87
Rishi Aurobindo Memorial Academy 134 P. K. Guha Road DumDum Kol-28 Study Material of 13/4/2020 For Class Nursery to XII Class Nursery Subject Topic Execution English Letter ‘A’ Golden rhymes Write in copy/ worksheet Humpty Dumpty listen and repeat Math Number ‘1’ Write in copy / workshhet Hindi Letter () Write in copy Bengali ऊऊऊ ऊऊऊऊ ऊऊऊऊ ऊऊऊऊ Pg 6 Write in book Class KG I Subject Topic Execution English Reading book Pg 14, 15 Write in Book Maths Reading Book pg 139 Write in Book Bengali ऊऊऊऊऊऊ ऊऊऊऊऊ ऊ ऊऊऊऊऊऊऊ(ऊऊऊऊऊऊऊ ऊऊऊऊऊ Pg 9) Listen & Repeat Class KG II Subject Topic Execution English (Term I) Pg 23, 24 Golden Nursery Rhymes C Write in cursive letters in book Peter, peter, pumpkin eater - (Read and Learn) Math (Term I) Pg 77 Write in book Science (Term I) Pg 126 Look at the pictures and learn. Class I Subject Topic Summary Execution Hindi ऊऊऊऊऊऊऊ ऊऊऊऊऊऊ ऊऊऊऊ ऊऊ ऊऊऊ ऊऊऊऊऊ 1) ऊऊऊ 2) ऊऊऊ 3) ऊऊऊ 4) ऊऊऊ 5) ऊऊऊऊऊ 6) ऊऊऊ 7) ऊऊऊ 8)ऊऊऊ 9)ऊऊऊ 10) ऊऊऊ Bengali ऊऊ–ऊऊऊऊ ऊऊऊऊऊ ऊऊऊऊऊऊ ऊऊऊ ऊऊऊऊ ১০ ऊऊऊऊऊ ऊऊऊ ऊऊऊऊऊऊ ऊ ऊऊऊऊ ऊ :- ऊऊऊऊ , ऊऊऊऊऊऊऊ , ऊऊऊऊऊऊऊऊऊऊ ১০ :- ऊऊऊऊऊऊ ऊ ऊऊऊऊऊ English 1 Chapter 16 the sentence Book page 56 continues C. Arrange the jumbled words:- 6)What is the name of your school? 7) My mother is an artist. 8) I love watching animals on television. 9) India is a big country. 10)I went to Delhi for a holiday. EVS Chapter 6 food Book page 120,121 B.Write true or false:- 1) Lunch is the first meal of the day.-- false 2)Healthy food makes the body weak.-- False3)Plant give us fruits

  · Web viewSubject. Topic. Summary. Execution. Hindi. व्याकरण. शरीरके अंगो के नाम लिखिए. 1) आँख 2) नाक 3

  • Upload
    others

  • View
    2

  • Download
    0

Embed Size (px)

Citation preview

Page 1:   · Web viewSubject. Topic. Summary. Execution. Hindi. व्याकरण. शरीरके अंगो के नाम लिखिए. 1) आँख 2) नाक 3

Rishi Aurobindo Memorial Academy134 P K Guha Road DumDum Kol-28

Study Material of 1342020For Class Nursery to XII

Class NurserySubject Topic Execution

English Letter lsquoArsquo Golden rhymes

Write in copy worksheet Humpty Dumpty listen and repeat

Math Number lsquo1rsquo Write in copy workshhetHindi Letter (ऊ) Write in copy

Bengali এসো লিলি লেো লিলি Pg 6 Write in bookClass KG I

Subject Topic ExecutionEnglish Reading book Pg 14 15 Write in BookMaths Reading Book pg 139 Write in Book

Bengali বস13র ধবলি ও উচচোর(োলিতয মক Pg 9)

Listen amp Repeat

Class KG IISubject Topic Execution

English (Term I) Pg 23 24 Golden Nursery Rhymes C

Write in cursive letters in book Peter peter pumpkin eater - (Read and Learn)

Math (Term I) Pg 77 Write in bookScience (Term I) Pg 126 Look at the pictures and learn Class I

Subject Topic Summary ExecutionHindi वयाकरण शरीरक अगो क नाम लिखिए 1) आ 2) नाक 3) कान 4) जीभ 5) घटना 6) पीठ 7)

पर 8)हाथ 9)गा 10) कमरBengali বইndashব13 পলিরচয় অ লেসক ঔ এবং ১ লেসক ১০ পব13 পোঠ অভযো

অ লেসক ঔ - লেো এসোসমসো যসথো১ লেসক ১০ - ংযো ও বোো

English 1 Chapter 16 the sentence

Book page 56 continues C Arrange the jumbled words- 6)What is the name of your school7) My mother is an artist8) I love watching animals on television9) India is a big country10)I went to Delhi for a holiday

EVS Chapter 6 food

Book page 120121 BWrite true or false-1) Lunch is the first meal of the day-- false 2)Healthy food makes the body weak-- False3)Plant give us fruits and vegetablesmdashTrue4)Animals give us milk and meatmdashtrueD) unscramble the letters-OTATOP- POTATO LKIM ndash MILKGGE ndash EGGERIC ndash RICE

Maths Chapter 4 Addition and subtraction

H W Book Do page 199 mental maths (Subtraction)

English 2 Book Orchid

The caring granny

This is a conversation between granny and his two grandchildren Sahil and NancyIn this chapter we come to know how the caring granny takes care of her little grandchild Nancy and by her clever tricks she fulfilled

Read the lesson

Nancyrsquos dream of getting the moonNow they all are happy together

Class IISubject Topic Summary Execution

Eng literature

Ch 4 Bob and the BathTub

REFER PG 46 DO IN THE COPY DO ndash IS USED FOR PRESENT TENSE ( NOW) DID ndash IS USED FOR PAST TENSE ( FINISHED)

COMPOUND WORDS ARE WORDS THAT ARE MADE BY JOINING TWO SEPARATE WORDS TOGETHER

III Write each word in the right boxAnswers DOget give keep drink make throwDIDgot gave kept drank made threw IV What small words make these compound wordsAnswers

1 bath and tub2 birth and day3 bath and room4 every and body5 water and melon6 mother and land

Eng language

Ch 3 A AN THE REFER TO PG 20DO IN THE BOOKPractise all that is done in the book

EX E] READ THE VERSE AND FILL IN THE BLANKS WITH A AN OR THEANSWERSthe an the the

EVS Ch 5 Eating Healthy REFER TO PG 118DO IN THE BOOK AND LEARN THEM

EX A TICK THE CORRECT STATEMENT PUT A CROSS NEXT TO THE WRONG STATEMENTANSWERS-1a] gram2c] milk3b] different kinds of food4a] energy- giving food

Mathematics CHAPTER-3NUMBERS UPTO 1000

PLACE VALUEPLACE VALUE OF A DIGIT IN A NUMERAL DEPENDS UPON ITS PLACE OR THE NUMERAL

FACE VALUEFACE VALUE OF A DIGIT IN A NUMERAL IS EQUAL TO THE DIGIT ITSELF

FIND THE PLACE VALUE OF EACH DIGIT OF 652H T O6 5 2Where place value of 6 is hundredPlace value if 5 is tensPlace value of 2 is ones

FIND THE FACE VALUE OF EACH DIGIT IN 213In 213 face value of 2 is 2Face value of 1 is 1Face value of 3 is 3

Bengali বইndashবোংো োলিতয পলিরচয়

পোঠndashলিং ও ইদসরর গলপ অীীর পরসোততর

৩ টিক দোও -ক) পোোস2র মস3য লি4 একটো ( গোলিদঘী)) ইদর লিংসর ( োসকর মসর ) মস3য ঢসক প2গ) লিংসর ( োসকর কোসর ) ফসটো লেসক ইদর লেবলিরসয় এসোঘ) লিংএক( লিকোলিররলেসর ) োস আটকো প2সো

ঙ)( ইদর লিকোলির )লিgtং লেক বাোলিচসয় লিদ৪ লিবপরীত -ব2 = লে4োট োকো = ভোরী দব13 = ব কষদর = বৎ দ13োম = োম উপকোর = অপকোর

Hindi वयाकरण Chapter 2 वणमाा एव वण क भद

वण-बोत समय हमार म स धवनिनया निनकती ह इन धवनिनयो कोlsquoवणrsquoया lsquoअकषर कहत

ह वण क टकड नही निकए जा सकत ह वण भाषा की सबस छोटी इकाई ह वण क निनशचिltत करम को वण माा

कहत ह हिहदी म वणA क दो भद होत ह- सवर

और वयजन सवण वण की सखया=11 वयजन वण की सखया = 33

सभी वयजन वण म lsquoअrsquoकी धवनिन मिमी हनिबनाlsquoअrsquoधवनिन वा वण कषतरजञशरndashसयकत वयजनड ढndashनई धवनिनयाअ-अनसवार अ- निवसग

Read the lesson

Class IIISubject Topic Summary Execution

COMPUTER CHAPTER 2 OPERATING SYSTEM

DONE IN THE PREVIOUS CLASS Q3)WHAT IS THE DIFFERENCE BETWEEN MULTI-TASKING AND MULTI-THREADING OPERATING SYSTEMSAns) THE DIFFERENCE BETWEEN MULTITASKING AND MULTITHREADING OPERATING SYSTEMS IS---

MULTITASKING OPERATING SYSTEM

MULTITHREADING OPERATING SYSTEM

A MULTITASKING OPERATING SYSTEM ALLOWS TO RUN MORE THAN ONE SOFTWARE PROCESSES AT THE SAME TIME

MULTITHREADING OPERATING SYSTEM ALLOWS DIFFERENT PARTS OF A SINGLE PROGRAM TO RUN AT THE SAME TIME

Q4) WHAT IS A GUI OPERATING SYSTEMAns) A GUI OPERATING SYSTEM ALLOWS USERS TO INTERACT USING A COMPUTER MOUSE OR ANY OTHER POINTING DEVICE FOR EXAMPLE MICROSOFT WINDOWS 7

ENGLISH I CHAPTER 4 ARTICLES

refer to pg 15 do in the rough copy and practice the whole chapter

Continuation pg 15 ex DANSWERS10You are the tallest person in the class11The sun rises in the east12An honest person always speaks the truth13The peacock is the national bird of India14The Nile is the longest river in the world15We have a beautiful garden The garden is full of roses16He is one of the best authors of the world17My brother lives in an apartment in the centre of the city18He quickly ate the cookies that was meant for the guests

SST INDIA-THE LAND OF FESTIVALS

Bihu it is a harvest festival celebrated in Assam Magh Bihu is celebrated every year in the month of January People feast sing and have fun

1In which state Bihu festival is celebratedAns Assam

2In which month Magh Bihu is celebrated

Pongal Itrsquos a four day long festival celebrated in Tamil Nadu It is celebrated on January 14 every year People cook sweet rice called Pongal

Ans January

3What is PongalAns Pongal is a harvest festival which is celebrated in Tamil Nadu It is a four day long festival

4In which month Pongal is celebratedAns Pongal is celebrated on January 14 every year

SCIENCE SOME PROPERTIES OF WATER

We get water from the nature so water is a natural resource Water is the most commonly found liquid on earth All living things need water to live Life on earth is not possible without water We need water for drinking cooking bathing washing etc

Fill in the blanks [pg no ndash 81]1We get fresh water from rain 2Boiling kills the germs present in water 3 Water changes its state when it is heated and cooled 4Water does not have a definite shape

BENGALI বই ndash বোংো োলিতয পলিরচয়

পোঠ ndash ১ একতোই বঅীীর পরসোততর

১ঠিক উততসর টিক দোও-ক) বস2ো লেোকটির ( লিত চোর পাোচ ) লে4স ) তোরো ব ময় ( কা োসদ োফোয় ঝগ2ো কসর ) গ) বস2ো লেোকটি লে4সসদর বস ( কমস2ো বাো কলিG ) আসত ঘ) বস2ো তোর লে4সসদর ( োটক মযোলিক লিসমো ) লেদোসব বসলি4সো ২ ক) লেোকটির কয়টি লে4স লি4 উঃ পাোচটি লে4স লি4 ) বস2ো লেোকটো লে4সসদর পরসম লিক আসত ব উঃ কলিG আসত বসোগ) বোলিJ বাো3োর পর তোসদর লিক করসত ব উঃ এক-এক কসর বোলিJটি ভোঙোর লেচষটো করসত ব ঘ) লে4সরো লিমসলিমস োকস লিক সব উঃ লে4সরো লিমসলিমস োকস বোইসরর লেোক

HINDI वयाकरण chapter 1 भाषा

BOOK PAGE 10 राजय भाषामहाराषटर मराठी गजरात गजराती पजाब पजाबी बगा बागा लिनिप भाषा दवनागरी-- हिहदीमराठी गजराती रोमन -- अगरजी गरमी -- पजाबी फारसी -- उरद

सही या गत निनशान गाइए-1पतर मौखिक भाषा का एक रप ह( )X 2भाषा क लिखित रप को लिनिप कहत ह( )3 ससार म सभी ोग एक ही भाषा बोत ह(X) 4गजरात म पजाबी बोी जाती ह(X) भाषा क दो

MATHEMATICS CHAPTER 4 SUBTRACTION

Subtraction without DecomposingSubtraction of 3-digit number (without decomposing)

The result obtained after subtraction is called difference

Note While finding the difference we write the greater of the two given numbers on the top and then subtract

Exercise 16Subtract9 409 10 876 11 641 - 103 - 443 - 510 306 433 131

12 829 - 113 716

Write in columns and find the difference14 603 ndash 401 15 840 ndash 610

Solution

14 H T O 15 H T O 6 0 3 8 4 0 - 4 0 1 - 6 1 0 2 0 2 2 3 0

Fill in the place holders17 _ 3 7 18 6 _ 5

- 2 - 6 - 2 4 3 7 2 -- -- 4 ndash

Solution

179 3 7 18 6 8 5 - 2 1 6 - 2 4 3 7 2 1 4 4 2

Class IVSubject Topic Summary Execution

English language

Personal pronouns

The different forms of the three personal pronounsFirst personSingular I me my minePlural we us our oursSecond person Singular thou thee thy thinePlural ye you your yoursThird personSingular he him his her hers she it itsPlural they them theirs their

Pick out the personal pronouns Mention of what kind each (first second or third person)1 You should always try to keep a promise that you have madeYou second person personal pronoun

2 I listened patiently to what he had to say about themI first person personal pronounHe them third person personal pronoun

3 They packed their belongings and left I do not know where they have goneThey their third person personal pronounI first person personal pronoun

Social studies Major landforms on earth

Explained in previous lesson Answer the following questions3 Differentiate between hills and mountainsAns Mountains has a peak whereas hills have rounded topsMountains are steeper than hills whereas hills are lower and less steep4 Give two usefulness of mountainsAns Mountains are storehouse of waterMountains have a rich variety of flora and fauna

MATHEMATICS

Ch 5Subtraction

Estimating the differenceWe have learnt estimation in addition Applying the same rule We estimate the difference

Exercise 15 Example 2

A fruit-seller has 38210 bananas Out of these he sold 21799 bananas Estimate the unsold bananas

Solution Estimating to ten-thousandsTotal bananas = 40000Bananas sold out =- 20000Bananas unsold = 20000

Exercise1 Find the actual and estimated difference by rounding off to the nearest thousands(b) ActualEstimated

16190 - 2979

Solution (b) ActualEstimated 16190 16000 -2979 -30001321113000

2 Find the actual and estimated difference by rounding off to the nearest ten-thousands(b) ActualEstimated 73012 -28790

Solution (b) ActualEstimated 73012 70000 -28790-3000044222 40000

4 A bakery shop has an order to supply 6705 packets of biscuits There are only 4920 packets in the shop Estimate the number of packets needed to complete the supply

Solution Estimating to thousandsEstimated order = 7000Packets available= -5000Packets needed = 2000

there4 Estimated packets needed to complete the order were 2000

Hindi 2ndlang पतर पतर लिना भी एक का ह पतर लित समय धयान दना होगा निक पतर निकस लि रह ह कयोनिक पतर या तो वयवहारिरक होगा या वयावहारिरक होगापतर दो परकार क होत ह

क) औपचारिरक पतर जस परधानाचाय को नगर निनगम सपादक आदिद

ख) अनौपचारिरक पतर जस पापा को दोसत को बहन को अपन परिरजनो को

1 औपचारिरक पतर-अपन निवदयाय म अवकाश क लिए परधानाचाय को पतर लिखिए १०३कसीबीदम दमक टकोकाता-७०००६५सवा मपरधानाचायऋनिष अरहिवदो ममोरिरय एकडमीदिदनाक-१३४२०२०निवषय-अवकाश हत पतरमहोदया जीसनिवनय निनवदन ह निक म आपकी ककषा चौथी का निवदयाथj ह क रात स मझ तज बार ह डॉकटर न मझ आराम करन को कहा हअतः म निवदयाय आन म असमथ ह आपस निनवदन ह निक आप मझ दिदनाक १३स १७ तक की अवकाश दन की कपा कर हम आप क आभारी रहग

धनयवादआपका आजञाकारी लिशषयसीमा सिसहककषा-४

Bengali বইndashবোংো ভোষো পলিরচয়

পোঠndash৮ লিঙগ ৩ পংলিঙগ কোসক বসউঃ পরোলিবোচক লেP লিবসষয পসদর দবোরো পরষ োতীয় কোউসক লেবোঝোয় তোসক পংলিঙগ বস লেPম -বোবো 4োতর৪ সতরীলিঙগ কোসক বসউঃ পরোলিবোচক লেP লিবসষয পসদর তোরো সতরী োতীয় কোউসক লেবোঝোয় তোসক সতরীলিঙগ বস লেPম - মো লিলিকষকো৫ উভয়লিঙগ কোসক বস উঃ পরোলিবোচক লেPলিবসষয পসদর দবোরোপরষ ও সতরীউভয়োলিতসক লেবোঝোয় তোসক উভয়লিঙগ বস লেPম - লিশ মনতরী ৬ কলীবলিঙগ কোসক বস উঃ অপরোলিবোচক লেP লিবসষয পসদর দবোরো সতরী বো পরষ কোউসক ো বলিঝসয় লেকো2 পদো13 লেক লেবোঝোয়তোসক কলীবলিঙগ বস লেPম - বই

COMPUTER CHAPTER 3 EDITING IN MS WORD

MOVING THE TEXTMS WORD ALLOWS US TO MOVE A BLOCK OF TEXT OR GRAPHICS WITHIN A DOCUMENT

Q6) HOW TO WE USE THE COPY AND PASTE OPTIONAns) WE CAN CREATE A DUPLICATE COPY OF THE TEXTTHE STEPS TO COPY AND PASTE THE TEXT AS FOLLOWS----

SELECT THE PART OF THE TEXT THAT WE WANT TO COPY

SELECT THE COPY BUTTON IN THE CLIPBOARD GROUP UNDER THE HOME TAB OR PRESS CTRL + C KEYS

NOW PLACE THE CURSOR AT THE PLACE WHERE THE TEXT NEEDS TO BE COPIED

CLICK THE PASTE BUTTON IN THE CLIPBOARD GROUP UNDER THE HOME TAB OR PRESS CTRL + V KEYS

Science Adaptations in Animals

We find different kinds of animals in our surroundings We find birds flying in the sky monkeys jumping on trees fish in water camels in desert and so on Animals live in their natural homes which are called their habitats The habitat of a camel is a dry sandy desert and that of a tiger is a dense forest So different animals live in different habitats In the world a plant or an animal has to adapt or change itself to suits its surroundings This changes happens over hundreds and thousands of years A change that a living thing undergoes to become better suited to its surrounding is called adaptation

Fill in the blanks [pg no 40]1 Terrestrial animals live on

land 2 Polar bears are found in the

cold polar regions 3 Frogs have webbed feet that

help them to swim 4 Monkeys and koalas are

arboreal animals

Class VSubject Topic Summary Execution

COMPUTER130420

CHAPTER 2 APPLICATIONS OF COMPUTERS

PAGE NO-15C FILL IN THE BLANKS

INPUTS ANIMATION BANKS SOFTWARE SUPER COMPUTERS

D WRITE TRUE OR FALSE TRUE TRUE TRUE TRUE FALSE

Science Chapter 2 - The Skeletal System

Joints A joint is a place where two or more bones are joined with each other

There are two types of joints 1) Fixed joints 2) Movable joints

The movable joints are mainly four types ndash 1) Hinge joint 2) Pivot joint 3) Gliding joint 4) Ball and socket joint

DAnswer these questions

1)What are the functions of the skeleton

Ans ndash The functions of skeleton are -

i The skeleton gives our body shape ii The skeleton provide our body strength iii The skeleton protect our inner organs from outside injuries iv The skeleton gives our body support

2)List the type of movable joint in our body giving one example of each

Ans - The movable joints are mainly four types ndash a Hinge joint

Example of hinge joints are elbows nice fingers and toes

b Pivot joint Example - the joint between the skull and the backbone is the example of pivot joint

c Gliding joint Example of gliding joints

are wrist and ankle joints d Ball and socket joint

Example of ball and socket joints are shoulder joint and hip joint

English language

Transitive and intransitive verb

Pick out the verbs from the following sentences and say whether they are transitive or intransitive (page 21 ex B)1 gave ndash verb Transitive verb2 Has planted- verb Intransitive verb3 Were- verb Intransitive verb4 Asked- verb Transitive verb5 Told- verb Transitive verb

Social studies Conquering distances

Airways The only airline owned by the government is Air India which handles both domestic and international flightsAdvantages of airways Air transport is the fastestIt can access remote areasIt is the best means of transport in case of emergencies

1 Which is the only airline owned by the governmentAns Air India

2 Give two advantages of airwaysAns The two advantages of airways areAir transport is the fastest transportIt can access remote areas

DisadvantageIt is the most expensive of all other means of transport

3 What is the disadvantage of airwaysAns The only disadvantage of airways is that it is the most expensive of all other means of transport

MATHEMATICS

Ch 3Addition and Subtraction

Properties of Addition1 The sum of two numbers does not change when we change their order This property known as Commutative Property of addition

2 The sum of three numbers does not change when we change their grouping This property is called Associative Property of addition

3 The sum of the numbers and zero is the number itself This property is called Identity Property of addition and the integer 0 is called identity

Exercise 11Fill in the blanks1 2730815 + 8319293 = ____ + 27308152 18219 + 1850308 = 1850308 + ____3 (27815 + 85919) + 95985 = (85919 + ____) + 278154 13227 + (25983 + 73607) = (____ + 25983) + 736075 91389 + 0 = ____ + 91389

Solution 1 83192932 182193 959854 132275 0

6 Which of the following are true statements(a) Any number added to zero is zero

(b) The sum of two numbers does not change when we change their order

(c) 1 is the identity element of addition

(d) Given any three numbers their sum does not change when we change their grouping

Solution (a) False(b) True(c) False(d) True

Class VISubject Topic Summary Execution

HISTORY AND CIVICS

CHAPTER 3

MAHAVIRA AND BUDDHA ndash GREAT PREACHERS BUDDHA

Decline of Buddhism1 Revival of the Brahmanical Hinduism ndash Brahmin Scholars like Shankaracharya and Kumarila Bhatta led the revival of Hinduism and established the supremacy of Vedic religion2 Loss of Royal Patronage ndash Gupta period marked the decline of Buddhism as Gupta rulers were followers of Hinduism3 Split in Buddhism ndash division into Hinayana and Mahayana sects and rise of Mahayana sects blurred the line between Hinduism and Buddhism4 Corruption in Buddhist Sangha ndash due to generation of large revenue from large estates Buddhist monks and nuns started living luxurious

Answer the Following 1 During which dynasty Buddhism was split During the reign of Kanishka

2 Name the two sects of Buddhism Mahayana and Hinayana

3 Name two Vedic scholars who led the revival of Brahmanical Hinduism Shankaracharya and Kumarila Bhatta

life in rich monasteries Hence corruption crept in5 Adoption of Sanskrit ndash when Buddhist scriptures began to be written in Sanskrit in place of peoples language like Pali or Prakrit people started drifting away from Buddhism6 The Turkish Invasion ndash As Muslim conquerors invaded India immensely wealthy Buddhist Monasteries and temples were looted and destroyed and Buddhists were persecuted and killed

4 During which period decline of Buddhism began The Gupta Period

ENGLISH 2 The great train journey- Ruskin Bond

The great journey by Ruskin Bond is a story about Suraj who loved trains and wanted to go to places One day while wandering along the railway tracks he enters into a carriage compartment The train suddenly starts moving with him in the compartment and after a journey returns back to the same place from where it had begun The story is about his experience during that journey

4 Answer the following questionsf Who else is in the carriageA ragged hippy with a dirty beard face was in the carriageg Where does Suraj say that he would like to go toSuraj said that he would like to go to England and China and Africa and Greenland He wanted to go all over the worldh What warning does the man give to SurajThe man said Suraj to keep out of sight so that he doesnrsquot get caught by the ticket collectorsiWhen Suraj thinks about his parents for the first time what does he imagines that they will thinkSuraj thought that if he failed to come home that night his parents would think that he had run away or been kidnapped or been involved in an accidentJ What presents does Suraj imagine that he will bring back for his friendSuraj imagines that he would bring an African lion or a transistor- radio for his friend

CHEMISTRY

Chapter 2 ndashElement and Compound

ATOMAn atom can be defined as the smallest constituent particle of an element which showcases independent existence Example Ne OMOLECULEA molecule can be defined as the combinations of two or more atoms which are held together by chemical bonds A molecule is the smallest portion of a substance which showcases all the properties of the substance On breaking down a molecule further we see properties of the constituent elements Example HCl NaCl O2

Answer the following Q3) What is a moleculeAns - A molecule can be defined as the combinations of two or more atoms which are held together by chemical bonds A molecule is the smallest portion of a substance which showcases all the properties of the substance On breaking down a molecule further we see properties of the constituent elements Example HCl NaCl O2

Q4) Which can exist independently ndash atom or moleculesAns ndash Molecules can exist independently

PHYSICS Physical quantities

Guidelines for writing SI units correctly1 The units named after scientists are not written with a capital initial letter For example newton henry watt2 The symbols of the units named after scientist should be written by a capital letter For example N for newton H for henry W for watt3 Small letters are used as symbols for units not derived from a proper name For example m for metre kg for kilogram4 No full stop or other punctuation marks should be used within or at the end of symbols For example 50 m and not as 50 m5 The symbols of the units do not take plural form For example 10 kg not as 10 kgs6 When temperature is expressed in kelvin the degree sign is omitted For example 273 K not as 273o K (If expressed in Celsius scale degree sign is to be included For example 100o C and not 100 C)7 Use of solidus is recommended only for indicating a division of one letter unit symbol by another unit

Fill in the blanks

1) Length and mass are examples of fundamental physical quantities

2) The measurement of a physical quantity consists of two part magnitude and unit

3) A foot consist of 32 inches 4) The unit of temperature in the SI system is

Kelvin

Write true or false Correct the false statements

1) In ancient times cubit was used to measure the mass of an object FalseCorrect statement ndash In ancient times cubit was used to measure the length of an object

2) There are 7 fundamental physical quantities True

symbol Not more than one solidus is used For example m s-1 or m s J K mol or J K-1 mol-1 but not J K mol8 Some space is always to be left between the number and the symbol of the unit and also between the symbols for compound units such as force momentum etc For example it is not correct to write 23m The correct representation is 23 m kg m s-2 and not as kgms-29 Only accepted symbols should be used For example ampere is represented as A and not as amp or am second is represented as s and not as sec10 Numerical value of any physical quantity should be expressed in scientific notationFor an example density of mercury is 136 x 104 kg m-3 and not as 13600 kg m-3

3) Second is the unit of time in both the CGS and MKS systems True

4) The symbol used for a unit is always written in capital letters False Correct statement -The symbol used for a unit is normally written in small letters

Hindi 2nd language

वाकय निवचार भागवत निवचारो को परकट करन वा साथक एव वयवसथिtत शबद समह को वाकय कहत ह वाकय दो परकार क होत ह ndash

1 उददशय- वाकय म जिजसक बार म कछ बताया जाता ह उस उददशय कहत ह जस राधा एक नतकी ह2 निवधय- वाकय म उददशय क बार म बताया जाता ह उस निवधयक कहत ह जस- राधा एक नतकी ह रचना क आधार पर वाकय क तीन भद होत ह ndash१सर वाकय- राम बाजार गया २ सयकत वाकय- राम बाजार गया और वहा जाकर दोसत स मिमा३ मिमशर वाकय- यह वही tान ह जहा उनका बचपन बीता

helliphellipContinue to nextBengali 2nd language

লিZ সবরপ ও সবরলিZ

সবরলিZর লিয়ম - ১ অ-কোর লিকংবো আ-কোসরর পসর অ-কোর লিকংবো আ - কোর োকস উভয় লিমস আ ndashকোর য় এবং ওই আ ndash কোর পব13বস13 Pকত য়

২ ই - কোর লিকংবো ঈ - কোসরর পসর ই - কোর লিকংবো ঈ - কোর োকস উভয় লিমস ঈ - কোর য় এবং ওই ঈ - কোর পব13বস13 Pকত য়

৩ উ - কোর লিকংবো ঊ - কোসরর পসর উ - কোর লিকংবো ঊ - কোর োকস উভয় লিমস ঊ - কোর য় এবং ওই ঊ - কোর পব13বস13 Pকত য়

৪ অ - কোর লিকংবো আ ndash কোসরর পসর ই - কোর লিকংবো ঈ - কোর োকস উভয় লিমস এ - কোর য় এবং ওই এ - কোর পব13বস13 Pকত য়

১ অ + অ = আ ( gtো ) লিম + অচ = লিমোচ সব + অ3ী = সবো3ী অ + আ = আ ( gtো )পদম + আ = পদমো শভ + আলি = শভোলি আ + আ = আ ( gtো )4োয়ো + আবত = 4োয়োবত মো + আতমো = মোতমো আ + অ = আ ( gtো )লিবদযো + অংকোর = লিবদযোংকোর Pো + অ13 = Pো13 ২ ই + ই = ঈ ( gtী )অলিত + ইব = অতীব লিগলির + ইনদর = লিগরীনদর ই + ঈ = ঈ ( gtী )পলির + ইকষো = পরীকষো অলি3 + ঈশবর = অ3ীশবর ঈ + ঈ = ঈ ( gtী )মী + ঈশবর = মীশবর 3ী + ঈ = 3ী ঈ + ই = ঈ ( gtী )রী + ইনদর = রীনদর মী + ইনদর = মীনদর ৩ উ + উ = ঊ ( gt )মর + উদযো = মরদযো কট + উলিকত = কটলিকত উ + ঊ = ঊ ( gt )ঘ + ঊলিম13 = ঘলিম13 লিZ + ঊলিম13 = লিZলিম13 ঊ + ঊ = ঊ ( gt )রP + ঊলিম13 = রPলিম13

৪ অ + ই = এ ( লেgt )র + ইনদর = সরনদর লেPোগ + ইনদর = লেPোসগনদর অ + ঈ = এ ( লেgt )গ + ঈ = গস

র + ঈ = সর আ + ই = এ ( লেgt )Pো + ইষট = Pসষট 3ো + ইনদ = স3নদ আ + ঈ = এ ( লেgt )রমো + ঈ = রসম দবোরকো + ঈশবর = দবোরসকশবর

COMPUTER THE WORLD OF WINDOWS 10

DONE IN THE PREVIOUS CLASS PAGE NO-83A TICK THE CORRECT OPTION BACKGROUND DISPLAY AREA RESTORE THREE

MATHEMATICS Topic ndash NumbersChapter ndash Natural numbers and whole numbers

Study item Properties of whole numbers for subtraction1) Closure property When we do subtraction of two whole numbers we can not get a whole number in all time Example 8 ndash 3 = 5 a whole number 0 ndash 6 = -6 is not a whole numberTherefore the subtraction of two whole numbers is not satisfying closure property2) Commutative property If x and y are two whole numbers then x ndash y is not equal to y ndash xExample If x=16 and y = 7 then x ndash y = 16 ndash 7 = 9Again y ndash x = 7 ndash 16 = - 9 Therefore x ndash y not equal to y ndash x Therefore the subtraction of two whole numbers is not satisfy commutative3) Associative property If x y and z are three whole numbersThen x ndash ( y ndash z ) not equal to ( x ndash y ) ndash z Example If x = 20 y = 10 and z = 6Therefore x ndash (y ndash z ) = 20 ndash(10 ndash 6 ) = 20 ndash 4 = 16(X ndash y ) ndash z = (20 ndash 10) ndash 4 = 10 -4 =6Therefore x ndash(y ndash z) not equal to ( x ndash y) ndash zTherefore subtraction of whole numbers is not satisfying associativity4) Distributive property If x y and z are three whole numbersThen x (y ndash z ) = xy ndash xzAnd (y ndash z)x = yx ndash zxExample If x = 10 y = 6 and z = 4x(y ndash z ) = 10(6 ndash 4 ) = 10times6 ndash 10times4 = 60 ndash 40 = 20( 6 ndash 4 )times 10 = 6times10 ndash 4times10 = 60 ndash 40 = 20Therefore the subtraction of whole numbers is satisfying distributive property5) Existence of identity For any whole number x X ndash 0 = x but 0 ndash x = - x not equal to xThus for subtraction no identity number existsException 0 ndash 0 = 0 so 0 is its own identity for subtraction

Class VIISubject Topic Summary Execution

Hindi 2ndlang वचन जो सजञा शबद निकसी वसत या पराणी क एक या अनक होन का बोध कराया उनह वचन कहत ह जस डका- डकयह दो परकार की होती ह-

क) एकवचन-शबद क जिजस रप स उसक एक होन का बोध हो उस एक वचन कहत ह जस निकताब गमा आदिद

) बहवचन-शबद क जिजस रप स उसक आन ोन का पता च उस बहवचन कहत ह जस डक निकताब निततलियाआदिद

निनमनलिखित शबदो को एकवचन स बहवचन म बदोम- हमजानित- जानितयानारी- नारिरयामिमतर ndashमिमतरोपसतक -पसतकसडक-सडकबोत-बोतनाहर-नहररपए-रपया

Bengali বইndashবোংো োলিতয পলিরচয়

পোঠndash১৪ গলপ - অপর কলপো পর

লেক - লিবভলিতভষ বসনদযোপো3যোয়লেকndash রবীনদর পরবতf বোংো কোোলিতয 3োরোর উসgসPোগয োম পরকলিতসপরমী লিবভলিতভষ বসনদযোপো3যোয় তোর লেীসত লেPম বোসর বোসর লিফসর এসস4 গরোম বোংোর পরকলিতর কো লেতমলি এসস4 গরোমী মো লিচতরগলপndash অপর কলপো গলপোংটি লিবভলিতভষ বসনদযোপো3যোসয়র লিবযোত উপযো পসর পাোচোী লেসক গীত অপ অ13োৎ পসর পাোচোী তো অপর কলপোর লেকনদরীয় চলিরতর এই অংস আমরো পোই বোক অপসক বোক অপ কলপো লিবোী লে দসরর অ গো4 লেদস মোসয়র মস লেোো রপকোর রোসয পোলি2 লেদয় দপরসবো মোসয়র মসর কসর কোীদোী মোভোরত এর করসকষতর Pসjর ব13ো শস তোর মোবীর কস13র পরলিত ব2 মমতো য় আবোরপালিসত বলি13ত Pসjর অমোপত অং লে লিসই মোপত কসর বোলি2র লিপ4স বাো বোগোস লিকংবো উঠোসর লিশমসর কলপো লিবো এোস পরকো লেপসয়স4

১ অপর কলপো গসলপর লেক লেক তোর মপসক13 লেসো২ অপর কলপো গলপটি লেকোো লেসক গীত গলপটির ম ভোব লেসো

GEOGRAPHY CHAPTER 7EUROPE

CHAPTER COMPLETE EXERCISEFill in the blanks1 Europe is a continent that comprises the western part of Eurasia2 Eurasia and Africa are connected into one large land mass known as Afroeurasia3 The Strait of Gibraltar separates Europe and Africa4 Europe is surrounded by the Arctic Ocean to the north5 The British Isles includes the island countries of Great Britain and Ireland

Name the following 1 Connects Africa to Eurasia - Isthmus of Suez2 Largest country in the world in terms of area ndash Russia3 A term used collectively for the five countries in northern Europe ndash Nordic Countries4 The capital of Montenegro - Podgorica5 the largest fjord in Norway ndash Sognefjord

Match the following Column 1 Column 2a Albania iii Tiranab Belgium i Brusselsc Denmark v Copenhagend Finland ii Helsinkie Hungary iv Budapest

CHEMISTRY Chapter 2 ndashElement and Compound

Atom - An atom is the basic unit of an element or the smallest particle of an element non capable of independent existence Atom is built up of three sub atomic particles electron proton and neutron

Nucleus-It is the centre of an atom In the centre of the atom contains proton (positively charged particles ) and neutrons ( particles carrying no charge )

Orbits- It surround the nucleus in which revolve electrons (negatively charged particles)

Answer the following

1) What are MetalloidsAns - Certain elements using properties of both metal and non-metals are called metalloids Example Silicon arsenic and antimony

2) What are Noble gasesAns - Certain elements are present in the air and are chemically inert or unreactive Such elements are called rare gases or noble gases Example helium neon argon and Krypton

English 2 Sentences based on meanings

Kinds of sentences

Assertive or declarative to convey information or simply make a statement

Interrogative to ask different types of questions

Imperative to command or instruct someone or make a request

Exclamatory to express strong feelings and emotions

Exercise B1 Stop it ( Exclamatory)2 May you always be happy

together ( Exclamatory)3 He does not like sports

( Assertive)4 Please pass me the salt

( Imperative)5 How dare she talk to me like

that ( Exclamatory)6 May success bless your effort

( Exclamatory)7 Canrsquot you wait for sometime

(Interrogative)8 Did anybody tell you about it

( Interrogative)9 I saw her waiting for the bus

( Assertive)10 Could you please take a

message for me ( Interrogative)

Homework Ex ABiology Chp -2

Classification of Plants

Today we discuss about usefulness of bacteria We also discuss what the harmful effects of bacteria are

89 How bacteria are useful for usbull Bacteria is helpful in many ways forhuman being i) Production of medicine - antibiotics vaccine etcii) Formation of curd by lactobacillusiii)Nitrogen fixation in Leguminousplant by Rhizobiumiv) Increase soil fertility by absorbingatmospheric nitrogen and convert it into nitrates and nitritesv) Cleaning the environment by converting the complex substances into simple substancesvi) Tanning of leathervii) Retting of Fibersviii) Formation of compost by acting onanimal dung and agricultual cases1x) Biogas production by decomposingplant and animal wastex)Help In Nutrition by producing vitamiacutemBand kx1) Some bacteria are used to give specialflavour to tea coffee and coccaQ10- Name some diseases and there causativebacteriabull Diseasescausative bacteria1 CholeraVibrio cholerae2 Tuberculosis - Mycobacterium tuberculosis3 Diptheria -Corynebacteriumdiphtheriae4 Pneumonia - Streptococcus pneumoniae

Math Number system

Chapter Fraction

Study item Using lsquoofrsquoThe word lsquoofrsquo between any two fractions is to be used as multiplicationExample 57 of 56 = 57 times 56 = 5times8 = 40Study item Using BODMASThe word lsquoBODMASrsquo is the abbreviation formed by taking the initial letters of six operations(i)Bracket (ii) of (iii) Division (iv) Multiplication (v) Addition (vi) SubtractionAccording to BODMAS rule First of all the terms inside Bracket must be simplified then lsquoofrsquo lsquoDivisionrsquo lsquoMultiplicationrsquo lsquoAdditionrsquo lsquosubtractionrsquo

Study item Removal of Brackets

There are four Brackets of algebra in Mathematics In a complex expression four types of brackets are used Order of removing the brackets is first ----- then ( ) then finally [ ]

Class VIIISubject Topic Summary Execution

Chemistry Hydrogen General methods of preparation of hydrogen

By the action of dilute acids on metals

Calcium Reacts readily to form chloride salt and hydrogen

Ca + 2HCl rarr CaCl₂ + H₂uarr

Magnesium

Aluminium

Zinc

React readily to form salt and hydrogen

Mg + 2HCl rarr MgCl₂ + H₂uarr2Al + 6HCl rarr 2AlCl₃ + 3H₂uarrZn + 2HCl rarr ZnCl₂ + H₂uarr

Question 4 ) Give reasons for the following

(a) Hydrogen be used as a fuel

Solution

Hydrogen is used as a fuel because it has a high heat of combustion Some significant fuels are coal gas water gas and liquid hydrogen

(b) Though hydrogen is lighter than air it cannot be collected by downward displacement of air

Solution

Hydrogen is lighter than air so it is possible to collect the gas by downward displacement of air But it is not safe to do so since a mixture of hydrogen and air can lead to an explosion

(c) A pop sound produced when hydrogen is burnt

Solution

Impure hydrogen gas burns in air with a pop sound This is because of the presence of impurities in it

(d) Helium replaced hydrogen in weather observation balloons

Solution

It forms a mixture with air that can explode when there is a small leakage of hydrogen in a balloon So helium has replaced hydrogen

(e) Nitric acid not used for the preparation of hydrogen gas

Solution

(e) By the action of nitric acid on metals hydrogen cannot be produced because it also releases nitrous oxide and nitric oxide and oxides the hydrogen to form water

Biology Chp-2 Reproduction in plants

Today we discuss different methods of artificial propagation like cutting-rose sugercane Layering ndashguava lemon china rose etc Grafting- mango apple etcMicropropagation ndashorchid asparagus etcWe also discuss about advantages and disadvantages of vegetative propagation

Q7 Define the following terms i) Explant In tissue culture techniquea tiny piece of bud shoot or any other partof plant from where new tissue develop ii) Callus The cells of the tissue divide andgrow into a mass of undifferentiated cells from explant iii) Plantlet After few days callus differentiate into a small plant with roots and shootQ8 what are the advantages and limitations of tissue culture or micropropagation

Advantages i ) It produacuteces superior quality plantsii)It can be applied to interspecifie hybridsiii) It is useful to grow seedless plants bull Limitations i) It cannot be used for all plantsii)It is not easy to handleQ9 Write advantages of vegetative propagationi) It is a quick and easy method ofproducing new plantsii) This method need less time to matureiii) The new plants are exact copies of the parentiv) it is extremly useful for growing seedlessplants like banana grapes etc Q10 Write some disadvantages of vegetativepropagationi) Dišeases present in the parent plant gettransferred to all in new plantsii) Overcrowding of new plants causes competition for sunlight water and nutrients which affects growth of plantsplant

Physics Chapter 2 Physical Quatites and Measurements

Here We Will Do Some QuestionsRelated To Chapter 2

Select the correct alternative A block of wood of density 08gcm-3 has a volume of 60cm3 The mass of the block is

1 608 g

2 75 g

3 48 g

4 0013 g

Solution 348 g

The density of aluminium is 27g and that of brass The correct statement is

1 Equal masses of aluminium and brass have equal volumes

2 The mass of a certain volume of brass is more than the mass of an equal volume of aluminium

3 The volume of a certain mass of brass is more than the volume of an equal mass of aluminium

4 Equal volumes of aluminium and brass have equal masses

Solution 2 The mass of a certain volume of brass is more than the mass of an equal volume of aluminium

MATHEMATICS Ch 6Sets

Exercise 6(C)1 Find all the subset of each the following sets(i) A = 57 (iii) C = x xisin W x le 2(iv) p p is a letter in the word lsquopoorrsquo

Solution (i) All the subsets of A are ϕ 5 7 57

(iii) All the subsets of C are ϕ 0 1 2 01 02 12 012

(iv) All the subsets are ϕ p o r po or por

4 Given the universal set = -7-3-105689 find (i) A = x xlt2 (ii) B = x -4ltxlt6 Solution

(i) A = -7-3-10(ii) B = -3-105

5 Given the universal set = x xisin N and xlt20 find

(i) A = x x = 3p pisin N (iii) C = x x is divisible by 4 Solution

(i) 369121518 (iii) 481216

6 Find the proper subset of x x2-9x-10 = 0 Solution

ϕ 10 -1

Working x2-9x-10 = 0 rArr x2-(10-1)x-10 = 0

rArr x2-10x+x-10 = 0 rArrx(x-10)+1(x-10) = 0

rArr (x+1) (x-10) = 0

11 Let M = letters of the word REAL and N = letters of the word LARE Write sets M and N in roster form and then state whether (i) M sube N is true (ii) N sube M is true (iii) M = N is true

Solution M = real and N = lareSo (i) Yes (ii) Yes (iii) Yes

English 2 Twelfth Night ndash Shakespeare

A noble man named Orsino in the kingdom of Illyria is deeply in love with a lady called lady Olivia She is in mourning for her dead brother so she will not even think about marriage At this time a sea storm causes a terrible shipwreck and a young lady called Viola is swept onto the shore She thinks that her twin brother Sebastian is drowned A sea captain tells her about Orsino and his love for Olivia Viola wishes to work in Oliviarsquos home but feels she will not be employed So she dresses as a man calls herself Cesario and gets work at the house of OrsinoViola (now Ceasario) is much liked by Orsino and becomes his page She falls in love with Orsino Orsino sends Ceasario to deliver messages to Olivia Olivia herself falls for the beautiful young Ceasario believing Viola to be a man

2 Answer the following questionsa Why does Orsino ask the musicians to play onOrsino asks the musicians to play on because music feeds his desire He calls upon the musicians to play music so that his hunger for love could be replenished with an excess of musicb What does Valentine tell about OliviaWe learn from Valentine that Olivia is in mourning for her brother she wears a veil and has vowed that no one will see her face for another seven yearsand she refuses to marry anyone until thenc From the exchange between Orsino and Valentine what do you think their relationship isValentine is one of orsinod attendants He was sent to Olivia as a messenger of love but was not allowed to speak to here Who is Olivia mourning for and whyOlivia is mourning for her dead brother

Homework Q fHistory and Civics

Growth of Nationalism

Important dates to remember1769-Napoleon born on 15thAugust1789-Fall of Bastille on 14th July and the beginning of the French revolution declaration of the rights of Man on 26thAugust1793-King Louis XVI executed on January 211764-The Sugar Act passed1765-The Stamp act passed1774-The first congress of Philadelphia1776-The declaration of American Independence of on 4th July1777-Defeat of the British at Saratoga1781-Surrender of lord Cornwallis at Yorktown1783-The treaty of Versailles1804-Napoleon becomes the emperor1813-Battle of Leipzig or Battle of nations in which Napoleon was defeated by the Allies1815-Battle of Waterloo June 18 in which Napoleon was defeated and captured1821-Death of Napoleon in StHelena1860-Abraham Lincoln elected President of the USA1861-The civil war began 1864-Abraham Lincoln elected President of the USA for the second time1865-Slavery abolished in the US

Name the following- The queen of Louis XVI

Marie Antoinette The three philosophers of France

VoltaireMontesquieuJean Jacques Rousseau

The British general whose surrender brought the war in America to an endLord Cornwallis

The first president of the USAGeorge Washington (1732-1799)

The first southern state to secede from the unionSouth Carolina

The author of the book lsquoUncle Toms CabinHarriet Beecher Stowe

Homework-Learn

Class IXSubject Topic Summary Execution

Economics

Types of economies Today I am going to share you the concept of economic growth and economic development Few questions will be given from the previous study material dated 942020

Meaning of economic growthAnswer) The term economic growth generally means anincrease in national income or per capita output or income over time It indicates towards quantitative growth of a country

Meaning of economic developmentAnswer) Economic development is defined

as a process whereby the real per capita income of a country increases over time along with fall in poverty ratio unemployment and income inequality etc

Distinguish between economic growth and economic development

Basis Economic growth

Economic development

Scope It has narrow scope as it refer only to rise in per capita income

It has wide concept since it includes qualitative changes as well

Concerned matter

It is concerned with the rise in income

It is concerned with not only rise in income but also reduction of poverty income inequality and unemployment

Focus Economic growth does not focus on economic development

Economic development focus on economic growth plus qualitative changes

Distinguish between capitalist economy and socialist economy

Ownership

Motive

Tool

Means of production are owned and managed by private people

Self interest and profit earning is the main motive

Price mechanism is a main tool to solve the economic problems

Means of production are owned and managed by the government

Social welfare is the main motive

Economic planning by the government is the main tool to solve the economic problem

Competition

Distribution of income

There exist large competition among buyers and sellers

There is existence of large inequalities of income

There is no such competition

There exist less inequalities of income

Math Topic ndash AlgebraChapter -Factorisation

Study item Factorising by taking out common factorSome solved sums from exercise 41

1) (i) 8xy3 + 12x2y2

= HCF of 8xy3 and 12x2y2 is 4xy2

= 4xy2(2y + 3x )

4) (ii) 28p2q2r ndash 42pq2r2

= HCF of 28p2q2r and 42pq2r2 is 14pq2r = 14pq2r (2p - 3r )5) (ii) 14mn + 22m - 62p=HCF of 14mn 22m and 62p is 2= 2(7mn + 11m - 31p)7) (ii) 3a(x2 + y2) + 6b (x2 + y2) = HCF of 3a(x2 + y2) and 6b(x2 + y2 ) is (x2 + y2)= ( x2+ y2 )(3a + 6b )9) (ii) x(x2 + y2 ndash z2 ) + y(-x2ndashy2 + z2 ) ndash z(x2+ y2 ndash z2 )= x(x2 + y2 -z2) ndash y-(x2 + y2 -z2) -z(x2 + y2 ndash z2)=x(x2 + y2-z2) -y( x2 + y2-z2) ndash z (x2 + y2 -z2)= (x2+ y2 ndash z2)(x ndash y ndash z )

Commercial Studies

Introduction to Accounting and Book-keeping

Today I am going to share you the meaning of Accounting and Book-keeping and its related terms bullAccounting bullBook Keeping bullAccountsbullTypes Of Accounts bullAccounting Cycle

bull Meaning of accounting

Ans )Accounting is the art and science of recording classifying and summarising monetary transactions

bull Meaning of Book-keeping

Ans) Bookkeeping is the art of recording business transactions with the view of having a permanent record of them and showing their effect on wealth

bull Meaning of account

Ans) The term account means a record of business transactions concern a particular person of firm asset or income or expense It is a summarised record of all transactions which take place in an accounting year

bull Types of accountsPersonal accounts ndash Personal accounts relating

to person and Organisation are known

as personal accounts Example Ramrsquos Account ABC amp Co Account etc

Real account - The accounts related to tangible and intangible assets are called real accountsExample Cash Account Furniture Account etc

Nominal account- Accounts related to expenses losses incomes and gains are known as nominal accountsExample Wages Account Salary Account Discount Account etc

bull Accounting cycle Accounting cycle refers to a complete sequence of accounting activities It begins with recording of transactions and ends with the preparation of a balance sheet

English 1 Transformation of sentences

Sentences A sentence is a group of words which makes complete sense

a Assertive sentencesb Imperative sentencesc Interrogative

sentencesd Exclamatory sentences

Sentences can be changed from one grammatical form to another without changing the meaning of the sentence This is known as transformation of sentences

Exercise 6Rewrite the following sentences according to the instructions given below without changing their meanings

1 As soon as he saw the beer he jumped into the river ( Begin No sooner)

2 None but brave deserve the fair (Begin the bravehellip)

3 This box is too heavy for me to lift ( Use so hellip That instead of too)

4 No one other than a king can live like James Luxurious ( Begin only James)

5 Oh for the wings of a dove (Begin I wishhellip)

BENGALI(2ND LANGUAGE)

ldquo বঙগভমির পরমি ldquo াইকেল ধসদন দতত

পব13পোসঠ আসোলিচত ৩ পরবোস দৈদসবর বস ীবতোরো Pলিদ স এ লেদ -আকো সত-োলি লেদ তোস - ক) বকতো লেক কোর লেো লেকো কলিবতোর অং ) কোর পরলিত বকতোর এই উলিকত গ) এ লেদ আকো সত বসত কী বলিঝসয়স4 ীবতোরো বসত কী লেবোঝ ঘ ) আসোচয অংসর তোৎপP13 কী

উ -ক ) বকতো স কলিব মোইসক ম3দ দতত

Types of AccountPersonal AccountReal AccountNominal AccountBalance Sheet (opening)

কলিব মোইসক ম3দ দসততর রলিচত বঙগভলিমর পরলিত কলিবতোর অং ) কলিব বঙগী অ13োৎ লেদমোতোর পরলিত কলিবর এই উলিকত গ ) এ লেদ আকো বসত কলিবর মোব লেদী রপ আকো লেক লেবোঝোসো সয়স4 আকো লেসক লেPম তোরো স পসর লেতমলি ীব লেদ রপ আকো লেসক পরো রপ তোরো স পরসত পোসর এই মভোবোর কোই কলিব বসস4 ঘ ) পরবো Pোতরোয় Pলিদ কলিবর লেদ আকো লেসক ীব তোরো রপ পরো স পসর তোসত কলিব লিবনদমোতর দঃলিত কোর মতয লিবসর সবোভোলিবক পলিরলিত এবং মোষ মরী তোই পরবোস Pলিদ তা োর মতয য় তবও কলিব লিবচলিত সব ো কোর পলিবীসত লেকউ অমর য় লিক4ই অকষয় য় দীর লেPম লিচরপরবোমো লেতমলি মোসষর ীবও চমোতোই ীব - সতবধতোই মতয ীব দীসত মোষ লিতয পরবোমো তবও লেPব মোষ আপ কতকসম13র মো3যসম মোসষর মস লিসসদর সথো কসর লিসত পোসর তোরো লিচরভোসবর সয় মোসষর মস লিবরো কসর তোসদর মস3য লেকউ পGভসত লিবী সয় গোসও মোসষর মস তোরো লিতযপলিত লিতযবলিনদত

Hindi 2ndlang

काकीी(लिसयारामशरणगपत)

इस कहानी म क न यह बतान का परयास निकया ह निक बचच अपनी मा स निकतना परम करत ह शयाम अबोध बाक ह वह अपनी मा क मरन क बाद उसन अपनी मा क लिए बहत रोया बाद म उस पता चा निक उसकी मा राम क घर ची गई ह आकाश म उडती हई पतग दकर उस हष हआ निक पतग क दवारा वह अपनी मा को नीच उतारगा इसक लिए वह अपनी निपता की जब स दो बार सवा रपया निनकाकर पतग और दो मोटी सी मन वाी अपन भाई स काकी एक कागज पर लिवा कर पतग म लिशव का दिदयानिनकाकर पतग और दो मोटी सी मन वाी अपन भाई स काकी एक कागज पर लिवा कर पतग म लिचपका दिदयाभोा और शयाम कोठरी म रससी बाधनी रह थ तभी उसक निपता करोध म आकर उन स पछ निक कया उनकी जब स रपया निनकाा हभोा डर क मार बताया निक शयाम इस पतग क दवारा अपनी काकी को राम क यहा स उतारना चाहता हनिवशशवर(शयाम क निपता)न फटी पतग उठाकर दी तो उस पर काकी लिा थावह हत बजि होकर वही ड रह गएउनहोन सोचा निक मन अपन पतर को मारा जोनिक अनजान और निनदष थावह अपनी मा कोनिकतना पयार करता ह

helliphellipContinue to next

Computer Application

Java Programming Prog 1Write a java program to input two numbers from user and display the sum or product of them as per user choice Use switch case statementSolve public class sum_product public static void main(String args[]) Scanner sc=new Scanner(Systemin) int abc Systemoutprintln(ldquoEnter two numbersrdquo) a=scnextInt() b=scnextInt() Systemoutprintln(ldquoPress 1 for sum or 2 for productrdquo)

c=scnextInt() switch(c) case 1 Systemoutprintln(ldquoThe sum will be =rdquo+(a+b)) break case 2 Systemoutprintln(ldquoThe product will be =rdquo+(ab)) break default Systemoutprintln(ldquoWrong Inputrdquo) Home Work - Practice in your computer using bluej

Subject Eng Literature (The Merchant of Venice ndash William Shakespeare)Topic Act I Scene 2 Lines 92 to 126 (End of scene) Date 13th April 2020 (5th Period)

[Students should read the original play and also the paraphrase given in the school prescribed textbook]Summary Questions amp Answers

o After Portia has expressed her opinion about the suitors Nerissa informs that she need not bother about any one of them as they have decided to quit Belmont at the earliest opportunity because they do not believe in trying their luck by the caskets which is the only way of winning Portia

o Nerissa then enquires of Portiarsquos opinion about Bassanio who once visited her in the company of the Marquis of Montferrat and says that she had never come across such an ideal love deserving the fairest lady for his bride

o Portia seems to remember Bassanio quite correctly and says that she agrees with Nerissa At this moment a servant informs Portia that the Prince of Morocco has arrived to try his luck by the caskets

o Portia tells Nerissa that if she could welcome this new suitor as gladly as she says farewell to the previous ones she would be glad of his arrival However if he happens to have the virtues of a saint but the black complexion of a devil she would prefer to have him for religious consolation rather than as a husband

(1) NERISSA You need not fear lady (Line 97-103)

the having any of these lords they have acquainted me with their determinations

which is indeed to return to their home and to

trouble you with no more suit unless you may be wonby some other sort than your fathers imposition depending on the caskets

PORTIA If I live to be as old as Sibylla I will die as chaste asDiana unless I be obtained by the manner of my fatherswill I am glad this parcel of wooers are so reasonablefor there is not one among them but I dote on his veryabsence and I pray God grant them a fair departure

(a) Elucidate the idea expressed in the first speech of the above dialogue

In the first speech Nerissa assures Portia that she need not have any fear of being compelled to marry anyone of the suitors who had lately come to Belmont She informs her that they have all decided to return to their respective countries(b) Illuminate the meaning of the phrase ldquoyour fatherrsquos imposition depending on the casketsrdquo

Nerissa means that the suitors of Portia do not find the conditions imposed by the will of her father to their liking They are too hard for them These conditions are that in the event of a suitor failing to choose the right casket (i) he should never disclose to anybody which casket he chose (ii) he can never marry and (iii) he should take his departure immediately(c) Explain the meaning of the term lsquoSibyllarsquo

lsquoSibyllarsquo is the name given by Romans and Greeks to a prophetess inspired by some deity usually the sun-god Apollo She had a very long life The god Apollo granted her as many years of life as she could hold grains of sand in her hand(d) Elucidate the meaning of the term lsquoDianarsquo

lsquoDianarsquo is the goddess of hunting She is also regarded as a symbol of virginity because she never fell in love and never

married(e) Explain the meaning of the first two lines of Portiarsquos speech

Portia says that even if she is to live for centuries like Sibylla she would not marry except in accordance to her fatherrsquos will She asserts that she would not mind remaining unmarried and untouched by a man like Diana the virgin the goddess of hunting unless a man is able to win her by passing the test laid down by her father

Class XSubject Topic Summary Execution

Hindi 2nd

Langबड घर की बटी( मशी परमचद)

lsquoबड घर की बीटीrsquo कहानी का उददशय मधयम वग की घर समसया को सझा कर सगदिठत परिरवार म मिम जकर परम स रहन का सदश दना ह घर म शानित tानिपत करन की जिजममदारी नारी की होती ह यदिद नारी समझदार ह उसम धय और परिरवार क परनित परम ह तो कोई भी घटना परिरवार को निवघदिटत नही कर सकती या कहानी परिरवार को सगदिठत करत हए परम सौहाद स एक रदसर की भावनाओ को समझ करउनका सहयोग करत हए जीवन यापन करन की पररणा दती ह मशीपरमचदर जी न इस कहानी म सयकत परिरवार का परनितनिनमिधतव निकया ह यह कहानी बनी माधव सिसह जो गौरी पर क जमीदार क उनक दो पतरो की हशरी कठ ा निबहारीशरीकात का निववाह एकजमीदार घरान की पतरी आनदी स हआ थाआनदी न द को ससरा क वातावरण म ढालिया थाएक दिदन आनदी का अपन दवर ा निबहारी स झगडा हो जाता ह दोनो भाई एक रदसर स अग होन की कोलिशश करत हसभी बह आनदी न अपन मधर वयवहार स ा निबहारी को घर छोडकर जान स रोक लिया| इस पर बनी माधव सिसह न कहा निक बड घर की बटी ऐसी ही होती ह जो निबगडा काम बना ती ह अतः शीषक साथक ह बड घर की बटी आनदी ह

helliphelliphelliphellipContinue to nextBiology Topic ndash Chp-1

CellWelcome to new session 2020-21Today we will start with Chpter 1 cell CELL

Protoplasm+Cellmembrane Or Cell wall

Cytoplasm+Neucleus

Cytoplasmic+ CytoplasmicOrganelles Inclutions(mitochondria (food Golgi bodies pigments)Ribosome)

What is cellbull Cell is the structural and functional unit of living organismbull According to number of cells organisms areUnicellular - Amoeba bacteria Multicellular - Rose Mango Tiger HumanSmallest cell -bacteria Longest cell - Nerve cellLargest cell - Ostrich egg cellCells are of different size and shapes according to their functionsQ2Write chief functions of following cellorganelles

Q3What is tonoplastVacuoles covered by a covering called tonoplast

Bengali(2Nd

Language)

ফ ফটক ো ফটক (কলিবতো ) ভোষ মসোপো3 gtPোয়

একটি লেমসয়র ীবস লেপরম লিকভোসব ফসট ওসঠ তো লেদলিসয়স4 কলিব লেপরম Pই য় লেই ময়ই বনত কোস পলিরত য় ফ লেফোটো বো োসফোটো লেটো ব2 কো য় লেমসয়সদর ব gtয13 লেপরসমর 4লিব ফসট উসঠস4 এই কলিবতোয় লেপরম মোষসক মত gtযর মস লেফস লিদসয় পরকষস বাোচোসোর gtয োত বো2োয় কলিবতোয় লেমসয়টির পসব13র দঃসর কো বো সও লেমসয়টি লেই পসর পলিক সত চোয়ো োরী ীবসর কোস4 পরম লেPৌবস লেপরমসক পোবোর পরব ইচছো োকসও তো পসর লেলিতবোচকতোয় পলিরত য় কলিব ভোষ মসোপো3 যোয় লেP ক লেপরসমর

কলিবতোয় ব gtযবহত লিবসষ লিক4 সvর অ13 লেদওয়ো ১) রসবোো= লেP লিবলিভনন রকম ডোকসত পোসর২) ো= পোর ৩) ঠলি = লেচোসর বZ৪)আই বস2ো=অলিববোলিত৫)শইসয় = োলিয়ত কসর৬)োতপাোচ= লিবলিভনন পরকোর৭)দ2োম = v কসর বZ কসর লেদওয়ো৮)লেরলিং =লেোোর দৈতরী লেব2ো৯) বনত= একঋত১০) পাোর = বসকরো2

Organelles Functions

1 Endoplasmic reticulum

2 Mitochondria

3Golgibodies

4 Ribosome5Lysosome

6Plastids

7 Centrosome

i) Supportive framework for the cellii) Synthesis and transpost of proteinsRelease of energy in the form of ATPi) Synthesis and secretion of enzymes hormoneii) Formation of vacuoles lysosomei) Protein Synthesisi) Intracellular digestionii) Destroy foreign substancei )Leucoplast - stores starchii)chloroplast - trap solar energyiii) Chromoplast - imparts colour toflowers amp fruitsi) Initiates and requlates cell division

কলিবতো তোর অ13সক ভোষোয় পরকোো কসর ঘলিরসয় ব যকত কসরস4 লেপরসমর ফতো আর লিবফতো লেক গোঢ় কসর লেদোসো কলিব ভোষ মসোপো3 যোসয়র অলিভবসর অ যলিদক

Economics

Factors of Production

Welcome to the new sessionToday we are going to start the first chapter of Class XThe name of the chapter is Factors of productionBy the name I hope you all can recall a glimpse of what you have learnt in the second chapter of Class IX

NowProduction is the process of creating the various goods and services which are consumed by the people of the country to satisfy their wants

Thus it is the process in which some materials are transformed from one form to another to create utility and value in goods

For example utility can be created by changing the form of a commodity ie

Making of table out of wood by a carpenter for his customer here the wood is getting transformed into table creating utility for his customer and he can also command a price for it

On the other hand Housewives perform very

useful activities at home which create utility but their domestic activities are not included in production because they have no money value

So we can also say that Production denotes two things firstly creation of utility and secondly creation of value

Production is not complete unless it reaches the consumer

An increase in production will increase the economic welfare of the consumers and hence the aim is to raise the production level of the country

Again production of a good or service is only possible if certain resources or

Questions

1 What do you mean by production

Answer Production means the creation of goods and services for the purpose of selling in the market

In fact production involves the transformation of inputs into outputs

Hence production denotes two thingsCreation of utility and creation of valueUtility and value can be created by changing the form by changing the place by changing the time and by rendering services

Example Transformation of raw

materials into finish goods such as potter creates utility by converting mud into utensils assembling of small parts to make bigger machinery

Production also includes services such as distribution and marketing

2 What are the factors of production

Answer Factors of Production refers to the resources and inputs needed for producing goods and servicesThese inputs can be classified as

Land Labour

Capital Enterprise

Land Land is defined to include not only the surface of the earth but also all other free gifts of nature(for example mineral resources forest resources and indeed anything that helps us to carry out the production of goods and services but is provided by

inputs are used together in right proportion

A resource or an input which helps in the process of production to obtain an output is called FACTOR OF PRODUCTION

These factors of production can broadly be categorized into four parts 1LAND 2LABOUR3CAPITAL4ENTERPRISE (ORGANISATION)or Entrepreneur

The above factors are all interdependent on each other and they play a major role in production process

FACTORS OF PRODUCTION

LANDCAPITAL

LABOUR ENTREPRENEUR

nature free of cost)LabourLabour refers to the human efforts that need to be combined with other factors of production for creating an output

CapitalAll man ndash made means of production is called capita example machineries which help in further production Money when used for starting any business for purchasing raw materials machinery tools etc it is regarded as capitalCapital also includes physical capital like factories machineriestoolsbuildingsequipments etcEnterpriseThe task of bearing risks is called enterprise and the person who bears these risks of business is called the entrepreneurThus an entrepreneur is one who organises production takes important decisions regarding production hires and purchases factors of production and bears the risk and uncertainty involved in productionOrganisation refers to the services of an entrepreneur who controls organises and undertakes all risks One who plans organises and manages a business enterprise is an organiser

Physics Chapter 1 Force

Force is an external agent capable of changing the state of rest or motion of a particular body It has a magnitude and a direction The direction towards which the force is applied is known as the direction of the force and the application of force is the point where force is applied The Force can be measured using a spring balance The SI unit of force is Newton (N)

Question 1

State the condition when on applying a force the body has

(a) the translational motion

(b) The rotational motion

Solutions

(a) Translational motion is produced when the body is free to move

(b) Rotational motion is produced when the body is pivoted at a point

Question 2

Define moment of force and state its SI unit

Solutions

The moment of force is equal to the product of the magnitude of the force and the perpendicular distance of the line of action of force from the axis

of rotation

The SI unit of moment of force is Newton times meter

= Newton meter (Nm)

Commercial Studies

Stake holders In this topic you will be come to know about the meaning and concept of stakeholders

How stakeholders are different from shareholders

Questions1 What do you mean by the term stake holdersAnswer) The term stake holders have developed from the words which mean an interest or expected benefit Stakeholders mean all those individuals groups and Institutions which have a state (interest) in the functioning and performance of a commercial organisation or a business enterprise2 What do you mean by share holdersAnswer) The person and Groups who own the shares of the joint stock company by providing capital to the company are called shareholders Shareholders are the internal stakeholders shareholders are one out of several stake holders3 How are shareholders different from stakeholdersAnswer)i) The term shareholders is related to only joint stock company whereas stakeholders are related with all business organisationsii) Stakeholders maybe any individual having financial stake in business organisation whereas a shareholders are those individuals who are holding shares in the company4) How are shareholders different from creditorsAnswer) i) Shareholders are internal stakeholders while creditors are external stakeholdersii) Shareholders invest in the capital of the company whereas creditors give loan to the companyiii) Shareholders are the members of the company with voting rights but creditors are not the members of the company

English 1 Transformation of sentences

Sentences A sentence is a group of words which makes complete sense

e Assertive sentencesf Imperative sentencesg Interrogative sentencesh Exclamatory sentences

Sentences can be changed from one grammatical form to another without changing the meaning of the sentence This is known as transformation of sentences

Exercise 1 Change the following affirmative sentences into Negative sentences

a He is a good manHe is not a bad man

b Ram loves SitaRam is not without love for Sita

c Only he stood first in the classNone but he stood first in the class

d Ankit was wiser than he

He was not so wise as Ankite He did it

He did not fail to do itf As soon as I reached college the

bell rangNo sooner did I reach college than the bell rang

g He finished everythingHe left nothing unfinished

h It always pours when it rainsIt never rains but it pours

Math Topic Commercial MathematicsChapter ndash Goods and services Tax

What is GSTAns It is a abbreviated term of Goods and Service Text which is an indirect tax levied on the sale of goods and rendering servicesSome terms related to GSTDelar Any person who buys goods or services For resale is known as a delar A delar Can be a firm or a companyIntra-state sales Sales of goods and services within the same state or same union territory are called intra- state salesInter-state sales Sales of goods and services outside the state or union territory are called Inter-state sales4) Input GST GST is paid by dealers on purchase of goods and services are called input GST5) Output GST GST is collected from customers on sale of goods and services are called output GST6) Types of GST There are three taxes applicable under GST(i) Central Goods and Services Tax (CGST)(ii) State Goods and Services Tax (SGST) or Union Territory Goods and Services Tax (UTGST) Both these taxes are levied on intra-state sales Here GST is divided equally among central and state governments(iii) Integrated Goods and Services Tax (IGST) IGST is levied on inter- state sales It is also levied on import of goods and services into India and export of goods and services from India

Subject Eng Literature (The Merchant of Venice ndash William Shakespeare)Topic Act III Scene 4 Lines 1 to 44 (Portia hellip To wish it back on you fare you well Jessica)[Students should read the original play and also the paraphrase given in the school prescribed textbook]

Summary Questions amp AnswersIn this scene we suddenly find a new element in the character of Portia We have already seen her possessed of every graceful womanly quality but now she shows that she is capable of rapid decision and determined action She shows this by her sudden resolve to hasten to Venice with a daring scheme for the rescue of Antonio This is an important scene in the dramatic action for it leads up to and renders possible the striking events of the famous trial scene which is one of the greatest striking elements of the play Moreover the fact that all the characters of importance are now assembled together in Venice makes the union of the main plot and the secondary story complete

(1) LORENZO Madam although I speak it in your presence(Line 1-9)

You have a noble and a true conceit

Of god-like amity which appears most strongly

In bearing thus the absence of your lordBut if you knew to whom you show this honourHow true a gentleman you send reliefHow dear a lover of my lord your husbandI know you would be prouder of the workThan customary bounty can enforce you

(a) Where is Lorenzo Why is he here To whom is he referring as lsquoMadamrsquo

Lorenzo is at Portiarsquos residence He had met Salerio on the way and Salerio had begged him to come along with him to

o In this scene Portia Nerissa Lorenzo Jessica and Balthazar appear

o Portia requests Lorenzo and Jessica to be in charge of her house during her absence from Belmont because she and Nerissa have decided to spend the days in meditation and also in visiting the holy places in the neighbourhood of Belmont She has already instructed her people to acknowledge both Lorenzo and Jessica as master and mistress of house during her absence Lorenzo and Jessica gladly agree to look after the house of Portia

handover the letter from Antonio to Bassanio The letter carried the bad news about Antoniorsquos arrest for non-payment of loan taken from Shylock Hence Salerio might have preferred company to break this bad news to Bassanio He is referring to Portia as Madam(b) What does Portia say on hearing the above extract

Portia says that she has never regretted doing good to others Friends who spend a lot of time together and really are there for each other have many traits in common As Antonio is Bassaniorsquos best friend saving him is like saving Bassanio who is like her own soul She asks Lorenzo to take care of management of the house till Bassanio is back(c) What does Portia send with Bassanio and why

On hearing about Antoniorsquos troubles on account of Bassanio her husband Portia immediately sends him with enough gold to repay the debt many times over to Venice to help Antonio out of his misfortune

(2) Lorenzo Madam with all my heart (Line 36-40)

I shall obey you in all fair commands

Portia My people do already know my mindAnd will acknowledge you and JessicaIn place of Lord Bassanio and myselfSo fare you well till we shall meet again

(a) Where are Lorenzo and Portia at this time What lsquofair commandsrsquo are given to Lorenzo

Lorenzo and Portia are at Belmont during this scenePortia reveals to Lorenzo that she has sworn to contemplate in prayer at a monastery around two miles away until her husband returns from Venice She tells him that Nerissa would accompany her and asks him to manage the house with Jessica till things are settled In response Lorenzo tells her that he would be obliged to do whatever she asks him to do(b) Where is Portia actually going and why

Portia tells Lorenzo that she would live a life of contemplation and pray at a monastery which is two miles away from her place In reality Portia plans to go to Venice in disguise with Nerissa and argue the case in defense of Antonio She is very sure that her plan would succeed

ClassXI (ScienceHumanitiesCommerce)Subject Topic Summary Execution

Computer Science

(APC)

Ch ndash 1 Numbers

(Numbers in different bases and

their Arithmatical operations)

Number System In computers Number System is defined as a writing system to represent the numbers in different ways ie we are using different symbols and notations to represent numbers There are four ways we can represent the number ndash Binary Decimal Octal and Hexadecimal

Decimal Number SystemThis number system consist 10 digits These are 0 1 2 3 4 5 6 7 8 amp 9

Binary Number SystemThis number system has only two digits these are 0 and 1 Here 0 stands for off while 1 stands for on

Octal Number SystemThis number system has 8 digits these are 0 1 2 3 4 5 6 amp 7

Hexadecimal Number SystemThis number system has 16 digits these are 0 1 2 3 4 5 6 7 8 9 A B C D E F Here the value of the alphabets are as follows A=10 B=11 C=12 D=13 E=14 F=15

Rules for conversion decimal number to Binary1 Divide the decimal number by 22 If the number will not divide equally by 2 then round down the answer to the nearest whole number (integer)3 Keep a note of the remainder it should be either 0 or 14 Keep repeating the above steps dividing each answer by 2 until you reach zero5 Write out all the remainders from bottom to top This is your binary solution

For example Lets convert 32 to binary 2 32 2 16 - 0 2 8 - 0 2 4 - 0 2 2 - 0 2 1 - 0 0 - 1

The binary equivalent of 3210 is 1000002

Try the follwing youself1 2410

2 4810

3 1210

History GROWTH OF NATIONALISM

The second half of the 19th century witnessed growth of political consciousness and a sense of Nationalism among the IndiansThere were various factors for growth of Indian Nationalism- As a result various political associations were formed in different provinces by the educated Indians Surendranath Banerjee organized a meeting of National conference at Calcutta Ultimately the National Congress was founded in Bombay in 1885This body became the vanguard of Indian struggle for freedom The congress leaders were known as moderates because they followed a policy of prayer and petition A large number of Indian leaders had experienced in political agitation The Political situation of England was also changed Moreover increasing revolutionary activities in Maharashtra Punjab and Bengal became serious concern to the British Government In this

QUESTION1 What do you mean by Nationalism ANSWER 1 Nationalism is defined as loyalty and devotion to own nation especially a sense of national consciousnessQUESTION 2 What are the causes of nationalism ANSWER 2 There were various factors for growth of nationalism

1 Spread of western education2 The progress of vernacular press and

patriotic literature3 The economic exploitation of our

country by the colonial rulers4 International affairs

QUESTION 3 Who organized National conference in Calcutta in 1883 ANSWER 3 Surendranath BanerjeeQUESTION 4 When did Indian National Congress formANSWER 4 Indian National Congress was formed in 1885 in BombayQUESTION 5 Who were ModeratesANSWER 5 The Early Nationalists were also known as Moderates Their emergence marked

background Lord Curzon became Viceroy in India He had no respect for the Indian National Congress

the beginning of the organized national movement in India They believed in British justice and were loyal to them They followed a policy of prayer and petition They demanded constitutional reforms of our country Impotant Moderate leaders were Pherozshah Mehta Dadabhai Naorozi and Surendranath Banerjee etcQUESTION 6 What do you know about Extremism in Indian National movementANSWER 6 In the beginning of 20th century a new class of national leaders emerged in India which was different from the moderate groups They started more aggressive movement against the British empire The goal of extremists was ldquoswarajrdquo Important extremist leaders were Bal Gangadhar Tilak Lala Lajpat Rai Bipin Chandra Pal etcQUESTION 7 Mention the places which were the main centres of Revolutionary movementANSWER 7 Maharashtra Bengal and Punjab

Physics

Chapter Dimensional Analysis

(Summary)

The dimensions of a physical quantity are the powers to which the fundamental units are raised in order to obtain the derived unit of that quantit

The physical quantites lengthmasstime are represented by [L] [M] [T] resp let they are raised to powers ( dimesions) abc resp then any physical quantity can be represented by [ La Mb Tc ] Examples

1 Area area = L x B = [L] x [L] = [M0 L2 T0 ]

2 Density density = massvolume = [M][L3] = [ M L-3]

3 Velocity velocity = distancetime = [L][T] = [LT-1]HW Try to find out dimension of acceleration Acceleration = velocity timeNB One can find the SI Units Using Dimension Analysis Such as for area we have [L2] so its SI unit is m2

Biology Topic ndash Chp-1 The living world

Today we will start the first chapter the living world Here we discuss about the characteristics of living organism and what are the difference between them and nonliving substances We also discuss about the contribution of different Scientists

There are over 500000 species of plants andover a million species of animal are present on earth Some 15000 new species were discovered every yearQ1 What is a living organismbull A living organism is primarily physico -chemical material that demonstrate a high degree of complexity is capable of selfRegulation possesses a metabolism and perpetuates itself through timeQ2 What are the differences between livingand non-livingsi) Compared with non-living living organisms

have more complex organised structure and their use of energy is more controlled amp efficientii) Living things reproduce their own kind by forming new cells which contains copies of their genesiii) Each organism has some degree of homeostasisie it is able to make adjustments so that internal environment remains constantQ3 Write contributions of following Scientists i) Aristotle - One of the first theories in Biology places all living things in a hiearchieii) AV Leeuwenhoek - was the first to observe living single celled organisms under microscopeii) Carolus Linnaeus - developed the binary system for naming of organisms and classificationiii) Geregor Johann Mendel ndash discoverbasic principles of inheritanceHomework i) C Darwin ii)Schleiden

Math Trigonometric functions

1 Overviewi) Trigonometry The word lsquotrigonometryrsquo is derived from the Greek words lsquotrigonrsquo and lsquometronrsquo which means measuring the sides of a triangle An angle is the amount of rotation of a revolving line with respect to a fixed line Usually we follow two types of conventions for measuring angles ie a) Sexagesimal system b) Circular system In Sexagesimal system the unit of measurement is Degree In Circular system the unit of measurement is Radian ii) Relation between degree and radianThe ratio of circumference of a circle to its diameter is always a constant This constant ratio is a number denoted by π which is taken approximately as 227The relationship between degree amp radian measurements is as follows2 right angles = 180deg= π radians1radian = 180degπ=57deg16(approx) 1deg=π180 radianiii) Length of an arc of a circleIf an arc of length s subtends an angle θ radians at the center of a circle of radius r then s=rθiv) Area of a sector of a circleA sector is like a pizza slice of the

Q) Express the following angles in radiana) 45deg b) 40deg3730Ans a) We have 180deg=π radiansi e 45deg= πtimes45180 radian = π4 radiansb) 40deg3730= 40deg37+3060 minute= 40deg 37 +12 minute= 40deg+ 752 minute=40 + 75(2times60) degree=3258 degreeNow 180deg=π radianie 3258 degree= (πtimes325) (180times8) radians = 65π288 radiansQ) A circle has a radius of r=12 meters What is the length of an arc traced out by a 60deg angle in the center of the circleAns In this problem we know both the central angle (60deg) and the radius of the circle (12) All we have to do is plug those values into our equation and we get

s = 2π(12)(60360)s = 24π6s = 4πSo the length of an arc traced out by a 60deg angle in a circle with a radius of 12 meters equals 4π meters asymp 1257 metersQ) Find the area of the sector with a central angle 30deg and a radius of 9cmAns GivenRadius r = 9 cmAngle θ = 30degArea of the sector = θ360degtimesπr2

= 30360degtimes227times92=2121cm2

circle It consists of a region bounded by two radii and an arc lying between the radiiThe area of a sector is a fraction of the area of the circle This area is proportional to the central angle In other words the bigger the central angle the larger is the area of the sectorArea of Sector = θ2 times r2 (when θ is in radians)

Area of Sector = θ times π360 times r2 (when θ is in degrees)

COMMERCE

CLASSIFICTION OF HUMAN ACTIVITIES-ECONOMIC AND NON-ECONOMIC

Welcome to the new sessiontoday we are going to start the first chapter of Class XI The name of the chapter that we are going to start is

lsquoClassification of Human Activities ndasheconomic and non-economicrsquo

Now let us start the chapter by considering human beings and the activities they perform throughout the day

Human activities means all those activities that human beings undertake to satisfy their wants

Human wants on the other hand are the desire of human beings for goods (vegetables fruits rice etc) and services (services of doctors teachers lawyers etc) that they require to live

Now these human activities continue throughout life as human wants are unending unlimited and recurring as human beings desire for better living throughout their lives

Now human activities can be classified into two categories

Human activities

Economic activities Non-economic activities

Economic activities are

Questions1 What are human activities

Answer Human activities mean all those activities that human beings undertake to satisfy their wants

Example A man working in an office

A boy playing in the garden

2What are the characteristics of human activitiesAnswer the characteristics of human activities are as follows

Human activities are undertaken by men women and children and these activities involve human efforts

Human activities are undertaken to satisfy human wants which are unlimited

Human activities continue throughout life

Human activities are performed for both earning money and personal satisfaction

3What is economic activitiesGive example

Answer Economic activities are undertaken by human beings with the object of earning money acquiring wealth and thereby satisfying human wantsExample

Selling of goods by a shop keeper to his customer

A clinic run by a doctor Service of a teacher in school or college

undertaken by human beings with the object of earning money and acquiring wealth

These activities result in the production of economic goods and services

Example Human activities(ie working in factories officesshops) which produce direct economic benefits

Non-economic activities are inspired by human sentiments and emotions such as love for the family desire to help the poor and love for the country

Thus these human activities (eg praying playing sleeping) produce no direct economic benefits and they are also not related to earning money and acquiring wealth

4 What are the characteristics of economic activities

Answer The characteristics of economic activities are as follows

Economic motiveEconomic activities are undertaken to earn money and acquire wealth

ProductiveEconomic activities involve productiondistribution and exchange of goods and services to create wealth

Economic growthEconomic activities determine the level of economic development of a country and standard of living of its citizens

Socially desirableEconomic activities are socially desirable for society

Economic resourcesEconomic activities make use of all the economic resources such landlabourcapital etc

5 What do you mean by non-economic activitiesExampleAnswerNon-economic activities are inspired by human sentiments and emotions such as love for the family desire to help the poor and love for the countryThese activities are not undertaken for monetary gain but for onersquos satisfaction and happinessExample

a mother looks after her children

a student donates blood8 Differentiate between Economic activities and Non-economic activities

Economic activities

Non-economic activities

1to earn living and acquiring wealth2Result can be measured in terms of money

3ExampleBusinessprofession and employment

1 to obtain some satisfaction

2Result cannot be measured in terms of money

3ExampleFamily-orientedreligious socialCultural and national

BUSINESS STUDIES

BUSINESS ENVIRONMENT

Welcome to the new sessionToday we are going to start the first chapter and the name of the chapter is Business Environment

In todayrsquos world every business enterprise is a part of the society It exists and operates in association with various groups in society such as customers suppliers competitors banks and financial institutions government agencies trade unions media and so on All these groups influence the functioning of business in one way or the other They constitute the environment of businessConcept of Business Environment

The term lsquobusiness environmentrsquo refers to the sum total of all individuals institutions and other forces that lie outside a business enterprise but that may influence its functioning and performance

The main features of business environment

Totality of External forces General and Specific forces Interrelatedness Complexity Dynamic Uncertainty Relativity

The Interrelation between business and its environment

The business enterprise is an open system It continuously interacts with its environment It takes inputs

Prepare the following questions from todayrsquos assignment

1 What do you mean by business environment

The term lsquobusiness environmentrsquo means the aggregate of all forces factors and institutions which are external to and beyond the control of an individual business enterprise but they may influence its functioning and performance Business environment is the macro framework within which a business firm a micro unit operates It consists of several interrelated and interacting elements

2 Explain the main features of business environment in brief

Totality of External forces-Business environment is the sum total of all things external to a business environment

General and Specific forces-It includes both the forces general forces are the economic social political legal and technological conditions which indirectly influence all business enterprise Specific forces are the investors customers competitors and suppliers which influence individual enterprise directly

Interrelatedness-Different elements of environment are interrelated for an example growing awareness for health care has increased the demand for health foods

Complexity- Business environment id

(such as raw materials capital labour energy and so on) from its environment transforms them into goods and services and sends them back to the environment

Fig 1 Business Environment Relationship

complex in nature as the elements keep on changing example economic technological and other forces changes in demand for a product and service

Dynamic-Business environment is not static it keeps on changing

Uncertainty- Itrsquos very difficult to predict future events such as technology and fashion which occur fast and frequently

Economics Basic Economic ConceptsSub topic

Microeconomics and

Macroeconomics

Welcome to the new sessiontoday we are going to start the first chapter of Class XI The name of the chapter that we are going to start is Basic Economic concepts

Now Economics covers the study of human activities Human activities are those activities which are performed by humans to satisfy their wants

Thus Human wants are unlimited and therefore economic activities such as production exchange and consumption are needed in order to satisfy those wants

The study of economics is divided largely in two parts which areMicroeconomics and Macroeconomics

SUBJECT- MATTER OF ECONOMICS

MICROECONOMICS MACROECONOMICS

Questions1Who has coined the words micro and macro economics

Answer Ranger Frisch coined the words lsquomicrorsquo and lsquomacrorsquo in 1933 to denote the two branches of economic theory namely microeconomics and macroeconomics

2What is microeconomicsAnswer It is the study of behaviour of individual decision ndash making unit such as consumers firms etc

3 What is macroeconomicsAnswer Macroeonomics is the study of overall economic phenomena like employment national income etc

4 What is the importance of microeconomicsAnswer

Microeconomics helps in formulating economic policies which enhance productive efficiency and results in greater social welfare

It helps the government in formulating correct price policies

It explains the working of a capitalistic economy where individual units(producers and consumers ) are free to take their own decision

Micro means a small part in

microeconomics we do not study the whole economy Hence we study an individual consumer and his or her choices and a producer and his or her profit maximizing decisions in the market Thus it does not mirror what happens in the economy as a whole

Macroeconomics on the other hand studies the economy as a whole It is concerned with aggregate and depicts the entire picture of the economyMacroeconomics deals with the national income aggregate investment aggregate consumption etc

Features of Microeconomics It deals with small

parts of the country Hence it looks at

individual consumers firms and industries

It deals with individual income consumption and savings

It studies the determination of price of any product or factors of production

It deals with the working of market via the price mechanism which is nothing but the determination of price and quantity of a commodity by the forces of demand and supply

Features of Macroeconomics

It deals with the study of the economy as a whole

It is concerned with

5 Give a limitation of microeconomics Microeconomics fails to explain the

functioning of an economy as a whole It cannot explain unemployment illiteracy and other problems prevailing in the country

6 What is the importance of macroeconomics It gives overall view of the growing

complexities of an economic system It provides the basic and logical

framework for formulating appropriate macroeconomic policies (eg for inflation poverty etc )to direct and regulate economy towards desirable goals

7What is the limitation of macroeconomics It ignores structural changes in an

individual unit of the aggregate

8 Differentiate between Microeconomics and Macroeconomics

Microeconomics Macroeconomics

the study of aggregates

National income aggregate savings and aggregate investments are major concepts dealt within macroeconomics style

It studies the determination of general price levels

It investigates into the problem of unemployment and the achievement of employment

It studies the aspect of decision making at the aggregate and national levels

It includes all growth theories whether related to developed or developing economies it also includes the study of economic systems and the working of the economy under different systems

Note Both Micro and macro economics are complementary and should be fully utilized for proper understanding of an economy

1It studies economic aspect of an individual unit2It deals with individual incomeConsumption and savings

3 It facilitates determination of price of any product or factors of production

4 Itrsquos scope is narrow and restricted to individual unit

1It studies the economy as a whole

2It deals with the national income aggregate consumption and aggregate savings3 It facilitates determination of general price level in an economy

4 Itrsquos scope is wide as it deals with economic units on the national level

ACCOUNTS

Introduction to Accounting and Book-keeping

Today I am going to share you the meaning of Accounting and Book-keeping and its related terms bullAccounting bullBook Keeping bullAccountsbullTypes Of Accounts bullAccounting Cycle

bull Meaning of accounting

Ans ) Accounting is the art and science of recording classifying and summarising monetary transactions

bull Meaning of Book-keeping

Ans) Bookkeeping is the art of recording business transactions with the view of having a permanent record of them and showing their effect on wealth

bull Meaning of account

Ans) The term account means a record of

business transactions concern a particular person of firm asset or income or expense It is a summarised record of all transactions which take place in an accounting year

bull Types of accountsPersonal accounts ndash Personal accounts relating

to person and Organisation are known as personal accounts Example Ramrsquos Account ABC amp Co Account etc

Real account - The accounts related to tangible and intangible assets are called real accounts Example Cash Account Furniture Account etc

Nominal account- Accounts related to expenses losses incomes and gains are known as nominal accounts Example Wages Account Salary Account Discount Account etc

bull Accounting cycle Accounting cycle refers to a complete sequence of accounting activities It begins with recording of transactions and ends with the preparation of a balance sheet

Chemistry TopicAtomic Structure

Thomsonrsquos atomic modelThomson (1898) was the first to propose the model of an atomHe proposed that an atom can be regarded as a uniform sphere of positive electricity in which requisite number of electrons are embedded evently to neutralize the positive chargeThis is just like plums embedded in a pudding or seeds evently distributed in red spongy mass of a watermelonThis model of atom is known as ldquoPlum-Pudding modelrdquo or

Q1)What is the fundamental constituents of atomAns Electron Proton and neutrons are the fundamental constituents of atomQ2)What is the value of fundamental unit of electricityAnsThe charge carried by one electron is sad to be the fundamental unit of electricityIts magnitude is 48times10-10esuOr 1602times10-19C Q3)Name the element containing no neutronAnsOrdinary hydrogen atom or protium 1H1

Types of AccountPersonal AccountReal AccountNominal AccountBalance Sheet (opening)

ldquowatermelon modelrdquoThis model could explain the electrical neutrality of an atom but failed to explain the result of scattering experiment carried out by Rutherford in 1911So it was rejected ultimately

Q4)Why is an electron called universal particleAns Itrsquos mass and Charge are independent of its source

EVS Chapter 1 ndash Modes of Existence

Modes of existence When one speaks normally about the mode of existence of some group or individual one refers to their customs their mode of being their ethology their habitat in some way their feeling for a placeDifferent modes of exixtence are ndash

1 Hunting ndashGathering2 Pastoral3 Agricultural4 Industrial

1 Hunting and gathering Hunting and gathering mode of existence is characterized by obtaining food from hunting wild animals including fishing and gathering wild plants From their earliest days the hunter-gatherer diet included various grasses tubers fruits seeds and nuts Lacking the means to kill larger animals they procured meat from smaller game or through scavenging

Societies that rely primarily or exclusively on hunting wild animals fishing and gathering wild fruits berries nuts and vegetables to support their diet are called hunting and gathering societies

At least this used to be practice of human beings before agriculture is invented As their brains evolved hominids developed more intricate knowledge of edible plant life and growth cycles

Q) Write the features of Hunting ndash gathering societiesAns - There are five basic characteristics of hunting and gathering societies

i The primary institution is the family which decides how food is to be shared and how children are to be socialized and which provides for the protection of its members

ii They tend to be small with fewer than fifty members

iii They tend to be nomadic moving to new areas when the current food supply in a given area has been exhausted

iv Members display a high level of interdependence

v Labor division is based on sex men hunt and women gather

Political Science

Introduction to political science

Political science occasionally called politology is a social science which deals with systems of governance and the analysis of political activities political thoughts associated constitutions and political behaviorThe study of political science involves the study of both the

Answer the following questions-1 What is political science

Political science occasionally called politology is a social science which deals with systems of governance and the analysis of political activities political thoughts associated constitutions and political behavior

2 Short notes-

traditional and modern theories of politicsTraditionalClassical political sciencepolitical theory-Traditional political science is the study of politics before Second World War The methodology to study Politics was traditional (legal formaletc) the definition of politics traditional (Politics begins and end with state)area of study (constitution state machinery)was traditionalModern Political scienceModern political theory-Modern Political Theory critically examines the contemporary state of political theory making an assessment of the achievement and limitations of the Behavioural Revolution in its totality and reviews objectively the major paradigms and conceptual frameworks adopted by the disciplineContemporary attempts at the development of an integrated political theory involving the use of both traditional and modern concepts approaches and theories-Around late 1960s several political scientists realized the importance of both the traditional political theory and modern Political theory They began building an integrated theory of politics involving a systematic mixture of traditional and modern studies of politics It was held that the study of a complex and vast field like politics needs both traditional as well as

Classical political theory Modern Political theory

Homework-Learn

modern concepts and approaches for studying itrsquos all aspects

Subject Eng Literature (The Tempest ndash William Shakespeare) Topic Act I Scene 1 Lines 1 to 32 (Line 32 ndash Gonzalo hellip If he be not born to be hanged our case is miserable) Date 13th April 2020 (3rd Period)

[Students should read the original play and also the paraphrase given in the school prescribed textbook]Summary Questions amp Answers

[SUMMARY OF THE ENTIRE SCENE]

o The play starts with the scene of a severe storm at sea Alonso (King of Naples) Sebastian (Alonsorsquos brother) Ferdinand (Alonsorsquos son) Gonzalo Antonio (the usurping Duke of Milan) are in a ship in the midst of the storm

o The mariners are trying their best to control the vessel from running aground and are totally following the orders of their Master the Boatswain They have scant success

o The mariners become extremely unhappy and annoyed when most of the passengers arrive on the deck thereby hampering their effort to save the ship There is serious confrontation between them and the passengers who are part of the Kingrsquos entourage

o The mariners could not save the ship

SUMMING-UP

(i) Vivid description of the scene which gives a realistic description of terror and confusion of a tropical storm

(ii) Shows Shakespearersquos accuracy of knowledge in describing the naval operations and also matters of seamanship

(iii) The opening scene justifies the title ndash The Tempest

UNANSWERED QUESTIONS

(i) The King always travels with his entire fleet including his soldiers Where

(1) GONZALO Nay good be patient (Line 15-26)BOATSWAIN When the sea is Hence What cares these

roarers for the name of the king To cabin silence Trouble us not

GONZALO Good yet remember whom thou has aboardBOATSWAIN None that I more love than myself You are a

councillor if you can command these elements to silence and work

the peace of the present we will not hand a rope more use your authority If you cannot give thanks you have

lived so long and make yourself ready in your cabin for the mischance of the hour if it so hap [To the Mariners]

Cheerly good hearts [To Gonzalo] Out of our way I say

(a) To whom is the boatswain speaking What does he mean by lsquoNone that I more love than myselfrsquo

The Boatswain is speaking to Gonzalo the honest old councilor of the Duke of MilanBy using the words ndash lsquoNone that I love more than I love myselfrsquo means that for the Boatswain nobody is dearer to him than his own life

(b) What were the conditions that made the boatswain react in this way

The Boatswain reacts in this way because the storm is at sea and Alonso King of Naples Sebastian his brother Ferdinand his son Gonzalo Antonio the usurping Duke of Milan on board are in distress and in panic Thus they have rushed to the deck interrupting the work of the mariners

(c) What hope does Gonzalo take from the attitude of the boatswain

The insolent and authoritative attitude of Boatswain makes Gonzalo feel comforted He tells that there are no signs that the Boatswain will be drowned But his facial appearance and attitude shows that he is destined to die on land by hanging which in effect means that all on board will be saved Otherwise all the persons on board are doomed

(d) How can they lsquomake yourself ready in your cabinrsquo For what were they asked to make ready themselves

In order to make themselves ready in their cabin the

were the other ships

(ii) Why was the ship in that area Where was it coming from or going where

(iii) The ship broke apart What happened to those who were in the ship

passengers on board must prepare for death which they will possibly soon have to meetThey can retire to their cabins and offer prayers to the Almighty to save them from drowning

(e) What does the boatswain say when he is asked to be patient What does he order to the royal party

When the boatswain is asked to be patient and remain calm he says that he will be patient only when the storm will be over and the sea will be calm but as long as the storm blows and there is danger to the ship he cannot think of being patient He orders the royal party to go to the cabin and leave the mariners to their work

(2) GONZALO I have great comfort from this fellow (Line 27-36)

Methinks he hath no drowning mark upon him his complexion is perfect

gallows Stand fast good Fate to his hanging Make the rope of his destiny our cable for our own doth little advantage If he be not born to be hanged our case is miserable

(a) Why does Gonzalo regard the Boatswain in the midst of danger

In the midst of danger Gonzalo regards the boatswain because he feels that the Boatswain is a source of comfort and is bent upon to do his work sincerely which in this case is saving the ship and its passengers from the severest of raging storm

(b) What reasons does Gonzalo give when he says that none in the ship will die of drowning

Gonzalo is almost sure that none in the ship will die by drowning His says that there is no mark on the face of the boatswain that indicates that he will die by drowning On the other hand the lines on his face are strong indications that he will be hanged to death Therefore there shall be no danger of the shiprsquos sinking

(c) Explain the following ldquoStand fast good Fate to his hanging Make the rope of his destiny our cable for our own doth little advantage If he be not born to be hanged our case is miserablerdquo

The stated lines mean that if the will of destiny is to be carried out then the ship will not get wrecked and all the passengers will be saved The safety of the passengers therefore depends upon the will of fate being carried out in the case of the boatswain If however the boatswain is not to die by hanging then the passengers are also very unsafe because in that case the ship is likely to sink

(d) What order does the Boatswain give to the sailors

when he re-enters What does he say about the crying of the fellows inside the cabin

The boatswain orders the sailors to bring the topmast lower and bring the ship close to a stationary position with the help of the main sail He says that the fellows inside the cabin are moaning and crying in their distress louder than his voice and louder even than the roaring of the storm

Class XII (ScienceCommerceHumanities) Subject Topic Summary Execution

Computer Science

PropositionalLogic

Propositional logic is a procedure to provide reasoning through statementProposition A ststement that results in True or False is said to be proposition There are two types of propositionSimple proposition amp compound propositionSimple proposioton A simple proposition is one that is not a part of any other proposition Such sentential form of proposition is symbolized with english letters in short For example Ram is a claver student (TrueFalse)Where do you live (Not in True or False)Grapes are sweet (TrueFalse)It rains today (TrueFalse)Here we can see some statements anwer would be true or false but some staements answer can not give in terms of true or false Thus the sentences which can be answered in true or false are known as simple propositionAssigning propositon to a variableThe general syntax to assign propostion to a variable is as followsVariable = Simple propositonFor example A=Ram is a clever studentB= Grapes are sweetC= it rains todayCompound proposition

helliphellipto be continued in next classhelliphellipMath Relation Relation If A and B are two non-empty sets

then a relation R from A to B is a subset of AxB If R A x B and (a b) R then we say that a sube isinis related to b by the relation R written as aRbeg Let A be the set of students of class XII and B be the set of students of class XI Then some of the examples of relation from A to B arei) (a b) AXB a is brother of bisinii) (a b) AXB age of a is more than age of isinb Types of relation In this section we would like to study different types of relations We know that a relation in a set A is a subset of A times A Thus the empty set φ and A times A are two extreme relations For illustration consider a relation R in the set A = 1 2 3 4 given by R = (a b) a ndash b = 10 This is the empty set as no pair (a b) satisfies the condition a ndash b = 10 Similarly R = (a b) | a ndash b | ge 0 is the whole primeset A times A as all pairs (a b) in A times A satisfy | a ndash

Example 1 Let A be the set of all students of a boys school Show that the relation R in A given by R = (a b) a is sister of b is the empty relation and R = (a b) the primedifference between heights of a and b is less than 3 meters is the universal relationSolution Since the school is boys school no student of the school can be sister of any student of the school Hence R = φ showing that R is the empty relation It is also obvious that the difference between heights of any two students of the school has to be less than 3 meters This shows that R = A times A is primethe universal relation Example 2 Show that the relation R in the set 1 2 3 given by R = (1 1) (2 2) (3 3) (1 2) (2 3) is reflexive

b | ge 0 These two extreme examples lead us to the following definitionsDefinition 1 A relation R in a set A is called empty relation if no element of A isrelated to any element of A ie R = φ A times AsubDefinition 2 A relation R in a set A is called universal relation if each element of A is related to every element of A ie R = A times A Both the empty relation and the universal relation are some times called trivial relation Definition 3 A relation R in a set A is called(i) reflexive if (a a) R for every a Aisin isin(ii) symmetric if (a1 a2) R implies that (aisin 2a1)

R for all aisin 1 a2 Aisin(iii) transitive if (a1 a2) R and (aisin 2 a3) R isinimplies that (a1 a3) R for all aisin 1 a2 a3 AisinDefinition 4 A relation R in a set A is said to be an equivalence relation if R is reflexive symmetric and transitive

but neither symmetric nor transitiveSolution R is reflexive since (1 1) (2 2) and (3 3) lie in R Also R is not symmetric as (1 2) R but (2 1) isin notinR Similarly R is not transitive as (1 2) R and (2 3) R but (1 3) R isin isin notinExample 3 Show that the relation R in the set Z of integers given byR = (a b) 2 divides a ndash b is an equivalence relationSolution R is reflexive as 2 divides (a ndash a) for all a Z isinFurther if (a b) R then 2 divides a isinndash b Therefore 2 divides b ndash a Hence (b a) R which shows that R is isinsymmetric Similarly if (a b) R and (b c) R isin isinthen a ndash b and b ndash c are divisible by 2 Now a ndash c = (a ndash b) + (b ndash c) is even (Why) So (a ndash c) is divisible by 2 This shows that R is transitive Thus R is an equivalence relation in ZExample 4 Let L be the set of all lines in a plane and R be the relation in L defined as R = (L1 L2) L1 is perpendicular to L2 Show that R is symmetric but neither reflexive nor transitiveSolution R is not reflexive as a line L1 can not be perpendicular to itself ie (L1 L1) R notinR is symmetric as (L1 L2) Risin

L1 is perpendicular to L2rArr L2 is perpendicular to L1rArr (L2 L1) RrArr isin

R is not transitive Indeed if L1 is perpendicular to L2 and L2 is perpendicular to L3 then L1 can never be perpendicular to L3 In fact L1 is parallel to L3 ie (L1 L2) R isin(L2 L3) R but (L1 L3) Risin notin

Chemistry Solid state Characteristics if Solids(i)The particles are locked in fixed positions they are unable to change their relative positions and this brings a definite shape and volume of a solid(ii)In a solid the constituent particles are held by strong forces of attractionThe forces of attraction may be bonding or non bonding(iii)The constituent particles in a solid pack together as closely as possibleoccupying most of the available space within the solidThus the empty space in a solid is very smallThis makes a solid highly rigid and nearly incompressibleThis also explains why a solid has high density and exhibits slow diffusionClassification of Solids

Q1)Define Crystalline solids AnsA Solid that has a definite geometrical shape and a sharp melting pointand whose constituent particles (atomsmolecules or ions) are arranged in a long range order of definite pattern extending throughout the solid is called a crystalline solidExNaClQ2)Define Amorphous solids AnsA solid that does not have a definite shape and a sharp melting pointand whose constituent particles (atomsmolecules or ions) are not arranged in a definite pattern is called an amorphoussolid

Crystalline solidsAmorphous solids

ExGlassRubberQ3)Classify Crystalline Solids Crystalline Solids

Physics Coloumbrsquos Law (Summary)

Before Going Into Coloumbrsquos Law We Will First Learn What is Charge Properties of Charge and Always remember that charge is quantized ie a body always have static charge of magnitude equal to some integral multiple of fundamental electronic charge e= 16 x 10- 19 C

Charge is the property of matter that causes it to produce and experience electrical and magnetic effects The study of the electrical charges at rest is called electrostatics When both electrical and magnetic effects are present the interaction between charges is referred to as electromagnetic

There exist two types of charges in nature positive and negative Like charges repel and unlike charges attract each other

The type of charge on an electron is negative The charge of a proton is the same as that of an electron but with a positive sign In an atom the number of electrons and the number of protons are equal The atom is therefore electrically neutral If one or more electrons are added to it it becomes negatively charged and is designated as negative ion However if one or more electrons are removed from an atom it becomes positively charged and is called a positive ion

The excess or deficiency of electrons in a body gives the concept of charge If there is an excess of electrons in a body it is negatively charged And if there is deficiency of electrons the body becomes positively charged Whenever addition or removal of electrons takes places the body acquires a charge

The SI Unit of charge is coulomb (C) In SI units the current is a fundamental quantity having a unit of ampere (A) The unit of charge is defined in terms of the unit of current Thus one coulomb is the charge transferred in one second across the section of a wire carrying a

Ionic SolidsMetallicSolids

Molecular Solids

current of one ampere

As q = It we have1 C = (1 A) (1 s)

The dimensions of charge are [A T]

Properties of Charge

(1) Quantization of Charge Electric charge can have only discrete values rather than any value That is charge is quantized The smallest discrete value of charge that can exist in nature is the charge on an electron given as

e = plusmn 16 x 10- 19 C

This is the charge attained by an electron and a protonA charge q must be an integral multiple of this basic unit That is

Q = plusmn ne where n = 1 2 hellip

Charge on a body can never be (frac12)e (23)e or 57e etcWhen we rub a glass rod with silk some electrons are transferred from the rod to the silk The rod becomes positively charged The silk becomes negatively charged The coulomb is a very large amount of charge A typical charge acquired by a rubbed body is 10 - 8 C

Biology Reproduction in organisms

Welcome to this new session 2020-21Today in this first chapter we mainly discuss about reproduction types needs and life span of some organismsWe also discuss about difference between sexual and asexual reproduction

Q1 What is reproductionReproduction is defined as a biological processin which an organism gives rise to young onessimilar to itselfQ2 What are the needs of reproductionbulli) Reproduction maintain life on earthii) It enables the continuity of the species generation after generationiii) It creates genetic variation among populationsQ3 Define Life span and write some orgnisms life spanbull Life span is the period from birth to

the natural death of an organism- OrganismsLife span1 Butterfly 1 - 2 weeks2 Fruit fly 30 days3Dog 10-13 years4 Rose5-7 years5 Tortoise100-150 years6 Banyan Tree -200 - 250 yearsQ4 Reproduction is of two types in case ofanimals but in case of plants vegetative propagation is also present

Asexual Reproduction Sexual Reproductioni) Always uniparentalii) Gametes are not involvediii) Only mitotic division involvediv) Somatic cells of parents are involvedv) Offsprings are genetically similar to the parents

i) Usually biparentalii) Gametes are involvediii) Meiosis occurs during gametogenesis Mitosis occurs after fertilisationiv) Germ cells of the parents are involvedv) offsprings are genetically different from the parents

COMMERCE BUSINESS ENVIRONMENT

Welcome to the new sessiontoday we are going to start the first chapter of Class XII The name of the chapter is Business Environment

Already many of you have got some idea about the word business environment form the first chapter of business studies in class XI

In todayrsquos world every business enterprise is a part of the society It exists and operates in association with various groups in society such as customers suppliers competitors banks and financial institutions government agencies trade unions media and so on All these groups influence the functioning of business in one way or the other They constitute the environment of businessConcept of Business Environment

The term lsquobusiness environmentrsquo refers to the sum total of all individuals institutions and other forces that lie outside a business enterprise but that may influence its functioning and performance

The main features of business environment Totality of External forces General and Specific forces Interrelatedness Complexity Dynamic Uncertainty

Prepare the following questions from todayrsquos assignment

2 What do you mean by business environment

The term lsquobusiness environmentrsquo means the aggregate of all forces factors and institutions which are external to and beyond the control of an individual business enterprise but they may influence its functioning and performance Business environment is the macro framework within which a business firm a micro unit operates It consists of several interrelated and interacting elements

2 Explain the main features of business environment in brief

Totality of External forces-Business environment is the sum total of all things external to a business environment

General and Specific forces-It

Relativity

The Interrelation between business and its environment

The business enterprise is an open system It continuously interacts with its environment It takes inputs (such as raw materials capital labour energy and so on) from its environment transforms them into goods and services and sends them back to the environment

Fig 1 Business Environment Relationship

includes both the forces general forces are the economic social political legal and technological conditions which indirectly influence all business enterprise Specific forces are the investors customers competitors and suppliers which influence individual enterprise directly

Interrelatedness-Different elements of environment are interrelated for an example growing awareness for health care has increased the demand for health foods

Complexity- Business environment id complex in nature as the elements keep on changing example economic technological and other forces changes in demand for a product and service

Dynamic-Business environment is not static it keeps on changing

Uncertainty- Itrsquos very difficult to predict future events such as technology and fashion which occur fast and frequently

Business Studies

Human Resources Management

Human resource of an organisation are the aggregate of knowledge skills attitudes of people working in it

The management system which deals with human resources is called human resource management

Features of HRMbullComprehensive functionbullPeople-oriented

Question1) What do you mean by human

resource management Answer) Human resource management may be defined as that field of Management which has to do with planning organising and controlling the functions of procuring developing maintaining and utilising the labour force

bullAction oriented bullPervasive function bullContinuous function

2) Explain the features of HRM in brief

Answer)bullHuman Resource Management is concerned with managing people at work bull Human Resource Management is concerned with employees which bring people and organisations together so that the goals of each are met bullHuman resource management considered every employees as an individual and also promote their satisfaction and growth bull Human resource management is inherent in all organisations and at all levelsbullManagement of human resources are ongoing on never ending process which requires a constant alertness and Awareness of human relations

3) ldquoHR function is said to be pervasiverdquowhy

Answer) Human resource management is required in all organisations whether it is private or government organisations armed forces sports organisations etc It permeatsall the functional areas like production marketing finance research etc This from this feature of human resource management it can be said that it is pervasive in nature

Economics Demand Q1DEFINITION OF DEMANDIn economics demand is the quantity of a good that consumers are willing and able to purchase at various prices during a given period of timeQ2DEMAND CURVEIn economics a demand curve is a graph depicting the relationship between the price of a certain commodity and the quantity of that commodity that is demanded at that pricQ3LAW OF DEMANDIn microeconomics the law of demand states that conditional on all else being equal as the price of a good increases quantity demanded decreases conversely as the price of a good decreases quantity demanded increasesQ4ASSUMPTION of LAW OF DEMAND(i)No change in price of related commodities(ii) No change in income of the consumer(iii) No change in taste and preferences customs habit and fashion of the consumer( No expectation regarding future change in priceQ5MARKET DEMAND SCHEDULEIn economics a market demand schedule is a tabulation of the quantity of a good that all consumers in a market will purchase at a

given price At any given price the corresponding value on the demand schedule is the sum of all consumersrsquo quantities demanded at that priceQ6INDIVIDUAL DEMAND SCHEDULEIndividual demand schedule refers to a tabular statement showing various quantities of a commodity that a consumer is willing to buy at various levels of price during a given period of timeQ7 FACTORS AFFECTING INDIVIDUAL DEMAND FOR A COMMODITY

The factors that influence a consumerrsquos decision to purchase a commodity are also known as determinants of demand The following factors affect the individual demand for a commodity1 price of the commodity2 price of related goods3 income of buyer of the commodity4 tastes and preferences of the buyer1 Price of the CommodityYou must have observed that when price of a commodity falls you tend to buy more of it and when its price rises you tend to buy less of it when all other factors remain constant (lsquoother things remaining the samersquo) In other words other things remaining the same there is an inverse relationship between the price of a commodity and its quantity demanded by its buyers This statement is in accordance with law of demand which you will study in the later part of this lesson Price of a commodity and its quantity demanded by its buyers are inversely related only when lsquoother things remain the samersquo So lsquoother things remaining the samersquo is an assumption when we study the effect of changes in the price of a commodity on its quantity demanded2 Price of Related goodsA consumer may demand a particular good But while buying that good heshe also asks the price of its related goods Related goods can be of two types-(i) Substitute goods(ii) Complementary goods While purchasing a good prices of its substitutes and complements do affect its quantity purchased(i) Price of Substitute Goods Substitute goods are those goods which can easily be used in place of one another for satisfaction of a particular want like tea and coffee An increase in price of substitute good leads to an increase in demand for the given commodity and a decrease in price of substitute good leads to a decrease in demand for the given commodity It means demand for a given commodity is directly affected by change in price of substitute goods For example if price of coffee increases the demand for tea will rise as tea will become relatively cheaper in comparison to coffee(ii) Price of Complementary goods Complementary goods are those goods which are used together to satisfy a particular want like car and petrol An increase in the price of complementary goods leads to a decrease in demand for the given commodity and a decrease in the price of complementary goods leads to an increase in demand for the given commodity For example if price of petrol falls then the demand for cars will increase as it will be relatively cheaper to use both the goods together So demand for a given commodity is inversely affected by change in price of complementary goods3 Income of the Buyer of CommodityDemand for a commodity is also affected by income of its buyer However the effect of change in income on demand depends on the nature of the commodity under consideration In case of some goods like full cream milk fine quality of rice (Basmati rice) etc demand for these commodities increases when income of the buyer increases and

demand for these commodities decreases when income of the buyer decreases Such goods whose demand increases with the increase in income of the buyer are called normal goods But there are some goods like coarse rice toned milk etc whose demand decreases when income of buyer increases and their demand increases when income of the buyer decreases Such goods whose demand decreases with the increase in income of the buyer are called inferior goods Suppose a consumer buys 10 Kgs of rice whose price is ` 25 per Kg He cannot afford to buy better quality of rice because the price of such rice is ` 50 per Kg The consumer is spending ` 250 per month on the purchase of rice Now if income of the consumer increases and he can afford ` 350 on purchase of 10 Kg of rice Now he can afford to buy some quantity of rice say 6 Kgs whose price is ` 25 per Kg and may buy 4 Kgs of rice whose price is ` 50 per Kg Thus he will buy 10 Kgs of rice by spending ` 350 per month Therefore we may conclude that demand for normal goods is directly related to the income of the buyer but demand for inferior goods is inversely related to the income of the buyer4 Tastes and Preferences of the BuyerThe demand for a commodity is also affected by the tastes and preferences of the buyers They include change in fashion customs habits etc Those commodities are preferred by the consumers which are in fashion So demand for those commodities rises which are in fashion On the other hand if a commodity goes out of the fashion its demand falls because no consumer will like to buy it(5) Number of Buyers in the Market(Population)Increase in population raises the market demand whereas decrease in population reduces the market demand for a commodity Not only the size of population but its composition like age (ratio of males females children and old people in population) also affects the demand for a commodity It is because of needs of children young old male and female population differs(6) Distribution of Income and WealthIf the distribution of income and wealth is more in favour of the rich demand for the commodities preferred by the rich such as comforts and luxuries is likely to be higher On the other hand if the distribution of income and wealth is more in favour of poor demand for commodities preferred by the poor such as necessities will be more(7) Season and Weather ConditionsThis is generally observed that the demand for woolens increases during winter whereas demand for ice creams and cold drinks increases during summer Similarly market demand for umbrellas rain coats increases during rainy seasonQ8 REASONS FOR OPERATION OF LAW OF DEMAND WHY DEMAND CURVE SLOPES DOWNWARDNow we will try to explain why does a consumer purchase more quantity of a commodity at a lower price and less of it at a higher price or why does the law of demand operate ie why does the demand curve slope downwards from left to right The main reasons for operation of law of demand are1 Law of Diminishing Marginal UtilityAs you have studied earlier law of diminishing marginal utility states that as we consume more and more units of a commodity the utility derived from each successive unit goes on decreasing The consumer will be ready to pay more for those units which provide him more utility and less for those which provide him less utility It implies that he will purchase more only when the price of the commodity falls2 Income Effect

When price of a commodity falls purchasing power or real income of the consumer increases which enables him to purchase more quantity of the commodity with the same money income Let us take an example Suppose you buy 4 ice creams when price of each ice cream is ` 25 If price of ice creams falls to ` 20 then with same money income you can buy 5 ice creams now3 Substitution EffectWhen price of a commodity falls it becomes comparatively cheaper as compared to its substitutes (although price of substitutes has not been changed) This will lead to rise in demand for the given commodity For example if coke and Pepsi both are sold at ` 10 each and price of coke falls Now coke has become relatively cheaper and will be substituted for Pepsi It will lead to rise in demand for coke4 Change in Number of BuyersWhen price of a commodity falls some old buyers may demand more of the commodity at the reduced price and some new buyers may also start buying this commodity who were not in a position to buy it earlier due to higher price This will lead to increase in number of buyers when price of the commodity falls As a result demand for the commodity rises when its price falls5 Diverse Uses of a CommoditySome commodities have diverse uses like milk It can be used for drinking for sweet preparation for ice cream preparation etc If price of milk rises its use may be restricted to important purpose only This will lead to reduction in demand for other less important uses When price of milk falls it can be put to other uses also leading to rise n demand for itQ9 EXCEPTIONS TO THE LAW OF DEMANDYou have studied in law of demand that a buyer is willing to buy more quantity of a commodity at a lower price and less of it at a higher price But in certain circumstances a rise in price may lead to rise in demand These circumstances are called Exceptions to the Law of Demand Some important exceptions are1 Giffen GoodsGiffen goods are special type of inferior goods in which negative income effect is stronger than negative substitution effect Giffen goods do not follow law of demand as their demand rises when their price rises Examples of Giffen goods are jowar and bajra etc2 Status Symbol GoodsSome goods are used by rich people as status symbols eg diamonds gold jewellary etc The higher the price the higher will be the demand for these goods When price of such goods falls these goods are no longer looked at as status symbol goods and tehrefore therir demand falls3 NecessitiesCommodities such as medicines salt wheat etc do not follow law of demandbecause we have to purchase them in minimum required quantity whatever their price may be4 Goods Expected to be ScarceWhen the buyers expect a scarcity of a particular good in near future they start buying more and more of that good even if their prices are rising For example during war famines etc people tend to buy more of some goods even at higher prices due to fear of their scarcity in near future

Political Science

Constitution of India-The

Preamble

The preamble-

Preamble-

The preamble is the most precious part of the constitution We the people of India having solemnly resolved to constitute India into a Sovereign Socialist Secular Democratic Republic and to secure to all its citizensA preamble is an introductory and expressionary statement in a document that explains the documents purpose and underlying philosophy When applied to the opening paragraphs of a statute it may recite historical facts pertinent to the subject of the statuteNature and purpose of the constitution-Purpose of the Constitution dictates permanent framework of the government to form a more perfect union to establish justice and ensure peace of thenationconstitution provide principles how the government can run itself following the rules and laws written in the constitution of each state keeps them balanced

Answer the following questions-

1 What is preambleA preamble is an introductory and expressionary statement in a document that explains the documents purpose and underlying philosophy2 What is the nature and

purpose of the constitutionConstitution dictatespermanent framework of the government to form a more perfect union to establish justice and ensure peace of the nation

Homework-Learn

Accounts Compatibilty mode

1MEANING OF PARTNERSHIPPartnership is a form of business organisation where two or more persons join hands to run a business They share the profits and losses according to the agreement amongst them According to the Indian Partnership Act 1932 ldquoPartnership is relation between persons who have agreed to share profits of a business carried on by all or any one of them acting for allrdquo For example one of your friends has passed class XII from National Institute of Open Schooling (NIOS) and wants to start a business Heshe approaches you to join in this venture Heshe wants you to contribute some money and participate in the business activities Both of you if join hands constitute a partnership2CHARACTERISTICS1048698 Agreement A partnership is formed by an agreement The agreement may be either oral or in writing It defines the relationship between the persons who agree to carry on business It may contain the terms of sharing profit and the capital to be invested by each partner etc The written agreement is known as partnership deed1048698 Number of persons There must be at least two persons to form a partnership

The maximum number of partners in a partnership firm can be 50 according toCompanies Act 20131048698 Business The Partnership is formed to carry on business with a purpose of earning profits The business should be lawful Thus if two or more persons agree to carry on unlawful activities it will not be termed as partnership1048698 Sharing Profits The partners agree to share profits in the agreed ratio In caseof loss all the partners have to bear it in the same agreed profit sharing ratio10486981048698Mutual Agency Every partner is an agent of the other partners Every partner can bind the firm and all other partners by hisher acts Each partner will be responsible and liable for the acts of all other partners10486981048698Unlimited liability The liability of each partner except that of a minor is unlimited Their liability extends to their personal assets also If the assets of the firm are insufficient to pay off its debts the partnersrsquo personal property can be used to satisfy the claim of the creditors of the partnership firm10486981048698Management All the partners have a right to mange the business However they may authorize one or more partners to manage the affairs of the business on their behalf10486981048698Transferability of Share No partner can transfer hisher share to any one including hisher family member without the consent of all other partners3PARTNERSHIP DEEDAgreement forms the basis of partnership The written form of the agreement is which a document of partnership is It contains terms and conditions regarding the conduct of the business It also explains relationship between the partners This document is called partnership deed Every firm can frame its own partnership deed in which the rights duties and liabilities of the partners are stated in detail It helps in settling the disputes arising among the partners during the general conduct of business 4CONTENTS OF PARTNERSHIP DEEDThe partnership deed generally contains the following (i) Name and address of the partnership firm(ii) Nature and objectives of the business(iii) Name and address of each partner(iv) Ratio in which profits is to be shared(v) Capital contribution by each partner(vi) Rate of Interest on capital if allowed(vii) Salary or any other remuneration to partners if allowed(viii) Rate of interest on loans and advances by a partner to the firm(ix) Drawings of partners and interest thereon if any(x) Method of valuation of goodwill and revaluation of assets and liabilities on the reconstitution of the partnership ie on the admission retirement or death of a partner(xi) Settlement of disputes by arbitration(xii) Settlement of accounts at the time of retirement or death of a partner5IN ABSENCE OF PARTNERSHIP DEEDThe partnership deed lays down the terms and conditions of partnership in regard to rights duties and obligations of the partners In the absence of partnership deed there may arise a controversy on certain issues like profit sharing ratio interest on

capital interest on drawings interest on loan and salary of the partners In such cases the provisions of the Indian Partnership Act becomes applicableSome of the Issues are(i) Distribution of Profit Partners are entitled to share profits equally(ii) Interest on Capital Interest on capital is not allowed(iii) Interest on Drawings No interest on drawing of the partners is to be charged(iv) Interest on Partnerrsquos Loan A Partner is allowed interest 6 per annum on the amount of loan given to the firm by himher(v) Salary and Commission to Partner A partner is not entitled to anysalary or commission or any other remuneration for managing the business

History TOPIC-TOWARDS INDEPENDENCE AND PARTITION THE LAST PHASE (1935-1947)

SUB TOPIC-IMPORTANT POLITICAL DEVELOPMENTS ndash GROWTH OF SOCIAL IDEAS

Socialism is a political social and economic philosophyLike in other parts of the world the Russian revolution of 1917 served as a great inspiration for revolutionaries in India who at that time were engaged in the struggle for liberation from British ruleSocialist ideas led to the formation of communist party of IndiaJAWAHARLAL NEHRU Among the early Congress leaders Jawaharlal Nehru was very much impressed and influenced by the Socialist ideas He also learnt about the Economic activities of the Soviet Union after the Bolshevic Revolution 1917 He made full use of them in IndiaThe election of Jawaharlal Nehru and Subhas Chandra Bose showed the Left wing tendency within CongressJawaharlal Nehru demanded economic freedom along with political freedom of the people in order to end the exploitation of masses

Nehrus working committee included three socialists leaders The Lucknow session was a landmark in the evolution of socialist ideas of the congressSUBHAS CHANDRA BOSE ndash Subhas Chandra Bose had socialist leaning Both Jawaharlal Nehru and Subhas Chandra Bose were known as leftist Congress men Later on National Congress divided into Leftist and rightist campCONGRESS SOCIALIST Within the Congress some leaders formed the Congress Socialist partyPattavi Sitaramyya Sardar Patel Rajendra Prasad had hostile attitude towards the Congress Socialist partyJawaharlals attitude was hesitant

1 QUESTION ndash Mention name of two Congress leaders who had socialist leaning

1ANSWER ndash Subhas Chandra Bose and Jawaharlal Nehru2QUESTION- In which session of the congress Jawaharlal elaborated his Socialist ideas2 ANSWER ndash Lucknow and Faizpur Session in December 1935 and 19363QUESTION ndash Why Congress was sharply divided into leftist and rightist camp 3ANSWER ndash Subhas Chandra Bosersquos attempt to seek re election for congress presidentship in 1939sharply divided the National Congress into Leftist and Rightist camp4 QUESTION ndash Who was MN Roy 4 ANSWER ndash Manabendra Roy first formed the Communist Party of India outside the country at Tashkent in 19205QUESTION ndash Who formed the Congress Socialist Party within the Congress5 ANSWER ndash Jaya Prakash Narayan Achyut Patwardhan Acharya Narendra Dev Ram Mohan Lohia Aruna Asaf Ali6QUESTION ndash When was the Congress Socialist Party formed What was its object6 ANSWER ndash 1934The Congress Socialist Party sought to work out socialist programme through the Congress They joined hands with the Congress and wanted to carry

Subhas Chandra Bose being expelled from the congress after the Tripuri rift he formed Forward BlockThere were basic differences between the Congress Socialists and the communistsTRADE UNION ACTIVITIES Maximum working class people lived in Bombay and Calcutta The working and living conditions of those workers were very miserable In this situation Shasipada Banerjee NM Lokhande protested against the oppression of the working class peopleThe first Trade Union Madras Labour Union was formed in 1918 by BP WadiaIndustrial strikes took place in Kanpur Calcutta Madras Jamshedpur and Ahmedabad AITUC was formed in Bombay in 1927 The growth of Trade union among the workers was slow because of the fear of the dismissal of the jobIn the mean time the Moderates as well as Communists left AITUC and formed separate organization

on National struggle with the help of workers and peasant class of the society7 QUESTION ndash What was the name of the party founded by Subhas Chandra Bose7 ANSWER- Forward Block8QUESTION ndash Who was Shasipada Banerjee8 ANSWER ndash Shasipada Banerjee was a radical Brahmo He founded a working menrsquos club to protest against exploitation of the British rulers towards the working class of India9 QUESTION ndash What was the weekly published by NM Lokhande9ANSWER- Dinabandhu10 QUESTION ndash Who founded Bombay Mill-Hands Association and in which year10 ANSWER- NM Lokhande in189011 QUESTION- Who was BP WadiaANSWER- BPWadia was the founder of Madras Labour Union in191812 QUESTION- What was the name of the first labour union of India12 ANSWER- Madras Labour Union13 QUESTION Who founded the Majur Mahajan 13 ANSWER GANDHIJI14 QUESTION What was the full form of AITUC When it was formed14 ANSWER All India Trade Union Congressin 192715QUESTION Who formed the Red Trade Union Congress and in which year15ANSWER The Communists formed the Red Trade Union Congress16 QUESTION What do you mean by Socialism16 ANSWER Socialism describes any political and economic theory that says the community rather than individuals should own and manage property and natural resources

Subject Eng Literature (The Tempest ndash William Shakespeare) Topic Act III Scene 3 Lines 1 to 52 (Line 52 ndash Brother my lord the Duke Stand to and do as we) Date 13th April 2020 (4th Period)

[Students should read the original play and also the paraphrase given in the school prescribed textbook]Summary Questions amp Answers

o Alonso Sebastian Antonio Gonzalo Adrian Francisco and others wandered about the island in search of Ferdinand and gets tired and hungry of the toil and at the same time gives up all hope of finding him

o Antonio and Sebastian are happy that Alonso is out of hope and decide to make another attempt on his life that night when being so tired they will be sleeping soundly

o Suddenly a solemn and strange music is heard in the air and several strange shapes enter bringing in a banquet These strange shapes then dance round it with gestures of salutation and then inviting the King to eat they depart

o Seeing this strange scene all are inclined to believe the tales told by travelers that there truly are ldquounicornsrdquo and ldquothe phoenixrsquo thronerdquo

1 ALONSO What harmony is this My good friends hark (L18-27)

GONZALO Marvellous sweet music

[Enter several strange shapes bringing in a banquet

they dance about it with gentle actions of salutation

and inviting the King and his companions to eat they depart]ALONSO Give us kind keepers heavens What were theseSEBASTIAN A living drollery Now I will believe

That there are unicorns that in Arabia

There is one tree the phoenixrsquo throne one phoenix

At this hour reigning thereANTONIO Ill believe both

And what does else want credit come to me

And Ill be sworn rsquotis true Travellers neer did lie

Though fools at home condemn rsquoem

(a) How did Prospero present an amazing spectacle before Alonso and his companions

Using his magic powers Prospero ordered strange shapes to lay a banquet before Alonso and his companions The shapes brought several dishes with tasty eatables in them They placed the dishes on a table before Alonso and his companions Then the strange shapes began to dance gracefully around the banquet While dancing they made gestures inviting them to eat the food Then suddenly the shapes disappeared(b) Who were the guests at the strange banquet Describe the lsquoliving drolleryrsquo

Alonso Sebastian Antonio Gonzalo Adrian and Francisco were the guests at the strange banquet

The term ldquoliving drolleryrdquo refers to live entertainment show In this context when Alonso the King of Naples Sebastian his brother Antonio the treacherous brother of Prospero Gonzalo the kind and loyal councillor to the King Adrian and Francisco came to the island they were hungry and weary in their spirits They heard a solemn and strange music They were shocked to see several strange shapes bringing in a banquet and these shapes danced about it with gentle action of salutation inviting the King and his companions to eat After this Sebastian described this show as lsquoliving drolleryrsquo(c) What is lsquophoenixrsquo What are lsquoUnicornsrdquo

The term lsquophoenixrsquo refers to a mythical Arabian bird which lived alone and perched on a solitary tree After one hundred years it expired in flames and rose again from its own ashes

lsquoUnicornsrsquo refers to the mythological four-footed beasts having horns in the centre of their foreheads When the horns are ground into powder the powder was believed to be

an aphrodisiac(d) How does Sebastian explain the puppet show OR Why does the speaker now believe in unicorns and phoenix

Sebastian finds several strange shapes bringing in the banquet They invite the king and his party for dinner and soon depart He tells that if such a strange sight can be a reality there is nothing incredible in the world and from the present moment he will believe anything He says that it is a strange dumb show enacted not by puppets but by living beings It is stranger than a travellerrsquos tale Seeing such a thing

before his own eyes he will no longer disbelieve the story about unicorns and phoenix(e) How do the other characters present respond to this living drollery

At the sight of the lsquoliving drolleryrsquo like Sebastian Gonzalo and Antonio too acted strangely Antonio told that he too now believes in unicorns and phoenix and anything else that seems to be incredible He too now believes in travellersrsquo tales Gonzalo told that if he would report those happenings in Naples nobody will believe him He considers that those gentle shapes were gentler in manner in comparison to the living beings Alonso was at first sight suspicious and told them that those strange shapes conveyed their meaning in expressive gestures when they seemed to lack speech by their movements and sounds Francisco was amazed at their mysterious disappearance

2 ALONSO Not I

(Line 43-52)GONZALO Faith sir you need not fear When we

were boysWho would believe that there were mountaineers

Dewlapped like bulls whose throats had hanging at rsquoem

Wallets of flesh Or that there were such men

Whose heads stood in their breasts Which now we find

Each putter-out of five for one will bring us

Good warrant ofALONSO I will stand to and feed

Although my lastmdashno matter since I feel

The best is past Brother my lord the Duke

Stand to and do as we

(a) How does Alonso respond at the spectacle of the shapes which were sent to them at the instruction of Prospero

After seeing the strange sight of appearing and disappearing of the shapes sent by Prospero to arrange a banquet for them Alonso says that his surprise at having seen those creatures is infinite and he is fully justified in feeling so much surprise He thinks that their shapes their gestures and the sounds they made were indeed amazing Although they do not possess the gift of speech yet they were able to convey their

thoughts by means of their gestures only

(b) What does Prospero say about the views expressed by Alonso regarding the shapes What does Francisco think about the shapesAfter hearing Alonsorsquos views about the shapes Prospero says that this manrsquos praise of the spirits is rather hasty He means to say that Alonso has shown great haste in reaching the conclusion about the shapes Francisco is amazed to see that those shapes disappeared in a mysterious way(c) What does Sebastian ask Alonso to doSebastian tells Alonso that the shapes having disappeared should not matter to them because they have left the eatables behind He asks Alonso to enjoy eating as they are extremely hungry but the king does not accept his offer of enjoying the dishes(d) How does Gonzalo try to dispel Alonsorsquos fear of those strange shapes What kind of references does he give to AlonsoGonzalo says that those who have travelled abroad have reported seeing even stranger sights than these shapes that Alonso and his companions have beheld Hence there is no reason to feel afraid of these shapes Gonzalo further adds that in his younger days he had heard strange stories from travelers and Alonso might have heard similar stories For instance it was said that there existed a certain race of

human beings who had huge lumps of flesh hanging at their throats and who therefore resembled bulls Then Gonzalo tells about a race of human beings whose heads were located at their breasts Gonzalo says that such stories were not believed by most people in those days but now-a-days these stories have become common(e) Explain the following lsquoEach putter-out of five for onersquoEnglish travellers often insured their trips with London brokers Those that went on foreign travels those days used to deposit a certain amount with some firm or company in London before their departure If the travelers failed to return the money was forfeited by the company with which it had been deposited But this money was repaid five-fold if the travelers returned safe and sound In this way a traveler stood a great chance of recovering the entire cost of his

travels(f) Give the explanatory meanings of the following expressions in the context of the above extract (i) Dewlapped (ii) Wallets of flesh

(iii) Putter-out(i) Dewlapped having big lumps of flesh at the necks(ii) Wallets of flesh large masses of flesh looking like bags(iii) Putter-out to invest money before commencing the travel

  • General methods of preparation of hydrogen
  • Chapter Dimensional Analysis (Summary)
    • Properties of Charge
Page 2:   · Web viewSubject. Topic. Summary. Execution. Hindi. व्याकरण. शरीरके अंगो के नाम लिखिए. 1) आँख 2) नाक 3

Nancyrsquos dream of getting the moonNow they all are happy together

Class IISubject Topic Summary Execution

Eng literature

Ch 4 Bob and the BathTub

REFER PG 46 DO IN THE COPY DO ndash IS USED FOR PRESENT TENSE ( NOW) DID ndash IS USED FOR PAST TENSE ( FINISHED)

COMPOUND WORDS ARE WORDS THAT ARE MADE BY JOINING TWO SEPARATE WORDS TOGETHER

III Write each word in the right boxAnswers DOget give keep drink make throwDIDgot gave kept drank made threw IV What small words make these compound wordsAnswers

1 bath and tub2 birth and day3 bath and room4 every and body5 water and melon6 mother and land

Eng language

Ch 3 A AN THE REFER TO PG 20DO IN THE BOOKPractise all that is done in the book

EX E] READ THE VERSE AND FILL IN THE BLANKS WITH A AN OR THEANSWERSthe an the the

EVS Ch 5 Eating Healthy REFER TO PG 118DO IN THE BOOK AND LEARN THEM

EX A TICK THE CORRECT STATEMENT PUT A CROSS NEXT TO THE WRONG STATEMENTANSWERS-1a] gram2c] milk3b] different kinds of food4a] energy- giving food

Mathematics CHAPTER-3NUMBERS UPTO 1000

PLACE VALUEPLACE VALUE OF A DIGIT IN A NUMERAL DEPENDS UPON ITS PLACE OR THE NUMERAL

FACE VALUEFACE VALUE OF A DIGIT IN A NUMERAL IS EQUAL TO THE DIGIT ITSELF

FIND THE PLACE VALUE OF EACH DIGIT OF 652H T O6 5 2Where place value of 6 is hundredPlace value if 5 is tensPlace value of 2 is ones

FIND THE FACE VALUE OF EACH DIGIT IN 213In 213 face value of 2 is 2Face value of 1 is 1Face value of 3 is 3

Bengali বইndashবোংো োলিতয পলিরচয়

পোঠndashলিং ও ইদসরর গলপ অীীর পরসোততর

৩ টিক দোও -ক) পোোস2র মস3য লি4 একটো ( গোলিদঘী)) ইদর লিংসর ( োসকর মসর ) মস3য ঢসক প2গ) লিংসর ( োসকর কোসর ) ফসটো লেসক ইদর লেবলিরসয় এসোঘ) লিংএক( লিকোলিররলেসর ) োস আটকো প2সো

ঙ)( ইদর লিকোলির )লিgtং লেক বাোলিচসয় লিদ৪ লিবপরীত -ব2 = লে4োট োকো = ভোরী দব13 = ব কষদর = বৎ দ13োম = োম উপকোর = অপকোর

Hindi वयाकरण Chapter 2 वणमाा एव वण क भद

वण-बोत समय हमार म स धवनिनया निनकती ह इन धवनिनयो कोlsquoवणrsquoया lsquoअकषर कहत

ह वण क टकड नही निकए जा सकत ह वण भाषा की सबस छोटी इकाई ह वण क निनशचिltत करम को वण माा

कहत ह हिहदी म वणA क दो भद होत ह- सवर

और वयजन सवण वण की सखया=11 वयजन वण की सखया = 33

सभी वयजन वण म lsquoअrsquoकी धवनिन मिमी हनिबनाlsquoअrsquoधवनिन वा वण कषतरजञशरndashसयकत वयजनड ढndashनई धवनिनयाअ-अनसवार अ- निवसग

Read the lesson

Class IIISubject Topic Summary Execution

COMPUTER CHAPTER 2 OPERATING SYSTEM

DONE IN THE PREVIOUS CLASS Q3)WHAT IS THE DIFFERENCE BETWEEN MULTI-TASKING AND MULTI-THREADING OPERATING SYSTEMSAns) THE DIFFERENCE BETWEEN MULTITASKING AND MULTITHREADING OPERATING SYSTEMS IS---

MULTITASKING OPERATING SYSTEM

MULTITHREADING OPERATING SYSTEM

A MULTITASKING OPERATING SYSTEM ALLOWS TO RUN MORE THAN ONE SOFTWARE PROCESSES AT THE SAME TIME

MULTITHREADING OPERATING SYSTEM ALLOWS DIFFERENT PARTS OF A SINGLE PROGRAM TO RUN AT THE SAME TIME

Q4) WHAT IS A GUI OPERATING SYSTEMAns) A GUI OPERATING SYSTEM ALLOWS USERS TO INTERACT USING A COMPUTER MOUSE OR ANY OTHER POINTING DEVICE FOR EXAMPLE MICROSOFT WINDOWS 7

ENGLISH I CHAPTER 4 ARTICLES

refer to pg 15 do in the rough copy and practice the whole chapter

Continuation pg 15 ex DANSWERS10You are the tallest person in the class11The sun rises in the east12An honest person always speaks the truth13The peacock is the national bird of India14The Nile is the longest river in the world15We have a beautiful garden The garden is full of roses16He is one of the best authors of the world17My brother lives in an apartment in the centre of the city18He quickly ate the cookies that was meant for the guests

SST INDIA-THE LAND OF FESTIVALS

Bihu it is a harvest festival celebrated in Assam Magh Bihu is celebrated every year in the month of January People feast sing and have fun

1In which state Bihu festival is celebratedAns Assam

2In which month Magh Bihu is celebrated

Pongal Itrsquos a four day long festival celebrated in Tamil Nadu It is celebrated on January 14 every year People cook sweet rice called Pongal

Ans January

3What is PongalAns Pongal is a harvest festival which is celebrated in Tamil Nadu It is a four day long festival

4In which month Pongal is celebratedAns Pongal is celebrated on January 14 every year

SCIENCE SOME PROPERTIES OF WATER

We get water from the nature so water is a natural resource Water is the most commonly found liquid on earth All living things need water to live Life on earth is not possible without water We need water for drinking cooking bathing washing etc

Fill in the blanks [pg no ndash 81]1We get fresh water from rain 2Boiling kills the germs present in water 3 Water changes its state when it is heated and cooled 4Water does not have a definite shape

BENGALI বই ndash বোংো োলিতয পলিরচয়

পোঠ ndash ১ একতোই বঅীীর পরসোততর

১ঠিক উততসর টিক দোও-ক) বস2ো লেোকটির ( লিত চোর পাোচ ) লে4স ) তোরো ব ময় ( কা োসদ োফোয় ঝগ2ো কসর ) গ) বস2ো লেোকটি লে4সসদর বস ( কমস2ো বাো কলিG ) আসত ঘ) বস2ো তোর লে4সসদর ( োটক মযোলিক লিসমো ) লেদোসব বসলি4সো ২ ক) লেোকটির কয়টি লে4স লি4 উঃ পাোচটি লে4স লি4 ) বস2ো লেোকটো লে4সসদর পরসম লিক আসত ব উঃ কলিG আসত বসোগ) বোলিJ বাো3োর পর তোসদর লিক করসত ব উঃ এক-এক কসর বোলিJটি ভোঙোর লেচষটো করসত ব ঘ) লে4সরো লিমসলিমস োকস লিক সব উঃ লে4সরো লিমসলিমস োকস বোইসরর লেোক

HINDI वयाकरण chapter 1 भाषा

BOOK PAGE 10 राजय भाषामहाराषटर मराठी गजरात गजराती पजाब पजाबी बगा बागा लिनिप भाषा दवनागरी-- हिहदीमराठी गजराती रोमन -- अगरजी गरमी -- पजाबी फारसी -- उरद

सही या गत निनशान गाइए-1पतर मौखिक भाषा का एक रप ह( )X 2भाषा क लिखित रप को लिनिप कहत ह( )3 ससार म सभी ोग एक ही भाषा बोत ह(X) 4गजरात म पजाबी बोी जाती ह(X) भाषा क दो

MATHEMATICS CHAPTER 4 SUBTRACTION

Subtraction without DecomposingSubtraction of 3-digit number (without decomposing)

The result obtained after subtraction is called difference

Note While finding the difference we write the greater of the two given numbers on the top and then subtract

Exercise 16Subtract9 409 10 876 11 641 - 103 - 443 - 510 306 433 131

12 829 - 113 716

Write in columns and find the difference14 603 ndash 401 15 840 ndash 610

Solution

14 H T O 15 H T O 6 0 3 8 4 0 - 4 0 1 - 6 1 0 2 0 2 2 3 0

Fill in the place holders17 _ 3 7 18 6 _ 5

- 2 - 6 - 2 4 3 7 2 -- -- 4 ndash

Solution

179 3 7 18 6 8 5 - 2 1 6 - 2 4 3 7 2 1 4 4 2

Class IVSubject Topic Summary Execution

English language

Personal pronouns

The different forms of the three personal pronounsFirst personSingular I me my minePlural we us our oursSecond person Singular thou thee thy thinePlural ye you your yoursThird personSingular he him his her hers she it itsPlural they them theirs their

Pick out the personal pronouns Mention of what kind each (first second or third person)1 You should always try to keep a promise that you have madeYou second person personal pronoun

2 I listened patiently to what he had to say about themI first person personal pronounHe them third person personal pronoun

3 They packed their belongings and left I do not know where they have goneThey their third person personal pronounI first person personal pronoun

Social studies Major landforms on earth

Explained in previous lesson Answer the following questions3 Differentiate between hills and mountainsAns Mountains has a peak whereas hills have rounded topsMountains are steeper than hills whereas hills are lower and less steep4 Give two usefulness of mountainsAns Mountains are storehouse of waterMountains have a rich variety of flora and fauna

MATHEMATICS

Ch 5Subtraction

Estimating the differenceWe have learnt estimation in addition Applying the same rule We estimate the difference

Exercise 15 Example 2

A fruit-seller has 38210 bananas Out of these he sold 21799 bananas Estimate the unsold bananas

Solution Estimating to ten-thousandsTotal bananas = 40000Bananas sold out =- 20000Bananas unsold = 20000

Exercise1 Find the actual and estimated difference by rounding off to the nearest thousands(b) ActualEstimated

16190 - 2979

Solution (b) ActualEstimated 16190 16000 -2979 -30001321113000

2 Find the actual and estimated difference by rounding off to the nearest ten-thousands(b) ActualEstimated 73012 -28790

Solution (b) ActualEstimated 73012 70000 -28790-3000044222 40000

4 A bakery shop has an order to supply 6705 packets of biscuits There are only 4920 packets in the shop Estimate the number of packets needed to complete the supply

Solution Estimating to thousandsEstimated order = 7000Packets available= -5000Packets needed = 2000

there4 Estimated packets needed to complete the order were 2000

Hindi 2ndlang पतर पतर लिना भी एक का ह पतर लित समय धयान दना होगा निक पतर निकस लि रह ह कयोनिक पतर या तो वयवहारिरक होगा या वयावहारिरक होगापतर दो परकार क होत ह

क) औपचारिरक पतर जस परधानाचाय को नगर निनगम सपादक आदिद

ख) अनौपचारिरक पतर जस पापा को दोसत को बहन को अपन परिरजनो को

1 औपचारिरक पतर-अपन निवदयाय म अवकाश क लिए परधानाचाय को पतर लिखिए १०३कसीबीदम दमक टकोकाता-७०००६५सवा मपरधानाचायऋनिष अरहिवदो ममोरिरय एकडमीदिदनाक-१३४२०२०निवषय-अवकाश हत पतरमहोदया जीसनिवनय निनवदन ह निक म आपकी ककषा चौथी का निवदयाथj ह क रात स मझ तज बार ह डॉकटर न मझ आराम करन को कहा हअतः म निवदयाय आन म असमथ ह आपस निनवदन ह निक आप मझ दिदनाक १३स १७ तक की अवकाश दन की कपा कर हम आप क आभारी रहग

धनयवादआपका आजञाकारी लिशषयसीमा सिसहककषा-४

Bengali বইndashবোংো ভোষো পলিরচয়

পোঠndash৮ লিঙগ ৩ পংলিঙগ কোসক বসউঃ পরোলিবোচক লেP লিবসষয পসদর দবোরো পরষ োতীয় কোউসক লেবোঝোয় তোসক পংলিঙগ বস লেPম -বোবো 4োতর৪ সতরীলিঙগ কোসক বসউঃ পরোলিবোচক লেP লিবসষয পসদর তোরো সতরী োতীয় কোউসক লেবোঝোয় তোসক সতরীলিঙগ বস লেPম - মো লিলিকষকো৫ উভয়লিঙগ কোসক বস উঃ পরোলিবোচক লেPলিবসষয পসদর দবোরোপরষ ও সতরীউভয়োলিতসক লেবোঝোয় তোসক উভয়লিঙগ বস লেPম - লিশ মনতরী ৬ কলীবলিঙগ কোসক বস উঃ অপরোলিবোচক লেP লিবসষয পসদর দবোরো সতরী বো পরষ কোউসক ো বলিঝসয় লেকো2 পদো13 লেক লেবোঝোয়তোসক কলীবলিঙগ বস লেPম - বই

COMPUTER CHAPTER 3 EDITING IN MS WORD

MOVING THE TEXTMS WORD ALLOWS US TO MOVE A BLOCK OF TEXT OR GRAPHICS WITHIN A DOCUMENT

Q6) HOW TO WE USE THE COPY AND PASTE OPTIONAns) WE CAN CREATE A DUPLICATE COPY OF THE TEXTTHE STEPS TO COPY AND PASTE THE TEXT AS FOLLOWS----

SELECT THE PART OF THE TEXT THAT WE WANT TO COPY

SELECT THE COPY BUTTON IN THE CLIPBOARD GROUP UNDER THE HOME TAB OR PRESS CTRL + C KEYS

NOW PLACE THE CURSOR AT THE PLACE WHERE THE TEXT NEEDS TO BE COPIED

CLICK THE PASTE BUTTON IN THE CLIPBOARD GROUP UNDER THE HOME TAB OR PRESS CTRL + V KEYS

Science Adaptations in Animals

We find different kinds of animals in our surroundings We find birds flying in the sky monkeys jumping on trees fish in water camels in desert and so on Animals live in their natural homes which are called their habitats The habitat of a camel is a dry sandy desert and that of a tiger is a dense forest So different animals live in different habitats In the world a plant or an animal has to adapt or change itself to suits its surroundings This changes happens over hundreds and thousands of years A change that a living thing undergoes to become better suited to its surrounding is called adaptation

Fill in the blanks [pg no 40]1 Terrestrial animals live on

land 2 Polar bears are found in the

cold polar regions 3 Frogs have webbed feet that

help them to swim 4 Monkeys and koalas are

arboreal animals

Class VSubject Topic Summary Execution

COMPUTER130420

CHAPTER 2 APPLICATIONS OF COMPUTERS

PAGE NO-15C FILL IN THE BLANKS

INPUTS ANIMATION BANKS SOFTWARE SUPER COMPUTERS

D WRITE TRUE OR FALSE TRUE TRUE TRUE TRUE FALSE

Science Chapter 2 - The Skeletal System

Joints A joint is a place where two or more bones are joined with each other

There are two types of joints 1) Fixed joints 2) Movable joints

The movable joints are mainly four types ndash 1) Hinge joint 2) Pivot joint 3) Gliding joint 4) Ball and socket joint

DAnswer these questions

1)What are the functions of the skeleton

Ans ndash The functions of skeleton are -

i The skeleton gives our body shape ii The skeleton provide our body strength iii The skeleton protect our inner organs from outside injuries iv The skeleton gives our body support

2)List the type of movable joint in our body giving one example of each

Ans - The movable joints are mainly four types ndash a Hinge joint

Example of hinge joints are elbows nice fingers and toes

b Pivot joint Example - the joint between the skull and the backbone is the example of pivot joint

c Gliding joint Example of gliding joints

are wrist and ankle joints d Ball and socket joint

Example of ball and socket joints are shoulder joint and hip joint

English language

Transitive and intransitive verb

Pick out the verbs from the following sentences and say whether they are transitive or intransitive (page 21 ex B)1 gave ndash verb Transitive verb2 Has planted- verb Intransitive verb3 Were- verb Intransitive verb4 Asked- verb Transitive verb5 Told- verb Transitive verb

Social studies Conquering distances

Airways The only airline owned by the government is Air India which handles both domestic and international flightsAdvantages of airways Air transport is the fastestIt can access remote areasIt is the best means of transport in case of emergencies

1 Which is the only airline owned by the governmentAns Air India

2 Give two advantages of airwaysAns The two advantages of airways areAir transport is the fastest transportIt can access remote areas

DisadvantageIt is the most expensive of all other means of transport

3 What is the disadvantage of airwaysAns The only disadvantage of airways is that it is the most expensive of all other means of transport

MATHEMATICS

Ch 3Addition and Subtraction

Properties of Addition1 The sum of two numbers does not change when we change their order This property known as Commutative Property of addition

2 The sum of three numbers does not change when we change their grouping This property is called Associative Property of addition

3 The sum of the numbers and zero is the number itself This property is called Identity Property of addition and the integer 0 is called identity

Exercise 11Fill in the blanks1 2730815 + 8319293 = ____ + 27308152 18219 + 1850308 = 1850308 + ____3 (27815 + 85919) + 95985 = (85919 + ____) + 278154 13227 + (25983 + 73607) = (____ + 25983) + 736075 91389 + 0 = ____ + 91389

Solution 1 83192932 182193 959854 132275 0

6 Which of the following are true statements(a) Any number added to zero is zero

(b) The sum of two numbers does not change when we change their order

(c) 1 is the identity element of addition

(d) Given any three numbers their sum does not change when we change their grouping

Solution (a) False(b) True(c) False(d) True

Class VISubject Topic Summary Execution

HISTORY AND CIVICS

CHAPTER 3

MAHAVIRA AND BUDDHA ndash GREAT PREACHERS BUDDHA

Decline of Buddhism1 Revival of the Brahmanical Hinduism ndash Brahmin Scholars like Shankaracharya and Kumarila Bhatta led the revival of Hinduism and established the supremacy of Vedic religion2 Loss of Royal Patronage ndash Gupta period marked the decline of Buddhism as Gupta rulers were followers of Hinduism3 Split in Buddhism ndash division into Hinayana and Mahayana sects and rise of Mahayana sects blurred the line between Hinduism and Buddhism4 Corruption in Buddhist Sangha ndash due to generation of large revenue from large estates Buddhist monks and nuns started living luxurious

Answer the Following 1 During which dynasty Buddhism was split During the reign of Kanishka

2 Name the two sects of Buddhism Mahayana and Hinayana

3 Name two Vedic scholars who led the revival of Brahmanical Hinduism Shankaracharya and Kumarila Bhatta

life in rich monasteries Hence corruption crept in5 Adoption of Sanskrit ndash when Buddhist scriptures began to be written in Sanskrit in place of peoples language like Pali or Prakrit people started drifting away from Buddhism6 The Turkish Invasion ndash As Muslim conquerors invaded India immensely wealthy Buddhist Monasteries and temples were looted and destroyed and Buddhists were persecuted and killed

4 During which period decline of Buddhism began The Gupta Period

ENGLISH 2 The great train journey- Ruskin Bond

The great journey by Ruskin Bond is a story about Suraj who loved trains and wanted to go to places One day while wandering along the railway tracks he enters into a carriage compartment The train suddenly starts moving with him in the compartment and after a journey returns back to the same place from where it had begun The story is about his experience during that journey

4 Answer the following questionsf Who else is in the carriageA ragged hippy with a dirty beard face was in the carriageg Where does Suraj say that he would like to go toSuraj said that he would like to go to England and China and Africa and Greenland He wanted to go all over the worldh What warning does the man give to SurajThe man said Suraj to keep out of sight so that he doesnrsquot get caught by the ticket collectorsiWhen Suraj thinks about his parents for the first time what does he imagines that they will thinkSuraj thought that if he failed to come home that night his parents would think that he had run away or been kidnapped or been involved in an accidentJ What presents does Suraj imagine that he will bring back for his friendSuraj imagines that he would bring an African lion or a transistor- radio for his friend

CHEMISTRY

Chapter 2 ndashElement and Compound

ATOMAn atom can be defined as the smallest constituent particle of an element which showcases independent existence Example Ne OMOLECULEA molecule can be defined as the combinations of two or more atoms which are held together by chemical bonds A molecule is the smallest portion of a substance which showcases all the properties of the substance On breaking down a molecule further we see properties of the constituent elements Example HCl NaCl O2

Answer the following Q3) What is a moleculeAns - A molecule can be defined as the combinations of two or more atoms which are held together by chemical bonds A molecule is the smallest portion of a substance which showcases all the properties of the substance On breaking down a molecule further we see properties of the constituent elements Example HCl NaCl O2

Q4) Which can exist independently ndash atom or moleculesAns ndash Molecules can exist independently

PHYSICS Physical quantities

Guidelines for writing SI units correctly1 The units named after scientists are not written with a capital initial letter For example newton henry watt2 The symbols of the units named after scientist should be written by a capital letter For example N for newton H for henry W for watt3 Small letters are used as symbols for units not derived from a proper name For example m for metre kg for kilogram4 No full stop or other punctuation marks should be used within or at the end of symbols For example 50 m and not as 50 m5 The symbols of the units do not take plural form For example 10 kg not as 10 kgs6 When temperature is expressed in kelvin the degree sign is omitted For example 273 K not as 273o K (If expressed in Celsius scale degree sign is to be included For example 100o C and not 100 C)7 Use of solidus is recommended only for indicating a division of one letter unit symbol by another unit

Fill in the blanks

1) Length and mass are examples of fundamental physical quantities

2) The measurement of a physical quantity consists of two part magnitude and unit

3) A foot consist of 32 inches 4) The unit of temperature in the SI system is

Kelvin

Write true or false Correct the false statements

1) In ancient times cubit was used to measure the mass of an object FalseCorrect statement ndash In ancient times cubit was used to measure the length of an object

2) There are 7 fundamental physical quantities True

symbol Not more than one solidus is used For example m s-1 or m s J K mol or J K-1 mol-1 but not J K mol8 Some space is always to be left between the number and the symbol of the unit and also between the symbols for compound units such as force momentum etc For example it is not correct to write 23m The correct representation is 23 m kg m s-2 and not as kgms-29 Only accepted symbols should be used For example ampere is represented as A and not as amp or am second is represented as s and not as sec10 Numerical value of any physical quantity should be expressed in scientific notationFor an example density of mercury is 136 x 104 kg m-3 and not as 13600 kg m-3

3) Second is the unit of time in both the CGS and MKS systems True

4) The symbol used for a unit is always written in capital letters False Correct statement -The symbol used for a unit is normally written in small letters

Hindi 2nd language

वाकय निवचार भागवत निवचारो को परकट करन वा साथक एव वयवसथिtत शबद समह को वाकय कहत ह वाकय दो परकार क होत ह ndash

1 उददशय- वाकय म जिजसक बार म कछ बताया जाता ह उस उददशय कहत ह जस राधा एक नतकी ह2 निवधय- वाकय म उददशय क बार म बताया जाता ह उस निवधयक कहत ह जस- राधा एक नतकी ह रचना क आधार पर वाकय क तीन भद होत ह ndash१सर वाकय- राम बाजार गया २ सयकत वाकय- राम बाजार गया और वहा जाकर दोसत स मिमा३ मिमशर वाकय- यह वही tान ह जहा उनका बचपन बीता

helliphellipContinue to nextBengali 2nd language

লিZ সবরপ ও সবরলিZ

সবরলিZর লিয়ম - ১ অ-কোর লিকংবো আ-কোসরর পসর অ-কোর লিকংবো আ - কোর োকস উভয় লিমস আ ndashকোর য় এবং ওই আ ndash কোর পব13বস13 Pকত য়

২ ই - কোর লিকংবো ঈ - কোসরর পসর ই - কোর লিকংবো ঈ - কোর োকস উভয় লিমস ঈ - কোর য় এবং ওই ঈ - কোর পব13বস13 Pকত য়

৩ উ - কোর লিকংবো ঊ - কোসরর পসর উ - কোর লিকংবো ঊ - কোর োকস উভয় লিমস ঊ - কোর য় এবং ওই ঊ - কোর পব13বস13 Pকত য়

৪ অ - কোর লিকংবো আ ndash কোসরর পসর ই - কোর লিকংবো ঈ - কোর োকস উভয় লিমস এ - কোর য় এবং ওই এ - কোর পব13বস13 Pকত য়

১ অ + অ = আ ( gtো ) লিম + অচ = লিমোচ সব + অ3ী = সবো3ী অ + আ = আ ( gtো )পদম + আ = পদমো শভ + আলি = শভোলি আ + আ = আ ( gtো )4োয়ো + আবত = 4োয়োবত মো + আতমো = মোতমো আ + অ = আ ( gtো )লিবদযো + অংকোর = লিবদযোংকোর Pো + অ13 = Pো13 ২ ই + ই = ঈ ( gtী )অলিত + ইব = অতীব লিগলির + ইনদর = লিগরীনদর ই + ঈ = ঈ ( gtী )পলির + ইকষো = পরীকষো অলি3 + ঈশবর = অ3ীশবর ঈ + ঈ = ঈ ( gtী )মী + ঈশবর = মীশবর 3ী + ঈ = 3ী ঈ + ই = ঈ ( gtী )রী + ইনদর = রীনদর মী + ইনদর = মীনদর ৩ উ + উ = ঊ ( gt )মর + উদযো = মরদযো কট + উলিকত = কটলিকত উ + ঊ = ঊ ( gt )ঘ + ঊলিম13 = ঘলিম13 লিZ + ঊলিম13 = লিZলিম13 ঊ + ঊ = ঊ ( gt )রP + ঊলিম13 = রPলিম13

৪ অ + ই = এ ( লেgt )র + ইনদর = সরনদর লেPোগ + ইনদর = লেPোসগনদর অ + ঈ = এ ( লেgt )গ + ঈ = গস

র + ঈ = সর আ + ই = এ ( লেgt )Pো + ইষট = Pসষট 3ো + ইনদ = স3নদ আ + ঈ = এ ( লেgt )রমো + ঈ = রসম দবোরকো + ঈশবর = দবোরসকশবর

COMPUTER THE WORLD OF WINDOWS 10

DONE IN THE PREVIOUS CLASS PAGE NO-83A TICK THE CORRECT OPTION BACKGROUND DISPLAY AREA RESTORE THREE

MATHEMATICS Topic ndash NumbersChapter ndash Natural numbers and whole numbers

Study item Properties of whole numbers for subtraction1) Closure property When we do subtraction of two whole numbers we can not get a whole number in all time Example 8 ndash 3 = 5 a whole number 0 ndash 6 = -6 is not a whole numberTherefore the subtraction of two whole numbers is not satisfying closure property2) Commutative property If x and y are two whole numbers then x ndash y is not equal to y ndash xExample If x=16 and y = 7 then x ndash y = 16 ndash 7 = 9Again y ndash x = 7 ndash 16 = - 9 Therefore x ndash y not equal to y ndash x Therefore the subtraction of two whole numbers is not satisfy commutative3) Associative property If x y and z are three whole numbersThen x ndash ( y ndash z ) not equal to ( x ndash y ) ndash z Example If x = 20 y = 10 and z = 6Therefore x ndash (y ndash z ) = 20 ndash(10 ndash 6 ) = 20 ndash 4 = 16(X ndash y ) ndash z = (20 ndash 10) ndash 4 = 10 -4 =6Therefore x ndash(y ndash z) not equal to ( x ndash y) ndash zTherefore subtraction of whole numbers is not satisfying associativity4) Distributive property If x y and z are three whole numbersThen x (y ndash z ) = xy ndash xzAnd (y ndash z)x = yx ndash zxExample If x = 10 y = 6 and z = 4x(y ndash z ) = 10(6 ndash 4 ) = 10times6 ndash 10times4 = 60 ndash 40 = 20( 6 ndash 4 )times 10 = 6times10 ndash 4times10 = 60 ndash 40 = 20Therefore the subtraction of whole numbers is satisfying distributive property5) Existence of identity For any whole number x X ndash 0 = x but 0 ndash x = - x not equal to xThus for subtraction no identity number existsException 0 ndash 0 = 0 so 0 is its own identity for subtraction

Class VIISubject Topic Summary Execution

Hindi 2ndlang वचन जो सजञा शबद निकसी वसत या पराणी क एक या अनक होन का बोध कराया उनह वचन कहत ह जस डका- डकयह दो परकार की होती ह-

क) एकवचन-शबद क जिजस रप स उसक एक होन का बोध हो उस एक वचन कहत ह जस निकताब गमा आदिद

) बहवचन-शबद क जिजस रप स उसक आन ोन का पता च उस बहवचन कहत ह जस डक निकताब निततलियाआदिद

निनमनलिखित शबदो को एकवचन स बहवचन म बदोम- हमजानित- जानितयानारी- नारिरयामिमतर ndashमिमतरोपसतक -पसतकसडक-सडकबोत-बोतनाहर-नहररपए-रपया

Bengali বইndashবোংো োলিতয পলিরচয়

পোঠndash১৪ গলপ - অপর কলপো পর

লেক - লিবভলিতভষ বসনদযোপো3যোয়লেকndash রবীনদর পরবতf বোংো কোোলিতয 3োরোর উসgসPোগয োম পরকলিতসপরমী লিবভলিতভষ বসনদযোপো3যোয় তোর লেীসত লেPম বোসর বোসর লিফসর এসস4 গরোম বোংোর পরকলিতর কো লেতমলি এসস4 গরোমী মো লিচতরগলপndash অপর কলপো গলপোংটি লিবভলিতভষ বসনদযোপো3যোসয়র লিবযোত উপযো পসর পাোচোী লেসক গীত অপ অ13োৎ পসর পাোচোী তো অপর কলপোর লেকনদরীয় চলিরতর এই অংস আমরো পোই বোক অপসক বোক অপ কলপো লিবোী লে দসরর অ গো4 লেদস মোসয়র মস লেোো রপকোর রোসয পোলি2 লেদয় দপরসবো মোসয়র মসর কসর কোীদোী মোভোরত এর করসকষতর Pসjর ব13ো শস তোর মোবীর কস13র পরলিত ব2 মমতো য় আবোরপালিসত বলি13ত Pসjর অমোপত অং লে লিসই মোপত কসর বোলি2র লিপ4স বাো বোগোস লিকংবো উঠোসর লিশমসর কলপো লিবো এোস পরকো লেপসয়স4

১ অপর কলপো গসলপর লেক লেক তোর মপসক13 লেসো২ অপর কলপো গলপটি লেকোো লেসক গীত গলপটির ম ভোব লেসো

GEOGRAPHY CHAPTER 7EUROPE

CHAPTER COMPLETE EXERCISEFill in the blanks1 Europe is a continent that comprises the western part of Eurasia2 Eurasia and Africa are connected into one large land mass known as Afroeurasia3 The Strait of Gibraltar separates Europe and Africa4 Europe is surrounded by the Arctic Ocean to the north5 The British Isles includes the island countries of Great Britain and Ireland

Name the following 1 Connects Africa to Eurasia - Isthmus of Suez2 Largest country in the world in terms of area ndash Russia3 A term used collectively for the five countries in northern Europe ndash Nordic Countries4 The capital of Montenegro - Podgorica5 the largest fjord in Norway ndash Sognefjord

Match the following Column 1 Column 2a Albania iii Tiranab Belgium i Brusselsc Denmark v Copenhagend Finland ii Helsinkie Hungary iv Budapest

CHEMISTRY Chapter 2 ndashElement and Compound

Atom - An atom is the basic unit of an element or the smallest particle of an element non capable of independent existence Atom is built up of three sub atomic particles electron proton and neutron

Nucleus-It is the centre of an atom In the centre of the atom contains proton (positively charged particles ) and neutrons ( particles carrying no charge )

Orbits- It surround the nucleus in which revolve electrons (negatively charged particles)

Answer the following

1) What are MetalloidsAns - Certain elements using properties of both metal and non-metals are called metalloids Example Silicon arsenic and antimony

2) What are Noble gasesAns - Certain elements are present in the air and are chemically inert or unreactive Such elements are called rare gases or noble gases Example helium neon argon and Krypton

English 2 Sentences based on meanings

Kinds of sentences

Assertive or declarative to convey information or simply make a statement

Interrogative to ask different types of questions

Imperative to command or instruct someone or make a request

Exclamatory to express strong feelings and emotions

Exercise B1 Stop it ( Exclamatory)2 May you always be happy

together ( Exclamatory)3 He does not like sports

( Assertive)4 Please pass me the salt

( Imperative)5 How dare she talk to me like

that ( Exclamatory)6 May success bless your effort

( Exclamatory)7 Canrsquot you wait for sometime

(Interrogative)8 Did anybody tell you about it

( Interrogative)9 I saw her waiting for the bus

( Assertive)10 Could you please take a

message for me ( Interrogative)

Homework Ex ABiology Chp -2

Classification of Plants

Today we discuss about usefulness of bacteria We also discuss what the harmful effects of bacteria are

89 How bacteria are useful for usbull Bacteria is helpful in many ways forhuman being i) Production of medicine - antibiotics vaccine etcii) Formation of curd by lactobacillusiii)Nitrogen fixation in Leguminousplant by Rhizobiumiv) Increase soil fertility by absorbingatmospheric nitrogen and convert it into nitrates and nitritesv) Cleaning the environment by converting the complex substances into simple substancesvi) Tanning of leathervii) Retting of Fibersviii) Formation of compost by acting onanimal dung and agricultual cases1x) Biogas production by decomposingplant and animal wastex)Help In Nutrition by producing vitamiacutemBand kx1) Some bacteria are used to give specialflavour to tea coffee and coccaQ10- Name some diseases and there causativebacteriabull Diseasescausative bacteria1 CholeraVibrio cholerae2 Tuberculosis - Mycobacterium tuberculosis3 Diptheria -Corynebacteriumdiphtheriae4 Pneumonia - Streptococcus pneumoniae

Math Number system

Chapter Fraction

Study item Using lsquoofrsquoThe word lsquoofrsquo between any two fractions is to be used as multiplicationExample 57 of 56 = 57 times 56 = 5times8 = 40Study item Using BODMASThe word lsquoBODMASrsquo is the abbreviation formed by taking the initial letters of six operations(i)Bracket (ii) of (iii) Division (iv) Multiplication (v) Addition (vi) SubtractionAccording to BODMAS rule First of all the terms inside Bracket must be simplified then lsquoofrsquo lsquoDivisionrsquo lsquoMultiplicationrsquo lsquoAdditionrsquo lsquosubtractionrsquo

Study item Removal of Brackets

There are four Brackets of algebra in Mathematics In a complex expression four types of brackets are used Order of removing the brackets is first ----- then ( ) then finally [ ]

Class VIIISubject Topic Summary Execution

Chemistry Hydrogen General methods of preparation of hydrogen

By the action of dilute acids on metals

Calcium Reacts readily to form chloride salt and hydrogen

Ca + 2HCl rarr CaCl₂ + H₂uarr

Magnesium

Aluminium

Zinc

React readily to form salt and hydrogen

Mg + 2HCl rarr MgCl₂ + H₂uarr2Al + 6HCl rarr 2AlCl₃ + 3H₂uarrZn + 2HCl rarr ZnCl₂ + H₂uarr

Question 4 ) Give reasons for the following

(a) Hydrogen be used as a fuel

Solution

Hydrogen is used as a fuel because it has a high heat of combustion Some significant fuels are coal gas water gas and liquid hydrogen

(b) Though hydrogen is lighter than air it cannot be collected by downward displacement of air

Solution

Hydrogen is lighter than air so it is possible to collect the gas by downward displacement of air But it is not safe to do so since a mixture of hydrogen and air can lead to an explosion

(c) A pop sound produced when hydrogen is burnt

Solution

Impure hydrogen gas burns in air with a pop sound This is because of the presence of impurities in it

(d) Helium replaced hydrogen in weather observation balloons

Solution

It forms a mixture with air that can explode when there is a small leakage of hydrogen in a balloon So helium has replaced hydrogen

(e) Nitric acid not used for the preparation of hydrogen gas

Solution

(e) By the action of nitric acid on metals hydrogen cannot be produced because it also releases nitrous oxide and nitric oxide and oxides the hydrogen to form water

Biology Chp-2 Reproduction in plants

Today we discuss different methods of artificial propagation like cutting-rose sugercane Layering ndashguava lemon china rose etc Grafting- mango apple etcMicropropagation ndashorchid asparagus etcWe also discuss about advantages and disadvantages of vegetative propagation

Q7 Define the following terms i) Explant In tissue culture techniquea tiny piece of bud shoot or any other partof plant from where new tissue develop ii) Callus The cells of the tissue divide andgrow into a mass of undifferentiated cells from explant iii) Plantlet After few days callus differentiate into a small plant with roots and shootQ8 what are the advantages and limitations of tissue culture or micropropagation

Advantages i ) It produacuteces superior quality plantsii)It can be applied to interspecifie hybridsiii) It is useful to grow seedless plants bull Limitations i) It cannot be used for all plantsii)It is not easy to handleQ9 Write advantages of vegetative propagationi) It is a quick and easy method ofproducing new plantsii) This method need less time to matureiii) The new plants are exact copies of the parentiv) it is extremly useful for growing seedlessplants like banana grapes etc Q10 Write some disadvantages of vegetativepropagationi) Dišeases present in the parent plant gettransferred to all in new plantsii) Overcrowding of new plants causes competition for sunlight water and nutrients which affects growth of plantsplant

Physics Chapter 2 Physical Quatites and Measurements

Here We Will Do Some QuestionsRelated To Chapter 2

Select the correct alternative A block of wood of density 08gcm-3 has a volume of 60cm3 The mass of the block is

1 608 g

2 75 g

3 48 g

4 0013 g

Solution 348 g

The density of aluminium is 27g and that of brass The correct statement is

1 Equal masses of aluminium and brass have equal volumes

2 The mass of a certain volume of brass is more than the mass of an equal volume of aluminium

3 The volume of a certain mass of brass is more than the volume of an equal mass of aluminium

4 Equal volumes of aluminium and brass have equal masses

Solution 2 The mass of a certain volume of brass is more than the mass of an equal volume of aluminium

MATHEMATICS Ch 6Sets

Exercise 6(C)1 Find all the subset of each the following sets(i) A = 57 (iii) C = x xisin W x le 2(iv) p p is a letter in the word lsquopoorrsquo

Solution (i) All the subsets of A are ϕ 5 7 57

(iii) All the subsets of C are ϕ 0 1 2 01 02 12 012

(iv) All the subsets are ϕ p o r po or por

4 Given the universal set = -7-3-105689 find (i) A = x xlt2 (ii) B = x -4ltxlt6 Solution

(i) A = -7-3-10(ii) B = -3-105

5 Given the universal set = x xisin N and xlt20 find

(i) A = x x = 3p pisin N (iii) C = x x is divisible by 4 Solution

(i) 369121518 (iii) 481216

6 Find the proper subset of x x2-9x-10 = 0 Solution

ϕ 10 -1

Working x2-9x-10 = 0 rArr x2-(10-1)x-10 = 0

rArr x2-10x+x-10 = 0 rArrx(x-10)+1(x-10) = 0

rArr (x+1) (x-10) = 0

11 Let M = letters of the word REAL and N = letters of the word LARE Write sets M and N in roster form and then state whether (i) M sube N is true (ii) N sube M is true (iii) M = N is true

Solution M = real and N = lareSo (i) Yes (ii) Yes (iii) Yes

English 2 Twelfth Night ndash Shakespeare

A noble man named Orsino in the kingdom of Illyria is deeply in love with a lady called lady Olivia She is in mourning for her dead brother so she will not even think about marriage At this time a sea storm causes a terrible shipwreck and a young lady called Viola is swept onto the shore She thinks that her twin brother Sebastian is drowned A sea captain tells her about Orsino and his love for Olivia Viola wishes to work in Oliviarsquos home but feels she will not be employed So she dresses as a man calls herself Cesario and gets work at the house of OrsinoViola (now Ceasario) is much liked by Orsino and becomes his page She falls in love with Orsino Orsino sends Ceasario to deliver messages to Olivia Olivia herself falls for the beautiful young Ceasario believing Viola to be a man

2 Answer the following questionsa Why does Orsino ask the musicians to play onOrsino asks the musicians to play on because music feeds his desire He calls upon the musicians to play music so that his hunger for love could be replenished with an excess of musicb What does Valentine tell about OliviaWe learn from Valentine that Olivia is in mourning for her brother she wears a veil and has vowed that no one will see her face for another seven yearsand she refuses to marry anyone until thenc From the exchange between Orsino and Valentine what do you think their relationship isValentine is one of orsinod attendants He was sent to Olivia as a messenger of love but was not allowed to speak to here Who is Olivia mourning for and whyOlivia is mourning for her dead brother

Homework Q fHistory and Civics

Growth of Nationalism

Important dates to remember1769-Napoleon born on 15thAugust1789-Fall of Bastille on 14th July and the beginning of the French revolution declaration of the rights of Man on 26thAugust1793-King Louis XVI executed on January 211764-The Sugar Act passed1765-The Stamp act passed1774-The first congress of Philadelphia1776-The declaration of American Independence of on 4th July1777-Defeat of the British at Saratoga1781-Surrender of lord Cornwallis at Yorktown1783-The treaty of Versailles1804-Napoleon becomes the emperor1813-Battle of Leipzig or Battle of nations in which Napoleon was defeated by the Allies1815-Battle of Waterloo June 18 in which Napoleon was defeated and captured1821-Death of Napoleon in StHelena1860-Abraham Lincoln elected President of the USA1861-The civil war began 1864-Abraham Lincoln elected President of the USA for the second time1865-Slavery abolished in the US

Name the following- The queen of Louis XVI

Marie Antoinette The three philosophers of France

VoltaireMontesquieuJean Jacques Rousseau

The British general whose surrender brought the war in America to an endLord Cornwallis

The first president of the USAGeorge Washington (1732-1799)

The first southern state to secede from the unionSouth Carolina

The author of the book lsquoUncle Toms CabinHarriet Beecher Stowe

Homework-Learn

Class IXSubject Topic Summary Execution

Economics

Types of economies Today I am going to share you the concept of economic growth and economic development Few questions will be given from the previous study material dated 942020

Meaning of economic growthAnswer) The term economic growth generally means anincrease in national income or per capita output or income over time It indicates towards quantitative growth of a country

Meaning of economic developmentAnswer) Economic development is defined

as a process whereby the real per capita income of a country increases over time along with fall in poverty ratio unemployment and income inequality etc

Distinguish between economic growth and economic development

Basis Economic growth

Economic development

Scope It has narrow scope as it refer only to rise in per capita income

It has wide concept since it includes qualitative changes as well

Concerned matter

It is concerned with the rise in income

It is concerned with not only rise in income but also reduction of poverty income inequality and unemployment

Focus Economic growth does not focus on economic development

Economic development focus on economic growth plus qualitative changes

Distinguish between capitalist economy and socialist economy

Ownership

Motive

Tool

Means of production are owned and managed by private people

Self interest and profit earning is the main motive

Price mechanism is a main tool to solve the economic problems

Means of production are owned and managed by the government

Social welfare is the main motive

Economic planning by the government is the main tool to solve the economic problem

Competition

Distribution of income

There exist large competition among buyers and sellers

There is existence of large inequalities of income

There is no such competition

There exist less inequalities of income

Math Topic ndash AlgebraChapter -Factorisation

Study item Factorising by taking out common factorSome solved sums from exercise 41

1) (i) 8xy3 + 12x2y2

= HCF of 8xy3 and 12x2y2 is 4xy2

= 4xy2(2y + 3x )

4) (ii) 28p2q2r ndash 42pq2r2

= HCF of 28p2q2r and 42pq2r2 is 14pq2r = 14pq2r (2p - 3r )5) (ii) 14mn + 22m - 62p=HCF of 14mn 22m and 62p is 2= 2(7mn + 11m - 31p)7) (ii) 3a(x2 + y2) + 6b (x2 + y2) = HCF of 3a(x2 + y2) and 6b(x2 + y2 ) is (x2 + y2)= ( x2+ y2 )(3a + 6b )9) (ii) x(x2 + y2 ndash z2 ) + y(-x2ndashy2 + z2 ) ndash z(x2+ y2 ndash z2 )= x(x2 + y2 -z2) ndash y-(x2 + y2 -z2) -z(x2 + y2 ndash z2)=x(x2 + y2-z2) -y( x2 + y2-z2) ndash z (x2 + y2 -z2)= (x2+ y2 ndash z2)(x ndash y ndash z )

Commercial Studies

Introduction to Accounting and Book-keeping

Today I am going to share you the meaning of Accounting and Book-keeping and its related terms bullAccounting bullBook Keeping bullAccountsbullTypes Of Accounts bullAccounting Cycle

bull Meaning of accounting

Ans )Accounting is the art and science of recording classifying and summarising monetary transactions

bull Meaning of Book-keeping

Ans) Bookkeeping is the art of recording business transactions with the view of having a permanent record of them and showing their effect on wealth

bull Meaning of account

Ans) The term account means a record of business transactions concern a particular person of firm asset or income or expense It is a summarised record of all transactions which take place in an accounting year

bull Types of accountsPersonal accounts ndash Personal accounts relating

to person and Organisation are known

as personal accounts Example Ramrsquos Account ABC amp Co Account etc

Real account - The accounts related to tangible and intangible assets are called real accountsExample Cash Account Furniture Account etc

Nominal account- Accounts related to expenses losses incomes and gains are known as nominal accountsExample Wages Account Salary Account Discount Account etc

bull Accounting cycle Accounting cycle refers to a complete sequence of accounting activities It begins with recording of transactions and ends with the preparation of a balance sheet

English 1 Transformation of sentences

Sentences A sentence is a group of words which makes complete sense

a Assertive sentencesb Imperative sentencesc Interrogative

sentencesd Exclamatory sentences

Sentences can be changed from one grammatical form to another without changing the meaning of the sentence This is known as transformation of sentences

Exercise 6Rewrite the following sentences according to the instructions given below without changing their meanings

1 As soon as he saw the beer he jumped into the river ( Begin No sooner)

2 None but brave deserve the fair (Begin the bravehellip)

3 This box is too heavy for me to lift ( Use so hellip That instead of too)

4 No one other than a king can live like James Luxurious ( Begin only James)

5 Oh for the wings of a dove (Begin I wishhellip)

BENGALI(2ND LANGUAGE)

ldquo বঙগভমির পরমি ldquo াইকেল ধসদন দতত

পব13পোসঠ আসোলিচত ৩ পরবোস দৈদসবর বস ীবতোরো Pলিদ স এ লেদ -আকো সত-োলি লেদ তোস - ক) বকতো লেক কোর লেো লেকো কলিবতোর অং ) কোর পরলিত বকতোর এই উলিকত গ) এ লেদ আকো সত বসত কী বলিঝসয়স4 ীবতোরো বসত কী লেবোঝ ঘ ) আসোচয অংসর তোৎপP13 কী

উ -ক ) বকতো স কলিব মোইসক ম3দ দতত

Types of AccountPersonal AccountReal AccountNominal AccountBalance Sheet (opening)

কলিব মোইসক ম3দ দসততর রলিচত বঙগভলিমর পরলিত কলিবতোর অং ) কলিব বঙগী অ13োৎ লেদমোতোর পরলিত কলিবর এই উলিকত গ ) এ লেদ আকো বসত কলিবর মোব লেদী রপ আকো লেক লেবোঝোসো সয়স4 আকো লেসক লেPম তোরো স পসর লেতমলি ীব লেদ রপ আকো লেসক পরো রপ তোরো স পরসত পোসর এই মভোবোর কোই কলিব বসস4 ঘ ) পরবো Pোতরোয় Pলিদ কলিবর লেদ আকো লেসক ীব তোরো রপ পরো স পসর তোসত কলিব লিবনদমোতর দঃলিত কোর মতয লিবসর সবোভোলিবক পলিরলিত এবং মোষ মরী তোই পরবোস Pলিদ তা োর মতয য় তবও কলিব লিবচলিত সব ো কোর পলিবীসত লেকউ অমর য় লিক4ই অকষয় য় দীর লেPম লিচরপরবোমো লেতমলি মোসষর ীবও চমোতোই ীব - সতবধতোই মতয ীব দীসত মোষ লিতয পরবোমো তবও লেPব মোষ আপ কতকসম13র মো3যসম মোসষর মস লিসসদর সথো কসর লিসত পোসর তোরো লিচরভোসবর সয় মোসষর মস লিবরো কসর তোসদর মস3য লেকউ পGভসত লিবী সয় গোসও মোসষর মস তোরো লিতযপলিত লিতযবলিনদত

Hindi 2ndlang

काकीी(लिसयारामशरणगपत)

इस कहानी म क न यह बतान का परयास निकया ह निक बचच अपनी मा स निकतना परम करत ह शयाम अबोध बाक ह वह अपनी मा क मरन क बाद उसन अपनी मा क लिए बहत रोया बाद म उस पता चा निक उसकी मा राम क घर ची गई ह आकाश म उडती हई पतग दकर उस हष हआ निक पतग क दवारा वह अपनी मा को नीच उतारगा इसक लिए वह अपनी निपता की जब स दो बार सवा रपया निनकाकर पतग और दो मोटी सी मन वाी अपन भाई स काकी एक कागज पर लिवा कर पतग म लिशव का दिदयानिनकाकर पतग और दो मोटी सी मन वाी अपन भाई स काकी एक कागज पर लिवा कर पतग म लिचपका दिदयाभोा और शयाम कोठरी म रससी बाधनी रह थ तभी उसक निपता करोध म आकर उन स पछ निक कया उनकी जब स रपया निनकाा हभोा डर क मार बताया निक शयाम इस पतग क दवारा अपनी काकी को राम क यहा स उतारना चाहता हनिवशशवर(शयाम क निपता)न फटी पतग उठाकर दी तो उस पर काकी लिा थावह हत बजि होकर वही ड रह गएउनहोन सोचा निक मन अपन पतर को मारा जोनिक अनजान और निनदष थावह अपनी मा कोनिकतना पयार करता ह

helliphellipContinue to next

Computer Application

Java Programming Prog 1Write a java program to input two numbers from user and display the sum or product of them as per user choice Use switch case statementSolve public class sum_product public static void main(String args[]) Scanner sc=new Scanner(Systemin) int abc Systemoutprintln(ldquoEnter two numbersrdquo) a=scnextInt() b=scnextInt() Systemoutprintln(ldquoPress 1 for sum or 2 for productrdquo)

c=scnextInt() switch(c) case 1 Systemoutprintln(ldquoThe sum will be =rdquo+(a+b)) break case 2 Systemoutprintln(ldquoThe product will be =rdquo+(ab)) break default Systemoutprintln(ldquoWrong Inputrdquo) Home Work - Practice in your computer using bluej

Subject Eng Literature (The Merchant of Venice ndash William Shakespeare)Topic Act I Scene 2 Lines 92 to 126 (End of scene) Date 13th April 2020 (5th Period)

[Students should read the original play and also the paraphrase given in the school prescribed textbook]Summary Questions amp Answers

o After Portia has expressed her opinion about the suitors Nerissa informs that she need not bother about any one of them as they have decided to quit Belmont at the earliest opportunity because they do not believe in trying their luck by the caskets which is the only way of winning Portia

o Nerissa then enquires of Portiarsquos opinion about Bassanio who once visited her in the company of the Marquis of Montferrat and says that she had never come across such an ideal love deserving the fairest lady for his bride

o Portia seems to remember Bassanio quite correctly and says that she agrees with Nerissa At this moment a servant informs Portia that the Prince of Morocco has arrived to try his luck by the caskets

o Portia tells Nerissa that if she could welcome this new suitor as gladly as she says farewell to the previous ones she would be glad of his arrival However if he happens to have the virtues of a saint but the black complexion of a devil she would prefer to have him for religious consolation rather than as a husband

(1) NERISSA You need not fear lady (Line 97-103)

the having any of these lords they have acquainted me with their determinations

which is indeed to return to their home and to

trouble you with no more suit unless you may be wonby some other sort than your fathers imposition depending on the caskets

PORTIA If I live to be as old as Sibylla I will die as chaste asDiana unless I be obtained by the manner of my fatherswill I am glad this parcel of wooers are so reasonablefor there is not one among them but I dote on his veryabsence and I pray God grant them a fair departure

(a) Elucidate the idea expressed in the first speech of the above dialogue

In the first speech Nerissa assures Portia that she need not have any fear of being compelled to marry anyone of the suitors who had lately come to Belmont She informs her that they have all decided to return to their respective countries(b) Illuminate the meaning of the phrase ldquoyour fatherrsquos imposition depending on the casketsrdquo

Nerissa means that the suitors of Portia do not find the conditions imposed by the will of her father to their liking They are too hard for them These conditions are that in the event of a suitor failing to choose the right casket (i) he should never disclose to anybody which casket he chose (ii) he can never marry and (iii) he should take his departure immediately(c) Explain the meaning of the term lsquoSibyllarsquo

lsquoSibyllarsquo is the name given by Romans and Greeks to a prophetess inspired by some deity usually the sun-god Apollo She had a very long life The god Apollo granted her as many years of life as she could hold grains of sand in her hand(d) Elucidate the meaning of the term lsquoDianarsquo

lsquoDianarsquo is the goddess of hunting She is also regarded as a symbol of virginity because she never fell in love and never

married(e) Explain the meaning of the first two lines of Portiarsquos speech

Portia says that even if she is to live for centuries like Sibylla she would not marry except in accordance to her fatherrsquos will She asserts that she would not mind remaining unmarried and untouched by a man like Diana the virgin the goddess of hunting unless a man is able to win her by passing the test laid down by her father

Class XSubject Topic Summary Execution

Hindi 2nd

Langबड घर की बटी( मशी परमचद)

lsquoबड घर की बीटीrsquo कहानी का उददशय मधयम वग की घर समसया को सझा कर सगदिठत परिरवार म मिम जकर परम स रहन का सदश दना ह घर म शानित tानिपत करन की जिजममदारी नारी की होती ह यदिद नारी समझदार ह उसम धय और परिरवार क परनित परम ह तो कोई भी घटना परिरवार को निवघदिटत नही कर सकती या कहानी परिरवार को सगदिठत करत हए परम सौहाद स एक रदसर की भावनाओ को समझ करउनका सहयोग करत हए जीवन यापन करन की पररणा दती ह मशीपरमचदर जी न इस कहानी म सयकत परिरवार का परनितनिनमिधतव निकया ह यह कहानी बनी माधव सिसह जो गौरी पर क जमीदार क उनक दो पतरो की हशरी कठ ा निबहारीशरीकात का निववाह एकजमीदार घरान की पतरी आनदी स हआ थाआनदी न द को ससरा क वातावरण म ढालिया थाएक दिदन आनदी का अपन दवर ा निबहारी स झगडा हो जाता ह दोनो भाई एक रदसर स अग होन की कोलिशश करत हसभी बह आनदी न अपन मधर वयवहार स ा निबहारी को घर छोडकर जान स रोक लिया| इस पर बनी माधव सिसह न कहा निक बड घर की बटी ऐसी ही होती ह जो निबगडा काम बना ती ह अतः शीषक साथक ह बड घर की बटी आनदी ह

helliphelliphelliphellipContinue to nextBiology Topic ndash Chp-1

CellWelcome to new session 2020-21Today we will start with Chpter 1 cell CELL

Protoplasm+Cellmembrane Or Cell wall

Cytoplasm+Neucleus

Cytoplasmic+ CytoplasmicOrganelles Inclutions(mitochondria (food Golgi bodies pigments)Ribosome)

What is cellbull Cell is the structural and functional unit of living organismbull According to number of cells organisms areUnicellular - Amoeba bacteria Multicellular - Rose Mango Tiger HumanSmallest cell -bacteria Longest cell - Nerve cellLargest cell - Ostrich egg cellCells are of different size and shapes according to their functionsQ2Write chief functions of following cellorganelles

Q3What is tonoplastVacuoles covered by a covering called tonoplast

Bengali(2Nd

Language)

ফ ফটক ো ফটক (কলিবতো ) ভোষ মসোপো3 gtPোয়

একটি লেমসয়র ীবস লেপরম লিকভোসব ফসট ওসঠ তো লেদলিসয়স4 কলিব লেপরম Pই য় লেই ময়ই বনত কোস পলিরত য় ফ লেফোটো বো োসফোটো লেটো ব2 কো য় লেমসয়সদর ব gtয13 লেপরসমর 4লিব ফসট উসঠস4 এই কলিবতোয় লেপরম মোষসক মত gtযর মস লেফস লিদসয় পরকষস বাোচোসোর gtয োত বো2োয় কলিবতোয় লেমসয়টির পসব13র দঃসর কো বো সও লেমসয়টি লেই পসর পলিক সত চোয়ো োরী ীবসর কোস4 পরম লেPৌবস লেপরমসক পোবোর পরব ইচছো োকসও তো পসর লেলিতবোচকতোয় পলিরত য় কলিব ভোষ মসোপো3 যোয় লেP ক লেপরসমর

কলিবতোয় ব gtযবহত লিবসষ লিক4 সvর অ13 লেদওয়ো ১) রসবোো= লেP লিবলিভনন রকম ডোকসত পোসর২) ো= পোর ৩) ঠলি = লেচোসর বZ৪)আই বস2ো=অলিববোলিত৫)শইসয় = োলিয়ত কসর৬)োতপাোচ= লিবলিভনন পরকোর৭)দ2োম = v কসর বZ কসর লেদওয়ো৮)লেরলিং =লেোোর দৈতরী লেব2ো৯) বনত= একঋত১০) পাোর = বসকরো2

Organelles Functions

1 Endoplasmic reticulum

2 Mitochondria

3Golgibodies

4 Ribosome5Lysosome

6Plastids

7 Centrosome

i) Supportive framework for the cellii) Synthesis and transpost of proteinsRelease of energy in the form of ATPi) Synthesis and secretion of enzymes hormoneii) Formation of vacuoles lysosomei) Protein Synthesisi) Intracellular digestionii) Destroy foreign substancei )Leucoplast - stores starchii)chloroplast - trap solar energyiii) Chromoplast - imparts colour toflowers amp fruitsi) Initiates and requlates cell division

কলিবতো তোর অ13সক ভোষোয় পরকোো কসর ঘলিরসয় ব যকত কসরস4 লেপরসমর ফতো আর লিবফতো লেক গোঢ় কসর লেদোসো কলিব ভোষ মসোপো3 যোসয়র অলিভবসর অ যলিদক

Economics

Factors of Production

Welcome to the new sessionToday we are going to start the first chapter of Class XThe name of the chapter is Factors of productionBy the name I hope you all can recall a glimpse of what you have learnt in the second chapter of Class IX

NowProduction is the process of creating the various goods and services which are consumed by the people of the country to satisfy their wants

Thus it is the process in which some materials are transformed from one form to another to create utility and value in goods

For example utility can be created by changing the form of a commodity ie

Making of table out of wood by a carpenter for his customer here the wood is getting transformed into table creating utility for his customer and he can also command a price for it

On the other hand Housewives perform very

useful activities at home which create utility but their domestic activities are not included in production because they have no money value

So we can also say that Production denotes two things firstly creation of utility and secondly creation of value

Production is not complete unless it reaches the consumer

An increase in production will increase the economic welfare of the consumers and hence the aim is to raise the production level of the country

Again production of a good or service is only possible if certain resources or

Questions

1 What do you mean by production

Answer Production means the creation of goods and services for the purpose of selling in the market

In fact production involves the transformation of inputs into outputs

Hence production denotes two thingsCreation of utility and creation of valueUtility and value can be created by changing the form by changing the place by changing the time and by rendering services

Example Transformation of raw

materials into finish goods such as potter creates utility by converting mud into utensils assembling of small parts to make bigger machinery

Production also includes services such as distribution and marketing

2 What are the factors of production

Answer Factors of Production refers to the resources and inputs needed for producing goods and servicesThese inputs can be classified as

Land Labour

Capital Enterprise

Land Land is defined to include not only the surface of the earth but also all other free gifts of nature(for example mineral resources forest resources and indeed anything that helps us to carry out the production of goods and services but is provided by

inputs are used together in right proportion

A resource or an input which helps in the process of production to obtain an output is called FACTOR OF PRODUCTION

These factors of production can broadly be categorized into four parts 1LAND 2LABOUR3CAPITAL4ENTERPRISE (ORGANISATION)or Entrepreneur

The above factors are all interdependent on each other and they play a major role in production process

FACTORS OF PRODUCTION

LANDCAPITAL

LABOUR ENTREPRENEUR

nature free of cost)LabourLabour refers to the human efforts that need to be combined with other factors of production for creating an output

CapitalAll man ndash made means of production is called capita example machineries which help in further production Money when used for starting any business for purchasing raw materials machinery tools etc it is regarded as capitalCapital also includes physical capital like factories machineriestoolsbuildingsequipments etcEnterpriseThe task of bearing risks is called enterprise and the person who bears these risks of business is called the entrepreneurThus an entrepreneur is one who organises production takes important decisions regarding production hires and purchases factors of production and bears the risk and uncertainty involved in productionOrganisation refers to the services of an entrepreneur who controls organises and undertakes all risks One who plans organises and manages a business enterprise is an organiser

Physics Chapter 1 Force

Force is an external agent capable of changing the state of rest or motion of a particular body It has a magnitude and a direction The direction towards which the force is applied is known as the direction of the force and the application of force is the point where force is applied The Force can be measured using a spring balance The SI unit of force is Newton (N)

Question 1

State the condition when on applying a force the body has

(a) the translational motion

(b) The rotational motion

Solutions

(a) Translational motion is produced when the body is free to move

(b) Rotational motion is produced when the body is pivoted at a point

Question 2

Define moment of force and state its SI unit

Solutions

The moment of force is equal to the product of the magnitude of the force and the perpendicular distance of the line of action of force from the axis

of rotation

The SI unit of moment of force is Newton times meter

= Newton meter (Nm)

Commercial Studies

Stake holders In this topic you will be come to know about the meaning and concept of stakeholders

How stakeholders are different from shareholders

Questions1 What do you mean by the term stake holdersAnswer) The term stake holders have developed from the words which mean an interest or expected benefit Stakeholders mean all those individuals groups and Institutions which have a state (interest) in the functioning and performance of a commercial organisation or a business enterprise2 What do you mean by share holdersAnswer) The person and Groups who own the shares of the joint stock company by providing capital to the company are called shareholders Shareholders are the internal stakeholders shareholders are one out of several stake holders3 How are shareholders different from stakeholdersAnswer)i) The term shareholders is related to only joint stock company whereas stakeholders are related with all business organisationsii) Stakeholders maybe any individual having financial stake in business organisation whereas a shareholders are those individuals who are holding shares in the company4) How are shareholders different from creditorsAnswer) i) Shareholders are internal stakeholders while creditors are external stakeholdersii) Shareholders invest in the capital of the company whereas creditors give loan to the companyiii) Shareholders are the members of the company with voting rights but creditors are not the members of the company

English 1 Transformation of sentences

Sentences A sentence is a group of words which makes complete sense

e Assertive sentencesf Imperative sentencesg Interrogative sentencesh Exclamatory sentences

Sentences can be changed from one grammatical form to another without changing the meaning of the sentence This is known as transformation of sentences

Exercise 1 Change the following affirmative sentences into Negative sentences

a He is a good manHe is not a bad man

b Ram loves SitaRam is not without love for Sita

c Only he stood first in the classNone but he stood first in the class

d Ankit was wiser than he

He was not so wise as Ankite He did it

He did not fail to do itf As soon as I reached college the

bell rangNo sooner did I reach college than the bell rang

g He finished everythingHe left nothing unfinished

h It always pours when it rainsIt never rains but it pours

Math Topic Commercial MathematicsChapter ndash Goods and services Tax

What is GSTAns It is a abbreviated term of Goods and Service Text which is an indirect tax levied on the sale of goods and rendering servicesSome terms related to GSTDelar Any person who buys goods or services For resale is known as a delar A delar Can be a firm or a companyIntra-state sales Sales of goods and services within the same state or same union territory are called intra- state salesInter-state sales Sales of goods and services outside the state or union territory are called Inter-state sales4) Input GST GST is paid by dealers on purchase of goods and services are called input GST5) Output GST GST is collected from customers on sale of goods and services are called output GST6) Types of GST There are three taxes applicable under GST(i) Central Goods and Services Tax (CGST)(ii) State Goods and Services Tax (SGST) or Union Territory Goods and Services Tax (UTGST) Both these taxes are levied on intra-state sales Here GST is divided equally among central and state governments(iii) Integrated Goods and Services Tax (IGST) IGST is levied on inter- state sales It is also levied on import of goods and services into India and export of goods and services from India

Subject Eng Literature (The Merchant of Venice ndash William Shakespeare)Topic Act III Scene 4 Lines 1 to 44 (Portia hellip To wish it back on you fare you well Jessica)[Students should read the original play and also the paraphrase given in the school prescribed textbook]

Summary Questions amp AnswersIn this scene we suddenly find a new element in the character of Portia We have already seen her possessed of every graceful womanly quality but now she shows that she is capable of rapid decision and determined action She shows this by her sudden resolve to hasten to Venice with a daring scheme for the rescue of Antonio This is an important scene in the dramatic action for it leads up to and renders possible the striking events of the famous trial scene which is one of the greatest striking elements of the play Moreover the fact that all the characters of importance are now assembled together in Venice makes the union of the main plot and the secondary story complete

(1) LORENZO Madam although I speak it in your presence(Line 1-9)

You have a noble and a true conceit

Of god-like amity which appears most strongly

In bearing thus the absence of your lordBut if you knew to whom you show this honourHow true a gentleman you send reliefHow dear a lover of my lord your husbandI know you would be prouder of the workThan customary bounty can enforce you

(a) Where is Lorenzo Why is he here To whom is he referring as lsquoMadamrsquo

Lorenzo is at Portiarsquos residence He had met Salerio on the way and Salerio had begged him to come along with him to

o In this scene Portia Nerissa Lorenzo Jessica and Balthazar appear

o Portia requests Lorenzo and Jessica to be in charge of her house during her absence from Belmont because she and Nerissa have decided to spend the days in meditation and also in visiting the holy places in the neighbourhood of Belmont She has already instructed her people to acknowledge both Lorenzo and Jessica as master and mistress of house during her absence Lorenzo and Jessica gladly agree to look after the house of Portia

handover the letter from Antonio to Bassanio The letter carried the bad news about Antoniorsquos arrest for non-payment of loan taken from Shylock Hence Salerio might have preferred company to break this bad news to Bassanio He is referring to Portia as Madam(b) What does Portia say on hearing the above extract

Portia says that she has never regretted doing good to others Friends who spend a lot of time together and really are there for each other have many traits in common As Antonio is Bassaniorsquos best friend saving him is like saving Bassanio who is like her own soul She asks Lorenzo to take care of management of the house till Bassanio is back(c) What does Portia send with Bassanio and why

On hearing about Antoniorsquos troubles on account of Bassanio her husband Portia immediately sends him with enough gold to repay the debt many times over to Venice to help Antonio out of his misfortune

(2) Lorenzo Madam with all my heart (Line 36-40)

I shall obey you in all fair commands

Portia My people do already know my mindAnd will acknowledge you and JessicaIn place of Lord Bassanio and myselfSo fare you well till we shall meet again

(a) Where are Lorenzo and Portia at this time What lsquofair commandsrsquo are given to Lorenzo

Lorenzo and Portia are at Belmont during this scenePortia reveals to Lorenzo that she has sworn to contemplate in prayer at a monastery around two miles away until her husband returns from Venice She tells him that Nerissa would accompany her and asks him to manage the house with Jessica till things are settled In response Lorenzo tells her that he would be obliged to do whatever she asks him to do(b) Where is Portia actually going and why

Portia tells Lorenzo that she would live a life of contemplation and pray at a monastery which is two miles away from her place In reality Portia plans to go to Venice in disguise with Nerissa and argue the case in defense of Antonio She is very sure that her plan would succeed

ClassXI (ScienceHumanitiesCommerce)Subject Topic Summary Execution

Computer Science

(APC)

Ch ndash 1 Numbers

(Numbers in different bases and

their Arithmatical operations)

Number System In computers Number System is defined as a writing system to represent the numbers in different ways ie we are using different symbols and notations to represent numbers There are four ways we can represent the number ndash Binary Decimal Octal and Hexadecimal

Decimal Number SystemThis number system consist 10 digits These are 0 1 2 3 4 5 6 7 8 amp 9

Binary Number SystemThis number system has only two digits these are 0 and 1 Here 0 stands for off while 1 stands for on

Octal Number SystemThis number system has 8 digits these are 0 1 2 3 4 5 6 amp 7

Hexadecimal Number SystemThis number system has 16 digits these are 0 1 2 3 4 5 6 7 8 9 A B C D E F Here the value of the alphabets are as follows A=10 B=11 C=12 D=13 E=14 F=15

Rules for conversion decimal number to Binary1 Divide the decimal number by 22 If the number will not divide equally by 2 then round down the answer to the nearest whole number (integer)3 Keep a note of the remainder it should be either 0 or 14 Keep repeating the above steps dividing each answer by 2 until you reach zero5 Write out all the remainders from bottom to top This is your binary solution

For example Lets convert 32 to binary 2 32 2 16 - 0 2 8 - 0 2 4 - 0 2 2 - 0 2 1 - 0 0 - 1

The binary equivalent of 3210 is 1000002

Try the follwing youself1 2410

2 4810

3 1210

History GROWTH OF NATIONALISM

The second half of the 19th century witnessed growth of political consciousness and a sense of Nationalism among the IndiansThere were various factors for growth of Indian Nationalism- As a result various political associations were formed in different provinces by the educated Indians Surendranath Banerjee organized a meeting of National conference at Calcutta Ultimately the National Congress was founded in Bombay in 1885This body became the vanguard of Indian struggle for freedom The congress leaders were known as moderates because they followed a policy of prayer and petition A large number of Indian leaders had experienced in political agitation The Political situation of England was also changed Moreover increasing revolutionary activities in Maharashtra Punjab and Bengal became serious concern to the British Government In this

QUESTION1 What do you mean by Nationalism ANSWER 1 Nationalism is defined as loyalty and devotion to own nation especially a sense of national consciousnessQUESTION 2 What are the causes of nationalism ANSWER 2 There were various factors for growth of nationalism

1 Spread of western education2 The progress of vernacular press and

patriotic literature3 The economic exploitation of our

country by the colonial rulers4 International affairs

QUESTION 3 Who organized National conference in Calcutta in 1883 ANSWER 3 Surendranath BanerjeeQUESTION 4 When did Indian National Congress formANSWER 4 Indian National Congress was formed in 1885 in BombayQUESTION 5 Who were ModeratesANSWER 5 The Early Nationalists were also known as Moderates Their emergence marked

background Lord Curzon became Viceroy in India He had no respect for the Indian National Congress

the beginning of the organized national movement in India They believed in British justice and were loyal to them They followed a policy of prayer and petition They demanded constitutional reforms of our country Impotant Moderate leaders were Pherozshah Mehta Dadabhai Naorozi and Surendranath Banerjee etcQUESTION 6 What do you know about Extremism in Indian National movementANSWER 6 In the beginning of 20th century a new class of national leaders emerged in India which was different from the moderate groups They started more aggressive movement against the British empire The goal of extremists was ldquoswarajrdquo Important extremist leaders were Bal Gangadhar Tilak Lala Lajpat Rai Bipin Chandra Pal etcQUESTION 7 Mention the places which were the main centres of Revolutionary movementANSWER 7 Maharashtra Bengal and Punjab

Physics

Chapter Dimensional Analysis

(Summary)

The dimensions of a physical quantity are the powers to which the fundamental units are raised in order to obtain the derived unit of that quantit

The physical quantites lengthmasstime are represented by [L] [M] [T] resp let they are raised to powers ( dimesions) abc resp then any physical quantity can be represented by [ La Mb Tc ] Examples

1 Area area = L x B = [L] x [L] = [M0 L2 T0 ]

2 Density density = massvolume = [M][L3] = [ M L-3]

3 Velocity velocity = distancetime = [L][T] = [LT-1]HW Try to find out dimension of acceleration Acceleration = velocity timeNB One can find the SI Units Using Dimension Analysis Such as for area we have [L2] so its SI unit is m2

Biology Topic ndash Chp-1 The living world

Today we will start the first chapter the living world Here we discuss about the characteristics of living organism and what are the difference between them and nonliving substances We also discuss about the contribution of different Scientists

There are over 500000 species of plants andover a million species of animal are present on earth Some 15000 new species were discovered every yearQ1 What is a living organismbull A living organism is primarily physico -chemical material that demonstrate a high degree of complexity is capable of selfRegulation possesses a metabolism and perpetuates itself through timeQ2 What are the differences between livingand non-livingsi) Compared with non-living living organisms

have more complex organised structure and their use of energy is more controlled amp efficientii) Living things reproduce their own kind by forming new cells which contains copies of their genesiii) Each organism has some degree of homeostasisie it is able to make adjustments so that internal environment remains constantQ3 Write contributions of following Scientists i) Aristotle - One of the first theories in Biology places all living things in a hiearchieii) AV Leeuwenhoek - was the first to observe living single celled organisms under microscopeii) Carolus Linnaeus - developed the binary system for naming of organisms and classificationiii) Geregor Johann Mendel ndash discoverbasic principles of inheritanceHomework i) C Darwin ii)Schleiden

Math Trigonometric functions

1 Overviewi) Trigonometry The word lsquotrigonometryrsquo is derived from the Greek words lsquotrigonrsquo and lsquometronrsquo which means measuring the sides of a triangle An angle is the amount of rotation of a revolving line with respect to a fixed line Usually we follow two types of conventions for measuring angles ie a) Sexagesimal system b) Circular system In Sexagesimal system the unit of measurement is Degree In Circular system the unit of measurement is Radian ii) Relation between degree and radianThe ratio of circumference of a circle to its diameter is always a constant This constant ratio is a number denoted by π which is taken approximately as 227The relationship between degree amp radian measurements is as follows2 right angles = 180deg= π radians1radian = 180degπ=57deg16(approx) 1deg=π180 radianiii) Length of an arc of a circleIf an arc of length s subtends an angle θ radians at the center of a circle of radius r then s=rθiv) Area of a sector of a circleA sector is like a pizza slice of the

Q) Express the following angles in radiana) 45deg b) 40deg3730Ans a) We have 180deg=π radiansi e 45deg= πtimes45180 radian = π4 radiansb) 40deg3730= 40deg37+3060 minute= 40deg 37 +12 minute= 40deg+ 752 minute=40 + 75(2times60) degree=3258 degreeNow 180deg=π radianie 3258 degree= (πtimes325) (180times8) radians = 65π288 radiansQ) A circle has a radius of r=12 meters What is the length of an arc traced out by a 60deg angle in the center of the circleAns In this problem we know both the central angle (60deg) and the radius of the circle (12) All we have to do is plug those values into our equation and we get

s = 2π(12)(60360)s = 24π6s = 4πSo the length of an arc traced out by a 60deg angle in a circle with a radius of 12 meters equals 4π meters asymp 1257 metersQ) Find the area of the sector with a central angle 30deg and a radius of 9cmAns GivenRadius r = 9 cmAngle θ = 30degArea of the sector = θ360degtimesπr2

= 30360degtimes227times92=2121cm2

circle It consists of a region bounded by two radii and an arc lying between the radiiThe area of a sector is a fraction of the area of the circle This area is proportional to the central angle In other words the bigger the central angle the larger is the area of the sectorArea of Sector = θ2 times r2 (when θ is in radians)

Area of Sector = θ times π360 times r2 (when θ is in degrees)

COMMERCE

CLASSIFICTION OF HUMAN ACTIVITIES-ECONOMIC AND NON-ECONOMIC

Welcome to the new sessiontoday we are going to start the first chapter of Class XI The name of the chapter that we are going to start is

lsquoClassification of Human Activities ndasheconomic and non-economicrsquo

Now let us start the chapter by considering human beings and the activities they perform throughout the day

Human activities means all those activities that human beings undertake to satisfy their wants

Human wants on the other hand are the desire of human beings for goods (vegetables fruits rice etc) and services (services of doctors teachers lawyers etc) that they require to live

Now these human activities continue throughout life as human wants are unending unlimited and recurring as human beings desire for better living throughout their lives

Now human activities can be classified into two categories

Human activities

Economic activities Non-economic activities

Economic activities are

Questions1 What are human activities

Answer Human activities mean all those activities that human beings undertake to satisfy their wants

Example A man working in an office

A boy playing in the garden

2What are the characteristics of human activitiesAnswer the characteristics of human activities are as follows

Human activities are undertaken by men women and children and these activities involve human efforts

Human activities are undertaken to satisfy human wants which are unlimited

Human activities continue throughout life

Human activities are performed for both earning money and personal satisfaction

3What is economic activitiesGive example

Answer Economic activities are undertaken by human beings with the object of earning money acquiring wealth and thereby satisfying human wantsExample

Selling of goods by a shop keeper to his customer

A clinic run by a doctor Service of a teacher in school or college

undertaken by human beings with the object of earning money and acquiring wealth

These activities result in the production of economic goods and services

Example Human activities(ie working in factories officesshops) which produce direct economic benefits

Non-economic activities are inspired by human sentiments and emotions such as love for the family desire to help the poor and love for the country

Thus these human activities (eg praying playing sleeping) produce no direct economic benefits and they are also not related to earning money and acquiring wealth

4 What are the characteristics of economic activities

Answer The characteristics of economic activities are as follows

Economic motiveEconomic activities are undertaken to earn money and acquire wealth

ProductiveEconomic activities involve productiondistribution and exchange of goods and services to create wealth

Economic growthEconomic activities determine the level of economic development of a country and standard of living of its citizens

Socially desirableEconomic activities are socially desirable for society

Economic resourcesEconomic activities make use of all the economic resources such landlabourcapital etc

5 What do you mean by non-economic activitiesExampleAnswerNon-economic activities are inspired by human sentiments and emotions such as love for the family desire to help the poor and love for the countryThese activities are not undertaken for monetary gain but for onersquos satisfaction and happinessExample

a mother looks after her children

a student donates blood8 Differentiate between Economic activities and Non-economic activities

Economic activities

Non-economic activities

1to earn living and acquiring wealth2Result can be measured in terms of money

3ExampleBusinessprofession and employment

1 to obtain some satisfaction

2Result cannot be measured in terms of money

3ExampleFamily-orientedreligious socialCultural and national

BUSINESS STUDIES

BUSINESS ENVIRONMENT

Welcome to the new sessionToday we are going to start the first chapter and the name of the chapter is Business Environment

In todayrsquos world every business enterprise is a part of the society It exists and operates in association with various groups in society such as customers suppliers competitors banks and financial institutions government agencies trade unions media and so on All these groups influence the functioning of business in one way or the other They constitute the environment of businessConcept of Business Environment

The term lsquobusiness environmentrsquo refers to the sum total of all individuals institutions and other forces that lie outside a business enterprise but that may influence its functioning and performance

The main features of business environment

Totality of External forces General and Specific forces Interrelatedness Complexity Dynamic Uncertainty Relativity

The Interrelation between business and its environment

The business enterprise is an open system It continuously interacts with its environment It takes inputs

Prepare the following questions from todayrsquos assignment

1 What do you mean by business environment

The term lsquobusiness environmentrsquo means the aggregate of all forces factors and institutions which are external to and beyond the control of an individual business enterprise but they may influence its functioning and performance Business environment is the macro framework within which a business firm a micro unit operates It consists of several interrelated and interacting elements

2 Explain the main features of business environment in brief

Totality of External forces-Business environment is the sum total of all things external to a business environment

General and Specific forces-It includes both the forces general forces are the economic social political legal and technological conditions which indirectly influence all business enterprise Specific forces are the investors customers competitors and suppliers which influence individual enterprise directly

Interrelatedness-Different elements of environment are interrelated for an example growing awareness for health care has increased the demand for health foods

Complexity- Business environment id

(such as raw materials capital labour energy and so on) from its environment transforms them into goods and services and sends them back to the environment

Fig 1 Business Environment Relationship

complex in nature as the elements keep on changing example economic technological and other forces changes in demand for a product and service

Dynamic-Business environment is not static it keeps on changing

Uncertainty- Itrsquos very difficult to predict future events such as technology and fashion which occur fast and frequently

Economics Basic Economic ConceptsSub topic

Microeconomics and

Macroeconomics

Welcome to the new sessiontoday we are going to start the first chapter of Class XI The name of the chapter that we are going to start is Basic Economic concepts

Now Economics covers the study of human activities Human activities are those activities which are performed by humans to satisfy their wants

Thus Human wants are unlimited and therefore economic activities such as production exchange and consumption are needed in order to satisfy those wants

The study of economics is divided largely in two parts which areMicroeconomics and Macroeconomics

SUBJECT- MATTER OF ECONOMICS

MICROECONOMICS MACROECONOMICS

Questions1Who has coined the words micro and macro economics

Answer Ranger Frisch coined the words lsquomicrorsquo and lsquomacrorsquo in 1933 to denote the two branches of economic theory namely microeconomics and macroeconomics

2What is microeconomicsAnswer It is the study of behaviour of individual decision ndash making unit such as consumers firms etc

3 What is macroeconomicsAnswer Macroeonomics is the study of overall economic phenomena like employment national income etc

4 What is the importance of microeconomicsAnswer

Microeconomics helps in formulating economic policies which enhance productive efficiency and results in greater social welfare

It helps the government in formulating correct price policies

It explains the working of a capitalistic economy where individual units(producers and consumers ) are free to take their own decision

Micro means a small part in

microeconomics we do not study the whole economy Hence we study an individual consumer and his or her choices and a producer and his or her profit maximizing decisions in the market Thus it does not mirror what happens in the economy as a whole

Macroeconomics on the other hand studies the economy as a whole It is concerned with aggregate and depicts the entire picture of the economyMacroeconomics deals with the national income aggregate investment aggregate consumption etc

Features of Microeconomics It deals with small

parts of the country Hence it looks at

individual consumers firms and industries

It deals with individual income consumption and savings

It studies the determination of price of any product or factors of production

It deals with the working of market via the price mechanism which is nothing but the determination of price and quantity of a commodity by the forces of demand and supply

Features of Macroeconomics

It deals with the study of the economy as a whole

It is concerned with

5 Give a limitation of microeconomics Microeconomics fails to explain the

functioning of an economy as a whole It cannot explain unemployment illiteracy and other problems prevailing in the country

6 What is the importance of macroeconomics It gives overall view of the growing

complexities of an economic system It provides the basic and logical

framework for formulating appropriate macroeconomic policies (eg for inflation poverty etc )to direct and regulate economy towards desirable goals

7What is the limitation of macroeconomics It ignores structural changes in an

individual unit of the aggregate

8 Differentiate between Microeconomics and Macroeconomics

Microeconomics Macroeconomics

the study of aggregates

National income aggregate savings and aggregate investments are major concepts dealt within macroeconomics style

It studies the determination of general price levels

It investigates into the problem of unemployment and the achievement of employment

It studies the aspect of decision making at the aggregate and national levels

It includes all growth theories whether related to developed or developing economies it also includes the study of economic systems and the working of the economy under different systems

Note Both Micro and macro economics are complementary and should be fully utilized for proper understanding of an economy

1It studies economic aspect of an individual unit2It deals with individual incomeConsumption and savings

3 It facilitates determination of price of any product or factors of production

4 Itrsquos scope is narrow and restricted to individual unit

1It studies the economy as a whole

2It deals with the national income aggregate consumption and aggregate savings3 It facilitates determination of general price level in an economy

4 Itrsquos scope is wide as it deals with economic units on the national level

ACCOUNTS

Introduction to Accounting and Book-keeping

Today I am going to share you the meaning of Accounting and Book-keeping and its related terms bullAccounting bullBook Keeping bullAccountsbullTypes Of Accounts bullAccounting Cycle

bull Meaning of accounting

Ans ) Accounting is the art and science of recording classifying and summarising monetary transactions

bull Meaning of Book-keeping

Ans) Bookkeeping is the art of recording business transactions with the view of having a permanent record of them and showing their effect on wealth

bull Meaning of account

Ans) The term account means a record of

business transactions concern a particular person of firm asset or income or expense It is a summarised record of all transactions which take place in an accounting year

bull Types of accountsPersonal accounts ndash Personal accounts relating

to person and Organisation are known as personal accounts Example Ramrsquos Account ABC amp Co Account etc

Real account - The accounts related to tangible and intangible assets are called real accounts Example Cash Account Furniture Account etc

Nominal account- Accounts related to expenses losses incomes and gains are known as nominal accounts Example Wages Account Salary Account Discount Account etc

bull Accounting cycle Accounting cycle refers to a complete sequence of accounting activities It begins with recording of transactions and ends with the preparation of a balance sheet

Chemistry TopicAtomic Structure

Thomsonrsquos atomic modelThomson (1898) was the first to propose the model of an atomHe proposed that an atom can be regarded as a uniform sphere of positive electricity in which requisite number of electrons are embedded evently to neutralize the positive chargeThis is just like plums embedded in a pudding or seeds evently distributed in red spongy mass of a watermelonThis model of atom is known as ldquoPlum-Pudding modelrdquo or

Q1)What is the fundamental constituents of atomAns Electron Proton and neutrons are the fundamental constituents of atomQ2)What is the value of fundamental unit of electricityAnsThe charge carried by one electron is sad to be the fundamental unit of electricityIts magnitude is 48times10-10esuOr 1602times10-19C Q3)Name the element containing no neutronAnsOrdinary hydrogen atom or protium 1H1

Types of AccountPersonal AccountReal AccountNominal AccountBalance Sheet (opening)

ldquowatermelon modelrdquoThis model could explain the electrical neutrality of an atom but failed to explain the result of scattering experiment carried out by Rutherford in 1911So it was rejected ultimately

Q4)Why is an electron called universal particleAns Itrsquos mass and Charge are independent of its source

EVS Chapter 1 ndash Modes of Existence

Modes of existence When one speaks normally about the mode of existence of some group or individual one refers to their customs their mode of being their ethology their habitat in some way their feeling for a placeDifferent modes of exixtence are ndash

1 Hunting ndashGathering2 Pastoral3 Agricultural4 Industrial

1 Hunting and gathering Hunting and gathering mode of existence is characterized by obtaining food from hunting wild animals including fishing and gathering wild plants From their earliest days the hunter-gatherer diet included various grasses tubers fruits seeds and nuts Lacking the means to kill larger animals they procured meat from smaller game or through scavenging

Societies that rely primarily or exclusively on hunting wild animals fishing and gathering wild fruits berries nuts and vegetables to support their diet are called hunting and gathering societies

At least this used to be practice of human beings before agriculture is invented As their brains evolved hominids developed more intricate knowledge of edible plant life and growth cycles

Q) Write the features of Hunting ndash gathering societiesAns - There are five basic characteristics of hunting and gathering societies

i The primary institution is the family which decides how food is to be shared and how children are to be socialized and which provides for the protection of its members

ii They tend to be small with fewer than fifty members

iii They tend to be nomadic moving to new areas when the current food supply in a given area has been exhausted

iv Members display a high level of interdependence

v Labor division is based on sex men hunt and women gather

Political Science

Introduction to political science

Political science occasionally called politology is a social science which deals with systems of governance and the analysis of political activities political thoughts associated constitutions and political behaviorThe study of political science involves the study of both the

Answer the following questions-1 What is political science

Political science occasionally called politology is a social science which deals with systems of governance and the analysis of political activities political thoughts associated constitutions and political behavior

2 Short notes-

traditional and modern theories of politicsTraditionalClassical political sciencepolitical theory-Traditional political science is the study of politics before Second World War The methodology to study Politics was traditional (legal formaletc) the definition of politics traditional (Politics begins and end with state)area of study (constitution state machinery)was traditionalModern Political scienceModern political theory-Modern Political Theory critically examines the contemporary state of political theory making an assessment of the achievement and limitations of the Behavioural Revolution in its totality and reviews objectively the major paradigms and conceptual frameworks adopted by the disciplineContemporary attempts at the development of an integrated political theory involving the use of both traditional and modern concepts approaches and theories-Around late 1960s several political scientists realized the importance of both the traditional political theory and modern Political theory They began building an integrated theory of politics involving a systematic mixture of traditional and modern studies of politics It was held that the study of a complex and vast field like politics needs both traditional as well as

Classical political theory Modern Political theory

Homework-Learn

modern concepts and approaches for studying itrsquos all aspects

Subject Eng Literature (The Tempest ndash William Shakespeare) Topic Act I Scene 1 Lines 1 to 32 (Line 32 ndash Gonzalo hellip If he be not born to be hanged our case is miserable) Date 13th April 2020 (3rd Period)

[Students should read the original play and also the paraphrase given in the school prescribed textbook]Summary Questions amp Answers

[SUMMARY OF THE ENTIRE SCENE]

o The play starts with the scene of a severe storm at sea Alonso (King of Naples) Sebastian (Alonsorsquos brother) Ferdinand (Alonsorsquos son) Gonzalo Antonio (the usurping Duke of Milan) are in a ship in the midst of the storm

o The mariners are trying their best to control the vessel from running aground and are totally following the orders of their Master the Boatswain They have scant success

o The mariners become extremely unhappy and annoyed when most of the passengers arrive on the deck thereby hampering their effort to save the ship There is serious confrontation between them and the passengers who are part of the Kingrsquos entourage

o The mariners could not save the ship

SUMMING-UP

(i) Vivid description of the scene which gives a realistic description of terror and confusion of a tropical storm

(ii) Shows Shakespearersquos accuracy of knowledge in describing the naval operations and also matters of seamanship

(iii) The opening scene justifies the title ndash The Tempest

UNANSWERED QUESTIONS

(i) The King always travels with his entire fleet including his soldiers Where

(1) GONZALO Nay good be patient (Line 15-26)BOATSWAIN When the sea is Hence What cares these

roarers for the name of the king To cabin silence Trouble us not

GONZALO Good yet remember whom thou has aboardBOATSWAIN None that I more love than myself You are a

councillor if you can command these elements to silence and work

the peace of the present we will not hand a rope more use your authority If you cannot give thanks you have

lived so long and make yourself ready in your cabin for the mischance of the hour if it so hap [To the Mariners]

Cheerly good hearts [To Gonzalo] Out of our way I say

(a) To whom is the boatswain speaking What does he mean by lsquoNone that I more love than myselfrsquo

The Boatswain is speaking to Gonzalo the honest old councilor of the Duke of MilanBy using the words ndash lsquoNone that I love more than I love myselfrsquo means that for the Boatswain nobody is dearer to him than his own life

(b) What were the conditions that made the boatswain react in this way

The Boatswain reacts in this way because the storm is at sea and Alonso King of Naples Sebastian his brother Ferdinand his son Gonzalo Antonio the usurping Duke of Milan on board are in distress and in panic Thus they have rushed to the deck interrupting the work of the mariners

(c) What hope does Gonzalo take from the attitude of the boatswain

The insolent and authoritative attitude of Boatswain makes Gonzalo feel comforted He tells that there are no signs that the Boatswain will be drowned But his facial appearance and attitude shows that he is destined to die on land by hanging which in effect means that all on board will be saved Otherwise all the persons on board are doomed

(d) How can they lsquomake yourself ready in your cabinrsquo For what were they asked to make ready themselves

In order to make themselves ready in their cabin the

were the other ships

(ii) Why was the ship in that area Where was it coming from or going where

(iii) The ship broke apart What happened to those who were in the ship

passengers on board must prepare for death which they will possibly soon have to meetThey can retire to their cabins and offer prayers to the Almighty to save them from drowning

(e) What does the boatswain say when he is asked to be patient What does he order to the royal party

When the boatswain is asked to be patient and remain calm he says that he will be patient only when the storm will be over and the sea will be calm but as long as the storm blows and there is danger to the ship he cannot think of being patient He orders the royal party to go to the cabin and leave the mariners to their work

(2) GONZALO I have great comfort from this fellow (Line 27-36)

Methinks he hath no drowning mark upon him his complexion is perfect

gallows Stand fast good Fate to his hanging Make the rope of his destiny our cable for our own doth little advantage If he be not born to be hanged our case is miserable

(a) Why does Gonzalo regard the Boatswain in the midst of danger

In the midst of danger Gonzalo regards the boatswain because he feels that the Boatswain is a source of comfort and is bent upon to do his work sincerely which in this case is saving the ship and its passengers from the severest of raging storm

(b) What reasons does Gonzalo give when he says that none in the ship will die of drowning

Gonzalo is almost sure that none in the ship will die by drowning His says that there is no mark on the face of the boatswain that indicates that he will die by drowning On the other hand the lines on his face are strong indications that he will be hanged to death Therefore there shall be no danger of the shiprsquos sinking

(c) Explain the following ldquoStand fast good Fate to his hanging Make the rope of his destiny our cable for our own doth little advantage If he be not born to be hanged our case is miserablerdquo

The stated lines mean that if the will of destiny is to be carried out then the ship will not get wrecked and all the passengers will be saved The safety of the passengers therefore depends upon the will of fate being carried out in the case of the boatswain If however the boatswain is not to die by hanging then the passengers are also very unsafe because in that case the ship is likely to sink

(d) What order does the Boatswain give to the sailors

when he re-enters What does he say about the crying of the fellows inside the cabin

The boatswain orders the sailors to bring the topmast lower and bring the ship close to a stationary position with the help of the main sail He says that the fellows inside the cabin are moaning and crying in their distress louder than his voice and louder even than the roaring of the storm

Class XII (ScienceCommerceHumanities) Subject Topic Summary Execution

Computer Science

PropositionalLogic

Propositional logic is a procedure to provide reasoning through statementProposition A ststement that results in True or False is said to be proposition There are two types of propositionSimple proposition amp compound propositionSimple proposioton A simple proposition is one that is not a part of any other proposition Such sentential form of proposition is symbolized with english letters in short For example Ram is a claver student (TrueFalse)Where do you live (Not in True or False)Grapes are sweet (TrueFalse)It rains today (TrueFalse)Here we can see some statements anwer would be true or false but some staements answer can not give in terms of true or false Thus the sentences which can be answered in true or false are known as simple propositionAssigning propositon to a variableThe general syntax to assign propostion to a variable is as followsVariable = Simple propositonFor example A=Ram is a clever studentB= Grapes are sweetC= it rains todayCompound proposition

helliphellipto be continued in next classhelliphellipMath Relation Relation If A and B are two non-empty sets

then a relation R from A to B is a subset of AxB If R A x B and (a b) R then we say that a sube isinis related to b by the relation R written as aRbeg Let A be the set of students of class XII and B be the set of students of class XI Then some of the examples of relation from A to B arei) (a b) AXB a is brother of bisinii) (a b) AXB age of a is more than age of isinb Types of relation In this section we would like to study different types of relations We know that a relation in a set A is a subset of A times A Thus the empty set φ and A times A are two extreme relations For illustration consider a relation R in the set A = 1 2 3 4 given by R = (a b) a ndash b = 10 This is the empty set as no pair (a b) satisfies the condition a ndash b = 10 Similarly R = (a b) | a ndash b | ge 0 is the whole primeset A times A as all pairs (a b) in A times A satisfy | a ndash

Example 1 Let A be the set of all students of a boys school Show that the relation R in A given by R = (a b) a is sister of b is the empty relation and R = (a b) the primedifference between heights of a and b is less than 3 meters is the universal relationSolution Since the school is boys school no student of the school can be sister of any student of the school Hence R = φ showing that R is the empty relation It is also obvious that the difference between heights of any two students of the school has to be less than 3 meters This shows that R = A times A is primethe universal relation Example 2 Show that the relation R in the set 1 2 3 given by R = (1 1) (2 2) (3 3) (1 2) (2 3) is reflexive

b | ge 0 These two extreme examples lead us to the following definitionsDefinition 1 A relation R in a set A is called empty relation if no element of A isrelated to any element of A ie R = φ A times AsubDefinition 2 A relation R in a set A is called universal relation if each element of A is related to every element of A ie R = A times A Both the empty relation and the universal relation are some times called trivial relation Definition 3 A relation R in a set A is called(i) reflexive if (a a) R for every a Aisin isin(ii) symmetric if (a1 a2) R implies that (aisin 2a1)

R for all aisin 1 a2 Aisin(iii) transitive if (a1 a2) R and (aisin 2 a3) R isinimplies that (a1 a3) R for all aisin 1 a2 a3 AisinDefinition 4 A relation R in a set A is said to be an equivalence relation if R is reflexive symmetric and transitive

but neither symmetric nor transitiveSolution R is reflexive since (1 1) (2 2) and (3 3) lie in R Also R is not symmetric as (1 2) R but (2 1) isin notinR Similarly R is not transitive as (1 2) R and (2 3) R but (1 3) R isin isin notinExample 3 Show that the relation R in the set Z of integers given byR = (a b) 2 divides a ndash b is an equivalence relationSolution R is reflexive as 2 divides (a ndash a) for all a Z isinFurther if (a b) R then 2 divides a isinndash b Therefore 2 divides b ndash a Hence (b a) R which shows that R is isinsymmetric Similarly if (a b) R and (b c) R isin isinthen a ndash b and b ndash c are divisible by 2 Now a ndash c = (a ndash b) + (b ndash c) is even (Why) So (a ndash c) is divisible by 2 This shows that R is transitive Thus R is an equivalence relation in ZExample 4 Let L be the set of all lines in a plane and R be the relation in L defined as R = (L1 L2) L1 is perpendicular to L2 Show that R is symmetric but neither reflexive nor transitiveSolution R is not reflexive as a line L1 can not be perpendicular to itself ie (L1 L1) R notinR is symmetric as (L1 L2) Risin

L1 is perpendicular to L2rArr L2 is perpendicular to L1rArr (L2 L1) RrArr isin

R is not transitive Indeed if L1 is perpendicular to L2 and L2 is perpendicular to L3 then L1 can never be perpendicular to L3 In fact L1 is parallel to L3 ie (L1 L2) R isin(L2 L3) R but (L1 L3) Risin notin

Chemistry Solid state Characteristics if Solids(i)The particles are locked in fixed positions they are unable to change their relative positions and this brings a definite shape and volume of a solid(ii)In a solid the constituent particles are held by strong forces of attractionThe forces of attraction may be bonding or non bonding(iii)The constituent particles in a solid pack together as closely as possibleoccupying most of the available space within the solidThus the empty space in a solid is very smallThis makes a solid highly rigid and nearly incompressibleThis also explains why a solid has high density and exhibits slow diffusionClassification of Solids

Q1)Define Crystalline solids AnsA Solid that has a definite geometrical shape and a sharp melting pointand whose constituent particles (atomsmolecules or ions) are arranged in a long range order of definite pattern extending throughout the solid is called a crystalline solidExNaClQ2)Define Amorphous solids AnsA solid that does not have a definite shape and a sharp melting pointand whose constituent particles (atomsmolecules or ions) are not arranged in a definite pattern is called an amorphoussolid

Crystalline solidsAmorphous solids

ExGlassRubberQ3)Classify Crystalline Solids Crystalline Solids

Physics Coloumbrsquos Law (Summary)

Before Going Into Coloumbrsquos Law We Will First Learn What is Charge Properties of Charge and Always remember that charge is quantized ie a body always have static charge of magnitude equal to some integral multiple of fundamental electronic charge e= 16 x 10- 19 C

Charge is the property of matter that causes it to produce and experience electrical and magnetic effects The study of the electrical charges at rest is called electrostatics When both electrical and magnetic effects are present the interaction between charges is referred to as electromagnetic

There exist two types of charges in nature positive and negative Like charges repel and unlike charges attract each other

The type of charge on an electron is negative The charge of a proton is the same as that of an electron but with a positive sign In an atom the number of electrons and the number of protons are equal The atom is therefore electrically neutral If one or more electrons are added to it it becomes negatively charged and is designated as negative ion However if one or more electrons are removed from an atom it becomes positively charged and is called a positive ion

The excess or deficiency of electrons in a body gives the concept of charge If there is an excess of electrons in a body it is negatively charged And if there is deficiency of electrons the body becomes positively charged Whenever addition or removal of electrons takes places the body acquires a charge

The SI Unit of charge is coulomb (C) In SI units the current is a fundamental quantity having a unit of ampere (A) The unit of charge is defined in terms of the unit of current Thus one coulomb is the charge transferred in one second across the section of a wire carrying a

Ionic SolidsMetallicSolids

Molecular Solids

current of one ampere

As q = It we have1 C = (1 A) (1 s)

The dimensions of charge are [A T]

Properties of Charge

(1) Quantization of Charge Electric charge can have only discrete values rather than any value That is charge is quantized The smallest discrete value of charge that can exist in nature is the charge on an electron given as

e = plusmn 16 x 10- 19 C

This is the charge attained by an electron and a protonA charge q must be an integral multiple of this basic unit That is

Q = plusmn ne where n = 1 2 hellip

Charge on a body can never be (frac12)e (23)e or 57e etcWhen we rub a glass rod with silk some electrons are transferred from the rod to the silk The rod becomes positively charged The silk becomes negatively charged The coulomb is a very large amount of charge A typical charge acquired by a rubbed body is 10 - 8 C

Biology Reproduction in organisms

Welcome to this new session 2020-21Today in this first chapter we mainly discuss about reproduction types needs and life span of some organismsWe also discuss about difference between sexual and asexual reproduction

Q1 What is reproductionReproduction is defined as a biological processin which an organism gives rise to young onessimilar to itselfQ2 What are the needs of reproductionbulli) Reproduction maintain life on earthii) It enables the continuity of the species generation after generationiii) It creates genetic variation among populationsQ3 Define Life span and write some orgnisms life spanbull Life span is the period from birth to

the natural death of an organism- OrganismsLife span1 Butterfly 1 - 2 weeks2 Fruit fly 30 days3Dog 10-13 years4 Rose5-7 years5 Tortoise100-150 years6 Banyan Tree -200 - 250 yearsQ4 Reproduction is of two types in case ofanimals but in case of plants vegetative propagation is also present

Asexual Reproduction Sexual Reproductioni) Always uniparentalii) Gametes are not involvediii) Only mitotic division involvediv) Somatic cells of parents are involvedv) Offsprings are genetically similar to the parents

i) Usually biparentalii) Gametes are involvediii) Meiosis occurs during gametogenesis Mitosis occurs after fertilisationiv) Germ cells of the parents are involvedv) offsprings are genetically different from the parents

COMMERCE BUSINESS ENVIRONMENT

Welcome to the new sessiontoday we are going to start the first chapter of Class XII The name of the chapter is Business Environment

Already many of you have got some idea about the word business environment form the first chapter of business studies in class XI

In todayrsquos world every business enterprise is a part of the society It exists and operates in association with various groups in society such as customers suppliers competitors banks and financial institutions government agencies trade unions media and so on All these groups influence the functioning of business in one way or the other They constitute the environment of businessConcept of Business Environment

The term lsquobusiness environmentrsquo refers to the sum total of all individuals institutions and other forces that lie outside a business enterprise but that may influence its functioning and performance

The main features of business environment Totality of External forces General and Specific forces Interrelatedness Complexity Dynamic Uncertainty

Prepare the following questions from todayrsquos assignment

2 What do you mean by business environment

The term lsquobusiness environmentrsquo means the aggregate of all forces factors and institutions which are external to and beyond the control of an individual business enterprise but they may influence its functioning and performance Business environment is the macro framework within which a business firm a micro unit operates It consists of several interrelated and interacting elements

2 Explain the main features of business environment in brief

Totality of External forces-Business environment is the sum total of all things external to a business environment

General and Specific forces-It

Relativity

The Interrelation between business and its environment

The business enterprise is an open system It continuously interacts with its environment It takes inputs (such as raw materials capital labour energy and so on) from its environment transforms them into goods and services and sends them back to the environment

Fig 1 Business Environment Relationship

includes both the forces general forces are the economic social political legal and technological conditions which indirectly influence all business enterprise Specific forces are the investors customers competitors and suppliers which influence individual enterprise directly

Interrelatedness-Different elements of environment are interrelated for an example growing awareness for health care has increased the demand for health foods

Complexity- Business environment id complex in nature as the elements keep on changing example economic technological and other forces changes in demand for a product and service

Dynamic-Business environment is not static it keeps on changing

Uncertainty- Itrsquos very difficult to predict future events such as technology and fashion which occur fast and frequently

Business Studies

Human Resources Management

Human resource of an organisation are the aggregate of knowledge skills attitudes of people working in it

The management system which deals with human resources is called human resource management

Features of HRMbullComprehensive functionbullPeople-oriented

Question1) What do you mean by human

resource management Answer) Human resource management may be defined as that field of Management which has to do with planning organising and controlling the functions of procuring developing maintaining and utilising the labour force

bullAction oriented bullPervasive function bullContinuous function

2) Explain the features of HRM in brief

Answer)bullHuman Resource Management is concerned with managing people at work bull Human Resource Management is concerned with employees which bring people and organisations together so that the goals of each are met bullHuman resource management considered every employees as an individual and also promote their satisfaction and growth bull Human resource management is inherent in all organisations and at all levelsbullManagement of human resources are ongoing on never ending process which requires a constant alertness and Awareness of human relations

3) ldquoHR function is said to be pervasiverdquowhy

Answer) Human resource management is required in all organisations whether it is private or government organisations armed forces sports organisations etc It permeatsall the functional areas like production marketing finance research etc This from this feature of human resource management it can be said that it is pervasive in nature

Economics Demand Q1DEFINITION OF DEMANDIn economics demand is the quantity of a good that consumers are willing and able to purchase at various prices during a given period of timeQ2DEMAND CURVEIn economics a demand curve is a graph depicting the relationship between the price of a certain commodity and the quantity of that commodity that is demanded at that pricQ3LAW OF DEMANDIn microeconomics the law of demand states that conditional on all else being equal as the price of a good increases quantity demanded decreases conversely as the price of a good decreases quantity demanded increasesQ4ASSUMPTION of LAW OF DEMAND(i)No change in price of related commodities(ii) No change in income of the consumer(iii) No change in taste and preferences customs habit and fashion of the consumer( No expectation regarding future change in priceQ5MARKET DEMAND SCHEDULEIn economics a market demand schedule is a tabulation of the quantity of a good that all consumers in a market will purchase at a

given price At any given price the corresponding value on the demand schedule is the sum of all consumersrsquo quantities demanded at that priceQ6INDIVIDUAL DEMAND SCHEDULEIndividual demand schedule refers to a tabular statement showing various quantities of a commodity that a consumer is willing to buy at various levels of price during a given period of timeQ7 FACTORS AFFECTING INDIVIDUAL DEMAND FOR A COMMODITY

The factors that influence a consumerrsquos decision to purchase a commodity are also known as determinants of demand The following factors affect the individual demand for a commodity1 price of the commodity2 price of related goods3 income of buyer of the commodity4 tastes and preferences of the buyer1 Price of the CommodityYou must have observed that when price of a commodity falls you tend to buy more of it and when its price rises you tend to buy less of it when all other factors remain constant (lsquoother things remaining the samersquo) In other words other things remaining the same there is an inverse relationship between the price of a commodity and its quantity demanded by its buyers This statement is in accordance with law of demand which you will study in the later part of this lesson Price of a commodity and its quantity demanded by its buyers are inversely related only when lsquoother things remain the samersquo So lsquoother things remaining the samersquo is an assumption when we study the effect of changes in the price of a commodity on its quantity demanded2 Price of Related goodsA consumer may demand a particular good But while buying that good heshe also asks the price of its related goods Related goods can be of two types-(i) Substitute goods(ii) Complementary goods While purchasing a good prices of its substitutes and complements do affect its quantity purchased(i) Price of Substitute Goods Substitute goods are those goods which can easily be used in place of one another for satisfaction of a particular want like tea and coffee An increase in price of substitute good leads to an increase in demand for the given commodity and a decrease in price of substitute good leads to a decrease in demand for the given commodity It means demand for a given commodity is directly affected by change in price of substitute goods For example if price of coffee increases the demand for tea will rise as tea will become relatively cheaper in comparison to coffee(ii) Price of Complementary goods Complementary goods are those goods which are used together to satisfy a particular want like car and petrol An increase in the price of complementary goods leads to a decrease in demand for the given commodity and a decrease in the price of complementary goods leads to an increase in demand for the given commodity For example if price of petrol falls then the demand for cars will increase as it will be relatively cheaper to use both the goods together So demand for a given commodity is inversely affected by change in price of complementary goods3 Income of the Buyer of CommodityDemand for a commodity is also affected by income of its buyer However the effect of change in income on demand depends on the nature of the commodity under consideration In case of some goods like full cream milk fine quality of rice (Basmati rice) etc demand for these commodities increases when income of the buyer increases and

demand for these commodities decreases when income of the buyer decreases Such goods whose demand increases with the increase in income of the buyer are called normal goods But there are some goods like coarse rice toned milk etc whose demand decreases when income of buyer increases and their demand increases when income of the buyer decreases Such goods whose demand decreases with the increase in income of the buyer are called inferior goods Suppose a consumer buys 10 Kgs of rice whose price is ` 25 per Kg He cannot afford to buy better quality of rice because the price of such rice is ` 50 per Kg The consumer is spending ` 250 per month on the purchase of rice Now if income of the consumer increases and he can afford ` 350 on purchase of 10 Kg of rice Now he can afford to buy some quantity of rice say 6 Kgs whose price is ` 25 per Kg and may buy 4 Kgs of rice whose price is ` 50 per Kg Thus he will buy 10 Kgs of rice by spending ` 350 per month Therefore we may conclude that demand for normal goods is directly related to the income of the buyer but demand for inferior goods is inversely related to the income of the buyer4 Tastes and Preferences of the BuyerThe demand for a commodity is also affected by the tastes and preferences of the buyers They include change in fashion customs habits etc Those commodities are preferred by the consumers which are in fashion So demand for those commodities rises which are in fashion On the other hand if a commodity goes out of the fashion its demand falls because no consumer will like to buy it(5) Number of Buyers in the Market(Population)Increase in population raises the market demand whereas decrease in population reduces the market demand for a commodity Not only the size of population but its composition like age (ratio of males females children and old people in population) also affects the demand for a commodity It is because of needs of children young old male and female population differs(6) Distribution of Income and WealthIf the distribution of income and wealth is more in favour of the rich demand for the commodities preferred by the rich such as comforts and luxuries is likely to be higher On the other hand if the distribution of income and wealth is more in favour of poor demand for commodities preferred by the poor such as necessities will be more(7) Season and Weather ConditionsThis is generally observed that the demand for woolens increases during winter whereas demand for ice creams and cold drinks increases during summer Similarly market demand for umbrellas rain coats increases during rainy seasonQ8 REASONS FOR OPERATION OF LAW OF DEMAND WHY DEMAND CURVE SLOPES DOWNWARDNow we will try to explain why does a consumer purchase more quantity of a commodity at a lower price and less of it at a higher price or why does the law of demand operate ie why does the demand curve slope downwards from left to right The main reasons for operation of law of demand are1 Law of Diminishing Marginal UtilityAs you have studied earlier law of diminishing marginal utility states that as we consume more and more units of a commodity the utility derived from each successive unit goes on decreasing The consumer will be ready to pay more for those units which provide him more utility and less for those which provide him less utility It implies that he will purchase more only when the price of the commodity falls2 Income Effect

When price of a commodity falls purchasing power or real income of the consumer increases which enables him to purchase more quantity of the commodity with the same money income Let us take an example Suppose you buy 4 ice creams when price of each ice cream is ` 25 If price of ice creams falls to ` 20 then with same money income you can buy 5 ice creams now3 Substitution EffectWhen price of a commodity falls it becomes comparatively cheaper as compared to its substitutes (although price of substitutes has not been changed) This will lead to rise in demand for the given commodity For example if coke and Pepsi both are sold at ` 10 each and price of coke falls Now coke has become relatively cheaper and will be substituted for Pepsi It will lead to rise in demand for coke4 Change in Number of BuyersWhen price of a commodity falls some old buyers may demand more of the commodity at the reduced price and some new buyers may also start buying this commodity who were not in a position to buy it earlier due to higher price This will lead to increase in number of buyers when price of the commodity falls As a result demand for the commodity rises when its price falls5 Diverse Uses of a CommoditySome commodities have diverse uses like milk It can be used for drinking for sweet preparation for ice cream preparation etc If price of milk rises its use may be restricted to important purpose only This will lead to reduction in demand for other less important uses When price of milk falls it can be put to other uses also leading to rise n demand for itQ9 EXCEPTIONS TO THE LAW OF DEMANDYou have studied in law of demand that a buyer is willing to buy more quantity of a commodity at a lower price and less of it at a higher price But in certain circumstances a rise in price may lead to rise in demand These circumstances are called Exceptions to the Law of Demand Some important exceptions are1 Giffen GoodsGiffen goods are special type of inferior goods in which negative income effect is stronger than negative substitution effect Giffen goods do not follow law of demand as their demand rises when their price rises Examples of Giffen goods are jowar and bajra etc2 Status Symbol GoodsSome goods are used by rich people as status symbols eg diamonds gold jewellary etc The higher the price the higher will be the demand for these goods When price of such goods falls these goods are no longer looked at as status symbol goods and tehrefore therir demand falls3 NecessitiesCommodities such as medicines salt wheat etc do not follow law of demandbecause we have to purchase them in minimum required quantity whatever their price may be4 Goods Expected to be ScarceWhen the buyers expect a scarcity of a particular good in near future they start buying more and more of that good even if their prices are rising For example during war famines etc people tend to buy more of some goods even at higher prices due to fear of their scarcity in near future

Political Science

Constitution of India-The

Preamble

The preamble-

Preamble-

The preamble is the most precious part of the constitution We the people of India having solemnly resolved to constitute India into a Sovereign Socialist Secular Democratic Republic and to secure to all its citizensA preamble is an introductory and expressionary statement in a document that explains the documents purpose and underlying philosophy When applied to the opening paragraphs of a statute it may recite historical facts pertinent to the subject of the statuteNature and purpose of the constitution-Purpose of the Constitution dictates permanent framework of the government to form a more perfect union to establish justice and ensure peace of thenationconstitution provide principles how the government can run itself following the rules and laws written in the constitution of each state keeps them balanced

Answer the following questions-

1 What is preambleA preamble is an introductory and expressionary statement in a document that explains the documents purpose and underlying philosophy2 What is the nature and

purpose of the constitutionConstitution dictatespermanent framework of the government to form a more perfect union to establish justice and ensure peace of the nation

Homework-Learn

Accounts Compatibilty mode

1MEANING OF PARTNERSHIPPartnership is a form of business organisation where two or more persons join hands to run a business They share the profits and losses according to the agreement amongst them According to the Indian Partnership Act 1932 ldquoPartnership is relation between persons who have agreed to share profits of a business carried on by all or any one of them acting for allrdquo For example one of your friends has passed class XII from National Institute of Open Schooling (NIOS) and wants to start a business Heshe approaches you to join in this venture Heshe wants you to contribute some money and participate in the business activities Both of you if join hands constitute a partnership2CHARACTERISTICS1048698 Agreement A partnership is formed by an agreement The agreement may be either oral or in writing It defines the relationship between the persons who agree to carry on business It may contain the terms of sharing profit and the capital to be invested by each partner etc The written agreement is known as partnership deed1048698 Number of persons There must be at least two persons to form a partnership

The maximum number of partners in a partnership firm can be 50 according toCompanies Act 20131048698 Business The Partnership is formed to carry on business with a purpose of earning profits The business should be lawful Thus if two or more persons agree to carry on unlawful activities it will not be termed as partnership1048698 Sharing Profits The partners agree to share profits in the agreed ratio In caseof loss all the partners have to bear it in the same agreed profit sharing ratio10486981048698Mutual Agency Every partner is an agent of the other partners Every partner can bind the firm and all other partners by hisher acts Each partner will be responsible and liable for the acts of all other partners10486981048698Unlimited liability The liability of each partner except that of a minor is unlimited Their liability extends to their personal assets also If the assets of the firm are insufficient to pay off its debts the partnersrsquo personal property can be used to satisfy the claim of the creditors of the partnership firm10486981048698Management All the partners have a right to mange the business However they may authorize one or more partners to manage the affairs of the business on their behalf10486981048698Transferability of Share No partner can transfer hisher share to any one including hisher family member without the consent of all other partners3PARTNERSHIP DEEDAgreement forms the basis of partnership The written form of the agreement is which a document of partnership is It contains terms and conditions regarding the conduct of the business It also explains relationship between the partners This document is called partnership deed Every firm can frame its own partnership deed in which the rights duties and liabilities of the partners are stated in detail It helps in settling the disputes arising among the partners during the general conduct of business 4CONTENTS OF PARTNERSHIP DEEDThe partnership deed generally contains the following (i) Name and address of the partnership firm(ii) Nature and objectives of the business(iii) Name and address of each partner(iv) Ratio in which profits is to be shared(v) Capital contribution by each partner(vi) Rate of Interest on capital if allowed(vii) Salary or any other remuneration to partners if allowed(viii) Rate of interest on loans and advances by a partner to the firm(ix) Drawings of partners and interest thereon if any(x) Method of valuation of goodwill and revaluation of assets and liabilities on the reconstitution of the partnership ie on the admission retirement or death of a partner(xi) Settlement of disputes by arbitration(xii) Settlement of accounts at the time of retirement or death of a partner5IN ABSENCE OF PARTNERSHIP DEEDThe partnership deed lays down the terms and conditions of partnership in regard to rights duties and obligations of the partners In the absence of partnership deed there may arise a controversy on certain issues like profit sharing ratio interest on

capital interest on drawings interest on loan and salary of the partners In such cases the provisions of the Indian Partnership Act becomes applicableSome of the Issues are(i) Distribution of Profit Partners are entitled to share profits equally(ii) Interest on Capital Interest on capital is not allowed(iii) Interest on Drawings No interest on drawing of the partners is to be charged(iv) Interest on Partnerrsquos Loan A Partner is allowed interest 6 per annum on the amount of loan given to the firm by himher(v) Salary and Commission to Partner A partner is not entitled to anysalary or commission or any other remuneration for managing the business

History TOPIC-TOWARDS INDEPENDENCE AND PARTITION THE LAST PHASE (1935-1947)

SUB TOPIC-IMPORTANT POLITICAL DEVELOPMENTS ndash GROWTH OF SOCIAL IDEAS

Socialism is a political social and economic philosophyLike in other parts of the world the Russian revolution of 1917 served as a great inspiration for revolutionaries in India who at that time were engaged in the struggle for liberation from British ruleSocialist ideas led to the formation of communist party of IndiaJAWAHARLAL NEHRU Among the early Congress leaders Jawaharlal Nehru was very much impressed and influenced by the Socialist ideas He also learnt about the Economic activities of the Soviet Union after the Bolshevic Revolution 1917 He made full use of them in IndiaThe election of Jawaharlal Nehru and Subhas Chandra Bose showed the Left wing tendency within CongressJawaharlal Nehru demanded economic freedom along with political freedom of the people in order to end the exploitation of masses

Nehrus working committee included three socialists leaders The Lucknow session was a landmark in the evolution of socialist ideas of the congressSUBHAS CHANDRA BOSE ndash Subhas Chandra Bose had socialist leaning Both Jawaharlal Nehru and Subhas Chandra Bose were known as leftist Congress men Later on National Congress divided into Leftist and rightist campCONGRESS SOCIALIST Within the Congress some leaders formed the Congress Socialist partyPattavi Sitaramyya Sardar Patel Rajendra Prasad had hostile attitude towards the Congress Socialist partyJawaharlals attitude was hesitant

1 QUESTION ndash Mention name of two Congress leaders who had socialist leaning

1ANSWER ndash Subhas Chandra Bose and Jawaharlal Nehru2QUESTION- In which session of the congress Jawaharlal elaborated his Socialist ideas2 ANSWER ndash Lucknow and Faizpur Session in December 1935 and 19363QUESTION ndash Why Congress was sharply divided into leftist and rightist camp 3ANSWER ndash Subhas Chandra Bosersquos attempt to seek re election for congress presidentship in 1939sharply divided the National Congress into Leftist and Rightist camp4 QUESTION ndash Who was MN Roy 4 ANSWER ndash Manabendra Roy first formed the Communist Party of India outside the country at Tashkent in 19205QUESTION ndash Who formed the Congress Socialist Party within the Congress5 ANSWER ndash Jaya Prakash Narayan Achyut Patwardhan Acharya Narendra Dev Ram Mohan Lohia Aruna Asaf Ali6QUESTION ndash When was the Congress Socialist Party formed What was its object6 ANSWER ndash 1934The Congress Socialist Party sought to work out socialist programme through the Congress They joined hands with the Congress and wanted to carry

Subhas Chandra Bose being expelled from the congress after the Tripuri rift he formed Forward BlockThere were basic differences between the Congress Socialists and the communistsTRADE UNION ACTIVITIES Maximum working class people lived in Bombay and Calcutta The working and living conditions of those workers were very miserable In this situation Shasipada Banerjee NM Lokhande protested against the oppression of the working class peopleThe first Trade Union Madras Labour Union was formed in 1918 by BP WadiaIndustrial strikes took place in Kanpur Calcutta Madras Jamshedpur and Ahmedabad AITUC was formed in Bombay in 1927 The growth of Trade union among the workers was slow because of the fear of the dismissal of the jobIn the mean time the Moderates as well as Communists left AITUC and formed separate organization

on National struggle with the help of workers and peasant class of the society7 QUESTION ndash What was the name of the party founded by Subhas Chandra Bose7 ANSWER- Forward Block8QUESTION ndash Who was Shasipada Banerjee8 ANSWER ndash Shasipada Banerjee was a radical Brahmo He founded a working menrsquos club to protest against exploitation of the British rulers towards the working class of India9 QUESTION ndash What was the weekly published by NM Lokhande9ANSWER- Dinabandhu10 QUESTION ndash Who founded Bombay Mill-Hands Association and in which year10 ANSWER- NM Lokhande in189011 QUESTION- Who was BP WadiaANSWER- BPWadia was the founder of Madras Labour Union in191812 QUESTION- What was the name of the first labour union of India12 ANSWER- Madras Labour Union13 QUESTION Who founded the Majur Mahajan 13 ANSWER GANDHIJI14 QUESTION What was the full form of AITUC When it was formed14 ANSWER All India Trade Union Congressin 192715QUESTION Who formed the Red Trade Union Congress and in which year15ANSWER The Communists formed the Red Trade Union Congress16 QUESTION What do you mean by Socialism16 ANSWER Socialism describes any political and economic theory that says the community rather than individuals should own and manage property and natural resources

Subject Eng Literature (The Tempest ndash William Shakespeare) Topic Act III Scene 3 Lines 1 to 52 (Line 52 ndash Brother my lord the Duke Stand to and do as we) Date 13th April 2020 (4th Period)

[Students should read the original play and also the paraphrase given in the school prescribed textbook]Summary Questions amp Answers

o Alonso Sebastian Antonio Gonzalo Adrian Francisco and others wandered about the island in search of Ferdinand and gets tired and hungry of the toil and at the same time gives up all hope of finding him

o Antonio and Sebastian are happy that Alonso is out of hope and decide to make another attempt on his life that night when being so tired they will be sleeping soundly

o Suddenly a solemn and strange music is heard in the air and several strange shapes enter bringing in a banquet These strange shapes then dance round it with gestures of salutation and then inviting the King to eat they depart

o Seeing this strange scene all are inclined to believe the tales told by travelers that there truly are ldquounicornsrdquo and ldquothe phoenixrsquo thronerdquo

1 ALONSO What harmony is this My good friends hark (L18-27)

GONZALO Marvellous sweet music

[Enter several strange shapes bringing in a banquet

they dance about it with gentle actions of salutation

and inviting the King and his companions to eat they depart]ALONSO Give us kind keepers heavens What were theseSEBASTIAN A living drollery Now I will believe

That there are unicorns that in Arabia

There is one tree the phoenixrsquo throne one phoenix

At this hour reigning thereANTONIO Ill believe both

And what does else want credit come to me

And Ill be sworn rsquotis true Travellers neer did lie

Though fools at home condemn rsquoem

(a) How did Prospero present an amazing spectacle before Alonso and his companions

Using his magic powers Prospero ordered strange shapes to lay a banquet before Alonso and his companions The shapes brought several dishes with tasty eatables in them They placed the dishes on a table before Alonso and his companions Then the strange shapes began to dance gracefully around the banquet While dancing they made gestures inviting them to eat the food Then suddenly the shapes disappeared(b) Who were the guests at the strange banquet Describe the lsquoliving drolleryrsquo

Alonso Sebastian Antonio Gonzalo Adrian and Francisco were the guests at the strange banquet

The term ldquoliving drolleryrdquo refers to live entertainment show In this context when Alonso the King of Naples Sebastian his brother Antonio the treacherous brother of Prospero Gonzalo the kind and loyal councillor to the King Adrian and Francisco came to the island they were hungry and weary in their spirits They heard a solemn and strange music They were shocked to see several strange shapes bringing in a banquet and these shapes danced about it with gentle action of salutation inviting the King and his companions to eat After this Sebastian described this show as lsquoliving drolleryrsquo(c) What is lsquophoenixrsquo What are lsquoUnicornsrdquo

The term lsquophoenixrsquo refers to a mythical Arabian bird which lived alone and perched on a solitary tree After one hundred years it expired in flames and rose again from its own ashes

lsquoUnicornsrsquo refers to the mythological four-footed beasts having horns in the centre of their foreheads When the horns are ground into powder the powder was believed to be

an aphrodisiac(d) How does Sebastian explain the puppet show OR Why does the speaker now believe in unicorns and phoenix

Sebastian finds several strange shapes bringing in the banquet They invite the king and his party for dinner and soon depart He tells that if such a strange sight can be a reality there is nothing incredible in the world and from the present moment he will believe anything He says that it is a strange dumb show enacted not by puppets but by living beings It is stranger than a travellerrsquos tale Seeing such a thing

before his own eyes he will no longer disbelieve the story about unicorns and phoenix(e) How do the other characters present respond to this living drollery

At the sight of the lsquoliving drolleryrsquo like Sebastian Gonzalo and Antonio too acted strangely Antonio told that he too now believes in unicorns and phoenix and anything else that seems to be incredible He too now believes in travellersrsquo tales Gonzalo told that if he would report those happenings in Naples nobody will believe him He considers that those gentle shapes were gentler in manner in comparison to the living beings Alonso was at first sight suspicious and told them that those strange shapes conveyed their meaning in expressive gestures when they seemed to lack speech by their movements and sounds Francisco was amazed at their mysterious disappearance

2 ALONSO Not I

(Line 43-52)GONZALO Faith sir you need not fear When we

were boysWho would believe that there were mountaineers

Dewlapped like bulls whose throats had hanging at rsquoem

Wallets of flesh Or that there were such men

Whose heads stood in their breasts Which now we find

Each putter-out of five for one will bring us

Good warrant ofALONSO I will stand to and feed

Although my lastmdashno matter since I feel

The best is past Brother my lord the Duke

Stand to and do as we

(a) How does Alonso respond at the spectacle of the shapes which were sent to them at the instruction of Prospero

After seeing the strange sight of appearing and disappearing of the shapes sent by Prospero to arrange a banquet for them Alonso says that his surprise at having seen those creatures is infinite and he is fully justified in feeling so much surprise He thinks that their shapes their gestures and the sounds they made were indeed amazing Although they do not possess the gift of speech yet they were able to convey their

thoughts by means of their gestures only

(b) What does Prospero say about the views expressed by Alonso regarding the shapes What does Francisco think about the shapesAfter hearing Alonsorsquos views about the shapes Prospero says that this manrsquos praise of the spirits is rather hasty He means to say that Alonso has shown great haste in reaching the conclusion about the shapes Francisco is amazed to see that those shapes disappeared in a mysterious way(c) What does Sebastian ask Alonso to doSebastian tells Alonso that the shapes having disappeared should not matter to them because they have left the eatables behind He asks Alonso to enjoy eating as they are extremely hungry but the king does not accept his offer of enjoying the dishes(d) How does Gonzalo try to dispel Alonsorsquos fear of those strange shapes What kind of references does he give to AlonsoGonzalo says that those who have travelled abroad have reported seeing even stranger sights than these shapes that Alonso and his companions have beheld Hence there is no reason to feel afraid of these shapes Gonzalo further adds that in his younger days he had heard strange stories from travelers and Alonso might have heard similar stories For instance it was said that there existed a certain race of

human beings who had huge lumps of flesh hanging at their throats and who therefore resembled bulls Then Gonzalo tells about a race of human beings whose heads were located at their breasts Gonzalo says that such stories were not believed by most people in those days but now-a-days these stories have become common(e) Explain the following lsquoEach putter-out of five for onersquoEnglish travellers often insured their trips with London brokers Those that went on foreign travels those days used to deposit a certain amount with some firm or company in London before their departure If the travelers failed to return the money was forfeited by the company with which it had been deposited But this money was repaid five-fold if the travelers returned safe and sound In this way a traveler stood a great chance of recovering the entire cost of his

travels(f) Give the explanatory meanings of the following expressions in the context of the above extract (i) Dewlapped (ii) Wallets of flesh

(iii) Putter-out(i) Dewlapped having big lumps of flesh at the necks(ii) Wallets of flesh large masses of flesh looking like bags(iii) Putter-out to invest money before commencing the travel

  • General methods of preparation of hydrogen
  • Chapter Dimensional Analysis (Summary)
    • Properties of Charge
Page 3:   · Web viewSubject. Topic. Summary. Execution. Hindi. व्याकरण. शरीरके अंगो के नाम लिखिए. 1) आँख 2) नाक 3

ঙ)( ইদর লিকোলির )লিgtং লেক বাোলিচসয় লিদ৪ লিবপরীত -ব2 = লে4োট োকো = ভোরী দব13 = ব কষদর = বৎ দ13োম = োম উপকোর = অপকোর

Hindi वयाकरण Chapter 2 वणमाा एव वण क भद

वण-बोत समय हमार म स धवनिनया निनकती ह इन धवनिनयो कोlsquoवणrsquoया lsquoअकषर कहत

ह वण क टकड नही निकए जा सकत ह वण भाषा की सबस छोटी इकाई ह वण क निनशचिltत करम को वण माा

कहत ह हिहदी म वणA क दो भद होत ह- सवर

और वयजन सवण वण की सखया=11 वयजन वण की सखया = 33

सभी वयजन वण म lsquoअrsquoकी धवनिन मिमी हनिबनाlsquoअrsquoधवनिन वा वण कषतरजञशरndashसयकत वयजनड ढndashनई धवनिनयाअ-अनसवार अ- निवसग

Read the lesson

Class IIISubject Topic Summary Execution

COMPUTER CHAPTER 2 OPERATING SYSTEM

DONE IN THE PREVIOUS CLASS Q3)WHAT IS THE DIFFERENCE BETWEEN MULTI-TASKING AND MULTI-THREADING OPERATING SYSTEMSAns) THE DIFFERENCE BETWEEN MULTITASKING AND MULTITHREADING OPERATING SYSTEMS IS---

MULTITASKING OPERATING SYSTEM

MULTITHREADING OPERATING SYSTEM

A MULTITASKING OPERATING SYSTEM ALLOWS TO RUN MORE THAN ONE SOFTWARE PROCESSES AT THE SAME TIME

MULTITHREADING OPERATING SYSTEM ALLOWS DIFFERENT PARTS OF A SINGLE PROGRAM TO RUN AT THE SAME TIME

Q4) WHAT IS A GUI OPERATING SYSTEMAns) A GUI OPERATING SYSTEM ALLOWS USERS TO INTERACT USING A COMPUTER MOUSE OR ANY OTHER POINTING DEVICE FOR EXAMPLE MICROSOFT WINDOWS 7

ENGLISH I CHAPTER 4 ARTICLES

refer to pg 15 do in the rough copy and practice the whole chapter

Continuation pg 15 ex DANSWERS10You are the tallest person in the class11The sun rises in the east12An honest person always speaks the truth13The peacock is the national bird of India14The Nile is the longest river in the world15We have a beautiful garden The garden is full of roses16He is one of the best authors of the world17My brother lives in an apartment in the centre of the city18He quickly ate the cookies that was meant for the guests

SST INDIA-THE LAND OF FESTIVALS

Bihu it is a harvest festival celebrated in Assam Magh Bihu is celebrated every year in the month of January People feast sing and have fun

1In which state Bihu festival is celebratedAns Assam

2In which month Magh Bihu is celebrated

Pongal Itrsquos a four day long festival celebrated in Tamil Nadu It is celebrated on January 14 every year People cook sweet rice called Pongal

Ans January

3What is PongalAns Pongal is a harvest festival which is celebrated in Tamil Nadu It is a four day long festival

4In which month Pongal is celebratedAns Pongal is celebrated on January 14 every year

SCIENCE SOME PROPERTIES OF WATER

We get water from the nature so water is a natural resource Water is the most commonly found liquid on earth All living things need water to live Life on earth is not possible without water We need water for drinking cooking bathing washing etc

Fill in the blanks [pg no ndash 81]1We get fresh water from rain 2Boiling kills the germs present in water 3 Water changes its state when it is heated and cooled 4Water does not have a definite shape

BENGALI বই ndash বোংো োলিতয পলিরচয়

পোঠ ndash ১ একতোই বঅীীর পরসোততর

১ঠিক উততসর টিক দোও-ক) বস2ো লেোকটির ( লিত চোর পাোচ ) লে4স ) তোরো ব ময় ( কা োসদ োফোয় ঝগ2ো কসর ) গ) বস2ো লেোকটি লে4সসদর বস ( কমস2ো বাো কলিG ) আসত ঘ) বস2ো তোর লে4সসদর ( োটক মযোলিক লিসমো ) লেদোসব বসলি4সো ২ ক) লেোকটির কয়টি লে4স লি4 উঃ পাোচটি লে4স লি4 ) বস2ো লেোকটো লে4সসদর পরসম লিক আসত ব উঃ কলিG আসত বসোগ) বোলিJ বাো3োর পর তোসদর লিক করসত ব উঃ এক-এক কসর বোলিJটি ভোঙোর লেচষটো করসত ব ঘ) লে4সরো লিমসলিমস োকস লিক সব উঃ লে4সরো লিমসলিমস োকস বোইসরর লেোক

HINDI वयाकरण chapter 1 भाषा

BOOK PAGE 10 राजय भाषामहाराषटर मराठी गजरात गजराती पजाब पजाबी बगा बागा लिनिप भाषा दवनागरी-- हिहदीमराठी गजराती रोमन -- अगरजी गरमी -- पजाबी फारसी -- उरद

सही या गत निनशान गाइए-1पतर मौखिक भाषा का एक रप ह( )X 2भाषा क लिखित रप को लिनिप कहत ह( )3 ससार म सभी ोग एक ही भाषा बोत ह(X) 4गजरात म पजाबी बोी जाती ह(X) भाषा क दो

MATHEMATICS CHAPTER 4 SUBTRACTION

Subtraction without DecomposingSubtraction of 3-digit number (without decomposing)

The result obtained after subtraction is called difference

Note While finding the difference we write the greater of the two given numbers on the top and then subtract

Exercise 16Subtract9 409 10 876 11 641 - 103 - 443 - 510 306 433 131

12 829 - 113 716

Write in columns and find the difference14 603 ndash 401 15 840 ndash 610

Solution

14 H T O 15 H T O 6 0 3 8 4 0 - 4 0 1 - 6 1 0 2 0 2 2 3 0

Fill in the place holders17 _ 3 7 18 6 _ 5

- 2 - 6 - 2 4 3 7 2 -- -- 4 ndash

Solution

179 3 7 18 6 8 5 - 2 1 6 - 2 4 3 7 2 1 4 4 2

Class IVSubject Topic Summary Execution

English language

Personal pronouns

The different forms of the three personal pronounsFirst personSingular I me my minePlural we us our oursSecond person Singular thou thee thy thinePlural ye you your yoursThird personSingular he him his her hers she it itsPlural they them theirs their

Pick out the personal pronouns Mention of what kind each (first second or third person)1 You should always try to keep a promise that you have madeYou second person personal pronoun

2 I listened patiently to what he had to say about themI first person personal pronounHe them third person personal pronoun

3 They packed their belongings and left I do not know where they have goneThey their third person personal pronounI first person personal pronoun

Social studies Major landforms on earth

Explained in previous lesson Answer the following questions3 Differentiate between hills and mountainsAns Mountains has a peak whereas hills have rounded topsMountains are steeper than hills whereas hills are lower and less steep4 Give two usefulness of mountainsAns Mountains are storehouse of waterMountains have a rich variety of flora and fauna

MATHEMATICS

Ch 5Subtraction

Estimating the differenceWe have learnt estimation in addition Applying the same rule We estimate the difference

Exercise 15 Example 2

A fruit-seller has 38210 bananas Out of these he sold 21799 bananas Estimate the unsold bananas

Solution Estimating to ten-thousandsTotal bananas = 40000Bananas sold out =- 20000Bananas unsold = 20000

Exercise1 Find the actual and estimated difference by rounding off to the nearest thousands(b) ActualEstimated

16190 - 2979

Solution (b) ActualEstimated 16190 16000 -2979 -30001321113000

2 Find the actual and estimated difference by rounding off to the nearest ten-thousands(b) ActualEstimated 73012 -28790

Solution (b) ActualEstimated 73012 70000 -28790-3000044222 40000

4 A bakery shop has an order to supply 6705 packets of biscuits There are only 4920 packets in the shop Estimate the number of packets needed to complete the supply

Solution Estimating to thousandsEstimated order = 7000Packets available= -5000Packets needed = 2000

there4 Estimated packets needed to complete the order were 2000

Hindi 2ndlang पतर पतर लिना भी एक का ह पतर लित समय धयान दना होगा निक पतर निकस लि रह ह कयोनिक पतर या तो वयवहारिरक होगा या वयावहारिरक होगापतर दो परकार क होत ह

क) औपचारिरक पतर जस परधानाचाय को नगर निनगम सपादक आदिद

ख) अनौपचारिरक पतर जस पापा को दोसत को बहन को अपन परिरजनो को

1 औपचारिरक पतर-अपन निवदयाय म अवकाश क लिए परधानाचाय को पतर लिखिए १०३कसीबीदम दमक टकोकाता-७०००६५सवा मपरधानाचायऋनिष अरहिवदो ममोरिरय एकडमीदिदनाक-१३४२०२०निवषय-अवकाश हत पतरमहोदया जीसनिवनय निनवदन ह निक म आपकी ककषा चौथी का निवदयाथj ह क रात स मझ तज बार ह डॉकटर न मझ आराम करन को कहा हअतः म निवदयाय आन म असमथ ह आपस निनवदन ह निक आप मझ दिदनाक १३स १७ तक की अवकाश दन की कपा कर हम आप क आभारी रहग

धनयवादआपका आजञाकारी लिशषयसीमा सिसहककषा-४

Bengali বইndashবোংো ভোষো পলিরচয়

পোঠndash৮ লিঙগ ৩ পংলিঙগ কোসক বসউঃ পরোলিবোচক লেP লিবসষয পসদর দবোরো পরষ োতীয় কোউসক লেবোঝোয় তোসক পংলিঙগ বস লেPম -বোবো 4োতর৪ সতরীলিঙগ কোসক বসউঃ পরোলিবোচক লেP লিবসষয পসদর তোরো সতরী োতীয় কোউসক লেবোঝোয় তোসক সতরীলিঙগ বস লেPম - মো লিলিকষকো৫ উভয়লিঙগ কোসক বস উঃ পরোলিবোচক লেPলিবসষয পসদর দবোরোপরষ ও সতরীউভয়োলিতসক লেবোঝোয় তোসক উভয়লিঙগ বস লেPম - লিশ মনতরী ৬ কলীবলিঙগ কোসক বস উঃ অপরোলিবোচক লেP লিবসষয পসদর দবোরো সতরী বো পরষ কোউসক ো বলিঝসয় লেকো2 পদো13 লেক লেবোঝোয়তোসক কলীবলিঙগ বস লেPম - বই

COMPUTER CHAPTER 3 EDITING IN MS WORD

MOVING THE TEXTMS WORD ALLOWS US TO MOVE A BLOCK OF TEXT OR GRAPHICS WITHIN A DOCUMENT

Q6) HOW TO WE USE THE COPY AND PASTE OPTIONAns) WE CAN CREATE A DUPLICATE COPY OF THE TEXTTHE STEPS TO COPY AND PASTE THE TEXT AS FOLLOWS----

SELECT THE PART OF THE TEXT THAT WE WANT TO COPY

SELECT THE COPY BUTTON IN THE CLIPBOARD GROUP UNDER THE HOME TAB OR PRESS CTRL + C KEYS

NOW PLACE THE CURSOR AT THE PLACE WHERE THE TEXT NEEDS TO BE COPIED

CLICK THE PASTE BUTTON IN THE CLIPBOARD GROUP UNDER THE HOME TAB OR PRESS CTRL + V KEYS

Science Adaptations in Animals

We find different kinds of animals in our surroundings We find birds flying in the sky monkeys jumping on trees fish in water camels in desert and so on Animals live in their natural homes which are called their habitats The habitat of a camel is a dry sandy desert and that of a tiger is a dense forest So different animals live in different habitats In the world a plant or an animal has to adapt or change itself to suits its surroundings This changes happens over hundreds and thousands of years A change that a living thing undergoes to become better suited to its surrounding is called adaptation

Fill in the blanks [pg no 40]1 Terrestrial animals live on

land 2 Polar bears are found in the

cold polar regions 3 Frogs have webbed feet that

help them to swim 4 Monkeys and koalas are

arboreal animals

Class VSubject Topic Summary Execution

COMPUTER130420

CHAPTER 2 APPLICATIONS OF COMPUTERS

PAGE NO-15C FILL IN THE BLANKS

INPUTS ANIMATION BANKS SOFTWARE SUPER COMPUTERS

D WRITE TRUE OR FALSE TRUE TRUE TRUE TRUE FALSE

Science Chapter 2 - The Skeletal System

Joints A joint is a place where two or more bones are joined with each other

There are two types of joints 1) Fixed joints 2) Movable joints

The movable joints are mainly four types ndash 1) Hinge joint 2) Pivot joint 3) Gliding joint 4) Ball and socket joint

DAnswer these questions

1)What are the functions of the skeleton

Ans ndash The functions of skeleton are -

i The skeleton gives our body shape ii The skeleton provide our body strength iii The skeleton protect our inner organs from outside injuries iv The skeleton gives our body support

2)List the type of movable joint in our body giving one example of each

Ans - The movable joints are mainly four types ndash a Hinge joint

Example of hinge joints are elbows nice fingers and toes

b Pivot joint Example - the joint between the skull and the backbone is the example of pivot joint

c Gliding joint Example of gliding joints

are wrist and ankle joints d Ball and socket joint

Example of ball and socket joints are shoulder joint and hip joint

English language

Transitive and intransitive verb

Pick out the verbs from the following sentences and say whether they are transitive or intransitive (page 21 ex B)1 gave ndash verb Transitive verb2 Has planted- verb Intransitive verb3 Were- verb Intransitive verb4 Asked- verb Transitive verb5 Told- verb Transitive verb

Social studies Conquering distances

Airways The only airline owned by the government is Air India which handles both domestic and international flightsAdvantages of airways Air transport is the fastestIt can access remote areasIt is the best means of transport in case of emergencies

1 Which is the only airline owned by the governmentAns Air India

2 Give two advantages of airwaysAns The two advantages of airways areAir transport is the fastest transportIt can access remote areas

DisadvantageIt is the most expensive of all other means of transport

3 What is the disadvantage of airwaysAns The only disadvantage of airways is that it is the most expensive of all other means of transport

MATHEMATICS

Ch 3Addition and Subtraction

Properties of Addition1 The sum of two numbers does not change when we change their order This property known as Commutative Property of addition

2 The sum of three numbers does not change when we change their grouping This property is called Associative Property of addition

3 The sum of the numbers and zero is the number itself This property is called Identity Property of addition and the integer 0 is called identity

Exercise 11Fill in the blanks1 2730815 + 8319293 = ____ + 27308152 18219 + 1850308 = 1850308 + ____3 (27815 + 85919) + 95985 = (85919 + ____) + 278154 13227 + (25983 + 73607) = (____ + 25983) + 736075 91389 + 0 = ____ + 91389

Solution 1 83192932 182193 959854 132275 0

6 Which of the following are true statements(a) Any number added to zero is zero

(b) The sum of two numbers does not change when we change their order

(c) 1 is the identity element of addition

(d) Given any three numbers their sum does not change when we change their grouping

Solution (a) False(b) True(c) False(d) True

Class VISubject Topic Summary Execution

HISTORY AND CIVICS

CHAPTER 3

MAHAVIRA AND BUDDHA ndash GREAT PREACHERS BUDDHA

Decline of Buddhism1 Revival of the Brahmanical Hinduism ndash Brahmin Scholars like Shankaracharya and Kumarila Bhatta led the revival of Hinduism and established the supremacy of Vedic religion2 Loss of Royal Patronage ndash Gupta period marked the decline of Buddhism as Gupta rulers were followers of Hinduism3 Split in Buddhism ndash division into Hinayana and Mahayana sects and rise of Mahayana sects blurred the line between Hinduism and Buddhism4 Corruption in Buddhist Sangha ndash due to generation of large revenue from large estates Buddhist monks and nuns started living luxurious

Answer the Following 1 During which dynasty Buddhism was split During the reign of Kanishka

2 Name the two sects of Buddhism Mahayana and Hinayana

3 Name two Vedic scholars who led the revival of Brahmanical Hinduism Shankaracharya and Kumarila Bhatta

life in rich monasteries Hence corruption crept in5 Adoption of Sanskrit ndash when Buddhist scriptures began to be written in Sanskrit in place of peoples language like Pali or Prakrit people started drifting away from Buddhism6 The Turkish Invasion ndash As Muslim conquerors invaded India immensely wealthy Buddhist Monasteries and temples were looted and destroyed and Buddhists were persecuted and killed

4 During which period decline of Buddhism began The Gupta Period

ENGLISH 2 The great train journey- Ruskin Bond

The great journey by Ruskin Bond is a story about Suraj who loved trains and wanted to go to places One day while wandering along the railway tracks he enters into a carriage compartment The train suddenly starts moving with him in the compartment and after a journey returns back to the same place from where it had begun The story is about his experience during that journey

4 Answer the following questionsf Who else is in the carriageA ragged hippy with a dirty beard face was in the carriageg Where does Suraj say that he would like to go toSuraj said that he would like to go to England and China and Africa and Greenland He wanted to go all over the worldh What warning does the man give to SurajThe man said Suraj to keep out of sight so that he doesnrsquot get caught by the ticket collectorsiWhen Suraj thinks about his parents for the first time what does he imagines that they will thinkSuraj thought that if he failed to come home that night his parents would think that he had run away or been kidnapped or been involved in an accidentJ What presents does Suraj imagine that he will bring back for his friendSuraj imagines that he would bring an African lion or a transistor- radio for his friend

CHEMISTRY

Chapter 2 ndashElement and Compound

ATOMAn atom can be defined as the smallest constituent particle of an element which showcases independent existence Example Ne OMOLECULEA molecule can be defined as the combinations of two or more atoms which are held together by chemical bonds A molecule is the smallest portion of a substance which showcases all the properties of the substance On breaking down a molecule further we see properties of the constituent elements Example HCl NaCl O2

Answer the following Q3) What is a moleculeAns - A molecule can be defined as the combinations of two or more atoms which are held together by chemical bonds A molecule is the smallest portion of a substance which showcases all the properties of the substance On breaking down a molecule further we see properties of the constituent elements Example HCl NaCl O2

Q4) Which can exist independently ndash atom or moleculesAns ndash Molecules can exist independently

PHYSICS Physical quantities

Guidelines for writing SI units correctly1 The units named after scientists are not written with a capital initial letter For example newton henry watt2 The symbols of the units named after scientist should be written by a capital letter For example N for newton H for henry W for watt3 Small letters are used as symbols for units not derived from a proper name For example m for metre kg for kilogram4 No full stop or other punctuation marks should be used within or at the end of symbols For example 50 m and not as 50 m5 The symbols of the units do not take plural form For example 10 kg not as 10 kgs6 When temperature is expressed in kelvin the degree sign is omitted For example 273 K not as 273o K (If expressed in Celsius scale degree sign is to be included For example 100o C and not 100 C)7 Use of solidus is recommended only for indicating a division of one letter unit symbol by another unit

Fill in the blanks

1) Length and mass are examples of fundamental physical quantities

2) The measurement of a physical quantity consists of two part magnitude and unit

3) A foot consist of 32 inches 4) The unit of temperature in the SI system is

Kelvin

Write true or false Correct the false statements

1) In ancient times cubit was used to measure the mass of an object FalseCorrect statement ndash In ancient times cubit was used to measure the length of an object

2) There are 7 fundamental physical quantities True

symbol Not more than one solidus is used For example m s-1 or m s J K mol or J K-1 mol-1 but not J K mol8 Some space is always to be left between the number and the symbol of the unit and also between the symbols for compound units such as force momentum etc For example it is not correct to write 23m The correct representation is 23 m kg m s-2 and not as kgms-29 Only accepted symbols should be used For example ampere is represented as A and not as amp or am second is represented as s and not as sec10 Numerical value of any physical quantity should be expressed in scientific notationFor an example density of mercury is 136 x 104 kg m-3 and not as 13600 kg m-3

3) Second is the unit of time in both the CGS and MKS systems True

4) The symbol used for a unit is always written in capital letters False Correct statement -The symbol used for a unit is normally written in small letters

Hindi 2nd language

वाकय निवचार भागवत निवचारो को परकट करन वा साथक एव वयवसथिtत शबद समह को वाकय कहत ह वाकय दो परकार क होत ह ndash

1 उददशय- वाकय म जिजसक बार म कछ बताया जाता ह उस उददशय कहत ह जस राधा एक नतकी ह2 निवधय- वाकय म उददशय क बार म बताया जाता ह उस निवधयक कहत ह जस- राधा एक नतकी ह रचना क आधार पर वाकय क तीन भद होत ह ndash१सर वाकय- राम बाजार गया २ सयकत वाकय- राम बाजार गया और वहा जाकर दोसत स मिमा३ मिमशर वाकय- यह वही tान ह जहा उनका बचपन बीता

helliphellipContinue to nextBengali 2nd language

লিZ সবরপ ও সবরলিZ

সবরলিZর লিয়ম - ১ অ-কোর লিকংবো আ-কোসরর পসর অ-কোর লিকংবো আ - কোর োকস উভয় লিমস আ ndashকোর য় এবং ওই আ ndash কোর পব13বস13 Pকত য়

২ ই - কোর লিকংবো ঈ - কোসরর পসর ই - কোর লিকংবো ঈ - কোর োকস উভয় লিমস ঈ - কোর য় এবং ওই ঈ - কোর পব13বস13 Pকত য়

৩ উ - কোর লিকংবো ঊ - কোসরর পসর উ - কোর লিকংবো ঊ - কোর োকস উভয় লিমস ঊ - কোর য় এবং ওই ঊ - কোর পব13বস13 Pকত য়

৪ অ - কোর লিকংবো আ ndash কোসরর পসর ই - কোর লিকংবো ঈ - কোর োকস উভয় লিমস এ - কোর য় এবং ওই এ - কোর পব13বস13 Pকত য়

১ অ + অ = আ ( gtো ) লিম + অচ = লিমোচ সব + অ3ী = সবো3ী অ + আ = আ ( gtো )পদম + আ = পদমো শভ + আলি = শভোলি আ + আ = আ ( gtো )4োয়ো + আবত = 4োয়োবত মো + আতমো = মোতমো আ + অ = আ ( gtো )লিবদযো + অংকোর = লিবদযোংকোর Pো + অ13 = Pো13 ২ ই + ই = ঈ ( gtী )অলিত + ইব = অতীব লিগলির + ইনদর = লিগরীনদর ই + ঈ = ঈ ( gtী )পলির + ইকষো = পরীকষো অলি3 + ঈশবর = অ3ীশবর ঈ + ঈ = ঈ ( gtী )মী + ঈশবর = মীশবর 3ী + ঈ = 3ী ঈ + ই = ঈ ( gtী )রী + ইনদর = রীনদর মী + ইনদর = মীনদর ৩ উ + উ = ঊ ( gt )মর + উদযো = মরদযো কট + উলিকত = কটলিকত উ + ঊ = ঊ ( gt )ঘ + ঊলিম13 = ঘলিম13 লিZ + ঊলিম13 = লিZলিম13 ঊ + ঊ = ঊ ( gt )রP + ঊলিম13 = রPলিম13

৪ অ + ই = এ ( লেgt )র + ইনদর = সরনদর লেPোগ + ইনদর = লেPোসগনদর অ + ঈ = এ ( লেgt )গ + ঈ = গস

র + ঈ = সর আ + ই = এ ( লেgt )Pো + ইষট = Pসষট 3ো + ইনদ = স3নদ আ + ঈ = এ ( লেgt )রমো + ঈ = রসম দবোরকো + ঈশবর = দবোরসকশবর

COMPUTER THE WORLD OF WINDOWS 10

DONE IN THE PREVIOUS CLASS PAGE NO-83A TICK THE CORRECT OPTION BACKGROUND DISPLAY AREA RESTORE THREE

MATHEMATICS Topic ndash NumbersChapter ndash Natural numbers and whole numbers

Study item Properties of whole numbers for subtraction1) Closure property When we do subtraction of two whole numbers we can not get a whole number in all time Example 8 ndash 3 = 5 a whole number 0 ndash 6 = -6 is not a whole numberTherefore the subtraction of two whole numbers is not satisfying closure property2) Commutative property If x and y are two whole numbers then x ndash y is not equal to y ndash xExample If x=16 and y = 7 then x ndash y = 16 ndash 7 = 9Again y ndash x = 7 ndash 16 = - 9 Therefore x ndash y not equal to y ndash x Therefore the subtraction of two whole numbers is not satisfy commutative3) Associative property If x y and z are three whole numbersThen x ndash ( y ndash z ) not equal to ( x ndash y ) ndash z Example If x = 20 y = 10 and z = 6Therefore x ndash (y ndash z ) = 20 ndash(10 ndash 6 ) = 20 ndash 4 = 16(X ndash y ) ndash z = (20 ndash 10) ndash 4 = 10 -4 =6Therefore x ndash(y ndash z) not equal to ( x ndash y) ndash zTherefore subtraction of whole numbers is not satisfying associativity4) Distributive property If x y and z are three whole numbersThen x (y ndash z ) = xy ndash xzAnd (y ndash z)x = yx ndash zxExample If x = 10 y = 6 and z = 4x(y ndash z ) = 10(6 ndash 4 ) = 10times6 ndash 10times4 = 60 ndash 40 = 20( 6 ndash 4 )times 10 = 6times10 ndash 4times10 = 60 ndash 40 = 20Therefore the subtraction of whole numbers is satisfying distributive property5) Existence of identity For any whole number x X ndash 0 = x but 0 ndash x = - x not equal to xThus for subtraction no identity number existsException 0 ndash 0 = 0 so 0 is its own identity for subtraction

Class VIISubject Topic Summary Execution

Hindi 2ndlang वचन जो सजञा शबद निकसी वसत या पराणी क एक या अनक होन का बोध कराया उनह वचन कहत ह जस डका- डकयह दो परकार की होती ह-

क) एकवचन-शबद क जिजस रप स उसक एक होन का बोध हो उस एक वचन कहत ह जस निकताब गमा आदिद

) बहवचन-शबद क जिजस रप स उसक आन ोन का पता च उस बहवचन कहत ह जस डक निकताब निततलियाआदिद

निनमनलिखित शबदो को एकवचन स बहवचन म बदोम- हमजानित- जानितयानारी- नारिरयामिमतर ndashमिमतरोपसतक -पसतकसडक-सडकबोत-बोतनाहर-नहररपए-रपया

Bengali বইndashবোংো োলিতয পলিরচয়

পোঠndash১৪ গলপ - অপর কলপো পর

লেক - লিবভলিতভষ বসনদযোপো3যোয়লেকndash রবীনদর পরবতf বোংো কোোলিতয 3োরোর উসgসPোগয োম পরকলিতসপরমী লিবভলিতভষ বসনদযোপো3যোয় তোর লেীসত লেPম বোসর বোসর লিফসর এসস4 গরোম বোংোর পরকলিতর কো লেতমলি এসস4 গরোমী মো লিচতরগলপndash অপর কলপো গলপোংটি লিবভলিতভষ বসনদযোপো3যোসয়র লিবযোত উপযো পসর পাোচোী লেসক গীত অপ অ13োৎ পসর পাোচোী তো অপর কলপোর লেকনদরীয় চলিরতর এই অংস আমরো পোই বোক অপসক বোক অপ কলপো লিবোী লে দসরর অ গো4 লেদস মোসয়র মস লেোো রপকোর রোসয পোলি2 লেদয় দপরসবো মোসয়র মসর কসর কোীদোী মোভোরত এর করসকষতর Pসjর ব13ো শস তোর মোবীর কস13র পরলিত ব2 মমতো য় আবোরপালিসত বলি13ত Pসjর অমোপত অং লে লিসই মোপত কসর বোলি2র লিপ4স বাো বোগোস লিকংবো উঠোসর লিশমসর কলপো লিবো এোস পরকো লেপসয়স4

১ অপর কলপো গসলপর লেক লেক তোর মপসক13 লেসো২ অপর কলপো গলপটি লেকোো লেসক গীত গলপটির ম ভোব লেসো

GEOGRAPHY CHAPTER 7EUROPE

CHAPTER COMPLETE EXERCISEFill in the blanks1 Europe is a continent that comprises the western part of Eurasia2 Eurasia and Africa are connected into one large land mass known as Afroeurasia3 The Strait of Gibraltar separates Europe and Africa4 Europe is surrounded by the Arctic Ocean to the north5 The British Isles includes the island countries of Great Britain and Ireland

Name the following 1 Connects Africa to Eurasia - Isthmus of Suez2 Largest country in the world in terms of area ndash Russia3 A term used collectively for the five countries in northern Europe ndash Nordic Countries4 The capital of Montenegro - Podgorica5 the largest fjord in Norway ndash Sognefjord

Match the following Column 1 Column 2a Albania iii Tiranab Belgium i Brusselsc Denmark v Copenhagend Finland ii Helsinkie Hungary iv Budapest

CHEMISTRY Chapter 2 ndashElement and Compound

Atom - An atom is the basic unit of an element or the smallest particle of an element non capable of independent existence Atom is built up of three sub atomic particles electron proton and neutron

Nucleus-It is the centre of an atom In the centre of the atom contains proton (positively charged particles ) and neutrons ( particles carrying no charge )

Orbits- It surround the nucleus in which revolve electrons (negatively charged particles)

Answer the following

1) What are MetalloidsAns - Certain elements using properties of both metal and non-metals are called metalloids Example Silicon arsenic and antimony

2) What are Noble gasesAns - Certain elements are present in the air and are chemically inert or unreactive Such elements are called rare gases or noble gases Example helium neon argon and Krypton

English 2 Sentences based on meanings

Kinds of sentences

Assertive or declarative to convey information or simply make a statement

Interrogative to ask different types of questions

Imperative to command or instruct someone or make a request

Exclamatory to express strong feelings and emotions

Exercise B1 Stop it ( Exclamatory)2 May you always be happy

together ( Exclamatory)3 He does not like sports

( Assertive)4 Please pass me the salt

( Imperative)5 How dare she talk to me like

that ( Exclamatory)6 May success bless your effort

( Exclamatory)7 Canrsquot you wait for sometime

(Interrogative)8 Did anybody tell you about it

( Interrogative)9 I saw her waiting for the bus

( Assertive)10 Could you please take a

message for me ( Interrogative)

Homework Ex ABiology Chp -2

Classification of Plants

Today we discuss about usefulness of bacteria We also discuss what the harmful effects of bacteria are

89 How bacteria are useful for usbull Bacteria is helpful in many ways forhuman being i) Production of medicine - antibiotics vaccine etcii) Formation of curd by lactobacillusiii)Nitrogen fixation in Leguminousplant by Rhizobiumiv) Increase soil fertility by absorbingatmospheric nitrogen and convert it into nitrates and nitritesv) Cleaning the environment by converting the complex substances into simple substancesvi) Tanning of leathervii) Retting of Fibersviii) Formation of compost by acting onanimal dung and agricultual cases1x) Biogas production by decomposingplant and animal wastex)Help In Nutrition by producing vitamiacutemBand kx1) Some bacteria are used to give specialflavour to tea coffee and coccaQ10- Name some diseases and there causativebacteriabull Diseasescausative bacteria1 CholeraVibrio cholerae2 Tuberculosis - Mycobacterium tuberculosis3 Diptheria -Corynebacteriumdiphtheriae4 Pneumonia - Streptococcus pneumoniae

Math Number system

Chapter Fraction

Study item Using lsquoofrsquoThe word lsquoofrsquo between any two fractions is to be used as multiplicationExample 57 of 56 = 57 times 56 = 5times8 = 40Study item Using BODMASThe word lsquoBODMASrsquo is the abbreviation formed by taking the initial letters of six operations(i)Bracket (ii) of (iii) Division (iv) Multiplication (v) Addition (vi) SubtractionAccording to BODMAS rule First of all the terms inside Bracket must be simplified then lsquoofrsquo lsquoDivisionrsquo lsquoMultiplicationrsquo lsquoAdditionrsquo lsquosubtractionrsquo

Study item Removal of Brackets

There are four Brackets of algebra in Mathematics In a complex expression four types of brackets are used Order of removing the brackets is first ----- then ( ) then finally [ ]

Class VIIISubject Topic Summary Execution

Chemistry Hydrogen General methods of preparation of hydrogen

By the action of dilute acids on metals

Calcium Reacts readily to form chloride salt and hydrogen

Ca + 2HCl rarr CaCl₂ + H₂uarr

Magnesium

Aluminium

Zinc

React readily to form salt and hydrogen

Mg + 2HCl rarr MgCl₂ + H₂uarr2Al + 6HCl rarr 2AlCl₃ + 3H₂uarrZn + 2HCl rarr ZnCl₂ + H₂uarr

Question 4 ) Give reasons for the following

(a) Hydrogen be used as a fuel

Solution

Hydrogen is used as a fuel because it has a high heat of combustion Some significant fuels are coal gas water gas and liquid hydrogen

(b) Though hydrogen is lighter than air it cannot be collected by downward displacement of air

Solution

Hydrogen is lighter than air so it is possible to collect the gas by downward displacement of air But it is not safe to do so since a mixture of hydrogen and air can lead to an explosion

(c) A pop sound produced when hydrogen is burnt

Solution

Impure hydrogen gas burns in air with a pop sound This is because of the presence of impurities in it

(d) Helium replaced hydrogen in weather observation balloons

Solution

It forms a mixture with air that can explode when there is a small leakage of hydrogen in a balloon So helium has replaced hydrogen

(e) Nitric acid not used for the preparation of hydrogen gas

Solution

(e) By the action of nitric acid on metals hydrogen cannot be produced because it also releases nitrous oxide and nitric oxide and oxides the hydrogen to form water

Biology Chp-2 Reproduction in plants

Today we discuss different methods of artificial propagation like cutting-rose sugercane Layering ndashguava lemon china rose etc Grafting- mango apple etcMicropropagation ndashorchid asparagus etcWe also discuss about advantages and disadvantages of vegetative propagation

Q7 Define the following terms i) Explant In tissue culture techniquea tiny piece of bud shoot or any other partof plant from where new tissue develop ii) Callus The cells of the tissue divide andgrow into a mass of undifferentiated cells from explant iii) Plantlet After few days callus differentiate into a small plant with roots and shootQ8 what are the advantages and limitations of tissue culture or micropropagation

Advantages i ) It produacuteces superior quality plantsii)It can be applied to interspecifie hybridsiii) It is useful to grow seedless plants bull Limitations i) It cannot be used for all plantsii)It is not easy to handleQ9 Write advantages of vegetative propagationi) It is a quick and easy method ofproducing new plantsii) This method need less time to matureiii) The new plants are exact copies of the parentiv) it is extremly useful for growing seedlessplants like banana grapes etc Q10 Write some disadvantages of vegetativepropagationi) Dišeases present in the parent plant gettransferred to all in new plantsii) Overcrowding of new plants causes competition for sunlight water and nutrients which affects growth of plantsplant

Physics Chapter 2 Physical Quatites and Measurements

Here We Will Do Some QuestionsRelated To Chapter 2

Select the correct alternative A block of wood of density 08gcm-3 has a volume of 60cm3 The mass of the block is

1 608 g

2 75 g

3 48 g

4 0013 g

Solution 348 g

The density of aluminium is 27g and that of brass The correct statement is

1 Equal masses of aluminium and brass have equal volumes

2 The mass of a certain volume of brass is more than the mass of an equal volume of aluminium

3 The volume of a certain mass of brass is more than the volume of an equal mass of aluminium

4 Equal volumes of aluminium and brass have equal masses

Solution 2 The mass of a certain volume of brass is more than the mass of an equal volume of aluminium

MATHEMATICS Ch 6Sets

Exercise 6(C)1 Find all the subset of each the following sets(i) A = 57 (iii) C = x xisin W x le 2(iv) p p is a letter in the word lsquopoorrsquo

Solution (i) All the subsets of A are ϕ 5 7 57

(iii) All the subsets of C are ϕ 0 1 2 01 02 12 012

(iv) All the subsets are ϕ p o r po or por

4 Given the universal set = -7-3-105689 find (i) A = x xlt2 (ii) B = x -4ltxlt6 Solution

(i) A = -7-3-10(ii) B = -3-105

5 Given the universal set = x xisin N and xlt20 find

(i) A = x x = 3p pisin N (iii) C = x x is divisible by 4 Solution

(i) 369121518 (iii) 481216

6 Find the proper subset of x x2-9x-10 = 0 Solution

ϕ 10 -1

Working x2-9x-10 = 0 rArr x2-(10-1)x-10 = 0

rArr x2-10x+x-10 = 0 rArrx(x-10)+1(x-10) = 0

rArr (x+1) (x-10) = 0

11 Let M = letters of the word REAL and N = letters of the word LARE Write sets M and N in roster form and then state whether (i) M sube N is true (ii) N sube M is true (iii) M = N is true

Solution M = real and N = lareSo (i) Yes (ii) Yes (iii) Yes

English 2 Twelfth Night ndash Shakespeare

A noble man named Orsino in the kingdom of Illyria is deeply in love with a lady called lady Olivia She is in mourning for her dead brother so she will not even think about marriage At this time a sea storm causes a terrible shipwreck and a young lady called Viola is swept onto the shore She thinks that her twin brother Sebastian is drowned A sea captain tells her about Orsino and his love for Olivia Viola wishes to work in Oliviarsquos home but feels she will not be employed So she dresses as a man calls herself Cesario and gets work at the house of OrsinoViola (now Ceasario) is much liked by Orsino and becomes his page She falls in love with Orsino Orsino sends Ceasario to deliver messages to Olivia Olivia herself falls for the beautiful young Ceasario believing Viola to be a man

2 Answer the following questionsa Why does Orsino ask the musicians to play onOrsino asks the musicians to play on because music feeds his desire He calls upon the musicians to play music so that his hunger for love could be replenished with an excess of musicb What does Valentine tell about OliviaWe learn from Valentine that Olivia is in mourning for her brother she wears a veil and has vowed that no one will see her face for another seven yearsand she refuses to marry anyone until thenc From the exchange between Orsino and Valentine what do you think their relationship isValentine is one of orsinod attendants He was sent to Olivia as a messenger of love but was not allowed to speak to here Who is Olivia mourning for and whyOlivia is mourning for her dead brother

Homework Q fHistory and Civics

Growth of Nationalism

Important dates to remember1769-Napoleon born on 15thAugust1789-Fall of Bastille on 14th July and the beginning of the French revolution declaration of the rights of Man on 26thAugust1793-King Louis XVI executed on January 211764-The Sugar Act passed1765-The Stamp act passed1774-The first congress of Philadelphia1776-The declaration of American Independence of on 4th July1777-Defeat of the British at Saratoga1781-Surrender of lord Cornwallis at Yorktown1783-The treaty of Versailles1804-Napoleon becomes the emperor1813-Battle of Leipzig or Battle of nations in which Napoleon was defeated by the Allies1815-Battle of Waterloo June 18 in which Napoleon was defeated and captured1821-Death of Napoleon in StHelena1860-Abraham Lincoln elected President of the USA1861-The civil war began 1864-Abraham Lincoln elected President of the USA for the second time1865-Slavery abolished in the US

Name the following- The queen of Louis XVI

Marie Antoinette The three philosophers of France

VoltaireMontesquieuJean Jacques Rousseau

The British general whose surrender brought the war in America to an endLord Cornwallis

The first president of the USAGeorge Washington (1732-1799)

The first southern state to secede from the unionSouth Carolina

The author of the book lsquoUncle Toms CabinHarriet Beecher Stowe

Homework-Learn

Class IXSubject Topic Summary Execution

Economics

Types of economies Today I am going to share you the concept of economic growth and economic development Few questions will be given from the previous study material dated 942020

Meaning of economic growthAnswer) The term economic growth generally means anincrease in national income or per capita output or income over time It indicates towards quantitative growth of a country

Meaning of economic developmentAnswer) Economic development is defined

as a process whereby the real per capita income of a country increases over time along with fall in poverty ratio unemployment and income inequality etc

Distinguish between economic growth and economic development

Basis Economic growth

Economic development

Scope It has narrow scope as it refer only to rise in per capita income

It has wide concept since it includes qualitative changes as well

Concerned matter

It is concerned with the rise in income

It is concerned with not only rise in income but also reduction of poverty income inequality and unemployment

Focus Economic growth does not focus on economic development

Economic development focus on economic growth plus qualitative changes

Distinguish between capitalist economy and socialist economy

Ownership

Motive

Tool

Means of production are owned and managed by private people

Self interest and profit earning is the main motive

Price mechanism is a main tool to solve the economic problems

Means of production are owned and managed by the government

Social welfare is the main motive

Economic planning by the government is the main tool to solve the economic problem

Competition

Distribution of income

There exist large competition among buyers and sellers

There is existence of large inequalities of income

There is no such competition

There exist less inequalities of income

Math Topic ndash AlgebraChapter -Factorisation

Study item Factorising by taking out common factorSome solved sums from exercise 41

1) (i) 8xy3 + 12x2y2

= HCF of 8xy3 and 12x2y2 is 4xy2

= 4xy2(2y + 3x )

4) (ii) 28p2q2r ndash 42pq2r2

= HCF of 28p2q2r and 42pq2r2 is 14pq2r = 14pq2r (2p - 3r )5) (ii) 14mn + 22m - 62p=HCF of 14mn 22m and 62p is 2= 2(7mn + 11m - 31p)7) (ii) 3a(x2 + y2) + 6b (x2 + y2) = HCF of 3a(x2 + y2) and 6b(x2 + y2 ) is (x2 + y2)= ( x2+ y2 )(3a + 6b )9) (ii) x(x2 + y2 ndash z2 ) + y(-x2ndashy2 + z2 ) ndash z(x2+ y2 ndash z2 )= x(x2 + y2 -z2) ndash y-(x2 + y2 -z2) -z(x2 + y2 ndash z2)=x(x2 + y2-z2) -y( x2 + y2-z2) ndash z (x2 + y2 -z2)= (x2+ y2 ndash z2)(x ndash y ndash z )

Commercial Studies

Introduction to Accounting and Book-keeping

Today I am going to share you the meaning of Accounting and Book-keeping and its related terms bullAccounting bullBook Keeping bullAccountsbullTypes Of Accounts bullAccounting Cycle

bull Meaning of accounting

Ans )Accounting is the art and science of recording classifying and summarising monetary transactions

bull Meaning of Book-keeping

Ans) Bookkeeping is the art of recording business transactions with the view of having a permanent record of them and showing their effect on wealth

bull Meaning of account

Ans) The term account means a record of business transactions concern a particular person of firm asset or income or expense It is a summarised record of all transactions which take place in an accounting year

bull Types of accountsPersonal accounts ndash Personal accounts relating

to person and Organisation are known

as personal accounts Example Ramrsquos Account ABC amp Co Account etc

Real account - The accounts related to tangible and intangible assets are called real accountsExample Cash Account Furniture Account etc

Nominal account- Accounts related to expenses losses incomes and gains are known as nominal accountsExample Wages Account Salary Account Discount Account etc

bull Accounting cycle Accounting cycle refers to a complete sequence of accounting activities It begins with recording of transactions and ends with the preparation of a balance sheet

English 1 Transformation of sentences

Sentences A sentence is a group of words which makes complete sense

a Assertive sentencesb Imperative sentencesc Interrogative

sentencesd Exclamatory sentences

Sentences can be changed from one grammatical form to another without changing the meaning of the sentence This is known as transformation of sentences

Exercise 6Rewrite the following sentences according to the instructions given below without changing their meanings

1 As soon as he saw the beer he jumped into the river ( Begin No sooner)

2 None but brave deserve the fair (Begin the bravehellip)

3 This box is too heavy for me to lift ( Use so hellip That instead of too)

4 No one other than a king can live like James Luxurious ( Begin only James)

5 Oh for the wings of a dove (Begin I wishhellip)

BENGALI(2ND LANGUAGE)

ldquo বঙগভমির পরমি ldquo াইকেল ধসদন দতত

পব13পোসঠ আসোলিচত ৩ পরবোস দৈদসবর বস ীবতোরো Pলিদ স এ লেদ -আকো সত-োলি লেদ তোস - ক) বকতো লেক কোর লেো লেকো কলিবতোর অং ) কোর পরলিত বকতোর এই উলিকত গ) এ লেদ আকো সত বসত কী বলিঝসয়স4 ীবতোরো বসত কী লেবোঝ ঘ ) আসোচয অংসর তোৎপP13 কী

উ -ক ) বকতো স কলিব মোইসক ম3দ দতত

Types of AccountPersonal AccountReal AccountNominal AccountBalance Sheet (opening)

কলিব মোইসক ম3দ দসততর রলিচত বঙগভলিমর পরলিত কলিবতোর অং ) কলিব বঙগী অ13োৎ লেদমোতোর পরলিত কলিবর এই উলিকত গ ) এ লেদ আকো বসত কলিবর মোব লেদী রপ আকো লেক লেবোঝোসো সয়স4 আকো লেসক লেPম তোরো স পসর লেতমলি ীব লেদ রপ আকো লেসক পরো রপ তোরো স পরসত পোসর এই মভোবোর কোই কলিব বসস4 ঘ ) পরবো Pোতরোয় Pলিদ কলিবর লেদ আকো লেসক ীব তোরো রপ পরো স পসর তোসত কলিব লিবনদমোতর দঃলিত কোর মতয লিবসর সবোভোলিবক পলিরলিত এবং মোষ মরী তোই পরবোস Pলিদ তা োর মতয য় তবও কলিব লিবচলিত সব ো কোর পলিবীসত লেকউ অমর য় লিক4ই অকষয় য় দীর লেPম লিচরপরবোমো লেতমলি মোসষর ীবও চমোতোই ীব - সতবধতোই মতয ীব দীসত মোষ লিতয পরবোমো তবও লেPব মোষ আপ কতকসম13র মো3যসম মোসষর মস লিসসদর সথো কসর লিসত পোসর তোরো লিচরভোসবর সয় মোসষর মস লিবরো কসর তোসদর মস3য লেকউ পGভসত লিবী সয় গোসও মোসষর মস তোরো লিতযপলিত লিতযবলিনদত

Hindi 2ndlang

काकीी(लिसयारामशरणगपत)

इस कहानी म क न यह बतान का परयास निकया ह निक बचच अपनी मा स निकतना परम करत ह शयाम अबोध बाक ह वह अपनी मा क मरन क बाद उसन अपनी मा क लिए बहत रोया बाद म उस पता चा निक उसकी मा राम क घर ची गई ह आकाश म उडती हई पतग दकर उस हष हआ निक पतग क दवारा वह अपनी मा को नीच उतारगा इसक लिए वह अपनी निपता की जब स दो बार सवा रपया निनकाकर पतग और दो मोटी सी मन वाी अपन भाई स काकी एक कागज पर लिवा कर पतग म लिशव का दिदयानिनकाकर पतग और दो मोटी सी मन वाी अपन भाई स काकी एक कागज पर लिवा कर पतग म लिचपका दिदयाभोा और शयाम कोठरी म रससी बाधनी रह थ तभी उसक निपता करोध म आकर उन स पछ निक कया उनकी जब स रपया निनकाा हभोा डर क मार बताया निक शयाम इस पतग क दवारा अपनी काकी को राम क यहा स उतारना चाहता हनिवशशवर(शयाम क निपता)न फटी पतग उठाकर दी तो उस पर काकी लिा थावह हत बजि होकर वही ड रह गएउनहोन सोचा निक मन अपन पतर को मारा जोनिक अनजान और निनदष थावह अपनी मा कोनिकतना पयार करता ह

helliphellipContinue to next

Computer Application

Java Programming Prog 1Write a java program to input two numbers from user and display the sum or product of them as per user choice Use switch case statementSolve public class sum_product public static void main(String args[]) Scanner sc=new Scanner(Systemin) int abc Systemoutprintln(ldquoEnter two numbersrdquo) a=scnextInt() b=scnextInt() Systemoutprintln(ldquoPress 1 for sum or 2 for productrdquo)

c=scnextInt() switch(c) case 1 Systemoutprintln(ldquoThe sum will be =rdquo+(a+b)) break case 2 Systemoutprintln(ldquoThe product will be =rdquo+(ab)) break default Systemoutprintln(ldquoWrong Inputrdquo) Home Work - Practice in your computer using bluej

Subject Eng Literature (The Merchant of Venice ndash William Shakespeare)Topic Act I Scene 2 Lines 92 to 126 (End of scene) Date 13th April 2020 (5th Period)

[Students should read the original play and also the paraphrase given in the school prescribed textbook]Summary Questions amp Answers

o After Portia has expressed her opinion about the suitors Nerissa informs that she need not bother about any one of them as they have decided to quit Belmont at the earliest opportunity because they do not believe in trying their luck by the caskets which is the only way of winning Portia

o Nerissa then enquires of Portiarsquos opinion about Bassanio who once visited her in the company of the Marquis of Montferrat and says that she had never come across such an ideal love deserving the fairest lady for his bride

o Portia seems to remember Bassanio quite correctly and says that she agrees with Nerissa At this moment a servant informs Portia that the Prince of Morocco has arrived to try his luck by the caskets

o Portia tells Nerissa that if she could welcome this new suitor as gladly as she says farewell to the previous ones she would be glad of his arrival However if he happens to have the virtues of a saint but the black complexion of a devil she would prefer to have him for religious consolation rather than as a husband

(1) NERISSA You need not fear lady (Line 97-103)

the having any of these lords they have acquainted me with their determinations

which is indeed to return to their home and to

trouble you with no more suit unless you may be wonby some other sort than your fathers imposition depending on the caskets

PORTIA If I live to be as old as Sibylla I will die as chaste asDiana unless I be obtained by the manner of my fatherswill I am glad this parcel of wooers are so reasonablefor there is not one among them but I dote on his veryabsence and I pray God grant them a fair departure

(a) Elucidate the idea expressed in the first speech of the above dialogue

In the first speech Nerissa assures Portia that she need not have any fear of being compelled to marry anyone of the suitors who had lately come to Belmont She informs her that they have all decided to return to their respective countries(b) Illuminate the meaning of the phrase ldquoyour fatherrsquos imposition depending on the casketsrdquo

Nerissa means that the suitors of Portia do not find the conditions imposed by the will of her father to their liking They are too hard for them These conditions are that in the event of a suitor failing to choose the right casket (i) he should never disclose to anybody which casket he chose (ii) he can never marry and (iii) he should take his departure immediately(c) Explain the meaning of the term lsquoSibyllarsquo

lsquoSibyllarsquo is the name given by Romans and Greeks to a prophetess inspired by some deity usually the sun-god Apollo She had a very long life The god Apollo granted her as many years of life as she could hold grains of sand in her hand(d) Elucidate the meaning of the term lsquoDianarsquo

lsquoDianarsquo is the goddess of hunting She is also regarded as a symbol of virginity because she never fell in love and never

married(e) Explain the meaning of the first two lines of Portiarsquos speech

Portia says that even if she is to live for centuries like Sibylla she would not marry except in accordance to her fatherrsquos will She asserts that she would not mind remaining unmarried and untouched by a man like Diana the virgin the goddess of hunting unless a man is able to win her by passing the test laid down by her father

Class XSubject Topic Summary Execution

Hindi 2nd

Langबड घर की बटी( मशी परमचद)

lsquoबड घर की बीटीrsquo कहानी का उददशय मधयम वग की घर समसया को सझा कर सगदिठत परिरवार म मिम जकर परम स रहन का सदश दना ह घर म शानित tानिपत करन की जिजममदारी नारी की होती ह यदिद नारी समझदार ह उसम धय और परिरवार क परनित परम ह तो कोई भी घटना परिरवार को निवघदिटत नही कर सकती या कहानी परिरवार को सगदिठत करत हए परम सौहाद स एक रदसर की भावनाओ को समझ करउनका सहयोग करत हए जीवन यापन करन की पररणा दती ह मशीपरमचदर जी न इस कहानी म सयकत परिरवार का परनितनिनमिधतव निकया ह यह कहानी बनी माधव सिसह जो गौरी पर क जमीदार क उनक दो पतरो की हशरी कठ ा निबहारीशरीकात का निववाह एकजमीदार घरान की पतरी आनदी स हआ थाआनदी न द को ससरा क वातावरण म ढालिया थाएक दिदन आनदी का अपन दवर ा निबहारी स झगडा हो जाता ह दोनो भाई एक रदसर स अग होन की कोलिशश करत हसभी बह आनदी न अपन मधर वयवहार स ा निबहारी को घर छोडकर जान स रोक लिया| इस पर बनी माधव सिसह न कहा निक बड घर की बटी ऐसी ही होती ह जो निबगडा काम बना ती ह अतः शीषक साथक ह बड घर की बटी आनदी ह

helliphelliphelliphellipContinue to nextBiology Topic ndash Chp-1

CellWelcome to new session 2020-21Today we will start with Chpter 1 cell CELL

Protoplasm+Cellmembrane Or Cell wall

Cytoplasm+Neucleus

Cytoplasmic+ CytoplasmicOrganelles Inclutions(mitochondria (food Golgi bodies pigments)Ribosome)

What is cellbull Cell is the structural and functional unit of living organismbull According to number of cells organisms areUnicellular - Amoeba bacteria Multicellular - Rose Mango Tiger HumanSmallest cell -bacteria Longest cell - Nerve cellLargest cell - Ostrich egg cellCells are of different size and shapes according to their functionsQ2Write chief functions of following cellorganelles

Q3What is tonoplastVacuoles covered by a covering called tonoplast

Bengali(2Nd

Language)

ফ ফটক ো ফটক (কলিবতো ) ভোষ মসোপো3 gtPোয়

একটি লেমসয়র ীবস লেপরম লিকভোসব ফসট ওসঠ তো লেদলিসয়স4 কলিব লেপরম Pই য় লেই ময়ই বনত কোস পলিরত য় ফ লেফোটো বো োসফোটো লেটো ব2 কো য় লেমসয়সদর ব gtয13 লেপরসমর 4লিব ফসট উসঠস4 এই কলিবতোয় লেপরম মোষসক মত gtযর মস লেফস লিদসয় পরকষস বাোচোসোর gtয োত বো2োয় কলিবতোয় লেমসয়টির পসব13র দঃসর কো বো সও লেমসয়টি লেই পসর পলিক সত চোয়ো োরী ীবসর কোস4 পরম লেPৌবস লেপরমসক পোবোর পরব ইচছো োকসও তো পসর লেলিতবোচকতোয় পলিরত য় কলিব ভোষ মসোপো3 যোয় লেP ক লেপরসমর

কলিবতোয় ব gtযবহত লিবসষ লিক4 সvর অ13 লেদওয়ো ১) রসবোো= লেP লিবলিভনন রকম ডোকসত পোসর২) ো= পোর ৩) ঠলি = লেচোসর বZ৪)আই বস2ো=অলিববোলিত৫)শইসয় = োলিয়ত কসর৬)োতপাোচ= লিবলিভনন পরকোর৭)দ2োম = v কসর বZ কসর লেদওয়ো৮)লেরলিং =লেোোর দৈতরী লেব2ো৯) বনত= একঋত১০) পাোর = বসকরো2

Organelles Functions

1 Endoplasmic reticulum

2 Mitochondria

3Golgibodies

4 Ribosome5Lysosome

6Plastids

7 Centrosome

i) Supportive framework for the cellii) Synthesis and transpost of proteinsRelease of energy in the form of ATPi) Synthesis and secretion of enzymes hormoneii) Formation of vacuoles lysosomei) Protein Synthesisi) Intracellular digestionii) Destroy foreign substancei )Leucoplast - stores starchii)chloroplast - trap solar energyiii) Chromoplast - imparts colour toflowers amp fruitsi) Initiates and requlates cell division

কলিবতো তোর অ13সক ভোষোয় পরকোো কসর ঘলিরসয় ব যকত কসরস4 লেপরসমর ফতো আর লিবফতো লেক গোঢ় কসর লেদোসো কলিব ভোষ মসোপো3 যোসয়র অলিভবসর অ যলিদক

Economics

Factors of Production

Welcome to the new sessionToday we are going to start the first chapter of Class XThe name of the chapter is Factors of productionBy the name I hope you all can recall a glimpse of what you have learnt in the second chapter of Class IX

NowProduction is the process of creating the various goods and services which are consumed by the people of the country to satisfy their wants

Thus it is the process in which some materials are transformed from one form to another to create utility and value in goods

For example utility can be created by changing the form of a commodity ie

Making of table out of wood by a carpenter for his customer here the wood is getting transformed into table creating utility for his customer and he can also command a price for it

On the other hand Housewives perform very

useful activities at home which create utility but their domestic activities are not included in production because they have no money value

So we can also say that Production denotes two things firstly creation of utility and secondly creation of value

Production is not complete unless it reaches the consumer

An increase in production will increase the economic welfare of the consumers and hence the aim is to raise the production level of the country

Again production of a good or service is only possible if certain resources or

Questions

1 What do you mean by production

Answer Production means the creation of goods and services for the purpose of selling in the market

In fact production involves the transformation of inputs into outputs

Hence production denotes two thingsCreation of utility and creation of valueUtility and value can be created by changing the form by changing the place by changing the time and by rendering services

Example Transformation of raw

materials into finish goods such as potter creates utility by converting mud into utensils assembling of small parts to make bigger machinery

Production also includes services such as distribution and marketing

2 What are the factors of production

Answer Factors of Production refers to the resources and inputs needed for producing goods and servicesThese inputs can be classified as

Land Labour

Capital Enterprise

Land Land is defined to include not only the surface of the earth but also all other free gifts of nature(for example mineral resources forest resources and indeed anything that helps us to carry out the production of goods and services but is provided by

inputs are used together in right proportion

A resource or an input which helps in the process of production to obtain an output is called FACTOR OF PRODUCTION

These factors of production can broadly be categorized into four parts 1LAND 2LABOUR3CAPITAL4ENTERPRISE (ORGANISATION)or Entrepreneur

The above factors are all interdependent on each other and they play a major role in production process

FACTORS OF PRODUCTION

LANDCAPITAL

LABOUR ENTREPRENEUR

nature free of cost)LabourLabour refers to the human efforts that need to be combined with other factors of production for creating an output

CapitalAll man ndash made means of production is called capita example machineries which help in further production Money when used for starting any business for purchasing raw materials machinery tools etc it is regarded as capitalCapital also includes physical capital like factories machineriestoolsbuildingsequipments etcEnterpriseThe task of bearing risks is called enterprise and the person who bears these risks of business is called the entrepreneurThus an entrepreneur is one who organises production takes important decisions regarding production hires and purchases factors of production and bears the risk and uncertainty involved in productionOrganisation refers to the services of an entrepreneur who controls organises and undertakes all risks One who plans organises and manages a business enterprise is an organiser

Physics Chapter 1 Force

Force is an external agent capable of changing the state of rest or motion of a particular body It has a magnitude and a direction The direction towards which the force is applied is known as the direction of the force and the application of force is the point where force is applied The Force can be measured using a spring balance The SI unit of force is Newton (N)

Question 1

State the condition when on applying a force the body has

(a) the translational motion

(b) The rotational motion

Solutions

(a) Translational motion is produced when the body is free to move

(b) Rotational motion is produced when the body is pivoted at a point

Question 2

Define moment of force and state its SI unit

Solutions

The moment of force is equal to the product of the magnitude of the force and the perpendicular distance of the line of action of force from the axis

of rotation

The SI unit of moment of force is Newton times meter

= Newton meter (Nm)

Commercial Studies

Stake holders In this topic you will be come to know about the meaning and concept of stakeholders

How stakeholders are different from shareholders

Questions1 What do you mean by the term stake holdersAnswer) The term stake holders have developed from the words which mean an interest or expected benefit Stakeholders mean all those individuals groups and Institutions which have a state (interest) in the functioning and performance of a commercial organisation or a business enterprise2 What do you mean by share holdersAnswer) The person and Groups who own the shares of the joint stock company by providing capital to the company are called shareholders Shareholders are the internal stakeholders shareholders are one out of several stake holders3 How are shareholders different from stakeholdersAnswer)i) The term shareholders is related to only joint stock company whereas stakeholders are related with all business organisationsii) Stakeholders maybe any individual having financial stake in business organisation whereas a shareholders are those individuals who are holding shares in the company4) How are shareholders different from creditorsAnswer) i) Shareholders are internal stakeholders while creditors are external stakeholdersii) Shareholders invest in the capital of the company whereas creditors give loan to the companyiii) Shareholders are the members of the company with voting rights but creditors are not the members of the company

English 1 Transformation of sentences

Sentences A sentence is a group of words which makes complete sense

e Assertive sentencesf Imperative sentencesg Interrogative sentencesh Exclamatory sentences

Sentences can be changed from one grammatical form to another without changing the meaning of the sentence This is known as transformation of sentences

Exercise 1 Change the following affirmative sentences into Negative sentences

a He is a good manHe is not a bad man

b Ram loves SitaRam is not without love for Sita

c Only he stood first in the classNone but he stood first in the class

d Ankit was wiser than he

He was not so wise as Ankite He did it

He did not fail to do itf As soon as I reached college the

bell rangNo sooner did I reach college than the bell rang

g He finished everythingHe left nothing unfinished

h It always pours when it rainsIt never rains but it pours

Math Topic Commercial MathematicsChapter ndash Goods and services Tax

What is GSTAns It is a abbreviated term of Goods and Service Text which is an indirect tax levied on the sale of goods and rendering servicesSome terms related to GSTDelar Any person who buys goods or services For resale is known as a delar A delar Can be a firm or a companyIntra-state sales Sales of goods and services within the same state or same union territory are called intra- state salesInter-state sales Sales of goods and services outside the state or union territory are called Inter-state sales4) Input GST GST is paid by dealers on purchase of goods and services are called input GST5) Output GST GST is collected from customers on sale of goods and services are called output GST6) Types of GST There are three taxes applicable under GST(i) Central Goods and Services Tax (CGST)(ii) State Goods and Services Tax (SGST) or Union Territory Goods and Services Tax (UTGST) Both these taxes are levied on intra-state sales Here GST is divided equally among central and state governments(iii) Integrated Goods and Services Tax (IGST) IGST is levied on inter- state sales It is also levied on import of goods and services into India and export of goods and services from India

Subject Eng Literature (The Merchant of Venice ndash William Shakespeare)Topic Act III Scene 4 Lines 1 to 44 (Portia hellip To wish it back on you fare you well Jessica)[Students should read the original play and also the paraphrase given in the school prescribed textbook]

Summary Questions amp AnswersIn this scene we suddenly find a new element in the character of Portia We have already seen her possessed of every graceful womanly quality but now she shows that she is capable of rapid decision and determined action She shows this by her sudden resolve to hasten to Venice with a daring scheme for the rescue of Antonio This is an important scene in the dramatic action for it leads up to and renders possible the striking events of the famous trial scene which is one of the greatest striking elements of the play Moreover the fact that all the characters of importance are now assembled together in Venice makes the union of the main plot and the secondary story complete

(1) LORENZO Madam although I speak it in your presence(Line 1-9)

You have a noble and a true conceit

Of god-like amity which appears most strongly

In bearing thus the absence of your lordBut if you knew to whom you show this honourHow true a gentleman you send reliefHow dear a lover of my lord your husbandI know you would be prouder of the workThan customary bounty can enforce you

(a) Where is Lorenzo Why is he here To whom is he referring as lsquoMadamrsquo

Lorenzo is at Portiarsquos residence He had met Salerio on the way and Salerio had begged him to come along with him to

o In this scene Portia Nerissa Lorenzo Jessica and Balthazar appear

o Portia requests Lorenzo and Jessica to be in charge of her house during her absence from Belmont because she and Nerissa have decided to spend the days in meditation and also in visiting the holy places in the neighbourhood of Belmont She has already instructed her people to acknowledge both Lorenzo and Jessica as master and mistress of house during her absence Lorenzo and Jessica gladly agree to look after the house of Portia

handover the letter from Antonio to Bassanio The letter carried the bad news about Antoniorsquos arrest for non-payment of loan taken from Shylock Hence Salerio might have preferred company to break this bad news to Bassanio He is referring to Portia as Madam(b) What does Portia say on hearing the above extract

Portia says that she has never regretted doing good to others Friends who spend a lot of time together and really are there for each other have many traits in common As Antonio is Bassaniorsquos best friend saving him is like saving Bassanio who is like her own soul She asks Lorenzo to take care of management of the house till Bassanio is back(c) What does Portia send with Bassanio and why

On hearing about Antoniorsquos troubles on account of Bassanio her husband Portia immediately sends him with enough gold to repay the debt many times over to Venice to help Antonio out of his misfortune

(2) Lorenzo Madam with all my heart (Line 36-40)

I shall obey you in all fair commands

Portia My people do already know my mindAnd will acknowledge you and JessicaIn place of Lord Bassanio and myselfSo fare you well till we shall meet again

(a) Where are Lorenzo and Portia at this time What lsquofair commandsrsquo are given to Lorenzo

Lorenzo and Portia are at Belmont during this scenePortia reveals to Lorenzo that she has sworn to contemplate in prayer at a monastery around two miles away until her husband returns from Venice She tells him that Nerissa would accompany her and asks him to manage the house with Jessica till things are settled In response Lorenzo tells her that he would be obliged to do whatever she asks him to do(b) Where is Portia actually going and why

Portia tells Lorenzo that she would live a life of contemplation and pray at a monastery which is two miles away from her place In reality Portia plans to go to Venice in disguise with Nerissa and argue the case in defense of Antonio She is very sure that her plan would succeed

ClassXI (ScienceHumanitiesCommerce)Subject Topic Summary Execution

Computer Science

(APC)

Ch ndash 1 Numbers

(Numbers in different bases and

their Arithmatical operations)

Number System In computers Number System is defined as a writing system to represent the numbers in different ways ie we are using different symbols and notations to represent numbers There are four ways we can represent the number ndash Binary Decimal Octal and Hexadecimal

Decimal Number SystemThis number system consist 10 digits These are 0 1 2 3 4 5 6 7 8 amp 9

Binary Number SystemThis number system has only two digits these are 0 and 1 Here 0 stands for off while 1 stands for on

Octal Number SystemThis number system has 8 digits these are 0 1 2 3 4 5 6 amp 7

Hexadecimal Number SystemThis number system has 16 digits these are 0 1 2 3 4 5 6 7 8 9 A B C D E F Here the value of the alphabets are as follows A=10 B=11 C=12 D=13 E=14 F=15

Rules for conversion decimal number to Binary1 Divide the decimal number by 22 If the number will not divide equally by 2 then round down the answer to the nearest whole number (integer)3 Keep a note of the remainder it should be either 0 or 14 Keep repeating the above steps dividing each answer by 2 until you reach zero5 Write out all the remainders from bottom to top This is your binary solution

For example Lets convert 32 to binary 2 32 2 16 - 0 2 8 - 0 2 4 - 0 2 2 - 0 2 1 - 0 0 - 1

The binary equivalent of 3210 is 1000002

Try the follwing youself1 2410

2 4810

3 1210

History GROWTH OF NATIONALISM

The second half of the 19th century witnessed growth of political consciousness and a sense of Nationalism among the IndiansThere were various factors for growth of Indian Nationalism- As a result various political associations were formed in different provinces by the educated Indians Surendranath Banerjee organized a meeting of National conference at Calcutta Ultimately the National Congress was founded in Bombay in 1885This body became the vanguard of Indian struggle for freedom The congress leaders were known as moderates because they followed a policy of prayer and petition A large number of Indian leaders had experienced in political agitation The Political situation of England was also changed Moreover increasing revolutionary activities in Maharashtra Punjab and Bengal became serious concern to the British Government In this

QUESTION1 What do you mean by Nationalism ANSWER 1 Nationalism is defined as loyalty and devotion to own nation especially a sense of national consciousnessQUESTION 2 What are the causes of nationalism ANSWER 2 There were various factors for growth of nationalism

1 Spread of western education2 The progress of vernacular press and

patriotic literature3 The economic exploitation of our

country by the colonial rulers4 International affairs

QUESTION 3 Who organized National conference in Calcutta in 1883 ANSWER 3 Surendranath BanerjeeQUESTION 4 When did Indian National Congress formANSWER 4 Indian National Congress was formed in 1885 in BombayQUESTION 5 Who were ModeratesANSWER 5 The Early Nationalists were also known as Moderates Their emergence marked

background Lord Curzon became Viceroy in India He had no respect for the Indian National Congress

the beginning of the organized national movement in India They believed in British justice and were loyal to them They followed a policy of prayer and petition They demanded constitutional reforms of our country Impotant Moderate leaders were Pherozshah Mehta Dadabhai Naorozi and Surendranath Banerjee etcQUESTION 6 What do you know about Extremism in Indian National movementANSWER 6 In the beginning of 20th century a new class of national leaders emerged in India which was different from the moderate groups They started more aggressive movement against the British empire The goal of extremists was ldquoswarajrdquo Important extremist leaders were Bal Gangadhar Tilak Lala Lajpat Rai Bipin Chandra Pal etcQUESTION 7 Mention the places which were the main centres of Revolutionary movementANSWER 7 Maharashtra Bengal and Punjab

Physics

Chapter Dimensional Analysis

(Summary)

The dimensions of a physical quantity are the powers to which the fundamental units are raised in order to obtain the derived unit of that quantit

The physical quantites lengthmasstime are represented by [L] [M] [T] resp let they are raised to powers ( dimesions) abc resp then any physical quantity can be represented by [ La Mb Tc ] Examples

1 Area area = L x B = [L] x [L] = [M0 L2 T0 ]

2 Density density = massvolume = [M][L3] = [ M L-3]

3 Velocity velocity = distancetime = [L][T] = [LT-1]HW Try to find out dimension of acceleration Acceleration = velocity timeNB One can find the SI Units Using Dimension Analysis Such as for area we have [L2] so its SI unit is m2

Biology Topic ndash Chp-1 The living world

Today we will start the first chapter the living world Here we discuss about the characteristics of living organism and what are the difference between them and nonliving substances We also discuss about the contribution of different Scientists

There are over 500000 species of plants andover a million species of animal are present on earth Some 15000 new species were discovered every yearQ1 What is a living organismbull A living organism is primarily physico -chemical material that demonstrate a high degree of complexity is capable of selfRegulation possesses a metabolism and perpetuates itself through timeQ2 What are the differences between livingand non-livingsi) Compared with non-living living organisms

have more complex organised structure and their use of energy is more controlled amp efficientii) Living things reproduce their own kind by forming new cells which contains copies of their genesiii) Each organism has some degree of homeostasisie it is able to make adjustments so that internal environment remains constantQ3 Write contributions of following Scientists i) Aristotle - One of the first theories in Biology places all living things in a hiearchieii) AV Leeuwenhoek - was the first to observe living single celled organisms under microscopeii) Carolus Linnaeus - developed the binary system for naming of organisms and classificationiii) Geregor Johann Mendel ndash discoverbasic principles of inheritanceHomework i) C Darwin ii)Schleiden

Math Trigonometric functions

1 Overviewi) Trigonometry The word lsquotrigonometryrsquo is derived from the Greek words lsquotrigonrsquo and lsquometronrsquo which means measuring the sides of a triangle An angle is the amount of rotation of a revolving line with respect to a fixed line Usually we follow two types of conventions for measuring angles ie a) Sexagesimal system b) Circular system In Sexagesimal system the unit of measurement is Degree In Circular system the unit of measurement is Radian ii) Relation between degree and radianThe ratio of circumference of a circle to its diameter is always a constant This constant ratio is a number denoted by π which is taken approximately as 227The relationship between degree amp radian measurements is as follows2 right angles = 180deg= π radians1radian = 180degπ=57deg16(approx) 1deg=π180 radianiii) Length of an arc of a circleIf an arc of length s subtends an angle θ radians at the center of a circle of radius r then s=rθiv) Area of a sector of a circleA sector is like a pizza slice of the

Q) Express the following angles in radiana) 45deg b) 40deg3730Ans a) We have 180deg=π radiansi e 45deg= πtimes45180 radian = π4 radiansb) 40deg3730= 40deg37+3060 minute= 40deg 37 +12 minute= 40deg+ 752 minute=40 + 75(2times60) degree=3258 degreeNow 180deg=π radianie 3258 degree= (πtimes325) (180times8) radians = 65π288 radiansQ) A circle has a radius of r=12 meters What is the length of an arc traced out by a 60deg angle in the center of the circleAns In this problem we know both the central angle (60deg) and the radius of the circle (12) All we have to do is plug those values into our equation and we get

s = 2π(12)(60360)s = 24π6s = 4πSo the length of an arc traced out by a 60deg angle in a circle with a radius of 12 meters equals 4π meters asymp 1257 metersQ) Find the area of the sector with a central angle 30deg and a radius of 9cmAns GivenRadius r = 9 cmAngle θ = 30degArea of the sector = θ360degtimesπr2

= 30360degtimes227times92=2121cm2

circle It consists of a region bounded by two radii and an arc lying between the radiiThe area of a sector is a fraction of the area of the circle This area is proportional to the central angle In other words the bigger the central angle the larger is the area of the sectorArea of Sector = θ2 times r2 (when θ is in radians)

Area of Sector = θ times π360 times r2 (when θ is in degrees)

COMMERCE

CLASSIFICTION OF HUMAN ACTIVITIES-ECONOMIC AND NON-ECONOMIC

Welcome to the new sessiontoday we are going to start the first chapter of Class XI The name of the chapter that we are going to start is

lsquoClassification of Human Activities ndasheconomic and non-economicrsquo

Now let us start the chapter by considering human beings and the activities they perform throughout the day

Human activities means all those activities that human beings undertake to satisfy their wants

Human wants on the other hand are the desire of human beings for goods (vegetables fruits rice etc) and services (services of doctors teachers lawyers etc) that they require to live

Now these human activities continue throughout life as human wants are unending unlimited and recurring as human beings desire for better living throughout their lives

Now human activities can be classified into two categories

Human activities

Economic activities Non-economic activities

Economic activities are

Questions1 What are human activities

Answer Human activities mean all those activities that human beings undertake to satisfy their wants

Example A man working in an office

A boy playing in the garden

2What are the characteristics of human activitiesAnswer the characteristics of human activities are as follows

Human activities are undertaken by men women and children and these activities involve human efforts

Human activities are undertaken to satisfy human wants which are unlimited

Human activities continue throughout life

Human activities are performed for both earning money and personal satisfaction

3What is economic activitiesGive example

Answer Economic activities are undertaken by human beings with the object of earning money acquiring wealth and thereby satisfying human wantsExample

Selling of goods by a shop keeper to his customer

A clinic run by a doctor Service of a teacher in school or college

undertaken by human beings with the object of earning money and acquiring wealth

These activities result in the production of economic goods and services

Example Human activities(ie working in factories officesshops) which produce direct economic benefits

Non-economic activities are inspired by human sentiments and emotions such as love for the family desire to help the poor and love for the country

Thus these human activities (eg praying playing sleeping) produce no direct economic benefits and they are also not related to earning money and acquiring wealth

4 What are the characteristics of economic activities

Answer The characteristics of economic activities are as follows

Economic motiveEconomic activities are undertaken to earn money and acquire wealth

ProductiveEconomic activities involve productiondistribution and exchange of goods and services to create wealth

Economic growthEconomic activities determine the level of economic development of a country and standard of living of its citizens

Socially desirableEconomic activities are socially desirable for society

Economic resourcesEconomic activities make use of all the economic resources such landlabourcapital etc

5 What do you mean by non-economic activitiesExampleAnswerNon-economic activities are inspired by human sentiments and emotions such as love for the family desire to help the poor and love for the countryThese activities are not undertaken for monetary gain but for onersquos satisfaction and happinessExample

a mother looks after her children

a student donates blood8 Differentiate between Economic activities and Non-economic activities

Economic activities

Non-economic activities

1to earn living and acquiring wealth2Result can be measured in terms of money

3ExampleBusinessprofession and employment

1 to obtain some satisfaction

2Result cannot be measured in terms of money

3ExampleFamily-orientedreligious socialCultural and national

BUSINESS STUDIES

BUSINESS ENVIRONMENT

Welcome to the new sessionToday we are going to start the first chapter and the name of the chapter is Business Environment

In todayrsquos world every business enterprise is a part of the society It exists and operates in association with various groups in society such as customers suppliers competitors banks and financial institutions government agencies trade unions media and so on All these groups influence the functioning of business in one way or the other They constitute the environment of businessConcept of Business Environment

The term lsquobusiness environmentrsquo refers to the sum total of all individuals institutions and other forces that lie outside a business enterprise but that may influence its functioning and performance

The main features of business environment

Totality of External forces General and Specific forces Interrelatedness Complexity Dynamic Uncertainty Relativity

The Interrelation between business and its environment

The business enterprise is an open system It continuously interacts with its environment It takes inputs

Prepare the following questions from todayrsquos assignment

1 What do you mean by business environment

The term lsquobusiness environmentrsquo means the aggregate of all forces factors and institutions which are external to and beyond the control of an individual business enterprise but they may influence its functioning and performance Business environment is the macro framework within which a business firm a micro unit operates It consists of several interrelated and interacting elements

2 Explain the main features of business environment in brief

Totality of External forces-Business environment is the sum total of all things external to a business environment

General and Specific forces-It includes both the forces general forces are the economic social political legal and technological conditions which indirectly influence all business enterprise Specific forces are the investors customers competitors and suppliers which influence individual enterprise directly

Interrelatedness-Different elements of environment are interrelated for an example growing awareness for health care has increased the demand for health foods

Complexity- Business environment id

(such as raw materials capital labour energy and so on) from its environment transforms them into goods and services and sends them back to the environment

Fig 1 Business Environment Relationship

complex in nature as the elements keep on changing example economic technological and other forces changes in demand for a product and service

Dynamic-Business environment is not static it keeps on changing

Uncertainty- Itrsquos very difficult to predict future events such as technology and fashion which occur fast and frequently

Economics Basic Economic ConceptsSub topic

Microeconomics and

Macroeconomics

Welcome to the new sessiontoday we are going to start the first chapter of Class XI The name of the chapter that we are going to start is Basic Economic concepts

Now Economics covers the study of human activities Human activities are those activities which are performed by humans to satisfy their wants

Thus Human wants are unlimited and therefore economic activities such as production exchange and consumption are needed in order to satisfy those wants

The study of economics is divided largely in two parts which areMicroeconomics and Macroeconomics

SUBJECT- MATTER OF ECONOMICS

MICROECONOMICS MACROECONOMICS

Questions1Who has coined the words micro and macro economics

Answer Ranger Frisch coined the words lsquomicrorsquo and lsquomacrorsquo in 1933 to denote the two branches of economic theory namely microeconomics and macroeconomics

2What is microeconomicsAnswer It is the study of behaviour of individual decision ndash making unit such as consumers firms etc

3 What is macroeconomicsAnswer Macroeonomics is the study of overall economic phenomena like employment national income etc

4 What is the importance of microeconomicsAnswer

Microeconomics helps in formulating economic policies which enhance productive efficiency and results in greater social welfare

It helps the government in formulating correct price policies

It explains the working of a capitalistic economy where individual units(producers and consumers ) are free to take their own decision

Micro means a small part in

microeconomics we do not study the whole economy Hence we study an individual consumer and his or her choices and a producer and his or her profit maximizing decisions in the market Thus it does not mirror what happens in the economy as a whole

Macroeconomics on the other hand studies the economy as a whole It is concerned with aggregate and depicts the entire picture of the economyMacroeconomics deals with the national income aggregate investment aggregate consumption etc

Features of Microeconomics It deals with small

parts of the country Hence it looks at

individual consumers firms and industries

It deals with individual income consumption and savings

It studies the determination of price of any product or factors of production

It deals with the working of market via the price mechanism which is nothing but the determination of price and quantity of a commodity by the forces of demand and supply

Features of Macroeconomics

It deals with the study of the economy as a whole

It is concerned with

5 Give a limitation of microeconomics Microeconomics fails to explain the

functioning of an economy as a whole It cannot explain unemployment illiteracy and other problems prevailing in the country

6 What is the importance of macroeconomics It gives overall view of the growing

complexities of an economic system It provides the basic and logical

framework for formulating appropriate macroeconomic policies (eg for inflation poverty etc )to direct and regulate economy towards desirable goals

7What is the limitation of macroeconomics It ignores structural changes in an

individual unit of the aggregate

8 Differentiate between Microeconomics and Macroeconomics

Microeconomics Macroeconomics

the study of aggregates

National income aggregate savings and aggregate investments are major concepts dealt within macroeconomics style

It studies the determination of general price levels

It investigates into the problem of unemployment and the achievement of employment

It studies the aspect of decision making at the aggregate and national levels

It includes all growth theories whether related to developed or developing economies it also includes the study of economic systems and the working of the economy under different systems

Note Both Micro and macro economics are complementary and should be fully utilized for proper understanding of an economy

1It studies economic aspect of an individual unit2It deals with individual incomeConsumption and savings

3 It facilitates determination of price of any product or factors of production

4 Itrsquos scope is narrow and restricted to individual unit

1It studies the economy as a whole

2It deals with the national income aggregate consumption and aggregate savings3 It facilitates determination of general price level in an economy

4 Itrsquos scope is wide as it deals with economic units on the national level

ACCOUNTS

Introduction to Accounting and Book-keeping

Today I am going to share you the meaning of Accounting and Book-keeping and its related terms bullAccounting bullBook Keeping bullAccountsbullTypes Of Accounts bullAccounting Cycle

bull Meaning of accounting

Ans ) Accounting is the art and science of recording classifying and summarising monetary transactions

bull Meaning of Book-keeping

Ans) Bookkeeping is the art of recording business transactions with the view of having a permanent record of them and showing their effect on wealth

bull Meaning of account

Ans) The term account means a record of

business transactions concern a particular person of firm asset or income or expense It is a summarised record of all transactions which take place in an accounting year

bull Types of accountsPersonal accounts ndash Personal accounts relating

to person and Organisation are known as personal accounts Example Ramrsquos Account ABC amp Co Account etc

Real account - The accounts related to tangible and intangible assets are called real accounts Example Cash Account Furniture Account etc

Nominal account- Accounts related to expenses losses incomes and gains are known as nominal accounts Example Wages Account Salary Account Discount Account etc

bull Accounting cycle Accounting cycle refers to a complete sequence of accounting activities It begins with recording of transactions and ends with the preparation of a balance sheet

Chemistry TopicAtomic Structure

Thomsonrsquos atomic modelThomson (1898) was the first to propose the model of an atomHe proposed that an atom can be regarded as a uniform sphere of positive electricity in which requisite number of electrons are embedded evently to neutralize the positive chargeThis is just like plums embedded in a pudding or seeds evently distributed in red spongy mass of a watermelonThis model of atom is known as ldquoPlum-Pudding modelrdquo or

Q1)What is the fundamental constituents of atomAns Electron Proton and neutrons are the fundamental constituents of atomQ2)What is the value of fundamental unit of electricityAnsThe charge carried by one electron is sad to be the fundamental unit of electricityIts magnitude is 48times10-10esuOr 1602times10-19C Q3)Name the element containing no neutronAnsOrdinary hydrogen atom or protium 1H1

Types of AccountPersonal AccountReal AccountNominal AccountBalance Sheet (opening)

ldquowatermelon modelrdquoThis model could explain the electrical neutrality of an atom but failed to explain the result of scattering experiment carried out by Rutherford in 1911So it was rejected ultimately

Q4)Why is an electron called universal particleAns Itrsquos mass and Charge are independent of its source

EVS Chapter 1 ndash Modes of Existence

Modes of existence When one speaks normally about the mode of existence of some group or individual one refers to their customs their mode of being their ethology their habitat in some way their feeling for a placeDifferent modes of exixtence are ndash

1 Hunting ndashGathering2 Pastoral3 Agricultural4 Industrial

1 Hunting and gathering Hunting and gathering mode of existence is characterized by obtaining food from hunting wild animals including fishing and gathering wild plants From their earliest days the hunter-gatherer diet included various grasses tubers fruits seeds and nuts Lacking the means to kill larger animals they procured meat from smaller game or through scavenging

Societies that rely primarily or exclusively on hunting wild animals fishing and gathering wild fruits berries nuts and vegetables to support their diet are called hunting and gathering societies

At least this used to be practice of human beings before agriculture is invented As their brains evolved hominids developed more intricate knowledge of edible plant life and growth cycles

Q) Write the features of Hunting ndash gathering societiesAns - There are five basic characteristics of hunting and gathering societies

i The primary institution is the family which decides how food is to be shared and how children are to be socialized and which provides for the protection of its members

ii They tend to be small with fewer than fifty members

iii They tend to be nomadic moving to new areas when the current food supply in a given area has been exhausted

iv Members display a high level of interdependence

v Labor division is based on sex men hunt and women gather

Political Science

Introduction to political science

Political science occasionally called politology is a social science which deals with systems of governance and the analysis of political activities political thoughts associated constitutions and political behaviorThe study of political science involves the study of both the

Answer the following questions-1 What is political science

Political science occasionally called politology is a social science which deals with systems of governance and the analysis of political activities political thoughts associated constitutions and political behavior

2 Short notes-

traditional and modern theories of politicsTraditionalClassical political sciencepolitical theory-Traditional political science is the study of politics before Second World War The methodology to study Politics was traditional (legal formaletc) the definition of politics traditional (Politics begins and end with state)area of study (constitution state machinery)was traditionalModern Political scienceModern political theory-Modern Political Theory critically examines the contemporary state of political theory making an assessment of the achievement and limitations of the Behavioural Revolution in its totality and reviews objectively the major paradigms and conceptual frameworks adopted by the disciplineContemporary attempts at the development of an integrated political theory involving the use of both traditional and modern concepts approaches and theories-Around late 1960s several political scientists realized the importance of both the traditional political theory and modern Political theory They began building an integrated theory of politics involving a systematic mixture of traditional and modern studies of politics It was held that the study of a complex and vast field like politics needs both traditional as well as

Classical political theory Modern Political theory

Homework-Learn

modern concepts and approaches for studying itrsquos all aspects

Subject Eng Literature (The Tempest ndash William Shakespeare) Topic Act I Scene 1 Lines 1 to 32 (Line 32 ndash Gonzalo hellip If he be not born to be hanged our case is miserable) Date 13th April 2020 (3rd Period)

[Students should read the original play and also the paraphrase given in the school prescribed textbook]Summary Questions amp Answers

[SUMMARY OF THE ENTIRE SCENE]

o The play starts with the scene of a severe storm at sea Alonso (King of Naples) Sebastian (Alonsorsquos brother) Ferdinand (Alonsorsquos son) Gonzalo Antonio (the usurping Duke of Milan) are in a ship in the midst of the storm

o The mariners are trying their best to control the vessel from running aground and are totally following the orders of their Master the Boatswain They have scant success

o The mariners become extremely unhappy and annoyed when most of the passengers arrive on the deck thereby hampering their effort to save the ship There is serious confrontation between them and the passengers who are part of the Kingrsquos entourage

o The mariners could not save the ship

SUMMING-UP

(i) Vivid description of the scene which gives a realistic description of terror and confusion of a tropical storm

(ii) Shows Shakespearersquos accuracy of knowledge in describing the naval operations and also matters of seamanship

(iii) The opening scene justifies the title ndash The Tempest

UNANSWERED QUESTIONS

(i) The King always travels with his entire fleet including his soldiers Where

(1) GONZALO Nay good be patient (Line 15-26)BOATSWAIN When the sea is Hence What cares these

roarers for the name of the king To cabin silence Trouble us not

GONZALO Good yet remember whom thou has aboardBOATSWAIN None that I more love than myself You are a

councillor if you can command these elements to silence and work

the peace of the present we will not hand a rope more use your authority If you cannot give thanks you have

lived so long and make yourself ready in your cabin for the mischance of the hour if it so hap [To the Mariners]

Cheerly good hearts [To Gonzalo] Out of our way I say

(a) To whom is the boatswain speaking What does he mean by lsquoNone that I more love than myselfrsquo

The Boatswain is speaking to Gonzalo the honest old councilor of the Duke of MilanBy using the words ndash lsquoNone that I love more than I love myselfrsquo means that for the Boatswain nobody is dearer to him than his own life

(b) What were the conditions that made the boatswain react in this way

The Boatswain reacts in this way because the storm is at sea and Alonso King of Naples Sebastian his brother Ferdinand his son Gonzalo Antonio the usurping Duke of Milan on board are in distress and in panic Thus they have rushed to the deck interrupting the work of the mariners

(c) What hope does Gonzalo take from the attitude of the boatswain

The insolent and authoritative attitude of Boatswain makes Gonzalo feel comforted He tells that there are no signs that the Boatswain will be drowned But his facial appearance and attitude shows that he is destined to die on land by hanging which in effect means that all on board will be saved Otherwise all the persons on board are doomed

(d) How can they lsquomake yourself ready in your cabinrsquo For what were they asked to make ready themselves

In order to make themselves ready in their cabin the

were the other ships

(ii) Why was the ship in that area Where was it coming from or going where

(iii) The ship broke apart What happened to those who were in the ship

passengers on board must prepare for death which they will possibly soon have to meetThey can retire to their cabins and offer prayers to the Almighty to save them from drowning

(e) What does the boatswain say when he is asked to be patient What does he order to the royal party

When the boatswain is asked to be patient and remain calm he says that he will be patient only when the storm will be over and the sea will be calm but as long as the storm blows and there is danger to the ship he cannot think of being patient He orders the royal party to go to the cabin and leave the mariners to their work

(2) GONZALO I have great comfort from this fellow (Line 27-36)

Methinks he hath no drowning mark upon him his complexion is perfect

gallows Stand fast good Fate to his hanging Make the rope of his destiny our cable for our own doth little advantage If he be not born to be hanged our case is miserable

(a) Why does Gonzalo regard the Boatswain in the midst of danger

In the midst of danger Gonzalo regards the boatswain because he feels that the Boatswain is a source of comfort and is bent upon to do his work sincerely which in this case is saving the ship and its passengers from the severest of raging storm

(b) What reasons does Gonzalo give when he says that none in the ship will die of drowning

Gonzalo is almost sure that none in the ship will die by drowning His says that there is no mark on the face of the boatswain that indicates that he will die by drowning On the other hand the lines on his face are strong indications that he will be hanged to death Therefore there shall be no danger of the shiprsquos sinking

(c) Explain the following ldquoStand fast good Fate to his hanging Make the rope of his destiny our cable for our own doth little advantage If he be not born to be hanged our case is miserablerdquo

The stated lines mean that if the will of destiny is to be carried out then the ship will not get wrecked and all the passengers will be saved The safety of the passengers therefore depends upon the will of fate being carried out in the case of the boatswain If however the boatswain is not to die by hanging then the passengers are also very unsafe because in that case the ship is likely to sink

(d) What order does the Boatswain give to the sailors

when he re-enters What does he say about the crying of the fellows inside the cabin

The boatswain orders the sailors to bring the topmast lower and bring the ship close to a stationary position with the help of the main sail He says that the fellows inside the cabin are moaning and crying in their distress louder than his voice and louder even than the roaring of the storm

Class XII (ScienceCommerceHumanities) Subject Topic Summary Execution

Computer Science

PropositionalLogic

Propositional logic is a procedure to provide reasoning through statementProposition A ststement that results in True or False is said to be proposition There are two types of propositionSimple proposition amp compound propositionSimple proposioton A simple proposition is one that is not a part of any other proposition Such sentential form of proposition is symbolized with english letters in short For example Ram is a claver student (TrueFalse)Where do you live (Not in True or False)Grapes are sweet (TrueFalse)It rains today (TrueFalse)Here we can see some statements anwer would be true or false but some staements answer can not give in terms of true or false Thus the sentences which can be answered in true or false are known as simple propositionAssigning propositon to a variableThe general syntax to assign propostion to a variable is as followsVariable = Simple propositonFor example A=Ram is a clever studentB= Grapes are sweetC= it rains todayCompound proposition

helliphellipto be continued in next classhelliphellipMath Relation Relation If A and B are two non-empty sets

then a relation R from A to B is a subset of AxB If R A x B and (a b) R then we say that a sube isinis related to b by the relation R written as aRbeg Let A be the set of students of class XII and B be the set of students of class XI Then some of the examples of relation from A to B arei) (a b) AXB a is brother of bisinii) (a b) AXB age of a is more than age of isinb Types of relation In this section we would like to study different types of relations We know that a relation in a set A is a subset of A times A Thus the empty set φ and A times A are two extreme relations For illustration consider a relation R in the set A = 1 2 3 4 given by R = (a b) a ndash b = 10 This is the empty set as no pair (a b) satisfies the condition a ndash b = 10 Similarly R = (a b) | a ndash b | ge 0 is the whole primeset A times A as all pairs (a b) in A times A satisfy | a ndash

Example 1 Let A be the set of all students of a boys school Show that the relation R in A given by R = (a b) a is sister of b is the empty relation and R = (a b) the primedifference between heights of a and b is less than 3 meters is the universal relationSolution Since the school is boys school no student of the school can be sister of any student of the school Hence R = φ showing that R is the empty relation It is also obvious that the difference between heights of any two students of the school has to be less than 3 meters This shows that R = A times A is primethe universal relation Example 2 Show that the relation R in the set 1 2 3 given by R = (1 1) (2 2) (3 3) (1 2) (2 3) is reflexive

b | ge 0 These two extreme examples lead us to the following definitionsDefinition 1 A relation R in a set A is called empty relation if no element of A isrelated to any element of A ie R = φ A times AsubDefinition 2 A relation R in a set A is called universal relation if each element of A is related to every element of A ie R = A times A Both the empty relation and the universal relation are some times called trivial relation Definition 3 A relation R in a set A is called(i) reflexive if (a a) R for every a Aisin isin(ii) symmetric if (a1 a2) R implies that (aisin 2a1)

R for all aisin 1 a2 Aisin(iii) transitive if (a1 a2) R and (aisin 2 a3) R isinimplies that (a1 a3) R for all aisin 1 a2 a3 AisinDefinition 4 A relation R in a set A is said to be an equivalence relation if R is reflexive symmetric and transitive

but neither symmetric nor transitiveSolution R is reflexive since (1 1) (2 2) and (3 3) lie in R Also R is not symmetric as (1 2) R but (2 1) isin notinR Similarly R is not transitive as (1 2) R and (2 3) R but (1 3) R isin isin notinExample 3 Show that the relation R in the set Z of integers given byR = (a b) 2 divides a ndash b is an equivalence relationSolution R is reflexive as 2 divides (a ndash a) for all a Z isinFurther if (a b) R then 2 divides a isinndash b Therefore 2 divides b ndash a Hence (b a) R which shows that R is isinsymmetric Similarly if (a b) R and (b c) R isin isinthen a ndash b and b ndash c are divisible by 2 Now a ndash c = (a ndash b) + (b ndash c) is even (Why) So (a ndash c) is divisible by 2 This shows that R is transitive Thus R is an equivalence relation in ZExample 4 Let L be the set of all lines in a plane and R be the relation in L defined as R = (L1 L2) L1 is perpendicular to L2 Show that R is symmetric but neither reflexive nor transitiveSolution R is not reflexive as a line L1 can not be perpendicular to itself ie (L1 L1) R notinR is symmetric as (L1 L2) Risin

L1 is perpendicular to L2rArr L2 is perpendicular to L1rArr (L2 L1) RrArr isin

R is not transitive Indeed if L1 is perpendicular to L2 and L2 is perpendicular to L3 then L1 can never be perpendicular to L3 In fact L1 is parallel to L3 ie (L1 L2) R isin(L2 L3) R but (L1 L3) Risin notin

Chemistry Solid state Characteristics if Solids(i)The particles are locked in fixed positions they are unable to change their relative positions and this brings a definite shape and volume of a solid(ii)In a solid the constituent particles are held by strong forces of attractionThe forces of attraction may be bonding or non bonding(iii)The constituent particles in a solid pack together as closely as possibleoccupying most of the available space within the solidThus the empty space in a solid is very smallThis makes a solid highly rigid and nearly incompressibleThis also explains why a solid has high density and exhibits slow diffusionClassification of Solids

Q1)Define Crystalline solids AnsA Solid that has a definite geometrical shape and a sharp melting pointand whose constituent particles (atomsmolecules or ions) are arranged in a long range order of definite pattern extending throughout the solid is called a crystalline solidExNaClQ2)Define Amorphous solids AnsA solid that does not have a definite shape and a sharp melting pointand whose constituent particles (atomsmolecules or ions) are not arranged in a definite pattern is called an amorphoussolid

Crystalline solidsAmorphous solids

ExGlassRubberQ3)Classify Crystalline Solids Crystalline Solids

Physics Coloumbrsquos Law (Summary)

Before Going Into Coloumbrsquos Law We Will First Learn What is Charge Properties of Charge and Always remember that charge is quantized ie a body always have static charge of magnitude equal to some integral multiple of fundamental electronic charge e= 16 x 10- 19 C

Charge is the property of matter that causes it to produce and experience electrical and magnetic effects The study of the electrical charges at rest is called electrostatics When both electrical and magnetic effects are present the interaction between charges is referred to as electromagnetic

There exist two types of charges in nature positive and negative Like charges repel and unlike charges attract each other

The type of charge on an electron is negative The charge of a proton is the same as that of an electron but with a positive sign In an atom the number of electrons and the number of protons are equal The atom is therefore electrically neutral If one or more electrons are added to it it becomes negatively charged and is designated as negative ion However if one or more electrons are removed from an atom it becomes positively charged and is called a positive ion

The excess or deficiency of electrons in a body gives the concept of charge If there is an excess of electrons in a body it is negatively charged And if there is deficiency of electrons the body becomes positively charged Whenever addition or removal of electrons takes places the body acquires a charge

The SI Unit of charge is coulomb (C) In SI units the current is a fundamental quantity having a unit of ampere (A) The unit of charge is defined in terms of the unit of current Thus one coulomb is the charge transferred in one second across the section of a wire carrying a

Ionic SolidsMetallicSolids

Molecular Solids

current of one ampere

As q = It we have1 C = (1 A) (1 s)

The dimensions of charge are [A T]

Properties of Charge

(1) Quantization of Charge Electric charge can have only discrete values rather than any value That is charge is quantized The smallest discrete value of charge that can exist in nature is the charge on an electron given as

e = plusmn 16 x 10- 19 C

This is the charge attained by an electron and a protonA charge q must be an integral multiple of this basic unit That is

Q = plusmn ne where n = 1 2 hellip

Charge on a body can never be (frac12)e (23)e or 57e etcWhen we rub a glass rod with silk some electrons are transferred from the rod to the silk The rod becomes positively charged The silk becomes negatively charged The coulomb is a very large amount of charge A typical charge acquired by a rubbed body is 10 - 8 C

Biology Reproduction in organisms

Welcome to this new session 2020-21Today in this first chapter we mainly discuss about reproduction types needs and life span of some organismsWe also discuss about difference between sexual and asexual reproduction

Q1 What is reproductionReproduction is defined as a biological processin which an organism gives rise to young onessimilar to itselfQ2 What are the needs of reproductionbulli) Reproduction maintain life on earthii) It enables the continuity of the species generation after generationiii) It creates genetic variation among populationsQ3 Define Life span and write some orgnisms life spanbull Life span is the period from birth to

the natural death of an organism- OrganismsLife span1 Butterfly 1 - 2 weeks2 Fruit fly 30 days3Dog 10-13 years4 Rose5-7 years5 Tortoise100-150 years6 Banyan Tree -200 - 250 yearsQ4 Reproduction is of two types in case ofanimals but in case of plants vegetative propagation is also present

Asexual Reproduction Sexual Reproductioni) Always uniparentalii) Gametes are not involvediii) Only mitotic division involvediv) Somatic cells of parents are involvedv) Offsprings are genetically similar to the parents

i) Usually biparentalii) Gametes are involvediii) Meiosis occurs during gametogenesis Mitosis occurs after fertilisationiv) Germ cells of the parents are involvedv) offsprings are genetically different from the parents

COMMERCE BUSINESS ENVIRONMENT

Welcome to the new sessiontoday we are going to start the first chapter of Class XII The name of the chapter is Business Environment

Already many of you have got some idea about the word business environment form the first chapter of business studies in class XI

In todayrsquos world every business enterprise is a part of the society It exists and operates in association with various groups in society such as customers suppliers competitors banks and financial institutions government agencies trade unions media and so on All these groups influence the functioning of business in one way or the other They constitute the environment of businessConcept of Business Environment

The term lsquobusiness environmentrsquo refers to the sum total of all individuals institutions and other forces that lie outside a business enterprise but that may influence its functioning and performance

The main features of business environment Totality of External forces General and Specific forces Interrelatedness Complexity Dynamic Uncertainty

Prepare the following questions from todayrsquos assignment

2 What do you mean by business environment

The term lsquobusiness environmentrsquo means the aggregate of all forces factors and institutions which are external to and beyond the control of an individual business enterprise but they may influence its functioning and performance Business environment is the macro framework within which a business firm a micro unit operates It consists of several interrelated and interacting elements

2 Explain the main features of business environment in brief

Totality of External forces-Business environment is the sum total of all things external to a business environment

General and Specific forces-It

Relativity

The Interrelation between business and its environment

The business enterprise is an open system It continuously interacts with its environment It takes inputs (such as raw materials capital labour energy and so on) from its environment transforms them into goods and services and sends them back to the environment

Fig 1 Business Environment Relationship

includes both the forces general forces are the economic social political legal and technological conditions which indirectly influence all business enterprise Specific forces are the investors customers competitors and suppliers which influence individual enterprise directly

Interrelatedness-Different elements of environment are interrelated for an example growing awareness for health care has increased the demand for health foods

Complexity- Business environment id complex in nature as the elements keep on changing example economic technological and other forces changes in demand for a product and service

Dynamic-Business environment is not static it keeps on changing

Uncertainty- Itrsquos very difficult to predict future events such as technology and fashion which occur fast and frequently

Business Studies

Human Resources Management

Human resource of an organisation are the aggregate of knowledge skills attitudes of people working in it

The management system which deals with human resources is called human resource management

Features of HRMbullComprehensive functionbullPeople-oriented

Question1) What do you mean by human

resource management Answer) Human resource management may be defined as that field of Management which has to do with planning organising and controlling the functions of procuring developing maintaining and utilising the labour force

bullAction oriented bullPervasive function bullContinuous function

2) Explain the features of HRM in brief

Answer)bullHuman Resource Management is concerned with managing people at work bull Human Resource Management is concerned with employees which bring people and organisations together so that the goals of each are met bullHuman resource management considered every employees as an individual and also promote their satisfaction and growth bull Human resource management is inherent in all organisations and at all levelsbullManagement of human resources are ongoing on never ending process which requires a constant alertness and Awareness of human relations

3) ldquoHR function is said to be pervasiverdquowhy

Answer) Human resource management is required in all organisations whether it is private or government organisations armed forces sports organisations etc It permeatsall the functional areas like production marketing finance research etc This from this feature of human resource management it can be said that it is pervasive in nature

Economics Demand Q1DEFINITION OF DEMANDIn economics demand is the quantity of a good that consumers are willing and able to purchase at various prices during a given period of timeQ2DEMAND CURVEIn economics a demand curve is a graph depicting the relationship between the price of a certain commodity and the quantity of that commodity that is demanded at that pricQ3LAW OF DEMANDIn microeconomics the law of demand states that conditional on all else being equal as the price of a good increases quantity demanded decreases conversely as the price of a good decreases quantity demanded increasesQ4ASSUMPTION of LAW OF DEMAND(i)No change in price of related commodities(ii) No change in income of the consumer(iii) No change in taste and preferences customs habit and fashion of the consumer( No expectation regarding future change in priceQ5MARKET DEMAND SCHEDULEIn economics a market demand schedule is a tabulation of the quantity of a good that all consumers in a market will purchase at a

given price At any given price the corresponding value on the demand schedule is the sum of all consumersrsquo quantities demanded at that priceQ6INDIVIDUAL DEMAND SCHEDULEIndividual demand schedule refers to a tabular statement showing various quantities of a commodity that a consumer is willing to buy at various levels of price during a given period of timeQ7 FACTORS AFFECTING INDIVIDUAL DEMAND FOR A COMMODITY

The factors that influence a consumerrsquos decision to purchase a commodity are also known as determinants of demand The following factors affect the individual demand for a commodity1 price of the commodity2 price of related goods3 income of buyer of the commodity4 tastes and preferences of the buyer1 Price of the CommodityYou must have observed that when price of a commodity falls you tend to buy more of it and when its price rises you tend to buy less of it when all other factors remain constant (lsquoother things remaining the samersquo) In other words other things remaining the same there is an inverse relationship between the price of a commodity and its quantity demanded by its buyers This statement is in accordance with law of demand which you will study in the later part of this lesson Price of a commodity and its quantity demanded by its buyers are inversely related only when lsquoother things remain the samersquo So lsquoother things remaining the samersquo is an assumption when we study the effect of changes in the price of a commodity on its quantity demanded2 Price of Related goodsA consumer may demand a particular good But while buying that good heshe also asks the price of its related goods Related goods can be of two types-(i) Substitute goods(ii) Complementary goods While purchasing a good prices of its substitutes and complements do affect its quantity purchased(i) Price of Substitute Goods Substitute goods are those goods which can easily be used in place of one another for satisfaction of a particular want like tea and coffee An increase in price of substitute good leads to an increase in demand for the given commodity and a decrease in price of substitute good leads to a decrease in demand for the given commodity It means demand for a given commodity is directly affected by change in price of substitute goods For example if price of coffee increases the demand for tea will rise as tea will become relatively cheaper in comparison to coffee(ii) Price of Complementary goods Complementary goods are those goods which are used together to satisfy a particular want like car and petrol An increase in the price of complementary goods leads to a decrease in demand for the given commodity and a decrease in the price of complementary goods leads to an increase in demand for the given commodity For example if price of petrol falls then the demand for cars will increase as it will be relatively cheaper to use both the goods together So demand for a given commodity is inversely affected by change in price of complementary goods3 Income of the Buyer of CommodityDemand for a commodity is also affected by income of its buyer However the effect of change in income on demand depends on the nature of the commodity under consideration In case of some goods like full cream milk fine quality of rice (Basmati rice) etc demand for these commodities increases when income of the buyer increases and

demand for these commodities decreases when income of the buyer decreases Such goods whose demand increases with the increase in income of the buyer are called normal goods But there are some goods like coarse rice toned milk etc whose demand decreases when income of buyer increases and their demand increases when income of the buyer decreases Such goods whose demand decreases with the increase in income of the buyer are called inferior goods Suppose a consumer buys 10 Kgs of rice whose price is ` 25 per Kg He cannot afford to buy better quality of rice because the price of such rice is ` 50 per Kg The consumer is spending ` 250 per month on the purchase of rice Now if income of the consumer increases and he can afford ` 350 on purchase of 10 Kg of rice Now he can afford to buy some quantity of rice say 6 Kgs whose price is ` 25 per Kg and may buy 4 Kgs of rice whose price is ` 50 per Kg Thus he will buy 10 Kgs of rice by spending ` 350 per month Therefore we may conclude that demand for normal goods is directly related to the income of the buyer but demand for inferior goods is inversely related to the income of the buyer4 Tastes and Preferences of the BuyerThe demand for a commodity is also affected by the tastes and preferences of the buyers They include change in fashion customs habits etc Those commodities are preferred by the consumers which are in fashion So demand for those commodities rises which are in fashion On the other hand if a commodity goes out of the fashion its demand falls because no consumer will like to buy it(5) Number of Buyers in the Market(Population)Increase in population raises the market demand whereas decrease in population reduces the market demand for a commodity Not only the size of population but its composition like age (ratio of males females children and old people in population) also affects the demand for a commodity It is because of needs of children young old male and female population differs(6) Distribution of Income and WealthIf the distribution of income and wealth is more in favour of the rich demand for the commodities preferred by the rich such as comforts and luxuries is likely to be higher On the other hand if the distribution of income and wealth is more in favour of poor demand for commodities preferred by the poor such as necessities will be more(7) Season and Weather ConditionsThis is generally observed that the demand for woolens increases during winter whereas demand for ice creams and cold drinks increases during summer Similarly market demand for umbrellas rain coats increases during rainy seasonQ8 REASONS FOR OPERATION OF LAW OF DEMAND WHY DEMAND CURVE SLOPES DOWNWARDNow we will try to explain why does a consumer purchase more quantity of a commodity at a lower price and less of it at a higher price or why does the law of demand operate ie why does the demand curve slope downwards from left to right The main reasons for operation of law of demand are1 Law of Diminishing Marginal UtilityAs you have studied earlier law of diminishing marginal utility states that as we consume more and more units of a commodity the utility derived from each successive unit goes on decreasing The consumer will be ready to pay more for those units which provide him more utility and less for those which provide him less utility It implies that he will purchase more only when the price of the commodity falls2 Income Effect

When price of a commodity falls purchasing power or real income of the consumer increases which enables him to purchase more quantity of the commodity with the same money income Let us take an example Suppose you buy 4 ice creams when price of each ice cream is ` 25 If price of ice creams falls to ` 20 then with same money income you can buy 5 ice creams now3 Substitution EffectWhen price of a commodity falls it becomes comparatively cheaper as compared to its substitutes (although price of substitutes has not been changed) This will lead to rise in demand for the given commodity For example if coke and Pepsi both are sold at ` 10 each and price of coke falls Now coke has become relatively cheaper and will be substituted for Pepsi It will lead to rise in demand for coke4 Change in Number of BuyersWhen price of a commodity falls some old buyers may demand more of the commodity at the reduced price and some new buyers may also start buying this commodity who were not in a position to buy it earlier due to higher price This will lead to increase in number of buyers when price of the commodity falls As a result demand for the commodity rises when its price falls5 Diverse Uses of a CommoditySome commodities have diverse uses like milk It can be used for drinking for sweet preparation for ice cream preparation etc If price of milk rises its use may be restricted to important purpose only This will lead to reduction in demand for other less important uses When price of milk falls it can be put to other uses also leading to rise n demand for itQ9 EXCEPTIONS TO THE LAW OF DEMANDYou have studied in law of demand that a buyer is willing to buy more quantity of a commodity at a lower price and less of it at a higher price But in certain circumstances a rise in price may lead to rise in demand These circumstances are called Exceptions to the Law of Demand Some important exceptions are1 Giffen GoodsGiffen goods are special type of inferior goods in which negative income effect is stronger than negative substitution effect Giffen goods do not follow law of demand as their demand rises when their price rises Examples of Giffen goods are jowar and bajra etc2 Status Symbol GoodsSome goods are used by rich people as status symbols eg diamonds gold jewellary etc The higher the price the higher will be the demand for these goods When price of such goods falls these goods are no longer looked at as status symbol goods and tehrefore therir demand falls3 NecessitiesCommodities such as medicines salt wheat etc do not follow law of demandbecause we have to purchase them in minimum required quantity whatever their price may be4 Goods Expected to be ScarceWhen the buyers expect a scarcity of a particular good in near future they start buying more and more of that good even if their prices are rising For example during war famines etc people tend to buy more of some goods even at higher prices due to fear of their scarcity in near future

Political Science

Constitution of India-The

Preamble

The preamble-

Preamble-

The preamble is the most precious part of the constitution We the people of India having solemnly resolved to constitute India into a Sovereign Socialist Secular Democratic Republic and to secure to all its citizensA preamble is an introductory and expressionary statement in a document that explains the documents purpose and underlying philosophy When applied to the opening paragraphs of a statute it may recite historical facts pertinent to the subject of the statuteNature and purpose of the constitution-Purpose of the Constitution dictates permanent framework of the government to form a more perfect union to establish justice and ensure peace of thenationconstitution provide principles how the government can run itself following the rules and laws written in the constitution of each state keeps them balanced

Answer the following questions-

1 What is preambleA preamble is an introductory and expressionary statement in a document that explains the documents purpose and underlying philosophy2 What is the nature and

purpose of the constitutionConstitution dictatespermanent framework of the government to form a more perfect union to establish justice and ensure peace of the nation

Homework-Learn

Accounts Compatibilty mode

1MEANING OF PARTNERSHIPPartnership is a form of business organisation where two or more persons join hands to run a business They share the profits and losses according to the agreement amongst them According to the Indian Partnership Act 1932 ldquoPartnership is relation between persons who have agreed to share profits of a business carried on by all or any one of them acting for allrdquo For example one of your friends has passed class XII from National Institute of Open Schooling (NIOS) and wants to start a business Heshe approaches you to join in this venture Heshe wants you to contribute some money and participate in the business activities Both of you if join hands constitute a partnership2CHARACTERISTICS1048698 Agreement A partnership is formed by an agreement The agreement may be either oral or in writing It defines the relationship between the persons who agree to carry on business It may contain the terms of sharing profit and the capital to be invested by each partner etc The written agreement is known as partnership deed1048698 Number of persons There must be at least two persons to form a partnership

The maximum number of partners in a partnership firm can be 50 according toCompanies Act 20131048698 Business The Partnership is formed to carry on business with a purpose of earning profits The business should be lawful Thus if two or more persons agree to carry on unlawful activities it will not be termed as partnership1048698 Sharing Profits The partners agree to share profits in the agreed ratio In caseof loss all the partners have to bear it in the same agreed profit sharing ratio10486981048698Mutual Agency Every partner is an agent of the other partners Every partner can bind the firm and all other partners by hisher acts Each partner will be responsible and liable for the acts of all other partners10486981048698Unlimited liability The liability of each partner except that of a minor is unlimited Their liability extends to their personal assets also If the assets of the firm are insufficient to pay off its debts the partnersrsquo personal property can be used to satisfy the claim of the creditors of the partnership firm10486981048698Management All the partners have a right to mange the business However they may authorize one or more partners to manage the affairs of the business on their behalf10486981048698Transferability of Share No partner can transfer hisher share to any one including hisher family member without the consent of all other partners3PARTNERSHIP DEEDAgreement forms the basis of partnership The written form of the agreement is which a document of partnership is It contains terms and conditions regarding the conduct of the business It also explains relationship between the partners This document is called partnership deed Every firm can frame its own partnership deed in which the rights duties and liabilities of the partners are stated in detail It helps in settling the disputes arising among the partners during the general conduct of business 4CONTENTS OF PARTNERSHIP DEEDThe partnership deed generally contains the following (i) Name and address of the partnership firm(ii) Nature and objectives of the business(iii) Name and address of each partner(iv) Ratio in which profits is to be shared(v) Capital contribution by each partner(vi) Rate of Interest on capital if allowed(vii) Salary or any other remuneration to partners if allowed(viii) Rate of interest on loans and advances by a partner to the firm(ix) Drawings of partners and interest thereon if any(x) Method of valuation of goodwill and revaluation of assets and liabilities on the reconstitution of the partnership ie on the admission retirement or death of a partner(xi) Settlement of disputes by arbitration(xii) Settlement of accounts at the time of retirement or death of a partner5IN ABSENCE OF PARTNERSHIP DEEDThe partnership deed lays down the terms and conditions of partnership in regard to rights duties and obligations of the partners In the absence of partnership deed there may arise a controversy on certain issues like profit sharing ratio interest on

capital interest on drawings interest on loan and salary of the partners In such cases the provisions of the Indian Partnership Act becomes applicableSome of the Issues are(i) Distribution of Profit Partners are entitled to share profits equally(ii) Interest on Capital Interest on capital is not allowed(iii) Interest on Drawings No interest on drawing of the partners is to be charged(iv) Interest on Partnerrsquos Loan A Partner is allowed interest 6 per annum on the amount of loan given to the firm by himher(v) Salary and Commission to Partner A partner is not entitled to anysalary or commission or any other remuneration for managing the business

History TOPIC-TOWARDS INDEPENDENCE AND PARTITION THE LAST PHASE (1935-1947)

SUB TOPIC-IMPORTANT POLITICAL DEVELOPMENTS ndash GROWTH OF SOCIAL IDEAS

Socialism is a political social and economic philosophyLike in other parts of the world the Russian revolution of 1917 served as a great inspiration for revolutionaries in India who at that time were engaged in the struggle for liberation from British ruleSocialist ideas led to the formation of communist party of IndiaJAWAHARLAL NEHRU Among the early Congress leaders Jawaharlal Nehru was very much impressed and influenced by the Socialist ideas He also learnt about the Economic activities of the Soviet Union after the Bolshevic Revolution 1917 He made full use of them in IndiaThe election of Jawaharlal Nehru and Subhas Chandra Bose showed the Left wing tendency within CongressJawaharlal Nehru demanded economic freedom along with political freedom of the people in order to end the exploitation of masses

Nehrus working committee included three socialists leaders The Lucknow session was a landmark in the evolution of socialist ideas of the congressSUBHAS CHANDRA BOSE ndash Subhas Chandra Bose had socialist leaning Both Jawaharlal Nehru and Subhas Chandra Bose were known as leftist Congress men Later on National Congress divided into Leftist and rightist campCONGRESS SOCIALIST Within the Congress some leaders formed the Congress Socialist partyPattavi Sitaramyya Sardar Patel Rajendra Prasad had hostile attitude towards the Congress Socialist partyJawaharlals attitude was hesitant

1 QUESTION ndash Mention name of two Congress leaders who had socialist leaning

1ANSWER ndash Subhas Chandra Bose and Jawaharlal Nehru2QUESTION- In which session of the congress Jawaharlal elaborated his Socialist ideas2 ANSWER ndash Lucknow and Faizpur Session in December 1935 and 19363QUESTION ndash Why Congress was sharply divided into leftist and rightist camp 3ANSWER ndash Subhas Chandra Bosersquos attempt to seek re election for congress presidentship in 1939sharply divided the National Congress into Leftist and Rightist camp4 QUESTION ndash Who was MN Roy 4 ANSWER ndash Manabendra Roy first formed the Communist Party of India outside the country at Tashkent in 19205QUESTION ndash Who formed the Congress Socialist Party within the Congress5 ANSWER ndash Jaya Prakash Narayan Achyut Patwardhan Acharya Narendra Dev Ram Mohan Lohia Aruna Asaf Ali6QUESTION ndash When was the Congress Socialist Party formed What was its object6 ANSWER ndash 1934The Congress Socialist Party sought to work out socialist programme through the Congress They joined hands with the Congress and wanted to carry

Subhas Chandra Bose being expelled from the congress after the Tripuri rift he formed Forward BlockThere were basic differences between the Congress Socialists and the communistsTRADE UNION ACTIVITIES Maximum working class people lived in Bombay and Calcutta The working and living conditions of those workers were very miserable In this situation Shasipada Banerjee NM Lokhande protested against the oppression of the working class peopleThe first Trade Union Madras Labour Union was formed in 1918 by BP WadiaIndustrial strikes took place in Kanpur Calcutta Madras Jamshedpur and Ahmedabad AITUC was formed in Bombay in 1927 The growth of Trade union among the workers was slow because of the fear of the dismissal of the jobIn the mean time the Moderates as well as Communists left AITUC and formed separate organization

on National struggle with the help of workers and peasant class of the society7 QUESTION ndash What was the name of the party founded by Subhas Chandra Bose7 ANSWER- Forward Block8QUESTION ndash Who was Shasipada Banerjee8 ANSWER ndash Shasipada Banerjee was a radical Brahmo He founded a working menrsquos club to protest against exploitation of the British rulers towards the working class of India9 QUESTION ndash What was the weekly published by NM Lokhande9ANSWER- Dinabandhu10 QUESTION ndash Who founded Bombay Mill-Hands Association and in which year10 ANSWER- NM Lokhande in189011 QUESTION- Who was BP WadiaANSWER- BPWadia was the founder of Madras Labour Union in191812 QUESTION- What was the name of the first labour union of India12 ANSWER- Madras Labour Union13 QUESTION Who founded the Majur Mahajan 13 ANSWER GANDHIJI14 QUESTION What was the full form of AITUC When it was formed14 ANSWER All India Trade Union Congressin 192715QUESTION Who formed the Red Trade Union Congress and in which year15ANSWER The Communists formed the Red Trade Union Congress16 QUESTION What do you mean by Socialism16 ANSWER Socialism describes any political and economic theory that says the community rather than individuals should own and manage property and natural resources

Subject Eng Literature (The Tempest ndash William Shakespeare) Topic Act III Scene 3 Lines 1 to 52 (Line 52 ndash Brother my lord the Duke Stand to and do as we) Date 13th April 2020 (4th Period)

[Students should read the original play and also the paraphrase given in the school prescribed textbook]Summary Questions amp Answers

o Alonso Sebastian Antonio Gonzalo Adrian Francisco and others wandered about the island in search of Ferdinand and gets tired and hungry of the toil and at the same time gives up all hope of finding him

o Antonio and Sebastian are happy that Alonso is out of hope and decide to make another attempt on his life that night when being so tired they will be sleeping soundly

o Suddenly a solemn and strange music is heard in the air and several strange shapes enter bringing in a banquet These strange shapes then dance round it with gestures of salutation and then inviting the King to eat they depart

o Seeing this strange scene all are inclined to believe the tales told by travelers that there truly are ldquounicornsrdquo and ldquothe phoenixrsquo thronerdquo

1 ALONSO What harmony is this My good friends hark (L18-27)

GONZALO Marvellous sweet music

[Enter several strange shapes bringing in a banquet

they dance about it with gentle actions of salutation

and inviting the King and his companions to eat they depart]ALONSO Give us kind keepers heavens What were theseSEBASTIAN A living drollery Now I will believe

That there are unicorns that in Arabia

There is one tree the phoenixrsquo throne one phoenix

At this hour reigning thereANTONIO Ill believe both

And what does else want credit come to me

And Ill be sworn rsquotis true Travellers neer did lie

Though fools at home condemn rsquoem

(a) How did Prospero present an amazing spectacle before Alonso and his companions

Using his magic powers Prospero ordered strange shapes to lay a banquet before Alonso and his companions The shapes brought several dishes with tasty eatables in them They placed the dishes on a table before Alonso and his companions Then the strange shapes began to dance gracefully around the banquet While dancing they made gestures inviting them to eat the food Then suddenly the shapes disappeared(b) Who were the guests at the strange banquet Describe the lsquoliving drolleryrsquo

Alonso Sebastian Antonio Gonzalo Adrian and Francisco were the guests at the strange banquet

The term ldquoliving drolleryrdquo refers to live entertainment show In this context when Alonso the King of Naples Sebastian his brother Antonio the treacherous brother of Prospero Gonzalo the kind and loyal councillor to the King Adrian and Francisco came to the island they were hungry and weary in their spirits They heard a solemn and strange music They were shocked to see several strange shapes bringing in a banquet and these shapes danced about it with gentle action of salutation inviting the King and his companions to eat After this Sebastian described this show as lsquoliving drolleryrsquo(c) What is lsquophoenixrsquo What are lsquoUnicornsrdquo

The term lsquophoenixrsquo refers to a mythical Arabian bird which lived alone and perched on a solitary tree After one hundred years it expired in flames and rose again from its own ashes

lsquoUnicornsrsquo refers to the mythological four-footed beasts having horns in the centre of their foreheads When the horns are ground into powder the powder was believed to be

an aphrodisiac(d) How does Sebastian explain the puppet show OR Why does the speaker now believe in unicorns and phoenix

Sebastian finds several strange shapes bringing in the banquet They invite the king and his party for dinner and soon depart He tells that if such a strange sight can be a reality there is nothing incredible in the world and from the present moment he will believe anything He says that it is a strange dumb show enacted not by puppets but by living beings It is stranger than a travellerrsquos tale Seeing such a thing

before his own eyes he will no longer disbelieve the story about unicorns and phoenix(e) How do the other characters present respond to this living drollery

At the sight of the lsquoliving drolleryrsquo like Sebastian Gonzalo and Antonio too acted strangely Antonio told that he too now believes in unicorns and phoenix and anything else that seems to be incredible He too now believes in travellersrsquo tales Gonzalo told that if he would report those happenings in Naples nobody will believe him He considers that those gentle shapes were gentler in manner in comparison to the living beings Alonso was at first sight suspicious and told them that those strange shapes conveyed their meaning in expressive gestures when they seemed to lack speech by their movements and sounds Francisco was amazed at their mysterious disappearance

2 ALONSO Not I

(Line 43-52)GONZALO Faith sir you need not fear When we

were boysWho would believe that there were mountaineers

Dewlapped like bulls whose throats had hanging at rsquoem

Wallets of flesh Or that there were such men

Whose heads stood in their breasts Which now we find

Each putter-out of five for one will bring us

Good warrant ofALONSO I will stand to and feed

Although my lastmdashno matter since I feel

The best is past Brother my lord the Duke

Stand to and do as we

(a) How does Alonso respond at the spectacle of the shapes which were sent to them at the instruction of Prospero

After seeing the strange sight of appearing and disappearing of the shapes sent by Prospero to arrange a banquet for them Alonso says that his surprise at having seen those creatures is infinite and he is fully justified in feeling so much surprise He thinks that their shapes their gestures and the sounds they made were indeed amazing Although they do not possess the gift of speech yet they were able to convey their

thoughts by means of their gestures only

(b) What does Prospero say about the views expressed by Alonso regarding the shapes What does Francisco think about the shapesAfter hearing Alonsorsquos views about the shapes Prospero says that this manrsquos praise of the spirits is rather hasty He means to say that Alonso has shown great haste in reaching the conclusion about the shapes Francisco is amazed to see that those shapes disappeared in a mysterious way(c) What does Sebastian ask Alonso to doSebastian tells Alonso that the shapes having disappeared should not matter to them because they have left the eatables behind He asks Alonso to enjoy eating as they are extremely hungry but the king does not accept his offer of enjoying the dishes(d) How does Gonzalo try to dispel Alonsorsquos fear of those strange shapes What kind of references does he give to AlonsoGonzalo says that those who have travelled abroad have reported seeing even stranger sights than these shapes that Alonso and his companions have beheld Hence there is no reason to feel afraid of these shapes Gonzalo further adds that in his younger days he had heard strange stories from travelers and Alonso might have heard similar stories For instance it was said that there existed a certain race of

human beings who had huge lumps of flesh hanging at their throats and who therefore resembled bulls Then Gonzalo tells about a race of human beings whose heads were located at their breasts Gonzalo says that such stories were not believed by most people in those days but now-a-days these stories have become common(e) Explain the following lsquoEach putter-out of five for onersquoEnglish travellers often insured their trips with London brokers Those that went on foreign travels those days used to deposit a certain amount with some firm or company in London before their departure If the travelers failed to return the money was forfeited by the company with which it had been deposited But this money was repaid five-fold if the travelers returned safe and sound In this way a traveler stood a great chance of recovering the entire cost of his

travels(f) Give the explanatory meanings of the following expressions in the context of the above extract (i) Dewlapped (ii) Wallets of flesh

(iii) Putter-out(i) Dewlapped having big lumps of flesh at the necks(ii) Wallets of flesh large masses of flesh looking like bags(iii) Putter-out to invest money before commencing the travel

  • General methods of preparation of hydrogen
  • Chapter Dimensional Analysis (Summary)
    • Properties of Charge
Page 4:   · Web viewSubject. Topic. Summary. Execution. Hindi. व्याकरण. शरीरके अंगो के नाम लिखिए. 1) आँख 2) नाक 3

Pongal Itrsquos a four day long festival celebrated in Tamil Nadu It is celebrated on January 14 every year People cook sweet rice called Pongal

Ans January

3What is PongalAns Pongal is a harvest festival which is celebrated in Tamil Nadu It is a four day long festival

4In which month Pongal is celebratedAns Pongal is celebrated on January 14 every year

SCIENCE SOME PROPERTIES OF WATER

We get water from the nature so water is a natural resource Water is the most commonly found liquid on earth All living things need water to live Life on earth is not possible without water We need water for drinking cooking bathing washing etc

Fill in the blanks [pg no ndash 81]1We get fresh water from rain 2Boiling kills the germs present in water 3 Water changes its state when it is heated and cooled 4Water does not have a definite shape

BENGALI বই ndash বোংো োলিতয পলিরচয়

পোঠ ndash ১ একতোই বঅীীর পরসোততর

১ঠিক উততসর টিক দোও-ক) বস2ো লেোকটির ( লিত চোর পাোচ ) লে4স ) তোরো ব ময় ( কা োসদ োফোয় ঝগ2ো কসর ) গ) বস2ো লেোকটি লে4সসদর বস ( কমস2ো বাো কলিG ) আসত ঘ) বস2ো তোর লে4সসদর ( োটক মযোলিক লিসমো ) লেদোসব বসলি4সো ২ ক) লেোকটির কয়টি লে4স লি4 উঃ পাোচটি লে4স লি4 ) বস2ো লেোকটো লে4সসদর পরসম লিক আসত ব উঃ কলিG আসত বসোগ) বোলিJ বাো3োর পর তোসদর লিক করসত ব উঃ এক-এক কসর বোলিJটি ভোঙোর লেচষটো করসত ব ঘ) লে4সরো লিমসলিমস োকস লিক সব উঃ লে4সরো লিমসলিমস োকস বোইসরর লেোক

HINDI वयाकरण chapter 1 भाषा

BOOK PAGE 10 राजय भाषामहाराषटर मराठी गजरात गजराती पजाब पजाबी बगा बागा लिनिप भाषा दवनागरी-- हिहदीमराठी गजराती रोमन -- अगरजी गरमी -- पजाबी फारसी -- उरद

सही या गत निनशान गाइए-1पतर मौखिक भाषा का एक रप ह( )X 2भाषा क लिखित रप को लिनिप कहत ह( )3 ससार म सभी ोग एक ही भाषा बोत ह(X) 4गजरात म पजाबी बोी जाती ह(X) भाषा क दो

MATHEMATICS CHAPTER 4 SUBTRACTION

Subtraction without DecomposingSubtraction of 3-digit number (without decomposing)

The result obtained after subtraction is called difference

Note While finding the difference we write the greater of the two given numbers on the top and then subtract

Exercise 16Subtract9 409 10 876 11 641 - 103 - 443 - 510 306 433 131

12 829 - 113 716

Write in columns and find the difference14 603 ndash 401 15 840 ndash 610

Solution

14 H T O 15 H T O 6 0 3 8 4 0 - 4 0 1 - 6 1 0 2 0 2 2 3 0

Fill in the place holders17 _ 3 7 18 6 _ 5

- 2 - 6 - 2 4 3 7 2 -- -- 4 ndash

Solution

179 3 7 18 6 8 5 - 2 1 6 - 2 4 3 7 2 1 4 4 2

Class IVSubject Topic Summary Execution

English language

Personal pronouns

The different forms of the three personal pronounsFirst personSingular I me my minePlural we us our oursSecond person Singular thou thee thy thinePlural ye you your yoursThird personSingular he him his her hers she it itsPlural they them theirs their

Pick out the personal pronouns Mention of what kind each (first second or third person)1 You should always try to keep a promise that you have madeYou second person personal pronoun

2 I listened patiently to what he had to say about themI first person personal pronounHe them third person personal pronoun

3 They packed their belongings and left I do not know where they have goneThey their third person personal pronounI first person personal pronoun

Social studies Major landforms on earth

Explained in previous lesson Answer the following questions3 Differentiate between hills and mountainsAns Mountains has a peak whereas hills have rounded topsMountains are steeper than hills whereas hills are lower and less steep4 Give two usefulness of mountainsAns Mountains are storehouse of waterMountains have a rich variety of flora and fauna

MATHEMATICS

Ch 5Subtraction

Estimating the differenceWe have learnt estimation in addition Applying the same rule We estimate the difference

Exercise 15 Example 2

A fruit-seller has 38210 bananas Out of these he sold 21799 bananas Estimate the unsold bananas

Solution Estimating to ten-thousandsTotal bananas = 40000Bananas sold out =- 20000Bananas unsold = 20000

Exercise1 Find the actual and estimated difference by rounding off to the nearest thousands(b) ActualEstimated

16190 - 2979

Solution (b) ActualEstimated 16190 16000 -2979 -30001321113000

2 Find the actual and estimated difference by rounding off to the nearest ten-thousands(b) ActualEstimated 73012 -28790

Solution (b) ActualEstimated 73012 70000 -28790-3000044222 40000

4 A bakery shop has an order to supply 6705 packets of biscuits There are only 4920 packets in the shop Estimate the number of packets needed to complete the supply

Solution Estimating to thousandsEstimated order = 7000Packets available= -5000Packets needed = 2000

there4 Estimated packets needed to complete the order were 2000

Hindi 2ndlang पतर पतर लिना भी एक का ह पतर लित समय धयान दना होगा निक पतर निकस लि रह ह कयोनिक पतर या तो वयवहारिरक होगा या वयावहारिरक होगापतर दो परकार क होत ह

क) औपचारिरक पतर जस परधानाचाय को नगर निनगम सपादक आदिद

ख) अनौपचारिरक पतर जस पापा को दोसत को बहन को अपन परिरजनो को

1 औपचारिरक पतर-अपन निवदयाय म अवकाश क लिए परधानाचाय को पतर लिखिए १०३कसीबीदम दमक टकोकाता-७०००६५सवा मपरधानाचायऋनिष अरहिवदो ममोरिरय एकडमीदिदनाक-१३४२०२०निवषय-अवकाश हत पतरमहोदया जीसनिवनय निनवदन ह निक म आपकी ककषा चौथी का निवदयाथj ह क रात स मझ तज बार ह डॉकटर न मझ आराम करन को कहा हअतः म निवदयाय आन म असमथ ह आपस निनवदन ह निक आप मझ दिदनाक १३स १७ तक की अवकाश दन की कपा कर हम आप क आभारी रहग

धनयवादआपका आजञाकारी लिशषयसीमा सिसहककषा-४

Bengali বইndashবোংো ভোষো পলিরচয়

পোঠndash৮ লিঙগ ৩ পংলিঙগ কোসক বসউঃ পরোলিবোচক লেP লিবসষয পসদর দবোরো পরষ োতীয় কোউসক লেবোঝোয় তোসক পংলিঙগ বস লেPম -বোবো 4োতর৪ সতরীলিঙগ কোসক বসউঃ পরোলিবোচক লেP লিবসষয পসদর তোরো সতরী োতীয় কোউসক লেবোঝোয় তোসক সতরীলিঙগ বস লেPম - মো লিলিকষকো৫ উভয়লিঙগ কোসক বস উঃ পরোলিবোচক লেPলিবসষয পসদর দবোরোপরষ ও সতরীউভয়োলিতসক লেবোঝোয় তোসক উভয়লিঙগ বস লেPম - লিশ মনতরী ৬ কলীবলিঙগ কোসক বস উঃ অপরোলিবোচক লেP লিবসষয পসদর দবোরো সতরী বো পরষ কোউসক ো বলিঝসয় লেকো2 পদো13 লেক লেবোঝোয়তোসক কলীবলিঙগ বস লেPম - বই

COMPUTER CHAPTER 3 EDITING IN MS WORD

MOVING THE TEXTMS WORD ALLOWS US TO MOVE A BLOCK OF TEXT OR GRAPHICS WITHIN A DOCUMENT

Q6) HOW TO WE USE THE COPY AND PASTE OPTIONAns) WE CAN CREATE A DUPLICATE COPY OF THE TEXTTHE STEPS TO COPY AND PASTE THE TEXT AS FOLLOWS----

SELECT THE PART OF THE TEXT THAT WE WANT TO COPY

SELECT THE COPY BUTTON IN THE CLIPBOARD GROUP UNDER THE HOME TAB OR PRESS CTRL + C KEYS

NOW PLACE THE CURSOR AT THE PLACE WHERE THE TEXT NEEDS TO BE COPIED

CLICK THE PASTE BUTTON IN THE CLIPBOARD GROUP UNDER THE HOME TAB OR PRESS CTRL + V KEYS

Science Adaptations in Animals

We find different kinds of animals in our surroundings We find birds flying in the sky monkeys jumping on trees fish in water camels in desert and so on Animals live in their natural homes which are called their habitats The habitat of a camel is a dry sandy desert and that of a tiger is a dense forest So different animals live in different habitats In the world a plant or an animal has to adapt or change itself to suits its surroundings This changes happens over hundreds and thousands of years A change that a living thing undergoes to become better suited to its surrounding is called adaptation

Fill in the blanks [pg no 40]1 Terrestrial animals live on

land 2 Polar bears are found in the

cold polar regions 3 Frogs have webbed feet that

help them to swim 4 Monkeys and koalas are

arboreal animals

Class VSubject Topic Summary Execution

COMPUTER130420

CHAPTER 2 APPLICATIONS OF COMPUTERS

PAGE NO-15C FILL IN THE BLANKS

INPUTS ANIMATION BANKS SOFTWARE SUPER COMPUTERS

D WRITE TRUE OR FALSE TRUE TRUE TRUE TRUE FALSE

Science Chapter 2 - The Skeletal System

Joints A joint is a place where two or more bones are joined with each other

There are two types of joints 1) Fixed joints 2) Movable joints

The movable joints are mainly four types ndash 1) Hinge joint 2) Pivot joint 3) Gliding joint 4) Ball and socket joint

DAnswer these questions

1)What are the functions of the skeleton

Ans ndash The functions of skeleton are -

i The skeleton gives our body shape ii The skeleton provide our body strength iii The skeleton protect our inner organs from outside injuries iv The skeleton gives our body support

2)List the type of movable joint in our body giving one example of each

Ans - The movable joints are mainly four types ndash a Hinge joint

Example of hinge joints are elbows nice fingers and toes

b Pivot joint Example - the joint between the skull and the backbone is the example of pivot joint

c Gliding joint Example of gliding joints

are wrist and ankle joints d Ball and socket joint

Example of ball and socket joints are shoulder joint and hip joint

English language

Transitive and intransitive verb

Pick out the verbs from the following sentences and say whether they are transitive or intransitive (page 21 ex B)1 gave ndash verb Transitive verb2 Has planted- verb Intransitive verb3 Were- verb Intransitive verb4 Asked- verb Transitive verb5 Told- verb Transitive verb

Social studies Conquering distances

Airways The only airline owned by the government is Air India which handles both domestic and international flightsAdvantages of airways Air transport is the fastestIt can access remote areasIt is the best means of transport in case of emergencies

1 Which is the only airline owned by the governmentAns Air India

2 Give two advantages of airwaysAns The two advantages of airways areAir transport is the fastest transportIt can access remote areas

DisadvantageIt is the most expensive of all other means of transport

3 What is the disadvantage of airwaysAns The only disadvantage of airways is that it is the most expensive of all other means of transport

MATHEMATICS

Ch 3Addition and Subtraction

Properties of Addition1 The sum of two numbers does not change when we change their order This property known as Commutative Property of addition

2 The sum of three numbers does not change when we change their grouping This property is called Associative Property of addition

3 The sum of the numbers and zero is the number itself This property is called Identity Property of addition and the integer 0 is called identity

Exercise 11Fill in the blanks1 2730815 + 8319293 = ____ + 27308152 18219 + 1850308 = 1850308 + ____3 (27815 + 85919) + 95985 = (85919 + ____) + 278154 13227 + (25983 + 73607) = (____ + 25983) + 736075 91389 + 0 = ____ + 91389

Solution 1 83192932 182193 959854 132275 0

6 Which of the following are true statements(a) Any number added to zero is zero

(b) The sum of two numbers does not change when we change their order

(c) 1 is the identity element of addition

(d) Given any three numbers their sum does not change when we change their grouping

Solution (a) False(b) True(c) False(d) True

Class VISubject Topic Summary Execution

HISTORY AND CIVICS

CHAPTER 3

MAHAVIRA AND BUDDHA ndash GREAT PREACHERS BUDDHA

Decline of Buddhism1 Revival of the Brahmanical Hinduism ndash Brahmin Scholars like Shankaracharya and Kumarila Bhatta led the revival of Hinduism and established the supremacy of Vedic religion2 Loss of Royal Patronage ndash Gupta period marked the decline of Buddhism as Gupta rulers were followers of Hinduism3 Split in Buddhism ndash division into Hinayana and Mahayana sects and rise of Mahayana sects blurred the line between Hinduism and Buddhism4 Corruption in Buddhist Sangha ndash due to generation of large revenue from large estates Buddhist monks and nuns started living luxurious

Answer the Following 1 During which dynasty Buddhism was split During the reign of Kanishka

2 Name the two sects of Buddhism Mahayana and Hinayana

3 Name two Vedic scholars who led the revival of Brahmanical Hinduism Shankaracharya and Kumarila Bhatta

life in rich monasteries Hence corruption crept in5 Adoption of Sanskrit ndash when Buddhist scriptures began to be written in Sanskrit in place of peoples language like Pali or Prakrit people started drifting away from Buddhism6 The Turkish Invasion ndash As Muslim conquerors invaded India immensely wealthy Buddhist Monasteries and temples were looted and destroyed and Buddhists were persecuted and killed

4 During which period decline of Buddhism began The Gupta Period

ENGLISH 2 The great train journey- Ruskin Bond

The great journey by Ruskin Bond is a story about Suraj who loved trains and wanted to go to places One day while wandering along the railway tracks he enters into a carriage compartment The train suddenly starts moving with him in the compartment and after a journey returns back to the same place from where it had begun The story is about his experience during that journey

4 Answer the following questionsf Who else is in the carriageA ragged hippy with a dirty beard face was in the carriageg Where does Suraj say that he would like to go toSuraj said that he would like to go to England and China and Africa and Greenland He wanted to go all over the worldh What warning does the man give to SurajThe man said Suraj to keep out of sight so that he doesnrsquot get caught by the ticket collectorsiWhen Suraj thinks about his parents for the first time what does he imagines that they will thinkSuraj thought that if he failed to come home that night his parents would think that he had run away or been kidnapped or been involved in an accidentJ What presents does Suraj imagine that he will bring back for his friendSuraj imagines that he would bring an African lion or a transistor- radio for his friend

CHEMISTRY

Chapter 2 ndashElement and Compound

ATOMAn atom can be defined as the smallest constituent particle of an element which showcases independent existence Example Ne OMOLECULEA molecule can be defined as the combinations of two or more atoms which are held together by chemical bonds A molecule is the smallest portion of a substance which showcases all the properties of the substance On breaking down a molecule further we see properties of the constituent elements Example HCl NaCl O2

Answer the following Q3) What is a moleculeAns - A molecule can be defined as the combinations of two or more atoms which are held together by chemical bonds A molecule is the smallest portion of a substance which showcases all the properties of the substance On breaking down a molecule further we see properties of the constituent elements Example HCl NaCl O2

Q4) Which can exist independently ndash atom or moleculesAns ndash Molecules can exist independently

PHYSICS Physical quantities

Guidelines for writing SI units correctly1 The units named after scientists are not written with a capital initial letter For example newton henry watt2 The symbols of the units named after scientist should be written by a capital letter For example N for newton H for henry W for watt3 Small letters are used as symbols for units not derived from a proper name For example m for metre kg for kilogram4 No full stop or other punctuation marks should be used within or at the end of symbols For example 50 m and not as 50 m5 The symbols of the units do not take plural form For example 10 kg not as 10 kgs6 When temperature is expressed in kelvin the degree sign is omitted For example 273 K not as 273o K (If expressed in Celsius scale degree sign is to be included For example 100o C and not 100 C)7 Use of solidus is recommended only for indicating a division of one letter unit symbol by another unit

Fill in the blanks

1) Length and mass are examples of fundamental physical quantities

2) The measurement of a physical quantity consists of two part magnitude and unit

3) A foot consist of 32 inches 4) The unit of temperature in the SI system is

Kelvin

Write true or false Correct the false statements

1) In ancient times cubit was used to measure the mass of an object FalseCorrect statement ndash In ancient times cubit was used to measure the length of an object

2) There are 7 fundamental physical quantities True

symbol Not more than one solidus is used For example m s-1 or m s J K mol or J K-1 mol-1 but not J K mol8 Some space is always to be left between the number and the symbol of the unit and also between the symbols for compound units such as force momentum etc For example it is not correct to write 23m The correct representation is 23 m kg m s-2 and not as kgms-29 Only accepted symbols should be used For example ampere is represented as A and not as amp or am second is represented as s and not as sec10 Numerical value of any physical quantity should be expressed in scientific notationFor an example density of mercury is 136 x 104 kg m-3 and not as 13600 kg m-3

3) Second is the unit of time in both the CGS and MKS systems True

4) The symbol used for a unit is always written in capital letters False Correct statement -The symbol used for a unit is normally written in small letters

Hindi 2nd language

वाकय निवचार भागवत निवचारो को परकट करन वा साथक एव वयवसथिtत शबद समह को वाकय कहत ह वाकय दो परकार क होत ह ndash

1 उददशय- वाकय म जिजसक बार म कछ बताया जाता ह उस उददशय कहत ह जस राधा एक नतकी ह2 निवधय- वाकय म उददशय क बार म बताया जाता ह उस निवधयक कहत ह जस- राधा एक नतकी ह रचना क आधार पर वाकय क तीन भद होत ह ndash१सर वाकय- राम बाजार गया २ सयकत वाकय- राम बाजार गया और वहा जाकर दोसत स मिमा३ मिमशर वाकय- यह वही tान ह जहा उनका बचपन बीता

helliphellipContinue to nextBengali 2nd language

লিZ সবরপ ও সবরলিZ

সবরলিZর লিয়ম - ১ অ-কোর লিকংবো আ-কোসরর পসর অ-কোর লিকংবো আ - কোর োকস উভয় লিমস আ ndashকোর য় এবং ওই আ ndash কোর পব13বস13 Pকত য়

২ ই - কোর লিকংবো ঈ - কোসরর পসর ই - কোর লিকংবো ঈ - কোর োকস উভয় লিমস ঈ - কোর য় এবং ওই ঈ - কোর পব13বস13 Pকত য়

৩ উ - কোর লিকংবো ঊ - কোসরর পসর উ - কোর লিকংবো ঊ - কোর োকস উভয় লিমস ঊ - কোর য় এবং ওই ঊ - কোর পব13বস13 Pকত য়

৪ অ - কোর লিকংবো আ ndash কোসরর পসর ই - কোর লিকংবো ঈ - কোর োকস উভয় লিমস এ - কোর য় এবং ওই এ - কোর পব13বস13 Pকত য়

১ অ + অ = আ ( gtো ) লিম + অচ = লিমোচ সব + অ3ী = সবো3ী অ + আ = আ ( gtো )পদম + আ = পদমো শভ + আলি = শভোলি আ + আ = আ ( gtো )4োয়ো + আবত = 4োয়োবত মো + আতমো = মোতমো আ + অ = আ ( gtো )লিবদযো + অংকোর = লিবদযোংকোর Pো + অ13 = Pো13 ২ ই + ই = ঈ ( gtী )অলিত + ইব = অতীব লিগলির + ইনদর = লিগরীনদর ই + ঈ = ঈ ( gtী )পলির + ইকষো = পরীকষো অলি3 + ঈশবর = অ3ীশবর ঈ + ঈ = ঈ ( gtী )মী + ঈশবর = মীশবর 3ী + ঈ = 3ী ঈ + ই = ঈ ( gtী )রী + ইনদর = রীনদর মী + ইনদর = মীনদর ৩ উ + উ = ঊ ( gt )মর + উদযো = মরদযো কট + উলিকত = কটলিকত উ + ঊ = ঊ ( gt )ঘ + ঊলিম13 = ঘলিম13 লিZ + ঊলিম13 = লিZলিম13 ঊ + ঊ = ঊ ( gt )রP + ঊলিম13 = রPলিম13

৪ অ + ই = এ ( লেgt )র + ইনদর = সরনদর লেPোগ + ইনদর = লেPোসগনদর অ + ঈ = এ ( লেgt )গ + ঈ = গস

র + ঈ = সর আ + ই = এ ( লেgt )Pো + ইষট = Pসষট 3ো + ইনদ = স3নদ আ + ঈ = এ ( লেgt )রমো + ঈ = রসম দবোরকো + ঈশবর = দবোরসকশবর

COMPUTER THE WORLD OF WINDOWS 10

DONE IN THE PREVIOUS CLASS PAGE NO-83A TICK THE CORRECT OPTION BACKGROUND DISPLAY AREA RESTORE THREE

MATHEMATICS Topic ndash NumbersChapter ndash Natural numbers and whole numbers

Study item Properties of whole numbers for subtraction1) Closure property When we do subtraction of two whole numbers we can not get a whole number in all time Example 8 ndash 3 = 5 a whole number 0 ndash 6 = -6 is not a whole numberTherefore the subtraction of two whole numbers is not satisfying closure property2) Commutative property If x and y are two whole numbers then x ndash y is not equal to y ndash xExample If x=16 and y = 7 then x ndash y = 16 ndash 7 = 9Again y ndash x = 7 ndash 16 = - 9 Therefore x ndash y not equal to y ndash x Therefore the subtraction of two whole numbers is not satisfy commutative3) Associative property If x y and z are three whole numbersThen x ndash ( y ndash z ) not equal to ( x ndash y ) ndash z Example If x = 20 y = 10 and z = 6Therefore x ndash (y ndash z ) = 20 ndash(10 ndash 6 ) = 20 ndash 4 = 16(X ndash y ) ndash z = (20 ndash 10) ndash 4 = 10 -4 =6Therefore x ndash(y ndash z) not equal to ( x ndash y) ndash zTherefore subtraction of whole numbers is not satisfying associativity4) Distributive property If x y and z are three whole numbersThen x (y ndash z ) = xy ndash xzAnd (y ndash z)x = yx ndash zxExample If x = 10 y = 6 and z = 4x(y ndash z ) = 10(6 ndash 4 ) = 10times6 ndash 10times4 = 60 ndash 40 = 20( 6 ndash 4 )times 10 = 6times10 ndash 4times10 = 60 ndash 40 = 20Therefore the subtraction of whole numbers is satisfying distributive property5) Existence of identity For any whole number x X ndash 0 = x but 0 ndash x = - x not equal to xThus for subtraction no identity number existsException 0 ndash 0 = 0 so 0 is its own identity for subtraction

Class VIISubject Topic Summary Execution

Hindi 2ndlang वचन जो सजञा शबद निकसी वसत या पराणी क एक या अनक होन का बोध कराया उनह वचन कहत ह जस डका- डकयह दो परकार की होती ह-

क) एकवचन-शबद क जिजस रप स उसक एक होन का बोध हो उस एक वचन कहत ह जस निकताब गमा आदिद

) बहवचन-शबद क जिजस रप स उसक आन ोन का पता च उस बहवचन कहत ह जस डक निकताब निततलियाआदिद

निनमनलिखित शबदो को एकवचन स बहवचन म बदोम- हमजानित- जानितयानारी- नारिरयामिमतर ndashमिमतरोपसतक -पसतकसडक-सडकबोत-बोतनाहर-नहररपए-रपया

Bengali বইndashবোংো োলিতয পলিরচয়

পোঠndash১৪ গলপ - অপর কলপো পর

লেক - লিবভলিতভষ বসনদযোপো3যোয়লেকndash রবীনদর পরবতf বোংো কোোলিতয 3োরোর উসgসPোগয োম পরকলিতসপরমী লিবভলিতভষ বসনদযোপো3যোয় তোর লেীসত লেPম বোসর বোসর লিফসর এসস4 গরোম বোংোর পরকলিতর কো লেতমলি এসস4 গরোমী মো লিচতরগলপndash অপর কলপো গলপোংটি লিবভলিতভষ বসনদযোপো3যোসয়র লিবযোত উপযো পসর পাোচোী লেসক গীত অপ অ13োৎ পসর পাোচোী তো অপর কলপোর লেকনদরীয় চলিরতর এই অংস আমরো পোই বোক অপসক বোক অপ কলপো লিবোী লে দসরর অ গো4 লেদস মোসয়র মস লেোো রপকোর রোসয পোলি2 লেদয় দপরসবো মোসয়র মসর কসর কোীদোী মোভোরত এর করসকষতর Pসjর ব13ো শস তোর মোবীর কস13র পরলিত ব2 মমতো য় আবোরপালিসত বলি13ত Pসjর অমোপত অং লে লিসই মোপত কসর বোলি2র লিপ4স বাো বোগোস লিকংবো উঠোসর লিশমসর কলপো লিবো এোস পরকো লেপসয়স4

১ অপর কলপো গসলপর লেক লেক তোর মপসক13 লেসো২ অপর কলপো গলপটি লেকোো লেসক গীত গলপটির ম ভোব লেসো

GEOGRAPHY CHAPTER 7EUROPE

CHAPTER COMPLETE EXERCISEFill in the blanks1 Europe is a continent that comprises the western part of Eurasia2 Eurasia and Africa are connected into one large land mass known as Afroeurasia3 The Strait of Gibraltar separates Europe and Africa4 Europe is surrounded by the Arctic Ocean to the north5 The British Isles includes the island countries of Great Britain and Ireland

Name the following 1 Connects Africa to Eurasia - Isthmus of Suez2 Largest country in the world in terms of area ndash Russia3 A term used collectively for the five countries in northern Europe ndash Nordic Countries4 The capital of Montenegro - Podgorica5 the largest fjord in Norway ndash Sognefjord

Match the following Column 1 Column 2a Albania iii Tiranab Belgium i Brusselsc Denmark v Copenhagend Finland ii Helsinkie Hungary iv Budapest

CHEMISTRY Chapter 2 ndashElement and Compound

Atom - An atom is the basic unit of an element or the smallest particle of an element non capable of independent existence Atom is built up of three sub atomic particles electron proton and neutron

Nucleus-It is the centre of an atom In the centre of the atom contains proton (positively charged particles ) and neutrons ( particles carrying no charge )

Orbits- It surround the nucleus in which revolve electrons (negatively charged particles)

Answer the following

1) What are MetalloidsAns - Certain elements using properties of both metal and non-metals are called metalloids Example Silicon arsenic and antimony

2) What are Noble gasesAns - Certain elements are present in the air and are chemically inert or unreactive Such elements are called rare gases or noble gases Example helium neon argon and Krypton

English 2 Sentences based on meanings

Kinds of sentences

Assertive or declarative to convey information or simply make a statement

Interrogative to ask different types of questions

Imperative to command or instruct someone or make a request

Exclamatory to express strong feelings and emotions

Exercise B1 Stop it ( Exclamatory)2 May you always be happy

together ( Exclamatory)3 He does not like sports

( Assertive)4 Please pass me the salt

( Imperative)5 How dare she talk to me like

that ( Exclamatory)6 May success bless your effort

( Exclamatory)7 Canrsquot you wait for sometime

(Interrogative)8 Did anybody tell you about it

( Interrogative)9 I saw her waiting for the bus

( Assertive)10 Could you please take a

message for me ( Interrogative)

Homework Ex ABiology Chp -2

Classification of Plants

Today we discuss about usefulness of bacteria We also discuss what the harmful effects of bacteria are

89 How bacteria are useful for usbull Bacteria is helpful in many ways forhuman being i) Production of medicine - antibiotics vaccine etcii) Formation of curd by lactobacillusiii)Nitrogen fixation in Leguminousplant by Rhizobiumiv) Increase soil fertility by absorbingatmospheric nitrogen and convert it into nitrates and nitritesv) Cleaning the environment by converting the complex substances into simple substancesvi) Tanning of leathervii) Retting of Fibersviii) Formation of compost by acting onanimal dung and agricultual cases1x) Biogas production by decomposingplant and animal wastex)Help In Nutrition by producing vitamiacutemBand kx1) Some bacteria are used to give specialflavour to tea coffee and coccaQ10- Name some diseases and there causativebacteriabull Diseasescausative bacteria1 CholeraVibrio cholerae2 Tuberculosis - Mycobacterium tuberculosis3 Diptheria -Corynebacteriumdiphtheriae4 Pneumonia - Streptococcus pneumoniae

Math Number system

Chapter Fraction

Study item Using lsquoofrsquoThe word lsquoofrsquo between any two fractions is to be used as multiplicationExample 57 of 56 = 57 times 56 = 5times8 = 40Study item Using BODMASThe word lsquoBODMASrsquo is the abbreviation formed by taking the initial letters of six operations(i)Bracket (ii) of (iii) Division (iv) Multiplication (v) Addition (vi) SubtractionAccording to BODMAS rule First of all the terms inside Bracket must be simplified then lsquoofrsquo lsquoDivisionrsquo lsquoMultiplicationrsquo lsquoAdditionrsquo lsquosubtractionrsquo

Study item Removal of Brackets

There are four Brackets of algebra in Mathematics In a complex expression four types of brackets are used Order of removing the brackets is first ----- then ( ) then finally [ ]

Class VIIISubject Topic Summary Execution

Chemistry Hydrogen General methods of preparation of hydrogen

By the action of dilute acids on metals

Calcium Reacts readily to form chloride salt and hydrogen

Ca + 2HCl rarr CaCl₂ + H₂uarr

Magnesium

Aluminium

Zinc

React readily to form salt and hydrogen

Mg + 2HCl rarr MgCl₂ + H₂uarr2Al + 6HCl rarr 2AlCl₃ + 3H₂uarrZn + 2HCl rarr ZnCl₂ + H₂uarr

Question 4 ) Give reasons for the following

(a) Hydrogen be used as a fuel

Solution

Hydrogen is used as a fuel because it has a high heat of combustion Some significant fuels are coal gas water gas and liquid hydrogen

(b) Though hydrogen is lighter than air it cannot be collected by downward displacement of air

Solution

Hydrogen is lighter than air so it is possible to collect the gas by downward displacement of air But it is not safe to do so since a mixture of hydrogen and air can lead to an explosion

(c) A pop sound produced when hydrogen is burnt

Solution

Impure hydrogen gas burns in air with a pop sound This is because of the presence of impurities in it

(d) Helium replaced hydrogen in weather observation balloons

Solution

It forms a mixture with air that can explode when there is a small leakage of hydrogen in a balloon So helium has replaced hydrogen

(e) Nitric acid not used for the preparation of hydrogen gas

Solution

(e) By the action of nitric acid on metals hydrogen cannot be produced because it also releases nitrous oxide and nitric oxide and oxides the hydrogen to form water

Biology Chp-2 Reproduction in plants

Today we discuss different methods of artificial propagation like cutting-rose sugercane Layering ndashguava lemon china rose etc Grafting- mango apple etcMicropropagation ndashorchid asparagus etcWe also discuss about advantages and disadvantages of vegetative propagation

Q7 Define the following terms i) Explant In tissue culture techniquea tiny piece of bud shoot or any other partof plant from where new tissue develop ii) Callus The cells of the tissue divide andgrow into a mass of undifferentiated cells from explant iii) Plantlet After few days callus differentiate into a small plant with roots and shootQ8 what are the advantages and limitations of tissue culture or micropropagation

Advantages i ) It produacuteces superior quality plantsii)It can be applied to interspecifie hybridsiii) It is useful to grow seedless plants bull Limitations i) It cannot be used for all plantsii)It is not easy to handleQ9 Write advantages of vegetative propagationi) It is a quick and easy method ofproducing new plantsii) This method need less time to matureiii) The new plants are exact copies of the parentiv) it is extremly useful for growing seedlessplants like banana grapes etc Q10 Write some disadvantages of vegetativepropagationi) Dišeases present in the parent plant gettransferred to all in new plantsii) Overcrowding of new plants causes competition for sunlight water and nutrients which affects growth of plantsplant

Physics Chapter 2 Physical Quatites and Measurements

Here We Will Do Some QuestionsRelated To Chapter 2

Select the correct alternative A block of wood of density 08gcm-3 has a volume of 60cm3 The mass of the block is

1 608 g

2 75 g

3 48 g

4 0013 g

Solution 348 g

The density of aluminium is 27g and that of brass The correct statement is

1 Equal masses of aluminium and brass have equal volumes

2 The mass of a certain volume of brass is more than the mass of an equal volume of aluminium

3 The volume of a certain mass of brass is more than the volume of an equal mass of aluminium

4 Equal volumes of aluminium and brass have equal masses

Solution 2 The mass of a certain volume of brass is more than the mass of an equal volume of aluminium

MATHEMATICS Ch 6Sets

Exercise 6(C)1 Find all the subset of each the following sets(i) A = 57 (iii) C = x xisin W x le 2(iv) p p is a letter in the word lsquopoorrsquo

Solution (i) All the subsets of A are ϕ 5 7 57

(iii) All the subsets of C are ϕ 0 1 2 01 02 12 012

(iv) All the subsets are ϕ p o r po or por

4 Given the universal set = -7-3-105689 find (i) A = x xlt2 (ii) B = x -4ltxlt6 Solution

(i) A = -7-3-10(ii) B = -3-105

5 Given the universal set = x xisin N and xlt20 find

(i) A = x x = 3p pisin N (iii) C = x x is divisible by 4 Solution

(i) 369121518 (iii) 481216

6 Find the proper subset of x x2-9x-10 = 0 Solution

ϕ 10 -1

Working x2-9x-10 = 0 rArr x2-(10-1)x-10 = 0

rArr x2-10x+x-10 = 0 rArrx(x-10)+1(x-10) = 0

rArr (x+1) (x-10) = 0

11 Let M = letters of the word REAL and N = letters of the word LARE Write sets M and N in roster form and then state whether (i) M sube N is true (ii) N sube M is true (iii) M = N is true

Solution M = real and N = lareSo (i) Yes (ii) Yes (iii) Yes

English 2 Twelfth Night ndash Shakespeare

A noble man named Orsino in the kingdom of Illyria is deeply in love with a lady called lady Olivia She is in mourning for her dead brother so she will not even think about marriage At this time a sea storm causes a terrible shipwreck and a young lady called Viola is swept onto the shore She thinks that her twin brother Sebastian is drowned A sea captain tells her about Orsino and his love for Olivia Viola wishes to work in Oliviarsquos home but feels she will not be employed So she dresses as a man calls herself Cesario and gets work at the house of OrsinoViola (now Ceasario) is much liked by Orsino and becomes his page She falls in love with Orsino Orsino sends Ceasario to deliver messages to Olivia Olivia herself falls for the beautiful young Ceasario believing Viola to be a man

2 Answer the following questionsa Why does Orsino ask the musicians to play onOrsino asks the musicians to play on because music feeds his desire He calls upon the musicians to play music so that his hunger for love could be replenished with an excess of musicb What does Valentine tell about OliviaWe learn from Valentine that Olivia is in mourning for her brother she wears a veil and has vowed that no one will see her face for another seven yearsand she refuses to marry anyone until thenc From the exchange between Orsino and Valentine what do you think their relationship isValentine is one of orsinod attendants He was sent to Olivia as a messenger of love but was not allowed to speak to here Who is Olivia mourning for and whyOlivia is mourning for her dead brother

Homework Q fHistory and Civics

Growth of Nationalism

Important dates to remember1769-Napoleon born on 15thAugust1789-Fall of Bastille on 14th July and the beginning of the French revolution declaration of the rights of Man on 26thAugust1793-King Louis XVI executed on January 211764-The Sugar Act passed1765-The Stamp act passed1774-The first congress of Philadelphia1776-The declaration of American Independence of on 4th July1777-Defeat of the British at Saratoga1781-Surrender of lord Cornwallis at Yorktown1783-The treaty of Versailles1804-Napoleon becomes the emperor1813-Battle of Leipzig or Battle of nations in which Napoleon was defeated by the Allies1815-Battle of Waterloo June 18 in which Napoleon was defeated and captured1821-Death of Napoleon in StHelena1860-Abraham Lincoln elected President of the USA1861-The civil war began 1864-Abraham Lincoln elected President of the USA for the second time1865-Slavery abolished in the US

Name the following- The queen of Louis XVI

Marie Antoinette The three philosophers of France

VoltaireMontesquieuJean Jacques Rousseau

The British general whose surrender brought the war in America to an endLord Cornwallis

The first president of the USAGeorge Washington (1732-1799)

The first southern state to secede from the unionSouth Carolina

The author of the book lsquoUncle Toms CabinHarriet Beecher Stowe

Homework-Learn

Class IXSubject Topic Summary Execution

Economics

Types of economies Today I am going to share you the concept of economic growth and economic development Few questions will be given from the previous study material dated 942020

Meaning of economic growthAnswer) The term economic growth generally means anincrease in national income or per capita output or income over time It indicates towards quantitative growth of a country

Meaning of economic developmentAnswer) Economic development is defined

as a process whereby the real per capita income of a country increases over time along with fall in poverty ratio unemployment and income inequality etc

Distinguish between economic growth and economic development

Basis Economic growth

Economic development

Scope It has narrow scope as it refer only to rise in per capita income

It has wide concept since it includes qualitative changes as well

Concerned matter

It is concerned with the rise in income

It is concerned with not only rise in income but also reduction of poverty income inequality and unemployment

Focus Economic growth does not focus on economic development

Economic development focus on economic growth plus qualitative changes

Distinguish between capitalist economy and socialist economy

Ownership

Motive

Tool

Means of production are owned and managed by private people

Self interest and profit earning is the main motive

Price mechanism is a main tool to solve the economic problems

Means of production are owned and managed by the government

Social welfare is the main motive

Economic planning by the government is the main tool to solve the economic problem

Competition

Distribution of income

There exist large competition among buyers and sellers

There is existence of large inequalities of income

There is no such competition

There exist less inequalities of income

Math Topic ndash AlgebraChapter -Factorisation

Study item Factorising by taking out common factorSome solved sums from exercise 41

1) (i) 8xy3 + 12x2y2

= HCF of 8xy3 and 12x2y2 is 4xy2

= 4xy2(2y + 3x )

4) (ii) 28p2q2r ndash 42pq2r2

= HCF of 28p2q2r and 42pq2r2 is 14pq2r = 14pq2r (2p - 3r )5) (ii) 14mn + 22m - 62p=HCF of 14mn 22m and 62p is 2= 2(7mn + 11m - 31p)7) (ii) 3a(x2 + y2) + 6b (x2 + y2) = HCF of 3a(x2 + y2) and 6b(x2 + y2 ) is (x2 + y2)= ( x2+ y2 )(3a + 6b )9) (ii) x(x2 + y2 ndash z2 ) + y(-x2ndashy2 + z2 ) ndash z(x2+ y2 ndash z2 )= x(x2 + y2 -z2) ndash y-(x2 + y2 -z2) -z(x2 + y2 ndash z2)=x(x2 + y2-z2) -y( x2 + y2-z2) ndash z (x2 + y2 -z2)= (x2+ y2 ndash z2)(x ndash y ndash z )

Commercial Studies

Introduction to Accounting and Book-keeping

Today I am going to share you the meaning of Accounting and Book-keeping and its related terms bullAccounting bullBook Keeping bullAccountsbullTypes Of Accounts bullAccounting Cycle

bull Meaning of accounting

Ans )Accounting is the art and science of recording classifying and summarising monetary transactions

bull Meaning of Book-keeping

Ans) Bookkeeping is the art of recording business transactions with the view of having a permanent record of them and showing their effect on wealth

bull Meaning of account

Ans) The term account means a record of business transactions concern a particular person of firm asset or income or expense It is a summarised record of all transactions which take place in an accounting year

bull Types of accountsPersonal accounts ndash Personal accounts relating

to person and Organisation are known

as personal accounts Example Ramrsquos Account ABC amp Co Account etc

Real account - The accounts related to tangible and intangible assets are called real accountsExample Cash Account Furniture Account etc

Nominal account- Accounts related to expenses losses incomes and gains are known as nominal accountsExample Wages Account Salary Account Discount Account etc

bull Accounting cycle Accounting cycle refers to a complete sequence of accounting activities It begins with recording of transactions and ends with the preparation of a balance sheet

English 1 Transformation of sentences

Sentences A sentence is a group of words which makes complete sense

a Assertive sentencesb Imperative sentencesc Interrogative

sentencesd Exclamatory sentences

Sentences can be changed from one grammatical form to another without changing the meaning of the sentence This is known as transformation of sentences

Exercise 6Rewrite the following sentences according to the instructions given below without changing their meanings

1 As soon as he saw the beer he jumped into the river ( Begin No sooner)

2 None but brave deserve the fair (Begin the bravehellip)

3 This box is too heavy for me to lift ( Use so hellip That instead of too)

4 No one other than a king can live like James Luxurious ( Begin only James)

5 Oh for the wings of a dove (Begin I wishhellip)

BENGALI(2ND LANGUAGE)

ldquo বঙগভমির পরমি ldquo াইকেল ধসদন দতত

পব13পোসঠ আসোলিচত ৩ পরবোস দৈদসবর বস ীবতোরো Pলিদ স এ লেদ -আকো সত-োলি লেদ তোস - ক) বকতো লেক কোর লেো লেকো কলিবতোর অং ) কোর পরলিত বকতোর এই উলিকত গ) এ লেদ আকো সত বসত কী বলিঝসয়স4 ীবতোরো বসত কী লেবোঝ ঘ ) আসোচয অংসর তোৎপP13 কী

উ -ক ) বকতো স কলিব মোইসক ম3দ দতত

Types of AccountPersonal AccountReal AccountNominal AccountBalance Sheet (opening)

কলিব মোইসক ম3দ দসততর রলিচত বঙগভলিমর পরলিত কলিবতোর অং ) কলিব বঙগী অ13োৎ লেদমোতোর পরলিত কলিবর এই উলিকত গ ) এ লেদ আকো বসত কলিবর মোব লেদী রপ আকো লেক লেবোঝোসো সয়স4 আকো লেসক লেPম তোরো স পসর লেতমলি ীব লেদ রপ আকো লেসক পরো রপ তোরো স পরসত পোসর এই মভোবোর কোই কলিব বসস4 ঘ ) পরবো Pোতরোয় Pলিদ কলিবর লেদ আকো লেসক ীব তোরো রপ পরো স পসর তোসত কলিব লিবনদমোতর দঃলিত কোর মতয লিবসর সবোভোলিবক পলিরলিত এবং মোষ মরী তোই পরবোস Pলিদ তা োর মতয য় তবও কলিব লিবচলিত সব ো কোর পলিবীসত লেকউ অমর য় লিক4ই অকষয় য় দীর লেPম লিচরপরবোমো লেতমলি মোসষর ীবও চমোতোই ীব - সতবধতোই মতয ীব দীসত মোষ লিতয পরবোমো তবও লেPব মোষ আপ কতকসম13র মো3যসম মোসষর মস লিসসদর সথো কসর লিসত পোসর তোরো লিচরভোসবর সয় মোসষর মস লিবরো কসর তোসদর মস3য লেকউ পGভসত লিবী সয় গোসও মোসষর মস তোরো লিতযপলিত লিতযবলিনদত

Hindi 2ndlang

काकीी(लिसयारामशरणगपत)

इस कहानी म क न यह बतान का परयास निकया ह निक बचच अपनी मा स निकतना परम करत ह शयाम अबोध बाक ह वह अपनी मा क मरन क बाद उसन अपनी मा क लिए बहत रोया बाद म उस पता चा निक उसकी मा राम क घर ची गई ह आकाश म उडती हई पतग दकर उस हष हआ निक पतग क दवारा वह अपनी मा को नीच उतारगा इसक लिए वह अपनी निपता की जब स दो बार सवा रपया निनकाकर पतग और दो मोटी सी मन वाी अपन भाई स काकी एक कागज पर लिवा कर पतग म लिशव का दिदयानिनकाकर पतग और दो मोटी सी मन वाी अपन भाई स काकी एक कागज पर लिवा कर पतग म लिचपका दिदयाभोा और शयाम कोठरी म रससी बाधनी रह थ तभी उसक निपता करोध म आकर उन स पछ निक कया उनकी जब स रपया निनकाा हभोा डर क मार बताया निक शयाम इस पतग क दवारा अपनी काकी को राम क यहा स उतारना चाहता हनिवशशवर(शयाम क निपता)न फटी पतग उठाकर दी तो उस पर काकी लिा थावह हत बजि होकर वही ड रह गएउनहोन सोचा निक मन अपन पतर को मारा जोनिक अनजान और निनदष थावह अपनी मा कोनिकतना पयार करता ह

helliphellipContinue to next

Computer Application

Java Programming Prog 1Write a java program to input two numbers from user and display the sum or product of them as per user choice Use switch case statementSolve public class sum_product public static void main(String args[]) Scanner sc=new Scanner(Systemin) int abc Systemoutprintln(ldquoEnter two numbersrdquo) a=scnextInt() b=scnextInt() Systemoutprintln(ldquoPress 1 for sum or 2 for productrdquo)

c=scnextInt() switch(c) case 1 Systemoutprintln(ldquoThe sum will be =rdquo+(a+b)) break case 2 Systemoutprintln(ldquoThe product will be =rdquo+(ab)) break default Systemoutprintln(ldquoWrong Inputrdquo) Home Work - Practice in your computer using bluej

Subject Eng Literature (The Merchant of Venice ndash William Shakespeare)Topic Act I Scene 2 Lines 92 to 126 (End of scene) Date 13th April 2020 (5th Period)

[Students should read the original play and also the paraphrase given in the school prescribed textbook]Summary Questions amp Answers

o After Portia has expressed her opinion about the suitors Nerissa informs that she need not bother about any one of them as they have decided to quit Belmont at the earliest opportunity because they do not believe in trying their luck by the caskets which is the only way of winning Portia

o Nerissa then enquires of Portiarsquos opinion about Bassanio who once visited her in the company of the Marquis of Montferrat and says that she had never come across such an ideal love deserving the fairest lady for his bride

o Portia seems to remember Bassanio quite correctly and says that she agrees with Nerissa At this moment a servant informs Portia that the Prince of Morocco has arrived to try his luck by the caskets

o Portia tells Nerissa that if she could welcome this new suitor as gladly as she says farewell to the previous ones she would be glad of his arrival However if he happens to have the virtues of a saint but the black complexion of a devil she would prefer to have him for religious consolation rather than as a husband

(1) NERISSA You need not fear lady (Line 97-103)

the having any of these lords they have acquainted me with their determinations

which is indeed to return to their home and to

trouble you with no more suit unless you may be wonby some other sort than your fathers imposition depending on the caskets

PORTIA If I live to be as old as Sibylla I will die as chaste asDiana unless I be obtained by the manner of my fatherswill I am glad this parcel of wooers are so reasonablefor there is not one among them but I dote on his veryabsence and I pray God grant them a fair departure

(a) Elucidate the idea expressed in the first speech of the above dialogue

In the first speech Nerissa assures Portia that she need not have any fear of being compelled to marry anyone of the suitors who had lately come to Belmont She informs her that they have all decided to return to their respective countries(b) Illuminate the meaning of the phrase ldquoyour fatherrsquos imposition depending on the casketsrdquo

Nerissa means that the suitors of Portia do not find the conditions imposed by the will of her father to their liking They are too hard for them These conditions are that in the event of a suitor failing to choose the right casket (i) he should never disclose to anybody which casket he chose (ii) he can never marry and (iii) he should take his departure immediately(c) Explain the meaning of the term lsquoSibyllarsquo

lsquoSibyllarsquo is the name given by Romans and Greeks to a prophetess inspired by some deity usually the sun-god Apollo She had a very long life The god Apollo granted her as many years of life as she could hold grains of sand in her hand(d) Elucidate the meaning of the term lsquoDianarsquo

lsquoDianarsquo is the goddess of hunting She is also regarded as a symbol of virginity because she never fell in love and never

married(e) Explain the meaning of the first two lines of Portiarsquos speech

Portia says that even if she is to live for centuries like Sibylla she would not marry except in accordance to her fatherrsquos will She asserts that she would not mind remaining unmarried and untouched by a man like Diana the virgin the goddess of hunting unless a man is able to win her by passing the test laid down by her father

Class XSubject Topic Summary Execution

Hindi 2nd

Langबड घर की बटी( मशी परमचद)

lsquoबड घर की बीटीrsquo कहानी का उददशय मधयम वग की घर समसया को सझा कर सगदिठत परिरवार म मिम जकर परम स रहन का सदश दना ह घर म शानित tानिपत करन की जिजममदारी नारी की होती ह यदिद नारी समझदार ह उसम धय और परिरवार क परनित परम ह तो कोई भी घटना परिरवार को निवघदिटत नही कर सकती या कहानी परिरवार को सगदिठत करत हए परम सौहाद स एक रदसर की भावनाओ को समझ करउनका सहयोग करत हए जीवन यापन करन की पररणा दती ह मशीपरमचदर जी न इस कहानी म सयकत परिरवार का परनितनिनमिधतव निकया ह यह कहानी बनी माधव सिसह जो गौरी पर क जमीदार क उनक दो पतरो की हशरी कठ ा निबहारीशरीकात का निववाह एकजमीदार घरान की पतरी आनदी स हआ थाआनदी न द को ससरा क वातावरण म ढालिया थाएक दिदन आनदी का अपन दवर ा निबहारी स झगडा हो जाता ह दोनो भाई एक रदसर स अग होन की कोलिशश करत हसभी बह आनदी न अपन मधर वयवहार स ा निबहारी को घर छोडकर जान स रोक लिया| इस पर बनी माधव सिसह न कहा निक बड घर की बटी ऐसी ही होती ह जो निबगडा काम बना ती ह अतः शीषक साथक ह बड घर की बटी आनदी ह

helliphelliphelliphellipContinue to nextBiology Topic ndash Chp-1

CellWelcome to new session 2020-21Today we will start with Chpter 1 cell CELL

Protoplasm+Cellmembrane Or Cell wall

Cytoplasm+Neucleus

Cytoplasmic+ CytoplasmicOrganelles Inclutions(mitochondria (food Golgi bodies pigments)Ribosome)

What is cellbull Cell is the structural and functional unit of living organismbull According to number of cells organisms areUnicellular - Amoeba bacteria Multicellular - Rose Mango Tiger HumanSmallest cell -bacteria Longest cell - Nerve cellLargest cell - Ostrich egg cellCells are of different size and shapes according to their functionsQ2Write chief functions of following cellorganelles

Q3What is tonoplastVacuoles covered by a covering called tonoplast

Bengali(2Nd

Language)

ফ ফটক ো ফটক (কলিবতো ) ভোষ মসোপো3 gtPোয়

একটি লেমসয়র ীবস লেপরম লিকভোসব ফসট ওসঠ তো লেদলিসয়স4 কলিব লেপরম Pই য় লেই ময়ই বনত কোস পলিরত য় ফ লেফোটো বো োসফোটো লেটো ব2 কো য় লেমসয়সদর ব gtয13 লেপরসমর 4লিব ফসট উসঠস4 এই কলিবতোয় লেপরম মোষসক মত gtযর মস লেফস লিদসয় পরকষস বাোচোসোর gtয োত বো2োয় কলিবতোয় লেমসয়টির পসব13র দঃসর কো বো সও লেমসয়টি লেই পসর পলিক সত চোয়ো োরী ীবসর কোস4 পরম লেPৌবস লেপরমসক পোবোর পরব ইচছো োকসও তো পসর লেলিতবোচকতোয় পলিরত য় কলিব ভোষ মসোপো3 যোয় লেP ক লেপরসমর

কলিবতোয় ব gtযবহত লিবসষ লিক4 সvর অ13 লেদওয়ো ১) রসবোো= লেP লিবলিভনন রকম ডোকসত পোসর২) ো= পোর ৩) ঠলি = লেচোসর বZ৪)আই বস2ো=অলিববোলিত৫)শইসয় = োলিয়ত কসর৬)োতপাোচ= লিবলিভনন পরকোর৭)দ2োম = v কসর বZ কসর লেদওয়ো৮)লেরলিং =লেোোর দৈতরী লেব2ো৯) বনত= একঋত১০) পাোর = বসকরো2

Organelles Functions

1 Endoplasmic reticulum

2 Mitochondria

3Golgibodies

4 Ribosome5Lysosome

6Plastids

7 Centrosome

i) Supportive framework for the cellii) Synthesis and transpost of proteinsRelease of energy in the form of ATPi) Synthesis and secretion of enzymes hormoneii) Formation of vacuoles lysosomei) Protein Synthesisi) Intracellular digestionii) Destroy foreign substancei )Leucoplast - stores starchii)chloroplast - trap solar energyiii) Chromoplast - imparts colour toflowers amp fruitsi) Initiates and requlates cell division

কলিবতো তোর অ13সক ভোষোয় পরকোো কসর ঘলিরসয় ব যকত কসরস4 লেপরসমর ফতো আর লিবফতো লেক গোঢ় কসর লেদোসো কলিব ভোষ মসোপো3 যোসয়র অলিভবসর অ যলিদক

Economics

Factors of Production

Welcome to the new sessionToday we are going to start the first chapter of Class XThe name of the chapter is Factors of productionBy the name I hope you all can recall a glimpse of what you have learnt in the second chapter of Class IX

NowProduction is the process of creating the various goods and services which are consumed by the people of the country to satisfy their wants

Thus it is the process in which some materials are transformed from one form to another to create utility and value in goods

For example utility can be created by changing the form of a commodity ie

Making of table out of wood by a carpenter for his customer here the wood is getting transformed into table creating utility for his customer and he can also command a price for it

On the other hand Housewives perform very

useful activities at home which create utility but their domestic activities are not included in production because they have no money value

So we can also say that Production denotes two things firstly creation of utility and secondly creation of value

Production is not complete unless it reaches the consumer

An increase in production will increase the economic welfare of the consumers and hence the aim is to raise the production level of the country

Again production of a good or service is only possible if certain resources or

Questions

1 What do you mean by production

Answer Production means the creation of goods and services for the purpose of selling in the market

In fact production involves the transformation of inputs into outputs

Hence production denotes two thingsCreation of utility and creation of valueUtility and value can be created by changing the form by changing the place by changing the time and by rendering services

Example Transformation of raw

materials into finish goods such as potter creates utility by converting mud into utensils assembling of small parts to make bigger machinery

Production also includes services such as distribution and marketing

2 What are the factors of production

Answer Factors of Production refers to the resources and inputs needed for producing goods and servicesThese inputs can be classified as

Land Labour

Capital Enterprise

Land Land is defined to include not only the surface of the earth but also all other free gifts of nature(for example mineral resources forest resources and indeed anything that helps us to carry out the production of goods and services but is provided by

inputs are used together in right proportion

A resource or an input which helps in the process of production to obtain an output is called FACTOR OF PRODUCTION

These factors of production can broadly be categorized into four parts 1LAND 2LABOUR3CAPITAL4ENTERPRISE (ORGANISATION)or Entrepreneur

The above factors are all interdependent on each other and they play a major role in production process

FACTORS OF PRODUCTION

LANDCAPITAL

LABOUR ENTREPRENEUR

nature free of cost)LabourLabour refers to the human efforts that need to be combined with other factors of production for creating an output

CapitalAll man ndash made means of production is called capita example machineries which help in further production Money when used for starting any business for purchasing raw materials machinery tools etc it is regarded as capitalCapital also includes physical capital like factories machineriestoolsbuildingsequipments etcEnterpriseThe task of bearing risks is called enterprise and the person who bears these risks of business is called the entrepreneurThus an entrepreneur is one who organises production takes important decisions regarding production hires and purchases factors of production and bears the risk and uncertainty involved in productionOrganisation refers to the services of an entrepreneur who controls organises and undertakes all risks One who plans organises and manages a business enterprise is an organiser

Physics Chapter 1 Force

Force is an external agent capable of changing the state of rest or motion of a particular body It has a magnitude and a direction The direction towards which the force is applied is known as the direction of the force and the application of force is the point where force is applied The Force can be measured using a spring balance The SI unit of force is Newton (N)

Question 1

State the condition when on applying a force the body has

(a) the translational motion

(b) The rotational motion

Solutions

(a) Translational motion is produced when the body is free to move

(b) Rotational motion is produced when the body is pivoted at a point

Question 2

Define moment of force and state its SI unit

Solutions

The moment of force is equal to the product of the magnitude of the force and the perpendicular distance of the line of action of force from the axis

of rotation

The SI unit of moment of force is Newton times meter

= Newton meter (Nm)

Commercial Studies

Stake holders In this topic you will be come to know about the meaning and concept of stakeholders

How stakeholders are different from shareholders

Questions1 What do you mean by the term stake holdersAnswer) The term stake holders have developed from the words which mean an interest or expected benefit Stakeholders mean all those individuals groups and Institutions which have a state (interest) in the functioning and performance of a commercial organisation or a business enterprise2 What do you mean by share holdersAnswer) The person and Groups who own the shares of the joint stock company by providing capital to the company are called shareholders Shareholders are the internal stakeholders shareholders are one out of several stake holders3 How are shareholders different from stakeholdersAnswer)i) The term shareholders is related to only joint stock company whereas stakeholders are related with all business organisationsii) Stakeholders maybe any individual having financial stake in business organisation whereas a shareholders are those individuals who are holding shares in the company4) How are shareholders different from creditorsAnswer) i) Shareholders are internal stakeholders while creditors are external stakeholdersii) Shareholders invest in the capital of the company whereas creditors give loan to the companyiii) Shareholders are the members of the company with voting rights but creditors are not the members of the company

English 1 Transformation of sentences

Sentences A sentence is a group of words which makes complete sense

e Assertive sentencesf Imperative sentencesg Interrogative sentencesh Exclamatory sentences

Sentences can be changed from one grammatical form to another without changing the meaning of the sentence This is known as transformation of sentences

Exercise 1 Change the following affirmative sentences into Negative sentences

a He is a good manHe is not a bad man

b Ram loves SitaRam is not without love for Sita

c Only he stood first in the classNone but he stood first in the class

d Ankit was wiser than he

He was not so wise as Ankite He did it

He did not fail to do itf As soon as I reached college the

bell rangNo sooner did I reach college than the bell rang

g He finished everythingHe left nothing unfinished

h It always pours when it rainsIt never rains but it pours

Math Topic Commercial MathematicsChapter ndash Goods and services Tax

What is GSTAns It is a abbreviated term of Goods and Service Text which is an indirect tax levied on the sale of goods and rendering servicesSome terms related to GSTDelar Any person who buys goods or services For resale is known as a delar A delar Can be a firm or a companyIntra-state sales Sales of goods and services within the same state or same union territory are called intra- state salesInter-state sales Sales of goods and services outside the state or union territory are called Inter-state sales4) Input GST GST is paid by dealers on purchase of goods and services are called input GST5) Output GST GST is collected from customers on sale of goods and services are called output GST6) Types of GST There are three taxes applicable under GST(i) Central Goods and Services Tax (CGST)(ii) State Goods and Services Tax (SGST) or Union Territory Goods and Services Tax (UTGST) Both these taxes are levied on intra-state sales Here GST is divided equally among central and state governments(iii) Integrated Goods and Services Tax (IGST) IGST is levied on inter- state sales It is also levied on import of goods and services into India and export of goods and services from India

Subject Eng Literature (The Merchant of Venice ndash William Shakespeare)Topic Act III Scene 4 Lines 1 to 44 (Portia hellip To wish it back on you fare you well Jessica)[Students should read the original play and also the paraphrase given in the school prescribed textbook]

Summary Questions amp AnswersIn this scene we suddenly find a new element in the character of Portia We have already seen her possessed of every graceful womanly quality but now she shows that she is capable of rapid decision and determined action She shows this by her sudden resolve to hasten to Venice with a daring scheme for the rescue of Antonio This is an important scene in the dramatic action for it leads up to and renders possible the striking events of the famous trial scene which is one of the greatest striking elements of the play Moreover the fact that all the characters of importance are now assembled together in Venice makes the union of the main plot and the secondary story complete

(1) LORENZO Madam although I speak it in your presence(Line 1-9)

You have a noble and a true conceit

Of god-like amity which appears most strongly

In bearing thus the absence of your lordBut if you knew to whom you show this honourHow true a gentleman you send reliefHow dear a lover of my lord your husbandI know you would be prouder of the workThan customary bounty can enforce you

(a) Where is Lorenzo Why is he here To whom is he referring as lsquoMadamrsquo

Lorenzo is at Portiarsquos residence He had met Salerio on the way and Salerio had begged him to come along with him to

o In this scene Portia Nerissa Lorenzo Jessica and Balthazar appear

o Portia requests Lorenzo and Jessica to be in charge of her house during her absence from Belmont because she and Nerissa have decided to spend the days in meditation and also in visiting the holy places in the neighbourhood of Belmont She has already instructed her people to acknowledge both Lorenzo and Jessica as master and mistress of house during her absence Lorenzo and Jessica gladly agree to look after the house of Portia

handover the letter from Antonio to Bassanio The letter carried the bad news about Antoniorsquos arrest for non-payment of loan taken from Shylock Hence Salerio might have preferred company to break this bad news to Bassanio He is referring to Portia as Madam(b) What does Portia say on hearing the above extract

Portia says that she has never regretted doing good to others Friends who spend a lot of time together and really are there for each other have many traits in common As Antonio is Bassaniorsquos best friend saving him is like saving Bassanio who is like her own soul She asks Lorenzo to take care of management of the house till Bassanio is back(c) What does Portia send with Bassanio and why

On hearing about Antoniorsquos troubles on account of Bassanio her husband Portia immediately sends him with enough gold to repay the debt many times over to Venice to help Antonio out of his misfortune

(2) Lorenzo Madam with all my heart (Line 36-40)

I shall obey you in all fair commands

Portia My people do already know my mindAnd will acknowledge you and JessicaIn place of Lord Bassanio and myselfSo fare you well till we shall meet again

(a) Where are Lorenzo and Portia at this time What lsquofair commandsrsquo are given to Lorenzo

Lorenzo and Portia are at Belmont during this scenePortia reveals to Lorenzo that she has sworn to contemplate in prayer at a monastery around two miles away until her husband returns from Venice She tells him that Nerissa would accompany her and asks him to manage the house with Jessica till things are settled In response Lorenzo tells her that he would be obliged to do whatever she asks him to do(b) Where is Portia actually going and why

Portia tells Lorenzo that she would live a life of contemplation and pray at a monastery which is two miles away from her place In reality Portia plans to go to Venice in disguise with Nerissa and argue the case in defense of Antonio She is very sure that her plan would succeed

ClassXI (ScienceHumanitiesCommerce)Subject Topic Summary Execution

Computer Science

(APC)

Ch ndash 1 Numbers

(Numbers in different bases and

their Arithmatical operations)

Number System In computers Number System is defined as a writing system to represent the numbers in different ways ie we are using different symbols and notations to represent numbers There are four ways we can represent the number ndash Binary Decimal Octal and Hexadecimal

Decimal Number SystemThis number system consist 10 digits These are 0 1 2 3 4 5 6 7 8 amp 9

Binary Number SystemThis number system has only two digits these are 0 and 1 Here 0 stands for off while 1 stands for on

Octal Number SystemThis number system has 8 digits these are 0 1 2 3 4 5 6 amp 7

Hexadecimal Number SystemThis number system has 16 digits these are 0 1 2 3 4 5 6 7 8 9 A B C D E F Here the value of the alphabets are as follows A=10 B=11 C=12 D=13 E=14 F=15

Rules for conversion decimal number to Binary1 Divide the decimal number by 22 If the number will not divide equally by 2 then round down the answer to the nearest whole number (integer)3 Keep a note of the remainder it should be either 0 or 14 Keep repeating the above steps dividing each answer by 2 until you reach zero5 Write out all the remainders from bottom to top This is your binary solution

For example Lets convert 32 to binary 2 32 2 16 - 0 2 8 - 0 2 4 - 0 2 2 - 0 2 1 - 0 0 - 1

The binary equivalent of 3210 is 1000002

Try the follwing youself1 2410

2 4810

3 1210

History GROWTH OF NATIONALISM

The second half of the 19th century witnessed growth of political consciousness and a sense of Nationalism among the IndiansThere were various factors for growth of Indian Nationalism- As a result various political associations were formed in different provinces by the educated Indians Surendranath Banerjee organized a meeting of National conference at Calcutta Ultimately the National Congress was founded in Bombay in 1885This body became the vanguard of Indian struggle for freedom The congress leaders were known as moderates because they followed a policy of prayer and petition A large number of Indian leaders had experienced in political agitation The Political situation of England was also changed Moreover increasing revolutionary activities in Maharashtra Punjab and Bengal became serious concern to the British Government In this

QUESTION1 What do you mean by Nationalism ANSWER 1 Nationalism is defined as loyalty and devotion to own nation especially a sense of national consciousnessQUESTION 2 What are the causes of nationalism ANSWER 2 There were various factors for growth of nationalism

1 Spread of western education2 The progress of vernacular press and

patriotic literature3 The economic exploitation of our

country by the colonial rulers4 International affairs

QUESTION 3 Who organized National conference in Calcutta in 1883 ANSWER 3 Surendranath BanerjeeQUESTION 4 When did Indian National Congress formANSWER 4 Indian National Congress was formed in 1885 in BombayQUESTION 5 Who were ModeratesANSWER 5 The Early Nationalists were also known as Moderates Their emergence marked

background Lord Curzon became Viceroy in India He had no respect for the Indian National Congress

the beginning of the organized national movement in India They believed in British justice and were loyal to them They followed a policy of prayer and petition They demanded constitutional reforms of our country Impotant Moderate leaders were Pherozshah Mehta Dadabhai Naorozi and Surendranath Banerjee etcQUESTION 6 What do you know about Extremism in Indian National movementANSWER 6 In the beginning of 20th century a new class of national leaders emerged in India which was different from the moderate groups They started more aggressive movement against the British empire The goal of extremists was ldquoswarajrdquo Important extremist leaders were Bal Gangadhar Tilak Lala Lajpat Rai Bipin Chandra Pal etcQUESTION 7 Mention the places which were the main centres of Revolutionary movementANSWER 7 Maharashtra Bengal and Punjab

Physics

Chapter Dimensional Analysis

(Summary)

The dimensions of a physical quantity are the powers to which the fundamental units are raised in order to obtain the derived unit of that quantit

The physical quantites lengthmasstime are represented by [L] [M] [T] resp let they are raised to powers ( dimesions) abc resp then any physical quantity can be represented by [ La Mb Tc ] Examples

1 Area area = L x B = [L] x [L] = [M0 L2 T0 ]

2 Density density = massvolume = [M][L3] = [ M L-3]

3 Velocity velocity = distancetime = [L][T] = [LT-1]HW Try to find out dimension of acceleration Acceleration = velocity timeNB One can find the SI Units Using Dimension Analysis Such as for area we have [L2] so its SI unit is m2

Biology Topic ndash Chp-1 The living world

Today we will start the first chapter the living world Here we discuss about the characteristics of living organism and what are the difference between them and nonliving substances We also discuss about the contribution of different Scientists

There are over 500000 species of plants andover a million species of animal are present on earth Some 15000 new species were discovered every yearQ1 What is a living organismbull A living organism is primarily physico -chemical material that demonstrate a high degree of complexity is capable of selfRegulation possesses a metabolism and perpetuates itself through timeQ2 What are the differences between livingand non-livingsi) Compared with non-living living organisms

have more complex organised structure and their use of energy is more controlled amp efficientii) Living things reproduce their own kind by forming new cells which contains copies of their genesiii) Each organism has some degree of homeostasisie it is able to make adjustments so that internal environment remains constantQ3 Write contributions of following Scientists i) Aristotle - One of the first theories in Biology places all living things in a hiearchieii) AV Leeuwenhoek - was the first to observe living single celled organisms under microscopeii) Carolus Linnaeus - developed the binary system for naming of organisms and classificationiii) Geregor Johann Mendel ndash discoverbasic principles of inheritanceHomework i) C Darwin ii)Schleiden

Math Trigonometric functions

1 Overviewi) Trigonometry The word lsquotrigonometryrsquo is derived from the Greek words lsquotrigonrsquo and lsquometronrsquo which means measuring the sides of a triangle An angle is the amount of rotation of a revolving line with respect to a fixed line Usually we follow two types of conventions for measuring angles ie a) Sexagesimal system b) Circular system In Sexagesimal system the unit of measurement is Degree In Circular system the unit of measurement is Radian ii) Relation between degree and radianThe ratio of circumference of a circle to its diameter is always a constant This constant ratio is a number denoted by π which is taken approximately as 227The relationship between degree amp radian measurements is as follows2 right angles = 180deg= π radians1radian = 180degπ=57deg16(approx) 1deg=π180 radianiii) Length of an arc of a circleIf an arc of length s subtends an angle θ radians at the center of a circle of radius r then s=rθiv) Area of a sector of a circleA sector is like a pizza slice of the

Q) Express the following angles in radiana) 45deg b) 40deg3730Ans a) We have 180deg=π radiansi e 45deg= πtimes45180 radian = π4 radiansb) 40deg3730= 40deg37+3060 minute= 40deg 37 +12 minute= 40deg+ 752 minute=40 + 75(2times60) degree=3258 degreeNow 180deg=π radianie 3258 degree= (πtimes325) (180times8) radians = 65π288 radiansQ) A circle has a radius of r=12 meters What is the length of an arc traced out by a 60deg angle in the center of the circleAns In this problem we know both the central angle (60deg) and the radius of the circle (12) All we have to do is plug those values into our equation and we get

s = 2π(12)(60360)s = 24π6s = 4πSo the length of an arc traced out by a 60deg angle in a circle with a radius of 12 meters equals 4π meters asymp 1257 metersQ) Find the area of the sector with a central angle 30deg and a radius of 9cmAns GivenRadius r = 9 cmAngle θ = 30degArea of the sector = θ360degtimesπr2

= 30360degtimes227times92=2121cm2

circle It consists of a region bounded by two radii and an arc lying between the radiiThe area of a sector is a fraction of the area of the circle This area is proportional to the central angle In other words the bigger the central angle the larger is the area of the sectorArea of Sector = θ2 times r2 (when θ is in radians)

Area of Sector = θ times π360 times r2 (when θ is in degrees)

COMMERCE

CLASSIFICTION OF HUMAN ACTIVITIES-ECONOMIC AND NON-ECONOMIC

Welcome to the new sessiontoday we are going to start the first chapter of Class XI The name of the chapter that we are going to start is

lsquoClassification of Human Activities ndasheconomic and non-economicrsquo

Now let us start the chapter by considering human beings and the activities they perform throughout the day

Human activities means all those activities that human beings undertake to satisfy their wants

Human wants on the other hand are the desire of human beings for goods (vegetables fruits rice etc) and services (services of doctors teachers lawyers etc) that they require to live

Now these human activities continue throughout life as human wants are unending unlimited and recurring as human beings desire for better living throughout their lives

Now human activities can be classified into two categories

Human activities

Economic activities Non-economic activities

Economic activities are

Questions1 What are human activities

Answer Human activities mean all those activities that human beings undertake to satisfy their wants

Example A man working in an office

A boy playing in the garden

2What are the characteristics of human activitiesAnswer the characteristics of human activities are as follows

Human activities are undertaken by men women and children and these activities involve human efforts

Human activities are undertaken to satisfy human wants which are unlimited

Human activities continue throughout life

Human activities are performed for both earning money and personal satisfaction

3What is economic activitiesGive example

Answer Economic activities are undertaken by human beings with the object of earning money acquiring wealth and thereby satisfying human wantsExample

Selling of goods by a shop keeper to his customer

A clinic run by a doctor Service of a teacher in school or college

undertaken by human beings with the object of earning money and acquiring wealth

These activities result in the production of economic goods and services

Example Human activities(ie working in factories officesshops) which produce direct economic benefits

Non-economic activities are inspired by human sentiments and emotions such as love for the family desire to help the poor and love for the country

Thus these human activities (eg praying playing sleeping) produce no direct economic benefits and they are also not related to earning money and acquiring wealth

4 What are the characteristics of economic activities

Answer The characteristics of economic activities are as follows

Economic motiveEconomic activities are undertaken to earn money and acquire wealth

ProductiveEconomic activities involve productiondistribution and exchange of goods and services to create wealth

Economic growthEconomic activities determine the level of economic development of a country and standard of living of its citizens

Socially desirableEconomic activities are socially desirable for society

Economic resourcesEconomic activities make use of all the economic resources such landlabourcapital etc

5 What do you mean by non-economic activitiesExampleAnswerNon-economic activities are inspired by human sentiments and emotions such as love for the family desire to help the poor and love for the countryThese activities are not undertaken for monetary gain but for onersquos satisfaction and happinessExample

a mother looks after her children

a student donates blood8 Differentiate between Economic activities and Non-economic activities

Economic activities

Non-economic activities

1to earn living and acquiring wealth2Result can be measured in terms of money

3ExampleBusinessprofession and employment

1 to obtain some satisfaction

2Result cannot be measured in terms of money

3ExampleFamily-orientedreligious socialCultural and national

BUSINESS STUDIES

BUSINESS ENVIRONMENT

Welcome to the new sessionToday we are going to start the first chapter and the name of the chapter is Business Environment

In todayrsquos world every business enterprise is a part of the society It exists and operates in association with various groups in society such as customers suppliers competitors banks and financial institutions government agencies trade unions media and so on All these groups influence the functioning of business in one way or the other They constitute the environment of businessConcept of Business Environment

The term lsquobusiness environmentrsquo refers to the sum total of all individuals institutions and other forces that lie outside a business enterprise but that may influence its functioning and performance

The main features of business environment

Totality of External forces General and Specific forces Interrelatedness Complexity Dynamic Uncertainty Relativity

The Interrelation between business and its environment

The business enterprise is an open system It continuously interacts with its environment It takes inputs

Prepare the following questions from todayrsquos assignment

1 What do you mean by business environment

The term lsquobusiness environmentrsquo means the aggregate of all forces factors and institutions which are external to and beyond the control of an individual business enterprise but they may influence its functioning and performance Business environment is the macro framework within which a business firm a micro unit operates It consists of several interrelated and interacting elements

2 Explain the main features of business environment in brief

Totality of External forces-Business environment is the sum total of all things external to a business environment

General and Specific forces-It includes both the forces general forces are the economic social political legal and technological conditions which indirectly influence all business enterprise Specific forces are the investors customers competitors and suppliers which influence individual enterprise directly

Interrelatedness-Different elements of environment are interrelated for an example growing awareness for health care has increased the demand for health foods

Complexity- Business environment id

(such as raw materials capital labour energy and so on) from its environment transforms them into goods and services and sends them back to the environment

Fig 1 Business Environment Relationship

complex in nature as the elements keep on changing example economic technological and other forces changes in demand for a product and service

Dynamic-Business environment is not static it keeps on changing

Uncertainty- Itrsquos very difficult to predict future events such as technology and fashion which occur fast and frequently

Economics Basic Economic ConceptsSub topic

Microeconomics and

Macroeconomics

Welcome to the new sessiontoday we are going to start the first chapter of Class XI The name of the chapter that we are going to start is Basic Economic concepts

Now Economics covers the study of human activities Human activities are those activities which are performed by humans to satisfy their wants

Thus Human wants are unlimited and therefore economic activities such as production exchange and consumption are needed in order to satisfy those wants

The study of economics is divided largely in two parts which areMicroeconomics and Macroeconomics

SUBJECT- MATTER OF ECONOMICS

MICROECONOMICS MACROECONOMICS

Questions1Who has coined the words micro and macro economics

Answer Ranger Frisch coined the words lsquomicrorsquo and lsquomacrorsquo in 1933 to denote the two branches of economic theory namely microeconomics and macroeconomics

2What is microeconomicsAnswer It is the study of behaviour of individual decision ndash making unit such as consumers firms etc

3 What is macroeconomicsAnswer Macroeonomics is the study of overall economic phenomena like employment national income etc

4 What is the importance of microeconomicsAnswer

Microeconomics helps in formulating economic policies which enhance productive efficiency and results in greater social welfare

It helps the government in formulating correct price policies

It explains the working of a capitalistic economy where individual units(producers and consumers ) are free to take their own decision

Micro means a small part in

microeconomics we do not study the whole economy Hence we study an individual consumer and his or her choices and a producer and his or her profit maximizing decisions in the market Thus it does not mirror what happens in the economy as a whole

Macroeconomics on the other hand studies the economy as a whole It is concerned with aggregate and depicts the entire picture of the economyMacroeconomics deals with the national income aggregate investment aggregate consumption etc

Features of Microeconomics It deals with small

parts of the country Hence it looks at

individual consumers firms and industries

It deals with individual income consumption and savings

It studies the determination of price of any product or factors of production

It deals with the working of market via the price mechanism which is nothing but the determination of price and quantity of a commodity by the forces of demand and supply

Features of Macroeconomics

It deals with the study of the economy as a whole

It is concerned with

5 Give a limitation of microeconomics Microeconomics fails to explain the

functioning of an economy as a whole It cannot explain unemployment illiteracy and other problems prevailing in the country

6 What is the importance of macroeconomics It gives overall view of the growing

complexities of an economic system It provides the basic and logical

framework for formulating appropriate macroeconomic policies (eg for inflation poverty etc )to direct and regulate economy towards desirable goals

7What is the limitation of macroeconomics It ignores structural changes in an

individual unit of the aggregate

8 Differentiate between Microeconomics and Macroeconomics

Microeconomics Macroeconomics

the study of aggregates

National income aggregate savings and aggregate investments are major concepts dealt within macroeconomics style

It studies the determination of general price levels

It investigates into the problem of unemployment and the achievement of employment

It studies the aspect of decision making at the aggregate and national levels

It includes all growth theories whether related to developed or developing economies it also includes the study of economic systems and the working of the economy under different systems

Note Both Micro and macro economics are complementary and should be fully utilized for proper understanding of an economy

1It studies economic aspect of an individual unit2It deals with individual incomeConsumption and savings

3 It facilitates determination of price of any product or factors of production

4 Itrsquos scope is narrow and restricted to individual unit

1It studies the economy as a whole

2It deals with the national income aggregate consumption and aggregate savings3 It facilitates determination of general price level in an economy

4 Itrsquos scope is wide as it deals with economic units on the national level

ACCOUNTS

Introduction to Accounting and Book-keeping

Today I am going to share you the meaning of Accounting and Book-keeping and its related terms bullAccounting bullBook Keeping bullAccountsbullTypes Of Accounts bullAccounting Cycle

bull Meaning of accounting

Ans ) Accounting is the art and science of recording classifying and summarising monetary transactions

bull Meaning of Book-keeping

Ans) Bookkeeping is the art of recording business transactions with the view of having a permanent record of them and showing their effect on wealth

bull Meaning of account

Ans) The term account means a record of

business transactions concern a particular person of firm asset or income or expense It is a summarised record of all transactions which take place in an accounting year

bull Types of accountsPersonal accounts ndash Personal accounts relating

to person and Organisation are known as personal accounts Example Ramrsquos Account ABC amp Co Account etc

Real account - The accounts related to tangible and intangible assets are called real accounts Example Cash Account Furniture Account etc

Nominal account- Accounts related to expenses losses incomes and gains are known as nominal accounts Example Wages Account Salary Account Discount Account etc

bull Accounting cycle Accounting cycle refers to a complete sequence of accounting activities It begins with recording of transactions and ends with the preparation of a balance sheet

Chemistry TopicAtomic Structure

Thomsonrsquos atomic modelThomson (1898) was the first to propose the model of an atomHe proposed that an atom can be regarded as a uniform sphere of positive electricity in which requisite number of electrons are embedded evently to neutralize the positive chargeThis is just like plums embedded in a pudding or seeds evently distributed in red spongy mass of a watermelonThis model of atom is known as ldquoPlum-Pudding modelrdquo or

Q1)What is the fundamental constituents of atomAns Electron Proton and neutrons are the fundamental constituents of atomQ2)What is the value of fundamental unit of electricityAnsThe charge carried by one electron is sad to be the fundamental unit of electricityIts magnitude is 48times10-10esuOr 1602times10-19C Q3)Name the element containing no neutronAnsOrdinary hydrogen atom or protium 1H1

Types of AccountPersonal AccountReal AccountNominal AccountBalance Sheet (opening)

ldquowatermelon modelrdquoThis model could explain the electrical neutrality of an atom but failed to explain the result of scattering experiment carried out by Rutherford in 1911So it was rejected ultimately

Q4)Why is an electron called universal particleAns Itrsquos mass and Charge are independent of its source

EVS Chapter 1 ndash Modes of Existence

Modes of existence When one speaks normally about the mode of existence of some group or individual one refers to their customs their mode of being their ethology their habitat in some way their feeling for a placeDifferent modes of exixtence are ndash

1 Hunting ndashGathering2 Pastoral3 Agricultural4 Industrial

1 Hunting and gathering Hunting and gathering mode of existence is characterized by obtaining food from hunting wild animals including fishing and gathering wild plants From their earliest days the hunter-gatherer diet included various grasses tubers fruits seeds and nuts Lacking the means to kill larger animals they procured meat from smaller game or through scavenging

Societies that rely primarily or exclusively on hunting wild animals fishing and gathering wild fruits berries nuts and vegetables to support their diet are called hunting and gathering societies

At least this used to be practice of human beings before agriculture is invented As their brains evolved hominids developed more intricate knowledge of edible plant life and growth cycles

Q) Write the features of Hunting ndash gathering societiesAns - There are five basic characteristics of hunting and gathering societies

i The primary institution is the family which decides how food is to be shared and how children are to be socialized and which provides for the protection of its members

ii They tend to be small with fewer than fifty members

iii They tend to be nomadic moving to new areas when the current food supply in a given area has been exhausted

iv Members display a high level of interdependence

v Labor division is based on sex men hunt and women gather

Political Science

Introduction to political science

Political science occasionally called politology is a social science which deals with systems of governance and the analysis of political activities political thoughts associated constitutions and political behaviorThe study of political science involves the study of both the

Answer the following questions-1 What is political science

Political science occasionally called politology is a social science which deals with systems of governance and the analysis of political activities political thoughts associated constitutions and political behavior

2 Short notes-

traditional and modern theories of politicsTraditionalClassical political sciencepolitical theory-Traditional political science is the study of politics before Second World War The methodology to study Politics was traditional (legal formaletc) the definition of politics traditional (Politics begins and end with state)area of study (constitution state machinery)was traditionalModern Political scienceModern political theory-Modern Political Theory critically examines the contemporary state of political theory making an assessment of the achievement and limitations of the Behavioural Revolution in its totality and reviews objectively the major paradigms and conceptual frameworks adopted by the disciplineContemporary attempts at the development of an integrated political theory involving the use of both traditional and modern concepts approaches and theories-Around late 1960s several political scientists realized the importance of both the traditional political theory and modern Political theory They began building an integrated theory of politics involving a systematic mixture of traditional and modern studies of politics It was held that the study of a complex and vast field like politics needs both traditional as well as

Classical political theory Modern Political theory

Homework-Learn

modern concepts and approaches for studying itrsquos all aspects

Subject Eng Literature (The Tempest ndash William Shakespeare) Topic Act I Scene 1 Lines 1 to 32 (Line 32 ndash Gonzalo hellip If he be not born to be hanged our case is miserable) Date 13th April 2020 (3rd Period)

[Students should read the original play and also the paraphrase given in the school prescribed textbook]Summary Questions amp Answers

[SUMMARY OF THE ENTIRE SCENE]

o The play starts with the scene of a severe storm at sea Alonso (King of Naples) Sebastian (Alonsorsquos brother) Ferdinand (Alonsorsquos son) Gonzalo Antonio (the usurping Duke of Milan) are in a ship in the midst of the storm

o The mariners are trying their best to control the vessel from running aground and are totally following the orders of their Master the Boatswain They have scant success

o The mariners become extremely unhappy and annoyed when most of the passengers arrive on the deck thereby hampering their effort to save the ship There is serious confrontation between them and the passengers who are part of the Kingrsquos entourage

o The mariners could not save the ship

SUMMING-UP

(i) Vivid description of the scene which gives a realistic description of terror and confusion of a tropical storm

(ii) Shows Shakespearersquos accuracy of knowledge in describing the naval operations and also matters of seamanship

(iii) The opening scene justifies the title ndash The Tempest

UNANSWERED QUESTIONS

(i) The King always travels with his entire fleet including his soldiers Where

(1) GONZALO Nay good be patient (Line 15-26)BOATSWAIN When the sea is Hence What cares these

roarers for the name of the king To cabin silence Trouble us not

GONZALO Good yet remember whom thou has aboardBOATSWAIN None that I more love than myself You are a

councillor if you can command these elements to silence and work

the peace of the present we will not hand a rope more use your authority If you cannot give thanks you have

lived so long and make yourself ready in your cabin for the mischance of the hour if it so hap [To the Mariners]

Cheerly good hearts [To Gonzalo] Out of our way I say

(a) To whom is the boatswain speaking What does he mean by lsquoNone that I more love than myselfrsquo

The Boatswain is speaking to Gonzalo the honest old councilor of the Duke of MilanBy using the words ndash lsquoNone that I love more than I love myselfrsquo means that for the Boatswain nobody is dearer to him than his own life

(b) What were the conditions that made the boatswain react in this way

The Boatswain reacts in this way because the storm is at sea and Alonso King of Naples Sebastian his brother Ferdinand his son Gonzalo Antonio the usurping Duke of Milan on board are in distress and in panic Thus they have rushed to the deck interrupting the work of the mariners

(c) What hope does Gonzalo take from the attitude of the boatswain

The insolent and authoritative attitude of Boatswain makes Gonzalo feel comforted He tells that there are no signs that the Boatswain will be drowned But his facial appearance and attitude shows that he is destined to die on land by hanging which in effect means that all on board will be saved Otherwise all the persons on board are doomed

(d) How can they lsquomake yourself ready in your cabinrsquo For what were they asked to make ready themselves

In order to make themselves ready in their cabin the

were the other ships

(ii) Why was the ship in that area Where was it coming from or going where

(iii) The ship broke apart What happened to those who were in the ship

passengers on board must prepare for death which they will possibly soon have to meetThey can retire to their cabins and offer prayers to the Almighty to save them from drowning

(e) What does the boatswain say when he is asked to be patient What does he order to the royal party

When the boatswain is asked to be patient and remain calm he says that he will be patient only when the storm will be over and the sea will be calm but as long as the storm blows and there is danger to the ship he cannot think of being patient He orders the royal party to go to the cabin and leave the mariners to their work

(2) GONZALO I have great comfort from this fellow (Line 27-36)

Methinks he hath no drowning mark upon him his complexion is perfect

gallows Stand fast good Fate to his hanging Make the rope of his destiny our cable for our own doth little advantage If he be not born to be hanged our case is miserable

(a) Why does Gonzalo regard the Boatswain in the midst of danger

In the midst of danger Gonzalo regards the boatswain because he feels that the Boatswain is a source of comfort and is bent upon to do his work sincerely which in this case is saving the ship and its passengers from the severest of raging storm

(b) What reasons does Gonzalo give when he says that none in the ship will die of drowning

Gonzalo is almost sure that none in the ship will die by drowning His says that there is no mark on the face of the boatswain that indicates that he will die by drowning On the other hand the lines on his face are strong indications that he will be hanged to death Therefore there shall be no danger of the shiprsquos sinking

(c) Explain the following ldquoStand fast good Fate to his hanging Make the rope of his destiny our cable for our own doth little advantage If he be not born to be hanged our case is miserablerdquo

The stated lines mean that if the will of destiny is to be carried out then the ship will not get wrecked and all the passengers will be saved The safety of the passengers therefore depends upon the will of fate being carried out in the case of the boatswain If however the boatswain is not to die by hanging then the passengers are also very unsafe because in that case the ship is likely to sink

(d) What order does the Boatswain give to the sailors

when he re-enters What does he say about the crying of the fellows inside the cabin

The boatswain orders the sailors to bring the topmast lower and bring the ship close to a stationary position with the help of the main sail He says that the fellows inside the cabin are moaning and crying in their distress louder than his voice and louder even than the roaring of the storm

Class XII (ScienceCommerceHumanities) Subject Topic Summary Execution

Computer Science

PropositionalLogic

Propositional logic is a procedure to provide reasoning through statementProposition A ststement that results in True or False is said to be proposition There are two types of propositionSimple proposition amp compound propositionSimple proposioton A simple proposition is one that is not a part of any other proposition Such sentential form of proposition is symbolized with english letters in short For example Ram is a claver student (TrueFalse)Where do you live (Not in True or False)Grapes are sweet (TrueFalse)It rains today (TrueFalse)Here we can see some statements anwer would be true or false but some staements answer can not give in terms of true or false Thus the sentences which can be answered in true or false are known as simple propositionAssigning propositon to a variableThe general syntax to assign propostion to a variable is as followsVariable = Simple propositonFor example A=Ram is a clever studentB= Grapes are sweetC= it rains todayCompound proposition

helliphellipto be continued in next classhelliphellipMath Relation Relation If A and B are two non-empty sets

then a relation R from A to B is a subset of AxB If R A x B and (a b) R then we say that a sube isinis related to b by the relation R written as aRbeg Let A be the set of students of class XII and B be the set of students of class XI Then some of the examples of relation from A to B arei) (a b) AXB a is brother of bisinii) (a b) AXB age of a is more than age of isinb Types of relation In this section we would like to study different types of relations We know that a relation in a set A is a subset of A times A Thus the empty set φ and A times A are two extreme relations For illustration consider a relation R in the set A = 1 2 3 4 given by R = (a b) a ndash b = 10 This is the empty set as no pair (a b) satisfies the condition a ndash b = 10 Similarly R = (a b) | a ndash b | ge 0 is the whole primeset A times A as all pairs (a b) in A times A satisfy | a ndash

Example 1 Let A be the set of all students of a boys school Show that the relation R in A given by R = (a b) a is sister of b is the empty relation and R = (a b) the primedifference between heights of a and b is less than 3 meters is the universal relationSolution Since the school is boys school no student of the school can be sister of any student of the school Hence R = φ showing that R is the empty relation It is also obvious that the difference between heights of any two students of the school has to be less than 3 meters This shows that R = A times A is primethe universal relation Example 2 Show that the relation R in the set 1 2 3 given by R = (1 1) (2 2) (3 3) (1 2) (2 3) is reflexive

b | ge 0 These two extreme examples lead us to the following definitionsDefinition 1 A relation R in a set A is called empty relation if no element of A isrelated to any element of A ie R = φ A times AsubDefinition 2 A relation R in a set A is called universal relation if each element of A is related to every element of A ie R = A times A Both the empty relation and the universal relation are some times called trivial relation Definition 3 A relation R in a set A is called(i) reflexive if (a a) R for every a Aisin isin(ii) symmetric if (a1 a2) R implies that (aisin 2a1)

R for all aisin 1 a2 Aisin(iii) transitive if (a1 a2) R and (aisin 2 a3) R isinimplies that (a1 a3) R for all aisin 1 a2 a3 AisinDefinition 4 A relation R in a set A is said to be an equivalence relation if R is reflexive symmetric and transitive

but neither symmetric nor transitiveSolution R is reflexive since (1 1) (2 2) and (3 3) lie in R Also R is not symmetric as (1 2) R but (2 1) isin notinR Similarly R is not transitive as (1 2) R and (2 3) R but (1 3) R isin isin notinExample 3 Show that the relation R in the set Z of integers given byR = (a b) 2 divides a ndash b is an equivalence relationSolution R is reflexive as 2 divides (a ndash a) for all a Z isinFurther if (a b) R then 2 divides a isinndash b Therefore 2 divides b ndash a Hence (b a) R which shows that R is isinsymmetric Similarly if (a b) R and (b c) R isin isinthen a ndash b and b ndash c are divisible by 2 Now a ndash c = (a ndash b) + (b ndash c) is even (Why) So (a ndash c) is divisible by 2 This shows that R is transitive Thus R is an equivalence relation in ZExample 4 Let L be the set of all lines in a plane and R be the relation in L defined as R = (L1 L2) L1 is perpendicular to L2 Show that R is symmetric but neither reflexive nor transitiveSolution R is not reflexive as a line L1 can not be perpendicular to itself ie (L1 L1) R notinR is symmetric as (L1 L2) Risin

L1 is perpendicular to L2rArr L2 is perpendicular to L1rArr (L2 L1) RrArr isin

R is not transitive Indeed if L1 is perpendicular to L2 and L2 is perpendicular to L3 then L1 can never be perpendicular to L3 In fact L1 is parallel to L3 ie (L1 L2) R isin(L2 L3) R but (L1 L3) Risin notin

Chemistry Solid state Characteristics if Solids(i)The particles are locked in fixed positions they are unable to change their relative positions and this brings a definite shape and volume of a solid(ii)In a solid the constituent particles are held by strong forces of attractionThe forces of attraction may be bonding or non bonding(iii)The constituent particles in a solid pack together as closely as possibleoccupying most of the available space within the solidThus the empty space in a solid is very smallThis makes a solid highly rigid and nearly incompressibleThis also explains why a solid has high density and exhibits slow diffusionClassification of Solids

Q1)Define Crystalline solids AnsA Solid that has a definite geometrical shape and a sharp melting pointand whose constituent particles (atomsmolecules or ions) are arranged in a long range order of definite pattern extending throughout the solid is called a crystalline solidExNaClQ2)Define Amorphous solids AnsA solid that does not have a definite shape and a sharp melting pointand whose constituent particles (atomsmolecules or ions) are not arranged in a definite pattern is called an amorphoussolid

Crystalline solidsAmorphous solids

ExGlassRubberQ3)Classify Crystalline Solids Crystalline Solids

Physics Coloumbrsquos Law (Summary)

Before Going Into Coloumbrsquos Law We Will First Learn What is Charge Properties of Charge and Always remember that charge is quantized ie a body always have static charge of magnitude equal to some integral multiple of fundamental electronic charge e= 16 x 10- 19 C

Charge is the property of matter that causes it to produce and experience electrical and magnetic effects The study of the electrical charges at rest is called electrostatics When both electrical and magnetic effects are present the interaction between charges is referred to as electromagnetic

There exist two types of charges in nature positive and negative Like charges repel and unlike charges attract each other

The type of charge on an electron is negative The charge of a proton is the same as that of an electron but with a positive sign In an atom the number of electrons and the number of protons are equal The atom is therefore electrically neutral If one or more electrons are added to it it becomes negatively charged and is designated as negative ion However if one or more electrons are removed from an atom it becomes positively charged and is called a positive ion

The excess or deficiency of electrons in a body gives the concept of charge If there is an excess of electrons in a body it is negatively charged And if there is deficiency of electrons the body becomes positively charged Whenever addition or removal of electrons takes places the body acquires a charge

The SI Unit of charge is coulomb (C) In SI units the current is a fundamental quantity having a unit of ampere (A) The unit of charge is defined in terms of the unit of current Thus one coulomb is the charge transferred in one second across the section of a wire carrying a

Ionic SolidsMetallicSolids

Molecular Solids

current of one ampere

As q = It we have1 C = (1 A) (1 s)

The dimensions of charge are [A T]

Properties of Charge

(1) Quantization of Charge Electric charge can have only discrete values rather than any value That is charge is quantized The smallest discrete value of charge that can exist in nature is the charge on an electron given as

e = plusmn 16 x 10- 19 C

This is the charge attained by an electron and a protonA charge q must be an integral multiple of this basic unit That is

Q = plusmn ne where n = 1 2 hellip

Charge on a body can never be (frac12)e (23)e or 57e etcWhen we rub a glass rod with silk some electrons are transferred from the rod to the silk The rod becomes positively charged The silk becomes negatively charged The coulomb is a very large amount of charge A typical charge acquired by a rubbed body is 10 - 8 C

Biology Reproduction in organisms

Welcome to this new session 2020-21Today in this first chapter we mainly discuss about reproduction types needs and life span of some organismsWe also discuss about difference between sexual and asexual reproduction

Q1 What is reproductionReproduction is defined as a biological processin which an organism gives rise to young onessimilar to itselfQ2 What are the needs of reproductionbulli) Reproduction maintain life on earthii) It enables the continuity of the species generation after generationiii) It creates genetic variation among populationsQ3 Define Life span and write some orgnisms life spanbull Life span is the period from birth to

the natural death of an organism- OrganismsLife span1 Butterfly 1 - 2 weeks2 Fruit fly 30 days3Dog 10-13 years4 Rose5-7 years5 Tortoise100-150 years6 Banyan Tree -200 - 250 yearsQ4 Reproduction is of two types in case ofanimals but in case of plants vegetative propagation is also present

Asexual Reproduction Sexual Reproductioni) Always uniparentalii) Gametes are not involvediii) Only mitotic division involvediv) Somatic cells of parents are involvedv) Offsprings are genetically similar to the parents

i) Usually biparentalii) Gametes are involvediii) Meiosis occurs during gametogenesis Mitosis occurs after fertilisationiv) Germ cells of the parents are involvedv) offsprings are genetically different from the parents

COMMERCE BUSINESS ENVIRONMENT

Welcome to the new sessiontoday we are going to start the first chapter of Class XII The name of the chapter is Business Environment

Already many of you have got some idea about the word business environment form the first chapter of business studies in class XI

In todayrsquos world every business enterprise is a part of the society It exists and operates in association with various groups in society such as customers suppliers competitors banks and financial institutions government agencies trade unions media and so on All these groups influence the functioning of business in one way or the other They constitute the environment of businessConcept of Business Environment

The term lsquobusiness environmentrsquo refers to the sum total of all individuals institutions and other forces that lie outside a business enterprise but that may influence its functioning and performance

The main features of business environment Totality of External forces General and Specific forces Interrelatedness Complexity Dynamic Uncertainty

Prepare the following questions from todayrsquos assignment

2 What do you mean by business environment

The term lsquobusiness environmentrsquo means the aggregate of all forces factors and institutions which are external to and beyond the control of an individual business enterprise but they may influence its functioning and performance Business environment is the macro framework within which a business firm a micro unit operates It consists of several interrelated and interacting elements

2 Explain the main features of business environment in brief

Totality of External forces-Business environment is the sum total of all things external to a business environment

General and Specific forces-It

Relativity

The Interrelation between business and its environment

The business enterprise is an open system It continuously interacts with its environment It takes inputs (such as raw materials capital labour energy and so on) from its environment transforms them into goods and services and sends them back to the environment

Fig 1 Business Environment Relationship

includes both the forces general forces are the economic social political legal and technological conditions which indirectly influence all business enterprise Specific forces are the investors customers competitors and suppliers which influence individual enterprise directly

Interrelatedness-Different elements of environment are interrelated for an example growing awareness for health care has increased the demand for health foods

Complexity- Business environment id complex in nature as the elements keep on changing example economic technological and other forces changes in demand for a product and service

Dynamic-Business environment is not static it keeps on changing

Uncertainty- Itrsquos very difficult to predict future events such as technology and fashion which occur fast and frequently

Business Studies

Human Resources Management

Human resource of an organisation are the aggregate of knowledge skills attitudes of people working in it

The management system which deals with human resources is called human resource management

Features of HRMbullComprehensive functionbullPeople-oriented

Question1) What do you mean by human

resource management Answer) Human resource management may be defined as that field of Management which has to do with planning organising and controlling the functions of procuring developing maintaining and utilising the labour force

bullAction oriented bullPervasive function bullContinuous function

2) Explain the features of HRM in brief

Answer)bullHuman Resource Management is concerned with managing people at work bull Human Resource Management is concerned with employees which bring people and organisations together so that the goals of each are met bullHuman resource management considered every employees as an individual and also promote their satisfaction and growth bull Human resource management is inherent in all organisations and at all levelsbullManagement of human resources are ongoing on never ending process which requires a constant alertness and Awareness of human relations

3) ldquoHR function is said to be pervasiverdquowhy

Answer) Human resource management is required in all organisations whether it is private or government organisations armed forces sports organisations etc It permeatsall the functional areas like production marketing finance research etc This from this feature of human resource management it can be said that it is pervasive in nature

Economics Demand Q1DEFINITION OF DEMANDIn economics demand is the quantity of a good that consumers are willing and able to purchase at various prices during a given period of timeQ2DEMAND CURVEIn economics a demand curve is a graph depicting the relationship between the price of a certain commodity and the quantity of that commodity that is demanded at that pricQ3LAW OF DEMANDIn microeconomics the law of demand states that conditional on all else being equal as the price of a good increases quantity demanded decreases conversely as the price of a good decreases quantity demanded increasesQ4ASSUMPTION of LAW OF DEMAND(i)No change in price of related commodities(ii) No change in income of the consumer(iii) No change in taste and preferences customs habit and fashion of the consumer( No expectation regarding future change in priceQ5MARKET DEMAND SCHEDULEIn economics a market demand schedule is a tabulation of the quantity of a good that all consumers in a market will purchase at a

given price At any given price the corresponding value on the demand schedule is the sum of all consumersrsquo quantities demanded at that priceQ6INDIVIDUAL DEMAND SCHEDULEIndividual demand schedule refers to a tabular statement showing various quantities of a commodity that a consumer is willing to buy at various levels of price during a given period of timeQ7 FACTORS AFFECTING INDIVIDUAL DEMAND FOR A COMMODITY

The factors that influence a consumerrsquos decision to purchase a commodity are also known as determinants of demand The following factors affect the individual demand for a commodity1 price of the commodity2 price of related goods3 income of buyer of the commodity4 tastes and preferences of the buyer1 Price of the CommodityYou must have observed that when price of a commodity falls you tend to buy more of it and when its price rises you tend to buy less of it when all other factors remain constant (lsquoother things remaining the samersquo) In other words other things remaining the same there is an inverse relationship between the price of a commodity and its quantity demanded by its buyers This statement is in accordance with law of demand which you will study in the later part of this lesson Price of a commodity and its quantity demanded by its buyers are inversely related only when lsquoother things remain the samersquo So lsquoother things remaining the samersquo is an assumption when we study the effect of changes in the price of a commodity on its quantity demanded2 Price of Related goodsA consumer may demand a particular good But while buying that good heshe also asks the price of its related goods Related goods can be of two types-(i) Substitute goods(ii) Complementary goods While purchasing a good prices of its substitutes and complements do affect its quantity purchased(i) Price of Substitute Goods Substitute goods are those goods which can easily be used in place of one another for satisfaction of a particular want like tea and coffee An increase in price of substitute good leads to an increase in demand for the given commodity and a decrease in price of substitute good leads to a decrease in demand for the given commodity It means demand for a given commodity is directly affected by change in price of substitute goods For example if price of coffee increases the demand for tea will rise as tea will become relatively cheaper in comparison to coffee(ii) Price of Complementary goods Complementary goods are those goods which are used together to satisfy a particular want like car and petrol An increase in the price of complementary goods leads to a decrease in demand for the given commodity and a decrease in the price of complementary goods leads to an increase in demand for the given commodity For example if price of petrol falls then the demand for cars will increase as it will be relatively cheaper to use both the goods together So demand for a given commodity is inversely affected by change in price of complementary goods3 Income of the Buyer of CommodityDemand for a commodity is also affected by income of its buyer However the effect of change in income on demand depends on the nature of the commodity under consideration In case of some goods like full cream milk fine quality of rice (Basmati rice) etc demand for these commodities increases when income of the buyer increases and

demand for these commodities decreases when income of the buyer decreases Such goods whose demand increases with the increase in income of the buyer are called normal goods But there are some goods like coarse rice toned milk etc whose demand decreases when income of buyer increases and their demand increases when income of the buyer decreases Such goods whose demand decreases with the increase in income of the buyer are called inferior goods Suppose a consumer buys 10 Kgs of rice whose price is ` 25 per Kg He cannot afford to buy better quality of rice because the price of such rice is ` 50 per Kg The consumer is spending ` 250 per month on the purchase of rice Now if income of the consumer increases and he can afford ` 350 on purchase of 10 Kg of rice Now he can afford to buy some quantity of rice say 6 Kgs whose price is ` 25 per Kg and may buy 4 Kgs of rice whose price is ` 50 per Kg Thus he will buy 10 Kgs of rice by spending ` 350 per month Therefore we may conclude that demand for normal goods is directly related to the income of the buyer but demand for inferior goods is inversely related to the income of the buyer4 Tastes and Preferences of the BuyerThe demand for a commodity is also affected by the tastes and preferences of the buyers They include change in fashion customs habits etc Those commodities are preferred by the consumers which are in fashion So demand for those commodities rises which are in fashion On the other hand if a commodity goes out of the fashion its demand falls because no consumer will like to buy it(5) Number of Buyers in the Market(Population)Increase in population raises the market demand whereas decrease in population reduces the market demand for a commodity Not only the size of population but its composition like age (ratio of males females children and old people in population) also affects the demand for a commodity It is because of needs of children young old male and female population differs(6) Distribution of Income and WealthIf the distribution of income and wealth is more in favour of the rich demand for the commodities preferred by the rich such as comforts and luxuries is likely to be higher On the other hand if the distribution of income and wealth is more in favour of poor demand for commodities preferred by the poor such as necessities will be more(7) Season and Weather ConditionsThis is generally observed that the demand for woolens increases during winter whereas demand for ice creams and cold drinks increases during summer Similarly market demand for umbrellas rain coats increases during rainy seasonQ8 REASONS FOR OPERATION OF LAW OF DEMAND WHY DEMAND CURVE SLOPES DOWNWARDNow we will try to explain why does a consumer purchase more quantity of a commodity at a lower price and less of it at a higher price or why does the law of demand operate ie why does the demand curve slope downwards from left to right The main reasons for operation of law of demand are1 Law of Diminishing Marginal UtilityAs you have studied earlier law of diminishing marginal utility states that as we consume more and more units of a commodity the utility derived from each successive unit goes on decreasing The consumer will be ready to pay more for those units which provide him more utility and less for those which provide him less utility It implies that he will purchase more only when the price of the commodity falls2 Income Effect

When price of a commodity falls purchasing power or real income of the consumer increases which enables him to purchase more quantity of the commodity with the same money income Let us take an example Suppose you buy 4 ice creams when price of each ice cream is ` 25 If price of ice creams falls to ` 20 then with same money income you can buy 5 ice creams now3 Substitution EffectWhen price of a commodity falls it becomes comparatively cheaper as compared to its substitutes (although price of substitutes has not been changed) This will lead to rise in demand for the given commodity For example if coke and Pepsi both are sold at ` 10 each and price of coke falls Now coke has become relatively cheaper and will be substituted for Pepsi It will lead to rise in demand for coke4 Change in Number of BuyersWhen price of a commodity falls some old buyers may demand more of the commodity at the reduced price and some new buyers may also start buying this commodity who were not in a position to buy it earlier due to higher price This will lead to increase in number of buyers when price of the commodity falls As a result demand for the commodity rises when its price falls5 Diverse Uses of a CommoditySome commodities have diverse uses like milk It can be used for drinking for sweet preparation for ice cream preparation etc If price of milk rises its use may be restricted to important purpose only This will lead to reduction in demand for other less important uses When price of milk falls it can be put to other uses also leading to rise n demand for itQ9 EXCEPTIONS TO THE LAW OF DEMANDYou have studied in law of demand that a buyer is willing to buy more quantity of a commodity at a lower price and less of it at a higher price But in certain circumstances a rise in price may lead to rise in demand These circumstances are called Exceptions to the Law of Demand Some important exceptions are1 Giffen GoodsGiffen goods are special type of inferior goods in which negative income effect is stronger than negative substitution effect Giffen goods do not follow law of demand as their demand rises when their price rises Examples of Giffen goods are jowar and bajra etc2 Status Symbol GoodsSome goods are used by rich people as status symbols eg diamonds gold jewellary etc The higher the price the higher will be the demand for these goods When price of such goods falls these goods are no longer looked at as status symbol goods and tehrefore therir demand falls3 NecessitiesCommodities such as medicines salt wheat etc do not follow law of demandbecause we have to purchase them in minimum required quantity whatever their price may be4 Goods Expected to be ScarceWhen the buyers expect a scarcity of a particular good in near future they start buying more and more of that good even if their prices are rising For example during war famines etc people tend to buy more of some goods even at higher prices due to fear of their scarcity in near future

Political Science

Constitution of India-The

Preamble

The preamble-

Preamble-

The preamble is the most precious part of the constitution We the people of India having solemnly resolved to constitute India into a Sovereign Socialist Secular Democratic Republic and to secure to all its citizensA preamble is an introductory and expressionary statement in a document that explains the documents purpose and underlying philosophy When applied to the opening paragraphs of a statute it may recite historical facts pertinent to the subject of the statuteNature and purpose of the constitution-Purpose of the Constitution dictates permanent framework of the government to form a more perfect union to establish justice and ensure peace of thenationconstitution provide principles how the government can run itself following the rules and laws written in the constitution of each state keeps them balanced

Answer the following questions-

1 What is preambleA preamble is an introductory and expressionary statement in a document that explains the documents purpose and underlying philosophy2 What is the nature and

purpose of the constitutionConstitution dictatespermanent framework of the government to form a more perfect union to establish justice and ensure peace of the nation

Homework-Learn

Accounts Compatibilty mode

1MEANING OF PARTNERSHIPPartnership is a form of business organisation where two or more persons join hands to run a business They share the profits and losses according to the agreement amongst them According to the Indian Partnership Act 1932 ldquoPartnership is relation between persons who have agreed to share profits of a business carried on by all or any one of them acting for allrdquo For example one of your friends has passed class XII from National Institute of Open Schooling (NIOS) and wants to start a business Heshe approaches you to join in this venture Heshe wants you to contribute some money and participate in the business activities Both of you if join hands constitute a partnership2CHARACTERISTICS1048698 Agreement A partnership is formed by an agreement The agreement may be either oral or in writing It defines the relationship between the persons who agree to carry on business It may contain the terms of sharing profit and the capital to be invested by each partner etc The written agreement is known as partnership deed1048698 Number of persons There must be at least two persons to form a partnership

The maximum number of partners in a partnership firm can be 50 according toCompanies Act 20131048698 Business The Partnership is formed to carry on business with a purpose of earning profits The business should be lawful Thus if two or more persons agree to carry on unlawful activities it will not be termed as partnership1048698 Sharing Profits The partners agree to share profits in the agreed ratio In caseof loss all the partners have to bear it in the same agreed profit sharing ratio10486981048698Mutual Agency Every partner is an agent of the other partners Every partner can bind the firm and all other partners by hisher acts Each partner will be responsible and liable for the acts of all other partners10486981048698Unlimited liability The liability of each partner except that of a minor is unlimited Their liability extends to their personal assets also If the assets of the firm are insufficient to pay off its debts the partnersrsquo personal property can be used to satisfy the claim of the creditors of the partnership firm10486981048698Management All the partners have a right to mange the business However they may authorize one or more partners to manage the affairs of the business on their behalf10486981048698Transferability of Share No partner can transfer hisher share to any one including hisher family member without the consent of all other partners3PARTNERSHIP DEEDAgreement forms the basis of partnership The written form of the agreement is which a document of partnership is It contains terms and conditions regarding the conduct of the business It also explains relationship between the partners This document is called partnership deed Every firm can frame its own partnership deed in which the rights duties and liabilities of the partners are stated in detail It helps in settling the disputes arising among the partners during the general conduct of business 4CONTENTS OF PARTNERSHIP DEEDThe partnership deed generally contains the following (i) Name and address of the partnership firm(ii) Nature and objectives of the business(iii) Name and address of each partner(iv) Ratio in which profits is to be shared(v) Capital contribution by each partner(vi) Rate of Interest on capital if allowed(vii) Salary or any other remuneration to partners if allowed(viii) Rate of interest on loans and advances by a partner to the firm(ix) Drawings of partners and interest thereon if any(x) Method of valuation of goodwill and revaluation of assets and liabilities on the reconstitution of the partnership ie on the admission retirement or death of a partner(xi) Settlement of disputes by arbitration(xii) Settlement of accounts at the time of retirement or death of a partner5IN ABSENCE OF PARTNERSHIP DEEDThe partnership deed lays down the terms and conditions of partnership in regard to rights duties and obligations of the partners In the absence of partnership deed there may arise a controversy on certain issues like profit sharing ratio interest on

capital interest on drawings interest on loan and salary of the partners In such cases the provisions of the Indian Partnership Act becomes applicableSome of the Issues are(i) Distribution of Profit Partners are entitled to share profits equally(ii) Interest on Capital Interest on capital is not allowed(iii) Interest on Drawings No interest on drawing of the partners is to be charged(iv) Interest on Partnerrsquos Loan A Partner is allowed interest 6 per annum on the amount of loan given to the firm by himher(v) Salary and Commission to Partner A partner is not entitled to anysalary or commission or any other remuneration for managing the business

History TOPIC-TOWARDS INDEPENDENCE AND PARTITION THE LAST PHASE (1935-1947)

SUB TOPIC-IMPORTANT POLITICAL DEVELOPMENTS ndash GROWTH OF SOCIAL IDEAS

Socialism is a political social and economic philosophyLike in other parts of the world the Russian revolution of 1917 served as a great inspiration for revolutionaries in India who at that time were engaged in the struggle for liberation from British ruleSocialist ideas led to the formation of communist party of IndiaJAWAHARLAL NEHRU Among the early Congress leaders Jawaharlal Nehru was very much impressed and influenced by the Socialist ideas He also learnt about the Economic activities of the Soviet Union after the Bolshevic Revolution 1917 He made full use of them in IndiaThe election of Jawaharlal Nehru and Subhas Chandra Bose showed the Left wing tendency within CongressJawaharlal Nehru demanded economic freedom along with political freedom of the people in order to end the exploitation of masses

Nehrus working committee included three socialists leaders The Lucknow session was a landmark in the evolution of socialist ideas of the congressSUBHAS CHANDRA BOSE ndash Subhas Chandra Bose had socialist leaning Both Jawaharlal Nehru and Subhas Chandra Bose were known as leftist Congress men Later on National Congress divided into Leftist and rightist campCONGRESS SOCIALIST Within the Congress some leaders formed the Congress Socialist partyPattavi Sitaramyya Sardar Patel Rajendra Prasad had hostile attitude towards the Congress Socialist partyJawaharlals attitude was hesitant

1 QUESTION ndash Mention name of two Congress leaders who had socialist leaning

1ANSWER ndash Subhas Chandra Bose and Jawaharlal Nehru2QUESTION- In which session of the congress Jawaharlal elaborated his Socialist ideas2 ANSWER ndash Lucknow and Faizpur Session in December 1935 and 19363QUESTION ndash Why Congress was sharply divided into leftist and rightist camp 3ANSWER ndash Subhas Chandra Bosersquos attempt to seek re election for congress presidentship in 1939sharply divided the National Congress into Leftist and Rightist camp4 QUESTION ndash Who was MN Roy 4 ANSWER ndash Manabendra Roy first formed the Communist Party of India outside the country at Tashkent in 19205QUESTION ndash Who formed the Congress Socialist Party within the Congress5 ANSWER ndash Jaya Prakash Narayan Achyut Patwardhan Acharya Narendra Dev Ram Mohan Lohia Aruna Asaf Ali6QUESTION ndash When was the Congress Socialist Party formed What was its object6 ANSWER ndash 1934The Congress Socialist Party sought to work out socialist programme through the Congress They joined hands with the Congress and wanted to carry

Subhas Chandra Bose being expelled from the congress after the Tripuri rift he formed Forward BlockThere were basic differences between the Congress Socialists and the communistsTRADE UNION ACTIVITIES Maximum working class people lived in Bombay and Calcutta The working and living conditions of those workers were very miserable In this situation Shasipada Banerjee NM Lokhande protested against the oppression of the working class peopleThe first Trade Union Madras Labour Union was formed in 1918 by BP WadiaIndustrial strikes took place in Kanpur Calcutta Madras Jamshedpur and Ahmedabad AITUC was formed in Bombay in 1927 The growth of Trade union among the workers was slow because of the fear of the dismissal of the jobIn the mean time the Moderates as well as Communists left AITUC and formed separate organization

on National struggle with the help of workers and peasant class of the society7 QUESTION ndash What was the name of the party founded by Subhas Chandra Bose7 ANSWER- Forward Block8QUESTION ndash Who was Shasipada Banerjee8 ANSWER ndash Shasipada Banerjee was a radical Brahmo He founded a working menrsquos club to protest against exploitation of the British rulers towards the working class of India9 QUESTION ndash What was the weekly published by NM Lokhande9ANSWER- Dinabandhu10 QUESTION ndash Who founded Bombay Mill-Hands Association and in which year10 ANSWER- NM Lokhande in189011 QUESTION- Who was BP WadiaANSWER- BPWadia was the founder of Madras Labour Union in191812 QUESTION- What was the name of the first labour union of India12 ANSWER- Madras Labour Union13 QUESTION Who founded the Majur Mahajan 13 ANSWER GANDHIJI14 QUESTION What was the full form of AITUC When it was formed14 ANSWER All India Trade Union Congressin 192715QUESTION Who formed the Red Trade Union Congress and in which year15ANSWER The Communists formed the Red Trade Union Congress16 QUESTION What do you mean by Socialism16 ANSWER Socialism describes any political and economic theory that says the community rather than individuals should own and manage property and natural resources

Subject Eng Literature (The Tempest ndash William Shakespeare) Topic Act III Scene 3 Lines 1 to 52 (Line 52 ndash Brother my lord the Duke Stand to and do as we) Date 13th April 2020 (4th Period)

[Students should read the original play and also the paraphrase given in the school prescribed textbook]Summary Questions amp Answers

o Alonso Sebastian Antonio Gonzalo Adrian Francisco and others wandered about the island in search of Ferdinand and gets tired and hungry of the toil and at the same time gives up all hope of finding him

o Antonio and Sebastian are happy that Alonso is out of hope and decide to make another attempt on his life that night when being so tired they will be sleeping soundly

o Suddenly a solemn and strange music is heard in the air and several strange shapes enter bringing in a banquet These strange shapes then dance round it with gestures of salutation and then inviting the King to eat they depart

o Seeing this strange scene all are inclined to believe the tales told by travelers that there truly are ldquounicornsrdquo and ldquothe phoenixrsquo thronerdquo

1 ALONSO What harmony is this My good friends hark (L18-27)

GONZALO Marvellous sweet music

[Enter several strange shapes bringing in a banquet

they dance about it with gentle actions of salutation

and inviting the King and his companions to eat they depart]ALONSO Give us kind keepers heavens What were theseSEBASTIAN A living drollery Now I will believe

That there are unicorns that in Arabia

There is one tree the phoenixrsquo throne one phoenix

At this hour reigning thereANTONIO Ill believe both

And what does else want credit come to me

And Ill be sworn rsquotis true Travellers neer did lie

Though fools at home condemn rsquoem

(a) How did Prospero present an amazing spectacle before Alonso and his companions

Using his magic powers Prospero ordered strange shapes to lay a banquet before Alonso and his companions The shapes brought several dishes with tasty eatables in them They placed the dishes on a table before Alonso and his companions Then the strange shapes began to dance gracefully around the banquet While dancing they made gestures inviting them to eat the food Then suddenly the shapes disappeared(b) Who were the guests at the strange banquet Describe the lsquoliving drolleryrsquo

Alonso Sebastian Antonio Gonzalo Adrian and Francisco were the guests at the strange banquet

The term ldquoliving drolleryrdquo refers to live entertainment show In this context when Alonso the King of Naples Sebastian his brother Antonio the treacherous brother of Prospero Gonzalo the kind and loyal councillor to the King Adrian and Francisco came to the island they were hungry and weary in their spirits They heard a solemn and strange music They were shocked to see several strange shapes bringing in a banquet and these shapes danced about it with gentle action of salutation inviting the King and his companions to eat After this Sebastian described this show as lsquoliving drolleryrsquo(c) What is lsquophoenixrsquo What are lsquoUnicornsrdquo

The term lsquophoenixrsquo refers to a mythical Arabian bird which lived alone and perched on a solitary tree After one hundred years it expired in flames and rose again from its own ashes

lsquoUnicornsrsquo refers to the mythological four-footed beasts having horns in the centre of their foreheads When the horns are ground into powder the powder was believed to be

an aphrodisiac(d) How does Sebastian explain the puppet show OR Why does the speaker now believe in unicorns and phoenix

Sebastian finds several strange shapes bringing in the banquet They invite the king and his party for dinner and soon depart He tells that if such a strange sight can be a reality there is nothing incredible in the world and from the present moment he will believe anything He says that it is a strange dumb show enacted not by puppets but by living beings It is stranger than a travellerrsquos tale Seeing such a thing

before his own eyes he will no longer disbelieve the story about unicorns and phoenix(e) How do the other characters present respond to this living drollery

At the sight of the lsquoliving drolleryrsquo like Sebastian Gonzalo and Antonio too acted strangely Antonio told that he too now believes in unicorns and phoenix and anything else that seems to be incredible He too now believes in travellersrsquo tales Gonzalo told that if he would report those happenings in Naples nobody will believe him He considers that those gentle shapes were gentler in manner in comparison to the living beings Alonso was at first sight suspicious and told them that those strange shapes conveyed their meaning in expressive gestures when they seemed to lack speech by their movements and sounds Francisco was amazed at their mysterious disappearance

2 ALONSO Not I

(Line 43-52)GONZALO Faith sir you need not fear When we

were boysWho would believe that there were mountaineers

Dewlapped like bulls whose throats had hanging at rsquoem

Wallets of flesh Or that there were such men

Whose heads stood in their breasts Which now we find

Each putter-out of five for one will bring us

Good warrant ofALONSO I will stand to and feed

Although my lastmdashno matter since I feel

The best is past Brother my lord the Duke

Stand to and do as we

(a) How does Alonso respond at the spectacle of the shapes which were sent to them at the instruction of Prospero

After seeing the strange sight of appearing and disappearing of the shapes sent by Prospero to arrange a banquet for them Alonso says that his surprise at having seen those creatures is infinite and he is fully justified in feeling so much surprise He thinks that their shapes their gestures and the sounds they made were indeed amazing Although they do not possess the gift of speech yet they were able to convey their

thoughts by means of their gestures only

(b) What does Prospero say about the views expressed by Alonso regarding the shapes What does Francisco think about the shapesAfter hearing Alonsorsquos views about the shapes Prospero says that this manrsquos praise of the spirits is rather hasty He means to say that Alonso has shown great haste in reaching the conclusion about the shapes Francisco is amazed to see that those shapes disappeared in a mysterious way(c) What does Sebastian ask Alonso to doSebastian tells Alonso that the shapes having disappeared should not matter to them because they have left the eatables behind He asks Alonso to enjoy eating as they are extremely hungry but the king does not accept his offer of enjoying the dishes(d) How does Gonzalo try to dispel Alonsorsquos fear of those strange shapes What kind of references does he give to AlonsoGonzalo says that those who have travelled abroad have reported seeing even stranger sights than these shapes that Alonso and his companions have beheld Hence there is no reason to feel afraid of these shapes Gonzalo further adds that in his younger days he had heard strange stories from travelers and Alonso might have heard similar stories For instance it was said that there existed a certain race of

human beings who had huge lumps of flesh hanging at their throats and who therefore resembled bulls Then Gonzalo tells about a race of human beings whose heads were located at their breasts Gonzalo says that such stories were not believed by most people in those days but now-a-days these stories have become common(e) Explain the following lsquoEach putter-out of five for onersquoEnglish travellers often insured their trips with London brokers Those that went on foreign travels those days used to deposit a certain amount with some firm or company in London before their departure If the travelers failed to return the money was forfeited by the company with which it had been deposited But this money was repaid five-fold if the travelers returned safe and sound In this way a traveler stood a great chance of recovering the entire cost of his

travels(f) Give the explanatory meanings of the following expressions in the context of the above extract (i) Dewlapped (ii) Wallets of flesh

(iii) Putter-out(i) Dewlapped having big lumps of flesh at the necks(ii) Wallets of flesh large masses of flesh looking like bags(iii) Putter-out to invest money before commencing the travel

  • General methods of preparation of hydrogen
  • Chapter Dimensional Analysis (Summary)
    • Properties of Charge
Page 5:   · Web viewSubject. Topic. Summary. Execution. Hindi. व्याकरण. शरीरके अंगो के नाम लिखिए. 1) आँख 2) नाक 3

- 2 - 6 - 2 4 3 7 2 -- -- 4 ndash

Solution

179 3 7 18 6 8 5 - 2 1 6 - 2 4 3 7 2 1 4 4 2

Class IVSubject Topic Summary Execution

English language

Personal pronouns

The different forms of the three personal pronounsFirst personSingular I me my minePlural we us our oursSecond person Singular thou thee thy thinePlural ye you your yoursThird personSingular he him his her hers she it itsPlural they them theirs their

Pick out the personal pronouns Mention of what kind each (first second or third person)1 You should always try to keep a promise that you have madeYou second person personal pronoun

2 I listened patiently to what he had to say about themI first person personal pronounHe them third person personal pronoun

3 They packed their belongings and left I do not know where they have goneThey their third person personal pronounI first person personal pronoun

Social studies Major landforms on earth

Explained in previous lesson Answer the following questions3 Differentiate between hills and mountainsAns Mountains has a peak whereas hills have rounded topsMountains are steeper than hills whereas hills are lower and less steep4 Give two usefulness of mountainsAns Mountains are storehouse of waterMountains have a rich variety of flora and fauna

MATHEMATICS

Ch 5Subtraction

Estimating the differenceWe have learnt estimation in addition Applying the same rule We estimate the difference

Exercise 15 Example 2

A fruit-seller has 38210 bananas Out of these he sold 21799 bananas Estimate the unsold bananas

Solution Estimating to ten-thousandsTotal bananas = 40000Bananas sold out =- 20000Bananas unsold = 20000

Exercise1 Find the actual and estimated difference by rounding off to the nearest thousands(b) ActualEstimated

16190 - 2979

Solution (b) ActualEstimated 16190 16000 -2979 -30001321113000

2 Find the actual and estimated difference by rounding off to the nearest ten-thousands(b) ActualEstimated 73012 -28790

Solution (b) ActualEstimated 73012 70000 -28790-3000044222 40000

4 A bakery shop has an order to supply 6705 packets of biscuits There are only 4920 packets in the shop Estimate the number of packets needed to complete the supply

Solution Estimating to thousandsEstimated order = 7000Packets available= -5000Packets needed = 2000

there4 Estimated packets needed to complete the order were 2000

Hindi 2ndlang पतर पतर लिना भी एक का ह पतर लित समय धयान दना होगा निक पतर निकस लि रह ह कयोनिक पतर या तो वयवहारिरक होगा या वयावहारिरक होगापतर दो परकार क होत ह

क) औपचारिरक पतर जस परधानाचाय को नगर निनगम सपादक आदिद

ख) अनौपचारिरक पतर जस पापा को दोसत को बहन को अपन परिरजनो को

1 औपचारिरक पतर-अपन निवदयाय म अवकाश क लिए परधानाचाय को पतर लिखिए १०३कसीबीदम दमक टकोकाता-७०००६५सवा मपरधानाचायऋनिष अरहिवदो ममोरिरय एकडमीदिदनाक-१३४२०२०निवषय-अवकाश हत पतरमहोदया जीसनिवनय निनवदन ह निक म आपकी ककषा चौथी का निवदयाथj ह क रात स मझ तज बार ह डॉकटर न मझ आराम करन को कहा हअतः म निवदयाय आन म असमथ ह आपस निनवदन ह निक आप मझ दिदनाक १३स १७ तक की अवकाश दन की कपा कर हम आप क आभारी रहग

धनयवादआपका आजञाकारी लिशषयसीमा सिसहककषा-४

Bengali বইndashবোংো ভোষো পলিরচয়

পোঠndash৮ লিঙগ ৩ পংলিঙগ কোসক বসউঃ পরোলিবোচক লেP লিবসষয পসদর দবোরো পরষ োতীয় কোউসক লেবোঝোয় তোসক পংলিঙগ বস লেPম -বোবো 4োতর৪ সতরীলিঙগ কোসক বসউঃ পরোলিবোচক লেP লিবসষয পসদর তোরো সতরী োতীয় কোউসক লেবোঝোয় তোসক সতরীলিঙগ বস লেPম - মো লিলিকষকো৫ উভয়লিঙগ কোসক বস উঃ পরোলিবোচক লেPলিবসষয পসদর দবোরোপরষ ও সতরীউভয়োলিতসক লেবোঝোয় তোসক উভয়লিঙগ বস লেPম - লিশ মনতরী ৬ কলীবলিঙগ কোসক বস উঃ অপরোলিবোচক লেP লিবসষয পসদর দবোরো সতরী বো পরষ কোউসক ো বলিঝসয় লেকো2 পদো13 লেক লেবোঝোয়তোসক কলীবলিঙগ বস লেPম - বই

COMPUTER CHAPTER 3 EDITING IN MS WORD

MOVING THE TEXTMS WORD ALLOWS US TO MOVE A BLOCK OF TEXT OR GRAPHICS WITHIN A DOCUMENT

Q6) HOW TO WE USE THE COPY AND PASTE OPTIONAns) WE CAN CREATE A DUPLICATE COPY OF THE TEXTTHE STEPS TO COPY AND PASTE THE TEXT AS FOLLOWS----

SELECT THE PART OF THE TEXT THAT WE WANT TO COPY

SELECT THE COPY BUTTON IN THE CLIPBOARD GROUP UNDER THE HOME TAB OR PRESS CTRL + C KEYS

NOW PLACE THE CURSOR AT THE PLACE WHERE THE TEXT NEEDS TO BE COPIED

CLICK THE PASTE BUTTON IN THE CLIPBOARD GROUP UNDER THE HOME TAB OR PRESS CTRL + V KEYS

Science Adaptations in Animals

We find different kinds of animals in our surroundings We find birds flying in the sky monkeys jumping on trees fish in water camels in desert and so on Animals live in their natural homes which are called their habitats The habitat of a camel is a dry sandy desert and that of a tiger is a dense forest So different animals live in different habitats In the world a plant or an animal has to adapt or change itself to suits its surroundings This changes happens over hundreds and thousands of years A change that a living thing undergoes to become better suited to its surrounding is called adaptation

Fill in the blanks [pg no 40]1 Terrestrial animals live on

land 2 Polar bears are found in the

cold polar regions 3 Frogs have webbed feet that

help them to swim 4 Monkeys and koalas are

arboreal animals

Class VSubject Topic Summary Execution

COMPUTER130420

CHAPTER 2 APPLICATIONS OF COMPUTERS

PAGE NO-15C FILL IN THE BLANKS

INPUTS ANIMATION BANKS SOFTWARE SUPER COMPUTERS

D WRITE TRUE OR FALSE TRUE TRUE TRUE TRUE FALSE

Science Chapter 2 - The Skeletal System

Joints A joint is a place where two or more bones are joined with each other

There are two types of joints 1) Fixed joints 2) Movable joints

The movable joints are mainly four types ndash 1) Hinge joint 2) Pivot joint 3) Gliding joint 4) Ball and socket joint

DAnswer these questions

1)What are the functions of the skeleton

Ans ndash The functions of skeleton are -

i The skeleton gives our body shape ii The skeleton provide our body strength iii The skeleton protect our inner organs from outside injuries iv The skeleton gives our body support

2)List the type of movable joint in our body giving one example of each

Ans - The movable joints are mainly four types ndash a Hinge joint

Example of hinge joints are elbows nice fingers and toes

b Pivot joint Example - the joint between the skull and the backbone is the example of pivot joint

c Gliding joint Example of gliding joints

are wrist and ankle joints d Ball and socket joint

Example of ball and socket joints are shoulder joint and hip joint

English language

Transitive and intransitive verb

Pick out the verbs from the following sentences and say whether they are transitive or intransitive (page 21 ex B)1 gave ndash verb Transitive verb2 Has planted- verb Intransitive verb3 Were- verb Intransitive verb4 Asked- verb Transitive verb5 Told- verb Transitive verb

Social studies Conquering distances

Airways The only airline owned by the government is Air India which handles both domestic and international flightsAdvantages of airways Air transport is the fastestIt can access remote areasIt is the best means of transport in case of emergencies

1 Which is the only airline owned by the governmentAns Air India

2 Give two advantages of airwaysAns The two advantages of airways areAir transport is the fastest transportIt can access remote areas

DisadvantageIt is the most expensive of all other means of transport

3 What is the disadvantage of airwaysAns The only disadvantage of airways is that it is the most expensive of all other means of transport

MATHEMATICS

Ch 3Addition and Subtraction

Properties of Addition1 The sum of two numbers does not change when we change their order This property known as Commutative Property of addition

2 The sum of three numbers does not change when we change their grouping This property is called Associative Property of addition

3 The sum of the numbers and zero is the number itself This property is called Identity Property of addition and the integer 0 is called identity

Exercise 11Fill in the blanks1 2730815 + 8319293 = ____ + 27308152 18219 + 1850308 = 1850308 + ____3 (27815 + 85919) + 95985 = (85919 + ____) + 278154 13227 + (25983 + 73607) = (____ + 25983) + 736075 91389 + 0 = ____ + 91389

Solution 1 83192932 182193 959854 132275 0

6 Which of the following are true statements(a) Any number added to zero is zero

(b) The sum of two numbers does not change when we change their order

(c) 1 is the identity element of addition

(d) Given any three numbers their sum does not change when we change their grouping

Solution (a) False(b) True(c) False(d) True

Class VISubject Topic Summary Execution

HISTORY AND CIVICS

CHAPTER 3

MAHAVIRA AND BUDDHA ndash GREAT PREACHERS BUDDHA

Decline of Buddhism1 Revival of the Brahmanical Hinduism ndash Brahmin Scholars like Shankaracharya and Kumarila Bhatta led the revival of Hinduism and established the supremacy of Vedic religion2 Loss of Royal Patronage ndash Gupta period marked the decline of Buddhism as Gupta rulers were followers of Hinduism3 Split in Buddhism ndash division into Hinayana and Mahayana sects and rise of Mahayana sects blurred the line between Hinduism and Buddhism4 Corruption in Buddhist Sangha ndash due to generation of large revenue from large estates Buddhist monks and nuns started living luxurious

Answer the Following 1 During which dynasty Buddhism was split During the reign of Kanishka

2 Name the two sects of Buddhism Mahayana and Hinayana

3 Name two Vedic scholars who led the revival of Brahmanical Hinduism Shankaracharya and Kumarila Bhatta

life in rich monasteries Hence corruption crept in5 Adoption of Sanskrit ndash when Buddhist scriptures began to be written in Sanskrit in place of peoples language like Pali or Prakrit people started drifting away from Buddhism6 The Turkish Invasion ndash As Muslim conquerors invaded India immensely wealthy Buddhist Monasteries and temples were looted and destroyed and Buddhists were persecuted and killed

4 During which period decline of Buddhism began The Gupta Period

ENGLISH 2 The great train journey- Ruskin Bond

The great journey by Ruskin Bond is a story about Suraj who loved trains and wanted to go to places One day while wandering along the railway tracks he enters into a carriage compartment The train suddenly starts moving with him in the compartment and after a journey returns back to the same place from where it had begun The story is about his experience during that journey

4 Answer the following questionsf Who else is in the carriageA ragged hippy with a dirty beard face was in the carriageg Where does Suraj say that he would like to go toSuraj said that he would like to go to England and China and Africa and Greenland He wanted to go all over the worldh What warning does the man give to SurajThe man said Suraj to keep out of sight so that he doesnrsquot get caught by the ticket collectorsiWhen Suraj thinks about his parents for the first time what does he imagines that they will thinkSuraj thought that if he failed to come home that night his parents would think that he had run away or been kidnapped or been involved in an accidentJ What presents does Suraj imagine that he will bring back for his friendSuraj imagines that he would bring an African lion or a transistor- radio for his friend

CHEMISTRY

Chapter 2 ndashElement and Compound

ATOMAn atom can be defined as the smallest constituent particle of an element which showcases independent existence Example Ne OMOLECULEA molecule can be defined as the combinations of two or more atoms which are held together by chemical bonds A molecule is the smallest portion of a substance which showcases all the properties of the substance On breaking down a molecule further we see properties of the constituent elements Example HCl NaCl O2

Answer the following Q3) What is a moleculeAns - A molecule can be defined as the combinations of two or more atoms which are held together by chemical bonds A molecule is the smallest portion of a substance which showcases all the properties of the substance On breaking down a molecule further we see properties of the constituent elements Example HCl NaCl O2

Q4) Which can exist independently ndash atom or moleculesAns ndash Molecules can exist independently

PHYSICS Physical quantities

Guidelines for writing SI units correctly1 The units named after scientists are not written with a capital initial letter For example newton henry watt2 The symbols of the units named after scientist should be written by a capital letter For example N for newton H for henry W for watt3 Small letters are used as symbols for units not derived from a proper name For example m for metre kg for kilogram4 No full stop or other punctuation marks should be used within or at the end of symbols For example 50 m and not as 50 m5 The symbols of the units do not take plural form For example 10 kg not as 10 kgs6 When temperature is expressed in kelvin the degree sign is omitted For example 273 K not as 273o K (If expressed in Celsius scale degree sign is to be included For example 100o C and not 100 C)7 Use of solidus is recommended only for indicating a division of one letter unit symbol by another unit

Fill in the blanks

1) Length and mass are examples of fundamental physical quantities

2) The measurement of a physical quantity consists of two part magnitude and unit

3) A foot consist of 32 inches 4) The unit of temperature in the SI system is

Kelvin

Write true or false Correct the false statements

1) In ancient times cubit was used to measure the mass of an object FalseCorrect statement ndash In ancient times cubit was used to measure the length of an object

2) There are 7 fundamental physical quantities True

symbol Not more than one solidus is used For example m s-1 or m s J K mol or J K-1 mol-1 but not J K mol8 Some space is always to be left between the number and the symbol of the unit and also between the symbols for compound units such as force momentum etc For example it is not correct to write 23m The correct representation is 23 m kg m s-2 and not as kgms-29 Only accepted symbols should be used For example ampere is represented as A and not as amp or am second is represented as s and not as sec10 Numerical value of any physical quantity should be expressed in scientific notationFor an example density of mercury is 136 x 104 kg m-3 and not as 13600 kg m-3

3) Second is the unit of time in both the CGS and MKS systems True

4) The symbol used for a unit is always written in capital letters False Correct statement -The symbol used for a unit is normally written in small letters

Hindi 2nd language

वाकय निवचार भागवत निवचारो को परकट करन वा साथक एव वयवसथिtत शबद समह को वाकय कहत ह वाकय दो परकार क होत ह ndash

1 उददशय- वाकय म जिजसक बार म कछ बताया जाता ह उस उददशय कहत ह जस राधा एक नतकी ह2 निवधय- वाकय म उददशय क बार म बताया जाता ह उस निवधयक कहत ह जस- राधा एक नतकी ह रचना क आधार पर वाकय क तीन भद होत ह ndash१सर वाकय- राम बाजार गया २ सयकत वाकय- राम बाजार गया और वहा जाकर दोसत स मिमा३ मिमशर वाकय- यह वही tान ह जहा उनका बचपन बीता

helliphellipContinue to nextBengali 2nd language

লিZ সবরপ ও সবরলিZ

সবরলিZর লিয়ম - ১ অ-কোর লিকংবো আ-কোসরর পসর অ-কোর লিকংবো আ - কোর োকস উভয় লিমস আ ndashকোর য় এবং ওই আ ndash কোর পব13বস13 Pকত য়

২ ই - কোর লিকংবো ঈ - কোসরর পসর ই - কোর লিকংবো ঈ - কোর োকস উভয় লিমস ঈ - কোর য় এবং ওই ঈ - কোর পব13বস13 Pকত য়

৩ উ - কোর লিকংবো ঊ - কোসরর পসর উ - কোর লিকংবো ঊ - কোর োকস উভয় লিমস ঊ - কোর য় এবং ওই ঊ - কোর পব13বস13 Pকত য়

৪ অ - কোর লিকংবো আ ndash কোসরর পসর ই - কোর লিকংবো ঈ - কোর োকস উভয় লিমস এ - কোর য় এবং ওই এ - কোর পব13বস13 Pকত য়

১ অ + অ = আ ( gtো ) লিম + অচ = লিমোচ সব + অ3ী = সবো3ী অ + আ = আ ( gtো )পদম + আ = পদমো শভ + আলি = শভোলি আ + আ = আ ( gtো )4োয়ো + আবত = 4োয়োবত মো + আতমো = মোতমো আ + অ = আ ( gtো )লিবদযো + অংকোর = লিবদযোংকোর Pো + অ13 = Pো13 ২ ই + ই = ঈ ( gtী )অলিত + ইব = অতীব লিগলির + ইনদর = লিগরীনদর ই + ঈ = ঈ ( gtী )পলির + ইকষো = পরীকষো অলি3 + ঈশবর = অ3ীশবর ঈ + ঈ = ঈ ( gtী )মী + ঈশবর = মীশবর 3ী + ঈ = 3ী ঈ + ই = ঈ ( gtী )রী + ইনদর = রীনদর মী + ইনদর = মীনদর ৩ উ + উ = ঊ ( gt )মর + উদযো = মরদযো কট + উলিকত = কটলিকত উ + ঊ = ঊ ( gt )ঘ + ঊলিম13 = ঘলিম13 লিZ + ঊলিম13 = লিZলিম13 ঊ + ঊ = ঊ ( gt )রP + ঊলিম13 = রPলিম13

৪ অ + ই = এ ( লেgt )র + ইনদর = সরনদর লেPোগ + ইনদর = লেPোসগনদর অ + ঈ = এ ( লেgt )গ + ঈ = গস

র + ঈ = সর আ + ই = এ ( লেgt )Pো + ইষট = Pসষট 3ো + ইনদ = স3নদ আ + ঈ = এ ( লেgt )রমো + ঈ = রসম দবোরকো + ঈশবর = দবোরসকশবর

COMPUTER THE WORLD OF WINDOWS 10

DONE IN THE PREVIOUS CLASS PAGE NO-83A TICK THE CORRECT OPTION BACKGROUND DISPLAY AREA RESTORE THREE

MATHEMATICS Topic ndash NumbersChapter ndash Natural numbers and whole numbers

Study item Properties of whole numbers for subtraction1) Closure property When we do subtraction of two whole numbers we can not get a whole number in all time Example 8 ndash 3 = 5 a whole number 0 ndash 6 = -6 is not a whole numberTherefore the subtraction of two whole numbers is not satisfying closure property2) Commutative property If x and y are two whole numbers then x ndash y is not equal to y ndash xExample If x=16 and y = 7 then x ndash y = 16 ndash 7 = 9Again y ndash x = 7 ndash 16 = - 9 Therefore x ndash y not equal to y ndash x Therefore the subtraction of two whole numbers is not satisfy commutative3) Associative property If x y and z are three whole numbersThen x ndash ( y ndash z ) not equal to ( x ndash y ) ndash z Example If x = 20 y = 10 and z = 6Therefore x ndash (y ndash z ) = 20 ndash(10 ndash 6 ) = 20 ndash 4 = 16(X ndash y ) ndash z = (20 ndash 10) ndash 4 = 10 -4 =6Therefore x ndash(y ndash z) not equal to ( x ndash y) ndash zTherefore subtraction of whole numbers is not satisfying associativity4) Distributive property If x y and z are three whole numbersThen x (y ndash z ) = xy ndash xzAnd (y ndash z)x = yx ndash zxExample If x = 10 y = 6 and z = 4x(y ndash z ) = 10(6 ndash 4 ) = 10times6 ndash 10times4 = 60 ndash 40 = 20( 6 ndash 4 )times 10 = 6times10 ndash 4times10 = 60 ndash 40 = 20Therefore the subtraction of whole numbers is satisfying distributive property5) Existence of identity For any whole number x X ndash 0 = x but 0 ndash x = - x not equal to xThus for subtraction no identity number existsException 0 ndash 0 = 0 so 0 is its own identity for subtraction

Class VIISubject Topic Summary Execution

Hindi 2ndlang वचन जो सजञा शबद निकसी वसत या पराणी क एक या अनक होन का बोध कराया उनह वचन कहत ह जस डका- डकयह दो परकार की होती ह-

क) एकवचन-शबद क जिजस रप स उसक एक होन का बोध हो उस एक वचन कहत ह जस निकताब गमा आदिद

) बहवचन-शबद क जिजस रप स उसक आन ोन का पता च उस बहवचन कहत ह जस डक निकताब निततलियाआदिद

निनमनलिखित शबदो को एकवचन स बहवचन म बदोम- हमजानित- जानितयानारी- नारिरयामिमतर ndashमिमतरोपसतक -पसतकसडक-सडकबोत-बोतनाहर-नहररपए-रपया

Bengali বইndashবোংো োলিতয পলিরচয়

পোঠndash১৪ গলপ - অপর কলপো পর

লেক - লিবভলিতভষ বসনদযোপো3যোয়লেকndash রবীনদর পরবতf বোংো কোোলিতয 3োরোর উসgসPোগয োম পরকলিতসপরমী লিবভলিতভষ বসনদযোপো3যোয় তোর লেীসত লেPম বোসর বোসর লিফসর এসস4 গরোম বোংোর পরকলিতর কো লেতমলি এসস4 গরোমী মো লিচতরগলপndash অপর কলপো গলপোংটি লিবভলিতভষ বসনদযোপো3যোসয়র লিবযোত উপযো পসর পাোচোী লেসক গীত অপ অ13োৎ পসর পাোচোী তো অপর কলপোর লেকনদরীয় চলিরতর এই অংস আমরো পোই বোক অপসক বোক অপ কলপো লিবোী লে দসরর অ গো4 লেদস মোসয়র মস লেোো রপকোর রোসয পোলি2 লেদয় দপরসবো মোসয়র মসর কসর কোীদোী মোভোরত এর করসকষতর Pসjর ব13ো শস তোর মোবীর কস13র পরলিত ব2 মমতো য় আবোরপালিসত বলি13ত Pসjর অমোপত অং লে লিসই মোপত কসর বোলি2র লিপ4স বাো বোগোস লিকংবো উঠোসর লিশমসর কলপো লিবো এোস পরকো লেপসয়স4

১ অপর কলপো গসলপর লেক লেক তোর মপসক13 লেসো২ অপর কলপো গলপটি লেকোো লেসক গীত গলপটির ম ভোব লেসো

GEOGRAPHY CHAPTER 7EUROPE

CHAPTER COMPLETE EXERCISEFill in the blanks1 Europe is a continent that comprises the western part of Eurasia2 Eurasia and Africa are connected into one large land mass known as Afroeurasia3 The Strait of Gibraltar separates Europe and Africa4 Europe is surrounded by the Arctic Ocean to the north5 The British Isles includes the island countries of Great Britain and Ireland

Name the following 1 Connects Africa to Eurasia - Isthmus of Suez2 Largest country in the world in terms of area ndash Russia3 A term used collectively for the five countries in northern Europe ndash Nordic Countries4 The capital of Montenegro - Podgorica5 the largest fjord in Norway ndash Sognefjord

Match the following Column 1 Column 2a Albania iii Tiranab Belgium i Brusselsc Denmark v Copenhagend Finland ii Helsinkie Hungary iv Budapest

CHEMISTRY Chapter 2 ndashElement and Compound

Atom - An atom is the basic unit of an element or the smallest particle of an element non capable of independent existence Atom is built up of three sub atomic particles electron proton and neutron

Nucleus-It is the centre of an atom In the centre of the atom contains proton (positively charged particles ) and neutrons ( particles carrying no charge )

Orbits- It surround the nucleus in which revolve electrons (negatively charged particles)

Answer the following

1) What are MetalloidsAns - Certain elements using properties of both metal and non-metals are called metalloids Example Silicon arsenic and antimony

2) What are Noble gasesAns - Certain elements are present in the air and are chemically inert or unreactive Such elements are called rare gases or noble gases Example helium neon argon and Krypton

English 2 Sentences based on meanings

Kinds of sentences

Assertive or declarative to convey information or simply make a statement

Interrogative to ask different types of questions

Imperative to command or instruct someone or make a request

Exclamatory to express strong feelings and emotions

Exercise B1 Stop it ( Exclamatory)2 May you always be happy

together ( Exclamatory)3 He does not like sports

( Assertive)4 Please pass me the salt

( Imperative)5 How dare she talk to me like

that ( Exclamatory)6 May success bless your effort

( Exclamatory)7 Canrsquot you wait for sometime

(Interrogative)8 Did anybody tell you about it

( Interrogative)9 I saw her waiting for the bus

( Assertive)10 Could you please take a

message for me ( Interrogative)

Homework Ex ABiology Chp -2

Classification of Plants

Today we discuss about usefulness of bacteria We also discuss what the harmful effects of bacteria are

89 How bacteria are useful for usbull Bacteria is helpful in many ways forhuman being i) Production of medicine - antibiotics vaccine etcii) Formation of curd by lactobacillusiii)Nitrogen fixation in Leguminousplant by Rhizobiumiv) Increase soil fertility by absorbingatmospheric nitrogen and convert it into nitrates and nitritesv) Cleaning the environment by converting the complex substances into simple substancesvi) Tanning of leathervii) Retting of Fibersviii) Formation of compost by acting onanimal dung and agricultual cases1x) Biogas production by decomposingplant and animal wastex)Help In Nutrition by producing vitamiacutemBand kx1) Some bacteria are used to give specialflavour to tea coffee and coccaQ10- Name some diseases and there causativebacteriabull Diseasescausative bacteria1 CholeraVibrio cholerae2 Tuberculosis - Mycobacterium tuberculosis3 Diptheria -Corynebacteriumdiphtheriae4 Pneumonia - Streptococcus pneumoniae

Math Number system

Chapter Fraction

Study item Using lsquoofrsquoThe word lsquoofrsquo between any two fractions is to be used as multiplicationExample 57 of 56 = 57 times 56 = 5times8 = 40Study item Using BODMASThe word lsquoBODMASrsquo is the abbreviation formed by taking the initial letters of six operations(i)Bracket (ii) of (iii) Division (iv) Multiplication (v) Addition (vi) SubtractionAccording to BODMAS rule First of all the terms inside Bracket must be simplified then lsquoofrsquo lsquoDivisionrsquo lsquoMultiplicationrsquo lsquoAdditionrsquo lsquosubtractionrsquo

Study item Removal of Brackets

There are four Brackets of algebra in Mathematics In a complex expression four types of brackets are used Order of removing the brackets is first ----- then ( ) then finally [ ]

Class VIIISubject Topic Summary Execution

Chemistry Hydrogen General methods of preparation of hydrogen

By the action of dilute acids on metals

Calcium Reacts readily to form chloride salt and hydrogen

Ca + 2HCl rarr CaCl₂ + H₂uarr

Magnesium

Aluminium

Zinc

React readily to form salt and hydrogen

Mg + 2HCl rarr MgCl₂ + H₂uarr2Al + 6HCl rarr 2AlCl₃ + 3H₂uarrZn + 2HCl rarr ZnCl₂ + H₂uarr

Question 4 ) Give reasons for the following

(a) Hydrogen be used as a fuel

Solution

Hydrogen is used as a fuel because it has a high heat of combustion Some significant fuels are coal gas water gas and liquid hydrogen

(b) Though hydrogen is lighter than air it cannot be collected by downward displacement of air

Solution

Hydrogen is lighter than air so it is possible to collect the gas by downward displacement of air But it is not safe to do so since a mixture of hydrogen and air can lead to an explosion

(c) A pop sound produced when hydrogen is burnt

Solution

Impure hydrogen gas burns in air with a pop sound This is because of the presence of impurities in it

(d) Helium replaced hydrogen in weather observation balloons

Solution

It forms a mixture with air that can explode when there is a small leakage of hydrogen in a balloon So helium has replaced hydrogen

(e) Nitric acid not used for the preparation of hydrogen gas

Solution

(e) By the action of nitric acid on metals hydrogen cannot be produced because it also releases nitrous oxide and nitric oxide and oxides the hydrogen to form water

Biology Chp-2 Reproduction in plants

Today we discuss different methods of artificial propagation like cutting-rose sugercane Layering ndashguava lemon china rose etc Grafting- mango apple etcMicropropagation ndashorchid asparagus etcWe also discuss about advantages and disadvantages of vegetative propagation

Q7 Define the following terms i) Explant In tissue culture techniquea tiny piece of bud shoot or any other partof plant from where new tissue develop ii) Callus The cells of the tissue divide andgrow into a mass of undifferentiated cells from explant iii) Plantlet After few days callus differentiate into a small plant with roots and shootQ8 what are the advantages and limitations of tissue culture or micropropagation

Advantages i ) It produacuteces superior quality plantsii)It can be applied to interspecifie hybridsiii) It is useful to grow seedless plants bull Limitations i) It cannot be used for all plantsii)It is not easy to handleQ9 Write advantages of vegetative propagationi) It is a quick and easy method ofproducing new plantsii) This method need less time to matureiii) The new plants are exact copies of the parentiv) it is extremly useful for growing seedlessplants like banana grapes etc Q10 Write some disadvantages of vegetativepropagationi) Dišeases present in the parent plant gettransferred to all in new plantsii) Overcrowding of new plants causes competition for sunlight water and nutrients which affects growth of plantsplant

Physics Chapter 2 Physical Quatites and Measurements

Here We Will Do Some QuestionsRelated To Chapter 2

Select the correct alternative A block of wood of density 08gcm-3 has a volume of 60cm3 The mass of the block is

1 608 g

2 75 g

3 48 g

4 0013 g

Solution 348 g

The density of aluminium is 27g and that of brass The correct statement is

1 Equal masses of aluminium and brass have equal volumes

2 The mass of a certain volume of brass is more than the mass of an equal volume of aluminium

3 The volume of a certain mass of brass is more than the volume of an equal mass of aluminium

4 Equal volumes of aluminium and brass have equal masses

Solution 2 The mass of a certain volume of brass is more than the mass of an equal volume of aluminium

MATHEMATICS Ch 6Sets

Exercise 6(C)1 Find all the subset of each the following sets(i) A = 57 (iii) C = x xisin W x le 2(iv) p p is a letter in the word lsquopoorrsquo

Solution (i) All the subsets of A are ϕ 5 7 57

(iii) All the subsets of C are ϕ 0 1 2 01 02 12 012

(iv) All the subsets are ϕ p o r po or por

4 Given the universal set = -7-3-105689 find (i) A = x xlt2 (ii) B = x -4ltxlt6 Solution

(i) A = -7-3-10(ii) B = -3-105

5 Given the universal set = x xisin N and xlt20 find

(i) A = x x = 3p pisin N (iii) C = x x is divisible by 4 Solution

(i) 369121518 (iii) 481216

6 Find the proper subset of x x2-9x-10 = 0 Solution

ϕ 10 -1

Working x2-9x-10 = 0 rArr x2-(10-1)x-10 = 0

rArr x2-10x+x-10 = 0 rArrx(x-10)+1(x-10) = 0

rArr (x+1) (x-10) = 0

11 Let M = letters of the word REAL and N = letters of the word LARE Write sets M and N in roster form and then state whether (i) M sube N is true (ii) N sube M is true (iii) M = N is true

Solution M = real and N = lareSo (i) Yes (ii) Yes (iii) Yes

English 2 Twelfth Night ndash Shakespeare

A noble man named Orsino in the kingdom of Illyria is deeply in love with a lady called lady Olivia She is in mourning for her dead brother so she will not even think about marriage At this time a sea storm causes a terrible shipwreck and a young lady called Viola is swept onto the shore She thinks that her twin brother Sebastian is drowned A sea captain tells her about Orsino and his love for Olivia Viola wishes to work in Oliviarsquos home but feels she will not be employed So she dresses as a man calls herself Cesario and gets work at the house of OrsinoViola (now Ceasario) is much liked by Orsino and becomes his page She falls in love with Orsino Orsino sends Ceasario to deliver messages to Olivia Olivia herself falls for the beautiful young Ceasario believing Viola to be a man

2 Answer the following questionsa Why does Orsino ask the musicians to play onOrsino asks the musicians to play on because music feeds his desire He calls upon the musicians to play music so that his hunger for love could be replenished with an excess of musicb What does Valentine tell about OliviaWe learn from Valentine that Olivia is in mourning for her brother she wears a veil and has vowed that no one will see her face for another seven yearsand she refuses to marry anyone until thenc From the exchange between Orsino and Valentine what do you think their relationship isValentine is one of orsinod attendants He was sent to Olivia as a messenger of love but was not allowed to speak to here Who is Olivia mourning for and whyOlivia is mourning for her dead brother

Homework Q fHistory and Civics

Growth of Nationalism

Important dates to remember1769-Napoleon born on 15thAugust1789-Fall of Bastille on 14th July and the beginning of the French revolution declaration of the rights of Man on 26thAugust1793-King Louis XVI executed on January 211764-The Sugar Act passed1765-The Stamp act passed1774-The first congress of Philadelphia1776-The declaration of American Independence of on 4th July1777-Defeat of the British at Saratoga1781-Surrender of lord Cornwallis at Yorktown1783-The treaty of Versailles1804-Napoleon becomes the emperor1813-Battle of Leipzig or Battle of nations in which Napoleon was defeated by the Allies1815-Battle of Waterloo June 18 in which Napoleon was defeated and captured1821-Death of Napoleon in StHelena1860-Abraham Lincoln elected President of the USA1861-The civil war began 1864-Abraham Lincoln elected President of the USA for the second time1865-Slavery abolished in the US

Name the following- The queen of Louis XVI

Marie Antoinette The three philosophers of France

VoltaireMontesquieuJean Jacques Rousseau

The British general whose surrender brought the war in America to an endLord Cornwallis

The first president of the USAGeorge Washington (1732-1799)

The first southern state to secede from the unionSouth Carolina

The author of the book lsquoUncle Toms CabinHarriet Beecher Stowe

Homework-Learn

Class IXSubject Topic Summary Execution

Economics

Types of economies Today I am going to share you the concept of economic growth and economic development Few questions will be given from the previous study material dated 942020

Meaning of economic growthAnswer) The term economic growth generally means anincrease in national income or per capita output or income over time It indicates towards quantitative growth of a country

Meaning of economic developmentAnswer) Economic development is defined

as a process whereby the real per capita income of a country increases over time along with fall in poverty ratio unemployment and income inequality etc

Distinguish between economic growth and economic development

Basis Economic growth

Economic development

Scope It has narrow scope as it refer only to rise in per capita income

It has wide concept since it includes qualitative changes as well

Concerned matter

It is concerned with the rise in income

It is concerned with not only rise in income but also reduction of poverty income inequality and unemployment

Focus Economic growth does not focus on economic development

Economic development focus on economic growth plus qualitative changes

Distinguish between capitalist economy and socialist economy

Ownership

Motive

Tool

Means of production are owned and managed by private people

Self interest and profit earning is the main motive

Price mechanism is a main tool to solve the economic problems

Means of production are owned and managed by the government

Social welfare is the main motive

Economic planning by the government is the main tool to solve the economic problem

Competition

Distribution of income

There exist large competition among buyers and sellers

There is existence of large inequalities of income

There is no such competition

There exist less inequalities of income

Math Topic ndash AlgebraChapter -Factorisation

Study item Factorising by taking out common factorSome solved sums from exercise 41

1) (i) 8xy3 + 12x2y2

= HCF of 8xy3 and 12x2y2 is 4xy2

= 4xy2(2y + 3x )

4) (ii) 28p2q2r ndash 42pq2r2

= HCF of 28p2q2r and 42pq2r2 is 14pq2r = 14pq2r (2p - 3r )5) (ii) 14mn + 22m - 62p=HCF of 14mn 22m and 62p is 2= 2(7mn + 11m - 31p)7) (ii) 3a(x2 + y2) + 6b (x2 + y2) = HCF of 3a(x2 + y2) and 6b(x2 + y2 ) is (x2 + y2)= ( x2+ y2 )(3a + 6b )9) (ii) x(x2 + y2 ndash z2 ) + y(-x2ndashy2 + z2 ) ndash z(x2+ y2 ndash z2 )= x(x2 + y2 -z2) ndash y-(x2 + y2 -z2) -z(x2 + y2 ndash z2)=x(x2 + y2-z2) -y( x2 + y2-z2) ndash z (x2 + y2 -z2)= (x2+ y2 ndash z2)(x ndash y ndash z )

Commercial Studies

Introduction to Accounting and Book-keeping

Today I am going to share you the meaning of Accounting and Book-keeping and its related terms bullAccounting bullBook Keeping bullAccountsbullTypes Of Accounts bullAccounting Cycle

bull Meaning of accounting

Ans )Accounting is the art and science of recording classifying and summarising monetary transactions

bull Meaning of Book-keeping

Ans) Bookkeeping is the art of recording business transactions with the view of having a permanent record of them and showing their effect on wealth

bull Meaning of account

Ans) The term account means a record of business transactions concern a particular person of firm asset or income or expense It is a summarised record of all transactions which take place in an accounting year

bull Types of accountsPersonal accounts ndash Personal accounts relating

to person and Organisation are known

as personal accounts Example Ramrsquos Account ABC amp Co Account etc

Real account - The accounts related to tangible and intangible assets are called real accountsExample Cash Account Furniture Account etc

Nominal account- Accounts related to expenses losses incomes and gains are known as nominal accountsExample Wages Account Salary Account Discount Account etc

bull Accounting cycle Accounting cycle refers to a complete sequence of accounting activities It begins with recording of transactions and ends with the preparation of a balance sheet

English 1 Transformation of sentences

Sentences A sentence is a group of words which makes complete sense

a Assertive sentencesb Imperative sentencesc Interrogative

sentencesd Exclamatory sentences

Sentences can be changed from one grammatical form to another without changing the meaning of the sentence This is known as transformation of sentences

Exercise 6Rewrite the following sentences according to the instructions given below without changing their meanings

1 As soon as he saw the beer he jumped into the river ( Begin No sooner)

2 None but brave deserve the fair (Begin the bravehellip)

3 This box is too heavy for me to lift ( Use so hellip That instead of too)

4 No one other than a king can live like James Luxurious ( Begin only James)

5 Oh for the wings of a dove (Begin I wishhellip)

BENGALI(2ND LANGUAGE)

ldquo বঙগভমির পরমি ldquo াইকেল ধসদন দতত

পব13পোসঠ আসোলিচত ৩ পরবোস দৈদসবর বস ীবতোরো Pলিদ স এ লেদ -আকো সত-োলি লেদ তোস - ক) বকতো লেক কোর লেো লেকো কলিবতোর অং ) কোর পরলিত বকতোর এই উলিকত গ) এ লেদ আকো সত বসত কী বলিঝসয়স4 ীবতোরো বসত কী লেবোঝ ঘ ) আসোচয অংসর তোৎপP13 কী

উ -ক ) বকতো স কলিব মোইসক ম3দ দতত

Types of AccountPersonal AccountReal AccountNominal AccountBalance Sheet (opening)

কলিব মোইসক ম3দ দসততর রলিচত বঙগভলিমর পরলিত কলিবতোর অং ) কলিব বঙগী অ13োৎ লেদমোতোর পরলিত কলিবর এই উলিকত গ ) এ লেদ আকো বসত কলিবর মোব লেদী রপ আকো লেক লেবোঝোসো সয়স4 আকো লেসক লেPম তোরো স পসর লেতমলি ীব লেদ রপ আকো লেসক পরো রপ তোরো স পরসত পোসর এই মভোবোর কোই কলিব বসস4 ঘ ) পরবো Pোতরোয় Pলিদ কলিবর লেদ আকো লেসক ীব তোরো রপ পরো স পসর তোসত কলিব লিবনদমোতর দঃলিত কোর মতয লিবসর সবোভোলিবক পলিরলিত এবং মোষ মরী তোই পরবোস Pলিদ তা োর মতয য় তবও কলিব লিবচলিত সব ো কোর পলিবীসত লেকউ অমর য় লিক4ই অকষয় য় দীর লেPম লিচরপরবোমো লেতমলি মোসষর ীবও চমোতোই ীব - সতবধতোই মতয ীব দীসত মোষ লিতয পরবোমো তবও লেPব মোষ আপ কতকসম13র মো3যসম মোসষর মস লিসসদর সথো কসর লিসত পোসর তোরো লিচরভোসবর সয় মোসষর মস লিবরো কসর তোসদর মস3য লেকউ পGভসত লিবী সয় গোসও মোসষর মস তোরো লিতযপলিত লিতযবলিনদত

Hindi 2ndlang

काकीी(लिसयारामशरणगपत)

इस कहानी म क न यह बतान का परयास निकया ह निक बचच अपनी मा स निकतना परम करत ह शयाम अबोध बाक ह वह अपनी मा क मरन क बाद उसन अपनी मा क लिए बहत रोया बाद म उस पता चा निक उसकी मा राम क घर ची गई ह आकाश म उडती हई पतग दकर उस हष हआ निक पतग क दवारा वह अपनी मा को नीच उतारगा इसक लिए वह अपनी निपता की जब स दो बार सवा रपया निनकाकर पतग और दो मोटी सी मन वाी अपन भाई स काकी एक कागज पर लिवा कर पतग म लिशव का दिदयानिनकाकर पतग और दो मोटी सी मन वाी अपन भाई स काकी एक कागज पर लिवा कर पतग म लिचपका दिदयाभोा और शयाम कोठरी म रससी बाधनी रह थ तभी उसक निपता करोध म आकर उन स पछ निक कया उनकी जब स रपया निनकाा हभोा डर क मार बताया निक शयाम इस पतग क दवारा अपनी काकी को राम क यहा स उतारना चाहता हनिवशशवर(शयाम क निपता)न फटी पतग उठाकर दी तो उस पर काकी लिा थावह हत बजि होकर वही ड रह गएउनहोन सोचा निक मन अपन पतर को मारा जोनिक अनजान और निनदष थावह अपनी मा कोनिकतना पयार करता ह

helliphellipContinue to next

Computer Application

Java Programming Prog 1Write a java program to input two numbers from user and display the sum or product of them as per user choice Use switch case statementSolve public class sum_product public static void main(String args[]) Scanner sc=new Scanner(Systemin) int abc Systemoutprintln(ldquoEnter two numbersrdquo) a=scnextInt() b=scnextInt() Systemoutprintln(ldquoPress 1 for sum or 2 for productrdquo)

c=scnextInt() switch(c) case 1 Systemoutprintln(ldquoThe sum will be =rdquo+(a+b)) break case 2 Systemoutprintln(ldquoThe product will be =rdquo+(ab)) break default Systemoutprintln(ldquoWrong Inputrdquo) Home Work - Practice in your computer using bluej

Subject Eng Literature (The Merchant of Venice ndash William Shakespeare)Topic Act I Scene 2 Lines 92 to 126 (End of scene) Date 13th April 2020 (5th Period)

[Students should read the original play and also the paraphrase given in the school prescribed textbook]Summary Questions amp Answers

o After Portia has expressed her opinion about the suitors Nerissa informs that she need not bother about any one of them as they have decided to quit Belmont at the earliest opportunity because they do not believe in trying their luck by the caskets which is the only way of winning Portia

o Nerissa then enquires of Portiarsquos opinion about Bassanio who once visited her in the company of the Marquis of Montferrat and says that she had never come across such an ideal love deserving the fairest lady for his bride

o Portia seems to remember Bassanio quite correctly and says that she agrees with Nerissa At this moment a servant informs Portia that the Prince of Morocco has arrived to try his luck by the caskets

o Portia tells Nerissa that if she could welcome this new suitor as gladly as she says farewell to the previous ones she would be glad of his arrival However if he happens to have the virtues of a saint but the black complexion of a devil she would prefer to have him for religious consolation rather than as a husband

(1) NERISSA You need not fear lady (Line 97-103)

the having any of these lords they have acquainted me with their determinations

which is indeed to return to their home and to

trouble you with no more suit unless you may be wonby some other sort than your fathers imposition depending on the caskets

PORTIA If I live to be as old as Sibylla I will die as chaste asDiana unless I be obtained by the manner of my fatherswill I am glad this parcel of wooers are so reasonablefor there is not one among them but I dote on his veryabsence and I pray God grant them a fair departure

(a) Elucidate the idea expressed in the first speech of the above dialogue

In the first speech Nerissa assures Portia that she need not have any fear of being compelled to marry anyone of the suitors who had lately come to Belmont She informs her that they have all decided to return to their respective countries(b) Illuminate the meaning of the phrase ldquoyour fatherrsquos imposition depending on the casketsrdquo

Nerissa means that the suitors of Portia do not find the conditions imposed by the will of her father to their liking They are too hard for them These conditions are that in the event of a suitor failing to choose the right casket (i) he should never disclose to anybody which casket he chose (ii) he can never marry and (iii) he should take his departure immediately(c) Explain the meaning of the term lsquoSibyllarsquo

lsquoSibyllarsquo is the name given by Romans and Greeks to a prophetess inspired by some deity usually the sun-god Apollo She had a very long life The god Apollo granted her as many years of life as she could hold grains of sand in her hand(d) Elucidate the meaning of the term lsquoDianarsquo

lsquoDianarsquo is the goddess of hunting She is also regarded as a symbol of virginity because she never fell in love and never

married(e) Explain the meaning of the first two lines of Portiarsquos speech

Portia says that even if she is to live for centuries like Sibylla she would not marry except in accordance to her fatherrsquos will She asserts that she would not mind remaining unmarried and untouched by a man like Diana the virgin the goddess of hunting unless a man is able to win her by passing the test laid down by her father

Class XSubject Topic Summary Execution

Hindi 2nd

Langबड घर की बटी( मशी परमचद)

lsquoबड घर की बीटीrsquo कहानी का उददशय मधयम वग की घर समसया को सझा कर सगदिठत परिरवार म मिम जकर परम स रहन का सदश दना ह घर म शानित tानिपत करन की जिजममदारी नारी की होती ह यदिद नारी समझदार ह उसम धय और परिरवार क परनित परम ह तो कोई भी घटना परिरवार को निवघदिटत नही कर सकती या कहानी परिरवार को सगदिठत करत हए परम सौहाद स एक रदसर की भावनाओ को समझ करउनका सहयोग करत हए जीवन यापन करन की पररणा दती ह मशीपरमचदर जी न इस कहानी म सयकत परिरवार का परनितनिनमिधतव निकया ह यह कहानी बनी माधव सिसह जो गौरी पर क जमीदार क उनक दो पतरो की हशरी कठ ा निबहारीशरीकात का निववाह एकजमीदार घरान की पतरी आनदी स हआ थाआनदी न द को ससरा क वातावरण म ढालिया थाएक दिदन आनदी का अपन दवर ा निबहारी स झगडा हो जाता ह दोनो भाई एक रदसर स अग होन की कोलिशश करत हसभी बह आनदी न अपन मधर वयवहार स ा निबहारी को घर छोडकर जान स रोक लिया| इस पर बनी माधव सिसह न कहा निक बड घर की बटी ऐसी ही होती ह जो निबगडा काम बना ती ह अतः शीषक साथक ह बड घर की बटी आनदी ह

helliphelliphelliphellipContinue to nextBiology Topic ndash Chp-1

CellWelcome to new session 2020-21Today we will start with Chpter 1 cell CELL

Protoplasm+Cellmembrane Or Cell wall

Cytoplasm+Neucleus

Cytoplasmic+ CytoplasmicOrganelles Inclutions(mitochondria (food Golgi bodies pigments)Ribosome)

What is cellbull Cell is the structural and functional unit of living organismbull According to number of cells organisms areUnicellular - Amoeba bacteria Multicellular - Rose Mango Tiger HumanSmallest cell -bacteria Longest cell - Nerve cellLargest cell - Ostrich egg cellCells are of different size and shapes according to their functionsQ2Write chief functions of following cellorganelles

Q3What is tonoplastVacuoles covered by a covering called tonoplast

Bengali(2Nd

Language)

ফ ফটক ো ফটক (কলিবতো ) ভোষ মসোপো3 gtPোয়

একটি লেমসয়র ীবস লেপরম লিকভোসব ফসট ওসঠ তো লেদলিসয়স4 কলিব লেপরম Pই য় লেই ময়ই বনত কোস পলিরত য় ফ লেফোটো বো োসফোটো লেটো ব2 কো য় লেমসয়সদর ব gtয13 লেপরসমর 4লিব ফসট উসঠস4 এই কলিবতোয় লেপরম মোষসক মত gtযর মস লেফস লিদসয় পরকষস বাোচোসোর gtয োত বো2োয় কলিবতোয় লেমসয়টির পসব13র দঃসর কো বো সও লেমসয়টি লেই পসর পলিক সত চোয়ো োরী ীবসর কোস4 পরম লেPৌবস লেপরমসক পোবোর পরব ইচছো োকসও তো পসর লেলিতবোচকতোয় পলিরত য় কলিব ভোষ মসোপো3 যোয় লেP ক লেপরসমর

কলিবতোয় ব gtযবহত লিবসষ লিক4 সvর অ13 লেদওয়ো ১) রসবোো= লেP লিবলিভনন রকম ডোকসত পোসর২) ো= পোর ৩) ঠলি = লেচোসর বZ৪)আই বস2ো=অলিববোলিত৫)শইসয় = োলিয়ত কসর৬)োতপাোচ= লিবলিভনন পরকোর৭)দ2োম = v কসর বZ কসর লেদওয়ো৮)লেরলিং =লেোোর দৈতরী লেব2ো৯) বনত= একঋত১০) পাোর = বসকরো2

Organelles Functions

1 Endoplasmic reticulum

2 Mitochondria

3Golgibodies

4 Ribosome5Lysosome

6Plastids

7 Centrosome

i) Supportive framework for the cellii) Synthesis and transpost of proteinsRelease of energy in the form of ATPi) Synthesis and secretion of enzymes hormoneii) Formation of vacuoles lysosomei) Protein Synthesisi) Intracellular digestionii) Destroy foreign substancei )Leucoplast - stores starchii)chloroplast - trap solar energyiii) Chromoplast - imparts colour toflowers amp fruitsi) Initiates and requlates cell division

কলিবতো তোর অ13সক ভোষোয় পরকোো কসর ঘলিরসয় ব যকত কসরস4 লেপরসমর ফতো আর লিবফতো লেক গোঢ় কসর লেদোসো কলিব ভোষ মসোপো3 যোসয়র অলিভবসর অ যলিদক

Economics

Factors of Production

Welcome to the new sessionToday we are going to start the first chapter of Class XThe name of the chapter is Factors of productionBy the name I hope you all can recall a glimpse of what you have learnt in the second chapter of Class IX

NowProduction is the process of creating the various goods and services which are consumed by the people of the country to satisfy their wants

Thus it is the process in which some materials are transformed from one form to another to create utility and value in goods

For example utility can be created by changing the form of a commodity ie

Making of table out of wood by a carpenter for his customer here the wood is getting transformed into table creating utility for his customer and he can also command a price for it

On the other hand Housewives perform very

useful activities at home which create utility but their domestic activities are not included in production because they have no money value

So we can also say that Production denotes two things firstly creation of utility and secondly creation of value

Production is not complete unless it reaches the consumer

An increase in production will increase the economic welfare of the consumers and hence the aim is to raise the production level of the country

Again production of a good or service is only possible if certain resources or

Questions

1 What do you mean by production

Answer Production means the creation of goods and services for the purpose of selling in the market

In fact production involves the transformation of inputs into outputs

Hence production denotes two thingsCreation of utility and creation of valueUtility and value can be created by changing the form by changing the place by changing the time and by rendering services

Example Transformation of raw

materials into finish goods such as potter creates utility by converting mud into utensils assembling of small parts to make bigger machinery

Production also includes services such as distribution and marketing

2 What are the factors of production

Answer Factors of Production refers to the resources and inputs needed for producing goods and servicesThese inputs can be classified as

Land Labour

Capital Enterprise

Land Land is defined to include not only the surface of the earth but also all other free gifts of nature(for example mineral resources forest resources and indeed anything that helps us to carry out the production of goods and services but is provided by

inputs are used together in right proportion

A resource or an input which helps in the process of production to obtain an output is called FACTOR OF PRODUCTION

These factors of production can broadly be categorized into four parts 1LAND 2LABOUR3CAPITAL4ENTERPRISE (ORGANISATION)or Entrepreneur

The above factors are all interdependent on each other and they play a major role in production process

FACTORS OF PRODUCTION

LANDCAPITAL

LABOUR ENTREPRENEUR

nature free of cost)LabourLabour refers to the human efforts that need to be combined with other factors of production for creating an output

CapitalAll man ndash made means of production is called capita example machineries which help in further production Money when used for starting any business for purchasing raw materials machinery tools etc it is regarded as capitalCapital also includes physical capital like factories machineriestoolsbuildingsequipments etcEnterpriseThe task of bearing risks is called enterprise and the person who bears these risks of business is called the entrepreneurThus an entrepreneur is one who organises production takes important decisions regarding production hires and purchases factors of production and bears the risk and uncertainty involved in productionOrganisation refers to the services of an entrepreneur who controls organises and undertakes all risks One who plans organises and manages a business enterprise is an organiser

Physics Chapter 1 Force

Force is an external agent capable of changing the state of rest or motion of a particular body It has a magnitude and a direction The direction towards which the force is applied is known as the direction of the force and the application of force is the point where force is applied The Force can be measured using a spring balance The SI unit of force is Newton (N)

Question 1

State the condition when on applying a force the body has

(a) the translational motion

(b) The rotational motion

Solutions

(a) Translational motion is produced when the body is free to move

(b) Rotational motion is produced when the body is pivoted at a point

Question 2

Define moment of force and state its SI unit

Solutions

The moment of force is equal to the product of the magnitude of the force and the perpendicular distance of the line of action of force from the axis

of rotation

The SI unit of moment of force is Newton times meter

= Newton meter (Nm)

Commercial Studies

Stake holders In this topic you will be come to know about the meaning and concept of stakeholders

How stakeholders are different from shareholders

Questions1 What do you mean by the term stake holdersAnswer) The term stake holders have developed from the words which mean an interest or expected benefit Stakeholders mean all those individuals groups and Institutions which have a state (interest) in the functioning and performance of a commercial organisation or a business enterprise2 What do you mean by share holdersAnswer) The person and Groups who own the shares of the joint stock company by providing capital to the company are called shareholders Shareholders are the internal stakeholders shareholders are one out of several stake holders3 How are shareholders different from stakeholdersAnswer)i) The term shareholders is related to only joint stock company whereas stakeholders are related with all business organisationsii) Stakeholders maybe any individual having financial stake in business organisation whereas a shareholders are those individuals who are holding shares in the company4) How are shareholders different from creditorsAnswer) i) Shareholders are internal stakeholders while creditors are external stakeholdersii) Shareholders invest in the capital of the company whereas creditors give loan to the companyiii) Shareholders are the members of the company with voting rights but creditors are not the members of the company

English 1 Transformation of sentences

Sentences A sentence is a group of words which makes complete sense

e Assertive sentencesf Imperative sentencesg Interrogative sentencesh Exclamatory sentences

Sentences can be changed from one grammatical form to another without changing the meaning of the sentence This is known as transformation of sentences

Exercise 1 Change the following affirmative sentences into Negative sentences

a He is a good manHe is not a bad man

b Ram loves SitaRam is not without love for Sita

c Only he stood first in the classNone but he stood first in the class

d Ankit was wiser than he

He was not so wise as Ankite He did it

He did not fail to do itf As soon as I reached college the

bell rangNo sooner did I reach college than the bell rang

g He finished everythingHe left nothing unfinished

h It always pours when it rainsIt never rains but it pours

Math Topic Commercial MathematicsChapter ndash Goods and services Tax

What is GSTAns It is a abbreviated term of Goods and Service Text which is an indirect tax levied on the sale of goods and rendering servicesSome terms related to GSTDelar Any person who buys goods or services For resale is known as a delar A delar Can be a firm or a companyIntra-state sales Sales of goods and services within the same state or same union territory are called intra- state salesInter-state sales Sales of goods and services outside the state or union territory are called Inter-state sales4) Input GST GST is paid by dealers on purchase of goods and services are called input GST5) Output GST GST is collected from customers on sale of goods and services are called output GST6) Types of GST There are three taxes applicable under GST(i) Central Goods and Services Tax (CGST)(ii) State Goods and Services Tax (SGST) or Union Territory Goods and Services Tax (UTGST) Both these taxes are levied on intra-state sales Here GST is divided equally among central and state governments(iii) Integrated Goods and Services Tax (IGST) IGST is levied on inter- state sales It is also levied on import of goods and services into India and export of goods and services from India

Subject Eng Literature (The Merchant of Venice ndash William Shakespeare)Topic Act III Scene 4 Lines 1 to 44 (Portia hellip To wish it back on you fare you well Jessica)[Students should read the original play and also the paraphrase given in the school prescribed textbook]

Summary Questions amp AnswersIn this scene we suddenly find a new element in the character of Portia We have already seen her possessed of every graceful womanly quality but now she shows that she is capable of rapid decision and determined action She shows this by her sudden resolve to hasten to Venice with a daring scheme for the rescue of Antonio This is an important scene in the dramatic action for it leads up to and renders possible the striking events of the famous trial scene which is one of the greatest striking elements of the play Moreover the fact that all the characters of importance are now assembled together in Venice makes the union of the main plot and the secondary story complete

(1) LORENZO Madam although I speak it in your presence(Line 1-9)

You have a noble and a true conceit

Of god-like amity which appears most strongly

In bearing thus the absence of your lordBut if you knew to whom you show this honourHow true a gentleman you send reliefHow dear a lover of my lord your husbandI know you would be prouder of the workThan customary bounty can enforce you

(a) Where is Lorenzo Why is he here To whom is he referring as lsquoMadamrsquo

Lorenzo is at Portiarsquos residence He had met Salerio on the way and Salerio had begged him to come along with him to

o In this scene Portia Nerissa Lorenzo Jessica and Balthazar appear

o Portia requests Lorenzo and Jessica to be in charge of her house during her absence from Belmont because she and Nerissa have decided to spend the days in meditation and also in visiting the holy places in the neighbourhood of Belmont She has already instructed her people to acknowledge both Lorenzo and Jessica as master and mistress of house during her absence Lorenzo and Jessica gladly agree to look after the house of Portia

handover the letter from Antonio to Bassanio The letter carried the bad news about Antoniorsquos arrest for non-payment of loan taken from Shylock Hence Salerio might have preferred company to break this bad news to Bassanio He is referring to Portia as Madam(b) What does Portia say on hearing the above extract

Portia says that she has never regretted doing good to others Friends who spend a lot of time together and really are there for each other have many traits in common As Antonio is Bassaniorsquos best friend saving him is like saving Bassanio who is like her own soul She asks Lorenzo to take care of management of the house till Bassanio is back(c) What does Portia send with Bassanio and why

On hearing about Antoniorsquos troubles on account of Bassanio her husband Portia immediately sends him with enough gold to repay the debt many times over to Venice to help Antonio out of his misfortune

(2) Lorenzo Madam with all my heart (Line 36-40)

I shall obey you in all fair commands

Portia My people do already know my mindAnd will acknowledge you and JessicaIn place of Lord Bassanio and myselfSo fare you well till we shall meet again

(a) Where are Lorenzo and Portia at this time What lsquofair commandsrsquo are given to Lorenzo

Lorenzo and Portia are at Belmont during this scenePortia reveals to Lorenzo that she has sworn to contemplate in prayer at a monastery around two miles away until her husband returns from Venice She tells him that Nerissa would accompany her and asks him to manage the house with Jessica till things are settled In response Lorenzo tells her that he would be obliged to do whatever she asks him to do(b) Where is Portia actually going and why

Portia tells Lorenzo that she would live a life of contemplation and pray at a monastery which is two miles away from her place In reality Portia plans to go to Venice in disguise with Nerissa and argue the case in defense of Antonio She is very sure that her plan would succeed

ClassXI (ScienceHumanitiesCommerce)Subject Topic Summary Execution

Computer Science

(APC)

Ch ndash 1 Numbers

(Numbers in different bases and

their Arithmatical operations)

Number System In computers Number System is defined as a writing system to represent the numbers in different ways ie we are using different symbols and notations to represent numbers There are four ways we can represent the number ndash Binary Decimal Octal and Hexadecimal

Decimal Number SystemThis number system consist 10 digits These are 0 1 2 3 4 5 6 7 8 amp 9

Binary Number SystemThis number system has only two digits these are 0 and 1 Here 0 stands for off while 1 stands for on

Octal Number SystemThis number system has 8 digits these are 0 1 2 3 4 5 6 amp 7

Hexadecimal Number SystemThis number system has 16 digits these are 0 1 2 3 4 5 6 7 8 9 A B C D E F Here the value of the alphabets are as follows A=10 B=11 C=12 D=13 E=14 F=15

Rules for conversion decimal number to Binary1 Divide the decimal number by 22 If the number will not divide equally by 2 then round down the answer to the nearest whole number (integer)3 Keep a note of the remainder it should be either 0 or 14 Keep repeating the above steps dividing each answer by 2 until you reach zero5 Write out all the remainders from bottom to top This is your binary solution

For example Lets convert 32 to binary 2 32 2 16 - 0 2 8 - 0 2 4 - 0 2 2 - 0 2 1 - 0 0 - 1

The binary equivalent of 3210 is 1000002

Try the follwing youself1 2410

2 4810

3 1210

History GROWTH OF NATIONALISM

The second half of the 19th century witnessed growth of political consciousness and a sense of Nationalism among the IndiansThere were various factors for growth of Indian Nationalism- As a result various political associations were formed in different provinces by the educated Indians Surendranath Banerjee organized a meeting of National conference at Calcutta Ultimately the National Congress was founded in Bombay in 1885This body became the vanguard of Indian struggle for freedom The congress leaders were known as moderates because they followed a policy of prayer and petition A large number of Indian leaders had experienced in political agitation The Political situation of England was also changed Moreover increasing revolutionary activities in Maharashtra Punjab and Bengal became serious concern to the British Government In this

QUESTION1 What do you mean by Nationalism ANSWER 1 Nationalism is defined as loyalty and devotion to own nation especially a sense of national consciousnessQUESTION 2 What are the causes of nationalism ANSWER 2 There were various factors for growth of nationalism

1 Spread of western education2 The progress of vernacular press and

patriotic literature3 The economic exploitation of our

country by the colonial rulers4 International affairs

QUESTION 3 Who organized National conference in Calcutta in 1883 ANSWER 3 Surendranath BanerjeeQUESTION 4 When did Indian National Congress formANSWER 4 Indian National Congress was formed in 1885 in BombayQUESTION 5 Who were ModeratesANSWER 5 The Early Nationalists were also known as Moderates Their emergence marked

background Lord Curzon became Viceroy in India He had no respect for the Indian National Congress

the beginning of the organized national movement in India They believed in British justice and were loyal to them They followed a policy of prayer and petition They demanded constitutional reforms of our country Impotant Moderate leaders were Pherozshah Mehta Dadabhai Naorozi and Surendranath Banerjee etcQUESTION 6 What do you know about Extremism in Indian National movementANSWER 6 In the beginning of 20th century a new class of national leaders emerged in India which was different from the moderate groups They started more aggressive movement against the British empire The goal of extremists was ldquoswarajrdquo Important extremist leaders were Bal Gangadhar Tilak Lala Lajpat Rai Bipin Chandra Pal etcQUESTION 7 Mention the places which were the main centres of Revolutionary movementANSWER 7 Maharashtra Bengal and Punjab

Physics

Chapter Dimensional Analysis

(Summary)

The dimensions of a physical quantity are the powers to which the fundamental units are raised in order to obtain the derived unit of that quantit

The physical quantites lengthmasstime are represented by [L] [M] [T] resp let they are raised to powers ( dimesions) abc resp then any physical quantity can be represented by [ La Mb Tc ] Examples

1 Area area = L x B = [L] x [L] = [M0 L2 T0 ]

2 Density density = massvolume = [M][L3] = [ M L-3]

3 Velocity velocity = distancetime = [L][T] = [LT-1]HW Try to find out dimension of acceleration Acceleration = velocity timeNB One can find the SI Units Using Dimension Analysis Such as for area we have [L2] so its SI unit is m2

Biology Topic ndash Chp-1 The living world

Today we will start the first chapter the living world Here we discuss about the characteristics of living organism and what are the difference between them and nonliving substances We also discuss about the contribution of different Scientists

There are over 500000 species of plants andover a million species of animal are present on earth Some 15000 new species were discovered every yearQ1 What is a living organismbull A living organism is primarily physico -chemical material that demonstrate a high degree of complexity is capable of selfRegulation possesses a metabolism and perpetuates itself through timeQ2 What are the differences between livingand non-livingsi) Compared with non-living living organisms

have more complex organised structure and their use of energy is more controlled amp efficientii) Living things reproduce their own kind by forming new cells which contains copies of their genesiii) Each organism has some degree of homeostasisie it is able to make adjustments so that internal environment remains constantQ3 Write contributions of following Scientists i) Aristotle - One of the first theories in Biology places all living things in a hiearchieii) AV Leeuwenhoek - was the first to observe living single celled organisms under microscopeii) Carolus Linnaeus - developed the binary system for naming of organisms and classificationiii) Geregor Johann Mendel ndash discoverbasic principles of inheritanceHomework i) C Darwin ii)Schleiden

Math Trigonometric functions

1 Overviewi) Trigonometry The word lsquotrigonometryrsquo is derived from the Greek words lsquotrigonrsquo and lsquometronrsquo which means measuring the sides of a triangle An angle is the amount of rotation of a revolving line with respect to a fixed line Usually we follow two types of conventions for measuring angles ie a) Sexagesimal system b) Circular system In Sexagesimal system the unit of measurement is Degree In Circular system the unit of measurement is Radian ii) Relation between degree and radianThe ratio of circumference of a circle to its diameter is always a constant This constant ratio is a number denoted by π which is taken approximately as 227The relationship between degree amp radian measurements is as follows2 right angles = 180deg= π radians1radian = 180degπ=57deg16(approx) 1deg=π180 radianiii) Length of an arc of a circleIf an arc of length s subtends an angle θ radians at the center of a circle of radius r then s=rθiv) Area of a sector of a circleA sector is like a pizza slice of the

Q) Express the following angles in radiana) 45deg b) 40deg3730Ans a) We have 180deg=π radiansi e 45deg= πtimes45180 radian = π4 radiansb) 40deg3730= 40deg37+3060 minute= 40deg 37 +12 minute= 40deg+ 752 minute=40 + 75(2times60) degree=3258 degreeNow 180deg=π radianie 3258 degree= (πtimes325) (180times8) radians = 65π288 radiansQ) A circle has a radius of r=12 meters What is the length of an arc traced out by a 60deg angle in the center of the circleAns In this problem we know both the central angle (60deg) and the radius of the circle (12) All we have to do is plug those values into our equation and we get

s = 2π(12)(60360)s = 24π6s = 4πSo the length of an arc traced out by a 60deg angle in a circle with a radius of 12 meters equals 4π meters asymp 1257 metersQ) Find the area of the sector with a central angle 30deg and a radius of 9cmAns GivenRadius r = 9 cmAngle θ = 30degArea of the sector = θ360degtimesπr2

= 30360degtimes227times92=2121cm2

circle It consists of a region bounded by two radii and an arc lying between the radiiThe area of a sector is a fraction of the area of the circle This area is proportional to the central angle In other words the bigger the central angle the larger is the area of the sectorArea of Sector = θ2 times r2 (when θ is in radians)

Area of Sector = θ times π360 times r2 (when θ is in degrees)

COMMERCE

CLASSIFICTION OF HUMAN ACTIVITIES-ECONOMIC AND NON-ECONOMIC

Welcome to the new sessiontoday we are going to start the first chapter of Class XI The name of the chapter that we are going to start is

lsquoClassification of Human Activities ndasheconomic and non-economicrsquo

Now let us start the chapter by considering human beings and the activities they perform throughout the day

Human activities means all those activities that human beings undertake to satisfy their wants

Human wants on the other hand are the desire of human beings for goods (vegetables fruits rice etc) and services (services of doctors teachers lawyers etc) that they require to live

Now these human activities continue throughout life as human wants are unending unlimited and recurring as human beings desire for better living throughout their lives

Now human activities can be classified into two categories

Human activities

Economic activities Non-economic activities

Economic activities are

Questions1 What are human activities

Answer Human activities mean all those activities that human beings undertake to satisfy their wants

Example A man working in an office

A boy playing in the garden

2What are the characteristics of human activitiesAnswer the characteristics of human activities are as follows

Human activities are undertaken by men women and children and these activities involve human efforts

Human activities are undertaken to satisfy human wants which are unlimited

Human activities continue throughout life

Human activities are performed for both earning money and personal satisfaction

3What is economic activitiesGive example

Answer Economic activities are undertaken by human beings with the object of earning money acquiring wealth and thereby satisfying human wantsExample

Selling of goods by a shop keeper to his customer

A clinic run by a doctor Service of a teacher in school or college

undertaken by human beings with the object of earning money and acquiring wealth

These activities result in the production of economic goods and services

Example Human activities(ie working in factories officesshops) which produce direct economic benefits

Non-economic activities are inspired by human sentiments and emotions such as love for the family desire to help the poor and love for the country

Thus these human activities (eg praying playing sleeping) produce no direct economic benefits and they are also not related to earning money and acquiring wealth

4 What are the characteristics of economic activities

Answer The characteristics of economic activities are as follows

Economic motiveEconomic activities are undertaken to earn money and acquire wealth

ProductiveEconomic activities involve productiondistribution and exchange of goods and services to create wealth

Economic growthEconomic activities determine the level of economic development of a country and standard of living of its citizens

Socially desirableEconomic activities are socially desirable for society

Economic resourcesEconomic activities make use of all the economic resources such landlabourcapital etc

5 What do you mean by non-economic activitiesExampleAnswerNon-economic activities are inspired by human sentiments and emotions such as love for the family desire to help the poor and love for the countryThese activities are not undertaken for monetary gain but for onersquos satisfaction and happinessExample

a mother looks after her children

a student donates blood8 Differentiate between Economic activities and Non-economic activities

Economic activities

Non-economic activities

1to earn living and acquiring wealth2Result can be measured in terms of money

3ExampleBusinessprofession and employment

1 to obtain some satisfaction

2Result cannot be measured in terms of money

3ExampleFamily-orientedreligious socialCultural and national

BUSINESS STUDIES

BUSINESS ENVIRONMENT

Welcome to the new sessionToday we are going to start the first chapter and the name of the chapter is Business Environment

In todayrsquos world every business enterprise is a part of the society It exists and operates in association with various groups in society such as customers suppliers competitors banks and financial institutions government agencies trade unions media and so on All these groups influence the functioning of business in one way or the other They constitute the environment of businessConcept of Business Environment

The term lsquobusiness environmentrsquo refers to the sum total of all individuals institutions and other forces that lie outside a business enterprise but that may influence its functioning and performance

The main features of business environment

Totality of External forces General and Specific forces Interrelatedness Complexity Dynamic Uncertainty Relativity

The Interrelation between business and its environment

The business enterprise is an open system It continuously interacts with its environment It takes inputs

Prepare the following questions from todayrsquos assignment

1 What do you mean by business environment

The term lsquobusiness environmentrsquo means the aggregate of all forces factors and institutions which are external to and beyond the control of an individual business enterprise but they may influence its functioning and performance Business environment is the macro framework within which a business firm a micro unit operates It consists of several interrelated and interacting elements

2 Explain the main features of business environment in brief

Totality of External forces-Business environment is the sum total of all things external to a business environment

General and Specific forces-It includes both the forces general forces are the economic social political legal and technological conditions which indirectly influence all business enterprise Specific forces are the investors customers competitors and suppliers which influence individual enterprise directly

Interrelatedness-Different elements of environment are interrelated for an example growing awareness for health care has increased the demand for health foods

Complexity- Business environment id

(such as raw materials capital labour energy and so on) from its environment transforms them into goods and services and sends them back to the environment

Fig 1 Business Environment Relationship

complex in nature as the elements keep on changing example economic technological and other forces changes in demand for a product and service

Dynamic-Business environment is not static it keeps on changing

Uncertainty- Itrsquos very difficult to predict future events such as technology and fashion which occur fast and frequently

Economics Basic Economic ConceptsSub topic

Microeconomics and

Macroeconomics

Welcome to the new sessiontoday we are going to start the first chapter of Class XI The name of the chapter that we are going to start is Basic Economic concepts

Now Economics covers the study of human activities Human activities are those activities which are performed by humans to satisfy their wants

Thus Human wants are unlimited and therefore economic activities such as production exchange and consumption are needed in order to satisfy those wants

The study of economics is divided largely in two parts which areMicroeconomics and Macroeconomics

SUBJECT- MATTER OF ECONOMICS

MICROECONOMICS MACROECONOMICS

Questions1Who has coined the words micro and macro economics

Answer Ranger Frisch coined the words lsquomicrorsquo and lsquomacrorsquo in 1933 to denote the two branches of economic theory namely microeconomics and macroeconomics

2What is microeconomicsAnswer It is the study of behaviour of individual decision ndash making unit such as consumers firms etc

3 What is macroeconomicsAnswer Macroeonomics is the study of overall economic phenomena like employment national income etc

4 What is the importance of microeconomicsAnswer

Microeconomics helps in formulating economic policies which enhance productive efficiency and results in greater social welfare

It helps the government in formulating correct price policies

It explains the working of a capitalistic economy where individual units(producers and consumers ) are free to take their own decision

Micro means a small part in

microeconomics we do not study the whole economy Hence we study an individual consumer and his or her choices and a producer and his or her profit maximizing decisions in the market Thus it does not mirror what happens in the economy as a whole

Macroeconomics on the other hand studies the economy as a whole It is concerned with aggregate and depicts the entire picture of the economyMacroeconomics deals with the national income aggregate investment aggregate consumption etc

Features of Microeconomics It deals with small

parts of the country Hence it looks at

individual consumers firms and industries

It deals with individual income consumption and savings

It studies the determination of price of any product or factors of production

It deals with the working of market via the price mechanism which is nothing but the determination of price and quantity of a commodity by the forces of demand and supply

Features of Macroeconomics

It deals with the study of the economy as a whole

It is concerned with

5 Give a limitation of microeconomics Microeconomics fails to explain the

functioning of an economy as a whole It cannot explain unemployment illiteracy and other problems prevailing in the country

6 What is the importance of macroeconomics It gives overall view of the growing

complexities of an economic system It provides the basic and logical

framework for formulating appropriate macroeconomic policies (eg for inflation poverty etc )to direct and regulate economy towards desirable goals

7What is the limitation of macroeconomics It ignores structural changes in an

individual unit of the aggregate

8 Differentiate between Microeconomics and Macroeconomics

Microeconomics Macroeconomics

the study of aggregates

National income aggregate savings and aggregate investments are major concepts dealt within macroeconomics style

It studies the determination of general price levels

It investigates into the problem of unemployment and the achievement of employment

It studies the aspect of decision making at the aggregate and national levels

It includes all growth theories whether related to developed or developing economies it also includes the study of economic systems and the working of the economy under different systems

Note Both Micro and macro economics are complementary and should be fully utilized for proper understanding of an economy

1It studies economic aspect of an individual unit2It deals with individual incomeConsumption and savings

3 It facilitates determination of price of any product or factors of production

4 Itrsquos scope is narrow and restricted to individual unit

1It studies the economy as a whole

2It deals with the national income aggregate consumption and aggregate savings3 It facilitates determination of general price level in an economy

4 Itrsquos scope is wide as it deals with economic units on the national level

ACCOUNTS

Introduction to Accounting and Book-keeping

Today I am going to share you the meaning of Accounting and Book-keeping and its related terms bullAccounting bullBook Keeping bullAccountsbullTypes Of Accounts bullAccounting Cycle

bull Meaning of accounting

Ans ) Accounting is the art and science of recording classifying and summarising monetary transactions

bull Meaning of Book-keeping

Ans) Bookkeeping is the art of recording business transactions with the view of having a permanent record of them and showing their effect on wealth

bull Meaning of account

Ans) The term account means a record of

business transactions concern a particular person of firm asset or income or expense It is a summarised record of all transactions which take place in an accounting year

bull Types of accountsPersonal accounts ndash Personal accounts relating

to person and Organisation are known as personal accounts Example Ramrsquos Account ABC amp Co Account etc

Real account - The accounts related to tangible and intangible assets are called real accounts Example Cash Account Furniture Account etc

Nominal account- Accounts related to expenses losses incomes and gains are known as nominal accounts Example Wages Account Salary Account Discount Account etc

bull Accounting cycle Accounting cycle refers to a complete sequence of accounting activities It begins with recording of transactions and ends with the preparation of a balance sheet

Chemistry TopicAtomic Structure

Thomsonrsquos atomic modelThomson (1898) was the first to propose the model of an atomHe proposed that an atom can be regarded as a uniform sphere of positive electricity in which requisite number of electrons are embedded evently to neutralize the positive chargeThis is just like plums embedded in a pudding or seeds evently distributed in red spongy mass of a watermelonThis model of atom is known as ldquoPlum-Pudding modelrdquo or

Q1)What is the fundamental constituents of atomAns Electron Proton and neutrons are the fundamental constituents of atomQ2)What is the value of fundamental unit of electricityAnsThe charge carried by one electron is sad to be the fundamental unit of electricityIts magnitude is 48times10-10esuOr 1602times10-19C Q3)Name the element containing no neutronAnsOrdinary hydrogen atom or protium 1H1

Types of AccountPersonal AccountReal AccountNominal AccountBalance Sheet (opening)

ldquowatermelon modelrdquoThis model could explain the electrical neutrality of an atom but failed to explain the result of scattering experiment carried out by Rutherford in 1911So it was rejected ultimately

Q4)Why is an electron called universal particleAns Itrsquos mass and Charge are independent of its source

EVS Chapter 1 ndash Modes of Existence

Modes of existence When one speaks normally about the mode of existence of some group or individual one refers to their customs their mode of being their ethology their habitat in some way their feeling for a placeDifferent modes of exixtence are ndash

1 Hunting ndashGathering2 Pastoral3 Agricultural4 Industrial

1 Hunting and gathering Hunting and gathering mode of existence is characterized by obtaining food from hunting wild animals including fishing and gathering wild plants From their earliest days the hunter-gatherer diet included various grasses tubers fruits seeds and nuts Lacking the means to kill larger animals they procured meat from smaller game or through scavenging

Societies that rely primarily or exclusively on hunting wild animals fishing and gathering wild fruits berries nuts and vegetables to support their diet are called hunting and gathering societies

At least this used to be practice of human beings before agriculture is invented As their brains evolved hominids developed more intricate knowledge of edible plant life and growth cycles

Q) Write the features of Hunting ndash gathering societiesAns - There are five basic characteristics of hunting and gathering societies

i The primary institution is the family which decides how food is to be shared and how children are to be socialized and which provides for the protection of its members

ii They tend to be small with fewer than fifty members

iii They tend to be nomadic moving to new areas when the current food supply in a given area has been exhausted

iv Members display a high level of interdependence

v Labor division is based on sex men hunt and women gather

Political Science

Introduction to political science

Political science occasionally called politology is a social science which deals with systems of governance and the analysis of political activities political thoughts associated constitutions and political behaviorThe study of political science involves the study of both the

Answer the following questions-1 What is political science

Political science occasionally called politology is a social science which deals with systems of governance and the analysis of political activities political thoughts associated constitutions and political behavior

2 Short notes-

traditional and modern theories of politicsTraditionalClassical political sciencepolitical theory-Traditional political science is the study of politics before Second World War The methodology to study Politics was traditional (legal formaletc) the definition of politics traditional (Politics begins and end with state)area of study (constitution state machinery)was traditionalModern Political scienceModern political theory-Modern Political Theory critically examines the contemporary state of political theory making an assessment of the achievement and limitations of the Behavioural Revolution in its totality and reviews objectively the major paradigms and conceptual frameworks adopted by the disciplineContemporary attempts at the development of an integrated political theory involving the use of both traditional and modern concepts approaches and theories-Around late 1960s several political scientists realized the importance of both the traditional political theory and modern Political theory They began building an integrated theory of politics involving a systematic mixture of traditional and modern studies of politics It was held that the study of a complex and vast field like politics needs both traditional as well as

Classical political theory Modern Political theory

Homework-Learn

modern concepts and approaches for studying itrsquos all aspects

Subject Eng Literature (The Tempest ndash William Shakespeare) Topic Act I Scene 1 Lines 1 to 32 (Line 32 ndash Gonzalo hellip If he be not born to be hanged our case is miserable) Date 13th April 2020 (3rd Period)

[Students should read the original play and also the paraphrase given in the school prescribed textbook]Summary Questions amp Answers

[SUMMARY OF THE ENTIRE SCENE]

o The play starts with the scene of a severe storm at sea Alonso (King of Naples) Sebastian (Alonsorsquos brother) Ferdinand (Alonsorsquos son) Gonzalo Antonio (the usurping Duke of Milan) are in a ship in the midst of the storm

o The mariners are trying their best to control the vessel from running aground and are totally following the orders of their Master the Boatswain They have scant success

o The mariners become extremely unhappy and annoyed when most of the passengers arrive on the deck thereby hampering their effort to save the ship There is serious confrontation between them and the passengers who are part of the Kingrsquos entourage

o The mariners could not save the ship

SUMMING-UP

(i) Vivid description of the scene which gives a realistic description of terror and confusion of a tropical storm

(ii) Shows Shakespearersquos accuracy of knowledge in describing the naval operations and also matters of seamanship

(iii) The opening scene justifies the title ndash The Tempest

UNANSWERED QUESTIONS

(i) The King always travels with his entire fleet including his soldiers Where

(1) GONZALO Nay good be patient (Line 15-26)BOATSWAIN When the sea is Hence What cares these

roarers for the name of the king To cabin silence Trouble us not

GONZALO Good yet remember whom thou has aboardBOATSWAIN None that I more love than myself You are a

councillor if you can command these elements to silence and work

the peace of the present we will not hand a rope more use your authority If you cannot give thanks you have

lived so long and make yourself ready in your cabin for the mischance of the hour if it so hap [To the Mariners]

Cheerly good hearts [To Gonzalo] Out of our way I say

(a) To whom is the boatswain speaking What does he mean by lsquoNone that I more love than myselfrsquo

The Boatswain is speaking to Gonzalo the honest old councilor of the Duke of MilanBy using the words ndash lsquoNone that I love more than I love myselfrsquo means that for the Boatswain nobody is dearer to him than his own life

(b) What were the conditions that made the boatswain react in this way

The Boatswain reacts in this way because the storm is at sea and Alonso King of Naples Sebastian his brother Ferdinand his son Gonzalo Antonio the usurping Duke of Milan on board are in distress and in panic Thus they have rushed to the deck interrupting the work of the mariners

(c) What hope does Gonzalo take from the attitude of the boatswain

The insolent and authoritative attitude of Boatswain makes Gonzalo feel comforted He tells that there are no signs that the Boatswain will be drowned But his facial appearance and attitude shows that he is destined to die on land by hanging which in effect means that all on board will be saved Otherwise all the persons on board are doomed

(d) How can they lsquomake yourself ready in your cabinrsquo For what were they asked to make ready themselves

In order to make themselves ready in their cabin the

were the other ships

(ii) Why was the ship in that area Where was it coming from or going where

(iii) The ship broke apart What happened to those who were in the ship

passengers on board must prepare for death which they will possibly soon have to meetThey can retire to their cabins and offer prayers to the Almighty to save them from drowning

(e) What does the boatswain say when he is asked to be patient What does he order to the royal party

When the boatswain is asked to be patient and remain calm he says that he will be patient only when the storm will be over and the sea will be calm but as long as the storm blows and there is danger to the ship he cannot think of being patient He orders the royal party to go to the cabin and leave the mariners to their work

(2) GONZALO I have great comfort from this fellow (Line 27-36)

Methinks he hath no drowning mark upon him his complexion is perfect

gallows Stand fast good Fate to his hanging Make the rope of his destiny our cable for our own doth little advantage If he be not born to be hanged our case is miserable

(a) Why does Gonzalo regard the Boatswain in the midst of danger

In the midst of danger Gonzalo regards the boatswain because he feels that the Boatswain is a source of comfort and is bent upon to do his work sincerely which in this case is saving the ship and its passengers from the severest of raging storm

(b) What reasons does Gonzalo give when he says that none in the ship will die of drowning

Gonzalo is almost sure that none in the ship will die by drowning His says that there is no mark on the face of the boatswain that indicates that he will die by drowning On the other hand the lines on his face are strong indications that he will be hanged to death Therefore there shall be no danger of the shiprsquos sinking

(c) Explain the following ldquoStand fast good Fate to his hanging Make the rope of his destiny our cable for our own doth little advantage If he be not born to be hanged our case is miserablerdquo

The stated lines mean that if the will of destiny is to be carried out then the ship will not get wrecked and all the passengers will be saved The safety of the passengers therefore depends upon the will of fate being carried out in the case of the boatswain If however the boatswain is not to die by hanging then the passengers are also very unsafe because in that case the ship is likely to sink

(d) What order does the Boatswain give to the sailors

when he re-enters What does he say about the crying of the fellows inside the cabin

The boatswain orders the sailors to bring the topmast lower and bring the ship close to a stationary position with the help of the main sail He says that the fellows inside the cabin are moaning and crying in their distress louder than his voice and louder even than the roaring of the storm

Class XII (ScienceCommerceHumanities) Subject Topic Summary Execution

Computer Science

PropositionalLogic

Propositional logic is a procedure to provide reasoning through statementProposition A ststement that results in True or False is said to be proposition There are two types of propositionSimple proposition amp compound propositionSimple proposioton A simple proposition is one that is not a part of any other proposition Such sentential form of proposition is symbolized with english letters in short For example Ram is a claver student (TrueFalse)Where do you live (Not in True or False)Grapes are sweet (TrueFalse)It rains today (TrueFalse)Here we can see some statements anwer would be true or false but some staements answer can not give in terms of true or false Thus the sentences which can be answered in true or false are known as simple propositionAssigning propositon to a variableThe general syntax to assign propostion to a variable is as followsVariable = Simple propositonFor example A=Ram is a clever studentB= Grapes are sweetC= it rains todayCompound proposition

helliphellipto be continued in next classhelliphellipMath Relation Relation If A and B are two non-empty sets

then a relation R from A to B is a subset of AxB If R A x B and (a b) R then we say that a sube isinis related to b by the relation R written as aRbeg Let A be the set of students of class XII and B be the set of students of class XI Then some of the examples of relation from A to B arei) (a b) AXB a is brother of bisinii) (a b) AXB age of a is more than age of isinb Types of relation In this section we would like to study different types of relations We know that a relation in a set A is a subset of A times A Thus the empty set φ and A times A are two extreme relations For illustration consider a relation R in the set A = 1 2 3 4 given by R = (a b) a ndash b = 10 This is the empty set as no pair (a b) satisfies the condition a ndash b = 10 Similarly R = (a b) | a ndash b | ge 0 is the whole primeset A times A as all pairs (a b) in A times A satisfy | a ndash

Example 1 Let A be the set of all students of a boys school Show that the relation R in A given by R = (a b) a is sister of b is the empty relation and R = (a b) the primedifference between heights of a and b is less than 3 meters is the universal relationSolution Since the school is boys school no student of the school can be sister of any student of the school Hence R = φ showing that R is the empty relation It is also obvious that the difference between heights of any two students of the school has to be less than 3 meters This shows that R = A times A is primethe universal relation Example 2 Show that the relation R in the set 1 2 3 given by R = (1 1) (2 2) (3 3) (1 2) (2 3) is reflexive

b | ge 0 These two extreme examples lead us to the following definitionsDefinition 1 A relation R in a set A is called empty relation if no element of A isrelated to any element of A ie R = φ A times AsubDefinition 2 A relation R in a set A is called universal relation if each element of A is related to every element of A ie R = A times A Both the empty relation and the universal relation are some times called trivial relation Definition 3 A relation R in a set A is called(i) reflexive if (a a) R for every a Aisin isin(ii) symmetric if (a1 a2) R implies that (aisin 2a1)

R for all aisin 1 a2 Aisin(iii) transitive if (a1 a2) R and (aisin 2 a3) R isinimplies that (a1 a3) R for all aisin 1 a2 a3 AisinDefinition 4 A relation R in a set A is said to be an equivalence relation if R is reflexive symmetric and transitive

but neither symmetric nor transitiveSolution R is reflexive since (1 1) (2 2) and (3 3) lie in R Also R is not symmetric as (1 2) R but (2 1) isin notinR Similarly R is not transitive as (1 2) R and (2 3) R but (1 3) R isin isin notinExample 3 Show that the relation R in the set Z of integers given byR = (a b) 2 divides a ndash b is an equivalence relationSolution R is reflexive as 2 divides (a ndash a) for all a Z isinFurther if (a b) R then 2 divides a isinndash b Therefore 2 divides b ndash a Hence (b a) R which shows that R is isinsymmetric Similarly if (a b) R and (b c) R isin isinthen a ndash b and b ndash c are divisible by 2 Now a ndash c = (a ndash b) + (b ndash c) is even (Why) So (a ndash c) is divisible by 2 This shows that R is transitive Thus R is an equivalence relation in ZExample 4 Let L be the set of all lines in a plane and R be the relation in L defined as R = (L1 L2) L1 is perpendicular to L2 Show that R is symmetric but neither reflexive nor transitiveSolution R is not reflexive as a line L1 can not be perpendicular to itself ie (L1 L1) R notinR is symmetric as (L1 L2) Risin

L1 is perpendicular to L2rArr L2 is perpendicular to L1rArr (L2 L1) RrArr isin

R is not transitive Indeed if L1 is perpendicular to L2 and L2 is perpendicular to L3 then L1 can never be perpendicular to L3 In fact L1 is parallel to L3 ie (L1 L2) R isin(L2 L3) R but (L1 L3) Risin notin

Chemistry Solid state Characteristics if Solids(i)The particles are locked in fixed positions they are unable to change their relative positions and this brings a definite shape and volume of a solid(ii)In a solid the constituent particles are held by strong forces of attractionThe forces of attraction may be bonding or non bonding(iii)The constituent particles in a solid pack together as closely as possibleoccupying most of the available space within the solidThus the empty space in a solid is very smallThis makes a solid highly rigid and nearly incompressibleThis also explains why a solid has high density and exhibits slow diffusionClassification of Solids

Q1)Define Crystalline solids AnsA Solid that has a definite geometrical shape and a sharp melting pointand whose constituent particles (atomsmolecules or ions) are arranged in a long range order of definite pattern extending throughout the solid is called a crystalline solidExNaClQ2)Define Amorphous solids AnsA solid that does not have a definite shape and a sharp melting pointand whose constituent particles (atomsmolecules or ions) are not arranged in a definite pattern is called an amorphoussolid

Crystalline solidsAmorphous solids

ExGlassRubberQ3)Classify Crystalline Solids Crystalline Solids

Physics Coloumbrsquos Law (Summary)

Before Going Into Coloumbrsquos Law We Will First Learn What is Charge Properties of Charge and Always remember that charge is quantized ie a body always have static charge of magnitude equal to some integral multiple of fundamental electronic charge e= 16 x 10- 19 C

Charge is the property of matter that causes it to produce and experience electrical and magnetic effects The study of the electrical charges at rest is called electrostatics When both electrical and magnetic effects are present the interaction between charges is referred to as electromagnetic

There exist two types of charges in nature positive and negative Like charges repel and unlike charges attract each other

The type of charge on an electron is negative The charge of a proton is the same as that of an electron but with a positive sign In an atom the number of electrons and the number of protons are equal The atom is therefore electrically neutral If one or more electrons are added to it it becomes negatively charged and is designated as negative ion However if one or more electrons are removed from an atom it becomes positively charged and is called a positive ion

The excess or deficiency of electrons in a body gives the concept of charge If there is an excess of electrons in a body it is negatively charged And if there is deficiency of electrons the body becomes positively charged Whenever addition or removal of electrons takes places the body acquires a charge

The SI Unit of charge is coulomb (C) In SI units the current is a fundamental quantity having a unit of ampere (A) The unit of charge is defined in terms of the unit of current Thus one coulomb is the charge transferred in one second across the section of a wire carrying a

Ionic SolidsMetallicSolids

Molecular Solids

current of one ampere

As q = It we have1 C = (1 A) (1 s)

The dimensions of charge are [A T]

Properties of Charge

(1) Quantization of Charge Electric charge can have only discrete values rather than any value That is charge is quantized The smallest discrete value of charge that can exist in nature is the charge on an electron given as

e = plusmn 16 x 10- 19 C

This is the charge attained by an electron and a protonA charge q must be an integral multiple of this basic unit That is

Q = plusmn ne where n = 1 2 hellip

Charge on a body can never be (frac12)e (23)e or 57e etcWhen we rub a glass rod with silk some electrons are transferred from the rod to the silk The rod becomes positively charged The silk becomes negatively charged The coulomb is a very large amount of charge A typical charge acquired by a rubbed body is 10 - 8 C

Biology Reproduction in organisms

Welcome to this new session 2020-21Today in this first chapter we mainly discuss about reproduction types needs and life span of some organismsWe also discuss about difference between sexual and asexual reproduction

Q1 What is reproductionReproduction is defined as a biological processin which an organism gives rise to young onessimilar to itselfQ2 What are the needs of reproductionbulli) Reproduction maintain life on earthii) It enables the continuity of the species generation after generationiii) It creates genetic variation among populationsQ3 Define Life span and write some orgnisms life spanbull Life span is the period from birth to

the natural death of an organism- OrganismsLife span1 Butterfly 1 - 2 weeks2 Fruit fly 30 days3Dog 10-13 years4 Rose5-7 years5 Tortoise100-150 years6 Banyan Tree -200 - 250 yearsQ4 Reproduction is of two types in case ofanimals but in case of plants vegetative propagation is also present

Asexual Reproduction Sexual Reproductioni) Always uniparentalii) Gametes are not involvediii) Only mitotic division involvediv) Somatic cells of parents are involvedv) Offsprings are genetically similar to the parents

i) Usually biparentalii) Gametes are involvediii) Meiosis occurs during gametogenesis Mitosis occurs after fertilisationiv) Germ cells of the parents are involvedv) offsprings are genetically different from the parents

COMMERCE BUSINESS ENVIRONMENT

Welcome to the new sessiontoday we are going to start the first chapter of Class XII The name of the chapter is Business Environment

Already many of you have got some idea about the word business environment form the first chapter of business studies in class XI

In todayrsquos world every business enterprise is a part of the society It exists and operates in association with various groups in society such as customers suppliers competitors banks and financial institutions government agencies trade unions media and so on All these groups influence the functioning of business in one way or the other They constitute the environment of businessConcept of Business Environment

The term lsquobusiness environmentrsquo refers to the sum total of all individuals institutions and other forces that lie outside a business enterprise but that may influence its functioning and performance

The main features of business environment Totality of External forces General and Specific forces Interrelatedness Complexity Dynamic Uncertainty

Prepare the following questions from todayrsquos assignment

2 What do you mean by business environment

The term lsquobusiness environmentrsquo means the aggregate of all forces factors and institutions which are external to and beyond the control of an individual business enterprise but they may influence its functioning and performance Business environment is the macro framework within which a business firm a micro unit operates It consists of several interrelated and interacting elements

2 Explain the main features of business environment in brief

Totality of External forces-Business environment is the sum total of all things external to a business environment

General and Specific forces-It

Relativity

The Interrelation between business and its environment

The business enterprise is an open system It continuously interacts with its environment It takes inputs (such as raw materials capital labour energy and so on) from its environment transforms them into goods and services and sends them back to the environment

Fig 1 Business Environment Relationship

includes both the forces general forces are the economic social political legal and technological conditions which indirectly influence all business enterprise Specific forces are the investors customers competitors and suppliers which influence individual enterprise directly

Interrelatedness-Different elements of environment are interrelated for an example growing awareness for health care has increased the demand for health foods

Complexity- Business environment id complex in nature as the elements keep on changing example economic technological and other forces changes in demand for a product and service

Dynamic-Business environment is not static it keeps on changing

Uncertainty- Itrsquos very difficult to predict future events such as technology and fashion which occur fast and frequently

Business Studies

Human Resources Management

Human resource of an organisation are the aggregate of knowledge skills attitudes of people working in it

The management system which deals with human resources is called human resource management

Features of HRMbullComprehensive functionbullPeople-oriented

Question1) What do you mean by human

resource management Answer) Human resource management may be defined as that field of Management which has to do with planning organising and controlling the functions of procuring developing maintaining and utilising the labour force

bullAction oriented bullPervasive function bullContinuous function

2) Explain the features of HRM in brief

Answer)bullHuman Resource Management is concerned with managing people at work bull Human Resource Management is concerned with employees which bring people and organisations together so that the goals of each are met bullHuman resource management considered every employees as an individual and also promote their satisfaction and growth bull Human resource management is inherent in all organisations and at all levelsbullManagement of human resources are ongoing on never ending process which requires a constant alertness and Awareness of human relations

3) ldquoHR function is said to be pervasiverdquowhy

Answer) Human resource management is required in all organisations whether it is private or government organisations armed forces sports organisations etc It permeatsall the functional areas like production marketing finance research etc This from this feature of human resource management it can be said that it is pervasive in nature

Economics Demand Q1DEFINITION OF DEMANDIn economics demand is the quantity of a good that consumers are willing and able to purchase at various prices during a given period of timeQ2DEMAND CURVEIn economics a demand curve is a graph depicting the relationship between the price of a certain commodity and the quantity of that commodity that is demanded at that pricQ3LAW OF DEMANDIn microeconomics the law of demand states that conditional on all else being equal as the price of a good increases quantity demanded decreases conversely as the price of a good decreases quantity demanded increasesQ4ASSUMPTION of LAW OF DEMAND(i)No change in price of related commodities(ii) No change in income of the consumer(iii) No change in taste and preferences customs habit and fashion of the consumer( No expectation regarding future change in priceQ5MARKET DEMAND SCHEDULEIn economics a market demand schedule is a tabulation of the quantity of a good that all consumers in a market will purchase at a

given price At any given price the corresponding value on the demand schedule is the sum of all consumersrsquo quantities demanded at that priceQ6INDIVIDUAL DEMAND SCHEDULEIndividual demand schedule refers to a tabular statement showing various quantities of a commodity that a consumer is willing to buy at various levels of price during a given period of timeQ7 FACTORS AFFECTING INDIVIDUAL DEMAND FOR A COMMODITY

The factors that influence a consumerrsquos decision to purchase a commodity are also known as determinants of demand The following factors affect the individual demand for a commodity1 price of the commodity2 price of related goods3 income of buyer of the commodity4 tastes and preferences of the buyer1 Price of the CommodityYou must have observed that when price of a commodity falls you tend to buy more of it and when its price rises you tend to buy less of it when all other factors remain constant (lsquoother things remaining the samersquo) In other words other things remaining the same there is an inverse relationship between the price of a commodity and its quantity demanded by its buyers This statement is in accordance with law of demand which you will study in the later part of this lesson Price of a commodity and its quantity demanded by its buyers are inversely related only when lsquoother things remain the samersquo So lsquoother things remaining the samersquo is an assumption when we study the effect of changes in the price of a commodity on its quantity demanded2 Price of Related goodsA consumer may demand a particular good But while buying that good heshe also asks the price of its related goods Related goods can be of two types-(i) Substitute goods(ii) Complementary goods While purchasing a good prices of its substitutes and complements do affect its quantity purchased(i) Price of Substitute Goods Substitute goods are those goods which can easily be used in place of one another for satisfaction of a particular want like tea and coffee An increase in price of substitute good leads to an increase in demand for the given commodity and a decrease in price of substitute good leads to a decrease in demand for the given commodity It means demand for a given commodity is directly affected by change in price of substitute goods For example if price of coffee increases the demand for tea will rise as tea will become relatively cheaper in comparison to coffee(ii) Price of Complementary goods Complementary goods are those goods which are used together to satisfy a particular want like car and petrol An increase in the price of complementary goods leads to a decrease in demand for the given commodity and a decrease in the price of complementary goods leads to an increase in demand for the given commodity For example if price of petrol falls then the demand for cars will increase as it will be relatively cheaper to use both the goods together So demand for a given commodity is inversely affected by change in price of complementary goods3 Income of the Buyer of CommodityDemand for a commodity is also affected by income of its buyer However the effect of change in income on demand depends on the nature of the commodity under consideration In case of some goods like full cream milk fine quality of rice (Basmati rice) etc demand for these commodities increases when income of the buyer increases and

demand for these commodities decreases when income of the buyer decreases Such goods whose demand increases with the increase in income of the buyer are called normal goods But there are some goods like coarse rice toned milk etc whose demand decreases when income of buyer increases and their demand increases when income of the buyer decreases Such goods whose demand decreases with the increase in income of the buyer are called inferior goods Suppose a consumer buys 10 Kgs of rice whose price is ` 25 per Kg He cannot afford to buy better quality of rice because the price of such rice is ` 50 per Kg The consumer is spending ` 250 per month on the purchase of rice Now if income of the consumer increases and he can afford ` 350 on purchase of 10 Kg of rice Now he can afford to buy some quantity of rice say 6 Kgs whose price is ` 25 per Kg and may buy 4 Kgs of rice whose price is ` 50 per Kg Thus he will buy 10 Kgs of rice by spending ` 350 per month Therefore we may conclude that demand for normal goods is directly related to the income of the buyer but demand for inferior goods is inversely related to the income of the buyer4 Tastes and Preferences of the BuyerThe demand for a commodity is also affected by the tastes and preferences of the buyers They include change in fashion customs habits etc Those commodities are preferred by the consumers which are in fashion So demand for those commodities rises which are in fashion On the other hand if a commodity goes out of the fashion its demand falls because no consumer will like to buy it(5) Number of Buyers in the Market(Population)Increase in population raises the market demand whereas decrease in population reduces the market demand for a commodity Not only the size of population but its composition like age (ratio of males females children and old people in population) also affects the demand for a commodity It is because of needs of children young old male and female population differs(6) Distribution of Income and WealthIf the distribution of income and wealth is more in favour of the rich demand for the commodities preferred by the rich such as comforts and luxuries is likely to be higher On the other hand if the distribution of income and wealth is more in favour of poor demand for commodities preferred by the poor such as necessities will be more(7) Season and Weather ConditionsThis is generally observed that the demand for woolens increases during winter whereas demand for ice creams and cold drinks increases during summer Similarly market demand for umbrellas rain coats increases during rainy seasonQ8 REASONS FOR OPERATION OF LAW OF DEMAND WHY DEMAND CURVE SLOPES DOWNWARDNow we will try to explain why does a consumer purchase more quantity of a commodity at a lower price and less of it at a higher price or why does the law of demand operate ie why does the demand curve slope downwards from left to right The main reasons for operation of law of demand are1 Law of Diminishing Marginal UtilityAs you have studied earlier law of diminishing marginal utility states that as we consume more and more units of a commodity the utility derived from each successive unit goes on decreasing The consumer will be ready to pay more for those units which provide him more utility and less for those which provide him less utility It implies that he will purchase more only when the price of the commodity falls2 Income Effect

When price of a commodity falls purchasing power or real income of the consumer increases which enables him to purchase more quantity of the commodity with the same money income Let us take an example Suppose you buy 4 ice creams when price of each ice cream is ` 25 If price of ice creams falls to ` 20 then with same money income you can buy 5 ice creams now3 Substitution EffectWhen price of a commodity falls it becomes comparatively cheaper as compared to its substitutes (although price of substitutes has not been changed) This will lead to rise in demand for the given commodity For example if coke and Pepsi both are sold at ` 10 each and price of coke falls Now coke has become relatively cheaper and will be substituted for Pepsi It will lead to rise in demand for coke4 Change in Number of BuyersWhen price of a commodity falls some old buyers may demand more of the commodity at the reduced price and some new buyers may also start buying this commodity who were not in a position to buy it earlier due to higher price This will lead to increase in number of buyers when price of the commodity falls As a result demand for the commodity rises when its price falls5 Diverse Uses of a CommoditySome commodities have diverse uses like milk It can be used for drinking for sweet preparation for ice cream preparation etc If price of milk rises its use may be restricted to important purpose only This will lead to reduction in demand for other less important uses When price of milk falls it can be put to other uses also leading to rise n demand for itQ9 EXCEPTIONS TO THE LAW OF DEMANDYou have studied in law of demand that a buyer is willing to buy more quantity of a commodity at a lower price and less of it at a higher price But in certain circumstances a rise in price may lead to rise in demand These circumstances are called Exceptions to the Law of Demand Some important exceptions are1 Giffen GoodsGiffen goods are special type of inferior goods in which negative income effect is stronger than negative substitution effect Giffen goods do not follow law of demand as their demand rises when their price rises Examples of Giffen goods are jowar and bajra etc2 Status Symbol GoodsSome goods are used by rich people as status symbols eg diamonds gold jewellary etc The higher the price the higher will be the demand for these goods When price of such goods falls these goods are no longer looked at as status symbol goods and tehrefore therir demand falls3 NecessitiesCommodities such as medicines salt wheat etc do not follow law of demandbecause we have to purchase them in minimum required quantity whatever their price may be4 Goods Expected to be ScarceWhen the buyers expect a scarcity of a particular good in near future they start buying more and more of that good even if their prices are rising For example during war famines etc people tend to buy more of some goods even at higher prices due to fear of their scarcity in near future

Political Science

Constitution of India-The

Preamble

The preamble-

Preamble-

The preamble is the most precious part of the constitution We the people of India having solemnly resolved to constitute India into a Sovereign Socialist Secular Democratic Republic and to secure to all its citizensA preamble is an introductory and expressionary statement in a document that explains the documents purpose and underlying philosophy When applied to the opening paragraphs of a statute it may recite historical facts pertinent to the subject of the statuteNature and purpose of the constitution-Purpose of the Constitution dictates permanent framework of the government to form a more perfect union to establish justice and ensure peace of thenationconstitution provide principles how the government can run itself following the rules and laws written in the constitution of each state keeps them balanced

Answer the following questions-

1 What is preambleA preamble is an introductory and expressionary statement in a document that explains the documents purpose and underlying philosophy2 What is the nature and

purpose of the constitutionConstitution dictatespermanent framework of the government to form a more perfect union to establish justice and ensure peace of the nation

Homework-Learn

Accounts Compatibilty mode

1MEANING OF PARTNERSHIPPartnership is a form of business organisation where two or more persons join hands to run a business They share the profits and losses according to the agreement amongst them According to the Indian Partnership Act 1932 ldquoPartnership is relation between persons who have agreed to share profits of a business carried on by all or any one of them acting for allrdquo For example one of your friends has passed class XII from National Institute of Open Schooling (NIOS) and wants to start a business Heshe approaches you to join in this venture Heshe wants you to contribute some money and participate in the business activities Both of you if join hands constitute a partnership2CHARACTERISTICS1048698 Agreement A partnership is formed by an agreement The agreement may be either oral or in writing It defines the relationship between the persons who agree to carry on business It may contain the terms of sharing profit and the capital to be invested by each partner etc The written agreement is known as partnership deed1048698 Number of persons There must be at least two persons to form a partnership

The maximum number of partners in a partnership firm can be 50 according toCompanies Act 20131048698 Business The Partnership is formed to carry on business with a purpose of earning profits The business should be lawful Thus if two or more persons agree to carry on unlawful activities it will not be termed as partnership1048698 Sharing Profits The partners agree to share profits in the agreed ratio In caseof loss all the partners have to bear it in the same agreed profit sharing ratio10486981048698Mutual Agency Every partner is an agent of the other partners Every partner can bind the firm and all other partners by hisher acts Each partner will be responsible and liable for the acts of all other partners10486981048698Unlimited liability The liability of each partner except that of a minor is unlimited Their liability extends to their personal assets also If the assets of the firm are insufficient to pay off its debts the partnersrsquo personal property can be used to satisfy the claim of the creditors of the partnership firm10486981048698Management All the partners have a right to mange the business However they may authorize one or more partners to manage the affairs of the business on their behalf10486981048698Transferability of Share No partner can transfer hisher share to any one including hisher family member without the consent of all other partners3PARTNERSHIP DEEDAgreement forms the basis of partnership The written form of the agreement is which a document of partnership is It contains terms and conditions regarding the conduct of the business It also explains relationship between the partners This document is called partnership deed Every firm can frame its own partnership deed in which the rights duties and liabilities of the partners are stated in detail It helps in settling the disputes arising among the partners during the general conduct of business 4CONTENTS OF PARTNERSHIP DEEDThe partnership deed generally contains the following (i) Name and address of the partnership firm(ii) Nature and objectives of the business(iii) Name and address of each partner(iv) Ratio in which profits is to be shared(v) Capital contribution by each partner(vi) Rate of Interest on capital if allowed(vii) Salary or any other remuneration to partners if allowed(viii) Rate of interest on loans and advances by a partner to the firm(ix) Drawings of partners and interest thereon if any(x) Method of valuation of goodwill and revaluation of assets and liabilities on the reconstitution of the partnership ie on the admission retirement or death of a partner(xi) Settlement of disputes by arbitration(xii) Settlement of accounts at the time of retirement or death of a partner5IN ABSENCE OF PARTNERSHIP DEEDThe partnership deed lays down the terms and conditions of partnership in regard to rights duties and obligations of the partners In the absence of partnership deed there may arise a controversy on certain issues like profit sharing ratio interest on

capital interest on drawings interest on loan and salary of the partners In such cases the provisions of the Indian Partnership Act becomes applicableSome of the Issues are(i) Distribution of Profit Partners are entitled to share profits equally(ii) Interest on Capital Interest on capital is not allowed(iii) Interest on Drawings No interest on drawing of the partners is to be charged(iv) Interest on Partnerrsquos Loan A Partner is allowed interest 6 per annum on the amount of loan given to the firm by himher(v) Salary and Commission to Partner A partner is not entitled to anysalary or commission or any other remuneration for managing the business

History TOPIC-TOWARDS INDEPENDENCE AND PARTITION THE LAST PHASE (1935-1947)

SUB TOPIC-IMPORTANT POLITICAL DEVELOPMENTS ndash GROWTH OF SOCIAL IDEAS

Socialism is a political social and economic philosophyLike in other parts of the world the Russian revolution of 1917 served as a great inspiration for revolutionaries in India who at that time were engaged in the struggle for liberation from British ruleSocialist ideas led to the formation of communist party of IndiaJAWAHARLAL NEHRU Among the early Congress leaders Jawaharlal Nehru was very much impressed and influenced by the Socialist ideas He also learnt about the Economic activities of the Soviet Union after the Bolshevic Revolution 1917 He made full use of them in IndiaThe election of Jawaharlal Nehru and Subhas Chandra Bose showed the Left wing tendency within CongressJawaharlal Nehru demanded economic freedom along with political freedom of the people in order to end the exploitation of masses

Nehrus working committee included three socialists leaders The Lucknow session was a landmark in the evolution of socialist ideas of the congressSUBHAS CHANDRA BOSE ndash Subhas Chandra Bose had socialist leaning Both Jawaharlal Nehru and Subhas Chandra Bose were known as leftist Congress men Later on National Congress divided into Leftist and rightist campCONGRESS SOCIALIST Within the Congress some leaders formed the Congress Socialist partyPattavi Sitaramyya Sardar Patel Rajendra Prasad had hostile attitude towards the Congress Socialist partyJawaharlals attitude was hesitant

1 QUESTION ndash Mention name of two Congress leaders who had socialist leaning

1ANSWER ndash Subhas Chandra Bose and Jawaharlal Nehru2QUESTION- In which session of the congress Jawaharlal elaborated his Socialist ideas2 ANSWER ndash Lucknow and Faizpur Session in December 1935 and 19363QUESTION ndash Why Congress was sharply divided into leftist and rightist camp 3ANSWER ndash Subhas Chandra Bosersquos attempt to seek re election for congress presidentship in 1939sharply divided the National Congress into Leftist and Rightist camp4 QUESTION ndash Who was MN Roy 4 ANSWER ndash Manabendra Roy first formed the Communist Party of India outside the country at Tashkent in 19205QUESTION ndash Who formed the Congress Socialist Party within the Congress5 ANSWER ndash Jaya Prakash Narayan Achyut Patwardhan Acharya Narendra Dev Ram Mohan Lohia Aruna Asaf Ali6QUESTION ndash When was the Congress Socialist Party formed What was its object6 ANSWER ndash 1934The Congress Socialist Party sought to work out socialist programme through the Congress They joined hands with the Congress and wanted to carry

Subhas Chandra Bose being expelled from the congress after the Tripuri rift he formed Forward BlockThere were basic differences between the Congress Socialists and the communistsTRADE UNION ACTIVITIES Maximum working class people lived in Bombay and Calcutta The working and living conditions of those workers were very miserable In this situation Shasipada Banerjee NM Lokhande protested against the oppression of the working class peopleThe first Trade Union Madras Labour Union was formed in 1918 by BP WadiaIndustrial strikes took place in Kanpur Calcutta Madras Jamshedpur and Ahmedabad AITUC was formed in Bombay in 1927 The growth of Trade union among the workers was slow because of the fear of the dismissal of the jobIn the mean time the Moderates as well as Communists left AITUC and formed separate organization

on National struggle with the help of workers and peasant class of the society7 QUESTION ndash What was the name of the party founded by Subhas Chandra Bose7 ANSWER- Forward Block8QUESTION ndash Who was Shasipada Banerjee8 ANSWER ndash Shasipada Banerjee was a radical Brahmo He founded a working menrsquos club to protest against exploitation of the British rulers towards the working class of India9 QUESTION ndash What was the weekly published by NM Lokhande9ANSWER- Dinabandhu10 QUESTION ndash Who founded Bombay Mill-Hands Association and in which year10 ANSWER- NM Lokhande in189011 QUESTION- Who was BP WadiaANSWER- BPWadia was the founder of Madras Labour Union in191812 QUESTION- What was the name of the first labour union of India12 ANSWER- Madras Labour Union13 QUESTION Who founded the Majur Mahajan 13 ANSWER GANDHIJI14 QUESTION What was the full form of AITUC When it was formed14 ANSWER All India Trade Union Congressin 192715QUESTION Who formed the Red Trade Union Congress and in which year15ANSWER The Communists formed the Red Trade Union Congress16 QUESTION What do you mean by Socialism16 ANSWER Socialism describes any political and economic theory that says the community rather than individuals should own and manage property and natural resources

Subject Eng Literature (The Tempest ndash William Shakespeare) Topic Act III Scene 3 Lines 1 to 52 (Line 52 ndash Brother my lord the Duke Stand to and do as we) Date 13th April 2020 (4th Period)

[Students should read the original play and also the paraphrase given in the school prescribed textbook]Summary Questions amp Answers

o Alonso Sebastian Antonio Gonzalo Adrian Francisco and others wandered about the island in search of Ferdinand and gets tired and hungry of the toil and at the same time gives up all hope of finding him

o Antonio and Sebastian are happy that Alonso is out of hope and decide to make another attempt on his life that night when being so tired they will be sleeping soundly

o Suddenly a solemn and strange music is heard in the air and several strange shapes enter bringing in a banquet These strange shapes then dance round it with gestures of salutation and then inviting the King to eat they depart

o Seeing this strange scene all are inclined to believe the tales told by travelers that there truly are ldquounicornsrdquo and ldquothe phoenixrsquo thronerdquo

1 ALONSO What harmony is this My good friends hark (L18-27)

GONZALO Marvellous sweet music

[Enter several strange shapes bringing in a banquet

they dance about it with gentle actions of salutation

and inviting the King and his companions to eat they depart]ALONSO Give us kind keepers heavens What were theseSEBASTIAN A living drollery Now I will believe

That there are unicorns that in Arabia

There is one tree the phoenixrsquo throne one phoenix

At this hour reigning thereANTONIO Ill believe both

And what does else want credit come to me

And Ill be sworn rsquotis true Travellers neer did lie

Though fools at home condemn rsquoem

(a) How did Prospero present an amazing spectacle before Alonso and his companions

Using his magic powers Prospero ordered strange shapes to lay a banquet before Alonso and his companions The shapes brought several dishes with tasty eatables in them They placed the dishes on a table before Alonso and his companions Then the strange shapes began to dance gracefully around the banquet While dancing they made gestures inviting them to eat the food Then suddenly the shapes disappeared(b) Who were the guests at the strange banquet Describe the lsquoliving drolleryrsquo

Alonso Sebastian Antonio Gonzalo Adrian and Francisco were the guests at the strange banquet

The term ldquoliving drolleryrdquo refers to live entertainment show In this context when Alonso the King of Naples Sebastian his brother Antonio the treacherous brother of Prospero Gonzalo the kind and loyal councillor to the King Adrian and Francisco came to the island they were hungry and weary in their spirits They heard a solemn and strange music They were shocked to see several strange shapes bringing in a banquet and these shapes danced about it with gentle action of salutation inviting the King and his companions to eat After this Sebastian described this show as lsquoliving drolleryrsquo(c) What is lsquophoenixrsquo What are lsquoUnicornsrdquo

The term lsquophoenixrsquo refers to a mythical Arabian bird which lived alone and perched on a solitary tree After one hundred years it expired in flames and rose again from its own ashes

lsquoUnicornsrsquo refers to the mythological four-footed beasts having horns in the centre of their foreheads When the horns are ground into powder the powder was believed to be

an aphrodisiac(d) How does Sebastian explain the puppet show OR Why does the speaker now believe in unicorns and phoenix

Sebastian finds several strange shapes bringing in the banquet They invite the king and his party for dinner and soon depart He tells that if such a strange sight can be a reality there is nothing incredible in the world and from the present moment he will believe anything He says that it is a strange dumb show enacted not by puppets but by living beings It is stranger than a travellerrsquos tale Seeing such a thing

before his own eyes he will no longer disbelieve the story about unicorns and phoenix(e) How do the other characters present respond to this living drollery

At the sight of the lsquoliving drolleryrsquo like Sebastian Gonzalo and Antonio too acted strangely Antonio told that he too now believes in unicorns and phoenix and anything else that seems to be incredible He too now believes in travellersrsquo tales Gonzalo told that if he would report those happenings in Naples nobody will believe him He considers that those gentle shapes were gentler in manner in comparison to the living beings Alonso was at first sight suspicious and told them that those strange shapes conveyed their meaning in expressive gestures when they seemed to lack speech by their movements and sounds Francisco was amazed at their mysterious disappearance

2 ALONSO Not I

(Line 43-52)GONZALO Faith sir you need not fear When we

were boysWho would believe that there were mountaineers

Dewlapped like bulls whose throats had hanging at rsquoem

Wallets of flesh Or that there were such men

Whose heads stood in their breasts Which now we find

Each putter-out of five for one will bring us

Good warrant ofALONSO I will stand to and feed

Although my lastmdashno matter since I feel

The best is past Brother my lord the Duke

Stand to and do as we

(a) How does Alonso respond at the spectacle of the shapes which were sent to them at the instruction of Prospero

After seeing the strange sight of appearing and disappearing of the shapes sent by Prospero to arrange a banquet for them Alonso says that his surprise at having seen those creatures is infinite and he is fully justified in feeling so much surprise He thinks that their shapes their gestures and the sounds they made were indeed amazing Although they do not possess the gift of speech yet they were able to convey their

thoughts by means of their gestures only

(b) What does Prospero say about the views expressed by Alonso regarding the shapes What does Francisco think about the shapesAfter hearing Alonsorsquos views about the shapes Prospero says that this manrsquos praise of the spirits is rather hasty He means to say that Alonso has shown great haste in reaching the conclusion about the shapes Francisco is amazed to see that those shapes disappeared in a mysterious way(c) What does Sebastian ask Alonso to doSebastian tells Alonso that the shapes having disappeared should not matter to them because they have left the eatables behind He asks Alonso to enjoy eating as they are extremely hungry but the king does not accept his offer of enjoying the dishes(d) How does Gonzalo try to dispel Alonsorsquos fear of those strange shapes What kind of references does he give to AlonsoGonzalo says that those who have travelled abroad have reported seeing even stranger sights than these shapes that Alonso and his companions have beheld Hence there is no reason to feel afraid of these shapes Gonzalo further adds that in his younger days he had heard strange stories from travelers and Alonso might have heard similar stories For instance it was said that there existed a certain race of

human beings who had huge lumps of flesh hanging at their throats and who therefore resembled bulls Then Gonzalo tells about a race of human beings whose heads were located at their breasts Gonzalo says that such stories were not believed by most people in those days but now-a-days these stories have become common(e) Explain the following lsquoEach putter-out of five for onersquoEnglish travellers often insured their trips with London brokers Those that went on foreign travels those days used to deposit a certain amount with some firm or company in London before their departure If the travelers failed to return the money was forfeited by the company with which it had been deposited But this money was repaid five-fold if the travelers returned safe and sound In this way a traveler stood a great chance of recovering the entire cost of his

travels(f) Give the explanatory meanings of the following expressions in the context of the above extract (i) Dewlapped (ii) Wallets of flesh

(iii) Putter-out(i) Dewlapped having big lumps of flesh at the necks(ii) Wallets of flesh large masses of flesh looking like bags(iii) Putter-out to invest money before commencing the travel

  • General methods of preparation of hydrogen
  • Chapter Dimensional Analysis (Summary)
    • Properties of Charge
Page 6:   · Web viewSubject. Topic. Summary. Execution. Hindi. व्याकरण. शरीरके अंगो के नाम लिखिए. 1) आँख 2) नाक 3

16190 - 2979

Solution (b) ActualEstimated 16190 16000 -2979 -30001321113000

2 Find the actual and estimated difference by rounding off to the nearest ten-thousands(b) ActualEstimated 73012 -28790

Solution (b) ActualEstimated 73012 70000 -28790-3000044222 40000

4 A bakery shop has an order to supply 6705 packets of biscuits There are only 4920 packets in the shop Estimate the number of packets needed to complete the supply

Solution Estimating to thousandsEstimated order = 7000Packets available= -5000Packets needed = 2000

there4 Estimated packets needed to complete the order were 2000

Hindi 2ndlang पतर पतर लिना भी एक का ह पतर लित समय धयान दना होगा निक पतर निकस लि रह ह कयोनिक पतर या तो वयवहारिरक होगा या वयावहारिरक होगापतर दो परकार क होत ह

क) औपचारिरक पतर जस परधानाचाय को नगर निनगम सपादक आदिद

ख) अनौपचारिरक पतर जस पापा को दोसत को बहन को अपन परिरजनो को

1 औपचारिरक पतर-अपन निवदयाय म अवकाश क लिए परधानाचाय को पतर लिखिए १०३कसीबीदम दमक टकोकाता-७०००६५सवा मपरधानाचायऋनिष अरहिवदो ममोरिरय एकडमीदिदनाक-१३४२०२०निवषय-अवकाश हत पतरमहोदया जीसनिवनय निनवदन ह निक म आपकी ककषा चौथी का निवदयाथj ह क रात स मझ तज बार ह डॉकटर न मझ आराम करन को कहा हअतः म निवदयाय आन म असमथ ह आपस निनवदन ह निक आप मझ दिदनाक १३स १७ तक की अवकाश दन की कपा कर हम आप क आभारी रहग

धनयवादआपका आजञाकारी लिशषयसीमा सिसहककषा-४

Bengali বইndashবোংো ভোষো পলিরচয়

পোঠndash৮ লিঙগ ৩ পংলিঙগ কোসক বসউঃ পরোলিবোচক লেP লিবসষয পসদর দবোরো পরষ োতীয় কোউসক লেবোঝোয় তোসক পংলিঙগ বস লেPম -বোবো 4োতর৪ সতরীলিঙগ কোসক বসউঃ পরোলিবোচক লেP লিবসষয পসদর তোরো সতরী োতীয় কোউসক লেবোঝোয় তোসক সতরীলিঙগ বস লেPম - মো লিলিকষকো৫ উভয়লিঙগ কোসক বস উঃ পরোলিবোচক লেPলিবসষয পসদর দবোরোপরষ ও সতরীউভয়োলিতসক লেবোঝোয় তোসক উভয়লিঙগ বস লেPম - লিশ মনতরী ৬ কলীবলিঙগ কোসক বস উঃ অপরোলিবোচক লেP লিবসষয পসদর দবোরো সতরী বো পরষ কোউসক ো বলিঝসয় লেকো2 পদো13 লেক লেবোঝোয়তোসক কলীবলিঙগ বস লেPম - বই

COMPUTER CHAPTER 3 EDITING IN MS WORD

MOVING THE TEXTMS WORD ALLOWS US TO MOVE A BLOCK OF TEXT OR GRAPHICS WITHIN A DOCUMENT

Q6) HOW TO WE USE THE COPY AND PASTE OPTIONAns) WE CAN CREATE A DUPLICATE COPY OF THE TEXTTHE STEPS TO COPY AND PASTE THE TEXT AS FOLLOWS----

SELECT THE PART OF THE TEXT THAT WE WANT TO COPY

SELECT THE COPY BUTTON IN THE CLIPBOARD GROUP UNDER THE HOME TAB OR PRESS CTRL + C KEYS

NOW PLACE THE CURSOR AT THE PLACE WHERE THE TEXT NEEDS TO BE COPIED

CLICK THE PASTE BUTTON IN THE CLIPBOARD GROUP UNDER THE HOME TAB OR PRESS CTRL + V KEYS

Science Adaptations in Animals

We find different kinds of animals in our surroundings We find birds flying in the sky monkeys jumping on trees fish in water camels in desert and so on Animals live in their natural homes which are called their habitats The habitat of a camel is a dry sandy desert and that of a tiger is a dense forest So different animals live in different habitats In the world a plant or an animal has to adapt or change itself to suits its surroundings This changes happens over hundreds and thousands of years A change that a living thing undergoes to become better suited to its surrounding is called adaptation

Fill in the blanks [pg no 40]1 Terrestrial animals live on

land 2 Polar bears are found in the

cold polar regions 3 Frogs have webbed feet that

help them to swim 4 Monkeys and koalas are

arboreal animals

Class VSubject Topic Summary Execution

COMPUTER130420

CHAPTER 2 APPLICATIONS OF COMPUTERS

PAGE NO-15C FILL IN THE BLANKS

INPUTS ANIMATION BANKS SOFTWARE SUPER COMPUTERS

D WRITE TRUE OR FALSE TRUE TRUE TRUE TRUE FALSE

Science Chapter 2 - The Skeletal System

Joints A joint is a place where two or more bones are joined with each other

There are two types of joints 1) Fixed joints 2) Movable joints

The movable joints are mainly four types ndash 1) Hinge joint 2) Pivot joint 3) Gliding joint 4) Ball and socket joint

DAnswer these questions

1)What are the functions of the skeleton

Ans ndash The functions of skeleton are -

i The skeleton gives our body shape ii The skeleton provide our body strength iii The skeleton protect our inner organs from outside injuries iv The skeleton gives our body support

2)List the type of movable joint in our body giving one example of each

Ans - The movable joints are mainly four types ndash a Hinge joint

Example of hinge joints are elbows nice fingers and toes

b Pivot joint Example - the joint between the skull and the backbone is the example of pivot joint

c Gliding joint Example of gliding joints

are wrist and ankle joints d Ball and socket joint

Example of ball and socket joints are shoulder joint and hip joint

English language

Transitive and intransitive verb

Pick out the verbs from the following sentences and say whether they are transitive or intransitive (page 21 ex B)1 gave ndash verb Transitive verb2 Has planted- verb Intransitive verb3 Were- verb Intransitive verb4 Asked- verb Transitive verb5 Told- verb Transitive verb

Social studies Conquering distances

Airways The only airline owned by the government is Air India which handles both domestic and international flightsAdvantages of airways Air transport is the fastestIt can access remote areasIt is the best means of transport in case of emergencies

1 Which is the only airline owned by the governmentAns Air India

2 Give two advantages of airwaysAns The two advantages of airways areAir transport is the fastest transportIt can access remote areas

DisadvantageIt is the most expensive of all other means of transport

3 What is the disadvantage of airwaysAns The only disadvantage of airways is that it is the most expensive of all other means of transport

MATHEMATICS

Ch 3Addition and Subtraction

Properties of Addition1 The sum of two numbers does not change when we change their order This property known as Commutative Property of addition

2 The sum of three numbers does not change when we change their grouping This property is called Associative Property of addition

3 The sum of the numbers and zero is the number itself This property is called Identity Property of addition and the integer 0 is called identity

Exercise 11Fill in the blanks1 2730815 + 8319293 = ____ + 27308152 18219 + 1850308 = 1850308 + ____3 (27815 + 85919) + 95985 = (85919 + ____) + 278154 13227 + (25983 + 73607) = (____ + 25983) + 736075 91389 + 0 = ____ + 91389

Solution 1 83192932 182193 959854 132275 0

6 Which of the following are true statements(a) Any number added to zero is zero

(b) The sum of two numbers does not change when we change their order

(c) 1 is the identity element of addition

(d) Given any three numbers their sum does not change when we change their grouping

Solution (a) False(b) True(c) False(d) True

Class VISubject Topic Summary Execution

HISTORY AND CIVICS

CHAPTER 3

MAHAVIRA AND BUDDHA ndash GREAT PREACHERS BUDDHA

Decline of Buddhism1 Revival of the Brahmanical Hinduism ndash Brahmin Scholars like Shankaracharya and Kumarila Bhatta led the revival of Hinduism and established the supremacy of Vedic religion2 Loss of Royal Patronage ndash Gupta period marked the decline of Buddhism as Gupta rulers were followers of Hinduism3 Split in Buddhism ndash division into Hinayana and Mahayana sects and rise of Mahayana sects blurred the line between Hinduism and Buddhism4 Corruption in Buddhist Sangha ndash due to generation of large revenue from large estates Buddhist monks and nuns started living luxurious

Answer the Following 1 During which dynasty Buddhism was split During the reign of Kanishka

2 Name the two sects of Buddhism Mahayana and Hinayana

3 Name two Vedic scholars who led the revival of Brahmanical Hinduism Shankaracharya and Kumarila Bhatta

life in rich monasteries Hence corruption crept in5 Adoption of Sanskrit ndash when Buddhist scriptures began to be written in Sanskrit in place of peoples language like Pali or Prakrit people started drifting away from Buddhism6 The Turkish Invasion ndash As Muslim conquerors invaded India immensely wealthy Buddhist Monasteries and temples were looted and destroyed and Buddhists were persecuted and killed

4 During which period decline of Buddhism began The Gupta Period

ENGLISH 2 The great train journey- Ruskin Bond

The great journey by Ruskin Bond is a story about Suraj who loved trains and wanted to go to places One day while wandering along the railway tracks he enters into a carriage compartment The train suddenly starts moving with him in the compartment and after a journey returns back to the same place from where it had begun The story is about his experience during that journey

4 Answer the following questionsf Who else is in the carriageA ragged hippy with a dirty beard face was in the carriageg Where does Suraj say that he would like to go toSuraj said that he would like to go to England and China and Africa and Greenland He wanted to go all over the worldh What warning does the man give to SurajThe man said Suraj to keep out of sight so that he doesnrsquot get caught by the ticket collectorsiWhen Suraj thinks about his parents for the first time what does he imagines that they will thinkSuraj thought that if he failed to come home that night his parents would think that he had run away or been kidnapped or been involved in an accidentJ What presents does Suraj imagine that he will bring back for his friendSuraj imagines that he would bring an African lion or a transistor- radio for his friend

CHEMISTRY

Chapter 2 ndashElement and Compound

ATOMAn atom can be defined as the smallest constituent particle of an element which showcases independent existence Example Ne OMOLECULEA molecule can be defined as the combinations of two or more atoms which are held together by chemical bonds A molecule is the smallest portion of a substance which showcases all the properties of the substance On breaking down a molecule further we see properties of the constituent elements Example HCl NaCl O2

Answer the following Q3) What is a moleculeAns - A molecule can be defined as the combinations of two or more atoms which are held together by chemical bonds A molecule is the smallest portion of a substance which showcases all the properties of the substance On breaking down a molecule further we see properties of the constituent elements Example HCl NaCl O2

Q4) Which can exist independently ndash atom or moleculesAns ndash Molecules can exist independently

PHYSICS Physical quantities

Guidelines for writing SI units correctly1 The units named after scientists are not written with a capital initial letter For example newton henry watt2 The symbols of the units named after scientist should be written by a capital letter For example N for newton H for henry W for watt3 Small letters are used as symbols for units not derived from a proper name For example m for metre kg for kilogram4 No full stop or other punctuation marks should be used within or at the end of symbols For example 50 m and not as 50 m5 The symbols of the units do not take plural form For example 10 kg not as 10 kgs6 When temperature is expressed in kelvin the degree sign is omitted For example 273 K not as 273o K (If expressed in Celsius scale degree sign is to be included For example 100o C and not 100 C)7 Use of solidus is recommended only for indicating a division of one letter unit symbol by another unit

Fill in the blanks

1) Length and mass are examples of fundamental physical quantities

2) The measurement of a physical quantity consists of two part magnitude and unit

3) A foot consist of 32 inches 4) The unit of temperature in the SI system is

Kelvin

Write true or false Correct the false statements

1) In ancient times cubit was used to measure the mass of an object FalseCorrect statement ndash In ancient times cubit was used to measure the length of an object

2) There are 7 fundamental physical quantities True

symbol Not more than one solidus is used For example m s-1 or m s J K mol or J K-1 mol-1 but not J K mol8 Some space is always to be left between the number and the symbol of the unit and also between the symbols for compound units such as force momentum etc For example it is not correct to write 23m The correct representation is 23 m kg m s-2 and not as kgms-29 Only accepted symbols should be used For example ampere is represented as A and not as amp or am second is represented as s and not as sec10 Numerical value of any physical quantity should be expressed in scientific notationFor an example density of mercury is 136 x 104 kg m-3 and not as 13600 kg m-3

3) Second is the unit of time in both the CGS and MKS systems True

4) The symbol used for a unit is always written in capital letters False Correct statement -The symbol used for a unit is normally written in small letters

Hindi 2nd language

वाकय निवचार भागवत निवचारो को परकट करन वा साथक एव वयवसथिtत शबद समह को वाकय कहत ह वाकय दो परकार क होत ह ndash

1 उददशय- वाकय म जिजसक बार म कछ बताया जाता ह उस उददशय कहत ह जस राधा एक नतकी ह2 निवधय- वाकय म उददशय क बार म बताया जाता ह उस निवधयक कहत ह जस- राधा एक नतकी ह रचना क आधार पर वाकय क तीन भद होत ह ndash१सर वाकय- राम बाजार गया २ सयकत वाकय- राम बाजार गया और वहा जाकर दोसत स मिमा३ मिमशर वाकय- यह वही tान ह जहा उनका बचपन बीता

helliphellipContinue to nextBengali 2nd language

লিZ সবরপ ও সবরলিZ

সবরলিZর লিয়ম - ১ অ-কোর লিকংবো আ-কোসরর পসর অ-কোর লিকংবো আ - কোর োকস উভয় লিমস আ ndashকোর য় এবং ওই আ ndash কোর পব13বস13 Pকত য়

২ ই - কোর লিকংবো ঈ - কোসরর পসর ই - কোর লিকংবো ঈ - কোর োকস উভয় লিমস ঈ - কোর য় এবং ওই ঈ - কোর পব13বস13 Pকত য়

৩ উ - কোর লিকংবো ঊ - কোসরর পসর উ - কোর লিকংবো ঊ - কোর োকস উভয় লিমস ঊ - কোর য় এবং ওই ঊ - কোর পব13বস13 Pকত য়

৪ অ - কোর লিকংবো আ ndash কোসরর পসর ই - কোর লিকংবো ঈ - কোর োকস উভয় লিমস এ - কোর য় এবং ওই এ - কোর পব13বস13 Pকত য়

১ অ + অ = আ ( gtো ) লিম + অচ = লিমোচ সব + অ3ী = সবো3ী অ + আ = আ ( gtো )পদম + আ = পদমো শভ + আলি = শভোলি আ + আ = আ ( gtো )4োয়ো + আবত = 4োয়োবত মো + আতমো = মোতমো আ + অ = আ ( gtো )লিবদযো + অংকোর = লিবদযোংকোর Pো + অ13 = Pো13 ২ ই + ই = ঈ ( gtী )অলিত + ইব = অতীব লিগলির + ইনদর = লিগরীনদর ই + ঈ = ঈ ( gtী )পলির + ইকষো = পরীকষো অলি3 + ঈশবর = অ3ীশবর ঈ + ঈ = ঈ ( gtী )মী + ঈশবর = মীশবর 3ী + ঈ = 3ী ঈ + ই = ঈ ( gtী )রী + ইনদর = রীনদর মী + ইনদর = মীনদর ৩ উ + উ = ঊ ( gt )মর + উদযো = মরদযো কট + উলিকত = কটলিকত উ + ঊ = ঊ ( gt )ঘ + ঊলিম13 = ঘলিম13 লিZ + ঊলিম13 = লিZলিম13 ঊ + ঊ = ঊ ( gt )রP + ঊলিম13 = রPলিম13

৪ অ + ই = এ ( লেgt )র + ইনদর = সরনদর লেPোগ + ইনদর = লেPোসগনদর অ + ঈ = এ ( লেgt )গ + ঈ = গস

র + ঈ = সর আ + ই = এ ( লেgt )Pো + ইষট = Pসষট 3ো + ইনদ = স3নদ আ + ঈ = এ ( লেgt )রমো + ঈ = রসম দবোরকো + ঈশবর = দবোরসকশবর

COMPUTER THE WORLD OF WINDOWS 10

DONE IN THE PREVIOUS CLASS PAGE NO-83A TICK THE CORRECT OPTION BACKGROUND DISPLAY AREA RESTORE THREE

MATHEMATICS Topic ndash NumbersChapter ndash Natural numbers and whole numbers

Study item Properties of whole numbers for subtraction1) Closure property When we do subtraction of two whole numbers we can not get a whole number in all time Example 8 ndash 3 = 5 a whole number 0 ndash 6 = -6 is not a whole numberTherefore the subtraction of two whole numbers is not satisfying closure property2) Commutative property If x and y are two whole numbers then x ndash y is not equal to y ndash xExample If x=16 and y = 7 then x ndash y = 16 ndash 7 = 9Again y ndash x = 7 ndash 16 = - 9 Therefore x ndash y not equal to y ndash x Therefore the subtraction of two whole numbers is not satisfy commutative3) Associative property If x y and z are three whole numbersThen x ndash ( y ndash z ) not equal to ( x ndash y ) ndash z Example If x = 20 y = 10 and z = 6Therefore x ndash (y ndash z ) = 20 ndash(10 ndash 6 ) = 20 ndash 4 = 16(X ndash y ) ndash z = (20 ndash 10) ndash 4 = 10 -4 =6Therefore x ndash(y ndash z) not equal to ( x ndash y) ndash zTherefore subtraction of whole numbers is not satisfying associativity4) Distributive property If x y and z are three whole numbersThen x (y ndash z ) = xy ndash xzAnd (y ndash z)x = yx ndash zxExample If x = 10 y = 6 and z = 4x(y ndash z ) = 10(6 ndash 4 ) = 10times6 ndash 10times4 = 60 ndash 40 = 20( 6 ndash 4 )times 10 = 6times10 ndash 4times10 = 60 ndash 40 = 20Therefore the subtraction of whole numbers is satisfying distributive property5) Existence of identity For any whole number x X ndash 0 = x but 0 ndash x = - x not equal to xThus for subtraction no identity number existsException 0 ndash 0 = 0 so 0 is its own identity for subtraction

Class VIISubject Topic Summary Execution

Hindi 2ndlang वचन जो सजञा शबद निकसी वसत या पराणी क एक या अनक होन का बोध कराया उनह वचन कहत ह जस डका- डकयह दो परकार की होती ह-

क) एकवचन-शबद क जिजस रप स उसक एक होन का बोध हो उस एक वचन कहत ह जस निकताब गमा आदिद

) बहवचन-शबद क जिजस रप स उसक आन ोन का पता च उस बहवचन कहत ह जस डक निकताब निततलियाआदिद

निनमनलिखित शबदो को एकवचन स बहवचन म बदोम- हमजानित- जानितयानारी- नारिरयामिमतर ndashमिमतरोपसतक -पसतकसडक-सडकबोत-बोतनाहर-नहररपए-रपया

Bengali বইndashবোংো োলিতয পলিরচয়

পোঠndash১৪ গলপ - অপর কলপো পর

লেক - লিবভলিতভষ বসনদযোপো3যোয়লেকndash রবীনদর পরবতf বোংো কোোলিতয 3োরোর উসgসPোগয োম পরকলিতসপরমী লিবভলিতভষ বসনদযোপো3যোয় তোর লেীসত লেPম বোসর বোসর লিফসর এসস4 গরোম বোংোর পরকলিতর কো লেতমলি এসস4 গরোমী মো লিচতরগলপndash অপর কলপো গলপোংটি লিবভলিতভষ বসনদযোপো3যোসয়র লিবযোত উপযো পসর পাোচোী লেসক গীত অপ অ13োৎ পসর পাোচোী তো অপর কলপোর লেকনদরীয় চলিরতর এই অংস আমরো পোই বোক অপসক বোক অপ কলপো লিবোী লে দসরর অ গো4 লেদস মোসয়র মস লেোো রপকোর রোসয পোলি2 লেদয় দপরসবো মোসয়র মসর কসর কোীদোী মোভোরত এর করসকষতর Pসjর ব13ো শস তোর মোবীর কস13র পরলিত ব2 মমতো য় আবোরপালিসত বলি13ত Pসjর অমোপত অং লে লিসই মোপত কসর বোলি2র লিপ4স বাো বোগোস লিকংবো উঠোসর লিশমসর কলপো লিবো এোস পরকো লেপসয়স4

১ অপর কলপো গসলপর লেক লেক তোর মপসক13 লেসো২ অপর কলপো গলপটি লেকোো লেসক গীত গলপটির ম ভোব লেসো

GEOGRAPHY CHAPTER 7EUROPE

CHAPTER COMPLETE EXERCISEFill in the blanks1 Europe is a continent that comprises the western part of Eurasia2 Eurasia and Africa are connected into one large land mass known as Afroeurasia3 The Strait of Gibraltar separates Europe and Africa4 Europe is surrounded by the Arctic Ocean to the north5 The British Isles includes the island countries of Great Britain and Ireland

Name the following 1 Connects Africa to Eurasia - Isthmus of Suez2 Largest country in the world in terms of area ndash Russia3 A term used collectively for the five countries in northern Europe ndash Nordic Countries4 The capital of Montenegro - Podgorica5 the largest fjord in Norway ndash Sognefjord

Match the following Column 1 Column 2a Albania iii Tiranab Belgium i Brusselsc Denmark v Copenhagend Finland ii Helsinkie Hungary iv Budapest

CHEMISTRY Chapter 2 ndashElement and Compound

Atom - An atom is the basic unit of an element or the smallest particle of an element non capable of independent existence Atom is built up of three sub atomic particles electron proton and neutron

Nucleus-It is the centre of an atom In the centre of the atom contains proton (positively charged particles ) and neutrons ( particles carrying no charge )

Orbits- It surround the nucleus in which revolve electrons (negatively charged particles)

Answer the following

1) What are MetalloidsAns - Certain elements using properties of both metal and non-metals are called metalloids Example Silicon arsenic and antimony

2) What are Noble gasesAns - Certain elements are present in the air and are chemically inert or unreactive Such elements are called rare gases or noble gases Example helium neon argon and Krypton

English 2 Sentences based on meanings

Kinds of sentences

Assertive or declarative to convey information or simply make a statement

Interrogative to ask different types of questions

Imperative to command or instruct someone or make a request

Exclamatory to express strong feelings and emotions

Exercise B1 Stop it ( Exclamatory)2 May you always be happy

together ( Exclamatory)3 He does not like sports

( Assertive)4 Please pass me the salt

( Imperative)5 How dare she talk to me like

that ( Exclamatory)6 May success bless your effort

( Exclamatory)7 Canrsquot you wait for sometime

(Interrogative)8 Did anybody tell you about it

( Interrogative)9 I saw her waiting for the bus

( Assertive)10 Could you please take a

message for me ( Interrogative)

Homework Ex ABiology Chp -2

Classification of Plants

Today we discuss about usefulness of bacteria We also discuss what the harmful effects of bacteria are

89 How bacteria are useful for usbull Bacteria is helpful in many ways forhuman being i) Production of medicine - antibiotics vaccine etcii) Formation of curd by lactobacillusiii)Nitrogen fixation in Leguminousplant by Rhizobiumiv) Increase soil fertility by absorbingatmospheric nitrogen and convert it into nitrates and nitritesv) Cleaning the environment by converting the complex substances into simple substancesvi) Tanning of leathervii) Retting of Fibersviii) Formation of compost by acting onanimal dung and agricultual cases1x) Biogas production by decomposingplant and animal wastex)Help In Nutrition by producing vitamiacutemBand kx1) Some bacteria are used to give specialflavour to tea coffee and coccaQ10- Name some diseases and there causativebacteriabull Diseasescausative bacteria1 CholeraVibrio cholerae2 Tuberculosis - Mycobacterium tuberculosis3 Diptheria -Corynebacteriumdiphtheriae4 Pneumonia - Streptococcus pneumoniae

Math Number system

Chapter Fraction

Study item Using lsquoofrsquoThe word lsquoofrsquo between any two fractions is to be used as multiplicationExample 57 of 56 = 57 times 56 = 5times8 = 40Study item Using BODMASThe word lsquoBODMASrsquo is the abbreviation formed by taking the initial letters of six operations(i)Bracket (ii) of (iii) Division (iv) Multiplication (v) Addition (vi) SubtractionAccording to BODMAS rule First of all the terms inside Bracket must be simplified then lsquoofrsquo lsquoDivisionrsquo lsquoMultiplicationrsquo lsquoAdditionrsquo lsquosubtractionrsquo

Study item Removal of Brackets

There are four Brackets of algebra in Mathematics In a complex expression four types of brackets are used Order of removing the brackets is first ----- then ( ) then finally [ ]

Class VIIISubject Topic Summary Execution

Chemistry Hydrogen General methods of preparation of hydrogen

By the action of dilute acids on metals

Calcium Reacts readily to form chloride salt and hydrogen

Ca + 2HCl rarr CaCl₂ + H₂uarr

Magnesium

Aluminium

Zinc

React readily to form salt and hydrogen

Mg + 2HCl rarr MgCl₂ + H₂uarr2Al + 6HCl rarr 2AlCl₃ + 3H₂uarrZn + 2HCl rarr ZnCl₂ + H₂uarr

Question 4 ) Give reasons for the following

(a) Hydrogen be used as a fuel

Solution

Hydrogen is used as a fuel because it has a high heat of combustion Some significant fuels are coal gas water gas and liquid hydrogen

(b) Though hydrogen is lighter than air it cannot be collected by downward displacement of air

Solution

Hydrogen is lighter than air so it is possible to collect the gas by downward displacement of air But it is not safe to do so since a mixture of hydrogen and air can lead to an explosion

(c) A pop sound produced when hydrogen is burnt

Solution

Impure hydrogen gas burns in air with a pop sound This is because of the presence of impurities in it

(d) Helium replaced hydrogen in weather observation balloons

Solution

It forms a mixture with air that can explode when there is a small leakage of hydrogen in a balloon So helium has replaced hydrogen

(e) Nitric acid not used for the preparation of hydrogen gas

Solution

(e) By the action of nitric acid on metals hydrogen cannot be produced because it also releases nitrous oxide and nitric oxide and oxides the hydrogen to form water

Biology Chp-2 Reproduction in plants

Today we discuss different methods of artificial propagation like cutting-rose sugercane Layering ndashguava lemon china rose etc Grafting- mango apple etcMicropropagation ndashorchid asparagus etcWe also discuss about advantages and disadvantages of vegetative propagation

Q7 Define the following terms i) Explant In tissue culture techniquea tiny piece of bud shoot or any other partof plant from where new tissue develop ii) Callus The cells of the tissue divide andgrow into a mass of undifferentiated cells from explant iii) Plantlet After few days callus differentiate into a small plant with roots and shootQ8 what are the advantages and limitations of tissue culture or micropropagation

Advantages i ) It produacuteces superior quality plantsii)It can be applied to interspecifie hybridsiii) It is useful to grow seedless plants bull Limitations i) It cannot be used for all plantsii)It is not easy to handleQ9 Write advantages of vegetative propagationi) It is a quick and easy method ofproducing new plantsii) This method need less time to matureiii) The new plants are exact copies of the parentiv) it is extremly useful for growing seedlessplants like banana grapes etc Q10 Write some disadvantages of vegetativepropagationi) Dišeases present in the parent plant gettransferred to all in new plantsii) Overcrowding of new plants causes competition for sunlight water and nutrients which affects growth of plantsplant

Physics Chapter 2 Physical Quatites and Measurements

Here We Will Do Some QuestionsRelated To Chapter 2

Select the correct alternative A block of wood of density 08gcm-3 has a volume of 60cm3 The mass of the block is

1 608 g

2 75 g

3 48 g

4 0013 g

Solution 348 g

The density of aluminium is 27g and that of brass The correct statement is

1 Equal masses of aluminium and brass have equal volumes

2 The mass of a certain volume of brass is more than the mass of an equal volume of aluminium

3 The volume of a certain mass of brass is more than the volume of an equal mass of aluminium

4 Equal volumes of aluminium and brass have equal masses

Solution 2 The mass of a certain volume of brass is more than the mass of an equal volume of aluminium

MATHEMATICS Ch 6Sets

Exercise 6(C)1 Find all the subset of each the following sets(i) A = 57 (iii) C = x xisin W x le 2(iv) p p is a letter in the word lsquopoorrsquo

Solution (i) All the subsets of A are ϕ 5 7 57

(iii) All the subsets of C are ϕ 0 1 2 01 02 12 012

(iv) All the subsets are ϕ p o r po or por

4 Given the universal set = -7-3-105689 find (i) A = x xlt2 (ii) B = x -4ltxlt6 Solution

(i) A = -7-3-10(ii) B = -3-105

5 Given the universal set = x xisin N and xlt20 find

(i) A = x x = 3p pisin N (iii) C = x x is divisible by 4 Solution

(i) 369121518 (iii) 481216

6 Find the proper subset of x x2-9x-10 = 0 Solution

ϕ 10 -1

Working x2-9x-10 = 0 rArr x2-(10-1)x-10 = 0

rArr x2-10x+x-10 = 0 rArrx(x-10)+1(x-10) = 0

rArr (x+1) (x-10) = 0

11 Let M = letters of the word REAL and N = letters of the word LARE Write sets M and N in roster form and then state whether (i) M sube N is true (ii) N sube M is true (iii) M = N is true

Solution M = real and N = lareSo (i) Yes (ii) Yes (iii) Yes

English 2 Twelfth Night ndash Shakespeare

A noble man named Orsino in the kingdom of Illyria is deeply in love with a lady called lady Olivia She is in mourning for her dead brother so she will not even think about marriage At this time a sea storm causes a terrible shipwreck and a young lady called Viola is swept onto the shore She thinks that her twin brother Sebastian is drowned A sea captain tells her about Orsino and his love for Olivia Viola wishes to work in Oliviarsquos home but feels she will not be employed So she dresses as a man calls herself Cesario and gets work at the house of OrsinoViola (now Ceasario) is much liked by Orsino and becomes his page She falls in love with Orsino Orsino sends Ceasario to deliver messages to Olivia Olivia herself falls for the beautiful young Ceasario believing Viola to be a man

2 Answer the following questionsa Why does Orsino ask the musicians to play onOrsino asks the musicians to play on because music feeds his desire He calls upon the musicians to play music so that his hunger for love could be replenished with an excess of musicb What does Valentine tell about OliviaWe learn from Valentine that Olivia is in mourning for her brother she wears a veil and has vowed that no one will see her face for another seven yearsand she refuses to marry anyone until thenc From the exchange between Orsino and Valentine what do you think their relationship isValentine is one of orsinod attendants He was sent to Olivia as a messenger of love but was not allowed to speak to here Who is Olivia mourning for and whyOlivia is mourning for her dead brother

Homework Q fHistory and Civics

Growth of Nationalism

Important dates to remember1769-Napoleon born on 15thAugust1789-Fall of Bastille on 14th July and the beginning of the French revolution declaration of the rights of Man on 26thAugust1793-King Louis XVI executed on January 211764-The Sugar Act passed1765-The Stamp act passed1774-The first congress of Philadelphia1776-The declaration of American Independence of on 4th July1777-Defeat of the British at Saratoga1781-Surrender of lord Cornwallis at Yorktown1783-The treaty of Versailles1804-Napoleon becomes the emperor1813-Battle of Leipzig or Battle of nations in which Napoleon was defeated by the Allies1815-Battle of Waterloo June 18 in which Napoleon was defeated and captured1821-Death of Napoleon in StHelena1860-Abraham Lincoln elected President of the USA1861-The civil war began 1864-Abraham Lincoln elected President of the USA for the second time1865-Slavery abolished in the US

Name the following- The queen of Louis XVI

Marie Antoinette The three philosophers of France

VoltaireMontesquieuJean Jacques Rousseau

The British general whose surrender brought the war in America to an endLord Cornwallis

The first president of the USAGeorge Washington (1732-1799)

The first southern state to secede from the unionSouth Carolina

The author of the book lsquoUncle Toms CabinHarriet Beecher Stowe

Homework-Learn

Class IXSubject Topic Summary Execution

Economics

Types of economies Today I am going to share you the concept of economic growth and economic development Few questions will be given from the previous study material dated 942020

Meaning of economic growthAnswer) The term economic growth generally means anincrease in national income or per capita output or income over time It indicates towards quantitative growth of a country

Meaning of economic developmentAnswer) Economic development is defined

as a process whereby the real per capita income of a country increases over time along with fall in poverty ratio unemployment and income inequality etc

Distinguish between economic growth and economic development

Basis Economic growth

Economic development

Scope It has narrow scope as it refer only to rise in per capita income

It has wide concept since it includes qualitative changes as well

Concerned matter

It is concerned with the rise in income

It is concerned with not only rise in income but also reduction of poverty income inequality and unemployment

Focus Economic growth does not focus on economic development

Economic development focus on economic growth plus qualitative changes

Distinguish between capitalist economy and socialist economy

Ownership

Motive

Tool

Means of production are owned and managed by private people

Self interest and profit earning is the main motive

Price mechanism is a main tool to solve the economic problems

Means of production are owned and managed by the government

Social welfare is the main motive

Economic planning by the government is the main tool to solve the economic problem

Competition

Distribution of income

There exist large competition among buyers and sellers

There is existence of large inequalities of income

There is no such competition

There exist less inequalities of income

Math Topic ndash AlgebraChapter -Factorisation

Study item Factorising by taking out common factorSome solved sums from exercise 41

1) (i) 8xy3 + 12x2y2

= HCF of 8xy3 and 12x2y2 is 4xy2

= 4xy2(2y + 3x )

4) (ii) 28p2q2r ndash 42pq2r2

= HCF of 28p2q2r and 42pq2r2 is 14pq2r = 14pq2r (2p - 3r )5) (ii) 14mn + 22m - 62p=HCF of 14mn 22m and 62p is 2= 2(7mn + 11m - 31p)7) (ii) 3a(x2 + y2) + 6b (x2 + y2) = HCF of 3a(x2 + y2) and 6b(x2 + y2 ) is (x2 + y2)= ( x2+ y2 )(3a + 6b )9) (ii) x(x2 + y2 ndash z2 ) + y(-x2ndashy2 + z2 ) ndash z(x2+ y2 ndash z2 )= x(x2 + y2 -z2) ndash y-(x2 + y2 -z2) -z(x2 + y2 ndash z2)=x(x2 + y2-z2) -y( x2 + y2-z2) ndash z (x2 + y2 -z2)= (x2+ y2 ndash z2)(x ndash y ndash z )

Commercial Studies

Introduction to Accounting and Book-keeping

Today I am going to share you the meaning of Accounting and Book-keeping and its related terms bullAccounting bullBook Keeping bullAccountsbullTypes Of Accounts bullAccounting Cycle

bull Meaning of accounting

Ans )Accounting is the art and science of recording classifying and summarising monetary transactions

bull Meaning of Book-keeping

Ans) Bookkeeping is the art of recording business transactions with the view of having a permanent record of them and showing their effect on wealth

bull Meaning of account

Ans) The term account means a record of business transactions concern a particular person of firm asset or income or expense It is a summarised record of all transactions which take place in an accounting year

bull Types of accountsPersonal accounts ndash Personal accounts relating

to person and Organisation are known

as personal accounts Example Ramrsquos Account ABC amp Co Account etc

Real account - The accounts related to tangible and intangible assets are called real accountsExample Cash Account Furniture Account etc

Nominal account- Accounts related to expenses losses incomes and gains are known as nominal accountsExample Wages Account Salary Account Discount Account etc

bull Accounting cycle Accounting cycle refers to a complete sequence of accounting activities It begins with recording of transactions and ends with the preparation of a balance sheet

English 1 Transformation of sentences

Sentences A sentence is a group of words which makes complete sense

a Assertive sentencesb Imperative sentencesc Interrogative

sentencesd Exclamatory sentences

Sentences can be changed from one grammatical form to another without changing the meaning of the sentence This is known as transformation of sentences

Exercise 6Rewrite the following sentences according to the instructions given below without changing their meanings

1 As soon as he saw the beer he jumped into the river ( Begin No sooner)

2 None but brave deserve the fair (Begin the bravehellip)

3 This box is too heavy for me to lift ( Use so hellip That instead of too)

4 No one other than a king can live like James Luxurious ( Begin only James)

5 Oh for the wings of a dove (Begin I wishhellip)

BENGALI(2ND LANGUAGE)

ldquo বঙগভমির পরমি ldquo াইকেল ধসদন দতত

পব13পোসঠ আসোলিচত ৩ পরবোস দৈদসবর বস ীবতোরো Pলিদ স এ লেদ -আকো সত-োলি লেদ তোস - ক) বকতো লেক কোর লেো লেকো কলিবতোর অং ) কোর পরলিত বকতোর এই উলিকত গ) এ লেদ আকো সত বসত কী বলিঝসয়স4 ীবতোরো বসত কী লেবোঝ ঘ ) আসোচয অংসর তোৎপP13 কী

উ -ক ) বকতো স কলিব মোইসক ম3দ দতত

Types of AccountPersonal AccountReal AccountNominal AccountBalance Sheet (opening)

কলিব মোইসক ম3দ দসততর রলিচত বঙগভলিমর পরলিত কলিবতোর অং ) কলিব বঙগী অ13োৎ লেদমোতোর পরলিত কলিবর এই উলিকত গ ) এ লেদ আকো বসত কলিবর মোব লেদী রপ আকো লেক লেবোঝোসো সয়স4 আকো লেসক লেPম তোরো স পসর লেতমলি ীব লেদ রপ আকো লেসক পরো রপ তোরো স পরসত পোসর এই মভোবোর কোই কলিব বসস4 ঘ ) পরবো Pোতরোয় Pলিদ কলিবর লেদ আকো লেসক ীব তোরো রপ পরো স পসর তোসত কলিব লিবনদমোতর দঃলিত কোর মতয লিবসর সবোভোলিবক পলিরলিত এবং মোষ মরী তোই পরবোস Pলিদ তা োর মতয য় তবও কলিব লিবচলিত সব ো কোর পলিবীসত লেকউ অমর য় লিক4ই অকষয় য় দীর লেPম লিচরপরবোমো লেতমলি মোসষর ীবও চমোতোই ীব - সতবধতোই মতয ীব দীসত মোষ লিতয পরবোমো তবও লেPব মোষ আপ কতকসম13র মো3যসম মোসষর মস লিসসদর সথো কসর লিসত পোসর তোরো লিচরভোসবর সয় মোসষর মস লিবরো কসর তোসদর মস3য লেকউ পGভসত লিবী সয় গোসও মোসষর মস তোরো লিতযপলিত লিতযবলিনদত

Hindi 2ndlang

काकीी(लिसयारामशरणगपत)

इस कहानी म क न यह बतान का परयास निकया ह निक बचच अपनी मा स निकतना परम करत ह शयाम अबोध बाक ह वह अपनी मा क मरन क बाद उसन अपनी मा क लिए बहत रोया बाद म उस पता चा निक उसकी मा राम क घर ची गई ह आकाश म उडती हई पतग दकर उस हष हआ निक पतग क दवारा वह अपनी मा को नीच उतारगा इसक लिए वह अपनी निपता की जब स दो बार सवा रपया निनकाकर पतग और दो मोटी सी मन वाी अपन भाई स काकी एक कागज पर लिवा कर पतग म लिशव का दिदयानिनकाकर पतग और दो मोटी सी मन वाी अपन भाई स काकी एक कागज पर लिवा कर पतग म लिचपका दिदयाभोा और शयाम कोठरी म रससी बाधनी रह थ तभी उसक निपता करोध म आकर उन स पछ निक कया उनकी जब स रपया निनकाा हभोा डर क मार बताया निक शयाम इस पतग क दवारा अपनी काकी को राम क यहा स उतारना चाहता हनिवशशवर(शयाम क निपता)न फटी पतग उठाकर दी तो उस पर काकी लिा थावह हत बजि होकर वही ड रह गएउनहोन सोचा निक मन अपन पतर को मारा जोनिक अनजान और निनदष थावह अपनी मा कोनिकतना पयार करता ह

helliphellipContinue to next

Computer Application

Java Programming Prog 1Write a java program to input two numbers from user and display the sum or product of them as per user choice Use switch case statementSolve public class sum_product public static void main(String args[]) Scanner sc=new Scanner(Systemin) int abc Systemoutprintln(ldquoEnter two numbersrdquo) a=scnextInt() b=scnextInt() Systemoutprintln(ldquoPress 1 for sum or 2 for productrdquo)

c=scnextInt() switch(c) case 1 Systemoutprintln(ldquoThe sum will be =rdquo+(a+b)) break case 2 Systemoutprintln(ldquoThe product will be =rdquo+(ab)) break default Systemoutprintln(ldquoWrong Inputrdquo) Home Work - Practice in your computer using bluej

Subject Eng Literature (The Merchant of Venice ndash William Shakespeare)Topic Act I Scene 2 Lines 92 to 126 (End of scene) Date 13th April 2020 (5th Period)

[Students should read the original play and also the paraphrase given in the school prescribed textbook]Summary Questions amp Answers

o After Portia has expressed her opinion about the suitors Nerissa informs that she need not bother about any one of them as they have decided to quit Belmont at the earliest opportunity because they do not believe in trying their luck by the caskets which is the only way of winning Portia

o Nerissa then enquires of Portiarsquos opinion about Bassanio who once visited her in the company of the Marquis of Montferrat and says that she had never come across such an ideal love deserving the fairest lady for his bride

o Portia seems to remember Bassanio quite correctly and says that she agrees with Nerissa At this moment a servant informs Portia that the Prince of Morocco has arrived to try his luck by the caskets

o Portia tells Nerissa that if she could welcome this new suitor as gladly as she says farewell to the previous ones she would be glad of his arrival However if he happens to have the virtues of a saint but the black complexion of a devil she would prefer to have him for religious consolation rather than as a husband

(1) NERISSA You need not fear lady (Line 97-103)

the having any of these lords they have acquainted me with their determinations

which is indeed to return to their home and to

trouble you with no more suit unless you may be wonby some other sort than your fathers imposition depending on the caskets

PORTIA If I live to be as old as Sibylla I will die as chaste asDiana unless I be obtained by the manner of my fatherswill I am glad this parcel of wooers are so reasonablefor there is not one among them but I dote on his veryabsence and I pray God grant them a fair departure

(a) Elucidate the idea expressed in the first speech of the above dialogue

In the first speech Nerissa assures Portia that she need not have any fear of being compelled to marry anyone of the suitors who had lately come to Belmont She informs her that they have all decided to return to their respective countries(b) Illuminate the meaning of the phrase ldquoyour fatherrsquos imposition depending on the casketsrdquo

Nerissa means that the suitors of Portia do not find the conditions imposed by the will of her father to their liking They are too hard for them These conditions are that in the event of a suitor failing to choose the right casket (i) he should never disclose to anybody which casket he chose (ii) he can never marry and (iii) he should take his departure immediately(c) Explain the meaning of the term lsquoSibyllarsquo

lsquoSibyllarsquo is the name given by Romans and Greeks to a prophetess inspired by some deity usually the sun-god Apollo She had a very long life The god Apollo granted her as many years of life as she could hold grains of sand in her hand(d) Elucidate the meaning of the term lsquoDianarsquo

lsquoDianarsquo is the goddess of hunting She is also regarded as a symbol of virginity because she never fell in love and never

married(e) Explain the meaning of the first two lines of Portiarsquos speech

Portia says that even if she is to live for centuries like Sibylla she would not marry except in accordance to her fatherrsquos will She asserts that she would not mind remaining unmarried and untouched by a man like Diana the virgin the goddess of hunting unless a man is able to win her by passing the test laid down by her father

Class XSubject Topic Summary Execution

Hindi 2nd

Langबड घर की बटी( मशी परमचद)

lsquoबड घर की बीटीrsquo कहानी का उददशय मधयम वग की घर समसया को सझा कर सगदिठत परिरवार म मिम जकर परम स रहन का सदश दना ह घर म शानित tानिपत करन की जिजममदारी नारी की होती ह यदिद नारी समझदार ह उसम धय और परिरवार क परनित परम ह तो कोई भी घटना परिरवार को निवघदिटत नही कर सकती या कहानी परिरवार को सगदिठत करत हए परम सौहाद स एक रदसर की भावनाओ को समझ करउनका सहयोग करत हए जीवन यापन करन की पररणा दती ह मशीपरमचदर जी न इस कहानी म सयकत परिरवार का परनितनिनमिधतव निकया ह यह कहानी बनी माधव सिसह जो गौरी पर क जमीदार क उनक दो पतरो की हशरी कठ ा निबहारीशरीकात का निववाह एकजमीदार घरान की पतरी आनदी स हआ थाआनदी न द को ससरा क वातावरण म ढालिया थाएक दिदन आनदी का अपन दवर ा निबहारी स झगडा हो जाता ह दोनो भाई एक रदसर स अग होन की कोलिशश करत हसभी बह आनदी न अपन मधर वयवहार स ा निबहारी को घर छोडकर जान स रोक लिया| इस पर बनी माधव सिसह न कहा निक बड घर की बटी ऐसी ही होती ह जो निबगडा काम बना ती ह अतः शीषक साथक ह बड घर की बटी आनदी ह

helliphelliphelliphellipContinue to nextBiology Topic ndash Chp-1

CellWelcome to new session 2020-21Today we will start with Chpter 1 cell CELL

Protoplasm+Cellmembrane Or Cell wall

Cytoplasm+Neucleus

Cytoplasmic+ CytoplasmicOrganelles Inclutions(mitochondria (food Golgi bodies pigments)Ribosome)

What is cellbull Cell is the structural and functional unit of living organismbull According to number of cells organisms areUnicellular - Amoeba bacteria Multicellular - Rose Mango Tiger HumanSmallest cell -bacteria Longest cell - Nerve cellLargest cell - Ostrich egg cellCells are of different size and shapes according to their functionsQ2Write chief functions of following cellorganelles

Q3What is tonoplastVacuoles covered by a covering called tonoplast

Bengali(2Nd

Language)

ফ ফটক ো ফটক (কলিবতো ) ভোষ মসোপো3 gtPোয়

একটি লেমসয়র ীবস লেপরম লিকভোসব ফসট ওসঠ তো লেদলিসয়স4 কলিব লেপরম Pই য় লেই ময়ই বনত কোস পলিরত য় ফ লেফোটো বো োসফোটো লেটো ব2 কো য় লেমসয়সদর ব gtয13 লেপরসমর 4লিব ফসট উসঠস4 এই কলিবতোয় লেপরম মোষসক মত gtযর মস লেফস লিদসয় পরকষস বাোচোসোর gtয োত বো2োয় কলিবতোয় লেমসয়টির পসব13র দঃসর কো বো সও লেমসয়টি লেই পসর পলিক সত চোয়ো োরী ীবসর কোস4 পরম লেPৌবস লেপরমসক পোবোর পরব ইচছো োকসও তো পসর লেলিতবোচকতোয় পলিরত য় কলিব ভোষ মসোপো3 যোয় লেP ক লেপরসমর

কলিবতোয় ব gtযবহত লিবসষ লিক4 সvর অ13 লেদওয়ো ১) রসবোো= লেP লিবলিভনন রকম ডোকসত পোসর২) ো= পোর ৩) ঠলি = লেচোসর বZ৪)আই বস2ো=অলিববোলিত৫)শইসয় = োলিয়ত কসর৬)োতপাোচ= লিবলিভনন পরকোর৭)দ2োম = v কসর বZ কসর লেদওয়ো৮)লেরলিং =লেোোর দৈতরী লেব2ো৯) বনত= একঋত১০) পাোর = বসকরো2

Organelles Functions

1 Endoplasmic reticulum

2 Mitochondria

3Golgibodies

4 Ribosome5Lysosome

6Plastids

7 Centrosome

i) Supportive framework for the cellii) Synthesis and transpost of proteinsRelease of energy in the form of ATPi) Synthesis and secretion of enzymes hormoneii) Formation of vacuoles lysosomei) Protein Synthesisi) Intracellular digestionii) Destroy foreign substancei )Leucoplast - stores starchii)chloroplast - trap solar energyiii) Chromoplast - imparts colour toflowers amp fruitsi) Initiates and requlates cell division

কলিবতো তোর অ13সক ভোষোয় পরকোো কসর ঘলিরসয় ব যকত কসরস4 লেপরসমর ফতো আর লিবফতো লেক গোঢ় কসর লেদোসো কলিব ভোষ মসোপো3 যোসয়র অলিভবসর অ যলিদক

Economics

Factors of Production

Welcome to the new sessionToday we are going to start the first chapter of Class XThe name of the chapter is Factors of productionBy the name I hope you all can recall a glimpse of what you have learnt in the second chapter of Class IX

NowProduction is the process of creating the various goods and services which are consumed by the people of the country to satisfy their wants

Thus it is the process in which some materials are transformed from one form to another to create utility and value in goods

For example utility can be created by changing the form of a commodity ie

Making of table out of wood by a carpenter for his customer here the wood is getting transformed into table creating utility for his customer and he can also command a price for it

On the other hand Housewives perform very

useful activities at home which create utility but their domestic activities are not included in production because they have no money value

So we can also say that Production denotes two things firstly creation of utility and secondly creation of value

Production is not complete unless it reaches the consumer

An increase in production will increase the economic welfare of the consumers and hence the aim is to raise the production level of the country

Again production of a good or service is only possible if certain resources or

Questions

1 What do you mean by production

Answer Production means the creation of goods and services for the purpose of selling in the market

In fact production involves the transformation of inputs into outputs

Hence production denotes two thingsCreation of utility and creation of valueUtility and value can be created by changing the form by changing the place by changing the time and by rendering services

Example Transformation of raw

materials into finish goods such as potter creates utility by converting mud into utensils assembling of small parts to make bigger machinery

Production also includes services such as distribution and marketing

2 What are the factors of production

Answer Factors of Production refers to the resources and inputs needed for producing goods and servicesThese inputs can be classified as

Land Labour

Capital Enterprise

Land Land is defined to include not only the surface of the earth but also all other free gifts of nature(for example mineral resources forest resources and indeed anything that helps us to carry out the production of goods and services but is provided by

inputs are used together in right proportion

A resource or an input which helps in the process of production to obtain an output is called FACTOR OF PRODUCTION

These factors of production can broadly be categorized into four parts 1LAND 2LABOUR3CAPITAL4ENTERPRISE (ORGANISATION)or Entrepreneur

The above factors are all interdependent on each other and they play a major role in production process

FACTORS OF PRODUCTION

LANDCAPITAL

LABOUR ENTREPRENEUR

nature free of cost)LabourLabour refers to the human efforts that need to be combined with other factors of production for creating an output

CapitalAll man ndash made means of production is called capita example machineries which help in further production Money when used for starting any business for purchasing raw materials machinery tools etc it is regarded as capitalCapital also includes physical capital like factories machineriestoolsbuildingsequipments etcEnterpriseThe task of bearing risks is called enterprise and the person who bears these risks of business is called the entrepreneurThus an entrepreneur is one who organises production takes important decisions regarding production hires and purchases factors of production and bears the risk and uncertainty involved in productionOrganisation refers to the services of an entrepreneur who controls organises and undertakes all risks One who plans organises and manages a business enterprise is an organiser

Physics Chapter 1 Force

Force is an external agent capable of changing the state of rest or motion of a particular body It has a magnitude and a direction The direction towards which the force is applied is known as the direction of the force and the application of force is the point where force is applied The Force can be measured using a spring balance The SI unit of force is Newton (N)

Question 1

State the condition when on applying a force the body has

(a) the translational motion

(b) The rotational motion

Solutions

(a) Translational motion is produced when the body is free to move

(b) Rotational motion is produced when the body is pivoted at a point

Question 2

Define moment of force and state its SI unit

Solutions

The moment of force is equal to the product of the magnitude of the force and the perpendicular distance of the line of action of force from the axis

of rotation

The SI unit of moment of force is Newton times meter

= Newton meter (Nm)

Commercial Studies

Stake holders In this topic you will be come to know about the meaning and concept of stakeholders

How stakeholders are different from shareholders

Questions1 What do you mean by the term stake holdersAnswer) The term stake holders have developed from the words which mean an interest or expected benefit Stakeholders mean all those individuals groups and Institutions which have a state (interest) in the functioning and performance of a commercial organisation or a business enterprise2 What do you mean by share holdersAnswer) The person and Groups who own the shares of the joint stock company by providing capital to the company are called shareholders Shareholders are the internal stakeholders shareholders are one out of several stake holders3 How are shareholders different from stakeholdersAnswer)i) The term shareholders is related to only joint stock company whereas stakeholders are related with all business organisationsii) Stakeholders maybe any individual having financial stake in business organisation whereas a shareholders are those individuals who are holding shares in the company4) How are shareholders different from creditorsAnswer) i) Shareholders are internal stakeholders while creditors are external stakeholdersii) Shareholders invest in the capital of the company whereas creditors give loan to the companyiii) Shareholders are the members of the company with voting rights but creditors are not the members of the company

English 1 Transformation of sentences

Sentences A sentence is a group of words which makes complete sense

e Assertive sentencesf Imperative sentencesg Interrogative sentencesh Exclamatory sentences

Sentences can be changed from one grammatical form to another without changing the meaning of the sentence This is known as transformation of sentences

Exercise 1 Change the following affirmative sentences into Negative sentences

a He is a good manHe is not a bad man

b Ram loves SitaRam is not without love for Sita

c Only he stood first in the classNone but he stood first in the class

d Ankit was wiser than he

He was not so wise as Ankite He did it

He did not fail to do itf As soon as I reached college the

bell rangNo sooner did I reach college than the bell rang

g He finished everythingHe left nothing unfinished

h It always pours when it rainsIt never rains but it pours

Math Topic Commercial MathematicsChapter ndash Goods and services Tax

What is GSTAns It is a abbreviated term of Goods and Service Text which is an indirect tax levied on the sale of goods and rendering servicesSome terms related to GSTDelar Any person who buys goods or services For resale is known as a delar A delar Can be a firm or a companyIntra-state sales Sales of goods and services within the same state or same union territory are called intra- state salesInter-state sales Sales of goods and services outside the state or union territory are called Inter-state sales4) Input GST GST is paid by dealers on purchase of goods and services are called input GST5) Output GST GST is collected from customers on sale of goods and services are called output GST6) Types of GST There are three taxes applicable under GST(i) Central Goods and Services Tax (CGST)(ii) State Goods and Services Tax (SGST) or Union Territory Goods and Services Tax (UTGST) Both these taxes are levied on intra-state sales Here GST is divided equally among central and state governments(iii) Integrated Goods and Services Tax (IGST) IGST is levied on inter- state sales It is also levied on import of goods and services into India and export of goods and services from India

Subject Eng Literature (The Merchant of Venice ndash William Shakespeare)Topic Act III Scene 4 Lines 1 to 44 (Portia hellip To wish it back on you fare you well Jessica)[Students should read the original play and also the paraphrase given in the school prescribed textbook]

Summary Questions amp AnswersIn this scene we suddenly find a new element in the character of Portia We have already seen her possessed of every graceful womanly quality but now she shows that she is capable of rapid decision and determined action She shows this by her sudden resolve to hasten to Venice with a daring scheme for the rescue of Antonio This is an important scene in the dramatic action for it leads up to and renders possible the striking events of the famous trial scene which is one of the greatest striking elements of the play Moreover the fact that all the characters of importance are now assembled together in Venice makes the union of the main plot and the secondary story complete

(1) LORENZO Madam although I speak it in your presence(Line 1-9)

You have a noble and a true conceit

Of god-like amity which appears most strongly

In bearing thus the absence of your lordBut if you knew to whom you show this honourHow true a gentleman you send reliefHow dear a lover of my lord your husbandI know you would be prouder of the workThan customary bounty can enforce you

(a) Where is Lorenzo Why is he here To whom is he referring as lsquoMadamrsquo

Lorenzo is at Portiarsquos residence He had met Salerio on the way and Salerio had begged him to come along with him to

o In this scene Portia Nerissa Lorenzo Jessica and Balthazar appear

o Portia requests Lorenzo and Jessica to be in charge of her house during her absence from Belmont because she and Nerissa have decided to spend the days in meditation and also in visiting the holy places in the neighbourhood of Belmont She has already instructed her people to acknowledge both Lorenzo and Jessica as master and mistress of house during her absence Lorenzo and Jessica gladly agree to look after the house of Portia

handover the letter from Antonio to Bassanio The letter carried the bad news about Antoniorsquos arrest for non-payment of loan taken from Shylock Hence Salerio might have preferred company to break this bad news to Bassanio He is referring to Portia as Madam(b) What does Portia say on hearing the above extract

Portia says that she has never regretted doing good to others Friends who spend a lot of time together and really are there for each other have many traits in common As Antonio is Bassaniorsquos best friend saving him is like saving Bassanio who is like her own soul She asks Lorenzo to take care of management of the house till Bassanio is back(c) What does Portia send with Bassanio and why

On hearing about Antoniorsquos troubles on account of Bassanio her husband Portia immediately sends him with enough gold to repay the debt many times over to Venice to help Antonio out of his misfortune

(2) Lorenzo Madam with all my heart (Line 36-40)

I shall obey you in all fair commands

Portia My people do already know my mindAnd will acknowledge you and JessicaIn place of Lord Bassanio and myselfSo fare you well till we shall meet again

(a) Where are Lorenzo and Portia at this time What lsquofair commandsrsquo are given to Lorenzo

Lorenzo and Portia are at Belmont during this scenePortia reveals to Lorenzo that she has sworn to contemplate in prayer at a monastery around two miles away until her husband returns from Venice She tells him that Nerissa would accompany her and asks him to manage the house with Jessica till things are settled In response Lorenzo tells her that he would be obliged to do whatever she asks him to do(b) Where is Portia actually going and why

Portia tells Lorenzo that she would live a life of contemplation and pray at a monastery which is two miles away from her place In reality Portia plans to go to Venice in disguise with Nerissa and argue the case in defense of Antonio She is very sure that her plan would succeed

ClassXI (ScienceHumanitiesCommerce)Subject Topic Summary Execution

Computer Science

(APC)

Ch ndash 1 Numbers

(Numbers in different bases and

their Arithmatical operations)

Number System In computers Number System is defined as a writing system to represent the numbers in different ways ie we are using different symbols and notations to represent numbers There are four ways we can represent the number ndash Binary Decimal Octal and Hexadecimal

Decimal Number SystemThis number system consist 10 digits These are 0 1 2 3 4 5 6 7 8 amp 9

Binary Number SystemThis number system has only two digits these are 0 and 1 Here 0 stands for off while 1 stands for on

Octal Number SystemThis number system has 8 digits these are 0 1 2 3 4 5 6 amp 7

Hexadecimal Number SystemThis number system has 16 digits these are 0 1 2 3 4 5 6 7 8 9 A B C D E F Here the value of the alphabets are as follows A=10 B=11 C=12 D=13 E=14 F=15

Rules for conversion decimal number to Binary1 Divide the decimal number by 22 If the number will not divide equally by 2 then round down the answer to the nearest whole number (integer)3 Keep a note of the remainder it should be either 0 or 14 Keep repeating the above steps dividing each answer by 2 until you reach zero5 Write out all the remainders from bottom to top This is your binary solution

For example Lets convert 32 to binary 2 32 2 16 - 0 2 8 - 0 2 4 - 0 2 2 - 0 2 1 - 0 0 - 1

The binary equivalent of 3210 is 1000002

Try the follwing youself1 2410

2 4810

3 1210

History GROWTH OF NATIONALISM

The second half of the 19th century witnessed growth of political consciousness and a sense of Nationalism among the IndiansThere were various factors for growth of Indian Nationalism- As a result various political associations were formed in different provinces by the educated Indians Surendranath Banerjee organized a meeting of National conference at Calcutta Ultimately the National Congress was founded in Bombay in 1885This body became the vanguard of Indian struggle for freedom The congress leaders were known as moderates because they followed a policy of prayer and petition A large number of Indian leaders had experienced in political agitation The Political situation of England was also changed Moreover increasing revolutionary activities in Maharashtra Punjab and Bengal became serious concern to the British Government In this

QUESTION1 What do you mean by Nationalism ANSWER 1 Nationalism is defined as loyalty and devotion to own nation especially a sense of national consciousnessQUESTION 2 What are the causes of nationalism ANSWER 2 There were various factors for growth of nationalism

1 Spread of western education2 The progress of vernacular press and

patriotic literature3 The economic exploitation of our

country by the colonial rulers4 International affairs

QUESTION 3 Who organized National conference in Calcutta in 1883 ANSWER 3 Surendranath BanerjeeQUESTION 4 When did Indian National Congress formANSWER 4 Indian National Congress was formed in 1885 in BombayQUESTION 5 Who were ModeratesANSWER 5 The Early Nationalists were also known as Moderates Their emergence marked

background Lord Curzon became Viceroy in India He had no respect for the Indian National Congress

the beginning of the organized national movement in India They believed in British justice and were loyal to them They followed a policy of prayer and petition They demanded constitutional reforms of our country Impotant Moderate leaders were Pherozshah Mehta Dadabhai Naorozi and Surendranath Banerjee etcQUESTION 6 What do you know about Extremism in Indian National movementANSWER 6 In the beginning of 20th century a new class of national leaders emerged in India which was different from the moderate groups They started more aggressive movement against the British empire The goal of extremists was ldquoswarajrdquo Important extremist leaders were Bal Gangadhar Tilak Lala Lajpat Rai Bipin Chandra Pal etcQUESTION 7 Mention the places which were the main centres of Revolutionary movementANSWER 7 Maharashtra Bengal and Punjab

Physics

Chapter Dimensional Analysis

(Summary)

The dimensions of a physical quantity are the powers to which the fundamental units are raised in order to obtain the derived unit of that quantit

The physical quantites lengthmasstime are represented by [L] [M] [T] resp let they are raised to powers ( dimesions) abc resp then any physical quantity can be represented by [ La Mb Tc ] Examples

1 Area area = L x B = [L] x [L] = [M0 L2 T0 ]

2 Density density = massvolume = [M][L3] = [ M L-3]

3 Velocity velocity = distancetime = [L][T] = [LT-1]HW Try to find out dimension of acceleration Acceleration = velocity timeNB One can find the SI Units Using Dimension Analysis Such as for area we have [L2] so its SI unit is m2

Biology Topic ndash Chp-1 The living world

Today we will start the first chapter the living world Here we discuss about the characteristics of living organism and what are the difference between them and nonliving substances We also discuss about the contribution of different Scientists

There are over 500000 species of plants andover a million species of animal are present on earth Some 15000 new species were discovered every yearQ1 What is a living organismbull A living organism is primarily physico -chemical material that demonstrate a high degree of complexity is capable of selfRegulation possesses a metabolism and perpetuates itself through timeQ2 What are the differences between livingand non-livingsi) Compared with non-living living organisms

have more complex organised structure and their use of energy is more controlled amp efficientii) Living things reproduce their own kind by forming new cells which contains copies of their genesiii) Each organism has some degree of homeostasisie it is able to make adjustments so that internal environment remains constantQ3 Write contributions of following Scientists i) Aristotle - One of the first theories in Biology places all living things in a hiearchieii) AV Leeuwenhoek - was the first to observe living single celled organisms under microscopeii) Carolus Linnaeus - developed the binary system for naming of organisms and classificationiii) Geregor Johann Mendel ndash discoverbasic principles of inheritanceHomework i) C Darwin ii)Schleiden

Math Trigonometric functions

1 Overviewi) Trigonometry The word lsquotrigonometryrsquo is derived from the Greek words lsquotrigonrsquo and lsquometronrsquo which means measuring the sides of a triangle An angle is the amount of rotation of a revolving line with respect to a fixed line Usually we follow two types of conventions for measuring angles ie a) Sexagesimal system b) Circular system In Sexagesimal system the unit of measurement is Degree In Circular system the unit of measurement is Radian ii) Relation between degree and radianThe ratio of circumference of a circle to its diameter is always a constant This constant ratio is a number denoted by π which is taken approximately as 227The relationship between degree amp radian measurements is as follows2 right angles = 180deg= π radians1radian = 180degπ=57deg16(approx) 1deg=π180 radianiii) Length of an arc of a circleIf an arc of length s subtends an angle θ radians at the center of a circle of radius r then s=rθiv) Area of a sector of a circleA sector is like a pizza slice of the

Q) Express the following angles in radiana) 45deg b) 40deg3730Ans a) We have 180deg=π radiansi e 45deg= πtimes45180 radian = π4 radiansb) 40deg3730= 40deg37+3060 minute= 40deg 37 +12 minute= 40deg+ 752 minute=40 + 75(2times60) degree=3258 degreeNow 180deg=π radianie 3258 degree= (πtimes325) (180times8) radians = 65π288 radiansQ) A circle has a radius of r=12 meters What is the length of an arc traced out by a 60deg angle in the center of the circleAns In this problem we know both the central angle (60deg) and the radius of the circle (12) All we have to do is plug those values into our equation and we get

s = 2π(12)(60360)s = 24π6s = 4πSo the length of an arc traced out by a 60deg angle in a circle with a radius of 12 meters equals 4π meters asymp 1257 metersQ) Find the area of the sector with a central angle 30deg and a radius of 9cmAns GivenRadius r = 9 cmAngle θ = 30degArea of the sector = θ360degtimesπr2

= 30360degtimes227times92=2121cm2

circle It consists of a region bounded by two radii and an arc lying between the radiiThe area of a sector is a fraction of the area of the circle This area is proportional to the central angle In other words the bigger the central angle the larger is the area of the sectorArea of Sector = θ2 times r2 (when θ is in radians)

Area of Sector = θ times π360 times r2 (when θ is in degrees)

COMMERCE

CLASSIFICTION OF HUMAN ACTIVITIES-ECONOMIC AND NON-ECONOMIC

Welcome to the new sessiontoday we are going to start the first chapter of Class XI The name of the chapter that we are going to start is

lsquoClassification of Human Activities ndasheconomic and non-economicrsquo

Now let us start the chapter by considering human beings and the activities they perform throughout the day

Human activities means all those activities that human beings undertake to satisfy their wants

Human wants on the other hand are the desire of human beings for goods (vegetables fruits rice etc) and services (services of doctors teachers lawyers etc) that they require to live

Now these human activities continue throughout life as human wants are unending unlimited and recurring as human beings desire for better living throughout their lives

Now human activities can be classified into two categories

Human activities

Economic activities Non-economic activities

Economic activities are

Questions1 What are human activities

Answer Human activities mean all those activities that human beings undertake to satisfy their wants

Example A man working in an office

A boy playing in the garden

2What are the characteristics of human activitiesAnswer the characteristics of human activities are as follows

Human activities are undertaken by men women and children and these activities involve human efforts

Human activities are undertaken to satisfy human wants which are unlimited

Human activities continue throughout life

Human activities are performed for both earning money and personal satisfaction

3What is economic activitiesGive example

Answer Economic activities are undertaken by human beings with the object of earning money acquiring wealth and thereby satisfying human wantsExample

Selling of goods by a shop keeper to his customer

A clinic run by a doctor Service of a teacher in school or college

undertaken by human beings with the object of earning money and acquiring wealth

These activities result in the production of economic goods and services

Example Human activities(ie working in factories officesshops) which produce direct economic benefits

Non-economic activities are inspired by human sentiments and emotions such as love for the family desire to help the poor and love for the country

Thus these human activities (eg praying playing sleeping) produce no direct economic benefits and they are also not related to earning money and acquiring wealth

4 What are the characteristics of economic activities

Answer The characteristics of economic activities are as follows

Economic motiveEconomic activities are undertaken to earn money and acquire wealth

ProductiveEconomic activities involve productiondistribution and exchange of goods and services to create wealth

Economic growthEconomic activities determine the level of economic development of a country and standard of living of its citizens

Socially desirableEconomic activities are socially desirable for society

Economic resourcesEconomic activities make use of all the economic resources such landlabourcapital etc

5 What do you mean by non-economic activitiesExampleAnswerNon-economic activities are inspired by human sentiments and emotions such as love for the family desire to help the poor and love for the countryThese activities are not undertaken for monetary gain but for onersquos satisfaction and happinessExample

a mother looks after her children

a student donates blood8 Differentiate between Economic activities and Non-economic activities

Economic activities

Non-economic activities

1to earn living and acquiring wealth2Result can be measured in terms of money

3ExampleBusinessprofession and employment

1 to obtain some satisfaction

2Result cannot be measured in terms of money

3ExampleFamily-orientedreligious socialCultural and national

BUSINESS STUDIES

BUSINESS ENVIRONMENT

Welcome to the new sessionToday we are going to start the first chapter and the name of the chapter is Business Environment

In todayrsquos world every business enterprise is a part of the society It exists and operates in association with various groups in society such as customers suppliers competitors banks and financial institutions government agencies trade unions media and so on All these groups influence the functioning of business in one way or the other They constitute the environment of businessConcept of Business Environment

The term lsquobusiness environmentrsquo refers to the sum total of all individuals institutions and other forces that lie outside a business enterprise but that may influence its functioning and performance

The main features of business environment

Totality of External forces General and Specific forces Interrelatedness Complexity Dynamic Uncertainty Relativity

The Interrelation between business and its environment

The business enterprise is an open system It continuously interacts with its environment It takes inputs

Prepare the following questions from todayrsquos assignment

1 What do you mean by business environment

The term lsquobusiness environmentrsquo means the aggregate of all forces factors and institutions which are external to and beyond the control of an individual business enterprise but they may influence its functioning and performance Business environment is the macro framework within which a business firm a micro unit operates It consists of several interrelated and interacting elements

2 Explain the main features of business environment in brief

Totality of External forces-Business environment is the sum total of all things external to a business environment

General and Specific forces-It includes both the forces general forces are the economic social political legal and technological conditions which indirectly influence all business enterprise Specific forces are the investors customers competitors and suppliers which influence individual enterprise directly

Interrelatedness-Different elements of environment are interrelated for an example growing awareness for health care has increased the demand for health foods

Complexity- Business environment id

(such as raw materials capital labour energy and so on) from its environment transforms them into goods and services and sends them back to the environment

Fig 1 Business Environment Relationship

complex in nature as the elements keep on changing example economic technological and other forces changes in demand for a product and service

Dynamic-Business environment is not static it keeps on changing

Uncertainty- Itrsquos very difficult to predict future events such as technology and fashion which occur fast and frequently

Economics Basic Economic ConceptsSub topic

Microeconomics and

Macroeconomics

Welcome to the new sessiontoday we are going to start the first chapter of Class XI The name of the chapter that we are going to start is Basic Economic concepts

Now Economics covers the study of human activities Human activities are those activities which are performed by humans to satisfy their wants

Thus Human wants are unlimited and therefore economic activities such as production exchange and consumption are needed in order to satisfy those wants

The study of economics is divided largely in two parts which areMicroeconomics and Macroeconomics

SUBJECT- MATTER OF ECONOMICS

MICROECONOMICS MACROECONOMICS

Questions1Who has coined the words micro and macro economics

Answer Ranger Frisch coined the words lsquomicrorsquo and lsquomacrorsquo in 1933 to denote the two branches of economic theory namely microeconomics and macroeconomics

2What is microeconomicsAnswer It is the study of behaviour of individual decision ndash making unit such as consumers firms etc

3 What is macroeconomicsAnswer Macroeonomics is the study of overall economic phenomena like employment national income etc

4 What is the importance of microeconomicsAnswer

Microeconomics helps in formulating economic policies which enhance productive efficiency and results in greater social welfare

It helps the government in formulating correct price policies

It explains the working of a capitalistic economy where individual units(producers and consumers ) are free to take their own decision

Micro means a small part in

microeconomics we do not study the whole economy Hence we study an individual consumer and his or her choices and a producer and his or her profit maximizing decisions in the market Thus it does not mirror what happens in the economy as a whole

Macroeconomics on the other hand studies the economy as a whole It is concerned with aggregate and depicts the entire picture of the economyMacroeconomics deals with the national income aggregate investment aggregate consumption etc

Features of Microeconomics It deals with small

parts of the country Hence it looks at

individual consumers firms and industries

It deals with individual income consumption and savings

It studies the determination of price of any product or factors of production

It deals with the working of market via the price mechanism which is nothing but the determination of price and quantity of a commodity by the forces of demand and supply

Features of Macroeconomics

It deals with the study of the economy as a whole

It is concerned with

5 Give a limitation of microeconomics Microeconomics fails to explain the

functioning of an economy as a whole It cannot explain unemployment illiteracy and other problems prevailing in the country

6 What is the importance of macroeconomics It gives overall view of the growing

complexities of an economic system It provides the basic and logical

framework for formulating appropriate macroeconomic policies (eg for inflation poverty etc )to direct and regulate economy towards desirable goals

7What is the limitation of macroeconomics It ignores structural changes in an

individual unit of the aggregate

8 Differentiate between Microeconomics and Macroeconomics

Microeconomics Macroeconomics

the study of aggregates

National income aggregate savings and aggregate investments are major concepts dealt within macroeconomics style

It studies the determination of general price levels

It investigates into the problem of unemployment and the achievement of employment

It studies the aspect of decision making at the aggregate and national levels

It includes all growth theories whether related to developed or developing economies it also includes the study of economic systems and the working of the economy under different systems

Note Both Micro and macro economics are complementary and should be fully utilized for proper understanding of an economy

1It studies economic aspect of an individual unit2It deals with individual incomeConsumption and savings

3 It facilitates determination of price of any product or factors of production

4 Itrsquos scope is narrow and restricted to individual unit

1It studies the economy as a whole

2It deals with the national income aggregate consumption and aggregate savings3 It facilitates determination of general price level in an economy

4 Itrsquos scope is wide as it deals with economic units on the national level

ACCOUNTS

Introduction to Accounting and Book-keeping

Today I am going to share you the meaning of Accounting and Book-keeping and its related terms bullAccounting bullBook Keeping bullAccountsbullTypes Of Accounts bullAccounting Cycle

bull Meaning of accounting

Ans ) Accounting is the art and science of recording classifying and summarising monetary transactions

bull Meaning of Book-keeping

Ans) Bookkeeping is the art of recording business transactions with the view of having a permanent record of them and showing their effect on wealth

bull Meaning of account

Ans) The term account means a record of

business transactions concern a particular person of firm asset or income or expense It is a summarised record of all transactions which take place in an accounting year

bull Types of accountsPersonal accounts ndash Personal accounts relating

to person and Organisation are known as personal accounts Example Ramrsquos Account ABC amp Co Account etc

Real account - The accounts related to tangible and intangible assets are called real accounts Example Cash Account Furniture Account etc

Nominal account- Accounts related to expenses losses incomes and gains are known as nominal accounts Example Wages Account Salary Account Discount Account etc

bull Accounting cycle Accounting cycle refers to a complete sequence of accounting activities It begins with recording of transactions and ends with the preparation of a balance sheet

Chemistry TopicAtomic Structure

Thomsonrsquos atomic modelThomson (1898) was the first to propose the model of an atomHe proposed that an atom can be regarded as a uniform sphere of positive electricity in which requisite number of electrons are embedded evently to neutralize the positive chargeThis is just like plums embedded in a pudding or seeds evently distributed in red spongy mass of a watermelonThis model of atom is known as ldquoPlum-Pudding modelrdquo or

Q1)What is the fundamental constituents of atomAns Electron Proton and neutrons are the fundamental constituents of atomQ2)What is the value of fundamental unit of electricityAnsThe charge carried by one electron is sad to be the fundamental unit of electricityIts magnitude is 48times10-10esuOr 1602times10-19C Q3)Name the element containing no neutronAnsOrdinary hydrogen atom or protium 1H1

Types of AccountPersonal AccountReal AccountNominal AccountBalance Sheet (opening)

ldquowatermelon modelrdquoThis model could explain the electrical neutrality of an atom but failed to explain the result of scattering experiment carried out by Rutherford in 1911So it was rejected ultimately

Q4)Why is an electron called universal particleAns Itrsquos mass and Charge are independent of its source

EVS Chapter 1 ndash Modes of Existence

Modes of existence When one speaks normally about the mode of existence of some group or individual one refers to their customs their mode of being their ethology their habitat in some way their feeling for a placeDifferent modes of exixtence are ndash

1 Hunting ndashGathering2 Pastoral3 Agricultural4 Industrial

1 Hunting and gathering Hunting and gathering mode of existence is characterized by obtaining food from hunting wild animals including fishing and gathering wild plants From their earliest days the hunter-gatherer diet included various grasses tubers fruits seeds and nuts Lacking the means to kill larger animals they procured meat from smaller game or through scavenging

Societies that rely primarily or exclusively on hunting wild animals fishing and gathering wild fruits berries nuts and vegetables to support their diet are called hunting and gathering societies

At least this used to be practice of human beings before agriculture is invented As their brains evolved hominids developed more intricate knowledge of edible plant life and growth cycles

Q) Write the features of Hunting ndash gathering societiesAns - There are five basic characteristics of hunting and gathering societies

i The primary institution is the family which decides how food is to be shared and how children are to be socialized and which provides for the protection of its members

ii They tend to be small with fewer than fifty members

iii They tend to be nomadic moving to new areas when the current food supply in a given area has been exhausted

iv Members display a high level of interdependence

v Labor division is based on sex men hunt and women gather

Political Science

Introduction to political science

Political science occasionally called politology is a social science which deals with systems of governance and the analysis of political activities political thoughts associated constitutions and political behaviorThe study of political science involves the study of both the

Answer the following questions-1 What is political science

Political science occasionally called politology is a social science which deals with systems of governance and the analysis of political activities political thoughts associated constitutions and political behavior

2 Short notes-

traditional and modern theories of politicsTraditionalClassical political sciencepolitical theory-Traditional political science is the study of politics before Second World War The methodology to study Politics was traditional (legal formaletc) the definition of politics traditional (Politics begins and end with state)area of study (constitution state machinery)was traditionalModern Political scienceModern political theory-Modern Political Theory critically examines the contemporary state of political theory making an assessment of the achievement and limitations of the Behavioural Revolution in its totality and reviews objectively the major paradigms and conceptual frameworks adopted by the disciplineContemporary attempts at the development of an integrated political theory involving the use of both traditional and modern concepts approaches and theories-Around late 1960s several political scientists realized the importance of both the traditional political theory and modern Political theory They began building an integrated theory of politics involving a systematic mixture of traditional and modern studies of politics It was held that the study of a complex and vast field like politics needs both traditional as well as

Classical political theory Modern Political theory

Homework-Learn

modern concepts and approaches for studying itrsquos all aspects

Subject Eng Literature (The Tempest ndash William Shakespeare) Topic Act I Scene 1 Lines 1 to 32 (Line 32 ndash Gonzalo hellip If he be not born to be hanged our case is miserable) Date 13th April 2020 (3rd Period)

[Students should read the original play and also the paraphrase given in the school prescribed textbook]Summary Questions amp Answers

[SUMMARY OF THE ENTIRE SCENE]

o The play starts with the scene of a severe storm at sea Alonso (King of Naples) Sebastian (Alonsorsquos brother) Ferdinand (Alonsorsquos son) Gonzalo Antonio (the usurping Duke of Milan) are in a ship in the midst of the storm

o The mariners are trying their best to control the vessel from running aground and are totally following the orders of their Master the Boatswain They have scant success

o The mariners become extremely unhappy and annoyed when most of the passengers arrive on the deck thereby hampering their effort to save the ship There is serious confrontation between them and the passengers who are part of the Kingrsquos entourage

o The mariners could not save the ship

SUMMING-UP

(i) Vivid description of the scene which gives a realistic description of terror and confusion of a tropical storm

(ii) Shows Shakespearersquos accuracy of knowledge in describing the naval operations and also matters of seamanship

(iii) The opening scene justifies the title ndash The Tempest

UNANSWERED QUESTIONS

(i) The King always travels with his entire fleet including his soldiers Where

(1) GONZALO Nay good be patient (Line 15-26)BOATSWAIN When the sea is Hence What cares these

roarers for the name of the king To cabin silence Trouble us not

GONZALO Good yet remember whom thou has aboardBOATSWAIN None that I more love than myself You are a

councillor if you can command these elements to silence and work

the peace of the present we will not hand a rope more use your authority If you cannot give thanks you have

lived so long and make yourself ready in your cabin for the mischance of the hour if it so hap [To the Mariners]

Cheerly good hearts [To Gonzalo] Out of our way I say

(a) To whom is the boatswain speaking What does he mean by lsquoNone that I more love than myselfrsquo

The Boatswain is speaking to Gonzalo the honest old councilor of the Duke of MilanBy using the words ndash lsquoNone that I love more than I love myselfrsquo means that for the Boatswain nobody is dearer to him than his own life

(b) What were the conditions that made the boatswain react in this way

The Boatswain reacts in this way because the storm is at sea and Alonso King of Naples Sebastian his brother Ferdinand his son Gonzalo Antonio the usurping Duke of Milan on board are in distress and in panic Thus they have rushed to the deck interrupting the work of the mariners

(c) What hope does Gonzalo take from the attitude of the boatswain

The insolent and authoritative attitude of Boatswain makes Gonzalo feel comforted He tells that there are no signs that the Boatswain will be drowned But his facial appearance and attitude shows that he is destined to die on land by hanging which in effect means that all on board will be saved Otherwise all the persons on board are doomed

(d) How can they lsquomake yourself ready in your cabinrsquo For what were they asked to make ready themselves

In order to make themselves ready in their cabin the

were the other ships

(ii) Why was the ship in that area Where was it coming from or going where

(iii) The ship broke apart What happened to those who were in the ship

passengers on board must prepare for death which they will possibly soon have to meetThey can retire to their cabins and offer prayers to the Almighty to save them from drowning

(e) What does the boatswain say when he is asked to be patient What does he order to the royal party

When the boatswain is asked to be patient and remain calm he says that he will be patient only when the storm will be over and the sea will be calm but as long as the storm blows and there is danger to the ship he cannot think of being patient He orders the royal party to go to the cabin and leave the mariners to their work

(2) GONZALO I have great comfort from this fellow (Line 27-36)

Methinks he hath no drowning mark upon him his complexion is perfect

gallows Stand fast good Fate to his hanging Make the rope of his destiny our cable for our own doth little advantage If he be not born to be hanged our case is miserable

(a) Why does Gonzalo regard the Boatswain in the midst of danger

In the midst of danger Gonzalo regards the boatswain because he feels that the Boatswain is a source of comfort and is bent upon to do his work sincerely which in this case is saving the ship and its passengers from the severest of raging storm

(b) What reasons does Gonzalo give when he says that none in the ship will die of drowning

Gonzalo is almost sure that none in the ship will die by drowning His says that there is no mark on the face of the boatswain that indicates that he will die by drowning On the other hand the lines on his face are strong indications that he will be hanged to death Therefore there shall be no danger of the shiprsquos sinking

(c) Explain the following ldquoStand fast good Fate to his hanging Make the rope of his destiny our cable for our own doth little advantage If he be not born to be hanged our case is miserablerdquo

The stated lines mean that if the will of destiny is to be carried out then the ship will not get wrecked and all the passengers will be saved The safety of the passengers therefore depends upon the will of fate being carried out in the case of the boatswain If however the boatswain is not to die by hanging then the passengers are also very unsafe because in that case the ship is likely to sink

(d) What order does the Boatswain give to the sailors

when he re-enters What does he say about the crying of the fellows inside the cabin

The boatswain orders the sailors to bring the topmast lower and bring the ship close to a stationary position with the help of the main sail He says that the fellows inside the cabin are moaning and crying in their distress louder than his voice and louder even than the roaring of the storm

Class XII (ScienceCommerceHumanities) Subject Topic Summary Execution

Computer Science

PropositionalLogic

Propositional logic is a procedure to provide reasoning through statementProposition A ststement that results in True or False is said to be proposition There are two types of propositionSimple proposition amp compound propositionSimple proposioton A simple proposition is one that is not a part of any other proposition Such sentential form of proposition is symbolized with english letters in short For example Ram is a claver student (TrueFalse)Where do you live (Not in True or False)Grapes are sweet (TrueFalse)It rains today (TrueFalse)Here we can see some statements anwer would be true or false but some staements answer can not give in terms of true or false Thus the sentences which can be answered in true or false are known as simple propositionAssigning propositon to a variableThe general syntax to assign propostion to a variable is as followsVariable = Simple propositonFor example A=Ram is a clever studentB= Grapes are sweetC= it rains todayCompound proposition

helliphellipto be continued in next classhelliphellipMath Relation Relation If A and B are two non-empty sets

then a relation R from A to B is a subset of AxB If R A x B and (a b) R then we say that a sube isinis related to b by the relation R written as aRbeg Let A be the set of students of class XII and B be the set of students of class XI Then some of the examples of relation from A to B arei) (a b) AXB a is brother of bisinii) (a b) AXB age of a is more than age of isinb Types of relation In this section we would like to study different types of relations We know that a relation in a set A is a subset of A times A Thus the empty set φ and A times A are two extreme relations For illustration consider a relation R in the set A = 1 2 3 4 given by R = (a b) a ndash b = 10 This is the empty set as no pair (a b) satisfies the condition a ndash b = 10 Similarly R = (a b) | a ndash b | ge 0 is the whole primeset A times A as all pairs (a b) in A times A satisfy | a ndash

Example 1 Let A be the set of all students of a boys school Show that the relation R in A given by R = (a b) a is sister of b is the empty relation and R = (a b) the primedifference between heights of a and b is less than 3 meters is the universal relationSolution Since the school is boys school no student of the school can be sister of any student of the school Hence R = φ showing that R is the empty relation It is also obvious that the difference between heights of any two students of the school has to be less than 3 meters This shows that R = A times A is primethe universal relation Example 2 Show that the relation R in the set 1 2 3 given by R = (1 1) (2 2) (3 3) (1 2) (2 3) is reflexive

b | ge 0 These two extreme examples lead us to the following definitionsDefinition 1 A relation R in a set A is called empty relation if no element of A isrelated to any element of A ie R = φ A times AsubDefinition 2 A relation R in a set A is called universal relation if each element of A is related to every element of A ie R = A times A Both the empty relation and the universal relation are some times called trivial relation Definition 3 A relation R in a set A is called(i) reflexive if (a a) R for every a Aisin isin(ii) symmetric if (a1 a2) R implies that (aisin 2a1)

R for all aisin 1 a2 Aisin(iii) transitive if (a1 a2) R and (aisin 2 a3) R isinimplies that (a1 a3) R for all aisin 1 a2 a3 AisinDefinition 4 A relation R in a set A is said to be an equivalence relation if R is reflexive symmetric and transitive

but neither symmetric nor transitiveSolution R is reflexive since (1 1) (2 2) and (3 3) lie in R Also R is not symmetric as (1 2) R but (2 1) isin notinR Similarly R is not transitive as (1 2) R and (2 3) R but (1 3) R isin isin notinExample 3 Show that the relation R in the set Z of integers given byR = (a b) 2 divides a ndash b is an equivalence relationSolution R is reflexive as 2 divides (a ndash a) for all a Z isinFurther if (a b) R then 2 divides a isinndash b Therefore 2 divides b ndash a Hence (b a) R which shows that R is isinsymmetric Similarly if (a b) R and (b c) R isin isinthen a ndash b and b ndash c are divisible by 2 Now a ndash c = (a ndash b) + (b ndash c) is even (Why) So (a ndash c) is divisible by 2 This shows that R is transitive Thus R is an equivalence relation in ZExample 4 Let L be the set of all lines in a plane and R be the relation in L defined as R = (L1 L2) L1 is perpendicular to L2 Show that R is symmetric but neither reflexive nor transitiveSolution R is not reflexive as a line L1 can not be perpendicular to itself ie (L1 L1) R notinR is symmetric as (L1 L2) Risin

L1 is perpendicular to L2rArr L2 is perpendicular to L1rArr (L2 L1) RrArr isin

R is not transitive Indeed if L1 is perpendicular to L2 and L2 is perpendicular to L3 then L1 can never be perpendicular to L3 In fact L1 is parallel to L3 ie (L1 L2) R isin(L2 L3) R but (L1 L3) Risin notin

Chemistry Solid state Characteristics if Solids(i)The particles are locked in fixed positions they are unable to change their relative positions and this brings a definite shape and volume of a solid(ii)In a solid the constituent particles are held by strong forces of attractionThe forces of attraction may be bonding or non bonding(iii)The constituent particles in a solid pack together as closely as possibleoccupying most of the available space within the solidThus the empty space in a solid is very smallThis makes a solid highly rigid and nearly incompressibleThis also explains why a solid has high density and exhibits slow diffusionClassification of Solids

Q1)Define Crystalline solids AnsA Solid that has a definite geometrical shape and a sharp melting pointand whose constituent particles (atomsmolecules or ions) are arranged in a long range order of definite pattern extending throughout the solid is called a crystalline solidExNaClQ2)Define Amorphous solids AnsA solid that does not have a definite shape and a sharp melting pointand whose constituent particles (atomsmolecules or ions) are not arranged in a definite pattern is called an amorphoussolid

Crystalline solidsAmorphous solids

ExGlassRubberQ3)Classify Crystalline Solids Crystalline Solids

Physics Coloumbrsquos Law (Summary)

Before Going Into Coloumbrsquos Law We Will First Learn What is Charge Properties of Charge and Always remember that charge is quantized ie a body always have static charge of magnitude equal to some integral multiple of fundamental electronic charge e= 16 x 10- 19 C

Charge is the property of matter that causes it to produce and experience electrical and magnetic effects The study of the electrical charges at rest is called electrostatics When both electrical and magnetic effects are present the interaction between charges is referred to as electromagnetic

There exist two types of charges in nature positive and negative Like charges repel and unlike charges attract each other

The type of charge on an electron is negative The charge of a proton is the same as that of an electron but with a positive sign In an atom the number of electrons and the number of protons are equal The atom is therefore electrically neutral If one or more electrons are added to it it becomes negatively charged and is designated as negative ion However if one or more electrons are removed from an atom it becomes positively charged and is called a positive ion

The excess or deficiency of electrons in a body gives the concept of charge If there is an excess of electrons in a body it is negatively charged And if there is deficiency of electrons the body becomes positively charged Whenever addition or removal of electrons takes places the body acquires a charge

The SI Unit of charge is coulomb (C) In SI units the current is a fundamental quantity having a unit of ampere (A) The unit of charge is defined in terms of the unit of current Thus one coulomb is the charge transferred in one second across the section of a wire carrying a

Ionic SolidsMetallicSolids

Molecular Solids

current of one ampere

As q = It we have1 C = (1 A) (1 s)

The dimensions of charge are [A T]

Properties of Charge

(1) Quantization of Charge Electric charge can have only discrete values rather than any value That is charge is quantized The smallest discrete value of charge that can exist in nature is the charge on an electron given as

e = plusmn 16 x 10- 19 C

This is the charge attained by an electron and a protonA charge q must be an integral multiple of this basic unit That is

Q = plusmn ne where n = 1 2 hellip

Charge on a body can never be (frac12)e (23)e or 57e etcWhen we rub a glass rod with silk some electrons are transferred from the rod to the silk The rod becomes positively charged The silk becomes negatively charged The coulomb is a very large amount of charge A typical charge acquired by a rubbed body is 10 - 8 C

Biology Reproduction in organisms

Welcome to this new session 2020-21Today in this first chapter we mainly discuss about reproduction types needs and life span of some organismsWe also discuss about difference between sexual and asexual reproduction

Q1 What is reproductionReproduction is defined as a biological processin which an organism gives rise to young onessimilar to itselfQ2 What are the needs of reproductionbulli) Reproduction maintain life on earthii) It enables the continuity of the species generation after generationiii) It creates genetic variation among populationsQ3 Define Life span and write some orgnisms life spanbull Life span is the period from birth to

the natural death of an organism- OrganismsLife span1 Butterfly 1 - 2 weeks2 Fruit fly 30 days3Dog 10-13 years4 Rose5-7 years5 Tortoise100-150 years6 Banyan Tree -200 - 250 yearsQ4 Reproduction is of two types in case ofanimals but in case of plants vegetative propagation is also present

Asexual Reproduction Sexual Reproductioni) Always uniparentalii) Gametes are not involvediii) Only mitotic division involvediv) Somatic cells of parents are involvedv) Offsprings are genetically similar to the parents

i) Usually biparentalii) Gametes are involvediii) Meiosis occurs during gametogenesis Mitosis occurs after fertilisationiv) Germ cells of the parents are involvedv) offsprings are genetically different from the parents

COMMERCE BUSINESS ENVIRONMENT

Welcome to the new sessiontoday we are going to start the first chapter of Class XII The name of the chapter is Business Environment

Already many of you have got some idea about the word business environment form the first chapter of business studies in class XI

In todayrsquos world every business enterprise is a part of the society It exists and operates in association with various groups in society such as customers suppliers competitors banks and financial institutions government agencies trade unions media and so on All these groups influence the functioning of business in one way or the other They constitute the environment of businessConcept of Business Environment

The term lsquobusiness environmentrsquo refers to the sum total of all individuals institutions and other forces that lie outside a business enterprise but that may influence its functioning and performance

The main features of business environment Totality of External forces General and Specific forces Interrelatedness Complexity Dynamic Uncertainty

Prepare the following questions from todayrsquos assignment

2 What do you mean by business environment

The term lsquobusiness environmentrsquo means the aggregate of all forces factors and institutions which are external to and beyond the control of an individual business enterprise but they may influence its functioning and performance Business environment is the macro framework within which a business firm a micro unit operates It consists of several interrelated and interacting elements

2 Explain the main features of business environment in brief

Totality of External forces-Business environment is the sum total of all things external to a business environment

General and Specific forces-It

Relativity

The Interrelation between business and its environment

The business enterprise is an open system It continuously interacts with its environment It takes inputs (such as raw materials capital labour energy and so on) from its environment transforms them into goods and services and sends them back to the environment

Fig 1 Business Environment Relationship

includes both the forces general forces are the economic social political legal and technological conditions which indirectly influence all business enterprise Specific forces are the investors customers competitors and suppliers which influence individual enterprise directly

Interrelatedness-Different elements of environment are interrelated for an example growing awareness for health care has increased the demand for health foods

Complexity- Business environment id complex in nature as the elements keep on changing example economic technological and other forces changes in demand for a product and service

Dynamic-Business environment is not static it keeps on changing

Uncertainty- Itrsquos very difficult to predict future events such as technology and fashion which occur fast and frequently

Business Studies

Human Resources Management

Human resource of an organisation are the aggregate of knowledge skills attitudes of people working in it

The management system which deals with human resources is called human resource management

Features of HRMbullComprehensive functionbullPeople-oriented

Question1) What do you mean by human

resource management Answer) Human resource management may be defined as that field of Management which has to do with planning organising and controlling the functions of procuring developing maintaining and utilising the labour force

bullAction oriented bullPervasive function bullContinuous function

2) Explain the features of HRM in brief

Answer)bullHuman Resource Management is concerned with managing people at work bull Human Resource Management is concerned with employees which bring people and organisations together so that the goals of each are met bullHuman resource management considered every employees as an individual and also promote their satisfaction and growth bull Human resource management is inherent in all organisations and at all levelsbullManagement of human resources are ongoing on never ending process which requires a constant alertness and Awareness of human relations

3) ldquoHR function is said to be pervasiverdquowhy

Answer) Human resource management is required in all organisations whether it is private or government organisations armed forces sports organisations etc It permeatsall the functional areas like production marketing finance research etc This from this feature of human resource management it can be said that it is pervasive in nature

Economics Demand Q1DEFINITION OF DEMANDIn economics demand is the quantity of a good that consumers are willing and able to purchase at various prices during a given period of timeQ2DEMAND CURVEIn economics a demand curve is a graph depicting the relationship between the price of a certain commodity and the quantity of that commodity that is demanded at that pricQ3LAW OF DEMANDIn microeconomics the law of demand states that conditional on all else being equal as the price of a good increases quantity demanded decreases conversely as the price of a good decreases quantity demanded increasesQ4ASSUMPTION of LAW OF DEMAND(i)No change in price of related commodities(ii) No change in income of the consumer(iii) No change in taste and preferences customs habit and fashion of the consumer( No expectation regarding future change in priceQ5MARKET DEMAND SCHEDULEIn economics a market demand schedule is a tabulation of the quantity of a good that all consumers in a market will purchase at a

given price At any given price the corresponding value on the demand schedule is the sum of all consumersrsquo quantities demanded at that priceQ6INDIVIDUAL DEMAND SCHEDULEIndividual demand schedule refers to a tabular statement showing various quantities of a commodity that a consumer is willing to buy at various levels of price during a given period of timeQ7 FACTORS AFFECTING INDIVIDUAL DEMAND FOR A COMMODITY

The factors that influence a consumerrsquos decision to purchase a commodity are also known as determinants of demand The following factors affect the individual demand for a commodity1 price of the commodity2 price of related goods3 income of buyer of the commodity4 tastes and preferences of the buyer1 Price of the CommodityYou must have observed that when price of a commodity falls you tend to buy more of it and when its price rises you tend to buy less of it when all other factors remain constant (lsquoother things remaining the samersquo) In other words other things remaining the same there is an inverse relationship between the price of a commodity and its quantity demanded by its buyers This statement is in accordance with law of demand which you will study in the later part of this lesson Price of a commodity and its quantity demanded by its buyers are inversely related only when lsquoother things remain the samersquo So lsquoother things remaining the samersquo is an assumption when we study the effect of changes in the price of a commodity on its quantity demanded2 Price of Related goodsA consumer may demand a particular good But while buying that good heshe also asks the price of its related goods Related goods can be of two types-(i) Substitute goods(ii) Complementary goods While purchasing a good prices of its substitutes and complements do affect its quantity purchased(i) Price of Substitute Goods Substitute goods are those goods which can easily be used in place of one another for satisfaction of a particular want like tea and coffee An increase in price of substitute good leads to an increase in demand for the given commodity and a decrease in price of substitute good leads to a decrease in demand for the given commodity It means demand for a given commodity is directly affected by change in price of substitute goods For example if price of coffee increases the demand for tea will rise as tea will become relatively cheaper in comparison to coffee(ii) Price of Complementary goods Complementary goods are those goods which are used together to satisfy a particular want like car and petrol An increase in the price of complementary goods leads to a decrease in demand for the given commodity and a decrease in the price of complementary goods leads to an increase in demand for the given commodity For example if price of petrol falls then the demand for cars will increase as it will be relatively cheaper to use both the goods together So demand for a given commodity is inversely affected by change in price of complementary goods3 Income of the Buyer of CommodityDemand for a commodity is also affected by income of its buyer However the effect of change in income on demand depends on the nature of the commodity under consideration In case of some goods like full cream milk fine quality of rice (Basmati rice) etc demand for these commodities increases when income of the buyer increases and

demand for these commodities decreases when income of the buyer decreases Such goods whose demand increases with the increase in income of the buyer are called normal goods But there are some goods like coarse rice toned milk etc whose demand decreases when income of buyer increases and their demand increases when income of the buyer decreases Such goods whose demand decreases with the increase in income of the buyer are called inferior goods Suppose a consumer buys 10 Kgs of rice whose price is ` 25 per Kg He cannot afford to buy better quality of rice because the price of such rice is ` 50 per Kg The consumer is spending ` 250 per month on the purchase of rice Now if income of the consumer increases and he can afford ` 350 on purchase of 10 Kg of rice Now he can afford to buy some quantity of rice say 6 Kgs whose price is ` 25 per Kg and may buy 4 Kgs of rice whose price is ` 50 per Kg Thus he will buy 10 Kgs of rice by spending ` 350 per month Therefore we may conclude that demand for normal goods is directly related to the income of the buyer but demand for inferior goods is inversely related to the income of the buyer4 Tastes and Preferences of the BuyerThe demand for a commodity is also affected by the tastes and preferences of the buyers They include change in fashion customs habits etc Those commodities are preferred by the consumers which are in fashion So demand for those commodities rises which are in fashion On the other hand if a commodity goes out of the fashion its demand falls because no consumer will like to buy it(5) Number of Buyers in the Market(Population)Increase in population raises the market demand whereas decrease in population reduces the market demand for a commodity Not only the size of population but its composition like age (ratio of males females children and old people in population) also affects the demand for a commodity It is because of needs of children young old male and female population differs(6) Distribution of Income and WealthIf the distribution of income and wealth is more in favour of the rich demand for the commodities preferred by the rich such as comforts and luxuries is likely to be higher On the other hand if the distribution of income and wealth is more in favour of poor demand for commodities preferred by the poor such as necessities will be more(7) Season and Weather ConditionsThis is generally observed that the demand for woolens increases during winter whereas demand for ice creams and cold drinks increases during summer Similarly market demand for umbrellas rain coats increases during rainy seasonQ8 REASONS FOR OPERATION OF LAW OF DEMAND WHY DEMAND CURVE SLOPES DOWNWARDNow we will try to explain why does a consumer purchase more quantity of a commodity at a lower price and less of it at a higher price or why does the law of demand operate ie why does the demand curve slope downwards from left to right The main reasons for operation of law of demand are1 Law of Diminishing Marginal UtilityAs you have studied earlier law of diminishing marginal utility states that as we consume more and more units of a commodity the utility derived from each successive unit goes on decreasing The consumer will be ready to pay more for those units which provide him more utility and less for those which provide him less utility It implies that he will purchase more only when the price of the commodity falls2 Income Effect

When price of a commodity falls purchasing power or real income of the consumer increases which enables him to purchase more quantity of the commodity with the same money income Let us take an example Suppose you buy 4 ice creams when price of each ice cream is ` 25 If price of ice creams falls to ` 20 then with same money income you can buy 5 ice creams now3 Substitution EffectWhen price of a commodity falls it becomes comparatively cheaper as compared to its substitutes (although price of substitutes has not been changed) This will lead to rise in demand for the given commodity For example if coke and Pepsi both are sold at ` 10 each and price of coke falls Now coke has become relatively cheaper and will be substituted for Pepsi It will lead to rise in demand for coke4 Change in Number of BuyersWhen price of a commodity falls some old buyers may demand more of the commodity at the reduced price and some new buyers may also start buying this commodity who were not in a position to buy it earlier due to higher price This will lead to increase in number of buyers when price of the commodity falls As a result demand for the commodity rises when its price falls5 Diverse Uses of a CommoditySome commodities have diverse uses like milk It can be used for drinking for sweet preparation for ice cream preparation etc If price of milk rises its use may be restricted to important purpose only This will lead to reduction in demand for other less important uses When price of milk falls it can be put to other uses also leading to rise n demand for itQ9 EXCEPTIONS TO THE LAW OF DEMANDYou have studied in law of demand that a buyer is willing to buy more quantity of a commodity at a lower price and less of it at a higher price But in certain circumstances a rise in price may lead to rise in demand These circumstances are called Exceptions to the Law of Demand Some important exceptions are1 Giffen GoodsGiffen goods are special type of inferior goods in which negative income effect is stronger than negative substitution effect Giffen goods do not follow law of demand as their demand rises when their price rises Examples of Giffen goods are jowar and bajra etc2 Status Symbol GoodsSome goods are used by rich people as status symbols eg diamonds gold jewellary etc The higher the price the higher will be the demand for these goods When price of such goods falls these goods are no longer looked at as status symbol goods and tehrefore therir demand falls3 NecessitiesCommodities such as medicines salt wheat etc do not follow law of demandbecause we have to purchase them in minimum required quantity whatever their price may be4 Goods Expected to be ScarceWhen the buyers expect a scarcity of a particular good in near future they start buying more and more of that good even if their prices are rising For example during war famines etc people tend to buy more of some goods even at higher prices due to fear of their scarcity in near future

Political Science

Constitution of India-The

Preamble

The preamble-

Preamble-

The preamble is the most precious part of the constitution We the people of India having solemnly resolved to constitute India into a Sovereign Socialist Secular Democratic Republic and to secure to all its citizensA preamble is an introductory and expressionary statement in a document that explains the documents purpose and underlying philosophy When applied to the opening paragraphs of a statute it may recite historical facts pertinent to the subject of the statuteNature and purpose of the constitution-Purpose of the Constitution dictates permanent framework of the government to form a more perfect union to establish justice and ensure peace of thenationconstitution provide principles how the government can run itself following the rules and laws written in the constitution of each state keeps them balanced

Answer the following questions-

1 What is preambleA preamble is an introductory and expressionary statement in a document that explains the documents purpose and underlying philosophy2 What is the nature and

purpose of the constitutionConstitution dictatespermanent framework of the government to form a more perfect union to establish justice and ensure peace of the nation

Homework-Learn

Accounts Compatibilty mode

1MEANING OF PARTNERSHIPPartnership is a form of business organisation where two or more persons join hands to run a business They share the profits and losses according to the agreement amongst them According to the Indian Partnership Act 1932 ldquoPartnership is relation between persons who have agreed to share profits of a business carried on by all or any one of them acting for allrdquo For example one of your friends has passed class XII from National Institute of Open Schooling (NIOS) and wants to start a business Heshe approaches you to join in this venture Heshe wants you to contribute some money and participate in the business activities Both of you if join hands constitute a partnership2CHARACTERISTICS1048698 Agreement A partnership is formed by an agreement The agreement may be either oral or in writing It defines the relationship between the persons who agree to carry on business It may contain the terms of sharing profit and the capital to be invested by each partner etc The written agreement is known as partnership deed1048698 Number of persons There must be at least two persons to form a partnership

The maximum number of partners in a partnership firm can be 50 according toCompanies Act 20131048698 Business The Partnership is formed to carry on business with a purpose of earning profits The business should be lawful Thus if two or more persons agree to carry on unlawful activities it will not be termed as partnership1048698 Sharing Profits The partners agree to share profits in the agreed ratio In caseof loss all the partners have to bear it in the same agreed profit sharing ratio10486981048698Mutual Agency Every partner is an agent of the other partners Every partner can bind the firm and all other partners by hisher acts Each partner will be responsible and liable for the acts of all other partners10486981048698Unlimited liability The liability of each partner except that of a minor is unlimited Their liability extends to their personal assets also If the assets of the firm are insufficient to pay off its debts the partnersrsquo personal property can be used to satisfy the claim of the creditors of the partnership firm10486981048698Management All the partners have a right to mange the business However they may authorize one or more partners to manage the affairs of the business on their behalf10486981048698Transferability of Share No partner can transfer hisher share to any one including hisher family member without the consent of all other partners3PARTNERSHIP DEEDAgreement forms the basis of partnership The written form of the agreement is which a document of partnership is It contains terms and conditions regarding the conduct of the business It also explains relationship between the partners This document is called partnership deed Every firm can frame its own partnership deed in which the rights duties and liabilities of the partners are stated in detail It helps in settling the disputes arising among the partners during the general conduct of business 4CONTENTS OF PARTNERSHIP DEEDThe partnership deed generally contains the following (i) Name and address of the partnership firm(ii) Nature and objectives of the business(iii) Name and address of each partner(iv) Ratio in which profits is to be shared(v) Capital contribution by each partner(vi) Rate of Interest on capital if allowed(vii) Salary or any other remuneration to partners if allowed(viii) Rate of interest on loans and advances by a partner to the firm(ix) Drawings of partners and interest thereon if any(x) Method of valuation of goodwill and revaluation of assets and liabilities on the reconstitution of the partnership ie on the admission retirement or death of a partner(xi) Settlement of disputes by arbitration(xii) Settlement of accounts at the time of retirement or death of a partner5IN ABSENCE OF PARTNERSHIP DEEDThe partnership deed lays down the terms and conditions of partnership in regard to rights duties and obligations of the partners In the absence of partnership deed there may arise a controversy on certain issues like profit sharing ratio interest on

capital interest on drawings interest on loan and salary of the partners In such cases the provisions of the Indian Partnership Act becomes applicableSome of the Issues are(i) Distribution of Profit Partners are entitled to share profits equally(ii) Interest on Capital Interest on capital is not allowed(iii) Interest on Drawings No interest on drawing of the partners is to be charged(iv) Interest on Partnerrsquos Loan A Partner is allowed interest 6 per annum on the amount of loan given to the firm by himher(v) Salary and Commission to Partner A partner is not entitled to anysalary or commission or any other remuneration for managing the business

History TOPIC-TOWARDS INDEPENDENCE AND PARTITION THE LAST PHASE (1935-1947)

SUB TOPIC-IMPORTANT POLITICAL DEVELOPMENTS ndash GROWTH OF SOCIAL IDEAS

Socialism is a political social and economic philosophyLike in other parts of the world the Russian revolution of 1917 served as a great inspiration for revolutionaries in India who at that time were engaged in the struggle for liberation from British ruleSocialist ideas led to the formation of communist party of IndiaJAWAHARLAL NEHRU Among the early Congress leaders Jawaharlal Nehru was very much impressed and influenced by the Socialist ideas He also learnt about the Economic activities of the Soviet Union after the Bolshevic Revolution 1917 He made full use of them in IndiaThe election of Jawaharlal Nehru and Subhas Chandra Bose showed the Left wing tendency within CongressJawaharlal Nehru demanded economic freedom along with political freedom of the people in order to end the exploitation of masses

Nehrus working committee included three socialists leaders The Lucknow session was a landmark in the evolution of socialist ideas of the congressSUBHAS CHANDRA BOSE ndash Subhas Chandra Bose had socialist leaning Both Jawaharlal Nehru and Subhas Chandra Bose were known as leftist Congress men Later on National Congress divided into Leftist and rightist campCONGRESS SOCIALIST Within the Congress some leaders formed the Congress Socialist partyPattavi Sitaramyya Sardar Patel Rajendra Prasad had hostile attitude towards the Congress Socialist partyJawaharlals attitude was hesitant

1 QUESTION ndash Mention name of two Congress leaders who had socialist leaning

1ANSWER ndash Subhas Chandra Bose and Jawaharlal Nehru2QUESTION- In which session of the congress Jawaharlal elaborated his Socialist ideas2 ANSWER ndash Lucknow and Faizpur Session in December 1935 and 19363QUESTION ndash Why Congress was sharply divided into leftist and rightist camp 3ANSWER ndash Subhas Chandra Bosersquos attempt to seek re election for congress presidentship in 1939sharply divided the National Congress into Leftist and Rightist camp4 QUESTION ndash Who was MN Roy 4 ANSWER ndash Manabendra Roy first formed the Communist Party of India outside the country at Tashkent in 19205QUESTION ndash Who formed the Congress Socialist Party within the Congress5 ANSWER ndash Jaya Prakash Narayan Achyut Patwardhan Acharya Narendra Dev Ram Mohan Lohia Aruna Asaf Ali6QUESTION ndash When was the Congress Socialist Party formed What was its object6 ANSWER ndash 1934The Congress Socialist Party sought to work out socialist programme through the Congress They joined hands with the Congress and wanted to carry

Subhas Chandra Bose being expelled from the congress after the Tripuri rift he formed Forward BlockThere were basic differences between the Congress Socialists and the communistsTRADE UNION ACTIVITIES Maximum working class people lived in Bombay and Calcutta The working and living conditions of those workers were very miserable In this situation Shasipada Banerjee NM Lokhande protested against the oppression of the working class peopleThe first Trade Union Madras Labour Union was formed in 1918 by BP WadiaIndustrial strikes took place in Kanpur Calcutta Madras Jamshedpur and Ahmedabad AITUC was formed in Bombay in 1927 The growth of Trade union among the workers was slow because of the fear of the dismissal of the jobIn the mean time the Moderates as well as Communists left AITUC and formed separate organization

on National struggle with the help of workers and peasant class of the society7 QUESTION ndash What was the name of the party founded by Subhas Chandra Bose7 ANSWER- Forward Block8QUESTION ndash Who was Shasipada Banerjee8 ANSWER ndash Shasipada Banerjee was a radical Brahmo He founded a working menrsquos club to protest against exploitation of the British rulers towards the working class of India9 QUESTION ndash What was the weekly published by NM Lokhande9ANSWER- Dinabandhu10 QUESTION ndash Who founded Bombay Mill-Hands Association and in which year10 ANSWER- NM Lokhande in189011 QUESTION- Who was BP WadiaANSWER- BPWadia was the founder of Madras Labour Union in191812 QUESTION- What was the name of the first labour union of India12 ANSWER- Madras Labour Union13 QUESTION Who founded the Majur Mahajan 13 ANSWER GANDHIJI14 QUESTION What was the full form of AITUC When it was formed14 ANSWER All India Trade Union Congressin 192715QUESTION Who formed the Red Trade Union Congress and in which year15ANSWER The Communists formed the Red Trade Union Congress16 QUESTION What do you mean by Socialism16 ANSWER Socialism describes any political and economic theory that says the community rather than individuals should own and manage property and natural resources

Subject Eng Literature (The Tempest ndash William Shakespeare) Topic Act III Scene 3 Lines 1 to 52 (Line 52 ndash Brother my lord the Duke Stand to and do as we) Date 13th April 2020 (4th Period)

[Students should read the original play and also the paraphrase given in the school prescribed textbook]Summary Questions amp Answers

o Alonso Sebastian Antonio Gonzalo Adrian Francisco and others wandered about the island in search of Ferdinand and gets tired and hungry of the toil and at the same time gives up all hope of finding him

o Antonio and Sebastian are happy that Alonso is out of hope and decide to make another attempt on his life that night when being so tired they will be sleeping soundly

o Suddenly a solemn and strange music is heard in the air and several strange shapes enter bringing in a banquet These strange shapes then dance round it with gestures of salutation and then inviting the King to eat they depart

o Seeing this strange scene all are inclined to believe the tales told by travelers that there truly are ldquounicornsrdquo and ldquothe phoenixrsquo thronerdquo

1 ALONSO What harmony is this My good friends hark (L18-27)

GONZALO Marvellous sweet music

[Enter several strange shapes bringing in a banquet

they dance about it with gentle actions of salutation

and inviting the King and his companions to eat they depart]ALONSO Give us kind keepers heavens What were theseSEBASTIAN A living drollery Now I will believe

That there are unicorns that in Arabia

There is one tree the phoenixrsquo throne one phoenix

At this hour reigning thereANTONIO Ill believe both

And what does else want credit come to me

And Ill be sworn rsquotis true Travellers neer did lie

Though fools at home condemn rsquoem

(a) How did Prospero present an amazing spectacle before Alonso and his companions

Using his magic powers Prospero ordered strange shapes to lay a banquet before Alonso and his companions The shapes brought several dishes with tasty eatables in them They placed the dishes on a table before Alonso and his companions Then the strange shapes began to dance gracefully around the banquet While dancing they made gestures inviting them to eat the food Then suddenly the shapes disappeared(b) Who were the guests at the strange banquet Describe the lsquoliving drolleryrsquo

Alonso Sebastian Antonio Gonzalo Adrian and Francisco were the guests at the strange banquet

The term ldquoliving drolleryrdquo refers to live entertainment show In this context when Alonso the King of Naples Sebastian his brother Antonio the treacherous brother of Prospero Gonzalo the kind and loyal councillor to the King Adrian and Francisco came to the island they were hungry and weary in their spirits They heard a solemn and strange music They were shocked to see several strange shapes bringing in a banquet and these shapes danced about it with gentle action of salutation inviting the King and his companions to eat After this Sebastian described this show as lsquoliving drolleryrsquo(c) What is lsquophoenixrsquo What are lsquoUnicornsrdquo

The term lsquophoenixrsquo refers to a mythical Arabian bird which lived alone and perched on a solitary tree After one hundred years it expired in flames and rose again from its own ashes

lsquoUnicornsrsquo refers to the mythological four-footed beasts having horns in the centre of their foreheads When the horns are ground into powder the powder was believed to be

an aphrodisiac(d) How does Sebastian explain the puppet show OR Why does the speaker now believe in unicorns and phoenix

Sebastian finds several strange shapes bringing in the banquet They invite the king and his party for dinner and soon depart He tells that if such a strange sight can be a reality there is nothing incredible in the world and from the present moment he will believe anything He says that it is a strange dumb show enacted not by puppets but by living beings It is stranger than a travellerrsquos tale Seeing such a thing

before his own eyes he will no longer disbelieve the story about unicorns and phoenix(e) How do the other characters present respond to this living drollery

At the sight of the lsquoliving drolleryrsquo like Sebastian Gonzalo and Antonio too acted strangely Antonio told that he too now believes in unicorns and phoenix and anything else that seems to be incredible He too now believes in travellersrsquo tales Gonzalo told that if he would report those happenings in Naples nobody will believe him He considers that those gentle shapes were gentler in manner in comparison to the living beings Alonso was at first sight suspicious and told them that those strange shapes conveyed their meaning in expressive gestures when they seemed to lack speech by their movements and sounds Francisco was amazed at their mysterious disappearance

2 ALONSO Not I

(Line 43-52)GONZALO Faith sir you need not fear When we

were boysWho would believe that there were mountaineers

Dewlapped like bulls whose throats had hanging at rsquoem

Wallets of flesh Or that there were such men

Whose heads stood in their breasts Which now we find

Each putter-out of five for one will bring us

Good warrant ofALONSO I will stand to and feed

Although my lastmdashno matter since I feel

The best is past Brother my lord the Duke

Stand to and do as we

(a) How does Alonso respond at the spectacle of the shapes which were sent to them at the instruction of Prospero

After seeing the strange sight of appearing and disappearing of the shapes sent by Prospero to arrange a banquet for them Alonso says that his surprise at having seen those creatures is infinite and he is fully justified in feeling so much surprise He thinks that their shapes their gestures and the sounds they made were indeed amazing Although they do not possess the gift of speech yet they were able to convey their

thoughts by means of their gestures only

(b) What does Prospero say about the views expressed by Alonso regarding the shapes What does Francisco think about the shapesAfter hearing Alonsorsquos views about the shapes Prospero says that this manrsquos praise of the spirits is rather hasty He means to say that Alonso has shown great haste in reaching the conclusion about the shapes Francisco is amazed to see that those shapes disappeared in a mysterious way(c) What does Sebastian ask Alonso to doSebastian tells Alonso that the shapes having disappeared should not matter to them because they have left the eatables behind He asks Alonso to enjoy eating as they are extremely hungry but the king does not accept his offer of enjoying the dishes(d) How does Gonzalo try to dispel Alonsorsquos fear of those strange shapes What kind of references does he give to AlonsoGonzalo says that those who have travelled abroad have reported seeing even stranger sights than these shapes that Alonso and his companions have beheld Hence there is no reason to feel afraid of these shapes Gonzalo further adds that in his younger days he had heard strange stories from travelers and Alonso might have heard similar stories For instance it was said that there existed a certain race of

human beings who had huge lumps of flesh hanging at their throats and who therefore resembled bulls Then Gonzalo tells about a race of human beings whose heads were located at their breasts Gonzalo says that such stories were not believed by most people in those days but now-a-days these stories have become common(e) Explain the following lsquoEach putter-out of five for onersquoEnglish travellers often insured their trips with London brokers Those that went on foreign travels those days used to deposit a certain amount with some firm or company in London before their departure If the travelers failed to return the money was forfeited by the company with which it had been deposited But this money was repaid five-fold if the travelers returned safe and sound In this way a traveler stood a great chance of recovering the entire cost of his

travels(f) Give the explanatory meanings of the following expressions in the context of the above extract (i) Dewlapped (ii) Wallets of flesh

(iii) Putter-out(i) Dewlapped having big lumps of flesh at the necks(ii) Wallets of flesh large masses of flesh looking like bags(iii) Putter-out to invest money before commencing the travel

  • General methods of preparation of hydrogen
  • Chapter Dimensional Analysis (Summary)
    • Properties of Charge
Page 7:   · Web viewSubject. Topic. Summary. Execution. Hindi. व्याकरण. शरीरके अंगो के नाम लिखिए. 1) आँख 2) नाक 3

Bengali বইndashবোংো ভোষো পলিরচয়

পোঠndash৮ লিঙগ ৩ পংলিঙগ কোসক বসউঃ পরোলিবোচক লেP লিবসষয পসদর দবোরো পরষ োতীয় কোউসক লেবোঝোয় তোসক পংলিঙগ বস লেPম -বোবো 4োতর৪ সতরীলিঙগ কোসক বসউঃ পরোলিবোচক লেP লিবসষয পসদর তোরো সতরী োতীয় কোউসক লেবোঝোয় তোসক সতরীলিঙগ বস লেPম - মো লিলিকষকো৫ উভয়লিঙগ কোসক বস উঃ পরোলিবোচক লেPলিবসষয পসদর দবোরোপরষ ও সতরীউভয়োলিতসক লেবোঝোয় তোসক উভয়লিঙগ বস লেPম - লিশ মনতরী ৬ কলীবলিঙগ কোসক বস উঃ অপরোলিবোচক লেP লিবসষয পসদর দবোরো সতরী বো পরষ কোউসক ো বলিঝসয় লেকো2 পদো13 লেক লেবোঝোয়তোসক কলীবলিঙগ বস লেPম - বই

COMPUTER CHAPTER 3 EDITING IN MS WORD

MOVING THE TEXTMS WORD ALLOWS US TO MOVE A BLOCK OF TEXT OR GRAPHICS WITHIN A DOCUMENT

Q6) HOW TO WE USE THE COPY AND PASTE OPTIONAns) WE CAN CREATE A DUPLICATE COPY OF THE TEXTTHE STEPS TO COPY AND PASTE THE TEXT AS FOLLOWS----

SELECT THE PART OF THE TEXT THAT WE WANT TO COPY

SELECT THE COPY BUTTON IN THE CLIPBOARD GROUP UNDER THE HOME TAB OR PRESS CTRL + C KEYS

NOW PLACE THE CURSOR AT THE PLACE WHERE THE TEXT NEEDS TO BE COPIED

CLICK THE PASTE BUTTON IN THE CLIPBOARD GROUP UNDER THE HOME TAB OR PRESS CTRL + V KEYS

Science Adaptations in Animals

We find different kinds of animals in our surroundings We find birds flying in the sky monkeys jumping on trees fish in water camels in desert and so on Animals live in their natural homes which are called their habitats The habitat of a camel is a dry sandy desert and that of a tiger is a dense forest So different animals live in different habitats In the world a plant or an animal has to adapt or change itself to suits its surroundings This changes happens over hundreds and thousands of years A change that a living thing undergoes to become better suited to its surrounding is called adaptation

Fill in the blanks [pg no 40]1 Terrestrial animals live on

land 2 Polar bears are found in the

cold polar regions 3 Frogs have webbed feet that

help them to swim 4 Monkeys and koalas are

arboreal animals

Class VSubject Topic Summary Execution

COMPUTER130420

CHAPTER 2 APPLICATIONS OF COMPUTERS

PAGE NO-15C FILL IN THE BLANKS

INPUTS ANIMATION BANKS SOFTWARE SUPER COMPUTERS

D WRITE TRUE OR FALSE TRUE TRUE TRUE TRUE FALSE

Science Chapter 2 - The Skeletal System

Joints A joint is a place where two or more bones are joined with each other

There are two types of joints 1) Fixed joints 2) Movable joints

The movable joints are mainly four types ndash 1) Hinge joint 2) Pivot joint 3) Gliding joint 4) Ball and socket joint

DAnswer these questions

1)What are the functions of the skeleton

Ans ndash The functions of skeleton are -

i The skeleton gives our body shape ii The skeleton provide our body strength iii The skeleton protect our inner organs from outside injuries iv The skeleton gives our body support

2)List the type of movable joint in our body giving one example of each

Ans - The movable joints are mainly four types ndash a Hinge joint

Example of hinge joints are elbows nice fingers and toes

b Pivot joint Example - the joint between the skull and the backbone is the example of pivot joint

c Gliding joint Example of gliding joints

are wrist and ankle joints d Ball and socket joint

Example of ball and socket joints are shoulder joint and hip joint

English language

Transitive and intransitive verb

Pick out the verbs from the following sentences and say whether they are transitive or intransitive (page 21 ex B)1 gave ndash verb Transitive verb2 Has planted- verb Intransitive verb3 Were- verb Intransitive verb4 Asked- verb Transitive verb5 Told- verb Transitive verb

Social studies Conquering distances

Airways The only airline owned by the government is Air India which handles both domestic and international flightsAdvantages of airways Air transport is the fastestIt can access remote areasIt is the best means of transport in case of emergencies

1 Which is the only airline owned by the governmentAns Air India

2 Give two advantages of airwaysAns The two advantages of airways areAir transport is the fastest transportIt can access remote areas

DisadvantageIt is the most expensive of all other means of transport

3 What is the disadvantage of airwaysAns The only disadvantage of airways is that it is the most expensive of all other means of transport

MATHEMATICS

Ch 3Addition and Subtraction

Properties of Addition1 The sum of two numbers does not change when we change their order This property known as Commutative Property of addition

2 The sum of three numbers does not change when we change their grouping This property is called Associative Property of addition

3 The sum of the numbers and zero is the number itself This property is called Identity Property of addition and the integer 0 is called identity

Exercise 11Fill in the blanks1 2730815 + 8319293 = ____ + 27308152 18219 + 1850308 = 1850308 + ____3 (27815 + 85919) + 95985 = (85919 + ____) + 278154 13227 + (25983 + 73607) = (____ + 25983) + 736075 91389 + 0 = ____ + 91389

Solution 1 83192932 182193 959854 132275 0

6 Which of the following are true statements(a) Any number added to zero is zero

(b) The sum of two numbers does not change when we change their order

(c) 1 is the identity element of addition

(d) Given any three numbers their sum does not change when we change their grouping

Solution (a) False(b) True(c) False(d) True

Class VISubject Topic Summary Execution

HISTORY AND CIVICS

CHAPTER 3

MAHAVIRA AND BUDDHA ndash GREAT PREACHERS BUDDHA

Decline of Buddhism1 Revival of the Brahmanical Hinduism ndash Brahmin Scholars like Shankaracharya and Kumarila Bhatta led the revival of Hinduism and established the supremacy of Vedic religion2 Loss of Royal Patronage ndash Gupta period marked the decline of Buddhism as Gupta rulers were followers of Hinduism3 Split in Buddhism ndash division into Hinayana and Mahayana sects and rise of Mahayana sects blurred the line between Hinduism and Buddhism4 Corruption in Buddhist Sangha ndash due to generation of large revenue from large estates Buddhist monks and nuns started living luxurious

Answer the Following 1 During which dynasty Buddhism was split During the reign of Kanishka

2 Name the two sects of Buddhism Mahayana and Hinayana

3 Name two Vedic scholars who led the revival of Brahmanical Hinduism Shankaracharya and Kumarila Bhatta

life in rich monasteries Hence corruption crept in5 Adoption of Sanskrit ndash when Buddhist scriptures began to be written in Sanskrit in place of peoples language like Pali or Prakrit people started drifting away from Buddhism6 The Turkish Invasion ndash As Muslim conquerors invaded India immensely wealthy Buddhist Monasteries and temples were looted and destroyed and Buddhists were persecuted and killed

4 During which period decline of Buddhism began The Gupta Period

ENGLISH 2 The great train journey- Ruskin Bond

The great journey by Ruskin Bond is a story about Suraj who loved trains and wanted to go to places One day while wandering along the railway tracks he enters into a carriage compartment The train suddenly starts moving with him in the compartment and after a journey returns back to the same place from where it had begun The story is about his experience during that journey

4 Answer the following questionsf Who else is in the carriageA ragged hippy with a dirty beard face was in the carriageg Where does Suraj say that he would like to go toSuraj said that he would like to go to England and China and Africa and Greenland He wanted to go all over the worldh What warning does the man give to SurajThe man said Suraj to keep out of sight so that he doesnrsquot get caught by the ticket collectorsiWhen Suraj thinks about his parents for the first time what does he imagines that they will thinkSuraj thought that if he failed to come home that night his parents would think that he had run away or been kidnapped or been involved in an accidentJ What presents does Suraj imagine that he will bring back for his friendSuraj imagines that he would bring an African lion or a transistor- radio for his friend

CHEMISTRY

Chapter 2 ndashElement and Compound

ATOMAn atom can be defined as the smallest constituent particle of an element which showcases independent existence Example Ne OMOLECULEA molecule can be defined as the combinations of two or more atoms which are held together by chemical bonds A molecule is the smallest portion of a substance which showcases all the properties of the substance On breaking down a molecule further we see properties of the constituent elements Example HCl NaCl O2

Answer the following Q3) What is a moleculeAns - A molecule can be defined as the combinations of two or more atoms which are held together by chemical bonds A molecule is the smallest portion of a substance which showcases all the properties of the substance On breaking down a molecule further we see properties of the constituent elements Example HCl NaCl O2

Q4) Which can exist independently ndash atom or moleculesAns ndash Molecules can exist independently

PHYSICS Physical quantities

Guidelines for writing SI units correctly1 The units named after scientists are not written with a capital initial letter For example newton henry watt2 The symbols of the units named after scientist should be written by a capital letter For example N for newton H for henry W for watt3 Small letters are used as symbols for units not derived from a proper name For example m for metre kg for kilogram4 No full stop or other punctuation marks should be used within or at the end of symbols For example 50 m and not as 50 m5 The symbols of the units do not take plural form For example 10 kg not as 10 kgs6 When temperature is expressed in kelvin the degree sign is omitted For example 273 K not as 273o K (If expressed in Celsius scale degree sign is to be included For example 100o C and not 100 C)7 Use of solidus is recommended only for indicating a division of one letter unit symbol by another unit

Fill in the blanks

1) Length and mass are examples of fundamental physical quantities

2) The measurement of a physical quantity consists of two part magnitude and unit

3) A foot consist of 32 inches 4) The unit of temperature in the SI system is

Kelvin

Write true or false Correct the false statements

1) In ancient times cubit was used to measure the mass of an object FalseCorrect statement ndash In ancient times cubit was used to measure the length of an object

2) There are 7 fundamental physical quantities True

symbol Not more than one solidus is used For example m s-1 or m s J K mol or J K-1 mol-1 but not J K mol8 Some space is always to be left between the number and the symbol of the unit and also between the symbols for compound units such as force momentum etc For example it is not correct to write 23m The correct representation is 23 m kg m s-2 and not as kgms-29 Only accepted symbols should be used For example ampere is represented as A and not as amp or am second is represented as s and not as sec10 Numerical value of any physical quantity should be expressed in scientific notationFor an example density of mercury is 136 x 104 kg m-3 and not as 13600 kg m-3

3) Second is the unit of time in both the CGS and MKS systems True

4) The symbol used for a unit is always written in capital letters False Correct statement -The symbol used for a unit is normally written in small letters

Hindi 2nd language

वाकय निवचार भागवत निवचारो को परकट करन वा साथक एव वयवसथिtत शबद समह को वाकय कहत ह वाकय दो परकार क होत ह ndash

1 उददशय- वाकय म जिजसक बार म कछ बताया जाता ह उस उददशय कहत ह जस राधा एक नतकी ह2 निवधय- वाकय म उददशय क बार म बताया जाता ह उस निवधयक कहत ह जस- राधा एक नतकी ह रचना क आधार पर वाकय क तीन भद होत ह ndash१सर वाकय- राम बाजार गया २ सयकत वाकय- राम बाजार गया और वहा जाकर दोसत स मिमा३ मिमशर वाकय- यह वही tान ह जहा उनका बचपन बीता

helliphellipContinue to nextBengali 2nd language

লিZ সবরপ ও সবরলিZ

সবরলিZর লিয়ম - ১ অ-কোর লিকংবো আ-কোসরর পসর অ-কোর লিকংবো আ - কোর োকস উভয় লিমস আ ndashকোর য় এবং ওই আ ndash কোর পব13বস13 Pকত য়

২ ই - কোর লিকংবো ঈ - কোসরর পসর ই - কোর লিকংবো ঈ - কোর োকস উভয় লিমস ঈ - কোর য় এবং ওই ঈ - কোর পব13বস13 Pকত য়

৩ উ - কোর লিকংবো ঊ - কোসরর পসর উ - কোর লিকংবো ঊ - কোর োকস উভয় লিমস ঊ - কোর য় এবং ওই ঊ - কোর পব13বস13 Pকত য়

৪ অ - কোর লিকংবো আ ndash কোসরর পসর ই - কোর লিকংবো ঈ - কোর োকস উভয় লিমস এ - কোর য় এবং ওই এ - কোর পব13বস13 Pকত য়

১ অ + অ = আ ( gtো ) লিম + অচ = লিমোচ সব + অ3ী = সবো3ী অ + আ = আ ( gtো )পদম + আ = পদমো শভ + আলি = শভোলি আ + আ = আ ( gtো )4োয়ো + আবত = 4োয়োবত মো + আতমো = মোতমো আ + অ = আ ( gtো )লিবদযো + অংকোর = লিবদযোংকোর Pো + অ13 = Pো13 ২ ই + ই = ঈ ( gtী )অলিত + ইব = অতীব লিগলির + ইনদর = লিগরীনদর ই + ঈ = ঈ ( gtী )পলির + ইকষো = পরীকষো অলি3 + ঈশবর = অ3ীশবর ঈ + ঈ = ঈ ( gtী )মী + ঈশবর = মীশবর 3ী + ঈ = 3ী ঈ + ই = ঈ ( gtী )রী + ইনদর = রীনদর মী + ইনদর = মীনদর ৩ উ + উ = ঊ ( gt )মর + উদযো = মরদযো কট + উলিকত = কটলিকত উ + ঊ = ঊ ( gt )ঘ + ঊলিম13 = ঘলিম13 লিZ + ঊলিম13 = লিZলিম13 ঊ + ঊ = ঊ ( gt )রP + ঊলিম13 = রPলিম13

৪ অ + ই = এ ( লেgt )র + ইনদর = সরনদর লেPোগ + ইনদর = লেPোসগনদর অ + ঈ = এ ( লেgt )গ + ঈ = গস

র + ঈ = সর আ + ই = এ ( লেgt )Pো + ইষট = Pসষট 3ো + ইনদ = স3নদ আ + ঈ = এ ( লেgt )রমো + ঈ = রসম দবোরকো + ঈশবর = দবোরসকশবর

COMPUTER THE WORLD OF WINDOWS 10

DONE IN THE PREVIOUS CLASS PAGE NO-83A TICK THE CORRECT OPTION BACKGROUND DISPLAY AREA RESTORE THREE

MATHEMATICS Topic ndash NumbersChapter ndash Natural numbers and whole numbers

Study item Properties of whole numbers for subtraction1) Closure property When we do subtraction of two whole numbers we can not get a whole number in all time Example 8 ndash 3 = 5 a whole number 0 ndash 6 = -6 is not a whole numberTherefore the subtraction of two whole numbers is not satisfying closure property2) Commutative property If x and y are two whole numbers then x ndash y is not equal to y ndash xExample If x=16 and y = 7 then x ndash y = 16 ndash 7 = 9Again y ndash x = 7 ndash 16 = - 9 Therefore x ndash y not equal to y ndash x Therefore the subtraction of two whole numbers is not satisfy commutative3) Associative property If x y and z are three whole numbersThen x ndash ( y ndash z ) not equal to ( x ndash y ) ndash z Example If x = 20 y = 10 and z = 6Therefore x ndash (y ndash z ) = 20 ndash(10 ndash 6 ) = 20 ndash 4 = 16(X ndash y ) ndash z = (20 ndash 10) ndash 4 = 10 -4 =6Therefore x ndash(y ndash z) not equal to ( x ndash y) ndash zTherefore subtraction of whole numbers is not satisfying associativity4) Distributive property If x y and z are three whole numbersThen x (y ndash z ) = xy ndash xzAnd (y ndash z)x = yx ndash zxExample If x = 10 y = 6 and z = 4x(y ndash z ) = 10(6 ndash 4 ) = 10times6 ndash 10times4 = 60 ndash 40 = 20( 6 ndash 4 )times 10 = 6times10 ndash 4times10 = 60 ndash 40 = 20Therefore the subtraction of whole numbers is satisfying distributive property5) Existence of identity For any whole number x X ndash 0 = x but 0 ndash x = - x not equal to xThus for subtraction no identity number existsException 0 ndash 0 = 0 so 0 is its own identity for subtraction

Class VIISubject Topic Summary Execution

Hindi 2ndlang वचन जो सजञा शबद निकसी वसत या पराणी क एक या अनक होन का बोध कराया उनह वचन कहत ह जस डका- डकयह दो परकार की होती ह-

क) एकवचन-शबद क जिजस रप स उसक एक होन का बोध हो उस एक वचन कहत ह जस निकताब गमा आदिद

) बहवचन-शबद क जिजस रप स उसक आन ोन का पता च उस बहवचन कहत ह जस डक निकताब निततलियाआदिद

निनमनलिखित शबदो को एकवचन स बहवचन म बदोम- हमजानित- जानितयानारी- नारिरयामिमतर ndashमिमतरोपसतक -पसतकसडक-सडकबोत-बोतनाहर-नहररपए-रपया

Bengali বইndashবোংো োলিতয পলিরচয়

পোঠndash১৪ গলপ - অপর কলপো পর

লেক - লিবভলিতভষ বসনদযোপো3যোয়লেকndash রবীনদর পরবতf বোংো কোোলিতয 3োরোর উসgসPোগয োম পরকলিতসপরমী লিবভলিতভষ বসনদযোপো3যোয় তোর লেীসত লেPম বোসর বোসর লিফসর এসস4 গরোম বোংোর পরকলিতর কো লেতমলি এসস4 গরোমী মো লিচতরগলপndash অপর কলপো গলপোংটি লিবভলিতভষ বসনদযোপো3যোসয়র লিবযোত উপযো পসর পাোচোী লেসক গীত অপ অ13োৎ পসর পাোচোী তো অপর কলপোর লেকনদরীয় চলিরতর এই অংস আমরো পোই বোক অপসক বোক অপ কলপো লিবোী লে দসরর অ গো4 লেদস মোসয়র মস লেোো রপকোর রোসয পোলি2 লেদয় দপরসবো মোসয়র মসর কসর কোীদোী মোভোরত এর করসকষতর Pসjর ব13ো শস তোর মোবীর কস13র পরলিত ব2 মমতো য় আবোরপালিসত বলি13ত Pসjর অমোপত অং লে লিসই মোপত কসর বোলি2র লিপ4স বাো বোগোস লিকংবো উঠোসর লিশমসর কলপো লিবো এোস পরকো লেপসয়স4

১ অপর কলপো গসলপর লেক লেক তোর মপসক13 লেসো২ অপর কলপো গলপটি লেকোো লেসক গীত গলপটির ম ভোব লেসো

GEOGRAPHY CHAPTER 7EUROPE

CHAPTER COMPLETE EXERCISEFill in the blanks1 Europe is a continent that comprises the western part of Eurasia2 Eurasia and Africa are connected into one large land mass known as Afroeurasia3 The Strait of Gibraltar separates Europe and Africa4 Europe is surrounded by the Arctic Ocean to the north5 The British Isles includes the island countries of Great Britain and Ireland

Name the following 1 Connects Africa to Eurasia - Isthmus of Suez2 Largest country in the world in terms of area ndash Russia3 A term used collectively for the five countries in northern Europe ndash Nordic Countries4 The capital of Montenegro - Podgorica5 the largest fjord in Norway ndash Sognefjord

Match the following Column 1 Column 2a Albania iii Tiranab Belgium i Brusselsc Denmark v Copenhagend Finland ii Helsinkie Hungary iv Budapest

CHEMISTRY Chapter 2 ndashElement and Compound

Atom - An atom is the basic unit of an element or the smallest particle of an element non capable of independent existence Atom is built up of three sub atomic particles electron proton and neutron

Nucleus-It is the centre of an atom In the centre of the atom contains proton (positively charged particles ) and neutrons ( particles carrying no charge )

Orbits- It surround the nucleus in which revolve electrons (negatively charged particles)

Answer the following

1) What are MetalloidsAns - Certain elements using properties of both metal and non-metals are called metalloids Example Silicon arsenic and antimony

2) What are Noble gasesAns - Certain elements are present in the air and are chemically inert or unreactive Such elements are called rare gases or noble gases Example helium neon argon and Krypton

English 2 Sentences based on meanings

Kinds of sentences

Assertive or declarative to convey information or simply make a statement

Interrogative to ask different types of questions

Imperative to command or instruct someone or make a request

Exclamatory to express strong feelings and emotions

Exercise B1 Stop it ( Exclamatory)2 May you always be happy

together ( Exclamatory)3 He does not like sports

( Assertive)4 Please pass me the salt

( Imperative)5 How dare she talk to me like

that ( Exclamatory)6 May success bless your effort

( Exclamatory)7 Canrsquot you wait for sometime

(Interrogative)8 Did anybody tell you about it

( Interrogative)9 I saw her waiting for the bus

( Assertive)10 Could you please take a

message for me ( Interrogative)

Homework Ex ABiology Chp -2

Classification of Plants

Today we discuss about usefulness of bacteria We also discuss what the harmful effects of bacteria are

89 How bacteria are useful for usbull Bacteria is helpful in many ways forhuman being i) Production of medicine - antibiotics vaccine etcii) Formation of curd by lactobacillusiii)Nitrogen fixation in Leguminousplant by Rhizobiumiv) Increase soil fertility by absorbingatmospheric nitrogen and convert it into nitrates and nitritesv) Cleaning the environment by converting the complex substances into simple substancesvi) Tanning of leathervii) Retting of Fibersviii) Formation of compost by acting onanimal dung and agricultual cases1x) Biogas production by decomposingplant and animal wastex)Help In Nutrition by producing vitamiacutemBand kx1) Some bacteria are used to give specialflavour to tea coffee and coccaQ10- Name some diseases and there causativebacteriabull Diseasescausative bacteria1 CholeraVibrio cholerae2 Tuberculosis - Mycobacterium tuberculosis3 Diptheria -Corynebacteriumdiphtheriae4 Pneumonia - Streptococcus pneumoniae

Math Number system

Chapter Fraction

Study item Using lsquoofrsquoThe word lsquoofrsquo between any two fractions is to be used as multiplicationExample 57 of 56 = 57 times 56 = 5times8 = 40Study item Using BODMASThe word lsquoBODMASrsquo is the abbreviation formed by taking the initial letters of six operations(i)Bracket (ii) of (iii) Division (iv) Multiplication (v) Addition (vi) SubtractionAccording to BODMAS rule First of all the terms inside Bracket must be simplified then lsquoofrsquo lsquoDivisionrsquo lsquoMultiplicationrsquo lsquoAdditionrsquo lsquosubtractionrsquo

Study item Removal of Brackets

There are four Brackets of algebra in Mathematics In a complex expression four types of brackets are used Order of removing the brackets is first ----- then ( ) then finally [ ]

Class VIIISubject Topic Summary Execution

Chemistry Hydrogen General methods of preparation of hydrogen

By the action of dilute acids on metals

Calcium Reacts readily to form chloride salt and hydrogen

Ca + 2HCl rarr CaCl₂ + H₂uarr

Magnesium

Aluminium

Zinc

React readily to form salt and hydrogen

Mg + 2HCl rarr MgCl₂ + H₂uarr2Al + 6HCl rarr 2AlCl₃ + 3H₂uarrZn + 2HCl rarr ZnCl₂ + H₂uarr

Question 4 ) Give reasons for the following

(a) Hydrogen be used as a fuel

Solution

Hydrogen is used as a fuel because it has a high heat of combustion Some significant fuels are coal gas water gas and liquid hydrogen

(b) Though hydrogen is lighter than air it cannot be collected by downward displacement of air

Solution

Hydrogen is lighter than air so it is possible to collect the gas by downward displacement of air But it is not safe to do so since a mixture of hydrogen and air can lead to an explosion

(c) A pop sound produced when hydrogen is burnt

Solution

Impure hydrogen gas burns in air with a pop sound This is because of the presence of impurities in it

(d) Helium replaced hydrogen in weather observation balloons

Solution

It forms a mixture with air that can explode when there is a small leakage of hydrogen in a balloon So helium has replaced hydrogen

(e) Nitric acid not used for the preparation of hydrogen gas

Solution

(e) By the action of nitric acid on metals hydrogen cannot be produced because it also releases nitrous oxide and nitric oxide and oxides the hydrogen to form water

Biology Chp-2 Reproduction in plants

Today we discuss different methods of artificial propagation like cutting-rose sugercane Layering ndashguava lemon china rose etc Grafting- mango apple etcMicropropagation ndashorchid asparagus etcWe also discuss about advantages and disadvantages of vegetative propagation

Q7 Define the following terms i) Explant In tissue culture techniquea tiny piece of bud shoot or any other partof plant from where new tissue develop ii) Callus The cells of the tissue divide andgrow into a mass of undifferentiated cells from explant iii) Plantlet After few days callus differentiate into a small plant with roots and shootQ8 what are the advantages and limitations of tissue culture or micropropagation

Advantages i ) It produacuteces superior quality plantsii)It can be applied to interspecifie hybridsiii) It is useful to grow seedless plants bull Limitations i) It cannot be used for all plantsii)It is not easy to handleQ9 Write advantages of vegetative propagationi) It is a quick and easy method ofproducing new plantsii) This method need less time to matureiii) The new plants are exact copies of the parentiv) it is extremly useful for growing seedlessplants like banana grapes etc Q10 Write some disadvantages of vegetativepropagationi) Dišeases present in the parent plant gettransferred to all in new plantsii) Overcrowding of new plants causes competition for sunlight water and nutrients which affects growth of plantsplant

Physics Chapter 2 Physical Quatites and Measurements

Here We Will Do Some QuestionsRelated To Chapter 2

Select the correct alternative A block of wood of density 08gcm-3 has a volume of 60cm3 The mass of the block is

1 608 g

2 75 g

3 48 g

4 0013 g

Solution 348 g

The density of aluminium is 27g and that of brass The correct statement is

1 Equal masses of aluminium and brass have equal volumes

2 The mass of a certain volume of brass is more than the mass of an equal volume of aluminium

3 The volume of a certain mass of brass is more than the volume of an equal mass of aluminium

4 Equal volumes of aluminium and brass have equal masses

Solution 2 The mass of a certain volume of brass is more than the mass of an equal volume of aluminium

MATHEMATICS Ch 6Sets

Exercise 6(C)1 Find all the subset of each the following sets(i) A = 57 (iii) C = x xisin W x le 2(iv) p p is a letter in the word lsquopoorrsquo

Solution (i) All the subsets of A are ϕ 5 7 57

(iii) All the subsets of C are ϕ 0 1 2 01 02 12 012

(iv) All the subsets are ϕ p o r po or por

4 Given the universal set = -7-3-105689 find (i) A = x xlt2 (ii) B = x -4ltxlt6 Solution

(i) A = -7-3-10(ii) B = -3-105

5 Given the universal set = x xisin N and xlt20 find

(i) A = x x = 3p pisin N (iii) C = x x is divisible by 4 Solution

(i) 369121518 (iii) 481216

6 Find the proper subset of x x2-9x-10 = 0 Solution

ϕ 10 -1

Working x2-9x-10 = 0 rArr x2-(10-1)x-10 = 0

rArr x2-10x+x-10 = 0 rArrx(x-10)+1(x-10) = 0

rArr (x+1) (x-10) = 0

11 Let M = letters of the word REAL and N = letters of the word LARE Write sets M and N in roster form and then state whether (i) M sube N is true (ii) N sube M is true (iii) M = N is true

Solution M = real and N = lareSo (i) Yes (ii) Yes (iii) Yes

English 2 Twelfth Night ndash Shakespeare

A noble man named Orsino in the kingdom of Illyria is deeply in love with a lady called lady Olivia She is in mourning for her dead brother so she will not even think about marriage At this time a sea storm causes a terrible shipwreck and a young lady called Viola is swept onto the shore She thinks that her twin brother Sebastian is drowned A sea captain tells her about Orsino and his love for Olivia Viola wishes to work in Oliviarsquos home but feels she will not be employed So she dresses as a man calls herself Cesario and gets work at the house of OrsinoViola (now Ceasario) is much liked by Orsino and becomes his page She falls in love with Orsino Orsino sends Ceasario to deliver messages to Olivia Olivia herself falls for the beautiful young Ceasario believing Viola to be a man

2 Answer the following questionsa Why does Orsino ask the musicians to play onOrsino asks the musicians to play on because music feeds his desire He calls upon the musicians to play music so that his hunger for love could be replenished with an excess of musicb What does Valentine tell about OliviaWe learn from Valentine that Olivia is in mourning for her brother she wears a veil and has vowed that no one will see her face for another seven yearsand she refuses to marry anyone until thenc From the exchange between Orsino and Valentine what do you think their relationship isValentine is one of orsinod attendants He was sent to Olivia as a messenger of love but was not allowed to speak to here Who is Olivia mourning for and whyOlivia is mourning for her dead brother

Homework Q fHistory and Civics

Growth of Nationalism

Important dates to remember1769-Napoleon born on 15thAugust1789-Fall of Bastille on 14th July and the beginning of the French revolution declaration of the rights of Man on 26thAugust1793-King Louis XVI executed on January 211764-The Sugar Act passed1765-The Stamp act passed1774-The first congress of Philadelphia1776-The declaration of American Independence of on 4th July1777-Defeat of the British at Saratoga1781-Surrender of lord Cornwallis at Yorktown1783-The treaty of Versailles1804-Napoleon becomes the emperor1813-Battle of Leipzig or Battle of nations in which Napoleon was defeated by the Allies1815-Battle of Waterloo June 18 in which Napoleon was defeated and captured1821-Death of Napoleon in StHelena1860-Abraham Lincoln elected President of the USA1861-The civil war began 1864-Abraham Lincoln elected President of the USA for the second time1865-Slavery abolished in the US

Name the following- The queen of Louis XVI

Marie Antoinette The three philosophers of France

VoltaireMontesquieuJean Jacques Rousseau

The British general whose surrender brought the war in America to an endLord Cornwallis

The first president of the USAGeorge Washington (1732-1799)

The first southern state to secede from the unionSouth Carolina

The author of the book lsquoUncle Toms CabinHarriet Beecher Stowe

Homework-Learn

Class IXSubject Topic Summary Execution

Economics

Types of economies Today I am going to share you the concept of economic growth and economic development Few questions will be given from the previous study material dated 942020

Meaning of economic growthAnswer) The term economic growth generally means anincrease in national income or per capita output or income over time It indicates towards quantitative growth of a country

Meaning of economic developmentAnswer) Economic development is defined

as a process whereby the real per capita income of a country increases over time along with fall in poverty ratio unemployment and income inequality etc

Distinguish between economic growth and economic development

Basis Economic growth

Economic development

Scope It has narrow scope as it refer only to rise in per capita income

It has wide concept since it includes qualitative changes as well

Concerned matter

It is concerned with the rise in income

It is concerned with not only rise in income but also reduction of poverty income inequality and unemployment

Focus Economic growth does not focus on economic development

Economic development focus on economic growth plus qualitative changes

Distinguish between capitalist economy and socialist economy

Ownership

Motive

Tool

Means of production are owned and managed by private people

Self interest and profit earning is the main motive

Price mechanism is a main tool to solve the economic problems

Means of production are owned and managed by the government

Social welfare is the main motive

Economic planning by the government is the main tool to solve the economic problem

Competition

Distribution of income

There exist large competition among buyers and sellers

There is existence of large inequalities of income

There is no such competition

There exist less inequalities of income

Math Topic ndash AlgebraChapter -Factorisation

Study item Factorising by taking out common factorSome solved sums from exercise 41

1) (i) 8xy3 + 12x2y2

= HCF of 8xy3 and 12x2y2 is 4xy2

= 4xy2(2y + 3x )

4) (ii) 28p2q2r ndash 42pq2r2

= HCF of 28p2q2r and 42pq2r2 is 14pq2r = 14pq2r (2p - 3r )5) (ii) 14mn + 22m - 62p=HCF of 14mn 22m and 62p is 2= 2(7mn + 11m - 31p)7) (ii) 3a(x2 + y2) + 6b (x2 + y2) = HCF of 3a(x2 + y2) and 6b(x2 + y2 ) is (x2 + y2)= ( x2+ y2 )(3a + 6b )9) (ii) x(x2 + y2 ndash z2 ) + y(-x2ndashy2 + z2 ) ndash z(x2+ y2 ndash z2 )= x(x2 + y2 -z2) ndash y-(x2 + y2 -z2) -z(x2 + y2 ndash z2)=x(x2 + y2-z2) -y( x2 + y2-z2) ndash z (x2 + y2 -z2)= (x2+ y2 ndash z2)(x ndash y ndash z )

Commercial Studies

Introduction to Accounting and Book-keeping

Today I am going to share you the meaning of Accounting and Book-keeping and its related terms bullAccounting bullBook Keeping bullAccountsbullTypes Of Accounts bullAccounting Cycle

bull Meaning of accounting

Ans )Accounting is the art and science of recording classifying and summarising monetary transactions

bull Meaning of Book-keeping

Ans) Bookkeeping is the art of recording business transactions with the view of having a permanent record of them and showing their effect on wealth

bull Meaning of account

Ans) The term account means a record of business transactions concern a particular person of firm asset or income or expense It is a summarised record of all transactions which take place in an accounting year

bull Types of accountsPersonal accounts ndash Personal accounts relating

to person and Organisation are known

as personal accounts Example Ramrsquos Account ABC amp Co Account etc

Real account - The accounts related to tangible and intangible assets are called real accountsExample Cash Account Furniture Account etc

Nominal account- Accounts related to expenses losses incomes and gains are known as nominal accountsExample Wages Account Salary Account Discount Account etc

bull Accounting cycle Accounting cycle refers to a complete sequence of accounting activities It begins with recording of transactions and ends with the preparation of a balance sheet

English 1 Transformation of sentences

Sentences A sentence is a group of words which makes complete sense

a Assertive sentencesb Imperative sentencesc Interrogative

sentencesd Exclamatory sentences

Sentences can be changed from one grammatical form to another without changing the meaning of the sentence This is known as transformation of sentences

Exercise 6Rewrite the following sentences according to the instructions given below without changing their meanings

1 As soon as he saw the beer he jumped into the river ( Begin No sooner)

2 None but brave deserve the fair (Begin the bravehellip)

3 This box is too heavy for me to lift ( Use so hellip That instead of too)

4 No one other than a king can live like James Luxurious ( Begin only James)

5 Oh for the wings of a dove (Begin I wishhellip)

BENGALI(2ND LANGUAGE)

ldquo বঙগভমির পরমি ldquo াইকেল ধসদন দতত

পব13পোসঠ আসোলিচত ৩ পরবোস দৈদসবর বস ীবতোরো Pলিদ স এ লেদ -আকো সত-োলি লেদ তোস - ক) বকতো লেক কোর লেো লেকো কলিবতোর অং ) কোর পরলিত বকতোর এই উলিকত গ) এ লেদ আকো সত বসত কী বলিঝসয়স4 ীবতোরো বসত কী লেবোঝ ঘ ) আসোচয অংসর তোৎপP13 কী

উ -ক ) বকতো স কলিব মোইসক ম3দ দতত

Types of AccountPersonal AccountReal AccountNominal AccountBalance Sheet (opening)

কলিব মোইসক ম3দ দসততর রলিচত বঙগভলিমর পরলিত কলিবতোর অং ) কলিব বঙগী অ13োৎ লেদমোতোর পরলিত কলিবর এই উলিকত গ ) এ লেদ আকো বসত কলিবর মোব লেদী রপ আকো লেক লেবোঝোসো সয়স4 আকো লেসক লেPম তোরো স পসর লেতমলি ীব লেদ রপ আকো লেসক পরো রপ তোরো স পরসত পোসর এই মভোবোর কোই কলিব বসস4 ঘ ) পরবো Pোতরোয় Pলিদ কলিবর লেদ আকো লেসক ীব তোরো রপ পরো স পসর তোসত কলিব লিবনদমোতর দঃলিত কোর মতয লিবসর সবোভোলিবক পলিরলিত এবং মোষ মরী তোই পরবোস Pলিদ তা োর মতয য় তবও কলিব লিবচলিত সব ো কোর পলিবীসত লেকউ অমর য় লিক4ই অকষয় য় দীর লেPম লিচরপরবোমো লেতমলি মোসষর ীবও চমোতোই ীব - সতবধতোই মতয ীব দীসত মোষ লিতয পরবোমো তবও লেPব মোষ আপ কতকসম13র মো3যসম মোসষর মস লিসসদর সথো কসর লিসত পোসর তোরো লিচরভোসবর সয় মোসষর মস লিবরো কসর তোসদর মস3য লেকউ পGভসত লিবী সয় গোসও মোসষর মস তোরো লিতযপলিত লিতযবলিনদত

Hindi 2ndlang

काकीी(लिसयारामशरणगपत)

इस कहानी म क न यह बतान का परयास निकया ह निक बचच अपनी मा स निकतना परम करत ह शयाम अबोध बाक ह वह अपनी मा क मरन क बाद उसन अपनी मा क लिए बहत रोया बाद म उस पता चा निक उसकी मा राम क घर ची गई ह आकाश म उडती हई पतग दकर उस हष हआ निक पतग क दवारा वह अपनी मा को नीच उतारगा इसक लिए वह अपनी निपता की जब स दो बार सवा रपया निनकाकर पतग और दो मोटी सी मन वाी अपन भाई स काकी एक कागज पर लिवा कर पतग म लिशव का दिदयानिनकाकर पतग और दो मोटी सी मन वाी अपन भाई स काकी एक कागज पर लिवा कर पतग म लिचपका दिदयाभोा और शयाम कोठरी म रससी बाधनी रह थ तभी उसक निपता करोध म आकर उन स पछ निक कया उनकी जब स रपया निनकाा हभोा डर क मार बताया निक शयाम इस पतग क दवारा अपनी काकी को राम क यहा स उतारना चाहता हनिवशशवर(शयाम क निपता)न फटी पतग उठाकर दी तो उस पर काकी लिा थावह हत बजि होकर वही ड रह गएउनहोन सोचा निक मन अपन पतर को मारा जोनिक अनजान और निनदष थावह अपनी मा कोनिकतना पयार करता ह

helliphellipContinue to next

Computer Application

Java Programming Prog 1Write a java program to input two numbers from user and display the sum or product of them as per user choice Use switch case statementSolve public class sum_product public static void main(String args[]) Scanner sc=new Scanner(Systemin) int abc Systemoutprintln(ldquoEnter two numbersrdquo) a=scnextInt() b=scnextInt() Systemoutprintln(ldquoPress 1 for sum or 2 for productrdquo)

c=scnextInt() switch(c) case 1 Systemoutprintln(ldquoThe sum will be =rdquo+(a+b)) break case 2 Systemoutprintln(ldquoThe product will be =rdquo+(ab)) break default Systemoutprintln(ldquoWrong Inputrdquo) Home Work - Practice in your computer using bluej

Subject Eng Literature (The Merchant of Venice ndash William Shakespeare)Topic Act I Scene 2 Lines 92 to 126 (End of scene) Date 13th April 2020 (5th Period)

[Students should read the original play and also the paraphrase given in the school prescribed textbook]Summary Questions amp Answers

o After Portia has expressed her opinion about the suitors Nerissa informs that she need not bother about any one of them as they have decided to quit Belmont at the earliest opportunity because they do not believe in trying their luck by the caskets which is the only way of winning Portia

o Nerissa then enquires of Portiarsquos opinion about Bassanio who once visited her in the company of the Marquis of Montferrat and says that she had never come across such an ideal love deserving the fairest lady for his bride

o Portia seems to remember Bassanio quite correctly and says that she agrees with Nerissa At this moment a servant informs Portia that the Prince of Morocco has arrived to try his luck by the caskets

o Portia tells Nerissa that if she could welcome this new suitor as gladly as she says farewell to the previous ones she would be glad of his arrival However if he happens to have the virtues of a saint but the black complexion of a devil she would prefer to have him for religious consolation rather than as a husband

(1) NERISSA You need not fear lady (Line 97-103)

the having any of these lords they have acquainted me with their determinations

which is indeed to return to their home and to

trouble you with no more suit unless you may be wonby some other sort than your fathers imposition depending on the caskets

PORTIA If I live to be as old as Sibylla I will die as chaste asDiana unless I be obtained by the manner of my fatherswill I am glad this parcel of wooers are so reasonablefor there is not one among them but I dote on his veryabsence and I pray God grant them a fair departure

(a) Elucidate the idea expressed in the first speech of the above dialogue

In the first speech Nerissa assures Portia that she need not have any fear of being compelled to marry anyone of the suitors who had lately come to Belmont She informs her that they have all decided to return to their respective countries(b) Illuminate the meaning of the phrase ldquoyour fatherrsquos imposition depending on the casketsrdquo

Nerissa means that the suitors of Portia do not find the conditions imposed by the will of her father to their liking They are too hard for them These conditions are that in the event of a suitor failing to choose the right casket (i) he should never disclose to anybody which casket he chose (ii) he can never marry and (iii) he should take his departure immediately(c) Explain the meaning of the term lsquoSibyllarsquo

lsquoSibyllarsquo is the name given by Romans and Greeks to a prophetess inspired by some deity usually the sun-god Apollo She had a very long life The god Apollo granted her as many years of life as she could hold grains of sand in her hand(d) Elucidate the meaning of the term lsquoDianarsquo

lsquoDianarsquo is the goddess of hunting She is also regarded as a symbol of virginity because she never fell in love and never

married(e) Explain the meaning of the first two lines of Portiarsquos speech

Portia says that even if she is to live for centuries like Sibylla she would not marry except in accordance to her fatherrsquos will She asserts that she would not mind remaining unmarried and untouched by a man like Diana the virgin the goddess of hunting unless a man is able to win her by passing the test laid down by her father

Class XSubject Topic Summary Execution

Hindi 2nd

Langबड घर की बटी( मशी परमचद)

lsquoबड घर की बीटीrsquo कहानी का उददशय मधयम वग की घर समसया को सझा कर सगदिठत परिरवार म मिम जकर परम स रहन का सदश दना ह घर म शानित tानिपत करन की जिजममदारी नारी की होती ह यदिद नारी समझदार ह उसम धय और परिरवार क परनित परम ह तो कोई भी घटना परिरवार को निवघदिटत नही कर सकती या कहानी परिरवार को सगदिठत करत हए परम सौहाद स एक रदसर की भावनाओ को समझ करउनका सहयोग करत हए जीवन यापन करन की पररणा दती ह मशीपरमचदर जी न इस कहानी म सयकत परिरवार का परनितनिनमिधतव निकया ह यह कहानी बनी माधव सिसह जो गौरी पर क जमीदार क उनक दो पतरो की हशरी कठ ा निबहारीशरीकात का निववाह एकजमीदार घरान की पतरी आनदी स हआ थाआनदी न द को ससरा क वातावरण म ढालिया थाएक दिदन आनदी का अपन दवर ा निबहारी स झगडा हो जाता ह दोनो भाई एक रदसर स अग होन की कोलिशश करत हसभी बह आनदी न अपन मधर वयवहार स ा निबहारी को घर छोडकर जान स रोक लिया| इस पर बनी माधव सिसह न कहा निक बड घर की बटी ऐसी ही होती ह जो निबगडा काम बना ती ह अतः शीषक साथक ह बड घर की बटी आनदी ह

helliphelliphelliphellipContinue to nextBiology Topic ndash Chp-1

CellWelcome to new session 2020-21Today we will start with Chpter 1 cell CELL

Protoplasm+Cellmembrane Or Cell wall

Cytoplasm+Neucleus

Cytoplasmic+ CytoplasmicOrganelles Inclutions(mitochondria (food Golgi bodies pigments)Ribosome)

What is cellbull Cell is the structural and functional unit of living organismbull According to number of cells organisms areUnicellular - Amoeba bacteria Multicellular - Rose Mango Tiger HumanSmallest cell -bacteria Longest cell - Nerve cellLargest cell - Ostrich egg cellCells are of different size and shapes according to their functionsQ2Write chief functions of following cellorganelles

Q3What is tonoplastVacuoles covered by a covering called tonoplast

Bengali(2Nd

Language)

ফ ফটক ো ফটক (কলিবতো ) ভোষ মসোপো3 gtPোয়

একটি লেমসয়র ীবস লেপরম লিকভোসব ফসট ওসঠ তো লেদলিসয়স4 কলিব লেপরম Pই য় লেই ময়ই বনত কোস পলিরত য় ফ লেফোটো বো োসফোটো লেটো ব2 কো য় লেমসয়সদর ব gtয13 লেপরসমর 4লিব ফসট উসঠস4 এই কলিবতোয় লেপরম মোষসক মত gtযর মস লেফস লিদসয় পরকষস বাোচোসোর gtয োত বো2োয় কলিবতোয় লেমসয়টির পসব13র দঃসর কো বো সও লেমসয়টি লেই পসর পলিক সত চোয়ো োরী ীবসর কোস4 পরম লেPৌবস লেপরমসক পোবোর পরব ইচছো োকসও তো পসর লেলিতবোচকতোয় পলিরত য় কলিব ভোষ মসোপো3 যোয় লেP ক লেপরসমর

কলিবতোয় ব gtযবহত লিবসষ লিক4 সvর অ13 লেদওয়ো ১) রসবোো= লেP লিবলিভনন রকম ডোকসত পোসর২) ো= পোর ৩) ঠলি = লেচোসর বZ৪)আই বস2ো=অলিববোলিত৫)শইসয় = োলিয়ত কসর৬)োতপাোচ= লিবলিভনন পরকোর৭)দ2োম = v কসর বZ কসর লেদওয়ো৮)লেরলিং =লেোোর দৈতরী লেব2ো৯) বনত= একঋত১০) পাোর = বসকরো2

Organelles Functions

1 Endoplasmic reticulum

2 Mitochondria

3Golgibodies

4 Ribosome5Lysosome

6Plastids

7 Centrosome

i) Supportive framework for the cellii) Synthesis and transpost of proteinsRelease of energy in the form of ATPi) Synthesis and secretion of enzymes hormoneii) Formation of vacuoles lysosomei) Protein Synthesisi) Intracellular digestionii) Destroy foreign substancei )Leucoplast - stores starchii)chloroplast - trap solar energyiii) Chromoplast - imparts colour toflowers amp fruitsi) Initiates and requlates cell division

কলিবতো তোর অ13সক ভোষোয় পরকোো কসর ঘলিরসয় ব যকত কসরস4 লেপরসমর ফতো আর লিবফতো লেক গোঢ় কসর লেদোসো কলিব ভোষ মসোপো3 যোসয়র অলিভবসর অ যলিদক

Economics

Factors of Production

Welcome to the new sessionToday we are going to start the first chapter of Class XThe name of the chapter is Factors of productionBy the name I hope you all can recall a glimpse of what you have learnt in the second chapter of Class IX

NowProduction is the process of creating the various goods and services which are consumed by the people of the country to satisfy their wants

Thus it is the process in which some materials are transformed from one form to another to create utility and value in goods

For example utility can be created by changing the form of a commodity ie

Making of table out of wood by a carpenter for his customer here the wood is getting transformed into table creating utility for his customer and he can also command a price for it

On the other hand Housewives perform very

useful activities at home which create utility but their domestic activities are not included in production because they have no money value

So we can also say that Production denotes two things firstly creation of utility and secondly creation of value

Production is not complete unless it reaches the consumer

An increase in production will increase the economic welfare of the consumers and hence the aim is to raise the production level of the country

Again production of a good or service is only possible if certain resources or

Questions

1 What do you mean by production

Answer Production means the creation of goods and services for the purpose of selling in the market

In fact production involves the transformation of inputs into outputs

Hence production denotes two thingsCreation of utility and creation of valueUtility and value can be created by changing the form by changing the place by changing the time and by rendering services

Example Transformation of raw

materials into finish goods such as potter creates utility by converting mud into utensils assembling of small parts to make bigger machinery

Production also includes services such as distribution and marketing

2 What are the factors of production

Answer Factors of Production refers to the resources and inputs needed for producing goods and servicesThese inputs can be classified as

Land Labour

Capital Enterprise

Land Land is defined to include not only the surface of the earth but also all other free gifts of nature(for example mineral resources forest resources and indeed anything that helps us to carry out the production of goods and services but is provided by

inputs are used together in right proportion

A resource or an input which helps in the process of production to obtain an output is called FACTOR OF PRODUCTION

These factors of production can broadly be categorized into four parts 1LAND 2LABOUR3CAPITAL4ENTERPRISE (ORGANISATION)or Entrepreneur

The above factors are all interdependent on each other and they play a major role in production process

FACTORS OF PRODUCTION

LANDCAPITAL

LABOUR ENTREPRENEUR

nature free of cost)LabourLabour refers to the human efforts that need to be combined with other factors of production for creating an output

CapitalAll man ndash made means of production is called capita example machineries which help in further production Money when used for starting any business for purchasing raw materials machinery tools etc it is regarded as capitalCapital also includes physical capital like factories machineriestoolsbuildingsequipments etcEnterpriseThe task of bearing risks is called enterprise and the person who bears these risks of business is called the entrepreneurThus an entrepreneur is one who organises production takes important decisions regarding production hires and purchases factors of production and bears the risk and uncertainty involved in productionOrganisation refers to the services of an entrepreneur who controls organises and undertakes all risks One who plans organises and manages a business enterprise is an organiser

Physics Chapter 1 Force

Force is an external agent capable of changing the state of rest or motion of a particular body It has a magnitude and a direction The direction towards which the force is applied is known as the direction of the force and the application of force is the point where force is applied The Force can be measured using a spring balance The SI unit of force is Newton (N)

Question 1

State the condition when on applying a force the body has

(a) the translational motion

(b) The rotational motion

Solutions

(a) Translational motion is produced when the body is free to move

(b) Rotational motion is produced when the body is pivoted at a point

Question 2

Define moment of force and state its SI unit

Solutions

The moment of force is equal to the product of the magnitude of the force and the perpendicular distance of the line of action of force from the axis

of rotation

The SI unit of moment of force is Newton times meter

= Newton meter (Nm)

Commercial Studies

Stake holders In this topic you will be come to know about the meaning and concept of stakeholders

How stakeholders are different from shareholders

Questions1 What do you mean by the term stake holdersAnswer) The term stake holders have developed from the words which mean an interest or expected benefit Stakeholders mean all those individuals groups and Institutions which have a state (interest) in the functioning and performance of a commercial organisation or a business enterprise2 What do you mean by share holdersAnswer) The person and Groups who own the shares of the joint stock company by providing capital to the company are called shareholders Shareholders are the internal stakeholders shareholders are one out of several stake holders3 How are shareholders different from stakeholdersAnswer)i) The term shareholders is related to only joint stock company whereas stakeholders are related with all business organisationsii) Stakeholders maybe any individual having financial stake in business organisation whereas a shareholders are those individuals who are holding shares in the company4) How are shareholders different from creditorsAnswer) i) Shareholders are internal stakeholders while creditors are external stakeholdersii) Shareholders invest in the capital of the company whereas creditors give loan to the companyiii) Shareholders are the members of the company with voting rights but creditors are not the members of the company

English 1 Transformation of sentences

Sentences A sentence is a group of words which makes complete sense

e Assertive sentencesf Imperative sentencesg Interrogative sentencesh Exclamatory sentences

Sentences can be changed from one grammatical form to another without changing the meaning of the sentence This is known as transformation of sentences

Exercise 1 Change the following affirmative sentences into Negative sentences

a He is a good manHe is not a bad man

b Ram loves SitaRam is not without love for Sita

c Only he stood first in the classNone but he stood first in the class

d Ankit was wiser than he

He was not so wise as Ankite He did it

He did not fail to do itf As soon as I reached college the

bell rangNo sooner did I reach college than the bell rang

g He finished everythingHe left nothing unfinished

h It always pours when it rainsIt never rains but it pours

Math Topic Commercial MathematicsChapter ndash Goods and services Tax

What is GSTAns It is a abbreviated term of Goods and Service Text which is an indirect tax levied on the sale of goods and rendering servicesSome terms related to GSTDelar Any person who buys goods or services For resale is known as a delar A delar Can be a firm or a companyIntra-state sales Sales of goods and services within the same state or same union territory are called intra- state salesInter-state sales Sales of goods and services outside the state or union territory are called Inter-state sales4) Input GST GST is paid by dealers on purchase of goods and services are called input GST5) Output GST GST is collected from customers on sale of goods and services are called output GST6) Types of GST There are three taxes applicable under GST(i) Central Goods and Services Tax (CGST)(ii) State Goods and Services Tax (SGST) or Union Territory Goods and Services Tax (UTGST) Both these taxes are levied on intra-state sales Here GST is divided equally among central and state governments(iii) Integrated Goods and Services Tax (IGST) IGST is levied on inter- state sales It is also levied on import of goods and services into India and export of goods and services from India

Subject Eng Literature (The Merchant of Venice ndash William Shakespeare)Topic Act III Scene 4 Lines 1 to 44 (Portia hellip To wish it back on you fare you well Jessica)[Students should read the original play and also the paraphrase given in the school prescribed textbook]

Summary Questions amp AnswersIn this scene we suddenly find a new element in the character of Portia We have already seen her possessed of every graceful womanly quality but now she shows that she is capable of rapid decision and determined action She shows this by her sudden resolve to hasten to Venice with a daring scheme for the rescue of Antonio This is an important scene in the dramatic action for it leads up to and renders possible the striking events of the famous trial scene which is one of the greatest striking elements of the play Moreover the fact that all the characters of importance are now assembled together in Venice makes the union of the main plot and the secondary story complete

(1) LORENZO Madam although I speak it in your presence(Line 1-9)

You have a noble and a true conceit

Of god-like amity which appears most strongly

In bearing thus the absence of your lordBut if you knew to whom you show this honourHow true a gentleman you send reliefHow dear a lover of my lord your husbandI know you would be prouder of the workThan customary bounty can enforce you

(a) Where is Lorenzo Why is he here To whom is he referring as lsquoMadamrsquo

Lorenzo is at Portiarsquos residence He had met Salerio on the way and Salerio had begged him to come along with him to

o In this scene Portia Nerissa Lorenzo Jessica and Balthazar appear

o Portia requests Lorenzo and Jessica to be in charge of her house during her absence from Belmont because she and Nerissa have decided to spend the days in meditation and also in visiting the holy places in the neighbourhood of Belmont She has already instructed her people to acknowledge both Lorenzo and Jessica as master and mistress of house during her absence Lorenzo and Jessica gladly agree to look after the house of Portia

handover the letter from Antonio to Bassanio The letter carried the bad news about Antoniorsquos arrest for non-payment of loan taken from Shylock Hence Salerio might have preferred company to break this bad news to Bassanio He is referring to Portia as Madam(b) What does Portia say on hearing the above extract

Portia says that she has never regretted doing good to others Friends who spend a lot of time together and really are there for each other have many traits in common As Antonio is Bassaniorsquos best friend saving him is like saving Bassanio who is like her own soul She asks Lorenzo to take care of management of the house till Bassanio is back(c) What does Portia send with Bassanio and why

On hearing about Antoniorsquos troubles on account of Bassanio her husband Portia immediately sends him with enough gold to repay the debt many times over to Venice to help Antonio out of his misfortune

(2) Lorenzo Madam with all my heart (Line 36-40)

I shall obey you in all fair commands

Portia My people do already know my mindAnd will acknowledge you and JessicaIn place of Lord Bassanio and myselfSo fare you well till we shall meet again

(a) Where are Lorenzo and Portia at this time What lsquofair commandsrsquo are given to Lorenzo

Lorenzo and Portia are at Belmont during this scenePortia reveals to Lorenzo that she has sworn to contemplate in prayer at a monastery around two miles away until her husband returns from Venice She tells him that Nerissa would accompany her and asks him to manage the house with Jessica till things are settled In response Lorenzo tells her that he would be obliged to do whatever she asks him to do(b) Where is Portia actually going and why

Portia tells Lorenzo that she would live a life of contemplation and pray at a monastery which is two miles away from her place In reality Portia plans to go to Venice in disguise with Nerissa and argue the case in defense of Antonio She is very sure that her plan would succeed

ClassXI (ScienceHumanitiesCommerce)Subject Topic Summary Execution

Computer Science

(APC)

Ch ndash 1 Numbers

(Numbers in different bases and

their Arithmatical operations)

Number System In computers Number System is defined as a writing system to represent the numbers in different ways ie we are using different symbols and notations to represent numbers There are four ways we can represent the number ndash Binary Decimal Octal and Hexadecimal

Decimal Number SystemThis number system consist 10 digits These are 0 1 2 3 4 5 6 7 8 amp 9

Binary Number SystemThis number system has only two digits these are 0 and 1 Here 0 stands for off while 1 stands for on

Octal Number SystemThis number system has 8 digits these are 0 1 2 3 4 5 6 amp 7

Hexadecimal Number SystemThis number system has 16 digits these are 0 1 2 3 4 5 6 7 8 9 A B C D E F Here the value of the alphabets are as follows A=10 B=11 C=12 D=13 E=14 F=15

Rules for conversion decimal number to Binary1 Divide the decimal number by 22 If the number will not divide equally by 2 then round down the answer to the nearest whole number (integer)3 Keep a note of the remainder it should be either 0 or 14 Keep repeating the above steps dividing each answer by 2 until you reach zero5 Write out all the remainders from bottom to top This is your binary solution

For example Lets convert 32 to binary 2 32 2 16 - 0 2 8 - 0 2 4 - 0 2 2 - 0 2 1 - 0 0 - 1

The binary equivalent of 3210 is 1000002

Try the follwing youself1 2410

2 4810

3 1210

History GROWTH OF NATIONALISM

The second half of the 19th century witnessed growth of political consciousness and a sense of Nationalism among the IndiansThere were various factors for growth of Indian Nationalism- As a result various political associations were formed in different provinces by the educated Indians Surendranath Banerjee organized a meeting of National conference at Calcutta Ultimately the National Congress was founded in Bombay in 1885This body became the vanguard of Indian struggle for freedom The congress leaders were known as moderates because they followed a policy of prayer and petition A large number of Indian leaders had experienced in political agitation The Political situation of England was also changed Moreover increasing revolutionary activities in Maharashtra Punjab and Bengal became serious concern to the British Government In this

QUESTION1 What do you mean by Nationalism ANSWER 1 Nationalism is defined as loyalty and devotion to own nation especially a sense of national consciousnessQUESTION 2 What are the causes of nationalism ANSWER 2 There were various factors for growth of nationalism

1 Spread of western education2 The progress of vernacular press and

patriotic literature3 The economic exploitation of our

country by the colonial rulers4 International affairs

QUESTION 3 Who organized National conference in Calcutta in 1883 ANSWER 3 Surendranath BanerjeeQUESTION 4 When did Indian National Congress formANSWER 4 Indian National Congress was formed in 1885 in BombayQUESTION 5 Who were ModeratesANSWER 5 The Early Nationalists were also known as Moderates Their emergence marked

background Lord Curzon became Viceroy in India He had no respect for the Indian National Congress

the beginning of the organized national movement in India They believed in British justice and were loyal to them They followed a policy of prayer and petition They demanded constitutional reforms of our country Impotant Moderate leaders were Pherozshah Mehta Dadabhai Naorozi and Surendranath Banerjee etcQUESTION 6 What do you know about Extremism in Indian National movementANSWER 6 In the beginning of 20th century a new class of national leaders emerged in India which was different from the moderate groups They started more aggressive movement against the British empire The goal of extremists was ldquoswarajrdquo Important extremist leaders were Bal Gangadhar Tilak Lala Lajpat Rai Bipin Chandra Pal etcQUESTION 7 Mention the places which were the main centres of Revolutionary movementANSWER 7 Maharashtra Bengal and Punjab

Physics

Chapter Dimensional Analysis

(Summary)

The dimensions of a physical quantity are the powers to which the fundamental units are raised in order to obtain the derived unit of that quantit

The physical quantites lengthmasstime are represented by [L] [M] [T] resp let they are raised to powers ( dimesions) abc resp then any physical quantity can be represented by [ La Mb Tc ] Examples

1 Area area = L x B = [L] x [L] = [M0 L2 T0 ]

2 Density density = massvolume = [M][L3] = [ M L-3]

3 Velocity velocity = distancetime = [L][T] = [LT-1]HW Try to find out dimension of acceleration Acceleration = velocity timeNB One can find the SI Units Using Dimension Analysis Such as for area we have [L2] so its SI unit is m2

Biology Topic ndash Chp-1 The living world

Today we will start the first chapter the living world Here we discuss about the characteristics of living organism and what are the difference between them and nonliving substances We also discuss about the contribution of different Scientists

There are over 500000 species of plants andover a million species of animal are present on earth Some 15000 new species were discovered every yearQ1 What is a living organismbull A living organism is primarily physico -chemical material that demonstrate a high degree of complexity is capable of selfRegulation possesses a metabolism and perpetuates itself through timeQ2 What are the differences between livingand non-livingsi) Compared with non-living living organisms

have more complex organised structure and their use of energy is more controlled amp efficientii) Living things reproduce their own kind by forming new cells which contains copies of their genesiii) Each organism has some degree of homeostasisie it is able to make adjustments so that internal environment remains constantQ3 Write contributions of following Scientists i) Aristotle - One of the first theories in Biology places all living things in a hiearchieii) AV Leeuwenhoek - was the first to observe living single celled organisms under microscopeii) Carolus Linnaeus - developed the binary system for naming of organisms and classificationiii) Geregor Johann Mendel ndash discoverbasic principles of inheritanceHomework i) C Darwin ii)Schleiden

Math Trigonometric functions

1 Overviewi) Trigonometry The word lsquotrigonometryrsquo is derived from the Greek words lsquotrigonrsquo and lsquometronrsquo which means measuring the sides of a triangle An angle is the amount of rotation of a revolving line with respect to a fixed line Usually we follow two types of conventions for measuring angles ie a) Sexagesimal system b) Circular system In Sexagesimal system the unit of measurement is Degree In Circular system the unit of measurement is Radian ii) Relation between degree and radianThe ratio of circumference of a circle to its diameter is always a constant This constant ratio is a number denoted by π which is taken approximately as 227The relationship between degree amp radian measurements is as follows2 right angles = 180deg= π radians1radian = 180degπ=57deg16(approx) 1deg=π180 radianiii) Length of an arc of a circleIf an arc of length s subtends an angle θ radians at the center of a circle of radius r then s=rθiv) Area of a sector of a circleA sector is like a pizza slice of the

Q) Express the following angles in radiana) 45deg b) 40deg3730Ans a) We have 180deg=π radiansi e 45deg= πtimes45180 radian = π4 radiansb) 40deg3730= 40deg37+3060 minute= 40deg 37 +12 minute= 40deg+ 752 minute=40 + 75(2times60) degree=3258 degreeNow 180deg=π radianie 3258 degree= (πtimes325) (180times8) radians = 65π288 radiansQ) A circle has a radius of r=12 meters What is the length of an arc traced out by a 60deg angle in the center of the circleAns In this problem we know both the central angle (60deg) and the radius of the circle (12) All we have to do is plug those values into our equation and we get

s = 2π(12)(60360)s = 24π6s = 4πSo the length of an arc traced out by a 60deg angle in a circle with a radius of 12 meters equals 4π meters asymp 1257 metersQ) Find the area of the sector with a central angle 30deg and a radius of 9cmAns GivenRadius r = 9 cmAngle θ = 30degArea of the sector = θ360degtimesπr2

= 30360degtimes227times92=2121cm2

circle It consists of a region bounded by two radii and an arc lying between the radiiThe area of a sector is a fraction of the area of the circle This area is proportional to the central angle In other words the bigger the central angle the larger is the area of the sectorArea of Sector = θ2 times r2 (when θ is in radians)

Area of Sector = θ times π360 times r2 (when θ is in degrees)

COMMERCE

CLASSIFICTION OF HUMAN ACTIVITIES-ECONOMIC AND NON-ECONOMIC

Welcome to the new sessiontoday we are going to start the first chapter of Class XI The name of the chapter that we are going to start is

lsquoClassification of Human Activities ndasheconomic and non-economicrsquo

Now let us start the chapter by considering human beings and the activities they perform throughout the day

Human activities means all those activities that human beings undertake to satisfy their wants

Human wants on the other hand are the desire of human beings for goods (vegetables fruits rice etc) and services (services of doctors teachers lawyers etc) that they require to live

Now these human activities continue throughout life as human wants are unending unlimited and recurring as human beings desire for better living throughout their lives

Now human activities can be classified into two categories

Human activities

Economic activities Non-economic activities

Economic activities are

Questions1 What are human activities

Answer Human activities mean all those activities that human beings undertake to satisfy their wants

Example A man working in an office

A boy playing in the garden

2What are the characteristics of human activitiesAnswer the characteristics of human activities are as follows

Human activities are undertaken by men women and children and these activities involve human efforts

Human activities are undertaken to satisfy human wants which are unlimited

Human activities continue throughout life

Human activities are performed for both earning money and personal satisfaction

3What is economic activitiesGive example

Answer Economic activities are undertaken by human beings with the object of earning money acquiring wealth and thereby satisfying human wantsExample

Selling of goods by a shop keeper to his customer

A clinic run by a doctor Service of a teacher in school or college

undertaken by human beings with the object of earning money and acquiring wealth

These activities result in the production of economic goods and services

Example Human activities(ie working in factories officesshops) which produce direct economic benefits

Non-economic activities are inspired by human sentiments and emotions such as love for the family desire to help the poor and love for the country

Thus these human activities (eg praying playing sleeping) produce no direct economic benefits and they are also not related to earning money and acquiring wealth

4 What are the characteristics of economic activities

Answer The characteristics of economic activities are as follows

Economic motiveEconomic activities are undertaken to earn money and acquire wealth

ProductiveEconomic activities involve productiondistribution and exchange of goods and services to create wealth

Economic growthEconomic activities determine the level of economic development of a country and standard of living of its citizens

Socially desirableEconomic activities are socially desirable for society

Economic resourcesEconomic activities make use of all the economic resources such landlabourcapital etc

5 What do you mean by non-economic activitiesExampleAnswerNon-economic activities are inspired by human sentiments and emotions such as love for the family desire to help the poor and love for the countryThese activities are not undertaken for monetary gain but for onersquos satisfaction and happinessExample

a mother looks after her children

a student donates blood8 Differentiate between Economic activities and Non-economic activities

Economic activities

Non-economic activities

1to earn living and acquiring wealth2Result can be measured in terms of money

3ExampleBusinessprofession and employment

1 to obtain some satisfaction

2Result cannot be measured in terms of money

3ExampleFamily-orientedreligious socialCultural and national

BUSINESS STUDIES

BUSINESS ENVIRONMENT

Welcome to the new sessionToday we are going to start the first chapter and the name of the chapter is Business Environment

In todayrsquos world every business enterprise is a part of the society It exists and operates in association with various groups in society such as customers suppliers competitors banks and financial institutions government agencies trade unions media and so on All these groups influence the functioning of business in one way or the other They constitute the environment of businessConcept of Business Environment

The term lsquobusiness environmentrsquo refers to the sum total of all individuals institutions and other forces that lie outside a business enterprise but that may influence its functioning and performance

The main features of business environment

Totality of External forces General and Specific forces Interrelatedness Complexity Dynamic Uncertainty Relativity

The Interrelation between business and its environment

The business enterprise is an open system It continuously interacts with its environment It takes inputs

Prepare the following questions from todayrsquos assignment

1 What do you mean by business environment

The term lsquobusiness environmentrsquo means the aggregate of all forces factors and institutions which are external to and beyond the control of an individual business enterprise but they may influence its functioning and performance Business environment is the macro framework within which a business firm a micro unit operates It consists of several interrelated and interacting elements

2 Explain the main features of business environment in brief

Totality of External forces-Business environment is the sum total of all things external to a business environment

General and Specific forces-It includes both the forces general forces are the economic social political legal and technological conditions which indirectly influence all business enterprise Specific forces are the investors customers competitors and suppliers which influence individual enterprise directly

Interrelatedness-Different elements of environment are interrelated for an example growing awareness for health care has increased the demand for health foods

Complexity- Business environment id

(such as raw materials capital labour energy and so on) from its environment transforms them into goods and services and sends them back to the environment

Fig 1 Business Environment Relationship

complex in nature as the elements keep on changing example economic technological and other forces changes in demand for a product and service

Dynamic-Business environment is not static it keeps on changing

Uncertainty- Itrsquos very difficult to predict future events such as technology and fashion which occur fast and frequently

Economics Basic Economic ConceptsSub topic

Microeconomics and

Macroeconomics

Welcome to the new sessiontoday we are going to start the first chapter of Class XI The name of the chapter that we are going to start is Basic Economic concepts

Now Economics covers the study of human activities Human activities are those activities which are performed by humans to satisfy their wants

Thus Human wants are unlimited and therefore economic activities such as production exchange and consumption are needed in order to satisfy those wants

The study of economics is divided largely in two parts which areMicroeconomics and Macroeconomics

SUBJECT- MATTER OF ECONOMICS

MICROECONOMICS MACROECONOMICS

Questions1Who has coined the words micro and macro economics

Answer Ranger Frisch coined the words lsquomicrorsquo and lsquomacrorsquo in 1933 to denote the two branches of economic theory namely microeconomics and macroeconomics

2What is microeconomicsAnswer It is the study of behaviour of individual decision ndash making unit such as consumers firms etc

3 What is macroeconomicsAnswer Macroeonomics is the study of overall economic phenomena like employment national income etc

4 What is the importance of microeconomicsAnswer

Microeconomics helps in formulating economic policies which enhance productive efficiency and results in greater social welfare

It helps the government in formulating correct price policies

It explains the working of a capitalistic economy where individual units(producers and consumers ) are free to take their own decision

Micro means a small part in

microeconomics we do not study the whole economy Hence we study an individual consumer and his or her choices and a producer and his or her profit maximizing decisions in the market Thus it does not mirror what happens in the economy as a whole

Macroeconomics on the other hand studies the economy as a whole It is concerned with aggregate and depicts the entire picture of the economyMacroeconomics deals with the national income aggregate investment aggregate consumption etc

Features of Microeconomics It deals with small

parts of the country Hence it looks at

individual consumers firms and industries

It deals with individual income consumption and savings

It studies the determination of price of any product or factors of production

It deals with the working of market via the price mechanism which is nothing but the determination of price and quantity of a commodity by the forces of demand and supply

Features of Macroeconomics

It deals with the study of the economy as a whole

It is concerned with

5 Give a limitation of microeconomics Microeconomics fails to explain the

functioning of an economy as a whole It cannot explain unemployment illiteracy and other problems prevailing in the country

6 What is the importance of macroeconomics It gives overall view of the growing

complexities of an economic system It provides the basic and logical

framework for formulating appropriate macroeconomic policies (eg for inflation poverty etc )to direct and regulate economy towards desirable goals

7What is the limitation of macroeconomics It ignores structural changes in an

individual unit of the aggregate

8 Differentiate between Microeconomics and Macroeconomics

Microeconomics Macroeconomics

the study of aggregates

National income aggregate savings and aggregate investments are major concepts dealt within macroeconomics style

It studies the determination of general price levels

It investigates into the problem of unemployment and the achievement of employment

It studies the aspect of decision making at the aggregate and national levels

It includes all growth theories whether related to developed or developing economies it also includes the study of economic systems and the working of the economy under different systems

Note Both Micro and macro economics are complementary and should be fully utilized for proper understanding of an economy

1It studies economic aspect of an individual unit2It deals with individual incomeConsumption and savings

3 It facilitates determination of price of any product or factors of production

4 Itrsquos scope is narrow and restricted to individual unit

1It studies the economy as a whole

2It deals with the national income aggregate consumption and aggregate savings3 It facilitates determination of general price level in an economy

4 Itrsquos scope is wide as it deals with economic units on the national level

ACCOUNTS

Introduction to Accounting and Book-keeping

Today I am going to share you the meaning of Accounting and Book-keeping and its related terms bullAccounting bullBook Keeping bullAccountsbullTypes Of Accounts bullAccounting Cycle

bull Meaning of accounting

Ans ) Accounting is the art and science of recording classifying and summarising monetary transactions

bull Meaning of Book-keeping

Ans) Bookkeeping is the art of recording business transactions with the view of having a permanent record of them and showing their effect on wealth

bull Meaning of account

Ans) The term account means a record of

business transactions concern a particular person of firm asset or income or expense It is a summarised record of all transactions which take place in an accounting year

bull Types of accountsPersonal accounts ndash Personal accounts relating

to person and Organisation are known as personal accounts Example Ramrsquos Account ABC amp Co Account etc

Real account - The accounts related to tangible and intangible assets are called real accounts Example Cash Account Furniture Account etc

Nominal account- Accounts related to expenses losses incomes and gains are known as nominal accounts Example Wages Account Salary Account Discount Account etc

bull Accounting cycle Accounting cycle refers to a complete sequence of accounting activities It begins with recording of transactions and ends with the preparation of a balance sheet

Chemistry TopicAtomic Structure

Thomsonrsquos atomic modelThomson (1898) was the first to propose the model of an atomHe proposed that an atom can be regarded as a uniform sphere of positive electricity in which requisite number of electrons are embedded evently to neutralize the positive chargeThis is just like plums embedded in a pudding or seeds evently distributed in red spongy mass of a watermelonThis model of atom is known as ldquoPlum-Pudding modelrdquo or

Q1)What is the fundamental constituents of atomAns Electron Proton and neutrons are the fundamental constituents of atomQ2)What is the value of fundamental unit of electricityAnsThe charge carried by one electron is sad to be the fundamental unit of electricityIts magnitude is 48times10-10esuOr 1602times10-19C Q3)Name the element containing no neutronAnsOrdinary hydrogen atom or protium 1H1

Types of AccountPersonal AccountReal AccountNominal AccountBalance Sheet (opening)

ldquowatermelon modelrdquoThis model could explain the electrical neutrality of an atom but failed to explain the result of scattering experiment carried out by Rutherford in 1911So it was rejected ultimately

Q4)Why is an electron called universal particleAns Itrsquos mass and Charge are independent of its source

EVS Chapter 1 ndash Modes of Existence

Modes of existence When one speaks normally about the mode of existence of some group or individual one refers to their customs their mode of being their ethology their habitat in some way their feeling for a placeDifferent modes of exixtence are ndash

1 Hunting ndashGathering2 Pastoral3 Agricultural4 Industrial

1 Hunting and gathering Hunting and gathering mode of existence is characterized by obtaining food from hunting wild animals including fishing and gathering wild plants From their earliest days the hunter-gatherer diet included various grasses tubers fruits seeds and nuts Lacking the means to kill larger animals they procured meat from smaller game or through scavenging

Societies that rely primarily or exclusively on hunting wild animals fishing and gathering wild fruits berries nuts and vegetables to support their diet are called hunting and gathering societies

At least this used to be practice of human beings before agriculture is invented As their brains evolved hominids developed more intricate knowledge of edible plant life and growth cycles

Q) Write the features of Hunting ndash gathering societiesAns - There are five basic characteristics of hunting and gathering societies

i The primary institution is the family which decides how food is to be shared and how children are to be socialized and which provides for the protection of its members

ii They tend to be small with fewer than fifty members

iii They tend to be nomadic moving to new areas when the current food supply in a given area has been exhausted

iv Members display a high level of interdependence

v Labor division is based on sex men hunt and women gather

Political Science

Introduction to political science

Political science occasionally called politology is a social science which deals with systems of governance and the analysis of political activities political thoughts associated constitutions and political behaviorThe study of political science involves the study of both the

Answer the following questions-1 What is political science

Political science occasionally called politology is a social science which deals with systems of governance and the analysis of political activities political thoughts associated constitutions and political behavior

2 Short notes-

traditional and modern theories of politicsTraditionalClassical political sciencepolitical theory-Traditional political science is the study of politics before Second World War The methodology to study Politics was traditional (legal formaletc) the definition of politics traditional (Politics begins and end with state)area of study (constitution state machinery)was traditionalModern Political scienceModern political theory-Modern Political Theory critically examines the contemporary state of political theory making an assessment of the achievement and limitations of the Behavioural Revolution in its totality and reviews objectively the major paradigms and conceptual frameworks adopted by the disciplineContemporary attempts at the development of an integrated political theory involving the use of both traditional and modern concepts approaches and theories-Around late 1960s several political scientists realized the importance of both the traditional political theory and modern Political theory They began building an integrated theory of politics involving a systematic mixture of traditional and modern studies of politics It was held that the study of a complex and vast field like politics needs both traditional as well as

Classical political theory Modern Political theory

Homework-Learn

modern concepts and approaches for studying itrsquos all aspects

Subject Eng Literature (The Tempest ndash William Shakespeare) Topic Act I Scene 1 Lines 1 to 32 (Line 32 ndash Gonzalo hellip If he be not born to be hanged our case is miserable) Date 13th April 2020 (3rd Period)

[Students should read the original play and also the paraphrase given in the school prescribed textbook]Summary Questions amp Answers

[SUMMARY OF THE ENTIRE SCENE]

o The play starts with the scene of a severe storm at sea Alonso (King of Naples) Sebastian (Alonsorsquos brother) Ferdinand (Alonsorsquos son) Gonzalo Antonio (the usurping Duke of Milan) are in a ship in the midst of the storm

o The mariners are trying their best to control the vessel from running aground and are totally following the orders of their Master the Boatswain They have scant success

o The mariners become extremely unhappy and annoyed when most of the passengers arrive on the deck thereby hampering their effort to save the ship There is serious confrontation between them and the passengers who are part of the Kingrsquos entourage

o The mariners could not save the ship

SUMMING-UP

(i) Vivid description of the scene which gives a realistic description of terror and confusion of a tropical storm

(ii) Shows Shakespearersquos accuracy of knowledge in describing the naval operations and also matters of seamanship

(iii) The opening scene justifies the title ndash The Tempest

UNANSWERED QUESTIONS

(i) The King always travels with his entire fleet including his soldiers Where

(1) GONZALO Nay good be patient (Line 15-26)BOATSWAIN When the sea is Hence What cares these

roarers for the name of the king To cabin silence Trouble us not

GONZALO Good yet remember whom thou has aboardBOATSWAIN None that I more love than myself You are a

councillor if you can command these elements to silence and work

the peace of the present we will not hand a rope more use your authority If you cannot give thanks you have

lived so long and make yourself ready in your cabin for the mischance of the hour if it so hap [To the Mariners]

Cheerly good hearts [To Gonzalo] Out of our way I say

(a) To whom is the boatswain speaking What does he mean by lsquoNone that I more love than myselfrsquo

The Boatswain is speaking to Gonzalo the honest old councilor of the Duke of MilanBy using the words ndash lsquoNone that I love more than I love myselfrsquo means that for the Boatswain nobody is dearer to him than his own life

(b) What were the conditions that made the boatswain react in this way

The Boatswain reacts in this way because the storm is at sea and Alonso King of Naples Sebastian his brother Ferdinand his son Gonzalo Antonio the usurping Duke of Milan on board are in distress and in panic Thus they have rushed to the deck interrupting the work of the mariners

(c) What hope does Gonzalo take from the attitude of the boatswain

The insolent and authoritative attitude of Boatswain makes Gonzalo feel comforted He tells that there are no signs that the Boatswain will be drowned But his facial appearance and attitude shows that he is destined to die on land by hanging which in effect means that all on board will be saved Otherwise all the persons on board are doomed

(d) How can they lsquomake yourself ready in your cabinrsquo For what were they asked to make ready themselves

In order to make themselves ready in their cabin the

were the other ships

(ii) Why was the ship in that area Where was it coming from or going where

(iii) The ship broke apart What happened to those who were in the ship

passengers on board must prepare for death which they will possibly soon have to meetThey can retire to their cabins and offer prayers to the Almighty to save them from drowning

(e) What does the boatswain say when he is asked to be patient What does he order to the royal party

When the boatswain is asked to be patient and remain calm he says that he will be patient only when the storm will be over and the sea will be calm but as long as the storm blows and there is danger to the ship he cannot think of being patient He orders the royal party to go to the cabin and leave the mariners to their work

(2) GONZALO I have great comfort from this fellow (Line 27-36)

Methinks he hath no drowning mark upon him his complexion is perfect

gallows Stand fast good Fate to his hanging Make the rope of his destiny our cable for our own doth little advantage If he be not born to be hanged our case is miserable

(a) Why does Gonzalo regard the Boatswain in the midst of danger

In the midst of danger Gonzalo regards the boatswain because he feels that the Boatswain is a source of comfort and is bent upon to do his work sincerely which in this case is saving the ship and its passengers from the severest of raging storm

(b) What reasons does Gonzalo give when he says that none in the ship will die of drowning

Gonzalo is almost sure that none in the ship will die by drowning His says that there is no mark on the face of the boatswain that indicates that he will die by drowning On the other hand the lines on his face are strong indications that he will be hanged to death Therefore there shall be no danger of the shiprsquos sinking

(c) Explain the following ldquoStand fast good Fate to his hanging Make the rope of his destiny our cable for our own doth little advantage If he be not born to be hanged our case is miserablerdquo

The stated lines mean that if the will of destiny is to be carried out then the ship will not get wrecked and all the passengers will be saved The safety of the passengers therefore depends upon the will of fate being carried out in the case of the boatswain If however the boatswain is not to die by hanging then the passengers are also very unsafe because in that case the ship is likely to sink

(d) What order does the Boatswain give to the sailors

when he re-enters What does he say about the crying of the fellows inside the cabin

The boatswain orders the sailors to bring the topmast lower and bring the ship close to a stationary position with the help of the main sail He says that the fellows inside the cabin are moaning and crying in their distress louder than his voice and louder even than the roaring of the storm

Class XII (ScienceCommerceHumanities) Subject Topic Summary Execution

Computer Science

PropositionalLogic

Propositional logic is a procedure to provide reasoning through statementProposition A ststement that results in True or False is said to be proposition There are two types of propositionSimple proposition amp compound propositionSimple proposioton A simple proposition is one that is not a part of any other proposition Such sentential form of proposition is symbolized with english letters in short For example Ram is a claver student (TrueFalse)Where do you live (Not in True or False)Grapes are sweet (TrueFalse)It rains today (TrueFalse)Here we can see some statements anwer would be true or false but some staements answer can not give in terms of true or false Thus the sentences which can be answered in true or false are known as simple propositionAssigning propositon to a variableThe general syntax to assign propostion to a variable is as followsVariable = Simple propositonFor example A=Ram is a clever studentB= Grapes are sweetC= it rains todayCompound proposition

helliphellipto be continued in next classhelliphellipMath Relation Relation If A and B are two non-empty sets

then a relation R from A to B is a subset of AxB If R A x B and (a b) R then we say that a sube isinis related to b by the relation R written as aRbeg Let A be the set of students of class XII and B be the set of students of class XI Then some of the examples of relation from A to B arei) (a b) AXB a is brother of bisinii) (a b) AXB age of a is more than age of isinb Types of relation In this section we would like to study different types of relations We know that a relation in a set A is a subset of A times A Thus the empty set φ and A times A are two extreme relations For illustration consider a relation R in the set A = 1 2 3 4 given by R = (a b) a ndash b = 10 This is the empty set as no pair (a b) satisfies the condition a ndash b = 10 Similarly R = (a b) | a ndash b | ge 0 is the whole primeset A times A as all pairs (a b) in A times A satisfy | a ndash

Example 1 Let A be the set of all students of a boys school Show that the relation R in A given by R = (a b) a is sister of b is the empty relation and R = (a b) the primedifference between heights of a and b is less than 3 meters is the universal relationSolution Since the school is boys school no student of the school can be sister of any student of the school Hence R = φ showing that R is the empty relation It is also obvious that the difference between heights of any two students of the school has to be less than 3 meters This shows that R = A times A is primethe universal relation Example 2 Show that the relation R in the set 1 2 3 given by R = (1 1) (2 2) (3 3) (1 2) (2 3) is reflexive

b | ge 0 These two extreme examples lead us to the following definitionsDefinition 1 A relation R in a set A is called empty relation if no element of A isrelated to any element of A ie R = φ A times AsubDefinition 2 A relation R in a set A is called universal relation if each element of A is related to every element of A ie R = A times A Both the empty relation and the universal relation are some times called trivial relation Definition 3 A relation R in a set A is called(i) reflexive if (a a) R for every a Aisin isin(ii) symmetric if (a1 a2) R implies that (aisin 2a1)

R for all aisin 1 a2 Aisin(iii) transitive if (a1 a2) R and (aisin 2 a3) R isinimplies that (a1 a3) R for all aisin 1 a2 a3 AisinDefinition 4 A relation R in a set A is said to be an equivalence relation if R is reflexive symmetric and transitive

but neither symmetric nor transitiveSolution R is reflexive since (1 1) (2 2) and (3 3) lie in R Also R is not symmetric as (1 2) R but (2 1) isin notinR Similarly R is not transitive as (1 2) R and (2 3) R but (1 3) R isin isin notinExample 3 Show that the relation R in the set Z of integers given byR = (a b) 2 divides a ndash b is an equivalence relationSolution R is reflexive as 2 divides (a ndash a) for all a Z isinFurther if (a b) R then 2 divides a isinndash b Therefore 2 divides b ndash a Hence (b a) R which shows that R is isinsymmetric Similarly if (a b) R and (b c) R isin isinthen a ndash b and b ndash c are divisible by 2 Now a ndash c = (a ndash b) + (b ndash c) is even (Why) So (a ndash c) is divisible by 2 This shows that R is transitive Thus R is an equivalence relation in ZExample 4 Let L be the set of all lines in a plane and R be the relation in L defined as R = (L1 L2) L1 is perpendicular to L2 Show that R is symmetric but neither reflexive nor transitiveSolution R is not reflexive as a line L1 can not be perpendicular to itself ie (L1 L1) R notinR is symmetric as (L1 L2) Risin

L1 is perpendicular to L2rArr L2 is perpendicular to L1rArr (L2 L1) RrArr isin

R is not transitive Indeed if L1 is perpendicular to L2 and L2 is perpendicular to L3 then L1 can never be perpendicular to L3 In fact L1 is parallel to L3 ie (L1 L2) R isin(L2 L3) R but (L1 L3) Risin notin

Chemistry Solid state Characteristics if Solids(i)The particles are locked in fixed positions they are unable to change their relative positions and this brings a definite shape and volume of a solid(ii)In a solid the constituent particles are held by strong forces of attractionThe forces of attraction may be bonding or non bonding(iii)The constituent particles in a solid pack together as closely as possibleoccupying most of the available space within the solidThus the empty space in a solid is very smallThis makes a solid highly rigid and nearly incompressibleThis also explains why a solid has high density and exhibits slow diffusionClassification of Solids

Q1)Define Crystalline solids AnsA Solid that has a definite geometrical shape and a sharp melting pointand whose constituent particles (atomsmolecules or ions) are arranged in a long range order of definite pattern extending throughout the solid is called a crystalline solidExNaClQ2)Define Amorphous solids AnsA solid that does not have a definite shape and a sharp melting pointand whose constituent particles (atomsmolecules or ions) are not arranged in a definite pattern is called an amorphoussolid

Crystalline solidsAmorphous solids

ExGlassRubberQ3)Classify Crystalline Solids Crystalline Solids

Physics Coloumbrsquos Law (Summary)

Before Going Into Coloumbrsquos Law We Will First Learn What is Charge Properties of Charge and Always remember that charge is quantized ie a body always have static charge of magnitude equal to some integral multiple of fundamental electronic charge e= 16 x 10- 19 C

Charge is the property of matter that causes it to produce and experience electrical and magnetic effects The study of the electrical charges at rest is called electrostatics When both electrical and magnetic effects are present the interaction between charges is referred to as electromagnetic

There exist two types of charges in nature positive and negative Like charges repel and unlike charges attract each other

The type of charge on an electron is negative The charge of a proton is the same as that of an electron but with a positive sign In an atom the number of electrons and the number of protons are equal The atom is therefore electrically neutral If one or more electrons are added to it it becomes negatively charged and is designated as negative ion However if one or more electrons are removed from an atom it becomes positively charged and is called a positive ion

The excess or deficiency of electrons in a body gives the concept of charge If there is an excess of electrons in a body it is negatively charged And if there is deficiency of electrons the body becomes positively charged Whenever addition or removal of electrons takes places the body acquires a charge

The SI Unit of charge is coulomb (C) In SI units the current is a fundamental quantity having a unit of ampere (A) The unit of charge is defined in terms of the unit of current Thus one coulomb is the charge transferred in one second across the section of a wire carrying a

Ionic SolidsMetallicSolids

Molecular Solids

current of one ampere

As q = It we have1 C = (1 A) (1 s)

The dimensions of charge are [A T]

Properties of Charge

(1) Quantization of Charge Electric charge can have only discrete values rather than any value That is charge is quantized The smallest discrete value of charge that can exist in nature is the charge on an electron given as

e = plusmn 16 x 10- 19 C

This is the charge attained by an electron and a protonA charge q must be an integral multiple of this basic unit That is

Q = plusmn ne where n = 1 2 hellip

Charge on a body can never be (frac12)e (23)e or 57e etcWhen we rub a glass rod with silk some electrons are transferred from the rod to the silk The rod becomes positively charged The silk becomes negatively charged The coulomb is a very large amount of charge A typical charge acquired by a rubbed body is 10 - 8 C

Biology Reproduction in organisms

Welcome to this new session 2020-21Today in this first chapter we mainly discuss about reproduction types needs and life span of some organismsWe also discuss about difference between sexual and asexual reproduction

Q1 What is reproductionReproduction is defined as a biological processin which an organism gives rise to young onessimilar to itselfQ2 What are the needs of reproductionbulli) Reproduction maintain life on earthii) It enables the continuity of the species generation after generationiii) It creates genetic variation among populationsQ3 Define Life span and write some orgnisms life spanbull Life span is the period from birth to

the natural death of an organism- OrganismsLife span1 Butterfly 1 - 2 weeks2 Fruit fly 30 days3Dog 10-13 years4 Rose5-7 years5 Tortoise100-150 years6 Banyan Tree -200 - 250 yearsQ4 Reproduction is of two types in case ofanimals but in case of plants vegetative propagation is also present

Asexual Reproduction Sexual Reproductioni) Always uniparentalii) Gametes are not involvediii) Only mitotic division involvediv) Somatic cells of parents are involvedv) Offsprings are genetically similar to the parents

i) Usually biparentalii) Gametes are involvediii) Meiosis occurs during gametogenesis Mitosis occurs after fertilisationiv) Germ cells of the parents are involvedv) offsprings are genetically different from the parents

COMMERCE BUSINESS ENVIRONMENT

Welcome to the new sessiontoday we are going to start the first chapter of Class XII The name of the chapter is Business Environment

Already many of you have got some idea about the word business environment form the first chapter of business studies in class XI

In todayrsquos world every business enterprise is a part of the society It exists and operates in association with various groups in society such as customers suppliers competitors banks and financial institutions government agencies trade unions media and so on All these groups influence the functioning of business in one way or the other They constitute the environment of businessConcept of Business Environment

The term lsquobusiness environmentrsquo refers to the sum total of all individuals institutions and other forces that lie outside a business enterprise but that may influence its functioning and performance

The main features of business environment Totality of External forces General and Specific forces Interrelatedness Complexity Dynamic Uncertainty

Prepare the following questions from todayrsquos assignment

2 What do you mean by business environment

The term lsquobusiness environmentrsquo means the aggregate of all forces factors and institutions which are external to and beyond the control of an individual business enterprise but they may influence its functioning and performance Business environment is the macro framework within which a business firm a micro unit operates It consists of several interrelated and interacting elements

2 Explain the main features of business environment in brief

Totality of External forces-Business environment is the sum total of all things external to a business environment

General and Specific forces-It

Relativity

The Interrelation between business and its environment

The business enterprise is an open system It continuously interacts with its environment It takes inputs (such as raw materials capital labour energy and so on) from its environment transforms them into goods and services and sends them back to the environment

Fig 1 Business Environment Relationship

includes both the forces general forces are the economic social political legal and technological conditions which indirectly influence all business enterprise Specific forces are the investors customers competitors and suppliers which influence individual enterprise directly

Interrelatedness-Different elements of environment are interrelated for an example growing awareness for health care has increased the demand for health foods

Complexity- Business environment id complex in nature as the elements keep on changing example economic technological and other forces changes in demand for a product and service

Dynamic-Business environment is not static it keeps on changing

Uncertainty- Itrsquos very difficult to predict future events such as technology and fashion which occur fast and frequently

Business Studies

Human Resources Management

Human resource of an organisation are the aggregate of knowledge skills attitudes of people working in it

The management system which deals with human resources is called human resource management

Features of HRMbullComprehensive functionbullPeople-oriented

Question1) What do you mean by human

resource management Answer) Human resource management may be defined as that field of Management which has to do with planning organising and controlling the functions of procuring developing maintaining and utilising the labour force

bullAction oriented bullPervasive function bullContinuous function

2) Explain the features of HRM in brief

Answer)bullHuman Resource Management is concerned with managing people at work bull Human Resource Management is concerned with employees which bring people and organisations together so that the goals of each are met bullHuman resource management considered every employees as an individual and also promote their satisfaction and growth bull Human resource management is inherent in all organisations and at all levelsbullManagement of human resources are ongoing on never ending process which requires a constant alertness and Awareness of human relations

3) ldquoHR function is said to be pervasiverdquowhy

Answer) Human resource management is required in all organisations whether it is private or government organisations armed forces sports organisations etc It permeatsall the functional areas like production marketing finance research etc This from this feature of human resource management it can be said that it is pervasive in nature

Economics Demand Q1DEFINITION OF DEMANDIn economics demand is the quantity of a good that consumers are willing and able to purchase at various prices during a given period of timeQ2DEMAND CURVEIn economics a demand curve is a graph depicting the relationship between the price of a certain commodity and the quantity of that commodity that is demanded at that pricQ3LAW OF DEMANDIn microeconomics the law of demand states that conditional on all else being equal as the price of a good increases quantity demanded decreases conversely as the price of a good decreases quantity demanded increasesQ4ASSUMPTION of LAW OF DEMAND(i)No change in price of related commodities(ii) No change in income of the consumer(iii) No change in taste and preferences customs habit and fashion of the consumer( No expectation regarding future change in priceQ5MARKET DEMAND SCHEDULEIn economics a market demand schedule is a tabulation of the quantity of a good that all consumers in a market will purchase at a

given price At any given price the corresponding value on the demand schedule is the sum of all consumersrsquo quantities demanded at that priceQ6INDIVIDUAL DEMAND SCHEDULEIndividual demand schedule refers to a tabular statement showing various quantities of a commodity that a consumer is willing to buy at various levels of price during a given period of timeQ7 FACTORS AFFECTING INDIVIDUAL DEMAND FOR A COMMODITY

The factors that influence a consumerrsquos decision to purchase a commodity are also known as determinants of demand The following factors affect the individual demand for a commodity1 price of the commodity2 price of related goods3 income of buyer of the commodity4 tastes and preferences of the buyer1 Price of the CommodityYou must have observed that when price of a commodity falls you tend to buy more of it and when its price rises you tend to buy less of it when all other factors remain constant (lsquoother things remaining the samersquo) In other words other things remaining the same there is an inverse relationship between the price of a commodity and its quantity demanded by its buyers This statement is in accordance with law of demand which you will study in the later part of this lesson Price of a commodity and its quantity demanded by its buyers are inversely related only when lsquoother things remain the samersquo So lsquoother things remaining the samersquo is an assumption when we study the effect of changes in the price of a commodity on its quantity demanded2 Price of Related goodsA consumer may demand a particular good But while buying that good heshe also asks the price of its related goods Related goods can be of two types-(i) Substitute goods(ii) Complementary goods While purchasing a good prices of its substitutes and complements do affect its quantity purchased(i) Price of Substitute Goods Substitute goods are those goods which can easily be used in place of one another for satisfaction of a particular want like tea and coffee An increase in price of substitute good leads to an increase in demand for the given commodity and a decrease in price of substitute good leads to a decrease in demand for the given commodity It means demand for a given commodity is directly affected by change in price of substitute goods For example if price of coffee increases the demand for tea will rise as tea will become relatively cheaper in comparison to coffee(ii) Price of Complementary goods Complementary goods are those goods which are used together to satisfy a particular want like car and petrol An increase in the price of complementary goods leads to a decrease in demand for the given commodity and a decrease in the price of complementary goods leads to an increase in demand for the given commodity For example if price of petrol falls then the demand for cars will increase as it will be relatively cheaper to use both the goods together So demand for a given commodity is inversely affected by change in price of complementary goods3 Income of the Buyer of CommodityDemand for a commodity is also affected by income of its buyer However the effect of change in income on demand depends on the nature of the commodity under consideration In case of some goods like full cream milk fine quality of rice (Basmati rice) etc demand for these commodities increases when income of the buyer increases and

demand for these commodities decreases when income of the buyer decreases Such goods whose demand increases with the increase in income of the buyer are called normal goods But there are some goods like coarse rice toned milk etc whose demand decreases when income of buyer increases and their demand increases when income of the buyer decreases Such goods whose demand decreases with the increase in income of the buyer are called inferior goods Suppose a consumer buys 10 Kgs of rice whose price is ` 25 per Kg He cannot afford to buy better quality of rice because the price of such rice is ` 50 per Kg The consumer is spending ` 250 per month on the purchase of rice Now if income of the consumer increases and he can afford ` 350 on purchase of 10 Kg of rice Now he can afford to buy some quantity of rice say 6 Kgs whose price is ` 25 per Kg and may buy 4 Kgs of rice whose price is ` 50 per Kg Thus he will buy 10 Kgs of rice by spending ` 350 per month Therefore we may conclude that demand for normal goods is directly related to the income of the buyer but demand for inferior goods is inversely related to the income of the buyer4 Tastes and Preferences of the BuyerThe demand for a commodity is also affected by the tastes and preferences of the buyers They include change in fashion customs habits etc Those commodities are preferred by the consumers which are in fashion So demand for those commodities rises which are in fashion On the other hand if a commodity goes out of the fashion its demand falls because no consumer will like to buy it(5) Number of Buyers in the Market(Population)Increase in population raises the market demand whereas decrease in population reduces the market demand for a commodity Not only the size of population but its composition like age (ratio of males females children and old people in population) also affects the demand for a commodity It is because of needs of children young old male and female population differs(6) Distribution of Income and WealthIf the distribution of income and wealth is more in favour of the rich demand for the commodities preferred by the rich such as comforts and luxuries is likely to be higher On the other hand if the distribution of income and wealth is more in favour of poor demand for commodities preferred by the poor such as necessities will be more(7) Season and Weather ConditionsThis is generally observed that the demand for woolens increases during winter whereas demand for ice creams and cold drinks increases during summer Similarly market demand for umbrellas rain coats increases during rainy seasonQ8 REASONS FOR OPERATION OF LAW OF DEMAND WHY DEMAND CURVE SLOPES DOWNWARDNow we will try to explain why does a consumer purchase more quantity of a commodity at a lower price and less of it at a higher price or why does the law of demand operate ie why does the demand curve slope downwards from left to right The main reasons for operation of law of demand are1 Law of Diminishing Marginal UtilityAs you have studied earlier law of diminishing marginal utility states that as we consume more and more units of a commodity the utility derived from each successive unit goes on decreasing The consumer will be ready to pay more for those units which provide him more utility and less for those which provide him less utility It implies that he will purchase more only when the price of the commodity falls2 Income Effect

When price of a commodity falls purchasing power or real income of the consumer increases which enables him to purchase more quantity of the commodity with the same money income Let us take an example Suppose you buy 4 ice creams when price of each ice cream is ` 25 If price of ice creams falls to ` 20 then with same money income you can buy 5 ice creams now3 Substitution EffectWhen price of a commodity falls it becomes comparatively cheaper as compared to its substitutes (although price of substitutes has not been changed) This will lead to rise in demand for the given commodity For example if coke and Pepsi both are sold at ` 10 each and price of coke falls Now coke has become relatively cheaper and will be substituted for Pepsi It will lead to rise in demand for coke4 Change in Number of BuyersWhen price of a commodity falls some old buyers may demand more of the commodity at the reduced price and some new buyers may also start buying this commodity who were not in a position to buy it earlier due to higher price This will lead to increase in number of buyers when price of the commodity falls As a result demand for the commodity rises when its price falls5 Diverse Uses of a CommoditySome commodities have diverse uses like milk It can be used for drinking for sweet preparation for ice cream preparation etc If price of milk rises its use may be restricted to important purpose only This will lead to reduction in demand for other less important uses When price of milk falls it can be put to other uses also leading to rise n demand for itQ9 EXCEPTIONS TO THE LAW OF DEMANDYou have studied in law of demand that a buyer is willing to buy more quantity of a commodity at a lower price and less of it at a higher price But in certain circumstances a rise in price may lead to rise in demand These circumstances are called Exceptions to the Law of Demand Some important exceptions are1 Giffen GoodsGiffen goods are special type of inferior goods in which negative income effect is stronger than negative substitution effect Giffen goods do not follow law of demand as their demand rises when their price rises Examples of Giffen goods are jowar and bajra etc2 Status Symbol GoodsSome goods are used by rich people as status symbols eg diamonds gold jewellary etc The higher the price the higher will be the demand for these goods When price of such goods falls these goods are no longer looked at as status symbol goods and tehrefore therir demand falls3 NecessitiesCommodities such as medicines salt wheat etc do not follow law of demandbecause we have to purchase them in minimum required quantity whatever their price may be4 Goods Expected to be ScarceWhen the buyers expect a scarcity of a particular good in near future they start buying more and more of that good even if their prices are rising For example during war famines etc people tend to buy more of some goods even at higher prices due to fear of their scarcity in near future

Political Science

Constitution of India-The

Preamble

The preamble-

Preamble-

The preamble is the most precious part of the constitution We the people of India having solemnly resolved to constitute India into a Sovereign Socialist Secular Democratic Republic and to secure to all its citizensA preamble is an introductory and expressionary statement in a document that explains the documents purpose and underlying philosophy When applied to the opening paragraphs of a statute it may recite historical facts pertinent to the subject of the statuteNature and purpose of the constitution-Purpose of the Constitution dictates permanent framework of the government to form a more perfect union to establish justice and ensure peace of thenationconstitution provide principles how the government can run itself following the rules and laws written in the constitution of each state keeps them balanced

Answer the following questions-

1 What is preambleA preamble is an introductory and expressionary statement in a document that explains the documents purpose and underlying philosophy2 What is the nature and

purpose of the constitutionConstitution dictatespermanent framework of the government to form a more perfect union to establish justice and ensure peace of the nation

Homework-Learn

Accounts Compatibilty mode

1MEANING OF PARTNERSHIPPartnership is a form of business organisation where two or more persons join hands to run a business They share the profits and losses according to the agreement amongst them According to the Indian Partnership Act 1932 ldquoPartnership is relation between persons who have agreed to share profits of a business carried on by all or any one of them acting for allrdquo For example one of your friends has passed class XII from National Institute of Open Schooling (NIOS) and wants to start a business Heshe approaches you to join in this venture Heshe wants you to contribute some money and participate in the business activities Both of you if join hands constitute a partnership2CHARACTERISTICS1048698 Agreement A partnership is formed by an agreement The agreement may be either oral or in writing It defines the relationship between the persons who agree to carry on business It may contain the terms of sharing profit and the capital to be invested by each partner etc The written agreement is known as partnership deed1048698 Number of persons There must be at least two persons to form a partnership

The maximum number of partners in a partnership firm can be 50 according toCompanies Act 20131048698 Business The Partnership is formed to carry on business with a purpose of earning profits The business should be lawful Thus if two or more persons agree to carry on unlawful activities it will not be termed as partnership1048698 Sharing Profits The partners agree to share profits in the agreed ratio In caseof loss all the partners have to bear it in the same agreed profit sharing ratio10486981048698Mutual Agency Every partner is an agent of the other partners Every partner can bind the firm and all other partners by hisher acts Each partner will be responsible and liable for the acts of all other partners10486981048698Unlimited liability The liability of each partner except that of a minor is unlimited Their liability extends to their personal assets also If the assets of the firm are insufficient to pay off its debts the partnersrsquo personal property can be used to satisfy the claim of the creditors of the partnership firm10486981048698Management All the partners have a right to mange the business However they may authorize one or more partners to manage the affairs of the business on their behalf10486981048698Transferability of Share No partner can transfer hisher share to any one including hisher family member without the consent of all other partners3PARTNERSHIP DEEDAgreement forms the basis of partnership The written form of the agreement is which a document of partnership is It contains terms and conditions regarding the conduct of the business It also explains relationship between the partners This document is called partnership deed Every firm can frame its own partnership deed in which the rights duties and liabilities of the partners are stated in detail It helps in settling the disputes arising among the partners during the general conduct of business 4CONTENTS OF PARTNERSHIP DEEDThe partnership deed generally contains the following (i) Name and address of the partnership firm(ii) Nature and objectives of the business(iii) Name and address of each partner(iv) Ratio in which profits is to be shared(v) Capital contribution by each partner(vi) Rate of Interest on capital if allowed(vii) Salary or any other remuneration to partners if allowed(viii) Rate of interest on loans and advances by a partner to the firm(ix) Drawings of partners and interest thereon if any(x) Method of valuation of goodwill and revaluation of assets and liabilities on the reconstitution of the partnership ie on the admission retirement or death of a partner(xi) Settlement of disputes by arbitration(xii) Settlement of accounts at the time of retirement or death of a partner5IN ABSENCE OF PARTNERSHIP DEEDThe partnership deed lays down the terms and conditions of partnership in regard to rights duties and obligations of the partners In the absence of partnership deed there may arise a controversy on certain issues like profit sharing ratio interest on

capital interest on drawings interest on loan and salary of the partners In such cases the provisions of the Indian Partnership Act becomes applicableSome of the Issues are(i) Distribution of Profit Partners are entitled to share profits equally(ii) Interest on Capital Interest on capital is not allowed(iii) Interest on Drawings No interest on drawing of the partners is to be charged(iv) Interest on Partnerrsquos Loan A Partner is allowed interest 6 per annum on the amount of loan given to the firm by himher(v) Salary and Commission to Partner A partner is not entitled to anysalary or commission or any other remuneration for managing the business

History TOPIC-TOWARDS INDEPENDENCE AND PARTITION THE LAST PHASE (1935-1947)

SUB TOPIC-IMPORTANT POLITICAL DEVELOPMENTS ndash GROWTH OF SOCIAL IDEAS

Socialism is a political social and economic philosophyLike in other parts of the world the Russian revolution of 1917 served as a great inspiration for revolutionaries in India who at that time were engaged in the struggle for liberation from British ruleSocialist ideas led to the formation of communist party of IndiaJAWAHARLAL NEHRU Among the early Congress leaders Jawaharlal Nehru was very much impressed and influenced by the Socialist ideas He also learnt about the Economic activities of the Soviet Union after the Bolshevic Revolution 1917 He made full use of them in IndiaThe election of Jawaharlal Nehru and Subhas Chandra Bose showed the Left wing tendency within CongressJawaharlal Nehru demanded economic freedom along with political freedom of the people in order to end the exploitation of masses

Nehrus working committee included three socialists leaders The Lucknow session was a landmark in the evolution of socialist ideas of the congressSUBHAS CHANDRA BOSE ndash Subhas Chandra Bose had socialist leaning Both Jawaharlal Nehru and Subhas Chandra Bose were known as leftist Congress men Later on National Congress divided into Leftist and rightist campCONGRESS SOCIALIST Within the Congress some leaders formed the Congress Socialist partyPattavi Sitaramyya Sardar Patel Rajendra Prasad had hostile attitude towards the Congress Socialist partyJawaharlals attitude was hesitant

1 QUESTION ndash Mention name of two Congress leaders who had socialist leaning

1ANSWER ndash Subhas Chandra Bose and Jawaharlal Nehru2QUESTION- In which session of the congress Jawaharlal elaborated his Socialist ideas2 ANSWER ndash Lucknow and Faizpur Session in December 1935 and 19363QUESTION ndash Why Congress was sharply divided into leftist and rightist camp 3ANSWER ndash Subhas Chandra Bosersquos attempt to seek re election for congress presidentship in 1939sharply divided the National Congress into Leftist and Rightist camp4 QUESTION ndash Who was MN Roy 4 ANSWER ndash Manabendra Roy first formed the Communist Party of India outside the country at Tashkent in 19205QUESTION ndash Who formed the Congress Socialist Party within the Congress5 ANSWER ndash Jaya Prakash Narayan Achyut Patwardhan Acharya Narendra Dev Ram Mohan Lohia Aruna Asaf Ali6QUESTION ndash When was the Congress Socialist Party formed What was its object6 ANSWER ndash 1934The Congress Socialist Party sought to work out socialist programme through the Congress They joined hands with the Congress and wanted to carry

Subhas Chandra Bose being expelled from the congress after the Tripuri rift he formed Forward BlockThere were basic differences between the Congress Socialists and the communistsTRADE UNION ACTIVITIES Maximum working class people lived in Bombay and Calcutta The working and living conditions of those workers were very miserable In this situation Shasipada Banerjee NM Lokhande protested against the oppression of the working class peopleThe first Trade Union Madras Labour Union was formed in 1918 by BP WadiaIndustrial strikes took place in Kanpur Calcutta Madras Jamshedpur and Ahmedabad AITUC was formed in Bombay in 1927 The growth of Trade union among the workers was slow because of the fear of the dismissal of the jobIn the mean time the Moderates as well as Communists left AITUC and formed separate organization

on National struggle with the help of workers and peasant class of the society7 QUESTION ndash What was the name of the party founded by Subhas Chandra Bose7 ANSWER- Forward Block8QUESTION ndash Who was Shasipada Banerjee8 ANSWER ndash Shasipada Banerjee was a radical Brahmo He founded a working menrsquos club to protest against exploitation of the British rulers towards the working class of India9 QUESTION ndash What was the weekly published by NM Lokhande9ANSWER- Dinabandhu10 QUESTION ndash Who founded Bombay Mill-Hands Association and in which year10 ANSWER- NM Lokhande in189011 QUESTION- Who was BP WadiaANSWER- BPWadia was the founder of Madras Labour Union in191812 QUESTION- What was the name of the first labour union of India12 ANSWER- Madras Labour Union13 QUESTION Who founded the Majur Mahajan 13 ANSWER GANDHIJI14 QUESTION What was the full form of AITUC When it was formed14 ANSWER All India Trade Union Congressin 192715QUESTION Who formed the Red Trade Union Congress and in which year15ANSWER The Communists formed the Red Trade Union Congress16 QUESTION What do you mean by Socialism16 ANSWER Socialism describes any political and economic theory that says the community rather than individuals should own and manage property and natural resources

Subject Eng Literature (The Tempest ndash William Shakespeare) Topic Act III Scene 3 Lines 1 to 52 (Line 52 ndash Brother my lord the Duke Stand to and do as we) Date 13th April 2020 (4th Period)

[Students should read the original play and also the paraphrase given in the school prescribed textbook]Summary Questions amp Answers

o Alonso Sebastian Antonio Gonzalo Adrian Francisco and others wandered about the island in search of Ferdinand and gets tired and hungry of the toil and at the same time gives up all hope of finding him

o Antonio and Sebastian are happy that Alonso is out of hope and decide to make another attempt on his life that night when being so tired they will be sleeping soundly

o Suddenly a solemn and strange music is heard in the air and several strange shapes enter bringing in a banquet These strange shapes then dance round it with gestures of salutation and then inviting the King to eat they depart

o Seeing this strange scene all are inclined to believe the tales told by travelers that there truly are ldquounicornsrdquo and ldquothe phoenixrsquo thronerdquo

1 ALONSO What harmony is this My good friends hark (L18-27)

GONZALO Marvellous sweet music

[Enter several strange shapes bringing in a banquet

they dance about it with gentle actions of salutation

and inviting the King and his companions to eat they depart]ALONSO Give us kind keepers heavens What were theseSEBASTIAN A living drollery Now I will believe

That there are unicorns that in Arabia

There is one tree the phoenixrsquo throne one phoenix

At this hour reigning thereANTONIO Ill believe both

And what does else want credit come to me

And Ill be sworn rsquotis true Travellers neer did lie

Though fools at home condemn rsquoem

(a) How did Prospero present an amazing spectacle before Alonso and his companions

Using his magic powers Prospero ordered strange shapes to lay a banquet before Alonso and his companions The shapes brought several dishes with tasty eatables in them They placed the dishes on a table before Alonso and his companions Then the strange shapes began to dance gracefully around the banquet While dancing they made gestures inviting them to eat the food Then suddenly the shapes disappeared(b) Who were the guests at the strange banquet Describe the lsquoliving drolleryrsquo

Alonso Sebastian Antonio Gonzalo Adrian and Francisco were the guests at the strange banquet

The term ldquoliving drolleryrdquo refers to live entertainment show In this context when Alonso the King of Naples Sebastian his brother Antonio the treacherous brother of Prospero Gonzalo the kind and loyal councillor to the King Adrian and Francisco came to the island they were hungry and weary in their spirits They heard a solemn and strange music They were shocked to see several strange shapes bringing in a banquet and these shapes danced about it with gentle action of salutation inviting the King and his companions to eat After this Sebastian described this show as lsquoliving drolleryrsquo(c) What is lsquophoenixrsquo What are lsquoUnicornsrdquo

The term lsquophoenixrsquo refers to a mythical Arabian bird which lived alone and perched on a solitary tree After one hundred years it expired in flames and rose again from its own ashes

lsquoUnicornsrsquo refers to the mythological four-footed beasts having horns in the centre of their foreheads When the horns are ground into powder the powder was believed to be

an aphrodisiac(d) How does Sebastian explain the puppet show OR Why does the speaker now believe in unicorns and phoenix

Sebastian finds several strange shapes bringing in the banquet They invite the king and his party for dinner and soon depart He tells that if such a strange sight can be a reality there is nothing incredible in the world and from the present moment he will believe anything He says that it is a strange dumb show enacted not by puppets but by living beings It is stranger than a travellerrsquos tale Seeing such a thing

before his own eyes he will no longer disbelieve the story about unicorns and phoenix(e) How do the other characters present respond to this living drollery

At the sight of the lsquoliving drolleryrsquo like Sebastian Gonzalo and Antonio too acted strangely Antonio told that he too now believes in unicorns and phoenix and anything else that seems to be incredible He too now believes in travellersrsquo tales Gonzalo told that if he would report those happenings in Naples nobody will believe him He considers that those gentle shapes were gentler in manner in comparison to the living beings Alonso was at first sight suspicious and told them that those strange shapes conveyed their meaning in expressive gestures when they seemed to lack speech by their movements and sounds Francisco was amazed at their mysterious disappearance

2 ALONSO Not I

(Line 43-52)GONZALO Faith sir you need not fear When we

were boysWho would believe that there were mountaineers

Dewlapped like bulls whose throats had hanging at rsquoem

Wallets of flesh Or that there were such men

Whose heads stood in their breasts Which now we find

Each putter-out of five for one will bring us

Good warrant ofALONSO I will stand to and feed

Although my lastmdashno matter since I feel

The best is past Brother my lord the Duke

Stand to and do as we

(a) How does Alonso respond at the spectacle of the shapes which were sent to them at the instruction of Prospero

After seeing the strange sight of appearing and disappearing of the shapes sent by Prospero to arrange a banquet for them Alonso says that his surprise at having seen those creatures is infinite and he is fully justified in feeling so much surprise He thinks that their shapes their gestures and the sounds they made were indeed amazing Although they do not possess the gift of speech yet they were able to convey their

thoughts by means of their gestures only

(b) What does Prospero say about the views expressed by Alonso regarding the shapes What does Francisco think about the shapesAfter hearing Alonsorsquos views about the shapes Prospero says that this manrsquos praise of the spirits is rather hasty He means to say that Alonso has shown great haste in reaching the conclusion about the shapes Francisco is amazed to see that those shapes disappeared in a mysterious way(c) What does Sebastian ask Alonso to doSebastian tells Alonso that the shapes having disappeared should not matter to them because they have left the eatables behind He asks Alonso to enjoy eating as they are extremely hungry but the king does not accept his offer of enjoying the dishes(d) How does Gonzalo try to dispel Alonsorsquos fear of those strange shapes What kind of references does he give to AlonsoGonzalo says that those who have travelled abroad have reported seeing even stranger sights than these shapes that Alonso and his companions have beheld Hence there is no reason to feel afraid of these shapes Gonzalo further adds that in his younger days he had heard strange stories from travelers and Alonso might have heard similar stories For instance it was said that there existed a certain race of

human beings who had huge lumps of flesh hanging at their throats and who therefore resembled bulls Then Gonzalo tells about a race of human beings whose heads were located at their breasts Gonzalo says that such stories were not believed by most people in those days but now-a-days these stories have become common(e) Explain the following lsquoEach putter-out of five for onersquoEnglish travellers often insured their trips with London brokers Those that went on foreign travels those days used to deposit a certain amount with some firm or company in London before their departure If the travelers failed to return the money was forfeited by the company with which it had been deposited But this money was repaid five-fold if the travelers returned safe and sound In this way a traveler stood a great chance of recovering the entire cost of his

travels(f) Give the explanatory meanings of the following expressions in the context of the above extract (i) Dewlapped (ii) Wallets of flesh

(iii) Putter-out(i) Dewlapped having big lumps of flesh at the necks(ii) Wallets of flesh large masses of flesh looking like bags(iii) Putter-out to invest money before commencing the travel

  • General methods of preparation of hydrogen
  • Chapter Dimensional Analysis (Summary)
    • Properties of Charge
Page 8:   · Web viewSubject. Topic. Summary. Execution. Hindi. व्याकरण. शरीरके अंगो के नाम लिखिए. 1) आँख 2) नाक 3

Class VSubject Topic Summary Execution

COMPUTER130420

CHAPTER 2 APPLICATIONS OF COMPUTERS

PAGE NO-15C FILL IN THE BLANKS

INPUTS ANIMATION BANKS SOFTWARE SUPER COMPUTERS

D WRITE TRUE OR FALSE TRUE TRUE TRUE TRUE FALSE

Science Chapter 2 - The Skeletal System

Joints A joint is a place where two or more bones are joined with each other

There are two types of joints 1) Fixed joints 2) Movable joints

The movable joints are mainly four types ndash 1) Hinge joint 2) Pivot joint 3) Gliding joint 4) Ball and socket joint

DAnswer these questions

1)What are the functions of the skeleton

Ans ndash The functions of skeleton are -

i The skeleton gives our body shape ii The skeleton provide our body strength iii The skeleton protect our inner organs from outside injuries iv The skeleton gives our body support

2)List the type of movable joint in our body giving one example of each

Ans - The movable joints are mainly four types ndash a Hinge joint

Example of hinge joints are elbows nice fingers and toes

b Pivot joint Example - the joint between the skull and the backbone is the example of pivot joint

c Gliding joint Example of gliding joints

are wrist and ankle joints d Ball and socket joint

Example of ball and socket joints are shoulder joint and hip joint

English language

Transitive and intransitive verb

Pick out the verbs from the following sentences and say whether they are transitive or intransitive (page 21 ex B)1 gave ndash verb Transitive verb2 Has planted- verb Intransitive verb3 Were- verb Intransitive verb4 Asked- verb Transitive verb5 Told- verb Transitive verb

Social studies Conquering distances

Airways The only airline owned by the government is Air India which handles both domestic and international flightsAdvantages of airways Air transport is the fastestIt can access remote areasIt is the best means of transport in case of emergencies

1 Which is the only airline owned by the governmentAns Air India

2 Give two advantages of airwaysAns The two advantages of airways areAir transport is the fastest transportIt can access remote areas

DisadvantageIt is the most expensive of all other means of transport

3 What is the disadvantage of airwaysAns The only disadvantage of airways is that it is the most expensive of all other means of transport

MATHEMATICS

Ch 3Addition and Subtraction

Properties of Addition1 The sum of two numbers does not change when we change their order This property known as Commutative Property of addition

2 The sum of three numbers does not change when we change their grouping This property is called Associative Property of addition

3 The sum of the numbers and zero is the number itself This property is called Identity Property of addition and the integer 0 is called identity

Exercise 11Fill in the blanks1 2730815 + 8319293 = ____ + 27308152 18219 + 1850308 = 1850308 + ____3 (27815 + 85919) + 95985 = (85919 + ____) + 278154 13227 + (25983 + 73607) = (____ + 25983) + 736075 91389 + 0 = ____ + 91389

Solution 1 83192932 182193 959854 132275 0

6 Which of the following are true statements(a) Any number added to zero is zero

(b) The sum of two numbers does not change when we change their order

(c) 1 is the identity element of addition

(d) Given any three numbers their sum does not change when we change their grouping

Solution (a) False(b) True(c) False(d) True

Class VISubject Topic Summary Execution

HISTORY AND CIVICS

CHAPTER 3

MAHAVIRA AND BUDDHA ndash GREAT PREACHERS BUDDHA

Decline of Buddhism1 Revival of the Brahmanical Hinduism ndash Brahmin Scholars like Shankaracharya and Kumarila Bhatta led the revival of Hinduism and established the supremacy of Vedic religion2 Loss of Royal Patronage ndash Gupta period marked the decline of Buddhism as Gupta rulers were followers of Hinduism3 Split in Buddhism ndash division into Hinayana and Mahayana sects and rise of Mahayana sects blurred the line between Hinduism and Buddhism4 Corruption in Buddhist Sangha ndash due to generation of large revenue from large estates Buddhist monks and nuns started living luxurious

Answer the Following 1 During which dynasty Buddhism was split During the reign of Kanishka

2 Name the two sects of Buddhism Mahayana and Hinayana

3 Name two Vedic scholars who led the revival of Brahmanical Hinduism Shankaracharya and Kumarila Bhatta

life in rich monasteries Hence corruption crept in5 Adoption of Sanskrit ndash when Buddhist scriptures began to be written in Sanskrit in place of peoples language like Pali or Prakrit people started drifting away from Buddhism6 The Turkish Invasion ndash As Muslim conquerors invaded India immensely wealthy Buddhist Monasteries and temples were looted and destroyed and Buddhists were persecuted and killed

4 During which period decline of Buddhism began The Gupta Period

ENGLISH 2 The great train journey- Ruskin Bond

The great journey by Ruskin Bond is a story about Suraj who loved trains and wanted to go to places One day while wandering along the railway tracks he enters into a carriage compartment The train suddenly starts moving with him in the compartment and after a journey returns back to the same place from where it had begun The story is about his experience during that journey

4 Answer the following questionsf Who else is in the carriageA ragged hippy with a dirty beard face was in the carriageg Where does Suraj say that he would like to go toSuraj said that he would like to go to England and China and Africa and Greenland He wanted to go all over the worldh What warning does the man give to SurajThe man said Suraj to keep out of sight so that he doesnrsquot get caught by the ticket collectorsiWhen Suraj thinks about his parents for the first time what does he imagines that they will thinkSuraj thought that if he failed to come home that night his parents would think that he had run away or been kidnapped or been involved in an accidentJ What presents does Suraj imagine that he will bring back for his friendSuraj imagines that he would bring an African lion or a transistor- radio for his friend

CHEMISTRY

Chapter 2 ndashElement and Compound

ATOMAn atom can be defined as the smallest constituent particle of an element which showcases independent existence Example Ne OMOLECULEA molecule can be defined as the combinations of two or more atoms which are held together by chemical bonds A molecule is the smallest portion of a substance which showcases all the properties of the substance On breaking down a molecule further we see properties of the constituent elements Example HCl NaCl O2

Answer the following Q3) What is a moleculeAns - A molecule can be defined as the combinations of two or more atoms which are held together by chemical bonds A molecule is the smallest portion of a substance which showcases all the properties of the substance On breaking down a molecule further we see properties of the constituent elements Example HCl NaCl O2

Q4) Which can exist independently ndash atom or moleculesAns ndash Molecules can exist independently

PHYSICS Physical quantities

Guidelines for writing SI units correctly1 The units named after scientists are not written with a capital initial letter For example newton henry watt2 The symbols of the units named after scientist should be written by a capital letter For example N for newton H for henry W for watt3 Small letters are used as symbols for units not derived from a proper name For example m for metre kg for kilogram4 No full stop or other punctuation marks should be used within or at the end of symbols For example 50 m and not as 50 m5 The symbols of the units do not take plural form For example 10 kg not as 10 kgs6 When temperature is expressed in kelvin the degree sign is omitted For example 273 K not as 273o K (If expressed in Celsius scale degree sign is to be included For example 100o C and not 100 C)7 Use of solidus is recommended only for indicating a division of one letter unit symbol by another unit

Fill in the blanks

1) Length and mass are examples of fundamental physical quantities

2) The measurement of a physical quantity consists of two part magnitude and unit

3) A foot consist of 32 inches 4) The unit of temperature in the SI system is

Kelvin

Write true or false Correct the false statements

1) In ancient times cubit was used to measure the mass of an object FalseCorrect statement ndash In ancient times cubit was used to measure the length of an object

2) There are 7 fundamental physical quantities True

symbol Not more than one solidus is used For example m s-1 or m s J K mol or J K-1 mol-1 but not J K mol8 Some space is always to be left between the number and the symbol of the unit and also between the symbols for compound units such as force momentum etc For example it is not correct to write 23m The correct representation is 23 m kg m s-2 and not as kgms-29 Only accepted symbols should be used For example ampere is represented as A and not as amp or am second is represented as s and not as sec10 Numerical value of any physical quantity should be expressed in scientific notationFor an example density of mercury is 136 x 104 kg m-3 and not as 13600 kg m-3

3) Second is the unit of time in both the CGS and MKS systems True

4) The symbol used for a unit is always written in capital letters False Correct statement -The symbol used for a unit is normally written in small letters

Hindi 2nd language

वाकय निवचार भागवत निवचारो को परकट करन वा साथक एव वयवसथिtत शबद समह को वाकय कहत ह वाकय दो परकार क होत ह ndash

1 उददशय- वाकय म जिजसक बार म कछ बताया जाता ह उस उददशय कहत ह जस राधा एक नतकी ह2 निवधय- वाकय म उददशय क बार म बताया जाता ह उस निवधयक कहत ह जस- राधा एक नतकी ह रचना क आधार पर वाकय क तीन भद होत ह ndash१सर वाकय- राम बाजार गया २ सयकत वाकय- राम बाजार गया और वहा जाकर दोसत स मिमा३ मिमशर वाकय- यह वही tान ह जहा उनका बचपन बीता

helliphellipContinue to nextBengali 2nd language

লিZ সবরপ ও সবরলিZ

সবরলিZর লিয়ম - ১ অ-কোর লিকংবো আ-কোসরর পসর অ-কোর লিকংবো আ - কোর োকস উভয় লিমস আ ndashকোর য় এবং ওই আ ndash কোর পব13বস13 Pকত য়

২ ই - কোর লিকংবো ঈ - কোসরর পসর ই - কোর লিকংবো ঈ - কোর োকস উভয় লিমস ঈ - কোর য় এবং ওই ঈ - কোর পব13বস13 Pকত য়

৩ উ - কোর লিকংবো ঊ - কোসরর পসর উ - কোর লিকংবো ঊ - কোর োকস উভয় লিমস ঊ - কোর য় এবং ওই ঊ - কোর পব13বস13 Pকত য়

৪ অ - কোর লিকংবো আ ndash কোসরর পসর ই - কোর লিকংবো ঈ - কোর োকস উভয় লিমস এ - কোর য় এবং ওই এ - কোর পব13বস13 Pকত য়

১ অ + অ = আ ( gtো ) লিম + অচ = লিমোচ সব + অ3ী = সবো3ী অ + আ = আ ( gtো )পদম + আ = পদমো শভ + আলি = শভোলি আ + আ = আ ( gtো )4োয়ো + আবত = 4োয়োবত মো + আতমো = মোতমো আ + অ = আ ( gtো )লিবদযো + অংকোর = লিবদযোংকোর Pো + অ13 = Pো13 ২ ই + ই = ঈ ( gtী )অলিত + ইব = অতীব লিগলির + ইনদর = লিগরীনদর ই + ঈ = ঈ ( gtী )পলির + ইকষো = পরীকষো অলি3 + ঈশবর = অ3ীশবর ঈ + ঈ = ঈ ( gtী )মী + ঈশবর = মীশবর 3ী + ঈ = 3ী ঈ + ই = ঈ ( gtী )রী + ইনদর = রীনদর মী + ইনদর = মীনদর ৩ উ + উ = ঊ ( gt )মর + উদযো = মরদযো কট + উলিকত = কটলিকত উ + ঊ = ঊ ( gt )ঘ + ঊলিম13 = ঘলিম13 লিZ + ঊলিম13 = লিZলিম13 ঊ + ঊ = ঊ ( gt )রP + ঊলিম13 = রPলিম13

৪ অ + ই = এ ( লেgt )র + ইনদর = সরনদর লেPোগ + ইনদর = লেPোসগনদর অ + ঈ = এ ( লেgt )গ + ঈ = গস

র + ঈ = সর আ + ই = এ ( লেgt )Pো + ইষট = Pসষট 3ো + ইনদ = স3নদ আ + ঈ = এ ( লেgt )রমো + ঈ = রসম দবোরকো + ঈশবর = দবোরসকশবর

COMPUTER THE WORLD OF WINDOWS 10

DONE IN THE PREVIOUS CLASS PAGE NO-83A TICK THE CORRECT OPTION BACKGROUND DISPLAY AREA RESTORE THREE

MATHEMATICS Topic ndash NumbersChapter ndash Natural numbers and whole numbers

Study item Properties of whole numbers for subtraction1) Closure property When we do subtraction of two whole numbers we can not get a whole number in all time Example 8 ndash 3 = 5 a whole number 0 ndash 6 = -6 is not a whole numberTherefore the subtraction of two whole numbers is not satisfying closure property2) Commutative property If x and y are two whole numbers then x ndash y is not equal to y ndash xExample If x=16 and y = 7 then x ndash y = 16 ndash 7 = 9Again y ndash x = 7 ndash 16 = - 9 Therefore x ndash y not equal to y ndash x Therefore the subtraction of two whole numbers is not satisfy commutative3) Associative property If x y and z are three whole numbersThen x ndash ( y ndash z ) not equal to ( x ndash y ) ndash z Example If x = 20 y = 10 and z = 6Therefore x ndash (y ndash z ) = 20 ndash(10 ndash 6 ) = 20 ndash 4 = 16(X ndash y ) ndash z = (20 ndash 10) ndash 4 = 10 -4 =6Therefore x ndash(y ndash z) not equal to ( x ndash y) ndash zTherefore subtraction of whole numbers is not satisfying associativity4) Distributive property If x y and z are three whole numbersThen x (y ndash z ) = xy ndash xzAnd (y ndash z)x = yx ndash zxExample If x = 10 y = 6 and z = 4x(y ndash z ) = 10(6 ndash 4 ) = 10times6 ndash 10times4 = 60 ndash 40 = 20( 6 ndash 4 )times 10 = 6times10 ndash 4times10 = 60 ndash 40 = 20Therefore the subtraction of whole numbers is satisfying distributive property5) Existence of identity For any whole number x X ndash 0 = x but 0 ndash x = - x not equal to xThus for subtraction no identity number existsException 0 ndash 0 = 0 so 0 is its own identity for subtraction

Class VIISubject Topic Summary Execution

Hindi 2ndlang वचन जो सजञा शबद निकसी वसत या पराणी क एक या अनक होन का बोध कराया उनह वचन कहत ह जस डका- डकयह दो परकार की होती ह-

क) एकवचन-शबद क जिजस रप स उसक एक होन का बोध हो उस एक वचन कहत ह जस निकताब गमा आदिद

) बहवचन-शबद क जिजस रप स उसक आन ोन का पता च उस बहवचन कहत ह जस डक निकताब निततलियाआदिद

निनमनलिखित शबदो को एकवचन स बहवचन म बदोम- हमजानित- जानितयानारी- नारिरयामिमतर ndashमिमतरोपसतक -पसतकसडक-सडकबोत-बोतनाहर-नहररपए-रपया

Bengali বইndashবোংো োলিতয পলিরচয়

পোঠndash১৪ গলপ - অপর কলপো পর

লেক - লিবভলিতভষ বসনদযোপো3যোয়লেকndash রবীনদর পরবতf বোংো কোোলিতয 3োরোর উসgসPোগয োম পরকলিতসপরমী লিবভলিতভষ বসনদযোপো3যোয় তোর লেীসত লেPম বোসর বোসর লিফসর এসস4 গরোম বোংোর পরকলিতর কো লেতমলি এসস4 গরোমী মো লিচতরগলপndash অপর কলপো গলপোংটি লিবভলিতভষ বসনদযোপো3যোসয়র লিবযোত উপযো পসর পাোচোী লেসক গীত অপ অ13োৎ পসর পাোচোী তো অপর কলপোর লেকনদরীয় চলিরতর এই অংস আমরো পোই বোক অপসক বোক অপ কলপো লিবোী লে দসরর অ গো4 লেদস মোসয়র মস লেোো রপকোর রোসয পোলি2 লেদয় দপরসবো মোসয়র মসর কসর কোীদোী মোভোরত এর করসকষতর Pসjর ব13ো শস তোর মোবীর কস13র পরলিত ব2 মমতো য় আবোরপালিসত বলি13ত Pসjর অমোপত অং লে লিসই মোপত কসর বোলি2র লিপ4স বাো বোগোস লিকংবো উঠোসর লিশমসর কলপো লিবো এোস পরকো লেপসয়স4

১ অপর কলপো গসলপর লেক লেক তোর মপসক13 লেসো২ অপর কলপো গলপটি লেকোো লেসক গীত গলপটির ম ভোব লেসো

GEOGRAPHY CHAPTER 7EUROPE

CHAPTER COMPLETE EXERCISEFill in the blanks1 Europe is a continent that comprises the western part of Eurasia2 Eurasia and Africa are connected into one large land mass known as Afroeurasia3 The Strait of Gibraltar separates Europe and Africa4 Europe is surrounded by the Arctic Ocean to the north5 The British Isles includes the island countries of Great Britain and Ireland

Name the following 1 Connects Africa to Eurasia - Isthmus of Suez2 Largest country in the world in terms of area ndash Russia3 A term used collectively for the five countries in northern Europe ndash Nordic Countries4 The capital of Montenegro - Podgorica5 the largest fjord in Norway ndash Sognefjord

Match the following Column 1 Column 2a Albania iii Tiranab Belgium i Brusselsc Denmark v Copenhagend Finland ii Helsinkie Hungary iv Budapest

CHEMISTRY Chapter 2 ndashElement and Compound

Atom - An atom is the basic unit of an element or the smallest particle of an element non capable of independent existence Atom is built up of three sub atomic particles electron proton and neutron

Nucleus-It is the centre of an atom In the centre of the atom contains proton (positively charged particles ) and neutrons ( particles carrying no charge )

Orbits- It surround the nucleus in which revolve electrons (negatively charged particles)

Answer the following

1) What are MetalloidsAns - Certain elements using properties of both metal and non-metals are called metalloids Example Silicon arsenic and antimony

2) What are Noble gasesAns - Certain elements are present in the air and are chemically inert or unreactive Such elements are called rare gases or noble gases Example helium neon argon and Krypton

English 2 Sentences based on meanings

Kinds of sentences

Assertive or declarative to convey information or simply make a statement

Interrogative to ask different types of questions

Imperative to command or instruct someone or make a request

Exclamatory to express strong feelings and emotions

Exercise B1 Stop it ( Exclamatory)2 May you always be happy

together ( Exclamatory)3 He does not like sports

( Assertive)4 Please pass me the salt

( Imperative)5 How dare she talk to me like

that ( Exclamatory)6 May success bless your effort

( Exclamatory)7 Canrsquot you wait for sometime

(Interrogative)8 Did anybody tell you about it

( Interrogative)9 I saw her waiting for the bus

( Assertive)10 Could you please take a

message for me ( Interrogative)

Homework Ex ABiology Chp -2

Classification of Plants

Today we discuss about usefulness of bacteria We also discuss what the harmful effects of bacteria are

89 How bacteria are useful for usbull Bacteria is helpful in many ways forhuman being i) Production of medicine - antibiotics vaccine etcii) Formation of curd by lactobacillusiii)Nitrogen fixation in Leguminousplant by Rhizobiumiv) Increase soil fertility by absorbingatmospheric nitrogen and convert it into nitrates and nitritesv) Cleaning the environment by converting the complex substances into simple substancesvi) Tanning of leathervii) Retting of Fibersviii) Formation of compost by acting onanimal dung and agricultual cases1x) Biogas production by decomposingplant and animal wastex)Help In Nutrition by producing vitamiacutemBand kx1) Some bacteria are used to give specialflavour to tea coffee and coccaQ10- Name some diseases and there causativebacteriabull Diseasescausative bacteria1 CholeraVibrio cholerae2 Tuberculosis - Mycobacterium tuberculosis3 Diptheria -Corynebacteriumdiphtheriae4 Pneumonia - Streptococcus pneumoniae

Math Number system

Chapter Fraction

Study item Using lsquoofrsquoThe word lsquoofrsquo between any two fractions is to be used as multiplicationExample 57 of 56 = 57 times 56 = 5times8 = 40Study item Using BODMASThe word lsquoBODMASrsquo is the abbreviation formed by taking the initial letters of six operations(i)Bracket (ii) of (iii) Division (iv) Multiplication (v) Addition (vi) SubtractionAccording to BODMAS rule First of all the terms inside Bracket must be simplified then lsquoofrsquo lsquoDivisionrsquo lsquoMultiplicationrsquo lsquoAdditionrsquo lsquosubtractionrsquo

Study item Removal of Brackets

There are four Brackets of algebra in Mathematics In a complex expression four types of brackets are used Order of removing the brackets is first ----- then ( ) then finally [ ]

Class VIIISubject Topic Summary Execution

Chemistry Hydrogen General methods of preparation of hydrogen

By the action of dilute acids on metals

Calcium Reacts readily to form chloride salt and hydrogen

Ca + 2HCl rarr CaCl₂ + H₂uarr

Magnesium

Aluminium

Zinc

React readily to form salt and hydrogen

Mg + 2HCl rarr MgCl₂ + H₂uarr2Al + 6HCl rarr 2AlCl₃ + 3H₂uarrZn + 2HCl rarr ZnCl₂ + H₂uarr

Question 4 ) Give reasons for the following

(a) Hydrogen be used as a fuel

Solution

Hydrogen is used as a fuel because it has a high heat of combustion Some significant fuels are coal gas water gas and liquid hydrogen

(b) Though hydrogen is lighter than air it cannot be collected by downward displacement of air

Solution

Hydrogen is lighter than air so it is possible to collect the gas by downward displacement of air But it is not safe to do so since a mixture of hydrogen and air can lead to an explosion

(c) A pop sound produced when hydrogen is burnt

Solution

Impure hydrogen gas burns in air with a pop sound This is because of the presence of impurities in it

(d) Helium replaced hydrogen in weather observation balloons

Solution

It forms a mixture with air that can explode when there is a small leakage of hydrogen in a balloon So helium has replaced hydrogen

(e) Nitric acid not used for the preparation of hydrogen gas

Solution

(e) By the action of nitric acid on metals hydrogen cannot be produced because it also releases nitrous oxide and nitric oxide and oxides the hydrogen to form water

Biology Chp-2 Reproduction in plants

Today we discuss different methods of artificial propagation like cutting-rose sugercane Layering ndashguava lemon china rose etc Grafting- mango apple etcMicropropagation ndashorchid asparagus etcWe also discuss about advantages and disadvantages of vegetative propagation

Q7 Define the following terms i) Explant In tissue culture techniquea tiny piece of bud shoot or any other partof plant from where new tissue develop ii) Callus The cells of the tissue divide andgrow into a mass of undifferentiated cells from explant iii) Plantlet After few days callus differentiate into a small plant with roots and shootQ8 what are the advantages and limitations of tissue culture or micropropagation

Advantages i ) It produacuteces superior quality plantsii)It can be applied to interspecifie hybridsiii) It is useful to grow seedless plants bull Limitations i) It cannot be used for all plantsii)It is not easy to handleQ9 Write advantages of vegetative propagationi) It is a quick and easy method ofproducing new plantsii) This method need less time to matureiii) The new plants are exact copies of the parentiv) it is extremly useful for growing seedlessplants like banana grapes etc Q10 Write some disadvantages of vegetativepropagationi) Dišeases present in the parent plant gettransferred to all in new plantsii) Overcrowding of new plants causes competition for sunlight water and nutrients which affects growth of plantsplant

Physics Chapter 2 Physical Quatites and Measurements

Here We Will Do Some QuestionsRelated To Chapter 2

Select the correct alternative A block of wood of density 08gcm-3 has a volume of 60cm3 The mass of the block is

1 608 g

2 75 g

3 48 g

4 0013 g

Solution 348 g

The density of aluminium is 27g and that of brass The correct statement is

1 Equal masses of aluminium and brass have equal volumes

2 The mass of a certain volume of brass is more than the mass of an equal volume of aluminium

3 The volume of a certain mass of brass is more than the volume of an equal mass of aluminium

4 Equal volumes of aluminium and brass have equal masses

Solution 2 The mass of a certain volume of brass is more than the mass of an equal volume of aluminium

MATHEMATICS Ch 6Sets

Exercise 6(C)1 Find all the subset of each the following sets(i) A = 57 (iii) C = x xisin W x le 2(iv) p p is a letter in the word lsquopoorrsquo

Solution (i) All the subsets of A are ϕ 5 7 57

(iii) All the subsets of C are ϕ 0 1 2 01 02 12 012

(iv) All the subsets are ϕ p o r po or por

4 Given the universal set = -7-3-105689 find (i) A = x xlt2 (ii) B = x -4ltxlt6 Solution

(i) A = -7-3-10(ii) B = -3-105

5 Given the universal set = x xisin N and xlt20 find

(i) A = x x = 3p pisin N (iii) C = x x is divisible by 4 Solution

(i) 369121518 (iii) 481216

6 Find the proper subset of x x2-9x-10 = 0 Solution

ϕ 10 -1

Working x2-9x-10 = 0 rArr x2-(10-1)x-10 = 0

rArr x2-10x+x-10 = 0 rArrx(x-10)+1(x-10) = 0

rArr (x+1) (x-10) = 0

11 Let M = letters of the word REAL and N = letters of the word LARE Write sets M and N in roster form and then state whether (i) M sube N is true (ii) N sube M is true (iii) M = N is true

Solution M = real and N = lareSo (i) Yes (ii) Yes (iii) Yes

English 2 Twelfth Night ndash Shakespeare

A noble man named Orsino in the kingdom of Illyria is deeply in love with a lady called lady Olivia She is in mourning for her dead brother so she will not even think about marriage At this time a sea storm causes a terrible shipwreck and a young lady called Viola is swept onto the shore She thinks that her twin brother Sebastian is drowned A sea captain tells her about Orsino and his love for Olivia Viola wishes to work in Oliviarsquos home but feels she will not be employed So she dresses as a man calls herself Cesario and gets work at the house of OrsinoViola (now Ceasario) is much liked by Orsino and becomes his page She falls in love with Orsino Orsino sends Ceasario to deliver messages to Olivia Olivia herself falls for the beautiful young Ceasario believing Viola to be a man

2 Answer the following questionsa Why does Orsino ask the musicians to play onOrsino asks the musicians to play on because music feeds his desire He calls upon the musicians to play music so that his hunger for love could be replenished with an excess of musicb What does Valentine tell about OliviaWe learn from Valentine that Olivia is in mourning for her brother she wears a veil and has vowed that no one will see her face for another seven yearsand she refuses to marry anyone until thenc From the exchange between Orsino and Valentine what do you think their relationship isValentine is one of orsinod attendants He was sent to Olivia as a messenger of love but was not allowed to speak to here Who is Olivia mourning for and whyOlivia is mourning for her dead brother

Homework Q fHistory and Civics

Growth of Nationalism

Important dates to remember1769-Napoleon born on 15thAugust1789-Fall of Bastille on 14th July and the beginning of the French revolution declaration of the rights of Man on 26thAugust1793-King Louis XVI executed on January 211764-The Sugar Act passed1765-The Stamp act passed1774-The first congress of Philadelphia1776-The declaration of American Independence of on 4th July1777-Defeat of the British at Saratoga1781-Surrender of lord Cornwallis at Yorktown1783-The treaty of Versailles1804-Napoleon becomes the emperor1813-Battle of Leipzig or Battle of nations in which Napoleon was defeated by the Allies1815-Battle of Waterloo June 18 in which Napoleon was defeated and captured1821-Death of Napoleon in StHelena1860-Abraham Lincoln elected President of the USA1861-The civil war began 1864-Abraham Lincoln elected President of the USA for the second time1865-Slavery abolished in the US

Name the following- The queen of Louis XVI

Marie Antoinette The three philosophers of France

VoltaireMontesquieuJean Jacques Rousseau

The British general whose surrender brought the war in America to an endLord Cornwallis

The first president of the USAGeorge Washington (1732-1799)

The first southern state to secede from the unionSouth Carolina

The author of the book lsquoUncle Toms CabinHarriet Beecher Stowe

Homework-Learn

Class IXSubject Topic Summary Execution

Economics

Types of economies Today I am going to share you the concept of economic growth and economic development Few questions will be given from the previous study material dated 942020

Meaning of economic growthAnswer) The term economic growth generally means anincrease in national income or per capita output or income over time It indicates towards quantitative growth of a country

Meaning of economic developmentAnswer) Economic development is defined

as a process whereby the real per capita income of a country increases over time along with fall in poverty ratio unemployment and income inequality etc

Distinguish between economic growth and economic development

Basis Economic growth

Economic development

Scope It has narrow scope as it refer only to rise in per capita income

It has wide concept since it includes qualitative changes as well

Concerned matter

It is concerned with the rise in income

It is concerned with not only rise in income but also reduction of poverty income inequality and unemployment

Focus Economic growth does not focus on economic development

Economic development focus on economic growth plus qualitative changes

Distinguish between capitalist economy and socialist economy

Ownership

Motive

Tool

Means of production are owned and managed by private people

Self interest and profit earning is the main motive

Price mechanism is a main tool to solve the economic problems

Means of production are owned and managed by the government

Social welfare is the main motive

Economic planning by the government is the main tool to solve the economic problem

Competition

Distribution of income

There exist large competition among buyers and sellers

There is existence of large inequalities of income

There is no such competition

There exist less inequalities of income

Math Topic ndash AlgebraChapter -Factorisation

Study item Factorising by taking out common factorSome solved sums from exercise 41

1) (i) 8xy3 + 12x2y2

= HCF of 8xy3 and 12x2y2 is 4xy2

= 4xy2(2y + 3x )

4) (ii) 28p2q2r ndash 42pq2r2

= HCF of 28p2q2r and 42pq2r2 is 14pq2r = 14pq2r (2p - 3r )5) (ii) 14mn + 22m - 62p=HCF of 14mn 22m and 62p is 2= 2(7mn + 11m - 31p)7) (ii) 3a(x2 + y2) + 6b (x2 + y2) = HCF of 3a(x2 + y2) and 6b(x2 + y2 ) is (x2 + y2)= ( x2+ y2 )(3a + 6b )9) (ii) x(x2 + y2 ndash z2 ) + y(-x2ndashy2 + z2 ) ndash z(x2+ y2 ndash z2 )= x(x2 + y2 -z2) ndash y-(x2 + y2 -z2) -z(x2 + y2 ndash z2)=x(x2 + y2-z2) -y( x2 + y2-z2) ndash z (x2 + y2 -z2)= (x2+ y2 ndash z2)(x ndash y ndash z )

Commercial Studies

Introduction to Accounting and Book-keeping

Today I am going to share you the meaning of Accounting and Book-keeping and its related terms bullAccounting bullBook Keeping bullAccountsbullTypes Of Accounts bullAccounting Cycle

bull Meaning of accounting

Ans )Accounting is the art and science of recording classifying and summarising monetary transactions

bull Meaning of Book-keeping

Ans) Bookkeeping is the art of recording business transactions with the view of having a permanent record of them and showing their effect on wealth

bull Meaning of account

Ans) The term account means a record of business transactions concern a particular person of firm asset or income or expense It is a summarised record of all transactions which take place in an accounting year

bull Types of accountsPersonal accounts ndash Personal accounts relating

to person and Organisation are known

as personal accounts Example Ramrsquos Account ABC amp Co Account etc

Real account - The accounts related to tangible and intangible assets are called real accountsExample Cash Account Furniture Account etc

Nominal account- Accounts related to expenses losses incomes and gains are known as nominal accountsExample Wages Account Salary Account Discount Account etc

bull Accounting cycle Accounting cycle refers to a complete sequence of accounting activities It begins with recording of transactions and ends with the preparation of a balance sheet

English 1 Transformation of sentences

Sentences A sentence is a group of words which makes complete sense

a Assertive sentencesb Imperative sentencesc Interrogative

sentencesd Exclamatory sentences

Sentences can be changed from one grammatical form to another without changing the meaning of the sentence This is known as transformation of sentences

Exercise 6Rewrite the following sentences according to the instructions given below without changing their meanings

1 As soon as he saw the beer he jumped into the river ( Begin No sooner)

2 None but brave deserve the fair (Begin the bravehellip)

3 This box is too heavy for me to lift ( Use so hellip That instead of too)

4 No one other than a king can live like James Luxurious ( Begin only James)

5 Oh for the wings of a dove (Begin I wishhellip)

BENGALI(2ND LANGUAGE)

ldquo বঙগভমির পরমি ldquo াইকেল ধসদন দতত

পব13পোসঠ আসোলিচত ৩ পরবোস দৈদসবর বস ীবতোরো Pলিদ স এ লেদ -আকো সত-োলি লেদ তোস - ক) বকতো লেক কোর লেো লেকো কলিবতোর অং ) কোর পরলিত বকতোর এই উলিকত গ) এ লেদ আকো সত বসত কী বলিঝসয়স4 ীবতোরো বসত কী লেবোঝ ঘ ) আসোচয অংসর তোৎপP13 কী

উ -ক ) বকতো স কলিব মোইসক ম3দ দতত

Types of AccountPersonal AccountReal AccountNominal AccountBalance Sheet (opening)

কলিব মোইসক ম3দ দসততর রলিচত বঙগভলিমর পরলিত কলিবতোর অং ) কলিব বঙগী অ13োৎ লেদমোতোর পরলিত কলিবর এই উলিকত গ ) এ লেদ আকো বসত কলিবর মোব লেদী রপ আকো লেক লেবোঝোসো সয়স4 আকো লেসক লেPম তোরো স পসর লেতমলি ীব লেদ রপ আকো লেসক পরো রপ তোরো স পরসত পোসর এই মভোবোর কোই কলিব বসস4 ঘ ) পরবো Pোতরোয় Pলিদ কলিবর লেদ আকো লেসক ীব তোরো রপ পরো স পসর তোসত কলিব লিবনদমোতর দঃলিত কোর মতয লিবসর সবোভোলিবক পলিরলিত এবং মোষ মরী তোই পরবোস Pলিদ তা োর মতয য় তবও কলিব লিবচলিত সব ো কোর পলিবীসত লেকউ অমর য় লিক4ই অকষয় য় দীর লেPম লিচরপরবোমো লেতমলি মোসষর ীবও চমোতোই ীব - সতবধতোই মতয ীব দীসত মোষ লিতয পরবোমো তবও লেPব মোষ আপ কতকসম13র মো3যসম মোসষর মস লিসসদর সথো কসর লিসত পোসর তোরো লিচরভোসবর সয় মোসষর মস লিবরো কসর তোসদর মস3য লেকউ পGভসত লিবী সয় গোসও মোসষর মস তোরো লিতযপলিত লিতযবলিনদত

Hindi 2ndlang

काकीी(लिसयारामशरणगपत)

इस कहानी म क न यह बतान का परयास निकया ह निक बचच अपनी मा स निकतना परम करत ह शयाम अबोध बाक ह वह अपनी मा क मरन क बाद उसन अपनी मा क लिए बहत रोया बाद म उस पता चा निक उसकी मा राम क घर ची गई ह आकाश म उडती हई पतग दकर उस हष हआ निक पतग क दवारा वह अपनी मा को नीच उतारगा इसक लिए वह अपनी निपता की जब स दो बार सवा रपया निनकाकर पतग और दो मोटी सी मन वाी अपन भाई स काकी एक कागज पर लिवा कर पतग म लिशव का दिदयानिनकाकर पतग और दो मोटी सी मन वाी अपन भाई स काकी एक कागज पर लिवा कर पतग म लिचपका दिदयाभोा और शयाम कोठरी म रससी बाधनी रह थ तभी उसक निपता करोध म आकर उन स पछ निक कया उनकी जब स रपया निनकाा हभोा डर क मार बताया निक शयाम इस पतग क दवारा अपनी काकी को राम क यहा स उतारना चाहता हनिवशशवर(शयाम क निपता)न फटी पतग उठाकर दी तो उस पर काकी लिा थावह हत बजि होकर वही ड रह गएउनहोन सोचा निक मन अपन पतर को मारा जोनिक अनजान और निनदष थावह अपनी मा कोनिकतना पयार करता ह

helliphellipContinue to next

Computer Application

Java Programming Prog 1Write a java program to input two numbers from user and display the sum or product of them as per user choice Use switch case statementSolve public class sum_product public static void main(String args[]) Scanner sc=new Scanner(Systemin) int abc Systemoutprintln(ldquoEnter two numbersrdquo) a=scnextInt() b=scnextInt() Systemoutprintln(ldquoPress 1 for sum or 2 for productrdquo)

c=scnextInt() switch(c) case 1 Systemoutprintln(ldquoThe sum will be =rdquo+(a+b)) break case 2 Systemoutprintln(ldquoThe product will be =rdquo+(ab)) break default Systemoutprintln(ldquoWrong Inputrdquo) Home Work - Practice in your computer using bluej

Subject Eng Literature (The Merchant of Venice ndash William Shakespeare)Topic Act I Scene 2 Lines 92 to 126 (End of scene) Date 13th April 2020 (5th Period)

[Students should read the original play and also the paraphrase given in the school prescribed textbook]Summary Questions amp Answers

o After Portia has expressed her opinion about the suitors Nerissa informs that she need not bother about any one of them as they have decided to quit Belmont at the earliest opportunity because they do not believe in trying their luck by the caskets which is the only way of winning Portia

o Nerissa then enquires of Portiarsquos opinion about Bassanio who once visited her in the company of the Marquis of Montferrat and says that she had never come across such an ideal love deserving the fairest lady for his bride

o Portia seems to remember Bassanio quite correctly and says that she agrees with Nerissa At this moment a servant informs Portia that the Prince of Morocco has arrived to try his luck by the caskets

o Portia tells Nerissa that if she could welcome this new suitor as gladly as she says farewell to the previous ones she would be glad of his arrival However if he happens to have the virtues of a saint but the black complexion of a devil she would prefer to have him for religious consolation rather than as a husband

(1) NERISSA You need not fear lady (Line 97-103)

the having any of these lords they have acquainted me with their determinations

which is indeed to return to their home and to

trouble you with no more suit unless you may be wonby some other sort than your fathers imposition depending on the caskets

PORTIA If I live to be as old as Sibylla I will die as chaste asDiana unless I be obtained by the manner of my fatherswill I am glad this parcel of wooers are so reasonablefor there is not one among them but I dote on his veryabsence and I pray God grant them a fair departure

(a) Elucidate the idea expressed in the first speech of the above dialogue

In the first speech Nerissa assures Portia that she need not have any fear of being compelled to marry anyone of the suitors who had lately come to Belmont She informs her that they have all decided to return to their respective countries(b) Illuminate the meaning of the phrase ldquoyour fatherrsquos imposition depending on the casketsrdquo

Nerissa means that the suitors of Portia do not find the conditions imposed by the will of her father to their liking They are too hard for them These conditions are that in the event of a suitor failing to choose the right casket (i) he should never disclose to anybody which casket he chose (ii) he can never marry and (iii) he should take his departure immediately(c) Explain the meaning of the term lsquoSibyllarsquo

lsquoSibyllarsquo is the name given by Romans and Greeks to a prophetess inspired by some deity usually the sun-god Apollo She had a very long life The god Apollo granted her as many years of life as she could hold grains of sand in her hand(d) Elucidate the meaning of the term lsquoDianarsquo

lsquoDianarsquo is the goddess of hunting She is also regarded as a symbol of virginity because she never fell in love and never

married(e) Explain the meaning of the first two lines of Portiarsquos speech

Portia says that even if she is to live for centuries like Sibylla she would not marry except in accordance to her fatherrsquos will She asserts that she would not mind remaining unmarried and untouched by a man like Diana the virgin the goddess of hunting unless a man is able to win her by passing the test laid down by her father

Class XSubject Topic Summary Execution

Hindi 2nd

Langबड घर की बटी( मशी परमचद)

lsquoबड घर की बीटीrsquo कहानी का उददशय मधयम वग की घर समसया को सझा कर सगदिठत परिरवार म मिम जकर परम स रहन का सदश दना ह घर म शानित tानिपत करन की जिजममदारी नारी की होती ह यदिद नारी समझदार ह उसम धय और परिरवार क परनित परम ह तो कोई भी घटना परिरवार को निवघदिटत नही कर सकती या कहानी परिरवार को सगदिठत करत हए परम सौहाद स एक रदसर की भावनाओ को समझ करउनका सहयोग करत हए जीवन यापन करन की पररणा दती ह मशीपरमचदर जी न इस कहानी म सयकत परिरवार का परनितनिनमिधतव निकया ह यह कहानी बनी माधव सिसह जो गौरी पर क जमीदार क उनक दो पतरो की हशरी कठ ा निबहारीशरीकात का निववाह एकजमीदार घरान की पतरी आनदी स हआ थाआनदी न द को ससरा क वातावरण म ढालिया थाएक दिदन आनदी का अपन दवर ा निबहारी स झगडा हो जाता ह दोनो भाई एक रदसर स अग होन की कोलिशश करत हसभी बह आनदी न अपन मधर वयवहार स ा निबहारी को घर छोडकर जान स रोक लिया| इस पर बनी माधव सिसह न कहा निक बड घर की बटी ऐसी ही होती ह जो निबगडा काम बना ती ह अतः शीषक साथक ह बड घर की बटी आनदी ह

helliphelliphelliphellipContinue to nextBiology Topic ndash Chp-1

CellWelcome to new session 2020-21Today we will start with Chpter 1 cell CELL

Protoplasm+Cellmembrane Or Cell wall

Cytoplasm+Neucleus

Cytoplasmic+ CytoplasmicOrganelles Inclutions(mitochondria (food Golgi bodies pigments)Ribosome)

What is cellbull Cell is the structural and functional unit of living organismbull According to number of cells organisms areUnicellular - Amoeba bacteria Multicellular - Rose Mango Tiger HumanSmallest cell -bacteria Longest cell - Nerve cellLargest cell - Ostrich egg cellCells are of different size and shapes according to their functionsQ2Write chief functions of following cellorganelles

Q3What is tonoplastVacuoles covered by a covering called tonoplast

Bengali(2Nd

Language)

ফ ফটক ো ফটক (কলিবতো ) ভোষ মসোপো3 gtPোয়

একটি লেমসয়র ীবস লেপরম লিকভোসব ফসট ওসঠ তো লেদলিসয়স4 কলিব লেপরম Pই য় লেই ময়ই বনত কোস পলিরত য় ফ লেফোটো বো োসফোটো লেটো ব2 কো য় লেমসয়সদর ব gtয13 লেপরসমর 4লিব ফসট উসঠস4 এই কলিবতোয় লেপরম মোষসক মত gtযর মস লেফস লিদসয় পরকষস বাোচোসোর gtয োত বো2োয় কলিবতোয় লেমসয়টির পসব13র দঃসর কো বো সও লেমসয়টি লেই পসর পলিক সত চোয়ো োরী ীবসর কোস4 পরম লেPৌবস লেপরমসক পোবোর পরব ইচছো োকসও তো পসর লেলিতবোচকতোয় পলিরত য় কলিব ভোষ মসোপো3 যোয় লেP ক লেপরসমর

কলিবতোয় ব gtযবহত লিবসষ লিক4 সvর অ13 লেদওয়ো ১) রসবোো= লেP লিবলিভনন রকম ডোকসত পোসর২) ো= পোর ৩) ঠলি = লেচোসর বZ৪)আই বস2ো=অলিববোলিত৫)শইসয় = োলিয়ত কসর৬)োতপাোচ= লিবলিভনন পরকোর৭)দ2োম = v কসর বZ কসর লেদওয়ো৮)লেরলিং =লেোোর দৈতরী লেব2ো৯) বনত= একঋত১০) পাোর = বসকরো2

Organelles Functions

1 Endoplasmic reticulum

2 Mitochondria

3Golgibodies

4 Ribosome5Lysosome

6Plastids

7 Centrosome

i) Supportive framework for the cellii) Synthesis and transpost of proteinsRelease of energy in the form of ATPi) Synthesis and secretion of enzymes hormoneii) Formation of vacuoles lysosomei) Protein Synthesisi) Intracellular digestionii) Destroy foreign substancei )Leucoplast - stores starchii)chloroplast - trap solar energyiii) Chromoplast - imparts colour toflowers amp fruitsi) Initiates and requlates cell division

কলিবতো তোর অ13সক ভোষোয় পরকোো কসর ঘলিরসয় ব যকত কসরস4 লেপরসমর ফতো আর লিবফতো লেক গোঢ় কসর লেদোসো কলিব ভোষ মসোপো3 যোসয়র অলিভবসর অ যলিদক

Economics

Factors of Production

Welcome to the new sessionToday we are going to start the first chapter of Class XThe name of the chapter is Factors of productionBy the name I hope you all can recall a glimpse of what you have learnt in the second chapter of Class IX

NowProduction is the process of creating the various goods and services which are consumed by the people of the country to satisfy their wants

Thus it is the process in which some materials are transformed from one form to another to create utility and value in goods

For example utility can be created by changing the form of a commodity ie

Making of table out of wood by a carpenter for his customer here the wood is getting transformed into table creating utility for his customer and he can also command a price for it

On the other hand Housewives perform very

useful activities at home which create utility but their domestic activities are not included in production because they have no money value

So we can also say that Production denotes two things firstly creation of utility and secondly creation of value

Production is not complete unless it reaches the consumer

An increase in production will increase the economic welfare of the consumers and hence the aim is to raise the production level of the country

Again production of a good or service is only possible if certain resources or

Questions

1 What do you mean by production

Answer Production means the creation of goods and services for the purpose of selling in the market

In fact production involves the transformation of inputs into outputs

Hence production denotes two thingsCreation of utility and creation of valueUtility and value can be created by changing the form by changing the place by changing the time and by rendering services

Example Transformation of raw

materials into finish goods such as potter creates utility by converting mud into utensils assembling of small parts to make bigger machinery

Production also includes services such as distribution and marketing

2 What are the factors of production

Answer Factors of Production refers to the resources and inputs needed for producing goods and servicesThese inputs can be classified as

Land Labour

Capital Enterprise

Land Land is defined to include not only the surface of the earth but also all other free gifts of nature(for example mineral resources forest resources and indeed anything that helps us to carry out the production of goods and services but is provided by

inputs are used together in right proportion

A resource or an input which helps in the process of production to obtain an output is called FACTOR OF PRODUCTION

These factors of production can broadly be categorized into four parts 1LAND 2LABOUR3CAPITAL4ENTERPRISE (ORGANISATION)or Entrepreneur

The above factors are all interdependent on each other and they play a major role in production process

FACTORS OF PRODUCTION

LANDCAPITAL

LABOUR ENTREPRENEUR

nature free of cost)LabourLabour refers to the human efforts that need to be combined with other factors of production for creating an output

CapitalAll man ndash made means of production is called capita example machineries which help in further production Money when used for starting any business for purchasing raw materials machinery tools etc it is regarded as capitalCapital also includes physical capital like factories machineriestoolsbuildingsequipments etcEnterpriseThe task of bearing risks is called enterprise and the person who bears these risks of business is called the entrepreneurThus an entrepreneur is one who organises production takes important decisions regarding production hires and purchases factors of production and bears the risk and uncertainty involved in productionOrganisation refers to the services of an entrepreneur who controls organises and undertakes all risks One who plans organises and manages a business enterprise is an organiser

Physics Chapter 1 Force

Force is an external agent capable of changing the state of rest or motion of a particular body It has a magnitude and a direction The direction towards which the force is applied is known as the direction of the force and the application of force is the point where force is applied The Force can be measured using a spring balance The SI unit of force is Newton (N)

Question 1

State the condition when on applying a force the body has

(a) the translational motion

(b) The rotational motion

Solutions

(a) Translational motion is produced when the body is free to move

(b) Rotational motion is produced when the body is pivoted at a point

Question 2

Define moment of force and state its SI unit

Solutions

The moment of force is equal to the product of the magnitude of the force and the perpendicular distance of the line of action of force from the axis

of rotation

The SI unit of moment of force is Newton times meter

= Newton meter (Nm)

Commercial Studies

Stake holders In this topic you will be come to know about the meaning and concept of stakeholders

How stakeholders are different from shareholders

Questions1 What do you mean by the term stake holdersAnswer) The term stake holders have developed from the words which mean an interest or expected benefit Stakeholders mean all those individuals groups and Institutions which have a state (interest) in the functioning and performance of a commercial organisation or a business enterprise2 What do you mean by share holdersAnswer) The person and Groups who own the shares of the joint stock company by providing capital to the company are called shareholders Shareholders are the internal stakeholders shareholders are one out of several stake holders3 How are shareholders different from stakeholdersAnswer)i) The term shareholders is related to only joint stock company whereas stakeholders are related with all business organisationsii) Stakeholders maybe any individual having financial stake in business organisation whereas a shareholders are those individuals who are holding shares in the company4) How are shareholders different from creditorsAnswer) i) Shareholders are internal stakeholders while creditors are external stakeholdersii) Shareholders invest in the capital of the company whereas creditors give loan to the companyiii) Shareholders are the members of the company with voting rights but creditors are not the members of the company

English 1 Transformation of sentences

Sentences A sentence is a group of words which makes complete sense

e Assertive sentencesf Imperative sentencesg Interrogative sentencesh Exclamatory sentences

Sentences can be changed from one grammatical form to another without changing the meaning of the sentence This is known as transformation of sentences

Exercise 1 Change the following affirmative sentences into Negative sentences

a He is a good manHe is not a bad man

b Ram loves SitaRam is not without love for Sita

c Only he stood first in the classNone but he stood first in the class

d Ankit was wiser than he

He was not so wise as Ankite He did it

He did not fail to do itf As soon as I reached college the

bell rangNo sooner did I reach college than the bell rang

g He finished everythingHe left nothing unfinished

h It always pours when it rainsIt never rains but it pours

Math Topic Commercial MathematicsChapter ndash Goods and services Tax

What is GSTAns It is a abbreviated term of Goods and Service Text which is an indirect tax levied on the sale of goods and rendering servicesSome terms related to GSTDelar Any person who buys goods or services For resale is known as a delar A delar Can be a firm or a companyIntra-state sales Sales of goods and services within the same state or same union territory are called intra- state salesInter-state sales Sales of goods and services outside the state or union territory are called Inter-state sales4) Input GST GST is paid by dealers on purchase of goods and services are called input GST5) Output GST GST is collected from customers on sale of goods and services are called output GST6) Types of GST There are three taxes applicable under GST(i) Central Goods and Services Tax (CGST)(ii) State Goods and Services Tax (SGST) or Union Territory Goods and Services Tax (UTGST) Both these taxes are levied on intra-state sales Here GST is divided equally among central and state governments(iii) Integrated Goods and Services Tax (IGST) IGST is levied on inter- state sales It is also levied on import of goods and services into India and export of goods and services from India

Subject Eng Literature (The Merchant of Venice ndash William Shakespeare)Topic Act III Scene 4 Lines 1 to 44 (Portia hellip To wish it back on you fare you well Jessica)[Students should read the original play and also the paraphrase given in the school prescribed textbook]

Summary Questions amp AnswersIn this scene we suddenly find a new element in the character of Portia We have already seen her possessed of every graceful womanly quality but now she shows that she is capable of rapid decision and determined action She shows this by her sudden resolve to hasten to Venice with a daring scheme for the rescue of Antonio This is an important scene in the dramatic action for it leads up to and renders possible the striking events of the famous trial scene which is one of the greatest striking elements of the play Moreover the fact that all the characters of importance are now assembled together in Venice makes the union of the main plot and the secondary story complete

(1) LORENZO Madam although I speak it in your presence(Line 1-9)

You have a noble and a true conceit

Of god-like amity which appears most strongly

In bearing thus the absence of your lordBut if you knew to whom you show this honourHow true a gentleman you send reliefHow dear a lover of my lord your husbandI know you would be prouder of the workThan customary bounty can enforce you

(a) Where is Lorenzo Why is he here To whom is he referring as lsquoMadamrsquo

Lorenzo is at Portiarsquos residence He had met Salerio on the way and Salerio had begged him to come along with him to

o In this scene Portia Nerissa Lorenzo Jessica and Balthazar appear

o Portia requests Lorenzo and Jessica to be in charge of her house during her absence from Belmont because she and Nerissa have decided to spend the days in meditation and also in visiting the holy places in the neighbourhood of Belmont She has already instructed her people to acknowledge both Lorenzo and Jessica as master and mistress of house during her absence Lorenzo and Jessica gladly agree to look after the house of Portia

handover the letter from Antonio to Bassanio The letter carried the bad news about Antoniorsquos arrest for non-payment of loan taken from Shylock Hence Salerio might have preferred company to break this bad news to Bassanio He is referring to Portia as Madam(b) What does Portia say on hearing the above extract

Portia says that she has never regretted doing good to others Friends who spend a lot of time together and really are there for each other have many traits in common As Antonio is Bassaniorsquos best friend saving him is like saving Bassanio who is like her own soul She asks Lorenzo to take care of management of the house till Bassanio is back(c) What does Portia send with Bassanio and why

On hearing about Antoniorsquos troubles on account of Bassanio her husband Portia immediately sends him with enough gold to repay the debt many times over to Venice to help Antonio out of his misfortune

(2) Lorenzo Madam with all my heart (Line 36-40)

I shall obey you in all fair commands

Portia My people do already know my mindAnd will acknowledge you and JessicaIn place of Lord Bassanio and myselfSo fare you well till we shall meet again

(a) Where are Lorenzo and Portia at this time What lsquofair commandsrsquo are given to Lorenzo

Lorenzo and Portia are at Belmont during this scenePortia reveals to Lorenzo that she has sworn to contemplate in prayer at a monastery around two miles away until her husband returns from Venice She tells him that Nerissa would accompany her and asks him to manage the house with Jessica till things are settled In response Lorenzo tells her that he would be obliged to do whatever she asks him to do(b) Where is Portia actually going and why

Portia tells Lorenzo that she would live a life of contemplation and pray at a monastery which is two miles away from her place In reality Portia plans to go to Venice in disguise with Nerissa and argue the case in defense of Antonio She is very sure that her plan would succeed

ClassXI (ScienceHumanitiesCommerce)Subject Topic Summary Execution

Computer Science

(APC)

Ch ndash 1 Numbers

(Numbers in different bases and

their Arithmatical operations)

Number System In computers Number System is defined as a writing system to represent the numbers in different ways ie we are using different symbols and notations to represent numbers There are four ways we can represent the number ndash Binary Decimal Octal and Hexadecimal

Decimal Number SystemThis number system consist 10 digits These are 0 1 2 3 4 5 6 7 8 amp 9

Binary Number SystemThis number system has only two digits these are 0 and 1 Here 0 stands for off while 1 stands for on

Octal Number SystemThis number system has 8 digits these are 0 1 2 3 4 5 6 amp 7

Hexadecimal Number SystemThis number system has 16 digits these are 0 1 2 3 4 5 6 7 8 9 A B C D E F Here the value of the alphabets are as follows A=10 B=11 C=12 D=13 E=14 F=15

Rules for conversion decimal number to Binary1 Divide the decimal number by 22 If the number will not divide equally by 2 then round down the answer to the nearest whole number (integer)3 Keep a note of the remainder it should be either 0 or 14 Keep repeating the above steps dividing each answer by 2 until you reach zero5 Write out all the remainders from bottom to top This is your binary solution

For example Lets convert 32 to binary 2 32 2 16 - 0 2 8 - 0 2 4 - 0 2 2 - 0 2 1 - 0 0 - 1

The binary equivalent of 3210 is 1000002

Try the follwing youself1 2410

2 4810

3 1210

History GROWTH OF NATIONALISM

The second half of the 19th century witnessed growth of political consciousness and a sense of Nationalism among the IndiansThere were various factors for growth of Indian Nationalism- As a result various political associations were formed in different provinces by the educated Indians Surendranath Banerjee organized a meeting of National conference at Calcutta Ultimately the National Congress was founded in Bombay in 1885This body became the vanguard of Indian struggle for freedom The congress leaders were known as moderates because they followed a policy of prayer and petition A large number of Indian leaders had experienced in political agitation The Political situation of England was also changed Moreover increasing revolutionary activities in Maharashtra Punjab and Bengal became serious concern to the British Government In this

QUESTION1 What do you mean by Nationalism ANSWER 1 Nationalism is defined as loyalty and devotion to own nation especially a sense of national consciousnessQUESTION 2 What are the causes of nationalism ANSWER 2 There were various factors for growth of nationalism

1 Spread of western education2 The progress of vernacular press and

patriotic literature3 The economic exploitation of our

country by the colonial rulers4 International affairs

QUESTION 3 Who organized National conference in Calcutta in 1883 ANSWER 3 Surendranath BanerjeeQUESTION 4 When did Indian National Congress formANSWER 4 Indian National Congress was formed in 1885 in BombayQUESTION 5 Who were ModeratesANSWER 5 The Early Nationalists were also known as Moderates Their emergence marked

background Lord Curzon became Viceroy in India He had no respect for the Indian National Congress

the beginning of the organized national movement in India They believed in British justice and were loyal to them They followed a policy of prayer and petition They demanded constitutional reforms of our country Impotant Moderate leaders were Pherozshah Mehta Dadabhai Naorozi and Surendranath Banerjee etcQUESTION 6 What do you know about Extremism in Indian National movementANSWER 6 In the beginning of 20th century a new class of national leaders emerged in India which was different from the moderate groups They started more aggressive movement against the British empire The goal of extremists was ldquoswarajrdquo Important extremist leaders were Bal Gangadhar Tilak Lala Lajpat Rai Bipin Chandra Pal etcQUESTION 7 Mention the places which were the main centres of Revolutionary movementANSWER 7 Maharashtra Bengal and Punjab

Physics

Chapter Dimensional Analysis

(Summary)

The dimensions of a physical quantity are the powers to which the fundamental units are raised in order to obtain the derived unit of that quantit

The physical quantites lengthmasstime are represented by [L] [M] [T] resp let they are raised to powers ( dimesions) abc resp then any physical quantity can be represented by [ La Mb Tc ] Examples

1 Area area = L x B = [L] x [L] = [M0 L2 T0 ]

2 Density density = massvolume = [M][L3] = [ M L-3]

3 Velocity velocity = distancetime = [L][T] = [LT-1]HW Try to find out dimension of acceleration Acceleration = velocity timeNB One can find the SI Units Using Dimension Analysis Such as for area we have [L2] so its SI unit is m2

Biology Topic ndash Chp-1 The living world

Today we will start the first chapter the living world Here we discuss about the characteristics of living organism and what are the difference between them and nonliving substances We also discuss about the contribution of different Scientists

There are over 500000 species of plants andover a million species of animal are present on earth Some 15000 new species were discovered every yearQ1 What is a living organismbull A living organism is primarily physico -chemical material that demonstrate a high degree of complexity is capable of selfRegulation possesses a metabolism and perpetuates itself through timeQ2 What are the differences between livingand non-livingsi) Compared with non-living living organisms

have more complex organised structure and their use of energy is more controlled amp efficientii) Living things reproduce their own kind by forming new cells which contains copies of their genesiii) Each organism has some degree of homeostasisie it is able to make adjustments so that internal environment remains constantQ3 Write contributions of following Scientists i) Aristotle - One of the first theories in Biology places all living things in a hiearchieii) AV Leeuwenhoek - was the first to observe living single celled organisms under microscopeii) Carolus Linnaeus - developed the binary system for naming of organisms and classificationiii) Geregor Johann Mendel ndash discoverbasic principles of inheritanceHomework i) C Darwin ii)Schleiden

Math Trigonometric functions

1 Overviewi) Trigonometry The word lsquotrigonometryrsquo is derived from the Greek words lsquotrigonrsquo and lsquometronrsquo which means measuring the sides of a triangle An angle is the amount of rotation of a revolving line with respect to a fixed line Usually we follow two types of conventions for measuring angles ie a) Sexagesimal system b) Circular system In Sexagesimal system the unit of measurement is Degree In Circular system the unit of measurement is Radian ii) Relation between degree and radianThe ratio of circumference of a circle to its diameter is always a constant This constant ratio is a number denoted by π which is taken approximately as 227The relationship between degree amp radian measurements is as follows2 right angles = 180deg= π radians1radian = 180degπ=57deg16(approx) 1deg=π180 radianiii) Length of an arc of a circleIf an arc of length s subtends an angle θ radians at the center of a circle of radius r then s=rθiv) Area of a sector of a circleA sector is like a pizza slice of the

Q) Express the following angles in radiana) 45deg b) 40deg3730Ans a) We have 180deg=π radiansi e 45deg= πtimes45180 radian = π4 radiansb) 40deg3730= 40deg37+3060 minute= 40deg 37 +12 minute= 40deg+ 752 minute=40 + 75(2times60) degree=3258 degreeNow 180deg=π radianie 3258 degree= (πtimes325) (180times8) radians = 65π288 radiansQ) A circle has a radius of r=12 meters What is the length of an arc traced out by a 60deg angle in the center of the circleAns In this problem we know both the central angle (60deg) and the radius of the circle (12) All we have to do is plug those values into our equation and we get

s = 2π(12)(60360)s = 24π6s = 4πSo the length of an arc traced out by a 60deg angle in a circle with a radius of 12 meters equals 4π meters asymp 1257 metersQ) Find the area of the sector with a central angle 30deg and a radius of 9cmAns GivenRadius r = 9 cmAngle θ = 30degArea of the sector = θ360degtimesπr2

= 30360degtimes227times92=2121cm2

circle It consists of a region bounded by two radii and an arc lying between the radiiThe area of a sector is a fraction of the area of the circle This area is proportional to the central angle In other words the bigger the central angle the larger is the area of the sectorArea of Sector = θ2 times r2 (when θ is in radians)

Area of Sector = θ times π360 times r2 (when θ is in degrees)

COMMERCE

CLASSIFICTION OF HUMAN ACTIVITIES-ECONOMIC AND NON-ECONOMIC

Welcome to the new sessiontoday we are going to start the first chapter of Class XI The name of the chapter that we are going to start is

lsquoClassification of Human Activities ndasheconomic and non-economicrsquo

Now let us start the chapter by considering human beings and the activities they perform throughout the day

Human activities means all those activities that human beings undertake to satisfy their wants

Human wants on the other hand are the desire of human beings for goods (vegetables fruits rice etc) and services (services of doctors teachers lawyers etc) that they require to live

Now these human activities continue throughout life as human wants are unending unlimited and recurring as human beings desire for better living throughout their lives

Now human activities can be classified into two categories

Human activities

Economic activities Non-economic activities

Economic activities are

Questions1 What are human activities

Answer Human activities mean all those activities that human beings undertake to satisfy their wants

Example A man working in an office

A boy playing in the garden

2What are the characteristics of human activitiesAnswer the characteristics of human activities are as follows

Human activities are undertaken by men women and children and these activities involve human efforts

Human activities are undertaken to satisfy human wants which are unlimited

Human activities continue throughout life

Human activities are performed for both earning money and personal satisfaction

3What is economic activitiesGive example

Answer Economic activities are undertaken by human beings with the object of earning money acquiring wealth and thereby satisfying human wantsExample

Selling of goods by a shop keeper to his customer

A clinic run by a doctor Service of a teacher in school or college

undertaken by human beings with the object of earning money and acquiring wealth

These activities result in the production of economic goods and services

Example Human activities(ie working in factories officesshops) which produce direct economic benefits

Non-economic activities are inspired by human sentiments and emotions such as love for the family desire to help the poor and love for the country

Thus these human activities (eg praying playing sleeping) produce no direct economic benefits and they are also not related to earning money and acquiring wealth

4 What are the characteristics of economic activities

Answer The characteristics of economic activities are as follows

Economic motiveEconomic activities are undertaken to earn money and acquire wealth

ProductiveEconomic activities involve productiondistribution and exchange of goods and services to create wealth

Economic growthEconomic activities determine the level of economic development of a country and standard of living of its citizens

Socially desirableEconomic activities are socially desirable for society

Economic resourcesEconomic activities make use of all the economic resources such landlabourcapital etc

5 What do you mean by non-economic activitiesExampleAnswerNon-economic activities are inspired by human sentiments and emotions such as love for the family desire to help the poor and love for the countryThese activities are not undertaken for monetary gain but for onersquos satisfaction and happinessExample

a mother looks after her children

a student donates blood8 Differentiate between Economic activities and Non-economic activities

Economic activities

Non-economic activities

1to earn living and acquiring wealth2Result can be measured in terms of money

3ExampleBusinessprofession and employment

1 to obtain some satisfaction

2Result cannot be measured in terms of money

3ExampleFamily-orientedreligious socialCultural and national

BUSINESS STUDIES

BUSINESS ENVIRONMENT

Welcome to the new sessionToday we are going to start the first chapter and the name of the chapter is Business Environment

In todayrsquos world every business enterprise is a part of the society It exists and operates in association with various groups in society such as customers suppliers competitors banks and financial institutions government agencies trade unions media and so on All these groups influence the functioning of business in one way or the other They constitute the environment of businessConcept of Business Environment

The term lsquobusiness environmentrsquo refers to the sum total of all individuals institutions and other forces that lie outside a business enterprise but that may influence its functioning and performance

The main features of business environment

Totality of External forces General and Specific forces Interrelatedness Complexity Dynamic Uncertainty Relativity

The Interrelation between business and its environment

The business enterprise is an open system It continuously interacts with its environment It takes inputs

Prepare the following questions from todayrsquos assignment

1 What do you mean by business environment

The term lsquobusiness environmentrsquo means the aggregate of all forces factors and institutions which are external to and beyond the control of an individual business enterprise but they may influence its functioning and performance Business environment is the macro framework within which a business firm a micro unit operates It consists of several interrelated and interacting elements

2 Explain the main features of business environment in brief

Totality of External forces-Business environment is the sum total of all things external to a business environment

General and Specific forces-It includes both the forces general forces are the economic social political legal and technological conditions which indirectly influence all business enterprise Specific forces are the investors customers competitors and suppliers which influence individual enterprise directly

Interrelatedness-Different elements of environment are interrelated for an example growing awareness for health care has increased the demand for health foods

Complexity- Business environment id

(such as raw materials capital labour energy and so on) from its environment transforms them into goods and services and sends them back to the environment

Fig 1 Business Environment Relationship

complex in nature as the elements keep on changing example economic technological and other forces changes in demand for a product and service

Dynamic-Business environment is not static it keeps on changing

Uncertainty- Itrsquos very difficult to predict future events such as technology and fashion which occur fast and frequently

Economics Basic Economic ConceptsSub topic

Microeconomics and

Macroeconomics

Welcome to the new sessiontoday we are going to start the first chapter of Class XI The name of the chapter that we are going to start is Basic Economic concepts

Now Economics covers the study of human activities Human activities are those activities which are performed by humans to satisfy their wants

Thus Human wants are unlimited and therefore economic activities such as production exchange and consumption are needed in order to satisfy those wants

The study of economics is divided largely in two parts which areMicroeconomics and Macroeconomics

SUBJECT- MATTER OF ECONOMICS

MICROECONOMICS MACROECONOMICS

Questions1Who has coined the words micro and macro economics

Answer Ranger Frisch coined the words lsquomicrorsquo and lsquomacrorsquo in 1933 to denote the two branches of economic theory namely microeconomics and macroeconomics

2What is microeconomicsAnswer It is the study of behaviour of individual decision ndash making unit such as consumers firms etc

3 What is macroeconomicsAnswer Macroeonomics is the study of overall economic phenomena like employment national income etc

4 What is the importance of microeconomicsAnswer

Microeconomics helps in formulating economic policies which enhance productive efficiency and results in greater social welfare

It helps the government in formulating correct price policies

It explains the working of a capitalistic economy where individual units(producers and consumers ) are free to take their own decision

Micro means a small part in

microeconomics we do not study the whole economy Hence we study an individual consumer and his or her choices and a producer and his or her profit maximizing decisions in the market Thus it does not mirror what happens in the economy as a whole

Macroeconomics on the other hand studies the economy as a whole It is concerned with aggregate and depicts the entire picture of the economyMacroeconomics deals with the national income aggregate investment aggregate consumption etc

Features of Microeconomics It deals with small

parts of the country Hence it looks at

individual consumers firms and industries

It deals with individual income consumption and savings

It studies the determination of price of any product or factors of production

It deals with the working of market via the price mechanism which is nothing but the determination of price and quantity of a commodity by the forces of demand and supply

Features of Macroeconomics

It deals with the study of the economy as a whole

It is concerned with

5 Give a limitation of microeconomics Microeconomics fails to explain the

functioning of an economy as a whole It cannot explain unemployment illiteracy and other problems prevailing in the country

6 What is the importance of macroeconomics It gives overall view of the growing

complexities of an economic system It provides the basic and logical

framework for formulating appropriate macroeconomic policies (eg for inflation poverty etc )to direct and regulate economy towards desirable goals

7What is the limitation of macroeconomics It ignores structural changes in an

individual unit of the aggregate

8 Differentiate between Microeconomics and Macroeconomics

Microeconomics Macroeconomics

the study of aggregates

National income aggregate savings and aggregate investments are major concepts dealt within macroeconomics style

It studies the determination of general price levels

It investigates into the problem of unemployment and the achievement of employment

It studies the aspect of decision making at the aggregate and national levels

It includes all growth theories whether related to developed or developing economies it also includes the study of economic systems and the working of the economy under different systems

Note Both Micro and macro economics are complementary and should be fully utilized for proper understanding of an economy

1It studies economic aspect of an individual unit2It deals with individual incomeConsumption and savings

3 It facilitates determination of price of any product or factors of production

4 Itrsquos scope is narrow and restricted to individual unit

1It studies the economy as a whole

2It deals with the national income aggregate consumption and aggregate savings3 It facilitates determination of general price level in an economy

4 Itrsquos scope is wide as it deals with economic units on the national level

ACCOUNTS

Introduction to Accounting and Book-keeping

Today I am going to share you the meaning of Accounting and Book-keeping and its related terms bullAccounting bullBook Keeping bullAccountsbullTypes Of Accounts bullAccounting Cycle

bull Meaning of accounting

Ans ) Accounting is the art and science of recording classifying and summarising monetary transactions

bull Meaning of Book-keeping

Ans) Bookkeeping is the art of recording business transactions with the view of having a permanent record of them and showing their effect on wealth

bull Meaning of account

Ans) The term account means a record of

business transactions concern a particular person of firm asset or income or expense It is a summarised record of all transactions which take place in an accounting year

bull Types of accountsPersonal accounts ndash Personal accounts relating

to person and Organisation are known as personal accounts Example Ramrsquos Account ABC amp Co Account etc

Real account - The accounts related to tangible and intangible assets are called real accounts Example Cash Account Furniture Account etc

Nominal account- Accounts related to expenses losses incomes and gains are known as nominal accounts Example Wages Account Salary Account Discount Account etc

bull Accounting cycle Accounting cycle refers to a complete sequence of accounting activities It begins with recording of transactions and ends with the preparation of a balance sheet

Chemistry TopicAtomic Structure

Thomsonrsquos atomic modelThomson (1898) was the first to propose the model of an atomHe proposed that an atom can be regarded as a uniform sphere of positive electricity in which requisite number of electrons are embedded evently to neutralize the positive chargeThis is just like plums embedded in a pudding or seeds evently distributed in red spongy mass of a watermelonThis model of atom is known as ldquoPlum-Pudding modelrdquo or

Q1)What is the fundamental constituents of atomAns Electron Proton and neutrons are the fundamental constituents of atomQ2)What is the value of fundamental unit of electricityAnsThe charge carried by one electron is sad to be the fundamental unit of electricityIts magnitude is 48times10-10esuOr 1602times10-19C Q3)Name the element containing no neutronAnsOrdinary hydrogen atom or protium 1H1

Types of AccountPersonal AccountReal AccountNominal AccountBalance Sheet (opening)

ldquowatermelon modelrdquoThis model could explain the electrical neutrality of an atom but failed to explain the result of scattering experiment carried out by Rutherford in 1911So it was rejected ultimately

Q4)Why is an electron called universal particleAns Itrsquos mass and Charge are independent of its source

EVS Chapter 1 ndash Modes of Existence

Modes of existence When one speaks normally about the mode of existence of some group or individual one refers to their customs their mode of being their ethology their habitat in some way their feeling for a placeDifferent modes of exixtence are ndash

1 Hunting ndashGathering2 Pastoral3 Agricultural4 Industrial

1 Hunting and gathering Hunting and gathering mode of existence is characterized by obtaining food from hunting wild animals including fishing and gathering wild plants From their earliest days the hunter-gatherer diet included various grasses tubers fruits seeds and nuts Lacking the means to kill larger animals they procured meat from smaller game or through scavenging

Societies that rely primarily or exclusively on hunting wild animals fishing and gathering wild fruits berries nuts and vegetables to support their diet are called hunting and gathering societies

At least this used to be practice of human beings before agriculture is invented As their brains evolved hominids developed more intricate knowledge of edible plant life and growth cycles

Q) Write the features of Hunting ndash gathering societiesAns - There are five basic characteristics of hunting and gathering societies

i The primary institution is the family which decides how food is to be shared and how children are to be socialized and which provides for the protection of its members

ii They tend to be small with fewer than fifty members

iii They tend to be nomadic moving to new areas when the current food supply in a given area has been exhausted

iv Members display a high level of interdependence

v Labor division is based on sex men hunt and women gather

Political Science

Introduction to political science

Political science occasionally called politology is a social science which deals with systems of governance and the analysis of political activities political thoughts associated constitutions and political behaviorThe study of political science involves the study of both the

Answer the following questions-1 What is political science

Political science occasionally called politology is a social science which deals with systems of governance and the analysis of political activities political thoughts associated constitutions and political behavior

2 Short notes-

traditional and modern theories of politicsTraditionalClassical political sciencepolitical theory-Traditional political science is the study of politics before Second World War The methodology to study Politics was traditional (legal formaletc) the definition of politics traditional (Politics begins and end with state)area of study (constitution state machinery)was traditionalModern Political scienceModern political theory-Modern Political Theory critically examines the contemporary state of political theory making an assessment of the achievement and limitations of the Behavioural Revolution in its totality and reviews objectively the major paradigms and conceptual frameworks adopted by the disciplineContemporary attempts at the development of an integrated political theory involving the use of both traditional and modern concepts approaches and theories-Around late 1960s several political scientists realized the importance of both the traditional political theory and modern Political theory They began building an integrated theory of politics involving a systematic mixture of traditional and modern studies of politics It was held that the study of a complex and vast field like politics needs both traditional as well as

Classical political theory Modern Political theory

Homework-Learn

modern concepts and approaches for studying itrsquos all aspects

Subject Eng Literature (The Tempest ndash William Shakespeare) Topic Act I Scene 1 Lines 1 to 32 (Line 32 ndash Gonzalo hellip If he be not born to be hanged our case is miserable) Date 13th April 2020 (3rd Period)

[Students should read the original play and also the paraphrase given in the school prescribed textbook]Summary Questions amp Answers

[SUMMARY OF THE ENTIRE SCENE]

o The play starts with the scene of a severe storm at sea Alonso (King of Naples) Sebastian (Alonsorsquos brother) Ferdinand (Alonsorsquos son) Gonzalo Antonio (the usurping Duke of Milan) are in a ship in the midst of the storm

o The mariners are trying their best to control the vessel from running aground and are totally following the orders of their Master the Boatswain They have scant success

o The mariners become extremely unhappy and annoyed when most of the passengers arrive on the deck thereby hampering their effort to save the ship There is serious confrontation between them and the passengers who are part of the Kingrsquos entourage

o The mariners could not save the ship

SUMMING-UP

(i) Vivid description of the scene which gives a realistic description of terror and confusion of a tropical storm

(ii) Shows Shakespearersquos accuracy of knowledge in describing the naval operations and also matters of seamanship

(iii) The opening scene justifies the title ndash The Tempest

UNANSWERED QUESTIONS

(i) The King always travels with his entire fleet including his soldiers Where

(1) GONZALO Nay good be patient (Line 15-26)BOATSWAIN When the sea is Hence What cares these

roarers for the name of the king To cabin silence Trouble us not

GONZALO Good yet remember whom thou has aboardBOATSWAIN None that I more love than myself You are a

councillor if you can command these elements to silence and work

the peace of the present we will not hand a rope more use your authority If you cannot give thanks you have

lived so long and make yourself ready in your cabin for the mischance of the hour if it so hap [To the Mariners]

Cheerly good hearts [To Gonzalo] Out of our way I say

(a) To whom is the boatswain speaking What does he mean by lsquoNone that I more love than myselfrsquo

The Boatswain is speaking to Gonzalo the honest old councilor of the Duke of MilanBy using the words ndash lsquoNone that I love more than I love myselfrsquo means that for the Boatswain nobody is dearer to him than his own life

(b) What were the conditions that made the boatswain react in this way

The Boatswain reacts in this way because the storm is at sea and Alonso King of Naples Sebastian his brother Ferdinand his son Gonzalo Antonio the usurping Duke of Milan on board are in distress and in panic Thus they have rushed to the deck interrupting the work of the mariners

(c) What hope does Gonzalo take from the attitude of the boatswain

The insolent and authoritative attitude of Boatswain makes Gonzalo feel comforted He tells that there are no signs that the Boatswain will be drowned But his facial appearance and attitude shows that he is destined to die on land by hanging which in effect means that all on board will be saved Otherwise all the persons on board are doomed

(d) How can they lsquomake yourself ready in your cabinrsquo For what were they asked to make ready themselves

In order to make themselves ready in their cabin the

were the other ships

(ii) Why was the ship in that area Where was it coming from or going where

(iii) The ship broke apart What happened to those who were in the ship

passengers on board must prepare for death which they will possibly soon have to meetThey can retire to their cabins and offer prayers to the Almighty to save them from drowning

(e) What does the boatswain say when he is asked to be patient What does he order to the royal party

When the boatswain is asked to be patient and remain calm he says that he will be patient only when the storm will be over and the sea will be calm but as long as the storm blows and there is danger to the ship he cannot think of being patient He orders the royal party to go to the cabin and leave the mariners to their work

(2) GONZALO I have great comfort from this fellow (Line 27-36)

Methinks he hath no drowning mark upon him his complexion is perfect

gallows Stand fast good Fate to his hanging Make the rope of his destiny our cable for our own doth little advantage If he be not born to be hanged our case is miserable

(a) Why does Gonzalo regard the Boatswain in the midst of danger

In the midst of danger Gonzalo regards the boatswain because he feels that the Boatswain is a source of comfort and is bent upon to do his work sincerely which in this case is saving the ship and its passengers from the severest of raging storm

(b) What reasons does Gonzalo give when he says that none in the ship will die of drowning

Gonzalo is almost sure that none in the ship will die by drowning His says that there is no mark on the face of the boatswain that indicates that he will die by drowning On the other hand the lines on his face are strong indications that he will be hanged to death Therefore there shall be no danger of the shiprsquos sinking

(c) Explain the following ldquoStand fast good Fate to his hanging Make the rope of his destiny our cable for our own doth little advantage If he be not born to be hanged our case is miserablerdquo

The stated lines mean that if the will of destiny is to be carried out then the ship will not get wrecked and all the passengers will be saved The safety of the passengers therefore depends upon the will of fate being carried out in the case of the boatswain If however the boatswain is not to die by hanging then the passengers are also very unsafe because in that case the ship is likely to sink

(d) What order does the Boatswain give to the sailors

when he re-enters What does he say about the crying of the fellows inside the cabin

The boatswain orders the sailors to bring the topmast lower and bring the ship close to a stationary position with the help of the main sail He says that the fellows inside the cabin are moaning and crying in their distress louder than his voice and louder even than the roaring of the storm

Class XII (ScienceCommerceHumanities) Subject Topic Summary Execution

Computer Science

PropositionalLogic

Propositional logic is a procedure to provide reasoning through statementProposition A ststement that results in True or False is said to be proposition There are two types of propositionSimple proposition amp compound propositionSimple proposioton A simple proposition is one that is not a part of any other proposition Such sentential form of proposition is symbolized with english letters in short For example Ram is a claver student (TrueFalse)Where do you live (Not in True or False)Grapes are sweet (TrueFalse)It rains today (TrueFalse)Here we can see some statements anwer would be true or false but some staements answer can not give in terms of true or false Thus the sentences which can be answered in true or false are known as simple propositionAssigning propositon to a variableThe general syntax to assign propostion to a variable is as followsVariable = Simple propositonFor example A=Ram is a clever studentB= Grapes are sweetC= it rains todayCompound proposition

helliphellipto be continued in next classhelliphellipMath Relation Relation If A and B are two non-empty sets

then a relation R from A to B is a subset of AxB If R A x B and (a b) R then we say that a sube isinis related to b by the relation R written as aRbeg Let A be the set of students of class XII and B be the set of students of class XI Then some of the examples of relation from A to B arei) (a b) AXB a is brother of bisinii) (a b) AXB age of a is more than age of isinb Types of relation In this section we would like to study different types of relations We know that a relation in a set A is a subset of A times A Thus the empty set φ and A times A are two extreme relations For illustration consider a relation R in the set A = 1 2 3 4 given by R = (a b) a ndash b = 10 This is the empty set as no pair (a b) satisfies the condition a ndash b = 10 Similarly R = (a b) | a ndash b | ge 0 is the whole primeset A times A as all pairs (a b) in A times A satisfy | a ndash

Example 1 Let A be the set of all students of a boys school Show that the relation R in A given by R = (a b) a is sister of b is the empty relation and R = (a b) the primedifference between heights of a and b is less than 3 meters is the universal relationSolution Since the school is boys school no student of the school can be sister of any student of the school Hence R = φ showing that R is the empty relation It is also obvious that the difference between heights of any two students of the school has to be less than 3 meters This shows that R = A times A is primethe universal relation Example 2 Show that the relation R in the set 1 2 3 given by R = (1 1) (2 2) (3 3) (1 2) (2 3) is reflexive

b | ge 0 These two extreme examples lead us to the following definitionsDefinition 1 A relation R in a set A is called empty relation if no element of A isrelated to any element of A ie R = φ A times AsubDefinition 2 A relation R in a set A is called universal relation if each element of A is related to every element of A ie R = A times A Both the empty relation and the universal relation are some times called trivial relation Definition 3 A relation R in a set A is called(i) reflexive if (a a) R for every a Aisin isin(ii) symmetric if (a1 a2) R implies that (aisin 2a1)

R for all aisin 1 a2 Aisin(iii) transitive if (a1 a2) R and (aisin 2 a3) R isinimplies that (a1 a3) R for all aisin 1 a2 a3 AisinDefinition 4 A relation R in a set A is said to be an equivalence relation if R is reflexive symmetric and transitive

but neither symmetric nor transitiveSolution R is reflexive since (1 1) (2 2) and (3 3) lie in R Also R is not symmetric as (1 2) R but (2 1) isin notinR Similarly R is not transitive as (1 2) R and (2 3) R but (1 3) R isin isin notinExample 3 Show that the relation R in the set Z of integers given byR = (a b) 2 divides a ndash b is an equivalence relationSolution R is reflexive as 2 divides (a ndash a) for all a Z isinFurther if (a b) R then 2 divides a isinndash b Therefore 2 divides b ndash a Hence (b a) R which shows that R is isinsymmetric Similarly if (a b) R and (b c) R isin isinthen a ndash b and b ndash c are divisible by 2 Now a ndash c = (a ndash b) + (b ndash c) is even (Why) So (a ndash c) is divisible by 2 This shows that R is transitive Thus R is an equivalence relation in ZExample 4 Let L be the set of all lines in a plane and R be the relation in L defined as R = (L1 L2) L1 is perpendicular to L2 Show that R is symmetric but neither reflexive nor transitiveSolution R is not reflexive as a line L1 can not be perpendicular to itself ie (L1 L1) R notinR is symmetric as (L1 L2) Risin

L1 is perpendicular to L2rArr L2 is perpendicular to L1rArr (L2 L1) RrArr isin

R is not transitive Indeed if L1 is perpendicular to L2 and L2 is perpendicular to L3 then L1 can never be perpendicular to L3 In fact L1 is parallel to L3 ie (L1 L2) R isin(L2 L3) R but (L1 L3) Risin notin

Chemistry Solid state Characteristics if Solids(i)The particles are locked in fixed positions they are unable to change their relative positions and this brings a definite shape and volume of a solid(ii)In a solid the constituent particles are held by strong forces of attractionThe forces of attraction may be bonding or non bonding(iii)The constituent particles in a solid pack together as closely as possibleoccupying most of the available space within the solidThus the empty space in a solid is very smallThis makes a solid highly rigid and nearly incompressibleThis also explains why a solid has high density and exhibits slow diffusionClassification of Solids

Q1)Define Crystalline solids AnsA Solid that has a definite geometrical shape and a sharp melting pointand whose constituent particles (atomsmolecules or ions) are arranged in a long range order of definite pattern extending throughout the solid is called a crystalline solidExNaClQ2)Define Amorphous solids AnsA solid that does not have a definite shape and a sharp melting pointand whose constituent particles (atomsmolecules or ions) are not arranged in a definite pattern is called an amorphoussolid

Crystalline solidsAmorphous solids

ExGlassRubberQ3)Classify Crystalline Solids Crystalline Solids

Physics Coloumbrsquos Law (Summary)

Before Going Into Coloumbrsquos Law We Will First Learn What is Charge Properties of Charge and Always remember that charge is quantized ie a body always have static charge of magnitude equal to some integral multiple of fundamental electronic charge e= 16 x 10- 19 C

Charge is the property of matter that causes it to produce and experience electrical and magnetic effects The study of the electrical charges at rest is called electrostatics When both electrical and magnetic effects are present the interaction between charges is referred to as electromagnetic

There exist two types of charges in nature positive and negative Like charges repel and unlike charges attract each other

The type of charge on an electron is negative The charge of a proton is the same as that of an electron but with a positive sign In an atom the number of electrons and the number of protons are equal The atom is therefore electrically neutral If one or more electrons are added to it it becomes negatively charged and is designated as negative ion However if one or more electrons are removed from an atom it becomes positively charged and is called a positive ion

The excess or deficiency of electrons in a body gives the concept of charge If there is an excess of electrons in a body it is negatively charged And if there is deficiency of electrons the body becomes positively charged Whenever addition or removal of electrons takes places the body acquires a charge

The SI Unit of charge is coulomb (C) In SI units the current is a fundamental quantity having a unit of ampere (A) The unit of charge is defined in terms of the unit of current Thus one coulomb is the charge transferred in one second across the section of a wire carrying a

Ionic SolidsMetallicSolids

Molecular Solids

current of one ampere

As q = It we have1 C = (1 A) (1 s)

The dimensions of charge are [A T]

Properties of Charge

(1) Quantization of Charge Electric charge can have only discrete values rather than any value That is charge is quantized The smallest discrete value of charge that can exist in nature is the charge on an electron given as

e = plusmn 16 x 10- 19 C

This is the charge attained by an electron and a protonA charge q must be an integral multiple of this basic unit That is

Q = plusmn ne where n = 1 2 hellip

Charge on a body can never be (frac12)e (23)e or 57e etcWhen we rub a glass rod with silk some electrons are transferred from the rod to the silk The rod becomes positively charged The silk becomes negatively charged The coulomb is a very large amount of charge A typical charge acquired by a rubbed body is 10 - 8 C

Biology Reproduction in organisms

Welcome to this new session 2020-21Today in this first chapter we mainly discuss about reproduction types needs and life span of some organismsWe also discuss about difference between sexual and asexual reproduction

Q1 What is reproductionReproduction is defined as a biological processin which an organism gives rise to young onessimilar to itselfQ2 What are the needs of reproductionbulli) Reproduction maintain life on earthii) It enables the continuity of the species generation after generationiii) It creates genetic variation among populationsQ3 Define Life span and write some orgnisms life spanbull Life span is the period from birth to

the natural death of an organism- OrganismsLife span1 Butterfly 1 - 2 weeks2 Fruit fly 30 days3Dog 10-13 years4 Rose5-7 years5 Tortoise100-150 years6 Banyan Tree -200 - 250 yearsQ4 Reproduction is of two types in case ofanimals but in case of plants vegetative propagation is also present

Asexual Reproduction Sexual Reproductioni) Always uniparentalii) Gametes are not involvediii) Only mitotic division involvediv) Somatic cells of parents are involvedv) Offsprings are genetically similar to the parents

i) Usually biparentalii) Gametes are involvediii) Meiosis occurs during gametogenesis Mitosis occurs after fertilisationiv) Germ cells of the parents are involvedv) offsprings are genetically different from the parents

COMMERCE BUSINESS ENVIRONMENT

Welcome to the new sessiontoday we are going to start the first chapter of Class XII The name of the chapter is Business Environment

Already many of you have got some idea about the word business environment form the first chapter of business studies in class XI

In todayrsquos world every business enterprise is a part of the society It exists and operates in association with various groups in society such as customers suppliers competitors banks and financial institutions government agencies trade unions media and so on All these groups influence the functioning of business in one way or the other They constitute the environment of businessConcept of Business Environment

The term lsquobusiness environmentrsquo refers to the sum total of all individuals institutions and other forces that lie outside a business enterprise but that may influence its functioning and performance

The main features of business environment Totality of External forces General and Specific forces Interrelatedness Complexity Dynamic Uncertainty

Prepare the following questions from todayrsquos assignment

2 What do you mean by business environment

The term lsquobusiness environmentrsquo means the aggregate of all forces factors and institutions which are external to and beyond the control of an individual business enterprise but they may influence its functioning and performance Business environment is the macro framework within which a business firm a micro unit operates It consists of several interrelated and interacting elements

2 Explain the main features of business environment in brief

Totality of External forces-Business environment is the sum total of all things external to a business environment

General and Specific forces-It

Relativity

The Interrelation between business and its environment

The business enterprise is an open system It continuously interacts with its environment It takes inputs (such as raw materials capital labour energy and so on) from its environment transforms them into goods and services and sends them back to the environment

Fig 1 Business Environment Relationship

includes both the forces general forces are the economic social political legal and technological conditions which indirectly influence all business enterprise Specific forces are the investors customers competitors and suppliers which influence individual enterprise directly

Interrelatedness-Different elements of environment are interrelated for an example growing awareness for health care has increased the demand for health foods

Complexity- Business environment id complex in nature as the elements keep on changing example economic technological and other forces changes in demand for a product and service

Dynamic-Business environment is not static it keeps on changing

Uncertainty- Itrsquos very difficult to predict future events such as technology and fashion which occur fast and frequently

Business Studies

Human Resources Management

Human resource of an organisation are the aggregate of knowledge skills attitudes of people working in it

The management system which deals with human resources is called human resource management

Features of HRMbullComprehensive functionbullPeople-oriented

Question1) What do you mean by human

resource management Answer) Human resource management may be defined as that field of Management which has to do with planning organising and controlling the functions of procuring developing maintaining and utilising the labour force

bullAction oriented bullPervasive function bullContinuous function

2) Explain the features of HRM in brief

Answer)bullHuman Resource Management is concerned with managing people at work bull Human Resource Management is concerned with employees which bring people and organisations together so that the goals of each are met bullHuman resource management considered every employees as an individual and also promote their satisfaction and growth bull Human resource management is inherent in all organisations and at all levelsbullManagement of human resources are ongoing on never ending process which requires a constant alertness and Awareness of human relations

3) ldquoHR function is said to be pervasiverdquowhy

Answer) Human resource management is required in all organisations whether it is private or government organisations armed forces sports organisations etc It permeatsall the functional areas like production marketing finance research etc This from this feature of human resource management it can be said that it is pervasive in nature

Economics Demand Q1DEFINITION OF DEMANDIn economics demand is the quantity of a good that consumers are willing and able to purchase at various prices during a given period of timeQ2DEMAND CURVEIn economics a demand curve is a graph depicting the relationship between the price of a certain commodity and the quantity of that commodity that is demanded at that pricQ3LAW OF DEMANDIn microeconomics the law of demand states that conditional on all else being equal as the price of a good increases quantity demanded decreases conversely as the price of a good decreases quantity demanded increasesQ4ASSUMPTION of LAW OF DEMAND(i)No change in price of related commodities(ii) No change in income of the consumer(iii) No change in taste and preferences customs habit and fashion of the consumer( No expectation regarding future change in priceQ5MARKET DEMAND SCHEDULEIn economics a market demand schedule is a tabulation of the quantity of a good that all consumers in a market will purchase at a

given price At any given price the corresponding value on the demand schedule is the sum of all consumersrsquo quantities demanded at that priceQ6INDIVIDUAL DEMAND SCHEDULEIndividual demand schedule refers to a tabular statement showing various quantities of a commodity that a consumer is willing to buy at various levels of price during a given period of timeQ7 FACTORS AFFECTING INDIVIDUAL DEMAND FOR A COMMODITY

The factors that influence a consumerrsquos decision to purchase a commodity are also known as determinants of demand The following factors affect the individual demand for a commodity1 price of the commodity2 price of related goods3 income of buyer of the commodity4 tastes and preferences of the buyer1 Price of the CommodityYou must have observed that when price of a commodity falls you tend to buy more of it and when its price rises you tend to buy less of it when all other factors remain constant (lsquoother things remaining the samersquo) In other words other things remaining the same there is an inverse relationship between the price of a commodity and its quantity demanded by its buyers This statement is in accordance with law of demand which you will study in the later part of this lesson Price of a commodity and its quantity demanded by its buyers are inversely related only when lsquoother things remain the samersquo So lsquoother things remaining the samersquo is an assumption when we study the effect of changes in the price of a commodity on its quantity demanded2 Price of Related goodsA consumer may demand a particular good But while buying that good heshe also asks the price of its related goods Related goods can be of two types-(i) Substitute goods(ii) Complementary goods While purchasing a good prices of its substitutes and complements do affect its quantity purchased(i) Price of Substitute Goods Substitute goods are those goods which can easily be used in place of one another for satisfaction of a particular want like tea and coffee An increase in price of substitute good leads to an increase in demand for the given commodity and a decrease in price of substitute good leads to a decrease in demand for the given commodity It means demand for a given commodity is directly affected by change in price of substitute goods For example if price of coffee increases the demand for tea will rise as tea will become relatively cheaper in comparison to coffee(ii) Price of Complementary goods Complementary goods are those goods which are used together to satisfy a particular want like car and petrol An increase in the price of complementary goods leads to a decrease in demand for the given commodity and a decrease in the price of complementary goods leads to an increase in demand for the given commodity For example if price of petrol falls then the demand for cars will increase as it will be relatively cheaper to use both the goods together So demand for a given commodity is inversely affected by change in price of complementary goods3 Income of the Buyer of CommodityDemand for a commodity is also affected by income of its buyer However the effect of change in income on demand depends on the nature of the commodity under consideration In case of some goods like full cream milk fine quality of rice (Basmati rice) etc demand for these commodities increases when income of the buyer increases and

demand for these commodities decreases when income of the buyer decreases Such goods whose demand increases with the increase in income of the buyer are called normal goods But there are some goods like coarse rice toned milk etc whose demand decreases when income of buyer increases and their demand increases when income of the buyer decreases Such goods whose demand decreases with the increase in income of the buyer are called inferior goods Suppose a consumer buys 10 Kgs of rice whose price is ` 25 per Kg He cannot afford to buy better quality of rice because the price of such rice is ` 50 per Kg The consumer is spending ` 250 per month on the purchase of rice Now if income of the consumer increases and he can afford ` 350 on purchase of 10 Kg of rice Now he can afford to buy some quantity of rice say 6 Kgs whose price is ` 25 per Kg and may buy 4 Kgs of rice whose price is ` 50 per Kg Thus he will buy 10 Kgs of rice by spending ` 350 per month Therefore we may conclude that demand for normal goods is directly related to the income of the buyer but demand for inferior goods is inversely related to the income of the buyer4 Tastes and Preferences of the BuyerThe demand for a commodity is also affected by the tastes and preferences of the buyers They include change in fashion customs habits etc Those commodities are preferred by the consumers which are in fashion So demand for those commodities rises which are in fashion On the other hand if a commodity goes out of the fashion its demand falls because no consumer will like to buy it(5) Number of Buyers in the Market(Population)Increase in population raises the market demand whereas decrease in population reduces the market demand for a commodity Not only the size of population but its composition like age (ratio of males females children and old people in population) also affects the demand for a commodity It is because of needs of children young old male and female population differs(6) Distribution of Income and WealthIf the distribution of income and wealth is more in favour of the rich demand for the commodities preferred by the rich such as comforts and luxuries is likely to be higher On the other hand if the distribution of income and wealth is more in favour of poor demand for commodities preferred by the poor such as necessities will be more(7) Season and Weather ConditionsThis is generally observed that the demand for woolens increases during winter whereas demand for ice creams and cold drinks increases during summer Similarly market demand for umbrellas rain coats increases during rainy seasonQ8 REASONS FOR OPERATION OF LAW OF DEMAND WHY DEMAND CURVE SLOPES DOWNWARDNow we will try to explain why does a consumer purchase more quantity of a commodity at a lower price and less of it at a higher price or why does the law of demand operate ie why does the demand curve slope downwards from left to right The main reasons for operation of law of demand are1 Law of Diminishing Marginal UtilityAs you have studied earlier law of diminishing marginal utility states that as we consume more and more units of a commodity the utility derived from each successive unit goes on decreasing The consumer will be ready to pay more for those units which provide him more utility and less for those which provide him less utility It implies that he will purchase more only when the price of the commodity falls2 Income Effect

When price of a commodity falls purchasing power or real income of the consumer increases which enables him to purchase more quantity of the commodity with the same money income Let us take an example Suppose you buy 4 ice creams when price of each ice cream is ` 25 If price of ice creams falls to ` 20 then with same money income you can buy 5 ice creams now3 Substitution EffectWhen price of a commodity falls it becomes comparatively cheaper as compared to its substitutes (although price of substitutes has not been changed) This will lead to rise in demand for the given commodity For example if coke and Pepsi both are sold at ` 10 each and price of coke falls Now coke has become relatively cheaper and will be substituted for Pepsi It will lead to rise in demand for coke4 Change in Number of BuyersWhen price of a commodity falls some old buyers may demand more of the commodity at the reduced price and some new buyers may also start buying this commodity who were not in a position to buy it earlier due to higher price This will lead to increase in number of buyers when price of the commodity falls As a result demand for the commodity rises when its price falls5 Diverse Uses of a CommoditySome commodities have diverse uses like milk It can be used for drinking for sweet preparation for ice cream preparation etc If price of milk rises its use may be restricted to important purpose only This will lead to reduction in demand for other less important uses When price of milk falls it can be put to other uses also leading to rise n demand for itQ9 EXCEPTIONS TO THE LAW OF DEMANDYou have studied in law of demand that a buyer is willing to buy more quantity of a commodity at a lower price and less of it at a higher price But in certain circumstances a rise in price may lead to rise in demand These circumstances are called Exceptions to the Law of Demand Some important exceptions are1 Giffen GoodsGiffen goods are special type of inferior goods in which negative income effect is stronger than negative substitution effect Giffen goods do not follow law of demand as their demand rises when their price rises Examples of Giffen goods are jowar and bajra etc2 Status Symbol GoodsSome goods are used by rich people as status symbols eg diamonds gold jewellary etc The higher the price the higher will be the demand for these goods When price of such goods falls these goods are no longer looked at as status symbol goods and tehrefore therir demand falls3 NecessitiesCommodities such as medicines salt wheat etc do not follow law of demandbecause we have to purchase them in minimum required quantity whatever their price may be4 Goods Expected to be ScarceWhen the buyers expect a scarcity of a particular good in near future they start buying more and more of that good even if their prices are rising For example during war famines etc people tend to buy more of some goods even at higher prices due to fear of their scarcity in near future

Political Science

Constitution of India-The

Preamble

The preamble-

Preamble-

The preamble is the most precious part of the constitution We the people of India having solemnly resolved to constitute India into a Sovereign Socialist Secular Democratic Republic and to secure to all its citizensA preamble is an introductory and expressionary statement in a document that explains the documents purpose and underlying philosophy When applied to the opening paragraphs of a statute it may recite historical facts pertinent to the subject of the statuteNature and purpose of the constitution-Purpose of the Constitution dictates permanent framework of the government to form a more perfect union to establish justice and ensure peace of thenationconstitution provide principles how the government can run itself following the rules and laws written in the constitution of each state keeps them balanced

Answer the following questions-

1 What is preambleA preamble is an introductory and expressionary statement in a document that explains the documents purpose and underlying philosophy2 What is the nature and

purpose of the constitutionConstitution dictatespermanent framework of the government to form a more perfect union to establish justice and ensure peace of the nation

Homework-Learn

Accounts Compatibilty mode

1MEANING OF PARTNERSHIPPartnership is a form of business organisation where two or more persons join hands to run a business They share the profits and losses according to the agreement amongst them According to the Indian Partnership Act 1932 ldquoPartnership is relation between persons who have agreed to share profits of a business carried on by all or any one of them acting for allrdquo For example one of your friends has passed class XII from National Institute of Open Schooling (NIOS) and wants to start a business Heshe approaches you to join in this venture Heshe wants you to contribute some money and participate in the business activities Both of you if join hands constitute a partnership2CHARACTERISTICS1048698 Agreement A partnership is formed by an agreement The agreement may be either oral or in writing It defines the relationship between the persons who agree to carry on business It may contain the terms of sharing profit and the capital to be invested by each partner etc The written agreement is known as partnership deed1048698 Number of persons There must be at least two persons to form a partnership

The maximum number of partners in a partnership firm can be 50 according toCompanies Act 20131048698 Business The Partnership is formed to carry on business with a purpose of earning profits The business should be lawful Thus if two or more persons agree to carry on unlawful activities it will not be termed as partnership1048698 Sharing Profits The partners agree to share profits in the agreed ratio In caseof loss all the partners have to bear it in the same agreed profit sharing ratio10486981048698Mutual Agency Every partner is an agent of the other partners Every partner can bind the firm and all other partners by hisher acts Each partner will be responsible and liable for the acts of all other partners10486981048698Unlimited liability The liability of each partner except that of a minor is unlimited Their liability extends to their personal assets also If the assets of the firm are insufficient to pay off its debts the partnersrsquo personal property can be used to satisfy the claim of the creditors of the partnership firm10486981048698Management All the partners have a right to mange the business However they may authorize one or more partners to manage the affairs of the business on their behalf10486981048698Transferability of Share No partner can transfer hisher share to any one including hisher family member without the consent of all other partners3PARTNERSHIP DEEDAgreement forms the basis of partnership The written form of the agreement is which a document of partnership is It contains terms and conditions regarding the conduct of the business It also explains relationship between the partners This document is called partnership deed Every firm can frame its own partnership deed in which the rights duties and liabilities of the partners are stated in detail It helps in settling the disputes arising among the partners during the general conduct of business 4CONTENTS OF PARTNERSHIP DEEDThe partnership deed generally contains the following (i) Name and address of the partnership firm(ii) Nature and objectives of the business(iii) Name and address of each partner(iv) Ratio in which profits is to be shared(v) Capital contribution by each partner(vi) Rate of Interest on capital if allowed(vii) Salary or any other remuneration to partners if allowed(viii) Rate of interest on loans and advances by a partner to the firm(ix) Drawings of partners and interest thereon if any(x) Method of valuation of goodwill and revaluation of assets and liabilities on the reconstitution of the partnership ie on the admission retirement or death of a partner(xi) Settlement of disputes by arbitration(xii) Settlement of accounts at the time of retirement or death of a partner5IN ABSENCE OF PARTNERSHIP DEEDThe partnership deed lays down the terms and conditions of partnership in regard to rights duties and obligations of the partners In the absence of partnership deed there may arise a controversy on certain issues like profit sharing ratio interest on

capital interest on drawings interest on loan and salary of the partners In such cases the provisions of the Indian Partnership Act becomes applicableSome of the Issues are(i) Distribution of Profit Partners are entitled to share profits equally(ii) Interest on Capital Interest on capital is not allowed(iii) Interest on Drawings No interest on drawing of the partners is to be charged(iv) Interest on Partnerrsquos Loan A Partner is allowed interest 6 per annum on the amount of loan given to the firm by himher(v) Salary and Commission to Partner A partner is not entitled to anysalary or commission or any other remuneration for managing the business

History TOPIC-TOWARDS INDEPENDENCE AND PARTITION THE LAST PHASE (1935-1947)

SUB TOPIC-IMPORTANT POLITICAL DEVELOPMENTS ndash GROWTH OF SOCIAL IDEAS

Socialism is a political social and economic philosophyLike in other parts of the world the Russian revolution of 1917 served as a great inspiration for revolutionaries in India who at that time were engaged in the struggle for liberation from British ruleSocialist ideas led to the formation of communist party of IndiaJAWAHARLAL NEHRU Among the early Congress leaders Jawaharlal Nehru was very much impressed and influenced by the Socialist ideas He also learnt about the Economic activities of the Soviet Union after the Bolshevic Revolution 1917 He made full use of them in IndiaThe election of Jawaharlal Nehru and Subhas Chandra Bose showed the Left wing tendency within CongressJawaharlal Nehru demanded economic freedom along with political freedom of the people in order to end the exploitation of masses

Nehrus working committee included three socialists leaders The Lucknow session was a landmark in the evolution of socialist ideas of the congressSUBHAS CHANDRA BOSE ndash Subhas Chandra Bose had socialist leaning Both Jawaharlal Nehru and Subhas Chandra Bose were known as leftist Congress men Later on National Congress divided into Leftist and rightist campCONGRESS SOCIALIST Within the Congress some leaders formed the Congress Socialist partyPattavi Sitaramyya Sardar Patel Rajendra Prasad had hostile attitude towards the Congress Socialist partyJawaharlals attitude was hesitant

1 QUESTION ndash Mention name of two Congress leaders who had socialist leaning

1ANSWER ndash Subhas Chandra Bose and Jawaharlal Nehru2QUESTION- In which session of the congress Jawaharlal elaborated his Socialist ideas2 ANSWER ndash Lucknow and Faizpur Session in December 1935 and 19363QUESTION ndash Why Congress was sharply divided into leftist and rightist camp 3ANSWER ndash Subhas Chandra Bosersquos attempt to seek re election for congress presidentship in 1939sharply divided the National Congress into Leftist and Rightist camp4 QUESTION ndash Who was MN Roy 4 ANSWER ndash Manabendra Roy first formed the Communist Party of India outside the country at Tashkent in 19205QUESTION ndash Who formed the Congress Socialist Party within the Congress5 ANSWER ndash Jaya Prakash Narayan Achyut Patwardhan Acharya Narendra Dev Ram Mohan Lohia Aruna Asaf Ali6QUESTION ndash When was the Congress Socialist Party formed What was its object6 ANSWER ndash 1934The Congress Socialist Party sought to work out socialist programme through the Congress They joined hands with the Congress and wanted to carry

Subhas Chandra Bose being expelled from the congress after the Tripuri rift he formed Forward BlockThere were basic differences between the Congress Socialists and the communistsTRADE UNION ACTIVITIES Maximum working class people lived in Bombay and Calcutta The working and living conditions of those workers were very miserable In this situation Shasipada Banerjee NM Lokhande protested against the oppression of the working class peopleThe first Trade Union Madras Labour Union was formed in 1918 by BP WadiaIndustrial strikes took place in Kanpur Calcutta Madras Jamshedpur and Ahmedabad AITUC was formed in Bombay in 1927 The growth of Trade union among the workers was slow because of the fear of the dismissal of the jobIn the mean time the Moderates as well as Communists left AITUC and formed separate organization

on National struggle with the help of workers and peasant class of the society7 QUESTION ndash What was the name of the party founded by Subhas Chandra Bose7 ANSWER- Forward Block8QUESTION ndash Who was Shasipada Banerjee8 ANSWER ndash Shasipada Banerjee was a radical Brahmo He founded a working menrsquos club to protest against exploitation of the British rulers towards the working class of India9 QUESTION ndash What was the weekly published by NM Lokhande9ANSWER- Dinabandhu10 QUESTION ndash Who founded Bombay Mill-Hands Association and in which year10 ANSWER- NM Lokhande in189011 QUESTION- Who was BP WadiaANSWER- BPWadia was the founder of Madras Labour Union in191812 QUESTION- What was the name of the first labour union of India12 ANSWER- Madras Labour Union13 QUESTION Who founded the Majur Mahajan 13 ANSWER GANDHIJI14 QUESTION What was the full form of AITUC When it was formed14 ANSWER All India Trade Union Congressin 192715QUESTION Who formed the Red Trade Union Congress and in which year15ANSWER The Communists formed the Red Trade Union Congress16 QUESTION What do you mean by Socialism16 ANSWER Socialism describes any political and economic theory that says the community rather than individuals should own and manage property and natural resources

Subject Eng Literature (The Tempest ndash William Shakespeare) Topic Act III Scene 3 Lines 1 to 52 (Line 52 ndash Brother my lord the Duke Stand to and do as we) Date 13th April 2020 (4th Period)

[Students should read the original play and also the paraphrase given in the school prescribed textbook]Summary Questions amp Answers

o Alonso Sebastian Antonio Gonzalo Adrian Francisco and others wandered about the island in search of Ferdinand and gets tired and hungry of the toil and at the same time gives up all hope of finding him

o Antonio and Sebastian are happy that Alonso is out of hope and decide to make another attempt on his life that night when being so tired they will be sleeping soundly

o Suddenly a solemn and strange music is heard in the air and several strange shapes enter bringing in a banquet These strange shapes then dance round it with gestures of salutation and then inviting the King to eat they depart

o Seeing this strange scene all are inclined to believe the tales told by travelers that there truly are ldquounicornsrdquo and ldquothe phoenixrsquo thronerdquo

1 ALONSO What harmony is this My good friends hark (L18-27)

GONZALO Marvellous sweet music

[Enter several strange shapes bringing in a banquet

they dance about it with gentle actions of salutation

and inviting the King and his companions to eat they depart]ALONSO Give us kind keepers heavens What were theseSEBASTIAN A living drollery Now I will believe

That there are unicorns that in Arabia

There is one tree the phoenixrsquo throne one phoenix

At this hour reigning thereANTONIO Ill believe both

And what does else want credit come to me

And Ill be sworn rsquotis true Travellers neer did lie

Though fools at home condemn rsquoem

(a) How did Prospero present an amazing spectacle before Alonso and his companions

Using his magic powers Prospero ordered strange shapes to lay a banquet before Alonso and his companions The shapes brought several dishes with tasty eatables in them They placed the dishes on a table before Alonso and his companions Then the strange shapes began to dance gracefully around the banquet While dancing they made gestures inviting them to eat the food Then suddenly the shapes disappeared(b) Who were the guests at the strange banquet Describe the lsquoliving drolleryrsquo

Alonso Sebastian Antonio Gonzalo Adrian and Francisco were the guests at the strange banquet

The term ldquoliving drolleryrdquo refers to live entertainment show In this context when Alonso the King of Naples Sebastian his brother Antonio the treacherous brother of Prospero Gonzalo the kind and loyal councillor to the King Adrian and Francisco came to the island they were hungry and weary in their spirits They heard a solemn and strange music They were shocked to see several strange shapes bringing in a banquet and these shapes danced about it with gentle action of salutation inviting the King and his companions to eat After this Sebastian described this show as lsquoliving drolleryrsquo(c) What is lsquophoenixrsquo What are lsquoUnicornsrdquo

The term lsquophoenixrsquo refers to a mythical Arabian bird which lived alone and perched on a solitary tree After one hundred years it expired in flames and rose again from its own ashes

lsquoUnicornsrsquo refers to the mythological four-footed beasts having horns in the centre of their foreheads When the horns are ground into powder the powder was believed to be

an aphrodisiac(d) How does Sebastian explain the puppet show OR Why does the speaker now believe in unicorns and phoenix

Sebastian finds several strange shapes bringing in the banquet They invite the king and his party for dinner and soon depart He tells that if such a strange sight can be a reality there is nothing incredible in the world and from the present moment he will believe anything He says that it is a strange dumb show enacted not by puppets but by living beings It is stranger than a travellerrsquos tale Seeing such a thing

before his own eyes he will no longer disbelieve the story about unicorns and phoenix(e) How do the other characters present respond to this living drollery

At the sight of the lsquoliving drolleryrsquo like Sebastian Gonzalo and Antonio too acted strangely Antonio told that he too now believes in unicorns and phoenix and anything else that seems to be incredible He too now believes in travellersrsquo tales Gonzalo told that if he would report those happenings in Naples nobody will believe him He considers that those gentle shapes were gentler in manner in comparison to the living beings Alonso was at first sight suspicious and told them that those strange shapes conveyed their meaning in expressive gestures when they seemed to lack speech by their movements and sounds Francisco was amazed at their mysterious disappearance

2 ALONSO Not I

(Line 43-52)GONZALO Faith sir you need not fear When we

were boysWho would believe that there were mountaineers

Dewlapped like bulls whose throats had hanging at rsquoem

Wallets of flesh Or that there were such men

Whose heads stood in their breasts Which now we find

Each putter-out of five for one will bring us

Good warrant ofALONSO I will stand to and feed

Although my lastmdashno matter since I feel

The best is past Brother my lord the Duke

Stand to and do as we

(a) How does Alonso respond at the spectacle of the shapes which were sent to them at the instruction of Prospero

After seeing the strange sight of appearing and disappearing of the shapes sent by Prospero to arrange a banquet for them Alonso says that his surprise at having seen those creatures is infinite and he is fully justified in feeling so much surprise He thinks that their shapes their gestures and the sounds they made were indeed amazing Although they do not possess the gift of speech yet they were able to convey their

thoughts by means of their gestures only

(b) What does Prospero say about the views expressed by Alonso regarding the shapes What does Francisco think about the shapesAfter hearing Alonsorsquos views about the shapes Prospero says that this manrsquos praise of the spirits is rather hasty He means to say that Alonso has shown great haste in reaching the conclusion about the shapes Francisco is amazed to see that those shapes disappeared in a mysterious way(c) What does Sebastian ask Alonso to doSebastian tells Alonso that the shapes having disappeared should not matter to them because they have left the eatables behind He asks Alonso to enjoy eating as they are extremely hungry but the king does not accept his offer of enjoying the dishes(d) How does Gonzalo try to dispel Alonsorsquos fear of those strange shapes What kind of references does he give to AlonsoGonzalo says that those who have travelled abroad have reported seeing even stranger sights than these shapes that Alonso and his companions have beheld Hence there is no reason to feel afraid of these shapes Gonzalo further adds that in his younger days he had heard strange stories from travelers and Alonso might have heard similar stories For instance it was said that there existed a certain race of

human beings who had huge lumps of flesh hanging at their throats and who therefore resembled bulls Then Gonzalo tells about a race of human beings whose heads were located at their breasts Gonzalo says that such stories were not believed by most people in those days but now-a-days these stories have become common(e) Explain the following lsquoEach putter-out of five for onersquoEnglish travellers often insured their trips with London brokers Those that went on foreign travels those days used to deposit a certain amount with some firm or company in London before their departure If the travelers failed to return the money was forfeited by the company with which it had been deposited But this money was repaid five-fold if the travelers returned safe and sound In this way a traveler stood a great chance of recovering the entire cost of his

travels(f) Give the explanatory meanings of the following expressions in the context of the above extract (i) Dewlapped (ii) Wallets of flesh

(iii) Putter-out(i) Dewlapped having big lumps of flesh at the necks(ii) Wallets of flesh large masses of flesh looking like bags(iii) Putter-out to invest money before commencing the travel

  • General methods of preparation of hydrogen
  • Chapter Dimensional Analysis (Summary)
    • Properties of Charge
Page 9:   · Web viewSubject. Topic. Summary. Execution. Hindi. व्याकरण. शरीरके अंगो के नाम लिखिए. 1) आँख 2) नाक 3

DisadvantageIt is the most expensive of all other means of transport

3 What is the disadvantage of airwaysAns The only disadvantage of airways is that it is the most expensive of all other means of transport

MATHEMATICS

Ch 3Addition and Subtraction

Properties of Addition1 The sum of two numbers does not change when we change their order This property known as Commutative Property of addition

2 The sum of three numbers does not change when we change their grouping This property is called Associative Property of addition

3 The sum of the numbers and zero is the number itself This property is called Identity Property of addition and the integer 0 is called identity

Exercise 11Fill in the blanks1 2730815 + 8319293 = ____ + 27308152 18219 + 1850308 = 1850308 + ____3 (27815 + 85919) + 95985 = (85919 + ____) + 278154 13227 + (25983 + 73607) = (____ + 25983) + 736075 91389 + 0 = ____ + 91389

Solution 1 83192932 182193 959854 132275 0

6 Which of the following are true statements(a) Any number added to zero is zero

(b) The sum of two numbers does not change when we change their order

(c) 1 is the identity element of addition

(d) Given any three numbers their sum does not change when we change their grouping

Solution (a) False(b) True(c) False(d) True

Class VISubject Topic Summary Execution

HISTORY AND CIVICS

CHAPTER 3

MAHAVIRA AND BUDDHA ndash GREAT PREACHERS BUDDHA

Decline of Buddhism1 Revival of the Brahmanical Hinduism ndash Brahmin Scholars like Shankaracharya and Kumarila Bhatta led the revival of Hinduism and established the supremacy of Vedic religion2 Loss of Royal Patronage ndash Gupta period marked the decline of Buddhism as Gupta rulers were followers of Hinduism3 Split in Buddhism ndash division into Hinayana and Mahayana sects and rise of Mahayana sects blurred the line between Hinduism and Buddhism4 Corruption in Buddhist Sangha ndash due to generation of large revenue from large estates Buddhist monks and nuns started living luxurious

Answer the Following 1 During which dynasty Buddhism was split During the reign of Kanishka

2 Name the two sects of Buddhism Mahayana and Hinayana

3 Name two Vedic scholars who led the revival of Brahmanical Hinduism Shankaracharya and Kumarila Bhatta

life in rich monasteries Hence corruption crept in5 Adoption of Sanskrit ndash when Buddhist scriptures began to be written in Sanskrit in place of peoples language like Pali or Prakrit people started drifting away from Buddhism6 The Turkish Invasion ndash As Muslim conquerors invaded India immensely wealthy Buddhist Monasteries and temples were looted and destroyed and Buddhists were persecuted and killed

4 During which period decline of Buddhism began The Gupta Period

ENGLISH 2 The great train journey- Ruskin Bond

The great journey by Ruskin Bond is a story about Suraj who loved trains and wanted to go to places One day while wandering along the railway tracks he enters into a carriage compartment The train suddenly starts moving with him in the compartment and after a journey returns back to the same place from where it had begun The story is about his experience during that journey

4 Answer the following questionsf Who else is in the carriageA ragged hippy with a dirty beard face was in the carriageg Where does Suraj say that he would like to go toSuraj said that he would like to go to England and China and Africa and Greenland He wanted to go all over the worldh What warning does the man give to SurajThe man said Suraj to keep out of sight so that he doesnrsquot get caught by the ticket collectorsiWhen Suraj thinks about his parents for the first time what does he imagines that they will thinkSuraj thought that if he failed to come home that night his parents would think that he had run away or been kidnapped or been involved in an accidentJ What presents does Suraj imagine that he will bring back for his friendSuraj imagines that he would bring an African lion or a transistor- radio for his friend

CHEMISTRY

Chapter 2 ndashElement and Compound

ATOMAn atom can be defined as the smallest constituent particle of an element which showcases independent existence Example Ne OMOLECULEA molecule can be defined as the combinations of two or more atoms which are held together by chemical bonds A molecule is the smallest portion of a substance which showcases all the properties of the substance On breaking down a molecule further we see properties of the constituent elements Example HCl NaCl O2

Answer the following Q3) What is a moleculeAns - A molecule can be defined as the combinations of two or more atoms which are held together by chemical bonds A molecule is the smallest portion of a substance which showcases all the properties of the substance On breaking down a molecule further we see properties of the constituent elements Example HCl NaCl O2

Q4) Which can exist independently ndash atom or moleculesAns ndash Molecules can exist independently

PHYSICS Physical quantities

Guidelines for writing SI units correctly1 The units named after scientists are not written with a capital initial letter For example newton henry watt2 The symbols of the units named after scientist should be written by a capital letter For example N for newton H for henry W for watt3 Small letters are used as symbols for units not derived from a proper name For example m for metre kg for kilogram4 No full stop or other punctuation marks should be used within or at the end of symbols For example 50 m and not as 50 m5 The symbols of the units do not take plural form For example 10 kg not as 10 kgs6 When temperature is expressed in kelvin the degree sign is omitted For example 273 K not as 273o K (If expressed in Celsius scale degree sign is to be included For example 100o C and not 100 C)7 Use of solidus is recommended only for indicating a division of one letter unit symbol by another unit

Fill in the blanks

1) Length and mass are examples of fundamental physical quantities

2) The measurement of a physical quantity consists of two part magnitude and unit

3) A foot consist of 32 inches 4) The unit of temperature in the SI system is

Kelvin

Write true or false Correct the false statements

1) In ancient times cubit was used to measure the mass of an object FalseCorrect statement ndash In ancient times cubit was used to measure the length of an object

2) There are 7 fundamental physical quantities True

symbol Not more than one solidus is used For example m s-1 or m s J K mol or J K-1 mol-1 but not J K mol8 Some space is always to be left between the number and the symbol of the unit and also between the symbols for compound units such as force momentum etc For example it is not correct to write 23m The correct representation is 23 m kg m s-2 and not as kgms-29 Only accepted symbols should be used For example ampere is represented as A and not as amp or am second is represented as s and not as sec10 Numerical value of any physical quantity should be expressed in scientific notationFor an example density of mercury is 136 x 104 kg m-3 and not as 13600 kg m-3

3) Second is the unit of time in both the CGS and MKS systems True

4) The symbol used for a unit is always written in capital letters False Correct statement -The symbol used for a unit is normally written in small letters

Hindi 2nd language

वाकय निवचार भागवत निवचारो को परकट करन वा साथक एव वयवसथिtत शबद समह को वाकय कहत ह वाकय दो परकार क होत ह ndash

1 उददशय- वाकय म जिजसक बार म कछ बताया जाता ह उस उददशय कहत ह जस राधा एक नतकी ह2 निवधय- वाकय म उददशय क बार म बताया जाता ह उस निवधयक कहत ह जस- राधा एक नतकी ह रचना क आधार पर वाकय क तीन भद होत ह ndash१सर वाकय- राम बाजार गया २ सयकत वाकय- राम बाजार गया और वहा जाकर दोसत स मिमा३ मिमशर वाकय- यह वही tान ह जहा उनका बचपन बीता

helliphellipContinue to nextBengali 2nd language

লিZ সবরপ ও সবরলিZ

সবরলিZর লিয়ম - ১ অ-কোর লিকংবো আ-কোসরর পসর অ-কোর লিকংবো আ - কোর োকস উভয় লিমস আ ndashকোর য় এবং ওই আ ndash কোর পব13বস13 Pকত য়

২ ই - কোর লিকংবো ঈ - কোসরর পসর ই - কোর লিকংবো ঈ - কোর োকস উভয় লিমস ঈ - কোর য় এবং ওই ঈ - কোর পব13বস13 Pকত য়

৩ উ - কোর লিকংবো ঊ - কোসরর পসর উ - কোর লিকংবো ঊ - কোর োকস উভয় লিমস ঊ - কোর য় এবং ওই ঊ - কোর পব13বস13 Pকত য়

৪ অ - কোর লিকংবো আ ndash কোসরর পসর ই - কোর লিকংবো ঈ - কোর োকস উভয় লিমস এ - কোর য় এবং ওই এ - কোর পব13বস13 Pকত য়

১ অ + অ = আ ( gtো ) লিম + অচ = লিমোচ সব + অ3ী = সবো3ী অ + আ = আ ( gtো )পদম + আ = পদমো শভ + আলি = শভোলি আ + আ = আ ( gtো )4োয়ো + আবত = 4োয়োবত মো + আতমো = মোতমো আ + অ = আ ( gtো )লিবদযো + অংকোর = লিবদযোংকোর Pো + অ13 = Pো13 ২ ই + ই = ঈ ( gtী )অলিত + ইব = অতীব লিগলির + ইনদর = লিগরীনদর ই + ঈ = ঈ ( gtী )পলির + ইকষো = পরীকষো অলি3 + ঈশবর = অ3ীশবর ঈ + ঈ = ঈ ( gtী )মী + ঈশবর = মীশবর 3ী + ঈ = 3ী ঈ + ই = ঈ ( gtী )রী + ইনদর = রীনদর মী + ইনদর = মীনদর ৩ উ + উ = ঊ ( gt )মর + উদযো = মরদযো কট + উলিকত = কটলিকত উ + ঊ = ঊ ( gt )ঘ + ঊলিম13 = ঘলিম13 লিZ + ঊলিম13 = লিZলিম13 ঊ + ঊ = ঊ ( gt )রP + ঊলিম13 = রPলিম13

৪ অ + ই = এ ( লেgt )র + ইনদর = সরনদর লেPোগ + ইনদর = লেPোসগনদর অ + ঈ = এ ( লেgt )গ + ঈ = গস

র + ঈ = সর আ + ই = এ ( লেgt )Pো + ইষট = Pসষট 3ো + ইনদ = স3নদ আ + ঈ = এ ( লেgt )রমো + ঈ = রসম দবোরকো + ঈশবর = দবোরসকশবর

COMPUTER THE WORLD OF WINDOWS 10

DONE IN THE PREVIOUS CLASS PAGE NO-83A TICK THE CORRECT OPTION BACKGROUND DISPLAY AREA RESTORE THREE

MATHEMATICS Topic ndash NumbersChapter ndash Natural numbers and whole numbers

Study item Properties of whole numbers for subtraction1) Closure property When we do subtraction of two whole numbers we can not get a whole number in all time Example 8 ndash 3 = 5 a whole number 0 ndash 6 = -6 is not a whole numberTherefore the subtraction of two whole numbers is not satisfying closure property2) Commutative property If x and y are two whole numbers then x ndash y is not equal to y ndash xExample If x=16 and y = 7 then x ndash y = 16 ndash 7 = 9Again y ndash x = 7 ndash 16 = - 9 Therefore x ndash y not equal to y ndash x Therefore the subtraction of two whole numbers is not satisfy commutative3) Associative property If x y and z are three whole numbersThen x ndash ( y ndash z ) not equal to ( x ndash y ) ndash z Example If x = 20 y = 10 and z = 6Therefore x ndash (y ndash z ) = 20 ndash(10 ndash 6 ) = 20 ndash 4 = 16(X ndash y ) ndash z = (20 ndash 10) ndash 4 = 10 -4 =6Therefore x ndash(y ndash z) not equal to ( x ndash y) ndash zTherefore subtraction of whole numbers is not satisfying associativity4) Distributive property If x y and z are three whole numbersThen x (y ndash z ) = xy ndash xzAnd (y ndash z)x = yx ndash zxExample If x = 10 y = 6 and z = 4x(y ndash z ) = 10(6 ndash 4 ) = 10times6 ndash 10times4 = 60 ndash 40 = 20( 6 ndash 4 )times 10 = 6times10 ndash 4times10 = 60 ndash 40 = 20Therefore the subtraction of whole numbers is satisfying distributive property5) Existence of identity For any whole number x X ndash 0 = x but 0 ndash x = - x not equal to xThus for subtraction no identity number existsException 0 ndash 0 = 0 so 0 is its own identity for subtraction

Class VIISubject Topic Summary Execution

Hindi 2ndlang वचन जो सजञा शबद निकसी वसत या पराणी क एक या अनक होन का बोध कराया उनह वचन कहत ह जस डका- डकयह दो परकार की होती ह-

क) एकवचन-शबद क जिजस रप स उसक एक होन का बोध हो उस एक वचन कहत ह जस निकताब गमा आदिद

) बहवचन-शबद क जिजस रप स उसक आन ोन का पता च उस बहवचन कहत ह जस डक निकताब निततलियाआदिद

निनमनलिखित शबदो को एकवचन स बहवचन म बदोम- हमजानित- जानितयानारी- नारिरयामिमतर ndashमिमतरोपसतक -पसतकसडक-सडकबोत-बोतनाहर-नहररपए-रपया

Bengali বইndashবোংো োলিতয পলিরচয়

পোঠndash১৪ গলপ - অপর কলপো পর

লেক - লিবভলিতভষ বসনদযোপো3যোয়লেকndash রবীনদর পরবতf বোংো কোোলিতয 3োরোর উসgসPোগয োম পরকলিতসপরমী লিবভলিতভষ বসনদযোপো3যোয় তোর লেীসত লেPম বোসর বোসর লিফসর এসস4 গরোম বোংোর পরকলিতর কো লেতমলি এসস4 গরোমী মো লিচতরগলপndash অপর কলপো গলপোংটি লিবভলিতভষ বসনদযোপো3যোসয়র লিবযোত উপযো পসর পাোচোী লেসক গীত অপ অ13োৎ পসর পাোচোী তো অপর কলপোর লেকনদরীয় চলিরতর এই অংস আমরো পোই বোক অপসক বোক অপ কলপো লিবোী লে দসরর অ গো4 লেদস মোসয়র মস লেোো রপকোর রোসয পোলি2 লেদয় দপরসবো মোসয়র মসর কসর কোীদোী মোভোরত এর করসকষতর Pসjর ব13ো শস তোর মোবীর কস13র পরলিত ব2 মমতো য় আবোরপালিসত বলি13ত Pসjর অমোপত অং লে লিসই মোপত কসর বোলি2র লিপ4স বাো বোগোস লিকংবো উঠোসর লিশমসর কলপো লিবো এোস পরকো লেপসয়স4

১ অপর কলপো গসলপর লেক লেক তোর মপসক13 লেসো২ অপর কলপো গলপটি লেকোো লেসক গীত গলপটির ম ভোব লেসো

GEOGRAPHY CHAPTER 7EUROPE

CHAPTER COMPLETE EXERCISEFill in the blanks1 Europe is a continent that comprises the western part of Eurasia2 Eurasia and Africa are connected into one large land mass known as Afroeurasia3 The Strait of Gibraltar separates Europe and Africa4 Europe is surrounded by the Arctic Ocean to the north5 The British Isles includes the island countries of Great Britain and Ireland

Name the following 1 Connects Africa to Eurasia - Isthmus of Suez2 Largest country in the world in terms of area ndash Russia3 A term used collectively for the five countries in northern Europe ndash Nordic Countries4 The capital of Montenegro - Podgorica5 the largest fjord in Norway ndash Sognefjord

Match the following Column 1 Column 2a Albania iii Tiranab Belgium i Brusselsc Denmark v Copenhagend Finland ii Helsinkie Hungary iv Budapest

CHEMISTRY Chapter 2 ndashElement and Compound

Atom - An atom is the basic unit of an element or the smallest particle of an element non capable of independent existence Atom is built up of three sub atomic particles electron proton and neutron

Nucleus-It is the centre of an atom In the centre of the atom contains proton (positively charged particles ) and neutrons ( particles carrying no charge )

Orbits- It surround the nucleus in which revolve electrons (negatively charged particles)

Answer the following

1) What are MetalloidsAns - Certain elements using properties of both metal and non-metals are called metalloids Example Silicon arsenic and antimony

2) What are Noble gasesAns - Certain elements are present in the air and are chemically inert or unreactive Such elements are called rare gases or noble gases Example helium neon argon and Krypton

English 2 Sentences based on meanings

Kinds of sentences

Assertive or declarative to convey information or simply make a statement

Interrogative to ask different types of questions

Imperative to command or instruct someone or make a request

Exclamatory to express strong feelings and emotions

Exercise B1 Stop it ( Exclamatory)2 May you always be happy

together ( Exclamatory)3 He does not like sports

( Assertive)4 Please pass me the salt

( Imperative)5 How dare she talk to me like

that ( Exclamatory)6 May success bless your effort

( Exclamatory)7 Canrsquot you wait for sometime

(Interrogative)8 Did anybody tell you about it

( Interrogative)9 I saw her waiting for the bus

( Assertive)10 Could you please take a

message for me ( Interrogative)

Homework Ex ABiology Chp -2

Classification of Plants

Today we discuss about usefulness of bacteria We also discuss what the harmful effects of bacteria are

89 How bacteria are useful for usbull Bacteria is helpful in many ways forhuman being i) Production of medicine - antibiotics vaccine etcii) Formation of curd by lactobacillusiii)Nitrogen fixation in Leguminousplant by Rhizobiumiv) Increase soil fertility by absorbingatmospheric nitrogen and convert it into nitrates and nitritesv) Cleaning the environment by converting the complex substances into simple substancesvi) Tanning of leathervii) Retting of Fibersviii) Formation of compost by acting onanimal dung and agricultual cases1x) Biogas production by decomposingplant and animal wastex)Help In Nutrition by producing vitamiacutemBand kx1) Some bacteria are used to give specialflavour to tea coffee and coccaQ10- Name some diseases and there causativebacteriabull Diseasescausative bacteria1 CholeraVibrio cholerae2 Tuberculosis - Mycobacterium tuberculosis3 Diptheria -Corynebacteriumdiphtheriae4 Pneumonia - Streptococcus pneumoniae

Math Number system

Chapter Fraction

Study item Using lsquoofrsquoThe word lsquoofrsquo between any two fractions is to be used as multiplicationExample 57 of 56 = 57 times 56 = 5times8 = 40Study item Using BODMASThe word lsquoBODMASrsquo is the abbreviation formed by taking the initial letters of six operations(i)Bracket (ii) of (iii) Division (iv) Multiplication (v) Addition (vi) SubtractionAccording to BODMAS rule First of all the terms inside Bracket must be simplified then lsquoofrsquo lsquoDivisionrsquo lsquoMultiplicationrsquo lsquoAdditionrsquo lsquosubtractionrsquo

Study item Removal of Brackets

There are four Brackets of algebra in Mathematics In a complex expression four types of brackets are used Order of removing the brackets is first ----- then ( ) then finally [ ]

Class VIIISubject Topic Summary Execution

Chemistry Hydrogen General methods of preparation of hydrogen

By the action of dilute acids on metals

Calcium Reacts readily to form chloride salt and hydrogen

Ca + 2HCl rarr CaCl₂ + H₂uarr

Magnesium

Aluminium

Zinc

React readily to form salt and hydrogen

Mg + 2HCl rarr MgCl₂ + H₂uarr2Al + 6HCl rarr 2AlCl₃ + 3H₂uarrZn + 2HCl rarr ZnCl₂ + H₂uarr

Question 4 ) Give reasons for the following

(a) Hydrogen be used as a fuel

Solution

Hydrogen is used as a fuel because it has a high heat of combustion Some significant fuels are coal gas water gas and liquid hydrogen

(b) Though hydrogen is lighter than air it cannot be collected by downward displacement of air

Solution

Hydrogen is lighter than air so it is possible to collect the gas by downward displacement of air But it is not safe to do so since a mixture of hydrogen and air can lead to an explosion

(c) A pop sound produced when hydrogen is burnt

Solution

Impure hydrogen gas burns in air with a pop sound This is because of the presence of impurities in it

(d) Helium replaced hydrogen in weather observation balloons

Solution

It forms a mixture with air that can explode when there is a small leakage of hydrogen in a balloon So helium has replaced hydrogen

(e) Nitric acid not used for the preparation of hydrogen gas

Solution

(e) By the action of nitric acid on metals hydrogen cannot be produced because it also releases nitrous oxide and nitric oxide and oxides the hydrogen to form water

Biology Chp-2 Reproduction in plants

Today we discuss different methods of artificial propagation like cutting-rose sugercane Layering ndashguava lemon china rose etc Grafting- mango apple etcMicropropagation ndashorchid asparagus etcWe also discuss about advantages and disadvantages of vegetative propagation

Q7 Define the following terms i) Explant In tissue culture techniquea tiny piece of bud shoot or any other partof plant from where new tissue develop ii) Callus The cells of the tissue divide andgrow into a mass of undifferentiated cells from explant iii) Plantlet After few days callus differentiate into a small plant with roots and shootQ8 what are the advantages and limitations of tissue culture or micropropagation

Advantages i ) It produacuteces superior quality plantsii)It can be applied to interspecifie hybridsiii) It is useful to grow seedless plants bull Limitations i) It cannot be used for all plantsii)It is not easy to handleQ9 Write advantages of vegetative propagationi) It is a quick and easy method ofproducing new plantsii) This method need less time to matureiii) The new plants are exact copies of the parentiv) it is extremly useful for growing seedlessplants like banana grapes etc Q10 Write some disadvantages of vegetativepropagationi) Dišeases present in the parent plant gettransferred to all in new plantsii) Overcrowding of new plants causes competition for sunlight water and nutrients which affects growth of plantsplant

Physics Chapter 2 Physical Quatites and Measurements

Here We Will Do Some QuestionsRelated To Chapter 2

Select the correct alternative A block of wood of density 08gcm-3 has a volume of 60cm3 The mass of the block is

1 608 g

2 75 g

3 48 g

4 0013 g

Solution 348 g

The density of aluminium is 27g and that of brass The correct statement is

1 Equal masses of aluminium and brass have equal volumes

2 The mass of a certain volume of brass is more than the mass of an equal volume of aluminium

3 The volume of a certain mass of brass is more than the volume of an equal mass of aluminium

4 Equal volumes of aluminium and brass have equal masses

Solution 2 The mass of a certain volume of brass is more than the mass of an equal volume of aluminium

MATHEMATICS Ch 6Sets

Exercise 6(C)1 Find all the subset of each the following sets(i) A = 57 (iii) C = x xisin W x le 2(iv) p p is a letter in the word lsquopoorrsquo

Solution (i) All the subsets of A are ϕ 5 7 57

(iii) All the subsets of C are ϕ 0 1 2 01 02 12 012

(iv) All the subsets are ϕ p o r po or por

4 Given the universal set = -7-3-105689 find (i) A = x xlt2 (ii) B = x -4ltxlt6 Solution

(i) A = -7-3-10(ii) B = -3-105

5 Given the universal set = x xisin N and xlt20 find

(i) A = x x = 3p pisin N (iii) C = x x is divisible by 4 Solution

(i) 369121518 (iii) 481216

6 Find the proper subset of x x2-9x-10 = 0 Solution

ϕ 10 -1

Working x2-9x-10 = 0 rArr x2-(10-1)x-10 = 0

rArr x2-10x+x-10 = 0 rArrx(x-10)+1(x-10) = 0

rArr (x+1) (x-10) = 0

11 Let M = letters of the word REAL and N = letters of the word LARE Write sets M and N in roster form and then state whether (i) M sube N is true (ii) N sube M is true (iii) M = N is true

Solution M = real and N = lareSo (i) Yes (ii) Yes (iii) Yes

English 2 Twelfth Night ndash Shakespeare

A noble man named Orsino in the kingdom of Illyria is deeply in love with a lady called lady Olivia She is in mourning for her dead brother so she will not even think about marriage At this time a sea storm causes a terrible shipwreck and a young lady called Viola is swept onto the shore She thinks that her twin brother Sebastian is drowned A sea captain tells her about Orsino and his love for Olivia Viola wishes to work in Oliviarsquos home but feels she will not be employed So she dresses as a man calls herself Cesario and gets work at the house of OrsinoViola (now Ceasario) is much liked by Orsino and becomes his page She falls in love with Orsino Orsino sends Ceasario to deliver messages to Olivia Olivia herself falls for the beautiful young Ceasario believing Viola to be a man

2 Answer the following questionsa Why does Orsino ask the musicians to play onOrsino asks the musicians to play on because music feeds his desire He calls upon the musicians to play music so that his hunger for love could be replenished with an excess of musicb What does Valentine tell about OliviaWe learn from Valentine that Olivia is in mourning for her brother she wears a veil and has vowed that no one will see her face for another seven yearsand she refuses to marry anyone until thenc From the exchange between Orsino and Valentine what do you think their relationship isValentine is one of orsinod attendants He was sent to Olivia as a messenger of love but was not allowed to speak to here Who is Olivia mourning for and whyOlivia is mourning for her dead brother

Homework Q fHistory and Civics

Growth of Nationalism

Important dates to remember1769-Napoleon born on 15thAugust1789-Fall of Bastille on 14th July and the beginning of the French revolution declaration of the rights of Man on 26thAugust1793-King Louis XVI executed on January 211764-The Sugar Act passed1765-The Stamp act passed1774-The first congress of Philadelphia1776-The declaration of American Independence of on 4th July1777-Defeat of the British at Saratoga1781-Surrender of lord Cornwallis at Yorktown1783-The treaty of Versailles1804-Napoleon becomes the emperor1813-Battle of Leipzig or Battle of nations in which Napoleon was defeated by the Allies1815-Battle of Waterloo June 18 in which Napoleon was defeated and captured1821-Death of Napoleon in StHelena1860-Abraham Lincoln elected President of the USA1861-The civil war began 1864-Abraham Lincoln elected President of the USA for the second time1865-Slavery abolished in the US

Name the following- The queen of Louis XVI

Marie Antoinette The three philosophers of France

VoltaireMontesquieuJean Jacques Rousseau

The British general whose surrender brought the war in America to an endLord Cornwallis

The first president of the USAGeorge Washington (1732-1799)

The first southern state to secede from the unionSouth Carolina

The author of the book lsquoUncle Toms CabinHarriet Beecher Stowe

Homework-Learn

Class IXSubject Topic Summary Execution

Economics

Types of economies Today I am going to share you the concept of economic growth and economic development Few questions will be given from the previous study material dated 942020

Meaning of economic growthAnswer) The term economic growth generally means anincrease in national income or per capita output or income over time It indicates towards quantitative growth of a country

Meaning of economic developmentAnswer) Economic development is defined

as a process whereby the real per capita income of a country increases over time along with fall in poverty ratio unemployment and income inequality etc

Distinguish between economic growth and economic development

Basis Economic growth

Economic development

Scope It has narrow scope as it refer only to rise in per capita income

It has wide concept since it includes qualitative changes as well

Concerned matter

It is concerned with the rise in income

It is concerned with not only rise in income but also reduction of poverty income inequality and unemployment

Focus Economic growth does not focus on economic development

Economic development focus on economic growth plus qualitative changes

Distinguish between capitalist economy and socialist economy

Ownership

Motive

Tool

Means of production are owned and managed by private people

Self interest and profit earning is the main motive

Price mechanism is a main tool to solve the economic problems

Means of production are owned and managed by the government

Social welfare is the main motive

Economic planning by the government is the main tool to solve the economic problem

Competition

Distribution of income

There exist large competition among buyers and sellers

There is existence of large inequalities of income

There is no such competition

There exist less inequalities of income

Math Topic ndash AlgebraChapter -Factorisation

Study item Factorising by taking out common factorSome solved sums from exercise 41

1) (i) 8xy3 + 12x2y2

= HCF of 8xy3 and 12x2y2 is 4xy2

= 4xy2(2y + 3x )

4) (ii) 28p2q2r ndash 42pq2r2

= HCF of 28p2q2r and 42pq2r2 is 14pq2r = 14pq2r (2p - 3r )5) (ii) 14mn + 22m - 62p=HCF of 14mn 22m and 62p is 2= 2(7mn + 11m - 31p)7) (ii) 3a(x2 + y2) + 6b (x2 + y2) = HCF of 3a(x2 + y2) and 6b(x2 + y2 ) is (x2 + y2)= ( x2+ y2 )(3a + 6b )9) (ii) x(x2 + y2 ndash z2 ) + y(-x2ndashy2 + z2 ) ndash z(x2+ y2 ndash z2 )= x(x2 + y2 -z2) ndash y-(x2 + y2 -z2) -z(x2 + y2 ndash z2)=x(x2 + y2-z2) -y( x2 + y2-z2) ndash z (x2 + y2 -z2)= (x2+ y2 ndash z2)(x ndash y ndash z )

Commercial Studies

Introduction to Accounting and Book-keeping

Today I am going to share you the meaning of Accounting and Book-keeping and its related terms bullAccounting bullBook Keeping bullAccountsbullTypes Of Accounts bullAccounting Cycle

bull Meaning of accounting

Ans )Accounting is the art and science of recording classifying and summarising monetary transactions

bull Meaning of Book-keeping

Ans) Bookkeeping is the art of recording business transactions with the view of having a permanent record of them and showing their effect on wealth

bull Meaning of account

Ans) The term account means a record of business transactions concern a particular person of firm asset or income or expense It is a summarised record of all transactions which take place in an accounting year

bull Types of accountsPersonal accounts ndash Personal accounts relating

to person and Organisation are known

as personal accounts Example Ramrsquos Account ABC amp Co Account etc

Real account - The accounts related to tangible and intangible assets are called real accountsExample Cash Account Furniture Account etc

Nominal account- Accounts related to expenses losses incomes and gains are known as nominal accountsExample Wages Account Salary Account Discount Account etc

bull Accounting cycle Accounting cycle refers to a complete sequence of accounting activities It begins with recording of transactions and ends with the preparation of a balance sheet

English 1 Transformation of sentences

Sentences A sentence is a group of words which makes complete sense

a Assertive sentencesb Imperative sentencesc Interrogative

sentencesd Exclamatory sentences

Sentences can be changed from one grammatical form to another without changing the meaning of the sentence This is known as transformation of sentences

Exercise 6Rewrite the following sentences according to the instructions given below without changing their meanings

1 As soon as he saw the beer he jumped into the river ( Begin No sooner)

2 None but brave deserve the fair (Begin the bravehellip)

3 This box is too heavy for me to lift ( Use so hellip That instead of too)

4 No one other than a king can live like James Luxurious ( Begin only James)

5 Oh for the wings of a dove (Begin I wishhellip)

BENGALI(2ND LANGUAGE)

ldquo বঙগভমির পরমি ldquo াইকেল ধসদন দতত

পব13পোসঠ আসোলিচত ৩ পরবোস দৈদসবর বস ীবতোরো Pলিদ স এ লেদ -আকো সত-োলি লেদ তোস - ক) বকতো লেক কোর লেো লেকো কলিবতোর অং ) কোর পরলিত বকতোর এই উলিকত গ) এ লেদ আকো সত বসত কী বলিঝসয়স4 ীবতোরো বসত কী লেবোঝ ঘ ) আসোচয অংসর তোৎপP13 কী

উ -ক ) বকতো স কলিব মোইসক ম3দ দতত

Types of AccountPersonal AccountReal AccountNominal AccountBalance Sheet (opening)

কলিব মোইসক ম3দ দসততর রলিচত বঙগভলিমর পরলিত কলিবতোর অং ) কলিব বঙগী অ13োৎ লেদমোতোর পরলিত কলিবর এই উলিকত গ ) এ লেদ আকো বসত কলিবর মোব লেদী রপ আকো লেক লেবোঝোসো সয়স4 আকো লেসক লেPম তোরো স পসর লেতমলি ীব লেদ রপ আকো লেসক পরো রপ তোরো স পরসত পোসর এই মভোবোর কোই কলিব বসস4 ঘ ) পরবো Pোতরোয় Pলিদ কলিবর লেদ আকো লেসক ীব তোরো রপ পরো স পসর তোসত কলিব লিবনদমোতর দঃলিত কোর মতয লিবসর সবোভোলিবক পলিরলিত এবং মোষ মরী তোই পরবোস Pলিদ তা োর মতয য় তবও কলিব লিবচলিত সব ো কোর পলিবীসত লেকউ অমর য় লিক4ই অকষয় য় দীর লেPম লিচরপরবোমো লেতমলি মোসষর ীবও চমোতোই ীব - সতবধতোই মতয ীব দীসত মোষ লিতয পরবোমো তবও লেPব মোষ আপ কতকসম13র মো3যসম মোসষর মস লিসসদর সথো কসর লিসত পোসর তোরো লিচরভোসবর সয় মোসষর মস লিবরো কসর তোসদর মস3য লেকউ পGভসত লিবী সয় গোসও মোসষর মস তোরো লিতযপলিত লিতযবলিনদত

Hindi 2ndlang

काकीी(लिसयारामशरणगपत)

इस कहानी म क न यह बतान का परयास निकया ह निक बचच अपनी मा स निकतना परम करत ह शयाम अबोध बाक ह वह अपनी मा क मरन क बाद उसन अपनी मा क लिए बहत रोया बाद म उस पता चा निक उसकी मा राम क घर ची गई ह आकाश म उडती हई पतग दकर उस हष हआ निक पतग क दवारा वह अपनी मा को नीच उतारगा इसक लिए वह अपनी निपता की जब स दो बार सवा रपया निनकाकर पतग और दो मोटी सी मन वाी अपन भाई स काकी एक कागज पर लिवा कर पतग म लिशव का दिदयानिनकाकर पतग और दो मोटी सी मन वाी अपन भाई स काकी एक कागज पर लिवा कर पतग म लिचपका दिदयाभोा और शयाम कोठरी म रससी बाधनी रह थ तभी उसक निपता करोध म आकर उन स पछ निक कया उनकी जब स रपया निनकाा हभोा डर क मार बताया निक शयाम इस पतग क दवारा अपनी काकी को राम क यहा स उतारना चाहता हनिवशशवर(शयाम क निपता)न फटी पतग उठाकर दी तो उस पर काकी लिा थावह हत बजि होकर वही ड रह गएउनहोन सोचा निक मन अपन पतर को मारा जोनिक अनजान और निनदष थावह अपनी मा कोनिकतना पयार करता ह

helliphellipContinue to next

Computer Application

Java Programming Prog 1Write a java program to input two numbers from user and display the sum or product of them as per user choice Use switch case statementSolve public class sum_product public static void main(String args[]) Scanner sc=new Scanner(Systemin) int abc Systemoutprintln(ldquoEnter two numbersrdquo) a=scnextInt() b=scnextInt() Systemoutprintln(ldquoPress 1 for sum or 2 for productrdquo)

c=scnextInt() switch(c) case 1 Systemoutprintln(ldquoThe sum will be =rdquo+(a+b)) break case 2 Systemoutprintln(ldquoThe product will be =rdquo+(ab)) break default Systemoutprintln(ldquoWrong Inputrdquo) Home Work - Practice in your computer using bluej

Subject Eng Literature (The Merchant of Venice ndash William Shakespeare)Topic Act I Scene 2 Lines 92 to 126 (End of scene) Date 13th April 2020 (5th Period)

[Students should read the original play and also the paraphrase given in the school prescribed textbook]Summary Questions amp Answers

o After Portia has expressed her opinion about the suitors Nerissa informs that she need not bother about any one of them as they have decided to quit Belmont at the earliest opportunity because they do not believe in trying their luck by the caskets which is the only way of winning Portia

o Nerissa then enquires of Portiarsquos opinion about Bassanio who once visited her in the company of the Marquis of Montferrat and says that she had never come across such an ideal love deserving the fairest lady for his bride

o Portia seems to remember Bassanio quite correctly and says that she agrees with Nerissa At this moment a servant informs Portia that the Prince of Morocco has arrived to try his luck by the caskets

o Portia tells Nerissa that if she could welcome this new suitor as gladly as she says farewell to the previous ones she would be glad of his arrival However if he happens to have the virtues of a saint but the black complexion of a devil she would prefer to have him for religious consolation rather than as a husband

(1) NERISSA You need not fear lady (Line 97-103)

the having any of these lords they have acquainted me with their determinations

which is indeed to return to their home and to

trouble you with no more suit unless you may be wonby some other sort than your fathers imposition depending on the caskets

PORTIA If I live to be as old as Sibylla I will die as chaste asDiana unless I be obtained by the manner of my fatherswill I am glad this parcel of wooers are so reasonablefor there is not one among them but I dote on his veryabsence and I pray God grant them a fair departure

(a) Elucidate the idea expressed in the first speech of the above dialogue

In the first speech Nerissa assures Portia that she need not have any fear of being compelled to marry anyone of the suitors who had lately come to Belmont She informs her that they have all decided to return to their respective countries(b) Illuminate the meaning of the phrase ldquoyour fatherrsquos imposition depending on the casketsrdquo

Nerissa means that the suitors of Portia do not find the conditions imposed by the will of her father to their liking They are too hard for them These conditions are that in the event of a suitor failing to choose the right casket (i) he should never disclose to anybody which casket he chose (ii) he can never marry and (iii) he should take his departure immediately(c) Explain the meaning of the term lsquoSibyllarsquo

lsquoSibyllarsquo is the name given by Romans and Greeks to a prophetess inspired by some deity usually the sun-god Apollo She had a very long life The god Apollo granted her as many years of life as she could hold grains of sand in her hand(d) Elucidate the meaning of the term lsquoDianarsquo

lsquoDianarsquo is the goddess of hunting She is also regarded as a symbol of virginity because she never fell in love and never

married(e) Explain the meaning of the first two lines of Portiarsquos speech

Portia says that even if she is to live for centuries like Sibylla she would not marry except in accordance to her fatherrsquos will She asserts that she would not mind remaining unmarried and untouched by a man like Diana the virgin the goddess of hunting unless a man is able to win her by passing the test laid down by her father

Class XSubject Topic Summary Execution

Hindi 2nd

Langबड घर की बटी( मशी परमचद)

lsquoबड घर की बीटीrsquo कहानी का उददशय मधयम वग की घर समसया को सझा कर सगदिठत परिरवार म मिम जकर परम स रहन का सदश दना ह घर म शानित tानिपत करन की जिजममदारी नारी की होती ह यदिद नारी समझदार ह उसम धय और परिरवार क परनित परम ह तो कोई भी घटना परिरवार को निवघदिटत नही कर सकती या कहानी परिरवार को सगदिठत करत हए परम सौहाद स एक रदसर की भावनाओ को समझ करउनका सहयोग करत हए जीवन यापन करन की पररणा दती ह मशीपरमचदर जी न इस कहानी म सयकत परिरवार का परनितनिनमिधतव निकया ह यह कहानी बनी माधव सिसह जो गौरी पर क जमीदार क उनक दो पतरो की हशरी कठ ा निबहारीशरीकात का निववाह एकजमीदार घरान की पतरी आनदी स हआ थाआनदी न द को ससरा क वातावरण म ढालिया थाएक दिदन आनदी का अपन दवर ा निबहारी स झगडा हो जाता ह दोनो भाई एक रदसर स अग होन की कोलिशश करत हसभी बह आनदी न अपन मधर वयवहार स ा निबहारी को घर छोडकर जान स रोक लिया| इस पर बनी माधव सिसह न कहा निक बड घर की बटी ऐसी ही होती ह जो निबगडा काम बना ती ह अतः शीषक साथक ह बड घर की बटी आनदी ह

helliphelliphelliphellipContinue to nextBiology Topic ndash Chp-1

CellWelcome to new session 2020-21Today we will start with Chpter 1 cell CELL

Protoplasm+Cellmembrane Or Cell wall

Cytoplasm+Neucleus

Cytoplasmic+ CytoplasmicOrganelles Inclutions(mitochondria (food Golgi bodies pigments)Ribosome)

What is cellbull Cell is the structural and functional unit of living organismbull According to number of cells organisms areUnicellular - Amoeba bacteria Multicellular - Rose Mango Tiger HumanSmallest cell -bacteria Longest cell - Nerve cellLargest cell - Ostrich egg cellCells are of different size and shapes according to their functionsQ2Write chief functions of following cellorganelles

Q3What is tonoplastVacuoles covered by a covering called tonoplast

Bengali(2Nd

Language)

ফ ফটক ো ফটক (কলিবতো ) ভোষ মসোপো3 gtPোয়

একটি লেমসয়র ীবস লেপরম লিকভোসব ফসট ওসঠ তো লেদলিসয়স4 কলিব লেপরম Pই য় লেই ময়ই বনত কোস পলিরত য় ফ লেফোটো বো োসফোটো লেটো ব2 কো য় লেমসয়সদর ব gtয13 লেপরসমর 4লিব ফসট উসঠস4 এই কলিবতোয় লেপরম মোষসক মত gtযর মস লেফস লিদসয় পরকষস বাোচোসোর gtয োত বো2োয় কলিবতোয় লেমসয়টির পসব13র দঃসর কো বো সও লেমসয়টি লেই পসর পলিক সত চোয়ো োরী ীবসর কোস4 পরম লেPৌবস লেপরমসক পোবোর পরব ইচছো োকসও তো পসর লেলিতবোচকতোয় পলিরত য় কলিব ভোষ মসোপো3 যোয় লেP ক লেপরসমর

কলিবতোয় ব gtযবহত লিবসষ লিক4 সvর অ13 লেদওয়ো ১) রসবোো= লেP লিবলিভনন রকম ডোকসত পোসর২) ো= পোর ৩) ঠলি = লেচোসর বZ৪)আই বস2ো=অলিববোলিত৫)শইসয় = োলিয়ত কসর৬)োতপাোচ= লিবলিভনন পরকোর৭)দ2োম = v কসর বZ কসর লেদওয়ো৮)লেরলিং =লেোোর দৈতরী লেব2ো৯) বনত= একঋত১০) পাোর = বসকরো2

Organelles Functions

1 Endoplasmic reticulum

2 Mitochondria

3Golgibodies

4 Ribosome5Lysosome

6Plastids

7 Centrosome

i) Supportive framework for the cellii) Synthesis and transpost of proteinsRelease of energy in the form of ATPi) Synthesis and secretion of enzymes hormoneii) Formation of vacuoles lysosomei) Protein Synthesisi) Intracellular digestionii) Destroy foreign substancei )Leucoplast - stores starchii)chloroplast - trap solar energyiii) Chromoplast - imparts colour toflowers amp fruitsi) Initiates and requlates cell division

কলিবতো তোর অ13সক ভোষোয় পরকোো কসর ঘলিরসয় ব যকত কসরস4 লেপরসমর ফতো আর লিবফতো লেক গোঢ় কসর লেদোসো কলিব ভোষ মসোপো3 যোসয়র অলিভবসর অ যলিদক

Economics

Factors of Production

Welcome to the new sessionToday we are going to start the first chapter of Class XThe name of the chapter is Factors of productionBy the name I hope you all can recall a glimpse of what you have learnt in the second chapter of Class IX

NowProduction is the process of creating the various goods and services which are consumed by the people of the country to satisfy their wants

Thus it is the process in which some materials are transformed from one form to another to create utility and value in goods

For example utility can be created by changing the form of a commodity ie

Making of table out of wood by a carpenter for his customer here the wood is getting transformed into table creating utility for his customer and he can also command a price for it

On the other hand Housewives perform very

useful activities at home which create utility but their domestic activities are not included in production because they have no money value

So we can also say that Production denotes two things firstly creation of utility and secondly creation of value

Production is not complete unless it reaches the consumer

An increase in production will increase the economic welfare of the consumers and hence the aim is to raise the production level of the country

Again production of a good or service is only possible if certain resources or

Questions

1 What do you mean by production

Answer Production means the creation of goods and services for the purpose of selling in the market

In fact production involves the transformation of inputs into outputs

Hence production denotes two thingsCreation of utility and creation of valueUtility and value can be created by changing the form by changing the place by changing the time and by rendering services

Example Transformation of raw

materials into finish goods such as potter creates utility by converting mud into utensils assembling of small parts to make bigger machinery

Production also includes services such as distribution and marketing

2 What are the factors of production

Answer Factors of Production refers to the resources and inputs needed for producing goods and servicesThese inputs can be classified as

Land Labour

Capital Enterprise

Land Land is defined to include not only the surface of the earth but also all other free gifts of nature(for example mineral resources forest resources and indeed anything that helps us to carry out the production of goods and services but is provided by

inputs are used together in right proportion

A resource or an input which helps in the process of production to obtain an output is called FACTOR OF PRODUCTION

These factors of production can broadly be categorized into four parts 1LAND 2LABOUR3CAPITAL4ENTERPRISE (ORGANISATION)or Entrepreneur

The above factors are all interdependent on each other and they play a major role in production process

FACTORS OF PRODUCTION

LANDCAPITAL

LABOUR ENTREPRENEUR

nature free of cost)LabourLabour refers to the human efforts that need to be combined with other factors of production for creating an output

CapitalAll man ndash made means of production is called capita example machineries which help in further production Money when used for starting any business for purchasing raw materials machinery tools etc it is regarded as capitalCapital also includes physical capital like factories machineriestoolsbuildingsequipments etcEnterpriseThe task of bearing risks is called enterprise and the person who bears these risks of business is called the entrepreneurThus an entrepreneur is one who organises production takes important decisions regarding production hires and purchases factors of production and bears the risk and uncertainty involved in productionOrganisation refers to the services of an entrepreneur who controls organises and undertakes all risks One who plans organises and manages a business enterprise is an organiser

Physics Chapter 1 Force

Force is an external agent capable of changing the state of rest or motion of a particular body It has a magnitude and a direction The direction towards which the force is applied is known as the direction of the force and the application of force is the point where force is applied The Force can be measured using a spring balance The SI unit of force is Newton (N)

Question 1

State the condition when on applying a force the body has

(a) the translational motion

(b) The rotational motion

Solutions

(a) Translational motion is produced when the body is free to move

(b) Rotational motion is produced when the body is pivoted at a point

Question 2

Define moment of force and state its SI unit

Solutions

The moment of force is equal to the product of the magnitude of the force and the perpendicular distance of the line of action of force from the axis

of rotation

The SI unit of moment of force is Newton times meter

= Newton meter (Nm)

Commercial Studies

Stake holders In this topic you will be come to know about the meaning and concept of stakeholders

How stakeholders are different from shareholders

Questions1 What do you mean by the term stake holdersAnswer) The term stake holders have developed from the words which mean an interest or expected benefit Stakeholders mean all those individuals groups and Institutions which have a state (interest) in the functioning and performance of a commercial organisation or a business enterprise2 What do you mean by share holdersAnswer) The person and Groups who own the shares of the joint stock company by providing capital to the company are called shareholders Shareholders are the internal stakeholders shareholders are one out of several stake holders3 How are shareholders different from stakeholdersAnswer)i) The term shareholders is related to only joint stock company whereas stakeholders are related with all business organisationsii) Stakeholders maybe any individual having financial stake in business organisation whereas a shareholders are those individuals who are holding shares in the company4) How are shareholders different from creditorsAnswer) i) Shareholders are internal stakeholders while creditors are external stakeholdersii) Shareholders invest in the capital of the company whereas creditors give loan to the companyiii) Shareholders are the members of the company with voting rights but creditors are not the members of the company

English 1 Transformation of sentences

Sentences A sentence is a group of words which makes complete sense

e Assertive sentencesf Imperative sentencesg Interrogative sentencesh Exclamatory sentences

Sentences can be changed from one grammatical form to another without changing the meaning of the sentence This is known as transformation of sentences

Exercise 1 Change the following affirmative sentences into Negative sentences

a He is a good manHe is not a bad man

b Ram loves SitaRam is not without love for Sita

c Only he stood first in the classNone but he stood first in the class

d Ankit was wiser than he

He was not so wise as Ankite He did it

He did not fail to do itf As soon as I reached college the

bell rangNo sooner did I reach college than the bell rang

g He finished everythingHe left nothing unfinished

h It always pours when it rainsIt never rains but it pours

Math Topic Commercial MathematicsChapter ndash Goods and services Tax

What is GSTAns It is a abbreviated term of Goods and Service Text which is an indirect tax levied on the sale of goods and rendering servicesSome terms related to GSTDelar Any person who buys goods or services For resale is known as a delar A delar Can be a firm or a companyIntra-state sales Sales of goods and services within the same state or same union territory are called intra- state salesInter-state sales Sales of goods and services outside the state or union territory are called Inter-state sales4) Input GST GST is paid by dealers on purchase of goods and services are called input GST5) Output GST GST is collected from customers on sale of goods and services are called output GST6) Types of GST There are three taxes applicable under GST(i) Central Goods and Services Tax (CGST)(ii) State Goods and Services Tax (SGST) or Union Territory Goods and Services Tax (UTGST) Both these taxes are levied on intra-state sales Here GST is divided equally among central and state governments(iii) Integrated Goods and Services Tax (IGST) IGST is levied on inter- state sales It is also levied on import of goods and services into India and export of goods and services from India

Subject Eng Literature (The Merchant of Venice ndash William Shakespeare)Topic Act III Scene 4 Lines 1 to 44 (Portia hellip To wish it back on you fare you well Jessica)[Students should read the original play and also the paraphrase given in the school prescribed textbook]

Summary Questions amp AnswersIn this scene we suddenly find a new element in the character of Portia We have already seen her possessed of every graceful womanly quality but now she shows that she is capable of rapid decision and determined action She shows this by her sudden resolve to hasten to Venice with a daring scheme for the rescue of Antonio This is an important scene in the dramatic action for it leads up to and renders possible the striking events of the famous trial scene which is one of the greatest striking elements of the play Moreover the fact that all the characters of importance are now assembled together in Venice makes the union of the main plot and the secondary story complete

(1) LORENZO Madam although I speak it in your presence(Line 1-9)

You have a noble and a true conceit

Of god-like amity which appears most strongly

In bearing thus the absence of your lordBut if you knew to whom you show this honourHow true a gentleman you send reliefHow dear a lover of my lord your husbandI know you would be prouder of the workThan customary bounty can enforce you

(a) Where is Lorenzo Why is he here To whom is he referring as lsquoMadamrsquo

Lorenzo is at Portiarsquos residence He had met Salerio on the way and Salerio had begged him to come along with him to

o In this scene Portia Nerissa Lorenzo Jessica and Balthazar appear

o Portia requests Lorenzo and Jessica to be in charge of her house during her absence from Belmont because she and Nerissa have decided to spend the days in meditation and also in visiting the holy places in the neighbourhood of Belmont She has already instructed her people to acknowledge both Lorenzo and Jessica as master and mistress of house during her absence Lorenzo and Jessica gladly agree to look after the house of Portia

handover the letter from Antonio to Bassanio The letter carried the bad news about Antoniorsquos arrest for non-payment of loan taken from Shylock Hence Salerio might have preferred company to break this bad news to Bassanio He is referring to Portia as Madam(b) What does Portia say on hearing the above extract

Portia says that she has never regretted doing good to others Friends who spend a lot of time together and really are there for each other have many traits in common As Antonio is Bassaniorsquos best friend saving him is like saving Bassanio who is like her own soul She asks Lorenzo to take care of management of the house till Bassanio is back(c) What does Portia send with Bassanio and why

On hearing about Antoniorsquos troubles on account of Bassanio her husband Portia immediately sends him with enough gold to repay the debt many times over to Venice to help Antonio out of his misfortune

(2) Lorenzo Madam with all my heart (Line 36-40)

I shall obey you in all fair commands

Portia My people do already know my mindAnd will acknowledge you and JessicaIn place of Lord Bassanio and myselfSo fare you well till we shall meet again

(a) Where are Lorenzo and Portia at this time What lsquofair commandsrsquo are given to Lorenzo

Lorenzo and Portia are at Belmont during this scenePortia reveals to Lorenzo that she has sworn to contemplate in prayer at a monastery around two miles away until her husband returns from Venice She tells him that Nerissa would accompany her and asks him to manage the house with Jessica till things are settled In response Lorenzo tells her that he would be obliged to do whatever she asks him to do(b) Where is Portia actually going and why

Portia tells Lorenzo that she would live a life of contemplation and pray at a monastery which is two miles away from her place In reality Portia plans to go to Venice in disguise with Nerissa and argue the case in defense of Antonio She is very sure that her plan would succeed

ClassXI (ScienceHumanitiesCommerce)Subject Topic Summary Execution

Computer Science

(APC)

Ch ndash 1 Numbers

(Numbers in different bases and

their Arithmatical operations)

Number System In computers Number System is defined as a writing system to represent the numbers in different ways ie we are using different symbols and notations to represent numbers There are four ways we can represent the number ndash Binary Decimal Octal and Hexadecimal

Decimal Number SystemThis number system consist 10 digits These are 0 1 2 3 4 5 6 7 8 amp 9

Binary Number SystemThis number system has only two digits these are 0 and 1 Here 0 stands for off while 1 stands for on

Octal Number SystemThis number system has 8 digits these are 0 1 2 3 4 5 6 amp 7

Hexadecimal Number SystemThis number system has 16 digits these are 0 1 2 3 4 5 6 7 8 9 A B C D E F Here the value of the alphabets are as follows A=10 B=11 C=12 D=13 E=14 F=15

Rules for conversion decimal number to Binary1 Divide the decimal number by 22 If the number will not divide equally by 2 then round down the answer to the nearest whole number (integer)3 Keep a note of the remainder it should be either 0 or 14 Keep repeating the above steps dividing each answer by 2 until you reach zero5 Write out all the remainders from bottom to top This is your binary solution

For example Lets convert 32 to binary 2 32 2 16 - 0 2 8 - 0 2 4 - 0 2 2 - 0 2 1 - 0 0 - 1

The binary equivalent of 3210 is 1000002

Try the follwing youself1 2410

2 4810

3 1210

History GROWTH OF NATIONALISM

The second half of the 19th century witnessed growth of political consciousness and a sense of Nationalism among the IndiansThere were various factors for growth of Indian Nationalism- As a result various political associations were formed in different provinces by the educated Indians Surendranath Banerjee organized a meeting of National conference at Calcutta Ultimately the National Congress was founded in Bombay in 1885This body became the vanguard of Indian struggle for freedom The congress leaders were known as moderates because they followed a policy of prayer and petition A large number of Indian leaders had experienced in political agitation The Political situation of England was also changed Moreover increasing revolutionary activities in Maharashtra Punjab and Bengal became serious concern to the British Government In this

QUESTION1 What do you mean by Nationalism ANSWER 1 Nationalism is defined as loyalty and devotion to own nation especially a sense of national consciousnessQUESTION 2 What are the causes of nationalism ANSWER 2 There were various factors for growth of nationalism

1 Spread of western education2 The progress of vernacular press and

patriotic literature3 The economic exploitation of our

country by the colonial rulers4 International affairs

QUESTION 3 Who organized National conference in Calcutta in 1883 ANSWER 3 Surendranath BanerjeeQUESTION 4 When did Indian National Congress formANSWER 4 Indian National Congress was formed in 1885 in BombayQUESTION 5 Who were ModeratesANSWER 5 The Early Nationalists were also known as Moderates Their emergence marked

background Lord Curzon became Viceroy in India He had no respect for the Indian National Congress

the beginning of the organized national movement in India They believed in British justice and were loyal to them They followed a policy of prayer and petition They demanded constitutional reforms of our country Impotant Moderate leaders were Pherozshah Mehta Dadabhai Naorozi and Surendranath Banerjee etcQUESTION 6 What do you know about Extremism in Indian National movementANSWER 6 In the beginning of 20th century a new class of national leaders emerged in India which was different from the moderate groups They started more aggressive movement against the British empire The goal of extremists was ldquoswarajrdquo Important extremist leaders were Bal Gangadhar Tilak Lala Lajpat Rai Bipin Chandra Pal etcQUESTION 7 Mention the places which were the main centres of Revolutionary movementANSWER 7 Maharashtra Bengal and Punjab

Physics

Chapter Dimensional Analysis

(Summary)

The dimensions of a physical quantity are the powers to which the fundamental units are raised in order to obtain the derived unit of that quantit

The physical quantites lengthmasstime are represented by [L] [M] [T] resp let they are raised to powers ( dimesions) abc resp then any physical quantity can be represented by [ La Mb Tc ] Examples

1 Area area = L x B = [L] x [L] = [M0 L2 T0 ]

2 Density density = massvolume = [M][L3] = [ M L-3]

3 Velocity velocity = distancetime = [L][T] = [LT-1]HW Try to find out dimension of acceleration Acceleration = velocity timeNB One can find the SI Units Using Dimension Analysis Such as for area we have [L2] so its SI unit is m2

Biology Topic ndash Chp-1 The living world

Today we will start the first chapter the living world Here we discuss about the characteristics of living organism and what are the difference between them and nonliving substances We also discuss about the contribution of different Scientists

There are over 500000 species of plants andover a million species of animal are present on earth Some 15000 new species were discovered every yearQ1 What is a living organismbull A living organism is primarily physico -chemical material that demonstrate a high degree of complexity is capable of selfRegulation possesses a metabolism and perpetuates itself through timeQ2 What are the differences between livingand non-livingsi) Compared with non-living living organisms

have more complex organised structure and their use of energy is more controlled amp efficientii) Living things reproduce their own kind by forming new cells which contains copies of their genesiii) Each organism has some degree of homeostasisie it is able to make adjustments so that internal environment remains constantQ3 Write contributions of following Scientists i) Aristotle - One of the first theories in Biology places all living things in a hiearchieii) AV Leeuwenhoek - was the first to observe living single celled organisms under microscopeii) Carolus Linnaeus - developed the binary system for naming of organisms and classificationiii) Geregor Johann Mendel ndash discoverbasic principles of inheritanceHomework i) C Darwin ii)Schleiden

Math Trigonometric functions

1 Overviewi) Trigonometry The word lsquotrigonometryrsquo is derived from the Greek words lsquotrigonrsquo and lsquometronrsquo which means measuring the sides of a triangle An angle is the amount of rotation of a revolving line with respect to a fixed line Usually we follow two types of conventions for measuring angles ie a) Sexagesimal system b) Circular system In Sexagesimal system the unit of measurement is Degree In Circular system the unit of measurement is Radian ii) Relation between degree and radianThe ratio of circumference of a circle to its diameter is always a constant This constant ratio is a number denoted by π which is taken approximately as 227The relationship between degree amp radian measurements is as follows2 right angles = 180deg= π radians1radian = 180degπ=57deg16(approx) 1deg=π180 radianiii) Length of an arc of a circleIf an arc of length s subtends an angle θ radians at the center of a circle of radius r then s=rθiv) Area of a sector of a circleA sector is like a pizza slice of the

Q) Express the following angles in radiana) 45deg b) 40deg3730Ans a) We have 180deg=π radiansi e 45deg= πtimes45180 radian = π4 radiansb) 40deg3730= 40deg37+3060 minute= 40deg 37 +12 minute= 40deg+ 752 minute=40 + 75(2times60) degree=3258 degreeNow 180deg=π radianie 3258 degree= (πtimes325) (180times8) radians = 65π288 radiansQ) A circle has a radius of r=12 meters What is the length of an arc traced out by a 60deg angle in the center of the circleAns In this problem we know both the central angle (60deg) and the radius of the circle (12) All we have to do is plug those values into our equation and we get

s = 2π(12)(60360)s = 24π6s = 4πSo the length of an arc traced out by a 60deg angle in a circle with a radius of 12 meters equals 4π meters asymp 1257 metersQ) Find the area of the sector with a central angle 30deg and a radius of 9cmAns GivenRadius r = 9 cmAngle θ = 30degArea of the sector = θ360degtimesπr2

= 30360degtimes227times92=2121cm2

circle It consists of a region bounded by two radii and an arc lying between the radiiThe area of a sector is a fraction of the area of the circle This area is proportional to the central angle In other words the bigger the central angle the larger is the area of the sectorArea of Sector = θ2 times r2 (when θ is in radians)

Area of Sector = θ times π360 times r2 (when θ is in degrees)

COMMERCE

CLASSIFICTION OF HUMAN ACTIVITIES-ECONOMIC AND NON-ECONOMIC

Welcome to the new sessiontoday we are going to start the first chapter of Class XI The name of the chapter that we are going to start is

lsquoClassification of Human Activities ndasheconomic and non-economicrsquo

Now let us start the chapter by considering human beings and the activities they perform throughout the day

Human activities means all those activities that human beings undertake to satisfy their wants

Human wants on the other hand are the desire of human beings for goods (vegetables fruits rice etc) and services (services of doctors teachers lawyers etc) that they require to live

Now these human activities continue throughout life as human wants are unending unlimited and recurring as human beings desire for better living throughout their lives

Now human activities can be classified into two categories

Human activities

Economic activities Non-economic activities

Economic activities are

Questions1 What are human activities

Answer Human activities mean all those activities that human beings undertake to satisfy their wants

Example A man working in an office

A boy playing in the garden

2What are the characteristics of human activitiesAnswer the characteristics of human activities are as follows

Human activities are undertaken by men women and children and these activities involve human efforts

Human activities are undertaken to satisfy human wants which are unlimited

Human activities continue throughout life

Human activities are performed for both earning money and personal satisfaction

3What is economic activitiesGive example

Answer Economic activities are undertaken by human beings with the object of earning money acquiring wealth and thereby satisfying human wantsExample

Selling of goods by a shop keeper to his customer

A clinic run by a doctor Service of a teacher in school or college

undertaken by human beings with the object of earning money and acquiring wealth

These activities result in the production of economic goods and services

Example Human activities(ie working in factories officesshops) which produce direct economic benefits

Non-economic activities are inspired by human sentiments and emotions such as love for the family desire to help the poor and love for the country

Thus these human activities (eg praying playing sleeping) produce no direct economic benefits and they are also not related to earning money and acquiring wealth

4 What are the characteristics of economic activities

Answer The characteristics of economic activities are as follows

Economic motiveEconomic activities are undertaken to earn money and acquire wealth

ProductiveEconomic activities involve productiondistribution and exchange of goods and services to create wealth

Economic growthEconomic activities determine the level of economic development of a country and standard of living of its citizens

Socially desirableEconomic activities are socially desirable for society

Economic resourcesEconomic activities make use of all the economic resources such landlabourcapital etc

5 What do you mean by non-economic activitiesExampleAnswerNon-economic activities are inspired by human sentiments and emotions such as love for the family desire to help the poor and love for the countryThese activities are not undertaken for monetary gain but for onersquos satisfaction and happinessExample

a mother looks after her children

a student donates blood8 Differentiate between Economic activities and Non-economic activities

Economic activities

Non-economic activities

1to earn living and acquiring wealth2Result can be measured in terms of money

3ExampleBusinessprofession and employment

1 to obtain some satisfaction

2Result cannot be measured in terms of money

3ExampleFamily-orientedreligious socialCultural and national

BUSINESS STUDIES

BUSINESS ENVIRONMENT

Welcome to the new sessionToday we are going to start the first chapter and the name of the chapter is Business Environment

In todayrsquos world every business enterprise is a part of the society It exists and operates in association with various groups in society such as customers suppliers competitors banks and financial institutions government agencies trade unions media and so on All these groups influence the functioning of business in one way or the other They constitute the environment of businessConcept of Business Environment

The term lsquobusiness environmentrsquo refers to the sum total of all individuals institutions and other forces that lie outside a business enterprise but that may influence its functioning and performance

The main features of business environment

Totality of External forces General and Specific forces Interrelatedness Complexity Dynamic Uncertainty Relativity

The Interrelation between business and its environment

The business enterprise is an open system It continuously interacts with its environment It takes inputs

Prepare the following questions from todayrsquos assignment

1 What do you mean by business environment

The term lsquobusiness environmentrsquo means the aggregate of all forces factors and institutions which are external to and beyond the control of an individual business enterprise but they may influence its functioning and performance Business environment is the macro framework within which a business firm a micro unit operates It consists of several interrelated and interacting elements

2 Explain the main features of business environment in brief

Totality of External forces-Business environment is the sum total of all things external to a business environment

General and Specific forces-It includes both the forces general forces are the economic social political legal and technological conditions which indirectly influence all business enterprise Specific forces are the investors customers competitors and suppliers which influence individual enterprise directly

Interrelatedness-Different elements of environment are interrelated for an example growing awareness for health care has increased the demand for health foods

Complexity- Business environment id

(such as raw materials capital labour energy and so on) from its environment transforms them into goods and services and sends them back to the environment

Fig 1 Business Environment Relationship

complex in nature as the elements keep on changing example economic technological and other forces changes in demand for a product and service

Dynamic-Business environment is not static it keeps on changing

Uncertainty- Itrsquos very difficult to predict future events such as technology and fashion which occur fast and frequently

Economics Basic Economic ConceptsSub topic

Microeconomics and

Macroeconomics

Welcome to the new sessiontoday we are going to start the first chapter of Class XI The name of the chapter that we are going to start is Basic Economic concepts

Now Economics covers the study of human activities Human activities are those activities which are performed by humans to satisfy their wants

Thus Human wants are unlimited and therefore economic activities such as production exchange and consumption are needed in order to satisfy those wants

The study of economics is divided largely in two parts which areMicroeconomics and Macroeconomics

SUBJECT- MATTER OF ECONOMICS

MICROECONOMICS MACROECONOMICS

Questions1Who has coined the words micro and macro economics

Answer Ranger Frisch coined the words lsquomicrorsquo and lsquomacrorsquo in 1933 to denote the two branches of economic theory namely microeconomics and macroeconomics

2What is microeconomicsAnswer It is the study of behaviour of individual decision ndash making unit such as consumers firms etc

3 What is macroeconomicsAnswer Macroeonomics is the study of overall economic phenomena like employment national income etc

4 What is the importance of microeconomicsAnswer

Microeconomics helps in formulating economic policies which enhance productive efficiency and results in greater social welfare

It helps the government in formulating correct price policies

It explains the working of a capitalistic economy where individual units(producers and consumers ) are free to take their own decision

Micro means a small part in

microeconomics we do not study the whole economy Hence we study an individual consumer and his or her choices and a producer and his or her profit maximizing decisions in the market Thus it does not mirror what happens in the economy as a whole

Macroeconomics on the other hand studies the economy as a whole It is concerned with aggregate and depicts the entire picture of the economyMacroeconomics deals with the national income aggregate investment aggregate consumption etc

Features of Microeconomics It deals with small

parts of the country Hence it looks at

individual consumers firms and industries

It deals with individual income consumption and savings

It studies the determination of price of any product or factors of production

It deals with the working of market via the price mechanism which is nothing but the determination of price and quantity of a commodity by the forces of demand and supply

Features of Macroeconomics

It deals with the study of the economy as a whole

It is concerned with

5 Give a limitation of microeconomics Microeconomics fails to explain the

functioning of an economy as a whole It cannot explain unemployment illiteracy and other problems prevailing in the country

6 What is the importance of macroeconomics It gives overall view of the growing

complexities of an economic system It provides the basic and logical

framework for formulating appropriate macroeconomic policies (eg for inflation poverty etc )to direct and regulate economy towards desirable goals

7What is the limitation of macroeconomics It ignores structural changes in an

individual unit of the aggregate

8 Differentiate between Microeconomics and Macroeconomics

Microeconomics Macroeconomics

the study of aggregates

National income aggregate savings and aggregate investments are major concepts dealt within macroeconomics style

It studies the determination of general price levels

It investigates into the problem of unemployment and the achievement of employment

It studies the aspect of decision making at the aggregate and national levels

It includes all growth theories whether related to developed or developing economies it also includes the study of economic systems and the working of the economy under different systems

Note Both Micro and macro economics are complementary and should be fully utilized for proper understanding of an economy

1It studies economic aspect of an individual unit2It deals with individual incomeConsumption and savings

3 It facilitates determination of price of any product or factors of production

4 Itrsquos scope is narrow and restricted to individual unit

1It studies the economy as a whole

2It deals with the national income aggregate consumption and aggregate savings3 It facilitates determination of general price level in an economy

4 Itrsquos scope is wide as it deals with economic units on the national level

ACCOUNTS

Introduction to Accounting and Book-keeping

Today I am going to share you the meaning of Accounting and Book-keeping and its related terms bullAccounting bullBook Keeping bullAccountsbullTypes Of Accounts bullAccounting Cycle

bull Meaning of accounting

Ans ) Accounting is the art and science of recording classifying and summarising monetary transactions

bull Meaning of Book-keeping

Ans) Bookkeeping is the art of recording business transactions with the view of having a permanent record of them and showing their effect on wealth

bull Meaning of account

Ans) The term account means a record of

business transactions concern a particular person of firm asset or income or expense It is a summarised record of all transactions which take place in an accounting year

bull Types of accountsPersonal accounts ndash Personal accounts relating

to person and Organisation are known as personal accounts Example Ramrsquos Account ABC amp Co Account etc

Real account - The accounts related to tangible and intangible assets are called real accounts Example Cash Account Furniture Account etc

Nominal account- Accounts related to expenses losses incomes and gains are known as nominal accounts Example Wages Account Salary Account Discount Account etc

bull Accounting cycle Accounting cycle refers to a complete sequence of accounting activities It begins with recording of transactions and ends with the preparation of a balance sheet

Chemistry TopicAtomic Structure

Thomsonrsquos atomic modelThomson (1898) was the first to propose the model of an atomHe proposed that an atom can be regarded as a uniform sphere of positive electricity in which requisite number of electrons are embedded evently to neutralize the positive chargeThis is just like plums embedded in a pudding or seeds evently distributed in red spongy mass of a watermelonThis model of atom is known as ldquoPlum-Pudding modelrdquo or

Q1)What is the fundamental constituents of atomAns Electron Proton and neutrons are the fundamental constituents of atomQ2)What is the value of fundamental unit of electricityAnsThe charge carried by one electron is sad to be the fundamental unit of electricityIts magnitude is 48times10-10esuOr 1602times10-19C Q3)Name the element containing no neutronAnsOrdinary hydrogen atom or protium 1H1

Types of AccountPersonal AccountReal AccountNominal AccountBalance Sheet (opening)

ldquowatermelon modelrdquoThis model could explain the electrical neutrality of an atom but failed to explain the result of scattering experiment carried out by Rutherford in 1911So it was rejected ultimately

Q4)Why is an electron called universal particleAns Itrsquos mass and Charge are independent of its source

EVS Chapter 1 ndash Modes of Existence

Modes of existence When one speaks normally about the mode of existence of some group or individual one refers to their customs their mode of being their ethology their habitat in some way their feeling for a placeDifferent modes of exixtence are ndash

1 Hunting ndashGathering2 Pastoral3 Agricultural4 Industrial

1 Hunting and gathering Hunting and gathering mode of existence is characterized by obtaining food from hunting wild animals including fishing and gathering wild plants From their earliest days the hunter-gatherer diet included various grasses tubers fruits seeds and nuts Lacking the means to kill larger animals they procured meat from smaller game or through scavenging

Societies that rely primarily or exclusively on hunting wild animals fishing and gathering wild fruits berries nuts and vegetables to support their diet are called hunting and gathering societies

At least this used to be practice of human beings before agriculture is invented As their brains evolved hominids developed more intricate knowledge of edible plant life and growth cycles

Q) Write the features of Hunting ndash gathering societiesAns - There are five basic characteristics of hunting and gathering societies

i The primary institution is the family which decides how food is to be shared and how children are to be socialized and which provides for the protection of its members

ii They tend to be small with fewer than fifty members

iii They tend to be nomadic moving to new areas when the current food supply in a given area has been exhausted

iv Members display a high level of interdependence

v Labor division is based on sex men hunt and women gather

Political Science

Introduction to political science

Political science occasionally called politology is a social science which deals with systems of governance and the analysis of political activities political thoughts associated constitutions and political behaviorThe study of political science involves the study of both the

Answer the following questions-1 What is political science

Political science occasionally called politology is a social science which deals with systems of governance and the analysis of political activities political thoughts associated constitutions and political behavior

2 Short notes-

traditional and modern theories of politicsTraditionalClassical political sciencepolitical theory-Traditional political science is the study of politics before Second World War The methodology to study Politics was traditional (legal formaletc) the definition of politics traditional (Politics begins and end with state)area of study (constitution state machinery)was traditionalModern Political scienceModern political theory-Modern Political Theory critically examines the contemporary state of political theory making an assessment of the achievement and limitations of the Behavioural Revolution in its totality and reviews objectively the major paradigms and conceptual frameworks adopted by the disciplineContemporary attempts at the development of an integrated political theory involving the use of both traditional and modern concepts approaches and theories-Around late 1960s several political scientists realized the importance of both the traditional political theory and modern Political theory They began building an integrated theory of politics involving a systematic mixture of traditional and modern studies of politics It was held that the study of a complex and vast field like politics needs both traditional as well as

Classical political theory Modern Political theory

Homework-Learn

modern concepts and approaches for studying itrsquos all aspects

Subject Eng Literature (The Tempest ndash William Shakespeare) Topic Act I Scene 1 Lines 1 to 32 (Line 32 ndash Gonzalo hellip If he be not born to be hanged our case is miserable) Date 13th April 2020 (3rd Period)

[Students should read the original play and also the paraphrase given in the school prescribed textbook]Summary Questions amp Answers

[SUMMARY OF THE ENTIRE SCENE]

o The play starts with the scene of a severe storm at sea Alonso (King of Naples) Sebastian (Alonsorsquos brother) Ferdinand (Alonsorsquos son) Gonzalo Antonio (the usurping Duke of Milan) are in a ship in the midst of the storm

o The mariners are trying their best to control the vessel from running aground and are totally following the orders of their Master the Boatswain They have scant success

o The mariners become extremely unhappy and annoyed when most of the passengers arrive on the deck thereby hampering their effort to save the ship There is serious confrontation between them and the passengers who are part of the Kingrsquos entourage

o The mariners could not save the ship

SUMMING-UP

(i) Vivid description of the scene which gives a realistic description of terror and confusion of a tropical storm

(ii) Shows Shakespearersquos accuracy of knowledge in describing the naval operations and also matters of seamanship

(iii) The opening scene justifies the title ndash The Tempest

UNANSWERED QUESTIONS

(i) The King always travels with his entire fleet including his soldiers Where

(1) GONZALO Nay good be patient (Line 15-26)BOATSWAIN When the sea is Hence What cares these

roarers for the name of the king To cabin silence Trouble us not

GONZALO Good yet remember whom thou has aboardBOATSWAIN None that I more love than myself You are a

councillor if you can command these elements to silence and work

the peace of the present we will not hand a rope more use your authority If you cannot give thanks you have

lived so long and make yourself ready in your cabin for the mischance of the hour if it so hap [To the Mariners]

Cheerly good hearts [To Gonzalo] Out of our way I say

(a) To whom is the boatswain speaking What does he mean by lsquoNone that I more love than myselfrsquo

The Boatswain is speaking to Gonzalo the honest old councilor of the Duke of MilanBy using the words ndash lsquoNone that I love more than I love myselfrsquo means that for the Boatswain nobody is dearer to him than his own life

(b) What were the conditions that made the boatswain react in this way

The Boatswain reacts in this way because the storm is at sea and Alonso King of Naples Sebastian his brother Ferdinand his son Gonzalo Antonio the usurping Duke of Milan on board are in distress and in panic Thus they have rushed to the deck interrupting the work of the mariners

(c) What hope does Gonzalo take from the attitude of the boatswain

The insolent and authoritative attitude of Boatswain makes Gonzalo feel comforted He tells that there are no signs that the Boatswain will be drowned But his facial appearance and attitude shows that he is destined to die on land by hanging which in effect means that all on board will be saved Otherwise all the persons on board are doomed

(d) How can they lsquomake yourself ready in your cabinrsquo For what were they asked to make ready themselves

In order to make themselves ready in their cabin the

were the other ships

(ii) Why was the ship in that area Where was it coming from or going where

(iii) The ship broke apart What happened to those who were in the ship

passengers on board must prepare for death which they will possibly soon have to meetThey can retire to their cabins and offer prayers to the Almighty to save them from drowning

(e) What does the boatswain say when he is asked to be patient What does he order to the royal party

When the boatswain is asked to be patient and remain calm he says that he will be patient only when the storm will be over and the sea will be calm but as long as the storm blows and there is danger to the ship he cannot think of being patient He orders the royal party to go to the cabin and leave the mariners to their work

(2) GONZALO I have great comfort from this fellow (Line 27-36)

Methinks he hath no drowning mark upon him his complexion is perfect

gallows Stand fast good Fate to his hanging Make the rope of his destiny our cable for our own doth little advantage If he be not born to be hanged our case is miserable

(a) Why does Gonzalo regard the Boatswain in the midst of danger

In the midst of danger Gonzalo regards the boatswain because he feels that the Boatswain is a source of comfort and is bent upon to do his work sincerely which in this case is saving the ship and its passengers from the severest of raging storm

(b) What reasons does Gonzalo give when he says that none in the ship will die of drowning

Gonzalo is almost sure that none in the ship will die by drowning His says that there is no mark on the face of the boatswain that indicates that he will die by drowning On the other hand the lines on his face are strong indications that he will be hanged to death Therefore there shall be no danger of the shiprsquos sinking

(c) Explain the following ldquoStand fast good Fate to his hanging Make the rope of his destiny our cable for our own doth little advantage If he be not born to be hanged our case is miserablerdquo

The stated lines mean that if the will of destiny is to be carried out then the ship will not get wrecked and all the passengers will be saved The safety of the passengers therefore depends upon the will of fate being carried out in the case of the boatswain If however the boatswain is not to die by hanging then the passengers are also very unsafe because in that case the ship is likely to sink

(d) What order does the Boatswain give to the sailors

when he re-enters What does he say about the crying of the fellows inside the cabin

The boatswain orders the sailors to bring the topmast lower and bring the ship close to a stationary position with the help of the main sail He says that the fellows inside the cabin are moaning and crying in their distress louder than his voice and louder even than the roaring of the storm

Class XII (ScienceCommerceHumanities) Subject Topic Summary Execution

Computer Science

PropositionalLogic

Propositional logic is a procedure to provide reasoning through statementProposition A ststement that results in True or False is said to be proposition There are two types of propositionSimple proposition amp compound propositionSimple proposioton A simple proposition is one that is not a part of any other proposition Such sentential form of proposition is symbolized with english letters in short For example Ram is a claver student (TrueFalse)Where do you live (Not in True or False)Grapes are sweet (TrueFalse)It rains today (TrueFalse)Here we can see some statements anwer would be true or false but some staements answer can not give in terms of true or false Thus the sentences which can be answered in true or false are known as simple propositionAssigning propositon to a variableThe general syntax to assign propostion to a variable is as followsVariable = Simple propositonFor example A=Ram is a clever studentB= Grapes are sweetC= it rains todayCompound proposition

helliphellipto be continued in next classhelliphellipMath Relation Relation If A and B are two non-empty sets

then a relation R from A to B is a subset of AxB If R A x B and (a b) R then we say that a sube isinis related to b by the relation R written as aRbeg Let A be the set of students of class XII and B be the set of students of class XI Then some of the examples of relation from A to B arei) (a b) AXB a is brother of bisinii) (a b) AXB age of a is more than age of isinb Types of relation In this section we would like to study different types of relations We know that a relation in a set A is a subset of A times A Thus the empty set φ and A times A are two extreme relations For illustration consider a relation R in the set A = 1 2 3 4 given by R = (a b) a ndash b = 10 This is the empty set as no pair (a b) satisfies the condition a ndash b = 10 Similarly R = (a b) | a ndash b | ge 0 is the whole primeset A times A as all pairs (a b) in A times A satisfy | a ndash

Example 1 Let A be the set of all students of a boys school Show that the relation R in A given by R = (a b) a is sister of b is the empty relation and R = (a b) the primedifference between heights of a and b is less than 3 meters is the universal relationSolution Since the school is boys school no student of the school can be sister of any student of the school Hence R = φ showing that R is the empty relation It is also obvious that the difference between heights of any two students of the school has to be less than 3 meters This shows that R = A times A is primethe universal relation Example 2 Show that the relation R in the set 1 2 3 given by R = (1 1) (2 2) (3 3) (1 2) (2 3) is reflexive

b | ge 0 These two extreme examples lead us to the following definitionsDefinition 1 A relation R in a set A is called empty relation if no element of A isrelated to any element of A ie R = φ A times AsubDefinition 2 A relation R in a set A is called universal relation if each element of A is related to every element of A ie R = A times A Both the empty relation and the universal relation are some times called trivial relation Definition 3 A relation R in a set A is called(i) reflexive if (a a) R for every a Aisin isin(ii) symmetric if (a1 a2) R implies that (aisin 2a1)

R for all aisin 1 a2 Aisin(iii) transitive if (a1 a2) R and (aisin 2 a3) R isinimplies that (a1 a3) R for all aisin 1 a2 a3 AisinDefinition 4 A relation R in a set A is said to be an equivalence relation if R is reflexive symmetric and transitive

but neither symmetric nor transitiveSolution R is reflexive since (1 1) (2 2) and (3 3) lie in R Also R is not symmetric as (1 2) R but (2 1) isin notinR Similarly R is not transitive as (1 2) R and (2 3) R but (1 3) R isin isin notinExample 3 Show that the relation R in the set Z of integers given byR = (a b) 2 divides a ndash b is an equivalence relationSolution R is reflexive as 2 divides (a ndash a) for all a Z isinFurther if (a b) R then 2 divides a isinndash b Therefore 2 divides b ndash a Hence (b a) R which shows that R is isinsymmetric Similarly if (a b) R and (b c) R isin isinthen a ndash b and b ndash c are divisible by 2 Now a ndash c = (a ndash b) + (b ndash c) is even (Why) So (a ndash c) is divisible by 2 This shows that R is transitive Thus R is an equivalence relation in ZExample 4 Let L be the set of all lines in a plane and R be the relation in L defined as R = (L1 L2) L1 is perpendicular to L2 Show that R is symmetric but neither reflexive nor transitiveSolution R is not reflexive as a line L1 can not be perpendicular to itself ie (L1 L1) R notinR is symmetric as (L1 L2) Risin

L1 is perpendicular to L2rArr L2 is perpendicular to L1rArr (L2 L1) RrArr isin

R is not transitive Indeed if L1 is perpendicular to L2 and L2 is perpendicular to L3 then L1 can never be perpendicular to L3 In fact L1 is parallel to L3 ie (L1 L2) R isin(L2 L3) R but (L1 L3) Risin notin

Chemistry Solid state Characteristics if Solids(i)The particles are locked in fixed positions they are unable to change their relative positions and this brings a definite shape and volume of a solid(ii)In a solid the constituent particles are held by strong forces of attractionThe forces of attraction may be bonding or non bonding(iii)The constituent particles in a solid pack together as closely as possibleoccupying most of the available space within the solidThus the empty space in a solid is very smallThis makes a solid highly rigid and nearly incompressibleThis also explains why a solid has high density and exhibits slow diffusionClassification of Solids

Q1)Define Crystalline solids AnsA Solid that has a definite geometrical shape and a sharp melting pointand whose constituent particles (atomsmolecules or ions) are arranged in a long range order of definite pattern extending throughout the solid is called a crystalline solidExNaClQ2)Define Amorphous solids AnsA solid that does not have a definite shape and a sharp melting pointand whose constituent particles (atomsmolecules or ions) are not arranged in a definite pattern is called an amorphoussolid

Crystalline solidsAmorphous solids

ExGlassRubberQ3)Classify Crystalline Solids Crystalline Solids

Physics Coloumbrsquos Law (Summary)

Before Going Into Coloumbrsquos Law We Will First Learn What is Charge Properties of Charge and Always remember that charge is quantized ie a body always have static charge of magnitude equal to some integral multiple of fundamental electronic charge e= 16 x 10- 19 C

Charge is the property of matter that causes it to produce and experience electrical and magnetic effects The study of the electrical charges at rest is called electrostatics When both electrical and magnetic effects are present the interaction between charges is referred to as electromagnetic

There exist two types of charges in nature positive and negative Like charges repel and unlike charges attract each other

The type of charge on an electron is negative The charge of a proton is the same as that of an electron but with a positive sign In an atom the number of electrons and the number of protons are equal The atom is therefore electrically neutral If one or more electrons are added to it it becomes negatively charged and is designated as negative ion However if one or more electrons are removed from an atom it becomes positively charged and is called a positive ion

The excess or deficiency of electrons in a body gives the concept of charge If there is an excess of electrons in a body it is negatively charged And if there is deficiency of electrons the body becomes positively charged Whenever addition or removal of electrons takes places the body acquires a charge

The SI Unit of charge is coulomb (C) In SI units the current is a fundamental quantity having a unit of ampere (A) The unit of charge is defined in terms of the unit of current Thus one coulomb is the charge transferred in one second across the section of a wire carrying a

Ionic SolidsMetallicSolids

Molecular Solids

current of one ampere

As q = It we have1 C = (1 A) (1 s)

The dimensions of charge are [A T]

Properties of Charge

(1) Quantization of Charge Electric charge can have only discrete values rather than any value That is charge is quantized The smallest discrete value of charge that can exist in nature is the charge on an electron given as

e = plusmn 16 x 10- 19 C

This is the charge attained by an electron and a protonA charge q must be an integral multiple of this basic unit That is

Q = plusmn ne where n = 1 2 hellip

Charge on a body can never be (frac12)e (23)e or 57e etcWhen we rub a glass rod with silk some electrons are transferred from the rod to the silk The rod becomes positively charged The silk becomes negatively charged The coulomb is a very large amount of charge A typical charge acquired by a rubbed body is 10 - 8 C

Biology Reproduction in organisms

Welcome to this new session 2020-21Today in this first chapter we mainly discuss about reproduction types needs and life span of some organismsWe also discuss about difference between sexual and asexual reproduction

Q1 What is reproductionReproduction is defined as a biological processin which an organism gives rise to young onessimilar to itselfQ2 What are the needs of reproductionbulli) Reproduction maintain life on earthii) It enables the continuity of the species generation after generationiii) It creates genetic variation among populationsQ3 Define Life span and write some orgnisms life spanbull Life span is the period from birth to

the natural death of an organism- OrganismsLife span1 Butterfly 1 - 2 weeks2 Fruit fly 30 days3Dog 10-13 years4 Rose5-7 years5 Tortoise100-150 years6 Banyan Tree -200 - 250 yearsQ4 Reproduction is of two types in case ofanimals but in case of plants vegetative propagation is also present

Asexual Reproduction Sexual Reproductioni) Always uniparentalii) Gametes are not involvediii) Only mitotic division involvediv) Somatic cells of parents are involvedv) Offsprings are genetically similar to the parents

i) Usually biparentalii) Gametes are involvediii) Meiosis occurs during gametogenesis Mitosis occurs after fertilisationiv) Germ cells of the parents are involvedv) offsprings are genetically different from the parents

COMMERCE BUSINESS ENVIRONMENT

Welcome to the new sessiontoday we are going to start the first chapter of Class XII The name of the chapter is Business Environment

Already many of you have got some idea about the word business environment form the first chapter of business studies in class XI

In todayrsquos world every business enterprise is a part of the society It exists and operates in association with various groups in society such as customers suppliers competitors banks and financial institutions government agencies trade unions media and so on All these groups influence the functioning of business in one way or the other They constitute the environment of businessConcept of Business Environment

The term lsquobusiness environmentrsquo refers to the sum total of all individuals institutions and other forces that lie outside a business enterprise but that may influence its functioning and performance

The main features of business environment Totality of External forces General and Specific forces Interrelatedness Complexity Dynamic Uncertainty

Prepare the following questions from todayrsquos assignment

2 What do you mean by business environment

The term lsquobusiness environmentrsquo means the aggregate of all forces factors and institutions which are external to and beyond the control of an individual business enterprise but they may influence its functioning and performance Business environment is the macro framework within which a business firm a micro unit operates It consists of several interrelated and interacting elements

2 Explain the main features of business environment in brief

Totality of External forces-Business environment is the sum total of all things external to a business environment

General and Specific forces-It

Relativity

The Interrelation between business and its environment

The business enterprise is an open system It continuously interacts with its environment It takes inputs (such as raw materials capital labour energy and so on) from its environment transforms them into goods and services and sends them back to the environment

Fig 1 Business Environment Relationship

includes both the forces general forces are the economic social political legal and technological conditions which indirectly influence all business enterprise Specific forces are the investors customers competitors and suppliers which influence individual enterprise directly

Interrelatedness-Different elements of environment are interrelated for an example growing awareness for health care has increased the demand for health foods

Complexity- Business environment id complex in nature as the elements keep on changing example economic technological and other forces changes in demand for a product and service

Dynamic-Business environment is not static it keeps on changing

Uncertainty- Itrsquos very difficult to predict future events such as technology and fashion which occur fast and frequently

Business Studies

Human Resources Management

Human resource of an organisation are the aggregate of knowledge skills attitudes of people working in it

The management system which deals with human resources is called human resource management

Features of HRMbullComprehensive functionbullPeople-oriented

Question1) What do you mean by human

resource management Answer) Human resource management may be defined as that field of Management which has to do with planning organising and controlling the functions of procuring developing maintaining and utilising the labour force

bullAction oriented bullPervasive function bullContinuous function

2) Explain the features of HRM in brief

Answer)bullHuman Resource Management is concerned with managing people at work bull Human Resource Management is concerned with employees which bring people and organisations together so that the goals of each are met bullHuman resource management considered every employees as an individual and also promote their satisfaction and growth bull Human resource management is inherent in all organisations and at all levelsbullManagement of human resources are ongoing on never ending process which requires a constant alertness and Awareness of human relations

3) ldquoHR function is said to be pervasiverdquowhy

Answer) Human resource management is required in all organisations whether it is private or government organisations armed forces sports organisations etc It permeatsall the functional areas like production marketing finance research etc This from this feature of human resource management it can be said that it is pervasive in nature

Economics Demand Q1DEFINITION OF DEMANDIn economics demand is the quantity of a good that consumers are willing and able to purchase at various prices during a given period of timeQ2DEMAND CURVEIn economics a demand curve is a graph depicting the relationship between the price of a certain commodity and the quantity of that commodity that is demanded at that pricQ3LAW OF DEMANDIn microeconomics the law of demand states that conditional on all else being equal as the price of a good increases quantity demanded decreases conversely as the price of a good decreases quantity demanded increasesQ4ASSUMPTION of LAW OF DEMAND(i)No change in price of related commodities(ii) No change in income of the consumer(iii) No change in taste and preferences customs habit and fashion of the consumer( No expectation regarding future change in priceQ5MARKET DEMAND SCHEDULEIn economics a market demand schedule is a tabulation of the quantity of a good that all consumers in a market will purchase at a

given price At any given price the corresponding value on the demand schedule is the sum of all consumersrsquo quantities demanded at that priceQ6INDIVIDUAL DEMAND SCHEDULEIndividual demand schedule refers to a tabular statement showing various quantities of a commodity that a consumer is willing to buy at various levels of price during a given period of timeQ7 FACTORS AFFECTING INDIVIDUAL DEMAND FOR A COMMODITY

The factors that influence a consumerrsquos decision to purchase a commodity are also known as determinants of demand The following factors affect the individual demand for a commodity1 price of the commodity2 price of related goods3 income of buyer of the commodity4 tastes and preferences of the buyer1 Price of the CommodityYou must have observed that when price of a commodity falls you tend to buy more of it and when its price rises you tend to buy less of it when all other factors remain constant (lsquoother things remaining the samersquo) In other words other things remaining the same there is an inverse relationship between the price of a commodity and its quantity demanded by its buyers This statement is in accordance with law of demand which you will study in the later part of this lesson Price of a commodity and its quantity demanded by its buyers are inversely related only when lsquoother things remain the samersquo So lsquoother things remaining the samersquo is an assumption when we study the effect of changes in the price of a commodity on its quantity demanded2 Price of Related goodsA consumer may demand a particular good But while buying that good heshe also asks the price of its related goods Related goods can be of two types-(i) Substitute goods(ii) Complementary goods While purchasing a good prices of its substitutes and complements do affect its quantity purchased(i) Price of Substitute Goods Substitute goods are those goods which can easily be used in place of one another for satisfaction of a particular want like tea and coffee An increase in price of substitute good leads to an increase in demand for the given commodity and a decrease in price of substitute good leads to a decrease in demand for the given commodity It means demand for a given commodity is directly affected by change in price of substitute goods For example if price of coffee increases the demand for tea will rise as tea will become relatively cheaper in comparison to coffee(ii) Price of Complementary goods Complementary goods are those goods which are used together to satisfy a particular want like car and petrol An increase in the price of complementary goods leads to a decrease in demand for the given commodity and a decrease in the price of complementary goods leads to an increase in demand for the given commodity For example if price of petrol falls then the demand for cars will increase as it will be relatively cheaper to use both the goods together So demand for a given commodity is inversely affected by change in price of complementary goods3 Income of the Buyer of CommodityDemand for a commodity is also affected by income of its buyer However the effect of change in income on demand depends on the nature of the commodity under consideration In case of some goods like full cream milk fine quality of rice (Basmati rice) etc demand for these commodities increases when income of the buyer increases and

demand for these commodities decreases when income of the buyer decreases Such goods whose demand increases with the increase in income of the buyer are called normal goods But there are some goods like coarse rice toned milk etc whose demand decreases when income of buyer increases and their demand increases when income of the buyer decreases Such goods whose demand decreases with the increase in income of the buyer are called inferior goods Suppose a consumer buys 10 Kgs of rice whose price is ` 25 per Kg He cannot afford to buy better quality of rice because the price of such rice is ` 50 per Kg The consumer is spending ` 250 per month on the purchase of rice Now if income of the consumer increases and he can afford ` 350 on purchase of 10 Kg of rice Now he can afford to buy some quantity of rice say 6 Kgs whose price is ` 25 per Kg and may buy 4 Kgs of rice whose price is ` 50 per Kg Thus he will buy 10 Kgs of rice by spending ` 350 per month Therefore we may conclude that demand for normal goods is directly related to the income of the buyer but demand for inferior goods is inversely related to the income of the buyer4 Tastes and Preferences of the BuyerThe demand for a commodity is also affected by the tastes and preferences of the buyers They include change in fashion customs habits etc Those commodities are preferred by the consumers which are in fashion So demand for those commodities rises which are in fashion On the other hand if a commodity goes out of the fashion its demand falls because no consumer will like to buy it(5) Number of Buyers in the Market(Population)Increase in population raises the market demand whereas decrease in population reduces the market demand for a commodity Not only the size of population but its composition like age (ratio of males females children and old people in population) also affects the demand for a commodity It is because of needs of children young old male and female population differs(6) Distribution of Income and WealthIf the distribution of income and wealth is more in favour of the rich demand for the commodities preferred by the rich such as comforts and luxuries is likely to be higher On the other hand if the distribution of income and wealth is more in favour of poor demand for commodities preferred by the poor such as necessities will be more(7) Season and Weather ConditionsThis is generally observed that the demand for woolens increases during winter whereas demand for ice creams and cold drinks increases during summer Similarly market demand for umbrellas rain coats increases during rainy seasonQ8 REASONS FOR OPERATION OF LAW OF DEMAND WHY DEMAND CURVE SLOPES DOWNWARDNow we will try to explain why does a consumer purchase more quantity of a commodity at a lower price and less of it at a higher price or why does the law of demand operate ie why does the demand curve slope downwards from left to right The main reasons for operation of law of demand are1 Law of Diminishing Marginal UtilityAs you have studied earlier law of diminishing marginal utility states that as we consume more and more units of a commodity the utility derived from each successive unit goes on decreasing The consumer will be ready to pay more for those units which provide him more utility and less for those which provide him less utility It implies that he will purchase more only when the price of the commodity falls2 Income Effect

When price of a commodity falls purchasing power or real income of the consumer increases which enables him to purchase more quantity of the commodity with the same money income Let us take an example Suppose you buy 4 ice creams when price of each ice cream is ` 25 If price of ice creams falls to ` 20 then with same money income you can buy 5 ice creams now3 Substitution EffectWhen price of a commodity falls it becomes comparatively cheaper as compared to its substitutes (although price of substitutes has not been changed) This will lead to rise in demand for the given commodity For example if coke and Pepsi both are sold at ` 10 each and price of coke falls Now coke has become relatively cheaper and will be substituted for Pepsi It will lead to rise in demand for coke4 Change in Number of BuyersWhen price of a commodity falls some old buyers may demand more of the commodity at the reduced price and some new buyers may also start buying this commodity who were not in a position to buy it earlier due to higher price This will lead to increase in number of buyers when price of the commodity falls As a result demand for the commodity rises when its price falls5 Diverse Uses of a CommoditySome commodities have diverse uses like milk It can be used for drinking for sweet preparation for ice cream preparation etc If price of milk rises its use may be restricted to important purpose only This will lead to reduction in demand for other less important uses When price of milk falls it can be put to other uses also leading to rise n demand for itQ9 EXCEPTIONS TO THE LAW OF DEMANDYou have studied in law of demand that a buyer is willing to buy more quantity of a commodity at a lower price and less of it at a higher price But in certain circumstances a rise in price may lead to rise in demand These circumstances are called Exceptions to the Law of Demand Some important exceptions are1 Giffen GoodsGiffen goods are special type of inferior goods in which negative income effect is stronger than negative substitution effect Giffen goods do not follow law of demand as their demand rises when their price rises Examples of Giffen goods are jowar and bajra etc2 Status Symbol GoodsSome goods are used by rich people as status symbols eg diamonds gold jewellary etc The higher the price the higher will be the demand for these goods When price of such goods falls these goods are no longer looked at as status symbol goods and tehrefore therir demand falls3 NecessitiesCommodities such as medicines salt wheat etc do not follow law of demandbecause we have to purchase them in minimum required quantity whatever their price may be4 Goods Expected to be ScarceWhen the buyers expect a scarcity of a particular good in near future they start buying more and more of that good even if their prices are rising For example during war famines etc people tend to buy more of some goods even at higher prices due to fear of their scarcity in near future

Political Science

Constitution of India-The

Preamble

The preamble-

Preamble-

The preamble is the most precious part of the constitution We the people of India having solemnly resolved to constitute India into a Sovereign Socialist Secular Democratic Republic and to secure to all its citizensA preamble is an introductory and expressionary statement in a document that explains the documents purpose and underlying philosophy When applied to the opening paragraphs of a statute it may recite historical facts pertinent to the subject of the statuteNature and purpose of the constitution-Purpose of the Constitution dictates permanent framework of the government to form a more perfect union to establish justice and ensure peace of thenationconstitution provide principles how the government can run itself following the rules and laws written in the constitution of each state keeps them balanced

Answer the following questions-

1 What is preambleA preamble is an introductory and expressionary statement in a document that explains the documents purpose and underlying philosophy2 What is the nature and

purpose of the constitutionConstitution dictatespermanent framework of the government to form a more perfect union to establish justice and ensure peace of the nation

Homework-Learn

Accounts Compatibilty mode

1MEANING OF PARTNERSHIPPartnership is a form of business organisation where two or more persons join hands to run a business They share the profits and losses according to the agreement amongst them According to the Indian Partnership Act 1932 ldquoPartnership is relation between persons who have agreed to share profits of a business carried on by all or any one of them acting for allrdquo For example one of your friends has passed class XII from National Institute of Open Schooling (NIOS) and wants to start a business Heshe approaches you to join in this venture Heshe wants you to contribute some money and participate in the business activities Both of you if join hands constitute a partnership2CHARACTERISTICS1048698 Agreement A partnership is formed by an agreement The agreement may be either oral or in writing It defines the relationship between the persons who agree to carry on business It may contain the terms of sharing profit and the capital to be invested by each partner etc The written agreement is known as partnership deed1048698 Number of persons There must be at least two persons to form a partnership

The maximum number of partners in a partnership firm can be 50 according toCompanies Act 20131048698 Business The Partnership is formed to carry on business with a purpose of earning profits The business should be lawful Thus if two or more persons agree to carry on unlawful activities it will not be termed as partnership1048698 Sharing Profits The partners agree to share profits in the agreed ratio In caseof loss all the partners have to bear it in the same agreed profit sharing ratio10486981048698Mutual Agency Every partner is an agent of the other partners Every partner can bind the firm and all other partners by hisher acts Each partner will be responsible and liable for the acts of all other partners10486981048698Unlimited liability The liability of each partner except that of a minor is unlimited Their liability extends to their personal assets also If the assets of the firm are insufficient to pay off its debts the partnersrsquo personal property can be used to satisfy the claim of the creditors of the partnership firm10486981048698Management All the partners have a right to mange the business However they may authorize one or more partners to manage the affairs of the business on their behalf10486981048698Transferability of Share No partner can transfer hisher share to any one including hisher family member without the consent of all other partners3PARTNERSHIP DEEDAgreement forms the basis of partnership The written form of the agreement is which a document of partnership is It contains terms and conditions regarding the conduct of the business It also explains relationship between the partners This document is called partnership deed Every firm can frame its own partnership deed in which the rights duties and liabilities of the partners are stated in detail It helps in settling the disputes arising among the partners during the general conduct of business 4CONTENTS OF PARTNERSHIP DEEDThe partnership deed generally contains the following (i) Name and address of the partnership firm(ii) Nature and objectives of the business(iii) Name and address of each partner(iv) Ratio in which profits is to be shared(v) Capital contribution by each partner(vi) Rate of Interest on capital if allowed(vii) Salary or any other remuneration to partners if allowed(viii) Rate of interest on loans and advances by a partner to the firm(ix) Drawings of partners and interest thereon if any(x) Method of valuation of goodwill and revaluation of assets and liabilities on the reconstitution of the partnership ie on the admission retirement or death of a partner(xi) Settlement of disputes by arbitration(xii) Settlement of accounts at the time of retirement or death of a partner5IN ABSENCE OF PARTNERSHIP DEEDThe partnership deed lays down the terms and conditions of partnership in regard to rights duties and obligations of the partners In the absence of partnership deed there may arise a controversy on certain issues like profit sharing ratio interest on

capital interest on drawings interest on loan and salary of the partners In such cases the provisions of the Indian Partnership Act becomes applicableSome of the Issues are(i) Distribution of Profit Partners are entitled to share profits equally(ii) Interest on Capital Interest on capital is not allowed(iii) Interest on Drawings No interest on drawing of the partners is to be charged(iv) Interest on Partnerrsquos Loan A Partner is allowed interest 6 per annum on the amount of loan given to the firm by himher(v) Salary and Commission to Partner A partner is not entitled to anysalary or commission or any other remuneration for managing the business

History TOPIC-TOWARDS INDEPENDENCE AND PARTITION THE LAST PHASE (1935-1947)

SUB TOPIC-IMPORTANT POLITICAL DEVELOPMENTS ndash GROWTH OF SOCIAL IDEAS

Socialism is a political social and economic philosophyLike in other parts of the world the Russian revolution of 1917 served as a great inspiration for revolutionaries in India who at that time were engaged in the struggle for liberation from British ruleSocialist ideas led to the formation of communist party of IndiaJAWAHARLAL NEHRU Among the early Congress leaders Jawaharlal Nehru was very much impressed and influenced by the Socialist ideas He also learnt about the Economic activities of the Soviet Union after the Bolshevic Revolution 1917 He made full use of them in IndiaThe election of Jawaharlal Nehru and Subhas Chandra Bose showed the Left wing tendency within CongressJawaharlal Nehru demanded economic freedom along with political freedom of the people in order to end the exploitation of masses

Nehrus working committee included three socialists leaders The Lucknow session was a landmark in the evolution of socialist ideas of the congressSUBHAS CHANDRA BOSE ndash Subhas Chandra Bose had socialist leaning Both Jawaharlal Nehru and Subhas Chandra Bose were known as leftist Congress men Later on National Congress divided into Leftist and rightist campCONGRESS SOCIALIST Within the Congress some leaders formed the Congress Socialist partyPattavi Sitaramyya Sardar Patel Rajendra Prasad had hostile attitude towards the Congress Socialist partyJawaharlals attitude was hesitant

1 QUESTION ndash Mention name of two Congress leaders who had socialist leaning

1ANSWER ndash Subhas Chandra Bose and Jawaharlal Nehru2QUESTION- In which session of the congress Jawaharlal elaborated his Socialist ideas2 ANSWER ndash Lucknow and Faizpur Session in December 1935 and 19363QUESTION ndash Why Congress was sharply divided into leftist and rightist camp 3ANSWER ndash Subhas Chandra Bosersquos attempt to seek re election for congress presidentship in 1939sharply divided the National Congress into Leftist and Rightist camp4 QUESTION ndash Who was MN Roy 4 ANSWER ndash Manabendra Roy first formed the Communist Party of India outside the country at Tashkent in 19205QUESTION ndash Who formed the Congress Socialist Party within the Congress5 ANSWER ndash Jaya Prakash Narayan Achyut Patwardhan Acharya Narendra Dev Ram Mohan Lohia Aruna Asaf Ali6QUESTION ndash When was the Congress Socialist Party formed What was its object6 ANSWER ndash 1934The Congress Socialist Party sought to work out socialist programme through the Congress They joined hands with the Congress and wanted to carry

Subhas Chandra Bose being expelled from the congress after the Tripuri rift he formed Forward BlockThere were basic differences between the Congress Socialists and the communistsTRADE UNION ACTIVITIES Maximum working class people lived in Bombay and Calcutta The working and living conditions of those workers were very miserable In this situation Shasipada Banerjee NM Lokhande protested against the oppression of the working class peopleThe first Trade Union Madras Labour Union was formed in 1918 by BP WadiaIndustrial strikes took place in Kanpur Calcutta Madras Jamshedpur and Ahmedabad AITUC was formed in Bombay in 1927 The growth of Trade union among the workers was slow because of the fear of the dismissal of the jobIn the mean time the Moderates as well as Communists left AITUC and formed separate organization

on National struggle with the help of workers and peasant class of the society7 QUESTION ndash What was the name of the party founded by Subhas Chandra Bose7 ANSWER- Forward Block8QUESTION ndash Who was Shasipada Banerjee8 ANSWER ndash Shasipada Banerjee was a radical Brahmo He founded a working menrsquos club to protest against exploitation of the British rulers towards the working class of India9 QUESTION ndash What was the weekly published by NM Lokhande9ANSWER- Dinabandhu10 QUESTION ndash Who founded Bombay Mill-Hands Association and in which year10 ANSWER- NM Lokhande in189011 QUESTION- Who was BP WadiaANSWER- BPWadia was the founder of Madras Labour Union in191812 QUESTION- What was the name of the first labour union of India12 ANSWER- Madras Labour Union13 QUESTION Who founded the Majur Mahajan 13 ANSWER GANDHIJI14 QUESTION What was the full form of AITUC When it was formed14 ANSWER All India Trade Union Congressin 192715QUESTION Who formed the Red Trade Union Congress and in which year15ANSWER The Communists formed the Red Trade Union Congress16 QUESTION What do you mean by Socialism16 ANSWER Socialism describes any political and economic theory that says the community rather than individuals should own and manage property and natural resources

Subject Eng Literature (The Tempest ndash William Shakespeare) Topic Act III Scene 3 Lines 1 to 52 (Line 52 ndash Brother my lord the Duke Stand to and do as we) Date 13th April 2020 (4th Period)

[Students should read the original play and also the paraphrase given in the school prescribed textbook]Summary Questions amp Answers

o Alonso Sebastian Antonio Gonzalo Adrian Francisco and others wandered about the island in search of Ferdinand and gets tired and hungry of the toil and at the same time gives up all hope of finding him

o Antonio and Sebastian are happy that Alonso is out of hope and decide to make another attempt on his life that night when being so tired they will be sleeping soundly

o Suddenly a solemn and strange music is heard in the air and several strange shapes enter bringing in a banquet These strange shapes then dance round it with gestures of salutation and then inviting the King to eat they depart

o Seeing this strange scene all are inclined to believe the tales told by travelers that there truly are ldquounicornsrdquo and ldquothe phoenixrsquo thronerdquo

1 ALONSO What harmony is this My good friends hark (L18-27)

GONZALO Marvellous sweet music

[Enter several strange shapes bringing in a banquet

they dance about it with gentle actions of salutation

and inviting the King and his companions to eat they depart]ALONSO Give us kind keepers heavens What were theseSEBASTIAN A living drollery Now I will believe

That there are unicorns that in Arabia

There is one tree the phoenixrsquo throne one phoenix

At this hour reigning thereANTONIO Ill believe both

And what does else want credit come to me

And Ill be sworn rsquotis true Travellers neer did lie

Though fools at home condemn rsquoem

(a) How did Prospero present an amazing spectacle before Alonso and his companions

Using his magic powers Prospero ordered strange shapes to lay a banquet before Alonso and his companions The shapes brought several dishes with tasty eatables in them They placed the dishes on a table before Alonso and his companions Then the strange shapes began to dance gracefully around the banquet While dancing they made gestures inviting them to eat the food Then suddenly the shapes disappeared(b) Who were the guests at the strange banquet Describe the lsquoliving drolleryrsquo

Alonso Sebastian Antonio Gonzalo Adrian and Francisco were the guests at the strange banquet

The term ldquoliving drolleryrdquo refers to live entertainment show In this context when Alonso the King of Naples Sebastian his brother Antonio the treacherous brother of Prospero Gonzalo the kind and loyal councillor to the King Adrian and Francisco came to the island they were hungry and weary in their spirits They heard a solemn and strange music They were shocked to see several strange shapes bringing in a banquet and these shapes danced about it with gentle action of salutation inviting the King and his companions to eat After this Sebastian described this show as lsquoliving drolleryrsquo(c) What is lsquophoenixrsquo What are lsquoUnicornsrdquo

The term lsquophoenixrsquo refers to a mythical Arabian bird which lived alone and perched on a solitary tree After one hundred years it expired in flames and rose again from its own ashes

lsquoUnicornsrsquo refers to the mythological four-footed beasts having horns in the centre of their foreheads When the horns are ground into powder the powder was believed to be

an aphrodisiac(d) How does Sebastian explain the puppet show OR Why does the speaker now believe in unicorns and phoenix

Sebastian finds several strange shapes bringing in the banquet They invite the king and his party for dinner and soon depart He tells that if such a strange sight can be a reality there is nothing incredible in the world and from the present moment he will believe anything He says that it is a strange dumb show enacted not by puppets but by living beings It is stranger than a travellerrsquos tale Seeing such a thing

before his own eyes he will no longer disbelieve the story about unicorns and phoenix(e) How do the other characters present respond to this living drollery

At the sight of the lsquoliving drolleryrsquo like Sebastian Gonzalo and Antonio too acted strangely Antonio told that he too now believes in unicorns and phoenix and anything else that seems to be incredible He too now believes in travellersrsquo tales Gonzalo told that if he would report those happenings in Naples nobody will believe him He considers that those gentle shapes were gentler in manner in comparison to the living beings Alonso was at first sight suspicious and told them that those strange shapes conveyed their meaning in expressive gestures when they seemed to lack speech by their movements and sounds Francisco was amazed at their mysterious disappearance

2 ALONSO Not I

(Line 43-52)GONZALO Faith sir you need not fear When we

were boysWho would believe that there were mountaineers

Dewlapped like bulls whose throats had hanging at rsquoem

Wallets of flesh Or that there were such men

Whose heads stood in their breasts Which now we find

Each putter-out of five for one will bring us

Good warrant ofALONSO I will stand to and feed

Although my lastmdashno matter since I feel

The best is past Brother my lord the Duke

Stand to and do as we

(a) How does Alonso respond at the spectacle of the shapes which were sent to them at the instruction of Prospero

After seeing the strange sight of appearing and disappearing of the shapes sent by Prospero to arrange a banquet for them Alonso says that his surprise at having seen those creatures is infinite and he is fully justified in feeling so much surprise He thinks that their shapes their gestures and the sounds they made were indeed amazing Although they do not possess the gift of speech yet they were able to convey their

thoughts by means of their gestures only

(b) What does Prospero say about the views expressed by Alonso regarding the shapes What does Francisco think about the shapesAfter hearing Alonsorsquos views about the shapes Prospero says that this manrsquos praise of the spirits is rather hasty He means to say that Alonso has shown great haste in reaching the conclusion about the shapes Francisco is amazed to see that those shapes disappeared in a mysterious way(c) What does Sebastian ask Alonso to doSebastian tells Alonso that the shapes having disappeared should not matter to them because they have left the eatables behind He asks Alonso to enjoy eating as they are extremely hungry but the king does not accept his offer of enjoying the dishes(d) How does Gonzalo try to dispel Alonsorsquos fear of those strange shapes What kind of references does he give to AlonsoGonzalo says that those who have travelled abroad have reported seeing even stranger sights than these shapes that Alonso and his companions have beheld Hence there is no reason to feel afraid of these shapes Gonzalo further adds that in his younger days he had heard strange stories from travelers and Alonso might have heard similar stories For instance it was said that there existed a certain race of

human beings who had huge lumps of flesh hanging at their throats and who therefore resembled bulls Then Gonzalo tells about a race of human beings whose heads were located at their breasts Gonzalo says that such stories were not believed by most people in those days but now-a-days these stories have become common(e) Explain the following lsquoEach putter-out of five for onersquoEnglish travellers often insured their trips with London brokers Those that went on foreign travels those days used to deposit a certain amount with some firm or company in London before their departure If the travelers failed to return the money was forfeited by the company with which it had been deposited But this money was repaid five-fold if the travelers returned safe and sound In this way a traveler stood a great chance of recovering the entire cost of his

travels(f) Give the explanatory meanings of the following expressions in the context of the above extract (i) Dewlapped (ii) Wallets of flesh

(iii) Putter-out(i) Dewlapped having big lumps of flesh at the necks(ii) Wallets of flesh large masses of flesh looking like bags(iii) Putter-out to invest money before commencing the travel

  • General methods of preparation of hydrogen
  • Chapter Dimensional Analysis (Summary)
    • Properties of Charge
Page 10:   · Web viewSubject. Topic. Summary. Execution. Hindi. व्याकरण. शरीरके अंगो के नाम लिखिए. 1) आँख 2) नाक 3

life in rich monasteries Hence corruption crept in5 Adoption of Sanskrit ndash when Buddhist scriptures began to be written in Sanskrit in place of peoples language like Pali or Prakrit people started drifting away from Buddhism6 The Turkish Invasion ndash As Muslim conquerors invaded India immensely wealthy Buddhist Monasteries and temples were looted and destroyed and Buddhists were persecuted and killed

4 During which period decline of Buddhism began The Gupta Period

ENGLISH 2 The great train journey- Ruskin Bond

The great journey by Ruskin Bond is a story about Suraj who loved trains and wanted to go to places One day while wandering along the railway tracks he enters into a carriage compartment The train suddenly starts moving with him in the compartment and after a journey returns back to the same place from where it had begun The story is about his experience during that journey

4 Answer the following questionsf Who else is in the carriageA ragged hippy with a dirty beard face was in the carriageg Where does Suraj say that he would like to go toSuraj said that he would like to go to England and China and Africa and Greenland He wanted to go all over the worldh What warning does the man give to SurajThe man said Suraj to keep out of sight so that he doesnrsquot get caught by the ticket collectorsiWhen Suraj thinks about his parents for the first time what does he imagines that they will thinkSuraj thought that if he failed to come home that night his parents would think that he had run away or been kidnapped or been involved in an accidentJ What presents does Suraj imagine that he will bring back for his friendSuraj imagines that he would bring an African lion or a transistor- radio for his friend

CHEMISTRY

Chapter 2 ndashElement and Compound

ATOMAn atom can be defined as the smallest constituent particle of an element which showcases independent existence Example Ne OMOLECULEA molecule can be defined as the combinations of two or more atoms which are held together by chemical bonds A molecule is the smallest portion of a substance which showcases all the properties of the substance On breaking down a molecule further we see properties of the constituent elements Example HCl NaCl O2

Answer the following Q3) What is a moleculeAns - A molecule can be defined as the combinations of two or more atoms which are held together by chemical bonds A molecule is the smallest portion of a substance which showcases all the properties of the substance On breaking down a molecule further we see properties of the constituent elements Example HCl NaCl O2

Q4) Which can exist independently ndash atom or moleculesAns ndash Molecules can exist independently

PHYSICS Physical quantities

Guidelines for writing SI units correctly1 The units named after scientists are not written with a capital initial letter For example newton henry watt2 The symbols of the units named after scientist should be written by a capital letter For example N for newton H for henry W for watt3 Small letters are used as symbols for units not derived from a proper name For example m for metre kg for kilogram4 No full stop or other punctuation marks should be used within or at the end of symbols For example 50 m and not as 50 m5 The symbols of the units do not take plural form For example 10 kg not as 10 kgs6 When temperature is expressed in kelvin the degree sign is omitted For example 273 K not as 273o K (If expressed in Celsius scale degree sign is to be included For example 100o C and not 100 C)7 Use of solidus is recommended only for indicating a division of one letter unit symbol by another unit

Fill in the blanks

1) Length and mass are examples of fundamental physical quantities

2) The measurement of a physical quantity consists of two part magnitude and unit

3) A foot consist of 32 inches 4) The unit of temperature in the SI system is

Kelvin

Write true or false Correct the false statements

1) In ancient times cubit was used to measure the mass of an object FalseCorrect statement ndash In ancient times cubit was used to measure the length of an object

2) There are 7 fundamental physical quantities True

symbol Not more than one solidus is used For example m s-1 or m s J K mol or J K-1 mol-1 but not J K mol8 Some space is always to be left between the number and the symbol of the unit and also between the symbols for compound units such as force momentum etc For example it is not correct to write 23m The correct representation is 23 m kg m s-2 and not as kgms-29 Only accepted symbols should be used For example ampere is represented as A and not as amp or am second is represented as s and not as sec10 Numerical value of any physical quantity should be expressed in scientific notationFor an example density of mercury is 136 x 104 kg m-3 and not as 13600 kg m-3

3) Second is the unit of time in both the CGS and MKS systems True

4) The symbol used for a unit is always written in capital letters False Correct statement -The symbol used for a unit is normally written in small letters

Hindi 2nd language

वाकय निवचार भागवत निवचारो को परकट करन वा साथक एव वयवसथिtत शबद समह को वाकय कहत ह वाकय दो परकार क होत ह ndash

1 उददशय- वाकय म जिजसक बार म कछ बताया जाता ह उस उददशय कहत ह जस राधा एक नतकी ह2 निवधय- वाकय म उददशय क बार म बताया जाता ह उस निवधयक कहत ह जस- राधा एक नतकी ह रचना क आधार पर वाकय क तीन भद होत ह ndash१सर वाकय- राम बाजार गया २ सयकत वाकय- राम बाजार गया और वहा जाकर दोसत स मिमा३ मिमशर वाकय- यह वही tान ह जहा उनका बचपन बीता

helliphellipContinue to nextBengali 2nd language

লিZ সবরপ ও সবরলিZ

সবরলিZর লিয়ম - ১ অ-কোর লিকংবো আ-কোসরর পসর অ-কোর লিকংবো আ - কোর োকস উভয় লিমস আ ndashকোর য় এবং ওই আ ndash কোর পব13বস13 Pকত য়

২ ই - কোর লিকংবো ঈ - কোসরর পসর ই - কোর লিকংবো ঈ - কোর োকস উভয় লিমস ঈ - কোর য় এবং ওই ঈ - কোর পব13বস13 Pকত য়

৩ উ - কোর লিকংবো ঊ - কোসরর পসর উ - কোর লিকংবো ঊ - কোর োকস উভয় লিমস ঊ - কোর য় এবং ওই ঊ - কোর পব13বস13 Pকত য়

৪ অ - কোর লিকংবো আ ndash কোসরর পসর ই - কোর লিকংবো ঈ - কোর োকস উভয় লিমস এ - কোর য় এবং ওই এ - কোর পব13বস13 Pকত য়

১ অ + অ = আ ( gtো ) লিম + অচ = লিমোচ সব + অ3ী = সবো3ী অ + আ = আ ( gtো )পদম + আ = পদমো শভ + আলি = শভোলি আ + আ = আ ( gtো )4োয়ো + আবত = 4োয়োবত মো + আতমো = মোতমো আ + অ = আ ( gtো )লিবদযো + অংকোর = লিবদযোংকোর Pো + অ13 = Pো13 ২ ই + ই = ঈ ( gtী )অলিত + ইব = অতীব লিগলির + ইনদর = লিগরীনদর ই + ঈ = ঈ ( gtী )পলির + ইকষো = পরীকষো অলি3 + ঈশবর = অ3ীশবর ঈ + ঈ = ঈ ( gtী )মী + ঈশবর = মীশবর 3ী + ঈ = 3ী ঈ + ই = ঈ ( gtী )রী + ইনদর = রীনদর মী + ইনদর = মীনদর ৩ উ + উ = ঊ ( gt )মর + উদযো = মরদযো কট + উলিকত = কটলিকত উ + ঊ = ঊ ( gt )ঘ + ঊলিম13 = ঘলিম13 লিZ + ঊলিম13 = লিZলিম13 ঊ + ঊ = ঊ ( gt )রP + ঊলিম13 = রPলিম13

৪ অ + ই = এ ( লেgt )র + ইনদর = সরনদর লেPোগ + ইনদর = লেPোসগনদর অ + ঈ = এ ( লেgt )গ + ঈ = গস

র + ঈ = সর আ + ই = এ ( লেgt )Pো + ইষট = Pসষট 3ো + ইনদ = স3নদ আ + ঈ = এ ( লেgt )রমো + ঈ = রসম দবোরকো + ঈশবর = দবোরসকশবর

COMPUTER THE WORLD OF WINDOWS 10

DONE IN THE PREVIOUS CLASS PAGE NO-83A TICK THE CORRECT OPTION BACKGROUND DISPLAY AREA RESTORE THREE

MATHEMATICS Topic ndash NumbersChapter ndash Natural numbers and whole numbers

Study item Properties of whole numbers for subtraction1) Closure property When we do subtraction of two whole numbers we can not get a whole number in all time Example 8 ndash 3 = 5 a whole number 0 ndash 6 = -6 is not a whole numberTherefore the subtraction of two whole numbers is not satisfying closure property2) Commutative property If x and y are two whole numbers then x ndash y is not equal to y ndash xExample If x=16 and y = 7 then x ndash y = 16 ndash 7 = 9Again y ndash x = 7 ndash 16 = - 9 Therefore x ndash y not equal to y ndash x Therefore the subtraction of two whole numbers is not satisfy commutative3) Associative property If x y and z are three whole numbersThen x ndash ( y ndash z ) not equal to ( x ndash y ) ndash z Example If x = 20 y = 10 and z = 6Therefore x ndash (y ndash z ) = 20 ndash(10 ndash 6 ) = 20 ndash 4 = 16(X ndash y ) ndash z = (20 ndash 10) ndash 4 = 10 -4 =6Therefore x ndash(y ndash z) not equal to ( x ndash y) ndash zTherefore subtraction of whole numbers is not satisfying associativity4) Distributive property If x y and z are three whole numbersThen x (y ndash z ) = xy ndash xzAnd (y ndash z)x = yx ndash zxExample If x = 10 y = 6 and z = 4x(y ndash z ) = 10(6 ndash 4 ) = 10times6 ndash 10times4 = 60 ndash 40 = 20( 6 ndash 4 )times 10 = 6times10 ndash 4times10 = 60 ndash 40 = 20Therefore the subtraction of whole numbers is satisfying distributive property5) Existence of identity For any whole number x X ndash 0 = x but 0 ndash x = - x not equal to xThus for subtraction no identity number existsException 0 ndash 0 = 0 so 0 is its own identity for subtraction

Class VIISubject Topic Summary Execution

Hindi 2ndlang वचन जो सजञा शबद निकसी वसत या पराणी क एक या अनक होन का बोध कराया उनह वचन कहत ह जस डका- डकयह दो परकार की होती ह-

क) एकवचन-शबद क जिजस रप स उसक एक होन का बोध हो उस एक वचन कहत ह जस निकताब गमा आदिद

) बहवचन-शबद क जिजस रप स उसक आन ोन का पता च उस बहवचन कहत ह जस डक निकताब निततलियाआदिद

निनमनलिखित शबदो को एकवचन स बहवचन म बदोम- हमजानित- जानितयानारी- नारिरयामिमतर ndashमिमतरोपसतक -पसतकसडक-सडकबोत-बोतनाहर-नहररपए-रपया

Bengali বইndashবোংো োলিতয পলিরচয়

পোঠndash১৪ গলপ - অপর কলপো পর

লেক - লিবভলিতভষ বসনদযোপো3যোয়লেকndash রবীনদর পরবতf বোংো কোোলিতয 3োরোর উসgসPোগয োম পরকলিতসপরমী লিবভলিতভষ বসনদযোপো3যোয় তোর লেীসত লেPম বোসর বোসর লিফসর এসস4 গরোম বোংোর পরকলিতর কো লেতমলি এসস4 গরোমী মো লিচতরগলপndash অপর কলপো গলপোংটি লিবভলিতভষ বসনদযোপো3যোসয়র লিবযোত উপযো পসর পাোচোী লেসক গীত অপ অ13োৎ পসর পাোচোী তো অপর কলপোর লেকনদরীয় চলিরতর এই অংস আমরো পোই বোক অপসক বোক অপ কলপো লিবোী লে দসরর অ গো4 লেদস মোসয়র মস লেোো রপকোর রোসয পোলি2 লেদয় দপরসবো মোসয়র মসর কসর কোীদোী মোভোরত এর করসকষতর Pসjর ব13ো শস তোর মোবীর কস13র পরলিত ব2 মমতো য় আবোরপালিসত বলি13ত Pসjর অমোপত অং লে লিসই মোপত কসর বোলি2র লিপ4স বাো বোগোস লিকংবো উঠোসর লিশমসর কলপো লিবো এোস পরকো লেপসয়স4

১ অপর কলপো গসলপর লেক লেক তোর মপসক13 লেসো২ অপর কলপো গলপটি লেকোো লেসক গীত গলপটির ম ভোব লেসো

GEOGRAPHY CHAPTER 7EUROPE

CHAPTER COMPLETE EXERCISEFill in the blanks1 Europe is a continent that comprises the western part of Eurasia2 Eurasia and Africa are connected into one large land mass known as Afroeurasia3 The Strait of Gibraltar separates Europe and Africa4 Europe is surrounded by the Arctic Ocean to the north5 The British Isles includes the island countries of Great Britain and Ireland

Name the following 1 Connects Africa to Eurasia - Isthmus of Suez2 Largest country in the world in terms of area ndash Russia3 A term used collectively for the five countries in northern Europe ndash Nordic Countries4 The capital of Montenegro - Podgorica5 the largest fjord in Norway ndash Sognefjord

Match the following Column 1 Column 2a Albania iii Tiranab Belgium i Brusselsc Denmark v Copenhagend Finland ii Helsinkie Hungary iv Budapest

CHEMISTRY Chapter 2 ndashElement and Compound

Atom - An atom is the basic unit of an element or the smallest particle of an element non capable of independent existence Atom is built up of three sub atomic particles electron proton and neutron

Nucleus-It is the centre of an atom In the centre of the atom contains proton (positively charged particles ) and neutrons ( particles carrying no charge )

Orbits- It surround the nucleus in which revolve electrons (negatively charged particles)

Answer the following

1) What are MetalloidsAns - Certain elements using properties of both metal and non-metals are called metalloids Example Silicon arsenic and antimony

2) What are Noble gasesAns - Certain elements are present in the air and are chemically inert or unreactive Such elements are called rare gases or noble gases Example helium neon argon and Krypton

English 2 Sentences based on meanings

Kinds of sentences

Assertive or declarative to convey information or simply make a statement

Interrogative to ask different types of questions

Imperative to command or instruct someone or make a request

Exclamatory to express strong feelings and emotions

Exercise B1 Stop it ( Exclamatory)2 May you always be happy

together ( Exclamatory)3 He does not like sports

( Assertive)4 Please pass me the salt

( Imperative)5 How dare she talk to me like

that ( Exclamatory)6 May success bless your effort

( Exclamatory)7 Canrsquot you wait for sometime

(Interrogative)8 Did anybody tell you about it

( Interrogative)9 I saw her waiting for the bus

( Assertive)10 Could you please take a

message for me ( Interrogative)

Homework Ex ABiology Chp -2

Classification of Plants

Today we discuss about usefulness of bacteria We also discuss what the harmful effects of bacteria are

89 How bacteria are useful for usbull Bacteria is helpful in many ways forhuman being i) Production of medicine - antibiotics vaccine etcii) Formation of curd by lactobacillusiii)Nitrogen fixation in Leguminousplant by Rhizobiumiv) Increase soil fertility by absorbingatmospheric nitrogen and convert it into nitrates and nitritesv) Cleaning the environment by converting the complex substances into simple substancesvi) Tanning of leathervii) Retting of Fibersviii) Formation of compost by acting onanimal dung and agricultual cases1x) Biogas production by decomposingplant and animal wastex)Help In Nutrition by producing vitamiacutemBand kx1) Some bacteria are used to give specialflavour to tea coffee and coccaQ10- Name some diseases and there causativebacteriabull Diseasescausative bacteria1 CholeraVibrio cholerae2 Tuberculosis - Mycobacterium tuberculosis3 Diptheria -Corynebacteriumdiphtheriae4 Pneumonia - Streptococcus pneumoniae

Math Number system

Chapter Fraction

Study item Using lsquoofrsquoThe word lsquoofrsquo between any two fractions is to be used as multiplicationExample 57 of 56 = 57 times 56 = 5times8 = 40Study item Using BODMASThe word lsquoBODMASrsquo is the abbreviation formed by taking the initial letters of six operations(i)Bracket (ii) of (iii) Division (iv) Multiplication (v) Addition (vi) SubtractionAccording to BODMAS rule First of all the terms inside Bracket must be simplified then lsquoofrsquo lsquoDivisionrsquo lsquoMultiplicationrsquo lsquoAdditionrsquo lsquosubtractionrsquo

Study item Removal of Brackets

There are four Brackets of algebra in Mathematics In a complex expression four types of brackets are used Order of removing the brackets is first ----- then ( ) then finally [ ]

Class VIIISubject Topic Summary Execution

Chemistry Hydrogen General methods of preparation of hydrogen

By the action of dilute acids on metals

Calcium Reacts readily to form chloride salt and hydrogen

Ca + 2HCl rarr CaCl₂ + H₂uarr

Magnesium

Aluminium

Zinc

React readily to form salt and hydrogen

Mg + 2HCl rarr MgCl₂ + H₂uarr2Al + 6HCl rarr 2AlCl₃ + 3H₂uarrZn + 2HCl rarr ZnCl₂ + H₂uarr

Question 4 ) Give reasons for the following

(a) Hydrogen be used as a fuel

Solution

Hydrogen is used as a fuel because it has a high heat of combustion Some significant fuels are coal gas water gas and liquid hydrogen

(b) Though hydrogen is lighter than air it cannot be collected by downward displacement of air

Solution

Hydrogen is lighter than air so it is possible to collect the gas by downward displacement of air But it is not safe to do so since a mixture of hydrogen and air can lead to an explosion

(c) A pop sound produced when hydrogen is burnt

Solution

Impure hydrogen gas burns in air with a pop sound This is because of the presence of impurities in it

(d) Helium replaced hydrogen in weather observation balloons

Solution

It forms a mixture with air that can explode when there is a small leakage of hydrogen in a balloon So helium has replaced hydrogen

(e) Nitric acid not used for the preparation of hydrogen gas

Solution

(e) By the action of nitric acid on metals hydrogen cannot be produced because it also releases nitrous oxide and nitric oxide and oxides the hydrogen to form water

Biology Chp-2 Reproduction in plants

Today we discuss different methods of artificial propagation like cutting-rose sugercane Layering ndashguava lemon china rose etc Grafting- mango apple etcMicropropagation ndashorchid asparagus etcWe also discuss about advantages and disadvantages of vegetative propagation

Q7 Define the following terms i) Explant In tissue culture techniquea tiny piece of bud shoot or any other partof plant from where new tissue develop ii) Callus The cells of the tissue divide andgrow into a mass of undifferentiated cells from explant iii) Plantlet After few days callus differentiate into a small plant with roots and shootQ8 what are the advantages and limitations of tissue culture or micropropagation

Advantages i ) It produacuteces superior quality plantsii)It can be applied to interspecifie hybridsiii) It is useful to grow seedless plants bull Limitations i) It cannot be used for all plantsii)It is not easy to handleQ9 Write advantages of vegetative propagationi) It is a quick and easy method ofproducing new plantsii) This method need less time to matureiii) The new plants are exact copies of the parentiv) it is extremly useful for growing seedlessplants like banana grapes etc Q10 Write some disadvantages of vegetativepropagationi) Dišeases present in the parent plant gettransferred to all in new plantsii) Overcrowding of new plants causes competition for sunlight water and nutrients which affects growth of plantsplant

Physics Chapter 2 Physical Quatites and Measurements

Here We Will Do Some QuestionsRelated To Chapter 2

Select the correct alternative A block of wood of density 08gcm-3 has a volume of 60cm3 The mass of the block is

1 608 g

2 75 g

3 48 g

4 0013 g

Solution 348 g

The density of aluminium is 27g and that of brass The correct statement is

1 Equal masses of aluminium and brass have equal volumes

2 The mass of a certain volume of brass is more than the mass of an equal volume of aluminium

3 The volume of a certain mass of brass is more than the volume of an equal mass of aluminium

4 Equal volumes of aluminium and brass have equal masses

Solution 2 The mass of a certain volume of brass is more than the mass of an equal volume of aluminium

MATHEMATICS Ch 6Sets

Exercise 6(C)1 Find all the subset of each the following sets(i) A = 57 (iii) C = x xisin W x le 2(iv) p p is a letter in the word lsquopoorrsquo

Solution (i) All the subsets of A are ϕ 5 7 57

(iii) All the subsets of C are ϕ 0 1 2 01 02 12 012

(iv) All the subsets are ϕ p o r po or por

4 Given the universal set = -7-3-105689 find (i) A = x xlt2 (ii) B = x -4ltxlt6 Solution

(i) A = -7-3-10(ii) B = -3-105

5 Given the universal set = x xisin N and xlt20 find

(i) A = x x = 3p pisin N (iii) C = x x is divisible by 4 Solution

(i) 369121518 (iii) 481216

6 Find the proper subset of x x2-9x-10 = 0 Solution

ϕ 10 -1

Working x2-9x-10 = 0 rArr x2-(10-1)x-10 = 0

rArr x2-10x+x-10 = 0 rArrx(x-10)+1(x-10) = 0

rArr (x+1) (x-10) = 0

11 Let M = letters of the word REAL and N = letters of the word LARE Write sets M and N in roster form and then state whether (i) M sube N is true (ii) N sube M is true (iii) M = N is true

Solution M = real and N = lareSo (i) Yes (ii) Yes (iii) Yes

English 2 Twelfth Night ndash Shakespeare

A noble man named Orsino in the kingdom of Illyria is deeply in love with a lady called lady Olivia She is in mourning for her dead brother so she will not even think about marriage At this time a sea storm causes a terrible shipwreck and a young lady called Viola is swept onto the shore She thinks that her twin brother Sebastian is drowned A sea captain tells her about Orsino and his love for Olivia Viola wishes to work in Oliviarsquos home but feels she will not be employed So she dresses as a man calls herself Cesario and gets work at the house of OrsinoViola (now Ceasario) is much liked by Orsino and becomes his page She falls in love with Orsino Orsino sends Ceasario to deliver messages to Olivia Olivia herself falls for the beautiful young Ceasario believing Viola to be a man

2 Answer the following questionsa Why does Orsino ask the musicians to play onOrsino asks the musicians to play on because music feeds his desire He calls upon the musicians to play music so that his hunger for love could be replenished with an excess of musicb What does Valentine tell about OliviaWe learn from Valentine that Olivia is in mourning for her brother she wears a veil and has vowed that no one will see her face for another seven yearsand she refuses to marry anyone until thenc From the exchange between Orsino and Valentine what do you think their relationship isValentine is one of orsinod attendants He was sent to Olivia as a messenger of love but was not allowed to speak to here Who is Olivia mourning for and whyOlivia is mourning for her dead brother

Homework Q fHistory and Civics

Growth of Nationalism

Important dates to remember1769-Napoleon born on 15thAugust1789-Fall of Bastille on 14th July and the beginning of the French revolution declaration of the rights of Man on 26thAugust1793-King Louis XVI executed on January 211764-The Sugar Act passed1765-The Stamp act passed1774-The first congress of Philadelphia1776-The declaration of American Independence of on 4th July1777-Defeat of the British at Saratoga1781-Surrender of lord Cornwallis at Yorktown1783-The treaty of Versailles1804-Napoleon becomes the emperor1813-Battle of Leipzig or Battle of nations in which Napoleon was defeated by the Allies1815-Battle of Waterloo June 18 in which Napoleon was defeated and captured1821-Death of Napoleon in StHelena1860-Abraham Lincoln elected President of the USA1861-The civil war began 1864-Abraham Lincoln elected President of the USA for the second time1865-Slavery abolished in the US

Name the following- The queen of Louis XVI

Marie Antoinette The three philosophers of France

VoltaireMontesquieuJean Jacques Rousseau

The British general whose surrender brought the war in America to an endLord Cornwallis

The first president of the USAGeorge Washington (1732-1799)

The first southern state to secede from the unionSouth Carolina

The author of the book lsquoUncle Toms CabinHarriet Beecher Stowe

Homework-Learn

Class IXSubject Topic Summary Execution

Economics

Types of economies Today I am going to share you the concept of economic growth and economic development Few questions will be given from the previous study material dated 942020

Meaning of economic growthAnswer) The term economic growth generally means anincrease in national income or per capita output or income over time It indicates towards quantitative growth of a country

Meaning of economic developmentAnswer) Economic development is defined

as a process whereby the real per capita income of a country increases over time along with fall in poverty ratio unemployment and income inequality etc

Distinguish between economic growth and economic development

Basis Economic growth

Economic development

Scope It has narrow scope as it refer only to rise in per capita income

It has wide concept since it includes qualitative changes as well

Concerned matter

It is concerned with the rise in income

It is concerned with not only rise in income but also reduction of poverty income inequality and unemployment

Focus Economic growth does not focus on economic development

Economic development focus on economic growth plus qualitative changes

Distinguish between capitalist economy and socialist economy

Ownership

Motive

Tool

Means of production are owned and managed by private people

Self interest and profit earning is the main motive

Price mechanism is a main tool to solve the economic problems

Means of production are owned and managed by the government

Social welfare is the main motive

Economic planning by the government is the main tool to solve the economic problem

Competition

Distribution of income

There exist large competition among buyers and sellers

There is existence of large inequalities of income

There is no such competition

There exist less inequalities of income

Math Topic ndash AlgebraChapter -Factorisation

Study item Factorising by taking out common factorSome solved sums from exercise 41

1) (i) 8xy3 + 12x2y2

= HCF of 8xy3 and 12x2y2 is 4xy2

= 4xy2(2y + 3x )

4) (ii) 28p2q2r ndash 42pq2r2

= HCF of 28p2q2r and 42pq2r2 is 14pq2r = 14pq2r (2p - 3r )5) (ii) 14mn + 22m - 62p=HCF of 14mn 22m and 62p is 2= 2(7mn + 11m - 31p)7) (ii) 3a(x2 + y2) + 6b (x2 + y2) = HCF of 3a(x2 + y2) and 6b(x2 + y2 ) is (x2 + y2)= ( x2+ y2 )(3a + 6b )9) (ii) x(x2 + y2 ndash z2 ) + y(-x2ndashy2 + z2 ) ndash z(x2+ y2 ndash z2 )= x(x2 + y2 -z2) ndash y-(x2 + y2 -z2) -z(x2 + y2 ndash z2)=x(x2 + y2-z2) -y( x2 + y2-z2) ndash z (x2 + y2 -z2)= (x2+ y2 ndash z2)(x ndash y ndash z )

Commercial Studies

Introduction to Accounting and Book-keeping

Today I am going to share you the meaning of Accounting and Book-keeping and its related terms bullAccounting bullBook Keeping bullAccountsbullTypes Of Accounts bullAccounting Cycle

bull Meaning of accounting

Ans )Accounting is the art and science of recording classifying and summarising monetary transactions

bull Meaning of Book-keeping

Ans) Bookkeeping is the art of recording business transactions with the view of having a permanent record of them and showing their effect on wealth

bull Meaning of account

Ans) The term account means a record of business transactions concern a particular person of firm asset or income or expense It is a summarised record of all transactions which take place in an accounting year

bull Types of accountsPersonal accounts ndash Personal accounts relating

to person and Organisation are known

as personal accounts Example Ramrsquos Account ABC amp Co Account etc

Real account - The accounts related to tangible and intangible assets are called real accountsExample Cash Account Furniture Account etc

Nominal account- Accounts related to expenses losses incomes and gains are known as nominal accountsExample Wages Account Salary Account Discount Account etc

bull Accounting cycle Accounting cycle refers to a complete sequence of accounting activities It begins with recording of transactions and ends with the preparation of a balance sheet

English 1 Transformation of sentences

Sentences A sentence is a group of words which makes complete sense

a Assertive sentencesb Imperative sentencesc Interrogative

sentencesd Exclamatory sentences

Sentences can be changed from one grammatical form to another without changing the meaning of the sentence This is known as transformation of sentences

Exercise 6Rewrite the following sentences according to the instructions given below without changing their meanings

1 As soon as he saw the beer he jumped into the river ( Begin No sooner)

2 None but brave deserve the fair (Begin the bravehellip)

3 This box is too heavy for me to lift ( Use so hellip That instead of too)

4 No one other than a king can live like James Luxurious ( Begin only James)

5 Oh for the wings of a dove (Begin I wishhellip)

BENGALI(2ND LANGUAGE)

ldquo বঙগভমির পরমি ldquo াইকেল ধসদন দতত

পব13পোসঠ আসোলিচত ৩ পরবোস দৈদসবর বস ীবতোরো Pলিদ স এ লেদ -আকো সত-োলি লেদ তোস - ক) বকতো লেক কোর লেো লেকো কলিবতোর অং ) কোর পরলিত বকতোর এই উলিকত গ) এ লেদ আকো সত বসত কী বলিঝসয়স4 ীবতোরো বসত কী লেবোঝ ঘ ) আসোচয অংসর তোৎপP13 কী

উ -ক ) বকতো স কলিব মোইসক ম3দ দতত

Types of AccountPersonal AccountReal AccountNominal AccountBalance Sheet (opening)

কলিব মোইসক ম3দ দসততর রলিচত বঙগভলিমর পরলিত কলিবতোর অং ) কলিব বঙগী অ13োৎ লেদমোতোর পরলিত কলিবর এই উলিকত গ ) এ লেদ আকো বসত কলিবর মোব লেদী রপ আকো লেক লেবোঝোসো সয়স4 আকো লেসক লেPম তোরো স পসর লেতমলি ীব লেদ রপ আকো লেসক পরো রপ তোরো স পরসত পোসর এই মভোবোর কোই কলিব বসস4 ঘ ) পরবো Pোতরোয় Pলিদ কলিবর লেদ আকো লেসক ীব তোরো রপ পরো স পসর তোসত কলিব লিবনদমোতর দঃলিত কোর মতয লিবসর সবোভোলিবক পলিরলিত এবং মোষ মরী তোই পরবোস Pলিদ তা োর মতয য় তবও কলিব লিবচলিত সব ো কোর পলিবীসত লেকউ অমর য় লিক4ই অকষয় য় দীর লেPম লিচরপরবোমো লেতমলি মোসষর ীবও চমোতোই ীব - সতবধতোই মতয ীব দীসত মোষ লিতয পরবোমো তবও লেPব মোষ আপ কতকসম13র মো3যসম মোসষর মস লিসসদর সথো কসর লিসত পোসর তোরো লিচরভোসবর সয় মোসষর মস লিবরো কসর তোসদর মস3য লেকউ পGভসত লিবী সয় গোসও মোসষর মস তোরো লিতযপলিত লিতযবলিনদত

Hindi 2ndlang

काकीी(लिसयारामशरणगपत)

इस कहानी म क न यह बतान का परयास निकया ह निक बचच अपनी मा स निकतना परम करत ह शयाम अबोध बाक ह वह अपनी मा क मरन क बाद उसन अपनी मा क लिए बहत रोया बाद म उस पता चा निक उसकी मा राम क घर ची गई ह आकाश म उडती हई पतग दकर उस हष हआ निक पतग क दवारा वह अपनी मा को नीच उतारगा इसक लिए वह अपनी निपता की जब स दो बार सवा रपया निनकाकर पतग और दो मोटी सी मन वाी अपन भाई स काकी एक कागज पर लिवा कर पतग म लिशव का दिदयानिनकाकर पतग और दो मोटी सी मन वाी अपन भाई स काकी एक कागज पर लिवा कर पतग म लिचपका दिदयाभोा और शयाम कोठरी म रससी बाधनी रह थ तभी उसक निपता करोध म आकर उन स पछ निक कया उनकी जब स रपया निनकाा हभोा डर क मार बताया निक शयाम इस पतग क दवारा अपनी काकी को राम क यहा स उतारना चाहता हनिवशशवर(शयाम क निपता)न फटी पतग उठाकर दी तो उस पर काकी लिा थावह हत बजि होकर वही ड रह गएउनहोन सोचा निक मन अपन पतर को मारा जोनिक अनजान और निनदष थावह अपनी मा कोनिकतना पयार करता ह

helliphellipContinue to next

Computer Application

Java Programming Prog 1Write a java program to input two numbers from user and display the sum or product of them as per user choice Use switch case statementSolve public class sum_product public static void main(String args[]) Scanner sc=new Scanner(Systemin) int abc Systemoutprintln(ldquoEnter two numbersrdquo) a=scnextInt() b=scnextInt() Systemoutprintln(ldquoPress 1 for sum or 2 for productrdquo)

c=scnextInt() switch(c) case 1 Systemoutprintln(ldquoThe sum will be =rdquo+(a+b)) break case 2 Systemoutprintln(ldquoThe product will be =rdquo+(ab)) break default Systemoutprintln(ldquoWrong Inputrdquo) Home Work - Practice in your computer using bluej

Subject Eng Literature (The Merchant of Venice ndash William Shakespeare)Topic Act I Scene 2 Lines 92 to 126 (End of scene) Date 13th April 2020 (5th Period)

[Students should read the original play and also the paraphrase given in the school prescribed textbook]Summary Questions amp Answers

o After Portia has expressed her opinion about the suitors Nerissa informs that she need not bother about any one of them as they have decided to quit Belmont at the earliest opportunity because they do not believe in trying their luck by the caskets which is the only way of winning Portia

o Nerissa then enquires of Portiarsquos opinion about Bassanio who once visited her in the company of the Marquis of Montferrat and says that she had never come across such an ideal love deserving the fairest lady for his bride

o Portia seems to remember Bassanio quite correctly and says that she agrees with Nerissa At this moment a servant informs Portia that the Prince of Morocco has arrived to try his luck by the caskets

o Portia tells Nerissa that if she could welcome this new suitor as gladly as she says farewell to the previous ones she would be glad of his arrival However if he happens to have the virtues of a saint but the black complexion of a devil she would prefer to have him for religious consolation rather than as a husband

(1) NERISSA You need not fear lady (Line 97-103)

the having any of these lords they have acquainted me with their determinations

which is indeed to return to their home and to

trouble you with no more suit unless you may be wonby some other sort than your fathers imposition depending on the caskets

PORTIA If I live to be as old as Sibylla I will die as chaste asDiana unless I be obtained by the manner of my fatherswill I am glad this parcel of wooers are so reasonablefor there is not one among them but I dote on his veryabsence and I pray God grant them a fair departure

(a) Elucidate the idea expressed in the first speech of the above dialogue

In the first speech Nerissa assures Portia that she need not have any fear of being compelled to marry anyone of the suitors who had lately come to Belmont She informs her that they have all decided to return to their respective countries(b) Illuminate the meaning of the phrase ldquoyour fatherrsquos imposition depending on the casketsrdquo

Nerissa means that the suitors of Portia do not find the conditions imposed by the will of her father to their liking They are too hard for them These conditions are that in the event of a suitor failing to choose the right casket (i) he should never disclose to anybody which casket he chose (ii) he can never marry and (iii) he should take his departure immediately(c) Explain the meaning of the term lsquoSibyllarsquo

lsquoSibyllarsquo is the name given by Romans and Greeks to a prophetess inspired by some deity usually the sun-god Apollo She had a very long life The god Apollo granted her as many years of life as she could hold grains of sand in her hand(d) Elucidate the meaning of the term lsquoDianarsquo

lsquoDianarsquo is the goddess of hunting She is also regarded as a symbol of virginity because she never fell in love and never

married(e) Explain the meaning of the first two lines of Portiarsquos speech

Portia says that even if she is to live for centuries like Sibylla she would not marry except in accordance to her fatherrsquos will She asserts that she would not mind remaining unmarried and untouched by a man like Diana the virgin the goddess of hunting unless a man is able to win her by passing the test laid down by her father

Class XSubject Topic Summary Execution

Hindi 2nd

Langबड घर की बटी( मशी परमचद)

lsquoबड घर की बीटीrsquo कहानी का उददशय मधयम वग की घर समसया को सझा कर सगदिठत परिरवार म मिम जकर परम स रहन का सदश दना ह घर म शानित tानिपत करन की जिजममदारी नारी की होती ह यदिद नारी समझदार ह उसम धय और परिरवार क परनित परम ह तो कोई भी घटना परिरवार को निवघदिटत नही कर सकती या कहानी परिरवार को सगदिठत करत हए परम सौहाद स एक रदसर की भावनाओ को समझ करउनका सहयोग करत हए जीवन यापन करन की पररणा दती ह मशीपरमचदर जी न इस कहानी म सयकत परिरवार का परनितनिनमिधतव निकया ह यह कहानी बनी माधव सिसह जो गौरी पर क जमीदार क उनक दो पतरो की हशरी कठ ा निबहारीशरीकात का निववाह एकजमीदार घरान की पतरी आनदी स हआ थाआनदी न द को ससरा क वातावरण म ढालिया थाएक दिदन आनदी का अपन दवर ा निबहारी स झगडा हो जाता ह दोनो भाई एक रदसर स अग होन की कोलिशश करत हसभी बह आनदी न अपन मधर वयवहार स ा निबहारी को घर छोडकर जान स रोक लिया| इस पर बनी माधव सिसह न कहा निक बड घर की बटी ऐसी ही होती ह जो निबगडा काम बना ती ह अतः शीषक साथक ह बड घर की बटी आनदी ह

helliphelliphelliphellipContinue to nextBiology Topic ndash Chp-1

CellWelcome to new session 2020-21Today we will start with Chpter 1 cell CELL

Protoplasm+Cellmembrane Or Cell wall

Cytoplasm+Neucleus

Cytoplasmic+ CytoplasmicOrganelles Inclutions(mitochondria (food Golgi bodies pigments)Ribosome)

What is cellbull Cell is the structural and functional unit of living organismbull According to number of cells organisms areUnicellular - Amoeba bacteria Multicellular - Rose Mango Tiger HumanSmallest cell -bacteria Longest cell - Nerve cellLargest cell - Ostrich egg cellCells are of different size and shapes according to their functionsQ2Write chief functions of following cellorganelles

Q3What is tonoplastVacuoles covered by a covering called tonoplast

Bengali(2Nd

Language)

ফ ফটক ো ফটক (কলিবতো ) ভোষ মসোপো3 gtPোয়

একটি লেমসয়র ীবস লেপরম লিকভোসব ফসট ওসঠ তো লেদলিসয়স4 কলিব লেপরম Pই য় লেই ময়ই বনত কোস পলিরত য় ফ লেফোটো বো োসফোটো লেটো ব2 কো য় লেমসয়সদর ব gtয13 লেপরসমর 4লিব ফসট উসঠস4 এই কলিবতোয় লেপরম মোষসক মত gtযর মস লেফস লিদসয় পরকষস বাোচোসোর gtয োত বো2োয় কলিবতোয় লেমসয়টির পসব13র দঃসর কো বো সও লেমসয়টি লেই পসর পলিক সত চোয়ো োরী ীবসর কোস4 পরম লেPৌবস লেপরমসক পোবোর পরব ইচছো োকসও তো পসর লেলিতবোচকতোয় পলিরত য় কলিব ভোষ মসোপো3 যোয় লেP ক লেপরসমর

কলিবতোয় ব gtযবহত লিবসষ লিক4 সvর অ13 লেদওয়ো ১) রসবোো= লেP লিবলিভনন রকম ডোকসত পোসর২) ো= পোর ৩) ঠলি = লেচোসর বZ৪)আই বস2ো=অলিববোলিত৫)শইসয় = োলিয়ত কসর৬)োতপাোচ= লিবলিভনন পরকোর৭)দ2োম = v কসর বZ কসর লেদওয়ো৮)লেরলিং =লেোোর দৈতরী লেব2ো৯) বনত= একঋত১০) পাোর = বসকরো2

Organelles Functions

1 Endoplasmic reticulum

2 Mitochondria

3Golgibodies

4 Ribosome5Lysosome

6Plastids

7 Centrosome

i) Supportive framework for the cellii) Synthesis and transpost of proteinsRelease of energy in the form of ATPi) Synthesis and secretion of enzymes hormoneii) Formation of vacuoles lysosomei) Protein Synthesisi) Intracellular digestionii) Destroy foreign substancei )Leucoplast - stores starchii)chloroplast - trap solar energyiii) Chromoplast - imparts colour toflowers amp fruitsi) Initiates and requlates cell division

কলিবতো তোর অ13সক ভোষোয় পরকোো কসর ঘলিরসয় ব যকত কসরস4 লেপরসমর ফতো আর লিবফতো লেক গোঢ় কসর লেদোসো কলিব ভোষ মসোপো3 যোসয়র অলিভবসর অ যলিদক

Economics

Factors of Production

Welcome to the new sessionToday we are going to start the first chapter of Class XThe name of the chapter is Factors of productionBy the name I hope you all can recall a glimpse of what you have learnt in the second chapter of Class IX

NowProduction is the process of creating the various goods and services which are consumed by the people of the country to satisfy their wants

Thus it is the process in which some materials are transformed from one form to another to create utility and value in goods

For example utility can be created by changing the form of a commodity ie

Making of table out of wood by a carpenter for his customer here the wood is getting transformed into table creating utility for his customer and he can also command a price for it

On the other hand Housewives perform very

useful activities at home which create utility but their domestic activities are not included in production because they have no money value

So we can also say that Production denotes two things firstly creation of utility and secondly creation of value

Production is not complete unless it reaches the consumer

An increase in production will increase the economic welfare of the consumers and hence the aim is to raise the production level of the country

Again production of a good or service is only possible if certain resources or

Questions

1 What do you mean by production

Answer Production means the creation of goods and services for the purpose of selling in the market

In fact production involves the transformation of inputs into outputs

Hence production denotes two thingsCreation of utility and creation of valueUtility and value can be created by changing the form by changing the place by changing the time and by rendering services

Example Transformation of raw

materials into finish goods such as potter creates utility by converting mud into utensils assembling of small parts to make bigger machinery

Production also includes services such as distribution and marketing

2 What are the factors of production

Answer Factors of Production refers to the resources and inputs needed for producing goods and servicesThese inputs can be classified as

Land Labour

Capital Enterprise

Land Land is defined to include not only the surface of the earth but also all other free gifts of nature(for example mineral resources forest resources and indeed anything that helps us to carry out the production of goods and services but is provided by

inputs are used together in right proportion

A resource or an input which helps in the process of production to obtain an output is called FACTOR OF PRODUCTION

These factors of production can broadly be categorized into four parts 1LAND 2LABOUR3CAPITAL4ENTERPRISE (ORGANISATION)or Entrepreneur

The above factors are all interdependent on each other and they play a major role in production process

FACTORS OF PRODUCTION

LANDCAPITAL

LABOUR ENTREPRENEUR

nature free of cost)LabourLabour refers to the human efforts that need to be combined with other factors of production for creating an output

CapitalAll man ndash made means of production is called capita example machineries which help in further production Money when used for starting any business for purchasing raw materials machinery tools etc it is regarded as capitalCapital also includes physical capital like factories machineriestoolsbuildingsequipments etcEnterpriseThe task of bearing risks is called enterprise and the person who bears these risks of business is called the entrepreneurThus an entrepreneur is one who organises production takes important decisions regarding production hires and purchases factors of production and bears the risk and uncertainty involved in productionOrganisation refers to the services of an entrepreneur who controls organises and undertakes all risks One who plans organises and manages a business enterprise is an organiser

Physics Chapter 1 Force

Force is an external agent capable of changing the state of rest or motion of a particular body It has a magnitude and a direction The direction towards which the force is applied is known as the direction of the force and the application of force is the point where force is applied The Force can be measured using a spring balance The SI unit of force is Newton (N)

Question 1

State the condition when on applying a force the body has

(a) the translational motion

(b) The rotational motion

Solutions

(a) Translational motion is produced when the body is free to move

(b) Rotational motion is produced when the body is pivoted at a point

Question 2

Define moment of force and state its SI unit

Solutions

The moment of force is equal to the product of the magnitude of the force and the perpendicular distance of the line of action of force from the axis

of rotation

The SI unit of moment of force is Newton times meter

= Newton meter (Nm)

Commercial Studies

Stake holders In this topic you will be come to know about the meaning and concept of stakeholders

How stakeholders are different from shareholders

Questions1 What do you mean by the term stake holdersAnswer) The term stake holders have developed from the words which mean an interest or expected benefit Stakeholders mean all those individuals groups and Institutions which have a state (interest) in the functioning and performance of a commercial organisation or a business enterprise2 What do you mean by share holdersAnswer) The person and Groups who own the shares of the joint stock company by providing capital to the company are called shareholders Shareholders are the internal stakeholders shareholders are one out of several stake holders3 How are shareholders different from stakeholdersAnswer)i) The term shareholders is related to only joint stock company whereas stakeholders are related with all business organisationsii) Stakeholders maybe any individual having financial stake in business organisation whereas a shareholders are those individuals who are holding shares in the company4) How are shareholders different from creditorsAnswer) i) Shareholders are internal stakeholders while creditors are external stakeholdersii) Shareholders invest in the capital of the company whereas creditors give loan to the companyiii) Shareholders are the members of the company with voting rights but creditors are not the members of the company

English 1 Transformation of sentences

Sentences A sentence is a group of words which makes complete sense

e Assertive sentencesf Imperative sentencesg Interrogative sentencesh Exclamatory sentences

Sentences can be changed from one grammatical form to another without changing the meaning of the sentence This is known as transformation of sentences

Exercise 1 Change the following affirmative sentences into Negative sentences

a He is a good manHe is not a bad man

b Ram loves SitaRam is not without love for Sita

c Only he stood first in the classNone but he stood first in the class

d Ankit was wiser than he

He was not so wise as Ankite He did it

He did not fail to do itf As soon as I reached college the

bell rangNo sooner did I reach college than the bell rang

g He finished everythingHe left nothing unfinished

h It always pours when it rainsIt never rains but it pours

Math Topic Commercial MathematicsChapter ndash Goods and services Tax

What is GSTAns It is a abbreviated term of Goods and Service Text which is an indirect tax levied on the sale of goods and rendering servicesSome terms related to GSTDelar Any person who buys goods or services For resale is known as a delar A delar Can be a firm or a companyIntra-state sales Sales of goods and services within the same state or same union territory are called intra- state salesInter-state sales Sales of goods and services outside the state or union territory are called Inter-state sales4) Input GST GST is paid by dealers on purchase of goods and services are called input GST5) Output GST GST is collected from customers on sale of goods and services are called output GST6) Types of GST There are three taxes applicable under GST(i) Central Goods and Services Tax (CGST)(ii) State Goods and Services Tax (SGST) or Union Territory Goods and Services Tax (UTGST) Both these taxes are levied on intra-state sales Here GST is divided equally among central and state governments(iii) Integrated Goods and Services Tax (IGST) IGST is levied on inter- state sales It is also levied on import of goods and services into India and export of goods and services from India

Subject Eng Literature (The Merchant of Venice ndash William Shakespeare)Topic Act III Scene 4 Lines 1 to 44 (Portia hellip To wish it back on you fare you well Jessica)[Students should read the original play and also the paraphrase given in the school prescribed textbook]

Summary Questions amp AnswersIn this scene we suddenly find a new element in the character of Portia We have already seen her possessed of every graceful womanly quality but now she shows that she is capable of rapid decision and determined action She shows this by her sudden resolve to hasten to Venice with a daring scheme for the rescue of Antonio This is an important scene in the dramatic action for it leads up to and renders possible the striking events of the famous trial scene which is one of the greatest striking elements of the play Moreover the fact that all the characters of importance are now assembled together in Venice makes the union of the main plot and the secondary story complete

(1) LORENZO Madam although I speak it in your presence(Line 1-9)

You have a noble and a true conceit

Of god-like amity which appears most strongly

In bearing thus the absence of your lordBut if you knew to whom you show this honourHow true a gentleman you send reliefHow dear a lover of my lord your husbandI know you would be prouder of the workThan customary bounty can enforce you

(a) Where is Lorenzo Why is he here To whom is he referring as lsquoMadamrsquo

Lorenzo is at Portiarsquos residence He had met Salerio on the way and Salerio had begged him to come along with him to

o In this scene Portia Nerissa Lorenzo Jessica and Balthazar appear

o Portia requests Lorenzo and Jessica to be in charge of her house during her absence from Belmont because she and Nerissa have decided to spend the days in meditation and also in visiting the holy places in the neighbourhood of Belmont She has already instructed her people to acknowledge both Lorenzo and Jessica as master and mistress of house during her absence Lorenzo and Jessica gladly agree to look after the house of Portia

handover the letter from Antonio to Bassanio The letter carried the bad news about Antoniorsquos arrest for non-payment of loan taken from Shylock Hence Salerio might have preferred company to break this bad news to Bassanio He is referring to Portia as Madam(b) What does Portia say on hearing the above extract

Portia says that she has never regretted doing good to others Friends who spend a lot of time together and really are there for each other have many traits in common As Antonio is Bassaniorsquos best friend saving him is like saving Bassanio who is like her own soul She asks Lorenzo to take care of management of the house till Bassanio is back(c) What does Portia send with Bassanio and why

On hearing about Antoniorsquos troubles on account of Bassanio her husband Portia immediately sends him with enough gold to repay the debt many times over to Venice to help Antonio out of his misfortune

(2) Lorenzo Madam with all my heart (Line 36-40)

I shall obey you in all fair commands

Portia My people do already know my mindAnd will acknowledge you and JessicaIn place of Lord Bassanio and myselfSo fare you well till we shall meet again

(a) Where are Lorenzo and Portia at this time What lsquofair commandsrsquo are given to Lorenzo

Lorenzo and Portia are at Belmont during this scenePortia reveals to Lorenzo that she has sworn to contemplate in prayer at a monastery around two miles away until her husband returns from Venice She tells him that Nerissa would accompany her and asks him to manage the house with Jessica till things are settled In response Lorenzo tells her that he would be obliged to do whatever she asks him to do(b) Where is Portia actually going and why

Portia tells Lorenzo that she would live a life of contemplation and pray at a monastery which is two miles away from her place In reality Portia plans to go to Venice in disguise with Nerissa and argue the case in defense of Antonio She is very sure that her plan would succeed

ClassXI (ScienceHumanitiesCommerce)Subject Topic Summary Execution

Computer Science

(APC)

Ch ndash 1 Numbers

(Numbers in different bases and

their Arithmatical operations)

Number System In computers Number System is defined as a writing system to represent the numbers in different ways ie we are using different symbols and notations to represent numbers There are four ways we can represent the number ndash Binary Decimal Octal and Hexadecimal

Decimal Number SystemThis number system consist 10 digits These are 0 1 2 3 4 5 6 7 8 amp 9

Binary Number SystemThis number system has only two digits these are 0 and 1 Here 0 stands for off while 1 stands for on

Octal Number SystemThis number system has 8 digits these are 0 1 2 3 4 5 6 amp 7

Hexadecimal Number SystemThis number system has 16 digits these are 0 1 2 3 4 5 6 7 8 9 A B C D E F Here the value of the alphabets are as follows A=10 B=11 C=12 D=13 E=14 F=15

Rules for conversion decimal number to Binary1 Divide the decimal number by 22 If the number will not divide equally by 2 then round down the answer to the nearest whole number (integer)3 Keep a note of the remainder it should be either 0 or 14 Keep repeating the above steps dividing each answer by 2 until you reach zero5 Write out all the remainders from bottom to top This is your binary solution

For example Lets convert 32 to binary 2 32 2 16 - 0 2 8 - 0 2 4 - 0 2 2 - 0 2 1 - 0 0 - 1

The binary equivalent of 3210 is 1000002

Try the follwing youself1 2410

2 4810

3 1210

History GROWTH OF NATIONALISM

The second half of the 19th century witnessed growth of political consciousness and a sense of Nationalism among the IndiansThere were various factors for growth of Indian Nationalism- As a result various political associations were formed in different provinces by the educated Indians Surendranath Banerjee organized a meeting of National conference at Calcutta Ultimately the National Congress was founded in Bombay in 1885This body became the vanguard of Indian struggle for freedom The congress leaders were known as moderates because they followed a policy of prayer and petition A large number of Indian leaders had experienced in political agitation The Political situation of England was also changed Moreover increasing revolutionary activities in Maharashtra Punjab and Bengal became serious concern to the British Government In this

QUESTION1 What do you mean by Nationalism ANSWER 1 Nationalism is defined as loyalty and devotion to own nation especially a sense of national consciousnessQUESTION 2 What are the causes of nationalism ANSWER 2 There were various factors for growth of nationalism

1 Spread of western education2 The progress of vernacular press and

patriotic literature3 The economic exploitation of our

country by the colonial rulers4 International affairs

QUESTION 3 Who organized National conference in Calcutta in 1883 ANSWER 3 Surendranath BanerjeeQUESTION 4 When did Indian National Congress formANSWER 4 Indian National Congress was formed in 1885 in BombayQUESTION 5 Who were ModeratesANSWER 5 The Early Nationalists were also known as Moderates Their emergence marked

background Lord Curzon became Viceroy in India He had no respect for the Indian National Congress

the beginning of the organized national movement in India They believed in British justice and were loyal to them They followed a policy of prayer and petition They demanded constitutional reforms of our country Impotant Moderate leaders were Pherozshah Mehta Dadabhai Naorozi and Surendranath Banerjee etcQUESTION 6 What do you know about Extremism in Indian National movementANSWER 6 In the beginning of 20th century a new class of national leaders emerged in India which was different from the moderate groups They started more aggressive movement against the British empire The goal of extremists was ldquoswarajrdquo Important extremist leaders were Bal Gangadhar Tilak Lala Lajpat Rai Bipin Chandra Pal etcQUESTION 7 Mention the places which were the main centres of Revolutionary movementANSWER 7 Maharashtra Bengal and Punjab

Physics

Chapter Dimensional Analysis

(Summary)

The dimensions of a physical quantity are the powers to which the fundamental units are raised in order to obtain the derived unit of that quantit

The physical quantites lengthmasstime are represented by [L] [M] [T] resp let they are raised to powers ( dimesions) abc resp then any physical quantity can be represented by [ La Mb Tc ] Examples

1 Area area = L x B = [L] x [L] = [M0 L2 T0 ]

2 Density density = massvolume = [M][L3] = [ M L-3]

3 Velocity velocity = distancetime = [L][T] = [LT-1]HW Try to find out dimension of acceleration Acceleration = velocity timeNB One can find the SI Units Using Dimension Analysis Such as for area we have [L2] so its SI unit is m2

Biology Topic ndash Chp-1 The living world

Today we will start the first chapter the living world Here we discuss about the characteristics of living organism and what are the difference between them and nonliving substances We also discuss about the contribution of different Scientists

There are over 500000 species of plants andover a million species of animal are present on earth Some 15000 new species were discovered every yearQ1 What is a living organismbull A living organism is primarily physico -chemical material that demonstrate a high degree of complexity is capable of selfRegulation possesses a metabolism and perpetuates itself through timeQ2 What are the differences between livingand non-livingsi) Compared with non-living living organisms

have more complex organised structure and their use of energy is more controlled amp efficientii) Living things reproduce their own kind by forming new cells which contains copies of their genesiii) Each organism has some degree of homeostasisie it is able to make adjustments so that internal environment remains constantQ3 Write contributions of following Scientists i) Aristotle - One of the first theories in Biology places all living things in a hiearchieii) AV Leeuwenhoek - was the first to observe living single celled organisms under microscopeii) Carolus Linnaeus - developed the binary system for naming of organisms and classificationiii) Geregor Johann Mendel ndash discoverbasic principles of inheritanceHomework i) C Darwin ii)Schleiden

Math Trigonometric functions

1 Overviewi) Trigonometry The word lsquotrigonometryrsquo is derived from the Greek words lsquotrigonrsquo and lsquometronrsquo which means measuring the sides of a triangle An angle is the amount of rotation of a revolving line with respect to a fixed line Usually we follow two types of conventions for measuring angles ie a) Sexagesimal system b) Circular system In Sexagesimal system the unit of measurement is Degree In Circular system the unit of measurement is Radian ii) Relation between degree and radianThe ratio of circumference of a circle to its diameter is always a constant This constant ratio is a number denoted by π which is taken approximately as 227The relationship between degree amp radian measurements is as follows2 right angles = 180deg= π radians1radian = 180degπ=57deg16(approx) 1deg=π180 radianiii) Length of an arc of a circleIf an arc of length s subtends an angle θ radians at the center of a circle of radius r then s=rθiv) Area of a sector of a circleA sector is like a pizza slice of the

Q) Express the following angles in radiana) 45deg b) 40deg3730Ans a) We have 180deg=π radiansi e 45deg= πtimes45180 radian = π4 radiansb) 40deg3730= 40deg37+3060 minute= 40deg 37 +12 minute= 40deg+ 752 minute=40 + 75(2times60) degree=3258 degreeNow 180deg=π radianie 3258 degree= (πtimes325) (180times8) radians = 65π288 radiansQ) A circle has a radius of r=12 meters What is the length of an arc traced out by a 60deg angle in the center of the circleAns In this problem we know both the central angle (60deg) and the radius of the circle (12) All we have to do is plug those values into our equation and we get

s = 2π(12)(60360)s = 24π6s = 4πSo the length of an arc traced out by a 60deg angle in a circle with a radius of 12 meters equals 4π meters asymp 1257 metersQ) Find the area of the sector with a central angle 30deg and a radius of 9cmAns GivenRadius r = 9 cmAngle θ = 30degArea of the sector = θ360degtimesπr2

= 30360degtimes227times92=2121cm2

circle It consists of a region bounded by two radii and an arc lying between the radiiThe area of a sector is a fraction of the area of the circle This area is proportional to the central angle In other words the bigger the central angle the larger is the area of the sectorArea of Sector = θ2 times r2 (when θ is in radians)

Area of Sector = θ times π360 times r2 (when θ is in degrees)

COMMERCE

CLASSIFICTION OF HUMAN ACTIVITIES-ECONOMIC AND NON-ECONOMIC

Welcome to the new sessiontoday we are going to start the first chapter of Class XI The name of the chapter that we are going to start is

lsquoClassification of Human Activities ndasheconomic and non-economicrsquo

Now let us start the chapter by considering human beings and the activities they perform throughout the day

Human activities means all those activities that human beings undertake to satisfy their wants

Human wants on the other hand are the desire of human beings for goods (vegetables fruits rice etc) and services (services of doctors teachers lawyers etc) that they require to live

Now these human activities continue throughout life as human wants are unending unlimited and recurring as human beings desire for better living throughout their lives

Now human activities can be classified into two categories

Human activities

Economic activities Non-economic activities

Economic activities are

Questions1 What are human activities

Answer Human activities mean all those activities that human beings undertake to satisfy their wants

Example A man working in an office

A boy playing in the garden

2What are the characteristics of human activitiesAnswer the characteristics of human activities are as follows

Human activities are undertaken by men women and children and these activities involve human efforts

Human activities are undertaken to satisfy human wants which are unlimited

Human activities continue throughout life

Human activities are performed for both earning money and personal satisfaction

3What is economic activitiesGive example

Answer Economic activities are undertaken by human beings with the object of earning money acquiring wealth and thereby satisfying human wantsExample

Selling of goods by a shop keeper to his customer

A clinic run by a doctor Service of a teacher in school or college

undertaken by human beings with the object of earning money and acquiring wealth

These activities result in the production of economic goods and services

Example Human activities(ie working in factories officesshops) which produce direct economic benefits

Non-economic activities are inspired by human sentiments and emotions such as love for the family desire to help the poor and love for the country

Thus these human activities (eg praying playing sleeping) produce no direct economic benefits and they are also not related to earning money and acquiring wealth

4 What are the characteristics of economic activities

Answer The characteristics of economic activities are as follows

Economic motiveEconomic activities are undertaken to earn money and acquire wealth

ProductiveEconomic activities involve productiondistribution and exchange of goods and services to create wealth

Economic growthEconomic activities determine the level of economic development of a country and standard of living of its citizens

Socially desirableEconomic activities are socially desirable for society

Economic resourcesEconomic activities make use of all the economic resources such landlabourcapital etc

5 What do you mean by non-economic activitiesExampleAnswerNon-economic activities are inspired by human sentiments and emotions such as love for the family desire to help the poor and love for the countryThese activities are not undertaken for monetary gain but for onersquos satisfaction and happinessExample

a mother looks after her children

a student donates blood8 Differentiate between Economic activities and Non-economic activities

Economic activities

Non-economic activities

1to earn living and acquiring wealth2Result can be measured in terms of money

3ExampleBusinessprofession and employment

1 to obtain some satisfaction

2Result cannot be measured in terms of money

3ExampleFamily-orientedreligious socialCultural and national

BUSINESS STUDIES

BUSINESS ENVIRONMENT

Welcome to the new sessionToday we are going to start the first chapter and the name of the chapter is Business Environment

In todayrsquos world every business enterprise is a part of the society It exists and operates in association with various groups in society such as customers suppliers competitors banks and financial institutions government agencies trade unions media and so on All these groups influence the functioning of business in one way or the other They constitute the environment of businessConcept of Business Environment

The term lsquobusiness environmentrsquo refers to the sum total of all individuals institutions and other forces that lie outside a business enterprise but that may influence its functioning and performance

The main features of business environment

Totality of External forces General and Specific forces Interrelatedness Complexity Dynamic Uncertainty Relativity

The Interrelation between business and its environment

The business enterprise is an open system It continuously interacts with its environment It takes inputs

Prepare the following questions from todayrsquos assignment

1 What do you mean by business environment

The term lsquobusiness environmentrsquo means the aggregate of all forces factors and institutions which are external to and beyond the control of an individual business enterprise but they may influence its functioning and performance Business environment is the macro framework within which a business firm a micro unit operates It consists of several interrelated and interacting elements

2 Explain the main features of business environment in brief

Totality of External forces-Business environment is the sum total of all things external to a business environment

General and Specific forces-It includes both the forces general forces are the economic social political legal and technological conditions which indirectly influence all business enterprise Specific forces are the investors customers competitors and suppliers which influence individual enterprise directly

Interrelatedness-Different elements of environment are interrelated for an example growing awareness for health care has increased the demand for health foods

Complexity- Business environment id

(such as raw materials capital labour energy and so on) from its environment transforms them into goods and services and sends them back to the environment

Fig 1 Business Environment Relationship

complex in nature as the elements keep on changing example economic technological and other forces changes in demand for a product and service

Dynamic-Business environment is not static it keeps on changing

Uncertainty- Itrsquos very difficult to predict future events such as technology and fashion which occur fast and frequently

Economics Basic Economic ConceptsSub topic

Microeconomics and

Macroeconomics

Welcome to the new sessiontoday we are going to start the first chapter of Class XI The name of the chapter that we are going to start is Basic Economic concepts

Now Economics covers the study of human activities Human activities are those activities which are performed by humans to satisfy their wants

Thus Human wants are unlimited and therefore economic activities such as production exchange and consumption are needed in order to satisfy those wants

The study of economics is divided largely in two parts which areMicroeconomics and Macroeconomics

SUBJECT- MATTER OF ECONOMICS

MICROECONOMICS MACROECONOMICS

Questions1Who has coined the words micro and macro economics

Answer Ranger Frisch coined the words lsquomicrorsquo and lsquomacrorsquo in 1933 to denote the two branches of economic theory namely microeconomics and macroeconomics

2What is microeconomicsAnswer It is the study of behaviour of individual decision ndash making unit such as consumers firms etc

3 What is macroeconomicsAnswer Macroeonomics is the study of overall economic phenomena like employment national income etc

4 What is the importance of microeconomicsAnswer

Microeconomics helps in formulating economic policies which enhance productive efficiency and results in greater social welfare

It helps the government in formulating correct price policies

It explains the working of a capitalistic economy where individual units(producers and consumers ) are free to take their own decision

Micro means a small part in

microeconomics we do not study the whole economy Hence we study an individual consumer and his or her choices and a producer and his or her profit maximizing decisions in the market Thus it does not mirror what happens in the economy as a whole

Macroeconomics on the other hand studies the economy as a whole It is concerned with aggregate and depicts the entire picture of the economyMacroeconomics deals with the national income aggregate investment aggregate consumption etc

Features of Microeconomics It deals with small

parts of the country Hence it looks at

individual consumers firms and industries

It deals with individual income consumption and savings

It studies the determination of price of any product or factors of production

It deals with the working of market via the price mechanism which is nothing but the determination of price and quantity of a commodity by the forces of demand and supply

Features of Macroeconomics

It deals with the study of the economy as a whole

It is concerned with

5 Give a limitation of microeconomics Microeconomics fails to explain the

functioning of an economy as a whole It cannot explain unemployment illiteracy and other problems prevailing in the country

6 What is the importance of macroeconomics It gives overall view of the growing

complexities of an economic system It provides the basic and logical

framework for formulating appropriate macroeconomic policies (eg for inflation poverty etc )to direct and regulate economy towards desirable goals

7What is the limitation of macroeconomics It ignores structural changes in an

individual unit of the aggregate

8 Differentiate between Microeconomics and Macroeconomics

Microeconomics Macroeconomics

the study of aggregates

National income aggregate savings and aggregate investments are major concepts dealt within macroeconomics style

It studies the determination of general price levels

It investigates into the problem of unemployment and the achievement of employment

It studies the aspect of decision making at the aggregate and national levels

It includes all growth theories whether related to developed or developing economies it also includes the study of economic systems and the working of the economy under different systems

Note Both Micro and macro economics are complementary and should be fully utilized for proper understanding of an economy

1It studies economic aspect of an individual unit2It deals with individual incomeConsumption and savings

3 It facilitates determination of price of any product or factors of production

4 Itrsquos scope is narrow and restricted to individual unit

1It studies the economy as a whole

2It deals with the national income aggregate consumption and aggregate savings3 It facilitates determination of general price level in an economy

4 Itrsquos scope is wide as it deals with economic units on the national level

ACCOUNTS

Introduction to Accounting and Book-keeping

Today I am going to share you the meaning of Accounting and Book-keeping and its related terms bullAccounting bullBook Keeping bullAccountsbullTypes Of Accounts bullAccounting Cycle

bull Meaning of accounting

Ans ) Accounting is the art and science of recording classifying and summarising monetary transactions

bull Meaning of Book-keeping

Ans) Bookkeeping is the art of recording business transactions with the view of having a permanent record of them and showing their effect on wealth

bull Meaning of account

Ans) The term account means a record of

business transactions concern a particular person of firm asset or income or expense It is a summarised record of all transactions which take place in an accounting year

bull Types of accountsPersonal accounts ndash Personal accounts relating

to person and Organisation are known as personal accounts Example Ramrsquos Account ABC amp Co Account etc

Real account - The accounts related to tangible and intangible assets are called real accounts Example Cash Account Furniture Account etc

Nominal account- Accounts related to expenses losses incomes and gains are known as nominal accounts Example Wages Account Salary Account Discount Account etc

bull Accounting cycle Accounting cycle refers to a complete sequence of accounting activities It begins with recording of transactions and ends with the preparation of a balance sheet

Chemistry TopicAtomic Structure

Thomsonrsquos atomic modelThomson (1898) was the first to propose the model of an atomHe proposed that an atom can be regarded as a uniform sphere of positive electricity in which requisite number of electrons are embedded evently to neutralize the positive chargeThis is just like plums embedded in a pudding or seeds evently distributed in red spongy mass of a watermelonThis model of atom is known as ldquoPlum-Pudding modelrdquo or

Q1)What is the fundamental constituents of atomAns Electron Proton and neutrons are the fundamental constituents of atomQ2)What is the value of fundamental unit of electricityAnsThe charge carried by one electron is sad to be the fundamental unit of electricityIts magnitude is 48times10-10esuOr 1602times10-19C Q3)Name the element containing no neutronAnsOrdinary hydrogen atom or protium 1H1

Types of AccountPersonal AccountReal AccountNominal AccountBalance Sheet (opening)

ldquowatermelon modelrdquoThis model could explain the electrical neutrality of an atom but failed to explain the result of scattering experiment carried out by Rutherford in 1911So it was rejected ultimately

Q4)Why is an electron called universal particleAns Itrsquos mass and Charge are independent of its source

EVS Chapter 1 ndash Modes of Existence

Modes of existence When one speaks normally about the mode of existence of some group or individual one refers to their customs their mode of being their ethology their habitat in some way their feeling for a placeDifferent modes of exixtence are ndash

1 Hunting ndashGathering2 Pastoral3 Agricultural4 Industrial

1 Hunting and gathering Hunting and gathering mode of existence is characterized by obtaining food from hunting wild animals including fishing and gathering wild plants From their earliest days the hunter-gatherer diet included various grasses tubers fruits seeds and nuts Lacking the means to kill larger animals they procured meat from smaller game or through scavenging

Societies that rely primarily or exclusively on hunting wild animals fishing and gathering wild fruits berries nuts and vegetables to support their diet are called hunting and gathering societies

At least this used to be practice of human beings before agriculture is invented As their brains evolved hominids developed more intricate knowledge of edible plant life and growth cycles

Q) Write the features of Hunting ndash gathering societiesAns - There are five basic characteristics of hunting and gathering societies

i The primary institution is the family which decides how food is to be shared and how children are to be socialized and which provides for the protection of its members

ii They tend to be small with fewer than fifty members

iii They tend to be nomadic moving to new areas when the current food supply in a given area has been exhausted

iv Members display a high level of interdependence

v Labor division is based on sex men hunt and women gather

Political Science

Introduction to political science

Political science occasionally called politology is a social science which deals with systems of governance and the analysis of political activities political thoughts associated constitutions and political behaviorThe study of political science involves the study of both the

Answer the following questions-1 What is political science

Political science occasionally called politology is a social science which deals with systems of governance and the analysis of political activities political thoughts associated constitutions and political behavior

2 Short notes-

traditional and modern theories of politicsTraditionalClassical political sciencepolitical theory-Traditional political science is the study of politics before Second World War The methodology to study Politics was traditional (legal formaletc) the definition of politics traditional (Politics begins and end with state)area of study (constitution state machinery)was traditionalModern Political scienceModern political theory-Modern Political Theory critically examines the contemporary state of political theory making an assessment of the achievement and limitations of the Behavioural Revolution in its totality and reviews objectively the major paradigms and conceptual frameworks adopted by the disciplineContemporary attempts at the development of an integrated political theory involving the use of both traditional and modern concepts approaches and theories-Around late 1960s several political scientists realized the importance of both the traditional political theory and modern Political theory They began building an integrated theory of politics involving a systematic mixture of traditional and modern studies of politics It was held that the study of a complex and vast field like politics needs both traditional as well as

Classical political theory Modern Political theory

Homework-Learn

modern concepts and approaches for studying itrsquos all aspects

Subject Eng Literature (The Tempest ndash William Shakespeare) Topic Act I Scene 1 Lines 1 to 32 (Line 32 ndash Gonzalo hellip If he be not born to be hanged our case is miserable) Date 13th April 2020 (3rd Period)

[Students should read the original play and also the paraphrase given in the school prescribed textbook]Summary Questions amp Answers

[SUMMARY OF THE ENTIRE SCENE]

o The play starts with the scene of a severe storm at sea Alonso (King of Naples) Sebastian (Alonsorsquos brother) Ferdinand (Alonsorsquos son) Gonzalo Antonio (the usurping Duke of Milan) are in a ship in the midst of the storm

o The mariners are trying their best to control the vessel from running aground and are totally following the orders of their Master the Boatswain They have scant success

o The mariners become extremely unhappy and annoyed when most of the passengers arrive on the deck thereby hampering their effort to save the ship There is serious confrontation between them and the passengers who are part of the Kingrsquos entourage

o The mariners could not save the ship

SUMMING-UP

(i) Vivid description of the scene which gives a realistic description of terror and confusion of a tropical storm

(ii) Shows Shakespearersquos accuracy of knowledge in describing the naval operations and also matters of seamanship

(iii) The opening scene justifies the title ndash The Tempest

UNANSWERED QUESTIONS

(i) The King always travels with his entire fleet including his soldiers Where

(1) GONZALO Nay good be patient (Line 15-26)BOATSWAIN When the sea is Hence What cares these

roarers for the name of the king To cabin silence Trouble us not

GONZALO Good yet remember whom thou has aboardBOATSWAIN None that I more love than myself You are a

councillor if you can command these elements to silence and work

the peace of the present we will not hand a rope more use your authority If you cannot give thanks you have

lived so long and make yourself ready in your cabin for the mischance of the hour if it so hap [To the Mariners]

Cheerly good hearts [To Gonzalo] Out of our way I say

(a) To whom is the boatswain speaking What does he mean by lsquoNone that I more love than myselfrsquo

The Boatswain is speaking to Gonzalo the honest old councilor of the Duke of MilanBy using the words ndash lsquoNone that I love more than I love myselfrsquo means that for the Boatswain nobody is dearer to him than his own life

(b) What were the conditions that made the boatswain react in this way

The Boatswain reacts in this way because the storm is at sea and Alonso King of Naples Sebastian his brother Ferdinand his son Gonzalo Antonio the usurping Duke of Milan on board are in distress and in panic Thus they have rushed to the deck interrupting the work of the mariners

(c) What hope does Gonzalo take from the attitude of the boatswain

The insolent and authoritative attitude of Boatswain makes Gonzalo feel comforted He tells that there are no signs that the Boatswain will be drowned But his facial appearance and attitude shows that he is destined to die on land by hanging which in effect means that all on board will be saved Otherwise all the persons on board are doomed

(d) How can they lsquomake yourself ready in your cabinrsquo For what were they asked to make ready themselves

In order to make themselves ready in their cabin the

were the other ships

(ii) Why was the ship in that area Where was it coming from or going where

(iii) The ship broke apart What happened to those who were in the ship

passengers on board must prepare for death which they will possibly soon have to meetThey can retire to their cabins and offer prayers to the Almighty to save them from drowning

(e) What does the boatswain say when he is asked to be patient What does he order to the royal party

When the boatswain is asked to be patient and remain calm he says that he will be patient only when the storm will be over and the sea will be calm but as long as the storm blows and there is danger to the ship he cannot think of being patient He orders the royal party to go to the cabin and leave the mariners to their work

(2) GONZALO I have great comfort from this fellow (Line 27-36)

Methinks he hath no drowning mark upon him his complexion is perfect

gallows Stand fast good Fate to his hanging Make the rope of his destiny our cable for our own doth little advantage If he be not born to be hanged our case is miserable

(a) Why does Gonzalo regard the Boatswain in the midst of danger

In the midst of danger Gonzalo regards the boatswain because he feels that the Boatswain is a source of comfort and is bent upon to do his work sincerely which in this case is saving the ship and its passengers from the severest of raging storm

(b) What reasons does Gonzalo give when he says that none in the ship will die of drowning

Gonzalo is almost sure that none in the ship will die by drowning His says that there is no mark on the face of the boatswain that indicates that he will die by drowning On the other hand the lines on his face are strong indications that he will be hanged to death Therefore there shall be no danger of the shiprsquos sinking

(c) Explain the following ldquoStand fast good Fate to his hanging Make the rope of his destiny our cable for our own doth little advantage If he be not born to be hanged our case is miserablerdquo

The stated lines mean that if the will of destiny is to be carried out then the ship will not get wrecked and all the passengers will be saved The safety of the passengers therefore depends upon the will of fate being carried out in the case of the boatswain If however the boatswain is not to die by hanging then the passengers are also very unsafe because in that case the ship is likely to sink

(d) What order does the Boatswain give to the sailors

when he re-enters What does he say about the crying of the fellows inside the cabin

The boatswain orders the sailors to bring the topmast lower and bring the ship close to a stationary position with the help of the main sail He says that the fellows inside the cabin are moaning and crying in their distress louder than his voice and louder even than the roaring of the storm

Class XII (ScienceCommerceHumanities) Subject Topic Summary Execution

Computer Science

PropositionalLogic

Propositional logic is a procedure to provide reasoning through statementProposition A ststement that results in True or False is said to be proposition There are two types of propositionSimple proposition amp compound propositionSimple proposioton A simple proposition is one that is not a part of any other proposition Such sentential form of proposition is symbolized with english letters in short For example Ram is a claver student (TrueFalse)Where do you live (Not in True or False)Grapes are sweet (TrueFalse)It rains today (TrueFalse)Here we can see some statements anwer would be true or false but some staements answer can not give in terms of true or false Thus the sentences which can be answered in true or false are known as simple propositionAssigning propositon to a variableThe general syntax to assign propostion to a variable is as followsVariable = Simple propositonFor example A=Ram is a clever studentB= Grapes are sweetC= it rains todayCompound proposition

helliphellipto be continued in next classhelliphellipMath Relation Relation If A and B are two non-empty sets

then a relation R from A to B is a subset of AxB If R A x B and (a b) R then we say that a sube isinis related to b by the relation R written as aRbeg Let A be the set of students of class XII and B be the set of students of class XI Then some of the examples of relation from A to B arei) (a b) AXB a is brother of bisinii) (a b) AXB age of a is more than age of isinb Types of relation In this section we would like to study different types of relations We know that a relation in a set A is a subset of A times A Thus the empty set φ and A times A are two extreme relations For illustration consider a relation R in the set A = 1 2 3 4 given by R = (a b) a ndash b = 10 This is the empty set as no pair (a b) satisfies the condition a ndash b = 10 Similarly R = (a b) | a ndash b | ge 0 is the whole primeset A times A as all pairs (a b) in A times A satisfy | a ndash

Example 1 Let A be the set of all students of a boys school Show that the relation R in A given by R = (a b) a is sister of b is the empty relation and R = (a b) the primedifference between heights of a and b is less than 3 meters is the universal relationSolution Since the school is boys school no student of the school can be sister of any student of the school Hence R = φ showing that R is the empty relation It is also obvious that the difference between heights of any two students of the school has to be less than 3 meters This shows that R = A times A is primethe universal relation Example 2 Show that the relation R in the set 1 2 3 given by R = (1 1) (2 2) (3 3) (1 2) (2 3) is reflexive

b | ge 0 These two extreme examples lead us to the following definitionsDefinition 1 A relation R in a set A is called empty relation if no element of A isrelated to any element of A ie R = φ A times AsubDefinition 2 A relation R in a set A is called universal relation if each element of A is related to every element of A ie R = A times A Both the empty relation and the universal relation are some times called trivial relation Definition 3 A relation R in a set A is called(i) reflexive if (a a) R for every a Aisin isin(ii) symmetric if (a1 a2) R implies that (aisin 2a1)

R for all aisin 1 a2 Aisin(iii) transitive if (a1 a2) R and (aisin 2 a3) R isinimplies that (a1 a3) R for all aisin 1 a2 a3 AisinDefinition 4 A relation R in a set A is said to be an equivalence relation if R is reflexive symmetric and transitive

but neither symmetric nor transitiveSolution R is reflexive since (1 1) (2 2) and (3 3) lie in R Also R is not symmetric as (1 2) R but (2 1) isin notinR Similarly R is not transitive as (1 2) R and (2 3) R but (1 3) R isin isin notinExample 3 Show that the relation R in the set Z of integers given byR = (a b) 2 divides a ndash b is an equivalence relationSolution R is reflexive as 2 divides (a ndash a) for all a Z isinFurther if (a b) R then 2 divides a isinndash b Therefore 2 divides b ndash a Hence (b a) R which shows that R is isinsymmetric Similarly if (a b) R and (b c) R isin isinthen a ndash b and b ndash c are divisible by 2 Now a ndash c = (a ndash b) + (b ndash c) is even (Why) So (a ndash c) is divisible by 2 This shows that R is transitive Thus R is an equivalence relation in ZExample 4 Let L be the set of all lines in a plane and R be the relation in L defined as R = (L1 L2) L1 is perpendicular to L2 Show that R is symmetric but neither reflexive nor transitiveSolution R is not reflexive as a line L1 can not be perpendicular to itself ie (L1 L1) R notinR is symmetric as (L1 L2) Risin

L1 is perpendicular to L2rArr L2 is perpendicular to L1rArr (L2 L1) RrArr isin

R is not transitive Indeed if L1 is perpendicular to L2 and L2 is perpendicular to L3 then L1 can never be perpendicular to L3 In fact L1 is parallel to L3 ie (L1 L2) R isin(L2 L3) R but (L1 L3) Risin notin

Chemistry Solid state Characteristics if Solids(i)The particles are locked in fixed positions they are unable to change their relative positions and this brings a definite shape and volume of a solid(ii)In a solid the constituent particles are held by strong forces of attractionThe forces of attraction may be bonding or non bonding(iii)The constituent particles in a solid pack together as closely as possibleoccupying most of the available space within the solidThus the empty space in a solid is very smallThis makes a solid highly rigid and nearly incompressibleThis also explains why a solid has high density and exhibits slow diffusionClassification of Solids

Q1)Define Crystalline solids AnsA Solid that has a definite geometrical shape and a sharp melting pointand whose constituent particles (atomsmolecules or ions) are arranged in a long range order of definite pattern extending throughout the solid is called a crystalline solidExNaClQ2)Define Amorphous solids AnsA solid that does not have a definite shape and a sharp melting pointand whose constituent particles (atomsmolecules or ions) are not arranged in a definite pattern is called an amorphoussolid

Crystalline solidsAmorphous solids

ExGlassRubberQ3)Classify Crystalline Solids Crystalline Solids

Physics Coloumbrsquos Law (Summary)

Before Going Into Coloumbrsquos Law We Will First Learn What is Charge Properties of Charge and Always remember that charge is quantized ie a body always have static charge of magnitude equal to some integral multiple of fundamental electronic charge e= 16 x 10- 19 C

Charge is the property of matter that causes it to produce and experience electrical and magnetic effects The study of the electrical charges at rest is called electrostatics When both electrical and magnetic effects are present the interaction between charges is referred to as electromagnetic

There exist two types of charges in nature positive and negative Like charges repel and unlike charges attract each other

The type of charge on an electron is negative The charge of a proton is the same as that of an electron but with a positive sign In an atom the number of electrons and the number of protons are equal The atom is therefore electrically neutral If one or more electrons are added to it it becomes negatively charged and is designated as negative ion However if one or more electrons are removed from an atom it becomes positively charged and is called a positive ion

The excess or deficiency of electrons in a body gives the concept of charge If there is an excess of electrons in a body it is negatively charged And if there is deficiency of electrons the body becomes positively charged Whenever addition or removal of electrons takes places the body acquires a charge

The SI Unit of charge is coulomb (C) In SI units the current is a fundamental quantity having a unit of ampere (A) The unit of charge is defined in terms of the unit of current Thus one coulomb is the charge transferred in one second across the section of a wire carrying a

Ionic SolidsMetallicSolids

Molecular Solids

current of one ampere

As q = It we have1 C = (1 A) (1 s)

The dimensions of charge are [A T]

Properties of Charge

(1) Quantization of Charge Electric charge can have only discrete values rather than any value That is charge is quantized The smallest discrete value of charge that can exist in nature is the charge on an electron given as

e = plusmn 16 x 10- 19 C

This is the charge attained by an electron and a protonA charge q must be an integral multiple of this basic unit That is

Q = plusmn ne where n = 1 2 hellip

Charge on a body can never be (frac12)e (23)e or 57e etcWhen we rub a glass rod with silk some electrons are transferred from the rod to the silk The rod becomes positively charged The silk becomes negatively charged The coulomb is a very large amount of charge A typical charge acquired by a rubbed body is 10 - 8 C

Biology Reproduction in organisms

Welcome to this new session 2020-21Today in this first chapter we mainly discuss about reproduction types needs and life span of some organismsWe also discuss about difference between sexual and asexual reproduction

Q1 What is reproductionReproduction is defined as a biological processin which an organism gives rise to young onessimilar to itselfQ2 What are the needs of reproductionbulli) Reproduction maintain life on earthii) It enables the continuity of the species generation after generationiii) It creates genetic variation among populationsQ3 Define Life span and write some orgnisms life spanbull Life span is the period from birth to

the natural death of an organism- OrganismsLife span1 Butterfly 1 - 2 weeks2 Fruit fly 30 days3Dog 10-13 years4 Rose5-7 years5 Tortoise100-150 years6 Banyan Tree -200 - 250 yearsQ4 Reproduction is of two types in case ofanimals but in case of plants vegetative propagation is also present

Asexual Reproduction Sexual Reproductioni) Always uniparentalii) Gametes are not involvediii) Only mitotic division involvediv) Somatic cells of parents are involvedv) Offsprings are genetically similar to the parents

i) Usually biparentalii) Gametes are involvediii) Meiosis occurs during gametogenesis Mitosis occurs after fertilisationiv) Germ cells of the parents are involvedv) offsprings are genetically different from the parents

COMMERCE BUSINESS ENVIRONMENT

Welcome to the new sessiontoday we are going to start the first chapter of Class XII The name of the chapter is Business Environment

Already many of you have got some idea about the word business environment form the first chapter of business studies in class XI

In todayrsquos world every business enterprise is a part of the society It exists and operates in association with various groups in society such as customers suppliers competitors banks and financial institutions government agencies trade unions media and so on All these groups influence the functioning of business in one way or the other They constitute the environment of businessConcept of Business Environment

The term lsquobusiness environmentrsquo refers to the sum total of all individuals institutions and other forces that lie outside a business enterprise but that may influence its functioning and performance

The main features of business environment Totality of External forces General and Specific forces Interrelatedness Complexity Dynamic Uncertainty

Prepare the following questions from todayrsquos assignment

2 What do you mean by business environment

The term lsquobusiness environmentrsquo means the aggregate of all forces factors and institutions which are external to and beyond the control of an individual business enterprise but they may influence its functioning and performance Business environment is the macro framework within which a business firm a micro unit operates It consists of several interrelated and interacting elements

2 Explain the main features of business environment in brief

Totality of External forces-Business environment is the sum total of all things external to a business environment

General and Specific forces-It

Relativity

The Interrelation between business and its environment

The business enterprise is an open system It continuously interacts with its environment It takes inputs (such as raw materials capital labour energy and so on) from its environment transforms them into goods and services and sends them back to the environment

Fig 1 Business Environment Relationship

includes both the forces general forces are the economic social political legal and technological conditions which indirectly influence all business enterprise Specific forces are the investors customers competitors and suppliers which influence individual enterprise directly

Interrelatedness-Different elements of environment are interrelated for an example growing awareness for health care has increased the demand for health foods

Complexity- Business environment id complex in nature as the elements keep on changing example economic technological and other forces changes in demand for a product and service

Dynamic-Business environment is not static it keeps on changing

Uncertainty- Itrsquos very difficult to predict future events such as technology and fashion which occur fast and frequently

Business Studies

Human Resources Management

Human resource of an organisation are the aggregate of knowledge skills attitudes of people working in it

The management system which deals with human resources is called human resource management

Features of HRMbullComprehensive functionbullPeople-oriented

Question1) What do you mean by human

resource management Answer) Human resource management may be defined as that field of Management which has to do with planning organising and controlling the functions of procuring developing maintaining and utilising the labour force

bullAction oriented bullPervasive function bullContinuous function

2) Explain the features of HRM in brief

Answer)bullHuman Resource Management is concerned with managing people at work bull Human Resource Management is concerned with employees which bring people and organisations together so that the goals of each are met bullHuman resource management considered every employees as an individual and also promote their satisfaction and growth bull Human resource management is inherent in all organisations and at all levelsbullManagement of human resources are ongoing on never ending process which requires a constant alertness and Awareness of human relations

3) ldquoHR function is said to be pervasiverdquowhy

Answer) Human resource management is required in all organisations whether it is private or government organisations armed forces sports organisations etc It permeatsall the functional areas like production marketing finance research etc This from this feature of human resource management it can be said that it is pervasive in nature

Economics Demand Q1DEFINITION OF DEMANDIn economics demand is the quantity of a good that consumers are willing and able to purchase at various prices during a given period of timeQ2DEMAND CURVEIn economics a demand curve is a graph depicting the relationship between the price of a certain commodity and the quantity of that commodity that is demanded at that pricQ3LAW OF DEMANDIn microeconomics the law of demand states that conditional on all else being equal as the price of a good increases quantity demanded decreases conversely as the price of a good decreases quantity demanded increasesQ4ASSUMPTION of LAW OF DEMAND(i)No change in price of related commodities(ii) No change in income of the consumer(iii) No change in taste and preferences customs habit and fashion of the consumer( No expectation regarding future change in priceQ5MARKET DEMAND SCHEDULEIn economics a market demand schedule is a tabulation of the quantity of a good that all consumers in a market will purchase at a

given price At any given price the corresponding value on the demand schedule is the sum of all consumersrsquo quantities demanded at that priceQ6INDIVIDUAL DEMAND SCHEDULEIndividual demand schedule refers to a tabular statement showing various quantities of a commodity that a consumer is willing to buy at various levels of price during a given period of timeQ7 FACTORS AFFECTING INDIVIDUAL DEMAND FOR A COMMODITY

The factors that influence a consumerrsquos decision to purchase a commodity are also known as determinants of demand The following factors affect the individual demand for a commodity1 price of the commodity2 price of related goods3 income of buyer of the commodity4 tastes and preferences of the buyer1 Price of the CommodityYou must have observed that when price of a commodity falls you tend to buy more of it and when its price rises you tend to buy less of it when all other factors remain constant (lsquoother things remaining the samersquo) In other words other things remaining the same there is an inverse relationship between the price of a commodity and its quantity demanded by its buyers This statement is in accordance with law of demand which you will study in the later part of this lesson Price of a commodity and its quantity demanded by its buyers are inversely related only when lsquoother things remain the samersquo So lsquoother things remaining the samersquo is an assumption when we study the effect of changes in the price of a commodity on its quantity demanded2 Price of Related goodsA consumer may demand a particular good But while buying that good heshe also asks the price of its related goods Related goods can be of two types-(i) Substitute goods(ii) Complementary goods While purchasing a good prices of its substitutes and complements do affect its quantity purchased(i) Price of Substitute Goods Substitute goods are those goods which can easily be used in place of one another for satisfaction of a particular want like tea and coffee An increase in price of substitute good leads to an increase in demand for the given commodity and a decrease in price of substitute good leads to a decrease in demand for the given commodity It means demand for a given commodity is directly affected by change in price of substitute goods For example if price of coffee increases the demand for tea will rise as tea will become relatively cheaper in comparison to coffee(ii) Price of Complementary goods Complementary goods are those goods which are used together to satisfy a particular want like car and petrol An increase in the price of complementary goods leads to a decrease in demand for the given commodity and a decrease in the price of complementary goods leads to an increase in demand for the given commodity For example if price of petrol falls then the demand for cars will increase as it will be relatively cheaper to use both the goods together So demand for a given commodity is inversely affected by change in price of complementary goods3 Income of the Buyer of CommodityDemand for a commodity is also affected by income of its buyer However the effect of change in income on demand depends on the nature of the commodity under consideration In case of some goods like full cream milk fine quality of rice (Basmati rice) etc demand for these commodities increases when income of the buyer increases and

demand for these commodities decreases when income of the buyer decreases Such goods whose demand increases with the increase in income of the buyer are called normal goods But there are some goods like coarse rice toned milk etc whose demand decreases when income of buyer increases and their demand increases when income of the buyer decreases Such goods whose demand decreases with the increase in income of the buyer are called inferior goods Suppose a consumer buys 10 Kgs of rice whose price is ` 25 per Kg He cannot afford to buy better quality of rice because the price of such rice is ` 50 per Kg The consumer is spending ` 250 per month on the purchase of rice Now if income of the consumer increases and he can afford ` 350 on purchase of 10 Kg of rice Now he can afford to buy some quantity of rice say 6 Kgs whose price is ` 25 per Kg and may buy 4 Kgs of rice whose price is ` 50 per Kg Thus he will buy 10 Kgs of rice by spending ` 350 per month Therefore we may conclude that demand for normal goods is directly related to the income of the buyer but demand for inferior goods is inversely related to the income of the buyer4 Tastes and Preferences of the BuyerThe demand for a commodity is also affected by the tastes and preferences of the buyers They include change in fashion customs habits etc Those commodities are preferred by the consumers which are in fashion So demand for those commodities rises which are in fashion On the other hand if a commodity goes out of the fashion its demand falls because no consumer will like to buy it(5) Number of Buyers in the Market(Population)Increase in population raises the market demand whereas decrease in population reduces the market demand for a commodity Not only the size of population but its composition like age (ratio of males females children and old people in population) also affects the demand for a commodity It is because of needs of children young old male and female population differs(6) Distribution of Income and WealthIf the distribution of income and wealth is more in favour of the rich demand for the commodities preferred by the rich such as comforts and luxuries is likely to be higher On the other hand if the distribution of income and wealth is more in favour of poor demand for commodities preferred by the poor such as necessities will be more(7) Season and Weather ConditionsThis is generally observed that the demand for woolens increases during winter whereas demand for ice creams and cold drinks increases during summer Similarly market demand for umbrellas rain coats increases during rainy seasonQ8 REASONS FOR OPERATION OF LAW OF DEMAND WHY DEMAND CURVE SLOPES DOWNWARDNow we will try to explain why does a consumer purchase more quantity of a commodity at a lower price and less of it at a higher price or why does the law of demand operate ie why does the demand curve slope downwards from left to right The main reasons for operation of law of demand are1 Law of Diminishing Marginal UtilityAs you have studied earlier law of diminishing marginal utility states that as we consume more and more units of a commodity the utility derived from each successive unit goes on decreasing The consumer will be ready to pay more for those units which provide him more utility and less for those which provide him less utility It implies that he will purchase more only when the price of the commodity falls2 Income Effect

When price of a commodity falls purchasing power or real income of the consumer increases which enables him to purchase more quantity of the commodity with the same money income Let us take an example Suppose you buy 4 ice creams when price of each ice cream is ` 25 If price of ice creams falls to ` 20 then with same money income you can buy 5 ice creams now3 Substitution EffectWhen price of a commodity falls it becomes comparatively cheaper as compared to its substitutes (although price of substitutes has not been changed) This will lead to rise in demand for the given commodity For example if coke and Pepsi both are sold at ` 10 each and price of coke falls Now coke has become relatively cheaper and will be substituted for Pepsi It will lead to rise in demand for coke4 Change in Number of BuyersWhen price of a commodity falls some old buyers may demand more of the commodity at the reduced price and some new buyers may also start buying this commodity who were not in a position to buy it earlier due to higher price This will lead to increase in number of buyers when price of the commodity falls As a result demand for the commodity rises when its price falls5 Diverse Uses of a CommoditySome commodities have diverse uses like milk It can be used for drinking for sweet preparation for ice cream preparation etc If price of milk rises its use may be restricted to important purpose only This will lead to reduction in demand for other less important uses When price of milk falls it can be put to other uses also leading to rise n demand for itQ9 EXCEPTIONS TO THE LAW OF DEMANDYou have studied in law of demand that a buyer is willing to buy more quantity of a commodity at a lower price and less of it at a higher price But in certain circumstances a rise in price may lead to rise in demand These circumstances are called Exceptions to the Law of Demand Some important exceptions are1 Giffen GoodsGiffen goods are special type of inferior goods in which negative income effect is stronger than negative substitution effect Giffen goods do not follow law of demand as their demand rises when their price rises Examples of Giffen goods are jowar and bajra etc2 Status Symbol GoodsSome goods are used by rich people as status symbols eg diamonds gold jewellary etc The higher the price the higher will be the demand for these goods When price of such goods falls these goods are no longer looked at as status symbol goods and tehrefore therir demand falls3 NecessitiesCommodities such as medicines salt wheat etc do not follow law of demandbecause we have to purchase them in minimum required quantity whatever their price may be4 Goods Expected to be ScarceWhen the buyers expect a scarcity of a particular good in near future they start buying more and more of that good even if their prices are rising For example during war famines etc people tend to buy more of some goods even at higher prices due to fear of their scarcity in near future

Political Science

Constitution of India-The

Preamble

The preamble-

Preamble-

The preamble is the most precious part of the constitution We the people of India having solemnly resolved to constitute India into a Sovereign Socialist Secular Democratic Republic and to secure to all its citizensA preamble is an introductory and expressionary statement in a document that explains the documents purpose and underlying philosophy When applied to the opening paragraphs of a statute it may recite historical facts pertinent to the subject of the statuteNature and purpose of the constitution-Purpose of the Constitution dictates permanent framework of the government to form a more perfect union to establish justice and ensure peace of thenationconstitution provide principles how the government can run itself following the rules and laws written in the constitution of each state keeps them balanced

Answer the following questions-

1 What is preambleA preamble is an introductory and expressionary statement in a document that explains the documents purpose and underlying philosophy2 What is the nature and

purpose of the constitutionConstitution dictatespermanent framework of the government to form a more perfect union to establish justice and ensure peace of the nation

Homework-Learn

Accounts Compatibilty mode

1MEANING OF PARTNERSHIPPartnership is a form of business organisation where two or more persons join hands to run a business They share the profits and losses according to the agreement amongst them According to the Indian Partnership Act 1932 ldquoPartnership is relation between persons who have agreed to share profits of a business carried on by all or any one of them acting for allrdquo For example one of your friends has passed class XII from National Institute of Open Schooling (NIOS) and wants to start a business Heshe approaches you to join in this venture Heshe wants you to contribute some money and participate in the business activities Both of you if join hands constitute a partnership2CHARACTERISTICS1048698 Agreement A partnership is formed by an agreement The agreement may be either oral or in writing It defines the relationship between the persons who agree to carry on business It may contain the terms of sharing profit and the capital to be invested by each partner etc The written agreement is known as partnership deed1048698 Number of persons There must be at least two persons to form a partnership

The maximum number of partners in a partnership firm can be 50 according toCompanies Act 20131048698 Business The Partnership is formed to carry on business with a purpose of earning profits The business should be lawful Thus if two or more persons agree to carry on unlawful activities it will not be termed as partnership1048698 Sharing Profits The partners agree to share profits in the agreed ratio In caseof loss all the partners have to bear it in the same agreed profit sharing ratio10486981048698Mutual Agency Every partner is an agent of the other partners Every partner can bind the firm and all other partners by hisher acts Each partner will be responsible and liable for the acts of all other partners10486981048698Unlimited liability The liability of each partner except that of a minor is unlimited Their liability extends to their personal assets also If the assets of the firm are insufficient to pay off its debts the partnersrsquo personal property can be used to satisfy the claim of the creditors of the partnership firm10486981048698Management All the partners have a right to mange the business However they may authorize one or more partners to manage the affairs of the business on their behalf10486981048698Transferability of Share No partner can transfer hisher share to any one including hisher family member without the consent of all other partners3PARTNERSHIP DEEDAgreement forms the basis of partnership The written form of the agreement is which a document of partnership is It contains terms and conditions regarding the conduct of the business It also explains relationship between the partners This document is called partnership deed Every firm can frame its own partnership deed in which the rights duties and liabilities of the partners are stated in detail It helps in settling the disputes arising among the partners during the general conduct of business 4CONTENTS OF PARTNERSHIP DEEDThe partnership deed generally contains the following (i) Name and address of the partnership firm(ii) Nature and objectives of the business(iii) Name and address of each partner(iv) Ratio in which profits is to be shared(v) Capital contribution by each partner(vi) Rate of Interest on capital if allowed(vii) Salary or any other remuneration to partners if allowed(viii) Rate of interest on loans and advances by a partner to the firm(ix) Drawings of partners and interest thereon if any(x) Method of valuation of goodwill and revaluation of assets and liabilities on the reconstitution of the partnership ie on the admission retirement or death of a partner(xi) Settlement of disputes by arbitration(xii) Settlement of accounts at the time of retirement or death of a partner5IN ABSENCE OF PARTNERSHIP DEEDThe partnership deed lays down the terms and conditions of partnership in regard to rights duties and obligations of the partners In the absence of partnership deed there may arise a controversy on certain issues like profit sharing ratio interest on

capital interest on drawings interest on loan and salary of the partners In such cases the provisions of the Indian Partnership Act becomes applicableSome of the Issues are(i) Distribution of Profit Partners are entitled to share profits equally(ii) Interest on Capital Interest on capital is not allowed(iii) Interest on Drawings No interest on drawing of the partners is to be charged(iv) Interest on Partnerrsquos Loan A Partner is allowed interest 6 per annum on the amount of loan given to the firm by himher(v) Salary and Commission to Partner A partner is not entitled to anysalary or commission or any other remuneration for managing the business

History TOPIC-TOWARDS INDEPENDENCE AND PARTITION THE LAST PHASE (1935-1947)

SUB TOPIC-IMPORTANT POLITICAL DEVELOPMENTS ndash GROWTH OF SOCIAL IDEAS

Socialism is a political social and economic philosophyLike in other parts of the world the Russian revolution of 1917 served as a great inspiration for revolutionaries in India who at that time were engaged in the struggle for liberation from British ruleSocialist ideas led to the formation of communist party of IndiaJAWAHARLAL NEHRU Among the early Congress leaders Jawaharlal Nehru was very much impressed and influenced by the Socialist ideas He also learnt about the Economic activities of the Soviet Union after the Bolshevic Revolution 1917 He made full use of them in IndiaThe election of Jawaharlal Nehru and Subhas Chandra Bose showed the Left wing tendency within CongressJawaharlal Nehru demanded economic freedom along with political freedom of the people in order to end the exploitation of masses

Nehrus working committee included three socialists leaders The Lucknow session was a landmark in the evolution of socialist ideas of the congressSUBHAS CHANDRA BOSE ndash Subhas Chandra Bose had socialist leaning Both Jawaharlal Nehru and Subhas Chandra Bose were known as leftist Congress men Later on National Congress divided into Leftist and rightist campCONGRESS SOCIALIST Within the Congress some leaders formed the Congress Socialist partyPattavi Sitaramyya Sardar Patel Rajendra Prasad had hostile attitude towards the Congress Socialist partyJawaharlals attitude was hesitant

1 QUESTION ndash Mention name of two Congress leaders who had socialist leaning

1ANSWER ndash Subhas Chandra Bose and Jawaharlal Nehru2QUESTION- In which session of the congress Jawaharlal elaborated his Socialist ideas2 ANSWER ndash Lucknow and Faizpur Session in December 1935 and 19363QUESTION ndash Why Congress was sharply divided into leftist and rightist camp 3ANSWER ndash Subhas Chandra Bosersquos attempt to seek re election for congress presidentship in 1939sharply divided the National Congress into Leftist and Rightist camp4 QUESTION ndash Who was MN Roy 4 ANSWER ndash Manabendra Roy first formed the Communist Party of India outside the country at Tashkent in 19205QUESTION ndash Who formed the Congress Socialist Party within the Congress5 ANSWER ndash Jaya Prakash Narayan Achyut Patwardhan Acharya Narendra Dev Ram Mohan Lohia Aruna Asaf Ali6QUESTION ndash When was the Congress Socialist Party formed What was its object6 ANSWER ndash 1934The Congress Socialist Party sought to work out socialist programme through the Congress They joined hands with the Congress and wanted to carry

Subhas Chandra Bose being expelled from the congress after the Tripuri rift he formed Forward BlockThere were basic differences between the Congress Socialists and the communistsTRADE UNION ACTIVITIES Maximum working class people lived in Bombay and Calcutta The working and living conditions of those workers were very miserable In this situation Shasipada Banerjee NM Lokhande protested against the oppression of the working class peopleThe first Trade Union Madras Labour Union was formed in 1918 by BP WadiaIndustrial strikes took place in Kanpur Calcutta Madras Jamshedpur and Ahmedabad AITUC was formed in Bombay in 1927 The growth of Trade union among the workers was slow because of the fear of the dismissal of the jobIn the mean time the Moderates as well as Communists left AITUC and formed separate organization

on National struggle with the help of workers and peasant class of the society7 QUESTION ndash What was the name of the party founded by Subhas Chandra Bose7 ANSWER- Forward Block8QUESTION ndash Who was Shasipada Banerjee8 ANSWER ndash Shasipada Banerjee was a radical Brahmo He founded a working menrsquos club to protest against exploitation of the British rulers towards the working class of India9 QUESTION ndash What was the weekly published by NM Lokhande9ANSWER- Dinabandhu10 QUESTION ndash Who founded Bombay Mill-Hands Association and in which year10 ANSWER- NM Lokhande in189011 QUESTION- Who was BP WadiaANSWER- BPWadia was the founder of Madras Labour Union in191812 QUESTION- What was the name of the first labour union of India12 ANSWER- Madras Labour Union13 QUESTION Who founded the Majur Mahajan 13 ANSWER GANDHIJI14 QUESTION What was the full form of AITUC When it was formed14 ANSWER All India Trade Union Congressin 192715QUESTION Who formed the Red Trade Union Congress and in which year15ANSWER The Communists formed the Red Trade Union Congress16 QUESTION What do you mean by Socialism16 ANSWER Socialism describes any political and economic theory that says the community rather than individuals should own and manage property and natural resources

Subject Eng Literature (The Tempest ndash William Shakespeare) Topic Act III Scene 3 Lines 1 to 52 (Line 52 ndash Brother my lord the Duke Stand to and do as we) Date 13th April 2020 (4th Period)

[Students should read the original play and also the paraphrase given in the school prescribed textbook]Summary Questions amp Answers

o Alonso Sebastian Antonio Gonzalo Adrian Francisco and others wandered about the island in search of Ferdinand and gets tired and hungry of the toil and at the same time gives up all hope of finding him

o Antonio and Sebastian are happy that Alonso is out of hope and decide to make another attempt on his life that night when being so tired they will be sleeping soundly

o Suddenly a solemn and strange music is heard in the air and several strange shapes enter bringing in a banquet These strange shapes then dance round it with gestures of salutation and then inviting the King to eat they depart

o Seeing this strange scene all are inclined to believe the tales told by travelers that there truly are ldquounicornsrdquo and ldquothe phoenixrsquo thronerdquo

1 ALONSO What harmony is this My good friends hark (L18-27)

GONZALO Marvellous sweet music

[Enter several strange shapes bringing in a banquet

they dance about it with gentle actions of salutation

and inviting the King and his companions to eat they depart]ALONSO Give us kind keepers heavens What were theseSEBASTIAN A living drollery Now I will believe

That there are unicorns that in Arabia

There is one tree the phoenixrsquo throne one phoenix

At this hour reigning thereANTONIO Ill believe both

And what does else want credit come to me

And Ill be sworn rsquotis true Travellers neer did lie

Though fools at home condemn rsquoem

(a) How did Prospero present an amazing spectacle before Alonso and his companions

Using his magic powers Prospero ordered strange shapes to lay a banquet before Alonso and his companions The shapes brought several dishes with tasty eatables in them They placed the dishes on a table before Alonso and his companions Then the strange shapes began to dance gracefully around the banquet While dancing they made gestures inviting them to eat the food Then suddenly the shapes disappeared(b) Who were the guests at the strange banquet Describe the lsquoliving drolleryrsquo

Alonso Sebastian Antonio Gonzalo Adrian and Francisco were the guests at the strange banquet

The term ldquoliving drolleryrdquo refers to live entertainment show In this context when Alonso the King of Naples Sebastian his brother Antonio the treacherous brother of Prospero Gonzalo the kind and loyal councillor to the King Adrian and Francisco came to the island they were hungry and weary in their spirits They heard a solemn and strange music They were shocked to see several strange shapes bringing in a banquet and these shapes danced about it with gentle action of salutation inviting the King and his companions to eat After this Sebastian described this show as lsquoliving drolleryrsquo(c) What is lsquophoenixrsquo What are lsquoUnicornsrdquo

The term lsquophoenixrsquo refers to a mythical Arabian bird which lived alone and perched on a solitary tree After one hundred years it expired in flames and rose again from its own ashes

lsquoUnicornsrsquo refers to the mythological four-footed beasts having horns in the centre of their foreheads When the horns are ground into powder the powder was believed to be

an aphrodisiac(d) How does Sebastian explain the puppet show OR Why does the speaker now believe in unicorns and phoenix

Sebastian finds several strange shapes bringing in the banquet They invite the king and his party for dinner and soon depart He tells that if such a strange sight can be a reality there is nothing incredible in the world and from the present moment he will believe anything He says that it is a strange dumb show enacted not by puppets but by living beings It is stranger than a travellerrsquos tale Seeing such a thing

before his own eyes he will no longer disbelieve the story about unicorns and phoenix(e) How do the other characters present respond to this living drollery

At the sight of the lsquoliving drolleryrsquo like Sebastian Gonzalo and Antonio too acted strangely Antonio told that he too now believes in unicorns and phoenix and anything else that seems to be incredible He too now believes in travellersrsquo tales Gonzalo told that if he would report those happenings in Naples nobody will believe him He considers that those gentle shapes were gentler in manner in comparison to the living beings Alonso was at first sight suspicious and told them that those strange shapes conveyed their meaning in expressive gestures when they seemed to lack speech by their movements and sounds Francisco was amazed at their mysterious disappearance

2 ALONSO Not I

(Line 43-52)GONZALO Faith sir you need not fear When we

were boysWho would believe that there were mountaineers

Dewlapped like bulls whose throats had hanging at rsquoem

Wallets of flesh Or that there were such men

Whose heads stood in their breasts Which now we find

Each putter-out of five for one will bring us

Good warrant ofALONSO I will stand to and feed

Although my lastmdashno matter since I feel

The best is past Brother my lord the Duke

Stand to and do as we

(a) How does Alonso respond at the spectacle of the shapes which were sent to them at the instruction of Prospero

After seeing the strange sight of appearing and disappearing of the shapes sent by Prospero to arrange a banquet for them Alonso says that his surprise at having seen those creatures is infinite and he is fully justified in feeling so much surprise He thinks that their shapes their gestures and the sounds they made were indeed amazing Although they do not possess the gift of speech yet they were able to convey their

thoughts by means of their gestures only

(b) What does Prospero say about the views expressed by Alonso regarding the shapes What does Francisco think about the shapesAfter hearing Alonsorsquos views about the shapes Prospero says that this manrsquos praise of the spirits is rather hasty He means to say that Alonso has shown great haste in reaching the conclusion about the shapes Francisco is amazed to see that those shapes disappeared in a mysterious way(c) What does Sebastian ask Alonso to doSebastian tells Alonso that the shapes having disappeared should not matter to them because they have left the eatables behind He asks Alonso to enjoy eating as they are extremely hungry but the king does not accept his offer of enjoying the dishes(d) How does Gonzalo try to dispel Alonsorsquos fear of those strange shapes What kind of references does he give to AlonsoGonzalo says that those who have travelled abroad have reported seeing even stranger sights than these shapes that Alonso and his companions have beheld Hence there is no reason to feel afraid of these shapes Gonzalo further adds that in his younger days he had heard strange stories from travelers and Alonso might have heard similar stories For instance it was said that there existed a certain race of

human beings who had huge lumps of flesh hanging at their throats and who therefore resembled bulls Then Gonzalo tells about a race of human beings whose heads were located at their breasts Gonzalo says that such stories were not believed by most people in those days but now-a-days these stories have become common(e) Explain the following lsquoEach putter-out of five for onersquoEnglish travellers often insured their trips with London brokers Those that went on foreign travels those days used to deposit a certain amount with some firm or company in London before their departure If the travelers failed to return the money was forfeited by the company with which it had been deposited But this money was repaid five-fold if the travelers returned safe and sound In this way a traveler stood a great chance of recovering the entire cost of his

travels(f) Give the explanatory meanings of the following expressions in the context of the above extract (i) Dewlapped (ii) Wallets of flesh

(iii) Putter-out(i) Dewlapped having big lumps of flesh at the necks(ii) Wallets of flesh large masses of flesh looking like bags(iii) Putter-out to invest money before commencing the travel

  • General methods of preparation of hydrogen
  • Chapter Dimensional Analysis (Summary)
    • Properties of Charge
Page 11:   · Web viewSubject. Topic. Summary. Execution. Hindi. व्याकरण. शरीरके अंगो के नाम लिखिए. 1) आँख 2) नाक 3

symbol Not more than one solidus is used For example m s-1 or m s J K mol or J K-1 mol-1 but not J K mol8 Some space is always to be left between the number and the symbol of the unit and also between the symbols for compound units such as force momentum etc For example it is not correct to write 23m The correct representation is 23 m kg m s-2 and not as kgms-29 Only accepted symbols should be used For example ampere is represented as A and not as amp or am second is represented as s and not as sec10 Numerical value of any physical quantity should be expressed in scientific notationFor an example density of mercury is 136 x 104 kg m-3 and not as 13600 kg m-3

3) Second is the unit of time in both the CGS and MKS systems True

4) The symbol used for a unit is always written in capital letters False Correct statement -The symbol used for a unit is normally written in small letters

Hindi 2nd language

वाकय निवचार भागवत निवचारो को परकट करन वा साथक एव वयवसथिtत शबद समह को वाकय कहत ह वाकय दो परकार क होत ह ndash

1 उददशय- वाकय म जिजसक बार म कछ बताया जाता ह उस उददशय कहत ह जस राधा एक नतकी ह2 निवधय- वाकय म उददशय क बार म बताया जाता ह उस निवधयक कहत ह जस- राधा एक नतकी ह रचना क आधार पर वाकय क तीन भद होत ह ndash१सर वाकय- राम बाजार गया २ सयकत वाकय- राम बाजार गया और वहा जाकर दोसत स मिमा३ मिमशर वाकय- यह वही tान ह जहा उनका बचपन बीता

helliphellipContinue to nextBengali 2nd language

লিZ সবরপ ও সবরলিZ

সবরলিZর লিয়ম - ১ অ-কোর লিকংবো আ-কোসরর পসর অ-কোর লিকংবো আ - কোর োকস উভয় লিমস আ ndashকোর য় এবং ওই আ ndash কোর পব13বস13 Pকত য়

২ ই - কোর লিকংবো ঈ - কোসরর পসর ই - কোর লিকংবো ঈ - কোর োকস উভয় লিমস ঈ - কোর য় এবং ওই ঈ - কোর পব13বস13 Pকত য়

৩ উ - কোর লিকংবো ঊ - কোসরর পসর উ - কোর লিকংবো ঊ - কোর োকস উভয় লিমস ঊ - কোর য় এবং ওই ঊ - কোর পব13বস13 Pকত য়

৪ অ - কোর লিকংবো আ ndash কোসরর পসর ই - কোর লিকংবো ঈ - কোর োকস উভয় লিমস এ - কোর য় এবং ওই এ - কোর পব13বস13 Pকত য়

১ অ + অ = আ ( gtো ) লিম + অচ = লিমোচ সব + অ3ী = সবো3ী অ + আ = আ ( gtো )পদম + আ = পদমো শভ + আলি = শভোলি আ + আ = আ ( gtো )4োয়ো + আবত = 4োয়োবত মো + আতমো = মোতমো আ + অ = আ ( gtো )লিবদযো + অংকোর = লিবদযোংকোর Pো + অ13 = Pো13 ২ ই + ই = ঈ ( gtী )অলিত + ইব = অতীব লিগলির + ইনদর = লিগরীনদর ই + ঈ = ঈ ( gtী )পলির + ইকষো = পরীকষো অলি3 + ঈশবর = অ3ীশবর ঈ + ঈ = ঈ ( gtী )মী + ঈশবর = মীশবর 3ী + ঈ = 3ী ঈ + ই = ঈ ( gtী )রী + ইনদর = রীনদর মী + ইনদর = মীনদর ৩ উ + উ = ঊ ( gt )মর + উদযো = মরদযো কট + উলিকত = কটলিকত উ + ঊ = ঊ ( gt )ঘ + ঊলিম13 = ঘলিম13 লিZ + ঊলিম13 = লিZলিম13 ঊ + ঊ = ঊ ( gt )রP + ঊলিম13 = রPলিম13

৪ অ + ই = এ ( লেgt )র + ইনদর = সরনদর লেPোগ + ইনদর = লেPোসগনদর অ + ঈ = এ ( লেgt )গ + ঈ = গস

র + ঈ = সর আ + ই = এ ( লেgt )Pো + ইষট = Pসষট 3ো + ইনদ = স3নদ আ + ঈ = এ ( লেgt )রমো + ঈ = রসম দবোরকো + ঈশবর = দবোরসকশবর

COMPUTER THE WORLD OF WINDOWS 10

DONE IN THE PREVIOUS CLASS PAGE NO-83A TICK THE CORRECT OPTION BACKGROUND DISPLAY AREA RESTORE THREE

MATHEMATICS Topic ndash NumbersChapter ndash Natural numbers and whole numbers

Study item Properties of whole numbers for subtraction1) Closure property When we do subtraction of two whole numbers we can not get a whole number in all time Example 8 ndash 3 = 5 a whole number 0 ndash 6 = -6 is not a whole numberTherefore the subtraction of two whole numbers is not satisfying closure property2) Commutative property If x and y are two whole numbers then x ndash y is not equal to y ndash xExample If x=16 and y = 7 then x ndash y = 16 ndash 7 = 9Again y ndash x = 7 ndash 16 = - 9 Therefore x ndash y not equal to y ndash x Therefore the subtraction of two whole numbers is not satisfy commutative3) Associative property If x y and z are three whole numbersThen x ndash ( y ndash z ) not equal to ( x ndash y ) ndash z Example If x = 20 y = 10 and z = 6Therefore x ndash (y ndash z ) = 20 ndash(10 ndash 6 ) = 20 ndash 4 = 16(X ndash y ) ndash z = (20 ndash 10) ndash 4 = 10 -4 =6Therefore x ndash(y ndash z) not equal to ( x ndash y) ndash zTherefore subtraction of whole numbers is not satisfying associativity4) Distributive property If x y and z are three whole numbersThen x (y ndash z ) = xy ndash xzAnd (y ndash z)x = yx ndash zxExample If x = 10 y = 6 and z = 4x(y ndash z ) = 10(6 ndash 4 ) = 10times6 ndash 10times4 = 60 ndash 40 = 20( 6 ndash 4 )times 10 = 6times10 ndash 4times10 = 60 ndash 40 = 20Therefore the subtraction of whole numbers is satisfying distributive property5) Existence of identity For any whole number x X ndash 0 = x but 0 ndash x = - x not equal to xThus for subtraction no identity number existsException 0 ndash 0 = 0 so 0 is its own identity for subtraction

Class VIISubject Topic Summary Execution

Hindi 2ndlang वचन जो सजञा शबद निकसी वसत या पराणी क एक या अनक होन का बोध कराया उनह वचन कहत ह जस डका- डकयह दो परकार की होती ह-

क) एकवचन-शबद क जिजस रप स उसक एक होन का बोध हो उस एक वचन कहत ह जस निकताब गमा आदिद

) बहवचन-शबद क जिजस रप स उसक आन ोन का पता च उस बहवचन कहत ह जस डक निकताब निततलियाआदिद

निनमनलिखित शबदो को एकवचन स बहवचन म बदोम- हमजानित- जानितयानारी- नारिरयामिमतर ndashमिमतरोपसतक -पसतकसडक-सडकबोत-बोतनाहर-नहररपए-रपया

Bengali বইndashবোংো োলিতয পলিরচয়

পোঠndash১৪ গলপ - অপর কলপো পর

লেক - লিবভলিতভষ বসনদযোপো3যোয়লেকndash রবীনদর পরবতf বোংো কোোলিতয 3োরোর উসgসPোগয োম পরকলিতসপরমী লিবভলিতভষ বসনদযোপো3যোয় তোর লেীসত লেPম বোসর বোসর লিফসর এসস4 গরোম বোংোর পরকলিতর কো লেতমলি এসস4 গরোমী মো লিচতরগলপndash অপর কলপো গলপোংটি লিবভলিতভষ বসনদযোপো3যোসয়র লিবযোত উপযো পসর পাোচোী লেসক গীত অপ অ13োৎ পসর পাোচোী তো অপর কলপোর লেকনদরীয় চলিরতর এই অংস আমরো পোই বোক অপসক বোক অপ কলপো লিবোী লে দসরর অ গো4 লেদস মোসয়র মস লেোো রপকোর রোসয পোলি2 লেদয় দপরসবো মোসয়র মসর কসর কোীদোী মোভোরত এর করসকষতর Pসjর ব13ো শস তোর মোবীর কস13র পরলিত ব2 মমতো য় আবোরপালিসত বলি13ত Pসjর অমোপত অং লে লিসই মোপত কসর বোলি2র লিপ4স বাো বোগোস লিকংবো উঠোসর লিশমসর কলপো লিবো এোস পরকো লেপসয়স4

১ অপর কলপো গসলপর লেক লেক তোর মপসক13 লেসো২ অপর কলপো গলপটি লেকোো লেসক গীত গলপটির ম ভোব লেসো

GEOGRAPHY CHAPTER 7EUROPE

CHAPTER COMPLETE EXERCISEFill in the blanks1 Europe is a continent that comprises the western part of Eurasia2 Eurasia and Africa are connected into one large land mass known as Afroeurasia3 The Strait of Gibraltar separates Europe and Africa4 Europe is surrounded by the Arctic Ocean to the north5 The British Isles includes the island countries of Great Britain and Ireland

Name the following 1 Connects Africa to Eurasia - Isthmus of Suez2 Largest country in the world in terms of area ndash Russia3 A term used collectively for the five countries in northern Europe ndash Nordic Countries4 The capital of Montenegro - Podgorica5 the largest fjord in Norway ndash Sognefjord

Match the following Column 1 Column 2a Albania iii Tiranab Belgium i Brusselsc Denmark v Copenhagend Finland ii Helsinkie Hungary iv Budapest

CHEMISTRY Chapter 2 ndashElement and Compound

Atom - An atom is the basic unit of an element or the smallest particle of an element non capable of independent existence Atom is built up of three sub atomic particles electron proton and neutron

Nucleus-It is the centre of an atom In the centre of the atom contains proton (positively charged particles ) and neutrons ( particles carrying no charge )

Orbits- It surround the nucleus in which revolve electrons (negatively charged particles)

Answer the following

1) What are MetalloidsAns - Certain elements using properties of both metal and non-metals are called metalloids Example Silicon arsenic and antimony

2) What are Noble gasesAns - Certain elements are present in the air and are chemically inert or unreactive Such elements are called rare gases or noble gases Example helium neon argon and Krypton

English 2 Sentences based on meanings

Kinds of sentences

Assertive or declarative to convey information or simply make a statement

Interrogative to ask different types of questions

Imperative to command or instruct someone or make a request

Exclamatory to express strong feelings and emotions

Exercise B1 Stop it ( Exclamatory)2 May you always be happy

together ( Exclamatory)3 He does not like sports

( Assertive)4 Please pass me the salt

( Imperative)5 How dare she talk to me like

that ( Exclamatory)6 May success bless your effort

( Exclamatory)7 Canrsquot you wait for sometime

(Interrogative)8 Did anybody tell you about it

( Interrogative)9 I saw her waiting for the bus

( Assertive)10 Could you please take a

message for me ( Interrogative)

Homework Ex ABiology Chp -2

Classification of Plants

Today we discuss about usefulness of bacteria We also discuss what the harmful effects of bacteria are

89 How bacteria are useful for usbull Bacteria is helpful in many ways forhuman being i) Production of medicine - antibiotics vaccine etcii) Formation of curd by lactobacillusiii)Nitrogen fixation in Leguminousplant by Rhizobiumiv) Increase soil fertility by absorbingatmospheric nitrogen and convert it into nitrates and nitritesv) Cleaning the environment by converting the complex substances into simple substancesvi) Tanning of leathervii) Retting of Fibersviii) Formation of compost by acting onanimal dung and agricultual cases1x) Biogas production by decomposingplant and animal wastex)Help In Nutrition by producing vitamiacutemBand kx1) Some bacteria are used to give specialflavour to tea coffee and coccaQ10- Name some diseases and there causativebacteriabull Diseasescausative bacteria1 CholeraVibrio cholerae2 Tuberculosis - Mycobacterium tuberculosis3 Diptheria -Corynebacteriumdiphtheriae4 Pneumonia - Streptococcus pneumoniae

Math Number system

Chapter Fraction

Study item Using lsquoofrsquoThe word lsquoofrsquo between any two fractions is to be used as multiplicationExample 57 of 56 = 57 times 56 = 5times8 = 40Study item Using BODMASThe word lsquoBODMASrsquo is the abbreviation formed by taking the initial letters of six operations(i)Bracket (ii) of (iii) Division (iv) Multiplication (v) Addition (vi) SubtractionAccording to BODMAS rule First of all the terms inside Bracket must be simplified then lsquoofrsquo lsquoDivisionrsquo lsquoMultiplicationrsquo lsquoAdditionrsquo lsquosubtractionrsquo

Study item Removal of Brackets

There are four Brackets of algebra in Mathematics In a complex expression four types of brackets are used Order of removing the brackets is first ----- then ( ) then finally [ ]

Class VIIISubject Topic Summary Execution

Chemistry Hydrogen General methods of preparation of hydrogen

By the action of dilute acids on metals

Calcium Reacts readily to form chloride salt and hydrogen

Ca + 2HCl rarr CaCl₂ + H₂uarr

Magnesium

Aluminium

Zinc

React readily to form salt and hydrogen

Mg + 2HCl rarr MgCl₂ + H₂uarr2Al + 6HCl rarr 2AlCl₃ + 3H₂uarrZn + 2HCl rarr ZnCl₂ + H₂uarr

Question 4 ) Give reasons for the following

(a) Hydrogen be used as a fuel

Solution

Hydrogen is used as a fuel because it has a high heat of combustion Some significant fuels are coal gas water gas and liquid hydrogen

(b) Though hydrogen is lighter than air it cannot be collected by downward displacement of air

Solution

Hydrogen is lighter than air so it is possible to collect the gas by downward displacement of air But it is not safe to do so since a mixture of hydrogen and air can lead to an explosion

(c) A pop sound produced when hydrogen is burnt

Solution

Impure hydrogen gas burns in air with a pop sound This is because of the presence of impurities in it

(d) Helium replaced hydrogen in weather observation balloons

Solution

It forms a mixture with air that can explode when there is a small leakage of hydrogen in a balloon So helium has replaced hydrogen

(e) Nitric acid not used for the preparation of hydrogen gas

Solution

(e) By the action of nitric acid on metals hydrogen cannot be produced because it also releases nitrous oxide and nitric oxide and oxides the hydrogen to form water

Biology Chp-2 Reproduction in plants

Today we discuss different methods of artificial propagation like cutting-rose sugercane Layering ndashguava lemon china rose etc Grafting- mango apple etcMicropropagation ndashorchid asparagus etcWe also discuss about advantages and disadvantages of vegetative propagation

Q7 Define the following terms i) Explant In tissue culture techniquea tiny piece of bud shoot or any other partof plant from where new tissue develop ii) Callus The cells of the tissue divide andgrow into a mass of undifferentiated cells from explant iii) Plantlet After few days callus differentiate into a small plant with roots and shootQ8 what are the advantages and limitations of tissue culture or micropropagation

Advantages i ) It produacuteces superior quality plantsii)It can be applied to interspecifie hybridsiii) It is useful to grow seedless plants bull Limitations i) It cannot be used for all plantsii)It is not easy to handleQ9 Write advantages of vegetative propagationi) It is a quick and easy method ofproducing new plantsii) This method need less time to matureiii) The new plants are exact copies of the parentiv) it is extremly useful for growing seedlessplants like banana grapes etc Q10 Write some disadvantages of vegetativepropagationi) Dišeases present in the parent plant gettransferred to all in new plantsii) Overcrowding of new plants causes competition for sunlight water and nutrients which affects growth of plantsplant

Physics Chapter 2 Physical Quatites and Measurements

Here We Will Do Some QuestionsRelated To Chapter 2

Select the correct alternative A block of wood of density 08gcm-3 has a volume of 60cm3 The mass of the block is

1 608 g

2 75 g

3 48 g

4 0013 g

Solution 348 g

The density of aluminium is 27g and that of brass The correct statement is

1 Equal masses of aluminium and brass have equal volumes

2 The mass of a certain volume of brass is more than the mass of an equal volume of aluminium

3 The volume of a certain mass of brass is more than the volume of an equal mass of aluminium

4 Equal volumes of aluminium and brass have equal masses

Solution 2 The mass of a certain volume of brass is more than the mass of an equal volume of aluminium

MATHEMATICS Ch 6Sets

Exercise 6(C)1 Find all the subset of each the following sets(i) A = 57 (iii) C = x xisin W x le 2(iv) p p is a letter in the word lsquopoorrsquo

Solution (i) All the subsets of A are ϕ 5 7 57

(iii) All the subsets of C are ϕ 0 1 2 01 02 12 012

(iv) All the subsets are ϕ p o r po or por

4 Given the universal set = -7-3-105689 find (i) A = x xlt2 (ii) B = x -4ltxlt6 Solution

(i) A = -7-3-10(ii) B = -3-105

5 Given the universal set = x xisin N and xlt20 find

(i) A = x x = 3p pisin N (iii) C = x x is divisible by 4 Solution

(i) 369121518 (iii) 481216

6 Find the proper subset of x x2-9x-10 = 0 Solution

ϕ 10 -1

Working x2-9x-10 = 0 rArr x2-(10-1)x-10 = 0

rArr x2-10x+x-10 = 0 rArrx(x-10)+1(x-10) = 0

rArr (x+1) (x-10) = 0

11 Let M = letters of the word REAL and N = letters of the word LARE Write sets M and N in roster form and then state whether (i) M sube N is true (ii) N sube M is true (iii) M = N is true

Solution M = real and N = lareSo (i) Yes (ii) Yes (iii) Yes

English 2 Twelfth Night ndash Shakespeare

A noble man named Orsino in the kingdom of Illyria is deeply in love with a lady called lady Olivia She is in mourning for her dead brother so she will not even think about marriage At this time a sea storm causes a terrible shipwreck and a young lady called Viola is swept onto the shore She thinks that her twin brother Sebastian is drowned A sea captain tells her about Orsino and his love for Olivia Viola wishes to work in Oliviarsquos home but feels she will not be employed So she dresses as a man calls herself Cesario and gets work at the house of OrsinoViola (now Ceasario) is much liked by Orsino and becomes his page She falls in love with Orsino Orsino sends Ceasario to deliver messages to Olivia Olivia herself falls for the beautiful young Ceasario believing Viola to be a man

2 Answer the following questionsa Why does Orsino ask the musicians to play onOrsino asks the musicians to play on because music feeds his desire He calls upon the musicians to play music so that his hunger for love could be replenished with an excess of musicb What does Valentine tell about OliviaWe learn from Valentine that Olivia is in mourning for her brother she wears a veil and has vowed that no one will see her face for another seven yearsand she refuses to marry anyone until thenc From the exchange between Orsino and Valentine what do you think their relationship isValentine is one of orsinod attendants He was sent to Olivia as a messenger of love but was not allowed to speak to here Who is Olivia mourning for and whyOlivia is mourning for her dead brother

Homework Q fHistory and Civics

Growth of Nationalism

Important dates to remember1769-Napoleon born on 15thAugust1789-Fall of Bastille on 14th July and the beginning of the French revolution declaration of the rights of Man on 26thAugust1793-King Louis XVI executed on January 211764-The Sugar Act passed1765-The Stamp act passed1774-The first congress of Philadelphia1776-The declaration of American Independence of on 4th July1777-Defeat of the British at Saratoga1781-Surrender of lord Cornwallis at Yorktown1783-The treaty of Versailles1804-Napoleon becomes the emperor1813-Battle of Leipzig or Battle of nations in which Napoleon was defeated by the Allies1815-Battle of Waterloo June 18 in which Napoleon was defeated and captured1821-Death of Napoleon in StHelena1860-Abraham Lincoln elected President of the USA1861-The civil war began 1864-Abraham Lincoln elected President of the USA for the second time1865-Slavery abolished in the US

Name the following- The queen of Louis XVI

Marie Antoinette The three philosophers of France

VoltaireMontesquieuJean Jacques Rousseau

The British general whose surrender brought the war in America to an endLord Cornwallis

The first president of the USAGeorge Washington (1732-1799)

The first southern state to secede from the unionSouth Carolina

The author of the book lsquoUncle Toms CabinHarriet Beecher Stowe

Homework-Learn

Class IXSubject Topic Summary Execution

Economics

Types of economies Today I am going to share you the concept of economic growth and economic development Few questions will be given from the previous study material dated 942020

Meaning of economic growthAnswer) The term economic growth generally means anincrease in national income or per capita output or income over time It indicates towards quantitative growth of a country

Meaning of economic developmentAnswer) Economic development is defined

as a process whereby the real per capita income of a country increases over time along with fall in poverty ratio unemployment and income inequality etc

Distinguish between economic growth and economic development

Basis Economic growth

Economic development

Scope It has narrow scope as it refer only to rise in per capita income

It has wide concept since it includes qualitative changes as well

Concerned matter

It is concerned with the rise in income

It is concerned with not only rise in income but also reduction of poverty income inequality and unemployment

Focus Economic growth does not focus on economic development

Economic development focus on economic growth plus qualitative changes

Distinguish between capitalist economy and socialist economy

Ownership

Motive

Tool

Means of production are owned and managed by private people

Self interest and profit earning is the main motive

Price mechanism is a main tool to solve the economic problems

Means of production are owned and managed by the government

Social welfare is the main motive

Economic planning by the government is the main tool to solve the economic problem

Competition

Distribution of income

There exist large competition among buyers and sellers

There is existence of large inequalities of income

There is no such competition

There exist less inequalities of income

Math Topic ndash AlgebraChapter -Factorisation

Study item Factorising by taking out common factorSome solved sums from exercise 41

1) (i) 8xy3 + 12x2y2

= HCF of 8xy3 and 12x2y2 is 4xy2

= 4xy2(2y + 3x )

4) (ii) 28p2q2r ndash 42pq2r2

= HCF of 28p2q2r and 42pq2r2 is 14pq2r = 14pq2r (2p - 3r )5) (ii) 14mn + 22m - 62p=HCF of 14mn 22m and 62p is 2= 2(7mn + 11m - 31p)7) (ii) 3a(x2 + y2) + 6b (x2 + y2) = HCF of 3a(x2 + y2) and 6b(x2 + y2 ) is (x2 + y2)= ( x2+ y2 )(3a + 6b )9) (ii) x(x2 + y2 ndash z2 ) + y(-x2ndashy2 + z2 ) ndash z(x2+ y2 ndash z2 )= x(x2 + y2 -z2) ndash y-(x2 + y2 -z2) -z(x2 + y2 ndash z2)=x(x2 + y2-z2) -y( x2 + y2-z2) ndash z (x2 + y2 -z2)= (x2+ y2 ndash z2)(x ndash y ndash z )

Commercial Studies

Introduction to Accounting and Book-keeping

Today I am going to share you the meaning of Accounting and Book-keeping and its related terms bullAccounting bullBook Keeping bullAccountsbullTypes Of Accounts bullAccounting Cycle

bull Meaning of accounting

Ans )Accounting is the art and science of recording classifying and summarising monetary transactions

bull Meaning of Book-keeping

Ans) Bookkeeping is the art of recording business transactions with the view of having a permanent record of them and showing their effect on wealth

bull Meaning of account

Ans) The term account means a record of business transactions concern a particular person of firm asset or income or expense It is a summarised record of all transactions which take place in an accounting year

bull Types of accountsPersonal accounts ndash Personal accounts relating

to person and Organisation are known

as personal accounts Example Ramrsquos Account ABC amp Co Account etc

Real account - The accounts related to tangible and intangible assets are called real accountsExample Cash Account Furniture Account etc

Nominal account- Accounts related to expenses losses incomes and gains are known as nominal accountsExample Wages Account Salary Account Discount Account etc

bull Accounting cycle Accounting cycle refers to a complete sequence of accounting activities It begins with recording of transactions and ends with the preparation of a balance sheet

English 1 Transformation of sentences

Sentences A sentence is a group of words which makes complete sense

a Assertive sentencesb Imperative sentencesc Interrogative

sentencesd Exclamatory sentences

Sentences can be changed from one grammatical form to another without changing the meaning of the sentence This is known as transformation of sentences

Exercise 6Rewrite the following sentences according to the instructions given below without changing their meanings

1 As soon as he saw the beer he jumped into the river ( Begin No sooner)

2 None but brave deserve the fair (Begin the bravehellip)

3 This box is too heavy for me to lift ( Use so hellip That instead of too)

4 No one other than a king can live like James Luxurious ( Begin only James)

5 Oh for the wings of a dove (Begin I wishhellip)

BENGALI(2ND LANGUAGE)

ldquo বঙগভমির পরমি ldquo াইকেল ধসদন দতত

পব13পোসঠ আসোলিচত ৩ পরবোস দৈদসবর বস ীবতোরো Pলিদ স এ লেদ -আকো সত-োলি লেদ তোস - ক) বকতো লেক কোর লেো লেকো কলিবতোর অং ) কোর পরলিত বকতোর এই উলিকত গ) এ লেদ আকো সত বসত কী বলিঝসয়স4 ীবতোরো বসত কী লেবোঝ ঘ ) আসোচয অংসর তোৎপP13 কী

উ -ক ) বকতো স কলিব মোইসক ম3দ দতত

Types of AccountPersonal AccountReal AccountNominal AccountBalance Sheet (opening)

কলিব মোইসক ম3দ দসততর রলিচত বঙগভলিমর পরলিত কলিবতোর অং ) কলিব বঙগী অ13োৎ লেদমোতোর পরলিত কলিবর এই উলিকত গ ) এ লেদ আকো বসত কলিবর মোব লেদী রপ আকো লেক লেবোঝোসো সয়স4 আকো লেসক লেPম তোরো স পসর লেতমলি ীব লেদ রপ আকো লেসক পরো রপ তোরো স পরসত পোসর এই মভোবোর কোই কলিব বসস4 ঘ ) পরবো Pোতরোয় Pলিদ কলিবর লেদ আকো লেসক ীব তোরো রপ পরো স পসর তোসত কলিব লিবনদমোতর দঃলিত কোর মতয লিবসর সবোভোলিবক পলিরলিত এবং মোষ মরী তোই পরবোস Pলিদ তা োর মতয য় তবও কলিব লিবচলিত সব ো কোর পলিবীসত লেকউ অমর য় লিক4ই অকষয় য় দীর লেPম লিচরপরবোমো লেতমলি মোসষর ীবও চমোতোই ীব - সতবধতোই মতয ীব দীসত মোষ লিতয পরবোমো তবও লেPব মোষ আপ কতকসম13র মো3যসম মোসষর মস লিসসদর সথো কসর লিসত পোসর তোরো লিচরভোসবর সয় মোসষর মস লিবরো কসর তোসদর মস3য লেকউ পGভসত লিবী সয় গোসও মোসষর মস তোরো লিতযপলিত লিতযবলিনদত

Hindi 2ndlang

काकीी(लिसयारामशरणगपत)

इस कहानी म क न यह बतान का परयास निकया ह निक बचच अपनी मा स निकतना परम करत ह शयाम अबोध बाक ह वह अपनी मा क मरन क बाद उसन अपनी मा क लिए बहत रोया बाद म उस पता चा निक उसकी मा राम क घर ची गई ह आकाश म उडती हई पतग दकर उस हष हआ निक पतग क दवारा वह अपनी मा को नीच उतारगा इसक लिए वह अपनी निपता की जब स दो बार सवा रपया निनकाकर पतग और दो मोटी सी मन वाी अपन भाई स काकी एक कागज पर लिवा कर पतग म लिशव का दिदयानिनकाकर पतग और दो मोटी सी मन वाी अपन भाई स काकी एक कागज पर लिवा कर पतग म लिचपका दिदयाभोा और शयाम कोठरी म रससी बाधनी रह थ तभी उसक निपता करोध म आकर उन स पछ निक कया उनकी जब स रपया निनकाा हभोा डर क मार बताया निक शयाम इस पतग क दवारा अपनी काकी को राम क यहा स उतारना चाहता हनिवशशवर(शयाम क निपता)न फटी पतग उठाकर दी तो उस पर काकी लिा थावह हत बजि होकर वही ड रह गएउनहोन सोचा निक मन अपन पतर को मारा जोनिक अनजान और निनदष थावह अपनी मा कोनिकतना पयार करता ह

helliphellipContinue to next

Computer Application

Java Programming Prog 1Write a java program to input two numbers from user and display the sum or product of them as per user choice Use switch case statementSolve public class sum_product public static void main(String args[]) Scanner sc=new Scanner(Systemin) int abc Systemoutprintln(ldquoEnter two numbersrdquo) a=scnextInt() b=scnextInt() Systemoutprintln(ldquoPress 1 for sum or 2 for productrdquo)

c=scnextInt() switch(c) case 1 Systemoutprintln(ldquoThe sum will be =rdquo+(a+b)) break case 2 Systemoutprintln(ldquoThe product will be =rdquo+(ab)) break default Systemoutprintln(ldquoWrong Inputrdquo) Home Work - Practice in your computer using bluej

Subject Eng Literature (The Merchant of Venice ndash William Shakespeare)Topic Act I Scene 2 Lines 92 to 126 (End of scene) Date 13th April 2020 (5th Period)

[Students should read the original play and also the paraphrase given in the school prescribed textbook]Summary Questions amp Answers

o After Portia has expressed her opinion about the suitors Nerissa informs that she need not bother about any one of them as they have decided to quit Belmont at the earliest opportunity because they do not believe in trying their luck by the caskets which is the only way of winning Portia

o Nerissa then enquires of Portiarsquos opinion about Bassanio who once visited her in the company of the Marquis of Montferrat and says that she had never come across such an ideal love deserving the fairest lady for his bride

o Portia seems to remember Bassanio quite correctly and says that she agrees with Nerissa At this moment a servant informs Portia that the Prince of Morocco has arrived to try his luck by the caskets

o Portia tells Nerissa that if she could welcome this new suitor as gladly as she says farewell to the previous ones she would be glad of his arrival However if he happens to have the virtues of a saint but the black complexion of a devil she would prefer to have him for religious consolation rather than as a husband

(1) NERISSA You need not fear lady (Line 97-103)

the having any of these lords they have acquainted me with their determinations

which is indeed to return to their home and to

trouble you with no more suit unless you may be wonby some other sort than your fathers imposition depending on the caskets

PORTIA If I live to be as old as Sibylla I will die as chaste asDiana unless I be obtained by the manner of my fatherswill I am glad this parcel of wooers are so reasonablefor there is not one among them but I dote on his veryabsence and I pray God grant them a fair departure

(a) Elucidate the idea expressed in the first speech of the above dialogue

In the first speech Nerissa assures Portia that she need not have any fear of being compelled to marry anyone of the suitors who had lately come to Belmont She informs her that they have all decided to return to their respective countries(b) Illuminate the meaning of the phrase ldquoyour fatherrsquos imposition depending on the casketsrdquo

Nerissa means that the suitors of Portia do not find the conditions imposed by the will of her father to their liking They are too hard for them These conditions are that in the event of a suitor failing to choose the right casket (i) he should never disclose to anybody which casket he chose (ii) he can never marry and (iii) he should take his departure immediately(c) Explain the meaning of the term lsquoSibyllarsquo

lsquoSibyllarsquo is the name given by Romans and Greeks to a prophetess inspired by some deity usually the sun-god Apollo She had a very long life The god Apollo granted her as many years of life as she could hold grains of sand in her hand(d) Elucidate the meaning of the term lsquoDianarsquo

lsquoDianarsquo is the goddess of hunting She is also regarded as a symbol of virginity because she never fell in love and never

married(e) Explain the meaning of the first two lines of Portiarsquos speech

Portia says that even if she is to live for centuries like Sibylla she would not marry except in accordance to her fatherrsquos will She asserts that she would not mind remaining unmarried and untouched by a man like Diana the virgin the goddess of hunting unless a man is able to win her by passing the test laid down by her father

Class XSubject Topic Summary Execution

Hindi 2nd

Langबड घर की बटी( मशी परमचद)

lsquoबड घर की बीटीrsquo कहानी का उददशय मधयम वग की घर समसया को सझा कर सगदिठत परिरवार म मिम जकर परम स रहन का सदश दना ह घर म शानित tानिपत करन की जिजममदारी नारी की होती ह यदिद नारी समझदार ह उसम धय और परिरवार क परनित परम ह तो कोई भी घटना परिरवार को निवघदिटत नही कर सकती या कहानी परिरवार को सगदिठत करत हए परम सौहाद स एक रदसर की भावनाओ को समझ करउनका सहयोग करत हए जीवन यापन करन की पररणा दती ह मशीपरमचदर जी न इस कहानी म सयकत परिरवार का परनितनिनमिधतव निकया ह यह कहानी बनी माधव सिसह जो गौरी पर क जमीदार क उनक दो पतरो की हशरी कठ ा निबहारीशरीकात का निववाह एकजमीदार घरान की पतरी आनदी स हआ थाआनदी न द को ससरा क वातावरण म ढालिया थाएक दिदन आनदी का अपन दवर ा निबहारी स झगडा हो जाता ह दोनो भाई एक रदसर स अग होन की कोलिशश करत हसभी बह आनदी न अपन मधर वयवहार स ा निबहारी को घर छोडकर जान स रोक लिया| इस पर बनी माधव सिसह न कहा निक बड घर की बटी ऐसी ही होती ह जो निबगडा काम बना ती ह अतः शीषक साथक ह बड घर की बटी आनदी ह

helliphelliphelliphellipContinue to nextBiology Topic ndash Chp-1

CellWelcome to new session 2020-21Today we will start with Chpter 1 cell CELL

Protoplasm+Cellmembrane Or Cell wall

Cytoplasm+Neucleus

Cytoplasmic+ CytoplasmicOrganelles Inclutions(mitochondria (food Golgi bodies pigments)Ribosome)

What is cellbull Cell is the structural and functional unit of living organismbull According to number of cells organisms areUnicellular - Amoeba bacteria Multicellular - Rose Mango Tiger HumanSmallest cell -bacteria Longest cell - Nerve cellLargest cell - Ostrich egg cellCells are of different size and shapes according to their functionsQ2Write chief functions of following cellorganelles

Q3What is tonoplastVacuoles covered by a covering called tonoplast

Bengali(2Nd

Language)

ফ ফটক ো ফটক (কলিবতো ) ভোষ মসোপো3 gtPোয়

একটি লেমসয়র ীবস লেপরম লিকভোসব ফসট ওসঠ তো লেদলিসয়স4 কলিব লেপরম Pই য় লেই ময়ই বনত কোস পলিরত য় ফ লেফোটো বো োসফোটো লেটো ব2 কো য় লেমসয়সদর ব gtয13 লেপরসমর 4লিব ফসট উসঠস4 এই কলিবতোয় লেপরম মোষসক মত gtযর মস লেফস লিদসয় পরকষস বাোচোসোর gtয োত বো2োয় কলিবতোয় লেমসয়টির পসব13র দঃসর কো বো সও লেমসয়টি লেই পসর পলিক সত চোয়ো োরী ীবসর কোস4 পরম লেPৌবস লেপরমসক পোবোর পরব ইচছো োকসও তো পসর লেলিতবোচকতোয় পলিরত য় কলিব ভোষ মসোপো3 যোয় লেP ক লেপরসমর

কলিবতোয় ব gtযবহত লিবসষ লিক4 সvর অ13 লেদওয়ো ১) রসবোো= লেP লিবলিভনন রকম ডোকসত পোসর২) ো= পোর ৩) ঠলি = লেচোসর বZ৪)আই বস2ো=অলিববোলিত৫)শইসয় = োলিয়ত কসর৬)োতপাোচ= লিবলিভনন পরকোর৭)দ2োম = v কসর বZ কসর লেদওয়ো৮)লেরলিং =লেোোর দৈতরী লেব2ো৯) বনত= একঋত১০) পাোর = বসকরো2

Organelles Functions

1 Endoplasmic reticulum

2 Mitochondria

3Golgibodies

4 Ribosome5Lysosome

6Plastids

7 Centrosome

i) Supportive framework for the cellii) Synthesis and transpost of proteinsRelease of energy in the form of ATPi) Synthesis and secretion of enzymes hormoneii) Formation of vacuoles lysosomei) Protein Synthesisi) Intracellular digestionii) Destroy foreign substancei )Leucoplast - stores starchii)chloroplast - trap solar energyiii) Chromoplast - imparts colour toflowers amp fruitsi) Initiates and requlates cell division

কলিবতো তোর অ13সক ভোষোয় পরকোো কসর ঘলিরসয় ব যকত কসরস4 লেপরসমর ফতো আর লিবফতো লেক গোঢ় কসর লেদোসো কলিব ভোষ মসোপো3 যোসয়র অলিভবসর অ যলিদক

Economics

Factors of Production

Welcome to the new sessionToday we are going to start the first chapter of Class XThe name of the chapter is Factors of productionBy the name I hope you all can recall a glimpse of what you have learnt in the second chapter of Class IX

NowProduction is the process of creating the various goods and services which are consumed by the people of the country to satisfy their wants

Thus it is the process in which some materials are transformed from one form to another to create utility and value in goods

For example utility can be created by changing the form of a commodity ie

Making of table out of wood by a carpenter for his customer here the wood is getting transformed into table creating utility for his customer and he can also command a price for it

On the other hand Housewives perform very

useful activities at home which create utility but their domestic activities are not included in production because they have no money value

So we can also say that Production denotes two things firstly creation of utility and secondly creation of value

Production is not complete unless it reaches the consumer

An increase in production will increase the economic welfare of the consumers and hence the aim is to raise the production level of the country

Again production of a good or service is only possible if certain resources or

Questions

1 What do you mean by production

Answer Production means the creation of goods and services for the purpose of selling in the market

In fact production involves the transformation of inputs into outputs

Hence production denotes two thingsCreation of utility and creation of valueUtility and value can be created by changing the form by changing the place by changing the time and by rendering services

Example Transformation of raw

materials into finish goods such as potter creates utility by converting mud into utensils assembling of small parts to make bigger machinery

Production also includes services such as distribution and marketing

2 What are the factors of production

Answer Factors of Production refers to the resources and inputs needed for producing goods and servicesThese inputs can be classified as

Land Labour

Capital Enterprise

Land Land is defined to include not only the surface of the earth but also all other free gifts of nature(for example mineral resources forest resources and indeed anything that helps us to carry out the production of goods and services but is provided by

inputs are used together in right proportion

A resource or an input which helps in the process of production to obtain an output is called FACTOR OF PRODUCTION

These factors of production can broadly be categorized into four parts 1LAND 2LABOUR3CAPITAL4ENTERPRISE (ORGANISATION)or Entrepreneur

The above factors are all interdependent on each other and they play a major role in production process

FACTORS OF PRODUCTION

LANDCAPITAL

LABOUR ENTREPRENEUR

nature free of cost)LabourLabour refers to the human efforts that need to be combined with other factors of production for creating an output

CapitalAll man ndash made means of production is called capita example machineries which help in further production Money when used for starting any business for purchasing raw materials machinery tools etc it is regarded as capitalCapital also includes physical capital like factories machineriestoolsbuildingsequipments etcEnterpriseThe task of bearing risks is called enterprise and the person who bears these risks of business is called the entrepreneurThus an entrepreneur is one who organises production takes important decisions regarding production hires and purchases factors of production and bears the risk and uncertainty involved in productionOrganisation refers to the services of an entrepreneur who controls organises and undertakes all risks One who plans organises and manages a business enterprise is an organiser

Physics Chapter 1 Force

Force is an external agent capable of changing the state of rest or motion of a particular body It has a magnitude and a direction The direction towards which the force is applied is known as the direction of the force and the application of force is the point where force is applied The Force can be measured using a spring balance The SI unit of force is Newton (N)

Question 1

State the condition when on applying a force the body has

(a) the translational motion

(b) The rotational motion

Solutions

(a) Translational motion is produced when the body is free to move

(b) Rotational motion is produced when the body is pivoted at a point

Question 2

Define moment of force and state its SI unit

Solutions

The moment of force is equal to the product of the magnitude of the force and the perpendicular distance of the line of action of force from the axis

of rotation

The SI unit of moment of force is Newton times meter

= Newton meter (Nm)

Commercial Studies

Stake holders In this topic you will be come to know about the meaning and concept of stakeholders

How stakeholders are different from shareholders

Questions1 What do you mean by the term stake holdersAnswer) The term stake holders have developed from the words which mean an interest or expected benefit Stakeholders mean all those individuals groups and Institutions which have a state (interest) in the functioning and performance of a commercial organisation or a business enterprise2 What do you mean by share holdersAnswer) The person and Groups who own the shares of the joint stock company by providing capital to the company are called shareholders Shareholders are the internal stakeholders shareholders are one out of several stake holders3 How are shareholders different from stakeholdersAnswer)i) The term shareholders is related to only joint stock company whereas stakeholders are related with all business organisationsii) Stakeholders maybe any individual having financial stake in business organisation whereas a shareholders are those individuals who are holding shares in the company4) How are shareholders different from creditorsAnswer) i) Shareholders are internal stakeholders while creditors are external stakeholdersii) Shareholders invest in the capital of the company whereas creditors give loan to the companyiii) Shareholders are the members of the company with voting rights but creditors are not the members of the company

English 1 Transformation of sentences

Sentences A sentence is a group of words which makes complete sense

e Assertive sentencesf Imperative sentencesg Interrogative sentencesh Exclamatory sentences

Sentences can be changed from one grammatical form to another without changing the meaning of the sentence This is known as transformation of sentences

Exercise 1 Change the following affirmative sentences into Negative sentences

a He is a good manHe is not a bad man

b Ram loves SitaRam is not without love for Sita

c Only he stood first in the classNone but he stood first in the class

d Ankit was wiser than he

He was not so wise as Ankite He did it

He did not fail to do itf As soon as I reached college the

bell rangNo sooner did I reach college than the bell rang

g He finished everythingHe left nothing unfinished

h It always pours when it rainsIt never rains but it pours

Math Topic Commercial MathematicsChapter ndash Goods and services Tax

What is GSTAns It is a abbreviated term of Goods and Service Text which is an indirect tax levied on the sale of goods and rendering servicesSome terms related to GSTDelar Any person who buys goods or services For resale is known as a delar A delar Can be a firm or a companyIntra-state sales Sales of goods and services within the same state or same union territory are called intra- state salesInter-state sales Sales of goods and services outside the state or union territory are called Inter-state sales4) Input GST GST is paid by dealers on purchase of goods and services are called input GST5) Output GST GST is collected from customers on sale of goods and services are called output GST6) Types of GST There are three taxes applicable under GST(i) Central Goods and Services Tax (CGST)(ii) State Goods and Services Tax (SGST) or Union Territory Goods and Services Tax (UTGST) Both these taxes are levied on intra-state sales Here GST is divided equally among central and state governments(iii) Integrated Goods and Services Tax (IGST) IGST is levied on inter- state sales It is also levied on import of goods and services into India and export of goods and services from India

Subject Eng Literature (The Merchant of Venice ndash William Shakespeare)Topic Act III Scene 4 Lines 1 to 44 (Portia hellip To wish it back on you fare you well Jessica)[Students should read the original play and also the paraphrase given in the school prescribed textbook]

Summary Questions amp AnswersIn this scene we suddenly find a new element in the character of Portia We have already seen her possessed of every graceful womanly quality but now she shows that she is capable of rapid decision and determined action She shows this by her sudden resolve to hasten to Venice with a daring scheme for the rescue of Antonio This is an important scene in the dramatic action for it leads up to and renders possible the striking events of the famous trial scene which is one of the greatest striking elements of the play Moreover the fact that all the characters of importance are now assembled together in Venice makes the union of the main plot and the secondary story complete

(1) LORENZO Madam although I speak it in your presence(Line 1-9)

You have a noble and a true conceit

Of god-like amity which appears most strongly

In bearing thus the absence of your lordBut if you knew to whom you show this honourHow true a gentleman you send reliefHow dear a lover of my lord your husbandI know you would be prouder of the workThan customary bounty can enforce you

(a) Where is Lorenzo Why is he here To whom is he referring as lsquoMadamrsquo

Lorenzo is at Portiarsquos residence He had met Salerio on the way and Salerio had begged him to come along with him to

o In this scene Portia Nerissa Lorenzo Jessica and Balthazar appear

o Portia requests Lorenzo and Jessica to be in charge of her house during her absence from Belmont because she and Nerissa have decided to spend the days in meditation and also in visiting the holy places in the neighbourhood of Belmont She has already instructed her people to acknowledge both Lorenzo and Jessica as master and mistress of house during her absence Lorenzo and Jessica gladly agree to look after the house of Portia

handover the letter from Antonio to Bassanio The letter carried the bad news about Antoniorsquos arrest for non-payment of loan taken from Shylock Hence Salerio might have preferred company to break this bad news to Bassanio He is referring to Portia as Madam(b) What does Portia say on hearing the above extract

Portia says that she has never regretted doing good to others Friends who spend a lot of time together and really are there for each other have many traits in common As Antonio is Bassaniorsquos best friend saving him is like saving Bassanio who is like her own soul She asks Lorenzo to take care of management of the house till Bassanio is back(c) What does Portia send with Bassanio and why

On hearing about Antoniorsquos troubles on account of Bassanio her husband Portia immediately sends him with enough gold to repay the debt many times over to Venice to help Antonio out of his misfortune

(2) Lorenzo Madam with all my heart (Line 36-40)

I shall obey you in all fair commands

Portia My people do already know my mindAnd will acknowledge you and JessicaIn place of Lord Bassanio and myselfSo fare you well till we shall meet again

(a) Where are Lorenzo and Portia at this time What lsquofair commandsrsquo are given to Lorenzo

Lorenzo and Portia are at Belmont during this scenePortia reveals to Lorenzo that she has sworn to contemplate in prayer at a monastery around two miles away until her husband returns from Venice She tells him that Nerissa would accompany her and asks him to manage the house with Jessica till things are settled In response Lorenzo tells her that he would be obliged to do whatever she asks him to do(b) Where is Portia actually going and why

Portia tells Lorenzo that she would live a life of contemplation and pray at a monastery which is two miles away from her place In reality Portia plans to go to Venice in disguise with Nerissa and argue the case in defense of Antonio She is very sure that her plan would succeed

ClassXI (ScienceHumanitiesCommerce)Subject Topic Summary Execution

Computer Science

(APC)

Ch ndash 1 Numbers

(Numbers in different bases and

their Arithmatical operations)

Number System In computers Number System is defined as a writing system to represent the numbers in different ways ie we are using different symbols and notations to represent numbers There are four ways we can represent the number ndash Binary Decimal Octal and Hexadecimal

Decimal Number SystemThis number system consist 10 digits These are 0 1 2 3 4 5 6 7 8 amp 9

Binary Number SystemThis number system has only two digits these are 0 and 1 Here 0 stands for off while 1 stands for on

Octal Number SystemThis number system has 8 digits these are 0 1 2 3 4 5 6 amp 7

Hexadecimal Number SystemThis number system has 16 digits these are 0 1 2 3 4 5 6 7 8 9 A B C D E F Here the value of the alphabets are as follows A=10 B=11 C=12 D=13 E=14 F=15

Rules for conversion decimal number to Binary1 Divide the decimal number by 22 If the number will not divide equally by 2 then round down the answer to the nearest whole number (integer)3 Keep a note of the remainder it should be either 0 or 14 Keep repeating the above steps dividing each answer by 2 until you reach zero5 Write out all the remainders from bottom to top This is your binary solution

For example Lets convert 32 to binary 2 32 2 16 - 0 2 8 - 0 2 4 - 0 2 2 - 0 2 1 - 0 0 - 1

The binary equivalent of 3210 is 1000002

Try the follwing youself1 2410

2 4810

3 1210

History GROWTH OF NATIONALISM

The second half of the 19th century witnessed growth of political consciousness and a sense of Nationalism among the IndiansThere were various factors for growth of Indian Nationalism- As a result various political associations were formed in different provinces by the educated Indians Surendranath Banerjee organized a meeting of National conference at Calcutta Ultimately the National Congress was founded in Bombay in 1885This body became the vanguard of Indian struggle for freedom The congress leaders were known as moderates because they followed a policy of prayer and petition A large number of Indian leaders had experienced in political agitation The Political situation of England was also changed Moreover increasing revolutionary activities in Maharashtra Punjab and Bengal became serious concern to the British Government In this

QUESTION1 What do you mean by Nationalism ANSWER 1 Nationalism is defined as loyalty and devotion to own nation especially a sense of national consciousnessQUESTION 2 What are the causes of nationalism ANSWER 2 There were various factors for growth of nationalism

1 Spread of western education2 The progress of vernacular press and

patriotic literature3 The economic exploitation of our

country by the colonial rulers4 International affairs

QUESTION 3 Who organized National conference in Calcutta in 1883 ANSWER 3 Surendranath BanerjeeQUESTION 4 When did Indian National Congress formANSWER 4 Indian National Congress was formed in 1885 in BombayQUESTION 5 Who were ModeratesANSWER 5 The Early Nationalists were also known as Moderates Their emergence marked

background Lord Curzon became Viceroy in India He had no respect for the Indian National Congress

the beginning of the organized national movement in India They believed in British justice and were loyal to them They followed a policy of prayer and petition They demanded constitutional reforms of our country Impotant Moderate leaders were Pherozshah Mehta Dadabhai Naorozi and Surendranath Banerjee etcQUESTION 6 What do you know about Extremism in Indian National movementANSWER 6 In the beginning of 20th century a new class of national leaders emerged in India which was different from the moderate groups They started more aggressive movement against the British empire The goal of extremists was ldquoswarajrdquo Important extremist leaders were Bal Gangadhar Tilak Lala Lajpat Rai Bipin Chandra Pal etcQUESTION 7 Mention the places which were the main centres of Revolutionary movementANSWER 7 Maharashtra Bengal and Punjab

Physics

Chapter Dimensional Analysis

(Summary)

The dimensions of a physical quantity are the powers to which the fundamental units are raised in order to obtain the derived unit of that quantit

The physical quantites lengthmasstime are represented by [L] [M] [T] resp let they are raised to powers ( dimesions) abc resp then any physical quantity can be represented by [ La Mb Tc ] Examples

1 Area area = L x B = [L] x [L] = [M0 L2 T0 ]

2 Density density = massvolume = [M][L3] = [ M L-3]

3 Velocity velocity = distancetime = [L][T] = [LT-1]HW Try to find out dimension of acceleration Acceleration = velocity timeNB One can find the SI Units Using Dimension Analysis Such as for area we have [L2] so its SI unit is m2

Biology Topic ndash Chp-1 The living world

Today we will start the first chapter the living world Here we discuss about the characteristics of living organism and what are the difference between them and nonliving substances We also discuss about the contribution of different Scientists

There are over 500000 species of plants andover a million species of animal are present on earth Some 15000 new species were discovered every yearQ1 What is a living organismbull A living organism is primarily physico -chemical material that demonstrate a high degree of complexity is capable of selfRegulation possesses a metabolism and perpetuates itself through timeQ2 What are the differences between livingand non-livingsi) Compared with non-living living organisms

have more complex organised structure and their use of energy is more controlled amp efficientii) Living things reproduce their own kind by forming new cells which contains copies of their genesiii) Each organism has some degree of homeostasisie it is able to make adjustments so that internal environment remains constantQ3 Write contributions of following Scientists i) Aristotle - One of the first theories in Biology places all living things in a hiearchieii) AV Leeuwenhoek - was the first to observe living single celled organisms under microscopeii) Carolus Linnaeus - developed the binary system for naming of organisms and classificationiii) Geregor Johann Mendel ndash discoverbasic principles of inheritanceHomework i) C Darwin ii)Schleiden

Math Trigonometric functions

1 Overviewi) Trigonometry The word lsquotrigonometryrsquo is derived from the Greek words lsquotrigonrsquo and lsquometronrsquo which means measuring the sides of a triangle An angle is the amount of rotation of a revolving line with respect to a fixed line Usually we follow two types of conventions for measuring angles ie a) Sexagesimal system b) Circular system In Sexagesimal system the unit of measurement is Degree In Circular system the unit of measurement is Radian ii) Relation between degree and radianThe ratio of circumference of a circle to its diameter is always a constant This constant ratio is a number denoted by π which is taken approximately as 227The relationship between degree amp radian measurements is as follows2 right angles = 180deg= π radians1radian = 180degπ=57deg16(approx) 1deg=π180 radianiii) Length of an arc of a circleIf an arc of length s subtends an angle θ radians at the center of a circle of radius r then s=rθiv) Area of a sector of a circleA sector is like a pizza slice of the

Q) Express the following angles in radiana) 45deg b) 40deg3730Ans a) We have 180deg=π radiansi e 45deg= πtimes45180 radian = π4 radiansb) 40deg3730= 40deg37+3060 minute= 40deg 37 +12 minute= 40deg+ 752 minute=40 + 75(2times60) degree=3258 degreeNow 180deg=π radianie 3258 degree= (πtimes325) (180times8) radians = 65π288 radiansQ) A circle has a radius of r=12 meters What is the length of an arc traced out by a 60deg angle in the center of the circleAns In this problem we know both the central angle (60deg) and the radius of the circle (12) All we have to do is plug those values into our equation and we get

s = 2π(12)(60360)s = 24π6s = 4πSo the length of an arc traced out by a 60deg angle in a circle with a radius of 12 meters equals 4π meters asymp 1257 metersQ) Find the area of the sector with a central angle 30deg and a radius of 9cmAns GivenRadius r = 9 cmAngle θ = 30degArea of the sector = θ360degtimesπr2

= 30360degtimes227times92=2121cm2

circle It consists of a region bounded by two radii and an arc lying between the radiiThe area of a sector is a fraction of the area of the circle This area is proportional to the central angle In other words the bigger the central angle the larger is the area of the sectorArea of Sector = θ2 times r2 (when θ is in radians)

Area of Sector = θ times π360 times r2 (when θ is in degrees)

COMMERCE

CLASSIFICTION OF HUMAN ACTIVITIES-ECONOMIC AND NON-ECONOMIC

Welcome to the new sessiontoday we are going to start the first chapter of Class XI The name of the chapter that we are going to start is

lsquoClassification of Human Activities ndasheconomic and non-economicrsquo

Now let us start the chapter by considering human beings and the activities they perform throughout the day

Human activities means all those activities that human beings undertake to satisfy their wants

Human wants on the other hand are the desire of human beings for goods (vegetables fruits rice etc) and services (services of doctors teachers lawyers etc) that they require to live

Now these human activities continue throughout life as human wants are unending unlimited and recurring as human beings desire for better living throughout their lives

Now human activities can be classified into two categories

Human activities

Economic activities Non-economic activities

Economic activities are

Questions1 What are human activities

Answer Human activities mean all those activities that human beings undertake to satisfy their wants

Example A man working in an office

A boy playing in the garden

2What are the characteristics of human activitiesAnswer the characteristics of human activities are as follows

Human activities are undertaken by men women and children and these activities involve human efforts

Human activities are undertaken to satisfy human wants which are unlimited

Human activities continue throughout life

Human activities are performed for both earning money and personal satisfaction

3What is economic activitiesGive example

Answer Economic activities are undertaken by human beings with the object of earning money acquiring wealth and thereby satisfying human wantsExample

Selling of goods by a shop keeper to his customer

A clinic run by a doctor Service of a teacher in school or college

undertaken by human beings with the object of earning money and acquiring wealth

These activities result in the production of economic goods and services

Example Human activities(ie working in factories officesshops) which produce direct economic benefits

Non-economic activities are inspired by human sentiments and emotions such as love for the family desire to help the poor and love for the country

Thus these human activities (eg praying playing sleeping) produce no direct economic benefits and they are also not related to earning money and acquiring wealth

4 What are the characteristics of economic activities

Answer The characteristics of economic activities are as follows

Economic motiveEconomic activities are undertaken to earn money and acquire wealth

ProductiveEconomic activities involve productiondistribution and exchange of goods and services to create wealth

Economic growthEconomic activities determine the level of economic development of a country and standard of living of its citizens

Socially desirableEconomic activities are socially desirable for society

Economic resourcesEconomic activities make use of all the economic resources such landlabourcapital etc

5 What do you mean by non-economic activitiesExampleAnswerNon-economic activities are inspired by human sentiments and emotions such as love for the family desire to help the poor and love for the countryThese activities are not undertaken for monetary gain but for onersquos satisfaction and happinessExample

a mother looks after her children

a student donates blood8 Differentiate between Economic activities and Non-economic activities

Economic activities

Non-economic activities

1to earn living and acquiring wealth2Result can be measured in terms of money

3ExampleBusinessprofession and employment

1 to obtain some satisfaction

2Result cannot be measured in terms of money

3ExampleFamily-orientedreligious socialCultural and national

BUSINESS STUDIES

BUSINESS ENVIRONMENT

Welcome to the new sessionToday we are going to start the first chapter and the name of the chapter is Business Environment

In todayrsquos world every business enterprise is a part of the society It exists and operates in association with various groups in society such as customers suppliers competitors banks and financial institutions government agencies trade unions media and so on All these groups influence the functioning of business in one way or the other They constitute the environment of businessConcept of Business Environment

The term lsquobusiness environmentrsquo refers to the sum total of all individuals institutions and other forces that lie outside a business enterprise but that may influence its functioning and performance

The main features of business environment

Totality of External forces General and Specific forces Interrelatedness Complexity Dynamic Uncertainty Relativity

The Interrelation between business and its environment

The business enterprise is an open system It continuously interacts with its environment It takes inputs

Prepare the following questions from todayrsquos assignment

1 What do you mean by business environment

The term lsquobusiness environmentrsquo means the aggregate of all forces factors and institutions which are external to and beyond the control of an individual business enterprise but they may influence its functioning and performance Business environment is the macro framework within which a business firm a micro unit operates It consists of several interrelated and interacting elements

2 Explain the main features of business environment in brief

Totality of External forces-Business environment is the sum total of all things external to a business environment

General and Specific forces-It includes both the forces general forces are the economic social political legal and technological conditions which indirectly influence all business enterprise Specific forces are the investors customers competitors and suppliers which influence individual enterprise directly

Interrelatedness-Different elements of environment are interrelated for an example growing awareness for health care has increased the demand for health foods

Complexity- Business environment id

(such as raw materials capital labour energy and so on) from its environment transforms them into goods and services and sends them back to the environment

Fig 1 Business Environment Relationship

complex in nature as the elements keep on changing example economic technological and other forces changes in demand for a product and service

Dynamic-Business environment is not static it keeps on changing

Uncertainty- Itrsquos very difficult to predict future events such as technology and fashion which occur fast and frequently

Economics Basic Economic ConceptsSub topic

Microeconomics and

Macroeconomics

Welcome to the new sessiontoday we are going to start the first chapter of Class XI The name of the chapter that we are going to start is Basic Economic concepts

Now Economics covers the study of human activities Human activities are those activities which are performed by humans to satisfy their wants

Thus Human wants are unlimited and therefore economic activities such as production exchange and consumption are needed in order to satisfy those wants

The study of economics is divided largely in two parts which areMicroeconomics and Macroeconomics

SUBJECT- MATTER OF ECONOMICS

MICROECONOMICS MACROECONOMICS

Questions1Who has coined the words micro and macro economics

Answer Ranger Frisch coined the words lsquomicrorsquo and lsquomacrorsquo in 1933 to denote the two branches of economic theory namely microeconomics and macroeconomics

2What is microeconomicsAnswer It is the study of behaviour of individual decision ndash making unit such as consumers firms etc

3 What is macroeconomicsAnswer Macroeonomics is the study of overall economic phenomena like employment national income etc

4 What is the importance of microeconomicsAnswer

Microeconomics helps in formulating economic policies which enhance productive efficiency and results in greater social welfare

It helps the government in formulating correct price policies

It explains the working of a capitalistic economy where individual units(producers and consumers ) are free to take their own decision

Micro means a small part in

microeconomics we do not study the whole economy Hence we study an individual consumer and his or her choices and a producer and his or her profit maximizing decisions in the market Thus it does not mirror what happens in the economy as a whole

Macroeconomics on the other hand studies the economy as a whole It is concerned with aggregate and depicts the entire picture of the economyMacroeconomics deals with the national income aggregate investment aggregate consumption etc

Features of Microeconomics It deals with small

parts of the country Hence it looks at

individual consumers firms and industries

It deals with individual income consumption and savings

It studies the determination of price of any product or factors of production

It deals with the working of market via the price mechanism which is nothing but the determination of price and quantity of a commodity by the forces of demand and supply

Features of Macroeconomics

It deals with the study of the economy as a whole

It is concerned with

5 Give a limitation of microeconomics Microeconomics fails to explain the

functioning of an economy as a whole It cannot explain unemployment illiteracy and other problems prevailing in the country

6 What is the importance of macroeconomics It gives overall view of the growing

complexities of an economic system It provides the basic and logical

framework for formulating appropriate macroeconomic policies (eg for inflation poverty etc )to direct and regulate economy towards desirable goals

7What is the limitation of macroeconomics It ignores structural changes in an

individual unit of the aggregate

8 Differentiate between Microeconomics and Macroeconomics

Microeconomics Macroeconomics

the study of aggregates

National income aggregate savings and aggregate investments are major concepts dealt within macroeconomics style

It studies the determination of general price levels

It investigates into the problem of unemployment and the achievement of employment

It studies the aspect of decision making at the aggregate and national levels

It includes all growth theories whether related to developed or developing economies it also includes the study of economic systems and the working of the economy under different systems

Note Both Micro and macro economics are complementary and should be fully utilized for proper understanding of an economy

1It studies economic aspect of an individual unit2It deals with individual incomeConsumption and savings

3 It facilitates determination of price of any product or factors of production

4 Itrsquos scope is narrow and restricted to individual unit

1It studies the economy as a whole

2It deals with the national income aggregate consumption and aggregate savings3 It facilitates determination of general price level in an economy

4 Itrsquos scope is wide as it deals with economic units on the national level

ACCOUNTS

Introduction to Accounting and Book-keeping

Today I am going to share you the meaning of Accounting and Book-keeping and its related terms bullAccounting bullBook Keeping bullAccountsbullTypes Of Accounts bullAccounting Cycle

bull Meaning of accounting

Ans ) Accounting is the art and science of recording classifying and summarising monetary transactions

bull Meaning of Book-keeping

Ans) Bookkeeping is the art of recording business transactions with the view of having a permanent record of them and showing their effect on wealth

bull Meaning of account

Ans) The term account means a record of

business transactions concern a particular person of firm asset or income or expense It is a summarised record of all transactions which take place in an accounting year

bull Types of accountsPersonal accounts ndash Personal accounts relating

to person and Organisation are known as personal accounts Example Ramrsquos Account ABC amp Co Account etc

Real account - The accounts related to tangible and intangible assets are called real accounts Example Cash Account Furniture Account etc

Nominal account- Accounts related to expenses losses incomes and gains are known as nominal accounts Example Wages Account Salary Account Discount Account etc

bull Accounting cycle Accounting cycle refers to a complete sequence of accounting activities It begins with recording of transactions and ends with the preparation of a balance sheet

Chemistry TopicAtomic Structure

Thomsonrsquos atomic modelThomson (1898) was the first to propose the model of an atomHe proposed that an atom can be regarded as a uniform sphere of positive electricity in which requisite number of electrons are embedded evently to neutralize the positive chargeThis is just like plums embedded in a pudding or seeds evently distributed in red spongy mass of a watermelonThis model of atom is known as ldquoPlum-Pudding modelrdquo or

Q1)What is the fundamental constituents of atomAns Electron Proton and neutrons are the fundamental constituents of atomQ2)What is the value of fundamental unit of electricityAnsThe charge carried by one electron is sad to be the fundamental unit of electricityIts magnitude is 48times10-10esuOr 1602times10-19C Q3)Name the element containing no neutronAnsOrdinary hydrogen atom or protium 1H1

Types of AccountPersonal AccountReal AccountNominal AccountBalance Sheet (opening)

ldquowatermelon modelrdquoThis model could explain the electrical neutrality of an atom but failed to explain the result of scattering experiment carried out by Rutherford in 1911So it was rejected ultimately

Q4)Why is an electron called universal particleAns Itrsquos mass and Charge are independent of its source

EVS Chapter 1 ndash Modes of Existence

Modes of existence When one speaks normally about the mode of existence of some group or individual one refers to their customs their mode of being their ethology their habitat in some way their feeling for a placeDifferent modes of exixtence are ndash

1 Hunting ndashGathering2 Pastoral3 Agricultural4 Industrial

1 Hunting and gathering Hunting and gathering mode of existence is characterized by obtaining food from hunting wild animals including fishing and gathering wild plants From their earliest days the hunter-gatherer diet included various grasses tubers fruits seeds and nuts Lacking the means to kill larger animals they procured meat from smaller game or through scavenging

Societies that rely primarily or exclusively on hunting wild animals fishing and gathering wild fruits berries nuts and vegetables to support their diet are called hunting and gathering societies

At least this used to be practice of human beings before agriculture is invented As their brains evolved hominids developed more intricate knowledge of edible plant life and growth cycles

Q) Write the features of Hunting ndash gathering societiesAns - There are five basic characteristics of hunting and gathering societies

i The primary institution is the family which decides how food is to be shared and how children are to be socialized and which provides for the protection of its members

ii They tend to be small with fewer than fifty members

iii They tend to be nomadic moving to new areas when the current food supply in a given area has been exhausted

iv Members display a high level of interdependence

v Labor division is based on sex men hunt and women gather

Political Science

Introduction to political science

Political science occasionally called politology is a social science which deals with systems of governance and the analysis of political activities political thoughts associated constitutions and political behaviorThe study of political science involves the study of both the

Answer the following questions-1 What is political science

Political science occasionally called politology is a social science which deals with systems of governance and the analysis of political activities political thoughts associated constitutions and political behavior

2 Short notes-

traditional and modern theories of politicsTraditionalClassical political sciencepolitical theory-Traditional political science is the study of politics before Second World War The methodology to study Politics was traditional (legal formaletc) the definition of politics traditional (Politics begins and end with state)area of study (constitution state machinery)was traditionalModern Political scienceModern political theory-Modern Political Theory critically examines the contemporary state of political theory making an assessment of the achievement and limitations of the Behavioural Revolution in its totality and reviews objectively the major paradigms and conceptual frameworks adopted by the disciplineContemporary attempts at the development of an integrated political theory involving the use of both traditional and modern concepts approaches and theories-Around late 1960s several political scientists realized the importance of both the traditional political theory and modern Political theory They began building an integrated theory of politics involving a systematic mixture of traditional and modern studies of politics It was held that the study of a complex and vast field like politics needs both traditional as well as

Classical political theory Modern Political theory

Homework-Learn

modern concepts and approaches for studying itrsquos all aspects

Subject Eng Literature (The Tempest ndash William Shakespeare) Topic Act I Scene 1 Lines 1 to 32 (Line 32 ndash Gonzalo hellip If he be not born to be hanged our case is miserable) Date 13th April 2020 (3rd Period)

[Students should read the original play and also the paraphrase given in the school prescribed textbook]Summary Questions amp Answers

[SUMMARY OF THE ENTIRE SCENE]

o The play starts with the scene of a severe storm at sea Alonso (King of Naples) Sebastian (Alonsorsquos brother) Ferdinand (Alonsorsquos son) Gonzalo Antonio (the usurping Duke of Milan) are in a ship in the midst of the storm

o The mariners are trying their best to control the vessel from running aground and are totally following the orders of their Master the Boatswain They have scant success

o The mariners become extremely unhappy and annoyed when most of the passengers arrive on the deck thereby hampering their effort to save the ship There is serious confrontation between them and the passengers who are part of the Kingrsquos entourage

o The mariners could not save the ship

SUMMING-UP

(i) Vivid description of the scene which gives a realistic description of terror and confusion of a tropical storm

(ii) Shows Shakespearersquos accuracy of knowledge in describing the naval operations and also matters of seamanship

(iii) The opening scene justifies the title ndash The Tempest

UNANSWERED QUESTIONS

(i) The King always travels with his entire fleet including his soldiers Where

(1) GONZALO Nay good be patient (Line 15-26)BOATSWAIN When the sea is Hence What cares these

roarers for the name of the king To cabin silence Trouble us not

GONZALO Good yet remember whom thou has aboardBOATSWAIN None that I more love than myself You are a

councillor if you can command these elements to silence and work

the peace of the present we will not hand a rope more use your authority If you cannot give thanks you have

lived so long and make yourself ready in your cabin for the mischance of the hour if it so hap [To the Mariners]

Cheerly good hearts [To Gonzalo] Out of our way I say

(a) To whom is the boatswain speaking What does he mean by lsquoNone that I more love than myselfrsquo

The Boatswain is speaking to Gonzalo the honest old councilor of the Duke of MilanBy using the words ndash lsquoNone that I love more than I love myselfrsquo means that for the Boatswain nobody is dearer to him than his own life

(b) What were the conditions that made the boatswain react in this way

The Boatswain reacts in this way because the storm is at sea and Alonso King of Naples Sebastian his brother Ferdinand his son Gonzalo Antonio the usurping Duke of Milan on board are in distress and in panic Thus they have rushed to the deck interrupting the work of the mariners

(c) What hope does Gonzalo take from the attitude of the boatswain

The insolent and authoritative attitude of Boatswain makes Gonzalo feel comforted He tells that there are no signs that the Boatswain will be drowned But his facial appearance and attitude shows that he is destined to die on land by hanging which in effect means that all on board will be saved Otherwise all the persons on board are doomed

(d) How can they lsquomake yourself ready in your cabinrsquo For what were they asked to make ready themselves

In order to make themselves ready in their cabin the

were the other ships

(ii) Why was the ship in that area Where was it coming from or going where

(iii) The ship broke apart What happened to those who were in the ship

passengers on board must prepare for death which they will possibly soon have to meetThey can retire to their cabins and offer prayers to the Almighty to save them from drowning

(e) What does the boatswain say when he is asked to be patient What does he order to the royal party

When the boatswain is asked to be patient and remain calm he says that he will be patient only when the storm will be over and the sea will be calm but as long as the storm blows and there is danger to the ship he cannot think of being patient He orders the royal party to go to the cabin and leave the mariners to their work

(2) GONZALO I have great comfort from this fellow (Line 27-36)

Methinks he hath no drowning mark upon him his complexion is perfect

gallows Stand fast good Fate to his hanging Make the rope of his destiny our cable for our own doth little advantage If he be not born to be hanged our case is miserable

(a) Why does Gonzalo regard the Boatswain in the midst of danger

In the midst of danger Gonzalo regards the boatswain because he feels that the Boatswain is a source of comfort and is bent upon to do his work sincerely which in this case is saving the ship and its passengers from the severest of raging storm

(b) What reasons does Gonzalo give when he says that none in the ship will die of drowning

Gonzalo is almost sure that none in the ship will die by drowning His says that there is no mark on the face of the boatswain that indicates that he will die by drowning On the other hand the lines on his face are strong indications that he will be hanged to death Therefore there shall be no danger of the shiprsquos sinking

(c) Explain the following ldquoStand fast good Fate to his hanging Make the rope of his destiny our cable for our own doth little advantage If he be not born to be hanged our case is miserablerdquo

The stated lines mean that if the will of destiny is to be carried out then the ship will not get wrecked and all the passengers will be saved The safety of the passengers therefore depends upon the will of fate being carried out in the case of the boatswain If however the boatswain is not to die by hanging then the passengers are also very unsafe because in that case the ship is likely to sink

(d) What order does the Boatswain give to the sailors

when he re-enters What does he say about the crying of the fellows inside the cabin

The boatswain orders the sailors to bring the topmast lower and bring the ship close to a stationary position with the help of the main sail He says that the fellows inside the cabin are moaning and crying in their distress louder than his voice and louder even than the roaring of the storm

Class XII (ScienceCommerceHumanities) Subject Topic Summary Execution

Computer Science

PropositionalLogic

Propositional logic is a procedure to provide reasoning through statementProposition A ststement that results in True or False is said to be proposition There are two types of propositionSimple proposition amp compound propositionSimple proposioton A simple proposition is one that is not a part of any other proposition Such sentential form of proposition is symbolized with english letters in short For example Ram is a claver student (TrueFalse)Where do you live (Not in True or False)Grapes are sweet (TrueFalse)It rains today (TrueFalse)Here we can see some statements anwer would be true or false but some staements answer can not give in terms of true or false Thus the sentences which can be answered in true or false are known as simple propositionAssigning propositon to a variableThe general syntax to assign propostion to a variable is as followsVariable = Simple propositonFor example A=Ram is a clever studentB= Grapes are sweetC= it rains todayCompound proposition

helliphellipto be continued in next classhelliphellipMath Relation Relation If A and B are two non-empty sets

then a relation R from A to B is a subset of AxB If R A x B and (a b) R then we say that a sube isinis related to b by the relation R written as aRbeg Let A be the set of students of class XII and B be the set of students of class XI Then some of the examples of relation from A to B arei) (a b) AXB a is brother of bisinii) (a b) AXB age of a is more than age of isinb Types of relation In this section we would like to study different types of relations We know that a relation in a set A is a subset of A times A Thus the empty set φ and A times A are two extreme relations For illustration consider a relation R in the set A = 1 2 3 4 given by R = (a b) a ndash b = 10 This is the empty set as no pair (a b) satisfies the condition a ndash b = 10 Similarly R = (a b) | a ndash b | ge 0 is the whole primeset A times A as all pairs (a b) in A times A satisfy | a ndash

Example 1 Let A be the set of all students of a boys school Show that the relation R in A given by R = (a b) a is sister of b is the empty relation and R = (a b) the primedifference between heights of a and b is less than 3 meters is the universal relationSolution Since the school is boys school no student of the school can be sister of any student of the school Hence R = φ showing that R is the empty relation It is also obvious that the difference between heights of any two students of the school has to be less than 3 meters This shows that R = A times A is primethe universal relation Example 2 Show that the relation R in the set 1 2 3 given by R = (1 1) (2 2) (3 3) (1 2) (2 3) is reflexive

b | ge 0 These two extreme examples lead us to the following definitionsDefinition 1 A relation R in a set A is called empty relation if no element of A isrelated to any element of A ie R = φ A times AsubDefinition 2 A relation R in a set A is called universal relation if each element of A is related to every element of A ie R = A times A Both the empty relation and the universal relation are some times called trivial relation Definition 3 A relation R in a set A is called(i) reflexive if (a a) R for every a Aisin isin(ii) symmetric if (a1 a2) R implies that (aisin 2a1)

R for all aisin 1 a2 Aisin(iii) transitive if (a1 a2) R and (aisin 2 a3) R isinimplies that (a1 a3) R for all aisin 1 a2 a3 AisinDefinition 4 A relation R in a set A is said to be an equivalence relation if R is reflexive symmetric and transitive

but neither symmetric nor transitiveSolution R is reflexive since (1 1) (2 2) and (3 3) lie in R Also R is not symmetric as (1 2) R but (2 1) isin notinR Similarly R is not transitive as (1 2) R and (2 3) R but (1 3) R isin isin notinExample 3 Show that the relation R in the set Z of integers given byR = (a b) 2 divides a ndash b is an equivalence relationSolution R is reflexive as 2 divides (a ndash a) for all a Z isinFurther if (a b) R then 2 divides a isinndash b Therefore 2 divides b ndash a Hence (b a) R which shows that R is isinsymmetric Similarly if (a b) R and (b c) R isin isinthen a ndash b and b ndash c are divisible by 2 Now a ndash c = (a ndash b) + (b ndash c) is even (Why) So (a ndash c) is divisible by 2 This shows that R is transitive Thus R is an equivalence relation in ZExample 4 Let L be the set of all lines in a plane and R be the relation in L defined as R = (L1 L2) L1 is perpendicular to L2 Show that R is symmetric but neither reflexive nor transitiveSolution R is not reflexive as a line L1 can not be perpendicular to itself ie (L1 L1) R notinR is symmetric as (L1 L2) Risin

L1 is perpendicular to L2rArr L2 is perpendicular to L1rArr (L2 L1) RrArr isin

R is not transitive Indeed if L1 is perpendicular to L2 and L2 is perpendicular to L3 then L1 can never be perpendicular to L3 In fact L1 is parallel to L3 ie (L1 L2) R isin(L2 L3) R but (L1 L3) Risin notin

Chemistry Solid state Characteristics if Solids(i)The particles are locked in fixed positions they are unable to change their relative positions and this brings a definite shape and volume of a solid(ii)In a solid the constituent particles are held by strong forces of attractionThe forces of attraction may be bonding or non bonding(iii)The constituent particles in a solid pack together as closely as possibleoccupying most of the available space within the solidThus the empty space in a solid is very smallThis makes a solid highly rigid and nearly incompressibleThis also explains why a solid has high density and exhibits slow diffusionClassification of Solids

Q1)Define Crystalline solids AnsA Solid that has a definite geometrical shape and a sharp melting pointand whose constituent particles (atomsmolecules or ions) are arranged in a long range order of definite pattern extending throughout the solid is called a crystalline solidExNaClQ2)Define Amorphous solids AnsA solid that does not have a definite shape and a sharp melting pointand whose constituent particles (atomsmolecules or ions) are not arranged in a definite pattern is called an amorphoussolid

Crystalline solidsAmorphous solids

ExGlassRubberQ3)Classify Crystalline Solids Crystalline Solids

Physics Coloumbrsquos Law (Summary)

Before Going Into Coloumbrsquos Law We Will First Learn What is Charge Properties of Charge and Always remember that charge is quantized ie a body always have static charge of magnitude equal to some integral multiple of fundamental electronic charge e= 16 x 10- 19 C

Charge is the property of matter that causes it to produce and experience electrical and magnetic effects The study of the electrical charges at rest is called electrostatics When both electrical and magnetic effects are present the interaction between charges is referred to as electromagnetic

There exist two types of charges in nature positive and negative Like charges repel and unlike charges attract each other

The type of charge on an electron is negative The charge of a proton is the same as that of an electron but with a positive sign In an atom the number of electrons and the number of protons are equal The atom is therefore electrically neutral If one or more electrons are added to it it becomes negatively charged and is designated as negative ion However if one or more electrons are removed from an atom it becomes positively charged and is called a positive ion

The excess or deficiency of electrons in a body gives the concept of charge If there is an excess of electrons in a body it is negatively charged And if there is deficiency of electrons the body becomes positively charged Whenever addition or removal of electrons takes places the body acquires a charge

The SI Unit of charge is coulomb (C) In SI units the current is a fundamental quantity having a unit of ampere (A) The unit of charge is defined in terms of the unit of current Thus one coulomb is the charge transferred in one second across the section of a wire carrying a

Ionic SolidsMetallicSolids

Molecular Solids

current of one ampere

As q = It we have1 C = (1 A) (1 s)

The dimensions of charge are [A T]

Properties of Charge

(1) Quantization of Charge Electric charge can have only discrete values rather than any value That is charge is quantized The smallest discrete value of charge that can exist in nature is the charge on an electron given as

e = plusmn 16 x 10- 19 C

This is the charge attained by an electron and a protonA charge q must be an integral multiple of this basic unit That is

Q = plusmn ne where n = 1 2 hellip

Charge on a body can never be (frac12)e (23)e or 57e etcWhen we rub a glass rod with silk some electrons are transferred from the rod to the silk The rod becomes positively charged The silk becomes negatively charged The coulomb is a very large amount of charge A typical charge acquired by a rubbed body is 10 - 8 C

Biology Reproduction in organisms

Welcome to this new session 2020-21Today in this first chapter we mainly discuss about reproduction types needs and life span of some organismsWe also discuss about difference between sexual and asexual reproduction

Q1 What is reproductionReproduction is defined as a biological processin which an organism gives rise to young onessimilar to itselfQ2 What are the needs of reproductionbulli) Reproduction maintain life on earthii) It enables the continuity of the species generation after generationiii) It creates genetic variation among populationsQ3 Define Life span and write some orgnisms life spanbull Life span is the period from birth to

the natural death of an organism- OrganismsLife span1 Butterfly 1 - 2 weeks2 Fruit fly 30 days3Dog 10-13 years4 Rose5-7 years5 Tortoise100-150 years6 Banyan Tree -200 - 250 yearsQ4 Reproduction is of two types in case ofanimals but in case of plants vegetative propagation is also present

Asexual Reproduction Sexual Reproductioni) Always uniparentalii) Gametes are not involvediii) Only mitotic division involvediv) Somatic cells of parents are involvedv) Offsprings are genetically similar to the parents

i) Usually biparentalii) Gametes are involvediii) Meiosis occurs during gametogenesis Mitosis occurs after fertilisationiv) Germ cells of the parents are involvedv) offsprings are genetically different from the parents

COMMERCE BUSINESS ENVIRONMENT

Welcome to the new sessiontoday we are going to start the first chapter of Class XII The name of the chapter is Business Environment

Already many of you have got some idea about the word business environment form the first chapter of business studies in class XI

In todayrsquos world every business enterprise is a part of the society It exists and operates in association with various groups in society such as customers suppliers competitors banks and financial institutions government agencies trade unions media and so on All these groups influence the functioning of business in one way or the other They constitute the environment of businessConcept of Business Environment

The term lsquobusiness environmentrsquo refers to the sum total of all individuals institutions and other forces that lie outside a business enterprise but that may influence its functioning and performance

The main features of business environment Totality of External forces General and Specific forces Interrelatedness Complexity Dynamic Uncertainty

Prepare the following questions from todayrsquos assignment

2 What do you mean by business environment

The term lsquobusiness environmentrsquo means the aggregate of all forces factors and institutions which are external to and beyond the control of an individual business enterprise but they may influence its functioning and performance Business environment is the macro framework within which a business firm a micro unit operates It consists of several interrelated and interacting elements

2 Explain the main features of business environment in brief

Totality of External forces-Business environment is the sum total of all things external to a business environment

General and Specific forces-It

Relativity

The Interrelation between business and its environment

The business enterprise is an open system It continuously interacts with its environment It takes inputs (such as raw materials capital labour energy and so on) from its environment transforms them into goods and services and sends them back to the environment

Fig 1 Business Environment Relationship

includes both the forces general forces are the economic social political legal and technological conditions which indirectly influence all business enterprise Specific forces are the investors customers competitors and suppliers which influence individual enterprise directly

Interrelatedness-Different elements of environment are interrelated for an example growing awareness for health care has increased the demand for health foods

Complexity- Business environment id complex in nature as the elements keep on changing example economic technological and other forces changes in demand for a product and service

Dynamic-Business environment is not static it keeps on changing

Uncertainty- Itrsquos very difficult to predict future events such as technology and fashion which occur fast and frequently

Business Studies

Human Resources Management

Human resource of an organisation are the aggregate of knowledge skills attitudes of people working in it

The management system which deals with human resources is called human resource management

Features of HRMbullComprehensive functionbullPeople-oriented

Question1) What do you mean by human

resource management Answer) Human resource management may be defined as that field of Management which has to do with planning organising and controlling the functions of procuring developing maintaining and utilising the labour force

bullAction oriented bullPervasive function bullContinuous function

2) Explain the features of HRM in brief

Answer)bullHuman Resource Management is concerned with managing people at work bull Human Resource Management is concerned with employees which bring people and organisations together so that the goals of each are met bullHuman resource management considered every employees as an individual and also promote their satisfaction and growth bull Human resource management is inherent in all organisations and at all levelsbullManagement of human resources are ongoing on never ending process which requires a constant alertness and Awareness of human relations

3) ldquoHR function is said to be pervasiverdquowhy

Answer) Human resource management is required in all organisations whether it is private or government organisations armed forces sports organisations etc It permeatsall the functional areas like production marketing finance research etc This from this feature of human resource management it can be said that it is pervasive in nature

Economics Demand Q1DEFINITION OF DEMANDIn economics demand is the quantity of a good that consumers are willing and able to purchase at various prices during a given period of timeQ2DEMAND CURVEIn economics a demand curve is a graph depicting the relationship between the price of a certain commodity and the quantity of that commodity that is demanded at that pricQ3LAW OF DEMANDIn microeconomics the law of demand states that conditional on all else being equal as the price of a good increases quantity demanded decreases conversely as the price of a good decreases quantity demanded increasesQ4ASSUMPTION of LAW OF DEMAND(i)No change in price of related commodities(ii) No change in income of the consumer(iii) No change in taste and preferences customs habit and fashion of the consumer( No expectation regarding future change in priceQ5MARKET DEMAND SCHEDULEIn economics a market demand schedule is a tabulation of the quantity of a good that all consumers in a market will purchase at a

given price At any given price the corresponding value on the demand schedule is the sum of all consumersrsquo quantities demanded at that priceQ6INDIVIDUAL DEMAND SCHEDULEIndividual demand schedule refers to a tabular statement showing various quantities of a commodity that a consumer is willing to buy at various levels of price during a given period of timeQ7 FACTORS AFFECTING INDIVIDUAL DEMAND FOR A COMMODITY

The factors that influence a consumerrsquos decision to purchase a commodity are also known as determinants of demand The following factors affect the individual demand for a commodity1 price of the commodity2 price of related goods3 income of buyer of the commodity4 tastes and preferences of the buyer1 Price of the CommodityYou must have observed that when price of a commodity falls you tend to buy more of it and when its price rises you tend to buy less of it when all other factors remain constant (lsquoother things remaining the samersquo) In other words other things remaining the same there is an inverse relationship between the price of a commodity and its quantity demanded by its buyers This statement is in accordance with law of demand which you will study in the later part of this lesson Price of a commodity and its quantity demanded by its buyers are inversely related only when lsquoother things remain the samersquo So lsquoother things remaining the samersquo is an assumption when we study the effect of changes in the price of a commodity on its quantity demanded2 Price of Related goodsA consumer may demand a particular good But while buying that good heshe also asks the price of its related goods Related goods can be of two types-(i) Substitute goods(ii) Complementary goods While purchasing a good prices of its substitutes and complements do affect its quantity purchased(i) Price of Substitute Goods Substitute goods are those goods which can easily be used in place of one another for satisfaction of a particular want like tea and coffee An increase in price of substitute good leads to an increase in demand for the given commodity and a decrease in price of substitute good leads to a decrease in demand for the given commodity It means demand for a given commodity is directly affected by change in price of substitute goods For example if price of coffee increases the demand for tea will rise as tea will become relatively cheaper in comparison to coffee(ii) Price of Complementary goods Complementary goods are those goods which are used together to satisfy a particular want like car and petrol An increase in the price of complementary goods leads to a decrease in demand for the given commodity and a decrease in the price of complementary goods leads to an increase in demand for the given commodity For example if price of petrol falls then the demand for cars will increase as it will be relatively cheaper to use both the goods together So demand for a given commodity is inversely affected by change in price of complementary goods3 Income of the Buyer of CommodityDemand for a commodity is also affected by income of its buyer However the effect of change in income on demand depends on the nature of the commodity under consideration In case of some goods like full cream milk fine quality of rice (Basmati rice) etc demand for these commodities increases when income of the buyer increases and

demand for these commodities decreases when income of the buyer decreases Such goods whose demand increases with the increase in income of the buyer are called normal goods But there are some goods like coarse rice toned milk etc whose demand decreases when income of buyer increases and their demand increases when income of the buyer decreases Such goods whose demand decreases with the increase in income of the buyer are called inferior goods Suppose a consumer buys 10 Kgs of rice whose price is ` 25 per Kg He cannot afford to buy better quality of rice because the price of such rice is ` 50 per Kg The consumer is spending ` 250 per month on the purchase of rice Now if income of the consumer increases and he can afford ` 350 on purchase of 10 Kg of rice Now he can afford to buy some quantity of rice say 6 Kgs whose price is ` 25 per Kg and may buy 4 Kgs of rice whose price is ` 50 per Kg Thus he will buy 10 Kgs of rice by spending ` 350 per month Therefore we may conclude that demand for normal goods is directly related to the income of the buyer but demand for inferior goods is inversely related to the income of the buyer4 Tastes and Preferences of the BuyerThe demand for a commodity is also affected by the tastes and preferences of the buyers They include change in fashion customs habits etc Those commodities are preferred by the consumers which are in fashion So demand for those commodities rises which are in fashion On the other hand if a commodity goes out of the fashion its demand falls because no consumer will like to buy it(5) Number of Buyers in the Market(Population)Increase in population raises the market demand whereas decrease in population reduces the market demand for a commodity Not only the size of population but its composition like age (ratio of males females children and old people in population) also affects the demand for a commodity It is because of needs of children young old male and female population differs(6) Distribution of Income and WealthIf the distribution of income and wealth is more in favour of the rich demand for the commodities preferred by the rich such as comforts and luxuries is likely to be higher On the other hand if the distribution of income and wealth is more in favour of poor demand for commodities preferred by the poor such as necessities will be more(7) Season and Weather ConditionsThis is generally observed that the demand for woolens increases during winter whereas demand for ice creams and cold drinks increases during summer Similarly market demand for umbrellas rain coats increases during rainy seasonQ8 REASONS FOR OPERATION OF LAW OF DEMAND WHY DEMAND CURVE SLOPES DOWNWARDNow we will try to explain why does a consumer purchase more quantity of a commodity at a lower price and less of it at a higher price or why does the law of demand operate ie why does the demand curve slope downwards from left to right The main reasons for operation of law of demand are1 Law of Diminishing Marginal UtilityAs you have studied earlier law of diminishing marginal utility states that as we consume more and more units of a commodity the utility derived from each successive unit goes on decreasing The consumer will be ready to pay more for those units which provide him more utility and less for those which provide him less utility It implies that he will purchase more only when the price of the commodity falls2 Income Effect

When price of a commodity falls purchasing power or real income of the consumer increases which enables him to purchase more quantity of the commodity with the same money income Let us take an example Suppose you buy 4 ice creams when price of each ice cream is ` 25 If price of ice creams falls to ` 20 then with same money income you can buy 5 ice creams now3 Substitution EffectWhen price of a commodity falls it becomes comparatively cheaper as compared to its substitutes (although price of substitutes has not been changed) This will lead to rise in demand for the given commodity For example if coke and Pepsi both are sold at ` 10 each and price of coke falls Now coke has become relatively cheaper and will be substituted for Pepsi It will lead to rise in demand for coke4 Change in Number of BuyersWhen price of a commodity falls some old buyers may demand more of the commodity at the reduced price and some new buyers may also start buying this commodity who were not in a position to buy it earlier due to higher price This will lead to increase in number of buyers when price of the commodity falls As a result demand for the commodity rises when its price falls5 Diverse Uses of a CommoditySome commodities have diverse uses like milk It can be used for drinking for sweet preparation for ice cream preparation etc If price of milk rises its use may be restricted to important purpose only This will lead to reduction in demand for other less important uses When price of milk falls it can be put to other uses also leading to rise n demand for itQ9 EXCEPTIONS TO THE LAW OF DEMANDYou have studied in law of demand that a buyer is willing to buy more quantity of a commodity at a lower price and less of it at a higher price But in certain circumstances a rise in price may lead to rise in demand These circumstances are called Exceptions to the Law of Demand Some important exceptions are1 Giffen GoodsGiffen goods are special type of inferior goods in which negative income effect is stronger than negative substitution effect Giffen goods do not follow law of demand as their demand rises when their price rises Examples of Giffen goods are jowar and bajra etc2 Status Symbol GoodsSome goods are used by rich people as status symbols eg diamonds gold jewellary etc The higher the price the higher will be the demand for these goods When price of such goods falls these goods are no longer looked at as status symbol goods and tehrefore therir demand falls3 NecessitiesCommodities such as medicines salt wheat etc do not follow law of demandbecause we have to purchase them in minimum required quantity whatever their price may be4 Goods Expected to be ScarceWhen the buyers expect a scarcity of a particular good in near future they start buying more and more of that good even if their prices are rising For example during war famines etc people tend to buy more of some goods even at higher prices due to fear of their scarcity in near future

Political Science

Constitution of India-The

Preamble

The preamble-

Preamble-

The preamble is the most precious part of the constitution We the people of India having solemnly resolved to constitute India into a Sovereign Socialist Secular Democratic Republic and to secure to all its citizensA preamble is an introductory and expressionary statement in a document that explains the documents purpose and underlying philosophy When applied to the opening paragraphs of a statute it may recite historical facts pertinent to the subject of the statuteNature and purpose of the constitution-Purpose of the Constitution dictates permanent framework of the government to form a more perfect union to establish justice and ensure peace of thenationconstitution provide principles how the government can run itself following the rules and laws written in the constitution of each state keeps them balanced

Answer the following questions-

1 What is preambleA preamble is an introductory and expressionary statement in a document that explains the documents purpose and underlying philosophy2 What is the nature and

purpose of the constitutionConstitution dictatespermanent framework of the government to form a more perfect union to establish justice and ensure peace of the nation

Homework-Learn

Accounts Compatibilty mode

1MEANING OF PARTNERSHIPPartnership is a form of business organisation where two or more persons join hands to run a business They share the profits and losses according to the agreement amongst them According to the Indian Partnership Act 1932 ldquoPartnership is relation between persons who have agreed to share profits of a business carried on by all or any one of them acting for allrdquo For example one of your friends has passed class XII from National Institute of Open Schooling (NIOS) and wants to start a business Heshe approaches you to join in this venture Heshe wants you to contribute some money and participate in the business activities Both of you if join hands constitute a partnership2CHARACTERISTICS1048698 Agreement A partnership is formed by an agreement The agreement may be either oral or in writing It defines the relationship between the persons who agree to carry on business It may contain the terms of sharing profit and the capital to be invested by each partner etc The written agreement is known as partnership deed1048698 Number of persons There must be at least two persons to form a partnership

The maximum number of partners in a partnership firm can be 50 according toCompanies Act 20131048698 Business The Partnership is formed to carry on business with a purpose of earning profits The business should be lawful Thus if two or more persons agree to carry on unlawful activities it will not be termed as partnership1048698 Sharing Profits The partners agree to share profits in the agreed ratio In caseof loss all the partners have to bear it in the same agreed profit sharing ratio10486981048698Mutual Agency Every partner is an agent of the other partners Every partner can bind the firm and all other partners by hisher acts Each partner will be responsible and liable for the acts of all other partners10486981048698Unlimited liability The liability of each partner except that of a minor is unlimited Their liability extends to their personal assets also If the assets of the firm are insufficient to pay off its debts the partnersrsquo personal property can be used to satisfy the claim of the creditors of the partnership firm10486981048698Management All the partners have a right to mange the business However they may authorize one or more partners to manage the affairs of the business on their behalf10486981048698Transferability of Share No partner can transfer hisher share to any one including hisher family member without the consent of all other partners3PARTNERSHIP DEEDAgreement forms the basis of partnership The written form of the agreement is which a document of partnership is It contains terms and conditions regarding the conduct of the business It also explains relationship between the partners This document is called partnership deed Every firm can frame its own partnership deed in which the rights duties and liabilities of the partners are stated in detail It helps in settling the disputes arising among the partners during the general conduct of business 4CONTENTS OF PARTNERSHIP DEEDThe partnership deed generally contains the following (i) Name and address of the partnership firm(ii) Nature and objectives of the business(iii) Name and address of each partner(iv) Ratio in which profits is to be shared(v) Capital contribution by each partner(vi) Rate of Interest on capital if allowed(vii) Salary or any other remuneration to partners if allowed(viii) Rate of interest on loans and advances by a partner to the firm(ix) Drawings of partners and interest thereon if any(x) Method of valuation of goodwill and revaluation of assets and liabilities on the reconstitution of the partnership ie on the admission retirement or death of a partner(xi) Settlement of disputes by arbitration(xii) Settlement of accounts at the time of retirement or death of a partner5IN ABSENCE OF PARTNERSHIP DEEDThe partnership deed lays down the terms and conditions of partnership in regard to rights duties and obligations of the partners In the absence of partnership deed there may arise a controversy on certain issues like profit sharing ratio interest on

capital interest on drawings interest on loan and salary of the partners In such cases the provisions of the Indian Partnership Act becomes applicableSome of the Issues are(i) Distribution of Profit Partners are entitled to share profits equally(ii) Interest on Capital Interest on capital is not allowed(iii) Interest on Drawings No interest on drawing of the partners is to be charged(iv) Interest on Partnerrsquos Loan A Partner is allowed interest 6 per annum on the amount of loan given to the firm by himher(v) Salary and Commission to Partner A partner is not entitled to anysalary or commission or any other remuneration for managing the business

History TOPIC-TOWARDS INDEPENDENCE AND PARTITION THE LAST PHASE (1935-1947)

SUB TOPIC-IMPORTANT POLITICAL DEVELOPMENTS ndash GROWTH OF SOCIAL IDEAS

Socialism is a political social and economic philosophyLike in other parts of the world the Russian revolution of 1917 served as a great inspiration for revolutionaries in India who at that time were engaged in the struggle for liberation from British ruleSocialist ideas led to the formation of communist party of IndiaJAWAHARLAL NEHRU Among the early Congress leaders Jawaharlal Nehru was very much impressed and influenced by the Socialist ideas He also learnt about the Economic activities of the Soviet Union after the Bolshevic Revolution 1917 He made full use of them in IndiaThe election of Jawaharlal Nehru and Subhas Chandra Bose showed the Left wing tendency within CongressJawaharlal Nehru demanded economic freedom along with political freedom of the people in order to end the exploitation of masses

Nehrus working committee included three socialists leaders The Lucknow session was a landmark in the evolution of socialist ideas of the congressSUBHAS CHANDRA BOSE ndash Subhas Chandra Bose had socialist leaning Both Jawaharlal Nehru and Subhas Chandra Bose were known as leftist Congress men Later on National Congress divided into Leftist and rightist campCONGRESS SOCIALIST Within the Congress some leaders formed the Congress Socialist partyPattavi Sitaramyya Sardar Patel Rajendra Prasad had hostile attitude towards the Congress Socialist partyJawaharlals attitude was hesitant

1 QUESTION ndash Mention name of two Congress leaders who had socialist leaning

1ANSWER ndash Subhas Chandra Bose and Jawaharlal Nehru2QUESTION- In which session of the congress Jawaharlal elaborated his Socialist ideas2 ANSWER ndash Lucknow and Faizpur Session in December 1935 and 19363QUESTION ndash Why Congress was sharply divided into leftist and rightist camp 3ANSWER ndash Subhas Chandra Bosersquos attempt to seek re election for congress presidentship in 1939sharply divided the National Congress into Leftist and Rightist camp4 QUESTION ndash Who was MN Roy 4 ANSWER ndash Manabendra Roy first formed the Communist Party of India outside the country at Tashkent in 19205QUESTION ndash Who formed the Congress Socialist Party within the Congress5 ANSWER ndash Jaya Prakash Narayan Achyut Patwardhan Acharya Narendra Dev Ram Mohan Lohia Aruna Asaf Ali6QUESTION ndash When was the Congress Socialist Party formed What was its object6 ANSWER ndash 1934The Congress Socialist Party sought to work out socialist programme through the Congress They joined hands with the Congress and wanted to carry

Subhas Chandra Bose being expelled from the congress after the Tripuri rift he formed Forward BlockThere were basic differences between the Congress Socialists and the communistsTRADE UNION ACTIVITIES Maximum working class people lived in Bombay and Calcutta The working and living conditions of those workers were very miserable In this situation Shasipada Banerjee NM Lokhande protested against the oppression of the working class peopleThe first Trade Union Madras Labour Union was formed in 1918 by BP WadiaIndustrial strikes took place in Kanpur Calcutta Madras Jamshedpur and Ahmedabad AITUC was formed in Bombay in 1927 The growth of Trade union among the workers was slow because of the fear of the dismissal of the jobIn the mean time the Moderates as well as Communists left AITUC and formed separate organization

on National struggle with the help of workers and peasant class of the society7 QUESTION ndash What was the name of the party founded by Subhas Chandra Bose7 ANSWER- Forward Block8QUESTION ndash Who was Shasipada Banerjee8 ANSWER ndash Shasipada Banerjee was a radical Brahmo He founded a working menrsquos club to protest against exploitation of the British rulers towards the working class of India9 QUESTION ndash What was the weekly published by NM Lokhande9ANSWER- Dinabandhu10 QUESTION ndash Who founded Bombay Mill-Hands Association and in which year10 ANSWER- NM Lokhande in189011 QUESTION- Who was BP WadiaANSWER- BPWadia was the founder of Madras Labour Union in191812 QUESTION- What was the name of the first labour union of India12 ANSWER- Madras Labour Union13 QUESTION Who founded the Majur Mahajan 13 ANSWER GANDHIJI14 QUESTION What was the full form of AITUC When it was formed14 ANSWER All India Trade Union Congressin 192715QUESTION Who formed the Red Trade Union Congress and in which year15ANSWER The Communists formed the Red Trade Union Congress16 QUESTION What do you mean by Socialism16 ANSWER Socialism describes any political and economic theory that says the community rather than individuals should own and manage property and natural resources

Subject Eng Literature (The Tempest ndash William Shakespeare) Topic Act III Scene 3 Lines 1 to 52 (Line 52 ndash Brother my lord the Duke Stand to and do as we) Date 13th April 2020 (4th Period)

[Students should read the original play and also the paraphrase given in the school prescribed textbook]Summary Questions amp Answers

o Alonso Sebastian Antonio Gonzalo Adrian Francisco and others wandered about the island in search of Ferdinand and gets tired and hungry of the toil and at the same time gives up all hope of finding him

o Antonio and Sebastian are happy that Alonso is out of hope and decide to make another attempt on his life that night when being so tired they will be sleeping soundly

o Suddenly a solemn and strange music is heard in the air and several strange shapes enter bringing in a banquet These strange shapes then dance round it with gestures of salutation and then inviting the King to eat they depart

o Seeing this strange scene all are inclined to believe the tales told by travelers that there truly are ldquounicornsrdquo and ldquothe phoenixrsquo thronerdquo

1 ALONSO What harmony is this My good friends hark (L18-27)

GONZALO Marvellous sweet music

[Enter several strange shapes bringing in a banquet

they dance about it with gentle actions of salutation

and inviting the King and his companions to eat they depart]ALONSO Give us kind keepers heavens What were theseSEBASTIAN A living drollery Now I will believe

That there are unicorns that in Arabia

There is one tree the phoenixrsquo throne one phoenix

At this hour reigning thereANTONIO Ill believe both

And what does else want credit come to me

And Ill be sworn rsquotis true Travellers neer did lie

Though fools at home condemn rsquoem

(a) How did Prospero present an amazing spectacle before Alonso and his companions

Using his magic powers Prospero ordered strange shapes to lay a banquet before Alonso and his companions The shapes brought several dishes with tasty eatables in them They placed the dishes on a table before Alonso and his companions Then the strange shapes began to dance gracefully around the banquet While dancing they made gestures inviting them to eat the food Then suddenly the shapes disappeared(b) Who were the guests at the strange banquet Describe the lsquoliving drolleryrsquo

Alonso Sebastian Antonio Gonzalo Adrian and Francisco were the guests at the strange banquet

The term ldquoliving drolleryrdquo refers to live entertainment show In this context when Alonso the King of Naples Sebastian his brother Antonio the treacherous brother of Prospero Gonzalo the kind and loyal councillor to the King Adrian and Francisco came to the island they were hungry and weary in their spirits They heard a solemn and strange music They were shocked to see several strange shapes bringing in a banquet and these shapes danced about it with gentle action of salutation inviting the King and his companions to eat After this Sebastian described this show as lsquoliving drolleryrsquo(c) What is lsquophoenixrsquo What are lsquoUnicornsrdquo

The term lsquophoenixrsquo refers to a mythical Arabian bird which lived alone and perched on a solitary tree After one hundred years it expired in flames and rose again from its own ashes

lsquoUnicornsrsquo refers to the mythological four-footed beasts having horns in the centre of their foreheads When the horns are ground into powder the powder was believed to be

an aphrodisiac(d) How does Sebastian explain the puppet show OR Why does the speaker now believe in unicorns and phoenix

Sebastian finds several strange shapes bringing in the banquet They invite the king and his party for dinner and soon depart He tells that if such a strange sight can be a reality there is nothing incredible in the world and from the present moment he will believe anything He says that it is a strange dumb show enacted not by puppets but by living beings It is stranger than a travellerrsquos tale Seeing such a thing

before his own eyes he will no longer disbelieve the story about unicorns and phoenix(e) How do the other characters present respond to this living drollery

At the sight of the lsquoliving drolleryrsquo like Sebastian Gonzalo and Antonio too acted strangely Antonio told that he too now believes in unicorns and phoenix and anything else that seems to be incredible He too now believes in travellersrsquo tales Gonzalo told that if he would report those happenings in Naples nobody will believe him He considers that those gentle shapes were gentler in manner in comparison to the living beings Alonso was at first sight suspicious and told them that those strange shapes conveyed their meaning in expressive gestures when they seemed to lack speech by their movements and sounds Francisco was amazed at their mysterious disappearance

2 ALONSO Not I

(Line 43-52)GONZALO Faith sir you need not fear When we

were boysWho would believe that there were mountaineers

Dewlapped like bulls whose throats had hanging at rsquoem

Wallets of flesh Or that there were such men

Whose heads stood in their breasts Which now we find

Each putter-out of five for one will bring us

Good warrant ofALONSO I will stand to and feed

Although my lastmdashno matter since I feel

The best is past Brother my lord the Duke

Stand to and do as we

(a) How does Alonso respond at the spectacle of the shapes which were sent to them at the instruction of Prospero

After seeing the strange sight of appearing and disappearing of the shapes sent by Prospero to arrange a banquet for them Alonso says that his surprise at having seen those creatures is infinite and he is fully justified in feeling so much surprise He thinks that their shapes their gestures and the sounds they made were indeed amazing Although they do not possess the gift of speech yet they were able to convey their

thoughts by means of their gestures only

(b) What does Prospero say about the views expressed by Alonso regarding the shapes What does Francisco think about the shapesAfter hearing Alonsorsquos views about the shapes Prospero says that this manrsquos praise of the spirits is rather hasty He means to say that Alonso has shown great haste in reaching the conclusion about the shapes Francisco is amazed to see that those shapes disappeared in a mysterious way(c) What does Sebastian ask Alonso to doSebastian tells Alonso that the shapes having disappeared should not matter to them because they have left the eatables behind He asks Alonso to enjoy eating as they are extremely hungry but the king does not accept his offer of enjoying the dishes(d) How does Gonzalo try to dispel Alonsorsquos fear of those strange shapes What kind of references does he give to AlonsoGonzalo says that those who have travelled abroad have reported seeing even stranger sights than these shapes that Alonso and his companions have beheld Hence there is no reason to feel afraid of these shapes Gonzalo further adds that in his younger days he had heard strange stories from travelers and Alonso might have heard similar stories For instance it was said that there existed a certain race of

human beings who had huge lumps of flesh hanging at their throats and who therefore resembled bulls Then Gonzalo tells about a race of human beings whose heads were located at their breasts Gonzalo says that such stories were not believed by most people in those days but now-a-days these stories have become common(e) Explain the following lsquoEach putter-out of five for onersquoEnglish travellers often insured their trips with London brokers Those that went on foreign travels those days used to deposit a certain amount with some firm or company in London before their departure If the travelers failed to return the money was forfeited by the company with which it had been deposited But this money was repaid five-fold if the travelers returned safe and sound In this way a traveler stood a great chance of recovering the entire cost of his

travels(f) Give the explanatory meanings of the following expressions in the context of the above extract (i) Dewlapped (ii) Wallets of flesh

(iii) Putter-out(i) Dewlapped having big lumps of flesh at the necks(ii) Wallets of flesh large masses of flesh looking like bags(iii) Putter-out to invest money before commencing the travel

  • General methods of preparation of hydrogen
  • Chapter Dimensional Analysis (Summary)
    • Properties of Charge
Page 12:   · Web viewSubject. Topic. Summary. Execution. Hindi. व्याकरण. शरीरके अंगो के नाम लिखिए. 1) आँख 2) नाक 3

র + ঈ = সর আ + ই = এ ( লেgt )Pো + ইষট = Pসষট 3ো + ইনদ = স3নদ আ + ঈ = এ ( লেgt )রমো + ঈ = রসম দবোরকো + ঈশবর = দবোরসকশবর

COMPUTER THE WORLD OF WINDOWS 10

DONE IN THE PREVIOUS CLASS PAGE NO-83A TICK THE CORRECT OPTION BACKGROUND DISPLAY AREA RESTORE THREE

MATHEMATICS Topic ndash NumbersChapter ndash Natural numbers and whole numbers

Study item Properties of whole numbers for subtraction1) Closure property When we do subtraction of two whole numbers we can not get a whole number in all time Example 8 ndash 3 = 5 a whole number 0 ndash 6 = -6 is not a whole numberTherefore the subtraction of two whole numbers is not satisfying closure property2) Commutative property If x and y are two whole numbers then x ndash y is not equal to y ndash xExample If x=16 and y = 7 then x ndash y = 16 ndash 7 = 9Again y ndash x = 7 ndash 16 = - 9 Therefore x ndash y not equal to y ndash x Therefore the subtraction of two whole numbers is not satisfy commutative3) Associative property If x y and z are three whole numbersThen x ndash ( y ndash z ) not equal to ( x ndash y ) ndash z Example If x = 20 y = 10 and z = 6Therefore x ndash (y ndash z ) = 20 ndash(10 ndash 6 ) = 20 ndash 4 = 16(X ndash y ) ndash z = (20 ndash 10) ndash 4 = 10 -4 =6Therefore x ndash(y ndash z) not equal to ( x ndash y) ndash zTherefore subtraction of whole numbers is not satisfying associativity4) Distributive property If x y and z are three whole numbersThen x (y ndash z ) = xy ndash xzAnd (y ndash z)x = yx ndash zxExample If x = 10 y = 6 and z = 4x(y ndash z ) = 10(6 ndash 4 ) = 10times6 ndash 10times4 = 60 ndash 40 = 20( 6 ndash 4 )times 10 = 6times10 ndash 4times10 = 60 ndash 40 = 20Therefore the subtraction of whole numbers is satisfying distributive property5) Existence of identity For any whole number x X ndash 0 = x but 0 ndash x = - x not equal to xThus for subtraction no identity number existsException 0 ndash 0 = 0 so 0 is its own identity for subtraction

Class VIISubject Topic Summary Execution

Hindi 2ndlang वचन जो सजञा शबद निकसी वसत या पराणी क एक या अनक होन का बोध कराया उनह वचन कहत ह जस डका- डकयह दो परकार की होती ह-

क) एकवचन-शबद क जिजस रप स उसक एक होन का बोध हो उस एक वचन कहत ह जस निकताब गमा आदिद

) बहवचन-शबद क जिजस रप स उसक आन ोन का पता च उस बहवचन कहत ह जस डक निकताब निततलियाआदिद

निनमनलिखित शबदो को एकवचन स बहवचन म बदोम- हमजानित- जानितयानारी- नारिरयामिमतर ndashमिमतरोपसतक -पसतकसडक-सडकबोत-बोतनाहर-नहररपए-रपया

Bengali বইndashবোংো োলিতয পলিরচয়

পোঠndash১৪ গলপ - অপর কলপো পর

লেক - লিবভলিতভষ বসনদযোপো3যোয়লেকndash রবীনদর পরবতf বোংো কোোলিতয 3োরোর উসgসPোগয োম পরকলিতসপরমী লিবভলিতভষ বসনদযোপো3যোয় তোর লেীসত লেPম বোসর বোসর লিফসর এসস4 গরোম বোংোর পরকলিতর কো লেতমলি এসস4 গরোমী মো লিচতরগলপndash অপর কলপো গলপোংটি লিবভলিতভষ বসনদযোপো3যোসয়র লিবযোত উপযো পসর পাোচোী লেসক গীত অপ অ13োৎ পসর পাোচোী তো অপর কলপোর লেকনদরীয় চলিরতর এই অংস আমরো পোই বোক অপসক বোক অপ কলপো লিবোী লে দসরর অ গো4 লেদস মোসয়র মস লেোো রপকোর রোসয পোলি2 লেদয় দপরসবো মোসয়র মসর কসর কোীদোী মোভোরত এর করসকষতর Pসjর ব13ো শস তোর মোবীর কস13র পরলিত ব2 মমতো য় আবোরপালিসত বলি13ত Pসjর অমোপত অং লে লিসই মোপত কসর বোলি2র লিপ4স বাো বোগোস লিকংবো উঠোসর লিশমসর কলপো লিবো এোস পরকো লেপসয়স4

১ অপর কলপো গসলপর লেক লেক তোর মপসক13 লেসো২ অপর কলপো গলপটি লেকোো লেসক গীত গলপটির ম ভোব লেসো

GEOGRAPHY CHAPTER 7EUROPE

CHAPTER COMPLETE EXERCISEFill in the blanks1 Europe is a continent that comprises the western part of Eurasia2 Eurasia and Africa are connected into one large land mass known as Afroeurasia3 The Strait of Gibraltar separates Europe and Africa4 Europe is surrounded by the Arctic Ocean to the north5 The British Isles includes the island countries of Great Britain and Ireland

Name the following 1 Connects Africa to Eurasia - Isthmus of Suez2 Largest country in the world in terms of area ndash Russia3 A term used collectively for the five countries in northern Europe ndash Nordic Countries4 The capital of Montenegro - Podgorica5 the largest fjord in Norway ndash Sognefjord

Match the following Column 1 Column 2a Albania iii Tiranab Belgium i Brusselsc Denmark v Copenhagend Finland ii Helsinkie Hungary iv Budapest

CHEMISTRY Chapter 2 ndashElement and Compound

Atom - An atom is the basic unit of an element or the smallest particle of an element non capable of independent existence Atom is built up of three sub atomic particles electron proton and neutron

Nucleus-It is the centre of an atom In the centre of the atom contains proton (positively charged particles ) and neutrons ( particles carrying no charge )

Orbits- It surround the nucleus in which revolve electrons (negatively charged particles)

Answer the following

1) What are MetalloidsAns - Certain elements using properties of both metal and non-metals are called metalloids Example Silicon arsenic and antimony

2) What are Noble gasesAns - Certain elements are present in the air and are chemically inert or unreactive Such elements are called rare gases or noble gases Example helium neon argon and Krypton

English 2 Sentences based on meanings

Kinds of sentences

Assertive or declarative to convey information or simply make a statement

Interrogative to ask different types of questions

Imperative to command or instruct someone or make a request

Exclamatory to express strong feelings and emotions

Exercise B1 Stop it ( Exclamatory)2 May you always be happy

together ( Exclamatory)3 He does not like sports

( Assertive)4 Please pass me the salt

( Imperative)5 How dare she talk to me like

that ( Exclamatory)6 May success bless your effort

( Exclamatory)7 Canrsquot you wait for sometime

(Interrogative)8 Did anybody tell you about it

( Interrogative)9 I saw her waiting for the bus

( Assertive)10 Could you please take a

message for me ( Interrogative)

Homework Ex ABiology Chp -2

Classification of Plants

Today we discuss about usefulness of bacteria We also discuss what the harmful effects of bacteria are

89 How bacteria are useful for usbull Bacteria is helpful in many ways forhuman being i) Production of medicine - antibiotics vaccine etcii) Formation of curd by lactobacillusiii)Nitrogen fixation in Leguminousplant by Rhizobiumiv) Increase soil fertility by absorbingatmospheric nitrogen and convert it into nitrates and nitritesv) Cleaning the environment by converting the complex substances into simple substancesvi) Tanning of leathervii) Retting of Fibersviii) Formation of compost by acting onanimal dung and agricultual cases1x) Biogas production by decomposingplant and animal wastex)Help In Nutrition by producing vitamiacutemBand kx1) Some bacteria are used to give specialflavour to tea coffee and coccaQ10- Name some diseases and there causativebacteriabull Diseasescausative bacteria1 CholeraVibrio cholerae2 Tuberculosis - Mycobacterium tuberculosis3 Diptheria -Corynebacteriumdiphtheriae4 Pneumonia - Streptococcus pneumoniae

Math Number system

Chapter Fraction

Study item Using lsquoofrsquoThe word lsquoofrsquo between any two fractions is to be used as multiplicationExample 57 of 56 = 57 times 56 = 5times8 = 40Study item Using BODMASThe word lsquoBODMASrsquo is the abbreviation formed by taking the initial letters of six operations(i)Bracket (ii) of (iii) Division (iv) Multiplication (v) Addition (vi) SubtractionAccording to BODMAS rule First of all the terms inside Bracket must be simplified then lsquoofrsquo lsquoDivisionrsquo lsquoMultiplicationrsquo lsquoAdditionrsquo lsquosubtractionrsquo

Study item Removal of Brackets

There are four Brackets of algebra in Mathematics In a complex expression four types of brackets are used Order of removing the brackets is first ----- then ( ) then finally [ ]

Class VIIISubject Topic Summary Execution

Chemistry Hydrogen General methods of preparation of hydrogen

By the action of dilute acids on metals

Calcium Reacts readily to form chloride salt and hydrogen

Ca + 2HCl rarr CaCl₂ + H₂uarr

Magnesium

Aluminium

Zinc

React readily to form salt and hydrogen

Mg + 2HCl rarr MgCl₂ + H₂uarr2Al + 6HCl rarr 2AlCl₃ + 3H₂uarrZn + 2HCl rarr ZnCl₂ + H₂uarr

Question 4 ) Give reasons for the following

(a) Hydrogen be used as a fuel

Solution

Hydrogen is used as a fuel because it has a high heat of combustion Some significant fuels are coal gas water gas and liquid hydrogen

(b) Though hydrogen is lighter than air it cannot be collected by downward displacement of air

Solution

Hydrogen is lighter than air so it is possible to collect the gas by downward displacement of air But it is not safe to do so since a mixture of hydrogen and air can lead to an explosion

(c) A pop sound produced when hydrogen is burnt

Solution

Impure hydrogen gas burns in air with a pop sound This is because of the presence of impurities in it

(d) Helium replaced hydrogen in weather observation balloons

Solution

It forms a mixture with air that can explode when there is a small leakage of hydrogen in a balloon So helium has replaced hydrogen

(e) Nitric acid not used for the preparation of hydrogen gas

Solution

(e) By the action of nitric acid on metals hydrogen cannot be produced because it also releases nitrous oxide and nitric oxide and oxides the hydrogen to form water

Biology Chp-2 Reproduction in plants

Today we discuss different methods of artificial propagation like cutting-rose sugercane Layering ndashguava lemon china rose etc Grafting- mango apple etcMicropropagation ndashorchid asparagus etcWe also discuss about advantages and disadvantages of vegetative propagation

Q7 Define the following terms i) Explant In tissue culture techniquea tiny piece of bud shoot or any other partof plant from where new tissue develop ii) Callus The cells of the tissue divide andgrow into a mass of undifferentiated cells from explant iii) Plantlet After few days callus differentiate into a small plant with roots and shootQ8 what are the advantages and limitations of tissue culture or micropropagation

Advantages i ) It produacuteces superior quality plantsii)It can be applied to interspecifie hybridsiii) It is useful to grow seedless plants bull Limitations i) It cannot be used for all plantsii)It is not easy to handleQ9 Write advantages of vegetative propagationi) It is a quick and easy method ofproducing new plantsii) This method need less time to matureiii) The new plants are exact copies of the parentiv) it is extremly useful for growing seedlessplants like banana grapes etc Q10 Write some disadvantages of vegetativepropagationi) Dišeases present in the parent plant gettransferred to all in new plantsii) Overcrowding of new plants causes competition for sunlight water and nutrients which affects growth of plantsplant

Physics Chapter 2 Physical Quatites and Measurements

Here We Will Do Some QuestionsRelated To Chapter 2

Select the correct alternative A block of wood of density 08gcm-3 has a volume of 60cm3 The mass of the block is

1 608 g

2 75 g

3 48 g

4 0013 g

Solution 348 g

The density of aluminium is 27g and that of brass The correct statement is

1 Equal masses of aluminium and brass have equal volumes

2 The mass of a certain volume of brass is more than the mass of an equal volume of aluminium

3 The volume of a certain mass of brass is more than the volume of an equal mass of aluminium

4 Equal volumes of aluminium and brass have equal masses

Solution 2 The mass of a certain volume of brass is more than the mass of an equal volume of aluminium

MATHEMATICS Ch 6Sets

Exercise 6(C)1 Find all the subset of each the following sets(i) A = 57 (iii) C = x xisin W x le 2(iv) p p is a letter in the word lsquopoorrsquo

Solution (i) All the subsets of A are ϕ 5 7 57

(iii) All the subsets of C are ϕ 0 1 2 01 02 12 012

(iv) All the subsets are ϕ p o r po or por

4 Given the universal set = -7-3-105689 find (i) A = x xlt2 (ii) B = x -4ltxlt6 Solution

(i) A = -7-3-10(ii) B = -3-105

5 Given the universal set = x xisin N and xlt20 find

(i) A = x x = 3p pisin N (iii) C = x x is divisible by 4 Solution

(i) 369121518 (iii) 481216

6 Find the proper subset of x x2-9x-10 = 0 Solution

ϕ 10 -1

Working x2-9x-10 = 0 rArr x2-(10-1)x-10 = 0

rArr x2-10x+x-10 = 0 rArrx(x-10)+1(x-10) = 0

rArr (x+1) (x-10) = 0

11 Let M = letters of the word REAL and N = letters of the word LARE Write sets M and N in roster form and then state whether (i) M sube N is true (ii) N sube M is true (iii) M = N is true

Solution M = real and N = lareSo (i) Yes (ii) Yes (iii) Yes

English 2 Twelfth Night ndash Shakespeare

A noble man named Orsino in the kingdom of Illyria is deeply in love with a lady called lady Olivia She is in mourning for her dead brother so she will not even think about marriage At this time a sea storm causes a terrible shipwreck and a young lady called Viola is swept onto the shore She thinks that her twin brother Sebastian is drowned A sea captain tells her about Orsino and his love for Olivia Viola wishes to work in Oliviarsquos home but feels she will not be employed So she dresses as a man calls herself Cesario and gets work at the house of OrsinoViola (now Ceasario) is much liked by Orsino and becomes his page She falls in love with Orsino Orsino sends Ceasario to deliver messages to Olivia Olivia herself falls for the beautiful young Ceasario believing Viola to be a man

2 Answer the following questionsa Why does Orsino ask the musicians to play onOrsino asks the musicians to play on because music feeds his desire He calls upon the musicians to play music so that his hunger for love could be replenished with an excess of musicb What does Valentine tell about OliviaWe learn from Valentine that Olivia is in mourning for her brother she wears a veil and has vowed that no one will see her face for another seven yearsand she refuses to marry anyone until thenc From the exchange between Orsino and Valentine what do you think their relationship isValentine is one of orsinod attendants He was sent to Olivia as a messenger of love but was not allowed to speak to here Who is Olivia mourning for and whyOlivia is mourning for her dead brother

Homework Q fHistory and Civics

Growth of Nationalism

Important dates to remember1769-Napoleon born on 15thAugust1789-Fall of Bastille on 14th July and the beginning of the French revolution declaration of the rights of Man on 26thAugust1793-King Louis XVI executed on January 211764-The Sugar Act passed1765-The Stamp act passed1774-The first congress of Philadelphia1776-The declaration of American Independence of on 4th July1777-Defeat of the British at Saratoga1781-Surrender of lord Cornwallis at Yorktown1783-The treaty of Versailles1804-Napoleon becomes the emperor1813-Battle of Leipzig or Battle of nations in which Napoleon was defeated by the Allies1815-Battle of Waterloo June 18 in which Napoleon was defeated and captured1821-Death of Napoleon in StHelena1860-Abraham Lincoln elected President of the USA1861-The civil war began 1864-Abraham Lincoln elected President of the USA for the second time1865-Slavery abolished in the US

Name the following- The queen of Louis XVI

Marie Antoinette The three philosophers of France

VoltaireMontesquieuJean Jacques Rousseau

The British general whose surrender brought the war in America to an endLord Cornwallis

The first president of the USAGeorge Washington (1732-1799)

The first southern state to secede from the unionSouth Carolina

The author of the book lsquoUncle Toms CabinHarriet Beecher Stowe

Homework-Learn

Class IXSubject Topic Summary Execution

Economics

Types of economies Today I am going to share you the concept of economic growth and economic development Few questions will be given from the previous study material dated 942020

Meaning of economic growthAnswer) The term economic growth generally means anincrease in national income or per capita output or income over time It indicates towards quantitative growth of a country

Meaning of economic developmentAnswer) Economic development is defined

as a process whereby the real per capita income of a country increases over time along with fall in poverty ratio unemployment and income inequality etc

Distinguish between economic growth and economic development

Basis Economic growth

Economic development

Scope It has narrow scope as it refer only to rise in per capita income

It has wide concept since it includes qualitative changes as well

Concerned matter

It is concerned with the rise in income

It is concerned with not only rise in income but also reduction of poverty income inequality and unemployment

Focus Economic growth does not focus on economic development

Economic development focus on economic growth plus qualitative changes

Distinguish between capitalist economy and socialist economy

Ownership

Motive

Tool

Means of production are owned and managed by private people

Self interest and profit earning is the main motive

Price mechanism is a main tool to solve the economic problems

Means of production are owned and managed by the government

Social welfare is the main motive

Economic planning by the government is the main tool to solve the economic problem

Competition

Distribution of income

There exist large competition among buyers and sellers

There is existence of large inequalities of income

There is no such competition

There exist less inequalities of income

Math Topic ndash AlgebraChapter -Factorisation

Study item Factorising by taking out common factorSome solved sums from exercise 41

1) (i) 8xy3 + 12x2y2

= HCF of 8xy3 and 12x2y2 is 4xy2

= 4xy2(2y + 3x )

4) (ii) 28p2q2r ndash 42pq2r2

= HCF of 28p2q2r and 42pq2r2 is 14pq2r = 14pq2r (2p - 3r )5) (ii) 14mn + 22m - 62p=HCF of 14mn 22m and 62p is 2= 2(7mn + 11m - 31p)7) (ii) 3a(x2 + y2) + 6b (x2 + y2) = HCF of 3a(x2 + y2) and 6b(x2 + y2 ) is (x2 + y2)= ( x2+ y2 )(3a + 6b )9) (ii) x(x2 + y2 ndash z2 ) + y(-x2ndashy2 + z2 ) ndash z(x2+ y2 ndash z2 )= x(x2 + y2 -z2) ndash y-(x2 + y2 -z2) -z(x2 + y2 ndash z2)=x(x2 + y2-z2) -y( x2 + y2-z2) ndash z (x2 + y2 -z2)= (x2+ y2 ndash z2)(x ndash y ndash z )

Commercial Studies

Introduction to Accounting and Book-keeping

Today I am going to share you the meaning of Accounting and Book-keeping and its related terms bullAccounting bullBook Keeping bullAccountsbullTypes Of Accounts bullAccounting Cycle

bull Meaning of accounting

Ans )Accounting is the art and science of recording classifying and summarising monetary transactions

bull Meaning of Book-keeping

Ans) Bookkeeping is the art of recording business transactions with the view of having a permanent record of them and showing their effect on wealth

bull Meaning of account

Ans) The term account means a record of business transactions concern a particular person of firm asset or income or expense It is a summarised record of all transactions which take place in an accounting year

bull Types of accountsPersonal accounts ndash Personal accounts relating

to person and Organisation are known

as personal accounts Example Ramrsquos Account ABC amp Co Account etc

Real account - The accounts related to tangible and intangible assets are called real accountsExample Cash Account Furniture Account etc

Nominal account- Accounts related to expenses losses incomes and gains are known as nominal accountsExample Wages Account Salary Account Discount Account etc

bull Accounting cycle Accounting cycle refers to a complete sequence of accounting activities It begins with recording of transactions and ends with the preparation of a balance sheet

English 1 Transformation of sentences

Sentences A sentence is a group of words which makes complete sense

a Assertive sentencesb Imperative sentencesc Interrogative

sentencesd Exclamatory sentences

Sentences can be changed from one grammatical form to another without changing the meaning of the sentence This is known as transformation of sentences

Exercise 6Rewrite the following sentences according to the instructions given below without changing their meanings

1 As soon as he saw the beer he jumped into the river ( Begin No sooner)

2 None but brave deserve the fair (Begin the bravehellip)

3 This box is too heavy for me to lift ( Use so hellip That instead of too)

4 No one other than a king can live like James Luxurious ( Begin only James)

5 Oh for the wings of a dove (Begin I wishhellip)

BENGALI(2ND LANGUAGE)

ldquo বঙগভমির পরমি ldquo াইকেল ধসদন দতত

পব13পোসঠ আসোলিচত ৩ পরবোস দৈদসবর বস ীবতোরো Pলিদ স এ লেদ -আকো সত-োলি লেদ তোস - ক) বকতো লেক কোর লেো লেকো কলিবতোর অং ) কোর পরলিত বকতোর এই উলিকত গ) এ লেদ আকো সত বসত কী বলিঝসয়স4 ীবতোরো বসত কী লেবোঝ ঘ ) আসোচয অংসর তোৎপP13 কী

উ -ক ) বকতো স কলিব মোইসক ম3দ দতত

Types of AccountPersonal AccountReal AccountNominal AccountBalance Sheet (opening)

কলিব মোইসক ম3দ দসততর রলিচত বঙগভলিমর পরলিত কলিবতোর অং ) কলিব বঙগী অ13োৎ লেদমোতোর পরলিত কলিবর এই উলিকত গ ) এ লেদ আকো বসত কলিবর মোব লেদী রপ আকো লেক লেবোঝোসো সয়স4 আকো লেসক লেPম তোরো স পসর লেতমলি ীব লেদ রপ আকো লেসক পরো রপ তোরো স পরসত পোসর এই মভোবোর কোই কলিব বসস4 ঘ ) পরবো Pোতরোয় Pলিদ কলিবর লেদ আকো লেসক ীব তোরো রপ পরো স পসর তোসত কলিব লিবনদমোতর দঃলিত কোর মতয লিবসর সবোভোলিবক পলিরলিত এবং মোষ মরী তোই পরবোস Pলিদ তা োর মতয য় তবও কলিব লিবচলিত সব ো কোর পলিবীসত লেকউ অমর য় লিক4ই অকষয় য় দীর লেPম লিচরপরবোমো লেতমলি মোসষর ীবও চমোতোই ীব - সতবধতোই মতয ীব দীসত মোষ লিতয পরবোমো তবও লেPব মোষ আপ কতকসম13র মো3যসম মোসষর মস লিসসদর সথো কসর লিসত পোসর তোরো লিচরভোসবর সয় মোসষর মস লিবরো কসর তোসদর মস3য লেকউ পGভসত লিবী সয় গোসও মোসষর মস তোরো লিতযপলিত লিতযবলিনদত

Hindi 2ndlang

काकीी(लिसयारामशरणगपत)

इस कहानी म क न यह बतान का परयास निकया ह निक बचच अपनी मा स निकतना परम करत ह शयाम अबोध बाक ह वह अपनी मा क मरन क बाद उसन अपनी मा क लिए बहत रोया बाद म उस पता चा निक उसकी मा राम क घर ची गई ह आकाश म उडती हई पतग दकर उस हष हआ निक पतग क दवारा वह अपनी मा को नीच उतारगा इसक लिए वह अपनी निपता की जब स दो बार सवा रपया निनकाकर पतग और दो मोटी सी मन वाी अपन भाई स काकी एक कागज पर लिवा कर पतग म लिशव का दिदयानिनकाकर पतग और दो मोटी सी मन वाी अपन भाई स काकी एक कागज पर लिवा कर पतग म लिचपका दिदयाभोा और शयाम कोठरी म रससी बाधनी रह थ तभी उसक निपता करोध म आकर उन स पछ निक कया उनकी जब स रपया निनकाा हभोा डर क मार बताया निक शयाम इस पतग क दवारा अपनी काकी को राम क यहा स उतारना चाहता हनिवशशवर(शयाम क निपता)न फटी पतग उठाकर दी तो उस पर काकी लिा थावह हत बजि होकर वही ड रह गएउनहोन सोचा निक मन अपन पतर को मारा जोनिक अनजान और निनदष थावह अपनी मा कोनिकतना पयार करता ह

helliphellipContinue to next

Computer Application

Java Programming Prog 1Write a java program to input two numbers from user and display the sum or product of them as per user choice Use switch case statementSolve public class sum_product public static void main(String args[]) Scanner sc=new Scanner(Systemin) int abc Systemoutprintln(ldquoEnter two numbersrdquo) a=scnextInt() b=scnextInt() Systemoutprintln(ldquoPress 1 for sum or 2 for productrdquo)

c=scnextInt() switch(c) case 1 Systemoutprintln(ldquoThe sum will be =rdquo+(a+b)) break case 2 Systemoutprintln(ldquoThe product will be =rdquo+(ab)) break default Systemoutprintln(ldquoWrong Inputrdquo) Home Work - Practice in your computer using bluej

Subject Eng Literature (The Merchant of Venice ndash William Shakespeare)Topic Act I Scene 2 Lines 92 to 126 (End of scene) Date 13th April 2020 (5th Period)

[Students should read the original play and also the paraphrase given in the school prescribed textbook]Summary Questions amp Answers

o After Portia has expressed her opinion about the suitors Nerissa informs that she need not bother about any one of them as they have decided to quit Belmont at the earliest opportunity because they do not believe in trying their luck by the caskets which is the only way of winning Portia

o Nerissa then enquires of Portiarsquos opinion about Bassanio who once visited her in the company of the Marquis of Montferrat and says that she had never come across such an ideal love deserving the fairest lady for his bride

o Portia seems to remember Bassanio quite correctly and says that she agrees with Nerissa At this moment a servant informs Portia that the Prince of Morocco has arrived to try his luck by the caskets

o Portia tells Nerissa that if she could welcome this new suitor as gladly as she says farewell to the previous ones she would be glad of his arrival However if he happens to have the virtues of a saint but the black complexion of a devil she would prefer to have him for religious consolation rather than as a husband

(1) NERISSA You need not fear lady (Line 97-103)

the having any of these lords they have acquainted me with their determinations

which is indeed to return to their home and to

trouble you with no more suit unless you may be wonby some other sort than your fathers imposition depending on the caskets

PORTIA If I live to be as old as Sibylla I will die as chaste asDiana unless I be obtained by the manner of my fatherswill I am glad this parcel of wooers are so reasonablefor there is not one among them but I dote on his veryabsence and I pray God grant them a fair departure

(a) Elucidate the idea expressed in the first speech of the above dialogue

In the first speech Nerissa assures Portia that she need not have any fear of being compelled to marry anyone of the suitors who had lately come to Belmont She informs her that they have all decided to return to their respective countries(b) Illuminate the meaning of the phrase ldquoyour fatherrsquos imposition depending on the casketsrdquo

Nerissa means that the suitors of Portia do not find the conditions imposed by the will of her father to their liking They are too hard for them These conditions are that in the event of a suitor failing to choose the right casket (i) he should never disclose to anybody which casket he chose (ii) he can never marry and (iii) he should take his departure immediately(c) Explain the meaning of the term lsquoSibyllarsquo

lsquoSibyllarsquo is the name given by Romans and Greeks to a prophetess inspired by some deity usually the sun-god Apollo She had a very long life The god Apollo granted her as many years of life as she could hold grains of sand in her hand(d) Elucidate the meaning of the term lsquoDianarsquo

lsquoDianarsquo is the goddess of hunting She is also regarded as a symbol of virginity because she never fell in love and never

married(e) Explain the meaning of the first two lines of Portiarsquos speech

Portia says that even if she is to live for centuries like Sibylla she would not marry except in accordance to her fatherrsquos will She asserts that she would not mind remaining unmarried and untouched by a man like Diana the virgin the goddess of hunting unless a man is able to win her by passing the test laid down by her father

Class XSubject Topic Summary Execution

Hindi 2nd

Langबड घर की बटी( मशी परमचद)

lsquoबड घर की बीटीrsquo कहानी का उददशय मधयम वग की घर समसया को सझा कर सगदिठत परिरवार म मिम जकर परम स रहन का सदश दना ह घर म शानित tानिपत करन की जिजममदारी नारी की होती ह यदिद नारी समझदार ह उसम धय और परिरवार क परनित परम ह तो कोई भी घटना परिरवार को निवघदिटत नही कर सकती या कहानी परिरवार को सगदिठत करत हए परम सौहाद स एक रदसर की भावनाओ को समझ करउनका सहयोग करत हए जीवन यापन करन की पररणा दती ह मशीपरमचदर जी न इस कहानी म सयकत परिरवार का परनितनिनमिधतव निकया ह यह कहानी बनी माधव सिसह जो गौरी पर क जमीदार क उनक दो पतरो की हशरी कठ ा निबहारीशरीकात का निववाह एकजमीदार घरान की पतरी आनदी स हआ थाआनदी न द को ससरा क वातावरण म ढालिया थाएक दिदन आनदी का अपन दवर ा निबहारी स झगडा हो जाता ह दोनो भाई एक रदसर स अग होन की कोलिशश करत हसभी बह आनदी न अपन मधर वयवहार स ा निबहारी को घर छोडकर जान स रोक लिया| इस पर बनी माधव सिसह न कहा निक बड घर की बटी ऐसी ही होती ह जो निबगडा काम बना ती ह अतः शीषक साथक ह बड घर की बटी आनदी ह

helliphelliphelliphellipContinue to nextBiology Topic ndash Chp-1

CellWelcome to new session 2020-21Today we will start with Chpter 1 cell CELL

Protoplasm+Cellmembrane Or Cell wall

Cytoplasm+Neucleus

Cytoplasmic+ CytoplasmicOrganelles Inclutions(mitochondria (food Golgi bodies pigments)Ribosome)

What is cellbull Cell is the structural and functional unit of living organismbull According to number of cells organisms areUnicellular - Amoeba bacteria Multicellular - Rose Mango Tiger HumanSmallest cell -bacteria Longest cell - Nerve cellLargest cell - Ostrich egg cellCells are of different size and shapes according to their functionsQ2Write chief functions of following cellorganelles

Q3What is tonoplastVacuoles covered by a covering called tonoplast

Bengali(2Nd

Language)

ফ ফটক ো ফটক (কলিবতো ) ভোষ মসোপো3 gtPোয়

একটি লেমসয়র ীবস লেপরম লিকভোসব ফসট ওসঠ তো লেদলিসয়স4 কলিব লেপরম Pই য় লেই ময়ই বনত কোস পলিরত য় ফ লেফোটো বো োসফোটো লেটো ব2 কো য় লেমসয়সদর ব gtয13 লেপরসমর 4লিব ফসট উসঠস4 এই কলিবতোয় লেপরম মোষসক মত gtযর মস লেফস লিদসয় পরকষস বাোচোসোর gtয োত বো2োয় কলিবতোয় লেমসয়টির পসব13র দঃসর কো বো সও লেমসয়টি লেই পসর পলিক সত চোয়ো োরী ীবসর কোস4 পরম লেPৌবস লেপরমসক পোবোর পরব ইচছো োকসও তো পসর লেলিতবোচকতোয় পলিরত য় কলিব ভোষ মসোপো3 যোয় লেP ক লেপরসমর

কলিবতোয় ব gtযবহত লিবসষ লিক4 সvর অ13 লেদওয়ো ১) রসবোো= লেP লিবলিভনন রকম ডোকসত পোসর২) ো= পোর ৩) ঠলি = লেচোসর বZ৪)আই বস2ো=অলিববোলিত৫)শইসয় = োলিয়ত কসর৬)োতপাোচ= লিবলিভনন পরকোর৭)দ2োম = v কসর বZ কসর লেদওয়ো৮)লেরলিং =লেোোর দৈতরী লেব2ো৯) বনত= একঋত১০) পাোর = বসকরো2

Organelles Functions

1 Endoplasmic reticulum

2 Mitochondria

3Golgibodies

4 Ribosome5Lysosome

6Plastids

7 Centrosome

i) Supportive framework for the cellii) Synthesis and transpost of proteinsRelease of energy in the form of ATPi) Synthesis and secretion of enzymes hormoneii) Formation of vacuoles lysosomei) Protein Synthesisi) Intracellular digestionii) Destroy foreign substancei )Leucoplast - stores starchii)chloroplast - trap solar energyiii) Chromoplast - imparts colour toflowers amp fruitsi) Initiates and requlates cell division

কলিবতো তোর অ13সক ভোষোয় পরকোো কসর ঘলিরসয় ব যকত কসরস4 লেপরসমর ফতো আর লিবফতো লেক গোঢ় কসর লেদোসো কলিব ভোষ মসোপো3 যোসয়র অলিভবসর অ যলিদক

Economics

Factors of Production

Welcome to the new sessionToday we are going to start the first chapter of Class XThe name of the chapter is Factors of productionBy the name I hope you all can recall a glimpse of what you have learnt in the second chapter of Class IX

NowProduction is the process of creating the various goods and services which are consumed by the people of the country to satisfy their wants

Thus it is the process in which some materials are transformed from one form to another to create utility and value in goods

For example utility can be created by changing the form of a commodity ie

Making of table out of wood by a carpenter for his customer here the wood is getting transformed into table creating utility for his customer and he can also command a price for it

On the other hand Housewives perform very

useful activities at home which create utility but their domestic activities are not included in production because they have no money value

So we can also say that Production denotes two things firstly creation of utility and secondly creation of value

Production is not complete unless it reaches the consumer

An increase in production will increase the economic welfare of the consumers and hence the aim is to raise the production level of the country

Again production of a good or service is only possible if certain resources or

Questions

1 What do you mean by production

Answer Production means the creation of goods and services for the purpose of selling in the market

In fact production involves the transformation of inputs into outputs

Hence production denotes two thingsCreation of utility and creation of valueUtility and value can be created by changing the form by changing the place by changing the time and by rendering services

Example Transformation of raw

materials into finish goods such as potter creates utility by converting mud into utensils assembling of small parts to make bigger machinery

Production also includes services such as distribution and marketing

2 What are the factors of production

Answer Factors of Production refers to the resources and inputs needed for producing goods and servicesThese inputs can be classified as

Land Labour

Capital Enterprise

Land Land is defined to include not only the surface of the earth but also all other free gifts of nature(for example mineral resources forest resources and indeed anything that helps us to carry out the production of goods and services but is provided by

inputs are used together in right proportion

A resource or an input which helps in the process of production to obtain an output is called FACTOR OF PRODUCTION

These factors of production can broadly be categorized into four parts 1LAND 2LABOUR3CAPITAL4ENTERPRISE (ORGANISATION)or Entrepreneur

The above factors are all interdependent on each other and they play a major role in production process

FACTORS OF PRODUCTION

LANDCAPITAL

LABOUR ENTREPRENEUR

nature free of cost)LabourLabour refers to the human efforts that need to be combined with other factors of production for creating an output

CapitalAll man ndash made means of production is called capita example machineries which help in further production Money when used for starting any business for purchasing raw materials machinery tools etc it is regarded as capitalCapital also includes physical capital like factories machineriestoolsbuildingsequipments etcEnterpriseThe task of bearing risks is called enterprise and the person who bears these risks of business is called the entrepreneurThus an entrepreneur is one who organises production takes important decisions regarding production hires and purchases factors of production and bears the risk and uncertainty involved in productionOrganisation refers to the services of an entrepreneur who controls organises and undertakes all risks One who plans organises and manages a business enterprise is an organiser

Physics Chapter 1 Force

Force is an external agent capable of changing the state of rest or motion of a particular body It has a magnitude and a direction The direction towards which the force is applied is known as the direction of the force and the application of force is the point where force is applied The Force can be measured using a spring balance The SI unit of force is Newton (N)

Question 1

State the condition when on applying a force the body has

(a) the translational motion

(b) The rotational motion

Solutions

(a) Translational motion is produced when the body is free to move

(b) Rotational motion is produced when the body is pivoted at a point

Question 2

Define moment of force and state its SI unit

Solutions

The moment of force is equal to the product of the magnitude of the force and the perpendicular distance of the line of action of force from the axis

of rotation

The SI unit of moment of force is Newton times meter

= Newton meter (Nm)

Commercial Studies

Stake holders In this topic you will be come to know about the meaning and concept of stakeholders

How stakeholders are different from shareholders

Questions1 What do you mean by the term stake holdersAnswer) The term stake holders have developed from the words which mean an interest or expected benefit Stakeholders mean all those individuals groups and Institutions which have a state (interest) in the functioning and performance of a commercial organisation or a business enterprise2 What do you mean by share holdersAnswer) The person and Groups who own the shares of the joint stock company by providing capital to the company are called shareholders Shareholders are the internal stakeholders shareholders are one out of several stake holders3 How are shareholders different from stakeholdersAnswer)i) The term shareholders is related to only joint stock company whereas stakeholders are related with all business organisationsii) Stakeholders maybe any individual having financial stake in business organisation whereas a shareholders are those individuals who are holding shares in the company4) How are shareholders different from creditorsAnswer) i) Shareholders are internal stakeholders while creditors are external stakeholdersii) Shareholders invest in the capital of the company whereas creditors give loan to the companyiii) Shareholders are the members of the company with voting rights but creditors are not the members of the company

English 1 Transformation of sentences

Sentences A sentence is a group of words which makes complete sense

e Assertive sentencesf Imperative sentencesg Interrogative sentencesh Exclamatory sentences

Sentences can be changed from one grammatical form to another without changing the meaning of the sentence This is known as transformation of sentences

Exercise 1 Change the following affirmative sentences into Negative sentences

a He is a good manHe is not a bad man

b Ram loves SitaRam is not without love for Sita

c Only he stood first in the classNone but he stood first in the class

d Ankit was wiser than he

He was not so wise as Ankite He did it

He did not fail to do itf As soon as I reached college the

bell rangNo sooner did I reach college than the bell rang

g He finished everythingHe left nothing unfinished

h It always pours when it rainsIt never rains but it pours

Math Topic Commercial MathematicsChapter ndash Goods and services Tax

What is GSTAns It is a abbreviated term of Goods and Service Text which is an indirect tax levied on the sale of goods and rendering servicesSome terms related to GSTDelar Any person who buys goods or services For resale is known as a delar A delar Can be a firm or a companyIntra-state sales Sales of goods and services within the same state or same union territory are called intra- state salesInter-state sales Sales of goods and services outside the state or union territory are called Inter-state sales4) Input GST GST is paid by dealers on purchase of goods and services are called input GST5) Output GST GST is collected from customers on sale of goods and services are called output GST6) Types of GST There are three taxes applicable under GST(i) Central Goods and Services Tax (CGST)(ii) State Goods and Services Tax (SGST) or Union Territory Goods and Services Tax (UTGST) Both these taxes are levied on intra-state sales Here GST is divided equally among central and state governments(iii) Integrated Goods and Services Tax (IGST) IGST is levied on inter- state sales It is also levied on import of goods and services into India and export of goods and services from India

Subject Eng Literature (The Merchant of Venice ndash William Shakespeare)Topic Act III Scene 4 Lines 1 to 44 (Portia hellip To wish it back on you fare you well Jessica)[Students should read the original play and also the paraphrase given in the school prescribed textbook]

Summary Questions amp AnswersIn this scene we suddenly find a new element in the character of Portia We have already seen her possessed of every graceful womanly quality but now she shows that she is capable of rapid decision and determined action She shows this by her sudden resolve to hasten to Venice with a daring scheme for the rescue of Antonio This is an important scene in the dramatic action for it leads up to and renders possible the striking events of the famous trial scene which is one of the greatest striking elements of the play Moreover the fact that all the characters of importance are now assembled together in Venice makes the union of the main plot and the secondary story complete

(1) LORENZO Madam although I speak it in your presence(Line 1-9)

You have a noble and a true conceit

Of god-like amity which appears most strongly

In bearing thus the absence of your lordBut if you knew to whom you show this honourHow true a gentleman you send reliefHow dear a lover of my lord your husbandI know you would be prouder of the workThan customary bounty can enforce you

(a) Where is Lorenzo Why is he here To whom is he referring as lsquoMadamrsquo

Lorenzo is at Portiarsquos residence He had met Salerio on the way and Salerio had begged him to come along with him to

o In this scene Portia Nerissa Lorenzo Jessica and Balthazar appear

o Portia requests Lorenzo and Jessica to be in charge of her house during her absence from Belmont because she and Nerissa have decided to spend the days in meditation and also in visiting the holy places in the neighbourhood of Belmont She has already instructed her people to acknowledge both Lorenzo and Jessica as master and mistress of house during her absence Lorenzo and Jessica gladly agree to look after the house of Portia

handover the letter from Antonio to Bassanio The letter carried the bad news about Antoniorsquos arrest for non-payment of loan taken from Shylock Hence Salerio might have preferred company to break this bad news to Bassanio He is referring to Portia as Madam(b) What does Portia say on hearing the above extract

Portia says that she has never regretted doing good to others Friends who spend a lot of time together and really are there for each other have many traits in common As Antonio is Bassaniorsquos best friend saving him is like saving Bassanio who is like her own soul She asks Lorenzo to take care of management of the house till Bassanio is back(c) What does Portia send with Bassanio and why

On hearing about Antoniorsquos troubles on account of Bassanio her husband Portia immediately sends him with enough gold to repay the debt many times over to Venice to help Antonio out of his misfortune

(2) Lorenzo Madam with all my heart (Line 36-40)

I shall obey you in all fair commands

Portia My people do already know my mindAnd will acknowledge you and JessicaIn place of Lord Bassanio and myselfSo fare you well till we shall meet again

(a) Where are Lorenzo and Portia at this time What lsquofair commandsrsquo are given to Lorenzo

Lorenzo and Portia are at Belmont during this scenePortia reveals to Lorenzo that she has sworn to contemplate in prayer at a monastery around two miles away until her husband returns from Venice She tells him that Nerissa would accompany her and asks him to manage the house with Jessica till things are settled In response Lorenzo tells her that he would be obliged to do whatever she asks him to do(b) Where is Portia actually going and why

Portia tells Lorenzo that she would live a life of contemplation and pray at a monastery which is two miles away from her place In reality Portia plans to go to Venice in disguise with Nerissa and argue the case in defense of Antonio She is very sure that her plan would succeed

ClassXI (ScienceHumanitiesCommerce)Subject Topic Summary Execution

Computer Science

(APC)

Ch ndash 1 Numbers

(Numbers in different bases and

their Arithmatical operations)

Number System In computers Number System is defined as a writing system to represent the numbers in different ways ie we are using different symbols and notations to represent numbers There are four ways we can represent the number ndash Binary Decimal Octal and Hexadecimal

Decimal Number SystemThis number system consist 10 digits These are 0 1 2 3 4 5 6 7 8 amp 9

Binary Number SystemThis number system has only two digits these are 0 and 1 Here 0 stands for off while 1 stands for on

Octal Number SystemThis number system has 8 digits these are 0 1 2 3 4 5 6 amp 7

Hexadecimal Number SystemThis number system has 16 digits these are 0 1 2 3 4 5 6 7 8 9 A B C D E F Here the value of the alphabets are as follows A=10 B=11 C=12 D=13 E=14 F=15

Rules for conversion decimal number to Binary1 Divide the decimal number by 22 If the number will not divide equally by 2 then round down the answer to the nearest whole number (integer)3 Keep a note of the remainder it should be either 0 or 14 Keep repeating the above steps dividing each answer by 2 until you reach zero5 Write out all the remainders from bottom to top This is your binary solution

For example Lets convert 32 to binary 2 32 2 16 - 0 2 8 - 0 2 4 - 0 2 2 - 0 2 1 - 0 0 - 1

The binary equivalent of 3210 is 1000002

Try the follwing youself1 2410

2 4810

3 1210

History GROWTH OF NATIONALISM

The second half of the 19th century witnessed growth of political consciousness and a sense of Nationalism among the IndiansThere were various factors for growth of Indian Nationalism- As a result various political associations were formed in different provinces by the educated Indians Surendranath Banerjee organized a meeting of National conference at Calcutta Ultimately the National Congress was founded in Bombay in 1885This body became the vanguard of Indian struggle for freedom The congress leaders were known as moderates because they followed a policy of prayer and petition A large number of Indian leaders had experienced in political agitation The Political situation of England was also changed Moreover increasing revolutionary activities in Maharashtra Punjab and Bengal became serious concern to the British Government In this

QUESTION1 What do you mean by Nationalism ANSWER 1 Nationalism is defined as loyalty and devotion to own nation especially a sense of national consciousnessQUESTION 2 What are the causes of nationalism ANSWER 2 There were various factors for growth of nationalism

1 Spread of western education2 The progress of vernacular press and

patriotic literature3 The economic exploitation of our

country by the colonial rulers4 International affairs

QUESTION 3 Who organized National conference in Calcutta in 1883 ANSWER 3 Surendranath BanerjeeQUESTION 4 When did Indian National Congress formANSWER 4 Indian National Congress was formed in 1885 in BombayQUESTION 5 Who were ModeratesANSWER 5 The Early Nationalists were also known as Moderates Their emergence marked

background Lord Curzon became Viceroy in India He had no respect for the Indian National Congress

the beginning of the organized national movement in India They believed in British justice and were loyal to them They followed a policy of prayer and petition They demanded constitutional reforms of our country Impotant Moderate leaders were Pherozshah Mehta Dadabhai Naorozi and Surendranath Banerjee etcQUESTION 6 What do you know about Extremism in Indian National movementANSWER 6 In the beginning of 20th century a new class of national leaders emerged in India which was different from the moderate groups They started more aggressive movement against the British empire The goal of extremists was ldquoswarajrdquo Important extremist leaders were Bal Gangadhar Tilak Lala Lajpat Rai Bipin Chandra Pal etcQUESTION 7 Mention the places which were the main centres of Revolutionary movementANSWER 7 Maharashtra Bengal and Punjab

Physics

Chapter Dimensional Analysis

(Summary)

The dimensions of a physical quantity are the powers to which the fundamental units are raised in order to obtain the derived unit of that quantit

The physical quantites lengthmasstime are represented by [L] [M] [T] resp let they are raised to powers ( dimesions) abc resp then any physical quantity can be represented by [ La Mb Tc ] Examples

1 Area area = L x B = [L] x [L] = [M0 L2 T0 ]

2 Density density = massvolume = [M][L3] = [ M L-3]

3 Velocity velocity = distancetime = [L][T] = [LT-1]HW Try to find out dimension of acceleration Acceleration = velocity timeNB One can find the SI Units Using Dimension Analysis Such as for area we have [L2] so its SI unit is m2

Biology Topic ndash Chp-1 The living world

Today we will start the first chapter the living world Here we discuss about the characteristics of living organism and what are the difference between them and nonliving substances We also discuss about the contribution of different Scientists

There are over 500000 species of plants andover a million species of animal are present on earth Some 15000 new species were discovered every yearQ1 What is a living organismbull A living organism is primarily physico -chemical material that demonstrate a high degree of complexity is capable of selfRegulation possesses a metabolism and perpetuates itself through timeQ2 What are the differences between livingand non-livingsi) Compared with non-living living organisms

have more complex organised structure and their use of energy is more controlled amp efficientii) Living things reproduce their own kind by forming new cells which contains copies of their genesiii) Each organism has some degree of homeostasisie it is able to make adjustments so that internal environment remains constantQ3 Write contributions of following Scientists i) Aristotle - One of the first theories in Biology places all living things in a hiearchieii) AV Leeuwenhoek - was the first to observe living single celled organisms under microscopeii) Carolus Linnaeus - developed the binary system for naming of organisms and classificationiii) Geregor Johann Mendel ndash discoverbasic principles of inheritanceHomework i) C Darwin ii)Schleiden

Math Trigonometric functions

1 Overviewi) Trigonometry The word lsquotrigonometryrsquo is derived from the Greek words lsquotrigonrsquo and lsquometronrsquo which means measuring the sides of a triangle An angle is the amount of rotation of a revolving line with respect to a fixed line Usually we follow two types of conventions for measuring angles ie a) Sexagesimal system b) Circular system In Sexagesimal system the unit of measurement is Degree In Circular system the unit of measurement is Radian ii) Relation between degree and radianThe ratio of circumference of a circle to its diameter is always a constant This constant ratio is a number denoted by π which is taken approximately as 227The relationship between degree amp radian measurements is as follows2 right angles = 180deg= π radians1radian = 180degπ=57deg16(approx) 1deg=π180 radianiii) Length of an arc of a circleIf an arc of length s subtends an angle θ radians at the center of a circle of radius r then s=rθiv) Area of a sector of a circleA sector is like a pizza slice of the

Q) Express the following angles in radiana) 45deg b) 40deg3730Ans a) We have 180deg=π radiansi e 45deg= πtimes45180 radian = π4 radiansb) 40deg3730= 40deg37+3060 minute= 40deg 37 +12 minute= 40deg+ 752 minute=40 + 75(2times60) degree=3258 degreeNow 180deg=π radianie 3258 degree= (πtimes325) (180times8) radians = 65π288 radiansQ) A circle has a radius of r=12 meters What is the length of an arc traced out by a 60deg angle in the center of the circleAns In this problem we know both the central angle (60deg) and the radius of the circle (12) All we have to do is plug those values into our equation and we get

s = 2π(12)(60360)s = 24π6s = 4πSo the length of an arc traced out by a 60deg angle in a circle with a radius of 12 meters equals 4π meters asymp 1257 metersQ) Find the area of the sector with a central angle 30deg and a radius of 9cmAns GivenRadius r = 9 cmAngle θ = 30degArea of the sector = θ360degtimesπr2

= 30360degtimes227times92=2121cm2

circle It consists of a region bounded by two radii and an arc lying between the radiiThe area of a sector is a fraction of the area of the circle This area is proportional to the central angle In other words the bigger the central angle the larger is the area of the sectorArea of Sector = θ2 times r2 (when θ is in radians)

Area of Sector = θ times π360 times r2 (when θ is in degrees)

COMMERCE

CLASSIFICTION OF HUMAN ACTIVITIES-ECONOMIC AND NON-ECONOMIC

Welcome to the new sessiontoday we are going to start the first chapter of Class XI The name of the chapter that we are going to start is

lsquoClassification of Human Activities ndasheconomic and non-economicrsquo

Now let us start the chapter by considering human beings and the activities they perform throughout the day

Human activities means all those activities that human beings undertake to satisfy their wants

Human wants on the other hand are the desire of human beings for goods (vegetables fruits rice etc) and services (services of doctors teachers lawyers etc) that they require to live

Now these human activities continue throughout life as human wants are unending unlimited and recurring as human beings desire for better living throughout their lives

Now human activities can be classified into two categories

Human activities

Economic activities Non-economic activities

Economic activities are

Questions1 What are human activities

Answer Human activities mean all those activities that human beings undertake to satisfy their wants

Example A man working in an office

A boy playing in the garden

2What are the characteristics of human activitiesAnswer the characteristics of human activities are as follows

Human activities are undertaken by men women and children and these activities involve human efforts

Human activities are undertaken to satisfy human wants which are unlimited

Human activities continue throughout life

Human activities are performed for both earning money and personal satisfaction

3What is economic activitiesGive example

Answer Economic activities are undertaken by human beings with the object of earning money acquiring wealth and thereby satisfying human wantsExample

Selling of goods by a shop keeper to his customer

A clinic run by a doctor Service of a teacher in school or college

undertaken by human beings with the object of earning money and acquiring wealth

These activities result in the production of economic goods and services

Example Human activities(ie working in factories officesshops) which produce direct economic benefits

Non-economic activities are inspired by human sentiments and emotions such as love for the family desire to help the poor and love for the country

Thus these human activities (eg praying playing sleeping) produce no direct economic benefits and they are also not related to earning money and acquiring wealth

4 What are the characteristics of economic activities

Answer The characteristics of economic activities are as follows

Economic motiveEconomic activities are undertaken to earn money and acquire wealth

ProductiveEconomic activities involve productiondistribution and exchange of goods and services to create wealth

Economic growthEconomic activities determine the level of economic development of a country and standard of living of its citizens

Socially desirableEconomic activities are socially desirable for society

Economic resourcesEconomic activities make use of all the economic resources such landlabourcapital etc

5 What do you mean by non-economic activitiesExampleAnswerNon-economic activities are inspired by human sentiments and emotions such as love for the family desire to help the poor and love for the countryThese activities are not undertaken for monetary gain but for onersquos satisfaction and happinessExample

a mother looks after her children

a student donates blood8 Differentiate between Economic activities and Non-economic activities

Economic activities

Non-economic activities

1to earn living and acquiring wealth2Result can be measured in terms of money

3ExampleBusinessprofession and employment

1 to obtain some satisfaction

2Result cannot be measured in terms of money

3ExampleFamily-orientedreligious socialCultural and national

BUSINESS STUDIES

BUSINESS ENVIRONMENT

Welcome to the new sessionToday we are going to start the first chapter and the name of the chapter is Business Environment

In todayrsquos world every business enterprise is a part of the society It exists and operates in association with various groups in society such as customers suppliers competitors banks and financial institutions government agencies trade unions media and so on All these groups influence the functioning of business in one way or the other They constitute the environment of businessConcept of Business Environment

The term lsquobusiness environmentrsquo refers to the sum total of all individuals institutions and other forces that lie outside a business enterprise but that may influence its functioning and performance

The main features of business environment

Totality of External forces General and Specific forces Interrelatedness Complexity Dynamic Uncertainty Relativity

The Interrelation between business and its environment

The business enterprise is an open system It continuously interacts with its environment It takes inputs

Prepare the following questions from todayrsquos assignment

1 What do you mean by business environment

The term lsquobusiness environmentrsquo means the aggregate of all forces factors and institutions which are external to and beyond the control of an individual business enterprise but they may influence its functioning and performance Business environment is the macro framework within which a business firm a micro unit operates It consists of several interrelated and interacting elements

2 Explain the main features of business environment in brief

Totality of External forces-Business environment is the sum total of all things external to a business environment

General and Specific forces-It includes both the forces general forces are the economic social political legal and technological conditions which indirectly influence all business enterprise Specific forces are the investors customers competitors and suppliers which influence individual enterprise directly

Interrelatedness-Different elements of environment are interrelated for an example growing awareness for health care has increased the demand for health foods

Complexity- Business environment id

(such as raw materials capital labour energy and so on) from its environment transforms them into goods and services and sends them back to the environment

Fig 1 Business Environment Relationship

complex in nature as the elements keep on changing example economic technological and other forces changes in demand for a product and service

Dynamic-Business environment is not static it keeps on changing

Uncertainty- Itrsquos very difficult to predict future events such as technology and fashion which occur fast and frequently

Economics Basic Economic ConceptsSub topic

Microeconomics and

Macroeconomics

Welcome to the new sessiontoday we are going to start the first chapter of Class XI The name of the chapter that we are going to start is Basic Economic concepts

Now Economics covers the study of human activities Human activities are those activities which are performed by humans to satisfy their wants

Thus Human wants are unlimited and therefore economic activities such as production exchange and consumption are needed in order to satisfy those wants

The study of economics is divided largely in two parts which areMicroeconomics and Macroeconomics

SUBJECT- MATTER OF ECONOMICS

MICROECONOMICS MACROECONOMICS

Questions1Who has coined the words micro and macro economics

Answer Ranger Frisch coined the words lsquomicrorsquo and lsquomacrorsquo in 1933 to denote the two branches of economic theory namely microeconomics and macroeconomics

2What is microeconomicsAnswer It is the study of behaviour of individual decision ndash making unit such as consumers firms etc

3 What is macroeconomicsAnswer Macroeonomics is the study of overall economic phenomena like employment national income etc

4 What is the importance of microeconomicsAnswer

Microeconomics helps in formulating economic policies which enhance productive efficiency and results in greater social welfare

It helps the government in formulating correct price policies

It explains the working of a capitalistic economy where individual units(producers and consumers ) are free to take their own decision

Micro means a small part in

microeconomics we do not study the whole economy Hence we study an individual consumer and his or her choices and a producer and his or her profit maximizing decisions in the market Thus it does not mirror what happens in the economy as a whole

Macroeconomics on the other hand studies the economy as a whole It is concerned with aggregate and depicts the entire picture of the economyMacroeconomics deals with the national income aggregate investment aggregate consumption etc

Features of Microeconomics It deals with small

parts of the country Hence it looks at

individual consumers firms and industries

It deals with individual income consumption and savings

It studies the determination of price of any product or factors of production

It deals with the working of market via the price mechanism which is nothing but the determination of price and quantity of a commodity by the forces of demand and supply

Features of Macroeconomics

It deals with the study of the economy as a whole

It is concerned with

5 Give a limitation of microeconomics Microeconomics fails to explain the

functioning of an economy as a whole It cannot explain unemployment illiteracy and other problems prevailing in the country

6 What is the importance of macroeconomics It gives overall view of the growing

complexities of an economic system It provides the basic and logical

framework for formulating appropriate macroeconomic policies (eg for inflation poverty etc )to direct and regulate economy towards desirable goals

7What is the limitation of macroeconomics It ignores structural changes in an

individual unit of the aggregate

8 Differentiate between Microeconomics and Macroeconomics

Microeconomics Macroeconomics

the study of aggregates

National income aggregate savings and aggregate investments are major concepts dealt within macroeconomics style

It studies the determination of general price levels

It investigates into the problem of unemployment and the achievement of employment

It studies the aspect of decision making at the aggregate and national levels

It includes all growth theories whether related to developed or developing economies it also includes the study of economic systems and the working of the economy under different systems

Note Both Micro and macro economics are complementary and should be fully utilized for proper understanding of an economy

1It studies economic aspect of an individual unit2It deals with individual incomeConsumption and savings

3 It facilitates determination of price of any product or factors of production

4 Itrsquos scope is narrow and restricted to individual unit

1It studies the economy as a whole

2It deals with the national income aggregate consumption and aggregate savings3 It facilitates determination of general price level in an economy

4 Itrsquos scope is wide as it deals with economic units on the national level

ACCOUNTS

Introduction to Accounting and Book-keeping

Today I am going to share you the meaning of Accounting and Book-keeping and its related terms bullAccounting bullBook Keeping bullAccountsbullTypes Of Accounts bullAccounting Cycle

bull Meaning of accounting

Ans ) Accounting is the art and science of recording classifying and summarising monetary transactions

bull Meaning of Book-keeping

Ans) Bookkeeping is the art of recording business transactions with the view of having a permanent record of them and showing their effect on wealth

bull Meaning of account

Ans) The term account means a record of

business transactions concern a particular person of firm asset or income or expense It is a summarised record of all transactions which take place in an accounting year

bull Types of accountsPersonal accounts ndash Personal accounts relating

to person and Organisation are known as personal accounts Example Ramrsquos Account ABC amp Co Account etc

Real account - The accounts related to tangible and intangible assets are called real accounts Example Cash Account Furniture Account etc

Nominal account- Accounts related to expenses losses incomes and gains are known as nominal accounts Example Wages Account Salary Account Discount Account etc

bull Accounting cycle Accounting cycle refers to a complete sequence of accounting activities It begins with recording of transactions and ends with the preparation of a balance sheet

Chemistry TopicAtomic Structure

Thomsonrsquos atomic modelThomson (1898) was the first to propose the model of an atomHe proposed that an atom can be regarded as a uniform sphere of positive electricity in which requisite number of electrons are embedded evently to neutralize the positive chargeThis is just like plums embedded in a pudding or seeds evently distributed in red spongy mass of a watermelonThis model of atom is known as ldquoPlum-Pudding modelrdquo or

Q1)What is the fundamental constituents of atomAns Electron Proton and neutrons are the fundamental constituents of atomQ2)What is the value of fundamental unit of electricityAnsThe charge carried by one electron is sad to be the fundamental unit of electricityIts magnitude is 48times10-10esuOr 1602times10-19C Q3)Name the element containing no neutronAnsOrdinary hydrogen atom or protium 1H1

Types of AccountPersonal AccountReal AccountNominal AccountBalance Sheet (opening)

ldquowatermelon modelrdquoThis model could explain the electrical neutrality of an atom but failed to explain the result of scattering experiment carried out by Rutherford in 1911So it was rejected ultimately

Q4)Why is an electron called universal particleAns Itrsquos mass and Charge are independent of its source

EVS Chapter 1 ndash Modes of Existence

Modes of existence When one speaks normally about the mode of existence of some group or individual one refers to their customs their mode of being their ethology their habitat in some way their feeling for a placeDifferent modes of exixtence are ndash

1 Hunting ndashGathering2 Pastoral3 Agricultural4 Industrial

1 Hunting and gathering Hunting and gathering mode of existence is characterized by obtaining food from hunting wild animals including fishing and gathering wild plants From their earliest days the hunter-gatherer diet included various grasses tubers fruits seeds and nuts Lacking the means to kill larger animals they procured meat from smaller game or through scavenging

Societies that rely primarily or exclusively on hunting wild animals fishing and gathering wild fruits berries nuts and vegetables to support their diet are called hunting and gathering societies

At least this used to be practice of human beings before agriculture is invented As their brains evolved hominids developed more intricate knowledge of edible plant life and growth cycles

Q) Write the features of Hunting ndash gathering societiesAns - There are five basic characteristics of hunting and gathering societies

i The primary institution is the family which decides how food is to be shared and how children are to be socialized and which provides for the protection of its members

ii They tend to be small with fewer than fifty members

iii They tend to be nomadic moving to new areas when the current food supply in a given area has been exhausted

iv Members display a high level of interdependence

v Labor division is based on sex men hunt and women gather

Political Science

Introduction to political science

Political science occasionally called politology is a social science which deals with systems of governance and the analysis of political activities political thoughts associated constitutions and political behaviorThe study of political science involves the study of both the

Answer the following questions-1 What is political science

Political science occasionally called politology is a social science which deals with systems of governance and the analysis of political activities political thoughts associated constitutions and political behavior

2 Short notes-

traditional and modern theories of politicsTraditionalClassical political sciencepolitical theory-Traditional political science is the study of politics before Second World War The methodology to study Politics was traditional (legal formaletc) the definition of politics traditional (Politics begins and end with state)area of study (constitution state machinery)was traditionalModern Political scienceModern political theory-Modern Political Theory critically examines the contemporary state of political theory making an assessment of the achievement and limitations of the Behavioural Revolution in its totality and reviews objectively the major paradigms and conceptual frameworks adopted by the disciplineContemporary attempts at the development of an integrated political theory involving the use of both traditional and modern concepts approaches and theories-Around late 1960s several political scientists realized the importance of both the traditional political theory and modern Political theory They began building an integrated theory of politics involving a systematic mixture of traditional and modern studies of politics It was held that the study of a complex and vast field like politics needs both traditional as well as

Classical political theory Modern Political theory

Homework-Learn

modern concepts and approaches for studying itrsquos all aspects

Subject Eng Literature (The Tempest ndash William Shakespeare) Topic Act I Scene 1 Lines 1 to 32 (Line 32 ndash Gonzalo hellip If he be not born to be hanged our case is miserable) Date 13th April 2020 (3rd Period)

[Students should read the original play and also the paraphrase given in the school prescribed textbook]Summary Questions amp Answers

[SUMMARY OF THE ENTIRE SCENE]

o The play starts with the scene of a severe storm at sea Alonso (King of Naples) Sebastian (Alonsorsquos brother) Ferdinand (Alonsorsquos son) Gonzalo Antonio (the usurping Duke of Milan) are in a ship in the midst of the storm

o The mariners are trying their best to control the vessel from running aground and are totally following the orders of their Master the Boatswain They have scant success

o The mariners become extremely unhappy and annoyed when most of the passengers arrive on the deck thereby hampering their effort to save the ship There is serious confrontation between them and the passengers who are part of the Kingrsquos entourage

o The mariners could not save the ship

SUMMING-UP

(i) Vivid description of the scene which gives a realistic description of terror and confusion of a tropical storm

(ii) Shows Shakespearersquos accuracy of knowledge in describing the naval operations and also matters of seamanship

(iii) The opening scene justifies the title ndash The Tempest

UNANSWERED QUESTIONS

(i) The King always travels with his entire fleet including his soldiers Where

(1) GONZALO Nay good be patient (Line 15-26)BOATSWAIN When the sea is Hence What cares these

roarers for the name of the king To cabin silence Trouble us not

GONZALO Good yet remember whom thou has aboardBOATSWAIN None that I more love than myself You are a

councillor if you can command these elements to silence and work

the peace of the present we will not hand a rope more use your authority If you cannot give thanks you have

lived so long and make yourself ready in your cabin for the mischance of the hour if it so hap [To the Mariners]

Cheerly good hearts [To Gonzalo] Out of our way I say

(a) To whom is the boatswain speaking What does he mean by lsquoNone that I more love than myselfrsquo

The Boatswain is speaking to Gonzalo the honest old councilor of the Duke of MilanBy using the words ndash lsquoNone that I love more than I love myselfrsquo means that for the Boatswain nobody is dearer to him than his own life

(b) What were the conditions that made the boatswain react in this way

The Boatswain reacts in this way because the storm is at sea and Alonso King of Naples Sebastian his brother Ferdinand his son Gonzalo Antonio the usurping Duke of Milan on board are in distress and in panic Thus they have rushed to the deck interrupting the work of the mariners

(c) What hope does Gonzalo take from the attitude of the boatswain

The insolent and authoritative attitude of Boatswain makes Gonzalo feel comforted He tells that there are no signs that the Boatswain will be drowned But his facial appearance and attitude shows that he is destined to die on land by hanging which in effect means that all on board will be saved Otherwise all the persons on board are doomed

(d) How can they lsquomake yourself ready in your cabinrsquo For what were they asked to make ready themselves

In order to make themselves ready in their cabin the

were the other ships

(ii) Why was the ship in that area Where was it coming from or going where

(iii) The ship broke apart What happened to those who were in the ship

passengers on board must prepare for death which they will possibly soon have to meetThey can retire to their cabins and offer prayers to the Almighty to save them from drowning

(e) What does the boatswain say when he is asked to be patient What does he order to the royal party

When the boatswain is asked to be patient and remain calm he says that he will be patient only when the storm will be over and the sea will be calm but as long as the storm blows and there is danger to the ship he cannot think of being patient He orders the royal party to go to the cabin and leave the mariners to their work

(2) GONZALO I have great comfort from this fellow (Line 27-36)

Methinks he hath no drowning mark upon him his complexion is perfect

gallows Stand fast good Fate to his hanging Make the rope of his destiny our cable for our own doth little advantage If he be not born to be hanged our case is miserable

(a) Why does Gonzalo regard the Boatswain in the midst of danger

In the midst of danger Gonzalo regards the boatswain because he feels that the Boatswain is a source of comfort and is bent upon to do his work sincerely which in this case is saving the ship and its passengers from the severest of raging storm

(b) What reasons does Gonzalo give when he says that none in the ship will die of drowning

Gonzalo is almost sure that none in the ship will die by drowning His says that there is no mark on the face of the boatswain that indicates that he will die by drowning On the other hand the lines on his face are strong indications that he will be hanged to death Therefore there shall be no danger of the shiprsquos sinking

(c) Explain the following ldquoStand fast good Fate to his hanging Make the rope of his destiny our cable for our own doth little advantage If he be not born to be hanged our case is miserablerdquo

The stated lines mean that if the will of destiny is to be carried out then the ship will not get wrecked and all the passengers will be saved The safety of the passengers therefore depends upon the will of fate being carried out in the case of the boatswain If however the boatswain is not to die by hanging then the passengers are also very unsafe because in that case the ship is likely to sink

(d) What order does the Boatswain give to the sailors

when he re-enters What does he say about the crying of the fellows inside the cabin

The boatswain orders the sailors to bring the topmast lower and bring the ship close to a stationary position with the help of the main sail He says that the fellows inside the cabin are moaning and crying in their distress louder than his voice and louder even than the roaring of the storm

Class XII (ScienceCommerceHumanities) Subject Topic Summary Execution

Computer Science

PropositionalLogic

Propositional logic is a procedure to provide reasoning through statementProposition A ststement that results in True or False is said to be proposition There are two types of propositionSimple proposition amp compound propositionSimple proposioton A simple proposition is one that is not a part of any other proposition Such sentential form of proposition is symbolized with english letters in short For example Ram is a claver student (TrueFalse)Where do you live (Not in True or False)Grapes are sweet (TrueFalse)It rains today (TrueFalse)Here we can see some statements anwer would be true or false but some staements answer can not give in terms of true or false Thus the sentences which can be answered in true or false are known as simple propositionAssigning propositon to a variableThe general syntax to assign propostion to a variable is as followsVariable = Simple propositonFor example A=Ram is a clever studentB= Grapes are sweetC= it rains todayCompound proposition

helliphellipto be continued in next classhelliphellipMath Relation Relation If A and B are two non-empty sets

then a relation R from A to B is a subset of AxB If R A x B and (a b) R then we say that a sube isinis related to b by the relation R written as aRbeg Let A be the set of students of class XII and B be the set of students of class XI Then some of the examples of relation from A to B arei) (a b) AXB a is brother of bisinii) (a b) AXB age of a is more than age of isinb Types of relation In this section we would like to study different types of relations We know that a relation in a set A is a subset of A times A Thus the empty set φ and A times A are two extreme relations For illustration consider a relation R in the set A = 1 2 3 4 given by R = (a b) a ndash b = 10 This is the empty set as no pair (a b) satisfies the condition a ndash b = 10 Similarly R = (a b) | a ndash b | ge 0 is the whole primeset A times A as all pairs (a b) in A times A satisfy | a ndash

Example 1 Let A be the set of all students of a boys school Show that the relation R in A given by R = (a b) a is sister of b is the empty relation and R = (a b) the primedifference between heights of a and b is less than 3 meters is the universal relationSolution Since the school is boys school no student of the school can be sister of any student of the school Hence R = φ showing that R is the empty relation It is also obvious that the difference between heights of any two students of the school has to be less than 3 meters This shows that R = A times A is primethe universal relation Example 2 Show that the relation R in the set 1 2 3 given by R = (1 1) (2 2) (3 3) (1 2) (2 3) is reflexive

b | ge 0 These two extreme examples lead us to the following definitionsDefinition 1 A relation R in a set A is called empty relation if no element of A isrelated to any element of A ie R = φ A times AsubDefinition 2 A relation R in a set A is called universal relation if each element of A is related to every element of A ie R = A times A Both the empty relation and the universal relation are some times called trivial relation Definition 3 A relation R in a set A is called(i) reflexive if (a a) R for every a Aisin isin(ii) symmetric if (a1 a2) R implies that (aisin 2a1)

R for all aisin 1 a2 Aisin(iii) transitive if (a1 a2) R and (aisin 2 a3) R isinimplies that (a1 a3) R for all aisin 1 a2 a3 AisinDefinition 4 A relation R in a set A is said to be an equivalence relation if R is reflexive symmetric and transitive

but neither symmetric nor transitiveSolution R is reflexive since (1 1) (2 2) and (3 3) lie in R Also R is not symmetric as (1 2) R but (2 1) isin notinR Similarly R is not transitive as (1 2) R and (2 3) R but (1 3) R isin isin notinExample 3 Show that the relation R in the set Z of integers given byR = (a b) 2 divides a ndash b is an equivalence relationSolution R is reflexive as 2 divides (a ndash a) for all a Z isinFurther if (a b) R then 2 divides a isinndash b Therefore 2 divides b ndash a Hence (b a) R which shows that R is isinsymmetric Similarly if (a b) R and (b c) R isin isinthen a ndash b and b ndash c are divisible by 2 Now a ndash c = (a ndash b) + (b ndash c) is even (Why) So (a ndash c) is divisible by 2 This shows that R is transitive Thus R is an equivalence relation in ZExample 4 Let L be the set of all lines in a plane and R be the relation in L defined as R = (L1 L2) L1 is perpendicular to L2 Show that R is symmetric but neither reflexive nor transitiveSolution R is not reflexive as a line L1 can not be perpendicular to itself ie (L1 L1) R notinR is symmetric as (L1 L2) Risin

L1 is perpendicular to L2rArr L2 is perpendicular to L1rArr (L2 L1) RrArr isin

R is not transitive Indeed if L1 is perpendicular to L2 and L2 is perpendicular to L3 then L1 can never be perpendicular to L3 In fact L1 is parallel to L3 ie (L1 L2) R isin(L2 L3) R but (L1 L3) Risin notin

Chemistry Solid state Characteristics if Solids(i)The particles are locked in fixed positions they are unable to change their relative positions and this brings a definite shape and volume of a solid(ii)In a solid the constituent particles are held by strong forces of attractionThe forces of attraction may be bonding or non bonding(iii)The constituent particles in a solid pack together as closely as possibleoccupying most of the available space within the solidThus the empty space in a solid is very smallThis makes a solid highly rigid and nearly incompressibleThis also explains why a solid has high density and exhibits slow diffusionClassification of Solids

Q1)Define Crystalline solids AnsA Solid that has a definite geometrical shape and a sharp melting pointand whose constituent particles (atomsmolecules or ions) are arranged in a long range order of definite pattern extending throughout the solid is called a crystalline solidExNaClQ2)Define Amorphous solids AnsA solid that does not have a definite shape and a sharp melting pointand whose constituent particles (atomsmolecules or ions) are not arranged in a definite pattern is called an amorphoussolid

Crystalline solidsAmorphous solids

ExGlassRubberQ3)Classify Crystalline Solids Crystalline Solids

Physics Coloumbrsquos Law (Summary)

Before Going Into Coloumbrsquos Law We Will First Learn What is Charge Properties of Charge and Always remember that charge is quantized ie a body always have static charge of magnitude equal to some integral multiple of fundamental electronic charge e= 16 x 10- 19 C

Charge is the property of matter that causes it to produce and experience electrical and magnetic effects The study of the electrical charges at rest is called electrostatics When both electrical and magnetic effects are present the interaction between charges is referred to as electromagnetic

There exist two types of charges in nature positive and negative Like charges repel and unlike charges attract each other

The type of charge on an electron is negative The charge of a proton is the same as that of an electron but with a positive sign In an atom the number of electrons and the number of protons are equal The atom is therefore electrically neutral If one or more electrons are added to it it becomes negatively charged and is designated as negative ion However if one or more electrons are removed from an atom it becomes positively charged and is called a positive ion

The excess or deficiency of electrons in a body gives the concept of charge If there is an excess of electrons in a body it is negatively charged And if there is deficiency of electrons the body becomes positively charged Whenever addition or removal of electrons takes places the body acquires a charge

The SI Unit of charge is coulomb (C) In SI units the current is a fundamental quantity having a unit of ampere (A) The unit of charge is defined in terms of the unit of current Thus one coulomb is the charge transferred in one second across the section of a wire carrying a

Ionic SolidsMetallicSolids

Molecular Solids

current of one ampere

As q = It we have1 C = (1 A) (1 s)

The dimensions of charge are [A T]

Properties of Charge

(1) Quantization of Charge Electric charge can have only discrete values rather than any value That is charge is quantized The smallest discrete value of charge that can exist in nature is the charge on an electron given as

e = plusmn 16 x 10- 19 C

This is the charge attained by an electron and a protonA charge q must be an integral multiple of this basic unit That is

Q = plusmn ne where n = 1 2 hellip

Charge on a body can never be (frac12)e (23)e or 57e etcWhen we rub a glass rod with silk some electrons are transferred from the rod to the silk The rod becomes positively charged The silk becomes negatively charged The coulomb is a very large amount of charge A typical charge acquired by a rubbed body is 10 - 8 C

Biology Reproduction in organisms

Welcome to this new session 2020-21Today in this first chapter we mainly discuss about reproduction types needs and life span of some organismsWe also discuss about difference between sexual and asexual reproduction

Q1 What is reproductionReproduction is defined as a biological processin which an organism gives rise to young onessimilar to itselfQ2 What are the needs of reproductionbulli) Reproduction maintain life on earthii) It enables the continuity of the species generation after generationiii) It creates genetic variation among populationsQ3 Define Life span and write some orgnisms life spanbull Life span is the period from birth to

the natural death of an organism- OrganismsLife span1 Butterfly 1 - 2 weeks2 Fruit fly 30 days3Dog 10-13 years4 Rose5-7 years5 Tortoise100-150 years6 Banyan Tree -200 - 250 yearsQ4 Reproduction is of two types in case ofanimals but in case of plants vegetative propagation is also present

Asexual Reproduction Sexual Reproductioni) Always uniparentalii) Gametes are not involvediii) Only mitotic division involvediv) Somatic cells of parents are involvedv) Offsprings are genetically similar to the parents

i) Usually biparentalii) Gametes are involvediii) Meiosis occurs during gametogenesis Mitosis occurs after fertilisationiv) Germ cells of the parents are involvedv) offsprings are genetically different from the parents

COMMERCE BUSINESS ENVIRONMENT

Welcome to the new sessiontoday we are going to start the first chapter of Class XII The name of the chapter is Business Environment

Already many of you have got some idea about the word business environment form the first chapter of business studies in class XI

In todayrsquos world every business enterprise is a part of the society It exists and operates in association with various groups in society such as customers suppliers competitors banks and financial institutions government agencies trade unions media and so on All these groups influence the functioning of business in one way or the other They constitute the environment of businessConcept of Business Environment

The term lsquobusiness environmentrsquo refers to the sum total of all individuals institutions and other forces that lie outside a business enterprise but that may influence its functioning and performance

The main features of business environment Totality of External forces General and Specific forces Interrelatedness Complexity Dynamic Uncertainty

Prepare the following questions from todayrsquos assignment

2 What do you mean by business environment

The term lsquobusiness environmentrsquo means the aggregate of all forces factors and institutions which are external to and beyond the control of an individual business enterprise but they may influence its functioning and performance Business environment is the macro framework within which a business firm a micro unit operates It consists of several interrelated and interacting elements

2 Explain the main features of business environment in brief

Totality of External forces-Business environment is the sum total of all things external to a business environment

General and Specific forces-It

Relativity

The Interrelation between business and its environment

The business enterprise is an open system It continuously interacts with its environment It takes inputs (such as raw materials capital labour energy and so on) from its environment transforms them into goods and services and sends them back to the environment

Fig 1 Business Environment Relationship

includes both the forces general forces are the economic social political legal and technological conditions which indirectly influence all business enterprise Specific forces are the investors customers competitors and suppliers which influence individual enterprise directly

Interrelatedness-Different elements of environment are interrelated for an example growing awareness for health care has increased the demand for health foods

Complexity- Business environment id complex in nature as the elements keep on changing example economic technological and other forces changes in demand for a product and service

Dynamic-Business environment is not static it keeps on changing

Uncertainty- Itrsquos very difficult to predict future events such as technology and fashion which occur fast and frequently

Business Studies

Human Resources Management

Human resource of an organisation are the aggregate of knowledge skills attitudes of people working in it

The management system which deals with human resources is called human resource management

Features of HRMbullComprehensive functionbullPeople-oriented

Question1) What do you mean by human

resource management Answer) Human resource management may be defined as that field of Management which has to do with planning organising and controlling the functions of procuring developing maintaining and utilising the labour force

bullAction oriented bullPervasive function bullContinuous function

2) Explain the features of HRM in brief

Answer)bullHuman Resource Management is concerned with managing people at work bull Human Resource Management is concerned with employees which bring people and organisations together so that the goals of each are met bullHuman resource management considered every employees as an individual and also promote their satisfaction and growth bull Human resource management is inherent in all organisations and at all levelsbullManagement of human resources are ongoing on never ending process which requires a constant alertness and Awareness of human relations

3) ldquoHR function is said to be pervasiverdquowhy

Answer) Human resource management is required in all organisations whether it is private or government organisations armed forces sports organisations etc It permeatsall the functional areas like production marketing finance research etc This from this feature of human resource management it can be said that it is pervasive in nature

Economics Demand Q1DEFINITION OF DEMANDIn economics demand is the quantity of a good that consumers are willing and able to purchase at various prices during a given period of timeQ2DEMAND CURVEIn economics a demand curve is a graph depicting the relationship between the price of a certain commodity and the quantity of that commodity that is demanded at that pricQ3LAW OF DEMANDIn microeconomics the law of demand states that conditional on all else being equal as the price of a good increases quantity demanded decreases conversely as the price of a good decreases quantity demanded increasesQ4ASSUMPTION of LAW OF DEMAND(i)No change in price of related commodities(ii) No change in income of the consumer(iii) No change in taste and preferences customs habit and fashion of the consumer( No expectation regarding future change in priceQ5MARKET DEMAND SCHEDULEIn economics a market demand schedule is a tabulation of the quantity of a good that all consumers in a market will purchase at a

given price At any given price the corresponding value on the demand schedule is the sum of all consumersrsquo quantities demanded at that priceQ6INDIVIDUAL DEMAND SCHEDULEIndividual demand schedule refers to a tabular statement showing various quantities of a commodity that a consumer is willing to buy at various levels of price during a given period of timeQ7 FACTORS AFFECTING INDIVIDUAL DEMAND FOR A COMMODITY

The factors that influence a consumerrsquos decision to purchase a commodity are also known as determinants of demand The following factors affect the individual demand for a commodity1 price of the commodity2 price of related goods3 income of buyer of the commodity4 tastes and preferences of the buyer1 Price of the CommodityYou must have observed that when price of a commodity falls you tend to buy more of it and when its price rises you tend to buy less of it when all other factors remain constant (lsquoother things remaining the samersquo) In other words other things remaining the same there is an inverse relationship between the price of a commodity and its quantity demanded by its buyers This statement is in accordance with law of demand which you will study in the later part of this lesson Price of a commodity and its quantity demanded by its buyers are inversely related only when lsquoother things remain the samersquo So lsquoother things remaining the samersquo is an assumption when we study the effect of changes in the price of a commodity on its quantity demanded2 Price of Related goodsA consumer may demand a particular good But while buying that good heshe also asks the price of its related goods Related goods can be of two types-(i) Substitute goods(ii) Complementary goods While purchasing a good prices of its substitutes and complements do affect its quantity purchased(i) Price of Substitute Goods Substitute goods are those goods which can easily be used in place of one another for satisfaction of a particular want like tea and coffee An increase in price of substitute good leads to an increase in demand for the given commodity and a decrease in price of substitute good leads to a decrease in demand for the given commodity It means demand for a given commodity is directly affected by change in price of substitute goods For example if price of coffee increases the demand for tea will rise as tea will become relatively cheaper in comparison to coffee(ii) Price of Complementary goods Complementary goods are those goods which are used together to satisfy a particular want like car and petrol An increase in the price of complementary goods leads to a decrease in demand for the given commodity and a decrease in the price of complementary goods leads to an increase in demand for the given commodity For example if price of petrol falls then the demand for cars will increase as it will be relatively cheaper to use both the goods together So demand for a given commodity is inversely affected by change in price of complementary goods3 Income of the Buyer of CommodityDemand for a commodity is also affected by income of its buyer However the effect of change in income on demand depends on the nature of the commodity under consideration In case of some goods like full cream milk fine quality of rice (Basmati rice) etc demand for these commodities increases when income of the buyer increases and

demand for these commodities decreases when income of the buyer decreases Such goods whose demand increases with the increase in income of the buyer are called normal goods But there are some goods like coarse rice toned milk etc whose demand decreases when income of buyer increases and their demand increases when income of the buyer decreases Such goods whose demand decreases with the increase in income of the buyer are called inferior goods Suppose a consumer buys 10 Kgs of rice whose price is ` 25 per Kg He cannot afford to buy better quality of rice because the price of such rice is ` 50 per Kg The consumer is spending ` 250 per month on the purchase of rice Now if income of the consumer increases and he can afford ` 350 on purchase of 10 Kg of rice Now he can afford to buy some quantity of rice say 6 Kgs whose price is ` 25 per Kg and may buy 4 Kgs of rice whose price is ` 50 per Kg Thus he will buy 10 Kgs of rice by spending ` 350 per month Therefore we may conclude that demand for normal goods is directly related to the income of the buyer but demand for inferior goods is inversely related to the income of the buyer4 Tastes and Preferences of the BuyerThe demand for a commodity is also affected by the tastes and preferences of the buyers They include change in fashion customs habits etc Those commodities are preferred by the consumers which are in fashion So demand for those commodities rises which are in fashion On the other hand if a commodity goes out of the fashion its demand falls because no consumer will like to buy it(5) Number of Buyers in the Market(Population)Increase in population raises the market demand whereas decrease in population reduces the market demand for a commodity Not only the size of population but its composition like age (ratio of males females children and old people in population) also affects the demand for a commodity It is because of needs of children young old male and female population differs(6) Distribution of Income and WealthIf the distribution of income and wealth is more in favour of the rich demand for the commodities preferred by the rich such as comforts and luxuries is likely to be higher On the other hand if the distribution of income and wealth is more in favour of poor demand for commodities preferred by the poor such as necessities will be more(7) Season and Weather ConditionsThis is generally observed that the demand for woolens increases during winter whereas demand for ice creams and cold drinks increases during summer Similarly market demand for umbrellas rain coats increases during rainy seasonQ8 REASONS FOR OPERATION OF LAW OF DEMAND WHY DEMAND CURVE SLOPES DOWNWARDNow we will try to explain why does a consumer purchase more quantity of a commodity at a lower price and less of it at a higher price or why does the law of demand operate ie why does the demand curve slope downwards from left to right The main reasons for operation of law of demand are1 Law of Diminishing Marginal UtilityAs you have studied earlier law of diminishing marginal utility states that as we consume more and more units of a commodity the utility derived from each successive unit goes on decreasing The consumer will be ready to pay more for those units which provide him more utility and less for those which provide him less utility It implies that he will purchase more only when the price of the commodity falls2 Income Effect

When price of a commodity falls purchasing power or real income of the consumer increases which enables him to purchase more quantity of the commodity with the same money income Let us take an example Suppose you buy 4 ice creams when price of each ice cream is ` 25 If price of ice creams falls to ` 20 then with same money income you can buy 5 ice creams now3 Substitution EffectWhen price of a commodity falls it becomes comparatively cheaper as compared to its substitutes (although price of substitutes has not been changed) This will lead to rise in demand for the given commodity For example if coke and Pepsi both are sold at ` 10 each and price of coke falls Now coke has become relatively cheaper and will be substituted for Pepsi It will lead to rise in demand for coke4 Change in Number of BuyersWhen price of a commodity falls some old buyers may demand more of the commodity at the reduced price and some new buyers may also start buying this commodity who were not in a position to buy it earlier due to higher price This will lead to increase in number of buyers when price of the commodity falls As a result demand for the commodity rises when its price falls5 Diverse Uses of a CommoditySome commodities have diverse uses like milk It can be used for drinking for sweet preparation for ice cream preparation etc If price of milk rises its use may be restricted to important purpose only This will lead to reduction in demand for other less important uses When price of milk falls it can be put to other uses also leading to rise n demand for itQ9 EXCEPTIONS TO THE LAW OF DEMANDYou have studied in law of demand that a buyer is willing to buy more quantity of a commodity at a lower price and less of it at a higher price But in certain circumstances a rise in price may lead to rise in demand These circumstances are called Exceptions to the Law of Demand Some important exceptions are1 Giffen GoodsGiffen goods are special type of inferior goods in which negative income effect is stronger than negative substitution effect Giffen goods do not follow law of demand as their demand rises when their price rises Examples of Giffen goods are jowar and bajra etc2 Status Symbol GoodsSome goods are used by rich people as status symbols eg diamonds gold jewellary etc The higher the price the higher will be the demand for these goods When price of such goods falls these goods are no longer looked at as status symbol goods and tehrefore therir demand falls3 NecessitiesCommodities such as medicines salt wheat etc do not follow law of demandbecause we have to purchase them in minimum required quantity whatever their price may be4 Goods Expected to be ScarceWhen the buyers expect a scarcity of a particular good in near future they start buying more and more of that good even if their prices are rising For example during war famines etc people tend to buy more of some goods even at higher prices due to fear of their scarcity in near future

Political Science

Constitution of India-The

Preamble

The preamble-

Preamble-

The preamble is the most precious part of the constitution We the people of India having solemnly resolved to constitute India into a Sovereign Socialist Secular Democratic Republic and to secure to all its citizensA preamble is an introductory and expressionary statement in a document that explains the documents purpose and underlying philosophy When applied to the opening paragraphs of a statute it may recite historical facts pertinent to the subject of the statuteNature and purpose of the constitution-Purpose of the Constitution dictates permanent framework of the government to form a more perfect union to establish justice and ensure peace of thenationconstitution provide principles how the government can run itself following the rules and laws written in the constitution of each state keeps them balanced

Answer the following questions-

1 What is preambleA preamble is an introductory and expressionary statement in a document that explains the documents purpose and underlying philosophy2 What is the nature and

purpose of the constitutionConstitution dictatespermanent framework of the government to form a more perfect union to establish justice and ensure peace of the nation

Homework-Learn

Accounts Compatibilty mode

1MEANING OF PARTNERSHIPPartnership is a form of business organisation where two or more persons join hands to run a business They share the profits and losses according to the agreement amongst them According to the Indian Partnership Act 1932 ldquoPartnership is relation between persons who have agreed to share profits of a business carried on by all or any one of them acting for allrdquo For example one of your friends has passed class XII from National Institute of Open Schooling (NIOS) and wants to start a business Heshe approaches you to join in this venture Heshe wants you to contribute some money and participate in the business activities Both of you if join hands constitute a partnership2CHARACTERISTICS1048698 Agreement A partnership is formed by an agreement The agreement may be either oral or in writing It defines the relationship between the persons who agree to carry on business It may contain the terms of sharing profit and the capital to be invested by each partner etc The written agreement is known as partnership deed1048698 Number of persons There must be at least two persons to form a partnership

The maximum number of partners in a partnership firm can be 50 according toCompanies Act 20131048698 Business The Partnership is formed to carry on business with a purpose of earning profits The business should be lawful Thus if two or more persons agree to carry on unlawful activities it will not be termed as partnership1048698 Sharing Profits The partners agree to share profits in the agreed ratio In caseof loss all the partners have to bear it in the same agreed profit sharing ratio10486981048698Mutual Agency Every partner is an agent of the other partners Every partner can bind the firm and all other partners by hisher acts Each partner will be responsible and liable for the acts of all other partners10486981048698Unlimited liability The liability of each partner except that of a minor is unlimited Their liability extends to their personal assets also If the assets of the firm are insufficient to pay off its debts the partnersrsquo personal property can be used to satisfy the claim of the creditors of the partnership firm10486981048698Management All the partners have a right to mange the business However they may authorize one or more partners to manage the affairs of the business on their behalf10486981048698Transferability of Share No partner can transfer hisher share to any one including hisher family member without the consent of all other partners3PARTNERSHIP DEEDAgreement forms the basis of partnership The written form of the agreement is which a document of partnership is It contains terms and conditions regarding the conduct of the business It also explains relationship between the partners This document is called partnership deed Every firm can frame its own partnership deed in which the rights duties and liabilities of the partners are stated in detail It helps in settling the disputes arising among the partners during the general conduct of business 4CONTENTS OF PARTNERSHIP DEEDThe partnership deed generally contains the following (i) Name and address of the partnership firm(ii) Nature and objectives of the business(iii) Name and address of each partner(iv) Ratio in which profits is to be shared(v) Capital contribution by each partner(vi) Rate of Interest on capital if allowed(vii) Salary or any other remuneration to partners if allowed(viii) Rate of interest on loans and advances by a partner to the firm(ix) Drawings of partners and interest thereon if any(x) Method of valuation of goodwill and revaluation of assets and liabilities on the reconstitution of the partnership ie on the admission retirement or death of a partner(xi) Settlement of disputes by arbitration(xii) Settlement of accounts at the time of retirement or death of a partner5IN ABSENCE OF PARTNERSHIP DEEDThe partnership deed lays down the terms and conditions of partnership in regard to rights duties and obligations of the partners In the absence of partnership deed there may arise a controversy on certain issues like profit sharing ratio interest on

capital interest on drawings interest on loan and salary of the partners In such cases the provisions of the Indian Partnership Act becomes applicableSome of the Issues are(i) Distribution of Profit Partners are entitled to share profits equally(ii) Interest on Capital Interest on capital is not allowed(iii) Interest on Drawings No interest on drawing of the partners is to be charged(iv) Interest on Partnerrsquos Loan A Partner is allowed interest 6 per annum on the amount of loan given to the firm by himher(v) Salary and Commission to Partner A partner is not entitled to anysalary or commission or any other remuneration for managing the business

History TOPIC-TOWARDS INDEPENDENCE AND PARTITION THE LAST PHASE (1935-1947)

SUB TOPIC-IMPORTANT POLITICAL DEVELOPMENTS ndash GROWTH OF SOCIAL IDEAS

Socialism is a political social and economic philosophyLike in other parts of the world the Russian revolution of 1917 served as a great inspiration for revolutionaries in India who at that time were engaged in the struggle for liberation from British ruleSocialist ideas led to the formation of communist party of IndiaJAWAHARLAL NEHRU Among the early Congress leaders Jawaharlal Nehru was very much impressed and influenced by the Socialist ideas He also learnt about the Economic activities of the Soviet Union after the Bolshevic Revolution 1917 He made full use of them in IndiaThe election of Jawaharlal Nehru and Subhas Chandra Bose showed the Left wing tendency within CongressJawaharlal Nehru demanded economic freedom along with political freedom of the people in order to end the exploitation of masses

Nehrus working committee included three socialists leaders The Lucknow session was a landmark in the evolution of socialist ideas of the congressSUBHAS CHANDRA BOSE ndash Subhas Chandra Bose had socialist leaning Both Jawaharlal Nehru and Subhas Chandra Bose were known as leftist Congress men Later on National Congress divided into Leftist and rightist campCONGRESS SOCIALIST Within the Congress some leaders formed the Congress Socialist partyPattavi Sitaramyya Sardar Patel Rajendra Prasad had hostile attitude towards the Congress Socialist partyJawaharlals attitude was hesitant

1 QUESTION ndash Mention name of two Congress leaders who had socialist leaning

1ANSWER ndash Subhas Chandra Bose and Jawaharlal Nehru2QUESTION- In which session of the congress Jawaharlal elaborated his Socialist ideas2 ANSWER ndash Lucknow and Faizpur Session in December 1935 and 19363QUESTION ndash Why Congress was sharply divided into leftist and rightist camp 3ANSWER ndash Subhas Chandra Bosersquos attempt to seek re election for congress presidentship in 1939sharply divided the National Congress into Leftist and Rightist camp4 QUESTION ndash Who was MN Roy 4 ANSWER ndash Manabendra Roy first formed the Communist Party of India outside the country at Tashkent in 19205QUESTION ndash Who formed the Congress Socialist Party within the Congress5 ANSWER ndash Jaya Prakash Narayan Achyut Patwardhan Acharya Narendra Dev Ram Mohan Lohia Aruna Asaf Ali6QUESTION ndash When was the Congress Socialist Party formed What was its object6 ANSWER ndash 1934The Congress Socialist Party sought to work out socialist programme through the Congress They joined hands with the Congress and wanted to carry

Subhas Chandra Bose being expelled from the congress after the Tripuri rift he formed Forward BlockThere were basic differences between the Congress Socialists and the communistsTRADE UNION ACTIVITIES Maximum working class people lived in Bombay and Calcutta The working and living conditions of those workers were very miserable In this situation Shasipada Banerjee NM Lokhande protested against the oppression of the working class peopleThe first Trade Union Madras Labour Union was formed in 1918 by BP WadiaIndustrial strikes took place in Kanpur Calcutta Madras Jamshedpur and Ahmedabad AITUC was formed in Bombay in 1927 The growth of Trade union among the workers was slow because of the fear of the dismissal of the jobIn the mean time the Moderates as well as Communists left AITUC and formed separate organization

on National struggle with the help of workers and peasant class of the society7 QUESTION ndash What was the name of the party founded by Subhas Chandra Bose7 ANSWER- Forward Block8QUESTION ndash Who was Shasipada Banerjee8 ANSWER ndash Shasipada Banerjee was a radical Brahmo He founded a working menrsquos club to protest against exploitation of the British rulers towards the working class of India9 QUESTION ndash What was the weekly published by NM Lokhande9ANSWER- Dinabandhu10 QUESTION ndash Who founded Bombay Mill-Hands Association and in which year10 ANSWER- NM Lokhande in189011 QUESTION- Who was BP WadiaANSWER- BPWadia was the founder of Madras Labour Union in191812 QUESTION- What was the name of the first labour union of India12 ANSWER- Madras Labour Union13 QUESTION Who founded the Majur Mahajan 13 ANSWER GANDHIJI14 QUESTION What was the full form of AITUC When it was formed14 ANSWER All India Trade Union Congressin 192715QUESTION Who formed the Red Trade Union Congress and in which year15ANSWER The Communists formed the Red Trade Union Congress16 QUESTION What do you mean by Socialism16 ANSWER Socialism describes any political and economic theory that says the community rather than individuals should own and manage property and natural resources

Subject Eng Literature (The Tempest ndash William Shakespeare) Topic Act III Scene 3 Lines 1 to 52 (Line 52 ndash Brother my lord the Duke Stand to and do as we) Date 13th April 2020 (4th Period)

[Students should read the original play and also the paraphrase given in the school prescribed textbook]Summary Questions amp Answers

o Alonso Sebastian Antonio Gonzalo Adrian Francisco and others wandered about the island in search of Ferdinand and gets tired and hungry of the toil and at the same time gives up all hope of finding him

o Antonio and Sebastian are happy that Alonso is out of hope and decide to make another attempt on his life that night when being so tired they will be sleeping soundly

o Suddenly a solemn and strange music is heard in the air and several strange shapes enter bringing in a banquet These strange shapes then dance round it with gestures of salutation and then inviting the King to eat they depart

o Seeing this strange scene all are inclined to believe the tales told by travelers that there truly are ldquounicornsrdquo and ldquothe phoenixrsquo thronerdquo

1 ALONSO What harmony is this My good friends hark (L18-27)

GONZALO Marvellous sweet music

[Enter several strange shapes bringing in a banquet

they dance about it with gentle actions of salutation

and inviting the King and his companions to eat they depart]ALONSO Give us kind keepers heavens What were theseSEBASTIAN A living drollery Now I will believe

That there are unicorns that in Arabia

There is one tree the phoenixrsquo throne one phoenix

At this hour reigning thereANTONIO Ill believe both

And what does else want credit come to me

And Ill be sworn rsquotis true Travellers neer did lie

Though fools at home condemn rsquoem

(a) How did Prospero present an amazing spectacle before Alonso and his companions

Using his magic powers Prospero ordered strange shapes to lay a banquet before Alonso and his companions The shapes brought several dishes with tasty eatables in them They placed the dishes on a table before Alonso and his companions Then the strange shapes began to dance gracefully around the banquet While dancing they made gestures inviting them to eat the food Then suddenly the shapes disappeared(b) Who were the guests at the strange banquet Describe the lsquoliving drolleryrsquo

Alonso Sebastian Antonio Gonzalo Adrian and Francisco were the guests at the strange banquet

The term ldquoliving drolleryrdquo refers to live entertainment show In this context when Alonso the King of Naples Sebastian his brother Antonio the treacherous brother of Prospero Gonzalo the kind and loyal councillor to the King Adrian and Francisco came to the island they were hungry and weary in their spirits They heard a solemn and strange music They were shocked to see several strange shapes bringing in a banquet and these shapes danced about it with gentle action of salutation inviting the King and his companions to eat After this Sebastian described this show as lsquoliving drolleryrsquo(c) What is lsquophoenixrsquo What are lsquoUnicornsrdquo

The term lsquophoenixrsquo refers to a mythical Arabian bird which lived alone and perched on a solitary tree After one hundred years it expired in flames and rose again from its own ashes

lsquoUnicornsrsquo refers to the mythological four-footed beasts having horns in the centre of their foreheads When the horns are ground into powder the powder was believed to be

an aphrodisiac(d) How does Sebastian explain the puppet show OR Why does the speaker now believe in unicorns and phoenix

Sebastian finds several strange shapes bringing in the banquet They invite the king and his party for dinner and soon depart He tells that if such a strange sight can be a reality there is nothing incredible in the world and from the present moment he will believe anything He says that it is a strange dumb show enacted not by puppets but by living beings It is stranger than a travellerrsquos tale Seeing such a thing

before his own eyes he will no longer disbelieve the story about unicorns and phoenix(e) How do the other characters present respond to this living drollery

At the sight of the lsquoliving drolleryrsquo like Sebastian Gonzalo and Antonio too acted strangely Antonio told that he too now believes in unicorns and phoenix and anything else that seems to be incredible He too now believes in travellersrsquo tales Gonzalo told that if he would report those happenings in Naples nobody will believe him He considers that those gentle shapes were gentler in manner in comparison to the living beings Alonso was at first sight suspicious and told them that those strange shapes conveyed their meaning in expressive gestures when they seemed to lack speech by their movements and sounds Francisco was amazed at their mysterious disappearance

2 ALONSO Not I

(Line 43-52)GONZALO Faith sir you need not fear When we

were boysWho would believe that there were mountaineers

Dewlapped like bulls whose throats had hanging at rsquoem

Wallets of flesh Or that there were such men

Whose heads stood in their breasts Which now we find

Each putter-out of five for one will bring us

Good warrant ofALONSO I will stand to and feed

Although my lastmdashno matter since I feel

The best is past Brother my lord the Duke

Stand to and do as we

(a) How does Alonso respond at the spectacle of the shapes which were sent to them at the instruction of Prospero

After seeing the strange sight of appearing and disappearing of the shapes sent by Prospero to arrange a banquet for them Alonso says that his surprise at having seen those creatures is infinite and he is fully justified in feeling so much surprise He thinks that their shapes their gestures and the sounds they made were indeed amazing Although they do not possess the gift of speech yet they were able to convey their

thoughts by means of their gestures only

(b) What does Prospero say about the views expressed by Alonso regarding the shapes What does Francisco think about the shapesAfter hearing Alonsorsquos views about the shapes Prospero says that this manrsquos praise of the spirits is rather hasty He means to say that Alonso has shown great haste in reaching the conclusion about the shapes Francisco is amazed to see that those shapes disappeared in a mysterious way(c) What does Sebastian ask Alonso to doSebastian tells Alonso that the shapes having disappeared should not matter to them because they have left the eatables behind He asks Alonso to enjoy eating as they are extremely hungry but the king does not accept his offer of enjoying the dishes(d) How does Gonzalo try to dispel Alonsorsquos fear of those strange shapes What kind of references does he give to AlonsoGonzalo says that those who have travelled abroad have reported seeing even stranger sights than these shapes that Alonso and his companions have beheld Hence there is no reason to feel afraid of these shapes Gonzalo further adds that in his younger days he had heard strange stories from travelers and Alonso might have heard similar stories For instance it was said that there existed a certain race of

human beings who had huge lumps of flesh hanging at their throats and who therefore resembled bulls Then Gonzalo tells about a race of human beings whose heads were located at their breasts Gonzalo says that such stories were not believed by most people in those days but now-a-days these stories have become common(e) Explain the following lsquoEach putter-out of five for onersquoEnglish travellers often insured their trips with London brokers Those that went on foreign travels those days used to deposit a certain amount with some firm or company in London before their departure If the travelers failed to return the money was forfeited by the company with which it had been deposited But this money was repaid five-fold if the travelers returned safe and sound In this way a traveler stood a great chance of recovering the entire cost of his

travels(f) Give the explanatory meanings of the following expressions in the context of the above extract (i) Dewlapped (ii) Wallets of flesh

(iii) Putter-out(i) Dewlapped having big lumps of flesh at the necks(ii) Wallets of flesh large masses of flesh looking like bags(iii) Putter-out to invest money before commencing the travel

  • General methods of preparation of hydrogen
  • Chapter Dimensional Analysis (Summary)
    • Properties of Charge
Page 13:   · Web viewSubject. Topic. Summary. Execution. Hindi. व्याकरण. शरीरके अंगो के नाम लिखिए. 1) आँख 2) नाक 3

লেক - লিবভলিতভষ বসনদযোপো3যোয়লেকndash রবীনদর পরবতf বোংো কোোলিতয 3োরোর উসgসPোগয োম পরকলিতসপরমী লিবভলিতভষ বসনদযোপো3যোয় তোর লেীসত লেPম বোসর বোসর লিফসর এসস4 গরোম বোংোর পরকলিতর কো লেতমলি এসস4 গরোমী মো লিচতরগলপndash অপর কলপো গলপোংটি লিবভলিতভষ বসনদযোপো3যোসয়র লিবযোত উপযো পসর পাোচোী লেসক গীত অপ অ13োৎ পসর পাোচোী তো অপর কলপোর লেকনদরীয় চলিরতর এই অংস আমরো পোই বোক অপসক বোক অপ কলপো লিবোী লে দসরর অ গো4 লেদস মোসয়র মস লেোো রপকোর রোসয পোলি2 লেদয় দপরসবো মোসয়র মসর কসর কোীদোী মোভোরত এর করসকষতর Pসjর ব13ো শস তোর মোবীর কস13র পরলিত ব2 মমতো য় আবোরপালিসত বলি13ত Pসjর অমোপত অং লে লিসই মোপত কসর বোলি2র লিপ4স বাো বোগোস লিকংবো উঠোসর লিশমসর কলপো লিবো এোস পরকো লেপসয়স4

১ অপর কলপো গসলপর লেক লেক তোর মপসক13 লেসো২ অপর কলপো গলপটি লেকোো লেসক গীত গলপটির ম ভোব লেসো

GEOGRAPHY CHAPTER 7EUROPE

CHAPTER COMPLETE EXERCISEFill in the blanks1 Europe is a continent that comprises the western part of Eurasia2 Eurasia and Africa are connected into one large land mass known as Afroeurasia3 The Strait of Gibraltar separates Europe and Africa4 Europe is surrounded by the Arctic Ocean to the north5 The British Isles includes the island countries of Great Britain and Ireland

Name the following 1 Connects Africa to Eurasia - Isthmus of Suez2 Largest country in the world in terms of area ndash Russia3 A term used collectively for the five countries in northern Europe ndash Nordic Countries4 The capital of Montenegro - Podgorica5 the largest fjord in Norway ndash Sognefjord

Match the following Column 1 Column 2a Albania iii Tiranab Belgium i Brusselsc Denmark v Copenhagend Finland ii Helsinkie Hungary iv Budapest

CHEMISTRY Chapter 2 ndashElement and Compound

Atom - An atom is the basic unit of an element or the smallest particle of an element non capable of independent existence Atom is built up of three sub atomic particles electron proton and neutron

Nucleus-It is the centre of an atom In the centre of the atom contains proton (positively charged particles ) and neutrons ( particles carrying no charge )

Orbits- It surround the nucleus in which revolve electrons (negatively charged particles)

Answer the following

1) What are MetalloidsAns - Certain elements using properties of both metal and non-metals are called metalloids Example Silicon arsenic and antimony

2) What are Noble gasesAns - Certain elements are present in the air and are chemically inert or unreactive Such elements are called rare gases or noble gases Example helium neon argon and Krypton

English 2 Sentences based on meanings

Kinds of sentences

Assertive or declarative to convey information or simply make a statement

Interrogative to ask different types of questions

Imperative to command or instruct someone or make a request

Exclamatory to express strong feelings and emotions

Exercise B1 Stop it ( Exclamatory)2 May you always be happy

together ( Exclamatory)3 He does not like sports

( Assertive)4 Please pass me the salt

( Imperative)5 How dare she talk to me like

that ( Exclamatory)6 May success bless your effort

( Exclamatory)7 Canrsquot you wait for sometime

(Interrogative)8 Did anybody tell you about it

( Interrogative)9 I saw her waiting for the bus

( Assertive)10 Could you please take a

message for me ( Interrogative)

Homework Ex ABiology Chp -2

Classification of Plants

Today we discuss about usefulness of bacteria We also discuss what the harmful effects of bacteria are

89 How bacteria are useful for usbull Bacteria is helpful in many ways forhuman being i) Production of medicine - antibiotics vaccine etcii) Formation of curd by lactobacillusiii)Nitrogen fixation in Leguminousplant by Rhizobiumiv) Increase soil fertility by absorbingatmospheric nitrogen and convert it into nitrates and nitritesv) Cleaning the environment by converting the complex substances into simple substancesvi) Tanning of leathervii) Retting of Fibersviii) Formation of compost by acting onanimal dung and agricultual cases1x) Biogas production by decomposingplant and animal wastex)Help In Nutrition by producing vitamiacutemBand kx1) Some bacteria are used to give specialflavour to tea coffee and coccaQ10- Name some diseases and there causativebacteriabull Diseasescausative bacteria1 CholeraVibrio cholerae2 Tuberculosis - Mycobacterium tuberculosis3 Diptheria -Corynebacteriumdiphtheriae4 Pneumonia - Streptococcus pneumoniae

Math Number system

Chapter Fraction

Study item Using lsquoofrsquoThe word lsquoofrsquo between any two fractions is to be used as multiplicationExample 57 of 56 = 57 times 56 = 5times8 = 40Study item Using BODMASThe word lsquoBODMASrsquo is the abbreviation formed by taking the initial letters of six operations(i)Bracket (ii) of (iii) Division (iv) Multiplication (v) Addition (vi) SubtractionAccording to BODMAS rule First of all the terms inside Bracket must be simplified then lsquoofrsquo lsquoDivisionrsquo lsquoMultiplicationrsquo lsquoAdditionrsquo lsquosubtractionrsquo

Study item Removal of Brackets

There are four Brackets of algebra in Mathematics In a complex expression four types of brackets are used Order of removing the brackets is first ----- then ( ) then finally [ ]

Class VIIISubject Topic Summary Execution

Chemistry Hydrogen General methods of preparation of hydrogen

By the action of dilute acids on metals

Calcium Reacts readily to form chloride salt and hydrogen

Ca + 2HCl rarr CaCl₂ + H₂uarr

Magnesium

Aluminium

Zinc

React readily to form salt and hydrogen

Mg + 2HCl rarr MgCl₂ + H₂uarr2Al + 6HCl rarr 2AlCl₃ + 3H₂uarrZn + 2HCl rarr ZnCl₂ + H₂uarr

Question 4 ) Give reasons for the following

(a) Hydrogen be used as a fuel

Solution

Hydrogen is used as a fuel because it has a high heat of combustion Some significant fuels are coal gas water gas and liquid hydrogen

(b) Though hydrogen is lighter than air it cannot be collected by downward displacement of air

Solution

Hydrogen is lighter than air so it is possible to collect the gas by downward displacement of air But it is not safe to do so since a mixture of hydrogen and air can lead to an explosion

(c) A pop sound produced when hydrogen is burnt

Solution

Impure hydrogen gas burns in air with a pop sound This is because of the presence of impurities in it

(d) Helium replaced hydrogen in weather observation balloons

Solution

It forms a mixture with air that can explode when there is a small leakage of hydrogen in a balloon So helium has replaced hydrogen

(e) Nitric acid not used for the preparation of hydrogen gas

Solution

(e) By the action of nitric acid on metals hydrogen cannot be produced because it also releases nitrous oxide and nitric oxide and oxides the hydrogen to form water

Biology Chp-2 Reproduction in plants

Today we discuss different methods of artificial propagation like cutting-rose sugercane Layering ndashguava lemon china rose etc Grafting- mango apple etcMicropropagation ndashorchid asparagus etcWe also discuss about advantages and disadvantages of vegetative propagation

Q7 Define the following terms i) Explant In tissue culture techniquea tiny piece of bud shoot or any other partof plant from where new tissue develop ii) Callus The cells of the tissue divide andgrow into a mass of undifferentiated cells from explant iii) Plantlet After few days callus differentiate into a small plant with roots and shootQ8 what are the advantages and limitations of tissue culture or micropropagation

Advantages i ) It produacuteces superior quality plantsii)It can be applied to interspecifie hybridsiii) It is useful to grow seedless plants bull Limitations i) It cannot be used for all plantsii)It is not easy to handleQ9 Write advantages of vegetative propagationi) It is a quick and easy method ofproducing new plantsii) This method need less time to matureiii) The new plants are exact copies of the parentiv) it is extremly useful for growing seedlessplants like banana grapes etc Q10 Write some disadvantages of vegetativepropagationi) Dišeases present in the parent plant gettransferred to all in new plantsii) Overcrowding of new plants causes competition for sunlight water and nutrients which affects growth of plantsplant

Physics Chapter 2 Physical Quatites and Measurements

Here We Will Do Some QuestionsRelated To Chapter 2

Select the correct alternative A block of wood of density 08gcm-3 has a volume of 60cm3 The mass of the block is

1 608 g

2 75 g

3 48 g

4 0013 g

Solution 348 g

The density of aluminium is 27g and that of brass The correct statement is

1 Equal masses of aluminium and brass have equal volumes

2 The mass of a certain volume of brass is more than the mass of an equal volume of aluminium

3 The volume of a certain mass of brass is more than the volume of an equal mass of aluminium

4 Equal volumes of aluminium and brass have equal masses

Solution 2 The mass of a certain volume of brass is more than the mass of an equal volume of aluminium

MATHEMATICS Ch 6Sets

Exercise 6(C)1 Find all the subset of each the following sets(i) A = 57 (iii) C = x xisin W x le 2(iv) p p is a letter in the word lsquopoorrsquo

Solution (i) All the subsets of A are ϕ 5 7 57

(iii) All the subsets of C are ϕ 0 1 2 01 02 12 012

(iv) All the subsets are ϕ p o r po or por

4 Given the universal set = -7-3-105689 find (i) A = x xlt2 (ii) B = x -4ltxlt6 Solution

(i) A = -7-3-10(ii) B = -3-105

5 Given the universal set = x xisin N and xlt20 find

(i) A = x x = 3p pisin N (iii) C = x x is divisible by 4 Solution

(i) 369121518 (iii) 481216

6 Find the proper subset of x x2-9x-10 = 0 Solution

ϕ 10 -1

Working x2-9x-10 = 0 rArr x2-(10-1)x-10 = 0

rArr x2-10x+x-10 = 0 rArrx(x-10)+1(x-10) = 0

rArr (x+1) (x-10) = 0

11 Let M = letters of the word REAL and N = letters of the word LARE Write sets M and N in roster form and then state whether (i) M sube N is true (ii) N sube M is true (iii) M = N is true

Solution M = real and N = lareSo (i) Yes (ii) Yes (iii) Yes

English 2 Twelfth Night ndash Shakespeare

A noble man named Orsino in the kingdom of Illyria is deeply in love with a lady called lady Olivia She is in mourning for her dead brother so she will not even think about marriage At this time a sea storm causes a terrible shipwreck and a young lady called Viola is swept onto the shore She thinks that her twin brother Sebastian is drowned A sea captain tells her about Orsino and his love for Olivia Viola wishes to work in Oliviarsquos home but feels she will not be employed So she dresses as a man calls herself Cesario and gets work at the house of OrsinoViola (now Ceasario) is much liked by Orsino and becomes his page She falls in love with Orsino Orsino sends Ceasario to deliver messages to Olivia Olivia herself falls for the beautiful young Ceasario believing Viola to be a man

2 Answer the following questionsa Why does Orsino ask the musicians to play onOrsino asks the musicians to play on because music feeds his desire He calls upon the musicians to play music so that his hunger for love could be replenished with an excess of musicb What does Valentine tell about OliviaWe learn from Valentine that Olivia is in mourning for her brother she wears a veil and has vowed that no one will see her face for another seven yearsand she refuses to marry anyone until thenc From the exchange between Orsino and Valentine what do you think their relationship isValentine is one of orsinod attendants He was sent to Olivia as a messenger of love but was not allowed to speak to here Who is Olivia mourning for and whyOlivia is mourning for her dead brother

Homework Q fHistory and Civics

Growth of Nationalism

Important dates to remember1769-Napoleon born on 15thAugust1789-Fall of Bastille on 14th July and the beginning of the French revolution declaration of the rights of Man on 26thAugust1793-King Louis XVI executed on January 211764-The Sugar Act passed1765-The Stamp act passed1774-The first congress of Philadelphia1776-The declaration of American Independence of on 4th July1777-Defeat of the British at Saratoga1781-Surrender of lord Cornwallis at Yorktown1783-The treaty of Versailles1804-Napoleon becomes the emperor1813-Battle of Leipzig or Battle of nations in which Napoleon was defeated by the Allies1815-Battle of Waterloo June 18 in which Napoleon was defeated and captured1821-Death of Napoleon in StHelena1860-Abraham Lincoln elected President of the USA1861-The civil war began 1864-Abraham Lincoln elected President of the USA for the second time1865-Slavery abolished in the US

Name the following- The queen of Louis XVI

Marie Antoinette The three philosophers of France

VoltaireMontesquieuJean Jacques Rousseau

The British general whose surrender brought the war in America to an endLord Cornwallis

The first president of the USAGeorge Washington (1732-1799)

The first southern state to secede from the unionSouth Carolina

The author of the book lsquoUncle Toms CabinHarriet Beecher Stowe

Homework-Learn

Class IXSubject Topic Summary Execution

Economics

Types of economies Today I am going to share you the concept of economic growth and economic development Few questions will be given from the previous study material dated 942020

Meaning of economic growthAnswer) The term economic growth generally means anincrease in national income or per capita output or income over time It indicates towards quantitative growth of a country

Meaning of economic developmentAnswer) Economic development is defined

as a process whereby the real per capita income of a country increases over time along with fall in poverty ratio unemployment and income inequality etc

Distinguish between economic growth and economic development

Basis Economic growth

Economic development

Scope It has narrow scope as it refer only to rise in per capita income

It has wide concept since it includes qualitative changes as well

Concerned matter

It is concerned with the rise in income

It is concerned with not only rise in income but also reduction of poverty income inequality and unemployment

Focus Economic growth does not focus on economic development

Economic development focus on economic growth plus qualitative changes

Distinguish between capitalist economy and socialist economy

Ownership

Motive

Tool

Means of production are owned and managed by private people

Self interest and profit earning is the main motive

Price mechanism is a main tool to solve the economic problems

Means of production are owned and managed by the government

Social welfare is the main motive

Economic planning by the government is the main tool to solve the economic problem

Competition

Distribution of income

There exist large competition among buyers and sellers

There is existence of large inequalities of income

There is no such competition

There exist less inequalities of income

Math Topic ndash AlgebraChapter -Factorisation

Study item Factorising by taking out common factorSome solved sums from exercise 41

1) (i) 8xy3 + 12x2y2

= HCF of 8xy3 and 12x2y2 is 4xy2

= 4xy2(2y + 3x )

4) (ii) 28p2q2r ndash 42pq2r2

= HCF of 28p2q2r and 42pq2r2 is 14pq2r = 14pq2r (2p - 3r )5) (ii) 14mn + 22m - 62p=HCF of 14mn 22m and 62p is 2= 2(7mn + 11m - 31p)7) (ii) 3a(x2 + y2) + 6b (x2 + y2) = HCF of 3a(x2 + y2) and 6b(x2 + y2 ) is (x2 + y2)= ( x2+ y2 )(3a + 6b )9) (ii) x(x2 + y2 ndash z2 ) + y(-x2ndashy2 + z2 ) ndash z(x2+ y2 ndash z2 )= x(x2 + y2 -z2) ndash y-(x2 + y2 -z2) -z(x2 + y2 ndash z2)=x(x2 + y2-z2) -y( x2 + y2-z2) ndash z (x2 + y2 -z2)= (x2+ y2 ndash z2)(x ndash y ndash z )

Commercial Studies

Introduction to Accounting and Book-keeping

Today I am going to share you the meaning of Accounting and Book-keeping and its related terms bullAccounting bullBook Keeping bullAccountsbullTypes Of Accounts bullAccounting Cycle

bull Meaning of accounting

Ans )Accounting is the art and science of recording classifying and summarising monetary transactions

bull Meaning of Book-keeping

Ans) Bookkeeping is the art of recording business transactions with the view of having a permanent record of them and showing their effect on wealth

bull Meaning of account

Ans) The term account means a record of business transactions concern a particular person of firm asset or income or expense It is a summarised record of all transactions which take place in an accounting year

bull Types of accountsPersonal accounts ndash Personal accounts relating

to person and Organisation are known

as personal accounts Example Ramrsquos Account ABC amp Co Account etc

Real account - The accounts related to tangible and intangible assets are called real accountsExample Cash Account Furniture Account etc

Nominal account- Accounts related to expenses losses incomes and gains are known as nominal accountsExample Wages Account Salary Account Discount Account etc

bull Accounting cycle Accounting cycle refers to a complete sequence of accounting activities It begins with recording of transactions and ends with the preparation of a balance sheet

English 1 Transformation of sentences

Sentences A sentence is a group of words which makes complete sense

a Assertive sentencesb Imperative sentencesc Interrogative

sentencesd Exclamatory sentences

Sentences can be changed from one grammatical form to another without changing the meaning of the sentence This is known as transformation of sentences

Exercise 6Rewrite the following sentences according to the instructions given below without changing their meanings

1 As soon as he saw the beer he jumped into the river ( Begin No sooner)

2 None but brave deserve the fair (Begin the bravehellip)

3 This box is too heavy for me to lift ( Use so hellip That instead of too)

4 No one other than a king can live like James Luxurious ( Begin only James)

5 Oh for the wings of a dove (Begin I wishhellip)

BENGALI(2ND LANGUAGE)

ldquo বঙগভমির পরমি ldquo াইকেল ধসদন দতত

পব13পোসঠ আসোলিচত ৩ পরবোস দৈদসবর বস ীবতোরো Pলিদ স এ লেদ -আকো সত-োলি লেদ তোস - ক) বকতো লেক কোর লেো লেকো কলিবতোর অং ) কোর পরলিত বকতোর এই উলিকত গ) এ লেদ আকো সত বসত কী বলিঝসয়স4 ীবতোরো বসত কী লেবোঝ ঘ ) আসোচয অংসর তোৎপP13 কী

উ -ক ) বকতো স কলিব মোইসক ম3দ দতত

Types of AccountPersonal AccountReal AccountNominal AccountBalance Sheet (opening)

কলিব মোইসক ম3দ দসততর রলিচত বঙগভলিমর পরলিত কলিবতোর অং ) কলিব বঙগী অ13োৎ লেদমোতোর পরলিত কলিবর এই উলিকত গ ) এ লেদ আকো বসত কলিবর মোব লেদী রপ আকো লেক লেবোঝোসো সয়স4 আকো লেসক লেPম তোরো স পসর লেতমলি ীব লেদ রপ আকো লেসক পরো রপ তোরো স পরসত পোসর এই মভোবোর কোই কলিব বসস4 ঘ ) পরবো Pোতরোয় Pলিদ কলিবর লেদ আকো লেসক ীব তোরো রপ পরো স পসর তোসত কলিব লিবনদমোতর দঃলিত কোর মতয লিবসর সবোভোলিবক পলিরলিত এবং মোষ মরী তোই পরবোস Pলিদ তা োর মতয য় তবও কলিব লিবচলিত সব ো কোর পলিবীসত লেকউ অমর য় লিক4ই অকষয় য় দীর লেPম লিচরপরবোমো লেতমলি মোসষর ীবও চমোতোই ীব - সতবধতোই মতয ীব দীসত মোষ লিতয পরবোমো তবও লেPব মোষ আপ কতকসম13র মো3যসম মোসষর মস লিসসদর সথো কসর লিসত পোসর তোরো লিচরভোসবর সয় মোসষর মস লিবরো কসর তোসদর মস3য লেকউ পGভসত লিবী সয় গোসও মোসষর মস তোরো লিতযপলিত লিতযবলিনদত

Hindi 2ndlang

काकीी(लिसयारामशरणगपत)

इस कहानी म क न यह बतान का परयास निकया ह निक बचच अपनी मा स निकतना परम करत ह शयाम अबोध बाक ह वह अपनी मा क मरन क बाद उसन अपनी मा क लिए बहत रोया बाद म उस पता चा निक उसकी मा राम क घर ची गई ह आकाश म उडती हई पतग दकर उस हष हआ निक पतग क दवारा वह अपनी मा को नीच उतारगा इसक लिए वह अपनी निपता की जब स दो बार सवा रपया निनकाकर पतग और दो मोटी सी मन वाी अपन भाई स काकी एक कागज पर लिवा कर पतग म लिशव का दिदयानिनकाकर पतग और दो मोटी सी मन वाी अपन भाई स काकी एक कागज पर लिवा कर पतग म लिचपका दिदयाभोा और शयाम कोठरी म रससी बाधनी रह थ तभी उसक निपता करोध म आकर उन स पछ निक कया उनकी जब स रपया निनकाा हभोा डर क मार बताया निक शयाम इस पतग क दवारा अपनी काकी को राम क यहा स उतारना चाहता हनिवशशवर(शयाम क निपता)न फटी पतग उठाकर दी तो उस पर काकी लिा थावह हत बजि होकर वही ड रह गएउनहोन सोचा निक मन अपन पतर को मारा जोनिक अनजान और निनदष थावह अपनी मा कोनिकतना पयार करता ह

helliphellipContinue to next

Computer Application

Java Programming Prog 1Write a java program to input two numbers from user and display the sum or product of them as per user choice Use switch case statementSolve public class sum_product public static void main(String args[]) Scanner sc=new Scanner(Systemin) int abc Systemoutprintln(ldquoEnter two numbersrdquo) a=scnextInt() b=scnextInt() Systemoutprintln(ldquoPress 1 for sum or 2 for productrdquo)

c=scnextInt() switch(c) case 1 Systemoutprintln(ldquoThe sum will be =rdquo+(a+b)) break case 2 Systemoutprintln(ldquoThe product will be =rdquo+(ab)) break default Systemoutprintln(ldquoWrong Inputrdquo) Home Work - Practice in your computer using bluej

Subject Eng Literature (The Merchant of Venice ndash William Shakespeare)Topic Act I Scene 2 Lines 92 to 126 (End of scene) Date 13th April 2020 (5th Period)

[Students should read the original play and also the paraphrase given in the school prescribed textbook]Summary Questions amp Answers

o After Portia has expressed her opinion about the suitors Nerissa informs that she need not bother about any one of them as they have decided to quit Belmont at the earliest opportunity because they do not believe in trying their luck by the caskets which is the only way of winning Portia

o Nerissa then enquires of Portiarsquos opinion about Bassanio who once visited her in the company of the Marquis of Montferrat and says that she had never come across such an ideal love deserving the fairest lady for his bride

o Portia seems to remember Bassanio quite correctly and says that she agrees with Nerissa At this moment a servant informs Portia that the Prince of Morocco has arrived to try his luck by the caskets

o Portia tells Nerissa that if she could welcome this new suitor as gladly as she says farewell to the previous ones she would be glad of his arrival However if he happens to have the virtues of a saint but the black complexion of a devil she would prefer to have him for religious consolation rather than as a husband

(1) NERISSA You need not fear lady (Line 97-103)

the having any of these lords they have acquainted me with their determinations

which is indeed to return to their home and to

trouble you with no more suit unless you may be wonby some other sort than your fathers imposition depending on the caskets

PORTIA If I live to be as old as Sibylla I will die as chaste asDiana unless I be obtained by the manner of my fatherswill I am glad this parcel of wooers are so reasonablefor there is not one among them but I dote on his veryabsence and I pray God grant them a fair departure

(a) Elucidate the idea expressed in the first speech of the above dialogue

In the first speech Nerissa assures Portia that she need not have any fear of being compelled to marry anyone of the suitors who had lately come to Belmont She informs her that they have all decided to return to their respective countries(b) Illuminate the meaning of the phrase ldquoyour fatherrsquos imposition depending on the casketsrdquo

Nerissa means that the suitors of Portia do not find the conditions imposed by the will of her father to their liking They are too hard for them These conditions are that in the event of a suitor failing to choose the right casket (i) he should never disclose to anybody which casket he chose (ii) he can never marry and (iii) he should take his departure immediately(c) Explain the meaning of the term lsquoSibyllarsquo

lsquoSibyllarsquo is the name given by Romans and Greeks to a prophetess inspired by some deity usually the sun-god Apollo She had a very long life The god Apollo granted her as many years of life as she could hold grains of sand in her hand(d) Elucidate the meaning of the term lsquoDianarsquo

lsquoDianarsquo is the goddess of hunting She is also regarded as a symbol of virginity because she never fell in love and never

married(e) Explain the meaning of the first two lines of Portiarsquos speech

Portia says that even if she is to live for centuries like Sibylla she would not marry except in accordance to her fatherrsquos will She asserts that she would not mind remaining unmarried and untouched by a man like Diana the virgin the goddess of hunting unless a man is able to win her by passing the test laid down by her father

Class XSubject Topic Summary Execution

Hindi 2nd

Langबड घर की बटी( मशी परमचद)

lsquoबड घर की बीटीrsquo कहानी का उददशय मधयम वग की घर समसया को सझा कर सगदिठत परिरवार म मिम जकर परम स रहन का सदश दना ह घर म शानित tानिपत करन की जिजममदारी नारी की होती ह यदिद नारी समझदार ह उसम धय और परिरवार क परनित परम ह तो कोई भी घटना परिरवार को निवघदिटत नही कर सकती या कहानी परिरवार को सगदिठत करत हए परम सौहाद स एक रदसर की भावनाओ को समझ करउनका सहयोग करत हए जीवन यापन करन की पररणा दती ह मशीपरमचदर जी न इस कहानी म सयकत परिरवार का परनितनिनमिधतव निकया ह यह कहानी बनी माधव सिसह जो गौरी पर क जमीदार क उनक दो पतरो की हशरी कठ ा निबहारीशरीकात का निववाह एकजमीदार घरान की पतरी आनदी स हआ थाआनदी न द को ससरा क वातावरण म ढालिया थाएक दिदन आनदी का अपन दवर ा निबहारी स झगडा हो जाता ह दोनो भाई एक रदसर स अग होन की कोलिशश करत हसभी बह आनदी न अपन मधर वयवहार स ा निबहारी को घर छोडकर जान स रोक लिया| इस पर बनी माधव सिसह न कहा निक बड घर की बटी ऐसी ही होती ह जो निबगडा काम बना ती ह अतः शीषक साथक ह बड घर की बटी आनदी ह

helliphelliphelliphellipContinue to nextBiology Topic ndash Chp-1

CellWelcome to new session 2020-21Today we will start with Chpter 1 cell CELL

Protoplasm+Cellmembrane Or Cell wall

Cytoplasm+Neucleus

Cytoplasmic+ CytoplasmicOrganelles Inclutions(mitochondria (food Golgi bodies pigments)Ribosome)

What is cellbull Cell is the structural and functional unit of living organismbull According to number of cells organisms areUnicellular - Amoeba bacteria Multicellular - Rose Mango Tiger HumanSmallest cell -bacteria Longest cell - Nerve cellLargest cell - Ostrich egg cellCells are of different size and shapes according to their functionsQ2Write chief functions of following cellorganelles

Q3What is tonoplastVacuoles covered by a covering called tonoplast

Bengali(2Nd

Language)

ফ ফটক ো ফটক (কলিবতো ) ভোষ মসোপো3 gtPোয়

একটি লেমসয়র ীবস লেপরম লিকভোসব ফসট ওসঠ তো লেদলিসয়স4 কলিব লেপরম Pই য় লেই ময়ই বনত কোস পলিরত য় ফ লেফোটো বো োসফোটো লেটো ব2 কো য় লেমসয়সদর ব gtয13 লেপরসমর 4লিব ফসট উসঠস4 এই কলিবতোয় লেপরম মোষসক মত gtযর মস লেফস লিদসয় পরকষস বাোচোসোর gtয োত বো2োয় কলিবতোয় লেমসয়টির পসব13র দঃসর কো বো সও লেমসয়টি লেই পসর পলিক সত চোয়ো োরী ীবসর কোস4 পরম লেPৌবস লেপরমসক পোবোর পরব ইচছো োকসও তো পসর লেলিতবোচকতোয় পলিরত য় কলিব ভোষ মসোপো3 যোয় লেP ক লেপরসমর

কলিবতোয় ব gtযবহত লিবসষ লিক4 সvর অ13 লেদওয়ো ১) রসবোো= লেP লিবলিভনন রকম ডোকসত পোসর২) ো= পোর ৩) ঠলি = লেচোসর বZ৪)আই বস2ো=অলিববোলিত৫)শইসয় = োলিয়ত কসর৬)োতপাোচ= লিবলিভনন পরকোর৭)দ2োম = v কসর বZ কসর লেদওয়ো৮)লেরলিং =লেোোর দৈতরী লেব2ো৯) বনত= একঋত১০) পাোর = বসকরো2

Organelles Functions

1 Endoplasmic reticulum

2 Mitochondria

3Golgibodies

4 Ribosome5Lysosome

6Plastids

7 Centrosome

i) Supportive framework for the cellii) Synthesis and transpost of proteinsRelease of energy in the form of ATPi) Synthesis and secretion of enzymes hormoneii) Formation of vacuoles lysosomei) Protein Synthesisi) Intracellular digestionii) Destroy foreign substancei )Leucoplast - stores starchii)chloroplast - trap solar energyiii) Chromoplast - imparts colour toflowers amp fruitsi) Initiates and requlates cell division

কলিবতো তোর অ13সক ভোষোয় পরকোো কসর ঘলিরসয় ব যকত কসরস4 লেপরসমর ফতো আর লিবফতো লেক গোঢ় কসর লেদোসো কলিব ভোষ মসোপো3 যোসয়র অলিভবসর অ যলিদক

Economics

Factors of Production

Welcome to the new sessionToday we are going to start the first chapter of Class XThe name of the chapter is Factors of productionBy the name I hope you all can recall a glimpse of what you have learnt in the second chapter of Class IX

NowProduction is the process of creating the various goods and services which are consumed by the people of the country to satisfy their wants

Thus it is the process in which some materials are transformed from one form to another to create utility and value in goods

For example utility can be created by changing the form of a commodity ie

Making of table out of wood by a carpenter for his customer here the wood is getting transformed into table creating utility for his customer and he can also command a price for it

On the other hand Housewives perform very

useful activities at home which create utility but their domestic activities are not included in production because they have no money value

So we can also say that Production denotes two things firstly creation of utility and secondly creation of value

Production is not complete unless it reaches the consumer

An increase in production will increase the economic welfare of the consumers and hence the aim is to raise the production level of the country

Again production of a good or service is only possible if certain resources or

Questions

1 What do you mean by production

Answer Production means the creation of goods and services for the purpose of selling in the market

In fact production involves the transformation of inputs into outputs

Hence production denotes two thingsCreation of utility and creation of valueUtility and value can be created by changing the form by changing the place by changing the time and by rendering services

Example Transformation of raw

materials into finish goods such as potter creates utility by converting mud into utensils assembling of small parts to make bigger machinery

Production also includes services such as distribution and marketing

2 What are the factors of production

Answer Factors of Production refers to the resources and inputs needed for producing goods and servicesThese inputs can be classified as

Land Labour

Capital Enterprise

Land Land is defined to include not only the surface of the earth but also all other free gifts of nature(for example mineral resources forest resources and indeed anything that helps us to carry out the production of goods and services but is provided by

inputs are used together in right proportion

A resource or an input which helps in the process of production to obtain an output is called FACTOR OF PRODUCTION

These factors of production can broadly be categorized into four parts 1LAND 2LABOUR3CAPITAL4ENTERPRISE (ORGANISATION)or Entrepreneur

The above factors are all interdependent on each other and they play a major role in production process

FACTORS OF PRODUCTION

LANDCAPITAL

LABOUR ENTREPRENEUR

nature free of cost)LabourLabour refers to the human efforts that need to be combined with other factors of production for creating an output

CapitalAll man ndash made means of production is called capita example machineries which help in further production Money when used for starting any business for purchasing raw materials machinery tools etc it is regarded as capitalCapital also includes physical capital like factories machineriestoolsbuildingsequipments etcEnterpriseThe task of bearing risks is called enterprise and the person who bears these risks of business is called the entrepreneurThus an entrepreneur is one who organises production takes important decisions regarding production hires and purchases factors of production and bears the risk and uncertainty involved in productionOrganisation refers to the services of an entrepreneur who controls organises and undertakes all risks One who plans organises and manages a business enterprise is an organiser

Physics Chapter 1 Force

Force is an external agent capable of changing the state of rest or motion of a particular body It has a magnitude and a direction The direction towards which the force is applied is known as the direction of the force and the application of force is the point where force is applied The Force can be measured using a spring balance The SI unit of force is Newton (N)

Question 1

State the condition when on applying a force the body has

(a) the translational motion

(b) The rotational motion

Solutions

(a) Translational motion is produced when the body is free to move

(b) Rotational motion is produced when the body is pivoted at a point

Question 2

Define moment of force and state its SI unit

Solutions

The moment of force is equal to the product of the magnitude of the force and the perpendicular distance of the line of action of force from the axis

of rotation

The SI unit of moment of force is Newton times meter

= Newton meter (Nm)

Commercial Studies

Stake holders In this topic you will be come to know about the meaning and concept of stakeholders

How stakeholders are different from shareholders

Questions1 What do you mean by the term stake holdersAnswer) The term stake holders have developed from the words which mean an interest or expected benefit Stakeholders mean all those individuals groups and Institutions which have a state (interest) in the functioning and performance of a commercial organisation or a business enterprise2 What do you mean by share holdersAnswer) The person and Groups who own the shares of the joint stock company by providing capital to the company are called shareholders Shareholders are the internal stakeholders shareholders are one out of several stake holders3 How are shareholders different from stakeholdersAnswer)i) The term shareholders is related to only joint stock company whereas stakeholders are related with all business organisationsii) Stakeholders maybe any individual having financial stake in business organisation whereas a shareholders are those individuals who are holding shares in the company4) How are shareholders different from creditorsAnswer) i) Shareholders are internal stakeholders while creditors are external stakeholdersii) Shareholders invest in the capital of the company whereas creditors give loan to the companyiii) Shareholders are the members of the company with voting rights but creditors are not the members of the company

English 1 Transformation of sentences

Sentences A sentence is a group of words which makes complete sense

e Assertive sentencesf Imperative sentencesg Interrogative sentencesh Exclamatory sentences

Sentences can be changed from one grammatical form to another without changing the meaning of the sentence This is known as transformation of sentences

Exercise 1 Change the following affirmative sentences into Negative sentences

a He is a good manHe is not a bad man

b Ram loves SitaRam is not without love for Sita

c Only he stood first in the classNone but he stood first in the class

d Ankit was wiser than he

He was not so wise as Ankite He did it

He did not fail to do itf As soon as I reached college the

bell rangNo sooner did I reach college than the bell rang

g He finished everythingHe left nothing unfinished

h It always pours when it rainsIt never rains but it pours

Math Topic Commercial MathematicsChapter ndash Goods and services Tax

What is GSTAns It is a abbreviated term of Goods and Service Text which is an indirect tax levied on the sale of goods and rendering servicesSome terms related to GSTDelar Any person who buys goods or services For resale is known as a delar A delar Can be a firm or a companyIntra-state sales Sales of goods and services within the same state or same union territory are called intra- state salesInter-state sales Sales of goods and services outside the state or union territory are called Inter-state sales4) Input GST GST is paid by dealers on purchase of goods and services are called input GST5) Output GST GST is collected from customers on sale of goods and services are called output GST6) Types of GST There are three taxes applicable under GST(i) Central Goods and Services Tax (CGST)(ii) State Goods and Services Tax (SGST) or Union Territory Goods and Services Tax (UTGST) Both these taxes are levied on intra-state sales Here GST is divided equally among central and state governments(iii) Integrated Goods and Services Tax (IGST) IGST is levied on inter- state sales It is also levied on import of goods and services into India and export of goods and services from India

Subject Eng Literature (The Merchant of Venice ndash William Shakespeare)Topic Act III Scene 4 Lines 1 to 44 (Portia hellip To wish it back on you fare you well Jessica)[Students should read the original play and also the paraphrase given in the school prescribed textbook]

Summary Questions amp AnswersIn this scene we suddenly find a new element in the character of Portia We have already seen her possessed of every graceful womanly quality but now she shows that she is capable of rapid decision and determined action She shows this by her sudden resolve to hasten to Venice with a daring scheme for the rescue of Antonio This is an important scene in the dramatic action for it leads up to and renders possible the striking events of the famous trial scene which is one of the greatest striking elements of the play Moreover the fact that all the characters of importance are now assembled together in Venice makes the union of the main plot and the secondary story complete

(1) LORENZO Madam although I speak it in your presence(Line 1-9)

You have a noble and a true conceit

Of god-like amity which appears most strongly

In bearing thus the absence of your lordBut if you knew to whom you show this honourHow true a gentleman you send reliefHow dear a lover of my lord your husbandI know you would be prouder of the workThan customary bounty can enforce you

(a) Where is Lorenzo Why is he here To whom is he referring as lsquoMadamrsquo

Lorenzo is at Portiarsquos residence He had met Salerio on the way and Salerio had begged him to come along with him to

o In this scene Portia Nerissa Lorenzo Jessica and Balthazar appear

o Portia requests Lorenzo and Jessica to be in charge of her house during her absence from Belmont because she and Nerissa have decided to spend the days in meditation and also in visiting the holy places in the neighbourhood of Belmont She has already instructed her people to acknowledge both Lorenzo and Jessica as master and mistress of house during her absence Lorenzo and Jessica gladly agree to look after the house of Portia

handover the letter from Antonio to Bassanio The letter carried the bad news about Antoniorsquos arrest for non-payment of loan taken from Shylock Hence Salerio might have preferred company to break this bad news to Bassanio He is referring to Portia as Madam(b) What does Portia say on hearing the above extract

Portia says that she has never regretted doing good to others Friends who spend a lot of time together and really are there for each other have many traits in common As Antonio is Bassaniorsquos best friend saving him is like saving Bassanio who is like her own soul She asks Lorenzo to take care of management of the house till Bassanio is back(c) What does Portia send with Bassanio and why

On hearing about Antoniorsquos troubles on account of Bassanio her husband Portia immediately sends him with enough gold to repay the debt many times over to Venice to help Antonio out of his misfortune

(2) Lorenzo Madam with all my heart (Line 36-40)

I shall obey you in all fair commands

Portia My people do already know my mindAnd will acknowledge you and JessicaIn place of Lord Bassanio and myselfSo fare you well till we shall meet again

(a) Where are Lorenzo and Portia at this time What lsquofair commandsrsquo are given to Lorenzo

Lorenzo and Portia are at Belmont during this scenePortia reveals to Lorenzo that she has sworn to contemplate in prayer at a monastery around two miles away until her husband returns from Venice She tells him that Nerissa would accompany her and asks him to manage the house with Jessica till things are settled In response Lorenzo tells her that he would be obliged to do whatever she asks him to do(b) Where is Portia actually going and why

Portia tells Lorenzo that she would live a life of contemplation and pray at a monastery which is two miles away from her place In reality Portia plans to go to Venice in disguise with Nerissa and argue the case in defense of Antonio She is very sure that her plan would succeed

ClassXI (ScienceHumanitiesCommerce)Subject Topic Summary Execution

Computer Science

(APC)

Ch ndash 1 Numbers

(Numbers in different bases and

their Arithmatical operations)

Number System In computers Number System is defined as a writing system to represent the numbers in different ways ie we are using different symbols and notations to represent numbers There are four ways we can represent the number ndash Binary Decimal Octal and Hexadecimal

Decimal Number SystemThis number system consist 10 digits These are 0 1 2 3 4 5 6 7 8 amp 9

Binary Number SystemThis number system has only two digits these are 0 and 1 Here 0 stands for off while 1 stands for on

Octal Number SystemThis number system has 8 digits these are 0 1 2 3 4 5 6 amp 7

Hexadecimal Number SystemThis number system has 16 digits these are 0 1 2 3 4 5 6 7 8 9 A B C D E F Here the value of the alphabets are as follows A=10 B=11 C=12 D=13 E=14 F=15

Rules for conversion decimal number to Binary1 Divide the decimal number by 22 If the number will not divide equally by 2 then round down the answer to the nearest whole number (integer)3 Keep a note of the remainder it should be either 0 or 14 Keep repeating the above steps dividing each answer by 2 until you reach zero5 Write out all the remainders from bottom to top This is your binary solution

For example Lets convert 32 to binary 2 32 2 16 - 0 2 8 - 0 2 4 - 0 2 2 - 0 2 1 - 0 0 - 1

The binary equivalent of 3210 is 1000002

Try the follwing youself1 2410

2 4810

3 1210

History GROWTH OF NATIONALISM

The second half of the 19th century witnessed growth of political consciousness and a sense of Nationalism among the IndiansThere were various factors for growth of Indian Nationalism- As a result various political associations were formed in different provinces by the educated Indians Surendranath Banerjee organized a meeting of National conference at Calcutta Ultimately the National Congress was founded in Bombay in 1885This body became the vanguard of Indian struggle for freedom The congress leaders were known as moderates because they followed a policy of prayer and petition A large number of Indian leaders had experienced in political agitation The Political situation of England was also changed Moreover increasing revolutionary activities in Maharashtra Punjab and Bengal became serious concern to the British Government In this

QUESTION1 What do you mean by Nationalism ANSWER 1 Nationalism is defined as loyalty and devotion to own nation especially a sense of national consciousnessQUESTION 2 What are the causes of nationalism ANSWER 2 There were various factors for growth of nationalism

1 Spread of western education2 The progress of vernacular press and

patriotic literature3 The economic exploitation of our

country by the colonial rulers4 International affairs

QUESTION 3 Who organized National conference in Calcutta in 1883 ANSWER 3 Surendranath BanerjeeQUESTION 4 When did Indian National Congress formANSWER 4 Indian National Congress was formed in 1885 in BombayQUESTION 5 Who were ModeratesANSWER 5 The Early Nationalists were also known as Moderates Their emergence marked

background Lord Curzon became Viceroy in India He had no respect for the Indian National Congress

the beginning of the organized national movement in India They believed in British justice and were loyal to them They followed a policy of prayer and petition They demanded constitutional reforms of our country Impotant Moderate leaders were Pherozshah Mehta Dadabhai Naorozi and Surendranath Banerjee etcQUESTION 6 What do you know about Extremism in Indian National movementANSWER 6 In the beginning of 20th century a new class of national leaders emerged in India which was different from the moderate groups They started more aggressive movement against the British empire The goal of extremists was ldquoswarajrdquo Important extremist leaders were Bal Gangadhar Tilak Lala Lajpat Rai Bipin Chandra Pal etcQUESTION 7 Mention the places which were the main centres of Revolutionary movementANSWER 7 Maharashtra Bengal and Punjab

Physics

Chapter Dimensional Analysis

(Summary)

The dimensions of a physical quantity are the powers to which the fundamental units are raised in order to obtain the derived unit of that quantit

The physical quantites lengthmasstime are represented by [L] [M] [T] resp let they are raised to powers ( dimesions) abc resp then any physical quantity can be represented by [ La Mb Tc ] Examples

1 Area area = L x B = [L] x [L] = [M0 L2 T0 ]

2 Density density = massvolume = [M][L3] = [ M L-3]

3 Velocity velocity = distancetime = [L][T] = [LT-1]HW Try to find out dimension of acceleration Acceleration = velocity timeNB One can find the SI Units Using Dimension Analysis Such as for area we have [L2] so its SI unit is m2

Biology Topic ndash Chp-1 The living world

Today we will start the first chapter the living world Here we discuss about the characteristics of living organism and what are the difference between them and nonliving substances We also discuss about the contribution of different Scientists

There are over 500000 species of plants andover a million species of animal are present on earth Some 15000 new species were discovered every yearQ1 What is a living organismbull A living organism is primarily physico -chemical material that demonstrate a high degree of complexity is capable of selfRegulation possesses a metabolism and perpetuates itself through timeQ2 What are the differences between livingand non-livingsi) Compared with non-living living organisms

have more complex organised structure and their use of energy is more controlled amp efficientii) Living things reproduce their own kind by forming new cells which contains copies of their genesiii) Each organism has some degree of homeostasisie it is able to make adjustments so that internal environment remains constantQ3 Write contributions of following Scientists i) Aristotle - One of the first theories in Biology places all living things in a hiearchieii) AV Leeuwenhoek - was the first to observe living single celled organisms under microscopeii) Carolus Linnaeus - developed the binary system for naming of organisms and classificationiii) Geregor Johann Mendel ndash discoverbasic principles of inheritanceHomework i) C Darwin ii)Schleiden

Math Trigonometric functions

1 Overviewi) Trigonometry The word lsquotrigonometryrsquo is derived from the Greek words lsquotrigonrsquo and lsquometronrsquo which means measuring the sides of a triangle An angle is the amount of rotation of a revolving line with respect to a fixed line Usually we follow two types of conventions for measuring angles ie a) Sexagesimal system b) Circular system In Sexagesimal system the unit of measurement is Degree In Circular system the unit of measurement is Radian ii) Relation between degree and radianThe ratio of circumference of a circle to its diameter is always a constant This constant ratio is a number denoted by π which is taken approximately as 227The relationship between degree amp radian measurements is as follows2 right angles = 180deg= π radians1radian = 180degπ=57deg16(approx) 1deg=π180 radianiii) Length of an arc of a circleIf an arc of length s subtends an angle θ radians at the center of a circle of radius r then s=rθiv) Area of a sector of a circleA sector is like a pizza slice of the

Q) Express the following angles in radiana) 45deg b) 40deg3730Ans a) We have 180deg=π radiansi e 45deg= πtimes45180 radian = π4 radiansb) 40deg3730= 40deg37+3060 minute= 40deg 37 +12 minute= 40deg+ 752 minute=40 + 75(2times60) degree=3258 degreeNow 180deg=π radianie 3258 degree= (πtimes325) (180times8) radians = 65π288 radiansQ) A circle has a radius of r=12 meters What is the length of an arc traced out by a 60deg angle in the center of the circleAns In this problem we know both the central angle (60deg) and the radius of the circle (12) All we have to do is plug those values into our equation and we get

s = 2π(12)(60360)s = 24π6s = 4πSo the length of an arc traced out by a 60deg angle in a circle with a radius of 12 meters equals 4π meters asymp 1257 metersQ) Find the area of the sector with a central angle 30deg and a radius of 9cmAns GivenRadius r = 9 cmAngle θ = 30degArea of the sector = θ360degtimesπr2

= 30360degtimes227times92=2121cm2

circle It consists of a region bounded by two radii and an arc lying between the radiiThe area of a sector is a fraction of the area of the circle This area is proportional to the central angle In other words the bigger the central angle the larger is the area of the sectorArea of Sector = θ2 times r2 (when θ is in radians)

Area of Sector = θ times π360 times r2 (when θ is in degrees)

COMMERCE

CLASSIFICTION OF HUMAN ACTIVITIES-ECONOMIC AND NON-ECONOMIC

Welcome to the new sessiontoday we are going to start the first chapter of Class XI The name of the chapter that we are going to start is

lsquoClassification of Human Activities ndasheconomic and non-economicrsquo

Now let us start the chapter by considering human beings and the activities they perform throughout the day

Human activities means all those activities that human beings undertake to satisfy their wants

Human wants on the other hand are the desire of human beings for goods (vegetables fruits rice etc) and services (services of doctors teachers lawyers etc) that they require to live

Now these human activities continue throughout life as human wants are unending unlimited and recurring as human beings desire for better living throughout their lives

Now human activities can be classified into two categories

Human activities

Economic activities Non-economic activities

Economic activities are

Questions1 What are human activities

Answer Human activities mean all those activities that human beings undertake to satisfy their wants

Example A man working in an office

A boy playing in the garden

2What are the characteristics of human activitiesAnswer the characteristics of human activities are as follows

Human activities are undertaken by men women and children and these activities involve human efforts

Human activities are undertaken to satisfy human wants which are unlimited

Human activities continue throughout life

Human activities are performed for both earning money and personal satisfaction

3What is economic activitiesGive example

Answer Economic activities are undertaken by human beings with the object of earning money acquiring wealth and thereby satisfying human wantsExample

Selling of goods by a shop keeper to his customer

A clinic run by a doctor Service of a teacher in school or college

undertaken by human beings with the object of earning money and acquiring wealth

These activities result in the production of economic goods and services

Example Human activities(ie working in factories officesshops) which produce direct economic benefits

Non-economic activities are inspired by human sentiments and emotions such as love for the family desire to help the poor and love for the country

Thus these human activities (eg praying playing sleeping) produce no direct economic benefits and they are also not related to earning money and acquiring wealth

4 What are the characteristics of economic activities

Answer The characteristics of economic activities are as follows

Economic motiveEconomic activities are undertaken to earn money and acquire wealth

ProductiveEconomic activities involve productiondistribution and exchange of goods and services to create wealth

Economic growthEconomic activities determine the level of economic development of a country and standard of living of its citizens

Socially desirableEconomic activities are socially desirable for society

Economic resourcesEconomic activities make use of all the economic resources such landlabourcapital etc

5 What do you mean by non-economic activitiesExampleAnswerNon-economic activities are inspired by human sentiments and emotions such as love for the family desire to help the poor and love for the countryThese activities are not undertaken for monetary gain but for onersquos satisfaction and happinessExample

a mother looks after her children

a student donates blood8 Differentiate between Economic activities and Non-economic activities

Economic activities

Non-economic activities

1to earn living and acquiring wealth2Result can be measured in terms of money

3ExampleBusinessprofession and employment

1 to obtain some satisfaction

2Result cannot be measured in terms of money

3ExampleFamily-orientedreligious socialCultural and national

BUSINESS STUDIES

BUSINESS ENVIRONMENT

Welcome to the new sessionToday we are going to start the first chapter and the name of the chapter is Business Environment

In todayrsquos world every business enterprise is a part of the society It exists and operates in association with various groups in society such as customers suppliers competitors banks and financial institutions government agencies trade unions media and so on All these groups influence the functioning of business in one way or the other They constitute the environment of businessConcept of Business Environment

The term lsquobusiness environmentrsquo refers to the sum total of all individuals institutions and other forces that lie outside a business enterprise but that may influence its functioning and performance

The main features of business environment

Totality of External forces General and Specific forces Interrelatedness Complexity Dynamic Uncertainty Relativity

The Interrelation between business and its environment

The business enterprise is an open system It continuously interacts with its environment It takes inputs

Prepare the following questions from todayrsquos assignment

1 What do you mean by business environment

The term lsquobusiness environmentrsquo means the aggregate of all forces factors and institutions which are external to and beyond the control of an individual business enterprise but they may influence its functioning and performance Business environment is the macro framework within which a business firm a micro unit operates It consists of several interrelated and interacting elements

2 Explain the main features of business environment in brief

Totality of External forces-Business environment is the sum total of all things external to a business environment

General and Specific forces-It includes both the forces general forces are the economic social political legal and technological conditions which indirectly influence all business enterprise Specific forces are the investors customers competitors and suppliers which influence individual enterprise directly

Interrelatedness-Different elements of environment are interrelated for an example growing awareness for health care has increased the demand for health foods

Complexity- Business environment id

(such as raw materials capital labour energy and so on) from its environment transforms them into goods and services and sends them back to the environment

Fig 1 Business Environment Relationship

complex in nature as the elements keep on changing example economic technological and other forces changes in demand for a product and service

Dynamic-Business environment is not static it keeps on changing

Uncertainty- Itrsquos very difficult to predict future events such as technology and fashion which occur fast and frequently

Economics Basic Economic ConceptsSub topic

Microeconomics and

Macroeconomics

Welcome to the new sessiontoday we are going to start the first chapter of Class XI The name of the chapter that we are going to start is Basic Economic concepts

Now Economics covers the study of human activities Human activities are those activities which are performed by humans to satisfy their wants

Thus Human wants are unlimited and therefore economic activities such as production exchange and consumption are needed in order to satisfy those wants

The study of economics is divided largely in two parts which areMicroeconomics and Macroeconomics

SUBJECT- MATTER OF ECONOMICS

MICROECONOMICS MACROECONOMICS

Questions1Who has coined the words micro and macro economics

Answer Ranger Frisch coined the words lsquomicrorsquo and lsquomacrorsquo in 1933 to denote the two branches of economic theory namely microeconomics and macroeconomics

2What is microeconomicsAnswer It is the study of behaviour of individual decision ndash making unit such as consumers firms etc

3 What is macroeconomicsAnswer Macroeonomics is the study of overall economic phenomena like employment national income etc

4 What is the importance of microeconomicsAnswer

Microeconomics helps in formulating economic policies which enhance productive efficiency and results in greater social welfare

It helps the government in formulating correct price policies

It explains the working of a capitalistic economy where individual units(producers and consumers ) are free to take their own decision

Micro means a small part in

microeconomics we do not study the whole economy Hence we study an individual consumer and his or her choices and a producer and his or her profit maximizing decisions in the market Thus it does not mirror what happens in the economy as a whole

Macroeconomics on the other hand studies the economy as a whole It is concerned with aggregate and depicts the entire picture of the economyMacroeconomics deals with the national income aggregate investment aggregate consumption etc

Features of Microeconomics It deals with small

parts of the country Hence it looks at

individual consumers firms and industries

It deals with individual income consumption and savings

It studies the determination of price of any product or factors of production

It deals with the working of market via the price mechanism which is nothing but the determination of price and quantity of a commodity by the forces of demand and supply

Features of Macroeconomics

It deals with the study of the economy as a whole

It is concerned with

5 Give a limitation of microeconomics Microeconomics fails to explain the

functioning of an economy as a whole It cannot explain unemployment illiteracy and other problems prevailing in the country

6 What is the importance of macroeconomics It gives overall view of the growing

complexities of an economic system It provides the basic and logical

framework for formulating appropriate macroeconomic policies (eg for inflation poverty etc )to direct and regulate economy towards desirable goals

7What is the limitation of macroeconomics It ignores structural changes in an

individual unit of the aggregate

8 Differentiate between Microeconomics and Macroeconomics

Microeconomics Macroeconomics

the study of aggregates

National income aggregate savings and aggregate investments are major concepts dealt within macroeconomics style

It studies the determination of general price levels

It investigates into the problem of unemployment and the achievement of employment

It studies the aspect of decision making at the aggregate and national levels

It includes all growth theories whether related to developed or developing economies it also includes the study of economic systems and the working of the economy under different systems

Note Both Micro and macro economics are complementary and should be fully utilized for proper understanding of an economy

1It studies economic aspect of an individual unit2It deals with individual incomeConsumption and savings

3 It facilitates determination of price of any product or factors of production

4 Itrsquos scope is narrow and restricted to individual unit

1It studies the economy as a whole

2It deals with the national income aggregate consumption and aggregate savings3 It facilitates determination of general price level in an economy

4 Itrsquos scope is wide as it deals with economic units on the national level

ACCOUNTS

Introduction to Accounting and Book-keeping

Today I am going to share you the meaning of Accounting and Book-keeping and its related terms bullAccounting bullBook Keeping bullAccountsbullTypes Of Accounts bullAccounting Cycle

bull Meaning of accounting

Ans ) Accounting is the art and science of recording classifying and summarising monetary transactions

bull Meaning of Book-keeping

Ans) Bookkeeping is the art of recording business transactions with the view of having a permanent record of them and showing their effect on wealth

bull Meaning of account

Ans) The term account means a record of

business transactions concern a particular person of firm asset or income or expense It is a summarised record of all transactions which take place in an accounting year

bull Types of accountsPersonal accounts ndash Personal accounts relating

to person and Organisation are known as personal accounts Example Ramrsquos Account ABC amp Co Account etc

Real account - The accounts related to tangible and intangible assets are called real accounts Example Cash Account Furniture Account etc

Nominal account- Accounts related to expenses losses incomes and gains are known as nominal accounts Example Wages Account Salary Account Discount Account etc

bull Accounting cycle Accounting cycle refers to a complete sequence of accounting activities It begins with recording of transactions and ends with the preparation of a balance sheet

Chemistry TopicAtomic Structure

Thomsonrsquos atomic modelThomson (1898) was the first to propose the model of an atomHe proposed that an atom can be regarded as a uniform sphere of positive electricity in which requisite number of electrons are embedded evently to neutralize the positive chargeThis is just like plums embedded in a pudding or seeds evently distributed in red spongy mass of a watermelonThis model of atom is known as ldquoPlum-Pudding modelrdquo or

Q1)What is the fundamental constituents of atomAns Electron Proton and neutrons are the fundamental constituents of atomQ2)What is the value of fundamental unit of electricityAnsThe charge carried by one electron is sad to be the fundamental unit of electricityIts magnitude is 48times10-10esuOr 1602times10-19C Q3)Name the element containing no neutronAnsOrdinary hydrogen atom or protium 1H1

Types of AccountPersonal AccountReal AccountNominal AccountBalance Sheet (opening)

ldquowatermelon modelrdquoThis model could explain the electrical neutrality of an atom but failed to explain the result of scattering experiment carried out by Rutherford in 1911So it was rejected ultimately

Q4)Why is an electron called universal particleAns Itrsquos mass and Charge are independent of its source

EVS Chapter 1 ndash Modes of Existence

Modes of existence When one speaks normally about the mode of existence of some group or individual one refers to their customs their mode of being their ethology their habitat in some way their feeling for a placeDifferent modes of exixtence are ndash

1 Hunting ndashGathering2 Pastoral3 Agricultural4 Industrial

1 Hunting and gathering Hunting and gathering mode of existence is characterized by obtaining food from hunting wild animals including fishing and gathering wild plants From their earliest days the hunter-gatherer diet included various grasses tubers fruits seeds and nuts Lacking the means to kill larger animals they procured meat from smaller game or through scavenging

Societies that rely primarily or exclusively on hunting wild animals fishing and gathering wild fruits berries nuts and vegetables to support their diet are called hunting and gathering societies

At least this used to be practice of human beings before agriculture is invented As their brains evolved hominids developed more intricate knowledge of edible plant life and growth cycles

Q) Write the features of Hunting ndash gathering societiesAns - There are five basic characteristics of hunting and gathering societies

i The primary institution is the family which decides how food is to be shared and how children are to be socialized and which provides for the protection of its members

ii They tend to be small with fewer than fifty members

iii They tend to be nomadic moving to new areas when the current food supply in a given area has been exhausted

iv Members display a high level of interdependence

v Labor division is based on sex men hunt and women gather

Political Science

Introduction to political science

Political science occasionally called politology is a social science which deals with systems of governance and the analysis of political activities political thoughts associated constitutions and political behaviorThe study of political science involves the study of both the

Answer the following questions-1 What is political science

Political science occasionally called politology is a social science which deals with systems of governance and the analysis of political activities political thoughts associated constitutions and political behavior

2 Short notes-

traditional and modern theories of politicsTraditionalClassical political sciencepolitical theory-Traditional political science is the study of politics before Second World War The methodology to study Politics was traditional (legal formaletc) the definition of politics traditional (Politics begins and end with state)area of study (constitution state machinery)was traditionalModern Political scienceModern political theory-Modern Political Theory critically examines the contemporary state of political theory making an assessment of the achievement and limitations of the Behavioural Revolution in its totality and reviews objectively the major paradigms and conceptual frameworks adopted by the disciplineContemporary attempts at the development of an integrated political theory involving the use of both traditional and modern concepts approaches and theories-Around late 1960s several political scientists realized the importance of both the traditional political theory and modern Political theory They began building an integrated theory of politics involving a systematic mixture of traditional and modern studies of politics It was held that the study of a complex and vast field like politics needs both traditional as well as

Classical political theory Modern Political theory

Homework-Learn

modern concepts and approaches for studying itrsquos all aspects

Subject Eng Literature (The Tempest ndash William Shakespeare) Topic Act I Scene 1 Lines 1 to 32 (Line 32 ndash Gonzalo hellip If he be not born to be hanged our case is miserable) Date 13th April 2020 (3rd Period)

[Students should read the original play and also the paraphrase given in the school prescribed textbook]Summary Questions amp Answers

[SUMMARY OF THE ENTIRE SCENE]

o The play starts with the scene of a severe storm at sea Alonso (King of Naples) Sebastian (Alonsorsquos brother) Ferdinand (Alonsorsquos son) Gonzalo Antonio (the usurping Duke of Milan) are in a ship in the midst of the storm

o The mariners are trying their best to control the vessel from running aground and are totally following the orders of their Master the Boatswain They have scant success

o The mariners become extremely unhappy and annoyed when most of the passengers arrive on the deck thereby hampering their effort to save the ship There is serious confrontation between them and the passengers who are part of the Kingrsquos entourage

o The mariners could not save the ship

SUMMING-UP

(i) Vivid description of the scene which gives a realistic description of terror and confusion of a tropical storm

(ii) Shows Shakespearersquos accuracy of knowledge in describing the naval operations and also matters of seamanship

(iii) The opening scene justifies the title ndash The Tempest

UNANSWERED QUESTIONS

(i) The King always travels with his entire fleet including his soldiers Where

(1) GONZALO Nay good be patient (Line 15-26)BOATSWAIN When the sea is Hence What cares these

roarers for the name of the king To cabin silence Trouble us not

GONZALO Good yet remember whom thou has aboardBOATSWAIN None that I more love than myself You are a

councillor if you can command these elements to silence and work

the peace of the present we will not hand a rope more use your authority If you cannot give thanks you have

lived so long and make yourself ready in your cabin for the mischance of the hour if it so hap [To the Mariners]

Cheerly good hearts [To Gonzalo] Out of our way I say

(a) To whom is the boatswain speaking What does he mean by lsquoNone that I more love than myselfrsquo

The Boatswain is speaking to Gonzalo the honest old councilor of the Duke of MilanBy using the words ndash lsquoNone that I love more than I love myselfrsquo means that for the Boatswain nobody is dearer to him than his own life

(b) What were the conditions that made the boatswain react in this way

The Boatswain reacts in this way because the storm is at sea and Alonso King of Naples Sebastian his brother Ferdinand his son Gonzalo Antonio the usurping Duke of Milan on board are in distress and in panic Thus they have rushed to the deck interrupting the work of the mariners

(c) What hope does Gonzalo take from the attitude of the boatswain

The insolent and authoritative attitude of Boatswain makes Gonzalo feel comforted He tells that there are no signs that the Boatswain will be drowned But his facial appearance and attitude shows that he is destined to die on land by hanging which in effect means that all on board will be saved Otherwise all the persons on board are doomed

(d) How can they lsquomake yourself ready in your cabinrsquo For what were they asked to make ready themselves

In order to make themselves ready in their cabin the

were the other ships

(ii) Why was the ship in that area Where was it coming from or going where

(iii) The ship broke apart What happened to those who were in the ship

passengers on board must prepare for death which they will possibly soon have to meetThey can retire to their cabins and offer prayers to the Almighty to save them from drowning

(e) What does the boatswain say when he is asked to be patient What does he order to the royal party

When the boatswain is asked to be patient and remain calm he says that he will be patient only when the storm will be over and the sea will be calm but as long as the storm blows and there is danger to the ship he cannot think of being patient He orders the royal party to go to the cabin and leave the mariners to their work

(2) GONZALO I have great comfort from this fellow (Line 27-36)

Methinks he hath no drowning mark upon him his complexion is perfect

gallows Stand fast good Fate to his hanging Make the rope of his destiny our cable for our own doth little advantage If he be not born to be hanged our case is miserable

(a) Why does Gonzalo regard the Boatswain in the midst of danger

In the midst of danger Gonzalo regards the boatswain because he feels that the Boatswain is a source of comfort and is bent upon to do his work sincerely which in this case is saving the ship and its passengers from the severest of raging storm

(b) What reasons does Gonzalo give when he says that none in the ship will die of drowning

Gonzalo is almost sure that none in the ship will die by drowning His says that there is no mark on the face of the boatswain that indicates that he will die by drowning On the other hand the lines on his face are strong indications that he will be hanged to death Therefore there shall be no danger of the shiprsquos sinking

(c) Explain the following ldquoStand fast good Fate to his hanging Make the rope of his destiny our cable for our own doth little advantage If he be not born to be hanged our case is miserablerdquo

The stated lines mean that if the will of destiny is to be carried out then the ship will not get wrecked and all the passengers will be saved The safety of the passengers therefore depends upon the will of fate being carried out in the case of the boatswain If however the boatswain is not to die by hanging then the passengers are also very unsafe because in that case the ship is likely to sink

(d) What order does the Boatswain give to the sailors

when he re-enters What does he say about the crying of the fellows inside the cabin

The boatswain orders the sailors to bring the topmast lower and bring the ship close to a stationary position with the help of the main sail He says that the fellows inside the cabin are moaning and crying in their distress louder than his voice and louder even than the roaring of the storm

Class XII (ScienceCommerceHumanities) Subject Topic Summary Execution

Computer Science

PropositionalLogic

Propositional logic is a procedure to provide reasoning through statementProposition A ststement that results in True or False is said to be proposition There are two types of propositionSimple proposition amp compound propositionSimple proposioton A simple proposition is one that is not a part of any other proposition Such sentential form of proposition is symbolized with english letters in short For example Ram is a claver student (TrueFalse)Where do you live (Not in True or False)Grapes are sweet (TrueFalse)It rains today (TrueFalse)Here we can see some statements anwer would be true or false but some staements answer can not give in terms of true or false Thus the sentences which can be answered in true or false are known as simple propositionAssigning propositon to a variableThe general syntax to assign propostion to a variable is as followsVariable = Simple propositonFor example A=Ram is a clever studentB= Grapes are sweetC= it rains todayCompound proposition

helliphellipto be continued in next classhelliphellipMath Relation Relation If A and B are two non-empty sets

then a relation R from A to B is a subset of AxB If R A x B and (a b) R then we say that a sube isinis related to b by the relation R written as aRbeg Let A be the set of students of class XII and B be the set of students of class XI Then some of the examples of relation from A to B arei) (a b) AXB a is brother of bisinii) (a b) AXB age of a is more than age of isinb Types of relation In this section we would like to study different types of relations We know that a relation in a set A is a subset of A times A Thus the empty set φ and A times A are two extreme relations For illustration consider a relation R in the set A = 1 2 3 4 given by R = (a b) a ndash b = 10 This is the empty set as no pair (a b) satisfies the condition a ndash b = 10 Similarly R = (a b) | a ndash b | ge 0 is the whole primeset A times A as all pairs (a b) in A times A satisfy | a ndash

Example 1 Let A be the set of all students of a boys school Show that the relation R in A given by R = (a b) a is sister of b is the empty relation and R = (a b) the primedifference between heights of a and b is less than 3 meters is the universal relationSolution Since the school is boys school no student of the school can be sister of any student of the school Hence R = φ showing that R is the empty relation It is also obvious that the difference between heights of any two students of the school has to be less than 3 meters This shows that R = A times A is primethe universal relation Example 2 Show that the relation R in the set 1 2 3 given by R = (1 1) (2 2) (3 3) (1 2) (2 3) is reflexive

b | ge 0 These two extreme examples lead us to the following definitionsDefinition 1 A relation R in a set A is called empty relation if no element of A isrelated to any element of A ie R = φ A times AsubDefinition 2 A relation R in a set A is called universal relation if each element of A is related to every element of A ie R = A times A Both the empty relation and the universal relation are some times called trivial relation Definition 3 A relation R in a set A is called(i) reflexive if (a a) R for every a Aisin isin(ii) symmetric if (a1 a2) R implies that (aisin 2a1)

R for all aisin 1 a2 Aisin(iii) transitive if (a1 a2) R and (aisin 2 a3) R isinimplies that (a1 a3) R for all aisin 1 a2 a3 AisinDefinition 4 A relation R in a set A is said to be an equivalence relation if R is reflexive symmetric and transitive

but neither symmetric nor transitiveSolution R is reflexive since (1 1) (2 2) and (3 3) lie in R Also R is not symmetric as (1 2) R but (2 1) isin notinR Similarly R is not transitive as (1 2) R and (2 3) R but (1 3) R isin isin notinExample 3 Show that the relation R in the set Z of integers given byR = (a b) 2 divides a ndash b is an equivalence relationSolution R is reflexive as 2 divides (a ndash a) for all a Z isinFurther if (a b) R then 2 divides a isinndash b Therefore 2 divides b ndash a Hence (b a) R which shows that R is isinsymmetric Similarly if (a b) R and (b c) R isin isinthen a ndash b and b ndash c are divisible by 2 Now a ndash c = (a ndash b) + (b ndash c) is even (Why) So (a ndash c) is divisible by 2 This shows that R is transitive Thus R is an equivalence relation in ZExample 4 Let L be the set of all lines in a plane and R be the relation in L defined as R = (L1 L2) L1 is perpendicular to L2 Show that R is symmetric but neither reflexive nor transitiveSolution R is not reflexive as a line L1 can not be perpendicular to itself ie (L1 L1) R notinR is symmetric as (L1 L2) Risin

L1 is perpendicular to L2rArr L2 is perpendicular to L1rArr (L2 L1) RrArr isin

R is not transitive Indeed if L1 is perpendicular to L2 and L2 is perpendicular to L3 then L1 can never be perpendicular to L3 In fact L1 is parallel to L3 ie (L1 L2) R isin(L2 L3) R but (L1 L3) Risin notin

Chemistry Solid state Characteristics if Solids(i)The particles are locked in fixed positions they are unable to change their relative positions and this brings a definite shape and volume of a solid(ii)In a solid the constituent particles are held by strong forces of attractionThe forces of attraction may be bonding or non bonding(iii)The constituent particles in a solid pack together as closely as possibleoccupying most of the available space within the solidThus the empty space in a solid is very smallThis makes a solid highly rigid and nearly incompressibleThis also explains why a solid has high density and exhibits slow diffusionClassification of Solids

Q1)Define Crystalline solids AnsA Solid that has a definite geometrical shape and a sharp melting pointand whose constituent particles (atomsmolecules or ions) are arranged in a long range order of definite pattern extending throughout the solid is called a crystalline solidExNaClQ2)Define Amorphous solids AnsA solid that does not have a definite shape and a sharp melting pointand whose constituent particles (atomsmolecules or ions) are not arranged in a definite pattern is called an amorphoussolid

Crystalline solidsAmorphous solids

ExGlassRubberQ3)Classify Crystalline Solids Crystalline Solids

Physics Coloumbrsquos Law (Summary)

Before Going Into Coloumbrsquos Law We Will First Learn What is Charge Properties of Charge and Always remember that charge is quantized ie a body always have static charge of magnitude equal to some integral multiple of fundamental electronic charge e= 16 x 10- 19 C

Charge is the property of matter that causes it to produce and experience electrical and magnetic effects The study of the electrical charges at rest is called electrostatics When both electrical and magnetic effects are present the interaction between charges is referred to as electromagnetic

There exist two types of charges in nature positive and negative Like charges repel and unlike charges attract each other

The type of charge on an electron is negative The charge of a proton is the same as that of an electron but with a positive sign In an atom the number of electrons and the number of protons are equal The atom is therefore electrically neutral If one or more electrons are added to it it becomes negatively charged and is designated as negative ion However if one or more electrons are removed from an atom it becomes positively charged and is called a positive ion

The excess or deficiency of electrons in a body gives the concept of charge If there is an excess of electrons in a body it is negatively charged And if there is deficiency of electrons the body becomes positively charged Whenever addition or removal of electrons takes places the body acquires a charge

The SI Unit of charge is coulomb (C) In SI units the current is a fundamental quantity having a unit of ampere (A) The unit of charge is defined in terms of the unit of current Thus one coulomb is the charge transferred in one second across the section of a wire carrying a

Ionic SolidsMetallicSolids

Molecular Solids

current of one ampere

As q = It we have1 C = (1 A) (1 s)

The dimensions of charge are [A T]

Properties of Charge

(1) Quantization of Charge Electric charge can have only discrete values rather than any value That is charge is quantized The smallest discrete value of charge that can exist in nature is the charge on an electron given as

e = plusmn 16 x 10- 19 C

This is the charge attained by an electron and a protonA charge q must be an integral multiple of this basic unit That is

Q = plusmn ne where n = 1 2 hellip

Charge on a body can never be (frac12)e (23)e or 57e etcWhen we rub a glass rod with silk some electrons are transferred from the rod to the silk The rod becomes positively charged The silk becomes negatively charged The coulomb is a very large amount of charge A typical charge acquired by a rubbed body is 10 - 8 C

Biology Reproduction in organisms

Welcome to this new session 2020-21Today in this first chapter we mainly discuss about reproduction types needs and life span of some organismsWe also discuss about difference between sexual and asexual reproduction

Q1 What is reproductionReproduction is defined as a biological processin which an organism gives rise to young onessimilar to itselfQ2 What are the needs of reproductionbulli) Reproduction maintain life on earthii) It enables the continuity of the species generation after generationiii) It creates genetic variation among populationsQ3 Define Life span and write some orgnisms life spanbull Life span is the period from birth to

the natural death of an organism- OrganismsLife span1 Butterfly 1 - 2 weeks2 Fruit fly 30 days3Dog 10-13 years4 Rose5-7 years5 Tortoise100-150 years6 Banyan Tree -200 - 250 yearsQ4 Reproduction is of two types in case ofanimals but in case of plants vegetative propagation is also present

Asexual Reproduction Sexual Reproductioni) Always uniparentalii) Gametes are not involvediii) Only mitotic division involvediv) Somatic cells of parents are involvedv) Offsprings are genetically similar to the parents

i) Usually biparentalii) Gametes are involvediii) Meiosis occurs during gametogenesis Mitosis occurs after fertilisationiv) Germ cells of the parents are involvedv) offsprings are genetically different from the parents

COMMERCE BUSINESS ENVIRONMENT

Welcome to the new sessiontoday we are going to start the first chapter of Class XII The name of the chapter is Business Environment

Already many of you have got some idea about the word business environment form the first chapter of business studies in class XI

In todayrsquos world every business enterprise is a part of the society It exists and operates in association with various groups in society such as customers suppliers competitors banks and financial institutions government agencies trade unions media and so on All these groups influence the functioning of business in one way or the other They constitute the environment of businessConcept of Business Environment

The term lsquobusiness environmentrsquo refers to the sum total of all individuals institutions and other forces that lie outside a business enterprise but that may influence its functioning and performance

The main features of business environment Totality of External forces General and Specific forces Interrelatedness Complexity Dynamic Uncertainty

Prepare the following questions from todayrsquos assignment

2 What do you mean by business environment

The term lsquobusiness environmentrsquo means the aggregate of all forces factors and institutions which are external to and beyond the control of an individual business enterprise but they may influence its functioning and performance Business environment is the macro framework within which a business firm a micro unit operates It consists of several interrelated and interacting elements

2 Explain the main features of business environment in brief

Totality of External forces-Business environment is the sum total of all things external to a business environment

General and Specific forces-It

Relativity

The Interrelation between business and its environment

The business enterprise is an open system It continuously interacts with its environment It takes inputs (such as raw materials capital labour energy and so on) from its environment transforms them into goods and services and sends them back to the environment

Fig 1 Business Environment Relationship

includes both the forces general forces are the economic social political legal and technological conditions which indirectly influence all business enterprise Specific forces are the investors customers competitors and suppliers which influence individual enterprise directly

Interrelatedness-Different elements of environment are interrelated for an example growing awareness for health care has increased the demand for health foods

Complexity- Business environment id complex in nature as the elements keep on changing example economic technological and other forces changes in demand for a product and service

Dynamic-Business environment is not static it keeps on changing

Uncertainty- Itrsquos very difficult to predict future events such as technology and fashion which occur fast and frequently

Business Studies

Human Resources Management

Human resource of an organisation are the aggregate of knowledge skills attitudes of people working in it

The management system which deals with human resources is called human resource management

Features of HRMbullComprehensive functionbullPeople-oriented

Question1) What do you mean by human

resource management Answer) Human resource management may be defined as that field of Management which has to do with planning organising and controlling the functions of procuring developing maintaining and utilising the labour force

bullAction oriented bullPervasive function bullContinuous function

2) Explain the features of HRM in brief

Answer)bullHuman Resource Management is concerned with managing people at work bull Human Resource Management is concerned with employees which bring people and organisations together so that the goals of each are met bullHuman resource management considered every employees as an individual and also promote their satisfaction and growth bull Human resource management is inherent in all organisations and at all levelsbullManagement of human resources are ongoing on never ending process which requires a constant alertness and Awareness of human relations

3) ldquoHR function is said to be pervasiverdquowhy

Answer) Human resource management is required in all organisations whether it is private or government organisations armed forces sports organisations etc It permeatsall the functional areas like production marketing finance research etc This from this feature of human resource management it can be said that it is pervasive in nature

Economics Demand Q1DEFINITION OF DEMANDIn economics demand is the quantity of a good that consumers are willing and able to purchase at various prices during a given period of timeQ2DEMAND CURVEIn economics a demand curve is a graph depicting the relationship between the price of a certain commodity and the quantity of that commodity that is demanded at that pricQ3LAW OF DEMANDIn microeconomics the law of demand states that conditional on all else being equal as the price of a good increases quantity demanded decreases conversely as the price of a good decreases quantity demanded increasesQ4ASSUMPTION of LAW OF DEMAND(i)No change in price of related commodities(ii) No change in income of the consumer(iii) No change in taste and preferences customs habit and fashion of the consumer( No expectation regarding future change in priceQ5MARKET DEMAND SCHEDULEIn economics a market demand schedule is a tabulation of the quantity of a good that all consumers in a market will purchase at a

given price At any given price the corresponding value on the demand schedule is the sum of all consumersrsquo quantities demanded at that priceQ6INDIVIDUAL DEMAND SCHEDULEIndividual demand schedule refers to a tabular statement showing various quantities of a commodity that a consumer is willing to buy at various levels of price during a given period of timeQ7 FACTORS AFFECTING INDIVIDUAL DEMAND FOR A COMMODITY

The factors that influence a consumerrsquos decision to purchase a commodity are also known as determinants of demand The following factors affect the individual demand for a commodity1 price of the commodity2 price of related goods3 income of buyer of the commodity4 tastes and preferences of the buyer1 Price of the CommodityYou must have observed that when price of a commodity falls you tend to buy more of it and when its price rises you tend to buy less of it when all other factors remain constant (lsquoother things remaining the samersquo) In other words other things remaining the same there is an inverse relationship between the price of a commodity and its quantity demanded by its buyers This statement is in accordance with law of demand which you will study in the later part of this lesson Price of a commodity and its quantity demanded by its buyers are inversely related only when lsquoother things remain the samersquo So lsquoother things remaining the samersquo is an assumption when we study the effect of changes in the price of a commodity on its quantity demanded2 Price of Related goodsA consumer may demand a particular good But while buying that good heshe also asks the price of its related goods Related goods can be of two types-(i) Substitute goods(ii) Complementary goods While purchasing a good prices of its substitutes and complements do affect its quantity purchased(i) Price of Substitute Goods Substitute goods are those goods which can easily be used in place of one another for satisfaction of a particular want like tea and coffee An increase in price of substitute good leads to an increase in demand for the given commodity and a decrease in price of substitute good leads to a decrease in demand for the given commodity It means demand for a given commodity is directly affected by change in price of substitute goods For example if price of coffee increases the demand for tea will rise as tea will become relatively cheaper in comparison to coffee(ii) Price of Complementary goods Complementary goods are those goods which are used together to satisfy a particular want like car and petrol An increase in the price of complementary goods leads to a decrease in demand for the given commodity and a decrease in the price of complementary goods leads to an increase in demand for the given commodity For example if price of petrol falls then the demand for cars will increase as it will be relatively cheaper to use both the goods together So demand for a given commodity is inversely affected by change in price of complementary goods3 Income of the Buyer of CommodityDemand for a commodity is also affected by income of its buyer However the effect of change in income on demand depends on the nature of the commodity under consideration In case of some goods like full cream milk fine quality of rice (Basmati rice) etc demand for these commodities increases when income of the buyer increases and

demand for these commodities decreases when income of the buyer decreases Such goods whose demand increases with the increase in income of the buyer are called normal goods But there are some goods like coarse rice toned milk etc whose demand decreases when income of buyer increases and their demand increases when income of the buyer decreases Such goods whose demand decreases with the increase in income of the buyer are called inferior goods Suppose a consumer buys 10 Kgs of rice whose price is ` 25 per Kg He cannot afford to buy better quality of rice because the price of such rice is ` 50 per Kg The consumer is spending ` 250 per month on the purchase of rice Now if income of the consumer increases and he can afford ` 350 on purchase of 10 Kg of rice Now he can afford to buy some quantity of rice say 6 Kgs whose price is ` 25 per Kg and may buy 4 Kgs of rice whose price is ` 50 per Kg Thus he will buy 10 Kgs of rice by spending ` 350 per month Therefore we may conclude that demand for normal goods is directly related to the income of the buyer but demand for inferior goods is inversely related to the income of the buyer4 Tastes and Preferences of the BuyerThe demand for a commodity is also affected by the tastes and preferences of the buyers They include change in fashion customs habits etc Those commodities are preferred by the consumers which are in fashion So demand for those commodities rises which are in fashion On the other hand if a commodity goes out of the fashion its demand falls because no consumer will like to buy it(5) Number of Buyers in the Market(Population)Increase in population raises the market demand whereas decrease in population reduces the market demand for a commodity Not only the size of population but its composition like age (ratio of males females children and old people in population) also affects the demand for a commodity It is because of needs of children young old male and female population differs(6) Distribution of Income and WealthIf the distribution of income and wealth is more in favour of the rich demand for the commodities preferred by the rich such as comforts and luxuries is likely to be higher On the other hand if the distribution of income and wealth is more in favour of poor demand for commodities preferred by the poor such as necessities will be more(7) Season and Weather ConditionsThis is generally observed that the demand for woolens increases during winter whereas demand for ice creams and cold drinks increases during summer Similarly market demand for umbrellas rain coats increases during rainy seasonQ8 REASONS FOR OPERATION OF LAW OF DEMAND WHY DEMAND CURVE SLOPES DOWNWARDNow we will try to explain why does a consumer purchase more quantity of a commodity at a lower price and less of it at a higher price or why does the law of demand operate ie why does the demand curve slope downwards from left to right The main reasons for operation of law of demand are1 Law of Diminishing Marginal UtilityAs you have studied earlier law of diminishing marginal utility states that as we consume more and more units of a commodity the utility derived from each successive unit goes on decreasing The consumer will be ready to pay more for those units which provide him more utility and less for those which provide him less utility It implies that he will purchase more only when the price of the commodity falls2 Income Effect

When price of a commodity falls purchasing power or real income of the consumer increases which enables him to purchase more quantity of the commodity with the same money income Let us take an example Suppose you buy 4 ice creams when price of each ice cream is ` 25 If price of ice creams falls to ` 20 then with same money income you can buy 5 ice creams now3 Substitution EffectWhen price of a commodity falls it becomes comparatively cheaper as compared to its substitutes (although price of substitutes has not been changed) This will lead to rise in demand for the given commodity For example if coke and Pepsi both are sold at ` 10 each and price of coke falls Now coke has become relatively cheaper and will be substituted for Pepsi It will lead to rise in demand for coke4 Change in Number of BuyersWhen price of a commodity falls some old buyers may demand more of the commodity at the reduced price and some new buyers may also start buying this commodity who were not in a position to buy it earlier due to higher price This will lead to increase in number of buyers when price of the commodity falls As a result demand for the commodity rises when its price falls5 Diverse Uses of a CommoditySome commodities have diverse uses like milk It can be used for drinking for sweet preparation for ice cream preparation etc If price of milk rises its use may be restricted to important purpose only This will lead to reduction in demand for other less important uses When price of milk falls it can be put to other uses also leading to rise n demand for itQ9 EXCEPTIONS TO THE LAW OF DEMANDYou have studied in law of demand that a buyer is willing to buy more quantity of a commodity at a lower price and less of it at a higher price But in certain circumstances a rise in price may lead to rise in demand These circumstances are called Exceptions to the Law of Demand Some important exceptions are1 Giffen GoodsGiffen goods are special type of inferior goods in which negative income effect is stronger than negative substitution effect Giffen goods do not follow law of demand as their demand rises when their price rises Examples of Giffen goods are jowar and bajra etc2 Status Symbol GoodsSome goods are used by rich people as status symbols eg diamonds gold jewellary etc The higher the price the higher will be the demand for these goods When price of such goods falls these goods are no longer looked at as status symbol goods and tehrefore therir demand falls3 NecessitiesCommodities such as medicines salt wheat etc do not follow law of demandbecause we have to purchase them in minimum required quantity whatever their price may be4 Goods Expected to be ScarceWhen the buyers expect a scarcity of a particular good in near future they start buying more and more of that good even if their prices are rising For example during war famines etc people tend to buy more of some goods even at higher prices due to fear of their scarcity in near future

Political Science

Constitution of India-The

Preamble

The preamble-

Preamble-

The preamble is the most precious part of the constitution We the people of India having solemnly resolved to constitute India into a Sovereign Socialist Secular Democratic Republic and to secure to all its citizensA preamble is an introductory and expressionary statement in a document that explains the documents purpose and underlying philosophy When applied to the opening paragraphs of a statute it may recite historical facts pertinent to the subject of the statuteNature and purpose of the constitution-Purpose of the Constitution dictates permanent framework of the government to form a more perfect union to establish justice and ensure peace of thenationconstitution provide principles how the government can run itself following the rules and laws written in the constitution of each state keeps them balanced

Answer the following questions-

1 What is preambleA preamble is an introductory and expressionary statement in a document that explains the documents purpose and underlying philosophy2 What is the nature and

purpose of the constitutionConstitution dictatespermanent framework of the government to form a more perfect union to establish justice and ensure peace of the nation

Homework-Learn

Accounts Compatibilty mode

1MEANING OF PARTNERSHIPPartnership is a form of business organisation where two or more persons join hands to run a business They share the profits and losses according to the agreement amongst them According to the Indian Partnership Act 1932 ldquoPartnership is relation between persons who have agreed to share profits of a business carried on by all or any one of them acting for allrdquo For example one of your friends has passed class XII from National Institute of Open Schooling (NIOS) and wants to start a business Heshe approaches you to join in this venture Heshe wants you to contribute some money and participate in the business activities Both of you if join hands constitute a partnership2CHARACTERISTICS1048698 Agreement A partnership is formed by an agreement The agreement may be either oral or in writing It defines the relationship between the persons who agree to carry on business It may contain the terms of sharing profit and the capital to be invested by each partner etc The written agreement is known as partnership deed1048698 Number of persons There must be at least two persons to form a partnership

The maximum number of partners in a partnership firm can be 50 according toCompanies Act 20131048698 Business The Partnership is formed to carry on business with a purpose of earning profits The business should be lawful Thus if two or more persons agree to carry on unlawful activities it will not be termed as partnership1048698 Sharing Profits The partners agree to share profits in the agreed ratio In caseof loss all the partners have to bear it in the same agreed profit sharing ratio10486981048698Mutual Agency Every partner is an agent of the other partners Every partner can bind the firm and all other partners by hisher acts Each partner will be responsible and liable for the acts of all other partners10486981048698Unlimited liability The liability of each partner except that of a minor is unlimited Their liability extends to their personal assets also If the assets of the firm are insufficient to pay off its debts the partnersrsquo personal property can be used to satisfy the claim of the creditors of the partnership firm10486981048698Management All the partners have a right to mange the business However they may authorize one or more partners to manage the affairs of the business on their behalf10486981048698Transferability of Share No partner can transfer hisher share to any one including hisher family member without the consent of all other partners3PARTNERSHIP DEEDAgreement forms the basis of partnership The written form of the agreement is which a document of partnership is It contains terms and conditions regarding the conduct of the business It also explains relationship between the partners This document is called partnership deed Every firm can frame its own partnership deed in which the rights duties and liabilities of the partners are stated in detail It helps in settling the disputes arising among the partners during the general conduct of business 4CONTENTS OF PARTNERSHIP DEEDThe partnership deed generally contains the following (i) Name and address of the partnership firm(ii) Nature and objectives of the business(iii) Name and address of each partner(iv) Ratio in which profits is to be shared(v) Capital contribution by each partner(vi) Rate of Interest on capital if allowed(vii) Salary or any other remuneration to partners if allowed(viii) Rate of interest on loans and advances by a partner to the firm(ix) Drawings of partners and interest thereon if any(x) Method of valuation of goodwill and revaluation of assets and liabilities on the reconstitution of the partnership ie on the admission retirement or death of a partner(xi) Settlement of disputes by arbitration(xii) Settlement of accounts at the time of retirement or death of a partner5IN ABSENCE OF PARTNERSHIP DEEDThe partnership deed lays down the terms and conditions of partnership in regard to rights duties and obligations of the partners In the absence of partnership deed there may arise a controversy on certain issues like profit sharing ratio interest on

capital interest on drawings interest on loan and salary of the partners In such cases the provisions of the Indian Partnership Act becomes applicableSome of the Issues are(i) Distribution of Profit Partners are entitled to share profits equally(ii) Interest on Capital Interest on capital is not allowed(iii) Interest on Drawings No interest on drawing of the partners is to be charged(iv) Interest on Partnerrsquos Loan A Partner is allowed interest 6 per annum on the amount of loan given to the firm by himher(v) Salary and Commission to Partner A partner is not entitled to anysalary or commission or any other remuneration for managing the business

History TOPIC-TOWARDS INDEPENDENCE AND PARTITION THE LAST PHASE (1935-1947)

SUB TOPIC-IMPORTANT POLITICAL DEVELOPMENTS ndash GROWTH OF SOCIAL IDEAS

Socialism is a political social and economic philosophyLike in other parts of the world the Russian revolution of 1917 served as a great inspiration for revolutionaries in India who at that time were engaged in the struggle for liberation from British ruleSocialist ideas led to the formation of communist party of IndiaJAWAHARLAL NEHRU Among the early Congress leaders Jawaharlal Nehru was very much impressed and influenced by the Socialist ideas He also learnt about the Economic activities of the Soviet Union after the Bolshevic Revolution 1917 He made full use of them in IndiaThe election of Jawaharlal Nehru and Subhas Chandra Bose showed the Left wing tendency within CongressJawaharlal Nehru demanded economic freedom along with political freedom of the people in order to end the exploitation of masses

Nehrus working committee included three socialists leaders The Lucknow session was a landmark in the evolution of socialist ideas of the congressSUBHAS CHANDRA BOSE ndash Subhas Chandra Bose had socialist leaning Both Jawaharlal Nehru and Subhas Chandra Bose were known as leftist Congress men Later on National Congress divided into Leftist and rightist campCONGRESS SOCIALIST Within the Congress some leaders formed the Congress Socialist partyPattavi Sitaramyya Sardar Patel Rajendra Prasad had hostile attitude towards the Congress Socialist partyJawaharlals attitude was hesitant

1 QUESTION ndash Mention name of two Congress leaders who had socialist leaning

1ANSWER ndash Subhas Chandra Bose and Jawaharlal Nehru2QUESTION- In which session of the congress Jawaharlal elaborated his Socialist ideas2 ANSWER ndash Lucknow and Faizpur Session in December 1935 and 19363QUESTION ndash Why Congress was sharply divided into leftist and rightist camp 3ANSWER ndash Subhas Chandra Bosersquos attempt to seek re election for congress presidentship in 1939sharply divided the National Congress into Leftist and Rightist camp4 QUESTION ndash Who was MN Roy 4 ANSWER ndash Manabendra Roy first formed the Communist Party of India outside the country at Tashkent in 19205QUESTION ndash Who formed the Congress Socialist Party within the Congress5 ANSWER ndash Jaya Prakash Narayan Achyut Patwardhan Acharya Narendra Dev Ram Mohan Lohia Aruna Asaf Ali6QUESTION ndash When was the Congress Socialist Party formed What was its object6 ANSWER ndash 1934The Congress Socialist Party sought to work out socialist programme through the Congress They joined hands with the Congress and wanted to carry

Subhas Chandra Bose being expelled from the congress after the Tripuri rift he formed Forward BlockThere were basic differences between the Congress Socialists and the communistsTRADE UNION ACTIVITIES Maximum working class people lived in Bombay and Calcutta The working and living conditions of those workers were very miserable In this situation Shasipada Banerjee NM Lokhande protested against the oppression of the working class peopleThe first Trade Union Madras Labour Union was formed in 1918 by BP WadiaIndustrial strikes took place in Kanpur Calcutta Madras Jamshedpur and Ahmedabad AITUC was formed in Bombay in 1927 The growth of Trade union among the workers was slow because of the fear of the dismissal of the jobIn the mean time the Moderates as well as Communists left AITUC and formed separate organization

on National struggle with the help of workers and peasant class of the society7 QUESTION ndash What was the name of the party founded by Subhas Chandra Bose7 ANSWER- Forward Block8QUESTION ndash Who was Shasipada Banerjee8 ANSWER ndash Shasipada Banerjee was a radical Brahmo He founded a working menrsquos club to protest against exploitation of the British rulers towards the working class of India9 QUESTION ndash What was the weekly published by NM Lokhande9ANSWER- Dinabandhu10 QUESTION ndash Who founded Bombay Mill-Hands Association and in which year10 ANSWER- NM Lokhande in189011 QUESTION- Who was BP WadiaANSWER- BPWadia was the founder of Madras Labour Union in191812 QUESTION- What was the name of the first labour union of India12 ANSWER- Madras Labour Union13 QUESTION Who founded the Majur Mahajan 13 ANSWER GANDHIJI14 QUESTION What was the full form of AITUC When it was formed14 ANSWER All India Trade Union Congressin 192715QUESTION Who formed the Red Trade Union Congress and in which year15ANSWER The Communists formed the Red Trade Union Congress16 QUESTION What do you mean by Socialism16 ANSWER Socialism describes any political and economic theory that says the community rather than individuals should own and manage property and natural resources

Subject Eng Literature (The Tempest ndash William Shakespeare) Topic Act III Scene 3 Lines 1 to 52 (Line 52 ndash Brother my lord the Duke Stand to and do as we) Date 13th April 2020 (4th Period)

[Students should read the original play and also the paraphrase given in the school prescribed textbook]Summary Questions amp Answers

o Alonso Sebastian Antonio Gonzalo Adrian Francisco and others wandered about the island in search of Ferdinand and gets tired and hungry of the toil and at the same time gives up all hope of finding him

o Antonio and Sebastian are happy that Alonso is out of hope and decide to make another attempt on his life that night when being so tired they will be sleeping soundly

o Suddenly a solemn and strange music is heard in the air and several strange shapes enter bringing in a banquet These strange shapes then dance round it with gestures of salutation and then inviting the King to eat they depart

o Seeing this strange scene all are inclined to believe the tales told by travelers that there truly are ldquounicornsrdquo and ldquothe phoenixrsquo thronerdquo

1 ALONSO What harmony is this My good friends hark (L18-27)

GONZALO Marvellous sweet music

[Enter several strange shapes bringing in a banquet

they dance about it with gentle actions of salutation

and inviting the King and his companions to eat they depart]ALONSO Give us kind keepers heavens What were theseSEBASTIAN A living drollery Now I will believe

That there are unicorns that in Arabia

There is one tree the phoenixrsquo throne one phoenix

At this hour reigning thereANTONIO Ill believe both

And what does else want credit come to me

And Ill be sworn rsquotis true Travellers neer did lie

Though fools at home condemn rsquoem

(a) How did Prospero present an amazing spectacle before Alonso and his companions

Using his magic powers Prospero ordered strange shapes to lay a banquet before Alonso and his companions The shapes brought several dishes with tasty eatables in them They placed the dishes on a table before Alonso and his companions Then the strange shapes began to dance gracefully around the banquet While dancing they made gestures inviting them to eat the food Then suddenly the shapes disappeared(b) Who were the guests at the strange banquet Describe the lsquoliving drolleryrsquo

Alonso Sebastian Antonio Gonzalo Adrian and Francisco were the guests at the strange banquet

The term ldquoliving drolleryrdquo refers to live entertainment show In this context when Alonso the King of Naples Sebastian his brother Antonio the treacherous brother of Prospero Gonzalo the kind and loyal councillor to the King Adrian and Francisco came to the island they were hungry and weary in their spirits They heard a solemn and strange music They were shocked to see several strange shapes bringing in a banquet and these shapes danced about it with gentle action of salutation inviting the King and his companions to eat After this Sebastian described this show as lsquoliving drolleryrsquo(c) What is lsquophoenixrsquo What are lsquoUnicornsrdquo

The term lsquophoenixrsquo refers to a mythical Arabian bird which lived alone and perched on a solitary tree After one hundred years it expired in flames and rose again from its own ashes

lsquoUnicornsrsquo refers to the mythological four-footed beasts having horns in the centre of their foreheads When the horns are ground into powder the powder was believed to be

an aphrodisiac(d) How does Sebastian explain the puppet show OR Why does the speaker now believe in unicorns and phoenix

Sebastian finds several strange shapes bringing in the banquet They invite the king and his party for dinner and soon depart He tells that if such a strange sight can be a reality there is nothing incredible in the world and from the present moment he will believe anything He says that it is a strange dumb show enacted not by puppets but by living beings It is stranger than a travellerrsquos tale Seeing such a thing

before his own eyes he will no longer disbelieve the story about unicorns and phoenix(e) How do the other characters present respond to this living drollery

At the sight of the lsquoliving drolleryrsquo like Sebastian Gonzalo and Antonio too acted strangely Antonio told that he too now believes in unicorns and phoenix and anything else that seems to be incredible He too now believes in travellersrsquo tales Gonzalo told that if he would report those happenings in Naples nobody will believe him He considers that those gentle shapes were gentler in manner in comparison to the living beings Alonso was at first sight suspicious and told them that those strange shapes conveyed their meaning in expressive gestures when they seemed to lack speech by their movements and sounds Francisco was amazed at their mysterious disappearance

2 ALONSO Not I

(Line 43-52)GONZALO Faith sir you need not fear When we

were boysWho would believe that there were mountaineers

Dewlapped like bulls whose throats had hanging at rsquoem

Wallets of flesh Or that there were such men

Whose heads stood in their breasts Which now we find

Each putter-out of five for one will bring us

Good warrant ofALONSO I will stand to and feed

Although my lastmdashno matter since I feel

The best is past Brother my lord the Duke

Stand to and do as we

(a) How does Alonso respond at the spectacle of the shapes which were sent to them at the instruction of Prospero

After seeing the strange sight of appearing and disappearing of the shapes sent by Prospero to arrange a banquet for them Alonso says that his surprise at having seen those creatures is infinite and he is fully justified in feeling so much surprise He thinks that their shapes their gestures and the sounds they made were indeed amazing Although they do not possess the gift of speech yet they were able to convey their

thoughts by means of their gestures only

(b) What does Prospero say about the views expressed by Alonso regarding the shapes What does Francisco think about the shapesAfter hearing Alonsorsquos views about the shapes Prospero says that this manrsquos praise of the spirits is rather hasty He means to say that Alonso has shown great haste in reaching the conclusion about the shapes Francisco is amazed to see that those shapes disappeared in a mysterious way(c) What does Sebastian ask Alonso to doSebastian tells Alonso that the shapes having disappeared should not matter to them because they have left the eatables behind He asks Alonso to enjoy eating as they are extremely hungry but the king does not accept his offer of enjoying the dishes(d) How does Gonzalo try to dispel Alonsorsquos fear of those strange shapes What kind of references does he give to AlonsoGonzalo says that those who have travelled abroad have reported seeing even stranger sights than these shapes that Alonso and his companions have beheld Hence there is no reason to feel afraid of these shapes Gonzalo further adds that in his younger days he had heard strange stories from travelers and Alonso might have heard similar stories For instance it was said that there existed a certain race of

human beings who had huge lumps of flesh hanging at their throats and who therefore resembled bulls Then Gonzalo tells about a race of human beings whose heads were located at their breasts Gonzalo says that such stories were not believed by most people in those days but now-a-days these stories have become common(e) Explain the following lsquoEach putter-out of five for onersquoEnglish travellers often insured their trips with London brokers Those that went on foreign travels those days used to deposit a certain amount with some firm or company in London before their departure If the travelers failed to return the money was forfeited by the company with which it had been deposited But this money was repaid five-fold if the travelers returned safe and sound In this way a traveler stood a great chance of recovering the entire cost of his

travels(f) Give the explanatory meanings of the following expressions in the context of the above extract (i) Dewlapped (ii) Wallets of flesh

(iii) Putter-out(i) Dewlapped having big lumps of flesh at the necks(ii) Wallets of flesh large masses of flesh looking like bags(iii) Putter-out to invest money before commencing the travel

  • General methods of preparation of hydrogen
  • Chapter Dimensional Analysis (Summary)
    • Properties of Charge
Page 14:   · Web viewSubject. Topic. Summary. Execution. Hindi. व्याकरण. शरीरके अंगो के नाम लिखिए. 1) आँख 2) नाक 3

English 2 Sentences based on meanings

Kinds of sentences

Assertive or declarative to convey information or simply make a statement

Interrogative to ask different types of questions

Imperative to command or instruct someone or make a request

Exclamatory to express strong feelings and emotions

Exercise B1 Stop it ( Exclamatory)2 May you always be happy

together ( Exclamatory)3 He does not like sports

( Assertive)4 Please pass me the salt

( Imperative)5 How dare she talk to me like

that ( Exclamatory)6 May success bless your effort

( Exclamatory)7 Canrsquot you wait for sometime

(Interrogative)8 Did anybody tell you about it

( Interrogative)9 I saw her waiting for the bus

( Assertive)10 Could you please take a

message for me ( Interrogative)

Homework Ex ABiology Chp -2

Classification of Plants

Today we discuss about usefulness of bacteria We also discuss what the harmful effects of bacteria are

89 How bacteria are useful for usbull Bacteria is helpful in many ways forhuman being i) Production of medicine - antibiotics vaccine etcii) Formation of curd by lactobacillusiii)Nitrogen fixation in Leguminousplant by Rhizobiumiv) Increase soil fertility by absorbingatmospheric nitrogen and convert it into nitrates and nitritesv) Cleaning the environment by converting the complex substances into simple substancesvi) Tanning of leathervii) Retting of Fibersviii) Formation of compost by acting onanimal dung and agricultual cases1x) Biogas production by decomposingplant and animal wastex)Help In Nutrition by producing vitamiacutemBand kx1) Some bacteria are used to give specialflavour to tea coffee and coccaQ10- Name some diseases and there causativebacteriabull Diseasescausative bacteria1 CholeraVibrio cholerae2 Tuberculosis - Mycobacterium tuberculosis3 Diptheria -Corynebacteriumdiphtheriae4 Pneumonia - Streptococcus pneumoniae

Math Number system

Chapter Fraction

Study item Using lsquoofrsquoThe word lsquoofrsquo between any two fractions is to be used as multiplicationExample 57 of 56 = 57 times 56 = 5times8 = 40Study item Using BODMASThe word lsquoBODMASrsquo is the abbreviation formed by taking the initial letters of six operations(i)Bracket (ii) of (iii) Division (iv) Multiplication (v) Addition (vi) SubtractionAccording to BODMAS rule First of all the terms inside Bracket must be simplified then lsquoofrsquo lsquoDivisionrsquo lsquoMultiplicationrsquo lsquoAdditionrsquo lsquosubtractionrsquo

Study item Removal of Brackets

There are four Brackets of algebra in Mathematics In a complex expression four types of brackets are used Order of removing the brackets is first ----- then ( ) then finally [ ]

Class VIIISubject Topic Summary Execution

Chemistry Hydrogen General methods of preparation of hydrogen

By the action of dilute acids on metals

Calcium Reacts readily to form chloride salt and hydrogen

Ca + 2HCl rarr CaCl₂ + H₂uarr

Magnesium

Aluminium

Zinc

React readily to form salt and hydrogen

Mg + 2HCl rarr MgCl₂ + H₂uarr2Al + 6HCl rarr 2AlCl₃ + 3H₂uarrZn + 2HCl rarr ZnCl₂ + H₂uarr

Question 4 ) Give reasons for the following

(a) Hydrogen be used as a fuel

Solution

Hydrogen is used as a fuel because it has a high heat of combustion Some significant fuels are coal gas water gas and liquid hydrogen

(b) Though hydrogen is lighter than air it cannot be collected by downward displacement of air

Solution

Hydrogen is lighter than air so it is possible to collect the gas by downward displacement of air But it is not safe to do so since a mixture of hydrogen and air can lead to an explosion

(c) A pop sound produced when hydrogen is burnt

Solution

Impure hydrogen gas burns in air with a pop sound This is because of the presence of impurities in it

(d) Helium replaced hydrogen in weather observation balloons

Solution

It forms a mixture with air that can explode when there is a small leakage of hydrogen in a balloon So helium has replaced hydrogen

(e) Nitric acid not used for the preparation of hydrogen gas

Solution

(e) By the action of nitric acid on metals hydrogen cannot be produced because it also releases nitrous oxide and nitric oxide and oxides the hydrogen to form water

Biology Chp-2 Reproduction in plants

Today we discuss different methods of artificial propagation like cutting-rose sugercane Layering ndashguava lemon china rose etc Grafting- mango apple etcMicropropagation ndashorchid asparagus etcWe also discuss about advantages and disadvantages of vegetative propagation

Q7 Define the following terms i) Explant In tissue culture techniquea tiny piece of bud shoot or any other partof plant from where new tissue develop ii) Callus The cells of the tissue divide andgrow into a mass of undifferentiated cells from explant iii) Plantlet After few days callus differentiate into a small plant with roots and shootQ8 what are the advantages and limitations of tissue culture or micropropagation

Advantages i ) It produacuteces superior quality plantsii)It can be applied to interspecifie hybridsiii) It is useful to grow seedless plants bull Limitations i) It cannot be used for all plantsii)It is not easy to handleQ9 Write advantages of vegetative propagationi) It is a quick and easy method ofproducing new plantsii) This method need less time to matureiii) The new plants are exact copies of the parentiv) it is extremly useful for growing seedlessplants like banana grapes etc Q10 Write some disadvantages of vegetativepropagationi) Dišeases present in the parent plant gettransferred to all in new plantsii) Overcrowding of new plants causes competition for sunlight water and nutrients which affects growth of plantsplant

Physics Chapter 2 Physical Quatites and Measurements

Here We Will Do Some QuestionsRelated To Chapter 2

Select the correct alternative A block of wood of density 08gcm-3 has a volume of 60cm3 The mass of the block is

1 608 g

2 75 g

3 48 g

4 0013 g

Solution 348 g

The density of aluminium is 27g and that of brass The correct statement is

1 Equal masses of aluminium and brass have equal volumes

2 The mass of a certain volume of brass is more than the mass of an equal volume of aluminium

3 The volume of a certain mass of brass is more than the volume of an equal mass of aluminium

4 Equal volumes of aluminium and brass have equal masses

Solution 2 The mass of a certain volume of brass is more than the mass of an equal volume of aluminium

MATHEMATICS Ch 6Sets

Exercise 6(C)1 Find all the subset of each the following sets(i) A = 57 (iii) C = x xisin W x le 2(iv) p p is a letter in the word lsquopoorrsquo

Solution (i) All the subsets of A are ϕ 5 7 57

(iii) All the subsets of C are ϕ 0 1 2 01 02 12 012

(iv) All the subsets are ϕ p o r po or por

4 Given the universal set = -7-3-105689 find (i) A = x xlt2 (ii) B = x -4ltxlt6 Solution

(i) A = -7-3-10(ii) B = -3-105

5 Given the universal set = x xisin N and xlt20 find

(i) A = x x = 3p pisin N (iii) C = x x is divisible by 4 Solution

(i) 369121518 (iii) 481216

6 Find the proper subset of x x2-9x-10 = 0 Solution

ϕ 10 -1

Working x2-9x-10 = 0 rArr x2-(10-1)x-10 = 0

rArr x2-10x+x-10 = 0 rArrx(x-10)+1(x-10) = 0

rArr (x+1) (x-10) = 0

11 Let M = letters of the word REAL and N = letters of the word LARE Write sets M and N in roster form and then state whether (i) M sube N is true (ii) N sube M is true (iii) M = N is true

Solution M = real and N = lareSo (i) Yes (ii) Yes (iii) Yes

English 2 Twelfth Night ndash Shakespeare

A noble man named Orsino in the kingdom of Illyria is deeply in love with a lady called lady Olivia She is in mourning for her dead brother so she will not even think about marriage At this time a sea storm causes a terrible shipwreck and a young lady called Viola is swept onto the shore She thinks that her twin brother Sebastian is drowned A sea captain tells her about Orsino and his love for Olivia Viola wishes to work in Oliviarsquos home but feels she will not be employed So she dresses as a man calls herself Cesario and gets work at the house of OrsinoViola (now Ceasario) is much liked by Orsino and becomes his page She falls in love with Orsino Orsino sends Ceasario to deliver messages to Olivia Olivia herself falls for the beautiful young Ceasario believing Viola to be a man

2 Answer the following questionsa Why does Orsino ask the musicians to play onOrsino asks the musicians to play on because music feeds his desire He calls upon the musicians to play music so that his hunger for love could be replenished with an excess of musicb What does Valentine tell about OliviaWe learn from Valentine that Olivia is in mourning for her brother she wears a veil and has vowed that no one will see her face for another seven yearsand she refuses to marry anyone until thenc From the exchange between Orsino and Valentine what do you think their relationship isValentine is one of orsinod attendants He was sent to Olivia as a messenger of love but was not allowed to speak to here Who is Olivia mourning for and whyOlivia is mourning for her dead brother

Homework Q fHistory and Civics

Growth of Nationalism

Important dates to remember1769-Napoleon born on 15thAugust1789-Fall of Bastille on 14th July and the beginning of the French revolution declaration of the rights of Man on 26thAugust1793-King Louis XVI executed on January 211764-The Sugar Act passed1765-The Stamp act passed1774-The first congress of Philadelphia1776-The declaration of American Independence of on 4th July1777-Defeat of the British at Saratoga1781-Surrender of lord Cornwallis at Yorktown1783-The treaty of Versailles1804-Napoleon becomes the emperor1813-Battle of Leipzig or Battle of nations in which Napoleon was defeated by the Allies1815-Battle of Waterloo June 18 in which Napoleon was defeated and captured1821-Death of Napoleon in StHelena1860-Abraham Lincoln elected President of the USA1861-The civil war began 1864-Abraham Lincoln elected President of the USA for the second time1865-Slavery abolished in the US

Name the following- The queen of Louis XVI

Marie Antoinette The three philosophers of France

VoltaireMontesquieuJean Jacques Rousseau

The British general whose surrender brought the war in America to an endLord Cornwallis

The first president of the USAGeorge Washington (1732-1799)

The first southern state to secede from the unionSouth Carolina

The author of the book lsquoUncle Toms CabinHarriet Beecher Stowe

Homework-Learn

Class IXSubject Topic Summary Execution

Economics

Types of economies Today I am going to share you the concept of economic growth and economic development Few questions will be given from the previous study material dated 942020

Meaning of economic growthAnswer) The term economic growth generally means anincrease in national income or per capita output or income over time It indicates towards quantitative growth of a country

Meaning of economic developmentAnswer) Economic development is defined

as a process whereby the real per capita income of a country increases over time along with fall in poverty ratio unemployment and income inequality etc

Distinguish between economic growth and economic development

Basis Economic growth

Economic development

Scope It has narrow scope as it refer only to rise in per capita income

It has wide concept since it includes qualitative changes as well

Concerned matter

It is concerned with the rise in income

It is concerned with not only rise in income but also reduction of poverty income inequality and unemployment

Focus Economic growth does not focus on economic development

Economic development focus on economic growth plus qualitative changes

Distinguish between capitalist economy and socialist economy

Ownership

Motive

Tool

Means of production are owned and managed by private people

Self interest and profit earning is the main motive

Price mechanism is a main tool to solve the economic problems

Means of production are owned and managed by the government

Social welfare is the main motive

Economic planning by the government is the main tool to solve the economic problem

Competition

Distribution of income

There exist large competition among buyers and sellers

There is existence of large inequalities of income

There is no such competition

There exist less inequalities of income

Math Topic ndash AlgebraChapter -Factorisation

Study item Factorising by taking out common factorSome solved sums from exercise 41

1) (i) 8xy3 + 12x2y2

= HCF of 8xy3 and 12x2y2 is 4xy2

= 4xy2(2y + 3x )

4) (ii) 28p2q2r ndash 42pq2r2

= HCF of 28p2q2r and 42pq2r2 is 14pq2r = 14pq2r (2p - 3r )5) (ii) 14mn + 22m - 62p=HCF of 14mn 22m and 62p is 2= 2(7mn + 11m - 31p)7) (ii) 3a(x2 + y2) + 6b (x2 + y2) = HCF of 3a(x2 + y2) and 6b(x2 + y2 ) is (x2 + y2)= ( x2+ y2 )(3a + 6b )9) (ii) x(x2 + y2 ndash z2 ) + y(-x2ndashy2 + z2 ) ndash z(x2+ y2 ndash z2 )= x(x2 + y2 -z2) ndash y-(x2 + y2 -z2) -z(x2 + y2 ndash z2)=x(x2 + y2-z2) -y( x2 + y2-z2) ndash z (x2 + y2 -z2)= (x2+ y2 ndash z2)(x ndash y ndash z )

Commercial Studies

Introduction to Accounting and Book-keeping

Today I am going to share you the meaning of Accounting and Book-keeping and its related terms bullAccounting bullBook Keeping bullAccountsbullTypes Of Accounts bullAccounting Cycle

bull Meaning of accounting

Ans )Accounting is the art and science of recording classifying and summarising monetary transactions

bull Meaning of Book-keeping

Ans) Bookkeeping is the art of recording business transactions with the view of having a permanent record of them and showing their effect on wealth

bull Meaning of account

Ans) The term account means a record of business transactions concern a particular person of firm asset or income or expense It is a summarised record of all transactions which take place in an accounting year

bull Types of accountsPersonal accounts ndash Personal accounts relating

to person and Organisation are known

as personal accounts Example Ramrsquos Account ABC amp Co Account etc

Real account - The accounts related to tangible and intangible assets are called real accountsExample Cash Account Furniture Account etc

Nominal account- Accounts related to expenses losses incomes and gains are known as nominal accountsExample Wages Account Salary Account Discount Account etc

bull Accounting cycle Accounting cycle refers to a complete sequence of accounting activities It begins with recording of transactions and ends with the preparation of a balance sheet

English 1 Transformation of sentences

Sentences A sentence is a group of words which makes complete sense

a Assertive sentencesb Imperative sentencesc Interrogative

sentencesd Exclamatory sentences

Sentences can be changed from one grammatical form to another without changing the meaning of the sentence This is known as transformation of sentences

Exercise 6Rewrite the following sentences according to the instructions given below without changing their meanings

1 As soon as he saw the beer he jumped into the river ( Begin No sooner)

2 None but brave deserve the fair (Begin the bravehellip)

3 This box is too heavy for me to lift ( Use so hellip That instead of too)

4 No one other than a king can live like James Luxurious ( Begin only James)

5 Oh for the wings of a dove (Begin I wishhellip)

BENGALI(2ND LANGUAGE)

ldquo বঙগভমির পরমি ldquo াইকেল ধসদন দতত

পব13পোসঠ আসোলিচত ৩ পরবোস দৈদসবর বস ীবতোরো Pলিদ স এ লেদ -আকো সত-োলি লেদ তোস - ক) বকতো লেক কোর লেো লেকো কলিবতোর অং ) কোর পরলিত বকতোর এই উলিকত গ) এ লেদ আকো সত বসত কী বলিঝসয়স4 ীবতোরো বসত কী লেবোঝ ঘ ) আসোচয অংসর তোৎপP13 কী

উ -ক ) বকতো স কলিব মোইসক ম3দ দতত

Types of AccountPersonal AccountReal AccountNominal AccountBalance Sheet (opening)

কলিব মোইসক ম3দ দসততর রলিচত বঙগভলিমর পরলিত কলিবতোর অং ) কলিব বঙগী অ13োৎ লেদমোতোর পরলিত কলিবর এই উলিকত গ ) এ লেদ আকো বসত কলিবর মোব লেদী রপ আকো লেক লেবোঝোসো সয়স4 আকো লেসক লেPম তোরো স পসর লেতমলি ীব লেদ রপ আকো লেসক পরো রপ তোরো স পরসত পোসর এই মভোবোর কোই কলিব বসস4 ঘ ) পরবো Pোতরোয় Pলিদ কলিবর লেদ আকো লেসক ীব তোরো রপ পরো স পসর তোসত কলিব লিবনদমোতর দঃলিত কোর মতয লিবসর সবোভোলিবক পলিরলিত এবং মোষ মরী তোই পরবোস Pলিদ তা োর মতয য় তবও কলিব লিবচলিত সব ো কোর পলিবীসত লেকউ অমর য় লিক4ই অকষয় য় দীর লেPম লিচরপরবোমো লেতমলি মোসষর ীবও চমোতোই ীব - সতবধতোই মতয ীব দীসত মোষ লিতয পরবোমো তবও লেPব মোষ আপ কতকসম13র মো3যসম মোসষর মস লিসসদর সথো কসর লিসত পোসর তোরো লিচরভোসবর সয় মোসষর মস লিবরো কসর তোসদর মস3য লেকউ পGভসত লিবী সয় গোসও মোসষর মস তোরো লিতযপলিত লিতযবলিনদত

Hindi 2ndlang

काकीी(लिसयारामशरणगपत)

इस कहानी म क न यह बतान का परयास निकया ह निक बचच अपनी मा स निकतना परम करत ह शयाम अबोध बाक ह वह अपनी मा क मरन क बाद उसन अपनी मा क लिए बहत रोया बाद म उस पता चा निक उसकी मा राम क घर ची गई ह आकाश म उडती हई पतग दकर उस हष हआ निक पतग क दवारा वह अपनी मा को नीच उतारगा इसक लिए वह अपनी निपता की जब स दो बार सवा रपया निनकाकर पतग और दो मोटी सी मन वाी अपन भाई स काकी एक कागज पर लिवा कर पतग म लिशव का दिदयानिनकाकर पतग और दो मोटी सी मन वाी अपन भाई स काकी एक कागज पर लिवा कर पतग म लिचपका दिदयाभोा और शयाम कोठरी म रससी बाधनी रह थ तभी उसक निपता करोध म आकर उन स पछ निक कया उनकी जब स रपया निनकाा हभोा डर क मार बताया निक शयाम इस पतग क दवारा अपनी काकी को राम क यहा स उतारना चाहता हनिवशशवर(शयाम क निपता)न फटी पतग उठाकर दी तो उस पर काकी लिा थावह हत बजि होकर वही ड रह गएउनहोन सोचा निक मन अपन पतर को मारा जोनिक अनजान और निनदष थावह अपनी मा कोनिकतना पयार करता ह

helliphellipContinue to next

Computer Application

Java Programming Prog 1Write a java program to input two numbers from user and display the sum or product of them as per user choice Use switch case statementSolve public class sum_product public static void main(String args[]) Scanner sc=new Scanner(Systemin) int abc Systemoutprintln(ldquoEnter two numbersrdquo) a=scnextInt() b=scnextInt() Systemoutprintln(ldquoPress 1 for sum or 2 for productrdquo)

c=scnextInt() switch(c) case 1 Systemoutprintln(ldquoThe sum will be =rdquo+(a+b)) break case 2 Systemoutprintln(ldquoThe product will be =rdquo+(ab)) break default Systemoutprintln(ldquoWrong Inputrdquo) Home Work - Practice in your computer using bluej

Subject Eng Literature (The Merchant of Venice ndash William Shakespeare)Topic Act I Scene 2 Lines 92 to 126 (End of scene) Date 13th April 2020 (5th Period)

[Students should read the original play and also the paraphrase given in the school prescribed textbook]Summary Questions amp Answers

o After Portia has expressed her opinion about the suitors Nerissa informs that she need not bother about any one of them as they have decided to quit Belmont at the earliest opportunity because they do not believe in trying their luck by the caskets which is the only way of winning Portia

o Nerissa then enquires of Portiarsquos opinion about Bassanio who once visited her in the company of the Marquis of Montferrat and says that she had never come across such an ideal love deserving the fairest lady for his bride

o Portia seems to remember Bassanio quite correctly and says that she agrees with Nerissa At this moment a servant informs Portia that the Prince of Morocco has arrived to try his luck by the caskets

o Portia tells Nerissa that if she could welcome this new suitor as gladly as she says farewell to the previous ones she would be glad of his arrival However if he happens to have the virtues of a saint but the black complexion of a devil she would prefer to have him for religious consolation rather than as a husband

(1) NERISSA You need not fear lady (Line 97-103)

the having any of these lords they have acquainted me with their determinations

which is indeed to return to their home and to

trouble you with no more suit unless you may be wonby some other sort than your fathers imposition depending on the caskets

PORTIA If I live to be as old as Sibylla I will die as chaste asDiana unless I be obtained by the manner of my fatherswill I am glad this parcel of wooers are so reasonablefor there is not one among them but I dote on his veryabsence and I pray God grant them a fair departure

(a) Elucidate the idea expressed in the first speech of the above dialogue

In the first speech Nerissa assures Portia that she need not have any fear of being compelled to marry anyone of the suitors who had lately come to Belmont She informs her that they have all decided to return to their respective countries(b) Illuminate the meaning of the phrase ldquoyour fatherrsquos imposition depending on the casketsrdquo

Nerissa means that the suitors of Portia do not find the conditions imposed by the will of her father to their liking They are too hard for them These conditions are that in the event of a suitor failing to choose the right casket (i) he should never disclose to anybody which casket he chose (ii) he can never marry and (iii) he should take his departure immediately(c) Explain the meaning of the term lsquoSibyllarsquo

lsquoSibyllarsquo is the name given by Romans and Greeks to a prophetess inspired by some deity usually the sun-god Apollo She had a very long life The god Apollo granted her as many years of life as she could hold grains of sand in her hand(d) Elucidate the meaning of the term lsquoDianarsquo

lsquoDianarsquo is the goddess of hunting She is also regarded as a symbol of virginity because she never fell in love and never

married(e) Explain the meaning of the first two lines of Portiarsquos speech

Portia says that even if she is to live for centuries like Sibylla she would not marry except in accordance to her fatherrsquos will She asserts that she would not mind remaining unmarried and untouched by a man like Diana the virgin the goddess of hunting unless a man is able to win her by passing the test laid down by her father

Class XSubject Topic Summary Execution

Hindi 2nd

Langबड घर की बटी( मशी परमचद)

lsquoबड घर की बीटीrsquo कहानी का उददशय मधयम वग की घर समसया को सझा कर सगदिठत परिरवार म मिम जकर परम स रहन का सदश दना ह घर म शानित tानिपत करन की जिजममदारी नारी की होती ह यदिद नारी समझदार ह उसम धय और परिरवार क परनित परम ह तो कोई भी घटना परिरवार को निवघदिटत नही कर सकती या कहानी परिरवार को सगदिठत करत हए परम सौहाद स एक रदसर की भावनाओ को समझ करउनका सहयोग करत हए जीवन यापन करन की पररणा दती ह मशीपरमचदर जी न इस कहानी म सयकत परिरवार का परनितनिनमिधतव निकया ह यह कहानी बनी माधव सिसह जो गौरी पर क जमीदार क उनक दो पतरो की हशरी कठ ा निबहारीशरीकात का निववाह एकजमीदार घरान की पतरी आनदी स हआ थाआनदी न द को ससरा क वातावरण म ढालिया थाएक दिदन आनदी का अपन दवर ा निबहारी स झगडा हो जाता ह दोनो भाई एक रदसर स अग होन की कोलिशश करत हसभी बह आनदी न अपन मधर वयवहार स ा निबहारी को घर छोडकर जान स रोक लिया| इस पर बनी माधव सिसह न कहा निक बड घर की बटी ऐसी ही होती ह जो निबगडा काम बना ती ह अतः शीषक साथक ह बड घर की बटी आनदी ह

helliphelliphelliphellipContinue to nextBiology Topic ndash Chp-1

CellWelcome to new session 2020-21Today we will start with Chpter 1 cell CELL

Protoplasm+Cellmembrane Or Cell wall

Cytoplasm+Neucleus

Cytoplasmic+ CytoplasmicOrganelles Inclutions(mitochondria (food Golgi bodies pigments)Ribosome)

What is cellbull Cell is the structural and functional unit of living organismbull According to number of cells organisms areUnicellular - Amoeba bacteria Multicellular - Rose Mango Tiger HumanSmallest cell -bacteria Longest cell - Nerve cellLargest cell - Ostrich egg cellCells are of different size and shapes according to their functionsQ2Write chief functions of following cellorganelles

Q3What is tonoplastVacuoles covered by a covering called tonoplast

Bengali(2Nd

Language)

ফ ফটক ো ফটক (কলিবতো ) ভোষ মসোপো3 gtPোয়

একটি লেমসয়র ীবস লেপরম লিকভোসব ফসট ওসঠ তো লেদলিসয়স4 কলিব লেপরম Pই য় লেই ময়ই বনত কোস পলিরত য় ফ লেফোটো বো োসফোটো লেটো ব2 কো য় লেমসয়সদর ব gtয13 লেপরসমর 4লিব ফসট উসঠস4 এই কলিবতোয় লেপরম মোষসক মত gtযর মস লেফস লিদসয় পরকষস বাোচোসোর gtয োত বো2োয় কলিবতোয় লেমসয়টির পসব13র দঃসর কো বো সও লেমসয়টি লেই পসর পলিক সত চোয়ো োরী ীবসর কোস4 পরম লেPৌবস লেপরমসক পোবোর পরব ইচছো োকসও তো পসর লেলিতবোচকতোয় পলিরত য় কলিব ভোষ মসোপো3 যোয় লেP ক লেপরসমর

কলিবতোয় ব gtযবহত লিবসষ লিক4 সvর অ13 লেদওয়ো ১) রসবোো= লেP লিবলিভনন রকম ডোকসত পোসর২) ো= পোর ৩) ঠলি = লেচোসর বZ৪)আই বস2ো=অলিববোলিত৫)শইসয় = োলিয়ত কসর৬)োতপাোচ= লিবলিভনন পরকোর৭)দ2োম = v কসর বZ কসর লেদওয়ো৮)লেরলিং =লেোোর দৈতরী লেব2ো৯) বনত= একঋত১০) পাোর = বসকরো2

Organelles Functions

1 Endoplasmic reticulum

2 Mitochondria

3Golgibodies

4 Ribosome5Lysosome

6Plastids

7 Centrosome

i) Supportive framework for the cellii) Synthesis and transpost of proteinsRelease of energy in the form of ATPi) Synthesis and secretion of enzymes hormoneii) Formation of vacuoles lysosomei) Protein Synthesisi) Intracellular digestionii) Destroy foreign substancei )Leucoplast - stores starchii)chloroplast - trap solar energyiii) Chromoplast - imparts colour toflowers amp fruitsi) Initiates and requlates cell division

কলিবতো তোর অ13সক ভোষোয় পরকোো কসর ঘলিরসয় ব যকত কসরস4 লেপরসমর ফতো আর লিবফতো লেক গোঢ় কসর লেদোসো কলিব ভোষ মসোপো3 যোসয়র অলিভবসর অ যলিদক

Economics

Factors of Production

Welcome to the new sessionToday we are going to start the first chapter of Class XThe name of the chapter is Factors of productionBy the name I hope you all can recall a glimpse of what you have learnt in the second chapter of Class IX

NowProduction is the process of creating the various goods and services which are consumed by the people of the country to satisfy their wants

Thus it is the process in which some materials are transformed from one form to another to create utility and value in goods

For example utility can be created by changing the form of a commodity ie

Making of table out of wood by a carpenter for his customer here the wood is getting transformed into table creating utility for his customer and he can also command a price for it

On the other hand Housewives perform very

useful activities at home which create utility but their domestic activities are not included in production because they have no money value

So we can also say that Production denotes two things firstly creation of utility and secondly creation of value

Production is not complete unless it reaches the consumer

An increase in production will increase the economic welfare of the consumers and hence the aim is to raise the production level of the country

Again production of a good or service is only possible if certain resources or

Questions

1 What do you mean by production

Answer Production means the creation of goods and services for the purpose of selling in the market

In fact production involves the transformation of inputs into outputs

Hence production denotes two thingsCreation of utility and creation of valueUtility and value can be created by changing the form by changing the place by changing the time and by rendering services

Example Transformation of raw

materials into finish goods such as potter creates utility by converting mud into utensils assembling of small parts to make bigger machinery

Production also includes services such as distribution and marketing

2 What are the factors of production

Answer Factors of Production refers to the resources and inputs needed for producing goods and servicesThese inputs can be classified as

Land Labour

Capital Enterprise

Land Land is defined to include not only the surface of the earth but also all other free gifts of nature(for example mineral resources forest resources and indeed anything that helps us to carry out the production of goods and services but is provided by

inputs are used together in right proportion

A resource or an input which helps in the process of production to obtain an output is called FACTOR OF PRODUCTION

These factors of production can broadly be categorized into four parts 1LAND 2LABOUR3CAPITAL4ENTERPRISE (ORGANISATION)or Entrepreneur

The above factors are all interdependent on each other and they play a major role in production process

FACTORS OF PRODUCTION

LANDCAPITAL

LABOUR ENTREPRENEUR

nature free of cost)LabourLabour refers to the human efforts that need to be combined with other factors of production for creating an output

CapitalAll man ndash made means of production is called capita example machineries which help in further production Money when used for starting any business for purchasing raw materials machinery tools etc it is regarded as capitalCapital also includes physical capital like factories machineriestoolsbuildingsequipments etcEnterpriseThe task of bearing risks is called enterprise and the person who bears these risks of business is called the entrepreneurThus an entrepreneur is one who organises production takes important decisions regarding production hires and purchases factors of production and bears the risk and uncertainty involved in productionOrganisation refers to the services of an entrepreneur who controls organises and undertakes all risks One who plans organises and manages a business enterprise is an organiser

Physics Chapter 1 Force

Force is an external agent capable of changing the state of rest or motion of a particular body It has a magnitude and a direction The direction towards which the force is applied is known as the direction of the force and the application of force is the point where force is applied The Force can be measured using a spring balance The SI unit of force is Newton (N)

Question 1

State the condition when on applying a force the body has

(a) the translational motion

(b) The rotational motion

Solutions

(a) Translational motion is produced when the body is free to move

(b) Rotational motion is produced when the body is pivoted at a point

Question 2

Define moment of force and state its SI unit

Solutions

The moment of force is equal to the product of the magnitude of the force and the perpendicular distance of the line of action of force from the axis

of rotation

The SI unit of moment of force is Newton times meter

= Newton meter (Nm)

Commercial Studies

Stake holders In this topic you will be come to know about the meaning and concept of stakeholders

How stakeholders are different from shareholders

Questions1 What do you mean by the term stake holdersAnswer) The term stake holders have developed from the words which mean an interest or expected benefit Stakeholders mean all those individuals groups and Institutions which have a state (interest) in the functioning and performance of a commercial organisation or a business enterprise2 What do you mean by share holdersAnswer) The person and Groups who own the shares of the joint stock company by providing capital to the company are called shareholders Shareholders are the internal stakeholders shareholders are one out of several stake holders3 How are shareholders different from stakeholdersAnswer)i) The term shareholders is related to only joint stock company whereas stakeholders are related with all business organisationsii) Stakeholders maybe any individual having financial stake in business organisation whereas a shareholders are those individuals who are holding shares in the company4) How are shareholders different from creditorsAnswer) i) Shareholders are internal stakeholders while creditors are external stakeholdersii) Shareholders invest in the capital of the company whereas creditors give loan to the companyiii) Shareholders are the members of the company with voting rights but creditors are not the members of the company

English 1 Transformation of sentences

Sentences A sentence is a group of words which makes complete sense

e Assertive sentencesf Imperative sentencesg Interrogative sentencesh Exclamatory sentences

Sentences can be changed from one grammatical form to another without changing the meaning of the sentence This is known as transformation of sentences

Exercise 1 Change the following affirmative sentences into Negative sentences

a He is a good manHe is not a bad man

b Ram loves SitaRam is not without love for Sita

c Only he stood first in the classNone but he stood first in the class

d Ankit was wiser than he

He was not so wise as Ankite He did it

He did not fail to do itf As soon as I reached college the

bell rangNo sooner did I reach college than the bell rang

g He finished everythingHe left nothing unfinished

h It always pours when it rainsIt never rains but it pours

Math Topic Commercial MathematicsChapter ndash Goods and services Tax

What is GSTAns It is a abbreviated term of Goods and Service Text which is an indirect tax levied on the sale of goods and rendering servicesSome terms related to GSTDelar Any person who buys goods or services For resale is known as a delar A delar Can be a firm or a companyIntra-state sales Sales of goods and services within the same state or same union territory are called intra- state salesInter-state sales Sales of goods and services outside the state or union territory are called Inter-state sales4) Input GST GST is paid by dealers on purchase of goods and services are called input GST5) Output GST GST is collected from customers on sale of goods and services are called output GST6) Types of GST There are three taxes applicable under GST(i) Central Goods and Services Tax (CGST)(ii) State Goods and Services Tax (SGST) or Union Territory Goods and Services Tax (UTGST) Both these taxes are levied on intra-state sales Here GST is divided equally among central and state governments(iii) Integrated Goods and Services Tax (IGST) IGST is levied on inter- state sales It is also levied on import of goods and services into India and export of goods and services from India

Subject Eng Literature (The Merchant of Venice ndash William Shakespeare)Topic Act III Scene 4 Lines 1 to 44 (Portia hellip To wish it back on you fare you well Jessica)[Students should read the original play and also the paraphrase given in the school prescribed textbook]

Summary Questions amp AnswersIn this scene we suddenly find a new element in the character of Portia We have already seen her possessed of every graceful womanly quality but now she shows that she is capable of rapid decision and determined action She shows this by her sudden resolve to hasten to Venice with a daring scheme for the rescue of Antonio This is an important scene in the dramatic action for it leads up to and renders possible the striking events of the famous trial scene which is one of the greatest striking elements of the play Moreover the fact that all the characters of importance are now assembled together in Venice makes the union of the main plot and the secondary story complete

(1) LORENZO Madam although I speak it in your presence(Line 1-9)

You have a noble and a true conceit

Of god-like amity which appears most strongly

In bearing thus the absence of your lordBut if you knew to whom you show this honourHow true a gentleman you send reliefHow dear a lover of my lord your husbandI know you would be prouder of the workThan customary bounty can enforce you

(a) Where is Lorenzo Why is he here To whom is he referring as lsquoMadamrsquo

Lorenzo is at Portiarsquos residence He had met Salerio on the way and Salerio had begged him to come along with him to

o In this scene Portia Nerissa Lorenzo Jessica and Balthazar appear

o Portia requests Lorenzo and Jessica to be in charge of her house during her absence from Belmont because she and Nerissa have decided to spend the days in meditation and also in visiting the holy places in the neighbourhood of Belmont She has already instructed her people to acknowledge both Lorenzo and Jessica as master and mistress of house during her absence Lorenzo and Jessica gladly agree to look after the house of Portia

handover the letter from Antonio to Bassanio The letter carried the bad news about Antoniorsquos arrest for non-payment of loan taken from Shylock Hence Salerio might have preferred company to break this bad news to Bassanio He is referring to Portia as Madam(b) What does Portia say on hearing the above extract

Portia says that she has never regretted doing good to others Friends who spend a lot of time together and really are there for each other have many traits in common As Antonio is Bassaniorsquos best friend saving him is like saving Bassanio who is like her own soul She asks Lorenzo to take care of management of the house till Bassanio is back(c) What does Portia send with Bassanio and why

On hearing about Antoniorsquos troubles on account of Bassanio her husband Portia immediately sends him with enough gold to repay the debt many times over to Venice to help Antonio out of his misfortune

(2) Lorenzo Madam with all my heart (Line 36-40)

I shall obey you in all fair commands

Portia My people do already know my mindAnd will acknowledge you and JessicaIn place of Lord Bassanio and myselfSo fare you well till we shall meet again

(a) Where are Lorenzo and Portia at this time What lsquofair commandsrsquo are given to Lorenzo

Lorenzo and Portia are at Belmont during this scenePortia reveals to Lorenzo that she has sworn to contemplate in prayer at a monastery around two miles away until her husband returns from Venice She tells him that Nerissa would accompany her and asks him to manage the house with Jessica till things are settled In response Lorenzo tells her that he would be obliged to do whatever she asks him to do(b) Where is Portia actually going and why

Portia tells Lorenzo that she would live a life of contemplation and pray at a monastery which is two miles away from her place In reality Portia plans to go to Venice in disguise with Nerissa and argue the case in defense of Antonio She is very sure that her plan would succeed

ClassXI (ScienceHumanitiesCommerce)Subject Topic Summary Execution

Computer Science

(APC)

Ch ndash 1 Numbers

(Numbers in different bases and

their Arithmatical operations)

Number System In computers Number System is defined as a writing system to represent the numbers in different ways ie we are using different symbols and notations to represent numbers There are four ways we can represent the number ndash Binary Decimal Octal and Hexadecimal

Decimal Number SystemThis number system consist 10 digits These are 0 1 2 3 4 5 6 7 8 amp 9

Binary Number SystemThis number system has only two digits these are 0 and 1 Here 0 stands for off while 1 stands for on

Octal Number SystemThis number system has 8 digits these are 0 1 2 3 4 5 6 amp 7

Hexadecimal Number SystemThis number system has 16 digits these are 0 1 2 3 4 5 6 7 8 9 A B C D E F Here the value of the alphabets are as follows A=10 B=11 C=12 D=13 E=14 F=15

Rules for conversion decimal number to Binary1 Divide the decimal number by 22 If the number will not divide equally by 2 then round down the answer to the nearest whole number (integer)3 Keep a note of the remainder it should be either 0 or 14 Keep repeating the above steps dividing each answer by 2 until you reach zero5 Write out all the remainders from bottom to top This is your binary solution

For example Lets convert 32 to binary 2 32 2 16 - 0 2 8 - 0 2 4 - 0 2 2 - 0 2 1 - 0 0 - 1

The binary equivalent of 3210 is 1000002

Try the follwing youself1 2410

2 4810

3 1210

History GROWTH OF NATIONALISM

The second half of the 19th century witnessed growth of political consciousness and a sense of Nationalism among the IndiansThere were various factors for growth of Indian Nationalism- As a result various political associations were formed in different provinces by the educated Indians Surendranath Banerjee organized a meeting of National conference at Calcutta Ultimately the National Congress was founded in Bombay in 1885This body became the vanguard of Indian struggle for freedom The congress leaders were known as moderates because they followed a policy of prayer and petition A large number of Indian leaders had experienced in political agitation The Political situation of England was also changed Moreover increasing revolutionary activities in Maharashtra Punjab and Bengal became serious concern to the British Government In this

QUESTION1 What do you mean by Nationalism ANSWER 1 Nationalism is defined as loyalty and devotion to own nation especially a sense of national consciousnessQUESTION 2 What are the causes of nationalism ANSWER 2 There were various factors for growth of nationalism

1 Spread of western education2 The progress of vernacular press and

patriotic literature3 The economic exploitation of our

country by the colonial rulers4 International affairs

QUESTION 3 Who organized National conference in Calcutta in 1883 ANSWER 3 Surendranath BanerjeeQUESTION 4 When did Indian National Congress formANSWER 4 Indian National Congress was formed in 1885 in BombayQUESTION 5 Who were ModeratesANSWER 5 The Early Nationalists were also known as Moderates Their emergence marked

background Lord Curzon became Viceroy in India He had no respect for the Indian National Congress

the beginning of the organized national movement in India They believed in British justice and were loyal to them They followed a policy of prayer and petition They demanded constitutional reforms of our country Impotant Moderate leaders were Pherozshah Mehta Dadabhai Naorozi and Surendranath Banerjee etcQUESTION 6 What do you know about Extremism in Indian National movementANSWER 6 In the beginning of 20th century a new class of national leaders emerged in India which was different from the moderate groups They started more aggressive movement against the British empire The goal of extremists was ldquoswarajrdquo Important extremist leaders were Bal Gangadhar Tilak Lala Lajpat Rai Bipin Chandra Pal etcQUESTION 7 Mention the places which were the main centres of Revolutionary movementANSWER 7 Maharashtra Bengal and Punjab

Physics

Chapter Dimensional Analysis

(Summary)

The dimensions of a physical quantity are the powers to which the fundamental units are raised in order to obtain the derived unit of that quantit

The physical quantites lengthmasstime are represented by [L] [M] [T] resp let they are raised to powers ( dimesions) abc resp then any physical quantity can be represented by [ La Mb Tc ] Examples

1 Area area = L x B = [L] x [L] = [M0 L2 T0 ]

2 Density density = massvolume = [M][L3] = [ M L-3]

3 Velocity velocity = distancetime = [L][T] = [LT-1]HW Try to find out dimension of acceleration Acceleration = velocity timeNB One can find the SI Units Using Dimension Analysis Such as for area we have [L2] so its SI unit is m2

Biology Topic ndash Chp-1 The living world

Today we will start the first chapter the living world Here we discuss about the characteristics of living organism and what are the difference between them and nonliving substances We also discuss about the contribution of different Scientists

There are over 500000 species of plants andover a million species of animal are present on earth Some 15000 new species were discovered every yearQ1 What is a living organismbull A living organism is primarily physico -chemical material that demonstrate a high degree of complexity is capable of selfRegulation possesses a metabolism and perpetuates itself through timeQ2 What are the differences between livingand non-livingsi) Compared with non-living living organisms

have more complex organised structure and their use of energy is more controlled amp efficientii) Living things reproduce their own kind by forming new cells which contains copies of their genesiii) Each organism has some degree of homeostasisie it is able to make adjustments so that internal environment remains constantQ3 Write contributions of following Scientists i) Aristotle - One of the first theories in Biology places all living things in a hiearchieii) AV Leeuwenhoek - was the first to observe living single celled organisms under microscopeii) Carolus Linnaeus - developed the binary system for naming of organisms and classificationiii) Geregor Johann Mendel ndash discoverbasic principles of inheritanceHomework i) C Darwin ii)Schleiden

Math Trigonometric functions

1 Overviewi) Trigonometry The word lsquotrigonometryrsquo is derived from the Greek words lsquotrigonrsquo and lsquometronrsquo which means measuring the sides of a triangle An angle is the amount of rotation of a revolving line with respect to a fixed line Usually we follow two types of conventions for measuring angles ie a) Sexagesimal system b) Circular system In Sexagesimal system the unit of measurement is Degree In Circular system the unit of measurement is Radian ii) Relation between degree and radianThe ratio of circumference of a circle to its diameter is always a constant This constant ratio is a number denoted by π which is taken approximately as 227The relationship between degree amp radian measurements is as follows2 right angles = 180deg= π radians1radian = 180degπ=57deg16(approx) 1deg=π180 radianiii) Length of an arc of a circleIf an arc of length s subtends an angle θ radians at the center of a circle of radius r then s=rθiv) Area of a sector of a circleA sector is like a pizza slice of the

Q) Express the following angles in radiana) 45deg b) 40deg3730Ans a) We have 180deg=π radiansi e 45deg= πtimes45180 radian = π4 radiansb) 40deg3730= 40deg37+3060 minute= 40deg 37 +12 minute= 40deg+ 752 minute=40 + 75(2times60) degree=3258 degreeNow 180deg=π radianie 3258 degree= (πtimes325) (180times8) radians = 65π288 radiansQ) A circle has a radius of r=12 meters What is the length of an arc traced out by a 60deg angle in the center of the circleAns In this problem we know both the central angle (60deg) and the radius of the circle (12) All we have to do is plug those values into our equation and we get

s = 2π(12)(60360)s = 24π6s = 4πSo the length of an arc traced out by a 60deg angle in a circle with a radius of 12 meters equals 4π meters asymp 1257 metersQ) Find the area of the sector with a central angle 30deg and a radius of 9cmAns GivenRadius r = 9 cmAngle θ = 30degArea of the sector = θ360degtimesπr2

= 30360degtimes227times92=2121cm2

circle It consists of a region bounded by two radii and an arc lying between the radiiThe area of a sector is a fraction of the area of the circle This area is proportional to the central angle In other words the bigger the central angle the larger is the area of the sectorArea of Sector = θ2 times r2 (when θ is in radians)

Area of Sector = θ times π360 times r2 (when θ is in degrees)

COMMERCE

CLASSIFICTION OF HUMAN ACTIVITIES-ECONOMIC AND NON-ECONOMIC

Welcome to the new sessiontoday we are going to start the first chapter of Class XI The name of the chapter that we are going to start is

lsquoClassification of Human Activities ndasheconomic and non-economicrsquo

Now let us start the chapter by considering human beings and the activities they perform throughout the day

Human activities means all those activities that human beings undertake to satisfy their wants

Human wants on the other hand are the desire of human beings for goods (vegetables fruits rice etc) and services (services of doctors teachers lawyers etc) that they require to live

Now these human activities continue throughout life as human wants are unending unlimited and recurring as human beings desire for better living throughout their lives

Now human activities can be classified into two categories

Human activities

Economic activities Non-economic activities

Economic activities are

Questions1 What are human activities

Answer Human activities mean all those activities that human beings undertake to satisfy their wants

Example A man working in an office

A boy playing in the garden

2What are the characteristics of human activitiesAnswer the characteristics of human activities are as follows

Human activities are undertaken by men women and children and these activities involve human efforts

Human activities are undertaken to satisfy human wants which are unlimited

Human activities continue throughout life

Human activities are performed for both earning money and personal satisfaction

3What is economic activitiesGive example

Answer Economic activities are undertaken by human beings with the object of earning money acquiring wealth and thereby satisfying human wantsExample

Selling of goods by a shop keeper to his customer

A clinic run by a doctor Service of a teacher in school or college

undertaken by human beings with the object of earning money and acquiring wealth

These activities result in the production of economic goods and services

Example Human activities(ie working in factories officesshops) which produce direct economic benefits

Non-economic activities are inspired by human sentiments and emotions such as love for the family desire to help the poor and love for the country

Thus these human activities (eg praying playing sleeping) produce no direct economic benefits and they are also not related to earning money and acquiring wealth

4 What are the characteristics of economic activities

Answer The characteristics of economic activities are as follows

Economic motiveEconomic activities are undertaken to earn money and acquire wealth

ProductiveEconomic activities involve productiondistribution and exchange of goods and services to create wealth

Economic growthEconomic activities determine the level of economic development of a country and standard of living of its citizens

Socially desirableEconomic activities are socially desirable for society

Economic resourcesEconomic activities make use of all the economic resources such landlabourcapital etc

5 What do you mean by non-economic activitiesExampleAnswerNon-economic activities are inspired by human sentiments and emotions such as love for the family desire to help the poor and love for the countryThese activities are not undertaken for monetary gain but for onersquos satisfaction and happinessExample

a mother looks after her children

a student donates blood8 Differentiate between Economic activities and Non-economic activities

Economic activities

Non-economic activities

1to earn living and acquiring wealth2Result can be measured in terms of money

3ExampleBusinessprofession and employment

1 to obtain some satisfaction

2Result cannot be measured in terms of money

3ExampleFamily-orientedreligious socialCultural and national

BUSINESS STUDIES

BUSINESS ENVIRONMENT

Welcome to the new sessionToday we are going to start the first chapter and the name of the chapter is Business Environment

In todayrsquos world every business enterprise is a part of the society It exists and operates in association with various groups in society such as customers suppliers competitors banks and financial institutions government agencies trade unions media and so on All these groups influence the functioning of business in one way or the other They constitute the environment of businessConcept of Business Environment

The term lsquobusiness environmentrsquo refers to the sum total of all individuals institutions and other forces that lie outside a business enterprise but that may influence its functioning and performance

The main features of business environment

Totality of External forces General and Specific forces Interrelatedness Complexity Dynamic Uncertainty Relativity

The Interrelation between business and its environment

The business enterprise is an open system It continuously interacts with its environment It takes inputs

Prepare the following questions from todayrsquos assignment

1 What do you mean by business environment

The term lsquobusiness environmentrsquo means the aggregate of all forces factors and institutions which are external to and beyond the control of an individual business enterprise but they may influence its functioning and performance Business environment is the macro framework within which a business firm a micro unit operates It consists of several interrelated and interacting elements

2 Explain the main features of business environment in brief

Totality of External forces-Business environment is the sum total of all things external to a business environment

General and Specific forces-It includes both the forces general forces are the economic social political legal and technological conditions which indirectly influence all business enterprise Specific forces are the investors customers competitors and suppliers which influence individual enterprise directly

Interrelatedness-Different elements of environment are interrelated for an example growing awareness for health care has increased the demand for health foods

Complexity- Business environment id

(such as raw materials capital labour energy and so on) from its environment transforms them into goods and services and sends them back to the environment

Fig 1 Business Environment Relationship

complex in nature as the elements keep on changing example economic technological and other forces changes in demand for a product and service

Dynamic-Business environment is not static it keeps on changing

Uncertainty- Itrsquos very difficult to predict future events such as technology and fashion which occur fast and frequently

Economics Basic Economic ConceptsSub topic

Microeconomics and

Macroeconomics

Welcome to the new sessiontoday we are going to start the first chapter of Class XI The name of the chapter that we are going to start is Basic Economic concepts

Now Economics covers the study of human activities Human activities are those activities which are performed by humans to satisfy their wants

Thus Human wants are unlimited and therefore economic activities such as production exchange and consumption are needed in order to satisfy those wants

The study of economics is divided largely in two parts which areMicroeconomics and Macroeconomics

SUBJECT- MATTER OF ECONOMICS

MICROECONOMICS MACROECONOMICS

Questions1Who has coined the words micro and macro economics

Answer Ranger Frisch coined the words lsquomicrorsquo and lsquomacrorsquo in 1933 to denote the two branches of economic theory namely microeconomics and macroeconomics

2What is microeconomicsAnswer It is the study of behaviour of individual decision ndash making unit such as consumers firms etc

3 What is macroeconomicsAnswer Macroeonomics is the study of overall economic phenomena like employment national income etc

4 What is the importance of microeconomicsAnswer

Microeconomics helps in formulating economic policies which enhance productive efficiency and results in greater social welfare

It helps the government in formulating correct price policies

It explains the working of a capitalistic economy where individual units(producers and consumers ) are free to take their own decision

Micro means a small part in

microeconomics we do not study the whole economy Hence we study an individual consumer and his or her choices and a producer and his or her profit maximizing decisions in the market Thus it does not mirror what happens in the economy as a whole

Macroeconomics on the other hand studies the economy as a whole It is concerned with aggregate and depicts the entire picture of the economyMacroeconomics deals with the national income aggregate investment aggregate consumption etc

Features of Microeconomics It deals with small

parts of the country Hence it looks at

individual consumers firms and industries

It deals with individual income consumption and savings

It studies the determination of price of any product or factors of production

It deals with the working of market via the price mechanism which is nothing but the determination of price and quantity of a commodity by the forces of demand and supply

Features of Macroeconomics

It deals with the study of the economy as a whole

It is concerned with

5 Give a limitation of microeconomics Microeconomics fails to explain the

functioning of an economy as a whole It cannot explain unemployment illiteracy and other problems prevailing in the country

6 What is the importance of macroeconomics It gives overall view of the growing

complexities of an economic system It provides the basic and logical

framework for formulating appropriate macroeconomic policies (eg for inflation poverty etc )to direct and regulate economy towards desirable goals

7What is the limitation of macroeconomics It ignores structural changes in an

individual unit of the aggregate

8 Differentiate between Microeconomics and Macroeconomics

Microeconomics Macroeconomics

the study of aggregates

National income aggregate savings and aggregate investments are major concepts dealt within macroeconomics style

It studies the determination of general price levels

It investigates into the problem of unemployment and the achievement of employment

It studies the aspect of decision making at the aggregate and national levels

It includes all growth theories whether related to developed or developing economies it also includes the study of economic systems and the working of the economy under different systems

Note Both Micro and macro economics are complementary and should be fully utilized for proper understanding of an economy

1It studies economic aspect of an individual unit2It deals with individual incomeConsumption and savings

3 It facilitates determination of price of any product or factors of production

4 Itrsquos scope is narrow and restricted to individual unit

1It studies the economy as a whole

2It deals with the national income aggregate consumption and aggregate savings3 It facilitates determination of general price level in an economy

4 Itrsquos scope is wide as it deals with economic units on the national level

ACCOUNTS

Introduction to Accounting and Book-keeping

Today I am going to share you the meaning of Accounting and Book-keeping and its related terms bullAccounting bullBook Keeping bullAccountsbullTypes Of Accounts bullAccounting Cycle

bull Meaning of accounting

Ans ) Accounting is the art and science of recording classifying and summarising monetary transactions

bull Meaning of Book-keeping

Ans) Bookkeeping is the art of recording business transactions with the view of having a permanent record of them and showing their effect on wealth

bull Meaning of account

Ans) The term account means a record of

business transactions concern a particular person of firm asset or income or expense It is a summarised record of all transactions which take place in an accounting year

bull Types of accountsPersonal accounts ndash Personal accounts relating

to person and Organisation are known as personal accounts Example Ramrsquos Account ABC amp Co Account etc

Real account - The accounts related to tangible and intangible assets are called real accounts Example Cash Account Furniture Account etc

Nominal account- Accounts related to expenses losses incomes and gains are known as nominal accounts Example Wages Account Salary Account Discount Account etc

bull Accounting cycle Accounting cycle refers to a complete sequence of accounting activities It begins with recording of transactions and ends with the preparation of a balance sheet

Chemistry TopicAtomic Structure

Thomsonrsquos atomic modelThomson (1898) was the first to propose the model of an atomHe proposed that an atom can be regarded as a uniform sphere of positive electricity in which requisite number of electrons are embedded evently to neutralize the positive chargeThis is just like plums embedded in a pudding or seeds evently distributed in red spongy mass of a watermelonThis model of atom is known as ldquoPlum-Pudding modelrdquo or

Q1)What is the fundamental constituents of atomAns Electron Proton and neutrons are the fundamental constituents of atomQ2)What is the value of fundamental unit of electricityAnsThe charge carried by one electron is sad to be the fundamental unit of electricityIts magnitude is 48times10-10esuOr 1602times10-19C Q3)Name the element containing no neutronAnsOrdinary hydrogen atom or protium 1H1

Types of AccountPersonal AccountReal AccountNominal AccountBalance Sheet (opening)

ldquowatermelon modelrdquoThis model could explain the electrical neutrality of an atom but failed to explain the result of scattering experiment carried out by Rutherford in 1911So it was rejected ultimately

Q4)Why is an electron called universal particleAns Itrsquos mass and Charge are independent of its source

EVS Chapter 1 ndash Modes of Existence

Modes of existence When one speaks normally about the mode of existence of some group or individual one refers to their customs their mode of being their ethology their habitat in some way their feeling for a placeDifferent modes of exixtence are ndash

1 Hunting ndashGathering2 Pastoral3 Agricultural4 Industrial

1 Hunting and gathering Hunting and gathering mode of existence is characterized by obtaining food from hunting wild animals including fishing and gathering wild plants From their earliest days the hunter-gatherer diet included various grasses tubers fruits seeds and nuts Lacking the means to kill larger animals they procured meat from smaller game or through scavenging

Societies that rely primarily or exclusively on hunting wild animals fishing and gathering wild fruits berries nuts and vegetables to support their diet are called hunting and gathering societies

At least this used to be practice of human beings before agriculture is invented As their brains evolved hominids developed more intricate knowledge of edible plant life and growth cycles

Q) Write the features of Hunting ndash gathering societiesAns - There are five basic characteristics of hunting and gathering societies

i The primary institution is the family which decides how food is to be shared and how children are to be socialized and which provides for the protection of its members

ii They tend to be small with fewer than fifty members

iii They tend to be nomadic moving to new areas when the current food supply in a given area has been exhausted

iv Members display a high level of interdependence

v Labor division is based on sex men hunt and women gather

Political Science

Introduction to political science

Political science occasionally called politology is a social science which deals with systems of governance and the analysis of political activities political thoughts associated constitutions and political behaviorThe study of political science involves the study of both the

Answer the following questions-1 What is political science

Political science occasionally called politology is a social science which deals with systems of governance and the analysis of political activities political thoughts associated constitutions and political behavior

2 Short notes-

traditional and modern theories of politicsTraditionalClassical political sciencepolitical theory-Traditional political science is the study of politics before Second World War The methodology to study Politics was traditional (legal formaletc) the definition of politics traditional (Politics begins and end with state)area of study (constitution state machinery)was traditionalModern Political scienceModern political theory-Modern Political Theory critically examines the contemporary state of political theory making an assessment of the achievement and limitations of the Behavioural Revolution in its totality and reviews objectively the major paradigms and conceptual frameworks adopted by the disciplineContemporary attempts at the development of an integrated political theory involving the use of both traditional and modern concepts approaches and theories-Around late 1960s several political scientists realized the importance of both the traditional political theory and modern Political theory They began building an integrated theory of politics involving a systematic mixture of traditional and modern studies of politics It was held that the study of a complex and vast field like politics needs both traditional as well as

Classical political theory Modern Political theory

Homework-Learn

modern concepts and approaches for studying itrsquos all aspects

Subject Eng Literature (The Tempest ndash William Shakespeare) Topic Act I Scene 1 Lines 1 to 32 (Line 32 ndash Gonzalo hellip If he be not born to be hanged our case is miserable) Date 13th April 2020 (3rd Period)

[Students should read the original play and also the paraphrase given in the school prescribed textbook]Summary Questions amp Answers

[SUMMARY OF THE ENTIRE SCENE]

o The play starts with the scene of a severe storm at sea Alonso (King of Naples) Sebastian (Alonsorsquos brother) Ferdinand (Alonsorsquos son) Gonzalo Antonio (the usurping Duke of Milan) are in a ship in the midst of the storm

o The mariners are trying their best to control the vessel from running aground and are totally following the orders of their Master the Boatswain They have scant success

o The mariners become extremely unhappy and annoyed when most of the passengers arrive on the deck thereby hampering their effort to save the ship There is serious confrontation between them and the passengers who are part of the Kingrsquos entourage

o The mariners could not save the ship

SUMMING-UP

(i) Vivid description of the scene which gives a realistic description of terror and confusion of a tropical storm

(ii) Shows Shakespearersquos accuracy of knowledge in describing the naval operations and also matters of seamanship

(iii) The opening scene justifies the title ndash The Tempest

UNANSWERED QUESTIONS

(i) The King always travels with his entire fleet including his soldiers Where

(1) GONZALO Nay good be patient (Line 15-26)BOATSWAIN When the sea is Hence What cares these

roarers for the name of the king To cabin silence Trouble us not

GONZALO Good yet remember whom thou has aboardBOATSWAIN None that I more love than myself You are a

councillor if you can command these elements to silence and work

the peace of the present we will not hand a rope more use your authority If you cannot give thanks you have

lived so long and make yourself ready in your cabin for the mischance of the hour if it so hap [To the Mariners]

Cheerly good hearts [To Gonzalo] Out of our way I say

(a) To whom is the boatswain speaking What does he mean by lsquoNone that I more love than myselfrsquo

The Boatswain is speaking to Gonzalo the honest old councilor of the Duke of MilanBy using the words ndash lsquoNone that I love more than I love myselfrsquo means that for the Boatswain nobody is dearer to him than his own life

(b) What were the conditions that made the boatswain react in this way

The Boatswain reacts in this way because the storm is at sea and Alonso King of Naples Sebastian his brother Ferdinand his son Gonzalo Antonio the usurping Duke of Milan on board are in distress and in panic Thus they have rushed to the deck interrupting the work of the mariners

(c) What hope does Gonzalo take from the attitude of the boatswain

The insolent and authoritative attitude of Boatswain makes Gonzalo feel comforted He tells that there are no signs that the Boatswain will be drowned But his facial appearance and attitude shows that he is destined to die on land by hanging which in effect means that all on board will be saved Otherwise all the persons on board are doomed

(d) How can they lsquomake yourself ready in your cabinrsquo For what were they asked to make ready themselves

In order to make themselves ready in their cabin the

were the other ships

(ii) Why was the ship in that area Where was it coming from or going where

(iii) The ship broke apart What happened to those who were in the ship

passengers on board must prepare for death which they will possibly soon have to meetThey can retire to their cabins and offer prayers to the Almighty to save them from drowning

(e) What does the boatswain say when he is asked to be patient What does he order to the royal party

When the boatswain is asked to be patient and remain calm he says that he will be patient only when the storm will be over and the sea will be calm but as long as the storm blows and there is danger to the ship he cannot think of being patient He orders the royal party to go to the cabin and leave the mariners to their work

(2) GONZALO I have great comfort from this fellow (Line 27-36)

Methinks he hath no drowning mark upon him his complexion is perfect

gallows Stand fast good Fate to his hanging Make the rope of his destiny our cable for our own doth little advantage If he be not born to be hanged our case is miserable

(a) Why does Gonzalo regard the Boatswain in the midst of danger

In the midst of danger Gonzalo regards the boatswain because he feels that the Boatswain is a source of comfort and is bent upon to do his work sincerely which in this case is saving the ship and its passengers from the severest of raging storm

(b) What reasons does Gonzalo give when he says that none in the ship will die of drowning

Gonzalo is almost sure that none in the ship will die by drowning His says that there is no mark on the face of the boatswain that indicates that he will die by drowning On the other hand the lines on his face are strong indications that he will be hanged to death Therefore there shall be no danger of the shiprsquos sinking

(c) Explain the following ldquoStand fast good Fate to his hanging Make the rope of his destiny our cable for our own doth little advantage If he be not born to be hanged our case is miserablerdquo

The stated lines mean that if the will of destiny is to be carried out then the ship will not get wrecked and all the passengers will be saved The safety of the passengers therefore depends upon the will of fate being carried out in the case of the boatswain If however the boatswain is not to die by hanging then the passengers are also very unsafe because in that case the ship is likely to sink

(d) What order does the Boatswain give to the sailors

when he re-enters What does he say about the crying of the fellows inside the cabin

The boatswain orders the sailors to bring the topmast lower and bring the ship close to a stationary position with the help of the main sail He says that the fellows inside the cabin are moaning and crying in their distress louder than his voice and louder even than the roaring of the storm

Class XII (ScienceCommerceHumanities) Subject Topic Summary Execution

Computer Science

PropositionalLogic

Propositional logic is a procedure to provide reasoning through statementProposition A ststement that results in True or False is said to be proposition There are two types of propositionSimple proposition amp compound propositionSimple proposioton A simple proposition is one that is not a part of any other proposition Such sentential form of proposition is symbolized with english letters in short For example Ram is a claver student (TrueFalse)Where do you live (Not in True or False)Grapes are sweet (TrueFalse)It rains today (TrueFalse)Here we can see some statements anwer would be true or false but some staements answer can not give in terms of true or false Thus the sentences which can be answered in true or false are known as simple propositionAssigning propositon to a variableThe general syntax to assign propostion to a variable is as followsVariable = Simple propositonFor example A=Ram is a clever studentB= Grapes are sweetC= it rains todayCompound proposition

helliphellipto be continued in next classhelliphellipMath Relation Relation If A and B are two non-empty sets

then a relation R from A to B is a subset of AxB If R A x B and (a b) R then we say that a sube isinis related to b by the relation R written as aRbeg Let A be the set of students of class XII and B be the set of students of class XI Then some of the examples of relation from A to B arei) (a b) AXB a is brother of bisinii) (a b) AXB age of a is more than age of isinb Types of relation In this section we would like to study different types of relations We know that a relation in a set A is a subset of A times A Thus the empty set φ and A times A are two extreme relations For illustration consider a relation R in the set A = 1 2 3 4 given by R = (a b) a ndash b = 10 This is the empty set as no pair (a b) satisfies the condition a ndash b = 10 Similarly R = (a b) | a ndash b | ge 0 is the whole primeset A times A as all pairs (a b) in A times A satisfy | a ndash

Example 1 Let A be the set of all students of a boys school Show that the relation R in A given by R = (a b) a is sister of b is the empty relation and R = (a b) the primedifference between heights of a and b is less than 3 meters is the universal relationSolution Since the school is boys school no student of the school can be sister of any student of the school Hence R = φ showing that R is the empty relation It is also obvious that the difference between heights of any two students of the school has to be less than 3 meters This shows that R = A times A is primethe universal relation Example 2 Show that the relation R in the set 1 2 3 given by R = (1 1) (2 2) (3 3) (1 2) (2 3) is reflexive

b | ge 0 These two extreme examples lead us to the following definitionsDefinition 1 A relation R in a set A is called empty relation if no element of A isrelated to any element of A ie R = φ A times AsubDefinition 2 A relation R in a set A is called universal relation if each element of A is related to every element of A ie R = A times A Both the empty relation and the universal relation are some times called trivial relation Definition 3 A relation R in a set A is called(i) reflexive if (a a) R for every a Aisin isin(ii) symmetric if (a1 a2) R implies that (aisin 2a1)

R for all aisin 1 a2 Aisin(iii) transitive if (a1 a2) R and (aisin 2 a3) R isinimplies that (a1 a3) R for all aisin 1 a2 a3 AisinDefinition 4 A relation R in a set A is said to be an equivalence relation if R is reflexive symmetric and transitive

but neither symmetric nor transitiveSolution R is reflexive since (1 1) (2 2) and (3 3) lie in R Also R is not symmetric as (1 2) R but (2 1) isin notinR Similarly R is not transitive as (1 2) R and (2 3) R but (1 3) R isin isin notinExample 3 Show that the relation R in the set Z of integers given byR = (a b) 2 divides a ndash b is an equivalence relationSolution R is reflexive as 2 divides (a ndash a) for all a Z isinFurther if (a b) R then 2 divides a isinndash b Therefore 2 divides b ndash a Hence (b a) R which shows that R is isinsymmetric Similarly if (a b) R and (b c) R isin isinthen a ndash b and b ndash c are divisible by 2 Now a ndash c = (a ndash b) + (b ndash c) is even (Why) So (a ndash c) is divisible by 2 This shows that R is transitive Thus R is an equivalence relation in ZExample 4 Let L be the set of all lines in a plane and R be the relation in L defined as R = (L1 L2) L1 is perpendicular to L2 Show that R is symmetric but neither reflexive nor transitiveSolution R is not reflexive as a line L1 can not be perpendicular to itself ie (L1 L1) R notinR is symmetric as (L1 L2) Risin

L1 is perpendicular to L2rArr L2 is perpendicular to L1rArr (L2 L1) RrArr isin

R is not transitive Indeed if L1 is perpendicular to L2 and L2 is perpendicular to L3 then L1 can never be perpendicular to L3 In fact L1 is parallel to L3 ie (L1 L2) R isin(L2 L3) R but (L1 L3) Risin notin

Chemistry Solid state Characteristics if Solids(i)The particles are locked in fixed positions they are unable to change their relative positions and this brings a definite shape and volume of a solid(ii)In a solid the constituent particles are held by strong forces of attractionThe forces of attraction may be bonding or non bonding(iii)The constituent particles in a solid pack together as closely as possibleoccupying most of the available space within the solidThus the empty space in a solid is very smallThis makes a solid highly rigid and nearly incompressibleThis also explains why a solid has high density and exhibits slow diffusionClassification of Solids

Q1)Define Crystalline solids AnsA Solid that has a definite geometrical shape and a sharp melting pointand whose constituent particles (atomsmolecules or ions) are arranged in a long range order of definite pattern extending throughout the solid is called a crystalline solidExNaClQ2)Define Amorphous solids AnsA solid that does not have a definite shape and a sharp melting pointand whose constituent particles (atomsmolecules or ions) are not arranged in a definite pattern is called an amorphoussolid

Crystalline solidsAmorphous solids

ExGlassRubberQ3)Classify Crystalline Solids Crystalline Solids

Physics Coloumbrsquos Law (Summary)

Before Going Into Coloumbrsquos Law We Will First Learn What is Charge Properties of Charge and Always remember that charge is quantized ie a body always have static charge of magnitude equal to some integral multiple of fundamental electronic charge e= 16 x 10- 19 C

Charge is the property of matter that causes it to produce and experience electrical and magnetic effects The study of the electrical charges at rest is called electrostatics When both electrical and magnetic effects are present the interaction between charges is referred to as electromagnetic

There exist two types of charges in nature positive and negative Like charges repel and unlike charges attract each other

The type of charge on an electron is negative The charge of a proton is the same as that of an electron but with a positive sign In an atom the number of electrons and the number of protons are equal The atom is therefore electrically neutral If one or more electrons are added to it it becomes negatively charged and is designated as negative ion However if one or more electrons are removed from an atom it becomes positively charged and is called a positive ion

The excess or deficiency of electrons in a body gives the concept of charge If there is an excess of electrons in a body it is negatively charged And if there is deficiency of electrons the body becomes positively charged Whenever addition or removal of electrons takes places the body acquires a charge

The SI Unit of charge is coulomb (C) In SI units the current is a fundamental quantity having a unit of ampere (A) The unit of charge is defined in terms of the unit of current Thus one coulomb is the charge transferred in one second across the section of a wire carrying a

Ionic SolidsMetallicSolids

Molecular Solids

current of one ampere

As q = It we have1 C = (1 A) (1 s)

The dimensions of charge are [A T]

Properties of Charge

(1) Quantization of Charge Electric charge can have only discrete values rather than any value That is charge is quantized The smallest discrete value of charge that can exist in nature is the charge on an electron given as

e = plusmn 16 x 10- 19 C

This is the charge attained by an electron and a protonA charge q must be an integral multiple of this basic unit That is

Q = plusmn ne where n = 1 2 hellip

Charge on a body can never be (frac12)e (23)e or 57e etcWhen we rub a glass rod with silk some electrons are transferred from the rod to the silk The rod becomes positively charged The silk becomes negatively charged The coulomb is a very large amount of charge A typical charge acquired by a rubbed body is 10 - 8 C

Biology Reproduction in organisms

Welcome to this new session 2020-21Today in this first chapter we mainly discuss about reproduction types needs and life span of some organismsWe also discuss about difference between sexual and asexual reproduction

Q1 What is reproductionReproduction is defined as a biological processin which an organism gives rise to young onessimilar to itselfQ2 What are the needs of reproductionbulli) Reproduction maintain life on earthii) It enables the continuity of the species generation after generationiii) It creates genetic variation among populationsQ3 Define Life span and write some orgnisms life spanbull Life span is the period from birth to

the natural death of an organism- OrganismsLife span1 Butterfly 1 - 2 weeks2 Fruit fly 30 days3Dog 10-13 years4 Rose5-7 years5 Tortoise100-150 years6 Banyan Tree -200 - 250 yearsQ4 Reproduction is of two types in case ofanimals but in case of plants vegetative propagation is also present

Asexual Reproduction Sexual Reproductioni) Always uniparentalii) Gametes are not involvediii) Only mitotic division involvediv) Somatic cells of parents are involvedv) Offsprings are genetically similar to the parents

i) Usually biparentalii) Gametes are involvediii) Meiosis occurs during gametogenesis Mitosis occurs after fertilisationiv) Germ cells of the parents are involvedv) offsprings are genetically different from the parents

COMMERCE BUSINESS ENVIRONMENT

Welcome to the new sessiontoday we are going to start the first chapter of Class XII The name of the chapter is Business Environment

Already many of you have got some idea about the word business environment form the first chapter of business studies in class XI

In todayrsquos world every business enterprise is a part of the society It exists and operates in association with various groups in society such as customers suppliers competitors banks and financial institutions government agencies trade unions media and so on All these groups influence the functioning of business in one way or the other They constitute the environment of businessConcept of Business Environment

The term lsquobusiness environmentrsquo refers to the sum total of all individuals institutions and other forces that lie outside a business enterprise but that may influence its functioning and performance

The main features of business environment Totality of External forces General and Specific forces Interrelatedness Complexity Dynamic Uncertainty

Prepare the following questions from todayrsquos assignment

2 What do you mean by business environment

The term lsquobusiness environmentrsquo means the aggregate of all forces factors and institutions which are external to and beyond the control of an individual business enterprise but they may influence its functioning and performance Business environment is the macro framework within which a business firm a micro unit operates It consists of several interrelated and interacting elements

2 Explain the main features of business environment in brief

Totality of External forces-Business environment is the sum total of all things external to a business environment

General and Specific forces-It

Relativity

The Interrelation between business and its environment

The business enterprise is an open system It continuously interacts with its environment It takes inputs (such as raw materials capital labour energy and so on) from its environment transforms them into goods and services and sends them back to the environment

Fig 1 Business Environment Relationship

includes both the forces general forces are the economic social political legal and technological conditions which indirectly influence all business enterprise Specific forces are the investors customers competitors and suppliers which influence individual enterprise directly

Interrelatedness-Different elements of environment are interrelated for an example growing awareness for health care has increased the demand for health foods

Complexity- Business environment id complex in nature as the elements keep on changing example economic technological and other forces changes in demand for a product and service

Dynamic-Business environment is not static it keeps on changing

Uncertainty- Itrsquos very difficult to predict future events such as technology and fashion which occur fast and frequently

Business Studies

Human Resources Management

Human resource of an organisation are the aggregate of knowledge skills attitudes of people working in it

The management system which deals with human resources is called human resource management

Features of HRMbullComprehensive functionbullPeople-oriented

Question1) What do you mean by human

resource management Answer) Human resource management may be defined as that field of Management which has to do with planning organising and controlling the functions of procuring developing maintaining and utilising the labour force

bullAction oriented bullPervasive function bullContinuous function

2) Explain the features of HRM in brief

Answer)bullHuman Resource Management is concerned with managing people at work bull Human Resource Management is concerned with employees which bring people and organisations together so that the goals of each are met bullHuman resource management considered every employees as an individual and also promote their satisfaction and growth bull Human resource management is inherent in all organisations and at all levelsbullManagement of human resources are ongoing on never ending process which requires a constant alertness and Awareness of human relations

3) ldquoHR function is said to be pervasiverdquowhy

Answer) Human resource management is required in all organisations whether it is private or government organisations armed forces sports organisations etc It permeatsall the functional areas like production marketing finance research etc This from this feature of human resource management it can be said that it is pervasive in nature

Economics Demand Q1DEFINITION OF DEMANDIn economics demand is the quantity of a good that consumers are willing and able to purchase at various prices during a given period of timeQ2DEMAND CURVEIn economics a demand curve is a graph depicting the relationship between the price of a certain commodity and the quantity of that commodity that is demanded at that pricQ3LAW OF DEMANDIn microeconomics the law of demand states that conditional on all else being equal as the price of a good increases quantity demanded decreases conversely as the price of a good decreases quantity demanded increasesQ4ASSUMPTION of LAW OF DEMAND(i)No change in price of related commodities(ii) No change in income of the consumer(iii) No change in taste and preferences customs habit and fashion of the consumer( No expectation regarding future change in priceQ5MARKET DEMAND SCHEDULEIn economics a market demand schedule is a tabulation of the quantity of a good that all consumers in a market will purchase at a

given price At any given price the corresponding value on the demand schedule is the sum of all consumersrsquo quantities demanded at that priceQ6INDIVIDUAL DEMAND SCHEDULEIndividual demand schedule refers to a tabular statement showing various quantities of a commodity that a consumer is willing to buy at various levels of price during a given period of timeQ7 FACTORS AFFECTING INDIVIDUAL DEMAND FOR A COMMODITY

The factors that influence a consumerrsquos decision to purchase a commodity are also known as determinants of demand The following factors affect the individual demand for a commodity1 price of the commodity2 price of related goods3 income of buyer of the commodity4 tastes and preferences of the buyer1 Price of the CommodityYou must have observed that when price of a commodity falls you tend to buy more of it and when its price rises you tend to buy less of it when all other factors remain constant (lsquoother things remaining the samersquo) In other words other things remaining the same there is an inverse relationship between the price of a commodity and its quantity demanded by its buyers This statement is in accordance with law of demand which you will study in the later part of this lesson Price of a commodity and its quantity demanded by its buyers are inversely related only when lsquoother things remain the samersquo So lsquoother things remaining the samersquo is an assumption when we study the effect of changes in the price of a commodity on its quantity demanded2 Price of Related goodsA consumer may demand a particular good But while buying that good heshe also asks the price of its related goods Related goods can be of two types-(i) Substitute goods(ii) Complementary goods While purchasing a good prices of its substitutes and complements do affect its quantity purchased(i) Price of Substitute Goods Substitute goods are those goods which can easily be used in place of one another for satisfaction of a particular want like tea and coffee An increase in price of substitute good leads to an increase in demand for the given commodity and a decrease in price of substitute good leads to a decrease in demand for the given commodity It means demand for a given commodity is directly affected by change in price of substitute goods For example if price of coffee increases the demand for tea will rise as tea will become relatively cheaper in comparison to coffee(ii) Price of Complementary goods Complementary goods are those goods which are used together to satisfy a particular want like car and petrol An increase in the price of complementary goods leads to a decrease in demand for the given commodity and a decrease in the price of complementary goods leads to an increase in demand for the given commodity For example if price of petrol falls then the demand for cars will increase as it will be relatively cheaper to use both the goods together So demand for a given commodity is inversely affected by change in price of complementary goods3 Income of the Buyer of CommodityDemand for a commodity is also affected by income of its buyer However the effect of change in income on demand depends on the nature of the commodity under consideration In case of some goods like full cream milk fine quality of rice (Basmati rice) etc demand for these commodities increases when income of the buyer increases and

demand for these commodities decreases when income of the buyer decreases Such goods whose demand increases with the increase in income of the buyer are called normal goods But there are some goods like coarse rice toned milk etc whose demand decreases when income of buyer increases and their demand increases when income of the buyer decreases Such goods whose demand decreases with the increase in income of the buyer are called inferior goods Suppose a consumer buys 10 Kgs of rice whose price is ` 25 per Kg He cannot afford to buy better quality of rice because the price of such rice is ` 50 per Kg The consumer is spending ` 250 per month on the purchase of rice Now if income of the consumer increases and he can afford ` 350 on purchase of 10 Kg of rice Now he can afford to buy some quantity of rice say 6 Kgs whose price is ` 25 per Kg and may buy 4 Kgs of rice whose price is ` 50 per Kg Thus he will buy 10 Kgs of rice by spending ` 350 per month Therefore we may conclude that demand for normal goods is directly related to the income of the buyer but demand for inferior goods is inversely related to the income of the buyer4 Tastes and Preferences of the BuyerThe demand for a commodity is also affected by the tastes and preferences of the buyers They include change in fashion customs habits etc Those commodities are preferred by the consumers which are in fashion So demand for those commodities rises which are in fashion On the other hand if a commodity goes out of the fashion its demand falls because no consumer will like to buy it(5) Number of Buyers in the Market(Population)Increase in population raises the market demand whereas decrease in population reduces the market demand for a commodity Not only the size of population but its composition like age (ratio of males females children and old people in population) also affects the demand for a commodity It is because of needs of children young old male and female population differs(6) Distribution of Income and WealthIf the distribution of income and wealth is more in favour of the rich demand for the commodities preferred by the rich such as comforts and luxuries is likely to be higher On the other hand if the distribution of income and wealth is more in favour of poor demand for commodities preferred by the poor such as necessities will be more(7) Season and Weather ConditionsThis is generally observed that the demand for woolens increases during winter whereas demand for ice creams and cold drinks increases during summer Similarly market demand for umbrellas rain coats increases during rainy seasonQ8 REASONS FOR OPERATION OF LAW OF DEMAND WHY DEMAND CURVE SLOPES DOWNWARDNow we will try to explain why does a consumer purchase more quantity of a commodity at a lower price and less of it at a higher price or why does the law of demand operate ie why does the demand curve slope downwards from left to right The main reasons for operation of law of demand are1 Law of Diminishing Marginal UtilityAs you have studied earlier law of diminishing marginal utility states that as we consume more and more units of a commodity the utility derived from each successive unit goes on decreasing The consumer will be ready to pay more for those units which provide him more utility and less for those which provide him less utility It implies that he will purchase more only when the price of the commodity falls2 Income Effect

When price of a commodity falls purchasing power or real income of the consumer increases which enables him to purchase more quantity of the commodity with the same money income Let us take an example Suppose you buy 4 ice creams when price of each ice cream is ` 25 If price of ice creams falls to ` 20 then with same money income you can buy 5 ice creams now3 Substitution EffectWhen price of a commodity falls it becomes comparatively cheaper as compared to its substitutes (although price of substitutes has not been changed) This will lead to rise in demand for the given commodity For example if coke and Pepsi both are sold at ` 10 each and price of coke falls Now coke has become relatively cheaper and will be substituted for Pepsi It will lead to rise in demand for coke4 Change in Number of BuyersWhen price of a commodity falls some old buyers may demand more of the commodity at the reduced price and some new buyers may also start buying this commodity who were not in a position to buy it earlier due to higher price This will lead to increase in number of buyers when price of the commodity falls As a result demand for the commodity rises when its price falls5 Diverse Uses of a CommoditySome commodities have diverse uses like milk It can be used for drinking for sweet preparation for ice cream preparation etc If price of milk rises its use may be restricted to important purpose only This will lead to reduction in demand for other less important uses When price of milk falls it can be put to other uses also leading to rise n demand for itQ9 EXCEPTIONS TO THE LAW OF DEMANDYou have studied in law of demand that a buyer is willing to buy more quantity of a commodity at a lower price and less of it at a higher price But in certain circumstances a rise in price may lead to rise in demand These circumstances are called Exceptions to the Law of Demand Some important exceptions are1 Giffen GoodsGiffen goods are special type of inferior goods in which negative income effect is stronger than negative substitution effect Giffen goods do not follow law of demand as their demand rises when their price rises Examples of Giffen goods are jowar and bajra etc2 Status Symbol GoodsSome goods are used by rich people as status symbols eg diamonds gold jewellary etc The higher the price the higher will be the demand for these goods When price of such goods falls these goods are no longer looked at as status symbol goods and tehrefore therir demand falls3 NecessitiesCommodities such as medicines salt wheat etc do not follow law of demandbecause we have to purchase them in minimum required quantity whatever their price may be4 Goods Expected to be ScarceWhen the buyers expect a scarcity of a particular good in near future they start buying more and more of that good even if their prices are rising For example during war famines etc people tend to buy more of some goods even at higher prices due to fear of their scarcity in near future

Political Science

Constitution of India-The

Preamble

The preamble-

Preamble-

The preamble is the most precious part of the constitution We the people of India having solemnly resolved to constitute India into a Sovereign Socialist Secular Democratic Republic and to secure to all its citizensA preamble is an introductory and expressionary statement in a document that explains the documents purpose and underlying philosophy When applied to the opening paragraphs of a statute it may recite historical facts pertinent to the subject of the statuteNature and purpose of the constitution-Purpose of the Constitution dictates permanent framework of the government to form a more perfect union to establish justice and ensure peace of thenationconstitution provide principles how the government can run itself following the rules and laws written in the constitution of each state keeps them balanced

Answer the following questions-

1 What is preambleA preamble is an introductory and expressionary statement in a document that explains the documents purpose and underlying philosophy2 What is the nature and

purpose of the constitutionConstitution dictatespermanent framework of the government to form a more perfect union to establish justice and ensure peace of the nation

Homework-Learn

Accounts Compatibilty mode

1MEANING OF PARTNERSHIPPartnership is a form of business organisation where two or more persons join hands to run a business They share the profits and losses according to the agreement amongst them According to the Indian Partnership Act 1932 ldquoPartnership is relation between persons who have agreed to share profits of a business carried on by all or any one of them acting for allrdquo For example one of your friends has passed class XII from National Institute of Open Schooling (NIOS) and wants to start a business Heshe approaches you to join in this venture Heshe wants you to contribute some money and participate in the business activities Both of you if join hands constitute a partnership2CHARACTERISTICS1048698 Agreement A partnership is formed by an agreement The agreement may be either oral or in writing It defines the relationship between the persons who agree to carry on business It may contain the terms of sharing profit and the capital to be invested by each partner etc The written agreement is known as partnership deed1048698 Number of persons There must be at least two persons to form a partnership

The maximum number of partners in a partnership firm can be 50 according toCompanies Act 20131048698 Business The Partnership is formed to carry on business with a purpose of earning profits The business should be lawful Thus if two or more persons agree to carry on unlawful activities it will not be termed as partnership1048698 Sharing Profits The partners agree to share profits in the agreed ratio In caseof loss all the partners have to bear it in the same agreed profit sharing ratio10486981048698Mutual Agency Every partner is an agent of the other partners Every partner can bind the firm and all other partners by hisher acts Each partner will be responsible and liable for the acts of all other partners10486981048698Unlimited liability The liability of each partner except that of a minor is unlimited Their liability extends to their personal assets also If the assets of the firm are insufficient to pay off its debts the partnersrsquo personal property can be used to satisfy the claim of the creditors of the partnership firm10486981048698Management All the partners have a right to mange the business However they may authorize one or more partners to manage the affairs of the business on their behalf10486981048698Transferability of Share No partner can transfer hisher share to any one including hisher family member without the consent of all other partners3PARTNERSHIP DEEDAgreement forms the basis of partnership The written form of the agreement is which a document of partnership is It contains terms and conditions regarding the conduct of the business It also explains relationship between the partners This document is called partnership deed Every firm can frame its own partnership deed in which the rights duties and liabilities of the partners are stated in detail It helps in settling the disputes arising among the partners during the general conduct of business 4CONTENTS OF PARTNERSHIP DEEDThe partnership deed generally contains the following (i) Name and address of the partnership firm(ii) Nature and objectives of the business(iii) Name and address of each partner(iv) Ratio in which profits is to be shared(v) Capital contribution by each partner(vi) Rate of Interest on capital if allowed(vii) Salary or any other remuneration to partners if allowed(viii) Rate of interest on loans and advances by a partner to the firm(ix) Drawings of partners and interest thereon if any(x) Method of valuation of goodwill and revaluation of assets and liabilities on the reconstitution of the partnership ie on the admission retirement or death of a partner(xi) Settlement of disputes by arbitration(xii) Settlement of accounts at the time of retirement or death of a partner5IN ABSENCE OF PARTNERSHIP DEEDThe partnership deed lays down the terms and conditions of partnership in regard to rights duties and obligations of the partners In the absence of partnership deed there may arise a controversy on certain issues like profit sharing ratio interest on

capital interest on drawings interest on loan and salary of the partners In such cases the provisions of the Indian Partnership Act becomes applicableSome of the Issues are(i) Distribution of Profit Partners are entitled to share profits equally(ii) Interest on Capital Interest on capital is not allowed(iii) Interest on Drawings No interest on drawing of the partners is to be charged(iv) Interest on Partnerrsquos Loan A Partner is allowed interest 6 per annum on the amount of loan given to the firm by himher(v) Salary and Commission to Partner A partner is not entitled to anysalary or commission or any other remuneration for managing the business

History TOPIC-TOWARDS INDEPENDENCE AND PARTITION THE LAST PHASE (1935-1947)

SUB TOPIC-IMPORTANT POLITICAL DEVELOPMENTS ndash GROWTH OF SOCIAL IDEAS

Socialism is a political social and economic philosophyLike in other parts of the world the Russian revolution of 1917 served as a great inspiration for revolutionaries in India who at that time were engaged in the struggle for liberation from British ruleSocialist ideas led to the formation of communist party of IndiaJAWAHARLAL NEHRU Among the early Congress leaders Jawaharlal Nehru was very much impressed and influenced by the Socialist ideas He also learnt about the Economic activities of the Soviet Union after the Bolshevic Revolution 1917 He made full use of them in IndiaThe election of Jawaharlal Nehru and Subhas Chandra Bose showed the Left wing tendency within CongressJawaharlal Nehru demanded economic freedom along with political freedom of the people in order to end the exploitation of masses

Nehrus working committee included three socialists leaders The Lucknow session was a landmark in the evolution of socialist ideas of the congressSUBHAS CHANDRA BOSE ndash Subhas Chandra Bose had socialist leaning Both Jawaharlal Nehru and Subhas Chandra Bose were known as leftist Congress men Later on National Congress divided into Leftist and rightist campCONGRESS SOCIALIST Within the Congress some leaders formed the Congress Socialist partyPattavi Sitaramyya Sardar Patel Rajendra Prasad had hostile attitude towards the Congress Socialist partyJawaharlals attitude was hesitant

1 QUESTION ndash Mention name of two Congress leaders who had socialist leaning

1ANSWER ndash Subhas Chandra Bose and Jawaharlal Nehru2QUESTION- In which session of the congress Jawaharlal elaborated his Socialist ideas2 ANSWER ndash Lucknow and Faizpur Session in December 1935 and 19363QUESTION ndash Why Congress was sharply divided into leftist and rightist camp 3ANSWER ndash Subhas Chandra Bosersquos attempt to seek re election for congress presidentship in 1939sharply divided the National Congress into Leftist and Rightist camp4 QUESTION ndash Who was MN Roy 4 ANSWER ndash Manabendra Roy first formed the Communist Party of India outside the country at Tashkent in 19205QUESTION ndash Who formed the Congress Socialist Party within the Congress5 ANSWER ndash Jaya Prakash Narayan Achyut Patwardhan Acharya Narendra Dev Ram Mohan Lohia Aruna Asaf Ali6QUESTION ndash When was the Congress Socialist Party formed What was its object6 ANSWER ndash 1934The Congress Socialist Party sought to work out socialist programme through the Congress They joined hands with the Congress and wanted to carry

Subhas Chandra Bose being expelled from the congress after the Tripuri rift he formed Forward BlockThere were basic differences between the Congress Socialists and the communistsTRADE UNION ACTIVITIES Maximum working class people lived in Bombay and Calcutta The working and living conditions of those workers were very miserable In this situation Shasipada Banerjee NM Lokhande protested against the oppression of the working class peopleThe first Trade Union Madras Labour Union was formed in 1918 by BP WadiaIndustrial strikes took place in Kanpur Calcutta Madras Jamshedpur and Ahmedabad AITUC was formed in Bombay in 1927 The growth of Trade union among the workers was slow because of the fear of the dismissal of the jobIn the mean time the Moderates as well as Communists left AITUC and formed separate organization

on National struggle with the help of workers and peasant class of the society7 QUESTION ndash What was the name of the party founded by Subhas Chandra Bose7 ANSWER- Forward Block8QUESTION ndash Who was Shasipada Banerjee8 ANSWER ndash Shasipada Banerjee was a radical Brahmo He founded a working menrsquos club to protest against exploitation of the British rulers towards the working class of India9 QUESTION ndash What was the weekly published by NM Lokhande9ANSWER- Dinabandhu10 QUESTION ndash Who founded Bombay Mill-Hands Association and in which year10 ANSWER- NM Lokhande in189011 QUESTION- Who was BP WadiaANSWER- BPWadia was the founder of Madras Labour Union in191812 QUESTION- What was the name of the first labour union of India12 ANSWER- Madras Labour Union13 QUESTION Who founded the Majur Mahajan 13 ANSWER GANDHIJI14 QUESTION What was the full form of AITUC When it was formed14 ANSWER All India Trade Union Congressin 192715QUESTION Who formed the Red Trade Union Congress and in which year15ANSWER The Communists formed the Red Trade Union Congress16 QUESTION What do you mean by Socialism16 ANSWER Socialism describes any political and economic theory that says the community rather than individuals should own and manage property and natural resources

Subject Eng Literature (The Tempest ndash William Shakespeare) Topic Act III Scene 3 Lines 1 to 52 (Line 52 ndash Brother my lord the Duke Stand to and do as we) Date 13th April 2020 (4th Period)

[Students should read the original play and also the paraphrase given in the school prescribed textbook]Summary Questions amp Answers

o Alonso Sebastian Antonio Gonzalo Adrian Francisco and others wandered about the island in search of Ferdinand and gets tired and hungry of the toil and at the same time gives up all hope of finding him

o Antonio and Sebastian are happy that Alonso is out of hope and decide to make another attempt on his life that night when being so tired they will be sleeping soundly

o Suddenly a solemn and strange music is heard in the air and several strange shapes enter bringing in a banquet These strange shapes then dance round it with gestures of salutation and then inviting the King to eat they depart

o Seeing this strange scene all are inclined to believe the tales told by travelers that there truly are ldquounicornsrdquo and ldquothe phoenixrsquo thronerdquo

1 ALONSO What harmony is this My good friends hark (L18-27)

GONZALO Marvellous sweet music

[Enter several strange shapes bringing in a banquet

they dance about it with gentle actions of salutation

and inviting the King and his companions to eat they depart]ALONSO Give us kind keepers heavens What were theseSEBASTIAN A living drollery Now I will believe

That there are unicorns that in Arabia

There is one tree the phoenixrsquo throne one phoenix

At this hour reigning thereANTONIO Ill believe both

And what does else want credit come to me

And Ill be sworn rsquotis true Travellers neer did lie

Though fools at home condemn rsquoem

(a) How did Prospero present an amazing spectacle before Alonso and his companions

Using his magic powers Prospero ordered strange shapes to lay a banquet before Alonso and his companions The shapes brought several dishes with tasty eatables in them They placed the dishes on a table before Alonso and his companions Then the strange shapes began to dance gracefully around the banquet While dancing they made gestures inviting them to eat the food Then suddenly the shapes disappeared(b) Who were the guests at the strange banquet Describe the lsquoliving drolleryrsquo

Alonso Sebastian Antonio Gonzalo Adrian and Francisco were the guests at the strange banquet

The term ldquoliving drolleryrdquo refers to live entertainment show In this context when Alonso the King of Naples Sebastian his brother Antonio the treacherous brother of Prospero Gonzalo the kind and loyal councillor to the King Adrian and Francisco came to the island they were hungry and weary in their spirits They heard a solemn and strange music They were shocked to see several strange shapes bringing in a banquet and these shapes danced about it with gentle action of salutation inviting the King and his companions to eat After this Sebastian described this show as lsquoliving drolleryrsquo(c) What is lsquophoenixrsquo What are lsquoUnicornsrdquo

The term lsquophoenixrsquo refers to a mythical Arabian bird which lived alone and perched on a solitary tree After one hundred years it expired in flames and rose again from its own ashes

lsquoUnicornsrsquo refers to the mythological four-footed beasts having horns in the centre of their foreheads When the horns are ground into powder the powder was believed to be

an aphrodisiac(d) How does Sebastian explain the puppet show OR Why does the speaker now believe in unicorns and phoenix

Sebastian finds several strange shapes bringing in the banquet They invite the king and his party for dinner and soon depart He tells that if such a strange sight can be a reality there is nothing incredible in the world and from the present moment he will believe anything He says that it is a strange dumb show enacted not by puppets but by living beings It is stranger than a travellerrsquos tale Seeing such a thing

before his own eyes he will no longer disbelieve the story about unicorns and phoenix(e) How do the other characters present respond to this living drollery

At the sight of the lsquoliving drolleryrsquo like Sebastian Gonzalo and Antonio too acted strangely Antonio told that he too now believes in unicorns and phoenix and anything else that seems to be incredible He too now believes in travellersrsquo tales Gonzalo told that if he would report those happenings in Naples nobody will believe him He considers that those gentle shapes were gentler in manner in comparison to the living beings Alonso was at first sight suspicious and told them that those strange shapes conveyed their meaning in expressive gestures when they seemed to lack speech by their movements and sounds Francisco was amazed at their mysterious disappearance

2 ALONSO Not I

(Line 43-52)GONZALO Faith sir you need not fear When we

were boysWho would believe that there were mountaineers

Dewlapped like bulls whose throats had hanging at rsquoem

Wallets of flesh Or that there were such men

Whose heads stood in their breasts Which now we find

Each putter-out of five for one will bring us

Good warrant ofALONSO I will stand to and feed

Although my lastmdashno matter since I feel

The best is past Brother my lord the Duke

Stand to and do as we

(a) How does Alonso respond at the spectacle of the shapes which were sent to them at the instruction of Prospero

After seeing the strange sight of appearing and disappearing of the shapes sent by Prospero to arrange a banquet for them Alonso says that his surprise at having seen those creatures is infinite and he is fully justified in feeling so much surprise He thinks that their shapes their gestures and the sounds they made were indeed amazing Although they do not possess the gift of speech yet they were able to convey their

thoughts by means of their gestures only

(b) What does Prospero say about the views expressed by Alonso regarding the shapes What does Francisco think about the shapesAfter hearing Alonsorsquos views about the shapes Prospero says that this manrsquos praise of the spirits is rather hasty He means to say that Alonso has shown great haste in reaching the conclusion about the shapes Francisco is amazed to see that those shapes disappeared in a mysterious way(c) What does Sebastian ask Alonso to doSebastian tells Alonso that the shapes having disappeared should not matter to them because they have left the eatables behind He asks Alonso to enjoy eating as they are extremely hungry but the king does not accept his offer of enjoying the dishes(d) How does Gonzalo try to dispel Alonsorsquos fear of those strange shapes What kind of references does he give to AlonsoGonzalo says that those who have travelled abroad have reported seeing even stranger sights than these shapes that Alonso and his companions have beheld Hence there is no reason to feel afraid of these shapes Gonzalo further adds that in his younger days he had heard strange stories from travelers and Alonso might have heard similar stories For instance it was said that there existed a certain race of

human beings who had huge lumps of flesh hanging at their throats and who therefore resembled bulls Then Gonzalo tells about a race of human beings whose heads were located at their breasts Gonzalo says that such stories were not believed by most people in those days but now-a-days these stories have become common(e) Explain the following lsquoEach putter-out of five for onersquoEnglish travellers often insured their trips with London brokers Those that went on foreign travels those days used to deposit a certain amount with some firm or company in London before their departure If the travelers failed to return the money was forfeited by the company with which it had been deposited But this money was repaid five-fold if the travelers returned safe and sound In this way a traveler stood a great chance of recovering the entire cost of his

travels(f) Give the explanatory meanings of the following expressions in the context of the above extract (i) Dewlapped (ii) Wallets of flesh

(iii) Putter-out(i) Dewlapped having big lumps of flesh at the necks(ii) Wallets of flesh large masses of flesh looking like bags(iii) Putter-out to invest money before commencing the travel

  • General methods of preparation of hydrogen
  • Chapter Dimensional Analysis (Summary)
    • Properties of Charge
Page 15:   · Web viewSubject. Topic. Summary. Execution. Hindi. व्याकरण. शरीरके अंगो के नाम लिखिए. 1) आँख 2) नाक 3

Math Number system

Chapter Fraction

Study item Using lsquoofrsquoThe word lsquoofrsquo between any two fractions is to be used as multiplicationExample 57 of 56 = 57 times 56 = 5times8 = 40Study item Using BODMASThe word lsquoBODMASrsquo is the abbreviation formed by taking the initial letters of six operations(i)Bracket (ii) of (iii) Division (iv) Multiplication (v) Addition (vi) SubtractionAccording to BODMAS rule First of all the terms inside Bracket must be simplified then lsquoofrsquo lsquoDivisionrsquo lsquoMultiplicationrsquo lsquoAdditionrsquo lsquosubtractionrsquo

Study item Removal of Brackets

There are four Brackets of algebra in Mathematics In a complex expression four types of brackets are used Order of removing the brackets is first ----- then ( ) then finally [ ]

Class VIIISubject Topic Summary Execution

Chemistry Hydrogen General methods of preparation of hydrogen

By the action of dilute acids on metals

Calcium Reacts readily to form chloride salt and hydrogen

Ca + 2HCl rarr CaCl₂ + H₂uarr

Magnesium

Aluminium

Zinc

React readily to form salt and hydrogen

Mg + 2HCl rarr MgCl₂ + H₂uarr2Al + 6HCl rarr 2AlCl₃ + 3H₂uarrZn + 2HCl rarr ZnCl₂ + H₂uarr

Question 4 ) Give reasons for the following

(a) Hydrogen be used as a fuel

Solution

Hydrogen is used as a fuel because it has a high heat of combustion Some significant fuels are coal gas water gas and liquid hydrogen

(b) Though hydrogen is lighter than air it cannot be collected by downward displacement of air

Solution

Hydrogen is lighter than air so it is possible to collect the gas by downward displacement of air But it is not safe to do so since a mixture of hydrogen and air can lead to an explosion

(c) A pop sound produced when hydrogen is burnt

Solution

Impure hydrogen gas burns in air with a pop sound This is because of the presence of impurities in it

(d) Helium replaced hydrogen in weather observation balloons

Solution

It forms a mixture with air that can explode when there is a small leakage of hydrogen in a balloon So helium has replaced hydrogen

(e) Nitric acid not used for the preparation of hydrogen gas

Solution

(e) By the action of nitric acid on metals hydrogen cannot be produced because it also releases nitrous oxide and nitric oxide and oxides the hydrogen to form water

Biology Chp-2 Reproduction in plants

Today we discuss different methods of artificial propagation like cutting-rose sugercane Layering ndashguava lemon china rose etc Grafting- mango apple etcMicropropagation ndashorchid asparagus etcWe also discuss about advantages and disadvantages of vegetative propagation

Q7 Define the following terms i) Explant In tissue culture techniquea tiny piece of bud shoot or any other partof plant from where new tissue develop ii) Callus The cells of the tissue divide andgrow into a mass of undifferentiated cells from explant iii) Plantlet After few days callus differentiate into a small plant with roots and shootQ8 what are the advantages and limitations of tissue culture or micropropagation

Advantages i ) It produacuteces superior quality plantsii)It can be applied to interspecifie hybridsiii) It is useful to grow seedless plants bull Limitations i) It cannot be used for all plantsii)It is not easy to handleQ9 Write advantages of vegetative propagationi) It is a quick and easy method ofproducing new plantsii) This method need less time to matureiii) The new plants are exact copies of the parentiv) it is extremly useful for growing seedlessplants like banana grapes etc Q10 Write some disadvantages of vegetativepropagationi) Dišeases present in the parent plant gettransferred to all in new plantsii) Overcrowding of new plants causes competition for sunlight water and nutrients which affects growth of plantsplant

Physics Chapter 2 Physical Quatites and Measurements

Here We Will Do Some QuestionsRelated To Chapter 2

Select the correct alternative A block of wood of density 08gcm-3 has a volume of 60cm3 The mass of the block is

1 608 g

2 75 g

3 48 g

4 0013 g

Solution 348 g

The density of aluminium is 27g and that of brass The correct statement is

1 Equal masses of aluminium and brass have equal volumes

2 The mass of a certain volume of brass is more than the mass of an equal volume of aluminium

3 The volume of a certain mass of brass is more than the volume of an equal mass of aluminium

4 Equal volumes of aluminium and brass have equal masses

Solution 2 The mass of a certain volume of brass is more than the mass of an equal volume of aluminium

MATHEMATICS Ch 6Sets

Exercise 6(C)1 Find all the subset of each the following sets(i) A = 57 (iii) C = x xisin W x le 2(iv) p p is a letter in the word lsquopoorrsquo

Solution (i) All the subsets of A are ϕ 5 7 57

(iii) All the subsets of C are ϕ 0 1 2 01 02 12 012

(iv) All the subsets are ϕ p o r po or por

4 Given the universal set = -7-3-105689 find (i) A = x xlt2 (ii) B = x -4ltxlt6 Solution

(i) A = -7-3-10(ii) B = -3-105

5 Given the universal set = x xisin N and xlt20 find

(i) A = x x = 3p pisin N (iii) C = x x is divisible by 4 Solution

(i) 369121518 (iii) 481216

6 Find the proper subset of x x2-9x-10 = 0 Solution

ϕ 10 -1

Working x2-9x-10 = 0 rArr x2-(10-1)x-10 = 0

rArr x2-10x+x-10 = 0 rArrx(x-10)+1(x-10) = 0

rArr (x+1) (x-10) = 0

11 Let M = letters of the word REAL and N = letters of the word LARE Write sets M and N in roster form and then state whether (i) M sube N is true (ii) N sube M is true (iii) M = N is true

Solution M = real and N = lareSo (i) Yes (ii) Yes (iii) Yes

English 2 Twelfth Night ndash Shakespeare

A noble man named Orsino in the kingdom of Illyria is deeply in love with a lady called lady Olivia She is in mourning for her dead brother so she will not even think about marriage At this time a sea storm causes a terrible shipwreck and a young lady called Viola is swept onto the shore She thinks that her twin brother Sebastian is drowned A sea captain tells her about Orsino and his love for Olivia Viola wishes to work in Oliviarsquos home but feels she will not be employed So she dresses as a man calls herself Cesario and gets work at the house of OrsinoViola (now Ceasario) is much liked by Orsino and becomes his page She falls in love with Orsino Orsino sends Ceasario to deliver messages to Olivia Olivia herself falls for the beautiful young Ceasario believing Viola to be a man

2 Answer the following questionsa Why does Orsino ask the musicians to play onOrsino asks the musicians to play on because music feeds his desire He calls upon the musicians to play music so that his hunger for love could be replenished with an excess of musicb What does Valentine tell about OliviaWe learn from Valentine that Olivia is in mourning for her brother she wears a veil and has vowed that no one will see her face for another seven yearsand she refuses to marry anyone until thenc From the exchange between Orsino and Valentine what do you think their relationship isValentine is one of orsinod attendants He was sent to Olivia as a messenger of love but was not allowed to speak to here Who is Olivia mourning for and whyOlivia is mourning for her dead brother

Homework Q fHistory and Civics

Growth of Nationalism

Important dates to remember1769-Napoleon born on 15thAugust1789-Fall of Bastille on 14th July and the beginning of the French revolution declaration of the rights of Man on 26thAugust1793-King Louis XVI executed on January 211764-The Sugar Act passed1765-The Stamp act passed1774-The first congress of Philadelphia1776-The declaration of American Independence of on 4th July1777-Defeat of the British at Saratoga1781-Surrender of lord Cornwallis at Yorktown1783-The treaty of Versailles1804-Napoleon becomes the emperor1813-Battle of Leipzig or Battle of nations in which Napoleon was defeated by the Allies1815-Battle of Waterloo June 18 in which Napoleon was defeated and captured1821-Death of Napoleon in StHelena1860-Abraham Lincoln elected President of the USA1861-The civil war began 1864-Abraham Lincoln elected President of the USA for the second time1865-Slavery abolished in the US

Name the following- The queen of Louis XVI

Marie Antoinette The three philosophers of France

VoltaireMontesquieuJean Jacques Rousseau

The British general whose surrender brought the war in America to an endLord Cornwallis

The first president of the USAGeorge Washington (1732-1799)

The first southern state to secede from the unionSouth Carolina

The author of the book lsquoUncle Toms CabinHarriet Beecher Stowe

Homework-Learn

Class IXSubject Topic Summary Execution

Economics

Types of economies Today I am going to share you the concept of economic growth and economic development Few questions will be given from the previous study material dated 942020

Meaning of economic growthAnswer) The term economic growth generally means anincrease in national income or per capita output or income over time It indicates towards quantitative growth of a country

Meaning of economic developmentAnswer) Economic development is defined

as a process whereby the real per capita income of a country increases over time along with fall in poverty ratio unemployment and income inequality etc

Distinguish between economic growth and economic development

Basis Economic growth

Economic development

Scope It has narrow scope as it refer only to rise in per capita income

It has wide concept since it includes qualitative changes as well

Concerned matter

It is concerned with the rise in income

It is concerned with not only rise in income but also reduction of poverty income inequality and unemployment

Focus Economic growth does not focus on economic development

Economic development focus on economic growth plus qualitative changes

Distinguish between capitalist economy and socialist economy

Ownership

Motive

Tool

Means of production are owned and managed by private people

Self interest and profit earning is the main motive

Price mechanism is a main tool to solve the economic problems

Means of production are owned and managed by the government

Social welfare is the main motive

Economic planning by the government is the main tool to solve the economic problem

Competition

Distribution of income

There exist large competition among buyers and sellers

There is existence of large inequalities of income

There is no such competition

There exist less inequalities of income

Math Topic ndash AlgebraChapter -Factorisation

Study item Factorising by taking out common factorSome solved sums from exercise 41

1) (i) 8xy3 + 12x2y2

= HCF of 8xy3 and 12x2y2 is 4xy2

= 4xy2(2y + 3x )

4) (ii) 28p2q2r ndash 42pq2r2

= HCF of 28p2q2r and 42pq2r2 is 14pq2r = 14pq2r (2p - 3r )5) (ii) 14mn + 22m - 62p=HCF of 14mn 22m and 62p is 2= 2(7mn + 11m - 31p)7) (ii) 3a(x2 + y2) + 6b (x2 + y2) = HCF of 3a(x2 + y2) and 6b(x2 + y2 ) is (x2 + y2)= ( x2+ y2 )(3a + 6b )9) (ii) x(x2 + y2 ndash z2 ) + y(-x2ndashy2 + z2 ) ndash z(x2+ y2 ndash z2 )= x(x2 + y2 -z2) ndash y-(x2 + y2 -z2) -z(x2 + y2 ndash z2)=x(x2 + y2-z2) -y( x2 + y2-z2) ndash z (x2 + y2 -z2)= (x2+ y2 ndash z2)(x ndash y ndash z )

Commercial Studies

Introduction to Accounting and Book-keeping

Today I am going to share you the meaning of Accounting and Book-keeping and its related terms bullAccounting bullBook Keeping bullAccountsbullTypes Of Accounts bullAccounting Cycle

bull Meaning of accounting

Ans )Accounting is the art and science of recording classifying and summarising monetary transactions

bull Meaning of Book-keeping

Ans) Bookkeeping is the art of recording business transactions with the view of having a permanent record of them and showing their effect on wealth

bull Meaning of account

Ans) The term account means a record of business transactions concern a particular person of firm asset or income or expense It is a summarised record of all transactions which take place in an accounting year

bull Types of accountsPersonal accounts ndash Personal accounts relating

to person and Organisation are known

as personal accounts Example Ramrsquos Account ABC amp Co Account etc

Real account - The accounts related to tangible and intangible assets are called real accountsExample Cash Account Furniture Account etc

Nominal account- Accounts related to expenses losses incomes and gains are known as nominal accountsExample Wages Account Salary Account Discount Account etc

bull Accounting cycle Accounting cycle refers to a complete sequence of accounting activities It begins with recording of transactions and ends with the preparation of a balance sheet

English 1 Transformation of sentences

Sentences A sentence is a group of words which makes complete sense

a Assertive sentencesb Imperative sentencesc Interrogative

sentencesd Exclamatory sentences

Sentences can be changed from one grammatical form to another without changing the meaning of the sentence This is known as transformation of sentences

Exercise 6Rewrite the following sentences according to the instructions given below without changing their meanings

1 As soon as he saw the beer he jumped into the river ( Begin No sooner)

2 None but brave deserve the fair (Begin the bravehellip)

3 This box is too heavy for me to lift ( Use so hellip That instead of too)

4 No one other than a king can live like James Luxurious ( Begin only James)

5 Oh for the wings of a dove (Begin I wishhellip)

BENGALI(2ND LANGUAGE)

ldquo বঙগভমির পরমি ldquo াইকেল ধসদন দতত

পব13পোসঠ আসোলিচত ৩ পরবোস দৈদসবর বস ীবতোরো Pলিদ স এ লেদ -আকো সত-োলি লেদ তোস - ক) বকতো লেক কোর লেো লেকো কলিবতোর অং ) কোর পরলিত বকতোর এই উলিকত গ) এ লেদ আকো সত বসত কী বলিঝসয়স4 ীবতোরো বসত কী লেবোঝ ঘ ) আসোচয অংসর তোৎপP13 কী

উ -ক ) বকতো স কলিব মোইসক ম3দ দতত

Types of AccountPersonal AccountReal AccountNominal AccountBalance Sheet (opening)

কলিব মোইসক ম3দ দসততর রলিচত বঙগভলিমর পরলিত কলিবতোর অং ) কলিব বঙগী অ13োৎ লেদমোতোর পরলিত কলিবর এই উলিকত গ ) এ লেদ আকো বসত কলিবর মোব লেদী রপ আকো লেক লেবোঝোসো সয়স4 আকো লেসক লেPম তোরো স পসর লেতমলি ীব লেদ রপ আকো লেসক পরো রপ তোরো স পরসত পোসর এই মভোবোর কোই কলিব বসস4 ঘ ) পরবো Pোতরোয় Pলিদ কলিবর লেদ আকো লেসক ীব তোরো রপ পরো স পসর তোসত কলিব লিবনদমোতর দঃলিত কোর মতয লিবসর সবোভোলিবক পলিরলিত এবং মোষ মরী তোই পরবোস Pলিদ তা োর মতয য় তবও কলিব লিবচলিত সব ো কোর পলিবীসত লেকউ অমর য় লিক4ই অকষয় য় দীর লেPম লিচরপরবোমো লেতমলি মোসষর ীবও চমোতোই ীব - সতবধতোই মতয ীব দীসত মোষ লিতয পরবোমো তবও লেPব মোষ আপ কতকসম13র মো3যসম মোসষর মস লিসসদর সথো কসর লিসত পোসর তোরো লিচরভোসবর সয় মোসষর মস লিবরো কসর তোসদর মস3য লেকউ পGভসত লিবী সয় গোসও মোসষর মস তোরো লিতযপলিত লিতযবলিনদত

Hindi 2ndlang

काकीी(लिसयारामशरणगपत)

इस कहानी म क न यह बतान का परयास निकया ह निक बचच अपनी मा स निकतना परम करत ह शयाम अबोध बाक ह वह अपनी मा क मरन क बाद उसन अपनी मा क लिए बहत रोया बाद म उस पता चा निक उसकी मा राम क घर ची गई ह आकाश म उडती हई पतग दकर उस हष हआ निक पतग क दवारा वह अपनी मा को नीच उतारगा इसक लिए वह अपनी निपता की जब स दो बार सवा रपया निनकाकर पतग और दो मोटी सी मन वाी अपन भाई स काकी एक कागज पर लिवा कर पतग म लिशव का दिदयानिनकाकर पतग और दो मोटी सी मन वाी अपन भाई स काकी एक कागज पर लिवा कर पतग म लिचपका दिदयाभोा और शयाम कोठरी म रससी बाधनी रह थ तभी उसक निपता करोध म आकर उन स पछ निक कया उनकी जब स रपया निनकाा हभोा डर क मार बताया निक शयाम इस पतग क दवारा अपनी काकी को राम क यहा स उतारना चाहता हनिवशशवर(शयाम क निपता)न फटी पतग उठाकर दी तो उस पर काकी लिा थावह हत बजि होकर वही ड रह गएउनहोन सोचा निक मन अपन पतर को मारा जोनिक अनजान और निनदष थावह अपनी मा कोनिकतना पयार करता ह

helliphellipContinue to next

Computer Application

Java Programming Prog 1Write a java program to input two numbers from user and display the sum or product of them as per user choice Use switch case statementSolve public class sum_product public static void main(String args[]) Scanner sc=new Scanner(Systemin) int abc Systemoutprintln(ldquoEnter two numbersrdquo) a=scnextInt() b=scnextInt() Systemoutprintln(ldquoPress 1 for sum or 2 for productrdquo)

c=scnextInt() switch(c) case 1 Systemoutprintln(ldquoThe sum will be =rdquo+(a+b)) break case 2 Systemoutprintln(ldquoThe product will be =rdquo+(ab)) break default Systemoutprintln(ldquoWrong Inputrdquo) Home Work - Practice in your computer using bluej

Subject Eng Literature (The Merchant of Venice ndash William Shakespeare)Topic Act I Scene 2 Lines 92 to 126 (End of scene) Date 13th April 2020 (5th Period)

[Students should read the original play and also the paraphrase given in the school prescribed textbook]Summary Questions amp Answers

o After Portia has expressed her opinion about the suitors Nerissa informs that she need not bother about any one of them as they have decided to quit Belmont at the earliest opportunity because they do not believe in trying their luck by the caskets which is the only way of winning Portia

o Nerissa then enquires of Portiarsquos opinion about Bassanio who once visited her in the company of the Marquis of Montferrat and says that she had never come across such an ideal love deserving the fairest lady for his bride

o Portia seems to remember Bassanio quite correctly and says that she agrees with Nerissa At this moment a servant informs Portia that the Prince of Morocco has arrived to try his luck by the caskets

o Portia tells Nerissa that if she could welcome this new suitor as gladly as she says farewell to the previous ones she would be glad of his arrival However if he happens to have the virtues of a saint but the black complexion of a devil she would prefer to have him for religious consolation rather than as a husband

(1) NERISSA You need not fear lady (Line 97-103)

the having any of these lords they have acquainted me with their determinations

which is indeed to return to their home and to

trouble you with no more suit unless you may be wonby some other sort than your fathers imposition depending on the caskets

PORTIA If I live to be as old as Sibylla I will die as chaste asDiana unless I be obtained by the manner of my fatherswill I am glad this parcel of wooers are so reasonablefor there is not one among them but I dote on his veryabsence and I pray God grant them a fair departure

(a) Elucidate the idea expressed in the first speech of the above dialogue

In the first speech Nerissa assures Portia that she need not have any fear of being compelled to marry anyone of the suitors who had lately come to Belmont She informs her that they have all decided to return to their respective countries(b) Illuminate the meaning of the phrase ldquoyour fatherrsquos imposition depending on the casketsrdquo

Nerissa means that the suitors of Portia do not find the conditions imposed by the will of her father to their liking They are too hard for them These conditions are that in the event of a suitor failing to choose the right casket (i) he should never disclose to anybody which casket he chose (ii) he can never marry and (iii) he should take his departure immediately(c) Explain the meaning of the term lsquoSibyllarsquo

lsquoSibyllarsquo is the name given by Romans and Greeks to a prophetess inspired by some deity usually the sun-god Apollo She had a very long life The god Apollo granted her as many years of life as she could hold grains of sand in her hand(d) Elucidate the meaning of the term lsquoDianarsquo

lsquoDianarsquo is the goddess of hunting She is also regarded as a symbol of virginity because she never fell in love and never

married(e) Explain the meaning of the first two lines of Portiarsquos speech

Portia says that even if she is to live for centuries like Sibylla she would not marry except in accordance to her fatherrsquos will She asserts that she would not mind remaining unmarried and untouched by a man like Diana the virgin the goddess of hunting unless a man is able to win her by passing the test laid down by her father

Class XSubject Topic Summary Execution

Hindi 2nd

Langबड घर की बटी( मशी परमचद)

lsquoबड घर की बीटीrsquo कहानी का उददशय मधयम वग की घर समसया को सझा कर सगदिठत परिरवार म मिम जकर परम स रहन का सदश दना ह घर म शानित tानिपत करन की जिजममदारी नारी की होती ह यदिद नारी समझदार ह उसम धय और परिरवार क परनित परम ह तो कोई भी घटना परिरवार को निवघदिटत नही कर सकती या कहानी परिरवार को सगदिठत करत हए परम सौहाद स एक रदसर की भावनाओ को समझ करउनका सहयोग करत हए जीवन यापन करन की पररणा दती ह मशीपरमचदर जी न इस कहानी म सयकत परिरवार का परनितनिनमिधतव निकया ह यह कहानी बनी माधव सिसह जो गौरी पर क जमीदार क उनक दो पतरो की हशरी कठ ा निबहारीशरीकात का निववाह एकजमीदार घरान की पतरी आनदी स हआ थाआनदी न द को ससरा क वातावरण म ढालिया थाएक दिदन आनदी का अपन दवर ा निबहारी स झगडा हो जाता ह दोनो भाई एक रदसर स अग होन की कोलिशश करत हसभी बह आनदी न अपन मधर वयवहार स ा निबहारी को घर छोडकर जान स रोक लिया| इस पर बनी माधव सिसह न कहा निक बड घर की बटी ऐसी ही होती ह जो निबगडा काम बना ती ह अतः शीषक साथक ह बड घर की बटी आनदी ह

helliphelliphelliphellipContinue to nextBiology Topic ndash Chp-1

CellWelcome to new session 2020-21Today we will start with Chpter 1 cell CELL

Protoplasm+Cellmembrane Or Cell wall

Cytoplasm+Neucleus

Cytoplasmic+ CytoplasmicOrganelles Inclutions(mitochondria (food Golgi bodies pigments)Ribosome)

What is cellbull Cell is the structural and functional unit of living organismbull According to number of cells organisms areUnicellular - Amoeba bacteria Multicellular - Rose Mango Tiger HumanSmallest cell -bacteria Longest cell - Nerve cellLargest cell - Ostrich egg cellCells are of different size and shapes according to their functionsQ2Write chief functions of following cellorganelles

Q3What is tonoplastVacuoles covered by a covering called tonoplast

Bengali(2Nd

Language)

ফ ফটক ো ফটক (কলিবতো ) ভোষ মসোপো3 gtPোয়

একটি লেমসয়র ীবস লেপরম লিকভোসব ফসট ওসঠ তো লেদলিসয়স4 কলিব লেপরম Pই য় লেই ময়ই বনত কোস পলিরত য় ফ লেফোটো বো োসফোটো লেটো ব2 কো য় লেমসয়সদর ব gtয13 লেপরসমর 4লিব ফসট উসঠস4 এই কলিবতোয় লেপরম মোষসক মত gtযর মস লেফস লিদসয় পরকষস বাোচোসোর gtয োত বো2োয় কলিবতোয় লেমসয়টির পসব13র দঃসর কো বো সও লেমসয়টি লেই পসর পলিক সত চোয়ো োরী ীবসর কোস4 পরম লেPৌবস লেপরমসক পোবোর পরব ইচছো োকসও তো পসর লেলিতবোচকতোয় পলিরত য় কলিব ভোষ মসোপো3 যোয় লেP ক লেপরসমর

কলিবতোয় ব gtযবহত লিবসষ লিক4 সvর অ13 লেদওয়ো ১) রসবোো= লেP লিবলিভনন রকম ডোকসত পোসর২) ো= পোর ৩) ঠলি = লেচোসর বZ৪)আই বস2ো=অলিববোলিত৫)শইসয় = োলিয়ত কসর৬)োতপাোচ= লিবলিভনন পরকোর৭)দ2োম = v কসর বZ কসর লেদওয়ো৮)লেরলিং =লেোোর দৈতরী লেব2ো৯) বনত= একঋত১০) পাোর = বসকরো2

Organelles Functions

1 Endoplasmic reticulum

2 Mitochondria

3Golgibodies

4 Ribosome5Lysosome

6Plastids

7 Centrosome

i) Supportive framework for the cellii) Synthesis and transpost of proteinsRelease of energy in the form of ATPi) Synthesis and secretion of enzymes hormoneii) Formation of vacuoles lysosomei) Protein Synthesisi) Intracellular digestionii) Destroy foreign substancei )Leucoplast - stores starchii)chloroplast - trap solar energyiii) Chromoplast - imparts colour toflowers amp fruitsi) Initiates and requlates cell division

কলিবতো তোর অ13সক ভোষোয় পরকোো কসর ঘলিরসয় ব যকত কসরস4 লেপরসমর ফতো আর লিবফতো লেক গোঢ় কসর লেদোসো কলিব ভোষ মসোপো3 যোসয়র অলিভবসর অ যলিদক

Economics

Factors of Production

Welcome to the new sessionToday we are going to start the first chapter of Class XThe name of the chapter is Factors of productionBy the name I hope you all can recall a glimpse of what you have learnt in the second chapter of Class IX

NowProduction is the process of creating the various goods and services which are consumed by the people of the country to satisfy their wants

Thus it is the process in which some materials are transformed from one form to another to create utility and value in goods

For example utility can be created by changing the form of a commodity ie

Making of table out of wood by a carpenter for his customer here the wood is getting transformed into table creating utility for his customer and he can also command a price for it

On the other hand Housewives perform very

useful activities at home which create utility but their domestic activities are not included in production because they have no money value

So we can also say that Production denotes two things firstly creation of utility and secondly creation of value

Production is not complete unless it reaches the consumer

An increase in production will increase the economic welfare of the consumers and hence the aim is to raise the production level of the country

Again production of a good or service is only possible if certain resources or

Questions

1 What do you mean by production

Answer Production means the creation of goods and services for the purpose of selling in the market

In fact production involves the transformation of inputs into outputs

Hence production denotes two thingsCreation of utility and creation of valueUtility and value can be created by changing the form by changing the place by changing the time and by rendering services

Example Transformation of raw

materials into finish goods such as potter creates utility by converting mud into utensils assembling of small parts to make bigger machinery

Production also includes services such as distribution and marketing

2 What are the factors of production

Answer Factors of Production refers to the resources and inputs needed for producing goods and servicesThese inputs can be classified as

Land Labour

Capital Enterprise

Land Land is defined to include not only the surface of the earth but also all other free gifts of nature(for example mineral resources forest resources and indeed anything that helps us to carry out the production of goods and services but is provided by

inputs are used together in right proportion

A resource or an input which helps in the process of production to obtain an output is called FACTOR OF PRODUCTION

These factors of production can broadly be categorized into four parts 1LAND 2LABOUR3CAPITAL4ENTERPRISE (ORGANISATION)or Entrepreneur

The above factors are all interdependent on each other and they play a major role in production process

FACTORS OF PRODUCTION

LANDCAPITAL

LABOUR ENTREPRENEUR

nature free of cost)LabourLabour refers to the human efforts that need to be combined with other factors of production for creating an output

CapitalAll man ndash made means of production is called capita example machineries which help in further production Money when used for starting any business for purchasing raw materials machinery tools etc it is regarded as capitalCapital also includes physical capital like factories machineriestoolsbuildingsequipments etcEnterpriseThe task of bearing risks is called enterprise and the person who bears these risks of business is called the entrepreneurThus an entrepreneur is one who organises production takes important decisions regarding production hires and purchases factors of production and bears the risk and uncertainty involved in productionOrganisation refers to the services of an entrepreneur who controls organises and undertakes all risks One who plans organises and manages a business enterprise is an organiser

Physics Chapter 1 Force

Force is an external agent capable of changing the state of rest or motion of a particular body It has a magnitude and a direction The direction towards which the force is applied is known as the direction of the force and the application of force is the point where force is applied The Force can be measured using a spring balance The SI unit of force is Newton (N)

Question 1

State the condition when on applying a force the body has

(a) the translational motion

(b) The rotational motion

Solutions

(a) Translational motion is produced when the body is free to move

(b) Rotational motion is produced when the body is pivoted at a point

Question 2

Define moment of force and state its SI unit

Solutions

The moment of force is equal to the product of the magnitude of the force and the perpendicular distance of the line of action of force from the axis

of rotation

The SI unit of moment of force is Newton times meter

= Newton meter (Nm)

Commercial Studies

Stake holders In this topic you will be come to know about the meaning and concept of stakeholders

How stakeholders are different from shareholders

Questions1 What do you mean by the term stake holdersAnswer) The term stake holders have developed from the words which mean an interest or expected benefit Stakeholders mean all those individuals groups and Institutions which have a state (interest) in the functioning and performance of a commercial organisation or a business enterprise2 What do you mean by share holdersAnswer) The person and Groups who own the shares of the joint stock company by providing capital to the company are called shareholders Shareholders are the internal stakeholders shareholders are one out of several stake holders3 How are shareholders different from stakeholdersAnswer)i) The term shareholders is related to only joint stock company whereas stakeholders are related with all business organisationsii) Stakeholders maybe any individual having financial stake in business organisation whereas a shareholders are those individuals who are holding shares in the company4) How are shareholders different from creditorsAnswer) i) Shareholders are internal stakeholders while creditors are external stakeholdersii) Shareholders invest in the capital of the company whereas creditors give loan to the companyiii) Shareholders are the members of the company with voting rights but creditors are not the members of the company

English 1 Transformation of sentences

Sentences A sentence is a group of words which makes complete sense

e Assertive sentencesf Imperative sentencesg Interrogative sentencesh Exclamatory sentences

Sentences can be changed from one grammatical form to another without changing the meaning of the sentence This is known as transformation of sentences

Exercise 1 Change the following affirmative sentences into Negative sentences

a He is a good manHe is not a bad man

b Ram loves SitaRam is not without love for Sita

c Only he stood first in the classNone but he stood first in the class

d Ankit was wiser than he

He was not so wise as Ankite He did it

He did not fail to do itf As soon as I reached college the

bell rangNo sooner did I reach college than the bell rang

g He finished everythingHe left nothing unfinished

h It always pours when it rainsIt never rains but it pours

Math Topic Commercial MathematicsChapter ndash Goods and services Tax

What is GSTAns It is a abbreviated term of Goods and Service Text which is an indirect tax levied on the sale of goods and rendering servicesSome terms related to GSTDelar Any person who buys goods or services For resale is known as a delar A delar Can be a firm or a companyIntra-state sales Sales of goods and services within the same state or same union territory are called intra- state salesInter-state sales Sales of goods and services outside the state or union territory are called Inter-state sales4) Input GST GST is paid by dealers on purchase of goods and services are called input GST5) Output GST GST is collected from customers on sale of goods and services are called output GST6) Types of GST There are three taxes applicable under GST(i) Central Goods and Services Tax (CGST)(ii) State Goods and Services Tax (SGST) or Union Territory Goods and Services Tax (UTGST) Both these taxes are levied on intra-state sales Here GST is divided equally among central and state governments(iii) Integrated Goods and Services Tax (IGST) IGST is levied on inter- state sales It is also levied on import of goods and services into India and export of goods and services from India

Subject Eng Literature (The Merchant of Venice ndash William Shakespeare)Topic Act III Scene 4 Lines 1 to 44 (Portia hellip To wish it back on you fare you well Jessica)[Students should read the original play and also the paraphrase given in the school prescribed textbook]

Summary Questions amp AnswersIn this scene we suddenly find a new element in the character of Portia We have already seen her possessed of every graceful womanly quality but now she shows that she is capable of rapid decision and determined action She shows this by her sudden resolve to hasten to Venice with a daring scheme for the rescue of Antonio This is an important scene in the dramatic action for it leads up to and renders possible the striking events of the famous trial scene which is one of the greatest striking elements of the play Moreover the fact that all the characters of importance are now assembled together in Venice makes the union of the main plot and the secondary story complete

(1) LORENZO Madam although I speak it in your presence(Line 1-9)

You have a noble and a true conceit

Of god-like amity which appears most strongly

In bearing thus the absence of your lordBut if you knew to whom you show this honourHow true a gentleman you send reliefHow dear a lover of my lord your husbandI know you would be prouder of the workThan customary bounty can enforce you

(a) Where is Lorenzo Why is he here To whom is he referring as lsquoMadamrsquo

Lorenzo is at Portiarsquos residence He had met Salerio on the way and Salerio had begged him to come along with him to

o In this scene Portia Nerissa Lorenzo Jessica and Balthazar appear

o Portia requests Lorenzo and Jessica to be in charge of her house during her absence from Belmont because she and Nerissa have decided to spend the days in meditation and also in visiting the holy places in the neighbourhood of Belmont She has already instructed her people to acknowledge both Lorenzo and Jessica as master and mistress of house during her absence Lorenzo and Jessica gladly agree to look after the house of Portia

handover the letter from Antonio to Bassanio The letter carried the bad news about Antoniorsquos arrest for non-payment of loan taken from Shylock Hence Salerio might have preferred company to break this bad news to Bassanio He is referring to Portia as Madam(b) What does Portia say on hearing the above extract

Portia says that she has never regretted doing good to others Friends who spend a lot of time together and really are there for each other have many traits in common As Antonio is Bassaniorsquos best friend saving him is like saving Bassanio who is like her own soul She asks Lorenzo to take care of management of the house till Bassanio is back(c) What does Portia send with Bassanio and why

On hearing about Antoniorsquos troubles on account of Bassanio her husband Portia immediately sends him with enough gold to repay the debt many times over to Venice to help Antonio out of his misfortune

(2) Lorenzo Madam with all my heart (Line 36-40)

I shall obey you in all fair commands

Portia My people do already know my mindAnd will acknowledge you and JessicaIn place of Lord Bassanio and myselfSo fare you well till we shall meet again

(a) Where are Lorenzo and Portia at this time What lsquofair commandsrsquo are given to Lorenzo

Lorenzo and Portia are at Belmont during this scenePortia reveals to Lorenzo that she has sworn to contemplate in prayer at a monastery around two miles away until her husband returns from Venice She tells him that Nerissa would accompany her and asks him to manage the house with Jessica till things are settled In response Lorenzo tells her that he would be obliged to do whatever she asks him to do(b) Where is Portia actually going and why

Portia tells Lorenzo that she would live a life of contemplation and pray at a monastery which is two miles away from her place In reality Portia plans to go to Venice in disguise with Nerissa and argue the case in defense of Antonio She is very sure that her plan would succeed

ClassXI (ScienceHumanitiesCommerce)Subject Topic Summary Execution

Computer Science

(APC)

Ch ndash 1 Numbers

(Numbers in different bases and

their Arithmatical operations)

Number System In computers Number System is defined as a writing system to represent the numbers in different ways ie we are using different symbols and notations to represent numbers There are four ways we can represent the number ndash Binary Decimal Octal and Hexadecimal

Decimal Number SystemThis number system consist 10 digits These are 0 1 2 3 4 5 6 7 8 amp 9

Binary Number SystemThis number system has only two digits these are 0 and 1 Here 0 stands for off while 1 stands for on

Octal Number SystemThis number system has 8 digits these are 0 1 2 3 4 5 6 amp 7

Hexadecimal Number SystemThis number system has 16 digits these are 0 1 2 3 4 5 6 7 8 9 A B C D E F Here the value of the alphabets are as follows A=10 B=11 C=12 D=13 E=14 F=15

Rules for conversion decimal number to Binary1 Divide the decimal number by 22 If the number will not divide equally by 2 then round down the answer to the nearest whole number (integer)3 Keep a note of the remainder it should be either 0 or 14 Keep repeating the above steps dividing each answer by 2 until you reach zero5 Write out all the remainders from bottom to top This is your binary solution

For example Lets convert 32 to binary 2 32 2 16 - 0 2 8 - 0 2 4 - 0 2 2 - 0 2 1 - 0 0 - 1

The binary equivalent of 3210 is 1000002

Try the follwing youself1 2410

2 4810

3 1210

History GROWTH OF NATIONALISM

The second half of the 19th century witnessed growth of political consciousness and a sense of Nationalism among the IndiansThere were various factors for growth of Indian Nationalism- As a result various political associations were formed in different provinces by the educated Indians Surendranath Banerjee organized a meeting of National conference at Calcutta Ultimately the National Congress was founded in Bombay in 1885This body became the vanguard of Indian struggle for freedom The congress leaders were known as moderates because they followed a policy of prayer and petition A large number of Indian leaders had experienced in political agitation The Political situation of England was also changed Moreover increasing revolutionary activities in Maharashtra Punjab and Bengal became serious concern to the British Government In this

QUESTION1 What do you mean by Nationalism ANSWER 1 Nationalism is defined as loyalty and devotion to own nation especially a sense of national consciousnessQUESTION 2 What are the causes of nationalism ANSWER 2 There were various factors for growth of nationalism

1 Spread of western education2 The progress of vernacular press and

patriotic literature3 The economic exploitation of our

country by the colonial rulers4 International affairs

QUESTION 3 Who organized National conference in Calcutta in 1883 ANSWER 3 Surendranath BanerjeeQUESTION 4 When did Indian National Congress formANSWER 4 Indian National Congress was formed in 1885 in BombayQUESTION 5 Who were ModeratesANSWER 5 The Early Nationalists were also known as Moderates Their emergence marked

background Lord Curzon became Viceroy in India He had no respect for the Indian National Congress

the beginning of the organized national movement in India They believed in British justice and were loyal to them They followed a policy of prayer and petition They demanded constitutional reforms of our country Impotant Moderate leaders were Pherozshah Mehta Dadabhai Naorozi and Surendranath Banerjee etcQUESTION 6 What do you know about Extremism in Indian National movementANSWER 6 In the beginning of 20th century a new class of national leaders emerged in India which was different from the moderate groups They started more aggressive movement against the British empire The goal of extremists was ldquoswarajrdquo Important extremist leaders were Bal Gangadhar Tilak Lala Lajpat Rai Bipin Chandra Pal etcQUESTION 7 Mention the places which were the main centres of Revolutionary movementANSWER 7 Maharashtra Bengal and Punjab

Physics

Chapter Dimensional Analysis

(Summary)

The dimensions of a physical quantity are the powers to which the fundamental units are raised in order to obtain the derived unit of that quantit

The physical quantites lengthmasstime are represented by [L] [M] [T] resp let they are raised to powers ( dimesions) abc resp then any physical quantity can be represented by [ La Mb Tc ] Examples

1 Area area = L x B = [L] x [L] = [M0 L2 T0 ]

2 Density density = massvolume = [M][L3] = [ M L-3]

3 Velocity velocity = distancetime = [L][T] = [LT-1]HW Try to find out dimension of acceleration Acceleration = velocity timeNB One can find the SI Units Using Dimension Analysis Such as for area we have [L2] so its SI unit is m2

Biology Topic ndash Chp-1 The living world

Today we will start the first chapter the living world Here we discuss about the characteristics of living organism and what are the difference between them and nonliving substances We also discuss about the contribution of different Scientists

There are over 500000 species of plants andover a million species of animal are present on earth Some 15000 new species were discovered every yearQ1 What is a living organismbull A living organism is primarily physico -chemical material that demonstrate a high degree of complexity is capable of selfRegulation possesses a metabolism and perpetuates itself through timeQ2 What are the differences between livingand non-livingsi) Compared with non-living living organisms

have more complex organised structure and their use of energy is more controlled amp efficientii) Living things reproduce their own kind by forming new cells which contains copies of their genesiii) Each organism has some degree of homeostasisie it is able to make adjustments so that internal environment remains constantQ3 Write contributions of following Scientists i) Aristotle - One of the first theories in Biology places all living things in a hiearchieii) AV Leeuwenhoek - was the first to observe living single celled organisms under microscopeii) Carolus Linnaeus - developed the binary system for naming of organisms and classificationiii) Geregor Johann Mendel ndash discoverbasic principles of inheritanceHomework i) C Darwin ii)Schleiden

Math Trigonometric functions

1 Overviewi) Trigonometry The word lsquotrigonometryrsquo is derived from the Greek words lsquotrigonrsquo and lsquometronrsquo which means measuring the sides of a triangle An angle is the amount of rotation of a revolving line with respect to a fixed line Usually we follow two types of conventions for measuring angles ie a) Sexagesimal system b) Circular system In Sexagesimal system the unit of measurement is Degree In Circular system the unit of measurement is Radian ii) Relation between degree and radianThe ratio of circumference of a circle to its diameter is always a constant This constant ratio is a number denoted by π which is taken approximately as 227The relationship between degree amp radian measurements is as follows2 right angles = 180deg= π radians1radian = 180degπ=57deg16(approx) 1deg=π180 radianiii) Length of an arc of a circleIf an arc of length s subtends an angle θ radians at the center of a circle of radius r then s=rθiv) Area of a sector of a circleA sector is like a pizza slice of the

Q) Express the following angles in radiana) 45deg b) 40deg3730Ans a) We have 180deg=π radiansi e 45deg= πtimes45180 radian = π4 radiansb) 40deg3730= 40deg37+3060 minute= 40deg 37 +12 minute= 40deg+ 752 minute=40 + 75(2times60) degree=3258 degreeNow 180deg=π radianie 3258 degree= (πtimes325) (180times8) radians = 65π288 radiansQ) A circle has a radius of r=12 meters What is the length of an arc traced out by a 60deg angle in the center of the circleAns In this problem we know both the central angle (60deg) and the radius of the circle (12) All we have to do is plug those values into our equation and we get

s = 2π(12)(60360)s = 24π6s = 4πSo the length of an arc traced out by a 60deg angle in a circle with a radius of 12 meters equals 4π meters asymp 1257 metersQ) Find the area of the sector with a central angle 30deg and a radius of 9cmAns GivenRadius r = 9 cmAngle θ = 30degArea of the sector = θ360degtimesπr2

= 30360degtimes227times92=2121cm2

circle It consists of a region bounded by two radii and an arc lying between the radiiThe area of a sector is a fraction of the area of the circle This area is proportional to the central angle In other words the bigger the central angle the larger is the area of the sectorArea of Sector = θ2 times r2 (when θ is in radians)

Area of Sector = θ times π360 times r2 (when θ is in degrees)

COMMERCE

CLASSIFICTION OF HUMAN ACTIVITIES-ECONOMIC AND NON-ECONOMIC

Welcome to the new sessiontoday we are going to start the first chapter of Class XI The name of the chapter that we are going to start is

lsquoClassification of Human Activities ndasheconomic and non-economicrsquo

Now let us start the chapter by considering human beings and the activities they perform throughout the day

Human activities means all those activities that human beings undertake to satisfy their wants

Human wants on the other hand are the desire of human beings for goods (vegetables fruits rice etc) and services (services of doctors teachers lawyers etc) that they require to live

Now these human activities continue throughout life as human wants are unending unlimited and recurring as human beings desire for better living throughout their lives

Now human activities can be classified into two categories

Human activities

Economic activities Non-economic activities

Economic activities are

Questions1 What are human activities

Answer Human activities mean all those activities that human beings undertake to satisfy their wants

Example A man working in an office

A boy playing in the garden

2What are the characteristics of human activitiesAnswer the characteristics of human activities are as follows

Human activities are undertaken by men women and children and these activities involve human efforts

Human activities are undertaken to satisfy human wants which are unlimited

Human activities continue throughout life

Human activities are performed for both earning money and personal satisfaction

3What is economic activitiesGive example

Answer Economic activities are undertaken by human beings with the object of earning money acquiring wealth and thereby satisfying human wantsExample

Selling of goods by a shop keeper to his customer

A clinic run by a doctor Service of a teacher in school or college

undertaken by human beings with the object of earning money and acquiring wealth

These activities result in the production of economic goods and services

Example Human activities(ie working in factories officesshops) which produce direct economic benefits

Non-economic activities are inspired by human sentiments and emotions such as love for the family desire to help the poor and love for the country

Thus these human activities (eg praying playing sleeping) produce no direct economic benefits and they are also not related to earning money and acquiring wealth

4 What are the characteristics of economic activities

Answer The characteristics of economic activities are as follows

Economic motiveEconomic activities are undertaken to earn money and acquire wealth

ProductiveEconomic activities involve productiondistribution and exchange of goods and services to create wealth

Economic growthEconomic activities determine the level of economic development of a country and standard of living of its citizens

Socially desirableEconomic activities are socially desirable for society

Economic resourcesEconomic activities make use of all the economic resources such landlabourcapital etc

5 What do you mean by non-economic activitiesExampleAnswerNon-economic activities are inspired by human sentiments and emotions such as love for the family desire to help the poor and love for the countryThese activities are not undertaken for monetary gain but for onersquos satisfaction and happinessExample

a mother looks after her children

a student donates blood8 Differentiate between Economic activities and Non-economic activities

Economic activities

Non-economic activities

1to earn living and acquiring wealth2Result can be measured in terms of money

3ExampleBusinessprofession and employment

1 to obtain some satisfaction

2Result cannot be measured in terms of money

3ExampleFamily-orientedreligious socialCultural and national

BUSINESS STUDIES

BUSINESS ENVIRONMENT

Welcome to the new sessionToday we are going to start the first chapter and the name of the chapter is Business Environment

In todayrsquos world every business enterprise is a part of the society It exists and operates in association with various groups in society such as customers suppliers competitors banks and financial institutions government agencies trade unions media and so on All these groups influence the functioning of business in one way or the other They constitute the environment of businessConcept of Business Environment

The term lsquobusiness environmentrsquo refers to the sum total of all individuals institutions and other forces that lie outside a business enterprise but that may influence its functioning and performance

The main features of business environment

Totality of External forces General and Specific forces Interrelatedness Complexity Dynamic Uncertainty Relativity

The Interrelation between business and its environment

The business enterprise is an open system It continuously interacts with its environment It takes inputs

Prepare the following questions from todayrsquos assignment

1 What do you mean by business environment

The term lsquobusiness environmentrsquo means the aggregate of all forces factors and institutions which are external to and beyond the control of an individual business enterprise but they may influence its functioning and performance Business environment is the macro framework within which a business firm a micro unit operates It consists of several interrelated and interacting elements

2 Explain the main features of business environment in brief

Totality of External forces-Business environment is the sum total of all things external to a business environment

General and Specific forces-It includes both the forces general forces are the economic social political legal and technological conditions which indirectly influence all business enterprise Specific forces are the investors customers competitors and suppliers which influence individual enterprise directly

Interrelatedness-Different elements of environment are interrelated for an example growing awareness for health care has increased the demand for health foods

Complexity- Business environment id

(such as raw materials capital labour energy and so on) from its environment transforms them into goods and services and sends them back to the environment

Fig 1 Business Environment Relationship

complex in nature as the elements keep on changing example economic technological and other forces changes in demand for a product and service

Dynamic-Business environment is not static it keeps on changing

Uncertainty- Itrsquos very difficult to predict future events such as technology and fashion which occur fast and frequently

Economics Basic Economic ConceptsSub topic

Microeconomics and

Macroeconomics

Welcome to the new sessiontoday we are going to start the first chapter of Class XI The name of the chapter that we are going to start is Basic Economic concepts

Now Economics covers the study of human activities Human activities are those activities which are performed by humans to satisfy their wants

Thus Human wants are unlimited and therefore economic activities such as production exchange and consumption are needed in order to satisfy those wants

The study of economics is divided largely in two parts which areMicroeconomics and Macroeconomics

SUBJECT- MATTER OF ECONOMICS

MICROECONOMICS MACROECONOMICS

Questions1Who has coined the words micro and macro economics

Answer Ranger Frisch coined the words lsquomicrorsquo and lsquomacrorsquo in 1933 to denote the two branches of economic theory namely microeconomics and macroeconomics

2What is microeconomicsAnswer It is the study of behaviour of individual decision ndash making unit such as consumers firms etc

3 What is macroeconomicsAnswer Macroeonomics is the study of overall economic phenomena like employment national income etc

4 What is the importance of microeconomicsAnswer

Microeconomics helps in formulating economic policies which enhance productive efficiency and results in greater social welfare

It helps the government in formulating correct price policies

It explains the working of a capitalistic economy where individual units(producers and consumers ) are free to take their own decision

Micro means a small part in

microeconomics we do not study the whole economy Hence we study an individual consumer and his or her choices and a producer and his or her profit maximizing decisions in the market Thus it does not mirror what happens in the economy as a whole

Macroeconomics on the other hand studies the economy as a whole It is concerned with aggregate and depicts the entire picture of the economyMacroeconomics deals with the national income aggregate investment aggregate consumption etc

Features of Microeconomics It deals with small

parts of the country Hence it looks at

individual consumers firms and industries

It deals with individual income consumption and savings

It studies the determination of price of any product or factors of production

It deals with the working of market via the price mechanism which is nothing but the determination of price and quantity of a commodity by the forces of demand and supply

Features of Macroeconomics

It deals with the study of the economy as a whole

It is concerned with

5 Give a limitation of microeconomics Microeconomics fails to explain the

functioning of an economy as a whole It cannot explain unemployment illiteracy and other problems prevailing in the country

6 What is the importance of macroeconomics It gives overall view of the growing

complexities of an economic system It provides the basic and logical

framework for formulating appropriate macroeconomic policies (eg for inflation poverty etc )to direct and regulate economy towards desirable goals

7What is the limitation of macroeconomics It ignores structural changes in an

individual unit of the aggregate

8 Differentiate between Microeconomics and Macroeconomics

Microeconomics Macroeconomics

the study of aggregates

National income aggregate savings and aggregate investments are major concepts dealt within macroeconomics style

It studies the determination of general price levels

It investigates into the problem of unemployment and the achievement of employment

It studies the aspect of decision making at the aggregate and national levels

It includes all growth theories whether related to developed or developing economies it also includes the study of economic systems and the working of the economy under different systems

Note Both Micro and macro economics are complementary and should be fully utilized for proper understanding of an economy

1It studies economic aspect of an individual unit2It deals with individual incomeConsumption and savings

3 It facilitates determination of price of any product or factors of production

4 Itrsquos scope is narrow and restricted to individual unit

1It studies the economy as a whole

2It deals with the national income aggregate consumption and aggregate savings3 It facilitates determination of general price level in an economy

4 Itrsquos scope is wide as it deals with economic units on the national level

ACCOUNTS

Introduction to Accounting and Book-keeping

Today I am going to share you the meaning of Accounting and Book-keeping and its related terms bullAccounting bullBook Keeping bullAccountsbullTypes Of Accounts bullAccounting Cycle

bull Meaning of accounting

Ans ) Accounting is the art and science of recording classifying and summarising monetary transactions

bull Meaning of Book-keeping

Ans) Bookkeeping is the art of recording business transactions with the view of having a permanent record of them and showing their effect on wealth

bull Meaning of account

Ans) The term account means a record of

business transactions concern a particular person of firm asset or income or expense It is a summarised record of all transactions which take place in an accounting year

bull Types of accountsPersonal accounts ndash Personal accounts relating

to person and Organisation are known as personal accounts Example Ramrsquos Account ABC amp Co Account etc

Real account - The accounts related to tangible and intangible assets are called real accounts Example Cash Account Furniture Account etc

Nominal account- Accounts related to expenses losses incomes and gains are known as nominal accounts Example Wages Account Salary Account Discount Account etc

bull Accounting cycle Accounting cycle refers to a complete sequence of accounting activities It begins with recording of transactions and ends with the preparation of a balance sheet

Chemistry TopicAtomic Structure

Thomsonrsquos atomic modelThomson (1898) was the first to propose the model of an atomHe proposed that an atom can be regarded as a uniform sphere of positive electricity in which requisite number of electrons are embedded evently to neutralize the positive chargeThis is just like plums embedded in a pudding or seeds evently distributed in red spongy mass of a watermelonThis model of atom is known as ldquoPlum-Pudding modelrdquo or

Q1)What is the fundamental constituents of atomAns Electron Proton and neutrons are the fundamental constituents of atomQ2)What is the value of fundamental unit of electricityAnsThe charge carried by one electron is sad to be the fundamental unit of electricityIts magnitude is 48times10-10esuOr 1602times10-19C Q3)Name the element containing no neutronAnsOrdinary hydrogen atom or protium 1H1

Types of AccountPersonal AccountReal AccountNominal AccountBalance Sheet (opening)

ldquowatermelon modelrdquoThis model could explain the electrical neutrality of an atom but failed to explain the result of scattering experiment carried out by Rutherford in 1911So it was rejected ultimately

Q4)Why is an electron called universal particleAns Itrsquos mass and Charge are independent of its source

EVS Chapter 1 ndash Modes of Existence

Modes of existence When one speaks normally about the mode of existence of some group or individual one refers to their customs their mode of being their ethology their habitat in some way their feeling for a placeDifferent modes of exixtence are ndash

1 Hunting ndashGathering2 Pastoral3 Agricultural4 Industrial

1 Hunting and gathering Hunting and gathering mode of existence is characterized by obtaining food from hunting wild animals including fishing and gathering wild plants From their earliest days the hunter-gatherer diet included various grasses tubers fruits seeds and nuts Lacking the means to kill larger animals they procured meat from smaller game or through scavenging

Societies that rely primarily or exclusively on hunting wild animals fishing and gathering wild fruits berries nuts and vegetables to support their diet are called hunting and gathering societies

At least this used to be practice of human beings before agriculture is invented As their brains evolved hominids developed more intricate knowledge of edible plant life and growth cycles

Q) Write the features of Hunting ndash gathering societiesAns - There are five basic characteristics of hunting and gathering societies

i The primary institution is the family which decides how food is to be shared and how children are to be socialized and which provides for the protection of its members

ii They tend to be small with fewer than fifty members

iii They tend to be nomadic moving to new areas when the current food supply in a given area has been exhausted

iv Members display a high level of interdependence

v Labor division is based on sex men hunt and women gather

Political Science

Introduction to political science

Political science occasionally called politology is a social science which deals with systems of governance and the analysis of political activities political thoughts associated constitutions and political behaviorThe study of political science involves the study of both the

Answer the following questions-1 What is political science

Political science occasionally called politology is a social science which deals with systems of governance and the analysis of political activities political thoughts associated constitutions and political behavior

2 Short notes-

traditional and modern theories of politicsTraditionalClassical political sciencepolitical theory-Traditional political science is the study of politics before Second World War The methodology to study Politics was traditional (legal formaletc) the definition of politics traditional (Politics begins and end with state)area of study (constitution state machinery)was traditionalModern Political scienceModern political theory-Modern Political Theory critically examines the contemporary state of political theory making an assessment of the achievement and limitations of the Behavioural Revolution in its totality and reviews objectively the major paradigms and conceptual frameworks adopted by the disciplineContemporary attempts at the development of an integrated political theory involving the use of both traditional and modern concepts approaches and theories-Around late 1960s several political scientists realized the importance of both the traditional political theory and modern Political theory They began building an integrated theory of politics involving a systematic mixture of traditional and modern studies of politics It was held that the study of a complex and vast field like politics needs both traditional as well as

Classical political theory Modern Political theory

Homework-Learn

modern concepts and approaches for studying itrsquos all aspects

Subject Eng Literature (The Tempest ndash William Shakespeare) Topic Act I Scene 1 Lines 1 to 32 (Line 32 ndash Gonzalo hellip If he be not born to be hanged our case is miserable) Date 13th April 2020 (3rd Period)

[Students should read the original play and also the paraphrase given in the school prescribed textbook]Summary Questions amp Answers

[SUMMARY OF THE ENTIRE SCENE]

o The play starts with the scene of a severe storm at sea Alonso (King of Naples) Sebastian (Alonsorsquos brother) Ferdinand (Alonsorsquos son) Gonzalo Antonio (the usurping Duke of Milan) are in a ship in the midst of the storm

o The mariners are trying their best to control the vessel from running aground and are totally following the orders of their Master the Boatswain They have scant success

o The mariners become extremely unhappy and annoyed when most of the passengers arrive on the deck thereby hampering their effort to save the ship There is serious confrontation between them and the passengers who are part of the Kingrsquos entourage

o The mariners could not save the ship

SUMMING-UP

(i) Vivid description of the scene which gives a realistic description of terror and confusion of a tropical storm

(ii) Shows Shakespearersquos accuracy of knowledge in describing the naval operations and also matters of seamanship

(iii) The opening scene justifies the title ndash The Tempest

UNANSWERED QUESTIONS

(i) The King always travels with his entire fleet including his soldiers Where

(1) GONZALO Nay good be patient (Line 15-26)BOATSWAIN When the sea is Hence What cares these

roarers for the name of the king To cabin silence Trouble us not

GONZALO Good yet remember whom thou has aboardBOATSWAIN None that I more love than myself You are a

councillor if you can command these elements to silence and work

the peace of the present we will not hand a rope more use your authority If you cannot give thanks you have

lived so long and make yourself ready in your cabin for the mischance of the hour if it so hap [To the Mariners]

Cheerly good hearts [To Gonzalo] Out of our way I say

(a) To whom is the boatswain speaking What does he mean by lsquoNone that I more love than myselfrsquo

The Boatswain is speaking to Gonzalo the honest old councilor of the Duke of MilanBy using the words ndash lsquoNone that I love more than I love myselfrsquo means that for the Boatswain nobody is dearer to him than his own life

(b) What were the conditions that made the boatswain react in this way

The Boatswain reacts in this way because the storm is at sea and Alonso King of Naples Sebastian his brother Ferdinand his son Gonzalo Antonio the usurping Duke of Milan on board are in distress and in panic Thus they have rushed to the deck interrupting the work of the mariners

(c) What hope does Gonzalo take from the attitude of the boatswain

The insolent and authoritative attitude of Boatswain makes Gonzalo feel comforted He tells that there are no signs that the Boatswain will be drowned But his facial appearance and attitude shows that he is destined to die on land by hanging which in effect means that all on board will be saved Otherwise all the persons on board are doomed

(d) How can they lsquomake yourself ready in your cabinrsquo For what were they asked to make ready themselves

In order to make themselves ready in their cabin the

were the other ships

(ii) Why was the ship in that area Where was it coming from or going where

(iii) The ship broke apart What happened to those who were in the ship

passengers on board must prepare for death which they will possibly soon have to meetThey can retire to their cabins and offer prayers to the Almighty to save them from drowning

(e) What does the boatswain say when he is asked to be patient What does he order to the royal party

When the boatswain is asked to be patient and remain calm he says that he will be patient only when the storm will be over and the sea will be calm but as long as the storm blows and there is danger to the ship he cannot think of being patient He orders the royal party to go to the cabin and leave the mariners to their work

(2) GONZALO I have great comfort from this fellow (Line 27-36)

Methinks he hath no drowning mark upon him his complexion is perfect

gallows Stand fast good Fate to his hanging Make the rope of his destiny our cable for our own doth little advantage If he be not born to be hanged our case is miserable

(a) Why does Gonzalo regard the Boatswain in the midst of danger

In the midst of danger Gonzalo regards the boatswain because he feels that the Boatswain is a source of comfort and is bent upon to do his work sincerely which in this case is saving the ship and its passengers from the severest of raging storm

(b) What reasons does Gonzalo give when he says that none in the ship will die of drowning

Gonzalo is almost sure that none in the ship will die by drowning His says that there is no mark on the face of the boatswain that indicates that he will die by drowning On the other hand the lines on his face are strong indications that he will be hanged to death Therefore there shall be no danger of the shiprsquos sinking

(c) Explain the following ldquoStand fast good Fate to his hanging Make the rope of his destiny our cable for our own doth little advantage If he be not born to be hanged our case is miserablerdquo

The stated lines mean that if the will of destiny is to be carried out then the ship will not get wrecked and all the passengers will be saved The safety of the passengers therefore depends upon the will of fate being carried out in the case of the boatswain If however the boatswain is not to die by hanging then the passengers are also very unsafe because in that case the ship is likely to sink

(d) What order does the Boatswain give to the sailors

when he re-enters What does he say about the crying of the fellows inside the cabin

The boatswain orders the sailors to bring the topmast lower and bring the ship close to a stationary position with the help of the main sail He says that the fellows inside the cabin are moaning and crying in their distress louder than his voice and louder even than the roaring of the storm

Class XII (ScienceCommerceHumanities) Subject Topic Summary Execution

Computer Science

PropositionalLogic

Propositional logic is a procedure to provide reasoning through statementProposition A ststement that results in True or False is said to be proposition There are two types of propositionSimple proposition amp compound propositionSimple proposioton A simple proposition is one that is not a part of any other proposition Such sentential form of proposition is symbolized with english letters in short For example Ram is a claver student (TrueFalse)Where do you live (Not in True or False)Grapes are sweet (TrueFalse)It rains today (TrueFalse)Here we can see some statements anwer would be true or false but some staements answer can not give in terms of true or false Thus the sentences which can be answered in true or false are known as simple propositionAssigning propositon to a variableThe general syntax to assign propostion to a variable is as followsVariable = Simple propositonFor example A=Ram is a clever studentB= Grapes are sweetC= it rains todayCompound proposition

helliphellipto be continued in next classhelliphellipMath Relation Relation If A and B are two non-empty sets

then a relation R from A to B is a subset of AxB If R A x B and (a b) R then we say that a sube isinis related to b by the relation R written as aRbeg Let A be the set of students of class XII and B be the set of students of class XI Then some of the examples of relation from A to B arei) (a b) AXB a is brother of bisinii) (a b) AXB age of a is more than age of isinb Types of relation In this section we would like to study different types of relations We know that a relation in a set A is a subset of A times A Thus the empty set φ and A times A are two extreme relations For illustration consider a relation R in the set A = 1 2 3 4 given by R = (a b) a ndash b = 10 This is the empty set as no pair (a b) satisfies the condition a ndash b = 10 Similarly R = (a b) | a ndash b | ge 0 is the whole primeset A times A as all pairs (a b) in A times A satisfy | a ndash

Example 1 Let A be the set of all students of a boys school Show that the relation R in A given by R = (a b) a is sister of b is the empty relation and R = (a b) the primedifference between heights of a and b is less than 3 meters is the universal relationSolution Since the school is boys school no student of the school can be sister of any student of the school Hence R = φ showing that R is the empty relation It is also obvious that the difference between heights of any two students of the school has to be less than 3 meters This shows that R = A times A is primethe universal relation Example 2 Show that the relation R in the set 1 2 3 given by R = (1 1) (2 2) (3 3) (1 2) (2 3) is reflexive

b | ge 0 These two extreme examples lead us to the following definitionsDefinition 1 A relation R in a set A is called empty relation if no element of A isrelated to any element of A ie R = φ A times AsubDefinition 2 A relation R in a set A is called universal relation if each element of A is related to every element of A ie R = A times A Both the empty relation and the universal relation are some times called trivial relation Definition 3 A relation R in a set A is called(i) reflexive if (a a) R for every a Aisin isin(ii) symmetric if (a1 a2) R implies that (aisin 2a1)

R for all aisin 1 a2 Aisin(iii) transitive if (a1 a2) R and (aisin 2 a3) R isinimplies that (a1 a3) R for all aisin 1 a2 a3 AisinDefinition 4 A relation R in a set A is said to be an equivalence relation if R is reflexive symmetric and transitive

but neither symmetric nor transitiveSolution R is reflexive since (1 1) (2 2) and (3 3) lie in R Also R is not symmetric as (1 2) R but (2 1) isin notinR Similarly R is not transitive as (1 2) R and (2 3) R but (1 3) R isin isin notinExample 3 Show that the relation R in the set Z of integers given byR = (a b) 2 divides a ndash b is an equivalence relationSolution R is reflexive as 2 divides (a ndash a) for all a Z isinFurther if (a b) R then 2 divides a isinndash b Therefore 2 divides b ndash a Hence (b a) R which shows that R is isinsymmetric Similarly if (a b) R and (b c) R isin isinthen a ndash b and b ndash c are divisible by 2 Now a ndash c = (a ndash b) + (b ndash c) is even (Why) So (a ndash c) is divisible by 2 This shows that R is transitive Thus R is an equivalence relation in ZExample 4 Let L be the set of all lines in a plane and R be the relation in L defined as R = (L1 L2) L1 is perpendicular to L2 Show that R is symmetric but neither reflexive nor transitiveSolution R is not reflexive as a line L1 can not be perpendicular to itself ie (L1 L1) R notinR is symmetric as (L1 L2) Risin

L1 is perpendicular to L2rArr L2 is perpendicular to L1rArr (L2 L1) RrArr isin

R is not transitive Indeed if L1 is perpendicular to L2 and L2 is perpendicular to L3 then L1 can never be perpendicular to L3 In fact L1 is parallel to L3 ie (L1 L2) R isin(L2 L3) R but (L1 L3) Risin notin

Chemistry Solid state Characteristics if Solids(i)The particles are locked in fixed positions they are unable to change their relative positions and this brings a definite shape and volume of a solid(ii)In a solid the constituent particles are held by strong forces of attractionThe forces of attraction may be bonding or non bonding(iii)The constituent particles in a solid pack together as closely as possibleoccupying most of the available space within the solidThus the empty space in a solid is very smallThis makes a solid highly rigid and nearly incompressibleThis also explains why a solid has high density and exhibits slow diffusionClassification of Solids

Q1)Define Crystalline solids AnsA Solid that has a definite geometrical shape and a sharp melting pointand whose constituent particles (atomsmolecules or ions) are arranged in a long range order of definite pattern extending throughout the solid is called a crystalline solidExNaClQ2)Define Amorphous solids AnsA solid that does not have a definite shape and a sharp melting pointand whose constituent particles (atomsmolecules or ions) are not arranged in a definite pattern is called an amorphoussolid

Crystalline solidsAmorphous solids

ExGlassRubberQ3)Classify Crystalline Solids Crystalline Solids

Physics Coloumbrsquos Law (Summary)

Before Going Into Coloumbrsquos Law We Will First Learn What is Charge Properties of Charge and Always remember that charge is quantized ie a body always have static charge of magnitude equal to some integral multiple of fundamental electronic charge e= 16 x 10- 19 C

Charge is the property of matter that causes it to produce and experience electrical and magnetic effects The study of the electrical charges at rest is called electrostatics When both electrical and magnetic effects are present the interaction between charges is referred to as electromagnetic

There exist two types of charges in nature positive and negative Like charges repel and unlike charges attract each other

The type of charge on an electron is negative The charge of a proton is the same as that of an electron but with a positive sign In an atom the number of electrons and the number of protons are equal The atom is therefore electrically neutral If one or more electrons are added to it it becomes negatively charged and is designated as negative ion However if one or more electrons are removed from an atom it becomes positively charged and is called a positive ion

The excess or deficiency of electrons in a body gives the concept of charge If there is an excess of electrons in a body it is negatively charged And if there is deficiency of electrons the body becomes positively charged Whenever addition or removal of electrons takes places the body acquires a charge

The SI Unit of charge is coulomb (C) In SI units the current is a fundamental quantity having a unit of ampere (A) The unit of charge is defined in terms of the unit of current Thus one coulomb is the charge transferred in one second across the section of a wire carrying a

Ionic SolidsMetallicSolids

Molecular Solids

current of one ampere

As q = It we have1 C = (1 A) (1 s)

The dimensions of charge are [A T]

Properties of Charge

(1) Quantization of Charge Electric charge can have only discrete values rather than any value That is charge is quantized The smallest discrete value of charge that can exist in nature is the charge on an electron given as

e = plusmn 16 x 10- 19 C

This is the charge attained by an electron and a protonA charge q must be an integral multiple of this basic unit That is

Q = plusmn ne where n = 1 2 hellip

Charge on a body can never be (frac12)e (23)e or 57e etcWhen we rub a glass rod with silk some electrons are transferred from the rod to the silk The rod becomes positively charged The silk becomes negatively charged The coulomb is a very large amount of charge A typical charge acquired by a rubbed body is 10 - 8 C

Biology Reproduction in organisms

Welcome to this new session 2020-21Today in this first chapter we mainly discuss about reproduction types needs and life span of some organismsWe also discuss about difference between sexual and asexual reproduction

Q1 What is reproductionReproduction is defined as a biological processin which an organism gives rise to young onessimilar to itselfQ2 What are the needs of reproductionbulli) Reproduction maintain life on earthii) It enables the continuity of the species generation after generationiii) It creates genetic variation among populationsQ3 Define Life span and write some orgnisms life spanbull Life span is the period from birth to

the natural death of an organism- OrganismsLife span1 Butterfly 1 - 2 weeks2 Fruit fly 30 days3Dog 10-13 years4 Rose5-7 years5 Tortoise100-150 years6 Banyan Tree -200 - 250 yearsQ4 Reproduction is of two types in case ofanimals but in case of plants vegetative propagation is also present

Asexual Reproduction Sexual Reproductioni) Always uniparentalii) Gametes are not involvediii) Only mitotic division involvediv) Somatic cells of parents are involvedv) Offsprings are genetically similar to the parents

i) Usually biparentalii) Gametes are involvediii) Meiosis occurs during gametogenesis Mitosis occurs after fertilisationiv) Germ cells of the parents are involvedv) offsprings are genetically different from the parents

COMMERCE BUSINESS ENVIRONMENT

Welcome to the new sessiontoday we are going to start the first chapter of Class XII The name of the chapter is Business Environment

Already many of you have got some idea about the word business environment form the first chapter of business studies in class XI

In todayrsquos world every business enterprise is a part of the society It exists and operates in association with various groups in society such as customers suppliers competitors banks and financial institutions government agencies trade unions media and so on All these groups influence the functioning of business in one way or the other They constitute the environment of businessConcept of Business Environment

The term lsquobusiness environmentrsquo refers to the sum total of all individuals institutions and other forces that lie outside a business enterprise but that may influence its functioning and performance

The main features of business environment Totality of External forces General and Specific forces Interrelatedness Complexity Dynamic Uncertainty

Prepare the following questions from todayrsquos assignment

2 What do you mean by business environment

The term lsquobusiness environmentrsquo means the aggregate of all forces factors and institutions which are external to and beyond the control of an individual business enterprise but they may influence its functioning and performance Business environment is the macro framework within which a business firm a micro unit operates It consists of several interrelated and interacting elements

2 Explain the main features of business environment in brief

Totality of External forces-Business environment is the sum total of all things external to a business environment

General and Specific forces-It

Relativity

The Interrelation between business and its environment

The business enterprise is an open system It continuously interacts with its environment It takes inputs (such as raw materials capital labour energy and so on) from its environment transforms them into goods and services and sends them back to the environment

Fig 1 Business Environment Relationship

includes both the forces general forces are the economic social political legal and technological conditions which indirectly influence all business enterprise Specific forces are the investors customers competitors and suppliers which influence individual enterprise directly

Interrelatedness-Different elements of environment are interrelated for an example growing awareness for health care has increased the demand for health foods

Complexity- Business environment id complex in nature as the elements keep on changing example economic technological and other forces changes in demand for a product and service

Dynamic-Business environment is not static it keeps on changing

Uncertainty- Itrsquos very difficult to predict future events such as technology and fashion which occur fast and frequently

Business Studies

Human Resources Management

Human resource of an organisation are the aggregate of knowledge skills attitudes of people working in it

The management system which deals with human resources is called human resource management

Features of HRMbullComprehensive functionbullPeople-oriented

Question1) What do you mean by human

resource management Answer) Human resource management may be defined as that field of Management which has to do with planning organising and controlling the functions of procuring developing maintaining and utilising the labour force

bullAction oriented bullPervasive function bullContinuous function

2) Explain the features of HRM in brief

Answer)bullHuman Resource Management is concerned with managing people at work bull Human Resource Management is concerned with employees which bring people and organisations together so that the goals of each are met bullHuman resource management considered every employees as an individual and also promote their satisfaction and growth bull Human resource management is inherent in all organisations and at all levelsbullManagement of human resources are ongoing on never ending process which requires a constant alertness and Awareness of human relations

3) ldquoHR function is said to be pervasiverdquowhy

Answer) Human resource management is required in all organisations whether it is private or government organisations armed forces sports organisations etc It permeatsall the functional areas like production marketing finance research etc This from this feature of human resource management it can be said that it is pervasive in nature

Economics Demand Q1DEFINITION OF DEMANDIn economics demand is the quantity of a good that consumers are willing and able to purchase at various prices during a given period of timeQ2DEMAND CURVEIn economics a demand curve is a graph depicting the relationship between the price of a certain commodity and the quantity of that commodity that is demanded at that pricQ3LAW OF DEMANDIn microeconomics the law of demand states that conditional on all else being equal as the price of a good increases quantity demanded decreases conversely as the price of a good decreases quantity demanded increasesQ4ASSUMPTION of LAW OF DEMAND(i)No change in price of related commodities(ii) No change in income of the consumer(iii) No change in taste and preferences customs habit and fashion of the consumer( No expectation regarding future change in priceQ5MARKET DEMAND SCHEDULEIn economics a market demand schedule is a tabulation of the quantity of a good that all consumers in a market will purchase at a

given price At any given price the corresponding value on the demand schedule is the sum of all consumersrsquo quantities demanded at that priceQ6INDIVIDUAL DEMAND SCHEDULEIndividual demand schedule refers to a tabular statement showing various quantities of a commodity that a consumer is willing to buy at various levels of price during a given period of timeQ7 FACTORS AFFECTING INDIVIDUAL DEMAND FOR A COMMODITY

The factors that influence a consumerrsquos decision to purchase a commodity are also known as determinants of demand The following factors affect the individual demand for a commodity1 price of the commodity2 price of related goods3 income of buyer of the commodity4 tastes and preferences of the buyer1 Price of the CommodityYou must have observed that when price of a commodity falls you tend to buy more of it and when its price rises you tend to buy less of it when all other factors remain constant (lsquoother things remaining the samersquo) In other words other things remaining the same there is an inverse relationship between the price of a commodity and its quantity demanded by its buyers This statement is in accordance with law of demand which you will study in the later part of this lesson Price of a commodity and its quantity demanded by its buyers are inversely related only when lsquoother things remain the samersquo So lsquoother things remaining the samersquo is an assumption when we study the effect of changes in the price of a commodity on its quantity demanded2 Price of Related goodsA consumer may demand a particular good But while buying that good heshe also asks the price of its related goods Related goods can be of two types-(i) Substitute goods(ii) Complementary goods While purchasing a good prices of its substitutes and complements do affect its quantity purchased(i) Price of Substitute Goods Substitute goods are those goods which can easily be used in place of one another for satisfaction of a particular want like tea and coffee An increase in price of substitute good leads to an increase in demand for the given commodity and a decrease in price of substitute good leads to a decrease in demand for the given commodity It means demand for a given commodity is directly affected by change in price of substitute goods For example if price of coffee increases the demand for tea will rise as tea will become relatively cheaper in comparison to coffee(ii) Price of Complementary goods Complementary goods are those goods which are used together to satisfy a particular want like car and petrol An increase in the price of complementary goods leads to a decrease in demand for the given commodity and a decrease in the price of complementary goods leads to an increase in demand for the given commodity For example if price of petrol falls then the demand for cars will increase as it will be relatively cheaper to use both the goods together So demand for a given commodity is inversely affected by change in price of complementary goods3 Income of the Buyer of CommodityDemand for a commodity is also affected by income of its buyer However the effect of change in income on demand depends on the nature of the commodity under consideration In case of some goods like full cream milk fine quality of rice (Basmati rice) etc demand for these commodities increases when income of the buyer increases and

demand for these commodities decreases when income of the buyer decreases Such goods whose demand increases with the increase in income of the buyer are called normal goods But there are some goods like coarse rice toned milk etc whose demand decreases when income of buyer increases and their demand increases when income of the buyer decreases Such goods whose demand decreases with the increase in income of the buyer are called inferior goods Suppose a consumer buys 10 Kgs of rice whose price is ` 25 per Kg He cannot afford to buy better quality of rice because the price of such rice is ` 50 per Kg The consumer is spending ` 250 per month on the purchase of rice Now if income of the consumer increases and he can afford ` 350 on purchase of 10 Kg of rice Now he can afford to buy some quantity of rice say 6 Kgs whose price is ` 25 per Kg and may buy 4 Kgs of rice whose price is ` 50 per Kg Thus he will buy 10 Kgs of rice by spending ` 350 per month Therefore we may conclude that demand for normal goods is directly related to the income of the buyer but demand for inferior goods is inversely related to the income of the buyer4 Tastes and Preferences of the BuyerThe demand for a commodity is also affected by the tastes and preferences of the buyers They include change in fashion customs habits etc Those commodities are preferred by the consumers which are in fashion So demand for those commodities rises which are in fashion On the other hand if a commodity goes out of the fashion its demand falls because no consumer will like to buy it(5) Number of Buyers in the Market(Population)Increase in population raises the market demand whereas decrease in population reduces the market demand for a commodity Not only the size of population but its composition like age (ratio of males females children and old people in population) also affects the demand for a commodity It is because of needs of children young old male and female population differs(6) Distribution of Income and WealthIf the distribution of income and wealth is more in favour of the rich demand for the commodities preferred by the rich such as comforts and luxuries is likely to be higher On the other hand if the distribution of income and wealth is more in favour of poor demand for commodities preferred by the poor such as necessities will be more(7) Season and Weather ConditionsThis is generally observed that the demand for woolens increases during winter whereas demand for ice creams and cold drinks increases during summer Similarly market demand for umbrellas rain coats increases during rainy seasonQ8 REASONS FOR OPERATION OF LAW OF DEMAND WHY DEMAND CURVE SLOPES DOWNWARDNow we will try to explain why does a consumer purchase more quantity of a commodity at a lower price and less of it at a higher price or why does the law of demand operate ie why does the demand curve slope downwards from left to right The main reasons for operation of law of demand are1 Law of Diminishing Marginal UtilityAs you have studied earlier law of diminishing marginal utility states that as we consume more and more units of a commodity the utility derived from each successive unit goes on decreasing The consumer will be ready to pay more for those units which provide him more utility and less for those which provide him less utility It implies that he will purchase more only when the price of the commodity falls2 Income Effect

When price of a commodity falls purchasing power or real income of the consumer increases which enables him to purchase more quantity of the commodity with the same money income Let us take an example Suppose you buy 4 ice creams when price of each ice cream is ` 25 If price of ice creams falls to ` 20 then with same money income you can buy 5 ice creams now3 Substitution EffectWhen price of a commodity falls it becomes comparatively cheaper as compared to its substitutes (although price of substitutes has not been changed) This will lead to rise in demand for the given commodity For example if coke and Pepsi both are sold at ` 10 each and price of coke falls Now coke has become relatively cheaper and will be substituted for Pepsi It will lead to rise in demand for coke4 Change in Number of BuyersWhen price of a commodity falls some old buyers may demand more of the commodity at the reduced price and some new buyers may also start buying this commodity who were not in a position to buy it earlier due to higher price This will lead to increase in number of buyers when price of the commodity falls As a result demand for the commodity rises when its price falls5 Diverse Uses of a CommoditySome commodities have diverse uses like milk It can be used for drinking for sweet preparation for ice cream preparation etc If price of milk rises its use may be restricted to important purpose only This will lead to reduction in demand for other less important uses When price of milk falls it can be put to other uses also leading to rise n demand for itQ9 EXCEPTIONS TO THE LAW OF DEMANDYou have studied in law of demand that a buyer is willing to buy more quantity of a commodity at a lower price and less of it at a higher price But in certain circumstances a rise in price may lead to rise in demand These circumstances are called Exceptions to the Law of Demand Some important exceptions are1 Giffen GoodsGiffen goods are special type of inferior goods in which negative income effect is stronger than negative substitution effect Giffen goods do not follow law of demand as their demand rises when their price rises Examples of Giffen goods are jowar and bajra etc2 Status Symbol GoodsSome goods are used by rich people as status symbols eg diamonds gold jewellary etc The higher the price the higher will be the demand for these goods When price of such goods falls these goods are no longer looked at as status symbol goods and tehrefore therir demand falls3 NecessitiesCommodities such as medicines salt wheat etc do not follow law of demandbecause we have to purchase them in minimum required quantity whatever their price may be4 Goods Expected to be ScarceWhen the buyers expect a scarcity of a particular good in near future they start buying more and more of that good even if their prices are rising For example during war famines etc people tend to buy more of some goods even at higher prices due to fear of their scarcity in near future

Political Science

Constitution of India-The

Preamble

The preamble-

Preamble-

The preamble is the most precious part of the constitution We the people of India having solemnly resolved to constitute India into a Sovereign Socialist Secular Democratic Republic and to secure to all its citizensA preamble is an introductory and expressionary statement in a document that explains the documents purpose and underlying philosophy When applied to the opening paragraphs of a statute it may recite historical facts pertinent to the subject of the statuteNature and purpose of the constitution-Purpose of the Constitution dictates permanent framework of the government to form a more perfect union to establish justice and ensure peace of thenationconstitution provide principles how the government can run itself following the rules and laws written in the constitution of each state keeps them balanced

Answer the following questions-

1 What is preambleA preamble is an introductory and expressionary statement in a document that explains the documents purpose and underlying philosophy2 What is the nature and

purpose of the constitutionConstitution dictatespermanent framework of the government to form a more perfect union to establish justice and ensure peace of the nation

Homework-Learn

Accounts Compatibilty mode

1MEANING OF PARTNERSHIPPartnership is a form of business organisation where two or more persons join hands to run a business They share the profits and losses according to the agreement amongst them According to the Indian Partnership Act 1932 ldquoPartnership is relation between persons who have agreed to share profits of a business carried on by all or any one of them acting for allrdquo For example one of your friends has passed class XII from National Institute of Open Schooling (NIOS) and wants to start a business Heshe approaches you to join in this venture Heshe wants you to contribute some money and participate in the business activities Both of you if join hands constitute a partnership2CHARACTERISTICS1048698 Agreement A partnership is formed by an agreement The agreement may be either oral or in writing It defines the relationship between the persons who agree to carry on business It may contain the terms of sharing profit and the capital to be invested by each partner etc The written agreement is known as partnership deed1048698 Number of persons There must be at least two persons to form a partnership

The maximum number of partners in a partnership firm can be 50 according toCompanies Act 20131048698 Business The Partnership is formed to carry on business with a purpose of earning profits The business should be lawful Thus if two or more persons agree to carry on unlawful activities it will not be termed as partnership1048698 Sharing Profits The partners agree to share profits in the agreed ratio In caseof loss all the partners have to bear it in the same agreed profit sharing ratio10486981048698Mutual Agency Every partner is an agent of the other partners Every partner can bind the firm and all other partners by hisher acts Each partner will be responsible and liable for the acts of all other partners10486981048698Unlimited liability The liability of each partner except that of a minor is unlimited Their liability extends to their personal assets also If the assets of the firm are insufficient to pay off its debts the partnersrsquo personal property can be used to satisfy the claim of the creditors of the partnership firm10486981048698Management All the partners have a right to mange the business However they may authorize one or more partners to manage the affairs of the business on their behalf10486981048698Transferability of Share No partner can transfer hisher share to any one including hisher family member without the consent of all other partners3PARTNERSHIP DEEDAgreement forms the basis of partnership The written form of the agreement is which a document of partnership is It contains terms and conditions regarding the conduct of the business It also explains relationship between the partners This document is called partnership deed Every firm can frame its own partnership deed in which the rights duties and liabilities of the partners are stated in detail It helps in settling the disputes arising among the partners during the general conduct of business 4CONTENTS OF PARTNERSHIP DEEDThe partnership deed generally contains the following (i) Name and address of the partnership firm(ii) Nature and objectives of the business(iii) Name and address of each partner(iv) Ratio in which profits is to be shared(v) Capital contribution by each partner(vi) Rate of Interest on capital if allowed(vii) Salary or any other remuneration to partners if allowed(viii) Rate of interest on loans and advances by a partner to the firm(ix) Drawings of partners and interest thereon if any(x) Method of valuation of goodwill and revaluation of assets and liabilities on the reconstitution of the partnership ie on the admission retirement or death of a partner(xi) Settlement of disputes by arbitration(xii) Settlement of accounts at the time of retirement or death of a partner5IN ABSENCE OF PARTNERSHIP DEEDThe partnership deed lays down the terms and conditions of partnership in regard to rights duties and obligations of the partners In the absence of partnership deed there may arise a controversy on certain issues like profit sharing ratio interest on

capital interest on drawings interest on loan and salary of the partners In such cases the provisions of the Indian Partnership Act becomes applicableSome of the Issues are(i) Distribution of Profit Partners are entitled to share profits equally(ii) Interest on Capital Interest on capital is not allowed(iii) Interest on Drawings No interest on drawing of the partners is to be charged(iv) Interest on Partnerrsquos Loan A Partner is allowed interest 6 per annum on the amount of loan given to the firm by himher(v) Salary and Commission to Partner A partner is not entitled to anysalary or commission or any other remuneration for managing the business

History TOPIC-TOWARDS INDEPENDENCE AND PARTITION THE LAST PHASE (1935-1947)

SUB TOPIC-IMPORTANT POLITICAL DEVELOPMENTS ndash GROWTH OF SOCIAL IDEAS

Socialism is a political social and economic philosophyLike in other parts of the world the Russian revolution of 1917 served as a great inspiration for revolutionaries in India who at that time were engaged in the struggle for liberation from British ruleSocialist ideas led to the formation of communist party of IndiaJAWAHARLAL NEHRU Among the early Congress leaders Jawaharlal Nehru was very much impressed and influenced by the Socialist ideas He also learnt about the Economic activities of the Soviet Union after the Bolshevic Revolution 1917 He made full use of them in IndiaThe election of Jawaharlal Nehru and Subhas Chandra Bose showed the Left wing tendency within CongressJawaharlal Nehru demanded economic freedom along with political freedom of the people in order to end the exploitation of masses

Nehrus working committee included three socialists leaders The Lucknow session was a landmark in the evolution of socialist ideas of the congressSUBHAS CHANDRA BOSE ndash Subhas Chandra Bose had socialist leaning Both Jawaharlal Nehru and Subhas Chandra Bose were known as leftist Congress men Later on National Congress divided into Leftist and rightist campCONGRESS SOCIALIST Within the Congress some leaders formed the Congress Socialist partyPattavi Sitaramyya Sardar Patel Rajendra Prasad had hostile attitude towards the Congress Socialist partyJawaharlals attitude was hesitant

1 QUESTION ndash Mention name of two Congress leaders who had socialist leaning

1ANSWER ndash Subhas Chandra Bose and Jawaharlal Nehru2QUESTION- In which session of the congress Jawaharlal elaborated his Socialist ideas2 ANSWER ndash Lucknow and Faizpur Session in December 1935 and 19363QUESTION ndash Why Congress was sharply divided into leftist and rightist camp 3ANSWER ndash Subhas Chandra Bosersquos attempt to seek re election for congress presidentship in 1939sharply divided the National Congress into Leftist and Rightist camp4 QUESTION ndash Who was MN Roy 4 ANSWER ndash Manabendra Roy first formed the Communist Party of India outside the country at Tashkent in 19205QUESTION ndash Who formed the Congress Socialist Party within the Congress5 ANSWER ndash Jaya Prakash Narayan Achyut Patwardhan Acharya Narendra Dev Ram Mohan Lohia Aruna Asaf Ali6QUESTION ndash When was the Congress Socialist Party formed What was its object6 ANSWER ndash 1934The Congress Socialist Party sought to work out socialist programme through the Congress They joined hands with the Congress and wanted to carry

Subhas Chandra Bose being expelled from the congress after the Tripuri rift he formed Forward BlockThere were basic differences between the Congress Socialists and the communistsTRADE UNION ACTIVITIES Maximum working class people lived in Bombay and Calcutta The working and living conditions of those workers were very miserable In this situation Shasipada Banerjee NM Lokhande protested against the oppression of the working class peopleThe first Trade Union Madras Labour Union was formed in 1918 by BP WadiaIndustrial strikes took place in Kanpur Calcutta Madras Jamshedpur and Ahmedabad AITUC was formed in Bombay in 1927 The growth of Trade union among the workers was slow because of the fear of the dismissal of the jobIn the mean time the Moderates as well as Communists left AITUC and formed separate organization

on National struggle with the help of workers and peasant class of the society7 QUESTION ndash What was the name of the party founded by Subhas Chandra Bose7 ANSWER- Forward Block8QUESTION ndash Who was Shasipada Banerjee8 ANSWER ndash Shasipada Banerjee was a radical Brahmo He founded a working menrsquos club to protest against exploitation of the British rulers towards the working class of India9 QUESTION ndash What was the weekly published by NM Lokhande9ANSWER- Dinabandhu10 QUESTION ndash Who founded Bombay Mill-Hands Association and in which year10 ANSWER- NM Lokhande in189011 QUESTION- Who was BP WadiaANSWER- BPWadia was the founder of Madras Labour Union in191812 QUESTION- What was the name of the first labour union of India12 ANSWER- Madras Labour Union13 QUESTION Who founded the Majur Mahajan 13 ANSWER GANDHIJI14 QUESTION What was the full form of AITUC When it was formed14 ANSWER All India Trade Union Congressin 192715QUESTION Who formed the Red Trade Union Congress and in which year15ANSWER The Communists formed the Red Trade Union Congress16 QUESTION What do you mean by Socialism16 ANSWER Socialism describes any political and economic theory that says the community rather than individuals should own and manage property and natural resources

Subject Eng Literature (The Tempest ndash William Shakespeare) Topic Act III Scene 3 Lines 1 to 52 (Line 52 ndash Brother my lord the Duke Stand to and do as we) Date 13th April 2020 (4th Period)

[Students should read the original play and also the paraphrase given in the school prescribed textbook]Summary Questions amp Answers

o Alonso Sebastian Antonio Gonzalo Adrian Francisco and others wandered about the island in search of Ferdinand and gets tired and hungry of the toil and at the same time gives up all hope of finding him

o Antonio and Sebastian are happy that Alonso is out of hope and decide to make another attempt on his life that night when being so tired they will be sleeping soundly

o Suddenly a solemn and strange music is heard in the air and several strange shapes enter bringing in a banquet These strange shapes then dance round it with gestures of salutation and then inviting the King to eat they depart

o Seeing this strange scene all are inclined to believe the tales told by travelers that there truly are ldquounicornsrdquo and ldquothe phoenixrsquo thronerdquo

1 ALONSO What harmony is this My good friends hark (L18-27)

GONZALO Marvellous sweet music

[Enter several strange shapes bringing in a banquet

they dance about it with gentle actions of salutation

and inviting the King and his companions to eat they depart]ALONSO Give us kind keepers heavens What were theseSEBASTIAN A living drollery Now I will believe

That there are unicorns that in Arabia

There is one tree the phoenixrsquo throne one phoenix

At this hour reigning thereANTONIO Ill believe both

And what does else want credit come to me

And Ill be sworn rsquotis true Travellers neer did lie

Though fools at home condemn rsquoem

(a) How did Prospero present an amazing spectacle before Alonso and his companions

Using his magic powers Prospero ordered strange shapes to lay a banquet before Alonso and his companions The shapes brought several dishes with tasty eatables in them They placed the dishes on a table before Alonso and his companions Then the strange shapes began to dance gracefully around the banquet While dancing they made gestures inviting them to eat the food Then suddenly the shapes disappeared(b) Who were the guests at the strange banquet Describe the lsquoliving drolleryrsquo

Alonso Sebastian Antonio Gonzalo Adrian and Francisco were the guests at the strange banquet

The term ldquoliving drolleryrdquo refers to live entertainment show In this context when Alonso the King of Naples Sebastian his brother Antonio the treacherous brother of Prospero Gonzalo the kind and loyal councillor to the King Adrian and Francisco came to the island they were hungry and weary in their spirits They heard a solemn and strange music They were shocked to see several strange shapes bringing in a banquet and these shapes danced about it with gentle action of salutation inviting the King and his companions to eat After this Sebastian described this show as lsquoliving drolleryrsquo(c) What is lsquophoenixrsquo What are lsquoUnicornsrdquo

The term lsquophoenixrsquo refers to a mythical Arabian bird which lived alone and perched on a solitary tree After one hundred years it expired in flames and rose again from its own ashes

lsquoUnicornsrsquo refers to the mythological four-footed beasts having horns in the centre of their foreheads When the horns are ground into powder the powder was believed to be

an aphrodisiac(d) How does Sebastian explain the puppet show OR Why does the speaker now believe in unicorns and phoenix

Sebastian finds several strange shapes bringing in the banquet They invite the king and his party for dinner and soon depart He tells that if such a strange sight can be a reality there is nothing incredible in the world and from the present moment he will believe anything He says that it is a strange dumb show enacted not by puppets but by living beings It is stranger than a travellerrsquos tale Seeing such a thing

before his own eyes he will no longer disbelieve the story about unicorns and phoenix(e) How do the other characters present respond to this living drollery

At the sight of the lsquoliving drolleryrsquo like Sebastian Gonzalo and Antonio too acted strangely Antonio told that he too now believes in unicorns and phoenix and anything else that seems to be incredible He too now believes in travellersrsquo tales Gonzalo told that if he would report those happenings in Naples nobody will believe him He considers that those gentle shapes were gentler in manner in comparison to the living beings Alonso was at first sight suspicious and told them that those strange shapes conveyed their meaning in expressive gestures when they seemed to lack speech by their movements and sounds Francisco was amazed at their mysterious disappearance

2 ALONSO Not I

(Line 43-52)GONZALO Faith sir you need not fear When we

were boysWho would believe that there were mountaineers

Dewlapped like bulls whose throats had hanging at rsquoem

Wallets of flesh Or that there were such men

Whose heads stood in their breasts Which now we find

Each putter-out of five for one will bring us

Good warrant ofALONSO I will stand to and feed

Although my lastmdashno matter since I feel

The best is past Brother my lord the Duke

Stand to and do as we

(a) How does Alonso respond at the spectacle of the shapes which were sent to them at the instruction of Prospero

After seeing the strange sight of appearing and disappearing of the shapes sent by Prospero to arrange a banquet for them Alonso says that his surprise at having seen those creatures is infinite and he is fully justified in feeling so much surprise He thinks that their shapes their gestures and the sounds they made were indeed amazing Although they do not possess the gift of speech yet they were able to convey their

thoughts by means of their gestures only

(b) What does Prospero say about the views expressed by Alonso regarding the shapes What does Francisco think about the shapesAfter hearing Alonsorsquos views about the shapes Prospero says that this manrsquos praise of the spirits is rather hasty He means to say that Alonso has shown great haste in reaching the conclusion about the shapes Francisco is amazed to see that those shapes disappeared in a mysterious way(c) What does Sebastian ask Alonso to doSebastian tells Alonso that the shapes having disappeared should not matter to them because they have left the eatables behind He asks Alonso to enjoy eating as they are extremely hungry but the king does not accept his offer of enjoying the dishes(d) How does Gonzalo try to dispel Alonsorsquos fear of those strange shapes What kind of references does he give to AlonsoGonzalo says that those who have travelled abroad have reported seeing even stranger sights than these shapes that Alonso and his companions have beheld Hence there is no reason to feel afraid of these shapes Gonzalo further adds that in his younger days he had heard strange stories from travelers and Alonso might have heard similar stories For instance it was said that there existed a certain race of

human beings who had huge lumps of flesh hanging at their throats and who therefore resembled bulls Then Gonzalo tells about a race of human beings whose heads were located at their breasts Gonzalo says that such stories were not believed by most people in those days but now-a-days these stories have become common(e) Explain the following lsquoEach putter-out of five for onersquoEnglish travellers often insured their trips with London brokers Those that went on foreign travels those days used to deposit a certain amount with some firm or company in London before their departure If the travelers failed to return the money was forfeited by the company with which it had been deposited But this money was repaid five-fold if the travelers returned safe and sound In this way a traveler stood a great chance of recovering the entire cost of his

travels(f) Give the explanatory meanings of the following expressions in the context of the above extract (i) Dewlapped (ii) Wallets of flesh

(iii) Putter-out(i) Dewlapped having big lumps of flesh at the necks(ii) Wallets of flesh large masses of flesh looking like bags(iii) Putter-out to invest money before commencing the travel

  • General methods of preparation of hydrogen
  • Chapter Dimensional Analysis (Summary)
    • Properties of Charge
Page 16:   · Web viewSubject. Topic. Summary. Execution. Hindi. व्याकरण. शरीरके अंगो के नाम लिखिए. 1) आँख 2) नाक 3

Advantages i ) It produacuteces superior quality plantsii)It can be applied to interspecifie hybridsiii) It is useful to grow seedless plants bull Limitations i) It cannot be used for all plantsii)It is not easy to handleQ9 Write advantages of vegetative propagationi) It is a quick and easy method ofproducing new plantsii) This method need less time to matureiii) The new plants are exact copies of the parentiv) it is extremly useful for growing seedlessplants like banana grapes etc Q10 Write some disadvantages of vegetativepropagationi) Dišeases present in the parent plant gettransferred to all in new plantsii) Overcrowding of new plants causes competition for sunlight water and nutrients which affects growth of plantsplant

Physics Chapter 2 Physical Quatites and Measurements

Here We Will Do Some QuestionsRelated To Chapter 2

Select the correct alternative A block of wood of density 08gcm-3 has a volume of 60cm3 The mass of the block is

1 608 g

2 75 g

3 48 g

4 0013 g

Solution 348 g

The density of aluminium is 27g and that of brass The correct statement is

1 Equal masses of aluminium and brass have equal volumes

2 The mass of a certain volume of brass is more than the mass of an equal volume of aluminium

3 The volume of a certain mass of brass is more than the volume of an equal mass of aluminium

4 Equal volumes of aluminium and brass have equal masses

Solution 2 The mass of a certain volume of brass is more than the mass of an equal volume of aluminium

MATHEMATICS Ch 6Sets

Exercise 6(C)1 Find all the subset of each the following sets(i) A = 57 (iii) C = x xisin W x le 2(iv) p p is a letter in the word lsquopoorrsquo

Solution (i) All the subsets of A are ϕ 5 7 57

(iii) All the subsets of C are ϕ 0 1 2 01 02 12 012

(iv) All the subsets are ϕ p o r po or por

4 Given the universal set = -7-3-105689 find (i) A = x xlt2 (ii) B = x -4ltxlt6 Solution

(i) A = -7-3-10(ii) B = -3-105

5 Given the universal set = x xisin N and xlt20 find

(i) A = x x = 3p pisin N (iii) C = x x is divisible by 4 Solution

(i) 369121518 (iii) 481216

6 Find the proper subset of x x2-9x-10 = 0 Solution

ϕ 10 -1

Working x2-9x-10 = 0 rArr x2-(10-1)x-10 = 0

rArr x2-10x+x-10 = 0 rArrx(x-10)+1(x-10) = 0

rArr (x+1) (x-10) = 0

11 Let M = letters of the word REAL and N = letters of the word LARE Write sets M and N in roster form and then state whether (i) M sube N is true (ii) N sube M is true (iii) M = N is true

Solution M = real and N = lareSo (i) Yes (ii) Yes (iii) Yes

English 2 Twelfth Night ndash Shakespeare

A noble man named Orsino in the kingdom of Illyria is deeply in love with a lady called lady Olivia She is in mourning for her dead brother so she will not even think about marriage At this time a sea storm causes a terrible shipwreck and a young lady called Viola is swept onto the shore She thinks that her twin brother Sebastian is drowned A sea captain tells her about Orsino and his love for Olivia Viola wishes to work in Oliviarsquos home but feels she will not be employed So she dresses as a man calls herself Cesario and gets work at the house of OrsinoViola (now Ceasario) is much liked by Orsino and becomes his page She falls in love with Orsino Orsino sends Ceasario to deliver messages to Olivia Olivia herself falls for the beautiful young Ceasario believing Viola to be a man

2 Answer the following questionsa Why does Orsino ask the musicians to play onOrsino asks the musicians to play on because music feeds his desire He calls upon the musicians to play music so that his hunger for love could be replenished with an excess of musicb What does Valentine tell about OliviaWe learn from Valentine that Olivia is in mourning for her brother she wears a veil and has vowed that no one will see her face for another seven yearsand she refuses to marry anyone until thenc From the exchange between Orsino and Valentine what do you think their relationship isValentine is one of orsinod attendants He was sent to Olivia as a messenger of love but was not allowed to speak to here Who is Olivia mourning for and whyOlivia is mourning for her dead brother

Homework Q fHistory and Civics

Growth of Nationalism

Important dates to remember1769-Napoleon born on 15thAugust1789-Fall of Bastille on 14th July and the beginning of the French revolution declaration of the rights of Man on 26thAugust1793-King Louis XVI executed on January 211764-The Sugar Act passed1765-The Stamp act passed1774-The first congress of Philadelphia1776-The declaration of American Independence of on 4th July1777-Defeat of the British at Saratoga1781-Surrender of lord Cornwallis at Yorktown1783-The treaty of Versailles1804-Napoleon becomes the emperor1813-Battle of Leipzig or Battle of nations in which Napoleon was defeated by the Allies1815-Battle of Waterloo June 18 in which Napoleon was defeated and captured1821-Death of Napoleon in StHelena1860-Abraham Lincoln elected President of the USA1861-The civil war began 1864-Abraham Lincoln elected President of the USA for the second time1865-Slavery abolished in the US

Name the following- The queen of Louis XVI

Marie Antoinette The three philosophers of France

VoltaireMontesquieuJean Jacques Rousseau

The British general whose surrender brought the war in America to an endLord Cornwallis

The first president of the USAGeorge Washington (1732-1799)

The first southern state to secede from the unionSouth Carolina

The author of the book lsquoUncle Toms CabinHarriet Beecher Stowe

Homework-Learn

Class IXSubject Topic Summary Execution

Economics

Types of economies Today I am going to share you the concept of economic growth and economic development Few questions will be given from the previous study material dated 942020

Meaning of economic growthAnswer) The term economic growth generally means anincrease in national income or per capita output or income over time It indicates towards quantitative growth of a country

Meaning of economic developmentAnswer) Economic development is defined

as a process whereby the real per capita income of a country increases over time along with fall in poverty ratio unemployment and income inequality etc

Distinguish between economic growth and economic development

Basis Economic growth

Economic development

Scope It has narrow scope as it refer only to rise in per capita income

It has wide concept since it includes qualitative changes as well

Concerned matter

It is concerned with the rise in income

It is concerned with not only rise in income but also reduction of poverty income inequality and unemployment

Focus Economic growth does not focus on economic development

Economic development focus on economic growth plus qualitative changes

Distinguish between capitalist economy and socialist economy

Ownership

Motive

Tool

Means of production are owned and managed by private people

Self interest and profit earning is the main motive

Price mechanism is a main tool to solve the economic problems

Means of production are owned and managed by the government

Social welfare is the main motive

Economic planning by the government is the main tool to solve the economic problem

Competition

Distribution of income

There exist large competition among buyers and sellers

There is existence of large inequalities of income

There is no such competition

There exist less inequalities of income

Math Topic ndash AlgebraChapter -Factorisation

Study item Factorising by taking out common factorSome solved sums from exercise 41

1) (i) 8xy3 + 12x2y2

= HCF of 8xy3 and 12x2y2 is 4xy2

= 4xy2(2y + 3x )

4) (ii) 28p2q2r ndash 42pq2r2

= HCF of 28p2q2r and 42pq2r2 is 14pq2r = 14pq2r (2p - 3r )5) (ii) 14mn + 22m - 62p=HCF of 14mn 22m and 62p is 2= 2(7mn + 11m - 31p)7) (ii) 3a(x2 + y2) + 6b (x2 + y2) = HCF of 3a(x2 + y2) and 6b(x2 + y2 ) is (x2 + y2)= ( x2+ y2 )(3a + 6b )9) (ii) x(x2 + y2 ndash z2 ) + y(-x2ndashy2 + z2 ) ndash z(x2+ y2 ndash z2 )= x(x2 + y2 -z2) ndash y-(x2 + y2 -z2) -z(x2 + y2 ndash z2)=x(x2 + y2-z2) -y( x2 + y2-z2) ndash z (x2 + y2 -z2)= (x2+ y2 ndash z2)(x ndash y ndash z )

Commercial Studies

Introduction to Accounting and Book-keeping

Today I am going to share you the meaning of Accounting and Book-keeping and its related terms bullAccounting bullBook Keeping bullAccountsbullTypes Of Accounts bullAccounting Cycle

bull Meaning of accounting

Ans )Accounting is the art and science of recording classifying and summarising monetary transactions

bull Meaning of Book-keeping

Ans) Bookkeeping is the art of recording business transactions with the view of having a permanent record of them and showing their effect on wealth

bull Meaning of account

Ans) The term account means a record of business transactions concern a particular person of firm asset or income or expense It is a summarised record of all transactions which take place in an accounting year

bull Types of accountsPersonal accounts ndash Personal accounts relating

to person and Organisation are known

as personal accounts Example Ramrsquos Account ABC amp Co Account etc

Real account - The accounts related to tangible and intangible assets are called real accountsExample Cash Account Furniture Account etc

Nominal account- Accounts related to expenses losses incomes and gains are known as nominal accountsExample Wages Account Salary Account Discount Account etc

bull Accounting cycle Accounting cycle refers to a complete sequence of accounting activities It begins with recording of transactions and ends with the preparation of a balance sheet

English 1 Transformation of sentences

Sentences A sentence is a group of words which makes complete sense

a Assertive sentencesb Imperative sentencesc Interrogative

sentencesd Exclamatory sentences

Sentences can be changed from one grammatical form to another without changing the meaning of the sentence This is known as transformation of sentences

Exercise 6Rewrite the following sentences according to the instructions given below without changing their meanings

1 As soon as he saw the beer he jumped into the river ( Begin No sooner)

2 None but brave deserve the fair (Begin the bravehellip)

3 This box is too heavy for me to lift ( Use so hellip That instead of too)

4 No one other than a king can live like James Luxurious ( Begin only James)

5 Oh for the wings of a dove (Begin I wishhellip)

BENGALI(2ND LANGUAGE)

ldquo বঙগভমির পরমি ldquo াইকেল ধসদন দতত

পব13পোসঠ আসোলিচত ৩ পরবোস দৈদসবর বস ীবতোরো Pলিদ স এ লেদ -আকো সত-োলি লেদ তোস - ক) বকতো লেক কোর লেো লেকো কলিবতোর অং ) কোর পরলিত বকতোর এই উলিকত গ) এ লেদ আকো সত বসত কী বলিঝসয়স4 ীবতোরো বসত কী লেবোঝ ঘ ) আসোচয অংসর তোৎপP13 কী

উ -ক ) বকতো স কলিব মোইসক ম3দ দতত

Types of AccountPersonal AccountReal AccountNominal AccountBalance Sheet (opening)

কলিব মোইসক ম3দ দসততর রলিচত বঙগভলিমর পরলিত কলিবতোর অং ) কলিব বঙগী অ13োৎ লেদমোতোর পরলিত কলিবর এই উলিকত গ ) এ লেদ আকো বসত কলিবর মোব লেদী রপ আকো লেক লেবোঝোসো সয়স4 আকো লেসক লেPম তোরো স পসর লেতমলি ীব লেদ রপ আকো লেসক পরো রপ তোরো স পরসত পোসর এই মভোবোর কোই কলিব বসস4 ঘ ) পরবো Pোতরোয় Pলিদ কলিবর লেদ আকো লেসক ীব তোরো রপ পরো স পসর তোসত কলিব লিবনদমোতর দঃলিত কোর মতয লিবসর সবোভোলিবক পলিরলিত এবং মোষ মরী তোই পরবোস Pলিদ তা োর মতয য় তবও কলিব লিবচলিত সব ো কোর পলিবীসত লেকউ অমর য় লিক4ই অকষয় য় দীর লেPম লিচরপরবোমো লেতমলি মোসষর ীবও চমোতোই ীব - সতবধতোই মতয ীব দীসত মোষ লিতয পরবোমো তবও লেPব মোষ আপ কতকসম13র মো3যসম মোসষর মস লিসসদর সথো কসর লিসত পোসর তোরো লিচরভোসবর সয় মোসষর মস লিবরো কসর তোসদর মস3য লেকউ পGভসত লিবী সয় গোসও মোসষর মস তোরো লিতযপলিত লিতযবলিনদত

Hindi 2ndlang

काकीी(लिसयारामशरणगपत)

इस कहानी म क न यह बतान का परयास निकया ह निक बचच अपनी मा स निकतना परम करत ह शयाम अबोध बाक ह वह अपनी मा क मरन क बाद उसन अपनी मा क लिए बहत रोया बाद म उस पता चा निक उसकी मा राम क घर ची गई ह आकाश म उडती हई पतग दकर उस हष हआ निक पतग क दवारा वह अपनी मा को नीच उतारगा इसक लिए वह अपनी निपता की जब स दो बार सवा रपया निनकाकर पतग और दो मोटी सी मन वाी अपन भाई स काकी एक कागज पर लिवा कर पतग म लिशव का दिदयानिनकाकर पतग और दो मोटी सी मन वाी अपन भाई स काकी एक कागज पर लिवा कर पतग म लिचपका दिदयाभोा और शयाम कोठरी म रससी बाधनी रह थ तभी उसक निपता करोध म आकर उन स पछ निक कया उनकी जब स रपया निनकाा हभोा डर क मार बताया निक शयाम इस पतग क दवारा अपनी काकी को राम क यहा स उतारना चाहता हनिवशशवर(शयाम क निपता)न फटी पतग उठाकर दी तो उस पर काकी लिा थावह हत बजि होकर वही ड रह गएउनहोन सोचा निक मन अपन पतर को मारा जोनिक अनजान और निनदष थावह अपनी मा कोनिकतना पयार करता ह

helliphellipContinue to next

Computer Application

Java Programming Prog 1Write a java program to input two numbers from user and display the sum or product of them as per user choice Use switch case statementSolve public class sum_product public static void main(String args[]) Scanner sc=new Scanner(Systemin) int abc Systemoutprintln(ldquoEnter two numbersrdquo) a=scnextInt() b=scnextInt() Systemoutprintln(ldquoPress 1 for sum or 2 for productrdquo)

c=scnextInt() switch(c) case 1 Systemoutprintln(ldquoThe sum will be =rdquo+(a+b)) break case 2 Systemoutprintln(ldquoThe product will be =rdquo+(ab)) break default Systemoutprintln(ldquoWrong Inputrdquo) Home Work - Practice in your computer using bluej

Subject Eng Literature (The Merchant of Venice ndash William Shakespeare)Topic Act I Scene 2 Lines 92 to 126 (End of scene) Date 13th April 2020 (5th Period)

[Students should read the original play and also the paraphrase given in the school prescribed textbook]Summary Questions amp Answers

o After Portia has expressed her opinion about the suitors Nerissa informs that she need not bother about any one of them as they have decided to quit Belmont at the earliest opportunity because they do not believe in trying their luck by the caskets which is the only way of winning Portia

o Nerissa then enquires of Portiarsquos opinion about Bassanio who once visited her in the company of the Marquis of Montferrat and says that she had never come across such an ideal love deserving the fairest lady for his bride

o Portia seems to remember Bassanio quite correctly and says that she agrees with Nerissa At this moment a servant informs Portia that the Prince of Morocco has arrived to try his luck by the caskets

o Portia tells Nerissa that if she could welcome this new suitor as gladly as she says farewell to the previous ones she would be glad of his arrival However if he happens to have the virtues of a saint but the black complexion of a devil she would prefer to have him for religious consolation rather than as a husband

(1) NERISSA You need not fear lady (Line 97-103)

the having any of these lords they have acquainted me with their determinations

which is indeed to return to their home and to

trouble you with no more suit unless you may be wonby some other sort than your fathers imposition depending on the caskets

PORTIA If I live to be as old as Sibylla I will die as chaste asDiana unless I be obtained by the manner of my fatherswill I am glad this parcel of wooers are so reasonablefor there is not one among them but I dote on his veryabsence and I pray God grant them a fair departure

(a) Elucidate the idea expressed in the first speech of the above dialogue

In the first speech Nerissa assures Portia that she need not have any fear of being compelled to marry anyone of the suitors who had lately come to Belmont She informs her that they have all decided to return to their respective countries(b) Illuminate the meaning of the phrase ldquoyour fatherrsquos imposition depending on the casketsrdquo

Nerissa means that the suitors of Portia do not find the conditions imposed by the will of her father to their liking They are too hard for them These conditions are that in the event of a suitor failing to choose the right casket (i) he should never disclose to anybody which casket he chose (ii) he can never marry and (iii) he should take his departure immediately(c) Explain the meaning of the term lsquoSibyllarsquo

lsquoSibyllarsquo is the name given by Romans and Greeks to a prophetess inspired by some deity usually the sun-god Apollo She had a very long life The god Apollo granted her as many years of life as she could hold grains of sand in her hand(d) Elucidate the meaning of the term lsquoDianarsquo

lsquoDianarsquo is the goddess of hunting She is also regarded as a symbol of virginity because she never fell in love and never

married(e) Explain the meaning of the first two lines of Portiarsquos speech

Portia says that even if she is to live for centuries like Sibylla she would not marry except in accordance to her fatherrsquos will She asserts that she would not mind remaining unmarried and untouched by a man like Diana the virgin the goddess of hunting unless a man is able to win her by passing the test laid down by her father

Class XSubject Topic Summary Execution

Hindi 2nd

Langबड घर की बटी( मशी परमचद)

lsquoबड घर की बीटीrsquo कहानी का उददशय मधयम वग की घर समसया को सझा कर सगदिठत परिरवार म मिम जकर परम स रहन का सदश दना ह घर म शानित tानिपत करन की जिजममदारी नारी की होती ह यदिद नारी समझदार ह उसम धय और परिरवार क परनित परम ह तो कोई भी घटना परिरवार को निवघदिटत नही कर सकती या कहानी परिरवार को सगदिठत करत हए परम सौहाद स एक रदसर की भावनाओ को समझ करउनका सहयोग करत हए जीवन यापन करन की पररणा दती ह मशीपरमचदर जी न इस कहानी म सयकत परिरवार का परनितनिनमिधतव निकया ह यह कहानी बनी माधव सिसह जो गौरी पर क जमीदार क उनक दो पतरो की हशरी कठ ा निबहारीशरीकात का निववाह एकजमीदार घरान की पतरी आनदी स हआ थाआनदी न द को ससरा क वातावरण म ढालिया थाएक दिदन आनदी का अपन दवर ा निबहारी स झगडा हो जाता ह दोनो भाई एक रदसर स अग होन की कोलिशश करत हसभी बह आनदी न अपन मधर वयवहार स ा निबहारी को घर छोडकर जान स रोक लिया| इस पर बनी माधव सिसह न कहा निक बड घर की बटी ऐसी ही होती ह जो निबगडा काम बना ती ह अतः शीषक साथक ह बड घर की बटी आनदी ह

helliphelliphelliphellipContinue to nextBiology Topic ndash Chp-1

CellWelcome to new session 2020-21Today we will start with Chpter 1 cell CELL

Protoplasm+Cellmembrane Or Cell wall

Cytoplasm+Neucleus

Cytoplasmic+ CytoplasmicOrganelles Inclutions(mitochondria (food Golgi bodies pigments)Ribosome)

What is cellbull Cell is the structural and functional unit of living organismbull According to number of cells organisms areUnicellular - Amoeba bacteria Multicellular - Rose Mango Tiger HumanSmallest cell -bacteria Longest cell - Nerve cellLargest cell - Ostrich egg cellCells are of different size and shapes according to their functionsQ2Write chief functions of following cellorganelles

Q3What is tonoplastVacuoles covered by a covering called tonoplast

Bengali(2Nd

Language)

ফ ফটক ো ফটক (কলিবতো ) ভোষ মসোপো3 gtPোয়

একটি লেমসয়র ীবস লেপরম লিকভোসব ফসট ওসঠ তো লেদলিসয়স4 কলিব লেপরম Pই য় লেই ময়ই বনত কোস পলিরত য় ফ লেফোটো বো োসফোটো লেটো ব2 কো য় লেমসয়সদর ব gtয13 লেপরসমর 4লিব ফসট উসঠস4 এই কলিবতোয় লেপরম মোষসক মত gtযর মস লেফস লিদসয় পরকষস বাোচোসোর gtয োত বো2োয় কলিবতোয় লেমসয়টির পসব13র দঃসর কো বো সও লেমসয়টি লেই পসর পলিক সত চোয়ো োরী ীবসর কোস4 পরম লেPৌবস লেপরমসক পোবোর পরব ইচছো োকসও তো পসর লেলিতবোচকতোয় পলিরত য় কলিব ভোষ মসোপো3 যোয় লেP ক লেপরসমর

কলিবতোয় ব gtযবহত লিবসষ লিক4 সvর অ13 লেদওয়ো ১) রসবোো= লেP লিবলিভনন রকম ডোকসত পোসর২) ো= পোর ৩) ঠলি = লেচোসর বZ৪)আই বস2ো=অলিববোলিত৫)শইসয় = োলিয়ত কসর৬)োতপাোচ= লিবলিভনন পরকোর৭)দ2োম = v কসর বZ কসর লেদওয়ো৮)লেরলিং =লেোোর দৈতরী লেব2ো৯) বনত= একঋত১০) পাোর = বসকরো2

Organelles Functions

1 Endoplasmic reticulum

2 Mitochondria

3Golgibodies

4 Ribosome5Lysosome

6Plastids

7 Centrosome

i) Supportive framework for the cellii) Synthesis and transpost of proteinsRelease of energy in the form of ATPi) Synthesis and secretion of enzymes hormoneii) Formation of vacuoles lysosomei) Protein Synthesisi) Intracellular digestionii) Destroy foreign substancei )Leucoplast - stores starchii)chloroplast - trap solar energyiii) Chromoplast - imparts colour toflowers amp fruitsi) Initiates and requlates cell division

কলিবতো তোর অ13সক ভোষোয় পরকোো কসর ঘলিরসয় ব যকত কসরস4 লেপরসমর ফতো আর লিবফতো লেক গোঢ় কসর লেদোসো কলিব ভোষ মসোপো3 যোসয়র অলিভবসর অ যলিদক

Economics

Factors of Production

Welcome to the new sessionToday we are going to start the first chapter of Class XThe name of the chapter is Factors of productionBy the name I hope you all can recall a glimpse of what you have learnt in the second chapter of Class IX

NowProduction is the process of creating the various goods and services which are consumed by the people of the country to satisfy their wants

Thus it is the process in which some materials are transformed from one form to another to create utility and value in goods

For example utility can be created by changing the form of a commodity ie

Making of table out of wood by a carpenter for his customer here the wood is getting transformed into table creating utility for his customer and he can also command a price for it

On the other hand Housewives perform very

useful activities at home which create utility but their domestic activities are not included in production because they have no money value

So we can also say that Production denotes two things firstly creation of utility and secondly creation of value

Production is not complete unless it reaches the consumer

An increase in production will increase the economic welfare of the consumers and hence the aim is to raise the production level of the country

Again production of a good or service is only possible if certain resources or

Questions

1 What do you mean by production

Answer Production means the creation of goods and services for the purpose of selling in the market

In fact production involves the transformation of inputs into outputs

Hence production denotes two thingsCreation of utility and creation of valueUtility and value can be created by changing the form by changing the place by changing the time and by rendering services

Example Transformation of raw

materials into finish goods such as potter creates utility by converting mud into utensils assembling of small parts to make bigger machinery

Production also includes services such as distribution and marketing

2 What are the factors of production

Answer Factors of Production refers to the resources and inputs needed for producing goods and servicesThese inputs can be classified as

Land Labour

Capital Enterprise

Land Land is defined to include not only the surface of the earth but also all other free gifts of nature(for example mineral resources forest resources and indeed anything that helps us to carry out the production of goods and services but is provided by

inputs are used together in right proportion

A resource or an input which helps in the process of production to obtain an output is called FACTOR OF PRODUCTION

These factors of production can broadly be categorized into four parts 1LAND 2LABOUR3CAPITAL4ENTERPRISE (ORGANISATION)or Entrepreneur

The above factors are all interdependent on each other and they play a major role in production process

FACTORS OF PRODUCTION

LANDCAPITAL

LABOUR ENTREPRENEUR

nature free of cost)LabourLabour refers to the human efforts that need to be combined with other factors of production for creating an output

CapitalAll man ndash made means of production is called capita example machineries which help in further production Money when used for starting any business for purchasing raw materials machinery tools etc it is regarded as capitalCapital also includes physical capital like factories machineriestoolsbuildingsequipments etcEnterpriseThe task of bearing risks is called enterprise and the person who bears these risks of business is called the entrepreneurThus an entrepreneur is one who organises production takes important decisions regarding production hires and purchases factors of production and bears the risk and uncertainty involved in productionOrganisation refers to the services of an entrepreneur who controls organises and undertakes all risks One who plans organises and manages a business enterprise is an organiser

Physics Chapter 1 Force

Force is an external agent capable of changing the state of rest or motion of a particular body It has a magnitude and a direction The direction towards which the force is applied is known as the direction of the force and the application of force is the point where force is applied The Force can be measured using a spring balance The SI unit of force is Newton (N)

Question 1

State the condition when on applying a force the body has

(a) the translational motion

(b) The rotational motion

Solutions

(a) Translational motion is produced when the body is free to move

(b) Rotational motion is produced when the body is pivoted at a point

Question 2

Define moment of force and state its SI unit

Solutions

The moment of force is equal to the product of the magnitude of the force and the perpendicular distance of the line of action of force from the axis

of rotation

The SI unit of moment of force is Newton times meter

= Newton meter (Nm)

Commercial Studies

Stake holders In this topic you will be come to know about the meaning and concept of stakeholders

How stakeholders are different from shareholders

Questions1 What do you mean by the term stake holdersAnswer) The term stake holders have developed from the words which mean an interest or expected benefit Stakeholders mean all those individuals groups and Institutions which have a state (interest) in the functioning and performance of a commercial organisation or a business enterprise2 What do you mean by share holdersAnswer) The person and Groups who own the shares of the joint stock company by providing capital to the company are called shareholders Shareholders are the internal stakeholders shareholders are one out of several stake holders3 How are shareholders different from stakeholdersAnswer)i) The term shareholders is related to only joint stock company whereas stakeholders are related with all business organisationsii) Stakeholders maybe any individual having financial stake in business organisation whereas a shareholders are those individuals who are holding shares in the company4) How are shareholders different from creditorsAnswer) i) Shareholders are internal stakeholders while creditors are external stakeholdersii) Shareholders invest in the capital of the company whereas creditors give loan to the companyiii) Shareholders are the members of the company with voting rights but creditors are not the members of the company

English 1 Transformation of sentences

Sentences A sentence is a group of words which makes complete sense

e Assertive sentencesf Imperative sentencesg Interrogative sentencesh Exclamatory sentences

Sentences can be changed from one grammatical form to another without changing the meaning of the sentence This is known as transformation of sentences

Exercise 1 Change the following affirmative sentences into Negative sentences

a He is a good manHe is not a bad man

b Ram loves SitaRam is not without love for Sita

c Only he stood first in the classNone but he stood first in the class

d Ankit was wiser than he

He was not so wise as Ankite He did it

He did not fail to do itf As soon as I reached college the

bell rangNo sooner did I reach college than the bell rang

g He finished everythingHe left nothing unfinished

h It always pours when it rainsIt never rains but it pours

Math Topic Commercial MathematicsChapter ndash Goods and services Tax

What is GSTAns It is a abbreviated term of Goods and Service Text which is an indirect tax levied on the sale of goods and rendering servicesSome terms related to GSTDelar Any person who buys goods or services For resale is known as a delar A delar Can be a firm or a companyIntra-state sales Sales of goods and services within the same state or same union territory are called intra- state salesInter-state sales Sales of goods and services outside the state or union territory are called Inter-state sales4) Input GST GST is paid by dealers on purchase of goods and services are called input GST5) Output GST GST is collected from customers on sale of goods and services are called output GST6) Types of GST There are three taxes applicable under GST(i) Central Goods and Services Tax (CGST)(ii) State Goods and Services Tax (SGST) or Union Territory Goods and Services Tax (UTGST) Both these taxes are levied on intra-state sales Here GST is divided equally among central and state governments(iii) Integrated Goods and Services Tax (IGST) IGST is levied on inter- state sales It is also levied on import of goods and services into India and export of goods and services from India

Subject Eng Literature (The Merchant of Venice ndash William Shakespeare)Topic Act III Scene 4 Lines 1 to 44 (Portia hellip To wish it back on you fare you well Jessica)[Students should read the original play and also the paraphrase given in the school prescribed textbook]

Summary Questions amp AnswersIn this scene we suddenly find a new element in the character of Portia We have already seen her possessed of every graceful womanly quality but now she shows that she is capable of rapid decision and determined action She shows this by her sudden resolve to hasten to Venice with a daring scheme for the rescue of Antonio This is an important scene in the dramatic action for it leads up to and renders possible the striking events of the famous trial scene which is one of the greatest striking elements of the play Moreover the fact that all the characters of importance are now assembled together in Venice makes the union of the main plot and the secondary story complete

(1) LORENZO Madam although I speak it in your presence(Line 1-9)

You have a noble and a true conceit

Of god-like amity which appears most strongly

In bearing thus the absence of your lordBut if you knew to whom you show this honourHow true a gentleman you send reliefHow dear a lover of my lord your husbandI know you would be prouder of the workThan customary bounty can enforce you

(a) Where is Lorenzo Why is he here To whom is he referring as lsquoMadamrsquo

Lorenzo is at Portiarsquos residence He had met Salerio on the way and Salerio had begged him to come along with him to

o In this scene Portia Nerissa Lorenzo Jessica and Balthazar appear

o Portia requests Lorenzo and Jessica to be in charge of her house during her absence from Belmont because she and Nerissa have decided to spend the days in meditation and also in visiting the holy places in the neighbourhood of Belmont She has already instructed her people to acknowledge both Lorenzo and Jessica as master and mistress of house during her absence Lorenzo and Jessica gladly agree to look after the house of Portia

handover the letter from Antonio to Bassanio The letter carried the bad news about Antoniorsquos arrest for non-payment of loan taken from Shylock Hence Salerio might have preferred company to break this bad news to Bassanio He is referring to Portia as Madam(b) What does Portia say on hearing the above extract

Portia says that she has never regretted doing good to others Friends who spend a lot of time together and really are there for each other have many traits in common As Antonio is Bassaniorsquos best friend saving him is like saving Bassanio who is like her own soul She asks Lorenzo to take care of management of the house till Bassanio is back(c) What does Portia send with Bassanio and why

On hearing about Antoniorsquos troubles on account of Bassanio her husband Portia immediately sends him with enough gold to repay the debt many times over to Venice to help Antonio out of his misfortune

(2) Lorenzo Madam with all my heart (Line 36-40)

I shall obey you in all fair commands

Portia My people do already know my mindAnd will acknowledge you and JessicaIn place of Lord Bassanio and myselfSo fare you well till we shall meet again

(a) Where are Lorenzo and Portia at this time What lsquofair commandsrsquo are given to Lorenzo

Lorenzo and Portia are at Belmont during this scenePortia reveals to Lorenzo that she has sworn to contemplate in prayer at a monastery around two miles away until her husband returns from Venice She tells him that Nerissa would accompany her and asks him to manage the house with Jessica till things are settled In response Lorenzo tells her that he would be obliged to do whatever she asks him to do(b) Where is Portia actually going and why

Portia tells Lorenzo that she would live a life of contemplation and pray at a monastery which is two miles away from her place In reality Portia plans to go to Venice in disguise with Nerissa and argue the case in defense of Antonio She is very sure that her plan would succeed

ClassXI (ScienceHumanitiesCommerce)Subject Topic Summary Execution

Computer Science

(APC)

Ch ndash 1 Numbers

(Numbers in different bases and

their Arithmatical operations)

Number System In computers Number System is defined as a writing system to represent the numbers in different ways ie we are using different symbols and notations to represent numbers There are four ways we can represent the number ndash Binary Decimal Octal and Hexadecimal

Decimal Number SystemThis number system consist 10 digits These are 0 1 2 3 4 5 6 7 8 amp 9

Binary Number SystemThis number system has only two digits these are 0 and 1 Here 0 stands for off while 1 stands for on

Octal Number SystemThis number system has 8 digits these are 0 1 2 3 4 5 6 amp 7

Hexadecimal Number SystemThis number system has 16 digits these are 0 1 2 3 4 5 6 7 8 9 A B C D E F Here the value of the alphabets are as follows A=10 B=11 C=12 D=13 E=14 F=15

Rules for conversion decimal number to Binary1 Divide the decimal number by 22 If the number will not divide equally by 2 then round down the answer to the nearest whole number (integer)3 Keep a note of the remainder it should be either 0 or 14 Keep repeating the above steps dividing each answer by 2 until you reach zero5 Write out all the remainders from bottom to top This is your binary solution

For example Lets convert 32 to binary 2 32 2 16 - 0 2 8 - 0 2 4 - 0 2 2 - 0 2 1 - 0 0 - 1

The binary equivalent of 3210 is 1000002

Try the follwing youself1 2410

2 4810

3 1210

History GROWTH OF NATIONALISM

The second half of the 19th century witnessed growth of political consciousness and a sense of Nationalism among the IndiansThere were various factors for growth of Indian Nationalism- As a result various political associations were formed in different provinces by the educated Indians Surendranath Banerjee organized a meeting of National conference at Calcutta Ultimately the National Congress was founded in Bombay in 1885This body became the vanguard of Indian struggle for freedom The congress leaders were known as moderates because they followed a policy of prayer and petition A large number of Indian leaders had experienced in political agitation The Political situation of England was also changed Moreover increasing revolutionary activities in Maharashtra Punjab and Bengal became serious concern to the British Government In this

QUESTION1 What do you mean by Nationalism ANSWER 1 Nationalism is defined as loyalty and devotion to own nation especially a sense of national consciousnessQUESTION 2 What are the causes of nationalism ANSWER 2 There were various factors for growth of nationalism

1 Spread of western education2 The progress of vernacular press and

patriotic literature3 The economic exploitation of our

country by the colonial rulers4 International affairs

QUESTION 3 Who organized National conference in Calcutta in 1883 ANSWER 3 Surendranath BanerjeeQUESTION 4 When did Indian National Congress formANSWER 4 Indian National Congress was formed in 1885 in BombayQUESTION 5 Who were ModeratesANSWER 5 The Early Nationalists were also known as Moderates Their emergence marked

background Lord Curzon became Viceroy in India He had no respect for the Indian National Congress

the beginning of the organized national movement in India They believed in British justice and were loyal to them They followed a policy of prayer and petition They demanded constitutional reforms of our country Impotant Moderate leaders were Pherozshah Mehta Dadabhai Naorozi and Surendranath Banerjee etcQUESTION 6 What do you know about Extremism in Indian National movementANSWER 6 In the beginning of 20th century a new class of national leaders emerged in India which was different from the moderate groups They started more aggressive movement against the British empire The goal of extremists was ldquoswarajrdquo Important extremist leaders were Bal Gangadhar Tilak Lala Lajpat Rai Bipin Chandra Pal etcQUESTION 7 Mention the places which were the main centres of Revolutionary movementANSWER 7 Maharashtra Bengal and Punjab

Physics

Chapter Dimensional Analysis

(Summary)

The dimensions of a physical quantity are the powers to which the fundamental units are raised in order to obtain the derived unit of that quantit

The physical quantites lengthmasstime are represented by [L] [M] [T] resp let they are raised to powers ( dimesions) abc resp then any physical quantity can be represented by [ La Mb Tc ] Examples

1 Area area = L x B = [L] x [L] = [M0 L2 T0 ]

2 Density density = massvolume = [M][L3] = [ M L-3]

3 Velocity velocity = distancetime = [L][T] = [LT-1]HW Try to find out dimension of acceleration Acceleration = velocity timeNB One can find the SI Units Using Dimension Analysis Such as for area we have [L2] so its SI unit is m2

Biology Topic ndash Chp-1 The living world

Today we will start the first chapter the living world Here we discuss about the characteristics of living organism and what are the difference between them and nonliving substances We also discuss about the contribution of different Scientists

There are over 500000 species of plants andover a million species of animal are present on earth Some 15000 new species were discovered every yearQ1 What is a living organismbull A living organism is primarily physico -chemical material that demonstrate a high degree of complexity is capable of selfRegulation possesses a metabolism and perpetuates itself through timeQ2 What are the differences between livingand non-livingsi) Compared with non-living living organisms

have more complex organised structure and their use of energy is more controlled amp efficientii) Living things reproduce their own kind by forming new cells which contains copies of their genesiii) Each organism has some degree of homeostasisie it is able to make adjustments so that internal environment remains constantQ3 Write contributions of following Scientists i) Aristotle - One of the first theories in Biology places all living things in a hiearchieii) AV Leeuwenhoek - was the first to observe living single celled organisms under microscopeii) Carolus Linnaeus - developed the binary system for naming of organisms and classificationiii) Geregor Johann Mendel ndash discoverbasic principles of inheritanceHomework i) C Darwin ii)Schleiden

Math Trigonometric functions

1 Overviewi) Trigonometry The word lsquotrigonometryrsquo is derived from the Greek words lsquotrigonrsquo and lsquometronrsquo which means measuring the sides of a triangle An angle is the amount of rotation of a revolving line with respect to a fixed line Usually we follow two types of conventions for measuring angles ie a) Sexagesimal system b) Circular system In Sexagesimal system the unit of measurement is Degree In Circular system the unit of measurement is Radian ii) Relation between degree and radianThe ratio of circumference of a circle to its diameter is always a constant This constant ratio is a number denoted by π which is taken approximately as 227The relationship between degree amp radian measurements is as follows2 right angles = 180deg= π radians1radian = 180degπ=57deg16(approx) 1deg=π180 radianiii) Length of an arc of a circleIf an arc of length s subtends an angle θ radians at the center of a circle of radius r then s=rθiv) Area of a sector of a circleA sector is like a pizza slice of the

Q) Express the following angles in radiana) 45deg b) 40deg3730Ans a) We have 180deg=π radiansi e 45deg= πtimes45180 radian = π4 radiansb) 40deg3730= 40deg37+3060 minute= 40deg 37 +12 minute= 40deg+ 752 minute=40 + 75(2times60) degree=3258 degreeNow 180deg=π radianie 3258 degree= (πtimes325) (180times8) radians = 65π288 radiansQ) A circle has a radius of r=12 meters What is the length of an arc traced out by a 60deg angle in the center of the circleAns In this problem we know both the central angle (60deg) and the radius of the circle (12) All we have to do is plug those values into our equation and we get

s = 2π(12)(60360)s = 24π6s = 4πSo the length of an arc traced out by a 60deg angle in a circle with a radius of 12 meters equals 4π meters asymp 1257 metersQ) Find the area of the sector with a central angle 30deg and a radius of 9cmAns GivenRadius r = 9 cmAngle θ = 30degArea of the sector = θ360degtimesπr2

= 30360degtimes227times92=2121cm2

circle It consists of a region bounded by two radii and an arc lying between the radiiThe area of a sector is a fraction of the area of the circle This area is proportional to the central angle In other words the bigger the central angle the larger is the area of the sectorArea of Sector = θ2 times r2 (when θ is in radians)

Area of Sector = θ times π360 times r2 (when θ is in degrees)

COMMERCE

CLASSIFICTION OF HUMAN ACTIVITIES-ECONOMIC AND NON-ECONOMIC

Welcome to the new sessiontoday we are going to start the first chapter of Class XI The name of the chapter that we are going to start is

lsquoClassification of Human Activities ndasheconomic and non-economicrsquo

Now let us start the chapter by considering human beings and the activities they perform throughout the day

Human activities means all those activities that human beings undertake to satisfy their wants

Human wants on the other hand are the desire of human beings for goods (vegetables fruits rice etc) and services (services of doctors teachers lawyers etc) that they require to live

Now these human activities continue throughout life as human wants are unending unlimited and recurring as human beings desire for better living throughout their lives

Now human activities can be classified into two categories

Human activities

Economic activities Non-economic activities

Economic activities are

Questions1 What are human activities

Answer Human activities mean all those activities that human beings undertake to satisfy their wants

Example A man working in an office

A boy playing in the garden

2What are the characteristics of human activitiesAnswer the characteristics of human activities are as follows

Human activities are undertaken by men women and children and these activities involve human efforts

Human activities are undertaken to satisfy human wants which are unlimited

Human activities continue throughout life

Human activities are performed for both earning money and personal satisfaction

3What is economic activitiesGive example

Answer Economic activities are undertaken by human beings with the object of earning money acquiring wealth and thereby satisfying human wantsExample

Selling of goods by a shop keeper to his customer

A clinic run by a doctor Service of a teacher in school or college

undertaken by human beings with the object of earning money and acquiring wealth

These activities result in the production of economic goods and services

Example Human activities(ie working in factories officesshops) which produce direct economic benefits

Non-economic activities are inspired by human sentiments and emotions such as love for the family desire to help the poor and love for the country

Thus these human activities (eg praying playing sleeping) produce no direct economic benefits and they are also not related to earning money and acquiring wealth

4 What are the characteristics of economic activities

Answer The characteristics of economic activities are as follows

Economic motiveEconomic activities are undertaken to earn money and acquire wealth

ProductiveEconomic activities involve productiondistribution and exchange of goods and services to create wealth

Economic growthEconomic activities determine the level of economic development of a country and standard of living of its citizens

Socially desirableEconomic activities are socially desirable for society

Economic resourcesEconomic activities make use of all the economic resources such landlabourcapital etc

5 What do you mean by non-economic activitiesExampleAnswerNon-economic activities are inspired by human sentiments and emotions such as love for the family desire to help the poor and love for the countryThese activities are not undertaken for monetary gain but for onersquos satisfaction and happinessExample

a mother looks after her children

a student donates blood8 Differentiate between Economic activities and Non-economic activities

Economic activities

Non-economic activities

1to earn living and acquiring wealth2Result can be measured in terms of money

3ExampleBusinessprofession and employment

1 to obtain some satisfaction

2Result cannot be measured in terms of money

3ExampleFamily-orientedreligious socialCultural and national

BUSINESS STUDIES

BUSINESS ENVIRONMENT

Welcome to the new sessionToday we are going to start the first chapter and the name of the chapter is Business Environment

In todayrsquos world every business enterprise is a part of the society It exists and operates in association with various groups in society such as customers suppliers competitors banks and financial institutions government agencies trade unions media and so on All these groups influence the functioning of business in one way or the other They constitute the environment of businessConcept of Business Environment

The term lsquobusiness environmentrsquo refers to the sum total of all individuals institutions and other forces that lie outside a business enterprise but that may influence its functioning and performance

The main features of business environment

Totality of External forces General and Specific forces Interrelatedness Complexity Dynamic Uncertainty Relativity

The Interrelation between business and its environment

The business enterprise is an open system It continuously interacts with its environment It takes inputs

Prepare the following questions from todayrsquos assignment

1 What do you mean by business environment

The term lsquobusiness environmentrsquo means the aggregate of all forces factors and institutions which are external to and beyond the control of an individual business enterprise but they may influence its functioning and performance Business environment is the macro framework within which a business firm a micro unit operates It consists of several interrelated and interacting elements

2 Explain the main features of business environment in brief

Totality of External forces-Business environment is the sum total of all things external to a business environment

General and Specific forces-It includes both the forces general forces are the economic social political legal and technological conditions which indirectly influence all business enterprise Specific forces are the investors customers competitors and suppliers which influence individual enterprise directly

Interrelatedness-Different elements of environment are interrelated for an example growing awareness for health care has increased the demand for health foods

Complexity- Business environment id

(such as raw materials capital labour energy and so on) from its environment transforms them into goods and services and sends them back to the environment

Fig 1 Business Environment Relationship

complex in nature as the elements keep on changing example economic technological and other forces changes in demand for a product and service

Dynamic-Business environment is not static it keeps on changing

Uncertainty- Itrsquos very difficult to predict future events such as technology and fashion which occur fast and frequently

Economics Basic Economic ConceptsSub topic

Microeconomics and

Macroeconomics

Welcome to the new sessiontoday we are going to start the first chapter of Class XI The name of the chapter that we are going to start is Basic Economic concepts

Now Economics covers the study of human activities Human activities are those activities which are performed by humans to satisfy their wants

Thus Human wants are unlimited and therefore economic activities such as production exchange and consumption are needed in order to satisfy those wants

The study of economics is divided largely in two parts which areMicroeconomics and Macroeconomics

SUBJECT- MATTER OF ECONOMICS

MICROECONOMICS MACROECONOMICS

Questions1Who has coined the words micro and macro economics

Answer Ranger Frisch coined the words lsquomicrorsquo and lsquomacrorsquo in 1933 to denote the two branches of economic theory namely microeconomics and macroeconomics

2What is microeconomicsAnswer It is the study of behaviour of individual decision ndash making unit such as consumers firms etc

3 What is macroeconomicsAnswer Macroeonomics is the study of overall economic phenomena like employment national income etc

4 What is the importance of microeconomicsAnswer

Microeconomics helps in formulating economic policies which enhance productive efficiency and results in greater social welfare

It helps the government in formulating correct price policies

It explains the working of a capitalistic economy where individual units(producers and consumers ) are free to take their own decision

Micro means a small part in

microeconomics we do not study the whole economy Hence we study an individual consumer and his or her choices and a producer and his or her profit maximizing decisions in the market Thus it does not mirror what happens in the economy as a whole

Macroeconomics on the other hand studies the economy as a whole It is concerned with aggregate and depicts the entire picture of the economyMacroeconomics deals with the national income aggregate investment aggregate consumption etc

Features of Microeconomics It deals with small

parts of the country Hence it looks at

individual consumers firms and industries

It deals with individual income consumption and savings

It studies the determination of price of any product or factors of production

It deals with the working of market via the price mechanism which is nothing but the determination of price and quantity of a commodity by the forces of demand and supply

Features of Macroeconomics

It deals with the study of the economy as a whole

It is concerned with

5 Give a limitation of microeconomics Microeconomics fails to explain the

functioning of an economy as a whole It cannot explain unemployment illiteracy and other problems prevailing in the country

6 What is the importance of macroeconomics It gives overall view of the growing

complexities of an economic system It provides the basic and logical

framework for formulating appropriate macroeconomic policies (eg for inflation poverty etc )to direct and regulate economy towards desirable goals

7What is the limitation of macroeconomics It ignores structural changes in an

individual unit of the aggregate

8 Differentiate between Microeconomics and Macroeconomics

Microeconomics Macroeconomics

the study of aggregates

National income aggregate savings and aggregate investments are major concepts dealt within macroeconomics style

It studies the determination of general price levels

It investigates into the problem of unemployment and the achievement of employment

It studies the aspect of decision making at the aggregate and national levels

It includes all growth theories whether related to developed or developing economies it also includes the study of economic systems and the working of the economy under different systems

Note Both Micro and macro economics are complementary and should be fully utilized for proper understanding of an economy

1It studies economic aspect of an individual unit2It deals with individual incomeConsumption and savings

3 It facilitates determination of price of any product or factors of production

4 Itrsquos scope is narrow and restricted to individual unit

1It studies the economy as a whole

2It deals with the national income aggregate consumption and aggregate savings3 It facilitates determination of general price level in an economy

4 Itrsquos scope is wide as it deals with economic units on the national level

ACCOUNTS

Introduction to Accounting and Book-keeping

Today I am going to share you the meaning of Accounting and Book-keeping and its related terms bullAccounting bullBook Keeping bullAccountsbullTypes Of Accounts bullAccounting Cycle

bull Meaning of accounting

Ans ) Accounting is the art and science of recording classifying and summarising monetary transactions

bull Meaning of Book-keeping

Ans) Bookkeeping is the art of recording business transactions with the view of having a permanent record of them and showing their effect on wealth

bull Meaning of account

Ans) The term account means a record of

business transactions concern a particular person of firm asset or income or expense It is a summarised record of all transactions which take place in an accounting year

bull Types of accountsPersonal accounts ndash Personal accounts relating

to person and Organisation are known as personal accounts Example Ramrsquos Account ABC amp Co Account etc

Real account - The accounts related to tangible and intangible assets are called real accounts Example Cash Account Furniture Account etc

Nominal account- Accounts related to expenses losses incomes and gains are known as nominal accounts Example Wages Account Salary Account Discount Account etc

bull Accounting cycle Accounting cycle refers to a complete sequence of accounting activities It begins with recording of transactions and ends with the preparation of a balance sheet

Chemistry TopicAtomic Structure

Thomsonrsquos atomic modelThomson (1898) was the first to propose the model of an atomHe proposed that an atom can be regarded as a uniform sphere of positive electricity in which requisite number of electrons are embedded evently to neutralize the positive chargeThis is just like plums embedded in a pudding or seeds evently distributed in red spongy mass of a watermelonThis model of atom is known as ldquoPlum-Pudding modelrdquo or

Q1)What is the fundamental constituents of atomAns Electron Proton and neutrons are the fundamental constituents of atomQ2)What is the value of fundamental unit of electricityAnsThe charge carried by one electron is sad to be the fundamental unit of electricityIts magnitude is 48times10-10esuOr 1602times10-19C Q3)Name the element containing no neutronAnsOrdinary hydrogen atom or protium 1H1

Types of AccountPersonal AccountReal AccountNominal AccountBalance Sheet (opening)

ldquowatermelon modelrdquoThis model could explain the electrical neutrality of an atom but failed to explain the result of scattering experiment carried out by Rutherford in 1911So it was rejected ultimately

Q4)Why is an electron called universal particleAns Itrsquos mass and Charge are independent of its source

EVS Chapter 1 ndash Modes of Existence

Modes of existence When one speaks normally about the mode of existence of some group or individual one refers to their customs their mode of being their ethology their habitat in some way their feeling for a placeDifferent modes of exixtence are ndash

1 Hunting ndashGathering2 Pastoral3 Agricultural4 Industrial

1 Hunting and gathering Hunting and gathering mode of existence is characterized by obtaining food from hunting wild animals including fishing and gathering wild plants From their earliest days the hunter-gatherer diet included various grasses tubers fruits seeds and nuts Lacking the means to kill larger animals they procured meat from smaller game or through scavenging

Societies that rely primarily or exclusively on hunting wild animals fishing and gathering wild fruits berries nuts and vegetables to support their diet are called hunting and gathering societies

At least this used to be practice of human beings before agriculture is invented As their brains evolved hominids developed more intricate knowledge of edible plant life and growth cycles

Q) Write the features of Hunting ndash gathering societiesAns - There are five basic characteristics of hunting and gathering societies

i The primary institution is the family which decides how food is to be shared and how children are to be socialized and which provides for the protection of its members

ii They tend to be small with fewer than fifty members

iii They tend to be nomadic moving to new areas when the current food supply in a given area has been exhausted

iv Members display a high level of interdependence

v Labor division is based on sex men hunt and women gather

Political Science

Introduction to political science

Political science occasionally called politology is a social science which deals with systems of governance and the analysis of political activities political thoughts associated constitutions and political behaviorThe study of political science involves the study of both the

Answer the following questions-1 What is political science

Political science occasionally called politology is a social science which deals with systems of governance and the analysis of political activities political thoughts associated constitutions and political behavior

2 Short notes-

traditional and modern theories of politicsTraditionalClassical political sciencepolitical theory-Traditional political science is the study of politics before Second World War The methodology to study Politics was traditional (legal formaletc) the definition of politics traditional (Politics begins and end with state)area of study (constitution state machinery)was traditionalModern Political scienceModern political theory-Modern Political Theory critically examines the contemporary state of political theory making an assessment of the achievement and limitations of the Behavioural Revolution in its totality and reviews objectively the major paradigms and conceptual frameworks adopted by the disciplineContemporary attempts at the development of an integrated political theory involving the use of both traditional and modern concepts approaches and theories-Around late 1960s several political scientists realized the importance of both the traditional political theory and modern Political theory They began building an integrated theory of politics involving a systematic mixture of traditional and modern studies of politics It was held that the study of a complex and vast field like politics needs both traditional as well as

Classical political theory Modern Political theory

Homework-Learn

modern concepts and approaches for studying itrsquos all aspects

Subject Eng Literature (The Tempest ndash William Shakespeare) Topic Act I Scene 1 Lines 1 to 32 (Line 32 ndash Gonzalo hellip If he be not born to be hanged our case is miserable) Date 13th April 2020 (3rd Period)

[Students should read the original play and also the paraphrase given in the school prescribed textbook]Summary Questions amp Answers

[SUMMARY OF THE ENTIRE SCENE]

o The play starts with the scene of a severe storm at sea Alonso (King of Naples) Sebastian (Alonsorsquos brother) Ferdinand (Alonsorsquos son) Gonzalo Antonio (the usurping Duke of Milan) are in a ship in the midst of the storm

o The mariners are trying their best to control the vessel from running aground and are totally following the orders of their Master the Boatswain They have scant success

o The mariners become extremely unhappy and annoyed when most of the passengers arrive on the deck thereby hampering their effort to save the ship There is serious confrontation between them and the passengers who are part of the Kingrsquos entourage

o The mariners could not save the ship

SUMMING-UP

(i) Vivid description of the scene which gives a realistic description of terror and confusion of a tropical storm

(ii) Shows Shakespearersquos accuracy of knowledge in describing the naval operations and also matters of seamanship

(iii) The opening scene justifies the title ndash The Tempest

UNANSWERED QUESTIONS

(i) The King always travels with his entire fleet including his soldiers Where

(1) GONZALO Nay good be patient (Line 15-26)BOATSWAIN When the sea is Hence What cares these

roarers for the name of the king To cabin silence Trouble us not

GONZALO Good yet remember whom thou has aboardBOATSWAIN None that I more love than myself You are a

councillor if you can command these elements to silence and work

the peace of the present we will not hand a rope more use your authority If you cannot give thanks you have

lived so long and make yourself ready in your cabin for the mischance of the hour if it so hap [To the Mariners]

Cheerly good hearts [To Gonzalo] Out of our way I say

(a) To whom is the boatswain speaking What does he mean by lsquoNone that I more love than myselfrsquo

The Boatswain is speaking to Gonzalo the honest old councilor of the Duke of MilanBy using the words ndash lsquoNone that I love more than I love myselfrsquo means that for the Boatswain nobody is dearer to him than his own life

(b) What were the conditions that made the boatswain react in this way

The Boatswain reacts in this way because the storm is at sea and Alonso King of Naples Sebastian his brother Ferdinand his son Gonzalo Antonio the usurping Duke of Milan on board are in distress and in panic Thus they have rushed to the deck interrupting the work of the mariners

(c) What hope does Gonzalo take from the attitude of the boatswain

The insolent and authoritative attitude of Boatswain makes Gonzalo feel comforted He tells that there are no signs that the Boatswain will be drowned But his facial appearance and attitude shows that he is destined to die on land by hanging which in effect means that all on board will be saved Otherwise all the persons on board are doomed

(d) How can they lsquomake yourself ready in your cabinrsquo For what were they asked to make ready themselves

In order to make themselves ready in their cabin the

were the other ships

(ii) Why was the ship in that area Where was it coming from or going where

(iii) The ship broke apart What happened to those who were in the ship

passengers on board must prepare for death which they will possibly soon have to meetThey can retire to their cabins and offer prayers to the Almighty to save them from drowning

(e) What does the boatswain say when he is asked to be patient What does he order to the royal party

When the boatswain is asked to be patient and remain calm he says that he will be patient only when the storm will be over and the sea will be calm but as long as the storm blows and there is danger to the ship he cannot think of being patient He orders the royal party to go to the cabin and leave the mariners to their work

(2) GONZALO I have great comfort from this fellow (Line 27-36)

Methinks he hath no drowning mark upon him his complexion is perfect

gallows Stand fast good Fate to his hanging Make the rope of his destiny our cable for our own doth little advantage If he be not born to be hanged our case is miserable

(a) Why does Gonzalo regard the Boatswain in the midst of danger

In the midst of danger Gonzalo regards the boatswain because he feels that the Boatswain is a source of comfort and is bent upon to do his work sincerely which in this case is saving the ship and its passengers from the severest of raging storm

(b) What reasons does Gonzalo give when he says that none in the ship will die of drowning

Gonzalo is almost sure that none in the ship will die by drowning His says that there is no mark on the face of the boatswain that indicates that he will die by drowning On the other hand the lines on his face are strong indications that he will be hanged to death Therefore there shall be no danger of the shiprsquos sinking

(c) Explain the following ldquoStand fast good Fate to his hanging Make the rope of his destiny our cable for our own doth little advantage If he be not born to be hanged our case is miserablerdquo

The stated lines mean that if the will of destiny is to be carried out then the ship will not get wrecked and all the passengers will be saved The safety of the passengers therefore depends upon the will of fate being carried out in the case of the boatswain If however the boatswain is not to die by hanging then the passengers are also very unsafe because in that case the ship is likely to sink

(d) What order does the Boatswain give to the sailors

when he re-enters What does he say about the crying of the fellows inside the cabin

The boatswain orders the sailors to bring the topmast lower and bring the ship close to a stationary position with the help of the main sail He says that the fellows inside the cabin are moaning and crying in their distress louder than his voice and louder even than the roaring of the storm

Class XII (ScienceCommerceHumanities) Subject Topic Summary Execution

Computer Science

PropositionalLogic

Propositional logic is a procedure to provide reasoning through statementProposition A ststement that results in True or False is said to be proposition There are two types of propositionSimple proposition amp compound propositionSimple proposioton A simple proposition is one that is not a part of any other proposition Such sentential form of proposition is symbolized with english letters in short For example Ram is a claver student (TrueFalse)Where do you live (Not in True or False)Grapes are sweet (TrueFalse)It rains today (TrueFalse)Here we can see some statements anwer would be true or false but some staements answer can not give in terms of true or false Thus the sentences which can be answered in true or false are known as simple propositionAssigning propositon to a variableThe general syntax to assign propostion to a variable is as followsVariable = Simple propositonFor example A=Ram is a clever studentB= Grapes are sweetC= it rains todayCompound proposition

helliphellipto be continued in next classhelliphellipMath Relation Relation If A and B are two non-empty sets

then a relation R from A to B is a subset of AxB If R A x B and (a b) R then we say that a sube isinis related to b by the relation R written as aRbeg Let A be the set of students of class XII and B be the set of students of class XI Then some of the examples of relation from A to B arei) (a b) AXB a is brother of bisinii) (a b) AXB age of a is more than age of isinb Types of relation In this section we would like to study different types of relations We know that a relation in a set A is a subset of A times A Thus the empty set φ and A times A are two extreme relations For illustration consider a relation R in the set A = 1 2 3 4 given by R = (a b) a ndash b = 10 This is the empty set as no pair (a b) satisfies the condition a ndash b = 10 Similarly R = (a b) | a ndash b | ge 0 is the whole primeset A times A as all pairs (a b) in A times A satisfy | a ndash

Example 1 Let A be the set of all students of a boys school Show that the relation R in A given by R = (a b) a is sister of b is the empty relation and R = (a b) the primedifference between heights of a and b is less than 3 meters is the universal relationSolution Since the school is boys school no student of the school can be sister of any student of the school Hence R = φ showing that R is the empty relation It is also obvious that the difference between heights of any two students of the school has to be less than 3 meters This shows that R = A times A is primethe universal relation Example 2 Show that the relation R in the set 1 2 3 given by R = (1 1) (2 2) (3 3) (1 2) (2 3) is reflexive

b | ge 0 These two extreme examples lead us to the following definitionsDefinition 1 A relation R in a set A is called empty relation if no element of A isrelated to any element of A ie R = φ A times AsubDefinition 2 A relation R in a set A is called universal relation if each element of A is related to every element of A ie R = A times A Both the empty relation and the universal relation are some times called trivial relation Definition 3 A relation R in a set A is called(i) reflexive if (a a) R for every a Aisin isin(ii) symmetric if (a1 a2) R implies that (aisin 2a1)

R for all aisin 1 a2 Aisin(iii) transitive if (a1 a2) R and (aisin 2 a3) R isinimplies that (a1 a3) R for all aisin 1 a2 a3 AisinDefinition 4 A relation R in a set A is said to be an equivalence relation if R is reflexive symmetric and transitive

but neither symmetric nor transitiveSolution R is reflexive since (1 1) (2 2) and (3 3) lie in R Also R is not symmetric as (1 2) R but (2 1) isin notinR Similarly R is not transitive as (1 2) R and (2 3) R but (1 3) R isin isin notinExample 3 Show that the relation R in the set Z of integers given byR = (a b) 2 divides a ndash b is an equivalence relationSolution R is reflexive as 2 divides (a ndash a) for all a Z isinFurther if (a b) R then 2 divides a isinndash b Therefore 2 divides b ndash a Hence (b a) R which shows that R is isinsymmetric Similarly if (a b) R and (b c) R isin isinthen a ndash b and b ndash c are divisible by 2 Now a ndash c = (a ndash b) + (b ndash c) is even (Why) So (a ndash c) is divisible by 2 This shows that R is transitive Thus R is an equivalence relation in ZExample 4 Let L be the set of all lines in a plane and R be the relation in L defined as R = (L1 L2) L1 is perpendicular to L2 Show that R is symmetric but neither reflexive nor transitiveSolution R is not reflexive as a line L1 can not be perpendicular to itself ie (L1 L1) R notinR is symmetric as (L1 L2) Risin

L1 is perpendicular to L2rArr L2 is perpendicular to L1rArr (L2 L1) RrArr isin

R is not transitive Indeed if L1 is perpendicular to L2 and L2 is perpendicular to L3 then L1 can never be perpendicular to L3 In fact L1 is parallel to L3 ie (L1 L2) R isin(L2 L3) R but (L1 L3) Risin notin

Chemistry Solid state Characteristics if Solids(i)The particles are locked in fixed positions they are unable to change their relative positions and this brings a definite shape and volume of a solid(ii)In a solid the constituent particles are held by strong forces of attractionThe forces of attraction may be bonding or non bonding(iii)The constituent particles in a solid pack together as closely as possibleoccupying most of the available space within the solidThus the empty space in a solid is very smallThis makes a solid highly rigid and nearly incompressibleThis also explains why a solid has high density and exhibits slow diffusionClassification of Solids

Q1)Define Crystalline solids AnsA Solid that has a definite geometrical shape and a sharp melting pointand whose constituent particles (atomsmolecules or ions) are arranged in a long range order of definite pattern extending throughout the solid is called a crystalline solidExNaClQ2)Define Amorphous solids AnsA solid that does not have a definite shape and a sharp melting pointand whose constituent particles (atomsmolecules or ions) are not arranged in a definite pattern is called an amorphoussolid

Crystalline solidsAmorphous solids

ExGlassRubberQ3)Classify Crystalline Solids Crystalline Solids

Physics Coloumbrsquos Law (Summary)

Before Going Into Coloumbrsquos Law We Will First Learn What is Charge Properties of Charge and Always remember that charge is quantized ie a body always have static charge of magnitude equal to some integral multiple of fundamental electronic charge e= 16 x 10- 19 C

Charge is the property of matter that causes it to produce and experience electrical and magnetic effects The study of the electrical charges at rest is called electrostatics When both electrical and magnetic effects are present the interaction between charges is referred to as electromagnetic

There exist two types of charges in nature positive and negative Like charges repel and unlike charges attract each other

The type of charge on an electron is negative The charge of a proton is the same as that of an electron but with a positive sign In an atom the number of electrons and the number of protons are equal The atom is therefore electrically neutral If one or more electrons are added to it it becomes negatively charged and is designated as negative ion However if one or more electrons are removed from an atom it becomes positively charged and is called a positive ion

The excess or deficiency of electrons in a body gives the concept of charge If there is an excess of electrons in a body it is negatively charged And if there is deficiency of electrons the body becomes positively charged Whenever addition or removal of electrons takes places the body acquires a charge

The SI Unit of charge is coulomb (C) In SI units the current is a fundamental quantity having a unit of ampere (A) The unit of charge is defined in terms of the unit of current Thus one coulomb is the charge transferred in one second across the section of a wire carrying a

Ionic SolidsMetallicSolids

Molecular Solids

current of one ampere

As q = It we have1 C = (1 A) (1 s)

The dimensions of charge are [A T]

Properties of Charge

(1) Quantization of Charge Electric charge can have only discrete values rather than any value That is charge is quantized The smallest discrete value of charge that can exist in nature is the charge on an electron given as

e = plusmn 16 x 10- 19 C

This is the charge attained by an electron and a protonA charge q must be an integral multiple of this basic unit That is

Q = plusmn ne where n = 1 2 hellip

Charge on a body can never be (frac12)e (23)e or 57e etcWhen we rub a glass rod with silk some electrons are transferred from the rod to the silk The rod becomes positively charged The silk becomes negatively charged The coulomb is a very large amount of charge A typical charge acquired by a rubbed body is 10 - 8 C

Biology Reproduction in organisms

Welcome to this new session 2020-21Today in this first chapter we mainly discuss about reproduction types needs and life span of some organismsWe also discuss about difference between sexual and asexual reproduction

Q1 What is reproductionReproduction is defined as a biological processin which an organism gives rise to young onessimilar to itselfQ2 What are the needs of reproductionbulli) Reproduction maintain life on earthii) It enables the continuity of the species generation after generationiii) It creates genetic variation among populationsQ3 Define Life span and write some orgnisms life spanbull Life span is the period from birth to

the natural death of an organism- OrganismsLife span1 Butterfly 1 - 2 weeks2 Fruit fly 30 days3Dog 10-13 years4 Rose5-7 years5 Tortoise100-150 years6 Banyan Tree -200 - 250 yearsQ4 Reproduction is of two types in case ofanimals but in case of plants vegetative propagation is also present

Asexual Reproduction Sexual Reproductioni) Always uniparentalii) Gametes are not involvediii) Only mitotic division involvediv) Somatic cells of parents are involvedv) Offsprings are genetically similar to the parents

i) Usually biparentalii) Gametes are involvediii) Meiosis occurs during gametogenesis Mitosis occurs after fertilisationiv) Germ cells of the parents are involvedv) offsprings are genetically different from the parents

COMMERCE BUSINESS ENVIRONMENT

Welcome to the new sessiontoday we are going to start the first chapter of Class XII The name of the chapter is Business Environment

Already many of you have got some idea about the word business environment form the first chapter of business studies in class XI

In todayrsquos world every business enterprise is a part of the society It exists and operates in association with various groups in society such as customers suppliers competitors banks and financial institutions government agencies trade unions media and so on All these groups influence the functioning of business in one way or the other They constitute the environment of businessConcept of Business Environment

The term lsquobusiness environmentrsquo refers to the sum total of all individuals institutions and other forces that lie outside a business enterprise but that may influence its functioning and performance

The main features of business environment Totality of External forces General and Specific forces Interrelatedness Complexity Dynamic Uncertainty

Prepare the following questions from todayrsquos assignment

2 What do you mean by business environment

The term lsquobusiness environmentrsquo means the aggregate of all forces factors and institutions which are external to and beyond the control of an individual business enterprise but they may influence its functioning and performance Business environment is the macro framework within which a business firm a micro unit operates It consists of several interrelated and interacting elements

2 Explain the main features of business environment in brief

Totality of External forces-Business environment is the sum total of all things external to a business environment

General and Specific forces-It

Relativity

The Interrelation between business and its environment

The business enterprise is an open system It continuously interacts with its environment It takes inputs (such as raw materials capital labour energy and so on) from its environment transforms them into goods and services and sends them back to the environment

Fig 1 Business Environment Relationship

includes both the forces general forces are the economic social political legal and technological conditions which indirectly influence all business enterprise Specific forces are the investors customers competitors and suppliers which influence individual enterprise directly

Interrelatedness-Different elements of environment are interrelated for an example growing awareness for health care has increased the demand for health foods

Complexity- Business environment id complex in nature as the elements keep on changing example economic technological and other forces changes in demand for a product and service

Dynamic-Business environment is not static it keeps on changing

Uncertainty- Itrsquos very difficult to predict future events such as technology and fashion which occur fast and frequently

Business Studies

Human Resources Management

Human resource of an organisation are the aggregate of knowledge skills attitudes of people working in it

The management system which deals with human resources is called human resource management

Features of HRMbullComprehensive functionbullPeople-oriented

Question1) What do you mean by human

resource management Answer) Human resource management may be defined as that field of Management which has to do with planning organising and controlling the functions of procuring developing maintaining and utilising the labour force

bullAction oriented bullPervasive function bullContinuous function

2) Explain the features of HRM in brief

Answer)bullHuman Resource Management is concerned with managing people at work bull Human Resource Management is concerned with employees which bring people and organisations together so that the goals of each are met bullHuman resource management considered every employees as an individual and also promote their satisfaction and growth bull Human resource management is inherent in all organisations and at all levelsbullManagement of human resources are ongoing on never ending process which requires a constant alertness and Awareness of human relations

3) ldquoHR function is said to be pervasiverdquowhy

Answer) Human resource management is required in all organisations whether it is private or government organisations armed forces sports organisations etc It permeatsall the functional areas like production marketing finance research etc This from this feature of human resource management it can be said that it is pervasive in nature

Economics Demand Q1DEFINITION OF DEMANDIn economics demand is the quantity of a good that consumers are willing and able to purchase at various prices during a given period of timeQ2DEMAND CURVEIn economics a demand curve is a graph depicting the relationship between the price of a certain commodity and the quantity of that commodity that is demanded at that pricQ3LAW OF DEMANDIn microeconomics the law of demand states that conditional on all else being equal as the price of a good increases quantity demanded decreases conversely as the price of a good decreases quantity demanded increasesQ4ASSUMPTION of LAW OF DEMAND(i)No change in price of related commodities(ii) No change in income of the consumer(iii) No change in taste and preferences customs habit and fashion of the consumer( No expectation regarding future change in priceQ5MARKET DEMAND SCHEDULEIn economics a market demand schedule is a tabulation of the quantity of a good that all consumers in a market will purchase at a

given price At any given price the corresponding value on the demand schedule is the sum of all consumersrsquo quantities demanded at that priceQ6INDIVIDUAL DEMAND SCHEDULEIndividual demand schedule refers to a tabular statement showing various quantities of a commodity that a consumer is willing to buy at various levels of price during a given period of timeQ7 FACTORS AFFECTING INDIVIDUAL DEMAND FOR A COMMODITY

The factors that influence a consumerrsquos decision to purchase a commodity are also known as determinants of demand The following factors affect the individual demand for a commodity1 price of the commodity2 price of related goods3 income of buyer of the commodity4 tastes and preferences of the buyer1 Price of the CommodityYou must have observed that when price of a commodity falls you tend to buy more of it and when its price rises you tend to buy less of it when all other factors remain constant (lsquoother things remaining the samersquo) In other words other things remaining the same there is an inverse relationship between the price of a commodity and its quantity demanded by its buyers This statement is in accordance with law of demand which you will study in the later part of this lesson Price of a commodity and its quantity demanded by its buyers are inversely related only when lsquoother things remain the samersquo So lsquoother things remaining the samersquo is an assumption when we study the effect of changes in the price of a commodity on its quantity demanded2 Price of Related goodsA consumer may demand a particular good But while buying that good heshe also asks the price of its related goods Related goods can be of two types-(i) Substitute goods(ii) Complementary goods While purchasing a good prices of its substitutes and complements do affect its quantity purchased(i) Price of Substitute Goods Substitute goods are those goods which can easily be used in place of one another for satisfaction of a particular want like tea and coffee An increase in price of substitute good leads to an increase in demand for the given commodity and a decrease in price of substitute good leads to a decrease in demand for the given commodity It means demand for a given commodity is directly affected by change in price of substitute goods For example if price of coffee increases the demand for tea will rise as tea will become relatively cheaper in comparison to coffee(ii) Price of Complementary goods Complementary goods are those goods which are used together to satisfy a particular want like car and petrol An increase in the price of complementary goods leads to a decrease in demand for the given commodity and a decrease in the price of complementary goods leads to an increase in demand for the given commodity For example if price of petrol falls then the demand for cars will increase as it will be relatively cheaper to use both the goods together So demand for a given commodity is inversely affected by change in price of complementary goods3 Income of the Buyer of CommodityDemand for a commodity is also affected by income of its buyer However the effect of change in income on demand depends on the nature of the commodity under consideration In case of some goods like full cream milk fine quality of rice (Basmati rice) etc demand for these commodities increases when income of the buyer increases and

demand for these commodities decreases when income of the buyer decreases Such goods whose demand increases with the increase in income of the buyer are called normal goods But there are some goods like coarse rice toned milk etc whose demand decreases when income of buyer increases and their demand increases when income of the buyer decreases Such goods whose demand decreases with the increase in income of the buyer are called inferior goods Suppose a consumer buys 10 Kgs of rice whose price is ` 25 per Kg He cannot afford to buy better quality of rice because the price of such rice is ` 50 per Kg The consumer is spending ` 250 per month on the purchase of rice Now if income of the consumer increases and he can afford ` 350 on purchase of 10 Kg of rice Now he can afford to buy some quantity of rice say 6 Kgs whose price is ` 25 per Kg and may buy 4 Kgs of rice whose price is ` 50 per Kg Thus he will buy 10 Kgs of rice by spending ` 350 per month Therefore we may conclude that demand for normal goods is directly related to the income of the buyer but demand for inferior goods is inversely related to the income of the buyer4 Tastes and Preferences of the BuyerThe demand for a commodity is also affected by the tastes and preferences of the buyers They include change in fashion customs habits etc Those commodities are preferred by the consumers which are in fashion So demand for those commodities rises which are in fashion On the other hand if a commodity goes out of the fashion its demand falls because no consumer will like to buy it(5) Number of Buyers in the Market(Population)Increase in population raises the market demand whereas decrease in population reduces the market demand for a commodity Not only the size of population but its composition like age (ratio of males females children and old people in population) also affects the demand for a commodity It is because of needs of children young old male and female population differs(6) Distribution of Income and WealthIf the distribution of income and wealth is more in favour of the rich demand for the commodities preferred by the rich such as comforts and luxuries is likely to be higher On the other hand if the distribution of income and wealth is more in favour of poor demand for commodities preferred by the poor such as necessities will be more(7) Season and Weather ConditionsThis is generally observed that the demand for woolens increases during winter whereas demand for ice creams and cold drinks increases during summer Similarly market demand for umbrellas rain coats increases during rainy seasonQ8 REASONS FOR OPERATION OF LAW OF DEMAND WHY DEMAND CURVE SLOPES DOWNWARDNow we will try to explain why does a consumer purchase more quantity of a commodity at a lower price and less of it at a higher price or why does the law of demand operate ie why does the demand curve slope downwards from left to right The main reasons for operation of law of demand are1 Law of Diminishing Marginal UtilityAs you have studied earlier law of diminishing marginal utility states that as we consume more and more units of a commodity the utility derived from each successive unit goes on decreasing The consumer will be ready to pay more for those units which provide him more utility and less for those which provide him less utility It implies that he will purchase more only when the price of the commodity falls2 Income Effect

When price of a commodity falls purchasing power or real income of the consumer increases which enables him to purchase more quantity of the commodity with the same money income Let us take an example Suppose you buy 4 ice creams when price of each ice cream is ` 25 If price of ice creams falls to ` 20 then with same money income you can buy 5 ice creams now3 Substitution EffectWhen price of a commodity falls it becomes comparatively cheaper as compared to its substitutes (although price of substitutes has not been changed) This will lead to rise in demand for the given commodity For example if coke and Pepsi both are sold at ` 10 each and price of coke falls Now coke has become relatively cheaper and will be substituted for Pepsi It will lead to rise in demand for coke4 Change in Number of BuyersWhen price of a commodity falls some old buyers may demand more of the commodity at the reduced price and some new buyers may also start buying this commodity who were not in a position to buy it earlier due to higher price This will lead to increase in number of buyers when price of the commodity falls As a result demand for the commodity rises when its price falls5 Diverse Uses of a CommoditySome commodities have diverse uses like milk It can be used for drinking for sweet preparation for ice cream preparation etc If price of milk rises its use may be restricted to important purpose only This will lead to reduction in demand for other less important uses When price of milk falls it can be put to other uses also leading to rise n demand for itQ9 EXCEPTIONS TO THE LAW OF DEMANDYou have studied in law of demand that a buyer is willing to buy more quantity of a commodity at a lower price and less of it at a higher price But in certain circumstances a rise in price may lead to rise in demand These circumstances are called Exceptions to the Law of Demand Some important exceptions are1 Giffen GoodsGiffen goods are special type of inferior goods in which negative income effect is stronger than negative substitution effect Giffen goods do not follow law of demand as their demand rises when their price rises Examples of Giffen goods are jowar and bajra etc2 Status Symbol GoodsSome goods are used by rich people as status symbols eg diamonds gold jewellary etc The higher the price the higher will be the demand for these goods When price of such goods falls these goods are no longer looked at as status symbol goods and tehrefore therir demand falls3 NecessitiesCommodities such as medicines salt wheat etc do not follow law of demandbecause we have to purchase them in minimum required quantity whatever their price may be4 Goods Expected to be ScarceWhen the buyers expect a scarcity of a particular good in near future they start buying more and more of that good even if their prices are rising For example during war famines etc people tend to buy more of some goods even at higher prices due to fear of their scarcity in near future

Political Science

Constitution of India-The

Preamble

The preamble-

Preamble-

The preamble is the most precious part of the constitution We the people of India having solemnly resolved to constitute India into a Sovereign Socialist Secular Democratic Republic and to secure to all its citizensA preamble is an introductory and expressionary statement in a document that explains the documents purpose and underlying philosophy When applied to the opening paragraphs of a statute it may recite historical facts pertinent to the subject of the statuteNature and purpose of the constitution-Purpose of the Constitution dictates permanent framework of the government to form a more perfect union to establish justice and ensure peace of thenationconstitution provide principles how the government can run itself following the rules and laws written in the constitution of each state keeps them balanced

Answer the following questions-

1 What is preambleA preamble is an introductory and expressionary statement in a document that explains the documents purpose and underlying philosophy2 What is the nature and

purpose of the constitutionConstitution dictatespermanent framework of the government to form a more perfect union to establish justice and ensure peace of the nation

Homework-Learn

Accounts Compatibilty mode

1MEANING OF PARTNERSHIPPartnership is a form of business organisation where two or more persons join hands to run a business They share the profits and losses according to the agreement amongst them According to the Indian Partnership Act 1932 ldquoPartnership is relation between persons who have agreed to share profits of a business carried on by all or any one of them acting for allrdquo For example one of your friends has passed class XII from National Institute of Open Schooling (NIOS) and wants to start a business Heshe approaches you to join in this venture Heshe wants you to contribute some money and participate in the business activities Both of you if join hands constitute a partnership2CHARACTERISTICS1048698 Agreement A partnership is formed by an agreement The agreement may be either oral or in writing It defines the relationship between the persons who agree to carry on business It may contain the terms of sharing profit and the capital to be invested by each partner etc The written agreement is known as partnership deed1048698 Number of persons There must be at least two persons to form a partnership

The maximum number of partners in a partnership firm can be 50 according toCompanies Act 20131048698 Business The Partnership is formed to carry on business with a purpose of earning profits The business should be lawful Thus if two or more persons agree to carry on unlawful activities it will not be termed as partnership1048698 Sharing Profits The partners agree to share profits in the agreed ratio In caseof loss all the partners have to bear it in the same agreed profit sharing ratio10486981048698Mutual Agency Every partner is an agent of the other partners Every partner can bind the firm and all other partners by hisher acts Each partner will be responsible and liable for the acts of all other partners10486981048698Unlimited liability The liability of each partner except that of a minor is unlimited Their liability extends to their personal assets also If the assets of the firm are insufficient to pay off its debts the partnersrsquo personal property can be used to satisfy the claim of the creditors of the partnership firm10486981048698Management All the partners have a right to mange the business However they may authorize one or more partners to manage the affairs of the business on their behalf10486981048698Transferability of Share No partner can transfer hisher share to any one including hisher family member without the consent of all other partners3PARTNERSHIP DEEDAgreement forms the basis of partnership The written form of the agreement is which a document of partnership is It contains terms and conditions regarding the conduct of the business It also explains relationship between the partners This document is called partnership deed Every firm can frame its own partnership deed in which the rights duties and liabilities of the partners are stated in detail It helps in settling the disputes arising among the partners during the general conduct of business 4CONTENTS OF PARTNERSHIP DEEDThe partnership deed generally contains the following (i) Name and address of the partnership firm(ii) Nature and objectives of the business(iii) Name and address of each partner(iv) Ratio in which profits is to be shared(v) Capital contribution by each partner(vi) Rate of Interest on capital if allowed(vii) Salary or any other remuneration to partners if allowed(viii) Rate of interest on loans and advances by a partner to the firm(ix) Drawings of partners and interest thereon if any(x) Method of valuation of goodwill and revaluation of assets and liabilities on the reconstitution of the partnership ie on the admission retirement or death of a partner(xi) Settlement of disputes by arbitration(xii) Settlement of accounts at the time of retirement or death of a partner5IN ABSENCE OF PARTNERSHIP DEEDThe partnership deed lays down the terms and conditions of partnership in regard to rights duties and obligations of the partners In the absence of partnership deed there may arise a controversy on certain issues like profit sharing ratio interest on

capital interest on drawings interest on loan and salary of the partners In such cases the provisions of the Indian Partnership Act becomes applicableSome of the Issues are(i) Distribution of Profit Partners are entitled to share profits equally(ii) Interest on Capital Interest on capital is not allowed(iii) Interest on Drawings No interest on drawing of the partners is to be charged(iv) Interest on Partnerrsquos Loan A Partner is allowed interest 6 per annum on the amount of loan given to the firm by himher(v) Salary and Commission to Partner A partner is not entitled to anysalary or commission or any other remuneration for managing the business

History TOPIC-TOWARDS INDEPENDENCE AND PARTITION THE LAST PHASE (1935-1947)

SUB TOPIC-IMPORTANT POLITICAL DEVELOPMENTS ndash GROWTH OF SOCIAL IDEAS

Socialism is a political social and economic philosophyLike in other parts of the world the Russian revolution of 1917 served as a great inspiration for revolutionaries in India who at that time were engaged in the struggle for liberation from British ruleSocialist ideas led to the formation of communist party of IndiaJAWAHARLAL NEHRU Among the early Congress leaders Jawaharlal Nehru was very much impressed and influenced by the Socialist ideas He also learnt about the Economic activities of the Soviet Union after the Bolshevic Revolution 1917 He made full use of them in IndiaThe election of Jawaharlal Nehru and Subhas Chandra Bose showed the Left wing tendency within CongressJawaharlal Nehru demanded economic freedom along with political freedom of the people in order to end the exploitation of masses

Nehrus working committee included three socialists leaders The Lucknow session was a landmark in the evolution of socialist ideas of the congressSUBHAS CHANDRA BOSE ndash Subhas Chandra Bose had socialist leaning Both Jawaharlal Nehru and Subhas Chandra Bose were known as leftist Congress men Later on National Congress divided into Leftist and rightist campCONGRESS SOCIALIST Within the Congress some leaders formed the Congress Socialist partyPattavi Sitaramyya Sardar Patel Rajendra Prasad had hostile attitude towards the Congress Socialist partyJawaharlals attitude was hesitant

1 QUESTION ndash Mention name of two Congress leaders who had socialist leaning

1ANSWER ndash Subhas Chandra Bose and Jawaharlal Nehru2QUESTION- In which session of the congress Jawaharlal elaborated his Socialist ideas2 ANSWER ndash Lucknow and Faizpur Session in December 1935 and 19363QUESTION ndash Why Congress was sharply divided into leftist and rightist camp 3ANSWER ndash Subhas Chandra Bosersquos attempt to seek re election for congress presidentship in 1939sharply divided the National Congress into Leftist and Rightist camp4 QUESTION ndash Who was MN Roy 4 ANSWER ndash Manabendra Roy first formed the Communist Party of India outside the country at Tashkent in 19205QUESTION ndash Who formed the Congress Socialist Party within the Congress5 ANSWER ndash Jaya Prakash Narayan Achyut Patwardhan Acharya Narendra Dev Ram Mohan Lohia Aruna Asaf Ali6QUESTION ndash When was the Congress Socialist Party formed What was its object6 ANSWER ndash 1934The Congress Socialist Party sought to work out socialist programme through the Congress They joined hands with the Congress and wanted to carry

Subhas Chandra Bose being expelled from the congress after the Tripuri rift he formed Forward BlockThere were basic differences between the Congress Socialists and the communistsTRADE UNION ACTIVITIES Maximum working class people lived in Bombay and Calcutta The working and living conditions of those workers were very miserable In this situation Shasipada Banerjee NM Lokhande protested against the oppression of the working class peopleThe first Trade Union Madras Labour Union was formed in 1918 by BP WadiaIndustrial strikes took place in Kanpur Calcutta Madras Jamshedpur and Ahmedabad AITUC was formed in Bombay in 1927 The growth of Trade union among the workers was slow because of the fear of the dismissal of the jobIn the mean time the Moderates as well as Communists left AITUC and formed separate organization

on National struggle with the help of workers and peasant class of the society7 QUESTION ndash What was the name of the party founded by Subhas Chandra Bose7 ANSWER- Forward Block8QUESTION ndash Who was Shasipada Banerjee8 ANSWER ndash Shasipada Banerjee was a radical Brahmo He founded a working menrsquos club to protest against exploitation of the British rulers towards the working class of India9 QUESTION ndash What was the weekly published by NM Lokhande9ANSWER- Dinabandhu10 QUESTION ndash Who founded Bombay Mill-Hands Association and in which year10 ANSWER- NM Lokhande in189011 QUESTION- Who was BP WadiaANSWER- BPWadia was the founder of Madras Labour Union in191812 QUESTION- What was the name of the first labour union of India12 ANSWER- Madras Labour Union13 QUESTION Who founded the Majur Mahajan 13 ANSWER GANDHIJI14 QUESTION What was the full form of AITUC When it was formed14 ANSWER All India Trade Union Congressin 192715QUESTION Who formed the Red Trade Union Congress and in which year15ANSWER The Communists formed the Red Trade Union Congress16 QUESTION What do you mean by Socialism16 ANSWER Socialism describes any political and economic theory that says the community rather than individuals should own and manage property and natural resources

Subject Eng Literature (The Tempest ndash William Shakespeare) Topic Act III Scene 3 Lines 1 to 52 (Line 52 ndash Brother my lord the Duke Stand to and do as we) Date 13th April 2020 (4th Period)

[Students should read the original play and also the paraphrase given in the school prescribed textbook]Summary Questions amp Answers

o Alonso Sebastian Antonio Gonzalo Adrian Francisco and others wandered about the island in search of Ferdinand and gets tired and hungry of the toil and at the same time gives up all hope of finding him

o Antonio and Sebastian are happy that Alonso is out of hope and decide to make another attempt on his life that night when being so tired they will be sleeping soundly

o Suddenly a solemn and strange music is heard in the air and several strange shapes enter bringing in a banquet These strange shapes then dance round it with gestures of salutation and then inviting the King to eat they depart

o Seeing this strange scene all are inclined to believe the tales told by travelers that there truly are ldquounicornsrdquo and ldquothe phoenixrsquo thronerdquo

1 ALONSO What harmony is this My good friends hark (L18-27)

GONZALO Marvellous sweet music

[Enter several strange shapes bringing in a banquet

they dance about it with gentle actions of salutation

and inviting the King and his companions to eat they depart]ALONSO Give us kind keepers heavens What were theseSEBASTIAN A living drollery Now I will believe

That there are unicorns that in Arabia

There is one tree the phoenixrsquo throne one phoenix

At this hour reigning thereANTONIO Ill believe both

And what does else want credit come to me

And Ill be sworn rsquotis true Travellers neer did lie

Though fools at home condemn rsquoem

(a) How did Prospero present an amazing spectacle before Alonso and his companions

Using his magic powers Prospero ordered strange shapes to lay a banquet before Alonso and his companions The shapes brought several dishes with tasty eatables in them They placed the dishes on a table before Alonso and his companions Then the strange shapes began to dance gracefully around the banquet While dancing they made gestures inviting them to eat the food Then suddenly the shapes disappeared(b) Who were the guests at the strange banquet Describe the lsquoliving drolleryrsquo

Alonso Sebastian Antonio Gonzalo Adrian and Francisco were the guests at the strange banquet

The term ldquoliving drolleryrdquo refers to live entertainment show In this context when Alonso the King of Naples Sebastian his brother Antonio the treacherous brother of Prospero Gonzalo the kind and loyal councillor to the King Adrian and Francisco came to the island they were hungry and weary in their spirits They heard a solemn and strange music They were shocked to see several strange shapes bringing in a banquet and these shapes danced about it with gentle action of salutation inviting the King and his companions to eat After this Sebastian described this show as lsquoliving drolleryrsquo(c) What is lsquophoenixrsquo What are lsquoUnicornsrdquo

The term lsquophoenixrsquo refers to a mythical Arabian bird which lived alone and perched on a solitary tree After one hundred years it expired in flames and rose again from its own ashes

lsquoUnicornsrsquo refers to the mythological four-footed beasts having horns in the centre of their foreheads When the horns are ground into powder the powder was believed to be

an aphrodisiac(d) How does Sebastian explain the puppet show OR Why does the speaker now believe in unicorns and phoenix

Sebastian finds several strange shapes bringing in the banquet They invite the king and his party for dinner and soon depart He tells that if such a strange sight can be a reality there is nothing incredible in the world and from the present moment he will believe anything He says that it is a strange dumb show enacted not by puppets but by living beings It is stranger than a travellerrsquos tale Seeing such a thing

before his own eyes he will no longer disbelieve the story about unicorns and phoenix(e) How do the other characters present respond to this living drollery

At the sight of the lsquoliving drolleryrsquo like Sebastian Gonzalo and Antonio too acted strangely Antonio told that he too now believes in unicorns and phoenix and anything else that seems to be incredible He too now believes in travellersrsquo tales Gonzalo told that if he would report those happenings in Naples nobody will believe him He considers that those gentle shapes were gentler in manner in comparison to the living beings Alonso was at first sight suspicious and told them that those strange shapes conveyed their meaning in expressive gestures when they seemed to lack speech by their movements and sounds Francisco was amazed at their mysterious disappearance

2 ALONSO Not I

(Line 43-52)GONZALO Faith sir you need not fear When we

were boysWho would believe that there were mountaineers

Dewlapped like bulls whose throats had hanging at rsquoem

Wallets of flesh Or that there were such men

Whose heads stood in their breasts Which now we find

Each putter-out of five for one will bring us

Good warrant ofALONSO I will stand to and feed

Although my lastmdashno matter since I feel

The best is past Brother my lord the Duke

Stand to and do as we

(a) How does Alonso respond at the spectacle of the shapes which were sent to them at the instruction of Prospero

After seeing the strange sight of appearing and disappearing of the shapes sent by Prospero to arrange a banquet for them Alonso says that his surprise at having seen those creatures is infinite and he is fully justified in feeling so much surprise He thinks that their shapes their gestures and the sounds they made were indeed amazing Although they do not possess the gift of speech yet they were able to convey their

thoughts by means of their gestures only

(b) What does Prospero say about the views expressed by Alonso regarding the shapes What does Francisco think about the shapesAfter hearing Alonsorsquos views about the shapes Prospero says that this manrsquos praise of the spirits is rather hasty He means to say that Alonso has shown great haste in reaching the conclusion about the shapes Francisco is amazed to see that those shapes disappeared in a mysterious way(c) What does Sebastian ask Alonso to doSebastian tells Alonso that the shapes having disappeared should not matter to them because they have left the eatables behind He asks Alonso to enjoy eating as they are extremely hungry but the king does not accept his offer of enjoying the dishes(d) How does Gonzalo try to dispel Alonsorsquos fear of those strange shapes What kind of references does he give to AlonsoGonzalo says that those who have travelled abroad have reported seeing even stranger sights than these shapes that Alonso and his companions have beheld Hence there is no reason to feel afraid of these shapes Gonzalo further adds that in his younger days he had heard strange stories from travelers and Alonso might have heard similar stories For instance it was said that there existed a certain race of

human beings who had huge lumps of flesh hanging at their throats and who therefore resembled bulls Then Gonzalo tells about a race of human beings whose heads were located at their breasts Gonzalo says that such stories were not believed by most people in those days but now-a-days these stories have become common(e) Explain the following lsquoEach putter-out of five for onersquoEnglish travellers often insured their trips with London brokers Those that went on foreign travels those days used to deposit a certain amount with some firm or company in London before their departure If the travelers failed to return the money was forfeited by the company with which it had been deposited But this money was repaid five-fold if the travelers returned safe and sound In this way a traveler stood a great chance of recovering the entire cost of his

travels(f) Give the explanatory meanings of the following expressions in the context of the above extract (i) Dewlapped (ii) Wallets of flesh

(iii) Putter-out(i) Dewlapped having big lumps of flesh at the necks(ii) Wallets of flesh large masses of flesh looking like bags(iii) Putter-out to invest money before commencing the travel

  • General methods of preparation of hydrogen
  • Chapter Dimensional Analysis (Summary)
    • Properties of Charge
Page 17:   · Web viewSubject. Topic. Summary. Execution. Hindi. व्याकरण. शरीरके अंगो के नाम लिखिए. 1) आँख 2) नाक 3

6 Find the proper subset of x x2-9x-10 = 0 Solution

ϕ 10 -1

Working x2-9x-10 = 0 rArr x2-(10-1)x-10 = 0

rArr x2-10x+x-10 = 0 rArrx(x-10)+1(x-10) = 0

rArr (x+1) (x-10) = 0

11 Let M = letters of the word REAL and N = letters of the word LARE Write sets M and N in roster form and then state whether (i) M sube N is true (ii) N sube M is true (iii) M = N is true

Solution M = real and N = lareSo (i) Yes (ii) Yes (iii) Yes

English 2 Twelfth Night ndash Shakespeare

A noble man named Orsino in the kingdom of Illyria is deeply in love with a lady called lady Olivia She is in mourning for her dead brother so she will not even think about marriage At this time a sea storm causes a terrible shipwreck and a young lady called Viola is swept onto the shore She thinks that her twin brother Sebastian is drowned A sea captain tells her about Orsino and his love for Olivia Viola wishes to work in Oliviarsquos home but feels she will not be employed So she dresses as a man calls herself Cesario and gets work at the house of OrsinoViola (now Ceasario) is much liked by Orsino and becomes his page She falls in love with Orsino Orsino sends Ceasario to deliver messages to Olivia Olivia herself falls for the beautiful young Ceasario believing Viola to be a man

2 Answer the following questionsa Why does Orsino ask the musicians to play onOrsino asks the musicians to play on because music feeds his desire He calls upon the musicians to play music so that his hunger for love could be replenished with an excess of musicb What does Valentine tell about OliviaWe learn from Valentine that Olivia is in mourning for her brother she wears a veil and has vowed that no one will see her face for another seven yearsand she refuses to marry anyone until thenc From the exchange between Orsino and Valentine what do you think their relationship isValentine is one of orsinod attendants He was sent to Olivia as a messenger of love but was not allowed to speak to here Who is Olivia mourning for and whyOlivia is mourning for her dead brother

Homework Q fHistory and Civics

Growth of Nationalism

Important dates to remember1769-Napoleon born on 15thAugust1789-Fall of Bastille on 14th July and the beginning of the French revolution declaration of the rights of Man on 26thAugust1793-King Louis XVI executed on January 211764-The Sugar Act passed1765-The Stamp act passed1774-The first congress of Philadelphia1776-The declaration of American Independence of on 4th July1777-Defeat of the British at Saratoga1781-Surrender of lord Cornwallis at Yorktown1783-The treaty of Versailles1804-Napoleon becomes the emperor1813-Battle of Leipzig or Battle of nations in which Napoleon was defeated by the Allies1815-Battle of Waterloo June 18 in which Napoleon was defeated and captured1821-Death of Napoleon in StHelena1860-Abraham Lincoln elected President of the USA1861-The civil war began 1864-Abraham Lincoln elected President of the USA for the second time1865-Slavery abolished in the US

Name the following- The queen of Louis XVI

Marie Antoinette The three philosophers of France

VoltaireMontesquieuJean Jacques Rousseau

The British general whose surrender brought the war in America to an endLord Cornwallis

The first president of the USAGeorge Washington (1732-1799)

The first southern state to secede from the unionSouth Carolina

The author of the book lsquoUncle Toms CabinHarriet Beecher Stowe

Homework-Learn

Class IXSubject Topic Summary Execution

Economics

Types of economies Today I am going to share you the concept of economic growth and economic development Few questions will be given from the previous study material dated 942020

Meaning of economic growthAnswer) The term economic growth generally means anincrease in national income or per capita output or income over time It indicates towards quantitative growth of a country

Meaning of economic developmentAnswer) Economic development is defined

as a process whereby the real per capita income of a country increases over time along with fall in poverty ratio unemployment and income inequality etc

Distinguish between economic growth and economic development

Basis Economic growth

Economic development

Scope It has narrow scope as it refer only to rise in per capita income

It has wide concept since it includes qualitative changes as well

Concerned matter

It is concerned with the rise in income

It is concerned with not only rise in income but also reduction of poverty income inequality and unemployment

Focus Economic growth does not focus on economic development

Economic development focus on economic growth plus qualitative changes

Distinguish between capitalist economy and socialist economy

Ownership

Motive

Tool

Means of production are owned and managed by private people

Self interest and profit earning is the main motive

Price mechanism is a main tool to solve the economic problems

Means of production are owned and managed by the government

Social welfare is the main motive

Economic planning by the government is the main tool to solve the economic problem

Competition

Distribution of income

There exist large competition among buyers and sellers

There is existence of large inequalities of income

There is no such competition

There exist less inequalities of income

Math Topic ndash AlgebraChapter -Factorisation

Study item Factorising by taking out common factorSome solved sums from exercise 41

1) (i) 8xy3 + 12x2y2

= HCF of 8xy3 and 12x2y2 is 4xy2

= 4xy2(2y + 3x )

4) (ii) 28p2q2r ndash 42pq2r2

= HCF of 28p2q2r and 42pq2r2 is 14pq2r = 14pq2r (2p - 3r )5) (ii) 14mn + 22m - 62p=HCF of 14mn 22m and 62p is 2= 2(7mn + 11m - 31p)7) (ii) 3a(x2 + y2) + 6b (x2 + y2) = HCF of 3a(x2 + y2) and 6b(x2 + y2 ) is (x2 + y2)= ( x2+ y2 )(3a + 6b )9) (ii) x(x2 + y2 ndash z2 ) + y(-x2ndashy2 + z2 ) ndash z(x2+ y2 ndash z2 )= x(x2 + y2 -z2) ndash y-(x2 + y2 -z2) -z(x2 + y2 ndash z2)=x(x2 + y2-z2) -y( x2 + y2-z2) ndash z (x2 + y2 -z2)= (x2+ y2 ndash z2)(x ndash y ndash z )

Commercial Studies

Introduction to Accounting and Book-keeping

Today I am going to share you the meaning of Accounting and Book-keeping and its related terms bullAccounting bullBook Keeping bullAccountsbullTypes Of Accounts bullAccounting Cycle

bull Meaning of accounting

Ans )Accounting is the art and science of recording classifying and summarising monetary transactions

bull Meaning of Book-keeping

Ans) Bookkeeping is the art of recording business transactions with the view of having a permanent record of them and showing their effect on wealth

bull Meaning of account

Ans) The term account means a record of business transactions concern a particular person of firm asset or income or expense It is a summarised record of all transactions which take place in an accounting year

bull Types of accountsPersonal accounts ndash Personal accounts relating

to person and Organisation are known

as personal accounts Example Ramrsquos Account ABC amp Co Account etc

Real account - The accounts related to tangible and intangible assets are called real accountsExample Cash Account Furniture Account etc

Nominal account- Accounts related to expenses losses incomes and gains are known as nominal accountsExample Wages Account Salary Account Discount Account etc

bull Accounting cycle Accounting cycle refers to a complete sequence of accounting activities It begins with recording of transactions and ends with the preparation of a balance sheet

English 1 Transformation of sentences

Sentences A sentence is a group of words which makes complete sense

a Assertive sentencesb Imperative sentencesc Interrogative

sentencesd Exclamatory sentences

Sentences can be changed from one grammatical form to another without changing the meaning of the sentence This is known as transformation of sentences

Exercise 6Rewrite the following sentences according to the instructions given below without changing their meanings

1 As soon as he saw the beer he jumped into the river ( Begin No sooner)

2 None but brave deserve the fair (Begin the bravehellip)

3 This box is too heavy for me to lift ( Use so hellip That instead of too)

4 No one other than a king can live like James Luxurious ( Begin only James)

5 Oh for the wings of a dove (Begin I wishhellip)

BENGALI(2ND LANGUAGE)

ldquo বঙগভমির পরমি ldquo াইকেল ধসদন দতত

পব13পোসঠ আসোলিচত ৩ পরবোস দৈদসবর বস ীবতোরো Pলিদ স এ লেদ -আকো সত-োলি লেদ তোস - ক) বকতো লেক কোর লেো লেকো কলিবতোর অং ) কোর পরলিত বকতোর এই উলিকত গ) এ লেদ আকো সত বসত কী বলিঝসয়স4 ীবতোরো বসত কী লেবোঝ ঘ ) আসোচয অংসর তোৎপP13 কী

উ -ক ) বকতো স কলিব মোইসক ম3দ দতত

Types of AccountPersonal AccountReal AccountNominal AccountBalance Sheet (opening)

কলিব মোইসক ম3দ দসততর রলিচত বঙগভলিমর পরলিত কলিবতোর অং ) কলিব বঙগী অ13োৎ লেদমোতোর পরলিত কলিবর এই উলিকত গ ) এ লেদ আকো বসত কলিবর মোব লেদী রপ আকো লেক লেবোঝোসো সয়স4 আকো লেসক লেPম তোরো স পসর লেতমলি ীব লেদ রপ আকো লেসক পরো রপ তোরো স পরসত পোসর এই মভোবোর কোই কলিব বসস4 ঘ ) পরবো Pোতরোয় Pলিদ কলিবর লেদ আকো লেসক ীব তোরো রপ পরো স পসর তোসত কলিব লিবনদমোতর দঃলিত কোর মতয লিবসর সবোভোলিবক পলিরলিত এবং মোষ মরী তোই পরবোস Pলিদ তা োর মতয য় তবও কলিব লিবচলিত সব ো কোর পলিবীসত লেকউ অমর য় লিক4ই অকষয় য় দীর লেPম লিচরপরবোমো লেতমলি মোসষর ীবও চমোতোই ীব - সতবধতোই মতয ীব দীসত মোষ লিতয পরবোমো তবও লেPব মোষ আপ কতকসম13র মো3যসম মোসষর মস লিসসদর সথো কসর লিসত পোসর তোরো লিচরভোসবর সয় মোসষর মস লিবরো কসর তোসদর মস3য লেকউ পGভসত লিবী সয় গোসও মোসষর মস তোরো লিতযপলিত লিতযবলিনদত

Hindi 2ndlang

काकीी(लिसयारामशरणगपत)

इस कहानी म क न यह बतान का परयास निकया ह निक बचच अपनी मा स निकतना परम करत ह शयाम अबोध बाक ह वह अपनी मा क मरन क बाद उसन अपनी मा क लिए बहत रोया बाद म उस पता चा निक उसकी मा राम क घर ची गई ह आकाश म उडती हई पतग दकर उस हष हआ निक पतग क दवारा वह अपनी मा को नीच उतारगा इसक लिए वह अपनी निपता की जब स दो बार सवा रपया निनकाकर पतग और दो मोटी सी मन वाी अपन भाई स काकी एक कागज पर लिवा कर पतग म लिशव का दिदयानिनकाकर पतग और दो मोटी सी मन वाी अपन भाई स काकी एक कागज पर लिवा कर पतग म लिचपका दिदयाभोा और शयाम कोठरी म रससी बाधनी रह थ तभी उसक निपता करोध म आकर उन स पछ निक कया उनकी जब स रपया निनकाा हभोा डर क मार बताया निक शयाम इस पतग क दवारा अपनी काकी को राम क यहा स उतारना चाहता हनिवशशवर(शयाम क निपता)न फटी पतग उठाकर दी तो उस पर काकी लिा थावह हत बजि होकर वही ड रह गएउनहोन सोचा निक मन अपन पतर को मारा जोनिक अनजान और निनदष थावह अपनी मा कोनिकतना पयार करता ह

helliphellipContinue to next

Computer Application

Java Programming Prog 1Write a java program to input two numbers from user and display the sum or product of them as per user choice Use switch case statementSolve public class sum_product public static void main(String args[]) Scanner sc=new Scanner(Systemin) int abc Systemoutprintln(ldquoEnter two numbersrdquo) a=scnextInt() b=scnextInt() Systemoutprintln(ldquoPress 1 for sum or 2 for productrdquo)

c=scnextInt() switch(c) case 1 Systemoutprintln(ldquoThe sum will be =rdquo+(a+b)) break case 2 Systemoutprintln(ldquoThe product will be =rdquo+(ab)) break default Systemoutprintln(ldquoWrong Inputrdquo) Home Work - Practice in your computer using bluej

Subject Eng Literature (The Merchant of Venice ndash William Shakespeare)Topic Act I Scene 2 Lines 92 to 126 (End of scene) Date 13th April 2020 (5th Period)

[Students should read the original play and also the paraphrase given in the school prescribed textbook]Summary Questions amp Answers

o After Portia has expressed her opinion about the suitors Nerissa informs that she need not bother about any one of them as they have decided to quit Belmont at the earliest opportunity because they do not believe in trying their luck by the caskets which is the only way of winning Portia

o Nerissa then enquires of Portiarsquos opinion about Bassanio who once visited her in the company of the Marquis of Montferrat and says that she had never come across such an ideal love deserving the fairest lady for his bride

o Portia seems to remember Bassanio quite correctly and says that she agrees with Nerissa At this moment a servant informs Portia that the Prince of Morocco has arrived to try his luck by the caskets

o Portia tells Nerissa that if she could welcome this new suitor as gladly as she says farewell to the previous ones she would be glad of his arrival However if he happens to have the virtues of a saint but the black complexion of a devil she would prefer to have him for religious consolation rather than as a husband

(1) NERISSA You need not fear lady (Line 97-103)

the having any of these lords they have acquainted me with their determinations

which is indeed to return to their home and to

trouble you with no more suit unless you may be wonby some other sort than your fathers imposition depending on the caskets

PORTIA If I live to be as old as Sibylla I will die as chaste asDiana unless I be obtained by the manner of my fatherswill I am glad this parcel of wooers are so reasonablefor there is not one among them but I dote on his veryabsence and I pray God grant them a fair departure

(a) Elucidate the idea expressed in the first speech of the above dialogue

In the first speech Nerissa assures Portia that she need not have any fear of being compelled to marry anyone of the suitors who had lately come to Belmont She informs her that they have all decided to return to their respective countries(b) Illuminate the meaning of the phrase ldquoyour fatherrsquos imposition depending on the casketsrdquo

Nerissa means that the suitors of Portia do not find the conditions imposed by the will of her father to their liking They are too hard for them These conditions are that in the event of a suitor failing to choose the right casket (i) he should never disclose to anybody which casket he chose (ii) he can never marry and (iii) he should take his departure immediately(c) Explain the meaning of the term lsquoSibyllarsquo

lsquoSibyllarsquo is the name given by Romans and Greeks to a prophetess inspired by some deity usually the sun-god Apollo She had a very long life The god Apollo granted her as many years of life as she could hold grains of sand in her hand(d) Elucidate the meaning of the term lsquoDianarsquo

lsquoDianarsquo is the goddess of hunting She is also regarded as a symbol of virginity because she never fell in love and never

married(e) Explain the meaning of the first two lines of Portiarsquos speech

Portia says that even if she is to live for centuries like Sibylla she would not marry except in accordance to her fatherrsquos will She asserts that she would not mind remaining unmarried and untouched by a man like Diana the virgin the goddess of hunting unless a man is able to win her by passing the test laid down by her father

Class XSubject Topic Summary Execution

Hindi 2nd

Langबड घर की बटी( मशी परमचद)

lsquoबड घर की बीटीrsquo कहानी का उददशय मधयम वग की घर समसया को सझा कर सगदिठत परिरवार म मिम जकर परम स रहन का सदश दना ह घर म शानित tानिपत करन की जिजममदारी नारी की होती ह यदिद नारी समझदार ह उसम धय और परिरवार क परनित परम ह तो कोई भी घटना परिरवार को निवघदिटत नही कर सकती या कहानी परिरवार को सगदिठत करत हए परम सौहाद स एक रदसर की भावनाओ को समझ करउनका सहयोग करत हए जीवन यापन करन की पररणा दती ह मशीपरमचदर जी न इस कहानी म सयकत परिरवार का परनितनिनमिधतव निकया ह यह कहानी बनी माधव सिसह जो गौरी पर क जमीदार क उनक दो पतरो की हशरी कठ ा निबहारीशरीकात का निववाह एकजमीदार घरान की पतरी आनदी स हआ थाआनदी न द को ससरा क वातावरण म ढालिया थाएक दिदन आनदी का अपन दवर ा निबहारी स झगडा हो जाता ह दोनो भाई एक रदसर स अग होन की कोलिशश करत हसभी बह आनदी न अपन मधर वयवहार स ा निबहारी को घर छोडकर जान स रोक लिया| इस पर बनी माधव सिसह न कहा निक बड घर की बटी ऐसी ही होती ह जो निबगडा काम बना ती ह अतः शीषक साथक ह बड घर की बटी आनदी ह

helliphelliphelliphellipContinue to nextBiology Topic ndash Chp-1

CellWelcome to new session 2020-21Today we will start with Chpter 1 cell CELL

Protoplasm+Cellmembrane Or Cell wall

Cytoplasm+Neucleus

Cytoplasmic+ CytoplasmicOrganelles Inclutions(mitochondria (food Golgi bodies pigments)Ribosome)

What is cellbull Cell is the structural and functional unit of living organismbull According to number of cells organisms areUnicellular - Amoeba bacteria Multicellular - Rose Mango Tiger HumanSmallest cell -bacteria Longest cell - Nerve cellLargest cell - Ostrich egg cellCells are of different size and shapes according to their functionsQ2Write chief functions of following cellorganelles

Q3What is tonoplastVacuoles covered by a covering called tonoplast

Bengali(2Nd

Language)

ফ ফটক ো ফটক (কলিবতো ) ভোষ মসোপো3 gtPোয়

একটি লেমসয়র ীবস লেপরম লিকভোসব ফসট ওসঠ তো লেদলিসয়স4 কলিব লেপরম Pই য় লেই ময়ই বনত কোস পলিরত য় ফ লেফোটো বো োসফোটো লেটো ব2 কো য় লেমসয়সদর ব gtয13 লেপরসমর 4লিব ফসট উসঠস4 এই কলিবতোয় লেপরম মোষসক মত gtযর মস লেফস লিদসয় পরকষস বাোচোসোর gtয োত বো2োয় কলিবতোয় লেমসয়টির পসব13র দঃসর কো বো সও লেমসয়টি লেই পসর পলিক সত চোয়ো োরী ীবসর কোস4 পরম লেPৌবস লেপরমসক পোবোর পরব ইচছো োকসও তো পসর লেলিতবোচকতোয় পলিরত য় কলিব ভোষ মসোপো3 যোয় লেP ক লেপরসমর

কলিবতোয় ব gtযবহত লিবসষ লিক4 সvর অ13 লেদওয়ো ১) রসবোো= লেP লিবলিভনন রকম ডোকসত পোসর২) ো= পোর ৩) ঠলি = লেচোসর বZ৪)আই বস2ো=অলিববোলিত৫)শইসয় = োলিয়ত কসর৬)োতপাোচ= লিবলিভনন পরকোর৭)দ2োম = v কসর বZ কসর লেদওয়ো৮)লেরলিং =লেোোর দৈতরী লেব2ো৯) বনত= একঋত১০) পাোর = বসকরো2

Organelles Functions

1 Endoplasmic reticulum

2 Mitochondria

3Golgibodies

4 Ribosome5Lysosome

6Plastids

7 Centrosome

i) Supportive framework for the cellii) Synthesis and transpost of proteinsRelease of energy in the form of ATPi) Synthesis and secretion of enzymes hormoneii) Formation of vacuoles lysosomei) Protein Synthesisi) Intracellular digestionii) Destroy foreign substancei )Leucoplast - stores starchii)chloroplast - trap solar energyiii) Chromoplast - imparts colour toflowers amp fruitsi) Initiates and requlates cell division

কলিবতো তোর অ13সক ভোষোয় পরকোো কসর ঘলিরসয় ব যকত কসরস4 লেপরসমর ফতো আর লিবফতো লেক গোঢ় কসর লেদোসো কলিব ভোষ মসোপো3 যোসয়র অলিভবসর অ যলিদক

Economics

Factors of Production

Welcome to the new sessionToday we are going to start the first chapter of Class XThe name of the chapter is Factors of productionBy the name I hope you all can recall a glimpse of what you have learnt in the second chapter of Class IX

NowProduction is the process of creating the various goods and services which are consumed by the people of the country to satisfy their wants

Thus it is the process in which some materials are transformed from one form to another to create utility and value in goods

For example utility can be created by changing the form of a commodity ie

Making of table out of wood by a carpenter for his customer here the wood is getting transformed into table creating utility for his customer and he can also command a price for it

On the other hand Housewives perform very

useful activities at home which create utility but their domestic activities are not included in production because they have no money value

So we can also say that Production denotes two things firstly creation of utility and secondly creation of value

Production is not complete unless it reaches the consumer

An increase in production will increase the economic welfare of the consumers and hence the aim is to raise the production level of the country

Again production of a good or service is only possible if certain resources or

Questions

1 What do you mean by production

Answer Production means the creation of goods and services for the purpose of selling in the market

In fact production involves the transformation of inputs into outputs

Hence production denotes two thingsCreation of utility and creation of valueUtility and value can be created by changing the form by changing the place by changing the time and by rendering services

Example Transformation of raw

materials into finish goods such as potter creates utility by converting mud into utensils assembling of small parts to make bigger machinery

Production also includes services such as distribution and marketing

2 What are the factors of production

Answer Factors of Production refers to the resources and inputs needed for producing goods and servicesThese inputs can be classified as

Land Labour

Capital Enterprise

Land Land is defined to include not only the surface of the earth but also all other free gifts of nature(for example mineral resources forest resources and indeed anything that helps us to carry out the production of goods and services but is provided by

inputs are used together in right proportion

A resource or an input which helps in the process of production to obtain an output is called FACTOR OF PRODUCTION

These factors of production can broadly be categorized into four parts 1LAND 2LABOUR3CAPITAL4ENTERPRISE (ORGANISATION)or Entrepreneur

The above factors are all interdependent on each other and they play a major role in production process

FACTORS OF PRODUCTION

LANDCAPITAL

LABOUR ENTREPRENEUR

nature free of cost)LabourLabour refers to the human efforts that need to be combined with other factors of production for creating an output

CapitalAll man ndash made means of production is called capita example machineries which help in further production Money when used for starting any business for purchasing raw materials machinery tools etc it is regarded as capitalCapital also includes physical capital like factories machineriestoolsbuildingsequipments etcEnterpriseThe task of bearing risks is called enterprise and the person who bears these risks of business is called the entrepreneurThus an entrepreneur is one who organises production takes important decisions regarding production hires and purchases factors of production and bears the risk and uncertainty involved in productionOrganisation refers to the services of an entrepreneur who controls organises and undertakes all risks One who plans organises and manages a business enterprise is an organiser

Physics Chapter 1 Force

Force is an external agent capable of changing the state of rest or motion of a particular body It has a magnitude and a direction The direction towards which the force is applied is known as the direction of the force and the application of force is the point where force is applied The Force can be measured using a spring balance The SI unit of force is Newton (N)

Question 1

State the condition when on applying a force the body has

(a) the translational motion

(b) The rotational motion

Solutions

(a) Translational motion is produced when the body is free to move

(b) Rotational motion is produced when the body is pivoted at a point

Question 2

Define moment of force and state its SI unit

Solutions

The moment of force is equal to the product of the magnitude of the force and the perpendicular distance of the line of action of force from the axis

of rotation

The SI unit of moment of force is Newton times meter

= Newton meter (Nm)

Commercial Studies

Stake holders In this topic you will be come to know about the meaning and concept of stakeholders

How stakeholders are different from shareholders

Questions1 What do you mean by the term stake holdersAnswer) The term stake holders have developed from the words which mean an interest or expected benefit Stakeholders mean all those individuals groups and Institutions which have a state (interest) in the functioning and performance of a commercial organisation or a business enterprise2 What do you mean by share holdersAnswer) The person and Groups who own the shares of the joint stock company by providing capital to the company are called shareholders Shareholders are the internal stakeholders shareholders are one out of several stake holders3 How are shareholders different from stakeholdersAnswer)i) The term shareholders is related to only joint stock company whereas stakeholders are related with all business organisationsii) Stakeholders maybe any individual having financial stake in business organisation whereas a shareholders are those individuals who are holding shares in the company4) How are shareholders different from creditorsAnswer) i) Shareholders are internal stakeholders while creditors are external stakeholdersii) Shareholders invest in the capital of the company whereas creditors give loan to the companyiii) Shareholders are the members of the company with voting rights but creditors are not the members of the company

English 1 Transformation of sentences

Sentences A sentence is a group of words which makes complete sense

e Assertive sentencesf Imperative sentencesg Interrogative sentencesh Exclamatory sentences

Sentences can be changed from one grammatical form to another without changing the meaning of the sentence This is known as transformation of sentences

Exercise 1 Change the following affirmative sentences into Negative sentences

a He is a good manHe is not a bad man

b Ram loves SitaRam is not without love for Sita

c Only he stood first in the classNone but he stood first in the class

d Ankit was wiser than he

He was not so wise as Ankite He did it

He did not fail to do itf As soon as I reached college the

bell rangNo sooner did I reach college than the bell rang

g He finished everythingHe left nothing unfinished

h It always pours when it rainsIt never rains but it pours

Math Topic Commercial MathematicsChapter ndash Goods and services Tax

What is GSTAns It is a abbreviated term of Goods and Service Text which is an indirect tax levied on the sale of goods and rendering servicesSome terms related to GSTDelar Any person who buys goods or services For resale is known as a delar A delar Can be a firm or a companyIntra-state sales Sales of goods and services within the same state or same union territory are called intra- state salesInter-state sales Sales of goods and services outside the state or union territory are called Inter-state sales4) Input GST GST is paid by dealers on purchase of goods and services are called input GST5) Output GST GST is collected from customers on sale of goods and services are called output GST6) Types of GST There are three taxes applicable under GST(i) Central Goods and Services Tax (CGST)(ii) State Goods and Services Tax (SGST) or Union Territory Goods and Services Tax (UTGST) Both these taxes are levied on intra-state sales Here GST is divided equally among central and state governments(iii) Integrated Goods and Services Tax (IGST) IGST is levied on inter- state sales It is also levied on import of goods and services into India and export of goods and services from India

Subject Eng Literature (The Merchant of Venice ndash William Shakespeare)Topic Act III Scene 4 Lines 1 to 44 (Portia hellip To wish it back on you fare you well Jessica)[Students should read the original play and also the paraphrase given in the school prescribed textbook]

Summary Questions amp AnswersIn this scene we suddenly find a new element in the character of Portia We have already seen her possessed of every graceful womanly quality but now she shows that she is capable of rapid decision and determined action She shows this by her sudden resolve to hasten to Venice with a daring scheme for the rescue of Antonio This is an important scene in the dramatic action for it leads up to and renders possible the striking events of the famous trial scene which is one of the greatest striking elements of the play Moreover the fact that all the characters of importance are now assembled together in Venice makes the union of the main plot and the secondary story complete

(1) LORENZO Madam although I speak it in your presence(Line 1-9)

You have a noble and a true conceit

Of god-like amity which appears most strongly

In bearing thus the absence of your lordBut if you knew to whom you show this honourHow true a gentleman you send reliefHow dear a lover of my lord your husbandI know you would be prouder of the workThan customary bounty can enforce you

(a) Where is Lorenzo Why is he here To whom is he referring as lsquoMadamrsquo

Lorenzo is at Portiarsquos residence He had met Salerio on the way and Salerio had begged him to come along with him to

o In this scene Portia Nerissa Lorenzo Jessica and Balthazar appear

o Portia requests Lorenzo and Jessica to be in charge of her house during her absence from Belmont because she and Nerissa have decided to spend the days in meditation and also in visiting the holy places in the neighbourhood of Belmont She has already instructed her people to acknowledge both Lorenzo and Jessica as master and mistress of house during her absence Lorenzo and Jessica gladly agree to look after the house of Portia

handover the letter from Antonio to Bassanio The letter carried the bad news about Antoniorsquos arrest for non-payment of loan taken from Shylock Hence Salerio might have preferred company to break this bad news to Bassanio He is referring to Portia as Madam(b) What does Portia say on hearing the above extract

Portia says that she has never regretted doing good to others Friends who spend a lot of time together and really are there for each other have many traits in common As Antonio is Bassaniorsquos best friend saving him is like saving Bassanio who is like her own soul She asks Lorenzo to take care of management of the house till Bassanio is back(c) What does Portia send with Bassanio and why

On hearing about Antoniorsquos troubles on account of Bassanio her husband Portia immediately sends him with enough gold to repay the debt many times over to Venice to help Antonio out of his misfortune

(2) Lorenzo Madam with all my heart (Line 36-40)

I shall obey you in all fair commands

Portia My people do already know my mindAnd will acknowledge you and JessicaIn place of Lord Bassanio and myselfSo fare you well till we shall meet again

(a) Where are Lorenzo and Portia at this time What lsquofair commandsrsquo are given to Lorenzo

Lorenzo and Portia are at Belmont during this scenePortia reveals to Lorenzo that she has sworn to contemplate in prayer at a monastery around two miles away until her husband returns from Venice She tells him that Nerissa would accompany her and asks him to manage the house with Jessica till things are settled In response Lorenzo tells her that he would be obliged to do whatever she asks him to do(b) Where is Portia actually going and why

Portia tells Lorenzo that she would live a life of contemplation and pray at a monastery which is two miles away from her place In reality Portia plans to go to Venice in disguise with Nerissa and argue the case in defense of Antonio She is very sure that her plan would succeed

ClassXI (ScienceHumanitiesCommerce)Subject Topic Summary Execution

Computer Science

(APC)

Ch ndash 1 Numbers

(Numbers in different bases and

their Arithmatical operations)

Number System In computers Number System is defined as a writing system to represent the numbers in different ways ie we are using different symbols and notations to represent numbers There are four ways we can represent the number ndash Binary Decimal Octal and Hexadecimal

Decimal Number SystemThis number system consist 10 digits These are 0 1 2 3 4 5 6 7 8 amp 9

Binary Number SystemThis number system has only two digits these are 0 and 1 Here 0 stands for off while 1 stands for on

Octal Number SystemThis number system has 8 digits these are 0 1 2 3 4 5 6 amp 7

Hexadecimal Number SystemThis number system has 16 digits these are 0 1 2 3 4 5 6 7 8 9 A B C D E F Here the value of the alphabets are as follows A=10 B=11 C=12 D=13 E=14 F=15

Rules for conversion decimal number to Binary1 Divide the decimal number by 22 If the number will not divide equally by 2 then round down the answer to the nearest whole number (integer)3 Keep a note of the remainder it should be either 0 or 14 Keep repeating the above steps dividing each answer by 2 until you reach zero5 Write out all the remainders from bottom to top This is your binary solution

For example Lets convert 32 to binary 2 32 2 16 - 0 2 8 - 0 2 4 - 0 2 2 - 0 2 1 - 0 0 - 1

The binary equivalent of 3210 is 1000002

Try the follwing youself1 2410

2 4810

3 1210

History GROWTH OF NATIONALISM

The second half of the 19th century witnessed growth of political consciousness and a sense of Nationalism among the IndiansThere were various factors for growth of Indian Nationalism- As a result various political associations were formed in different provinces by the educated Indians Surendranath Banerjee organized a meeting of National conference at Calcutta Ultimately the National Congress was founded in Bombay in 1885This body became the vanguard of Indian struggle for freedom The congress leaders were known as moderates because they followed a policy of prayer and petition A large number of Indian leaders had experienced in political agitation The Political situation of England was also changed Moreover increasing revolutionary activities in Maharashtra Punjab and Bengal became serious concern to the British Government In this

QUESTION1 What do you mean by Nationalism ANSWER 1 Nationalism is defined as loyalty and devotion to own nation especially a sense of national consciousnessQUESTION 2 What are the causes of nationalism ANSWER 2 There were various factors for growth of nationalism

1 Spread of western education2 The progress of vernacular press and

patriotic literature3 The economic exploitation of our

country by the colonial rulers4 International affairs

QUESTION 3 Who organized National conference in Calcutta in 1883 ANSWER 3 Surendranath BanerjeeQUESTION 4 When did Indian National Congress formANSWER 4 Indian National Congress was formed in 1885 in BombayQUESTION 5 Who were ModeratesANSWER 5 The Early Nationalists were also known as Moderates Their emergence marked

background Lord Curzon became Viceroy in India He had no respect for the Indian National Congress

the beginning of the organized national movement in India They believed in British justice and were loyal to them They followed a policy of prayer and petition They demanded constitutional reforms of our country Impotant Moderate leaders were Pherozshah Mehta Dadabhai Naorozi and Surendranath Banerjee etcQUESTION 6 What do you know about Extremism in Indian National movementANSWER 6 In the beginning of 20th century a new class of national leaders emerged in India which was different from the moderate groups They started more aggressive movement against the British empire The goal of extremists was ldquoswarajrdquo Important extremist leaders were Bal Gangadhar Tilak Lala Lajpat Rai Bipin Chandra Pal etcQUESTION 7 Mention the places which were the main centres of Revolutionary movementANSWER 7 Maharashtra Bengal and Punjab

Physics

Chapter Dimensional Analysis

(Summary)

The dimensions of a physical quantity are the powers to which the fundamental units are raised in order to obtain the derived unit of that quantit

The physical quantites lengthmasstime are represented by [L] [M] [T] resp let they are raised to powers ( dimesions) abc resp then any physical quantity can be represented by [ La Mb Tc ] Examples

1 Area area = L x B = [L] x [L] = [M0 L2 T0 ]

2 Density density = massvolume = [M][L3] = [ M L-3]

3 Velocity velocity = distancetime = [L][T] = [LT-1]HW Try to find out dimension of acceleration Acceleration = velocity timeNB One can find the SI Units Using Dimension Analysis Such as for area we have [L2] so its SI unit is m2

Biology Topic ndash Chp-1 The living world

Today we will start the first chapter the living world Here we discuss about the characteristics of living organism and what are the difference between them and nonliving substances We also discuss about the contribution of different Scientists

There are over 500000 species of plants andover a million species of animal are present on earth Some 15000 new species were discovered every yearQ1 What is a living organismbull A living organism is primarily physico -chemical material that demonstrate a high degree of complexity is capable of selfRegulation possesses a metabolism and perpetuates itself through timeQ2 What are the differences between livingand non-livingsi) Compared with non-living living organisms

have more complex organised structure and their use of energy is more controlled amp efficientii) Living things reproduce their own kind by forming new cells which contains copies of their genesiii) Each organism has some degree of homeostasisie it is able to make adjustments so that internal environment remains constantQ3 Write contributions of following Scientists i) Aristotle - One of the first theories in Biology places all living things in a hiearchieii) AV Leeuwenhoek - was the first to observe living single celled organisms under microscopeii) Carolus Linnaeus - developed the binary system for naming of organisms and classificationiii) Geregor Johann Mendel ndash discoverbasic principles of inheritanceHomework i) C Darwin ii)Schleiden

Math Trigonometric functions

1 Overviewi) Trigonometry The word lsquotrigonometryrsquo is derived from the Greek words lsquotrigonrsquo and lsquometronrsquo which means measuring the sides of a triangle An angle is the amount of rotation of a revolving line with respect to a fixed line Usually we follow two types of conventions for measuring angles ie a) Sexagesimal system b) Circular system In Sexagesimal system the unit of measurement is Degree In Circular system the unit of measurement is Radian ii) Relation between degree and radianThe ratio of circumference of a circle to its diameter is always a constant This constant ratio is a number denoted by π which is taken approximately as 227The relationship between degree amp radian measurements is as follows2 right angles = 180deg= π radians1radian = 180degπ=57deg16(approx) 1deg=π180 radianiii) Length of an arc of a circleIf an arc of length s subtends an angle θ radians at the center of a circle of radius r then s=rθiv) Area of a sector of a circleA sector is like a pizza slice of the

Q) Express the following angles in radiana) 45deg b) 40deg3730Ans a) We have 180deg=π radiansi e 45deg= πtimes45180 radian = π4 radiansb) 40deg3730= 40deg37+3060 minute= 40deg 37 +12 minute= 40deg+ 752 minute=40 + 75(2times60) degree=3258 degreeNow 180deg=π radianie 3258 degree= (πtimes325) (180times8) radians = 65π288 radiansQ) A circle has a radius of r=12 meters What is the length of an arc traced out by a 60deg angle in the center of the circleAns In this problem we know both the central angle (60deg) and the radius of the circle (12) All we have to do is plug those values into our equation and we get

s = 2π(12)(60360)s = 24π6s = 4πSo the length of an arc traced out by a 60deg angle in a circle with a radius of 12 meters equals 4π meters asymp 1257 metersQ) Find the area of the sector with a central angle 30deg and a radius of 9cmAns GivenRadius r = 9 cmAngle θ = 30degArea of the sector = θ360degtimesπr2

= 30360degtimes227times92=2121cm2

circle It consists of a region bounded by two radii and an arc lying between the radiiThe area of a sector is a fraction of the area of the circle This area is proportional to the central angle In other words the bigger the central angle the larger is the area of the sectorArea of Sector = θ2 times r2 (when θ is in radians)

Area of Sector = θ times π360 times r2 (when θ is in degrees)

COMMERCE

CLASSIFICTION OF HUMAN ACTIVITIES-ECONOMIC AND NON-ECONOMIC

Welcome to the new sessiontoday we are going to start the first chapter of Class XI The name of the chapter that we are going to start is

lsquoClassification of Human Activities ndasheconomic and non-economicrsquo

Now let us start the chapter by considering human beings and the activities they perform throughout the day

Human activities means all those activities that human beings undertake to satisfy their wants

Human wants on the other hand are the desire of human beings for goods (vegetables fruits rice etc) and services (services of doctors teachers lawyers etc) that they require to live

Now these human activities continue throughout life as human wants are unending unlimited and recurring as human beings desire for better living throughout their lives

Now human activities can be classified into two categories

Human activities

Economic activities Non-economic activities

Economic activities are

Questions1 What are human activities

Answer Human activities mean all those activities that human beings undertake to satisfy their wants

Example A man working in an office

A boy playing in the garden

2What are the characteristics of human activitiesAnswer the characteristics of human activities are as follows

Human activities are undertaken by men women and children and these activities involve human efforts

Human activities are undertaken to satisfy human wants which are unlimited

Human activities continue throughout life

Human activities are performed for both earning money and personal satisfaction

3What is economic activitiesGive example

Answer Economic activities are undertaken by human beings with the object of earning money acquiring wealth and thereby satisfying human wantsExample

Selling of goods by a shop keeper to his customer

A clinic run by a doctor Service of a teacher in school or college

undertaken by human beings with the object of earning money and acquiring wealth

These activities result in the production of economic goods and services

Example Human activities(ie working in factories officesshops) which produce direct economic benefits

Non-economic activities are inspired by human sentiments and emotions such as love for the family desire to help the poor and love for the country

Thus these human activities (eg praying playing sleeping) produce no direct economic benefits and they are also not related to earning money and acquiring wealth

4 What are the characteristics of economic activities

Answer The characteristics of economic activities are as follows

Economic motiveEconomic activities are undertaken to earn money and acquire wealth

ProductiveEconomic activities involve productiondistribution and exchange of goods and services to create wealth

Economic growthEconomic activities determine the level of economic development of a country and standard of living of its citizens

Socially desirableEconomic activities are socially desirable for society

Economic resourcesEconomic activities make use of all the economic resources such landlabourcapital etc

5 What do you mean by non-economic activitiesExampleAnswerNon-economic activities are inspired by human sentiments and emotions such as love for the family desire to help the poor and love for the countryThese activities are not undertaken for monetary gain but for onersquos satisfaction and happinessExample

a mother looks after her children

a student donates blood8 Differentiate between Economic activities and Non-economic activities

Economic activities

Non-economic activities

1to earn living and acquiring wealth2Result can be measured in terms of money

3ExampleBusinessprofession and employment

1 to obtain some satisfaction

2Result cannot be measured in terms of money

3ExampleFamily-orientedreligious socialCultural and national

BUSINESS STUDIES

BUSINESS ENVIRONMENT

Welcome to the new sessionToday we are going to start the first chapter and the name of the chapter is Business Environment

In todayrsquos world every business enterprise is a part of the society It exists and operates in association with various groups in society such as customers suppliers competitors banks and financial institutions government agencies trade unions media and so on All these groups influence the functioning of business in one way or the other They constitute the environment of businessConcept of Business Environment

The term lsquobusiness environmentrsquo refers to the sum total of all individuals institutions and other forces that lie outside a business enterprise but that may influence its functioning and performance

The main features of business environment

Totality of External forces General and Specific forces Interrelatedness Complexity Dynamic Uncertainty Relativity

The Interrelation between business and its environment

The business enterprise is an open system It continuously interacts with its environment It takes inputs

Prepare the following questions from todayrsquos assignment

1 What do you mean by business environment

The term lsquobusiness environmentrsquo means the aggregate of all forces factors and institutions which are external to and beyond the control of an individual business enterprise but they may influence its functioning and performance Business environment is the macro framework within which a business firm a micro unit operates It consists of several interrelated and interacting elements

2 Explain the main features of business environment in brief

Totality of External forces-Business environment is the sum total of all things external to a business environment

General and Specific forces-It includes both the forces general forces are the economic social political legal and technological conditions which indirectly influence all business enterprise Specific forces are the investors customers competitors and suppliers which influence individual enterprise directly

Interrelatedness-Different elements of environment are interrelated for an example growing awareness for health care has increased the demand for health foods

Complexity- Business environment id

(such as raw materials capital labour energy and so on) from its environment transforms them into goods and services and sends them back to the environment

Fig 1 Business Environment Relationship

complex in nature as the elements keep on changing example economic technological and other forces changes in demand for a product and service

Dynamic-Business environment is not static it keeps on changing

Uncertainty- Itrsquos very difficult to predict future events such as technology and fashion which occur fast and frequently

Economics Basic Economic ConceptsSub topic

Microeconomics and

Macroeconomics

Welcome to the new sessiontoday we are going to start the first chapter of Class XI The name of the chapter that we are going to start is Basic Economic concepts

Now Economics covers the study of human activities Human activities are those activities which are performed by humans to satisfy their wants

Thus Human wants are unlimited and therefore economic activities such as production exchange and consumption are needed in order to satisfy those wants

The study of economics is divided largely in two parts which areMicroeconomics and Macroeconomics

SUBJECT- MATTER OF ECONOMICS

MICROECONOMICS MACROECONOMICS

Questions1Who has coined the words micro and macro economics

Answer Ranger Frisch coined the words lsquomicrorsquo and lsquomacrorsquo in 1933 to denote the two branches of economic theory namely microeconomics and macroeconomics

2What is microeconomicsAnswer It is the study of behaviour of individual decision ndash making unit such as consumers firms etc

3 What is macroeconomicsAnswer Macroeonomics is the study of overall economic phenomena like employment national income etc

4 What is the importance of microeconomicsAnswer

Microeconomics helps in formulating economic policies which enhance productive efficiency and results in greater social welfare

It helps the government in formulating correct price policies

It explains the working of a capitalistic economy where individual units(producers and consumers ) are free to take their own decision

Micro means a small part in

microeconomics we do not study the whole economy Hence we study an individual consumer and his or her choices and a producer and his or her profit maximizing decisions in the market Thus it does not mirror what happens in the economy as a whole

Macroeconomics on the other hand studies the economy as a whole It is concerned with aggregate and depicts the entire picture of the economyMacroeconomics deals with the national income aggregate investment aggregate consumption etc

Features of Microeconomics It deals with small

parts of the country Hence it looks at

individual consumers firms and industries

It deals with individual income consumption and savings

It studies the determination of price of any product or factors of production

It deals with the working of market via the price mechanism which is nothing but the determination of price and quantity of a commodity by the forces of demand and supply

Features of Macroeconomics

It deals with the study of the economy as a whole

It is concerned with

5 Give a limitation of microeconomics Microeconomics fails to explain the

functioning of an economy as a whole It cannot explain unemployment illiteracy and other problems prevailing in the country

6 What is the importance of macroeconomics It gives overall view of the growing

complexities of an economic system It provides the basic and logical

framework for formulating appropriate macroeconomic policies (eg for inflation poverty etc )to direct and regulate economy towards desirable goals

7What is the limitation of macroeconomics It ignores structural changes in an

individual unit of the aggregate

8 Differentiate between Microeconomics and Macroeconomics

Microeconomics Macroeconomics

the study of aggregates

National income aggregate savings and aggregate investments are major concepts dealt within macroeconomics style

It studies the determination of general price levels

It investigates into the problem of unemployment and the achievement of employment

It studies the aspect of decision making at the aggregate and national levels

It includes all growth theories whether related to developed or developing economies it also includes the study of economic systems and the working of the economy under different systems

Note Both Micro and macro economics are complementary and should be fully utilized for proper understanding of an economy

1It studies economic aspect of an individual unit2It deals with individual incomeConsumption and savings

3 It facilitates determination of price of any product or factors of production

4 Itrsquos scope is narrow and restricted to individual unit

1It studies the economy as a whole

2It deals with the national income aggregate consumption and aggregate savings3 It facilitates determination of general price level in an economy

4 Itrsquos scope is wide as it deals with economic units on the national level

ACCOUNTS

Introduction to Accounting and Book-keeping

Today I am going to share you the meaning of Accounting and Book-keeping and its related terms bullAccounting bullBook Keeping bullAccountsbullTypes Of Accounts bullAccounting Cycle

bull Meaning of accounting

Ans ) Accounting is the art and science of recording classifying and summarising monetary transactions

bull Meaning of Book-keeping

Ans) Bookkeeping is the art of recording business transactions with the view of having a permanent record of them and showing their effect on wealth

bull Meaning of account

Ans) The term account means a record of

business transactions concern a particular person of firm asset or income or expense It is a summarised record of all transactions which take place in an accounting year

bull Types of accountsPersonal accounts ndash Personal accounts relating

to person and Organisation are known as personal accounts Example Ramrsquos Account ABC amp Co Account etc

Real account - The accounts related to tangible and intangible assets are called real accounts Example Cash Account Furniture Account etc

Nominal account- Accounts related to expenses losses incomes and gains are known as nominal accounts Example Wages Account Salary Account Discount Account etc

bull Accounting cycle Accounting cycle refers to a complete sequence of accounting activities It begins with recording of transactions and ends with the preparation of a balance sheet

Chemistry TopicAtomic Structure

Thomsonrsquos atomic modelThomson (1898) was the first to propose the model of an atomHe proposed that an atom can be regarded as a uniform sphere of positive electricity in which requisite number of electrons are embedded evently to neutralize the positive chargeThis is just like plums embedded in a pudding or seeds evently distributed in red spongy mass of a watermelonThis model of atom is known as ldquoPlum-Pudding modelrdquo or

Q1)What is the fundamental constituents of atomAns Electron Proton and neutrons are the fundamental constituents of atomQ2)What is the value of fundamental unit of electricityAnsThe charge carried by one electron is sad to be the fundamental unit of electricityIts magnitude is 48times10-10esuOr 1602times10-19C Q3)Name the element containing no neutronAnsOrdinary hydrogen atom or protium 1H1

Types of AccountPersonal AccountReal AccountNominal AccountBalance Sheet (opening)

ldquowatermelon modelrdquoThis model could explain the electrical neutrality of an atom but failed to explain the result of scattering experiment carried out by Rutherford in 1911So it was rejected ultimately

Q4)Why is an electron called universal particleAns Itrsquos mass and Charge are independent of its source

EVS Chapter 1 ndash Modes of Existence

Modes of existence When one speaks normally about the mode of existence of some group or individual one refers to their customs their mode of being their ethology their habitat in some way their feeling for a placeDifferent modes of exixtence are ndash

1 Hunting ndashGathering2 Pastoral3 Agricultural4 Industrial

1 Hunting and gathering Hunting and gathering mode of existence is characterized by obtaining food from hunting wild animals including fishing and gathering wild plants From their earliest days the hunter-gatherer diet included various grasses tubers fruits seeds and nuts Lacking the means to kill larger animals they procured meat from smaller game or through scavenging

Societies that rely primarily or exclusively on hunting wild animals fishing and gathering wild fruits berries nuts and vegetables to support their diet are called hunting and gathering societies

At least this used to be practice of human beings before agriculture is invented As their brains evolved hominids developed more intricate knowledge of edible plant life and growth cycles

Q) Write the features of Hunting ndash gathering societiesAns - There are five basic characteristics of hunting and gathering societies

i The primary institution is the family which decides how food is to be shared and how children are to be socialized and which provides for the protection of its members

ii They tend to be small with fewer than fifty members

iii They tend to be nomadic moving to new areas when the current food supply in a given area has been exhausted

iv Members display a high level of interdependence

v Labor division is based on sex men hunt and women gather

Political Science

Introduction to political science

Political science occasionally called politology is a social science which deals with systems of governance and the analysis of political activities political thoughts associated constitutions and political behaviorThe study of political science involves the study of both the

Answer the following questions-1 What is political science

Political science occasionally called politology is a social science which deals with systems of governance and the analysis of political activities political thoughts associated constitutions and political behavior

2 Short notes-

traditional and modern theories of politicsTraditionalClassical political sciencepolitical theory-Traditional political science is the study of politics before Second World War The methodology to study Politics was traditional (legal formaletc) the definition of politics traditional (Politics begins and end with state)area of study (constitution state machinery)was traditionalModern Political scienceModern political theory-Modern Political Theory critically examines the contemporary state of political theory making an assessment of the achievement and limitations of the Behavioural Revolution in its totality and reviews objectively the major paradigms and conceptual frameworks adopted by the disciplineContemporary attempts at the development of an integrated political theory involving the use of both traditional and modern concepts approaches and theories-Around late 1960s several political scientists realized the importance of both the traditional political theory and modern Political theory They began building an integrated theory of politics involving a systematic mixture of traditional and modern studies of politics It was held that the study of a complex and vast field like politics needs both traditional as well as

Classical political theory Modern Political theory

Homework-Learn

modern concepts and approaches for studying itrsquos all aspects

Subject Eng Literature (The Tempest ndash William Shakespeare) Topic Act I Scene 1 Lines 1 to 32 (Line 32 ndash Gonzalo hellip If he be not born to be hanged our case is miserable) Date 13th April 2020 (3rd Period)

[Students should read the original play and also the paraphrase given in the school prescribed textbook]Summary Questions amp Answers

[SUMMARY OF THE ENTIRE SCENE]

o The play starts with the scene of a severe storm at sea Alonso (King of Naples) Sebastian (Alonsorsquos brother) Ferdinand (Alonsorsquos son) Gonzalo Antonio (the usurping Duke of Milan) are in a ship in the midst of the storm

o The mariners are trying their best to control the vessel from running aground and are totally following the orders of their Master the Boatswain They have scant success

o The mariners become extremely unhappy and annoyed when most of the passengers arrive on the deck thereby hampering their effort to save the ship There is serious confrontation between them and the passengers who are part of the Kingrsquos entourage

o The mariners could not save the ship

SUMMING-UP

(i) Vivid description of the scene which gives a realistic description of terror and confusion of a tropical storm

(ii) Shows Shakespearersquos accuracy of knowledge in describing the naval operations and also matters of seamanship

(iii) The opening scene justifies the title ndash The Tempest

UNANSWERED QUESTIONS

(i) The King always travels with his entire fleet including his soldiers Where

(1) GONZALO Nay good be patient (Line 15-26)BOATSWAIN When the sea is Hence What cares these

roarers for the name of the king To cabin silence Trouble us not

GONZALO Good yet remember whom thou has aboardBOATSWAIN None that I more love than myself You are a

councillor if you can command these elements to silence and work

the peace of the present we will not hand a rope more use your authority If you cannot give thanks you have

lived so long and make yourself ready in your cabin for the mischance of the hour if it so hap [To the Mariners]

Cheerly good hearts [To Gonzalo] Out of our way I say

(a) To whom is the boatswain speaking What does he mean by lsquoNone that I more love than myselfrsquo

The Boatswain is speaking to Gonzalo the honest old councilor of the Duke of MilanBy using the words ndash lsquoNone that I love more than I love myselfrsquo means that for the Boatswain nobody is dearer to him than his own life

(b) What were the conditions that made the boatswain react in this way

The Boatswain reacts in this way because the storm is at sea and Alonso King of Naples Sebastian his brother Ferdinand his son Gonzalo Antonio the usurping Duke of Milan on board are in distress and in panic Thus they have rushed to the deck interrupting the work of the mariners

(c) What hope does Gonzalo take from the attitude of the boatswain

The insolent and authoritative attitude of Boatswain makes Gonzalo feel comforted He tells that there are no signs that the Boatswain will be drowned But his facial appearance and attitude shows that he is destined to die on land by hanging which in effect means that all on board will be saved Otherwise all the persons on board are doomed

(d) How can they lsquomake yourself ready in your cabinrsquo For what were they asked to make ready themselves

In order to make themselves ready in their cabin the

were the other ships

(ii) Why was the ship in that area Where was it coming from or going where

(iii) The ship broke apart What happened to those who were in the ship

passengers on board must prepare for death which they will possibly soon have to meetThey can retire to their cabins and offer prayers to the Almighty to save them from drowning

(e) What does the boatswain say when he is asked to be patient What does he order to the royal party

When the boatswain is asked to be patient and remain calm he says that he will be patient only when the storm will be over and the sea will be calm but as long as the storm blows and there is danger to the ship he cannot think of being patient He orders the royal party to go to the cabin and leave the mariners to their work

(2) GONZALO I have great comfort from this fellow (Line 27-36)

Methinks he hath no drowning mark upon him his complexion is perfect

gallows Stand fast good Fate to his hanging Make the rope of his destiny our cable for our own doth little advantage If he be not born to be hanged our case is miserable

(a) Why does Gonzalo regard the Boatswain in the midst of danger

In the midst of danger Gonzalo regards the boatswain because he feels that the Boatswain is a source of comfort and is bent upon to do his work sincerely which in this case is saving the ship and its passengers from the severest of raging storm

(b) What reasons does Gonzalo give when he says that none in the ship will die of drowning

Gonzalo is almost sure that none in the ship will die by drowning His says that there is no mark on the face of the boatswain that indicates that he will die by drowning On the other hand the lines on his face are strong indications that he will be hanged to death Therefore there shall be no danger of the shiprsquos sinking

(c) Explain the following ldquoStand fast good Fate to his hanging Make the rope of his destiny our cable for our own doth little advantage If he be not born to be hanged our case is miserablerdquo

The stated lines mean that if the will of destiny is to be carried out then the ship will not get wrecked and all the passengers will be saved The safety of the passengers therefore depends upon the will of fate being carried out in the case of the boatswain If however the boatswain is not to die by hanging then the passengers are also very unsafe because in that case the ship is likely to sink

(d) What order does the Boatswain give to the sailors

when he re-enters What does he say about the crying of the fellows inside the cabin

The boatswain orders the sailors to bring the topmast lower and bring the ship close to a stationary position with the help of the main sail He says that the fellows inside the cabin are moaning and crying in their distress louder than his voice and louder even than the roaring of the storm

Class XII (ScienceCommerceHumanities) Subject Topic Summary Execution

Computer Science

PropositionalLogic

Propositional logic is a procedure to provide reasoning through statementProposition A ststement that results in True or False is said to be proposition There are two types of propositionSimple proposition amp compound propositionSimple proposioton A simple proposition is one that is not a part of any other proposition Such sentential form of proposition is symbolized with english letters in short For example Ram is a claver student (TrueFalse)Where do you live (Not in True or False)Grapes are sweet (TrueFalse)It rains today (TrueFalse)Here we can see some statements anwer would be true or false but some staements answer can not give in terms of true or false Thus the sentences which can be answered in true or false are known as simple propositionAssigning propositon to a variableThe general syntax to assign propostion to a variable is as followsVariable = Simple propositonFor example A=Ram is a clever studentB= Grapes are sweetC= it rains todayCompound proposition

helliphellipto be continued in next classhelliphellipMath Relation Relation If A and B are two non-empty sets

then a relation R from A to B is a subset of AxB If R A x B and (a b) R then we say that a sube isinis related to b by the relation R written as aRbeg Let A be the set of students of class XII and B be the set of students of class XI Then some of the examples of relation from A to B arei) (a b) AXB a is brother of bisinii) (a b) AXB age of a is more than age of isinb Types of relation In this section we would like to study different types of relations We know that a relation in a set A is a subset of A times A Thus the empty set φ and A times A are two extreme relations For illustration consider a relation R in the set A = 1 2 3 4 given by R = (a b) a ndash b = 10 This is the empty set as no pair (a b) satisfies the condition a ndash b = 10 Similarly R = (a b) | a ndash b | ge 0 is the whole primeset A times A as all pairs (a b) in A times A satisfy | a ndash

Example 1 Let A be the set of all students of a boys school Show that the relation R in A given by R = (a b) a is sister of b is the empty relation and R = (a b) the primedifference between heights of a and b is less than 3 meters is the universal relationSolution Since the school is boys school no student of the school can be sister of any student of the school Hence R = φ showing that R is the empty relation It is also obvious that the difference between heights of any two students of the school has to be less than 3 meters This shows that R = A times A is primethe universal relation Example 2 Show that the relation R in the set 1 2 3 given by R = (1 1) (2 2) (3 3) (1 2) (2 3) is reflexive

b | ge 0 These two extreme examples lead us to the following definitionsDefinition 1 A relation R in a set A is called empty relation if no element of A isrelated to any element of A ie R = φ A times AsubDefinition 2 A relation R in a set A is called universal relation if each element of A is related to every element of A ie R = A times A Both the empty relation and the universal relation are some times called trivial relation Definition 3 A relation R in a set A is called(i) reflexive if (a a) R for every a Aisin isin(ii) symmetric if (a1 a2) R implies that (aisin 2a1)

R for all aisin 1 a2 Aisin(iii) transitive if (a1 a2) R and (aisin 2 a3) R isinimplies that (a1 a3) R for all aisin 1 a2 a3 AisinDefinition 4 A relation R in a set A is said to be an equivalence relation if R is reflexive symmetric and transitive

but neither symmetric nor transitiveSolution R is reflexive since (1 1) (2 2) and (3 3) lie in R Also R is not symmetric as (1 2) R but (2 1) isin notinR Similarly R is not transitive as (1 2) R and (2 3) R but (1 3) R isin isin notinExample 3 Show that the relation R in the set Z of integers given byR = (a b) 2 divides a ndash b is an equivalence relationSolution R is reflexive as 2 divides (a ndash a) for all a Z isinFurther if (a b) R then 2 divides a isinndash b Therefore 2 divides b ndash a Hence (b a) R which shows that R is isinsymmetric Similarly if (a b) R and (b c) R isin isinthen a ndash b and b ndash c are divisible by 2 Now a ndash c = (a ndash b) + (b ndash c) is even (Why) So (a ndash c) is divisible by 2 This shows that R is transitive Thus R is an equivalence relation in ZExample 4 Let L be the set of all lines in a plane and R be the relation in L defined as R = (L1 L2) L1 is perpendicular to L2 Show that R is symmetric but neither reflexive nor transitiveSolution R is not reflexive as a line L1 can not be perpendicular to itself ie (L1 L1) R notinR is symmetric as (L1 L2) Risin

L1 is perpendicular to L2rArr L2 is perpendicular to L1rArr (L2 L1) RrArr isin

R is not transitive Indeed if L1 is perpendicular to L2 and L2 is perpendicular to L3 then L1 can never be perpendicular to L3 In fact L1 is parallel to L3 ie (L1 L2) R isin(L2 L3) R but (L1 L3) Risin notin

Chemistry Solid state Characteristics if Solids(i)The particles are locked in fixed positions they are unable to change their relative positions and this brings a definite shape and volume of a solid(ii)In a solid the constituent particles are held by strong forces of attractionThe forces of attraction may be bonding or non bonding(iii)The constituent particles in a solid pack together as closely as possibleoccupying most of the available space within the solidThus the empty space in a solid is very smallThis makes a solid highly rigid and nearly incompressibleThis also explains why a solid has high density and exhibits slow diffusionClassification of Solids

Q1)Define Crystalline solids AnsA Solid that has a definite geometrical shape and a sharp melting pointand whose constituent particles (atomsmolecules or ions) are arranged in a long range order of definite pattern extending throughout the solid is called a crystalline solidExNaClQ2)Define Amorphous solids AnsA solid that does not have a definite shape and a sharp melting pointand whose constituent particles (atomsmolecules or ions) are not arranged in a definite pattern is called an amorphoussolid

Crystalline solidsAmorphous solids

ExGlassRubberQ3)Classify Crystalline Solids Crystalline Solids

Physics Coloumbrsquos Law (Summary)

Before Going Into Coloumbrsquos Law We Will First Learn What is Charge Properties of Charge and Always remember that charge is quantized ie a body always have static charge of magnitude equal to some integral multiple of fundamental electronic charge e= 16 x 10- 19 C

Charge is the property of matter that causes it to produce and experience electrical and magnetic effects The study of the electrical charges at rest is called electrostatics When both electrical and magnetic effects are present the interaction between charges is referred to as electromagnetic

There exist two types of charges in nature positive and negative Like charges repel and unlike charges attract each other

The type of charge on an electron is negative The charge of a proton is the same as that of an electron but with a positive sign In an atom the number of electrons and the number of protons are equal The atom is therefore electrically neutral If one or more electrons are added to it it becomes negatively charged and is designated as negative ion However if one or more electrons are removed from an atom it becomes positively charged and is called a positive ion

The excess or deficiency of electrons in a body gives the concept of charge If there is an excess of electrons in a body it is negatively charged And if there is deficiency of electrons the body becomes positively charged Whenever addition or removal of electrons takes places the body acquires a charge

The SI Unit of charge is coulomb (C) In SI units the current is a fundamental quantity having a unit of ampere (A) The unit of charge is defined in terms of the unit of current Thus one coulomb is the charge transferred in one second across the section of a wire carrying a

Ionic SolidsMetallicSolids

Molecular Solids

current of one ampere

As q = It we have1 C = (1 A) (1 s)

The dimensions of charge are [A T]

Properties of Charge

(1) Quantization of Charge Electric charge can have only discrete values rather than any value That is charge is quantized The smallest discrete value of charge that can exist in nature is the charge on an electron given as

e = plusmn 16 x 10- 19 C

This is the charge attained by an electron and a protonA charge q must be an integral multiple of this basic unit That is

Q = plusmn ne where n = 1 2 hellip

Charge on a body can never be (frac12)e (23)e or 57e etcWhen we rub a glass rod with silk some electrons are transferred from the rod to the silk The rod becomes positively charged The silk becomes negatively charged The coulomb is a very large amount of charge A typical charge acquired by a rubbed body is 10 - 8 C

Biology Reproduction in organisms

Welcome to this new session 2020-21Today in this first chapter we mainly discuss about reproduction types needs and life span of some organismsWe also discuss about difference between sexual and asexual reproduction

Q1 What is reproductionReproduction is defined as a biological processin which an organism gives rise to young onessimilar to itselfQ2 What are the needs of reproductionbulli) Reproduction maintain life on earthii) It enables the continuity of the species generation after generationiii) It creates genetic variation among populationsQ3 Define Life span and write some orgnisms life spanbull Life span is the period from birth to

the natural death of an organism- OrganismsLife span1 Butterfly 1 - 2 weeks2 Fruit fly 30 days3Dog 10-13 years4 Rose5-7 years5 Tortoise100-150 years6 Banyan Tree -200 - 250 yearsQ4 Reproduction is of two types in case ofanimals but in case of plants vegetative propagation is also present

Asexual Reproduction Sexual Reproductioni) Always uniparentalii) Gametes are not involvediii) Only mitotic division involvediv) Somatic cells of parents are involvedv) Offsprings are genetically similar to the parents

i) Usually biparentalii) Gametes are involvediii) Meiosis occurs during gametogenesis Mitosis occurs after fertilisationiv) Germ cells of the parents are involvedv) offsprings are genetically different from the parents

COMMERCE BUSINESS ENVIRONMENT

Welcome to the new sessiontoday we are going to start the first chapter of Class XII The name of the chapter is Business Environment

Already many of you have got some idea about the word business environment form the first chapter of business studies in class XI

In todayrsquos world every business enterprise is a part of the society It exists and operates in association with various groups in society such as customers suppliers competitors banks and financial institutions government agencies trade unions media and so on All these groups influence the functioning of business in one way or the other They constitute the environment of businessConcept of Business Environment

The term lsquobusiness environmentrsquo refers to the sum total of all individuals institutions and other forces that lie outside a business enterprise but that may influence its functioning and performance

The main features of business environment Totality of External forces General and Specific forces Interrelatedness Complexity Dynamic Uncertainty

Prepare the following questions from todayrsquos assignment

2 What do you mean by business environment

The term lsquobusiness environmentrsquo means the aggregate of all forces factors and institutions which are external to and beyond the control of an individual business enterprise but they may influence its functioning and performance Business environment is the macro framework within which a business firm a micro unit operates It consists of several interrelated and interacting elements

2 Explain the main features of business environment in brief

Totality of External forces-Business environment is the sum total of all things external to a business environment

General and Specific forces-It

Relativity

The Interrelation between business and its environment

The business enterprise is an open system It continuously interacts with its environment It takes inputs (such as raw materials capital labour energy and so on) from its environment transforms them into goods and services and sends them back to the environment

Fig 1 Business Environment Relationship

includes both the forces general forces are the economic social political legal and technological conditions which indirectly influence all business enterprise Specific forces are the investors customers competitors and suppliers which influence individual enterprise directly

Interrelatedness-Different elements of environment are interrelated for an example growing awareness for health care has increased the demand for health foods

Complexity- Business environment id complex in nature as the elements keep on changing example economic technological and other forces changes in demand for a product and service

Dynamic-Business environment is not static it keeps on changing

Uncertainty- Itrsquos very difficult to predict future events such as technology and fashion which occur fast and frequently

Business Studies

Human Resources Management

Human resource of an organisation are the aggregate of knowledge skills attitudes of people working in it

The management system which deals with human resources is called human resource management

Features of HRMbullComprehensive functionbullPeople-oriented

Question1) What do you mean by human

resource management Answer) Human resource management may be defined as that field of Management which has to do with planning organising and controlling the functions of procuring developing maintaining and utilising the labour force

bullAction oriented bullPervasive function bullContinuous function

2) Explain the features of HRM in brief

Answer)bullHuman Resource Management is concerned with managing people at work bull Human Resource Management is concerned with employees which bring people and organisations together so that the goals of each are met bullHuman resource management considered every employees as an individual and also promote their satisfaction and growth bull Human resource management is inherent in all organisations and at all levelsbullManagement of human resources are ongoing on never ending process which requires a constant alertness and Awareness of human relations

3) ldquoHR function is said to be pervasiverdquowhy

Answer) Human resource management is required in all organisations whether it is private or government organisations armed forces sports organisations etc It permeatsall the functional areas like production marketing finance research etc This from this feature of human resource management it can be said that it is pervasive in nature

Economics Demand Q1DEFINITION OF DEMANDIn economics demand is the quantity of a good that consumers are willing and able to purchase at various prices during a given period of timeQ2DEMAND CURVEIn economics a demand curve is a graph depicting the relationship between the price of a certain commodity and the quantity of that commodity that is demanded at that pricQ3LAW OF DEMANDIn microeconomics the law of demand states that conditional on all else being equal as the price of a good increases quantity demanded decreases conversely as the price of a good decreases quantity demanded increasesQ4ASSUMPTION of LAW OF DEMAND(i)No change in price of related commodities(ii) No change in income of the consumer(iii) No change in taste and preferences customs habit and fashion of the consumer( No expectation regarding future change in priceQ5MARKET DEMAND SCHEDULEIn economics a market demand schedule is a tabulation of the quantity of a good that all consumers in a market will purchase at a

given price At any given price the corresponding value on the demand schedule is the sum of all consumersrsquo quantities demanded at that priceQ6INDIVIDUAL DEMAND SCHEDULEIndividual demand schedule refers to a tabular statement showing various quantities of a commodity that a consumer is willing to buy at various levels of price during a given period of timeQ7 FACTORS AFFECTING INDIVIDUAL DEMAND FOR A COMMODITY

The factors that influence a consumerrsquos decision to purchase a commodity are also known as determinants of demand The following factors affect the individual demand for a commodity1 price of the commodity2 price of related goods3 income of buyer of the commodity4 tastes and preferences of the buyer1 Price of the CommodityYou must have observed that when price of a commodity falls you tend to buy more of it and when its price rises you tend to buy less of it when all other factors remain constant (lsquoother things remaining the samersquo) In other words other things remaining the same there is an inverse relationship between the price of a commodity and its quantity demanded by its buyers This statement is in accordance with law of demand which you will study in the later part of this lesson Price of a commodity and its quantity demanded by its buyers are inversely related only when lsquoother things remain the samersquo So lsquoother things remaining the samersquo is an assumption when we study the effect of changes in the price of a commodity on its quantity demanded2 Price of Related goodsA consumer may demand a particular good But while buying that good heshe also asks the price of its related goods Related goods can be of two types-(i) Substitute goods(ii) Complementary goods While purchasing a good prices of its substitutes and complements do affect its quantity purchased(i) Price of Substitute Goods Substitute goods are those goods which can easily be used in place of one another for satisfaction of a particular want like tea and coffee An increase in price of substitute good leads to an increase in demand for the given commodity and a decrease in price of substitute good leads to a decrease in demand for the given commodity It means demand for a given commodity is directly affected by change in price of substitute goods For example if price of coffee increases the demand for tea will rise as tea will become relatively cheaper in comparison to coffee(ii) Price of Complementary goods Complementary goods are those goods which are used together to satisfy a particular want like car and petrol An increase in the price of complementary goods leads to a decrease in demand for the given commodity and a decrease in the price of complementary goods leads to an increase in demand for the given commodity For example if price of petrol falls then the demand for cars will increase as it will be relatively cheaper to use both the goods together So demand for a given commodity is inversely affected by change in price of complementary goods3 Income of the Buyer of CommodityDemand for a commodity is also affected by income of its buyer However the effect of change in income on demand depends on the nature of the commodity under consideration In case of some goods like full cream milk fine quality of rice (Basmati rice) etc demand for these commodities increases when income of the buyer increases and

demand for these commodities decreases when income of the buyer decreases Such goods whose demand increases with the increase in income of the buyer are called normal goods But there are some goods like coarse rice toned milk etc whose demand decreases when income of buyer increases and their demand increases when income of the buyer decreases Such goods whose demand decreases with the increase in income of the buyer are called inferior goods Suppose a consumer buys 10 Kgs of rice whose price is ` 25 per Kg He cannot afford to buy better quality of rice because the price of such rice is ` 50 per Kg The consumer is spending ` 250 per month on the purchase of rice Now if income of the consumer increases and he can afford ` 350 on purchase of 10 Kg of rice Now he can afford to buy some quantity of rice say 6 Kgs whose price is ` 25 per Kg and may buy 4 Kgs of rice whose price is ` 50 per Kg Thus he will buy 10 Kgs of rice by spending ` 350 per month Therefore we may conclude that demand for normal goods is directly related to the income of the buyer but demand for inferior goods is inversely related to the income of the buyer4 Tastes and Preferences of the BuyerThe demand for a commodity is also affected by the tastes and preferences of the buyers They include change in fashion customs habits etc Those commodities are preferred by the consumers which are in fashion So demand for those commodities rises which are in fashion On the other hand if a commodity goes out of the fashion its demand falls because no consumer will like to buy it(5) Number of Buyers in the Market(Population)Increase in population raises the market demand whereas decrease in population reduces the market demand for a commodity Not only the size of population but its composition like age (ratio of males females children and old people in population) also affects the demand for a commodity It is because of needs of children young old male and female population differs(6) Distribution of Income and WealthIf the distribution of income and wealth is more in favour of the rich demand for the commodities preferred by the rich such as comforts and luxuries is likely to be higher On the other hand if the distribution of income and wealth is more in favour of poor demand for commodities preferred by the poor such as necessities will be more(7) Season and Weather ConditionsThis is generally observed that the demand for woolens increases during winter whereas demand for ice creams and cold drinks increases during summer Similarly market demand for umbrellas rain coats increases during rainy seasonQ8 REASONS FOR OPERATION OF LAW OF DEMAND WHY DEMAND CURVE SLOPES DOWNWARDNow we will try to explain why does a consumer purchase more quantity of a commodity at a lower price and less of it at a higher price or why does the law of demand operate ie why does the demand curve slope downwards from left to right The main reasons for operation of law of demand are1 Law of Diminishing Marginal UtilityAs you have studied earlier law of diminishing marginal utility states that as we consume more and more units of a commodity the utility derived from each successive unit goes on decreasing The consumer will be ready to pay more for those units which provide him more utility and less for those which provide him less utility It implies that he will purchase more only when the price of the commodity falls2 Income Effect

When price of a commodity falls purchasing power or real income of the consumer increases which enables him to purchase more quantity of the commodity with the same money income Let us take an example Suppose you buy 4 ice creams when price of each ice cream is ` 25 If price of ice creams falls to ` 20 then with same money income you can buy 5 ice creams now3 Substitution EffectWhen price of a commodity falls it becomes comparatively cheaper as compared to its substitutes (although price of substitutes has not been changed) This will lead to rise in demand for the given commodity For example if coke and Pepsi both are sold at ` 10 each and price of coke falls Now coke has become relatively cheaper and will be substituted for Pepsi It will lead to rise in demand for coke4 Change in Number of BuyersWhen price of a commodity falls some old buyers may demand more of the commodity at the reduced price and some new buyers may also start buying this commodity who were not in a position to buy it earlier due to higher price This will lead to increase in number of buyers when price of the commodity falls As a result demand for the commodity rises when its price falls5 Diverse Uses of a CommoditySome commodities have diverse uses like milk It can be used for drinking for sweet preparation for ice cream preparation etc If price of milk rises its use may be restricted to important purpose only This will lead to reduction in demand for other less important uses When price of milk falls it can be put to other uses also leading to rise n demand for itQ9 EXCEPTIONS TO THE LAW OF DEMANDYou have studied in law of demand that a buyer is willing to buy more quantity of a commodity at a lower price and less of it at a higher price But in certain circumstances a rise in price may lead to rise in demand These circumstances are called Exceptions to the Law of Demand Some important exceptions are1 Giffen GoodsGiffen goods are special type of inferior goods in which negative income effect is stronger than negative substitution effect Giffen goods do not follow law of demand as their demand rises when their price rises Examples of Giffen goods are jowar and bajra etc2 Status Symbol GoodsSome goods are used by rich people as status symbols eg diamonds gold jewellary etc The higher the price the higher will be the demand for these goods When price of such goods falls these goods are no longer looked at as status symbol goods and tehrefore therir demand falls3 NecessitiesCommodities such as medicines salt wheat etc do not follow law of demandbecause we have to purchase them in minimum required quantity whatever their price may be4 Goods Expected to be ScarceWhen the buyers expect a scarcity of a particular good in near future they start buying more and more of that good even if their prices are rising For example during war famines etc people tend to buy more of some goods even at higher prices due to fear of their scarcity in near future

Political Science

Constitution of India-The

Preamble

The preamble-

Preamble-

The preamble is the most precious part of the constitution We the people of India having solemnly resolved to constitute India into a Sovereign Socialist Secular Democratic Republic and to secure to all its citizensA preamble is an introductory and expressionary statement in a document that explains the documents purpose and underlying philosophy When applied to the opening paragraphs of a statute it may recite historical facts pertinent to the subject of the statuteNature and purpose of the constitution-Purpose of the Constitution dictates permanent framework of the government to form a more perfect union to establish justice and ensure peace of thenationconstitution provide principles how the government can run itself following the rules and laws written in the constitution of each state keeps them balanced

Answer the following questions-

1 What is preambleA preamble is an introductory and expressionary statement in a document that explains the documents purpose and underlying philosophy2 What is the nature and

purpose of the constitutionConstitution dictatespermanent framework of the government to form a more perfect union to establish justice and ensure peace of the nation

Homework-Learn

Accounts Compatibilty mode

1MEANING OF PARTNERSHIPPartnership is a form of business organisation where two or more persons join hands to run a business They share the profits and losses according to the agreement amongst them According to the Indian Partnership Act 1932 ldquoPartnership is relation between persons who have agreed to share profits of a business carried on by all or any one of them acting for allrdquo For example one of your friends has passed class XII from National Institute of Open Schooling (NIOS) and wants to start a business Heshe approaches you to join in this venture Heshe wants you to contribute some money and participate in the business activities Both of you if join hands constitute a partnership2CHARACTERISTICS1048698 Agreement A partnership is formed by an agreement The agreement may be either oral or in writing It defines the relationship between the persons who agree to carry on business It may contain the terms of sharing profit and the capital to be invested by each partner etc The written agreement is known as partnership deed1048698 Number of persons There must be at least two persons to form a partnership

The maximum number of partners in a partnership firm can be 50 according toCompanies Act 20131048698 Business The Partnership is formed to carry on business with a purpose of earning profits The business should be lawful Thus if two or more persons agree to carry on unlawful activities it will not be termed as partnership1048698 Sharing Profits The partners agree to share profits in the agreed ratio In caseof loss all the partners have to bear it in the same agreed profit sharing ratio10486981048698Mutual Agency Every partner is an agent of the other partners Every partner can bind the firm and all other partners by hisher acts Each partner will be responsible and liable for the acts of all other partners10486981048698Unlimited liability The liability of each partner except that of a minor is unlimited Their liability extends to their personal assets also If the assets of the firm are insufficient to pay off its debts the partnersrsquo personal property can be used to satisfy the claim of the creditors of the partnership firm10486981048698Management All the partners have a right to mange the business However they may authorize one or more partners to manage the affairs of the business on their behalf10486981048698Transferability of Share No partner can transfer hisher share to any one including hisher family member without the consent of all other partners3PARTNERSHIP DEEDAgreement forms the basis of partnership The written form of the agreement is which a document of partnership is It contains terms and conditions regarding the conduct of the business It also explains relationship between the partners This document is called partnership deed Every firm can frame its own partnership deed in which the rights duties and liabilities of the partners are stated in detail It helps in settling the disputes arising among the partners during the general conduct of business 4CONTENTS OF PARTNERSHIP DEEDThe partnership deed generally contains the following (i) Name and address of the partnership firm(ii) Nature and objectives of the business(iii) Name and address of each partner(iv) Ratio in which profits is to be shared(v) Capital contribution by each partner(vi) Rate of Interest on capital if allowed(vii) Salary or any other remuneration to partners if allowed(viii) Rate of interest on loans and advances by a partner to the firm(ix) Drawings of partners and interest thereon if any(x) Method of valuation of goodwill and revaluation of assets and liabilities on the reconstitution of the partnership ie on the admission retirement or death of a partner(xi) Settlement of disputes by arbitration(xii) Settlement of accounts at the time of retirement or death of a partner5IN ABSENCE OF PARTNERSHIP DEEDThe partnership deed lays down the terms and conditions of partnership in regard to rights duties and obligations of the partners In the absence of partnership deed there may arise a controversy on certain issues like profit sharing ratio interest on

capital interest on drawings interest on loan and salary of the partners In such cases the provisions of the Indian Partnership Act becomes applicableSome of the Issues are(i) Distribution of Profit Partners are entitled to share profits equally(ii) Interest on Capital Interest on capital is not allowed(iii) Interest on Drawings No interest on drawing of the partners is to be charged(iv) Interest on Partnerrsquos Loan A Partner is allowed interest 6 per annum on the amount of loan given to the firm by himher(v) Salary and Commission to Partner A partner is not entitled to anysalary or commission or any other remuneration for managing the business

History TOPIC-TOWARDS INDEPENDENCE AND PARTITION THE LAST PHASE (1935-1947)

SUB TOPIC-IMPORTANT POLITICAL DEVELOPMENTS ndash GROWTH OF SOCIAL IDEAS

Socialism is a political social and economic philosophyLike in other parts of the world the Russian revolution of 1917 served as a great inspiration for revolutionaries in India who at that time were engaged in the struggle for liberation from British ruleSocialist ideas led to the formation of communist party of IndiaJAWAHARLAL NEHRU Among the early Congress leaders Jawaharlal Nehru was very much impressed and influenced by the Socialist ideas He also learnt about the Economic activities of the Soviet Union after the Bolshevic Revolution 1917 He made full use of them in IndiaThe election of Jawaharlal Nehru and Subhas Chandra Bose showed the Left wing tendency within CongressJawaharlal Nehru demanded economic freedom along with political freedom of the people in order to end the exploitation of masses

Nehrus working committee included three socialists leaders The Lucknow session was a landmark in the evolution of socialist ideas of the congressSUBHAS CHANDRA BOSE ndash Subhas Chandra Bose had socialist leaning Both Jawaharlal Nehru and Subhas Chandra Bose were known as leftist Congress men Later on National Congress divided into Leftist and rightist campCONGRESS SOCIALIST Within the Congress some leaders formed the Congress Socialist partyPattavi Sitaramyya Sardar Patel Rajendra Prasad had hostile attitude towards the Congress Socialist partyJawaharlals attitude was hesitant

1 QUESTION ndash Mention name of two Congress leaders who had socialist leaning

1ANSWER ndash Subhas Chandra Bose and Jawaharlal Nehru2QUESTION- In which session of the congress Jawaharlal elaborated his Socialist ideas2 ANSWER ndash Lucknow and Faizpur Session in December 1935 and 19363QUESTION ndash Why Congress was sharply divided into leftist and rightist camp 3ANSWER ndash Subhas Chandra Bosersquos attempt to seek re election for congress presidentship in 1939sharply divided the National Congress into Leftist and Rightist camp4 QUESTION ndash Who was MN Roy 4 ANSWER ndash Manabendra Roy first formed the Communist Party of India outside the country at Tashkent in 19205QUESTION ndash Who formed the Congress Socialist Party within the Congress5 ANSWER ndash Jaya Prakash Narayan Achyut Patwardhan Acharya Narendra Dev Ram Mohan Lohia Aruna Asaf Ali6QUESTION ndash When was the Congress Socialist Party formed What was its object6 ANSWER ndash 1934The Congress Socialist Party sought to work out socialist programme through the Congress They joined hands with the Congress and wanted to carry

Subhas Chandra Bose being expelled from the congress after the Tripuri rift he formed Forward BlockThere were basic differences between the Congress Socialists and the communistsTRADE UNION ACTIVITIES Maximum working class people lived in Bombay and Calcutta The working and living conditions of those workers were very miserable In this situation Shasipada Banerjee NM Lokhande protested against the oppression of the working class peopleThe first Trade Union Madras Labour Union was formed in 1918 by BP WadiaIndustrial strikes took place in Kanpur Calcutta Madras Jamshedpur and Ahmedabad AITUC was formed in Bombay in 1927 The growth of Trade union among the workers was slow because of the fear of the dismissal of the jobIn the mean time the Moderates as well as Communists left AITUC and formed separate organization

on National struggle with the help of workers and peasant class of the society7 QUESTION ndash What was the name of the party founded by Subhas Chandra Bose7 ANSWER- Forward Block8QUESTION ndash Who was Shasipada Banerjee8 ANSWER ndash Shasipada Banerjee was a radical Brahmo He founded a working menrsquos club to protest against exploitation of the British rulers towards the working class of India9 QUESTION ndash What was the weekly published by NM Lokhande9ANSWER- Dinabandhu10 QUESTION ndash Who founded Bombay Mill-Hands Association and in which year10 ANSWER- NM Lokhande in189011 QUESTION- Who was BP WadiaANSWER- BPWadia was the founder of Madras Labour Union in191812 QUESTION- What was the name of the first labour union of India12 ANSWER- Madras Labour Union13 QUESTION Who founded the Majur Mahajan 13 ANSWER GANDHIJI14 QUESTION What was the full form of AITUC When it was formed14 ANSWER All India Trade Union Congressin 192715QUESTION Who formed the Red Trade Union Congress and in which year15ANSWER The Communists formed the Red Trade Union Congress16 QUESTION What do you mean by Socialism16 ANSWER Socialism describes any political and economic theory that says the community rather than individuals should own and manage property and natural resources

Subject Eng Literature (The Tempest ndash William Shakespeare) Topic Act III Scene 3 Lines 1 to 52 (Line 52 ndash Brother my lord the Duke Stand to and do as we) Date 13th April 2020 (4th Period)

[Students should read the original play and also the paraphrase given in the school prescribed textbook]Summary Questions amp Answers

o Alonso Sebastian Antonio Gonzalo Adrian Francisco and others wandered about the island in search of Ferdinand and gets tired and hungry of the toil and at the same time gives up all hope of finding him

o Antonio and Sebastian are happy that Alonso is out of hope and decide to make another attempt on his life that night when being so tired they will be sleeping soundly

o Suddenly a solemn and strange music is heard in the air and several strange shapes enter bringing in a banquet These strange shapes then dance round it with gestures of salutation and then inviting the King to eat they depart

o Seeing this strange scene all are inclined to believe the tales told by travelers that there truly are ldquounicornsrdquo and ldquothe phoenixrsquo thronerdquo

1 ALONSO What harmony is this My good friends hark (L18-27)

GONZALO Marvellous sweet music

[Enter several strange shapes bringing in a banquet

they dance about it with gentle actions of salutation

and inviting the King and his companions to eat they depart]ALONSO Give us kind keepers heavens What were theseSEBASTIAN A living drollery Now I will believe

That there are unicorns that in Arabia

There is one tree the phoenixrsquo throne one phoenix

At this hour reigning thereANTONIO Ill believe both

And what does else want credit come to me

And Ill be sworn rsquotis true Travellers neer did lie

Though fools at home condemn rsquoem

(a) How did Prospero present an amazing spectacle before Alonso and his companions

Using his magic powers Prospero ordered strange shapes to lay a banquet before Alonso and his companions The shapes brought several dishes with tasty eatables in them They placed the dishes on a table before Alonso and his companions Then the strange shapes began to dance gracefully around the banquet While dancing they made gestures inviting them to eat the food Then suddenly the shapes disappeared(b) Who were the guests at the strange banquet Describe the lsquoliving drolleryrsquo

Alonso Sebastian Antonio Gonzalo Adrian and Francisco were the guests at the strange banquet

The term ldquoliving drolleryrdquo refers to live entertainment show In this context when Alonso the King of Naples Sebastian his brother Antonio the treacherous brother of Prospero Gonzalo the kind and loyal councillor to the King Adrian and Francisco came to the island they were hungry and weary in their spirits They heard a solemn and strange music They were shocked to see several strange shapes bringing in a banquet and these shapes danced about it with gentle action of salutation inviting the King and his companions to eat After this Sebastian described this show as lsquoliving drolleryrsquo(c) What is lsquophoenixrsquo What are lsquoUnicornsrdquo

The term lsquophoenixrsquo refers to a mythical Arabian bird which lived alone and perched on a solitary tree After one hundred years it expired in flames and rose again from its own ashes

lsquoUnicornsrsquo refers to the mythological four-footed beasts having horns in the centre of their foreheads When the horns are ground into powder the powder was believed to be

an aphrodisiac(d) How does Sebastian explain the puppet show OR Why does the speaker now believe in unicorns and phoenix

Sebastian finds several strange shapes bringing in the banquet They invite the king and his party for dinner and soon depart He tells that if such a strange sight can be a reality there is nothing incredible in the world and from the present moment he will believe anything He says that it is a strange dumb show enacted not by puppets but by living beings It is stranger than a travellerrsquos tale Seeing such a thing

before his own eyes he will no longer disbelieve the story about unicorns and phoenix(e) How do the other characters present respond to this living drollery

At the sight of the lsquoliving drolleryrsquo like Sebastian Gonzalo and Antonio too acted strangely Antonio told that he too now believes in unicorns and phoenix and anything else that seems to be incredible He too now believes in travellersrsquo tales Gonzalo told that if he would report those happenings in Naples nobody will believe him He considers that those gentle shapes were gentler in manner in comparison to the living beings Alonso was at first sight suspicious and told them that those strange shapes conveyed their meaning in expressive gestures when they seemed to lack speech by their movements and sounds Francisco was amazed at their mysterious disappearance

2 ALONSO Not I

(Line 43-52)GONZALO Faith sir you need not fear When we

were boysWho would believe that there were mountaineers

Dewlapped like bulls whose throats had hanging at rsquoem

Wallets of flesh Or that there were such men

Whose heads stood in their breasts Which now we find

Each putter-out of five for one will bring us

Good warrant ofALONSO I will stand to and feed

Although my lastmdashno matter since I feel

The best is past Brother my lord the Duke

Stand to and do as we

(a) How does Alonso respond at the spectacle of the shapes which were sent to them at the instruction of Prospero

After seeing the strange sight of appearing and disappearing of the shapes sent by Prospero to arrange a banquet for them Alonso says that his surprise at having seen those creatures is infinite and he is fully justified in feeling so much surprise He thinks that their shapes their gestures and the sounds they made were indeed amazing Although they do not possess the gift of speech yet they were able to convey their

thoughts by means of their gestures only

(b) What does Prospero say about the views expressed by Alonso regarding the shapes What does Francisco think about the shapesAfter hearing Alonsorsquos views about the shapes Prospero says that this manrsquos praise of the spirits is rather hasty He means to say that Alonso has shown great haste in reaching the conclusion about the shapes Francisco is amazed to see that those shapes disappeared in a mysterious way(c) What does Sebastian ask Alonso to doSebastian tells Alonso that the shapes having disappeared should not matter to them because they have left the eatables behind He asks Alonso to enjoy eating as they are extremely hungry but the king does not accept his offer of enjoying the dishes(d) How does Gonzalo try to dispel Alonsorsquos fear of those strange shapes What kind of references does he give to AlonsoGonzalo says that those who have travelled abroad have reported seeing even stranger sights than these shapes that Alonso and his companions have beheld Hence there is no reason to feel afraid of these shapes Gonzalo further adds that in his younger days he had heard strange stories from travelers and Alonso might have heard similar stories For instance it was said that there existed a certain race of

human beings who had huge lumps of flesh hanging at their throats and who therefore resembled bulls Then Gonzalo tells about a race of human beings whose heads were located at their breasts Gonzalo says that such stories were not believed by most people in those days but now-a-days these stories have become common(e) Explain the following lsquoEach putter-out of five for onersquoEnglish travellers often insured their trips with London brokers Those that went on foreign travels those days used to deposit a certain amount with some firm or company in London before their departure If the travelers failed to return the money was forfeited by the company with which it had been deposited But this money was repaid five-fold if the travelers returned safe and sound In this way a traveler stood a great chance of recovering the entire cost of his

travels(f) Give the explanatory meanings of the following expressions in the context of the above extract (i) Dewlapped (ii) Wallets of flesh

(iii) Putter-out(i) Dewlapped having big lumps of flesh at the necks(ii) Wallets of flesh large masses of flesh looking like bags(iii) Putter-out to invest money before commencing the travel

  • General methods of preparation of hydrogen
  • Chapter Dimensional Analysis (Summary)
    • Properties of Charge
Page 18:   · Web viewSubject. Topic. Summary. Execution. Hindi. व्याकरण. शरीरके अंगो के नाम लिखिए. 1) आँख 2) नाक 3

as a process whereby the real per capita income of a country increases over time along with fall in poverty ratio unemployment and income inequality etc

Distinguish between economic growth and economic development

Basis Economic growth

Economic development

Scope It has narrow scope as it refer only to rise in per capita income

It has wide concept since it includes qualitative changes as well

Concerned matter

It is concerned with the rise in income

It is concerned with not only rise in income but also reduction of poverty income inequality and unemployment

Focus Economic growth does not focus on economic development

Economic development focus on economic growth plus qualitative changes

Distinguish between capitalist economy and socialist economy

Ownership

Motive

Tool

Means of production are owned and managed by private people

Self interest and profit earning is the main motive

Price mechanism is a main tool to solve the economic problems

Means of production are owned and managed by the government

Social welfare is the main motive

Economic planning by the government is the main tool to solve the economic problem

Competition

Distribution of income

There exist large competition among buyers and sellers

There is existence of large inequalities of income

There is no such competition

There exist less inequalities of income

Math Topic ndash AlgebraChapter -Factorisation

Study item Factorising by taking out common factorSome solved sums from exercise 41

1) (i) 8xy3 + 12x2y2

= HCF of 8xy3 and 12x2y2 is 4xy2

= 4xy2(2y + 3x )

4) (ii) 28p2q2r ndash 42pq2r2

= HCF of 28p2q2r and 42pq2r2 is 14pq2r = 14pq2r (2p - 3r )5) (ii) 14mn + 22m - 62p=HCF of 14mn 22m and 62p is 2= 2(7mn + 11m - 31p)7) (ii) 3a(x2 + y2) + 6b (x2 + y2) = HCF of 3a(x2 + y2) and 6b(x2 + y2 ) is (x2 + y2)= ( x2+ y2 )(3a + 6b )9) (ii) x(x2 + y2 ndash z2 ) + y(-x2ndashy2 + z2 ) ndash z(x2+ y2 ndash z2 )= x(x2 + y2 -z2) ndash y-(x2 + y2 -z2) -z(x2 + y2 ndash z2)=x(x2 + y2-z2) -y( x2 + y2-z2) ndash z (x2 + y2 -z2)= (x2+ y2 ndash z2)(x ndash y ndash z )

Commercial Studies

Introduction to Accounting and Book-keeping

Today I am going to share you the meaning of Accounting and Book-keeping and its related terms bullAccounting bullBook Keeping bullAccountsbullTypes Of Accounts bullAccounting Cycle

bull Meaning of accounting

Ans )Accounting is the art and science of recording classifying and summarising monetary transactions

bull Meaning of Book-keeping

Ans) Bookkeeping is the art of recording business transactions with the view of having a permanent record of them and showing their effect on wealth

bull Meaning of account

Ans) The term account means a record of business transactions concern a particular person of firm asset or income or expense It is a summarised record of all transactions which take place in an accounting year

bull Types of accountsPersonal accounts ndash Personal accounts relating

to person and Organisation are known

as personal accounts Example Ramrsquos Account ABC amp Co Account etc

Real account - The accounts related to tangible and intangible assets are called real accountsExample Cash Account Furniture Account etc

Nominal account- Accounts related to expenses losses incomes and gains are known as nominal accountsExample Wages Account Salary Account Discount Account etc

bull Accounting cycle Accounting cycle refers to a complete sequence of accounting activities It begins with recording of transactions and ends with the preparation of a balance sheet

English 1 Transformation of sentences

Sentences A sentence is a group of words which makes complete sense

a Assertive sentencesb Imperative sentencesc Interrogative

sentencesd Exclamatory sentences

Sentences can be changed from one grammatical form to another without changing the meaning of the sentence This is known as transformation of sentences

Exercise 6Rewrite the following sentences according to the instructions given below without changing their meanings

1 As soon as he saw the beer he jumped into the river ( Begin No sooner)

2 None but brave deserve the fair (Begin the bravehellip)

3 This box is too heavy for me to lift ( Use so hellip That instead of too)

4 No one other than a king can live like James Luxurious ( Begin only James)

5 Oh for the wings of a dove (Begin I wishhellip)

BENGALI(2ND LANGUAGE)

ldquo বঙগভমির পরমি ldquo াইকেল ধসদন দতত

পব13পোসঠ আসোলিচত ৩ পরবোস দৈদসবর বস ীবতোরো Pলিদ স এ লেদ -আকো সত-োলি লেদ তোস - ক) বকতো লেক কোর লেো লেকো কলিবতোর অং ) কোর পরলিত বকতোর এই উলিকত গ) এ লেদ আকো সত বসত কী বলিঝসয়স4 ীবতোরো বসত কী লেবোঝ ঘ ) আসোচয অংসর তোৎপP13 কী

উ -ক ) বকতো স কলিব মোইসক ম3দ দতত

Types of AccountPersonal AccountReal AccountNominal AccountBalance Sheet (opening)

কলিব মোইসক ম3দ দসততর রলিচত বঙগভলিমর পরলিত কলিবতোর অং ) কলিব বঙগী অ13োৎ লেদমোতোর পরলিত কলিবর এই উলিকত গ ) এ লেদ আকো বসত কলিবর মোব লেদী রপ আকো লেক লেবোঝোসো সয়স4 আকো লেসক লেPম তোরো স পসর লেতমলি ীব লেদ রপ আকো লেসক পরো রপ তোরো স পরসত পোসর এই মভোবোর কোই কলিব বসস4 ঘ ) পরবো Pোতরোয় Pলিদ কলিবর লেদ আকো লেসক ীব তোরো রপ পরো স পসর তোসত কলিব লিবনদমোতর দঃলিত কোর মতয লিবসর সবোভোলিবক পলিরলিত এবং মোষ মরী তোই পরবোস Pলিদ তা োর মতয য় তবও কলিব লিবচলিত সব ো কোর পলিবীসত লেকউ অমর য় লিক4ই অকষয় য় দীর লেPম লিচরপরবোমো লেতমলি মোসষর ীবও চমোতোই ীব - সতবধতোই মতয ীব দীসত মোষ লিতয পরবোমো তবও লেPব মোষ আপ কতকসম13র মো3যসম মোসষর মস লিসসদর সথো কসর লিসত পোসর তোরো লিচরভোসবর সয় মোসষর মস লিবরো কসর তোসদর মস3য লেকউ পGভসত লিবী সয় গোসও মোসষর মস তোরো লিতযপলিত লিতযবলিনদত

Hindi 2ndlang

काकीी(लिसयारामशरणगपत)

इस कहानी म क न यह बतान का परयास निकया ह निक बचच अपनी मा स निकतना परम करत ह शयाम अबोध बाक ह वह अपनी मा क मरन क बाद उसन अपनी मा क लिए बहत रोया बाद म उस पता चा निक उसकी मा राम क घर ची गई ह आकाश म उडती हई पतग दकर उस हष हआ निक पतग क दवारा वह अपनी मा को नीच उतारगा इसक लिए वह अपनी निपता की जब स दो बार सवा रपया निनकाकर पतग और दो मोटी सी मन वाी अपन भाई स काकी एक कागज पर लिवा कर पतग म लिशव का दिदयानिनकाकर पतग और दो मोटी सी मन वाी अपन भाई स काकी एक कागज पर लिवा कर पतग म लिचपका दिदयाभोा और शयाम कोठरी म रससी बाधनी रह थ तभी उसक निपता करोध म आकर उन स पछ निक कया उनकी जब स रपया निनकाा हभोा डर क मार बताया निक शयाम इस पतग क दवारा अपनी काकी को राम क यहा स उतारना चाहता हनिवशशवर(शयाम क निपता)न फटी पतग उठाकर दी तो उस पर काकी लिा थावह हत बजि होकर वही ड रह गएउनहोन सोचा निक मन अपन पतर को मारा जोनिक अनजान और निनदष थावह अपनी मा कोनिकतना पयार करता ह

helliphellipContinue to next

Computer Application

Java Programming Prog 1Write a java program to input two numbers from user and display the sum or product of them as per user choice Use switch case statementSolve public class sum_product public static void main(String args[]) Scanner sc=new Scanner(Systemin) int abc Systemoutprintln(ldquoEnter two numbersrdquo) a=scnextInt() b=scnextInt() Systemoutprintln(ldquoPress 1 for sum or 2 for productrdquo)

c=scnextInt() switch(c) case 1 Systemoutprintln(ldquoThe sum will be =rdquo+(a+b)) break case 2 Systemoutprintln(ldquoThe product will be =rdquo+(ab)) break default Systemoutprintln(ldquoWrong Inputrdquo) Home Work - Practice in your computer using bluej

Subject Eng Literature (The Merchant of Venice ndash William Shakespeare)Topic Act I Scene 2 Lines 92 to 126 (End of scene) Date 13th April 2020 (5th Period)

[Students should read the original play and also the paraphrase given in the school prescribed textbook]Summary Questions amp Answers

o After Portia has expressed her opinion about the suitors Nerissa informs that she need not bother about any one of them as they have decided to quit Belmont at the earliest opportunity because they do not believe in trying their luck by the caskets which is the only way of winning Portia

o Nerissa then enquires of Portiarsquos opinion about Bassanio who once visited her in the company of the Marquis of Montferrat and says that she had never come across such an ideal love deserving the fairest lady for his bride

o Portia seems to remember Bassanio quite correctly and says that she agrees with Nerissa At this moment a servant informs Portia that the Prince of Morocco has arrived to try his luck by the caskets

o Portia tells Nerissa that if she could welcome this new suitor as gladly as she says farewell to the previous ones she would be glad of his arrival However if he happens to have the virtues of a saint but the black complexion of a devil she would prefer to have him for religious consolation rather than as a husband

(1) NERISSA You need not fear lady (Line 97-103)

the having any of these lords they have acquainted me with their determinations

which is indeed to return to their home and to

trouble you with no more suit unless you may be wonby some other sort than your fathers imposition depending on the caskets

PORTIA If I live to be as old as Sibylla I will die as chaste asDiana unless I be obtained by the manner of my fatherswill I am glad this parcel of wooers are so reasonablefor there is not one among them but I dote on his veryabsence and I pray God grant them a fair departure

(a) Elucidate the idea expressed in the first speech of the above dialogue

In the first speech Nerissa assures Portia that she need not have any fear of being compelled to marry anyone of the suitors who had lately come to Belmont She informs her that they have all decided to return to their respective countries(b) Illuminate the meaning of the phrase ldquoyour fatherrsquos imposition depending on the casketsrdquo

Nerissa means that the suitors of Portia do not find the conditions imposed by the will of her father to their liking They are too hard for them These conditions are that in the event of a suitor failing to choose the right casket (i) he should never disclose to anybody which casket he chose (ii) he can never marry and (iii) he should take his departure immediately(c) Explain the meaning of the term lsquoSibyllarsquo

lsquoSibyllarsquo is the name given by Romans and Greeks to a prophetess inspired by some deity usually the sun-god Apollo She had a very long life The god Apollo granted her as many years of life as she could hold grains of sand in her hand(d) Elucidate the meaning of the term lsquoDianarsquo

lsquoDianarsquo is the goddess of hunting She is also regarded as a symbol of virginity because she never fell in love and never

married(e) Explain the meaning of the first two lines of Portiarsquos speech

Portia says that even if she is to live for centuries like Sibylla she would not marry except in accordance to her fatherrsquos will She asserts that she would not mind remaining unmarried and untouched by a man like Diana the virgin the goddess of hunting unless a man is able to win her by passing the test laid down by her father

Class XSubject Topic Summary Execution

Hindi 2nd

Langबड घर की बटी( मशी परमचद)

lsquoबड घर की बीटीrsquo कहानी का उददशय मधयम वग की घर समसया को सझा कर सगदिठत परिरवार म मिम जकर परम स रहन का सदश दना ह घर म शानित tानिपत करन की जिजममदारी नारी की होती ह यदिद नारी समझदार ह उसम धय और परिरवार क परनित परम ह तो कोई भी घटना परिरवार को निवघदिटत नही कर सकती या कहानी परिरवार को सगदिठत करत हए परम सौहाद स एक रदसर की भावनाओ को समझ करउनका सहयोग करत हए जीवन यापन करन की पररणा दती ह मशीपरमचदर जी न इस कहानी म सयकत परिरवार का परनितनिनमिधतव निकया ह यह कहानी बनी माधव सिसह जो गौरी पर क जमीदार क उनक दो पतरो की हशरी कठ ा निबहारीशरीकात का निववाह एकजमीदार घरान की पतरी आनदी स हआ थाआनदी न द को ससरा क वातावरण म ढालिया थाएक दिदन आनदी का अपन दवर ा निबहारी स झगडा हो जाता ह दोनो भाई एक रदसर स अग होन की कोलिशश करत हसभी बह आनदी न अपन मधर वयवहार स ा निबहारी को घर छोडकर जान स रोक लिया| इस पर बनी माधव सिसह न कहा निक बड घर की बटी ऐसी ही होती ह जो निबगडा काम बना ती ह अतः शीषक साथक ह बड घर की बटी आनदी ह

helliphelliphelliphellipContinue to nextBiology Topic ndash Chp-1

CellWelcome to new session 2020-21Today we will start with Chpter 1 cell CELL

Protoplasm+Cellmembrane Or Cell wall

Cytoplasm+Neucleus

Cytoplasmic+ CytoplasmicOrganelles Inclutions(mitochondria (food Golgi bodies pigments)Ribosome)

What is cellbull Cell is the structural and functional unit of living organismbull According to number of cells organisms areUnicellular - Amoeba bacteria Multicellular - Rose Mango Tiger HumanSmallest cell -bacteria Longest cell - Nerve cellLargest cell - Ostrich egg cellCells are of different size and shapes according to their functionsQ2Write chief functions of following cellorganelles

Q3What is tonoplastVacuoles covered by a covering called tonoplast

Bengali(2Nd

Language)

ফ ফটক ো ফটক (কলিবতো ) ভোষ মসোপো3 gtPোয়

একটি লেমসয়র ীবস লেপরম লিকভোসব ফসট ওসঠ তো লেদলিসয়স4 কলিব লেপরম Pই য় লেই ময়ই বনত কোস পলিরত য় ফ লেফোটো বো োসফোটো লেটো ব2 কো য় লেমসয়সদর ব gtয13 লেপরসমর 4লিব ফসট উসঠস4 এই কলিবতোয় লেপরম মোষসক মত gtযর মস লেফস লিদসয় পরকষস বাোচোসোর gtয োত বো2োয় কলিবতোয় লেমসয়টির পসব13র দঃসর কো বো সও লেমসয়টি লেই পসর পলিক সত চোয়ো োরী ীবসর কোস4 পরম লেPৌবস লেপরমসক পোবোর পরব ইচছো োকসও তো পসর লেলিতবোচকতোয় পলিরত য় কলিব ভোষ মসোপো3 যোয় লেP ক লেপরসমর

কলিবতোয় ব gtযবহত লিবসষ লিক4 সvর অ13 লেদওয়ো ১) রসবোো= লেP লিবলিভনন রকম ডোকসত পোসর২) ো= পোর ৩) ঠলি = লেচোসর বZ৪)আই বস2ো=অলিববোলিত৫)শইসয় = োলিয়ত কসর৬)োতপাোচ= লিবলিভনন পরকোর৭)দ2োম = v কসর বZ কসর লেদওয়ো৮)লেরলিং =লেোোর দৈতরী লেব2ো৯) বনত= একঋত১০) পাোর = বসকরো2

Organelles Functions

1 Endoplasmic reticulum

2 Mitochondria

3Golgibodies

4 Ribosome5Lysosome

6Plastids

7 Centrosome

i) Supportive framework for the cellii) Synthesis and transpost of proteinsRelease of energy in the form of ATPi) Synthesis and secretion of enzymes hormoneii) Formation of vacuoles lysosomei) Protein Synthesisi) Intracellular digestionii) Destroy foreign substancei )Leucoplast - stores starchii)chloroplast - trap solar energyiii) Chromoplast - imparts colour toflowers amp fruitsi) Initiates and requlates cell division

কলিবতো তোর অ13সক ভোষোয় পরকোো কসর ঘলিরসয় ব যকত কসরস4 লেপরসমর ফতো আর লিবফতো লেক গোঢ় কসর লেদোসো কলিব ভোষ মসোপো3 যোসয়র অলিভবসর অ যলিদক

Economics

Factors of Production

Welcome to the new sessionToday we are going to start the first chapter of Class XThe name of the chapter is Factors of productionBy the name I hope you all can recall a glimpse of what you have learnt in the second chapter of Class IX

NowProduction is the process of creating the various goods and services which are consumed by the people of the country to satisfy their wants

Thus it is the process in which some materials are transformed from one form to another to create utility and value in goods

For example utility can be created by changing the form of a commodity ie

Making of table out of wood by a carpenter for his customer here the wood is getting transformed into table creating utility for his customer and he can also command a price for it

On the other hand Housewives perform very

useful activities at home which create utility but their domestic activities are not included in production because they have no money value

So we can also say that Production denotes two things firstly creation of utility and secondly creation of value

Production is not complete unless it reaches the consumer

An increase in production will increase the economic welfare of the consumers and hence the aim is to raise the production level of the country

Again production of a good or service is only possible if certain resources or

Questions

1 What do you mean by production

Answer Production means the creation of goods and services for the purpose of selling in the market

In fact production involves the transformation of inputs into outputs

Hence production denotes two thingsCreation of utility and creation of valueUtility and value can be created by changing the form by changing the place by changing the time and by rendering services

Example Transformation of raw

materials into finish goods such as potter creates utility by converting mud into utensils assembling of small parts to make bigger machinery

Production also includes services such as distribution and marketing

2 What are the factors of production

Answer Factors of Production refers to the resources and inputs needed for producing goods and servicesThese inputs can be classified as

Land Labour

Capital Enterprise

Land Land is defined to include not only the surface of the earth but also all other free gifts of nature(for example mineral resources forest resources and indeed anything that helps us to carry out the production of goods and services but is provided by

inputs are used together in right proportion

A resource or an input which helps in the process of production to obtain an output is called FACTOR OF PRODUCTION

These factors of production can broadly be categorized into four parts 1LAND 2LABOUR3CAPITAL4ENTERPRISE (ORGANISATION)or Entrepreneur

The above factors are all interdependent on each other and they play a major role in production process

FACTORS OF PRODUCTION

LANDCAPITAL

LABOUR ENTREPRENEUR

nature free of cost)LabourLabour refers to the human efforts that need to be combined with other factors of production for creating an output

CapitalAll man ndash made means of production is called capita example machineries which help in further production Money when used for starting any business for purchasing raw materials machinery tools etc it is regarded as capitalCapital also includes physical capital like factories machineriestoolsbuildingsequipments etcEnterpriseThe task of bearing risks is called enterprise and the person who bears these risks of business is called the entrepreneurThus an entrepreneur is one who organises production takes important decisions regarding production hires and purchases factors of production and bears the risk and uncertainty involved in productionOrganisation refers to the services of an entrepreneur who controls organises and undertakes all risks One who plans organises and manages a business enterprise is an organiser

Physics Chapter 1 Force

Force is an external agent capable of changing the state of rest or motion of a particular body It has a magnitude and a direction The direction towards which the force is applied is known as the direction of the force and the application of force is the point where force is applied The Force can be measured using a spring balance The SI unit of force is Newton (N)

Question 1

State the condition when on applying a force the body has

(a) the translational motion

(b) The rotational motion

Solutions

(a) Translational motion is produced when the body is free to move

(b) Rotational motion is produced when the body is pivoted at a point

Question 2

Define moment of force and state its SI unit

Solutions

The moment of force is equal to the product of the magnitude of the force and the perpendicular distance of the line of action of force from the axis

of rotation

The SI unit of moment of force is Newton times meter

= Newton meter (Nm)

Commercial Studies

Stake holders In this topic you will be come to know about the meaning and concept of stakeholders

How stakeholders are different from shareholders

Questions1 What do you mean by the term stake holdersAnswer) The term stake holders have developed from the words which mean an interest or expected benefit Stakeholders mean all those individuals groups and Institutions which have a state (interest) in the functioning and performance of a commercial organisation or a business enterprise2 What do you mean by share holdersAnswer) The person and Groups who own the shares of the joint stock company by providing capital to the company are called shareholders Shareholders are the internal stakeholders shareholders are one out of several stake holders3 How are shareholders different from stakeholdersAnswer)i) The term shareholders is related to only joint stock company whereas stakeholders are related with all business organisationsii) Stakeholders maybe any individual having financial stake in business organisation whereas a shareholders are those individuals who are holding shares in the company4) How are shareholders different from creditorsAnswer) i) Shareholders are internal stakeholders while creditors are external stakeholdersii) Shareholders invest in the capital of the company whereas creditors give loan to the companyiii) Shareholders are the members of the company with voting rights but creditors are not the members of the company

English 1 Transformation of sentences

Sentences A sentence is a group of words which makes complete sense

e Assertive sentencesf Imperative sentencesg Interrogative sentencesh Exclamatory sentences

Sentences can be changed from one grammatical form to another without changing the meaning of the sentence This is known as transformation of sentences

Exercise 1 Change the following affirmative sentences into Negative sentences

a He is a good manHe is not a bad man

b Ram loves SitaRam is not without love for Sita

c Only he stood first in the classNone but he stood first in the class

d Ankit was wiser than he

He was not so wise as Ankite He did it

He did not fail to do itf As soon as I reached college the

bell rangNo sooner did I reach college than the bell rang

g He finished everythingHe left nothing unfinished

h It always pours when it rainsIt never rains but it pours

Math Topic Commercial MathematicsChapter ndash Goods and services Tax

What is GSTAns It is a abbreviated term of Goods and Service Text which is an indirect tax levied on the sale of goods and rendering servicesSome terms related to GSTDelar Any person who buys goods or services For resale is known as a delar A delar Can be a firm or a companyIntra-state sales Sales of goods and services within the same state or same union territory are called intra- state salesInter-state sales Sales of goods and services outside the state or union territory are called Inter-state sales4) Input GST GST is paid by dealers on purchase of goods and services are called input GST5) Output GST GST is collected from customers on sale of goods and services are called output GST6) Types of GST There are three taxes applicable under GST(i) Central Goods and Services Tax (CGST)(ii) State Goods and Services Tax (SGST) or Union Territory Goods and Services Tax (UTGST) Both these taxes are levied on intra-state sales Here GST is divided equally among central and state governments(iii) Integrated Goods and Services Tax (IGST) IGST is levied on inter- state sales It is also levied on import of goods and services into India and export of goods and services from India

Subject Eng Literature (The Merchant of Venice ndash William Shakespeare)Topic Act III Scene 4 Lines 1 to 44 (Portia hellip To wish it back on you fare you well Jessica)[Students should read the original play and also the paraphrase given in the school prescribed textbook]

Summary Questions amp AnswersIn this scene we suddenly find a new element in the character of Portia We have already seen her possessed of every graceful womanly quality but now she shows that she is capable of rapid decision and determined action She shows this by her sudden resolve to hasten to Venice with a daring scheme for the rescue of Antonio This is an important scene in the dramatic action for it leads up to and renders possible the striking events of the famous trial scene which is one of the greatest striking elements of the play Moreover the fact that all the characters of importance are now assembled together in Venice makes the union of the main plot and the secondary story complete

(1) LORENZO Madam although I speak it in your presence(Line 1-9)

You have a noble and a true conceit

Of god-like amity which appears most strongly

In bearing thus the absence of your lordBut if you knew to whom you show this honourHow true a gentleman you send reliefHow dear a lover of my lord your husbandI know you would be prouder of the workThan customary bounty can enforce you

(a) Where is Lorenzo Why is he here To whom is he referring as lsquoMadamrsquo

Lorenzo is at Portiarsquos residence He had met Salerio on the way and Salerio had begged him to come along with him to

o In this scene Portia Nerissa Lorenzo Jessica and Balthazar appear

o Portia requests Lorenzo and Jessica to be in charge of her house during her absence from Belmont because she and Nerissa have decided to spend the days in meditation and also in visiting the holy places in the neighbourhood of Belmont She has already instructed her people to acknowledge both Lorenzo and Jessica as master and mistress of house during her absence Lorenzo and Jessica gladly agree to look after the house of Portia

handover the letter from Antonio to Bassanio The letter carried the bad news about Antoniorsquos arrest for non-payment of loan taken from Shylock Hence Salerio might have preferred company to break this bad news to Bassanio He is referring to Portia as Madam(b) What does Portia say on hearing the above extract

Portia says that she has never regretted doing good to others Friends who spend a lot of time together and really are there for each other have many traits in common As Antonio is Bassaniorsquos best friend saving him is like saving Bassanio who is like her own soul She asks Lorenzo to take care of management of the house till Bassanio is back(c) What does Portia send with Bassanio and why

On hearing about Antoniorsquos troubles on account of Bassanio her husband Portia immediately sends him with enough gold to repay the debt many times over to Venice to help Antonio out of his misfortune

(2) Lorenzo Madam with all my heart (Line 36-40)

I shall obey you in all fair commands

Portia My people do already know my mindAnd will acknowledge you and JessicaIn place of Lord Bassanio and myselfSo fare you well till we shall meet again

(a) Where are Lorenzo and Portia at this time What lsquofair commandsrsquo are given to Lorenzo

Lorenzo and Portia are at Belmont during this scenePortia reveals to Lorenzo that she has sworn to contemplate in prayer at a monastery around two miles away until her husband returns from Venice She tells him that Nerissa would accompany her and asks him to manage the house with Jessica till things are settled In response Lorenzo tells her that he would be obliged to do whatever she asks him to do(b) Where is Portia actually going and why

Portia tells Lorenzo that she would live a life of contemplation and pray at a monastery which is two miles away from her place In reality Portia plans to go to Venice in disguise with Nerissa and argue the case in defense of Antonio She is very sure that her plan would succeed

ClassXI (ScienceHumanitiesCommerce)Subject Topic Summary Execution

Computer Science

(APC)

Ch ndash 1 Numbers

(Numbers in different bases and

their Arithmatical operations)

Number System In computers Number System is defined as a writing system to represent the numbers in different ways ie we are using different symbols and notations to represent numbers There are four ways we can represent the number ndash Binary Decimal Octal and Hexadecimal

Decimal Number SystemThis number system consist 10 digits These are 0 1 2 3 4 5 6 7 8 amp 9

Binary Number SystemThis number system has only two digits these are 0 and 1 Here 0 stands for off while 1 stands for on

Octal Number SystemThis number system has 8 digits these are 0 1 2 3 4 5 6 amp 7

Hexadecimal Number SystemThis number system has 16 digits these are 0 1 2 3 4 5 6 7 8 9 A B C D E F Here the value of the alphabets are as follows A=10 B=11 C=12 D=13 E=14 F=15

Rules for conversion decimal number to Binary1 Divide the decimal number by 22 If the number will not divide equally by 2 then round down the answer to the nearest whole number (integer)3 Keep a note of the remainder it should be either 0 or 14 Keep repeating the above steps dividing each answer by 2 until you reach zero5 Write out all the remainders from bottom to top This is your binary solution

For example Lets convert 32 to binary 2 32 2 16 - 0 2 8 - 0 2 4 - 0 2 2 - 0 2 1 - 0 0 - 1

The binary equivalent of 3210 is 1000002

Try the follwing youself1 2410

2 4810

3 1210

History GROWTH OF NATIONALISM

The second half of the 19th century witnessed growth of political consciousness and a sense of Nationalism among the IndiansThere were various factors for growth of Indian Nationalism- As a result various political associations were formed in different provinces by the educated Indians Surendranath Banerjee organized a meeting of National conference at Calcutta Ultimately the National Congress was founded in Bombay in 1885This body became the vanguard of Indian struggle for freedom The congress leaders were known as moderates because they followed a policy of prayer and petition A large number of Indian leaders had experienced in political agitation The Political situation of England was also changed Moreover increasing revolutionary activities in Maharashtra Punjab and Bengal became serious concern to the British Government In this

QUESTION1 What do you mean by Nationalism ANSWER 1 Nationalism is defined as loyalty and devotion to own nation especially a sense of national consciousnessQUESTION 2 What are the causes of nationalism ANSWER 2 There were various factors for growth of nationalism

1 Spread of western education2 The progress of vernacular press and

patriotic literature3 The economic exploitation of our

country by the colonial rulers4 International affairs

QUESTION 3 Who organized National conference in Calcutta in 1883 ANSWER 3 Surendranath BanerjeeQUESTION 4 When did Indian National Congress formANSWER 4 Indian National Congress was formed in 1885 in BombayQUESTION 5 Who were ModeratesANSWER 5 The Early Nationalists were also known as Moderates Their emergence marked

background Lord Curzon became Viceroy in India He had no respect for the Indian National Congress

the beginning of the organized national movement in India They believed in British justice and were loyal to them They followed a policy of prayer and petition They demanded constitutional reforms of our country Impotant Moderate leaders were Pherozshah Mehta Dadabhai Naorozi and Surendranath Banerjee etcQUESTION 6 What do you know about Extremism in Indian National movementANSWER 6 In the beginning of 20th century a new class of national leaders emerged in India which was different from the moderate groups They started more aggressive movement against the British empire The goal of extremists was ldquoswarajrdquo Important extremist leaders were Bal Gangadhar Tilak Lala Lajpat Rai Bipin Chandra Pal etcQUESTION 7 Mention the places which were the main centres of Revolutionary movementANSWER 7 Maharashtra Bengal and Punjab

Physics

Chapter Dimensional Analysis

(Summary)

The dimensions of a physical quantity are the powers to which the fundamental units are raised in order to obtain the derived unit of that quantit

The physical quantites lengthmasstime are represented by [L] [M] [T] resp let they are raised to powers ( dimesions) abc resp then any physical quantity can be represented by [ La Mb Tc ] Examples

1 Area area = L x B = [L] x [L] = [M0 L2 T0 ]

2 Density density = massvolume = [M][L3] = [ M L-3]

3 Velocity velocity = distancetime = [L][T] = [LT-1]HW Try to find out dimension of acceleration Acceleration = velocity timeNB One can find the SI Units Using Dimension Analysis Such as for area we have [L2] so its SI unit is m2

Biology Topic ndash Chp-1 The living world

Today we will start the first chapter the living world Here we discuss about the characteristics of living organism and what are the difference between them and nonliving substances We also discuss about the contribution of different Scientists

There are over 500000 species of plants andover a million species of animal are present on earth Some 15000 new species were discovered every yearQ1 What is a living organismbull A living organism is primarily physico -chemical material that demonstrate a high degree of complexity is capable of selfRegulation possesses a metabolism and perpetuates itself through timeQ2 What are the differences between livingand non-livingsi) Compared with non-living living organisms

have more complex organised structure and their use of energy is more controlled amp efficientii) Living things reproduce their own kind by forming new cells which contains copies of their genesiii) Each organism has some degree of homeostasisie it is able to make adjustments so that internal environment remains constantQ3 Write contributions of following Scientists i) Aristotle - One of the first theories in Biology places all living things in a hiearchieii) AV Leeuwenhoek - was the first to observe living single celled organisms under microscopeii) Carolus Linnaeus - developed the binary system for naming of organisms and classificationiii) Geregor Johann Mendel ndash discoverbasic principles of inheritanceHomework i) C Darwin ii)Schleiden

Math Trigonometric functions

1 Overviewi) Trigonometry The word lsquotrigonometryrsquo is derived from the Greek words lsquotrigonrsquo and lsquometronrsquo which means measuring the sides of a triangle An angle is the amount of rotation of a revolving line with respect to a fixed line Usually we follow two types of conventions for measuring angles ie a) Sexagesimal system b) Circular system In Sexagesimal system the unit of measurement is Degree In Circular system the unit of measurement is Radian ii) Relation between degree and radianThe ratio of circumference of a circle to its diameter is always a constant This constant ratio is a number denoted by π which is taken approximately as 227The relationship between degree amp radian measurements is as follows2 right angles = 180deg= π radians1radian = 180degπ=57deg16(approx) 1deg=π180 radianiii) Length of an arc of a circleIf an arc of length s subtends an angle θ radians at the center of a circle of radius r then s=rθiv) Area of a sector of a circleA sector is like a pizza slice of the

Q) Express the following angles in radiana) 45deg b) 40deg3730Ans a) We have 180deg=π radiansi e 45deg= πtimes45180 radian = π4 radiansb) 40deg3730= 40deg37+3060 minute= 40deg 37 +12 minute= 40deg+ 752 minute=40 + 75(2times60) degree=3258 degreeNow 180deg=π radianie 3258 degree= (πtimes325) (180times8) radians = 65π288 radiansQ) A circle has a radius of r=12 meters What is the length of an arc traced out by a 60deg angle in the center of the circleAns In this problem we know both the central angle (60deg) and the radius of the circle (12) All we have to do is plug those values into our equation and we get

s = 2π(12)(60360)s = 24π6s = 4πSo the length of an arc traced out by a 60deg angle in a circle with a radius of 12 meters equals 4π meters asymp 1257 metersQ) Find the area of the sector with a central angle 30deg and a radius of 9cmAns GivenRadius r = 9 cmAngle θ = 30degArea of the sector = θ360degtimesπr2

= 30360degtimes227times92=2121cm2

circle It consists of a region bounded by two radii and an arc lying between the radiiThe area of a sector is a fraction of the area of the circle This area is proportional to the central angle In other words the bigger the central angle the larger is the area of the sectorArea of Sector = θ2 times r2 (when θ is in radians)

Area of Sector = θ times π360 times r2 (when θ is in degrees)

COMMERCE

CLASSIFICTION OF HUMAN ACTIVITIES-ECONOMIC AND NON-ECONOMIC

Welcome to the new sessiontoday we are going to start the first chapter of Class XI The name of the chapter that we are going to start is

lsquoClassification of Human Activities ndasheconomic and non-economicrsquo

Now let us start the chapter by considering human beings and the activities they perform throughout the day

Human activities means all those activities that human beings undertake to satisfy their wants

Human wants on the other hand are the desire of human beings for goods (vegetables fruits rice etc) and services (services of doctors teachers lawyers etc) that they require to live

Now these human activities continue throughout life as human wants are unending unlimited and recurring as human beings desire for better living throughout their lives

Now human activities can be classified into two categories

Human activities

Economic activities Non-economic activities

Economic activities are

Questions1 What are human activities

Answer Human activities mean all those activities that human beings undertake to satisfy their wants

Example A man working in an office

A boy playing in the garden

2What are the characteristics of human activitiesAnswer the characteristics of human activities are as follows

Human activities are undertaken by men women and children and these activities involve human efforts

Human activities are undertaken to satisfy human wants which are unlimited

Human activities continue throughout life

Human activities are performed for both earning money and personal satisfaction

3What is economic activitiesGive example

Answer Economic activities are undertaken by human beings with the object of earning money acquiring wealth and thereby satisfying human wantsExample

Selling of goods by a shop keeper to his customer

A clinic run by a doctor Service of a teacher in school or college

undertaken by human beings with the object of earning money and acquiring wealth

These activities result in the production of economic goods and services

Example Human activities(ie working in factories officesshops) which produce direct economic benefits

Non-economic activities are inspired by human sentiments and emotions such as love for the family desire to help the poor and love for the country

Thus these human activities (eg praying playing sleeping) produce no direct economic benefits and they are also not related to earning money and acquiring wealth

4 What are the characteristics of economic activities

Answer The characteristics of economic activities are as follows

Economic motiveEconomic activities are undertaken to earn money and acquire wealth

ProductiveEconomic activities involve productiondistribution and exchange of goods and services to create wealth

Economic growthEconomic activities determine the level of economic development of a country and standard of living of its citizens

Socially desirableEconomic activities are socially desirable for society

Economic resourcesEconomic activities make use of all the economic resources such landlabourcapital etc

5 What do you mean by non-economic activitiesExampleAnswerNon-economic activities are inspired by human sentiments and emotions such as love for the family desire to help the poor and love for the countryThese activities are not undertaken for monetary gain but for onersquos satisfaction and happinessExample

a mother looks after her children

a student donates blood8 Differentiate between Economic activities and Non-economic activities

Economic activities

Non-economic activities

1to earn living and acquiring wealth2Result can be measured in terms of money

3ExampleBusinessprofession and employment

1 to obtain some satisfaction

2Result cannot be measured in terms of money

3ExampleFamily-orientedreligious socialCultural and national

BUSINESS STUDIES

BUSINESS ENVIRONMENT

Welcome to the new sessionToday we are going to start the first chapter and the name of the chapter is Business Environment

In todayrsquos world every business enterprise is a part of the society It exists and operates in association with various groups in society such as customers suppliers competitors banks and financial institutions government agencies trade unions media and so on All these groups influence the functioning of business in one way or the other They constitute the environment of businessConcept of Business Environment

The term lsquobusiness environmentrsquo refers to the sum total of all individuals institutions and other forces that lie outside a business enterprise but that may influence its functioning and performance

The main features of business environment

Totality of External forces General and Specific forces Interrelatedness Complexity Dynamic Uncertainty Relativity

The Interrelation between business and its environment

The business enterprise is an open system It continuously interacts with its environment It takes inputs

Prepare the following questions from todayrsquos assignment

1 What do you mean by business environment

The term lsquobusiness environmentrsquo means the aggregate of all forces factors and institutions which are external to and beyond the control of an individual business enterprise but they may influence its functioning and performance Business environment is the macro framework within which a business firm a micro unit operates It consists of several interrelated and interacting elements

2 Explain the main features of business environment in brief

Totality of External forces-Business environment is the sum total of all things external to a business environment

General and Specific forces-It includes both the forces general forces are the economic social political legal and technological conditions which indirectly influence all business enterprise Specific forces are the investors customers competitors and suppliers which influence individual enterprise directly

Interrelatedness-Different elements of environment are interrelated for an example growing awareness for health care has increased the demand for health foods

Complexity- Business environment id

(such as raw materials capital labour energy and so on) from its environment transforms them into goods and services and sends them back to the environment

Fig 1 Business Environment Relationship

complex in nature as the elements keep on changing example economic technological and other forces changes in demand for a product and service

Dynamic-Business environment is not static it keeps on changing

Uncertainty- Itrsquos very difficult to predict future events such as technology and fashion which occur fast and frequently

Economics Basic Economic ConceptsSub topic

Microeconomics and

Macroeconomics

Welcome to the new sessiontoday we are going to start the first chapter of Class XI The name of the chapter that we are going to start is Basic Economic concepts

Now Economics covers the study of human activities Human activities are those activities which are performed by humans to satisfy their wants

Thus Human wants are unlimited and therefore economic activities such as production exchange and consumption are needed in order to satisfy those wants

The study of economics is divided largely in two parts which areMicroeconomics and Macroeconomics

SUBJECT- MATTER OF ECONOMICS

MICROECONOMICS MACROECONOMICS

Questions1Who has coined the words micro and macro economics

Answer Ranger Frisch coined the words lsquomicrorsquo and lsquomacrorsquo in 1933 to denote the two branches of economic theory namely microeconomics and macroeconomics

2What is microeconomicsAnswer It is the study of behaviour of individual decision ndash making unit such as consumers firms etc

3 What is macroeconomicsAnswer Macroeonomics is the study of overall economic phenomena like employment national income etc

4 What is the importance of microeconomicsAnswer

Microeconomics helps in formulating economic policies which enhance productive efficiency and results in greater social welfare

It helps the government in formulating correct price policies

It explains the working of a capitalistic economy where individual units(producers and consumers ) are free to take their own decision

Micro means a small part in

microeconomics we do not study the whole economy Hence we study an individual consumer and his or her choices and a producer and his or her profit maximizing decisions in the market Thus it does not mirror what happens in the economy as a whole

Macroeconomics on the other hand studies the economy as a whole It is concerned with aggregate and depicts the entire picture of the economyMacroeconomics deals with the national income aggregate investment aggregate consumption etc

Features of Microeconomics It deals with small

parts of the country Hence it looks at

individual consumers firms and industries

It deals with individual income consumption and savings

It studies the determination of price of any product or factors of production

It deals with the working of market via the price mechanism which is nothing but the determination of price and quantity of a commodity by the forces of demand and supply

Features of Macroeconomics

It deals with the study of the economy as a whole

It is concerned with

5 Give a limitation of microeconomics Microeconomics fails to explain the

functioning of an economy as a whole It cannot explain unemployment illiteracy and other problems prevailing in the country

6 What is the importance of macroeconomics It gives overall view of the growing

complexities of an economic system It provides the basic and logical

framework for formulating appropriate macroeconomic policies (eg for inflation poverty etc )to direct and regulate economy towards desirable goals

7What is the limitation of macroeconomics It ignores structural changes in an

individual unit of the aggregate

8 Differentiate between Microeconomics and Macroeconomics

Microeconomics Macroeconomics

the study of aggregates

National income aggregate savings and aggregate investments are major concepts dealt within macroeconomics style

It studies the determination of general price levels

It investigates into the problem of unemployment and the achievement of employment

It studies the aspect of decision making at the aggregate and national levels

It includes all growth theories whether related to developed or developing economies it also includes the study of economic systems and the working of the economy under different systems

Note Both Micro and macro economics are complementary and should be fully utilized for proper understanding of an economy

1It studies economic aspect of an individual unit2It deals with individual incomeConsumption and savings

3 It facilitates determination of price of any product or factors of production

4 Itrsquos scope is narrow and restricted to individual unit

1It studies the economy as a whole

2It deals with the national income aggregate consumption and aggregate savings3 It facilitates determination of general price level in an economy

4 Itrsquos scope is wide as it deals with economic units on the national level

ACCOUNTS

Introduction to Accounting and Book-keeping

Today I am going to share you the meaning of Accounting and Book-keeping and its related terms bullAccounting bullBook Keeping bullAccountsbullTypes Of Accounts bullAccounting Cycle

bull Meaning of accounting

Ans ) Accounting is the art and science of recording classifying and summarising monetary transactions

bull Meaning of Book-keeping

Ans) Bookkeeping is the art of recording business transactions with the view of having a permanent record of them and showing their effect on wealth

bull Meaning of account

Ans) The term account means a record of

business transactions concern a particular person of firm asset or income or expense It is a summarised record of all transactions which take place in an accounting year

bull Types of accountsPersonal accounts ndash Personal accounts relating

to person and Organisation are known as personal accounts Example Ramrsquos Account ABC amp Co Account etc

Real account - The accounts related to tangible and intangible assets are called real accounts Example Cash Account Furniture Account etc

Nominal account- Accounts related to expenses losses incomes and gains are known as nominal accounts Example Wages Account Salary Account Discount Account etc

bull Accounting cycle Accounting cycle refers to a complete sequence of accounting activities It begins with recording of transactions and ends with the preparation of a balance sheet

Chemistry TopicAtomic Structure

Thomsonrsquos atomic modelThomson (1898) was the first to propose the model of an atomHe proposed that an atom can be regarded as a uniform sphere of positive electricity in which requisite number of electrons are embedded evently to neutralize the positive chargeThis is just like plums embedded in a pudding or seeds evently distributed in red spongy mass of a watermelonThis model of atom is known as ldquoPlum-Pudding modelrdquo or

Q1)What is the fundamental constituents of atomAns Electron Proton and neutrons are the fundamental constituents of atomQ2)What is the value of fundamental unit of electricityAnsThe charge carried by one electron is sad to be the fundamental unit of electricityIts magnitude is 48times10-10esuOr 1602times10-19C Q3)Name the element containing no neutronAnsOrdinary hydrogen atom or protium 1H1

Types of AccountPersonal AccountReal AccountNominal AccountBalance Sheet (opening)

ldquowatermelon modelrdquoThis model could explain the electrical neutrality of an atom but failed to explain the result of scattering experiment carried out by Rutherford in 1911So it was rejected ultimately

Q4)Why is an electron called universal particleAns Itrsquos mass and Charge are independent of its source

EVS Chapter 1 ndash Modes of Existence

Modes of existence When one speaks normally about the mode of existence of some group or individual one refers to their customs their mode of being their ethology their habitat in some way their feeling for a placeDifferent modes of exixtence are ndash

1 Hunting ndashGathering2 Pastoral3 Agricultural4 Industrial

1 Hunting and gathering Hunting and gathering mode of existence is characterized by obtaining food from hunting wild animals including fishing and gathering wild plants From their earliest days the hunter-gatherer diet included various grasses tubers fruits seeds and nuts Lacking the means to kill larger animals they procured meat from smaller game or through scavenging

Societies that rely primarily or exclusively on hunting wild animals fishing and gathering wild fruits berries nuts and vegetables to support their diet are called hunting and gathering societies

At least this used to be practice of human beings before agriculture is invented As their brains evolved hominids developed more intricate knowledge of edible plant life and growth cycles

Q) Write the features of Hunting ndash gathering societiesAns - There are five basic characteristics of hunting and gathering societies

i The primary institution is the family which decides how food is to be shared and how children are to be socialized and which provides for the protection of its members

ii They tend to be small with fewer than fifty members

iii They tend to be nomadic moving to new areas when the current food supply in a given area has been exhausted

iv Members display a high level of interdependence

v Labor division is based on sex men hunt and women gather

Political Science

Introduction to political science

Political science occasionally called politology is a social science which deals with systems of governance and the analysis of political activities political thoughts associated constitutions and political behaviorThe study of political science involves the study of both the

Answer the following questions-1 What is political science

Political science occasionally called politology is a social science which deals with systems of governance and the analysis of political activities political thoughts associated constitutions and political behavior

2 Short notes-

traditional and modern theories of politicsTraditionalClassical political sciencepolitical theory-Traditional political science is the study of politics before Second World War The methodology to study Politics was traditional (legal formaletc) the definition of politics traditional (Politics begins and end with state)area of study (constitution state machinery)was traditionalModern Political scienceModern political theory-Modern Political Theory critically examines the contemporary state of political theory making an assessment of the achievement and limitations of the Behavioural Revolution in its totality and reviews objectively the major paradigms and conceptual frameworks adopted by the disciplineContemporary attempts at the development of an integrated political theory involving the use of both traditional and modern concepts approaches and theories-Around late 1960s several political scientists realized the importance of both the traditional political theory and modern Political theory They began building an integrated theory of politics involving a systematic mixture of traditional and modern studies of politics It was held that the study of a complex and vast field like politics needs both traditional as well as

Classical political theory Modern Political theory

Homework-Learn

modern concepts and approaches for studying itrsquos all aspects

Subject Eng Literature (The Tempest ndash William Shakespeare) Topic Act I Scene 1 Lines 1 to 32 (Line 32 ndash Gonzalo hellip If he be not born to be hanged our case is miserable) Date 13th April 2020 (3rd Period)

[Students should read the original play and also the paraphrase given in the school prescribed textbook]Summary Questions amp Answers

[SUMMARY OF THE ENTIRE SCENE]

o The play starts with the scene of a severe storm at sea Alonso (King of Naples) Sebastian (Alonsorsquos brother) Ferdinand (Alonsorsquos son) Gonzalo Antonio (the usurping Duke of Milan) are in a ship in the midst of the storm

o The mariners are trying their best to control the vessel from running aground and are totally following the orders of their Master the Boatswain They have scant success

o The mariners become extremely unhappy and annoyed when most of the passengers arrive on the deck thereby hampering their effort to save the ship There is serious confrontation between them and the passengers who are part of the Kingrsquos entourage

o The mariners could not save the ship

SUMMING-UP

(i) Vivid description of the scene which gives a realistic description of terror and confusion of a tropical storm

(ii) Shows Shakespearersquos accuracy of knowledge in describing the naval operations and also matters of seamanship

(iii) The opening scene justifies the title ndash The Tempest

UNANSWERED QUESTIONS

(i) The King always travels with his entire fleet including his soldiers Where

(1) GONZALO Nay good be patient (Line 15-26)BOATSWAIN When the sea is Hence What cares these

roarers for the name of the king To cabin silence Trouble us not

GONZALO Good yet remember whom thou has aboardBOATSWAIN None that I more love than myself You are a

councillor if you can command these elements to silence and work

the peace of the present we will not hand a rope more use your authority If you cannot give thanks you have

lived so long and make yourself ready in your cabin for the mischance of the hour if it so hap [To the Mariners]

Cheerly good hearts [To Gonzalo] Out of our way I say

(a) To whom is the boatswain speaking What does he mean by lsquoNone that I more love than myselfrsquo

The Boatswain is speaking to Gonzalo the honest old councilor of the Duke of MilanBy using the words ndash lsquoNone that I love more than I love myselfrsquo means that for the Boatswain nobody is dearer to him than his own life

(b) What were the conditions that made the boatswain react in this way

The Boatswain reacts in this way because the storm is at sea and Alonso King of Naples Sebastian his brother Ferdinand his son Gonzalo Antonio the usurping Duke of Milan on board are in distress and in panic Thus they have rushed to the deck interrupting the work of the mariners

(c) What hope does Gonzalo take from the attitude of the boatswain

The insolent and authoritative attitude of Boatswain makes Gonzalo feel comforted He tells that there are no signs that the Boatswain will be drowned But his facial appearance and attitude shows that he is destined to die on land by hanging which in effect means that all on board will be saved Otherwise all the persons on board are doomed

(d) How can they lsquomake yourself ready in your cabinrsquo For what were they asked to make ready themselves

In order to make themselves ready in their cabin the

were the other ships

(ii) Why was the ship in that area Where was it coming from or going where

(iii) The ship broke apart What happened to those who were in the ship

passengers on board must prepare for death which they will possibly soon have to meetThey can retire to their cabins and offer prayers to the Almighty to save them from drowning

(e) What does the boatswain say when he is asked to be patient What does he order to the royal party

When the boatswain is asked to be patient and remain calm he says that he will be patient only when the storm will be over and the sea will be calm but as long as the storm blows and there is danger to the ship he cannot think of being patient He orders the royal party to go to the cabin and leave the mariners to their work

(2) GONZALO I have great comfort from this fellow (Line 27-36)

Methinks he hath no drowning mark upon him his complexion is perfect

gallows Stand fast good Fate to his hanging Make the rope of his destiny our cable for our own doth little advantage If he be not born to be hanged our case is miserable

(a) Why does Gonzalo regard the Boatswain in the midst of danger

In the midst of danger Gonzalo regards the boatswain because he feels that the Boatswain is a source of comfort and is bent upon to do his work sincerely which in this case is saving the ship and its passengers from the severest of raging storm

(b) What reasons does Gonzalo give when he says that none in the ship will die of drowning

Gonzalo is almost sure that none in the ship will die by drowning His says that there is no mark on the face of the boatswain that indicates that he will die by drowning On the other hand the lines on his face are strong indications that he will be hanged to death Therefore there shall be no danger of the shiprsquos sinking

(c) Explain the following ldquoStand fast good Fate to his hanging Make the rope of his destiny our cable for our own doth little advantage If he be not born to be hanged our case is miserablerdquo

The stated lines mean that if the will of destiny is to be carried out then the ship will not get wrecked and all the passengers will be saved The safety of the passengers therefore depends upon the will of fate being carried out in the case of the boatswain If however the boatswain is not to die by hanging then the passengers are also very unsafe because in that case the ship is likely to sink

(d) What order does the Boatswain give to the sailors

when he re-enters What does he say about the crying of the fellows inside the cabin

The boatswain orders the sailors to bring the topmast lower and bring the ship close to a stationary position with the help of the main sail He says that the fellows inside the cabin are moaning and crying in their distress louder than his voice and louder even than the roaring of the storm

Class XII (ScienceCommerceHumanities) Subject Topic Summary Execution

Computer Science

PropositionalLogic

Propositional logic is a procedure to provide reasoning through statementProposition A ststement that results in True or False is said to be proposition There are two types of propositionSimple proposition amp compound propositionSimple proposioton A simple proposition is one that is not a part of any other proposition Such sentential form of proposition is symbolized with english letters in short For example Ram is a claver student (TrueFalse)Where do you live (Not in True or False)Grapes are sweet (TrueFalse)It rains today (TrueFalse)Here we can see some statements anwer would be true or false but some staements answer can not give in terms of true or false Thus the sentences which can be answered in true or false are known as simple propositionAssigning propositon to a variableThe general syntax to assign propostion to a variable is as followsVariable = Simple propositonFor example A=Ram is a clever studentB= Grapes are sweetC= it rains todayCompound proposition

helliphellipto be continued in next classhelliphellipMath Relation Relation If A and B are two non-empty sets

then a relation R from A to B is a subset of AxB If R A x B and (a b) R then we say that a sube isinis related to b by the relation R written as aRbeg Let A be the set of students of class XII and B be the set of students of class XI Then some of the examples of relation from A to B arei) (a b) AXB a is brother of bisinii) (a b) AXB age of a is more than age of isinb Types of relation In this section we would like to study different types of relations We know that a relation in a set A is a subset of A times A Thus the empty set φ and A times A are two extreme relations For illustration consider a relation R in the set A = 1 2 3 4 given by R = (a b) a ndash b = 10 This is the empty set as no pair (a b) satisfies the condition a ndash b = 10 Similarly R = (a b) | a ndash b | ge 0 is the whole primeset A times A as all pairs (a b) in A times A satisfy | a ndash

Example 1 Let A be the set of all students of a boys school Show that the relation R in A given by R = (a b) a is sister of b is the empty relation and R = (a b) the primedifference between heights of a and b is less than 3 meters is the universal relationSolution Since the school is boys school no student of the school can be sister of any student of the school Hence R = φ showing that R is the empty relation It is also obvious that the difference between heights of any two students of the school has to be less than 3 meters This shows that R = A times A is primethe universal relation Example 2 Show that the relation R in the set 1 2 3 given by R = (1 1) (2 2) (3 3) (1 2) (2 3) is reflexive

b | ge 0 These two extreme examples lead us to the following definitionsDefinition 1 A relation R in a set A is called empty relation if no element of A isrelated to any element of A ie R = φ A times AsubDefinition 2 A relation R in a set A is called universal relation if each element of A is related to every element of A ie R = A times A Both the empty relation and the universal relation are some times called trivial relation Definition 3 A relation R in a set A is called(i) reflexive if (a a) R for every a Aisin isin(ii) symmetric if (a1 a2) R implies that (aisin 2a1)

R for all aisin 1 a2 Aisin(iii) transitive if (a1 a2) R and (aisin 2 a3) R isinimplies that (a1 a3) R for all aisin 1 a2 a3 AisinDefinition 4 A relation R in a set A is said to be an equivalence relation if R is reflexive symmetric and transitive

but neither symmetric nor transitiveSolution R is reflexive since (1 1) (2 2) and (3 3) lie in R Also R is not symmetric as (1 2) R but (2 1) isin notinR Similarly R is not transitive as (1 2) R and (2 3) R but (1 3) R isin isin notinExample 3 Show that the relation R in the set Z of integers given byR = (a b) 2 divides a ndash b is an equivalence relationSolution R is reflexive as 2 divides (a ndash a) for all a Z isinFurther if (a b) R then 2 divides a isinndash b Therefore 2 divides b ndash a Hence (b a) R which shows that R is isinsymmetric Similarly if (a b) R and (b c) R isin isinthen a ndash b and b ndash c are divisible by 2 Now a ndash c = (a ndash b) + (b ndash c) is even (Why) So (a ndash c) is divisible by 2 This shows that R is transitive Thus R is an equivalence relation in ZExample 4 Let L be the set of all lines in a plane and R be the relation in L defined as R = (L1 L2) L1 is perpendicular to L2 Show that R is symmetric but neither reflexive nor transitiveSolution R is not reflexive as a line L1 can not be perpendicular to itself ie (L1 L1) R notinR is symmetric as (L1 L2) Risin

L1 is perpendicular to L2rArr L2 is perpendicular to L1rArr (L2 L1) RrArr isin

R is not transitive Indeed if L1 is perpendicular to L2 and L2 is perpendicular to L3 then L1 can never be perpendicular to L3 In fact L1 is parallel to L3 ie (L1 L2) R isin(L2 L3) R but (L1 L3) Risin notin

Chemistry Solid state Characteristics if Solids(i)The particles are locked in fixed positions they are unable to change their relative positions and this brings a definite shape and volume of a solid(ii)In a solid the constituent particles are held by strong forces of attractionThe forces of attraction may be bonding or non bonding(iii)The constituent particles in a solid pack together as closely as possibleoccupying most of the available space within the solidThus the empty space in a solid is very smallThis makes a solid highly rigid and nearly incompressibleThis also explains why a solid has high density and exhibits slow diffusionClassification of Solids

Q1)Define Crystalline solids AnsA Solid that has a definite geometrical shape and a sharp melting pointand whose constituent particles (atomsmolecules or ions) are arranged in a long range order of definite pattern extending throughout the solid is called a crystalline solidExNaClQ2)Define Amorphous solids AnsA solid that does not have a definite shape and a sharp melting pointand whose constituent particles (atomsmolecules or ions) are not arranged in a definite pattern is called an amorphoussolid

Crystalline solidsAmorphous solids

ExGlassRubberQ3)Classify Crystalline Solids Crystalline Solids

Physics Coloumbrsquos Law (Summary)

Before Going Into Coloumbrsquos Law We Will First Learn What is Charge Properties of Charge and Always remember that charge is quantized ie a body always have static charge of magnitude equal to some integral multiple of fundamental electronic charge e= 16 x 10- 19 C

Charge is the property of matter that causes it to produce and experience electrical and magnetic effects The study of the electrical charges at rest is called electrostatics When both electrical and magnetic effects are present the interaction between charges is referred to as electromagnetic

There exist two types of charges in nature positive and negative Like charges repel and unlike charges attract each other

The type of charge on an electron is negative The charge of a proton is the same as that of an electron but with a positive sign In an atom the number of electrons and the number of protons are equal The atom is therefore electrically neutral If one or more electrons are added to it it becomes negatively charged and is designated as negative ion However if one or more electrons are removed from an atom it becomes positively charged and is called a positive ion

The excess or deficiency of electrons in a body gives the concept of charge If there is an excess of electrons in a body it is negatively charged And if there is deficiency of electrons the body becomes positively charged Whenever addition or removal of electrons takes places the body acquires a charge

The SI Unit of charge is coulomb (C) In SI units the current is a fundamental quantity having a unit of ampere (A) The unit of charge is defined in terms of the unit of current Thus one coulomb is the charge transferred in one second across the section of a wire carrying a

Ionic SolidsMetallicSolids

Molecular Solids

current of one ampere

As q = It we have1 C = (1 A) (1 s)

The dimensions of charge are [A T]

Properties of Charge

(1) Quantization of Charge Electric charge can have only discrete values rather than any value That is charge is quantized The smallest discrete value of charge that can exist in nature is the charge on an electron given as

e = plusmn 16 x 10- 19 C

This is the charge attained by an electron and a protonA charge q must be an integral multiple of this basic unit That is

Q = plusmn ne where n = 1 2 hellip

Charge on a body can never be (frac12)e (23)e or 57e etcWhen we rub a glass rod with silk some electrons are transferred from the rod to the silk The rod becomes positively charged The silk becomes negatively charged The coulomb is a very large amount of charge A typical charge acquired by a rubbed body is 10 - 8 C

Biology Reproduction in organisms

Welcome to this new session 2020-21Today in this first chapter we mainly discuss about reproduction types needs and life span of some organismsWe also discuss about difference between sexual and asexual reproduction

Q1 What is reproductionReproduction is defined as a biological processin which an organism gives rise to young onessimilar to itselfQ2 What are the needs of reproductionbulli) Reproduction maintain life on earthii) It enables the continuity of the species generation after generationiii) It creates genetic variation among populationsQ3 Define Life span and write some orgnisms life spanbull Life span is the period from birth to

the natural death of an organism- OrganismsLife span1 Butterfly 1 - 2 weeks2 Fruit fly 30 days3Dog 10-13 years4 Rose5-7 years5 Tortoise100-150 years6 Banyan Tree -200 - 250 yearsQ4 Reproduction is of two types in case ofanimals but in case of plants vegetative propagation is also present

Asexual Reproduction Sexual Reproductioni) Always uniparentalii) Gametes are not involvediii) Only mitotic division involvediv) Somatic cells of parents are involvedv) Offsprings are genetically similar to the parents

i) Usually biparentalii) Gametes are involvediii) Meiosis occurs during gametogenesis Mitosis occurs after fertilisationiv) Germ cells of the parents are involvedv) offsprings are genetically different from the parents

COMMERCE BUSINESS ENVIRONMENT

Welcome to the new sessiontoday we are going to start the first chapter of Class XII The name of the chapter is Business Environment

Already many of you have got some idea about the word business environment form the first chapter of business studies in class XI

In todayrsquos world every business enterprise is a part of the society It exists and operates in association with various groups in society such as customers suppliers competitors banks and financial institutions government agencies trade unions media and so on All these groups influence the functioning of business in one way or the other They constitute the environment of businessConcept of Business Environment

The term lsquobusiness environmentrsquo refers to the sum total of all individuals institutions and other forces that lie outside a business enterprise but that may influence its functioning and performance

The main features of business environment Totality of External forces General and Specific forces Interrelatedness Complexity Dynamic Uncertainty

Prepare the following questions from todayrsquos assignment

2 What do you mean by business environment

The term lsquobusiness environmentrsquo means the aggregate of all forces factors and institutions which are external to and beyond the control of an individual business enterprise but they may influence its functioning and performance Business environment is the macro framework within which a business firm a micro unit operates It consists of several interrelated and interacting elements

2 Explain the main features of business environment in brief

Totality of External forces-Business environment is the sum total of all things external to a business environment

General and Specific forces-It

Relativity

The Interrelation between business and its environment

The business enterprise is an open system It continuously interacts with its environment It takes inputs (such as raw materials capital labour energy and so on) from its environment transforms them into goods and services and sends them back to the environment

Fig 1 Business Environment Relationship

includes both the forces general forces are the economic social political legal and technological conditions which indirectly influence all business enterprise Specific forces are the investors customers competitors and suppliers which influence individual enterprise directly

Interrelatedness-Different elements of environment are interrelated for an example growing awareness for health care has increased the demand for health foods

Complexity- Business environment id complex in nature as the elements keep on changing example economic technological and other forces changes in demand for a product and service

Dynamic-Business environment is not static it keeps on changing

Uncertainty- Itrsquos very difficult to predict future events such as technology and fashion which occur fast and frequently

Business Studies

Human Resources Management

Human resource of an organisation are the aggregate of knowledge skills attitudes of people working in it

The management system which deals with human resources is called human resource management

Features of HRMbullComprehensive functionbullPeople-oriented

Question1) What do you mean by human

resource management Answer) Human resource management may be defined as that field of Management which has to do with planning organising and controlling the functions of procuring developing maintaining and utilising the labour force

bullAction oriented bullPervasive function bullContinuous function

2) Explain the features of HRM in brief

Answer)bullHuman Resource Management is concerned with managing people at work bull Human Resource Management is concerned with employees which bring people and organisations together so that the goals of each are met bullHuman resource management considered every employees as an individual and also promote their satisfaction and growth bull Human resource management is inherent in all organisations and at all levelsbullManagement of human resources are ongoing on never ending process which requires a constant alertness and Awareness of human relations

3) ldquoHR function is said to be pervasiverdquowhy

Answer) Human resource management is required in all organisations whether it is private or government organisations armed forces sports organisations etc It permeatsall the functional areas like production marketing finance research etc This from this feature of human resource management it can be said that it is pervasive in nature

Economics Demand Q1DEFINITION OF DEMANDIn economics demand is the quantity of a good that consumers are willing and able to purchase at various prices during a given period of timeQ2DEMAND CURVEIn economics a demand curve is a graph depicting the relationship between the price of a certain commodity and the quantity of that commodity that is demanded at that pricQ3LAW OF DEMANDIn microeconomics the law of demand states that conditional on all else being equal as the price of a good increases quantity demanded decreases conversely as the price of a good decreases quantity demanded increasesQ4ASSUMPTION of LAW OF DEMAND(i)No change in price of related commodities(ii) No change in income of the consumer(iii) No change in taste and preferences customs habit and fashion of the consumer( No expectation regarding future change in priceQ5MARKET DEMAND SCHEDULEIn economics a market demand schedule is a tabulation of the quantity of a good that all consumers in a market will purchase at a

given price At any given price the corresponding value on the demand schedule is the sum of all consumersrsquo quantities demanded at that priceQ6INDIVIDUAL DEMAND SCHEDULEIndividual demand schedule refers to a tabular statement showing various quantities of a commodity that a consumer is willing to buy at various levels of price during a given period of timeQ7 FACTORS AFFECTING INDIVIDUAL DEMAND FOR A COMMODITY

The factors that influence a consumerrsquos decision to purchase a commodity are also known as determinants of demand The following factors affect the individual demand for a commodity1 price of the commodity2 price of related goods3 income of buyer of the commodity4 tastes and preferences of the buyer1 Price of the CommodityYou must have observed that when price of a commodity falls you tend to buy more of it and when its price rises you tend to buy less of it when all other factors remain constant (lsquoother things remaining the samersquo) In other words other things remaining the same there is an inverse relationship between the price of a commodity and its quantity demanded by its buyers This statement is in accordance with law of demand which you will study in the later part of this lesson Price of a commodity and its quantity demanded by its buyers are inversely related only when lsquoother things remain the samersquo So lsquoother things remaining the samersquo is an assumption when we study the effect of changes in the price of a commodity on its quantity demanded2 Price of Related goodsA consumer may demand a particular good But while buying that good heshe also asks the price of its related goods Related goods can be of two types-(i) Substitute goods(ii) Complementary goods While purchasing a good prices of its substitutes and complements do affect its quantity purchased(i) Price of Substitute Goods Substitute goods are those goods which can easily be used in place of one another for satisfaction of a particular want like tea and coffee An increase in price of substitute good leads to an increase in demand for the given commodity and a decrease in price of substitute good leads to a decrease in demand for the given commodity It means demand for a given commodity is directly affected by change in price of substitute goods For example if price of coffee increases the demand for tea will rise as tea will become relatively cheaper in comparison to coffee(ii) Price of Complementary goods Complementary goods are those goods which are used together to satisfy a particular want like car and petrol An increase in the price of complementary goods leads to a decrease in demand for the given commodity and a decrease in the price of complementary goods leads to an increase in demand for the given commodity For example if price of petrol falls then the demand for cars will increase as it will be relatively cheaper to use both the goods together So demand for a given commodity is inversely affected by change in price of complementary goods3 Income of the Buyer of CommodityDemand for a commodity is also affected by income of its buyer However the effect of change in income on demand depends on the nature of the commodity under consideration In case of some goods like full cream milk fine quality of rice (Basmati rice) etc demand for these commodities increases when income of the buyer increases and

demand for these commodities decreases when income of the buyer decreases Such goods whose demand increases with the increase in income of the buyer are called normal goods But there are some goods like coarse rice toned milk etc whose demand decreases when income of buyer increases and their demand increases when income of the buyer decreases Such goods whose demand decreases with the increase in income of the buyer are called inferior goods Suppose a consumer buys 10 Kgs of rice whose price is ` 25 per Kg He cannot afford to buy better quality of rice because the price of such rice is ` 50 per Kg The consumer is spending ` 250 per month on the purchase of rice Now if income of the consumer increases and he can afford ` 350 on purchase of 10 Kg of rice Now he can afford to buy some quantity of rice say 6 Kgs whose price is ` 25 per Kg and may buy 4 Kgs of rice whose price is ` 50 per Kg Thus he will buy 10 Kgs of rice by spending ` 350 per month Therefore we may conclude that demand for normal goods is directly related to the income of the buyer but demand for inferior goods is inversely related to the income of the buyer4 Tastes and Preferences of the BuyerThe demand for a commodity is also affected by the tastes and preferences of the buyers They include change in fashion customs habits etc Those commodities are preferred by the consumers which are in fashion So demand for those commodities rises which are in fashion On the other hand if a commodity goes out of the fashion its demand falls because no consumer will like to buy it(5) Number of Buyers in the Market(Population)Increase in population raises the market demand whereas decrease in population reduces the market demand for a commodity Not only the size of population but its composition like age (ratio of males females children and old people in population) also affects the demand for a commodity It is because of needs of children young old male and female population differs(6) Distribution of Income and WealthIf the distribution of income and wealth is more in favour of the rich demand for the commodities preferred by the rich such as comforts and luxuries is likely to be higher On the other hand if the distribution of income and wealth is more in favour of poor demand for commodities preferred by the poor such as necessities will be more(7) Season and Weather ConditionsThis is generally observed that the demand for woolens increases during winter whereas demand for ice creams and cold drinks increases during summer Similarly market demand for umbrellas rain coats increases during rainy seasonQ8 REASONS FOR OPERATION OF LAW OF DEMAND WHY DEMAND CURVE SLOPES DOWNWARDNow we will try to explain why does a consumer purchase more quantity of a commodity at a lower price and less of it at a higher price or why does the law of demand operate ie why does the demand curve slope downwards from left to right The main reasons for operation of law of demand are1 Law of Diminishing Marginal UtilityAs you have studied earlier law of diminishing marginal utility states that as we consume more and more units of a commodity the utility derived from each successive unit goes on decreasing The consumer will be ready to pay more for those units which provide him more utility and less for those which provide him less utility It implies that he will purchase more only when the price of the commodity falls2 Income Effect

When price of a commodity falls purchasing power or real income of the consumer increases which enables him to purchase more quantity of the commodity with the same money income Let us take an example Suppose you buy 4 ice creams when price of each ice cream is ` 25 If price of ice creams falls to ` 20 then with same money income you can buy 5 ice creams now3 Substitution EffectWhen price of a commodity falls it becomes comparatively cheaper as compared to its substitutes (although price of substitutes has not been changed) This will lead to rise in demand for the given commodity For example if coke and Pepsi both are sold at ` 10 each and price of coke falls Now coke has become relatively cheaper and will be substituted for Pepsi It will lead to rise in demand for coke4 Change in Number of BuyersWhen price of a commodity falls some old buyers may demand more of the commodity at the reduced price and some new buyers may also start buying this commodity who were not in a position to buy it earlier due to higher price This will lead to increase in number of buyers when price of the commodity falls As a result demand for the commodity rises when its price falls5 Diverse Uses of a CommoditySome commodities have diverse uses like milk It can be used for drinking for sweet preparation for ice cream preparation etc If price of milk rises its use may be restricted to important purpose only This will lead to reduction in demand for other less important uses When price of milk falls it can be put to other uses also leading to rise n demand for itQ9 EXCEPTIONS TO THE LAW OF DEMANDYou have studied in law of demand that a buyer is willing to buy more quantity of a commodity at a lower price and less of it at a higher price But in certain circumstances a rise in price may lead to rise in demand These circumstances are called Exceptions to the Law of Demand Some important exceptions are1 Giffen GoodsGiffen goods are special type of inferior goods in which negative income effect is stronger than negative substitution effect Giffen goods do not follow law of demand as their demand rises when their price rises Examples of Giffen goods are jowar and bajra etc2 Status Symbol GoodsSome goods are used by rich people as status symbols eg diamonds gold jewellary etc The higher the price the higher will be the demand for these goods When price of such goods falls these goods are no longer looked at as status symbol goods and tehrefore therir demand falls3 NecessitiesCommodities such as medicines salt wheat etc do not follow law of demandbecause we have to purchase them in minimum required quantity whatever their price may be4 Goods Expected to be ScarceWhen the buyers expect a scarcity of a particular good in near future they start buying more and more of that good even if their prices are rising For example during war famines etc people tend to buy more of some goods even at higher prices due to fear of their scarcity in near future

Political Science

Constitution of India-The

Preamble

The preamble-

Preamble-

The preamble is the most precious part of the constitution We the people of India having solemnly resolved to constitute India into a Sovereign Socialist Secular Democratic Republic and to secure to all its citizensA preamble is an introductory and expressionary statement in a document that explains the documents purpose and underlying philosophy When applied to the opening paragraphs of a statute it may recite historical facts pertinent to the subject of the statuteNature and purpose of the constitution-Purpose of the Constitution dictates permanent framework of the government to form a more perfect union to establish justice and ensure peace of thenationconstitution provide principles how the government can run itself following the rules and laws written in the constitution of each state keeps them balanced

Answer the following questions-

1 What is preambleA preamble is an introductory and expressionary statement in a document that explains the documents purpose and underlying philosophy2 What is the nature and

purpose of the constitutionConstitution dictatespermanent framework of the government to form a more perfect union to establish justice and ensure peace of the nation

Homework-Learn

Accounts Compatibilty mode

1MEANING OF PARTNERSHIPPartnership is a form of business organisation where two or more persons join hands to run a business They share the profits and losses according to the agreement amongst them According to the Indian Partnership Act 1932 ldquoPartnership is relation between persons who have agreed to share profits of a business carried on by all or any one of them acting for allrdquo For example one of your friends has passed class XII from National Institute of Open Schooling (NIOS) and wants to start a business Heshe approaches you to join in this venture Heshe wants you to contribute some money and participate in the business activities Both of you if join hands constitute a partnership2CHARACTERISTICS1048698 Agreement A partnership is formed by an agreement The agreement may be either oral or in writing It defines the relationship between the persons who agree to carry on business It may contain the terms of sharing profit and the capital to be invested by each partner etc The written agreement is known as partnership deed1048698 Number of persons There must be at least two persons to form a partnership

The maximum number of partners in a partnership firm can be 50 according toCompanies Act 20131048698 Business The Partnership is formed to carry on business with a purpose of earning profits The business should be lawful Thus if two or more persons agree to carry on unlawful activities it will not be termed as partnership1048698 Sharing Profits The partners agree to share profits in the agreed ratio In caseof loss all the partners have to bear it in the same agreed profit sharing ratio10486981048698Mutual Agency Every partner is an agent of the other partners Every partner can bind the firm and all other partners by hisher acts Each partner will be responsible and liable for the acts of all other partners10486981048698Unlimited liability The liability of each partner except that of a minor is unlimited Their liability extends to their personal assets also If the assets of the firm are insufficient to pay off its debts the partnersrsquo personal property can be used to satisfy the claim of the creditors of the partnership firm10486981048698Management All the partners have a right to mange the business However they may authorize one or more partners to manage the affairs of the business on their behalf10486981048698Transferability of Share No partner can transfer hisher share to any one including hisher family member without the consent of all other partners3PARTNERSHIP DEEDAgreement forms the basis of partnership The written form of the agreement is which a document of partnership is It contains terms and conditions regarding the conduct of the business It also explains relationship between the partners This document is called partnership deed Every firm can frame its own partnership deed in which the rights duties and liabilities of the partners are stated in detail It helps in settling the disputes arising among the partners during the general conduct of business 4CONTENTS OF PARTNERSHIP DEEDThe partnership deed generally contains the following (i) Name and address of the partnership firm(ii) Nature and objectives of the business(iii) Name and address of each partner(iv) Ratio in which profits is to be shared(v) Capital contribution by each partner(vi) Rate of Interest on capital if allowed(vii) Salary or any other remuneration to partners if allowed(viii) Rate of interest on loans and advances by a partner to the firm(ix) Drawings of partners and interest thereon if any(x) Method of valuation of goodwill and revaluation of assets and liabilities on the reconstitution of the partnership ie on the admission retirement or death of a partner(xi) Settlement of disputes by arbitration(xii) Settlement of accounts at the time of retirement or death of a partner5IN ABSENCE OF PARTNERSHIP DEEDThe partnership deed lays down the terms and conditions of partnership in regard to rights duties and obligations of the partners In the absence of partnership deed there may arise a controversy on certain issues like profit sharing ratio interest on

capital interest on drawings interest on loan and salary of the partners In such cases the provisions of the Indian Partnership Act becomes applicableSome of the Issues are(i) Distribution of Profit Partners are entitled to share profits equally(ii) Interest on Capital Interest on capital is not allowed(iii) Interest on Drawings No interest on drawing of the partners is to be charged(iv) Interest on Partnerrsquos Loan A Partner is allowed interest 6 per annum on the amount of loan given to the firm by himher(v) Salary and Commission to Partner A partner is not entitled to anysalary or commission or any other remuneration for managing the business

History TOPIC-TOWARDS INDEPENDENCE AND PARTITION THE LAST PHASE (1935-1947)

SUB TOPIC-IMPORTANT POLITICAL DEVELOPMENTS ndash GROWTH OF SOCIAL IDEAS

Socialism is a political social and economic philosophyLike in other parts of the world the Russian revolution of 1917 served as a great inspiration for revolutionaries in India who at that time were engaged in the struggle for liberation from British ruleSocialist ideas led to the formation of communist party of IndiaJAWAHARLAL NEHRU Among the early Congress leaders Jawaharlal Nehru was very much impressed and influenced by the Socialist ideas He also learnt about the Economic activities of the Soviet Union after the Bolshevic Revolution 1917 He made full use of them in IndiaThe election of Jawaharlal Nehru and Subhas Chandra Bose showed the Left wing tendency within CongressJawaharlal Nehru demanded economic freedom along with political freedom of the people in order to end the exploitation of masses

Nehrus working committee included three socialists leaders The Lucknow session was a landmark in the evolution of socialist ideas of the congressSUBHAS CHANDRA BOSE ndash Subhas Chandra Bose had socialist leaning Both Jawaharlal Nehru and Subhas Chandra Bose were known as leftist Congress men Later on National Congress divided into Leftist and rightist campCONGRESS SOCIALIST Within the Congress some leaders formed the Congress Socialist partyPattavi Sitaramyya Sardar Patel Rajendra Prasad had hostile attitude towards the Congress Socialist partyJawaharlals attitude was hesitant

1 QUESTION ndash Mention name of two Congress leaders who had socialist leaning

1ANSWER ndash Subhas Chandra Bose and Jawaharlal Nehru2QUESTION- In which session of the congress Jawaharlal elaborated his Socialist ideas2 ANSWER ndash Lucknow and Faizpur Session in December 1935 and 19363QUESTION ndash Why Congress was sharply divided into leftist and rightist camp 3ANSWER ndash Subhas Chandra Bosersquos attempt to seek re election for congress presidentship in 1939sharply divided the National Congress into Leftist and Rightist camp4 QUESTION ndash Who was MN Roy 4 ANSWER ndash Manabendra Roy first formed the Communist Party of India outside the country at Tashkent in 19205QUESTION ndash Who formed the Congress Socialist Party within the Congress5 ANSWER ndash Jaya Prakash Narayan Achyut Patwardhan Acharya Narendra Dev Ram Mohan Lohia Aruna Asaf Ali6QUESTION ndash When was the Congress Socialist Party formed What was its object6 ANSWER ndash 1934The Congress Socialist Party sought to work out socialist programme through the Congress They joined hands with the Congress and wanted to carry

Subhas Chandra Bose being expelled from the congress after the Tripuri rift he formed Forward BlockThere were basic differences between the Congress Socialists and the communistsTRADE UNION ACTIVITIES Maximum working class people lived in Bombay and Calcutta The working and living conditions of those workers were very miserable In this situation Shasipada Banerjee NM Lokhande protested against the oppression of the working class peopleThe first Trade Union Madras Labour Union was formed in 1918 by BP WadiaIndustrial strikes took place in Kanpur Calcutta Madras Jamshedpur and Ahmedabad AITUC was formed in Bombay in 1927 The growth of Trade union among the workers was slow because of the fear of the dismissal of the jobIn the mean time the Moderates as well as Communists left AITUC and formed separate organization

on National struggle with the help of workers and peasant class of the society7 QUESTION ndash What was the name of the party founded by Subhas Chandra Bose7 ANSWER- Forward Block8QUESTION ndash Who was Shasipada Banerjee8 ANSWER ndash Shasipada Banerjee was a radical Brahmo He founded a working menrsquos club to protest against exploitation of the British rulers towards the working class of India9 QUESTION ndash What was the weekly published by NM Lokhande9ANSWER- Dinabandhu10 QUESTION ndash Who founded Bombay Mill-Hands Association and in which year10 ANSWER- NM Lokhande in189011 QUESTION- Who was BP WadiaANSWER- BPWadia was the founder of Madras Labour Union in191812 QUESTION- What was the name of the first labour union of India12 ANSWER- Madras Labour Union13 QUESTION Who founded the Majur Mahajan 13 ANSWER GANDHIJI14 QUESTION What was the full form of AITUC When it was formed14 ANSWER All India Trade Union Congressin 192715QUESTION Who formed the Red Trade Union Congress and in which year15ANSWER The Communists formed the Red Trade Union Congress16 QUESTION What do you mean by Socialism16 ANSWER Socialism describes any political and economic theory that says the community rather than individuals should own and manage property and natural resources

Subject Eng Literature (The Tempest ndash William Shakespeare) Topic Act III Scene 3 Lines 1 to 52 (Line 52 ndash Brother my lord the Duke Stand to and do as we) Date 13th April 2020 (4th Period)

[Students should read the original play and also the paraphrase given in the school prescribed textbook]Summary Questions amp Answers

o Alonso Sebastian Antonio Gonzalo Adrian Francisco and others wandered about the island in search of Ferdinand and gets tired and hungry of the toil and at the same time gives up all hope of finding him

o Antonio and Sebastian are happy that Alonso is out of hope and decide to make another attempt on his life that night when being so tired they will be sleeping soundly

o Suddenly a solemn and strange music is heard in the air and several strange shapes enter bringing in a banquet These strange shapes then dance round it with gestures of salutation and then inviting the King to eat they depart

o Seeing this strange scene all are inclined to believe the tales told by travelers that there truly are ldquounicornsrdquo and ldquothe phoenixrsquo thronerdquo

1 ALONSO What harmony is this My good friends hark (L18-27)

GONZALO Marvellous sweet music

[Enter several strange shapes bringing in a banquet

they dance about it with gentle actions of salutation

and inviting the King and his companions to eat they depart]ALONSO Give us kind keepers heavens What were theseSEBASTIAN A living drollery Now I will believe

That there are unicorns that in Arabia

There is one tree the phoenixrsquo throne one phoenix

At this hour reigning thereANTONIO Ill believe both

And what does else want credit come to me

And Ill be sworn rsquotis true Travellers neer did lie

Though fools at home condemn rsquoem

(a) How did Prospero present an amazing spectacle before Alonso and his companions

Using his magic powers Prospero ordered strange shapes to lay a banquet before Alonso and his companions The shapes brought several dishes with tasty eatables in them They placed the dishes on a table before Alonso and his companions Then the strange shapes began to dance gracefully around the banquet While dancing they made gestures inviting them to eat the food Then suddenly the shapes disappeared(b) Who were the guests at the strange banquet Describe the lsquoliving drolleryrsquo

Alonso Sebastian Antonio Gonzalo Adrian and Francisco were the guests at the strange banquet

The term ldquoliving drolleryrdquo refers to live entertainment show In this context when Alonso the King of Naples Sebastian his brother Antonio the treacherous brother of Prospero Gonzalo the kind and loyal councillor to the King Adrian and Francisco came to the island they were hungry and weary in their spirits They heard a solemn and strange music They were shocked to see several strange shapes bringing in a banquet and these shapes danced about it with gentle action of salutation inviting the King and his companions to eat After this Sebastian described this show as lsquoliving drolleryrsquo(c) What is lsquophoenixrsquo What are lsquoUnicornsrdquo

The term lsquophoenixrsquo refers to a mythical Arabian bird which lived alone and perched on a solitary tree After one hundred years it expired in flames and rose again from its own ashes

lsquoUnicornsrsquo refers to the mythological four-footed beasts having horns in the centre of their foreheads When the horns are ground into powder the powder was believed to be

an aphrodisiac(d) How does Sebastian explain the puppet show OR Why does the speaker now believe in unicorns and phoenix

Sebastian finds several strange shapes bringing in the banquet They invite the king and his party for dinner and soon depart He tells that if such a strange sight can be a reality there is nothing incredible in the world and from the present moment he will believe anything He says that it is a strange dumb show enacted not by puppets but by living beings It is stranger than a travellerrsquos tale Seeing such a thing

before his own eyes he will no longer disbelieve the story about unicorns and phoenix(e) How do the other characters present respond to this living drollery

At the sight of the lsquoliving drolleryrsquo like Sebastian Gonzalo and Antonio too acted strangely Antonio told that he too now believes in unicorns and phoenix and anything else that seems to be incredible He too now believes in travellersrsquo tales Gonzalo told that if he would report those happenings in Naples nobody will believe him He considers that those gentle shapes were gentler in manner in comparison to the living beings Alonso was at first sight suspicious and told them that those strange shapes conveyed their meaning in expressive gestures when they seemed to lack speech by their movements and sounds Francisco was amazed at their mysterious disappearance

2 ALONSO Not I

(Line 43-52)GONZALO Faith sir you need not fear When we

were boysWho would believe that there were mountaineers

Dewlapped like bulls whose throats had hanging at rsquoem

Wallets of flesh Or that there were such men

Whose heads stood in their breasts Which now we find

Each putter-out of five for one will bring us

Good warrant ofALONSO I will stand to and feed

Although my lastmdashno matter since I feel

The best is past Brother my lord the Duke

Stand to and do as we

(a) How does Alonso respond at the spectacle of the shapes which were sent to them at the instruction of Prospero

After seeing the strange sight of appearing and disappearing of the shapes sent by Prospero to arrange a banquet for them Alonso says that his surprise at having seen those creatures is infinite and he is fully justified in feeling so much surprise He thinks that their shapes their gestures and the sounds they made were indeed amazing Although they do not possess the gift of speech yet they were able to convey their

thoughts by means of their gestures only

(b) What does Prospero say about the views expressed by Alonso regarding the shapes What does Francisco think about the shapesAfter hearing Alonsorsquos views about the shapes Prospero says that this manrsquos praise of the spirits is rather hasty He means to say that Alonso has shown great haste in reaching the conclusion about the shapes Francisco is amazed to see that those shapes disappeared in a mysterious way(c) What does Sebastian ask Alonso to doSebastian tells Alonso that the shapes having disappeared should not matter to them because they have left the eatables behind He asks Alonso to enjoy eating as they are extremely hungry but the king does not accept his offer of enjoying the dishes(d) How does Gonzalo try to dispel Alonsorsquos fear of those strange shapes What kind of references does he give to AlonsoGonzalo says that those who have travelled abroad have reported seeing even stranger sights than these shapes that Alonso and his companions have beheld Hence there is no reason to feel afraid of these shapes Gonzalo further adds that in his younger days he had heard strange stories from travelers and Alonso might have heard similar stories For instance it was said that there existed a certain race of

human beings who had huge lumps of flesh hanging at their throats and who therefore resembled bulls Then Gonzalo tells about a race of human beings whose heads were located at their breasts Gonzalo says that such stories were not believed by most people in those days but now-a-days these stories have become common(e) Explain the following lsquoEach putter-out of five for onersquoEnglish travellers often insured their trips with London brokers Those that went on foreign travels those days used to deposit a certain amount with some firm or company in London before their departure If the travelers failed to return the money was forfeited by the company with which it had been deposited But this money was repaid five-fold if the travelers returned safe and sound In this way a traveler stood a great chance of recovering the entire cost of his

travels(f) Give the explanatory meanings of the following expressions in the context of the above extract (i) Dewlapped (ii) Wallets of flesh

(iii) Putter-out(i) Dewlapped having big lumps of flesh at the necks(ii) Wallets of flesh large masses of flesh looking like bags(iii) Putter-out to invest money before commencing the travel

  • General methods of preparation of hydrogen
  • Chapter Dimensional Analysis (Summary)
    • Properties of Charge
Page 19:   · Web viewSubject. Topic. Summary. Execution. Hindi. व्याकरण. शरीरके अंगो के नाम लिखिए. 1) आँख 2) नाक 3

Competition

Distribution of income

There exist large competition among buyers and sellers

There is existence of large inequalities of income

There is no such competition

There exist less inequalities of income

Math Topic ndash AlgebraChapter -Factorisation

Study item Factorising by taking out common factorSome solved sums from exercise 41

1) (i) 8xy3 + 12x2y2

= HCF of 8xy3 and 12x2y2 is 4xy2

= 4xy2(2y + 3x )

4) (ii) 28p2q2r ndash 42pq2r2

= HCF of 28p2q2r and 42pq2r2 is 14pq2r = 14pq2r (2p - 3r )5) (ii) 14mn + 22m - 62p=HCF of 14mn 22m and 62p is 2= 2(7mn + 11m - 31p)7) (ii) 3a(x2 + y2) + 6b (x2 + y2) = HCF of 3a(x2 + y2) and 6b(x2 + y2 ) is (x2 + y2)= ( x2+ y2 )(3a + 6b )9) (ii) x(x2 + y2 ndash z2 ) + y(-x2ndashy2 + z2 ) ndash z(x2+ y2 ndash z2 )= x(x2 + y2 -z2) ndash y-(x2 + y2 -z2) -z(x2 + y2 ndash z2)=x(x2 + y2-z2) -y( x2 + y2-z2) ndash z (x2 + y2 -z2)= (x2+ y2 ndash z2)(x ndash y ndash z )

Commercial Studies

Introduction to Accounting and Book-keeping

Today I am going to share you the meaning of Accounting and Book-keeping and its related terms bullAccounting bullBook Keeping bullAccountsbullTypes Of Accounts bullAccounting Cycle

bull Meaning of accounting

Ans )Accounting is the art and science of recording classifying and summarising monetary transactions

bull Meaning of Book-keeping

Ans) Bookkeeping is the art of recording business transactions with the view of having a permanent record of them and showing their effect on wealth

bull Meaning of account

Ans) The term account means a record of business transactions concern a particular person of firm asset or income or expense It is a summarised record of all transactions which take place in an accounting year

bull Types of accountsPersonal accounts ndash Personal accounts relating

to person and Organisation are known

as personal accounts Example Ramrsquos Account ABC amp Co Account etc

Real account - The accounts related to tangible and intangible assets are called real accountsExample Cash Account Furniture Account etc

Nominal account- Accounts related to expenses losses incomes and gains are known as nominal accountsExample Wages Account Salary Account Discount Account etc

bull Accounting cycle Accounting cycle refers to a complete sequence of accounting activities It begins with recording of transactions and ends with the preparation of a balance sheet

English 1 Transformation of sentences

Sentences A sentence is a group of words which makes complete sense

a Assertive sentencesb Imperative sentencesc Interrogative

sentencesd Exclamatory sentences

Sentences can be changed from one grammatical form to another without changing the meaning of the sentence This is known as transformation of sentences

Exercise 6Rewrite the following sentences according to the instructions given below without changing their meanings

1 As soon as he saw the beer he jumped into the river ( Begin No sooner)

2 None but brave deserve the fair (Begin the bravehellip)

3 This box is too heavy for me to lift ( Use so hellip That instead of too)

4 No one other than a king can live like James Luxurious ( Begin only James)

5 Oh for the wings of a dove (Begin I wishhellip)

BENGALI(2ND LANGUAGE)

ldquo বঙগভমির পরমি ldquo াইকেল ধসদন দতত

পব13পোসঠ আসোলিচত ৩ পরবোস দৈদসবর বস ীবতোরো Pলিদ স এ লেদ -আকো সত-োলি লেদ তোস - ক) বকতো লেক কোর লেো লেকো কলিবতোর অং ) কোর পরলিত বকতোর এই উলিকত গ) এ লেদ আকো সত বসত কী বলিঝসয়স4 ীবতোরো বসত কী লেবোঝ ঘ ) আসোচয অংসর তোৎপP13 কী

উ -ক ) বকতো স কলিব মোইসক ম3দ দতত

Types of AccountPersonal AccountReal AccountNominal AccountBalance Sheet (opening)

কলিব মোইসক ম3দ দসততর রলিচত বঙগভলিমর পরলিত কলিবতোর অং ) কলিব বঙগী অ13োৎ লেদমোতোর পরলিত কলিবর এই উলিকত গ ) এ লেদ আকো বসত কলিবর মোব লেদী রপ আকো লেক লেবোঝোসো সয়স4 আকো লেসক লেPম তোরো স পসর লেতমলি ীব লেদ রপ আকো লেসক পরো রপ তোরো স পরসত পোসর এই মভোবোর কোই কলিব বসস4 ঘ ) পরবো Pোতরোয় Pলিদ কলিবর লেদ আকো লেসক ীব তোরো রপ পরো স পসর তোসত কলিব লিবনদমোতর দঃলিত কোর মতয লিবসর সবোভোলিবক পলিরলিত এবং মোষ মরী তোই পরবোস Pলিদ তা োর মতয য় তবও কলিব লিবচলিত সব ো কোর পলিবীসত লেকউ অমর য় লিক4ই অকষয় য় দীর লেPম লিচরপরবোমো লেতমলি মোসষর ীবও চমোতোই ীব - সতবধতোই মতয ীব দীসত মোষ লিতয পরবোমো তবও লেPব মোষ আপ কতকসম13র মো3যসম মোসষর মস লিসসদর সথো কসর লিসত পোসর তোরো লিচরভোসবর সয় মোসষর মস লিবরো কসর তোসদর মস3য লেকউ পGভসত লিবী সয় গোসও মোসষর মস তোরো লিতযপলিত লিতযবলিনদত

Hindi 2ndlang

काकीी(लिसयारामशरणगपत)

इस कहानी म क न यह बतान का परयास निकया ह निक बचच अपनी मा स निकतना परम करत ह शयाम अबोध बाक ह वह अपनी मा क मरन क बाद उसन अपनी मा क लिए बहत रोया बाद म उस पता चा निक उसकी मा राम क घर ची गई ह आकाश म उडती हई पतग दकर उस हष हआ निक पतग क दवारा वह अपनी मा को नीच उतारगा इसक लिए वह अपनी निपता की जब स दो बार सवा रपया निनकाकर पतग और दो मोटी सी मन वाी अपन भाई स काकी एक कागज पर लिवा कर पतग म लिशव का दिदयानिनकाकर पतग और दो मोटी सी मन वाी अपन भाई स काकी एक कागज पर लिवा कर पतग म लिचपका दिदयाभोा और शयाम कोठरी म रससी बाधनी रह थ तभी उसक निपता करोध म आकर उन स पछ निक कया उनकी जब स रपया निनकाा हभोा डर क मार बताया निक शयाम इस पतग क दवारा अपनी काकी को राम क यहा स उतारना चाहता हनिवशशवर(शयाम क निपता)न फटी पतग उठाकर दी तो उस पर काकी लिा थावह हत बजि होकर वही ड रह गएउनहोन सोचा निक मन अपन पतर को मारा जोनिक अनजान और निनदष थावह अपनी मा कोनिकतना पयार करता ह

helliphellipContinue to next

Computer Application

Java Programming Prog 1Write a java program to input two numbers from user and display the sum or product of them as per user choice Use switch case statementSolve public class sum_product public static void main(String args[]) Scanner sc=new Scanner(Systemin) int abc Systemoutprintln(ldquoEnter two numbersrdquo) a=scnextInt() b=scnextInt() Systemoutprintln(ldquoPress 1 for sum or 2 for productrdquo)

c=scnextInt() switch(c) case 1 Systemoutprintln(ldquoThe sum will be =rdquo+(a+b)) break case 2 Systemoutprintln(ldquoThe product will be =rdquo+(ab)) break default Systemoutprintln(ldquoWrong Inputrdquo) Home Work - Practice in your computer using bluej

Subject Eng Literature (The Merchant of Venice ndash William Shakespeare)Topic Act I Scene 2 Lines 92 to 126 (End of scene) Date 13th April 2020 (5th Period)

[Students should read the original play and also the paraphrase given in the school prescribed textbook]Summary Questions amp Answers

o After Portia has expressed her opinion about the suitors Nerissa informs that she need not bother about any one of them as they have decided to quit Belmont at the earliest opportunity because they do not believe in trying their luck by the caskets which is the only way of winning Portia

o Nerissa then enquires of Portiarsquos opinion about Bassanio who once visited her in the company of the Marquis of Montferrat and says that she had never come across such an ideal love deserving the fairest lady for his bride

o Portia seems to remember Bassanio quite correctly and says that she agrees with Nerissa At this moment a servant informs Portia that the Prince of Morocco has arrived to try his luck by the caskets

o Portia tells Nerissa that if she could welcome this new suitor as gladly as she says farewell to the previous ones she would be glad of his arrival However if he happens to have the virtues of a saint but the black complexion of a devil she would prefer to have him for religious consolation rather than as a husband

(1) NERISSA You need not fear lady (Line 97-103)

the having any of these lords they have acquainted me with their determinations

which is indeed to return to their home and to

trouble you with no more suit unless you may be wonby some other sort than your fathers imposition depending on the caskets

PORTIA If I live to be as old as Sibylla I will die as chaste asDiana unless I be obtained by the manner of my fatherswill I am glad this parcel of wooers are so reasonablefor there is not one among them but I dote on his veryabsence and I pray God grant them a fair departure

(a) Elucidate the idea expressed in the first speech of the above dialogue

In the first speech Nerissa assures Portia that she need not have any fear of being compelled to marry anyone of the suitors who had lately come to Belmont She informs her that they have all decided to return to their respective countries(b) Illuminate the meaning of the phrase ldquoyour fatherrsquos imposition depending on the casketsrdquo

Nerissa means that the suitors of Portia do not find the conditions imposed by the will of her father to their liking They are too hard for them These conditions are that in the event of a suitor failing to choose the right casket (i) he should never disclose to anybody which casket he chose (ii) he can never marry and (iii) he should take his departure immediately(c) Explain the meaning of the term lsquoSibyllarsquo

lsquoSibyllarsquo is the name given by Romans and Greeks to a prophetess inspired by some deity usually the sun-god Apollo She had a very long life The god Apollo granted her as many years of life as she could hold grains of sand in her hand(d) Elucidate the meaning of the term lsquoDianarsquo

lsquoDianarsquo is the goddess of hunting She is also regarded as a symbol of virginity because she never fell in love and never

married(e) Explain the meaning of the first two lines of Portiarsquos speech

Portia says that even if she is to live for centuries like Sibylla she would not marry except in accordance to her fatherrsquos will She asserts that she would not mind remaining unmarried and untouched by a man like Diana the virgin the goddess of hunting unless a man is able to win her by passing the test laid down by her father

Class XSubject Topic Summary Execution

Hindi 2nd

Langबड घर की बटी( मशी परमचद)

lsquoबड घर की बीटीrsquo कहानी का उददशय मधयम वग की घर समसया को सझा कर सगदिठत परिरवार म मिम जकर परम स रहन का सदश दना ह घर म शानित tानिपत करन की जिजममदारी नारी की होती ह यदिद नारी समझदार ह उसम धय और परिरवार क परनित परम ह तो कोई भी घटना परिरवार को निवघदिटत नही कर सकती या कहानी परिरवार को सगदिठत करत हए परम सौहाद स एक रदसर की भावनाओ को समझ करउनका सहयोग करत हए जीवन यापन करन की पररणा दती ह मशीपरमचदर जी न इस कहानी म सयकत परिरवार का परनितनिनमिधतव निकया ह यह कहानी बनी माधव सिसह जो गौरी पर क जमीदार क उनक दो पतरो की हशरी कठ ा निबहारीशरीकात का निववाह एकजमीदार घरान की पतरी आनदी स हआ थाआनदी न द को ससरा क वातावरण म ढालिया थाएक दिदन आनदी का अपन दवर ा निबहारी स झगडा हो जाता ह दोनो भाई एक रदसर स अग होन की कोलिशश करत हसभी बह आनदी न अपन मधर वयवहार स ा निबहारी को घर छोडकर जान स रोक लिया| इस पर बनी माधव सिसह न कहा निक बड घर की बटी ऐसी ही होती ह जो निबगडा काम बना ती ह अतः शीषक साथक ह बड घर की बटी आनदी ह

helliphelliphelliphellipContinue to nextBiology Topic ndash Chp-1

CellWelcome to new session 2020-21Today we will start with Chpter 1 cell CELL

Protoplasm+Cellmembrane Or Cell wall

Cytoplasm+Neucleus

Cytoplasmic+ CytoplasmicOrganelles Inclutions(mitochondria (food Golgi bodies pigments)Ribosome)

What is cellbull Cell is the structural and functional unit of living organismbull According to number of cells organisms areUnicellular - Amoeba bacteria Multicellular - Rose Mango Tiger HumanSmallest cell -bacteria Longest cell - Nerve cellLargest cell - Ostrich egg cellCells are of different size and shapes according to their functionsQ2Write chief functions of following cellorganelles

Q3What is tonoplastVacuoles covered by a covering called tonoplast

Bengali(2Nd

Language)

ফ ফটক ো ফটক (কলিবতো ) ভোষ মসোপো3 gtPোয়

একটি লেমসয়র ীবস লেপরম লিকভোসব ফসট ওসঠ তো লেদলিসয়স4 কলিব লেপরম Pই য় লেই ময়ই বনত কোস পলিরত য় ফ লেফোটো বো োসফোটো লেটো ব2 কো য় লেমসয়সদর ব gtয13 লেপরসমর 4লিব ফসট উসঠস4 এই কলিবতোয় লেপরম মোষসক মত gtযর মস লেফস লিদসয় পরকষস বাোচোসোর gtয োত বো2োয় কলিবতোয় লেমসয়টির পসব13র দঃসর কো বো সও লেমসয়টি লেই পসর পলিক সত চোয়ো োরী ীবসর কোস4 পরম লেPৌবস লেপরমসক পোবোর পরব ইচছো োকসও তো পসর লেলিতবোচকতোয় পলিরত য় কলিব ভোষ মসোপো3 যোয় লেP ক লেপরসমর

কলিবতোয় ব gtযবহত লিবসষ লিক4 সvর অ13 লেদওয়ো ১) রসবোো= লেP লিবলিভনন রকম ডোকসত পোসর২) ো= পোর ৩) ঠলি = লেচোসর বZ৪)আই বস2ো=অলিববোলিত৫)শইসয় = োলিয়ত কসর৬)োতপাোচ= লিবলিভনন পরকোর৭)দ2োম = v কসর বZ কসর লেদওয়ো৮)লেরলিং =লেোোর দৈতরী লেব2ো৯) বনত= একঋত১০) পাোর = বসকরো2

Organelles Functions

1 Endoplasmic reticulum

2 Mitochondria

3Golgibodies

4 Ribosome5Lysosome

6Plastids

7 Centrosome

i) Supportive framework for the cellii) Synthesis and transpost of proteinsRelease of energy in the form of ATPi) Synthesis and secretion of enzymes hormoneii) Formation of vacuoles lysosomei) Protein Synthesisi) Intracellular digestionii) Destroy foreign substancei )Leucoplast - stores starchii)chloroplast - trap solar energyiii) Chromoplast - imparts colour toflowers amp fruitsi) Initiates and requlates cell division

কলিবতো তোর অ13সক ভোষোয় পরকোো কসর ঘলিরসয় ব যকত কসরস4 লেপরসমর ফতো আর লিবফতো লেক গোঢ় কসর লেদোসো কলিব ভোষ মসোপো3 যোসয়র অলিভবসর অ যলিদক

Economics

Factors of Production

Welcome to the new sessionToday we are going to start the first chapter of Class XThe name of the chapter is Factors of productionBy the name I hope you all can recall a glimpse of what you have learnt in the second chapter of Class IX

NowProduction is the process of creating the various goods and services which are consumed by the people of the country to satisfy their wants

Thus it is the process in which some materials are transformed from one form to another to create utility and value in goods

For example utility can be created by changing the form of a commodity ie

Making of table out of wood by a carpenter for his customer here the wood is getting transformed into table creating utility for his customer and he can also command a price for it

On the other hand Housewives perform very

useful activities at home which create utility but their domestic activities are not included in production because they have no money value

So we can also say that Production denotes two things firstly creation of utility and secondly creation of value

Production is not complete unless it reaches the consumer

An increase in production will increase the economic welfare of the consumers and hence the aim is to raise the production level of the country

Again production of a good or service is only possible if certain resources or

Questions

1 What do you mean by production

Answer Production means the creation of goods and services for the purpose of selling in the market

In fact production involves the transformation of inputs into outputs

Hence production denotes two thingsCreation of utility and creation of valueUtility and value can be created by changing the form by changing the place by changing the time and by rendering services

Example Transformation of raw

materials into finish goods such as potter creates utility by converting mud into utensils assembling of small parts to make bigger machinery

Production also includes services such as distribution and marketing

2 What are the factors of production

Answer Factors of Production refers to the resources and inputs needed for producing goods and servicesThese inputs can be classified as

Land Labour

Capital Enterprise

Land Land is defined to include not only the surface of the earth but also all other free gifts of nature(for example mineral resources forest resources and indeed anything that helps us to carry out the production of goods and services but is provided by

inputs are used together in right proportion

A resource or an input which helps in the process of production to obtain an output is called FACTOR OF PRODUCTION

These factors of production can broadly be categorized into four parts 1LAND 2LABOUR3CAPITAL4ENTERPRISE (ORGANISATION)or Entrepreneur

The above factors are all interdependent on each other and they play a major role in production process

FACTORS OF PRODUCTION

LANDCAPITAL

LABOUR ENTREPRENEUR

nature free of cost)LabourLabour refers to the human efforts that need to be combined with other factors of production for creating an output

CapitalAll man ndash made means of production is called capita example machineries which help in further production Money when used for starting any business for purchasing raw materials machinery tools etc it is regarded as capitalCapital also includes physical capital like factories machineriestoolsbuildingsequipments etcEnterpriseThe task of bearing risks is called enterprise and the person who bears these risks of business is called the entrepreneurThus an entrepreneur is one who organises production takes important decisions regarding production hires and purchases factors of production and bears the risk and uncertainty involved in productionOrganisation refers to the services of an entrepreneur who controls organises and undertakes all risks One who plans organises and manages a business enterprise is an organiser

Physics Chapter 1 Force

Force is an external agent capable of changing the state of rest or motion of a particular body It has a magnitude and a direction The direction towards which the force is applied is known as the direction of the force and the application of force is the point where force is applied The Force can be measured using a spring balance The SI unit of force is Newton (N)

Question 1

State the condition when on applying a force the body has

(a) the translational motion

(b) The rotational motion

Solutions

(a) Translational motion is produced when the body is free to move

(b) Rotational motion is produced when the body is pivoted at a point

Question 2

Define moment of force and state its SI unit

Solutions

The moment of force is equal to the product of the magnitude of the force and the perpendicular distance of the line of action of force from the axis

of rotation

The SI unit of moment of force is Newton times meter

= Newton meter (Nm)

Commercial Studies

Stake holders In this topic you will be come to know about the meaning and concept of stakeholders

How stakeholders are different from shareholders

Questions1 What do you mean by the term stake holdersAnswer) The term stake holders have developed from the words which mean an interest or expected benefit Stakeholders mean all those individuals groups and Institutions which have a state (interest) in the functioning and performance of a commercial organisation or a business enterprise2 What do you mean by share holdersAnswer) The person and Groups who own the shares of the joint stock company by providing capital to the company are called shareholders Shareholders are the internal stakeholders shareholders are one out of several stake holders3 How are shareholders different from stakeholdersAnswer)i) The term shareholders is related to only joint stock company whereas stakeholders are related with all business organisationsii) Stakeholders maybe any individual having financial stake in business organisation whereas a shareholders are those individuals who are holding shares in the company4) How are shareholders different from creditorsAnswer) i) Shareholders are internal stakeholders while creditors are external stakeholdersii) Shareholders invest in the capital of the company whereas creditors give loan to the companyiii) Shareholders are the members of the company with voting rights but creditors are not the members of the company

English 1 Transformation of sentences

Sentences A sentence is a group of words which makes complete sense

e Assertive sentencesf Imperative sentencesg Interrogative sentencesh Exclamatory sentences

Sentences can be changed from one grammatical form to another without changing the meaning of the sentence This is known as transformation of sentences

Exercise 1 Change the following affirmative sentences into Negative sentences

a He is a good manHe is not a bad man

b Ram loves SitaRam is not without love for Sita

c Only he stood first in the classNone but he stood first in the class

d Ankit was wiser than he

He was not so wise as Ankite He did it

He did not fail to do itf As soon as I reached college the

bell rangNo sooner did I reach college than the bell rang

g He finished everythingHe left nothing unfinished

h It always pours when it rainsIt never rains but it pours

Math Topic Commercial MathematicsChapter ndash Goods and services Tax

What is GSTAns It is a abbreviated term of Goods and Service Text which is an indirect tax levied on the sale of goods and rendering servicesSome terms related to GSTDelar Any person who buys goods or services For resale is known as a delar A delar Can be a firm or a companyIntra-state sales Sales of goods and services within the same state or same union territory are called intra- state salesInter-state sales Sales of goods and services outside the state or union territory are called Inter-state sales4) Input GST GST is paid by dealers on purchase of goods and services are called input GST5) Output GST GST is collected from customers on sale of goods and services are called output GST6) Types of GST There are three taxes applicable under GST(i) Central Goods and Services Tax (CGST)(ii) State Goods and Services Tax (SGST) or Union Territory Goods and Services Tax (UTGST) Both these taxes are levied on intra-state sales Here GST is divided equally among central and state governments(iii) Integrated Goods and Services Tax (IGST) IGST is levied on inter- state sales It is also levied on import of goods and services into India and export of goods and services from India

Subject Eng Literature (The Merchant of Venice ndash William Shakespeare)Topic Act III Scene 4 Lines 1 to 44 (Portia hellip To wish it back on you fare you well Jessica)[Students should read the original play and also the paraphrase given in the school prescribed textbook]

Summary Questions amp AnswersIn this scene we suddenly find a new element in the character of Portia We have already seen her possessed of every graceful womanly quality but now she shows that she is capable of rapid decision and determined action She shows this by her sudden resolve to hasten to Venice with a daring scheme for the rescue of Antonio This is an important scene in the dramatic action for it leads up to and renders possible the striking events of the famous trial scene which is one of the greatest striking elements of the play Moreover the fact that all the characters of importance are now assembled together in Venice makes the union of the main plot and the secondary story complete

(1) LORENZO Madam although I speak it in your presence(Line 1-9)

You have a noble and a true conceit

Of god-like amity which appears most strongly

In bearing thus the absence of your lordBut if you knew to whom you show this honourHow true a gentleman you send reliefHow dear a lover of my lord your husbandI know you would be prouder of the workThan customary bounty can enforce you

(a) Where is Lorenzo Why is he here To whom is he referring as lsquoMadamrsquo

Lorenzo is at Portiarsquos residence He had met Salerio on the way and Salerio had begged him to come along with him to

o In this scene Portia Nerissa Lorenzo Jessica and Balthazar appear

o Portia requests Lorenzo and Jessica to be in charge of her house during her absence from Belmont because she and Nerissa have decided to spend the days in meditation and also in visiting the holy places in the neighbourhood of Belmont She has already instructed her people to acknowledge both Lorenzo and Jessica as master and mistress of house during her absence Lorenzo and Jessica gladly agree to look after the house of Portia

handover the letter from Antonio to Bassanio The letter carried the bad news about Antoniorsquos arrest for non-payment of loan taken from Shylock Hence Salerio might have preferred company to break this bad news to Bassanio He is referring to Portia as Madam(b) What does Portia say on hearing the above extract

Portia says that she has never regretted doing good to others Friends who spend a lot of time together and really are there for each other have many traits in common As Antonio is Bassaniorsquos best friend saving him is like saving Bassanio who is like her own soul She asks Lorenzo to take care of management of the house till Bassanio is back(c) What does Portia send with Bassanio and why

On hearing about Antoniorsquos troubles on account of Bassanio her husband Portia immediately sends him with enough gold to repay the debt many times over to Venice to help Antonio out of his misfortune

(2) Lorenzo Madam with all my heart (Line 36-40)

I shall obey you in all fair commands

Portia My people do already know my mindAnd will acknowledge you and JessicaIn place of Lord Bassanio and myselfSo fare you well till we shall meet again

(a) Where are Lorenzo and Portia at this time What lsquofair commandsrsquo are given to Lorenzo

Lorenzo and Portia are at Belmont during this scenePortia reveals to Lorenzo that she has sworn to contemplate in prayer at a monastery around two miles away until her husband returns from Venice She tells him that Nerissa would accompany her and asks him to manage the house with Jessica till things are settled In response Lorenzo tells her that he would be obliged to do whatever she asks him to do(b) Where is Portia actually going and why

Portia tells Lorenzo that she would live a life of contemplation and pray at a monastery which is two miles away from her place In reality Portia plans to go to Venice in disguise with Nerissa and argue the case in defense of Antonio She is very sure that her plan would succeed

ClassXI (ScienceHumanitiesCommerce)Subject Topic Summary Execution

Computer Science

(APC)

Ch ndash 1 Numbers

(Numbers in different bases and

their Arithmatical operations)

Number System In computers Number System is defined as a writing system to represent the numbers in different ways ie we are using different symbols and notations to represent numbers There are four ways we can represent the number ndash Binary Decimal Octal and Hexadecimal

Decimal Number SystemThis number system consist 10 digits These are 0 1 2 3 4 5 6 7 8 amp 9

Binary Number SystemThis number system has only two digits these are 0 and 1 Here 0 stands for off while 1 stands for on

Octal Number SystemThis number system has 8 digits these are 0 1 2 3 4 5 6 amp 7

Hexadecimal Number SystemThis number system has 16 digits these are 0 1 2 3 4 5 6 7 8 9 A B C D E F Here the value of the alphabets are as follows A=10 B=11 C=12 D=13 E=14 F=15

Rules for conversion decimal number to Binary1 Divide the decimal number by 22 If the number will not divide equally by 2 then round down the answer to the nearest whole number (integer)3 Keep a note of the remainder it should be either 0 or 14 Keep repeating the above steps dividing each answer by 2 until you reach zero5 Write out all the remainders from bottom to top This is your binary solution

For example Lets convert 32 to binary 2 32 2 16 - 0 2 8 - 0 2 4 - 0 2 2 - 0 2 1 - 0 0 - 1

The binary equivalent of 3210 is 1000002

Try the follwing youself1 2410

2 4810

3 1210

History GROWTH OF NATIONALISM

The second half of the 19th century witnessed growth of political consciousness and a sense of Nationalism among the IndiansThere were various factors for growth of Indian Nationalism- As a result various political associations were formed in different provinces by the educated Indians Surendranath Banerjee organized a meeting of National conference at Calcutta Ultimately the National Congress was founded in Bombay in 1885This body became the vanguard of Indian struggle for freedom The congress leaders were known as moderates because they followed a policy of prayer and petition A large number of Indian leaders had experienced in political agitation The Political situation of England was also changed Moreover increasing revolutionary activities in Maharashtra Punjab and Bengal became serious concern to the British Government In this

QUESTION1 What do you mean by Nationalism ANSWER 1 Nationalism is defined as loyalty and devotion to own nation especially a sense of national consciousnessQUESTION 2 What are the causes of nationalism ANSWER 2 There were various factors for growth of nationalism

1 Spread of western education2 The progress of vernacular press and

patriotic literature3 The economic exploitation of our

country by the colonial rulers4 International affairs

QUESTION 3 Who organized National conference in Calcutta in 1883 ANSWER 3 Surendranath BanerjeeQUESTION 4 When did Indian National Congress formANSWER 4 Indian National Congress was formed in 1885 in BombayQUESTION 5 Who were ModeratesANSWER 5 The Early Nationalists were also known as Moderates Their emergence marked

background Lord Curzon became Viceroy in India He had no respect for the Indian National Congress

the beginning of the organized national movement in India They believed in British justice and were loyal to them They followed a policy of prayer and petition They demanded constitutional reforms of our country Impotant Moderate leaders were Pherozshah Mehta Dadabhai Naorozi and Surendranath Banerjee etcQUESTION 6 What do you know about Extremism in Indian National movementANSWER 6 In the beginning of 20th century a new class of national leaders emerged in India which was different from the moderate groups They started more aggressive movement against the British empire The goal of extremists was ldquoswarajrdquo Important extremist leaders were Bal Gangadhar Tilak Lala Lajpat Rai Bipin Chandra Pal etcQUESTION 7 Mention the places which were the main centres of Revolutionary movementANSWER 7 Maharashtra Bengal and Punjab

Physics

Chapter Dimensional Analysis

(Summary)

The dimensions of a physical quantity are the powers to which the fundamental units are raised in order to obtain the derived unit of that quantit

The physical quantites lengthmasstime are represented by [L] [M] [T] resp let they are raised to powers ( dimesions) abc resp then any physical quantity can be represented by [ La Mb Tc ] Examples

1 Area area = L x B = [L] x [L] = [M0 L2 T0 ]

2 Density density = massvolume = [M][L3] = [ M L-3]

3 Velocity velocity = distancetime = [L][T] = [LT-1]HW Try to find out dimension of acceleration Acceleration = velocity timeNB One can find the SI Units Using Dimension Analysis Such as for area we have [L2] so its SI unit is m2

Biology Topic ndash Chp-1 The living world

Today we will start the first chapter the living world Here we discuss about the characteristics of living organism and what are the difference between them and nonliving substances We also discuss about the contribution of different Scientists

There are over 500000 species of plants andover a million species of animal are present on earth Some 15000 new species were discovered every yearQ1 What is a living organismbull A living organism is primarily physico -chemical material that demonstrate a high degree of complexity is capable of selfRegulation possesses a metabolism and perpetuates itself through timeQ2 What are the differences between livingand non-livingsi) Compared with non-living living organisms

have more complex organised structure and their use of energy is more controlled amp efficientii) Living things reproduce their own kind by forming new cells which contains copies of their genesiii) Each organism has some degree of homeostasisie it is able to make adjustments so that internal environment remains constantQ3 Write contributions of following Scientists i) Aristotle - One of the first theories in Biology places all living things in a hiearchieii) AV Leeuwenhoek - was the first to observe living single celled organisms under microscopeii) Carolus Linnaeus - developed the binary system for naming of organisms and classificationiii) Geregor Johann Mendel ndash discoverbasic principles of inheritanceHomework i) C Darwin ii)Schleiden

Math Trigonometric functions

1 Overviewi) Trigonometry The word lsquotrigonometryrsquo is derived from the Greek words lsquotrigonrsquo and lsquometronrsquo which means measuring the sides of a triangle An angle is the amount of rotation of a revolving line with respect to a fixed line Usually we follow two types of conventions for measuring angles ie a) Sexagesimal system b) Circular system In Sexagesimal system the unit of measurement is Degree In Circular system the unit of measurement is Radian ii) Relation between degree and radianThe ratio of circumference of a circle to its diameter is always a constant This constant ratio is a number denoted by π which is taken approximately as 227The relationship between degree amp radian measurements is as follows2 right angles = 180deg= π radians1radian = 180degπ=57deg16(approx) 1deg=π180 radianiii) Length of an arc of a circleIf an arc of length s subtends an angle θ radians at the center of a circle of radius r then s=rθiv) Area of a sector of a circleA sector is like a pizza slice of the

Q) Express the following angles in radiana) 45deg b) 40deg3730Ans a) We have 180deg=π radiansi e 45deg= πtimes45180 radian = π4 radiansb) 40deg3730= 40deg37+3060 minute= 40deg 37 +12 minute= 40deg+ 752 minute=40 + 75(2times60) degree=3258 degreeNow 180deg=π radianie 3258 degree= (πtimes325) (180times8) radians = 65π288 radiansQ) A circle has a radius of r=12 meters What is the length of an arc traced out by a 60deg angle in the center of the circleAns In this problem we know both the central angle (60deg) and the radius of the circle (12) All we have to do is plug those values into our equation and we get

s = 2π(12)(60360)s = 24π6s = 4πSo the length of an arc traced out by a 60deg angle in a circle with a radius of 12 meters equals 4π meters asymp 1257 metersQ) Find the area of the sector with a central angle 30deg and a radius of 9cmAns GivenRadius r = 9 cmAngle θ = 30degArea of the sector = θ360degtimesπr2

= 30360degtimes227times92=2121cm2

circle It consists of a region bounded by two radii and an arc lying between the radiiThe area of a sector is a fraction of the area of the circle This area is proportional to the central angle In other words the bigger the central angle the larger is the area of the sectorArea of Sector = θ2 times r2 (when θ is in radians)

Area of Sector = θ times π360 times r2 (when θ is in degrees)

COMMERCE

CLASSIFICTION OF HUMAN ACTIVITIES-ECONOMIC AND NON-ECONOMIC

Welcome to the new sessiontoday we are going to start the first chapter of Class XI The name of the chapter that we are going to start is

lsquoClassification of Human Activities ndasheconomic and non-economicrsquo

Now let us start the chapter by considering human beings and the activities they perform throughout the day

Human activities means all those activities that human beings undertake to satisfy their wants

Human wants on the other hand are the desire of human beings for goods (vegetables fruits rice etc) and services (services of doctors teachers lawyers etc) that they require to live

Now these human activities continue throughout life as human wants are unending unlimited and recurring as human beings desire for better living throughout their lives

Now human activities can be classified into two categories

Human activities

Economic activities Non-economic activities

Economic activities are

Questions1 What are human activities

Answer Human activities mean all those activities that human beings undertake to satisfy their wants

Example A man working in an office

A boy playing in the garden

2What are the characteristics of human activitiesAnswer the characteristics of human activities are as follows

Human activities are undertaken by men women and children and these activities involve human efforts

Human activities are undertaken to satisfy human wants which are unlimited

Human activities continue throughout life

Human activities are performed for both earning money and personal satisfaction

3What is economic activitiesGive example

Answer Economic activities are undertaken by human beings with the object of earning money acquiring wealth and thereby satisfying human wantsExample

Selling of goods by a shop keeper to his customer

A clinic run by a doctor Service of a teacher in school or college

undertaken by human beings with the object of earning money and acquiring wealth

These activities result in the production of economic goods and services

Example Human activities(ie working in factories officesshops) which produce direct economic benefits

Non-economic activities are inspired by human sentiments and emotions such as love for the family desire to help the poor and love for the country

Thus these human activities (eg praying playing sleeping) produce no direct economic benefits and they are also not related to earning money and acquiring wealth

4 What are the characteristics of economic activities

Answer The characteristics of economic activities are as follows

Economic motiveEconomic activities are undertaken to earn money and acquire wealth

ProductiveEconomic activities involve productiondistribution and exchange of goods and services to create wealth

Economic growthEconomic activities determine the level of economic development of a country and standard of living of its citizens

Socially desirableEconomic activities are socially desirable for society

Economic resourcesEconomic activities make use of all the economic resources such landlabourcapital etc

5 What do you mean by non-economic activitiesExampleAnswerNon-economic activities are inspired by human sentiments and emotions such as love for the family desire to help the poor and love for the countryThese activities are not undertaken for monetary gain but for onersquos satisfaction and happinessExample

a mother looks after her children

a student donates blood8 Differentiate between Economic activities and Non-economic activities

Economic activities

Non-economic activities

1to earn living and acquiring wealth2Result can be measured in terms of money

3ExampleBusinessprofession and employment

1 to obtain some satisfaction

2Result cannot be measured in terms of money

3ExampleFamily-orientedreligious socialCultural and national

BUSINESS STUDIES

BUSINESS ENVIRONMENT

Welcome to the new sessionToday we are going to start the first chapter and the name of the chapter is Business Environment

In todayrsquos world every business enterprise is a part of the society It exists and operates in association with various groups in society such as customers suppliers competitors banks and financial institutions government agencies trade unions media and so on All these groups influence the functioning of business in one way or the other They constitute the environment of businessConcept of Business Environment

The term lsquobusiness environmentrsquo refers to the sum total of all individuals institutions and other forces that lie outside a business enterprise but that may influence its functioning and performance

The main features of business environment

Totality of External forces General and Specific forces Interrelatedness Complexity Dynamic Uncertainty Relativity

The Interrelation between business and its environment

The business enterprise is an open system It continuously interacts with its environment It takes inputs

Prepare the following questions from todayrsquos assignment

1 What do you mean by business environment

The term lsquobusiness environmentrsquo means the aggregate of all forces factors and institutions which are external to and beyond the control of an individual business enterprise but they may influence its functioning and performance Business environment is the macro framework within which a business firm a micro unit operates It consists of several interrelated and interacting elements

2 Explain the main features of business environment in brief

Totality of External forces-Business environment is the sum total of all things external to a business environment

General and Specific forces-It includes both the forces general forces are the economic social political legal and technological conditions which indirectly influence all business enterprise Specific forces are the investors customers competitors and suppliers which influence individual enterprise directly

Interrelatedness-Different elements of environment are interrelated for an example growing awareness for health care has increased the demand for health foods

Complexity- Business environment id

(such as raw materials capital labour energy and so on) from its environment transforms them into goods and services and sends them back to the environment

Fig 1 Business Environment Relationship

complex in nature as the elements keep on changing example economic technological and other forces changes in demand for a product and service

Dynamic-Business environment is not static it keeps on changing

Uncertainty- Itrsquos very difficult to predict future events such as technology and fashion which occur fast and frequently

Economics Basic Economic ConceptsSub topic

Microeconomics and

Macroeconomics

Welcome to the new sessiontoday we are going to start the first chapter of Class XI The name of the chapter that we are going to start is Basic Economic concepts

Now Economics covers the study of human activities Human activities are those activities which are performed by humans to satisfy their wants

Thus Human wants are unlimited and therefore economic activities such as production exchange and consumption are needed in order to satisfy those wants

The study of economics is divided largely in two parts which areMicroeconomics and Macroeconomics

SUBJECT- MATTER OF ECONOMICS

MICROECONOMICS MACROECONOMICS

Questions1Who has coined the words micro and macro economics

Answer Ranger Frisch coined the words lsquomicrorsquo and lsquomacrorsquo in 1933 to denote the two branches of economic theory namely microeconomics and macroeconomics

2What is microeconomicsAnswer It is the study of behaviour of individual decision ndash making unit such as consumers firms etc

3 What is macroeconomicsAnswer Macroeonomics is the study of overall economic phenomena like employment national income etc

4 What is the importance of microeconomicsAnswer

Microeconomics helps in formulating economic policies which enhance productive efficiency and results in greater social welfare

It helps the government in formulating correct price policies

It explains the working of a capitalistic economy where individual units(producers and consumers ) are free to take their own decision

Micro means a small part in

microeconomics we do not study the whole economy Hence we study an individual consumer and his or her choices and a producer and his or her profit maximizing decisions in the market Thus it does not mirror what happens in the economy as a whole

Macroeconomics on the other hand studies the economy as a whole It is concerned with aggregate and depicts the entire picture of the economyMacroeconomics deals with the national income aggregate investment aggregate consumption etc

Features of Microeconomics It deals with small

parts of the country Hence it looks at

individual consumers firms and industries

It deals with individual income consumption and savings

It studies the determination of price of any product or factors of production

It deals with the working of market via the price mechanism which is nothing but the determination of price and quantity of a commodity by the forces of demand and supply

Features of Macroeconomics

It deals with the study of the economy as a whole

It is concerned with

5 Give a limitation of microeconomics Microeconomics fails to explain the

functioning of an economy as a whole It cannot explain unemployment illiteracy and other problems prevailing in the country

6 What is the importance of macroeconomics It gives overall view of the growing

complexities of an economic system It provides the basic and logical

framework for formulating appropriate macroeconomic policies (eg for inflation poverty etc )to direct and regulate economy towards desirable goals

7What is the limitation of macroeconomics It ignores structural changes in an

individual unit of the aggregate

8 Differentiate between Microeconomics and Macroeconomics

Microeconomics Macroeconomics

the study of aggregates

National income aggregate savings and aggregate investments are major concepts dealt within macroeconomics style

It studies the determination of general price levels

It investigates into the problem of unemployment and the achievement of employment

It studies the aspect of decision making at the aggregate and national levels

It includes all growth theories whether related to developed or developing economies it also includes the study of economic systems and the working of the economy under different systems

Note Both Micro and macro economics are complementary and should be fully utilized for proper understanding of an economy

1It studies economic aspect of an individual unit2It deals with individual incomeConsumption and savings

3 It facilitates determination of price of any product or factors of production

4 Itrsquos scope is narrow and restricted to individual unit

1It studies the economy as a whole

2It deals with the national income aggregate consumption and aggregate savings3 It facilitates determination of general price level in an economy

4 Itrsquos scope is wide as it deals with economic units on the national level

ACCOUNTS

Introduction to Accounting and Book-keeping

Today I am going to share you the meaning of Accounting and Book-keeping and its related terms bullAccounting bullBook Keeping bullAccountsbullTypes Of Accounts bullAccounting Cycle

bull Meaning of accounting

Ans ) Accounting is the art and science of recording classifying and summarising monetary transactions

bull Meaning of Book-keeping

Ans) Bookkeeping is the art of recording business transactions with the view of having a permanent record of them and showing their effect on wealth

bull Meaning of account

Ans) The term account means a record of

business transactions concern a particular person of firm asset or income or expense It is a summarised record of all transactions which take place in an accounting year

bull Types of accountsPersonal accounts ndash Personal accounts relating

to person and Organisation are known as personal accounts Example Ramrsquos Account ABC amp Co Account etc

Real account - The accounts related to tangible and intangible assets are called real accounts Example Cash Account Furniture Account etc

Nominal account- Accounts related to expenses losses incomes and gains are known as nominal accounts Example Wages Account Salary Account Discount Account etc

bull Accounting cycle Accounting cycle refers to a complete sequence of accounting activities It begins with recording of transactions and ends with the preparation of a balance sheet

Chemistry TopicAtomic Structure

Thomsonrsquos atomic modelThomson (1898) was the first to propose the model of an atomHe proposed that an atom can be regarded as a uniform sphere of positive electricity in which requisite number of electrons are embedded evently to neutralize the positive chargeThis is just like plums embedded in a pudding or seeds evently distributed in red spongy mass of a watermelonThis model of atom is known as ldquoPlum-Pudding modelrdquo or

Q1)What is the fundamental constituents of atomAns Electron Proton and neutrons are the fundamental constituents of atomQ2)What is the value of fundamental unit of electricityAnsThe charge carried by one electron is sad to be the fundamental unit of electricityIts magnitude is 48times10-10esuOr 1602times10-19C Q3)Name the element containing no neutronAnsOrdinary hydrogen atom or protium 1H1

Types of AccountPersonal AccountReal AccountNominal AccountBalance Sheet (opening)

ldquowatermelon modelrdquoThis model could explain the electrical neutrality of an atom but failed to explain the result of scattering experiment carried out by Rutherford in 1911So it was rejected ultimately

Q4)Why is an electron called universal particleAns Itrsquos mass and Charge are independent of its source

EVS Chapter 1 ndash Modes of Existence

Modes of existence When one speaks normally about the mode of existence of some group or individual one refers to their customs their mode of being their ethology their habitat in some way their feeling for a placeDifferent modes of exixtence are ndash

1 Hunting ndashGathering2 Pastoral3 Agricultural4 Industrial

1 Hunting and gathering Hunting and gathering mode of existence is characterized by obtaining food from hunting wild animals including fishing and gathering wild plants From their earliest days the hunter-gatherer diet included various grasses tubers fruits seeds and nuts Lacking the means to kill larger animals they procured meat from smaller game or through scavenging

Societies that rely primarily or exclusively on hunting wild animals fishing and gathering wild fruits berries nuts and vegetables to support their diet are called hunting and gathering societies

At least this used to be practice of human beings before agriculture is invented As their brains evolved hominids developed more intricate knowledge of edible plant life and growth cycles

Q) Write the features of Hunting ndash gathering societiesAns - There are five basic characteristics of hunting and gathering societies

i The primary institution is the family which decides how food is to be shared and how children are to be socialized and which provides for the protection of its members

ii They tend to be small with fewer than fifty members

iii They tend to be nomadic moving to new areas when the current food supply in a given area has been exhausted

iv Members display a high level of interdependence

v Labor division is based on sex men hunt and women gather

Political Science

Introduction to political science

Political science occasionally called politology is a social science which deals with systems of governance and the analysis of political activities political thoughts associated constitutions and political behaviorThe study of political science involves the study of both the

Answer the following questions-1 What is political science

Political science occasionally called politology is a social science which deals with systems of governance and the analysis of political activities political thoughts associated constitutions and political behavior

2 Short notes-

traditional and modern theories of politicsTraditionalClassical political sciencepolitical theory-Traditional political science is the study of politics before Second World War The methodology to study Politics was traditional (legal formaletc) the definition of politics traditional (Politics begins and end with state)area of study (constitution state machinery)was traditionalModern Political scienceModern political theory-Modern Political Theory critically examines the contemporary state of political theory making an assessment of the achievement and limitations of the Behavioural Revolution in its totality and reviews objectively the major paradigms and conceptual frameworks adopted by the disciplineContemporary attempts at the development of an integrated political theory involving the use of both traditional and modern concepts approaches and theories-Around late 1960s several political scientists realized the importance of both the traditional political theory and modern Political theory They began building an integrated theory of politics involving a systematic mixture of traditional and modern studies of politics It was held that the study of a complex and vast field like politics needs both traditional as well as

Classical political theory Modern Political theory

Homework-Learn

modern concepts and approaches for studying itrsquos all aspects

Subject Eng Literature (The Tempest ndash William Shakespeare) Topic Act I Scene 1 Lines 1 to 32 (Line 32 ndash Gonzalo hellip If he be not born to be hanged our case is miserable) Date 13th April 2020 (3rd Period)

[Students should read the original play and also the paraphrase given in the school prescribed textbook]Summary Questions amp Answers

[SUMMARY OF THE ENTIRE SCENE]

o The play starts with the scene of a severe storm at sea Alonso (King of Naples) Sebastian (Alonsorsquos brother) Ferdinand (Alonsorsquos son) Gonzalo Antonio (the usurping Duke of Milan) are in a ship in the midst of the storm

o The mariners are trying their best to control the vessel from running aground and are totally following the orders of their Master the Boatswain They have scant success

o The mariners become extremely unhappy and annoyed when most of the passengers arrive on the deck thereby hampering their effort to save the ship There is serious confrontation between them and the passengers who are part of the Kingrsquos entourage

o The mariners could not save the ship

SUMMING-UP

(i) Vivid description of the scene which gives a realistic description of terror and confusion of a tropical storm

(ii) Shows Shakespearersquos accuracy of knowledge in describing the naval operations and also matters of seamanship

(iii) The opening scene justifies the title ndash The Tempest

UNANSWERED QUESTIONS

(i) The King always travels with his entire fleet including his soldiers Where

(1) GONZALO Nay good be patient (Line 15-26)BOATSWAIN When the sea is Hence What cares these

roarers for the name of the king To cabin silence Trouble us not

GONZALO Good yet remember whom thou has aboardBOATSWAIN None that I more love than myself You are a

councillor if you can command these elements to silence and work

the peace of the present we will not hand a rope more use your authority If you cannot give thanks you have

lived so long and make yourself ready in your cabin for the mischance of the hour if it so hap [To the Mariners]

Cheerly good hearts [To Gonzalo] Out of our way I say

(a) To whom is the boatswain speaking What does he mean by lsquoNone that I more love than myselfrsquo

The Boatswain is speaking to Gonzalo the honest old councilor of the Duke of MilanBy using the words ndash lsquoNone that I love more than I love myselfrsquo means that for the Boatswain nobody is dearer to him than his own life

(b) What were the conditions that made the boatswain react in this way

The Boatswain reacts in this way because the storm is at sea and Alonso King of Naples Sebastian his brother Ferdinand his son Gonzalo Antonio the usurping Duke of Milan on board are in distress and in panic Thus they have rushed to the deck interrupting the work of the mariners

(c) What hope does Gonzalo take from the attitude of the boatswain

The insolent and authoritative attitude of Boatswain makes Gonzalo feel comforted He tells that there are no signs that the Boatswain will be drowned But his facial appearance and attitude shows that he is destined to die on land by hanging which in effect means that all on board will be saved Otherwise all the persons on board are doomed

(d) How can they lsquomake yourself ready in your cabinrsquo For what were they asked to make ready themselves

In order to make themselves ready in their cabin the

were the other ships

(ii) Why was the ship in that area Where was it coming from or going where

(iii) The ship broke apart What happened to those who were in the ship

passengers on board must prepare for death which they will possibly soon have to meetThey can retire to their cabins and offer prayers to the Almighty to save them from drowning

(e) What does the boatswain say when he is asked to be patient What does he order to the royal party

When the boatswain is asked to be patient and remain calm he says that he will be patient only when the storm will be over and the sea will be calm but as long as the storm blows and there is danger to the ship he cannot think of being patient He orders the royal party to go to the cabin and leave the mariners to their work

(2) GONZALO I have great comfort from this fellow (Line 27-36)

Methinks he hath no drowning mark upon him his complexion is perfect

gallows Stand fast good Fate to his hanging Make the rope of his destiny our cable for our own doth little advantage If he be not born to be hanged our case is miserable

(a) Why does Gonzalo regard the Boatswain in the midst of danger

In the midst of danger Gonzalo regards the boatswain because he feels that the Boatswain is a source of comfort and is bent upon to do his work sincerely which in this case is saving the ship and its passengers from the severest of raging storm

(b) What reasons does Gonzalo give when he says that none in the ship will die of drowning

Gonzalo is almost sure that none in the ship will die by drowning His says that there is no mark on the face of the boatswain that indicates that he will die by drowning On the other hand the lines on his face are strong indications that he will be hanged to death Therefore there shall be no danger of the shiprsquos sinking

(c) Explain the following ldquoStand fast good Fate to his hanging Make the rope of his destiny our cable for our own doth little advantage If he be not born to be hanged our case is miserablerdquo

The stated lines mean that if the will of destiny is to be carried out then the ship will not get wrecked and all the passengers will be saved The safety of the passengers therefore depends upon the will of fate being carried out in the case of the boatswain If however the boatswain is not to die by hanging then the passengers are also very unsafe because in that case the ship is likely to sink

(d) What order does the Boatswain give to the sailors

when he re-enters What does he say about the crying of the fellows inside the cabin

The boatswain orders the sailors to bring the topmast lower and bring the ship close to a stationary position with the help of the main sail He says that the fellows inside the cabin are moaning and crying in their distress louder than his voice and louder even than the roaring of the storm

Class XII (ScienceCommerceHumanities) Subject Topic Summary Execution

Computer Science

PropositionalLogic

Propositional logic is a procedure to provide reasoning through statementProposition A ststement that results in True or False is said to be proposition There are two types of propositionSimple proposition amp compound propositionSimple proposioton A simple proposition is one that is not a part of any other proposition Such sentential form of proposition is symbolized with english letters in short For example Ram is a claver student (TrueFalse)Where do you live (Not in True or False)Grapes are sweet (TrueFalse)It rains today (TrueFalse)Here we can see some statements anwer would be true or false but some staements answer can not give in terms of true or false Thus the sentences which can be answered in true or false are known as simple propositionAssigning propositon to a variableThe general syntax to assign propostion to a variable is as followsVariable = Simple propositonFor example A=Ram is a clever studentB= Grapes are sweetC= it rains todayCompound proposition

helliphellipto be continued in next classhelliphellipMath Relation Relation If A and B are two non-empty sets

then a relation R from A to B is a subset of AxB If R A x B and (a b) R then we say that a sube isinis related to b by the relation R written as aRbeg Let A be the set of students of class XII and B be the set of students of class XI Then some of the examples of relation from A to B arei) (a b) AXB a is brother of bisinii) (a b) AXB age of a is more than age of isinb Types of relation In this section we would like to study different types of relations We know that a relation in a set A is a subset of A times A Thus the empty set φ and A times A are two extreme relations For illustration consider a relation R in the set A = 1 2 3 4 given by R = (a b) a ndash b = 10 This is the empty set as no pair (a b) satisfies the condition a ndash b = 10 Similarly R = (a b) | a ndash b | ge 0 is the whole primeset A times A as all pairs (a b) in A times A satisfy | a ndash

Example 1 Let A be the set of all students of a boys school Show that the relation R in A given by R = (a b) a is sister of b is the empty relation and R = (a b) the primedifference between heights of a and b is less than 3 meters is the universal relationSolution Since the school is boys school no student of the school can be sister of any student of the school Hence R = φ showing that R is the empty relation It is also obvious that the difference between heights of any two students of the school has to be less than 3 meters This shows that R = A times A is primethe universal relation Example 2 Show that the relation R in the set 1 2 3 given by R = (1 1) (2 2) (3 3) (1 2) (2 3) is reflexive

b | ge 0 These two extreme examples lead us to the following definitionsDefinition 1 A relation R in a set A is called empty relation if no element of A isrelated to any element of A ie R = φ A times AsubDefinition 2 A relation R in a set A is called universal relation if each element of A is related to every element of A ie R = A times A Both the empty relation and the universal relation are some times called trivial relation Definition 3 A relation R in a set A is called(i) reflexive if (a a) R for every a Aisin isin(ii) symmetric if (a1 a2) R implies that (aisin 2a1)

R for all aisin 1 a2 Aisin(iii) transitive if (a1 a2) R and (aisin 2 a3) R isinimplies that (a1 a3) R for all aisin 1 a2 a3 AisinDefinition 4 A relation R in a set A is said to be an equivalence relation if R is reflexive symmetric and transitive

but neither symmetric nor transitiveSolution R is reflexive since (1 1) (2 2) and (3 3) lie in R Also R is not symmetric as (1 2) R but (2 1) isin notinR Similarly R is not transitive as (1 2) R and (2 3) R but (1 3) R isin isin notinExample 3 Show that the relation R in the set Z of integers given byR = (a b) 2 divides a ndash b is an equivalence relationSolution R is reflexive as 2 divides (a ndash a) for all a Z isinFurther if (a b) R then 2 divides a isinndash b Therefore 2 divides b ndash a Hence (b a) R which shows that R is isinsymmetric Similarly if (a b) R and (b c) R isin isinthen a ndash b and b ndash c are divisible by 2 Now a ndash c = (a ndash b) + (b ndash c) is even (Why) So (a ndash c) is divisible by 2 This shows that R is transitive Thus R is an equivalence relation in ZExample 4 Let L be the set of all lines in a plane and R be the relation in L defined as R = (L1 L2) L1 is perpendicular to L2 Show that R is symmetric but neither reflexive nor transitiveSolution R is not reflexive as a line L1 can not be perpendicular to itself ie (L1 L1) R notinR is symmetric as (L1 L2) Risin

L1 is perpendicular to L2rArr L2 is perpendicular to L1rArr (L2 L1) RrArr isin

R is not transitive Indeed if L1 is perpendicular to L2 and L2 is perpendicular to L3 then L1 can never be perpendicular to L3 In fact L1 is parallel to L3 ie (L1 L2) R isin(L2 L3) R but (L1 L3) Risin notin

Chemistry Solid state Characteristics if Solids(i)The particles are locked in fixed positions they are unable to change their relative positions and this brings a definite shape and volume of a solid(ii)In a solid the constituent particles are held by strong forces of attractionThe forces of attraction may be bonding or non bonding(iii)The constituent particles in a solid pack together as closely as possibleoccupying most of the available space within the solidThus the empty space in a solid is very smallThis makes a solid highly rigid and nearly incompressibleThis also explains why a solid has high density and exhibits slow diffusionClassification of Solids

Q1)Define Crystalline solids AnsA Solid that has a definite geometrical shape and a sharp melting pointand whose constituent particles (atomsmolecules or ions) are arranged in a long range order of definite pattern extending throughout the solid is called a crystalline solidExNaClQ2)Define Amorphous solids AnsA solid that does not have a definite shape and a sharp melting pointand whose constituent particles (atomsmolecules or ions) are not arranged in a definite pattern is called an amorphoussolid

Crystalline solidsAmorphous solids

ExGlassRubberQ3)Classify Crystalline Solids Crystalline Solids

Physics Coloumbrsquos Law (Summary)

Before Going Into Coloumbrsquos Law We Will First Learn What is Charge Properties of Charge and Always remember that charge is quantized ie a body always have static charge of magnitude equal to some integral multiple of fundamental electronic charge e= 16 x 10- 19 C

Charge is the property of matter that causes it to produce and experience electrical and magnetic effects The study of the electrical charges at rest is called electrostatics When both electrical and magnetic effects are present the interaction between charges is referred to as electromagnetic

There exist two types of charges in nature positive and negative Like charges repel and unlike charges attract each other

The type of charge on an electron is negative The charge of a proton is the same as that of an electron but with a positive sign In an atom the number of electrons and the number of protons are equal The atom is therefore electrically neutral If one or more electrons are added to it it becomes negatively charged and is designated as negative ion However if one or more electrons are removed from an atom it becomes positively charged and is called a positive ion

The excess or deficiency of electrons in a body gives the concept of charge If there is an excess of electrons in a body it is negatively charged And if there is deficiency of electrons the body becomes positively charged Whenever addition or removal of electrons takes places the body acquires a charge

The SI Unit of charge is coulomb (C) In SI units the current is a fundamental quantity having a unit of ampere (A) The unit of charge is defined in terms of the unit of current Thus one coulomb is the charge transferred in one second across the section of a wire carrying a

Ionic SolidsMetallicSolids

Molecular Solids

current of one ampere

As q = It we have1 C = (1 A) (1 s)

The dimensions of charge are [A T]

Properties of Charge

(1) Quantization of Charge Electric charge can have only discrete values rather than any value That is charge is quantized The smallest discrete value of charge that can exist in nature is the charge on an electron given as

e = plusmn 16 x 10- 19 C

This is the charge attained by an electron and a protonA charge q must be an integral multiple of this basic unit That is

Q = plusmn ne where n = 1 2 hellip

Charge on a body can never be (frac12)e (23)e or 57e etcWhen we rub a glass rod with silk some electrons are transferred from the rod to the silk The rod becomes positively charged The silk becomes negatively charged The coulomb is a very large amount of charge A typical charge acquired by a rubbed body is 10 - 8 C

Biology Reproduction in organisms

Welcome to this new session 2020-21Today in this first chapter we mainly discuss about reproduction types needs and life span of some organismsWe also discuss about difference between sexual and asexual reproduction

Q1 What is reproductionReproduction is defined as a biological processin which an organism gives rise to young onessimilar to itselfQ2 What are the needs of reproductionbulli) Reproduction maintain life on earthii) It enables the continuity of the species generation after generationiii) It creates genetic variation among populationsQ3 Define Life span and write some orgnisms life spanbull Life span is the period from birth to

the natural death of an organism- OrganismsLife span1 Butterfly 1 - 2 weeks2 Fruit fly 30 days3Dog 10-13 years4 Rose5-7 years5 Tortoise100-150 years6 Banyan Tree -200 - 250 yearsQ4 Reproduction is of two types in case ofanimals but in case of plants vegetative propagation is also present

Asexual Reproduction Sexual Reproductioni) Always uniparentalii) Gametes are not involvediii) Only mitotic division involvediv) Somatic cells of parents are involvedv) Offsprings are genetically similar to the parents

i) Usually biparentalii) Gametes are involvediii) Meiosis occurs during gametogenesis Mitosis occurs after fertilisationiv) Germ cells of the parents are involvedv) offsprings are genetically different from the parents

COMMERCE BUSINESS ENVIRONMENT

Welcome to the new sessiontoday we are going to start the first chapter of Class XII The name of the chapter is Business Environment

Already many of you have got some idea about the word business environment form the first chapter of business studies in class XI

In todayrsquos world every business enterprise is a part of the society It exists and operates in association with various groups in society such as customers suppliers competitors banks and financial institutions government agencies trade unions media and so on All these groups influence the functioning of business in one way or the other They constitute the environment of businessConcept of Business Environment

The term lsquobusiness environmentrsquo refers to the sum total of all individuals institutions and other forces that lie outside a business enterprise but that may influence its functioning and performance

The main features of business environment Totality of External forces General and Specific forces Interrelatedness Complexity Dynamic Uncertainty

Prepare the following questions from todayrsquos assignment

2 What do you mean by business environment

The term lsquobusiness environmentrsquo means the aggregate of all forces factors and institutions which are external to and beyond the control of an individual business enterprise but they may influence its functioning and performance Business environment is the macro framework within which a business firm a micro unit operates It consists of several interrelated and interacting elements

2 Explain the main features of business environment in brief

Totality of External forces-Business environment is the sum total of all things external to a business environment

General and Specific forces-It

Relativity

The Interrelation between business and its environment

The business enterprise is an open system It continuously interacts with its environment It takes inputs (such as raw materials capital labour energy and so on) from its environment transforms them into goods and services and sends them back to the environment

Fig 1 Business Environment Relationship

includes both the forces general forces are the economic social political legal and technological conditions which indirectly influence all business enterprise Specific forces are the investors customers competitors and suppliers which influence individual enterprise directly

Interrelatedness-Different elements of environment are interrelated for an example growing awareness for health care has increased the demand for health foods

Complexity- Business environment id complex in nature as the elements keep on changing example economic technological and other forces changes in demand for a product and service

Dynamic-Business environment is not static it keeps on changing

Uncertainty- Itrsquos very difficult to predict future events such as technology and fashion which occur fast and frequently

Business Studies

Human Resources Management

Human resource of an organisation are the aggregate of knowledge skills attitudes of people working in it

The management system which deals with human resources is called human resource management

Features of HRMbullComprehensive functionbullPeople-oriented

Question1) What do you mean by human

resource management Answer) Human resource management may be defined as that field of Management which has to do with planning organising and controlling the functions of procuring developing maintaining and utilising the labour force

bullAction oriented bullPervasive function bullContinuous function

2) Explain the features of HRM in brief

Answer)bullHuman Resource Management is concerned with managing people at work bull Human Resource Management is concerned with employees which bring people and organisations together so that the goals of each are met bullHuman resource management considered every employees as an individual and also promote their satisfaction and growth bull Human resource management is inherent in all organisations and at all levelsbullManagement of human resources are ongoing on never ending process which requires a constant alertness and Awareness of human relations

3) ldquoHR function is said to be pervasiverdquowhy

Answer) Human resource management is required in all organisations whether it is private or government organisations armed forces sports organisations etc It permeatsall the functional areas like production marketing finance research etc This from this feature of human resource management it can be said that it is pervasive in nature

Economics Demand Q1DEFINITION OF DEMANDIn economics demand is the quantity of a good that consumers are willing and able to purchase at various prices during a given period of timeQ2DEMAND CURVEIn economics a demand curve is a graph depicting the relationship between the price of a certain commodity and the quantity of that commodity that is demanded at that pricQ3LAW OF DEMANDIn microeconomics the law of demand states that conditional on all else being equal as the price of a good increases quantity demanded decreases conversely as the price of a good decreases quantity demanded increasesQ4ASSUMPTION of LAW OF DEMAND(i)No change in price of related commodities(ii) No change in income of the consumer(iii) No change in taste and preferences customs habit and fashion of the consumer( No expectation regarding future change in priceQ5MARKET DEMAND SCHEDULEIn economics a market demand schedule is a tabulation of the quantity of a good that all consumers in a market will purchase at a

given price At any given price the corresponding value on the demand schedule is the sum of all consumersrsquo quantities demanded at that priceQ6INDIVIDUAL DEMAND SCHEDULEIndividual demand schedule refers to a tabular statement showing various quantities of a commodity that a consumer is willing to buy at various levels of price during a given period of timeQ7 FACTORS AFFECTING INDIVIDUAL DEMAND FOR A COMMODITY

The factors that influence a consumerrsquos decision to purchase a commodity are also known as determinants of demand The following factors affect the individual demand for a commodity1 price of the commodity2 price of related goods3 income of buyer of the commodity4 tastes and preferences of the buyer1 Price of the CommodityYou must have observed that when price of a commodity falls you tend to buy more of it and when its price rises you tend to buy less of it when all other factors remain constant (lsquoother things remaining the samersquo) In other words other things remaining the same there is an inverse relationship between the price of a commodity and its quantity demanded by its buyers This statement is in accordance with law of demand which you will study in the later part of this lesson Price of a commodity and its quantity demanded by its buyers are inversely related only when lsquoother things remain the samersquo So lsquoother things remaining the samersquo is an assumption when we study the effect of changes in the price of a commodity on its quantity demanded2 Price of Related goodsA consumer may demand a particular good But while buying that good heshe also asks the price of its related goods Related goods can be of two types-(i) Substitute goods(ii) Complementary goods While purchasing a good prices of its substitutes and complements do affect its quantity purchased(i) Price of Substitute Goods Substitute goods are those goods which can easily be used in place of one another for satisfaction of a particular want like tea and coffee An increase in price of substitute good leads to an increase in demand for the given commodity and a decrease in price of substitute good leads to a decrease in demand for the given commodity It means demand for a given commodity is directly affected by change in price of substitute goods For example if price of coffee increases the demand for tea will rise as tea will become relatively cheaper in comparison to coffee(ii) Price of Complementary goods Complementary goods are those goods which are used together to satisfy a particular want like car and petrol An increase in the price of complementary goods leads to a decrease in demand for the given commodity and a decrease in the price of complementary goods leads to an increase in demand for the given commodity For example if price of petrol falls then the demand for cars will increase as it will be relatively cheaper to use both the goods together So demand for a given commodity is inversely affected by change in price of complementary goods3 Income of the Buyer of CommodityDemand for a commodity is also affected by income of its buyer However the effect of change in income on demand depends on the nature of the commodity under consideration In case of some goods like full cream milk fine quality of rice (Basmati rice) etc demand for these commodities increases when income of the buyer increases and

demand for these commodities decreases when income of the buyer decreases Such goods whose demand increases with the increase in income of the buyer are called normal goods But there are some goods like coarse rice toned milk etc whose demand decreases when income of buyer increases and their demand increases when income of the buyer decreases Such goods whose demand decreases with the increase in income of the buyer are called inferior goods Suppose a consumer buys 10 Kgs of rice whose price is ` 25 per Kg He cannot afford to buy better quality of rice because the price of such rice is ` 50 per Kg The consumer is spending ` 250 per month on the purchase of rice Now if income of the consumer increases and he can afford ` 350 on purchase of 10 Kg of rice Now he can afford to buy some quantity of rice say 6 Kgs whose price is ` 25 per Kg and may buy 4 Kgs of rice whose price is ` 50 per Kg Thus he will buy 10 Kgs of rice by spending ` 350 per month Therefore we may conclude that demand for normal goods is directly related to the income of the buyer but demand for inferior goods is inversely related to the income of the buyer4 Tastes and Preferences of the BuyerThe demand for a commodity is also affected by the tastes and preferences of the buyers They include change in fashion customs habits etc Those commodities are preferred by the consumers which are in fashion So demand for those commodities rises which are in fashion On the other hand if a commodity goes out of the fashion its demand falls because no consumer will like to buy it(5) Number of Buyers in the Market(Population)Increase in population raises the market demand whereas decrease in population reduces the market demand for a commodity Not only the size of population but its composition like age (ratio of males females children and old people in population) also affects the demand for a commodity It is because of needs of children young old male and female population differs(6) Distribution of Income and WealthIf the distribution of income and wealth is more in favour of the rich demand for the commodities preferred by the rich such as comforts and luxuries is likely to be higher On the other hand if the distribution of income and wealth is more in favour of poor demand for commodities preferred by the poor such as necessities will be more(7) Season and Weather ConditionsThis is generally observed that the demand for woolens increases during winter whereas demand for ice creams and cold drinks increases during summer Similarly market demand for umbrellas rain coats increases during rainy seasonQ8 REASONS FOR OPERATION OF LAW OF DEMAND WHY DEMAND CURVE SLOPES DOWNWARDNow we will try to explain why does a consumer purchase more quantity of a commodity at a lower price and less of it at a higher price or why does the law of demand operate ie why does the demand curve slope downwards from left to right The main reasons for operation of law of demand are1 Law of Diminishing Marginal UtilityAs you have studied earlier law of diminishing marginal utility states that as we consume more and more units of a commodity the utility derived from each successive unit goes on decreasing The consumer will be ready to pay more for those units which provide him more utility and less for those which provide him less utility It implies that he will purchase more only when the price of the commodity falls2 Income Effect

When price of a commodity falls purchasing power or real income of the consumer increases which enables him to purchase more quantity of the commodity with the same money income Let us take an example Suppose you buy 4 ice creams when price of each ice cream is ` 25 If price of ice creams falls to ` 20 then with same money income you can buy 5 ice creams now3 Substitution EffectWhen price of a commodity falls it becomes comparatively cheaper as compared to its substitutes (although price of substitutes has not been changed) This will lead to rise in demand for the given commodity For example if coke and Pepsi both are sold at ` 10 each and price of coke falls Now coke has become relatively cheaper and will be substituted for Pepsi It will lead to rise in demand for coke4 Change in Number of BuyersWhen price of a commodity falls some old buyers may demand more of the commodity at the reduced price and some new buyers may also start buying this commodity who were not in a position to buy it earlier due to higher price This will lead to increase in number of buyers when price of the commodity falls As a result demand for the commodity rises when its price falls5 Diverse Uses of a CommoditySome commodities have diverse uses like milk It can be used for drinking for sweet preparation for ice cream preparation etc If price of milk rises its use may be restricted to important purpose only This will lead to reduction in demand for other less important uses When price of milk falls it can be put to other uses also leading to rise n demand for itQ9 EXCEPTIONS TO THE LAW OF DEMANDYou have studied in law of demand that a buyer is willing to buy more quantity of a commodity at a lower price and less of it at a higher price But in certain circumstances a rise in price may lead to rise in demand These circumstances are called Exceptions to the Law of Demand Some important exceptions are1 Giffen GoodsGiffen goods are special type of inferior goods in which negative income effect is stronger than negative substitution effect Giffen goods do not follow law of demand as their demand rises when their price rises Examples of Giffen goods are jowar and bajra etc2 Status Symbol GoodsSome goods are used by rich people as status symbols eg diamonds gold jewellary etc The higher the price the higher will be the demand for these goods When price of such goods falls these goods are no longer looked at as status symbol goods and tehrefore therir demand falls3 NecessitiesCommodities such as medicines salt wheat etc do not follow law of demandbecause we have to purchase them in minimum required quantity whatever their price may be4 Goods Expected to be ScarceWhen the buyers expect a scarcity of a particular good in near future they start buying more and more of that good even if their prices are rising For example during war famines etc people tend to buy more of some goods even at higher prices due to fear of their scarcity in near future

Political Science

Constitution of India-The

Preamble

The preamble-

Preamble-

The preamble is the most precious part of the constitution We the people of India having solemnly resolved to constitute India into a Sovereign Socialist Secular Democratic Republic and to secure to all its citizensA preamble is an introductory and expressionary statement in a document that explains the documents purpose and underlying philosophy When applied to the opening paragraphs of a statute it may recite historical facts pertinent to the subject of the statuteNature and purpose of the constitution-Purpose of the Constitution dictates permanent framework of the government to form a more perfect union to establish justice and ensure peace of thenationconstitution provide principles how the government can run itself following the rules and laws written in the constitution of each state keeps them balanced

Answer the following questions-

1 What is preambleA preamble is an introductory and expressionary statement in a document that explains the documents purpose and underlying philosophy2 What is the nature and

purpose of the constitutionConstitution dictatespermanent framework of the government to form a more perfect union to establish justice and ensure peace of the nation

Homework-Learn

Accounts Compatibilty mode

1MEANING OF PARTNERSHIPPartnership is a form of business organisation where two or more persons join hands to run a business They share the profits and losses according to the agreement amongst them According to the Indian Partnership Act 1932 ldquoPartnership is relation between persons who have agreed to share profits of a business carried on by all or any one of them acting for allrdquo For example one of your friends has passed class XII from National Institute of Open Schooling (NIOS) and wants to start a business Heshe approaches you to join in this venture Heshe wants you to contribute some money and participate in the business activities Both of you if join hands constitute a partnership2CHARACTERISTICS1048698 Agreement A partnership is formed by an agreement The agreement may be either oral or in writing It defines the relationship between the persons who agree to carry on business It may contain the terms of sharing profit and the capital to be invested by each partner etc The written agreement is known as partnership deed1048698 Number of persons There must be at least two persons to form a partnership

The maximum number of partners in a partnership firm can be 50 according toCompanies Act 20131048698 Business The Partnership is formed to carry on business with a purpose of earning profits The business should be lawful Thus if two or more persons agree to carry on unlawful activities it will not be termed as partnership1048698 Sharing Profits The partners agree to share profits in the agreed ratio In caseof loss all the partners have to bear it in the same agreed profit sharing ratio10486981048698Mutual Agency Every partner is an agent of the other partners Every partner can bind the firm and all other partners by hisher acts Each partner will be responsible and liable for the acts of all other partners10486981048698Unlimited liability The liability of each partner except that of a minor is unlimited Their liability extends to their personal assets also If the assets of the firm are insufficient to pay off its debts the partnersrsquo personal property can be used to satisfy the claim of the creditors of the partnership firm10486981048698Management All the partners have a right to mange the business However they may authorize one or more partners to manage the affairs of the business on their behalf10486981048698Transferability of Share No partner can transfer hisher share to any one including hisher family member without the consent of all other partners3PARTNERSHIP DEEDAgreement forms the basis of partnership The written form of the agreement is which a document of partnership is It contains terms and conditions regarding the conduct of the business It also explains relationship between the partners This document is called partnership deed Every firm can frame its own partnership deed in which the rights duties and liabilities of the partners are stated in detail It helps in settling the disputes arising among the partners during the general conduct of business 4CONTENTS OF PARTNERSHIP DEEDThe partnership deed generally contains the following (i) Name and address of the partnership firm(ii) Nature and objectives of the business(iii) Name and address of each partner(iv) Ratio in which profits is to be shared(v) Capital contribution by each partner(vi) Rate of Interest on capital if allowed(vii) Salary or any other remuneration to partners if allowed(viii) Rate of interest on loans and advances by a partner to the firm(ix) Drawings of partners and interest thereon if any(x) Method of valuation of goodwill and revaluation of assets and liabilities on the reconstitution of the partnership ie on the admission retirement or death of a partner(xi) Settlement of disputes by arbitration(xii) Settlement of accounts at the time of retirement or death of a partner5IN ABSENCE OF PARTNERSHIP DEEDThe partnership deed lays down the terms and conditions of partnership in regard to rights duties and obligations of the partners In the absence of partnership deed there may arise a controversy on certain issues like profit sharing ratio interest on

capital interest on drawings interest on loan and salary of the partners In such cases the provisions of the Indian Partnership Act becomes applicableSome of the Issues are(i) Distribution of Profit Partners are entitled to share profits equally(ii) Interest on Capital Interest on capital is not allowed(iii) Interest on Drawings No interest on drawing of the partners is to be charged(iv) Interest on Partnerrsquos Loan A Partner is allowed interest 6 per annum on the amount of loan given to the firm by himher(v) Salary and Commission to Partner A partner is not entitled to anysalary or commission or any other remuneration for managing the business

History TOPIC-TOWARDS INDEPENDENCE AND PARTITION THE LAST PHASE (1935-1947)

SUB TOPIC-IMPORTANT POLITICAL DEVELOPMENTS ndash GROWTH OF SOCIAL IDEAS

Socialism is a political social and economic philosophyLike in other parts of the world the Russian revolution of 1917 served as a great inspiration for revolutionaries in India who at that time were engaged in the struggle for liberation from British ruleSocialist ideas led to the formation of communist party of IndiaJAWAHARLAL NEHRU Among the early Congress leaders Jawaharlal Nehru was very much impressed and influenced by the Socialist ideas He also learnt about the Economic activities of the Soviet Union after the Bolshevic Revolution 1917 He made full use of them in IndiaThe election of Jawaharlal Nehru and Subhas Chandra Bose showed the Left wing tendency within CongressJawaharlal Nehru demanded economic freedom along with political freedom of the people in order to end the exploitation of masses

Nehrus working committee included three socialists leaders The Lucknow session was a landmark in the evolution of socialist ideas of the congressSUBHAS CHANDRA BOSE ndash Subhas Chandra Bose had socialist leaning Both Jawaharlal Nehru and Subhas Chandra Bose were known as leftist Congress men Later on National Congress divided into Leftist and rightist campCONGRESS SOCIALIST Within the Congress some leaders formed the Congress Socialist partyPattavi Sitaramyya Sardar Patel Rajendra Prasad had hostile attitude towards the Congress Socialist partyJawaharlals attitude was hesitant

1 QUESTION ndash Mention name of two Congress leaders who had socialist leaning

1ANSWER ndash Subhas Chandra Bose and Jawaharlal Nehru2QUESTION- In which session of the congress Jawaharlal elaborated his Socialist ideas2 ANSWER ndash Lucknow and Faizpur Session in December 1935 and 19363QUESTION ndash Why Congress was sharply divided into leftist and rightist camp 3ANSWER ndash Subhas Chandra Bosersquos attempt to seek re election for congress presidentship in 1939sharply divided the National Congress into Leftist and Rightist camp4 QUESTION ndash Who was MN Roy 4 ANSWER ndash Manabendra Roy first formed the Communist Party of India outside the country at Tashkent in 19205QUESTION ndash Who formed the Congress Socialist Party within the Congress5 ANSWER ndash Jaya Prakash Narayan Achyut Patwardhan Acharya Narendra Dev Ram Mohan Lohia Aruna Asaf Ali6QUESTION ndash When was the Congress Socialist Party formed What was its object6 ANSWER ndash 1934The Congress Socialist Party sought to work out socialist programme through the Congress They joined hands with the Congress and wanted to carry

Subhas Chandra Bose being expelled from the congress after the Tripuri rift he formed Forward BlockThere were basic differences between the Congress Socialists and the communistsTRADE UNION ACTIVITIES Maximum working class people lived in Bombay and Calcutta The working and living conditions of those workers were very miserable In this situation Shasipada Banerjee NM Lokhande protested against the oppression of the working class peopleThe first Trade Union Madras Labour Union was formed in 1918 by BP WadiaIndustrial strikes took place in Kanpur Calcutta Madras Jamshedpur and Ahmedabad AITUC was formed in Bombay in 1927 The growth of Trade union among the workers was slow because of the fear of the dismissal of the jobIn the mean time the Moderates as well as Communists left AITUC and formed separate organization

on National struggle with the help of workers and peasant class of the society7 QUESTION ndash What was the name of the party founded by Subhas Chandra Bose7 ANSWER- Forward Block8QUESTION ndash Who was Shasipada Banerjee8 ANSWER ndash Shasipada Banerjee was a radical Brahmo He founded a working menrsquos club to protest against exploitation of the British rulers towards the working class of India9 QUESTION ndash What was the weekly published by NM Lokhande9ANSWER- Dinabandhu10 QUESTION ndash Who founded Bombay Mill-Hands Association and in which year10 ANSWER- NM Lokhande in189011 QUESTION- Who was BP WadiaANSWER- BPWadia was the founder of Madras Labour Union in191812 QUESTION- What was the name of the first labour union of India12 ANSWER- Madras Labour Union13 QUESTION Who founded the Majur Mahajan 13 ANSWER GANDHIJI14 QUESTION What was the full form of AITUC When it was formed14 ANSWER All India Trade Union Congressin 192715QUESTION Who formed the Red Trade Union Congress and in which year15ANSWER The Communists formed the Red Trade Union Congress16 QUESTION What do you mean by Socialism16 ANSWER Socialism describes any political and economic theory that says the community rather than individuals should own and manage property and natural resources

Subject Eng Literature (The Tempest ndash William Shakespeare) Topic Act III Scene 3 Lines 1 to 52 (Line 52 ndash Brother my lord the Duke Stand to and do as we) Date 13th April 2020 (4th Period)

[Students should read the original play and also the paraphrase given in the school prescribed textbook]Summary Questions amp Answers

o Alonso Sebastian Antonio Gonzalo Adrian Francisco and others wandered about the island in search of Ferdinand and gets tired and hungry of the toil and at the same time gives up all hope of finding him

o Antonio and Sebastian are happy that Alonso is out of hope and decide to make another attempt on his life that night when being so tired they will be sleeping soundly

o Suddenly a solemn and strange music is heard in the air and several strange shapes enter bringing in a banquet These strange shapes then dance round it with gestures of salutation and then inviting the King to eat they depart

o Seeing this strange scene all are inclined to believe the tales told by travelers that there truly are ldquounicornsrdquo and ldquothe phoenixrsquo thronerdquo

1 ALONSO What harmony is this My good friends hark (L18-27)

GONZALO Marvellous sweet music

[Enter several strange shapes bringing in a banquet

they dance about it with gentle actions of salutation

and inviting the King and his companions to eat they depart]ALONSO Give us kind keepers heavens What were theseSEBASTIAN A living drollery Now I will believe

That there are unicorns that in Arabia

There is one tree the phoenixrsquo throne one phoenix

At this hour reigning thereANTONIO Ill believe both

And what does else want credit come to me

And Ill be sworn rsquotis true Travellers neer did lie

Though fools at home condemn rsquoem

(a) How did Prospero present an amazing spectacle before Alonso and his companions

Using his magic powers Prospero ordered strange shapes to lay a banquet before Alonso and his companions The shapes brought several dishes with tasty eatables in them They placed the dishes on a table before Alonso and his companions Then the strange shapes began to dance gracefully around the banquet While dancing they made gestures inviting them to eat the food Then suddenly the shapes disappeared(b) Who were the guests at the strange banquet Describe the lsquoliving drolleryrsquo

Alonso Sebastian Antonio Gonzalo Adrian and Francisco were the guests at the strange banquet

The term ldquoliving drolleryrdquo refers to live entertainment show In this context when Alonso the King of Naples Sebastian his brother Antonio the treacherous brother of Prospero Gonzalo the kind and loyal councillor to the King Adrian and Francisco came to the island they were hungry and weary in their spirits They heard a solemn and strange music They were shocked to see several strange shapes bringing in a banquet and these shapes danced about it with gentle action of salutation inviting the King and his companions to eat After this Sebastian described this show as lsquoliving drolleryrsquo(c) What is lsquophoenixrsquo What are lsquoUnicornsrdquo

The term lsquophoenixrsquo refers to a mythical Arabian bird which lived alone and perched on a solitary tree After one hundred years it expired in flames and rose again from its own ashes

lsquoUnicornsrsquo refers to the mythological four-footed beasts having horns in the centre of their foreheads When the horns are ground into powder the powder was believed to be

an aphrodisiac(d) How does Sebastian explain the puppet show OR Why does the speaker now believe in unicorns and phoenix

Sebastian finds several strange shapes bringing in the banquet They invite the king and his party for dinner and soon depart He tells that if such a strange sight can be a reality there is nothing incredible in the world and from the present moment he will believe anything He says that it is a strange dumb show enacted not by puppets but by living beings It is stranger than a travellerrsquos tale Seeing such a thing

before his own eyes he will no longer disbelieve the story about unicorns and phoenix(e) How do the other characters present respond to this living drollery

At the sight of the lsquoliving drolleryrsquo like Sebastian Gonzalo and Antonio too acted strangely Antonio told that he too now believes in unicorns and phoenix and anything else that seems to be incredible He too now believes in travellersrsquo tales Gonzalo told that if he would report those happenings in Naples nobody will believe him He considers that those gentle shapes were gentler in manner in comparison to the living beings Alonso was at first sight suspicious and told them that those strange shapes conveyed their meaning in expressive gestures when they seemed to lack speech by their movements and sounds Francisco was amazed at their mysterious disappearance

2 ALONSO Not I

(Line 43-52)GONZALO Faith sir you need not fear When we

were boysWho would believe that there were mountaineers

Dewlapped like bulls whose throats had hanging at rsquoem

Wallets of flesh Or that there were such men

Whose heads stood in their breasts Which now we find

Each putter-out of five for one will bring us

Good warrant ofALONSO I will stand to and feed

Although my lastmdashno matter since I feel

The best is past Brother my lord the Duke

Stand to and do as we

(a) How does Alonso respond at the spectacle of the shapes which were sent to them at the instruction of Prospero

After seeing the strange sight of appearing and disappearing of the shapes sent by Prospero to arrange a banquet for them Alonso says that his surprise at having seen those creatures is infinite and he is fully justified in feeling so much surprise He thinks that their shapes their gestures and the sounds they made were indeed amazing Although they do not possess the gift of speech yet they were able to convey their

thoughts by means of their gestures only

(b) What does Prospero say about the views expressed by Alonso regarding the shapes What does Francisco think about the shapesAfter hearing Alonsorsquos views about the shapes Prospero says that this manrsquos praise of the spirits is rather hasty He means to say that Alonso has shown great haste in reaching the conclusion about the shapes Francisco is amazed to see that those shapes disappeared in a mysterious way(c) What does Sebastian ask Alonso to doSebastian tells Alonso that the shapes having disappeared should not matter to them because they have left the eatables behind He asks Alonso to enjoy eating as they are extremely hungry but the king does not accept his offer of enjoying the dishes(d) How does Gonzalo try to dispel Alonsorsquos fear of those strange shapes What kind of references does he give to AlonsoGonzalo says that those who have travelled abroad have reported seeing even stranger sights than these shapes that Alonso and his companions have beheld Hence there is no reason to feel afraid of these shapes Gonzalo further adds that in his younger days he had heard strange stories from travelers and Alonso might have heard similar stories For instance it was said that there existed a certain race of

human beings who had huge lumps of flesh hanging at their throats and who therefore resembled bulls Then Gonzalo tells about a race of human beings whose heads were located at their breasts Gonzalo says that such stories were not believed by most people in those days but now-a-days these stories have become common(e) Explain the following lsquoEach putter-out of five for onersquoEnglish travellers often insured their trips with London brokers Those that went on foreign travels those days used to deposit a certain amount with some firm or company in London before their departure If the travelers failed to return the money was forfeited by the company with which it had been deposited But this money was repaid five-fold if the travelers returned safe and sound In this way a traveler stood a great chance of recovering the entire cost of his

travels(f) Give the explanatory meanings of the following expressions in the context of the above extract (i) Dewlapped (ii) Wallets of flesh

(iii) Putter-out(i) Dewlapped having big lumps of flesh at the necks(ii) Wallets of flesh large masses of flesh looking like bags(iii) Putter-out to invest money before commencing the travel

  • General methods of preparation of hydrogen
  • Chapter Dimensional Analysis (Summary)
    • Properties of Charge
Page 20:   · Web viewSubject. Topic. Summary. Execution. Hindi. व्याकरण. शरीरके अंगो के नाम लिखिए. 1) आँख 2) नाक 3

as personal accounts Example Ramrsquos Account ABC amp Co Account etc

Real account - The accounts related to tangible and intangible assets are called real accountsExample Cash Account Furniture Account etc

Nominal account- Accounts related to expenses losses incomes and gains are known as nominal accountsExample Wages Account Salary Account Discount Account etc

bull Accounting cycle Accounting cycle refers to a complete sequence of accounting activities It begins with recording of transactions and ends with the preparation of a balance sheet

English 1 Transformation of sentences

Sentences A sentence is a group of words which makes complete sense

a Assertive sentencesb Imperative sentencesc Interrogative

sentencesd Exclamatory sentences

Sentences can be changed from one grammatical form to another without changing the meaning of the sentence This is known as transformation of sentences

Exercise 6Rewrite the following sentences according to the instructions given below without changing their meanings

1 As soon as he saw the beer he jumped into the river ( Begin No sooner)

2 None but brave deserve the fair (Begin the bravehellip)

3 This box is too heavy for me to lift ( Use so hellip That instead of too)

4 No one other than a king can live like James Luxurious ( Begin only James)

5 Oh for the wings of a dove (Begin I wishhellip)

BENGALI(2ND LANGUAGE)

ldquo বঙগভমির পরমি ldquo াইকেল ধসদন দতত

পব13পোসঠ আসোলিচত ৩ পরবোস দৈদসবর বস ীবতোরো Pলিদ স এ লেদ -আকো সত-োলি লেদ তোস - ক) বকতো লেক কোর লেো লেকো কলিবতোর অং ) কোর পরলিত বকতোর এই উলিকত গ) এ লেদ আকো সত বসত কী বলিঝসয়স4 ীবতোরো বসত কী লেবোঝ ঘ ) আসোচয অংসর তোৎপP13 কী

উ -ক ) বকতো স কলিব মোইসক ম3দ দতত

Types of AccountPersonal AccountReal AccountNominal AccountBalance Sheet (opening)

কলিব মোইসক ম3দ দসততর রলিচত বঙগভলিমর পরলিত কলিবতোর অং ) কলিব বঙগী অ13োৎ লেদমোতোর পরলিত কলিবর এই উলিকত গ ) এ লেদ আকো বসত কলিবর মোব লেদী রপ আকো লেক লেবোঝোসো সয়স4 আকো লেসক লেPম তোরো স পসর লেতমলি ীব লেদ রপ আকো লেসক পরো রপ তোরো স পরসত পোসর এই মভোবোর কোই কলিব বসস4 ঘ ) পরবো Pোতরোয় Pলিদ কলিবর লেদ আকো লেসক ীব তোরো রপ পরো স পসর তোসত কলিব লিবনদমোতর দঃলিত কোর মতয লিবসর সবোভোলিবক পলিরলিত এবং মোষ মরী তোই পরবোস Pলিদ তা োর মতয য় তবও কলিব লিবচলিত সব ো কোর পলিবীসত লেকউ অমর য় লিক4ই অকষয় য় দীর লেPম লিচরপরবোমো লেতমলি মোসষর ীবও চমোতোই ীব - সতবধতোই মতয ীব দীসত মোষ লিতয পরবোমো তবও লেPব মোষ আপ কতকসম13র মো3যসম মোসষর মস লিসসদর সথো কসর লিসত পোসর তোরো লিচরভোসবর সয় মোসষর মস লিবরো কসর তোসদর মস3য লেকউ পGভসত লিবী সয় গোসও মোসষর মস তোরো লিতযপলিত লিতযবলিনদত

Hindi 2ndlang

काकीी(लिसयारामशरणगपत)

इस कहानी म क न यह बतान का परयास निकया ह निक बचच अपनी मा स निकतना परम करत ह शयाम अबोध बाक ह वह अपनी मा क मरन क बाद उसन अपनी मा क लिए बहत रोया बाद म उस पता चा निक उसकी मा राम क घर ची गई ह आकाश म उडती हई पतग दकर उस हष हआ निक पतग क दवारा वह अपनी मा को नीच उतारगा इसक लिए वह अपनी निपता की जब स दो बार सवा रपया निनकाकर पतग और दो मोटी सी मन वाी अपन भाई स काकी एक कागज पर लिवा कर पतग म लिशव का दिदयानिनकाकर पतग और दो मोटी सी मन वाी अपन भाई स काकी एक कागज पर लिवा कर पतग म लिचपका दिदयाभोा और शयाम कोठरी म रससी बाधनी रह थ तभी उसक निपता करोध म आकर उन स पछ निक कया उनकी जब स रपया निनकाा हभोा डर क मार बताया निक शयाम इस पतग क दवारा अपनी काकी को राम क यहा स उतारना चाहता हनिवशशवर(शयाम क निपता)न फटी पतग उठाकर दी तो उस पर काकी लिा थावह हत बजि होकर वही ड रह गएउनहोन सोचा निक मन अपन पतर को मारा जोनिक अनजान और निनदष थावह अपनी मा कोनिकतना पयार करता ह

helliphellipContinue to next

Computer Application

Java Programming Prog 1Write a java program to input two numbers from user and display the sum or product of them as per user choice Use switch case statementSolve public class sum_product public static void main(String args[]) Scanner sc=new Scanner(Systemin) int abc Systemoutprintln(ldquoEnter two numbersrdquo) a=scnextInt() b=scnextInt() Systemoutprintln(ldquoPress 1 for sum or 2 for productrdquo)

c=scnextInt() switch(c) case 1 Systemoutprintln(ldquoThe sum will be =rdquo+(a+b)) break case 2 Systemoutprintln(ldquoThe product will be =rdquo+(ab)) break default Systemoutprintln(ldquoWrong Inputrdquo) Home Work - Practice in your computer using bluej

Subject Eng Literature (The Merchant of Venice ndash William Shakespeare)Topic Act I Scene 2 Lines 92 to 126 (End of scene) Date 13th April 2020 (5th Period)

[Students should read the original play and also the paraphrase given in the school prescribed textbook]Summary Questions amp Answers

o After Portia has expressed her opinion about the suitors Nerissa informs that she need not bother about any one of them as they have decided to quit Belmont at the earliest opportunity because they do not believe in trying their luck by the caskets which is the only way of winning Portia

o Nerissa then enquires of Portiarsquos opinion about Bassanio who once visited her in the company of the Marquis of Montferrat and says that she had never come across such an ideal love deserving the fairest lady for his bride

o Portia seems to remember Bassanio quite correctly and says that she agrees with Nerissa At this moment a servant informs Portia that the Prince of Morocco has arrived to try his luck by the caskets

o Portia tells Nerissa that if she could welcome this new suitor as gladly as she says farewell to the previous ones she would be glad of his arrival However if he happens to have the virtues of a saint but the black complexion of a devil she would prefer to have him for religious consolation rather than as a husband

(1) NERISSA You need not fear lady (Line 97-103)

the having any of these lords they have acquainted me with their determinations

which is indeed to return to their home and to

trouble you with no more suit unless you may be wonby some other sort than your fathers imposition depending on the caskets

PORTIA If I live to be as old as Sibylla I will die as chaste asDiana unless I be obtained by the manner of my fatherswill I am glad this parcel of wooers are so reasonablefor there is not one among them but I dote on his veryabsence and I pray God grant them a fair departure

(a) Elucidate the idea expressed in the first speech of the above dialogue

In the first speech Nerissa assures Portia that she need not have any fear of being compelled to marry anyone of the suitors who had lately come to Belmont She informs her that they have all decided to return to their respective countries(b) Illuminate the meaning of the phrase ldquoyour fatherrsquos imposition depending on the casketsrdquo

Nerissa means that the suitors of Portia do not find the conditions imposed by the will of her father to their liking They are too hard for them These conditions are that in the event of a suitor failing to choose the right casket (i) he should never disclose to anybody which casket he chose (ii) he can never marry and (iii) he should take his departure immediately(c) Explain the meaning of the term lsquoSibyllarsquo

lsquoSibyllarsquo is the name given by Romans and Greeks to a prophetess inspired by some deity usually the sun-god Apollo She had a very long life The god Apollo granted her as many years of life as she could hold grains of sand in her hand(d) Elucidate the meaning of the term lsquoDianarsquo

lsquoDianarsquo is the goddess of hunting She is also regarded as a symbol of virginity because she never fell in love and never

married(e) Explain the meaning of the first two lines of Portiarsquos speech

Portia says that even if she is to live for centuries like Sibylla she would not marry except in accordance to her fatherrsquos will She asserts that she would not mind remaining unmarried and untouched by a man like Diana the virgin the goddess of hunting unless a man is able to win her by passing the test laid down by her father

Class XSubject Topic Summary Execution

Hindi 2nd

Langबड घर की बटी( मशी परमचद)

lsquoबड घर की बीटीrsquo कहानी का उददशय मधयम वग की घर समसया को सझा कर सगदिठत परिरवार म मिम जकर परम स रहन का सदश दना ह घर म शानित tानिपत करन की जिजममदारी नारी की होती ह यदिद नारी समझदार ह उसम धय और परिरवार क परनित परम ह तो कोई भी घटना परिरवार को निवघदिटत नही कर सकती या कहानी परिरवार को सगदिठत करत हए परम सौहाद स एक रदसर की भावनाओ को समझ करउनका सहयोग करत हए जीवन यापन करन की पररणा दती ह मशीपरमचदर जी न इस कहानी म सयकत परिरवार का परनितनिनमिधतव निकया ह यह कहानी बनी माधव सिसह जो गौरी पर क जमीदार क उनक दो पतरो की हशरी कठ ा निबहारीशरीकात का निववाह एकजमीदार घरान की पतरी आनदी स हआ थाआनदी न द को ससरा क वातावरण म ढालिया थाएक दिदन आनदी का अपन दवर ा निबहारी स झगडा हो जाता ह दोनो भाई एक रदसर स अग होन की कोलिशश करत हसभी बह आनदी न अपन मधर वयवहार स ा निबहारी को घर छोडकर जान स रोक लिया| इस पर बनी माधव सिसह न कहा निक बड घर की बटी ऐसी ही होती ह जो निबगडा काम बना ती ह अतः शीषक साथक ह बड घर की बटी आनदी ह

helliphelliphelliphellipContinue to nextBiology Topic ndash Chp-1

CellWelcome to new session 2020-21Today we will start with Chpter 1 cell CELL

Protoplasm+Cellmembrane Or Cell wall

Cytoplasm+Neucleus

Cytoplasmic+ CytoplasmicOrganelles Inclutions(mitochondria (food Golgi bodies pigments)Ribosome)

What is cellbull Cell is the structural and functional unit of living organismbull According to number of cells organisms areUnicellular - Amoeba bacteria Multicellular - Rose Mango Tiger HumanSmallest cell -bacteria Longest cell - Nerve cellLargest cell - Ostrich egg cellCells are of different size and shapes according to their functionsQ2Write chief functions of following cellorganelles

Q3What is tonoplastVacuoles covered by a covering called tonoplast

Bengali(2Nd

Language)

ফ ফটক ো ফটক (কলিবতো ) ভোষ মসোপো3 gtPোয়

একটি লেমসয়র ীবস লেপরম লিকভোসব ফসট ওসঠ তো লেদলিসয়স4 কলিব লেপরম Pই য় লেই ময়ই বনত কোস পলিরত য় ফ লেফোটো বো োসফোটো লেটো ব2 কো য় লেমসয়সদর ব gtয13 লেপরসমর 4লিব ফসট উসঠস4 এই কলিবতোয় লেপরম মোষসক মত gtযর মস লেফস লিদসয় পরকষস বাোচোসোর gtয োত বো2োয় কলিবতোয় লেমসয়টির পসব13র দঃসর কো বো সও লেমসয়টি লেই পসর পলিক সত চোয়ো োরী ীবসর কোস4 পরম লেPৌবস লেপরমসক পোবোর পরব ইচছো োকসও তো পসর লেলিতবোচকতোয় পলিরত য় কলিব ভোষ মসোপো3 যোয় লেP ক লেপরসমর

কলিবতোয় ব gtযবহত লিবসষ লিক4 সvর অ13 লেদওয়ো ১) রসবোো= লেP লিবলিভনন রকম ডোকসত পোসর২) ো= পোর ৩) ঠলি = লেচোসর বZ৪)আই বস2ো=অলিববোলিত৫)শইসয় = োলিয়ত কসর৬)োতপাোচ= লিবলিভনন পরকোর৭)দ2োম = v কসর বZ কসর লেদওয়ো৮)লেরলিং =লেোোর দৈতরী লেব2ো৯) বনত= একঋত১০) পাোর = বসকরো2

Organelles Functions

1 Endoplasmic reticulum

2 Mitochondria

3Golgibodies

4 Ribosome5Lysosome

6Plastids

7 Centrosome

i) Supportive framework for the cellii) Synthesis and transpost of proteinsRelease of energy in the form of ATPi) Synthesis and secretion of enzymes hormoneii) Formation of vacuoles lysosomei) Protein Synthesisi) Intracellular digestionii) Destroy foreign substancei )Leucoplast - stores starchii)chloroplast - trap solar energyiii) Chromoplast - imparts colour toflowers amp fruitsi) Initiates and requlates cell division

কলিবতো তোর অ13সক ভোষোয় পরকোো কসর ঘলিরসয় ব যকত কসরস4 লেপরসমর ফতো আর লিবফতো লেক গোঢ় কসর লেদোসো কলিব ভোষ মসোপো3 যোসয়র অলিভবসর অ যলিদক

Economics

Factors of Production

Welcome to the new sessionToday we are going to start the first chapter of Class XThe name of the chapter is Factors of productionBy the name I hope you all can recall a glimpse of what you have learnt in the second chapter of Class IX

NowProduction is the process of creating the various goods and services which are consumed by the people of the country to satisfy their wants

Thus it is the process in which some materials are transformed from one form to another to create utility and value in goods

For example utility can be created by changing the form of a commodity ie

Making of table out of wood by a carpenter for his customer here the wood is getting transformed into table creating utility for his customer and he can also command a price for it

On the other hand Housewives perform very

useful activities at home which create utility but their domestic activities are not included in production because they have no money value

So we can also say that Production denotes two things firstly creation of utility and secondly creation of value

Production is not complete unless it reaches the consumer

An increase in production will increase the economic welfare of the consumers and hence the aim is to raise the production level of the country

Again production of a good or service is only possible if certain resources or

Questions

1 What do you mean by production

Answer Production means the creation of goods and services for the purpose of selling in the market

In fact production involves the transformation of inputs into outputs

Hence production denotes two thingsCreation of utility and creation of valueUtility and value can be created by changing the form by changing the place by changing the time and by rendering services

Example Transformation of raw

materials into finish goods such as potter creates utility by converting mud into utensils assembling of small parts to make bigger machinery

Production also includes services such as distribution and marketing

2 What are the factors of production

Answer Factors of Production refers to the resources and inputs needed for producing goods and servicesThese inputs can be classified as

Land Labour

Capital Enterprise

Land Land is defined to include not only the surface of the earth but also all other free gifts of nature(for example mineral resources forest resources and indeed anything that helps us to carry out the production of goods and services but is provided by

inputs are used together in right proportion

A resource or an input which helps in the process of production to obtain an output is called FACTOR OF PRODUCTION

These factors of production can broadly be categorized into four parts 1LAND 2LABOUR3CAPITAL4ENTERPRISE (ORGANISATION)or Entrepreneur

The above factors are all interdependent on each other and they play a major role in production process

FACTORS OF PRODUCTION

LANDCAPITAL

LABOUR ENTREPRENEUR

nature free of cost)LabourLabour refers to the human efforts that need to be combined with other factors of production for creating an output

CapitalAll man ndash made means of production is called capita example machineries which help in further production Money when used for starting any business for purchasing raw materials machinery tools etc it is regarded as capitalCapital also includes physical capital like factories machineriestoolsbuildingsequipments etcEnterpriseThe task of bearing risks is called enterprise and the person who bears these risks of business is called the entrepreneurThus an entrepreneur is one who organises production takes important decisions regarding production hires and purchases factors of production and bears the risk and uncertainty involved in productionOrganisation refers to the services of an entrepreneur who controls organises and undertakes all risks One who plans organises and manages a business enterprise is an organiser

Physics Chapter 1 Force

Force is an external agent capable of changing the state of rest or motion of a particular body It has a magnitude and a direction The direction towards which the force is applied is known as the direction of the force and the application of force is the point where force is applied The Force can be measured using a spring balance The SI unit of force is Newton (N)

Question 1

State the condition when on applying a force the body has

(a) the translational motion

(b) The rotational motion

Solutions

(a) Translational motion is produced when the body is free to move

(b) Rotational motion is produced when the body is pivoted at a point

Question 2

Define moment of force and state its SI unit

Solutions

The moment of force is equal to the product of the magnitude of the force and the perpendicular distance of the line of action of force from the axis

of rotation

The SI unit of moment of force is Newton times meter

= Newton meter (Nm)

Commercial Studies

Stake holders In this topic you will be come to know about the meaning and concept of stakeholders

How stakeholders are different from shareholders

Questions1 What do you mean by the term stake holdersAnswer) The term stake holders have developed from the words which mean an interest or expected benefit Stakeholders mean all those individuals groups and Institutions which have a state (interest) in the functioning and performance of a commercial organisation or a business enterprise2 What do you mean by share holdersAnswer) The person and Groups who own the shares of the joint stock company by providing capital to the company are called shareholders Shareholders are the internal stakeholders shareholders are one out of several stake holders3 How are shareholders different from stakeholdersAnswer)i) The term shareholders is related to only joint stock company whereas stakeholders are related with all business organisationsii) Stakeholders maybe any individual having financial stake in business organisation whereas a shareholders are those individuals who are holding shares in the company4) How are shareholders different from creditorsAnswer) i) Shareholders are internal stakeholders while creditors are external stakeholdersii) Shareholders invest in the capital of the company whereas creditors give loan to the companyiii) Shareholders are the members of the company with voting rights but creditors are not the members of the company

English 1 Transformation of sentences

Sentences A sentence is a group of words which makes complete sense

e Assertive sentencesf Imperative sentencesg Interrogative sentencesh Exclamatory sentences

Sentences can be changed from one grammatical form to another without changing the meaning of the sentence This is known as transformation of sentences

Exercise 1 Change the following affirmative sentences into Negative sentences

a He is a good manHe is not a bad man

b Ram loves SitaRam is not without love for Sita

c Only he stood first in the classNone but he stood first in the class

d Ankit was wiser than he

He was not so wise as Ankite He did it

He did not fail to do itf As soon as I reached college the

bell rangNo sooner did I reach college than the bell rang

g He finished everythingHe left nothing unfinished

h It always pours when it rainsIt never rains but it pours

Math Topic Commercial MathematicsChapter ndash Goods and services Tax

What is GSTAns It is a abbreviated term of Goods and Service Text which is an indirect tax levied on the sale of goods and rendering servicesSome terms related to GSTDelar Any person who buys goods or services For resale is known as a delar A delar Can be a firm or a companyIntra-state sales Sales of goods and services within the same state or same union territory are called intra- state salesInter-state sales Sales of goods and services outside the state or union territory are called Inter-state sales4) Input GST GST is paid by dealers on purchase of goods and services are called input GST5) Output GST GST is collected from customers on sale of goods and services are called output GST6) Types of GST There are three taxes applicable under GST(i) Central Goods and Services Tax (CGST)(ii) State Goods and Services Tax (SGST) or Union Territory Goods and Services Tax (UTGST) Both these taxes are levied on intra-state sales Here GST is divided equally among central and state governments(iii) Integrated Goods and Services Tax (IGST) IGST is levied on inter- state sales It is also levied on import of goods and services into India and export of goods and services from India

Subject Eng Literature (The Merchant of Venice ndash William Shakespeare)Topic Act III Scene 4 Lines 1 to 44 (Portia hellip To wish it back on you fare you well Jessica)[Students should read the original play and also the paraphrase given in the school prescribed textbook]

Summary Questions amp AnswersIn this scene we suddenly find a new element in the character of Portia We have already seen her possessed of every graceful womanly quality but now she shows that she is capable of rapid decision and determined action She shows this by her sudden resolve to hasten to Venice with a daring scheme for the rescue of Antonio This is an important scene in the dramatic action for it leads up to and renders possible the striking events of the famous trial scene which is one of the greatest striking elements of the play Moreover the fact that all the characters of importance are now assembled together in Venice makes the union of the main plot and the secondary story complete

(1) LORENZO Madam although I speak it in your presence(Line 1-9)

You have a noble and a true conceit

Of god-like amity which appears most strongly

In bearing thus the absence of your lordBut if you knew to whom you show this honourHow true a gentleman you send reliefHow dear a lover of my lord your husbandI know you would be prouder of the workThan customary bounty can enforce you

(a) Where is Lorenzo Why is he here To whom is he referring as lsquoMadamrsquo

Lorenzo is at Portiarsquos residence He had met Salerio on the way and Salerio had begged him to come along with him to

o In this scene Portia Nerissa Lorenzo Jessica and Balthazar appear

o Portia requests Lorenzo and Jessica to be in charge of her house during her absence from Belmont because she and Nerissa have decided to spend the days in meditation and also in visiting the holy places in the neighbourhood of Belmont She has already instructed her people to acknowledge both Lorenzo and Jessica as master and mistress of house during her absence Lorenzo and Jessica gladly agree to look after the house of Portia

handover the letter from Antonio to Bassanio The letter carried the bad news about Antoniorsquos arrest for non-payment of loan taken from Shylock Hence Salerio might have preferred company to break this bad news to Bassanio He is referring to Portia as Madam(b) What does Portia say on hearing the above extract

Portia says that she has never regretted doing good to others Friends who spend a lot of time together and really are there for each other have many traits in common As Antonio is Bassaniorsquos best friend saving him is like saving Bassanio who is like her own soul She asks Lorenzo to take care of management of the house till Bassanio is back(c) What does Portia send with Bassanio and why

On hearing about Antoniorsquos troubles on account of Bassanio her husband Portia immediately sends him with enough gold to repay the debt many times over to Venice to help Antonio out of his misfortune

(2) Lorenzo Madam with all my heart (Line 36-40)

I shall obey you in all fair commands

Portia My people do already know my mindAnd will acknowledge you and JessicaIn place of Lord Bassanio and myselfSo fare you well till we shall meet again

(a) Where are Lorenzo and Portia at this time What lsquofair commandsrsquo are given to Lorenzo

Lorenzo and Portia are at Belmont during this scenePortia reveals to Lorenzo that she has sworn to contemplate in prayer at a monastery around two miles away until her husband returns from Venice She tells him that Nerissa would accompany her and asks him to manage the house with Jessica till things are settled In response Lorenzo tells her that he would be obliged to do whatever she asks him to do(b) Where is Portia actually going and why

Portia tells Lorenzo that she would live a life of contemplation and pray at a monastery which is two miles away from her place In reality Portia plans to go to Venice in disguise with Nerissa and argue the case in defense of Antonio She is very sure that her plan would succeed

ClassXI (ScienceHumanitiesCommerce)Subject Topic Summary Execution

Computer Science

(APC)

Ch ndash 1 Numbers

(Numbers in different bases and

their Arithmatical operations)

Number System In computers Number System is defined as a writing system to represent the numbers in different ways ie we are using different symbols and notations to represent numbers There are four ways we can represent the number ndash Binary Decimal Octal and Hexadecimal

Decimal Number SystemThis number system consist 10 digits These are 0 1 2 3 4 5 6 7 8 amp 9

Binary Number SystemThis number system has only two digits these are 0 and 1 Here 0 stands for off while 1 stands for on

Octal Number SystemThis number system has 8 digits these are 0 1 2 3 4 5 6 amp 7

Hexadecimal Number SystemThis number system has 16 digits these are 0 1 2 3 4 5 6 7 8 9 A B C D E F Here the value of the alphabets are as follows A=10 B=11 C=12 D=13 E=14 F=15

Rules for conversion decimal number to Binary1 Divide the decimal number by 22 If the number will not divide equally by 2 then round down the answer to the nearest whole number (integer)3 Keep a note of the remainder it should be either 0 or 14 Keep repeating the above steps dividing each answer by 2 until you reach zero5 Write out all the remainders from bottom to top This is your binary solution

For example Lets convert 32 to binary 2 32 2 16 - 0 2 8 - 0 2 4 - 0 2 2 - 0 2 1 - 0 0 - 1

The binary equivalent of 3210 is 1000002

Try the follwing youself1 2410

2 4810

3 1210

History GROWTH OF NATIONALISM

The second half of the 19th century witnessed growth of political consciousness and a sense of Nationalism among the IndiansThere were various factors for growth of Indian Nationalism- As a result various political associations were formed in different provinces by the educated Indians Surendranath Banerjee organized a meeting of National conference at Calcutta Ultimately the National Congress was founded in Bombay in 1885This body became the vanguard of Indian struggle for freedom The congress leaders were known as moderates because they followed a policy of prayer and petition A large number of Indian leaders had experienced in political agitation The Political situation of England was also changed Moreover increasing revolutionary activities in Maharashtra Punjab and Bengal became serious concern to the British Government In this

QUESTION1 What do you mean by Nationalism ANSWER 1 Nationalism is defined as loyalty and devotion to own nation especially a sense of national consciousnessQUESTION 2 What are the causes of nationalism ANSWER 2 There were various factors for growth of nationalism

1 Spread of western education2 The progress of vernacular press and

patriotic literature3 The economic exploitation of our

country by the colonial rulers4 International affairs

QUESTION 3 Who organized National conference in Calcutta in 1883 ANSWER 3 Surendranath BanerjeeQUESTION 4 When did Indian National Congress formANSWER 4 Indian National Congress was formed in 1885 in BombayQUESTION 5 Who were ModeratesANSWER 5 The Early Nationalists were also known as Moderates Their emergence marked

background Lord Curzon became Viceroy in India He had no respect for the Indian National Congress

the beginning of the organized national movement in India They believed in British justice and were loyal to them They followed a policy of prayer and petition They demanded constitutional reforms of our country Impotant Moderate leaders were Pherozshah Mehta Dadabhai Naorozi and Surendranath Banerjee etcQUESTION 6 What do you know about Extremism in Indian National movementANSWER 6 In the beginning of 20th century a new class of national leaders emerged in India which was different from the moderate groups They started more aggressive movement against the British empire The goal of extremists was ldquoswarajrdquo Important extremist leaders were Bal Gangadhar Tilak Lala Lajpat Rai Bipin Chandra Pal etcQUESTION 7 Mention the places which were the main centres of Revolutionary movementANSWER 7 Maharashtra Bengal and Punjab

Physics

Chapter Dimensional Analysis

(Summary)

The dimensions of a physical quantity are the powers to which the fundamental units are raised in order to obtain the derived unit of that quantit

The physical quantites lengthmasstime are represented by [L] [M] [T] resp let they are raised to powers ( dimesions) abc resp then any physical quantity can be represented by [ La Mb Tc ] Examples

1 Area area = L x B = [L] x [L] = [M0 L2 T0 ]

2 Density density = massvolume = [M][L3] = [ M L-3]

3 Velocity velocity = distancetime = [L][T] = [LT-1]HW Try to find out dimension of acceleration Acceleration = velocity timeNB One can find the SI Units Using Dimension Analysis Such as for area we have [L2] so its SI unit is m2

Biology Topic ndash Chp-1 The living world

Today we will start the first chapter the living world Here we discuss about the characteristics of living organism and what are the difference between them and nonliving substances We also discuss about the contribution of different Scientists

There are over 500000 species of plants andover a million species of animal are present on earth Some 15000 new species were discovered every yearQ1 What is a living organismbull A living organism is primarily physico -chemical material that demonstrate a high degree of complexity is capable of selfRegulation possesses a metabolism and perpetuates itself through timeQ2 What are the differences between livingand non-livingsi) Compared with non-living living organisms

have more complex organised structure and their use of energy is more controlled amp efficientii) Living things reproduce their own kind by forming new cells which contains copies of their genesiii) Each organism has some degree of homeostasisie it is able to make adjustments so that internal environment remains constantQ3 Write contributions of following Scientists i) Aristotle - One of the first theories in Biology places all living things in a hiearchieii) AV Leeuwenhoek - was the first to observe living single celled organisms under microscopeii) Carolus Linnaeus - developed the binary system for naming of organisms and classificationiii) Geregor Johann Mendel ndash discoverbasic principles of inheritanceHomework i) C Darwin ii)Schleiden

Math Trigonometric functions

1 Overviewi) Trigonometry The word lsquotrigonometryrsquo is derived from the Greek words lsquotrigonrsquo and lsquometronrsquo which means measuring the sides of a triangle An angle is the amount of rotation of a revolving line with respect to a fixed line Usually we follow two types of conventions for measuring angles ie a) Sexagesimal system b) Circular system In Sexagesimal system the unit of measurement is Degree In Circular system the unit of measurement is Radian ii) Relation between degree and radianThe ratio of circumference of a circle to its diameter is always a constant This constant ratio is a number denoted by π which is taken approximately as 227The relationship between degree amp radian measurements is as follows2 right angles = 180deg= π radians1radian = 180degπ=57deg16(approx) 1deg=π180 radianiii) Length of an arc of a circleIf an arc of length s subtends an angle θ radians at the center of a circle of radius r then s=rθiv) Area of a sector of a circleA sector is like a pizza slice of the

Q) Express the following angles in radiana) 45deg b) 40deg3730Ans a) We have 180deg=π radiansi e 45deg= πtimes45180 radian = π4 radiansb) 40deg3730= 40deg37+3060 minute= 40deg 37 +12 minute= 40deg+ 752 minute=40 + 75(2times60) degree=3258 degreeNow 180deg=π radianie 3258 degree= (πtimes325) (180times8) radians = 65π288 radiansQ) A circle has a radius of r=12 meters What is the length of an arc traced out by a 60deg angle in the center of the circleAns In this problem we know both the central angle (60deg) and the radius of the circle (12) All we have to do is plug those values into our equation and we get

s = 2π(12)(60360)s = 24π6s = 4πSo the length of an arc traced out by a 60deg angle in a circle with a radius of 12 meters equals 4π meters asymp 1257 metersQ) Find the area of the sector with a central angle 30deg and a radius of 9cmAns GivenRadius r = 9 cmAngle θ = 30degArea of the sector = θ360degtimesπr2

= 30360degtimes227times92=2121cm2

circle It consists of a region bounded by two radii and an arc lying between the radiiThe area of a sector is a fraction of the area of the circle This area is proportional to the central angle In other words the bigger the central angle the larger is the area of the sectorArea of Sector = θ2 times r2 (when θ is in radians)

Area of Sector = θ times π360 times r2 (when θ is in degrees)

COMMERCE

CLASSIFICTION OF HUMAN ACTIVITIES-ECONOMIC AND NON-ECONOMIC

Welcome to the new sessiontoday we are going to start the first chapter of Class XI The name of the chapter that we are going to start is

lsquoClassification of Human Activities ndasheconomic and non-economicrsquo

Now let us start the chapter by considering human beings and the activities they perform throughout the day

Human activities means all those activities that human beings undertake to satisfy their wants

Human wants on the other hand are the desire of human beings for goods (vegetables fruits rice etc) and services (services of doctors teachers lawyers etc) that they require to live

Now these human activities continue throughout life as human wants are unending unlimited and recurring as human beings desire for better living throughout their lives

Now human activities can be classified into two categories

Human activities

Economic activities Non-economic activities

Economic activities are

Questions1 What are human activities

Answer Human activities mean all those activities that human beings undertake to satisfy their wants

Example A man working in an office

A boy playing in the garden

2What are the characteristics of human activitiesAnswer the characteristics of human activities are as follows

Human activities are undertaken by men women and children and these activities involve human efforts

Human activities are undertaken to satisfy human wants which are unlimited

Human activities continue throughout life

Human activities are performed for both earning money and personal satisfaction

3What is economic activitiesGive example

Answer Economic activities are undertaken by human beings with the object of earning money acquiring wealth and thereby satisfying human wantsExample

Selling of goods by a shop keeper to his customer

A clinic run by a doctor Service of a teacher in school or college

undertaken by human beings with the object of earning money and acquiring wealth

These activities result in the production of economic goods and services

Example Human activities(ie working in factories officesshops) which produce direct economic benefits

Non-economic activities are inspired by human sentiments and emotions such as love for the family desire to help the poor and love for the country

Thus these human activities (eg praying playing sleeping) produce no direct economic benefits and they are also not related to earning money and acquiring wealth

4 What are the characteristics of economic activities

Answer The characteristics of economic activities are as follows

Economic motiveEconomic activities are undertaken to earn money and acquire wealth

ProductiveEconomic activities involve productiondistribution and exchange of goods and services to create wealth

Economic growthEconomic activities determine the level of economic development of a country and standard of living of its citizens

Socially desirableEconomic activities are socially desirable for society

Economic resourcesEconomic activities make use of all the economic resources such landlabourcapital etc

5 What do you mean by non-economic activitiesExampleAnswerNon-economic activities are inspired by human sentiments and emotions such as love for the family desire to help the poor and love for the countryThese activities are not undertaken for monetary gain but for onersquos satisfaction and happinessExample

a mother looks after her children

a student donates blood8 Differentiate between Economic activities and Non-economic activities

Economic activities

Non-economic activities

1to earn living and acquiring wealth2Result can be measured in terms of money

3ExampleBusinessprofession and employment

1 to obtain some satisfaction

2Result cannot be measured in terms of money

3ExampleFamily-orientedreligious socialCultural and national

BUSINESS STUDIES

BUSINESS ENVIRONMENT

Welcome to the new sessionToday we are going to start the first chapter and the name of the chapter is Business Environment

In todayrsquos world every business enterprise is a part of the society It exists and operates in association with various groups in society such as customers suppliers competitors banks and financial institutions government agencies trade unions media and so on All these groups influence the functioning of business in one way or the other They constitute the environment of businessConcept of Business Environment

The term lsquobusiness environmentrsquo refers to the sum total of all individuals institutions and other forces that lie outside a business enterprise but that may influence its functioning and performance

The main features of business environment

Totality of External forces General and Specific forces Interrelatedness Complexity Dynamic Uncertainty Relativity

The Interrelation between business and its environment

The business enterprise is an open system It continuously interacts with its environment It takes inputs

Prepare the following questions from todayrsquos assignment

1 What do you mean by business environment

The term lsquobusiness environmentrsquo means the aggregate of all forces factors and institutions which are external to and beyond the control of an individual business enterprise but they may influence its functioning and performance Business environment is the macro framework within which a business firm a micro unit operates It consists of several interrelated and interacting elements

2 Explain the main features of business environment in brief

Totality of External forces-Business environment is the sum total of all things external to a business environment

General and Specific forces-It includes both the forces general forces are the economic social political legal and technological conditions which indirectly influence all business enterprise Specific forces are the investors customers competitors and suppliers which influence individual enterprise directly

Interrelatedness-Different elements of environment are interrelated for an example growing awareness for health care has increased the demand for health foods

Complexity- Business environment id

(such as raw materials capital labour energy and so on) from its environment transforms them into goods and services and sends them back to the environment

Fig 1 Business Environment Relationship

complex in nature as the elements keep on changing example economic technological and other forces changes in demand for a product and service

Dynamic-Business environment is not static it keeps on changing

Uncertainty- Itrsquos very difficult to predict future events such as technology and fashion which occur fast and frequently

Economics Basic Economic ConceptsSub topic

Microeconomics and

Macroeconomics

Welcome to the new sessiontoday we are going to start the first chapter of Class XI The name of the chapter that we are going to start is Basic Economic concepts

Now Economics covers the study of human activities Human activities are those activities which are performed by humans to satisfy their wants

Thus Human wants are unlimited and therefore economic activities such as production exchange and consumption are needed in order to satisfy those wants

The study of economics is divided largely in two parts which areMicroeconomics and Macroeconomics

SUBJECT- MATTER OF ECONOMICS

MICROECONOMICS MACROECONOMICS

Questions1Who has coined the words micro and macro economics

Answer Ranger Frisch coined the words lsquomicrorsquo and lsquomacrorsquo in 1933 to denote the two branches of economic theory namely microeconomics and macroeconomics

2What is microeconomicsAnswer It is the study of behaviour of individual decision ndash making unit such as consumers firms etc

3 What is macroeconomicsAnswer Macroeonomics is the study of overall economic phenomena like employment national income etc

4 What is the importance of microeconomicsAnswer

Microeconomics helps in formulating economic policies which enhance productive efficiency and results in greater social welfare

It helps the government in formulating correct price policies

It explains the working of a capitalistic economy where individual units(producers and consumers ) are free to take their own decision

Micro means a small part in

microeconomics we do not study the whole economy Hence we study an individual consumer and his or her choices and a producer and his or her profit maximizing decisions in the market Thus it does not mirror what happens in the economy as a whole

Macroeconomics on the other hand studies the economy as a whole It is concerned with aggregate and depicts the entire picture of the economyMacroeconomics deals with the national income aggregate investment aggregate consumption etc

Features of Microeconomics It deals with small

parts of the country Hence it looks at

individual consumers firms and industries

It deals with individual income consumption and savings

It studies the determination of price of any product or factors of production

It deals with the working of market via the price mechanism which is nothing but the determination of price and quantity of a commodity by the forces of demand and supply

Features of Macroeconomics

It deals with the study of the economy as a whole

It is concerned with

5 Give a limitation of microeconomics Microeconomics fails to explain the

functioning of an economy as a whole It cannot explain unemployment illiteracy and other problems prevailing in the country

6 What is the importance of macroeconomics It gives overall view of the growing

complexities of an economic system It provides the basic and logical

framework for formulating appropriate macroeconomic policies (eg for inflation poverty etc )to direct and regulate economy towards desirable goals

7What is the limitation of macroeconomics It ignores structural changes in an

individual unit of the aggregate

8 Differentiate between Microeconomics and Macroeconomics

Microeconomics Macroeconomics

the study of aggregates

National income aggregate savings and aggregate investments are major concepts dealt within macroeconomics style

It studies the determination of general price levels

It investigates into the problem of unemployment and the achievement of employment

It studies the aspect of decision making at the aggregate and national levels

It includes all growth theories whether related to developed or developing economies it also includes the study of economic systems and the working of the economy under different systems

Note Both Micro and macro economics are complementary and should be fully utilized for proper understanding of an economy

1It studies economic aspect of an individual unit2It deals with individual incomeConsumption and savings

3 It facilitates determination of price of any product or factors of production

4 Itrsquos scope is narrow and restricted to individual unit

1It studies the economy as a whole

2It deals with the national income aggregate consumption and aggregate savings3 It facilitates determination of general price level in an economy

4 Itrsquos scope is wide as it deals with economic units on the national level

ACCOUNTS

Introduction to Accounting and Book-keeping

Today I am going to share you the meaning of Accounting and Book-keeping and its related terms bullAccounting bullBook Keeping bullAccountsbullTypes Of Accounts bullAccounting Cycle

bull Meaning of accounting

Ans ) Accounting is the art and science of recording classifying and summarising monetary transactions

bull Meaning of Book-keeping

Ans) Bookkeeping is the art of recording business transactions with the view of having a permanent record of them and showing their effect on wealth

bull Meaning of account

Ans) The term account means a record of

business transactions concern a particular person of firm asset or income or expense It is a summarised record of all transactions which take place in an accounting year

bull Types of accountsPersonal accounts ndash Personal accounts relating

to person and Organisation are known as personal accounts Example Ramrsquos Account ABC amp Co Account etc

Real account - The accounts related to tangible and intangible assets are called real accounts Example Cash Account Furniture Account etc

Nominal account- Accounts related to expenses losses incomes and gains are known as nominal accounts Example Wages Account Salary Account Discount Account etc

bull Accounting cycle Accounting cycle refers to a complete sequence of accounting activities It begins with recording of transactions and ends with the preparation of a balance sheet

Chemistry TopicAtomic Structure

Thomsonrsquos atomic modelThomson (1898) was the first to propose the model of an atomHe proposed that an atom can be regarded as a uniform sphere of positive electricity in which requisite number of electrons are embedded evently to neutralize the positive chargeThis is just like plums embedded in a pudding or seeds evently distributed in red spongy mass of a watermelonThis model of atom is known as ldquoPlum-Pudding modelrdquo or

Q1)What is the fundamental constituents of atomAns Electron Proton and neutrons are the fundamental constituents of atomQ2)What is the value of fundamental unit of electricityAnsThe charge carried by one electron is sad to be the fundamental unit of electricityIts magnitude is 48times10-10esuOr 1602times10-19C Q3)Name the element containing no neutronAnsOrdinary hydrogen atom or protium 1H1

Types of AccountPersonal AccountReal AccountNominal AccountBalance Sheet (opening)

ldquowatermelon modelrdquoThis model could explain the electrical neutrality of an atom but failed to explain the result of scattering experiment carried out by Rutherford in 1911So it was rejected ultimately

Q4)Why is an electron called universal particleAns Itrsquos mass and Charge are independent of its source

EVS Chapter 1 ndash Modes of Existence

Modes of existence When one speaks normally about the mode of existence of some group or individual one refers to their customs their mode of being their ethology their habitat in some way their feeling for a placeDifferent modes of exixtence are ndash

1 Hunting ndashGathering2 Pastoral3 Agricultural4 Industrial

1 Hunting and gathering Hunting and gathering mode of existence is characterized by obtaining food from hunting wild animals including fishing and gathering wild plants From their earliest days the hunter-gatherer diet included various grasses tubers fruits seeds and nuts Lacking the means to kill larger animals they procured meat from smaller game or through scavenging

Societies that rely primarily or exclusively on hunting wild animals fishing and gathering wild fruits berries nuts and vegetables to support their diet are called hunting and gathering societies

At least this used to be practice of human beings before agriculture is invented As their brains evolved hominids developed more intricate knowledge of edible plant life and growth cycles

Q) Write the features of Hunting ndash gathering societiesAns - There are five basic characteristics of hunting and gathering societies

i The primary institution is the family which decides how food is to be shared and how children are to be socialized and which provides for the protection of its members

ii They tend to be small with fewer than fifty members

iii They tend to be nomadic moving to new areas when the current food supply in a given area has been exhausted

iv Members display a high level of interdependence

v Labor division is based on sex men hunt and women gather

Political Science

Introduction to political science

Political science occasionally called politology is a social science which deals with systems of governance and the analysis of political activities political thoughts associated constitutions and political behaviorThe study of political science involves the study of both the

Answer the following questions-1 What is political science

Political science occasionally called politology is a social science which deals with systems of governance and the analysis of political activities political thoughts associated constitutions and political behavior

2 Short notes-

traditional and modern theories of politicsTraditionalClassical political sciencepolitical theory-Traditional political science is the study of politics before Second World War The methodology to study Politics was traditional (legal formaletc) the definition of politics traditional (Politics begins and end with state)area of study (constitution state machinery)was traditionalModern Political scienceModern political theory-Modern Political Theory critically examines the contemporary state of political theory making an assessment of the achievement and limitations of the Behavioural Revolution in its totality and reviews objectively the major paradigms and conceptual frameworks adopted by the disciplineContemporary attempts at the development of an integrated political theory involving the use of both traditional and modern concepts approaches and theories-Around late 1960s several political scientists realized the importance of both the traditional political theory and modern Political theory They began building an integrated theory of politics involving a systematic mixture of traditional and modern studies of politics It was held that the study of a complex and vast field like politics needs both traditional as well as

Classical political theory Modern Political theory

Homework-Learn

modern concepts and approaches for studying itrsquos all aspects

Subject Eng Literature (The Tempest ndash William Shakespeare) Topic Act I Scene 1 Lines 1 to 32 (Line 32 ndash Gonzalo hellip If he be not born to be hanged our case is miserable) Date 13th April 2020 (3rd Period)

[Students should read the original play and also the paraphrase given in the school prescribed textbook]Summary Questions amp Answers

[SUMMARY OF THE ENTIRE SCENE]

o The play starts with the scene of a severe storm at sea Alonso (King of Naples) Sebastian (Alonsorsquos brother) Ferdinand (Alonsorsquos son) Gonzalo Antonio (the usurping Duke of Milan) are in a ship in the midst of the storm

o The mariners are trying their best to control the vessel from running aground and are totally following the orders of their Master the Boatswain They have scant success

o The mariners become extremely unhappy and annoyed when most of the passengers arrive on the deck thereby hampering their effort to save the ship There is serious confrontation between them and the passengers who are part of the Kingrsquos entourage

o The mariners could not save the ship

SUMMING-UP

(i) Vivid description of the scene which gives a realistic description of terror and confusion of a tropical storm

(ii) Shows Shakespearersquos accuracy of knowledge in describing the naval operations and also matters of seamanship

(iii) The opening scene justifies the title ndash The Tempest

UNANSWERED QUESTIONS

(i) The King always travels with his entire fleet including his soldiers Where

(1) GONZALO Nay good be patient (Line 15-26)BOATSWAIN When the sea is Hence What cares these

roarers for the name of the king To cabin silence Trouble us not

GONZALO Good yet remember whom thou has aboardBOATSWAIN None that I more love than myself You are a

councillor if you can command these elements to silence and work

the peace of the present we will not hand a rope more use your authority If you cannot give thanks you have

lived so long and make yourself ready in your cabin for the mischance of the hour if it so hap [To the Mariners]

Cheerly good hearts [To Gonzalo] Out of our way I say

(a) To whom is the boatswain speaking What does he mean by lsquoNone that I more love than myselfrsquo

The Boatswain is speaking to Gonzalo the honest old councilor of the Duke of MilanBy using the words ndash lsquoNone that I love more than I love myselfrsquo means that for the Boatswain nobody is dearer to him than his own life

(b) What were the conditions that made the boatswain react in this way

The Boatswain reacts in this way because the storm is at sea and Alonso King of Naples Sebastian his brother Ferdinand his son Gonzalo Antonio the usurping Duke of Milan on board are in distress and in panic Thus they have rushed to the deck interrupting the work of the mariners

(c) What hope does Gonzalo take from the attitude of the boatswain

The insolent and authoritative attitude of Boatswain makes Gonzalo feel comforted He tells that there are no signs that the Boatswain will be drowned But his facial appearance and attitude shows that he is destined to die on land by hanging which in effect means that all on board will be saved Otherwise all the persons on board are doomed

(d) How can they lsquomake yourself ready in your cabinrsquo For what were they asked to make ready themselves

In order to make themselves ready in their cabin the

were the other ships

(ii) Why was the ship in that area Where was it coming from or going where

(iii) The ship broke apart What happened to those who were in the ship

passengers on board must prepare for death which they will possibly soon have to meetThey can retire to their cabins and offer prayers to the Almighty to save them from drowning

(e) What does the boatswain say when he is asked to be patient What does he order to the royal party

When the boatswain is asked to be patient and remain calm he says that he will be patient only when the storm will be over and the sea will be calm but as long as the storm blows and there is danger to the ship he cannot think of being patient He orders the royal party to go to the cabin and leave the mariners to their work

(2) GONZALO I have great comfort from this fellow (Line 27-36)

Methinks he hath no drowning mark upon him his complexion is perfect

gallows Stand fast good Fate to his hanging Make the rope of his destiny our cable for our own doth little advantage If he be not born to be hanged our case is miserable

(a) Why does Gonzalo regard the Boatswain in the midst of danger

In the midst of danger Gonzalo regards the boatswain because he feels that the Boatswain is a source of comfort and is bent upon to do his work sincerely which in this case is saving the ship and its passengers from the severest of raging storm

(b) What reasons does Gonzalo give when he says that none in the ship will die of drowning

Gonzalo is almost sure that none in the ship will die by drowning His says that there is no mark on the face of the boatswain that indicates that he will die by drowning On the other hand the lines on his face are strong indications that he will be hanged to death Therefore there shall be no danger of the shiprsquos sinking

(c) Explain the following ldquoStand fast good Fate to his hanging Make the rope of his destiny our cable for our own doth little advantage If he be not born to be hanged our case is miserablerdquo

The stated lines mean that if the will of destiny is to be carried out then the ship will not get wrecked and all the passengers will be saved The safety of the passengers therefore depends upon the will of fate being carried out in the case of the boatswain If however the boatswain is not to die by hanging then the passengers are also very unsafe because in that case the ship is likely to sink

(d) What order does the Boatswain give to the sailors

when he re-enters What does he say about the crying of the fellows inside the cabin

The boatswain orders the sailors to bring the topmast lower and bring the ship close to a stationary position with the help of the main sail He says that the fellows inside the cabin are moaning and crying in their distress louder than his voice and louder even than the roaring of the storm

Class XII (ScienceCommerceHumanities) Subject Topic Summary Execution

Computer Science

PropositionalLogic

Propositional logic is a procedure to provide reasoning through statementProposition A ststement that results in True or False is said to be proposition There are two types of propositionSimple proposition amp compound propositionSimple proposioton A simple proposition is one that is not a part of any other proposition Such sentential form of proposition is symbolized with english letters in short For example Ram is a claver student (TrueFalse)Where do you live (Not in True or False)Grapes are sweet (TrueFalse)It rains today (TrueFalse)Here we can see some statements anwer would be true or false but some staements answer can not give in terms of true or false Thus the sentences which can be answered in true or false are known as simple propositionAssigning propositon to a variableThe general syntax to assign propostion to a variable is as followsVariable = Simple propositonFor example A=Ram is a clever studentB= Grapes are sweetC= it rains todayCompound proposition

helliphellipto be continued in next classhelliphellipMath Relation Relation If A and B are two non-empty sets

then a relation R from A to B is a subset of AxB If R A x B and (a b) R then we say that a sube isinis related to b by the relation R written as aRbeg Let A be the set of students of class XII and B be the set of students of class XI Then some of the examples of relation from A to B arei) (a b) AXB a is brother of bisinii) (a b) AXB age of a is more than age of isinb Types of relation In this section we would like to study different types of relations We know that a relation in a set A is a subset of A times A Thus the empty set φ and A times A are two extreme relations For illustration consider a relation R in the set A = 1 2 3 4 given by R = (a b) a ndash b = 10 This is the empty set as no pair (a b) satisfies the condition a ndash b = 10 Similarly R = (a b) | a ndash b | ge 0 is the whole primeset A times A as all pairs (a b) in A times A satisfy | a ndash

Example 1 Let A be the set of all students of a boys school Show that the relation R in A given by R = (a b) a is sister of b is the empty relation and R = (a b) the primedifference between heights of a and b is less than 3 meters is the universal relationSolution Since the school is boys school no student of the school can be sister of any student of the school Hence R = φ showing that R is the empty relation It is also obvious that the difference between heights of any two students of the school has to be less than 3 meters This shows that R = A times A is primethe universal relation Example 2 Show that the relation R in the set 1 2 3 given by R = (1 1) (2 2) (3 3) (1 2) (2 3) is reflexive

b | ge 0 These two extreme examples lead us to the following definitionsDefinition 1 A relation R in a set A is called empty relation if no element of A isrelated to any element of A ie R = φ A times AsubDefinition 2 A relation R in a set A is called universal relation if each element of A is related to every element of A ie R = A times A Both the empty relation and the universal relation are some times called trivial relation Definition 3 A relation R in a set A is called(i) reflexive if (a a) R for every a Aisin isin(ii) symmetric if (a1 a2) R implies that (aisin 2a1)

R for all aisin 1 a2 Aisin(iii) transitive if (a1 a2) R and (aisin 2 a3) R isinimplies that (a1 a3) R for all aisin 1 a2 a3 AisinDefinition 4 A relation R in a set A is said to be an equivalence relation if R is reflexive symmetric and transitive

but neither symmetric nor transitiveSolution R is reflexive since (1 1) (2 2) and (3 3) lie in R Also R is not symmetric as (1 2) R but (2 1) isin notinR Similarly R is not transitive as (1 2) R and (2 3) R but (1 3) R isin isin notinExample 3 Show that the relation R in the set Z of integers given byR = (a b) 2 divides a ndash b is an equivalence relationSolution R is reflexive as 2 divides (a ndash a) for all a Z isinFurther if (a b) R then 2 divides a isinndash b Therefore 2 divides b ndash a Hence (b a) R which shows that R is isinsymmetric Similarly if (a b) R and (b c) R isin isinthen a ndash b and b ndash c are divisible by 2 Now a ndash c = (a ndash b) + (b ndash c) is even (Why) So (a ndash c) is divisible by 2 This shows that R is transitive Thus R is an equivalence relation in ZExample 4 Let L be the set of all lines in a plane and R be the relation in L defined as R = (L1 L2) L1 is perpendicular to L2 Show that R is symmetric but neither reflexive nor transitiveSolution R is not reflexive as a line L1 can not be perpendicular to itself ie (L1 L1) R notinR is symmetric as (L1 L2) Risin

L1 is perpendicular to L2rArr L2 is perpendicular to L1rArr (L2 L1) RrArr isin

R is not transitive Indeed if L1 is perpendicular to L2 and L2 is perpendicular to L3 then L1 can never be perpendicular to L3 In fact L1 is parallel to L3 ie (L1 L2) R isin(L2 L3) R but (L1 L3) Risin notin

Chemistry Solid state Characteristics if Solids(i)The particles are locked in fixed positions they are unable to change their relative positions and this brings a definite shape and volume of a solid(ii)In a solid the constituent particles are held by strong forces of attractionThe forces of attraction may be bonding or non bonding(iii)The constituent particles in a solid pack together as closely as possibleoccupying most of the available space within the solidThus the empty space in a solid is very smallThis makes a solid highly rigid and nearly incompressibleThis also explains why a solid has high density and exhibits slow diffusionClassification of Solids

Q1)Define Crystalline solids AnsA Solid that has a definite geometrical shape and a sharp melting pointand whose constituent particles (atomsmolecules or ions) are arranged in a long range order of definite pattern extending throughout the solid is called a crystalline solidExNaClQ2)Define Amorphous solids AnsA solid that does not have a definite shape and a sharp melting pointand whose constituent particles (atomsmolecules or ions) are not arranged in a definite pattern is called an amorphoussolid

Crystalline solidsAmorphous solids

ExGlassRubberQ3)Classify Crystalline Solids Crystalline Solids

Physics Coloumbrsquos Law (Summary)

Before Going Into Coloumbrsquos Law We Will First Learn What is Charge Properties of Charge and Always remember that charge is quantized ie a body always have static charge of magnitude equal to some integral multiple of fundamental electronic charge e= 16 x 10- 19 C

Charge is the property of matter that causes it to produce and experience electrical and magnetic effects The study of the electrical charges at rest is called electrostatics When both electrical and magnetic effects are present the interaction between charges is referred to as electromagnetic

There exist two types of charges in nature positive and negative Like charges repel and unlike charges attract each other

The type of charge on an electron is negative The charge of a proton is the same as that of an electron but with a positive sign In an atom the number of electrons and the number of protons are equal The atom is therefore electrically neutral If one or more electrons are added to it it becomes negatively charged and is designated as negative ion However if one or more electrons are removed from an atom it becomes positively charged and is called a positive ion

The excess or deficiency of electrons in a body gives the concept of charge If there is an excess of electrons in a body it is negatively charged And if there is deficiency of electrons the body becomes positively charged Whenever addition or removal of electrons takes places the body acquires a charge

The SI Unit of charge is coulomb (C) In SI units the current is a fundamental quantity having a unit of ampere (A) The unit of charge is defined in terms of the unit of current Thus one coulomb is the charge transferred in one second across the section of a wire carrying a

Ionic SolidsMetallicSolids

Molecular Solids

current of one ampere

As q = It we have1 C = (1 A) (1 s)

The dimensions of charge are [A T]

Properties of Charge

(1) Quantization of Charge Electric charge can have only discrete values rather than any value That is charge is quantized The smallest discrete value of charge that can exist in nature is the charge on an electron given as

e = plusmn 16 x 10- 19 C

This is the charge attained by an electron and a protonA charge q must be an integral multiple of this basic unit That is

Q = plusmn ne where n = 1 2 hellip

Charge on a body can never be (frac12)e (23)e or 57e etcWhen we rub a glass rod with silk some electrons are transferred from the rod to the silk The rod becomes positively charged The silk becomes negatively charged The coulomb is a very large amount of charge A typical charge acquired by a rubbed body is 10 - 8 C

Biology Reproduction in organisms

Welcome to this new session 2020-21Today in this first chapter we mainly discuss about reproduction types needs and life span of some organismsWe also discuss about difference between sexual and asexual reproduction

Q1 What is reproductionReproduction is defined as a biological processin which an organism gives rise to young onessimilar to itselfQ2 What are the needs of reproductionbulli) Reproduction maintain life on earthii) It enables the continuity of the species generation after generationiii) It creates genetic variation among populationsQ3 Define Life span and write some orgnisms life spanbull Life span is the period from birth to

the natural death of an organism- OrganismsLife span1 Butterfly 1 - 2 weeks2 Fruit fly 30 days3Dog 10-13 years4 Rose5-7 years5 Tortoise100-150 years6 Banyan Tree -200 - 250 yearsQ4 Reproduction is of two types in case ofanimals but in case of plants vegetative propagation is also present

Asexual Reproduction Sexual Reproductioni) Always uniparentalii) Gametes are not involvediii) Only mitotic division involvediv) Somatic cells of parents are involvedv) Offsprings are genetically similar to the parents

i) Usually biparentalii) Gametes are involvediii) Meiosis occurs during gametogenesis Mitosis occurs after fertilisationiv) Germ cells of the parents are involvedv) offsprings are genetically different from the parents

COMMERCE BUSINESS ENVIRONMENT

Welcome to the new sessiontoday we are going to start the first chapter of Class XII The name of the chapter is Business Environment

Already many of you have got some idea about the word business environment form the first chapter of business studies in class XI

In todayrsquos world every business enterprise is a part of the society It exists and operates in association with various groups in society such as customers suppliers competitors banks and financial institutions government agencies trade unions media and so on All these groups influence the functioning of business in one way or the other They constitute the environment of businessConcept of Business Environment

The term lsquobusiness environmentrsquo refers to the sum total of all individuals institutions and other forces that lie outside a business enterprise but that may influence its functioning and performance

The main features of business environment Totality of External forces General and Specific forces Interrelatedness Complexity Dynamic Uncertainty

Prepare the following questions from todayrsquos assignment

2 What do you mean by business environment

The term lsquobusiness environmentrsquo means the aggregate of all forces factors and institutions which are external to and beyond the control of an individual business enterprise but they may influence its functioning and performance Business environment is the macro framework within which a business firm a micro unit operates It consists of several interrelated and interacting elements

2 Explain the main features of business environment in brief

Totality of External forces-Business environment is the sum total of all things external to a business environment

General and Specific forces-It

Relativity

The Interrelation between business and its environment

The business enterprise is an open system It continuously interacts with its environment It takes inputs (such as raw materials capital labour energy and so on) from its environment transforms them into goods and services and sends them back to the environment

Fig 1 Business Environment Relationship

includes both the forces general forces are the economic social political legal and technological conditions which indirectly influence all business enterprise Specific forces are the investors customers competitors and suppliers which influence individual enterprise directly

Interrelatedness-Different elements of environment are interrelated for an example growing awareness for health care has increased the demand for health foods

Complexity- Business environment id complex in nature as the elements keep on changing example economic technological and other forces changes in demand for a product and service

Dynamic-Business environment is not static it keeps on changing

Uncertainty- Itrsquos very difficult to predict future events such as technology and fashion which occur fast and frequently

Business Studies

Human Resources Management

Human resource of an organisation are the aggregate of knowledge skills attitudes of people working in it

The management system which deals with human resources is called human resource management

Features of HRMbullComprehensive functionbullPeople-oriented

Question1) What do you mean by human

resource management Answer) Human resource management may be defined as that field of Management which has to do with planning organising and controlling the functions of procuring developing maintaining and utilising the labour force

bullAction oriented bullPervasive function bullContinuous function

2) Explain the features of HRM in brief

Answer)bullHuman Resource Management is concerned with managing people at work bull Human Resource Management is concerned with employees which bring people and organisations together so that the goals of each are met bullHuman resource management considered every employees as an individual and also promote their satisfaction and growth bull Human resource management is inherent in all organisations and at all levelsbullManagement of human resources are ongoing on never ending process which requires a constant alertness and Awareness of human relations

3) ldquoHR function is said to be pervasiverdquowhy

Answer) Human resource management is required in all organisations whether it is private or government organisations armed forces sports organisations etc It permeatsall the functional areas like production marketing finance research etc This from this feature of human resource management it can be said that it is pervasive in nature

Economics Demand Q1DEFINITION OF DEMANDIn economics demand is the quantity of a good that consumers are willing and able to purchase at various prices during a given period of timeQ2DEMAND CURVEIn economics a demand curve is a graph depicting the relationship between the price of a certain commodity and the quantity of that commodity that is demanded at that pricQ3LAW OF DEMANDIn microeconomics the law of demand states that conditional on all else being equal as the price of a good increases quantity demanded decreases conversely as the price of a good decreases quantity demanded increasesQ4ASSUMPTION of LAW OF DEMAND(i)No change in price of related commodities(ii) No change in income of the consumer(iii) No change in taste and preferences customs habit and fashion of the consumer( No expectation regarding future change in priceQ5MARKET DEMAND SCHEDULEIn economics a market demand schedule is a tabulation of the quantity of a good that all consumers in a market will purchase at a

given price At any given price the corresponding value on the demand schedule is the sum of all consumersrsquo quantities demanded at that priceQ6INDIVIDUAL DEMAND SCHEDULEIndividual demand schedule refers to a tabular statement showing various quantities of a commodity that a consumer is willing to buy at various levels of price during a given period of timeQ7 FACTORS AFFECTING INDIVIDUAL DEMAND FOR A COMMODITY

The factors that influence a consumerrsquos decision to purchase a commodity are also known as determinants of demand The following factors affect the individual demand for a commodity1 price of the commodity2 price of related goods3 income of buyer of the commodity4 tastes and preferences of the buyer1 Price of the CommodityYou must have observed that when price of a commodity falls you tend to buy more of it and when its price rises you tend to buy less of it when all other factors remain constant (lsquoother things remaining the samersquo) In other words other things remaining the same there is an inverse relationship between the price of a commodity and its quantity demanded by its buyers This statement is in accordance with law of demand which you will study in the later part of this lesson Price of a commodity and its quantity demanded by its buyers are inversely related only when lsquoother things remain the samersquo So lsquoother things remaining the samersquo is an assumption when we study the effect of changes in the price of a commodity on its quantity demanded2 Price of Related goodsA consumer may demand a particular good But while buying that good heshe also asks the price of its related goods Related goods can be of two types-(i) Substitute goods(ii) Complementary goods While purchasing a good prices of its substitutes and complements do affect its quantity purchased(i) Price of Substitute Goods Substitute goods are those goods which can easily be used in place of one another for satisfaction of a particular want like tea and coffee An increase in price of substitute good leads to an increase in demand for the given commodity and a decrease in price of substitute good leads to a decrease in demand for the given commodity It means demand for a given commodity is directly affected by change in price of substitute goods For example if price of coffee increases the demand for tea will rise as tea will become relatively cheaper in comparison to coffee(ii) Price of Complementary goods Complementary goods are those goods which are used together to satisfy a particular want like car and petrol An increase in the price of complementary goods leads to a decrease in demand for the given commodity and a decrease in the price of complementary goods leads to an increase in demand for the given commodity For example if price of petrol falls then the demand for cars will increase as it will be relatively cheaper to use both the goods together So demand for a given commodity is inversely affected by change in price of complementary goods3 Income of the Buyer of CommodityDemand for a commodity is also affected by income of its buyer However the effect of change in income on demand depends on the nature of the commodity under consideration In case of some goods like full cream milk fine quality of rice (Basmati rice) etc demand for these commodities increases when income of the buyer increases and

demand for these commodities decreases when income of the buyer decreases Such goods whose demand increases with the increase in income of the buyer are called normal goods But there are some goods like coarse rice toned milk etc whose demand decreases when income of buyer increases and their demand increases when income of the buyer decreases Such goods whose demand decreases with the increase in income of the buyer are called inferior goods Suppose a consumer buys 10 Kgs of rice whose price is ` 25 per Kg He cannot afford to buy better quality of rice because the price of such rice is ` 50 per Kg The consumer is spending ` 250 per month on the purchase of rice Now if income of the consumer increases and he can afford ` 350 on purchase of 10 Kg of rice Now he can afford to buy some quantity of rice say 6 Kgs whose price is ` 25 per Kg and may buy 4 Kgs of rice whose price is ` 50 per Kg Thus he will buy 10 Kgs of rice by spending ` 350 per month Therefore we may conclude that demand for normal goods is directly related to the income of the buyer but demand for inferior goods is inversely related to the income of the buyer4 Tastes and Preferences of the BuyerThe demand for a commodity is also affected by the tastes and preferences of the buyers They include change in fashion customs habits etc Those commodities are preferred by the consumers which are in fashion So demand for those commodities rises which are in fashion On the other hand if a commodity goes out of the fashion its demand falls because no consumer will like to buy it(5) Number of Buyers in the Market(Population)Increase in population raises the market demand whereas decrease in population reduces the market demand for a commodity Not only the size of population but its composition like age (ratio of males females children and old people in population) also affects the demand for a commodity It is because of needs of children young old male and female population differs(6) Distribution of Income and WealthIf the distribution of income and wealth is more in favour of the rich demand for the commodities preferred by the rich such as comforts and luxuries is likely to be higher On the other hand if the distribution of income and wealth is more in favour of poor demand for commodities preferred by the poor such as necessities will be more(7) Season and Weather ConditionsThis is generally observed that the demand for woolens increases during winter whereas demand for ice creams and cold drinks increases during summer Similarly market demand for umbrellas rain coats increases during rainy seasonQ8 REASONS FOR OPERATION OF LAW OF DEMAND WHY DEMAND CURVE SLOPES DOWNWARDNow we will try to explain why does a consumer purchase more quantity of a commodity at a lower price and less of it at a higher price or why does the law of demand operate ie why does the demand curve slope downwards from left to right The main reasons for operation of law of demand are1 Law of Diminishing Marginal UtilityAs you have studied earlier law of diminishing marginal utility states that as we consume more and more units of a commodity the utility derived from each successive unit goes on decreasing The consumer will be ready to pay more for those units which provide him more utility and less for those which provide him less utility It implies that he will purchase more only when the price of the commodity falls2 Income Effect

When price of a commodity falls purchasing power or real income of the consumer increases which enables him to purchase more quantity of the commodity with the same money income Let us take an example Suppose you buy 4 ice creams when price of each ice cream is ` 25 If price of ice creams falls to ` 20 then with same money income you can buy 5 ice creams now3 Substitution EffectWhen price of a commodity falls it becomes comparatively cheaper as compared to its substitutes (although price of substitutes has not been changed) This will lead to rise in demand for the given commodity For example if coke and Pepsi both are sold at ` 10 each and price of coke falls Now coke has become relatively cheaper and will be substituted for Pepsi It will lead to rise in demand for coke4 Change in Number of BuyersWhen price of a commodity falls some old buyers may demand more of the commodity at the reduced price and some new buyers may also start buying this commodity who were not in a position to buy it earlier due to higher price This will lead to increase in number of buyers when price of the commodity falls As a result demand for the commodity rises when its price falls5 Diverse Uses of a CommoditySome commodities have diverse uses like milk It can be used for drinking for sweet preparation for ice cream preparation etc If price of milk rises its use may be restricted to important purpose only This will lead to reduction in demand for other less important uses When price of milk falls it can be put to other uses also leading to rise n demand for itQ9 EXCEPTIONS TO THE LAW OF DEMANDYou have studied in law of demand that a buyer is willing to buy more quantity of a commodity at a lower price and less of it at a higher price But in certain circumstances a rise in price may lead to rise in demand These circumstances are called Exceptions to the Law of Demand Some important exceptions are1 Giffen GoodsGiffen goods are special type of inferior goods in which negative income effect is stronger than negative substitution effect Giffen goods do not follow law of demand as their demand rises when their price rises Examples of Giffen goods are jowar and bajra etc2 Status Symbol GoodsSome goods are used by rich people as status symbols eg diamonds gold jewellary etc The higher the price the higher will be the demand for these goods When price of such goods falls these goods are no longer looked at as status symbol goods and tehrefore therir demand falls3 NecessitiesCommodities such as medicines salt wheat etc do not follow law of demandbecause we have to purchase them in minimum required quantity whatever their price may be4 Goods Expected to be ScarceWhen the buyers expect a scarcity of a particular good in near future they start buying more and more of that good even if their prices are rising For example during war famines etc people tend to buy more of some goods even at higher prices due to fear of their scarcity in near future

Political Science

Constitution of India-The

Preamble

The preamble-

Preamble-

The preamble is the most precious part of the constitution We the people of India having solemnly resolved to constitute India into a Sovereign Socialist Secular Democratic Republic and to secure to all its citizensA preamble is an introductory and expressionary statement in a document that explains the documents purpose and underlying philosophy When applied to the opening paragraphs of a statute it may recite historical facts pertinent to the subject of the statuteNature and purpose of the constitution-Purpose of the Constitution dictates permanent framework of the government to form a more perfect union to establish justice and ensure peace of thenationconstitution provide principles how the government can run itself following the rules and laws written in the constitution of each state keeps them balanced

Answer the following questions-

1 What is preambleA preamble is an introductory and expressionary statement in a document that explains the documents purpose and underlying philosophy2 What is the nature and

purpose of the constitutionConstitution dictatespermanent framework of the government to form a more perfect union to establish justice and ensure peace of the nation

Homework-Learn

Accounts Compatibilty mode

1MEANING OF PARTNERSHIPPartnership is a form of business organisation where two or more persons join hands to run a business They share the profits and losses according to the agreement amongst them According to the Indian Partnership Act 1932 ldquoPartnership is relation between persons who have agreed to share profits of a business carried on by all or any one of them acting for allrdquo For example one of your friends has passed class XII from National Institute of Open Schooling (NIOS) and wants to start a business Heshe approaches you to join in this venture Heshe wants you to contribute some money and participate in the business activities Both of you if join hands constitute a partnership2CHARACTERISTICS1048698 Agreement A partnership is formed by an agreement The agreement may be either oral or in writing It defines the relationship between the persons who agree to carry on business It may contain the terms of sharing profit and the capital to be invested by each partner etc The written agreement is known as partnership deed1048698 Number of persons There must be at least two persons to form a partnership

The maximum number of partners in a partnership firm can be 50 according toCompanies Act 20131048698 Business The Partnership is formed to carry on business with a purpose of earning profits The business should be lawful Thus if two or more persons agree to carry on unlawful activities it will not be termed as partnership1048698 Sharing Profits The partners agree to share profits in the agreed ratio In caseof loss all the partners have to bear it in the same agreed profit sharing ratio10486981048698Mutual Agency Every partner is an agent of the other partners Every partner can bind the firm and all other partners by hisher acts Each partner will be responsible and liable for the acts of all other partners10486981048698Unlimited liability The liability of each partner except that of a minor is unlimited Their liability extends to their personal assets also If the assets of the firm are insufficient to pay off its debts the partnersrsquo personal property can be used to satisfy the claim of the creditors of the partnership firm10486981048698Management All the partners have a right to mange the business However they may authorize one or more partners to manage the affairs of the business on their behalf10486981048698Transferability of Share No partner can transfer hisher share to any one including hisher family member without the consent of all other partners3PARTNERSHIP DEEDAgreement forms the basis of partnership The written form of the agreement is which a document of partnership is It contains terms and conditions regarding the conduct of the business It also explains relationship between the partners This document is called partnership deed Every firm can frame its own partnership deed in which the rights duties and liabilities of the partners are stated in detail It helps in settling the disputes arising among the partners during the general conduct of business 4CONTENTS OF PARTNERSHIP DEEDThe partnership deed generally contains the following (i) Name and address of the partnership firm(ii) Nature and objectives of the business(iii) Name and address of each partner(iv) Ratio in which profits is to be shared(v) Capital contribution by each partner(vi) Rate of Interest on capital if allowed(vii) Salary or any other remuneration to partners if allowed(viii) Rate of interest on loans and advances by a partner to the firm(ix) Drawings of partners and interest thereon if any(x) Method of valuation of goodwill and revaluation of assets and liabilities on the reconstitution of the partnership ie on the admission retirement or death of a partner(xi) Settlement of disputes by arbitration(xii) Settlement of accounts at the time of retirement or death of a partner5IN ABSENCE OF PARTNERSHIP DEEDThe partnership deed lays down the terms and conditions of partnership in regard to rights duties and obligations of the partners In the absence of partnership deed there may arise a controversy on certain issues like profit sharing ratio interest on

capital interest on drawings interest on loan and salary of the partners In such cases the provisions of the Indian Partnership Act becomes applicableSome of the Issues are(i) Distribution of Profit Partners are entitled to share profits equally(ii) Interest on Capital Interest on capital is not allowed(iii) Interest on Drawings No interest on drawing of the partners is to be charged(iv) Interest on Partnerrsquos Loan A Partner is allowed interest 6 per annum on the amount of loan given to the firm by himher(v) Salary and Commission to Partner A partner is not entitled to anysalary or commission or any other remuneration for managing the business

History TOPIC-TOWARDS INDEPENDENCE AND PARTITION THE LAST PHASE (1935-1947)

SUB TOPIC-IMPORTANT POLITICAL DEVELOPMENTS ndash GROWTH OF SOCIAL IDEAS

Socialism is a political social and economic philosophyLike in other parts of the world the Russian revolution of 1917 served as a great inspiration for revolutionaries in India who at that time were engaged in the struggle for liberation from British ruleSocialist ideas led to the formation of communist party of IndiaJAWAHARLAL NEHRU Among the early Congress leaders Jawaharlal Nehru was very much impressed and influenced by the Socialist ideas He also learnt about the Economic activities of the Soviet Union after the Bolshevic Revolution 1917 He made full use of them in IndiaThe election of Jawaharlal Nehru and Subhas Chandra Bose showed the Left wing tendency within CongressJawaharlal Nehru demanded economic freedom along with political freedom of the people in order to end the exploitation of masses

Nehrus working committee included three socialists leaders The Lucknow session was a landmark in the evolution of socialist ideas of the congressSUBHAS CHANDRA BOSE ndash Subhas Chandra Bose had socialist leaning Both Jawaharlal Nehru and Subhas Chandra Bose were known as leftist Congress men Later on National Congress divided into Leftist and rightist campCONGRESS SOCIALIST Within the Congress some leaders formed the Congress Socialist partyPattavi Sitaramyya Sardar Patel Rajendra Prasad had hostile attitude towards the Congress Socialist partyJawaharlals attitude was hesitant

1 QUESTION ndash Mention name of two Congress leaders who had socialist leaning

1ANSWER ndash Subhas Chandra Bose and Jawaharlal Nehru2QUESTION- In which session of the congress Jawaharlal elaborated his Socialist ideas2 ANSWER ndash Lucknow and Faizpur Session in December 1935 and 19363QUESTION ndash Why Congress was sharply divided into leftist and rightist camp 3ANSWER ndash Subhas Chandra Bosersquos attempt to seek re election for congress presidentship in 1939sharply divided the National Congress into Leftist and Rightist camp4 QUESTION ndash Who was MN Roy 4 ANSWER ndash Manabendra Roy first formed the Communist Party of India outside the country at Tashkent in 19205QUESTION ndash Who formed the Congress Socialist Party within the Congress5 ANSWER ndash Jaya Prakash Narayan Achyut Patwardhan Acharya Narendra Dev Ram Mohan Lohia Aruna Asaf Ali6QUESTION ndash When was the Congress Socialist Party formed What was its object6 ANSWER ndash 1934The Congress Socialist Party sought to work out socialist programme through the Congress They joined hands with the Congress and wanted to carry

Subhas Chandra Bose being expelled from the congress after the Tripuri rift he formed Forward BlockThere were basic differences between the Congress Socialists and the communistsTRADE UNION ACTIVITIES Maximum working class people lived in Bombay and Calcutta The working and living conditions of those workers were very miserable In this situation Shasipada Banerjee NM Lokhande protested against the oppression of the working class peopleThe first Trade Union Madras Labour Union was formed in 1918 by BP WadiaIndustrial strikes took place in Kanpur Calcutta Madras Jamshedpur and Ahmedabad AITUC was formed in Bombay in 1927 The growth of Trade union among the workers was slow because of the fear of the dismissal of the jobIn the mean time the Moderates as well as Communists left AITUC and formed separate organization

on National struggle with the help of workers and peasant class of the society7 QUESTION ndash What was the name of the party founded by Subhas Chandra Bose7 ANSWER- Forward Block8QUESTION ndash Who was Shasipada Banerjee8 ANSWER ndash Shasipada Banerjee was a radical Brahmo He founded a working menrsquos club to protest against exploitation of the British rulers towards the working class of India9 QUESTION ndash What was the weekly published by NM Lokhande9ANSWER- Dinabandhu10 QUESTION ndash Who founded Bombay Mill-Hands Association and in which year10 ANSWER- NM Lokhande in189011 QUESTION- Who was BP WadiaANSWER- BPWadia was the founder of Madras Labour Union in191812 QUESTION- What was the name of the first labour union of India12 ANSWER- Madras Labour Union13 QUESTION Who founded the Majur Mahajan 13 ANSWER GANDHIJI14 QUESTION What was the full form of AITUC When it was formed14 ANSWER All India Trade Union Congressin 192715QUESTION Who formed the Red Trade Union Congress and in which year15ANSWER The Communists formed the Red Trade Union Congress16 QUESTION What do you mean by Socialism16 ANSWER Socialism describes any political and economic theory that says the community rather than individuals should own and manage property and natural resources

Subject Eng Literature (The Tempest ndash William Shakespeare) Topic Act III Scene 3 Lines 1 to 52 (Line 52 ndash Brother my lord the Duke Stand to and do as we) Date 13th April 2020 (4th Period)

[Students should read the original play and also the paraphrase given in the school prescribed textbook]Summary Questions amp Answers

o Alonso Sebastian Antonio Gonzalo Adrian Francisco and others wandered about the island in search of Ferdinand and gets tired and hungry of the toil and at the same time gives up all hope of finding him

o Antonio and Sebastian are happy that Alonso is out of hope and decide to make another attempt on his life that night when being so tired they will be sleeping soundly

o Suddenly a solemn and strange music is heard in the air and several strange shapes enter bringing in a banquet These strange shapes then dance round it with gestures of salutation and then inviting the King to eat they depart

o Seeing this strange scene all are inclined to believe the tales told by travelers that there truly are ldquounicornsrdquo and ldquothe phoenixrsquo thronerdquo

1 ALONSO What harmony is this My good friends hark (L18-27)

GONZALO Marvellous sweet music

[Enter several strange shapes bringing in a banquet

they dance about it with gentle actions of salutation

and inviting the King and his companions to eat they depart]ALONSO Give us kind keepers heavens What were theseSEBASTIAN A living drollery Now I will believe

That there are unicorns that in Arabia

There is one tree the phoenixrsquo throne one phoenix

At this hour reigning thereANTONIO Ill believe both

And what does else want credit come to me

And Ill be sworn rsquotis true Travellers neer did lie

Though fools at home condemn rsquoem

(a) How did Prospero present an amazing spectacle before Alonso and his companions

Using his magic powers Prospero ordered strange shapes to lay a banquet before Alonso and his companions The shapes brought several dishes with tasty eatables in them They placed the dishes on a table before Alonso and his companions Then the strange shapes began to dance gracefully around the banquet While dancing they made gestures inviting them to eat the food Then suddenly the shapes disappeared(b) Who were the guests at the strange banquet Describe the lsquoliving drolleryrsquo

Alonso Sebastian Antonio Gonzalo Adrian and Francisco were the guests at the strange banquet

The term ldquoliving drolleryrdquo refers to live entertainment show In this context when Alonso the King of Naples Sebastian his brother Antonio the treacherous brother of Prospero Gonzalo the kind and loyal councillor to the King Adrian and Francisco came to the island they were hungry and weary in their spirits They heard a solemn and strange music They were shocked to see several strange shapes bringing in a banquet and these shapes danced about it with gentle action of salutation inviting the King and his companions to eat After this Sebastian described this show as lsquoliving drolleryrsquo(c) What is lsquophoenixrsquo What are lsquoUnicornsrdquo

The term lsquophoenixrsquo refers to a mythical Arabian bird which lived alone and perched on a solitary tree After one hundred years it expired in flames and rose again from its own ashes

lsquoUnicornsrsquo refers to the mythological four-footed beasts having horns in the centre of their foreheads When the horns are ground into powder the powder was believed to be

an aphrodisiac(d) How does Sebastian explain the puppet show OR Why does the speaker now believe in unicorns and phoenix

Sebastian finds several strange shapes bringing in the banquet They invite the king and his party for dinner and soon depart He tells that if such a strange sight can be a reality there is nothing incredible in the world and from the present moment he will believe anything He says that it is a strange dumb show enacted not by puppets but by living beings It is stranger than a travellerrsquos tale Seeing such a thing

before his own eyes he will no longer disbelieve the story about unicorns and phoenix(e) How do the other characters present respond to this living drollery

At the sight of the lsquoliving drolleryrsquo like Sebastian Gonzalo and Antonio too acted strangely Antonio told that he too now believes in unicorns and phoenix and anything else that seems to be incredible He too now believes in travellersrsquo tales Gonzalo told that if he would report those happenings in Naples nobody will believe him He considers that those gentle shapes were gentler in manner in comparison to the living beings Alonso was at first sight suspicious and told them that those strange shapes conveyed their meaning in expressive gestures when they seemed to lack speech by their movements and sounds Francisco was amazed at their mysterious disappearance

2 ALONSO Not I

(Line 43-52)GONZALO Faith sir you need not fear When we

were boysWho would believe that there were mountaineers

Dewlapped like bulls whose throats had hanging at rsquoem

Wallets of flesh Or that there were such men

Whose heads stood in their breasts Which now we find

Each putter-out of five for one will bring us

Good warrant ofALONSO I will stand to and feed

Although my lastmdashno matter since I feel

The best is past Brother my lord the Duke

Stand to and do as we

(a) How does Alonso respond at the spectacle of the shapes which were sent to them at the instruction of Prospero

After seeing the strange sight of appearing and disappearing of the shapes sent by Prospero to arrange a banquet for them Alonso says that his surprise at having seen those creatures is infinite and he is fully justified in feeling so much surprise He thinks that their shapes their gestures and the sounds they made were indeed amazing Although they do not possess the gift of speech yet they were able to convey their

thoughts by means of their gestures only

(b) What does Prospero say about the views expressed by Alonso regarding the shapes What does Francisco think about the shapesAfter hearing Alonsorsquos views about the shapes Prospero says that this manrsquos praise of the spirits is rather hasty He means to say that Alonso has shown great haste in reaching the conclusion about the shapes Francisco is amazed to see that those shapes disappeared in a mysterious way(c) What does Sebastian ask Alonso to doSebastian tells Alonso that the shapes having disappeared should not matter to them because they have left the eatables behind He asks Alonso to enjoy eating as they are extremely hungry but the king does not accept his offer of enjoying the dishes(d) How does Gonzalo try to dispel Alonsorsquos fear of those strange shapes What kind of references does he give to AlonsoGonzalo says that those who have travelled abroad have reported seeing even stranger sights than these shapes that Alonso and his companions have beheld Hence there is no reason to feel afraid of these shapes Gonzalo further adds that in his younger days he had heard strange stories from travelers and Alonso might have heard similar stories For instance it was said that there existed a certain race of

human beings who had huge lumps of flesh hanging at their throats and who therefore resembled bulls Then Gonzalo tells about a race of human beings whose heads were located at their breasts Gonzalo says that such stories were not believed by most people in those days but now-a-days these stories have become common(e) Explain the following lsquoEach putter-out of five for onersquoEnglish travellers often insured their trips with London brokers Those that went on foreign travels those days used to deposit a certain amount with some firm or company in London before their departure If the travelers failed to return the money was forfeited by the company with which it had been deposited But this money was repaid five-fold if the travelers returned safe and sound In this way a traveler stood a great chance of recovering the entire cost of his

travels(f) Give the explanatory meanings of the following expressions in the context of the above extract (i) Dewlapped (ii) Wallets of flesh

(iii) Putter-out(i) Dewlapped having big lumps of flesh at the necks(ii) Wallets of flesh large masses of flesh looking like bags(iii) Putter-out to invest money before commencing the travel

  • General methods of preparation of hydrogen
  • Chapter Dimensional Analysis (Summary)
    • Properties of Charge
Page 21:   · Web viewSubject. Topic. Summary. Execution. Hindi. व्याकरण. शरीरके अंगो के नाम लिखिए. 1) आँख 2) नाक 3

কলিব মোইসক ম3দ দসততর রলিচত বঙগভলিমর পরলিত কলিবতোর অং ) কলিব বঙগী অ13োৎ লেদমোতোর পরলিত কলিবর এই উলিকত গ ) এ লেদ আকো বসত কলিবর মোব লেদী রপ আকো লেক লেবোঝোসো সয়স4 আকো লেসক লেPম তোরো স পসর লেতমলি ীব লেদ রপ আকো লেসক পরো রপ তোরো স পরসত পোসর এই মভোবোর কোই কলিব বসস4 ঘ ) পরবো Pোতরোয় Pলিদ কলিবর লেদ আকো লেসক ীব তোরো রপ পরো স পসর তোসত কলিব লিবনদমোতর দঃলিত কোর মতয লিবসর সবোভোলিবক পলিরলিত এবং মোষ মরী তোই পরবোস Pলিদ তা োর মতয য় তবও কলিব লিবচলিত সব ো কোর পলিবীসত লেকউ অমর য় লিক4ই অকষয় য় দীর লেPম লিচরপরবোমো লেতমলি মোসষর ীবও চমোতোই ীব - সতবধতোই মতয ীব দীসত মোষ লিতয পরবোমো তবও লেPব মোষ আপ কতকসম13র মো3যসম মোসষর মস লিসসদর সথো কসর লিসত পোসর তোরো লিচরভোসবর সয় মোসষর মস লিবরো কসর তোসদর মস3য লেকউ পGভসত লিবী সয় গোসও মোসষর মস তোরো লিতযপলিত লিতযবলিনদত

Hindi 2ndlang

काकीी(लिसयारामशरणगपत)

इस कहानी म क न यह बतान का परयास निकया ह निक बचच अपनी मा स निकतना परम करत ह शयाम अबोध बाक ह वह अपनी मा क मरन क बाद उसन अपनी मा क लिए बहत रोया बाद म उस पता चा निक उसकी मा राम क घर ची गई ह आकाश म उडती हई पतग दकर उस हष हआ निक पतग क दवारा वह अपनी मा को नीच उतारगा इसक लिए वह अपनी निपता की जब स दो बार सवा रपया निनकाकर पतग और दो मोटी सी मन वाी अपन भाई स काकी एक कागज पर लिवा कर पतग म लिशव का दिदयानिनकाकर पतग और दो मोटी सी मन वाी अपन भाई स काकी एक कागज पर लिवा कर पतग म लिचपका दिदयाभोा और शयाम कोठरी म रससी बाधनी रह थ तभी उसक निपता करोध म आकर उन स पछ निक कया उनकी जब स रपया निनकाा हभोा डर क मार बताया निक शयाम इस पतग क दवारा अपनी काकी को राम क यहा स उतारना चाहता हनिवशशवर(शयाम क निपता)न फटी पतग उठाकर दी तो उस पर काकी लिा थावह हत बजि होकर वही ड रह गएउनहोन सोचा निक मन अपन पतर को मारा जोनिक अनजान और निनदष थावह अपनी मा कोनिकतना पयार करता ह

helliphellipContinue to next

Computer Application

Java Programming Prog 1Write a java program to input two numbers from user and display the sum or product of them as per user choice Use switch case statementSolve public class sum_product public static void main(String args[]) Scanner sc=new Scanner(Systemin) int abc Systemoutprintln(ldquoEnter two numbersrdquo) a=scnextInt() b=scnextInt() Systemoutprintln(ldquoPress 1 for sum or 2 for productrdquo)

c=scnextInt() switch(c) case 1 Systemoutprintln(ldquoThe sum will be =rdquo+(a+b)) break case 2 Systemoutprintln(ldquoThe product will be =rdquo+(ab)) break default Systemoutprintln(ldquoWrong Inputrdquo) Home Work - Practice in your computer using bluej

Subject Eng Literature (The Merchant of Venice ndash William Shakespeare)Topic Act I Scene 2 Lines 92 to 126 (End of scene) Date 13th April 2020 (5th Period)

[Students should read the original play and also the paraphrase given in the school prescribed textbook]Summary Questions amp Answers

o After Portia has expressed her opinion about the suitors Nerissa informs that she need not bother about any one of them as they have decided to quit Belmont at the earliest opportunity because they do not believe in trying their luck by the caskets which is the only way of winning Portia

o Nerissa then enquires of Portiarsquos opinion about Bassanio who once visited her in the company of the Marquis of Montferrat and says that she had never come across such an ideal love deserving the fairest lady for his bride

o Portia seems to remember Bassanio quite correctly and says that she agrees with Nerissa At this moment a servant informs Portia that the Prince of Morocco has arrived to try his luck by the caskets

o Portia tells Nerissa that if she could welcome this new suitor as gladly as she says farewell to the previous ones she would be glad of his arrival However if he happens to have the virtues of a saint but the black complexion of a devil she would prefer to have him for religious consolation rather than as a husband

(1) NERISSA You need not fear lady (Line 97-103)

the having any of these lords they have acquainted me with their determinations

which is indeed to return to their home and to

trouble you with no more suit unless you may be wonby some other sort than your fathers imposition depending on the caskets

PORTIA If I live to be as old as Sibylla I will die as chaste asDiana unless I be obtained by the manner of my fatherswill I am glad this parcel of wooers are so reasonablefor there is not one among them but I dote on his veryabsence and I pray God grant them a fair departure

(a) Elucidate the idea expressed in the first speech of the above dialogue

In the first speech Nerissa assures Portia that she need not have any fear of being compelled to marry anyone of the suitors who had lately come to Belmont She informs her that they have all decided to return to their respective countries(b) Illuminate the meaning of the phrase ldquoyour fatherrsquos imposition depending on the casketsrdquo

Nerissa means that the suitors of Portia do not find the conditions imposed by the will of her father to their liking They are too hard for them These conditions are that in the event of a suitor failing to choose the right casket (i) he should never disclose to anybody which casket he chose (ii) he can never marry and (iii) he should take his departure immediately(c) Explain the meaning of the term lsquoSibyllarsquo

lsquoSibyllarsquo is the name given by Romans and Greeks to a prophetess inspired by some deity usually the sun-god Apollo She had a very long life The god Apollo granted her as many years of life as she could hold grains of sand in her hand(d) Elucidate the meaning of the term lsquoDianarsquo

lsquoDianarsquo is the goddess of hunting She is also regarded as a symbol of virginity because she never fell in love and never

married(e) Explain the meaning of the first two lines of Portiarsquos speech

Portia says that even if she is to live for centuries like Sibylla she would not marry except in accordance to her fatherrsquos will She asserts that she would not mind remaining unmarried and untouched by a man like Diana the virgin the goddess of hunting unless a man is able to win her by passing the test laid down by her father

Class XSubject Topic Summary Execution

Hindi 2nd

Langबड घर की बटी( मशी परमचद)

lsquoबड घर की बीटीrsquo कहानी का उददशय मधयम वग की घर समसया को सझा कर सगदिठत परिरवार म मिम जकर परम स रहन का सदश दना ह घर म शानित tानिपत करन की जिजममदारी नारी की होती ह यदिद नारी समझदार ह उसम धय और परिरवार क परनित परम ह तो कोई भी घटना परिरवार को निवघदिटत नही कर सकती या कहानी परिरवार को सगदिठत करत हए परम सौहाद स एक रदसर की भावनाओ को समझ करउनका सहयोग करत हए जीवन यापन करन की पररणा दती ह मशीपरमचदर जी न इस कहानी म सयकत परिरवार का परनितनिनमिधतव निकया ह यह कहानी बनी माधव सिसह जो गौरी पर क जमीदार क उनक दो पतरो की हशरी कठ ा निबहारीशरीकात का निववाह एकजमीदार घरान की पतरी आनदी स हआ थाआनदी न द को ससरा क वातावरण म ढालिया थाएक दिदन आनदी का अपन दवर ा निबहारी स झगडा हो जाता ह दोनो भाई एक रदसर स अग होन की कोलिशश करत हसभी बह आनदी न अपन मधर वयवहार स ा निबहारी को घर छोडकर जान स रोक लिया| इस पर बनी माधव सिसह न कहा निक बड घर की बटी ऐसी ही होती ह जो निबगडा काम बना ती ह अतः शीषक साथक ह बड घर की बटी आनदी ह

helliphelliphelliphellipContinue to nextBiology Topic ndash Chp-1

CellWelcome to new session 2020-21Today we will start with Chpter 1 cell CELL

Protoplasm+Cellmembrane Or Cell wall

Cytoplasm+Neucleus

Cytoplasmic+ CytoplasmicOrganelles Inclutions(mitochondria (food Golgi bodies pigments)Ribosome)

What is cellbull Cell is the structural and functional unit of living organismbull According to number of cells organisms areUnicellular - Amoeba bacteria Multicellular - Rose Mango Tiger HumanSmallest cell -bacteria Longest cell - Nerve cellLargest cell - Ostrich egg cellCells are of different size and shapes according to their functionsQ2Write chief functions of following cellorganelles

Q3What is tonoplastVacuoles covered by a covering called tonoplast

Bengali(2Nd

Language)

ফ ফটক ো ফটক (কলিবতো ) ভোষ মসোপো3 gtPোয়

একটি লেমসয়র ীবস লেপরম লিকভোসব ফসট ওসঠ তো লেদলিসয়স4 কলিব লেপরম Pই য় লেই ময়ই বনত কোস পলিরত য় ফ লেফোটো বো োসফোটো লেটো ব2 কো য় লেমসয়সদর ব gtয13 লেপরসমর 4লিব ফসট উসঠস4 এই কলিবতোয় লেপরম মোষসক মত gtযর মস লেফস লিদসয় পরকষস বাোচোসোর gtয োত বো2োয় কলিবতোয় লেমসয়টির পসব13র দঃসর কো বো সও লেমসয়টি লেই পসর পলিক সত চোয়ো োরী ীবসর কোস4 পরম লেPৌবস লেপরমসক পোবোর পরব ইচছো োকসও তো পসর লেলিতবোচকতোয় পলিরত য় কলিব ভোষ মসোপো3 যোয় লেP ক লেপরসমর

কলিবতোয় ব gtযবহত লিবসষ লিক4 সvর অ13 লেদওয়ো ১) রসবোো= লেP লিবলিভনন রকম ডোকসত পোসর২) ো= পোর ৩) ঠলি = লেচোসর বZ৪)আই বস2ো=অলিববোলিত৫)শইসয় = োলিয়ত কসর৬)োতপাোচ= লিবলিভনন পরকোর৭)দ2োম = v কসর বZ কসর লেদওয়ো৮)লেরলিং =লেোোর দৈতরী লেব2ো৯) বনত= একঋত১০) পাোর = বসকরো2

Organelles Functions

1 Endoplasmic reticulum

2 Mitochondria

3Golgibodies

4 Ribosome5Lysosome

6Plastids

7 Centrosome

i) Supportive framework for the cellii) Synthesis and transpost of proteinsRelease of energy in the form of ATPi) Synthesis and secretion of enzymes hormoneii) Formation of vacuoles lysosomei) Protein Synthesisi) Intracellular digestionii) Destroy foreign substancei )Leucoplast - stores starchii)chloroplast - trap solar energyiii) Chromoplast - imparts colour toflowers amp fruitsi) Initiates and requlates cell division

কলিবতো তোর অ13সক ভোষোয় পরকোো কসর ঘলিরসয় ব যকত কসরস4 লেপরসমর ফতো আর লিবফতো লেক গোঢ় কসর লেদোসো কলিব ভোষ মসোপো3 যোসয়র অলিভবসর অ যলিদক

Economics

Factors of Production

Welcome to the new sessionToday we are going to start the first chapter of Class XThe name of the chapter is Factors of productionBy the name I hope you all can recall a glimpse of what you have learnt in the second chapter of Class IX

NowProduction is the process of creating the various goods and services which are consumed by the people of the country to satisfy their wants

Thus it is the process in which some materials are transformed from one form to another to create utility and value in goods

For example utility can be created by changing the form of a commodity ie

Making of table out of wood by a carpenter for his customer here the wood is getting transformed into table creating utility for his customer and he can also command a price for it

On the other hand Housewives perform very

useful activities at home which create utility but their domestic activities are not included in production because they have no money value

So we can also say that Production denotes two things firstly creation of utility and secondly creation of value

Production is not complete unless it reaches the consumer

An increase in production will increase the economic welfare of the consumers and hence the aim is to raise the production level of the country

Again production of a good or service is only possible if certain resources or

Questions

1 What do you mean by production

Answer Production means the creation of goods and services for the purpose of selling in the market

In fact production involves the transformation of inputs into outputs

Hence production denotes two thingsCreation of utility and creation of valueUtility and value can be created by changing the form by changing the place by changing the time and by rendering services

Example Transformation of raw

materials into finish goods such as potter creates utility by converting mud into utensils assembling of small parts to make bigger machinery

Production also includes services such as distribution and marketing

2 What are the factors of production

Answer Factors of Production refers to the resources and inputs needed for producing goods and servicesThese inputs can be classified as

Land Labour

Capital Enterprise

Land Land is defined to include not only the surface of the earth but also all other free gifts of nature(for example mineral resources forest resources and indeed anything that helps us to carry out the production of goods and services but is provided by

inputs are used together in right proportion

A resource or an input which helps in the process of production to obtain an output is called FACTOR OF PRODUCTION

These factors of production can broadly be categorized into four parts 1LAND 2LABOUR3CAPITAL4ENTERPRISE (ORGANISATION)or Entrepreneur

The above factors are all interdependent on each other and they play a major role in production process

FACTORS OF PRODUCTION

LANDCAPITAL

LABOUR ENTREPRENEUR

nature free of cost)LabourLabour refers to the human efforts that need to be combined with other factors of production for creating an output

CapitalAll man ndash made means of production is called capita example machineries which help in further production Money when used for starting any business for purchasing raw materials machinery tools etc it is regarded as capitalCapital also includes physical capital like factories machineriestoolsbuildingsequipments etcEnterpriseThe task of bearing risks is called enterprise and the person who bears these risks of business is called the entrepreneurThus an entrepreneur is one who organises production takes important decisions regarding production hires and purchases factors of production and bears the risk and uncertainty involved in productionOrganisation refers to the services of an entrepreneur who controls organises and undertakes all risks One who plans organises and manages a business enterprise is an organiser

Physics Chapter 1 Force

Force is an external agent capable of changing the state of rest or motion of a particular body It has a magnitude and a direction The direction towards which the force is applied is known as the direction of the force and the application of force is the point where force is applied The Force can be measured using a spring balance The SI unit of force is Newton (N)

Question 1

State the condition when on applying a force the body has

(a) the translational motion

(b) The rotational motion

Solutions

(a) Translational motion is produced when the body is free to move

(b) Rotational motion is produced when the body is pivoted at a point

Question 2

Define moment of force and state its SI unit

Solutions

The moment of force is equal to the product of the magnitude of the force and the perpendicular distance of the line of action of force from the axis

of rotation

The SI unit of moment of force is Newton times meter

= Newton meter (Nm)

Commercial Studies

Stake holders In this topic you will be come to know about the meaning and concept of stakeholders

How stakeholders are different from shareholders

Questions1 What do you mean by the term stake holdersAnswer) The term stake holders have developed from the words which mean an interest or expected benefit Stakeholders mean all those individuals groups and Institutions which have a state (interest) in the functioning and performance of a commercial organisation or a business enterprise2 What do you mean by share holdersAnswer) The person and Groups who own the shares of the joint stock company by providing capital to the company are called shareholders Shareholders are the internal stakeholders shareholders are one out of several stake holders3 How are shareholders different from stakeholdersAnswer)i) The term shareholders is related to only joint stock company whereas stakeholders are related with all business organisationsii) Stakeholders maybe any individual having financial stake in business organisation whereas a shareholders are those individuals who are holding shares in the company4) How are shareholders different from creditorsAnswer) i) Shareholders are internal stakeholders while creditors are external stakeholdersii) Shareholders invest in the capital of the company whereas creditors give loan to the companyiii) Shareholders are the members of the company with voting rights but creditors are not the members of the company

English 1 Transformation of sentences

Sentences A sentence is a group of words which makes complete sense

e Assertive sentencesf Imperative sentencesg Interrogative sentencesh Exclamatory sentences

Sentences can be changed from one grammatical form to another without changing the meaning of the sentence This is known as transformation of sentences

Exercise 1 Change the following affirmative sentences into Negative sentences

a He is a good manHe is not a bad man

b Ram loves SitaRam is not without love for Sita

c Only he stood first in the classNone but he stood first in the class

d Ankit was wiser than he

He was not so wise as Ankite He did it

He did not fail to do itf As soon as I reached college the

bell rangNo sooner did I reach college than the bell rang

g He finished everythingHe left nothing unfinished

h It always pours when it rainsIt never rains but it pours

Math Topic Commercial MathematicsChapter ndash Goods and services Tax

What is GSTAns It is a abbreviated term of Goods and Service Text which is an indirect tax levied on the sale of goods and rendering servicesSome terms related to GSTDelar Any person who buys goods or services For resale is known as a delar A delar Can be a firm or a companyIntra-state sales Sales of goods and services within the same state or same union territory are called intra- state salesInter-state sales Sales of goods and services outside the state or union territory are called Inter-state sales4) Input GST GST is paid by dealers on purchase of goods and services are called input GST5) Output GST GST is collected from customers on sale of goods and services are called output GST6) Types of GST There are three taxes applicable under GST(i) Central Goods and Services Tax (CGST)(ii) State Goods and Services Tax (SGST) or Union Territory Goods and Services Tax (UTGST) Both these taxes are levied on intra-state sales Here GST is divided equally among central and state governments(iii) Integrated Goods and Services Tax (IGST) IGST is levied on inter- state sales It is also levied on import of goods and services into India and export of goods and services from India

Subject Eng Literature (The Merchant of Venice ndash William Shakespeare)Topic Act III Scene 4 Lines 1 to 44 (Portia hellip To wish it back on you fare you well Jessica)[Students should read the original play and also the paraphrase given in the school prescribed textbook]

Summary Questions amp AnswersIn this scene we suddenly find a new element in the character of Portia We have already seen her possessed of every graceful womanly quality but now she shows that she is capable of rapid decision and determined action She shows this by her sudden resolve to hasten to Venice with a daring scheme for the rescue of Antonio This is an important scene in the dramatic action for it leads up to and renders possible the striking events of the famous trial scene which is one of the greatest striking elements of the play Moreover the fact that all the characters of importance are now assembled together in Venice makes the union of the main plot and the secondary story complete

(1) LORENZO Madam although I speak it in your presence(Line 1-9)

You have a noble and a true conceit

Of god-like amity which appears most strongly

In bearing thus the absence of your lordBut if you knew to whom you show this honourHow true a gentleman you send reliefHow dear a lover of my lord your husbandI know you would be prouder of the workThan customary bounty can enforce you

(a) Where is Lorenzo Why is he here To whom is he referring as lsquoMadamrsquo

Lorenzo is at Portiarsquos residence He had met Salerio on the way and Salerio had begged him to come along with him to

o In this scene Portia Nerissa Lorenzo Jessica and Balthazar appear

o Portia requests Lorenzo and Jessica to be in charge of her house during her absence from Belmont because she and Nerissa have decided to spend the days in meditation and also in visiting the holy places in the neighbourhood of Belmont She has already instructed her people to acknowledge both Lorenzo and Jessica as master and mistress of house during her absence Lorenzo and Jessica gladly agree to look after the house of Portia

handover the letter from Antonio to Bassanio The letter carried the bad news about Antoniorsquos arrest for non-payment of loan taken from Shylock Hence Salerio might have preferred company to break this bad news to Bassanio He is referring to Portia as Madam(b) What does Portia say on hearing the above extract

Portia says that she has never regretted doing good to others Friends who spend a lot of time together and really are there for each other have many traits in common As Antonio is Bassaniorsquos best friend saving him is like saving Bassanio who is like her own soul She asks Lorenzo to take care of management of the house till Bassanio is back(c) What does Portia send with Bassanio and why

On hearing about Antoniorsquos troubles on account of Bassanio her husband Portia immediately sends him with enough gold to repay the debt many times over to Venice to help Antonio out of his misfortune

(2) Lorenzo Madam with all my heart (Line 36-40)

I shall obey you in all fair commands

Portia My people do already know my mindAnd will acknowledge you and JessicaIn place of Lord Bassanio and myselfSo fare you well till we shall meet again

(a) Where are Lorenzo and Portia at this time What lsquofair commandsrsquo are given to Lorenzo

Lorenzo and Portia are at Belmont during this scenePortia reveals to Lorenzo that she has sworn to contemplate in prayer at a monastery around two miles away until her husband returns from Venice She tells him that Nerissa would accompany her and asks him to manage the house with Jessica till things are settled In response Lorenzo tells her that he would be obliged to do whatever she asks him to do(b) Where is Portia actually going and why

Portia tells Lorenzo that she would live a life of contemplation and pray at a monastery which is two miles away from her place In reality Portia plans to go to Venice in disguise with Nerissa and argue the case in defense of Antonio She is very sure that her plan would succeed

ClassXI (ScienceHumanitiesCommerce)Subject Topic Summary Execution

Computer Science

(APC)

Ch ndash 1 Numbers

(Numbers in different bases and

their Arithmatical operations)

Number System In computers Number System is defined as a writing system to represent the numbers in different ways ie we are using different symbols and notations to represent numbers There are four ways we can represent the number ndash Binary Decimal Octal and Hexadecimal

Decimal Number SystemThis number system consist 10 digits These are 0 1 2 3 4 5 6 7 8 amp 9

Binary Number SystemThis number system has only two digits these are 0 and 1 Here 0 stands for off while 1 stands for on

Octal Number SystemThis number system has 8 digits these are 0 1 2 3 4 5 6 amp 7

Hexadecimal Number SystemThis number system has 16 digits these are 0 1 2 3 4 5 6 7 8 9 A B C D E F Here the value of the alphabets are as follows A=10 B=11 C=12 D=13 E=14 F=15

Rules for conversion decimal number to Binary1 Divide the decimal number by 22 If the number will not divide equally by 2 then round down the answer to the nearest whole number (integer)3 Keep a note of the remainder it should be either 0 or 14 Keep repeating the above steps dividing each answer by 2 until you reach zero5 Write out all the remainders from bottom to top This is your binary solution

For example Lets convert 32 to binary 2 32 2 16 - 0 2 8 - 0 2 4 - 0 2 2 - 0 2 1 - 0 0 - 1

The binary equivalent of 3210 is 1000002

Try the follwing youself1 2410

2 4810

3 1210

History GROWTH OF NATIONALISM

The second half of the 19th century witnessed growth of political consciousness and a sense of Nationalism among the IndiansThere were various factors for growth of Indian Nationalism- As a result various political associations were formed in different provinces by the educated Indians Surendranath Banerjee organized a meeting of National conference at Calcutta Ultimately the National Congress was founded in Bombay in 1885This body became the vanguard of Indian struggle for freedom The congress leaders were known as moderates because they followed a policy of prayer and petition A large number of Indian leaders had experienced in political agitation The Political situation of England was also changed Moreover increasing revolutionary activities in Maharashtra Punjab and Bengal became serious concern to the British Government In this

QUESTION1 What do you mean by Nationalism ANSWER 1 Nationalism is defined as loyalty and devotion to own nation especially a sense of national consciousnessQUESTION 2 What are the causes of nationalism ANSWER 2 There were various factors for growth of nationalism

1 Spread of western education2 The progress of vernacular press and

patriotic literature3 The economic exploitation of our

country by the colonial rulers4 International affairs

QUESTION 3 Who organized National conference in Calcutta in 1883 ANSWER 3 Surendranath BanerjeeQUESTION 4 When did Indian National Congress formANSWER 4 Indian National Congress was formed in 1885 in BombayQUESTION 5 Who were ModeratesANSWER 5 The Early Nationalists were also known as Moderates Their emergence marked

background Lord Curzon became Viceroy in India He had no respect for the Indian National Congress

the beginning of the organized national movement in India They believed in British justice and were loyal to them They followed a policy of prayer and petition They demanded constitutional reforms of our country Impotant Moderate leaders were Pherozshah Mehta Dadabhai Naorozi and Surendranath Banerjee etcQUESTION 6 What do you know about Extremism in Indian National movementANSWER 6 In the beginning of 20th century a new class of national leaders emerged in India which was different from the moderate groups They started more aggressive movement against the British empire The goal of extremists was ldquoswarajrdquo Important extremist leaders were Bal Gangadhar Tilak Lala Lajpat Rai Bipin Chandra Pal etcQUESTION 7 Mention the places which were the main centres of Revolutionary movementANSWER 7 Maharashtra Bengal and Punjab

Physics

Chapter Dimensional Analysis

(Summary)

The dimensions of a physical quantity are the powers to which the fundamental units are raised in order to obtain the derived unit of that quantit

The physical quantites lengthmasstime are represented by [L] [M] [T] resp let they are raised to powers ( dimesions) abc resp then any physical quantity can be represented by [ La Mb Tc ] Examples

1 Area area = L x B = [L] x [L] = [M0 L2 T0 ]

2 Density density = massvolume = [M][L3] = [ M L-3]

3 Velocity velocity = distancetime = [L][T] = [LT-1]HW Try to find out dimension of acceleration Acceleration = velocity timeNB One can find the SI Units Using Dimension Analysis Such as for area we have [L2] so its SI unit is m2

Biology Topic ndash Chp-1 The living world

Today we will start the first chapter the living world Here we discuss about the characteristics of living organism and what are the difference between them and nonliving substances We also discuss about the contribution of different Scientists

There are over 500000 species of plants andover a million species of animal are present on earth Some 15000 new species were discovered every yearQ1 What is a living organismbull A living organism is primarily physico -chemical material that demonstrate a high degree of complexity is capable of selfRegulation possesses a metabolism and perpetuates itself through timeQ2 What are the differences between livingand non-livingsi) Compared with non-living living organisms

have more complex organised structure and their use of energy is more controlled amp efficientii) Living things reproduce their own kind by forming new cells which contains copies of their genesiii) Each organism has some degree of homeostasisie it is able to make adjustments so that internal environment remains constantQ3 Write contributions of following Scientists i) Aristotle - One of the first theories in Biology places all living things in a hiearchieii) AV Leeuwenhoek - was the first to observe living single celled organisms under microscopeii) Carolus Linnaeus - developed the binary system for naming of organisms and classificationiii) Geregor Johann Mendel ndash discoverbasic principles of inheritanceHomework i) C Darwin ii)Schleiden

Math Trigonometric functions

1 Overviewi) Trigonometry The word lsquotrigonometryrsquo is derived from the Greek words lsquotrigonrsquo and lsquometronrsquo which means measuring the sides of a triangle An angle is the amount of rotation of a revolving line with respect to a fixed line Usually we follow two types of conventions for measuring angles ie a) Sexagesimal system b) Circular system In Sexagesimal system the unit of measurement is Degree In Circular system the unit of measurement is Radian ii) Relation between degree and radianThe ratio of circumference of a circle to its diameter is always a constant This constant ratio is a number denoted by π which is taken approximately as 227The relationship between degree amp radian measurements is as follows2 right angles = 180deg= π radians1radian = 180degπ=57deg16(approx) 1deg=π180 radianiii) Length of an arc of a circleIf an arc of length s subtends an angle θ radians at the center of a circle of radius r then s=rθiv) Area of a sector of a circleA sector is like a pizza slice of the

Q) Express the following angles in radiana) 45deg b) 40deg3730Ans a) We have 180deg=π radiansi e 45deg= πtimes45180 radian = π4 radiansb) 40deg3730= 40deg37+3060 minute= 40deg 37 +12 minute= 40deg+ 752 minute=40 + 75(2times60) degree=3258 degreeNow 180deg=π radianie 3258 degree= (πtimes325) (180times8) radians = 65π288 radiansQ) A circle has a radius of r=12 meters What is the length of an arc traced out by a 60deg angle in the center of the circleAns In this problem we know both the central angle (60deg) and the radius of the circle (12) All we have to do is plug those values into our equation and we get

s = 2π(12)(60360)s = 24π6s = 4πSo the length of an arc traced out by a 60deg angle in a circle with a radius of 12 meters equals 4π meters asymp 1257 metersQ) Find the area of the sector with a central angle 30deg and a radius of 9cmAns GivenRadius r = 9 cmAngle θ = 30degArea of the sector = θ360degtimesπr2

= 30360degtimes227times92=2121cm2

circle It consists of a region bounded by two radii and an arc lying between the radiiThe area of a sector is a fraction of the area of the circle This area is proportional to the central angle In other words the bigger the central angle the larger is the area of the sectorArea of Sector = θ2 times r2 (when θ is in radians)

Area of Sector = θ times π360 times r2 (when θ is in degrees)

COMMERCE

CLASSIFICTION OF HUMAN ACTIVITIES-ECONOMIC AND NON-ECONOMIC

Welcome to the new sessiontoday we are going to start the first chapter of Class XI The name of the chapter that we are going to start is

lsquoClassification of Human Activities ndasheconomic and non-economicrsquo

Now let us start the chapter by considering human beings and the activities they perform throughout the day

Human activities means all those activities that human beings undertake to satisfy their wants

Human wants on the other hand are the desire of human beings for goods (vegetables fruits rice etc) and services (services of doctors teachers lawyers etc) that they require to live

Now these human activities continue throughout life as human wants are unending unlimited and recurring as human beings desire for better living throughout their lives

Now human activities can be classified into two categories

Human activities

Economic activities Non-economic activities

Economic activities are

Questions1 What are human activities

Answer Human activities mean all those activities that human beings undertake to satisfy their wants

Example A man working in an office

A boy playing in the garden

2What are the characteristics of human activitiesAnswer the characteristics of human activities are as follows

Human activities are undertaken by men women and children and these activities involve human efforts

Human activities are undertaken to satisfy human wants which are unlimited

Human activities continue throughout life

Human activities are performed for both earning money and personal satisfaction

3What is economic activitiesGive example

Answer Economic activities are undertaken by human beings with the object of earning money acquiring wealth and thereby satisfying human wantsExample

Selling of goods by a shop keeper to his customer

A clinic run by a doctor Service of a teacher in school or college

undertaken by human beings with the object of earning money and acquiring wealth

These activities result in the production of economic goods and services

Example Human activities(ie working in factories officesshops) which produce direct economic benefits

Non-economic activities are inspired by human sentiments and emotions such as love for the family desire to help the poor and love for the country

Thus these human activities (eg praying playing sleeping) produce no direct economic benefits and they are also not related to earning money and acquiring wealth

4 What are the characteristics of economic activities

Answer The characteristics of economic activities are as follows

Economic motiveEconomic activities are undertaken to earn money and acquire wealth

ProductiveEconomic activities involve productiondistribution and exchange of goods and services to create wealth

Economic growthEconomic activities determine the level of economic development of a country and standard of living of its citizens

Socially desirableEconomic activities are socially desirable for society

Economic resourcesEconomic activities make use of all the economic resources such landlabourcapital etc

5 What do you mean by non-economic activitiesExampleAnswerNon-economic activities are inspired by human sentiments and emotions such as love for the family desire to help the poor and love for the countryThese activities are not undertaken for monetary gain but for onersquos satisfaction and happinessExample

a mother looks after her children

a student donates blood8 Differentiate between Economic activities and Non-economic activities

Economic activities

Non-economic activities

1to earn living and acquiring wealth2Result can be measured in terms of money

3ExampleBusinessprofession and employment

1 to obtain some satisfaction

2Result cannot be measured in terms of money

3ExampleFamily-orientedreligious socialCultural and national

BUSINESS STUDIES

BUSINESS ENVIRONMENT

Welcome to the new sessionToday we are going to start the first chapter and the name of the chapter is Business Environment

In todayrsquos world every business enterprise is a part of the society It exists and operates in association with various groups in society such as customers suppliers competitors banks and financial institutions government agencies trade unions media and so on All these groups influence the functioning of business in one way or the other They constitute the environment of businessConcept of Business Environment

The term lsquobusiness environmentrsquo refers to the sum total of all individuals institutions and other forces that lie outside a business enterprise but that may influence its functioning and performance

The main features of business environment

Totality of External forces General and Specific forces Interrelatedness Complexity Dynamic Uncertainty Relativity

The Interrelation between business and its environment

The business enterprise is an open system It continuously interacts with its environment It takes inputs

Prepare the following questions from todayrsquos assignment

1 What do you mean by business environment

The term lsquobusiness environmentrsquo means the aggregate of all forces factors and institutions which are external to and beyond the control of an individual business enterprise but they may influence its functioning and performance Business environment is the macro framework within which a business firm a micro unit operates It consists of several interrelated and interacting elements

2 Explain the main features of business environment in brief

Totality of External forces-Business environment is the sum total of all things external to a business environment

General and Specific forces-It includes both the forces general forces are the economic social political legal and technological conditions which indirectly influence all business enterprise Specific forces are the investors customers competitors and suppliers which influence individual enterprise directly

Interrelatedness-Different elements of environment are interrelated for an example growing awareness for health care has increased the demand for health foods

Complexity- Business environment id

(such as raw materials capital labour energy and so on) from its environment transforms them into goods and services and sends them back to the environment

Fig 1 Business Environment Relationship

complex in nature as the elements keep on changing example economic technological and other forces changes in demand for a product and service

Dynamic-Business environment is not static it keeps on changing

Uncertainty- Itrsquos very difficult to predict future events such as technology and fashion which occur fast and frequently

Economics Basic Economic ConceptsSub topic

Microeconomics and

Macroeconomics

Welcome to the new sessiontoday we are going to start the first chapter of Class XI The name of the chapter that we are going to start is Basic Economic concepts

Now Economics covers the study of human activities Human activities are those activities which are performed by humans to satisfy their wants

Thus Human wants are unlimited and therefore economic activities such as production exchange and consumption are needed in order to satisfy those wants

The study of economics is divided largely in two parts which areMicroeconomics and Macroeconomics

SUBJECT- MATTER OF ECONOMICS

MICROECONOMICS MACROECONOMICS

Questions1Who has coined the words micro and macro economics

Answer Ranger Frisch coined the words lsquomicrorsquo and lsquomacrorsquo in 1933 to denote the two branches of economic theory namely microeconomics and macroeconomics

2What is microeconomicsAnswer It is the study of behaviour of individual decision ndash making unit such as consumers firms etc

3 What is macroeconomicsAnswer Macroeonomics is the study of overall economic phenomena like employment national income etc

4 What is the importance of microeconomicsAnswer

Microeconomics helps in formulating economic policies which enhance productive efficiency and results in greater social welfare

It helps the government in formulating correct price policies

It explains the working of a capitalistic economy where individual units(producers and consumers ) are free to take their own decision

Micro means a small part in

microeconomics we do not study the whole economy Hence we study an individual consumer and his or her choices and a producer and his or her profit maximizing decisions in the market Thus it does not mirror what happens in the economy as a whole

Macroeconomics on the other hand studies the economy as a whole It is concerned with aggregate and depicts the entire picture of the economyMacroeconomics deals with the national income aggregate investment aggregate consumption etc

Features of Microeconomics It deals with small

parts of the country Hence it looks at

individual consumers firms and industries

It deals with individual income consumption and savings

It studies the determination of price of any product or factors of production

It deals with the working of market via the price mechanism which is nothing but the determination of price and quantity of a commodity by the forces of demand and supply

Features of Macroeconomics

It deals with the study of the economy as a whole

It is concerned with

5 Give a limitation of microeconomics Microeconomics fails to explain the

functioning of an economy as a whole It cannot explain unemployment illiteracy and other problems prevailing in the country

6 What is the importance of macroeconomics It gives overall view of the growing

complexities of an economic system It provides the basic and logical

framework for formulating appropriate macroeconomic policies (eg for inflation poverty etc )to direct and regulate economy towards desirable goals

7What is the limitation of macroeconomics It ignores structural changes in an

individual unit of the aggregate

8 Differentiate between Microeconomics and Macroeconomics

Microeconomics Macroeconomics

the study of aggregates

National income aggregate savings and aggregate investments are major concepts dealt within macroeconomics style

It studies the determination of general price levels

It investigates into the problem of unemployment and the achievement of employment

It studies the aspect of decision making at the aggregate and national levels

It includes all growth theories whether related to developed or developing economies it also includes the study of economic systems and the working of the economy under different systems

Note Both Micro and macro economics are complementary and should be fully utilized for proper understanding of an economy

1It studies economic aspect of an individual unit2It deals with individual incomeConsumption and savings

3 It facilitates determination of price of any product or factors of production

4 Itrsquos scope is narrow and restricted to individual unit

1It studies the economy as a whole

2It deals with the national income aggregate consumption and aggregate savings3 It facilitates determination of general price level in an economy

4 Itrsquos scope is wide as it deals with economic units on the national level

ACCOUNTS

Introduction to Accounting and Book-keeping

Today I am going to share you the meaning of Accounting and Book-keeping and its related terms bullAccounting bullBook Keeping bullAccountsbullTypes Of Accounts bullAccounting Cycle

bull Meaning of accounting

Ans ) Accounting is the art and science of recording classifying and summarising monetary transactions

bull Meaning of Book-keeping

Ans) Bookkeeping is the art of recording business transactions with the view of having a permanent record of them and showing their effect on wealth

bull Meaning of account

Ans) The term account means a record of

business transactions concern a particular person of firm asset or income or expense It is a summarised record of all transactions which take place in an accounting year

bull Types of accountsPersonal accounts ndash Personal accounts relating

to person and Organisation are known as personal accounts Example Ramrsquos Account ABC amp Co Account etc

Real account - The accounts related to tangible and intangible assets are called real accounts Example Cash Account Furniture Account etc

Nominal account- Accounts related to expenses losses incomes and gains are known as nominal accounts Example Wages Account Salary Account Discount Account etc

bull Accounting cycle Accounting cycle refers to a complete sequence of accounting activities It begins with recording of transactions and ends with the preparation of a balance sheet

Chemistry TopicAtomic Structure

Thomsonrsquos atomic modelThomson (1898) was the first to propose the model of an atomHe proposed that an atom can be regarded as a uniform sphere of positive electricity in which requisite number of electrons are embedded evently to neutralize the positive chargeThis is just like plums embedded in a pudding or seeds evently distributed in red spongy mass of a watermelonThis model of atom is known as ldquoPlum-Pudding modelrdquo or

Q1)What is the fundamental constituents of atomAns Electron Proton and neutrons are the fundamental constituents of atomQ2)What is the value of fundamental unit of electricityAnsThe charge carried by one electron is sad to be the fundamental unit of electricityIts magnitude is 48times10-10esuOr 1602times10-19C Q3)Name the element containing no neutronAnsOrdinary hydrogen atom or protium 1H1

Types of AccountPersonal AccountReal AccountNominal AccountBalance Sheet (opening)

ldquowatermelon modelrdquoThis model could explain the electrical neutrality of an atom but failed to explain the result of scattering experiment carried out by Rutherford in 1911So it was rejected ultimately

Q4)Why is an electron called universal particleAns Itrsquos mass and Charge are independent of its source

EVS Chapter 1 ndash Modes of Existence

Modes of existence When one speaks normally about the mode of existence of some group or individual one refers to their customs their mode of being their ethology their habitat in some way their feeling for a placeDifferent modes of exixtence are ndash

1 Hunting ndashGathering2 Pastoral3 Agricultural4 Industrial

1 Hunting and gathering Hunting and gathering mode of existence is characterized by obtaining food from hunting wild animals including fishing and gathering wild plants From their earliest days the hunter-gatherer diet included various grasses tubers fruits seeds and nuts Lacking the means to kill larger animals they procured meat from smaller game or through scavenging

Societies that rely primarily or exclusively on hunting wild animals fishing and gathering wild fruits berries nuts and vegetables to support their diet are called hunting and gathering societies

At least this used to be practice of human beings before agriculture is invented As their brains evolved hominids developed more intricate knowledge of edible plant life and growth cycles

Q) Write the features of Hunting ndash gathering societiesAns - There are five basic characteristics of hunting and gathering societies

i The primary institution is the family which decides how food is to be shared and how children are to be socialized and which provides for the protection of its members

ii They tend to be small with fewer than fifty members

iii They tend to be nomadic moving to new areas when the current food supply in a given area has been exhausted

iv Members display a high level of interdependence

v Labor division is based on sex men hunt and women gather

Political Science

Introduction to political science

Political science occasionally called politology is a social science which deals with systems of governance and the analysis of political activities political thoughts associated constitutions and political behaviorThe study of political science involves the study of both the

Answer the following questions-1 What is political science

Political science occasionally called politology is a social science which deals with systems of governance and the analysis of political activities political thoughts associated constitutions and political behavior

2 Short notes-

traditional and modern theories of politicsTraditionalClassical political sciencepolitical theory-Traditional political science is the study of politics before Second World War The methodology to study Politics was traditional (legal formaletc) the definition of politics traditional (Politics begins and end with state)area of study (constitution state machinery)was traditionalModern Political scienceModern political theory-Modern Political Theory critically examines the contemporary state of political theory making an assessment of the achievement and limitations of the Behavioural Revolution in its totality and reviews objectively the major paradigms and conceptual frameworks adopted by the disciplineContemporary attempts at the development of an integrated political theory involving the use of both traditional and modern concepts approaches and theories-Around late 1960s several political scientists realized the importance of both the traditional political theory and modern Political theory They began building an integrated theory of politics involving a systematic mixture of traditional and modern studies of politics It was held that the study of a complex and vast field like politics needs both traditional as well as

Classical political theory Modern Political theory

Homework-Learn

modern concepts and approaches for studying itrsquos all aspects

Subject Eng Literature (The Tempest ndash William Shakespeare) Topic Act I Scene 1 Lines 1 to 32 (Line 32 ndash Gonzalo hellip If he be not born to be hanged our case is miserable) Date 13th April 2020 (3rd Period)

[Students should read the original play and also the paraphrase given in the school prescribed textbook]Summary Questions amp Answers

[SUMMARY OF THE ENTIRE SCENE]

o The play starts with the scene of a severe storm at sea Alonso (King of Naples) Sebastian (Alonsorsquos brother) Ferdinand (Alonsorsquos son) Gonzalo Antonio (the usurping Duke of Milan) are in a ship in the midst of the storm

o The mariners are trying their best to control the vessel from running aground and are totally following the orders of their Master the Boatswain They have scant success

o The mariners become extremely unhappy and annoyed when most of the passengers arrive on the deck thereby hampering their effort to save the ship There is serious confrontation between them and the passengers who are part of the Kingrsquos entourage

o The mariners could not save the ship

SUMMING-UP

(i) Vivid description of the scene which gives a realistic description of terror and confusion of a tropical storm

(ii) Shows Shakespearersquos accuracy of knowledge in describing the naval operations and also matters of seamanship

(iii) The opening scene justifies the title ndash The Tempest

UNANSWERED QUESTIONS

(i) The King always travels with his entire fleet including his soldiers Where

(1) GONZALO Nay good be patient (Line 15-26)BOATSWAIN When the sea is Hence What cares these

roarers for the name of the king To cabin silence Trouble us not

GONZALO Good yet remember whom thou has aboardBOATSWAIN None that I more love than myself You are a

councillor if you can command these elements to silence and work

the peace of the present we will not hand a rope more use your authority If you cannot give thanks you have

lived so long and make yourself ready in your cabin for the mischance of the hour if it so hap [To the Mariners]

Cheerly good hearts [To Gonzalo] Out of our way I say

(a) To whom is the boatswain speaking What does he mean by lsquoNone that I more love than myselfrsquo

The Boatswain is speaking to Gonzalo the honest old councilor of the Duke of MilanBy using the words ndash lsquoNone that I love more than I love myselfrsquo means that for the Boatswain nobody is dearer to him than his own life

(b) What were the conditions that made the boatswain react in this way

The Boatswain reacts in this way because the storm is at sea and Alonso King of Naples Sebastian his brother Ferdinand his son Gonzalo Antonio the usurping Duke of Milan on board are in distress and in panic Thus they have rushed to the deck interrupting the work of the mariners

(c) What hope does Gonzalo take from the attitude of the boatswain

The insolent and authoritative attitude of Boatswain makes Gonzalo feel comforted He tells that there are no signs that the Boatswain will be drowned But his facial appearance and attitude shows that he is destined to die on land by hanging which in effect means that all on board will be saved Otherwise all the persons on board are doomed

(d) How can they lsquomake yourself ready in your cabinrsquo For what were they asked to make ready themselves

In order to make themselves ready in their cabin the

were the other ships

(ii) Why was the ship in that area Where was it coming from or going where

(iii) The ship broke apart What happened to those who were in the ship

passengers on board must prepare for death which they will possibly soon have to meetThey can retire to their cabins and offer prayers to the Almighty to save them from drowning

(e) What does the boatswain say when he is asked to be patient What does he order to the royal party

When the boatswain is asked to be patient and remain calm he says that he will be patient only when the storm will be over and the sea will be calm but as long as the storm blows and there is danger to the ship he cannot think of being patient He orders the royal party to go to the cabin and leave the mariners to their work

(2) GONZALO I have great comfort from this fellow (Line 27-36)

Methinks he hath no drowning mark upon him his complexion is perfect

gallows Stand fast good Fate to his hanging Make the rope of his destiny our cable for our own doth little advantage If he be not born to be hanged our case is miserable

(a) Why does Gonzalo regard the Boatswain in the midst of danger

In the midst of danger Gonzalo regards the boatswain because he feels that the Boatswain is a source of comfort and is bent upon to do his work sincerely which in this case is saving the ship and its passengers from the severest of raging storm

(b) What reasons does Gonzalo give when he says that none in the ship will die of drowning

Gonzalo is almost sure that none in the ship will die by drowning His says that there is no mark on the face of the boatswain that indicates that he will die by drowning On the other hand the lines on his face are strong indications that he will be hanged to death Therefore there shall be no danger of the shiprsquos sinking

(c) Explain the following ldquoStand fast good Fate to his hanging Make the rope of his destiny our cable for our own doth little advantage If he be not born to be hanged our case is miserablerdquo

The stated lines mean that if the will of destiny is to be carried out then the ship will not get wrecked and all the passengers will be saved The safety of the passengers therefore depends upon the will of fate being carried out in the case of the boatswain If however the boatswain is not to die by hanging then the passengers are also very unsafe because in that case the ship is likely to sink

(d) What order does the Boatswain give to the sailors

when he re-enters What does he say about the crying of the fellows inside the cabin

The boatswain orders the sailors to bring the topmast lower and bring the ship close to a stationary position with the help of the main sail He says that the fellows inside the cabin are moaning and crying in their distress louder than his voice and louder even than the roaring of the storm

Class XII (ScienceCommerceHumanities) Subject Topic Summary Execution

Computer Science

PropositionalLogic

Propositional logic is a procedure to provide reasoning through statementProposition A ststement that results in True or False is said to be proposition There are two types of propositionSimple proposition amp compound propositionSimple proposioton A simple proposition is one that is not a part of any other proposition Such sentential form of proposition is symbolized with english letters in short For example Ram is a claver student (TrueFalse)Where do you live (Not in True or False)Grapes are sweet (TrueFalse)It rains today (TrueFalse)Here we can see some statements anwer would be true or false but some staements answer can not give in terms of true or false Thus the sentences which can be answered in true or false are known as simple propositionAssigning propositon to a variableThe general syntax to assign propostion to a variable is as followsVariable = Simple propositonFor example A=Ram is a clever studentB= Grapes are sweetC= it rains todayCompound proposition

helliphellipto be continued in next classhelliphellipMath Relation Relation If A and B are two non-empty sets

then a relation R from A to B is a subset of AxB If R A x B and (a b) R then we say that a sube isinis related to b by the relation R written as aRbeg Let A be the set of students of class XII and B be the set of students of class XI Then some of the examples of relation from A to B arei) (a b) AXB a is brother of bisinii) (a b) AXB age of a is more than age of isinb Types of relation In this section we would like to study different types of relations We know that a relation in a set A is a subset of A times A Thus the empty set φ and A times A are two extreme relations For illustration consider a relation R in the set A = 1 2 3 4 given by R = (a b) a ndash b = 10 This is the empty set as no pair (a b) satisfies the condition a ndash b = 10 Similarly R = (a b) | a ndash b | ge 0 is the whole primeset A times A as all pairs (a b) in A times A satisfy | a ndash

Example 1 Let A be the set of all students of a boys school Show that the relation R in A given by R = (a b) a is sister of b is the empty relation and R = (a b) the primedifference between heights of a and b is less than 3 meters is the universal relationSolution Since the school is boys school no student of the school can be sister of any student of the school Hence R = φ showing that R is the empty relation It is also obvious that the difference between heights of any two students of the school has to be less than 3 meters This shows that R = A times A is primethe universal relation Example 2 Show that the relation R in the set 1 2 3 given by R = (1 1) (2 2) (3 3) (1 2) (2 3) is reflexive

b | ge 0 These two extreme examples lead us to the following definitionsDefinition 1 A relation R in a set A is called empty relation if no element of A isrelated to any element of A ie R = φ A times AsubDefinition 2 A relation R in a set A is called universal relation if each element of A is related to every element of A ie R = A times A Both the empty relation and the universal relation are some times called trivial relation Definition 3 A relation R in a set A is called(i) reflexive if (a a) R for every a Aisin isin(ii) symmetric if (a1 a2) R implies that (aisin 2a1)

R for all aisin 1 a2 Aisin(iii) transitive if (a1 a2) R and (aisin 2 a3) R isinimplies that (a1 a3) R for all aisin 1 a2 a3 AisinDefinition 4 A relation R in a set A is said to be an equivalence relation if R is reflexive symmetric and transitive

but neither symmetric nor transitiveSolution R is reflexive since (1 1) (2 2) and (3 3) lie in R Also R is not symmetric as (1 2) R but (2 1) isin notinR Similarly R is not transitive as (1 2) R and (2 3) R but (1 3) R isin isin notinExample 3 Show that the relation R in the set Z of integers given byR = (a b) 2 divides a ndash b is an equivalence relationSolution R is reflexive as 2 divides (a ndash a) for all a Z isinFurther if (a b) R then 2 divides a isinndash b Therefore 2 divides b ndash a Hence (b a) R which shows that R is isinsymmetric Similarly if (a b) R and (b c) R isin isinthen a ndash b and b ndash c are divisible by 2 Now a ndash c = (a ndash b) + (b ndash c) is even (Why) So (a ndash c) is divisible by 2 This shows that R is transitive Thus R is an equivalence relation in ZExample 4 Let L be the set of all lines in a plane and R be the relation in L defined as R = (L1 L2) L1 is perpendicular to L2 Show that R is symmetric but neither reflexive nor transitiveSolution R is not reflexive as a line L1 can not be perpendicular to itself ie (L1 L1) R notinR is symmetric as (L1 L2) Risin

L1 is perpendicular to L2rArr L2 is perpendicular to L1rArr (L2 L1) RrArr isin

R is not transitive Indeed if L1 is perpendicular to L2 and L2 is perpendicular to L3 then L1 can never be perpendicular to L3 In fact L1 is parallel to L3 ie (L1 L2) R isin(L2 L3) R but (L1 L3) Risin notin

Chemistry Solid state Characteristics if Solids(i)The particles are locked in fixed positions they are unable to change their relative positions and this brings a definite shape and volume of a solid(ii)In a solid the constituent particles are held by strong forces of attractionThe forces of attraction may be bonding or non bonding(iii)The constituent particles in a solid pack together as closely as possibleoccupying most of the available space within the solidThus the empty space in a solid is very smallThis makes a solid highly rigid and nearly incompressibleThis also explains why a solid has high density and exhibits slow diffusionClassification of Solids

Q1)Define Crystalline solids AnsA Solid that has a definite geometrical shape and a sharp melting pointand whose constituent particles (atomsmolecules or ions) are arranged in a long range order of definite pattern extending throughout the solid is called a crystalline solidExNaClQ2)Define Amorphous solids AnsA solid that does not have a definite shape and a sharp melting pointand whose constituent particles (atomsmolecules or ions) are not arranged in a definite pattern is called an amorphoussolid

Crystalline solidsAmorphous solids

ExGlassRubberQ3)Classify Crystalline Solids Crystalline Solids

Physics Coloumbrsquos Law (Summary)

Before Going Into Coloumbrsquos Law We Will First Learn What is Charge Properties of Charge and Always remember that charge is quantized ie a body always have static charge of magnitude equal to some integral multiple of fundamental electronic charge e= 16 x 10- 19 C

Charge is the property of matter that causes it to produce and experience electrical and magnetic effects The study of the electrical charges at rest is called electrostatics When both electrical and magnetic effects are present the interaction between charges is referred to as electromagnetic

There exist two types of charges in nature positive and negative Like charges repel and unlike charges attract each other

The type of charge on an electron is negative The charge of a proton is the same as that of an electron but with a positive sign In an atom the number of electrons and the number of protons are equal The atom is therefore electrically neutral If one or more electrons are added to it it becomes negatively charged and is designated as negative ion However if one or more electrons are removed from an atom it becomes positively charged and is called a positive ion

The excess or deficiency of electrons in a body gives the concept of charge If there is an excess of electrons in a body it is negatively charged And if there is deficiency of electrons the body becomes positively charged Whenever addition or removal of electrons takes places the body acquires a charge

The SI Unit of charge is coulomb (C) In SI units the current is a fundamental quantity having a unit of ampere (A) The unit of charge is defined in terms of the unit of current Thus one coulomb is the charge transferred in one second across the section of a wire carrying a

Ionic SolidsMetallicSolids

Molecular Solids

current of one ampere

As q = It we have1 C = (1 A) (1 s)

The dimensions of charge are [A T]

Properties of Charge

(1) Quantization of Charge Electric charge can have only discrete values rather than any value That is charge is quantized The smallest discrete value of charge that can exist in nature is the charge on an electron given as

e = plusmn 16 x 10- 19 C

This is the charge attained by an electron and a protonA charge q must be an integral multiple of this basic unit That is

Q = plusmn ne where n = 1 2 hellip

Charge on a body can never be (frac12)e (23)e or 57e etcWhen we rub a glass rod with silk some electrons are transferred from the rod to the silk The rod becomes positively charged The silk becomes negatively charged The coulomb is a very large amount of charge A typical charge acquired by a rubbed body is 10 - 8 C

Biology Reproduction in organisms

Welcome to this new session 2020-21Today in this first chapter we mainly discuss about reproduction types needs and life span of some organismsWe also discuss about difference between sexual and asexual reproduction

Q1 What is reproductionReproduction is defined as a biological processin which an organism gives rise to young onessimilar to itselfQ2 What are the needs of reproductionbulli) Reproduction maintain life on earthii) It enables the continuity of the species generation after generationiii) It creates genetic variation among populationsQ3 Define Life span and write some orgnisms life spanbull Life span is the period from birth to

the natural death of an organism- OrganismsLife span1 Butterfly 1 - 2 weeks2 Fruit fly 30 days3Dog 10-13 years4 Rose5-7 years5 Tortoise100-150 years6 Banyan Tree -200 - 250 yearsQ4 Reproduction is of two types in case ofanimals but in case of plants vegetative propagation is also present

Asexual Reproduction Sexual Reproductioni) Always uniparentalii) Gametes are not involvediii) Only mitotic division involvediv) Somatic cells of parents are involvedv) Offsprings are genetically similar to the parents

i) Usually biparentalii) Gametes are involvediii) Meiosis occurs during gametogenesis Mitosis occurs after fertilisationiv) Germ cells of the parents are involvedv) offsprings are genetically different from the parents

COMMERCE BUSINESS ENVIRONMENT

Welcome to the new sessiontoday we are going to start the first chapter of Class XII The name of the chapter is Business Environment

Already many of you have got some idea about the word business environment form the first chapter of business studies in class XI

In todayrsquos world every business enterprise is a part of the society It exists and operates in association with various groups in society such as customers suppliers competitors banks and financial institutions government agencies trade unions media and so on All these groups influence the functioning of business in one way or the other They constitute the environment of businessConcept of Business Environment

The term lsquobusiness environmentrsquo refers to the sum total of all individuals institutions and other forces that lie outside a business enterprise but that may influence its functioning and performance

The main features of business environment Totality of External forces General and Specific forces Interrelatedness Complexity Dynamic Uncertainty

Prepare the following questions from todayrsquos assignment

2 What do you mean by business environment

The term lsquobusiness environmentrsquo means the aggregate of all forces factors and institutions which are external to and beyond the control of an individual business enterprise but they may influence its functioning and performance Business environment is the macro framework within which a business firm a micro unit operates It consists of several interrelated and interacting elements

2 Explain the main features of business environment in brief

Totality of External forces-Business environment is the sum total of all things external to a business environment

General and Specific forces-It

Relativity

The Interrelation between business and its environment

The business enterprise is an open system It continuously interacts with its environment It takes inputs (such as raw materials capital labour energy and so on) from its environment transforms them into goods and services and sends them back to the environment

Fig 1 Business Environment Relationship

includes both the forces general forces are the economic social political legal and technological conditions which indirectly influence all business enterprise Specific forces are the investors customers competitors and suppliers which influence individual enterprise directly

Interrelatedness-Different elements of environment are interrelated for an example growing awareness for health care has increased the demand for health foods

Complexity- Business environment id complex in nature as the elements keep on changing example economic technological and other forces changes in demand for a product and service

Dynamic-Business environment is not static it keeps on changing

Uncertainty- Itrsquos very difficult to predict future events such as technology and fashion which occur fast and frequently

Business Studies

Human Resources Management

Human resource of an organisation are the aggregate of knowledge skills attitudes of people working in it

The management system which deals with human resources is called human resource management

Features of HRMbullComprehensive functionbullPeople-oriented

Question1) What do you mean by human

resource management Answer) Human resource management may be defined as that field of Management which has to do with planning organising and controlling the functions of procuring developing maintaining and utilising the labour force

bullAction oriented bullPervasive function bullContinuous function

2) Explain the features of HRM in brief

Answer)bullHuman Resource Management is concerned with managing people at work bull Human Resource Management is concerned with employees which bring people and organisations together so that the goals of each are met bullHuman resource management considered every employees as an individual and also promote their satisfaction and growth bull Human resource management is inherent in all organisations and at all levelsbullManagement of human resources are ongoing on never ending process which requires a constant alertness and Awareness of human relations

3) ldquoHR function is said to be pervasiverdquowhy

Answer) Human resource management is required in all organisations whether it is private or government organisations armed forces sports organisations etc It permeatsall the functional areas like production marketing finance research etc This from this feature of human resource management it can be said that it is pervasive in nature

Economics Demand Q1DEFINITION OF DEMANDIn economics demand is the quantity of a good that consumers are willing and able to purchase at various prices during a given period of timeQ2DEMAND CURVEIn economics a demand curve is a graph depicting the relationship between the price of a certain commodity and the quantity of that commodity that is demanded at that pricQ3LAW OF DEMANDIn microeconomics the law of demand states that conditional on all else being equal as the price of a good increases quantity demanded decreases conversely as the price of a good decreases quantity demanded increasesQ4ASSUMPTION of LAW OF DEMAND(i)No change in price of related commodities(ii) No change in income of the consumer(iii) No change in taste and preferences customs habit and fashion of the consumer( No expectation regarding future change in priceQ5MARKET DEMAND SCHEDULEIn economics a market demand schedule is a tabulation of the quantity of a good that all consumers in a market will purchase at a

given price At any given price the corresponding value on the demand schedule is the sum of all consumersrsquo quantities demanded at that priceQ6INDIVIDUAL DEMAND SCHEDULEIndividual demand schedule refers to a tabular statement showing various quantities of a commodity that a consumer is willing to buy at various levels of price during a given period of timeQ7 FACTORS AFFECTING INDIVIDUAL DEMAND FOR A COMMODITY

The factors that influence a consumerrsquos decision to purchase a commodity are also known as determinants of demand The following factors affect the individual demand for a commodity1 price of the commodity2 price of related goods3 income of buyer of the commodity4 tastes and preferences of the buyer1 Price of the CommodityYou must have observed that when price of a commodity falls you tend to buy more of it and when its price rises you tend to buy less of it when all other factors remain constant (lsquoother things remaining the samersquo) In other words other things remaining the same there is an inverse relationship between the price of a commodity and its quantity demanded by its buyers This statement is in accordance with law of demand which you will study in the later part of this lesson Price of a commodity and its quantity demanded by its buyers are inversely related only when lsquoother things remain the samersquo So lsquoother things remaining the samersquo is an assumption when we study the effect of changes in the price of a commodity on its quantity demanded2 Price of Related goodsA consumer may demand a particular good But while buying that good heshe also asks the price of its related goods Related goods can be of two types-(i) Substitute goods(ii) Complementary goods While purchasing a good prices of its substitutes and complements do affect its quantity purchased(i) Price of Substitute Goods Substitute goods are those goods which can easily be used in place of one another for satisfaction of a particular want like tea and coffee An increase in price of substitute good leads to an increase in demand for the given commodity and a decrease in price of substitute good leads to a decrease in demand for the given commodity It means demand for a given commodity is directly affected by change in price of substitute goods For example if price of coffee increases the demand for tea will rise as tea will become relatively cheaper in comparison to coffee(ii) Price of Complementary goods Complementary goods are those goods which are used together to satisfy a particular want like car and petrol An increase in the price of complementary goods leads to a decrease in demand for the given commodity and a decrease in the price of complementary goods leads to an increase in demand for the given commodity For example if price of petrol falls then the demand for cars will increase as it will be relatively cheaper to use both the goods together So demand for a given commodity is inversely affected by change in price of complementary goods3 Income of the Buyer of CommodityDemand for a commodity is also affected by income of its buyer However the effect of change in income on demand depends on the nature of the commodity under consideration In case of some goods like full cream milk fine quality of rice (Basmati rice) etc demand for these commodities increases when income of the buyer increases and

demand for these commodities decreases when income of the buyer decreases Such goods whose demand increases with the increase in income of the buyer are called normal goods But there are some goods like coarse rice toned milk etc whose demand decreases when income of buyer increases and their demand increases when income of the buyer decreases Such goods whose demand decreases with the increase in income of the buyer are called inferior goods Suppose a consumer buys 10 Kgs of rice whose price is ` 25 per Kg He cannot afford to buy better quality of rice because the price of such rice is ` 50 per Kg The consumer is spending ` 250 per month on the purchase of rice Now if income of the consumer increases and he can afford ` 350 on purchase of 10 Kg of rice Now he can afford to buy some quantity of rice say 6 Kgs whose price is ` 25 per Kg and may buy 4 Kgs of rice whose price is ` 50 per Kg Thus he will buy 10 Kgs of rice by spending ` 350 per month Therefore we may conclude that demand for normal goods is directly related to the income of the buyer but demand for inferior goods is inversely related to the income of the buyer4 Tastes and Preferences of the BuyerThe demand for a commodity is also affected by the tastes and preferences of the buyers They include change in fashion customs habits etc Those commodities are preferred by the consumers which are in fashion So demand for those commodities rises which are in fashion On the other hand if a commodity goes out of the fashion its demand falls because no consumer will like to buy it(5) Number of Buyers in the Market(Population)Increase in population raises the market demand whereas decrease in population reduces the market demand for a commodity Not only the size of population but its composition like age (ratio of males females children and old people in population) also affects the demand for a commodity It is because of needs of children young old male and female population differs(6) Distribution of Income and WealthIf the distribution of income and wealth is more in favour of the rich demand for the commodities preferred by the rich such as comforts and luxuries is likely to be higher On the other hand if the distribution of income and wealth is more in favour of poor demand for commodities preferred by the poor such as necessities will be more(7) Season and Weather ConditionsThis is generally observed that the demand for woolens increases during winter whereas demand for ice creams and cold drinks increases during summer Similarly market demand for umbrellas rain coats increases during rainy seasonQ8 REASONS FOR OPERATION OF LAW OF DEMAND WHY DEMAND CURVE SLOPES DOWNWARDNow we will try to explain why does a consumer purchase more quantity of a commodity at a lower price and less of it at a higher price or why does the law of demand operate ie why does the demand curve slope downwards from left to right The main reasons for operation of law of demand are1 Law of Diminishing Marginal UtilityAs you have studied earlier law of diminishing marginal utility states that as we consume more and more units of a commodity the utility derived from each successive unit goes on decreasing The consumer will be ready to pay more for those units which provide him more utility and less for those which provide him less utility It implies that he will purchase more only when the price of the commodity falls2 Income Effect

When price of a commodity falls purchasing power or real income of the consumer increases which enables him to purchase more quantity of the commodity with the same money income Let us take an example Suppose you buy 4 ice creams when price of each ice cream is ` 25 If price of ice creams falls to ` 20 then with same money income you can buy 5 ice creams now3 Substitution EffectWhen price of a commodity falls it becomes comparatively cheaper as compared to its substitutes (although price of substitutes has not been changed) This will lead to rise in demand for the given commodity For example if coke and Pepsi both are sold at ` 10 each and price of coke falls Now coke has become relatively cheaper and will be substituted for Pepsi It will lead to rise in demand for coke4 Change in Number of BuyersWhen price of a commodity falls some old buyers may demand more of the commodity at the reduced price and some new buyers may also start buying this commodity who were not in a position to buy it earlier due to higher price This will lead to increase in number of buyers when price of the commodity falls As a result demand for the commodity rises when its price falls5 Diverse Uses of a CommoditySome commodities have diverse uses like milk It can be used for drinking for sweet preparation for ice cream preparation etc If price of milk rises its use may be restricted to important purpose only This will lead to reduction in demand for other less important uses When price of milk falls it can be put to other uses also leading to rise n demand for itQ9 EXCEPTIONS TO THE LAW OF DEMANDYou have studied in law of demand that a buyer is willing to buy more quantity of a commodity at a lower price and less of it at a higher price But in certain circumstances a rise in price may lead to rise in demand These circumstances are called Exceptions to the Law of Demand Some important exceptions are1 Giffen GoodsGiffen goods are special type of inferior goods in which negative income effect is stronger than negative substitution effect Giffen goods do not follow law of demand as their demand rises when their price rises Examples of Giffen goods are jowar and bajra etc2 Status Symbol GoodsSome goods are used by rich people as status symbols eg diamonds gold jewellary etc The higher the price the higher will be the demand for these goods When price of such goods falls these goods are no longer looked at as status symbol goods and tehrefore therir demand falls3 NecessitiesCommodities such as medicines salt wheat etc do not follow law of demandbecause we have to purchase them in minimum required quantity whatever their price may be4 Goods Expected to be ScarceWhen the buyers expect a scarcity of a particular good in near future they start buying more and more of that good even if their prices are rising For example during war famines etc people tend to buy more of some goods even at higher prices due to fear of their scarcity in near future

Political Science

Constitution of India-The

Preamble

The preamble-

Preamble-

The preamble is the most precious part of the constitution We the people of India having solemnly resolved to constitute India into a Sovereign Socialist Secular Democratic Republic and to secure to all its citizensA preamble is an introductory and expressionary statement in a document that explains the documents purpose and underlying philosophy When applied to the opening paragraphs of a statute it may recite historical facts pertinent to the subject of the statuteNature and purpose of the constitution-Purpose of the Constitution dictates permanent framework of the government to form a more perfect union to establish justice and ensure peace of thenationconstitution provide principles how the government can run itself following the rules and laws written in the constitution of each state keeps them balanced

Answer the following questions-

1 What is preambleA preamble is an introductory and expressionary statement in a document that explains the documents purpose and underlying philosophy2 What is the nature and

purpose of the constitutionConstitution dictatespermanent framework of the government to form a more perfect union to establish justice and ensure peace of the nation

Homework-Learn

Accounts Compatibilty mode

1MEANING OF PARTNERSHIPPartnership is a form of business organisation where two or more persons join hands to run a business They share the profits and losses according to the agreement amongst them According to the Indian Partnership Act 1932 ldquoPartnership is relation between persons who have agreed to share profits of a business carried on by all or any one of them acting for allrdquo For example one of your friends has passed class XII from National Institute of Open Schooling (NIOS) and wants to start a business Heshe approaches you to join in this venture Heshe wants you to contribute some money and participate in the business activities Both of you if join hands constitute a partnership2CHARACTERISTICS1048698 Agreement A partnership is formed by an agreement The agreement may be either oral or in writing It defines the relationship between the persons who agree to carry on business It may contain the terms of sharing profit and the capital to be invested by each partner etc The written agreement is known as partnership deed1048698 Number of persons There must be at least two persons to form a partnership

The maximum number of partners in a partnership firm can be 50 according toCompanies Act 20131048698 Business The Partnership is formed to carry on business with a purpose of earning profits The business should be lawful Thus if two or more persons agree to carry on unlawful activities it will not be termed as partnership1048698 Sharing Profits The partners agree to share profits in the agreed ratio In caseof loss all the partners have to bear it in the same agreed profit sharing ratio10486981048698Mutual Agency Every partner is an agent of the other partners Every partner can bind the firm and all other partners by hisher acts Each partner will be responsible and liable for the acts of all other partners10486981048698Unlimited liability The liability of each partner except that of a minor is unlimited Their liability extends to their personal assets also If the assets of the firm are insufficient to pay off its debts the partnersrsquo personal property can be used to satisfy the claim of the creditors of the partnership firm10486981048698Management All the partners have a right to mange the business However they may authorize one or more partners to manage the affairs of the business on their behalf10486981048698Transferability of Share No partner can transfer hisher share to any one including hisher family member without the consent of all other partners3PARTNERSHIP DEEDAgreement forms the basis of partnership The written form of the agreement is which a document of partnership is It contains terms and conditions regarding the conduct of the business It also explains relationship between the partners This document is called partnership deed Every firm can frame its own partnership deed in which the rights duties and liabilities of the partners are stated in detail It helps in settling the disputes arising among the partners during the general conduct of business 4CONTENTS OF PARTNERSHIP DEEDThe partnership deed generally contains the following (i) Name and address of the partnership firm(ii) Nature and objectives of the business(iii) Name and address of each partner(iv) Ratio in which profits is to be shared(v) Capital contribution by each partner(vi) Rate of Interest on capital if allowed(vii) Salary or any other remuneration to partners if allowed(viii) Rate of interest on loans and advances by a partner to the firm(ix) Drawings of partners and interest thereon if any(x) Method of valuation of goodwill and revaluation of assets and liabilities on the reconstitution of the partnership ie on the admission retirement or death of a partner(xi) Settlement of disputes by arbitration(xii) Settlement of accounts at the time of retirement or death of a partner5IN ABSENCE OF PARTNERSHIP DEEDThe partnership deed lays down the terms and conditions of partnership in regard to rights duties and obligations of the partners In the absence of partnership deed there may arise a controversy on certain issues like profit sharing ratio interest on

capital interest on drawings interest on loan and salary of the partners In such cases the provisions of the Indian Partnership Act becomes applicableSome of the Issues are(i) Distribution of Profit Partners are entitled to share profits equally(ii) Interest on Capital Interest on capital is not allowed(iii) Interest on Drawings No interest on drawing of the partners is to be charged(iv) Interest on Partnerrsquos Loan A Partner is allowed interest 6 per annum on the amount of loan given to the firm by himher(v) Salary and Commission to Partner A partner is not entitled to anysalary or commission or any other remuneration for managing the business

History TOPIC-TOWARDS INDEPENDENCE AND PARTITION THE LAST PHASE (1935-1947)

SUB TOPIC-IMPORTANT POLITICAL DEVELOPMENTS ndash GROWTH OF SOCIAL IDEAS

Socialism is a political social and economic philosophyLike in other parts of the world the Russian revolution of 1917 served as a great inspiration for revolutionaries in India who at that time were engaged in the struggle for liberation from British ruleSocialist ideas led to the formation of communist party of IndiaJAWAHARLAL NEHRU Among the early Congress leaders Jawaharlal Nehru was very much impressed and influenced by the Socialist ideas He also learnt about the Economic activities of the Soviet Union after the Bolshevic Revolution 1917 He made full use of them in IndiaThe election of Jawaharlal Nehru and Subhas Chandra Bose showed the Left wing tendency within CongressJawaharlal Nehru demanded economic freedom along with political freedom of the people in order to end the exploitation of masses

Nehrus working committee included three socialists leaders The Lucknow session was a landmark in the evolution of socialist ideas of the congressSUBHAS CHANDRA BOSE ndash Subhas Chandra Bose had socialist leaning Both Jawaharlal Nehru and Subhas Chandra Bose were known as leftist Congress men Later on National Congress divided into Leftist and rightist campCONGRESS SOCIALIST Within the Congress some leaders formed the Congress Socialist partyPattavi Sitaramyya Sardar Patel Rajendra Prasad had hostile attitude towards the Congress Socialist partyJawaharlals attitude was hesitant

1 QUESTION ndash Mention name of two Congress leaders who had socialist leaning

1ANSWER ndash Subhas Chandra Bose and Jawaharlal Nehru2QUESTION- In which session of the congress Jawaharlal elaborated his Socialist ideas2 ANSWER ndash Lucknow and Faizpur Session in December 1935 and 19363QUESTION ndash Why Congress was sharply divided into leftist and rightist camp 3ANSWER ndash Subhas Chandra Bosersquos attempt to seek re election for congress presidentship in 1939sharply divided the National Congress into Leftist and Rightist camp4 QUESTION ndash Who was MN Roy 4 ANSWER ndash Manabendra Roy first formed the Communist Party of India outside the country at Tashkent in 19205QUESTION ndash Who formed the Congress Socialist Party within the Congress5 ANSWER ndash Jaya Prakash Narayan Achyut Patwardhan Acharya Narendra Dev Ram Mohan Lohia Aruna Asaf Ali6QUESTION ndash When was the Congress Socialist Party formed What was its object6 ANSWER ndash 1934The Congress Socialist Party sought to work out socialist programme through the Congress They joined hands with the Congress and wanted to carry

Subhas Chandra Bose being expelled from the congress after the Tripuri rift he formed Forward BlockThere were basic differences between the Congress Socialists and the communistsTRADE UNION ACTIVITIES Maximum working class people lived in Bombay and Calcutta The working and living conditions of those workers were very miserable In this situation Shasipada Banerjee NM Lokhande protested against the oppression of the working class peopleThe first Trade Union Madras Labour Union was formed in 1918 by BP WadiaIndustrial strikes took place in Kanpur Calcutta Madras Jamshedpur and Ahmedabad AITUC was formed in Bombay in 1927 The growth of Trade union among the workers was slow because of the fear of the dismissal of the jobIn the mean time the Moderates as well as Communists left AITUC and formed separate organization

on National struggle with the help of workers and peasant class of the society7 QUESTION ndash What was the name of the party founded by Subhas Chandra Bose7 ANSWER- Forward Block8QUESTION ndash Who was Shasipada Banerjee8 ANSWER ndash Shasipada Banerjee was a radical Brahmo He founded a working menrsquos club to protest against exploitation of the British rulers towards the working class of India9 QUESTION ndash What was the weekly published by NM Lokhande9ANSWER- Dinabandhu10 QUESTION ndash Who founded Bombay Mill-Hands Association and in which year10 ANSWER- NM Lokhande in189011 QUESTION- Who was BP WadiaANSWER- BPWadia was the founder of Madras Labour Union in191812 QUESTION- What was the name of the first labour union of India12 ANSWER- Madras Labour Union13 QUESTION Who founded the Majur Mahajan 13 ANSWER GANDHIJI14 QUESTION What was the full form of AITUC When it was formed14 ANSWER All India Trade Union Congressin 192715QUESTION Who formed the Red Trade Union Congress and in which year15ANSWER The Communists formed the Red Trade Union Congress16 QUESTION What do you mean by Socialism16 ANSWER Socialism describes any political and economic theory that says the community rather than individuals should own and manage property and natural resources

Subject Eng Literature (The Tempest ndash William Shakespeare) Topic Act III Scene 3 Lines 1 to 52 (Line 52 ndash Brother my lord the Duke Stand to and do as we) Date 13th April 2020 (4th Period)

[Students should read the original play and also the paraphrase given in the school prescribed textbook]Summary Questions amp Answers

o Alonso Sebastian Antonio Gonzalo Adrian Francisco and others wandered about the island in search of Ferdinand and gets tired and hungry of the toil and at the same time gives up all hope of finding him

o Antonio and Sebastian are happy that Alonso is out of hope and decide to make another attempt on his life that night when being so tired they will be sleeping soundly

o Suddenly a solemn and strange music is heard in the air and several strange shapes enter bringing in a banquet These strange shapes then dance round it with gestures of salutation and then inviting the King to eat they depart

o Seeing this strange scene all are inclined to believe the tales told by travelers that there truly are ldquounicornsrdquo and ldquothe phoenixrsquo thronerdquo

1 ALONSO What harmony is this My good friends hark (L18-27)

GONZALO Marvellous sweet music

[Enter several strange shapes bringing in a banquet

they dance about it with gentle actions of salutation

and inviting the King and his companions to eat they depart]ALONSO Give us kind keepers heavens What were theseSEBASTIAN A living drollery Now I will believe

That there are unicorns that in Arabia

There is one tree the phoenixrsquo throne one phoenix

At this hour reigning thereANTONIO Ill believe both

And what does else want credit come to me

And Ill be sworn rsquotis true Travellers neer did lie

Though fools at home condemn rsquoem

(a) How did Prospero present an amazing spectacle before Alonso and his companions

Using his magic powers Prospero ordered strange shapes to lay a banquet before Alonso and his companions The shapes brought several dishes with tasty eatables in them They placed the dishes on a table before Alonso and his companions Then the strange shapes began to dance gracefully around the banquet While dancing they made gestures inviting them to eat the food Then suddenly the shapes disappeared(b) Who were the guests at the strange banquet Describe the lsquoliving drolleryrsquo

Alonso Sebastian Antonio Gonzalo Adrian and Francisco were the guests at the strange banquet

The term ldquoliving drolleryrdquo refers to live entertainment show In this context when Alonso the King of Naples Sebastian his brother Antonio the treacherous brother of Prospero Gonzalo the kind and loyal councillor to the King Adrian and Francisco came to the island they were hungry and weary in their spirits They heard a solemn and strange music They were shocked to see several strange shapes bringing in a banquet and these shapes danced about it with gentle action of salutation inviting the King and his companions to eat After this Sebastian described this show as lsquoliving drolleryrsquo(c) What is lsquophoenixrsquo What are lsquoUnicornsrdquo

The term lsquophoenixrsquo refers to a mythical Arabian bird which lived alone and perched on a solitary tree After one hundred years it expired in flames and rose again from its own ashes

lsquoUnicornsrsquo refers to the mythological four-footed beasts having horns in the centre of their foreheads When the horns are ground into powder the powder was believed to be

an aphrodisiac(d) How does Sebastian explain the puppet show OR Why does the speaker now believe in unicorns and phoenix

Sebastian finds several strange shapes bringing in the banquet They invite the king and his party for dinner and soon depart He tells that if such a strange sight can be a reality there is nothing incredible in the world and from the present moment he will believe anything He says that it is a strange dumb show enacted not by puppets but by living beings It is stranger than a travellerrsquos tale Seeing such a thing

before his own eyes he will no longer disbelieve the story about unicorns and phoenix(e) How do the other characters present respond to this living drollery

At the sight of the lsquoliving drolleryrsquo like Sebastian Gonzalo and Antonio too acted strangely Antonio told that he too now believes in unicorns and phoenix and anything else that seems to be incredible He too now believes in travellersrsquo tales Gonzalo told that if he would report those happenings in Naples nobody will believe him He considers that those gentle shapes were gentler in manner in comparison to the living beings Alonso was at first sight suspicious and told them that those strange shapes conveyed their meaning in expressive gestures when they seemed to lack speech by their movements and sounds Francisco was amazed at their mysterious disappearance

2 ALONSO Not I

(Line 43-52)GONZALO Faith sir you need not fear When we

were boysWho would believe that there were mountaineers

Dewlapped like bulls whose throats had hanging at rsquoem

Wallets of flesh Or that there were such men

Whose heads stood in their breasts Which now we find

Each putter-out of five for one will bring us

Good warrant ofALONSO I will stand to and feed

Although my lastmdashno matter since I feel

The best is past Brother my lord the Duke

Stand to and do as we

(a) How does Alonso respond at the spectacle of the shapes which were sent to them at the instruction of Prospero

After seeing the strange sight of appearing and disappearing of the shapes sent by Prospero to arrange a banquet for them Alonso says that his surprise at having seen those creatures is infinite and he is fully justified in feeling so much surprise He thinks that their shapes their gestures and the sounds they made were indeed amazing Although they do not possess the gift of speech yet they were able to convey their

thoughts by means of their gestures only

(b) What does Prospero say about the views expressed by Alonso regarding the shapes What does Francisco think about the shapesAfter hearing Alonsorsquos views about the shapes Prospero says that this manrsquos praise of the spirits is rather hasty He means to say that Alonso has shown great haste in reaching the conclusion about the shapes Francisco is amazed to see that those shapes disappeared in a mysterious way(c) What does Sebastian ask Alonso to doSebastian tells Alonso that the shapes having disappeared should not matter to them because they have left the eatables behind He asks Alonso to enjoy eating as they are extremely hungry but the king does not accept his offer of enjoying the dishes(d) How does Gonzalo try to dispel Alonsorsquos fear of those strange shapes What kind of references does he give to AlonsoGonzalo says that those who have travelled abroad have reported seeing even stranger sights than these shapes that Alonso and his companions have beheld Hence there is no reason to feel afraid of these shapes Gonzalo further adds that in his younger days he had heard strange stories from travelers and Alonso might have heard similar stories For instance it was said that there existed a certain race of

human beings who had huge lumps of flesh hanging at their throats and who therefore resembled bulls Then Gonzalo tells about a race of human beings whose heads were located at their breasts Gonzalo says that such stories were not believed by most people in those days but now-a-days these stories have become common(e) Explain the following lsquoEach putter-out of five for onersquoEnglish travellers often insured their trips with London brokers Those that went on foreign travels those days used to deposit a certain amount with some firm or company in London before their departure If the travelers failed to return the money was forfeited by the company with which it had been deposited But this money was repaid five-fold if the travelers returned safe and sound In this way a traveler stood a great chance of recovering the entire cost of his

travels(f) Give the explanatory meanings of the following expressions in the context of the above extract (i) Dewlapped (ii) Wallets of flesh

(iii) Putter-out(i) Dewlapped having big lumps of flesh at the necks(ii) Wallets of flesh large masses of flesh looking like bags(iii) Putter-out to invest money before commencing the travel

  • General methods of preparation of hydrogen
  • Chapter Dimensional Analysis (Summary)
    • Properties of Charge
Page 22:   · Web viewSubject. Topic. Summary. Execution. Hindi. व्याकरण. शरीरके अंगो के नाम लिखिए. 1) आँख 2) नाक 3

c=scnextInt() switch(c) case 1 Systemoutprintln(ldquoThe sum will be =rdquo+(a+b)) break case 2 Systemoutprintln(ldquoThe product will be =rdquo+(ab)) break default Systemoutprintln(ldquoWrong Inputrdquo) Home Work - Practice in your computer using bluej

Subject Eng Literature (The Merchant of Venice ndash William Shakespeare)Topic Act I Scene 2 Lines 92 to 126 (End of scene) Date 13th April 2020 (5th Period)

[Students should read the original play and also the paraphrase given in the school prescribed textbook]Summary Questions amp Answers

o After Portia has expressed her opinion about the suitors Nerissa informs that she need not bother about any one of them as they have decided to quit Belmont at the earliest opportunity because they do not believe in trying their luck by the caskets which is the only way of winning Portia

o Nerissa then enquires of Portiarsquos opinion about Bassanio who once visited her in the company of the Marquis of Montferrat and says that she had never come across such an ideal love deserving the fairest lady for his bride

o Portia seems to remember Bassanio quite correctly and says that she agrees with Nerissa At this moment a servant informs Portia that the Prince of Morocco has arrived to try his luck by the caskets

o Portia tells Nerissa that if she could welcome this new suitor as gladly as she says farewell to the previous ones she would be glad of his arrival However if he happens to have the virtues of a saint but the black complexion of a devil she would prefer to have him for religious consolation rather than as a husband

(1) NERISSA You need not fear lady (Line 97-103)

the having any of these lords they have acquainted me with their determinations

which is indeed to return to their home and to

trouble you with no more suit unless you may be wonby some other sort than your fathers imposition depending on the caskets

PORTIA If I live to be as old as Sibylla I will die as chaste asDiana unless I be obtained by the manner of my fatherswill I am glad this parcel of wooers are so reasonablefor there is not one among them but I dote on his veryabsence and I pray God grant them a fair departure

(a) Elucidate the idea expressed in the first speech of the above dialogue

In the first speech Nerissa assures Portia that she need not have any fear of being compelled to marry anyone of the suitors who had lately come to Belmont She informs her that they have all decided to return to their respective countries(b) Illuminate the meaning of the phrase ldquoyour fatherrsquos imposition depending on the casketsrdquo

Nerissa means that the suitors of Portia do not find the conditions imposed by the will of her father to their liking They are too hard for them These conditions are that in the event of a suitor failing to choose the right casket (i) he should never disclose to anybody which casket he chose (ii) he can never marry and (iii) he should take his departure immediately(c) Explain the meaning of the term lsquoSibyllarsquo

lsquoSibyllarsquo is the name given by Romans and Greeks to a prophetess inspired by some deity usually the sun-god Apollo She had a very long life The god Apollo granted her as many years of life as she could hold grains of sand in her hand(d) Elucidate the meaning of the term lsquoDianarsquo

lsquoDianarsquo is the goddess of hunting She is also regarded as a symbol of virginity because she never fell in love and never

married(e) Explain the meaning of the first two lines of Portiarsquos speech

Portia says that even if she is to live for centuries like Sibylla she would not marry except in accordance to her fatherrsquos will She asserts that she would not mind remaining unmarried and untouched by a man like Diana the virgin the goddess of hunting unless a man is able to win her by passing the test laid down by her father

Class XSubject Topic Summary Execution

Hindi 2nd

Langबड घर की बटी( मशी परमचद)

lsquoबड घर की बीटीrsquo कहानी का उददशय मधयम वग की घर समसया को सझा कर सगदिठत परिरवार म मिम जकर परम स रहन का सदश दना ह घर म शानित tानिपत करन की जिजममदारी नारी की होती ह यदिद नारी समझदार ह उसम धय और परिरवार क परनित परम ह तो कोई भी घटना परिरवार को निवघदिटत नही कर सकती या कहानी परिरवार को सगदिठत करत हए परम सौहाद स एक रदसर की भावनाओ को समझ करउनका सहयोग करत हए जीवन यापन करन की पररणा दती ह मशीपरमचदर जी न इस कहानी म सयकत परिरवार का परनितनिनमिधतव निकया ह यह कहानी बनी माधव सिसह जो गौरी पर क जमीदार क उनक दो पतरो की हशरी कठ ा निबहारीशरीकात का निववाह एकजमीदार घरान की पतरी आनदी स हआ थाआनदी न द को ससरा क वातावरण म ढालिया थाएक दिदन आनदी का अपन दवर ा निबहारी स झगडा हो जाता ह दोनो भाई एक रदसर स अग होन की कोलिशश करत हसभी बह आनदी न अपन मधर वयवहार स ा निबहारी को घर छोडकर जान स रोक लिया| इस पर बनी माधव सिसह न कहा निक बड घर की बटी ऐसी ही होती ह जो निबगडा काम बना ती ह अतः शीषक साथक ह बड घर की बटी आनदी ह

helliphelliphelliphellipContinue to nextBiology Topic ndash Chp-1

CellWelcome to new session 2020-21Today we will start with Chpter 1 cell CELL

Protoplasm+Cellmembrane Or Cell wall

Cytoplasm+Neucleus

Cytoplasmic+ CytoplasmicOrganelles Inclutions(mitochondria (food Golgi bodies pigments)Ribosome)

What is cellbull Cell is the structural and functional unit of living organismbull According to number of cells organisms areUnicellular - Amoeba bacteria Multicellular - Rose Mango Tiger HumanSmallest cell -bacteria Longest cell - Nerve cellLargest cell - Ostrich egg cellCells are of different size and shapes according to their functionsQ2Write chief functions of following cellorganelles

Q3What is tonoplastVacuoles covered by a covering called tonoplast

Bengali(2Nd

Language)

ফ ফটক ো ফটক (কলিবতো ) ভোষ মসোপো3 gtPোয়

একটি লেমসয়র ীবস লেপরম লিকভোসব ফসট ওসঠ তো লেদলিসয়স4 কলিব লেপরম Pই য় লেই ময়ই বনত কোস পলিরত য় ফ লেফোটো বো োসফোটো লেটো ব2 কো য় লেমসয়সদর ব gtয13 লেপরসমর 4লিব ফসট উসঠস4 এই কলিবতোয় লেপরম মোষসক মত gtযর মস লেফস লিদসয় পরকষস বাোচোসোর gtয োত বো2োয় কলিবতোয় লেমসয়টির পসব13র দঃসর কো বো সও লেমসয়টি লেই পসর পলিক সত চোয়ো োরী ীবসর কোস4 পরম লেPৌবস লেপরমসক পোবোর পরব ইচছো োকসও তো পসর লেলিতবোচকতোয় পলিরত য় কলিব ভোষ মসোপো3 যোয় লেP ক লেপরসমর

কলিবতোয় ব gtযবহত লিবসষ লিক4 সvর অ13 লেদওয়ো ১) রসবোো= লেP লিবলিভনন রকম ডোকসত পোসর২) ো= পোর ৩) ঠলি = লেচোসর বZ৪)আই বস2ো=অলিববোলিত৫)শইসয় = োলিয়ত কসর৬)োতপাোচ= লিবলিভনন পরকোর৭)দ2োম = v কসর বZ কসর লেদওয়ো৮)লেরলিং =লেোোর দৈতরী লেব2ো৯) বনত= একঋত১০) পাোর = বসকরো2

Organelles Functions

1 Endoplasmic reticulum

2 Mitochondria

3Golgibodies

4 Ribosome5Lysosome

6Plastids

7 Centrosome

i) Supportive framework for the cellii) Synthesis and transpost of proteinsRelease of energy in the form of ATPi) Synthesis and secretion of enzymes hormoneii) Formation of vacuoles lysosomei) Protein Synthesisi) Intracellular digestionii) Destroy foreign substancei )Leucoplast - stores starchii)chloroplast - trap solar energyiii) Chromoplast - imparts colour toflowers amp fruitsi) Initiates and requlates cell division

কলিবতো তোর অ13সক ভোষোয় পরকোো কসর ঘলিরসয় ব যকত কসরস4 লেপরসমর ফতো আর লিবফতো লেক গোঢ় কসর লেদোসো কলিব ভোষ মসোপো3 যোসয়র অলিভবসর অ যলিদক

Economics

Factors of Production

Welcome to the new sessionToday we are going to start the first chapter of Class XThe name of the chapter is Factors of productionBy the name I hope you all can recall a glimpse of what you have learnt in the second chapter of Class IX

NowProduction is the process of creating the various goods and services which are consumed by the people of the country to satisfy their wants

Thus it is the process in which some materials are transformed from one form to another to create utility and value in goods

For example utility can be created by changing the form of a commodity ie

Making of table out of wood by a carpenter for his customer here the wood is getting transformed into table creating utility for his customer and he can also command a price for it

On the other hand Housewives perform very

useful activities at home which create utility but their domestic activities are not included in production because they have no money value

So we can also say that Production denotes two things firstly creation of utility and secondly creation of value

Production is not complete unless it reaches the consumer

An increase in production will increase the economic welfare of the consumers and hence the aim is to raise the production level of the country

Again production of a good or service is only possible if certain resources or

Questions

1 What do you mean by production

Answer Production means the creation of goods and services for the purpose of selling in the market

In fact production involves the transformation of inputs into outputs

Hence production denotes two thingsCreation of utility and creation of valueUtility and value can be created by changing the form by changing the place by changing the time and by rendering services

Example Transformation of raw

materials into finish goods such as potter creates utility by converting mud into utensils assembling of small parts to make bigger machinery

Production also includes services such as distribution and marketing

2 What are the factors of production

Answer Factors of Production refers to the resources and inputs needed for producing goods and servicesThese inputs can be classified as

Land Labour

Capital Enterprise

Land Land is defined to include not only the surface of the earth but also all other free gifts of nature(for example mineral resources forest resources and indeed anything that helps us to carry out the production of goods and services but is provided by

inputs are used together in right proportion

A resource or an input which helps in the process of production to obtain an output is called FACTOR OF PRODUCTION

These factors of production can broadly be categorized into four parts 1LAND 2LABOUR3CAPITAL4ENTERPRISE (ORGANISATION)or Entrepreneur

The above factors are all interdependent on each other and they play a major role in production process

FACTORS OF PRODUCTION

LANDCAPITAL

LABOUR ENTREPRENEUR

nature free of cost)LabourLabour refers to the human efforts that need to be combined with other factors of production for creating an output

CapitalAll man ndash made means of production is called capita example machineries which help in further production Money when used for starting any business for purchasing raw materials machinery tools etc it is regarded as capitalCapital also includes physical capital like factories machineriestoolsbuildingsequipments etcEnterpriseThe task of bearing risks is called enterprise and the person who bears these risks of business is called the entrepreneurThus an entrepreneur is one who organises production takes important decisions regarding production hires and purchases factors of production and bears the risk and uncertainty involved in productionOrganisation refers to the services of an entrepreneur who controls organises and undertakes all risks One who plans organises and manages a business enterprise is an organiser

Physics Chapter 1 Force

Force is an external agent capable of changing the state of rest or motion of a particular body It has a magnitude and a direction The direction towards which the force is applied is known as the direction of the force and the application of force is the point where force is applied The Force can be measured using a spring balance The SI unit of force is Newton (N)

Question 1

State the condition when on applying a force the body has

(a) the translational motion

(b) The rotational motion

Solutions

(a) Translational motion is produced when the body is free to move

(b) Rotational motion is produced when the body is pivoted at a point

Question 2

Define moment of force and state its SI unit

Solutions

The moment of force is equal to the product of the magnitude of the force and the perpendicular distance of the line of action of force from the axis

of rotation

The SI unit of moment of force is Newton times meter

= Newton meter (Nm)

Commercial Studies

Stake holders In this topic you will be come to know about the meaning and concept of stakeholders

How stakeholders are different from shareholders

Questions1 What do you mean by the term stake holdersAnswer) The term stake holders have developed from the words which mean an interest or expected benefit Stakeholders mean all those individuals groups and Institutions which have a state (interest) in the functioning and performance of a commercial organisation or a business enterprise2 What do you mean by share holdersAnswer) The person and Groups who own the shares of the joint stock company by providing capital to the company are called shareholders Shareholders are the internal stakeholders shareholders are one out of several stake holders3 How are shareholders different from stakeholdersAnswer)i) The term shareholders is related to only joint stock company whereas stakeholders are related with all business organisationsii) Stakeholders maybe any individual having financial stake in business organisation whereas a shareholders are those individuals who are holding shares in the company4) How are shareholders different from creditorsAnswer) i) Shareholders are internal stakeholders while creditors are external stakeholdersii) Shareholders invest in the capital of the company whereas creditors give loan to the companyiii) Shareholders are the members of the company with voting rights but creditors are not the members of the company

English 1 Transformation of sentences

Sentences A sentence is a group of words which makes complete sense

e Assertive sentencesf Imperative sentencesg Interrogative sentencesh Exclamatory sentences

Sentences can be changed from one grammatical form to another without changing the meaning of the sentence This is known as transformation of sentences

Exercise 1 Change the following affirmative sentences into Negative sentences

a He is a good manHe is not a bad man

b Ram loves SitaRam is not without love for Sita

c Only he stood first in the classNone but he stood first in the class

d Ankit was wiser than he

He was not so wise as Ankite He did it

He did not fail to do itf As soon as I reached college the

bell rangNo sooner did I reach college than the bell rang

g He finished everythingHe left nothing unfinished

h It always pours when it rainsIt never rains but it pours

Math Topic Commercial MathematicsChapter ndash Goods and services Tax

What is GSTAns It is a abbreviated term of Goods and Service Text which is an indirect tax levied on the sale of goods and rendering servicesSome terms related to GSTDelar Any person who buys goods or services For resale is known as a delar A delar Can be a firm or a companyIntra-state sales Sales of goods and services within the same state or same union territory are called intra- state salesInter-state sales Sales of goods and services outside the state or union territory are called Inter-state sales4) Input GST GST is paid by dealers on purchase of goods and services are called input GST5) Output GST GST is collected from customers on sale of goods and services are called output GST6) Types of GST There are three taxes applicable under GST(i) Central Goods and Services Tax (CGST)(ii) State Goods and Services Tax (SGST) or Union Territory Goods and Services Tax (UTGST) Both these taxes are levied on intra-state sales Here GST is divided equally among central and state governments(iii) Integrated Goods and Services Tax (IGST) IGST is levied on inter- state sales It is also levied on import of goods and services into India and export of goods and services from India

Subject Eng Literature (The Merchant of Venice ndash William Shakespeare)Topic Act III Scene 4 Lines 1 to 44 (Portia hellip To wish it back on you fare you well Jessica)[Students should read the original play and also the paraphrase given in the school prescribed textbook]

Summary Questions amp AnswersIn this scene we suddenly find a new element in the character of Portia We have already seen her possessed of every graceful womanly quality but now she shows that she is capable of rapid decision and determined action She shows this by her sudden resolve to hasten to Venice with a daring scheme for the rescue of Antonio This is an important scene in the dramatic action for it leads up to and renders possible the striking events of the famous trial scene which is one of the greatest striking elements of the play Moreover the fact that all the characters of importance are now assembled together in Venice makes the union of the main plot and the secondary story complete

(1) LORENZO Madam although I speak it in your presence(Line 1-9)

You have a noble and a true conceit

Of god-like amity which appears most strongly

In bearing thus the absence of your lordBut if you knew to whom you show this honourHow true a gentleman you send reliefHow dear a lover of my lord your husbandI know you would be prouder of the workThan customary bounty can enforce you

(a) Where is Lorenzo Why is he here To whom is he referring as lsquoMadamrsquo

Lorenzo is at Portiarsquos residence He had met Salerio on the way and Salerio had begged him to come along with him to

o In this scene Portia Nerissa Lorenzo Jessica and Balthazar appear

o Portia requests Lorenzo and Jessica to be in charge of her house during her absence from Belmont because she and Nerissa have decided to spend the days in meditation and also in visiting the holy places in the neighbourhood of Belmont She has already instructed her people to acknowledge both Lorenzo and Jessica as master and mistress of house during her absence Lorenzo and Jessica gladly agree to look after the house of Portia

handover the letter from Antonio to Bassanio The letter carried the bad news about Antoniorsquos arrest for non-payment of loan taken from Shylock Hence Salerio might have preferred company to break this bad news to Bassanio He is referring to Portia as Madam(b) What does Portia say on hearing the above extract

Portia says that she has never regretted doing good to others Friends who spend a lot of time together and really are there for each other have many traits in common As Antonio is Bassaniorsquos best friend saving him is like saving Bassanio who is like her own soul She asks Lorenzo to take care of management of the house till Bassanio is back(c) What does Portia send with Bassanio and why

On hearing about Antoniorsquos troubles on account of Bassanio her husband Portia immediately sends him with enough gold to repay the debt many times over to Venice to help Antonio out of his misfortune

(2) Lorenzo Madam with all my heart (Line 36-40)

I shall obey you in all fair commands

Portia My people do already know my mindAnd will acknowledge you and JessicaIn place of Lord Bassanio and myselfSo fare you well till we shall meet again

(a) Where are Lorenzo and Portia at this time What lsquofair commandsrsquo are given to Lorenzo

Lorenzo and Portia are at Belmont during this scenePortia reveals to Lorenzo that she has sworn to contemplate in prayer at a monastery around two miles away until her husband returns from Venice She tells him that Nerissa would accompany her and asks him to manage the house with Jessica till things are settled In response Lorenzo tells her that he would be obliged to do whatever she asks him to do(b) Where is Portia actually going and why

Portia tells Lorenzo that she would live a life of contemplation and pray at a monastery which is two miles away from her place In reality Portia plans to go to Venice in disguise with Nerissa and argue the case in defense of Antonio She is very sure that her plan would succeed

ClassXI (ScienceHumanitiesCommerce)Subject Topic Summary Execution

Computer Science

(APC)

Ch ndash 1 Numbers

(Numbers in different bases and

their Arithmatical operations)

Number System In computers Number System is defined as a writing system to represent the numbers in different ways ie we are using different symbols and notations to represent numbers There are four ways we can represent the number ndash Binary Decimal Octal and Hexadecimal

Decimal Number SystemThis number system consist 10 digits These are 0 1 2 3 4 5 6 7 8 amp 9

Binary Number SystemThis number system has only two digits these are 0 and 1 Here 0 stands for off while 1 stands for on

Octal Number SystemThis number system has 8 digits these are 0 1 2 3 4 5 6 amp 7

Hexadecimal Number SystemThis number system has 16 digits these are 0 1 2 3 4 5 6 7 8 9 A B C D E F Here the value of the alphabets are as follows A=10 B=11 C=12 D=13 E=14 F=15

Rules for conversion decimal number to Binary1 Divide the decimal number by 22 If the number will not divide equally by 2 then round down the answer to the nearest whole number (integer)3 Keep a note of the remainder it should be either 0 or 14 Keep repeating the above steps dividing each answer by 2 until you reach zero5 Write out all the remainders from bottom to top This is your binary solution

For example Lets convert 32 to binary 2 32 2 16 - 0 2 8 - 0 2 4 - 0 2 2 - 0 2 1 - 0 0 - 1

The binary equivalent of 3210 is 1000002

Try the follwing youself1 2410

2 4810

3 1210

History GROWTH OF NATIONALISM

The second half of the 19th century witnessed growth of political consciousness and a sense of Nationalism among the IndiansThere were various factors for growth of Indian Nationalism- As a result various political associations were formed in different provinces by the educated Indians Surendranath Banerjee organized a meeting of National conference at Calcutta Ultimately the National Congress was founded in Bombay in 1885This body became the vanguard of Indian struggle for freedom The congress leaders were known as moderates because they followed a policy of prayer and petition A large number of Indian leaders had experienced in political agitation The Political situation of England was also changed Moreover increasing revolutionary activities in Maharashtra Punjab and Bengal became serious concern to the British Government In this

QUESTION1 What do you mean by Nationalism ANSWER 1 Nationalism is defined as loyalty and devotion to own nation especially a sense of national consciousnessQUESTION 2 What are the causes of nationalism ANSWER 2 There were various factors for growth of nationalism

1 Spread of western education2 The progress of vernacular press and

patriotic literature3 The economic exploitation of our

country by the colonial rulers4 International affairs

QUESTION 3 Who organized National conference in Calcutta in 1883 ANSWER 3 Surendranath BanerjeeQUESTION 4 When did Indian National Congress formANSWER 4 Indian National Congress was formed in 1885 in BombayQUESTION 5 Who were ModeratesANSWER 5 The Early Nationalists were also known as Moderates Their emergence marked

background Lord Curzon became Viceroy in India He had no respect for the Indian National Congress

the beginning of the organized national movement in India They believed in British justice and were loyal to them They followed a policy of prayer and petition They demanded constitutional reforms of our country Impotant Moderate leaders were Pherozshah Mehta Dadabhai Naorozi and Surendranath Banerjee etcQUESTION 6 What do you know about Extremism in Indian National movementANSWER 6 In the beginning of 20th century a new class of national leaders emerged in India which was different from the moderate groups They started more aggressive movement against the British empire The goal of extremists was ldquoswarajrdquo Important extremist leaders were Bal Gangadhar Tilak Lala Lajpat Rai Bipin Chandra Pal etcQUESTION 7 Mention the places which were the main centres of Revolutionary movementANSWER 7 Maharashtra Bengal and Punjab

Physics

Chapter Dimensional Analysis

(Summary)

The dimensions of a physical quantity are the powers to which the fundamental units are raised in order to obtain the derived unit of that quantit

The physical quantites lengthmasstime are represented by [L] [M] [T] resp let they are raised to powers ( dimesions) abc resp then any physical quantity can be represented by [ La Mb Tc ] Examples

1 Area area = L x B = [L] x [L] = [M0 L2 T0 ]

2 Density density = massvolume = [M][L3] = [ M L-3]

3 Velocity velocity = distancetime = [L][T] = [LT-1]HW Try to find out dimension of acceleration Acceleration = velocity timeNB One can find the SI Units Using Dimension Analysis Such as for area we have [L2] so its SI unit is m2

Biology Topic ndash Chp-1 The living world

Today we will start the first chapter the living world Here we discuss about the characteristics of living organism and what are the difference between them and nonliving substances We also discuss about the contribution of different Scientists

There are over 500000 species of plants andover a million species of animal are present on earth Some 15000 new species were discovered every yearQ1 What is a living organismbull A living organism is primarily physico -chemical material that demonstrate a high degree of complexity is capable of selfRegulation possesses a metabolism and perpetuates itself through timeQ2 What are the differences between livingand non-livingsi) Compared with non-living living organisms

have more complex organised structure and their use of energy is more controlled amp efficientii) Living things reproduce their own kind by forming new cells which contains copies of their genesiii) Each organism has some degree of homeostasisie it is able to make adjustments so that internal environment remains constantQ3 Write contributions of following Scientists i) Aristotle - One of the first theories in Biology places all living things in a hiearchieii) AV Leeuwenhoek - was the first to observe living single celled organisms under microscopeii) Carolus Linnaeus - developed the binary system for naming of organisms and classificationiii) Geregor Johann Mendel ndash discoverbasic principles of inheritanceHomework i) C Darwin ii)Schleiden

Math Trigonometric functions

1 Overviewi) Trigonometry The word lsquotrigonometryrsquo is derived from the Greek words lsquotrigonrsquo and lsquometronrsquo which means measuring the sides of a triangle An angle is the amount of rotation of a revolving line with respect to a fixed line Usually we follow two types of conventions for measuring angles ie a) Sexagesimal system b) Circular system In Sexagesimal system the unit of measurement is Degree In Circular system the unit of measurement is Radian ii) Relation between degree and radianThe ratio of circumference of a circle to its diameter is always a constant This constant ratio is a number denoted by π which is taken approximately as 227The relationship between degree amp radian measurements is as follows2 right angles = 180deg= π radians1radian = 180degπ=57deg16(approx) 1deg=π180 radianiii) Length of an arc of a circleIf an arc of length s subtends an angle θ radians at the center of a circle of radius r then s=rθiv) Area of a sector of a circleA sector is like a pizza slice of the

Q) Express the following angles in radiana) 45deg b) 40deg3730Ans a) We have 180deg=π radiansi e 45deg= πtimes45180 radian = π4 radiansb) 40deg3730= 40deg37+3060 minute= 40deg 37 +12 minute= 40deg+ 752 minute=40 + 75(2times60) degree=3258 degreeNow 180deg=π radianie 3258 degree= (πtimes325) (180times8) radians = 65π288 radiansQ) A circle has a radius of r=12 meters What is the length of an arc traced out by a 60deg angle in the center of the circleAns In this problem we know both the central angle (60deg) and the radius of the circle (12) All we have to do is plug those values into our equation and we get

s = 2π(12)(60360)s = 24π6s = 4πSo the length of an arc traced out by a 60deg angle in a circle with a radius of 12 meters equals 4π meters asymp 1257 metersQ) Find the area of the sector with a central angle 30deg and a radius of 9cmAns GivenRadius r = 9 cmAngle θ = 30degArea of the sector = θ360degtimesπr2

= 30360degtimes227times92=2121cm2

circle It consists of a region bounded by two radii and an arc lying between the radiiThe area of a sector is a fraction of the area of the circle This area is proportional to the central angle In other words the bigger the central angle the larger is the area of the sectorArea of Sector = θ2 times r2 (when θ is in radians)

Area of Sector = θ times π360 times r2 (when θ is in degrees)

COMMERCE

CLASSIFICTION OF HUMAN ACTIVITIES-ECONOMIC AND NON-ECONOMIC

Welcome to the new sessiontoday we are going to start the first chapter of Class XI The name of the chapter that we are going to start is

lsquoClassification of Human Activities ndasheconomic and non-economicrsquo

Now let us start the chapter by considering human beings and the activities they perform throughout the day

Human activities means all those activities that human beings undertake to satisfy their wants

Human wants on the other hand are the desire of human beings for goods (vegetables fruits rice etc) and services (services of doctors teachers lawyers etc) that they require to live

Now these human activities continue throughout life as human wants are unending unlimited and recurring as human beings desire for better living throughout their lives

Now human activities can be classified into two categories

Human activities

Economic activities Non-economic activities

Economic activities are

Questions1 What are human activities

Answer Human activities mean all those activities that human beings undertake to satisfy their wants

Example A man working in an office

A boy playing in the garden

2What are the characteristics of human activitiesAnswer the characteristics of human activities are as follows

Human activities are undertaken by men women and children and these activities involve human efforts

Human activities are undertaken to satisfy human wants which are unlimited

Human activities continue throughout life

Human activities are performed for both earning money and personal satisfaction

3What is economic activitiesGive example

Answer Economic activities are undertaken by human beings with the object of earning money acquiring wealth and thereby satisfying human wantsExample

Selling of goods by a shop keeper to his customer

A clinic run by a doctor Service of a teacher in school or college

undertaken by human beings with the object of earning money and acquiring wealth

These activities result in the production of economic goods and services

Example Human activities(ie working in factories officesshops) which produce direct economic benefits

Non-economic activities are inspired by human sentiments and emotions such as love for the family desire to help the poor and love for the country

Thus these human activities (eg praying playing sleeping) produce no direct economic benefits and they are also not related to earning money and acquiring wealth

4 What are the characteristics of economic activities

Answer The characteristics of economic activities are as follows

Economic motiveEconomic activities are undertaken to earn money and acquire wealth

ProductiveEconomic activities involve productiondistribution and exchange of goods and services to create wealth

Economic growthEconomic activities determine the level of economic development of a country and standard of living of its citizens

Socially desirableEconomic activities are socially desirable for society

Economic resourcesEconomic activities make use of all the economic resources such landlabourcapital etc

5 What do you mean by non-economic activitiesExampleAnswerNon-economic activities are inspired by human sentiments and emotions such as love for the family desire to help the poor and love for the countryThese activities are not undertaken for monetary gain but for onersquos satisfaction and happinessExample

a mother looks after her children

a student donates blood8 Differentiate between Economic activities and Non-economic activities

Economic activities

Non-economic activities

1to earn living and acquiring wealth2Result can be measured in terms of money

3ExampleBusinessprofession and employment

1 to obtain some satisfaction

2Result cannot be measured in terms of money

3ExampleFamily-orientedreligious socialCultural and national

BUSINESS STUDIES

BUSINESS ENVIRONMENT

Welcome to the new sessionToday we are going to start the first chapter and the name of the chapter is Business Environment

In todayrsquos world every business enterprise is a part of the society It exists and operates in association with various groups in society such as customers suppliers competitors banks and financial institutions government agencies trade unions media and so on All these groups influence the functioning of business in one way or the other They constitute the environment of businessConcept of Business Environment

The term lsquobusiness environmentrsquo refers to the sum total of all individuals institutions and other forces that lie outside a business enterprise but that may influence its functioning and performance

The main features of business environment

Totality of External forces General and Specific forces Interrelatedness Complexity Dynamic Uncertainty Relativity

The Interrelation between business and its environment

The business enterprise is an open system It continuously interacts with its environment It takes inputs

Prepare the following questions from todayrsquos assignment

1 What do you mean by business environment

The term lsquobusiness environmentrsquo means the aggregate of all forces factors and institutions which are external to and beyond the control of an individual business enterprise but they may influence its functioning and performance Business environment is the macro framework within which a business firm a micro unit operates It consists of several interrelated and interacting elements

2 Explain the main features of business environment in brief

Totality of External forces-Business environment is the sum total of all things external to a business environment

General and Specific forces-It includes both the forces general forces are the economic social political legal and technological conditions which indirectly influence all business enterprise Specific forces are the investors customers competitors and suppliers which influence individual enterprise directly

Interrelatedness-Different elements of environment are interrelated for an example growing awareness for health care has increased the demand for health foods

Complexity- Business environment id

(such as raw materials capital labour energy and so on) from its environment transforms them into goods and services and sends them back to the environment

Fig 1 Business Environment Relationship

complex in nature as the elements keep on changing example economic technological and other forces changes in demand for a product and service

Dynamic-Business environment is not static it keeps on changing

Uncertainty- Itrsquos very difficult to predict future events such as technology and fashion which occur fast and frequently

Economics Basic Economic ConceptsSub topic

Microeconomics and

Macroeconomics

Welcome to the new sessiontoday we are going to start the first chapter of Class XI The name of the chapter that we are going to start is Basic Economic concepts

Now Economics covers the study of human activities Human activities are those activities which are performed by humans to satisfy their wants

Thus Human wants are unlimited and therefore economic activities such as production exchange and consumption are needed in order to satisfy those wants

The study of economics is divided largely in two parts which areMicroeconomics and Macroeconomics

SUBJECT- MATTER OF ECONOMICS

MICROECONOMICS MACROECONOMICS

Questions1Who has coined the words micro and macro economics

Answer Ranger Frisch coined the words lsquomicrorsquo and lsquomacrorsquo in 1933 to denote the two branches of economic theory namely microeconomics and macroeconomics

2What is microeconomicsAnswer It is the study of behaviour of individual decision ndash making unit such as consumers firms etc

3 What is macroeconomicsAnswer Macroeonomics is the study of overall economic phenomena like employment national income etc

4 What is the importance of microeconomicsAnswer

Microeconomics helps in formulating economic policies which enhance productive efficiency and results in greater social welfare

It helps the government in formulating correct price policies

It explains the working of a capitalistic economy where individual units(producers and consumers ) are free to take their own decision

Micro means a small part in

microeconomics we do not study the whole economy Hence we study an individual consumer and his or her choices and a producer and his or her profit maximizing decisions in the market Thus it does not mirror what happens in the economy as a whole

Macroeconomics on the other hand studies the economy as a whole It is concerned with aggregate and depicts the entire picture of the economyMacroeconomics deals with the national income aggregate investment aggregate consumption etc

Features of Microeconomics It deals with small

parts of the country Hence it looks at

individual consumers firms and industries

It deals with individual income consumption and savings

It studies the determination of price of any product or factors of production

It deals with the working of market via the price mechanism which is nothing but the determination of price and quantity of a commodity by the forces of demand and supply

Features of Macroeconomics

It deals with the study of the economy as a whole

It is concerned with

5 Give a limitation of microeconomics Microeconomics fails to explain the

functioning of an economy as a whole It cannot explain unemployment illiteracy and other problems prevailing in the country

6 What is the importance of macroeconomics It gives overall view of the growing

complexities of an economic system It provides the basic and logical

framework for formulating appropriate macroeconomic policies (eg for inflation poverty etc )to direct and regulate economy towards desirable goals

7What is the limitation of macroeconomics It ignores structural changes in an

individual unit of the aggregate

8 Differentiate between Microeconomics and Macroeconomics

Microeconomics Macroeconomics

the study of aggregates

National income aggregate savings and aggregate investments are major concepts dealt within macroeconomics style

It studies the determination of general price levels

It investigates into the problem of unemployment and the achievement of employment

It studies the aspect of decision making at the aggregate and national levels

It includes all growth theories whether related to developed or developing economies it also includes the study of economic systems and the working of the economy under different systems

Note Both Micro and macro economics are complementary and should be fully utilized for proper understanding of an economy

1It studies economic aspect of an individual unit2It deals with individual incomeConsumption and savings

3 It facilitates determination of price of any product or factors of production

4 Itrsquos scope is narrow and restricted to individual unit

1It studies the economy as a whole

2It deals with the national income aggregate consumption and aggregate savings3 It facilitates determination of general price level in an economy

4 Itrsquos scope is wide as it deals with economic units on the national level

ACCOUNTS

Introduction to Accounting and Book-keeping

Today I am going to share you the meaning of Accounting and Book-keeping and its related terms bullAccounting bullBook Keeping bullAccountsbullTypes Of Accounts bullAccounting Cycle

bull Meaning of accounting

Ans ) Accounting is the art and science of recording classifying and summarising monetary transactions

bull Meaning of Book-keeping

Ans) Bookkeeping is the art of recording business transactions with the view of having a permanent record of them and showing their effect on wealth

bull Meaning of account

Ans) The term account means a record of

business transactions concern a particular person of firm asset or income or expense It is a summarised record of all transactions which take place in an accounting year

bull Types of accountsPersonal accounts ndash Personal accounts relating

to person and Organisation are known as personal accounts Example Ramrsquos Account ABC amp Co Account etc

Real account - The accounts related to tangible and intangible assets are called real accounts Example Cash Account Furniture Account etc

Nominal account- Accounts related to expenses losses incomes and gains are known as nominal accounts Example Wages Account Salary Account Discount Account etc

bull Accounting cycle Accounting cycle refers to a complete sequence of accounting activities It begins with recording of transactions and ends with the preparation of a balance sheet

Chemistry TopicAtomic Structure

Thomsonrsquos atomic modelThomson (1898) was the first to propose the model of an atomHe proposed that an atom can be regarded as a uniform sphere of positive electricity in which requisite number of electrons are embedded evently to neutralize the positive chargeThis is just like plums embedded in a pudding or seeds evently distributed in red spongy mass of a watermelonThis model of atom is known as ldquoPlum-Pudding modelrdquo or

Q1)What is the fundamental constituents of atomAns Electron Proton and neutrons are the fundamental constituents of atomQ2)What is the value of fundamental unit of electricityAnsThe charge carried by one electron is sad to be the fundamental unit of electricityIts magnitude is 48times10-10esuOr 1602times10-19C Q3)Name the element containing no neutronAnsOrdinary hydrogen atom or protium 1H1

Types of AccountPersonal AccountReal AccountNominal AccountBalance Sheet (opening)

ldquowatermelon modelrdquoThis model could explain the electrical neutrality of an atom but failed to explain the result of scattering experiment carried out by Rutherford in 1911So it was rejected ultimately

Q4)Why is an electron called universal particleAns Itrsquos mass and Charge are independent of its source

EVS Chapter 1 ndash Modes of Existence

Modes of existence When one speaks normally about the mode of existence of some group or individual one refers to their customs their mode of being their ethology their habitat in some way their feeling for a placeDifferent modes of exixtence are ndash

1 Hunting ndashGathering2 Pastoral3 Agricultural4 Industrial

1 Hunting and gathering Hunting and gathering mode of existence is characterized by obtaining food from hunting wild animals including fishing and gathering wild plants From their earliest days the hunter-gatherer diet included various grasses tubers fruits seeds and nuts Lacking the means to kill larger animals they procured meat from smaller game or through scavenging

Societies that rely primarily or exclusively on hunting wild animals fishing and gathering wild fruits berries nuts and vegetables to support their diet are called hunting and gathering societies

At least this used to be practice of human beings before agriculture is invented As their brains evolved hominids developed more intricate knowledge of edible plant life and growth cycles

Q) Write the features of Hunting ndash gathering societiesAns - There are five basic characteristics of hunting and gathering societies

i The primary institution is the family which decides how food is to be shared and how children are to be socialized and which provides for the protection of its members

ii They tend to be small with fewer than fifty members

iii They tend to be nomadic moving to new areas when the current food supply in a given area has been exhausted

iv Members display a high level of interdependence

v Labor division is based on sex men hunt and women gather

Political Science

Introduction to political science

Political science occasionally called politology is a social science which deals with systems of governance and the analysis of political activities political thoughts associated constitutions and political behaviorThe study of political science involves the study of both the

Answer the following questions-1 What is political science

Political science occasionally called politology is a social science which deals with systems of governance and the analysis of political activities political thoughts associated constitutions and political behavior

2 Short notes-

traditional and modern theories of politicsTraditionalClassical political sciencepolitical theory-Traditional political science is the study of politics before Second World War The methodology to study Politics was traditional (legal formaletc) the definition of politics traditional (Politics begins and end with state)area of study (constitution state machinery)was traditionalModern Political scienceModern political theory-Modern Political Theory critically examines the contemporary state of political theory making an assessment of the achievement and limitations of the Behavioural Revolution in its totality and reviews objectively the major paradigms and conceptual frameworks adopted by the disciplineContemporary attempts at the development of an integrated political theory involving the use of both traditional and modern concepts approaches and theories-Around late 1960s several political scientists realized the importance of both the traditional political theory and modern Political theory They began building an integrated theory of politics involving a systematic mixture of traditional and modern studies of politics It was held that the study of a complex and vast field like politics needs both traditional as well as

Classical political theory Modern Political theory

Homework-Learn

modern concepts and approaches for studying itrsquos all aspects

Subject Eng Literature (The Tempest ndash William Shakespeare) Topic Act I Scene 1 Lines 1 to 32 (Line 32 ndash Gonzalo hellip If he be not born to be hanged our case is miserable) Date 13th April 2020 (3rd Period)

[Students should read the original play and also the paraphrase given in the school prescribed textbook]Summary Questions amp Answers

[SUMMARY OF THE ENTIRE SCENE]

o The play starts with the scene of a severe storm at sea Alonso (King of Naples) Sebastian (Alonsorsquos brother) Ferdinand (Alonsorsquos son) Gonzalo Antonio (the usurping Duke of Milan) are in a ship in the midst of the storm

o The mariners are trying their best to control the vessel from running aground and are totally following the orders of their Master the Boatswain They have scant success

o The mariners become extremely unhappy and annoyed when most of the passengers arrive on the deck thereby hampering their effort to save the ship There is serious confrontation between them and the passengers who are part of the Kingrsquos entourage

o The mariners could not save the ship

SUMMING-UP

(i) Vivid description of the scene which gives a realistic description of terror and confusion of a tropical storm

(ii) Shows Shakespearersquos accuracy of knowledge in describing the naval operations and also matters of seamanship

(iii) The opening scene justifies the title ndash The Tempest

UNANSWERED QUESTIONS

(i) The King always travels with his entire fleet including his soldiers Where

(1) GONZALO Nay good be patient (Line 15-26)BOATSWAIN When the sea is Hence What cares these

roarers for the name of the king To cabin silence Trouble us not

GONZALO Good yet remember whom thou has aboardBOATSWAIN None that I more love than myself You are a

councillor if you can command these elements to silence and work

the peace of the present we will not hand a rope more use your authority If you cannot give thanks you have

lived so long and make yourself ready in your cabin for the mischance of the hour if it so hap [To the Mariners]

Cheerly good hearts [To Gonzalo] Out of our way I say

(a) To whom is the boatswain speaking What does he mean by lsquoNone that I more love than myselfrsquo

The Boatswain is speaking to Gonzalo the honest old councilor of the Duke of MilanBy using the words ndash lsquoNone that I love more than I love myselfrsquo means that for the Boatswain nobody is dearer to him than his own life

(b) What were the conditions that made the boatswain react in this way

The Boatswain reacts in this way because the storm is at sea and Alonso King of Naples Sebastian his brother Ferdinand his son Gonzalo Antonio the usurping Duke of Milan on board are in distress and in panic Thus they have rushed to the deck interrupting the work of the mariners

(c) What hope does Gonzalo take from the attitude of the boatswain

The insolent and authoritative attitude of Boatswain makes Gonzalo feel comforted He tells that there are no signs that the Boatswain will be drowned But his facial appearance and attitude shows that he is destined to die on land by hanging which in effect means that all on board will be saved Otherwise all the persons on board are doomed

(d) How can they lsquomake yourself ready in your cabinrsquo For what were they asked to make ready themselves

In order to make themselves ready in their cabin the

were the other ships

(ii) Why was the ship in that area Where was it coming from or going where

(iii) The ship broke apart What happened to those who were in the ship

passengers on board must prepare for death which they will possibly soon have to meetThey can retire to their cabins and offer prayers to the Almighty to save them from drowning

(e) What does the boatswain say when he is asked to be patient What does he order to the royal party

When the boatswain is asked to be patient and remain calm he says that he will be patient only when the storm will be over and the sea will be calm but as long as the storm blows and there is danger to the ship he cannot think of being patient He orders the royal party to go to the cabin and leave the mariners to their work

(2) GONZALO I have great comfort from this fellow (Line 27-36)

Methinks he hath no drowning mark upon him his complexion is perfect

gallows Stand fast good Fate to his hanging Make the rope of his destiny our cable for our own doth little advantage If he be not born to be hanged our case is miserable

(a) Why does Gonzalo regard the Boatswain in the midst of danger

In the midst of danger Gonzalo regards the boatswain because he feels that the Boatswain is a source of comfort and is bent upon to do his work sincerely which in this case is saving the ship and its passengers from the severest of raging storm

(b) What reasons does Gonzalo give when he says that none in the ship will die of drowning

Gonzalo is almost sure that none in the ship will die by drowning His says that there is no mark on the face of the boatswain that indicates that he will die by drowning On the other hand the lines on his face are strong indications that he will be hanged to death Therefore there shall be no danger of the shiprsquos sinking

(c) Explain the following ldquoStand fast good Fate to his hanging Make the rope of his destiny our cable for our own doth little advantage If he be not born to be hanged our case is miserablerdquo

The stated lines mean that if the will of destiny is to be carried out then the ship will not get wrecked and all the passengers will be saved The safety of the passengers therefore depends upon the will of fate being carried out in the case of the boatswain If however the boatswain is not to die by hanging then the passengers are also very unsafe because in that case the ship is likely to sink

(d) What order does the Boatswain give to the sailors

when he re-enters What does he say about the crying of the fellows inside the cabin

The boatswain orders the sailors to bring the topmast lower and bring the ship close to a stationary position with the help of the main sail He says that the fellows inside the cabin are moaning and crying in their distress louder than his voice and louder even than the roaring of the storm

Class XII (ScienceCommerceHumanities) Subject Topic Summary Execution

Computer Science

PropositionalLogic

Propositional logic is a procedure to provide reasoning through statementProposition A ststement that results in True or False is said to be proposition There are two types of propositionSimple proposition amp compound propositionSimple proposioton A simple proposition is one that is not a part of any other proposition Such sentential form of proposition is symbolized with english letters in short For example Ram is a claver student (TrueFalse)Where do you live (Not in True or False)Grapes are sweet (TrueFalse)It rains today (TrueFalse)Here we can see some statements anwer would be true or false but some staements answer can not give in terms of true or false Thus the sentences which can be answered in true or false are known as simple propositionAssigning propositon to a variableThe general syntax to assign propostion to a variable is as followsVariable = Simple propositonFor example A=Ram is a clever studentB= Grapes are sweetC= it rains todayCompound proposition

helliphellipto be continued in next classhelliphellipMath Relation Relation If A and B are two non-empty sets

then a relation R from A to B is a subset of AxB If R A x B and (a b) R then we say that a sube isinis related to b by the relation R written as aRbeg Let A be the set of students of class XII and B be the set of students of class XI Then some of the examples of relation from A to B arei) (a b) AXB a is brother of bisinii) (a b) AXB age of a is more than age of isinb Types of relation In this section we would like to study different types of relations We know that a relation in a set A is a subset of A times A Thus the empty set φ and A times A are two extreme relations For illustration consider a relation R in the set A = 1 2 3 4 given by R = (a b) a ndash b = 10 This is the empty set as no pair (a b) satisfies the condition a ndash b = 10 Similarly R = (a b) | a ndash b | ge 0 is the whole primeset A times A as all pairs (a b) in A times A satisfy | a ndash

Example 1 Let A be the set of all students of a boys school Show that the relation R in A given by R = (a b) a is sister of b is the empty relation and R = (a b) the primedifference between heights of a and b is less than 3 meters is the universal relationSolution Since the school is boys school no student of the school can be sister of any student of the school Hence R = φ showing that R is the empty relation It is also obvious that the difference between heights of any two students of the school has to be less than 3 meters This shows that R = A times A is primethe universal relation Example 2 Show that the relation R in the set 1 2 3 given by R = (1 1) (2 2) (3 3) (1 2) (2 3) is reflexive

b | ge 0 These two extreme examples lead us to the following definitionsDefinition 1 A relation R in a set A is called empty relation if no element of A isrelated to any element of A ie R = φ A times AsubDefinition 2 A relation R in a set A is called universal relation if each element of A is related to every element of A ie R = A times A Both the empty relation and the universal relation are some times called trivial relation Definition 3 A relation R in a set A is called(i) reflexive if (a a) R for every a Aisin isin(ii) symmetric if (a1 a2) R implies that (aisin 2a1)

R for all aisin 1 a2 Aisin(iii) transitive if (a1 a2) R and (aisin 2 a3) R isinimplies that (a1 a3) R for all aisin 1 a2 a3 AisinDefinition 4 A relation R in a set A is said to be an equivalence relation if R is reflexive symmetric and transitive

but neither symmetric nor transitiveSolution R is reflexive since (1 1) (2 2) and (3 3) lie in R Also R is not symmetric as (1 2) R but (2 1) isin notinR Similarly R is not transitive as (1 2) R and (2 3) R but (1 3) R isin isin notinExample 3 Show that the relation R in the set Z of integers given byR = (a b) 2 divides a ndash b is an equivalence relationSolution R is reflexive as 2 divides (a ndash a) for all a Z isinFurther if (a b) R then 2 divides a isinndash b Therefore 2 divides b ndash a Hence (b a) R which shows that R is isinsymmetric Similarly if (a b) R and (b c) R isin isinthen a ndash b and b ndash c are divisible by 2 Now a ndash c = (a ndash b) + (b ndash c) is even (Why) So (a ndash c) is divisible by 2 This shows that R is transitive Thus R is an equivalence relation in ZExample 4 Let L be the set of all lines in a plane and R be the relation in L defined as R = (L1 L2) L1 is perpendicular to L2 Show that R is symmetric but neither reflexive nor transitiveSolution R is not reflexive as a line L1 can not be perpendicular to itself ie (L1 L1) R notinR is symmetric as (L1 L2) Risin

L1 is perpendicular to L2rArr L2 is perpendicular to L1rArr (L2 L1) RrArr isin

R is not transitive Indeed if L1 is perpendicular to L2 and L2 is perpendicular to L3 then L1 can never be perpendicular to L3 In fact L1 is parallel to L3 ie (L1 L2) R isin(L2 L3) R but (L1 L3) Risin notin

Chemistry Solid state Characteristics if Solids(i)The particles are locked in fixed positions they are unable to change their relative positions and this brings a definite shape and volume of a solid(ii)In a solid the constituent particles are held by strong forces of attractionThe forces of attraction may be bonding or non bonding(iii)The constituent particles in a solid pack together as closely as possibleoccupying most of the available space within the solidThus the empty space in a solid is very smallThis makes a solid highly rigid and nearly incompressibleThis also explains why a solid has high density and exhibits slow diffusionClassification of Solids

Q1)Define Crystalline solids AnsA Solid that has a definite geometrical shape and a sharp melting pointand whose constituent particles (atomsmolecules or ions) are arranged in a long range order of definite pattern extending throughout the solid is called a crystalline solidExNaClQ2)Define Amorphous solids AnsA solid that does not have a definite shape and a sharp melting pointand whose constituent particles (atomsmolecules or ions) are not arranged in a definite pattern is called an amorphoussolid

Crystalline solidsAmorphous solids

ExGlassRubberQ3)Classify Crystalline Solids Crystalline Solids

Physics Coloumbrsquos Law (Summary)

Before Going Into Coloumbrsquos Law We Will First Learn What is Charge Properties of Charge and Always remember that charge is quantized ie a body always have static charge of magnitude equal to some integral multiple of fundamental electronic charge e= 16 x 10- 19 C

Charge is the property of matter that causes it to produce and experience electrical and magnetic effects The study of the electrical charges at rest is called electrostatics When both electrical and magnetic effects are present the interaction between charges is referred to as electromagnetic

There exist two types of charges in nature positive and negative Like charges repel and unlike charges attract each other

The type of charge on an electron is negative The charge of a proton is the same as that of an electron but with a positive sign In an atom the number of electrons and the number of protons are equal The atom is therefore electrically neutral If one or more electrons are added to it it becomes negatively charged and is designated as negative ion However if one or more electrons are removed from an atom it becomes positively charged and is called a positive ion

The excess or deficiency of electrons in a body gives the concept of charge If there is an excess of electrons in a body it is negatively charged And if there is deficiency of electrons the body becomes positively charged Whenever addition or removal of electrons takes places the body acquires a charge

The SI Unit of charge is coulomb (C) In SI units the current is a fundamental quantity having a unit of ampere (A) The unit of charge is defined in terms of the unit of current Thus one coulomb is the charge transferred in one second across the section of a wire carrying a

Ionic SolidsMetallicSolids

Molecular Solids

current of one ampere

As q = It we have1 C = (1 A) (1 s)

The dimensions of charge are [A T]

Properties of Charge

(1) Quantization of Charge Electric charge can have only discrete values rather than any value That is charge is quantized The smallest discrete value of charge that can exist in nature is the charge on an electron given as

e = plusmn 16 x 10- 19 C

This is the charge attained by an electron and a protonA charge q must be an integral multiple of this basic unit That is

Q = plusmn ne where n = 1 2 hellip

Charge on a body can never be (frac12)e (23)e or 57e etcWhen we rub a glass rod with silk some electrons are transferred from the rod to the silk The rod becomes positively charged The silk becomes negatively charged The coulomb is a very large amount of charge A typical charge acquired by a rubbed body is 10 - 8 C

Biology Reproduction in organisms

Welcome to this new session 2020-21Today in this first chapter we mainly discuss about reproduction types needs and life span of some organismsWe also discuss about difference between sexual and asexual reproduction

Q1 What is reproductionReproduction is defined as a biological processin which an organism gives rise to young onessimilar to itselfQ2 What are the needs of reproductionbulli) Reproduction maintain life on earthii) It enables the continuity of the species generation after generationiii) It creates genetic variation among populationsQ3 Define Life span and write some orgnisms life spanbull Life span is the period from birth to

the natural death of an organism- OrganismsLife span1 Butterfly 1 - 2 weeks2 Fruit fly 30 days3Dog 10-13 years4 Rose5-7 years5 Tortoise100-150 years6 Banyan Tree -200 - 250 yearsQ4 Reproduction is of two types in case ofanimals but in case of plants vegetative propagation is also present

Asexual Reproduction Sexual Reproductioni) Always uniparentalii) Gametes are not involvediii) Only mitotic division involvediv) Somatic cells of parents are involvedv) Offsprings are genetically similar to the parents

i) Usually biparentalii) Gametes are involvediii) Meiosis occurs during gametogenesis Mitosis occurs after fertilisationiv) Germ cells of the parents are involvedv) offsprings are genetically different from the parents

COMMERCE BUSINESS ENVIRONMENT

Welcome to the new sessiontoday we are going to start the first chapter of Class XII The name of the chapter is Business Environment

Already many of you have got some idea about the word business environment form the first chapter of business studies in class XI

In todayrsquos world every business enterprise is a part of the society It exists and operates in association with various groups in society such as customers suppliers competitors banks and financial institutions government agencies trade unions media and so on All these groups influence the functioning of business in one way or the other They constitute the environment of businessConcept of Business Environment

The term lsquobusiness environmentrsquo refers to the sum total of all individuals institutions and other forces that lie outside a business enterprise but that may influence its functioning and performance

The main features of business environment Totality of External forces General and Specific forces Interrelatedness Complexity Dynamic Uncertainty

Prepare the following questions from todayrsquos assignment

2 What do you mean by business environment

The term lsquobusiness environmentrsquo means the aggregate of all forces factors and institutions which are external to and beyond the control of an individual business enterprise but they may influence its functioning and performance Business environment is the macro framework within which a business firm a micro unit operates It consists of several interrelated and interacting elements

2 Explain the main features of business environment in brief

Totality of External forces-Business environment is the sum total of all things external to a business environment

General and Specific forces-It

Relativity

The Interrelation between business and its environment

The business enterprise is an open system It continuously interacts with its environment It takes inputs (such as raw materials capital labour energy and so on) from its environment transforms them into goods and services and sends them back to the environment

Fig 1 Business Environment Relationship

includes both the forces general forces are the economic social political legal and technological conditions which indirectly influence all business enterprise Specific forces are the investors customers competitors and suppliers which influence individual enterprise directly

Interrelatedness-Different elements of environment are interrelated for an example growing awareness for health care has increased the demand for health foods

Complexity- Business environment id complex in nature as the elements keep on changing example economic technological and other forces changes in demand for a product and service

Dynamic-Business environment is not static it keeps on changing

Uncertainty- Itrsquos very difficult to predict future events such as technology and fashion which occur fast and frequently

Business Studies

Human Resources Management

Human resource of an organisation are the aggregate of knowledge skills attitudes of people working in it

The management system which deals with human resources is called human resource management

Features of HRMbullComprehensive functionbullPeople-oriented

Question1) What do you mean by human

resource management Answer) Human resource management may be defined as that field of Management which has to do with planning organising and controlling the functions of procuring developing maintaining and utilising the labour force

bullAction oriented bullPervasive function bullContinuous function

2) Explain the features of HRM in brief

Answer)bullHuman Resource Management is concerned with managing people at work bull Human Resource Management is concerned with employees which bring people and organisations together so that the goals of each are met bullHuman resource management considered every employees as an individual and also promote their satisfaction and growth bull Human resource management is inherent in all organisations and at all levelsbullManagement of human resources are ongoing on never ending process which requires a constant alertness and Awareness of human relations

3) ldquoHR function is said to be pervasiverdquowhy

Answer) Human resource management is required in all organisations whether it is private or government organisations armed forces sports organisations etc It permeatsall the functional areas like production marketing finance research etc This from this feature of human resource management it can be said that it is pervasive in nature

Economics Demand Q1DEFINITION OF DEMANDIn economics demand is the quantity of a good that consumers are willing and able to purchase at various prices during a given period of timeQ2DEMAND CURVEIn economics a demand curve is a graph depicting the relationship between the price of a certain commodity and the quantity of that commodity that is demanded at that pricQ3LAW OF DEMANDIn microeconomics the law of demand states that conditional on all else being equal as the price of a good increases quantity demanded decreases conversely as the price of a good decreases quantity demanded increasesQ4ASSUMPTION of LAW OF DEMAND(i)No change in price of related commodities(ii) No change in income of the consumer(iii) No change in taste and preferences customs habit and fashion of the consumer( No expectation regarding future change in priceQ5MARKET DEMAND SCHEDULEIn economics a market demand schedule is a tabulation of the quantity of a good that all consumers in a market will purchase at a

given price At any given price the corresponding value on the demand schedule is the sum of all consumersrsquo quantities demanded at that priceQ6INDIVIDUAL DEMAND SCHEDULEIndividual demand schedule refers to a tabular statement showing various quantities of a commodity that a consumer is willing to buy at various levels of price during a given period of timeQ7 FACTORS AFFECTING INDIVIDUAL DEMAND FOR A COMMODITY

The factors that influence a consumerrsquos decision to purchase a commodity are also known as determinants of demand The following factors affect the individual demand for a commodity1 price of the commodity2 price of related goods3 income of buyer of the commodity4 tastes and preferences of the buyer1 Price of the CommodityYou must have observed that when price of a commodity falls you tend to buy more of it and when its price rises you tend to buy less of it when all other factors remain constant (lsquoother things remaining the samersquo) In other words other things remaining the same there is an inverse relationship between the price of a commodity and its quantity demanded by its buyers This statement is in accordance with law of demand which you will study in the later part of this lesson Price of a commodity and its quantity demanded by its buyers are inversely related only when lsquoother things remain the samersquo So lsquoother things remaining the samersquo is an assumption when we study the effect of changes in the price of a commodity on its quantity demanded2 Price of Related goodsA consumer may demand a particular good But while buying that good heshe also asks the price of its related goods Related goods can be of two types-(i) Substitute goods(ii) Complementary goods While purchasing a good prices of its substitutes and complements do affect its quantity purchased(i) Price of Substitute Goods Substitute goods are those goods which can easily be used in place of one another for satisfaction of a particular want like tea and coffee An increase in price of substitute good leads to an increase in demand for the given commodity and a decrease in price of substitute good leads to a decrease in demand for the given commodity It means demand for a given commodity is directly affected by change in price of substitute goods For example if price of coffee increases the demand for tea will rise as tea will become relatively cheaper in comparison to coffee(ii) Price of Complementary goods Complementary goods are those goods which are used together to satisfy a particular want like car and petrol An increase in the price of complementary goods leads to a decrease in demand for the given commodity and a decrease in the price of complementary goods leads to an increase in demand for the given commodity For example if price of petrol falls then the demand for cars will increase as it will be relatively cheaper to use both the goods together So demand for a given commodity is inversely affected by change in price of complementary goods3 Income of the Buyer of CommodityDemand for a commodity is also affected by income of its buyer However the effect of change in income on demand depends on the nature of the commodity under consideration In case of some goods like full cream milk fine quality of rice (Basmati rice) etc demand for these commodities increases when income of the buyer increases and

demand for these commodities decreases when income of the buyer decreases Such goods whose demand increases with the increase in income of the buyer are called normal goods But there are some goods like coarse rice toned milk etc whose demand decreases when income of buyer increases and their demand increases when income of the buyer decreases Such goods whose demand decreases with the increase in income of the buyer are called inferior goods Suppose a consumer buys 10 Kgs of rice whose price is ` 25 per Kg He cannot afford to buy better quality of rice because the price of such rice is ` 50 per Kg The consumer is spending ` 250 per month on the purchase of rice Now if income of the consumer increases and he can afford ` 350 on purchase of 10 Kg of rice Now he can afford to buy some quantity of rice say 6 Kgs whose price is ` 25 per Kg and may buy 4 Kgs of rice whose price is ` 50 per Kg Thus he will buy 10 Kgs of rice by spending ` 350 per month Therefore we may conclude that demand for normal goods is directly related to the income of the buyer but demand for inferior goods is inversely related to the income of the buyer4 Tastes and Preferences of the BuyerThe demand for a commodity is also affected by the tastes and preferences of the buyers They include change in fashion customs habits etc Those commodities are preferred by the consumers which are in fashion So demand for those commodities rises which are in fashion On the other hand if a commodity goes out of the fashion its demand falls because no consumer will like to buy it(5) Number of Buyers in the Market(Population)Increase in population raises the market demand whereas decrease in population reduces the market demand for a commodity Not only the size of population but its composition like age (ratio of males females children and old people in population) also affects the demand for a commodity It is because of needs of children young old male and female population differs(6) Distribution of Income and WealthIf the distribution of income and wealth is more in favour of the rich demand for the commodities preferred by the rich such as comforts and luxuries is likely to be higher On the other hand if the distribution of income and wealth is more in favour of poor demand for commodities preferred by the poor such as necessities will be more(7) Season and Weather ConditionsThis is generally observed that the demand for woolens increases during winter whereas demand for ice creams and cold drinks increases during summer Similarly market demand for umbrellas rain coats increases during rainy seasonQ8 REASONS FOR OPERATION OF LAW OF DEMAND WHY DEMAND CURVE SLOPES DOWNWARDNow we will try to explain why does a consumer purchase more quantity of a commodity at a lower price and less of it at a higher price or why does the law of demand operate ie why does the demand curve slope downwards from left to right The main reasons for operation of law of demand are1 Law of Diminishing Marginal UtilityAs you have studied earlier law of diminishing marginal utility states that as we consume more and more units of a commodity the utility derived from each successive unit goes on decreasing The consumer will be ready to pay more for those units which provide him more utility and less for those which provide him less utility It implies that he will purchase more only when the price of the commodity falls2 Income Effect

When price of a commodity falls purchasing power or real income of the consumer increases which enables him to purchase more quantity of the commodity with the same money income Let us take an example Suppose you buy 4 ice creams when price of each ice cream is ` 25 If price of ice creams falls to ` 20 then with same money income you can buy 5 ice creams now3 Substitution EffectWhen price of a commodity falls it becomes comparatively cheaper as compared to its substitutes (although price of substitutes has not been changed) This will lead to rise in demand for the given commodity For example if coke and Pepsi both are sold at ` 10 each and price of coke falls Now coke has become relatively cheaper and will be substituted for Pepsi It will lead to rise in demand for coke4 Change in Number of BuyersWhen price of a commodity falls some old buyers may demand more of the commodity at the reduced price and some new buyers may also start buying this commodity who were not in a position to buy it earlier due to higher price This will lead to increase in number of buyers when price of the commodity falls As a result demand for the commodity rises when its price falls5 Diverse Uses of a CommoditySome commodities have diverse uses like milk It can be used for drinking for sweet preparation for ice cream preparation etc If price of milk rises its use may be restricted to important purpose only This will lead to reduction in demand for other less important uses When price of milk falls it can be put to other uses also leading to rise n demand for itQ9 EXCEPTIONS TO THE LAW OF DEMANDYou have studied in law of demand that a buyer is willing to buy more quantity of a commodity at a lower price and less of it at a higher price But in certain circumstances a rise in price may lead to rise in demand These circumstances are called Exceptions to the Law of Demand Some important exceptions are1 Giffen GoodsGiffen goods are special type of inferior goods in which negative income effect is stronger than negative substitution effect Giffen goods do not follow law of demand as their demand rises when their price rises Examples of Giffen goods are jowar and bajra etc2 Status Symbol GoodsSome goods are used by rich people as status symbols eg diamonds gold jewellary etc The higher the price the higher will be the demand for these goods When price of such goods falls these goods are no longer looked at as status symbol goods and tehrefore therir demand falls3 NecessitiesCommodities such as medicines salt wheat etc do not follow law of demandbecause we have to purchase them in minimum required quantity whatever their price may be4 Goods Expected to be ScarceWhen the buyers expect a scarcity of a particular good in near future they start buying more and more of that good even if their prices are rising For example during war famines etc people tend to buy more of some goods even at higher prices due to fear of their scarcity in near future

Political Science

Constitution of India-The

Preamble

The preamble-

Preamble-

The preamble is the most precious part of the constitution We the people of India having solemnly resolved to constitute India into a Sovereign Socialist Secular Democratic Republic and to secure to all its citizensA preamble is an introductory and expressionary statement in a document that explains the documents purpose and underlying philosophy When applied to the opening paragraphs of a statute it may recite historical facts pertinent to the subject of the statuteNature and purpose of the constitution-Purpose of the Constitution dictates permanent framework of the government to form a more perfect union to establish justice and ensure peace of thenationconstitution provide principles how the government can run itself following the rules and laws written in the constitution of each state keeps them balanced

Answer the following questions-

1 What is preambleA preamble is an introductory and expressionary statement in a document that explains the documents purpose and underlying philosophy2 What is the nature and

purpose of the constitutionConstitution dictatespermanent framework of the government to form a more perfect union to establish justice and ensure peace of the nation

Homework-Learn

Accounts Compatibilty mode

1MEANING OF PARTNERSHIPPartnership is a form of business organisation where two or more persons join hands to run a business They share the profits and losses according to the agreement amongst them According to the Indian Partnership Act 1932 ldquoPartnership is relation between persons who have agreed to share profits of a business carried on by all or any one of them acting for allrdquo For example one of your friends has passed class XII from National Institute of Open Schooling (NIOS) and wants to start a business Heshe approaches you to join in this venture Heshe wants you to contribute some money and participate in the business activities Both of you if join hands constitute a partnership2CHARACTERISTICS1048698 Agreement A partnership is formed by an agreement The agreement may be either oral or in writing It defines the relationship between the persons who agree to carry on business It may contain the terms of sharing profit and the capital to be invested by each partner etc The written agreement is known as partnership deed1048698 Number of persons There must be at least two persons to form a partnership

The maximum number of partners in a partnership firm can be 50 according toCompanies Act 20131048698 Business The Partnership is formed to carry on business with a purpose of earning profits The business should be lawful Thus if two or more persons agree to carry on unlawful activities it will not be termed as partnership1048698 Sharing Profits The partners agree to share profits in the agreed ratio In caseof loss all the partners have to bear it in the same agreed profit sharing ratio10486981048698Mutual Agency Every partner is an agent of the other partners Every partner can bind the firm and all other partners by hisher acts Each partner will be responsible and liable for the acts of all other partners10486981048698Unlimited liability The liability of each partner except that of a minor is unlimited Their liability extends to their personal assets also If the assets of the firm are insufficient to pay off its debts the partnersrsquo personal property can be used to satisfy the claim of the creditors of the partnership firm10486981048698Management All the partners have a right to mange the business However they may authorize one or more partners to manage the affairs of the business on their behalf10486981048698Transferability of Share No partner can transfer hisher share to any one including hisher family member without the consent of all other partners3PARTNERSHIP DEEDAgreement forms the basis of partnership The written form of the agreement is which a document of partnership is It contains terms and conditions regarding the conduct of the business It also explains relationship between the partners This document is called partnership deed Every firm can frame its own partnership deed in which the rights duties and liabilities of the partners are stated in detail It helps in settling the disputes arising among the partners during the general conduct of business 4CONTENTS OF PARTNERSHIP DEEDThe partnership deed generally contains the following (i) Name and address of the partnership firm(ii) Nature and objectives of the business(iii) Name and address of each partner(iv) Ratio in which profits is to be shared(v) Capital contribution by each partner(vi) Rate of Interest on capital if allowed(vii) Salary or any other remuneration to partners if allowed(viii) Rate of interest on loans and advances by a partner to the firm(ix) Drawings of partners and interest thereon if any(x) Method of valuation of goodwill and revaluation of assets and liabilities on the reconstitution of the partnership ie on the admission retirement or death of a partner(xi) Settlement of disputes by arbitration(xii) Settlement of accounts at the time of retirement or death of a partner5IN ABSENCE OF PARTNERSHIP DEEDThe partnership deed lays down the terms and conditions of partnership in regard to rights duties and obligations of the partners In the absence of partnership deed there may arise a controversy on certain issues like profit sharing ratio interest on

capital interest on drawings interest on loan and salary of the partners In such cases the provisions of the Indian Partnership Act becomes applicableSome of the Issues are(i) Distribution of Profit Partners are entitled to share profits equally(ii) Interest on Capital Interest on capital is not allowed(iii) Interest on Drawings No interest on drawing of the partners is to be charged(iv) Interest on Partnerrsquos Loan A Partner is allowed interest 6 per annum on the amount of loan given to the firm by himher(v) Salary and Commission to Partner A partner is not entitled to anysalary or commission or any other remuneration for managing the business

History TOPIC-TOWARDS INDEPENDENCE AND PARTITION THE LAST PHASE (1935-1947)

SUB TOPIC-IMPORTANT POLITICAL DEVELOPMENTS ndash GROWTH OF SOCIAL IDEAS

Socialism is a political social and economic philosophyLike in other parts of the world the Russian revolution of 1917 served as a great inspiration for revolutionaries in India who at that time were engaged in the struggle for liberation from British ruleSocialist ideas led to the formation of communist party of IndiaJAWAHARLAL NEHRU Among the early Congress leaders Jawaharlal Nehru was very much impressed and influenced by the Socialist ideas He also learnt about the Economic activities of the Soviet Union after the Bolshevic Revolution 1917 He made full use of them in IndiaThe election of Jawaharlal Nehru and Subhas Chandra Bose showed the Left wing tendency within CongressJawaharlal Nehru demanded economic freedom along with political freedom of the people in order to end the exploitation of masses

Nehrus working committee included three socialists leaders The Lucknow session was a landmark in the evolution of socialist ideas of the congressSUBHAS CHANDRA BOSE ndash Subhas Chandra Bose had socialist leaning Both Jawaharlal Nehru and Subhas Chandra Bose were known as leftist Congress men Later on National Congress divided into Leftist and rightist campCONGRESS SOCIALIST Within the Congress some leaders formed the Congress Socialist partyPattavi Sitaramyya Sardar Patel Rajendra Prasad had hostile attitude towards the Congress Socialist partyJawaharlals attitude was hesitant

1 QUESTION ndash Mention name of two Congress leaders who had socialist leaning

1ANSWER ndash Subhas Chandra Bose and Jawaharlal Nehru2QUESTION- In which session of the congress Jawaharlal elaborated his Socialist ideas2 ANSWER ndash Lucknow and Faizpur Session in December 1935 and 19363QUESTION ndash Why Congress was sharply divided into leftist and rightist camp 3ANSWER ndash Subhas Chandra Bosersquos attempt to seek re election for congress presidentship in 1939sharply divided the National Congress into Leftist and Rightist camp4 QUESTION ndash Who was MN Roy 4 ANSWER ndash Manabendra Roy first formed the Communist Party of India outside the country at Tashkent in 19205QUESTION ndash Who formed the Congress Socialist Party within the Congress5 ANSWER ndash Jaya Prakash Narayan Achyut Patwardhan Acharya Narendra Dev Ram Mohan Lohia Aruna Asaf Ali6QUESTION ndash When was the Congress Socialist Party formed What was its object6 ANSWER ndash 1934The Congress Socialist Party sought to work out socialist programme through the Congress They joined hands with the Congress and wanted to carry

Subhas Chandra Bose being expelled from the congress after the Tripuri rift he formed Forward BlockThere were basic differences between the Congress Socialists and the communistsTRADE UNION ACTIVITIES Maximum working class people lived in Bombay and Calcutta The working and living conditions of those workers were very miserable In this situation Shasipada Banerjee NM Lokhande protested against the oppression of the working class peopleThe first Trade Union Madras Labour Union was formed in 1918 by BP WadiaIndustrial strikes took place in Kanpur Calcutta Madras Jamshedpur and Ahmedabad AITUC was formed in Bombay in 1927 The growth of Trade union among the workers was slow because of the fear of the dismissal of the jobIn the mean time the Moderates as well as Communists left AITUC and formed separate organization

on National struggle with the help of workers and peasant class of the society7 QUESTION ndash What was the name of the party founded by Subhas Chandra Bose7 ANSWER- Forward Block8QUESTION ndash Who was Shasipada Banerjee8 ANSWER ndash Shasipada Banerjee was a radical Brahmo He founded a working menrsquos club to protest against exploitation of the British rulers towards the working class of India9 QUESTION ndash What was the weekly published by NM Lokhande9ANSWER- Dinabandhu10 QUESTION ndash Who founded Bombay Mill-Hands Association and in which year10 ANSWER- NM Lokhande in189011 QUESTION- Who was BP WadiaANSWER- BPWadia was the founder of Madras Labour Union in191812 QUESTION- What was the name of the first labour union of India12 ANSWER- Madras Labour Union13 QUESTION Who founded the Majur Mahajan 13 ANSWER GANDHIJI14 QUESTION What was the full form of AITUC When it was formed14 ANSWER All India Trade Union Congressin 192715QUESTION Who formed the Red Trade Union Congress and in which year15ANSWER The Communists formed the Red Trade Union Congress16 QUESTION What do you mean by Socialism16 ANSWER Socialism describes any political and economic theory that says the community rather than individuals should own and manage property and natural resources

Subject Eng Literature (The Tempest ndash William Shakespeare) Topic Act III Scene 3 Lines 1 to 52 (Line 52 ndash Brother my lord the Duke Stand to and do as we) Date 13th April 2020 (4th Period)

[Students should read the original play and also the paraphrase given in the school prescribed textbook]Summary Questions amp Answers

o Alonso Sebastian Antonio Gonzalo Adrian Francisco and others wandered about the island in search of Ferdinand and gets tired and hungry of the toil and at the same time gives up all hope of finding him

o Antonio and Sebastian are happy that Alonso is out of hope and decide to make another attempt on his life that night when being so tired they will be sleeping soundly

o Suddenly a solemn and strange music is heard in the air and several strange shapes enter bringing in a banquet These strange shapes then dance round it with gestures of salutation and then inviting the King to eat they depart

o Seeing this strange scene all are inclined to believe the tales told by travelers that there truly are ldquounicornsrdquo and ldquothe phoenixrsquo thronerdquo

1 ALONSO What harmony is this My good friends hark (L18-27)

GONZALO Marvellous sweet music

[Enter several strange shapes bringing in a banquet

they dance about it with gentle actions of salutation

and inviting the King and his companions to eat they depart]ALONSO Give us kind keepers heavens What were theseSEBASTIAN A living drollery Now I will believe

That there are unicorns that in Arabia

There is one tree the phoenixrsquo throne one phoenix

At this hour reigning thereANTONIO Ill believe both

And what does else want credit come to me

And Ill be sworn rsquotis true Travellers neer did lie

Though fools at home condemn rsquoem

(a) How did Prospero present an amazing spectacle before Alonso and his companions

Using his magic powers Prospero ordered strange shapes to lay a banquet before Alonso and his companions The shapes brought several dishes with tasty eatables in them They placed the dishes on a table before Alonso and his companions Then the strange shapes began to dance gracefully around the banquet While dancing they made gestures inviting them to eat the food Then suddenly the shapes disappeared(b) Who were the guests at the strange banquet Describe the lsquoliving drolleryrsquo

Alonso Sebastian Antonio Gonzalo Adrian and Francisco were the guests at the strange banquet

The term ldquoliving drolleryrdquo refers to live entertainment show In this context when Alonso the King of Naples Sebastian his brother Antonio the treacherous brother of Prospero Gonzalo the kind and loyal councillor to the King Adrian and Francisco came to the island they were hungry and weary in their spirits They heard a solemn and strange music They were shocked to see several strange shapes bringing in a banquet and these shapes danced about it with gentle action of salutation inviting the King and his companions to eat After this Sebastian described this show as lsquoliving drolleryrsquo(c) What is lsquophoenixrsquo What are lsquoUnicornsrdquo

The term lsquophoenixrsquo refers to a mythical Arabian bird which lived alone and perched on a solitary tree After one hundred years it expired in flames and rose again from its own ashes

lsquoUnicornsrsquo refers to the mythological four-footed beasts having horns in the centre of their foreheads When the horns are ground into powder the powder was believed to be

an aphrodisiac(d) How does Sebastian explain the puppet show OR Why does the speaker now believe in unicorns and phoenix

Sebastian finds several strange shapes bringing in the banquet They invite the king and his party for dinner and soon depart He tells that if such a strange sight can be a reality there is nothing incredible in the world and from the present moment he will believe anything He says that it is a strange dumb show enacted not by puppets but by living beings It is stranger than a travellerrsquos tale Seeing such a thing

before his own eyes he will no longer disbelieve the story about unicorns and phoenix(e) How do the other characters present respond to this living drollery

At the sight of the lsquoliving drolleryrsquo like Sebastian Gonzalo and Antonio too acted strangely Antonio told that he too now believes in unicorns and phoenix and anything else that seems to be incredible He too now believes in travellersrsquo tales Gonzalo told that if he would report those happenings in Naples nobody will believe him He considers that those gentle shapes were gentler in manner in comparison to the living beings Alonso was at first sight suspicious and told them that those strange shapes conveyed their meaning in expressive gestures when they seemed to lack speech by their movements and sounds Francisco was amazed at their mysterious disappearance

2 ALONSO Not I

(Line 43-52)GONZALO Faith sir you need not fear When we

were boysWho would believe that there were mountaineers

Dewlapped like bulls whose throats had hanging at rsquoem

Wallets of flesh Or that there were such men

Whose heads stood in their breasts Which now we find

Each putter-out of five for one will bring us

Good warrant ofALONSO I will stand to and feed

Although my lastmdashno matter since I feel

The best is past Brother my lord the Duke

Stand to and do as we

(a) How does Alonso respond at the spectacle of the shapes which were sent to them at the instruction of Prospero

After seeing the strange sight of appearing and disappearing of the shapes sent by Prospero to arrange a banquet for them Alonso says that his surprise at having seen those creatures is infinite and he is fully justified in feeling so much surprise He thinks that their shapes their gestures and the sounds they made were indeed amazing Although they do not possess the gift of speech yet they were able to convey their

thoughts by means of their gestures only

(b) What does Prospero say about the views expressed by Alonso regarding the shapes What does Francisco think about the shapesAfter hearing Alonsorsquos views about the shapes Prospero says that this manrsquos praise of the spirits is rather hasty He means to say that Alonso has shown great haste in reaching the conclusion about the shapes Francisco is amazed to see that those shapes disappeared in a mysterious way(c) What does Sebastian ask Alonso to doSebastian tells Alonso that the shapes having disappeared should not matter to them because they have left the eatables behind He asks Alonso to enjoy eating as they are extremely hungry but the king does not accept his offer of enjoying the dishes(d) How does Gonzalo try to dispel Alonsorsquos fear of those strange shapes What kind of references does he give to AlonsoGonzalo says that those who have travelled abroad have reported seeing even stranger sights than these shapes that Alonso and his companions have beheld Hence there is no reason to feel afraid of these shapes Gonzalo further adds that in his younger days he had heard strange stories from travelers and Alonso might have heard similar stories For instance it was said that there existed a certain race of

human beings who had huge lumps of flesh hanging at their throats and who therefore resembled bulls Then Gonzalo tells about a race of human beings whose heads were located at their breasts Gonzalo says that such stories were not believed by most people in those days but now-a-days these stories have become common(e) Explain the following lsquoEach putter-out of five for onersquoEnglish travellers often insured their trips with London brokers Those that went on foreign travels those days used to deposit a certain amount with some firm or company in London before their departure If the travelers failed to return the money was forfeited by the company with which it had been deposited But this money was repaid five-fold if the travelers returned safe and sound In this way a traveler stood a great chance of recovering the entire cost of his

travels(f) Give the explanatory meanings of the following expressions in the context of the above extract (i) Dewlapped (ii) Wallets of flesh

(iii) Putter-out(i) Dewlapped having big lumps of flesh at the necks(ii) Wallets of flesh large masses of flesh looking like bags(iii) Putter-out to invest money before commencing the travel

  • General methods of preparation of hydrogen
  • Chapter Dimensional Analysis (Summary)
    • Properties of Charge
Page 23:   · Web viewSubject. Topic. Summary. Execution. Hindi. व्याकरण. शरीरके अंगो के नाम लिखिए. 1) आँख 2) नाक 3

married(e) Explain the meaning of the first two lines of Portiarsquos speech

Portia says that even if she is to live for centuries like Sibylla she would not marry except in accordance to her fatherrsquos will She asserts that she would not mind remaining unmarried and untouched by a man like Diana the virgin the goddess of hunting unless a man is able to win her by passing the test laid down by her father

Class XSubject Topic Summary Execution

Hindi 2nd

Langबड घर की बटी( मशी परमचद)

lsquoबड घर की बीटीrsquo कहानी का उददशय मधयम वग की घर समसया को सझा कर सगदिठत परिरवार म मिम जकर परम स रहन का सदश दना ह घर म शानित tानिपत करन की जिजममदारी नारी की होती ह यदिद नारी समझदार ह उसम धय और परिरवार क परनित परम ह तो कोई भी घटना परिरवार को निवघदिटत नही कर सकती या कहानी परिरवार को सगदिठत करत हए परम सौहाद स एक रदसर की भावनाओ को समझ करउनका सहयोग करत हए जीवन यापन करन की पररणा दती ह मशीपरमचदर जी न इस कहानी म सयकत परिरवार का परनितनिनमिधतव निकया ह यह कहानी बनी माधव सिसह जो गौरी पर क जमीदार क उनक दो पतरो की हशरी कठ ा निबहारीशरीकात का निववाह एकजमीदार घरान की पतरी आनदी स हआ थाआनदी न द को ससरा क वातावरण म ढालिया थाएक दिदन आनदी का अपन दवर ा निबहारी स झगडा हो जाता ह दोनो भाई एक रदसर स अग होन की कोलिशश करत हसभी बह आनदी न अपन मधर वयवहार स ा निबहारी को घर छोडकर जान स रोक लिया| इस पर बनी माधव सिसह न कहा निक बड घर की बटी ऐसी ही होती ह जो निबगडा काम बना ती ह अतः शीषक साथक ह बड घर की बटी आनदी ह

helliphelliphelliphellipContinue to nextBiology Topic ndash Chp-1

CellWelcome to new session 2020-21Today we will start with Chpter 1 cell CELL

Protoplasm+Cellmembrane Or Cell wall

Cytoplasm+Neucleus

Cytoplasmic+ CytoplasmicOrganelles Inclutions(mitochondria (food Golgi bodies pigments)Ribosome)

What is cellbull Cell is the structural and functional unit of living organismbull According to number of cells organisms areUnicellular - Amoeba bacteria Multicellular - Rose Mango Tiger HumanSmallest cell -bacteria Longest cell - Nerve cellLargest cell - Ostrich egg cellCells are of different size and shapes according to their functionsQ2Write chief functions of following cellorganelles

Q3What is tonoplastVacuoles covered by a covering called tonoplast

Bengali(2Nd

Language)

ফ ফটক ো ফটক (কলিবতো ) ভোষ মসোপো3 gtPোয়

একটি লেমসয়র ীবস লেপরম লিকভোসব ফসট ওসঠ তো লেদলিসয়স4 কলিব লেপরম Pই য় লেই ময়ই বনত কোস পলিরত য় ফ লেফোটো বো োসফোটো লেটো ব2 কো য় লেমসয়সদর ব gtয13 লেপরসমর 4লিব ফসট উসঠস4 এই কলিবতোয় লেপরম মোষসক মত gtযর মস লেফস লিদসয় পরকষস বাোচোসোর gtয োত বো2োয় কলিবতোয় লেমসয়টির পসব13র দঃসর কো বো সও লেমসয়টি লেই পসর পলিক সত চোয়ো োরী ীবসর কোস4 পরম লেPৌবস লেপরমসক পোবোর পরব ইচছো োকসও তো পসর লেলিতবোচকতোয় পলিরত য় কলিব ভোষ মসোপো3 যোয় লেP ক লেপরসমর

কলিবতোয় ব gtযবহত লিবসষ লিক4 সvর অ13 লেদওয়ো ১) রসবোো= লেP লিবলিভনন রকম ডোকসত পোসর২) ো= পোর ৩) ঠলি = লেচোসর বZ৪)আই বস2ো=অলিববোলিত৫)শইসয় = োলিয়ত কসর৬)োতপাোচ= লিবলিভনন পরকোর৭)দ2োম = v কসর বZ কসর লেদওয়ো৮)লেরলিং =লেোোর দৈতরী লেব2ো৯) বনত= একঋত১০) পাোর = বসকরো2

Organelles Functions

1 Endoplasmic reticulum

2 Mitochondria

3Golgibodies

4 Ribosome5Lysosome

6Plastids

7 Centrosome

i) Supportive framework for the cellii) Synthesis and transpost of proteinsRelease of energy in the form of ATPi) Synthesis and secretion of enzymes hormoneii) Formation of vacuoles lysosomei) Protein Synthesisi) Intracellular digestionii) Destroy foreign substancei )Leucoplast - stores starchii)chloroplast - trap solar energyiii) Chromoplast - imparts colour toflowers amp fruitsi) Initiates and requlates cell division

কলিবতো তোর অ13সক ভোষোয় পরকোো কসর ঘলিরসয় ব যকত কসরস4 লেপরসমর ফতো আর লিবফতো লেক গোঢ় কসর লেদোসো কলিব ভোষ মসোপো3 যোসয়র অলিভবসর অ যলিদক

Economics

Factors of Production

Welcome to the new sessionToday we are going to start the first chapter of Class XThe name of the chapter is Factors of productionBy the name I hope you all can recall a glimpse of what you have learnt in the second chapter of Class IX

NowProduction is the process of creating the various goods and services which are consumed by the people of the country to satisfy their wants

Thus it is the process in which some materials are transformed from one form to another to create utility and value in goods

For example utility can be created by changing the form of a commodity ie

Making of table out of wood by a carpenter for his customer here the wood is getting transformed into table creating utility for his customer and he can also command a price for it

On the other hand Housewives perform very

useful activities at home which create utility but their domestic activities are not included in production because they have no money value

So we can also say that Production denotes two things firstly creation of utility and secondly creation of value

Production is not complete unless it reaches the consumer

An increase in production will increase the economic welfare of the consumers and hence the aim is to raise the production level of the country

Again production of a good or service is only possible if certain resources or

Questions

1 What do you mean by production

Answer Production means the creation of goods and services for the purpose of selling in the market

In fact production involves the transformation of inputs into outputs

Hence production denotes two thingsCreation of utility and creation of valueUtility and value can be created by changing the form by changing the place by changing the time and by rendering services

Example Transformation of raw

materials into finish goods such as potter creates utility by converting mud into utensils assembling of small parts to make bigger machinery

Production also includes services such as distribution and marketing

2 What are the factors of production

Answer Factors of Production refers to the resources and inputs needed for producing goods and servicesThese inputs can be classified as

Land Labour

Capital Enterprise

Land Land is defined to include not only the surface of the earth but also all other free gifts of nature(for example mineral resources forest resources and indeed anything that helps us to carry out the production of goods and services but is provided by

inputs are used together in right proportion

A resource or an input which helps in the process of production to obtain an output is called FACTOR OF PRODUCTION

These factors of production can broadly be categorized into four parts 1LAND 2LABOUR3CAPITAL4ENTERPRISE (ORGANISATION)or Entrepreneur

The above factors are all interdependent on each other and they play a major role in production process

FACTORS OF PRODUCTION

LANDCAPITAL

LABOUR ENTREPRENEUR

nature free of cost)LabourLabour refers to the human efforts that need to be combined with other factors of production for creating an output

CapitalAll man ndash made means of production is called capita example machineries which help in further production Money when used for starting any business for purchasing raw materials machinery tools etc it is regarded as capitalCapital also includes physical capital like factories machineriestoolsbuildingsequipments etcEnterpriseThe task of bearing risks is called enterprise and the person who bears these risks of business is called the entrepreneurThus an entrepreneur is one who organises production takes important decisions regarding production hires and purchases factors of production and bears the risk and uncertainty involved in productionOrganisation refers to the services of an entrepreneur who controls organises and undertakes all risks One who plans organises and manages a business enterprise is an organiser

Physics Chapter 1 Force

Force is an external agent capable of changing the state of rest or motion of a particular body It has a magnitude and a direction The direction towards which the force is applied is known as the direction of the force and the application of force is the point where force is applied The Force can be measured using a spring balance The SI unit of force is Newton (N)

Question 1

State the condition when on applying a force the body has

(a) the translational motion

(b) The rotational motion

Solutions

(a) Translational motion is produced when the body is free to move

(b) Rotational motion is produced when the body is pivoted at a point

Question 2

Define moment of force and state its SI unit

Solutions

The moment of force is equal to the product of the magnitude of the force and the perpendicular distance of the line of action of force from the axis

of rotation

The SI unit of moment of force is Newton times meter

= Newton meter (Nm)

Commercial Studies

Stake holders In this topic you will be come to know about the meaning and concept of stakeholders

How stakeholders are different from shareholders

Questions1 What do you mean by the term stake holdersAnswer) The term stake holders have developed from the words which mean an interest or expected benefit Stakeholders mean all those individuals groups and Institutions which have a state (interest) in the functioning and performance of a commercial organisation or a business enterprise2 What do you mean by share holdersAnswer) The person and Groups who own the shares of the joint stock company by providing capital to the company are called shareholders Shareholders are the internal stakeholders shareholders are one out of several stake holders3 How are shareholders different from stakeholdersAnswer)i) The term shareholders is related to only joint stock company whereas stakeholders are related with all business organisationsii) Stakeholders maybe any individual having financial stake in business organisation whereas a shareholders are those individuals who are holding shares in the company4) How are shareholders different from creditorsAnswer) i) Shareholders are internal stakeholders while creditors are external stakeholdersii) Shareholders invest in the capital of the company whereas creditors give loan to the companyiii) Shareholders are the members of the company with voting rights but creditors are not the members of the company

English 1 Transformation of sentences

Sentences A sentence is a group of words which makes complete sense

e Assertive sentencesf Imperative sentencesg Interrogative sentencesh Exclamatory sentences

Sentences can be changed from one grammatical form to another without changing the meaning of the sentence This is known as transformation of sentences

Exercise 1 Change the following affirmative sentences into Negative sentences

a He is a good manHe is not a bad man

b Ram loves SitaRam is not without love for Sita

c Only he stood first in the classNone but he stood first in the class

d Ankit was wiser than he

He was not so wise as Ankite He did it

He did not fail to do itf As soon as I reached college the

bell rangNo sooner did I reach college than the bell rang

g He finished everythingHe left nothing unfinished

h It always pours when it rainsIt never rains but it pours

Math Topic Commercial MathematicsChapter ndash Goods and services Tax

What is GSTAns It is a abbreviated term of Goods and Service Text which is an indirect tax levied on the sale of goods and rendering servicesSome terms related to GSTDelar Any person who buys goods or services For resale is known as a delar A delar Can be a firm or a companyIntra-state sales Sales of goods and services within the same state or same union territory are called intra- state salesInter-state sales Sales of goods and services outside the state or union territory are called Inter-state sales4) Input GST GST is paid by dealers on purchase of goods and services are called input GST5) Output GST GST is collected from customers on sale of goods and services are called output GST6) Types of GST There are three taxes applicable under GST(i) Central Goods and Services Tax (CGST)(ii) State Goods and Services Tax (SGST) or Union Territory Goods and Services Tax (UTGST) Both these taxes are levied on intra-state sales Here GST is divided equally among central and state governments(iii) Integrated Goods and Services Tax (IGST) IGST is levied on inter- state sales It is also levied on import of goods and services into India and export of goods and services from India

Subject Eng Literature (The Merchant of Venice ndash William Shakespeare)Topic Act III Scene 4 Lines 1 to 44 (Portia hellip To wish it back on you fare you well Jessica)[Students should read the original play and also the paraphrase given in the school prescribed textbook]

Summary Questions amp AnswersIn this scene we suddenly find a new element in the character of Portia We have already seen her possessed of every graceful womanly quality but now she shows that she is capable of rapid decision and determined action She shows this by her sudden resolve to hasten to Venice with a daring scheme for the rescue of Antonio This is an important scene in the dramatic action for it leads up to and renders possible the striking events of the famous trial scene which is one of the greatest striking elements of the play Moreover the fact that all the characters of importance are now assembled together in Venice makes the union of the main plot and the secondary story complete

(1) LORENZO Madam although I speak it in your presence(Line 1-9)

You have a noble and a true conceit

Of god-like amity which appears most strongly

In bearing thus the absence of your lordBut if you knew to whom you show this honourHow true a gentleman you send reliefHow dear a lover of my lord your husbandI know you would be prouder of the workThan customary bounty can enforce you

(a) Where is Lorenzo Why is he here To whom is he referring as lsquoMadamrsquo

Lorenzo is at Portiarsquos residence He had met Salerio on the way and Salerio had begged him to come along with him to

o In this scene Portia Nerissa Lorenzo Jessica and Balthazar appear

o Portia requests Lorenzo and Jessica to be in charge of her house during her absence from Belmont because she and Nerissa have decided to spend the days in meditation and also in visiting the holy places in the neighbourhood of Belmont She has already instructed her people to acknowledge both Lorenzo and Jessica as master and mistress of house during her absence Lorenzo and Jessica gladly agree to look after the house of Portia

handover the letter from Antonio to Bassanio The letter carried the bad news about Antoniorsquos arrest for non-payment of loan taken from Shylock Hence Salerio might have preferred company to break this bad news to Bassanio He is referring to Portia as Madam(b) What does Portia say on hearing the above extract

Portia says that she has never regretted doing good to others Friends who spend a lot of time together and really are there for each other have many traits in common As Antonio is Bassaniorsquos best friend saving him is like saving Bassanio who is like her own soul She asks Lorenzo to take care of management of the house till Bassanio is back(c) What does Portia send with Bassanio and why

On hearing about Antoniorsquos troubles on account of Bassanio her husband Portia immediately sends him with enough gold to repay the debt many times over to Venice to help Antonio out of his misfortune

(2) Lorenzo Madam with all my heart (Line 36-40)

I shall obey you in all fair commands

Portia My people do already know my mindAnd will acknowledge you and JessicaIn place of Lord Bassanio and myselfSo fare you well till we shall meet again

(a) Where are Lorenzo and Portia at this time What lsquofair commandsrsquo are given to Lorenzo

Lorenzo and Portia are at Belmont during this scenePortia reveals to Lorenzo that she has sworn to contemplate in prayer at a monastery around two miles away until her husband returns from Venice She tells him that Nerissa would accompany her and asks him to manage the house with Jessica till things are settled In response Lorenzo tells her that he would be obliged to do whatever she asks him to do(b) Where is Portia actually going and why

Portia tells Lorenzo that she would live a life of contemplation and pray at a monastery which is two miles away from her place In reality Portia plans to go to Venice in disguise with Nerissa and argue the case in defense of Antonio She is very sure that her plan would succeed

ClassXI (ScienceHumanitiesCommerce)Subject Topic Summary Execution

Computer Science

(APC)

Ch ndash 1 Numbers

(Numbers in different bases and

their Arithmatical operations)

Number System In computers Number System is defined as a writing system to represent the numbers in different ways ie we are using different symbols and notations to represent numbers There are four ways we can represent the number ndash Binary Decimal Octal and Hexadecimal

Decimal Number SystemThis number system consist 10 digits These are 0 1 2 3 4 5 6 7 8 amp 9

Binary Number SystemThis number system has only two digits these are 0 and 1 Here 0 stands for off while 1 stands for on

Octal Number SystemThis number system has 8 digits these are 0 1 2 3 4 5 6 amp 7

Hexadecimal Number SystemThis number system has 16 digits these are 0 1 2 3 4 5 6 7 8 9 A B C D E F Here the value of the alphabets are as follows A=10 B=11 C=12 D=13 E=14 F=15

Rules for conversion decimal number to Binary1 Divide the decimal number by 22 If the number will not divide equally by 2 then round down the answer to the nearest whole number (integer)3 Keep a note of the remainder it should be either 0 or 14 Keep repeating the above steps dividing each answer by 2 until you reach zero5 Write out all the remainders from bottom to top This is your binary solution

For example Lets convert 32 to binary 2 32 2 16 - 0 2 8 - 0 2 4 - 0 2 2 - 0 2 1 - 0 0 - 1

The binary equivalent of 3210 is 1000002

Try the follwing youself1 2410

2 4810

3 1210

History GROWTH OF NATIONALISM

The second half of the 19th century witnessed growth of political consciousness and a sense of Nationalism among the IndiansThere were various factors for growth of Indian Nationalism- As a result various political associations were formed in different provinces by the educated Indians Surendranath Banerjee organized a meeting of National conference at Calcutta Ultimately the National Congress was founded in Bombay in 1885This body became the vanguard of Indian struggle for freedom The congress leaders were known as moderates because they followed a policy of prayer and petition A large number of Indian leaders had experienced in political agitation The Political situation of England was also changed Moreover increasing revolutionary activities in Maharashtra Punjab and Bengal became serious concern to the British Government In this

QUESTION1 What do you mean by Nationalism ANSWER 1 Nationalism is defined as loyalty and devotion to own nation especially a sense of national consciousnessQUESTION 2 What are the causes of nationalism ANSWER 2 There were various factors for growth of nationalism

1 Spread of western education2 The progress of vernacular press and

patriotic literature3 The economic exploitation of our

country by the colonial rulers4 International affairs

QUESTION 3 Who organized National conference in Calcutta in 1883 ANSWER 3 Surendranath BanerjeeQUESTION 4 When did Indian National Congress formANSWER 4 Indian National Congress was formed in 1885 in BombayQUESTION 5 Who were ModeratesANSWER 5 The Early Nationalists were also known as Moderates Their emergence marked

background Lord Curzon became Viceroy in India He had no respect for the Indian National Congress

the beginning of the organized national movement in India They believed in British justice and were loyal to them They followed a policy of prayer and petition They demanded constitutional reforms of our country Impotant Moderate leaders were Pherozshah Mehta Dadabhai Naorozi and Surendranath Banerjee etcQUESTION 6 What do you know about Extremism in Indian National movementANSWER 6 In the beginning of 20th century a new class of national leaders emerged in India which was different from the moderate groups They started more aggressive movement against the British empire The goal of extremists was ldquoswarajrdquo Important extremist leaders were Bal Gangadhar Tilak Lala Lajpat Rai Bipin Chandra Pal etcQUESTION 7 Mention the places which were the main centres of Revolutionary movementANSWER 7 Maharashtra Bengal and Punjab

Physics

Chapter Dimensional Analysis

(Summary)

The dimensions of a physical quantity are the powers to which the fundamental units are raised in order to obtain the derived unit of that quantit

The physical quantites lengthmasstime are represented by [L] [M] [T] resp let they are raised to powers ( dimesions) abc resp then any physical quantity can be represented by [ La Mb Tc ] Examples

1 Area area = L x B = [L] x [L] = [M0 L2 T0 ]

2 Density density = massvolume = [M][L3] = [ M L-3]

3 Velocity velocity = distancetime = [L][T] = [LT-1]HW Try to find out dimension of acceleration Acceleration = velocity timeNB One can find the SI Units Using Dimension Analysis Such as for area we have [L2] so its SI unit is m2

Biology Topic ndash Chp-1 The living world

Today we will start the first chapter the living world Here we discuss about the characteristics of living organism and what are the difference between them and nonliving substances We also discuss about the contribution of different Scientists

There are over 500000 species of plants andover a million species of animal are present on earth Some 15000 new species were discovered every yearQ1 What is a living organismbull A living organism is primarily physico -chemical material that demonstrate a high degree of complexity is capable of selfRegulation possesses a metabolism and perpetuates itself through timeQ2 What are the differences between livingand non-livingsi) Compared with non-living living organisms

have more complex organised structure and their use of energy is more controlled amp efficientii) Living things reproduce their own kind by forming new cells which contains copies of their genesiii) Each organism has some degree of homeostasisie it is able to make adjustments so that internal environment remains constantQ3 Write contributions of following Scientists i) Aristotle - One of the first theories in Biology places all living things in a hiearchieii) AV Leeuwenhoek - was the first to observe living single celled organisms under microscopeii) Carolus Linnaeus - developed the binary system for naming of organisms and classificationiii) Geregor Johann Mendel ndash discoverbasic principles of inheritanceHomework i) C Darwin ii)Schleiden

Math Trigonometric functions

1 Overviewi) Trigonometry The word lsquotrigonometryrsquo is derived from the Greek words lsquotrigonrsquo and lsquometronrsquo which means measuring the sides of a triangle An angle is the amount of rotation of a revolving line with respect to a fixed line Usually we follow two types of conventions for measuring angles ie a) Sexagesimal system b) Circular system In Sexagesimal system the unit of measurement is Degree In Circular system the unit of measurement is Radian ii) Relation between degree and radianThe ratio of circumference of a circle to its diameter is always a constant This constant ratio is a number denoted by π which is taken approximately as 227The relationship between degree amp radian measurements is as follows2 right angles = 180deg= π radians1radian = 180degπ=57deg16(approx) 1deg=π180 radianiii) Length of an arc of a circleIf an arc of length s subtends an angle θ radians at the center of a circle of radius r then s=rθiv) Area of a sector of a circleA sector is like a pizza slice of the

Q) Express the following angles in radiana) 45deg b) 40deg3730Ans a) We have 180deg=π radiansi e 45deg= πtimes45180 radian = π4 radiansb) 40deg3730= 40deg37+3060 minute= 40deg 37 +12 minute= 40deg+ 752 minute=40 + 75(2times60) degree=3258 degreeNow 180deg=π radianie 3258 degree= (πtimes325) (180times8) radians = 65π288 radiansQ) A circle has a radius of r=12 meters What is the length of an arc traced out by a 60deg angle in the center of the circleAns In this problem we know both the central angle (60deg) and the radius of the circle (12) All we have to do is plug those values into our equation and we get

s = 2π(12)(60360)s = 24π6s = 4πSo the length of an arc traced out by a 60deg angle in a circle with a radius of 12 meters equals 4π meters asymp 1257 metersQ) Find the area of the sector with a central angle 30deg and a radius of 9cmAns GivenRadius r = 9 cmAngle θ = 30degArea of the sector = θ360degtimesπr2

= 30360degtimes227times92=2121cm2

circle It consists of a region bounded by two radii and an arc lying between the radiiThe area of a sector is a fraction of the area of the circle This area is proportional to the central angle In other words the bigger the central angle the larger is the area of the sectorArea of Sector = θ2 times r2 (when θ is in radians)

Area of Sector = θ times π360 times r2 (when θ is in degrees)

COMMERCE

CLASSIFICTION OF HUMAN ACTIVITIES-ECONOMIC AND NON-ECONOMIC

Welcome to the new sessiontoday we are going to start the first chapter of Class XI The name of the chapter that we are going to start is

lsquoClassification of Human Activities ndasheconomic and non-economicrsquo

Now let us start the chapter by considering human beings and the activities they perform throughout the day

Human activities means all those activities that human beings undertake to satisfy their wants

Human wants on the other hand are the desire of human beings for goods (vegetables fruits rice etc) and services (services of doctors teachers lawyers etc) that they require to live

Now these human activities continue throughout life as human wants are unending unlimited and recurring as human beings desire for better living throughout their lives

Now human activities can be classified into two categories

Human activities

Economic activities Non-economic activities

Economic activities are

Questions1 What are human activities

Answer Human activities mean all those activities that human beings undertake to satisfy their wants

Example A man working in an office

A boy playing in the garden

2What are the characteristics of human activitiesAnswer the characteristics of human activities are as follows

Human activities are undertaken by men women and children and these activities involve human efforts

Human activities are undertaken to satisfy human wants which are unlimited

Human activities continue throughout life

Human activities are performed for both earning money and personal satisfaction

3What is economic activitiesGive example

Answer Economic activities are undertaken by human beings with the object of earning money acquiring wealth and thereby satisfying human wantsExample

Selling of goods by a shop keeper to his customer

A clinic run by a doctor Service of a teacher in school or college

undertaken by human beings with the object of earning money and acquiring wealth

These activities result in the production of economic goods and services

Example Human activities(ie working in factories officesshops) which produce direct economic benefits

Non-economic activities are inspired by human sentiments and emotions such as love for the family desire to help the poor and love for the country

Thus these human activities (eg praying playing sleeping) produce no direct economic benefits and they are also not related to earning money and acquiring wealth

4 What are the characteristics of economic activities

Answer The characteristics of economic activities are as follows

Economic motiveEconomic activities are undertaken to earn money and acquire wealth

ProductiveEconomic activities involve productiondistribution and exchange of goods and services to create wealth

Economic growthEconomic activities determine the level of economic development of a country and standard of living of its citizens

Socially desirableEconomic activities are socially desirable for society

Economic resourcesEconomic activities make use of all the economic resources such landlabourcapital etc

5 What do you mean by non-economic activitiesExampleAnswerNon-economic activities are inspired by human sentiments and emotions such as love for the family desire to help the poor and love for the countryThese activities are not undertaken for monetary gain but for onersquos satisfaction and happinessExample

a mother looks after her children

a student donates blood8 Differentiate between Economic activities and Non-economic activities

Economic activities

Non-economic activities

1to earn living and acquiring wealth2Result can be measured in terms of money

3ExampleBusinessprofession and employment

1 to obtain some satisfaction

2Result cannot be measured in terms of money

3ExampleFamily-orientedreligious socialCultural and national

BUSINESS STUDIES

BUSINESS ENVIRONMENT

Welcome to the new sessionToday we are going to start the first chapter and the name of the chapter is Business Environment

In todayrsquos world every business enterprise is a part of the society It exists and operates in association with various groups in society such as customers suppliers competitors banks and financial institutions government agencies trade unions media and so on All these groups influence the functioning of business in one way or the other They constitute the environment of businessConcept of Business Environment

The term lsquobusiness environmentrsquo refers to the sum total of all individuals institutions and other forces that lie outside a business enterprise but that may influence its functioning and performance

The main features of business environment

Totality of External forces General and Specific forces Interrelatedness Complexity Dynamic Uncertainty Relativity

The Interrelation between business and its environment

The business enterprise is an open system It continuously interacts with its environment It takes inputs

Prepare the following questions from todayrsquos assignment

1 What do you mean by business environment

The term lsquobusiness environmentrsquo means the aggregate of all forces factors and institutions which are external to and beyond the control of an individual business enterprise but they may influence its functioning and performance Business environment is the macro framework within which a business firm a micro unit operates It consists of several interrelated and interacting elements

2 Explain the main features of business environment in brief

Totality of External forces-Business environment is the sum total of all things external to a business environment

General and Specific forces-It includes both the forces general forces are the economic social political legal and technological conditions which indirectly influence all business enterprise Specific forces are the investors customers competitors and suppliers which influence individual enterprise directly

Interrelatedness-Different elements of environment are interrelated for an example growing awareness for health care has increased the demand for health foods

Complexity- Business environment id

(such as raw materials capital labour energy and so on) from its environment transforms them into goods and services and sends them back to the environment

Fig 1 Business Environment Relationship

complex in nature as the elements keep on changing example economic technological and other forces changes in demand for a product and service

Dynamic-Business environment is not static it keeps on changing

Uncertainty- Itrsquos very difficult to predict future events such as technology and fashion which occur fast and frequently

Economics Basic Economic ConceptsSub topic

Microeconomics and

Macroeconomics

Welcome to the new sessiontoday we are going to start the first chapter of Class XI The name of the chapter that we are going to start is Basic Economic concepts

Now Economics covers the study of human activities Human activities are those activities which are performed by humans to satisfy their wants

Thus Human wants are unlimited and therefore economic activities such as production exchange and consumption are needed in order to satisfy those wants

The study of economics is divided largely in two parts which areMicroeconomics and Macroeconomics

SUBJECT- MATTER OF ECONOMICS

MICROECONOMICS MACROECONOMICS

Questions1Who has coined the words micro and macro economics

Answer Ranger Frisch coined the words lsquomicrorsquo and lsquomacrorsquo in 1933 to denote the two branches of economic theory namely microeconomics and macroeconomics

2What is microeconomicsAnswer It is the study of behaviour of individual decision ndash making unit such as consumers firms etc

3 What is macroeconomicsAnswer Macroeonomics is the study of overall economic phenomena like employment national income etc

4 What is the importance of microeconomicsAnswer

Microeconomics helps in formulating economic policies which enhance productive efficiency and results in greater social welfare

It helps the government in formulating correct price policies

It explains the working of a capitalistic economy where individual units(producers and consumers ) are free to take their own decision

Micro means a small part in

microeconomics we do not study the whole economy Hence we study an individual consumer and his or her choices and a producer and his or her profit maximizing decisions in the market Thus it does not mirror what happens in the economy as a whole

Macroeconomics on the other hand studies the economy as a whole It is concerned with aggregate and depicts the entire picture of the economyMacroeconomics deals with the national income aggregate investment aggregate consumption etc

Features of Microeconomics It deals with small

parts of the country Hence it looks at

individual consumers firms and industries

It deals with individual income consumption and savings

It studies the determination of price of any product or factors of production

It deals with the working of market via the price mechanism which is nothing but the determination of price and quantity of a commodity by the forces of demand and supply

Features of Macroeconomics

It deals with the study of the economy as a whole

It is concerned with

5 Give a limitation of microeconomics Microeconomics fails to explain the

functioning of an economy as a whole It cannot explain unemployment illiteracy and other problems prevailing in the country

6 What is the importance of macroeconomics It gives overall view of the growing

complexities of an economic system It provides the basic and logical

framework for formulating appropriate macroeconomic policies (eg for inflation poverty etc )to direct and regulate economy towards desirable goals

7What is the limitation of macroeconomics It ignores structural changes in an

individual unit of the aggregate

8 Differentiate between Microeconomics and Macroeconomics

Microeconomics Macroeconomics

the study of aggregates

National income aggregate savings and aggregate investments are major concepts dealt within macroeconomics style

It studies the determination of general price levels

It investigates into the problem of unemployment and the achievement of employment

It studies the aspect of decision making at the aggregate and national levels

It includes all growth theories whether related to developed or developing economies it also includes the study of economic systems and the working of the economy under different systems

Note Both Micro and macro economics are complementary and should be fully utilized for proper understanding of an economy

1It studies economic aspect of an individual unit2It deals with individual incomeConsumption and savings

3 It facilitates determination of price of any product or factors of production

4 Itrsquos scope is narrow and restricted to individual unit

1It studies the economy as a whole

2It deals with the national income aggregate consumption and aggregate savings3 It facilitates determination of general price level in an economy

4 Itrsquos scope is wide as it deals with economic units on the national level

ACCOUNTS

Introduction to Accounting and Book-keeping

Today I am going to share you the meaning of Accounting and Book-keeping and its related terms bullAccounting bullBook Keeping bullAccountsbullTypes Of Accounts bullAccounting Cycle

bull Meaning of accounting

Ans ) Accounting is the art and science of recording classifying and summarising monetary transactions

bull Meaning of Book-keeping

Ans) Bookkeeping is the art of recording business transactions with the view of having a permanent record of them and showing their effect on wealth

bull Meaning of account

Ans) The term account means a record of

business transactions concern a particular person of firm asset or income or expense It is a summarised record of all transactions which take place in an accounting year

bull Types of accountsPersonal accounts ndash Personal accounts relating

to person and Organisation are known as personal accounts Example Ramrsquos Account ABC amp Co Account etc

Real account - The accounts related to tangible and intangible assets are called real accounts Example Cash Account Furniture Account etc

Nominal account- Accounts related to expenses losses incomes and gains are known as nominal accounts Example Wages Account Salary Account Discount Account etc

bull Accounting cycle Accounting cycle refers to a complete sequence of accounting activities It begins with recording of transactions and ends with the preparation of a balance sheet

Chemistry TopicAtomic Structure

Thomsonrsquos atomic modelThomson (1898) was the first to propose the model of an atomHe proposed that an atom can be regarded as a uniform sphere of positive electricity in which requisite number of electrons are embedded evently to neutralize the positive chargeThis is just like plums embedded in a pudding or seeds evently distributed in red spongy mass of a watermelonThis model of atom is known as ldquoPlum-Pudding modelrdquo or

Q1)What is the fundamental constituents of atomAns Electron Proton and neutrons are the fundamental constituents of atomQ2)What is the value of fundamental unit of electricityAnsThe charge carried by one electron is sad to be the fundamental unit of electricityIts magnitude is 48times10-10esuOr 1602times10-19C Q3)Name the element containing no neutronAnsOrdinary hydrogen atom or protium 1H1

Types of AccountPersonal AccountReal AccountNominal AccountBalance Sheet (opening)

ldquowatermelon modelrdquoThis model could explain the electrical neutrality of an atom but failed to explain the result of scattering experiment carried out by Rutherford in 1911So it was rejected ultimately

Q4)Why is an electron called universal particleAns Itrsquos mass and Charge are independent of its source

EVS Chapter 1 ndash Modes of Existence

Modes of existence When one speaks normally about the mode of existence of some group or individual one refers to their customs their mode of being their ethology their habitat in some way their feeling for a placeDifferent modes of exixtence are ndash

1 Hunting ndashGathering2 Pastoral3 Agricultural4 Industrial

1 Hunting and gathering Hunting and gathering mode of existence is characterized by obtaining food from hunting wild animals including fishing and gathering wild plants From their earliest days the hunter-gatherer diet included various grasses tubers fruits seeds and nuts Lacking the means to kill larger animals they procured meat from smaller game or through scavenging

Societies that rely primarily or exclusively on hunting wild animals fishing and gathering wild fruits berries nuts and vegetables to support their diet are called hunting and gathering societies

At least this used to be practice of human beings before agriculture is invented As their brains evolved hominids developed more intricate knowledge of edible plant life and growth cycles

Q) Write the features of Hunting ndash gathering societiesAns - There are five basic characteristics of hunting and gathering societies

i The primary institution is the family which decides how food is to be shared and how children are to be socialized and which provides for the protection of its members

ii They tend to be small with fewer than fifty members

iii They tend to be nomadic moving to new areas when the current food supply in a given area has been exhausted

iv Members display a high level of interdependence

v Labor division is based on sex men hunt and women gather

Political Science

Introduction to political science

Political science occasionally called politology is a social science which deals with systems of governance and the analysis of political activities political thoughts associated constitutions and political behaviorThe study of political science involves the study of both the

Answer the following questions-1 What is political science

Political science occasionally called politology is a social science which deals with systems of governance and the analysis of political activities political thoughts associated constitutions and political behavior

2 Short notes-

traditional and modern theories of politicsTraditionalClassical political sciencepolitical theory-Traditional political science is the study of politics before Second World War The methodology to study Politics was traditional (legal formaletc) the definition of politics traditional (Politics begins and end with state)area of study (constitution state machinery)was traditionalModern Political scienceModern political theory-Modern Political Theory critically examines the contemporary state of political theory making an assessment of the achievement and limitations of the Behavioural Revolution in its totality and reviews objectively the major paradigms and conceptual frameworks adopted by the disciplineContemporary attempts at the development of an integrated political theory involving the use of both traditional and modern concepts approaches and theories-Around late 1960s several political scientists realized the importance of both the traditional political theory and modern Political theory They began building an integrated theory of politics involving a systematic mixture of traditional and modern studies of politics It was held that the study of a complex and vast field like politics needs both traditional as well as

Classical political theory Modern Political theory

Homework-Learn

modern concepts and approaches for studying itrsquos all aspects

Subject Eng Literature (The Tempest ndash William Shakespeare) Topic Act I Scene 1 Lines 1 to 32 (Line 32 ndash Gonzalo hellip If he be not born to be hanged our case is miserable) Date 13th April 2020 (3rd Period)

[Students should read the original play and also the paraphrase given in the school prescribed textbook]Summary Questions amp Answers

[SUMMARY OF THE ENTIRE SCENE]

o The play starts with the scene of a severe storm at sea Alonso (King of Naples) Sebastian (Alonsorsquos brother) Ferdinand (Alonsorsquos son) Gonzalo Antonio (the usurping Duke of Milan) are in a ship in the midst of the storm

o The mariners are trying their best to control the vessel from running aground and are totally following the orders of their Master the Boatswain They have scant success

o The mariners become extremely unhappy and annoyed when most of the passengers arrive on the deck thereby hampering their effort to save the ship There is serious confrontation between them and the passengers who are part of the Kingrsquos entourage

o The mariners could not save the ship

SUMMING-UP

(i) Vivid description of the scene which gives a realistic description of terror and confusion of a tropical storm

(ii) Shows Shakespearersquos accuracy of knowledge in describing the naval operations and also matters of seamanship

(iii) The opening scene justifies the title ndash The Tempest

UNANSWERED QUESTIONS

(i) The King always travels with his entire fleet including his soldiers Where

(1) GONZALO Nay good be patient (Line 15-26)BOATSWAIN When the sea is Hence What cares these

roarers for the name of the king To cabin silence Trouble us not

GONZALO Good yet remember whom thou has aboardBOATSWAIN None that I more love than myself You are a

councillor if you can command these elements to silence and work

the peace of the present we will not hand a rope more use your authority If you cannot give thanks you have

lived so long and make yourself ready in your cabin for the mischance of the hour if it so hap [To the Mariners]

Cheerly good hearts [To Gonzalo] Out of our way I say

(a) To whom is the boatswain speaking What does he mean by lsquoNone that I more love than myselfrsquo

The Boatswain is speaking to Gonzalo the honest old councilor of the Duke of MilanBy using the words ndash lsquoNone that I love more than I love myselfrsquo means that for the Boatswain nobody is dearer to him than his own life

(b) What were the conditions that made the boatswain react in this way

The Boatswain reacts in this way because the storm is at sea and Alonso King of Naples Sebastian his brother Ferdinand his son Gonzalo Antonio the usurping Duke of Milan on board are in distress and in panic Thus they have rushed to the deck interrupting the work of the mariners

(c) What hope does Gonzalo take from the attitude of the boatswain

The insolent and authoritative attitude of Boatswain makes Gonzalo feel comforted He tells that there are no signs that the Boatswain will be drowned But his facial appearance and attitude shows that he is destined to die on land by hanging which in effect means that all on board will be saved Otherwise all the persons on board are doomed

(d) How can they lsquomake yourself ready in your cabinrsquo For what were they asked to make ready themselves

In order to make themselves ready in their cabin the

were the other ships

(ii) Why was the ship in that area Where was it coming from or going where

(iii) The ship broke apart What happened to those who were in the ship

passengers on board must prepare for death which they will possibly soon have to meetThey can retire to their cabins and offer prayers to the Almighty to save them from drowning

(e) What does the boatswain say when he is asked to be patient What does he order to the royal party

When the boatswain is asked to be patient and remain calm he says that he will be patient only when the storm will be over and the sea will be calm but as long as the storm blows and there is danger to the ship he cannot think of being patient He orders the royal party to go to the cabin and leave the mariners to their work

(2) GONZALO I have great comfort from this fellow (Line 27-36)

Methinks he hath no drowning mark upon him his complexion is perfect

gallows Stand fast good Fate to his hanging Make the rope of his destiny our cable for our own doth little advantage If he be not born to be hanged our case is miserable

(a) Why does Gonzalo regard the Boatswain in the midst of danger

In the midst of danger Gonzalo regards the boatswain because he feels that the Boatswain is a source of comfort and is bent upon to do his work sincerely which in this case is saving the ship and its passengers from the severest of raging storm

(b) What reasons does Gonzalo give when he says that none in the ship will die of drowning

Gonzalo is almost sure that none in the ship will die by drowning His says that there is no mark on the face of the boatswain that indicates that he will die by drowning On the other hand the lines on his face are strong indications that he will be hanged to death Therefore there shall be no danger of the shiprsquos sinking

(c) Explain the following ldquoStand fast good Fate to his hanging Make the rope of his destiny our cable for our own doth little advantage If he be not born to be hanged our case is miserablerdquo

The stated lines mean that if the will of destiny is to be carried out then the ship will not get wrecked and all the passengers will be saved The safety of the passengers therefore depends upon the will of fate being carried out in the case of the boatswain If however the boatswain is not to die by hanging then the passengers are also very unsafe because in that case the ship is likely to sink

(d) What order does the Boatswain give to the sailors

when he re-enters What does he say about the crying of the fellows inside the cabin

The boatswain orders the sailors to bring the topmast lower and bring the ship close to a stationary position with the help of the main sail He says that the fellows inside the cabin are moaning and crying in their distress louder than his voice and louder even than the roaring of the storm

Class XII (ScienceCommerceHumanities) Subject Topic Summary Execution

Computer Science

PropositionalLogic

Propositional logic is a procedure to provide reasoning through statementProposition A ststement that results in True or False is said to be proposition There are two types of propositionSimple proposition amp compound propositionSimple proposioton A simple proposition is one that is not a part of any other proposition Such sentential form of proposition is symbolized with english letters in short For example Ram is a claver student (TrueFalse)Where do you live (Not in True or False)Grapes are sweet (TrueFalse)It rains today (TrueFalse)Here we can see some statements anwer would be true or false but some staements answer can not give in terms of true or false Thus the sentences which can be answered in true or false are known as simple propositionAssigning propositon to a variableThe general syntax to assign propostion to a variable is as followsVariable = Simple propositonFor example A=Ram is a clever studentB= Grapes are sweetC= it rains todayCompound proposition

helliphellipto be continued in next classhelliphellipMath Relation Relation If A and B are two non-empty sets

then a relation R from A to B is a subset of AxB If R A x B and (a b) R then we say that a sube isinis related to b by the relation R written as aRbeg Let A be the set of students of class XII and B be the set of students of class XI Then some of the examples of relation from A to B arei) (a b) AXB a is brother of bisinii) (a b) AXB age of a is more than age of isinb Types of relation In this section we would like to study different types of relations We know that a relation in a set A is a subset of A times A Thus the empty set φ and A times A are two extreme relations For illustration consider a relation R in the set A = 1 2 3 4 given by R = (a b) a ndash b = 10 This is the empty set as no pair (a b) satisfies the condition a ndash b = 10 Similarly R = (a b) | a ndash b | ge 0 is the whole primeset A times A as all pairs (a b) in A times A satisfy | a ndash

Example 1 Let A be the set of all students of a boys school Show that the relation R in A given by R = (a b) a is sister of b is the empty relation and R = (a b) the primedifference between heights of a and b is less than 3 meters is the universal relationSolution Since the school is boys school no student of the school can be sister of any student of the school Hence R = φ showing that R is the empty relation It is also obvious that the difference between heights of any two students of the school has to be less than 3 meters This shows that R = A times A is primethe universal relation Example 2 Show that the relation R in the set 1 2 3 given by R = (1 1) (2 2) (3 3) (1 2) (2 3) is reflexive

b | ge 0 These two extreme examples lead us to the following definitionsDefinition 1 A relation R in a set A is called empty relation if no element of A isrelated to any element of A ie R = φ A times AsubDefinition 2 A relation R in a set A is called universal relation if each element of A is related to every element of A ie R = A times A Both the empty relation and the universal relation are some times called trivial relation Definition 3 A relation R in a set A is called(i) reflexive if (a a) R for every a Aisin isin(ii) symmetric if (a1 a2) R implies that (aisin 2a1)

R for all aisin 1 a2 Aisin(iii) transitive if (a1 a2) R and (aisin 2 a3) R isinimplies that (a1 a3) R for all aisin 1 a2 a3 AisinDefinition 4 A relation R in a set A is said to be an equivalence relation if R is reflexive symmetric and transitive

but neither symmetric nor transitiveSolution R is reflexive since (1 1) (2 2) and (3 3) lie in R Also R is not symmetric as (1 2) R but (2 1) isin notinR Similarly R is not transitive as (1 2) R and (2 3) R but (1 3) R isin isin notinExample 3 Show that the relation R in the set Z of integers given byR = (a b) 2 divides a ndash b is an equivalence relationSolution R is reflexive as 2 divides (a ndash a) for all a Z isinFurther if (a b) R then 2 divides a isinndash b Therefore 2 divides b ndash a Hence (b a) R which shows that R is isinsymmetric Similarly if (a b) R and (b c) R isin isinthen a ndash b and b ndash c are divisible by 2 Now a ndash c = (a ndash b) + (b ndash c) is even (Why) So (a ndash c) is divisible by 2 This shows that R is transitive Thus R is an equivalence relation in ZExample 4 Let L be the set of all lines in a plane and R be the relation in L defined as R = (L1 L2) L1 is perpendicular to L2 Show that R is symmetric but neither reflexive nor transitiveSolution R is not reflexive as a line L1 can not be perpendicular to itself ie (L1 L1) R notinR is symmetric as (L1 L2) Risin

L1 is perpendicular to L2rArr L2 is perpendicular to L1rArr (L2 L1) RrArr isin

R is not transitive Indeed if L1 is perpendicular to L2 and L2 is perpendicular to L3 then L1 can never be perpendicular to L3 In fact L1 is parallel to L3 ie (L1 L2) R isin(L2 L3) R but (L1 L3) Risin notin

Chemistry Solid state Characteristics if Solids(i)The particles are locked in fixed positions they are unable to change their relative positions and this brings a definite shape and volume of a solid(ii)In a solid the constituent particles are held by strong forces of attractionThe forces of attraction may be bonding or non bonding(iii)The constituent particles in a solid pack together as closely as possibleoccupying most of the available space within the solidThus the empty space in a solid is very smallThis makes a solid highly rigid and nearly incompressibleThis also explains why a solid has high density and exhibits slow diffusionClassification of Solids

Q1)Define Crystalline solids AnsA Solid that has a definite geometrical shape and a sharp melting pointand whose constituent particles (atomsmolecules or ions) are arranged in a long range order of definite pattern extending throughout the solid is called a crystalline solidExNaClQ2)Define Amorphous solids AnsA solid that does not have a definite shape and a sharp melting pointand whose constituent particles (atomsmolecules or ions) are not arranged in a definite pattern is called an amorphoussolid

Crystalline solidsAmorphous solids

ExGlassRubberQ3)Classify Crystalline Solids Crystalline Solids

Physics Coloumbrsquos Law (Summary)

Before Going Into Coloumbrsquos Law We Will First Learn What is Charge Properties of Charge and Always remember that charge is quantized ie a body always have static charge of magnitude equal to some integral multiple of fundamental electronic charge e= 16 x 10- 19 C

Charge is the property of matter that causes it to produce and experience electrical and magnetic effects The study of the electrical charges at rest is called electrostatics When both electrical and magnetic effects are present the interaction between charges is referred to as electromagnetic

There exist two types of charges in nature positive and negative Like charges repel and unlike charges attract each other

The type of charge on an electron is negative The charge of a proton is the same as that of an electron but with a positive sign In an atom the number of electrons and the number of protons are equal The atom is therefore electrically neutral If one or more electrons are added to it it becomes negatively charged and is designated as negative ion However if one or more electrons are removed from an atom it becomes positively charged and is called a positive ion

The excess or deficiency of electrons in a body gives the concept of charge If there is an excess of electrons in a body it is negatively charged And if there is deficiency of electrons the body becomes positively charged Whenever addition or removal of electrons takes places the body acquires a charge

The SI Unit of charge is coulomb (C) In SI units the current is a fundamental quantity having a unit of ampere (A) The unit of charge is defined in terms of the unit of current Thus one coulomb is the charge transferred in one second across the section of a wire carrying a

Ionic SolidsMetallicSolids

Molecular Solids

current of one ampere

As q = It we have1 C = (1 A) (1 s)

The dimensions of charge are [A T]

Properties of Charge

(1) Quantization of Charge Electric charge can have only discrete values rather than any value That is charge is quantized The smallest discrete value of charge that can exist in nature is the charge on an electron given as

e = plusmn 16 x 10- 19 C

This is the charge attained by an electron and a protonA charge q must be an integral multiple of this basic unit That is

Q = plusmn ne where n = 1 2 hellip

Charge on a body can never be (frac12)e (23)e or 57e etcWhen we rub a glass rod with silk some electrons are transferred from the rod to the silk The rod becomes positively charged The silk becomes negatively charged The coulomb is a very large amount of charge A typical charge acquired by a rubbed body is 10 - 8 C

Biology Reproduction in organisms

Welcome to this new session 2020-21Today in this first chapter we mainly discuss about reproduction types needs and life span of some organismsWe also discuss about difference between sexual and asexual reproduction

Q1 What is reproductionReproduction is defined as a biological processin which an organism gives rise to young onessimilar to itselfQ2 What are the needs of reproductionbulli) Reproduction maintain life on earthii) It enables the continuity of the species generation after generationiii) It creates genetic variation among populationsQ3 Define Life span and write some orgnisms life spanbull Life span is the period from birth to

the natural death of an organism- OrganismsLife span1 Butterfly 1 - 2 weeks2 Fruit fly 30 days3Dog 10-13 years4 Rose5-7 years5 Tortoise100-150 years6 Banyan Tree -200 - 250 yearsQ4 Reproduction is of two types in case ofanimals but in case of plants vegetative propagation is also present

Asexual Reproduction Sexual Reproductioni) Always uniparentalii) Gametes are not involvediii) Only mitotic division involvediv) Somatic cells of parents are involvedv) Offsprings are genetically similar to the parents

i) Usually biparentalii) Gametes are involvediii) Meiosis occurs during gametogenesis Mitosis occurs after fertilisationiv) Germ cells of the parents are involvedv) offsprings are genetically different from the parents

COMMERCE BUSINESS ENVIRONMENT

Welcome to the new sessiontoday we are going to start the first chapter of Class XII The name of the chapter is Business Environment

Already many of you have got some idea about the word business environment form the first chapter of business studies in class XI

In todayrsquos world every business enterprise is a part of the society It exists and operates in association with various groups in society such as customers suppliers competitors banks and financial institutions government agencies trade unions media and so on All these groups influence the functioning of business in one way or the other They constitute the environment of businessConcept of Business Environment

The term lsquobusiness environmentrsquo refers to the sum total of all individuals institutions and other forces that lie outside a business enterprise but that may influence its functioning and performance

The main features of business environment Totality of External forces General and Specific forces Interrelatedness Complexity Dynamic Uncertainty

Prepare the following questions from todayrsquos assignment

2 What do you mean by business environment

The term lsquobusiness environmentrsquo means the aggregate of all forces factors and institutions which are external to and beyond the control of an individual business enterprise but they may influence its functioning and performance Business environment is the macro framework within which a business firm a micro unit operates It consists of several interrelated and interacting elements

2 Explain the main features of business environment in brief

Totality of External forces-Business environment is the sum total of all things external to a business environment

General and Specific forces-It

Relativity

The Interrelation between business and its environment

The business enterprise is an open system It continuously interacts with its environment It takes inputs (such as raw materials capital labour energy and so on) from its environment transforms them into goods and services and sends them back to the environment

Fig 1 Business Environment Relationship

includes both the forces general forces are the economic social political legal and technological conditions which indirectly influence all business enterprise Specific forces are the investors customers competitors and suppliers which influence individual enterprise directly

Interrelatedness-Different elements of environment are interrelated for an example growing awareness for health care has increased the demand for health foods

Complexity- Business environment id complex in nature as the elements keep on changing example economic technological and other forces changes in demand for a product and service

Dynamic-Business environment is not static it keeps on changing

Uncertainty- Itrsquos very difficult to predict future events such as technology and fashion which occur fast and frequently

Business Studies

Human Resources Management

Human resource of an organisation are the aggregate of knowledge skills attitudes of people working in it

The management system which deals with human resources is called human resource management

Features of HRMbullComprehensive functionbullPeople-oriented

Question1) What do you mean by human

resource management Answer) Human resource management may be defined as that field of Management which has to do with planning organising and controlling the functions of procuring developing maintaining and utilising the labour force

bullAction oriented bullPervasive function bullContinuous function

2) Explain the features of HRM in brief

Answer)bullHuman Resource Management is concerned with managing people at work bull Human Resource Management is concerned with employees which bring people and organisations together so that the goals of each are met bullHuman resource management considered every employees as an individual and also promote their satisfaction and growth bull Human resource management is inherent in all organisations and at all levelsbullManagement of human resources are ongoing on never ending process which requires a constant alertness and Awareness of human relations

3) ldquoHR function is said to be pervasiverdquowhy

Answer) Human resource management is required in all organisations whether it is private or government organisations armed forces sports organisations etc It permeatsall the functional areas like production marketing finance research etc This from this feature of human resource management it can be said that it is pervasive in nature

Economics Demand Q1DEFINITION OF DEMANDIn economics demand is the quantity of a good that consumers are willing and able to purchase at various prices during a given period of timeQ2DEMAND CURVEIn economics a demand curve is a graph depicting the relationship between the price of a certain commodity and the quantity of that commodity that is demanded at that pricQ3LAW OF DEMANDIn microeconomics the law of demand states that conditional on all else being equal as the price of a good increases quantity demanded decreases conversely as the price of a good decreases quantity demanded increasesQ4ASSUMPTION of LAW OF DEMAND(i)No change in price of related commodities(ii) No change in income of the consumer(iii) No change in taste and preferences customs habit and fashion of the consumer( No expectation regarding future change in priceQ5MARKET DEMAND SCHEDULEIn economics a market demand schedule is a tabulation of the quantity of a good that all consumers in a market will purchase at a

given price At any given price the corresponding value on the demand schedule is the sum of all consumersrsquo quantities demanded at that priceQ6INDIVIDUAL DEMAND SCHEDULEIndividual demand schedule refers to a tabular statement showing various quantities of a commodity that a consumer is willing to buy at various levels of price during a given period of timeQ7 FACTORS AFFECTING INDIVIDUAL DEMAND FOR A COMMODITY

The factors that influence a consumerrsquos decision to purchase a commodity are also known as determinants of demand The following factors affect the individual demand for a commodity1 price of the commodity2 price of related goods3 income of buyer of the commodity4 tastes and preferences of the buyer1 Price of the CommodityYou must have observed that when price of a commodity falls you tend to buy more of it and when its price rises you tend to buy less of it when all other factors remain constant (lsquoother things remaining the samersquo) In other words other things remaining the same there is an inverse relationship between the price of a commodity and its quantity demanded by its buyers This statement is in accordance with law of demand which you will study in the later part of this lesson Price of a commodity and its quantity demanded by its buyers are inversely related only when lsquoother things remain the samersquo So lsquoother things remaining the samersquo is an assumption when we study the effect of changes in the price of a commodity on its quantity demanded2 Price of Related goodsA consumer may demand a particular good But while buying that good heshe also asks the price of its related goods Related goods can be of two types-(i) Substitute goods(ii) Complementary goods While purchasing a good prices of its substitutes and complements do affect its quantity purchased(i) Price of Substitute Goods Substitute goods are those goods which can easily be used in place of one another for satisfaction of a particular want like tea and coffee An increase in price of substitute good leads to an increase in demand for the given commodity and a decrease in price of substitute good leads to a decrease in demand for the given commodity It means demand for a given commodity is directly affected by change in price of substitute goods For example if price of coffee increases the demand for tea will rise as tea will become relatively cheaper in comparison to coffee(ii) Price of Complementary goods Complementary goods are those goods which are used together to satisfy a particular want like car and petrol An increase in the price of complementary goods leads to a decrease in demand for the given commodity and a decrease in the price of complementary goods leads to an increase in demand for the given commodity For example if price of petrol falls then the demand for cars will increase as it will be relatively cheaper to use both the goods together So demand for a given commodity is inversely affected by change in price of complementary goods3 Income of the Buyer of CommodityDemand for a commodity is also affected by income of its buyer However the effect of change in income on demand depends on the nature of the commodity under consideration In case of some goods like full cream milk fine quality of rice (Basmati rice) etc demand for these commodities increases when income of the buyer increases and

demand for these commodities decreases when income of the buyer decreases Such goods whose demand increases with the increase in income of the buyer are called normal goods But there are some goods like coarse rice toned milk etc whose demand decreases when income of buyer increases and their demand increases when income of the buyer decreases Such goods whose demand decreases with the increase in income of the buyer are called inferior goods Suppose a consumer buys 10 Kgs of rice whose price is ` 25 per Kg He cannot afford to buy better quality of rice because the price of such rice is ` 50 per Kg The consumer is spending ` 250 per month on the purchase of rice Now if income of the consumer increases and he can afford ` 350 on purchase of 10 Kg of rice Now he can afford to buy some quantity of rice say 6 Kgs whose price is ` 25 per Kg and may buy 4 Kgs of rice whose price is ` 50 per Kg Thus he will buy 10 Kgs of rice by spending ` 350 per month Therefore we may conclude that demand for normal goods is directly related to the income of the buyer but demand for inferior goods is inversely related to the income of the buyer4 Tastes and Preferences of the BuyerThe demand for a commodity is also affected by the tastes and preferences of the buyers They include change in fashion customs habits etc Those commodities are preferred by the consumers which are in fashion So demand for those commodities rises which are in fashion On the other hand if a commodity goes out of the fashion its demand falls because no consumer will like to buy it(5) Number of Buyers in the Market(Population)Increase in population raises the market demand whereas decrease in population reduces the market demand for a commodity Not only the size of population but its composition like age (ratio of males females children and old people in population) also affects the demand for a commodity It is because of needs of children young old male and female population differs(6) Distribution of Income and WealthIf the distribution of income and wealth is more in favour of the rich demand for the commodities preferred by the rich such as comforts and luxuries is likely to be higher On the other hand if the distribution of income and wealth is more in favour of poor demand for commodities preferred by the poor such as necessities will be more(7) Season and Weather ConditionsThis is generally observed that the demand for woolens increases during winter whereas demand for ice creams and cold drinks increases during summer Similarly market demand for umbrellas rain coats increases during rainy seasonQ8 REASONS FOR OPERATION OF LAW OF DEMAND WHY DEMAND CURVE SLOPES DOWNWARDNow we will try to explain why does a consumer purchase more quantity of a commodity at a lower price and less of it at a higher price or why does the law of demand operate ie why does the demand curve slope downwards from left to right The main reasons for operation of law of demand are1 Law of Diminishing Marginal UtilityAs you have studied earlier law of diminishing marginal utility states that as we consume more and more units of a commodity the utility derived from each successive unit goes on decreasing The consumer will be ready to pay more for those units which provide him more utility and less for those which provide him less utility It implies that he will purchase more only when the price of the commodity falls2 Income Effect

When price of a commodity falls purchasing power or real income of the consumer increases which enables him to purchase more quantity of the commodity with the same money income Let us take an example Suppose you buy 4 ice creams when price of each ice cream is ` 25 If price of ice creams falls to ` 20 then with same money income you can buy 5 ice creams now3 Substitution EffectWhen price of a commodity falls it becomes comparatively cheaper as compared to its substitutes (although price of substitutes has not been changed) This will lead to rise in demand for the given commodity For example if coke and Pepsi both are sold at ` 10 each and price of coke falls Now coke has become relatively cheaper and will be substituted for Pepsi It will lead to rise in demand for coke4 Change in Number of BuyersWhen price of a commodity falls some old buyers may demand more of the commodity at the reduced price and some new buyers may also start buying this commodity who were not in a position to buy it earlier due to higher price This will lead to increase in number of buyers when price of the commodity falls As a result demand for the commodity rises when its price falls5 Diverse Uses of a CommoditySome commodities have diverse uses like milk It can be used for drinking for sweet preparation for ice cream preparation etc If price of milk rises its use may be restricted to important purpose only This will lead to reduction in demand for other less important uses When price of milk falls it can be put to other uses also leading to rise n demand for itQ9 EXCEPTIONS TO THE LAW OF DEMANDYou have studied in law of demand that a buyer is willing to buy more quantity of a commodity at a lower price and less of it at a higher price But in certain circumstances a rise in price may lead to rise in demand These circumstances are called Exceptions to the Law of Demand Some important exceptions are1 Giffen GoodsGiffen goods are special type of inferior goods in which negative income effect is stronger than negative substitution effect Giffen goods do not follow law of demand as their demand rises when their price rises Examples of Giffen goods are jowar and bajra etc2 Status Symbol GoodsSome goods are used by rich people as status symbols eg diamonds gold jewellary etc The higher the price the higher will be the demand for these goods When price of such goods falls these goods are no longer looked at as status symbol goods and tehrefore therir demand falls3 NecessitiesCommodities such as medicines salt wheat etc do not follow law of demandbecause we have to purchase them in minimum required quantity whatever their price may be4 Goods Expected to be ScarceWhen the buyers expect a scarcity of a particular good in near future they start buying more and more of that good even if their prices are rising For example during war famines etc people tend to buy more of some goods even at higher prices due to fear of their scarcity in near future

Political Science

Constitution of India-The

Preamble

The preamble-

Preamble-

The preamble is the most precious part of the constitution We the people of India having solemnly resolved to constitute India into a Sovereign Socialist Secular Democratic Republic and to secure to all its citizensA preamble is an introductory and expressionary statement in a document that explains the documents purpose and underlying philosophy When applied to the opening paragraphs of a statute it may recite historical facts pertinent to the subject of the statuteNature and purpose of the constitution-Purpose of the Constitution dictates permanent framework of the government to form a more perfect union to establish justice and ensure peace of thenationconstitution provide principles how the government can run itself following the rules and laws written in the constitution of each state keeps them balanced

Answer the following questions-

1 What is preambleA preamble is an introductory and expressionary statement in a document that explains the documents purpose and underlying philosophy2 What is the nature and

purpose of the constitutionConstitution dictatespermanent framework of the government to form a more perfect union to establish justice and ensure peace of the nation

Homework-Learn

Accounts Compatibilty mode

1MEANING OF PARTNERSHIPPartnership is a form of business organisation where two or more persons join hands to run a business They share the profits and losses according to the agreement amongst them According to the Indian Partnership Act 1932 ldquoPartnership is relation between persons who have agreed to share profits of a business carried on by all or any one of them acting for allrdquo For example one of your friends has passed class XII from National Institute of Open Schooling (NIOS) and wants to start a business Heshe approaches you to join in this venture Heshe wants you to contribute some money and participate in the business activities Both of you if join hands constitute a partnership2CHARACTERISTICS1048698 Agreement A partnership is formed by an agreement The agreement may be either oral or in writing It defines the relationship between the persons who agree to carry on business It may contain the terms of sharing profit and the capital to be invested by each partner etc The written agreement is known as partnership deed1048698 Number of persons There must be at least two persons to form a partnership

The maximum number of partners in a partnership firm can be 50 according toCompanies Act 20131048698 Business The Partnership is formed to carry on business with a purpose of earning profits The business should be lawful Thus if two or more persons agree to carry on unlawful activities it will not be termed as partnership1048698 Sharing Profits The partners agree to share profits in the agreed ratio In caseof loss all the partners have to bear it in the same agreed profit sharing ratio10486981048698Mutual Agency Every partner is an agent of the other partners Every partner can bind the firm and all other partners by hisher acts Each partner will be responsible and liable for the acts of all other partners10486981048698Unlimited liability The liability of each partner except that of a minor is unlimited Their liability extends to their personal assets also If the assets of the firm are insufficient to pay off its debts the partnersrsquo personal property can be used to satisfy the claim of the creditors of the partnership firm10486981048698Management All the partners have a right to mange the business However they may authorize one or more partners to manage the affairs of the business on their behalf10486981048698Transferability of Share No partner can transfer hisher share to any one including hisher family member without the consent of all other partners3PARTNERSHIP DEEDAgreement forms the basis of partnership The written form of the agreement is which a document of partnership is It contains terms and conditions regarding the conduct of the business It also explains relationship between the partners This document is called partnership deed Every firm can frame its own partnership deed in which the rights duties and liabilities of the partners are stated in detail It helps in settling the disputes arising among the partners during the general conduct of business 4CONTENTS OF PARTNERSHIP DEEDThe partnership deed generally contains the following (i) Name and address of the partnership firm(ii) Nature and objectives of the business(iii) Name and address of each partner(iv) Ratio in which profits is to be shared(v) Capital contribution by each partner(vi) Rate of Interest on capital if allowed(vii) Salary or any other remuneration to partners if allowed(viii) Rate of interest on loans and advances by a partner to the firm(ix) Drawings of partners and interest thereon if any(x) Method of valuation of goodwill and revaluation of assets and liabilities on the reconstitution of the partnership ie on the admission retirement or death of a partner(xi) Settlement of disputes by arbitration(xii) Settlement of accounts at the time of retirement or death of a partner5IN ABSENCE OF PARTNERSHIP DEEDThe partnership deed lays down the terms and conditions of partnership in regard to rights duties and obligations of the partners In the absence of partnership deed there may arise a controversy on certain issues like profit sharing ratio interest on

capital interest on drawings interest on loan and salary of the partners In such cases the provisions of the Indian Partnership Act becomes applicableSome of the Issues are(i) Distribution of Profit Partners are entitled to share profits equally(ii) Interest on Capital Interest on capital is not allowed(iii) Interest on Drawings No interest on drawing of the partners is to be charged(iv) Interest on Partnerrsquos Loan A Partner is allowed interest 6 per annum on the amount of loan given to the firm by himher(v) Salary and Commission to Partner A partner is not entitled to anysalary or commission or any other remuneration for managing the business

History TOPIC-TOWARDS INDEPENDENCE AND PARTITION THE LAST PHASE (1935-1947)

SUB TOPIC-IMPORTANT POLITICAL DEVELOPMENTS ndash GROWTH OF SOCIAL IDEAS

Socialism is a political social and economic philosophyLike in other parts of the world the Russian revolution of 1917 served as a great inspiration for revolutionaries in India who at that time were engaged in the struggle for liberation from British ruleSocialist ideas led to the formation of communist party of IndiaJAWAHARLAL NEHRU Among the early Congress leaders Jawaharlal Nehru was very much impressed and influenced by the Socialist ideas He also learnt about the Economic activities of the Soviet Union after the Bolshevic Revolution 1917 He made full use of them in IndiaThe election of Jawaharlal Nehru and Subhas Chandra Bose showed the Left wing tendency within CongressJawaharlal Nehru demanded economic freedom along with political freedom of the people in order to end the exploitation of masses

Nehrus working committee included three socialists leaders The Lucknow session was a landmark in the evolution of socialist ideas of the congressSUBHAS CHANDRA BOSE ndash Subhas Chandra Bose had socialist leaning Both Jawaharlal Nehru and Subhas Chandra Bose were known as leftist Congress men Later on National Congress divided into Leftist and rightist campCONGRESS SOCIALIST Within the Congress some leaders formed the Congress Socialist partyPattavi Sitaramyya Sardar Patel Rajendra Prasad had hostile attitude towards the Congress Socialist partyJawaharlals attitude was hesitant

1 QUESTION ndash Mention name of two Congress leaders who had socialist leaning

1ANSWER ndash Subhas Chandra Bose and Jawaharlal Nehru2QUESTION- In which session of the congress Jawaharlal elaborated his Socialist ideas2 ANSWER ndash Lucknow and Faizpur Session in December 1935 and 19363QUESTION ndash Why Congress was sharply divided into leftist and rightist camp 3ANSWER ndash Subhas Chandra Bosersquos attempt to seek re election for congress presidentship in 1939sharply divided the National Congress into Leftist and Rightist camp4 QUESTION ndash Who was MN Roy 4 ANSWER ndash Manabendra Roy first formed the Communist Party of India outside the country at Tashkent in 19205QUESTION ndash Who formed the Congress Socialist Party within the Congress5 ANSWER ndash Jaya Prakash Narayan Achyut Patwardhan Acharya Narendra Dev Ram Mohan Lohia Aruna Asaf Ali6QUESTION ndash When was the Congress Socialist Party formed What was its object6 ANSWER ndash 1934The Congress Socialist Party sought to work out socialist programme through the Congress They joined hands with the Congress and wanted to carry

Subhas Chandra Bose being expelled from the congress after the Tripuri rift he formed Forward BlockThere were basic differences between the Congress Socialists and the communistsTRADE UNION ACTIVITIES Maximum working class people lived in Bombay and Calcutta The working and living conditions of those workers were very miserable In this situation Shasipada Banerjee NM Lokhande protested against the oppression of the working class peopleThe first Trade Union Madras Labour Union was formed in 1918 by BP WadiaIndustrial strikes took place in Kanpur Calcutta Madras Jamshedpur and Ahmedabad AITUC was formed in Bombay in 1927 The growth of Trade union among the workers was slow because of the fear of the dismissal of the jobIn the mean time the Moderates as well as Communists left AITUC and formed separate organization

on National struggle with the help of workers and peasant class of the society7 QUESTION ndash What was the name of the party founded by Subhas Chandra Bose7 ANSWER- Forward Block8QUESTION ndash Who was Shasipada Banerjee8 ANSWER ndash Shasipada Banerjee was a radical Brahmo He founded a working menrsquos club to protest against exploitation of the British rulers towards the working class of India9 QUESTION ndash What was the weekly published by NM Lokhande9ANSWER- Dinabandhu10 QUESTION ndash Who founded Bombay Mill-Hands Association and in which year10 ANSWER- NM Lokhande in189011 QUESTION- Who was BP WadiaANSWER- BPWadia was the founder of Madras Labour Union in191812 QUESTION- What was the name of the first labour union of India12 ANSWER- Madras Labour Union13 QUESTION Who founded the Majur Mahajan 13 ANSWER GANDHIJI14 QUESTION What was the full form of AITUC When it was formed14 ANSWER All India Trade Union Congressin 192715QUESTION Who formed the Red Trade Union Congress and in which year15ANSWER The Communists formed the Red Trade Union Congress16 QUESTION What do you mean by Socialism16 ANSWER Socialism describes any political and economic theory that says the community rather than individuals should own and manage property and natural resources

Subject Eng Literature (The Tempest ndash William Shakespeare) Topic Act III Scene 3 Lines 1 to 52 (Line 52 ndash Brother my lord the Duke Stand to and do as we) Date 13th April 2020 (4th Period)

[Students should read the original play and also the paraphrase given in the school prescribed textbook]Summary Questions amp Answers

o Alonso Sebastian Antonio Gonzalo Adrian Francisco and others wandered about the island in search of Ferdinand and gets tired and hungry of the toil and at the same time gives up all hope of finding him

o Antonio and Sebastian are happy that Alonso is out of hope and decide to make another attempt on his life that night when being so tired they will be sleeping soundly

o Suddenly a solemn and strange music is heard in the air and several strange shapes enter bringing in a banquet These strange shapes then dance round it with gestures of salutation and then inviting the King to eat they depart

o Seeing this strange scene all are inclined to believe the tales told by travelers that there truly are ldquounicornsrdquo and ldquothe phoenixrsquo thronerdquo

1 ALONSO What harmony is this My good friends hark (L18-27)

GONZALO Marvellous sweet music

[Enter several strange shapes bringing in a banquet

they dance about it with gentle actions of salutation

and inviting the King and his companions to eat they depart]ALONSO Give us kind keepers heavens What were theseSEBASTIAN A living drollery Now I will believe

That there are unicorns that in Arabia

There is one tree the phoenixrsquo throne one phoenix

At this hour reigning thereANTONIO Ill believe both

And what does else want credit come to me

And Ill be sworn rsquotis true Travellers neer did lie

Though fools at home condemn rsquoem

(a) How did Prospero present an amazing spectacle before Alonso and his companions

Using his magic powers Prospero ordered strange shapes to lay a banquet before Alonso and his companions The shapes brought several dishes with tasty eatables in them They placed the dishes on a table before Alonso and his companions Then the strange shapes began to dance gracefully around the banquet While dancing they made gestures inviting them to eat the food Then suddenly the shapes disappeared(b) Who were the guests at the strange banquet Describe the lsquoliving drolleryrsquo

Alonso Sebastian Antonio Gonzalo Adrian and Francisco were the guests at the strange banquet

The term ldquoliving drolleryrdquo refers to live entertainment show In this context when Alonso the King of Naples Sebastian his brother Antonio the treacherous brother of Prospero Gonzalo the kind and loyal councillor to the King Adrian and Francisco came to the island they were hungry and weary in their spirits They heard a solemn and strange music They were shocked to see several strange shapes bringing in a banquet and these shapes danced about it with gentle action of salutation inviting the King and his companions to eat After this Sebastian described this show as lsquoliving drolleryrsquo(c) What is lsquophoenixrsquo What are lsquoUnicornsrdquo

The term lsquophoenixrsquo refers to a mythical Arabian bird which lived alone and perched on a solitary tree After one hundred years it expired in flames and rose again from its own ashes

lsquoUnicornsrsquo refers to the mythological four-footed beasts having horns in the centre of their foreheads When the horns are ground into powder the powder was believed to be

an aphrodisiac(d) How does Sebastian explain the puppet show OR Why does the speaker now believe in unicorns and phoenix

Sebastian finds several strange shapes bringing in the banquet They invite the king and his party for dinner and soon depart He tells that if such a strange sight can be a reality there is nothing incredible in the world and from the present moment he will believe anything He says that it is a strange dumb show enacted not by puppets but by living beings It is stranger than a travellerrsquos tale Seeing such a thing

before his own eyes he will no longer disbelieve the story about unicorns and phoenix(e) How do the other characters present respond to this living drollery

At the sight of the lsquoliving drolleryrsquo like Sebastian Gonzalo and Antonio too acted strangely Antonio told that he too now believes in unicorns and phoenix and anything else that seems to be incredible He too now believes in travellersrsquo tales Gonzalo told that if he would report those happenings in Naples nobody will believe him He considers that those gentle shapes were gentler in manner in comparison to the living beings Alonso was at first sight suspicious and told them that those strange shapes conveyed their meaning in expressive gestures when they seemed to lack speech by their movements and sounds Francisco was amazed at their mysterious disappearance

2 ALONSO Not I

(Line 43-52)GONZALO Faith sir you need not fear When we

were boysWho would believe that there were mountaineers

Dewlapped like bulls whose throats had hanging at rsquoem

Wallets of flesh Or that there were such men

Whose heads stood in their breasts Which now we find

Each putter-out of five for one will bring us

Good warrant ofALONSO I will stand to and feed

Although my lastmdashno matter since I feel

The best is past Brother my lord the Duke

Stand to and do as we

(a) How does Alonso respond at the spectacle of the shapes which were sent to them at the instruction of Prospero

After seeing the strange sight of appearing and disappearing of the shapes sent by Prospero to arrange a banquet for them Alonso says that his surprise at having seen those creatures is infinite and he is fully justified in feeling so much surprise He thinks that their shapes their gestures and the sounds they made were indeed amazing Although they do not possess the gift of speech yet they were able to convey their

thoughts by means of their gestures only

(b) What does Prospero say about the views expressed by Alonso regarding the shapes What does Francisco think about the shapesAfter hearing Alonsorsquos views about the shapes Prospero says that this manrsquos praise of the spirits is rather hasty He means to say that Alonso has shown great haste in reaching the conclusion about the shapes Francisco is amazed to see that those shapes disappeared in a mysterious way(c) What does Sebastian ask Alonso to doSebastian tells Alonso that the shapes having disappeared should not matter to them because they have left the eatables behind He asks Alonso to enjoy eating as they are extremely hungry but the king does not accept his offer of enjoying the dishes(d) How does Gonzalo try to dispel Alonsorsquos fear of those strange shapes What kind of references does he give to AlonsoGonzalo says that those who have travelled abroad have reported seeing even stranger sights than these shapes that Alonso and his companions have beheld Hence there is no reason to feel afraid of these shapes Gonzalo further adds that in his younger days he had heard strange stories from travelers and Alonso might have heard similar stories For instance it was said that there existed a certain race of

human beings who had huge lumps of flesh hanging at their throats and who therefore resembled bulls Then Gonzalo tells about a race of human beings whose heads were located at their breasts Gonzalo says that such stories were not believed by most people in those days but now-a-days these stories have become common(e) Explain the following lsquoEach putter-out of five for onersquoEnglish travellers often insured their trips with London brokers Those that went on foreign travels those days used to deposit a certain amount with some firm or company in London before their departure If the travelers failed to return the money was forfeited by the company with which it had been deposited But this money was repaid five-fold if the travelers returned safe and sound In this way a traveler stood a great chance of recovering the entire cost of his

travels(f) Give the explanatory meanings of the following expressions in the context of the above extract (i) Dewlapped (ii) Wallets of flesh

(iii) Putter-out(i) Dewlapped having big lumps of flesh at the necks(ii) Wallets of flesh large masses of flesh looking like bags(iii) Putter-out to invest money before commencing the travel

  • General methods of preparation of hydrogen
  • Chapter Dimensional Analysis (Summary)
    • Properties of Charge
Page 24:   · Web viewSubject. Topic. Summary. Execution. Hindi. व्याकरण. शरीरके अंगो के नाम लिखिए. 1) आँख 2) नाक 3

Q3What is tonoplastVacuoles covered by a covering called tonoplast

Bengali(2Nd

Language)

ফ ফটক ো ফটক (কলিবতো ) ভোষ মসোপো3 gtPোয়

একটি লেমসয়র ীবস লেপরম লিকভোসব ফসট ওসঠ তো লেদলিসয়স4 কলিব লেপরম Pই য় লেই ময়ই বনত কোস পলিরত য় ফ লেফোটো বো োসফোটো লেটো ব2 কো য় লেমসয়সদর ব gtয13 লেপরসমর 4লিব ফসট উসঠস4 এই কলিবতোয় লেপরম মোষসক মত gtযর মস লেফস লিদসয় পরকষস বাোচোসোর gtয োত বো2োয় কলিবতোয় লেমসয়টির পসব13র দঃসর কো বো সও লেমসয়টি লেই পসর পলিক সত চোয়ো োরী ীবসর কোস4 পরম লেPৌবস লেপরমসক পোবোর পরব ইচছো োকসও তো পসর লেলিতবোচকতোয় পলিরত য় কলিব ভোষ মসোপো3 যোয় লেP ক লেপরসমর

কলিবতোয় ব gtযবহত লিবসষ লিক4 সvর অ13 লেদওয়ো ১) রসবোো= লেP লিবলিভনন রকম ডোকসত পোসর২) ো= পোর ৩) ঠলি = লেচোসর বZ৪)আই বস2ো=অলিববোলিত৫)শইসয় = োলিয়ত কসর৬)োতপাোচ= লিবলিভনন পরকোর৭)দ2োম = v কসর বZ কসর লেদওয়ো৮)লেরলিং =লেোোর দৈতরী লেব2ো৯) বনত= একঋত১০) পাোর = বসকরো2

Organelles Functions

1 Endoplasmic reticulum

2 Mitochondria

3Golgibodies

4 Ribosome5Lysosome

6Plastids

7 Centrosome

i) Supportive framework for the cellii) Synthesis and transpost of proteinsRelease of energy in the form of ATPi) Synthesis and secretion of enzymes hormoneii) Formation of vacuoles lysosomei) Protein Synthesisi) Intracellular digestionii) Destroy foreign substancei )Leucoplast - stores starchii)chloroplast - trap solar energyiii) Chromoplast - imparts colour toflowers amp fruitsi) Initiates and requlates cell division

কলিবতো তোর অ13সক ভোষোয় পরকোো কসর ঘলিরসয় ব যকত কসরস4 লেপরসমর ফতো আর লিবফতো লেক গোঢ় কসর লেদোসো কলিব ভোষ মসোপো3 যোসয়র অলিভবসর অ যলিদক

Economics

Factors of Production

Welcome to the new sessionToday we are going to start the first chapter of Class XThe name of the chapter is Factors of productionBy the name I hope you all can recall a glimpse of what you have learnt in the second chapter of Class IX

NowProduction is the process of creating the various goods and services which are consumed by the people of the country to satisfy their wants

Thus it is the process in which some materials are transformed from one form to another to create utility and value in goods

For example utility can be created by changing the form of a commodity ie

Making of table out of wood by a carpenter for his customer here the wood is getting transformed into table creating utility for his customer and he can also command a price for it

On the other hand Housewives perform very

useful activities at home which create utility but their domestic activities are not included in production because they have no money value

So we can also say that Production denotes two things firstly creation of utility and secondly creation of value

Production is not complete unless it reaches the consumer

An increase in production will increase the economic welfare of the consumers and hence the aim is to raise the production level of the country

Again production of a good or service is only possible if certain resources or

Questions

1 What do you mean by production

Answer Production means the creation of goods and services for the purpose of selling in the market

In fact production involves the transformation of inputs into outputs

Hence production denotes two thingsCreation of utility and creation of valueUtility and value can be created by changing the form by changing the place by changing the time and by rendering services

Example Transformation of raw

materials into finish goods such as potter creates utility by converting mud into utensils assembling of small parts to make bigger machinery

Production also includes services such as distribution and marketing

2 What are the factors of production

Answer Factors of Production refers to the resources and inputs needed for producing goods and servicesThese inputs can be classified as

Land Labour

Capital Enterprise

Land Land is defined to include not only the surface of the earth but also all other free gifts of nature(for example mineral resources forest resources and indeed anything that helps us to carry out the production of goods and services but is provided by

inputs are used together in right proportion

A resource or an input which helps in the process of production to obtain an output is called FACTOR OF PRODUCTION

These factors of production can broadly be categorized into four parts 1LAND 2LABOUR3CAPITAL4ENTERPRISE (ORGANISATION)or Entrepreneur

The above factors are all interdependent on each other and they play a major role in production process

FACTORS OF PRODUCTION

LANDCAPITAL

LABOUR ENTREPRENEUR

nature free of cost)LabourLabour refers to the human efforts that need to be combined with other factors of production for creating an output

CapitalAll man ndash made means of production is called capita example machineries which help in further production Money when used for starting any business for purchasing raw materials machinery tools etc it is regarded as capitalCapital also includes physical capital like factories machineriestoolsbuildingsequipments etcEnterpriseThe task of bearing risks is called enterprise and the person who bears these risks of business is called the entrepreneurThus an entrepreneur is one who organises production takes important decisions regarding production hires and purchases factors of production and bears the risk and uncertainty involved in productionOrganisation refers to the services of an entrepreneur who controls organises and undertakes all risks One who plans organises and manages a business enterprise is an organiser

Physics Chapter 1 Force

Force is an external agent capable of changing the state of rest or motion of a particular body It has a magnitude and a direction The direction towards which the force is applied is known as the direction of the force and the application of force is the point where force is applied The Force can be measured using a spring balance The SI unit of force is Newton (N)

Question 1

State the condition when on applying a force the body has

(a) the translational motion

(b) The rotational motion

Solutions

(a) Translational motion is produced when the body is free to move

(b) Rotational motion is produced when the body is pivoted at a point

Question 2

Define moment of force and state its SI unit

Solutions

The moment of force is equal to the product of the magnitude of the force and the perpendicular distance of the line of action of force from the axis

of rotation

The SI unit of moment of force is Newton times meter

= Newton meter (Nm)

Commercial Studies

Stake holders In this topic you will be come to know about the meaning and concept of stakeholders

How stakeholders are different from shareholders

Questions1 What do you mean by the term stake holdersAnswer) The term stake holders have developed from the words which mean an interest or expected benefit Stakeholders mean all those individuals groups and Institutions which have a state (interest) in the functioning and performance of a commercial organisation or a business enterprise2 What do you mean by share holdersAnswer) The person and Groups who own the shares of the joint stock company by providing capital to the company are called shareholders Shareholders are the internal stakeholders shareholders are one out of several stake holders3 How are shareholders different from stakeholdersAnswer)i) The term shareholders is related to only joint stock company whereas stakeholders are related with all business organisationsii) Stakeholders maybe any individual having financial stake in business organisation whereas a shareholders are those individuals who are holding shares in the company4) How are shareholders different from creditorsAnswer) i) Shareholders are internal stakeholders while creditors are external stakeholdersii) Shareholders invest in the capital of the company whereas creditors give loan to the companyiii) Shareholders are the members of the company with voting rights but creditors are not the members of the company

English 1 Transformation of sentences

Sentences A sentence is a group of words which makes complete sense

e Assertive sentencesf Imperative sentencesg Interrogative sentencesh Exclamatory sentences

Sentences can be changed from one grammatical form to another without changing the meaning of the sentence This is known as transformation of sentences

Exercise 1 Change the following affirmative sentences into Negative sentences

a He is a good manHe is not a bad man

b Ram loves SitaRam is not without love for Sita

c Only he stood first in the classNone but he stood first in the class

d Ankit was wiser than he

He was not so wise as Ankite He did it

He did not fail to do itf As soon as I reached college the

bell rangNo sooner did I reach college than the bell rang

g He finished everythingHe left nothing unfinished

h It always pours when it rainsIt never rains but it pours

Math Topic Commercial MathematicsChapter ndash Goods and services Tax

What is GSTAns It is a abbreviated term of Goods and Service Text which is an indirect tax levied on the sale of goods and rendering servicesSome terms related to GSTDelar Any person who buys goods or services For resale is known as a delar A delar Can be a firm or a companyIntra-state sales Sales of goods and services within the same state or same union territory are called intra- state salesInter-state sales Sales of goods and services outside the state or union territory are called Inter-state sales4) Input GST GST is paid by dealers on purchase of goods and services are called input GST5) Output GST GST is collected from customers on sale of goods and services are called output GST6) Types of GST There are three taxes applicable under GST(i) Central Goods and Services Tax (CGST)(ii) State Goods and Services Tax (SGST) or Union Territory Goods and Services Tax (UTGST) Both these taxes are levied on intra-state sales Here GST is divided equally among central and state governments(iii) Integrated Goods and Services Tax (IGST) IGST is levied on inter- state sales It is also levied on import of goods and services into India and export of goods and services from India

Subject Eng Literature (The Merchant of Venice ndash William Shakespeare)Topic Act III Scene 4 Lines 1 to 44 (Portia hellip To wish it back on you fare you well Jessica)[Students should read the original play and also the paraphrase given in the school prescribed textbook]

Summary Questions amp AnswersIn this scene we suddenly find a new element in the character of Portia We have already seen her possessed of every graceful womanly quality but now she shows that she is capable of rapid decision and determined action She shows this by her sudden resolve to hasten to Venice with a daring scheme for the rescue of Antonio This is an important scene in the dramatic action for it leads up to and renders possible the striking events of the famous trial scene which is one of the greatest striking elements of the play Moreover the fact that all the characters of importance are now assembled together in Venice makes the union of the main plot and the secondary story complete

(1) LORENZO Madam although I speak it in your presence(Line 1-9)

You have a noble and a true conceit

Of god-like amity which appears most strongly

In bearing thus the absence of your lordBut if you knew to whom you show this honourHow true a gentleman you send reliefHow dear a lover of my lord your husbandI know you would be prouder of the workThan customary bounty can enforce you

(a) Where is Lorenzo Why is he here To whom is he referring as lsquoMadamrsquo

Lorenzo is at Portiarsquos residence He had met Salerio on the way and Salerio had begged him to come along with him to

o In this scene Portia Nerissa Lorenzo Jessica and Balthazar appear

o Portia requests Lorenzo and Jessica to be in charge of her house during her absence from Belmont because she and Nerissa have decided to spend the days in meditation and also in visiting the holy places in the neighbourhood of Belmont She has already instructed her people to acknowledge both Lorenzo and Jessica as master and mistress of house during her absence Lorenzo and Jessica gladly agree to look after the house of Portia

handover the letter from Antonio to Bassanio The letter carried the bad news about Antoniorsquos arrest for non-payment of loan taken from Shylock Hence Salerio might have preferred company to break this bad news to Bassanio He is referring to Portia as Madam(b) What does Portia say on hearing the above extract

Portia says that she has never regretted doing good to others Friends who spend a lot of time together and really are there for each other have many traits in common As Antonio is Bassaniorsquos best friend saving him is like saving Bassanio who is like her own soul She asks Lorenzo to take care of management of the house till Bassanio is back(c) What does Portia send with Bassanio and why

On hearing about Antoniorsquos troubles on account of Bassanio her husband Portia immediately sends him with enough gold to repay the debt many times over to Venice to help Antonio out of his misfortune

(2) Lorenzo Madam with all my heart (Line 36-40)

I shall obey you in all fair commands

Portia My people do already know my mindAnd will acknowledge you and JessicaIn place of Lord Bassanio and myselfSo fare you well till we shall meet again

(a) Where are Lorenzo and Portia at this time What lsquofair commandsrsquo are given to Lorenzo

Lorenzo and Portia are at Belmont during this scenePortia reveals to Lorenzo that she has sworn to contemplate in prayer at a monastery around two miles away until her husband returns from Venice She tells him that Nerissa would accompany her and asks him to manage the house with Jessica till things are settled In response Lorenzo tells her that he would be obliged to do whatever she asks him to do(b) Where is Portia actually going and why

Portia tells Lorenzo that she would live a life of contemplation and pray at a monastery which is two miles away from her place In reality Portia plans to go to Venice in disguise with Nerissa and argue the case in defense of Antonio She is very sure that her plan would succeed

ClassXI (ScienceHumanitiesCommerce)Subject Topic Summary Execution

Computer Science

(APC)

Ch ndash 1 Numbers

(Numbers in different bases and

their Arithmatical operations)

Number System In computers Number System is defined as a writing system to represent the numbers in different ways ie we are using different symbols and notations to represent numbers There are four ways we can represent the number ndash Binary Decimal Octal and Hexadecimal

Decimal Number SystemThis number system consist 10 digits These are 0 1 2 3 4 5 6 7 8 amp 9

Binary Number SystemThis number system has only two digits these are 0 and 1 Here 0 stands for off while 1 stands for on

Octal Number SystemThis number system has 8 digits these are 0 1 2 3 4 5 6 amp 7

Hexadecimal Number SystemThis number system has 16 digits these are 0 1 2 3 4 5 6 7 8 9 A B C D E F Here the value of the alphabets are as follows A=10 B=11 C=12 D=13 E=14 F=15

Rules for conversion decimal number to Binary1 Divide the decimal number by 22 If the number will not divide equally by 2 then round down the answer to the nearest whole number (integer)3 Keep a note of the remainder it should be either 0 or 14 Keep repeating the above steps dividing each answer by 2 until you reach zero5 Write out all the remainders from bottom to top This is your binary solution

For example Lets convert 32 to binary 2 32 2 16 - 0 2 8 - 0 2 4 - 0 2 2 - 0 2 1 - 0 0 - 1

The binary equivalent of 3210 is 1000002

Try the follwing youself1 2410

2 4810

3 1210

History GROWTH OF NATIONALISM

The second half of the 19th century witnessed growth of political consciousness and a sense of Nationalism among the IndiansThere were various factors for growth of Indian Nationalism- As a result various political associations were formed in different provinces by the educated Indians Surendranath Banerjee organized a meeting of National conference at Calcutta Ultimately the National Congress was founded in Bombay in 1885This body became the vanguard of Indian struggle for freedom The congress leaders were known as moderates because they followed a policy of prayer and petition A large number of Indian leaders had experienced in political agitation The Political situation of England was also changed Moreover increasing revolutionary activities in Maharashtra Punjab and Bengal became serious concern to the British Government In this

QUESTION1 What do you mean by Nationalism ANSWER 1 Nationalism is defined as loyalty and devotion to own nation especially a sense of national consciousnessQUESTION 2 What are the causes of nationalism ANSWER 2 There were various factors for growth of nationalism

1 Spread of western education2 The progress of vernacular press and

patriotic literature3 The economic exploitation of our

country by the colonial rulers4 International affairs

QUESTION 3 Who organized National conference in Calcutta in 1883 ANSWER 3 Surendranath BanerjeeQUESTION 4 When did Indian National Congress formANSWER 4 Indian National Congress was formed in 1885 in BombayQUESTION 5 Who were ModeratesANSWER 5 The Early Nationalists were also known as Moderates Their emergence marked

background Lord Curzon became Viceroy in India He had no respect for the Indian National Congress

the beginning of the organized national movement in India They believed in British justice and were loyal to them They followed a policy of prayer and petition They demanded constitutional reforms of our country Impotant Moderate leaders were Pherozshah Mehta Dadabhai Naorozi and Surendranath Banerjee etcQUESTION 6 What do you know about Extremism in Indian National movementANSWER 6 In the beginning of 20th century a new class of national leaders emerged in India which was different from the moderate groups They started more aggressive movement against the British empire The goal of extremists was ldquoswarajrdquo Important extremist leaders were Bal Gangadhar Tilak Lala Lajpat Rai Bipin Chandra Pal etcQUESTION 7 Mention the places which were the main centres of Revolutionary movementANSWER 7 Maharashtra Bengal and Punjab

Physics

Chapter Dimensional Analysis

(Summary)

The dimensions of a physical quantity are the powers to which the fundamental units are raised in order to obtain the derived unit of that quantit

The physical quantites lengthmasstime are represented by [L] [M] [T] resp let they are raised to powers ( dimesions) abc resp then any physical quantity can be represented by [ La Mb Tc ] Examples

1 Area area = L x B = [L] x [L] = [M0 L2 T0 ]

2 Density density = massvolume = [M][L3] = [ M L-3]

3 Velocity velocity = distancetime = [L][T] = [LT-1]HW Try to find out dimension of acceleration Acceleration = velocity timeNB One can find the SI Units Using Dimension Analysis Such as for area we have [L2] so its SI unit is m2

Biology Topic ndash Chp-1 The living world

Today we will start the first chapter the living world Here we discuss about the characteristics of living organism and what are the difference between them and nonliving substances We also discuss about the contribution of different Scientists

There are over 500000 species of plants andover a million species of animal are present on earth Some 15000 new species were discovered every yearQ1 What is a living organismbull A living organism is primarily physico -chemical material that demonstrate a high degree of complexity is capable of selfRegulation possesses a metabolism and perpetuates itself through timeQ2 What are the differences between livingand non-livingsi) Compared with non-living living organisms

have more complex organised structure and their use of energy is more controlled amp efficientii) Living things reproduce their own kind by forming new cells which contains copies of their genesiii) Each organism has some degree of homeostasisie it is able to make adjustments so that internal environment remains constantQ3 Write contributions of following Scientists i) Aristotle - One of the first theories in Biology places all living things in a hiearchieii) AV Leeuwenhoek - was the first to observe living single celled organisms under microscopeii) Carolus Linnaeus - developed the binary system for naming of organisms and classificationiii) Geregor Johann Mendel ndash discoverbasic principles of inheritanceHomework i) C Darwin ii)Schleiden

Math Trigonometric functions

1 Overviewi) Trigonometry The word lsquotrigonometryrsquo is derived from the Greek words lsquotrigonrsquo and lsquometronrsquo which means measuring the sides of a triangle An angle is the amount of rotation of a revolving line with respect to a fixed line Usually we follow two types of conventions for measuring angles ie a) Sexagesimal system b) Circular system In Sexagesimal system the unit of measurement is Degree In Circular system the unit of measurement is Radian ii) Relation between degree and radianThe ratio of circumference of a circle to its diameter is always a constant This constant ratio is a number denoted by π which is taken approximately as 227The relationship between degree amp radian measurements is as follows2 right angles = 180deg= π radians1radian = 180degπ=57deg16(approx) 1deg=π180 radianiii) Length of an arc of a circleIf an arc of length s subtends an angle θ radians at the center of a circle of radius r then s=rθiv) Area of a sector of a circleA sector is like a pizza slice of the

Q) Express the following angles in radiana) 45deg b) 40deg3730Ans a) We have 180deg=π radiansi e 45deg= πtimes45180 radian = π4 radiansb) 40deg3730= 40deg37+3060 minute= 40deg 37 +12 minute= 40deg+ 752 minute=40 + 75(2times60) degree=3258 degreeNow 180deg=π radianie 3258 degree= (πtimes325) (180times8) radians = 65π288 radiansQ) A circle has a radius of r=12 meters What is the length of an arc traced out by a 60deg angle in the center of the circleAns In this problem we know both the central angle (60deg) and the radius of the circle (12) All we have to do is plug those values into our equation and we get

s = 2π(12)(60360)s = 24π6s = 4πSo the length of an arc traced out by a 60deg angle in a circle with a radius of 12 meters equals 4π meters asymp 1257 metersQ) Find the area of the sector with a central angle 30deg and a radius of 9cmAns GivenRadius r = 9 cmAngle θ = 30degArea of the sector = θ360degtimesπr2

= 30360degtimes227times92=2121cm2

circle It consists of a region bounded by two radii and an arc lying between the radiiThe area of a sector is a fraction of the area of the circle This area is proportional to the central angle In other words the bigger the central angle the larger is the area of the sectorArea of Sector = θ2 times r2 (when θ is in radians)

Area of Sector = θ times π360 times r2 (when θ is in degrees)

COMMERCE

CLASSIFICTION OF HUMAN ACTIVITIES-ECONOMIC AND NON-ECONOMIC

Welcome to the new sessiontoday we are going to start the first chapter of Class XI The name of the chapter that we are going to start is

lsquoClassification of Human Activities ndasheconomic and non-economicrsquo

Now let us start the chapter by considering human beings and the activities they perform throughout the day

Human activities means all those activities that human beings undertake to satisfy their wants

Human wants on the other hand are the desire of human beings for goods (vegetables fruits rice etc) and services (services of doctors teachers lawyers etc) that they require to live

Now these human activities continue throughout life as human wants are unending unlimited and recurring as human beings desire for better living throughout their lives

Now human activities can be classified into two categories

Human activities

Economic activities Non-economic activities

Economic activities are

Questions1 What are human activities

Answer Human activities mean all those activities that human beings undertake to satisfy their wants

Example A man working in an office

A boy playing in the garden

2What are the characteristics of human activitiesAnswer the characteristics of human activities are as follows

Human activities are undertaken by men women and children and these activities involve human efforts

Human activities are undertaken to satisfy human wants which are unlimited

Human activities continue throughout life

Human activities are performed for both earning money and personal satisfaction

3What is economic activitiesGive example

Answer Economic activities are undertaken by human beings with the object of earning money acquiring wealth and thereby satisfying human wantsExample

Selling of goods by a shop keeper to his customer

A clinic run by a doctor Service of a teacher in school or college

undertaken by human beings with the object of earning money and acquiring wealth

These activities result in the production of economic goods and services

Example Human activities(ie working in factories officesshops) which produce direct economic benefits

Non-economic activities are inspired by human sentiments and emotions such as love for the family desire to help the poor and love for the country

Thus these human activities (eg praying playing sleeping) produce no direct economic benefits and they are also not related to earning money and acquiring wealth

4 What are the characteristics of economic activities

Answer The characteristics of economic activities are as follows

Economic motiveEconomic activities are undertaken to earn money and acquire wealth

ProductiveEconomic activities involve productiondistribution and exchange of goods and services to create wealth

Economic growthEconomic activities determine the level of economic development of a country and standard of living of its citizens

Socially desirableEconomic activities are socially desirable for society

Economic resourcesEconomic activities make use of all the economic resources such landlabourcapital etc

5 What do you mean by non-economic activitiesExampleAnswerNon-economic activities are inspired by human sentiments and emotions such as love for the family desire to help the poor and love for the countryThese activities are not undertaken for monetary gain but for onersquos satisfaction and happinessExample

a mother looks after her children

a student donates blood8 Differentiate between Economic activities and Non-economic activities

Economic activities

Non-economic activities

1to earn living and acquiring wealth2Result can be measured in terms of money

3ExampleBusinessprofession and employment

1 to obtain some satisfaction

2Result cannot be measured in terms of money

3ExampleFamily-orientedreligious socialCultural and national

BUSINESS STUDIES

BUSINESS ENVIRONMENT

Welcome to the new sessionToday we are going to start the first chapter and the name of the chapter is Business Environment

In todayrsquos world every business enterprise is a part of the society It exists and operates in association with various groups in society such as customers suppliers competitors banks and financial institutions government agencies trade unions media and so on All these groups influence the functioning of business in one way or the other They constitute the environment of businessConcept of Business Environment

The term lsquobusiness environmentrsquo refers to the sum total of all individuals institutions and other forces that lie outside a business enterprise but that may influence its functioning and performance

The main features of business environment

Totality of External forces General and Specific forces Interrelatedness Complexity Dynamic Uncertainty Relativity

The Interrelation between business and its environment

The business enterprise is an open system It continuously interacts with its environment It takes inputs

Prepare the following questions from todayrsquos assignment

1 What do you mean by business environment

The term lsquobusiness environmentrsquo means the aggregate of all forces factors and institutions which are external to and beyond the control of an individual business enterprise but they may influence its functioning and performance Business environment is the macro framework within which a business firm a micro unit operates It consists of several interrelated and interacting elements

2 Explain the main features of business environment in brief

Totality of External forces-Business environment is the sum total of all things external to a business environment

General and Specific forces-It includes both the forces general forces are the economic social political legal and technological conditions which indirectly influence all business enterprise Specific forces are the investors customers competitors and suppliers which influence individual enterprise directly

Interrelatedness-Different elements of environment are interrelated for an example growing awareness for health care has increased the demand for health foods

Complexity- Business environment id

(such as raw materials capital labour energy and so on) from its environment transforms them into goods and services and sends them back to the environment

Fig 1 Business Environment Relationship

complex in nature as the elements keep on changing example economic technological and other forces changes in demand for a product and service

Dynamic-Business environment is not static it keeps on changing

Uncertainty- Itrsquos very difficult to predict future events such as technology and fashion which occur fast and frequently

Economics Basic Economic ConceptsSub topic

Microeconomics and

Macroeconomics

Welcome to the new sessiontoday we are going to start the first chapter of Class XI The name of the chapter that we are going to start is Basic Economic concepts

Now Economics covers the study of human activities Human activities are those activities which are performed by humans to satisfy their wants

Thus Human wants are unlimited and therefore economic activities such as production exchange and consumption are needed in order to satisfy those wants

The study of economics is divided largely in two parts which areMicroeconomics and Macroeconomics

SUBJECT- MATTER OF ECONOMICS

MICROECONOMICS MACROECONOMICS

Questions1Who has coined the words micro and macro economics

Answer Ranger Frisch coined the words lsquomicrorsquo and lsquomacrorsquo in 1933 to denote the two branches of economic theory namely microeconomics and macroeconomics

2What is microeconomicsAnswer It is the study of behaviour of individual decision ndash making unit such as consumers firms etc

3 What is macroeconomicsAnswer Macroeonomics is the study of overall economic phenomena like employment national income etc

4 What is the importance of microeconomicsAnswer

Microeconomics helps in formulating economic policies which enhance productive efficiency and results in greater social welfare

It helps the government in formulating correct price policies

It explains the working of a capitalistic economy where individual units(producers and consumers ) are free to take their own decision

Micro means a small part in

microeconomics we do not study the whole economy Hence we study an individual consumer and his or her choices and a producer and his or her profit maximizing decisions in the market Thus it does not mirror what happens in the economy as a whole

Macroeconomics on the other hand studies the economy as a whole It is concerned with aggregate and depicts the entire picture of the economyMacroeconomics deals with the national income aggregate investment aggregate consumption etc

Features of Microeconomics It deals with small

parts of the country Hence it looks at

individual consumers firms and industries

It deals with individual income consumption and savings

It studies the determination of price of any product or factors of production

It deals with the working of market via the price mechanism which is nothing but the determination of price and quantity of a commodity by the forces of demand and supply

Features of Macroeconomics

It deals with the study of the economy as a whole

It is concerned with

5 Give a limitation of microeconomics Microeconomics fails to explain the

functioning of an economy as a whole It cannot explain unemployment illiteracy and other problems prevailing in the country

6 What is the importance of macroeconomics It gives overall view of the growing

complexities of an economic system It provides the basic and logical

framework for formulating appropriate macroeconomic policies (eg for inflation poverty etc )to direct and regulate economy towards desirable goals

7What is the limitation of macroeconomics It ignores structural changes in an

individual unit of the aggregate

8 Differentiate between Microeconomics and Macroeconomics

Microeconomics Macroeconomics

the study of aggregates

National income aggregate savings and aggregate investments are major concepts dealt within macroeconomics style

It studies the determination of general price levels

It investigates into the problem of unemployment and the achievement of employment

It studies the aspect of decision making at the aggregate and national levels

It includes all growth theories whether related to developed or developing economies it also includes the study of economic systems and the working of the economy under different systems

Note Both Micro and macro economics are complementary and should be fully utilized for proper understanding of an economy

1It studies economic aspect of an individual unit2It deals with individual incomeConsumption and savings

3 It facilitates determination of price of any product or factors of production

4 Itrsquos scope is narrow and restricted to individual unit

1It studies the economy as a whole

2It deals with the national income aggregate consumption and aggregate savings3 It facilitates determination of general price level in an economy

4 Itrsquos scope is wide as it deals with economic units on the national level

ACCOUNTS

Introduction to Accounting and Book-keeping

Today I am going to share you the meaning of Accounting and Book-keeping and its related terms bullAccounting bullBook Keeping bullAccountsbullTypes Of Accounts bullAccounting Cycle

bull Meaning of accounting

Ans ) Accounting is the art and science of recording classifying and summarising monetary transactions

bull Meaning of Book-keeping

Ans) Bookkeeping is the art of recording business transactions with the view of having a permanent record of them and showing their effect on wealth

bull Meaning of account

Ans) The term account means a record of

business transactions concern a particular person of firm asset or income or expense It is a summarised record of all transactions which take place in an accounting year

bull Types of accountsPersonal accounts ndash Personal accounts relating

to person and Organisation are known as personal accounts Example Ramrsquos Account ABC amp Co Account etc

Real account - The accounts related to tangible and intangible assets are called real accounts Example Cash Account Furniture Account etc

Nominal account- Accounts related to expenses losses incomes and gains are known as nominal accounts Example Wages Account Salary Account Discount Account etc

bull Accounting cycle Accounting cycle refers to a complete sequence of accounting activities It begins with recording of transactions and ends with the preparation of a balance sheet

Chemistry TopicAtomic Structure

Thomsonrsquos atomic modelThomson (1898) was the first to propose the model of an atomHe proposed that an atom can be regarded as a uniform sphere of positive electricity in which requisite number of electrons are embedded evently to neutralize the positive chargeThis is just like plums embedded in a pudding or seeds evently distributed in red spongy mass of a watermelonThis model of atom is known as ldquoPlum-Pudding modelrdquo or

Q1)What is the fundamental constituents of atomAns Electron Proton and neutrons are the fundamental constituents of atomQ2)What is the value of fundamental unit of electricityAnsThe charge carried by one electron is sad to be the fundamental unit of electricityIts magnitude is 48times10-10esuOr 1602times10-19C Q3)Name the element containing no neutronAnsOrdinary hydrogen atom or protium 1H1

Types of AccountPersonal AccountReal AccountNominal AccountBalance Sheet (opening)

ldquowatermelon modelrdquoThis model could explain the electrical neutrality of an atom but failed to explain the result of scattering experiment carried out by Rutherford in 1911So it was rejected ultimately

Q4)Why is an electron called universal particleAns Itrsquos mass and Charge are independent of its source

EVS Chapter 1 ndash Modes of Existence

Modes of existence When one speaks normally about the mode of existence of some group or individual one refers to their customs their mode of being their ethology their habitat in some way their feeling for a placeDifferent modes of exixtence are ndash

1 Hunting ndashGathering2 Pastoral3 Agricultural4 Industrial

1 Hunting and gathering Hunting and gathering mode of existence is characterized by obtaining food from hunting wild animals including fishing and gathering wild plants From their earliest days the hunter-gatherer diet included various grasses tubers fruits seeds and nuts Lacking the means to kill larger animals they procured meat from smaller game or through scavenging

Societies that rely primarily or exclusively on hunting wild animals fishing and gathering wild fruits berries nuts and vegetables to support their diet are called hunting and gathering societies

At least this used to be practice of human beings before agriculture is invented As their brains evolved hominids developed more intricate knowledge of edible plant life and growth cycles

Q) Write the features of Hunting ndash gathering societiesAns - There are five basic characteristics of hunting and gathering societies

i The primary institution is the family which decides how food is to be shared and how children are to be socialized and which provides for the protection of its members

ii They tend to be small with fewer than fifty members

iii They tend to be nomadic moving to new areas when the current food supply in a given area has been exhausted

iv Members display a high level of interdependence

v Labor division is based on sex men hunt and women gather

Political Science

Introduction to political science

Political science occasionally called politology is a social science which deals with systems of governance and the analysis of political activities political thoughts associated constitutions and political behaviorThe study of political science involves the study of both the

Answer the following questions-1 What is political science

Political science occasionally called politology is a social science which deals with systems of governance and the analysis of political activities political thoughts associated constitutions and political behavior

2 Short notes-

traditional and modern theories of politicsTraditionalClassical political sciencepolitical theory-Traditional political science is the study of politics before Second World War The methodology to study Politics was traditional (legal formaletc) the definition of politics traditional (Politics begins and end with state)area of study (constitution state machinery)was traditionalModern Political scienceModern political theory-Modern Political Theory critically examines the contemporary state of political theory making an assessment of the achievement and limitations of the Behavioural Revolution in its totality and reviews objectively the major paradigms and conceptual frameworks adopted by the disciplineContemporary attempts at the development of an integrated political theory involving the use of both traditional and modern concepts approaches and theories-Around late 1960s several political scientists realized the importance of both the traditional political theory and modern Political theory They began building an integrated theory of politics involving a systematic mixture of traditional and modern studies of politics It was held that the study of a complex and vast field like politics needs both traditional as well as

Classical political theory Modern Political theory

Homework-Learn

modern concepts and approaches for studying itrsquos all aspects

Subject Eng Literature (The Tempest ndash William Shakespeare) Topic Act I Scene 1 Lines 1 to 32 (Line 32 ndash Gonzalo hellip If he be not born to be hanged our case is miserable) Date 13th April 2020 (3rd Period)

[Students should read the original play and also the paraphrase given in the school prescribed textbook]Summary Questions amp Answers

[SUMMARY OF THE ENTIRE SCENE]

o The play starts with the scene of a severe storm at sea Alonso (King of Naples) Sebastian (Alonsorsquos brother) Ferdinand (Alonsorsquos son) Gonzalo Antonio (the usurping Duke of Milan) are in a ship in the midst of the storm

o The mariners are trying their best to control the vessel from running aground and are totally following the orders of their Master the Boatswain They have scant success

o The mariners become extremely unhappy and annoyed when most of the passengers arrive on the deck thereby hampering their effort to save the ship There is serious confrontation between them and the passengers who are part of the Kingrsquos entourage

o The mariners could not save the ship

SUMMING-UP

(i) Vivid description of the scene which gives a realistic description of terror and confusion of a tropical storm

(ii) Shows Shakespearersquos accuracy of knowledge in describing the naval operations and also matters of seamanship

(iii) The opening scene justifies the title ndash The Tempest

UNANSWERED QUESTIONS

(i) The King always travels with his entire fleet including his soldiers Where

(1) GONZALO Nay good be patient (Line 15-26)BOATSWAIN When the sea is Hence What cares these

roarers for the name of the king To cabin silence Trouble us not

GONZALO Good yet remember whom thou has aboardBOATSWAIN None that I more love than myself You are a

councillor if you can command these elements to silence and work

the peace of the present we will not hand a rope more use your authority If you cannot give thanks you have

lived so long and make yourself ready in your cabin for the mischance of the hour if it so hap [To the Mariners]

Cheerly good hearts [To Gonzalo] Out of our way I say

(a) To whom is the boatswain speaking What does he mean by lsquoNone that I more love than myselfrsquo

The Boatswain is speaking to Gonzalo the honest old councilor of the Duke of MilanBy using the words ndash lsquoNone that I love more than I love myselfrsquo means that for the Boatswain nobody is dearer to him than his own life

(b) What were the conditions that made the boatswain react in this way

The Boatswain reacts in this way because the storm is at sea and Alonso King of Naples Sebastian his brother Ferdinand his son Gonzalo Antonio the usurping Duke of Milan on board are in distress and in panic Thus they have rushed to the deck interrupting the work of the mariners

(c) What hope does Gonzalo take from the attitude of the boatswain

The insolent and authoritative attitude of Boatswain makes Gonzalo feel comforted He tells that there are no signs that the Boatswain will be drowned But his facial appearance and attitude shows that he is destined to die on land by hanging which in effect means that all on board will be saved Otherwise all the persons on board are doomed

(d) How can they lsquomake yourself ready in your cabinrsquo For what were they asked to make ready themselves

In order to make themselves ready in their cabin the

were the other ships

(ii) Why was the ship in that area Where was it coming from or going where

(iii) The ship broke apart What happened to those who were in the ship

passengers on board must prepare for death which they will possibly soon have to meetThey can retire to their cabins and offer prayers to the Almighty to save them from drowning

(e) What does the boatswain say when he is asked to be patient What does he order to the royal party

When the boatswain is asked to be patient and remain calm he says that he will be patient only when the storm will be over and the sea will be calm but as long as the storm blows and there is danger to the ship he cannot think of being patient He orders the royal party to go to the cabin and leave the mariners to their work

(2) GONZALO I have great comfort from this fellow (Line 27-36)

Methinks he hath no drowning mark upon him his complexion is perfect

gallows Stand fast good Fate to his hanging Make the rope of his destiny our cable for our own doth little advantage If he be not born to be hanged our case is miserable

(a) Why does Gonzalo regard the Boatswain in the midst of danger

In the midst of danger Gonzalo regards the boatswain because he feels that the Boatswain is a source of comfort and is bent upon to do his work sincerely which in this case is saving the ship and its passengers from the severest of raging storm

(b) What reasons does Gonzalo give when he says that none in the ship will die of drowning

Gonzalo is almost sure that none in the ship will die by drowning His says that there is no mark on the face of the boatswain that indicates that he will die by drowning On the other hand the lines on his face are strong indications that he will be hanged to death Therefore there shall be no danger of the shiprsquos sinking

(c) Explain the following ldquoStand fast good Fate to his hanging Make the rope of his destiny our cable for our own doth little advantage If he be not born to be hanged our case is miserablerdquo

The stated lines mean that if the will of destiny is to be carried out then the ship will not get wrecked and all the passengers will be saved The safety of the passengers therefore depends upon the will of fate being carried out in the case of the boatswain If however the boatswain is not to die by hanging then the passengers are also very unsafe because in that case the ship is likely to sink

(d) What order does the Boatswain give to the sailors

when he re-enters What does he say about the crying of the fellows inside the cabin

The boatswain orders the sailors to bring the topmast lower and bring the ship close to a stationary position with the help of the main sail He says that the fellows inside the cabin are moaning and crying in their distress louder than his voice and louder even than the roaring of the storm

Class XII (ScienceCommerceHumanities) Subject Topic Summary Execution

Computer Science

PropositionalLogic

Propositional logic is a procedure to provide reasoning through statementProposition A ststement that results in True or False is said to be proposition There are two types of propositionSimple proposition amp compound propositionSimple proposioton A simple proposition is one that is not a part of any other proposition Such sentential form of proposition is symbolized with english letters in short For example Ram is a claver student (TrueFalse)Where do you live (Not in True or False)Grapes are sweet (TrueFalse)It rains today (TrueFalse)Here we can see some statements anwer would be true or false but some staements answer can not give in terms of true or false Thus the sentences which can be answered in true or false are known as simple propositionAssigning propositon to a variableThe general syntax to assign propostion to a variable is as followsVariable = Simple propositonFor example A=Ram is a clever studentB= Grapes are sweetC= it rains todayCompound proposition

helliphellipto be continued in next classhelliphellipMath Relation Relation If A and B are two non-empty sets

then a relation R from A to B is a subset of AxB If R A x B and (a b) R then we say that a sube isinis related to b by the relation R written as aRbeg Let A be the set of students of class XII and B be the set of students of class XI Then some of the examples of relation from A to B arei) (a b) AXB a is brother of bisinii) (a b) AXB age of a is more than age of isinb Types of relation In this section we would like to study different types of relations We know that a relation in a set A is a subset of A times A Thus the empty set φ and A times A are two extreme relations For illustration consider a relation R in the set A = 1 2 3 4 given by R = (a b) a ndash b = 10 This is the empty set as no pair (a b) satisfies the condition a ndash b = 10 Similarly R = (a b) | a ndash b | ge 0 is the whole primeset A times A as all pairs (a b) in A times A satisfy | a ndash

Example 1 Let A be the set of all students of a boys school Show that the relation R in A given by R = (a b) a is sister of b is the empty relation and R = (a b) the primedifference between heights of a and b is less than 3 meters is the universal relationSolution Since the school is boys school no student of the school can be sister of any student of the school Hence R = φ showing that R is the empty relation It is also obvious that the difference between heights of any two students of the school has to be less than 3 meters This shows that R = A times A is primethe universal relation Example 2 Show that the relation R in the set 1 2 3 given by R = (1 1) (2 2) (3 3) (1 2) (2 3) is reflexive

b | ge 0 These two extreme examples lead us to the following definitionsDefinition 1 A relation R in a set A is called empty relation if no element of A isrelated to any element of A ie R = φ A times AsubDefinition 2 A relation R in a set A is called universal relation if each element of A is related to every element of A ie R = A times A Both the empty relation and the universal relation are some times called trivial relation Definition 3 A relation R in a set A is called(i) reflexive if (a a) R for every a Aisin isin(ii) symmetric if (a1 a2) R implies that (aisin 2a1)

R for all aisin 1 a2 Aisin(iii) transitive if (a1 a2) R and (aisin 2 a3) R isinimplies that (a1 a3) R for all aisin 1 a2 a3 AisinDefinition 4 A relation R in a set A is said to be an equivalence relation if R is reflexive symmetric and transitive

but neither symmetric nor transitiveSolution R is reflexive since (1 1) (2 2) and (3 3) lie in R Also R is not symmetric as (1 2) R but (2 1) isin notinR Similarly R is not transitive as (1 2) R and (2 3) R but (1 3) R isin isin notinExample 3 Show that the relation R in the set Z of integers given byR = (a b) 2 divides a ndash b is an equivalence relationSolution R is reflexive as 2 divides (a ndash a) for all a Z isinFurther if (a b) R then 2 divides a isinndash b Therefore 2 divides b ndash a Hence (b a) R which shows that R is isinsymmetric Similarly if (a b) R and (b c) R isin isinthen a ndash b and b ndash c are divisible by 2 Now a ndash c = (a ndash b) + (b ndash c) is even (Why) So (a ndash c) is divisible by 2 This shows that R is transitive Thus R is an equivalence relation in ZExample 4 Let L be the set of all lines in a plane and R be the relation in L defined as R = (L1 L2) L1 is perpendicular to L2 Show that R is symmetric but neither reflexive nor transitiveSolution R is not reflexive as a line L1 can not be perpendicular to itself ie (L1 L1) R notinR is symmetric as (L1 L2) Risin

L1 is perpendicular to L2rArr L2 is perpendicular to L1rArr (L2 L1) RrArr isin

R is not transitive Indeed if L1 is perpendicular to L2 and L2 is perpendicular to L3 then L1 can never be perpendicular to L3 In fact L1 is parallel to L3 ie (L1 L2) R isin(L2 L3) R but (L1 L3) Risin notin

Chemistry Solid state Characteristics if Solids(i)The particles are locked in fixed positions they are unable to change their relative positions and this brings a definite shape and volume of a solid(ii)In a solid the constituent particles are held by strong forces of attractionThe forces of attraction may be bonding or non bonding(iii)The constituent particles in a solid pack together as closely as possibleoccupying most of the available space within the solidThus the empty space in a solid is very smallThis makes a solid highly rigid and nearly incompressibleThis also explains why a solid has high density and exhibits slow diffusionClassification of Solids

Q1)Define Crystalline solids AnsA Solid that has a definite geometrical shape and a sharp melting pointand whose constituent particles (atomsmolecules or ions) are arranged in a long range order of definite pattern extending throughout the solid is called a crystalline solidExNaClQ2)Define Amorphous solids AnsA solid that does not have a definite shape and a sharp melting pointand whose constituent particles (atomsmolecules or ions) are not arranged in a definite pattern is called an amorphoussolid

Crystalline solidsAmorphous solids

ExGlassRubberQ3)Classify Crystalline Solids Crystalline Solids

Physics Coloumbrsquos Law (Summary)

Before Going Into Coloumbrsquos Law We Will First Learn What is Charge Properties of Charge and Always remember that charge is quantized ie a body always have static charge of magnitude equal to some integral multiple of fundamental electronic charge e= 16 x 10- 19 C

Charge is the property of matter that causes it to produce and experience electrical and magnetic effects The study of the electrical charges at rest is called electrostatics When both electrical and magnetic effects are present the interaction between charges is referred to as electromagnetic

There exist two types of charges in nature positive and negative Like charges repel and unlike charges attract each other

The type of charge on an electron is negative The charge of a proton is the same as that of an electron but with a positive sign In an atom the number of electrons and the number of protons are equal The atom is therefore electrically neutral If one or more electrons are added to it it becomes negatively charged and is designated as negative ion However if one or more electrons are removed from an atom it becomes positively charged and is called a positive ion

The excess or deficiency of electrons in a body gives the concept of charge If there is an excess of electrons in a body it is negatively charged And if there is deficiency of electrons the body becomes positively charged Whenever addition or removal of electrons takes places the body acquires a charge

The SI Unit of charge is coulomb (C) In SI units the current is a fundamental quantity having a unit of ampere (A) The unit of charge is defined in terms of the unit of current Thus one coulomb is the charge transferred in one second across the section of a wire carrying a

Ionic SolidsMetallicSolids

Molecular Solids

current of one ampere

As q = It we have1 C = (1 A) (1 s)

The dimensions of charge are [A T]

Properties of Charge

(1) Quantization of Charge Electric charge can have only discrete values rather than any value That is charge is quantized The smallest discrete value of charge that can exist in nature is the charge on an electron given as

e = plusmn 16 x 10- 19 C

This is the charge attained by an electron and a protonA charge q must be an integral multiple of this basic unit That is

Q = plusmn ne where n = 1 2 hellip

Charge on a body can never be (frac12)e (23)e or 57e etcWhen we rub a glass rod with silk some electrons are transferred from the rod to the silk The rod becomes positively charged The silk becomes negatively charged The coulomb is a very large amount of charge A typical charge acquired by a rubbed body is 10 - 8 C

Biology Reproduction in organisms

Welcome to this new session 2020-21Today in this first chapter we mainly discuss about reproduction types needs and life span of some organismsWe also discuss about difference between sexual and asexual reproduction

Q1 What is reproductionReproduction is defined as a biological processin which an organism gives rise to young onessimilar to itselfQ2 What are the needs of reproductionbulli) Reproduction maintain life on earthii) It enables the continuity of the species generation after generationiii) It creates genetic variation among populationsQ3 Define Life span and write some orgnisms life spanbull Life span is the period from birth to

the natural death of an organism- OrganismsLife span1 Butterfly 1 - 2 weeks2 Fruit fly 30 days3Dog 10-13 years4 Rose5-7 years5 Tortoise100-150 years6 Banyan Tree -200 - 250 yearsQ4 Reproduction is of two types in case ofanimals but in case of plants vegetative propagation is also present

Asexual Reproduction Sexual Reproductioni) Always uniparentalii) Gametes are not involvediii) Only mitotic division involvediv) Somatic cells of parents are involvedv) Offsprings are genetically similar to the parents

i) Usually biparentalii) Gametes are involvediii) Meiosis occurs during gametogenesis Mitosis occurs after fertilisationiv) Germ cells of the parents are involvedv) offsprings are genetically different from the parents

COMMERCE BUSINESS ENVIRONMENT

Welcome to the new sessiontoday we are going to start the first chapter of Class XII The name of the chapter is Business Environment

Already many of you have got some idea about the word business environment form the first chapter of business studies in class XI

In todayrsquos world every business enterprise is a part of the society It exists and operates in association with various groups in society such as customers suppliers competitors banks and financial institutions government agencies trade unions media and so on All these groups influence the functioning of business in one way or the other They constitute the environment of businessConcept of Business Environment

The term lsquobusiness environmentrsquo refers to the sum total of all individuals institutions and other forces that lie outside a business enterprise but that may influence its functioning and performance

The main features of business environment Totality of External forces General and Specific forces Interrelatedness Complexity Dynamic Uncertainty

Prepare the following questions from todayrsquos assignment

2 What do you mean by business environment

The term lsquobusiness environmentrsquo means the aggregate of all forces factors and institutions which are external to and beyond the control of an individual business enterprise but they may influence its functioning and performance Business environment is the macro framework within which a business firm a micro unit operates It consists of several interrelated and interacting elements

2 Explain the main features of business environment in brief

Totality of External forces-Business environment is the sum total of all things external to a business environment

General and Specific forces-It

Relativity

The Interrelation between business and its environment

The business enterprise is an open system It continuously interacts with its environment It takes inputs (such as raw materials capital labour energy and so on) from its environment transforms them into goods and services and sends them back to the environment

Fig 1 Business Environment Relationship

includes both the forces general forces are the economic social political legal and technological conditions which indirectly influence all business enterprise Specific forces are the investors customers competitors and suppliers which influence individual enterprise directly

Interrelatedness-Different elements of environment are interrelated for an example growing awareness for health care has increased the demand for health foods

Complexity- Business environment id complex in nature as the elements keep on changing example economic technological and other forces changes in demand for a product and service

Dynamic-Business environment is not static it keeps on changing

Uncertainty- Itrsquos very difficult to predict future events such as technology and fashion which occur fast and frequently

Business Studies

Human Resources Management

Human resource of an organisation are the aggregate of knowledge skills attitudes of people working in it

The management system which deals with human resources is called human resource management

Features of HRMbullComprehensive functionbullPeople-oriented

Question1) What do you mean by human

resource management Answer) Human resource management may be defined as that field of Management which has to do with planning organising and controlling the functions of procuring developing maintaining and utilising the labour force

bullAction oriented bullPervasive function bullContinuous function

2) Explain the features of HRM in brief

Answer)bullHuman Resource Management is concerned with managing people at work bull Human Resource Management is concerned with employees which bring people and organisations together so that the goals of each are met bullHuman resource management considered every employees as an individual and also promote their satisfaction and growth bull Human resource management is inherent in all organisations and at all levelsbullManagement of human resources are ongoing on never ending process which requires a constant alertness and Awareness of human relations

3) ldquoHR function is said to be pervasiverdquowhy

Answer) Human resource management is required in all organisations whether it is private or government organisations armed forces sports organisations etc It permeatsall the functional areas like production marketing finance research etc This from this feature of human resource management it can be said that it is pervasive in nature

Economics Demand Q1DEFINITION OF DEMANDIn economics demand is the quantity of a good that consumers are willing and able to purchase at various prices during a given period of timeQ2DEMAND CURVEIn economics a demand curve is a graph depicting the relationship between the price of a certain commodity and the quantity of that commodity that is demanded at that pricQ3LAW OF DEMANDIn microeconomics the law of demand states that conditional on all else being equal as the price of a good increases quantity demanded decreases conversely as the price of a good decreases quantity demanded increasesQ4ASSUMPTION of LAW OF DEMAND(i)No change in price of related commodities(ii) No change in income of the consumer(iii) No change in taste and preferences customs habit and fashion of the consumer( No expectation regarding future change in priceQ5MARKET DEMAND SCHEDULEIn economics a market demand schedule is a tabulation of the quantity of a good that all consumers in a market will purchase at a

given price At any given price the corresponding value on the demand schedule is the sum of all consumersrsquo quantities demanded at that priceQ6INDIVIDUAL DEMAND SCHEDULEIndividual demand schedule refers to a tabular statement showing various quantities of a commodity that a consumer is willing to buy at various levels of price during a given period of timeQ7 FACTORS AFFECTING INDIVIDUAL DEMAND FOR A COMMODITY

The factors that influence a consumerrsquos decision to purchase a commodity are also known as determinants of demand The following factors affect the individual demand for a commodity1 price of the commodity2 price of related goods3 income of buyer of the commodity4 tastes and preferences of the buyer1 Price of the CommodityYou must have observed that when price of a commodity falls you tend to buy more of it and when its price rises you tend to buy less of it when all other factors remain constant (lsquoother things remaining the samersquo) In other words other things remaining the same there is an inverse relationship between the price of a commodity and its quantity demanded by its buyers This statement is in accordance with law of demand which you will study in the later part of this lesson Price of a commodity and its quantity demanded by its buyers are inversely related only when lsquoother things remain the samersquo So lsquoother things remaining the samersquo is an assumption when we study the effect of changes in the price of a commodity on its quantity demanded2 Price of Related goodsA consumer may demand a particular good But while buying that good heshe also asks the price of its related goods Related goods can be of two types-(i) Substitute goods(ii) Complementary goods While purchasing a good prices of its substitutes and complements do affect its quantity purchased(i) Price of Substitute Goods Substitute goods are those goods which can easily be used in place of one another for satisfaction of a particular want like tea and coffee An increase in price of substitute good leads to an increase in demand for the given commodity and a decrease in price of substitute good leads to a decrease in demand for the given commodity It means demand for a given commodity is directly affected by change in price of substitute goods For example if price of coffee increases the demand for tea will rise as tea will become relatively cheaper in comparison to coffee(ii) Price of Complementary goods Complementary goods are those goods which are used together to satisfy a particular want like car and petrol An increase in the price of complementary goods leads to a decrease in demand for the given commodity and a decrease in the price of complementary goods leads to an increase in demand for the given commodity For example if price of petrol falls then the demand for cars will increase as it will be relatively cheaper to use both the goods together So demand for a given commodity is inversely affected by change in price of complementary goods3 Income of the Buyer of CommodityDemand for a commodity is also affected by income of its buyer However the effect of change in income on demand depends on the nature of the commodity under consideration In case of some goods like full cream milk fine quality of rice (Basmati rice) etc demand for these commodities increases when income of the buyer increases and

demand for these commodities decreases when income of the buyer decreases Such goods whose demand increases with the increase in income of the buyer are called normal goods But there are some goods like coarse rice toned milk etc whose demand decreases when income of buyer increases and their demand increases when income of the buyer decreases Such goods whose demand decreases with the increase in income of the buyer are called inferior goods Suppose a consumer buys 10 Kgs of rice whose price is ` 25 per Kg He cannot afford to buy better quality of rice because the price of such rice is ` 50 per Kg The consumer is spending ` 250 per month on the purchase of rice Now if income of the consumer increases and he can afford ` 350 on purchase of 10 Kg of rice Now he can afford to buy some quantity of rice say 6 Kgs whose price is ` 25 per Kg and may buy 4 Kgs of rice whose price is ` 50 per Kg Thus he will buy 10 Kgs of rice by spending ` 350 per month Therefore we may conclude that demand for normal goods is directly related to the income of the buyer but demand for inferior goods is inversely related to the income of the buyer4 Tastes and Preferences of the BuyerThe demand for a commodity is also affected by the tastes and preferences of the buyers They include change in fashion customs habits etc Those commodities are preferred by the consumers which are in fashion So demand for those commodities rises which are in fashion On the other hand if a commodity goes out of the fashion its demand falls because no consumer will like to buy it(5) Number of Buyers in the Market(Population)Increase in population raises the market demand whereas decrease in population reduces the market demand for a commodity Not only the size of population but its composition like age (ratio of males females children and old people in population) also affects the demand for a commodity It is because of needs of children young old male and female population differs(6) Distribution of Income and WealthIf the distribution of income and wealth is more in favour of the rich demand for the commodities preferred by the rich such as comforts and luxuries is likely to be higher On the other hand if the distribution of income and wealth is more in favour of poor demand for commodities preferred by the poor such as necessities will be more(7) Season and Weather ConditionsThis is generally observed that the demand for woolens increases during winter whereas demand for ice creams and cold drinks increases during summer Similarly market demand for umbrellas rain coats increases during rainy seasonQ8 REASONS FOR OPERATION OF LAW OF DEMAND WHY DEMAND CURVE SLOPES DOWNWARDNow we will try to explain why does a consumer purchase more quantity of a commodity at a lower price and less of it at a higher price or why does the law of demand operate ie why does the demand curve slope downwards from left to right The main reasons for operation of law of demand are1 Law of Diminishing Marginal UtilityAs you have studied earlier law of diminishing marginal utility states that as we consume more and more units of a commodity the utility derived from each successive unit goes on decreasing The consumer will be ready to pay more for those units which provide him more utility and less for those which provide him less utility It implies that he will purchase more only when the price of the commodity falls2 Income Effect

When price of a commodity falls purchasing power or real income of the consumer increases which enables him to purchase more quantity of the commodity with the same money income Let us take an example Suppose you buy 4 ice creams when price of each ice cream is ` 25 If price of ice creams falls to ` 20 then with same money income you can buy 5 ice creams now3 Substitution EffectWhen price of a commodity falls it becomes comparatively cheaper as compared to its substitutes (although price of substitutes has not been changed) This will lead to rise in demand for the given commodity For example if coke and Pepsi both are sold at ` 10 each and price of coke falls Now coke has become relatively cheaper and will be substituted for Pepsi It will lead to rise in demand for coke4 Change in Number of BuyersWhen price of a commodity falls some old buyers may demand more of the commodity at the reduced price and some new buyers may also start buying this commodity who were not in a position to buy it earlier due to higher price This will lead to increase in number of buyers when price of the commodity falls As a result demand for the commodity rises when its price falls5 Diverse Uses of a CommoditySome commodities have diverse uses like milk It can be used for drinking for sweet preparation for ice cream preparation etc If price of milk rises its use may be restricted to important purpose only This will lead to reduction in demand for other less important uses When price of milk falls it can be put to other uses also leading to rise n demand for itQ9 EXCEPTIONS TO THE LAW OF DEMANDYou have studied in law of demand that a buyer is willing to buy more quantity of a commodity at a lower price and less of it at a higher price But in certain circumstances a rise in price may lead to rise in demand These circumstances are called Exceptions to the Law of Demand Some important exceptions are1 Giffen GoodsGiffen goods are special type of inferior goods in which negative income effect is stronger than negative substitution effect Giffen goods do not follow law of demand as their demand rises when their price rises Examples of Giffen goods are jowar and bajra etc2 Status Symbol GoodsSome goods are used by rich people as status symbols eg diamonds gold jewellary etc The higher the price the higher will be the demand for these goods When price of such goods falls these goods are no longer looked at as status symbol goods and tehrefore therir demand falls3 NecessitiesCommodities such as medicines salt wheat etc do not follow law of demandbecause we have to purchase them in minimum required quantity whatever their price may be4 Goods Expected to be ScarceWhen the buyers expect a scarcity of a particular good in near future they start buying more and more of that good even if their prices are rising For example during war famines etc people tend to buy more of some goods even at higher prices due to fear of their scarcity in near future

Political Science

Constitution of India-The

Preamble

The preamble-

Preamble-

The preamble is the most precious part of the constitution We the people of India having solemnly resolved to constitute India into a Sovereign Socialist Secular Democratic Republic and to secure to all its citizensA preamble is an introductory and expressionary statement in a document that explains the documents purpose and underlying philosophy When applied to the opening paragraphs of a statute it may recite historical facts pertinent to the subject of the statuteNature and purpose of the constitution-Purpose of the Constitution dictates permanent framework of the government to form a more perfect union to establish justice and ensure peace of thenationconstitution provide principles how the government can run itself following the rules and laws written in the constitution of each state keeps them balanced

Answer the following questions-

1 What is preambleA preamble is an introductory and expressionary statement in a document that explains the documents purpose and underlying philosophy2 What is the nature and

purpose of the constitutionConstitution dictatespermanent framework of the government to form a more perfect union to establish justice and ensure peace of the nation

Homework-Learn

Accounts Compatibilty mode

1MEANING OF PARTNERSHIPPartnership is a form of business organisation where two or more persons join hands to run a business They share the profits and losses according to the agreement amongst them According to the Indian Partnership Act 1932 ldquoPartnership is relation between persons who have agreed to share profits of a business carried on by all or any one of them acting for allrdquo For example one of your friends has passed class XII from National Institute of Open Schooling (NIOS) and wants to start a business Heshe approaches you to join in this venture Heshe wants you to contribute some money and participate in the business activities Both of you if join hands constitute a partnership2CHARACTERISTICS1048698 Agreement A partnership is formed by an agreement The agreement may be either oral or in writing It defines the relationship between the persons who agree to carry on business It may contain the terms of sharing profit and the capital to be invested by each partner etc The written agreement is known as partnership deed1048698 Number of persons There must be at least two persons to form a partnership

The maximum number of partners in a partnership firm can be 50 according toCompanies Act 20131048698 Business The Partnership is formed to carry on business with a purpose of earning profits The business should be lawful Thus if two or more persons agree to carry on unlawful activities it will not be termed as partnership1048698 Sharing Profits The partners agree to share profits in the agreed ratio In caseof loss all the partners have to bear it in the same agreed profit sharing ratio10486981048698Mutual Agency Every partner is an agent of the other partners Every partner can bind the firm and all other partners by hisher acts Each partner will be responsible and liable for the acts of all other partners10486981048698Unlimited liability The liability of each partner except that of a minor is unlimited Their liability extends to their personal assets also If the assets of the firm are insufficient to pay off its debts the partnersrsquo personal property can be used to satisfy the claim of the creditors of the partnership firm10486981048698Management All the partners have a right to mange the business However they may authorize one or more partners to manage the affairs of the business on their behalf10486981048698Transferability of Share No partner can transfer hisher share to any one including hisher family member without the consent of all other partners3PARTNERSHIP DEEDAgreement forms the basis of partnership The written form of the agreement is which a document of partnership is It contains terms and conditions regarding the conduct of the business It also explains relationship between the partners This document is called partnership deed Every firm can frame its own partnership deed in which the rights duties and liabilities of the partners are stated in detail It helps in settling the disputes arising among the partners during the general conduct of business 4CONTENTS OF PARTNERSHIP DEEDThe partnership deed generally contains the following (i) Name and address of the partnership firm(ii) Nature and objectives of the business(iii) Name and address of each partner(iv) Ratio in which profits is to be shared(v) Capital contribution by each partner(vi) Rate of Interest on capital if allowed(vii) Salary or any other remuneration to partners if allowed(viii) Rate of interest on loans and advances by a partner to the firm(ix) Drawings of partners and interest thereon if any(x) Method of valuation of goodwill and revaluation of assets and liabilities on the reconstitution of the partnership ie on the admission retirement or death of a partner(xi) Settlement of disputes by arbitration(xii) Settlement of accounts at the time of retirement or death of a partner5IN ABSENCE OF PARTNERSHIP DEEDThe partnership deed lays down the terms and conditions of partnership in regard to rights duties and obligations of the partners In the absence of partnership deed there may arise a controversy on certain issues like profit sharing ratio interest on

capital interest on drawings interest on loan and salary of the partners In such cases the provisions of the Indian Partnership Act becomes applicableSome of the Issues are(i) Distribution of Profit Partners are entitled to share profits equally(ii) Interest on Capital Interest on capital is not allowed(iii) Interest on Drawings No interest on drawing of the partners is to be charged(iv) Interest on Partnerrsquos Loan A Partner is allowed interest 6 per annum on the amount of loan given to the firm by himher(v) Salary and Commission to Partner A partner is not entitled to anysalary or commission or any other remuneration for managing the business

History TOPIC-TOWARDS INDEPENDENCE AND PARTITION THE LAST PHASE (1935-1947)

SUB TOPIC-IMPORTANT POLITICAL DEVELOPMENTS ndash GROWTH OF SOCIAL IDEAS

Socialism is a political social and economic philosophyLike in other parts of the world the Russian revolution of 1917 served as a great inspiration for revolutionaries in India who at that time were engaged in the struggle for liberation from British ruleSocialist ideas led to the formation of communist party of IndiaJAWAHARLAL NEHRU Among the early Congress leaders Jawaharlal Nehru was very much impressed and influenced by the Socialist ideas He also learnt about the Economic activities of the Soviet Union after the Bolshevic Revolution 1917 He made full use of them in IndiaThe election of Jawaharlal Nehru and Subhas Chandra Bose showed the Left wing tendency within CongressJawaharlal Nehru demanded economic freedom along with political freedom of the people in order to end the exploitation of masses

Nehrus working committee included three socialists leaders The Lucknow session was a landmark in the evolution of socialist ideas of the congressSUBHAS CHANDRA BOSE ndash Subhas Chandra Bose had socialist leaning Both Jawaharlal Nehru and Subhas Chandra Bose were known as leftist Congress men Later on National Congress divided into Leftist and rightist campCONGRESS SOCIALIST Within the Congress some leaders formed the Congress Socialist partyPattavi Sitaramyya Sardar Patel Rajendra Prasad had hostile attitude towards the Congress Socialist partyJawaharlals attitude was hesitant

1 QUESTION ndash Mention name of two Congress leaders who had socialist leaning

1ANSWER ndash Subhas Chandra Bose and Jawaharlal Nehru2QUESTION- In which session of the congress Jawaharlal elaborated his Socialist ideas2 ANSWER ndash Lucknow and Faizpur Session in December 1935 and 19363QUESTION ndash Why Congress was sharply divided into leftist and rightist camp 3ANSWER ndash Subhas Chandra Bosersquos attempt to seek re election for congress presidentship in 1939sharply divided the National Congress into Leftist and Rightist camp4 QUESTION ndash Who was MN Roy 4 ANSWER ndash Manabendra Roy first formed the Communist Party of India outside the country at Tashkent in 19205QUESTION ndash Who formed the Congress Socialist Party within the Congress5 ANSWER ndash Jaya Prakash Narayan Achyut Patwardhan Acharya Narendra Dev Ram Mohan Lohia Aruna Asaf Ali6QUESTION ndash When was the Congress Socialist Party formed What was its object6 ANSWER ndash 1934The Congress Socialist Party sought to work out socialist programme through the Congress They joined hands with the Congress and wanted to carry

Subhas Chandra Bose being expelled from the congress after the Tripuri rift he formed Forward BlockThere were basic differences between the Congress Socialists and the communistsTRADE UNION ACTIVITIES Maximum working class people lived in Bombay and Calcutta The working and living conditions of those workers were very miserable In this situation Shasipada Banerjee NM Lokhande protested against the oppression of the working class peopleThe first Trade Union Madras Labour Union was formed in 1918 by BP WadiaIndustrial strikes took place in Kanpur Calcutta Madras Jamshedpur and Ahmedabad AITUC was formed in Bombay in 1927 The growth of Trade union among the workers was slow because of the fear of the dismissal of the jobIn the mean time the Moderates as well as Communists left AITUC and formed separate organization

on National struggle with the help of workers and peasant class of the society7 QUESTION ndash What was the name of the party founded by Subhas Chandra Bose7 ANSWER- Forward Block8QUESTION ndash Who was Shasipada Banerjee8 ANSWER ndash Shasipada Banerjee was a radical Brahmo He founded a working menrsquos club to protest against exploitation of the British rulers towards the working class of India9 QUESTION ndash What was the weekly published by NM Lokhande9ANSWER- Dinabandhu10 QUESTION ndash Who founded Bombay Mill-Hands Association and in which year10 ANSWER- NM Lokhande in189011 QUESTION- Who was BP WadiaANSWER- BPWadia was the founder of Madras Labour Union in191812 QUESTION- What was the name of the first labour union of India12 ANSWER- Madras Labour Union13 QUESTION Who founded the Majur Mahajan 13 ANSWER GANDHIJI14 QUESTION What was the full form of AITUC When it was formed14 ANSWER All India Trade Union Congressin 192715QUESTION Who formed the Red Trade Union Congress and in which year15ANSWER The Communists formed the Red Trade Union Congress16 QUESTION What do you mean by Socialism16 ANSWER Socialism describes any political and economic theory that says the community rather than individuals should own and manage property and natural resources

Subject Eng Literature (The Tempest ndash William Shakespeare) Topic Act III Scene 3 Lines 1 to 52 (Line 52 ndash Brother my lord the Duke Stand to and do as we) Date 13th April 2020 (4th Period)

[Students should read the original play and also the paraphrase given in the school prescribed textbook]Summary Questions amp Answers

o Alonso Sebastian Antonio Gonzalo Adrian Francisco and others wandered about the island in search of Ferdinand and gets tired and hungry of the toil and at the same time gives up all hope of finding him

o Antonio and Sebastian are happy that Alonso is out of hope and decide to make another attempt on his life that night when being so tired they will be sleeping soundly

o Suddenly a solemn and strange music is heard in the air and several strange shapes enter bringing in a banquet These strange shapes then dance round it with gestures of salutation and then inviting the King to eat they depart

o Seeing this strange scene all are inclined to believe the tales told by travelers that there truly are ldquounicornsrdquo and ldquothe phoenixrsquo thronerdquo

1 ALONSO What harmony is this My good friends hark (L18-27)

GONZALO Marvellous sweet music

[Enter several strange shapes bringing in a banquet

they dance about it with gentle actions of salutation

and inviting the King and his companions to eat they depart]ALONSO Give us kind keepers heavens What were theseSEBASTIAN A living drollery Now I will believe

That there are unicorns that in Arabia

There is one tree the phoenixrsquo throne one phoenix

At this hour reigning thereANTONIO Ill believe both

And what does else want credit come to me

And Ill be sworn rsquotis true Travellers neer did lie

Though fools at home condemn rsquoem

(a) How did Prospero present an amazing spectacle before Alonso and his companions

Using his magic powers Prospero ordered strange shapes to lay a banquet before Alonso and his companions The shapes brought several dishes with tasty eatables in them They placed the dishes on a table before Alonso and his companions Then the strange shapes began to dance gracefully around the banquet While dancing they made gestures inviting them to eat the food Then suddenly the shapes disappeared(b) Who were the guests at the strange banquet Describe the lsquoliving drolleryrsquo

Alonso Sebastian Antonio Gonzalo Adrian and Francisco were the guests at the strange banquet

The term ldquoliving drolleryrdquo refers to live entertainment show In this context when Alonso the King of Naples Sebastian his brother Antonio the treacherous brother of Prospero Gonzalo the kind and loyal councillor to the King Adrian and Francisco came to the island they were hungry and weary in their spirits They heard a solemn and strange music They were shocked to see several strange shapes bringing in a banquet and these shapes danced about it with gentle action of salutation inviting the King and his companions to eat After this Sebastian described this show as lsquoliving drolleryrsquo(c) What is lsquophoenixrsquo What are lsquoUnicornsrdquo

The term lsquophoenixrsquo refers to a mythical Arabian bird which lived alone and perched on a solitary tree After one hundred years it expired in flames and rose again from its own ashes

lsquoUnicornsrsquo refers to the mythological four-footed beasts having horns in the centre of their foreheads When the horns are ground into powder the powder was believed to be

an aphrodisiac(d) How does Sebastian explain the puppet show OR Why does the speaker now believe in unicorns and phoenix

Sebastian finds several strange shapes bringing in the banquet They invite the king and his party for dinner and soon depart He tells that if such a strange sight can be a reality there is nothing incredible in the world and from the present moment he will believe anything He says that it is a strange dumb show enacted not by puppets but by living beings It is stranger than a travellerrsquos tale Seeing such a thing

before his own eyes he will no longer disbelieve the story about unicorns and phoenix(e) How do the other characters present respond to this living drollery

At the sight of the lsquoliving drolleryrsquo like Sebastian Gonzalo and Antonio too acted strangely Antonio told that he too now believes in unicorns and phoenix and anything else that seems to be incredible He too now believes in travellersrsquo tales Gonzalo told that if he would report those happenings in Naples nobody will believe him He considers that those gentle shapes were gentler in manner in comparison to the living beings Alonso was at first sight suspicious and told them that those strange shapes conveyed their meaning in expressive gestures when they seemed to lack speech by their movements and sounds Francisco was amazed at their mysterious disappearance

2 ALONSO Not I

(Line 43-52)GONZALO Faith sir you need not fear When we

were boysWho would believe that there were mountaineers

Dewlapped like bulls whose throats had hanging at rsquoem

Wallets of flesh Or that there were such men

Whose heads stood in their breasts Which now we find

Each putter-out of five for one will bring us

Good warrant ofALONSO I will stand to and feed

Although my lastmdashno matter since I feel

The best is past Brother my lord the Duke

Stand to and do as we

(a) How does Alonso respond at the spectacle of the shapes which were sent to them at the instruction of Prospero

After seeing the strange sight of appearing and disappearing of the shapes sent by Prospero to arrange a banquet for them Alonso says that his surprise at having seen those creatures is infinite and he is fully justified in feeling so much surprise He thinks that their shapes their gestures and the sounds they made were indeed amazing Although they do not possess the gift of speech yet they were able to convey their

thoughts by means of their gestures only

(b) What does Prospero say about the views expressed by Alonso regarding the shapes What does Francisco think about the shapesAfter hearing Alonsorsquos views about the shapes Prospero says that this manrsquos praise of the spirits is rather hasty He means to say that Alonso has shown great haste in reaching the conclusion about the shapes Francisco is amazed to see that those shapes disappeared in a mysterious way(c) What does Sebastian ask Alonso to doSebastian tells Alonso that the shapes having disappeared should not matter to them because they have left the eatables behind He asks Alonso to enjoy eating as they are extremely hungry but the king does not accept his offer of enjoying the dishes(d) How does Gonzalo try to dispel Alonsorsquos fear of those strange shapes What kind of references does he give to AlonsoGonzalo says that those who have travelled abroad have reported seeing even stranger sights than these shapes that Alonso and his companions have beheld Hence there is no reason to feel afraid of these shapes Gonzalo further adds that in his younger days he had heard strange stories from travelers and Alonso might have heard similar stories For instance it was said that there existed a certain race of

human beings who had huge lumps of flesh hanging at their throats and who therefore resembled bulls Then Gonzalo tells about a race of human beings whose heads were located at their breasts Gonzalo says that such stories were not believed by most people in those days but now-a-days these stories have become common(e) Explain the following lsquoEach putter-out of five for onersquoEnglish travellers often insured their trips with London brokers Those that went on foreign travels those days used to deposit a certain amount with some firm or company in London before their departure If the travelers failed to return the money was forfeited by the company with which it had been deposited But this money was repaid five-fold if the travelers returned safe and sound In this way a traveler stood a great chance of recovering the entire cost of his

travels(f) Give the explanatory meanings of the following expressions in the context of the above extract (i) Dewlapped (ii) Wallets of flesh

(iii) Putter-out(i) Dewlapped having big lumps of flesh at the necks(ii) Wallets of flesh large masses of flesh looking like bags(iii) Putter-out to invest money before commencing the travel

  • General methods of preparation of hydrogen
  • Chapter Dimensional Analysis (Summary)
    • Properties of Charge
Page 25:   · Web viewSubject. Topic. Summary. Execution. Hindi. व्याकरण. शरीरके अंगो के नाम लिखिए. 1) आँख 2) नाक 3

কলিবতো তোর অ13সক ভোষোয় পরকোো কসর ঘলিরসয় ব যকত কসরস4 লেপরসমর ফতো আর লিবফতো লেক গোঢ় কসর লেদোসো কলিব ভোষ মসোপো3 যোসয়র অলিভবসর অ যলিদক

Economics

Factors of Production

Welcome to the new sessionToday we are going to start the first chapter of Class XThe name of the chapter is Factors of productionBy the name I hope you all can recall a glimpse of what you have learnt in the second chapter of Class IX

NowProduction is the process of creating the various goods and services which are consumed by the people of the country to satisfy their wants

Thus it is the process in which some materials are transformed from one form to another to create utility and value in goods

For example utility can be created by changing the form of a commodity ie

Making of table out of wood by a carpenter for his customer here the wood is getting transformed into table creating utility for his customer and he can also command a price for it

On the other hand Housewives perform very

useful activities at home which create utility but their domestic activities are not included in production because they have no money value

So we can also say that Production denotes two things firstly creation of utility and secondly creation of value

Production is not complete unless it reaches the consumer

An increase in production will increase the economic welfare of the consumers and hence the aim is to raise the production level of the country

Again production of a good or service is only possible if certain resources or

Questions

1 What do you mean by production

Answer Production means the creation of goods and services for the purpose of selling in the market

In fact production involves the transformation of inputs into outputs

Hence production denotes two thingsCreation of utility and creation of valueUtility and value can be created by changing the form by changing the place by changing the time and by rendering services

Example Transformation of raw

materials into finish goods such as potter creates utility by converting mud into utensils assembling of small parts to make bigger machinery

Production also includes services such as distribution and marketing

2 What are the factors of production

Answer Factors of Production refers to the resources and inputs needed for producing goods and servicesThese inputs can be classified as

Land Labour

Capital Enterprise

Land Land is defined to include not only the surface of the earth but also all other free gifts of nature(for example mineral resources forest resources and indeed anything that helps us to carry out the production of goods and services but is provided by

inputs are used together in right proportion

A resource or an input which helps in the process of production to obtain an output is called FACTOR OF PRODUCTION

These factors of production can broadly be categorized into four parts 1LAND 2LABOUR3CAPITAL4ENTERPRISE (ORGANISATION)or Entrepreneur

The above factors are all interdependent on each other and they play a major role in production process

FACTORS OF PRODUCTION

LANDCAPITAL

LABOUR ENTREPRENEUR

nature free of cost)LabourLabour refers to the human efforts that need to be combined with other factors of production for creating an output

CapitalAll man ndash made means of production is called capita example machineries which help in further production Money when used for starting any business for purchasing raw materials machinery tools etc it is regarded as capitalCapital also includes physical capital like factories machineriestoolsbuildingsequipments etcEnterpriseThe task of bearing risks is called enterprise and the person who bears these risks of business is called the entrepreneurThus an entrepreneur is one who organises production takes important decisions regarding production hires and purchases factors of production and bears the risk and uncertainty involved in productionOrganisation refers to the services of an entrepreneur who controls organises and undertakes all risks One who plans organises and manages a business enterprise is an organiser

Physics Chapter 1 Force

Force is an external agent capable of changing the state of rest or motion of a particular body It has a magnitude and a direction The direction towards which the force is applied is known as the direction of the force and the application of force is the point where force is applied The Force can be measured using a spring balance The SI unit of force is Newton (N)

Question 1

State the condition when on applying a force the body has

(a) the translational motion

(b) The rotational motion

Solutions

(a) Translational motion is produced when the body is free to move

(b) Rotational motion is produced when the body is pivoted at a point

Question 2

Define moment of force and state its SI unit

Solutions

The moment of force is equal to the product of the magnitude of the force and the perpendicular distance of the line of action of force from the axis

of rotation

The SI unit of moment of force is Newton times meter

= Newton meter (Nm)

Commercial Studies

Stake holders In this topic you will be come to know about the meaning and concept of stakeholders

How stakeholders are different from shareholders

Questions1 What do you mean by the term stake holdersAnswer) The term stake holders have developed from the words which mean an interest or expected benefit Stakeholders mean all those individuals groups and Institutions which have a state (interest) in the functioning and performance of a commercial organisation or a business enterprise2 What do you mean by share holdersAnswer) The person and Groups who own the shares of the joint stock company by providing capital to the company are called shareholders Shareholders are the internal stakeholders shareholders are one out of several stake holders3 How are shareholders different from stakeholdersAnswer)i) The term shareholders is related to only joint stock company whereas stakeholders are related with all business organisationsii) Stakeholders maybe any individual having financial stake in business organisation whereas a shareholders are those individuals who are holding shares in the company4) How are shareholders different from creditorsAnswer) i) Shareholders are internal stakeholders while creditors are external stakeholdersii) Shareholders invest in the capital of the company whereas creditors give loan to the companyiii) Shareholders are the members of the company with voting rights but creditors are not the members of the company

English 1 Transformation of sentences

Sentences A sentence is a group of words which makes complete sense

e Assertive sentencesf Imperative sentencesg Interrogative sentencesh Exclamatory sentences

Sentences can be changed from one grammatical form to another without changing the meaning of the sentence This is known as transformation of sentences

Exercise 1 Change the following affirmative sentences into Negative sentences

a He is a good manHe is not a bad man

b Ram loves SitaRam is not without love for Sita

c Only he stood first in the classNone but he stood first in the class

d Ankit was wiser than he

He was not so wise as Ankite He did it

He did not fail to do itf As soon as I reached college the

bell rangNo sooner did I reach college than the bell rang

g He finished everythingHe left nothing unfinished

h It always pours when it rainsIt never rains but it pours

Math Topic Commercial MathematicsChapter ndash Goods and services Tax

What is GSTAns It is a abbreviated term of Goods and Service Text which is an indirect tax levied on the sale of goods and rendering servicesSome terms related to GSTDelar Any person who buys goods or services For resale is known as a delar A delar Can be a firm or a companyIntra-state sales Sales of goods and services within the same state or same union territory are called intra- state salesInter-state sales Sales of goods and services outside the state or union territory are called Inter-state sales4) Input GST GST is paid by dealers on purchase of goods and services are called input GST5) Output GST GST is collected from customers on sale of goods and services are called output GST6) Types of GST There are three taxes applicable under GST(i) Central Goods and Services Tax (CGST)(ii) State Goods and Services Tax (SGST) or Union Territory Goods and Services Tax (UTGST) Both these taxes are levied on intra-state sales Here GST is divided equally among central and state governments(iii) Integrated Goods and Services Tax (IGST) IGST is levied on inter- state sales It is also levied on import of goods and services into India and export of goods and services from India

Subject Eng Literature (The Merchant of Venice ndash William Shakespeare)Topic Act III Scene 4 Lines 1 to 44 (Portia hellip To wish it back on you fare you well Jessica)[Students should read the original play and also the paraphrase given in the school prescribed textbook]

Summary Questions amp AnswersIn this scene we suddenly find a new element in the character of Portia We have already seen her possessed of every graceful womanly quality but now she shows that she is capable of rapid decision and determined action She shows this by her sudden resolve to hasten to Venice with a daring scheme for the rescue of Antonio This is an important scene in the dramatic action for it leads up to and renders possible the striking events of the famous trial scene which is one of the greatest striking elements of the play Moreover the fact that all the characters of importance are now assembled together in Venice makes the union of the main plot and the secondary story complete

(1) LORENZO Madam although I speak it in your presence(Line 1-9)

You have a noble and a true conceit

Of god-like amity which appears most strongly

In bearing thus the absence of your lordBut if you knew to whom you show this honourHow true a gentleman you send reliefHow dear a lover of my lord your husbandI know you would be prouder of the workThan customary bounty can enforce you

(a) Where is Lorenzo Why is he here To whom is he referring as lsquoMadamrsquo

Lorenzo is at Portiarsquos residence He had met Salerio on the way and Salerio had begged him to come along with him to

o In this scene Portia Nerissa Lorenzo Jessica and Balthazar appear

o Portia requests Lorenzo and Jessica to be in charge of her house during her absence from Belmont because she and Nerissa have decided to spend the days in meditation and also in visiting the holy places in the neighbourhood of Belmont She has already instructed her people to acknowledge both Lorenzo and Jessica as master and mistress of house during her absence Lorenzo and Jessica gladly agree to look after the house of Portia

handover the letter from Antonio to Bassanio The letter carried the bad news about Antoniorsquos arrest for non-payment of loan taken from Shylock Hence Salerio might have preferred company to break this bad news to Bassanio He is referring to Portia as Madam(b) What does Portia say on hearing the above extract

Portia says that she has never regretted doing good to others Friends who spend a lot of time together and really are there for each other have many traits in common As Antonio is Bassaniorsquos best friend saving him is like saving Bassanio who is like her own soul She asks Lorenzo to take care of management of the house till Bassanio is back(c) What does Portia send with Bassanio and why

On hearing about Antoniorsquos troubles on account of Bassanio her husband Portia immediately sends him with enough gold to repay the debt many times over to Venice to help Antonio out of his misfortune

(2) Lorenzo Madam with all my heart (Line 36-40)

I shall obey you in all fair commands

Portia My people do already know my mindAnd will acknowledge you and JessicaIn place of Lord Bassanio and myselfSo fare you well till we shall meet again

(a) Where are Lorenzo and Portia at this time What lsquofair commandsrsquo are given to Lorenzo

Lorenzo and Portia are at Belmont during this scenePortia reveals to Lorenzo that she has sworn to contemplate in prayer at a monastery around two miles away until her husband returns from Venice She tells him that Nerissa would accompany her and asks him to manage the house with Jessica till things are settled In response Lorenzo tells her that he would be obliged to do whatever she asks him to do(b) Where is Portia actually going and why

Portia tells Lorenzo that she would live a life of contemplation and pray at a monastery which is two miles away from her place In reality Portia plans to go to Venice in disguise with Nerissa and argue the case in defense of Antonio She is very sure that her plan would succeed

ClassXI (ScienceHumanitiesCommerce)Subject Topic Summary Execution

Computer Science

(APC)

Ch ndash 1 Numbers

(Numbers in different bases and

their Arithmatical operations)

Number System In computers Number System is defined as a writing system to represent the numbers in different ways ie we are using different symbols and notations to represent numbers There are four ways we can represent the number ndash Binary Decimal Octal and Hexadecimal

Decimal Number SystemThis number system consist 10 digits These are 0 1 2 3 4 5 6 7 8 amp 9

Binary Number SystemThis number system has only two digits these are 0 and 1 Here 0 stands for off while 1 stands for on

Octal Number SystemThis number system has 8 digits these are 0 1 2 3 4 5 6 amp 7

Hexadecimal Number SystemThis number system has 16 digits these are 0 1 2 3 4 5 6 7 8 9 A B C D E F Here the value of the alphabets are as follows A=10 B=11 C=12 D=13 E=14 F=15

Rules for conversion decimal number to Binary1 Divide the decimal number by 22 If the number will not divide equally by 2 then round down the answer to the nearest whole number (integer)3 Keep a note of the remainder it should be either 0 or 14 Keep repeating the above steps dividing each answer by 2 until you reach zero5 Write out all the remainders from bottom to top This is your binary solution

For example Lets convert 32 to binary 2 32 2 16 - 0 2 8 - 0 2 4 - 0 2 2 - 0 2 1 - 0 0 - 1

The binary equivalent of 3210 is 1000002

Try the follwing youself1 2410

2 4810

3 1210

History GROWTH OF NATIONALISM

The second half of the 19th century witnessed growth of political consciousness and a sense of Nationalism among the IndiansThere were various factors for growth of Indian Nationalism- As a result various political associations were formed in different provinces by the educated Indians Surendranath Banerjee organized a meeting of National conference at Calcutta Ultimately the National Congress was founded in Bombay in 1885This body became the vanguard of Indian struggle for freedom The congress leaders were known as moderates because they followed a policy of prayer and petition A large number of Indian leaders had experienced in political agitation The Political situation of England was also changed Moreover increasing revolutionary activities in Maharashtra Punjab and Bengal became serious concern to the British Government In this

QUESTION1 What do you mean by Nationalism ANSWER 1 Nationalism is defined as loyalty and devotion to own nation especially a sense of national consciousnessQUESTION 2 What are the causes of nationalism ANSWER 2 There were various factors for growth of nationalism

1 Spread of western education2 The progress of vernacular press and

patriotic literature3 The economic exploitation of our

country by the colonial rulers4 International affairs

QUESTION 3 Who organized National conference in Calcutta in 1883 ANSWER 3 Surendranath BanerjeeQUESTION 4 When did Indian National Congress formANSWER 4 Indian National Congress was formed in 1885 in BombayQUESTION 5 Who were ModeratesANSWER 5 The Early Nationalists were also known as Moderates Their emergence marked

background Lord Curzon became Viceroy in India He had no respect for the Indian National Congress

the beginning of the organized national movement in India They believed in British justice and were loyal to them They followed a policy of prayer and petition They demanded constitutional reforms of our country Impotant Moderate leaders were Pherozshah Mehta Dadabhai Naorozi and Surendranath Banerjee etcQUESTION 6 What do you know about Extremism in Indian National movementANSWER 6 In the beginning of 20th century a new class of national leaders emerged in India which was different from the moderate groups They started more aggressive movement against the British empire The goal of extremists was ldquoswarajrdquo Important extremist leaders were Bal Gangadhar Tilak Lala Lajpat Rai Bipin Chandra Pal etcQUESTION 7 Mention the places which were the main centres of Revolutionary movementANSWER 7 Maharashtra Bengal and Punjab

Physics

Chapter Dimensional Analysis

(Summary)

The dimensions of a physical quantity are the powers to which the fundamental units are raised in order to obtain the derived unit of that quantit

The physical quantites lengthmasstime are represented by [L] [M] [T] resp let they are raised to powers ( dimesions) abc resp then any physical quantity can be represented by [ La Mb Tc ] Examples

1 Area area = L x B = [L] x [L] = [M0 L2 T0 ]

2 Density density = massvolume = [M][L3] = [ M L-3]

3 Velocity velocity = distancetime = [L][T] = [LT-1]HW Try to find out dimension of acceleration Acceleration = velocity timeNB One can find the SI Units Using Dimension Analysis Such as for area we have [L2] so its SI unit is m2

Biology Topic ndash Chp-1 The living world

Today we will start the first chapter the living world Here we discuss about the characteristics of living organism and what are the difference between them and nonliving substances We also discuss about the contribution of different Scientists

There are over 500000 species of plants andover a million species of animal are present on earth Some 15000 new species were discovered every yearQ1 What is a living organismbull A living organism is primarily physico -chemical material that demonstrate a high degree of complexity is capable of selfRegulation possesses a metabolism and perpetuates itself through timeQ2 What are the differences between livingand non-livingsi) Compared with non-living living organisms

have more complex organised structure and their use of energy is more controlled amp efficientii) Living things reproduce their own kind by forming new cells which contains copies of their genesiii) Each organism has some degree of homeostasisie it is able to make adjustments so that internal environment remains constantQ3 Write contributions of following Scientists i) Aristotle - One of the first theories in Biology places all living things in a hiearchieii) AV Leeuwenhoek - was the first to observe living single celled organisms under microscopeii) Carolus Linnaeus - developed the binary system for naming of organisms and classificationiii) Geregor Johann Mendel ndash discoverbasic principles of inheritanceHomework i) C Darwin ii)Schleiden

Math Trigonometric functions

1 Overviewi) Trigonometry The word lsquotrigonometryrsquo is derived from the Greek words lsquotrigonrsquo and lsquometronrsquo which means measuring the sides of a triangle An angle is the amount of rotation of a revolving line with respect to a fixed line Usually we follow two types of conventions for measuring angles ie a) Sexagesimal system b) Circular system In Sexagesimal system the unit of measurement is Degree In Circular system the unit of measurement is Radian ii) Relation between degree and radianThe ratio of circumference of a circle to its diameter is always a constant This constant ratio is a number denoted by π which is taken approximately as 227The relationship between degree amp radian measurements is as follows2 right angles = 180deg= π radians1radian = 180degπ=57deg16(approx) 1deg=π180 radianiii) Length of an arc of a circleIf an arc of length s subtends an angle θ radians at the center of a circle of radius r then s=rθiv) Area of a sector of a circleA sector is like a pizza slice of the

Q) Express the following angles in radiana) 45deg b) 40deg3730Ans a) We have 180deg=π radiansi e 45deg= πtimes45180 radian = π4 radiansb) 40deg3730= 40deg37+3060 minute= 40deg 37 +12 minute= 40deg+ 752 minute=40 + 75(2times60) degree=3258 degreeNow 180deg=π radianie 3258 degree= (πtimes325) (180times8) radians = 65π288 radiansQ) A circle has a radius of r=12 meters What is the length of an arc traced out by a 60deg angle in the center of the circleAns In this problem we know both the central angle (60deg) and the radius of the circle (12) All we have to do is plug those values into our equation and we get

s = 2π(12)(60360)s = 24π6s = 4πSo the length of an arc traced out by a 60deg angle in a circle with a radius of 12 meters equals 4π meters asymp 1257 metersQ) Find the area of the sector with a central angle 30deg and a radius of 9cmAns GivenRadius r = 9 cmAngle θ = 30degArea of the sector = θ360degtimesπr2

= 30360degtimes227times92=2121cm2

circle It consists of a region bounded by two radii and an arc lying between the radiiThe area of a sector is a fraction of the area of the circle This area is proportional to the central angle In other words the bigger the central angle the larger is the area of the sectorArea of Sector = θ2 times r2 (when θ is in radians)

Area of Sector = θ times π360 times r2 (when θ is in degrees)

COMMERCE

CLASSIFICTION OF HUMAN ACTIVITIES-ECONOMIC AND NON-ECONOMIC

Welcome to the new sessiontoday we are going to start the first chapter of Class XI The name of the chapter that we are going to start is

lsquoClassification of Human Activities ndasheconomic and non-economicrsquo

Now let us start the chapter by considering human beings and the activities they perform throughout the day

Human activities means all those activities that human beings undertake to satisfy their wants

Human wants on the other hand are the desire of human beings for goods (vegetables fruits rice etc) and services (services of doctors teachers lawyers etc) that they require to live

Now these human activities continue throughout life as human wants are unending unlimited and recurring as human beings desire for better living throughout their lives

Now human activities can be classified into two categories

Human activities

Economic activities Non-economic activities

Economic activities are

Questions1 What are human activities

Answer Human activities mean all those activities that human beings undertake to satisfy their wants

Example A man working in an office

A boy playing in the garden

2What are the characteristics of human activitiesAnswer the characteristics of human activities are as follows

Human activities are undertaken by men women and children and these activities involve human efforts

Human activities are undertaken to satisfy human wants which are unlimited

Human activities continue throughout life

Human activities are performed for both earning money and personal satisfaction

3What is economic activitiesGive example

Answer Economic activities are undertaken by human beings with the object of earning money acquiring wealth and thereby satisfying human wantsExample

Selling of goods by a shop keeper to his customer

A clinic run by a doctor Service of a teacher in school or college

undertaken by human beings with the object of earning money and acquiring wealth

These activities result in the production of economic goods and services

Example Human activities(ie working in factories officesshops) which produce direct economic benefits

Non-economic activities are inspired by human sentiments and emotions such as love for the family desire to help the poor and love for the country

Thus these human activities (eg praying playing sleeping) produce no direct economic benefits and they are also not related to earning money and acquiring wealth

4 What are the characteristics of economic activities

Answer The characteristics of economic activities are as follows

Economic motiveEconomic activities are undertaken to earn money and acquire wealth

ProductiveEconomic activities involve productiondistribution and exchange of goods and services to create wealth

Economic growthEconomic activities determine the level of economic development of a country and standard of living of its citizens

Socially desirableEconomic activities are socially desirable for society

Economic resourcesEconomic activities make use of all the economic resources such landlabourcapital etc

5 What do you mean by non-economic activitiesExampleAnswerNon-economic activities are inspired by human sentiments and emotions such as love for the family desire to help the poor and love for the countryThese activities are not undertaken for monetary gain but for onersquos satisfaction and happinessExample

a mother looks after her children

a student donates blood8 Differentiate between Economic activities and Non-economic activities

Economic activities

Non-economic activities

1to earn living and acquiring wealth2Result can be measured in terms of money

3ExampleBusinessprofession and employment

1 to obtain some satisfaction

2Result cannot be measured in terms of money

3ExampleFamily-orientedreligious socialCultural and national

BUSINESS STUDIES

BUSINESS ENVIRONMENT

Welcome to the new sessionToday we are going to start the first chapter and the name of the chapter is Business Environment

In todayrsquos world every business enterprise is a part of the society It exists and operates in association with various groups in society such as customers suppliers competitors banks and financial institutions government agencies trade unions media and so on All these groups influence the functioning of business in one way or the other They constitute the environment of businessConcept of Business Environment

The term lsquobusiness environmentrsquo refers to the sum total of all individuals institutions and other forces that lie outside a business enterprise but that may influence its functioning and performance

The main features of business environment

Totality of External forces General and Specific forces Interrelatedness Complexity Dynamic Uncertainty Relativity

The Interrelation between business and its environment

The business enterprise is an open system It continuously interacts with its environment It takes inputs

Prepare the following questions from todayrsquos assignment

1 What do you mean by business environment

The term lsquobusiness environmentrsquo means the aggregate of all forces factors and institutions which are external to and beyond the control of an individual business enterprise but they may influence its functioning and performance Business environment is the macro framework within which a business firm a micro unit operates It consists of several interrelated and interacting elements

2 Explain the main features of business environment in brief

Totality of External forces-Business environment is the sum total of all things external to a business environment

General and Specific forces-It includes both the forces general forces are the economic social political legal and technological conditions which indirectly influence all business enterprise Specific forces are the investors customers competitors and suppliers which influence individual enterprise directly

Interrelatedness-Different elements of environment are interrelated for an example growing awareness for health care has increased the demand for health foods

Complexity- Business environment id

(such as raw materials capital labour energy and so on) from its environment transforms them into goods and services and sends them back to the environment

Fig 1 Business Environment Relationship

complex in nature as the elements keep on changing example economic technological and other forces changes in demand for a product and service

Dynamic-Business environment is not static it keeps on changing

Uncertainty- Itrsquos very difficult to predict future events such as technology and fashion which occur fast and frequently

Economics Basic Economic ConceptsSub topic

Microeconomics and

Macroeconomics

Welcome to the new sessiontoday we are going to start the first chapter of Class XI The name of the chapter that we are going to start is Basic Economic concepts

Now Economics covers the study of human activities Human activities are those activities which are performed by humans to satisfy their wants

Thus Human wants are unlimited and therefore economic activities such as production exchange and consumption are needed in order to satisfy those wants

The study of economics is divided largely in two parts which areMicroeconomics and Macroeconomics

SUBJECT- MATTER OF ECONOMICS

MICROECONOMICS MACROECONOMICS

Questions1Who has coined the words micro and macro economics

Answer Ranger Frisch coined the words lsquomicrorsquo and lsquomacrorsquo in 1933 to denote the two branches of economic theory namely microeconomics and macroeconomics

2What is microeconomicsAnswer It is the study of behaviour of individual decision ndash making unit such as consumers firms etc

3 What is macroeconomicsAnswer Macroeonomics is the study of overall economic phenomena like employment national income etc

4 What is the importance of microeconomicsAnswer

Microeconomics helps in formulating economic policies which enhance productive efficiency and results in greater social welfare

It helps the government in formulating correct price policies

It explains the working of a capitalistic economy where individual units(producers and consumers ) are free to take their own decision

Micro means a small part in

microeconomics we do not study the whole economy Hence we study an individual consumer and his or her choices and a producer and his or her profit maximizing decisions in the market Thus it does not mirror what happens in the economy as a whole

Macroeconomics on the other hand studies the economy as a whole It is concerned with aggregate and depicts the entire picture of the economyMacroeconomics deals with the national income aggregate investment aggregate consumption etc

Features of Microeconomics It deals with small

parts of the country Hence it looks at

individual consumers firms and industries

It deals with individual income consumption and savings

It studies the determination of price of any product or factors of production

It deals with the working of market via the price mechanism which is nothing but the determination of price and quantity of a commodity by the forces of demand and supply

Features of Macroeconomics

It deals with the study of the economy as a whole

It is concerned with

5 Give a limitation of microeconomics Microeconomics fails to explain the

functioning of an economy as a whole It cannot explain unemployment illiteracy and other problems prevailing in the country

6 What is the importance of macroeconomics It gives overall view of the growing

complexities of an economic system It provides the basic and logical

framework for formulating appropriate macroeconomic policies (eg for inflation poverty etc )to direct and regulate economy towards desirable goals

7What is the limitation of macroeconomics It ignores structural changes in an

individual unit of the aggregate

8 Differentiate between Microeconomics and Macroeconomics

Microeconomics Macroeconomics

the study of aggregates

National income aggregate savings and aggregate investments are major concepts dealt within macroeconomics style

It studies the determination of general price levels

It investigates into the problem of unemployment and the achievement of employment

It studies the aspect of decision making at the aggregate and national levels

It includes all growth theories whether related to developed or developing economies it also includes the study of economic systems and the working of the economy under different systems

Note Both Micro and macro economics are complementary and should be fully utilized for proper understanding of an economy

1It studies economic aspect of an individual unit2It deals with individual incomeConsumption and savings

3 It facilitates determination of price of any product or factors of production

4 Itrsquos scope is narrow and restricted to individual unit

1It studies the economy as a whole

2It deals with the national income aggregate consumption and aggregate savings3 It facilitates determination of general price level in an economy

4 Itrsquos scope is wide as it deals with economic units on the national level

ACCOUNTS

Introduction to Accounting and Book-keeping

Today I am going to share you the meaning of Accounting and Book-keeping and its related terms bullAccounting bullBook Keeping bullAccountsbullTypes Of Accounts bullAccounting Cycle

bull Meaning of accounting

Ans ) Accounting is the art and science of recording classifying and summarising monetary transactions

bull Meaning of Book-keeping

Ans) Bookkeeping is the art of recording business transactions with the view of having a permanent record of them and showing their effect on wealth

bull Meaning of account

Ans) The term account means a record of

business transactions concern a particular person of firm asset or income or expense It is a summarised record of all transactions which take place in an accounting year

bull Types of accountsPersonal accounts ndash Personal accounts relating

to person and Organisation are known as personal accounts Example Ramrsquos Account ABC amp Co Account etc

Real account - The accounts related to tangible and intangible assets are called real accounts Example Cash Account Furniture Account etc

Nominal account- Accounts related to expenses losses incomes and gains are known as nominal accounts Example Wages Account Salary Account Discount Account etc

bull Accounting cycle Accounting cycle refers to a complete sequence of accounting activities It begins with recording of transactions and ends with the preparation of a balance sheet

Chemistry TopicAtomic Structure

Thomsonrsquos atomic modelThomson (1898) was the first to propose the model of an atomHe proposed that an atom can be regarded as a uniform sphere of positive electricity in which requisite number of electrons are embedded evently to neutralize the positive chargeThis is just like plums embedded in a pudding or seeds evently distributed in red spongy mass of a watermelonThis model of atom is known as ldquoPlum-Pudding modelrdquo or

Q1)What is the fundamental constituents of atomAns Electron Proton and neutrons are the fundamental constituents of atomQ2)What is the value of fundamental unit of electricityAnsThe charge carried by one electron is sad to be the fundamental unit of electricityIts magnitude is 48times10-10esuOr 1602times10-19C Q3)Name the element containing no neutronAnsOrdinary hydrogen atom or protium 1H1

Types of AccountPersonal AccountReal AccountNominal AccountBalance Sheet (opening)

ldquowatermelon modelrdquoThis model could explain the electrical neutrality of an atom but failed to explain the result of scattering experiment carried out by Rutherford in 1911So it was rejected ultimately

Q4)Why is an electron called universal particleAns Itrsquos mass and Charge are independent of its source

EVS Chapter 1 ndash Modes of Existence

Modes of existence When one speaks normally about the mode of existence of some group or individual one refers to their customs their mode of being their ethology their habitat in some way their feeling for a placeDifferent modes of exixtence are ndash

1 Hunting ndashGathering2 Pastoral3 Agricultural4 Industrial

1 Hunting and gathering Hunting and gathering mode of existence is characterized by obtaining food from hunting wild animals including fishing and gathering wild plants From their earliest days the hunter-gatherer diet included various grasses tubers fruits seeds and nuts Lacking the means to kill larger animals they procured meat from smaller game or through scavenging

Societies that rely primarily or exclusively on hunting wild animals fishing and gathering wild fruits berries nuts and vegetables to support their diet are called hunting and gathering societies

At least this used to be practice of human beings before agriculture is invented As their brains evolved hominids developed more intricate knowledge of edible plant life and growth cycles

Q) Write the features of Hunting ndash gathering societiesAns - There are five basic characteristics of hunting and gathering societies

i The primary institution is the family which decides how food is to be shared and how children are to be socialized and which provides for the protection of its members

ii They tend to be small with fewer than fifty members

iii They tend to be nomadic moving to new areas when the current food supply in a given area has been exhausted

iv Members display a high level of interdependence

v Labor division is based on sex men hunt and women gather

Political Science

Introduction to political science

Political science occasionally called politology is a social science which deals with systems of governance and the analysis of political activities political thoughts associated constitutions and political behaviorThe study of political science involves the study of both the

Answer the following questions-1 What is political science

Political science occasionally called politology is a social science which deals with systems of governance and the analysis of political activities political thoughts associated constitutions and political behavior

2 Short notes-

traditional and modern theories of politicsTraditionalClassical political sciencepolitical theory-Traditional political science is the study of politics before Second World War The methodology to study Politics was traditional (legal formaletc) the definition of politics traditional (Politics begins and end with state)area of study (constitution state machinery)was traditionalModern Political scienceModern political theory-Modern Political Theory critically examines the contemporary state of political theory making an assessment of the achievement and limitations of the Behavioural Revolution in its totality and reviews objectively the major paradigms and conceptual frameworks adopted by the disciplineContemporary attempts at the development of an integrated political theory involving the use of both traditional and modern concepts approaches and theories-Around late 1960s several political scientists realized the importance of both the traditional political theory and modern Political theory They began building an integrated theory of politics involving a systematic mixture of traditional and modern studies of politics It was held that the study of a complex and vast field like politics needs both traditional as well as

Classical political theory Modern Political theory

Homework-Learn

modern concepts and approaches for studying itrsquos all aspects

Subject Eng Literature (The Tempest ndash William Shakespeare) Topic Act I Scene 1 Lines 1 to 32 (Line 32 ndash Gonzalo hellip If he be not born to be hanged our case is miserable) Date 13th April 2020 (3rd Period)

[Students should read the original play and also the paraphrase given in the school prescribed textbook]Summary Questions amp Answers

[SUMMARY OF THE ENTIRE SCENE]

o The play starts with the scene of a severe storm at sea Alonso (King of Naples) Sebastian (Alonsorsquos brother) Ferdinand (Alonsorsquos son) Gonzalo Antonio (the usurping Duke of Milan) are in a ship in the midst of the storm

o The mariners are trying their best to control the vessel from running aground and are totally following the orders of their Master the Boatswain They have scant success

o The mariners become extremely unhappy and annoyed when most of the passengers arrive on the deck thereby hampering their effort to save the ship There is serious confrontation between them and the passengers who are part of the Kingrsquos entourage

o The mariners could not save the ship

SUMMING-UP

(i) Vivid description of the scene which gives a realistic description of terror and confusion of a tropical storm

(ii) Shows Shakespearersquos accuracy of knowledge in describing the naval operations and also matters of seamanship

(iii) The opening scene justifies the title ndash The Tempest

UNANSWERED QUESTIONS

(i) The King always travels with his entire fleet including his soldiers Where

(1) GONZALO Nay good be patient (Line 15-26)BOATSWAIN When the sea is Hence What cares these

roarers for the name of the king To cabin silence Trouble us not

GONZALO Good yet remember whom thou has aboardBOATSWAIN None that I more love than myself You are a

councillor if you can command these elements to silence and work

the peace of the present we will not hand a rope more use your authority If you cannot give thanks you have

lived so long and make yourself ready in your cabin for the mischance of the hour if it so hap [To the Mariners]

Cheerly good hearts [To Gonzalo] Out of our way I say

(a) To whom is the boatswain speaking What does he mean by lsquoNone that I more love than myselfrsquo

The Boatswain is speaking to Gonzalo the honest old councilor of the Duke of MilanBy using the words ndash lsquoNone that I love more than I love myselfrsquo means that for the Boatswain nobody is dearer to him than his own life

(b) What were the conditions that made the boatswain react in this way

The Boatswain reacts in this way because the storm is at sea and Alonso King of Naples Sebastian his brother Ferdinand his son Gonzalo Antonio the usurping Duke of Milan on board are in distress and in panic Thus they have rushed to the deck interrupting the work of the mariners

(c) What hope does Gonzalo take from the attitude of the boatswain

The insolent and authoritative attitude of Boatswain makes Gonzalo feel comforted He tells that there are no signs that the Boatswain will be drowned But his facial appearance and attitude shows that he is destined to die on land by hanging which in effect means that all on board will be saved Otherwise all the persons on board are doomed

(d) How can they lsquomake yourself ready in your cabinrsquo For what were they asked to make ready themselves

In order to make themselves ready in their cabin the

were the other ships

(ii) Why was the ship in that area Where was it coming from or going where

(iii) The ship broke apart What happened to those who were in the ship

passengers on board must prepare for death which they will possibly soon have to meetThey can retire to their cabins and offer prayers to the Almighty to save them from drowning

(e) What does the boatswain say when he is asked to be patient What does he order to the royal party

When the boatswain is asked to be patient and remain calm he says that he will be patient only when the storm will be over and the sea will be calm but as long as the storm blows and there is danger to the ship he cannot think of being patient He orders the royal party to go to the cabin and leave the mariners to their work

(2) GONZALO I have great comfort from this fellow (Line 27-36)

Methinks he hath no drowning mark upon him his complexion is perfect

gallows Stand fast good Fate to his hanging Make the rope of his destiny our cable for our own doth little advantage If he be not born to be hanged our case is miserable

(a) Why does Gonzalo regard the Boatswain in the midst of danger

In the midst of danger Gonzalo regards the boatswain because he feels that the Boatswain is a source of comfort and is bent upon to do his work sincerely which in this case is saving the ship and its passengers from the severest of raging storm

(b) What reasons does Gonzalo give when he says that none in the ship will die of drowning

Gonzalo is almost sure that none in the ship will die by drowning His says that there is no mark on the face of the boatswain that indicates that he will die by drowning On the other hand the lines on his face are strong indications that he will be hanged to death Therefore there shall be no danger of the shiprsquos sinking

(c) Explain the following ldquoStand fast good Fate to his hanging Make the rope of his destiny our cable for our own doth little advantage If he be not born to be hanged our case is miserablerdquo

The stated lines mean that if the will of destiny is to be carried out then the ship will not get wrecked and all the passengers will be saved The safety of the passengers therefore depends upon the will of fate being carried out in the case of the boatswain If however the boatswain is not to die by hanging then the passengers are also very unsafe because in that case the ship is likely to sink

(d) What order does the Boatswain give to the sailors

when he re-enters What does he say about the crying of the fellows inside the cabin

The boatswain orders the sailors to bring the topmast lower and bring the ship close to a stationary position with the help of the main sail He says that the fellows inside the cabin are moaning and crying in their distress louder than his voice and louder even than the roaring of the storm

Class XII (ScienceCommerceHumanities) Subject Topic Summary Execution

Computer Science

PropositionalLogic

Propositional logic is a procedure to provide reasoning through statementProposition A ststement that results in True or False is said to be proposition There are two types of propositionSimple proposition amp compound propositionSimple proposioton A simple proposition is one that is not a part of any other proposition Such sentential form of proposition is symbolized with english letters in short For example Ram is a claver student (TrueFalse)Where do you live (Not in True or False)Grapes are sweet (TrueFalse)It rains today (TrueFalse)Here we can see some statements anwer would be true or false but some staements answer can not give in terms of true or false Thus the sentences which can be answered in true or false are known as simple propositionAssigning propositon to a variableThe general syntax to assign propostion to a variable is as followsVariable = Simple propositonFor example A=Ram is a clever studentB= Grapes are sweetC= it rains todayCompound proposition

helliphellipto be continued in next classhelliphellipMath Relation Relation If A and B are two non-empty sets

then a relation R from A to B is a subset of AxB If R A x B and (a b) R then we say that a sube isinis related to b by the relation R written as aRbeg Let A be the set of students of class XII and B be the set of students of class XI Then some of the examples of relation from A to B arei) (a b) AXB a is brother of bisinii) (a b) AXB age of a is more than age of isinb Types of relation In this section we would like to study different types of relations We know that a relation in a set A is a subset of A times A Thus the empty set φ and A times A are two extreme relations For illustration consider a relation R in the set A = 1 2 3 4 given by R = (a b) a ndash b = 10 This is the empty set as no pair (a b) satisfies the condition a ndash b = 10 Similarly R = (a b) | a ndash b | ge 0 is the whole primeset A times A as all pairs (a b) in A times A satisfy | a ndash

Example 1 Let A be the set of all students of a boys school Show that the relation R in A given by R = (a b) a is sister of b is the empty relation and R = (a b) the primedifference between heights of a and b is less than 3 meters is the universal relationSolution Since the school is boys school no student of the school can be sister of any student of the school Hence R = φ showing that R is the empty relation It is also obvious that the difference between heights of any two students of the school has to be less than 3 meters This shows that R = A times A is primethe universal relation Example 2 Show that the relation R in the set 1 2 3 given by R = (1 1) (2 2) (3 3) (1 2) (2 3) is reflexive

b | ge 0 These two extreme examples lead us to the following definitionsDefinition 1 A relation R in a set A is called empty relation if no element of A isrelated to any element of A ie R = φ A times AsubDefinition 2 A relation R in a set A is called universal relation if each element of A is related to every element of A ie R = A times A Both the empty relation and the universal relation are some times called trivial relation Definition 3 A relation R in a set A is called(i) reflexive if (a a) R for every a Aisin isin(ii) symmetric if (a1 a2) R implies that (aisin 2a1)

R for all aisin 1 a2 Aisin(iii) transitive if (a1 a2) R and (aisin 2 a3) R isinimplies that (a1 a3) R for all aisin 1 a2 a3 AisinDefinition 4 A relation R in a set A is said to be an equivalence relation if R is reflexive symmetric and transitive

but neither symmetric nor transitiveSolution R is reflexive since (1 1) (2 2) and (3 3) lie in R Also R is not symmetric as (1 2) R but (2 1) isin notinR Similarly R is not transitive as (1 2) R and (2 3) R but (1 3) R isin isin notinExample 3 Show that the relation R in the set Z of integers given byR = (a b) 2 divides a ndash b is an equivalence relationSolution R is reflexive as 2 divides (a ndash a) for all a Z isinFurther if (a b) R then 2 divides a isinndash b Therefore 2 divides b ndash a Hence (b a) R which shows that R is isinsymmetric Similarly if (a b) R and (b c) R isin isinthen a ndash b and b ndash c are divisible by 2 Now a ndash c = (a ndash b) + (b ndash c) is even (Why) So (a ndash c) is divisible by 2 This shows that R is transitive Thus R is an equivalence relation in ZExample 4 Let L be the set of all lines in a plane and R be the relation in L defined as R = (L1 L2) L1 is perpendicular to L2 Show that R is symmetric but neither reflexive nor transitiveSolution R is not reflexive as a line L1 can not be perpendicular to itself ie (L1 L1) R notinR is symmetric as (L1 L2) Risin

L1 is perpendicular to L2rArr L2 is perpendicular to L1rArr (L2 L1) RrArr isin

R is not transitive Indeed if L1 is perpendicular to L2 and L2 is perpendicular to L3 then L1 can never be perpendicular to L3 In fact L1 is parallel to L3 ie (L1 L2) R isin(L2 L3) R but (L1 L3) Risin notin

Chemistry Solid state Characteristics if Solids(i)The particles are locked in fixed positions they are unable to change their relative positions and this brings a definite shape and volume of a solid(ii)In a solid the constituent particles are held by strong forces of attractionThe forces of attraction may be bonding or non bonding(iii)The constituent particles in a solid pack together as closely as possibleoccupying most of the available space within the solidThus the empty space in a solid is very smallThis makes a solid highly rigid and nearly incompressibleThis also explains why a solid has high density and exhibits slow diffusionClassification of Solids

Q1)Define Crystalline solids AnsA Solid that has a definite geometrical shape and a sharp melting pointand whose constituent particles (atomsmolecules or ions) are arranged in a long range order of definite pattern extending throughout the solid is called a crystalline solidExNaClQ2)Define Amorphous solids AnsA solid that does not have a definite shape and a sharp melting pointand whose constituent particles (atomsmolecules or ions) are not arranged in a definite pattern is called an amorphoussolid

Crystalline solidsAmorphous solids

ExGlassRubberQ3)Classify Crystalline Solids Crystalline Solids

Physics Coloumbrsquos Law (Summary)

Before Going Into Coloumbrsquos Law We Will First Learn What is Charge Properties of Charge and Always remember that charge is quantized ie a body always have static charge of magnitude equal to some integral multiple of fundamental electronic charge e= 16 x 10- 19 C

Charge is the property of matter that causes it to produce and experience electrical and magnetic effects The study of the electrical charges at rest is called electrostatics When both electrical and magnetic effects are present the interaction between charges is referred to as electromagnetic

There exist two types of charges in nature positive and negative Like charges repel and unlike charges attract each other

The type of charge on an electron is negative The charge of a proton is the same as that of an electron but with a positive sign In an atom the number of electrons and the number of protons are equal The atom is therefore electrically neutral If one or more electrons are added to it it becomes negatively charged and is designated as negative ion However if one or more electrons are removed from an atom it becomes positively charged and is called a positive ion

The excess or deficiency of electrons in a body gives the concept of charge If there is an excess of electrons in a body it is negatively charged And if there is deficiency of electrons the body becomes positively charged Whenever addition or removal of electrons takes places the body acquires a charge

The SI Unit of charge is coulomb (C) In SI units the current is a fundamental quantity having a unit of ampere (A) The unit of charge is defined in terms of the unit of current Thus one coulomb is the charge transferred in one second across the section of a wire carrying a

Ionic SolidsMetallicSolids

Molecular Solids

current of one ampere

As q = It we have1 C = (1 A) (1 s)

The dimensions of charge are [A T]

Properties of Charge

(1) Quantization of Charge Electric charge can have only discrete values rather than any value That is charge is quantized The smallest discrete value of charge that can exist in nature is the charge on an electron given as

e = plusmn 16 x 10- 19 C

This is the charge attained by an electron and a protonA charge q must be an integral multiple of this basic unit That is

Q = plusmn ne where n = 1 2 hellip

Charge on a body can never be (frac12)e (23)e or 57e etcWhen we rub a glass rod with silk some electrons are transferred from the rod to the silk The rod becomes positively charged The silk becomes negatively charged The coulomb is a very large amount of charge A typical charge acquired by a rubbed body is 10 - 8 C

Biology Reproduction in organisms

Welcome to this new session 2020-21Today in this first chapter we mainly discuss about reproduction types needs and life span of some organismsWe also discuss about difference between sexual and asexual reproduction

Q1 What is reproductionReproduction is defined as a biological processin which an organism gives rise to young onessimilar to itselfQ2 What are the needs of reproductionbulli) Reproduction maintain life on earthii) It enables the continuity of the species generation after generationiii) It creates genetic variation among populationsQ3 Define Life span and write some orgnisms life spanbull Life span is the period from birth to

the natural death of an organism- OrganismsLife span1 Butterfly 1 - 2 weeks2 Fruit fly 30 days3Dog 10-13 years4 Rose5-7 years5 Tortoise100-150 years6 Banyan Tree -200 - 250 yearsQ4 Reproduction is of two types in case ofanimals but in case of plants vegetative propagation is also present

Asexual Reproduction Sexual Reproductioni) Always uniparentalii) Gametes are not involvediii) Only mitotic division involvediv) Somatic cells of parents are involvedv) Offsprings are genetically similar to the parents

i) Usually biparentalii) Gametes are involvediii) Meiosis occurs during gametogenesis Mitosis occurs after fertilisationiv) Germ cells of the parents are involvedv) offsprings are genetically different from the parents

COMMERCE BUSINESS ENVIRONMENT

Welcome to the new sessiontoday we are going to start the first chapter of Class XII The name of the chapter is Business Environment

Already many of you have got some idea about the word business environment form the first chapter of business studies in class XI

In todayrsquos world every business enterprise is a part of the society It exists and operates in association with various groups in society such as customers suppliers competitors banks and financial institutions government agencies trade unions media and so on All these groups influence the functioning of business in one way or the other They constitute the environment of businessConcept of Business Environment

The term lsquobusiness environmentrsquo refers to the sum total of all individuals institutions and other forces that lie outside a business enterprise but that may influence its functioning and performance

The main features of business environment Totality of External forces General and Specific forces Interrelatedness Complexity Dynamic Uncertainty

Prepare the following questions from todayrsquos assignment

2 What do you mean by business environment

The term lsquobusiness environmentrsquo means the aggregate of all forces factors and institutions which are external to and beyond the control of an individual business enterprise but they may influence its functioning and performance Business environment is the macro framework within which a business firm a micro unit operates It consists of several interrelated and interacting elements

2 Explain the main features of business environment in brief

Totality of External forces-Business environment is the sum total of all things external to a business environment

General and Specific forces-It

Relativity

The Interrelation between business and its environment

The business enterprise is an open system It continuously interacts with its environment It takes inputs (such as raw materials capital labour energy and so on) from its environment transforms them into goods and services and sends them back to the environment

Fig 1 Business Environment Relationship

includes both the forces general forces are the economic social political legal and technological conditions which indirectly influence all business enterprise Specific forces are the investors customers competitors and suppliers which influence individual enterprise directly

Interrelatedness-Different elements of environment are interrelated for an example growing awareness for health care has increased the demand for health foods

Complexity- Business environment id complex in nature as the elements keep on changing example economic technological and other forces changes in demand for a product and service

Dynamic-Business environment is not static it keeps on changing

Uncertainty- Itrsquos very difficult to predict future events such as technology and fashion which occur fast and frequently

Business Studies

Human Resources Management

Human resource of an organisation are the aggregate of knowledge skills attitudes of people working in it

The management system which deals with human resources is called human resource management

Features of HRMbullComprehensive functionbullPeople-oriented

Question1) What do you mean by human

resource management Answer) Human resource management may be defined as that field of Management which has to do with planning organising and controlling the functions of procuring developing maintaining and utilising the labour force

bullAction oriented bullPervasive function bullContinuous function

2) Explain the features of HRM in brief

Answer)bullHuman Resource Management is concerned with managing people at work bull Human Resource Management is concerned with employees which bring people and organisations together so that the goals of each are met bullHuman resource management considered every employees as an individual and also promote their satisfaction and growth bull Human resource management is inherent in all organisations and at all levelsbullManagement of human resources are ongoing on never ending process which requires a constant alertness and Awareness of human relations

3) ldquoHR function is said to be pervasiverdquowhy

Answer) Human resource management is required in all organisations whether it is private or government organisations armed forces sports organisations etc It permeatsall the functional areas like production marketing finance research etc This from this feature of human resource management it can be said that it is pervasive in nature

Economics Demand Q1DEFINITION OF DEMANDIn economics demand is the quantity of a good that consumers are willing and able to purchase at various prices during a given period of timeQ2DEMAND CURVEIn economics a demand curve is a graph depicting the relationship between the price of a certain commodity and the quantity of that commodity that is demanded at that pricQ3LAW OF DEMANDIn microeconomics the law of demand states that conditional on all else being equal as the price of a good increases quantity demanded decreases conversely as the price of a good decreases quantity demanded increasesQ4ASSUMPTION of LAW OF DEMAND(i)No change in price of related commodities(ii) No change in income of the consumer(iii) No change in taste and preferences customs habit and fashion of the consumer( No expectation regarding future change in priceQ5MARKET DEMAND SCHEDULEIn economics a market demand schedule is a tabulation of the quantity of a good that all consumers in a market will purchase at a

given price At any given price the corresponding value on the demand schedule is the sum of all consumersrsquo quantities demanded at that priceQ6INDIVIDUAL DEMAND SCHEDULEIndividual demand schedule refers to a tabular statement showing various quantities of a commodity that a consumer is willing to buy at various levels of price during a given period of timeQ7 FACTORS AFFECTING INDIVIDUAL DEMAND FOR A COMMODITY

The factors that influence a consumerrsquos decision to purchase a commodity are also known as determinants of demand The following factors affect the individual demand for a commodity1 price of the commodity2 price of related goods3 income of buyer of the commodity4 tastes and preferences of the buyer1 Price of the CommodityYou must have observed that when price of a commodity falls you tend to buy more of it and when its price rises you tend to buy less of it when all other factors remain constant (lsquoother things remaining the samersquo) In other words other things remaining the same there is an inverse relationship between the price of a commodity and its quantity demanded by its buyers This statement is in accordance with law of demand which you will study in the later part of this lesson Price of a commodity and its quantity demanded by its buyers are inversely related only when lsquoother things remain the samersquo So lsquoother things remaining the samersquo is an assumption when we study the effect of changes in the price of a commodity on its quantity demanded2 Price of Related goodsA consumer may demand a particular good But while buying that good heshe also asks the price of its related goods Related goods can be of two types-(i) Substitute goods(ii) Complementary goods While purchasing a good prices of its substitutes and complements do affect its quantity purchased(i) Price of Substitute Goods Substitute goods are those goods which can easily be used in place of one another for satisfaction of a particular want like tea and coffee An increase in price of substitute good leads to an increase in demand for the given commodity and a decrease in price of substitute good leads to a decrease in demand for the given commodity It means demand for a given commodity is directly affected by change in price of substitute goods For example if price of coffee increases the demand for tea will rise as tea will become relatively cheaper in comparison to coffee(ii) Price of Complementary goods Complementary goods are those goods which are used together to satisfy a particular want like car and petrol An increase in the price of complementary goods leads to a decrease in demand for the given commodity and a decrease in the price of complementary goods leads to an increase in demand for the given commodity For example if price of petrol falls then the demand for cars will increase as it will be relatively cheaper to use both the goods together So demand for a given commodity is inversely affected by change in price of complementary goods3 Income of the Buyer of CommodityDemand for a commodity is also affected by income of its buyer However the effect of change in income on demand depends on the nature of the commodity under consideration In case of some goods like full cream milk fine quality of rice (Basmati rice) etc demand for these commodities increases when income of the buyer increases and

demand for these commodities decreases when income of the buyer decreases Such goods whose demand increases with the increase in income of the buyer are called normal goods But there are some goods like coarse rice toned milk etc whose demand decreases when income of buyer increases and their demand increases when income of the buyer decreases Such goods whose demand decreases with the increase in income of the buyer are called inferior goods Suppose a consumer buys 10 Kgs of rice whose price is ` 25 per Kg He cannot afford to buy better quality of rice because the price of such rice is ` 50 per Kg The consumer is spending ` 250 per month on the purchase of rice Now if income of the consumer increases and he can afford ` 350 on purchase of 10 Kg of rice Now he can afford to buy some quantity of rice say 6 Kgs whose price is ` 25 per Kg and may buy 4 Kgs of rice whose price is ` 50 per Kg Thus he will buy 10 Kgs of rice by spending ` 350 per month Therefore we may conclude that demand for normal goods is directly related to the income of the buyer but demand for inferior goods is inversely related to the income of the buyer4 Tastes and Preferences of the BuyerThe demand for a commodity is also affected by the tastes and preferences of the buyers They include change in fashion customs habits etc Those commodities are preferred by the consumers which are in fashion So demand for those commodities rises which are in fashion On the other hand if a commodity goes out of the fashion its demand falls because no consumer will like to buy it(5) Number of Buyers in the Market(Population)Increase in population raises the market demand whereas decrease in population reduces the market demand for a commodity Not only the size of population but its composition like age (ratio of males females children and old people in population) also affects the demand for a commodity It is because of needs of children young old male and female population differs(6) Distribution of Income and WealthIf the distribution of income and wealth is more in favour of the rich demand for the commodities preferred by the rich such as comforts and luxuries is likely to be higher On the other hand if the distribution of income and wealth is more in favour of poor demand for commodities preferred by the poor such as necessities will be more(7) Season and Weather ConditionsThis is generally observed that the demand for woolens increases during winter whereas demand for ice creams and cold drinks increases during summer Similarly market demand for umbrellas rain coats increases during rainy seasonQ8 REASONS FOR OPERATION OF LAW OF DEMAND WHY DEMAND CURVE SLOPES DOWNWARDNow we will try to explain why does a consumer purchase more quantity of a commodity at a lower price and less of it at a higher price or why does the law of demand operate ie why does the demand curve slope downwards from left to right The main reasons for operation of law of demand are1 Law of Diminishing Marginal UtilityAs you have studied earlier law of diminishing marginal utility states that as we consume more and more units of a commodity the utility derived from each successive unit goes on decreasing The consumer will be ready to pay more for those units which provide him more utility and less for those which provide him less utility It implies that he will purchase more only when the price of the commodity falls2 Income Effect

When price of a commodity falls purchasing power or real income of the consumer increases which enables him to purchase more quantity of the commodity with the same money income Let us take an example Suppose you buy 4 ice creams when price of each ice cream is ` 25 If price of ice creams falls to ` 20 then with same money income you can buy 5 ice creams now3 Substitution EffectWhen price of a commodity falls it becomes comparatively cheaper as compared to its substitutes (although price of substitutes has not been changed) This will lead to rise in demand for the given commodity For example if coke and Pepsi both are sold at ` 10 each and price of coke falls Now coke has become relatively cheaper and will be substituted for Pepsi It will lead to rise in demand for coke4 Change in Number of BuyersWhen price of a commodity falls some old buyers may demand more of the commodity at the reduced price and some new buyers may also start buying this commodity who were not in a position to buy it earlier due to higher price This will lead to increase in number of buyers when price of the commodity falls As a result demand for the commodity rises when its price falls5 Diverse Uses of a CommoditySome commodities have diverse uses like milk It can be used for drinking for sweet preparation for ice cream preparation etc If price of milk rises its use may be restricted to important purpose only This will lead to reduction in demand for other less important uses When price of milk falls it can be put to other uses also leading to rise n demand for itQ9 EXCEPTIONS TO THE LAW OF DEMANDYou have studied in law of demand that a buyer is willing to buy more quantity of a commodity at a lower price and less of it at a higher price But in certain circumstances a rise in price may lead to rise in demand These circumstances are called Exceptions to the Law of Demand Some important exceptions are1 Giffen GoodsGiffen goods are special type of inferior goods in which negative income effect is stronger than negative substitution effect Giffen goods do not follow law of demand as their demand rises when their price rises Examples of Giffen goods are jowar and bajra etc2 Status Symbol GoodsSome goods are used by rich people as status symbols eg diamonds gold jewellary etc The higher the price the higher will be the demand for these goods When price of such goods falls these goods are no longer looked at as status symbol goods and tehrefore therir demand falls3 NecessitiesCommodities such as medicines salt wheat etc do not follow law of demandbecause we have to purchase them in minimum required quantity whatever their price may be4 Goods Expected to be ScarceWhen the buyers expect a scarcity of a particular good in near future they start buying more and more of that good even if their prices are rising For example during war famines etc people tend to buy more of some goods even at higher prices due to fear of their scarcity in near future

Political Science

Constitution of India-The

Preamble

The preamble-

Preamble-

The preamble is the most precious part of the constitution We the people of India having solemnly resolved to constitute India into a Sovereign Socialist Secular Democratic Republic and to secure to all its citizensA preamble is an introductory and expressionary statement in a document that explains the documents purpose and underlying philosophy When applied to the opening paragraphs of a statute it may recite historical facts pertinent to the subject of the statuteNature and purpose of the constitution-Purpose of the Constitution dictates permanent framework of the government to form a more perfect union to establish justice and ensure peace of thenationconstitution provide principles how the government can run itself following the rules and laws written in the constitution of each state keeps them balanced

Answer the following questions-

1 What is preambleA preamble is an introductory and expressionary statement in a document that explains the documents purpose and underlying philosophy2 What is the nature and

purpose of the constitutionConstitution dictatespermanent framework of the government to form a more perfect union to establish justice and ensure peace of the nation

Homework-Learn

Accounts Compatibilty mode

1MEANING OF PARTNERSHIPPartnership is a form of business organisation where two or more persons join hands to run a business They share the profits and losses according to the agreement amongst them According to the Indian Partnership Act 1932 ldquoPartnership is relation between persons who have agreed to share profits of a business carried on by all or any one of them acting for allrdquo For example one of your friends has passed class XII from National Institute of Open Schooling (NIOS) and wants to start a business Heshe approaches you to join in this venture Heshe wants you to contribute some money and participate in the business activities Both of you if join hands constitute a partnership2CHARACTERISTICS1048698 Agreement A partnership is formed by an agreement The agreement may be either oral or in writing It defines the relationship between the persons who agree to carry on business It may contain the terms of sharing profit and the capital to be invested by each partner etc The written agreement is known as partnership deed1048698 Number of persons There must be at least two persons to form a partnership

The maximum number of partners in a partnership firm can be 50 according toCompanies Act 20131048698 Business The Partnership is formed to carry on business with a purpose of earning profits The business should be lawful Thus if two or more persons agree to carry on unlawful activities it will not be termed as partnership1048698 Sharing Profits The partners agree to share profits in the agreed ratio In caseof loss all the partners have to bear it in the same agreed profit sharing ratio10486981048698Mutual Agency Every partner is an agent of the other partners Every partner can bind the firm and all other partners by hisher acts Each partner will be responsible and liable for the acts of all other partners10486981048698Unlimited liability The liability of each partner except that of a minor is unlimited Their liability extends to their personal assets also If the assets of the firm are insufficient to pay off its debts the partnersrsquo personal property can be used to satisfy the claim of the creditors of the partnership firm10486981048698Management All the partners have a right to mange the business However they may authorize one or more partners to manage the affairs of the business on their behalf10486981048698Transferability of Share No partner can transfer hisher share to any one including hisher family member without the consent of all other partners3PARTNERSHIP DEEDAgreement forms the basis of partnership The written form of the agreement is which a document of partnership is It contains terms and conditions regarding the conduct of the business It also explains relationship between the partners This document is called partnership deed Every firm can frame its own partnership deed in which the rights duties and liabilities of the partners are stated in detail It helps in settling the disputes arising among the partners during the general conduct of business 4CONTENTS OF PARTNERSHIP DEEDThe partnership deed generally contains the following (i) Name and address of the partnership firm(ii) Nature and objectives of the business(iii) Name and address of each partner(iv) Ratio in which profits is to be shared(v) Capital contribution by each partner(vi) Rate of Interest on capital if allowed(vii) Salary or any other remuneration to partners if allowed(viii) Rate of interest on loans and advances by a partner to the firm(ix) Drawings of partners and interest thereon if any(x) Method of valuation of goodwill and revaluation of assets and liabilities on the reconstitution of the partnership ie on the admission retirement or death of a partner(xi) Settlement of disputes by arbitration(xii) Settlement of accounts at the time of retirement or death of a partner5IN ABSENCE OF PARTNERSHIP DEEDThe partnership deed lays down the terms and conditions of partnership in regard to rights duties and obligations of the partners In the absence of partnership deed there may arise a controversy on certain issues like profit sharing ratio interest on

capital interest on drawings interest on loan and salary of the partners In such cases the provisions of the Indian Partnership Act becomes applicableSome of the Issues are(i) Distribution of Profit Partners are entitled to share profits equally(ii) Interest on Capital Interest on capital is not allowed(iii) Interest on Drawings No interest on drawing of the partners is to be charged(iv) Interest on Partnerrsquos Loan A Partner is allowed interest 6 per annum on the amount of loan given to the firm by himher(v) Salary and Commission to Partner A partner is not entitled to anysalary or commission or any other remuneration for managing the business

History TOPIC-TOWARDS INDEPENDENCE AND PARTITION THE LAST PHASE (1935-1947)

SUB TOPIC-IMPORTANT POLITICAL DEVELOPMENTS ndash GROWTH OF SOCIAL IDEAS

Socialism is a political social and economic philosophyLike in other parts of the world the Russian revolution of 1917 served as a great inspiration for revolutionaries in India who at that time were engaged in the struggle for liberation from British ruleSocialist ideas led to the formation of communist party of IndiaJAWAHARLAL NEHRU Among the early Congress leaders Jawaharlal Nehru was very much impressed and influenced by the Socialist ideas He also learnt about the Economic activities of the Soviet Union after the Bolshevic Revolution 1917 He made full use of them in IndiaThe election of Jawaharlal Nehru and Subhas Chandra Bose showed the Left wing tendency within CongressJawaharlal Nehru demanded economic freedom along with political freedom of the people in order to end the exploitation of masses

Nehrus working committee included three socialists leaders The Lucknow session was a landmark in the evolution of socialist ideas of the congressSUBHAS CHANDRA BOSE ndash Subhas Chandra Bose had socialist leaning Both Jawaharlal Nehru and Subhas Chandra Bose were known as leftist Congress men Later on National Congress divided into Leftist and rightist campCONGRESS SOCIALIST Within the Congress some leaders formed the Congress Socialist partyPattavi Sitaramyya Sardar Patel Rajendra Prasad had hostile attitude towards the Congress Socialist partyJawaharlals attitude was hesitant

1 QUESTION ndash Mention name of two Congress leaders who had socialist leaning

1ANSWER ndash Subhas Chandra Bose and Jawaharlal Nehru2QUESTION- In which session of the congress Jawaharlal elaborated his Socialist ideas2 ANSWER ndash Lucknow and Faizpur Session in December 1935 and 19363QUESTION ndash Why Congress was sharply divided into leftist and rightist camp 3ANSWER ndash Subhas Chandra Bosersquos attempt to seek re election for congress presidentship in 1939sharply divided the National Congress into Leftist and Rightist camp4 QUESTION ndash Who was MN Roy 4 ANSWER ndash Manabendra Roy first formed the Communist Party of India outside the country at Tashkent in 19205QUESTION ndash Who formed the Congress Socialist Party within the Congress5 ANSWER ndash Jaya Prakash Narayan Achyut Patwardhan Acharya Narendra Dev Ram Mohan Lohia Aruna Asaf Ali6QUESTION ndash When was the Congress Socialist Party formed What was its object6 ANSWER ndash 1934The Congress Socialist Party sought to work out socialist programme through the Congress They joined hands with the Congress and wanted to carry

Subhas Chandra Bose being expelled from the congress after the Tripuri rift he formed Forward BlockThere were basic differences between the Congress Socialists and the communistsTRADE UNION ACTIVITIES Maximum working class people lived in Bombay and Calcutta The working and living conditions of those workers were very miserable In this situation Shasipada Banerjee NM Lokhande protested against the oppression of the working class peopleThe first Trade Union Madras Labour Union was formed in 1918 by BP WadiaIndustrial strikes took place in Kanpur Calcutta Madras Jamshedpur and Ahmedabad AITUC was formed in Bombay in 1927 The growth of Trade union among the workers was slow because of the fear of the dismissal of the jobIn the mean time the Moderates as well as Communists left AITUC and formed separate organization

on National struggle with the help of workers and peasant class of the society7 QUESTION ndash What was the name of the party founded by Subhas Chandra Bose7 ANSWER- Forward Block8QUESTION ndash Who was Shasipada Banerjee8 ANSWER ndash Shasipada Banerjee was a radical Brahmo He founded a working menrsquos club to protest against exploitation of the British rulers towards the working class of India9 QUESTION ndash What was the weekly published by NM Lokhande9ANSWER- Dinabandhu10 QUESTION ndash Who founded Bombay Mill-Hands Association and in which year10 ANSWER- NM Lokhande in189011 QUESTION- Who was BP WadiaANSWER- BPWadia was the founder of Madras Labour Union in191812 QUESTION- What was the name of the first labour union of India12 ANSWER- Madras Labour Union13 QUESTION Who founded the Majur Mahajan 13 ANSWER GANDHIJI14 QUESTION What was the full form of AITUC When it was formed14 ANSWER All India Trade Union Congressin 192715QUESTION Who formed the Red Trade Union Congress and in which year15ANSWER The Communists formed the Red Trade Union Congress16 QUESTION What do you mean by Socialism16 ANSWER Socialism describes any political and economic theory that says the community rather than individuals should own and manage property and natural resources

Subject Eng Literature (The Tempest ndash William Shakespeare) Topic Act III Scene 3 Lines 1 to 52 (Line 52 ndash Brother my lord the Duke Stand to and do as we) Date 13th April 2020 (4th Period)

[Students should read the original play and also the paraphrase given in the school prescribed textbook]Summary Questions amp Answers

o Alonso Sebastian Antonio Gonzalo Adrian Francisco and others wandered about the island in search of Ferdinand and gets tired and hungry of the toil and at the same time gives up all hope of finding him

o Antonio and Sebastian are happy that Alonso is out of hope and decide to make another attempt on his life that night when being so tired they will be sleeping soundly

o Suddenly a solemn and strange music is heard in the air and several strange shapes enter bringing in a banquet These strange shapes then dance round it with gestures of salutation and then inviting the King to eat they depart

o Seeing this strange scene all are inclined to believe the tales told by travelers that there truly are ldquounicornsrdquo and ldquothe phoenixrsquo thronerdquo

1 ALONSO What harmony is this My good friends hark (L18-27)

GONZALO Marvellous sweet music

[Enter several strange shapes bringing in a banquet

they dance about it with gentle actions of salutation

and inviting the King and his companions to eat they depart]ALONSO Give us kind keepers heavens What were theseSEBASTIAN A living drollery Now I will believe

That there are unicorns that in Arabia

There is one tree the phoenixrsquo throne one phoenix

At this hour reigning thereANTONIO Ill believe both

And what does else want credit come to me

And Ill be sworn rsquotis true Travellers neer did lie

Though fools at home condemn rsquoem

(a) How did Prospero present an amazing spectacle before Alonso and his companions

Using his magic powers Prospero ordered strange shapes to lay a banquet before Alonso and his companions The shapes brought several dishes with tasty eatables in them They placed the dishes on a table before Alonso and his companions Then the strange shapes began to dance gracefully around the banquet While dancing they made gestures inviting them to eat the food Then suddenly the shapes disappeared(b) Who were the guests at the strange banquet Describe the lsquoliving drolleryrsquo

Alonso Sebastian Antonio Gonzalo Adrian and Francisco were the guests at the strange banquet

The term ldquoliving drolleryrdquo refers to live entertainment show In this context when Alonso the King of Naples Sebastian his brother Antonio the treacherous brother of Prospero Gonzalo the kind and loyal councillor to the King Adrian and Francisco came to the island they were hungry and weary in their spirits They heard a solemn and strange music They were shocked to see several strange shapes bringing in a banquet and these shapes danced about it with gentle action of salutation inviting the King and his companions to eat After this Sebastian described this show as lsquoliving drolleryrsquo(c) What is lsquophoenixrsquo What are lsquoUnicornsrdquo

The term lsquophoenixrsquo refers to a mythical Arabian bird which lived alone and perched on a solitary tree After one hundred years it expired in flames and rose again from its own ashes

lsquoUnicornsrsquo refers to the mythological four-footed beasts having horns in the centre of their foreheads When the horns are ground into powder the powder was believed to be

an aphrodisiac(d) How does Sebastian explain the puppet show OR Why does the speaker now believe in unicorns and phoenix

Sebastian finds several strange shapes bringing in the banquet They invite the king and his party for dinner and soon depart He tells that if such a strange sight can be a reality there is nothing incredible in the world and from the present moment he will believe anything He says that it is a strange dumb show enacted not by puppets but by living beings It is stranger than a travellerrsquos tale Seeing such a thing

before his own eyes he will no longer disbelieve the story about unicorns and phoenix(e) How do the other characters present respond to this living drollery

At the sight of the lsquoliving drolleryrsquo like Sebastian Gonzalo and Antonio too acted strangely Antonio told that he too now believes in unicorns and phoenix and anything else that seems to be incredible He too now believes in travellersrsquo tales Gonzalo told that if he would report those happenings in Naples nobody will believe him He considers that those gentle shapes were gentler in manner in comparison to the living beings Alonso was at first sight suspicious and told them that those strange shapes conveyed their meaning in expressive gestures when they seemed to lack speech by their movements and sounds Francisco was amazed at their mysterious disappearance

2 ALONSO Not I

(Line 43-52)GONZALO Faith sir you need not fear When we

were boysWho would believe that there were mountaineers

Dewlapped like bulls whose throats had hanging at rsquoem

Wallets of flesh Or that there were such men

Whose heads stood in their breasts Which now we find

Each putter-out of five for one will bring us

Good warrant ofALONSO I will stand to and feed

Although my lastmdashno matter since I feel

The best is past Brother my lord the Duke

Stand to and do as we

(a) How does Alonso respond at the spectacle of the shapes which were sent to them at the instruction of Prospero

After seeing the strange sight of appearing and disappearing of the shapes sent by Prospero to arrange a banquet for them Alonso says that his surprise at having seen those creatures is infinite and he is fully justified in feeling so much surprise He thinks that their shapes their gestures and the sounds they made were indeed amazing Although they do not possess the gift of speech yet they were able to convey their

thoughts by means of their gestures only

(b) What does Prospero say about the views expressed by Alonso regarding the shapes What does Francisco think about the shapesAfter hearing Alonsorsquos views about the shapes Prospero says that this manrsquos praise of the spirits is rather hasty He means to say that Alonso has shown great haste in reaching the conclusion about the shapes Francisco is amazed to see that those shapes disappeared in a mysterious way(c) What does Sebastian ask Alonso to doSebastian tells Alonso that the shapes having disappeared should not matter to them because they have left the eatables behind He asks Alonso to enjoy eating as they are extremely hungry but the king does not accept his offer of enjoying the dishes(d) How does Gonzalo try to dispel Alonsorsquos fear of those strange shapes What kind of references does he give to AlonsoGonzalo says that those who have travelled abroad have reported seeing even stranger sights than these shapes that Alonso and his companions have beheld Hence there is no reason to feel afraid of these shapes Gonzalo further adds that in his younger days he had heard strange stories from travelers and Alonso might have heard similar stories For instance it was said that there existed a certain race of

human beings who had huge lumps of flesh hanging at their throats and who therefore resembled bulls Then Gonzalo tells about a race of human beings whose heads were located at their breasts Gonzalo says that such stories were not believed by most people in those days but now-a-days these stories have become common(e) Explain the following lsquoEach putter-out of five for onersquoEnglish travellers often insured their trips with London brokers Those that went on foreign travels those days used to deposit a certain amount with some firm or company in London before their departure If the travelers failed to return the money was forfeited by the company with which it had been deposited But this money was repaid five-fold if the travelers returned safe and sound In this way a traveler stood a great chance of recovering the entire cost of his

travels(f) Give the explanatory meanings of the following expressions in the context of the above extract (i) Dewlapped (ii) Wallets of flesh

(iii) Putter-out(i) Dewlapped having big lumps of flesh at the necks(ii) Wallets of flesh large masses of flesh looking like bags(iii) Putter-out to invest money before commencing the travel

  • General methods of preparation of hydrogen
  • Chapter Dimensional Analysis (Summary)
    • Properties of Charge
Page 26:   · Web viewSubject. Topic. Summary. Execution. Hindi. व्याकरण. शरीरके अंगो के नाम लिखिए. 1) आँख 2) नाक 3

inputs are used together in right proportion

A resource or an input which helps in the process of production to obtain an output is called FACTOR OF PRODUCTION

These factors of production can broadly be categorized into four parts 1LAND 2LABOUR3CAPITAL4ENTERPRISE (ORGANISATION)or Entrepreneur

The above factors are all interdependent on each other and they play a major role in production process

FACTORS OF PRODUCTION

LANDCAPITAL

LABOUR ENTREPRENEUR

nature free of cost)LabourLabour refers to the human efforts that need to be combined with other factors of production for creating an output

CapitalAll man ndash made means of production is called capita example machineries which help in further production Money when used for starting any business for purchasing raw materials machinery tools etc it is regarded as capitalCapital also includes physical capital like factories machineriestoolsbuildingsequipments etcEnterpriseThe task of bearing risks is called enterprise and the person who bears these risks of business is called the entrepreneurThus an entrepreneur is one who organises production takes important decisions regarding production hires and purchases factors of production and bears the risk and uncertainty involved in productionOrganisation refers to the services of an entrepreneur who controls organises and undertakes all risks One who plans organises and manages a business enterprise is an organiser

Physics Chapter 1 Force

Force is an external agent capable of changing the state of rest or motion of a particular body It has a magnitude and a direction The direction towards which the force is applied is known as the direction of the force and the application of force is the point where force is applied The Force can be measured using a spring balance The SI unit of force is Newton (N)

Question 1

State the condition when on applying a force the body has

(a) the translational motion

(b) The rotational motion

Solutions

(a) Translational motion is produced when the body is free to move

(b) Rotational motion is produced when the body is pivoted at a point

Question 2

Define moment of force and state its SI unit

Solutions

The moment of force is equal to the product of the magnitude of the force and the perpendicular distance of the line of action of force from the axis

of rotation

The SI unit of moment of force is Newton times meter

= Newton meter (Nm)

Commercial Studies

Stake holders In this topic you will be come to know about the meaning and concept of stakeholders

How stakeholders are different from shareholders

Questions1 What do you mean by the term stake holdersAnswer) The term stake holders have developed from the words which mean an interest or expected benefit Stakeholders mean all those individuals groups and Institutions which have a state (interest) in the functioning and performance of a commercial organisation or a business enterprise2 What do you mean by share holdersAnswer) The person and Groups who own the shares of the joint stock company by providing capital to the company are called shareholders Shareholders are the internal stakeholders shareholders are one out of several stake holders3 How are shareholders different from stakeholdersAnswer)i) The term shareholders is related to only joint stock company whereas stakeholders are related with all business organisationsii) Stakeholders maybe any individual having financial stake in business organisation whereas a shareholders are those individuals who are holding shares in the company4) How are shareholders different from creditorsAnswer) i) Shareholders are internal stakeholders while creditors are external stakeholdersii) Shareholders invest in the capital of the company whereas creditors give loan to the companyiii) Shareholders are the members of the company with voting rights but creditors are not the members of the company

English 1 Transformation of sentences

Sentences A sentence is a group of words which makes complete sense

e Assertive sentencesf Imperative sentencesg Interrogative sentencesh Exclamatory sentences

Sentences can be changed from one grammatical form to another without changing the meaning of the sentence This is known as transformation of sentences

Exercise 1 Change the following affirmative sentences into Negative sentences

a He is a good manHe is not a bad man

b Ram loves SitaRam is not without love for Sita

c Only he stood first in the classNone but he stood first in the class

d Ankit was wiser than he

He was not so wise as Ankite He did it

He did not fail to do itf As soon as I reached college the

bell rangNo sooner did I reach college than the bell rang

g He finished everythingHe left nothing unfinished

h It always pours when it rainsIt never rains but it pours

Math Topic Commercial MathematicsChapter ndash Goods and services Tax

What is GSTAns It is a abbreviated term of Goods and Service Text which is an indirect tax levied on the sale of goods and rendering servicesSome terms related to GSTDelar Any person who buys goods or services For resale is known as a delar A delar Can be a firm or a companyIntra-state sales Sales of goods and services within the same state or same union territory are called intra- state salesInter-state sales Sales of goods and services outside the state or union territory are called Inter-state sales4) Input GST GST is paid by dealers on purchase of goods and services are called input GST5) Output GST GST is collected from customers on sale of goods and services are called output GST6) Types of GST There are three taxes applicable under GST(i) Central Goods and Services Tax (CGST)(ii) State Goods and Services Tax (SGST) or Union Territory Goods and Services Tax (UTGST) Both these taxes are levied on intra-state sales Here GST is divided equally among central and state governments(iii) Integrated Goods and Services Tax (IGST) IGST is levied on inter- state sales It is also levied on import of goods and services into India and export of goods and services from India

Subject Eng Literature (The Merchant of Venice ndash William Shakespeare)Topic Act III Scene 4 Lines 1 to 44 (Portia hellip To wish it back on you fare you well Jessica)[Students should read the original play and also the paraphrase given in the school prescribed textbook]

Summary Questions amp AnswersIn this scene we suddenly find a new element in the character of Portia We have already seen her possessed of every graceful womanly quality but now she shows that she is capable of rapid decision and determined action She shows this by her sudden resolve to hasten to Venice with a daring scheme for the rescue of Antonio This is an important scene in the dramatic action for it leads up to and renders possible the striking events of the famous trial scene which is one of the greatest striking elements of the play Moreover the fact that all the characters of importance are now assembled together in Venice makes the union of the main plot and the secondary story complete

(1) LORENZO Madam although I speak it in your presence(Line 1-9)

You have a noble and a true conceit

Of god-like amity which appears most strongly

In bearing thus the absence of your lordBut if you knew to whom you show this honourHow true a gentleman you send reliefHow dear a lover of my lord your husbandI know you would be prouder of the workThan customary bounty can enforce you

(a) Where is Lorenzo Why is he here To whom is he referring as lsquoMadamrsquo

Lorenzo is at Portiarsquos residence He had met Salerio on the way and Salerio had begged him to come along with him to

o In this scene Portia Nerissa Lorenzo Jessica and Balthazar appear

o Portia requests Lorenzo and Jessica to be in charge of her house during her absence from Belmont because she and Nerissa have decided to spend the days in meditation and also in visiting the holy places in the neighbourhood of Belmont She has already instructed her people to acknowledge both Lorenzo and Jessica as master and mistress of house during her absence Lorenzo and Jessica gladly agree to look after the house of Portia

handover the letter from Antonio to Bassanio The letter carried the bad news about Antoniorsquos arrest for non-payment of loan taken from Shylock Hence Salerio might have preferred company to break this bad news to Bassanio He is referring to Portia as Madam(b) What does Portia say on hearing the above extract

Portia says that she has never regretted doing good to others Friends who spend a lot of time together and really are there for each other have many traits in common As Antonio is Bassaniorsquos best friend saving him is like saving Bassanio who is like her own soul She asks Lorenzo to take care of management of the house till Bassanio is back(c) What does Portia send with Bassanio and why

On hearing about Antoniorsquos troubles on account of Bassanio her husband Portia immediately sends him with enough gold to repay the debt many times over to Venice to help Antonio out of his misfortune

(2) Lorenzo Madam with all my heart (Line 36-40)

I shall obey you in all fair commands

Portia My people do already know my mindAnd will acknowledge you and JessicaIn place of Lord Bassanio and myselfSo fare you well till we shall meet again

(a) Where are Lorenzo and Portia at this time What lsquofair commandsrsquo are given to Lorenzo

Lorenzo and Portia are at Belmont during this scenePortia reveals to Lorenzo that she has sworn to contemplate in prayer at a monastery around two miles away until her husband returns from Venice She tells him that Nerissa would accompany her and asks him to manage the house with Jessica till things are settled In response Lorenzo tells her that he would be obliged to do whatever she asks him to do(b) Where is Portia actually going and why

Portia tells Lorenzo that she would live a life of contemplation and pray at a monastery which is two miles away from her place In reality Portia plans to go to Venice in disguise with Nerissa and argue the case in defense of Antonio She is very sure that her plan would succeed

ClassXI (ScienceHumanitiesCommerce)Subject Topic Summary Execution

Computer Science

(APC)

Ch ndash 1 Numbers

(Numbers in different bases and

their Arithmatical operations)

Number System In computers Number System is defined as a writing system to represent the numbers in different ways ie we are using different symbols and notations to represent numbers There are four ways we can represent the number ndash Binary Decimal Octal and Hexadecimal

Decimal Number SystemThis number system consist 10 digits These are 0 1 2 3 4 5 6 7 8 amp 9

Binary Number SystemThis number system has only two digits these are 0 and 1 Here 0 stands for off while 1 stands for on

Octal Number SystemThis number system has 8 digits these are 0 1 2 3 4 5 6 amp 7

Hexadecimal Number SystemThis number system has 16 digits these are 0 1 2 3 4 5 6 7 8 9 A B C D E F Here the value of the alphabets are as follows A=10 B=11 C=12 D=13 E=14 F=15

Rules for conversion decimal number to Binary1 Divide the decimal number by 22 If the number will not divide equally by 2 then round down the answer to the nearest whole number (integer)3 Keep a note of the remainder it should be either 0 or 14 Keep repeating the above steps dividing each answer by 2 until you reach zero5 Write out all the remainders from bottom to top This is your binary solution

For example Lets convert 32 to binary 2 32 2 16 - 0 2 8 - 0 2 4 - 0 2 2 - 0 2 1 - 0 0 - 1

The binary equivalent of 3210 is 1000002

Try the follwing youself1 2410

2 4810

3 1210

History GROWTH OF NATIONALISM

The second half of the 19th century witnessed growth of political consciousness and a sense of Nationalism among the IndiansThere were various factors for growth of Indian Nationalism- As a result various political associations were formed in different provinces by the educated Indians Surendranath Banerjee organized a meeting of National conference at Calcutta Ultimately the National Congress was founded in Bombay in 1885This body became the vanguard of Indian struggle for freedom The congress leaders were known as moderates because they followed a policy of prayer and petition A large number of Indian leaders had experienced in political agitation The Political situation of England was also changed Moreover increasing revolutionary activities in Maharashtra Punjab and Bengal became serious concern to the British Government In this

QUESTION1 What do you mean by Nationalism ANSWER 1 Nationalism is defined as loyalty and devotion to own nation especially a sense of national consciousnessQUESTION 2 What are the causes of nationalism ANSWER 2 There were various factors for growth of nationalism

1 Spread of western education2 The progress of vernacular press and

patriotic literature3 The economic exploitation of our

country by the colonial rulers4 International affairs

QUESTION 3 Who organized National conference in Calcutta in 1883 ANSWER 3 Surendranath BanerjeeQUESTION 4 When did Indian National Congress formANSWER 4 Indian National Congress was formed in 1885 in BombayQUESTION 5 Who were ModeratesANSWER 5 The Early Nationalists were also known as Moderates Their emergence marked

background Lord Curzon became Viceroy in India He had no respect for the Indian National Congress

the beginning of the organized national movement in India They believed in British justice and were loyal to them They followed a policy of prayer and petition They demanded constitutional reforms of our country Impotant Moderate leaders were Pherozshah Mehta Dadabhai Naorozi and Surendranath Banerjee etcQUESTION 6 What do you know about Extremism in Indian National movementANSWER 6 In the beginning of 20th century a new class of national leaders emerged in India which was different from the moderate groups They started more aggressive movement against the British empire The goal of extremists was ldquoswarajrdquo Important extremist leaders were Bal Gangadhar Tilak Lala Lajpat Rai Bipin Chandra Pal etcQUESTION 7 Mention the places which were the main centres of Revolutionary movementANSWER 7 Maharashtra Bengal and Punjab

Physics

Chapter Dimensional Analysis

(Summary)

The dimensions of a physical quantity are the powers to which the fundamental units are raised in order to obtain the derived unit of that quantit

The physical quantites lengthmasstime are represented by [L] [M] [T] resp let they are raised to powers ( dimesions) abc resp then any physical quantity can be represented by [ La Mb Tc ] Examples

1 Area area = L x B = [L] x [L] = [M0 L2 T0 ]

2 Density density = massvolume = [M][L3] = [ M L-3]

3 Velocity velocity = distancetime = [L][T] = [LT-1]HW Try to find out dimension of acceleration Acceleration = velocity timeNB One can find the SI Units Using Dimension Analysis Such as for area we have [L2] so its SI unit is m2

Biology Topic ndash Chp-1 The living world

Today we will start the first chapter the living world Here we discuss about the characteristics of living organism and what are the difference between them and nonliving substances We also discuss about the contribution of different Scientists

There are over 500000 species of plants andover a million species of animal are present on earth Some 15000 new species were discovered every yearQ1 What is a living organismbull A living organism is primarily physico -chemical material that demonstrate a high degree of complexity is capable of selfRegulation possesses a metabolism and perpetuates itself through timeQ2 What are the differences between livingand non-livingsi) Compared with non-living living organisms

have more complex organised structure and their use of energy is more controlled amp efficientii) Living things reproduce their own kind by forming new cells which contains copies of their genesiii) Each organism has some degree of homeostasisie it is able to make adjustments so that internal environment remains constantQ3 Write contributions of following Scientists i) Aristotle - One of the first theories in Biology places all living things in a hiearchieii) AV Leeuwenhoek - was the first to observe living single celled organisms under microscopeii) Carolus Linnaeus - developed the binary system for naming of organisms and classificationiii) Geregor Johann Mendel ndash discoverbasic principles of inheritanceHomework i) C Darwin ii)Schleiden

Math Trigonometric functions

1 Overviewi) Trigonometry The word lsquotrigonometryrsquo is derived from the Greek words lsquotrigonrsquo and lsquometronrsquo which means measuring the sides of a triangle An angle is the amount of rotation of a revolving line with respect to a fixed line Usually we follow two types of conventions for measuring angles ie a) Sexagesimal system b) Circular system In Sexagesimal system the unit of measurement is Degree In Circular system the unit of measurement is Radian ii) Relation between degree and radianThe ratio of circumference of a circle to its diameter is always a constant This constant ratio is a number denoted by π which is taken approximately as 227The relationship between degree amp radian measurements is as follows2 right angles = 180deg= π radians1radian = 180degπ=57deg16(approx) 1deg=π180 radianiii) Length of an arc of a circleIf an arc of length s subtends an angle θ radians at the center of a circle of radius r then s=rθiv) Area of a sector of a circleA sector is like a pizza slice of the

Q) Express the following angles in radiana) 45deg b) 40deg3730Ans a) We have 180deg=π radiansi e 45deg= πtimes45180 radian = π4 radiansb) 40deg3730= 40deg37+3060 minute= 40deg 37 +12 minute= 40deg+ 752 minute=40 + 75(2times60) degree=3258 degreeNow 180deg=π radianie 3258 degree= (πtimes325) (180times8) radians = 65π288 radiansQ) A circle has a radius of r=12 meters What is the length of an arc traced out by a 60deg angle in the center of the circleAns In this problem we know both the central angle (60deg) and the radius of the circle (12) All we have to do is plug those values into our equation and we get

s = 2π(12)(60360)s = 24π6s = 4πSo the length of an arc traced out by a 60deg angle in a circle with a radius of 12 meters equals 4π meters asymp 1257 metersQ) Find the area of the sector with a central angle 30deg and a radius of 9cmAns GivenRadius r = 9 cmAngle θ = 30degArea of the sector = θ360degtimesπr2

= 30360degtimes227times92=2121cm2

circle It consists of a region bounded by two radii and an arc lying between the radiiThe area of a sector is a fraction of the area of the circle This area is proportional to the central angle In other words the bigger the central angle the larger is the area of the sectorArea of Sector = θ2 times r2 (when θ is in radians)

Area of Sector = θ times π360 times r2 (when θ is in degrees)

COMMERCE

CLASSIFICTION OF HUMAN ACTIVITIES-ECONOMIC AND NON-ECONOMIC

Welcome to the new sessiontoday we are going to start the first chapter of Class XI The name of the chapter that we are going to start is

lsquoClassification of Human Activities ndasheconomic and non-economicrsquo

Now let us start the chapter by considering human beings and the activities they perform throughout the day

Human activities means all those activities that human beings undertake to satisfy their wants

Human wants on the other hand are the desire of human beings for goods (vegetables fruits rice etc) and services (services of doctors teachers lawyers etc) that they require to live

Now these human activities continue throughout life as human wants are unending unlimited and recurring as human beings desire for better living throughout their lives

Now human activities can be classified into two categories

Human activities

Economic activities Non-economic activities

Economic activities are

Questions1 What are human activities

Answer Human activities mean all those activities that human beings undertake to satisfy their wants

Example A man working in an office

A boy playing in the garden

2What are the characteristics of human activitiesAnswer the characteristics of human activities are as follows

Human activities are undertaken by men women and children and these activities involve human efforts

Human activities are undertaken to satisfy human wants which are unlimited

Human activities continue throughout life

Human activities are performed for both earning money and personal satisfaction

3What is economic activitiesGive example

Answer Economic activities are undertaken by human beings with the object of earning money acquiring wealth and thereby satisfying human wantsExample

Selling of goods by a shop keeper to his customer

A clinic run by a doctor Service of a teacher in school or college

undertaken by human beings with the object of earning money and acquiring wealth

These activities result in the production of economic goods and services

Example Human activities(ie working in factories officesshops) which produce direct economic benefits

Non-economic activities are inspired by human sentiments and emotions such as love for the family desire to help the poor and love for the country

Thus these human activities (eg praying playing sleeping) produce no direct economic benefits and they are also not related to earning money and acquiring wealth

4 What are the characteristics of economic activities

Answer The characteristics of economic activities are as follows

Economic motiveEconomic activities are undertaken to earn money and acquire wealth

ProductiveEconomic activities involve productiondistribution and exchange of goods and services to create wealth

Economic growthEconomic activities determine the level of economic development of a country and standard of living of its citizens

Socially desirableEconomic activities are socially desirable for society

Economic resourcesEconomic activities make use of all the economic resources such landlabourcapital etc

5 What do you mean by non-economic activitiesExampleAnswerNon-economic activities are inspired by human sentiments and emotions such as love for the family desire to help the poor and love for the countryThese activities are not undertaken for monetary gain but for onersquos satisfaction and happinessExample

a mother looks after her children

a student donates blood8 Differentiate between Economic activities and Non-economic activities

Economic activities

Non-economic activities

1to earn living and acquiring wealth2Result can be measured in terms of money

3ExampleBusinessprofession and employment

1 to obtain some satisfaction

2Result cannot be measured in terms of money

3ExampleFamily-orientedreligious socialCultural and national

BUSINESS STUDIES

BUSINESS ENVIRONMENT

Welcome to the new sessionToday we are going to start the first chapter and the name of the chapter is Business Environment

In todayrsquos world every business enterprise is a part of the society It exists and operates in association with various groups in society such as customers suppliers competitors banks and financial institutions government agencies trade unions media and so on All these groups influence the functioning of business in one way or the other They constitute the environment of businessConcept of Business Environment

The term lsquobusiness environmentrsquo refers to the sum total of all individuals institutions and other forces that lie outside a business enterprise but that may influence its functioning and performance

The main features of business environment

Totality of External forces General and Specific forces Interrelatedness Complexity Dynamic Uncertainty Relativity

The Interrelation between business and its environment

The business enterprise is an open system It continuously interacts with its environment It takes inputs

Prepare the following questions from todayrsquos assignment

1 What do you mean by business environment

The term lsquobusiness environmentrsquo means the aggregate of all forces factors and institutions which are external to and beyond the control of an individual business enterprise but they may influence its functioning and performance Business environment is the macro framework within which a business firm a micro unit operates It consists of several interrelated and interacting elements

2 Explain the main features of business environment in brief

Totality of External forces-Business environment is the sum total of all things external to a business environment

General and Specific forces-It includes both the forces general forces are the economic social political legal and technological conditions which indirectly influence all business enterprise Specific forces are the investors customers competitors and suppliers which influence individual enterprise directly

Interrelatedness-Different elements of environment are interrelated for an example growing awareness for health care has increased the demand for health foods

Complexity- Business environment id

(such as raw materials capital labour energy and so on) from its environment transforms them into goods and services and sends them back to the environment

Fig 1 Business Environment Relationship

complex in nature as the elements keep on changing example economic technological and other forces changes in demand for a product and service

Dynamic-Business environment is not static it keeps on changing

Uncertainty- Itrsquos very difficult to predict future events such as technology and fashion which occur fast and frequently

Economics Basic Economic ConceptsSub topic

Microeconomics and

Macroeconomics

Welcome to the new sessiontoday we are going to start the first chapter of Class XI The name of the chapter that we are going to start is Basic Economic concepts

Now Economics covers the study of human activities Human activities are those activities which are performed by humans to satisfy their wants

Thus Human wants are unlimited and therefore economic activities such as production exchange and consumption are needed in order to satisfy those wants

The study of economics is divided largely in two parts which areMicroeconomics and Macroeconomics

SUBJECT- MATTER OF ECONOMICS

MICROECONOMICS MACROECONOMICS

Questions1Who has coined the words micro and macro economics

Answer Ranger Frisch coined the words lsquomicrorsquo and lsquomacrorsquo in 1933 to denote the two branches of economic theory namely microeconomics and macroeconomics

2What is microeconomicsAnswer It is the study of behaviour of individual decision ndash making unit such as consumers firms etc

3 What is macroeconomicsAnswer Macroeonomics is the study of overall economic phenomena like employment national income etc

4 What is the importance of microeconomicsAnswer

Microeconomics helps in formulating economic policies which enhance productive efficiency and results in greater social welfare

It helps the government in formulating correct price policies

It explains the working of a capitalistic economy where individual units(producers and consumers ) are free to take their own decision

Micro means a small part in

microeconomics we do not study the whole economy Hence we study an individual consumer and his or her choices and a producer and his or her profit maximizing decisions in the market Thus it does not mirror what happens in the economy as a whole

Macroeconomics on the other hand studies the economy as a whole It is concerned with aggregate and depicts the entire picture of the economyMacroeconomics deals with the national income aggregate investment aggregate consumption etc

Features of Microeconomics It deals with small

parts of the country Hence it looks at

individual consumers firms and industries

It deals with individual income consumption and savings

It studies the determination of price of any product or factors of production

It deals with the working of market via the price mechanism which is nothing but the determination of price and quantity of a commodity by the forces of demand and supply

Features of Macroeconomics

It deals with the study of the economy as a whole

It is concerned with

5 Give a limitation of microeconomics Microeconomics fails to explain the

functioning of an economy as a whole It cannot explain unemployment illiteracy and other problems prevailing in the country

6 What is the importance of macroeconomics It gives overall view of the growing

complexities of an economic system It provides the basic and logical

framework for formulating appropriate macroeconomic policies (eg for inflation poverty etc )to direct and regulate economy towards desirable goals

7What is the limitation of macroeconomics It ignores structural changes in an

individual unit of the aggregate

8 Differentiate between Microeconomics and Macroeconomics

Microeconomics Macroeconomics

the study of aggregates

National income aggregate savings and aggregate investments are major concepts dealt within macroeconomics style

It studies the determination of general price levels

It investigates into the problem of unemployment and the achievement of employment

It studies the aspect of decision making at the aggregate and national levels

It includes all growth theories whether related to developed or developing economies it also includes the study of economic systems and the working of the economy under different systems

Note Both Micro and macro economics are complementary and should be fully utilized for proper understanding of an economy

1It studies economic aspect of an individual unit2It deals with individual incomeConsumption and savings

3 It facilitates determination of price of any product or factors of production

4 Itrsquos scope is narrow and restricted to individual unit

1It studies the economy as a whole

2It deals with the national income aggregate consumption and aggregate savings3 It facilitates determination of general price level in an economy

4 Itrsquos scope is wide as it deals with economic units on the national level

ACCOUNTS

Introduction to Accounting and Book-keeping

Today I am going to share you the meaning of Accounting and Book-keeping and its related terms bullAccounting bullBook Keeping bullAccountsbullTypes Of Accounts bullAccounting Cycle

bull Meaning of accounting

Ans ) Accounting is the art and science of recording classifying and summarising monetary transactions

bull Meaning of Book-keeping

Ans) Bookkeeping is the art of recording business transactions with the view of having a permanent record of them and showing their effect on wealth

bull Meaning of account

Ans) The term account means a record of

business transactions concern a particular person of firm asset or income or expense It is a summarised record of all transactions which take place in an accounting year

bull Types of accountsPersonal accounts ndash Personal accounts relating

to person and Organisation are known as personal accounts Example Ramrsquos Account ABC amp Co Account etc

Real account - The accounts related to tangible and intangible assets are called real accounts Example Cash Account Furniture Account etc

Nominal account- Accounts related to expenses losses incomes and gains are known as nominal accounts Example Wages Account Salary Account Discount Account etc

bull Accounting cycle Accounting cycle refers to a complete sequence of accounting activities It begins with recording of transactions and ends with the preparation of a balance sheet

Chemistry TopicAtomic Structure

Thomsonrsquos atomic modelThomson (1898) was the first to propose the model of an atomHe proposed that an atom can be regarded as a uniform sphere of positive electricity in which requisite number of electrons are embedded evently to neutralize the positive chargeThis is just like plums embedded in a pudding or seeds evently distributed in red spongy mass of a watermelonThis model of atom is known as ldquoPlum-Pudding modelrdquo or

Q1)What is the fundamental constituents of atomAns Electron Proton and neutrons are the fundamental constituents of atomQ2)What is the value of fundamental unit of electricityAnsThe charge carried by one electron is sad to be the fundamental unit of electricityIts magnitude is 48times10-10esuOr 1602times10-19C Q3)Name the element containing no neutronAnsOrdinary hydrogen atom or protium 1H1

Types of AccountPersonal AccountReal AccountNominal AccountBalance Sheet (opening)

ldquowatermelon modelrdquoThis model could explain the electrical neutrality of an atom but failed to explain the result of scattering experiment carried out by Rutherford in 1911So it was rejected ultimately

Q4)Why is an electron called universal particleAns Itrsquos mass and Charge are independent of its source

EVS Chapter 1 ndash Modes of Existence

Modes of existence When one speaks normally about the mode of existence of some group or individual one refers to their customs their mode of being their ethology their habitat in some way their feeling for a placeDifferent modes of exixtence are ndash

1 Hunting ndashGathering2 Pastoral3 Agricultural4 Industrial

1 Hunting and gathering Hunting and gathering mode of existence is characterized by obtaining food from hunting wild animals including fishing and gathering wild plants From their earliest days the hunter-gatherer diet included various grasses tubers fruits seeds and nuts Lacking the means to kill larger animals they procured meat from smaller game or through scavenging

Societies that rely primarily or exclusively on hunting wild animals fishing and gathering wild fruits berries nuts and vegetables to support their diet are called hunting and gathering societies

At least this used to be practice of human beings before agriculture is invented As their brains evolved hominids developed more intricate knowledge of edible plant life and growth cycles

Q) Write the features of Hunting ndash gathering societiesAns - There are five basic characteristics of hunting and gathering societies

i The primary institution is the family which decides how food is to be shared and how children are to be socialized and which provides for the protection of its members

ii They tend to be small with fewer than fifty members

iii They tend to be nomadic moving to new areas when the current food supply in a given area has been exhausted

iv Members display a high level of interdependence

v Labor division is based on sex men hunt and women gather

Political Science

Introduction to political science

Political science occasionally called politology is a social science which deals with systems of governance and the analysis of political activities political thoughts associated constitutions and political behaviorThe study of political science involves the study of both the

Answer the following questions-1 What is political science

Political science occasionally called politology is a social science which deals with systems of governance and the analysis of political activities political thoughts associated constitutions and political behavior

2 Short notes-

traditional and modern theories of politicsTraditionalClassical political sciencepolitical theory-Traditional political science is the study of politics before Second World War The methodology to study Politics was traditional (legal formaletc) the definition of politics traditional (Politics begins and end with state)area of study (constitution state machinery)was traditionalModern Political scienceModern political theory-Modern Political Theory critically examines the contemporary state of political theory making an assessment of the achievement and limitations of the Behavioural Revolution in its totality and reviews objectively the major paradigms and conceptual frameworks adopted by the disciplineContemporary attempts at the development of an integrated political theory involving the use of both traditional and modern concepts approaches and theories-Around late 1960s several political scientists realized the importance of both the traditional political theory and modern Political theory They began building an integrated theory of politics involving a systematic mixture of traditional and modern studies of politics It was held that the study of a complex and vast field like politics needs both traditional as well as

Classical political theory Modern Political theory

Homework-Learn

modern concepts and approaches for studying itrsquos all aspects

Subject Eng Literature (The Tempest ndash William Shakespeare) Topic Act I Scene 1 Lines 1 to 32 (Line 32 ndash Gonzalo hellip If he be not born to be hanged our case is miserable) Date 13th April 2020 (3rd Period)

[Students should read the original play and also the paraphrase given in the school prescribed textbook]Summary Questions amp Answers

[SUMMARY OF THE ENTIRE SCENE]

o The play starts with the scene of a severe storm at sea Alonso (King of Naples) Sebastian (Alonsorsquos brother) Ferdinand (Alonsorsquos son) Gonzalo Antonio (the usurping Duke of Milan) are in a ship in the midst of the storm

o The mariners are trying their best to control the vessel from running aground and are totally following the orders of their Master the Boatswain They have scant success

o The mariners become extremely unhappy and annoyed when most of the passengers arrive on the deck thereby hampering their effort to save the ship There is serious confrontation between them and the passengers who are part of the Kingrsquos entourage

o The mariners could not save the ship

SUMMING-UP

(i) Vivid description of the scene which gives a realistic description of terror and confusion of a tropical storm

(ii) Shows Shakespearersquos accuracy of knowledge in describing the naval operations and also matters of seamanship

(iii) The opening scene justifies the title ndash The Tempest

UNANSWERED QUESTIONS

(i) The King always travels with his entire fleet including his soldiers Where

(1) GONZALO Nay good be patient (Line 15-26)BOATSWAIN When the sea is Hence What cares these

roarers for the name of the king To cabin silence Trouble us not

GONZALO Good yet remember whom thou has aboardBOATSWAIN None that I more love than myself You are a

councillor if you can command these elements to silence and work

the peace of the present we will not hand a rope more use your authority If you cannot give thanks you have

lived so long and make yourself ready in your cabin for the mischance of the hour if it so hap [To the Mariners]

Cheerly good hearts [To Gonzalo] Out of our way I say

(a) To whom is the boatswain speaking What does he mean by lsquoNone that I more love than myselfrsquo

The Boatswain is speaking to Gonzalo the honest old councilor of the Duke of MilanBy using the words ndash lsquoNone that I love more than I love myselfrsquo means that for the Boatswain nobody is dearer to him than his own life

(b) What were the conditions that made the boatswain react in this way

The Boatswain reacts in this way because the storm is at sea and Alonso King of Naples Sebastian his brother Ferdinand his son Gonzalo Antonio the usurping Duke of Milan on board are in distress and in panic Thus they have rushed to the deck interrupting the work of the mariners

(c) What hope does Gonzalo take from the attitude of the boatswain

The insolent and authoritative attitude of Boatswain makes Gonzalo feel comforted He tells that there are no signs that the Boatswain will be drowned But his facial appearance and attitude shows that he is destined to die on land by hanging which in effect means that all on board will be saved Otherwise all the persons on board are doomed

(d) How can they lsquomake yourself ready in your cabinrsquo For what were they asked to make ready themselves

In order to make themselves ready in their cabin the

were the other ships

(ii) Why was the ship in that area Where was it coming from or going where

(iii) The ship broke apart What happened to those who were in the ship

passengers on board must prepare for death which they will possibly soon have to meetThey can retire to their cabins and offer prayers to the Almighty to save them from drowning

(e) What does the boatswain say when he is asked to be patient What does he order to the royal party

When the boatswain is asked to be patient and remain calm he says that he will be patient only when the storm will be over and the sea will be calm but as long as the storm blows and there is danger to the ship he cannot think of being patient He orders the royal party to go to the cabin and leave the mariners to their work

(2) GONZALO I have great comfort from this fellow (Line 27-36)

Methinks he hath no drowning mark upon him his complexion is perfect

gallows Stand fast good Fate to his hanging Make the rope of his destiny our cable for our own doth little advantage If he be not born to be hanged our case is miserable

(a) Why does Gonzalo regard the Boatswain in the midst of danger

In the midst of danger Gonzalo regards the boatswain because he feels that the Boatswain is a source of comfort and is bent upon to do his work sincerely which in this case is saving the ship and its passengers from the severest of raging storm

(b) What reasons does Gonzalo give when he says that none in the ship will die of drowning

Gonzalo is almost sure that none in the ship will die by drowning His says that there is no mark on the face of the boatswain that indicates that he will die by drowning On the other hand the lines on his face are strong indications that he will be hanged to death Therefore there shall be no danger of the shiprsquos sinking

(c) Explain the following ldquoStand fast good Fate to his hanging Make the rope of his destiny our cable for our own doth little advantage If he be not born to be hanged our case is miserablerdquo

The stated lines mean that if the will of destiny is to be carried out then the ship will not get wrecked and all the passengers will be saved The safety of the passengers therefore depends upon the will of fate being carried out in the case of the boatswain If however the boatswain is not to die by hanging then the passengers are also very unsafe because in that case the ship is likely to sink

(d) What order does the Boatswain give to the sailors

when he re-enters What does he say about the crying of the fellows inside the cabin

The boatswain orders the sailors to bring the topmast lower and bring the ship close to a stationary position with the help of the main sail He says that the fellows inside the cabin are moaning and crying in their distress louder than his voice and louder even than the roaring of the storm

Class XII (ScienceCommerceHumanities) Subject Topic Summary Execution

Computer Science

PropositionalLogic

Propositional logic is a procedure to provide reasoning through statementProposition A ststement that results in True or False is said to be proposition There are two types of propositionSimple proposition amp compound propositionSimple proposioton A simple proposition is one that is not a part of any other proposition Such sentential form of proposition is symbolized with english letters in short For example Ram is a claver student (TrueFalse)Where do you live (Not in True or False)Grapes are sweet (TrueFalse)It rains today (TrueFalse)Here we can see some statements anwer would be true or false but some staements answer can not give in terms of true or false Thus the sentences which can be answered in true or false are known as simple propositionAssigning propositon to a variableThe general syntax to assign propostion to a variable is as followsVariable = Simple propositonFor example A=Ram is a clever studentB= Grapes are sweetC= it rains todayCompound proposition

helliphellipto be continued in next classhelliphellipMath Relation Relation If A and B are two non-empty sets

then a relation R from A to B is a subset of AxB If R A x B and (a b) R then we say that a sube isinis related to b by the relation R written as aRbeg Let A be the set of students of class XII and B be the set of students of class XI Then some of the examples of relation from A to B arei) (a b) AXB a is brother of bisinii) (a b) AXB age of a is more than age of isinb Types of relation In this section we would like to study different types of relations We know that a relation in a set A is a subset of A times A Thus the empty set φ and A times A are two extreme relations For illustration consider a relation R in the set A = 1 2 3 4 given by R = (a b) a ndash b = 10 This is the empty set as no pair (a b) satisfies the condition a ndash b = 10 Similarly R = (a b) | a ndash b | ge 0 is the whole primeset A times A as all pairs (a b) in A times A satisfy | a ndash

Example 1 Let A be the set of all students of a boys school Show that the relation R in A given by R = (a b) a is sister of b is the empty relation and R = (a b) the primedifference between heights of a and b is less than 3 meters is the universal relationSolution Since the school is boys school no student of the school can be sister of any student of the school Hence R = φ showing that R is the empty relation It is also obvious that the difference between heights of any two students of the school has to be less than 3 meters This shows that R = A times A is primethe universal relation Example 2 Show that the relation R in the set 1 2 3 given by R = (1 1) (2 2) (3 3) (1 2) (2 3) is reflexive

b | ge 0 These two extreme examples lead us to the following definitionsDefinition 1 A relation R in a set A is called empty relation if no element of A isrelated to any element of A ie R = φ A times AsubDefinition 2 A relation R in a set A is called universal relation if each element of A is related to every element of A ie R = A times A Both the empty relation and the universal relation are some times called trivial relation Definition 3 A relation R in a set A is called(i) reflexive if (a a) R for every a Aisin isin(ii) symmetric if (a1 a2) R implies that (aisin 2a1)

R for all aisin 1 a2 Aisin(iii) transitive if (a1 a2) R and (aisin 2 a3) R isinimplies that (a1 a3) R for all aisin 1 a2 a3 AisinDefinition 4 A relation R in a set A is said to be an equivalence relation if R is reflexive symmetric and transitive

but neither symmetric nor transitiveSolution R is reflexive since (1 1) (2 2) and (3 3) lie in R Also R is not symmetric as (1 2) R but (2 1) isin notinR Similarly R is not transitive as (1 2) R and (2 3) R but (1 3) R isin isin notinExample 3 Show that the relation R in the set Z of integers given byR = (a b) 2 divides a ndash b is an equivalence relationSolution R is reflexive as 2 divides (a ndash a) for all a Z isinFurther if (a b) R then 2 divides a isinndash b Therefore 2 divides b ndash a Hence (b a) R which shows that R is isinsymmetric Similarly if (a b) R and (b c) R isin isinthen a ndash b and b ndash c are divisible by 2 Now a ndash c = (a ndash b) + (b ndash c) is even (Why) So (a ndash c) is divisible by 2 This shows that R is transitive Thus R is an equivalence relation in ZExample 4 Let L be the set of all lines in a plane and R be the relation in L defined as R = (L1 L2) L1 is perpendicular to L2 Show that R is symmetric but neither reflexive nor transitiveSolution R is not reflexive as a line L1 can not be perpendicular to itself ie (L1 L1) R notinR is symmetric as (L1 L2) Risin

L1 is perpendicular to L2rArr L2 is perpendicular to L1rArr (L2 L1) RrArr isin

R is not transitive Indeed if L1 is perpendicular to L2 and L2 is perpendicular to L3 then L1 can never be perpendicular to L3 In fact L1 is parallel to L3 ie (L1 L2) R isin(L2 L3) R but (L1 L3) Risin notin

Chemistry Solid state Characteristics if Solids(i)The particles are locked in fixed positions they are unable to change their relative positions and this brings a definite shape and volume of a solid(ii)In a solid the constituent particles are held by strong forces of attractionThe forces of attraction may be bonding or non bonding(iii)The constituent particles in a solid pack together as closely as possibleoccupying most of the available space within the solidThus the empty space in a solid is very smallThis makes a solid highly rigid and nearly incompressibleThis also explains why a solid has high density and exhibits slow diffusionClassification of Solids

Q1)Define Crystalline solids AnsA Solid that has a definite geometrical shape and a sharp melting pointand whose constituent particles (atomsmolecules or ions) are arranged in a long range order of definite pattern extending throughout the solid is called a crystalline solidExNaClQ2)Define Amorphous solids AnsA solid that does not have a definite shape and a sharp melting pointand whose constituent particles (atomsmolecules or ions) are not arranged in a definite pattern is called an amorphoussolid

Crystalline solidsAmorphous solids

ExGlassRubberQ3)Classify Crystalline Solids Crystalline Solids

Physics Coloumbrsquos Law (Summary)

Before Going Into Coloumbrsquos Law We Will First Learn What is Charge Properties of Charge and Always remember that charge is quantized ie a body always have static charge of magnitude equal to some integral multiple of fundamental electronic charge e= 16 x 10- 19 C

Charge is the property of matter that causes it to produce and experience electrical and magnetic effects The study of the electrical charges at rest is called electrostatics When both electrical and magnetic effects are present the interaction between charges is referred to as electromagnetic

There exist two types of charges in nature positive and negative Like charges repel and unlike charges attract each other

The type of charge on an electron is negative The charge of a proton is the same as that of an electron but with a positive sign In an atom the number of electrons and the number of protons are equal The atom is therefore electrically neutral If one or more electrons are added to it it becomes negatively charged and is designated as negative ion However if one or more electrons are removed from an atom it becomes positively charged and is called a positive ion

The excess or deficiency of electrons in a body gives the concept of charge If there is an excess of electrons in a body it is negatively charged And if there is deficiency of electrons the body becomes positively charged Whenever addition or removal of electrons takes places the body acquires a charge

The SI Unit of charge is coulomb (C) In SI units the current is a fundamental quantity having a unit of ampere (A) The unit of charge is defined in terms of the unit of current Thus one coulomb is the charge transferred in one second across the section of a wire carrying a

Ionic SolidsMetallicSolids

Molecular Solids

current of one ampere

As q = It we have1 C = (1 A) (1 s)

The dimensions of charge are [A T]

Properties of Charge

(1) Quantization of Charge Electric charge can have only discrete values rather than any value That is charge is quantized The smallest discrete value of charge that can exist in nature is the charge on an electron given as

e = plusmn 16 x 10- 19 C

This is the charge attained by an electron and a protonA charge q must be an integral multiple of this basic unit That is

Q = plusmn ne where n = 1 2 hellip

Charge on a body can never be (frac12)e (23)e or 57e etcWhen we rub a glass rod with silk some electrons are transferred from the rod to the silk The rod becomes positively charged The silk becomes negatively charged The coulomb is a very large amount of charge A typical charge acquired by a rubbed body is 10 - 8 C

Biology Reproduction in organisms

Welcome to this new session 2020-21Today in this first chapter we mainly discuss about reproduction types needs and life span of some organismsWe also discuss about difference between sexual and asexual reproduction

Q1 What is reproductionReproduction is defined as a biological processin which an organism gives rise to young onessimilar to itselfQ2 What are the needs of reproductionbulli) Reproduction maintain life on earthii) It enables the continuity of the species generation after generationiii) It creates genetic variation among populationsQ3 Define Life span and write some orgnisms life spanbull Life span is the period from birth to

the natural death of an organism- OrganismsLife span1 Butterfly 1 - 2 weeks2 Fruit fly 30 days3Dog 10-13 years4 Rose5-7 years5 Tortoise100-150 years6 Banyan Tree -200 - 250 yearsQ4 Reproduction is of two types in case ofanimals but in case of plants vegetative propagation is also present

Asexual Reproduction Sexual Reproductioni) Always uniparentalii) Gametes are not involvediii) Only mitotic division involvediv) Somatic cells of parents are involvedv) Offsprings are genetically similar to the parents

i) Usually biparentalii) Gametes are involvediii) Meiosis occurs during gametogenesis Mitosis occurs after fertilisationiv) Germ cells of the parents are involvedv) offsprings are genetically different from the parents

COMMERCE BUSINESS ENVIRONMENT

Welcome to the new sessiontoday we are going to start the first chapter of Class XII The name of the chapter is Business Environment

Already many of you have got some idea about the word business environment form the first chapter of business studies in class XI

In todayrsquos world every business enterprise is a part of the society It exists and operates in association with various groups in society such as customers suppliers competitors banks and financial institutions government agencies trade unions media and so on All these groups influence the functioning of business in one way or the other They constitute the environment of businessConcept of Business Environment

The term lsquobusiness environmentrsquo refers to the sum total of all individuals institutions and other forces that lie outside a business enterprise but that may influence its functioning and performance

The main features of business environment Totality of External forces General and Specific forces Interrelatedness Complexity Dynamic Uncertainty

Prepare the following questions from todayrsquos assignment

2 What do you mean by business environment

The term lsquobusiness environmentrsquo means the aggregate of all forces factors and institutions which are external to and beyond the control of an individual business enterprise but they may influence its functioning and performance Business environment is the macro framework within which a business firm a micro unit operates It consists of several interrelated and interacting elements

2 Explain the main features of business environment in brief

Totality of External forces-Business environment is the sum total of all things external to a business environment

General and Specific forces-It

Relativity

The Interrelation between business and its environment

The business enterprise is an open system It continuously interacts with its environment It takes inputs (such as raw materials capital labour energy and so on) from its environment transforms them into goods and services and sends them back to the environment

Fig 1 Business Environment Relationship

includes both the forces general forces are the economic social political legal and technological conditions which indirectly influence all business enterprise Specific forces are the investors customers competitors and suppliers which influence individual enterprise directly

Interrelatedness-Different elements of environment are interrelated for an example growing awareness for health care has increased the demand for health foods

Complexity- Business environment id complex in nature as the elements keep on changing example economic technological and other forces changes in demand for a product and service

Dynamic-Business environment is not static it keeps on changing

Uncertainty- Itrsquos very difficult to predict future events such as technology and fashion which occur fast and frequently

Business Studies

Human Resources Management

Human resource of an organisation are the aggregate of knowledge skills attitudes of people working in it

The management system which deals with human resources is called human resource management

Features of HRMbullComprehensive functionbullPeople-oriented

Question1) What do you mean by human

resource management Answer) Human resource management may be defined as that field of Management which has to do with planning organising and controlling the functions of procuring developing maintaining and utilising the labour force

bullAction oriented bullPervasive function bullContinuous function

2) Explain the features of HRM in brief

Answer)bullHuman Resource Management is concerned with managing people at work bull Human Resource Management is concerned with employees which bring people and organisations together so that the goals of each are met bullHuman resource management considered every employees as an individual and also promote their satisfaction and growth bull Human resource management is inherent in all organisations and at all levelsbullManagement of human resources are ongoing on never ending process which requires a constant alertness and Awareness of human relations

3) ldquoHR function is said to be pervasiverdquowhy

Answer) Human resource management is required in all organisations whether it is private or government organisations armed forces sports organisations etc It permeatsall the functional areas like production marketing finance research etc This from this feature of human resource management it can be said that it is pervasive in nature

Economics Demand Q1DEFINITION OF DEMANDIn economics demand is the quantity of a good that consumers are willing and able to purchase at various prices during a given period of timeQ2DEMAND CURVEIn economics a demand curve is a graph depicting the relationship between the price of a certain commodity and the quantity of that commodity that is demanded at that pricQ3LAW OF DEMANDIn microeconomics the law of demand states that conditional on all else being equal as the price of a good increases quantity demanded decreases conversely as the price of a good decreases quantity demanded increasesQ4ASSUMPTION of LAW OF DEMAND(i)No change in price of related commodities(ii) No change in income of the consumer(iii) No change in taste and preferences customs habit and fashion of the consumer( No expectation regarding future change in priceQ5MARKET DEMAND SCHEDULEIn economics a market demand schedule is a tabulation of the quantity of a good that all consumers in a market will purchase at a

given price At any given price the corresponding value on the demand schedule is the sum of all consumersrsquo quantities demanded at that priceQ6INDIVIDUAL DEMAND SCHEDULEIndividual demand schedule refers to a tabular statement showing various quantities of a commodity that a consumer is willing to buy at various levels of price during a given period of timeQ7 FACTORS AFFECTING INDIVIDUAL DEMAND FOR A COMMODITY

The factors that influence a consumerrsquos decision to purchase a commodity are also known as determinants of demand The following factors affect the individual demand for a commodity1 price of the commodity2 price of related goods3 income of buyer of the commodity4 tastes and preferences of the buyer1 Price of the CommodityYou must have observed that when price of a commodity falls you tend to buy more of it and when its price rises you tend to buy less of it when all other factors remain constant (lsquoother things remaining the samersquo) In other words other things remaining the same there is an inverse relationship between the price of a commodity and its quantity demanded by its buyers This statement is in accordance with law of demand which you will study in the later part of this lesson Price of a commodity and its quantity demanded by its buyers are inversely related only when lsquoother things remain the samersquo So lsquoother things remaining the samersquo is an assumption when we study the effect of changes in the price of a commodity on its quantity demanded2 Price of Related goodsA consumer may demand a particular good But while buying that good heshe also asks the price of its related goods Related goods can be of two types-(i) Substitute goods(ii) Complementary goods While purchasing a good prices of its substitutes and complements do affect its quantity purchased(i) Price of Substitute Goods Substitute goods are those goods which can easily be used in place of one another for satisfaction of a particular want like tea and coffee An increase in price of substitute good leads to an increase in demand for the given commodity and a decrease in price of substitute good leads to a decrease in demand for the given commodity It means demand for a given commodity is directly affected by change in price of substitute goods For example if price of coffee increases the demand for tea will rise as tea will become relatively cheaper in comparison to coffee(ii) Price of Complementary goods Complementary goods are those goods which are used together to satisfy a particular want like car and petrol An increase in the price of complementary goods leads to a decrease in demand for the given commodity and a decrease in the price of complementary goods leads to an increase in demand for the given commodity For example if price of petrol falls then the demand for cars will increase as it will be relatively cheaper to use both the goods together So demand for a given commodity is inversely affected by change in price of complementary goods3 Income of the Buyer of CommodityDemand for a commodity is also affected by income of its buyer However the effect of change in income on demand depends on the nature of the commodity under consideration In case of some goods like full cream milk fine quality of rice (Basmati rice) etc demand for these commodities increases when income of the buyer increases and

demand for these commodities decreases when income of the buyer decreases Such goods whose demand increases with the increase in income of the buyer are called normal goods But there are some goods like coarse rice toned milk etc whose demand decreases when income of buyer increases and their demand increases when income of the buyer decreases Such goods whose demand decreases with the increase in income of the buyer are called inferior goods Suppose a consumer buys 10 Kgs of rice whose price is ` 25 per Kg He cannot afford to buy better quality of rice because the price of such rice is ` 50 per Kg The consumer is spending ` 250 per month on the purchase of rice Now if income of the consumer increases and he can afford ` 350 on purchase of 10 Kg of rice Now he can afford to buy some quantity of rice say 6 Kgs whose price is ` 25 per Kg and may buy 4 Kgs of rice whose price is ` 50 per Kg Thus he will buy 10 Kgs of rice by spending ` 350 per month Therefore we may conclude that demand for normal goods is directly related to the income of the buyer but demand for inferior goods is inversely related to the income of the buyer4 Tastes and Preferences of the BuyerThe demand for a commodity is also affected by the tastes and preferences of the buyers They include change in fashion customs habits etc Those commodities are preferred by the consumers which are in fashion So demand for those commodities rises which are in fashion On the other hand if a commodity goes out of the fashion its demand falls because no consumer will like to buy it(5) Number of Buyers in the Market(Population)Increase in population raises the market demand whereas decrease in population reduces the market demand for a commodity Not only the size of population but its composition like age (ratio of males females children and old people in population) also affects the demand for a commodity It is because of needs of children young old male and female population differs(6) Distribution of Income and WealthIf the distribution of income and wealth is more in favour of the rich demand for the commodities preferred by the rich such as comforts and luxuries is likely to be higher On the other hand if the distribution of income and wealth is more in favour of poor demand for commodities preferred by the poor such as necessities will be more(7) Season and Weather ConditionsThis is generally observed that the demand for woolens increases during winter whereas demand for ice creams and cold drinks increases during summer Similarly market demand for umbrellas rain coats increases during rainy seasonQ8 REASONS FOR OPERATION OF LAW OF DEMAND WHY DEMAND CURVE SLOPES DOWNWARDNow we will try to explain why does a consumer purchase more quantity of a commodity at a lower price and less of it at a higher price or why does the law of demand operate ie why does the demand curve slope downwards from left to right The main reasons for operation of law of demand are1 Law of Diminishing Marginal UtilityAs you have studied earlier law of diminishing marginal utility states that as we consume more and more units of a commodity the utility derived from each successive unit goes on decreasing The consumer will be ready to pay more for those units which provide him more utility and less for those which provide him less utility It implies that he will purchase more only when the price of the commodity falls2 Income Effect

When price of a commodity falls purchasing power or real income of the consumer increases which enables him to purchase more quantity of the commodity with the same money income Let us take an example Suppose you buy 4 ice creams when price of each ice cream is ` 25 If price of ice creams falls to ` 20 then with same money income you can buy 5 ice creams now3 Substitution EffectWhen price of a commodity falls it becomes comparatively cheaper as compared to its substitutes (although price of substitutes has not been changed) This will lead to rise in demand for the given commodity For example if coke and Pepsi both are sold at ` 10 each and price of coke falls Now coke has become relatively cheaper and will be substituted for Pepsi It will lead to rise in demand for coke4 Change in Number of BuyersWhen price of a commodity falls some old buyers may demand more of the commodity at the reduced price and some new buyers may also start buying this commodity who were not in a position to buy it earlier due to higher price This will lead to increase in number of buyers when price of the commodity falls As a result demand for the commodity rises when its price falls5 Diverse Uses of a CommoditySome commodities have diverse uses like milk It can be used for drinking for sweet preparation for ice cream preparation etc If price of milk rises its use may be restricted to important purpose only This will lead to reduction in demand for other less important uses When price of milk falls it can be put to other uses also leading to rise n demand for itQ9 EXCEPTIONS TO THE LAW OF DEMANDYou have studied in law of demand that a buyer is willing to buy more quantity of a commodity at a lower price and less of it at a higher price But in certain circumstances a rise in price may lead to rise in demand These circumstances are called Exceptions to the Law of Demand Some important exceptions are1 Giffen GoodsGiffen goods are special type of inferior goods in which negative income effect is stronger than negative substitution effect Giffen goods do not follow law of demand as their demand rises when their price rises Examples of Giffen goods are jowar and bajra etc2 Status Symbol GoodsSome goods are used by rich people as status symbols eg diamonds gold jewellary etc The higher the price the higher will be the demand for these goods When price of such goods falls these goods are no longer looked at as status symbol goods and tehrefore therir demand falls3 NecessitiesCommodities such as medicines salt wheat etc do not follow law of demandbecause we have to purchase them in minimum required quantity whatever their price may be4 Goods Expected to be ScarceWhen the buyers expect a scarcity of a particular good in near future they start buying more and more of that good even if their prices are rising For example during war famines etc people tend to buy more of some goods even at higher prices due to fear of their scarcity in near future

Political Science

Constitution of India-The

Preamble

The preamble-

Preamble-

The preamble is the most precious part of the constitution We the people of India having solemnly resolved to constitute India into a Sovereign Socialist Secular Democratic Republic and to secure to all its citizensA preamble is an introductory and expressionary statement in a document that explains the documents purpose and underlying philosophy When applied to the opening paragraphs of a statute it may recite historical facts pertinent to the subject of the statuteNature and purpose of the constitution-Purpose of the Constitution dictates permanent framework of the government to form a more perfect union to establish justice and ensure peace of thenationconstitution provide principles how the government can run itself following the rules and laws written in the constitution of each state keeps them balanced

Answer the following questions-

1 What is preambleA preamble is an introductory and expressionary statement in a document that explains the documents purpose and underlying philosophy2 What is the nature and

purpose of the constitutionConstitution dictatespermanent framework of the government to form a more perfect union to establish justice and ensure peace of the nation

Homework-Learn

Accounts Compatibilty mode

1MEANING OF PARTNERSHIPPartnership is a form of business organisation where two or more persons join hands to run a business They share the profits and losses according to the agreement amongst them According to the Indian Partnership Act 1932 ldquoPartnership is relation between persons who have agreed to share profits of a business carried on by all or any one of them acting for allrdquo For example one of your friends has passed class XII from National Institute of Open Schooling (NIOS) and wants to start a business Heshe approaches you to join in this venture Heshe wants you to contribute some money and participate in the business activities Both of you if join hands constitute a partnership2CHARACTERISTICS1048698 Agreement A partnership is formed by an agreement The agreement may be either oral or in writing It defines the relationship between the persons who agree to carry on business It may contain the terms of sharing profit and the capital to be invested by each partner etc The written agreement is known as partnership deed1048698 Number of persons There must be at least two persons to form a partnership

The maximum number of partners in a partnership firm can be 50 according toCompanies Act 20131048698 Business The Partnership is formed to carry on business with a purpose of earning profits The business should be lawful Thus if two or more persons agree to carry on unlawful activities it will not be termed as partnership1048698 Sharing Profits The partners agree to share profits in the agreed ratio In caseof loss all the partners have to bear it in the same agreed profit sharing ratio10486981048698Mutual Agency Every partner is an agent of the other partners Every partner can bind the firm and all other partners by hisher acts Each partner will be responsible and liable for the acts of all other partners10486981048698Unlimited liability The liability of each partner except that of a minor is unlimited Their liability extends to their personal assets also If the assets of the firm are insufficient to pay off its debts the partnersrsquo personal property can be used to satisfy the claim of the creditors of the partnership firm10486981048698Management All the partners have a right to mange the business However they may authorize one or more partners to manage the affairs of the business on their behalf10486981048698Transferability of Share No partner can transfer hisher share to any one including hisher family member without the consent of all other partners3PARTNERSHIP DEEDAgreement forms the basis of partnership The written form of the agreement is which a document of partnership is It contains terms and conditions regarding the conduct of the business It also explains relationship between the partners This document is called partnership deed Every firm can frame its own partnership deed in which the rights duties and liabilities of the partners are stated in detail It helps in settling the disputes arising among the partners during the general conduct of business 4CONTENTS OF PARTNERSHIP DEEDThe partnership deed generally contains the following (i) Name and address of the partnership firm(ii) Nature and objectives of the business(iii) Name and address of each partner(iv) Ratio in which profits is to be shared(v) Capital contribution by each partner(vi) Rate of Interest on capital if allowed(vii) Salary or any other remuneration to partners if allowed(viii) Rate of interest on loans and advances by a partner to the firm(ix) Drawings of partners and interest thereon if any(x) Method of valuation of goodwill and revaluation of assets and liabilities on the reconstitution of the partnership ie on the admission retirement or death of a partner(xi) Settlement of disputes by arbitration(xii) Settlement of accounts at the time of retirement or death of a partner5IN ABSENCE OF PARTNERSHIP DEEDThe partnership deed lays down the terms and conditions of partnership in regard to rights duties and obligations of the partners In the absence of partnership deed there may arise a controversy on certain issues like profit sharing ratio interest on

capital interest on drawings interest on loan and salary of the partners In such cases the provisions of the Indian Partnership Act becomes applicableSome of the Issues are(i) Distribution of Profit Partners are entitled to share profits equally(ii) Interest on Capital Interest on capital is not allowed(iii) Interest on Drawings No interest on drawing of the partners is to be charged(iv) Interest on Partnerrsquos Loan A Partner is allowed interest 6 per annum on the amount of loan given to the firm by himher(v) Salary and Commission to Partner A partner is not entitled to anysalary or commission or any other remuneration for managing the business

History TOPIC-TOWARDS INDEPENDENCE AND PARTITION THE LAST PHASE (1935-1947)

SUB TOPIC-IMPORTANT POLITICAL DEVELOPMENTS ndash GROWTH OF SOCIAL IDEAS

Socialism is a political social and economic philosophyLike in other parts of the world the Russian revolution of 1917 served as a great inspiration for revolutionaries in India who at that time were engaged in the struggle for liberation from British ruleSocialist ideas led to the formation of communist party of IndiaJAWAHARLAL NEHRU Among the early Congress leaders Jawaharlal Nehru was very much impressed and influenced by the Socialist ideas He also learnt about the Economic activities of the Soviet Union after the Bolshevic Revolution 1917 He made full use of them in IndiaThe election of Jawaharlal Nehru and Subhas Chandra Bose showed the Left wing tendency within CongressJawaharlal Nehru demanded economic freedom along with political freedom of the people in order to end the exploitation of masses

Nehrus working committee included three socialists leaders The Lucknow session was a landmark in the evolution of socialist ideas of the congressSUBHAS CHANDRA BOSE ndash Subhas Chandra Bose had socialist leaning Both Jawaharlal Nehru and Subhas Chandra Bose were known as leftist Congress men Later on National Congress divided into Leftist and rightist campCONGRESS SOCIALIST Within the Congress some leaders formed the Congress Socialist partyPattavi Sitaramyya Sardar Patel Rajendra Prasad had hostile attitude towards the Congress Socialist partyJawaharlals attitude was hesitant

1 QUESTION ndash Mention name of two Congress leaders who had socialist leaning

1ANSWER ndash Subhas Chandra Bose and Jawaharlal Nehru2QUESTION- In which session of the congress Jawaharlal elaborated his Socialist ideas2 ANSWER ndash Lucknow and Faizpur Session in December 1935 and 19363QUESTION ndash Why Congress was sharply divided into leftist and rightist camp 3ANSWER ndash Subhas Chandra Bosersquos attempt to seek re election for congress presidentship in 1939sharply divided the National Congress into Leftist and Rightist camp4 QUESTION ndash Who was MN Roy 4 ANSWER ndash Manabendra Roy first formed the Communist Party of India outside the country at Tashkent in 19205QUESTION ndash Who formed the Congress Socialist Party within the Congress5 ANSWER ndash Jaya Prakash Narayan Achyut Patwardhan Acharya Narendra Dev Ram Mohan Lohia Aruna Asaf Ali6QUESTION ndash When was the Congress Socialist Party formed What was its object6 ANSWER ndash 1934The Congress Socialist Party sought to work out socialist programme through the Congress They joined hands with the Congress and wanted to carry

Subhas Chandra Bose being expelled from the congress after the Tripuri rift he formed Forward BlockThere were basic differences between the Congress Socialists and the communistsTRADE UNION ACTIVITIES Maximum working class people lived in Bombay and Calcutta The working and living conditions of those workers were very miserable In this situation Shasipada Banerjee NM Lokhande protested against the oppression of the working class peopleThe first Trade Union Madras Labour Union was formed in 1918 by BP WadiaIndustrial strikes took place in Kanpur Calcutta Madras Jamshedpur and Ahmedabad AITUC was formed in Bombay in 1927 The growth of Trade union among the workers was slow because of the fear of the dismissal of the jobIn the mean time the Moderates as well as Communists left AITUC and formed separate organization

on National struggle with the help of workers and peasant class of the society7 QUESTION ndash What was the name of the party founded by Subhas Chandra Bose7 ANSWER- Forward Block8QUESTION ndash Who was Shasipada Banerjee8 ANSWER ndash Shasipada Banerjee was a radical Brahmo He founded a working menrsquos club to protest against exploitation of the British rulers towards the working class of India9 QUESTION ndash What was the weekly published by NM Lokhande9ANSWER- Dinabandhu10 QUESTION ndash Who founded Bombay Mill-Hands Association and in which year10 ANSWER- NM Lokhande in189011 QUESTION- Who was BP WadiaANSWER- BPWadia was the founder of Madras Labour Union in191812 QUESTION- What was the name of the first labour union of India12 ANSWER- Madras Labour Union13 QUESTION Who founded the Majur Mahajan 13 ANSWER GANDHIJI14 QUESTION What was the full form of AITUC When it was formed14 ANSWER All India Trade Union Congressin 192715QUESTION Who formed the Red Trade Union Congress and in which year15ANSWER The Communists formed the Red Trade Union Congress16 QUESTION What do you mean by Socialism16 ANSWER Socialism describes any political and economic theory that says the community rather than individuals should own and manage property and natural resources

Subject Eng Literature (The Tempest ndash William Shakespeare) Topic Act III Scene 3 Lines 1 to 52 (Line 52 ndash Brother my lord the Duke Stand to and do as we) Date 13th April 2020 (4th Period)

[Students should read the original play and also the paraphrase given in the school prescribed textbook]Summary Questions amp Answers

o Alonso Sebastian Antonio Gonzalo Adrian Francisco and others wandered about the island in search of Ferdinand and gets tired and hungry of the toil and at the same time gives up all hope of finding him

o Antonio and Sebastian are happy that Alonso is out of hope and decide to make another attempt on his life that night when being so tired they will be sleeping soundly

o Suddenly a solemn and strange music is heard in the air and several strange shapes enter bringing in a banquet These strange shapes then dance round it with gestures of salutation and then inviting the King to eat they depart

o Seeing this strange scene all are inclined to believe the tales told by travelers that there truly are ldquounicornsrdquo and ldquothe phoenixrsquo thronerdquo

1 ALONSO What harmony is this My good friends hark (L18-27)

GONZALO Marvellous sweet music

[Enter several strange shapes bringing in a banquet

they dance about it with gentle actions of salutation

and inviting the King and his companions to eat they depart]ALONSO Give us kind keepers heavens What were theseSEBASTIAN A living drollery Now I will believe

That there are unicorns that in Arabia

There is one tree the phoenixrsquo throne one phoenix

At this hour reigning thereANTONIO Ill believe both

And what does else want credit come to me

And Ill be sworn rsquotis true Travellers neer did lie

Though fools at home condemn rsquoem

(a) How did Prospero present an amazing spectacle before Alonso and his companions

Using his magic powers Prospero ordered strange shapes to lay a banquet before Alonso and his companions The shapes brought several dishes with tasty eatables in them They placed the dishes on a table before Alonso and his companions Then the strange shapes began to dance gracefully around the banquet While dancing they made gestures inviting them to eat the food Then suddenly the shapes disappeared(b) Who were the guests at the strange banquet Describe the lsquoliving drolleryrsquo

Alonso Sebastian Antonio Gonzalo Adrian and Francisco were the guests at the strange banquet

The term ldquoliving drolleryrdquo refers to live entertainment show In this context when Alonso the King of Naples Sebastian his brother Antonio the treacherous brother of Prospero Gonzalo the kind and loyal councillor to the King Adrian and Francisco came to the island they were hungry and weary in their spirits They heard a solemn and strange music They were shocked to see several strange shapes bringing in a banquet and these shapes danced about it with gentle action of salutation inviting the King and his companions to eat After this Sebastian described this show as lsquoliving drolleryrsquo(c) What is lsquophoenixrsquo What are lsquoUnicornsrdquo

The term lsquophoenixrsquo refers to a mythical Arabian bird which lived alone and perched on a solitary tree After one hundred years it expired in flames and rose again from its own ashes

lsquoUnicornsrsquo refers to the mythological four-footed beasts having horns in the centre of their foreheads When the horns are ground into powder the powder was believed to be

an aphrodisiac(d) How does Sebastian explain the puppet show OR Why does the speaker now believe in unicorns and phoenix

Sebastian finds several strange shapes bringing in the banquet They invite the king and his party for dinner and soon depart He tells that if such a strange sight can be a reality there is nothing incredible in the world and from the present moment he will believe anything He says that it is a strange dumb show enacted not by puppets but by living beings It is stranger than a travellerrsquos tale Seeing such a thing

before his own eyes he will no longer disbelieve the story about unicorns and phoenix(e) How do the other characters present respond to this living drollery

At the sight of the lsquoliving drolleryrsquo like Sebastian Gonzalo and Antonio too acted strangely Antonio told that he too now believes in unicorns and phoenix and anything else that seems to be incredible He too now believes in travellersrsquo tales Gonzalo told that if he would report those happenings in Naples nobody will believe him He considers that those gentle shapes were gentler in manner in comparison to the living beings Alonso was at first sight suspicious and told them that those strange shapes conveyed their meaning in expressive gestures when they seemed to lack speech by their movements and sounds Francisco was amazed at their mysterious disappearance

2 ALONSO Not I

(Line 43-52)GONZALO Faith sir you need not fear When we

were boysWho would believe that there were mountaineers

Dewlapped like bulls whose throats had hanging at rsquoem

Wallets of flesh Or that there were such men

Whose heads stood in their breasts Which now we find

Each putter-out of five for one will bring us

Good warrant ofALONSO I will stand to and feed

Although my lastmdashno matter since I feel

The best is past Brother my lord the Duke

Stand to and do as we

(a) How does Alonso respond at the spectacle of the shapes which were sent to them at the instruction of Prospero

After seeing the strange sight of appearing and disappearing of the shapes sent by Prospero to arrange a banquet for them Alonso says that his surprise at having seen those creatures is infinite and he is fully justified in feeling so much surprise He thinks that their shapes their gestures and the sounds they made were indeed amazing Although they do not possess the gift of speech yet they were able to convey their

thoughts by means of their gestures only

(b) What does Prospero say about the views expressed by Alonso regarding the shapes What does Francisco think about the shapesAfter hearing Alonsorsquos views about the shapes Prospero says that this manrsquos praise of the spirits is rather hasty He means to say that Alonso has shown great haste in reaching the conclusion about the shapes Francisco is amazed to see that those shapes disappeared in a mysterious way(c) What does Sebastian ask Alonso to doSebastian tells Alonso that the shapes having disappeared should not matter to them because they have left the eatables behind He asks Alonso to enjoy eating as they are extremely hungry but the king does not accept his offer of enjoying the dishes(d) How does Gonzalo try to dispel Alonsorsquos fear of those strange shapes What kind of references does he give to AlonsoGonzalo says that those who have travelled abroad have reported seeing even stranger sights than these shapes that Alonso and his companions have beheld Hence there is no reason to feel afraid of these shapes Gonzalo further adds that in his younger days he had heard strange stories from travelers and Alonso might have heard similar stories For instance it was said that there existed a certain race of

human beings who had huge lumps of flesh hanging at their throats and who therefore resembled bulls Then Gonzalo tells about a race of human beings whose heads were located at their breasts Gonzalo says that such stories were not believed by most people in those days but now-a-days these stories have become common(e) Explain the following lsquoEach putter-out of five for onersquoEnglish travellers often insured their trips with London brokers Those that went on foreign travels those days used to deposit a certain amount with some firm or company in London before their departure If the travelers failed to return the money was forfeited by the company with which it had been deposited But this money was repaid five-fold if the travelers returned safe and sound In this way a traveler stood a great chance of recovering the entire cost of his

travels(f) Give the explanatory meanings of the following expressions in the context of the above extract (i) Dewlapped (ii) Wallets of flesh

(iii) Putter-out(i) Dewlapped having big lumps of flesh at the necks(ii) Wallets of flesh large masses of flesh looking like bags(iii) Putter-out to invest money before commencing the travel

  • General methods of preparation of hydrogen
  • Chapter Dimensional Analysis (Summary)
    • Properties of Charge
Page 27:   · Web viewSubject. Topic. Summary. Execution. Hindi. व्याकरण. शरीरके अंगो के नाम लिखिए. 1) आँख 2) नाक 3

of rotation

The SI unit of moment of force is Newton times meter

= Newton meter (Nm)

Commercial Studies

Stake holders In this topic you will be come to know about the meaning and concept of stakeholders

How stakeholders are different from shareholders

Questions1 What do you mean by the term stake holdersAnswer) The term stake holders have developed from the words which mean an interest or expected benefit Stakeholders mean all those individuals groups and Institutions which have a state (interest) in the functioning and performance of a commercial organisation or a business enterprise2 What do you mean by share holdersAnswer) The person and Groups who own the shares of the joint stock company by providing capital to the company are called shareholders Shareholders are the internal stakeholders shareholders are one out of several stake holders3 How are shareholders different from stakeholdersAnswer)i) The term shareholders is related to only joint stock company whereas stakeholders are related with all business organisationsii) Stakeholders maybe any individual having financial stake in business organisation whereas a shareholders are those individuals who are holding shares in the company4) How are shareholders different from creditorsAnswer) i) Shareholders are internal stakeholders while creditors are external stakeholdersii) Shareholders invest in the capital of the company whereas creditors give loan to the companyiii) Shareholders are the members of the company with voting rights but creditors are not the members of the company

English 1 Transformation of sentences

Sentences A sentence is a group of words which makes complete sense

e Assertive sentencesf Imperative sentencesg Interrogative sentencesh Exclamatory sentences

Sentences can be changed from one grammatical form to another without changing the meaning of the sentence This is known as transformation of sentences

Exercise 1 Change the following affirmative sentences into Negative sentences

a He is a good manHe is not a bad man

b Ram loves SitaRam is not without love for Sita

c Only he stood first in the classNone but he stood first in the class

d Ankit was wiser than he

He was not so wise as Ankite He did it

He did not fail to do itf As soon as I reached college the

bell rangNo sooner did I reach college than the bell rang

g He finished everythingHe left nothing unfinished

h It always pours when it rainsIt never rains but it pours

Math Topic Commercial MathematicsChapter ndash Goods and services Tax

What is GSTAns It is a abbreviated term of Goods and Service Text which is an indirect tax levied on the sale of goods and rendering servicesSome terms related to GSTDelar Any person who buys goods or services For resale is known as a delar A delar Can be a firm or a companyIntra-state sales Sales of goods and services within the same state or same union territory are called intra- state salesInter-state sales Sales of goods and services outside the state or union territory are called Inter-state sales4) Input GST GST is paid by dealers on purchase of goods and services are called input GST5) Output GST GST is collected from customers on sale of goods and services are called output GST6) Types of GST There are three taxes applicable under GST(i) Central Goods and Services Tax (CGST)(ii) State Goods and Services Tax (SGST) or Union Territory Goods and Services Tax (UTGST) Both these taxes are levied on intra-state sales Here GST is divided equally among central and state governments(iii) Integrated Goods and Services Tax (IGST) IGST is levied on inter- state sales It is also levied on import of goods and services into India and export of goods and services from India

Subject Eng Literature (The Merchant of Venice ndash William Shakespeare)Topic Act III Scene 4 Lines 1 to 44 (Portia hellip To wish it back on you fare you well Jessica)[Students should read the original play and also the paraphrase given in the school prescribed textbook]

Summary Questions amp AnswersIn this scene we suddenly find a new element in the character of Portia We have already seen her possessed of every graceful womanly quality but now she shows that she is capable of rapid decision and determined action She shows this by her sudden resolve to hasten to Venice with a daring scheme for the rescue of Antonio This is an important scene in the dramatic action for it leads up to and renders possible the striking events of the famous trial scene which is one of the greatest striking elements of the play Moreover the fact that all the characters of importance are now assembled together in Venice makes the union of the main plot and the secondary story complete

(1) LORENZO Madam although I speak it in your presence(Line 1-9)

You have a noble and a true conceit

Of god-like amity which appears most strongly

In bearing thus the absence of your lordBut if you knew to whom you show this honourHow true a gentleman you send reliefHow dear a lover of my lord your husbandI know you would be prouder of the workThan customary bounty can enforce you

(a) Where is Lorenzo Why is he here To whom is he referring as lsquoMadamrsquo

Lorenzo is at Portiarsquos residence He had met Salerio on the way and Salerio had begged him to come along with him to

o In this scene Portia Nerissa Lorenzo Jessica and Balthazar appear

o Portia requests Lorenzo and Jessica to be in charge of her house during her absence from Belmont because she and Nerissa have decided to spend the days in meditation and also in visiting the holy places in the neighbourhood of Belmont She has already instructed her people to acknowledge both Lorenzo and Jessica as master and mistress of house during her absence Lorenzo and Jessica gladly agree to look after the house of Portia

handover the letter from Antonio to Bassanio The letter carried the bad news about Antoniorsquos arrest for non-payment of loan taken from Shylock Hence Salerio might have preferred company to break this bad news to Bassanio He is referring to Portia as Madam(b) What does Portia say on hearing the above extract

Portia says that she has never regretted doing good to others Friends who spend a lot of time together and really are there for each other have many traits in common As Antonio is Bassaniorsquos best friend saving him is like saving Bassanio who is like her own soul She asks Lorenzo to take care of management of the house till Bassanio is back(c) What does Portia send with Bassanio and why

On hearing about Antoniorsquos troubles on account of Bassanio her husband Portia immediately sends him with enough gold to repay the debt many times over to Venice to help Antonio out of his misfortune

(2) Lorenzo Madam with all my heart (Line 36-40)

I shall obey you in all fair commands

Portia My people do already know my mindAnd will acknowledge you and JessicaIn place of Lord Bassanio and myselfSo fare you well till we shall meet again

(a) Where are Lorenzo and Portia at this time What lsquofair commandsrsquo are given to Lorenzo

Lorenzo and Portia are at Belmont during this scenePortia reveals to Lorenzo that she has sworn to contemplate in prayer at a monastery around two miles away until her husband returns from Venice She tells him that Nerissa would accompany her and asks him to manage the house with Jessica till things are settled In response Lorenzo tells her that he would be obliged to do whatever she asks him to do(b) Where is Portia actually going and why

Portia tells Lorenzo that she would live a life of contemplation and pray at a monastery which is two miles away from her place In reality Portia plans to go to Venice in disguise with Nerissa and argue the case in defense of Antonio She is very sure that her plan would succeed

ClassXI (ScienceHumanitiesCommerce)Subject Topic Summary Execution

Computer Science

(APC)

Ch ndash 1 Numbers

(Numbers in different bases and

their Arithmatical operations)

Number System In computers Number System is defined as a writing system to represent the numbers in different ways ie we are using different symbols and notations to represent numbers There are four ways we can represent the number ndash Binary Decimal Octal and Hexadecimal

Decimal Number SystemThis number system consist 10 digits These are 0 1 2 3 4 5 6 7 8 amp 9

Binary Number SystemThis number system has only two digits these are 0 and 1 Here 0 stands for off while 1 stands for on

Octal Number SystemThis number system has 8 digits these are 0 1 2 3 4 5 6 amp 7

Hexadecimal Number SystemThis number system has 16 digits these are 0 1 2 3 4 5 6 7 8 9 A B C D E F Here the value of the alphabets are as follows A=10 B=11 C=12 D=13 E=14 F=15

Rules for conversion decimal number to Binary1 Divide the decimal number by 22 If the number will not divide equally by 2 then round down the answer to the nearest whole number (integer)3 Keep a note of the remainder it should be either 0 or 14 Keep repeating the above steps dividing each answer by 2 until you reach zero5 Write out all the remainders from bottom to top This is your binary solution

For example Lets convert 32 to binary 2 32 2 16 - 0 2 8 - 0 2 4 - 0 2 2 - 0 2 1 - 0 0 - 1

The binary equivalent of 3210 is 1000002

Try the follwing youself1 2410

2 4810

3 1210

History GROWTH OF NATIONALISM

The second half of the 19th century witnessed growth of political consciousness and a sense of Nationalism among the IndiansThere were various factors for growth of Indian Nationalism- As a result various political associations were formed in different provinces by the educated Indians Surendranath Banerjee organized a meeting of National conference at Calcutta Ultimately the National Congress was founded in Bombay in 1885This body became the vanguard of Indian struggle for freedom The congress leaders were known as moderates because they followed a policy of prayer and petition A large number of Indian leaders had experienced in political agitation The Political situation of England was also changed Moreover increasing revolutionary activities in Maharashtra Punjab and Bengal became serious concern to the British Government In this

QUESTION1 What do you mean by Nationalism ANSWER 1 Nationalism is defined as loyalty and devotion to own nation especially a sense of national consciousnessQUESTION 2 What are the causes of nationalism ANSWER 2 There were various factors for growth of nationalism

1 Spread of western education2 The progress of vernacular press and

patriotic literature3 The economic exploitation of our

country by the colonial rulers4 International affairs

QUESTION 3 Who organized National conference in Calcutta in 1883 ANSWER 3 Surendranath BanerjeeQUESTION 4 When did Indian National Congress formANSWER 4 Indian National Congress was formed in 1885 in BombayQUESTION 5 Who were ModeratesANSWER 5 The Early Nationalists were also known as Moderates Their emergence marked

background Lord Curzon became Viceroy in India He had no respect for the Indian National Congress

the beginning of the organized national movement in India They believed in British justice and were loyal to them They followed a policy of prayer and petition They demanded constitutional reforms of our country Impotant Moderate leaders were Pherozshah Mehta Dadabhai Naorozi and Surendranath Banerjee etcQUESTION 6 What do you know about Extremism in Indian National movementANSWER 6 In the beginning of 20th century a new class of national leaders emerged in India which was different from the moderate groups They started more aggressive movement against the British empire The goal of extremists was ldquoswarajrdquo Important extremist leaders were Bal Gangadhar Tilak Lala Lajpat Rai Bipin Chandra Pal etcQUESTION 7 Mention the places which were the main centres of Revolutionary movementANSWER 7 Maharashtra Bengal and Punjab

Physics

Chapter Dimensional Analysis

(Summary)

The dimensions of a physical quantity are the powers to which the fundamental units are raised in order to obtain the derived unit of that quantit

The physical quantites lengthmasstime are represented by [L] [M] [T] resp let they are raised to powers ( dimesions) abc resp then any physical quantity can be represented by [ La Mb Tc ] Examples

1 Area area = L x B = [L] x [L] = [M0 L2 T0 ]

2 Density density = massvolume = [M][L3] = [ M L-3]

3 Velocity velocity = distancetime = [L][T] = [LT-1]HW Try to find out dimension of acceleration Acceleration = velocity timeNB One can find the SI Units Using Dimension Analysis Such as for area we have [L2] so its SI unit is m2

Biology Topic ndash Chp-1 The living world

Today we will start the first chapter the living world Here we discuss about the characteristics of living organism and what are the difference between them and nonliving substances We also discuss about the contribution of different Scientists

There are over 500000 species of plants andover a million species of animal are present on earth Some 15000 new species were discovered every yearQ1 What is a living organismbull A living organism is primarily physico -chemical material that demonstrate a high degree of complexity is capable of selfRegulation possesses a metabolism and perpetuates itself through timeQ2 What are the differences between livingand non-livingsi) Compared with non-living living organisms

have more complex organised structure and their use of energy is more controlled amp efficientii) Living things reproduce their own kind by forming new cells which contains copies of their genesiii) Each organism has some degree of homeostasisie it is able to make adjustments so that internal environment remains constantQ3 Write contributions of following Scientists i) Aristotle - One of the first theories in Biology places all living things in a hiearchieii) AV Leeuwenhoek - was the first to observe living single celled organisms under microscopeii) Carolus Linnaeus - developed the binary system for naming of organisms and classificationiii) Geregor Johann Mendel ndash discoverbasic principles of inheritanceHomework i) C Darwin ii)Schleiden

Math Trigonometric functions

1 Overviewi) Trigonometry The word lsquotrigonometryrsquo is derived from the Greek words lsquotrigonrsquo and lsquometronrsquo which means measuring the sides of a triangle An angle is the amount of rotation of a revolving line with respect to a fixed line Usually we follow two types of conventions for measuring angles ie a) Sexagesimal system b) Circular system In Sexagesimal system the unit of measurement is Degree In Circular system the unit of measurement is Radian ii) Relation between degree and radianThe ratio of circumference of a circle to its diameter is always a constant This constant ratio is a number denoted by π which is taken approximately as 227The relationship between degree amp radian measurements is as follows2 right angles = 180deg= π radians1radian = 180degπ=57deg16(approx) 1deg=π180 radianiii) Length of an arc of a circleIf an arc of length s subtends an angle θ radians at the center of a circle of radius r then s=rθiv) Area of a sector of a circleA sector is like a pizza slice of the

Q) Express the following angles in radiana) 45deg b) 40deg3730Ans a) We have 180deg=π radiansi e 45deg= πtimes45180 radian = π4 radiansb) 40deg3730= 40deg37+3060 minute= 40deg 37 +12 minute= 40deg+ 752 minute=40 + 75(2times60) degree=3258 degreeNow 180deg=π radianie 3258 degree= (πtimes325) (180times8) radians = 65π288 radiansQ) A circle has a radius of r=12 meters What is the length of an arc traced out by a 60deg angle in the center of the circleAns In this problem we know both the central angle (60deg) and the radius of the circle (12) All we have to do is plug those values into our equation and we get

s = 2π(12)(60360)s = 24π6s = 4πSo the length of an arc traced out by a 60deg angle in a circle with a radius of 12 meters equals 4π meters asymp 1257 metersQ) Find the area of the sector with a central angle 30deg and a radius of 9cmAns GivenRadius r = 9 cmAngle θ = 30degArea of the sector = θ360degtimesπr2

= 30360degtimes227times92=2121cm2

circle It consists of a region bounded by two radii and an arc lying between the radiiThe area of a sector is a fraction of the area of the circle This area is proportional to the central angle In other words the bigger the central angle the larger is the area of the sectorArea of Sector = θ2 times r2 (when θ is in radians)

Area of Sector = θ times π360 times r2 (when θ is in degrees)

COMMERCE

CLASSIFICTION OF HUMAN ACTIVITIES-ECONOMIC AND NON-ECONOMIC

Welcome to the new sessiontoday we are going to start the first chapter of Class XI The name of the chapter that we are going to start is

lsquoClassification of Human Activities ndasheconomic and non-economicrsquo

Now let us start the chapter by considering human beings and the activities they perform throughout the day

Human activities means all those activities that human beings undertake to satisfy their wants

Human wants on the other hand are the desire of human beings for goods (vegetables fruits rice etc) and services (services of doctors teachers lawyers etc) that they require to live

Now these human activities continue throughout life as human wants are unending unlimited and recurring as human beings desire for better living throughout their lives

Now human activities can be classified into two categories

Human activities

Economic activities Non-economic activities

Economic activities are

Questions1 What are human activities

Answer Human activities mean all those activities that human beings undertake to satisfy their wants

Example A man working in an office

A boy playing in the garden

2What are the characteristics of human activitiesAnswer the characteristics of human activities are as follows

Human activities are undertaken by men women and children and these activities involve human efforts

Human activities are undertaken to satisfy human wants which are unlimited

Human activities continue throughout life

Human activities are performed for both earning money and personal satisfaction

3What is economic activitiesGive example

Answer Economic activities are undertaken by human beings with the object of earning money acquiring wealth and thereby satisfying human wantsExample

Selling of goods by a shop keeper to his customer

A clinic run by a doctor Service of a teacher in school or college

undertaken by human beings with the object of earning money and acquiring wealth

These activities result in the production of economic goods and services

Example Human activities(ie working in factories officesshops) which produce direct economic benefits

Non-economic activities are inspired by human sentiments and emotions such as love for the family desire to help the poor and love for the country

Thus these human activities (eg praying playing sleeping) produce no direct economic benefits and they are also not related to earning money and acquiring wealth

4 What are the characteristics of economic activities

Answer The characteristics of economic activities are as follows

Economic motiveEconomic activities are undertaken to earn money and acquire wealth

ProductiveEconomic activities involve productiondistribution and exchange of goods and services to create wealth

Economic growthEconomic activities determine the level of economic development of a country and standard of living of its citizens

Socially desirableEconomic activities are socially desirable for society

Economic resourcesEconomic activities make use of all the economic resources such landlabourcapital etc

5 What do you mean by non-economic activitiesExampleAnswerNon-economic activities are inspired by human sentiments and emotions such as love for the family desire to help the poor and love for the countryThese activities are not undertaken for monetary gain but for onersquos satisfaction and happinessExample

a mother looks after her children

a student donates blood8 Differentiate between Economic activities and Non-economic activities

Economic activities

Non-economic activities

1to earn living and acquiring wealth2Result can be measured in terms of money

3ExampleBusinessprofession and employment

1 to obtain some satisfaction

2Result cannot be measured in terms of money

3ExampleFamily-orientedreligious socialCultural and national

BUSINESS STUDIES

BUSINESS ENVIRONMENT

Welcome to the new sessionToday we are going to start the first chapter and the name of the chapter is Business Environment

In todayrsquos world every business enterprise is a part of the society It exists and operates in association with various groups in society such as customers suppliers competitors banks and financial institutions government agencies trade unions media and so on All these groups influence the functioning of business in one way or the other They constitute the environment of businessConcept of Business Environment

The term lsquobusiness environmentrsquo refers to the sum total of all individuals institutions and other forces that lie outside a business enterprise but that may influence its functioning and performance

The main features of business environment

Totality of External forces General and Specific forces Interrelatedness Complexity Dynamic Uncertainty Relativity

The Interrelation between business and its environment

The business enterprise is an open system It continuously interacts with its environment It takes inputs

Prepare the following questions from todayrsquos assignment

1 What do you mean by business environment

The term lsquobusiness environmentrsquo means the aggregate of all forces factors and institutions which are external to and beyond the control of an individual business enterprise but they may influence its functioning and performance Business environment is the macro framework within which a business firm a micro unit operates It consists of several interrelated and interacting elements

2 Explain the main features of business environment in brief

Totality of External forces-Business environment is the sum total of all things external to a business environment

General and Specific forces-It includes both the forces general forces are the economic social political legal and technological conditions which indirectly influence all business enterprise Specific forces are the investors customers competitors and suppliers which influence individual enterprise directly

Interrelatedness-Different elements of environment are interrelated for an example growing awareness for health care has increased the demand for health foods

Complexity- Business environment id

(such as raw materials capital labour energy and so on) from its environment transforms them into goods and services and sends them back to the environment

Fig 1 Business Environment Relationship

complex in nature as the elements keep on changing example economic technological and other forces changes in demand for a product and service

Dynamic-Business environment is not static it keeps on changing

Uncertainty- Itrsquos very difficult to predict future events such as technology and fashion which occur fast and frequently

Economics Basic Economic ConceptsSub topic

Microeconomics and

Macroeconomics

Welcome to the new sessiontoday we are going to start the first chapter of Class XI The name of the chapter that we are going to start is Basic Economic concepts

Now Economics covers the study of human activities Human activities are those activities which are performed by humans to satisfy their wants

Thus Human wants are unlimited and therefore economic activities such as production exchange and consumption are needed in order to satisfy those wants

The study of economics is divided largely in two parts which areMicroeconomics and Macroeconomics

SUBJECT- MATTER OF ECONOMICS

MICROECONOMICS MACROECONOMICS

Questions1Who has coined the words micro and macro economics

Answer Ranger Frisch coined the words lsquomicrorsquo and lsquomacrorsquo in 1933 to denote the two branches of economic theory namely microeconomics and macroeconomics

2What is microeconomicsAnswer It is the study of behaviour of individual decision ndash making unit such as consumers firms etc

3 What is macroeconomicsAnswer Macroeonomics is the study of overall economic phenomena like employment national income etc

4 What is the importance of microeconomicsAnswer

Microeconomics helps in formulating economic policies which enhance productive efficiency and results in greater social welfare

It helps the government in formulating correct price policies

It explains the working of a capitalistic economy where individual units(producers and consumers ) are free to take their own decision

Micro means a small part in

microeconomics we do not study the whole economy Hence we study an individual consumer and his or her choices and a producer and his or her profit maximizing decisions in the market Thus it does not mirror what happens in the economy as a whole

Macroeconomics on the other hand studies the economy as a whole It is concerned with aggregate and depicts the entire picture of the economyMacroeconomics deals with the national income aggregate investment aggregate consumption etc

Features of Microeconomics It deals with small

parts of the country Hence it looks at

individual consumers firms and industries

It deals with individual income consumption and savings

It studies the determination of price of any product or factors of production

It deals with the working of market via the price mechanism which is nothing but the determination of price and quantity of a commodity by the forces of demand and supply

Features of Macroeconomics

It deals with the study of the economy as a whole

It is concerned with

5 Give a limitation of microeconomics Microeconomics fails to explain the

functioning of an economy as a whole It cannot explain unemployment illiteracy and other problems prevailing in the country

6 What is the importance of macroeconomics It gives overall view of the growing

complexities of an economic system It provides the basic and logical

framework for formulating appropriate macroeconomic policies (eg for inflation poverty etc )to direct and regulate economy towards desirable goals

7What is the limitation of macroeconomics It ignores structural changes in an

individual unit of the aggregate

8 Differentiate between Microeconomics and Macroeconomics

Microeconomics Macroeconomics

the study of aggregates

National income aggregate savings and aggregate investments are major concepts dealt within macroeconomics style

It studies the determination of general price levels

It investigates into the problem of unemployment and the achievement of employment

It studies the aspect of decision making at the aggregate and national levels

It includes all growth theories whether related to developed or developing economies it also includes the study of economic systems and the working of the economy under different systems

Note Both Micro and macro economics are complementary and should be fully utilized for proper understanding of an economy

1It studies economic aspect of an individual unit2It deals with individual incomeConsumption and savings

3 It facilitates determination of price of any product or factors of production

4 Itrsquos scope is narrow and restricted to individual unit

1It studies the economy as a whole

2It deals with the national income aggregate consumption and aggregate savings3 It facilitates determination of general price level in an economy

4 Itrsquos scope is wide as it deals with economic units on the national level

ACCOUNTS

Introduction to Accounting and Book-keeping

Today I am going to share you the meaning of Accounting and Book-keeping and its related terms bullAccounting bullBook Keeping bullAccountsbullTypes Of Accounts bullAccounting Cycle

bull Meaning of accounting

Ans ) Accounting is the art and science of recording classifying and summarising monetary transactions

bull Meaning of Book-keeping

Ans) Bookkeeping is the art of recording business transactions with the view of having a permanent record of them and showing their effect on wealth

bull Meaning of account

Ans) The term account means a record of

business transactions concern a particular person of firm asset or income or expense It is a summarised record of all transactions which take place in an accounting year

bull Types of accountsPersonal accounts ndash Personal accounts relating

to person and Organisation are known as personal accounts Example Ramrsquos Account ABC amp Co Account etc

Real account - The accounts related to tangible and intangible assets are called real accounts Example Cash Account Furniture Account etc

Nominal account- Accounts related to expenses losses incomes and gains are known as nominal accounts Example Wages Account Salary Account Discount Account etc

bull Accounting cycle Accounting cycle refers to a complete sequence of accounting activities It begins with recording of transactions and ends with the preparation of a balance sheet

Chemistry TopicAtomic Structure

Thomsonrsquos atomic modelThomson (1898) was the first to propose the model of an atomHe proposed that an atom can be regarded as a uniform sphere of positive electricity in which requisite number of electrons are embedded evently to neutralize the positive chargeThis is just like plums embedded in a pudding or seeds evently distributed in red spongy mass of a watermelonThis model of atom is known as ldquoPlum-Pudding modelrdquo or

Q1)What is the fundamental constituents of atomAns Electron Proton and neutrons are the fundamental constituents of atomQ2)What is the value of fundamental unit of electricityAnsThe charge carried by one electron is sad to be the fundamental unit of electricityIts magnitude is 48times10-10esuOr 1602times10-19C Q3)Name the element containing no neutronAnsOrdinary hydrogen atom or protium 1H1

Types of AccountPersonal AccountReal AccountNominal AccountBalance Sheet (opening)

ldquowatermelon modelrdquoThis model could explain the electrical neutrality of an atom but failed to explain the result of scattering experiment carried out by Rutherford in 1911So it was rejected ultimately

Q4)Why is an electron called universal particleAns Itrsquos mass and Charge are independent of its source

EVS Chapter 1 ndash Modes of Existence

Modes of existence When one speaks normally about the mode of existence of some group or individual one refers to their customs their mode of being their ethology their habitat in some way their feeling for a placeDifferent modes of exixtence are ndash

1 Hunting ndashGathering2 Pastoral3 Agricultural4 Industrial

1 Hunting and gathering Hunting and gathering mode of existence is characterized by obtaining food from hunting wild animals including fishing and gathering wild plants From their earliest days the hunter-gatherer diet included various grasses tubers fruits seeds and nuts Lacking the means to kill larger animals they procured meat from smaller game or through scavenging

Societies that rely primarily or exclusively on hunting wild animals fishing and gathering wild fruits berries nuts and vegetables to support their diet are called hunting and gathering societies

At least this used to be practice of human beings before agriculture is invented As their brains evolved hominids developed more intricate knowledge of edible plant life and growth cycles

Q) Write the features of Hunting ndash gathering societiesAns - There are five basic characteristics of hunting and gathering societies

i The primary institution is the family which decides how food is to be shared and how children are to be socialized and which provides for the protection of its members

ii They tend to be small with fewer than fifty members

iii They tend to be nomadic moving to new areas when the current food supply in a given area has been exhausted

iv Members display a high level of interdependence

v Labor division is based on sex men hunt and women gather

Political Science

Introduction to political science

Political science occasionally called politology is a social science which deals with systems of governance and the analysis of political activities political thoughts associated constitutions and political behaviorThe study of political science involves the study of both the

Answer the following questions-1 What is political science

Political science occasionally called politology is a social science which deals with systems of governance and the analysis of political activities political thoughts associated constitutions and political behavior

2 Short notes-

traditional and modern theories of politicsTraditionalClassical political sciencepolitical theory-Traditional political science is the study of politics before Second World War The methodology to study Politics was traditional (legal formaletc) the definition of politics traditional (Politics begins and end with state)area of study (constitution state machinery)was traditionalModern Political scienceModern political theory-Modern Political Theory critically examines the contemporary state of political theory making an assessment of the achievement and limitations of the Behavioural Revolution in its totality and reviews objectively the major paradigms and conceptual frameworks adopted by the disciplineContemporary attempts at the development of an integrated political theory involving the use of both traditional and modern concepts approaches and theories-Around late 1960s several political scientists realized the importance of both the traditional political theory and modern Political theory They began building an integrated theory of politics involving a systematic mixture of traditional and modern studies of politics It was held that the study of a complex and vast field like politics needs both traditional as well as

Classical political theory Modern Political theory

Homework-Learn

modern concepts and approaches for studying itrsquos all aspects

Subject Eng Literature (The Tempest ndash William Shakespeare) Topic Act I Scene 1 Lines 1 to 32 (Line 32 ndash Gonzalo hellip If he be not born to be hanged our case is miserable) Date 13th April 2020 (3rd Period)

[Students should read the original play and also the paraphrase given in the school prescribed textbook]Summary Questions amp Answers

[SUMMARY OF THE ENTIRE SCENE]

o The play starts with the scene of a severe storm at sea Alonso (King of Naples) Sebastian (Alonsorsquos brother) Ferdinand (Alonsorsquos son) Gonzalo Antonio (the usurping Duke of Milan) are in a ship in the midst of the storm

o The mariners are trying their best to control the vessel from running aground and are totally following the orders of their Master the Boatswain They have scant success

o The mariners become extremely unhappy and annoyed when most of the passengers arrive on the deck thereby hampering their effort to save the ship There is serious confrontation between them and the passengers who are part of the Kingrsquos entourage

o The mariners could not save the ship

SUMMING-UP

(i) Vivid description of the scene which gives a realistic description of terror and confusion of a tropical storm

(ii) Shows Shakespearersquos accuracy of knowledge in describing the naval operations and also matters of seamanship

(iii) The opening scene justifies the title ndash The Tempest

UNANSWERED QUESTIONS

(i) The King always travels with his entire fleet including his soldiers Where

(1) GONZALO Nay good be patient (Line 15-26)BOATSWAIN When the sea is Hence What cares these

roarers for the name of the king To cabin silence Trouble us not

GONZALO Good yet remember whom thou has aboardBOATSWAIN None that I more love than myself You are a

councillor if you can command these elements to silence and work

the peace of the present we will not hand a rope more use your authority If you cannot give thanks you have

lived so long and make yourself ready in your cabin for the mischance of the hour if it so hap [To the Mariners]

Cheerly good hearts [To Gonzalo] Out of our way I say

(a) To whom is the boatswain speaking What does he mean by lsquoNone that I more love than myselfrsquo

The Boatswain is speaking to Gonzalo the honest old councilor of the Duke of MilanBy using the words ndash lsquoNone that I love more than I love myselfrsquo means that for the Boatswain nobody is dearer to him than his own life

(b) What were the conditions that made the boatswain react in this way

The Boatswain reacts in this way because the storm is at sea and Alonso King of Naples Sebastian his brother Ferdinand his son Gonzalo Antonio the usurping Duke of Milan on board are in distress and in panic Thus they have rushed to the deck interrupting the work of the mariners

(c) What hope does Gonzalo take from the attitude of the boatswain

The insolent and authoritative attitude of Boatswain makes Gonzalo feel comforted He tells that there are no signs that the Boatswain will be drowned But his facial appearance and attitude shows that he is destined to die on land by hanging which in effect means that all on board will be saved Otherwise all the persons on board are doomed

(d) How can they lsquomake yourself ready in your cabinrsquo For what were they asked to make ready themselves

In order to make themselves ready in their cabin the

were the other ships

(ii) Why was the ship in that area Where was it coming from or going where

(iii) The ship broke apart What happened to those who were in the ship

passengers on board must prepare for death which they will possibly soon have to meetThey can retire to their cabins and offer prayers to the Almighty to save them from drowning

(e) What does the boatswain say when he is asked to be patient What does he order to the royal party

When the boatswain is asked to be patient and remain calm he says that he will be patient only when the storm will be over and the sea will be calm but as long as the storm blows and there is danger to the ship he cannot think of being patient He orders the royal party to go to the cabin and leave the mariners to their work

(2) GONZALO I have great comfort from this fellow (Line 27-36)

Methinks he hath no drowning mark upon him his complexion is perfect

gallows Stand fast good Fate to his hanging Make the rope of his destiny our cable for our own doth little advantage If he be not born to be hanged our case is miserable

(a) Why does Gonzalo regard the Boatswain in the midst of danger

In the midst of danger Gonzalo regards the boatswain because he feels that the Boatswain is a source of comfort and is bent upon to do his work sincerely which in this case is saving the ship and its passengers from the severest of raging storm

(b) What reasons does Gonzalo give when he says that none in the ship will die of drowning

Gonzalo is almost sure that none in the ship will die by drowning His says that there is no mark on the face of the boatswain that indicates that he will die by drowning On the other hand the lines on his face are strong indications that he will be hanged to death Therefore there shall be no danger of the shiprsquos sinking

(c) Explain the following ldquoStand fast good Fate to his hanging Make the rope of his destiny our cable for our own doth little advantage If he be not born to be hanged our case is miserablerdquo

The stated lines mean that if the will of destiny is to be carried out then the ship will not get wrecked and all the passengers will be saved The safety of the passengers therefore depends upon the will of fate being carried out in the case of the boatswain If however the boatswain is not to die by hanging then the passengers are also very unsafe because in that case the ship is likely to sink

(d) What order does the Boatswain give to the sailors

when he re-enters What does he say about the crying of the fellows inside the cabin

The boatswain orders the sailors to bring the topmast lower and bring the ship close to a stationary position with the help of the main sail He says that the fellows inside the cabin are moaning and crying in their distress louder than his voice and louder even than the roaring of the storm

Class XII (ScienceCommerceHumanities) Subject Topic Summary Execution

Computer Science

PropositionalLogic

Propositional logic is a procedure to provide reasoning through statementProposition A ststement that results in True or False is said to be proposition There are two types of propositionSimple proposition amp compound propositionSimple proposioton A simple proposition is one that is not a part of any other proposition Such sentential form of proposition is symbolized with english letters in short For example Ram is a claver student (TrueFalse)Where do you live (Not in True or False)Grapes are sweet (TrueFalse)It rains today (TrueFalse)Here we can see some statements anwer would be true or false but some staements answer can not give in terms of true or false Thus the sentences which can be answered in true or false are known as simple propositionAssigning propositon to a variableThe general syntax to assign propostion to a variable is as followsVariable = Simple propositonFor example A=Ram is a clever studentB= Grapes are sweetC= it rains todayCompound proposition

helliphellipto be continued in next classhelliphellipMath Relation Relation If A and B are two non-empty sets

then a relation R from A to B is a subset of AxB If R A x B and (a b) R then we say that a sube isinis related to b by the relation R written as aRbeg Let A be the set of students of class XII and B be the set of students of class XI Then some of the examples of relation from A to B arei) (a b) AXB a is brother of bisinii) (a b) AXB age of a is more than age of isinb Types of relation In this section we would like to study different types of relations We know that a relation in a set A is a subset of A times A Thus the empty set φ and A times A are two extreme relations For illustration consider a relation R in the set A = 1 2 3 4 given by R = (a b) a ndash b = 10 This is the empty set as no pair (a b) satisfies the condition a ndash b = 10 Similarly R = (a b) | a ndash b | ge 0 is the whole primeset A times A as all pairs (a b) in A times A satisfy | a ndash

Example 1 Let A be the set of all students of a boys school Show that the relation R in A given by R = (a b) a is sister of b is the empty relation and R = (a b) the primedifference between heights of a and b is less than 3 meters is the universal relationSolution Since the school is boys school no student of the school can be sister of any student of the school Hence R = φ showing that R is the empty relation It is also obvious that the difference between heights of any two students of the school has to be less than 3 meters This shows that R = A times A is primethe universal relation Example 2 Show that the relation R in the set 1 2 3 given by R = (1 1) (2 2) (3 3) (1 2) (2 3) is reflexive

b | ge 0 These two extreme examples lead us to the following definitionsDefinition 1 A relation R in a set A is called empty relation if no element of A isrelated to any element of A ie R = φ A times AsubDefinition 2 A relation R in a set A is called universal relation if each element of A is related to every element of A ie R = A times A Both the empty relation and the universal relation are some times called trivial relation Definition 3 A relation R in a set A is called(i) reflexive if (a a) R for every a Aisin isin(ii) symmetric if (a1 a2) R implies that (aisin 2a1)

R for all aisin 1 a2 Aisin(iii) transitive if (a1 a2) R and (aisin 2 a3) R isinimplies that (a1 a3) R for all aisin 1 a2 a3 AisinDefinition 4 A relation R in a set A is said to be an equivalence relation if R is reflexive symmetric and transitive

but neither symmetric nor transitiveSolution R is reflexive since (1 1) (2 2) and (3 3) lie in R Also R is not symmetric as (1 2) R but (2 1) isin notinR Similarly R is not transitive as (1 2) R and (2 3) R but (1 3) R isin isin notinExample 3 Show that the relation R in the set Z of integers given byR = (a b) 2 divides a ndash b is an equivalence relationSolution R is reflexive as 2 divides (a ndash a) for all a Z isinFurther if (a b) R then 2 divides a isinndash b Therefore 2 divides b ndash a Hence (b a) R which shows that R is isinsymmetric Similarly if (a b) R and (b c) R isin isinthen a ndash b and b ndash c are divisible by 2 Now a ndash c = (a ndash b) + (b ndash c) is even (Why) So (a ndash c) is divisible by 2 This shows that R is transitive Thus R is an equivalence relation in ZExample 4 Let L be the set of all lines in a plane and R be the relation in L defined as R = (L1 L2) L1 is perpendicular to L2 Show that R is symmetric but neither reflexive nor transitiveSolution R is not reflexive as a line L1 can not be perpendicular to itself ie (L1 L1) R notinR is symmetric as (L1 L2) Risin

L1 is perpendicular to L2rArr L2 is perpendicular to L1rArr (L2 L1) RrArr isin

R is not transitive Indeed if L1 is perpendicular to L2 and L2 is perpendicular to L3 then L1 can never be perpendicular to L3 In fact L1 is parallel to L3 ie (L1 L2) R isin(L2 L3) R but (L1 L3) Risin notin

Chemistry Solid state Characteristics if Solids(i)The particles are locked in fixed positions they are unable to change their relative positions and this brings a definite shape and volume of a solid(ii)In a solid the constituent particles are held by strong forces of attractionThe forces of attraction may be bonding or non bonding(iii)The constituent particles in a solid pack together as closely as possibleoccupying most of the available space within the solidThus the empty space in a solid is very smallThis makes a solid highly rigid and nearly incompressibleThis also explains why a solid has high density and exhibits slow diffusionClassification of Solids

Q1)Define Crystalline solids AnsA Solid that has a definite geometrical shape and a sharp melting pointand whose constituent particles (atomsmolecules or ions) are arranged in a long range order of definite pattern extending throughout the solid is called a crystalline solidExNaClQ2)Define Amorphous solids AnsA solid that does not have a definite shape and a sharp melting pointand whose constituent particles (atomsmolecules or ions) are not arranged in a definite pattern is called an amorphoussolid

Crystalline solidsAmorphous solids

ExGlassRubberQ3)Classify Crystalline Solids Crystalline Solids

Physics Coloumbrsquos Law (Summary)

Before Going Into Coloumbrsquos Law We Will First Learn What is Charge Properties of Charge and Always remember that charge is quantized ie a body always have static charge of magnitude equal to some integral multiple of fundamental electronic charge e= 16 x 10- 19 C

Charge is the property of matter that causes it to produce and experience electrical and magnetic effects The study of the electrical charges at rest is called electrostatics When both electrical and magnetic effects are present the interaction between charges is referred to as electromagnetic

There exist two types of charges in nature positive and negative Like charges repel and unlike charges attract each other

The type of charge on an electron is negative The charge of a proton is the same as that of an electron but with a positive sign In an atom the number of electrons and the number of protons are equal The atom is therefore electrically neutral If one or more electrons are added to it it becomes negatively charged and is designated as negative ion However if one or more electrons are removed from an atom it becomes positively charged and is called a positive ion

The excess or deficiency of electrons in a body gives the concept of charge If there is an excess of electrons in a body it is negatively charged And if there is deficiency of electrons the body becomes positively charged Whenever addition or removal of electrons takes places the body acquires a charge

The SI Unit of charge is coulomb (C) In SI units the current is a fundamental quantity having a unit of ampere (A) The unit of charge is defined in terms of the unit of current Thus one coulomb is the charge transferred in one second across the section of a wire carrying a

Ionic SolidsMetallicSolids

Molecular Solids

current of one ampere

As q = It we have1 C = (1 A) (1 s)

The dimensions of charge are [A T]

Properties of Charge

(1) Quantization of Charge Electric charge can have only discrete values rather than any value That is charge is quantized The smallest discrete value of charge that can exist in nature is the charge on an electron given as

e = plusmn 16 x 10- 19 C

This is the charge attained by an electron and a protonA charge q must be an integral multiple of this basic unit That is

Q = plusmn ne where n = 1 2 hellip

Charge on a body can never be (frac12)e (23)e or 57e etcWhen we rub a glass rod with silk some electrons are transferred from the rod to the silk The rod becomes positively charged The silk becomes negatively charged The coulomb is a very large amount of charge A typical charge acquired by a rubbed body is 10 - 8 C

Biology Reproduction in organisms

Welcome to this new session 2020-21Today in this first chapter we mainly discuss about reproduction types needs and life span of some organismsWe also discuss about difference between sexual and asexual reproduction

Q1 What is reproductionReproduction is defined as a biological processin which an organism gives rise to young onessimilar to itselfQ2 What are the needs of reproductionbulli) Reproduction maintain life on earthii) It enables the continuity of the species generation after generationiii) It creates genetic variation among populationsQ3 Define Life span and write some orgnisms life spanbull Life span is the period from birth to

the natural death of an organism- OrganismsLife span1 Butterfly 1 - 2 weeks2 Fruit fly 30 days3Dog 10-13 years4 Rose5-7 years5 Tortoise100-150 years6 Banyan Tree -200 - 250 yearsQ4 Reproduction is of two types in case ofanimals but in case of plants vegetative propagation is also present

Asexual Reproduction Sexual Reproductioni) Always uniparentalii) Gametes are not involvediii) Only mitotic division involvediv) Somatic cells of parents are involvedv) Offsprings are genetically similar to the parents

i) Usually biparentalii) Gametes are involvediii) Meiosis occurs during gametogenesis Mitosis occurs after fertilisationiv) Germ cells of the parents are involvedv) offsprings are genetically different from the parents

COMMERCE BUSINESS ENVIRONMENT

Welcome to the new sessiontoday we are going to start the first chapter of Class XII The name of the chapter is Business Environment

Already many of you have got some idea about the word business environment form the first chapter of business studies in class XI

In todayrsquos world every business enterprise is a part of the society It exists and operates in association with various groups in society such as customers suppliers competitors banks and financial institutions government agencies trade unions media and so on All these groups influence the functioning of business in one way or the other They constitute the environment of businessConcept of Business Environment

The term lsquobusiness environmentrsquo refers to the sum total of all individuals institutions and other forces that lie outside a business enterprise but that may influence its functioning and performance

The main features of business environment Totality of External forces General and Specific forces Interrelatedness Complexity Dynamic Uncertainty

Prepare the following questions from todayrsquos assignment

2 What do you mean by business environment

The term lsquobusiness environmentrsquo means the aggregate of all forces factors and institutions which are external to and beyond the control of an individual business enterprise but they may influence its functioning and performance Business environment is the macro framework within which a business firm a micro unit operates It consists of several interrelated and interacting elements

2 Explain the main features of business environment in brief

Totality of External forces-Business environment is the sum total of all things external to a business environment

General and Specific forces-It

Relativity

The Interrelation between business and its environment

The business enterprise is an open system It continuously interacts with its environment It takes inputs (such as raw materials capital labour energy and so on) from its environment transforms them into goods and services and sends them back to the environment

Fig 1 Business Environment Relationship

includes both the forces general forces are the economic social political legal and technological conditions which indirectly influence all business enterprise Specific forces are the investors customers competitors and suppliers which influence individual enterprise directly

Interrelatedness-Different elements of environment are interrelated for an example growing awareness for health care has increased the demand for health foods

Complexity- Business environment id complex in nature as the elements keep on changing example economic technological and other forces changes in demand for a product and service

Dynamic-Business environment is not static it keeps on changing

Uncertainty- Itrsquos very difficult to predict future events such as technology and fashion which occur fast and frequently

Business Studies

Human Resources Management

Human resource of an organisation are the aggregate of knowledge skills attitudes of people working in it

The management system which deals with human resources is called human resource management

Features of HRMbullComprehensive functionbullPeople-oriented

Question1) What do you mean by human

resource management Answer) Human resource management may be defined as that field of Management which has to do with planning organising and controlling the functions of procuring developing maintaining and utilising the labour force

bullAction oriented bullPervasive function bullContinuous function

2) Explain the features of HRM in brief

Answer)bullHuman Resource Management is concerned with managing people at work bull Human Resource Management is concerned with employees which bring people and organisations together so that the goals of each are met bullHuman resource management considered every employees as an individual and also promote their satisfaction and growth bull Human resource management is inherent in all organisations and at all levelsbullManagement of human resources are ongoing on never ending process which requires a constant alertness and Awareness of human relations

3) ldquoHR function is said to be pervasiverdquowhy

Answer) Human resource management is required in all organisations whether it is private or government organisations armed forces sports organisations etc It permeatsall the functional areas like production marketing finance research etc This from this feature of human resource management it can be said that it is pervasive in nature

Economics Demand Q1DEFINITION OF DEMANDIn economics demand is the quantity of a good that consumers are willing and able to purchase at various prices during a given period of timeQ2DEMAND CURVEIn economics a demand curve is a graph depicting the relationship between the price of a certain commodity and the quantity of that commodity that is demanded at that pricQ3LAW OF DEMANDIn microeconomics the law of demand states that conditional on all else being equal as the price of a good increases quantity demanded decreases conversely as the price of a good decreases quantity demanded increasesQ4ASSUMPTION of LAW OF DEMAND(i)No change in price of related commodities(ii) No change in income of the consumer(iii) No change in taste and preferences customs habit and fashion of the consumer( No expectation regarding future change in priceQ5MARKET DEMAND SCHEDULEIn economics a market demand schedule is a tabulation of the quantity of a good that all consumers in a market will purchase at a

given price At any given price the corresponding value on the demand schedule is the sum of all consumersrsquo quantities demanded at that priceQ6INDIVIDUAL DEMAND SCHEDULEIndividual demand schedule refers to a tabular statement showing various quantities of a commodity that a consumer is willing to buy at various levels of price during a given period of timeQ7 FACTORS AFFECTING INDIVIDUAL DEMAND FOR A COMMODITY

The factors that influence a consumerrsquos decision to purchase a commodity are also known as determinants of demand The following factors affect the individual demand for a commodity1 price of the commodity2 price of related goods3 income of buyer of the commodity4 tastes and preferences of the buyer1 Price of the CommodityYou must have observed that when price of a commodity falls you tend to buy more of it and when its price rises you tend to buy less of it when all other factors remain constant (lsquoother things remaining the samersquo) In other words other things remaining the same there is an inverse relationship between the price of a commodity and its quantity demanded by its buyers This statement is in accordance with law of demand which you will study in the later part of this lesson Price of a commodity and its quantity demanded by its buyers are inversely related only when lsquoother things remain the samersquo So lsquoother things remaining the samersquo is an assumption when we study the effect of changes in the price of a commodity on its quantity demanded2 Price of Related goodsA consumer may demand a particular good But while buying that good heshe also asks the price of its related goods Related goods can be of two types-(i) Substitute goods(ii) Complementary goods While purchasing a good prices of its substitutes and complements do affect its quantity purchased(i) Price of Substitute Goods Substitute goods are those goods which can easily be used in place of one another for satisfaction of a particular want like tea and coffee An increase in price of substitute good leads to an increase in demand for the given commodity and a decrease in price of substitute good leads to a decrease in demand for the given commodity It means demand for a given commodity is directly affected by change in price of substitute goods For example if price of coffee increases the demand for tea will rise as tea will become relatively cheaper in comparison to coffee(ii) Price of Complementary goods Complementary goods are those goods which are used together to satisfy a particular want like car and petrol An increase in the price of complementary goods leads to a decrease in demand for the given commodity and a decrease in the price of complementary goods leads to an increase in demand for the given commodity For example if price of petrol falls then the demand for cars will increase as it will be relatively cheaper to use both the goods together So demand for a given commodity is inversely affected by change in price of complementary goods3 Income of the Buyer of CommodityDemand for a commodity is also affected by income of its buyer However the effect of change in income on demand depends on the nature of the commodity under consideration In case of some goods like full cream milk fine quality of rice (Basmati rice) etc demand for these commodities increases when income of the buyer increases and

demand for these commodities decreases when income of the buyer decreases Such goods whose demand increases with the increase in income of the buyer are called normal goods But there are some goods like coarse rice toned milk etc whose demand decreases when income of buyer increases and their demand increases when income of the buyer decreases Such goods whose demand decreases with the increase in income of the buyer are called inferior goods Suppose a consumer buys 10 Kgs of rice whose price is ` 25 per Kg He cannot afford to buy better quality of rice because the price of such rice is ` 50 per Kg The consumer is spending ` 250 per month on the purchase of rice Now if income of the consumer increases and he can afford ` 350 on purchase of 10 Kg of rice Now he can afford to buy some quantity of rice say 6 Kgs whose price is ` 25 per Kg and may buy 4 Kgs of rice whose price is ` 50 per Kg Thus he will buy 10 Kgs of rice by spending ` 350 per month Therefore we may conclude that demand for normal goods is directly related to the income of the buyer but demand for inferior goods is inversely related to the income of the buyer4 Tastes and Preferences of the BuyerThe demand for a commodity is also affected by the tastes and preferences of the buyers They include change in fashion customs habits etc Those commodities are preferred by the consumers which are in fashion So demand for those commodities rises which are in fashion On the other hand if a commodity goes out of the fashion its demand falls because no consumer will like to buy it(5) Number of Buyers in the Market(Population)Increase in population raises the market demand whereas decrease in population reduces the market demand for a commodity Not only the size of population but its composition like age (ratio of males females children and old people in population) also affects the demand for a commodity It is because of needs of children young old male and female population differs(6) Distribution of Income and WealthIf the distribution of income and wealth is more in favour of the rich demand for the commodities preferred by the rich such as comforts and luxuries is likely to be higher On the other hand if the distribution of income and wealth is more in favour of poor demand for commodities preferred by the poor such as necessities will be more(7) Season and Weather ConditionsThis is generally observed that the demand for woolens increases during winter whereas demand for ice creams and cold drinks increases during summer Similarly market demand for umbrellas rain coats increases during rainy seasonQ8 REASONS FOR OPERATION OF LAW OF DEMAND WHY DEMAND CURVE SLOPES DOWNWARDNow we will try to explain why does a consumer purchase more quantity of a commodity at a lower price and less of it at a higher price or why does the law of demand operate ie why does the demand curve slope downwards from left to right The main reasons for operation of law of demand are1 Law of Diminishing Marginal UtilityAs you have studied earlier law of diminishing marginal utility states that as we consume more and more units of a commodity the utility derived from each successive unit goes on decreasing The consumer will be ready to pay more for those units which provide him more utility and less for those which provide him less utility It implies that he will purchase more only when the price of the commodity falls2 Income Effect

When price of a commodity falls purchasing power or real income of the consumer increases which enables him to purchase more quantity of the commodity with the same money income Let us take an example Suppose you buy 4 ice creams when price of each ice cream is ` 25 If price of ice creams falls to ` 20 then with same money income you can buy 5 ice creams now3 Substitution EffectWhen price of a commodity falls it becomes comparatively cheaper as compared to its substitutes (although price of substitutes has not been changed) This will lead to rise in demand for the given commodity For example if coke and Pepsi both are sold at ` 10 each and price of coke falls Now coke has become relatively cheaper and will be substituted for Pepsi It will lead to rise in demand for coke4 Change in Number of BuyersWhen price of a commodity falls some old buyers may demand more of the commodity at the reduced price and some new buyers may also start buying this commodity who were not in a position to buy it earlier due to higher price This will lead to increase in number of buyers when price of the commodity falls As a result demand for the commodity rises when its price falls5 Diverse Uses of a CommoditySome commodities have diverse uses like milk It can be used for drinking for sweet preparation for ice cream preparation etc If price of milk rises its use may be restricted to important purpose only This will lead to reduction in demand for other less important uses When price of milk falls it can be put to other uses also leading to rise n demand for itQ9 EXCEPTIONS TO THE LAW OF DEMANDYou have studied in law of demand that a buyer is willing to buy more quantity of a commodity at a lower price and less of it at a higher price But in certain circumstances a rise in price may lead to rise in demand These circumstances are called Exceptions to the Law of Demand Some important exceptions are1 Giffen GoodsGiffen goods are special type of inferior goods in which negative income effect is stronger than negative substitution effect Giffen goods do not follow law of demand as their demand rises when their price rises Examples of Giffen goods are jowar and bajra etc2 Status Symbol GoodsSome goods are used by rich people as status symbols eg diamonds gold jewellary etc The higher the price the higher will be the demand for these goods When price of such goods falls these goods are no longer looked at as status symbol goods and tehrefore therir demand falls3 NecessitiesCommodities such as medicines salt wheat etc do not follow law of demandbecause we have to purchase them in minimum required quantity whatever their price may be4 Goods Expected to be ScarceWhen the buyers expect a scarcity of a particular good in near future they start buying more and more of that good even if their prices are rising For example during war famines etc people tend to buy more of some goods even at higher prices due to fear of their scarcity in near future

Political Science

Constitution of India-The

Preamble

The preamble-

Preamble-

The preamble is the most precious part of the constitution We the people of India having solemnly resolved to constitute India into a Sovereign Socialist Secular Democratic Republic and to secure to all its citizensA preamble is an introductory and expressionary statement in a document that explains the documents purpose and underlying philosophy When applied to the opening paragraphs of a statute it may recite historical facts pertinent to the subject of the statuteNature and purpose of the constitution-Purpose of the Constitution dictates permanent framework of the government to form a more perfect union to establish justice and ensure peace of thenationconstitution provide principles how the government can run itself following the rules and laws written in the constitution of each state keeps them balanced

Answer the following questions-

1 What is preambleA preamble is an introductory and expressionary statement in a document that explains the documents purpose and underlying philosophy2 What is the nature and

purpose of the constitutionConstitution dictatespermanent framework of the government to form a more perfect union to establish justice and ensure peace of the nation

Homework-Learn

Accounts Compatibilty mode

1MEANING OF PARTNERSHIPPartnership is a form of business organisation where two or more persons join hands to run a business They share the profits and losses according to the agreement amongst them According to the Indian Partnership Act 1932 ldquoPartnership is relation between persons who have agreed to share profits of a business carried on by all or any one of them acting for allrdquo For example one of your friends has passed class XII from National Institute of Open Schooling (NIOS) and wants to start a business Heshe approaches you to join in this venture Heshe wants you to contribute some money and participate in the business activities Both of you if join hands constitute a partnership2CHARACTERISTICS1048698 Agreement A partnership is formed by an agreement The agreement may be either oral or in writing It defines the relationship between the persons who agree to carry on business It may contain the terms of sharing profit and the capital to be invested by each partner etc The written agreement is known as partnership deed1048698 Number of persons There must be at least two persons to form a partnership

The maximum number of partners in a partnership firm can be 50 according toCompanies Act 20131048698 Business The Partnership is formed to carry on business with a purpose of earning profits The business should be lawful Thus if two or more persons agree to carry on unlawful activities it will not be termed as partnership1048698 Sharing Profits The partners agree to share profits in the agreed ratio In caseof loss all the partners have to bear it in the same agreed profit sharing ratio10486981048698Mutual Agency Every partner is an agent of the other partners Every partner can bind the firm and all other partners by hisher acts Each partner will be responsible and liable for the acts of all other partners10486981048698Unlimited liability The liability of each partner except that of a minor is unlimited Their liability extends to their personal assets also If the assets of the firm are insufficient to pay off its debts the partnersrsquo personal property can be used to satisfy the claim of the creditors of the partnership firm10486981048698Management All the partners have a right to mange the business However they may authorize one or more partners to manage the affairs of the business on their behalf10486981048698Transferability of Share No partner can transfer hisher share to any one including hisher family member without the consent of all other partners3PARTNERSHIP DEEDAgreement forms the basis of partnership The written form of the agreement is which a document of partnership is It contains terms and conditions regarding the conduct of the business It also explains relationship between the partners This document is called partnership deed Every firm can frame its own partnership deed in which the rights duties and liabilities of the partners are stated in detail It helps in settling the disputes arising among the partners during the general conduct of business 4CONTENTS OF PARTNERSHIP DEEDThe partnership deed generally contains the following (i) Name and address of the partnership firm(ii) Nature and objectives of the business(iii) Name and address of each partner(iv) Ratio in which profits is to be shared(v) Capital contribution by each partner(vi) Rate of Interest on capital if allowed(vii) Salary or any other remuneration to partners if allowed(viii) Rate of interest on loans and advances by a partner to the firm(ix) Drawings of partners and interest thereon if any(x) Method of valuation of goodwill and revaluation of assets and liabilities on the reconstitution of the partnership ie on the admission retirement or death of a partner(xi) Settlement of disputes by arbitration(xii) Settlement of accounts at the time of retirement or death of a partner5IN ABSENCE OF PARTNERSHIP DEEDThe partnership deed lays down the terms and conditions of partnership in regard to rights duties and obligations of the partners In the absence of partnership deed there may arise a controversy on certain issues like profit sharing ratio interest on

capital interest on drawings interest on loan and salary of the partners In such cases the provisions of the Indian Partnership Act becomes applicableSome of the Issues are(i) Distribution of Profit Partners are entitled to share profits equally(ii) Interest on Capital Interest on capital is not allowed(iii) Interest on Drawings No interest on drawing of the partners is to be charged(iv) Interest on Partnerrsquos Loan A Partner is allowed interest 6 per annum on the amount of loan given to the firm by himher(v) Salary and Commission to Partner A partner is not entitled to anysalary or commission or any other remuneration for managing the business

History TOPIC-TOWARDS INDEPENDENCE AND PARTITION THE LAST PHASE (1935-1947)

SUB TOPIC-IMPORTANT POLITICAL DEVELOPMENTS ndash GROWTH OF SOCIAL IDEAS

Socialism is a political social and economic philosophyLike in other parts of the world the Russian revolution of 1917 served as a great inspiration for revolutionaries in India who at that time were engaged in the struggle for liberation from British ruleSocialist ideas led to the formation of communist party of IndiaJAWAHARLAL NEHRU Among the early Congress leaders Jawaharlal Nehru was very much impressed and influenced by the Socialist ideas He also learnt about the Economic activities of the Soviet Union after the Bolshevic Revolution 1917 He made full use of them in IndiaThe election of Jawaharlal Nehru and Subhas Chandra Bose showed the Left wing tendency within CongressJawaharlal Nehru demanded economic freedom along with political freedom of the people in order to end the exploitation of masses

Nehrus working committee included three socialists leaders The Lucknow session was a landmark in the evolution of socialist ideas of the congressSUBHAS CHANDRA BOSE ndash Subhas Chandra Bose had socialist leaning Both Jawaharlal Nehru and Subhas Chandra Bose were known as leftist Congress men Later on National Congress divided into Leftist and rightist campCONGRESS SOCIALIST Within the Congress some leaders formed the Congress Socialist partyPattavi Sitaramyya Sardar Patel Rajendra Prasad had hostile attitude towards the Congress Socialist partyJawaharlals attitude was hesitant

1 QUESTION ndash Mention name of two Congress leaders who had socialist leaning

1ANSWER ndash Subhas Chandra Bose and Jawaharlal Nehru2QUESTION- In which session of the congress Jawaharlal elaborated his Socialist ideas2 ANSWER ndash Lucknow and Faizpur Session in December 1935 and 19363QUESTION ndash Why Congress was sharply divided into leftist and rightist camp 3ANSWER ndash Subhas Chandra Bosersquos attempt to seek re election for congress presidentship in 1939sharply divided the National Congress into Leftist and Rightist camp4 QUESTION ndash Who was MN Roy 4 ANSWER ndash Manabendra Roy first formed the Communist Party of India outside the country at Tashkent in 19205QUESTION ndash Who formed the Congress Socialist Party within the Congress5 ANSWER ndash Jaya Prakash Narayan Achyut Patwardhan Acharya Narendra Dev Ram Mohan Lohia Aruna Asaf Ali6QUESTION ndash When was the Congress Socialist Party formed What was its object6 ANSWER ndash 1934The Congress Socialist Party sought to work out socialist programme through the Congress They joined hands with the Congress and wanted to carry

Subhas Chandra Bose being expelled from the congress after the Tripuri rift he formed Forward BlockThere were basic differences between the Congress Socialists and the communistsTRADE UNION ACTIVITIES Maximum working class people lived in Bombay and Calcutta The working and living conditions of those workers were very miserable In this situation Shasipada Banerjee NM Lokhande protested against the oppression of the working class peopleThe first Trade Union Madras Labour Union was formed in 1918 by BP WadiaIndustrial strikes took place in Kanpur Calcutta Madras Jamshedpur and Ahmedabad AITUC was formed in Bombay in 1927 The growth of Trade union among the workers was slow because of the fear of the dismissal of the jobIn the mean time the Moderates as well as Communists left AITUC and formed separate organization

on National struggle with the help of workers and peasant class of the society7 QUESTION ndash What was the name of the party founded by Subhas Chandra Bose7 ANSWER- Forward Block8QUESTION ndash Who was Shasipada Banerjee8 ANSWER ndash Shasipada Banerjee was a radical Brahmo He founded a working menrsquos club to protest against exploitation of the British rulers towards the working class of India9 QUESTION ndash What was the weekly published by NM Lokhande9ANSWER- Dinabandhu10 QUESTION ndash Who founded Bombay Mill-Hands Association and in which year10 ANSWER- NM Lokhande in189011 QUESTION- Who was BP WadiaANSWER- BPWadia was the founder of Madras Labour Union in191812 QUESTION- What was the name of the first labour union of India12 ANSWER- Madras Labour Union13 QUESTION Who founded the Majur Mahajan 13 ANSWER GANDHIJI14 QUESTION What was the full form of AITUC When it was formed14 ANSWER All India Trade Union Congressin 192715QUESTION Who formed the Red Trade Union Congress and in which year15ANSWER The Communists formed the Red Trade Union Congress16 QUESTION What do you mean by Socialism16 ANSWER Socialism describes any political and economic theory that says the community rather than individuals should own and manage property and natural resources

Subject Eng Literature (The Tempest ndash William Shakespeare) Topic Act III Scene 3 Lines 1 to 52 (Line 52 ndash Brother my lord the Duke Stand to and do as we) Date 13th April 2020 (4th Period)

[Students should read the original play and also the paraphrase given in the school prescribed textbook]Summary Questions amp Answers

o Alonso Sebastian Antonio Gonzalo Adrian Francisco and others wandered about the island in search of Ferdinand and gets tired and hungry of the toil and at the same time gives up all hope of finding him

o Antonio and Sebastian are happy that Alonso is out of hope and decide to make another attempt on his life that night when being so tired they will be sleeping soundly

o Suddenly a solemn and strange music is heard in the air and several strange shapes enter bringing in a banquet These strange shapes then dance round it with gestures of salutation and then inviting the King to eat they depart

o Seeing this strange scene all are inclined to believe the tales told by travelers that there truly are ldquounicornsrdquo and ldquothe phoenixrsquo thronerdquo

1 ALONSO What harmony is this My good friends hark (L18-27)

GONZALO Marvellous sweet music

[Enter several strange shapes bringing in a banquet

they dance about it with gentle actions of salutation

and inviting the King and his companions to eat they depart]ALONSO Give us kind keepers heavens What were theseSEBASTIAN A living drollery Now I will believe

That there are unicorns that in Arabia

There is one tree the phoenixrsquo throne one phoenix

At this hour reigning thereANTONIO Ill believe both

And what does else want credit come to me

And Ill be sworn rsquotis true Travellers neer did lie

Though fools at home condemn rsquoem

(a) How did Prospero present an amazing spectacle before Alonso and his companions

Using his magic powers Prospero ordered strange shapes to lay a banquet before Alonso and his companions The shapes brought several dishes with tasty eatables in them They placed the dishes on a table before Alonso and his companions Then the strange shapes began to dance gracefully around the banquet While dancing they made gestures inviting them to eat the food Then suddenly the shapes disappeared(b) Who were the guests at the strange banquet Describe the lsquoliving drolleryrsquo

Alonso Sebastian Antonio Gonzalo Adrian and Francisco were the guests at the strange banquet

The term ldquoliving drolleryrdquo refers to live entertainment show In this context when Alonso the King of Naples Sebastian his brother Antonio the treacherous brother of Prospero Gonzalo the kind and loyal councillor to the King Adrian and Francisco came to the island they were hungry and weary in their spirits They heard a solemn and strange music They were shocked to see several strange shapes bringing in a banquet and these shapes danced about it with gentle action of salutation inviting the King and his companions to eat After this Sebastian described this show as lsquoliving drolleryrsquo(c) What is lsquophoenixrsquo What are lsquoUnicornsrdquo

The term lsquophoenixrsquo refers to a mythical Arabian bird which lived alone and perched on a solitary tree After one hundred years it expired in flames and rose again from its own ashes

lsquoUnicornsrsquo refers to the mythological four-footed beasts having horns in the centre of their foreheads When the horns are ground into powder the powder was believed to be

an aphrodisiac(d) How does Sebastian explain the puppet show OR Why does the speaker now believe in unicorns and phoenix

Sebastian finds several strange shapes bringing in the banquet They invite the king and his party for dinner and soon depart He tells that if such a strange sight can be a reality there is nothing incredible in the world and from the present moment he will believe anything He says that it is a strange dumb show enacted not by puppets but by living beings It is stranger than a travellerrsquos tale Seeing such a thing

before his own eyes he will no longer disbelieve the story about unicorns and phoenix(e) How do the other characters present respond to this living drollery

At the sight of the lsquoliving drolleryrsquo like Sebastian Gonzalo and Antonio too acted strangely Antonio told that he too now believes in unicorns and phoenix and anything else that seems to be incredible He too now believes in travellersrsquo tales Gonzalo told that if he would report those happenings in Naples nobody will believe him He considers that those gentle shapes were gentler in manner in comparison to the living beings Alonso was at first sight suspicious and told them that those strange shapes conveyed their meaning in expressive gestures when they seemed to lack speech by their movements and sounds Francisco was amazed at their mysterious disappearance

2 ALONSO Not I

(Line 43-52)GONZALO Faith sir you need not fear When we

were boysWho would believe that there were mountaineers

Dewlapped like bulls whose throats had hanging at rsquoem

Wallets of flesh Or that there were such men

Whose heads stood in their breasts Which now we find

Each putter-out of five for one will bring us

Good warrant ofALONSO I will stand to and feed

Although my lastmdashno matter since I feel

The best is past Brother my lord the Duke

Stand to and do as we

(a) How does Alonso respond at the spectacle of the shapes which were sent to them at the instruction of Prospero

After seeing the strange sight of appearing and disappearing of the shapes sent by Prospero to arrange a banquet for them Alonso says that his surprise at having seen those creatures is infinite and he is fully justified in feeling so much surprise He thinks that their shapes their gestures and the sounds they made were indeed amazing Although they do not possess the gift of speech yet they were able to convey their

thoughts by means of their gestures only

(b) What does Prospero say about the views expressed by Alonso regarding the shapes What does Francisco think about the shapesAfter hearing Alonsorsquos views about the shapes Prospero says that this manrsquos praise of the spirits is rather hasty He means to say that Alonso has shown great haste in reaching the conclusion about the shapes Francisco is amazed to see that those shapes disappeared in a mysterious way(c) What does Sebastian ask Alonso to doSebastian tells Alonso that the shapes having disappeared should not matter to them because they have left the eatables behind He asks Alonso to enjoy eating as they are extremely hungry but the king does not accept his offer of enjoying the dishes(d) How does Gonzalo try to dispel Alonsorsquos fear of those strange shapes What kind of references does he give to AlonsoGonzalo says that those who have travelled abroad have reported seeing even stranger sights than these shapes that Alonso and his companions have beheld Hence there is no reason to feel afraid of these shapes Gonzalo further adds that in his younger days he had heard strange stories from travelers and Alonso might have heard similar stories For instance it was said that there existed a certain race of

human beings who had huge lumps of flesh hanging at their throats and who therefore resembled bulls Then Gonzalo tells about a race of human beings whose heads were located at their breasts Gonzalo says that such stories were not believed by most people in those days but now-a-days these stories have become common(e) Explain the following lsquoEach putter-out of five for onersquoEnglish travellers often insured their trips with London brokers Those that went on foreign travels those days used to deposit a certain amount with some firm or company in London before their departure If the travelers failed to return the money was forfeited by the company with which it had been deposited But this money was repaid five-fold if the travelers returned safe and sound In this way a traveler stood a great chance of recovering the entire cost of his

travels(f) Give the explanatory meanings of the following expressions in the context of the above extract (i) Dewlapped (ii) Wallets of flesh

(iii) Putter-out(i) Dewlapped having big lumps of flesh at the necks(ii) Wallets of flesh large masses of flesh looking like bags(iii) Putter-out to invest money before commencing the travel

  • General methods of preparation of hydrogen
  • Chapter Dimensional Analysis (Summary)
    • Properties of Charge
Page 28:   · Web viewSubject. Topic. Summary. Execution. Hindi. व्याकरण. शरीरके अंगो के नाम लिखिए. 1) आँख 2) नाक 3

He was not so wise as Ankite He did it

He did not fail to do itf As soon as I reached college the

bell rangNo sooner did I reach college than the bell rang

g He finished everythingHe left nothing unfinished

h It always pours when it rainsIt never rains but it pours

Math Topic Commercial MathematicsChapter ndash Goods and services Tax

What is GSTAns It is a abbreviated term of Goods and Service Text which is an indirect tax levied on the sale of goods and rendering servicesSome terms related to GSTDelar Any person who buys goods or services For resale is known as a delar A delar Can be a firm or a companyIntra-state sales Sales of goods and services within the same state or same union territory are called intra- state salesInter-state sales Sales of goods and services outside the state or union territory are called Inter-state sales4) Input GST GST is paid by dealers on purchase of goods and services are called input GST5) Output GST GST is collected from customers on sale of goods and services are called output GST6) Types of GST There are three taxes applicable under GST(i) Central Goods and Services Tax (CGST)(ii) State Goods and Services Tax (SGST) or Union Territory Goods and Services Tax (UTGST) Both these taxes are levied on intra-state sales Here GST is divided equally among central and state governments(iii) Integrated Goods and Services Tax (IGST) IGST is levied on inter- state sales It is also levied on import of goods and services into India and export of goods and services from India

Subject Eng Literature (The Merchant of Venice ndash William Shakespeare)Topic Act III Scene 4 Lines 1 to 44 (Portia hellip To wish it back on you fare you well Jessica)[Students should read the original play and also the paraphrase given in the school prescribed textbook]

Summary Questions amp AnswersIn this scene we suddenly find a new element in the character of Portia We have already seen her possessed of every graceful womanly quality but now she shows that she is capable of rapid decision and determined action She shows this by her sudden resolve to hasten to Venice with a daring scheme for the rescue of Antonio This is an important scene in the dramatic action for it leads up to and renders possible the striking events of the famous trial scene which is one of the greatest striking elements of the play Moreover the fact that all the characters of importance are now assembled together in Venice makes the union of the main plot and the secondary story complete

(1) LORENZO Madam although I speak it in your presence(Line 1-9)

You have a noble and a true conceit

Of god-like amity which appears most strongly

In bearing thus the absence of your lordBut if you knew to whom you show this honourHow true a gentleman you send reliefHow dear a lover of my lord your husbandI know you would be prouder of the workThan customary bounty can enforce you

(a) Where is Lorenzo Why is he here To whom is he referring as lsquoMadamrsquo

Lorenzo is at Portiarsquos residence He had met Salerio on the way and Salerio had begged him to come along with him to

o In this scene Portia Nerissa Lorenzo Jessica and Balthazar appear

o Portia requests Lorenzo and Jessica to be in charge of her house during her absence from Belmont because she and Nerissa have decided to spend the days in meditation and also in visiting the holy places in the neighbourhood of Belmont She has already instructed her people to acknowledge both Lorenzo and Jessica as master and mistress of house during her absence Lorenzo and Jessica gladly agree to look after the house of Portia

handover the letter from Antonio to Bassanio The letter carried the bad news about Antoniorsquos arrest for non-payment of loan taken from Shylock Hence Salerio might have preferred company to break this bad news to Bassanio He is referring to Portia as Madam(b) What does Portia say on hearing the above extract

Portia says that she has never regretted doing good to others Friends who spend a lot of time together and really are there for each other have many traits in common As Antonio is Bassaniorsquos best friend saving him is like saving Bassanio who is like her own soul She asks Lorenzo to take care of management of the house till Bassanio is back(c) What does Portia send with Bassanio and why

On hearing about Antoniorsquos troubles on account of Bassanio her husband Portia immediately sends him with enough gold to repay the debt many times over to Venice to help Antonio out of his misfortune

(2) Lorenzo Madam with all my heart (Line 36-40)

I shall obey you in all fair commands

Portia My people do already know my mindAnd will acknowledge you and JessicaIn place of Lord Bassanio and myselfSo fare you well till we shall meet again

(a) Where are Lorenzo and Portia at this time What lsquofair commandsrsquo are given to Lorenzo

Lorenzo and Portia are at Belmont during this scenePortia reveals to Lorenzo that she has sworn to contemplate in prayer at a monastery around two miles away until her husband returns from Venice She tells him that Nerissa would accompany her and asks him to manage the house with Jessica till things are settled In response Lorenzo tells her that he would be obliged to do whatever she asks him to do(b) Where is Portia actually going and why

Portia tells Lorenzo that she would live a life of contemplation and pray at a monastery which is two miles away from her place In reality Portia plans to go to Venice in disguise with Nerissa and argue the case in defense of Antonio She is very sure that her plan would succeed

ClassXI (ScienceHumanitiesCommerce)Subject Topic Summary Execution

Computer Science

(APC)

Ch ndash 1 Numbers

(Numbers in different bases and

their Arithmatical operations)

Number System In computers Number System is defined as a writing system to represent the numbers in different ways ie we are using different symbols and notations to represent numbers There are four ways we can represent the number ndash Binary Decimal Octal and Hexadecimal

Decimal Number SystemThis number system consist 10 digits These are 0 1 2 3 4 5 6 7 8 amp 9

Binary Number SystemThis number system has only two digits these are 0 and 1 Here 0 stands for off while 1 stands for on

Octal Number SystemThis number system has 8 digits these are 0 1 2 3 4 5 6 amp 7

Hexadecimal Number SystemThis number system has 16 digits these are 0 1 2 3 4 5 6 7 8 9 A B C D E F Here the value of the alphabets are as follows A=10 B=11 C=12 D=13 E=14 F=15

Rules for conversion decimal number to Binary1 Divide the decimal number by 22 If the number will not divide equally by 2 then round down the answer to the nearest whole number (integer)3 Keep a note of the remainder it should be either 0 or 14 Keep repeating the above steps dividing each answer by 2 until you reach zero5 Write out all the remainders from bottom to top This is your binary solution

For example Lets convert 32 to binary 2 32 2 16 - 0 2 8 - 0 2 4 - 0 2 2 - 0 2 1 - 0 0 - 1

The binary equivalent of 3210 is 1000002

Try the follwing youself1 2410

2 4810

3 1210

History GROWTH OF NATIONALISM

The second half of the 19th century witnessed growth of political consciousness and a sense of Nationalism among the IndiansThere were various factors for growth of Indian Nationalism- As a result various political associations were formed in different provinces by the educated Indians Surendranath Banerjee organized a meeting of National conference at Calcutta Ultimately the National Congress was founded in Bombay in 1885This body became the vanguard of Indian struggle for freedom The congress leaders were known as moderates because they followed a policy of prayer and petition A large number of Indian leaders had experienced in political agitation The Political situation of England was also changed Moreover increasing revolutionary activities in Maharashtra Punjab and Bengal became serious concern to the British Government In this

QUESTION1 What do you mean by Nationalism ANSWER 1 Nationalism is defined as loyalty and devotion to own nation especially a sense of national consciousnessQUESTION 2 What are the causes of nationalism ANSWER 2 There were various factors for growth of nationalism

1 Spread of western education2 The progress of vernacular press and

patriotic literature3 The economic exploitation of our

country by the colonial rulers4 International affairs

QUESTION 3 Who organized National conference in Calcutta in 1883 ANSWER 3 Surendranath BanerjeeQUESTION 4 When did Indian National Congress formANSWER 4 Indian National Congress was formed in 1885 in BombayQUESTION 5 Who were ModeratesANSWER 5 The Early Nationalists were also known as Moderates Their emergence marked

background Lord Curzon became Viceroy in India He had no respect for the Indian National Congress

the beginning of the organized national movement in India They believed in British justice and were loyal to them They followed a policy of prayer and petition They demanded constitutional reforms of our country Impotant Moderate leaders were Pherozshah Mehta Dadabhai Naorozi and Surendranath Banerjee etcQUESTION 6 What do you know about Extremism in Indian National movementANSWER 6 In the beginning of 20th century a new class of national leaders emerged in India which was different from the moderate groups They started more aggressive movement against the British empire The goal of extremists was ldquoswarajrdquo Important extremist leaders were Bal Gangadhar Tilak Lala Lajpat Rai Bipin Chandra Pal etcQUESTION 7 Mention the places which were the main centres of Revolutionary movementANSWER 7 Maharashtra Bengal and Punjab

Physics

Chapter Dimensional Analysis

(Summary)

The dimensions of a physical quantity are the powers to which the fundamental units are raised in order to obtain the derived unit of that quantit

The physical quantites lengthmasstime are represented by [L] [M] [T] resp let they are raised to powers ( dimesions) abc resp then any physical quantity can be represented by [ La Mb Tc ] Examples

1 Area area = L x B = [L] x [L] = [M0 L2 T0 ]

2 Density density = massvolume = [M][L3] = [ M L-3]

3 Velocity velocity = distancetime = [L][T] = [LT-1]HW Try to find out dimension of acceleration Acceleration = velocity timeNB One can find the SI Units Using Dimension Analysis Such as for area we have [L2] so its SI unit is m2

Biology Topic ndash Chp-1 The living world

Today we will start the first chapter the living world Here we discuss about the characteristics of living organism and what are the difference between them and nonliving substances We also discuss about the contribution of different Scientists

There are over 500000 species of plants andover a million species of animal are present on earth Some 15000 new species were discovered every yearQ1 What is a living organismbull A living organism is primarily physico -chemical material that demonstrate a high degree of complexity is capable of selfRegulation possesses a metabolism and perpetuates itself through timeQ2 What are the differences between livingand non-livingsi) Compared with non-living living organisms

have more complex organised structure and their use of energy is more controlled amp efficientii) Living things reproduce their own kind by forming new cells which contains copies of their genesiii) Each organism has some degree of homeostasisie it is able to make adjustments so that internal environment remains constantQ3 Write contributions of following Scientists i) Aristotle - One of the first theories in Biology places all living things in a hiearchieii) AV Leeuwenhoek - was the first to observe living single celled organisms under microscopeii) Carolus Linnaeus - developed the binary system for naming of organisms and classificationiii) Geregor Johann Mendel ndash discoverbasic principles of inheritanceHomework i) C Darwin ii)Schleiden

Math Trigonometric functions

1 Overviewi) Trigonometry The word lsquotrigonometryrsquo is derived from the Greek words lsquotrigonrsquo and lsquometronrsquo which means measuring the sides of a triangle An angle is the amount of rotation of a revolving line with respect to a fixed line Usually we follow two types of conventions for measuring angles ie a) Sexagesimal system b) Circular system In Sexagesimal system the unit of measurement is Degree In Circular system the unit of measurement is Radian ii) Relation between degree and radianThe ratio of circumference of a circle to its diameter is always a constant This constant ratio is a number denoted by π which is taken approximately as 227The relationship between degree amp radian measurements is as follows2 right angles = 180deg= π radians1radian = 180degπ=57deg16(approx) 1deg=π180 radianiii) Length of an arc of a circleIf an arc of length s subtends an angle θ radians at the center of a circle of radius r then s=rθiv) Area of a sector of a circleA sector is like a pizza slice of the

Q) Express the following angles in radiana) 45deg b) 40deg3730Ans a) We have 180deg=π radiansi e 45deg= πtimes45180 radian = π4 radiansb) 40deg3730= 40deg37+3060 minute= 40deg 37 +12 minute= 40deg+ 752 minute=40 + 75(2times60) degree=3258 degreeNow 180deg=π radianie 3258 degree= (πtimes325) (180times8) radians = 65π288 radiansQ) A circle has a radius of r=12 meters What is the length of an arc traced out by a 60deg angle in the center of the circleAns In this problem we know both the central angle (60deg) and the radius of the circle (12) All we have to do is plug those values into our equation and we get

s = 2π(12)(60360)s = 24π6s = 4πSo the length of an arc traced out by a 60deg angle in a circle with a radius of 12 meters equals 4π meters asymp 1257 metersQ) Find the area of the sector with a central angle 30deg and a radius of 9cmAns GivenRadius r = 9 cmAngle θ = 30degArea of the sector = θ360degtimesπr2

= 30360degtimes227times92=2121cm2

circle It consists of a region bounded by two radii and an arc lying between the radiiThe area of a sector is a fraction of the area of the circle This area is proportional to the central angle In other words the bigger the central angle the larger is the area of the sectorArea of Sector = θ2 times r2 (when θ is in radians)

Area of Sector = θ times π360 times r2 (when θ is in degrees)

COMMERCE

CLASSIFICTION OF HUMAN ACTIVITIES-ECONOMIC AND NON-ECONOMIC

Welcome to the new sessiontoday we are going to start the first chapter of Class XI The name of the chapter that we are going to start is

lsquoClassification of Human Activities ndasheconomic and non-economicrsquo

Now let us start the chapter by considering human beings and the activities they perform throughout the day

Human activities means all those activities that human beings undertake to satisfy their wants

Human wants on the other hand are the desire of human beings for goods (vegetables fruits rice etc) and services (services of doctors teachers lawyers etc) that they require to live

Now these human activities continue throughout life as human wants are unending unlimited and recurring as human beings desire for better living throughout their lives

Now human activities can be classified into two categories

Human activities

Economic activities Non-economic activities

Economic activities are

Questions1 What are human activities

Answer Human activities mean all those activities that human beings undertake to satisfy their wants

Example A man working in an office

A boy playing in the garden

2What are the characteristics of human activitiesAnswer the characteristics of human activities are as follows

Human activities are undertaken by men women and children and these activities involve human efforts

Human activities are undertaken to satisfy human wants which are unlimited

Human activities continue throughout life

Human activities are performed for both earning money and personal satisfaction

3What is economic activitiesGive example

Answer Economic activities are undertaken by human beings with the object of earning money acquiring wealth and thereby satisfying human wantsExample

Selling of goods by a shop keeper to his customer

A clinic run by a doctor Service of a teacher in school or college

undertaken by human beings with the object of earning money and acquiring wealth

These activities result in the production of economic goods and services

Example Human activities(ie working in factories officesshops) which produce direct economic benefits

Non-economic activities are inspired by human sentiments and emotions such as love for the family desire to help the poor and love for the country

Thus these human activities (eg praying playing sleeping) produce no direct economic benefits and they are also not related to earning money and acquiring wealth

4 What are the characteristics of economic activities

Answer The characteristics of economic activities are as follows

Economic motiveEconomic activities are undertaken to earn money and acquire wealth

ProductiveEconomic activities involve productiondistribution and exchange of goods and services to create wealth

Economic growthEconomic activities determine the level of economic development of a country and standard of living of its citizens

Socially desirableEconomic activities are socially desirable for society

Economic resourcesEconomic activities make use of all the economic resources such landlabourcapital etc

5 What do you mean by non-economic activitiesExampleAnswerNon-economic activities are inspired by human sentiments and emotions such as love for the family desire to help the poor and love for the countryThese activities are not undertaken for monetary gain but for onersquos satisfaction and happinessExample

a mother looks after her children

a student donates blood8 Differentiate between Economic activities and Non-economic activities

Economic activities

Non-economic activities

1to earn living and acquiring wealth2Result can be measured in terms of money

3ExampleBusinessprofession and employment

1 to obtain some satisfaction

2Result cannot be measured in terms of money

3ExampleFamily-orientedreligious socialCultural and national

BUSINESS STUDIES

BUSINESS ENVIRONMENT

Welcome to the new sessionToday we are going to start the first chapter and the name of the chapter is Business Environment

In todayrsquos world every business enterprise is a part of the society It exists and operates in association with various groups in society such as customers suppliers competitors banks and financial institutions government agencies trade unions media and so on All these groups influence the functioning of business in one way or the other They constitute the environment of businessConcept of Business Environment

The term lsquobusiness environmentrsquo refers to the sum total of all individuals institutions and other forces that lie outside a business enterprise but that may influence its functioning and performance

The main features of business environment

Totality of External forces General and Specific forces Interrelatedness Complexity Dynamic Uncertainty Relativity

The Interrelation between business and its environment

The business enterprise is an open system It continuously interacts with its environment It takes inputs

Prepare the following questions from todayrsquos assignment

1 What do you mean by business environment

The term lsquobusiness environmentrsquo means the aggregate of all forces factors and institutions which are external to and beyond the control of an individual business enterprise but they may influence its functioning and performance Business environment is the macro framework within which a business firm a micro unit operates It consists of several interrelated and interacting elements

2 Explain the main features of business environment in brief

Totality of External forces-Business environment is the sum total of all things external to a business environment

General and Specific forces-It includes both the forces general forces are the economic social political legal and technological conditions which indirectly influence all business enterprise Specific forces are the investors customers competitors and suppliers which influence individual enterprise directly

Interrelatedness-Different elements of environment are interrelated for an example growing awareness for health care has increased the demand for health foods

Complexity- Business environment id

(such as raw materials capital labour energy and so on) from its environment transforms them into goods and services and sends them back to the environment

Fig 1 Business Environment Relationship

complex in nature as the elements keep on changing example economic technological and other forces changes in demand for a product and service

Dynamic-Business environment is not static it keeps on changing

Uncertainty- Itrsquos very difficult to predict future events such as technology and fashion which occur fast and frequently

Economics Basic Economic ConceptsSub topic

Microeconomics and

Macroeconomics

Welcome to the new sessiontoday we are going to start the first chapter of Class XI The name of the chapter that we are going to start is Basic Economic concepts

Now Economics covers the study of human activities Human activities are those activities which are performed by humans to satisfy their wants

Thus Human wants are unlimited and therefore economic activities such as production exchange and consumption are needed in order to satisfy those wants

The study of economics is divided largely in two parts which areMicroeconomics and Macroeconomics

SUBJECT- MATTER OF ECONOMICS

MICROECONOMICS MACROECONOMICS

Questions1Who has coined the words micro and macro economics

Answer Ranger Frisch coined the words lsquomicrorsquo and lsquomacrorsquo in 1933 to denote the two branches of economic theory namely microeconomics and macroeconomics

2What is microeconomicsAnswer It is the study of behaviour of individual decision ndash making unit such as consumers firms etc

3 What is macroeconomicsAnswer Macroeonomics is the study of overall economic phenomena like employment national income etc

4 What is the importance of microeconomicsAnswer

Microeconomics helps in formulating economic policies which enhance productive efficiency and results in greater social welfare

It helps the government in formulating correct price policies

It explains the working of a capitalistic economy where individual units(producers and consumers ) are free to take their own decision

Micro means a small part in

microeconomics we do not study the whole economy Hence we study an individual consumer and his or her choices and a producer and his or her profit maximizing decisions in the market Thus it does not mirror what happens in the economy as a whole

Macroeconomics on the other hand studies the economy as a whole It is concerned with aggregate and depicts the entire picture of the economyMacroeconomics deals with the national income aggregate investment aggregate consumption etc

Features of Microeconomics It deals with small

parts of the country Hence it looks at

individual consumers firms and industries

It deals with individual income consumption and savings

It studies the determination of price of any product or factors of production

It deals with the working of market via the price mechanism which is nothing but the determination of price and quantity of a commodity by the forces of demand and supply

Features of Macroeconomics

It deals with the study of the economy as a whole

It is concerned with

5 Give a limitation of microeconomics Microeconomics fails to explain the

functioning of an economy as a whole It cannot explain unemployment illiteracy and other problems prevailing in the country

6 What is the importance of macroeconomics It gives overall view of the growing

complexities of an economic system It provides the basic and logical

framework for formulating appropriate macroeconomic policies (eg for inflation poverty etc )to direct and regulate economy towards desirable goals

7What is the limitation of macroeconomics It ignores structural changes in an

individual unit of the aggregate

8 Differentiate between Microeconomics and Macroeconomics

Microeconomics Macroeconomics

the study of aggregates

National income aggregate savings and aggregate investments are major concepts dealt within macroeconomics style

It studies the determination of general price levels

It investigates into the problem of unemployment and the achievement of employment

It studies the aspect of decision making at the aggregate and national levels

It includes all growth theories whether related to developed or developing economies it also includes the study of economic systems and the working of the economy under different systems

Note Both Micro and macro economics are complementary and should be fully utilized for proper understanding of an economy

1It studies economic aspect of an individual unit2It deals with individual incomeConsumption and savings

3 It facilitates determination of price of any product or factors of production

4 Itrsquos scope is narrow and restricted to individual unit

1It studies the economy as a whole

2It deals with the national income aggregate consumption and aggregate savings3 It facilitates determination of general price level in an economy

4 Itrsquos scope is wide as it deals with economic units on the national level

ACCOUNTS

Introduction to Accounting and Book-keeping

Today I am going to share you the meaning of Accounting and Book-keeping and its related terms bullAccounting bullBook Keeping bullAccountsbullTypes Of Accounts bullAccounting Cycle

bull Meaning of accounting

Ans ) Accounting is the art and science of recording classifying and summarising monetary transactions

bull Meaning of Book-keeping

Ans) Bookkeeping is the art of recording business transactions with the view of having a permanent record of them and showing their effect on wealth

bull Meaning of account

Ans) The term account means a record of

business transactions concern a particular person of firm asset or income or expense It is a summarised record of all transactions which take place in an accounting year

bull Types of accountsPersonal accounts ndash Personal accounts relating

to person and Organisation are known as personal accounts Example Ramrsquos Account ABC amp Co Account etc

Real account - The accounts related to tangible and intangible assets are called real accounts Example Cash Account Furniture Account etc

Nominal account- Accounts related to expenses losses incomes and gains are known as nominal accounts Example Wages Account Salary Account Discount Account etc

bull Accounting cycle Accounting cycle refers to a complete sequence of accounting activities It begins with recording of transactions and ends with the preparation of a balance sheet

Chemistry TopicAtomic Structure

Thomsonrsquos atomic modelThomson (1898) was the first to propose the model of an atomHe proposed that an atom can be regarded as a uniform sphere of positive electricity in which requisite number of electrons are embedded evently to neutralize the positive chargeThis is just like plums embedded in a pudding or seeds evently distributed in red spongy mass of a watermelonThis model of atom is known as ldquoPlum-Pudding modelrdquo or

Q1)What is the fundamental constituents of atomAns Electron Proton and neutrons are the fundamental constituents of atomQ2)What is the value of fundamental unit of electricityAnsThe charge carried by one electron is sad to be the fundamental unit of electricityIts magnitude is 48times10-10esuOr 1602times10-19C Q3)Name the element containing no neutronAnsOrdinary hydrogen atom or protium 1H1

Types of AccountPersonal AccountReal AccountNominal AccountBalance Sheet (opening)

ldquowatermelon modelrdquoThis model could explain the electrical neutrality of an atom but failed to explain the result of scattering experiment carried out by Rutherford in 1911So it was rejected ultimately

Q4)Why is an electron called universal particleAns Itrsquos mass and Charge are independent of its source

EVS Chapter 1 ndash Modes of Existence

Modes of existence When one speaks normally about the mode of existence of some group or individual one refers to their customs their mode of being their ethology their habitat in some way their feeling for a placeDifferent modes of exixtence are ndash

1 Hunting ndashGathering2 Pastoral3 Agricultural4 Industrial

1 Hunting and gathering Hunting and gathering mode of existence is characterized by obtaining food from hunting wild animals including fishing and gathering wild plants From their earliest days the hunter-gatherer diet included various grasses tubers fruits seeds and nuts Lacking the means to kill larger animals they procured meat from smaller game or through scavenging

Societies that rely primarily or exclusively on hunting wild animals fishing and gathering wild fruits berries nuts and vegetables to support their diet are called hunting and gathering societies

At least this used to be practice of human beings before agriculture is invented As their brains evolved hominids developed more intricate knowledge of edible plant life and growth cycles

Q) Write the features of Hunting ndash gathering societiesAns - There are five basic characteristics of hunting and gathering societies

i The primary institution is the family which decides how food is to be shared and how children are to be socialized and which provides for the protection of its members

ii They tend to be small with fewer than fifty members

iii They tend to be nomadic moving to new areas when the current food supply in a given area has been exhausted

iv Members display a high level of interdependence

v Labor division is based on sex men hunt and women gather

Political Science

Introduction to political science

Political science occasionally called politology is a social science which deals with systems of governance and the analysis of political activities political thoughts associated constitutions and political behaviorThe study of political science involves the study of both the

Answer the following questions-1 What is political science

Political science occasionally called politology is a social science which deals with systems of governance and the analysis of political activities political thoughts associated constitutions and political behavior

2 Short notes-

traditional and modern theories of politicsTraditionalClassical political sciencepolitical theory-Traditional political science is the study of politics before Second World War The methodology to study Politics was traditional (legal formaletc) the definition of politics traditional (Politics begins and end with state)area of study (constitution state machinery)was traditionalModern Political scienceModern political theory-Modern Political Theory critically examines the contemporary state of political theory making an assessment of the achievement and limitations of the Behavioural Revolution in its totality and reviews objectively the major paradigms and conceptual frameworks adopted by the disciplineContemporary attempts at the development of an integrated political theory involving the use of both traditional and modern concepts approaches and theories-Around late 1960s several political scientists realized the importance of both the traditional political theory and modern Political theory They began building an integrated theory of politics involving a systematic mixture of traditional and modern studies of politics It was held that the study of a complex and vast field like politics needs both traditional as well as

Classical political theory Modern Political theory

Homework-Learn

modern concepts and approaches for studying itrsquos all aspects

Subject Eng Literature (The Tempest ndash William Shakespeare) Topic Act I Scene 1 Lines 1 to 32 (Line 32 ndash Gonzalo hellip If he be not born to be hanged our case is miserable) Date 13th April 2020 (3rd Period)

[Students should read the original play and also the paraphrase given in the school prescribed textbook]Summary Questions amp Answers

[SUMMARY OF THE ENTIRE SCENE]

o The play starts with the scene of a severe storm at sea Alonso (King of Naples) Sebastian (Alonsorsquos brother) Ferdinand (Alonsorsquos son) Gonzalo Antonio (the usurping Duke of Milan) are in a ship in the midst of the storm

o The mariners are trying their best to control the vessel from running aground and are totally following the orders of their Master the Boatswain They have scant success

o The mariners become extremely unhappy and annoyed when most of the passengers arrive on the deck thereby hampering their effort to save the ship There is serious confrontation between them and the passengers who are part of the Kingrsquos entourage

o The mariners could not save the ship

SUMMING-UP

(i) Vivid description of the scene which gives a realistic description of terror and confusion of a tropical storm

(ii) Shows Shakespearersquos accuracy of knowledge in describing the naval operations and also matters of seamanship

(iii) The opening scene justifies the title ndash The Tempest

UNANSWERED QUESTIONS

(i) The King always travels with his entire fleet including his soldiers Where

(1) GONZALO Nay good be patient (Line 15-26)BOATSWAIN When the sea is Hence What cares these

roarers for the name of the king To cabin silence Trouble us not

GONZALO Good yet remember whom thou has aboardBOATSWAIN None that I more love than myself You are a

councillor if you can command these elements to silence and work

the peace of the present we will not hand a rope more use your authority If you cannot give thanks you have

lived so long and make yourself ready in your cabin for the mischance of the hour if it so hap [To the Mariners]

Cheerly good hearts [To Gonzalo] Out of our way I say

(a) To whom is the boatswain speaking What does he mean by lsquoNone that I more love than myselfrsquo

The Boatswain is speaking to Gonzalo the honest old councilor of the Duke of MilanBy using the words ndash lsquoNone that I love more than I love myselfrsquo means that for the Boatswain nobody is dearer to him than his own life

(b) What were the conditions that made the boatswain react in this way

The Boatswain reacts in this way because the storm is at sea and Alonso King of Naples Sebastian his brother Ferdinand his son Gonzalo Antonio the usurping Duke of Milan on board are in distress and in panic Thus they have rushed to the deck interrupting the work of the mariners

(c) What hope does Gonzalo take from the attitude of the boatswain

The insolent and authoritative attitude of Boatswain makes Gonzalo feel comforted He tells that there are no signs that the Boatswain will be drowned But his facial appearance and attitude shows that he is destined to die on land by hanging which in effect means that all on board will be saved Otherwise all the persons on board are doomed

(d) How can they lsquomake yourself ready in your cabinrsquo For what were they asked to make ready themselves

In order to make themselves ready in their cabin the

were the other ships

(ii) Why was the ship in that area Where was it coming from or going where

(iii) The ship broke apart What happened to those who were in the ship

passengers on board must prepare for death which they will possibly soon have to meetThey can retire to their cabins and offer prayers to the Almighty to save them from drowning

(e) What does the boatswain say when he is asked to be patient What does he order to the royal party

When the boatswain is asked to be patient and remain calm he says that he will be patient only when the storm will be over and the sea will be calm but as long as the storm blows and there is danger to the ship he cannot think of being patient He orders the royal party to go to the cabin and leave the mariners to their work

(2) GONZALO I have great comfort from this fellow (Line 27-36)

Methinks he hath no drowning mark upon him his complexion is perfect

gallows Stand fast good Fate to his hanging Make the rope of his destiny our cable for our own doth little advantage If he be not born to be hanged our case is miserable

(a) Why does Gonzalo regard the Boatswain in the midst of danger

In the midst of danger Gonzalo regards the boatswain because he feels that the Boatswain is a source of comfort and is bent upon to do his work sincerely which in this case is saving the ship and its passengers from the severest of raging storm

(b) What reasons does Gonzalo give when he says that none in the ship will die of drowning

Gonzalo is almost sure that none in the ship will die by drowning His says that there is no mark on the face of the boatswain that indicates that he will die by drowning On the other hand the lines on his face are strong indications that he will be hanged to death Therefore there shall be no danger of the shiprsquos sinking

(c) Explain the following ldquoStand fast good Fate to his hanging Make the rope of his destiny our cable for our own doth little advantage If he be not born to be hanged our case is miserablerdquo

The stated lines mean that if the will of destiny is to be carried out then the ship will not get wrecked and all the passengers will be saved The safety of the passengers therefore depends upon the will of fate being carried out in the case of the boatswain If however the boatswain is not to die by hanging then the passengers are also very unsafe because in that case the ship is likely to sink

(d) What order does the Boatswain give to the sailors

when he re-enters What does he say about the crying of the fellows inside the cabin

The boatswain orders the sailors to bring the topmast lower and bring the ship close to a stationary position with the help of the main sail He says that the fellows inside the cabin are moaning and crying in their distress louder than his voice and louder even than the roaring of the storm

Class XII (ScienceCommerceHumanities) Subject Topic Summary Execution

Computer Science

PropositionalLogic

Propositional logic is a procedure to provide reasoning through statementProposition A ststement that results in True or False is said to be proposition There are two types of propositionSimple proposition amp compound propositionSimple proposioton A simple proposition is one that is not a part of any other proposition Such sentential form of proposition is symbolized with english letters in short For example Ram is a claver student (TrueFalse)Where do you live (Not in True or False)Grapes are sweet (TrueFalse)It rains today (TrueFalse)Here we can see some statements anwer would be true or false but some staements answer can not give in terms of true or false Thus the sentences which can be answered in true or false are known as simple propositionAssigning propositon to a variableThe general syntax to assign propostion to a variable is as followsVariable = Simple propositonFor example A=Ram is a clever studentB= Grapes are sweetC= it rains todayCompound proposition

helliphellipto be continued in next classhelliphellipMath Relation Relation If A and B are two non-empty sets

then a relation R from A to B is a subset of AxB If R A x B and (a b) R then we say that a sube isinis related to b by the relation R written as aRbeg Let A be the set of students of class XII and B be the set of students of class XI Then some of the examples of relation from A to B arei) (a b) AXB a is brother of bisinii) (a b) AXB age of a is more than age of isinb Types of relation In this section we would like to study different types of relations We know that a relation in a set A is a subset of A times A Thus the empty set φ and A times A are two extreme relations For illustration consider a relation R in the set A = 1 2 3 4 given by R = (a b) a ndash b = 10 This is the empty set as no pair (a b) satisfies the condition a ndash b = 10 Similarly R = (a b) | a ndash b | ge 0 is the whole primeset A times A as all pairs (a b) in A times A satisfy | a ndash

Example 1 Let A be the set of all students of a boys school Show that the relation R in A given by R = (a b) a is sister of b is the empty relation and R = (a b) the primedifference between heights of a and b is less than 3 meters is the universal relationSolution Since the school is boys school no student of the school can be sister of any student of the school Hence R = φ showing that R is the empty relation It is also obvious that the difference between heights of any two students of the school has to be less than 3 meters This shows that R = A times A is primethe universal relation Example 2 Show that the relation R in the set 1 2 3 given by R = (1 1) (2 2) (3 3) (1 2) (2 3) is reflexive

b | ge 0 These two extreme examples lead us to the following definitionsDefinition 1 A relation R in a set A is called empty relation if no element of A isrelated to any element of A ie R = φ A times AsubDefinition 2 A relation R in a set A is called universal relation if each element of A is related to every element of A ie R = A times A Both the empty relation and the universal relation are some times called trivial relation Definition 3 A relation R in a set A is called(i) reflexive if (a a) R for every a Aisin isin(ii) symmetric if (a1 a2) R implies that (aisin 2a1)

R for all aisin 1 a2 Aisin(iii) transitive if (a1 a2) R and (aisin 2 a3) R isinimplies that (a1 a3) R for all aisin 1 a2 a3 AisinDefinition 4 A relation R in a set A is said to be an equivalence relation if R is reflexive symmetric and transitive

but neither symmetric nor transitiveSolution R is reflexive since (1 1) (2 2) and (3 3) lie in R Also R is not symmetric as (1 2) R but (2 1) isin notinR Similarly R is not transitive as (1 2) R and (2 3) R but (1 3) R isin isin notinExample 3 Show that the relation R in the set Z of integers given byR = (a b) 2 divides a ndash b is an equivalence relationSolution R is reflexive as 2 divides (a ndash a) for all a Z isinFurther if (a b) R then 2 divides a isinndash b Therefore 2 divides b ndash a Hence (b a) R which shows that R is isinsymmetric Similarly if (a b) R and (b c) R isin isinthen a ndash b and b ndash c are divisible by 2 Now a ndash c = (a ndash b) + (b ndash c) is even (Why) So (a ndash c) is divisible by 2 This shows that R is transitive Thus R is an equivalence relation in ZExample 4 Let L be the set of all lines in a plane and R be the relation in L defined as R = (L1 L2) L1 is perpendicular to L2 Show that R is symmetric but neither reflexive nor transitiveSolution R is not reflexive as a line L1 can not be perpendicular to itself ie (L1 L1) R notinR is symmetric as (L1 L2) Risin

L1 is perpendicular to L2rArr L2 is perpendicular to L1rArr (L2 L1) RrArr isin

R is not transitive Indeed if L1 is perpendicular to L2 and L2 is perpendicular to L3 then L1 can never be perpendicular to L3 In fact L1 is parallel to L3 ie (L1 L2) R isin(L2 L3) R but (L1 L3) Risin notin

Chemistry Solid state Characteristics if Solids(i)The particles are locked in fixed positions they are unable to change their relative positions and this brings a definite shape and volume of a solid(ii)In a solid the constituent particles are held by strong forces of attractionThe forces of attraction may be bonding or non bonding(iii)The constituent particles in a solid pack together as closely as possibleoccupying most of the available space within the solidThus the empty space in a solid is very smallThis makes a solid highly rigid and nearly incompressibleThis also explains why a solid has high density and exhibits slow diffusionClassification of Solids

Q1)Define Crystalline solids AnsA Solid that has a definite geometrical shape and a sharp melting pointand whose constituent particles (atomsmolecules or ions) are arranged in a long range order of definite pattern extending throughout the solid is called a crystalline solidExNaClQ2)Define Amorphous solids AnsA solid that does not have a definite shape and a sharp melting pointand whose constituent particles (atomsmolecules or ions) are not arranged in a definite pattern is called an amorphoussolid

Crystalline solidsAmorphous solids

ExGlassRubberQ3)Classify Crystalline Solids Crystalline Solids

Physics Coloumbrsquos Law (Summary)

Before Going Into Coloumbrsquos Law We Will First Learn What is Charge Properties of Charge and Always remember that charge is quantized ie a body always have static charge of magnitude equal to some integral multiple of fundamental electronic charge e= 16 x 10- 19 C

Charge is the property of matter that causes it to produce and experience electrical and magnetic effects The study of the electrical charges at rest is called electrostatics When both electrical and magnetic effects are present the interaction between charges is referred to as electromagnetic

There exist two types of charges in nature positive and negative Like charges repel and unlike charges attract each other

The type of charge on an electron is negative The charge of a proton is the same as that of an electron but with a positive sign In an atom the number of electrons and the number of protons are equal The atom is therefore electrically neutral If one or more electrons are added to it it becomes negatively charged and is designated as negative ion However if one or more electrons are removed from an atom it becomes positively charged and is called a positive ion

The excess or deficiency of electrons in a body gives the concept of charge If there is an excess of electrons in a body it is negatively charged And if there is deficiency of electrons the body becomes positively charged Whenever addition or removal of electrons takes places the body acquires a charge

The SI Unit of charge is coulomb (C) In SI units the current is a fundamental quantity having a unit of ampere (A) The unit of charge is defined in terms of the unit of current Thus one coulomb is the charge transferred in one second across the section of a wire carrying a

Ionic SolidsMetallicSolids

Molecular Solids

current of one ampere

As q = It we have1 C = (1 A) (1 s)

The dimensions of charge are [A T]

Properties of Charge

(1) Quantization of Charge Electric charge can have only discrete values rather than any value That is charge is quantized The smallest discrete value of charge that can exist in nature is the charge on an electron given as

e = plusmn 16 x 10- 19 C

This is the charge attained by an electron and a protonA charge q must be an integral multiple of this basic unit That is

Q = plusmn ne where n = 1 2 hellip

Charge on a body can never be (frac12)e (23)e or 57e etcWhen we rub a glass rod with silk some electrons are transferred from the rod to the silk The rod becomes positively charged The silk becomes negatively charged The coulomb is a very large amount of charge A typical charge acquired by a rubbed body is 10 - 8 C

Biology Reproduction in organisms

Welcome to this new session 2020-21Today in this first chapter we mainly discuss about reproduction types needs and life span of some organismsWe also discuss about difference between sexual and asexual reproduction

Q1 What is reproductionReproduction is defined as a biological processin which an organism gives rise to young onessimilar to itselfQ2 What are the needs of reproductionbulli) Reproduction maintain life on earthii) It enables the continuity of the species generation after generationiii) It creates genetic variation among populationsQ3 Define Life span and write some orgnisms life spanbull Life span is the period from birth to

the natural death of an organism- OrganismsLife span1 Butterfly 1 - 2 weeks2 Fruit fly 30 days3Dog 10-13 years4 Rose5-7 years5 Tortoise100-150 years6 Banyan Tree -200 - 250 yearsQ4 Reproduction is of two types in case ofanimals but in case of plants vegetative propagation is also present

Asexual Reproduction Sexual Reproductioni) Always uniparentalii) Gametes are not involvediii) Only mitotic division involvediv) Somatic cells of parents are involvedv) Offsprings are genetically similar to the parents

i) Usually biparentalii) Gametes are involvediii) Meiosis occurs during gametogenesis Mitosis occurs after fertilisationiv) Germ cells of the parents are involvedv) offsprings are genetically different from the parents

COMMERCE BUSINESS ENVIRONMENT

Welcome to the new sessiontoday we are going to start the first chapter of Class XII The name of the chapter is Business Environment

Already many of you have got some idea about the word business environment form the first chapter of business studies in class XI

In todayrsquos world every business enterprise is a part of the society It exists and operates in association with various groups in society such as customers suppliers competitors banks and financial institutions government agencies trade unions media and so on All these groups influence the functioning of business in one way or the other They constitute the environment of businessConcept of Business Environment

The term lsquobusiness environmentrsquo refers to the sum total of all individuals institutions and other forces that lie outside a business enterprise but that may influence its functioning and performance

The main features of business environment Totality of External forces General and Specific forces Interrelatedness Complexity Dynamic Uncertainty

Prepare the following questions from todayrsquos assignment

2 What do you mean by business environment

The term lsquobusiness environmentrsquo means the aggregate of all forces factors and institutions which are external to and beyond the control of an individual business enterprise but they may influence its functioning and performance Business environment is the macro framework within which a business firm a micro unit operates It consists of several interrelated and interacting elements

2 Explain the main features of business environment in brief

Totality of External forces-Business environment is the sum total of all things external to a business environment

General and Specific forces-It

Relativity

The Interrelation between business and its environment

The business enterprise is an open system It continuously interacts with its environment It takes inputs (such as raw materials capital labour energy and so on) from its environment transforms them into goods and services and sends them back to the environment

Fig 1 Business Environment Relationship

includes both the forces general forces are the economic social political legal and technological conditions which indirectly influence all business enterprise Specific forces are the investors customers competitors and suppliers which influence individual enterprise directly

Interrelatedness-Different elements of environment are interrelated for an example growing awareness for health care has increased the demand for health foods

Complexity- Business environment id complex in nature as the elements keep on changing example economic technological and other forces changes in demand for a product and service

Dynamic-Business environment is not static it keeps on changing

Uncertainty- Itrsquos very difficult to predict future events such as technology and fashion which occur fast and frequently

Business Studies

Human Resources Management

Human resource of an organisation are the aggregate of knowledge skills attitudes of people working in it

The management system which deals with human resources is called human resource management

Features of HRMbullComprehensive functionbullPeople-oriented

Question1) What do you mean by human

resource management Answer) Human resource management may be defined as that field of Management which has to do with planning organising and controlling the functions of procuring developing maintaining and utilising the labour force

bullAction oriented bullPervasive function bullContinuous function

2) Explain the features of HRM in brief

Answer)bullHuman Resource Management is concerned with managing people at work bull Human Resource Management is concerned with employees which bring people and organisations together so that the goals of each are met bullHuman resource management considered every employees as an individual and also promote their satisfaction and growth bull Human resource management is inherent in all organisations and at all levelsbullManagement of human resources are ongoing on never ending process which requires a constant alertness and Awareness of human relations

3) ldquoHR function is said to be pervasiverdquowhy

Answer) Human resource management is required in all organisations whether it is private or government organisations armed forces sports organisations etc It permeatsall the functional areas like production marketing finance research etc This from this feature of human resource management it can be said that it is pervasive in nature

Economics Demand Q1DEFINITION OF DEMANDIn economics demand is the quantity of a good that consumers are willing and able to purchase at various prices during a given period of timeQ2DEMAND CURVEIn economics a demand curve is a graph depicting the relationship between the price of a certain commodity and the quantity of that commodity that is demanded at that pricQ3LAW OF DEMANDIn microeconomics the law of demand states that conditional on all else being equal as the price of a good increases quantity demanded decreases conversely as the price of a good decreases quantity demanded increasesQ4ASSUMPTION of LAW OF DEMAND(i)No change in price of related commodities(ii) No change in income of the consumer(iii) No change in taste and preferences customs habit and fashion of the consumer( No expectation regarding future change in priceQ5MARKET DEMAND SCHEDULEIn economics a market demand schedule is a tabulation of the quantity of a good that all consumers in a market will purchase at a

given price At any given price the corresponding value on the demand schedule is the sum of all consumersrsquo quantities demanded at that priceQ6INDIVIDUAL DEMAND SCHEDULEIndividual demand schedule refers to a tabular statement showing various quantities of a commodity that a consumer is willing to buy at various levels of price during a given period of timeQ7 FACTORS AFFECTING INDIVIDUAL DEMAND FOR A COMMODITY

The factors that influence a consumerrsquos decision to purchase a commodity are also known as determinants of demand The following factors affect the individual demand for a commodity1 price of the commodity2 price of related goods3 income of buyer of the commodity4 tastes and preferences of the buyer1 Price of the CommodityYou must have observed that when price of a commodity falls you tend to buy more of it and when its price rises you tend to buy less of it when all other factors remain constant (lsquoother things remaining the samersquo) In other words other things remaining the same there is an inverse relationship between the price of a commodity and its quantity demanded by its buyers This statement is in accordance with law of demand which you will study in the later part of this lesson Price of a commodity and its quantity demanded by its buyers are inversely related only when lsquoother things remain the samersquo So lsquoother things remaining the samersquo is an assumption when we study the effect of changes in the price of a commodity on its quantity demanded2 Price of Related goodsA consumer may demand a particular good But while buying that good heshe also asks the price of its related goods Related goods can be of two types-(i) Substitute goods(ii) Complementary goods While purchasing a good prices of its substitutes and complements do affect its quantity purchased(i) Price of Substitute Goods Substitute goods are those goods which can easily be used in place of one another for satisfaction of a particular want like tea and coffee An increase in price of substitute good leads to an increase in demand for the given commodity and a decrease in price of substitute good leads to a decrease in demand for the given commodity It means demand for a given commodity is directly affected by change in price of substitute goods For example if price of coffee increases the demand for tea will rise as tea will become relatively cheaper in comparison to coffee(ii) Price of Complementary goods Complementary goods are those goods which are used together to satisfy a particular want like car and petrol An increase in the price of complementary goods leads to a decrease in demand for the given commodity and a decrease in the price of complementary goods leads to an increase in demand for the given commodity For example if price of petrol falls then the demand for cars will increase as it will be relatively cheaper to use both the goods together So demand for a given commodity is inversely affected by change in price of complementary goods3 Income of the Buyer of CommodityDemand for a commodity is also affected by income of its buyer However the effect of change in income on demand depends on the nature of the commodity under consideration In case of some goods like full cream milk fine quality of rice (Basmati rice) etc demand for these commodities increases when income of the buyer increases and

demand for these commodities decreases when income of the buyer decreases Such goods whose demand increases with the increase in income of the buyer are called normal goods But there are some goods like coarse rice toned milk etc whose demand decreases when income of buyer increases and their demand increases when income of the buyer decreases Such goods whose demand decreases with the increase in income of the buyer are called inferior goods Suppose a consumer buys 10 Kgs of rice whose price is ` 25 per Kg He cannot afford to buy better quality of rice because the price of such rice is ` 50 per Kg The consumer is spending ` 250 per month on the purchase of rice Now if income of the consumer increases and he can afford ` 350 on purchase of 10 Kg of rice Now he can afford to buy some quantity of rice say 6 Kgs whose price is ` 25 per Kg and may buy 4 Kgs of rice whose price is ` 50 per Kg Thus he will buy 10 Kgs of rice by spending ` 350 per month Therefore we may conclude that demand for normal goods is directly related to the income of the buyer but demand for inferior goods is inversely related to the income of the buyer4 Tastes and Preferences of the BuyerThe demand for a commodity is also affected by the tastes and preferences of the buyers They include change in fashion customs habits etc Those commodities are preferred by the consumers which are in fashion So demand for those commodities rises which are in fashion On the other hand if a commodity goes out of the fashion its demand falls because no consumer will like to buy it(5) Number of Buyers in the Market(Population)Increase in population raises the market demand whereas decrease in population reduces the market demand for a commodity Not only the size of population but its composition like age (ratio of males females children and old people in population) also affects the demand for a commodity It is because of needs of children young old male and female population differs(6) Distribution of Income and WealthIf the distribution of income and wealth is more in favour of the rich demand for the commodities preferred by the rich such as comforts and luxuries is likely to be higher On the other hand if the distribution of income and wealth is more in favour of poor demand for commodities preferred by the poor such as necessities will be more(7) Season and Weather ConditionsThis is generally observed that the demand for woolens increases during winter whereas demand for ice creams and cold drinks increases during summer Similarly market demand for umbrellas rain coats increases during rainy seasonQ8 REASONS FOR OPERATION OF LAW OF DEMAND WHY DEMAND CURVE SLOPES DOWNWARDNow we will try to explain why does a consumer purchase more quantity of a commodity at a lower price and less of it at a higher price or why does the law of demand operate ie why does the demand curve slope downwards from left to right The main reasons for operation of law of demand are1 Law of Diminishing Marginal UtilityAs you have studied earlier law of diminishing marginal utility states that as we consume more and more units of a commodity the utility derived from each successive unit goes on decreasing The consumer will be ready to pay more for those units which provide him more utility and less for those which provide him less utility It implies that he will purchase more only when the price of the commodity falls2 Income Effect

When price of a commodity falls purchasing power or real income of the consumer increases which enables him to purchase more quantity of the commodity with the same money income Let us take an example Suppose you buy 4 ice creams when price of each ice cream is ` 25 If price of ice creams falls to ` 20 then with same money income you can buy 5 ice creams now3 Substitution EffectWhen price of a commodity falls it becomes comparatively cheaper as compared to its substitutes (although price of substitutes has not been changed) This will lead to rise in demand for the given commodity For example if coke and Pepsi both are sold at ` 10 each and price of coke falls Now coke has become relatively cheaper and will be substituted for Pepsi It will lead to rise in demand for coke4 Change in Number of BuyersWhen price of a commodity falls some old buyers may demand more of the commodity at the reduced price and some new buyers may also start buying this commodity who were not in a position to buy it earlier due to higher price This will lead to increase in number of buyers when price of the commodity falls As a result demand for the commodity rises when its price falls5 Diverse Uses of a CommoditySome commodities have diverse uses like milk It can be used for drinking for sweet preparation for ice cream preparation etc If price of milk rises its use may be restricted to important purpose only This will lead to reduction in demand for other less important uses When price of milk falls it can be put to other uses also leading to rise n demand for itQ9 EXCEPTIONS TO THE LAW OF DEMANDYou have studied in law of demand that a buyer is willing to buy more quantity of a commodity at a lower price and less of it at a higher price But in certain circumstances a rise in price may lead to rise in demand These circumstances are called Exceptions to the Law of Demand Some important exceptions are1 Giffen GoodsGiffen goods are special type of inferior goods in which negative income effect is stronger than negative substitution effect Giffen goods do not follow law of demand as their demand rises when their price rises Examples of Giffen goods are jowar and bajra etc2 Status Symbol GoodsSome goods are used by rich people as status symbols eg diamonds gold jewellary etc The higher the price the higher will be the demand for these goods When price of such goods falls these goods are no longer looked at as status symbol goods and tehrefore therir demand falls3 NecessitiesCommodities such as medicines salt wheat etc do not follow law of demandbecause we have to purchase them in minimum required quantity whatever their price may be4 Goods Expected to be ScarceWhen the buyers expect a scarcity of a particular good in near future they start buying more and more of that good even if their prices are rising For example during war famines etc people tend to buy more of some goods even at higher prices due to fear of their scarcity in near future

Political Science

Constitution of India-The

Preamble

The preamble-

Preamble-

The preamble is the most precious part of the constitution We the people of India having solemnly resolved to constitute India into a Sovereign Socialist Secular Democratic Republic and to secure to all its citizensA preamble is an introductory and expressionary statement in a document that explains the documents purpose and underlying philosophy When applied to the opening paragraphs of a statute it may recite historical facts pertinent to the subject of the statuteNature and purpose of the constitution-Purpose of the Constitution dictates permanent framework of the government to form a more perfect union to establish justice and ensure peace of thenationconstitution provide principles how the government can run itself following the rules and laws written in the constitution of each state keeps them balanced

Answer the following questions-

1 What is preambleA preamble is an introductory and expressionary statement in a document that explains the documents purpose and underlying philosophy2 What is the nature and

purpose of the constitutionConstitution dictatespermanent framework of the government to form a more perfect union to establish justice and ensure peace of the nation

Homework-Learn

Accounts Compatibilty mode

1MEANING OF PARTNERSHIPPartnership is a form of business organisation where two or more persons join hands to run a business They share the profits and losses according to the agreement amongst them According to the Indian Partnership Act 1932 ldquoPartnership is relation between persons who have agreed to share profits of a business carried on by all or any one of them acting for allrdquo For example one of your friends has passed class XII from National Institute of Open Schooling (NIOS) and wants to start a business Heshe approaches you to join in this venture Heshe wants you to contribute some money and participate in the business activities Both of you if join hands constitute a partnership2CHARACTERISTICS1048698 Agreement A partnership is formed by an agreement The agreement may be either oral or in writing It defines the relationship between the persons who agree to carry on business It may contain the terms of sharing profit and the capital to be invested by each partner etc The written agreement is known as partnership deed1048698 Number of persons There must be at least two persons to form a partnership

The maximum number of partners in a partnership firm can be 50 according toCompanies Act 20131048698 Business The Partnership is formed to carry on business with a purpose of earning profits The business should be lawful Thus if two or more persons agree to carry on unlawful activities it will not be termed as partnership1048698 Sharing Profits The partners agree to share profits in the agreed ratio In caseof loss all the partners have to bear it in the same agreed profit sharing ratio10486981048698Mutual Agency Every partner is an agent of the other partners Every partner can bind the firm and all other partners by hisher acts Each partner will be responsible and liable for the acts of all other partners10486981048698Unlimited liability The liability of each partner except that of a minor is unlimited Their liability extends to their personal assets also If the assets of the firm are insufficient to pay off its debts the partnersrsquo personal property can be used to satisfy the claim of the creditors of the partnership firm10486981048698Management All the partners have a right to mange the business However they may authorize one or more partners to manage the affairs of the business on their behalf10486981048698Transferability of Share No partner can transfer hisher share to any one including hisher family member without the consent of all other partners3PARTNERSHIP DEEDAgreement forms the basis of partnership The written form of the agreement is which a document of partnership is It contains terms and conditions regarding the conduct of the business It also explains relationship between the partners This document is called partnership deed Every firm can frame its own partnership deed in which the rights duties and liabilities of the partners are stated in detail It helps in settling the disputes arising among the partners during the general conduct of business 4CONTENTS OF PARTNERSHIP DEEDThe partnership deed generally contains the following (i) Name and address of the partnership firm(ii) Nature and objectives of the business(iii) Name and address of each partner(iv) Ratio in which profits is to be shared(v) Capital contribution by each partner(vi) Rate of Interest on capital if allowed(vii) Salary or any other remuneration to partners if allowed(viii) Rate of interest on loans and advances by a partner to the firm(ix) Drawings of partners and interest thereon if any(x) Method of valuation of goodwill and revaluation of assets and liabilities on the reconstitution of the partnership ie on the admission retirement or death of a partner(xi) Settlement of disputes by arbitration(xii) Settlement of accounts at the time of retirement or death of a partner5IN ABSENCE OF PARTNERSHIP DEEDThe partnership deed lays down the terms and conditions of partnership in regard to rights duties and obligations of the partners In the absence of partnership deed there may arise a controversy on certain issues like profit sharing ratio interest on

capital interest on drawings interest on loan and salary of the partners In such cases the provisions of the Indian Partnership Act becomes applicableSome of the Issues are(i) Distribution of Profit Partners are entitled to share profits equally(ii) Interest on Capital Interest on capital is not allowed(iii) Interest on Drawings No interest on drawing of the partners is to be charged(iv) Interest on Partnerrsquos Loan A Partner is allowed interest 6 per annum on the amount of loan given to the firm by himher(v) Salary and Commission to Partner A partner is not entitled to anysalary or commission or any other remuneration for managing the business

History TOPIC-TOWARDS INDEPENDENCE AND PARTITION THE LAST PHASE (1935-1947)

SUB TOPIC-IMPORTANT POLITICAL DEVELOPMENTS ndash GROWTH OF SOCIAL IDEAS

Socialism is a political social and economic philosophyLike in other parts of the world the Russian revolution of 1917 served as a great inspiration for revolutionaries in India who at that time were engaged in the struggle for liberation from British ruleSocialist ideas led to the formation of communist party of IndiaJAWAHARLAL NEHRU Among the early Congress leaders Jawaharlal Nehru was very much impressed and influenced by the Socialist ideas He also learnt about the Economic activities of the Soviet Union after the Bolshevic Revolution 1917 He made full use of them in IndiaThe election of Jawaharlal Nehru and Subhas Chandra Bose showed the Left wing tendency within CongressJawaharlal Nehru demanded economic freedom along with political freedom of the people in order to end the exploitation of masses

Nehrus working committee included three socialists leaders The Lucknow session was a landmark in the evolution of socialist ideas of the congressSUBHAS CHANDRA BOSE ndash Subhas Chandra Bose had socialist leaning Both Jawaharlal Nehru and Subhas Chandra Bose were known as leftist Congress men Later on National Congress divided into Leftist and rightist campCONGRESS SOCIALIST Within the Congress some leaders formed the Congress Socialist partyPattavi Sitaramyya Sardar Patel Rajendra Prasad had hostile attitude towards the Congress Socialist partyJawaharlals attitude was hesitant

1 QUESTION ndash Mention name of two Congress leaders who had socialist leaning

1ANSWER ndash Subhas Chandra Bose and Jawaharlal Nehru2QUESTION- In which session of the congress Jawaharlal elaborated his Socialist ideas2 ANSWER ndash Lucknow and Faizpur Session in December 1935 and 19363QUESTION ndash Why Congress was sharply divided into leftist and rightist camp 3ANSWER ndash Subhas Chandra Bosersquos attempt to seek re election for congress presidentship in 1939sharply divided the National Congress into Leftist and Rightist camp4 QUESTION ndash Who was MN Roy 4 ANSWER ndash Manabendra Roy first formed the Communist Party of India outside the country at Tashkent in 19205QUESTION ndash Who formed the Congress Socialist Party within the Congress5 ANSWER ndash Jaya Prakash Narayan Achyut Patwardhan Acharya Narendra Dev Ram Mohan Lohia Aruna Asaf Ali6QUESTION ndash When was the Congress Socialist Party formed What was its object6 ANSWER ndash 1934The Congress Socialist Party sought to work out socialist programme through the Congress They joined hands with the Congress and wanted to carry

Subhas Chandra Bose being expelled from the congress after the Tripuri rift he formed Forward BlockThere were basic differences between the Congress Socialists and the communistsTRADE UNION ACTIVITIES Maximum working class people lived in Bombay and Calcutta The working and living conditions of those workers were very miserable In this situation Shasipada Banerjee NM Lokhande protested against the oppression of the working class peopleThe first Trade Union Madras Labour Union was formed in 1918 by BP WadiaIndustrial strikes took place in Kanpur Calcutta Madras Jamshedpur and Ahmedabad AITUC was formed in Bombay in 1927 The growth of Trade union among the workers was slow because of the fear of the dismissal of the jobIn the mean time the Moderates as well as Communists left AITUC and formed separate organization

on National struggle with the help of workers and peasant class of the society7 QUESTION ndash What was the name of the party founded by Subhas Chandra Bose7 ANSWER- Forward Block8QUESTION ndash Who was Shasipada Banerjee8 ANSWER ndash Shasipada Banerjee was a radical Brahmo He founded a working menrsquos club to protest against exploitation of the British rulers towards the working class of India9 QUESTION ndash What was the weekly published by NM Lokhande9ANSWER- Dinabandhu10 QUESTION ndash Who founded Bombay Mill-Hands Association and in which year10 ANSWER- NM Lokhande in189011 QUESTION- Who was BP WadiaANSWER- BPWadia was the founder of Madras Labour Union in191812 QUESTION- What was the name of the first labour union of India12 ANSWER- Madras Labour Union13 QUESTION Who founded the Majur Mahajan 13 ANSWER GANDHIJI14 QUESTION What was the full form of AITUC When it was formed14 ANSWER All India Trade Union Congressin 192715QUESTION Who formed the Red Trade Union Congress and in which year15ANSWER The Communists formed the Red Trade Union Congress16 QUESTION What do you mean by Socialism16 ANSWER Socialism describes any political and economic theory that says the community rather than individuals should own and manage property and natural resources

Subject Eng Literature (The Tempest ndash William Shakespeare) Topic Act III Scene 3 Lines 1 to 52 (Line 52 ndash Brother my lord the Duke Stand to and do as we) Date 13th April 2020 (4th Period)

[Students should read the original play and also the paraphrase given in the school prescribed textbook]Summary Questions amp Answers

o Alonso Sebastian Antonio Gonzalo Adrian Francisco and others wandered about the island in search of Ferdinand and gets tired and hungry of the toil and at the same time gives up all hope of finding him

o Antonio and Sebastian are happy that Alonso is out of hope and decide to make another attempt on his life that night when being so tired they will be sleeping soundly

o Suddenly a solemn and strange music is heard in the air and several strange shapes enter bringing in a banquet These strange shapes then dance round it with gestures of salutation and then inviting the King to eat they depart

o Seeing this strange scene all are inclined to believe the tales told by travelers that there truly are ldquounicornsrdquo and ldquothe phoenixrsquo thronerdquo

1 ALONSO What harmony is this My good friends hark (L18-27)

GONZALO Marvellous sweet music

[Enter several strange shapes bringing in a banquet

they dance about it with gentle actions of salutation

and inviting the King and his companions to eat they depart]ALONSO Give us kind keepers heavens What were theseSEBASTIAN A living drollery Now I will believe

That there are unicorns that in Arabia

There is one tree the phoenixrsquo throne one phoenix

At this hour reigning thereANTONIO Ill believe both

And what does else want credit come to me

And Ill be sworn rsquotis true Travellers neer did lie

Though fools at home condemn rsquoem

(a) How did Prospero present an amazing spectacle before Alonso and his companions

Using his magic powers Prospero ordered strange shapes to lay a banquet before Alonso and his companions The shapes brought several dishes with tasty eatables in them They placed the dishes on a table before Alonso and his companions Then the strange shapes began to dance gracefully around the banquet While dancing they made gestures inviting them to eat the food Then suddenly the shapes disappeared(b) Who were the guests at the strange banquet Describe the lsquoliving drolleryrsquo

Alonso Sebastian Antonio Gonzalo Adrian and Francisco were the guests at the strange banquet

The term ldquoliving drolleryrdquo refers to live entertainment show In this context when Alonso the King of Naples Sebastian his brother Antonio the treacherous brother of Prospero Gonzalo the kind and loyal councillor to the King Adrian and Francisco came to the island they were hungry and weary in their spirits They heard a solemn and strange music They were shocked to see several strange shapes bringing in a banquet and these shapes danced about it with gentle action of salutation inviting the King and his companions to eat After this Sebastian described this show as lsquoliving drolleryrsquo(c) What is lsquophoenixrsquo What are lsquoUnicornsrdquo

The term lsquophoenixrsquo refers to a mythical Arabian bird which lived alone and perched on a solitary tree After one hundred years it expired in flames and rose again from its own ashes

lsquoUnicornsrsquo refers to the mythological four-footed beasts having horns in the centre of their foreheads When the horns are ground into powder the powder was believed to be

an aphrodisiac(d) How does Sebastian explain the puppet show OR Why does the speaker now believe in unicorns and phoenix

Sebastian finds several strange shapes bringing in the banquet They invite the king and his party for dinner and soon depart He tells that if such a strange sight can be a reality there is nothing incredible in the world and from the present moment he will believe anything He says that it is a strange dumb show enacted not by puppets but by living beings It is stranger than a travellerrsquos tale Seeing such a thing

before his own eyes he will no longer disbelieve the story about unicorns and phoenix(e) How do the other characters present respond to this living drollery

At the sight of the lsquoliving drolleryrsquo like Sebastian Gonzalo and Antonio too acted strangely Antonio told that he too now believes in unicorns and phoenix and anything else that seems to be incredible He too now believes in travellersrsquo tales Gonzalo told that if he would report those happenings in Naples nobody will believe him He considers that those gentle shapes were gentler in manner in comparison to the living beings Alonso was at first sight suspicious and told them that those strange shapes conveyed their meaning in expressive gestures when they seemed to lack speech by their movements and sounds Francisco was amazed at their mysterious disappearance

2 ALONSO Not I

(Line 43-52)GONZALO Faith sir you need not fear When we

were boysWho would believe that there were mountaineers

Dewlapped like bulls whose throats had hanging at rsquoem

Wallets of flesh Or that there were such men

Whose heads stood in their breasts Which now we find

Each putter-out of five for one will bring us

Good warrant ofALONSO I will stand to and feed

Although my lastmdashno matter since I feel

The best is past Brother my lord the Duke

Stand to and do as we

(a) How does Alonso respond at the spectacle of the shapes which were sent to them at the instruction of Prospero

After seeing the strange sight of appearing and disappearing of the shapes sent by Prospero to arrange a banquet for them Alonso says that his surprise at having seen those creatures is infinite and he is fully justified in feeling so much surprise He thinks that their shapes their gestures and the sounds they made were indeed amazing Although they do not possess the gift of speech yet they were able to convey their

thoughts by means of their gestures only

(b) What does Prospero say about the views expressed by Alonso regarding the shapes What does Francisco think about the shapesAfter hearing Alonsorsquos views about the shapes Prospero says that this manrsquos praise of the spirits is rather hasty He means to say that Alonso has shown great haste in reaching the conclusion about the shapes Francisco is amazed to see that those shapes disappeared in a mysterious way(c) What does Sebastian ask Alonso to doSebastian tells Alonso that the shapes having disappeared should not matter to them because they have left the eatables behind He asks Alonso to enjoy eating as they are extremely hungry but the king does not accept his offer of enjoying the dishes(d) How does Gonzalo try to dispel Alonsorsquos fear of those strange shapes What kind of references does he give to AlonsoGonzalo says that those who have travelled abroad have reported seeing even stranger sights than these shapes that Alonso and his companions have beheld Hence there is no reason to feel afraid of these shapes Gonzalo further adds that in his younger days he had heard strange stories from travelers and Alonso might have heard similar stories For instance it was said that there existed a certain race of

human beings who had huge lumps of flesh hanging at their throats and who therefore resembled bulls Then Gonzalo tells about a race of human beings whose heads were located at their breasts Gonzalo says that such stories were not believed by most people in those days but now-a-days these stories have become common(e) Explain the following lsquoEach putter-out of five for onersquoEnglish travellers often insured their trips with London brokers Those that went on foreign travels those days used to deposit a certain amount with some firm or company in London before their departure If the travelers failed to return the money was forfeited by the company with which it had been deposited But this money was repaid five-fold if the travelers returned safe and sound In this way a traveler stood a great chance of recovering the entire cost of his

travels(f) Give the explanatory meanings of the following expressions in the context of the above extract (i) Dewlapped (ii) Wallets of flesh

(iii) Putter-out(i) Dewlapped having big lumps of flesh at the necks(ii) Wallets of flesh large masses of flesh looking like bags(iii) Putter-out to invest money before commencing the travel

  • General methods of preparation of hydrogen
  • Chapter Dimensional Analysis (Summary)
    • Properties of Charge
Page 29:   · Web viewSubject. Topic. Summary. Execution. Hindi. व्याकरण. शरीरके अंगो के नाम लिखिए. 1) आँख 2) नाक 3

o In this scene Portia Nerissa Lorenzo Jessica and Balthazar appear

o Portia requests Lorenzo and Jessica to be in charge of her house during her absence from Belmont because she and Nerissa have decided to spend the days in meditation and also in visiting the holy places in the neighbourhood of Belmont She has already instructed her people to acknowledge both Lorenzo and Jessica as master and mistress of house during her absence Lorenzo and Jessica gladly agree to look after the house of Portia

handover the letter from Antonio to Bassanio The letter carried the bad news about Antoniorsquos arrest for non-payment of loan taken from Shylock Hence Salerio might have preferred company to break this bad news to Bassanio He is referring to Portia as Madam(b) What does Portia say on hearing the above extract

Portia says that she has never regretted doing good to others Friends who spend a lot of time together and really are there for each other have many traits in common As Antonio is Bassaniorsquos best friend saving him is like saving Bassanio who is like her own soul She asks Lorenzo to take care of management of the house till Bassanio is back(c) What does Portia send with Bassanio and why

On hearing about Antoniorsquos troubles on account of Bassanio her husband Portia immediately sends him with enough gold to repay the debt many times over to Venice to help Antonio out of his misfortune

(2) Lorenzo Madam with all my heart (Line 36-40)

I shall obey you in all fair commands

Portia My people do already know my mindAnd will acknowledge you and JessicaIn place of Lord Bassanio and myselfSo fare you well till we shall meet again

(a) Where are Lorenzo and Portia at this time What lsquofair commandsrsquo are given to Lorenzo

Lorenzo and Portia are at Belmont during this scenePortia reveals to Lorenzo that she has sworn to contemplate in prayer at a monastery around two miles away until her husband returns from Venice She tells him that Nerissa would accompany her and asks him to manage the house with Jessica till things are settled In response Lorenzo tells her that he would be obliged to do whatever she asks him to do(b) Where is Portia actually going and why

Portia tells Lorenzo that she would live a life of contemplation and pray at a monastery which is two miles away from her place In reality Portia plans to go to Venice in disguise with Nerissa and argue the case in defense of Antonio She is very sure that her plan would succeed

ClassXI (ScienceHumanitiesCommerce)Subject Topic Summary Execution

Computer Science

(APC)

Ch ndash 1 Numbers

(Numbers in different bases and

their Arithmatical operations)

Number System In computers Number System is defined as a writing system to represent the numbers in different ways ie we are using different symbols and notations to represent numbers There are four ways we can represent the number ndash Binary Decimal Octal and Hexadecimal

Decimal Number SystemThis number system consist 10 digits These are 0 1 2 3 4 5 6 7 8 amp 9

Binary Number SystemThis number system has only two digits these are 0 and 1 Here 0 stands for off while 1 stands for on

Octal Number SystemThis number system has 8 digits these are 0 1 2 3 4 5 6 amp 7

Hexadecimal Number SystemThis number system has 16 digits these are 0 1 2 3 4 5 6 7 8 9 A B C D E F Here the value of the alphabets are as follows A=10 B=11 C=12 D=13 E=14 F=15

Rules for conversion decimal number to Binary1 Divide the decimal number by 22 If the number will not divide equally by 2 then round down the answer to the nearest whole number (integer)3 Keep a note of the remainder it should be either 0 or 14 Keep repeating the above steps dividing each answer by 2 until you reach zero5 Write out all the remainders from bottom to top This is your binary solution

For example Lets convert 32 to binary 2 32 2 16 - 0 2 8 - 0 2 4 - 0 2 2 - 0 2 1 - 0 0 - 1

The binary equivalent of 3210 is 1000002

Try the follwing youself1 2410

2 4810

3 1210

History GROWTH OF NATIONALISM

The second half of the 19th century witnessed growth of political consciousness and a sense of Nationalism among the IndiansThere were various factors for growth of Indian Nationalism- As a result various political associations were formed in different provinces by the educated Indians Surendranath Banerjee organized a meeting of National conference at Calcutta Ultimately the National Congress was founded in Bombay in 1885This body became the vanguard of Indian struggle for freedom The congress leaders were known as moderates because they followed a policy of prayer and petition A large number of Indian leaders had experienced in political agitation The Political situation of England was also changed Moreover increasing revolutionary activities in Maharashtra Punjab and Bengal became serious concern to the British Government In this

QUESTION1 What do you mean by Nationalism ANSWER 1 Nationalism is defined as loyalty and devotion to own nation especially a sense of national consciousnessQUESTION 2 What are the causes of nationalism ANSWER 2 There were various factors for growth of nationalism

1 Spread of western education2 The progress of vernacular press and

patriotic literature3 The economic exploitation of our

country by the colonial rulers4 International affairs

QUESTION 3 Who organized National conference in Calcutta in 1883 ANSWER 3 Surendranath BanerjeeQUESTION 4 When did Indian National Congress formANSWER 4 Indian National Congress was formed in 1885 in BombayQUESTION 5 Who were ModeratesANSWER 5 The Early Nationalists were also known as Moderates Their emergence marked

background Lord Curzon became Viceroy in India He had no respect for the Indian National Congress

the beginning of the organized national movement in India They believed in British justice and were loyal to them They followed a policy of prayer and petition They demanded constitutional reforms of our country Impotant Moderate leaders were Pherozshah Mehta Dadabhai Naorozi and Surendranath Banerjee etcQUESTION 6 What do you know about Extremism in Indian National movementANSWER 6 In the beginning of 20th century a new class of national leaders emerged in India which was different from the moderate groups They started more aggressive movement against the British empire The goal of extremists was ldquoswarajrdquo Important extremist leaders were Bal Gangadhar Tilak Lala Lajpat Rai Bipin Chandra Pal etcQUESTION 7 Mention the places which were the main centres of Revolutionary movementANSWER 7 Maharashtra Bengal and Punjab

Physics

Chapter Dimensional Analysis

(Summary)

The dimensions of a physical quantity are the powers to which the fundamental units are raised in order to obtain the derived unit of that quantit

The physical quantites lengthmasstime are represented by [L] [M] [T] resp let they are raised to powers ( dimesions) abc resp then any physical quantity can be represented by [ La Mb Tc ] Examples

1 Area area = L x B = [L] x [L] = [M0 L2 T0 ]

2 Density density = massvolume = [M][L3] = [ M L-3]

3 Velocity velocity = distancetime = [L][T] = [LT-1]HW Try to find out dimension of acceleration Acceleration = velocity timeNB One can find the SI Units Using Dimension Analysis Such as for area we have [L2] so its SI unit is m2

Biology Topic ndash Chp-1 The living world

Today we will start the first chapter the living world Here we discuss about the characteristics of living organism and what are the difference between them and nonliving substances We also discuss about the contribution of different Scientists

There are over 500000 species of plants andover a million species of animal are present on earth Some 15000 new species were discovered every yearQ1 What is a living organismbull A living organism is primarily physico -chemical material that demonstrate a high degree of complexity is capable of selfRegulation possesses a metabolism and perpetuates itself through timeQ2 What are the differences between livingand non-livingsi) Compared with non-living living organisms

have more complex organised structure and their use of energy is more controlled amp efficientii) Living things reproduce their own kind by forming new cells which contains copies of their genesiii) Each organism has some degree of homeostasisie it is able to make adjustments so that internal environment remains constantQ3 Write contributions of following Scientists i) Aristotle - One of the first theories in Biology places all living things in a hiearchieii) AV Leeuwenhoek - was the first to observe living single celled organisms under microscopeii) Carolus Linnaeus - developed the binary system for naming of organisms and classificationiii) Geregor Johann Mendel ndash discoverbasic principles of inheritanceHomework i) C Darwin ii)Schleiden

Math Trigonometric functions

1 Overviewi) Trigonometry The word lsquotrigonometryrsquo is derived from the Greek words lsquotrigonrsquo and lsquometronrsquo which means measuring the sides of a triangle An angle is the amount of rotation of a revolving line with respect to a fixed line Usually we follow two types of conventions for measuring angles ie a) Sexagesimal system b) Circular system In Sexagesimal system the unit of measurement is Degree In Circular system the unit of measurement is Radian ii) Relation between degree and radianThe ratio of circumference of a circle to its diameter is always a constant This constant ratio is a number denoted by π which is taken approximately as 227The relationship between degree amp radian measurements is as follows2 right angles = 180deg= π radians1radian = 180degπ=57deg16(approx) 1deg=π180 radianiii) Length of an arc of a circleIf an arc of length s subtends an angle θ radians at the center of a circle of radius r then s=rθiv) Area of a sector of a circleA sector is like a pizza slice of the

Q) Express the following angles in radiana) 45deg b) 40deg3730Ans a) We have 180deg=π radiansi e 45deg= πtimes45180 radian = π4 radiansb) 40deg3730= 40deg37+3060 minute= 40deg 37 +12 minute= 40deg+ 752 minute=40 + 75(2times60) degree=3258 degreeNow 180deg=π radianie 3258 degree= (πtimes325) (180times8) radians = 65π288 radiansQ) A circle has a radius of r=12 meters What is the length of an arc traced out by a 60deg angle in the center of the circleAns In this problem we know both the central angle (60deg) and the radius of the circle (12) All we have to do is plug those values into our equation and we get

s = 2π(12)(60360)s = 24π6s = 4πSo the length of an arc traced out by a 60deg angle in a circle with a radius of 12 meters equals 4π meters asymp 1257 metersQ) Find the area of the sector with a central angle 30deg and a radius of 9cmAns GivenRadius r = 9 cmAngle θ = 30degArea of the sector = θ360degtimesπr2

= 30360degtimes227times92=2121cm2

circle It consists of a region bounded by two radii and an arc lying between the radiiThe area of a sector is a fraction of the area of the circle This area is proportional to the central angle In other words the bigger the central angle the larger is the area of the sectorArea of Sector = θ2 times r2 (when θ is in radians)

Area of Sector = θ times π360 times r2 (when θ is in degrees)

COMMERCE

CLASSIFICTION OF HUMAN ACTIVITIES-ECONOMIC AND NON-ECONOMIC

Welcome to the new sessiontoday we are going to start the first chapter of Class XI The name of the chapter that we are going to start is

lsquoClassification of Human Activities ndasheconomic and non-economicrsquo

Now let us start the chapter by considering human beings and the activities they perform throughout the day

Human activities means all those activities that human beings undertake to satisfy their wants

Human wants on the other hand are the desire of human beings for goods (vegetables fruits rice etc) and services (services of doctors teachers lawyers etc) that they require to live

Now these human activities continue throughout life as human wants are unending unlimited and recurring as human beings desire for better living throughout their lives

Now human activities can be classified into two categories

Human activities

Economic activities Non-economic activities

Economic activities are

Questions1 What are human activities

Answer Human activities mean all those activities that human beings undertake to satisfy their wants

Example A man working in an office

A boy playing in the garden

2What are the characteristics of human activitiesAnswer the characteristics of human activities are as follows

Human activities are undertaken by men women and children and these activities involve human efforts

Human activities are undertaken to satisfy human wants which are unlimited

Human activities continue throughout life

Human activities are performed for both earning money and personal satisfaction

3What is economic activitiesGive example

Answer Economic activities are undertaken by human beings with the object of earning money acquiring wealth and thereby satisfying human wantsExample

Selling of goods by a shop keeper to his customer

A clinic run by a doctor Service of a teacher in school or college

undertaken by human beings with the object of earning money and acquiring wealth

These activities result in the production of economic goods and services

Example Human activities(ie working in factories officesshops) which produce direct economic benefits

Non-economic activities are inspired by human sentiments and emotions such as love for the family desire to help the poor and love for the country

Thus these human activities (eg praying playing sleeping) produce no direct economic benefits and they are also not related to earning money and acquiring wealth

4 What are the characteristics of economic activities

Answer The characteristics of economic activities are as follows

Economic motiveEconomic activities are undertaken to earn money and acquire wealth

ProductiveEconomic activities involve productiondistribution and exchange of goods and services to create wealth

Economic growthEconomic activities determine the level of economic development of a country and standard of living of its citizens

Socially desirableEconomic activities are socially desirable for society

Economic resourcesEconomic activities make use of all the economic resources such landlabourcapital etc

5 What do you mean by non-economic activitiesExampleAnswerNon-economic activities are inspired by human sentiments and emotions such as love for the family desire to help the poor and love for the countryThese activities are not undertaken for monetary gain but for onersquos satisfaction and happinessExample

a mother looks after her children

a student donates blood8 Differentiate between Economic activities and Non-economic activities

Economic activities

Non-economic activities

1to earn living and acquiring wealth2Result can be measured in terms of money

3ExampleBusinessprofession and employment

1 to obtain some satisfaction

2Result cannot be measured in terms of money

3ExampleFamily-orientedreligious socialCultural and national

BUSINESS STUDIES

BUSINESS ENVIRONMENT

Welcome to the new sessionToday we are going to start the first chapter and the name of the chapter is Business Environment

In todayrsquos world every business enterprise is a part of the society It exists and operates in association with various groups in society such as customers suppliers competitors banks and financial institutions government agencies trade unions media and so on All these groups influence the functioning of business in one way or the other They constitute the environment of businessConcept of Business Environment

The term lsquobusiness environmentrsquo refers to the sum total of all individuals institutions and other forces that lie outside a business enterprise but that may influence its functioning and performance

The main features of business environment

Totality of External forces General and Specific forces Interrelatedness Complexity Dynamic Uncertainty Relativity

The Interrelation between business and its environment

The business enterprise is an open system It continuously interacts with its environment It takes inputs

Prepare the following questions from todayrsquos assignment

1 What do you mean by business environment

The term lsquobusiness environmentrsquo means the aggregate of all forces factors and institutions which are external to and beyond the control of an individual business enterprise but they may influence its functioning and performance Business environment is the macro framework within which a business firm a micro unit operates It consists of several interrelated and interacting elements

2 Explain the main features of business environment in brief

Totality of External forces-Business environment is the sum total of all things external to a business environment

General and Specific forces-It includes both the forces general forces are the economic social political legal and technological conditions which indirectly influence all business enterprise Specific forces are the investors customers competitors and suppliers which influence individual enterprise directly

Interrelatedness-Different elements of environment are interrelated for an example growing awareness for health care has increased the demand for health foods

Complexity- Business environment id

(such as raw materials capital labour energy and so on) from its environment transforms them into goods and services and sends them back to the environment

Fig 1 Business Environment Relationship

complex in nature as the elements keep on changing example economic technological and other forces changes in demand for a product and service

Dynamic-Business environment is not static it keeps on changing

Uncertainty- Itrsquos very difficult to predict future events such as technology and fashion which occur fast and frequently

Economics Basic Economic ConceptsSub topic

Microeconomics and

Macroeconomics

Welcome to the new sessiontoday we are going to start the first chapter of Class XI The name of the chapter that we are going to start is Basic Economic concepts

Now Economics covers the study of human activities Human activities are those activities which are performed by humans to satisfy their wants

Thus Human wants are unlimited and therefore economic activities such as production exchange and consumption are needed in order to satisfy those wants

The study of economics is divided largely in two parts which areMicroeconomics and Macroeconomics

SUBJECT- MATTER OF ECONOMICS

MICROECONOMICS MACROECONOMICS

Questions1Who has coined the words micro and macro economics

Answer Ranger Frisch coined the words lsquomicrorsquo and lsquomacrorsquo in 1933 to denote the two branches of economic theory namely microeconomics and macroeconomics

2What is microeconomicsAnswer It is the study of behaviour of individual decision ndash making unit such as consumers firms etc

3 What is macroeconomicsAnswer Macroeonomics is the study of overall economic phenomena like employment national income etc

4 What is the importance of microeconomicsAnswer

Microeconomics helps in formulating economic policies which enhance productive efficiency and results in greater social welfare

It helps the government in formulating correct price policies

It explains the working of a capitalistic economy where individual units(producers and consumers ) are free to take their own decision

Micro means a small part in

microeconomics we do not study the whole economy Hence we study an individual consumer and his or her choices and a producer and his or her profit maximizing decisions in the market Thus it does not mirror what happens in the economy as a whole

Macroeconomics on the other hand studies the economy as a whole It is concerned with aggregate and depicts the entire picture of the economyMacroeconomics deals with the national income aggregate investment aggregate consumption etc

Features of Microeconomics It deals with small

parts of the country Hence it looks at

individual consumers firms and industries

It deals with individual income consumption and savings

It studies the determination of price of any product or factors of production

It deals with the working of market via the price mechanism which is nothing but the determination of price and quantity of a commodity by the forces of demand and supply

Features of Macroeconomics

It deals with the study of the economy as a whole

It is concerned with

5 Give a limitation of microeconomics Microeconomics fails to explain the

functioning of an economy as a whole It cannot explain unemployment illiteracy and other problems prevailing in the country

6 What is the importance of macroeconomics It gives overall view of the growing

complexities of an economic system It provides the basic and logical

framework for formulating appropriate macroeconomic policies (eg for inflation poverty etc )to direct and regulate economy towards desirable goals

7What is the limitation of macroeconomics It ignores structural changes in an

individual unit of the aggregate

8 Differentiate between Microeconomics and Macroeconomics

Microeconomics Macroeconomics

the study of aggregates

National income aggregate savings and aggregate investments are major concepts dealt within macroeconomics style

It studies the determination of general price levels

It investigates into the problem of unemployment and the achievement of employment

It studies the aspect of decision making at the aggregate and national levels

It includes all growth theories whether related to developed or developing economies it also includes the study of economic systems and the working of the economy under different systems

Note Both Micro and macro economics are complementary and should be fully utilized for proper understanding of an economy

1It studies economic aspect of an individual unit2It deals with individual incomeConsumption and savings

3 It facilitates determination of price of any product or factors of production

4 Itrsquos scope is narrow and restricted to individual unit

1It studies the economy as a whole

2It deals with the national income aggregate consumption and aggregate savings3 It facilitates determination of general price level in an economy

4 Itrsquos scope is wide as it deals with economic units on the national level

ACCOUNTS

Introduction to Accounting and Book-keeping

Today I am going to share you the meaning of Accounting and Book-keeping and its related terms bullAccounting bullBook Keeping bullAccountsbullTypes Of Accounts bullAccounting Cycle

bull Meaning of accounting

Ans ) Accounting is the art and science of recording classifying and summarising monetary transactions

bull Meaning of Book-keeping

Ans) Bookkeeping is the art of recording business transactions with the view of having a permanent record of them and showing their effect on wealth

bull Meaning of account

Ans) The term account means a record of

business transactions concern a particular person of firm asset or income or expense It is a summarised record of all transactions which take place in an accounting year

bull Types of accountsPersonal accounts ndash Personal accounts relating

to person and Organisation are known as personal accounts Example Ramrsquos Account ABC amp Co Account etc

Real account - The accounts related to tangible and intangible assets are called real accounts Example Cash Account Furniture Account etc

Nominal account- Accounts related to expenses losses incomes and gains are known as nominal accounts Example Wages Account Salary Account Discount Account etc

bull Accounting cycle Accounting cycle refers to a complete sequence of accounting activities It begins with recording of transactions and ends with the preparation of a balance sheet

Chemistry TopicAtomic Structure

Thomsonrsquos atomic modelThomson (1898) was the first to propose the model of an atomHe proposed that an atom can be regarded as a uniform sphere of positive electricity in which requisite number of electrons are embedded evently to neutralize the positive chargeThis is just like plums embedded in a pudding or seeds evently distributed in red spongy mass of a watermelonThis model of atom is known as ldquoPlum-Pudding modelrdquo or

Q1)What is the fundamental constituents of atomAns Electron Proton and neutrons are the fundamental constituents of atomQ2)What is the value of fundamental unit of electricityAnsThe charge carried by one electron is sad to be the fundamental unit of electricityIts magnitude is 48times10-10esuOr 1602times10-19C Q3)Name the element containing no neutronAnsOrdinary hydrogen atom or protium 1H1

Types of AccountPersonal AccountReal AccountNominal AccountBalance Sheet (opening)

ldquowatermelon modelrdquoThis model could explain the electrical neutrality of an atom but failed to explain the result of scattering experiment carried out by Rutherford in 1911So it was rejected ultimately

Q4)Why is an electron called universal particleAns Itrsquos mass and Charge are independent of its source

EVS Chapter 1 ndash Modes of Existence

Modes of existence When one speaks normally about the mode of existence of some group or individual one refers to their customs their mode of being their ethology their habitat in some way their feeling for a placeDifferent modes of exixtence are ndash

1 Hunting ndashGathering2 Pastoral3 Agricultural4 Industrial

1 Hunting and gathering Hunting and gathering mode of existence is characterized by obtaining food from hunting wild animals including fishing and gathering wild plants From their earliest days the hunter-gatherer diet included various grasses tubers fruits seeds and nuts Lacking the means to kill larger animals they procured meat from smaller game or through scavenging

Societies that rely primarily or exclusively on hunting wild animals fishing and gathering wild fruits berries nuts and vegetables to support their diet are called hunting and gathering societies

At least this used to be practice of human beings before agriculture is invented As their brains evolved hominids developed more intricate knowledge of edible plant life and growth cycles

Q) Write the features of Hunting ndash gathering societiesAns - There are five basic characteristics of hunting and gathering societies

i The primary institution is the family which decides how food is to be shared and how children are to be socialized and which provides for the protection of its members

ii They tend to be small with fewer than fifty members

iii They tend to be nomadic moving to new areas when the current food supply in a given area has been exhausted

iv Members display a high level of interdependence

v Labor division is based on sex men hunt and women gather

Political Science

Introduction to political science

Political science occasionally called politology is a social science which deals with systems of governance and the analysis of political activities political thoughts associated constitutions and political behaviorThe study of political science involves the study of both the

Answer the following questions-1 What is political science

Political science occasionally called politology is a social science which deals with systems of governance and the analysis of political activities political thoughts associated constitutions and political behavior

2 Short notes-

traditional and modern theories of politicsTraditionalClassical political sciencepolitical theory-Traditional political science is the study of politics before Second World War The methodology to study Politics was traditional (legal formaletc) the definition of politics traditional (Politics begins and end with state)area of study (constitution state machinery)was traditionalModern Political scienceModern political theory-Modern Political Theory critically examines the contemporary state of political theory making an assessment of the achievement and limitations of the Behavioural Revolution in its totality and reviews objectively the major paradigms and conceptual frameworks adopted by the disciplineContemporary attempts at the development of an integrated political theory involving the use of both traditional and modern concepts approaches and theories-Around late 1960s several political scientists realized the importance of both the traditional political theory and modern Political theory They began building an integrated theory of politics involving a systematic mixture of traditional and modern studies of politics It was held that the study of a complex and vast field like politics needs both traditional as well as

Classical political theory Modern Political theory

Homework-Learn

modern concepts and approaches for studying itrsquos all aspects

Subject Eng Literature (The Tempest ndash William Shakespeare) Topic Act I Scene 1 Lines 1 to 32 (Line 32 ndash Gonzalo hellip If he be not born to be hanged our case is miserable) Date 13th April 2020 (3rd Period)

[Students should read the original play and also the paraphrase given in the school prescribed textbook]Summary Questions amp Answers

[SUMMARY OF THE ENTIRE SCENE]

o The play starts with the scene of a severe storm at sea Alonso (King of Naples) Sebastian (Alonsorsquos brother) Ferdinand (Alonsorsquos son) Gonzalo Antonio (the usurping Duke of Milan) are in a ship in the midst of the storm

o The mariners are trying their best to control the vessel from running aground and are totally following the orders of their Master the Boatswain They have scant success

o The mariners become extremely unhappy and annoyed when most of the passengers arrive on the deck thereby hampering their effort to save the ship There is serious confrontation between them and the passengers who are part of the Kingrsquos entourage

o The mariners could not save the ship

SUMMING-UP

(i) Vivid description of the scene which gives a realistic description of terror and confusion of a tropical storm

(ii) Shows Shakespearersquos accuracy of knowledge in describing the naval operations and also matters of seamanship

(iii) The opening scene justifies the title ndash The Tempest

UNANSWERED QUESTIONS

(i) The King always travels with his entire fleet including his soldiers Where

(1) GONZALO Nay good be patient (Line 15-26)BOATSWAIN When the sea is Hence What cares these

roarers for the name of the king To cabin silence Trouble us not

GONZALO Good yet remember whom thou has aboardBOATSWAIN None that I more love than myself You are a

councillor if you can command these elements to silence and work

the peace of the present we will not hand a rope more use your authority If you cannot give thanks you have

lived so long and make yourself ready in your cabin for the mischance of the hour if it so hap [To the Mariners]

Cheerly good hearts [To Gonzalo] Out of our way I say

(a) To whom is the boatswain speaking What does he mean by lsquoNone that I more love than myselfrsquo

The Boatswain is speaking to Gonzalo the honest old councilor of the Duke of MilanBy using the words ndash lsquoNone that I love more than I love myselfrsquo means that for the Boatswain nobody is dearer to him than his own life

(b) What were the conditions that made the boatswain react in this way

The Boatswain reacts in this way because the storm is at sea and Alonso King of Naples Sebastian his brother Ferdinand his son Gonzalo Antonio the usurping Duke of Milan on board are in distress and in panic Thus they have rushed to the deck interrupting the work of the mariners

(c) What hope does Gonzalo take from the attitude of the boatswain

The insolent and authoritative attitude of Boatswain makes Gonzalo feel comforted He tells that there are no signs that the Boatswain will be drowned But his facial appearance and attitude shows that he is destined to die on land by hanging which in effect means that all on board will be saved Otherwise all the persons on board are doomed

(d) How can they lsquomake yourself ready in your cabinrsquo For what were they asked to make ready themselves

In order to make themselves ready in their cabin the

were the other ships

(ii) Why was the ship in that area Where was it coming from or going where

(iii) The ship broke apart What happened to those who were in the ship

passengers on board must prepare for death which they will possibly soon have to meetThey can retire to their cabins and offer prayers to the Almighty to save them from drowning

(e) What does the boatswain say when he is asked to be patient What does he order to the royal party

When the boatswain is asked to be patient and remain calm he says that he will be patient only when the storm will be over and the sea will be calm but as long as the storm blows and there is danger to the ship he cannot think of being patient He orders the royal party to go to the cabin and leave the mariners to their work

(2) GONZALO I have great comfort from this fellow (Line 27-36)

Methinks he hath no drowning mark upon him his complexion is perfect

gallows Stand fast good Fate to his hanging Make the rope of his destiny our cable for our own doth little advantage If he be not born to be hanged our case is miserable

(a) Why does Gonzalo regard the Boatswain in the midst of danger

In the midst of danger Gonzalo regards the boatswain because he feels that the Boatswain is a source of comfort and is bent upon to do his work sincerely which in this case is saving the ship and its passengers from the severest of raging storm

(b) What reasons does Gonzalo give when he says that none in the ship will die of drowning

Gonzalo is almost sure that none in the ship will die by drowning His says that there is no mark on the face of the boatswain that indicates that he will die by drowning On the other hand the lines on his face are strong indications that he will be hanged to death Therefore there shall be no danger of the shiprsquos sinking

(c) Explain the following ldquoStand fast good Fate to his hanging Make the rope of his destiny our cable for our own doth little advantage If he be not born to be hanged our case is miserablerdquo

The stated lines mean that if the will of destiny is to be carried out then the ship will not get wrecked and all the passengers will be saved The safety of the passengers therefore depends upon the will of fate being carried out in the case of the boatswain If however the boatswain is not to die by hanging then the passengers are also very unsafe because in that case the ship is likely to sink

(d) What order does the Boatswain give to the sailors

when he re-enters What does he say about the crying of the fellows inside the cabin

The boatswain orders the sailors to bring the topmast lower and bring the ship close to a stationary position with the help of the main sail He says that the fellows inside the cabin are moaning and crying in their distress louder than his voice and louder even than the roaring of the storm

Class XII (ScienceCommerceHumanities) Subject Topic Summary Execution

Computer Science

PropositionalLogic

Propositional logic is a procedure to provide reasoning through statementProposition A ststement that results in True or False is said to be proposition There are two types of propositionSimple proposition amp compound propositionSimple proposioton A simple proposition is one that is not a part of any other proposition Such sentential form of proposition is symbolized with english letters in short For example Ram is a claver student (TrueFalse)Where do you live (Not in True or False)Grapes are sweet (TrueFalse)It rains today (TrueFalse)Here we can see some statements anwer would be true or false but some staements answer can not give in terms of true or false Thus the sentences which can be answered in true or false are known as simple propositionAssigning propositon to a variableThe general syntax to assign propostion to a variable is as followsVariable = Simple propositonFor example A=Ram is a clever studentB= Grapes are sweetC= it rains todayCompound proposition

helliphellipto be continued in next classhelliphellipMath Relation Relation If A and B are two non-empty sets

then a relation R from A to B is a subset of AxB If R A x B and (a b) R then we say that a sube isinis related to b by the relation R written as aRbeg Let A be the set of students of class XII and B be the set of students of class XI Then some of the examples of relation from A to B arei) (a b) AXB a is brother of bisinii) (a b) AXB age of a is more than age of isinb Types of relation In this section we would like to study different types of relations We know that a relation in a set A is a subset of A times A Thus the empty set φ and A times A are two extreme relations For illustration consider a relation R in the set A = 1 2 3 4 given by R = (a b) a ndash b = 10 This is the empty set as no pair (a b) satisfies the condition a ndash b = 10 Similarly R = (a b) | a ndash b | ge 0 is the whole primeset A times A as all pairs (a b) in A times A satisfy | a ndash

Example 1 Let A be the set of all students of a boys school Show that the relation R in A given by R = (a b) a is sister of b is the empty relation and R = (a b) the primedifference between heights of a and b is less than 3 meters is the universal relationSolution Since the school is boys school no student of the school can be sister of any student of the school Hence R = φ showing that R is the empty relation It is also obvious that the difference between heights of any two students of the school has to be less than 3 meters This shows that R = A times A is primethe universal relation Example 2 Show that the relation R in the set 1 2 3 given by R = (1 1) (2 2) (3 3) (1 2) (2 3) is reflexive

b | ge 0 These two extreme examples lead us to the following definitionsDefinition 1 A relation R in a set A is called empty relation if no element of A isrelated to any element of A ie R = φ A times AsubDefinition 2 A relation R in a set A is called universal relation if each element of A is related to every element of A ie R = A times A Both the empty relation and the universal relation are some times called trivial relation Definition 3 A relation R in a set A is called(i) reflexive if (a a) R for every a Aisin isin(ii) symmetric if (a1 a2) R implies that (aisin 2a1)

R for all aisin 1 a2 Aisin(iii) transitive if (a1 a2) R and (aisin 2 a3) R isinimplies that (a1 a3) R for all aisin 1 a2 a3 AisinDefinition 4 A relation R in a set A is said to be an equivalence relation if R is reflexive symmetric and transitive

but neither symmetric nor transitiveSolution R is reflexive since (1 1) (2 2) and (3 3) lie in R Also R is not symmetric as (1 2) R but (2 1) isin notinR Similarly R is not transitive as (1 2) R and (2 3) R but (1 3) R isin isin notinExample 3 Show that the relation R in the set Z of integers given byR = (a b) 2 divides a ndash b is an equivalence relationSolution R is reflexive as 2 divides (a ndash a) for all a Z isinFurther if (a b) R then 2 divides a isinndash b Therefore 2 divides b ndash a Hence (b a) R which shows that R is isinsymmetric Similarly if (a b) R and (b c) R isin isinthen a ndash b and b ndash c are divisible by 2 Now a ndash c = (a ndash b) + (b ndash c) is even (Why) So (a ndash c) is divisible by 2 This shows that R is transitive Thus R is an equivalence relation in ZExample 4 Let L be the set of all lines in a plane and R be the relation in L defined as R = (L1 L2) L1 is perpendicular to L2 Show that R is symmetric but neither reflexive nor transitiveSolution R is not reflexive as a line L1 can not be perpendicular to itself ie (L1 L1) R notinR is symmetric as (L1 L2) Risin

L1 is perpendicular to L2rArr L2 is perpendicular to L1rArr (L2 L1) RrArr isin

R is not transitive Indeed if L1 is perpendicular to L2 and L2 is perpendicular to L3 then L1 can never be perpendicular to L3 In fact L1 is parallel to L3 ie (L1 L2) R isin(L2 L3) R but (L1 L3) Risin notin

Chemistry Solid state Characteristics if Solids(i)The particles are locked in fixed positions they are unable to change their relative positions and this brings a definite shape and volume of a solid(ii)In a solid the constituent particles are held by strong forces of attractionThe forces of attraction may be bonding or non bonding(iii)The constituent particles in a solid pack together as closely as possibleoccupying most of the available space within the solidThus the empty space in a solid is very smallThis makes a solid highly rigid and nearly incompressibleThis also explains why a solid has high density and exhibits slow diffusionClassification of Solids

Q1)Define Crystalline solids AnsA Solid that has a definite geometrical shape and a sharp melting pointand whose constituent particles (atomsmolecules or ions) are arranged in a long range order of definite pattern extending throughout the solid is called a crystalline solidExNaClQ2)Define Amorphous solids AnsA solid that does not have a definite shape and a sharp melting pointand whose constituent particles (atomsmolecules or ions) are not arranged in a definite pattern is called an amorphoussolid

Crystalline solidsAmorphous solids

ExGlassRubberQ3)Classify Crystalline Solids Crystalline Solids

Physics Coloumbrsquos Law (Summary)

Before Going Into Coloumbrsquos Law We Will First Learn What is Charge Properties of Charge and Always remember that charge is quantized ie a body always have static charge of magnitude equal to some integral multiple of fundamental electronic charge e= 16 x 10- 19 C

Charge is the property of matter that causes it to produce and experience electrical and magnetic effects The study of the electrical charges at rest is called electrostatics When both electrical and magnetic effects are present the interaction between charges is referred to as electromagnetic

There exist two types of charges in nature positive and negative Like charges repel and unlike charges attract each other

The type of charge on an electron is negative The charge of a proton is the same as that of an electron but with a positive sign In an atom the number of electrons and the number of protons are equal The atom is therefore electrically neutral If one or more electrons are added to it it becomes negatively charged and is designated as negative ion However if one or more electrons are removed from an atom it becomes positively charged and is called a positive ion

The excess or deficiency of electrons in a body gives the concept of charge If there is an excess of electrons in a body it is negatively charged And if there is deficiency of electrons the body becomes positively charged Whenever addition or removal of electrons takes places the body acquires a charge

The SI Unit of charge is coulomb (C) In SI units the current is a fundamental quantity having a unit of ampere (A) The unit of charge is defined in terms of the unit of current Thus one coulomb is the charge transferred in one second across the section of a wire carrying a

Ionic SolidsMetallicSolids

Molecular Solids

current of one ampere

As q = It we have1 C = (1 A) (1 s)

The dimensions of charge are [A T]

Properties of Charge

(1) Quantization of Charge Electric charge can have only discrete values rather than any value That is charge is quantized The smallest discrete value of charge that can exist in nature is the charge on an electron given as

e = plusmn 16 x 10- 19 C

This is the charge attained by an electron and a protonA charge q must be an integral multiple of this basic unit That is

Q = plusmn ne where n = 1 2 hellip

Charge on a body can never be (frac12)e (23)e or 57e etcWhen we rub a glass rod with silk some electrons are transferred from the rod to the silk The rod becomes positively charged The silk becomes negatively charged The coulomb is a very large amount of charge A typical charge acquired by a rubbed body is 10 - 8 C

Biology Reproduction in organisms

Welcome to this new session 2020-21Today in this first chapter we mainly discuss about reproduction types needs and life span of some organismsWe also discuss about difference between sexual and asexual reproduction

Q1 What is reproductionReproduction is defined as a biological processin which an organism gives rise to young onessimilar to itselfQ2 What are the needs of reproductionbulli) Reproduction maintain life on earthii) It enables the continuity of the species generation after generationiii) It creates genetic variation among populationsQ3 Define Life span and write some orgnisms life spanbull Life span is the period from birth to

the natural death of an organism- OrganismsLife span1 Butterfly 1 - 2 weeks2 Fruit fly 30 days3Dog 10-13 years4 Rose5-7 years5 Tortoise100-150 years6 Banyan Tree -200 - 250 yearsQ4 Reproduction is of two types in case ofanimals but in case of plants vegetative propagation is also present

Asexual Reproduction Sexual Reproductioni) Always uniparentalii) Gametes are not involvediii) Only mitotic division involvediv) Somatic cells of parents are involvedv) Offsprings are genetically similar to the parents

i) Usually biparentalii) Gametes are involvediii) Meiosis occurs during gametogenesis Mitosis occurs after fertilisationiv) Germ cells of the parents are involvedv) offsprings are genetically different from the parents

COMMERCE BUSINESS ENVIRONMENT

Welcome to the new sessiontoday we are going to start the first chapter of Class XII The name of the chapter is Business Environment

Already many of you have got some idea about the word business environment form the first chapter of business studies in class XI

In todayrsquos world every business enterprise is a part of the society It exists and operates in association with various groups in society such as customers suppliers competitors banks and financial institutions government agencies trade unions media and so on All these groups influence the functioning of business in one way or the other They constitute the environment of businessConcept of Business Environment

The term lsquobusiness environmentrsquo refers to the sum total of all individuals institutions and other forces that lie outside a business enterprise but that may influence its functioning and performance

The main features of business environment Totality of External forces General and Specific forces Interrelatedness Complexity Dynamic Uncertainty

Prepare the following questions from todayrsquos assignment

2 What do you mean by business environment

The term lsquobusiness environmentrsquo means the aggregate of all forces factors and institutions which are external to and beyond the control of an individual business enterprise but they may influence its functioning and performance Business environment is the macro framework within which a business firm a micro unit operates It consists of several interrelated and interacting elements

2 Explain the main features of business environment in brief

Totality of External forces-Business environment is the sum total of all things external to a business environment

General and Specific forces-It

Relativity

The Interrelation between business and its environment

The business enterprise is an open system It continuously interacts with its environment It takes inputs (such as raw materials capital labour energy and so on) from its environment transforms them into goods and services and sends them back to the environment

Fig 1 Business Environment Relationship

includes both the forces general forces are the economic social political legal and technological conditions which indirectly influence all business enterprise Specific forces are the investors customers competitors and suppliers which influence individual enterprise directly

Interrelatedness-Different elements of environment are interrelated for an example growing awareness for health care has increased the demand for health foods

Complexity- Business environment id complex in nature as the elements keep on changing example economic technological and other forces changes in demand for a product and service

Dynamic-Business environment is not static it keeps on changing

Uncertainty- Itrsquos very difficult to predict future events such as technology and fashion which occur fast and frequently

Business Studies

Human Resources Management

Human resource of an organisation are the aggregate of knowledge skills attitudes of people working in it

The management system which deals with human resources is called human resource management

Features of HRMbullComprehensive functionbullPeople-oriented

Question1) What do you mean by human

resource management Answer) Human resource management may be defined as that field of Management which has to do with planning organising and controlling the functions of procuring developing maintaining and utilising the labour force

bullAction oriented bullPervasive function bullContinuous function

2) Explain the features of HRM in brief

Answer)bullHuman Resource Management is concerned with managing people at work bull Human Resource Management is concerned with employees which bring people and organisations together so that the goals of each are met bullHuman resource management considered every employees as an individual and also promote their satisfaction and growth bull Human resource management is inherent in all organisations and at all levelsbullManagement of human resources are ongoing on never ending process which requires a constant alertness and Awareness of human relations

3) ldquoHR function is said to be pervasiverdquowhy

Answer) Human resource management is required in all organisations whether it is private or government organisations armed forces sports organisations etc It permeatsall the functional areas like production marketing finance research etc This from this feature of human resource management it can be said that it is pervasive in nature

Economics Demand Q1DEFINITION OF DEMANDIn economics demand is the quantity of a good that consumers are willing and able to purchase at various prices during a given period of timeQ2DEMAND CURVEIn economics a demand curve is a graph depicting the relationship between the price of a certain commodity and the quantity of that commodity that is demanded at that pricQ3LAW OF DEMANDIn microeconomics the law of demand states that conditional on all else being equal as the price of a good increases quantity demanded decreases conversely as the price of a good decreases quantity demanded increasesQ4ASSUMPTION of LAW OF DEMAND(i)No change in price of related commodities(ii) No change in income of the consumer(iii) No change in taste and preferences customs habit and fashion of the consumer( No expectation regarding future change in priceQ5MARKET DEMAND SCHEDULEIn economics a market demand schedule is a tabulation of the quantity of a good that all consumers in a market will purchase at a

given price At any given price the corresponding value on the demand schedule is the sum of all consumersrsquo quantities demanded at that priceQ6INDIVIDUAL DEMAND SCHEDULEIndividual demand schedule refers to a tabular statement showing various quantities of a commodity that a consumer is willing to buy at various levels of price during a given period of timeQ7 FACTORS AFFECTING INDIVIDUAL DEMAND FOR A COMMODITY

The factors that influence a consumerrsquos decision to purchase a commodity are also known as determinants of demand The following factors affect the individual demand for a commodity1 price of the commodity2 price of related goods3 income of buyer of the commodity4 tastes and preferences of the buyer1 Price of the CommodityYou must have observed that when price of a commodity falls you tend to buy more of it and when its price rises you tend to buy less of it when all other factors remain constant (lsquoother things remaining the samersquo) In other words other things remaining the same there is an inverse relationship between the price of a commodity and its quantity demanded by its buyers This statement is in accordance with law of demand which you will study in the later part of this lesson Price of a commodity and its quantity demanded by its buyers are inversely related only when lsquoother things remain the samersquo So lsquoother things remaining the samersquo is an assumption when we study the effect of changes in the price of a commodity on its quantity demanded2 Price of Related goodsA consumer may demand a particular good But while buying that good heshe also asks the price of its related goods Related goods can be of two types-(i) Substitute goods(ii) Complementary goods While purchasing a good prices of its substitutes and complements do affect its quantity purchased(i) Price of Substitute Goods Substitute goods are those goods which can easily be used in place of one another for satisfaction of a particular want like tea and coffee An increase in price of substitute good leads to an increase in demand for the given commodity and a decrease in price of substitute good leads to a decrease in demand for the given commodity It means demand for a given commodity is directly affected by change in price of substitute goods For example if price of coffee increases the demand for tea will rise as tea will become relatively cheaper in comparison to coffee(ii) Price of Complementary goods Complementary goods are those goods which are used together to satisfy a particular want like car and petrol An increase in the price of complementary goods leads to a decrease in demand for the given commodity and a decrease in the price of complementary goods leads to an increase in demand for the given commodity For example if price of petrol falls then the demand for cars will increase as it will be relatively cheaper to use both the goods together So demand for a given commodity is inversely affected by change in price of complementary goods3 Income of the Buyer of CommodityDemand for a commodity is also affected by income of its buyer However the effect of change in income on demand depends on the nature of the commodity under consideration In case of some goods like full cream milk fine quality of rice (Basmati rice) etc demand for these commodities increases when income of the buyer increases and

demand for these commodities decreases when income of the buyer decreases Such goods whose demand increases with the increase in income of the buyer are called normal goods But there are some goods like coarse rice toned milk etc whose demand decreases when income of buyer increases and their demand increases when income of the buyer decreases Such goods whose demand decreases with the increase in income of the buyer are called inferior goods Suppose a consumer buys 10 Kgs of rice whose price is ` 25 per Kg He cannot afford to buy better quality of rice because the price of such rice is ` 50 per Kg The consumer is spending ` 250 per month on the purchase of rice Now if income of the consumer increases and he can afford ` 350 on purchase of 10 Kg of rice Now he can afford to buy some quantity of rice say 6 Kgs whose price is ` 25 per Kg and may buy 4 Kgs of rice whose price is ` 50 per Kg Thus he will buy 10 Kgs of rice by spending ` 350 per month Therefore we may conclude that demand for normal goods is directly related to the income of the buyer but demand for inferior goods is inversely related to the income of the buyer4 Tastes and Preferences of the BuyerThe demand for a commodity is also affected by the tastes and preferences of the buyers They include change in fashion customs habits etc Those commodities are preferred by the consumers which are in fashion So demand for those commodities rises which are in fashion On the other hand if a commodity goes out of the fashion its demand falls because no consumer will like to buy it(5) Number of Buyers in the Market(Population)Increase in population raises the market demand whereas decrease in population reduces the market demand for a commodity Not only the size of population but its composition like age (ratio of males females children and old people in population) also affects the demand for a commodity It is because of needs of children young old male and female population differs(6) Distribution of Income and WealthIf the distribution of income and wealth is more in favour of the rich demand for the commodities preferred by the rich such as comforts and luxuries is likely to be higher On the other hand if the distribution of income and wealth is more in favour of poor demand for commodities preferred by the poor such as necessities will be more(7) Season and Weather ConditionsThis is generally observed that the demand for woolens increases during winter whereas demand for ice creams and cold drinks increases during summer Similarly market demand for umbrellas rain coats increases during rainy seasonQ8 REASONS FOR OPERATION OF LAW OF DEMAND WHY DEMAND CURVE SLOPES DOWNWARDNow we will try to explain why does a consumer purchase more quantity of a commodity at a lower price and less of it at a higher price or why does the law of demand operate ie why does the demand curve slope downwards from left to right The main reasons for operation of law of demand are1 Law of Diminishing Marginal UtilityAs you have studied earlier law of diminishing marginal utility states that as we consume more and more units of a commodity the utility derived from each successive unit goes on decreasing The consumer will be ready to pay more for those units which provide him more utility and less for those which provide him less utility It implies that he will purchase more only when the price of the commodity falls2 Income Effect

When price of a commodity falls purchasing power or real income of the consumer increases which enables him to purchase more quantity of the commodity with the same money income Let us take an example Suppose you buy 4 ice creams when price of each ice cream is ` 25 If price of ice creams falls to ` 20 then with same money income you can buy 5 ice creams now3 Substitution EffectWhen price of a commodity falls it becomes comparatively cheaper as compared to its substitutes (although price of substitutes has not been changed) This will lead to rise in demand for the given commodity For example if coke and Pepsi both are sold at ` 10 each and price of coke falls Now coke has become relatively cheaper and will be substituted for Pepsi It will lead to rise in demand for coke4 Change in Number of BuyersWhen price of a commodity falls some old buyers may demand more of the commodity at the reduced price and some new buyers may also start buying this commodity who were not in a position to buy it earlier due to higher price This will lead to increase in number of buyers when price of the commodity falls As a result demand for the commodity rises when its price falls5 Diverse Uses of a CommoditySome commodities have diverse uses like milk It can be used for drinking for sweet preparation for ice cream preparation etc If price of milk rises its use may be restricted to important purpose only This will lead to reduction in demand for other less important uses When price of milk falls it can be put to other uses also leading to rise n demand for itQ9 EXCEPTIONS TO THE LAW OF DEMANDYou have studied in law of demand that a buyer is willing to buy more quantity of a commodity at a lower price and less of it at a higher price But in certain circumstances a rise in price may lead to rise in demand These circumstances are called Exceptions to the Law of Demand Some important exceptions are1 Giffen GoodsGiffen goods are special type of inferior goods in which negative income effect is stronger than negative substitution effect Giffen goods do not follow law of demand as their demand rises when their price rises Examples of Giffen goods are jowar and bajra etc2 Status Symbol GoodsSome goods are used by rich people as status symbols eg diamonds gold jewellary etc The higher the price the higher will be the demand for these goods When price of such goods falls these goods are no longer looked at as status symbol goods and tehrefore therir demand falls3 NecessitiesCommodities such as medicines salt wheat etc do not follow law of demandbecause we have to purchase them in minimum required quantity whatever their price may be4 Goods Expected to be ScarceWhen the buyers expect a scarcity of a particular good in near future they start buying more and more of that good even if their prices are rising For example during war famines etc people tend to buy more of some goods even at higher prices due to fear of their scarcity in near future

Political Science

Constitution of India-The

Preamble

The preamble-

Preamble-

The preamble is the most precious part of the constitution We the people of India having solemnly resolved to constitute India into a Sovereign Socialist Secular Democratic Republic and to secure to all its citizensA preamble is an introductory and expressionary statement in a document that explains the documents purpose and underlying philosophy When applied to the opening paragraphs of a statute it may recite historical facts pertinent to the subject of the statuteNature and purpose of the constitution-Purpose of the Constitution dictates permanent framework of the government to form a more perfect union to establish justice and ensure peace of thenationconstitution provide principles how the government can run itself following the rules and laws written in the constitution of each state keeps them balanced

Answer the following questions-

1 What is preambleA preamble is an introductory and expressionary statement in a document that explains the documents purpose and underlying philosophy2 What is the nature and

purpose of the constitutionConstitution dictatespermanent framework of the government to form a more perfect union to establish justice and ensure peace of the nation

Homework-Learn

Accounts Compatibilty mode

1MEANING OF PARTNERSHIPPartnership is a form of business organisation where two or more persons join hands to run a business They share the profits and losses according to the agreement amongst them According to the Indian Partnership Act 1932 ldquoPartnership is relation between persons who have agreed to share profits of a business carried on by all or any one of them acting for allrdquo For example one of your friends has passed class XII from National Institute of Open Schooling (NIOS) and wants to start a business Heshe approaches you to join in this venture Heshe wants you to contribute some money and participate in the business activities Both of you if join hands constitute a partnership2CHARACTERISTICS1048698 Agreement A partnership is formed by an agreement The agreement may be either oral or in writing It defines the relationship between the persons who agree to carry on business It may contain the terms of sharing profit and the capital to be invested by each partner etc The written agreement is known as partnership deed1048698 Number of persons There must be at least two persons to form a partnership

The maximum number of partners in a partnership firm can be 50 according toCompanies Act 20131048698 Business The Partnership is formed to carry on business with a purpose of earning profits The business should be lawful Thus if two or more persons agree to carry on unlawful activities it will not be termed as partnership1048698 Sharing Profits The partners agree to share profits in the agreed ratio In caseof loss all the partners have to bear it in the same agreed profit sharing ratio10486981048698Mutual Agency Every partner is an agent of the other partners Every partner can bind the firm and all other partners by hisher acts Each partner will be responsible and liable for the acts of all other partners10486981048698Unlimited liability The liability of each partner except that of a minor is unlimited Their liability extends to their personal assets also If the assets of the firm are insufficient to pay off its debts the partnersrsquo personal property can be used to satisfy the claim of the creditors of the partnership firm10486981048698Management All the partners have a right to mange the business However they may authorize one or more partners to manage the affairs of the business on their behalf10486981048698Transferability of Share No partner can transfer hisher share to any one including hisher family member without the consent of all other partners3PARTNERSHIP DEEDAgreement forms the basis of partnership The written form of the agreement is which a document of partnership is It contains terms and conditions regarding the conduct of the business It also explains relationship between the partners This document is called partnership deed Every firm can frame its own partnership deed in which the rights duties and liabilities of the partners are stated in detail It helps in settling the disputes arising among the partners during the general conduct of business 4CONTENTS OF PARTNERSHIP DEEDThe partnership deed generally contains the following (i) Name and address of the partnership firm(ii) Nature and objectives of the business(iii) Name and address of each partner(iv) Ratio in which profits is to be shared(v) Capital contribution by each partner(vi) Rate of Interest on capital if allowed(vii) Salary or any other remuneration to partners if allowed(viii) Rate of interest on loans and advances by a partner to the firm(ix) Drawings of partners and interest thereon if any(x) Method of valuation of goodwill and revaluation of assets and liabilities on the reconstitution of the partnership ie on the admission retirement or death of a partner(xi) Settlement of disputes by arbitration(xii) Settlement of accounts at the time of retirement or death of a partner5IN ABSENCE OF PARTNERSHIP DEEDThe partnership deed lays down the terms and conditions of partnership in regard to rights duties and obligations of the partners In the absence of partnership deed there may arise a controversy on certain issues like profit sharing ratio interest on

capital interest on drawings interest on loan and salary of the partners In such cases the provisions of the Indian Partnership Act becomes applicableSome of the Issues are(i) Distribution of Profit Partners are entitled to share profits equally(ii) Interest on Capital Interest on capital is not allowed(iii) Interest on Drawings No interest on drawing of the partners is to be charged(iv) Interest on Partnerrsquos Loan A Partner is allowed interest 6 per annum on the amount of loan given to the firm by himher(v) Salary and Commission to Partner A partner is not entitled to anysalary or commission or any other remuneration for managing the business

History TOPIC-TOWARDS INDEPENDENCE AND PARTITION THE LAST PHASE (1935-1947)

SUB TOPIC-IMPORTANT POLITICAL DEVELOPMENTS ndash GROWTH OF SOCIAL IDEAS

Socialism is a political social and economic philosophyLike in other parts of the world the Russian revolution of 1917 served as a great inspiration for revolutionaries in India who at that time were engaged in the struggle for liberation from British ruleSocialist ideas led to the formation of communist party of IndiaJAWAHARLAL NEHRU Among the early Congress leaders Jawaharlal Nehru was very much impressed and influenced by the Socialist ideas He also learnt about the Economic activities of the Soviet Union after the Bolshevic Revolution 1917 He made full use of them in IndiaThe election of Jawaharlal Nehru and Subhas Chandra Bose showed the Left wing tendency within CongressJawaharlal Nehru demanded economic freedom along with political freedom of the people in order to end the exploitation of masses

Nehrus working committee included three socialists leaders The Lucknow session was a landmark in the evolution of socialist ideas of the congressSUBHAS CHANDRA BOSE ndash Subhas Chandra Bose had socialist leaning Both Jawaharlal Nehru and Subhas Chandra Bose were known as leftist Congress men Later on National Congress divided into Leftist and rightist campCONGRESS SOCIALIST Within the Congress some leaders formed the Congress Socialist partyPattavi Sitaramyya Sardar Patel Rajendra Prasad had hostile attitude towards the Congress Socialist partyJawaharlals attitude was hesitant

1 QUESTION ndash Mention name of two Congress leaders who had socialist leaning

1ANSWER ndash Subhas Chandra Bose and Jawaharlal Nehru2QUESTION- In which session of the congress Jawaharlal elaborated his Socialist ideas2 ANSWER ndash Lucknow and Faizpur Session in December 1935 and 19363QUESTION ndash Why Congress was sharply divided into leftist and rightist camp 3ANSWER ndash Subhas Chandra Bosersquos attempt to seek re election for congress presidentship in 1939sharply divided the National Congress into Leftist and Rightist camp4 QUESTION ndash Who was MN Roy 4 ANSWER ndash Manabendra Roy first formed the Communist Party of India outside the country at Tashkent in 19205QUESTION ndash Who formed the Congress Socialist Party within the Congress5 ANSWER ndash Jaya Prakash Narayan Achyut Patwardhan Acharya Narendra Dev Ram Mohan Lohia Aruna Asaf Ali6QUESTION ndash When was the Congress Socialist Party formed What was its object6 ANSWER ndash 1934The Congress Socialist Party sought to work out socialist programme through the Congress They joined hands with the Congress and wanted to carry

Subhas Chandra Bose being expelled from the congress after the Tripuri rift he formed Forward BlockThere were basic differences between the Congress Socialists and the communistsTRADE UNION ACTIVITIES Maximum working class people lived in Bombay and Calcutta The working and living conditions of those workers were very miserable In this situation Shasipada Banerjee NM Lokhande protested against the oppression of the working class peopleThe first Trade Union Madras Labour Union was formed in 1918 by BP WadiaIndustrial strikes took place in Kanpur Calcutta Madras Jamshedpur and Ahmedabad AITUC was formed in Bombay in 1927 The growth of Trade union among the workers was slow because of the fear of the dismissal of the jobIn the mean time the Moderates as well as Communists left AITUC and formed separate organization

on National struggle with the help of workers and peasant class of the society7 QUESTION ndash What was the name of the party founded by Subhas Chandra Bose7 ANSWER- Forward Block8QUESTION ndash Who was Shasipada Banerjee8 ANSWER ndash Shasipada Banerjee was a radical Brahmo He founded a working menrsquos club to protest against exploitation of the British rulers towards the working class of India9 QUESTION ndash What was the weekly published by NM Lokhande9ANSWER- Dinabandhu10 QUESTION ndash Who founded Bombay Mill-Hands Association and in which year10 ANSWER- NM Lokhande in189011 QUESTION- Who was BP WadiaANSWER- BPWadia was the founder of Madras Labour Union in191812 QUESTION- What was the name of the first labour union of India12 ANSWER- Madras Labour Union13 QUESTION Who founded the Majur Mahajan 13 ANSWER GANDHIJI14 QUESTION What was the full form of AITUC When it was formed14 ANSWER All India Trade Union Congressin 192715QUESTION Who formed the Red Trade Union Congress and in which year15ANSWER The Communists formed the Red Trade Union Congress16 QUESTION What do you mean by Socialism16 ANSWER Socialism describes any political and economic theory that says the community rather than individuals should own and manage property and natural resources

Subject Eng Literature (The Tempest ndash William Shakespeare) Topic Act III Scene 3 Lines 1 to 52 (Line 52 ndash Brother my lord the Duke Stand to and do as we) Date 13th April 2020 (4th Period)

[Students should read the original play and also the paraphrase given in the school prescribed textbook]Summary Questions amp Answers

o Alonso Sebastian Antonio Gonzalo Adrian Francisco and others wandered about the island in search of Ferdinand and gets tired and hungry of the toil and at the same time gives up all hope of finding him

o Antonio and Sebastian are happy that Alonso is out of hope and decide to make another attempt on his life that night when being so tired they will be sleeping soundly

o Suddenly a solemn and strange music is heard in the air and several strange shapes enter bringing in a banquet These strange shapes then dance round it with gestures of salutation and then inviting the King to eat they depart

o Seeing this strange scene all are inclined to believe the tales told by travelers that there truly are ldquounicornsrdquo and ldquothe phoenixrsquo thronerdquo

1 ALONSO What harmony is this My good friends hark (L18-27)

GONZALO Marvellous sweet music

[Enter several strange shapes bringing in a banquet

they dance about it with gentle actions of salutation

and inviting the King and his companions to eat they depart]ALONSO Give us kind keepers heavens What were theseSEBASTIAN A living drollery Now I will believe

That there are unicorns that in Arabia

There is one tree the phoenixrsquo throne one phoenix

At this hour reigning thereANTONIO Ill believe both

And what does else want credit come to me

And Ill be sworn rsquotis true Travellers neer did lie

Though fools at home condemn rsquoem

(a) How did Prospero present an amazing spectacle before Alonso and his companions

Using his magic powers Prospero ordered strange shapes to lay a banquet before Alonso and his companions The shapes brought several dishes with tasty eatables in them They placed the dishes on a table before Alonso and his companions Then the strange shapes began to dance gracefully around the banquet While dancing they made gestures inviting them to eat the food Then suddenly the shapes disappeared(b) Who were the guests at the strange banquet Describe the lsquoliving drolleryrsquo

Alonso Sebastian Antonio Gonzalo Adrian and Francisco were the guests at the strange banquet

The term ldquoliving drolleryrdquo refers to live entertainment show In this context when Alonso the King of Naples Sebastian his brother Antonio the treacherous brother of Prospero Gonzalo the kind and loyal councillor to the King Adrian and Francisco came to the island they were hungry and weary in their spirits They heard a solemn and strange music They were shocked to see several strange shapes bringing in a banquet and these shapes danced about it with gentle action of salutation inviting the King and his companions to eat After this Sebastian described this show as lsquoliving drolleryrsquo(c) What is lsquophoenixrsquo What are lsquoUnicornsrdquo

The term lsquophoenixrsquo refers to a mythical Arabian bird which lived alone and perched on a solitary tree After one hundred years it expired in flames and rose again from its own ashes

lsquoUnicornsrsquo refers to the mythological four-footed beasts having horns in the centre of their foreheads When the horns are ground into powder the powder was believed to be

an aphrodisiac(d) How does Sebastian explain the puppet show OR Why does the speaker now believe in unicorns and phoenix

Sebastian finds several strange shapes bringing in the banquet They invite the king and his party for dinner and soon depart He tells that if such a strange sight can be a reality there is nothing incredible in the world and from the present moment he will believe anything He says that it is a strange dumb show enacted not by puppets but by living beings It is stranger than a travellerrsquos tale Seeing such a thing

before his own eyes he will no longer disbelieve the story about unicorns and phoenix(e) How do the other characters present respond to this living drollery

At the sight of the lsquoliving drolleryrsquo like Sebastian Gonzalo and Antonio too acted strangely Antonio told that he too now believes in unicorns and phoenix and anything else that seems to be incredible He too now believes in travellersrsquo tales Gonzalo told that if he would report those happenings in Naples nobody will believe him He considers that those gentle shapes were gentler in manner in comparison to the living beings Alonso was at first sight suspicious and told them that those strange shapes conveyed their meaning in expressive gestures when they seemed to lack speech by their movements and sounds Francisco was amazed at their mysterious disappearance

2 ALONSO Not I

(Line 43-52)GONZALO Faith sir you need not fear When we

were boysWho would believe that there were mountaineers

Dewlapped like bulls whose throats had hanging at rsquoem

Wallets of flesh Or that there were such men

Whose heads stood in their breasts Which now we find

Each putter-out of five for one will bring us

Good warrant ofALONSO I will stand to and feed

Although my lastmdashno matter since I feel

The best is past Brother my lord the Duke

Stand to and do as we

(a) How does Alonso respond at the spectacle of the shapes which were sent to them at the instruction of Prospero

After seeing the strange sight of appearing and disappearing of the shapes sent by Prospero to arrange a banquet for them Alonso says that his surprise at having seen those creatures is infinite and he is fully justified in feeling so much surprise He thinks that their shapes their gestures and the sounds they made were indeed amazing Although they do not possess the gift of speech yet they were able to convey their

thoughts by means of their gestures only

(b) What does Prospero say about the views expressed by Alonso regarding the shapes What does Francisco think about the shapesAfter hearing Alonsorsquos views about the shapes Prospero says that this manrsquos praise of the spirits is rather hasty He means to say that Alonso has shown great haste in reaching the conclusion about the shapes Francisco is amazed to see that those shapes disappeared in a mysterious way(c) What does Sebastian ask Alonso to doSebastian tells Alonso that the shapes having disappeared should not matter to them because they have left the eatables behind He asks Alonso to enjoy eating as they are extremely hungry but the king does not accept his offer of enjoying the dishes(d) How does Gonzalo try to dispel Alonsorsquos fear of those strange shapes What kind of references does he give to AlonsoGonzalo says that those who have travelled abroad have reported seeing even stranger sights than these shapes that Alonso and his companions have beheld Hence there is no reason to feel afraid of these shapes Gonzalo further adds that in his younger days he had heard strange stories from travelers and Alonso might have heard similar stories For instance it was said that there existed a certain race of

human beings who had huge lumps of flesh hanging at their throats and who therefore resembled bulls Then Gonzalo tells about a race of human beings whose heads were located at their breasts Gonzalo says that such stories were not believed by most people in those days but now-a-days these stories have become common(e) Explain the following lsquoEach putter-out of five for onersquoEnglish travellers often insured their trips with London brokers Those that went on foreign travels those days used to deposit a certain amount with some firm or company in London before their departure If the travelers failed to return the money was forfeited by the company with which it had been deposited But this money was repaid five-fold if the travelers returned safe and sound In this way a traveler stood a great chance of recovering the entire cost of his

travels(f) Give the explanatory meanings of the following expressions in the context of the above extract (i) Dewlapped (ii) Wallets of flesh

(iii) Putter-out(i) Dewlapped having big lumps of flesh at the necks(ii) Wallets of flesh large masses of flesh looking like bags(iii) Putter-out to invest money before commencing the travel

  • General methods of preparation of hydrogen
  • Chapter Dimensional Analysis (Summary)
    • Properties of Charge
Page 30:   · Web viewSubject. Topic. Summary. Execution. Hindi. व्याकरण. शरीरके अंगो के नाम लिखिए. 1) आँख 2) नाक 3

Binary Number SystemThis number system has only two digits these are 0 and 1 Here 0 stands for off while 1 stands for on

Octal Number SystemThis number system has 8 digits these are 0 1 2 3 4 5 6 amp 7

Hexadecimal Number SystemThis number system has 16 digits these are 0 1 2 3 4 5 6 7 8 9 A B C D E F Here the value of the alphabets are as follows A=10 B=11 C=12 D=13 E=14 F=15

Rules for conversion decimal number to Binary1 Divide the decimal number by 22 If the number will not divide equally by 2 then round down the answer to the nearest whole number (integer)3 Keep a note of the remainder it should be either 0 or 14 Keep repeating the above steps dividing each answer by 2 until you reach zero5 Write out all the remainders from bottom to top This is your binary solution

For example Lets convert 32 to binary 2 32 2 16 - 0 2 8 - 0 2 4 - 0 2 2 - 0 2 1 - 0 0 - 1

The binary equivalent of 3210 is 1000002

Try the follwing youself1 2410

2 4810

3 1210

History GROWTH OF NATIONALISM

The second half of the 19th century witnessed growth of political consciousness and a sense of Nationalism among the IndiansThere were various factors for growth of Indian Nationalism- As a result various political associations were formed in different provinces by the educated Indians Surendranath Banerjee organized a meeting of National conference at Calcutta Ultimately the National Congress was founded in Bombay in 1885This body became the vanguard of Indian struggle for freedom The congress leaders were known as moderates because they followed a policy of prayer and petition A large number of Indian leaders had experienced in political agitation The Political situation of England was also changed Moreover increasing revolutionary activities in Maharashtra Punjab and Bengal became serious concern to the British Government In this

QUESTION1 What do you mean by Nationalism ANSWER 1 Nationalism is defined as loyalty and devotion to own nation especially a sense of national consciousnessQUESTION 2 What are the causes of nationalism ANSWER 2 There were various factors for growth of nationalism

1 Spread of western education2 The progress of vernacular press and

patriotic literature3 The economic exploitation of our

country by the colonial rulers4 International affairs

QUESTION 3 Who organized National conference in Calcutta in 1883 ANSWER 3 Surendranath BanerjeeQUESTION 4 When did Indian National Congress formANSWER 4 Indian National Congress was formed in 1885 in BombayQUESTION 5 Who were ModeratesANSWER 5 The Early Nationalists were also known as Moderates Their emergence marked

background Lord Curzon became Viceroy in India He had no respect for the Indian National Congress

the beginning of the organized national movement in India They believed in British justice and were loyal to them They followed a policy of prayer and petition They demanded constitutional reforms of our country Impotant Moderate leaders were Pherozshah Mehta Dadabhai Naorozi and Surendranath Banerjee etcQUESTION 6 What do you know about Extremism in Indian National movementANSWER 6 In the beginning of 20th century a new class of national leaders emerged in India which was different from the moderate groups They started more aggressive movement against the British empire The goal of extremists was ldquoswarajrdquo Important extremist leaders were Bal Gangadhar Tilak Lala Lajpat Rai Bipin Chandra Pal etcQUESTION 7 Mention the places which were the main centres of Revolutionary movementANSWER 7 Maharashtra Bengal and Punjab

Physics

Chapter Dimensional Analysis

(Summary)

The dimensions of a physical quantity are the powers to which the fundamental units are raised in order to obtain the derived unit of that quantit

The physical quantites lengthmasstime are represented by [L] [M] [T] resp let they are raised to powers ( dimesions) abc resp then any physical quantity can be represented by [ La Mb Tc ] Examples

1 Area area = L x B = [L] x [L] = [M0 L2 T0 ]

2 Density density = massvolume = [M][L3] = [ M L-3]

3 Velocity velocity = distancetime = [L][T] = [LT-1]HW Try to find out dimension of acceleration Acceleration = velocity timeNB One can find the SI Units Using Dimension Analysis Such as for area we have [L2] so its SI unit is m2

Biology Topic ndash Chp-1 The living world

Today we will start the first chapter the living world Here we discuss about the characteristics of living organism and what are the difference between them and nonliving substances We also discuss about the contribution of different Scientists

There are over 500000 species of plants andover a million species of animal are present on earth Some 15000 new species were discovered every yearQ1 What is a living organismbull A living organism is primarily physico -chemical material that demonstrate a high degree of complexity is capable of selfRegulation possesses a metabolism and perpetuates itself through timeQ2 What are the differences between livingand non-livingsi) Compared with non-living living organisms

have more complex organised structure and their use of energy is more controlled amp efficientii) Living things reproduce their own kind by forming new cells which contains copies of their genesiii) Each organism has some degree of homeostasisie it is able to make adjustments so that internal environment remains constantQ3 Write contributions of following Scientists i) Aristotle - One of the first theories in Biology places all living things in a hiearchieii) AV Leeuwenhoek - was the first to observe living single celled organisms under microscopeii) Carolus Linnaeus - developed the binary system for naming of organisms and classificationiii) Geregor Johann Mendel ndash discoverbasic principles of inheritanceHomework i) C Darwin ii)Schleiden

Math Trigonometric functions

1 Overviewi) Trigonometry The word lsquotrigonometryrsquo is derived from the Greek words lsquotrigonrsquo and lsquometronrsquo which means measuring the sides of a triangle An angle is the amount of rotation of a revolving line with respect to a fixed line Usually we follow two types of conventions for measuring angles ie a) Sexagesimal system b) Circular system In Sexagesimal system the unit of measurement is Degree In Circular system the unit of measurement is Radian ii) Relation between degree and radianThe ratio of circumference of a circle to its diameter is always a constant This constant ratio is a number denoted by π which is taken approximately as 227The relationship between degree amp radian measurements is as follows2 right angles = 180deg= π radians1radian = 180degπ=57deg16(approx) 1deg=π180 radianiii) Length of an arc of a circleIf an arc of length s subtends an angle θ radians at the center of a circle of radius r then s=rθiv) Area of a sector of a circleA sector is like a pizza slice of the

Q) Express the following angles in radiana) 45deg b) 40deg3730Ans a) We have 180deg=π radiansi e 45deg= πtimes45180 radian = π4 radiansb) 40deg3730= 40deg37+3060 minute= 40deg 37 +12 minute= 40deg+ 752 minute=40 + 75(2times60) degree=3258 degreeNow 180deg=π radianie 3258 degree= (πtimes325) (180times8) radians = 65π288 radiansQ) A circle has a radius of r=12 meters What is the length of an arc traced out by a 60deg angle in the center of the circleAns In this problem we know both the central angle (60deg) and the radius of the circle (12) All we have to do is plug those values into our equation and we get

s = 2π(12)(60360)s = 24π6s = 4πSo the length of an arc traced out by a 60deg angle in a circle with a radius of 12 meters equals 4π meters asymp 1257 metersQ) Find the area of the sector with a central angle 30deg and a radius of 9cmAns GivenRadius r = 9 cmAngle θ = 30degArea of the sector = θ360degtimesπr2

= 30360degtimes227times92=2121cm2

circle It consists of a region bounded by two radii and an arc lying between the radiiThe area of a sector is a fraction of the area of the circle This area is proportional to the central angle In other words the bigger the central angle the larger is the area of the sectorArea of Sector = θ2 times r2 (when θ is in radians)

Area of Sector = θ times π360 times r2 (when θ is in degrees)

COMMERCE

CLASSIFICTION OF HUMAN ACTIVITIES-ECONOMIC AND NON-ECONOMIC

Welcome to the new sessiontoday we are going to start the first chapter of Class XI The name of the chapter that we are going to start is

lsquoClassification of Human Activities ndasheconomic and non-economicrsquo

Now let us start the chapter by considering human beings and the activities they perform throughout the day

Human activities means all those activities that human beings undertake to satisfy their wants

Human wants on the other hand are the desire of human beings for goods (vegetables fruits rice etc) and services (services of doctors teachers lawyers etc) that they require to live

Now these human activities continue throughout life as human wants are unending unlimited and recurring as human beings desire for better living throughout their lives

Now human activities can be classified into two categories

Human activities

Economic activities Non-economic activities

Economic activities are

Questions1 What are human activities

Answer Human activities mean all those activities that human beings undertake to satisfy their wants

Example A man working in an office

A boy playing in the garden

2What are the characteristics of human activitiesAnswer the characteristics of human activities are as follows

Human activities are undertaken by men women and children and these activities involve human efforts

Human activities are undertaken to satisfy human wants which are unlimited

Human activities continue throughout life

Human activities are performed for both earning money and personal satisfaction

3What is economic activitiesGive example

Answer Economic activities are undertaken by human beings with the object of earning money acquiring wealth and thereby satisfying human wantsExample

Selling of goods by a shop keeper to his customer

A clinic run by a doctor Service of a teacher in school or college

undertaken by human beings with the object of earning money and acquiring wealth

These activities result in the production of economic goods and services

Example Human activities(ie working in factories officesshops) which produce direct economic benefits

Non-economic activities are inspired by human sentiments and emotions such as love for the family desire to help the poor and love for the country

Thus these human activities (eg praying playing sleeping) produce no direct economic benefits and they are also not related to earning money and acquiring wealth

4 What are the characteristics of economic activities

Answer The characteristics of economic activities are as follows

Economic motiveEconomic activities are undertaken to earn money and acquire wealth

ProductiveEconomic activities involve productiondistribution and exchange of goods and services to create wealth

Economic growthEconomic activities determine the level of economic development of a country and standard of living of its citizens

Socially desirableEconomic activities are socially desirable for society

Economic resourcesEconomic activities make use of all the economic resources such landlabourcapital etc

5 What do you mean by non-economic activitiesExampleAnswerNon-economic activities are inspired by human sentiments and emotions such as love for the family desire to help the poor and love for the countryThese activities are not undertaken for monetary gain but for onersquos satisfaction and happinessExample

a mother looks after her children

a student donates blood8 Differentiate between Economic activities and Non-economic activities

Economic activities

Non-economic activities

1to earn living and acquiring wealth2Result can be measured in terms of money

3ExampleBusinessprofession and employment

1 to obtain some satisfaction

2Result cannot be measured in terms of money

3ExampleFamily-orientedreligious socialCultural and national

BUSINESS STUDIES

BUSINESS ENVIRONMENT

Welcome to the new sessionToday we are going to start the first chapter and the name of the chapter is Business Environment

In todayrsquos world every business enterprise is a part of the society It exists and operates in association with various groups in society such as customers suppliers competitors banks and financial institutions government agencies trade unions media and so on All these groups influence the functioning of business in one way or the other They constitute the environment of businessConcept of Business Environment

The term lsquobusiness environmentrsquo refers to the sum total of all individuals institutions and other forces that lie outside a business enterprise but that may influence its functioning and performance

The main features of business environment

Totality of External forces General and Specific forces Interrelatedness Complexity Dynamic Uncertainty Relativity

The Interrelation between business and its environment

The business enterprise is an open system It continuously interacts with its environment It takes inputs

Prepare the following questions from todayrsquos assignment

1 What do you mean by business environment

The term lsquobusiness environmentrsquo means the aggregate of all forces factors and institutions which are external to and beyond the control of an individual business enterprise but they may influence its functioning and performance Business environment is the macro framework within which a business firm a micro unit operates It consists of several interrelated and interacting elements

2 Explain the main features of business environment in brief

Totality of External forces-Business environment is the sum total of all things external to a business environment

General and Specific forces-It includes both the forces general forces are the economic social political legal and technological conditions which indirectly influence all business enterprise Specific forces are the investors customers competitors and suppliers which influence individual enterprise directly

Interrelatedness-Different elements of environment are interrelated for an example growing awareness for health care has increased the demand for health foods

Complexity- Business environment id

(such as raw materials capital labour energy and so on) from its environment transforms them into goods and services and sends them back to the environment

Fig 1 Business Environment Relationship

complex in nature as the elements keep on changing example economic technological and other forces changes in demand for a product and service

Dynamic-Business environment is not static it keeps on changing

Uncertainty- Itrsquos very difficult to predict future events such as technology and fashion which occur fast and frequently

Economics Basic Economic ConceptsSub topic

Microeconomics and

Macroeconomics

Welcome to the new sessiontoday we are going to start the first chapter of Class XI The name of the chapter that we are going to start is Basic Economic concepts

Now Economics covers the study of human activities Human activities are those activities which are performed by humans to satisfy their wants

Thus Human wants are unlimited and therefore economic activities such as production exchange and consumption are needed in order to satisfy those wants

The study of economics is divided largely in two parts which areMicroeconomics and Macroeconomics

SUBJECT- MATTER OF ECONOMICS

MICROECONOMICS MACROECONOMICS

Questions1Who has coined the words micro and macro economics

Answer Ranger Frisch coined the words lsquomicrorsquo and lsquomacrorsquo in 1933 to denote the two branches of economic theory namely microeconomics and macroeconomics

2What is microeconomicsAnswer It is the study of behaviour of individual decision ndash making unit such as consumers firms etc

3 What is macroeconomicsAnswer Macroeonomics is the study of overall economic phenomena like employment national income etc

4 What is the importance of microeconomicsAnswer

Microeconomics helps in formulating economic policies which enhance productive efficiency and results in greater social welfare

It helps the government in formulating correct price policies

It explains the working of a capitalistic economy where individual units(producers and consumers ) are free to take their own decision

Micro means a small part in

microeconomics we do not study the whole economy Hence we study an individual consumer and his or her choices and a producer and his or her profit maximizing decisions in the market Thus it does not mirror what happens in the economy as a whole

Macroeconomics on the other hand studies the economy as a whole It is concerned with aggregate and depicts the entire picture of the economyMacroeconomics deals with the national income aggregate investment aggregate consumption etc

Features of Microeconomics It deals with small

parts of the country Hence it looks at

individual consumers firms and industries

It deals with individual income consumption and savings

It studies the determination of price of any product or factors of production

It deals with the working of market via the price mechanism which is nothing but the determination of price and quantity of a commodity by the forces of demand and supply

Features of Macroeconomics

It deals with the study of the economy as a whole

It is concerned with

5 Give a limitation of microeconomics Microeconomics fails to explain the

functioning of an economy as a whole It cannot explain unemployment illiteracy and other problems prevailing in the country

6 What is the importance of macroeconomics It gives overall view of the growing

complexities of an economic system It provides the basic and logical

framework for formulating appropriate macroeconomic policies (eg for inflation poverty etc )to direct and regulate economy towards desirable goals

7What is the limitation of macroeconomics It ignores structural changes in an

individual unit of the aggregate

8 Differentiate between Microeconomics and Macroeconomics

Microeconomics Macroeconomics

the study of aggregates

National income aggregate savings and aggregate investments are major concepts dealt within macroeconomics style

It studies the determination of general price levels

It investigates into the problem of unemployment and the achievement of employment

It studies the aspect of decision making at the aggregate and national levels

It includes all growth theories whether related to developed or developing economies it also includes the study of economic systems and the working of the economy under different systems

Note Both Micro and macro economics are complementary and should be fully utilized for proper understanding of an economy

1It studies economic aspect of an individual unit2It deals with individual incomeConsumption and savings

3 It facilitates determination of price of any product or factors of production

4 Itrsquos scope is narrow and restricted to individual unit

1It studies the economy as a whole

2It deals with the national income aggregate consumption and aggregate savings3 It facilitates determination of general price level in an economy

4 Itrsquos scope is wide as it deals with economic units on the national level

ACCOUNTS

Introduction to Accounting and Book-keeping

Today I am going to share you the meaning of Accounting and Book-keeping and its related terms bullAccounting bullBook Keeping bullAccountsbullTypes Of Accounts bullAccounting Cycle

bull Meaning of accounting

Ans ) Accounting is the art and science of recording classifying and summarising monetary transactions

bull Meaning of Book-keeping

Ans) Bookkeeping is the art of recording business transactions with the view of having a permanent record of them and showing their effect on wealth

bull Meaning of account

Ans) The term account means a record of

business transactions concern a particular person of firm asset or income or expense It is a summarised record of all transactions which take place in an accounting year

bull Types of accountsPersonal accounts ndash Personal accounts relating

to person and Organisation are known as personal accounts Example Ramrsquos Account ABC amp Co Account etc

Real account - The accounts related to tangible and intangible assets are called real accounts Example Cash Account Furniture Account etc

Nominal account- Accounts related to expenses losses incomes and gains are known as nominal accounts Example Wages Account Salary Account Discount Account etc

bull Accounting cycle Accounting cycle refers to a complete sequence of accounting activities It begins with recording of transactions and ends with the preparation of a balance sheet

Chemistry TopicAtomic Structure

Thomsonrsquos atomic modelThomson (1898) was the first to propose the model of an atomHe proposed that an atom can be regarded as a uniform sphere of positive electricity in which requisite number of electrons are embedded evently to neutralize the positive chargeThis is just like plums embedded in a pudding or seeds evently distributed in red spongy mass of a watermelonThis model of atom is known as ldquoPlum-Pudding modelrdquo or

Q1)What is the fundamental constituents of atomAns Electron Proton and neutrons are the fundamental constituents of atomQ2)What is the value of fundamental unit of electricityAnsThe charge carried by one electron is sad to be the fundamental unit of electricityIts magnitude is 48times10-10esuOr 1602times10-19C Q3)Name the element containing no neutronAnsOrdinary hydrogen atom or protium 1H1

Types of AccountPersonal AccountReal AccountNominal AccountBalance Sheet (opening)

ldquowatermelon modelrdquoThis model could explain the electrical neutrality of an atom but failed to explain the result of scattering experiment carried out by Rutherford in 1911So it was rejected ultimately

Q4)Why is an electron called universal particleAns Itrsquos mass and Charge are independent of its source

EVS Chapter 1 ndash Modes of Existence

Modes of existence When one speaks normally about the mode of existence of some group or individual one refers to their customs their mode of being their ethology their habitat in some way their feeling for a placeDifferent modes of exixtence are ndash

1 Hunting ndashGathering2 Pastoral3 Agricultural4 Industrial

1 Hunting and gathering Hunting and gathering mode of existence is characterized by obtaining food from hunting wild animals including fishing and gathering wild plants From their earliest days the hunter-gatherer diet included various grasses tubers fruits seeds and nuts Lacking the means to kill larger animals they procured meat from smaller game or through scavenging

Societies that rely primarily or exclusively on hunting wild animals fishing and gathering wild fruits berries nuts and vegetables to support their diet are called hunting and gathering societies

At least this used to be practice of human beings before agriculture is invented As their brains evolved hominids developed more intricate knowledge of edible plant life and growth cycles

Q) Write the features of Hunting ndash gathering societiesAns - There are five basic characteristics of hunting and gathering societies

i The primary institution is the family which decides how food is to be shared and how children are to be socialized and which provides for the protection of its members

ii They tend to be small with fewer than fifty members

iii They tend to be nomadic moving to new areas when the current food supply in a given area has been exhausted

iv Members display a high level of interdependence

v Labor division is based on sex men hunt and women gather

Political Science

Introduction to political science

Political science occasionally called politology is a social science which deals with systems of governance and the analysis of political activities political thoughts associated constitutions and political behaviorThe study of political science involves the study of both the

Answer the following questions-1 What is political science

Political science occasionally called politology is a social science which deals with systems of governance and the analysis of political activities political thoughts associated constitutions and political behavior

2 Short notes-

traditional and modern theories of politicsTraditionalClassical political sciencepolitical theory-Traditional political science is the study of politics before Second World War The methodology to study Politics was traditional (legal formaletc) the definition of politics traditional (Politics begins and end with state)area of study (constitution state machinery)was traditionalModern Political scienceModern political theory-Modern Political Theory critically examines the contemporary state of political theory making an assessment of the achievement and limitations of the Behavioural Revolution in its totality and reviews objectively the major paradigms and conceptual frameworks adopted by the disciplineContemporary attempts at the development of an integrated political theory involving the use of both traditional and modern concepts approaches and theories-Around late 1960s several political scientists realized the importance of both the traditional political theory and modern Political theory They began building an integrated theory of politics involving a systematic mixture of traditional and modern studies of politics It was held that the study of a complex and vast field like politics needs both traditional as well as

Classical political theory Modern Political theory

Homework-Learn

modern concepts and approaches for studying itrsquos all aspects

Subject Eng Literature (The Tempest ndash William Shakespeare) Topic Act I Scene 1 Lines 1 to 32 (Line 32 ndash Gonzalo hellip If he be not born to be hanged our case is miserable) Date 13th April 2020 (3rd Period)

[Students should read the original play and also the paraphrase given in the school prescribed textbook]Summary Questions amp Answers

[SUMMARY OF THE ENTIRE SCENE]

o The play starts with the scene of a severe storm at sea Alonso (King of Naples) Sebastian (Alonsorsquos brother) Ferdinand (Alonsorsquos son) Gonzalo Antonio (the usurping Duke of Milan) are in a ship in the midst of the storm

o The mariners are trying their best to control the vessel from running aground and are totally following the orders of their Master the Boatswain They have scant success

o The mariners become extremely unhappy and annoyed when most of the passengers arrive on the deck thereby hampering their effort to save the ship There is serious confrontation between them and the passengers who are part of the Kingrsquos entourage

o The mariners could not save the ship

SUMMING-UP

(i) Vivid description of the scene which gives a realistic description of terror and confusion of a tropical storm

(ii) Shows Shakespearersquos accuracy of knowledge in describing the naval operations and also matters of seamanship

(iii) The opening scene justifies the title ndash The Tempest

UNANSWERED QUESTIONS

(i) The King always travels with his entire fleet including his soldiers Where

(1) GONZALO Nay good be patient (Line 15-26)BOATSWAIN When the sea is Hence What cares these

roarers for the name of the king To cabin silence Trouble us not

GONZALO Good yet remember whom thou has aboardBOATSWAIN None that I more love than myself You are a

councillor if you can command these elements to silence and work

the peace of the present we will not hand a rope more use your authority If you cannot give thanks you have

lived so long and make yourself ready in your cabin for the mischance of the hour if it so hap [To the Mariners]

Cheerly good hearts [To Gonzalo] Out of our way I say

(a) To whom is the boatswain speaking What does he mean by lsquoNone that I more love than myselfrsquo

The Boatswain is speaking to Gonzalo the honest old councilor of the Duke of MilanBy using the words ndash lsquoNone that I love more than I love myselfrsquo means that for the Boatswain nobody is dearer to him than his own life

(b) What were the conditions that made the boatswain react in this way

The Boatswain reacts in this way because the storm is at sea and Alonso King of Naples Sebastian his brother Ferdinand his son Gonzalo Antonio the usurping Duke of Milan on board are in distress and in panic Thus they have rushed to the deck interrupting the work of the mariners

(c) What hope does Gonzalo take from the attitude of the boatswain

The insolent and authoritative attitude of Boatswain makes Gonzalo feel comforted He tells that there are no signs that the Boatswain will be drowned But his facial appearance and attitude shows that he is destined to die on land by hanging which in effect means that all on board will be saved Otherwise all the persons on board are doomed

(d) How can they lsquomake yourself ready in your cabinrsquo For what were they asked to make ready themselves

In order to make themselves ready in their cabin the

were the other ships

(ii) Why was the ship in that area Where was it coming from or going where

(iii) The ship broke apart What happened to those who were in the ship

passengers on board must prepare for death which they will possibly soon have to meetThey can retire to their cabins and offer prayers to the Almighty to save them from drowning

(e) What does the boatswain say when he is asked to be patient What does he order to the royal party

When the boatswain is asked to be patient and remain calm he says that he will be patient only when the storm will be over and the sea will be calm but as long as the storm blows and there is danger to the ship he cannot think of being patient He orders the royal party to go to the cabin and leave the mariners to their work

(2) GONZALO I have great comfort from this fellow (Line 27-36)

Methinks he hath no drowning mark upon him his complexion is perfect

gallows Stand fast good Fate to his hanging Make the rope of his destiny our cable for our own doth little advantage If he be not born to be hanged our case is miserable

(a) Why does Gonzalo regard the Boatswain in the midst of danger

In the midst of danger Gonzalo regards the boatswain because he feels that the Boatswain is a source of comfort and is bent upon to do his work sincerely which in this case is saving the ship and its passengers from the severest of raging storm

(b) What reasons does Gonzalo give when he says that none in the ship will die of drowning

Gonzalo is almost sure that none in the ship will die by drowning His says that there is no mark on the face of the boatswain that indicates that he will die by drowning On the other hand the lines on his face are strong indications that he will be hanged to death Therefore there shall be no danger of the shiprsquos sinking

(c) Explain the following ldquoStand fast good Fate to his hanging Make the rope of his destiny our cable for our own doth little advantage If he be not born to be hanged our case is miserablerdquo

The stated lines mean that if the will of destiny is to be carried out then the ship will not get wrecked and all the passengers will be saved The safety of the passengers therefore depends upon the will of fate being carried out in the case of the boatswain If however the boatswain is not to die by hanging then the passengers are also very unsafe because in that case the ship is likely to sink

(d) What order does the Boatswain give to the sailors

when he re-enters What does he say about the crying of the fellows inside the cabin

The boatswain orders the sailors to bring the topmast lower and bring the ship close to a stationary position with the help of the main sail He says that the fellows inside the cabin are moaning and crying in their distress louder than his voice and louder even than the roaring of the storm

Class XII (ScienceCommerceHumanities) Subject Topic Summary Execution

Computer Science

PropositionalLogic

Propositional logic is a procedure to provide reasoning through statementProposition A ststement that results in True or False is said to be proposition There are two types of propositionSimple proposition amp compound propositionSimple proposioton A simple proposition is one that is not a part of any other proposition Such sentential form of proposition is symbolized with english letters in short For example Ram is a claver student (TrueFalse)Where do you live (Not in True or False)Grapes are sweet (TrueFalse)It rains today (TrueFalse)Here we can see some statements anwer would be true or false but some staements answer can not give in terms of true or false Thus the sentences which can be answered in true or false are known as simple propositionAssigning propositon to a variableThe general syntax to assign propostion to a variable is as followsVariable = Simple propositonFor example A=Ram is a clever studentB= Grapes are sweetC= it rains todayCompound proposition

helliphellipto be continued in next classhelliphellipMath Relation Relation If A and B are two non-empty sets

then a relation R from A to B is a subset of AxB If R A x B and (a b) R then we say that a sube isinis related to b by the relation R written as aRbeg Let A be the set of students of class XII and B be the set of students of class XI Then some of the examples of relation from A to B arei) (a b) AXB a is brother of bisinii) (a b) AXB age of a is more than age of isinb Types of relation In this section we would like to study different types of relations We know that a relation in a set A is a subset of A times A Thus the empty set φ and A times A are two extreme relations For illustration consider a relation R in the set A = 1 2 3 4 given by R = (a b) a ndash b = 10 This is the empty set as no pair (a b) satisfies the condition a ndash b = 10 Similarly R = (a b) | a ndash b | ge 0 is the whole primeset A times A as all pairs (a b) in A times A satisfy | a ndash

Example 1 Let A be the set of all students of a boys school Show that the relation R in A given by R = (a b) a is sister of b is the empty relation and R = (a b) the primedifference between heights of a and b is less than 3 meters is the universal relationSolution Since the school is boys school no student of the school can be sister of any student of the school Hence R = φ showing that R is the empty relation It is also obvious that the difference between heights of any two students of the school has to be less than 3 meters This shows that R = A times A is primethe universal relation Example 2 Show that the relation R in the set 1 2 3 given by R = (1 1) (2 2) (3 3) (1 2) (2 3) is reflexive

b | ge 0 These two extreme examples lead us to the following definitionsDefinition 1 A relation R in a set A is called empty relation if no element of A isrelated to any element of A ie R = φ A times AsubDefinition 2 A relation R in a set A is called universal relation if each element of A is related to every element of A ie R = A times A Both the empty relation and the universal relation are some times called trivial relation Definition 3 A relation R in a set A is called(i) reflexive if (a a) R for every a Aisin isin(ii) symmetric if (a1 a2) R implies that (aisin 2a1)

R for all aisin 1 a2 Aisin(iii) transitive if (a1 a2) R and (aisin 2 a3) R isinimplies that (a1 a3) R for all aisin 1 a2 a3 AisinDefinition 4 A relation R in a set A is said to be an equivalence relation if R is reflexive symmetric and transitive

but neither symmetric nor transitiveSolution R is reflexive since (1 1) (2 2) and (3 3) lie in R Also R is not symmetric as (1 2) R but (2 1) isin notinR Similarly R is not transitive as (1 2) R and (2 3) R but (1 3) R isin isin notinExample 3 Show that the relation R in the set Z of integers given byR = (a b) 2 divides a ndash b is an equivalence relationSolution R is reflexive as 2 divides (a ndash a) for all a Z isinFurther if (a b) R then 2 divides a isinndash b Therefore 2 divides b ndash a Hence (b a) R which shows that R is isinsymmetric Similarly if (a b) R and (b c) R isin isinthen a ndash b and b ndash c are divisible by 2 Now a ndash c = (a ndash b) + (b ndash c) is even (Why) So (a ndash c) is divisible by 2 This shows that R is transitive Thus R is an equivalence relation in ZExample 4 Let L be the set of all lines in a plane and R be the relation in L defined as R = (L1 L2) L1 is perpendicular to L2 Show that R is symmetric but neither reflexive nor transitiveSolution R is not reflexive as a line L1 can not be perpendicular to itself ie (L1 L1) R notinR is symmetric as (L1 L2) Risin

L1 is perpendicular to L2rArr L2 is perpendicular to L1rArr (L2 L1) RrArr isin

R is not transitive Indeed if L1 is perpendicular to L2 and L2 is perpendicular to L3 then L1 can never be perpendicular to L3 In fact L1 is parallel to L3 ie (L1 L2) R isin(L2 L3) R but (L1 L3) Risin notin

Chemistry Solid state Characteristics if Solids(i)The particles are locked in fixed positions they are unable to change their relative positions and this brings a definite shape and volume of a solid(ii)In a solid the constituent particles are held by strong forces of attractionThe forces of attraction may be bonding or non bonding(iii)The constituent particles in a solid pack together as closely as possibleoccupying most of the available space within the solidThus the empty space in a solid is very smallThis makes a solid highly rigid and nearly incompressibleThis also explains why a solid has high density and exhibits slow diffusionClassification of Solids

Q1)Define Crystalline solids AnsA Solid that has a definite geometrical shape and a sharp melting pointand whose constituent particles (atomsmolecules or ions) are arranged in a long range order of definite pattern extending throughout the solid is called a crystalline solidExNaClQ2)Define Amorphous solids AnsA solid that does not have a definite shape and a sharp melting pointand whose constituent particles (atomsmolecules or ions) are not arranged in a definite pattern is called an amorphoussolid

Crystalline solidsAmorphous solids

ExGlassRubberQ3)Classify Crystalline Solids Crystalline Solids

Physics Coloumbrsquos Law (Summary)

Before Going Into Coloumbrsquos Law We Will First Learn What is Charge Properties of Charge and Always remember that charge is quantized ie a body always have static charge of magnitude equal to some integral multiple of fundamental electronic charge e= 16 x 10- 19 C

Charge is the property of matter that causes it to produce and experience electrical and magnetic effects The study of the electrical charges at rest is called electrostatics When both electrical and magnetic effects are present the interaction between charges is referred to as electromagnetic

There exist two types of charges in nature positive and negative Like charges repel and unlike charges attract each other

The type of charge on an electron is negative The charge of a proton is the same as that of an electron but with a positive sign In an atom the number of electrons and the number of protons are equal The atom is therefore electrically neutral If one or more electrons are added to it it becomes negatively charged and is designated as negative ion However if one or more electrons are removed from an atom it becomes positively charged and is called a positive ion

The excess or deficiency of electrons in a body gives the concept of charge If there is an excess of electrons in a body it is negatively charged And if there is deficiency of electrons the body becomes positively charged Whenever addition or removal of electrons takes places the body acquires a charge

The SI Unit of charge is coulomb (C) In SI units the current is a fundamental quantity having a unit of ampere (A) The unit of charge is defined in terms of the unit of current Thus one coulomb is the charge transferred in one second across the section of a wire carrying a

Ionic SolidsMetallicSolids

Molecular Solids

current of one ampere

As q = It we have1 C = (1 A) (1 s)

The dimensions of charge are [A T]

Properties of Charge

(1) Quantization of Charge Electric charge can have only discrete values rather than any value That is charge is quantized The smallest discrete value of charge that can exist in nature is the charge on an electron given as

e = plusmn 16 x 10- 19 C

This is the charge attained by an electron and a protonA charge q must be an integral multiple of this basic unit That is

Q = plusmn ne where n = 1 2 hellip

Charge on a body can never be (frac12)e (23)e or 57e etcWhen we rub a glass rod with silk some electrons are transferred from the rod to the silk The rod becomes positively charged The silk becomes negatively charged The coulomb is a very large amount of charge A typical charge acquired by a rubbed body is 10 - 8 C

Biology Reproduction in organisms

Welcome to this new session 2020-21Today in this first chapter we mainly discuss about reproduction types needs and life span of some organismsWe also discuss about difference between sexual and asexual reproduction

Q1 What is reproductionReproduction is defined as a biological processin which an organism gives rise to young onessimilar to itselfQ2 What are the needs of reproductionbulli) Reproduction maintain life on earthii) It enables the continuity of the species generation after generationiii) It creates genetic variation among populationsQ3 Define Life span and write some orgnisms life spanbull Life span is the period from birth to

the natural death of an organism- OrganismsLife span1 Butterfly 1 - 2 weeks2 Fruit fly 30 days3Dog 10-13 years4 Rose5-7 years5 Tortoise100-150 years6 Banyan Tree -200 - 250 yearsQ4 Reproduction is of two types in case ofanimals but in case of plants vegetative propagation is also present

Asexual Reproduction Sexual Reproductioni) Always uniparentalii) Gametes are not involvediii) Only mitotic division involvediv) Somatic cells of parents are involvedv) Offsprings are genetically similar to the parents

i) Usually biparentalii) Gametes are involvediii) Meiosis occurs during gametogenesis Mitosis occurs after fertilisationiv) Germ cells of the parents are involvedv) offsprings are genetically different from the parents

COMMERCE BUSINESS ENVIRONMENT

Welcome to the new sessiontoday we are going to start the first chapter of Class XII The name of the chapter is Business Environment

Already many of you have got some idea about the word business environment form the first chapter of business studies in class XI

In todayrsquos world every business enterprise is a part of the society It exists and operates in association with various groups in society such as customers suppliers competitors banks and financial institutions government agencies trade unions media and so on All these groups influence the functioning of business in one way or the other They constitute the environment of businessConcept of Business Environment

The term lsquobusiness environmentrsquo refers to the sum total of all individuals institutions and other forces that lie outside a business enterprise but that may influence its functioning and performance

The main features of business environment Totality of External forces General and Specific forces Interrelatedness Complexity Dynamic Uncertainty

Prepare the following questions from todayrsquos assignment

2 What do you mean by business environment

The term lsquobusiness environmentrsquo means the aggregate of all forces factors and institutions which are external to and beyond the control of an individual business enterprise but they may influence its functioning and performance Business environment is the macro framework within which a business firm a micro unit operates It consists of several interrelated and interacting elements

2 Explain the main features of business environment in brief

Totality of External forces-Business environment is the sum total of all things external to a business environment

General and Specific forces-It

Relativity

The Interrelation between business and its environment

The business enterprise is an open system It continuously interacts with its environment It takes inputs (such as raw materials capital labour energy and so on) from its environment transforms them into goods and services and sends them back to the environment

Fig 1 Business Environment Relationship

includes both the forces general forces are the economic social political legal and technological conditions which indirectly influence all business enterprise Specific forces are the investors customers competitors and suppliers which influence individual enterprise directly

Interrelatedness-Different elements of environment are interrelated for an example growing awareness for health care has increased the demand for health foods

Complexity- Business environment id complex in nature as the elements keep on changing example economic technological and other forces changes in demand for a product and service

Dynamic-Business environment is not static it keeps on changing

Uncertainty- Itrsquos very difficult to predict future events such as technology and fashion which occur fast and frequently

Business Studies

Human Resources Management

Human resource of an organisation are the aggregate of knowledge skills attitudes of people working in it

The management system which deals with human resources is called human resource management

Features of HRMbullComprehensive functionbullPeople-oriented

Question1) What do you mean by human

resource management Answer) Human resource management may be defined as that field of Management which has to do with planning organising and controlling the functions of procuring developing maintaining and utilising the labour force

bullAction oriented bullPervasive function bullContinuous function

2) Explain the features of HRM in brief

Answer)bullHuman Resource Management is concerned with managing people at work bull Human Resource Management is concerned with employees which bring people and organisations together so that the goals of each are met bullHuman resource management considered every employees as an individual and also promote their satisfaction and growth bull Human resource management is inherent in all organisations and at all levelsbullManagement of human resources are ongoing on never ending process which requires a constant alertness and Awareness of human relations

3) ldquoHR function is said to be pervasiverdquowhy

Answer) Human resource management is required in all organisations whether it is private or government organisations armed forces sports organisations etc It permeatsall the functional areas like production marketing finance research etc This from this feature of human resource management it can be said that it is pervasive in nature

Economics Demand Q1DEFINITION OF DEMANDIn economics demand is the quantity of a good that consumers are willing and able to purchase at various prices during a given period of timeQ2DEMAND CURVEIn economics a demand curve is a graph depicting the relationship between the price of a certain commodity and the quantity of that commodity that is demanded at that pricQ3LAW OF DEMANDIn microeconomics the law of demand states that conditional on all else being equal as the price of a good increases quantity demanded decreases conversely as the price of a good decreases quantity demanded increasesQ4ASSUMPTION of LAW OF DEMAND(i)No change in price of related commodities(ii) No change in income of the consumer(iii) No change in taste and preferences customs habit and fashion of the consumer( No expectation regarding future change in priceQ5MARKET DEMAND SCHEDULEIn economics a market demand schedule is a tabulation of the quantity of a good that all consumers in a market will purchase at a

given price At any given price the corresponding value on the demand schedule is the sum of all consumersrsquo quantities demanded at that priceQ6INDIVIDUAL DEMAND SCHEDULEIndividual demand schedule refers to a tabular statement showing various quantities of a commodity that a consumer is willing to buy at various levels of price during a given period of timeQ7 FACTORS AFFECTING INDIVIDUAL DEMAND FOR A COMMODITY

The factors that influence a consumerrsquos decision to purchase a commodity are also known as determinants of demand The following factors affect the individual demand for a commodity1 price of the commodity2 price of related goods3 income of buyer of the commodity4 tastes and preferences of the buyer1 Price of the CommodityYou must have observed that when price of a commodity falls you tend to buy more of it and when its price rises you tend to buy less of it when all other factors remain constant (lsquoother things remaining the samersquo) In other words other things remaining the same there is an inverse relationship between the price of a commodity and its quantity demanded by its buyers This statement is in accordance with law of demand which you will study in the later part of this lesson Price of a commodity and its quantity demanded by its buyers are inversely related only when lsquoother things remain the samersquo So lsquoother things remaining the samersquo is an assumption when we study the effect of changes in the price of a commodity on its quantity demanded2 Price of Related goodsA consumer may demand a particular good But while buying that good heshe also asks the price of its related goods Related goods can be of two types-(i) Substitute goods(ii) Complementary goods While purchasing a good prices of its substitutes and complements do affect its quantity purchased(i) Price of Substitute Goods Substitute goods are those goods which can easily be used in place of one another for satisfaction of a particular want like tea and coffee An increase in price of substitute good leads to an increase in demand for the given commodity and a decrease in price of substitute good leads to a decrease in demand for the given commodity It means demand for a given commodity is directly affected by change in price of substitute goods For example if price of coffee increases the demand for tea will rise as tea will become relatively cheaper in comparison to coffee(ii) Price of Complementary goods Complementary goods are those goods which are used together to satisfy a particular want like car and petrol An increase in the price of complementary goods leads to a decrease in demand for the given commodity and a decrease in the price of complementary goods leads to an increase in demand for the given commodity For example if price of petrol falls then the demand for cars will increase as it will be relatively cheaper to use both the goods together So demand for a given commodity is inversely affected by change in price of complementary goods3 Income of the Buyer of CommodityDemand for a commodity is also affected by income of its buyer However the effect of change in income on demand depends on the nature of the commodity under consideration In case of some goods like full cream milk fine quality of rice (Basmati rice) etc demand for these commodities increases when income of the buyer increases and

demand for these commodities decreases when income of the buyer decreases Such goods whose demand increases with the increase in income of the buyer are called normal goods But there are some goods like coarse rice toned milk etc whose demand decreases when income of buyer increases and their demand increases when income of the buyer decreases Such goods whose demand decreases with the increase in income of the buyer are called inferior goods Suppose a consumer buys 10 Kgs of rice whose price is ` 25 per Kg He cannot afford to buy better quality of rice because the price of such rice is ` 50 per Kg The consumer is spending ` 250 per month on the purchase of rice Now if income of the consumer increases and he can afford ` 350 on purchase of 10 Kg of rice Now he can afford to buy some quantity of rice say 6 Kgs whose price is ` 25 per Kg and may buy 4 Kgs of rice whose price is ` 50 per Kg Thus he will buy 10 Kgs of rice by spending ` 350 per month Therefore we may conclude that demand for normal goods is directly related to the income of the buyer but demand for inferior goods is inversely related to the income of the buyer4 Tastes and Preferences of the BuyerThe demand for a commodity is also affected by the tastes and preferences of the buyers They include change in fashion customs habits etc Those commodities are preferred by the consumers which are in fashion So demand for those commodities rises which are in fashion On the other hand if a commodity goes out of the fashion its demand falls because no consumer will like to buy it(5) Number of Buyers in the Market(Population)Increase in population raises the market demand whereas decrease in population reduces the market demand for a commodity Not only the size of population but its composition like age (ratio of males females children and old people in population) also affects the demand for a commodity It is because of needs of children young old male and female population differs(6) Distribution of Income and WealthIf the distribution of income and wealth is more in favour of the rich demand for the commodities preferred by the rich such as comforts and luxuries is likely to be higher On the other hand if the distribution of income and wealth is more in favour of poor demand for commodities preferred by the poor such as necessities will be more(7) Season and Weather ConditionsThis is generally observed that the demand for woolens increases during winter whereas demand for ice creams and cold drinks increases during summer Similarly market demand for umbrellas rain coats increases during rainy seasonQ8 REASONS FOR OPERATION OF LAW OF DEMAND WHY DEMAND CURVE SLOPES DOWNWARDNow we will try to explain why does a consumer purchase more quantity of a commodity at a lower price and less of it at a higher price or why does the law of demand operate ie why does the demand curve slope downwards from left to right The main reasons for operation of law of demand are1 Law of Diminishing Marginal UtilityAs you have studied earlier law of diminishing marginal utility states that as we consume more and more units of a commodity the utility derived from each successive unit goes on decreasing The consumer will be ready to pay more for those units which provide him more utility and less for those which provide him less utility It implies that he will purchase more only when the price of the commodity falls2 Income Effect

When price of a commodity falls purchasing power or real income of the consumer increases which enables him to purchase more quantity of the commodity with the same money income Let us take an example Suppose you buy 4 ice creams when price of each ice cream is ` 25 If price of ice creams falls to ` 20 then with same money income you can buy 5 ice creams now3 Substitution EffectWhen price of a commodity falls it becomes comparatively cheaper as compared to its substitutes (although price of substitutes has not been changed) This will lead to rise in demand for the given commodity For example if coke and Pepsi both are sold at ` 10 each and price of coke falls Now coke has become relatively cheaper and will be substituted for Pepsi It will lead to rise in demand for coke4 Change in Number of BuyersWhen price of a commodity falls some old buyers may demand more of the commodity at the reduced price and some new buyers may also start buying this commodity who were not in a position to buy it earlier due to higher price This will lead to increase in number of buyers when price of the commodity falls As a result demand for the commodity rises when its price falls5 Diverse Uses of a CommoditySome commodities have diverse uses like milk It can be used for drinking for sweet preparation for ice cream preparation etc If price of milk rises its use may be restricted to important purpose only This will lead to reduction in demand for other less important uses When price of milk falls it can be put to other uses also leading to rise n demand for itQ9 EXCEPTIONS TO THE LAW OF DEMANDYou have studied in law of demand that a buyer is willing to buy more quantity of a commodity at a lower price and less of it at a higher price But in certain circumstances a rise in price may lead to rise in demand These circumstances are called Exceptions to the Law of Demand Some important exceptions are1 Giffen GoodsGiffen goods are special type of inferior goods in which negative income effect is stronger than negative substitution effect Giffen goods do not follow law of demand as their demand rises when their price rises Examples of Giffen goods are jowar and bajra etc2 Status Symbol GoodsSome goods are used by rich people as status symbols eg diamonds gold jewellary etc The higher the price the higher will be the demand for these goods When price of such goods falls these goods are no longer looked at as status symbol goods and tehrefore therir demand falls3 NecessitiesCommodities such as medicines salt wheat etc do not follow law of demandbecause we have to purchase them in minimum required quantity whatever their price may be4 Goods Expected to be ScarceWhen the buyers expect a scarcity of a particular good in near future they start buying more and more of that good even if their prices are rising For example during war famines etc people tend to buy more of some goods even at higher prices due to fear of their scarcity in near future

Political Science

Constitution of India-The

Preamble

The preamble-

Preamble-

The preamble is the most precious part of the constitution We the people of India having solemnly resolved to constitute India into a Sovereign Socialist Secular Democratic Republic and to secure to all its citizensA preamble is an introductory and expressionary statement in a document that explains the documents purpose and underlying philosophy When applied to the opening paragraphs of a statute it may recite historical facts pertinent to the subject of the statuteNature and purpose of the constitution-Purpose of the Constitution dictates permanent framework of the government to form a more perfect union to establish justice and ensure peace of thenationconstitution provide principles how the government can run itself following the rules and laws written in the constitution of each state keeps them balanced

Answer the following questions-

1 What is preambleA preamble is an introductory and expressionary statement in a document that explains the documents purpose and underlying philosophy2 What is the nature and

purpose of the constitutionConstitution dictatespermanent framework of the government to form a more perfect union to establish justice and ensure peace of the nation

Homework-Learn

Accounts Compatibilty mode

1MEANING OF PARTNERSHIPPartnership is a form of business organisation where two or more persons join hands to run a business They share the profits and losses according to the agreement amongst them According to the Indian Partnership Act 1932 ldquoPartnership is relation between persons who have agreed to share profits of a business carried on by all or any one of them acting for allrdquo For example one of your friends has passed class XII from National Institute of Open Schooling (NIOS) and wants to start a business Heshe approaches you to join in this venture Heshe wants you to contribute some money and participate in the business activities Both of you if join hands constitute a partnership2CHARACTERISTICS1048698 Agreement A partnership is formed by an agreement The agreement may be either oral or in writing It defines the relationship between the persons who agree to carry on business It may contain the terms of sharing profit and the capital to be invested by each partner etc The written agreement is known as partnership deed1048698 Number of persons There must be at least two persons to form a partnership

The maximum number of partners in a partnership firm can be 50 according toCompanies Act 20131048698 Business The Partnership is formed to carry on business with a purpose of earning profits The business should be lawful Thus if two or more persons agree to carry on unlawful activities it will not be termed as partnership1048698 Sharing Profits The partners agree to share profits in the agreed ratio In caseof loss all the partners have to bear it in the same agreed profit sharing ratio10486981048698Mutual Agency Every partner is an agent of the other partners Every partner can bind the firm and all other partners by hisher acts Each partner will be responsible and liable for the acts of all other partners10486981048698Unlimited liability The liability of each partner except that of a minor is unlimited Their liability extends to their personal assets also If the assets of the firm are insufficient to pay off its debts the partnersrsquo personal property can be used to satisfy the claim of the creditors of the partnership firm10486981048698Management All the partners have a right to mange the business However they may authorize one or more partners to manage the affairs of the business on their behalf10486981048698Transferability of Share No partner can transfer hisher share to any one including hisher family member without the consent of all other partners3PARTNERSHIP DEEDAgreement forms the basis of partnership The written form of the agreement is which a document of partnership is It contains terms and conditions regarding the conduct of the business It also explains relationship between the partners This document is called partnership deed Every firm can frame its own partnership deed in which the rights duties and liabilities of the partners are stated in detail It helps in settling the disputes arising among the partners during the general conduct of business 4CONTENTS OF PARTNERSHIP DEEDThe partnership deed generally contains the following (i) Name and address of the partnership firm(ii) Nature and objectives of the business(iii) Name and address of each partner(iv) Ratio in which profits is to be shared(v) Capital contribution by each partner(vi) Rate of Interest on capital if allowed(vii) Salary or any other remuneration to partners if allowed(viii) Rate of interest on loans and advances by a partner to the firm(ix) Drawings of partners and interest thereon if any(x) Method of valuation of goodwill and revaluation of assets and liabilities on the reconstitution of the partnership ie on the admission retirement or death of a partner(xi) Settlement of disputes by arbitration(xii) Settlement of accounts at the time of retirement or death of a partner5IN ABSENCE OF PARTNERSHIP DEEDThe partnership deed lays down the terms and conditions of partnership in regard to rights duties and obligations of the partners In the absence of partnership deed there may arise a controversy on certain issues like profit sharing ratio interest on

capital interest on drawings interest on loan and salary of the partners In such cases the provisions of the Indian Partnership Act becomes applicableSome of the Issues are(i) Distribution of Profit Partners are entitled to share profits equally(ii) Interest on Capital Interest on capital is not allowed(iii) Interest on Drawings No interest on drawing of the partners is to be charged(iv) Interest on Partnerrsquos Loan A Partner is allowed interest 6 per annum on the amount of loan given to the firm by himher(v) Salary and Commission to Partner A partner is not entitled to anysalary or commission or any other remuneration for managing the business

History TOPIC-TOWARDS INDEPENDENCE AND PARTITION THE LAST PHASE (1935-1947)

SUB TOPIC-IMPORTANT POLITICAL DEVELOPMENTS ndash GROWTH OF SOCIAL IDEAS

Socialism is a political social and economic philosophyLike in other parts of the world the Russian revolution of 1917 served as a great inspiration for revolutionaries in India who at that time were engaged in the struggle for liberation from British ruleSocialist ideas led to the formation of communist party of IndiaJAWAHARLAL NEHRU Among the early Congress leaders Jawaharlal Nehru was very much impressed and influenced by the Socialist ideas He also learnt about the Economic activities of the Soviet Union after the Bolshevic Revolution 1917 He made full use of them in IndiaThe election of Jawaharlal Nehru and Subhas Chandra Bose showed the Left wing tendency within CongressJawaharlal Nehru demanded economic freedom along with political freedom of the people in order to end the exploitation of masses

Nehrus working committee included three socialists leaders The Lucknow session was a landmark in the evolution of socialist ideas of the congressSUBHAS CHANDRA BOSE ndash Subhas Chandra Bose had socialist leaning Both Jawaharlal Nehru and Subhas Chandra Bose were known as leftist Congress men Later on National Congress divided into Leftist and rightist campCONGRESS SOCIALIST Within the Congress some leaders formed the Congress Socialist partyPattavi Sitaramyya Sardar Patel Rajendra Prasad had hostile attitude towards the Congress Socialist partyJawaharlals attitude was hesitant

1 QUESTION ndash Mention name of two Congress leaders who had socialist leaning

1ANSWER ndash Subhas Chandra Bose and Jawaharlal Nehru2QUESTION- In which session of the congress Jawaharlal elaborated his Socialist ideas2 ANSWER ndash Lucknow and Faizpur Session in December 1935 and 19363QUESTION ndash Why Congress was sharply divided into leftist and rightist camp 3ANSWER ndash Subhas Chandra Bosersquos attempt to seek re election for congress presidentship in 1939sharply divided the National Congress into Leftist and Rightist camp4 QUESTION ndash Who was MN Roy 4 ANSWER ndash Manabendra Roy first formed the Communist Party of India outside the country at Tashkent in 19205QUESTION ndash Who formed the Congress Socialist Party within the Congress5 ANSWER ndash Jaya Prakash Narayan Achyut Patwardhan Acharya Narendra Dev Ram Mohan Lohia Aruna Asaf Ali6QUESTION ndash When was the Congress Socialist Party formed What was its object6 ANSWER ndash 1934The Congress Socialist Party sought to work out socialist programme through the Congress They joined hands with the Congress and wanted to carry

Subhas Chandra Bose being expelled from the congress after the Tripuri rift he formed Forward BlockThere were basic differences between the Congress Socialists and the communistsTRADE UNION ACTIVITIES Maximum working class people lived in Bombay and Calcutta The working and living conditions of those workers were very miserable In this situation Shasipada Banerjee NM Lokhande protested against the oppression of the working class peopleThe first Trade Union Madras Labour Union was formed in 1918 by BP WadiaIndustrial strikes took place in Kanpur Calcutta Madras Jamshedpur and Ahmedabad AITUC was formed in Bombay in 1927 The growth of Trade union among the workers was slow because of the fear of the dismissal of the jobIn the mean time the Moderates as well as Communists left AITUC and formed separate organization

on National struggle with the help of workers and peasant class of the society7 QUESTION ndash What was the name of the party founded by Subhas Chandra Bose7 ANSWER- Forward Block8QUESTION ndash Who was Shasipada Banerjee8 ANSWER ndash Shasipada Banerjee was a radical Brahmo He founded a working menrsquos club to protest against exploitation of the British rulers towards the working class of India9 QUESTION ndash What was the weekly published by NM Lokhande9ANSWER- Dinabandhu10 QUESTION ndash Who founded Bombay Mill-Hands Association and in which year10 ANSWER- NM Lokhande in189011 QUESTION- Who was BP WadiaANSWER- BPWadia was the founder of Madras Labour Union in191812 QUESTION- What was the name of the first labour union of India12 ANSWER- Madras Labour Union13 QUESTION Who founded the Majur Mahajan 13 ANSWER GANDHIJI14 QUESTION What was the full form of AITUC When it was formed14 ANSWER All India Trade Union Congressin 192715QUESTION Who formed the Red Trade Union Congress and in which year15ANSWER The Communists formed the Red Trade Union Congress16 QUESTION What do you mean by Socialism16 ANSWER Socialism describes any political and economic theory that says the community rather than individuals should own and manage property and natural resources

Subject Eng Literature (The Tempest ndash William Shakespeare) Topic Act III Scene 3 Lines 1 to 52 (Line 52 ndash Brother my lord the Duke Stand to and do as we) Date 13th April 2020 (4th Period)

[Students should read the original play and also the paraphrase given in the school prescribed textbook]Summary Questions amp Answers

o Alonso Sebastian Antonio Gonzalo Adrian Francisco and others wandered about the island in search of Ferdinand and gets tired and hungry of the toil and at the same time gives up all hope of finding him

o Antonio and Sebastian are happy that Alonso is out of hope and decide to make another attempt on his life that night when being so tired they will be sleeping soundly

o Suddenly a solemn and strange music is heard in the air and several strange shapes enter bringing in a banquet These strange shapes then dance round it with gestures of salutation and then inviting the King to eat they depart

o Seeing this strange scene all are inclined to believe the tales told by travelers that there truly are ldquounicornsrdquo and ldquothe phoenixrsquo thronerdquo

1 ALONSO What harmony is this My good friends hark (L18-27)

GONZALO Marvellous sweet music

[Enter several strange shapes bringing in a banquet

they dance about it with gentle actions of salutation

and inviting the King and his companions to eat they depart]ALONSO Give us kind keepers heavens What were theseSEBASTIAN A living drollery Now I will believe

That there are unicorns that in Arabia

There is one tree the phoenixrsquo throne one phoenix

At this hour reigning thereANTONIO Ill believe both

And what does else want credit come to me

And Ill be sworn rsquotis true Travellers neer did lie

Though fools at home condemn rsquoem

(a) How did Prospero present an amazing spectacle before Alonso and his companions

Using his magic powers Prospero ordered strange shapes to lay a banquet before Alonso and his companions The shapes brought several dishes with tasty eatables in them They placed the dishes on a table before Alonso and his companions Then the strange shapes began to dance gracefully around the banquet While dancing they made gestures inviting them to eat the food Then suddenly the shapes disappeared(b) Who were the guests at the strange banquet Describe the lsquoliving drolleryrsquo

Alonso Sebastian Antonio Gonzalo Adrian and Francisco were the guests at the strange banquet

The term ldquoliving drolleryrdquo refers to live entertainment show In this context when Alonso the King of Naples Sebastian his brother Antonio the treacherous brother of Prospero Gonzalo the kind and loyal councillor to the King Adrian and Francisco came to the island they were hungry and weary in their spirits They heard a solemn and strange music They were shocked to see several strange shapes bringing in a banquet and these shapes danced about it with gentle action of salutation inviting the King and his companions to eat After this Sebastian described this show as lsquoliving drolleryrsquo(c) What is lsquophoenixrsquo What are lsquoUnicornsrdquo

The term lsquophoenixrsquo refers to a mythical Arabian bird which lived alone and perched on a solitary tree After one hundred years it expired in flames and rose again from its own ashes

lsquoUnicornsrsquo refers to the mythological four-footed beasts having horns in the centre of their foreheads When the horns are ground into powder the powder was believed to be

an aphrodisiac(d) How does Sebastian explain the puppet show OR Why does the speaker now believe in unicorns and phoenix

Sebastian finds several strange shapes bringing in the banquet They invite the king and his party for dinner and soon depart He tells that if such a strange sight can be a reality there is nothing incredible in the world and from the present moment he will believe anything He says that it is a strange dumb show enacted not by puppets but by living beings It is stranger than a travellerrsquos tale Seeing such a thing

before his own eyes he will no longer disbelieve the story about unicorns and phoenix(e) How do the other characters present respond to this living drollery

At the sight of the lsquoliving drolleryrsquo like Sebastian Gonzalo and Antonio too acted strangely Antonio told that he too now believes in unicorns and phoenix and anything else that seems to be incredible He too now believes in travellersrsquo tales Gonzalo told that if he would report those happenings in Naples nobody will believe him He considers that those gentle shapes were gentler in manner in comparison to the living beings Alonso was at first sight suspicious and told them that those strange shapes conveyed their meaning in expressive gestures when they seemed to lack speech by their movements and sounds Francisco was amazed at their mysterious disappearance

2 ALONSO Not I

(Line 43-52)GONZALO Faith sir you need not fear When we

were boysWho would believe that there were mountaineers

Dewlapped like bulls whose throats had hanging at rsquoem

Wallets of flesh Or that there were such men

Whose heads stood in their breasts Which now we find

Each putter-out of five for one will bring us

Good warrant ofALONSO I will stand to and feed

Although my lastmdashno matter since I feel

The best is past Brother my lord the Duke

Stand to and do as we

(a) How does Alonso respond at the spectacle of the shapes which were sent to them at the instruction of Prospero

After seeing the strange sight of appearing and disappearing of the shapes sent by Prospero to arrange a banquet for them Alonso says that his surprise at having seen those creatures is infinite and he is fully justified in feeling so much surprise He thinks that their shapes their gestures and the sounds they made were indeed amazing Although they do not possess the gift of speech yet they were able to convey their

thoughts by means of their gestures only

(b) What does Prospero say about the views expressed by Alonso regarding the shapes What does Francisco think about the shapesAfter hearing Alonsorsquos views about the shapes Prospero says that this manrsquos praise of the spirits is rather hasty He means to say that Alonso has shown great haste in reaching the conclusion about the shapes Francisco is amazed to see that those shapes disappeared in a mysterious way(c) What does Sebastian ask Alonso to doSebastian tells Alonso that the shapes having disappeared should not matter to them because they have left the eatables behind He asks Alonso to enjoy eating as they are extremely hungry but the king does not accept his offer of enjoying the dishes(d) How does Gonzalo try to dispel Alonsorsquos fear of those strange shapes What kind of references does he give to AlonsoGonzalo says that those who have travelled abroad have reported seeing even stranger sights than these shapes that Alonso and his companions have beheld Hence there is no reason to feel afraid of these shapes Gonzalo further adds that in his younger days he had heard strange stories from travelers and Alonso might have heard similar stories For instance it was said that there existed a certain race of

human beings who had huge lumps of flesh hanging at their throats and who therefore resembled bulls Then Gonzalo tells about a race of human beings whose heads were located at their breasts Gonzalo says that such stories were not believed by most people in those days but now-a-days these stories have become common(e) Explain the following lsquoEach putter-out of five for onersquoEnglish travellers often insured their trips with London brokers Those that went on foreign travels those days used to deposit a certain amount with some firm or company in London before their departure If the travelers failed to return the money was forfeited by the company with which it had been deposited But this money was repaid five-fold if the travelers returned safe and sound In this way a traveler stood a great chance of recovering the entire cost of his

travels(f) Give the explanatory meanings of the following expressions in the context of the above extract (i) Dewlapped (ii) Wallets of flesh

(iii) Putter-out(i) Dewlapped having big lumps of flesh at the necks(ii) Wallets of flesh large masses of flesh looking like bags(iii) Putter-out to invest money before commencing the travel

  • General methods of preparation of hydrogen
  • Chapter Dimensional Analysis (Summary)
    • Properties of Charge
Page 31:   · Web viewSubject. Topic. Summary. Execution. Hindi. व्याकरण. शरीरके अंगो के नाम लिखिए. 1) आँख 2) नाक 3

background Lord Curzon became Viceroy in India He had no respect for the Indian National Congress

the beginning of the organized national movement in India They believed in British justice and were loyal to them They followed a policy of prayer and petition They demanded constitutional reforms of our country Impotant Moderate leaders were Pherozshah Mehta Dadabhai Naorozi and Surendranath Banerjee etcQUESTION 6 What do you know about Extremism in Indian National movementANSWER 6 In the beginning of 20th century a new class of national leaders emerged in India which was different from the moderate groups They started more aggressive movement against the British empire The goal of extremists was ldquoswarajrdquo Important extremist leaders were Bal Gangadhar Tilak Lala Lajpat Rai Bipin Chandra Pal etcQUESTION 7 Mention the places which were the main centres of Revolutionary movementANSWER 7 Maharashtra Bengal and Punjab

Physics

Chapter Dimensional Analysis

(Summary)

The dimensions of a physical quantity are the powers to which the fundamental units are raised in order to obtain the derived unit of that quantit

The physical quantites lengthmasstime are represented by [L] [M] [T] resp let they are raised to powers ( dimesions) abc resp then any physical quantity can be represented by [ La Mb Tc ] Examples

1 Area area = L x B = [L] x [L] = [M0 L2 T0 ]

2 Density density = massvolume = [M][L3] = [ M L-3]

3 Velocity velocity = distancetime = [L][T] = [LT-1]HW Try to find out dimension of acceleration Acceleration = velocity timeNB One can find the SI Units Using Dimension Analysis Such as for area we have [L2] so its SI unit is m2

Biology Topic ndash Chp-1 The living world

Today we will start the first chapter the living world Here we discuss about the characteristics of living organism and what are the difference between them and nonliving substances We also discuss about the contribution of different Scientists

There are over 500000 species of plants andover a million species of animal are present on earth Some 15000 new species were discovered every yearQ1 What is a living organismbull A living organism is primarily physico -chemical material that demonstrate a high degree of complexity is capable of selfRegulation possesses a metabolism and perpetuates itself through timeQ2 What are the differences between livingand non-livingsi) Compared with non-living living organisms

have more complex organised structure and their use of energy is more controlled amp efficientii) Living things reproduce their own kind by forming new cells which contains copies of their genesiii) Each organism has some degree of homeostasisie it is able to make adjustments so that internal environment remains constantQ3 Write contributions of following Scientists i) Aristotle - One of the first theories in Biology places all living things in a hiearchieii) AV Leeuwenhoek - was the first to observe living single celled organisms under microscopeii) Carolus Linnaeus - developed the binary system for naming of organisms and classificationiii) Geregor Johann Mendel ndash discoverbasic principles of inheritanceHomework i) C Darwin ii)Schleiden

Math Trigonometric functions

1 Overviewi) Trigonometry The word lsquotrigonometryrsquo is derived from the Greek words lsquotrigonrsquo and lsquometronrsquo which means measuring the sides of a triangle An angle is the amount of rotation of a revolving line with respect to a fixed line Usually we follow two types of conventions for measuring angles ie a) Sexagesimal system b) Circular system In Sexagesimal system the unit of measurement is Degree In Circular system the unit of measurement is Radian ii) Relation between degree and radianThe ratio of circumference of a circle to its diameter is always a constant This constant ratio is a number denoted by π which is taken approximately as 227The relationship between degree amp radian measurements is as follows2 right angles = 180deg= π radians1radian = 180degπ=57deg16(approx) 1deg=π180 radianiii) Length of an arc of a circleIf an arc of length s subtends an angle θ radians at the center of a circle of radius r then s=rθiv) Area of a sector of a circleA sector is like a pizza slice of the

Q) Express the following angles in radiana) 45deg b) 40deg3730Ans a) We have 180deg=π radiansi e 45deg= πtimes45180 radian = π4 radiansb) 40deg3730= 40deg37+3060 minute= 40deg 37 +12 minute= 40deg+ 752 minute=40 + 75(2times60) degree=3258 degreeNow 180deg=π radianie 3258 degree= (πtimes325) (180times8) radians = 65π288 radiansQ) A circle has a radius of r=12 meters What is the length of an arc traced out by a 60deg angle in the center of the circleAns In this problem we know both the central angle (60deg) and the radius of the circle (12) All we have to do is plug those values into our equation and we get

s = 2π(12)(60360)s = 24π6s = 4πSo the length of an arc traced out by a 60deg angle in a circle with a radius of 12 meters equals 4π meters asymp 1257 metersQ) Find the area of the sector with a central angle 30deg and a radius of 9cmAns GivenRadius r = 9 cmAngle θ = 30degArea of the sector = θ360degtimesπr2

= 30360degtimes227times92=2121cm2

circle It consists of a region bounded by two radii and an arc lying between the radiiThe area of a sector is a fraction of the area of the circle This area is proportional to the central angle In other words the bigger the central angle the larger is the area of the sectorArea of Sector = θ2 times r2 (when θ is in radians)

Area of Sector = θ times π360 times r2 (when θ is in degrees)

COMMERCE

CLASSIFICTION OF HUMAN ACTIVITIES-ECONOMIC AND NON-ECONOMIC

Welcome to the new sessiontoday we are going to start the first chapter of Class XI The name of the chapter that we are going to start is

lsquoClassification of Human Activities ndasheconomic and non-economicrsquo

Now let us start the chapter by considering human beings and the activities they perform throughout the day

Human activities means all those activities that human beings undertake to satisfy their wants

Human wants on the other hand are the desire of human beings for goods (vegetables fruits rice etc) and services (services of doctors teachers lawyers etc) that they require to live

Now these human activities continue throughout life as human wants are unending unlimited and recurring as human beings desire for better living throughout their lives

Now human activities can be classified into two categories

Human activities

Economic activities Non-economic activities

Economic activities are

Questions1 What are human activities

Answer Human activities mean all those activities that human beings undertake to satisfy their wants

Example A man working in an office

A boy playing in the garden

2What are the characteristics of human activitiesAnswer the characteristics of human activities are as follows

Human activities are undertaken by men women and children and these activities involve human efforts

Human activities are undertaken to satisfy human wants which are unlimited

Human activities continue throughout life

Human activities are performed for both earning money and personal satisfaction

3What is economic activitiesGive example

Answer Economic activities are undertaken by human beings with the object of earning money acquiring wealth and thereby satisfying human wantsExample

Selling of goods by a shop keeper to his customer

A clinic run by a doctor Service of a teacher in school or college

undertaken by human beings with the object of earning money and acquiring wealth

These activities result in the production of economic goods and services

Example Human activities(ie working in factories officesshops) which produce direct economic benefits

Non-economic activities are inspired by human sentiments and emotions such as love for the family desire to help the poor and love for the country

Thus these human activities (eg praying playing sleeping) produce no direct economic benefits and they are also not related to earning money and acquiring wealth

4 What are the characteristics of economic activities

Answer The characteristics of economic activities are as follows

Economic motiveEconomic activities are undertaken to earn money and acquire wealth

ProductiveEconomic activities involve productiondistribution and exchange of goods and services to create wealth

Economic growthEconomic activities determine the level of economic development of a country and standard of living of its citizens

Socially desirableEconomic activities are socially desirable for society

Economic resourcesEconomic activities make use of all the economic resources such landlabourcapital etc

5 What do you mean by non-economic activitiesExampleAnswerNon-economic activities are inspired by human sentiments and emotions such as love for the family desire to help the poor and love for the countryThese activities are not undertaken for monetary gain but for onersquos satisfaction and happinessExample

a mother looks after her children

a student donates blood8 Differentiate between Economic activities and Non-economic activities

Economic activities

Non-economic activities

1to earn living and acquiring wealth2Result can be measured in terms of money

3ExampleBusinessprofession and employment

1 to obtain some satisfaction

2Result cannot be measured in terms of money

3ExampleFamily-orientedreligious socialCultural and national

BUSINESS STUDIES

BUSINESS ENVIRONMENT

Welcome to the new sessionToday we are going to start the first chapter and the name of the chapter is Business Environment

In todayrsquos world every business enterprise is a part of the society It exists and operates in association with various groups in society such as customers suppliers competitors banks and financial institutions government agencies trade unions media and so on All these groups influence the functioning of business in one way or the other They constitute the environment of businessConcept of Business Environment

The term lsquobusiness environmentrsquo refers to the sum total of all individuals institutions and other forces that lie outside a business enterprise but that may influence its functioning and performance

The main features of business environment

Totality of External forces General and Specific forces Interrelatedness Complexity Dynamic Uncertainty Relativity

The Interrelation between business and its environment

The business enterprise is an open system It continuously interacts with its environment It takes inputs

Prepare the following questions from todayrsquos assignment

1 What do you mean by business environment

The term lsquobusiness environmentrsquo means the aggregate of all forces factors and institutions which are external to and beyond the control of an individual business enterprise but they may influence its functioning and performance Business environment is the macro framework within which a business firm a micro unit operates It consists of several interrelated and interacting elements

2 Explain the main features of business environment in brief

Totality of External forces-Business environment is the sum total of all things external to a business environment

General and Specific forces-It includes both the forces general forces are the economic social political legal and technological conditions which indirectly influence all business enterprise Specific forces are the investors customers competitors and suppliers which influence individual enterprise directly

Interrelatedness-Different elements of environment are interrelated for an example growing awareness for health care has increased the demand for health foods

Complexity- Business environment id

(such as raw materials capital labour energy and so on) from its environment transforms them into goods and services and sends them back to the environment

Fig 1 Business Environment Relationship

complex in nature as the elements keep on changing example economic technological and other forces changes in demand for a product and service

Dynamic-Business environment is not static it keeps on changing

Uncertainty- Itrsquos very difficult to predict future events such as technology and fashion which occur fast and frequently

Economics Basic Economic ConceptsSub topic

Microeconomics and

Macroeconomics

Welcome to the new sessiontoday we are going to start the first chapter of Class XI The name of the chapter that we are going to start is Basic Economic concepts

Now Economics covers the study of human activities Human activities are those activities which are performed by humans to satisfy their wants

Thus Human wants are unlimited and therefore economic activities such as production exchange and consumption are needed in order to satisfy those wants

The study of economics is divided largely in two parts which areMicroeconomics and Macroeconomics

SUBJECT- MATTER OF ECONOMICS

MICROECONOMICS MACROECONOMICS

Questions1Who has coined the words micro and macro economics

Answer Ranger Frisch coined the words lsquomicrorsquo and lsquomacrorsquo in 1933 to denote the two branches of economic theory namely microeconomics and macroeconomics

2What is microeconomicsAnswer It is the study of behaviour of individual decision ndash making unit such as consumers firms etc

3 What is macroeconomicsAnswer Macroeonomics is the study of overall economic phenomena like employment national income etc

4 What is the importance of microeconomicsAnswer

Microeconomics helps in formulating economic policies which enhance productive efficiency and results in greater social welfare

It helps the government in formulating correct price policies

It explains the working of a capitalistic economy where individual units(producers and consumers ) are free to take their own decision

Micro means a small part in

microeconomics we do not study the whole economy Hence we study an individual consumer and his or her choices and a producer and his or her profit maximizing decisions in the market Thus it does not mirror what happens in the economy as a whole

Macroeconomics on the other hand studies the economy as a whole It is concerned with aggregate and depicts the entire picture of the economyMacroeconomics deals with the national income aggregate investment aggregate consumption etc

Features of Microeconomics It deals with small

parts of the country Hence it looks at

individual consumers firms and industries

It deals with individual income consumption and savings

It studies the determination of price of any product or factors of production

It deals with the working of market via the price mechanism which is nothing but the determination of price and quantity of a commodity by the forces of demand and supply

Features of Macroeconomics

It deals with the study of the economy as a whole

It is concerned with

5 Give a limitation of microeconomics Microeconomics fails to explain the

functioning of an economy as a whole It cannot explain unemployment illiteracy and other problems prevailing in the country

6 What is the importance of macroeconomics It gives overall view of the growing

complexities of an economic system It provides the basic and logical

framework for formulating appropriate macroeconomic policies (eg for inflation poverty etc )to direct and regulate economy towards desirable goals

7What is the limitation of macroeconomics It ignores structural changes in an

individual unit of the aggregate

8 Differentiate between Microeconomics and Macroeconomics

Microeconomics Macroeconomics

the study of aggregates

National income aggregate savings and aggregate investments are major concepts dealt within macroeconomics style

It studies the determination of general price levels

It investigates into the problem of unemployment and the achievement of employment

It studies the aspect of decision making at the aggregate and national levels

It includes all growth theories whether related to developed or developing economies it also includes the study of economic systems and the working of the economy under different systems

Note Both Micro and macro economics are complementary and should be fully utilized for proper understanding of an economy

1It studies economic aspect of an individual unit2It deals with individual incomeConsumption and savings

3 It facilitates determination of price of any product or factors of production

4 Itrsquos scope is narrow and restricted to individual unit

1It studies the economy as a whole

2It deals with the national income aggregate consumption and aggregate savings3 It facilitates determination of general price level in an economy

4 Itrsquos scope is wide as it deals with economic units on the national level

ACCOUNTS

Introduction to Accounting and Book-keeping

Today I am going to share you the meaning of Accounting and Book-keeping and its related terms bullAccounting bullBook Keeping bullAccountsbullTypes Of Accounts bullAccounting Cycle

bull Meaning of accounting

Ans ) Accounting is the art and science of recording classifying and summarising monetary transactions

bull Meaning of Book-keeping

Ans) Bookkeeping is the art of recording business transactions with the view of having a permanent record of them and showing their effect on wealth

bull Meaning of account

Ans) The term account means a record of

business transactions concern a particular person of firm asset or income or expense It is a summarised record of all transactions which take place in an accounting year

bull Types of accountsPersonal accounts ndash Personal accounts relating

to person and Organisation are known as personal accounts Example Ramrsquos Account ABC amp Co Account etc

Real account - The accounts related to tangible and intangible assets are called real accounts Example Cash Account Furniture Account etc

Nominal account- Accounts related to expenses losses incomes and gains are known as nominal accounts Example Wages Account Salary Account Discount Account etc

bull Accounting cycle Accounting cycle refers to a complete sequence of accounting activities It begins with recording of transactions and ends with the preparation of a balance sheet

Chemistry TopicAtomic Structure

Thomsonrsquos atomic modelThomson (1898) was the first to propose the model of an atomHe proposed that an atom can be regarded as a uniform sphere of positive electricity in which requisite number of electrons are embedded evently to neutralize the positive chargeThis is just like plums embedded in a pudding or seeds evently distributed in red spongy mass of a watermelonThis model of atom is known as ldquoPlum-Pudding modelrdquo or

Q1)What is the fundamental constituents of atomAns Electron Proton and neutrons are the fundamental constituents of atomQ2)What is the value of fundamental unit of electricityAnsThe charge carried by one electron is sad to be the fundamental unit of electricityIts magnitude is 48times10-10esuOr 1602times10-19C Q3)Name the element containing no neutronAnsOrdinary hydrogen atom or protium 1H1

Types of AccountPersonal AccountReal AccountNominal AccountBalance Sheet (opening)

ldquowatermelon modelrdquoThis model could explain the electrical neutrality of an atom but failed to explain the result of scattering experiment carried out by Rutherford in 1911So it was rejected ultimately

Q4)Why is an electron called universal particleAns Itrsquos mass and Charge are independent of its source

EVS Chapter 1 ndash Modes of Existence

Modes of existence When one speaks normally about the mode of existence of some group or individual one refers to their customs their mode of being their ethology their habitat in some way their feeling for a placeDifferent modes of exixtence are ndash

1 Hunting ndashGathering2 Pastoral3 Agricultural4 Industrial

1 Hunting and gathering Hunting and gathering mode of existence is characterized by obtaining food from hunting wild animals including fishing and gathering wild plants From their earliest days the hunter-gatherer diet included various grasses tubers fruits seeds and nuts Lacking the means to kill larger animals they procured meat from smaller game or through scavenging

Societies that rely primarily or exclusively on hunting wild animals fishing and gathering wild fruits berries nuts and vegetables to support their diet are called hunting and gathering societies

At least this used to be practice of human beings before agriculture is invented As their brains evolved hominids developed more intricate knowledge of edible plant life and growth cycles

Q) Write the features of Hunting ndash gathering societiesAns - There are five basic characteristics of hunting and gathering societies

i The primary institution is the family which decides how food is to be shared and how children are to be socialized and which provides for the protection of its members

ii They tend to be small with fewer than fifty members

iii They tend to be nomadic moving to new areas when the current food supply in a given area has been exhausted

iv Members display a high level of interdependence

v Labor division is based on sex men hunt and women gather

Political Science

Introduction to political science

Political science occasionally called politology is a social science which deals with systems of governance and the analysis of political activities political thoughts associated constitutions and political behaviorThe study of political science involves the study of both the

Answer the following questions-1 What is political science

Political science occasionally called politology is a social science which deals with systems of governance and the analysis of political activities political thoughts associated constitutions and political behavior

2 Short notes-

traditional and modern theories of politicsTraditionalClassical political sciencepolitical theory-Traditional political science is the study of politics before Second World War The methodology to study Politics was traditional (legal formaletc) the definition of politics traditional (Politics begins and end with state)area of study (constitution state machinery)was traditionalModern Political scienceModern political theory-Modern Political Theory critically examines the contemporary state of political theory making an assessment of the achievement and limitations of the Behavioural Revolution in its totality and reviews objectively the major paradigms and conceptual frameworks adopted by the disciplineContemporary attempts at the development of an integrated political theory involving the use of both traditional and modern concepts approaches and theories-Around late 1960s several political scientists realized the importance of both the traditional political theory and modern Political theory They began building an integrated theory of politics involving a systematic mixture of traditional and modern studies of politics It was held that the study of a complex and vast field like politics needs both traditional as well as

Classical political theory Modern Political theory

Homework-Learn

modern concepts and approaches for studying itrsquos all aspects

Subject Eng Literature (The Tempest ndash William Shakespeare) Topic Act I Scene 1 Lines 1 to 32 (Line 32 ndash Gonzalo hellip If he be not born to be hanged our case is miserable) Date 13th April 2020 (3rd Period)

[Students should read the original play and also the paraphrase given in the school prescribed textbook]Summary Questions amp Answers

[SUMMARY OF THE ENTIRE SCENE]

o The play starts with the scene of a severe storm at sea Alonso (King of Naples) Sebastian (Alonsorsquos brother) Ferdinand (Alonsorsquos son) Gonzalo Antonio (the usurping Duke of Milan) are in a ship in the midst of the storm

o The mariners are trying their best to control the vessel from running aground and are totally following the orders of their Master the Boatswain They have scant success

o The mariners become extremely unhappy and annoyed when most of the passengers arrive on the deck thereby hampering their effort to save the ship There is serious confrontation between them and the passengers who are part of the Kingrsquos entourage

o The mariners could not save the ship

SUMMING-UP

(i) Vivid description of the scene which gives a realistic description of terror and confusion of a tropical storm

(ii) Shows Shakespearersquos accuracy of knowledge in describing the naval operations and also matters of seamanship

(iii) The opening scene justifies the title ndash The Tempest

UNANSWERED QUESTIONS

(i) The King always travels with his entire fleet including his soldiers Where

(1) GONZALO Nay good be patient (Line 15-26)BOATSWAIN When the sea is Hence What cares these

roarers for the name of the king To cabin silence Trouble us not

GONZALO Good yet remember whom thou has aboardBOATSWAIN None that I more love than myself You are a

councillor if you can command these elements to silence and work

the peace of the present we will not hand a rope more use your authority If you cannot give thanks you have

lived so long and make yourself ready in your cabin for the mischance of the hour if it so hap [To the Mariners]

Cheerly good hearts [To Gonzalo] Out of our way I say

(a) To whom is the boatswain speaking What does he mean by lsquoNone that I more love than myselfrsquo

The Boatswain is speaking to Gonzalo the honest old councilor of the Duke of MilanBy using the words ndash lsquoNone that I love more than I love myselfrsquo means that for the Boatswain nobody is dearer to him than his own life

(b) What were the conditions that made the boatswain react in this way

The Boatswain reacts in this way because the storm is at sea and Alonso King of Naples Sebastian his brother Ferdinand his son Gonzalo Antonio the usurping Duke of Milan on board are in distress and in panic Thus they have rushed to the deck interrupting the work of the mariners

(c) What hope does Gonzalo take from the attitude of the boatswain

The insolent and authoritative attitude of Boatswain makes Gonzalo feel comforted He tells that there are no signs that the Boatswain will be drowned But his facial appearance and attitude shows that he is destined to die on land by hanging which in effect means that all on board will be saved Otherwise all the persons on board are doomed

(d) How can they lsquomake yourself ready in your cabinrsquo For what were they asked to make ready themselves

In order to make themselves ready in their cabin the

were the other ships

(ii) Why was the ship in that area Where was it coming from or going where

(iii) The ship broke apart What happened to those who were in the ship

passengers on board must prepare for death which they will possibly soon have to meetThey can retire to their cabins and offer prayers to the Almighty to save them from drowning

(e) What does the boatswain say when he is asked to be patient What does he order to the royal party

When the boatswain is asked to be patient and remain calm he says that he will be patient only when the storm will be over and the sea will be calm but as long as the storm blows and there is danger to the ship he cannot think of being patient He orders the royal party to go to the cabin and leave the mariners to their work

(2) GONZALO I have great comfort from this fellow (Line 27-36)

Methinks he hath no drowning mark upon him his complexion is perfect

gallows Stand fast good Fate to his hanging Make the rope of his destiny our cable for our own doth little advantage If he be not born to be hanged our case is miserable

(a) Why does Gonzalo regard the Boatswain in the midst of danger

In the midst of danger Gonzalo regards the boatswain because he feels that the Boatswain is a source of comfort and is bent upon to do his work sincerely which in this case is saving the ship and its passengers from the severest of raging storm

(b) What reasons does Gonzalo give when he says that none in the ship will die of drowning

Gonzalo is almost sure that none in the ship will die by drowning His says that there is no mark on the face of the boatswain that indicates that he will die by drowning On the other hand the lines on his face are strong indications that he will be hanged to death Therefore there shall be no danger of the shiprsquos sinking

(c) Explain the following ldquoStand fast good Fate to his hanging Make the rope of his destiny our cable for our own doth little advantage If he be not born to be hanged our case is miserablerdquo

The stated lines mean that if the will of destiny is to be carried out then the ship will not get wrecked and all the passengers will be saved The safety of the passengers therefore depends upon the will of fate being carried out in the case of the boatswain If however the boatswain is not to die by hanging then the passengers are also very unsafe because in that case the ship is likely to sink

(d) What order does the Boatswain give to the sailors

when he re-enters What does he say about the crying of the fellows inside the cabin

The boatswain orders the sailors to bring the topmast lower and bring the ship close to a stationary position with the help of the main sail He says that the fellows inside the cabin are moaning and crying in their distress louder than his voice and louder even than the roaring of the storm

Class XII (ScienceCommerceHumanities) Subject Topic Summary Execution

Computer Science

PropositionalLogic

Propositional logic is a procedure to provide reasoning through statementProposition A ststement that results in True or False is said to be proposition There are two types of propositionSimple proposition amp compound propositionSimple proposioton A simple proposition is one that is not a part of any other proposition Such sentential form of proposition is symbolized with english letters in short For example Ram is a claver student (TrueFalse)Where do you live (Not in True or False)Grapes are sweet (TrueFalse)It rains today (TrueFalse)Here we can see some statements anwer would be true or false but some staements answer can not give in terms of true or false Thus the sentences which can be answered in true or false are known as simple propositionAssigning propositon to a variableThe general syntax to assign propostion to a variable is as followsVariable = Simple propositonFor example A=Ram is a clever studentB= Grapes are sweetC= it rains todayCompound proposition

helliphellipto be continued in next classhelliphellipMath Relation Relation If A and B are two non-empty sets

then a relation R from A to B is a subset of AxB If R A x B and (a b) R then we say that a sube isinis related to b by the relation R written as aRbeg Let A be the set of students of class XII and B be the set of students of class XI Then some of the examples of relation from A to B arei) (a b) AXB a is brother of bisinii) (a b) AXB age of a is more than age of isinb Types of relation In this section we would like to study different types of relations We know that a relation in a set A is a subset of A times A Thus the empty set φ and A times A are two extreme relations For illustration consider a relation R in the set A = 1 2 3 4 given by R = (a b) a ndash b = 10 This is the empty set as no pair (a b) satisfies the condition a ndash b = 10 Similarly R = (a b) | a ndash b | ge 0 is the whole primeset A times A as all pairs (a b) in A times A satisfy | a ndash

Example 1 Let A be the set of all students of a boys school Show that the relation R in A given by R = (a b) a is sister of b is the empty relation and R = (a b) the primedifference between heights of a and b is less than 3 meters is the universal relationSolution Since the school is boys school no student of the school can be sister of any student of the school Hence R = φ showing that R is the empty relation It is also obvious that the difference between heights of any two students of the school has to be less than 3 meters This shows that R = A times A is primethe universal relation Example 2 Show that the relation R in the set 1 2 3 given by R = (1 1) (2 2) (3 3) (1 2) (2 3) is reflexive

b | ge 0 These two extreme examples lead us to the following definitionsDefinition 1 A relation R in a set A is called empty relation if no element of A isrelated to any element of A ie R = φ A times AsubDefinition 2 A relation R in a set A is called universal relation if each element of A is related to every element of A ie R = A times A Both the empty relation and the universal relation are some times called trivial relation Definition 3 A relation R in a set A is called(i) reflexive if (a a) R for every a Aisin isin(ii) symmetric if (a1 a2) R implies that (aisin 2a1)

R for all aisin 1 a2 Aisin(iii) transitive if (a1 a2) R and (aisin 2 a3) R isinimplies that (a1 a3) R for all aisin 1 a2 a3 AisinDefinition 4 A relation R in a set A is said to be an equivalence relation if R is reflexive symmetric and transitive

but neither symmetric nor transitiveSolution R is reflexive since (1 1) (2 2) and (3 3) lie in R Also R is not symmetric as (1 2) R but (2 1) isin notinR Similarly R is not transitive as (1 2) R and (2 3) R but (1 3) R isin isin notinExample 3 Show that the relation R in the set Z of integers given byR = (a b) 2 divides a ndash b is an equivalence relationSolution R is reflexive as 2 divides (a ndash a) for all a Z isinFurther if (a b) R then 2 divides a isinndash b Therefore 2 divides b ndash a Hence (b a) R which shows that R is isinsymmetric Similarly if (a b) R and (b c) R isin isinthen a ndash b and b ndash c are divisible by 2 Now a ndash c = (a ndash b) + (b ndash c) is even (Why) So (a ndash c) is divisible by 2 This shows that R is transitive Thus R is an equivalence relation in ZExample 4 Let L be the set of all lines in a plane and R be the relation in L defined as R = (L1 L2) L1 is perpendicular to L2 Show that R is symmetric but neither reflexive nor transitiveSolution R is not reflexive as a line L1 can not be perpendicular to itself ie (L1 L1) R notinR is symmetric as (L1 L2) Risin

L1 is perpendicular to L2rArr L2 is perpendicular to L1rArr (L2 L1) RrArr isin

R is not transitive Indeed if L1 is perpendicular to L2 and L2 is perpendicular to L3 then L1 can never be perpendicular to L3 In fact L1 is parallel to L3 ie (L1 L2) R isin(L2 L3) R but (L1 L3) Risin notin

Chemistry Solid state Characteristics if Solids(i)The particles are locked in fixed positions they are unable to change their relative positions and this brings a definite shape and volume of a solid(ii)In a solid the constituent particles are held by strong forces of attractionThe forces of attraction may be bonding or non bonding(iii)The constituent particles in a solid pack together as closely as possibleoccupying most of the available space within the solidThus the empty space in a solid is very smallThis makes a solid highly rigid and nearly incompressibleThis also explains why a solid has high density and exhibits slow diffusionClassification of Solids

Q1)Define Crystalline solids AnsA Solid that has a definite geometrical shape and a sharp melting pointand whose constituent particles (atomsmolecules or ions) are arranged in a long range order of definite pattern extending throughout the solid is called a crystalline solidExNaClQ2)Define Amorphous solids AnsA solid that does not have a definite shape and a sharp melting pointand whose constituent particles (atomsmolecules or ions) are not arranged in a definite pattern is called an amorphoussolid

Crystalline solidsAmorphous solids

ExGlassRubberQ3)Classify Crystalline Solids Crystalline Solids

Physics Coloumbrsquos Law (Summary)

Before Going Into Coloumbrsquos Law We Will First Learn What is Charge Properties of Charge and Always remember that charge is quantized ie a body always have static charge of magnitude equal to some integral multiple of fundamental electronic charge e= 16 x 10- 19 C

Charge is the property of matter that causes it to produce and experience electrical and magnetic effects The study of the electrical charges at rest is called electrostatics When both electrical and magnetic effects are present the interaction between charges is referred to as electromagnetic

There exist two types of charges in nature positive and negative Like charges repel and unlike charges attract each other

The type of charge on an electron is negative The charge of a proton is the same as that of an electron but with a positive sign In an atom the number of electrons and the number of protons are equal The atom is therefore electrically neutral If one or more electrons are added to it it becomes negatively charged and is designated as negative ion However if one or more electrons are removed from an atom it becomes positively charged and is called a positive ion

The excess or deficiency of electrons in a body gives the concept of charge If there is an excess of electrons in a body it is negatively charged And if there is deficiency of electrons the body becomes positively charged Whenever addition or removal of electrons takes places the body acquires a charge

The SI Unit of charge is coulomb (C) In SI units the current is a fundamental quantity having a unit of ampere (A) The unit of charge is defined in terms of the unit of current Thus one coulomb is the charge transferred in one second across the section of a wire carrying a

Ionic SolidsMetallicSolids

Molecular Solids

current of one ampere

As q = It we have1 C = (1 A) (1 s)

The dimensions of charge are [A T]

Properties of Charge

(1) Quantization of Charge Electric charge can have only discrete values rather than any value That is charge is quantized The smallest discrete value of charge that can exist in nature is the charge on an electron given as

e = plusmn 16 x 10- 19 C

This is the charge attained by an electron and a protonA charge q must be an integral multiple of this basic unit That is

Q = plusmn ne where n = 1 2 hellip

Charge on a body can never be (frac12)e (23)e or 57e etcWhen we rub a glass rod with silk some electrons are transferred from the rod to the silk The rod becomes positively charged The silk becomes negatively charged The coulomb is a very large amount of charge A typical charge acquired by a rubbed body is 10 - 8 C

Biology Reproduction in organisms

Welcome to this new session 2020-21Today in this first chapter we mainly discuss about reproduction types needs and life span of some organismsWe also discuss about difference between sexual and asexual reproduction

Q1 What is reproductionReproduction is defined as a biological processin which an organism gives rise to young onessimilar to itselfQ2 What are the needs of reproductionbulli) Reproduction maintain life on earthii) It enables the continuity of the species generation after generationiii) It creates genetic variation among populationsQ3 Define Life span and write some orgnisms life spanbull Life span is the period from birth to

the natural death of an organism- OrganismsLife span1 Butterfly 1 - 2 weeks2 Fruit fly 30 days3Dog 10-13 years4 Rose5-7 years5 Tortoise100-150 years6 Banyan Tree -200 - 250 yearsQ4 Reproduction is of two types in case ofanimals but in case of plants vegetative propagation is also present

Asexual Reproduction Sexual Reproductioni) Always uniparentalii) Gametes are not involvediii) Only mitotic division involvediv) Somatic cells of parents are involvedv) Offsprings are genetically similar to the parents

i) Usually biparentalii) Gametes are involvediii) Meiosis occurs during gametogenesis Mitosis occurs after fertilisationiv) Germ cells of the parents are involvedv) offsprings are genetically different from the parents

COMMERCE BUSINESS ENVIRONMENT

Welcome to the new sessiontoday we are going to start the first chapter of Class XII The name of the chapter is Business Environment

Already many of you have got some idea about the word business environment form the first chapter of business studies in class XI

In todayrsquos world every business enterprise is a part of the society It exists and operates in association with various groups in society such as customers suppliers competitors banks and financial institutions government agencies trade unions media and so on All these groups influence the functioning of business in one way or the other They constitute the environment of businessConcept of Business Environment

The term lsquobusiness environmentrsquo refers to the sum total of all individuals institutions and other forces that lie outside a business enterprise but that may influence its functioning and performance

The main features of business environment Totality of External forces General and Specific forces Interrelatedness Complexity Dynamic Uncertainty

Prepare the following questions from todayrsquos assignment

2 What do you mean by business environment

The term lsquobusiness environmentrsquo means the aggregate of all forces factors and institutions which are external to and beyond the control of an individual business enterprise but they may influence its functioning and performance Business environment is the macro framework within which a business firm a micro unit operates It consists of several interrelated and interacting elements

2 Explain the main features of business environment in brief

Totality of External forces-Business environment is the sum total of all things external to a business environment

General and Specific forces-It

Relativity

The Interrelation between business and its environment

The business enterprise is an open system It continuously interacts with its environment It takes inputs (such as raw materials capital labour energy and so on) from its environment transforms them into goods and services and sends them back to the environment

Fig 1 Business Environment Relationship

includes both the forces general forces are the economic social political legal and technological conditions which indirectly influence all business enterprise Specific forces are the investors customers competitors and suppliers which influence individual enterprise directly

Interrelatedness-Different elements of environment are interrelated for an example growing awareness for health care has increased the demand for health foods

Complexity- Business environment id complex in nature as the elements keep on changing example economic technological and other forces changes in demand for a product and service

Dynamic-Business environment is not static it keeps on changing

Uncertainty- Itrsquos very difficult to predict future events such as technology and fashion which occur fast and frequently

Business Studies

Human Resources Management

Human resource of an organisation are the aggregate of knowledge skills attitudes of people working in it

The management system which deals with human resources is called human resource management

Features of HRMbullComprehensive functionbullPeople-oriented

Question1) What do you mean by human

resource management Answer) Human resource management may be defined as that field of Management which has to do with planning organising and controlling the functions of procuring developing maintaining and utilising the labour force

bullAction oriented bullPervasive function bullContinuous function

2) Explain the features of HRM in brief

Answer)bullHuman Resource Management is concerned with managing people at work bull Human Resource Management is concerned with employees which bring people and organisations together so that the goals of each are met bullHuman resource management considered every employees as an individual and also promote their satisfaction and growth bull Human resource management is inherent in all organisations and at all levelsbullManagement of human resources are ongoing on never ending process which requires a constant alertness and Awareness of human relations

3) ldquoHR function is said to be pervasiverdquowhy

Answer) Human resource management is required in all organisations whether it is private or government organisations armed forces sports organisations etc It permeatsall the functional areas like production marketing finance research etc This from this feature of human resource management it can be said that it is pervasive in nature

Economics Demand Q1DEFINITION OF DEMANDIn economics demand is the quantity of a good that consumers are willing and able to purchase at various prices during a given period of timeQ2DEMAND CURVEIn economics a demand curve is a graph depicting the relationship between the price of a certain commodity and the quantity of that commodity that is demanded at that pricQ3LAW OF DEMANDIn microeconomics the law of demand states that conditional on all else being equal as the price of a good increases quantity demanded decreases conversely as the price of a good decreases quantity demanded increasesQ4ASSUMPTION of LAW OF DEMAND(i)No change in price of related commodities(ii) No change in income of the consumer(iii) No change in taste and preferences customs habit and fashion of the consumer( No expectation regarding future change in priceQ5MARKET DEMAND SCHEDULEIn economics a market demand schedule is a tabulation of the quantity of a good that all consumers in a market will purchase at a

given price At any given price the corresponding value on the demand schedule is the sum of all consumersrsquo quantities demanded at that priceQ6INDIVIDUAL DEMAND SCHEDULEIndividual demand schedule refers to a tabular statement showing various quantities of a commodity that a consumer is willing to buy at various levels of price during a given period of timeQ7 FACTORS AFFECTING INDIVIDUAL DEMAND FOR A COMMODITY

The factors that influence a consumerrsquos decision to purchase a commodity are also known as determinants of demand The following factors affect the individual demand for a commodity1 price of the commodity2 price of related goods3 income of buyer of the commodity4 tastes and preferences of the buyer1 Price of the CommodityYou must have observed that when price of a commodity falls you tend to buy more of it and when its price rises you tend to buy less of it when all other factors remain constant (lsquoother things remaining the samersquo) In other words other things remaining the same there is an inverse relationship between the price of a commodity and its quantity demanded by its buyers This statement is in accordance with law of demand which you will study in the later part of this lesson Price of a commodity and its quantity demanded by its buyers are inversely related only when lsquoother things remain the samersquo So lsquoother things remaining the samersquo is an assumption when we study the effect of changes in the price of a commodity on its quantity demanded2 Price of Related goodsA consumer may demand a particular good But while buying that good heshe also asks the price of its related goods Related goods can be of two types-(i) Substitute goods(ii) Complementary goods While purchasing a good prices of its substitutes and complements do affect its quantity purchased(i) Price of Substitute Goods Substitute goods are those goods which can easily be used in place of one another for satisfaction of a particular want like tea and coffee An increase in price of substitute good leads to an increase in demand for the given commodity and a decrease in price of substitute good leads to a decrease in demand for the given commodity It means demand for a given commodity is directly affected by change in price of substitute goods For example if price of coffee increases the demand for tea will rise as tea will become relatively cheaper in comparison to coffee(ii) Price of Complementary goods Complementary goods are those goods which are used together to satisfy a particular want like car and petrol An increase in the price of complementary goods leads to a decrease in demand for the given commodity and a decrease in the price of complementary goods leads to an increase in demand for the given commodity For example if price of petrol falls then the demand for cars will increase as it will be relatively cheaper to use both the goods together So demand for a given commodity is inversely affected by change in price of complementary goods3 Income of the Buyer of CommodityDemand for a commodity is also affected by income of its buyer However the effect of change in income on demand depends on the nature of the commodity under consideration In case of some goods like full cream milk fine quality of rice (Basmati rice) etc demand for these commodities increases when income of the buyer increases and

demand for these commodities decreases when income of the buyer decreases Such goods whose demand increases with the increase in income of the buyer are called normal goods But there are some goods like coarse rice toned milk etc whose demand decreases when income of buyer increases and their demand increases when income of the buyer decreases Such goods whose demand decreases with the increase in income of the buyer are called inferior goods Suppose a consumer buys 10 Kgs of rice whose price is ` 25 per Kg He cannot afford to buy better quality of rice because the price of such rice is ` 50 per Kg The consumer is spending ` 250 per month on the purchase of rice Now if income of the consumer increases and he can afford ` 350 on purchase of 10 Kg of rice Now he can afford to buy some quantity of rice say 6 Kgs whose price is ` 25 per Kg and may buy 4 Kgs of rice whose price is ` 50 per Kg Thus he will buy 10 Kgs of rice by spending ` 350 per month Therefore we may conclude that demand for normal goods is directly related to the income of the buyer but demand for inferior goods is inversely related to the income of the buyer4 Tastes and Preferences of the BuyerThe demand for a commodity is also affected by the tastes and preferences of the buyers They include change in fashion customs habits etc Those commodities are preferred by the consumers which are in fashion So demand for those commodities rises which are in fashion On the other hand if a commodity goes out of the fashion its demand falls because no consumer will like to buy it(5) Number of Buyers in the Market(Population)Increase in population raises the market demand whereas decrease in population reduces the market demand for a commodity Not only the size of population but its composition like age (ratio of males females children and old people in population) also affects the demand for a commodity It is because of needs of children young old male and female population differs(6) Distribution of Income and WealthIf the distribution of income and wealth is more in favour of the rich demand for the commodities preferred by the rich such as comforts and luxuries is likely to be higher On the other hand if the distribution of income and wealth is more in favour of poor demand for commodities preferred by the poor such as necessities will be more(7) Season and Weather ConditionsThis is generally observed that the demand for woolens increases during winter whereas demand for ice creams and cold drinks increases during summer Similarly market demand for umbrellas rain coats increases during rainy seasonQ8 REASONS FOR OPERATION OF LAW OF DEMAND WHY DEMAND CURVE SLOPES DOWNWARDNow we will try to explain why does a consumer purchase more quantity of a commodity at a lower price and less of it at a higher price or why does the law of demand operate ie why does the demand curve slope downwards from left to right The main reasons for operation of law of demand are1 Law of Diminishing Marginal UtilityAs you have studied earlier law of diminishing marginal utility states that as we consume more and more units of a commodity the utility derived from each successive unit goes on decreasing The consumer will be ready to pay more for those units which provide him more utility and less for those which provide him less utility It implies that he will purchase more only when the price of the commodity falls2 Income Effect

When price of a commodity falls purchasing power or real income of the consumer increases which enables him to purchase more quantity of the commodity with the same money income Let us take an example Suppose you buy 4 ice creams when price of each ice cream is ` 25 If price of ice creams falls to ` 20 then with same money income you can buy 5 ice creams now3 Substitution EffectWhen price of a commodity falls it becomes comparatively cheaper as compared to its substitutes (although price of substitutes has not been changed) This will lead to rise in demand for the given commodity For example if coke and Pepsi both are sold at ` 10 each and price of coke falls Now coke has become relatively cheaper and will be substituted for Pepsi It will lead to rise in demand for coke4 Change in Number of BuyersWhen price of a commodity falls some old buyers may demand more of the commodity at the reduced price and some new buyers may also start buying this commodity who were not in a position to buy it earlier due to higher price This will lead to increase in number of buyers when price of the commodity falls As a result demand for the commodity rises when its price falls5 Diverse Uses of a CommoditySome commodities have diverse uses like milk It can be used for drinking for sweet preparation for ice cream preparation etc If price of milk rises its use may be restricted to important purpose only This will lead to reduction in demand for other less important uses When price of milk falls it can be put to other uses also leading to rise n demand for itQ9 EXCEPTIONS TO THE LAW OF DEMANDYou have studied in law of demand that a buyer is willing to buy more quantity of a commodity at a lower price and less of it at a higher price But in certain circumstances a rise in price may lead to rise in demand These circumstances are called Exceptions to the Law of Demand Some important exceptions are1 Giffen GoodsGiffen goods are special type of inferior goods in which negative income effect is stronger than negative substitution effect Giffen goods do not follow law of demand as their demand rises when their price rises Examples of Giffen goods are jowar and bajra etc2 Status Symbol GoodsSome goods are used by rich people as status symbols eg diamonds gold jewellary etc The higher the price the higher will be the demand for these goods When price of such goods falls these goods are no longer looked at as status symbol goods and tehrefore therir demand falls3 NecessitiesCommodities such as medicines salt wheat etc do not follow law of demandbecause we have to purchase them in minimum required quantity whatever their price may be4 Goods Expected to be ScarceWhen the buyers expect a scarcity of a particular good in near future they start buying more and more of that good even if their prices are rising For example during war famines etc people tend to buy more of some goods even at higher prices due to fear of their scarcity in near future

Political Science

Constitution of India-The

Preamble

The preamble-

Preamble-

The preamble is the most precious part of the constitution We the people of India having solemnly resolved to constitute India into a Sovereign Socialist Secular Democratic Republic and to secure to all its citizensA preamble is an introductory and expressionary statement in a document that explains the documents purpose and underlying philosophy When applied to the opening paragraphs of a statute it may recite historical facts pertinent to the subject of the statuteNature and purpose of the constitution-Purpose of the Constitution dictates permanent framework of the government to form a more perfect union to establish justice and ensure peace of thenationconstitution provide principles how the government can run itself following the rules and laws written in the constitution of each state keeps them balanced

Answer the following questions-

1 What is preambleA preamble is an introductory and expressionary statement in a document that explains the documents purpose and underlying philosophy2 What is the nature and

purpose of the constitutionConstitution dictatespermanent framework of the government to form a more perfect union to establish justice and ensure peace of the nation

Homework-Learn

Accounts Compatibilty mode

1MEANING OF PARTNERSHIPPartnership is a form of business organisation where two or more persons join hands to run a business They share the profits and losses according to the agreement amongst them According to the Indian Partnership Act 1932 ldquoPartnership is relation between persons who have agreed to share profits of a business carried on by all or any one of them acting for allrdquo For example one of your friends has passed class XII from National Institute of Open Schooling (NIOS) and wants to start a business Heshe approaches you to join in this venture Heshe wants you to contribute some money and participate in the business activities Both of you if join hands constitute a partnership2CHARACTERISTICS1048698 Agreement A partnership is formed by an agreement The agreement may be either oral or in writing It defines the relationship between the persons who agree to carry on business It may contain the terms of sharing profit and the capital to be invested by each partner etc The written agreement is known as partnership deed1048698 Number of persons There must be at least two persons to form a partnership

The maximum number of partners in a partnership firm can be 50 according toCompanies Act 20131048698 Business The Partnership is formed to carry on business with a purpose of earning profits The business should be lawful Thus if two or more persons agree to carry on unlawful activities it will not be termed as partnership1048698 Sharing Profits The partners agree to share profits in the agreed ratio In caseof loss all the partners have to bear it in the same agreed profit sharing ratio10486981048698Mutual Agency Every partner is an agent of the other partners Every partner can bind the firm and all other partners by hisher acts Each partner will be responsible and liable for the acts of all other partners10486981048698Unlimited liability The liability of each partner except that of a minor is unlimited Their liability extends to their personal assets also If the assets of the firm are insufficient to pay off its debts the partnersrsquo personal property can be used to satisfy the claim of the creditors of the partnership firm10486981048698Management All the partners have a right to mange the business However they may authorize one or more partners to manage the affairs of the business on their behalf10486981048698Transferability of Share No partner can transfer hisher share to any one including hisher family member without the consent of all other partners3PARTNERSHIP DEEDAgreement forms the basis of partnership The written form of the agreement is which a document of partnership is It contains terms and conditions regarding the conduct of the business It also explains relationship between the partners This document is called partnership deed Every firm can frame its own partnership deed in which the rights duties and liabilities of the partners are stated in detail It helps in settling the disputes arising among the partners during the general conduct of business 4CONTENTS OF PARTNERSHIP DEEDThe partnership deed generally contains the following (i) Name and address of the partnership firm(ii) Nature and objectives of the business(iii) Name and address of each partner(iv) Ratio in which profits is to be shared(v) Capital contribution by each partner(vi) Rate of Interest on capital if allowed(vii) Salary or any other remuneration to partners if allowed(viii) Rate of interest on loans and advances by a partner to the firm(ix) Drawings of partners and interest thereon if any(x) Method of valuation of goodwill and revaluation of assets and liabilities on the reconstitution of the partnership ie on the admission retirement or death of a partner(xi) Settlement of disputes by arbitration(xii) Settlement of accounts at the time of retirement or death of a partner5IN ABSENCE OF PARTNERSHIP DEEDThe partnership deed lays down the terms and conditions of partnership in regard to rights duties and obligations of the partners In the absence of partnership deed there may arise a controversy on certain issues like profit sharing ratio interest on

capital interest on drawings interest on loan and salary of the partners In such cases the provisions of the Indian Partnership Act becomes applicableSome of the Issues are(i) Distribution of Profit Partners are entitled to share profits equally(ii) Interest on Capital Interest on capital is not allowed(iii) Interest on Drawings No interest on drawing of the partners is to be charged(iv) Interest on Partnerrsquos Loan A Partner is allowed interest 6 per annum on the amount of loan given to the firm by himher(v) Salary and Commission to Partner A partner is not entitled to anysalary or commission or any other remuneration for managing the business

History TOPIC-TOWARDS INDEPENDENCE AND PARTITION THE LAST PHASE (1935-1947)

SUB TOPIC-IMPORTANT POLITICAL DEVELOPMENTS ndash GROWTH OF SOCIAL IDEAS

Socialism is a political social and economic philosophyLike in other parts of the world the Russian revolution of 1917 served as a great inspiration for revolutionaries in India who at that time were engaged in the struggle for liberation from British ruleSocialist ideas led to the formation of communist party of IndiaJAWAHARLAL NEHRU Among the early Congress leaders Jawaharlal Nehru was very much impressed and influenced by the Socialist ideas He also learnt about the Economic activities of the Soviet Union after the Bolshevic Revolution 1917 He made full use of them in IndiaThe election of Jawaharlal Nehru and Subhas Chandra Bose showed the Left wing tendency within CongressJawaharlal Nehru demanded economic freedom along with political freedom of the people in order to end the exploitation of masses

Nehrus working committee included three socialists leaders The Lucknow session was a landmark in the evolution of socialist ideas of the congressSUBHAS CHANDRA BOSE ndash Subhas Chandra Bose had socialist leaning Both Jawaharlal Nehru and Subhas Chandra Bose were known as leftist Congress men Later on National Congress divided into Leftist and rightist campCONGRESS SOCIALIST Within the Congress some leaders formed the Congress Socialist partyPattavi Sitaramyya Sardar Patel Rajendra Prasad had hostile attitude towards the Congress Socialist partyJawaharlals attitude was hesitant

1 QUESTION ndash Mention name of two Congress leaders who had socialist leaning

1ANSWER ndash Subhas Chandra Bose and Jawaharlal Nehru2QUESTION- In which session of the congress Jawaharlal elaborated his Socialist ideas2 ANSWER ndash Lucknow and Faizpur Session in December 1935 and 19363QUESTION ndash Why Congress was sharply divided into leftist and rightist camp 3ANSWER ndash Subhas Chandra Bosersquos attempt to seek re election for congress presidentship in 1939sharply divided the National Congress into Leftist and Rightist camp4 QUESTION ndash Who was MN Roy 4 ANSWER ndash Manabendra Roy first formed the Communist Party of India outside the country at Tashkent in 19205QUESTION ndash Who formed the Congress Socialist Party within the Congress5 ANSWER ndash Jaya Prakash Narayan Achyut Patwardhan Acharya Narendra Dev Ram Mohan Lohia Aruna Asaf Ali6QUESTION ndash When was the Congress Socialist Party formed What was its object6 ANSWER ndash 1934The Congress Socialist Party sought to work out socialist programme through the Congress They joined hands with the Congress and wanted to carry

Subhas Chandra Bose being expelled from the congress after the Tripuri rift he formed Forward BlockThere were basic differences between the Congress Socialists and the communistsTRADE UNION ACTIVITIES Maximum working class people lived in Bombay and Calcutta The working and living conditions of those workers were very miserable In this situation Shasipada Banerjee NM Lokhande protested against the oppression of the working class peopleThe first Trade Union Madras Labour Union was formed in 1918 by BP WadiaIndustrial strikes took place in Kanpur Calcutta Madras Jamshedpur and Ahmedabad AITUC was formed in Bombay in 1927 The growth of Trade union among the workers was slow because of the fear of the dismissal of the jobIn the mean time the Moderates as well as Communists left AITUC and formed separate organization

on National struggle with the help of workers and peasant class of the society7 QUESTION ndash What was the name of the party founded by Subhas Chandra Bose7 ANSWER- Forward Block8QUESTION ndash Who was Shasipada Banerjee8 ANSWER ndash Shasipada Banerjee was a radical Brahmo He founded a working menrsquos club to protest against exploitation of the British rulers towards the working class of India9 QUESTION ndash What was the weekly published by NM Lokhande9ANSWER- Dinabandhu10 QUESTION ndash Who founded Bombay Mill-Hands Association and in which year10 ANSWER- NM Lokhande in189011 QUESTION- Who was BP WadiaANSWER- BPWadia was the founder of Madras Labour Union in191812 QUESTION- What was the name of the first labour union of India12 ANSWER- Madras Labour Union13 QUESTION Who founded the Majur Mahajan 13 ANSWER GANDHIJI14 QUESTION What was the full form of AITUC When it was formed14 ANSWER All India Trade Union Congressin 192715QUESTION Who formed the Red Trade Union Congress and in which year15ANSWER The Communists formed the Red Trade Union Congress16 QUESTION What do you mean by Socialism16 ANSWER Socialism describes any political and economic theory that says the community rather than individuals should own and manage property and natural resources

Subject Eng Literature (The Tempest ndash William Shakespeare) Topic Act III Scene 3 Lines 1 to 52 (Line 52 ndash Brother my lord the Duke Stand to and do as we) Date 13th April 2020 (4th Period)

[Students should read the original play and also the paraphrase given in the school prescribed textbook]Summary Questions amp Answers

o Alonso Sebastian Antonio Gonzalo Adrian Francisco and others wandered about the island in search of Ferdinand and gets tired and hungry of the toil and at the same time gives up all hope of finding him

o Antonio and Sebastian are happy that Alonso is out of hope and decide to make another attempt on his life that night when being so tired they will be sleeping soundly

o Suddenly a solemn and strange music is heard in the air and several strange shapes enter bringing in a banquet These strange shapes then dance round it with gestures of salutation and then inviting the King to eat they depart

o Seeing this strange scene all are inclined to believe the tales told by travelers that there truly are ldquounicornsrdquo and ldquothe phoenixrsquo thronerdquo

1 ALONSO What harmony is this My good friends hark (L18-27)

GONZALO Marvellous sweet music

[Enter several strange shapes bringing in a banquet

they dance about it with gentle actions of salutation

and inviting the King and his companions to eat they depart]ALONSO Give us kind keepers heavens What were theseSEBASTIAN A living drollery Now I will believe

That there are unicorns that in Arabia

There is one tree the phoenixrsquo throne one phoenix

At this hour reigning thereANTONIO Ill believe both

And what does else want credit come to me

And Ill be sworn rsquotis true Travellers neer did lie

Though fools at home condemn rsquoem

(a) How did Prospero present an amazing spectacle before Alonso and his companions

Using his magic powers Prospero ordered strange shapes to lay a banquet before Alonso and his companions The shapes brought several dishes with tasty eatables in them They placed the dishes on a table before Alonso and his companions Then the strange shapes began to dance gracefully around the banquet While dancing they made gestures inviting them to eat the food Then suddenly the shapes disappeared(b) Who were the guests at the strange banquet Describe the lsquoliving drolleryrsquo

Alonso Sebastian Antonio Gonzalo Adrian and Francisco were the guests at the strange banquet

The term ldquoliving drolleryrdquo refers to live entertainment show In this context when Alonso the King of Naples Sebastian his brother Antonio the treacherous brother of Prospero Gonzalo the kind and loyal councillor to the King Adrian and Francisco came to the island they were hungry and weary in their spirits They heard a solemn and strange music They were shocked to see several strange shapes bringing in a banquet and these shapes danced about it with gentle action of salutation inviting the King and his companions to eat After this Sebastian described this show as lsquoliving drolleryrsquo(c) What is lsquophoenixrsquo What are lsquoUnicornsrdquo

The term lsquophoenixrsquo refers to a mythical Arabian bird which lived alone and perched on a solitary tree After one hundred years it expired in flames and rose again from its own ashes

lsquoUnicornsrsquo refers to the mythological four-footed beasts having horns in the centre of their foreheads When the horns are ground into powder the powder was believed to be

an aphrodisiac(d) How does Sebastian explain the puppet show OR Why does the speaker now believe in unicorns and phoenix

Sebastian finds several strange shapes bringing in the banquet They invite the king and his party for dinner and soon depart He tells that if such a strange sight can be a reality there is nothing incredible in the world and from the present moment he will believe anything He says that it is a strange dumb show enacted not by puppets but by living beings It is stranger than a travellerrsquos tale Seeing such a thing

before his own eyes he will no longer disbelieve the story about unicorns and phoenix(e) How do the other characters present respond to this living drollery

At the sight of the lsquoliving drolleryrsquo like Sebastian Gonzalo and Antonio too acted strangely Antonio told that he too now believes in unicorns and phoenix and anything else that seems to be incredible He too now believes in travellersrsquo tales Gonzalo told that if he would report those happenings in Naples nobody will believe him He considers that those gentle shapes were gentler in manner in comparison to the living beings Alonso was at first sight suspicious and told them that those strange shapes conveyed their meaning in expressive gestures when they seemed to lack speech by their movements and sounds Francisco was amazed at their mysterious disappearance

2 ALONSO Not I

(Line 43-52)GONZALO Faith sir you need not fear When we

were boysWho would believe that there were mountaineers

Dewlapped like bulls whose throats had hanging at rsquoem

Wallets of flesh Or that there were such men

Whose heads stood in their breasts Which now we find

Each putter-out of five for one will bring us

Good warrant ofALONSO I will stand to and feed

Although my lastmdashno matter since I feel

The best is past Brother my lord the Duke

Stand to and do as we

(a) How does Alonso respond at the spectacle of the shapes which were sent to them at the instruction of Prospero

After seeing the strange sight of appearing and disappearing of the shapes sent by Prospero to arrange a banquet for them Alonso says that his surprise at having seen those creatures is infinite and he is fully justified in feeling so much surprise He thinks that their shapes their gestures and the sounds they made were indeed amazing Although they do not possess the gift of speech yet they were able to convey their

thoughts by means of their gestures only

(b) What does Prospero say about the views expressed by Alonso regarding the shapes What does Francisco think about the shapesAfter hearing Alonsorsquos views about the shapes Prospero says that this manrsquos praise of the spirits is rather hasty He means to say that Alonso has shown great haste in reaching the conclusion about the shapes Francisco is amazed to see that those shapes disappeared in a mysterious way(c) What does Sebastian ask Alonso to doSebastian tells Alonso that the shapes having disappeared should not matter to them because they have left the eatables behind He asks Alonso to enjoy eating as they are extremely hungry but the king does not accept his offer of enjoying the dishes(d) How does Gonzalo try to dispel Alonsorsquos fear of those strange shapes What kind of references does he give to AlonsoGonzalo says that those who have travelled abroad have reported seeing even stranger sights than these shapes that Alonso and his companions have beheld Hence there is no reason to feel afraid of these shapes Gonzalo further adds that in his younger days he had heard strange stories from travelers and Alonso might have heard similar stories For instance it was said that there existed a certain race of

human beings who had huge lumps of flesh hanging at their throats and who therefore resembled bulls Then Gonzalo tells about a race of human beings whose heads were located at their breasts Gonzalo says that such stories were not believed by most people in those days but now-a-days these stories have become common(e) Explain the following lsquoEach putter-out of five for onersquoEnglish travellers often insured their trips with London brokers Those that went on foreign travels those days used to deposit a certain amount with some firm or company in London before their departure If the travelers failed to return the money was forfeited by the company with which it had been deposited But this money was repaid five-fold if the travelers returned safe and sound In this way a traveler stood a great chance of recovering the entire cost of his

travels(f) Give the explanatory meanings of the following expressions in the context of the above extract (i) Dewlapped (ii) Wallets of flesh

(iii) Putter-out(i) Dewlapped having big lumps of flesh at the necks(ii) Wallets of flesh large masses of flesh looking like bags(iii) Putter-out to invest money before commencing the travel

  • General methods of preparation of hydrogen
  • Chapter Dimensional Analysis (Summary)
    • Properties of Charge
Page 32:   · Web viewSubject. Topic. Summary. Execution. Hindi. व्याकरण. शरीरके अंगो के नाम लिखिए. 1) आँख 2) नाक 3

have more complex organised structure and their use of energy is more controlled amp efficientii) Living things reproduce their own kind by forming new cells which contains copies of their genesiii) Each organism has some degree of homeostasisie it is able to make adjustments so that internal environment remains constantQ3 Write contributions of following Scientists i) Aristotle - One of the first theories in Biology places all living things in a hiearchieii) AV Leeuwenhoek - was the first to observe living single celled organisms under microscopeii) Carolus Linnaeus - developed the binary system for naming of organisms and classificationiii) Geregor Johann Mendel ndash discoverbasic principles of inheritanceHomework i) C Darwin ii)Schleiden

Math Trigonometric functions

1 Overviewi) Trigonometry The word lsquotrigonometryrsquo is derived from the Greek words lsquotrigonrsquo and lsquometronrsquo which means measuring the sides of a triangle An angle is the amount of rotation of a revolving line with respect to a fixed line Usually we follow two types of conventions for measuring angles ie a) Sexagesimal system b) Circular system In Sexagesimal system the unit of measurement is Degree In Circular system the unit of measurement is Radian ii) Relation between degree and radianThe ratio of circumference of a circle to its diameter is always a constant This constant ratio is a number denoted by π which is taken approximately as 227The relationship between degree amp radian measurements is as follows2 right angles = 180deg= π radians1radian = 180degπ=57deg16(approx) 1deg=π180 radianiii) Length of an arc of a circleIf an arc of length s subtends an angle θ radians at the center of a circle of radius r then s=rθiv) Area of a sector of a circleA sector is like a pizza slice of the

Q) Express the following angles in radiana) 45deg b) 40deg3730Ans a) We have 180deg=π radiansi e 45deg= πtimes45180 radian = π4 radiansb) 40deg3730= 40deg37+3060 minute= 40deg 37 +12 minute= 40deg+ 752 minute=40 + 75(2times60) degree=3258 degreeNow 180deg=π radianie 3258 degree= (πtimes325) (180times8) radians = 65π288 radiansQ) A circle has a radius of r=12 meters What is the length of an arc traced out by a 60deg angle in the center of the circleAns In this problem we know both the central angle (60deg) and the radius of the circle (12) All we have to do is plug those values into our equation and we get

s = 2π(12)(60360)s = 24π6s = 4πSo the length of an arc traced out by a 60deg angle in a circle with a radius of 12 meters equals 4π meters asymp 1257 metersQ) Find the area of the sector with a central angle 30deg and a radius of 9cmAns GivenRadius r = 9 cmAngle θ = 30degArea of the sector = θ360degtimesπr2

= 30360degtimes227times92=2121cm2

circle It consists of a region bounded by two radii and an arc lying between the radiiThe area of a sector is a fraction of the area of the circle This area is proportional to the central angle In other words the bigger the central angle the larger is the area of the sectorArea of Sector = θ2 times r2 (when θ is in radians)

Area of Sector = θ times π360 times r2 (when θ is in degrees)

COMMERCE

CLASSIFICTION OF HUMAN ACTIVITIES-ECONOMIC AND NON-ECONOMIC

Welcome to the new sessiontoday we are going to start the first chapter of Class XI The name of the chapter that we are going to start is

lsquoClassification of Human Activities ndasheconomic and non-economicrsquo

Now let us start the chapter by considering human beings and the activities they perform throughout the day

Human activities means all those activities that human beings undertake to satisfy their wants

Human wants on the other hand are the desire of human beings for goods (vegetables fruits rice etc) and services (services of doctors teachers lawyers etc) that they require to live

Now these human activities continue throughout life as human wants are unending unlimited and recurring as human beings desire for better living throughout their lives

Now human activities can be classified into two categories

Human activities

Economic activities Non-economic activities

Economic activities are

Questions1 What are human activities

Answer Human activities mean all those activities that human beings undertake to satisfy their wants

Example A man working in an office

A boy playing in the garden

2What are the characteristics of human activitiesAnswer the characteristics of human activities are as follows

Human activities are undertaken by men women and children and these activities involve human efforts

Human activities are undertaken to satisfy human wants which are unlimited

Human activities continue throughout life

Human activities are performed for both earning money and personal satisfaction

3What is economic activitiesGive example

Answer Economic activities are undertaken by human beings with the object of earning money acquiring wealth and thereby satisfying human wantsExample

Selling of goods by a shop keeper to his customer

A clinic run by a doctor Service of a teacher in school or college

undertaken by human beings with the object of earning money and acquiring wealth

These activities result in the production of economic goods and services

Example Human activities(ie working in factories officesshops) which produce direct economic benefits

Non-economic activities are inspired by human sentiments and emotions such as love for the family desire to help the poor and love for the country

Thus these human activities (eg praying playing sleeping) produce no direct economic benefits and they are also not related to earning money and acquiring wealth

4 What are the characteristics of economic activities

Answer The characteristics of economic activities are as follows

Economic motiveEconomic activities are undertaken to earn money and acquire wealth

ProductiveEconomic activities involve productiondistribution and exchange of goods and services to create wealth

Economic growthEconomic activities determine the level of economic development of a country and standard of living of its citizens

Socially desirableEconomic activities are socially desirable for society

Economic resourcesEconomic activities make use of all the economic resources such landlabourcapital etc

5 What do you mean by non-economic activitiesExampleAnswerNon-economic activities are inspired by human sentiments and emotions such as love for the family desire to help the poor and love for the countryThese activities are not undertaken for monetary gain but for onersquos satisfaction and happinessExample

a mother looks after her children

a student donates blood8 Differentiate between Economic activities and Non-economic activities

Economic activities

Non-economic activities

1to earn living and acquiring wealth2Result can be measured in terms of money

3ExampleBusinessprofession and employment

1 to obtain some satisfaction

2Result cannot be measured in terms of money

3ExampleFamily-orientedreligious socialCultural and national

BUSINESS STUDIES

BUSINESS ENVIRONMENT

Welcome to the new sessionToday we are going to start the first chapter and the name of the chapter is Business Environment

In todayrsquos world every business enterprise is a part of the society It exists and operates in association with various groups in society such as customers suppliers competitors banks and financial institutions government agencies trade unions media and so on All these groups influence the functioning of business in one way or the other They constitute the environment of businessConcept of Business Environment

The term lsquobusiness environmentrsquo refers to the sum total of all individuals institutions and other forces that lie outside a business enterprise but that may influence its functioning and performance

The main features of business environment

Totality of External forces General and Specific forces Interrelatedness Complexity Dynamic Uncertainty Relativity

The Interrelation between business and its environment

The business enterprise is an open system It continuously interacts with its environment It takes inputs

Prepare the following questions from todayrsquos assignment

1 What do you mean by business environment

The term lsquobusiness environmentrsquo means the aggregate of all forces factors and institutions which are external to and beyond the control of an individual business enterprise but they may influence its functioning and performance Business environment is the macro framework within which a business firm a micro unit operates It consists of several interrelated and interacting elements

2 Explain the main features of business environment in brief

Totality of External forces-Business environment is the sum total of all things external to a business environment

General and Specific forces-It includes both the forces general forces are the economic social political legal and technological conditions which indirectly influence all business enterprise Specific forces are the investors customers competitors and suppliers which influence individual enterprise directly

Interrelatedness-Different elements of environment are interrelated for an example growing awareness for health care has increased the demand for health foods

Complexity- Business environment id

(such as raw materials capital labour energy and so on) from its environment transforms them into goods and services and sends them back to the environment

Fig 1 Business Environment Relationship

complex in nature as the elements keep on changing example economic technological and other forces changes in demand for a product and service

Dynamic-Business environment is not static it keeps on changing

Uncertainty- Itrsquos very difficult to predict future events such as technology and fashion which occur fast and frequently

Economics Basic Economic ConceptsSub topic

Microeconomics and

Macroeconomics

Welcome to the new sessiontoday we are going to start the first chapter of Class XI The name of the chapter that we are going to start is Basic Economic concepts

Now Economics covers the study of human activities Human activities are those activities which are performed by humans to satisfy their wants

Thus Human wants are unlimited and therefore economic activities such as production exchange and consumption are needed in order to satisfy those wants

The study of economics is divided largely in two parts which areMicroeconomics and Macroeconomics

SUBJECT- MATTER OF ECONOMICS

MICROECONOMICS MACROECONOMICS

Questions1Who has coined the words micro and macro economics

Answer Ranger Frisch coined the words lsquomicrorsquo and lsquomacrorsquo in 1933 to denote the two branches of economic theory namely microeconomics and macroeconomics

2What is microeconomicsAnswer It is the study of behaviour of individual decision ndash making unit such as consumers firms etc

3 What is macroeconomicsAnswer Macroeonomics is the study of overall economic phenomena like employment national income etc

4 What is the importance of microeconomicsAnswer

Microeconomics helps in formulating economic policies which enhance productive efficiency and results in greater social welfare

It helps the government in formulating correct price policies

It explains the working of a capitalistic economy where individual units(producers and consumers ) are free to take their own decision

Micro means a small part in

microeconomics we do not study the whole economy Hence we study an individual consumer and his or her choices and a producer and his or her profit maximizing decisions in the market Thus it does not mirror what happens in the economy as a whole

Macroeconomics on the other hand studies the economy as a whole It is concerned with aggregate and depicts the entire picture of the economyMacroeconomics deals with the national income aggregate investment aggregate consumption etc

Features of Microeconomics It deals with small

parts of the country Hence it looks at

individual consumers firms and industries

It deals with individual income consumption and savings

It studies the determination of price of any product or factors of production

It deals with the working of market via the price mechanism which is nothing but the determination of price and quantity of a commodity by the forces of demand and supply

Features of Macroeconomics

It deals with the study of the economy as a whole

It is concerned with

5 Give a limitation of microeconomics Microeconomics fails to explain the

functioning of an economy as a whole It cannot explain unemployment illiteracy and other problems prevailing in the country

6 What is the importance of macroeconomics It gives overall view of the growing

complexities of an economic system It provides the basic and logical

framework for formulating appropriate macroeconomic policies (eg for inflation poverty etc )to direct and regulate economy towards desirable goals

7What is the limitation of macroeconomics It ignores structural changes in an

individual unit of the aggregate

8 Differentiate between Microeconomics and Macroeconomics

Microeconomics Macroeconomics

the study of aggregates

National income aggregate savings and aggregate investments are major concepts dealt within macroeconomics style

It studies the determination of general price levels

It investigates into the problem of unemployment and the achievement of employment

It studies the aspect of decision making at the aggregate and national levels

It includes all growth theories whether related to developed or developing economies it also includes the study of economic systems and the working of the economy under different systems

Note Both Micro and macro economics are complementary and should be fully utilized for proper understanding of an economy

1It studies economic aspect of an individual unit2It deals with individual incomeConsumption and savings

3 It facilitates determination of price of any product or factors of production

4 Itrsquos scope is narrow and restricted to individual unit

1It studies the economy as a whole

2It deals with the national income aggregate consumption and aggregate savings3 It facilitates determination of general price level in an economy

4 Itrsquos scope is wide as it deals with economic units on the national level

ACCOUNTS

Introduction to Accounting and Book-keeping

Today I am going to share you the meaning of Accounting and Book-keeping and its related terms bullAccounting bullBook Keeping bullAccountsbullTypes Of Accounts bullAccounting Cycle

bull Meaning of accounting

Ans ) Accounting is the art and science of recording classifying and summarising monetary transactions

bull Meaning of Book-keeping

Ans) Bookkeeping is the art of recording business transactions with the view of having a permanent record of them and showing their effect on wealth

bull Meaning of account

Ans) The term account means a record of

business transactions concern a particular person of firm asset or income or expense It is a summarised record of all transactions which take place in an accounting year

bull Types of accountsPersonal accounts ndash Personal accounts relating

to person and Organisation are known as personal accounts Example Ramrsquos Account ABC amp Co Account etc

Real account - The accounts related to tangible and intangible assets are called real accounts Example Cash Account Furniture Account etc

Nominal account- Accounts related to expenses losses incomes and gains are known as nominal accounts Example Wages Account Salary Account Discount Account etc

bull Accounting cycle Accounting cycle refers to a complete sequence of accounting activities It begins with recording of transactions and ends with the preparation of a balance sheet

Chemistry TopicAtomic Structure

Thomsonrsquos atomic modelThomson (1898) was the first to propose the model of an atomHe proposed that an atom can be regarded as a uniform sphere of positive electricity in which requisite number of electrons are embedded evently to neutralize the positive chargeThis is just like plums embedded in a pudding or seeds evently distributed in red spongy mass of a watermelonThis model of atom is known as ldquoPlum-Pudding modelrdquo or

Q1)What is the fundamental constituents of atomAns Electron Proton and neutrons are the fundamental constituents of atomQ2)What is the value of fundamental unit of electricityAnsThe charge carried by one electron is sad to be the fundamental unit of electricityIts magnitude is 48times10-10esuOr 1602times10-19C Q3)Name the element containing no neutronAnsOrdinary hydrogen atom or protium 1H1

Types of AccountPersonal AccountReal AccountNominal AccountBalance Sheet (opening)

ldquowatermelon modelrdquoThis model could explain the electrical neutrality of an atom but failed to explain the result of scattering experiment carried out by Rutherford in 1911So it was rejected ultimately

Q4)Why is an electron called universal particleAns Itrsquos mass and Charge are independent of its source

EVS Chapter 1 ndash Modes of Existence

Modes of existence When one speaks normally about the mode of existence of some group or individual one refers to their customs their mode of being their ethology their habitat in some way their feeling for a placeDifferent modes of exixtence are ndash

1 Hunting ndashGathering2 Pastoral3 Agricultural4 Industrial

1 Hunting and gathering Hunting and gathering mode of existence is characterized by obtaining food from hunting wild animals including fishing and gathering wild plants From their earliest days the hunter-gatherer diet included various grasses tubers fruits seeds and nuts Lacking the means to kill larger animals they procured meat from smaller game or through scavenging

Societies that rely primarily or exclusively on hunting wild animals fishing and gathering wild fruits berries nuts and vegetables to support their diet are called hunting and gathering societies

At least this used to be practice of human beings before agriculture is invented As their brains evolved hominids developed more intricate knowledge of edible plant life and growth cycles

Q) Write the features of Hunting ndash gathering societiesAns - There are five basic characteristics of hunting and gathering societies

i The primary institution is the family which decides how food is to be shared and how children are to be socialized and which provides for the protection of its members

ii They tend to be small with fewer than fifty members

iii They tend to be nomadic moving to new areas when the current food supply in a given area has been exhausted

iv Members display a high level of interdependence

v Labor division is based on sex men hunt and women gather

Political Science

Introduction to political science

Political science occasionally called politology is a social science which deals with systems of governance and the analysis of political activities political thoughts associated constitutions and political behaviorThe study of political science involves the study of both the

Answer the following questions-1 What is political science

Political science occasionally called politology is a social science which deals with systems of governance and the analysis of political activities political thoughts associated constitutions and political behavior

2 Short notes-

traditional and modern theories of politicsTraditionalClassical political sciencepolitical theory-Traditional political science is the study of politics before Second World War The methodology to study Politics was traditional (legal formaletc) the definition of politics traditional (Politics begins and end with state)area of study (constitution state machinery)was traditionalModern Political scienceModern political theory-Modern Political Theory critically examines the contemporary state of political theory making an assessment of the achievement and limitations of the Behavioural Revolution in its totality and reviews objectively the major paradigms and conceptual frameworks adopted by the disciplineContemporary attempts at the development of an integrated political theory involving the use of both traditional and modern concepts approaches and theories-Around late 1960s several political scientists realized the importance of both the traditional political theory and modern Political theory They began building an integrated theory of politics involving a systematic mixture of traditional and modern studies of politics It was held that the study of a complex and vast field like politics needs both traditional as well as

Classical political theory Modern Political theory

Homework-Learn

modern concepts and approaches for studying itrsquos all aspects

Subject Eng Literature (The Tempest ndash William Shakespeare) Topic Act I Scene 1 Lines 1 to 32 (Line 32 ndash Gonzalo hellip If he be not born to be hanged our case is miserable) Date 13th April 2020 (3rd Period)

[Students should read the original play and also the paraphrase given in the school prescribed textbook]Summary Questions amp Answers

[SUMMARY OF THE ENTIRE SCENE]

o The play starts with the scene of a severe storm at sea Alonso (King of Naples) Sebastian (Alonsorsquos brother) Ferdinand (Alonsorsquos son) Gonzalo Antonio (the usurping Duke of Milan) are in a ship in the midst of the storm

o The mariners are trying their best to control the vessel from running aground and are totally following the orders of their Master the Boatswain They have scant success

o The mariners become extremely unhappy and annoyed when most of the passengers arrive on the deck thereby hampering their effort to save the ship There is serious confrontation between them and the passengers who are part of the Kingrsquos entourage

o The mariners could not save the ship

SUMMING-UP

(i) Vivid description of the scene which gives a realistic description of terror and confusion of a tropical storm

(ii) Shows Shakespearersquos accuracy of knowledge in describing the naval operations and also matters of seamanship

(iii) The opening scene justifies the title ndash The Tempest

UNANSWERED QUESTIONS

(i) The King always travels with his entire fleet including his soldiers Where

(1) GONZALO Nay good be patient (Line 15-26)BOATSWAIN When the sea is Hence What cares these

roarers for the name of the king To cabin silence Trouble us not

GONZALO Good yet remember whom thou has aboardBOATSWAIN None that I more love than myself You are a

councillor if you can command these elements to silence and work

the peace of the present we will not hand a rope more use your authority If you cannot give thanks you have

lived so long and make yourself ready in your cabin for the mischance of the hour if it so hap [To the Mariners]

Cheerly good hearts [To Gonzalo] Out of our way I say

(a) To whom is the boatswain speaking What does he mean by lsquoNone that I more love than myselfrsquo

The Boatswain is speaking to Gonzalo the honest old councilor of the Duke of MilanBy using the words ndash lsquoNone that I love more than I love myselfrsquo means that for the Boatswain nobody is dearer to him than his own life

(b) What were the conditions that made the boatswain react in this way

The Boatswain reacts in this way because the storm is at sea and Alonso King of Naples Sebastian his brother Ferdinand his son Gonzalo Antonio the usurping Duke of Milan on board are in distress and in panic Thus they have rushed to the deck interrupting the work of the mariners

(c) What hope does Gonzalo take from the attitude of the boatswain

The insolent and authoritative attitude of Boatswain makes Gonzalo feel comforted He tells that there are no signs that the Boatswain will be drowned But his facial appearance and attitude shows that he is destined to die on land by hanging which in effect means that all on board will be saved Otherwise all the persons on board are doomed

(d) How can they lsquomake yourself ready in your cabinrsquo For what were they asked to make ready themselves

In order to make themselves ready in their cabin the

were the other ships

(ii) Why was the ship in that area Where was it coming from or going where

(iii) The ship broke apart What happened to those who were in the ship

passengers on board must prepare for death which they will possibly soon have to meetThey can retire to their cabins and offer prayers to the Almighty to save them from drowning

(e) What does the boatswain say when he is asked to be patient What does he order to the royal party

When the boatswain is asked to be patient and remain calm he says that he will be patient only when the storm will be over and the sea will be calm but as long as the storm blows and there is danger to the ship he cannot think of being patient He orders the royal party to go to the cabin and leave the mariners to their work

(2) GONZALO I have great comfort from this fellow (Line 27-36)

Methinks he hath no drowning mark upon him his complexion is perfect

gallows Stand fast good Fate to his hanging Make the rope of his destiny our cable for our own doth little advantage If he be not born to be hanged our case is miserable

(a) Why does Gonzalo regard the Boatswain in the midst of danger

In the midst of danger Gonzalo regards the boatswain because he feels that the Boatswain is a source of comfort and is bent upon to do his work sincerely which in this case is saving the ship and its passengers from the severest of raging storm

(b) What reasons does Gonzalo give when he says that none in the ship will die of drowning

Gonzalo is almost sure that none in the ship will die by drowning His says that there is no mark on the face of the boatswain that indicates that he will die by drowning On the other hand the lines on his face are strong indications that he will be hanged to death Therefore there shall be no danger of the shiprsquos sinking

(c) Explain the following ldquoStand fast good Fate to his hanging Make the rope of his destiny our cable for our own doth little advantage If he be not born to be hanged our case is miserablerdquo

The stated lines mean that if the will of destiny is to be carried out then the ship will not get wrecked and all the passengers will be saved The safety of the passengers therefore depends upon the will of fate being carried out in the case of the boatswain If however the boatswain is not to die by hanging then the passengers are also very unsafe because in that case the ship is likely to sink

(d) What order does the Boatswain give to the sailors

when he re-enters What does he say about the crying of the fellows inside the cabin

The boatswain orders the sailors to bring the topmast lower and bring the ship close to a stationary position with the help of the main sail He says that the fellows inside the cabin are moaning and crying in their distress louder than his voice and louder even than the roaring of the storm

Class XII (ScienceCommerceHumanities) Subject Topic Summary Execution

Computer Science

PropositionalLogic

Propositional logic is a procedure to provide reasoning through statementProposition A ststement that results in True or False is said to be proposition There are two types of propositionSimple proposition amp compound propositionSimple proposioton A simple proposition is one that is not a part of any other proposition Such sentential form of proposition is symbolized with english letters in short For example Ram is a claver student (TrueFalse)Where do you live (Not in True or False)Grapes are sweet (TrueFalse)It rains today (TrueFalse)Here we can see some statements anwer would be true or false but some staements answer can not give in terms of true or false Thus the sentences which can be answered in true or false are known as simple propositionAssigning propositon to a variableThe general syntax to assign propostion to a variable is as followsVariable = Simple propositonFor example A=Ram is a clever studentB= Grapes are sweetC= it rains todayCompound proposition

helliphellipto be continued in next classhelliphellipMath Relation Relation If A and B are two non-empty sets

then a relation R from A to B is a subset of AxB If R A x B and (a b) R then we say that a sube isinis related to b by the relation R written as aRbeg Let A be the set of students of class XII and B be the set of students of class XI Then some of the examples of relation from A to B arei) (a b) AXB a is brother of bisinii) (a b) AXB age of a is more than age of isinb Types of relation In this section we would like to study different types of relations We know that a relation in a set A is a subset of A times A Thus the empty set φ and A times A are two extreme relations For illustration consider a relation R in the set A = 1 2 3 4 given by R = (a b) a ndash b = 10 This is the empty set as no pair (a b) satisfies the condition a ndash b = 10 Similarly R = (a b) | a ndash b | ge 0 is the whole primeset A times A as all pairs (a b) in A times A satisfy | a ndash

Example 1 Let A be the set of all students of a boys school Show that the relation R in A given by R = (a b) a is sister of b is the empty relation and R = (a b) the primedifference between heights of a and b is less than 3 meters is the universal relationSolution Since the school is boys school no student of the school can be sister of any student of the school Hence R = φ showing that R is the empty relation It is also obvious that the difference between heights of any two students of the school has to be less than 3 meters This shows that R = A times A is primethe universal relation Example 2 Show that the relation R in the set 1 2 3 given by R = (1 1) (2 2) (3 3) (1 2) (2 3) is reflexive

b | ge 0 These two extreme examples lead us to the following definitionsDefinition 1 A relation R in a set A is called empty relation if no element of A isrelated to any element of A ie R = φ A times AsubDefinition 2 A relation R in a set A is called universal relation if each element of A is related to every element of A ie R = A times A Both the empty relation and the universal relation are some times called trivial relation Definition 3 A relation R in a set A is called(i) reflexive if (a a) R for every a Aisin isin(ii) symmetric if (a1 a2) R implies that (aisin 2a1)

R for all aisin 1 a2 Aisin(iii) transitive if (a1 a2) R and (aisin 2 a3) R isinimplies that (a1 a3) R for all aisin 1 a2 a3 AisinDefinition 4 A relation R in a set A is said to be an equivalence relation if R is reflexive symmetric and transitive

but neither symmetric nor transitiveSolution R is reflexive since (1 1) (2 2) and (3 3) lie in R Also R is not symmetric as (1 2) R but (2 1) isin notinR Similarly R is not transitive as (1 2) R and (2 3) R but (1 3) R isin isin notinExample 3 Show that the relation R in the set Z of integers given byR = (a b) 2 divides a ndash b is an equivalence relationSolution R is reflexive as 2 divides (a ndash a) for all a Z isinFurther if (a b) R then 2 divides a isinndash b Therefore 2 divides b ndash a Hence (b a) R which shows that R is isinsymmetric Similarly if (a b) R and (b c) R isin isinthen a ndash b and b ndash c are divisible by 2 Now a ndash c = (a ndash b) + (b ndash c) is even (Why) So (a ndash c) is divisible by 2 This shows that R is transitive Thus R is an equivalence relation in ZExample 4 Let L be the set of all lines in a plane and R be the relation in L defined as R = (L1 L2) L1 is perpendicular to L2 Show that R is symmetric but neither reflexive nor transitiveSolution R is not reflexive as a line L1 can not be perpendicular to itself ie (L1 L1) R notinR is symmetric as (L1 L2) Risin

L1 is perpendicular to L2rArr L2 is perpendicular to L1rArr (L2 L1) RrArr isin

R is not transitive Indeed if L1 is perpendicular to L2 and L2 is perpendicular to L3 then L1 can never be perpendicular to L3 In fact L1 is parallel to L3 ie (L1 L2) R isin(L2 L3) R but (L1 L3) Risin notin

Chemistry Solid state Characteristics if Solids(i)The particles are locked in fixed positions they are unable to change their relative positions and this brings a definite shape and volume of a solid(ii)In a solid the constituent particles are held by strong forces of attractionThe forces of attraction may be bonding or non bonding(iii)The constituent particles in a solid pack together as closely as possibleoccupying most of the available space within the solidThus the empty space in a solid is very smallThis makes a solid highly rigid and nearly incompressibleThis also explains why a solid has high density and exhibits slow diffusionClassification of Solids

Q1)Define Crystalline solids AnsA Solid that has a definite geometrical shape and a sharp melting pointand whose constituent particles (atomsmolecules or ions) are arranged in a long range order of definite pattern extending throughout the solid is called a crystalline solidExNaClQ2)Define Amorphous solids AnsA solid that does not have a definite shape and a sharp melting pointand whose constituent particles (atomsmolecules or ions) are not arranged in a definite pattern is called an amorphoussolid

Crystalline solidsAmorphous solids

ExGlassRubberQ3)Classify Crystalline Solids Crystalline Solids

Physics Coloumbrsquos Law (Summary)

Before Going Into Coloumbrsquos Law We Will First Learn What is Charge Properties of Charge and Always remember that charge is quantized ie a body always have static charge of magnitude equal to some integral multiple of fundamental electronic charge e= 16 x 10- 19 C

Charge is the property of matter that causes it to produce and experience electrical and magnetic effects The study of the electrical charges at rest is called electrostatics When both electrical and magnetic effects are present the interaction between charges is referred to as electromagnetic

There exist two types of charges in nature positive and negative Like charges repel and unlike charges attract each other

The type of charge on an electron is negative The charge of a proton is the same as that of an electron but with a positive sign In an atom the number of electrons and the number of protons are equal The atom is therefore electrically neutral If one or more electrons are added to it it becomes negatively charged and is designated as negative ion However if one or more electrons are removed from an atom it becomes positively charged and is called a positive ion

The excess or deficiency of electrons in a body gives the concept of charge If there is an excess of electrons in a body it is negatively charged And if there is deficiency of electrons the body becomes positively charged Whenever addition or removal of electrons takes places the body acquires a charge

The SI Unit of charge is coulomb (C) In SI units the current is a fundamental quantity having a unit of ampere (A) The unit of charge is defined in terms of the unit of current Thus one coulomb is the charge transferred in one second across the section of a wire carrying a

Ionic SolidsMetallicSolids

Molecular Solids

current of one ampere

As q = It we have1 C = (1 A) (1 s)

The dimensions of charge are [A T]

Properties of Charge

(1) Quantization of Charge Electric charge can have only discrete values rather than any value That is charge is quantized The smallest discrete value of charge that can exist in nature is the charge on an electron given as

e = plusmn 16 x 10- 19 C

This is the charge attained by an electron and a protonA charge q must be an integral multiple of this basic unit That is

Q = plusmn ne where n = 1 2 hellip

Charge on a body can never be (frac12)e (23)e or 57e etcWhen we rub a glass rod with silk some electrons are transferred from the rod to the silk The rod becomes positively charged The silk becomes negatively charged The coulomb is a very large amount of charge A typical charge acquired by a rubbed body is 10 - 8 C

Biology Reproduction in organisms

Welcome to this new session 2020-21Today in this first chapter we mainly discuss about reproduction types needs and life span of some organismsWe also discuss about difference between sexual and asexual reproduction

Q1 What is reproductionReproduction is defined as a biological processin which an organism gives rise to young onessimilar to itselfQ2 What are the needs of reproductionbulli) Reproduction maintain life on earthii) It enables the continuity of the species generation after generationiii) It creates genetic variation among populationsQ3 Define Life span and write some orgnisms life spanbull Life span is the period from birth to

the natural death of an organism- OrganismsLife span1 Butterfly 1 - 2 weeks2 Fruit fly 30 days3Dog 10-13 years4 Rose5-7 years5 Tortoise100-150 years6 Banyan Tree -200 - 250 yearsQ4 Reproduction is of two types in case ofanimals but in case of plants vegetative propagation is also present

Asexual Reproduction Sexual Reproductioni) Always uniparentalii) Gametes are not involvediii) Only mitotic division involvediv) Somatic cells of parents are involvedv) Offsprings are genetically similar to the parents

i) Usually biparentalii) Gametes are involvediii) Meiosis occurs during gametogenesis Mitosis occurs after fertilisationiv) Germ cells of the parents are involvedv) offsprings are genetically different from the parents

COMMERCE BUSINESS ENVIRONMENT

Welcome to the new sessiontoday we are going to start the first chapter of Class XII The name of the chapter is Business Environment

Already many of you have got some idea about the word business environment form the first chapter of business studies in class XI

In todayrsquos world every business enterprise is a part of the society It exists and operates in association with various groups in society such as customers suppliers competitors banks and financial institutions government agencies trade unions media and so on All these groups influence the functioning of business in one way or the other They constitute the environment of businessConcept of Business Environment

The term lsquobusiness environmentrsquo refers to the sum total of all individuals institutions and other forces that lie outside a business enterprise but that may influence its functioning and performance

The main features of business environment Totality of External forces General and Specific forces Interrelatedness Complexity Dynamic Uncertainty

Prepare the following questions from todayrsquos assignment

2 What do you mean by business environment

The term lsquobusiness environmentrsquo means the aggregate of all forces factors and institutions which are external to and beyond the control of an individual business enterprise but they may influence its functioning and performance Business environment is the macro framework within which a business firm a micro unit operates It consists of several interrelated and interacting elements

2 Explain the main features of business environment in brief

Totality of External forces-Business environment is the sum total of all things external to a business environment

General and Specific forces-It

Relativity

The Interrelation between business and its environment

The business enterprise is an open system It continuously interacts with its environment It takes inputs (such as raw materials capital labour energy and so on) from its environment transforms them into goods and services and sends them back to the environment

Fig 1 Business Environment Relationship

includes both the forces general forces are the economic social political legal and technological conditions which indirectly influence all business enterprise Specific forces are the investors customers competitors and suppliers which influence individual enterprise directly

Interrelatedness-Different elements of environment are interrelated for an example growing awareness for health care has increased the demand for health foods

Complexity- Business environment id complex in nature as the elements keep on changing example economic technological and other forces changes in demand for a product and service

Dynamic-Business environment is not static it keeps on changing

Uncertainty- Itrsquos very difficult to predict future events such as technology and fashion which occur fast and frequently

Business Studies

Human Resources Management

Human resource of an organisation are the aggregate of knowledge skills attitudes of people working in it

The management system which deals with human resources is called human resource management

Features of HRMbullComprehensive functionbullPeople-oriented

Question1) What do you mean by human

resource management Answer) Human resource management may be defined as that field of Management which has to do with planning organising and controlling the functions of procuring developing maintaining and utilising the labour force

bullAction oriented bullPervasive function bullContinuous function

2) Explain the features of HRM in brief

Answer)bullHuman Resource Management is concerned with managing people at work bull Human Resource Management is concerned with employees which bring people and organisations together so that the goals of each are met bullHuman resource management considered every employees as an individual and also promote their satisfaction and growth bull Human resource management is inherent in all organisations and at all levelsbullManagement of human resources are ongoing on never ending process which requires a constant alertness and Awareness of human relations

3) ldquoHR function is said to be pervasiverdquowhy

Answer) Human resource management is required in all organisations whether it is private or government organisations armed forces sports organisations etc It permeatsall the functional areas like production marketing finance research etc This from this feature of human resource management it can be said that it is pervasive in nature

Economics Demand Q1DEFINITION OF DEMANDIn economics demand is the quantity of a good that consumers are willing and able to purchase at various prices during a given period of timeQ2DEMAND CURVEIn economics a demand curve is a graph depicting the relationship between the price of a certain commodity and the quantity of that commodity that is demanded at that pricQ3LAW OF DEMANDIn microeconomics the law of demand states that conditional on all else being equal as the price of a good increases quantity demanded decreases conversely as the price of a good decreases quantity demanded increasesQ4ASSUMPTION of LAW OF DEMAND(i)No change in price of related commodities(ii) No change in income of the consumer(iii) No change in taste and preferences customs habit and fashion of the consumer( No expectation regarding future change in priceQ5MARKET DEMAND SCHEDULEIn economics a market demand schedule is a tabulation of the quantity of a good that all consumers in a market will purchase at a

given price At any given price the corresponding value on the demand schedule is the sum of all consumersrsquo quantities demanded at that priceQ6INDIVIDUAL DEMAND SCHEDULEIndividual demand schedule refers to a tabular statement showing various quantities of a commodity that a consumer is willing to buy at various levels of price during a given period of timeQ7 FACTORS AFFECTING INDIVIDUAL DEMAND FOR A COMMODITY

The factors that influence a consumerrsquos decision to purchase a commodity are also known as determinants of demand The following factors affect the individual demand for a commodity1 price of the commodity2 price of related goods3 income of buyer of the commodity4 tastes and preferences of the buyer1 Price of the CommodityYou must have observed that when price of a commodity falls you tend to buy more of it and when its price rises you tend to buy less of it when all other factors remain constant (lsquoother things remaining the samersquo) In other words other things remaining the same there is an inverse relationship between the price of a commodity and its quantity demanded by its buyers This statement is in accordance with law of demand which you will study in the later part of this lesson Price of a commodity and its quantity demanded by its buyers are inversely related only when lsquoother things remain the samersquo So lsquoother things remaining the samersquo is an assumption when we study the effect of changes in the price of a commodity on its quantity demanded2 Price of Related goodsA consumer may demand a particular good But while buying that good heshe also asks the price of its related goods Related goods can be of two types-(i) Substitute goods(ii) Complementary goods While purchasing a good prices of its substitutes and complements do affect its quantity purchased(i) Price of Substitute Goods Substitute goods are those goods which can easily be used in place of one another for satisfaction of a particular want like tea and coffee An increase in price of substitute good leads to an increase in demand for the given commodity and a decrease in price of substitute good leads to a decrease in demand for the given commodity It means demand for a given commodity is directly affected by change in price of substitute goods For example if price of coffee increases the demand for tea will rise as tea will become relatively cheaper in comparison to coffee(ii) Price of Complementary goods Complementary goods are those goods which are used together to satisfy a particular want like car and petrol An increase in the price of complementary goods leads to a decrease in demand for the given commodity and a decrease in the price of complementary goods leads to an increase in demand for the given commodity For example if price of petrol falls then the demand for cars will increase as it will be relatively cheaper to use both the goods together So demand for a given commodity is inversely affected by change in price of complementary goods3 Income of the Buyer of CommodityDemand for a commodity is also affected by income of its buyer However the effect of change in income on demand depends on the nature of the commodity under consideration In case of some goods like full cream milk fine quality of rice (Basmati rice) etc demand for these commodities increases when income of the buyer increases and

demand for these commodities decreases when income of the buyer decreases Such goods whose demand increases with the increase in income of the buyer are called normal goods But there are some goods like coarse rice toned milk etc whose demand decreases when income of buyer increases and their demand increases when income of the buyer decreases Such goods whose demand decreases with the increase in income of the buyer are called inferior goods Suppose a consumer buys 10 Kgs of rice whose price is ` 25 per Kg He cannot afford to buy better quality of rice because the price of such rice is ` 50 per Kg The consumer is spending ` 250 per month on the purchase of rice Now if income of the consumer increases and he can afford ` 350 on purchase of 10 Kg of rice Now he can afford to buy some quantity of rice say 6 Kgs whose price is ` 25 per Kg and may buy 4 Kgs of rice whose price is ` 50 per Kg Thus he will buy 10 Kgs of rice by spending ` 350 per month Therefore we may conclude that demand for normal goods is directly related to the income of the buyer but demand for inferior goods is inversely related to the income of the buyer4 Tastes and Preferences of the BuyerThe demand for a commodity is also affected by the tastes and preferences of the buyers They include change in fashion customs habits etc Those commodities are preferred by the consumers which are in fashion So demand for those commodities rises which are in fashion On the other hand if a commodity goes out of the fashion its demand falls because no consumer will like to buy it(5) Number of Buyers in the Market(Population)Increase in population raises the market demand whereas decrease in population reduces the market demand for a commodity Not only the size of population but its composition like age (ratio of males females children and old people in population) also affects the demand for a commodity It is because of needs of children young old male and female population differs(6) Distribution of Income and WealthIf the distribution of income and wealth is more in favour of the rich demand for the commodities preferred by the rich such as comforts and luxuries is likely to be higher On the other hand if the distribution of income and wealth is more in favour of poor demand for commodities preferred by the poor such as necessities will be more(7) Season and Weather ConditionsThis is generally observed that the demand for woolens increases during winter whereas demand for ice creams and cold drinks increases during summer Similarly market demand for umbrellas rain coats increases during rainy seasonQ8 REASONS FOR OPERATION OF LAW OF DEMAND WHY DEMAND CURVE SLOPES DOWNWARDNow we will try to explain why does a consumer purchase more quantity of a commodity at a lower price and less of it at a higher price or why does the law of demand operate ie why does the demand curve slope downwards from left to right The main reasons for operation of law of demand are1 Law of Diminishing Marginal UtilityAs you have studied earlier law of diminishing marginal utility states that as we consume more and more units of a commodity the utility derived from each successive unit goes on decreasing The consumer will be ready to pay more for those units which provide him more utility and less for those which provide him less utility It implies that he will purchase more only when the price of the commodity falls2 Income Effect

When price of a commodity falls purchasing power or real income of the consumer increases which enables him to purchase more quantity of the commodity with the same money income Let us take an example Suppose you buy 4 ice creams when price of each ice cream is ` 25 If price of ice creams falls to ` 20 then with same money income you can buy 5 ice creams now3 Substitution EffectWhen price of a commodity falls it becomes comparatively cheaper as compared to its substitutes (although price of substitutes has not been changed) This will lead to rise in demand for the given commodity For example if coke and Pepsi both are sold at ` 10 each and price of coke falls Now coke has become relatively cheaper and will be substituted for Pepsi It will lead to rise in demand for coke4 Change in Number of BuyersWhen price of a commodity falls some old buyers may demand more of the commodity at the reduced price and some new buyers may also start buying this commodity who were not in a position to buy it earlier due to higher price This will lead to increase in number of buyers when price of the commodity falls As a result demand for the commodity rises when its price falls5 Diverse Uses of a CommoditySome commodities have diverse uses like milk It can be used for drinking for sweet preparation for ice cream preparation etc If price of milk rises its use may be restricted to important purpose only This will lead to reduction in demand for other less important uses When price of milk falls it can be put to other uses also leading to rise n demand for itQ9 EXCEPTIONS TO THE LAW OF DEMANDYou have studied in law of demand that a buyer is willing to buy more quantity of a commodity at a lower price and less of it at a higher price But in certain circumstances a rise in price may lead to rise in demand These circumstances are called Exceptions to the Law of Demand Some important exceptions are1 Giffen GoodsGiffen goods are special type of inferior goods in which negative income effect is stronger than negative substitution effect Giffen goods do not follow law of demand as their demand rises when their price rises Examples of Giffen goods are jowar and bajra etc2 Status Symbol GoodsSome goods are used by rich people as status symbols eg diamonds gold jewellary etc The higher the price the higher will be the demand for these goods When price of such goods falls these goods are no longer looked at as status symbol goods and tehrefore therir demand falls3 NecessitiesCommodities such as medicines salt wheat etc do not follow law of demandbecause we have to purchase them in minimum required quantity whatever their price may be4 Goods Expected to be ScarceWhen the buyers expect a scarcity of a particular good in near future they start buying more and more of that good even if their prices are rising For example during war famines etc people tend to buy more of some goods even at higher prices due to fear of their scarcity in near future

Political Science

Constitution of India-The

Preamble

The preamble-

Preamble-

The preamble is the most precious part of the constitution We the people of India having solemnly resolved to constitute India into a Sovereign Socialist Secular Democratic Republic and to secure to all its citizensA preamble is an introductory and expressionary statement in a document that explains the documents purpose and underlying philosophy When applied to the opening paragraphs of a statute it may recite historical facts pertinent to the subject of the statuteNature and purpose of the constitution-Purpose of the Constitution dictates permanent framework of the government to form a more perfect union to establish justice and ensure peace of thenationconstitution provide principles how the government can run itself following the rules and laws written in the constitution of each state keeps them balanced

Answer the following questions-

1 What is preambleA preamble is an introductory and expressionary statement in a document that explains the documents purpose and underlying philosophy2 What is the nature and

purpose of the constitutionConstitution dictatespermanent framework of the government to form a more perfect union to establish justice and ensure peace of the nation

Homework-Learn

Accounts Compatibilty mode

1MEANING OF PARTNERSHIPPartnership is a form of business organisation where two or more persons join hands to run a business They share the profits and losses according to the agreement amongst them According to the Indian Partnership Act 1932 ldquoPartnership is relation between persons who have agreed to share profits of a business carried on by all or any one of them acting for allrdquo For example one of your friends has passed class XII from National Institute of Open Schooling (NIOS) and wants to start a business Heshe approaches you to join in this venture Heshe wants you to contribute some money and participate in the business activities Both of you if join hands constitute a partnership2CHARACTERISTICS1048698 Agreement A partnership is formed by an agreement The agreement may be either oral or in writing It defines the relationship between the persons who agree to carry on business It may contain the terms of sharing profit and the capital to be invested by each partner etc The written agreement is known as partnership deed1048698 Number of persons There must be at least two persons to form a partnership

The maximum number of partners in a partnership firm can be 50 according toCompanies Act 20131048698 Business The Partnership is formed to carry on business with a purpose of earning profits The business should be lawful Thus if two or more persons agree to carry on unlawful activities it will not be termed as partnership1048698 Sharing Profits The partners agree to share profits in the agreed ratio In caseof loss all the partners have to bear it in the same agreed profit sharing ratio10486981048698Mutual Agency Every partner is an agent of the other partners Every partner can bind the firm and all other partners by hisher acts Each partner will be responsible and liable for the acts of all other partners10486981048698Unlimited liability The liability of each partner except that of a minor is unlimited Their liability extends to their personal assets also If the assets of the firm are insufficient to pay off its debts the partnersrsquo personal property can be used to satisfy the claim of the creditors of the partnership firm10486981048698Management All the partners have a right to mange the business However they may authorize one or more partners to manage the affairs of the business on their behalf10486981048698Transferability of Share No partner can transfer hisher share to any one including hisher family member without the consent of all other partners3PARTNERSHIP DEEDAgreement forms the basis of partnership The written form of the agreement is which a document of partnership is It contains terms and conditions regarding the conduct of the business It also explains relationship between the partners This document is called partnership deed Every firm can frame its own partnership deed in which the rights duties and liabilities of the partners are stated in detail It helps in settling the disputes arising among the partners during the general conduct of business 4CONTENTS OF PARTNERSHIP DEEDThe partnership deed generally contains the following (i) Name and address of the partnership firm(ii) Nature and objectives of the business(iii) Name and address of each partner(iv) Ratio in which profits is to be shared(v) Capital contribution by each partner(vi) Rate of Interest on capital if allowed(vii) Salary or any other remuneration to partners if allowed(viii) Rate of interest on loans and advances by a partner to the firm(ix) Drawings of partners and interest thereon if any(x) Method of valuation of goodwill and revaluation of assets and liabilities on the reconstitution of the partnership ie on the admission retirement or death of a partner(xi) Settlement of disputes by arbitration(xii) Settlement of accounts at the time of retirement or death of a partner5IN ABSENCE OF PARTNERSHIP DEEDThe partnership deed lays down the terms and conditions of partnership in regard to rights duties and obligations of the partners In the absence of partnership deed there may arise a controversy on certain issues like profit sharing ratio interest on

capital interest on drawings interest on loan and salary of the partners In such cases the provisions of the Indian Partnership Act becomes applicableSome of the Issues are(i) Distribution of Profit Partners are entitled to share profits equally(ii) Interest on Capital Interest on capital is not allowed(iii) Interest on Drawings No interest on drawing of the partners is to be charged(iv) Interest on Partnerrsquos Loan A Partner is allowed interest 6 per annum on the amount of loan given to the firm by himher(v) Salary and Commission to Partner A partner is not entitled to anysalary or commission or any other remuneration for managing the business

History TOPIC-TOWARDS INDEPENDENCE AND PARTITION THE LAST PHASE (1935-1947)

SUB TOPIC-IMPORTANT POLITICAL DEVELOPMENTS ndash GROWTH OF SOCIAL IDEAS

Socialism is a political social and economic philosophyLike in other parts of the world the Russian revolution of 1917 served as a great inspiration for revolutionaries in India who at that time were engaged in the struggle for liberation from British ruleSocialist ideas led to the formation of communist party of IndiaJAWAHARLAL NEHRU Among the early Congress leaders Jawaharlal Nehru was very much impressed and influenced by the Socialist ideas He also learnt about the Economic activities of the Soviet Union after the Bolshevic Revolution 1917 He made full use of them in IndiaThe election of Jawaharlal Nehru and Subhas Chandra Bose showed the Left wing tendency within CongressJawaharlal Nehru demanded economic freedom along with political freedom of the people in order to end the exploitation of masses

Nehrus working committee included three socialists leaders The Lucknow session was a landmark in the evolution of socialist ideas of the congressSUBHAS CHANDRA BOSE ndash Subhas Chandra Bose had socialist leaning Both Jawaharlal Nehru and Subhas Chandra Bose were known as leftist Congress men Later on National Congress divided into Leftist and rightist campCONGRESS SOCIALIST Within the Congress some leaders formed the Congress Socialist partyPattavi Sitaramyya Sardar Patel Rajendra Prasad had hostile attitude towards the Congress Socialist partyJawaharlals attitude was hesitant

1 QUESTION ndash Mention name of two Congress leaders who had socialist leaning

1ANSWER ndash Subhas Chandra Bose and Jawaharlal Nehru2QUESTION- In which session of the congress Jawaharlal elaborated his Socialist ideas2 ANSWER ndash Lucknow and Faizpur Session in December 1935 and 19363QUESTION ndash Why Congress was sharply divided into leftist and rightist camp 3ANSWER ndash Subhas Chandra Bosersquos attempt to seek re election for congress presidentship in 1939sharply divided the National Congress into Leftist and Rightist camp4 QUESTION ndash Who was MN Roy 4 ANSWER ndash Manabendra Roy first formed the Communist Party of India outside the country at Tashkent in 19205QUESTION ndash Who formed the Congress Socialist Party within the Congress5 ANSWER ndash Jaya Prakash Narayan Achyut Patwardhan Acharya Narendra Dev Ram Mohan Lohia Aruna Asaf Ali6QUESTION ndash When was the Congress Socialist Party formed What was its object6 ANSWER ndash 1934The Congress Socialist Party sought to work out socialist programme through the Congress They joined hands with the Congress and wanted to carry

Subhas Chandra Bose being expelled from the congress after the Tripuri rift he formed Forward BlockThere were basic differences between the Congress Socialists and the communistsTRADE UNION ACTIVITIES Maximum working class people lived in Bombay and Calcutta The working and living conditions of those workers were very miserable In this situation Shasipada Banerjee NM Lokhande protested against the oppression of the working class peopleThe first Trade Union Madras Labour Union was formed in 1918 by BP WadiaIndustrial strikes took place in Kanpur Calcutta Madras Jamshedpur and Ahmedabad AITUC was formed in Bombay in 1927 The growth of Trade union among the workers was slow because of the fear of the dismissal of the jobIn the mean time the Moderates as well as Communists left AITUC and formed separate organization

on National struggle with the help of workers and peasant class of the society7 QUESTION ndash What was the name of the party founded by Subhas Chandra Bose7 ANSWER- Forward Block8QUESTION ndash Who was Shasipada Banerjee8 ANSWER ndash Shasipada Banerjee was a radical Brahmo He founded a working menrsquos club to protest against exploitation of the British rulers towards the working class of India9 QUESTION ndash What was the weekly published by NM Lokhande9ANSWER- Dinabandhu10 QUESTION ndash Who founded Bombay Mill-Hands Association and in which year10 ANSWER- NM Lokhande in189011 QUESTION- Who was BP WadiaANSWER- BPWadia was the founder of Madras Labour Union in191812 QUESTION- What was the name of the first labour union of India12 ANSWER- Madras Labour Union13 QUESTION Who founded the Majur Mahajan 13 ANSWER GANDHIJI14 QUESTION What was the full form of AITUC When it was formed14 ANSWER All India Trade Union Congressin 192715QUESTION Who formed the Red Trade Union Congress and in which year15ANSWER The Communists formed the Red Trade Union Congress16 QUESTION What do you mean by Socialism16 ANSWER Socialism describes any political and economic theory that says the community rather than individuals should own and manage property and natural resources

Subject Eng Literature (The Tempest ndash William Shakespeare) Topic Act III Scene 3 Lines 1 to 52 (Line 52 ndash Brother my lord the Duke Stand to and do as we) Date 13th April 2020 (4th Period)

[Students should read the original play and also the paraphrase given in the school prescribed textbook]Summary Questions amp Answers

o Alonso Sebastian Antonio Gonzalo Adrian Francisco and others wandered about the island in search of Ferdinand and gets tired and hungry of the toil and at the same time gives up all hope of finding him

o Antonio and Sebastian are happy that Alonso is out of hope and decide to make another attempt on his life that night when being so tired they will be sleeping soundly

o Suddenly a solemn and strange music is heard in the air and several strange shapes enter bringing in a banquet These strange shapes then dance round it with gestures of salutation and then inviting the King to eat they depart

o Seeing this strange scene all are inclined to believe the tales told by travelers that there truly are ldquounicornsrdquo and ldquothe phoenixrsquo thronerdquo

1 ALONSO What harmony is this My good friends hark (L18-27)

GONZALO Marvellous sweet music

[Enter several strange shapes bringing in a banquet

they dance about it with gentle actions of salutation

and inviting the King and his companions to eat they depart]ALONSO Give us kind keepers heavens What were theseSEBASTIAN A living drollery Now I will believe

That there are unicorns that in Arabia

There is one tree the phoenixrsquo throne one phoenix

At this hour reigning thereANTONIO Ill believe both

And what does else want credit come to me

And Ill be sworn rsquotis true Travellers neer did lie

Though fools at home condemn rsquoem

(a) How did Prospero present an amazing spectacle before Alonso and his companions

Using his magic powers Prospero ordered strange shapes to lay a banquet before Alonso and his companions The shapes brought several dishes with tasty eatables in them They placed the dishes on a table before Alonso and his companions Then the strange shapes began to dance gracefully around the banquet While dancing they made gestures inviting them to eat the food Then suddenly the shapes disappeared(b) Who were the guests at the strange banquet Describe the lsquoliving drolleryrsquo

Alonso Sebastian Antonio Gonzalo Adrian and Francisco were the guests at the strange banquet

The term ldquoliving drolleryrdquo refers to live entertainment show In this context when Alonso the King of Naples Sebastian his brother Antonio the treacherous brother of Prospero Gonzalo the kind and loyal councillor to the King Adrian and Francisco came to the island they were hungry and weary in their spirits They heard a solemn and strange music They were shocked to see several strange shapes bringing in a banquet and these shapes danced about it with gentle action of salutation inviting the King and his companions to eat After this Sebastian described this show as lsquoliving drolleryrsquo(c) What is lsquophoenixrsquo What are lsquoUnicornsrdquo

The term lsquophoenixrsquo refers to a mythical Arabian bird which lived alone and perched on a solitary tree After one hundred years it expired in flames and rose again from its own ashes

lsquoUnicornsrsquo refers to the mythological four-footed beasts having horns in the centre of their foreheads When the horns are ground into powder the powder was believed to be

an aphrodisiac(d) How does Sebastian explain the puppet show OR Why does the speaker now believe in unicorns and phoenix

Sebastian finds several strange shapes bringing in the banquet They invite the king and his party for dinner and soon depart He tells that if such a strange sight can be a reality there is nothing incredible in the world and from the present moment he will believe anything He says that it is a strange dumb show enacted not by puppets but by living beings It is stranger than a travellerrsquos tale Seeing such a thing

before his own eyes he will no longer disbelieve the story about unicorns and phoenix(e) How do the other characters present respond to this living drollery

At the sight of the lsquoliving drolleryrsquo like Sebastian Gonzalo and Antonio too acted strangely Antonio told that he too now believes in unicorns and phoenix and anything else that seems to be incredible He too now believes in travellersrsquo tales Gonzalo told that if he would report those happenings in Naples nobody will believe him He considers that those gentle shapes were gentler in manner in comparison to the living beings Alonso was at first sight suspicious and told them that those strange shapes conveyed their meaning in expressive gestures when they seemed to lack speech by their movements and sounds Francisco was amazed at their mysterious disappearance

2 ALONSO Not I

(Line 43-52)GONZALO Faith sir you need not fear When we

were boysWho would believe that there were mountaineers

Dewlapped like bulls whose throats had hanging at rsquoem

Wallets of flesh Or that there were such men

Whose heads stood in their breasts Which now we find

Each putter-out of five for one will bring us

Good warrant ofALONSO I will stand to and feed

Although my lastmdashno matter since I feel

The best is past Brother my lord the Duke

Stand to and do as we

(a) How does Alonso respond at the spectacle of the shapes which were sent to them at the instruction of Prospero

After seeing the strange sight of appearing and disappearing of the shapes sent by Prospero to arrange a banquet for them Alonso says that his surprise at having seen those creatures is infinite and he is fully justified in feeling so much surprise He thinks that their shapes their gestures and the sounds they made were indeed amazing Although they do not possess the gift of speech yet they were able to convey their

thoughts by means of their gestures only

(b) What does Prospero say about the views expressed by Alonso regarding the shapes What does Francisco think about the shapesAfter hearing Alonsorsquos views about the shapes Prospero says that this manrsquos praise of the spirits is rather hasty He means to say that Alonso has shown great haste in reaching the conclusion about the shapes Francisco is amazed to see that those shapes disappeared in a mysterious way(c) What does Sebastian ask Alonso to doSebastian tells Alonso that the shapes having disappeared should not matter to them because they have left the eatables behind He asks Alonso to enjoy eating as they are extremely hungry but the king does not accept his offer of enjoying the dishes(d) How does Gonzalo try to dispel Alonsorsquos fear of those strange shapes What kind of references does he give to AlonsoGonzalo says that those who have travelled abroad have reported seeing even stranger sights than these shapes that Alonso and his companions have beheld Hence there is no reason to feel afraid of these shapes Gonzalo further adds that in his younger days he had heard strange stories from travelers and Alonso might have heard similar stories For instance it was said that there existed a certain race of

human beings who had huge lumps of flesh hanging at their throats and who therefore resembled bulls Then Gonzalo tells about a race of human beings whose heads were located at their breasts Gonzalo says that such stories were not believed by most people in those days but now-a-days these stories have become common(e) Explain the following lsquoEach putter-out of five for onersquoEnglish travellers often insured their trips with London brokers Those that went on foreign travels those days used to deposit a certain amount with some firm or company in London before their departure If the travelers failed to return the money was forfeited by the company with which it had been deposited But this money was repaid five-fold if the travelers returned safe and sound In this way a traveler stood a great chance of recovering the entire cost of his

travels(f) Give the explanatory meanings of the following expressions in the context of the above extract (i) Dewlapped (ii) Wallets of flesh

(iii) Putter-out(i) Dewlapped having big lumps of flesh at the necks(ii) Wallets of flesh large masses of flesh looking like bags(iii) Putter-out to invest money before commencing the travel

  • General methods of preparation of hydrogen
  • Chapter Dimensional Analysis (Summary)
    • Properties of Charge
Page 33:   · Web viewSubject. Topic. Summary. Execution. Hindi. व्याकरण. शरीरके अंगो के नाम लिखिए. 1) आँख 2) नाक 3

circle It consists of a region bounded by two radii and an arc lying between the radiiThe area of a sector is a fraction of the area of the circle This area is proportional to the central angle In other words the bigger the central angle the larger is the area of the sectorArea of Sector = θ2 times r2 (when θ is in radians)

Area of Sector = θ times π360 times r2 (when θ is in degrees)

COMMERCE

CLASSIFICTION OF HUMAN ACTIVITIES-ECONOMIC AND NON-ECONOMIC

Welcome to the new sessiontoday we are going to start the first chapter of Class XI The name of the chapter that we are going to start is

lsquoClassification of Human Activities ndasheconomic and non-economicrsquo

Now let us start the chapter by considering human beings and the activities they perform throughout the day

Human activities means all those activities that human beings undertake to satisfy their wants

Human wants on the other hand are the desire of human beings for goods (vegetables fruits rice etc) and services (services of doctors teachers lawyers etc) that they require to live

Now these human activities continue throughout life as human wants are unending unlimited and recurring as human beings desire for better living throughout their lives

Now human activities can be classified into two categories

Human activities

Economic activities Non-economic activities

Economic activities are

Questions1 What are human activities

Answer Human activities mean all those activities that human beings undertake to satisfy their wants

Example A man working in an office

A boy playing in the garden

2What are the characteristics of human activitiesAnswer the characteristics of human activities are as follows

Human activities are undertaken by men women and children and these activities involve human efforts

Human activities are undertaken to satisfy human wants which are unlimited

Human activities continue throughout life

Human activities are performed for both earning money and personal satisfaction

3What is economic activitiesGive example

Answer Economic activities are undertaken by human beings with the object of earning money acquiring wealth and thereby satisfying human wantsExample

Selling of goods by a shop keeper to his customer

A clinic run by a doctor Service of a teacher in school or college

undertaken by human beings with the object of earning money and acquiring wealth

These activities result in the production of economic goods and services

Example Human activities(ie working in factories officesshops) which produce direct economic benefits

Non-economic activities are inspired by human sentiments and emotions such as love for the family desire to help the poor and love for the country

Thus these human activities (eg praying playing sleeping) produce no direct economic benefits and they are also not related to earning money and acquiring wealth

4 What are the characteristics of economic activities

Answer The characteristics of economic activities are as follows

Economic motiveEconomic activities are undertaken to earn money and acquire wealth

ProductiveEconomic activities involve productiondistribution and exchange of goods and services to create wealth

Economic growthEconomic activities determine the level of economic development of a country and standard of living of its citizens

Socially desirableEconomic activities are socially desirable for society

Economic resourcesEconomic activities make use of all the economic resources such landlabourcapital etc

5 What do you mean by non-economic activitiesExampleAnswerNon-economic activities are inspired by human sentiments and emotions such as love for the family desire to help the poor and love for the countryThese activities are not undertaken for monetary gain but for onersquos satisfaction and happinessExample

a mother looks after her children

a student donates blood8 Differentiate between Economic activities and Non-economic activities

Economic activities

Non-economic activities

1to earn living and acquiring wealth2Result can be measured in terms of money

3ExampleBusinessprofession and employment

1 to obtain some satisfaction

2Result cannot be measured in terms of money

3ExampleFamily-orientedreligious socialCultural and national

BUSINESS STUDIES

BUSINESS ENVIRONMENT

Welcome to the new sessionToday we are going to start the first chapter and the name of the chapter is Business Environment

In todayrsquos world every business enterprise is a part of the society It exists and operates in association with various groups in society such as customers suppliers competitors banks and financial institutions government agencies trade unions media and so on All these groups influence the functioning of business in one way or the other They constitute the environment of businessConcept of Business Environment

The term lsquobusiness environmentrsquo refers to the sum total of all individuals institutions and other forces that lie outside a business enterprise but that may influence its functioning and performance

The main features of business environment

Totality of External forces General and Specific forces Interrelatedness Complexity Dynamic Uncertainty Relativity

The Interrelation between business and its environment

The business enterprise is an open system It continuously interacts with its environment It takes inputs

Prepare the following questions from todayrsquos assignment

1 What do you mean by business environment

The term lsquobusiness environmentrsquo means the aggregate of all forces factors and institutions which are external to and beyond the control of an individual business enterprise but they may influence its functioning and performance Business environment is the macro framework within which a business firm a micro unit operates It consists of several interrelated and interacting elements

2 Explain the main features of business environment in brief

Totality of External forces-Business environment is the sum total of all things external to a business environment

General and Specific forces-It includes both the forces general forces are the economic social political legal and technological conditions which indirectly influence all business enterprise Specific forces are the investors customers competitors and suppliers which influence individual enterprise directly

Interrelatedness-Different elements of environment are interrelated for an example growing awareness for health care has increased the demand for health foods

Complexity- Business environment id

(such as raw materials capital labour energy and so on) from its environment transforms them into goods and services and sends them back to the environment

Fig 1 Business Environment Relationship

complex in nature as the elements keep on changing example economic technological and other forces changes in demand for a product and service

Dynamic-Business environment is not static it keeps on changing

Uncertainty- Itrsquos very difficult to predict future events such as technology and fashion which occur fast and frequently

Economics Basic Economic ConceptsSub topic

Microeconomics and

Macroeconomics

Welcome to the new sessiontoday we are going to start the first chapter of Class XI The name of the chapter that we are going to start is Basic Economic concepts

Now Economics covers the study of human activities Human activities are those activities which are performed by humans to satisfy their wants

Thus Human wants are unlimited and therefore economic activities such as production exchange and consumption are needed in order to satisfy those wants

The study of economics is divided largely in two parts which areMicroeconomics and Macroeconomics

SUBJECT- MATTER OF ECONOMICS

MICROECONOMICS MACROECONOMICS

Questions1Who has coined the words micro and macro economics

Answer Ranger Frisch coined the words lsquomicrorsquo and lsquomacrorsquo in 1933 to denote the two branches of economic theory namely microeconomics and macroeconomics

2What is microeconomicsAnswer It is the study of behaviour of individual decision ndash making unit such as consumers firms etc

3 What is macroeconomicsAnswer Macroeonomics is the study of overall economic phenomena like employment national income etc

4 What is the importance of microeconomicsAnswer

Microeconomics helps in formulating economic policies which enhance productive efficiency and results in greater social welfare

It helps the government in formulating correct price policies

It explains the working of a capitalistic economy where individual units(producers and consumers ) are free to take their own decision

Micro means a small part in

microeconomics we do not study the whole economy Hence we study an individual consumer and his or her choices and a producer and his or her profit maximizing decisions in the market Thus it does not mirror what happens in the economy as a whole

Macroeconomics on the other hand studies the economy as a whole It is concerned with aggregate and depicts the entire picture of the economyMacroeconomics deals with the national income aggregate investment aggregate consumption etc

Features of Microeconomics It deals with small

parts of the country Hence it looks at

individual consumers firms and industries

It deals with individual income consumption and savings

It studies the determination of price of any product or factors of production

It deals with the working of market via the price mechanism which is nothing but the determination of price and quantity of a commodity by the forces of demand and supply

Features of Macroeconomics

It deals with the study of the economy as a whole

It is concerned with

5 Give a limitation of microeconomics Microeconomics fails to explain the

functioning of an economy as a whole It cannot explain unemployment illiteracy and other problems prevailing in the country

6 What is the importance of macroeconomics It gives overall view of the growing

complexities of an economic system It provides the basic and logical

framework for formulating appropriate macroeconomic policies (eg for inflation poverty etc )to direct and regulate economy towards desirable goals

7What is the limitation of macroeconomics It ignores structural changes in an

individual unit of the aggregate

8 Differentiate between Microeconomics and Macroeconomics

Microeconomics Macroeconomics

the study of aggregates

National income aggregate savings and aggregate investments are major concepts dealt within macroeconomics style

It studies the determination of general price levels

It investigates into the problem of unemployment and the achievement of employment

It studies the aspect of decision making at the aggregate and national levels

It includes all growth theories whether related to developed or developing economies it also includes the study of economic systems and the working of the economy under different systems

Note Both Micro and macro economics are complementary and should be fully utilized for proper understanding of an economy

1It studies economic aspect of an individual unit2It deals with individual incomeConsumption and savings

3 It facilitates determination of price of any product or factors of production

4 Itrsquos scope is narrow and restricted to individual unit

1It studies the economy as a whole

2It deals with the national income aggregate consumption and aggregate savings3 It facilitates determination of general price level in an economy

4 Itrsquos scope is wide as it deals with economic units on the national level

ACCOUNTS

Introduction to Accounting and Book-keeping

Today I am going to share you the meaning of Accounting and Book-keeping and its related terms bullAccounting bullBook Keeping bullAccountsbullTypes Of Accounts bullAccounting Cycle

bull Meaning of accounting

Ans ) Accounting is the art and science of recording classifying and summarising monetary transactions

bull Meaning of Book-keeping

Ans) Bookkeeping is the art of recording business transactions with the view of having a permanent record of them and showing their effect on wealth

bull Meaning of account

Ans) The term account means a record of

business transactions concern a particular person of firm asset or income or expense It is a summarised record of all transactions which take place in an accounting year

bull Types of accountsPersonal accounts ndash Personal accounts relating

to person and Organisation are known as personal accounts Example Ramrsquos Account ABC amp Co Account etc

Real account - The accounts related to tangible and intangible assets are called real accounts Example Cash Account Furniture Account etc

Nominal account- Accounts related to expenses losses incomes and gains are known as nominal accounts Example Wages Account Salary Account Discount Account etc

bull Accounting cycle Accounting cycle refers to a complete sequence of accounting activities It begins with recording of transactions and ends with the preparation of a balance sheet

Chemistry TopicAtomic Structure

Thomsonrsquos atomic modelThomson (1898) was the first to propose the model of an atomHe proposed that an atom can be regarded as a uniform sphere of positive electricity in which requisite number of electrons are embedded evently to neutralize the positive chargeThis is just like plums embedded in a pudding or seeds evently distributed in red spongy mass of a watermelonThis model of atom is known as ldquoPlum-Pudding modelrdquo or

Q1)What is the fundamental constituents of atomAns Electron Proton and neutrons are the fundamental constituents of atomQ2)What is the value of fundamental unit of electricityAnsThe charge carried by one electron is sad to be the fundamental unit of electricityIts magnitude is 48times10-10esuOr 1602times10-19C Q3)Name the element containing no neutronAnsOrdinary hydrogen atom or protium 1H1

Types of AccountPersonal AccountReal AccountNominal AccountBalance Sheet (opening)

ldquowatermelon modelrdquoThis model could explain the electrical neutrality of an atom but failed to explain the result of scattering experiment carried out by Rutherford in 1911So it was rejected ultimately

Q4)Why is an electron called universal particleAns Itrsquos mass and Charge are independent of its source

EVS Chapter 1 ndash Modes of Existence

Modes of existence When one speaks normally about the mode of existence of some group or individual one refers to their customs their mode of being their ethology their habitat in some way their feeling for a placeDifferent modes of exixtence are ndash

1 Hunting ndashGathering2 Pastoral3 Agricultural4 Industrial

1 Hunting and gathering Hunting and gathering mode of existence is characterized by obtaining food from hunting wild animals including fishing and gathering wild plants From their earliest days the hunter-gatherer diet included various grasses tubers fruits seeds and nuts Lacking the means to kill larger animals they procured meat from smaller game or through scavenging

Societies that rely primarily or exclusively on hunting wild animals fishing and gathering wild fruits berries nuts and vegetables to support their diet are called hunting and gathering societies

At least this used to be practice of human beings before agriculture is invented As their brains evolved hominids developed more intricate knowledge of edible plant life and growth cycles

Q) Write the features of Hunting ndash gathering societiesAns - There are five basic characteristics of hunting and gathering societies

i The primary institution is the family which decides how food is to be shared and how children are to be socialized and which provides for the protection of its members

ii They tend to be small with fewer than fifty members

iii They tend to be nomadic moving to new areas when the current food supply in a given area has been exhausted

iv Members display a high level of interdependence

v Labor division is based on sex men hunt and women gather

Political Science

Introduction to political science

Political science occasionally called politology is a social science which deals with systems of governance and the analysis of political activities political thoughts associated constitutions and political behaviorThe study of political science involves the study of both the

Answer the following questions-1 What is political science

Political science occasionally called politology is a social science which deals with systems of governance and the analysis of political activities political thoughts associated constitutions and political behavior

2 Short notes-

traditional and modern theories of politicsTraditionalClassical political sciencepolitical theory-Traditional political science is the study of politics before Second World War The methodology to study Politics was traditional (legal formaletc) the definition of politics traditional (Politics begins and end with state)area of study (constitution state machinery)was traditionalModern Political scienceModern political theory-Modern Political Theory critically examines the contemporary state of political theory making an assessment of the achievement and limitations of the Behavioural Revolution in its totality and reviews objectively the major paradigms and conceptual frameworks adopted by the disciplineContemporary attempts at the development of an integrated political theory involving the use of both traditional and modern concepts approaches and theories-Around late 1960s several political scientists realized the importance of both the traditional political theory and modern Political theory They began building an integrated theory of politics involving a systematic mixture of traditional and modern studies of politics It was held that the study of a complex and vast field like politics needs both traditional as well as

Classical political theory Modern Political theory

Homework-Learn

modern concepts and approaches for studying itrsquos all aspects

Subject Eng Literature (The Tempest ndash William Shakespeare) Topic Act I Scene 1 Lines 1 to 32 (Line 32 ndash Gonzalo hellip If he be not born to be hanged our case is miserable) Date 13th April 2020 (3rd Period)

[Students should read the original play and also the paraphrase given in the school prescribed textbook]Summary Questions amp Answers

[SUMMARY OF THE ENTIRE SCENE]

o The play starts with the scene of a severe storm at sea Alonso (King of Naples) Sebastian (Alonsorsquos brother) Ferdinand (Alonsorsquos son) Gonzalo Antonio (the usurping Duke of Milan) are in a ship in the midst of the storm

o The mariners are trying their best to control the vessel from running aground and are totally following the orders of their Master the Boatswain They have scant success

o The mariners become extremely unhappy and annoyed when most of the passengers arrive on the deck thereby hampering their effort to save the ship There is serious confrontation between them and the passengers who are part of the Kingrsquos entourage

o The mariners could not save the ship

SUMMING-UP

(i) Vivid description of the scene which gives a realistic description of terror and confusion of a tropical storm

(ii) Shows Shakespearersquos accuracy of knowledge in describing the naval operations and also matters of seamanship

(iii) The opening scene justifies the title ndash The Tempest

UNANSWERED QUESTIONS

(i) The King always travels with his entire fleet including his soldiers Where

(1) GONZALO Nay good be patient (Line 15-26)BOATSWAIN When the sea is Hence What cares these

roarers for the name of the king To cabin silence Trouble us not

GONZALO Good yet remember whom thou has aboardBOATSWAIN None that I more love than myself You are a

councillor if you can command these elements to silence and work

the peace of the present we will not hand a rope more use your authority If you cannot give thanks you have

lived so long and make yourself ready in your cabin for the mischance of the hour if it so hap [To the Mariners]

Cheerly good hearts [To Gonzalo] Out of our way I say

(a) To whom is the boatswain speaking What does he mean by lsquoNone that I more love than myselfrsquo

The Boatswain is speaking to Gonzalo the honest old councilor of the Duke of MilanBy using the words ndash lsquoNone that I love more than I love myselfrsquo means that for the Boatswain nobody is dearer to him than his own life

(b) What were the conditions that made the boatswain react in this way

The Boatswain reacts in this way because the storm is at sea and Alonso King of Naples Sebastian his brother Ferdinand his son Gonzalo Antonio the usurping Duke of Milan on board are in distress and in panic Thus they have rushed to the deck interrupting the work of the mariners

(c) What hope does Gonzalo take from the attitude of the boatswain

The insolent and authoritative attitude of Boatswain makes Gonzalo feel comforted He tells that there are no signs that the Boatswain will be drowned But his facial appearance and attitude shows that he is destined to die on land by hanging which in effect means that all on board will be saved Otherwise all the persons on board are doomed

(d) How can they lsquomake yourself ready in your cabinrsquo For what were they asked to make ready themselves

In order to make themselves ready in their cabin the

were the other ships

(ii) Why was the ship in that area Where was it coming from or going where

(iii) The ship broke apart What happened to those who were in the ship

passengers on board must prepare for death which they will possibly soon have to meetThey can retire to their cabins and offer prayers to the Almighty to save them from drowning

(e) What does the boatswain say when he is asked to be patient What does he order to the royal party

When the boatswain is asked to be patient and remain calm he says that he will be patient only when the storm will be over and the sea will be calm but as long as the storm blows and there is danger to the ship he cannot think of being patient He orders the royal party to go to the cabin and leave the mariners to their work

(2) GONZALO I have great comfort from this fellow (Line 27-36)

Methinks he hath no drowning mark upon him his complexion is perfect

gallows Stand fast good Fate to his hanging Make the rope of his destiny our cable for our own doth little advantage If he be not born to be hanged our case is miserable

(a) Why does Gonzalo regard the Boatswain in the midst of danger

In the midst of danger Gonzalo regards the boatswain because he feels that the Boatswain is a source of comfort and is bent upon to do his work sincerely which in this case is saving the ship and its passengers from the severest of raging storm

(b) What reasons does Gonzalo give when he says that none in the ship will die of drowning

Gonzalo is almost sure that none in the ship will die by drowning His says that there is no mark on the face of the boatswain that indicates that he will die by drowning On the other hand the lines on his face are strong indications that he will be hanged to death Therefore there shall be no danger of the shiprsquos sinking

(c) Explain the following ldquoStand fast good Fate to his hanging Make the rope of his destiny our cable for our own doth little advantage If he be not born to be hanged our case is miserablerdquo

The stated lines mean that if the will of destiny is to be carried out then the ship will not get wrecked and all the passengers will be saved The safety of the passengers therefore depends upon the will of fate being carried out in the case of the boatswain If however the boatswain is not to die by hanging then the passengers are also very unsafe because in that case the ship is likely to sink

(d) What order does the Boatswain give to the sailors

when he re-enters What does he say about the crying of the fellows inside the cabin

The boatswain orders the sailors to bring the topmast lower and bring the ship close to a stationary position with the help of the main sail He says that the fellows inside the cabin are moaning and crying in their distress louder than his voice and louder even than the roaring of the storm

Class XII (ScienceCommerceHumanities) Subject Topic Summary Execution

Computer Science

PropositionalLogic

Propositional logic is a procedure to provide reasoning through statementProposition A ststement that results in True or False is said to be proposition There are two types of propositionSimple proposition amp compound propositionSimple proposioton A simple proposition is one that is not a part of any other proposition Such sentential form of proposition is symbolized with english letters in short For example Ram is a claver student (TrueFalse)Where do you live (Not in True or False)Grapes are sweet (TrueFalse)It rains today (TrueFalse)Here we can see some statements anwer would be true or false but some staements answer can not give in terms of true or false Thus the sentences which can be answered in true or false are known as simple propositionAssigning propositon to a variableThe general syntax to assign propostion to a variable is as followsVariable = Simple propositonFor example A=Ram is a clever studentB= Grapes are sweetC= it rains todayCompound proposition

helliphellipto be continued in next classhelliphellipMath Relation Relation If A and B are two non-empty sets

then a relation R from A to B is a subset of AxB If R A x B and (a b) R then we say that a sube isinis related to b by the relation R written as aRbeg Let A be the set of students of class XII and B be the set of students of class XI Then some of the examples of relation from A to B arei) (a b) AXB a is brother of bisinii) (a b) AXB age of a is more than age of isinb Types of relation In this section we would like to study different types of relations We know that a relation in a set A is a subset of A times A Thus the empty set φ and A times A are two extreme relations For illustration consider a relation R in the set A = 1 2 3 4 given by R = (a b) a ndash b = 10 This is the empty set as no pair (a b) satisfies the condition a ndash b = 10 Similarly R = (a b) | a ndash b | ge 0 is the whole primeset A times A as all pairs (a b) in A times A satisfy | a ndash

Example 1 Let A be the set of all students of a boys school Show that the relation R in A given by R = (a b) a is sister of b is the empty relation and R = (a b) the primedifference between heights of a and b is less than 3 meters is the universal relationSolution Since the school is boys school no student of the school can be sister of any student of the school Hence R = φ showing that R is the empty relation It is also obvious that the difference between heights of any two students of the school has to be less than 3 meters This shows that R = A times A is primethe universal relation Example 2 Show that the relation R in the set 1 2 3 given by R = (1 1) (2 2) (3 3) (1 2) (2 3) is reflexive

b | ge 0 These two extreme examples lead us to the following definitionsDefinition 1 A relation R in a set A is called empty relation if no element of A isrelated to any element of A ie R = φ A times AsubDefinition 2 A relation R in a set A is called universal relation if each element of A is related to every element of A ie R = A times A Both the empty relation and the universal relation are some times called trivial relation Definition 3 A relation R in a set A is called(i) reflexive if (a a) R for every a Aisin isin(ii) symmetric if (a1 a2) R implies that (aisin 2a1)

R for all aisin 1 a2 Aisin(iii) transitive if (a1 a2) R and (aisin 2 a3) R isinimplies that (a1 a3) R for all aisin 1 a2 a3 AisinDefinition 4 A relation R in a set A is said to be an equivalence relation if R is reflexive symmetric and transitive

but neither symmetric nor transitiveSolution R is reflexive since (1 1) (2 2) and (3 3) lie in R Also R is not symmetric as (1 2) R but (2 1) isin notinR Similarly R is not transitive as (1 2) R and (2 3) R but (1 3) R isin isin notinExample 3 Show that the relation R in the set Z of integers given byR = (a b) 2 divides a ndash b is an equivalence relationSolution R is reflexive as 2 divides (a ndash a) for all a Z isinFurther if (a b) R then 2 divides a isinndash b Therefore 2 divides b ndash a Hence (b a) R which shows that R is isinsymmetric Similarly if (a b) R and (b c) R isin isinthen a ndash b and b ndash c are divisible by 2 Now a ndash c = (a ndash b) + (b ndash c) is even (Why) So (a ndash c) is divisible by 2 This shows that R is transitive Thus R is an equivalence relation in ZExample 4 Let L be the set of all lines in a plane and R be the relation in L defined as R = (L1 L2) L1 is perpendicular to L2 Show that R is symmetric but neither reflexive nor transitiveSolution R is not reflexive as a line L1 can not be perpendicular to itself ie (L1 L1) R notinR is symmetric as (L1 L2) Risin

L1 is perpendicular to L2rArr L2 is perpendicular to L1rArr (L2 L1) RrArr isin

R is not transitive Indeed if L1 is perpendicular to L2 and L2 is perpendicular to L3 then L1 can never be perpendicular to L3 In fact L1 is parallel to L3 ie (L1 L2) R isin(L2 L3) R but (L1 L3) Risin notin

Chemistry Solid state Characteristics if Solids(i)The particles are locked in fixed positions they are unable to change their relative positions and this brings a definite shape and volume of a solid(ii)In a solid the constituent particles are held by strong forces of attractionThe forces of attraction may be bonding or non bonding(iii)The constituent particles in a solid pack together as closely as possibleoccupying most of the available space within the solidThus the empty space in a solid is very smallThis makes a solid highly rigid and nearly incompressibleThis also explains why a solid has high density and exhibits slow diffusionClassification of Solids

Q1)Define Crystalline solids AnsA Solid that has a definite geometrical shape and a sharp melting pointand whose constituent particles (atomsmolecules or ions) are arranged in a long range order of definite pattern extending throughout the solid is called a crystalline solidExNaClQ2)Define Amorphous solids AnsA solid that does not have a definite shape and a sharp melting pointand whose constituent particles (atomsmolecules or ions) are not arranged in a definite pattern is called an amorphoussolid

Crystalline solidsAmorphous solids

ExGlassRubberQ3)Classify Crystalline Solids Crystalline Solids

Physics Coloumbrsquos Law (Summary)

Before Going Into Coloumbrsquos Law We Will First Learn What is Charge Properties of Charge and Always remember that charge is quantized ie a body always have static charge of magnitude equal to some integral multiple of fundamental electronic charge e= 16 x 10- 19 C

Charge is the property of matter that causes it to produce and experience electrical and magnetic effects The study of the electrical charges at rest is called electrostatics When both electrical and magnetic effects are present the interaction between charges is referred to as electromagnetic

There exist two types of charges in nature positive and negative Like charges repel and unlike charges attract each other

The type of charge on an electron is negative The charge of a proton is the same as that of an electron but with a positive sign In an atom the number of electrons and the number of protons are equal The atom is therefore electrically neutral If one or more electrons are added to it it becomes negatively charged and is designated as negative ion However if one or more electrons are removed from an atom it becomes positively charged and is called a positive ion

The excess or deficiency of electrons in a body gives the concept of charge If there is an excess of electrons in a body it is negatively charged And if there is deficiency of electrons the body becomes positively charged Whenever addition or removal of electrons takes places the body acquires a charge

The SI Unit of charge is coulomb (C) In SI units the current is a fundamental quantity having a unit of ampere (A) The unit of charge is defined in terms of the unit of current Thus one coulomb is the charge transferred in one second across the section of a wire carrying a

Ionic SolidsMetallicSolids

Molecular Solids

current of one ampere

As q = It we have1 C = (1 A) (1 s)

The dimensions of charge are [A T]

Properties of Charge

(1) Quantization of Charge Electric charge can have only discrete values rather than any value That is charge is quantized The smallest discrete value of charge that can exist in nature is the charge on an electron given as

e = plusmn 16 x 10- 19 C

This is the charge attained by an electron and a protonA charge q must be an integral multiple of this basic unit That is

Q = plusmn ne where n = 1 2 hellip

Charge on a body can never be (frac12)e (23)e or 57e etcWhen we rub a glass rod with silk some electrons are transferred from the rod to the silk The rod becomes positively charged The silk becomes negatively charged The coulomb is a very large amount of charge A typical charge acquired by a rubbed body is 10 - 8 C

Biology Reproduction in organisms

Welcome to this new session 2020-21Today in this first chapter we mainly discuss about reproduction types needs and life span of some organismsWe also discuss about difference between sexual and asexual reproduction

Q1 What is reproductionReproduction is defined as a biological processin which an organism gives rise to young onessimilar to itselfQ2 What are the needs of reproductionbulli) Reproduction maintain life on earthii) It enables the continuity of the species generation after generationiii) It creates genetic variation among populationsQ3 Define Life span and write some orgnisms life spanbull Life span is the period from birth to

the natural death of an organism- OrganismsLife span1 Butterfly 1 - 2 weeks2 Fruit fly 30 days3Dog 10-13 years4 Rose5-7 years5 Tortoise100-150 years6 Banyan Tree -200 - 250 yearsQ4 Reproduction is of two types in case ofanimals but in case of plants vegetative propagation is also present

Asexual Reproduction Sexual Reproductioni) Always uniparentalii) Gametes are not involvediii) Only mitotic division involvediv) Somatic cells of parents are involvedv) Offsprings are genetically similar to the parents

i) Usually biparentalii) Gametes are involvediii) Meiosis occurs during gametogenesis Mitosis occurs after fertilisationiv) Germ cells of the parents are involvedv) offsprings are genetically different from the parents

COMMERCE BUSINESS ENVIRONMENT

Welcome to the new sessiontoday we are going to start the first chapter of Class XII The name of the chapter is Business Environment

Already many of you have got some idea about the word business environment form the first chapter of business studies in class XI

In todayrsquos world every business enterprise is a part of the society It exists and operates in association with various groups in society such as customers suppliers competitors banks and financial institutions government agencies trade unions media and so on All these groups influence the functioning of business in one way or the other They constitute the environment of businessConcept of Business Environment

The term lsquobusiness environmentrsquo refers to the sum total of all individuals institutions and other forces that lie outside a business enterprise but that may influence its functioning and performance

The main features of business environment Totality of External forces General and Specific forces Interrelatedness Complexity Dynamic Uncertainty

Prepare the following questions from todayrsquos assignment

2 What do you mean by business environment

The term lsquobusiness environmentrsquo means the aggregate of all forces factors and institutions which are external to and beyond the control of an individual business enterprise but they may influence its functioning and performance Business environment is the macro framework within which a business firm a micro unit operates It consists of several interrelated and interacting elements

2 Explain the main features of business environment in brief

Totality of External forces-Business environment is the sum total of all things external to a business environment

General and Specific forces-It

Relativity

The Interrelation between business and its environment

The business enterprise is an open system It continuously interacts with its environment It takes inputs (such as raw materials capital labour energy and so on) from its environment transforms them into goods and services and sends them back to the environment

Fig 1 Business Environment Relationship

includes both the forces general forces are the economic social political legal and technological conditions which indirectly influence all business enterprise Specific forces are the investors customers competitors and suppliers which influence individual enterprise directly

Interrelatedness-Different elements of environment are interrelated for an example growing awareness for health care has increased the demand for health foods

Complexity- Business environment id complex in nature as the elements keep on changing example economic technological and other forces changes in demand for a product and service

Dynamic-Business environment is not static it keeps on changing

Uncertainty- Itrsquos very difficult to predict future events such as technology and fashion which occur fast and frequently

Business Studies

Human Resources Management

Human resource of an organisation are the aggregate of knowledge skills attitudes of people working in it

The management system which deals with human resources is called human resource management

Features of HRMbullComprehensive functionbullPeople-oriented

Question1) What do you mean by human

resource management Answer) Human resource management may be defined as that field of Management which has to do with planning organising and controlling the functions of procuring developing maintaining and utilising the labour force

bullAction oriented bullPervasive function bullContinuous function

2) Explain the features of HRM in brief

Answer)bullHuman Resource Management is concerned with managing people at work bull Human Resource Management is concerned with employees which bring people and organisations together so that the goals of each are met bullHuman resource management considered every employees as an individual and also promote their satisfaction and growth bull Human resource management is inherent in all organisations and at all levelsbullManagement of human resources are ongoing on never ending process which requires a constant alertness and Awareness of human relations

3) ldquoHR function is said to be pervasiverdquowhy

Answer) Human resource management is required in all organisations whether it is private or government organisations armed forces sports organisations etc It permeatsall the functional areas like production marketing finance research etc This from this feature of human resource management it can be said that it is pervasive in nature

Economics Demand Q1DEFINITION OF DEMANDIn economics demand is the quantity of a good that consumers are willing and able to purchase at various prices during a given period of timeQ2DEMAND CURVEIn economics a demand curve is a graph depicting the relationship between the price of a certain commodity and the quantity of that commodity that is demanded at that pricQ3LAW OF DEMANDIn microeconomics the law of demand states that conditional on all else being equal as the price of a good increases quantity demanded decreases conversely as the price of a good decreases quantity demanded increasesQ4ASSUMPTION of LAW OF DEMAND(i)No change in price of related commodities(ii) No change in income of the consumer(iii) No change in taste and preferences customs habit and fashion of the consumer( No expectation regarding future change in priceQ5MARKET DEMAND SCHEDULEIn economics a market demand schedule is a tabulation of the quantity of a good that all consumers in a market will purchase at a

given price At any given price the corresponding value on the demand schedule is the sum of all consumersrsquo quantities demanded at that priceQ6INDIVIDUAL DEMAND SCHEDULEIndividual demand schedule refers to a tabular statement showing various quantities of a commodity that a consumer is willing to buy at various levels of price during a given period of timeQ7 FACTORS AFFECTING INDIVIDUAL DEMAND FOR A COMMODITY

The factors that influence a consumerrsquos decision to purchase a commodity are also known as determinants of demand The following factors affect the individual demand for a commodity1 price of the commodity2 price of related goods3 income of buyer of the commodity4 tastes and preferences of the buyer1 Price of the CommodityYou must have observed that when price of a commodity falls you tend to buy more of it and when its price rises you tend to buy less of it when all other factors remain constant (lsquoother things remaining the samersquo) In other words other things remaining the same there is an inverse relationship between the price of a commodity and its quantity demanded by its buyers This statement is in accordance with law of demand which you will study in the later part of this lesson Price of a commodity and its quantity demanded by its buyers are inversely related only when lsquoother things remain the samersquo So lsquoother things remaining the samersquo is an assumption when we study the effect of changes in the price of a commodity on its quantity demanded2 Price of Related goodsA consumer may demand a particular good But while buying that good heshe also asks the price of its related goods Related goods can be of two types-(i) Substitute goods(ii) Complementary goods While purchasing a good prices of its substitutes and complements do affect its quantity purchased(i) Price of Substitute Goods Substitute goods are those goods which can easily be used in place of one another for satisfaction of a particular want like tea and coffee An increase in price of substitute good leads to an increase in demand for the given commodity and a decrease in price of substitute good leads to a decrease in demand for the given commodity It means demand for a given commodity is directly affected by change in price of substitute goods For example if price of coffee increases the demand for tea will rise as tea will become relatively cheaper in comparison to coffee(ii) Price of Complementary goods Complementary goods are those goods which are used together to satisfy a particular want like car and petrol An increase in the price of complementary goods leads to a decrease in demand for the given commodity and a decrease in the price of complementary goods leads to an increase in demand for the given commodity For example if price of petrol falls then the demand for cars will increase as it will be relatively cheaper to use both the goods together So demand for a given commodity is inversely affected by change in price of complementary goods3 Income of the Buyer of CommodityDemand for a commodity is also affected by income of its buyer However the effect of change in income on demand depends on the nature of the commodity under consideration In case of some goods like full cream milk fine quality of rice (Basmati rice) etc demand for these commodities increases when income of the buyer increases and

demand for these commodities decreases when income of the buyer decreases Such goods whose demand increases with the increase in income of the buyer are called normal goods But there are some goods like coarse rice toned milk etc whose demand decreases when income of buyer increases and their demand increases when income of the buyer decreases Such goods whose demand decreases with the increase in income of the buyer are called inferior goods Suppose a consumer buys 10 Kgs of rice whose price is ` 25 per Kg He cannot afford to buy better quality of rice because the price of such rice is ` 50 per Kg The consumer is spending ` 250 per month on the purchase of rice Now if income of the consumer increases and he can afford ` 350 on purchase of 10 Kg of rice Now he can afford to buy some quantity of rice say 6 Kgs whose price is ` 25 per Kg and may buy 4 Kgs of rice whose price is ` 50 per Kg Thus he will buy 10 Kgs of rice by spending ` 350 per month Therefore we may conclude that demand for normal goods is directly related to the income of the buyer but demand for inferior goods is inversely related to the income of the buyer4 Tastes and Preferences of the BuyerThe demand for a commodity is also affected by the tastes and preferences of the buyers They include change in fashion customs habits etc Those commodities are preferred by the consumers which are in fashion So demand for those commodities rises which are in fashion On the other hand if a commodity goes out of the fashion its demand falls because no consumer will like to buy it(5) Number of Buyers in the Market(Population)Increase in population raises the market demand whereas decrease in population reduces the market demand for a commodity Not only the size of population but its composition like age (ratio of males females children and old people in population) also affects the demand for a commodity It is because of needs of children young old male and female population differs(6) Distribution of Income and WealthIf the distribution of income and wealth is more in favour of the rich demand for the commodities preferred by the rich such as comforts and luxuries is likely to be higher On the other hand if the distribution of income and wealth is more in favour of poor demand for commodities preferred by the poor such as necessities will be more(7) Season and Weather ConditionsThis is generally observed that the demand for woolens increases during winter whereas demand for ice creams and cold drinks increases during summer Similarly market demand for umbrellas rain coats increases during rainy seasonQ8 REASONS FOR OPERATION OF LAW OF DEMAND WHY DEMAND CURVE SLOPES DOWNWARDNow we will try to explain why does a consumer purchase more quantity of a commodity at a lower price and less of it at a higher price or why does the law of demand operate ie why does the demand curve slope downwards from left to right The main reasons for operation of law of demand are1 Law of Diminishing Marginal UtilityAs you have studied earlier law of diminishing marginal utility states that as we consume more and more units of a commodity the utility derived from each successive unit goes on decreasing The consumer will be ready to pay more for those units which provide him more utility and less for those which provide him less utility It implies that he will purchase more only when the price of the commodity falls2 Income Effect

When price of a commodity falls purchasing power or real income of the consumer increases which enables him to purchase more quantity of the commodity with the same money income Let us take an example Suppose you buy 4 ice creams when price of each ice cream is ` 25 If price of ice creams falls to ` 20 then with same money income you can buy 5 ice creams now3 Substitution EffectWhen price of a commodity falls it becomes comparatively cheaper as compared to its substitutes (although price of substitutes has not been changed) This will lead to rise in demand for the given commodity For example if coke and Pepsi both are sold at ` 10 each and price of coke falls Now coke has become relatively cheaper and will be substituted for Pepsi It will lead to rise in demand for coke4 Change in Number of BuyersWhen price of a commodity falls some old buyers may demand more of the commodity at the reduced price and some new buyers may also start buying this commodity who were not in a position to buy it earlier due to higher price This will lead to increase in number of buyers when price of the commodity falls As a result demand for the commodity rises when its price falls5 Diverse Uses of a CommoditySome commodities have diverse uses like milk It can be used for drinking for sweet preparation for ice cream preparation etc If price of milk rises its use may be restricted to important purpose only This will lead to reduction in demand for other less important uses When price of milk falls it can be put to other uses also leading to rise n demand for itQ9 EXCEPTIONS TO THE LAW OF DEMANDYou have studied in law of demand that a buyer is willing to buy more quantity of a commodity at a lower price and less of it at a higher price But in certain circumstances a rise in price may lead to rise in demand These circumstances are called Exceptions to the Law of Demand Some important exceptions are1 Giffen GoodsGiffen goods are special type of inferior goods in which negative income effect is stronger than negative substitution effect Giffen goods do not follow law of demand as their demand rises when their price rises Examples of Giffen goods are jowar and bajra etc2 Status Symbol GoodsSome goods are used by rich people as status symbols eg diamonds gold jewellary etc The higher the price the higher will be the demand for these goods When price of such goods falls these goods are no longer looked at as status symbol goods and tehrefore therir demand falls3 NecessitiesCommodities such as medicines salt wheat etc do not follow law of demandbecause we have to purchase them in minimum required quantity whatever their price may be4 Goods Expected to be ScarceWhen the buyers expect a scarcity of a particular good in near future they start buying more and more of that good even if their prices are rising For example during war famines etc people tend to buy more of some goods even at higher prices due to fear of their scarcity in near future

Political Science

Constitution of India-The

Preamble

The preamble-

Preamble-

The preamble is the most precious part of the constitution We the people of India having solemnly resolved to constitute India into a Sovereign Socialist Secular Democratic Republic and to secure to all its citizensA preamble is an introductory and expressionary statement in a document that explains the documents purpose and underlying philosophy When applied to the opening paragraphs of a statute it may recite historical facts pertinent to the subject of the statuteNature and purpose of the constitution-Purpose of the Constitution dictates permanent framework of the government to form a more perfect union to establish justice and ensure peace of thenationconstitution provide principles how the government can run itself following the rules and laws written in the constitution of each state keeps them balanced

Answer the following questions-

1 What is preambleA preamble is an introductory and expressionary statement in a document that explains the documents purpose and underlying philosophy2 What is the nature and

purpose of the constitutionConstitution dictatespermanent framework of the government to form a more perfect union to establish justice and ensure peace of the nation

Homework-Learn

Accounts Compatibilty mode

1MEANING OF PARTNERSHIPPartnership is a form of business organisation where two or more persons join hands to run a business They share the profits and losses according to the agreement amongst them According to the Indian Partnership Act 1932 ldquoPartnership is relation between persons who have agreed to share profits of a business carried on by all or any one of them acting for allrdquo For example one of your friends has passed class XII from National Institute of Open Schooling (NIOS) and wants to start a business Heshe approaches you to join in this venture Heshe wants you to contribute some money and participate in the business activities Both of you if join hands constitute a partnership2CHARACTERISTICS1048698 Agreement A partnership is formed by an agreement The agreement may be either oral or in writing It defines the relationship between the persons who agree to carry on business It may contain the terms of sharing profit and the capital to be invested by each partner etc The written agreement is known as partnership deed1048698 Number of persons There must be at least two persons to form a partnership

The maximum number of partners in a partnership firm can be 50 according toCompanies Act 20131048698 Business The Partnership is formed to carry on business with a purpose of earning profits The business should be lawful Thus if two or more persons agree to carry on unlawful activities it will not be termed as partnership1048698 Sharing Profits The partners agree to share profits in the agreed ratio In caseof loss all the partners have to bear it in the same agreed profit sharing ratio10486981048698Mutual Agency Every partner is an agent of the other partners Every partner can bind the firm and all other partners by hisher acts Each partner will be responsible and liable for the acts of all other partners10486981048698Unlimited liability The liability of each partner except that of a minor is unlimited Their liability extends to their personal assets also If the assets of the firm are insufficient to pay off its debts the partnersrsquo personal property can be used to satisfy the claim of the creditors of the partnership firm10486981048698Management All the partners have a right to mange the business However they may authorize one or more partners to manage the affairs of the business on their behalf10486981048698Transferability of Share No partner can transfer hisher share to any one including hisher family member without the consent of all other partners3PARTNERSHIP DEEDAgreement forms the basis of partnership The written form of the agreement is which a document of partnership is It contains terms and conditions regarding the conduct of the business It also explains relationship between the partners This document is called partnership deed Every firm can frame its own partnership deed in which the rights duties and liabilities of the partners are stated in detail It helps in settling the disputes arising among the partners during the general conduct of business 4CONTENTS OF PARTNERSHIP DEEDThe partnership deed generally contains the following (i) Name and address of the partnership firm(ii) Nature and objectives of the business(iii) Name and address of each partner(iv) Ratio in which profits is to be shared(v) Capital contribution by each partner(vi) Rate of Interest on capital if allowed(vii) Salary or any other remuneration to partners if allowed(viii) Rate of interest on loans and advances by a partner to the firm(ix) Drawings of partners and interest thereon if any(x) Method of valuation of goodwill and revaluation of assets and liabilities on the reconstitution of the partnership ie on the admission retirement or death of a partner(xi) Settlement of disputes by arbitration(xii) Settlement of accounts at the time of retirement or death of a partner5IN ABSENCE OF PARTNERSHIP DEEDThe partnership deed lays down the terms and conditions of partnership in regard to rights duties and obligations of the partners In the absence of partnership deed there may arise a controversy on certain issues like profit sharing ratio interest on

capital interest on drawings interest on loan and salary of the partners In such cases the provisions of the Indian Partnership Act becomes applicableSome of the Issues are(i) Distribution of Profit Partners are entitled to share profits equally(ii) Interest on Capital Interest on capital is not allowed(iii) Interest on Drawings No interest on drawing of the partners is to be charged(iv) Interest on Partnerrsquos Loan A Partner is allowed interest 6 per annum on the amount of loan given to the firm by himher(v) Salary and Commission to Partner A partner is not entitled to anysalary or commission or any other remuneration for managing the business

History TOPIC-TOWARDS INDEPENDENCE AND PARTITION THE LAST PHASE (1935-1947)

SUB TOPIC-IMPORTANT POLITICAL DEVELOPMENTS ndash GROWTH OF SOCIAL IDEAS

Socialism is a political social and economic philosophyLike in other parts of the world the Russian revolution of 1917 served as a great inspiration for revolutionaries in India who at that time were engaged in the struggle for liberation from British ruleSocialist ideas led to the formation of communist party of IndiaJAWAHARLAL NEHRU Among the early Congress leaders Jawaharlal Nehru was very much impressed and influenced by the Socialist ideas He also learnt about the Economic activities of the Soviet Union after the Bolshevic Revolution 1917 He made full use of them in IndiaThe election of Jawaharlal Nehru and Subhas Chandra Bose showed the Left wing tendency within CongressJawaharlal Nehru demanded economic freedom along with political freedom of the people in order to end the exploitation of masses

Nehrus working committee included three socialists leaders The Lucknow session was a landmark in the evolution of socialist ideas of the congressSUBHAS CHANDRA BOSE ndash Subhas Chandra Bose had socialist leaning Both Jawaharlal Nehru and Subhas Chandra Bose were known as leftist Congress men Later on National Congress divided into Leftist and rightist campCONGRESS SOCIALIST Within the Congress some leaders formed the Congress Socialist partyPattavi Sitaramyya Sardar Patel Rajendra Prasad had hostile attitude towards the Congress Socialist partyJawaharlals attitude was hesitant

1 QUESTION ndash Mention name of two Congress leaders who had socialist leaning

1ANSWER ndash Subhas Chandra Bose and Jawaharlal Nehru2QUESTION- In which session of the congress Jawaharlal elaborated his Socialist ideas2 ANSWER ndash Lucknow and Faizpur Session in December 1935 and 19363QUESTION ndash Why Congress was sharply divided into leftist and rightist camp 3ANSWER ndash Subhas Chandra Bosersquos attempt to seek re election for congress presidentship in 1939sharply divided the National Congress into Leftist and Rightist camp4 QUESTION ndash Who was MN Roy 4 ANSWER ndash Manabendra Roy first formed the Communist Party of India outside the country at Tashkent in 19205QUESTION ndash Who formed the Congress Socialist Party within the Congress5 ANSWER ndash Jaya Prakash Narayan Achyut Patwardhan Acharya Narendra Dev Ram Mohan Lohia Aruna Asaf Ali6QUESTION ndash When was the Congress Socialist Party formed What was its object6 ANSWER ndash 1934The Congress Socialist Party sought to work out socialist programme through the Congress They joined hands with the Congress and wanted to carry

Subhas Chandra Bose being expelled from the congress after the Tripuri rift he formed Forward BlockThere were basic differences between the Congress Socialists and the communistsTRADE UNION ACTIVITIES Maximum working class people lived in Bombay and Calcutta The working and living conditions of those workers were very miserable In this situation Shasipada Banerjee NM Lokhande protested against the oppression of the working class peopleThe first Trade Union Madras Labour Union was formed in 1918 by BP WadiaIndustrial strikes took place in Kanpur Calcutta Madras Jamshedpur and Ahmedabad AITUC was formed in Bombay in 1927 The growth of Trade union among the workers was slow because of the fear of the dismissal of the jobIn the mean time the Moderates as well as Communists left AITUC and formed separate organization

on National struggle with the help of workers and peasant class of the society7 QUESTION ndash What was the name of the party founded by Subhas Chandra Bose7 ANSWER- Forward Block8QUESTION ndash Who was Shasipada Banerjee8 ANSWER ndash Shasipada Banerjee was a radical Brahmo He founded a working menrsquos club to protest against exploitation of the British rulers towards the working class of India9 QUESTION ndash What was the weekly published by NM Lokhande9ANSWER- Dinabandhu10 QUESTION ndash Who founded Bombay Mill-Hands Association and in which year10 ANSWER- NM Lokhande in189011 QUESTION- Who was BP WadiaANSWER- BPWadia was the founder of Madras Labour Union in191812 QUESTION- What was the name of the first labour union of India12 ANSWER- Madras Labour Union13 QUESTION Who founded the Majur Mahajan 13 ANSWER GANDHIJI14 QUESTION What was the full form of AITUC When it was formed14 ANSWER All India Trade Union Congressin 192715QUESTION Who formed the Red Trade Union Congress and in which year15ANSWER The Communists formed the Red Trade Union Congress16 QUESTION What do you mean by Socialism16 ANSWER Socialism describes any political and economic theory that says the community rather than individuals should own and manage property and natural resources

Subject Eng Literature (The Tempest ndash William Shakespeare) Topic Act III Scene 3 Lines 1 to 52 (Line 52 ndash Brother my lord the Duke Stand to and do as we) Date 13th April 2020 (4th Period)

[Students should read the original play and also the paraphrase given in the school prescribed textbook]Summary Questions amp Answers

o Alonso Sebastian Antonio Gonzalo Adrian Francisco and others wandered about the island in search of Ferdinand and gets tired and hungry of the toil and at the same time gives up all hope of finding him

o Antonio and Sebastian are happy that Alonso is out of hope and decide to make another attempt on his life that night when being so tired they will be sleeping soundly

o Suddenly a solemn and strange music is heard in the air and several strange shapes enter bringing in a banquet These strange shapes then dance round it with gestures of salutation and then inviting the King to eat they depart

o Seeing this strange scene all are inclined to believe the tales told by travelers that there truly are ldquounicornsrdquo and ldquothe phoenixrsquo thronerdquo

1 ALONSO What harmony is this My good friends hark (L18-27)

GONZALO Marvellous sweet music

[Enter several strange shapes bringing in a banquet

they dance about it with gentle actions of salutation

and inviting the King and his companions to eat they depart]ALONSO Give us kind keepers heavens What were theseSEBASTIAN A living drollery Now I will believe

That there are unicorns that in Arabia

There is one tree the phoenixrsquo throne one phoenix

At this hour reigning thereANTONIO Ill believe both

And what does else want credit come to me

And Ill be sworn rsquotis true Travellers neer did lie

Though fools at home condemn rsquoem

(a) How did Prospero present an amazing spectacle before Alonso and his companions

Using his magic powers Prospero ordered strange shapes to lay a banquet before Alonso and his companions The shapes brought several dishes with tasty eatables in them They placed the dishes on a table before Alonso and his companions Then the strange shapes began to dance gracefully around the banquet While dancing they made gestures inviting them to eat the food Then suddenly the shapes disappeared(b) Who were the guests at the strange banquet Describe the lsquoliving drolleryrsquo

Alonso Sebastian Antonio Gonzalo Adrian and Francisco were the guests at the strange banquet

The term ldquoliving drolleryrdquo refers to live entertainment show In this context when Alonso the King of Naples Sebastian his brother Antonio the treacherous brother of Prospero Gonzalo the kind and loyal councillor to the King Adrian and Francisco came to the island they were hungry and weary in their spirits They heard a solemn and strange music They were shocked to see several strange shapes bringing in a banquet and these shapes danced about it with gentle action of salutation inviting the King and his companions to eat After this Sebastian described this show as lsquoliving drolleryrsquo(c) What is lsquophoenixrsquo What are lsquoUnicornsrdquo

The term lsquophoenixrsquo refers to a mythical Arabian bird which lived alone and perched on a solitary tree After one hundred years it expired in flames and rose again from its own ashes

lsquoUnicornsrsquo refers to the mythological four-footed beasts having horns in the centre of their foreheads When the horns are ground into powder the powder was believed to be

an aphrodisiac(d) How does Sebastian explain the puppet show OR Why does the speaker now believe in unicorns and phoenix

Sebastian finds several strange shapes bringing in the banquet They invite the king and his party for dinner and soon depart He tells that if such a strange sight can be a reality there is nothing incredible in the world and from the present moment he will believe anything He says that it is a strange dumb show enacted not by puppets but by living beings It is stranger than a travellerrsquos tale Seeing such a thing

before his own eyes he will no longer disbelieve the story about unicorns and phoenix(e) How do the other characters present respond to this living drollery

At the sight of the lsquoliving drolleryrsquo like Sebastian Gonzalo and Antonio too acted strangely Antonio told that he too now believes in unicorns and phoenix and anything else that seems to be incredible He too now believes in travellersrsquo tales Gonzalo told that if he would report those happenings in Naples nobody will believe him He considers that those gentle shapes were gentler in manner in comparison to the living beings Alonso was at first sight suspicious and told them that those strange shapes conveyed their meaning in expressive gestures when they seemed to lack speech by their movements and sounds Francisco was amazed at their mysterious disappearance

2 ALONSO Not I

(Line 43-52)GONZALO Faith sir you need not fear When we

were boysWho would believe that there were mountaineers

Dewlapped like bulls whose throats had hanging at rsquoem

Wallets of flesh Or that there were such men

Whose heads stood in their breasts Which now we find

Each putter-out of five for one will bring us

Good warrant ofALONSO I will stand to and feed

Although my lastmdashno matter since I feel

The best is past Brother my lord the Duke

Stand to and do as we

(a) How does Alonso respond at the spectacle of the shapes which were sent to them at the instruction of Prospero

After seeing the strange sight of appearing and disappearing of the shapes sent by Prospero to arrange a banquet for them Alonso says that his surprise at having seen those creatures is infinite and he is fully justified in feeling so much surprise He thinks that their shapes their gestures and the sounds they made were indeed amazing Although they do not possess the gift of speech yet they were able to convey their

thoughts by means of their gestures only

(b) What does Prospero say about the views expressed by Alonso regarding the shapes What does Francisco think about the shapesAfter hearing Alonsorsquos views about the shapes Prospero says that this manrsquos praise of the spirits is rather hasty He means to say that Alonso has shown great haste in reaching the conclusion about the shapes Francisco is amazed to see that those shapes disappeared in a mysterious way(c) What does Sebastian ask Alonso to doSebastian tells Alonso that the shapes having disappeared should not matter to them because they have left the eatables behind He asks Alonso to enjoy eating as they are extremely hungry but the king does not accept his offer of enjoying the dishes(d) How does Gonzalo try to dispel Alonsorsquos fear of those strange shapes What kind of references does he give to AlonsoGonzalo says that those who have travelled abroad have reported seeing even stranger sights than these shapes that Alonso and his companions have beheld Hence there is no reason to feel afraid of these shapes Gonzalo further adds that in his younger days he had heard strange stories from travelers and Alonso might have heard similar stories For instance it was said that there existed a certain race of

human beings who had huge lumps of flesh hanging at their throats and who therefore resembled bulls Then Gonzalo tells about a race of human beings whose heads were located at their breasts Gonzalo says that such stories were not believed by most people in those days but now-a-days these stories have become common(e) Explain the following lsquoEach putter-out of five for onersquoEnglish travellers often insured their trips with London brokers Those that went on foreign travels those days used to deposit a certain amount with some firm or company in London before their departure If the travelers failed to return the money was forfeited by the company with which it had been deposited But this money was repaid five-fold if the travelers returned safe and sound In this way a traveler stood a great chance of recovering the entire cost of his

travels(f) Give the explanatory meanings of the following expressions in the context of the above extract (i) Dewlapped (ii) Wallets of flesh

(iii) Putter-out(i) Dewlapped having big lumps of flesh at the necks(ii) Wallets of flesh large masses of flesh looking like bags(iii) Putter-out to invest money before commencing the travel

  • General methods of preparation of hydrogen
  • Chapter Dimensional Analysis (Summary)
    • Properties of Charge
Page 34:   · Web viewSubject. Topic. Summary. Execution. Hindi. व्याकरण. शरीरके अंगो के नाम लिखिए. 1) आँख 2) नाक 3

undertaken by human beings with the object of earning money and acquiring wealth

These activities result in the production of economic goods and services

Example Human activities(ie working in factories officesshops) which produce direct economic benefits

Non-economic activities are inspired by human sentiments and emotions such as love for the family desire to help the poor and love for the country

Thus these human activities (eg praying playing sleeping) produce no direct economic benefits and they are also not related to earning money and acquiring wealth

4 What are the characteristics of economic activities

Answer The characteristics of economic activities are as follows

Economic motiveEconomic activities are undertaken to earn money and acquire wealth

ProductiveEconomic activities involve productiondistribution and exchange of goods and services to create wealth

Economic growthEconomic activities determine the level of economic development of a country and standard of living of its citizens

Socially desirableEconomic activities are socially desirable for society

Economic resourcesEconomic activities make use of all the economic resources such landlabourcapital etc

5 What do you mean by non-economic activitiesExampleAnswerNon-economic activities are inspired by human sentiments and emotions such as love for the family desire to help the poor and love for the countryThese activities are not undertaken for monetary gain but for onersquos satisfaction and happinessExample

a mother looks after her children

a student donates blood8 Differentiate between Economic activities and Non-economic activities

Economic activities

Non-economic activities

1to earn living and acquiring wealth2Result can be measured in terms of money

3ExampleBusinessprofession and employment

1 to obtain some satisfaction

2Result cannot be measured in terms of money

3ExampleFamily-orientedreligious socialCultural and national

BUSINESS STUDIES

BUSINESS ENVIRONMENT

Welcome to the new sessionToday we are going to start the first chapter and the name of the chapter is Business Environment

In todayrsquos world every business enterprise is a part of the society It exists and operates in association with various groups in society such as customers suppliers competitors banks and financial institutions government agencies trade unions media and so on All these groups influence the functioning of business in one way or the other They constitute the environment of businessConcept of Business Environment

The term lsquobusiness environmentrsquo refers to the sum total of all individuals institutions and other forces that lie outside a business enterprise but that may influence its functioning and performance

The main features of business environment

Totality of External forces General and Specific forces Interrelatedness Complexity Dynamic Uncertainty Relativity

The Interrelation between business and its environment

The business enterprise is an open system It continuously interacts with its environment It takes inputs

Prepare the following questions from todayrsquos assignment

1 What do you mean by business environment

The term lsquobusiness environmentrsquo means the aggregate of all forces factors and institutions which are external to and beyond the control of an individual business enterprise but they may influence its functioning and performance Business environment is the macro framework within which a business firm a micro unit operates It consists of several interrelated and interacting elements

2 Explain the main features of business environment in brief

Totality of External forces-Business environment is the sum total of all things external to a business environment

General and Specific forces-It includes both the forces general forces are the economic social political legal and technological conditions which indirectly influence all business enterprise Specific forces are the investors customers competitors and suppliers which influence individual enterprise directly

Interrelatedness-Different elements of environment are interrelated for an example growing awareness for health care has increased the demand for health foods

Complexity- Business environment id

(such as raw materials capital labour energy and so on) from its environment transforms them into goods and services and sends them back to the environment

Fig 1 Business Environment Relationship

complex in nature as the elements keep on changing example economic technological and other forces changes in demand for a product and service

Dynamic-Business environment is not static it keeps on changing

Uncertainty- Itrsquos very difficult to predict future events such as technology and fashion which occur fast and frequently

Economics Basic Economic ConceptsSub topic

Microeconomics and

Macroeconomics

Welcome to the new sessiontoday we are going to start the first chapter of Class XI The name of the chapter that we are going to start is Basic Economic concepts

Now Economics covers the study of human activities Human activities are those activities which are performed by humans to satisfy their wants

Thus Human wants are unlimited and therefore economic activities such as production exchange and consumption are needed in order to satisfy those wants

The study of economics is divided largely in two parts which areMicroeconomics and Macroeconomics

SUBJECT- MATTER OF ECONOMICS

MICROECONOMICS MACROECONOMICS

Questions1Who has coined the words micro and macro economics

Answer Ranger Frisch coined the words lsquomicrorsquo and lsquomacrorsquo in 1933 to denote the two branches of economic theory namely microeconomics and macroeconomics

2What is microeconomicsAnswer It is the study of behaviour of individual decision ndash making unit such as consumers firms etc

3 What is macroeconomicsAnswer Macroeonomics is the study of overall economic phenomena like employment national income etc

4 What is the importance of microeconomicsAnswer

Microeconomics helps in formulating economic policies which enhance productive efficiency and results in greater social welfare

It helps the government in formulating correct price policies

It explains the working of a capitalistic economy where individual units(producers and consumers ) are free to take their own decision

Micro means a small part in

microeconomics we do not study the whole economy Hence we study an individual consumer and his or her choices and a producer and his or her profit maximizing decisions in the market Thus it does not mirror what happens in the economy as a whole

Macroeconomics on the other hand studies the economy as a whole It is concerned with aggregate and depicts the entire picture of the economyMacroeconomics deals with the national income aggregate investment aggregate consumption etc

Features of Microeconomics It deals with small

parts of the country Hence it looks at

individual consumers firms and industries

It deals with individual income consumption and savings

It studies the determination of price of any product or factors of production

It deals with the working of market via the price mechanism which is nothing but the determination of price and quantity of a commodity by the forces of demand and supply

Features of Macroeconomics

It deals with the study of the economy as a whole

It is concerned with

5 Give a limitation of microeconomics Microeconomics fails to explain the

functioning of an economy as a whole It cannot explain unemployment illiteracy and other problems prevailing in the country

6 What is the importance of macroeconomics It gives overall view of the growing

complexities of an economic system It provides the basic and logical

framework for formulating appropriate macroeconomic policies (eg for inflation poverty etc )to direct and regulate economy towards desirable goals

7What is the limitation of macroeconomics It ignores structural changes in an

individual unit of the aggregate

8 Differentiate between Microeconomics and Macroeconomics

Microeconomics Macroeconomics

the study of aggregates

National income aggregate savings and aggregate investments are major concepts dealt within macroeconomics style

It studies the determination of general price levels

It investigates into the problem of unemployment and the achievement of employment

It studies the aspect of decision making at the aggregate and national levels

It includes all growth theories whether related to developed or developing economies it also includes the study of economic systems and the working of the economy under different systems

Note Both Micro and macro economics are complementary and should be fully utilized for proper understanding of an economy

1It studies economic aspect of an individual unit2It deals with individual incomeConsumption and savings

3 It facilitates determination of price of any product or factors of production

4 Itrsquos scope is narrow and restricted to individual unit

1It studies the economy as a whole

2It deals with the national income aggregate consumption and aggregate savings3 It facilitates determination of general price level in an economy

4 Itrsquos scope is wide as it deals with economic units on the national level

ACCOUNTS

Introduction to Accounting and Book-keeping

Today I am going to share you the meaning of Accounting and Book-keeping and its related terms bullAccounting bullBook Keeping bullAccountsbullTypes Of Accounts bullAccounting Cycle

bull Meaning of accounting

Ans ) Accounting is the art and science of recording classifying and summarising monetary transactions

bull Meaning of Book-keeping

Ans) Bookkeeping is the art of recording business transactions with the view of having a permanent record of them and showing their effect on wealth

bull Meaning of account

Ans) The term account means a record of

business transactions concern a particular person of firm asset or income or expense It is a summarised record of all transactions which take place in an accounting year

bull Types of accountsPersonal accounts ndash Personal accounts relating

to person and Organisation are known as personal accounts Example Ramrsquos Account ABC amp Co Account etc

Real account - The accounts related to tangible and intangible assets are called real accounts Example Cash Account Furniture Account etc

Nominal account- Accounts related to expenses losses incomes and gains are known as nominal accounts Example Wages Account Salary Account Discount Account etc

bull Accounting cycle Accounting cycle refers to a complete sequence of accounting activities It begins with recording of transactions and ends with the preparation of a balance sheet

Chemistry TopicAtomic Structure

Thomsonrsquos atomic modelThomson (1898) was the first to propose the model of an atomHe proposed that an atom can be regarded as a uniform sphere of positive electricity in which requisite number of electrons are embedded evently to neutralize the positive chargeThis is just like plums embedded in a pudding or seeds evently distributed in red spongy mass of a watermelonThis model of atom is known as ldquoPlum-Pudding modelrdquo or

Q1)What is the fundamental constituents of atomAns Electron Proton and neutrons are the fundamental constituents of atomQ2)What is the value of fundamental unit of electricityAnsThe charge carried by one electron is sad to be the fundamental unit of electricityIts magnitude is 48times10-10esuOr 1602times10-19C Q3)Name the element containing no neutronAnsOrdinary hydrogen atom or protium 1H1

Types of AccountPersonal AccountReal AccountNominal AccountBalance Sheet (opening)

ldquowatermelon modelrdquoThis model could explain the electrical neutrality of an atom but failed to explain the result of scattering experiment carried out by Rutherford in 1911So it was rejected ultimately

Q4)Why is an electron called universal particleAns Itrsquos mass and Charge are independent of its source

EVS Chapter 1 ndash Modes of Existence

Modes of existence When one speaks normally about the mode of existence of some group or individual one refers to their customs their mode of being their ethology their habitat in some way their feeling for a placeDifferent modes of exixtence are ndash

1 Hunting ndashGathering2 Pastoral3 Agricultural4 Industrial

1 Hunting and gathering Hunting and gathering mode of existence is characterized by obtaining food from hunting wild animals including fishing and gathering wild plants From their earliest days the hunter-gatherer diet included various grasses tubers fruits seeds and nuts Lacking the means to kill larger animals they procured meat from smaller game or through scavenging

Societies that rely primarily or exclusively on hunting wild animals fishing and gathering wild fruits berries nuts and vegetables to support their diet are called hunting and gathering societies

At least this used to be practice of human beings before agriculture is invented As their brains evolved hominids developed more intricate knowledge of edible plant life and growth cycles

Q) Write the features of Hunting ndash gathering societiesAns - There are five basic characteristics of hunting and gathering societies

i The primary institution is the family which decides how food is to be shared and how children are to be socialized and which provides for the protection of its members

ii They tend to be small with fewer than fifty members

iii They tend to be nomadic moving to new areas when the current food supply in a given area has been exhausted

iv Members display a high level of interdependence

v Labor division is based on sex men hunt and women gather

Political Science

Introduction to political science

Political science occasionally called politology is a social science which deals with systems of governance and the analysis of political activities political thoughts associated constitutions and political behaviorThe study of political science involves the study of both the

Answer the following questions-1 What is political science

Political science occasionally called politology is a social science which deals with systems of governance and the analysis of political activities political thoughts associated constitutions and political behavior

2 Short notes-

traditional and modern theories of politicsTraditionalClassical political sciencepolitical theory-Traditional political science is the study of politics before Second World War The methodology to study Politics was traditional (legal formaletc) the definition of politics traditional (Politics begins and end with state)area of study (constitution state machinery)was traditionalModern Political scienceModern political theory-Modern Political Theory critically examines the contemporary state of political theory making an assessment of the achievement and limitations of the Behavioural Revolution in its totality and reviews objectively the major paradigms and conceptual frameworks adopted by the disciplineContemporary attempts at the development of an integrated political theory involving the use of both traditional and modern concepts approaches and theories-Around late 1960s several political scientists realized the importance of both the traditional political theory and modern Political theory They began building an integrated theory of politics involving a systematic mixture of traditional and modern studies of politics It was held that the study of a complex and vast field like politics needs both traditional as well as

Classical political theory Modern Political theory

Homework-Learn

modern concepts and approaches for studying itrsquos all aspects

Subject Eng Literature (The Tempest ndash William Shakespeare) Topic Act I Scene 1 Lines 1 to 32 (Line 32 ndash Gonzalo hellip If he be not born to be hanged our case is miserable) Date 13th April 2020 (3rd Period)

[Students should read the original play and also the paraphrase given in the school prescribed textbook]Summary Questions amp Answers

[SUMMARY OF THE ENTIRE SCENE]

o The play starts with the scene of a severe storm at sea Alonso (King of Naples) Sebastian (Alonsorsquos brother) Ferdinand (Alonsorsquos son) Gonzalo Antonio (the usurping Duke of Milan) are in a ship in the midst of the storm

o The mariners are trying their best to control the vessel from running aground and are totally following the orders of their Master the Boatswain They have scant success

o The mariners become extremely unhappy and annoyed when most of the passengers arrive on the deck thereby hampering their effort to save the ship There is serious confrontation between them and the passengers who are part of the Kingrsquos entourage

o The mariners could not save the ship

SUMMING-UP

(i) Vivid description of the scene which gives a realistic description of terror and confusion of a tropical storm

(ii) Shows Shakespearersquos accuracy of knowledge in describing the naval operations and also matters of seamanship

(iii) The opening scene justifies the title ndash The Tempest

UNANSWERED QUESTIONS

(i) The King always travels with his entire fleet including his soldiers Where

(1) GONZALO Nay good be patient (Line 15-26)BOATSWAIN When the sea is Hence What cares these

roarers for the name of the king To cabin silence Trouble us not

GONZALO Good yet remember whom thou has aboardBOATSWAIN None that I more love than myself You are a

councillor if you can command these elements to silence and work

the peace of the present we will not hand a rope more use your authority If you cannot give thanks you have

lived so long and make yourself ready in your cabin for the mischance of the hour if it so hap [To the Mariners]

Cheerly good hearts [To Gonzalo] Out of our way I say

(a) To whom is the boatswain speaking What does he mean by lsquoNone that I more love than myselfrsquo

The Boatswain is speaking to Gonzalo the honest old councilor of the Duke of MilanBy using the words ndash lsquoNone that I love more than I love myselfrsquo means that for the Boatswain nobody is dearer to him than his own life

(b) What were the conditions that made the boatswain react in this way

The Boatswain reacts in this way because the storm is at sea and Alonso King of Naples Sebastian his brother Ferdinand his son Gonzalo Antonio the usurping Duke of Milan on board are in distress and in panic Thus they have rushed to the deck interrupting the work of the mariners

(c) What hope does Gonzalo take from the attitude of the boatswain

The insolent and authoritative attitude of Boatswain makes Gonzalo feel comforted He tells that there are no signs that the Boatswain will be drowned But his facial appearance and attitude shows that he is destined to die on land by hanging which in effect means that all on board will be saved Otherwise all the persons on board are doomed

(d) How can they lsquomake yourself ready in your cabinrsquo For what were they asked to make ready themselves

In order to make themselves ready in their cabin the

were the other ships

(ii) Why was the ship in that area Where was it coming from or going where

(iii) The ship broke apart What happened to those who were in the ship

passengers on board must prepare for death which they will possibly soon have to meetThey can retire to their cabins and offer prayers to the Almighty to save them from drowning

(e) What does the boatswain say when he is asked to be patient What does he order to the royal party

When the boatswain is asked to be patient and remain calm he says that he will be patient only when the storm will be over and the sea will be calm but as long as the storm blows and there is danger to the ship he cannot think of being patient He orders the royal party to go to the cabin and leave the mariners to their work

(2) GONZALO I have great comfort from this fellow (Line 27-36)

Methinks he hath no drowning mark upon him his complexion is perfect

gallows Stand fast good Fate to his hanging Make the rope of his destiny our cable for our own doth little advantage If he be not born to be hanged our case is miserable

(a) Why does Gonzalo regard the Boatswain in the midst of danger

In the midst of danger Gonzalo regards the boatswain because he feels that the Boatswain is a source of comfort and is bent upon to do his work sincerely which in this case is saving the ship and its passengers from the severest of raging storm

(b) What reasons does Gonzalo give when he says that none in the ship will die of drowning

Gonzalo is almost sure that none in the ship will die by drowning His says that there is no mark on the face of the boatswain that indicates that he will die by drowning On the other hand the lines on his face are strong indications that he will be hanged to death Therefore there shall be no danger of the shiprsquos sinking

(c) Explain the following ldquoStand fast good Fate to his hanging Make the rope of his destiny our cable for our own doth little advantage If he be not born to be hanged our case is miserablerdquo

The stated lines mean that if the will of destiny is to be carried out then the ship will not get wrecked and all the passengers will be saved The safety of the passengers therefore depends upon the will of fate being carried out in the case of the boatswain If however the boatswain is not to die by hanging then the passengers are also very unsafe because in that case the ship is likely to sink

(d) What order does the Boatswain give to the sailors

when he re-enters What does he say about the crying of the fellows inside the cabin

The boatswain orders the sailors to bring the topmast lower and bring the ship close to a stationary position with the help of the main sail He says that the fellows inside the cabin are moaning and crying in their distress louder than his voice and louder even than the roaring of the storm

Class XII (ScienceCommerceHumanities) Subject Topic Summary Execution

Computer Science

PropositionalLogic

Propositional logic is a procedure to provide reasoning through statementProposition A ststement that results in True or False is said to be proposition There are two types of propositionSimple proposition amp compound propositionSimple proposioton A simple proposition is one that is not a part of any other proposition Such sentential form of proposition is symbolized with english letters in short For example Ram is a claver student (TrueFalse)Where do you live (Not in True or False)Grapes are sweet (TrueFalse)It rains today (TrueFalse)Here we can see some statements anwer would be true or false but some staements answer can not give in terms of true or false Thus the sentences which can be answered in true or false are known as simple propositionAssigning propositon to a variableThe general syntax to assign propostion to a variable is as followsVariable = Simple propositonFor example A=Ram is a clever studentB= Grapes are sweetC= it rains todayCompound proposition

helliphellipto be continued in next classhelliphellipMath Relation Relation If A and B are two non-empty sets

then a relation R from A to B is a subset of AxB If R A x B and (a b) R then we say that a sube isinis related to b by the relation R written as aRbeg Let A be the set of students of class XII and B be the set of students of class XI Then some of the examples of relation from A to B arei) (a b) AXB a is brother of bisinii) (a b) AXB age of a is more than age of isinb Types of relation In this section we would like to study different types of relations We know that a relation in a set A is a subset of A times A Thus the empty set φ and A times A are two extreme relations For illustration consider a relation R in the set A = 1 2 3 4 given by R = (a b) a ndash b = 10 This is the empty set as no pair (a b) satisfies the condition a ndash b = 10 Similarly R = (a b) | a ndash b | ge 0 is the whole primeset A times A as all pairs (a b) in A times A satisfy | a ndash

Example 1 Let A be the set of all students of a boys school Show that the relation R in A given by R = (a b) a is sister of b is the empty relation and R = (a b) the primedifference between heights of a and b is less than 3 meters is the universal relationSolution Since the school is boys school no student of the school can be sister of any student of the school Hence R = φ showing that R is the empty relation It is also obvious that the difference between heights of any two students of the school has to be less than 3 meters This shows that R = A times A is primethe universal relation Example 2 Show that the relation R in the set 1 2 3 given by R = (1 1) (2 2) (3 3) (1 2) (2 3) is reflexive

b | ge 0 These two extreme examples lead us to the following definitionsDefinition 1 A relation R in a set A is called empty relation if no element of A isrelated to any element of A ie R = φ A times AsubDefinition 2 A relation R in a set A is called universal relation if each element of A is related to every element of A ie R = A times A Both the empty relation and the universal relation are some times called trivial relation Definition 3 A relation R in a set A is called(i) reflexive if (a a) R for every a Aisin isin(ii) symmetric if (a1 a2) R implies that (aisin 2a1)

R for all aisin 1 a2 Aisin(iii) transitive if (a1 a2) R and (aisin 2 a3) R isinimplies that (a1 a3) R for all aisin 1 a2 a3 AisinDefinition 4 A relation R in a set A is said to be an equivalence relation if R is reflexive symmetric and transitive

but neither symmetric nor transitiveSolution R is reflexive since (1 1) (2 2) and (3 3) lie in R Also R is not symmetric as (1 2) R but (2 1) isin notinR Similarly R is not transitive as (1 2) R and (2 3) R but (1 3) R isin isin notinExample 3 Show that the relation R in the set Z of integers given byR = (a b) 2 divides a ndash b is an equivalence relationSolution R is reflexive as 2 divides (a ndash a) for all a Z isinFurther if (a b) R then 2 divides a isinndash b Therefore 2 divides b ndash a Hence (b a) R which shows that R is isinsymmetric Similarly if (a b) R and (b c) R isin isinthen a ndash b and b ndash c are divisible by 2 Now a ndash c = (a ndash b) + (b ndash c) is even (Why) So (a ndash c) is divisible by 2 This shows that R is transitive Thus R is an equivalence relation in ZExample 4 Let L be the set of all lines in a plane and R be the relation in L defined as R = (L1 L2) L1 is perpendicular to L2 Show that R is symmetric but neither reflexive nor transitiveSolution R is not reflexive as a line L1 can not be perpendicular to itself ie (L1 L1) R notinR is symmetric as (L1 L2) Risin

L1 is perpendicular to L2rArr L2 is perpendicular to L1rArr (L2 L1) RrArr isin

R is not transitive Indeed if L1 is perpendicular to L2 and L2 is perpendicular to L3 then L1 can never be perpendicular to L3 In fact L1 is parallel to L3 ie (L1 L2) R isin(L2 L3) R but (L1 L3) Risin notin

Chemistry Solid state Characteristics if Solids(i)The particles are locked in fixed positions they are unable to change their relative positions and this brings a definite shape and volume of a solid(ii)In a solid the constituent particles are held by strong forces of attractionThe forces of attraction may be bonding or non bonding(iii)The constituent particles in a solid pack together as closely as possibleoccupying most of the available space within the solidThus the empty space in a solid is very smallThis makes a solid highly rigid and nearly incompressibleThis also explains why a solid has high density and exhibits slow diffusionClassification of Solids

Q1)Define Crystalline solids AnsA Solid that has a definite geometrical shape and a sharp melting pointand whose constituent particles (atomsmolecules or ions) are arranged in a long range order of definite pattern extending throughout the solid is called a crystalline solidExNaClQ2)Define Amorphous solids AnsA solid that does not have a definite shape and a sharp melting pointand whose constituent particles (atomsmolecules or ions) are not arranged in a definite pattern is called an amorphoussolid

Crystalline solidsAmorphous solids

ExGlassRubberQ3)Classify Crystalline Solids Crystalline Solids

Physics Coloumbrsquos Law (Summary)

Before Going Into Coloumbrsquos Law We Will First Learn What is Charge Properties of Charge and Always remember that charge is quantized ie a body always have static charge of magnitude equal to some integral multiple of fundamental electronic charge e= 16 x 10- 19 C

Charge is the property of matter that causes it to produce and experience electrical and magnetic effects The study of the electrical charges at rest is called electrostatics When both electrical and magnetic effects are present the interaction between charges is referred to as electromagnetic

There exist two types of charges in nature positive and negative Like charges repel and unlike charges attract each other

The type of charge on an electron is negative The charge of a proton is the same as that of an electron but with a positive sign In an atom the number of electrons and the number of protons are equal The atom is therefore electrically neutral If one or more electrons are added to it it becomes negatively charged and is designated as negative ion However if one or more electrons are removed from an atom it becomes positively charged and is called a positive ion

The excess or deficiency of electrons in a body gives the concept of charge If there is an excess of electrons in a body it is negatively charged And if there is deficiency of electrons the body becomes positively charged Whenever addition or removal of electrons takes places the body acquires a charge

The SI Unit of charge is coulomb (C) In SI units the current is a fundamental quantity having a unit of ampere (A) The unit of charge is defined in terms of the unit of current Thus one coulomb is the charge transferred in one second across the section of a wire carrying a

Ionic SolidsMetallicSolids

Molecular Solids

current of one ampere

As q = It we have1 C = (1 A) (1 s)

The dimensions of charge are [A T]

Properties of Charge

(1) Quantization of Charge Electric charge can have only discrete values rather than any value That is charge is quantized The smallest discrete value of charge that can exist in nature is the charge on an electron given as

e = plusmn 16 x 10- 19 C

This is the charge attained by an electron and a protonA charge q must be an integral multiple of this basic unit That is

Q = plusmn ne where n = 1 2 hellip

Charge on a body can never be (frac12)e (23)e or 57e etcWhen we rub a glass rod with silk some electrons are transferred from the rod to the silk The rod becomes positively charged The silk becomes negatively charged The coulomb is a very large amount of charge A typical charge acquired by a rubbed body is 10 - 8 C

Biology Reproduction in organisms

Welcome to this new session 2020-21Today in this first chapter we mainly discuss about reproduction types needs and life span of some organismsWe also discuss about difference between sexual and asexual reproduction

Q1 What is reproductionReproduction is defined as a biological processin which an organism gives rise to young onessimilar to itselfQ2 What are the needs of reproductionbulli) Reproduction maintain life on earthii) It enables the continuity of the species generation after generationiii) It creates genetic variation among populationsQ3 Define Life span and write some orgnisms life spanbull Life span is the period from birth to

the natural death of an organism- OrganismsLife span1 Butterfly 1 - 2 weeks2 Fruit fly 30 days3Dog 10-13 years4 Rose5-7 years5 Tortoise100-150 years6 Banyan Tree -200 - 250 yearsQ4 Reproduction is of two types in case ofanimals but in case of plants vegetative propagation is also present

Asexual Reproduction Sexual Reproductioni) Always uniparentalii) Gametes are not involvediii) Only mitotic division involvediv) Somatic cells of parents are involvedv) Offsprings are genetically similar to the parents

i) Usually biparentalii) Gametes are involvediii) Meiosis occurs during gametogenesis Mitosis occurs after fertilisationiv) Germ cells of the parents are involvedv) offsprings are genetically different from the parents

COMMERCE BUSINESS ENVIRONMENT

Welcome to the new sessiontoday we are going to start the first chapter of Class XII The name of the chapter is Business Environment

Already many of you have got some idea about the word business environment form the first chapter of business studies in class XI

In todayrsquos world every business enterprise is a part of the society It exists and operates in association with various groups in society such as customers suppliers competitors banks and financial institutions government agencies trade unions media and so on All these groups influence the functioning of business in one way or the other They constitute the environment of businessConcept of Business Environment

The term lsquobusiness environmentrsquo refers to the sum total of all individuals institutions and other forces that lie outside a business enterprise but that may influence its functioning and performance

The main features of business environment Totality of External forces General and Specific forces Interrelatedness Complexity Dynamic Uncertainty

Prepare the following questions from todayrsquos assignment

2 What do you mean by business environment

The term lsquobusiness environmentrsquo means the aggregate of all forces factors and institutions which are external to and beyond the control of an individual business enterprise but they may influence its functioning and performance Business environment is the macro framework within which a business firm a micro unit operates It consists of several interrelated and interacting elements

2 Explain the main features of business environment in brief

Totality of External forces-Business environment is the sum total of all things external to a business environment

General and Specific forces-It

Relativity

The Interrelation between business and its environment

The business enterprise is an open system It continuously interacts with its environment It takes inputs (such as raw materials capital labour energy and so on) from its environment transforms them into goods and services and sends them back to the environment

Fig 1 Business Environment Relationship

includes both the forces general forces are the economic social political legal and technological conditions which indirectly influence all business enterprise Specific forces are the investors customers competitors and suppliers which influence individual enterprise directly

Interrelatedness-Different elements of environment are interrelated for an example growing awareness for health care has increased the demand for health foods

Complexity- Business environment id complex in nature as the elements keep on changing example economic technological and other forces changes in demand for a product and service

Dynamic-Business environment is not static it keeps on changing

Uncertainty- Itrsquos very difficult to predict future events such as technology and fashion which occur fast and frequently

Business Studies

Human Resources Management

Human resource of an organisation are the aggregate of knowledge skills attitudes of people working in it

The management system which deals with human resources is called human resource management

Features of HRMbullComprehensive functionbullPeople-oriented

Question1) What do you mean by human

resource management Answer) Human resource management may be defined as that field of Management which has to do with planning organising and controlling the functions of procuring developing maintaining and utilising the labour force

bullAction oriented bullPervasive function bullContinuous function

2) Explain the features of HRM in brief

Answer)bullHuman Resource Management is concerned with managing people at work bull Human Resource Management is concerned with employees which bring people and organisations together so that the goals of each are met bullHuman resource management considered every employees as an individual and also promote their satisfaction and growth bull Human resource management is inherent in all organisations and at all levelsbullManagement of human resources are ongoing on never ending process which requires a constant alertness and Awareness of human relations

3) ldquoHR function is said to be pervasiverdquowhy

Answer) Human resource management is required in all organisations whether it is private or government organisations armed forces sports organisations etc It permeatsall the functional areas like production marketing finance research etc This from this feature of human resource management it can be said that it is pervasive in nature

Economics Demand Q1DEFINITION OF DEMANDIn economics demand is the quantity of a good that consumers are willing and able to purchase at various prices during a given period of timeQ2DEMAND CURVEIn economics a demand curve is a graph depicting the relationship between the price of a certain commodity and the quantity of that commodity that is demanded at that pricQ3LAW OF DEMANDIn microeconomics the law of demand states that conditional on all else being equal as the price of a good increases quantity demanded decreases conversely as the price of a good decreases quantity demanded increasesQ4ASSUMPTION of LAW OF DEMAND(i)No change in price of related commodities(ii) No change in income of the consumer(iii) No change in taste and preferences customs habit and fashion of the consumer( No expectation regarding future change in priceQ5MARKET DEMAND SCHEDULEIn economics a market demand schedule is a tabulation of the quantity of a good that all consumers in a market will purchase at a

given price At any given price the corresponding value on the demand schedule is the sum of all consumersrsquo quantities demanded at that priceQ6INDIVIDUAL DEMAND SCHEDULEIndividual demand schedule refers to a tabular statement showing various quantities of a commodity that a consumer is willing to buy at various levels of price during a given period of timeQ7 FACTORS AFFECTING INDIVIDUAL DEMAND FOR A COMMODITY

The factors that influence a consumerrsquos decision to purchase a commodity are also known as determinants of demand The following factors affect the individual demand for a commodity1 price of the commodity2 price of related goods3 income of buyer of the commodity4 tastes and preferences of the buyer1 Price of the CommodityYou must have observed that when price of a commodity falls you tend to buy more of it and when its price rises you tend to buy less of it when all other factors remain constant (lsquoother things remaining the samersquo) In other words other things remaining the same there is an inverse relationship between the price of a commodity and its quantity demanded by its buyers This statement is in accordance with law of demand which you will study in the later part of this lesson Price of a commodity and its quantity demanded by its buyers are inversely related only when lsquoother things remain the samersquo So lsquoother things remaining the samersquo is an assumption when we study the effect of changes in the price of a commodity on its quantity demanded2 Price of Related goodsA consumer may demand a particular good But while buying that good heshe also asks the price of its related goods Related goods can be of two types-(i) Substitute goods(ii) Complementary goods While purchasing a good prices of its substitutes and complements do affect its quantity purchased(i) Price of Substitute Goods Substitute goods are those goods which can easily be used in place of one another for satisfaction of a particular want like tea and coffee An increase in price of substitute good leads to an increase in demand for the given commodity and a decrease in price of substitute good leads to a decrease in demand for the given commodity It means demand for a given commodity is directly affected by change in price of substitute goods For example if price of coffee increases the demand for tea will rise as tea will become relatively cheaper in comparison to coffee(ii) Price of Complementary goods Complementary goods are those goods which are used together to satisfy a particular want like car and petrol An increase in the price of complementary goods leads to a decrease in demand for the given commodity and a decrease in the price of complementary goods leads to an increase in demand for the given commodity For example if price of petrol falls then the demand for cars will increase as it will be relatively cheaper to use both the goods together So demand for a given commodity is inversely affected by change in price of complementary goods3 Income of the Buyer of CommodityDemand for a commodity is also affected by income of its buyer However the effect of change in income on demand depends on the nature of the commodity under consideration In case of some goods like full cream milk fine quality of rice (Basmati rice) etc demand for these commodities increases when income of the buyer increases and

demand for these commodities decreases when income of the buyer decreases Such goods whose demand increases with the increase in income of the buyer are called normal goods But there are some goods like coarse rice toned milk etc whose demand decreases when income of buyer increases and their demand increases when income of the buyer decreases Such goods whose demand decreases with the increase in income of the buyer are called inferior goods Suppose a consumer buys 10 Kgs of rice whose price is ` 25 per Kg He cannot afford to buy better quality of rice because the price of such rice is ` 50 per Kg The consumer is spending ` 250 per month on the purchase of rice Now if income of the consumer increases and he can afford ` 350 on purchase of 10 Kg of rice Now he can afford to buy some quantity of rice say 6 Kgs whose price is ` 25 per Kg and may buy 4 Kgs of rice whose price is ` 50 per Kg Thus he will buy 10 Kgs of rice by spending ` 350 per month Therefore we may conclude that demand for normal goods is directly related to the income of the buyer but demand for inferior goods is inversely related to the income of the buyer4 Tastes and Preferences of the BuyerThe demand for a commodity is also affected by the tastes and preferences of the buyers They include change in fashion customs habits etc Those commodities are preferred by the consumers which are in fashion So demand for those commodities rises which are in fashion On the other hand if a commodity goes out of the fashion its demand falls because no consumer will like to buy it(5) Number of Buyers in the Market(Population)Increase in population raises the market demand whereas decrease in population reduces the market demand for a commodity Not only the size of population but its composition like age (ratio of males females children and old people in population) also affects the demand for a commodity It is because of needs of children young old male and female population differs(6) Distribution of Income and WealthIf the distribution of income and wealth is more in favour of the rich demand for the commodities preferred by the rich such as comforts and luxuries is likely to be higher On the other hand if the distribution of income and wealth is more in favour of poor demand for commodities preferred by the poor such as necessities will be more(7) Season and Weather ConditionsThis is generally observed that the demand for woolens increases during winter whereas demand for ice creams and cold drinks increases during summer Similarly market demand for umbrellas rain coats increases during rainy seasonQ8 REASONS FOR OPERATION OF LAW OF DEMAND WHY DEMAND CURVE SLOPES DOWNWARDNow we will try to explain why does a consumer purchase more quantity of a commodity at a lower price and less of it at a higher price or why does the law of demand operate ie why does the demand curve slope downwards from left to right The main reasons for operation of law of demand are1 Law of Diminishing Marginal UtilityAs you have studied earlier law of diminishing marginal utility states that as we consume more and more units of a commodity the utility derived from each successive unit goes on decreasing The consumer will be ready to pay more for those units which provide him more utility and less for those which provide him less utility It implies that he will purchase more only when the price of the commodity falls2 Income Effect

When price of a commodity falls purchasing power or real income of the consumer increases which enables him to purchase more quantity of the commodity with the same money income Let us take an example Suppose you buy 4 ice creams when price of each ice cream is ` 25 If price of ice creams falls to ` 20 then with same money income you can buy 5 ice creams now3 Substitution EffectWhen price of a commodity falls it becomes comparatively cheaper as compared to its substitutes (although price of substitutes has not been changed) This will lead to rise in demand for the given commodity For example if coke and Pepsi both are sold at ` 10 each and price of coke falls Now coke has become relatively cheaper and will be substituted for Pepsi It will lead to rise in demand for coke4 Change in Number of BuyersWhen price of a commodity falls some old buyers may demand more of the commodity at the reduced price and some new buyers may also start buying this commodity who were not in a position to buy it earlier due to higher price This will lead to increase in number of buyers when price of the commodity falls As a result demand for the commodity rises when its price falls5 Diverse Uses of a CommoditySome commodities have diverse uses like milk It can be used for drinking for sweet preparation for ice cream preparation etc If price of milk rises its use may be restricted to important purpose only This will lead to reduction in demand for other less important uses When price of milk falls it can be put to other uses also leading to rise n demand for itQ9 EXCEPTIONS TO THE LAW OF DEMANDYou have studied in law of demand that a buyer is willing to buy more quantity of a commodity at a lower price and less of it at a higher price But in certain circumstances a rise in price may lead to rise in demand These circumstances are called Exceptions to the Law of Demand Some important exceptions are1 Giffen GoodsGiffen goods are special type of inferior goods in which negative income effect is stronger than negative substitution effect Giffen goods do not follow law of demand as their demand rises when their price rises Examples of Giffen goods are jowar and bajra etc2 Status Symbol GoodsSome goods are used by rich people as status symbols eg diamonds gold jewellary etc The higher the price the higher will be the demand for these goods When price of such goods falls these goods are no longer looked at as status symbol goods and tehrefore therir demand falls3 NecessitiesCommodities such as medicines salt wheat etc do not follow law of demandbecause we have to purchase them in minimum required quantity whatever their price may be4 Goods Expected to be ScarceWhen the buyers expect a scarcity of a particular good in near future they start buying more and more of that good even if their prices are rising For example during war famines etc people tend to buy more of some goods even at higher prices due to fear of their scarcity in near future

Political Science

Constitution of India-The

Preamble

The preamble-

Preamble-

The preamble is the most precious part of the constitution We the people of India having solemnly resolved to constitute India into a Sovereign Socialist Secular Democratic Republic and to secure to all its citizensA preamble is an introductory and expressionary statement in a document that explains the documents purpose and underlying philosophy When applied to the opening paragraphs of a statute it may recite historical facts pertinent to the subject of the statuteNature and purpose of the constitution-Purpose of the Constitution dictates permanent framework of the government to form a more perfect union to establish justice and ensure peace of thenationconstitution provide principles how the government can run itself following the rules and laws written in the constitution of each state keeps them balanced

Answer the following questions-

1 What is preambleA preamble is an introductory and expressionary statement in a document that explains the documents purpose and underlying philosophy2 What is the nature and

purpose of the constitutionConstitution dictatespermanent framework of the government to form a more perfect union to establish justice and ensure peace of the nation

Homework-Learn

Accounts Compatibilty mode

1MEANING OF PARTNERSHIPPartnership is a form of business organisation where two or more persons join hands to run a business They share the profits and losses according to the agreement amongst them According to the Indian Partnership Act 1932 ldquoPartnership is relation between persons who have agreed to share profits of a business carried on by all or any one of them acting for allrdquo For example one of your friends has passed class XII from National Institute of Open Schooling (NIOS) and wants to start a business Heshe approaches you to join in this venture Heshe wants you to contribute some money and participate in the business activities Both of you if join hands constitute a partnership2CHARACTERISTICS1048698 Agreement A partnership is formed by an agreement The agreement may be either oral or in writing It defines the relationship between the persons who agree to carry on business It may contain the terms of sharing profit and the capital to be invested by each partner etc The written agreement is known as partnership deed1048698 Number of persons There must be at least two persons to form a partnership

The maximum number of partners in a partnership firm can be 50 according toCompanies Act 20131048698 Business The Partnership is formed to carry on business with a purpose of earning profits The business should be lawful Thus if two or more persons agree to carry on unlawful activities it will not be termed as partnership1048698 Sharing Profits The partners agree to share profits in the agreed ratio In caseof loss all the partners have to bear it in the same agreed profit sharing ratio10486981048698Mutual Agency Every partner is an agent of the other partners Every partner can bind the firm and all other partners by hisher acts Each partner will be responsible and liable for the acts of all other partners10486981048698Unlimited liability The liability of each partner except that of a minor is unlimited Their liability extends to their personal assets also If the assets of the firm are insufficient to pay off its debts the partnersrsquo personal property can be used to satisfy the claim of the creditors of the partnership firm10486981048698Management All the partners have a right to mange the business However they may authorize one or more partners to manage the affairs of the business on their behalf10486981048698Transferability of Share No partner can transfer hisher share to any one including hisher family member without the consent of all other partners3PARTNERSHIP DEEDAgreement forms the basis of partnership The written form of the agreement is which a document of partnership is It contains terms and conditions regarding the conduct of the business It also explains relationship between the partners This document is called partnership deed Every firm can frame its own partnership deed in which the rights duties and liabilities of the partners are stated in detail It helps in settling the disputes arising among the partners during the general conduct of business 4CONTENTS OF PARTNERSHIP DEEDThe partnership deed generally contains the following (i) Name and address of the partnership firm(ii) Nature and objectives of the business(iii) Name and address of each partner(iv) Ratio in which profits is to be shared(v) Capital contribution by each partner(vi) Rate of Interest on capital if allowed(vii) Salary or any other remuneration to partners if allowed(viii) Rate of interest on loans and advances by a partner to the firm(ix) Drawings of partners and interest thereon if any(x) Method of valuation of goodwill and revaluation of assets and liabilities on the reconstitution of the partnership ie on the admission retirement or death of a partner(xi) Settlement of disputes by arbitration(xii) Settlement of accounts at the time of retirement or death of a partner5IN ABSENCE OF PARTNERSHIP DEEDThe partnership deed lays down the terms and conditions of partnership in regard to rights duties and obligations of the partners In the absence of partnership deed there may arise a controversy on certain issues like profit sharing ratio interest on

capital interest on drawings interest on loan and salary of the partners In such cases the provisions of the Indian Partnership Act becomes applicableSome of the Issues are(i) Distribution of Profit Partners are entitled to share profits equally(ii) Interest on Capital Interest on capital is not allowed(iii) Interest on Drawings No interest on drawing of the partners is to be charged(iv) Interest on Partnerrsquos Loan A Partner is allowed interest 6 per annum on the amount of loan given to the firm by himher(v) Salary and Commission to Partner A partner is not entitled to anysalary or commission or any other remuneration for managing the business

History TOPIC-TOWARDS INDEPENDENCE AND PARTITION THE LAST PHASE (1935-1947)

SUB TOPIC-IMPORTANT POLITICAL DEVELOPMENTS ndash GROWTH OF SOCIAL IDEAS

Socialism is a political social and economic philosophyLike in other parts of the world the Russian revolution of 1917 served as a great inspiration for revolutionaries in India who at that time were engaged in the struggle for liberation from British ruleSocialist ideas led to the formation of communist party of IndiaJAWAHARLAL NEHRU Among the early Congress leaders Jawaharlal Nehru was very much impressed and influenced by the Socialist ideas He also learnt about the Economic activities of the Soviet Union after the Bolshevic Revolution 1917 He made full use of them in IndiaThe election of Jawaharlal Nehru and Subhas Chandra Bose showed the Left wing tendency within CongressJawaharlal Nehru demanded economic freedom along with political freedom of the people in order to end the exploitation of masses

Nehrus working committee included three socialists leaders The Lucknow session was a landmark in the evolution of socialist ideas of the congressSUBHAS CHANDRA BOSE ndash Subhas Chandra Bose had socialist leaning Both Jawaharlal Nehru and Subhas Chandra Bose were known as leftist Congress men Later on National Congress divided into Leftist and rightist campCONGRESS SOCIALIST Within the Congress some leaders formed the Congress Socialist partyPattavi Sitaramyya Sardar Patel Rajendra Prasad had hostile attitude towards the Congress Socialist partyJawaharlals attitude was hesitant

1 QUESTION ndash Mention name of two Congress leaders who had socialist leaning

1ANSWER ndash Subhas Chandra Bose and Jawaharlal Nehru2QUESTION- In which session of the congress Jawaharlal elaborated his Socialist ideas2 ANSWER ndash Lucknow and Faizpur Session in December 1935 and 19363QUESTION ndash Why Congress was sharply divided into leftist and rightist camp 3ANSWER ndash Subhas Chandra Bosersquos attempt to seek re election for congress presidentship in 1939sharply divided the National Congress into Leftist and Rightist camp4 QUESTION ndash Who was MN Roy 4 ANSWER ndash Manabendra Roy first formed the Communist Party of India outside the country at Tashkent in 19205QUESTION ndash Who formed the Congress Socialist Party within the Congress5 ANSWER ndash Jaya Prakash Narayan Achyut Patwardhan Acharya Narendra Dev Ram Mohan Lohia Aruna Asaf Ali6QUESTION ndash When was the Congress Socialist Party formed What was its object6 ANSWER ndash 1934The Congress Socialist Party sought to work out socialist programme through the Congress They joined hands with the Congress and wanted to carry

Subhas Chandra Bose being expelled from the congress after the Tripuri rift he formed Forward BlockThere were basic differences between the Congress Socialists and the communistsTRADE UNION ACTIVITIES Maximum working class people lived in Bombay and Calcutta The working and living conditions of those workers were very miserable In this situation Shasipada Banerjee NM Lokhande protested against the oppression of the working class peopleThe first Trade Union Madras Labour Union was formed in 1918 by BP WadiaIndustrial strikes took place in Kanpur Calcutta Madras Jamshedpur and Ahmedabad AITUC was formed in Bombay in 1927 The growth of Trade union among the workers was slow because of the fear of the dismissal of the jobIn the mean time the Moderates as well as Communists left AITUC and formed separate organization

on National struggle with the help of workers and peasant class of the society7 QUESTION ndash What was the name of the party founded by Subhas Chandra Bose7 ANSWER- Forward Block8QUESTION ndash Who was Shasipada Banerjee8 ANSWER ndash Shasipada Banerjee was a radical Brahmo He founded a working menrsquos club to protest against exploitation of the British rulers towards the working class of India9 QUESTION ndash What was the weekly published by NM Lokhande9ANSWER- Dinabandhu10 QUESTION ndash Who founded Bombay Mill-Hands Association and in which year10 ANSWER- NM Lokhande in189011 QUESTION- Who was BP WadiaANSWER- BPWadia was the founder of Madras Labour Union in191812 QUESTION- What was the name of the first labour union of India12 ANSWER- Madras Labour Union13 QUESTION Who founded the Majur Mahajan 13 ANSWER GANDHIJI14 QUESTION What was the full form of AITUC When it was formed14 ANSWER All India Trade Union Congressin 192715QUESTION Who formed the Red Trade Union Congress and in which year15ANSWER The Communists formed the Red Trade Union Congress16 QUESTION What do you mean by Socialism16 ANSWER Socialism describes any political and economic theory that says the community rather than individuals should own and manage property and natural resources

Subject Eng Literature (The Tempest ndash William Shakespeare) Topic Act III Scene 3 Lines 1 to 52 (Line 52 ndash Brother my lord the Duke Stand to and do as we) Date 13th April 2020 (4th Period)

[Students should read the original play and also the paraphrase given in the school prescribed textbook]Summary Questions amp Answers

o Alonso Sebastian Antonio Gonzalo Adrian Francisco and others wandered about the island in search of Ferdinand and gets tired and hungry of the toil and at the same time gives up all hope of finding him

o Antonio and Sebastian are happy that Alonso is out of hope and decide to make another attempt on his life that night when being so tired they will be sleeping soundly

o Suddenly a solemn and strange music is heard in the air and several strange shapes enter bringing in a banquet These strange shapes then dance round it with gestures of salutation and then inviting the King to eat they depart

o Seeing this strange scene all are inclined to believe the tales told by travelers that there truly are ldquounicornsrdquo and ldquothe phoenixrsquo thronerdquo

1 ALONSO What harmony is this My good friends hark (L18-27)

GONZALO Marvellous sweet music

[Enter several strange shapes bringing in a banquet

they dance about it with gentle actions of salutation

and inviting the King and his companions to eat they depart]ALONSO Give us kind keepers heavens What were theseSEBASTIAN A living drollery Now I will believe

That there are unicorns that in Arabia

There is one tree the phoenixrsquo throne one phoenix

At this hour reigning thereANTONIO Ill believe both

And what does else want credit come to me

And Ill be sworn rsquotis true Travellers neer did lie

Though fools at home condemn rsquoem

(a) How did Prospero present an amazing spectacle before Alonso and his companions

Using his magic powers Prospero ordered strange shapes to lay a banquet before Alonso and his companions The shapes brought several dishes with tasty eatables in them They placed the dishes on a table before Alonso and his companions Then the strange shapes began to dance gracefully around the banquet While dancing they made gestures inviting them to eat the food Then suddenly the shapes disappeared(b) Who were the guests at the strange banquet Describe the lsquoliving drolleryrsquo

Alonso Sebastian Antonio Gonzalo Adrian and Francisco were the guests at the strange banquet

The term ldquoliving drolleryrdquo refers to live entertainment show In this context when Alonso the King of Naples Sebastian his brother Antonio the treacherous brother of Prospero Gonzalo the kind and loyal councillor to the King Adrian and Francisco came to the island they were hungry and weary in their spirits They heard a solemn and strange music They were shocked to see several strange shapes bringing in a banquet and these shapes danced about it with gentle action of salutation inviting the King and his companions to eat After this Sebastian described this show as lsquoliving drolleryrsquo(c) What is lsquophoenixrsquo What are lsquoUnicornsrdquo

The term lsquophoenixrsquo refers to a mythical Arabian bird which lived alone and perched on a solitary tree After one hundred years it expired in flames and rose again from its own ashes

lsquoUnicornsrsquo refers to the mythological four-footed beasts having horns in the centre of their foreheads When the horns are ground into powder the powder was believed to be

an aphrodisiac(d) How does Sebastian explain the puppet show OR Why does the speaker now believe in unicorns and phoenix

Sebastian finds several strange shapes bringing in the banquet They invite the king and his party for dinner and soon depart He tells that if such a strange sight can be a reality there is nothing incredible in the world and from the present moment he will believe anything He says that it is a strange dumb show enacted not by puppets but by living beings It is stranger than a travellerrsquos tale Seeing such a thing

before his own eyes he will no longer disbelieve the story about unicorns and phoenix(e) How do the other characters present respond to this living drollery

At the sight of the lsquoliving drolleryrsquo like Sebastian Gonzalo and Antonio too acted strangely Antonio told that he too now believes in unicorns and phoenix and anything else that seems to be incredible He too now believes in travellersrsquo tales Gonzalo told that if he would report those happenings in Naples nobody will believe him He considers that those gentle shapes were gentler in manner in comparison to the living beings Alonso was at first sight suspicious and told them that those strange shapes conveyed their meaning in expressive gestures when they seemed to lack speech by their movements and sounds Francisco was amazed at their mysterious disappearance

2 ALONSO Not I

(Line 43-52)GONZALO Faith sir you need not fear When we

were boysWho would believe that there were mountaineers

Dewlapped like bulls whose throats had hanging at rsquoem

Wallets of flesh Or that there were such men

Whose heads stood in their breasts Which now we find

Each putter-out of five for one will bring us

Good warrant ofALONSO I will stand to and feed

Although my lastmdashno matter since I feel

The best is past Brother my lord the Duke

Stand to and do as we

(a) How does Alonso respond at the spectacle of the shapes which were sent to them at the instruction of Prospero

After seeing the strange sight of appearing and disappearing of the shapes sent by Prospero to arrange a banquet for them Alonso says that his surprise at having seen those creatures is infinite and he is fully justified in feeling so much surprise He thinks that their shapes their gestures and the sounds they made were indeed amazing Although they do not possess the gift of speech yet they were able to convey their

thoughts by means of their gestures only

(b) What does Prospero say about the views expressed by Alonso regarding the shapes What does Francisco think about the shapesAfter hearing Alonsorsquos views about the shapes Prospero says that this manrsquos praise of the spirits is rather hasty He means to say that Alonso has shown great haste in reaching the conclusion about the shapes Francisco is amazed to see that those shapes disappeared in a mysterious way(c) What does Sebastian ask Alonso to doSebastian tells Alonso that the shapes having disappeared should not matter to them because they have left the eatables behind He asks Alonso to enjoy eating as they are extremely hungry but the king does not accept his offer of enjoying the dishes(d) How does Gonzalo try to dispel Alonsorsquos fear of those strange shapes What kind of references does he give to AlonsoGonzalo says that those who have travelled abroad have reported seeing even stranger sights than these shapes that Alonso and his companions have beheld Hence there is no reason to feel afraid of these shapes Gonzalo further adds that in his younger days he had heard strange stories from travelers and Alonso might have heard similar stories For instance it was said that there existed a certain race of

human beings who had huge lumps of flesh hanging at their throats and who therefore resembled bulls Then Gonzalo tells about a race of human beings whose heads were located at their breasts Gonzalo says that such stories were not believed by most people in those days but now-a-days these stories have become common(e) Explain the following lsquoEach putter-out of five for onersquoEnglish travellers often insured their trips with London brokers Those that went on foreign travels those days used to deposit a certain amount with some firm or company in London before their departure If the travelers failed to return the money was forfeited by the company with which it had been deposited But this money was repaid five-fold if the travelers returned safe and sound In this way a traveler stood a great chance of recovering the entire cost of his

travels(f) Give the explanatory meanings of the following expressions in the context of the above extract (i) Dewlapped (ii) Wallets of flesh

(iii) Putter-out(i) Dewlapped having big lumps of flesh at the necks(ii) Wallets of flesh large masses of flesh looking like bags(iii) Putter-out to invest money before commencing the travel

  • General methods of preparation of hydrogen
  • Chapter Dimensional Analysis (Summary)
    • Properties of Charge
Page 35:   · Web viewSubject. Topic. Summary. Execution. Hindi. व्याकरण. शरीरके अंगो के नाम लिखिए. 1) आँख 2) नाक 3

1to earn living and acquiring wealth2Result can be measured in terms of money

3ExampleBusinessprofession and employment

1 to obtain some satisfaction

2Result cannot be measured in terms of money

3ExampleFamily-orientedreligious socialCultural and national

BUSINESS STUDIES

BUSINESS ENVIRONMENT

Welcome to the new sessionToday we are going to start the first chapter and the name of the chapter is Business Environment

In todayrsquos world every business enterprise is a part of the society It exists and operates in association with various groups in society such as customers suppliers competitors banks and financial institutions government agencies trade unions media and so on All these groups influence the functioning of business in one way or the other They constitute the environment of businessConcept of Business Environment

The term lsquobusiness environmentrsquo refers to the sum total of all individuals institutions and other forces that lie outside a business enterprise but that may influence its functioning and performance

The main features of business environment

Totality of External forces General and Specific forces Interrelatedness Complexity Dynamic Uncertainty Relativity

The Interrelation between business and its environment

The business enterprise is an open system It continuously interacts with its environment It takes inputs

Prepare the following questions from todayrsquos assignment

1 What do you mean by business environment

The term lsquobusiness environmentrsquo means the aggregate of all forces factors and institutions which are external to and beyond the control of an individual business enterprise but they may influence its functioning and performance Business environment is the macro framework within which a business firm a micro unit operates It consists of several interrelated and interacting elements

2 Explain the main features of business environment in brief

Totality of External forces-Business environment is the sum total of all things external to a business environment

General and Specific forces-It includes both the forces general forces are the economic social political legal and technological conditions which indirectly influence all business enterprise Specific forces are the investors customers competitors and suppliers which influence individual enterprise directly

Interrelatedness-Different elements of environment are interrelated for an example growing awareness for health care has increased the demand for health foods

Complexity- Business environment id

(such as raw materials capital labour energy and so on) from its environment transforms them into goods and services and sends them back to the environment

Fig 1 Business Environment Relationship

complex in nature as the elements keep on changing example economic technological and other forces changes in demand for a product and service

Dynamic-Business environment is not static it keeps on changing

Uncertainty- Itrsquos very difficult to predict future events such as technology and fashion which occur fast and frequently

Economics Basic Economic ConceptsSub topic

Microeconomics and

Macroeconomics

Welcome to the new sessiontoday we are going to start the first chapter of Class XI The name of the chapter that we are going to start is Basic Economic concepts

Now Economics covers the study of human activities Human activities are those activities which are performed by humans to satisfy their wants

Thus Human wants are unlimited and therefore economic activities such as production exchange and consumption are needed in order to satisfy those wants

The study of economics is divided largely in two parts which areMicroeconomics and Macroeconomics

SUBJECT- MATTER OF ECONOMICS

MICROECONOMICS MACROECONOMICS

Questions1Who has coined the words micro and macro economics

Answer Ranger Frisch coined the words lsquomicrorsquo and lsquomacrorsquo in 1933 to denote the two branches of economic theory namely microeconomics and macroeconomics

2What is microeconomicsAnswer It is the study of behaviour of individual decision ndash making unit such as consumers firms etc

3 What is macroeconomicsAnswer Macroeonomics is the study of overall economic phenomena like employment national income etc

4 What is the importance of microeconomicsAnswer

Microeconomics helps in formulating economic policies which enhance productive efficiency and results in greater social welfare

It helps the government in formulating correct price policies

It explains the working of a capitalistic economy where individual units(producers and consumers ) are free to take their own decision

Micro means a small part in

microeconomics we do not study the whole economy Hence we study an individual consumer and his or her choices and a producer and his or her profit maximizing decisions in the market Thus it does not mirror what happens in the economy as a whole

Macroeconomics on the other hand studies the economy as a whole It is concerned with aggregate and depicts the entire picture of the economyMacroeconomics deals with the national income aggregate investment aggregate consumption etc

Features of Microeconomics It deals with small

parts of the country Hence it looks at

individual consumers firms and industries

It deals with individual income consumption and savings

It studies the determination of price of any product or factors of production

It deals with the working of market via the price mechanism which is nothing but the determination of price and quantity of a commodity by the forces of demand and supply

Features of Macroeconomics

It deals with the study of the economy as a whole

It is concerned with

5 Give a limitation of microeconomics Microeconomics fails to explain the

functioning of an economy as a whole It cannot explain unemployment illiteracy and other problems prevailing in the country

6 What is the importance of macroeconomics It gives overall view of the growing

complexities of an economic system It provides the basic and logical

framework for formulating appropriate macroeconomic policies (eg for inflation poverty etc )to direct and regulate economy towards desirable goals

7What is the limitation of macroeconomics It ignores structural changes in an

individual unit of the aggregate

8 Differentiate between Microeconomics and Macroeconomics

Microeconomics Macroeconomics

the study of aggregates

National income aggregate savings and aggregate investments are major concepts dealt within macroeconomics style

It studies the determination of general price levels

It investigates into the problem of unemployment and the achievement of employment

It studies the aspect of decision making at the aggregate and national levels

It includes all growth theories whether related to developed or developing economies it also includes the study of economic systems and the working of the economy under different systems

Note Both Micro and macro economics are complementary and should be fully utilized for proper understanding of an economy

1It studies economic aspect of an individual unit2It deals with individual incomeConsumption and savings

3 It facilitates determination of price of any product or factors of production

4 Itrsquos scope is narrow and restricted to individual unit

1It studies the economy as a whole

2It deals with the national income aggregate consumption and aggregate savings3 It facilitates determination of general price level in an economy

4 Itrsquos scope is wide as it deals with economic units on the national level

ACCOUNTS

Introduction to Accounting and Book-keeping

Today I am going to share you the meaning of Accounting and Book-keeping and its related terms bullAccounting bullBook Keeping bullAccountsbullTypes Of Accounts bullAccounting Cycle

bull Meaning of accounting

Ans ) Accounting is the art and science of recording classifying and summarising monetary transactions

bull Meaning of Book-keeping

Ans) Bookkeeping is the art of recording business transactions with the view of having a permanent record of them and showing their effect on wealth

bull Meaning of account

Ans) The term account means a record of

business transactions concern a particular person of firm asset or income or expense It is a summarised record of all transactions which take place in an accounting year

bull Types of accountsPersonal accounts ndash Personal accounts relating

to person and Organisation are known as personal accounts Example Ramrsquos Account ABC amp Co Account etc

Real account - The accounts related to tangible and intangible assets are called real accounts Example Cash Account Furniture Account etc

Nominal account- Accounts related to expenses losses incomes and gains are known as nominal accounts Example Wages Account Salary Account Discount Account etc

bull Accounting cycle Accounting cycle refers to a complete sequence of accounting activities It begins with recording of transactions and ends with the preparation of a balance sheet

Chemistry TopicAtomic Structure

Thomsonrsquos atomic modelThomson (1898) was the first to propose the model of an atomHe proposed that an atom can be regarded as a uniform sphere of positive electricity in which requisite number of electrons are embedded evently to neutralize the positive chargeThis is just like plums embedded in a pudding or seeds evently distributed in red spongy mass of a watermelonThis model of atom is known as ldquoPlum-Pudding modelrdquo or

Q1)What is the fundamental constituents of atomAns Electron Proton and neutrons are the fundamental constituents of atomQ2)What is the value of fundamental unit of electricityAnsThe charge carried by one electron is sad to be the fundamental unit of electricityIts magnitude is 48times10-10esuOr 1602times10-19C Q3)Name the element containing no neutronAnsOrdinary hydrogen atom or protium 1H1

Types of AccountPersonal AccountReal AccountNominal AccountBalance Sheet (opening)

ldquowatermelon modelrdquoThis model could explain the electrical neutrality of an atom but failed to explain the result of scattering experiment carried out by Rutherford in 1911So it was rejected ultimately

Q4)Why is an electron called universal particleAns Itrsquos mass and Charge are independent of its source

EVS Chapter 1 ndash Modes of Existence

Modes of existence When one speaks normally about the mode of existence of some group or individual one refers to their customs their mode of being their ethology their habitat in some way their feeling for a placeDifferent modes of exixtence are ndash

1 Hunting ndashGathering2 Pastoral3 Agricultural4 Industrial

1 Hunting and gathering Hunting and gathering mode of existence is characterized by obtaining food from hunting wild animals including fishing and gathering wild plants From their earliest days the hunter-gatherer diet included various grasses tubers fruits seeds and nuts Lacking the means to kill larger animals they procured meat from smaller game or through scavenging

Societies that rely primarily or exclusively on hunting wild animals fishing and gathering wild fruits berries nuts and vegetables to support their diet are called hunting and gathering societies

At least this used to be practice of human beings before agriculture is invented As their brains evolved hominids developed more intricate knowledge of edible plant life and growth cycles

Q) Write the features of Hunting ndash gathering societiesAns - There are five basic characteristics of hunting and gathering societies

i The primary institution is the family which decides how food is to be shared and how children are to be socialized and which provides for the protection of its members

ii They tend to be small with fewer than fifty members

iii They tend to be nomadic moving to new areas when the current food supply in a given area has been exhausted

iv Members display a high level of interdependence

v Labor division is based on sex men hunt and women gather

Political Science

Introduction to political science

Political science occasionally called politology is a social science which deals with systems of governance and the analysis of political activities political thoughts associated constitutions and political behaviorThe study of political science involves the study of both the

Answer the following questions-1 What is political science

Political science occasionally called politology is a social science which deals with systems of governance and the analysis of political activities political thoughts associated constitutions and political behavior

2 Short notes-

traditional and modern theories of politicsTraditionalClassical political sciencepolitical theory-Traditional political science is the study of politics before Second World War The methodology to study Politics was traditional (legal formaletc) the definition of politics traditional (Politics begins and end with state)area of study (constitution state machinery)was traditionalModern Political scienceModern political theory-Modern Political Theory critically examines the contemporary state of political theory making an assessment of the achievement and limitations of the Behavioural Revolution in its totality and reviews objectively the major paradigms and conceptual frameworks adopted by the disciplineContemporary attempts at the development of an integrated political theory involving the use of both traditional and modern concepts approaches and theories-Around late 1960s several political scientists realized the importance of both the traditional political theory and modern Political theory They began building an integrated theory of politics involving a systematic mixture of traditional and modern studies of politics It was held that the study of a complex and vast field like politics needs both traditional as well as

Classical political theory Modern Political theory

Homework-Learn

modern concepts and approaches for studying itrsquos all aspects

Subject Eng Literature (The Tempest ndash William Shakespeare) Topic Act I Scene 1 Lines 1 to 32 (Line 32 ndash Gonzalo hellip If he be not born to be hanged our case is miserable) Date 13th April 2020 (3rd Period)

[Students should read the original play and also the paraphrase given in the school prescribed textbook]Summary Questions amp Answers

[SUMMARY OF THE ENTIRE SCENE]

o The play starts with the scene of a severe storm at sea Alonso (King of Naples) Sebastian (Alonsorsquos brother) Ferdinand (Alonsorsquos son) Gonzalo Antonio (the usurping Duke of Milan) are in a ship in the midst of the storm

o The mariners are trying their best to control the vessel from running aground and are totally following the orders of their Master the Boatswain They have scant success

o The mariners become extremely unhappy and annoyed when most of the passengers arrive on the deck thereby hampering their effort to save the ship There is serious confrontation between them and the passengers who are part of the Kingrsquos entourage

o The mariners could not save the ship

SUMMING-UP

(i) Vivid description of the scene which gives a realistic description of terror and confusion of a tropical storm

(ii) Shows Shakespearersquos accuracy of knowledge in describing the naval operations and also matters of seamanship

(iii) The opening scene justifies the title ndash The Tempest

UNANSWERED QUESTIONS

(i) The King always travels with his entire fleet including his soldiers Where

(1) GONZALO Nay good be patient (Line 15-26)BOATSWAIN When the sea is Hence What cares these

roarers for the name of the king To cabin silence Trouble us not

GONZALO Good yet remember whom thou has aboardBOATSWAIN None that I more love than myself You are a

councillor if you can command these elements to silence and work

the peace of the present we will not hand a rope more use your authority If you cannot give thanks you have

lived so long and make yourself ready in your cabin for the mischance of the hour if it so hap [To the Mariners]

Cheerly good hearts [To Gonzalo] Out of our way I say

(a) To whom is the boatswain speaking What does he mean by lsquoNone that I more love than myselfrsquo

The Boatswain is speaking to Gonzalo the honest old councilor of the Duke of MilanBy using the words ndash lsquoNone that I love more than I love myselfrsquo means that for the Boatswain nobody is dearer to him than his own life

(b) What were the conditions that made the boatswain react in this way

The Boatswain reacts in this way because the storm is at sea and Alonso King of Naples Sebastian his brother Ferdinand his son Gonzalo Antonio the usurping Duke of Milan on board are in distress and in panic Thus they have rushed to the deck interrupting the work of the mariners

(c) What hope does Gonzalo take from the attitude of the boatswain

The insolent and authoritative attitude of Boatswain makes Gonzalo feel comforted He tells that there are no signs that the Boatswain will be drowned But his facial appearance and attitude shows that he is destined to die on land by hanging which in effect means that all on board will be saved Otherwise all the persons on board are doomed

(d) How can they lsquomake yourself ready in your cabinrsquo For what were they asked to make ready themselves

In order to make themselves ready in their cabin the

were the other ships

(ii) Why was the ship in that area Where was it coming from or going where

(iii) The ship broke apart What happened to those who were in the ship

passengers on board must prepare for death which they will possibly soon have to meetThey can retire to their cabins and offer prayers to the Almighty to save them from drowning

(e) What does the boatswain say when he is asked to be patient What does he order to the royal party

When the boatswain is asked to be patient and remain calm he says that he will be patient only when the storm will be over and the sea will be calm but as long as the storm blows and there is danger to the ship he cannot think of being patient He orders the royal party to go to the cabin and leave the mariners to their work

(2) GONZALO I have great comfort from this fellow (Line 27-36)

Methinks he hath no drowning mark upon him his complexion is perfect

gallows Stand fast good Fate to his hanging Make the rope of his destiny our cable for our own doth little advantage If he be not born to be hanged our case is miserable

(a) Why does Gonzalo regard the Boatswain in the midst of danger

In the midst of danger Gonzalo regards the boatswain because he feels that the Boatswain is a source of comfort and is bent upon to do his work sincerely which in this case is saving the ship and its passengers from the severest of raging storm

(b) What reasons does Gonzalo give when he says that none in the ship will die of drowning

Gonzalo is almost sure that none in the ship will die by drowning His says that there is no mark on the face of the boatswain that indicates that he will die by drowning On the other hand the lines on his face are strong indications that he will be hanged to death Therefore there shall be no danger of the shiprsquos sinking

(c) Explain the following ldquoStand fast good Fate to his hanging Make the rope of his destiny our cable for our own doth little advantage If he be not born to be hanged our case is miserablerdquo

The stated lines mean that if the will of destiny is to be carried out then the ship will not get wrecked and all the passengers will be saved The safety of the passengers therefore depends upon the will of fate being carried out in the case of the boatswain If however the boatswain is not to die by hanging then the passengers are also very unsafe because in that case the ship is likely to sink

(d) What order does the Boatswain give to the sailors

when he re-enters What does he say about the crying of the fellows inside the cabin

The boatswain orders the sailors to bring the topmast lower and bring the ship close to a stationary position with the help of the main sail He says that the fellows inside the cabin are moaning and crying in their distress louder than his voice and louder even than the roaring of the storm

Class XII (ScienceCommerceHumanities) Subject Topic Summary Execution

Computer Science

PropositionalLogic

Propositional logic is a procedure to provide reasoning through statementProposition A ststement that results in True or False is said to be proposition There are two types of propositionSimple proposition amp compound propositionSimple proposioton A simple proposition is one that is not a part of any other proposition Such sentential form of proposition is symbolized with english letters in short For example Ram is a claver student (TrueFalse)Where do you live (Not in True or False)Grapes are sweet (TrueFalse)It rains today (TrueFalse)Here we can see some statements anwer would be true or false but some staements answer can not give in terms of true or false Thus the sentences which can be answered in true or false are known as simple propositionAssigning propositon to a variableThe general syntax to assign propostion to a variable is as followsVariable = Simple propositonFor example A=Ram is a clever studentB= Grapes are sweetC= it rains todayCompound proposition

helliphellipto be continued in next classhelliphellipMath Relation Relation If A and B are two non-empty sets

then a relation R from A to B is a subset of AxB If R A x B and (a b) R then we say that a sube isinis related to b by the relation R written as aRbeg Let A be the set of students of class XII and B be the set of students of class XI Then some of the examples of relation from A to B arei) (a b) AXB a is brother of bisinii) (a b) AXB age of a is more than age of isinb Types of relation In this section we would like to study different types of relations We know that a relation in a set A is a subset of A times A Thus the empty set φ and A times A are two extreme relations For illustration consider a relation R in the set A = 1 2 3 4 given by R = (a b) a ndash b = 10 This is the empty set as no pair (a b) satisfies the condition a ndash b = 10 Similarly R = (a b) | a ndash b | ge 0 is the whole primeset A times A as all pairs (a b) in A times A satisfy | a ndash

Example 1 Let A be the set of all students of a boys school Show that the relation R in A given by R = (a b) a is sister of b is the empty relation and R = (a b) the primedifference between heights of a and b is less than 3 meters is the universal relationSolution Since the school is boys school no student of the school can be sister of any student of the school Hence R = φ showing that R is the empty relation It is also obvious that the difference between heights of any two students of the school has to be less than 3 meters This shows that R = A times A is primethe universal relation Example 2 Show that the relation R in the set 1 2 3 given by R = (1 1) (2 2) (3 3) (1 2) (2 3) is reflexive

b | ge 0 These two extreme examples lead us to the following definitionsDefinition 1 A relation R in a set A is called empty relation if no element of A isrelated to any element of A ie R = φ A times AsubDefinition 2 A relation R in a set A is called universal relation if each element of A is related to every element of A ie R = A times A Both the empty relation and the universal relation are some times called trivial relation Definition 3 A relation R in a set A is called(i) reflexive if (a a) R for every a Aisin isin(ii) symmetric if (a1 a2) R implies that (aisin 2a1)

R for all aisin 1 a2 Aisin(iii) transitive if (a1 a2) R and (aisin 2 a3) R isinimplies that (a1 a3) R for all aisin 1 a2 a3 AisinDefinition 4 A relation R in a set A is said to be an equivalence relation if R is reflexive symmetric and transitive

but neither symmetric nor transitiveSolution R is reflexive since (1 1) (2 2) and (3 3) lie in R Also R is not symmetric as (1 2) R but (2 1) isin notinR Similarly R is not transitive as (1 2) R and (2 3) R but (1 3) R isin isin notinExample 3 Show that the relation R in the set Z of integers given byR = (a b) 2 divides a ndash b is an equivalence relationSolution R is reflexive as 2 divides (a ndash a) for all a Z isinFurther if (a b) R then 2 divides a isinndash b Therefore 2 divides b ndash a Hence (b a) R which shows that R is isinsymmetric Similarly if (a b) R and (b c) R isin isinthen a ndash b and b ndash c are divisible by 2 Now a ndash c = (a ndash b) + (b ndash c) is even (Why) So (a ndash c) is divisible by 2 This shows that R is transitive Thus R is an equivalence relation in ZExample 4 Let L be the set of all lines in a plane and R be the relation in L defined as R = (L1 L2) L1 is perpendicular to L2 Show that R is symmetric but neither reflexive nor transitiveSolution R is not reflexive as a line L1 can not be perpendicular to itself ie (L1 L1) R notinR is symmetric as (L1 L2) Risin

L1 is perpendicular to L2rArr L2 is perpendicular to L1rArr (L2 L1) RrArr isin

R is not transitive Indeed if L1 is perpendicular to L2 and L2 is perpendicular to L3 then L1 can never be perpendicular to L3 In fact L1 is parallel to L3 ie (L1 L2) R isin(L2 L3) R but (L1 L3) Risin notin

Chemistry Solid state Characteristics if Solids(i)The particles are locked in fixed positions they are unable to change their relative positions and this brings a definite shape and volume of a solid(ii)In a solid the constituent particles are held by strong forces of attractionThe forces of attraction may be bonding or non bonding(iii)The constituent particles in a solid pack together as closely as possibleoccupying most of the available space within the solidThus the empty space in a solid is very smallThis makes a solid highly rigid and nearly incompressibleThis also explains why a solid has high density and exhibits slow diffusionClassification of Solids

Q1)Define Crystalline solids AnsA Solid that has a definite geometrical shape and a sharp melting pointand whose constituent particles (atomsmolecules or ions) are arranged in a long range order of definite pattern extending throughout the solid is called a crystalline solidExNaClQ2)Define Amorphous solids AnsA solid that does not have a definite shape and a sharp melting pointand whose constituent particles (atomsmolecules or ions) are not arranged in a definite pattern is called an amorphoussolid

Crystalline solidsAmorphous solids

ExGlassRubberQ3)Classify Crystalline Solids Crystalline Solids

Physics Coloumbrsquos Law (Summary)

Before Going Into Coloumbrsquos Law We Will First Learn What is Charge Properties of Charge and Always remember that charge is quantized ie a body always have static charge of magnitude equal to some integral multiple of fundamental electronic charge e= 16 x 10- 19 C

Charge is the property of matter that causes it to produce and experience electrical and magnetic effects The study of the electrical charges at rest is called electrostatics When both electrical and magnetic effects are present the interaction between charges is referred to as electromagnetic

There exist two types of charges in nature positive and negative Like charges repel and unlike charges attract each other

The type of charge on an electron is negative The charge of a proton is the same as that of an electron but with a positive sign In an atom the number of electrons and the number of protons are equal The atom is therefore electrically neutral If one or more electrons are added to it it becomes negatively charged and is designated as negative ion However if one or more electrons are removed from an atom it becomes positively charged and is called a positive ion

The excess or deficiency of electrons in a body gives the concept of charge If there is an excess of electrons in a body it is negatively charged And if there is deficiency of electrons the body becomes positively charged Whenever addition or removal of electrons takes places the body acquires a charge

The SI Unit of charge is coulomb (C) In SI units the current is a fundamental quantity having a unit of ampere (A) The unit of charge is defined in terms of the unit of current Thus one coulomb is the charge transferred in one second across the section of a wire carrying a

Ionic SolidsMetallicSolids

Molecular Solids

current of one ampere

As q = It we have1 C = (1 A) (1 s)

The dimensions of charge are [A T]

Properties of Charge

(1) Quantization of Charge Electric charge can have only discrete values rather than any value That is charge is quantized The smallest discrete value of charge that can exist in nature is the charge on an electron given as

e = plusmn 16 x 10- 19 C

This is the charge attained by an electron and a protonA charge q must be an integral multiple of this basic unit That is

Q = plusmn ne where n = 1 2 hellip

Charge on a body can never be (frac12)e (23)e or 57e etcWhen we rub a glass rod with silk some electrons are transferred from the rod to the silk The rod becomes positively charged The silk becomes negatively charged The coulomb is a very large amount of charge A typical charge acquired by a rubbed body is 10 - 8 C

Biology Reproduction in organisms

Welcome to this new session 2020-21Today in this first chapter we mainly discuss about reproduction types needs and life span of some organismsWe also discuss about difference between sexual and asexual reproduction

Q1 What is reproductionReproduction is defined as a biological processin which an organism gives rise to young onessimilar to itselfQ2 What are the needs of reproductionbulli) Reproduction maintain life on earthii) It enables the continuity of the species generation after generationiii) It creates genetic variation among populationsQ3 Define Life span and write some orgnisms life spanbull Life span is the period from birth to

the natural death of an organism- OrganismsLife span1 Butterfly 1 - 2 weeks2 Fruit fly 30 days3Dog 10-13 years4 Rose5-7 years5 Tortoise100-150 years6 Banyan Tree -200 - 250 yearsQ4 Reproduction is of two types in case ofanimals but in case of plants vegetative propagation is also present

Asexual Reproduction Sexual Reproductioni) Always uniparentalii) Gametes are not involvediii) Only mitotic division involvediv) Somatic cells of parents are involvedv) Offsprings are genetically similar to the parents

i) Usually biparentalii) Gametes are involvediii) Meiosis occurs during gametogenesis Mitosis occurs after fertilisationiv) Germ cells of the parents are involvedv) offsprings are genetically different from the parents

COMMERCE BUSINESS ENVIRONMENT

Welcome to the new sessiontoday we are going to start the first chapter of Class XII The name of the chapter is Business Environment

Already many of you have got some idea about the word business environment form the first chapter of business studies in class XI

In todayrsquos world every business enterprise is a part of the society It exists and operates in association with various groups in society such as customers suppliers competitors banks and financial institutions government agencies trade unions media and so on All these groups influence the functioning of business in one way or the other They constitute the environment of businessConcept of Business Environment

The term lsquobusiness environmentrsquo refers to the sum total of all individuals institutions and other forces that lie outside a business enterprise but that may influence its functioning and performance

The main features of business environment Totality of External forces General and Specific forces Interrelatedness Complexity Dynamic Uncertainty

Prepare the following questions from todayrsquos assignment

2 What do you mean by business environment

The term lsquobusiness environmentrsquo means the aggregate of all forces factors and institutions which are external to and beyond the control of an individual business enterprise but they may influence its functioning and performance Business environment is the macro framework within which a business firm a micro unit operates It consists of several interrelated and interacting elements

2 Explain the main features of business environment in brief

Totality of External forces-Business environment is the sum total of all things external to a business environment

General and Specific forces-It

Relativity

The Interrelation between business and its environment

The business enterprise is an open system It continuously interacts with its environment It takes inputs (such as raw materials capital labour energy and so on) from its environment transforms them into goods and services and sends them back to the environment

Fig 1 Business Environment Relationship

includes both the forces general forces are the economic social political legal and technological conditions which indirectly influence all business enterprise Specific forces are the investors customers competitors and suppliers which influence individual enterprise directly

Interrelatedness-Different elements of environment are interrelated for an example growing awareness for health care has increased the demand for health foods

Complexity- Business environment id complex in nature as the elements keep on changing example economic technological and other forces changes in demand for a product and service

Dynamic-Business environment is not static it keeps on changing

Uncertainty- Itrsquos very difficult to predict future events such as technology and fashion which occur fast and frequently

Business Studies

Human Resources Management

Human resource of an organisation are the aggregate of knowledge skills attitudes of people working in it

The management system which deals with human resources is called human resource management

Features of HRMbullComprehensive functionbullPeople-oriented

Question1) What do you mean by human

resource management Answer) Human resource management may be defined as that field of Management which has to do with planning organising and controlling the functions of procuring developing maintaining and utilising the labour force

bullAction oriented bullPervasive function bullContinuous function

2) Explain the features of HRM in brief

Answer)bullHuman Resource Management is concerned with managing people at work bull Human Resource Management is concerned with employees which bring people and organisations together so that the goals of each are met bullHuman resource management considered every employees as an individual and also promote their satisfaction and growth bull Human resource management is inherent in all organisations and at all levelsbullManagement of human resources are ongoing on never ending process which requires a constant alertness and Awareness of human relations

3) ldquoHR function is said to be pervasiverdquowhy

Answer) Human resource management is required in all organisations whether it is private or government organisations armed forces sports organisations etc It permeatsall the functional areas like production marketing finance research etc This from this feature of human resource management it can be said that it is pervasive in nature

Economics Demand Q1DEFINITION OF DEMANDIn economics demand is the quantity of a good that consumers are willing and able to purchase at various prices during a given period of timeQ2DEMAND CURVEIn economics a demand curve is a graph depicting the relationship between the price of a certain commodity and the quantity of that commodity that is demanded at that pricQ3LAW OF DEMANDIn microeconomics the law of demand states that conditional on all else being equal as the price of a good increases quantity demanded decreases conversely as the price of a good decreases quantity demanded increasesQ4ASSUMPTION of LAW OF DEMAND(i)No change in price of related commodities(ii) No change in income of the consumer(iii) No change in taste and preferences customs habit and fashion of the consumer( No expectation regarding future change in priceQ5MARKET DEMAND SCHEDULEIn economics a market demand schedule is a tabulation of the quantity of a good that all consumers in a market will purchase at a

given price At any given price the corresponding value on the demand schedule is the sum of all consumersrsquo quantities demanded at that priceQ6INDIVIDUAL DEMAND SCHEDULEIndividual demand schedule refers to a tabular statement showing various quantities of a commodity that a consumer is willing to buy at various levels of price during a given period of timeQ7 FACTORS AFFECTING INDIVIDUAL DEMAND FOR A COMMODITY

The factors that influence a consumerrsquos decision to purchase a commodity are also known as determinants of demand The following factors affect the individual demand for a commodity1 price of the commodity2 price of related goods3 income of buyer of the commodity4 tastes and preferences of the buyer1 Price of the CommodityYou must have observed that when price of a commodity falls you tend to buy more of it and when its price rises you tend to buy less of it when all other factors remain constant (lsquoother things remaining the samersquo) In other words other things remaining the same there is an inverse relationship between the price of a commodity and its quantity demanded by its buyers This statement is in accordance with law of demand which you will study in the later part of this lesson Price of a commodity and its quantity demanded by its buyers are inversely related only when lsquoother things remain the samersquo So lsquoother things remaining the samersquo is an assumption when we study the effect of changes in the price of a commodity on its quantity demanded2 Price of Related goodsA consumer may demand a particular good But while buying that good heshe also asks the price of its related goods Related goods can be of two types-(i) Substitute goods(ii) Complementary goods While purchasing a good prices of its substitutes and complements do affect its quantity purchased(i) Price of Substitute Goods Substitute goods are those goods which can easily be used in place of one another for satisfaction of a particular want like tea and coffee An increase in price of substitute good leads to an increase in demand for the given commodity and a decrease in price of substitute good leads to a decrease in demand for the given commodity It means demand for a given commodity is directly affected by change in price of substitute goods For example if price of coffee increases the demand for tea will rise as tea will become relatively cheaper in comparison to coffee(ii) Price of Complementary goods Complementary goods are those goods which are used together to satisfy a particular want like car and petrol An increase in the price of complementary goods leads to a decrease in demand for the given commodity and a decrease in the price of complementary goods leads to an increase in demand for the given commodity For example if price of petrol falls then the demand for cars will increase as it will be relatively cheaper to use both the goods together So demand for a given commodity is inversely affected by change in price of complementary goods3 Income of the Buyer of CommodityDemand for a commodity is also affected by income of its buyer However the effect of change in income on demand depends on the nature of the commodity under consideration In case of some goods like full cream milk fine quality of rice (Basmati rice) etc demand for these commodities increases when income of the buyer increases and

demand for these commodities decreases when income of the buyer decreases Such goods whose demand increases with the increase in income of the buyer are called normal goods But there are some goods like coarse rice toned milk etc whose demand decreases when income of buyer increases and their demand increases when income of the buyer decreases Such goods whose demand decreases with the increase in income of the buyer are called inferior goods Suppose a consumer buys 10 Kgs of rice whose price is ` 25 per Kg He cannot afford to buy better quality of rice because the price of such rice is ` 50 per Kg The consumer is spending ` 250 per month on the purchase of rice Now if income of the consumer increases and he can afford ` 350 on purchase of 10 Kg of rice Now he can afford to buy some quantity of rice say 6 Kgs whose price is ` 25 per Kg and may buy 4 Kgs of rice whose price is ` 50 per Kg Thus he will buy 10 Kgs of rice by spending ` 350 per month Therefore we may conclude that demand for normal goods is directly related to the income of the buyer but demand for inferior goods is inversely related to the income of the buyer4 Tastes and Preferences of the BuyerThe demand for a commodity is also affected by the tastes and preferences of the buyers They include change in fashion customs habits etc Those commodities are preferred by the consumers which are in fashion So demand for those commodities rises which are in fashion On the other hand if a commodity goes out of the fashion its demand falls because no consumer will like to buy it(5) Number of Buyers in the Market(Population)Increase in population raises the market demand whereas decrease in population reduces the market demand for a commodity Not only the size of population but its composition like age (ratio of males females children and old people in population) also affects the demand for a commodity It is because of needs of children young old male and female population differs(6) Distribution of Income and WealthIf the distribution of income and wealth is more in favour of the rich demand for the commodities preferred by the rich such as comforts and luxuries is likely to be higher On the other hand if the distribution of income and wealth is more in favour of poor demand for commodities preferred by the poor such as necessities will be more(7) Season and Weather ConditionsThis is generally observed that the demand for woolens increases during winter whereas demand for ice creams and cold drinks increases during summer Similarly market demand for umbrellas rain coats increases during rainy seasonQ8 REASONS FOR OPERATION OF LAW OF DEMAND WHY DEMAND CURVE SLOPES DOWNWARDNow we will try to explain why does a consumer purchase more quantity of a commodity at a lower price and less of it at a higher price or why does the law of demand operate ie why does the demand curve slope downwards from left to right The main reasons for operation of law of demand are1 Law of Diminishing Marginal UtilityAs you have studied earlier law of diminishing marginal utility states that as we consume more and more units of a commodity the utility derived from each successive unit goes on decreasing The consumer will be ready to pay more for those units which provide him more utility and less for those which provide him less utility It implies that he will purchase more only when the price of the commodity falls2 Income Effect

When price of a commodity falls purchasing power or real income of the consumer increases which enables him to purchase more quantity of the commodity with the same money income Let us take an example Suppose you buy 4 ice creams when price of each ice cream is ` 25 If price of ice creams falls to ` 20 then with same money income you can buy 5 ice creams now3 Substitution EffectWhen price of a commodity falls it becomes comparatively cheaper as compared to its substitutes (although price of substitutes has not been changed) This will lead to rise in demand for the given commodity For example if coke and Pepsi both are sold at ` 10 each and price of coke falls Now coke has become relatively cheaper and will be substituted for Pepsi It will lead to rise in demand for coke4 Change in Number of BuyersWhen price of a commodity falls some old buyers may demand more of the commodity at the reduced price and some new buyers may also start buying this commodity who were not in a position to buy it earlier due to higher price This will lead to increase in number of buyers when price of the commodity falls As a result demand for the commodity rises when its price falls5 Diverse Uses of a CommoditySome commodities have diverse uses like milk It can be used for drinking for sweet preparation for ice cream preparation etc If price of milk rises its use may be restricted to important purpose only This will lead to reduction in demand for other less important uses When price of milk falls it can be put to other uses also leading to rise n demand for itQ9 EXCEPTIONS TO THE LAW OF DEMANDYou have studied in law of demand that a buyer is willing to buy more quantity of a commodity at a lower price and less of it at a higher price But in certain circumstances a rise in price may lead to rise in demand These circumstances are called Exceptions to the Law of Demand Some important exceptions are1 Giffen GoodsGiffen goods are special type of inferior goods in which negative income effect is stronger than negative substitution effect Giffen goods do not follow law of demand as their demand rises when their price rises Examples of Giffen goods are jowar and bajra etc2 Status Symbol GoodsSome goods are used by rich people as status symbols eg diamonds gold jewellary etc The higher the price the higher will be the demand for these goods When price of such goods falls these goods are no longer looked at as status symbol goods and tehrefore therir demand falls3 NecessitiesCommodities such as medicines salt wheat etc do not follow law of demandbecause we have to purchase them in minimum required quantity whatever their price may be4 Goods Expected to be ScarceWhen the buyers expect a scarcity of a particular good in near future they start buying more and more of that good even if their prices are rising For example during war famines etc people tend to buy more of some goods even at higher prices due to fear of their scarcity in near future

Political Science

Constitution of India-The

Preamble

The preamble-

Preamble-

The preamble is the most precious part of the constitution We the people of India having solemnly resolved to constitute India into a Sovereign Socialist Secular Democratic Republic and to secure to all its citizensA preamble is an introductory and expressionary statement in a document that explains the documents purpose and underlying philosophy When applied to the opening paragraphs of a statute it may recite historical facts pertinent to the subject of the statuteNature and purpose of the constitution-Purpose of the Constitution dictates permanent framework of the government to form a more perfect union to establish justice and ensure peace of thenationconstitution provide principles how the government can run itself following the rules and laws written in the constitution of each state keeps them balanced

Answer the following questions-

1 What is preambleA preamble is an introductory and expressionary statement in a document that explains the documents purpose and underlying philosophy2 What is the nature and

purpose of the constitutionConstitution dictatespermanent framework of the government to form a more perfect union to establish justice and ensure peace of the nation

Homework-Learn

Accounts Compatibilty mode

1MEANING OF PARTNERSHIPPartnership is a form of business organisation where two or more persons join hands to run a business They share the profits and losses according to the agreement amongst them According to the Indian Partnership Act 1932 ldquoPartnership is relation between persons who have agreed to share profits of a business carried on by all or any one of them acting for allrdquo For example one of your friends has passed class XII from National Institute of Open Schooling (NIOS) and wants to start a business Heshe approaches you to join in this venture Heshe wants you to contribute some money and participate in the business activities Both of you if join hands constitute a partnership2CHARACTERISTICS1048698 Agreement A partnership is formed by an agreement The agreement may be either oral or in writing It defines the relationship between the persons who agree to carry on business It may contain the terms of sharing profit and the capital to be invested by each partner etc The written agreement is known as partnership deed1048698 Number of persons There must be at least two persons to form a partnership

The maximum number of partners in a partnership firm can be 50 according toCompanies Act 20131048698 Business The Partnership is formed to carry on business with a purpose of earning profits The business should be lawful Thus if two or more persons agree to carry on unlawful activities it will not be termed as partnership1048698 Sharing Profits The partners agree to share profits in the agreed ratio In caseof loss all the partners have to bear it in the same agreed profit sharing ratio10486981048698Mutual Agency Every partner is an agent of the other partners Every partner can bind the firm and all other partners by hisher acts Each partner will be responsible and liable for the acts of all other partners10486981048698Unlimited liability The liability of each partner except that of a minor is unlimited Their liability extends to their personal assets also If the assets of the firm are insufficient to pay off its debts the partnersrsquo personal property can be used to satisfy the claim of the creditors of the partnership firm10486981048698Management All the partners have a right to mange the business However they may authorize one or more partners to manage the affairs of the business on their behalf10486981048698Transferability of Share No partner can transfer hisher share to any one including hisher family member without the consent of all other partners3PARTNERSHIP DEEDAgreement forms the basis of partnership The written form of the agreement is which a document of partnership is It contains terms and conditions regarding the conduct of the business It also explains relationship between the partners This document is called partnership deed Every firm can frame its own partnership deed in which the rights duties and liabilities of the partners are stated in detail It helps in settling the disputes arising among the partners during the general conduct of business 4CONTENTS OF PARTNERSHIP DEEDThe partnership deed generally contains the following (i) Name and address of the partnership firm(ii) Nature and objectives of the business(iii) Name and address of each partner(iv) Ratio in which profits is to be shared(v) Capital contribution by each partner(vi) Rate of Interest on capital if allowed(vii) Salary or any other remuneration to partners if allowed(viii) Rate of interest on loans and advances by a partner to the firm(ix) Drawings of partners and interest thereon if any(x) Method of valuation of goodwill and revaluation of assets and liabilities on the reconstitution of the partnership ie on the admission retirement or death of a partner(xi) Settlement of disputes by arbitration(xii) Settlement of accounts at the time of retirement or death of a partner5IN ABSENCE OF PARTNERSHIP DEEDThe partnership deed lays down the terms and conditions of partnership in regard to rights duties and obligations of the partners In the absence of partnership deed there may arise a controversy on certain issues like profit sharing ratio interest on

capital interest on drawings interest on loan and salary of the partners In such cases the provisions of the Indian Partnership Act becomes applicableSome of the Issues are(i) Distribution of Profit Partners are entitled to share profits equally(ii) Interest on Capital Interest on capital is not allowed(iii) Interest on Drawings No interest on drawing of the partners is to be charged(iv) Interest on Partnerrsquos Loan A Partner is allowed interest 6 per annum on the amount of loan given to the firm by himher(v) Salary and Commission to Partner A partner is not entitled to anysalary or commission or any other remuneration for managing the business

History TOPIC-TOWARDS INDEPENDENCE AND PARTITION THE LAST PHASE (1935-1947)

SUB TOPIC-IMPORTANT POLITICAL DEVELOPMENTS ndash GROWTH OF SOCIAL IDEAS

Socialism is a political social and economic philosophyLike in other parts of the world the Russian revolution of 1917 served as a great inspiration for revolutionaries in India who at that time were engaged in the struggle for liberation from British ruleSocialist ideas led to the formation of communist party of IndiaJAWAHARLAL NEHRU Among the early Congress leaders Jawaharlal Nehru was very much impressed and influenced by the Socialist ideas He also learnt about the Economic activities of the Soviet Union after the Bolshevic Revolution 1917 He made full use of them in IndiaThe election of Jawaharlal Nehru and Subhas Chandra Bose showed the Left wing tendency within CongressJawaharlal Nehru demanded economic freedom along with political freedom of the people in order to end the exploitation of masses

Nehrus working committee included three socialists leaders The Lucknow session was a landmark in the evolution of socialist ideas of the congressSUBHAS CHANDRA BOSE ndash Subhas Chandra Bose had socialist leaning Both Jawaharlal Nehru and Subhas Chandra Bose were known as leftist Congress men Later on National Congress divided into Leftist and rightist campCONGRESS SOCIALIST Within the Congress some leaders formed the Congress Socialist partyPattavi Sitaramyya Sardar Patel Rajendra Prasad had hostile attitude towards the Congress Socialist partyJawaharlals attitude was hesitant

1 QUESTION ndash Mention name of two Congress leaders who had socialist leaning

1ANSWER ndash Subhas Chandra Bose and Jawaharlal Nehru2QUESTION- In which session of the congress Jawaharlal elaborated his Socialist ideas2 ANSWER ndash Lucknow and Faizpur Session in December 1935 and 19363QUESTION ndash Why Congress was sharply divided into leftist and rightist camp 3ANSWER ndash Subhas Chandra Bosersquos attempt to seek re election for congress presidentship in 1939sharply divided the National Congress into Leftist and Rightist camp4 QUESTION ndash Who was MN Roy 4 ANSWER ndash Manabendra Roy first formed the Communist Party of India outside the country at Tashkent in 19205QUESTION ndash Who formed the Congress Socialist Party within the Congress5 ANSWER ndash Jaya Prakash Narayan Achyut Patwardhan Acharya Narendra Dev Ram Mohan Lohia Aruna Asaf Ali6QUESTION ndash When was the Congress Socialist Party formed What was its object6 ANSWER ndash 1934The Congress Socialist Party sought to work out socialist programme through the Congress They joined hands with the Congress and wanted to carry

Subhas Chandra Bose being expelled from the congress after the Tripuri rift he formed Forward BlockThere were basic differences between the Congress Socialists and the communistsTRADE UNION ACTIVITIES Maximum working class people lived in Bombay and Calcutta The working and living conditions of those workers were very miserable In this situation Shasipada Banerjee NM Lokhande protested against the oppression of the working class peopleThe first Trade Union Madras Labour Union was formed in 1918 by BP WadiaIndustrial strikes took place in Kanpur Calcutta Madras Jamshedpur and Ahmedabad AITUC was formed in Bombay in 1927 The growth of Trade union among the workers was slow because of the fear of the dismissal of the jobIn the mean time the Moderates as well as Communists left AITUC and formed separate organization

on National struggle with the help of workers and peasant class of the society7 QUESTION ndash What was the name of the party founded by Subhas Chandra Bose7 ANSWER- Forward Block8QUESTION ndash Who was Shasipada Banerjee8 ANSWER ndash Shasipada Banerjee was a radical Brahmo He founded a working menrsquos club to protest against exploitation of the British rulers towards the working class of India9 QUESTION ndash What was the weekly published by NM Lokhande9ANSWER- Dinabandhu10 QUESTION ndash Who founded Bombay Mill-Hands Association and in which year10 ANSWER- NM Lokhande in189011 QUESTION- Who was BP WadiaANSWER- BPWadia was the founder of Madras Labour Union in191812 QUESTION- What was the name of the first labour union of India12 ANSWER- Madras Labour Union13 QUESTION Who founded the Majur Mahajan 13 ANSWER GANDHIJI14 QUESTION What was the full form of AITUC When it was formed14 ANSWER All India Trade Union Congressin 192715QUESTION Who formed the Red Trade Union Congress and in which year15ANSWER The Communists formed the Red Trade Union Congress16 QUESTION What do you mean by Socialism16 ANSWER Socialism describes any political and economic theory that says the community rather than individuals should own and manage property and natural resources

Subject Eng Literature (The Tempest ndash William Shakespeare) Topic Act III Scene 3 Lines 1 to 52 (Line 52 ndash Brother my lord the Duke Stand to and do as we) Date 13th April 2020 (4th Period)

[Students should read the original play and also the paraphrase given in the school prescribed textbook]Summary Questions amp Answers

o Alonso Sebastian Antonio Gonzalo Adrian Francisco and others wandered about the island in search of Ferdinand and gets tired and hungry of the toil and at the same time gives up all hope of finding him

o Antonio and Sebastian are happy that Alonso is out of hope and decide to make another attempt on his life that night when being so tired they will be sleeping soundly

o Suddenly a solemn and strange music is heard in the air and several strange shapes enter bringing in a banquet These strange shapes then dance round it with gestures of salutation and then inviting the King to eat they depart

o Seeing this strange scene all are inclined to believe the tales told by travelers that there truly are ldquounicornsrdquo and ldquothe phoenixrsquo thronerdquo

1 ALONSO What harmony is this My good friends hark (L18-27)

GONZALO Marvellous sweet music

[Enter several strange shapes bringing in a banquet

they dance about it with gentle actions of salutation

and inviting the King and his companions to eat they depart]ALONSO Give us kind keepers heavens What were theseSEBASTIAN A living drollery Now I will believe

That there are unicorns that in Arabia

There is one tree the phoenixrsquo throne one phoenix

At this hour reigning thereANTONIO Ill believe both

And what does else want credit come to me

And Ill be sworn rsquotis true Travellers neer did lie

Though fools at home condemn rsquoem

(a) How did Prospero present an amazing spectacle before Alonso and his companions

Using his magic powers Prospero ordered strange shapes to lay a banquet before Alonso and his companions The shapes brought several dishes with tasty eatables in them They placed the dishes on a table before Alonso and his companions Then the strange shapes began to dance gracefully around the banquet While dancing they made gestures inviting them to eat the food Then suddenly the shapes disappeared(b) Who were the guests at the strange banquet Describe the lsquoliving drolleryrsquo

Alonso Sebastian Antonio Gonzalo Adrian and Francisco were the guests at the strange banquet

The term ldquoliving drolleryrdquo refers to live entertainment show In this context when Alonso the King of Naples Sebastian his brother Antonio the treacherous brother of Prospero Gonzalo the kind and loyal councillor to the King Adrian and Francisco came to the island they were hungry and weary in their spirits They heard a solemn and strange music They were shocked to see several strange shapes bringing in a banquet and these shapes danced about it with gentle action of salutation inviting the King and his companions to eat After this Sebastian described this show as lsquoliving drolleryrsquo(c) What is lsquophoenixrsquo What are lsquoUnicornsrdquo

The term lsquophoenixrsquo refers to a mythical Arabian bird which lived alone and perched on a solitary tree After one hundred years it expired in flames and rose again from its own ashes

lsquoUnicornsrsquo refers to the mythological four-footed beasts having horns in the centre of their foreheads When the horns are ground into powder the powder was believed to be

an aphrodisiac(d) How does Sebastian explain the puppet show OR Why does the speaker now believe in unicorns and phoenix

Sebastian finds several strange shapes bringing in the banquet They invite the king and his party for dinner and soon depart He tells that if such a strange sight can be a reality there is nothing incredible in the world and from the present moment he will believe anything He says that it is a strange dumb show enacted not by puppets but by living beings It is stranger than a travellerrsquos tale Seeing such a thing

before his own eyes he will no longer disbelieve the story about unicorns and phoenix(e) How do the other characters present respond to this living drollery

At the sight of the lsquoliving drolleryrsquo like Sebastian Gonzalo and Antonio too acted strangely Antonio told that he too now believes in unicorns and phoenix and anything else that seems to be incredible He too now believes in travellersrsquo tales Gonzalo told that if he would report those happenings in Naples nobody will believe him He considers that those gentle shapes were gentler in manner in comparison to the living beings Alonso was at first sight suspicious and told them that those strange shapes conveyed their meaning in expressive gestures when they seemed to lack speech by their movements and sounds Francisco was amazed at their mysterious disappearance

2 ALONSO Not I

(Line 43-52)GONZALO Faith sir you need not fear When we

were boysWho would believe that there were mountaineers

Dewlapped like bulls whose throats had hanging at rsquoem

Wallets of flesh Or that there were such men

Whose heads stood in their breasts Which now we find

Each putter-out of five for one will bring us

Good warrant ofALONSO I will stand to and feed

Although my lastmdashno matter since I feel

The best is past Brother my lord the Duke

Stand to and do as we

(a) How does Alonso respond at the spectacle of the shapes which were sent to them at the instruction of Prospero

After seeing the strange sight of appearing and disappearing of the shapes sent by Prospero to arrange a banquet for them Alonso says that his surprise at having seen those creatures is infinite and he is fully justified in feeling so much surprise He thinks that their shapes their gestures and the sounds they made were indeed amazing Although they do not possess the gift of speech yet they were able to convey their

thoughts by means of their gestures only

(b) What does Prospero say about the views expressed by Alonso regarding the shapes What does Francisco think about the shapesAfter hearing Alonsorsquos views about the shapes Prospero says that this manrsquos praise of the spirits is rather hasty He means to say that Alonso has shown great haste in reaching the conclusion about the shapes Francisco is amazed to see that those shapes disappeared in a mysterious way(c) What does Sebastian ask Alonso to doSebastian tells Alonso that the shapes having disappeared should not matter to them because they have left the eatables behind He asks Alonso to enjoy eating as they are extremely hungry but the king does not accept his offer of enjoying the dishes(d) How does Gonzalo try to dispel Alonsorsquos fear of those strange shapes What kind of references does he give to AlonsoGonzalo says that those who have travelled abroad have reported seeing even stranger sights than these shapes that Alonso and his companions have beheld Hence there is no reason to feel afraid of these shapes Gonzalo further adds that in his younger days he had heard strange stories from travelers and Alonso might have heard similar stories For instance it was said that there existed a certain race of

human beings who had huge lumps of flesh hanging at their throats and who therefore resembled bulls Then Gonzalo tells about a race of human beings whose heads were located at their breasts Gonzalo says that such stories were not believed by most people in those days but now-a-days these stories have become common(e) Explain the following lsquoEach putter-out of five for onersquoEnglish travellers often insured their trips with London brokers Those that went on foreign travels those days used to deposit a certain amount with some firm or company in London before their departure If the travelers failed to return the money was forfeited by the company with which it had been deposited But this money was repaid five-fold if the travelers returned safe and sound In this way a traveler stood a great chance of recovering the entire cost of his

travels(f) Give the explanatory meanings of the following expressions in the context of the above extract (i) Dewlapped (ii) Wallets of flesh

(iii) Putter-out(i) Dewlapped having big lumps of flesh at the necks(ii) Wallets of flesh large masses of flesh looking like bags(iii) Putter-out to invest money before commencing the travel

  • General methods of preparation of hydrogen
  • Chapter Dimensional Analysis (Summary)
    • Properties of Charge
Page 36:   · Web viewSubject. Topic. Summary. Execution. Hindi. व्याकरण. शरीरके अंगो के नाम लिखिए. 1) आँख 2) नाक 3

(such as raw materials capital labour energy and so on) from its environment transforms them into goods and services and sends them back to the environment

Fig 1 Business Environment Relationship

complex in nature as the elements keep on changing example economic technological and other forces changes in demand for a product and service

Dynamic-Business environment is not static it keeps on changing

Uncertainty- Itrsquos very difficult to predict future events such as technology and fashion which occur fast and frequently

Economics Basic Economic ConceptsSub topic

Microeconomics and

Macroeconomics

Welcome to the new sessiontoday we are going to start the first chapter of Class XI The name of the chapter that we are going to start is Basic Economic concepts

Now Economics covers the study of human activities Human activities are those activities which are performed by humans to satisfy their wants

Thus Human wants are unlimited and therefore economic activities such as production exchange and consumption are needed in order to satisfy those wants

The study of economics is divided largely in two parts which areMicroeconomics and Macroeconomics

SUBJECT- MATTER OF ECONOMICS

MICROECONOMICS MACROECONOMICS

Questions1Who has coined the words micro and macro economics

Answer Ranger Frisch coined the words lsquomicrorsquo and lsquomacrorsquo in 1933 to denote the two branches of economic theory namely microeconomics and macroeconomics

2What is microeconomicsAnswer It is the study of behaviour of individual decision ndash making unit such as consumers firms etc

3 What is macroeconomicsAnswer Macroeonomics is the study of overall economic phenomena like employment national income etc

4 What is the importance of microeconomicsAnswer

Microeconomics helps in formulating economic policies which enhance productive efficiency and results in greater social welfare

It helps the government in formulating correct price policies

It explains the working of a capitalistic economy where individual units(producers and consumers ) are free to take their own decision

Micro means a small part in

microeconomics we do not study the whole economy Hence we study an individual consumer and his or her choices and a producer and his or her profit maximizing decisions in the market Thus it does not mirror what happens in the economy as a whole

Macroeconomics on the other hand studies the economy as a whole It is concerned with aggregate and depicts the entire picture of the economyMacroeconomics deals with the national income aggregate investment aggregate consumption etc

Features of Microeconomics It deals with small

parts of the country Hence it looks at

individual consumers firms and industries

It deals with individual income consumption and savings

It studies the determination of price of any product or factors of production

It deals with the working of market via the price mechanism which is nothing but the determination of price and quantity of a commodity by the forces of demand and supply

Features of Macroeconomics

It deals with the study of the economy as a whole

It is concerned with

5 Give a limitation of microeconomics Microeconomics fails to explain the

functioning of an economy as a whole It cannot explain unemployment illiteracy and other problems prevailing in the country

6 What is the importance of macroeconomics It gives overall view of the growing

complexities of an economic system It provides the basic and logical

framework for formulating appropriate macroeconomic policies (eg for inflation poverty etc )to direct and regulate economy towards desirable goals

7What is the limitation of macroeconomics It ignores structural changes in an

individual unit of the aggregate

8 Differentiate between Microeconomics and Macroeconomics

Microeconomics Macroeconomics

the study of aggregates

National income aggregate savings and aggregate investments are major concepts dealt within macroeconomics style

It studies the determination of general price levels

It investigates into the problem of unemployment and the achievement of employment

It studies the aspect of decision making at the aggregate and national levels

It includes all growth theories whether related to developed or developing economies it also includes the study of economic systems and the working of the economy under different systems

Note Both Micro and macro economics are complementary and should be fully utilized for proper understanding of an economy

1It studies economic aspect of an individual unit2It deals with individual incomeConsumption and savings

3 It facilitates determination of price of any product or factors of production

4 Itrsquos scope is narrow and restricted to individual unit

1It studies the economy as a whole

2It deals with the national income aggregate consumption and aggregate savings3 It facilitates determination of general price level in an economy

4 Itrsquos scope is wide as it deals with economic units on the national level

ACCOUNTS

Introduction to Accounting and Book-keeping

Today I am going to share you the meaning of Accounting and Book-keeping and its related terms bullAccounting bullBook Keeping bullAccountsbullTypes Of Accounts bullAccounting Cycle

bull Meaning of accounting

Ans ) Accounting is the art and science of recording classifying and summarising monetary transactions

bull Meaning of Book-keeping

Ans) Bookkeeping is the art of recording business transactions with the view of having a permanent record of them and showing their effect on wealth

bull Meaning of account

Ans) The term account means a record of

business transactions concern a particular person of firm asset or income or expense It is a summarised record of all transactions which take place in an accounting year

bull Types of accountsPersonal accounts ndash Personal accounts relating

to person and Organisation are known as personal accounts Example Ramrsquos Account ABC amp Co Account etc

Real account - The accounts related to tangible and intangible assets are called real accounts Example Cash Account Furniture Account etc

Nominal account- Accounts related to expenses losses incomes and gains are known as nominal accounts Example Wages Account Salary Account Discount Account etc

bull Accounting cycle Accounting cycle refers to a complete sequence of accounting activities It begins with recording of transactions and ends with the preparation of a balance sheet

Chemistry TopicAtomic Structure

Thomsonrsquos atomic modelThomson (1898) was the first to propose the model of an atomHe proposed that an atom can be regarded as a uniform sphere of positive electricity in which requisite number of electrons are embedded evently to neutralize the positive chargeThis is just like plums embedded in a pudding or seeds evently distributed in red spongy mass of a watermelonThis model of atom is known as ldquoPlum-Pudding modelrdquo or

Q1)What is the fundamental constituents of atomAns Electron Proton and neutrons are the fundamental constituents of atomQ2)What is the value of fundamental unit of electricityAnsThe charge carried by one electron is sad to be the fundamental unit of electricityIts magnitude is 48times10-10esuOr 1602times10-19C Q3)Name the element containing no neutronAnsOrdinary hydrogen atom or protium 1H1

Types of AccountPersonal AccountReal AccountNominal AccountBalance Sheet (opening)

ldquowatermelon modelrdquoThis model could explain the electrical neutrality of an atom but failed to explain the result of scattering experiment carried out by Rutherford in 1911So it was rejected ultimately

Q4)Why is an electron called universal particleAns Itrsquos mass and Charge are independent of its source

EVS Chapter 1 ndash Modes of Existence

Modes of existence When one speaks normally about the mode of existence of some group or individual one refers to their customs their mode of being their ethology their habitat in some way their feeling for a placeDifferent modes of exixtence are ndash

1 Hunting ndashGathering2 Pastoral3 Agricultural4 Industrial

1 Hunting and gathering Hunting and gathering mode of existence is characterized by obtaining food from hunting wild animals including fishing and gathering wild plants From their earliest days the hunter-gatherer diet included various grasses tubers fruits seeds and nuts Lacking the means to kill larger animals they procured meat from smaller game or through scavenging

Societies that rely primarily or exclusively on hunting wild animals fishing and gathering wild fruits berries nuts and vegetables to support their diet are called hunting and gathering societies

At least this used to be practice of human beings before agriculture is invented As their brains evolved hominids developed more intricate knowledge of edible plant life and growth cycles

Q) Write the features of Hunting ndash gathering societiesAns - There are five basic characteristics of hunting and gathering societies

i The primary institution is the family which decides how food is to be shared and how children are to be socialized and which provides for the protection of its members

ii They tend to be small with fewer than fifty members

iii They tend to be nomadic moving to new areas when the current food supply in a given area has been exhausted

iv Members display a high level of interdependence

v Labor division is based on sex men hunt and women gather

Political Science

Introduction to political science

Political science occasionally called politology is a social science which deals with systems of governance and the analysis of political activities political thoughts associated constitutions and political behaviorThe study of political science involves the study of both the

Answer the following questions-1 What is political science

Political science occasionally called politology is a social science which deals with systems of governance and the analysis of political activities political thoughts associated constitutions and political behavior

2 Short notes-

traditional and modern theories of politicsTraditionalClassical political sciencepolitical theory-Traditional political science is the study of politics before Second World War The methodology to study Politics was traditional (legal formaletc) the definition of politics traditional (Politics begins and end with state)area of study (constitution state machinery)was traditionalModern Political scienceModern political theory-Modern Political Theory critically examines the contemporary state of political theory making an assessment of the achievement and limitations of the Behavioural Revolution in its totality and reviews objectively the major paradigms and conceptual frameworks adopted by the disciplineContemporary attempts at the development of an integrated political theory involving the use of both traditional and modern concepts approaches and theories-Around late 1960s several political scientists realized the importance of both the traditional political theory and modern Political theory They began building an integrated theory of politics involving a systematic mixture of traditional and modern studies of politics It was held that the study of a complex and vast field like politics needs both traditional as well as

Classical political theory Modern Political theory

Homework-Learn

modern concepts and approaches for studying itrsquos all aspects

Subject Eng Literature (The Tempest ndash William Shakespeare) Topic Act I Scene 1 Lines 1 to 32 (Line 32 ndash Gonzalo hellip If he be not born to be hanged our case is miserable) Date 13th April 2020 (3rd Period)

[Students should read the original play and also the paraphrase given in the school prescribed textbook]Summary Questions amp Answers

[SUMMARY OF THE ENTIRE SCENE]

o The play starts with the scene of a severe storm at sea Alonso (King of Naples) Sebastian (Alonsorsquos brother) Ferdinand (Alonsorsquos son) Gonzalo Antonio (the usurping Duke of Milan) are in a ship in the midst of the storm

o The mariners are trying their best to control the vessel from running aground and are totally following the orders of their Master the Boatswain They have scant success

o The mariners become extremely unhappy and annoyed when most of the passengers arrive on the deck thereby hampering their effort to save the ship There is serious confrontation between them and the passengers who are part of the Kingrsquos entourage

o The mariners could not save the ship

SUMMING-UP

(i) Vivid description of the scene which gives a realistic description of terror and confusion of a tropical storm

(ii) Shows Shakespearersquos accuracy of knowledge in describing the naval operations and also matters of seamanship

(iii) The opening scene justifies the title ndash The Tempest

UNANSWERED QUESTIONS

(i) The King always travels with his entire fleet including his soldiers Where

(1) GONZALO Nay good be patient (Line 15-26)BOATSWAIN When the sea is Hence What cares these

roarers for the name of the king To cabin silence Trouble us not

GONZALO Good yet remember whom thou has aboardBOATSWAIN None that I more love than myself You are a

councillor if you can command these elements to silence and work

the peace of the present we will not hand a rope more use your authority If you cannot give thanks you have

lived so long and make yourself ready in your cabin for the mischance of the hour if it so hap [To the Mariners]

Cheerly good hearts [To Gonzalo] Out of our way I say

(a) To whom is the boatswain speaking What does he mean by lsquoNone that I more love than myselfrsquo

The Boatswain is speaking to Gonzalo the honest old councilor of the Duke of MilanBy using the words ndash lsquoNone that I love more than I love myselfrsquo means that for the Boatswain nobody is dearer to him than his own life

(b) What were the conditions that made the boatswain react in this way

The Boatswain reacts in this way because the storm is at sea and Alonso King of Naples Sebastian his brother Ferdinand his son Gonzalo Antonio the usurping Duke of Milan on board are in distress and in panic Thus they have rushed to the deck interrupting the work of the mariners

(c) What hope does Gonzalo take from the attitude of the boatswain

The insolent and authoritative attitude of Boatswain makes Gonzalo feel comforted He tells that there are no signs that the Boatswain will be drowned But his facial appearance and attitude shows that he is destined to die on land by hanging which in effect means that all on board will be saved Otherwise all the persons on board are doomed

(d) How can they lsquomake yourself ready in your cabinrsquo For what were they asked to make ready themselves

In order to make themselves ready in their cabin the

were the other ships

(ii) Why was the ship in that area Where was it coming from or going where

(iii) The ship broke apart What happened to those who were in the ship

passengers on board must prepare for death which they will possibly soon have to meetThey can retire to their cabins and offer prayers to the Almighty to save them from drowning

(e) What does the boatswain say when he is asked to be patient What does he order to the royal party

When the boatswain is asked to be patient and remain calm he says that he will be patient only when the storm will be over and the sea will be calm but as long as the storm blows and there is danger to the ship he cannot think of being patient He orders the royal party to go to the cabin and leave the mariners to their work

(2) GONZALO I have great comfort from this fellow (Line 27-36)

Methinks he hath no drowning mark upon him his complexion is perfect

gallows Stand fast good Fate to his hanging Make the rope of his destiny our cable for our own doth little advantage If he be not born to be hanged our case is miserable

(a) Why does Gonzalo regard the Boatswain in the midst of danger

In the midst of danger Gonzalo regards the boatswain because he feels that the Boatswain is a source of comfort and is bent upon to do his work sincerely which in this case is saving the ship and its passengers from the severest of raging storm

(b) What reasons does Gonzalo give when he says that none in the ship will die of drowning

Gonzalo is almost sure that none in the ship will die by drowning His says that there is no mark on the face of the boatswain that indicates that he will die by drowning On the other hand the lines on his face are strong indications that he will be hanged to death Therefore there shall be no danger of the shiprsquos sinking

(c) Explain the following ldquoStand fast good Fate to his hanging Make the rope of his destiny our cable for our own doth little advantage If he be not born to be hanged our case is miserablerdquo

The stated lines mean that if the will of destiny is to be carried out then the ship will not get wrecked and all the passengers will be saved The safety of the passengers therefore depends upon the will of fate being carried out in the case of the boatswain If however the boatswain is not to die by hanging then the passengers are also very unsafe because in that case the ship is likely to sink

(d) What order does the Boatswain give to the sailors

when he re-enters What does he say about the crying of the fellows inside the cabin

The boatswain orders the sailors to bring the topmast lower and bring the ship close to a stationary position with the help of the main sail He says that the fellows inside the cabin are moaning and crying in their distress louder than his voice and louder even than the roaring of the storm

Class XII (ScienceCommerceHumanities) Subject Topic Summary Execution

Computer Science

PropositionalLogic

Propositional logic is a procedure to provide reasoning through statementProposition A ststement that results in True or False is said to be proposition There are two types of propositionSimple proposition amp compound propositionSimple proposioton A simple proposition is one that is not a part of any other proposition Such sentential form of proposition is symbolized with english letters in short For example Ram is a claver student (TrueFalse)Where do you live (Not in True or False)Grapes are sweet (TrueFalse)It rains today (TrueFalse)Here we can see some statements anwer would be true or false but some staements answer can not give in terms of true or false Thus the sentences which can be answered in true or false are known as simple propositionAssigning propositon to a variableThe general syntax to assign propostion to a variable is as followsVariable = Simple propositonFor example A=Ram is a clever studentB= Grapes are sweetC= it rains todayCompound proposition

helliphellipto be continued in next classhelliphellipMath Relation Relation If A and B are two non-empty sets

then a relation R from A to B is a subset of AxB If R A x B and (a b) R then we say that a sube isinis related to b by the relation R written as aRbeg Let A be the set of students of class XII and B be the set of students of class XI Then some of the examples of relation from A to B arei) (a b) AXB a is brother of bisinii) (a b) AXB age of a is more than age of isinb Types of relation In this section we would like to study different types of relations We know that a relation in a set A is a subset of A times A Thus the empty set φ and A times A are two extreme relations For illustration consider a relation R in the set A = 1 2 3 4 given by R = (a b) a ndash b = 10 This is the empty set as no pair (a b) satisfies the condition a ndash b = 10 Similarly R = (a b) | a ndash b | ge 0 is the whole primeset A times A as all pairs (a b) in A times A satisfy | a ndash

Example 1 Let A be the set of all students of a boys school Show that the relation R in A given by R = (a b) a is sister of b is the empty relation and R = (a b) the primedifference between heights of a and b is less than 3 meters is the universal relationSolution Since the school is boys school no student of the school can be sister of any student of the school Hence R = φ showing that R is the empty relation It is also obvious that the difference between heights of any two students of the school has to be less than 3 meters This shows that R = A times A is primethe universal relation Example 2 Show that the relation R in the set 1 2 3 given by R = (1 1) (2 2) (3 3) (1 2) (2 3) is reflexive

b | ge 0 These two extreme examples lead us to the following definitionsDefinition 1 A relation R in a set A is called empty relation if no element of A isrelated to any element of A ie R = φ A times AsubDefinition 2 A relation R in a set A is called universal relation if each element of A is related to every element of A ie R = A times A Both the empty relation and the universal relation are some times called trivial relation Definition 3 A relation R in a set A is called(i) reflexive if (a a) R for every a Aisin isin(ii) symmetric if (a1 a2) R implies that (aisin 2a1)

R for all aisin 1 a2 Aisin(iii) transitive if (a1 a2) R and (aisin 2 a3) R isinimplies that (a1 a3) R for all aisin 1 a2 a3 AisinDefinition 4 A relation R in a set A is said to be an equivalence relation if R is reflexive symmetric and transitive

but neither symmetric nor transitiveSolution R is reflexive since (1 1) (2 2) and (3 3) lie in R Also R is not symmetric as (1 2) R but (2 1) isin notinR Similarly R is not transitive as (1 2) R and (2 3) R but (1 3) R isin isin notinExample 3 Show that the relation R in the set Z of integers given byR = (a b) 2 divides a ndash b is an equivalence relationSolution R is reflexive as 2 divides (a ndash a) for all a Z isinFurther if (a b) R then 2 divides a isinndash b Therefore 2 divides b ndash a Hence (b a) R which shows that R is isinsymmetric Similarly if (a b) R and (b c) R isin isinthen a ndash b and b ndash c are divisible by 2 Now a ndash c = (a ndash b) + (b ndash c) is even (Why) So (a ndash c) is divisible by 2 This shows that R is transitive Thus R is an equivalence relation in ZExample 4 Let L be the set of all lines in a plane and R be the relation in L defined as R = (L1 L2) L1 is perpendicular to L2 Show that R is symmetric but neither reflexive nor transitiveSolution R is not reflexive as a line L1 can not be perpendicular to itself ie (L1 L1) R notinR is symmetric as (L1 L2) Risin

L1 is perpendicular to L2rArr L2 is perpendicular to L1rArr (L2 L1) RrArr isin

R is not transitive Indeed if L1 is perpendicular to L2 and L2 is perpendicular to L3 then L1 can never be perpendicular to L3 In fact L1 is parallel to L3 ie (L1 L2) R isin(L2 L3) R but (L1 L3) Risin notin

Chemistry Solid state Characteristics if Solids(i)The particles are locked in fixed positions they are unable to change their relative positions and this brings a definite shape and volume of a solid(ii)In a solid the constituent particles are held by strong forces of attractionThe forces of attraction may be bonding or non bonding(iii)The constituent particles in a solid pack together as closely as possibleoccupying most of the available space within the solidThus the empty space in a solid is very smallThis makes a solid highly rigid and nearly incompressibleThis also explains why a solid has high density and exhibits slow diffusionClassification of Solids

Q1)Define Crystalline solids AnsA Solid that has a definite geometrical shape and a sharp melting pointand whose constituent particles (atomsmolecules or ions) are arranged in a long range order of definite pattern extending throughout the solid is called a crystalline solidExNaClQ2)Define Amorphous solids AnsA solid that does not have a definite shape and a sharp melting pointand whose constituent particles (atomsmolecules or ions) are not arranged in a definite pattern is called an amorphoussolid

Crystalline solidsAmorphous solids

ExGlassRubberQ3)Classify Crystalline Solids Crystalline Solids

Physics Coloumbrsquos Law (Summary)

Before Going Into Coloumbrsquos Law We Will First Learn What is Charge Properties of Charge and Always remember that charge is quantized ie a body always have static charge of magnitude equal to some integral multiple of fundamental electronic charge e= 16 x 10- 19 C

Charge is the property of matter that causes it to produce and experience electrical and magnetic effects The study of the electrical charges at rest is called electrostatics When both electrical and magnetic effects are present the interaction between charges is referred to as electromagnetic

There exist two types of charges in nature positive and negative Like charges repel and unlike charges attract each other

The type of charge on an electron is negative The charge of a proton is the same as that of an electron but with a positive sign In an atom the number of electrons and the number of protons are equal The atom is therefore electrically neutral If one or more electrons are added to it it becomes negatively charged and is designated as negative ion However if one or more electrons are removed from an atom it becomes positively charged and is called a positive ion

The excess or deficiency of electrons in a body gives the concept of charge If there is an excess of electrons in a body it is negatively charged And if there is deficiency of electrons the body becomes positively charged Whenever addition or removal of electrons takes places the body acquires a charge

The SI Unit of charge is coulomb (C) In SI units the current is a fundamental quantity having a unit of ampere (A) The unit of charge is defined in terms of the unit of current Thus one coulomb is the charge transferred in one second across the section of a wire carrying a

Ionic SolidsMetallicSolids

Molecular Solids

current of one ampere

As q = It we have1 C = (1 A) (1 s)

The dimensions of charge are [A T]

Properties of Charge

(1) Quantization of Charge Electric charge can have only discrete values rather than any value That is charge is quantized The smallest discrete value of charge that can exist in nature is the charge on an electron given as

e = plusmn 16 x 10- 19 C

This is the charge attained by an electron and a protonA charge q must be an integral multiple of this basic unit That is

Q = plusmn ne where n = 1 2 hellip

Charge on a body can never be (frac12)e (23)e or 57e etcWhen we rub a glass rod with silk some electrons are transferred from the rod to the silk The rod becomes positively charged The silk becomes negatively charged The coulomb is a very large amount of charge A typical charge acquired by a rubbed body is 10 - 8 C

Biology Reproduction in organisms

Welcome to this new session 2020-21Today in this first chapter we mainly discuss about reproduction types needs and life span of some organismsWe also discuss about difference between sexual and asexual reproduction

Q1 What is reproductionReproduction is defined as a biological processin which an organism gives rise to young onessimilar to itselfQ2 What are the needs of reproductionbulli) Reproduction maintain life on earthii) It enables the continuity of the species generation after generationiii) It creates genetic variation among populationsQ3 Define Life span and write some orgnisms life spanbull Life span is the period from birth to

the natural death of an organism- OrganismsLife span1 Butterfly 1 - 2 weeks2 Fruit fly 30 days3Dog 10-13 years4 Rose5-7 years5 Tortoise100-150 years6 Banyan Tree -200 - 250 yearsQ4 Reproduction is of two types in case ofanimals but in case of plants vegetative propagation is also present

Asexual Reproduction Sexual Reproductioni) Always uniparentalii) Gametes are not involvediii) Only mitotic division involvediv) Somatic cells of parents are involvedv) Offsprings are genetically similar to the parents

i) Usually biparentalii) Gametes are involvediii) Meiosis occurs during gametogenesis Mitosis occurs after fertilisationiv) Germ cells of the parents are involvedv) offsprings are genetically different from the parents

COMMERCE BUSINESS ENVIRONMENT

Welcome to the new sessiontoday we are going to start the first chapter of Class XII The name of the chapter is Business Environment

Already many of you have got some idea about the word business environment form the first chapter of business studies in class XI

In todayrsquos world every business enterprise is a part of the society It exists and operates in association with various groups in society such as customers suppliers competitors banks and financial institutions government agencies trade unions media and so on All these groups influence the functioning of business in one way or the other They constitute the environment of businessConcept of Business Environment

The term lsquobusiness environmentrsquo refers to the sum total of all individuals institutions and other forces that lie outside a business enterprise but that may influence its functioning and performance

The main features of business environment Totality of External forces General and Specific forces Interrelatedness Complexity Dynamic Uncertainty

Prepare the following questions from todayrsquos assignment

2 What do you mean by business environment

The term lsquobusiness environmentrsquo means the aggregate of all forces factors and institutions which are external to and beyond the control of an individual business enterprise but they may influence its functioning and performance Business environment is the macro framework within which a business firm a micro unit operates It consists of several interrelated and interacting elements

2 Explain the main features of business environment in brief

Totality of External forces-Business environment is the sum total of all things external to a business environment

General and Specific forces-It

Relativity

The Interrelation between business and its environment

The business enterprise is an open system It continuously interacts with its environment It takes inputs (such as raw materials capital labour energy and so on) from its environment transforms them into goods and services and sends them back to the environment

Fig 1 Business Environment Relationship

includes both the forces general forces are the economic social political legal and technological conditions which indirectly influence all business enterprise Specific forces are the investors customers competitors and suppliers which influence individual enterprise directly

Interrelatedness-Different elements of environment are interrelated for an example growing awareness for health care has increased the demand for health foods

Complexity- Business environment id complex in nature as the elements keep on changing example economic technological and other forces changes in demand for a product and service

Dynamic-Business environment is not static it keeps on changing

Uncertainty- Itrsquos very difficult to predict future events such as technology and fashion which occur fast and frequently

Business Studies

Human Resources Management

Human resource of an organisation are the aggregate of knowledge skills attitudes of people working in it

The management system which deals with human resources is called human resource management

Features of HRMbullComprehensive functionbullPeople-oriented

Question1) What do you mean by human

resource management Answer) Human resource management may be defined as that field of Management which has to do with planning organising and controlling the functions of procuring developing maintaining and utilising the labour force

bullAction oriented bullPervasive function bullContinuous function

2) Explain the features of HRM in brief

Answer)bullHuman Resource Management is concerned with managing people at work bull Human Resource Management is concerned with employees which bring people and organisations together so that the goals of each are met bullHuman resource management considered every employees as an individual and also promote their satisfaction and growth bull Human resource management is inherent in all organisations and at all levelsbullManagement of human resources are ongoing on never ending process which requires a constant alertness and Awareness of human relations

3) ldquoHR function is said to be pervasiverdquowhy

Answer) Human resource management is required in all organisations whether it is private or government organisations armed forces sports organisations etc It permeatsall the functional areas like production marketing finance research etc This from this feature of human resource management it can be said that it is pervasive in nature

Economics Demand Q1DEFINITION OF DEMANDIn economics demand is the quantity of a good that consumers are willing and able to purchase at various prices during a given period of timeQ2DEMAND CURVEIn economics a demand curve is a graph depicting the relationship between the price of a certain commodity and the quantity of that commodity that is demanded at that pricQ3LAW OF DEMANDIn microeconomics the law of demand states that conditional on all else being equal as the price of a good increases quantity demanded decreases conversely as the price of a good decreases quantity demanded increasesQ4ASSUMPTION of LAW OF DEMAND(i)No change in price of related commodities(ii) No change in income of the consumer(iii) No change in taste and preferences customs habit and fashion of the consumer( No expectation regarding future change in priceQ5MARKET DEMAND SCHEDULEIn economics a market demand schedule is a tabulation of the quantity of a good that all consumers in a market will purchase at a

given price At any given price the corresponding value on the demand schedule is the sum of all consumersrsquo quantities demanded at that priceQ6INDIVIDUAL DEMAND SCHEDULEIndividual demand schedule refers to a tabular statement showing various quantities of a commodity that a consumer is willing to buy at various levels of price during a given period of timeQ7 FACTORS AFFECTING INDIVIDUAL DEMAND FOR A COMMODITY

The factors that influence a consumerrsquos decision to purchase a commodity are also known as determinants of demand The following factors affect the individual demand for a commodity1 price of the commodity2 price of related goods3 income of buyer of the commodity4 tastes and preferences of the buyer1 Price of the CommodityYou must have observed that when price of a commodity falls you tend to buy more of it and when its price rises you tend to buy less of it when all other factors remain constant (lsquoother things remaining the samersquo) In other words other things remaining the same there is an inverse relationship between the price of a commodity and its quantity demanded by its buyers This statement is in accordance with law of demand which you will study in the later part of this lesson Price of a commodity and its quantity demanded by its buyers are inversely related only when lsquoother things remain the samersquo So lsquoother things remaining the samersquo is an assumption when we study the effect of changes in the price of a commodity on its quantity demanded2 Price of Related goodsA consumer may demand a particular good But while buying that good heshe also asks the price of its related goods Related goods can be of two types-(i) Substitute goods(ii) Complementary goods While purchasing a good prices of its substitutes and complements do affect its quantity purchased(i) Price of Substitute Goods Substitute goods are those goods which can easily be used in place of one another for satisfaction of a particular want like tea and coffee An increase in price of substitute good leads to an increase in demand for the given commodity and a decrease in price of substitute good leads to a decrease in demand for the given commodity It means demand for a given commodity is directly affected by change in price of substitute goods For example if price of coffee increases the demand for tea will rise as tea will become relatively cheaper in comparison to coffee(ii) Price of Complementary goods Complementary goods are those goods which are used together to satisfy a particular want like car and petrol An increase in the price of complementary goods leads to a decrease in demand for the given commodity and a decrease in the price of complementary goods leads to an increase in demand for the given commodity For example if price of petrol falls then the demand for cars will increase as it will be relatively cheaper to use both the goods together So demand for a given commodity is inversely affected by change in price of complementary goods3 Income of the Buyer of CommodityDemand for a commodity is also affected by income of its buyer However the effect of change in income on demand depends on the nature of the commodity under consideration In case of some goods like full cream milk fine quality of rice (Basmati rice) etc demand for these commodities increases when income of the buyer increases and

demand for these commodities decreases when income of the buyer decreases Such goods whose demand increases with the increase in income of the buyer are called normal goods But there are some goods like coarse rice toned milk etc whose demand decreases when income of buyer increases and their demand increases when income of the buyer decreases Such goods whose demand decreases with the increase in income of the buyer are called inferior goods Suppose a consumer buys 10 Kgs of rice whose price is ` 25 per Kg He cannot afford to buy better quality of rice because the price of such rice is ` 50 per Kg The consumer is spending ` 250 per month on the purchase of rice Now if income of the consumer increases and he can afford ` 350 on purchase of 10 Kg of rice Now he can afford to buy some quantity of rice say 6 Kgs whose price is ` 25 per Kg and may buy 4 Kgs of rice whose price is ` 50 per Kg Thus he will buy 10 Kgs of rice by spending ` 350 per month Therefore we may conclude that demand for normal goods is directly related to the income of the buyer but demand for inferior goods is inversely related to the income of the buyer4 Tastes and Preferences of the BuyerThe demand for a commodity is also affected by the tastes and preferences of the buyers They include change in fashion customs habits etc Those commodities are preferred by the consumers which are in fashion So demand for those commodities rises which are in fashion On the other hand if a commodity goes out of the fashion its demand falls because no consumer will like to buy it(5) Number of Buyers in the Market(Population)Increase in population raises the market demand whereas decrease in population reduces the market demand for a commodity Not only the size of population but its composition like age (ratio of males females children and old people in population) also affects the demand for a commodity It is because of needs of children young old male and female population differs(6) Distribution of Income and WealthIf the distribution of income and wealth is more in favour of the rich demand for the commodities preferred by the rich such as comforts and luxuries is likely to be higher On the other hand if the distribution of income and wealth is more in favour of poor demand for commodities preferred by the poor such as necessities will be more(7) Season and Weather ConditionsThis is generally observed that the demand for woolens increases during winter whereas demand for ice creams and cold drinks increases during summer Similarly market demand for umbrellas rain coats increases during rainy seasonQ8 REASONS FOR OPERATION OF LAW OF DEMAND WHY DEMAND CURVE SLOPES DOWNWARDNow we will try to explain why does a consumer purchase more quantity of a commodity at a lower price and less of it at a higher price or why does the law of demand operate ie why does the demand curve slope downwards from left to right The main reasons for operation of law of demand are1 Law of Diminishing Marginal UtilityAs you have studied earlier law of diminishing marginal utility states that as we consume more and more units of a commodity the utility derived from each successive unit goes on decreasing The consumer will be ready to pay more for those units which provide him more utility and less for those which provide him less utility It implies that he will purchase more only when the price of the commodity falls2 Income Effect

When price of a commodity falls purchasing power or real income of the consumer increases which enables him to purchase more quantity of the commodity with the same money income Let us take an example Suppose you buy 4 ice creams when price of each ice cream is ` 25 If price of ice creams falls to ` 20 then with same money income you can buy 5 ice creams now3 Substitution EffectWhen price of a commodity falls it becomes comparatively cheaper as compared to its substitutes (although price of substitutes has not been changed) This will lead to rise in demand for the given commodity For example if coke and Pepsi both are sold at ` 10 each and price of coke falls Now coke has become relatively cheaper and will be substituted for Pepsi It will lead to rise in demand for coke4 Change in Number of BuyersWhen price of a commodity falls some old buyers may demand more of the commodity at the reduced price and some new buyers may also start buying this commodity who were not in a position to buy it earlier due to higher price This will lead to increase in number of buyers when price of the commodity falls As a result demand for the commodity rises when its price falls5 Diverse Uses of a CommoditySome commodities have diverse uses like milk It can be used for drinking for sweet preparation for ice cream preparation etc If price of milk rises its use may be restricted to important purpose only This will lead to reduction in demand for other less important uses When price of milk falls it can be put to other uses also leading to rise n demand for itQ9 EXCEPTIONS TO THE LAW OF DEMANDYou have studied in law of demand that a buyer is willing to buy more quantity of a commodity at a lower price and less of it at a higher price But in certain circumstances a rise in price may lead to rise in demand These circumstances are called Exceptions to the Law of Demand Some important exceptions are1 Giffen GoodsGiffen goods are special type of inferior goods in which negative income effect is stronger than negative substitution effect Giffen goods do not follow law of demand as their demand rises when their price rises Examples of Giffen goods are jowar and bajra etc2 Status Symbol GoodsSome goods are used by rich people as status symbols eg diamonds gold jewellary etc The higher the price the higher will be the demand for these goods When price of such goods falls these goods are no longer looked at as status symbol goods and tehrefore therir demand falls3 NecessitiesCommodities such as medicines salt wheat etc do not follow law of demandbecause we have to purchase them in minimum required quantity whatever their price may be4 Goods Expected to be ScarceWhen the buyers expect a scarcity of a particular good in near future they start buying more and more of that good even if their prices are rising For example during war famines etc people tend to buy more of some goods even at higher prices due to fear of their scarcity in near future

Political Science

Constitution of India-The

Preamble

The preamble-

Preamble-

The preamble is the most precious part of the constitution We the people of India having solemnly resolved to constitute India into a Sovereign Socialist Secular Democratic Republic and to secure to all its citizensA preamble is an introductory and expressionary statement in a document that explains the documents purpose and underlying philosophy When applied to the opening paragraphs of a statute it may recite historical facts pertinent to the subject of the statuteNature and purpose of the constitution-Purpose of the Constitution dictates permanent framework of the government to form a more perfect union to establish justice and ensure peace of thenationconstitution provide principles how the government can run itself following the rules and laws written in the constitution of each state keeps them balanced

Answer the following questions-

1 What is preambleA preamble is an introductory and expressionary statement in a document that explains the documents purpose and underlying philosophy2 What is the nature and

purpose of the constitutionConstitution dictatespermanent framework of the government to form a more perfect union to establish justice and ensure peace of the nation

Homework-Learn

Accounts Compatibilty mode

1MEANING OF PARTNERSHIPPartnership is a form of business organisation where two or more persons join hands to run a business They share the profits and losses according to the agreement amongst them According to the Indian Partnership Act 1932 ldquoPartnership is relation between persons who have agreed to share profits of a business carried on by all or any one of them acting for allrdquo For example one of your friends has passed class XII from National Institute of Open Schooling (NIOS) and wants to start a business Heshe approaches you to join in this venture Heshe wants you to contribute some money and participate in the business activities Both of you if join hands constitute a partnership2CHARACTERISTICS1048698 Agreement A partnership is formed by an agreement The agreement may be either oral or in writing It defines the relationship between the persons who agree to carry on business It may contain the terms of sharing profit and the capital to be invested by each partner etc The written agreement is known as partnership deed1048698 Number of persons There must be at least two persons to form a partnership

The maximum number of partners in a partnership firm can be 50 according toCompanies Act 20131048698 Business The Partnership is formed to carry on business with a purpose of earning profits The business should be lawful Thus if two or more persons agree to carry on unlawful activities it will not be termed as partnership1048698 Sharing Profits The partners agree to share profits in the agreed ratio In caseof loss all the partners have to bear it in the same agreed profit sharing ratio10486981048698Mutual Agency Every partner is an agent of the other partners Every partner can bind the firm and all other partners by hisher acts Each partner will be responsible and liable for the acts of all other partners10486981048698Unlimited liability The liability of each partner except that of a minor is unlimited Their liability extends to their personal assets also If the assets of the firm are insufficient to pay off its debts the partnersrsquo personal property can be used to satisfy the claim of the creditors of the partnership firm10486981048698Management All the partners have a right to mange the business However they may authorize one or more partners to manage the affairs of the business on their behalf10486981048698Transferability of Share No partner can transfer hisher share to any one including hisher family member without the consent of all other partners3PARTNERSHIP DEEDAgreement forms the basis of partnership The written form of the agreement is which a document of partnership is It contains terms and conditions regarding the conduct of the business It also explains relationship between the partners This document is called partnership deed Every firm can frame its own partnership deed in which the rights duties and liabilities of the partners are stated in detail It helps in settling the disputes arising among the partners during the general conduct of business 4CONTENTS OF PARTNERSHIP DEEDThe partnership deed generally contains the following (i) Name and address of the partnership firm(ii) Nature and objectives of the business(iii) Name and address of each partner(iv) Ratio in which profits is to be shared(v) Capital contribution by each partner(vi) Rate of Interest on capital if allowed(vii) Salary or any other remuneration to partners if allowed(viii) Rate of interest on loans and advances by a partner to the firm(ix) Drawings of partners and interest thereon if any(x) Method of valuation of goodwill and revaluation of assets and liabilities on the reconstitution of the partnership ie on the admission retirement or death of a partner(xi) Settlement of disputes by arbitration(xii) Settlement of accounts at the time of retirement or death of a partner5IN ABSENCE OF PARTNERSHIP DEEDThe partnership deed lays down the terms and conditions of partnership in regard to rights duties and obligations of the partners In the absence of partnership deed there may arise a controversy on certain issues like profit sharing ratio interest on

capital interest on drawings interest on loan and salary of the partners In such cases the provisions of the Indian Partnership Act becomes applicableSome of the Issues are(i) Distribution of Profit Partners are entitled to share profits equally(ii) Interest on Capital Interest on capital is not allowed(iii) Interest on Drawings No interest on drawing of the partners is to be charged(iv) Interest on Partnerrsquos Loan A Partner is allowed interest 6 per annum on the amount of loan given to the firm by himher(v) Salary and Commission to Partner A partner is not entitled to anysalary or commission or any other remuneration for managing the business

History TOPIC-TOWARDS INDEPENDENCE AND PARTITION THE LAST PHASE (1935-1947)

SUB TOPIC-IMPORTANT POLITICAL DEVELOPMENTS ndash GROWTH OF SOCIAL IDEAS

Socialism is a political social and economic philosophyLike in other parts of the world the Russian revolution of 1917 served as a great inspiration for revolutionaries in India who at that time were engaged in the struggle for liberation from British ruleSocialist ideas led to the formation of communist party of IndiaJAWAHARLAL NEHRU Among the early Congress leaders Jawaharlal Nehru was very much impressed and influenced by the Socialist ideas He also learnt about the Economic activities of the Soviet Union after the Bolshevic Revolution 1917 He made full use of them in IndiaThe election of Jawaharlal Nehru and Subhas Chandra Bose showed the Left wing tendency within CongressJawaharlal Nehru demanded economic freedom along with political freedom of the people in order to end the exploitation of masses

Nehrus working committee included three socialists leaders The Lucknow session was a landmark in the evolution of socialist ideas of the congressSUBHAS CHANDRA BOSE ndash Subhas Chandra Bose had socialist leaning Both Jawaharlal Nehru and Subhas Chandra Bose were known as leftist Congress men Later on National Congress divided into Leftist and rightist campCONGRESS SOCIALIST Within the Congress some leaders formed the Congress Socialist partyPattavi Sitaramyya Sardar Patel Rajendra Prasad had hostile attitude towards the Congress Socialist partyJawaharlals attitude was hesitant

1 QUESTION ndash Mention name of two Congress leaders who had socialist leaning

1ANSWER ndash Subhas Chandra Bose and Jawaharlal Nehru2QUESTION- In which session of the congress Jawaharlal elaborated his Socialist ideas2 ANSWER ndash Lucknow and Faizpur Session in December 1935 and 19363QUESTION ndash Why Congress was sharply divided into leftist and rightist camp 3ANSWER ndash Subhas Chandra Bosersquos attempt to seek re election for congress presidentship in 1939sharply divided the National Congress into Leftist and Rightist camp4 QUESTION ndash Who was MN Roy 4 ANSWER ndash Manabendra Roy first formed the Communist Party of India outside the country at Tashkent in 19205QUESTION ndash Who formed the Congress Socialist Party within the Congress5 ANSWER ndash Jaya Prakash Narayan Achyut Patwardhan Acharya Narendra Dev Ram Mohan Lohia Aruna Asaf Ali6QUESTION ndash When was the Congress Socialist Party formed What was its object6 ANSWER ndash 1934The Congress Socialist Party sought to work out socialist programme through the Congress They joined hands with the Congress and wanted to carry

Subhas Chandra Bose being expelled from the congress after the Tripuri rift he formed Forward BlockThere were basic differences between the Congress Socialists and the communistsTRADE UNION ACTIVITIES Maximum working class people lived in Bombay and Calcutta The working and living conditions of those workers were very miserable In this situation Shasipada Banerjee NM Lokhande protested against the oppression of the working class peopleThe first Trade Union Madras Labour Union was formed in 1918 by BP WadiaIndustrial strikes took place in Kanpur Calcutta Madras Jamshedpur and Ahmedabad AITUC was formed in Bombay in 1927 The growth of Trade union among the workers was slow because of the fear of the dismissal of the jobIn the mean time the Moderates as well as Communists left AITUC and formed separate organization

on National struggle with the help of workers and peasant class of the society7 QUESTION ndash What was the name of the party founded by Subhas Chandra Bose7 ANSWER- Forward Block8QUESTION ndash Who was Shasipada Banerjee8 ANSWER ndash Shasipada Banerjee was a radical Brahmo He founded a working menrsquos club to protest against exploitation of the British rulers towards the working class of India9 QUESTION ndash What was the weekly published by NM Lokhande9ANSWER- Dinabandhu10 QUESTION ndash Who founded Bombay Mill-Hands Association and in which year10 ANSWER- NM Lokhande in189011 QUESTION- Who was BP WadiaANSWER- BPWadia was the founder of Madras Labour Union in191812 QUESTION- What was the name of the first labour union of India12 ANSWER- Madras Labour Union13 QUESTION Who founded the Majur Mahajan 13 ANSWER GANDHIJI14 QUESTION What was the full form of AITUC When it was formed14 ANSWER All India Trade Union Congressin 192715QUESTION Who formed the Red Trade Union Congress and in which year15ANSWER The Communists formed the Red Trade Union Congress16 QUESTION What do you mean by Socialism16 ANSWER Socialism describes any political and economic theory that says the community rather than individuals should own and manage property and natural resources

Subject Eng Literature (The Tempest ndash William Shakespeare) Topic Act III Scene 3 Lines 1 to 52 (Line 52 ndash Brother my lord the Duke Stand to and do as we) Date 13th April 2020 (4th Period)

[Students should read the original play and also the paraphrase given in the school prescribed textbook]Summary Questions amp Answers

o Alonso Sebastian Antonio Gonzalo Adrian Francisco and others wandered about the island in search of Ferdinand and gets tired and hungry of the toil and at the same time gives up all hope of finding him

o Antonio and Sebastian are happy that Alonso is out of hope and decide to make another attempt on his life that night when being so tired they will be sleeping soundly

o Suddenly a solemn and strange music is heard in the air and several strange shapes enter bringing in a banquet These strange shapes then dance round it with gestures of salutation and then inviting the King to eat they depart

o Seeing this strange scene all are inclined to believe the tales told by travelers that there truly are ldquounicornsrdquo and ldquothe phoenixrsquo thronerdquo

1 ALONSO What harmony is this My good friends hark (L18-27)

GONZALO Marvellous sweet music

[Enter several strange shapes bringing in a banquet

they dance about it with gentle actions of salutation

and inviting the King and his companions to eat they depart]ALONSO Give us kind keepers heavens What were theseSEBASTIAN A living drollery Now I will believe

That there are unicorns that in Arabia

There is one tree the phoenixrsquo throne one phoenix

At this hour reigning thereANTONIO Ill believe both

And what does else want credit come to me

And Ill be sworn rsquotis true Travellers neer did lie

Though fools at home condemn rsquoem

(a) How did Prospero present an amazing spectacle before Alonso and his companions

Using his magic powers Prospero ordered strange shapes to lay a banquet before Alonso and his companions The shapes brought several dishes with tasty eatables in them They placed the dishes on a table before Alonso and his companions Then the strange shapes began to dance gracefully around the banquet While dancing they made gestures inviting them to eat the food Then suddenly the shapes disappeared(b) Who were the guests at the strange banquet Describe the lsquoliving drolleryrsquo

Alonso Sebastian Antonio Gonzalo Adrian and Francisco were the guests at the strange banquet

The term ldquoliving drolleryrdquo refers to live entertainment show In this context when Alonso the King of Naples Sebastian his brother Antonio the treacherous brother of Prospero Gonzalo the kind and loyal councillor to the King Adrian and Francisco came to the island they were hungry and weary in their spirits They heard a solemn and strange music They were shocked to see several strange shapes bringing in a banquet and these shapes danced about it with gentle action of salutation inviting the King and his companions to eat After this Sebastian described this show as lsquoliving drolleryrsquo(c) What is lsquophoenixrsquo What are lsquoUnicornsrdquo

The term lsquophoenixrsquo refers to a mythical Arabian bird which lived alone and perched on a solitary tree After one hundred years it expired in flames and rose again from its own ashes

lsquoUnicornsrsquo refers to the mythological four-footed beasts having horns in the centre of their foreheads When the horns are ground into powder the powder was believed to be

an aphrodisiac(d) How does Sebastian explain the puppet show OR Why does the speaker now believe in unicorns and phoenix

Sebastian finds several strange shapes bringing in the banquet They invite the king and his party for dinner and soon depart He tells that if such a strange sight can be a reality there is nothing incredible in the world and from the present moment he will believe anything He says that it is a strange dumb show enacted not by puppets but by living beings It is stranger than a travellerrsquos tale Seeing such a thing

before his own eyes he will no longer disbelieve the story about unicorns and phoenix(e) How do the other characters present respond to this living drollery

At the sight of the lsquoliving drolleryrsquo like Sebastian Gonzalo and Antonio too acted strangely Antonio told that he too now believes in unicorns and phoenix and anything else that seems to be incredible He too now believes in travellersrsquo tales Gonzalo told that if he would report those happenings in Naples nobody will believe him He considers that those gentle shapes were gentler in manner in comparison to the living beings Alonso was at first sight suspicious and told them that those strange shapes conveyed their meaning in expressive gestures when they seemed to lack speech by their movements and sounds Francisco was amazed at their mysterious disappearance

2 ALONSO Not I

(Line 43-52)GONZALO Faith sir you need not fear When we

were boysWho would believe that there were mountaineers

Dewlapped like bulls whose throats had hanging at rsquoem

Wallets of flesh Or that there were such men

Whose heads stood in their breasts Which now we find

Each putter-out of five for one will bring us

Good warrant ofALONSO I will stand to and feed

Although my lastmdashno matter since I feel

The best is past Brother my lord the Duke

Stand to and do as we

(a) How does Alonso respond at the spectacle of the shapes which were sent to them at the instruction of Prospero

After seeing the strange sight of appearing and disappearing of the shapes sent by Prospero to arrange a banquet for them Alonso says that his surprise at having seen those creatures is infinite and he is fully justified in feeling so much surprise He thinks that their shapes their gestures and the sounds they made were indeed amazing Although they do not possess the gift of speech yet they were able to convey their

thoughts by means of their gestures only

(b) What does Prospero say about the views expressed by Alonso regarding the shapes What does Francisco think about the shapesAfter hearing Alonsorsquos views about the shapes Prospero says that this manrsquos praise of the spirits is rather hasty He means to say that Alonso has shown great haste in reaching the conclusion about the shapes Francisco is amazed to see that those shapes disappeared in a mysterious way(c) What does Sebastian ask Alonso to doSebastian tells Alonso that the shapes having disappeared should not matter to them because they have left the eatables behind He asks Alonso to enjoy eating as they are extremely hungry but the king does not accept his offer of enjoying the dishes(d) How does Gonzalo try to dispel Alonsorsquos fear of those strange shapes What kind of references does he give to AlonsoGonzalo says that those who have travelled abroad have reported seeing even stranger sights than these shapes that Alonso and his companions have beheld Hence there is no reason to feel afraid of these shapes Gonzalo further adds that in his younger days he had heard strange stories from travelers and Alonso might have heard similar stories For instance it was said that there existed a certain race of

human beings who had huge lumps of flesh hanging at their throats and who therefore resembled bulls Then Gonzalo tells about a race of human beings whose heads were located at their breasts Gonzalo says that such stories were not believed by most people in those days but now-a-days these stories have become common(e) Explain the following lsquoEach putter-out of five for onersquoEnglish travellers often insured their trips with London brokers Those that went on foreign travels those days used to deposit a certain amount with some firm or company in London before their departure If the travelers failed to return the money was forfeited by the company with which it had been deposited But this money was repaid five-fold if the travelers returned safe and sound In this way a traveler stood a great chance of recovering the entire cost of his

travels(f) Give the explanatory meanings of the following expressions in the context of the above extract (i) Dewlapped (ii) Wallets of flesh

(iii) Putter-out(i) Dewlapped having big lumps of flesh at the necks(ii) Wallets of flesh large masses of flesh looking like bags(iii) Putter-out to invest money before commencing the travel

  • General methods of preparation of hydrogen
  • Chapter Dimensional Analysis (Summary)
    • Properties of Charge
Page 37:   · Web viewSubject. Topic. Summary. Execution. Hindi. व्याकरण. शरीरके अंगो के नाम लिखिए. 1) आँख 2) नाक 3

Micro means a small part in

microeconomics we do not study the whole economy Hence we study an individual consumer and his or her choices and a producer and his or her profit maximizing decisions in the market Thus it does not mirror what happens in the economy as a whole

Macroeconomics on the other hand studies the economy as a whole It is concerned with aggregate and depicts the entire picture of the economyMacroeconomics deals with the national income aggregate investment aggregate consumption etc

Features of Microeconomics It deals with small

parts of the country Hence it looks at

individual consumers firms and industries

It deals with individual income consumption and savings

It studies the determination of price of any product or factors of production

It deals with the working of market via the price mechanism which is nothing but the determination of price and quantity of a commodity by the forces of demand and supply

Features of Macroeconomics

It deals with the study of the economy as a whole

It is concerned with

5 Give a limitation of microeconomics Microeconomics fails to explain the

functioning of an economy as a whole It cannot explain unemployment illiteracy and other problems prevailing in the country

6 What is the importance of macroeconomics It gives overall view of the growing

complexities of an economic system It provides the basic and logical

framework for formulating appropriate macroeconomic policies (eg for inflation poverty etc )to direct and regulate economy towards desirable goals

7What is the limitation of macroeconomics It ignores structural changes in an

individual unit of the aggregate

8 Differentiate between Microeconomics and Macroeconomics

Microeconomics Macroeconomics

the study of aggregates

National income aggregate savings and aggregate investments are major concepts dealt within macroeconomics style

It studies the determination of general price levels

It investigates into the problem of unemployment and the achievement of employment

It studies the aspect of decision making at the aggregate and national levels

It includes all growth theories whether related to developed or developing economies it also includes the study of economic systems and the working of the economy under different systems

Note Both Micro and macro economics are complementary and should be fully utilized for proper understanding of an economy

1It studies economic aspect of an individual unit2It deals with individual incomeConsumption and savings

3 It facilitates determination of price of any product or factors of production

4 Itrsquos scope is narrow and restricted to individual unit

1It studies the economy as a whole

2It deals with the national income aggregate consumption and aggregate savings3 It facilitates determination of general price level in an economy

4 Itrsquos scope is wide as it deals with economic units on the national level

ACCOUNTS

Introduction to Accounting and Book-keeping

Today I am going to share you the meaning of Accounting and Book-keeping and its related terms bullAccounting bullBook Keeping bullAccountsbullTypes Of Accounts bullAccounting Cycle

bull Meaning of accounting

Ans ) Accounting is the art and science of recording classifying and summarising monetary transactions

bull Meaning of Book-keeping

Ans) Bookkeeping is the art of recording business transactions with the view of having a permanent record of them and showing their effect on wealth

bull Meaning of account

Ans) The term account means a record of

business transactions concern a particular person of firm asset or income or expense It is a summarised record of all transactions which take place in an accounting year

bull Types of accountsPersonal accounts ndash Personal accounts relating

to person and Organisation are known as personal accounts Example Ramrsquos Account ABC amp Co Account etc

Real account - The accounts related to tangible and intangible assets are called real accounts Example Cash Account Furniture Account etc

Nominal account- Accounts related to expenses losses incomes and gains are known as nominal accounts Example Wages Account Salary Account Discount Account etc

bull Accounting cycle Accounting cycle refers to a complete sequence of accounting activities It begins with recording of transactions and ends with the preparation of a balance sheet

Chemistry TopicAtomic Structure

Thomsonrsquos atomic modelThomson (1898) was the first to propose the model of an atomHe proposed that an atom can be regarded as a uniform sphere of positive electricity in which requisite number of electrons are embedded evently to neutralize the positive chargeThis is just like plums embedded in a pudding or seeds evently distributed in red spongy mass of a watermelonThis model of atom is known as ldquoPlum-Pudding modelrdquo or

Q1)What is the fundamental constituents of atomAns Electron Proton and neutrons are the fundamental constituents of atomQ2)What is the value of fundamental unit of electricityAnsThe charge carried by one electron is sad to be the fundamental unit of electricityIts magnitude is 48times10-10esuOr 1602times10-19C Q3)Name the element containing no neutronAnsOrdinary hydrogen atom or protium 1H1

Types of AccountPersonal AccountReal AccountNominal AccountBalance Sheet (opening)

ldquowatermelon modelrdquoThis model could explain the electrical neutrality of an atom but failed to explain the result of scattering experiment carried out by Rutherford in 1911So it was rejected ultimately

Q4)Why is an electron called universal particleAns Itrsquos mass and Charge are independent of its source

EVS Chapter 1 ndash Modes of Existence

Modes of existence When one speaks normally about the mode of existence of some group or individual one refers to their customs their mode of being their ethology their habitat in some way their feeling for a placeDifferent modes of exixtence are ndash

1 Hunting ndashGathering2 Pastoral3 Agricultural4 Industrial

1 Hunting and gathering Hunting and gathering mode of existence is characterized by obtaining food from hunting wild animals including fishing and gathering wild plants From their earliest days the hunter-gatherer diet included various grasses tubers fruits seeds and nuts Lacking the means to kill larger animals they procured meat from smaller game or through scavenging

Societies that rely primarily or exclusively on hunting wild animals fishing and gathering wild fruits berries nuts and vegetables to support their diet are called hunting and gathering societies

At least this used to be practice of human beings before agriculture is invented As their brains evolved hominids developed more intricate knowledge of edible plant life and growth cycles

Q) Write the features of Hunting ndash gathering societiesAns - There are five basic characteristics of hunting and gathering societies

i The primary institution is the family which decides how food is to be shared and how children are to be socialized and which provides for the protection of its members

ii They tend to be small with fewer than fifty members

iii They tend to be nomadic moving to new areas when the current food supply in a given area has been exhausted

iv Members display a high level of interdependence

v Labor division is based on sex men hunt and women gather

Political Science

Introduction to political science

Political science occasionally called politology is a social science which deals with systems of governance and the analysis of political activities political thoughts associated constitutions and political behaviorThe study of political science involves the study of both the

Answer the following questions-1 What is political science

Political science occasionally called politology is a social science which deals with systems of governance and the analysis of political activities political thoughts associated constitutions and political behavior

2 Short notes-

traditional and modern theories of politicsTraditionalClassical political sciencepolitical theory-Traditional political science is the study of politics before Second World War The methodology to study Politics was traditional (legal formaletc) the definition of politics traditional (Politics begins and end with state)area of study (constitution state machinery)was traditionalModern Political scienceModern political theory-Modern Political Theory critically examines the contemporary state of political theory making an assessment of the achievement and limitations of the Behavioural Revolution in its totality and reviews objectively the major paradigms and conceptual frameworks adopted by the disciplineContemporary attempts at the development of an integrated political theory involving the use of both traditional and modern concepts approaches and theories-Around late 1960s several political scientists realized the importance of both the traditional political theory and modern Political theory They began building an integrated theory of politics involving a systematic mixture of traditional and modern studies of politics It was held that the study of a complex and vast field like politics needs both traditional as well as

Classical political theory Modern Political theory

Homework-Learn

modern concepts and approaches for studying itrsquos all aspects

Subject Eng Literature (The Tempest ndash William Shakespeare) Topic Act I Scene 1 Lines 1 to 32 (Line 32 ndash Gonzalo hellip If he be not born to be hanged our case is miserable) Date 13th April 2020 (3rd Period)

[Students should read the original play and also the paraphrase given in the school prescribed textbook]Summary Questions amp Answers

[SUMMARY OF THE ENTIRE SCENE]

o The play starts with the scene of a severe storm at sea Alonso (King of Naples) Sebastian (Alonsorsquos brother) Ferdinand (Alonsorsquos son) Gonzalo Antonio (the usurping Duke of Milan) are in a ship in the midst of the storm

o The mariners are trying their best to control the vessel from running aground and are totally following the orders of their Master the Boatswain They have scant success

o The mariners become extremely unhappy and annoyed when most of the passengers arrive on the deck thereby hampering their effort to save the ship There is serious confrontation between them and the passengers who are part of the Kingrsquos entourage

o The mariners could not save the ship

SUMMING-UP

(i) Vivid description of the scene which gives a realistic description of terror and confusion of a tropical storm

(ii) Shows Shakespearersquos accuracy of knowledge in describing the naval operations and also matters of seamanship

(iii) The opening scene justifies the title ndash The Tempest

UNANSWERED QUESTIONS

(i) The King always travels with his entire fleet including his soldiers Where

(1) GONZALO Nay good be patient (Line 15-26)BOATSWAIN When the sea is Hence What cares these

roarers for the name of the king To cabin silence Trouble us not

GONZALO Good yet remember whom thou has aboardBOATSWAIN None that I more love than myself You are a

councillor if you can command these elements to silence and work

the peace of the present we will not hand a rope more use your authority If you cannot give thanks you have

lived so long and make yourself ready in your cabin for the mischance of the hour if it so hap [To the Mariners]

Cheerly good hearts [To Gonzalo] Out of our way I say

(a) To whom is the boatswain speaking What does he mean by lsquoNone that I more love than myselfrsquo

The Boatswain is speaking to Gonzalo the honest old councilor of the Duke of MilanBy using the words ndash lsquoNone that I love more than I love myselfrsquo means that for the Boatswain nobody is dearer to him than his own life

(b) What were the conditions that made the boatswain react in this way

The Boatswain reacts in this way because the storm is at sea and Alonso King of Naples Sebastian his brother Ferdinand his son Gonzalo Antonio the usurping Duke of Milan on board are in distress and in panic Thus they have rushed to the deck interrupting the work of the mariners

(c) What hope does Gonzalo take from the attitude of the boatswain

The insolent and authoritative attitude of Boatswain makes Gonzalo feel comforted He tells that there are no signs that the Boatswain will be drowned But his facial appearance and attitude shows that he is destined to die on land by hanging which in effect means that all on board will be saved Otherwise all the persons on board are doomed

(d) How can they lsquomake yourself ready in your cabinrsquo For what were they asked to make ready themselves

In order to make themselves ready in their cabin the

were the other ships

(ii) Why was the ship in that area Where was it coming from or going where

(iii) The ship broke apart What happened to those who were in the ship

passengers on board must prepare for death which they will possibly soon have to meetThey can retire to their cabins and offer prayers to the Almighty to save them from drowning

(e) What does the boatswain say when he is asked to be patient What does he order to the royal party

When the boatswain is asked to be patient and remain calm he says that he will be patient only when the storm will be over and the sea will be calm but as long as the storm blows and there is danger to the ship he cannot think of being patient He orders the royal party to go to the cabin and leave the mariners to their work

(2) GONZALO I have great comfort from this fellow (Line 27-36)

Methinks he hath no drowning mark upon him his complexion is perfect

gallows Stand fast good Fate to his hanging Make the rope of his destiny our cable for our own doth little advantage If he be not born to be hanged our case is miserable

(a) Why does Gonzalo regard the Boatswain in the midst of danger

In the midst of danger Gonzalo regards the boatswain because he feels that the Boatswain is a source of comfort and is bent upon to do his work sincerely which in this case is saving the ship and its passengers from the severest of raging storm

(b) What reasons does Gonzalo give when he says that none in the ship will die of drowning

Gonzalo is almost sure that none in the ship will die by drowning His says that there is no mark on the face of the boatswain that indicates that he will die by drowning On the other hand the lines on his face are strong indications that he will be hanged to death Therefore there shall be no danger of the shiprsquos sinking

(c) Explain the following ldquoStand fast good Fate to his hanging Make the rope of his destiny our cable for our own doth little advantage If he be not born to be hanged our case is miserablerdquo

The stated lines mean that if the will of destiny is to be carried out then the ship will not get wrecked and all the passengers will be saved The safety of the passengers therefore depends upon the will of fate being carried out in the case of the boatswain If however the boatswain is not to die by hanging then the passengers are also very unsafe because in that case the ship is likely to sink

(d) What order does the Boatswain give to the sailors

when he re-enters What does he say about the crying of the fellows inside the cabin

The boatswain orders the sailors to bring the topmast lower and bring the ship close to a stationary position with the help of the main sail He says that the fellows inside the cabin are moaning and crying in their distress louder than his voice and louder even than the roaring of the storm

Class XII (ScienceCommerceHumanities) Subject Topic Summary Execution

Computer Science

PropositionalLogic

Propositional logic is a procedure to provide reasoning through statementProposition A ststement that results in True or False is said to be proposition There are two types of propositionSimple proposition amp compound propositionSimple proposioton A simple proposition is one that is not a part of any other proposition Such sentential form of proposition is symbolized with english letters in short For example Ram is a claver student (TrueFalse)Where do you live (Not in True or False)Grapes are sweet (TrueFalse)It rains today (TrueFalse)Here we can see some statements anwer would be true or false but some staements answer can not give in terms of true or false Thus the sentences which can be answered in true or false are known as simple propositionAssigning propositon to a variableThe general syntax to assign propostion to a variable is as followsVariable = Simple propositonFor example A=Ram is a clever studentB= Grapes are sweetC= it rains todayCompound proposition

helliphellipto be continued in next classhelliphellipMath Relation Relation If A and B are two non-empty sets

then a relation R from A to B is a subset of AxB If R A x B and (a b) R then we say that a sube isinis related to b by the relation R written as aRbeg Let A be the set of students of class XII and B be the set of students of class XI Then some of the examples of relation from A to B arei) (a b) AXB a is brother of bisinii) (a b) AXB age of a is more than age of isinb Types of relation In this section we would like to study different types of relations We know that a relation in a set A is a subset of A times A Thus the empty set φ and A times A are two extreme relations For illustration consider a relation R in the set A = 1 2 3 4 given by R = (a b) a ndash b = 10 This is the empty set as no pair (a b) satisfies the condition a ndash b = 10 Similarly R = (a b) | a ndash b | ge 0 is the whole primeset A times A as all pairs (a b) in A times A satisfy | a ndash

Example 1 Let A be the set of all students of a boys school Show that the relation R in A given by R = (a b) a is sister of b is the empty relation and R = (a b) the primedifference between heights of a and b is less than 3 meters is the universal relationSolution Since the school is boys school no student of the school can be sister of any student of the school Hence R = φ showing that R is the empty relation It is also obvious that the difference between heights of any two students of the school has to be less than 3 meters This shows that R = A times A is primethe universal relation Example 2 Show that the relation R in the set 1 2 3 given by R = (1 1) (2 2) (3 3) (1 2) (2 3) is reflexive

b | ge 0 These two extreme examples lead us to the following definitionsDefinition 1 A relation R in a set A is called empty relation if no element of A isrelated to any element of A ie R = φ A times AsubDefinition 2 A relation R in a set A is called universal relation if each element of A is related to every element of A ie R = A times A Both the empty relation and the universal relation are some times called trivial relation Definition 3 A relation R in a set A is called(i) reflexive if (a a) R for every a Aisin isin(ii) symmetric if (a1 a2) R implies that (aisin 2a1)

R for all aisin 1 a2 Aisin(iii) transitive if (a1 a2) R and (aisin 2 a3) R isinimplies that (a1 a3) R for all aisin 1 a2 a3 AisinDefinition 4 A relation R in a set A is said to be an equivalence relation if R is reflexive symmetric and transitive

but neither symmetric nor transitiveSolution R is reflexive since (1 1) (2 2) and (3 3) lie in R Also R is not symmetric as (1 2) R but (2 1) isin notinR Similarly R is not transitive as (1 2) R and (2 3) R but (1 3) R isin isin notinExample 3 Show that the relation R in the set Z of integers given byR = (a b) 2 divides a ndash b is an equivalence relationSolution R is reflexive as 2 divides (a ndash a) for all a Z isinFurther if (a b) R then 2 divides a isinndash b Therefore 2 divides b ndash a Hence (b a) R which shows that R is isinsymmetric Similarly if (a b) R and (b c) R isin isinthen a ndash b and b ndash c are divisible by 2 Now a ndash c = (a ndash b) + (b ndash c) is even (Why) So (a ndash c) is divisible by 2 This shows that R is transitive Thus R is an equivalence relation in ZExample 4 Let L be the set of all lines in a plane and R be the relation in L defined as R = (L1 L2) L1 is perpendicular to L2 Show that R is symmetric but neither reflexive nor transitiveSolution R is not reflexive as a line L1 can not be perpendicular to itself ie (L1 L1) R notinR is symmetric as (L1 L2) Risin

L1 is perpendicular to L2rArr L2 is perpendicular to L1rArr (L2 L1) RrArr isin

R is not transitive Indeed if L1 is perpendicular to L2 and L2 is perpendicular to L3 then L1 can never be perpendicular to L3 In fact L1 is parallel to L3 ie (L1 L2) R isin(L2 L3) R but (L1 L3) Risin notin

Chemistry Solid state Characteristics if Solids(i)The particles are locked in fixed positions they are unable to change their relative positions and this brings a definite shape and volume of a solid(ii)In a solid the constituent particles are held by strong forces of attractionThe forces of attraction may be bonding or non bonding(iii)The constituent particles in a solid pack together as closely as possibleoccupying most of the available space within the solidThus the empty space in a solid is very smallThis makes a solid highly rigid and nearly incompressibleThis also explains why a solid has high density and exhibits slow diffusionClassification of Solids

Q1)Define Crystalline solids AnsA Solid that has a definite geometrical shape and a sharp melting pointand whose constituent particles (atomsmolecules or ions) are arranged in a long range order of definite pattern extending throughout the solid is called a crystalline solidExNaClQ2)Define Amorphous solids AnsA solid that does not have a definite shape and a sharp melting pointand whose constituent particles (atomsmolecules or ions) are not arranged in a definite pattern is called an amorphoussolid

Crystalline solidsAmorphous solids

ExGlassRubberQ3)Classify Crystalline Solids Crystalline Solids

Physics Coloumbrsquos Law (Summary)

Before Going Into Coloumbrsquos Law We Will First Learn What is Charge Properties of Charge and Always remember that charge is quantized ie a body always have static charge of magnitude equal to some integral multiple of fundamental electronic charge e= 16 x 10- 19 C

Charge is the property of matter that causes it to produce and experience electrical and magnetic effects The study of the electrical charges at rest is called electrostatics When both electrical and magnetic effects are present the interaction between charges is referred to as electromagnetic

There exist two types of charges in nature positive and negative Like charges repel and unlike charges attract each other

The type of charge on an electron is negative The charge of a proton is the same as that of an electron but with a positive sign In an atom the number of electrons and the number of protons are equal The atom is therefore electrically neutral If one or more electrons are added to it it becomes negatively charged and is designated as negative ion However if one or more electrons are removed from an atom it becomes positively charged and is called a positive ion

The excess or deficiency of electrons in a body gives the concept of charge If there is an excess of electrons in a body it is negatively charged And if there is deficiency of electrons the body becomes positively charged Whenever addition or removal of electrons takes places the body acquires a charge

The SI Unit of charge is coulomb (C) In SI units the current is a fundamental quantity having a unit of ampere (A) The unit of charge is defined in terms of the unit of current Thus one coulomb is the charge transferred in one second across the section of a wire carrying a

Ionic SolidsMetallicSolids

Molecular Solids

current of one ampere

As q = It we have1 C = (1 A) (1 s)

The dimensions of charge are [A T]

Properties of Charge

(1) Quantization of Charge Electric charge can have only discrete values rather than any value That is charge is quantized The smallest discrete value of charge that can exist in nature is the charge on an electron given as

e = plusmn 16 x 10- 19 C

This is the charge attained by an electron and a protonA charge q must be an integral multiple of this basic unit That is

Q = plusmn ne where n = 1 2 hellip

Charge on a body can never be (frac12)e (23)e or 57e etcWhen we rub a glass rod with silk some electrons are transferred from the rod to the silk The rod becomes positively charged The silk becomes negatively charged The coulomb is a very large amount of charge A typical charge acquired by a rubbed body is 10 - 8 C

Biology Reproduction in organisms

Welcome to this new session 2020-21Today in this first chapter we mainly discuss about reproduction types needs and life span of some organismsWe also discuss about difference between sexual and asexual reproduction

Q1 What is reproductionReproduction is defined as a biological processin which an organism gives rise to young onessimilar to itselfQ2 What are the needs of reproductionbulli) Reproduction maintain life on earthii) It enables the continuity of the species generation after generationiii) It creates genetic variation among populationsQ3 Define Life span and write some orgnisms life spanbull Life span is the period from birth to

the natural death of an organism- OrganismsLife span1 Butterfly 1 - 2 weeks2 Fruit fly 30 days3Dog 10-13 years4 Rose5-7 years5 Tortoise100-150 years6 Banyan Tree -200 - 250 yearsQ4 Reproduction is of two types in case ofanimals but in case of plants vegetative propagation is also present

Asexual Reproduction Sexual Reproductioni) Always uniparentalii) Gametes are not involvediii) Only mitotic division involvediv) Somatic cells of parents are involvedv) Offsprings are genetically similar to the parents

i) Usually biparentalii) Gametes are involvediii) Meiosis occurs during gametogenesis Mitosis occurs after fertilisationiv) Germ cells of the parents are involvedv) offsprings are genetically different from the parents

COMMERCE BUSINESS ENVIRONMENT

Welcome to the new sessiontoday we are going to start the first chapter of Class XII The name of the chapter is Business Environment

Already many of you have got some idea about the word business environment form the first chapter of business studies in class XI

In todayrsquos world every business enterprise is a part of the society It exists and operates in association with various groups in society such as customers suppliers competitors banks and financial institutions government agencies trade unions media and so on All these groups influence the functioning of business in one way or the other They constitute the environment of businessConcept of Business Environment

The term lsquobusiness environmentrsquo refers to the sum total of all individuals institutions and other forces that lie outside a business enterprise but that may influence its functioning and performance

The main features of business environment Totality of External forces General and Specific forces Interrelatedness Complexity Dynamic Uncertainty

Prepare the following questions from todayrsquos assignment

2 What do you mean by business environment

The term lsquobusiness environmentrsquo means the aggregate of all forces factors and institutions which are external to and beyond the control of an individual business enterprise but they may influence its functioning and performance Business environment is the macro framework within which a business firm a micro unit operates It consists of several interrelated and interacting elements

2 Explain the main features of business environment in brief

Totality of External forces-Business environment is the sum total of all things external to a business environment

General and Specific forces-It

Relativity

The Interrelation between business and its environment

The business enterprise is an open system It continuously interacts with its environment It takes inputs (such as raw materials capital labour energy and so on) from its environment transforms them into goods and services and sends them back to the environment

Fig 1 Business Environment Relationship

includes both the forces general forces are the economic social political legal and technological conditions which indirectly influence all business enterprise Specific forces are the investors customers competitors and suppliers which influence individual enterprise directly

Interrelatedness-Different elements of environment are interrelated for an example growing awareness for health care has increased the demand for health foods

Complexity- Business environment id complex in nature as the elements keep on changing example economic technological and other forces changes in demand for a product and service

Dynamic-Business environment is not static it keeps on changing

Uncertainty- Itrsquos very difficult to predict future events such as technology and fashion which occur fast and frequently

Business Studies

Human Resources Management

Human resource of an organisation are the aggregate of knowledge skills attitudes of people working in it

The management system which deals with human resources is called human resource management

Features of HRMbullComprehensive functionbullPeople-oriented

Question1) What do you mean by human

resource management Answer) Human resource management may be defined as that field of Management which has to do with planning organising and controlling the functions of procuring developing maintaining and utilising the labour force

bullAction oriented bullPervasive function bullContinuous function

2) Explain the features of HRM in brief

Answer)bullHuman Resource Management is concerned with managing people at work bull Human Resource Management is concerned with employees which bring people and organisations together so that the goals of each are met bullHuman resource management considered every employees as an individual and also promote their satisfaction and growth bull Human resource management is inherent in all organisations and at all levelsbullManagement of human resources are ongoing on never ending process which requires a constant alertness and Awareness of human relations

3) ldquoHR function is said to be pervasiverdquowhy

Answer) Human resource management is required in all organisations whether it is private or government organisations armed forces sports organisations etc It permeatsall the functional areas like production marketing finance research etc This from this feature of human resource management it can be said that it is pervasive in nature

Economics Demand Q1DEFINITION OF DEMANDIn economics demand is the quantity of a good that consumers are willing and able to purchase at various prices during a given period of timeQ2DEMAND CURVEIn economics a demand curve is a graph depicting the relationship between the price of a certain commodity and the quantity of that commodity that is demanded at that pricQ3LAW OF DEMANDIn microeconomics the law of demand states that conditional on all else being equal as the price of a good increases quantity demanded decreases conversely as the price of a good decreases quantity demanded increasesQ4ASSUMPTION of LAW OF DEMAND(i)No change in price of related commodities(ii) No change in income of the consumer(iii) No change in taste and preferences customs habit and fashion of the consumer( No expectation regarding future change in priceQ5MARKET DEMAND SCHEDULEIn economics a market demand schedule is a tabulation of the quantity of a good that all consumers in a market will purchase at a

given price At any given price the corresponding value on the demand schedule is the sum of all consumersrsquo quantities demanded at that priceQ6INDIVIDUAL DEMAND SCHEDULEIndividual demand schedule refers to a tabular statement showing various quantities of a commodity that a consumer is willing to buy at various levels of price during a given period of timeQ7 FACTORS AFFECTING INDIVIDUAL DEMAND FOR A COMMODITY

The factors that influence a consumerrsquos decision to purchase a commodity are also known as determinants of demand The following factors affect the individual demand for a commodity1 price of the commodity2 price of related goods3 income of buyer of the commodity4 tastes and preferences of the buyer1 Price of the CommodityYou must have observed that when price of a commodity falls you tend to buy more of it and when its price rises you tend to buy less of it when all other factors remain constant (lsquoother things remaining the samersquo) In other words other things remaining the same there is an inverse relationship between the price of a commodity and its quantity demanded by its buyers This statement is in accordance with law of demand which you will study in the later part of this lesson Price of a commodity and its quantity demanded by its buyers are inversely related only when lsquoother things remain the samersquo So lsquoother things remaining the samersquo is an assumption when we study the effect of changes in the price of a commodity on its quantity demanded2 Price of Related goodsA consumer may demand a particular good But while buying that good heshe also asks the price of its related goods Related goods can be of two types-(i) Substitute goods(ii) Complementary goods While purchasing a good prices of its substitutes and complements do affect its quantity purchased(i) Price of Substitute Goods Substitute goods are those goods which can easily be used in place of one another for satisfaction of a particular want like tea and coffee An increase in price of substitute good leads to an increase in demand for the given commodity and a decrease in price of substitute good leads to a decrease in demand for the given commodity It means demand for a given commodity is directly affected by change in price of substitute goods For example if price of coffee increases the demand for tea will rise as tea will become relatively cheaper in comparison to coffee(ii) Price of Complementary goods Complementary goods are those goods which are used together to satisfy a particular want like car and petrol An increase in the price of complementary goods leads to a decrease in demand for the given commodity and a decrease in the price of complementary goods leads to an increase in demand for the given commodity For example if price of petrol falls then the demand for cars will increase as it will be relatively cheaper to use both the goods together So demand for a given commodity is inversely affected by change in price of complementary goods3 Income of the Buyer of CommodityDemand for a commodity is also affected by income of its buyer However the effect of change in income on demand depends on the nature of the commodity under consideration In case of some goods like full cream milk fine quality of rice (Basmati rice) etc demand for these commodities increases when income of the buyer increases and

demand for these commodities decreases when income of the buyer decreases Such goods whose demand increases with the increase in income of the buyer are called normal goods But there are some goods like coarse rice toned milk etc whose demand decreases when income of buyer increases and their demand increases when income of the buyer decreases Such goods whose demand decreases with the increase in income of the buyer are called inferior goods Suppose a consumer buys 10 Kgs of rice whose price is ` 25 per Kg He cannot afford to buy better quality of rice because the price of such rice is ` 50 per Kg The consumer is spending ` 250 per month on the purchase of rice Now if income of the consumer increases and he can afford ` 350 on purchase of 10 Kg of rice Now he can afford to buy some quantity of rice say 6 Kgs whose price is ` 25 per Kg and may buy 4 Kgs of rice whose price is ` 50 per Kg Thus he will buy 10 Kgs of rice by spending ` 350 per month Therefore we may conclude that demand for normal goods is directly related to the income of the buyer but demand for inferior goods is inversely related to the income of the buyer4 Tastes and Preferences of the BuyerThe demand for a commodity is also affected by the tastes and preferences of the buyers They include change in fashion customs habits etc Those commodities are preferred by the consumers which are in fashion So demand for those commodities rises which are in fashion On the other hand if a commodity goes out of the fashion its demand falls because no consumer will like to buy it(5) Number of Buyers in the Market(Population)Increase in population raises the market demand whereas decrease in population reduces the market demand for a commodity Not only the size of population but its composition like age (ratio of males females children and old people in population) also affects the demand for a commodity It is because of needs of children young old male and female population differs(6) Distribution of Income and WealthIf the distribution of income and wealth is more in favour of the rich demand for the commodities preferred by the rich such as comforts and luxuries is likely to be higher On the other hand if the distribution of income and wealth is more in favour of poor demand for commodities preferred by the poor such as necessities will be more(7) Season and Weather ConditionsThis is generally observed that the demand for woolens increases during winter whereas demand for ice creams and cold drinks increases during summer Similarly market demand for umbrellas rain coats increases during rainy seasonQ8 REASONS FOR OPERATION OF LAW OF DEMAND WHY DEMAND CURVE SLOPES DOWNWARDNow we will try to explain why does a consumer purchase more quantity of a commodity at a lower price and less of it at a higher price or why does the law of demand operate ie why does the demand curve slope downwards from left to right The main reasons for operation of law of demand are1 Law of Diminishing Marginal UtilityAs you have studied earlier law of diminishing marginal utility states that as we consume more and more units of a commodity the utility derived from each successive unit goes on decreasing The consumer will be ready to pay more for those units which provide him more utility and less for those which provide him less utility It implies that he will purchase more only when the price of the commodity falls2 Income Effect

When price of a commodity falls purchasing power or real income of the consumer increases which enables him to purchase more quantity of the commodity with the same money income Let us take an example Suppose you buy 4 ice creams when price of each ice cream is ` 25 If price of ice creams falls to ` 20 then with same money income you can buy 5 ice creams now3 Substitution EffectWhen price of a commodity falls it becomes comparatively cheaper as compared to its substitutes (although price of substitutes has not been changed) This will lead to rise in demand for the given commodity For example if coke and Pepsi both are sold at ` 10 each and price of coke falls Now coke has become relatively cheaper and will be substituted for Pepsi It will lead to rise in demand for coke4 Change in Number of BuyersWhen price of a commodity falls some old buyers may demand more of the commodity at the reduced price and some new buyers may also start buying this commodity who were not in a position to buy it earlier due to higher price This will lead to increase in number of buyers when price of the commodity falls As a result demand for the commodity rises when its price falls5 Diverse Uses of a CommoditySome commodities have diverse uses like milk It can be used for drinking for sweet preparation for ice cream preparation etc If price of milk rises its use may be restricted to important purpose only This will lead to reduction in demand for other less important uses When price of milk falls it can be put to other uses also leading to rise n demand for itQ9 EXCEPTIONS TO THE LAW OF DEMANDYou have studied in law of demand that a buyer is willing to buy more quantity of a commodity at a lower price and less of it at a higher price But in certain circumstances a rise in price may lead to rise in demand These circumstances are called Exceptions to the Law of Demand Some important exceptions are1 Giffen GoodsGiffen goods are special type of inferior goods in which negative income effect is stronger than negative substitution effect Giffen goods do not follow law of demand as their demand rises when their price rises Examples of Giffen goods are jowar and bajra etc2 Status Symbol GoodsSome goods are used by rich people as status symbols eg diamonds gold jewellary etc The higher the price the higher will be the demand for these goods When price of such goods falls these goods are no longer looked at as status symbol goods and tehrefore therir demand falls3 NecessitiesCommodities such as medicines salt wheat etc do not follow law of demandbecause we have to purchase them in minimum required quantity whatever their price may be4 Goods Expected to be ScarceWhen the buyers expect a scarcity of a particular good in near future they start buying more and more of that good even if their prices are rising For example during war famines etc people tend to buy more of some goods even at higher prices due to fear of their scarcity in near future

Political Science

Constitution of India-The

Preamble

The preamble-

Preamble-

The preamble is the most precious part of the constitution We the people of India having solemnly resolved to constitute India into a Sovereign Socialist Secular Democratic Republic and to secure to all its citizensA preamble is an introductory and expressionary statement in a document that explains the documents purpose and underlying philosophy When applied to the opening paragraphs of a statute it may recite historical facts pertinent to the subject of the statuteNature and purpose of the constitution-Purpose of the Constitution dictates permanent framework of the government to form a more perfect union to establish justice and ensure peace of thenationconstitution provide principles how the government can run itself following the rules and laws written in the constitution of each state keeps them balanced

Answer the following questions-

1 What is preambleA preamble is an introductory and expressionary statement in a document that explains the documents purpose and underlying philosophy2 What is the nature and

purpose of the constitutionConstitution dictatespermanent framework of the government to form a more perfect union to establish justice and ensure peace of the nation

Homework-Learn

Accounts Compatibilty mode

1MEANING OF PARTNERSHIPPartnership is a form of business organisation where two or more persons join hands to run a business They share the profits and losses according to the agreement amongst them According to the Indian Partnership Act 1932 ldquoPartnership is relation between persons who have agreed to share profits of a business carried on by all or any one of them acting for allrdquo For example one of your friends has passed class XII from National Institute of Open Schooling (NIOS) and wants to start a business Heshe approaches you to join in this venture Heshe wants you to contribute some money and participate in the business activities Both of you if join hands constitute a partnership2CHARACTERISTICS1048698 Agreement A partnership is formed by an agreement The agreement may be either oral or in writing It defines the relationship between the persons who agree to carry on business It may contain the terms of sharing profit and the capital to be invested by each partner etc The written agreement is known as partnership deed1048698 Number of persons There must be at least two persons to form a partnership

The maximum number of partners in a partnership firm can be 50 according toCompanies Act 20131048698 Business The Partnership is formed to carry on business with a purpose of earning profits The business should be lawful Thus if two or more persons agree to carry on unlawful activities it will not be termed as partnership1048698 Sharing Profits The partners agree to share profits in the agreed ratio In caseof loss all the partners have to bear it in the same agreed profit sharing ratio10486981048698Mutual Agency Every partner is an agent of the other partners Every partner can bind the firm and all other partners by hisher acts Each partner will be responsible and liable for the acts of all other partners10486981048698Unlimited liability The liability of each partner except that of a minor is unlimited Their liability extends to their personal assets also If the assets of the firm are insufficient to pay off its debts the partnersrsquo personal property can be used to satisfy the claim of the creditors of the partnership firm10486981048698Management All the partners have a right to mange the business However they may authorize one or more partners to manage the affairs of the business on their behalf10486981048698Transferability of Share No partner can transfer hisher share to any one including hisher family member without the consent of all other partners3PARTNERSHIP DEEDAgreement forms the basis of partnership The written form of the agreement is which a document of partnership is It contains terms and conditions regarding the conduct of the business It also explains relationship between the partners This document is called partnership deed Every firm can frame its own partnership deed in which the rights duties and liabilities of the partners are stated in detail It helps in settling the disputes arising among the partners during the general conduct of business 4CONTENTS OF PARTNERSHIP DEEDThe partnership deed generally contains the following (i) Name and address of the partnership firm(ii) Nature and objectives of the business(iii) Name and address of each partner(iv) Ratio in which profits is to be shared(v) Capital contribution by each partner(vi) Rate of Interest on capital if allowed(vii) Salary or any other remuneration to partners if allowed(viii) Rate of interest on loans and advances by a partner to the firm(ix) Drawings of partners and interest thereon if any(x) Method of valuation of goodwill and revaluation of assets and liabilities on the reconstitution of the partnership ie on the admission retirement or death of a partner(xi) Settlement of disputes by arbitration(xii) Settlement of accounts at the time of retirement or death of a partner5IN ABSENCE OF PARTNERSHIP DEEDThe partnership deed lays down the terms and conditions of partnership in regard to rights duties and obligations of the partners In the absence of partnership deed there may arise a controversy on certain issues like profit sharing ratio interest on

capital interest on drawings interest on loan and salary of the partners In such cases the provisions of the Indian Partnership Act becomes applicableSome of the Issues are(i) Distribution of Profit Partners are entitled to share profits equally(ii) Interest on Capital Interest on capital is not allowed(iii) Interest on Drawings No interest on drawing of the partners is to be charged(iv) Interest on Partnerrsquos Loan A Partner is allowed interest 6 per annum on the amount of loan given to the firm by himher(v) Salary and Commission to Partner A partner is not entitled to anysalary or commission or any other remuneration for managing the business

History TOPIC-TOWARDS INDEPENDENCE AND PARTITION THE LAST PHASE (1935-1947)

SUB TOPIC-IMPORTANT POLITICAL DEVELOPMENTS ndash GROWTH OF SOCIAL IDEAS

Socialism is a political social and economic philosophyLike in other parts of the world the Russian revolution of 1917 served as a great inspiration for revolutionaries in India who at that time were engaged in the struggle for liberation from British ruleSocialist ideas led to the formation of communist party of IndiaJAWAHARLAL NEHRU Among the early Congress leaders Jawaharlal Nehru was very much impressed and influenced by the Socialist ideas He also learnt about the Economic activities of the Soviet Union after the Bolshevic Revolution 1917 He made full use of them in IndiaThe election of Jawaharlal Nehru and Subhas Chandra Bose showed the Left wing tendency within CongressJawaharlal Nehru demanded economic freedom along with political freedom of the people in order to end the exploitation of masses

Nehrus working committee included three socialists leaders The Lucknow session was a landmark in the evolution of socialist ideas of the congressSUBHAS CHANDRA BOSE ndash Subhas Chandra Bose had socialist leaning Both Jawaharlal Nehru and Subhas Chandra Bose were known as leftist Congress men Later on National Congress divided into Leftist and rightist campCONGRESS SOCIALIST Within the Congress some leaders formed the Congress Socialist partyPattavi Sitaramyya Sardar Patel Rajendra Prasad had hostile attitude towards the Congress Socialist partyJawaharlals attitude was hesitant

1 QUESTION ndash Mention name of two Congress leaders who had socialist leaning

1ANSWER ndash Subhas Chandra Bose and Jawaharlal Nehru2QUESTION- In which session of the congress Jawaharlal elaborated his Socialist ideas2 ANSWER ndash Lucknow and Faizpur Session in December 1935 and 19363QUESTION ndash Why Congress was sharply divided into leftist and rightist camp 3ANSWER ndash Subhas Chandra Bosersquos attempt to seek re election for congress presidentship in 1939sharply divided the National Congress into Leftist and Rightist camp4 QUESTION ndash Who was MN Roy 4 ANSWER ndash Manabendra Roy first formed the Communist Party of India outside the country at Tashkent in 19205QUESTION ndash Who formed the Congress Socialist Party within the Congress5 ANSWER ndash Jaya Prakash Narayan Achyut Patwardhan Acharya Narendra Dev Ram Mohan Lohia Aruna Asaf Ali6QUESTION ndash When was the Congress Socialist Party formed What was its object6 ANSWER ndash 1934The Congress Socialist Party sought to work out socialist programme through the Congress They joined hands with the Congress and wanted to carry

Subhas Chandra Bose being expelled from the congress after the Tripuri rift he formed Forward BlockThere were basic differences between the Congress Socialists and the communistsTRADE UNION ACTIVITIES Maximum working class people lived in Bombay and Calcutta The working and living conditions of those workers were very miserable In this situation Shasipada Banerjee NM Lokhande protested against the oppression of the working class peopleThe first Trade Union Madras Labour Union was formed in 1918 by BP WadiaIndustrial strikes took place in Kanpur Calcutta Madras Jamshedpur and Ahmedabad AITUC was formed in Bombay in 1927 The growth of Trade union among the workers was slow because of the fear of the dismissal of the jobIn the mean time the Moderates as well as Communists left AITUC and formed separate organization

on National struggle with the help of workers and peasant class of the society7 QUESTION ndash What was the name of the party founded by Subhas Chandra Bose7 ANSWER- Forward Block8QUESTION ndash Who was Shasipada Banerjee8 ANSWER ndash Shasipada Banerjee was a radical Brahmo He founded a working menrsquos club to protest against exploitation of the British rulers towards the working class of India9 QUESTION ndash What was the weekly published by NM Lokhande9ANSWER- Dinabandhu10 QUESTION ndash Who founded Bombay Mill-Hands Association and in which year10 ANSWER- NM Lokhande in189011 QUESTION- Who was BP WadiaANSWER- BPWadia was the founder of Madras Labour Union in191812 QUESTION- What was the name of the first labour union of India12 ANSWER- Madras Labour Union13 QUESTION Who founded the Majur Mahajan 13 ANSWER GANDHIJI14 QUESTION What was the full form of AITUC When it was formed14 ANSWER All India Trade Union Congressin 192715QUESTION Who formed the Red Trade Union Congress and in which year15ANSWER The Communists formed the Red Trade Union Congress16 QUESTION What do you mean by Socialism16 ANSWER Socialism describes any political and economic theory that says the community rather than individuals should own and manage property and natural resources

Subject Eng Literature (The Tempest ndash William Shakespeare) Topic Act III Scene 3 Lines 1 to 52 (Line 52 ndash Brother my lord the Duke Stand to and do as we) Date 13th April 2020 (4th Period)

[Students should read the original play and also the paraphrase given in the school prescribed textbook]Summary Questions amp Answers

o Alonso Sebastian Antonio Gonzalo Adrian Francisco and others wandered about the island in search of Ferdinand and gets tired and hungry of the toil and at the same time gives up all hope of finding him

o Antonio and Sebastian are happy that Alonso is out of hope and decide to make another attempt on his life that night when being so tired they will be sleeping soundly

o Suddenly a solemn and strange music is heard in the air and several strange shapes enter bringing in a banquet These strange shapes then dance round it with gestures of salutation and then inviting the King to eat they depart

o Seeing this strange scene all are inclined to believe the tales told by travelers that there truly are ldquounicornsrdquo and ldquothe phoenixrsquo thronerdquo

1 ALONSO What harmony is this My good friends hark (L18-27)

GONZALO Marvellous sweet music

[Enter several strange shapes bringing in a banquet

they dance about it with gentle actions of salutation

and inviting the King and his companions to eat they depart]ALONSO Give us kind keepers heavens What were theseSEBASTIAN A living drollery Now I will believe

That there are unicorns that in Arabia

There is one tree the phoenixrsquo throne one phoenix

At this hour reigning thereANTONIO Ill believe both

And what does else want credit come to me

And Ill be sworn rsquotis true Travellers neer did lie

Though fools at home condemn rsquoem

(a) How did Prospero present an amazing spectacle before Alonso and his companions

Using his magic powers Prospero ordered strange shapes to lay a banquet before Alonso and his companions The shapes brought several dishes with tasty eatables in them They placed the dishes on a table before Alonso and his companions Then the strange shapes began to dance gracefully around the banquet While dancing they made gestures inviting them to eat the food Then suddenly the shapes disappeared(b) Who were the guests at the strange banquet Describe the lsquoliving drolleryrsquo

Alonso Sebastian Antonio Gonzalo Adrian and Francisco were the guests at the strange banquet

The term ldquoliving drolleryrdquo refers to live entertainment show In this context when Alonso the King of Naples Sebastian his brother Antonio the treacherous brother of Prospero Gonzalo the kind and loyal councillor to the King Adrian and Francisco came to the island they were hungry and weary in their spirits They heard a solemn and strange music They were shocked to see several strange shapes bringing in a banquet and these shapes danced about it with gentle action of salutation inviting the King and his companions to eat After this Sebastian described this show as lsquoliving drolleryrsquo(c) What is lsquophoenixrsquo What are lsquoUnicornsrdquo

The term lsquophoenixrsquo refers to a mythical Arabian bird which lived alone and perched on a solitary tree After one hundred years it expired in flames and rose again from its own ashes

lsquoUnicornsrsquo refers to the mythological four-footed beasts having horns in the centre of their foreheads When the horns are ground into powder the powder was believed to be

an aphrodisiac(d) How does Sebastian explain the puppet show OR Why does the speaker now believe in unicorns and phoenix

Sebastian finds several strange shapes bringing in the banquet They invite the king and his party for dinner and soon depart He tells that if such a strange sight can be a reality there is nothing incredible in the world and from the present moment he will believe anything He says that it is a strange dumb show enacted not by puppets but by living beings It is stranger than a travellerrsquos tale Seeing such a thing

before his own eyes he will no longer disbelieve the story about unicorns and phoenix(e) How do the other characters present respond to this living drollery

At the sight of the lsquoliving drolleryrsquo like Sebastian Gonzalo and Antonio too acted strangely Antonio told that he too now believes in unicorns and phoenix and anything else that seems to be incredible He too now believes in travellersrsquo tales Gonzalo told that if he would report those happenings in Naples nobody will believe him He considers that those gentle shapes were gentler in manner in comparison to the living beings Alonso was at first sight suspicious and told them that those strange shapes conveyed their meaning in expressive gestures when they seemed to lack speech by their movements and sounds Francisco was amazed at their mysterious disappearance

2 ALONSO Not I

(Line 43-52)GONZALO Faith sir you need not fear When we

were boysWho would believe that there were mountaineers

Dewlapped like bulls whose throats had hanging at rsquoem

Wallets of flesh Or that there were such men

Whose heads stood in their breasts Which now we find

Each putter-out of five for one will bring us

Good warrant ofALONSO I will stand to and feed

Although my lastmdashno matter since I feel

The best is past Brother my lord the Duke

Stand to and do as we

(a) How does Alonso respond at the spectacle of the shapes which were sent to them at the instruction of Prospero

After seeing the strange sight of appearing and disappearing of the shapes sent by Prospero to arrange a banquet for them Alonso says that his surprise at having seen those creatures is infinite and he is fully justified in feeling so much surprise He thinks that their shapes their gestures and the sounds they made were indeed amazing Although they do not possess the gift of speech yet they were able to convey their

thoughts by means of their gestures only

(b) What does Prospero say about the views expressed by Alonso regarding the shapes What does Francisco think about the shapesAfter hearing Alonsorsquos views about the shapes Prospero says that this manrsquos praise of the spirits is rather hasty He means to say that Alonso has shown great haste in reaching the conclusion about the shapes Francisco is amazed to see that those shapes disappeared in a mysterious way(c) What does Sebastian ask Alonso to doSebastian tells Alonso that the shapes having disappeared should not matter to them because they have left the eatables behind He asks Alonso to enjoy eating as they are extremely hungry but the king does not accept his offer of enjoying the dishes(d) How does Gonzalo try to dispel Alonsorsquos fear of those strange shapes What kind of references does he give to AlonsoGonzalo says that those who have travelled abroad have reported seeing even stranger sights than these shapes that Alonso and his companions have beheld Hence there is no reason to feel afraid of these shapes Gonzalo further adds that in his younger days he had heard strange stories from travelers and Alonso might have heard similar stories For instance it was said that there existed a certain race of

human beings who had huge lumps of flesh hanging at their throats and who therefore resembled bulls Then Gonzalo tells about a race of human beings whose heads were located at their breasts Gonzalo says that such stories were not believed by most people in those days but now-a-days these stories have become common(e) Explain the following lsquoEach putter-out of five for onersquoEnglish travellers often insured their trips with London brokers Those that went on foreign travels those days used to deposit a certain amount with some firm or company in London before their departure If the travelers failed to return the money was forfeited by the company with which it had been deposited But this money was repaid five-fold if the travelers returned safe and sound In this way a traveler stood a great chance of recovering the entire cost of his

travels(f) Give the explanatory meanings of the following expressions in the context of the above extract (i) Dewlapped (ii) Wallets of flesh

(iii) Putter-out(i) Dewlapped having big lumps of flesh at the necks(ii) Wallets of flesh large masses of flesh looking like bags(iii) Putter-out to invest money before commencing the travel

  • General methods of preparation of hydrogen
  • Chapter Dimensional Analysis (Summary)
    • Properties of Charge
Page 38:   · Web viewSubject. Topic. Summary. Execution. Hindi. व्याकरण. शरीरके अंगो के नाम लिखिए. 1) आँख 2) नाक 3

the study of aggregates

National income aggregate savings and aggregate investments are major concepts dealt within macroeconomics style

It studies the determination of general price levels

It investigates into the problem of unemployment and the achievement of employment

It studies the aspect of decision making at the aggregate and national levels

It includes all growth theories whether related to developed or developing economies it also includes the study of economic systems and the working of the economy under different systems

Note Both Micro and macro economics are complementary and should be fully utilized for proper understanding of an economy

1It studies economic aspect of an individual unit2It deals with individual incomeConsumption and savings

3 It facilitates determination of price of any product or factors of production

4 Itrsquos scope is narrow and restricted to individual unit

1It studies the economy as a whole

2It deals with the national income aggregate consumption and aggregate savings3 It facilitates determination of general price level in an economy

4 Itrsquos scope is wide as it deals with economic units on the national level

ACCOUNTS

Introduction to Accounting and Book-keeping

Today I am going to share you the meaning of Accounting and Book-keeping and its related terms bullAccounting bullBook Keeping bullAccountsbullTypes Of Accounts bullAccounting Cycle

bull Meaning of accounting

Ans ) Accounting is the art and science of recording classifying and summarising monetary transactions

bull Meaning of Book-keeping

Ans) Bookkeeping is the art of recording business transactions with the view of having a permanent record of them and showing their effect on wealth

bull Meaning of account

Ans) The term account means a record of

business transactions concern a particular person of firm asset or income or expense It is a summarised record of all transactions which take place in an accounting year

bull Types of accountsPersonal accounts ndash Personal accounts relating

to person and Organisation are known as personal accounts Example Ramrsquos Account ABC amp Co Account etc

Real account - The accounts related to tangible and intangible assets are called real accounts Example Cash Account Furniture Account etc

Nominal account- Accounts related to expenses losses incomes and gains are known as nominal accounts Example Wages Account Salary Account Discount Account etc

bull Accounting cycle Accounting cycle refers to a complete sequence of accounting activities It begins with recording of transactions and ends with the preparation of a balance sheet

Chemistry TopicAtomic Structure

Thomsonrsquos atomic modelThomson (1898) was the first to propose the model of an atomHe proposed that an atom can be regarded as a uniform sphere of positive electricity in which requisite number of electrons are embedded evently to neutralize the positive chargeThis is just like plums embedded in a pudding or seeds evently distributed in red spongy mass of a watermelonThis model of atom is known as ldquoPlum-Pudding modelrdquo or

Q1)What is the fundamental constituents of atomAns Electron Proton and neutrons are the fundamental constituents of atomQ2)What is the value of fundamental unit of electricityAnsThe charge carried by one electron is sad to be the fundamental unit of electricityIts magnitude is 48times10-10esuOr 1602times10-19C Q3)Name the element containing no neutronAnsOrdinary hydrogen atom or protium 1H1

Types of AccountPersonal AccountReal AccountNominal AccountBalance Sheet (opening)

ldquowatermelon modelrdquoThis model could explain the electrical neutrality of an atom but failed to explain the result of scattering experiment carried out by Rutherford in 1911So it was rejected ultimately

Q4)Why is an electron called universal particleAns Itrsquos mass and Charge are independent of its source

EVS Chapter 1 ndash Modes of Existence

Modes of existence When one speaks normally about the mode of existence of some group or individual one refers to their customs their mode of being their ethology their habitat in some way their feeling for a placeDifferent modes of exixtence are ndash

1 Hunting ndashGathering2 Pastoral3 Agricultural4 Industrial

1 Hunting and gathering Hunting and gathering mode of existence is characterized by obtaining food from hunting wild animals including fishing and gathering wild plants From their earliest days the hunter-gatherer diet included various grasses tubers fruits seeds and nuts Lacking the means to kill larger animals they procured meat from smaller game or through scavenging

Societies that rely primarily or exclusively on hunting wild animals fishing and gathering wild fruits berries nuts and vegetables to support their diet are called hunting and gathering societies

At least this used to be practice of human beings before agriculture is invented As their brains evolved hominids developed more intricate knowledge of edible plant life and growth cycles

Q) Write the features of Hunting ndash gathering societiesAns - There are five basic characteristics of hunting and gathering societies

i The primary institution is the family which decides how food is to be shared and how children are to be socialized and which provides for the protection of its members

ii They tend to be small with fewer than fifty members

iii They tend to be nomadic moving to new areas when the current food supply in a given area has been exhausted

iv Members display a high level of interdependence

v Labor division is based on sex men hunt and women gather

Political Science

Introduction to political science

Political science occasionally called politology is a social science which deals with systems of governance and the analysis of political activities political thoughts associated constitutions and political behaviorThe study of political science involves the study of both the

Answer the following questions-1 What is political science

Political science occasionally called politology is a social science which deals with systems of governance and the analysis of political activities political thoughts associated constitutions and political behavior

2 Short notes-

traditional and modern theories of politicsTraditionalClassical political sciencepolitical theory-Traditional political science is the study of politics before Second World War The methodology to study Politics was traditional (legal formaletc) the definition of politics traditional (Politics begins and end with state)area of study (constitution state machinery)was traditionalModern Political scienceModern political theory-Modern Political Theory critically examines the contemporary state of political theory making an assessment of the achievement and limitations of the Behavioural Revolution in its totality and reviews objectively the major paradigms and conceptual frameworks adopted by the disciplineContemporary attempts at the development of an integrated political theory involving the use of both traditional and modern concepts approaches and theories-Around late 1960s several political scientists realized the importance of both the traditional political theory and modern Political theory They began building an integrated theory of politics involving a systematic mixture of traditional and modern studies of politics It was held that the study of a complex and vast field like politics needs both traditional as well as

Classical political theory Modern Political theory

Homework-Learn

modern concepts and approaches for studying itrsquos all aspects

Subject Eng Literature (The Tempest ndash William Shakespeare) Topic Act I Scene 1 Lines 1 to 32 (Line 32 ndash Gonzalo hellip If he be not born to be hanged our case is miserable) Date 13th April 2020 (3rd Period)

[Students should read the original play and also the paraphrase given in the school prescribed textbook]Summary Questions amp Answers

[SUMMARY OF THE ENTIRE SCENE]

o The play starts with the scene of a severe storm at sea Alonso (King of Naples) Sebastian (Alonsorsquos brother) Ferdinand (Alonsorsquos son) Gonzalo Antonio (the usurping Duke of Milan) are in a ship in the midst of the storm

o The mariners are trying their best to control the vessel from running aground and are totally following the orders of their Master the Boatswain They have scant success

o The mariners become extremely unhappy and annoyed when most of the passengers arrive on the deck thereby hampering their effort to save the ship There is serious confrontation between them and the passengers who are part of the Kingrsquos entourage

o The mariners could not save the ship

SUMMING-UP

(i) Vivid description of the scene which gives a realistic description of terror and confusion of a tropical storm

(ii) Shows Shakespearersquos accuracy of knowledge in describing the naval operations and also matters of seamanship

(iii) The opening scene justifies the title ndash The Tempest

UNANSWERED QUESTIONS

(i) The King always travels with his entire fleet including his soldiers Where

(1) GONZALO Nay good be patient (Line 15-26)BOATSWAIN When the sea is Hence What cares these

roarers for the name of the king To cabin silence Trouble us not

GONZALO Good yet remember whom thou has aboardBOATSWAIN None that I more love than myself You are a

councillor if you can command these elements to silence and work

the peace of the present we will not hand a rope more use your authority If you cannot give thanks you have

lived so long and make yourself ready in your cabin for the mischance of the hour if it so hap [To the Mariners]

Cheerly good hearts [To Gonzalo] Out of our way I say

(a) To whom is the boatswain speaking What does he mean by lsquoNone that I more love than myselfrsquo

The Boatswain is speaking to Gonzalo the honest old councilor of the Duke of MilanBy using the words ndash lsquoNone that I love more than I love myselfrsquo means that for the Boatswain nobody is dearer to him than his own life

(b) What were the conditions that made the boatswain react in this way

The Boatswain reacts in this way because the storm is at sea and Alonso King of Naples Sebastian his brother Ferdinand his son Gonzalo Antonio the usurping Duke of Milan on board are in distress and in panic Thus they have rushed to the deck interrupting the work of the mariners

(c) What hope does Gonzalo take from the attitude of the boatswain

The insolent and authoritative attitude of Boatswain makes Gonzalo feel comforted He tells that there are no signs that the Boatswain will be drowned But his facial appearance and attitude shows that he is destined to die on land by hanging which in effect means that all on board will be saved Otherwise all the persons on board are doomed

(d) How can they lsquomake yourself ready in your cabinrsquo For what were they asked to make ready themselves

In order to make themselves ready in their cabin the

were the other ships

(ii) Why was the ship in that area Where was it coming from or going where

(iii) The ship broke apart What happened to those who were in the ship

passengers on board must prepare for death which they will possibly soon have to meetThey can retire to their cabins and offer prayers to the Almighty to save them from drowning

(e) What does the boatswain say when he is asked to be patient What does he order to the royal party

When the boatswain is asked to be patient and remain calm he says that he will be patient only when the storm will be over and the sea will be calm but as long as the storm blows and there is danger to the ship he cannot think of being patient He orders the royal party to go to the cabin and leave the mariners to their work

(2) GONZALO I have great comfort from this fellow (Line 27-36)

Methinks he hath no drowning mark upon him his complexion is perfect

gallows Stand fast good Fate to his hanging Make the rope of his destiny our cable for our own doth little advantage If he be not born to be hanged our case is miserable

(a) Why does Gonzalo regard the Boatswain in the midst of danger

In the midst of danger Gonzalo regards the boatswain because he feels that the Boatswain is a source of comfort and is bent upon to do his work sincerely which in this case is saving the ship and its passengers from the severest of raging storm

(b) What reasons does Gonzalo give when he says that none in the ship will die of drowning

Gonzalo is almost sure that none in the ship will die by drowning His says that there is no mark on the face of the boatswain that indicates that he will die by drowning On the other hand the lines on his face are strong indications that he will be hanged to death Therefore there shall be no danger of the shiprsquos sinking

(c) Explain the following ldquoStand fast good Fate to his hanging Make the rope of his destiny our cable for our own doth little advantage If he be not born to be hanged our case is miserablerdquo

The stated lines mean that if the will of destiny is to be carried out then the ship will not get wrecked and all the passengers will be saved The safety of the passengers therefore depends upon the will of fate being carried out in the case of the boatswain If however the boatswain is not to die by hanging then the passengers are also very unsafe because in that case the ship is likely to sink

(d) What order does the Boatswain give to the sailors

when he re-enters What does he say about the crying of the fellows inside the cabin

The boatswain orders the sailors to bring the topmast lower and bring the ship close to a stationary position with the help of the main sail He says that the fellows inside the cabin are moaning and crying in their distress louder than his voice and louder even than the roaring of the storm

Class XII (ScienceCommerceHumanities) Subject Topic Summary Execution

Computer Science

PropositionalLogic

Propositional logic is a procedure to provide reasoning through statementProposition A ststement that results in True or False is said to be proposition There are two types of propositionSimple proposition amp compound propositionSimple proposioton A simple proposition is one that is not a part of any other proposition Such sentential form of proposition is symbolized with english letters in short For example Ram is a claver student (TrueFalse)Where do you live (Not in True or False)Grapes are sweet (TrueFalse)It rains today (TrueFalse)Here we can see some statements anwer would be true or false but some staements answer can not give in terms of true or false Thus the sentences which can be answered in true or false are known as simple propositionAssigning propositon to a variableThe general syntax to assign propostion to a variable is as followsVariable = Simple propositonFor example A=Ram is a clever studentB= Grapes are sweetC= it rains todayCompound proposition

helliphellipto be continued in next classhelliphellipMath Relation Relation If A and B are two non-empty sets

then a relation R from A to B is a subset of AxB If R A x B and (a b) R then we say that a sube isinis related to b by the relation R written as aRbeg Let A be the set of students of class XII and B be the set of students of class XI Then some of the examples of relation from A to B arei) (a b) AXB a is brother of bisinii) (a b) AXB age of a is more than age of isinb Types of relation In this section we would like to study different types of relations We know that a relation in a set A is a subset of A times A Thus the empty set φ and A times A are two extreme relations For illustration consider a relation R in the set A = 1 2 3 4 given by R = (a b) a ndash b = 10 This is the empty set as no pair (a b) satisfies the condition a ndash b = 10 Similarly R = (a b) | a ndash b | ge 0 is the whole primeset A times A as all pairs (a b) in A times A satisfy | a ndash

Example 1 Let A be the set of all students of a boys school Show that the relation R in A given by R = (a b) a is sister of b is the empty relation and R = (a b) the primedifference between heights of a and b is less than 3 meters is the universal relationSolution Since the school is boys school no student of the school can be sister of any student of the school Hence R = φ showing that R is the empty relation It is also obvious that the difference between heights of any two students of the school has to be less than 3 meters This shows that R = A times A is primethe universal relation Example 2 Show that the relation R in the set 1 2 3 given by R = (1 1) (2 2) (3 3) (1 2) (2 3) is reflexive

b | ge 0 These two extreme examples lead us to the following definitionsDefinition 1 A relation R in a set A is called empty relation if no element of A isrelated to any element of A ie R = φ A times AsubDefinition 2 A relation R in a set A is called universal relation if each element of A is related to every element of A ie R = A times A Both the empty relation and the universal relation are some times called trivial relation Definition 3 A relation R in a set A is called(i) reflexive if (a a) R for every a Aisin isin(ii) symmetric if (a1 a2) R implies that (aisin 2a1)

R for all aisin 1 a2 Aisin(iii) transitive if (a1 a2) R and (aisin 2 a3) R isinimplies that (a1 a3) R for all aisin 1 a2 a3 AisinDefinition 4 A relation R in a set A is said to be an equivalence relation if R is reflexive symmetric and transitive

but neither symmetric nor transitiveSolution R is reflexive since (1 1) (2 2) and (3 3) lie in R Also R is not symmetric as (1 2) R but (2 1) isin notinR Similarly R is not transitive as (1 2) R and (2 3) R but (1 3) R isin isin notinExample 3 Show that the relation R in the set Z of integers given byR = (a b) 2 divides a ndash b is an equivalence relationSolution R is reflexive as 2 divides (a ndash a) for all a Z isinFurther if (a b) R then 2 divides a isinndash b Therefore 2 divides b ndash a Hence (b a) R which shows that R is isinsymmetric Similarly if (a b) R and (b c) R isin isinthen a ndash b and b ndash c are divisible by 2 Now a ndash c = (a ndash b) + (b ndash c) is even (Why) So (a ndash c) is divisible by 2 This shows that R is transitive Thus R is an equivalence relation in ZExample 4 Let L be the set of all lines in a plane and R be the relation in L defined as R = (L1 L2) L1 is perpendicular to L2 Show that R is symmetric but neither reflexive nor transitiveSolution R is not reflexive as a line L1 can not be perpendicular to itself ie (L1 L1) R notinR is symmetric as (L1 L2) Risin

L1 is perpendicular to L2rArr L2 is perpendicular to L1rArr (L2 L1) RrArr isin

R is not transitive Indeed if L1 is perpendicular to L2 and L2 is perpendicular to L3 then L1 can never be perpendicular to L3 In fact L1 is parallel to L3 ie (L1 L2) R isin(L2 L3) R but (L1 L3) Risin notin

Chemistry Solid state Characteristics if Solids(i)The particles are locked in fixed positions they are unable to change their relative positions and this brings a definite shape and volume of a solid(ii)In a solid the constituent particles are held by strong forces of attractionThe forces of attraction may be bonding or non bonding(iii)The constituent particles in a solid pack together as closely as possibleoccupying most of the available space within the solidThus the empty space in a solid is very smallThis makes a solid highly rigid and nearly incompressibleThis also explains why a solid has high density and exhibits slow diffusionClassification of Solids

Q1)Define Crystalline solids AnsA Solid that has a definite geometrical shape and a sharp melting pointand whose constituent particles (atomsmolecules or ions) are arranged in a long range order of definite pattern extending throughout the solid is called a crystalline solidExNaClQ2)Define Amorphous solids AnsA solid that does not have a definite shape and a sharp melting pointand whose constituent particles (atomsmolecules or ions) are not arranged in a definite pattern is called an amorphoussolid

Crystalline solidsAmorphous solids

ExGlassRubberQ3)Classify Crystalline Solids Crystalline Solids

Physics Coloumbrsquos Law (Summary)

Before Going Into Coloumbrsquos Law We Will First Learn What is Charge Properties of Charge and Always remember that charge is quantized ie a body always have static charge of magnitude equal to some integral multiple of fundamental electronic charge e= 16 x 10- 19 C

Charge is the property of matter that causes it to produce and experience electrical and magnetic effects The study of the electrical charges at rest is called electrostatics When both electrical and magnetic effects are present the interaction between charges is referred to as electromagnetic

There exist two types of charges in nature positive and negative Like charges repel and unlike charges attract each other

The type of charge on an electron is negative The charge of a proton is the same as that of an electron but with a positive sign In an atom the number of electrons and the number of protons are equal The atom is therefore electrically neutral If one or more electrons are added to it it becomes negatively charged and is designated as negative ion However if one or more electrons are removed from an atom it becomes positively charged and is called a positive ion

The excess or deficiency of electrons in a body gives the concept of charge If there is an excess of electrons in a body it is negatively charged And if there is deficiency of electrons the body becomes positively charged Whenever addition or removal of electrons takes places the body acquires a charge

The SI Unit of charge is coulomb (C) In SI units the current is a fundamental quantity having a unit of ampere (A) The unit of charge is defined in terms of the unit of current Thus one coulomb is the charge transferred in one second across the section of a wire carrying a

Ionic SolidsMetallicSolids

Molecular Solids

current of one ampere

As q = It we have1 C = (1 A) (1 s)

The dimensions of charge are [A T]

Properties of Charge

(1) Quantization of Charge Electric charge can have only discrete values rather than any value That is charge is quantized The smallest discrete value of charge that can exist in nature is the charge on an electron given as

e = plusmn 16 x 10- 19 C

This is the charge attained by an electron and a protonA charge q must be an integral multiple of this basic unit That is

Q = plusmn ne where n = 1 2 hellip

Charge on a body can never be (frac12)e (23)e or 57e etcWhen we rub a glass rod with silk some electrons are transferred from the rod to the silk The rod becomes positively charged The silk becomes negatively charged The coulomb is a very large amount of charge A typical charge acquired by a rubbed body is 10 - 8 C

Biology Reproduction in organisms

Welcome to this new session 2020-21Today in this first chapter we mainly discuss about reproduction types needs and life span of some organismsWe also discuss about difference between sexual and asexual reproduction

Q1 What is reproductionReproduction is defined as a biological processin which an organism gives rise to young onessimilar to itselfQ2 What are the needs of reproductionbulli) Reproduction maintain life on earthii) It enables the continuity of the species generation after generationiii) It creates genetic variation among populationsQ3 Define Life span and write some orgnisms life spanbull Life span is the period from birth to

the natural death of an organism- OrganismsLife span1 Butterfly 1 - 2 weeks2 Fruit fly 30 days3Dog 10-13 years4 Rose5-7 years5 Tortoise100-150 years6 Banyan Tree -200 - 250 yearsQ4 Reproduction is of two types in case ofanimals but in case of plants vegetative propagation is also present

Asexual Reproduction Sexual Reproductioni) Always uniparentalii) Gametes are not involvediii) Only mitotic division involvediv) Somatic cells of parents are involvedv) Offsprings are genetically similar to the parents

i) Usually biparentalii) Gametes are involvediii) Meiosis occurs during gametogenesis Mitosis occurs after fertilisationiv) Germ cells of the parents are involvedv) offsprings are genetically different from the parents

COMMERCE BUSINESS ENVIRONMENT

Welcome to the new sessiontoday we are going to start the first chapter of Class XII The name of the chapter is Business Environment

Already many of you have got some idea about the word business environment form the first chapter of business studies in class XI

In todayrsquos world every business enterprise is a part of the society It exists and operates in association with various groups in society such as customers suppliers competitors banks and financial institutions government agencies trade unions media and so on All these groups influence the functioning of business in one way or the other They constitute the environment of businessConcept of Business Environment

The term lsquobusiness environmentrsquo refers to the sum total of all individuals institutions and other forces that lie outside a business enterprise but that may influence its functioning and performance

The main features of business environment Totality of External forces General and Specific forces Interrelatedness Complexity Dynamic Uncertainty

Prepare the following questions from todayrsquos assignment

2 What do you mean by business environment

The term lsquobusiness environmentrsquo means the aggregate of all forces factors and institutions which are external to and beyond the control of an individual business enterprise but they may influence its functioning and performance Business environment is the macro framework within which a business firm a micro unit operates It consists of several interrelated and interacting elements

2 Explain the main features of business environment in brief

Totality of External forces-Business environment is the sum total of all things external to a business environment

General and Specific forces-It

Relativity

The Interrelation between business and its environment

The business enterprise is an open system It continuously interacts with its environment It takes inputs (such as raw materials capital labour energy and so on) from its environment transforms them into goods and services and sends them back to the environment

Fig 1 Business Environment Relationship

includes both the forces general forces are the economic social political legal and technological conditions which indirectly influence all business enterprise Specific forces are the investors customers competitors and suppliers which influence individual enterprise directly

Interrelatedness-Different elements of environment are interrelated for an example growing awareness for health care has increased the demand for health foods

Complexity- Business environment id complex in nature as the elements keep on changing example economic technological and other forces changes in demand for a product and service

Dynamic-Business environment is not static it keeps on changing

Uncertainty- Itrsquos very difficult to predict future events such as technology and fashion which occur fast and frequently

Business Studies

Human Resources Management

Human resource of an organisation are the aggregate of knowledge skills attitudes of people working in it

The management system which deals with human resources is called human resource management

Features of HRMbullComprehensive functionbullPeople-oriented

Question1) What do you mean by human

resource management Answer) Human resource management may be defined as that field of Management which has to do with planning organising and controlling the functions of procuring developing maintaining and utilising the labour force

bullAction oriented bullPervasive function bullContinuous function

2) Explain the features of HRM in brief

Answer)bullHuman Resource Management is concerned with managing people at work bull Human Resource Management is concerned with employees which bring people and organisations together so that the goals of each are met bullHuman resource management considered every employees as an individual and also promote their satisfaction and growth bull Human resource management is inherent in all organisations and at all levelsbullManagement of human resources are ongoing on never ending process which requires a constant alertness and Awareness of human relations

3) ldquoHR function is said to be pervasiverdquowhy

Answer) Human resource management is required in all organisations whether it is private or government organisations armed forces sports organisations etc It permeatsall the functional areas like production marketing finance research etc This from this feature of human resource management it can be said that it is pervasive in nature

Economics Demand Q1DEFINITION OF DEMANDIn economics demand is the quantity of a good that consumers are willing and able to purchase at various prices during a given period of timeQ2DEMAND CURVEIn economics a demand curve is a graph depicting the relationship between the price of a certain commodity and the quantity of that commodity that is demanded at that pricQ3LAW OF DEMANDIn microeconomics the law of demand states that conditional on all else being equal as the price of a good increases quantity demanded decreases conversely as the price of a good decreases quantity demanded increasesQ4ASSUMPTION of LAW OF DEMAND(i)No change in price of related commodities(ii) No change in income of the consumer(iii) No change in taste and preferences customs habit and fashion of the consumer( No expectation regarding future change in priceQ5MARKET DEMAND SCHEDULEIn economics a market demand schedule is a tabulation of the quantity of a good that all consumers in a market will purchase at a

given price At any given price the corresponding value on the demand schedule is the sum of all consumersrsquo quantities demanded at that priceQ6INDIVIDUAL DEMAND SCHEDULEIndividual demand schedule refers to a tabular statement showing various quantities of a commodity that a consumer is willing to buy at various levels of price during a given period of timeQ7 FACTORS AFFECTING INDIVIDUAL DEMAND FOR A COMMODITY

The factors that influence a consumerrsquos decision to purchase a commodity are also known as determinants of demand The following factors affect the individual demand for a commodity1 price of the commodity2 price of related goods3 income of buyer of the commodity4 tastes and preferences of the buyer1 Price of the CommodityYou must have observed that when price of a commodity falls you tend to buy more of it and when its price rises you tend to buy less of it when all other factors remain constant (lsquoother things remaining the samersquo) In other words other things remaining the same there is an inverse relationship between the price of a commodity and its quantity demanded by its buyers This statement is in accordance with law of demand which you will study in the later part of this lesson Price of a commodity and its quantity demanded by its buyers are inversely related only when lsquoother things remain the samersquo So lsquoother things remaining the samersquo is an assumption when we study the effect of changes in the price of a commodity on its quantity demanded2 Price of Related goodsA consumer may demand a particular good But while buying that good heshe also asks the price of its related goods Related goods can be of two types-(i) Substitute goods(ii) Complementary goods While purchasing a good prices of its substitutes and complements do affect its quantity purchased(i) Price of Substitute Goods Substitute goods are those goods which can easily be used in place of one another for satisfaction of a particular want like tea and coffee An increase in price of substitute good leads to an increase in demand for the given commodity and a decrease in price of substitute good leads to a decrease in demand for the given commodity It means demand for a given commodity is directly affected by change in price of substitute goods For example if price of coffee increases the demand for tea will rise as tea will become relatively cheaper in comparison to coffee(ii) Price of Complementary goods Complementary goods are those goods which are used together to satisfy a particular want like car and petrol An increase in the price of complementary goods leads to a decrease in demand for the given commodity and a decrease in the price of complementary goods leads to an increase in demand for the given commodity For example if price of petrol falls then the demand for cars will increase as it will be relatively cheaper to use both the goods together So demand for a given commodity is inversely affected by change in price of complementary goods3 Income of the Buyer of CommodityDemand for a commodity is also affected by income of its buyer However the effect of change in income on demand depends on the nature of the commodity under consideration In case of some goods like full cream milk fine quality of rice (Basmati rice) etc demand for these commodities increases when income of the buyer increases and

demand for these commodities decreases when income of the buyer decreases Such goods whose demand increases with the increase in income of the buyer are called normal goods But there are some goods like coarse rice toned milk etc whose demand decreases when income of buyer increases and their demand increases when income of the buyer decreases Such goods whose demand decreases with the increase in income of the buyer are called inferior goods Suppose a consumer buys 10 Kgs of rice whose price is ` 25 per Kg He cannot afford to buy better quality of rice because the price of such rice is ` 50 per Kg The consumer is spending ` 250 per month on the purchase of rice Now if income of the consumer increases and he can afford ` 350 on purchase of 10 Kg of rice Now he can afford to buy some quantity of rice say 6 Kgs whose price is ` 25 per Kg and may buy 4 Kgs of rice whose price is ` 50 per Kg Thus he will buy 10 Kgs of rice by spending ` 350 per month Therefore we may conclude that demand for normal goods is directly related to the income of the buyer but demand for inferior goods is inversely related to the income of the buyer4 Tastes and Preferences of the BuyerThe demand for a commodity is also affected by the tastes and preferences of the buyers They include change in fashion customs habits etc Those commodities are preferred by the consumers which are in fashion So demand for those commodities rises which are in fashion On the other hand if a commodity goes out of the fashion its demand falls because no consumer will like to buy it(5) Number of Buyers in the Market(Population)Increase in population raises the market demand whereas decrease in population reduces the market demand for a commodity Not only the size of population but its composition like age (ratio of males females children and old people in population) also affects the demand for a commodity It is because of needs of children young old male and female population differs(6) Distribution of Income and WealthIf the distribution of income and wealth is more in favour of the rich demand for the commodities preferred by the rich such as comforts and luxuries is likely to be higher On the other hand if the distribution of income and wealth is more in favour of poor demand for commodities preferred by the poor such as necessities will be more(7) Season and Weather ConditionsThis is generally observed that the demand for woolens increases during winter whereas demand for ice creams and cold drinks increases during summer Similarly market demand for umbrellas rain coats increases during rainy seasonQ8 REASONS FOR OPERATION OF LAW OF DEMAND WHY DEMAND CURVE SLOPES DOWNWARDNow we will try to explain why does a consumer purchase more quantity of a commodity at a lower price and less of it at a higher price or why does the law of demand operate ie why does the demand curve slope downwards from left to right The main reasons for operation of law of demand are1 Law of Diminishing Marginal UtilityAs you have studied earlier law of diminishing marginal utility states that as we consume more and more units of a commodity the utility derived from each successive unit goes on decreasing The consumer will be ready to pay more for those units which provide him more utility and less for those which provide him less utility It implies that he will purchase more only when the price of the commodity falls2 Income Effect

When price of a commodity falls purchasing power or real income of the consumer increases which enables him to purchase more quantity of the commodity with the same money income Let us take an example Suppose you buy 4 ice creams when price of each ice cream is ` 25 If price of ice creams falls to ` 20 then with same money income you can buy 5 ice creams now3 Substitution EffectWhen price of a commodity falls it becomes comparatively cheaper as compared to its substitutes (although price of substitutes has not been changed) This will lead to rise in demand for the given commodity For example if coke and Pepsi both are sold at ` 10 each and price of coke falls Now coke has become relatively cheaper and will be substituted for Pepsi It will lead to rise in demand for coke4 Change in Number of BuyersWhen price of a commodity falls some old buyers may demand more of the commodity at the reduced price and some new buyers may also start buying this commodity who were not in a position to buy it earlier due to higher price This will lead to increase in number of buyers when price of the commodity falls As a result demand for the commodity rises when its price falls5 Diverse Uses of a CommoditySome commodities have diverse uses like milk It can be used for drinking for sweet preparation for ice cream preparation etc If price of milk rises its use may be restricted to important purpose only This will lead to reduction in demand for other less important uses When price of milk falls it can be put to other uses also leading to rise n demand for itQ9 EXCEPTIONS TO THE LAW OF DEMANDYou have studied in law of demand that a buyer is willing to buy more quantity of a commodity at a lower price and less of it at a higher price But in certain circumstances a rise in price may lead to rise in demand These circumstances are called Exceptions to the Law of Demand Some important exceptions are1 Giffen GoodsGiffen goods are special type of inferior goods in which negative income effect is stronger than negative substitution effect Giffen goods do not follow law of demand as their demand rises when their price rises Examples of Giffen goods are jowar and bajra etc2 Status Symbol GoodsSome goods are used by rich people as status symbols eg diamonds gold jewellary etc The higher the price the higher will be the demand for these goods When price of such goods falls these goods are no longer looked at as status symbol goods and tehrefore therir demand falls3 NecessitiesCommodities such as medicines salt wheat etc do not follow law of demandbecause we have to purchase them in minimum required quantity whatever their price may be4 Goods Expected to be ScarceWhen the buyers expect a scarcity of a particular good in near future they start buying more and more of that good even if their prices are rising For example during war famines etc people tend to buy more of some goods even at higher prices due to fear of their scarcity in near future

Political Science

Constitution of India-The

Preamble

The preamble-

Preamble-

The preamble is the most precious part of the constitution We the people of India having solemnly resolved to constitute India into a Sovereign Socialist Secular Democratic Republic and to secure to all its citizensA preamble is an introductory and expressionary statement in a document that explains the documents purpose and underlying philosophy When applied to the opening paragraphs of a statute it may recite historical facts pertinent to the subject of the statuteNature and purpose of the constitution-Purpose of the Constitution dictates permanent framework of the government to form a more perfect union to establish justice and ensure peace of thenationconstitution provide principles how the government can run itself following the rules and laws written in the constitution of each state keeps them balanced

Answer the following questions-

1 What is preambleA preamble is an introductory and expressionary statement in a document that explains the documents purpose and underlying philosophy2 What is the nature and

purpose of the constitutionConstitution dictatespermanent framework of the government to form a more perfect union to establish justice and ensure peace of the nation

Homework-Learn

Accounts Compatibilty mode

1MEANING OF PARTNERSHIPPartnership is a form of business organisation where two or more persons join hands to run a business They share the profits and losses according to the agreement amongst them According to the Indian Partnership Act 1932 ldquoPartnership is relation between persons who have agreed to share profits of a business carried on by all or any one of them acting for allrdquo For example one of your friends has passed class XII from National Institute of Open Schooling (NIOS) and wants to start a business Heshe approaches you to join in this venture Heshe wants you to contribute some money and participate in the business activities Both of you if join hands constitute a partnership2CHARACTERISTICS1048698 Agreement A partnership is formed by an agreement The agreement may be either oral or in writing It defines the relationship between the persons who agree to carry on business It may contain the terms of sharing profit and the capital to be invested by each partner etc The written agreement is known as partnership deed1048698 Number of persons There must be at least two persons to form a partnership

The maximum number of partners in a partnership firm can be 50 according toCompanies Act 20131048698 Business The Partnership is formed to carry on business with a purpose of earning profits The business should be lawful Thus if two or more persons agree to carry on unlawful activities it will not be termed as partnership1048698 Sharing Profits The partners agree to share profits in the agreed ratio In caseof loss all the partners have to bear it in the same agreed profit sharing ratio10486981048698Mutual Agency Every partner is an agent of the other partners Every partner can bind the firm and all other partners by hisher acts Each partner will be responsible and liable for the acts of all other partners10486981048698Unlimited liability The liability of each partner except that of a minor is unlimited Their liability extends to their personal assets also If the assets of the firm are insufficient to pay off its debts the partnersrsquo personal property can be used to satisfy the claim of the creditors of the partnership firm10486981048698Management All the partners have a right to mange the business However they may authorize one or more partners to manage the affairs of the business on their behalf10486981048698Transferability of Share No partner can transfer hisher share to any one including hisher family member without the consent of all other partners3PARTNERSHIP DEEDAgreement forms the basis of partnership The written form of the agreement is which a document of partnership is It contains terms and conditions regarding the conduct of the business It also explains relationship between the partners This document is called partnership deed Every firm can frame its own partnership deed in which the rights duties and liabilities of the partners are stated in detail It helps in settling the disputes arising among the partners during the general conduct of business 4CONTENTS OF PARTNERSHIP DEEDThe partnership deed generally contains the following (i) Name and address of the partnership firm(ii) Nature and objectives of the business(iii) Name and address of each partner(iv) Ratio in which profits is to be shared(v) Capital contribution by each partner(vi) Rate of Interest on capital if allowed(vii) Salary or any other remuneration to partners if allowed(viii) Rate of interest on loans and advances by a partner to the firm(ix) Drawings of partners and interest thereon if any(x) Method of valuation of goodwill and revaluation of assets and liabilities on the reconstitution of the partnership ie on the admission retirement or death of a partner(xi) Settlement of disputes by arbitration(xii) Settlement of accounts at the time of retirement or death of a partner5IN ABSENCE OF PARTNERSHIP DEEDThe partnership deed lays down the terms and conditions of partnership in regard to rights duties and obligations of the partners In the absence of partnership deed there may arise a controversy on certain issues like profit sharing ratio interest on

capital interest on drawings interest on loan and salary of the partners In such cases the provisions of the Indian Partnership Act becomes applicableSome of the Issues are(i) Distribution of Profit Partners are entitled to share profits equally(ii) Interest on Capital Interest on capital is not allowed(iii) Interest on Drawings No interest on drawing of the partners is to be charged(iv) Interest on Partnerrsquos Loan A Partner is allowed interest 6 per annum on the amount of loan given to the firm by himher(v) Salary and Commission to Partner A partner is not entitled to anysalary or commission or any other remuneration for managing the business

History TOPIC-TOWARDS INDEPENDENCE AND PARTITION THE LAST PHASE (1935-1947)

SUB TOPIC-IMPORTANT POLITICAL DEVELOPMENTS ndash GROWTH OF SOCIAL IDEAS

Socialism is a political social and economic philosophyLike in other parts of the world the Russian revolution of 1917 served as a great inspiration for revolutionaries in India who at that time were engaged in the struggle for liberation from British ruleSocialist ideas led to the formation of communist party of IndiaJAWAHARLAL NEHRU Among the early Congress leaders Jawaharlal Nehru was very much impressed and influenced by the Socialist ideas He also learnt about the Economic activities of the Soviet Union after the Bolshevic Revolution 1917 He made full use of them in IndiaThe election of Jawaharlal Nehru and Subhas Chandra Bose showed the Left wing tendency within CongressJawaharlal Nehru demanded economic freedom along with political freedom of the people in order to end the exploitation of masses

Nehrus working committee included three socialists leaders The Lucknow session was a landmark in the evolution of socialist ideas of the congressSUBHAS CHANDRA BOSE ndash Subhas Chandra Bose had socialist leaning Both Jawaharlal Nehru and Subhas Chandra Bose were known as leftist Congress men Later on National Congress divided into Leftist and rightist campCONGRESS SOCIALIST Within the Congress some leaders formed the Congress Socialist partyPattavi Sitaramyya Sardar Patel Rajendra Prasad had hostile attitude towards the Congress Socialist partyJawaharlals attitude was hesitant

1 QUESTION ndash Mention name of two Congress leaders who had socialist leaning

1ANSWER ndash Subhas Chandra Bose and Jawaharlal Nehru2QUESTION- In which session of the congress Jawaharlal elaborated his Socialist ideas2 ANSWER ndash Lucknow and Faizpur Session in December 1935 and 19363QUESTION ndash Why Congress was sharply divided into leftist and rightist camp 3ANSWER ndash Subhas Chandra Bosersquos attempt to seek re election for congress presidentship in 1939sharply divided the National Congress into Leftist and Rightist camp4 QUESTION ndash Who was MN Roy 4 ANSWER ndash Manabendra Roy first formed the Communist Party of India outside the country at Tashkent in 19205QUESTION ndash Who formed the Congress Socialist Party within the Congress5 ANSWER ndash Jaya Prakash Narayan Achyut Patwardhan Acharya Narendra Dev Ram Mohan Lohia Aruna Asaf Ali6QUESTION ndash When was the Congress Socialist Party formed What was its object6 ANSWER ndash 1934The Congress Socialist Party sought to work out socialist programme through the Congress They joined hands with the Congress and wanted to carry

Subhas Chandra Bose being expelled from the congress after the Tripuri rift he formed Forward BlockThere were basic differences between the Congress Socialists and the communistsTRADE UNION ACTIVITIES Maximum working class people lived in Bombay and Calcutta The working and living conditions of those workers were very miserable In this situation Shasipada Banerjee NM Lokhande protested against the oppression of the working class peopleThe first Trade Union Madras Labour Union was formed in 1918 by BP WadiaIndustrial strikes took place in Kanpur Calcutta Madras Jamshedpur and Ahmedabad AITUC was formed in Bombay in 1927 The growth of Trade union among the workers was slow because of the fear of the dismissal of the jobIn the mean time the Moderates as well as Communists left AITUC and formed separate organization

on National struggle with the help of workers and peasant class of the society7 QUESTION ndash What was the name of the party founded by Subhas Chandra Bose7 ANSWER- Forward Block8QUESTION ndash Who was Shasipada Banerjee8 ANSWER ndash Shasipada Banerjee was a radical Brahmo He founded a working menrsquos club to protest against exploitation of the British rulers towards the working class of India9 QUESTION ndash What was the weekly published by NM Lokhande9ANSWER- Dinabandhu10 QUESTION ndash Who founded Bombay Mill-Hands Association and in which year10 ANSWER- NM Lokhande in189011 QUESTION- Who was BP WadiaANSWER- BPWadia was the founder of Madras Labour Union in191812 QUESTION- What was the name of the first labour union of India12 ANSWER- Madras Labour Union13 QUESTION Who founded the Majur Mahajan 13 ANSWER GANDHIJI14 QUESTION What was the full form of AITUC When it was formed14 ANSWER All India Trade Union Congressin 192715QUESTION Who formed the Red Trade Union Congress and in which year15ANSWER The Communists formed the Red Trade Union Congress16 QUESTION What do you mean by Socialism16 ANSWER Socialism describes any political and economic theory that says the community rather than individuals should own and manage property and natural resources

Subject Eng Literature (The Tempest ndash William Shakespeare) Topic Act III Scene 3 Lines 1 to 52 (Line 52 ndash Brother my lord the Duke Stand to and do as we) Date 13th April 2020 (4th Period)

[Students should read the original play and also the paraphrase given in the school prescribed textbook]Summary Questions amp Answers

o Alonso Sebastian Antonio Gonzalo Adrian Francisco and others wandered about the island in search of Ferdinand and gets tired and hungry of the toil and at the same time gives up all hope of finding him

o Antonio and Sebastian are happy that Alonso is out of hope and decide to make another attempt on his life that night when being so tired they will be sleeping soundly

o Suddenly a solemn and strange music is heard in the air and several strange shapes enter bringing in a banquet These strange shapes then dance round it with gestures of salutation and then inviting the King to eat they depart

o Seeing this strange scene all are inclined to believe the tales told by travelers that there truly are ldquounicornsrdquo and ldquothe phoenixrsquo thronerdquo

1 ALONSO What harmony is this My good friends hark (L18-27)

GONZALO Marvellous sweet music

[Enter several strange shapes bringing in a banquet

they dance about it with gentle actions of salutation

and inviting the King and his companions to eat they depart]ALONSO Give us kind keepers heavens What were theseSEBASTIAN A living drollery Now I will believe

That there are unicorns that in Arabia

There is one tree the phoenixrsquo throne one phoenix

At this hour reigning thereANTONIO Ill believe both

And what does else want credit come to me

And Ill be sworn rsquotis true Travellers neer did lie

Though fools at home condemn rsquoem

(a) How did Prospero present an amazing spectacle before Alonso and his companions

Using his magic powers Prospero ordered strange shapes to lay a banquet before Alonso and his companions The shapes brought several dishes with tasty eatables in them They placed the dishes on a table before Alonso and his companions Then the strange shapes began to dance gracefully around the banquet While dancing they made gestures inviting them to eat the food Then suddenly the shapes disappeared(b) Who were the guests at the strange banquet Describe the lsquoliving drolleryrsquo

Alonso Sebastian Antonio Gonzalo Adrian and Francisco were the guests at the strange banquet

The term ldquoliving drolleryrdquo refers to live entertainment show In this context when Alonso the King of Naples Sebastian his brother Antonio the treacherous brother of Prospero Gonzalo the kind and loyal councillor to the King Adrian and Francisco came to the island they were hungry and weary in their spirits They heard a solemn and strange music They were shocked to see several strange shapes bringing in a banquet and these shapes danced about it with gentle action of salutation inviting the King and his companions to eat After this Sebastian described this show as lsquoliving drolleryrsquo(c) What is lsquophoenixrsquo What are lsquoUnicornsrdquo

The term lsquophoenixrsquo refers to a mythical Arabian bird which lived alone and perched on a solitary tree After one hundred years it expired in flames and rose again from its own ashes

lsquoUnicornsrsquo refers to the mythological four-footed beasts having horns in the centre of their foreheads When the horns are ground into powder the powder was believed to be

an aphrodisiac(d) How does Sebastian explain the puppet show OR Why does the speaker now believe in unicorns and phoenix

Sebastian finds several strange shapes bringing in the banquet They invite the king and his party for dinner and soon depart He tells that if such a strange sight can be a reality there is nothing incredible in the world and from the present moment he will believe anything He says that it is a strange dumb show enacted not by puppets but by living beings It is stranger than a travellerrsquos tale Seeing such a thing

before his own eyes he will no longer disbelieve the story about unicorns and phoenix(e) How do the other characters present respond to this living drollery

At the sight of the lsquoliving drolleryrsquo like Sebastian Gonzalo and Antonio too acted strangely Antonio told that he too now believes in unicorns and phoenix and anything else that seems to be incredible He too now believes in travellersrsquo tales Gonzalo told that if he would report those happenings in Naples nobody will believe him He considers that those gentle shapes were gentler in manner in comparison to the living beings Alonso was at first sight suspicious and told them that those strange shapes conveyed their meaning in expressive gestures when they seemed to lack speech by their movements and sounds Francisco was amazed at their mysterious disappearance

2 ALONSO Not I

(Line 43-52)GONZALO Faith sir you need not fear When we

were boysWho would believe that there were mountaineers

Dewlapped like bulls whose throats had hanging at rsquoem

Wallets of flesh Or that there were such men

Whose heads stood in their breasts Which now we find

Each putter-out of five for one will bring us

Good warrant ofALONSO I will stand to and feed

Although my lastmdashno matter since I feel

The best is past Brother my lord the Duke

Stand to and do as we

(a) How does Alonso respond at the spectacle of the shapes which were sent to them at the instruction of Prospero

After seeing the strange sight of appearing and disappearing of the shapes sent by Prospero to arrange a banquet for them Alonso says that his surprise at having seen those creatures is infinite and he is fully justified in feeling so much surprise He thinks that their shapes their gestures and the sounds they made were indeed amazing Although they do not possess the gift of speech yet they were able to convey their

thoughts by means of their gestures only

(b) What does Prospero say about the views expressed by Alonso regarding the shapes What does Francisco think about the shapesAfter hearing Alonsorsquos views about the shapes Prospero says that this manrsquos praise of the spirits is rather hasty He means to say that Alonso has shown great haste in reaching the conclusion about the shapes Francisco is amazed to see that those shapes disappeared in a mysterious way(c) What does Sebastian ask Alonso to doSebastian tells Alonso that the shapes having disappeared should not matter to them because they have left the eatables behind He asks Alonso to enjoy eating as they are extremely hungry but the king does not accept his offer of enjoying the dishes(d) How does Gonzalo try to dispel Alonsorsquos fear of those strange shapes What kind of references does he give to AlonsoGonzalo says that those who have travelled abroad have reported seeing even stranger sights than these shapes that Alonso and his companions have beheld Hence there is no reason to feel afraid of these shapes Gonzalo further adds that in his younger days he had heard strange stories from travelers and Alonso might have heard similar stories For instance it was said that there existed a certain race of

human beings who had huge lumps of flesh hanging at their throats and who therefore resembled bulls Then Gonzalo tells about a race of human beings whose heads were located at their breasts Gonzalo says that such stories were not believed by most people in those days but now-a-days these stories have become common(e) Explain the following lsquoEach putter-out of five for onersquoEnglish travellers often insured their trips with London brokers Those that went on foreign travels those days used to deposit a certain amount with some firm or company in London before their departure If the travelers failed to return the money was forfeited by the company with which it had been deposited But this money was repaid five-fold if the travelers returned safe and sound In this way a traveler stood a great chance of recovering the entire cost of his

travels(f) Give the explanatory meanings of the following expressions in the context of the above extract (i) Dewlapped (ii) Wallets of flesh

(iii) Putter-out(i) Dewlapped having big lumps of flesh at the necks(ii) Wallets of flesh large masses of flesh looking like bags(iii) Putter-out to invest money before commencing the travel

  • General methods of preparation of hydrogen
  • Chapter Dimensional Analysis (Summary)
    • Properties of Charge
Page 39:   · Web viewSubject. Topic. Summary. Execution. Hindi. व्याकरण. शरीरके अंगो के नाम लिखिए. 1) आँख 2) नाक 3

business transactions concern a particular person of firm asset or income or expense It is a summarised record of all transactions which take place in an accounting year

bull Types of accountsPersonal accounts ndash Personal accounts relating

to person and Organisation are known as personal accounts Example Ramrsquos Account ABC amp Co Account etc

Real account - The accounts related to tangible and intangible assets are called real accounts Example Cash Account Furniture Account etc

Nominal account- Accounts related to expenses losses incomes and gains are known as nominal accounts Example Wages Account Salary Account Discount Account etc

bull Accounting cycle Accounting cycle refers to a complete sequence of accounting activities It begins with recording of transactions and ends with the preparation of a balance sheet

Chemistry TopicAtomic Structure

Thomsonrsquos atomic modelThomson (1898) was the first to propose the model of an atomHe proposed that an atom can be regarded as a uniform sphere of positive electricity in which requisite number of electrons are embedded evently to neutralize the positive chargeThis is just like plums embedded in a pudding or seeds evently distributed in red spongy mass of a watermelonThis model of atom is known as ldquoPlum-Pudding modelrdquo or

Q1)What is the fundamental constituents of atomAns Electron Proton and neutrons are the fundamental constituents of atomQ2)What is the value of fundamental unit of electricityAnsThe charge carried by one electron is sad to be the fundamental unit of electricityIts magnitude is 48times10-10esuOr 1602times10-19C Q3)Name the element containing no neutronAnsOrdinary hydrogen atom or protium 1H1

Types of AccountPersonal AccountReal AccountNominal AccountBalance Sheet (opening)

ldquowatermelon modelrdquoThis model could explain the electrical neutrality of an atom but failed to explain the result of scattering experiment carried out by Rutherford in 1911So it was rejected ultimately

Q4)Why is an electron called universal particleAns Itrsquos mass and Charge are independent of its source

EVS Chapter 1 ndash Modes of Existence

Modes of existence When one speaks normally about the mode of existence of some group or individual one refers to their customs their mode of being their ethology their habitat in some way their feeling for a placeDifferent modes of exixtence are ndash

1 Hunting ndashGathering2 Pastoral3 Agricultural4 Industrial

1 Hunting and gathering Hunting and gathering mode of existence is characterized by obtaining food from hunting wild animals including fishing and gathering wild plants From their earliest days the hunter-gatherer diet included various grasses tubers fruits seeds and nuts Lacking the means to kill larger animals they procured meat from smaller game or through scavenging

Societies that rely primarily or exclusively on hunting wild animals fishing and gathering wild fruits berries nuts and vegetables to support their diet are called hunting and gathering societies

At least this used to be practice of human beings before agriculture is invented As their brains evolved hominids developed more intricate knowledge of edible plant life and growth cycles

Q) Write the features of Hunting ndash gathering societiesAns - There are five basic characteristics of hunting and gathering societies

i The primary institution is the family which decides how food is to be shared and how children are to be socialized and which provides for the protection of its members

ii They tend to be small with fewer than fifty members

iii They tend to be nomadic moving to new areas when the current food supply in a given area has been exhausted

iv Members display a high level of interdependence

v Labor division is based on sex men hunt and women gather

Political Science

Introduction to political science

Political science occasionally called politology is a social science which deals with systems of governance and the analysis of political activities political thoughts associated constitutions and political behaviorThe study of political science involves the study of both the

Answer the following questions-1 What is political science

Political science occasionally called politology is a social science which deals with systems of governance and the analysis of political activities political thoughts associated constitutions and political behavior

2 Short notes-

traditional and modern theories of politicsTraditionalClassical political sciencepolitical theory-Traditional political science is the study of politics before Second World War The methodology to study Politics was traditional (legal formaletc) the definition of politics traditional (Politics begins and end with state)area of study (constitution state machinery)was traditionalModern Political scienceModern political theory-Modern Political Theory critically examines the contemporary state of political theory making an assessment of the achievement and limitations of the Behavioural Revolution in its totality and reviews objectively the major paradigms and conceptual frameworks adopted by the disciplineContemporary attempts at the development of an integrated political theory involving the use of both traditional and modern concepts approaches and theories-Around late 1960s several political scientists realized the importance of both the traditional political theory and modern Political theory They began building an integrated theory of politics involving a systematic mixture of traditional and modern studies of politics It was held that the study of a complex and vast field like politics needs both traditional as well as

Classical political theory Modern Political theory

Homework-Learn

modern concepts and approaches for studying itrsquos all aspects

Subject Eng Literature (The Tempest ndash William Shakespeare) Topic Act I Scene 1 Lines 1 to 32 (Line 32 ndash Gonzalo hellip If he be not born to be hanged our case is miserable) Date 13th April 2020 (3rd Period)

[Students should read the original play and also the paraphrase given in the school prescribed textbook]Summary Questions amp Answers

[SUMMARY OF THE ENTIRE SCENE]

o The play starts with the scene of a severe storm at sea Alonso (King of Naples) Sebastian (Alonsorsquos brother) Ferdinand (Alonsorsquos son) Gonzalo Antonio (the usurping Duke of Milan) are in a ship in the midst of the storm

o The mariners are trying their best to control the vessel from running aground and are totally following the orders of their Master the Boatswain They have scant success

o The mariners become extremely unhappy and annoyed when most of the passengers arrive on the deck thereby hampering their effort to save the ship There is serious confrontation between them and the passengers who are part of the Kingrsquos entourage

o The mariners could not save the ship

SUMMING-UP

(i) Vivid description of the scene which gives a realistic description of terror and confusion of a tropical storm

(ii) Shows Shakespearersquos accuracy of knowledge in describing the naval operations and also matters of seamanship

(iii) The opening scene justifies the title ndash The Tempest

UNANSWERED QUESTIONS

(i) The King always travels with his entire fleet including his soldiers Where

(1) GONZALO Nay good be patient (Line 15-26)BOATSWAIN When the sea is Hence What cares these

roarers for the name of the king To cabin silence Trouble us not

GONZALO Good yet remember whom thou has aboardBOATSWAIN None that I more love than myself You are a

councillor if you can command these elements to silence and work

the peace of the present we will not hand a rope more use your authority If you cannot give thanks you have

lived so long and make yourself ready in your cabin for the mischance of the hour if it so hap [To the Mariners]

Cheerly good hearts [To Gonzalo] Out of our way I say

(a) To whom is the boatswain speaking What does he mean by lsquoNone that I more love than myselfrsquo

The Boatswain is speaking to Gonzalo the honest old councilor of the Duke of MilanBy using the words ndash lsquoNone that I love more than I love myselfrsquo means that for the Boatswain nobody is dearer to him than his own life

(b) What were the conditions that made the boatswain react in this way

The Boatswain reacts in this way because the storm is at sea and Alonso King of Naples Sebastian his brother Ferdinand his son Gonzalo Antonio the usurping Duke of Milan on board are in distress and in panic Thus they have rushed to the deck interrupting the work of the mariners

(c) What hope does Gonzalo take from the attitude of the boatswain

The insolent and authoritative attitude of Boatswain makes Gonzalo feel comforted He tells that there are no signs that the Boatswain will be drowned But his facial appearance and attitude shows that he is destined to die on land by hanging which in effect means that all on board will be saved Otherwise all the persons on board are doomed

(d) How can they lsquomake yourself ready in your cabinrsquo For what were they asked to make ready themselves

In order to make themselves ready in their cabin the

were the other ships

(ii) Why was the ship in that area Where was it coming from or going where

(iii) The ship broke apart What happened to those who were in the ship

passengers on board must prepare for death which they will possibly soon have to meetThey can retire to their cabins and offer prayers to the Almighty to save them from drowning

(e) What does the boatswain say when he is asked to be patient What does he order to the royal party

When the boatswain is asked to be patient and remain calm he says that he will be patient only when the storm will be over and the sea will be calm but as long as the storm blows and there is danger to the ship he cannot think of being patient He orders the royal party to go to the cabin and leave the mariners to their work

(2) GONZALO I have great comfort from this fellow (Line 27-36)

Methinks he hath no drowning mark upon him his complexion is perfect

gallows Stand fast good Fate to his hanging Make the rope of his destiny our cable for our own doth little advantage If he be not born to be hanged our case is miserable

(a) Why does Gonzalo regard the Boatswain in the midst of danger

In the midst of danger Gonzalo regards the boatswain because he feels that the Boatswain is a source of comfort and is bent upon to do his work sincerely which in this case is saving the ship and its passengers from the severest of raging storm

(b) What reasons does Gonzalo give when he says that none in the ship will die of drowning

Gonzalo is almost sure that none in the ship will die by drowning His says that there is no mark on the face of the boatswain that indicates that he will die by drowning On the other hand the lines on his face are strong indications that he will be hanged to death Therefore there shall be no danger of the shiprsquos sinking

(c) Explain the following ldquoStand fast good Fate to his hanging Make the rope of his destiny our cable for our own doth little advantage If he be not born to be hanged our case is miserablerdquo

The stated lines mean that if the will of destiny is to be carried out then the ship will not get wrecked and all the passengers will be saved The safety of the passengers therefore depends upon the will of fate being carried out in the case of the boatswain If however the boatswain is not to die by hanging then the passengers are also very unsafe because in that case the ship is likely to sink

(d) What order does the Boatswain give to the sailors

when he re-enters What does he say about the crying of the fellows inside the cabin

The boatswain orders the sailors to bring the topmast lower and bring the ship close to a stationary position with the help of the main sail He says that the fellows inside the cabin are moaning and crying in their distress louder than his voice and louder even than the roaring of the storm

Class XII (ScienceCommerceHumanities) Subject Topic Summary Execution

Computer Science

PropositionalLogic

Propositional logic is a procedure to provide reasoning through statementProposition A ststement that results in True or False is said to be proposition There are two types of propositionSimple proposition amp compound propositionSimple proposioton A simple proposition is one that is not a part of any other proposition Such sentential form of proposition is symbolized with english letters in short For example Ram is a claver student (TrueFalse)Where do you live (Not in True or False)Grapes are sweet (TrueFalse)It rains today (TrueFalse)Here we can see some statements anwer would be true or false but some staements answer can not give in terms of true or false Thus the sentences which can be answered in true or false are known as simple propositionAssigning propositon to a variableThe general syntax to assign propostion to a variable is as followsVariable = Simple propositonFor example A=Ram is a clever studentB= Grapes are sweetC= it rains todayCompound proposition

helliphellipto be continued in next classhelliphellipMath Relation Relation If A and B are two non-empty sets

then a relation R from A to B is a subset of AxB If R A x B and (a b) R then we say that a sube isinis related to b by the relation R written as aRbeg Let A be the set of students of class XII and B be the set of students of class XI Then some of the examples of relation from A to B arei) (a b) AXB a is brother of bisinii) (a b) AXB age of a is more than age of isinb Types of relation In this section we would like to study different types of relations We know that a relation in a set A is a subset of A times A Thus the empty set φ and A times A are two extreme relations For illustration consider a relation R in the set A = 1 2 3 4 given by R = (a b) a ndash b = 10 This is the empty set as no pair (a b) satisfies the condition a ndash b = 10 Similarly R = (a b) | a ndash b | ge 0 is the whole primeset A times A as all pairs (a b) in A times A satisfy | a ndash

Example 1 Let A be the set of all students of a boys school Show that the relation R in A given by R = (a b) a is sister of b is the empty relation and R = (a b) the primedifference between heights of a and b is less than 3 meters is the universal relationSolution Since the school is boys school no student of the school can be sister of any student of the school Hence R = φ showing that R is the empty relation It is also obvious that the difference between heights of any two students of the school has to be less than 3 meters This shows that R = A times A is primethe universal relation Example 2 Show that the relation R in the set 1 2 3 given by R = (1 1) (2 2) (3 3) (1 2) (2 3) is reflexive

b | ge 0 These two extreme examples lead us to the following definitionsDefinition 1 A relation R in a set A is called empty relation if no element of A isrelated to any element of A ie R = φ A times AsubDefinition 2 A relation R in a set A is called universal relation if each element of A is related to every element of A ie R = A times A Both the empty relation and the universal relation are some times called trivial relation Definition 3 A relation R in a set A is called(i) reflexive if (a a) R for every a Aisin isin(ii) symmetric if (a1 a2) R implies that (aisin 2a1)

R for all aisin 1 a2 Aisin(iii) transitive if (a1 a2) R and (aisin 2 a3) R isinimplies that (a1 a3) R for all aisin 1 a2 a3 AisinDefinition 4 A relation R in a set A is said to be an equivalence relation if R is reflexive symmetric and transitive

but neither symmetric nor transitiveSolution R is reflexive since (1 1) (2 2) and (3 3) lie in R Also R is not symmetric as (1 2) R but (2 1) isin notinR Similarly R is not transitive as (1 2) R and (2 3) R but (1 3) R isin isin notinExample 3 Show that the relation R in the set Z of integers given byR = (a b) 2 divides a ndash b is an equivalence relationSolution R is reflexive as 2 divides (a ndash a) for all a Z isinFurther if (a b) R then 2 divides a isinndash b Therefore 2 divides b ndash a Hence (b a) R which shows that R is isinsymmetric Similarly if (a b) R and (b c) R isin isinthen a ndash b and b ndash c are divisible by 2 Now a ndash c = (a ndash b) + (b ndash c) is even (Why) So (a ndash c) is divisible by 2 This shows that R is transitive Thus R is an equivalence relation in ZExample 4 Let L be the set of all lines in a plane and R be the relation in L defined as R = (L1 L2) L1 is perpendicular to L2 Show that R is symmetric but neither reflexive nor transitiveSolution R is not reflexive as a line L1 can not be perpendicular to itself ie (L1 L1) R notinR is symmetric as (L1 L2) Risin

L1 is perpendicular to L2rArr L2 is perpendicular to L1rArr (L2 L1) RrArr isin

R is not transitive Indeed if L1 is perpendicular to L2 and L2 is perpendicular to L3 then L1 can never be perpendicular to L3 In fact L1 is parallel to L3 ie (L1 L2) R isin(L2 L3) R but (L1 L3) Risin notin

Chemistry Solid state Characteristics if Solids(i)The particles are locked in fixed positions they are unable to change their relative positions and this brings a definite shape and volume of a solid(ii)In a solid the constituent particles are held by strong forces of attractionThe forces of attraction may be bonding or non bonding(iii)The constituent particles in a solid pack together as closely as possibleoccupying most of the available space within the solidThus the empty space in a solid is very smallThis makes a solid highly rigid and nearly incompressibleThis also explains why a solid has high density and exhibits slow diffusionClassification of Solids

Q1)Define Crystalline solids AnsA Solid that has a definite geometrical shape and a sharp melting pointand whose constituent particles (atomsmolecules or ions) are arranged in a long range order of definite pattern extending throughout the solid is called a crystalline solidExNaClQ2)Define Amorphous solids AnsA solid that does not have a definite shape and a sharp melting pointand whose constituent particles (atomsmolecules or ions) are not arranged in a definite pattern is called an amorphoussolid

Crystalline solidsAmorphous solids

ExGlassRubberQ3)Classify Crystalline Solids Crystalline Solids

Physics Coloumbrsquos Law (Summary)

Before Going Into Coloumbrsquos Law We Will First Learn What is Charge Properties of Charge and Always remember that charge is quantized ie a body always have static charge of magnitude equal to some integral multiple of fundamental electronic charge e= 16 x 10- 19 C

Charge is the property of matter that causes it to produce and experience electrical and magnetic effects The study of the electrical charges at rest is called electrostatics When both electrical and magnetic effects are present the interaction between charges is referred to as electromagnetic

There exist two types of charges in nature positive and negative Like charges repel and unlike charges attract each other

The type of charge on an electron is negative The charge of a proton is the same as that of an electron but with a positive sign In an atom the number of electrons and the number of protons are equal The atom is therefore electrically neutral If one or more electrons are added to it it becomes negatively charged and is designated as negative ion However if one or more electrons are removed from an atom it becomes positively charged and is called a positive ion

The excess or deficiency of electrons in a body gives the concept of charge If there is an excess of electrons in a body it is negatively charged And if there is deficiency of electrons the body becomes positively charged Whenever addition or removal of electrons takes places the body acquires a charge

The SI Unit of charge is coulomb (C) In SI units the current is a fundamental quantity having a unit of ampere (A) The unit of charge is defined in terms of the unit of current Thus one coulomb is the charge transferred in one second across the section of a wire carrying a

Ionic SolidsMetallicSolids

Molecular Solids

current of one ampere

As q = It we have1 C = (1 A) (1 s)

The dimensions of charge are [A T]

Properties of Charge

(1) Quantization of Charge Electric charge can have only discrete values rather than any value That is charge is quantized The smallest discrete value of charge that can exist in nature is the charge on an electron given as

e = plusmn 16 x 10- 19 C

This is the charge attained by an electron and a protonA charge q must be an integral multiple of this basic unit That is

Q = plusmn ne where n = 1 2 hellip

Charge on a body can never be (frac12)e (23)e or 57e etcWhen we rub a glass rod with silk some electrons are transferred from the rod to the silk The rod becomes positively charged The silk becomes negatively charged The coulomb is a very large amount of charge A typical charge acquired by a rubbed body is 10 - 8 C

Biology Reproduction in organisms

Welcome to this new session 2020-21Today in this first chapter we mainly discuss about reproduction types needs and life span of some organismsWe also discuss about difference between sexual and asexual reproduction

Q1 What is reproductionReproduction is defined as a biological processin which an organism gives rise to young onessimilar to itselfQ2 What are the needs of reproductionbulli) Reproduction maintain life on earthii) It enables the continuity of the species generation after generationiii) It creates genetic variation among populationsQ3 Define Life span and write some orgnisms life spanbull Life span is the period from birth to

the natural death of an organism- OrganismsLife span1 Butterfly 1 - 2 weeks2 Fruit fly 30 days3Dog 10-13 years4 Rose5-7 years5 Tortoise100-150 years6 Banyan Tree -200 - 250 yearsQ4 Reproduction is of two types in case ofanimals but in case of plants vegetative propagation is also present

Asexual Reproduction Sexual Reproductioni) Always uniparentalii) Gametes are not involvediii) Only mitotic division involvediv) Somatic cells of parents are involvedv) Offsprings are genetically similar to the parents

i) Usually biparentalii) Gametes are involvediii) Meiosis occurs during gametogenesis Mitosis occurs after fertilisationiv) Germ cells of the parents are involvedv) offsprings are genetically different from the parents

COMMERCE BUSINESS ENVIRONMENT

Welcome to the new sessiontoday we are going to start the first chapter of Class XII The name of the chapter is Business Environment

Already many of you have got some idea about the word business environment form the first chapter of business studies in class XI

In todayrsquos world every business enterprise is a part of the society It exists and operates in association with various groups in society such as customers suppliers competitors banks and financial institutions government agencies trade unions media and so on All these groups influence the functioning of business in one way or the other They constitute the environment of businessConcept of Business Environment

The term lsquobusiness environmentrsquo refers to the sum total of all individuals institutions and other forces that lie outside a business enterprise but that may influence its functioning and performance

The main features of business environment Totality of External forces General and Specific forces Interrelatedness Complexity Dynamic Uncertainty

Prepare the following questions from todayrsquos assignment

2 What do you mean by business environment

The term lsquobusiness environmentrsquo means the aggregate of all forces factors and institutions which are external to and beyond the control of an individual business enterprise but they may influence its functioning and performance Business environment is the macro framework within which a business firm a micro unit operates It consists of several interrelated and interacting elements

2 Explain the main features of business environment in brief

Totality of External forces-Business environment is the sum total of all things external to a business environment

General and Specific forces-It

Relativity

The Interrelation between business and its environment

The business enterprise is an open system It continuously interacts with its environment It takes inputs (such as raw materials capital labour energy and so on) from its environment transforms them into goods and services and sends them back to the environment

Fig 1 Business Environment Relationship

includes both the forces general forces are the economic social political legal and technological conditions which indirectly influence all business enterprise Specific forces are the investors customers competitors and suppliers which influence individual enterprise directly

Interrelatedness-Different elements of environment are interrelated for an example growing awareness for health care has increased the demand for health foods

Complexity- Business environment id complex in nature as the elements keep on changing example economic technological and other forces changes in demand for a product and service

Dynamic-Business environment is not static it keeps on changing

Uncertainty- Itrsquos very difficult to predict future events such as technology and fashion which occur fast and frequently

Business Studies

Human Resources Management

Human resource of an organisation are the aggregate of knowledge skills attitudes of people working in it

The management system which deals with human resources is called human resource management

Features of HRMbullComprehensive functionbullPeople-oriented

Question1) What do you mean by human

resource management Answer) Human resource management may be defined as that field of Management which has to do with planning organising and controlling the functions of procuring developing maintaining and utilising the labour force

bullAction oriented bullPervasive function bullContinuous function

2) Explain the features of HRM in brief

Answer)bullHuman Resource Management is concerned with managing people at work bull Human Resource Management is concerned with employees which bring people and organisations together so that the goals of each are met bullHuman resource management considered every employees as an individual and also promote their satisfaction and growth bull Human resource management is inherent in all organisations and at all levelsbullManagement of human resources are ongoing on never ending process which requires a constant alertness and Awareness of human relations

3) ldquoHR function is said to be pervasiverdquowhy

Answer) Human resource management is required in all organisations whether it is private or government organisations armed forces sports organisations etc It permeatsall the functional areas like production marketing finance research etc This from this feature of human resource management it can be said that it is pervasive in nature

Economics Demand Q1DEFINITION OF DEMANDIn economics demand is the quantity of a good that consumers are willing and able to purchase at various prices during a given period of timeQ2DEMAND CURVEIn economics a demand curve is a graph depicting the relationship between the price of a certain commodity and the quantity of that commodity that is demanded at that pricQ3LAW OF DEMANDIn microeconomics the law of demand states that conditional on all else being equal as the price of a good increases quantity demanded decreases conversely as the price of a good decreases quantity demanded increasesQ4ASSUMPTION of LAW OF DEMAND(i)No change in price of related commodities(ii) No change in income of the consumer(iii) No change in taste and preferences customs habit and fashion of the consumer( No expectation regarding future change in priceQ5MARKET DEMAND SCHEDULEIn economics a market demand schedule is a tabulation of the quantity of a good that all consumers in a market will purchase at a

given price At any given price the corresponding value on the demand schedule is the sum of all consumersrsquo quantities demanded at that priceQ6INDIVIDUAL DEMAND SCHEDULEIndividual demand schedule refers to a tabular statement showing various quantities of a commodity that a consumer is willing to buy at various levels of price during a given period of timeQ7 FACTORS AFFECTING INDIVIDUAL DEMAND FOR A COMMODITY

The factors that influence a consumerrsquos decision to purchase a commodity are also known as determinants of demand The following factors affect the individual demand for a commodity1 price of the commodity2 price of related goods3 income of buyer of the commodity4 tastes and preferences of the buyer1 Price of the CommodityYou must have observed that when price of a commodity falls you tend to buy more of it and when its price rises you tend to buy less of it when all other factors remain constant (lsquoother things remaining the samersquo) In other words other things remaining the same there is an inverse relationship between the price of a commodity and its quantity demanded by its buyers This statement is in accordance with law of demand which you will study in the later part of this lesson Price of a commodity and its quantity demanded by its buyers are inversely related only when lsquoother things remain the samersquo So lsquoother things remaining the samersquo is an assumption when we study the effect of changes in the price of a commodity on its quantity demanded2 Price of Related goodsA consumer may demand a particular good But while buying that good heshe also asks the price of its related goods Related goods can be of two types-(i) Substitute goods(ii) Complementary goods While purchasing a good prices of its substitutes and complements do affect its quantity purchased(i) Price of Substitute Goods Substitute goods are those goods which can easily be used in place of one another for satisfaction of a particular want like tea and coffee An increase in price of substitute good leads to an increase in demand for the given commodity and a decrease in price of substitute good leads to a decrease in demand for the given commodity It means demand for a given commodity is directly affected by change in price of substitute goods For example if price of coffee increases the demand for tea will rise as tea will become relatively cheaper in comparison to coffee(ii) Price of Complementary goods Complementary goods are those goods which are used together to satisfy a particular want like car and petrol An increase in the price of complementary goods leads to a decrease in demand for the given commodity and a decrease in the price of complementary goods leads to an increase in demand for the given commodity For example if price of petrol falls then the demand for cars will increase as it will be relatively cheaper to use both the goods together So demand for a given commodity is inversely affected by change in price of complementary goods3 Income of the Buyer of CommodityDemand for a commodity is also affected by income of its buyer However the effect of change in income on demand depends on the nature of the commodity under consideration In case of some goods like full cream milk fine quality of rice (Basmati rice) etc demand for these commodities increases when income of the buyer increases and

demand for these commodities decreases when income of the buyer decreases Such goods whose demand increases with the increase in income of the buyer are called normal goods But there are some goods like coarse rice toned milk etc whose demand decreases when income of buyer increases and their demand increases when income of the buyer decreases Such goods whose demand decreases with the increase in income of the buyer are called inferior goods Suppose a consumer buys 10 Kgs of rice whose price is ` 25 per Kg He cannot afford to buy better quality of rice because the price of such rice is ` 50 per Kg The consumer is spending ` 250 per month on the purchase of rice Now if income of the consumer increases and he can afford ` 350 on purchase of 10 Kg of rice Now he can afford to buy some quantity of rice say 6 Kgs whose price is ` 25 per Kg and may buy 4 Kgs of rice whose price is ` 50 per Kg Thus he will buy 10 Kgs of rice by spending ` 350 per month Therefore we may conclude that demand for normal goods is directly related to the income of the buyer but demand for inferior goods is inversely related to the income of the buyer4 Tastes and Preferences of the BuyerThe demand for a commodity is also affected by the tastes and preferences of the buyers They include change in fashion customs habits etc Those commodities are preferred by the consumers which are in fashion So demand for those commodities rises which are in fashion On the other hand if a commodity goes out of the fashion its demand falls because no consumer will like to buy it(5) Number of Buyers in the Market(Population)Increase in population raises the market demand whereas decrease in population reduces the market demand for a commodity Not only the size of population but its composition like age (ratio of males females children and old people in population) also affects the demand for a commodity It is because of needs of children young old male and female population differs(6) Distribution of Income and WealthIf the distribution of income and wealth is more in favour of the rich demand for the commodities preferred by the rich such as comforts and luxuries is likely to be higher On the other hand if the distribution of income and wealth is more in favour of poor demand for commodities preferred by the poor such as necessities will be more(7) Season and Weather ConditionsThis is generally observed that the demand for woolens increases during winter whereas demand for ice creams and cold drinks increases during summer Similarly market demand for umbrellas rain coats increases during rainy seasonQ8 REASONS FOR OPERATION OF LAW OF DEMAND WHY DEMAND CURVE SLOPES DOWNWARDNow we will try to explain why does a consumer purchase more quantity of a commodity at a lower price and less of it at a higher price or why does the law of demand operate ie why does the demand curve slope downwards from left to right The main reasons for operation of law of demand are1 Law of Diminishing Marginal UtilityAs you have studied earlier law of diminishing marginal utility states that as we consume more and more units of a commodity the utility derived from each successive unit goes on decreasing The consumer will be ready to pay more for those units which provide him more utility and less for those which provide him less utility It implies that he will purchase more only when the price of the commodity falls2 Income Effect

When price of a commodity falls purchasing power or real income of the consumer increases which enables him to purchase more quantity of the commodity with the same money income Let us take an example Suppose you buy 4 ice creams when price of each ice cream is ` 25 If price of ice creams falls to ` 20 then with same money income you can buy 5 ice creams now3 Substitution EffectWhen price of a commodity falls it becomes comparatively cheaper as compared to its substitutes (although price of substitutes has not been changed) This will lead to rise in demand for the given commodity For example if coke and Pepsi both are sold at ` 10 each and price of coke falls Now coke has become relatively cheaper and will be substituted for Pepsi It will lead to rise in demand for coke4 Change in Number of BuyersWhen price of a commodity falls some old buyers may demand more of the commodity at the reduced price and some new buyers may also start buying this commodity who were not in a position to buy it earlier due to higher price This will lead to increase in number of buyers when price of the commodity falls As a result demand for the commodity rises when its price falls5 Diverse Uses of a CommoditySome commodities have diverse uses like milk It can be used for drinking for sweet preparation for ice cream preparation etc If price of milk rises its use may be restricted to important purpose only This will lead to reduction in demand for other less important uses When price of milk falls it can be put to other uses also leading to rise n demand for itQ9 EXCEPTIONS TO THE LAW OF DEMANDYou have studied in law of demand that a buyer is willing to buy more quantity of a commodity at a lower price and less of it at a higher price But in certain circumstances a rise in price may lead to rise in demand These circumstances are called Exceptions to the Law of Demand Some important exceptions are1 Giffen GoodsGiffen goods are special type of inferior goods in which negative income effect is stronger than negative substitution effect Giffen goods do not follow law of demand as their demand rises when their price rises Examples of Giffen goods are jowar and bajra etc2 Status Symbol GoodsSome goods are used by rich people as status symbols eg diamonds gold jewellary etc The higher the price the higher will be the demand for these goods When price of such goods falls these goods are no longer looked at as status symbol goods and tehrefore therir demand falls3 NecessitiesCommodities such as medicines salt wheat etc do not follow law of demandbecause we have to purchase them in minimum required quantity whatever their price may be4 Goods Expected to be ScarceWhen the buyers expect a scarcity of a particular good in near future they start buying more and more of that good even if their prices are rising For example during war famines etc people tend to buy more of some goods even at higher prices due to fear of their scarcity in near future

Political Science

Constitution of India-The

Preamble

The preamble-

Preamble-

The preamble is the most precious part of the constitution We the people of India having solemnly resolved to constitute India into a Sovereign Socialist Secular Democratic Republic and to secure to all its citizensA preamble is an introductory and expressionary statement in a document that explains the documents purpose and underlying philosophy When applied to the opening paragraphs of a statute it may recite historical facts pertinent to the subject of the statuteNature and purpose of the constitution-Purpose of the Constitution dictates permanent framework of the government to form a more perfect union to establish justice and ensure peace of thenationconstitution provide principles how the government can run itself following the rules and laws written in the constitution of each state keeps them balanced

Answer the following questions-

1 What is preambleA preamble is an introductory and expressionary statement in a document that explains the documents purpose and underlying philosophy2 What is the nature and

purpose of the constitutionConstitution dictatespermanent framework of the government to form a more perfect union to establish justice and ensure peace of the nation

Homework-Learn

Accounts Compatibilty mode

1MEANING OF PARTNERSHIPPartnership is a form of business organisation where two or more persons join hands to run a business They share the profits and losses according to the agreement amongst them According to the Indian Partnership Act 1932 ldquoPartnership is relation between persons who have agreed to share profits of a business carried on by all or any one of them acting for allrdquo For example one of your friends has passed class XII from National Institute of Open Schooling (NIOS) and wants to start a business Heshe approaches you to join in this venture Heshe wants you to contribute some money and participate in the business activities Both of you if join hands constitute a partnership2CHARACTERISTICS1048698 Agreement A partnership is formed by an agreement The agreement may be either oral or in writing It defines the relationship between the persons who agree to carry on business It may contain the terms of sharing profit and the capital to be invested by each partner etc The written agreement is known as partnership deed1048698 Number of persons There must be at least two persons to form a partnership

The maximum number of partners in a partnership firm can be 50 according toCompanies Act 20131048698 Business The Partnership is formed to carry on business with a purpose of earning profits The business should be lawful Thus if two or more persons agree to carry on unlawful activities it will not be termed as partnership1048698 Sharing Profits The partners agree to share profits in the agreed ratio In caseof loss all the partners have to bear it in the same agreed profit sharing ratio10486981048698Mutual Agency Every partner is an agent of the other partners Every partner can bind the firm and all other partners by hisher acts Each partner will be responsible and liable for the acts of all other partners10486981048698Unlimited liability The liability of each partner except that of a minor is unlimited Their liability extends to their personal assets also If the assets of the firm are insufficient to pay off its debts the partnersrsquo personal property can be used to satisfy the claim of the creditors of the partnership firm10486981048698Management All the partners have a right to mange the business However they may authorize one or more partners to manage the affairs of the business on their behalf10486981048698Transferability of Share No partner can transfer hisher share to any one including hisher family member without the consent of all other partners3PARTNERSHIP DEEDAgreement forms the basis of partnership The written form of the agreement is which a document of partnership is It contains terms and conditions regarding the conduct of the business It also explains relationship between the partners This document is called partnership deed Every firm can frame its own partnership deed in which the rights duties and liabilities of the partners are stated in detail It helps in settling the disputes arising among the partners during the general conduct of business 4CONTENTS OF PARTNERSHIP DEEDThe partnership deed generally contains the following (i) Name and address of the partnership firm(ii) Nature and objectives of the business(iii) Name and address of each partner(iv) Ratio in which profits is to be shared(v) Capital contribution by each partner(vi) Rate of Interest on capital if allowed(vii) Salary or any other remuneration to partners if allowed(viii) Rate of interest on loans and advances by a partner to the firm(ix) Drawings of partners and interest thereon if any(x) Method of valuation of goodwill and revaluation of assets and liabilities on the reconstitution of the partnership ie on the admission retirement or death of a partner(xi) Settlement of disputes by arbitration(xii) Settlement of accounts at the time of retirement or death of a partner5IN ABSENCE OF PARTNERSHIP DEEDThe partnership deed lays down the terms and conditions of partnership in regard to rights duties and obligations of the partners In the absence of partnership deed there may arise a controversy on certain issues like profit sharing ratio interest on

capital interest on drawings interest on loan and salary of the partners In such cases the provisions of the Indian Partnership Act becomes applicableSome of the Issues are(i) Distribution of Profit Partners are entitled to share profits equally(ii) Interest on Capital Interest on capital is not allowed(iii) Interest on Drawings No interest on drawing of the partners is to be charged(iv) Interest on Partnerrsquos Loan A Partner is allowed interest 6 per annum on the amount of loan given to the firm by himher(v) Salary and Commission to Partner A partner is not entitled to anysalary or commission or any other remuneration for managing the business

History TOPIC-TOWARDS INDEPENDENCE AND PARTITION THE LAST PHASE (1935-1947)

SUB TOPIC-IMPORTANT POLITICAL DEVELOPMENTS ndash GROWTH OF SOCIAL IDEAS

Socialism is a political social and economic philosophyLike in other parts of the world the Russian revolution of 1917 served as a great inspiration for revolutionaries in India who at that time were engaged in the struggle for liberation from British ruleSocialist ideas led to the formation of communist party of IndiaJAWAHARLAL NEHRU Among the early Congress leaders Jawaharlal Nehru was very much impressed and influenced by the Socialist ideas He also learnt about the Economic activities of the Soviet Union after the Bolshevic Revolution 1917 He made full use of them in IndiaThe election of Jawaharlal Nehru and Subhas Chandra Bose showed the Left wing tendency within CongressJawaharlal Nehru demanded economic freedom along with political freedom of the people in order to end the exploitation of masses

Nehrus working committee included three socialists leaders The Lucknow session was a landmark in the evolution of socialist ideas of the congressSUBHAS CHANDRA BOSE ndash Subhas Chandra Bose had socialist leaning Both Jawaharlal Nehru and Subhas Chandra Bose were known as leftist Congress men Later on National Congress divided into Leftist and rightist campCONGRESS SOCIALIST Within the Congress some leaders formed the Congress Socialist partyPattavi Sitaramyya Sardar Patel Rajendra Prasad had hostile attitude towards the Congress Socialist partyJawaharlals attitude was hesitant

1 QUESTION ndash Mention name of two Congress leaders who had socialist leaning

1ANSWER ndash Subhas Chandra Bose and Jawaharlal Nehru2QUESTION- In which session of the congress Jawaharlal elaborated his Socialist ideas2 ANSWER ndash Lucknow and Faizpur Session in December 1935 and 19363QUESTION ndash Why Congress was sharply divided into leftist and rightist camp 3ANSWER ndash Subhas Chandra Bosersquos attempt to seek re election for congress presidentship in 1939sharply divided the National Congress into Leftist and Rightist camp4 QUESTION ndash Who was MN Roy 4 ANSWER ndash Manabendra Roy first formed the Communist Party of India outside the country at Tashkent in 19205QUESTION ndash Who formed the Congress Socialist Party within the Congress5 ANSWER ndash Jaya Prakash Narayan Achyut Patwardhan Acharya Narendra Dev Ram Mohan Lohia Aruna Asaf Ali6QUESTION ndash When was the Congress Socialist Party formed What was its object6 ANSWER ndash 1934The Congress Socialist Party sought to work out socialist programme through the Congress They joined hands with the Congress and wanted to carry

Subhas Chandra Bose being expelled from the congress after the Tripuri rift he formed Forward BlockThere were basic differences between the Congress Socialists and the communistsTRADE UNION ACTIVITIES Maximum working class people lived in Bombay and Calcutta The working and living conditions of those workers were very miserable In this situation Shasipada Banerjee NM Lokhande protested against the oppression of the working class peopleThe first Trade Union Madras Labour Union was formed in 1918 by BP WadiaIndustrial strikes took place in Kanpur Calcutta Madras Jamshedpur and Ahmedabad AITUC was formed in Bombay in 1927 The growth of Trade union among the workers was slow because of the fear of the dismissal of the jobIn the mean time the Moderates as well as Communists left AITUC and formed separate organization

on National struggle with the help of workers and peasant class of the society7 QUESTION ndash What was the name of the party founded by Subhas Chandra Bose7 ANSWER- Forward Block8QUESTION ndash Who was Shasipada Banerjee8 ANSWER ndash Shasipada Banerjee was a radical Brahmo He founded a working menrsquos club to protest against exploitation of the British rulers towards the working class of India9 QUESTION ndash What was the weekly published by NM Lokhande9ANSWER- Dinabandhu10 QUESTION ndash Who founded Bombay Mill-Hands Association and in which year10 ANSWER- NM Lokhande in189011 QUESTION- Who was BP WadiaANSWER- BPWadia was the founder of Madras Labour Union in191812 QUESTION- What was the name of the first labour union of India12 ANSWER- Madras Labour Union13 QUESTION Who founded the Majur Mahajan 13 ANSWER GANDHIJI14 QUESTION What was the full form of AITUC When it was formed14 ANSWER All India Trade Union Congressin 192715QUESTION Who formed the Red Trade Union Congress and in which year15ANSWER The Communists formed the Red Trade Union Congress16 QUESTION What do you mean by Socialism16 ANSWER Socialism describes any political and economic theory that says the community rather than individuals should own and manage property and natural resources

Subject Eng Literature (The Tempest ndash William Shakespeare) Topic Act III Scene 3 Lines 1 to 52 (Line 52 ndash Brother my lord the Duke Stand to and do as we) Date 13th April 2020 (4th Period)

[Students should read the original play and also the paraphrase given in the school prescribed textbook]Summary Questions amp Answers

o Alonso Sebastian Antonio Gonzalo Adrian Francisco and others wandered about the island in search of Ferdinand and gets tired and hungry of the toil and at the same time gives up all hope of finding him

o Antonio and Sebastian are happy that Alonso is out of hope and decide to make another attempt on his life that night when being so tired they will be sleeping soundly

o Suddenly a solemn and strange music is heard in the air and several strange shapes enter bringing in a banquet These strange shapes then dance round it with gestures of salutation and then inviting the King to eat they depart

o Seeing this strange scene all are inclined to believe the tales told by travelers that there truly are ldquounicornsrdquo and ldquothe phoenixrsquo thronerdquo

1 ALONSO What harmony is this My good friends hark (L18-27)

GONZALO Marvellous sweet music

[Enter several strange shapes bringing in a banquet

they dance about it with gentle actions of salutation

and inviting the King and his companions to eat they depart]ALONSO Give us kind keepers heavens What were theseSEBASTIAN A living drollery Now I will believe

That there are unicorns that in Arabia

There is one tree the phoenixrsquo throne one phoenix

At this hour reigning thereANTONIO Ill believe both

And what does else want credit come to me

And Ill be sworn rsquotis true Travellers neer did lie

Though fools at home condemn rsquoem

(a) How did Prospero present an amazing spectacle before Alonso and his companions

Using his magic powers Prospero ordered strange shapes to lay a banquet before Alonso and his companions The shapes brought several dishes with tasty eatables in them They placed the dishes on a table before Alonso and his companions Then the strange shapes began to dance gracefully around the banquet While dancing they made gestures inviting them to eat the food Then suddenly the shapes disappeared(b) Who were the guests at the strange banquet Describe the lsquoliving drolleryrsquo

Alonso Sebastian Antonio Gonzalo Adrian and Francisco were the guests at the strange banquet

The term ldquoliving drolleryrdquo refers to live entertainment show In this context when Alonso the King of Naples Sebastian his brother Antonio the treacherous brother of Prospero Gonzalo the kind and loyal councillor to the King Adrian and Francisco came to the island they were hungry and weary in their spirits They heard a solemn and strange music They were shocked to see several strange shapes bringing in a banquet and these shapes danced about it with gentle action of salutation inviting the King and his companions to eat After this Sebastian described this show as lsquoliving drolleryrsquo(c) What is lsquophoenixrsquo What are lsquoUnicornsrdquo

The term lsquophoenixrsquo refers to a mythical Arabian bird which lived alone and perched on a solitary tree After one hundred years it expired in flames and rose again from its own ashes

lsquoUnicornsrsquo refers to the mythological four-footed beasts having horns in the centre of their foreheads When the horns are ground into powder the powder was believed to be

an aphrodisiac(d) How does Sebastian explain the puppet show OR Why does the speaker now believe in unicorns and phoenix

Sebastian finds several strange shapes bringing in the banquet They invite the king and his party for dinner and soon depart He tells that if such a strange sight can be a reality there is nothing incredible in the world and from the present moment he will believe anything He says that it is a strange dumb show enacted not by puppets but by living beings It is stranger than a travellerrsquos tale Seeing such a thing

before his own eyes he will no longer disbelieve the story about unicorns and phoenix(e) How do the other characters present respond to this living drollery

At the sight of the lsquoliving drolleryrsquo like Sebastian Gonzalo and Antonio too acted strangely Antonio told that he too now believes in unicorns and phoenix and anything else that seems to be incredible He too now believes in travellersrsquo tales Gonzalo told that if he would report those happenings in Naples nobody will believe him He considers that those gentle shapes were gentler in manner in comparison to the living beings Alonso was at first sight suspicious and told them that those strange shapes conveyed their meaning in expressive gestures when they seemed to lack speech by their movements and sounds Francisco was amazed at their mysterious disappearance

2 ALONSO Not I

(Line 43-52)GONZALO Faith sir you need not fear When we

were boysWho would believe that there were mountaineers

Dewlapped like bulls whose throats had hanging at rsquoem

Wallets of flesh Or that there were such men

Whose heads stood in their breasts Which now we find

Each putter-out of five for one will bring us

Good warrant ofALONSO I will stand to and feed

Although my lastmdashno matter since I feel

The best is past Brother my lord the Duke

Stand to and do as we

(a) How does Alonso respond at the spectacle of the shapes which were sent to them at the instruction of Prospero

After seeing the strange sight of appearing and disappearing of the shapes sent by Prospero to arrange a banquet for them Alonso says that his surprise at having seen those creatures is infinite and he is fully justified in feeling so much surprise He thinks that their shapes their gestures and the sounds they made were indeed amazing Although they do not possess the gift of speech yet they were able to convey their

thoughts by means of their gestures only

(b) What does Prospero say about the views expressed by Alonso regarding the shapes What does Francisco think about the shapesAfter hearing Alonsorsquos views about the shapes Prospero says that this manrsquos praise of the spirits is rather hasty He means to say that Alonso has shown great haste in reaching the conclusion about the shapes Francisco is amazed to see that those shapes disappeared in a mysterious way(c) What does Sebastian ask Alonso to doSebastian tells Alonso that the shapes having disappeared should not matter to them because they have left the eatables behind He asks Alonso to enjoy eating as they are extremely hungry but the king does not accept his offer of enjoying the dishes(d) How does Gonzalo try to dispel Alonsorsquos fear of those strange shapes What kind of references does he give to AlonsoGonzalo says that those who have travelled abroad have reported seeing even stranger sights than these shapes that Alonso and his companions have beheld Hence there is no reason to feel afraid of these shapes Gonzalo further adds that in his younger days he had heard strange stories from travelers and Alonso might have heard similar stories For instance it was said that there existed a certain race of

human beings who had huge lumps of flesh hanging at their throats and who therefore resembled bulls Then Gonzalo tells about a race of human beings whose heads were located at their breasts Gonzalo says that such stories were not believed by most people in those days but now-a-days these stories have become common(e) Explain the following lsquoEach putter-out of five for onersquoEnglish travellers often insured their trips with London brokers Those that went on foreign travels those days used to deposit a certain amount with some firm or company in London before their departure If the travelers failed to return the money was forfeited by the company with which it had been deposited But this money was repaid five-fold if the travelers returned safe and sound In this way a traveler stood a great chance of recovering the entire cost of his

travels(f) Give the explanatory meanings of the following expressions in the context of the above extract (i) Dewlapped (ii) Wallets of flesh

(iii) Putter-out(i) Dewlapped having big lumps of flesh at the necks(ii) Wallets of flesh large masses of flesh looking like bags(iii) Putter-out to invest money before commencing the travel

  • General methods of preparation of hydrogen
  • Chapter Dimensional Analysis (Summary)
    • Properties of Charge
Page 40:   · Web viewSubject. Topic. Summary. Execution. Hindi. व्याकरण. शरीरके अंगो के नाम लिखिए. 1) आँख 2) नाक 3

ldquowatermelon modelrdquoThis model could explain the electrical neutrality of an atom but failed to explain the result of scattering experiment carried out by Rutherford in 1911So it was rejected ultimately

Q4)Why is an electron called universal particleAns Itrsquos mass and Charge are independent of its source

EVS Chapter 1 ndash Modes of Existence

Modes of existence When one speaks normally about the mode of existence of some group or individual one refers to their customs their mode of being their ethology their habitat in some way their feeling for a placeDifferent modes of exixtence are ndash

1 Hunting ndashGathering2 Pastoral3 Agricultural4 Industrial

1 Hunting and gathering Hunting and gathering mode of existence is characterized by obtaining food from hunting wild animals including fishing and gathering wild plants From their earliest days the hunter-gatherer diet included various grasses tubers fruits seeds and nuts Lacking the means to kill larger animals they procured meat from smaller game or through scavenging

Societies that rely primarily or exclusively on hunting wild animals fishing and gathering wild fruits berries nuts and vegetables to support their diet are called hunting and gathering societies

At least this used to be practice of human beings before agriculture is invented As their brains evolved hominids developed more intricate knowledge of edible plant life and growth cycles

Q) Write the features of Hunting ndash gathering societiesAns - There are five basic characteristics of hunting and gathering societies

i The primary institution is the family which decides how food is to be shared and how children are to be socialized and which provides for the protection of its members

ii They tend to be small with fewer than fifty members

iii They tend to be nomadic moving to new areas when the current food supply in a given area has been exhausted

iv Members display a high level of interdependence

v Labor division is based on sex men hunt and women gather

Political Science

Introduction to political science

Political science occasionally called politology is a social science which deals with systems of governance and the analysis of political activities political thoughts associated constitutions and political behaviorThe study of political science involves the study of both the

Answer the following questions-1 What is political science

Political science occasionally called politology is a social science which deals with systems of governance and the analysis of political activities political thoughts associated constitutions and political behavior

2 Short notes-

traditional and modern theories of politicsTraditionalClassical political sciencepolitical theory-Traditional political science is the study of politics before Second World War The methodology to study Politics was traditional (legal formaletc) the definition of politics traditional (Politics begins and end with state)area of study (constitution state machinery)was traditionalModern Political scienceModern political theory-Modern Political Theory critically examines the contemporary state of political theory making an assessment of the achievement and limitations of the Behavioural Revolution in its totality and reviews objectively the major paradigms and conceptual frameworks adopted by the disciplineContemporary attempts at the development of an integrated political theory involving the use of both traditional and modern concepts approaches and theories-Around late 1960s several political scientists realized the importance of both the traditional political theory and modern Political theory They began building an integrated theory of politics involving a systematic mixture of traditional and modern studies of politics It was held that the study of a complex and vast field like politics needs both traditional as well as

Classical political theory Modern Political theory

Homework-Learn

modern concepts and approaches for studying itrsquos all aspects

Subject Eng Literature (The Tempest ndash William Shakespeare) Topic Act I Scene 1 Lines 1 to 32 (Line 32 ndash Gonzalo hellip If he be not born to be hanged our case is miserable) Date 13th April 2020 (3rd Period)

[Students should read the original play and also the paraphrase given in the school prescribed textbook]Summary Questions amp Answers

[SUMMARY OF THE ENTIRE SCENE]

o The play starts with the scene of a severe storm at sea Alonso (King of Naples) Sebastian (Alonsorsquos brother) Ferdinand (Alonsorsquos son) Gonzalo Antonio (the usurping Duke of Milan) are in a ship in the midst of the storm

o The mariners are trying their best to control the vessel from running aground and are totally following the orders of their Master the Boatswain They have scant success

o The mariners become extremely unhappy and annoyed when most of the passengers arrive on the deck thereby hampering their effort to save the ship There is serious confrontation between them and the passengers who are part of the Kingrsquos entourage

o The mariners could not save the ship

SUMMING-UP

(i) Vivid description of the scene which gives a realistic description of terror and confusion of a tropical storm

(ii) Shows Shakespearersquos accuracy of knowledge in describing the naval operations and also matters of seamanship

(iii) The opening scene justifies the title ndash The Tempest

UNANSWERED QUESTIONS

(i) The King always travels with his entire fleet including his soldiers Where

(1) GONZALO Nay good be patient (Line 15-26)BOATSWAIN When the sea is Hence What cares these

roarers for the name of the king To cabin silence Trouble us not

GONZALO Good yet remember whom thou has aboardBOATSWAIN None that I more love than myself You are a

councillor if you can command these elements to silence and work

the peace of the present we will not hand a rope more use your authority If you cannot give thanks you have

lived so long and make yourself ready in your cabin for the mischance of the hour if it so hap [To the Mariners]

Cheerly good hearts [To Gonzalo] Out of our way I say

(a) To whom is the boatswain speaking What does he mean by lsquoNone that I more love than myselfrsquo

The Boatswain is speaking to Gonzalo the honest old councilor of the Duke of MilanBy using the words ndash lsquoNone that I love more than I love myselfrsquo means that for the Boatswain nobody is dearer to him than his own life

(b) What were the conditions that made the boatswain react in this way

The Boatswain reacts in this way because the storm is at sea and Alonso King of Naples Sebastian his brother Ferdinand his son Gonzalo Antonio the usurping Duke of Milan on board are in distress and in panic Thus they have rushed to the deck interrupting the work of the mariners

(c) What hope does Gonzalo take from the attitude of the boatswain

The insolent and authoritative attitude of Boatswain makes Gonzalo feel comforted He tells that there are no signs that the Boatswain will be drowned But his facial appearance and attitude shows that he is destined to die on land by hanging which in effect means that all on board will be saved Otherwise all the persons on board are doomed

(d) How can they lsquomake yourself ready in your cabinrsquo For what were they asked to make ready themselves

In order to make themselves ready in their cabin the

were the other ships

(ii) Why was the ship in that area Where was it coming from or going where

(iii) The ship broke apart What happened to those who were in the ship

passengers on board must prepare for death which they will possibly soon have to meetThey can retire to their cabins and offer prayers to the Almighty to save them from drowning

(e) What does the boatswain say when he is asked to be patient What does he order to the royal party

When the boatswain is asked to be patient and remain calm he says that he will be patient only when the storm will be over and the sea will be calm but as long as the storm blows and there is danger to the ship he cannot think of being patient He orders the royal party to go to the cabin and leave the mariners to their work

(2) GONZALO I have great comfort from this fellow (Line 27-36)

Methinks he hath no drowning mark upon him his complexion is perfect

gallows Stand fast good Fate to his hanging Make the rope of his destiny our cable for our own doth little advantage If he be not born to be hanged our case is miserable

(a) Why does Gonzalo regard the Boatswain in the midst of danger

In the midst of danger Gonzalo regards the boatswain because he feels that the Boatswain is a source of comfort and is bent upon to do his work sincerely which in this case is saving the ship and its passengers from the severest of raging storm

(b) What reasons does Gonzalo give when he says that none in the ship will die of drowning

Gonzalo is almost sure that none in the ship will die by drowning His says that there is no mark on the face of the boatswain that indicates that he will die by drowning On the other hand the lines on his face are strong indications that he will be hanged to death Therefore there shall be no danger of the shiprsquos sinking

(c) Explain the following ldquoStand fast good Fate to his hanging Make the rope of his destiny our cable for our own doth little advantage If he be not born to be hanged our case is miserablerdquo

The stated lines mean that if the will of destiny is to be carried out then the ship will not get wrecked and all the passengers will be saved The safety of the passengers therefore depends upon the will of fate being carried out in the case of the boatswain If however the boatswain is not to die by hanging then the passengers are also very unsafe because in that case the ship is likely to sink

(d) What order does the Boatswain give to the sailors

when he re-enters What does he say about the crying of the fellows inside the cabin

The boatswain orders the sailors to bring the topmast lower and bring the ship close to a stationary position with the help of the main sail He says that the fellows inside the cabin are moaning and crying in their distress louder than his voice and louder even than the roaring of the storm

Class XII (ScienceCommerceHumanities) Subject Topic Summary Execution

Computer Science

PropositionalLogic

Propositional logic is a procedure to provide reasoning through statementProposition A ststement that results in True or False is said to be proposition There are two types of propositionSimple proposition amp compound propositionSimple proposioton A simple proposition is one that is not a part of any other proposition Such sentential form of proposition is symbolized with english letters in short For example Ram is a claver student (TrueFalse)Where do you live (Not in True or False)Grapes are sweet (TrueFalse)It rains today (TrueFalse)Here we can see some statements anwer would be true or false but some staements answer can not give in terms of true or false Thus the sentences which can be answered in true or false are known as simple propositionAssigning propositon to a variableThe general syntax to assign propostion to a variable is as followsVariable = Simple propositonFor example A=Ram is a clever studentB= Grapes are sweetC= it rains todayCompound proposition

helliphellipto be continued in next classhelliphellipMath Relation Relation If A and B are two non-empty sets

then a relation R from A to B is a subset of AxB If R A x B and (a b) R then we say that a sube isinis related to b by the relation R written as aRbeg Let A be the set of students of class XII and B be the set of students of class XI Then some of the examples of relation from A to B arei) (a b) AXB a is brother of bisinii) (a b) AXB age of a is more than age of isinb Types of relation In this section we would like to study different types of relations We know that a relation in a set A is a subset of A times A Thus the empty set φ and A times A are two extreme relations For illustration consider a relation R in the set A = 1 2 3 4 given by R = (a b) a ndash b = 10 This is the empty set as no pair (a b) satisfies the condition a ndash b = 10 Similarly R = (a b) | a ndash b | ge 0 is the whole primeset A times A as all pairs (a b) in A times A satisfy | a ndash

Example 1 Let A be the set of all students of a boys school Show that the relation R in A given by R = (a b) a is sister of b is the empty relation and R = (a b) the primedifference between heights of a and b is less than 3 meters is the universal relationSolution Since the school is boys school no student of the school can be sister of any student of the school Hence R = φ showing that R is the empty relation It is also obvious that the difference between heights of any two students of the school has to be less than 3 meters This shows that R = A times A is primethe universal relation Example 2 Show that the relation R in the set 1 2 3 given by R = (1 1) (2 2) (3 3) (1 2) (2 3) is reflexive

b | ge 0 These two extreme examples lead us to the following definitionsDefinition 1 A relation R in a set A is called empty relation if no element of A isrelated to any element of A ie R = φ A times AsubDefinition 2 A relation R in a set A is called universal relation if each element of A is related to every element of A ie R = A times A Both the empty relation and the universal relation are some times called trivial relation Definition 3 A relation R in a set A is called(i) reflexive if (a a) R for every a Aisin isin(ii) symmetric if (a1 a2) R implies that (aisin 2a1)

R for all aisin 1 a2 Aisin(iii) transitive if (a1 a2) R and (aisin 2 a3) R isinimplies that (a1 a3) R for all aisin 1 a2 a3 AisinDefinition 4 A relation R in a set A is said to be an equivalence relation if R is reflexive symmetric and transitive

but neither symmetric nor transitiveSolution R is reflexive since (1 1) (2 2) and (3 3) lie in R Also R is not symmetric as (1 2) R but (2 1) isin notinR Similarly R is not transitive as (1 2) R and (2 3) R but (1 3) R isin isin notinExample 3 Show that the relation R in the set Z of integers given byR = (a b) 2 divides a ndash b is an equivalence relationSolution R is reflexive as 2 divides (a ndash a) for all a Z isinFurther if (a b) R then 2 divides a isinndash b Therefore 2 divides b ndash a Hence (b a) R which shows that R is isinsymmetric Similarly if (a b) R and (b c) R isin isinthen a ndash b and b ndash c are divisible by 2 Now a ndash c = (a ndash b) + (b ndash c) is even (Why) So (a ndash c) is divisible by 2 This shows that R is transitive Thus R is an equivalence relation in ZExample 4 Let L be the set of all lines in a plane and R be the relation in L defined as R = (L1 L2) L1 is perpendicular to L2 Show that R is symmetric but neither reflexive nor transitiveSolution R is not reflexive as a line L1 can not be perpendicular to itself ie (L1 L1) R notinR is symmetric as (L1 L2) Risin

L1 is perpendicular to L2rArr L2 is perpendicular to L1rArr (L2 L1) RrArr isin

R is not transitive Indeed if L1 is perpendicular to L2 and L2 is perpendicular to L3 then L1 can never be perpendicular to L3 In fact L1 is parallel to L3 ie (L1 L2) R isin(L2 L3) R but (L1 L3) Risin notin

Chemistry Solid state Characteristics if Solids(i)The particles are locked in fixed positions they are unable to change their relative positions and this brings a definite shape and volume of a solid(ii)In a solid the constituent particles are held by strong forces of attractionThe forces of attraction may be bonding or non bonding(iii)The constituent particles in a solid pack together as closely as possibleoccupying most of the available space within the solidThus the empty space in a solid is very smallThis makes a solid highly rigid and nearly incompressibleThis also explains why a solid has high density and exhibits slow diffusionClassification of Solids

Q1)Define Crystalline solids AnsA Solid that has a definite geometrical shape and a sharp melting pointand whose constituent particles (atomsmolecules or ions) are arranged in a long range order of definite pattern extending throughout the solid is called a crystalline solidExNaClQ2)Define Amorphous solids AnsA solid that does not have a definite shape and a sharp melting pointand whose constituent particles (atomsmolecules or ions) are not arranged in a definite pattern is called an amorphoussolid

Crystalline solidsAmorphous solids

ExGlassRubberQ3)Classify Crystalline Solids Crystalline Solids

Physics Coloumbrsquos Law (Summary)

Before Going Into Coloumbrsquos Law We Will First Learn What is Charge Properties of Charge and Always remember that charge is quantized ie a body always have static charge of magnitude equal to some integral multiple of fundamental electronic charge e= 16 x 10- 19 C

Charge is the property of matter that causes it to produce and experience electrical and magnetic effects The study of the electrical charges at rest is called electrostatics When both electrical and magnetic effects are present the interaction between charges is referred to as electromagnetic

There exist two types of charges in nature positive and negative Like charges repel and unlike charges attract each other

The type of charge on an electron is negative The charge of a proton is the same as that of an electron but with a positive sign In an atom the number of electrons and the number of protons are equal The atom is therefore electrically neutral If one or more electrons are added to it it becomes negatively charged and is designated as negative ion However if one or more electrons are removed from an atom it becomes positively charged and is called a positive ion

The excess or deficiency of electrons in a body gives the concept of charge If there is an excess of electrons in a body it is negatively charged And if there is deficiency of electrons the body becomes positively charged Whenever addition or removal of electrons takes places the body acquires a charge

The SI Unit of charge is coulomb (C) In SI units the current is a fundamental quantity having a unit of ampere (A) The unit of charge is defined in terms of the unit of current Thus one coulomb is the charge transferred in one second across the section of a wire carrying a

Ionic SolidsMetallicSolids

Molecular Solids

current of one ampere

As q = It we have1 C = (1 A) (1 s)

The dimensions of charge are [A T]

Properties of Charge

(1) Quantization of Charge Electric charge can have only discrete values rather than any value That is charge is quantized The smallest discrete value of charge that can exist in nature is the charge on an electron given as

e = plusmn 16 x 10- 19 C

This is the charge attained by an electron and a protonA charge q must be an integral multiple of this basic unit That is

Q = plusmn ne where n = 1 2 hellip

Charge on a body can never be (frac12)e (23)e or 57e etcWhen we rub a glass rod with silk some electrons are transferred from the rod to the silk The rod becomes positively charged The silk becomes negatively charged The coulomb is a very large amount of charge A typical charge acquired by a rubbed body is 10 - 8 C

Biology Reproduction in organisms

Welcome to this new session 2020-21Today in this first chapter we mainly discuss about reproduction types needs and life span of some organismsWe also discuss about difference between sexual and asexual reproduction

Q1 What is reproductionReproduction is defined as a biological processin which an organism gives rise to young onessimilar to itselfQ2 What are the needs of reproductionbulli) Reproduction maintain life on earthii) It enables the continuity of the species generation after generationiii) It creates genetic variation among populationsQ3 Define Life span and write some orgnisms life spanbull Life span is the period from birth to

the natural death of an organism- OrganismsLife span1 Butterfly 1 - 2 weeks2 Fruit fly 30 days3Dog 10-13 years4 Rose5-7 years5 Tortoise100-150 years6 Banyan Tree -200 - 250 yearsQ4 Reproduction is of two types in case ofanimals but in case of plants vegetative propagation is also present

Asexual Reproduction Sexual Reproductioni) Always uniparentalii) Gametes are not involvediii) Only mitotic division involvediv) Somatic cells of parents are involvedv) Offsprings are genetically similar to the parents

i) Usually biparentalii) Gametes are involvediii) Meiosis occurs during gametogenesis Mitosis occurs after fertilisationiv) Germ cells of the parents are involvedv) offsprings are genetically different from the parents

COMMERCE BUSINESS ENVIRONMENT

Welcome to the new sessiontoday we are going to start the first chapter of Class XII The name of the chapter is Business Environment

Already many of you have got some idea about the word business environment form the first chapter of business studies in class XI

In todayrsquos world every business enterprise is a part of the society It exists and operates in association with various groups in society such as customers suppliers competitors banks and financial institutions government agencies trade unions media and so on All these groups influence the functioning of business in one way or the other They constitute the environment of businessConcept of Business Environment

The term lsquobusiness environmentrsquo refers to the sum total of all individuals institutions and other forces that lie outside a business enterprise but that may influence its functioning and performance

The main features of business environment Totality of External forces General and Specific forces Interrelatedness Complexity Dynamic Uncertainty

Prepare the following questions from todayrsquos assignment

2 What do you mean by business environment

The term lsquobusiness environmentrsquo means the aggregate of all forces factors and institutions which are external to and beyond the control of an individual business enterprise but they may influence its functioning and performance Business environment is the macro framework within which a business firm a micro unit operates It consists of several interrelated and interacting elements

2 Explain the main features of business environment in brief

Totality of External forces-Business environment is the sum total of all things external to a business environment

General and Specific forces-It

Relativity

The Interrelation between business and its environment

The business enterprise is an open system It continuously interacts with its environment It takes inputs (such as raw materials capital labour energy and so on) from its environment transforms them into goods and services and sends them back to the environment

Fig 1 Business Environment Relationship

includes both the forces general forces are the economic social political legal and technological conditions which indirectly influence all business enterprise Specific forces are the investors customers competitors and suppliers which influence individual enterprise directly

Interrelatedness-Different elements of environment are interrelated for an example growing awareness for health care has increased the demand for health foods

Complexity- Business environment id complex in nature as the elements keep on changing example economic technological and other forces changes in demand for a product and service

Dynamic-Business environment is not static it keeps on changing

Uncertainty- Itrsquos very difficult to predict future events such as technology and fashion which occur fast and frequently

Business Studies

Human Resources Management

Human resource of an organisation are the aggregate of knowledge skills attitudes of people working in it

The management system which deals with human resources is called human resource management

Features of HRMbullComprehensive functionbullPeople-oriented

Question1) What do you mean by human

resource management Answer) Human resource management may be defined as that field of Management which has to do with planning organising and controlling the functions of procuring developing maintaining and utilising the labour force

bullAction oriented bullPervasive function bullContinuous function

2) Explain the features of HRM in brief

Answer)bullHuman Resource Management is concerned with managing people at work bull Human Resource Management is concerned with employees which bring people and organisations together so that the goals of each are met bullHuman resource management considered every employees as an individual and also promote their satisfaction and growth bull Human resource management is inherent in all organisations and at all levelsbullManagement of human resources are ongoing on never ending process which requires a constant alertness and Awareness of human relations

3) ldquoHR function is said to be pervasiverdquowhy

Answer) Human resource management is required in all organisations whether it is private or government organisations armed forces sports organisations etc It permeatsall the functional areas like production marketing finance research etc This from this feature of human resource management it can be said that it is pervasive in nature

Economics Demand Q1DEFINITION OF DEMANDIn economics demand is the quantity of a good that consumers are willing and able to purchase at various prices during a given period of timeQ2DEMAND CURVEIn economics a demand curve is a graph depicting the relationship between the price of a certain commodity and the quantity of that commodity that is demanded at that pricQ3LAW OF DEMANDIn microeconomics the law of demand states that conditional on all else being equal as the price of a good increases quantity demanded decreases conversely as the price of a good decreases quantity demanded increasesQ4ASSUMPTION of LAW OF DEMAND(i)No change in price of related commodities(ii) No change in income of the consumer(iii) No change in taste and preferences customs habit and fashion of the consumer( No expectation regarding future change in priceQ5MARKET DEMAND SCHEDULEIn economics a market demand schedule is a tabulation of the quantity of a good that all consumers in a market will purchase at a

given price At any given price the corresponding value on the demand schedule is the sum of all consumersrsquo quantities demanded at that priceQ6INDIVIDUAL DEMAND SCHEDULEIndividual demand schedule refers to a tabular statement showing various quantities of a commodity that a consumer is willing to buy at various levels of price during a given period of timeQ7 FACTORS AFFECTING INDIVIDUAL DEMAND FOR A COMMODITY

The factors that influence a consumerrsquos decision to purchase a commodity are also known as determinants of demand The following factors affect the individual demand for a commodity1 price of the commodity2 price of related goods3 income of buyer of the commodity4 tastes and preferences of the buyer1 Price of the CommodityYou must have observed that when price of a commodity falls you tend to buy more of it and when its price rises you tend to buy less of it when all other factors remain constant (lsquoother things remaining the samersquo) In other words other things remaining the same there is an inverse relationship between the price of a commodity and its quantity demanded by its buyers This statement is in accordance with law of demand which you will study in the later part of this lesson Price of a commodity and its quantity demanded by its buyers are inversely related only when lsquoother things remain the samersquo So lsquoother things remaining the samersquo is an assumption when we study the effect of changes in the price of a commodity on its quantity demanded2 Price of Related goodsA consumer may demand a particular good But while buying that good heshe also asks the price of its related goods Related goods can be of two types-(i) Substitute goods(ii) Complementary goods While purchasing a good prices of its substitutes and complements do affect its quantity purchased(i) Price of Substitute Goods Substitute goods are those goods which can easily be used in place of one another for satisfaction of a particular want like tea and coffee An increase in price of substitute good leads to an increase in demand for the given commodity and a decrease in price of substitute good leads to a decrease in demand for the given commodity It means demand for a given commodity is directly affected by change in price of substitute goods For example if price of coffee increases the demand for tea will rise as tea will become relatively cheaper in comparison to coffee(ii) Price of Complementary goods Complementary goods are those goods which are used together to satisfy a particular want like car and petrol An increase in the price of complementary goods leads to a decrease in demand for the given commodity and a decrease in the price of complementary goods leads to an increase in demand for the given commodity For example if price of petrol falls then the demand for cars will increase as it will be relatively cheaper to use both the goods together So demand for a given commodity is inversely affected by change in price of complementary goods3 Income of the Buyer of CommodityDemand for a commodity is also affected by income of its buyer However the effect of change in income on demand depends on the nature of the commodity under consideration In case of some goods like full cream milk fine quality of rice (Basmati rice) etc demand for these commodities increases when income of the buyer increases and

demand for these commodities decreases when income of the buyer decreases Such goods whose demand increases with the increase in income of the buyer are called normal goods But there are some goods like coarse rice toned milk etc whose demand decreases when income of buyer increases and their demand increases when income of the buyer decreases Such goods whose demand decreases with the increase in income of the buyer are called inferior goods Suppose a consumer buys 10 Kgs of rice whose price is ` 25 per Kg He cannot afford to buy better quality of rice because the price of such rice is ` 50 per Kg The consumer is spending ` 250 per month on the purchase of rice Now if income of the consumer increases and he can afford ` 350 on purchase of 10 Kg of rice Now he can afford to buy some quantity of rice say 6 Kgs whose price is ` 25 per Kg and may buy 4 Kgs of rice whose price is ` 50 per Kg Thus he will buy 10 Kgs of rice by spending ` 350 per month Therefore we may conclude that demand for normal goods is directly related to the income of the buyer but demand for inferior goods is inversely related to the income of the buyer4 Tastes and Preferences of the BuyerThe demand for a commodity is also affected by the tastes and preferences of the buyers They include change in fashion customs habits etc Those commodities are preferred by the consumers which are in fashion So demand for those commodities rises which are in fashion On the other hand if a commodity goes out of the fashion its demand falls because no consumer will like to buy it(5) Number of Buyers in the Market(Population)Increase in population raises the market demand whereas decrease in population reduces the market demand for a commodity Not only the size of population but its composition like age (ratio of males females children and old people in population) also affects the demand for a commodity It is because of needs of children young old male and female population differs(6) Distribution of Income and WealthIf the distribution of income and wealth is more in favour of the rich demand for the commodities preferred by the rich such as comforts and luxuries is likely to be higher On the other hand if the distribution of income and wealth is more in favour of poor demand for commodities preferred by the poor such as necessities will be more(7) Season and Weather ConditionsThis is generally observed that the demand for woolens increases during winter whereas demand for ice creams and cold drinks increases during summer Similarly market demand for umbrellas rain coats increases during rainy seasonQ8 REASONS FOR OPERATION OF LAW OF DEMAND WHY DEMAND CURVE SLOPES DOWNWARDNow we will try to explain why does a consumer purchase more quantity of a commodity at a lower price and less of it at a higher price or why does the law of demand operate ie why does the demand curve slope downwards from left to right The main reasons for operation of law of demand are1 Law of Diminishing Marginal UtilityAs you have studied earlier law of diminishing marginal utility states that as we consume more and more units of a commodity the utility derived from each successive unit goes on decreasing The consumer will be ready to pay more for those units which provide him more utility and less for those which provide him less utility It implies that he will purchase more only when the price of the commodity falls2 Income Effect

When price of a commodity falls purchasing power or real income of the consumer increases which enables him to purchase more quantity of the commodity with the same money income Let us take an example Suppose you buy 4 ice creams when price of each ice cream is ` 25 If price of ice creams falls to ` 20 then with same money income you can buy 5 ice creams now3 Substitution EffectWhen price of a commodity falls it becomes comparatively cheaper as compared to its substitutes (although price of substitutes has not been changed) This will lead to rise in demand for the given commodity For example if coke and Pepsi both are sold at ` 10 each and price of coke falls Now coke has become relatively cheaper and will be substituted for Pepsi It will lead to rise in demand for coke4 Change in Number of BuyersWhen price of a commodity falls some old buyers may demand more of the commodity at the reduced price and some new buyers may also start buying this commodity who were not in a position to buy it earlier due to higher price This will lead to increase in number of buyers when price of the commodity falls As a result demand for the commodity rises when its price falls5 Diverse Uses of a CommoditySome commodities have diverse uses like milk It can be used for drinking for sweet preparation for ice cream preparation etc If price of milk rises its use may be restricted to important purpose only This will lead to reduction in demand for other less important uses When price of milk falls it can be put to other uses also leading to rise n demand for itQ9 EXCEPTIONS TO THE LAW OF DEMANDYou have studied in law of demand that a buyer is willing to buy more quantity of a commodity at a lower price and less of it at a higher price But in certain circumstances a rise in price may lead to rise in demand These circumstances are called Exceptions to the Law of Demand Some important exceptions are1 Giffen GoodsGiffen goods are special type of inferior goods in which negative income effect is stronger than negative substitution effect Giffen goods do not follow law of demand as their demand rises when their price rises Examples of Giffen goods are jowar and bajra etc2 Status Symbol GoodsSome goods are used by rich people as status symbols eg diamonds gold jewellary etc The higher the price the higher will be the demand for these goods When price of such goods falls these goods are no longer looked at as status symbol goods and tehrefore therir demand falls3 NecessitiesCommodities such as medicines salt wheat etc do not follow law of demandbecause we have to purchase them in minimum required quantity whatever their price may be4 Goods Expected to be ScarceWhen the buyers expect a scarcity of a particular good in near future they start buying more and more of that good even if their prices are rising For example during war famines etc people tend to buy more of some goods even at higher prices due to fear of their scarcity in near future

Political Science

Constitution of India-The

Preamble

The preamble-

Preamble-

The preamble is the most precious part of the constitution We the people of India having solemnly resolved to constitute India into a Sovereign Socialist Secular Democratic Republic and to secure to all its citizensA preamble is an introductory and expressionary statement in a document that explains the documents purpose and underlying philosophy When applied to the opening paragraphs of a statute it may recite historical facts pertinent to the subject of the statuteNature and purpose of the constitution-Purpose of the Constitution dictates permanent framework of the government to form a more perfect union to establish justice and ensure peace of thenationconstitution provide principles how the government can run itself following the rules and laws written in the constitution of each state keeps them balanced

Answer the following questions-

1 What is preambleA preamble is an introductory and expressionary statement in a document that explains the documents purpose and underlying philosophy2 What is the nature and

purpose of the constitutionConstitution dictatespermanent framework of the government to form a more perfect union to establish justice and ensure peace of the nation

Homework-Learn

Accounts Compatibilty mode

1MEANING OF PARTNERSHIPPartnership is a form of business organisation where two or more persons join hands to run a business They share the profits and losses according to the agreement amongst them According to the Indian Partnership Act 1932 ldquoPartnership is relation between persons who have agreed to share profits of a business carried on by all or any one of them acting for allrdquo For example one of your friends has passed class XII from National Institute of Open Schooling (NIOS) and wants to start a business Heshe approaches you to join in this venture Heshe wants you to contribute some money and participate in the business activities Both of you if join hands constitute a partnership2CHARACTERISTICS1048698 Agreement A partnership is formed by an agreement The agreement may be either oral or in writing It defines the relationship between the persons who agree to carry on business It may contain the terms of sharing profit and the capital to be invested by each partner etc The written agreement is known as partnership deed1048698 Number of persons There must be at least two persons to form a partnership

The maximum number of partners in a partnership firm can be 50 according toCompanies Act 20131048698 Business The Partnership is formed to carry on business with a purpose of earning profits The business should be lawful Thus if two or more persons agree to carry on unlawful activities it will not be termed as partnership1048698 Sharing Profits The partners agree to share profits in the agreed ratio In caseof loss all the partners have to bear it in the same agreed profit sharing ratio10486981048698Mutual Agency Every partner is an agent of the other partners Every partner can bind the firm and all other partners by hisher acts Each partner will be responsible and liable for the acts of all other partners10486981048698Unlimited liability The liability of each partner except that of a minor is unlimited Their liability extends to their personal assets also If the assets of the firm are insufficient to pay off its debts the partnersrsquo personal property can be used to satisfy the claim of the creditors of the partnership firm10486981048698Management All the partners have a right to mange the business However they may authorize one or more partners to manage the affairs of the business on their behalf10486981048698Transferability of Share No partner can transfer hisher share to any one including hisher family member without the consent of all other partners3PARTNERSHIP DEEDAgreement forms the basis of partnership The written form of the agreement is which a document of partnership is It contains terms and conditions regarding the conduct of the business It also explains relationship between the partners This document is called partnership deed Every firm can frame its own partnership deed in which the rights duties and liabilities of the partners are stated in detail It helps in settling the disputes arising among the partners during the general conduct of business 4CONTENTS OF PARTNERSHIP DEEDThe partnership deed generally contains the following (i) Name and address of the partnership firm(ii) Nature and objectives of the business(iii) Name and address of each partner(iv) Ratio in which profits is to be shared(v) Capital contribution by each partner(vi) Rate of Interest on capital if allowed(vii) Salary or any other remuneration to partners if allowed(viii) Rate of interest on loans and advances by a partner to the firm(ix) Drawings of partners and interest thereon if any(x) Method of valuation of goodwill and revaluation of assets and liabilities on the reconstitution of the partnership ie on the admission retirement or death of a partner(xi) Settlement of disputes by arbitration(xii) Settlement of accounts at the time of retirement or death of a partner5IN ABSENCE OF PARTNERSHIP DEEDThe partnership deed lays down the terms and conditions of partnership in regard to rights duties and obligations of the partners In the absence of partnership deed there may arise a controversy on certain issues like profit sharing ratio interest on

capital interest on drawings interest on loan and salary of the partners In such cases the provisions of the Indian Partnership Act becomes applicableSome of the Issues are(i) Distribution of Profit Partners are entitled to share profits equally(ii) Interest on Capital Interest on capital is not allowed(iii) Interest on Drawings No interest on drawing of the partners is to be charged(iv) Interest on Partnerrsquos Loan A Partner is allowed interest 6 per annum on the amount of loan given to the firm by himher(v) Salary and Commission to Partner A partner is not entitled to anysalary or commission or any other remuneration for managing the business

History TOPIC-TOWARDS INDEPENDENCE AND PARTITION THE LAST PHASE (1935-1947)

SUB TOPIC-IMPORTANT POLITICAL DEVELOPMENTS ndash GROWTH OF SOCIAL IDEAS

Socialism is a political social and economic philosophyLike in other parts of the world the Russian revolution of 1917 served as a great inspiration for revolutionaries in India who at that time were engaged in the struggle for liberation from British ruleSocialist ideas led to the formation of communist party of IndiaJAWAHARLAL NEHRU Among the early Congress leaders Jawaharlal Nehru was very much impressed and influenced by the Socialist ideas He also learnt about the Economic activities of the Soviet Union after the Bolshevic Revolution 1917 He made full use of them in IndiaThe election of Jawaharlal Nehru and Subhas Chandra Bose showed the Left wing tendency within CongressJawaharlal Nehru demanded economic freedom along with political freedom of the people in order to end the exploitation of masses

Nehrus working committee included three socialists leaders The Lucknow session was a landmark in the evolution of socialist ideas of the congressSUBHAS CHANDRA BOSE ndash Subhas Chandra Bose had socialist leaning Both Jawaharlal Nehru and Subhas Chandra Bose were known as leftist Congress men Later on National Congress divided into Leftist and rightist campCONGRESS SOCIALIST Within the Congress some leaders formed the Congress Socialist partyPattavi Sitaramyya Sardar Patel Rajendra Prasad had hostile attitude towards the Congress Socialist partyJawaharlals attitude was hesitant

1 QUESTION ndash Mention name of two Congress leaders who had socialist leaning

1ANSWER ndash Subhas Chandra Bose and Jawaharlal Nehru2QUESTION- In which session of the congress Jawaharlal elaborated his Socialist ideas2 ANSWER ndash Lucknow and Faizpur Session in December 1935 and 19363QUESTION ndash Why Congress was sharply divided into leftist and rightist camp 3ANSWER ndash Subhas Chandra Bosersquos attempt to seek re election for congress presidentship in 1939sharply divided the National Congress into Leftist and Rightist camp4 QUESTION ndash Who was MN Roy 4 ANSWER ndash Manabendra Roy first formed the Communist Party of India outside the country at Tashkent in 19205QUESTION ndash Who formed the Congress Socialist Party within the Congress5 ANSWER ndash Jaya Prakash Narayan Achyut Patwardhan Acharya Narendra Dev Ram Mohan Lohia Aruna Asaf Ali6QUESTION ndash When was the Congress Socialist Party formed What was its object6 ANSWER ndash 1934The Congress Socialist Party sought to work out socialist programme through the Congress They joined hands with the Congress and wanted to carry

Subhas Chandra Bose being expelled from the congress after the Tripuri rift he formed Forward BlockThere were basic differences between the Congress Socialists and the communistsTRADE UNION ACTIVITIES Maximum working class people lived in Bombay and Calcutta The working and living conditions of those workers were very miserable In this situation Shasipada Banerjee NM Lokhande protested against the oppression of the working class peopleThe first Trade Union Madras Labour Union was formed in 1918 by BP WadiaIndustrial strikes took place in Kanpur Calcutta Madras Jamshedpur and Ahmedabad AITUC was formed in Bombay in 1927 The growth of Trade union among the workers was slow because of the fear of the dismissal of the jobIn the mean time the Moderates as well as Communists left AITUC and formed separate organization

on National struggle with the help of workers and peasant class of the society7 QUESTION ndash What was the name of the party founded by Subhas Chandra Bose7 ANSWER- Forward Block8QUESTION ndash Who was Shasipada Banerjee8 ANSWER ndash Shasipada Banerjee was a radical Brahmo He founded a working menrsquos club to protest against exploitation of the British rulers towards the working class of India9 QUESTION ndash What was the weekly published by NM Lokhande9ANSWER- Dinabandhu10 QUESTION ndash Who founded Bombay Mill-Hands Association and in which year10 ANSWER- NM Lokhande in189011 QUESTION- Who was BP WadiaANSWER- BPWadia was the founder of Madras Labour Union in191812 QUESTION- What was the name of the first labour union of India12 ANSWER- Madras Labour Union13 QUESTION Who founded the Majur Mahajan 13 ANSWER GANDHIJI14 QUESTION What was the full form of AITUC When it was formed14 ANSWER All India Trade Union Congressin 192715QUESTION Who formed the Red Trade Union Congress and in which year15ANSWER The Communists formed the Red Trade Union Congress16 QUESTION What do you mean by Socialism16 ANSWER Socialism describes any political and economic theory that says the community rather than individuals should own and manage property and natural resources

Subject Eng Literature (The Tempest ndash William Shakespeare) Topic Act III Scene 3 Lines 1 to 52 (Line 52 ndash Brother my lord the Duke Stand to and do as we) Date 13th April 2020 (4th Period)

[Students should read the original play and also the paraphrase given in the school prescribed textbook]Summary Questions amp Answers

o Alonso Sebastian Antonio Gonzalo Adrian Francisco and others wandered about the island in search of Ferdinand and gets tired and hungry of the toil and at the same time gives up all hope of finding him

o Antonio and Sebastian are happy that Alonso is out of hope and decide to make another attempt on his life that night when being so tired they will be sleeping soundly

o Suddenly a solemn and strange music is heard in the air and several strange shapes enter bringing in a banquet These strange shapes then dance round it with gestures of salutation and then inviting the King to eat they depart

o Seeing this strange scene all are inclined to believe the tales told by travelers that there truly are ldquounicornsrdquo and ldquothe phoenixrsquo thronerdquo

1 ALONSO What harmony is this My good friends hark (L18-27)

GONZALO Marvellous sweet music

[Enter several strange shapes bringing in a banquet

they dance about it with gentle actions of salutation

and inviting the King and his companions to eat they depart]ALONSO Give us kind keepers heavens What were theseSEBASTIAN A living drollery Now I will believe

That there are unicorns that in Arabia

There is one tree the phoenixrsquo throne one phoenix

At this hour reigning thereANTONIO Ill believe both

And what does else want credit come to me

And Ill be sworn rsquotis true Travellers neer did lie

Though fools at home condemn rsquoem

(a) How did Prospero present an amazing spectacle before Alonso and his companions

Using his magic powers Prospero ordered strange shapes to lay a banquet before Alonso and his companions The shapes brought several dishes with tasty eatables in them They placed the dishes on a table before Alonso and his companions Then the strange shapes began to dance gracefully around the banquet While dancing they made gestures inviting them to eat the food Then suddenly the shapes disappeared(b) Who were the guests at the strange banquet Describe the lsquoliving drolleryrsquo

Alonso Sebastian Antonio Gonzalo Adrian and Francisco were the guests at the strange banquet

The term ldquoliving drolleryrdquo refers to live entertainment show In this context when Alonso the King of Naples Sebastian his brother Antonio the treacherous brother of Prospero Gonzalo the kind and loyal councillor to the King Adrian and Francisco came to the island they were hungry and weary in their spirits They heard a solemn and strange music They were shocked to see several strange shapes bringing in a banquet and these shapes danced about it with gentle action of salutation inviting the King and his companions to eat After this Sebastian described this show as lsquoliving drolleryrsquo(c) What is lsquophoenixrsquo What are lsquoUnicornsrdquo

The term lsquophoenixrsquo refers to a mythical Arabian bird which lived alone and perched on a solitary tree After one hundred years it expired in flames and rose again from its own ashes

lsquoUnicornsrsquo refers to the mythological four-footed beasts having horns in the centre of their foreheads When the horns are ground into powder the powder was believed to be

an aphrodisiac(d) How does Sebastian explain the puppet show OR Why does the speaker now believe in unicorns and phoenix

Sebastian finds several strange shapes bringing in the banquet They invite the king and his party for dinner and soon depart He tells that if such a strange sight can be a reality there is nothing incredible in the world and from the present moment he will believe anything He says that it is a strange dumb show enacted not by puppets but by living beings It is stranger than a travellerrsquos tale Seeing such a thing

before his own eyes he will no longer disbelieve the story about unicorns and phoenix(e) How do the other characters present respond to this living drollery

At the sight of the lsquoliving drolleryrsquo like Sebastian Gonzalo and Antonio too acted strangely Antonio told that he too now believes in unicorns and phoenix and anything else that seems to be incredible He too now believes in travellersrsquo tales Gonzalo told that if he would report those happenings in Naples nobody will believe him He considers that those gentle shapes were gentler in manner in comparison to the living beings Alonso was at first sight suspicious and told them that those strange shapes conveyed their meaning in expressive gestures when they seemed to lack speech by their movements and sounds Francisco was amazed at their mysterious disappearance

2 ALONSO Not I

(Line 43-52)GONZALO Faith sir you need not fear When we

were boysWho would believe that there were mountaineers

Dewlapped like bulls whose throats had hanging at rsquoem

Wallets of flesh Or that there were such men

Whose heads stood in their breasts Which now we find

Each putter-out of five for one will bring us

Good warrant ofALONSO I will stand to and feed

Although my lastmdashno matter since I feel

The best is past Brother my lord the Duke

Stand to and do as we

(a) How does Alonso respond at the spectacle of the shapes which were sent to them at the instruction of Prospero

After seeing the strange sight of appearing and disappearing of the shapes sent by Prospero to arrange a banquet for them Alonso says that his surprise at having seen those creatures is infinite and he is fully justified in feeling so much surprise He thinks that their shapes their gestures and the sounds they made were indeed amazing Although they do not possess the gift of speech yet they were able to convey their

thoughts by means of their gestures only

(b) What does Prospero say about the views expressed by Alonso regarding the shapes What does Francisco think about the shapesAfter hearing Alonsorsquos views about the shapes Prospero says that this manrsquos praise of the spirits is rather hasty He means to say that Alonso has shown great haste in reaching the conclusion about the shapes Francisco is amazed to see that those shapes disappeared in a mysterious way(c) What does Sebastian ask Alonso to doSebastian tells Alonso that the shapes having disappeared should not matter to them because they have left the eatables behind He asks Alonso to enjoy eating as they are extremely hungry but the king does not accept his offer of enjoying the dishes(d) How does Gonzalo try to dispel Alonsorsquos fear of those strange shapes What kind of references does he give to AlonsoGonzalo says that those who have travelled abroad have reported seeing even stranger sights than these shapes that Alonso and his companions have beheld Hence there is no reason to feel afraid of these shapes Gonzalo further adds that in his younger days he had heard strange stories from travelers and Alonso might have heard similar stories For instance it was said that there existed a certain race of

human beings who had huge lumps of flesh hanging at their throats and who therefore resembled bulls Then Gonzalo tells about a race of human beings whose heads were located at their breasts Gonzalo says that such stories were not believed by most people in those days but now-a-days these stories have become common(e) Explain the following lsquoEach putter-out of five for onersquoEnglish travellers often insured their trips with London brokers Those that went on foreign travels those days used to deposit a certain amount with some firm or company in London before their departure If the travelers failed to return the money was forfeited by the company with which it had been deposited But this money was repaid five-fold if the travelers returned safe and sound In this way a traveler stood a great chance of recovering the entire cost of his

travels(f) Give the explanatory meanings of the following expressions in the context of the above extract (i) Dewlapped (ii) Wallets of flesh

(iii) Putter-out(i) Dewlapped having big lumps of flesh at the necks(ii) Wallets of flesh large masses of flesh looking like bags(iii) Putter-out to invest money before commencing the travel

  • General methods of preparation of hydrogen
  • Chapter Dimensional Analysis (Summary)
    • Properties of Charge
Page 41:   · Web viewSubject. Topic. Summary. Execution. Hindi. व्याकरण. शरीरके अंगो के नाम लिखिए. 1) आँख 2) नाक 3

traditional and modern theories of politicsTraditionalClassical political sciencepolitical theory-Traditional political science is the study of politics before Second World War The methodology to study Politics was traditional (legal formaletc) the definition of politics traditional (Politics begins and end with state)area of study (constitution state machinery)was traditionalModern Political scienceModern political theory-Modern Political Theory critically examines the contemporary state of political theory making an assessment of the achievement and limitations of the Behavioural Revolution in its totality and reviews objectively the major paradigms and conceptual frameworks adopted by the disciplineContemporary attempts at the development of an integrated political theory involving the use of both traditional and modern concepts approaches and theories-Around late 1960s several political scientists realized the importance of both the traditional political theory and modern Political theory They began building an integrated theory of politics involving a systematic mixture of traditional and modern studies of politics It was held that the study of a complex and vast field like politics needs both traditional as well as

Classical political theory Modern Political theory

Homework-Learn

modern concepts and approaches for studying itrsquos all aspects

Subject Eng Literature (The Tempest ndash William Shakespeare) Topic Act I Scene 1 Lines 1 to 32 (Line 32 ndash Gonzalo hellip If he be not born to be hanged our case is miserable) Date 13th April 2020 (3rd Period)

[Students should read the original play and also the paraphrase given in the school prescribed textbook]Summary Questions amp Answers

[SUMMARY OF THE ENTIRE SCENE]

o The play starts with the scene of a severe storm at sea Alonso (King of Naples) Sebastian (Alonsorsquos brother) Ferdinand (Alonsorsquos son) Gonzalo Antonio (the usurping Duke of Milan) are in a ship in the midst of the storm

o The mariners are trying their best to control the vessel from running aground and are totally following the orders of their Master the Boatswain They have scant success

o The mariners become extremely unhappy and annoyed when most of the passengers arrive on the deck thereby hampering their effort to save the ship There is serious confrontation between them and the passengers who are part of the Kingrsquos entourage

o The mariners could not save the ship

SUMMING-UP

(i) Vivid description of the scene which gives a realistic description of terror and confusion of a tropical storm

(ii) Shows Shakespearersquos accuracy of knowledge in describing the naval operations and also matters of seamanship

(iii) The opening scene justifies the title ndash The Tempest

UNANSWERED QUESTIONS

(i) The King always travels with his entire fleet including his soldiers Where

(1) GONZALO Nay good be patient (Line 15-26)BOATSWAIN When the sea is Hence What cares these

roarers for the name of the king To cabin silence Trouble us not

GONZALO Good yet remember whom thou has aboardBOATSWAIN None that I more love than myself You are a

councillor if you can command these elements to silence and work

the peace of the present we will not hand a rope more use your authority If you cannot give thanks you have

lived so long and make yourself ready in your cabin for the mischance of the hour if it so hap [To the Mariners]

Cheerly good hearts [To Gonzalo] Out of our way I say

(a) To whom is the boatswain speaking What does he mean by lsquoNone that I more love than myselfrsquo

The Boatswain is speaking to Gonzalo the honest old councilor of the Duke of MilanBy using the words ndash lsquoNone that I love more than I love myselfrsquo means that for the Boatswain nobody is dearer to him than his own life

(b) What were the conditions that made the boatswain react in this way

The Boatswain reacts in this way because the storm is at sea and Alonso King of Naples Sebastian his brother Ferdinand his son Gonzalo Antonio the usurping Duke of Milan on board are in distress and in panic Thus they have rushed to the deck interrupting the work of the mariners

(c) What hope does Gonzalo take from the attitude of the boatswain

The insolent and authoritative attitude of Boatswain makes Gonzalo feel comforted He tells that there are no signs that the Boatswain will be drowned But his facial appearance and attitude shows that he is destined to die on land by hanging which in effect means that all on board will be saved Otherwise all the persons on board are doomed

(d) How can they lsquomake yourself ready in your cabinrsquo For what were they asked to make ready themselves

In order to make themselves ready in their cabin the

were the other ships

(ii) Why was the ship in that area Where was it coming from or going where

(iii) The ship broke apart What happened to those who were in the ship

passengers on board must prepare for death which they will possibly soon have to meetThey can retire to their cabins and offer prayers to the Almighty to save them from drowning

(e) What does the boatswain say when he is asked to be patient What does he order to the royal party

When the boatswain is asked to be patient and remain calm he says that he will be patient only when the storm will be over and the sea will be calm but as long as the storm blows and there is danger to the ship he cannot think of being patient He orders the royal party to go to the cabin and leave the mariners to their work

(2) GONZALO I have great comfort from this fellow (Line 27-36)

Methinks he hath no drowning mark upon him his complexion is perfect

gallows Stand fast good Fate to his hanging Make the rope of his destiny our cable for our own doth little advantage If he be not born to be hanged our case is miserable

(a) Why does Gonzalo regard the Boatswain in the midst of danger

In the midst of danger Gonzalo regards the boatswain because he feels that the Boatswain is a source of comfort and is bent upon to do his work sincerely which in this case is saving the ship and its passengers from the severest of raging storm

(b) What reasons does Gonzalo give when he says that none in the ship will die of drowning

Gonzalo is almost sure that none in the ship will die by drowning His says that there is no mark on the face of the boatswain that indicates that he will die by drowning On the other hand the lines on his face are strong indications that he will be hanged to death Therefore there shall be no danger of the shiprsquos sinking

(c) Explain the following ldquoStand fast good Fate to his hanging Make the rope of his destiny our cable for our own doth little advantage If he be not born to be hanged our case is miserablerdquo

The stated lines mean that if the will of destiny is to be carried out then the ship will not get wrecked and all the passengers will be saved The safety of the passengers therefore depends upon the will of fate being carried out in the case of the boatswain If however the boatswain is not to die by hanging then the passengers are also very unsafe because in that case the ship is likely to sink

(d) What order does the Boatswain give to the sailors

when he re-enters What does he say about the crying of the fellows inside the cabin

The boatswain orders the sailors to bring the topmast lower and bring the ship close to a stationary position with the help of the main sail He says that the fellows inside the cabin are moaning and crying in their distress louder than his voice and louder even than the roaring of the storm

Class XII (ScienceCommerceHumanities) Subject Topic Summary Execution

Computer Science

PropositionalLogic

Propositional logic is a procedure to provide reasoning through statementProposition A ststement that results in True or False is said to be proposition There are two types of propositionSimple proposition amp compound propositionSimple proposioton A simple proposition is one that is not a part of any other proposition Such sentential form of proposition is symbolized with english letters in short For example Ram is a claver student (TrueFalse)Where do you live (Not in True or False)Grapes are sweet (TrueFalse)It rains today (TrueFalse)Here we can see some statements anwer would be true or false but some staements answer can not give in terms of true or false Thus the sentences which can be answered in true or false are known as simple propositionAssigning propositon to a variableThe general syntax to assign propostion to a variable is as followsVariable = Simple propositonFor example A=Ram is a clever studentB= Grapes are sweetC= it rains todayCompound proposition

helliphellipto be continued in next classhelliphellipMath Relation Relation If A and B are two non-empty sets

then a relation R from A to B is a subset of AxB If R A x B and (a b) R then we say that a sube isinis related to b by the relation R written as aRbeg Let A be the set of students of class XII and B be the set of students of class XI Then some of the examples of relation from A to B arei) (a b) AXB a is brother of bisinii) (a b) AXB age of a is more than age of isinb Types of relation In this section we would like to study different types of relations We know that a relation in a set A is a subset of A times A Thus the empty set φ and A times A are two extreme relations For illustration consider a relation R in the set A = 1 2 3 4 given by R = (a b) a ndash b = 10 This is the empty set as no pair (a b) satisfies the condition a ndash b = 10 Similarly R = (a b) | a ndash b | ge 0 is the whole primeset A times A as all pairs (a b) in A times A satisfy | a ndash

Example 1 Let A be the set of all students of a boys school Show that the relation R in A given by R = (a b) a is sister of b is the empty relation and R = (a b) the primedifference between heights of a and b is less than 3 meters is the universal relationSolution Since the school is boys school no student of the school can be sister of any student of the school Hence R = φ showing that R is the empty relation It is also obvious that the difference between heights of any two students of the school has to be less than 3 meters This shows that R = A times A is primethe universal relation Example 2 Show that the relation R in the set 1 2 3 given by R = (1 1) (2 2) (3 3) (1 2) (2 3) is reflexive

b | ge 0 These two extreme examples lead us to the following definitionsDefinition 1 A relation R in a set A is called empty relation if no element of A isrelated to any element of A ie R = φ A times AsubDefinition 2 A relation R in a set A is called universal relation if each element of A is related to every element of A ie R = A times A Both the empty relation and the universal relation are some times called trivial relation Definition 3 A relation R in a set A is called(i) reflexive if (a a) R for every a Aisin isin(ii) symmetric if (a1 a2) R implies that (aisin 2a1)

R for all aisin 1 a2 Aisin(iii) transitive if (a1 a2) R and (aisin 2 a3) R isinimplies that (a1 a3) R for all aisin 1 a2 a3 AisinDefinition 4 A relation R in a set A is said to be an equivalence relation if R is reflexive symmetric and transitive

but neither symmetric nor transitiveSolution R is reflexive since (1 1) (2 2) and (3 3) lie in R Also R is not symmetric as (1 2) R but (2 1) isin notinR Similarly R is not transitive as (1 2) R and (2 3) R but (1 3) R isin isin notinExample 3 Show that the relation R in the set Z of integers given byR = (a b) 2 divides a ndash b is an equivalence relationSolution R is reflexive as 2 divides (a ndash a) for all a Z isinFurther if (a b) R then 2 divides a isinndash b Therefore 2 divides b ndash a Hence (b a) R which shows that R is isinsymmetric Similarly if (a b) R and (b c) R isin isinthen a ndash b and b ndash c are divisible by 2 Now a ndash c = (a ndash b) + (b ndash c) is even (Why) So (a ndash c) is divisible by 2 This shows that R is transitive Thus R is an equivalence relation in ZExample 4 Let L be the set of all lines in a plane and R be the relation in L defined as R = (L1 L2) L1 is perpendicular to L2 Show that R is symmetric but neither reflexive nor transitiveSolution R is not reflexive as a line L1 can not be perpendicular to itself ie (L1 L1) R notinR is symmetric as (L1 L2) Risin

L1 is perpendicular to L2rArr L2 is perpendicular to L1rArr (L2 L1) RrArr isin

R is not transitive Indeed if L1 is perpendicular to L2 and L2 is perpendicular to L3 then L1 can never be perpendicular to L3 In fact L1 is parallel to L3 ie (L1 L2) R isin(L2 L3) R but (L1 L3) Risin notin

Chemistry Solid state Characteristics if Solids(i)The particles are locked in fixed positions they are unable to change their relative positions and this brings a definite shape and volume of a solid(ii)In a solid the constituent particles are held by strong forces of attractionThe forces of attraction may be bonding or non bonding(iii)The constituent particles in a solid pack together as closely as possibleoccupying most of the available space within the solidThus the empty space in a solid is very smallThis makes a solid highly rigid and nearly incompressibleThis also explains why a solid has high density and exhibits slow diffusionClassification of Solids

Q1)Define Crystalline solids AnsA Solid that has a definite geometrical shape and a sharp melting pointand whose constituent particles (atomsmolecules or ions) are arranged in a long range order of definite pattern extending throughout the solid is called a crystalline solidExNaClQ2)Define Amorphous solids AnsA solid that does not have a definite shape and a sharp melting pointand whose constituent particles (atomsmolecules or ions) are not arranged in a definite pattern is called an amorphoussolid

Crystalline solidsAmorphous solids

ExGlassRubberQ3)Classify Crystalline Solids Crystalline Solids

Physics Coloumbrsquos Law (Summary)

Before Going Into Coloumbrsquos Law We Will First Learn What is Charge Properties of Charge and Always remember that charge is quantized ie a body always have static charge of magnitude equal to some integral multiple of fundamental electronic charge e= 16 x 10- 19 C

Charge is the property of matter that causes it to produce and experience electrical and magnetic effects The study of the electrical charges at rest is called electrostatics When both electrical and magnetic effects are present the interaction between charges is referred to as electromagnetic

There exist two types of charges in nature positive and negative Like charges repel and unlike charges attract each other

The type of charge on an electron is negative The charge of a proton is the same as that of an electron but with a positive sign In an atom the number of electrons and the number of protons are equal The atom is therefore electrically neutral If one or more electrons are added to it it becomes negatively charged and is designated as negative ion However if one or more electrons are removed from an atom it becomes positively charged and is called a positive ion

The excess or deficiency of electrons in a body gives the concept of charge If there is an excess of electrons in a body it is negatively charged And if there is deficiency of electrons the body becomes positively charged Whenever addition or removal of electrons takes places the body acquires a charge

The SI Unit of charge is coulomb (C) In SI units the current is a fundamental quantity having a unit of ampere (A) The unit of charge is defined in terms of the unit of current Thus one coulomb is the charge transferred in one second across the section of a wire carrying a

Ionic SolidsMetallicSolids

Molecular Solids

current of one ampere

As q = It we have1 C = (1 A) (1 s)

The dimensions of charge are [A T]

Properties of Charge

(1) Quantization of Charge Electric charge can have only discrete values rather than any value That is charge is quantized The smallest discrete value of charge that can exist in nature is the charge on an electron given as

e = plusmn 16 x 10- 19 C

This is the charge attained by an electron and a protonA charge q must be an integral multiple of this basic unit That is

Q = plusmn ne where n = 1 2 hellip

Charge on a body can never be (frac12)e (23)e or 57e etcWhen we rub a glass rod with silk some electrons are transferred from the rod to the silk The rod becomes positively charged The silk becomes negatively charged The coulomb is a very large amount of charge A typical charge acquired by a rubbed body is 10 - 8 C

Biology Reproduction in organisms

Welcome to this new session 2020-21Today in this first chapter we mainly discuss about reproduction types needs and life span of some organismsWe also discuss about difference between sexual and asexual reproduction

Q1 What is reproductionReproduction is defined as a biological processin which an organism gives rise to young onessimilar to itselfQ2 What are the needs of reproductionbulli) Reproduction maintain life on earthii) It enables the continuity of the species generation after generationiii) It creates genetic variation among populationsQ3 Define Life span and write some orgnisms life spanbull Life span is the period from birth to

the natural death of an organism- OrganismsLife span1 Butterfly 1 - 2 weeks2 Fruit fly 30 days3Dog 10-13 years4 Rose5-7 years5 Tortoise100-150 years6 Banyan Tree -200 - 250 yearsQ4 Reproduction is of two types in case ofanimals but in case of plants vegetative propagation is also present

Asexual Reproduction Sexual Reproductioni) Always uniparentalii) Gametes are not involvediii) Only mitotic division involvediv) Somatic cells of parents are involvedv) Offsprings are genetically similar to the parents

i) Usually biparentalii) Gametes are involvediii) Meiosis occurs during gametogenesis Mitosis occurs after fertilisationiv) Germ cells of the parents are involvedv) offsprings are genetically different from the parents

COMMERCE BUSINESS ENVIRONMENT

Welcome to the new sessiontoday we are going to start the first chapter of Class XII The name of the chapter is Business Environment

Already many of you have got some idea about the word business environment form the first chapter of business studies in class XI

In todayrsquos world every business enterprise is a part of the society It exists and operates in association with various groups in society such as customers suppliers competitors banks and financial institutions government agencies trade unions media and so on All these groups influence the functioning of business in one way or the other They constitute the environment of businessConcept of Business Environment

The term lsquobusiness environmentrsquo refers to the sum total of all individuals institutions and other forces that lie outside a business enterprise but that may influence its functioning and performance

The main features of business environment Totality of External forces General and Specific forces Interrelatedness Complexity Dynamic Uncertainty

Prepare the following questions from todayrsquos assignment

2 What do you mean by business environment

The term lsquobusiness environmentrsquo means the aggregate of all forces factors and institutions which are external to and beyond the control of an individual business enterprise but they may influence its functioning and performance Business environment is the macro framework within which a business firm a micro unit operates It consists of several interrelated and interacting elements

2 Explain the main features of business environment in brief

Totality of External forces-Business environment is the sum total of all things external to a business environment

General and Specific forces-It

Relativity

The Interrelation between business and its environment

The business enterprise is an open system It continuously interacts with its environment It takes inputs (such as raw materials capital labour energy and so on) from its environment transforms them into goods and services and sends them back to the environment

Fig 1 Business Environment Relationship

includes both the forces general forces are the economic social political legal and technological conditions which indirectly influence all business enterprise Specific forces are the investors customers competitors and suppliers which influence individual enterprise directly

Interrelatedness-Different elements of environment are interrelated for an example growing awareness for health care has increased the demand for health foods

Complexity- Business environment id complex in nature as the elements keep on changing example economic technological and other forces changes in demand for a product and service

Dynamic-Business environment is not static it keeps on changing

Uncertainty- Itrsquos very difficult to predict future events such as technology and fashion which occur fast and frequently

Business Studies

Human Resources Management

Human resource of an organisation are the aggregate of knowledge skills attitudes of people working in it

The management system which deals with human resources is called human resource management

Features of HRMbullComprehensive functionbullPeople-oriented

Question1) What do you mean by human

resource management Answer) Human resource management may be defined as that field of Management which has to do with planning organising and controlling the functions of procuring developing maintaining and utilising the labour force

bullAction oriented bullPervasive function bullContinuous function

2) Explain the features of HRM in brief

Answer)bullHuman Resource Management is concerned with managing people at work bull Human Resource Management is concerned with employees which bring people and organisations together so that the goals of each are met bullHuman resource management considered every employees as an individual and also promote their satisfaction and growth bull Human resource management is inherent in all organisations and at all levelsbullManagement of human resources are ongoing on never ending process which requires a constant alertness and Awareness of human relations

3) ldquoHR function is said to be pervasiverdquowhy

Answer) Human resource management is required in all organisations whether it is private or government organisations armed forces sports organisations etc It permeatsall the functional areas like production marketing finance research etc This from this feature of human resource management it can be said that it is pervasive in nature

Economics Demand Q1DEFINITION OF DEMANDIn economics demand is the quantity of a good that consumers are willing and able to purchase at various prices during a given period of timeQ2DEMAND CURVEIn economics a demand curve is a graph depicting the relationship between the price of a certain commodity and the quantity of that commodity that is demanded at that pricQ3LAW OF DEMANDIn microeconomics the law of demand states that conditional on all else being equal as the price of a good increases quantity demanded decreases conversely as the price of a good decreases quantity demanded increasesQ4ASSUMPTION of LAW OF DEMAND(i)No change in price of related commodities(ii) No change in income of the consumer(iii) No change in taste and preferences customs habit and fashion of the consumer( No expectation regarding future change in priceQ5MARKET DEMAND SCHEDULEIn economics a market demand schedule is a tabulation of the quantity of a good that all consumers in a market will purchase at a

given price At any given price the corresponding value on the demand schedule is the sum of all consumersrsquo quantities demanded at that priceQ6INDIVIDUAL DEMAND SCHEDULEIndividual demand schedule refers to a tabular statement showing various quantities of a commodity that a consumer is willing to buy at various levels of price during a given period of timeQ7 FACTORS AFFECTING INDIVIDUAL DEMAND FOR A COMMODITY

The factors that influence a consumerrsquos decision to purchase a commodity are also known as determinants of demand The following factors affect the individual demand for a commodity1 price of the commodity2 price of related goods3 income of buyer of the commodity4 tastes and preferences of the buyer1 Price of the CommodityYou must have observed that when price of a commodity falls you tend to buy more of it and when its price rises you tend to buy less of it when all other factors remain constant (lsquoother things remaining the samersquo) In other words other things remaining the same there is an inverse relationship between the price of a commodity and its quantity demanded by its buyers This statement is in accordance with law of demand which you will study in the later part of this lesson Price of a commodity and its quantity demanded by its buyers are inversely related only when lsquoother things remain the samersquo So lsquoother things remaining the samersquo is an assumption when we study the effect of changes in the price of a commodity on its quantity demanded2 Price of Related goodsA consumer may demand a particular good But while buying that good heshe also asks the price of its related goods Related goods can be of two types-(i) Substitute goods(ii) Complementary goods While purchasing a good prices of its substitutes and complements do affect its quantity purchased(i) Price of Substitute Goods Substitute goods are those goods which can easily be used in place of one another for satisfaction of a particular want like tea and coffee An increase in price of substitute good leads to an increase in demand for the given commodity and a decrease in price of substitute good leads to a decrease in demand for the given commodity It means demand for a given commodity is directly affected by change in price of substitute goods For example if price of coffee increases the demand for tea will rise as tea will become relatively cheaper in comparison to coffee(ii) Price of Complementary goods Complementary goods are those goods which are used together to satisfy a particular want like car and petrol An increase in the price of complementary goods leads to a decrease in demand for the given commodity and a decrease in the price of complementary goods leads to an increase in demand for the given commodity For example if price of petrol falls then the demand for cars will increase as it will be relatively cheaper to use both the goods together So demand for a given commodity is inversely affected by change in price of complementary goods3 Income of the Buyer of CommodityDemand for a commodity is also affected by income of its buyer However the effect of change in income on demand depends on the nature of the commodity under consideration In case of some goods like full cream milk fine quality of rice (Basmati rice) etc demand for these commodities increases when income of the buyer increases and

demand for these commodities decreases when income of the buyer decreases Such goods whose demand increases with the increase in income of the buyer are called normal goods But there are some goods like coarse rice toned milk etc whose demand decreases when income of buyer increases and their demand increases when income of the buyer decreases Such goods whose demand decreases with the increase in income of the buyer are called inferior goods Suppose a consumer buys 10 Kgs of rice whose price is ` 25 per Kg He cannot afford to buy better quality of rice because the price of such rice is ` 50 per Kg The consumer is spending ` 250 per month on the purchase of rice Now if income of the consumer increases and he can afford ` 350 on purchase of 10 Kg of rice Now he can afford to buy some quantity of rice say 6 Kgs whose price is ` 25 per Kg and may buy 4 Kgs of rice whose price is ` 50 per Kg Thus he will buy 10 Kgs of rice by spending ` 350 per month Therefore we may conclude that demand for normal goods is directly related to the income of the buyer but demand for inferior goods is inversely related to the income of the buyer4 Tastes and Preferences of the BuyerThe demand for a commodity is also affected by the tastes and preferences of the buyers They include change in fashion customs habits etc Those commodities are preferred by the consumers which are in fashion So demand for those commodities rises which are in fashion On the other hand if a commodity goes out of the fashion its demand falls because no consumer will like to buy it(5) Number of Buyers in the Market(Population)Increase in population raises the market demand whereas decrease in population reduces the market demand for a commodity Not only the size of population but its composition like age (ratio of males females children and old people in population) also affects the demand for a commodity It is because of needs of children young old male and female population differs(6) Distribution of Income and WealthIf the distribution of income and wealth is more in favour of the rich demand for the commodities preferred by the rich such as comforts and luxuries is likely to be higher On the other hand if the distribution of income and wealth is more in favour of poor demand for commodities preferred by the poor such as necessities will be more(7) Season and Weather ConditionsThis is generally observed that the demand for woolens increases during winter whereas demand for ice creams and cold drinks increases during summer Similarly market demand for umbrellas rain coats increases during rainy seasonQ8 REASONS FOR OPERATION OF LAW OF DEMAND WHY DEMAND CURVE SLOPES DOWNWARDNow we will try to explain why does a consumer purchase more quantity of a commodity at a lower price and less of it at a higher price or why does the law of demand operate ie why does the demand curve slope downwards from left to right The main reasons for operation of law of demand are1 Law of Diminishing Marginal UtilityAs you have studied earlier law of diminishing marginal utility states that as we consume more and more units of a commodity the utility derived from each successive unit goes on decreasing The consumer will be ready to pay more for those units which provide him more utility and less for those which provide him less utility It implies that he will purchase more only when the price of the commodity falls2 Income Effect

When price of a commodity falls purchasing power or real income of the consumer increases which enables him to purchase more quantity of the commodity with the same money income Let us take an example Suppose you buy 4 ice creams when price of each ice cream is ` 25 If price of ice creams falls to ` 20 then with same money income you can buy 5 ice creams now3 Substitution EffectWhen price of a commodity falls it becomes comparatively cheaper as compared to its substitutes (although price of substitutes has not been changed) This will lead to rise in demand for the given commodity For example if coke and Pepsi both are sold at ` 10 each and price of coke falls Now coke has become relatively cheaper and will be substituted for Pepsi It will lead to rise in demand for coke4 Change in Number of BuyersWhen price of a commodity falls some old buyers may demand more of the commodity at the reduced price and some new buyers may also start buying this commodity who were not in a position to buy it earlier due to higher price This will lead to increase in number of buyers when price of the commodity falls As a result demand for the commodity rises when its price falls5 Diverse Uses of a CommoditySome commodities have diverse uses like milk It can be used for drinking for sweet preparation for ice cream preparation etc If price of milk rises its use may be restricted to important purpose only This will lead to reduction in demand for other less important uses When price of milk falls it can be put to other uses also leading to rise n demand for itQ9 EXCEPTIONS TO THE LAW OF DEMANDYou have studied in law of demand that a buyer is willing to buy more quantity of a commodity at a lower price and less of it at a higher price But in certain circumstances a rise in price may lead to rise in demand These circumstances are called Exceptions to the Law of Demand Some important exceptions are1 Giffen GoodsGiffen goods are special type of inferior goods in which negative income effect is stronger than negative substitution effect Giffen goods do not follow law of demand as their demand rises when their price rises Examples of Giffen goods are jowar and bajra etc2 Status Symbol GoodsSome goods are used by rich people as status symbols eg diamonds gold jewellary etc The higher the price the higher will be the demand for these goods When price of such goods falls these goods are no longer looked at as status symbol goods and tehrefore therir demand falls3 NecessitiesCommodities such as medicines salt wheat etc do not follow law of demandbecause we have to purchase them in minimum required quantity whatever their price may be4 Goods Expected to be ScarceWhen the buyers expect a scarcity of a particular good in near future they start buying more and more of that good even if their prices are rising For example during war famines etc people tend to buy more of some goods even at higher prices due to fear of their scarcity in near future

Political Science

Constitution of India-The

Preamble

The preamble-

Preamble-

The preamble is the most precious part of the constitution We the people of India having solemnly resolved to constitute India into a Sovereign Socialist Secular Democratic Republic and to secure to all its citizensA preamble is an introductory and expressionary statement in a document that explains the documents purpose and underlying philosophy When applied to the opening paragraphs of a statute it may recite historical facts pertinent to the subject of the statuteNature and purpose of the constitution-Purpose of the Constitution dictates permanent framework of the government to form a more perfect union to establish justice and ensure peace of thenationconstitution provide principles how the government can run itself following the rules and laws written in the constitution of each state keeps them balanced

Answer the following questions-

1 What is preambleA preamble is an introductory and expressionary statement in a document that explains the documents purpose and underlying philosophy2 What is the nature and

purpose of the constitutionConstitution dictatespermanent framework of the government to form a more perfect union to establish justice and ensure peace of the nation

Homework-Learn

Accounts Compatibilty mode

1MEANING OF PARTNERSHIPPartnership is a form of business organisation where two or more persons join hands to run a business They share the profits and losses according to the agreement amongst them According to the Indian Partnership Act 1932 ldquoPartnership is relation between persons who have agreed to share profits of a business carried on by all or any one of them acting for allrdquo For example one of your friends has passed class XII from National Institute of Open Schooling (NIOS) and wants to start a business Heshe approaches you to join in this venture Heshe wants you to contribute some money and participate in the business activities Both of you if join hands constitute a partnership2CHARACTERISTICS1048698 Agreement A partnership is formed by an agreement The agreement may be either oral or in writing It defines the relationship between the persons who agree to carry on business It may contain the terms of sharing profit and the capital to be invested by each partner etc The written agreement is known as partnership deed1048698 Number of persons There must be at least two persons to form a partnership

The maximum number of partners in a partnership firm can be 50 according toCompanies Act 20131048698 Business The Partnership is formed to carry on business with a purpose of earning profits The business should be lawful Thus if two or more persons agree to carry on unlawful activities it will not be termed as partnership1048698 Sharing Profits The partners agree to share profits in the agreed ratio In caseof loss all the partners have to bear it in the same agreed profit sharing ratio10486981048698Mutual Agency Every partner is an agent of the other partners Every partner can bind the firm and all other partners by hisher acts Each partner will be responsible and liable for the acts of all other partners10486981048698Unlimited liability The liability of each partner except that of a minor is unlimited Their liability extends to their personal assets also If the assets of the firm are insufficient to pay off its debts the partnersrsquo personal property can be used to satisfy the claim of the creditors of the partnership firm10486981048698Management All the partners have a right to mange the business However they may authorize one or more partners to manage the affairs of the business on their behalf10486981048698Transferability of Share No partner can transfer hisher share to any one including hisher family member without the consent of all other partners3PARTNERSHIP DEEDAgreement forms the basis of partnership The written form of the agreement is which a document of partnership is It contains terms and conditions regarding the conduct of the business It also explains relationship between the partners This document is called partnership deed Every firm can frame its own partnership deed in which the rights duties and liabilities of the partners are stated in detail It helps in settling the disputes arising among the partners during the general conduct of business 4CONTENTS OF PARTNERSHIP DEEDThe partnership deed generally contains the following (i) Name and address of the partnership firm(ii) Nature and objectives of the business(iii) Name and address of each partner(iv) Ratio in which profits is to be shared(v) Capital contribution by each partner(vi) Rate of Interest on capital if allowed(vii) Salary or any other remuneration to partners if allowed(viii) Rate of interest on loans and advances by a partner to the firm(ix) Drawings of partners and interest thereon if any(x) Method of valuation of goodwill and revaluation of assets and liabilities on the reconstitution of the partnership ie on the admission retirement or death of a partner(xi) Settlement of disputes by arbitration(xii) Settlement of accounts at the time of retirement or death of a partner5IN ABSENCE OF PARTNERSHIP DEEDThe partnership deed lays down the terms and conditions of partnership in regard to rights duties and obligations of the partners In the absence of partnership deed there may arise a controversy on certain issues like profit sharing ratio interest on

capital interest on drawings interest on loan and salary of the partners In such cases the provisions of the Indian Partnership Act becomes applicableSome of the Issues are(i) Distribution of Profit Partners are entitled to share profits equally(ii) Interest on Capital Interest on capital is not allowed(iii) Interest on Drawings No interest on drawing of the partners is to be charged(iv) Interest on Partnerrsquos Loan A Partner is allowed interest 6 per annum on the amount of loan given to the firm by himher(v) Salary and Commission to Partner A partner is not entitled to anysalary or commission or any other remuneration for managing the business

History TOPIC-TOWARDS INDEPENDENCE AND PARTITION THE LAST PHASE (1935-1947)

SUB TOPIC-IMPORTANT POLITICAL DEVELOPMENTS ndash GROWTH OF SOCIAL IDEAS

Socialism is a political social and economic philosophyLike in other parts of the world the Russian revolution of 1917 served as a great inspiration for revolutionaries in India who at that time were engaged in the struggle for liberation from British ruleSocialist ideas led to the formation of communist party of IndiaJAWAHARLAL NEHRU Among the early Congress leaders Jawaharlal Nehru was very much impressed and influenced by the Socialist ideas He also learnt about the Economic activities of the Soviet Union after the Bolshevic Revolution 1917 He made full use of them in IndiaThe election of Jawaharlal Nehru and Subhas Chandra Bose showed the Left wing tendency within CongressJawaharlal Nehru demanded economic freedom along with political freedom of the people in order to end the exploitation of masses

Nehrus working committee included three socialists leaders The Lucknow session was a landmark in the evolution of socialist ideas of the congressSUBHAS CHANDRA BOSE ndash Subhas Chandra Bose had socialist leaning Both Jawaharlal Nehru and Subhas Chandra Bose were known as leftist Congress men Later on National Congress divided into Leftist and rightist campCONGRESS SOCIALIST Within the Congress some leaders formed the Congress Socialist partyPattavi Sitaramyya Sardar Patel Rajendra Prasad had hostile attitude towards the Congress Socialist partyJawaharlals attitude was hesitant

1 QUESTION ndash Mention name of two Congress leaders who had socialist leaning

1ANSWER ndash Subhas Chandra Bose and Jawaharlal Nehru2QUESTION- In which session of the congress Jawaharlal elaborated his Socialist ideas2 ANSWER ndash Lucknow and Faizpur Session in December 1935 and 19363QUESTION ndash Why Congress was sharply divided into leftist and rightist camp 3ANSWER ndash Subhas Chandra Bosersquos attempt to seek re election for congress presidentship in 1939sharply divided the National Congress into Leftist and Rightist camp4 QUESTION ndash Who was MN Roy 4 ANSWER ndash Manabendra Roy first formed the Communist Party of India outside the country at Tashkent in 19205QUESTION ndash Who formed the Congress Socialist Party within the Congress5 ANSWER ndash Jaya Prakash Narayan Achyut Patwardhan Acharya Narendra Dev Ram Mohan Lohia Aruna Asaf Ali6QUESTION ndash When was the Congress Socialist Party formed What was its object6 ANSWER ndash 1934The Congress Socialist Party sought to work out socialist programme through the Congress They joined hands with the Congress and wanted to carry

Subhas Chandra Bose being expelled from the congress after the Tripuri rift he formed Forward BlockThere were basic differences between the Congress Socialists and the communistsTRADE UNION ACTIVITIES Maximum working class people lived in Bombay and Calcutta The working and living conditions of those workers were very miserable In this situation Shasipada Banerjee NM Lokhande protested against the oppression of the working class peopleThe first Trade Union Madras Labour Union was formed in 1918 by BP WadiaIndustrial strikes took place in Kanpur Calcutta Madras Jamshedpur and Ahmedabad AITUC was formed in Bombay in 1927 The growth of Trade union among the workers was slow because of the fear of the dismissal of the jobIn the mean time the Moderates as well as Communists left AITUC and formed separate organization

on National struggle with the help of workers and peasant class of the society7 QUESTION ndash What was the name of the party founded by Subhas Chandra Bose7 ANSWER- Forward Block8QUESTION ndash Who was Shasipada Banerjee8 ANSWER ndash Shasipada Banerjee was a radical Brahmo He founded a working menrsquos club to protest against exploitation of the British rulers towards the working class of India9 QUESTION ndash What was the weekly published by NM Lokhande9ANSWER- Dinabandhu10 QUESTION ndash Who founded Bombay Mill-Hands Association and in which year10 ANSWER- NM Lokhande in189011 QUESTION- Who was BP WadiaANSWER- BPWadia was the founder of Madras Labour Union in191812 QUESTION- What was the name of the first labour union of India12 ANSWER- Madras Labour Union13 QUESTION Who founded the Majur Mahajan 13 ANSWER GANDHIJI14 QUESTION What was the full form of AITUC When it was formed14 ANSWER All India Trade Union Congressin 192715QUESTION Who formed the Red Trade Union Congress and in which year15ANSWER The Communists formed the Red Trade Union Congress16 QUESTION What do you mean by Socialism16 ANSWER Socialism describes any political and economic theory that says the community rather than individuals should own and manage property and natural resources

Subject Eng Literature (The Tempest ndash William Shakespeare) Topic Act III Scene 3 Lines 1 to 52 (Line 52 ndash Brother my lord the Duke Stand to and do as we) Date 13th April 2020 (4th Period)

[Students should read the original play and also the paraphrase given in the school prescribed textbook]Summary Questions amp Answers

o Alonso Sebastian Antonio Gonzalo Adrian Francisco and others wandered about the island in search of Ferdinand and gets tired and hungry of the toil and at the same time gives up all hope of finding him

o Antonio and Sebastian are happy that Alonso is out of hope and decide to make another attempt on his life that night when being so tired they will be sleeping soundly

o Suddenly a solemn and strange music is heard in the air and several strange shapes enter bringing in a banquet These strange shapes then dance round it with gestures of salutation and then inviting the King to eat they depart

o Seeing this strange scene all are inclined to believe the tales told by travelers that there truly are ldquounicornsrdquo and ldquothe phoenixrsquo thronerdquo

1 ALONSO What harmony is this My good friends hark (L18-27)

GONZALO Marvellous sweet music

[Enter several strange shapes bringing in a banquet

they dance about it with gentle actions of salutation

and inviting the King and his companions to eat they depart]ALONSO Give us kind keepers heavens What were theseSEBASTIAN A living drollery Now I will believe

That there are unicorns that in Arabia

There is one tree the phoenixrsquo throne one phoenix

At this hour reigning thereANTONIO Ill believe both

And what does else want credit come to me

And Ill be sworn rsquotis true Travellers neer did lie

Though fools at home condemn rsquoem

(a) How did Prospero present an amazing spectacle before Alonso and his companions

Using his magic powers Prospero ordered strange shapes to lay a banquet before Alonso and his companions The shapes brought several dishes with tasty eatables in them They placed the dishes on a table before Alonso and his companions Then the strange shapes began to dance gracefully around the banquet While dancing they made gestures inviting them to eat the food Then suddenly the shapes disappeared(b) Who were the guests at the strange banquet Describe the lsquoliving drolleryrsquo

Alonso Sebastian Antonio Gonzalo Adrian and Francisco were the guests at the strange banquet

The term ldquoliving drolleryrdquo refers to live entertainment show In this context when Alonso the King of Naples Sebastian his brother Antonio the treacherous brother of Prospero Gonzalo the kind and loyal councillor to the King Adrian and Francisco came to the island they were hungry and weary in their spirits They heard a solemn and strange music They were shocked to see several strange shapes bringing in a banquet and these shapes danced about it with gentle action of salutation inviting the King and his companions to eat After this Sebastian described this show as lsquoliving drolleryrsquo(c) What is lsquophoenixrsquo What are lsquoUnicornsrdquo

The term lsquophoenixrsquo refers to a mythical Arabian bird which lived alone and perched on a solitary tree After one hundred years it expired in flames and rose again from its own ashes

lsquoUnicornsrsquo refers to the mythological four-footed beasts having horns in the centre of their foreheads When the horns are ground into powder the powder was believed to be

an aphrodisiac(d) How does Sebastian explain the puppet show OR Why does the speaker now believe in unicorns and phoenix

Sebastian finds several strange shapes bringing in the banquet They invite the king and his party for dinner and soon depart He tells that if such a strange sight can be a reality there is nothing incredible in the world and from the present moment he will believe anything He says that it is a strange dumb show enacted not by puppets but by living beings It is stranger than a travellerrsquos tale Seeing such a thing

before his own eyes he will no longer disbelieve the story about unicorns and phoenix(e) How do the other characters present respond to this living drollery

At the sight of the lsquoliving drolleryrsquo like Sebastian Gonzalo and Antonio too acted strangely Antonio told that he too now believes in unicorns and phoenix and anything else that seems to be incredible He too now believes in travellersrsquo tales Gonzalo told that if he would report those happenings in Naples nobody will believe him He considers that those gentle shapes were gentler in manner in comparison to the living beings Alonso was at first sight suspicious and told them that those strange shapes conveyed their meaning in expressive gestures when they seemed to lack speech by their movements and sounds Francisco was amazed at their mysterious disappearance

2 ALONSO Not I

(Line 43-52)GONZALO Faith sir you need not fear When we

were boysWho would believe that there were mountaineers

Dewlapped like bulls whose throats had hanging at rsquoem

Wallets of flesh Or that there were such men

Whose heads stood in their breasts Which now we find

Each putter-out of five for one will bring us

Good warrant ofALONSO I will stand to and feed

Although my lastmdashno matter since I feel

The best is past Brother my lord the Duke

Stand to and do as we

(a) How does Alonso respond at the spectacle of the shapes which were sent to them at the instruction of Prospero

After seeing the strange sight of appearing and disappearing of the shapes sent by Prospero to arrange a banquet for them Alonso says that his surprise at having seen those creatures is infinite and he is fully justified in feeling so much surprise He thinks that their shapes their gestures and the sounds they made were indeed amazing Although they do not possess the gift of speech yet they were able to convey their

thoughts by means of their gestures only

(b) What does Prospero say about the views expressed by Alonso regarding the shapes What does Francisco think about the shapesAfter hearing Alonsorsquos views about the shapes Prospero says that this manrsquos praise of the spirits is rather hasty He means to say that Alonso has shown great haste in reaching the conclusion about the shapes Francisco is amazed to see that those shapes disappeared in a mysterious way(c) What does Sebastian ask Alonso to doSebastian tells Alonso that the shapes having disappeared should not matter to them because they have left the eatables behind He asks Alonso to enjoy eating as they are extremely hungry but the king does not accept his offer of enjoying the dishes(d) How does Gonzalo try to dispel Alonsorsquos fear of those strange shapes What kind of references does he give to AlonsoGonzalo says that those who have travelled abroad have reported seeing even stranger sights than these shapes that Alonso and his companions have beheld Hence there is no reason to feel afraid of these shapes Gonzalo further adds that in his younger days he had heard strange stories from travelers and Alonso might have heard similar stories For instance it was said that there existed a certain race of

human beings who had huge lumps of flesh hanging at their throats and who therefore resembled bulls Then Gonzalo tells about a race of human beings whose heads were located at their breasts Gonzalo says that such stories were not believed by most people in those days but now-a-days these stories have become common(e) Explain the following lsquoEach putter-out of five for onersquoEnglish travellers often insured their trips with London brokers Those that went on foreign travels those days used to deposit a certain amount with some firm or company in London before their departure If the travelers failed to return the money was forfeited by the company with which it had been deposited But this money was repaid five-fold if the travelers returned safe and sound In this way a traveler stood a great chance of recovering the entire cost of his

travels(f) Give the explanatory meanings of the following expressions in the context of the above extract (i) Dewlapped (ii) Wallets of flesh

(iii) Putter-out(i) Dewlapped having big lumps of flesh at the necks(ii) Wallets of flesh large masses of flesh looking like bags(iii) Putter-out to invest money before commencing the travel

  • General methods of preparation of hydrogen
  • Chapter Dimensional Analysis (Summary)
    • Properties of Charge
Page 42:   · Web viewSubject. Topic. Summary. Execution. Hindi. व्याकरण. शरीरके अंगो के नाम लिखिए. 1) आँख 2) नाक 3

modern concepts and approaches for studying itrsquos all aspects

Subject Eng Literature (The Tempest ndash William Shakespeare) Topic Act I Scene 1 Lines 1 to 32 (Line 32 ndash Gonzalo hellip If he be not born to be hanged our case is miserable) Date 13th April 2020 (3rd Period)

[Students should read the original play and also the paraphrase given in the school prescribed textbook]Summary Questions amp Answers

[SUMMARY OF THE ENTIRE SCENE]

o The play starts with the scene of a severe storm at sea Alonso (King of Naples) Sebastian (Alonsorsquos brother) Ferdinand (Alonsorsquos son) Gonzalo Antonio (the usurping Duke of Milan) are in a ship in the midst of the storm

o The mariners are trying their best to control the vessel from running aground and are totally following the orders of their Master the Boatswain They have scant success

o The mariners become extremely unhappy and annoyed when most of the passengers arrive on the deck thereby hampering their effort to save the ship There is serious confrontation between them and the passengers who are part of the Kingrsquos entourage

o The mariners could not save the ship

SUMMING-UP

(i) Vivid description of the scene which gives a realistic description of terror and confusion of a tropical storm

(ii) Shows Shakespearersquos accuracy of knowledge in describing the naval operations and also matters of seamanship

(iii) The opening scene justifies the title ndash The Tempest

UNANSWERED QUESTIONS

(i) The King always travels with his entire fleet including his soldiers Where

(1) GONZALO Nay good be patient (Line 15-26)BOATSWAIN When the sea is Hence What cares these

roarers for the name of the king To cabin silence Trouble us not

GONZALO Good yet remember whom thou has aboardBOATSWAIN None that I more love than myself You are a

councillor if you can command these elements to silence and work

the peace of the present we will not hand a rope more use your authority If you cannot give thanks you have

lived so long and make yourself ready in your cabin for the mischance of the hour if it so hap [To the Mariners]

Cheerly good hearts [To Gonzalo] Out of our way I say

(a) To whom is the boatswain speaking What does he mean by lsquoNone that I more love than myselfrsquo

The Boatswain is speaking to Gonzalo the honest old councilor of the Duke of MilanBy using the words ndash lsquoNone that I love more than I love myselfrsquo means that for the Boatswain nobody is dearer to him than his own life

(b) What were the conditions that made the boatswain react in this way

The Boatswain reacts in this way because the storm is at sea and Alonso King of Naples Sebastian his brother Ferdinand his son Gonzalo Antonio the usurping Duke of Milan on board are in distress and in panic Thus they have rushed to the deck interrupting the work of the mariners

(c) What hope does Gonzalo take from the attitude of the boatswain

The insolent and authoritative attitude of Boatswain makes Gonzalo feel comforted He tells that there are no signs that the Boatswain will be drowned But his facial appearance and attitude shows that he is destined to die on land by hanging which in effect means that all on board will be saved Otherwise all the persons on board are doomed

(d) How can they lsquomake yourself ready in your cabinrsquo For what were they asked to make ready themselves

In order to make themselves ready in their cabin the

were the other ships

(ii) Why was the ship in that area Where was it coming from or going where

(iii) The ship broke apart What happened to those who were in the ship

passengers on board must prepare for death which they will possibly soon have to meetThey can retire to their cabins and offer prayers to the Almighty to save them from drowning

(e) What does the boatswain say when he is asked to be patient What does he order to the royal party

When the boatswain is asked to be patient and remain calm he says that he will be patient only when the storm will be over and the sea will be calm but as long as the storm blows and there is danger to the ship he cannot think of being patient He orders the royal party to go to the cabin and leave the mariners to their work

(2) GONZALO I have great comfort from this fellow (Line 27-36)

Methinks he hath no drowning mark upon him his complexion is perfect

gallows Stand fast good Fate to his hanging Make the rope of his destiny our cable for our own doth little advantage If he be not born to be hanged our case is miserable

(a) Why does Gonzalo regard the Boatswain in the midst of danger

In the midst of danger Gonzalo regards the boatswain because he feels that the Boatswain is a source of comfort and is bent upon to do his work sincerely which in this case is saving the ship and its passengers from the severest of raging storm

(b) What reasons does Gonzalo give when he says that none in the ship will die of drowning

Gonzalo is almost sure that none in the ship will die by drowning His says that there is no mark on the face of the boatswain that indicates that he will die by drowning On the other hand the lines on his face are strong indications that he will be hanged to death Therefore there shall be no danger of the shiprsquos sinking

(c) Explain the following ldquoStand fast good Fate to his hanging Make the rope of his destiny our cable for our own doth little advantage If he be not born to be hanged our case is miserablerdquo

The stated lines mean that if the will of destiny is to be carried out then the ship will not get wrecked and all the passengers will be saved The safety of the passengers therefore depends upon the will of fate being carried out in the case of the boatswain If however the boatswain is not to die by hanging then the passengers are also very unsafe because in that case the ship is likely to sink

(d) What order does the Boatswain give to the sailors

when he re-enters What does he say about the crying of the fellows inside the cabin

The boatswain orders the sailors to bring the topmast lower and bring the ship close to a stationary position with the help of the main sail He says that the fellows inside the cabin are moaning and crying in their distress louder than his voice and louder even than the roaring of the storm

Class XII (ScienceCommerceHumanities) Subject Topic Summary Execution

Computer Science

PropositionalLogic

Propositional logic is a procedure to provide reasoning through statementProposition A ststement that results in True or False is said to be proposition There are two types of propositionSimple proposition amp compound propositionSimple proposioton A simple proposition is one that is not a part of any other proposition Such sentential form of proposition is symbolized with english letters in short For example Ram is a claver student (TrueFalse)Where do you live (Not in True or False)Grapes are sweet (TrueFalse)It rains today (TrueFalse)Here we can see some statements anwer would be true or false but some staements answer can not give in terms of true or false Thus the sentences which can be answered in true or false are known as simple propositionAssigning propositon to a variableThe general syntax to assign propostion to a variable is as followsVariable = Simple propositonFor example A=Ram is a clever studentB= Grapes are sweetC= it rains todayCompound proposition

helliphellipto be continued in next classhelliphellipMath Relation Relation If A and B are two non-empty sets

then a relation R from A to B is a subset of AxB If R A x B and (a b) R then we say that a sube isinis related to b by the relation R written as aRbeg Let A be the set of students of class XII and B be the set of students of class XI Then some of the examples of relation from A to B arei) (a b) AXB a is brother of bisinii) (a b) AXB age of a is more than age of isinb Types of relation In this section we would like to study different types of relations We know that a relation in a set A is a subset of A times A Thus the empty set φ and A times A are two extreme relations For illustration consider a relation R in the set A = 1 2 3 4 given by R = (a b) a ndash b = 10 This is the empty set as no pair (a b) satisfies the condition a ndash b = 10 Similarly R = (a b) | a ndash b | ge 0 is the whole primeset A times A as all pairs (a b) in A times A satisfy | a ndash

Example 1 Let A be the set of all students of a boys school Show that the relation R in A given by R = (a b) a is sister of b is the empty relation and R = (a b) the primedifference between heights of a and b is less than 3 meters is the universal relationSolution Since the school is boys school no student of the school can be sister of any student of the school Hence R = φ showing that R is the empty relation It is also obvious that the difference between heights of any two students of the school has to be less than 3 meters This shows that R = A times A is primethe universal relation Example 2 Show that the relation R in the set 1 2 3 given by R = (1 1) (2 2) (3 3) (1 2) (2 3) is reflexive

b | ge 0 These two extreme examples lead us to the following definitionsDefinition 1 A relation R in a set A is called empty relation if no element of A isrelated to any element of A ie R = φ A times AsubDefinition 2 A relation R in a set A is called universal relation if each element of A is related to every element of A ie R = A times A Both the empty relation and the universal relation are some times called trivial relation Definition 3 A relation R in a set A is called(i) reflexive if (a a) R for every a Aisin isin(ii) symmetric if (a1 a2) R implies that (aisin 2a1)

R for all aisin 1 a2 Aisin(iii) transitive if (a1 a2) R and (aisin 2 a3) R isinimplies that (a1 a3) R for all aisin 1 a2 a3 AisinDefinition 4 A relation R in a set A is said to be an equivalence relation if R is reflexive symmetric and transitive

but neither symmetric nor transitiveSolution R is reflexive since (1 1) (2 2) and (3 3) lie in R Also R is not symmetric as (1 2) R but (2 1) isin notinR Similarly R is not transitive as (1 2) R and (2 3) R but (1 3) R isin isin notinExample 3 Show that the relation R in the set Z of integers given byR = (a b) 2 divides a ndash b is an equivalence relationSolution R is reflexive as 2 divides (a ndash a) for all a Z isinFurther if (a b) R then 2 divides a isinndash b Therefore 2 divides b ndash a Hence (b a) R which shows that R is isinsymmetric Similarly if (a b) R and (b c) R isin isinthen a ndash b and b ndash c are divisible by 2 Now a ndash c = (a ndash b) + (b ndash c) is even (Why) So (a ndash c) is divisible by 2 This shows that R is transitive Thus R is an equivalence relation in ZExample 4 Let L be the set of all lines in a plane and R be the relation in L defined as R = (L1 L2) L1 is perpendicular to L2 Show that R is symmetric but neither reflexive nor transitiveSolution R is not reflexive as a line L1 can not be perpendicular to itself ie (L1 L1) R notinR is symmetric as (L1 L2) Risin

L1 is perpendicular to L2rArr L2 is perpendicular to L1rArr (L2 L1) RrArr isin

R is not transitive Indeed if L1 is perpendicular to L2 and L2 is perpendicular to L3 then L1 can never be perpendicular to L3 In fact L1 is parallel to L3 ie (L1 L2) R isin(L2 L3) R but (L1 L3) Risin notin

Chemistry Solid state Characteristics if Solids(i)The particles are locked in fixed positions they are unable to change their relative positions and this brings a definite shape and volume of a solid(ii)In a solid the constituent particles are held by strong forces of attractionThe forces of attraction may be bonding or non bonding(iii)The constituent particles in a solid pack together as closely as possibleoccupying most of the available space within the solidThus the empty space in a solid is very smallThis makes a solid highly rigid and nearly incompressibleThis also explains why a solid has high density and exhibits slow diffusionClassification of Solids

Q1)Define Crystalline solids AnsA Solid that has a definite geometrical shape and a sharp melting pointand whose constituent particles (atomsmolecules or ions) are arranged in a long range order of definite pattern extending throughout the solid is called a crystalline solidExNaClQ2)Define Amorphous solids AnsA solid that does not have a definite shape and a sharp melting pointand whose constituent particles (atomsmolecules or ions) are not arranged in a definite pattern is called an amorphoussolid

Crystalline solidsAmorphous solids

ExGlassRubberQ3)Classify Crystalline Solids Crystalline Solids

Physics Coloumbrsquos Law (Summary)

Before Going Into Coloumbrsquos Law We Will First Learn What is Charge Properties of Charge and Always remember that charge is quantized ie a body always have static charge of magnitude equal to some integral multiple of fundamental electronic charge e= 16 x 10- 19 C

Charge is the property of matter that causes it to produce and experience electrical and magnetic effects The study of the electrical charges at rest is called electrostatics When both electrical and magnetic effects are present the interaction between charges is referred to as electromagnetic

There exist two types of charges in nature positive and negative Like charges repel and unlike charges attract each other

The type of charge on an electron is negative The charge of a proton is the same as that of an electron but with a positive sign In an atom the number of electrons and the number of protons are equal The atom is therefore electrically neutral If one or more electrons are added to it it becomes negatively charged and is designated as negative ion However if one or more electrons are removed from an atom it becomes positively charged and is called a positive ion

The excess or deficiency of electrons in a body gives the concept of charge If there is an excess of electrons in a body it is negatively charged And if there is deficiency of electrons the body becomes positively charged Whenever addition or removal of electrons takes places the body acquires a charge

The SI Unit of charge is coulomb (C) In SI units the current is a fundamental quantity having a unit of ampere (A) The unit of charge is defined in terms of the unit of current Thus one coulomb is the charge transferred in one second across the section of a wire carrying a

Ionic SolidsMetallicSolids

Molecular Solids

current of one ampere

As q = It we have1 C = (1 A) (1 s)

The dimensions of charge are [A T]

Properties of Charge

(1) Quantization of Charge Electric charge can have only discrete values rather than any value That is charge is quantized The smallest discrete value of charge that can exist in nature is the charge on an electron given as

e = plusmn 16 x 10- 19 C

This is the charge attained by an electron and a protonA charge q must be an integral multiple of this basic unit That is

Q = plusmn ne where n = 1 2 hellip

Charge on a body can never be (frac12)e (23)e or 57e etcWhen we rub a glass rod with silk some electrons are transferred from the rod to the silk The rod becomes positively charged The silk becomes negatively charged The coulomb is a very large amount of charge A typical charge acquired by a rubbed body is 10 - 8 C

Biology Reproduction in organisms

Welcome to this new session 2020-21Today in this first chapter we mainly discuss about reproduction types needs and life span of some organismsWe also discuss about difference between sexual and asexual reproduction

Q1 What is reproductionReproduction is defined as a biological processin which an organism gives rise to young onessimilar to itselfQ2 What are the needs of reproductionbulli) Reproduction maintain life on earthii) It enables the continuity of the species generation after generationiii) It creates genetic variation among populationsQ3 Define Life span and write some orgnisms life spanbull Life span is the period from birth to

the natural death of an organism- OrganismsLife span1 Butterfly 1 - 2 weeks2 Fruit fly 30 days3Dog 10-13 years4 Rose5-7 years5 Tortoise100-150 years6 Banyan Tree -200 - 250 yearsQ4 Reproduction is of two types in case ofanimals but in case of plants vegetative propagation is also present

Asexual Reproduction Sexual Reproductioni) Always uniparentalii) Gametes are not involvediii) Only mitotic division involvediv) Somatic cells of parents are involvedv) Offsprings are genetically similar to the parents

i) Usually biparentalii) Gametes are involvediii) Meiosis occurs during gametogenesis Mitosis occurs after fertilisationiv) Germ cells of the parents are involvedv) offsprings are genetically different from the parents

COMMERCE BUSINESS ENVIRONMENT

Welcome to the new sessiontoday we are going to start the first chapter of Class XII The name of the chapter is Business Environment

Already many of you have got some idea about the word business environment form the first chapter of business studies in class XI

In todayrsquos world every business enterprise is a part of the society It exists and operates in association with various groups in society such as customers suppliers competitors banks and financial institutions government agencies trade unions media and so on All these groups influence the functioning of business in one way or the other They constitute the environment of businessConcept of Business Environment

The term lsquobusiness environmentrsquo refers to the sum total of all individuals institutions and other forces that lie outside a business enterprise but that may influence its functioning and performance

The main features of business environment Totality of External forces General and Specific forces Interrelatedness Complexity Dynamic Uncertainty

Prepare the following questions from todayrsquos assignment

2 What do you mean by business environment

The term lsquobusiness environmentrsquo means the aggregate of all forces factors and institutions which are external to and beyond the control of an individual business enterprise but they may influence its functioning and performance Business environment is the macro framework within which a business firm a micro unit operates It consists of several interrelated and interacting elements

2 Explain the main features of business environment in brief

Totality of External forces-Business environment is the sum total of all things external to a business environment

General and Specific forces-It

Relativity

The Interrelation between business and its environment

The business enterprise is an open system It continuously interacts with its environment It takes inputs (such as raw materials capital labour energy and so on) from its environment transforms them into goods and services and sends them back to the environment

Fig 1 Business Environment Relationship

includes both the forces general forces are the economic social political legal and technological conditions which indirectly influence all business enterprise Specific forces are the investors customers competitors and suppliers which influence individual enterprise directly

Interrelatedness-Different elements of environment are interrelated for an example growing awareness for health care has increased the demand for health foods

Complexity- Business environment id complex in nature as the elements keep on changing example economic technological and other forces changes in demand for a product and service

Dynamic-Business environment is not static it keeps on changing

Uncertainty- Itrsquos very difficult to predict future events such as technology and fashion which occur fast and frequently

Business Studies

Human Resources Management

Human resource of an organisation are the aggregate of knowledge skills attitudes of people working in it

The management system which deals with human resources is called human resource management

Features of HRMbullComprehensive functionbullPeople-oriented

Question1) What do you mean by human

resource management Answer) Human resource management may be defined as that field of Management which has to do with planning organising and controlling the functions of procuring developing maintaining and utilising the labour force

bullAction oriented bullPervasive function bullContinuous function

2) Explain the features of HRM in brief

Answer)bullHuman Resource Management is concerned with managing people at work bull Human Resource Management is concerned with employees which bring people and organisations together so that the goals of each are met bullHuman resource management considered every employees as an individual and also promote their satisfaction and growth bull Human resource management is inherent in all organisations and at all levelsbullManagement of human resources are ongoing on never ending process which requires a constant alertness and Awareness of human relations

3) ldquoHR function is said to be pervasiverdquowhy

Answer) Human resource management is required in all organisations whether it is private or government organisations armed forces sports organisations etc It permeatsall the functional areas like production marketing finance research etc This from this feature of human resource management it can be said that it is pervasive in nature

Economics Demand Q1DEFINITION OF DEMANDIn economics demand is the quantity of a good that consumers are willing and able to purchase at various prices during a given period of timeQ2DEMAND CURVEIn economics a demand curve is a graph depicting the relationship between the price of a certain commodity and the quantity of that commodity that is demanded at that pricQ3LAW OF DEMANDIn microeconomics the law of demand states that conditional on all else being equal as the price of a good increases quantity demanded decreases conversely as the price of a good decreases quantity demanded increasesQ4ASSUMPTION of LAW OF DEMAND(i)No change in price of related commodities(ii) No change in income of the consumer(iii) No change in taste and preferences customs habit and fashion of the consumer( No expectation regarding future change in priceQ5MARKET DEMAND SCHEDULEIn economics a market demand schedule is a tabulation of the quantity of a good that all consumers in a market will purchase at a

given price At any given price the corresponding value on the demand schedule is the sum of all consumersrsquo quantities demanded at that priceQ6INDIVIDUAL DEMAND SCHEDULEIndividual demand schedule refers to a tabular statement showing various quantities of a commodity that a consumer is willing to buy at various levels of price during a given period of timeQ7 FACTORS AFFECTING INDIVIDUAL DEMAND FOR A COMMODITY

The factors that influence a consumerrsquos decision to purchase a commodity are also known as determinants of demand The following factors affect the individual demand for a commodity1 price of the commodity2 price of related goods3 income of buyer of the commodity4 tastes and preferences of the buyer1 Price of the CommodityYou must have observed that when price of a commodity falls you tend to buy more of it and when its price rises you tend to buy less of it when all other factors remain constant (lsquoother things remaining the samersquo) In other words other things remaining the same there is an inverse relationship between the price of a commodity and its quantity demanded by its buyers This statement is in accordance with law of demand which you will study in the later part of this lesson Price of a commodity and its quantity demanded by its buyers are inversely related only when lsquoother things remain the samersquo So lsquoother things remaining the samersquo is an assumption when we study the effect of changes in the price of a commodity on its quantity demanded2 Price of Related goodsA consumer may demand a particular good But while buying that good heshe also asks the price of its related goods Related goods can be of two types-(i) Substitute goods(ii) Complementary goods While purchasing a good prices of its substitutes and complements do affect its quantity purchased(i) Price of Substitute Goods Substitute goods are those goods which can easily be used in place of one another for satisfaction of a particular want like tea and coffee An increase in price of substitute good leads to an increase in demand for the given commodity and a decrease in price of substitute good leads to a decrease in demand for the given commodity It means demand for a given commodity is directly affected by change in price of substitute goods For example if price of coffee increases the demand for tea will rise as tea will become relatively cheaper in comparison to coffee(ii) Price of Complementary goods Complementary goods are those goods which are used together to satisfy a particular want like car and petrol An increase in the price of complementary goods leads to a decrease in demand for the given commodity and a decrease in the price of complementary goods leads to an increase in demand for the given commodity For example if price of petrol falls then the demand for cars will increase as it will be relatively cheaper to use both the goods together So demand for a given commodity is inversely affected by change in price of complementary goods3 Income of the Buyer of CommodityDemand for a commodity is also affected by income of its buyer However the effect of change in income on demand depends on the nature of the commodity under consideration In case of some goods like full cream milk fine quality of rice (Basmati rice) etc demand for these commodities increases when income of the buyer increases and

demand for these commodities decreases when income of the buyer decreases Such goods whose demand increases with the increase in income of the buyer are called normal goods But there are some goods like coarse rice toned milk etc whose demand decreases when income of buyer increases and their demand increases when income of the buyer decreases Such goods whose demand decreases with the increase in income of the buyer are called inferior goods Suppose a consumer buys 10 Kgs of rice whose price is ` 25 per Kg He cannot afford to buy better quality of rice because the price of such rice is ` 50 per Kg The consumer is spending ` 250 per month on the purchase of rice Now if income of the consumer increases and he can afford ` 350 on purchase of 10 Kg of rice Now he can afford to buy some quantity of rice say 6 Kgs whose price is ` 25 per Kg and may buy 4 Kgs of rice whose price is ` 50 per Kg Thus he will buy 10 Kgs of rice by spending ` 350 per month Therefore we may conclude that demand for normal goods is directly related to the income of the buyer but demand for inferior goods is inversely related to the income of the buyer4 Tastes and Preferences of the BuyerThe demand for a commodity is also affected by the tastes and preferences of the buyers They include change in fashion customs habits etc Those commodities are preferred by the consumers which are in fashion So demand for those commodities rises which are in fashion On the other hand if a commodity goes out of the fashion its demand falls because no consumer will like to buy it(5) Number of Buyers in the Market(Population)Increase in population raises the market demand whereas decrease in population reduces the market demand for a commodity Not only the size of population but its composition like age (ratio of males females children and old people in population) also affects the demand for a commodity It is because of needs of children young old male and female population differs(6) Distribution of Income and WealthIf the distribution of income and wealth is more in favour of the rich demand for the commodities preferred by the rich such as comforts and luxuries is likely to be higher On the other hand if the distribution of income and wealth is more in favour of poor demand for commodities preferred by the poor such as necessities will be more(7) Season and Weather ConditionsThis is generally observed that the demand for woolens increases during winter whereas demand for ice creams and cold drinks increases during summer Similarly market demand for umbrellas rain coats increases during rainy seasonQ8 REASONS FOR OPERATION OF LAW OF DEMAND WHY DEMAND CURVE SLOPES DOWNWARDNow we will try to explain why does a consumer purchase more quantity of a commodity at a lower price and less of it at a higher price or why does the law of demand operate ie why does the demand curve slope downwards from left to right The main reasons for operation of law of demand are1 Law of Diminishing Marginal UtilityAs you have studied earlier law of diminishing marginal utility states that as we consume more and more units of a commodity the utility derived from each successive unit goes on decreasing The consumer will be ready to pay more for those units which provide him more utility and less for those which provide him less utility It implies that he will purchase more only when the price of the commodity falls2 Income Effect

When price of a commodity falls purchasing power or real income of the consumer increases which enables him to purchase more quantity of the commodity with the same money income Let us take an example Suppose you buy 4 ice creams when price of each ice cream is ` 25 If price of ice creams falls to ` 20 then with same money income you can buy 5 ice creams now3 Substitution EffectWhen price of a commodity falls it becomes comparatively cheaper as compared to its substitutes (although price of substitutes has not been changed) This will lead to rise in demand for the given commodity For example if coke and Pepsi both are sold at ` 10 each and price of coke falls Now coke has become relatively cheaper and will be substituted for Pepsi It will lead to rise in demand for coke4 Change in Number of BuyersWhen price of a commodity falls some old buyers may demand more of the commodity at the reduced price and some new buyers may also start buying this commodity who were not in a position to buy it earlier due to higher price This will lead to increase in number of buyers when price of the commodity falls As a result demand for the commodity rises when its price falls5 Diverse Uses of a CommoditySome commodities have diverse uses like milk It can be used for drinking for sweet preparation for ice cream preparation etc If price of milk rises its use may be restricted to important purpose only This will lead to reduction in demand for other less important uses When price of milk falls it can be put to other uses also leading to rise n demand for itQ9 EXCEPTIONS TO THE LAW OF DEMANDYou have studied in law of demand that a buyer is willing to buy more quantity of a commodity at a lower price and less of it at a higher price But in certain circumstances a rise in price may lead to rise in demand These circumstances are called Exceptions to the Law of Demand Some important exceptions are1 Giffen GoodsGiffen goods are special type of inferior goods in which negative income effect is stronger than negative substitution effect Giffen goods do not follow law of demand as their demand rises when their price rises Examples of Giffen goods are jowar and bajra etc2 Status Symbol GoodsSome goods are used by rich people as status symbols eg diamonds gold jewellary etc The higher the price the higher will be the demand for these goods When price of such goods falls these goods are no longer looked at as status symbol goods and tehrefore therir demand falls3 NecessitiesCommodities such as medicines salt wheat etc do not follow law of demandbecause we have to purchase them in minimum required quantity whatever their price may be4 Goods Expected to be ScarceWhen the buyers expect a scarcity of a particular good in near future they start buying more and more of that good even if their prices are rising For example during war famines etc people tend to buy more of some goods even at higher prices due to fear of their scarcity in near future

Political Science

Constitution of India-The

Preamble

The preamble-

Preamble-

The preamble is the most precious part of the constitution We the people of India having solemnly resolved to constitute India into a Sovereign Socialist Secular Democratic Republic and to secure to all its citizensA preamble is an introductory and expressionary statement in a document that explains the documents purpose and underlying philosophy When applied to the opening paragraphs of a statute it may recite historical facts pertinent to the subject of the statuteNature and purpose of the constitution-Purpose of the Constitution dictates permanent framework of the government to form a more perfect union to establish justice and ensure peace of thenationconstitution provide principles how the government can run itself following the rules and laws written in the constitution of each state keeps them balanced

Answer the following questions-

1 What is preambleA preamble is an introductory and expressionary statement in a document that explains the documents purpose and underlying philosophy2 What is the nature and

purpose of the constitutionConstitution dictatespermanent framework of the government to form a more perfect union to establish justice and ensure peace of the nation

Homework-Learn

Accounts Compatibilty mode

1MEANING OF PARTNERSHIPPartnership is a form of business organisation where two or more persons join hands to run a business They share the profits and losses according to the agreement amongst them According to the Indian Partnership Act 1932 ldquoPartnership is relation between persons who have agreed to share profits of a business carried on by all or any one of them acting for allrdquo For example one of your friends has passed class XII from National Institute of Open Schooling (NIOS) and wants to start a business Heshe approaches you to join in this venture Heshe wants you to contribute some money and participate in the business activities Both of you if join hands constitute a partnership2CHARACTERISTICS1048698 Agreement A partnership is formed by an agreement The agreement may be either oral or in writing It defines the relationship between the persons who agree to carry on business It may contain the terms of sharing profit and the capital to be invested by each partner etc The written agreement is known as partnership deed1048698 Number of persons There must be at least two persons to form a partnership

The maximum number of partners in a partnership firm can be 50 according toCompanies Act 20131048698 Business The Partnership is formed to carry on business with a purpose of earning profits The business should be lawful Thus if two or more persons agree to carry on unlawful activities it will not be termed as partnership1048698 Sharing Profits The partners agree to share profits in the agreed ratio In caseof loss all the partners have to bear it in the same agreed profit sharing ratio10486981048698Mutual Agency Every partner is an agent of the other partners Every partner can bind the firm and all other partners by hisher acts Each partner will be responsible and liable for the acts of all other partners10486981048698Unlimited liability The liability of each partner except that of a minor is unlimited Their liability extends to their personal assets also If the assets of the firm are insufficient to pay off its debts the partnersrsquo personal property can be used to satisfy the claim of the creditors of the partnership firm10486981048698Management All the partners have a right to mange the business However they may authorize one or more partners to manage the affairs of the business on their behalf10486981048698Transferability of Share No partner can transfer hisher share to any one including hisher family member without the consent of all other partners3PARTNERSHIP DEEDAgreement forms the basis of partnership The written form of the agreement is which a document of partnership is It contains terms and conditions regarding the conduct of the business It also explains relationship between the partners This document is called partnership deed Every firm can frame its own partnership deed in which the rights duties and liabilities of the partners are stated in detail It helps in settling the disputes arising among the partners during the general conduct of business 4CONTENTS OF PARTNERSHIP DEEDThe partnership deed generally contains the following (i) Name and address of the partnership firm(ii) Nature and objectives of the business(iii) Name and address of each partner(iv) Ratio in which profits is to be shared(v) Capital contribution by each partner(vi) Rate of Interest on capital if allowed(vii) Salary or any other remuneration to partners if allowed(viii) Rate of interest on loans and advances by a partner to the firm(ix) Drawings of partners and interest thereon if any(x) Method of valuation of goodwill and revaluation of assets and liabilities on the reconstitution of the partnership ie on the admission retirement or death of a partner(xi) Settlement of disputes by arbitration(xii) Settlement of accounts at the time of retirement or death of a partner5IN ABSENCE OF PARTNERSHIP DEEDThe partnership deed lays down the terms and conditions of partnership in regard to rights duties and obligations of the partners In the absence of partnership deed there may arise a controversy on certain issues like profit sharing ratio interest on

capital interest on drawings interest on loan and salary of the partners In such cases the provisions of the Indian Partnership Act becomes applicableSome of the Issues are(i) Distribution of Profit Partners are entitled to share profits equally(ii) Interest on Capital Interest on capital is not allowed(iii) Interest on Drawings No interest on drawing of the partners is to be charged(iv) Interest on Partnerrsquos Loan A Partner is allowed interest 6 per annum on the amount of loan given to the firm by himher(v) Salary and Commission to Partner A partner is not entitled to anysalary or commission or any other remuneration for managing the business

History TOPIC-TOWARDS INDEPENDENCE AND PARTITION THE LAST PHASE (1935-1947)

SUB TOPIC-IMPORTANT POLITICAL DEVELOPMENTS ndash GROWTH OF SOCIAL IDEAS

Socialism is a political social and economic philosophyLike in other parts of the world the Russian revolution of 1917 served as a great inspiration for revolutionaries in India who at that time were engaged in the struggle for liberation from British ruleSocialist ideas led to the formation of communist party of IndiaJAWAHARLAL NEHRU Among the early Congress leaders Jawaharlal Nehru was very much impressed and influenced by the Socialist ideas He also learnt about the Economic activities of the Soviet Union after the Bolshevic Revolution 1917 He made full use of them in IndiaThe election of Jawaharlal Nehru and Subhas Chandra Bose showed the Left wing tendency within CongressJawaharlal Nehru demanded economic freedom along with political freedom of the people in order to end the exploitation of masses

Nehrus working committee included three socialists leaders The Lucknow session was a landmark in the evolution of socialist ideas of the congressSUBHAS CHANDRA BOSE ndash Subhas Chandra Bose had socialist leaning Both Jawaharlal Nehru and Subhas Chandra Bose were known as leftist Congress men Later on National Congress divided into Leftist and rightist campCONGRESS SOCIALIST Within the Congress some leaders formed the Congress Socialist partyPattavi Sitaramyya Sardar Patel Rajendra Prasad had hostile attitude towards the Congress Socialist partyJawaharlals attitude was hesitant

1 QUESTION ndash Mention name of two Congress leaders who had socialist leaning

1ANSWER ndash Subhas Chandra Bose and Jawaharlal Nehru2QUESTION- In which session of the congress Jawaharlal elaborated his Socialist ideas2 ANSWER ndash Lucknow and Faizpur Session in December 1935 and 19363QUESTION ndash Why Congress was sharply divided into leftist and rightist camp 3ANSWER ndash Subhas Chandra Bosersquos attempt to seek re election for congress presidentship in 1939sharply divided the National Congress into Leftist and Rightist camp4 QUESTION ndash Who was MN Roy 4 ANSWER ndash Manabendra Roy first formed the Communist Party of India outside the country at Tashkent in 19205QUESTION ndash Who formed the Congress Socialist Party within the Congress5 ANSWER ndash Jaya Prakash Narayan Achyut Patwardhan Acharya Narendra Dev Ram Mohan Lohia Aruna Asaf Ali6QUESTION ndash When was the Congress Socialist Party formed What was its object6 ANSWER ndash 1934The Congress Socialist Party sought to work out socialist programme through the Congress They joined hands with the Congress and wanted to carry

Subhas Chandra Bose being expelled from the congress after the Tripuri rift he formed Forward BlockThere were basic differences between the Congress Socialists and the communistsTRADE UNION ACTIVITIES Maximum working class people lived in Bombay and Calcutta The working and living conditions of those workers were very miserable In this situation Shasipada Banerjee NM Lokhande protested against the oppression of the working class peopleThe first Trade Union Madras Labour Union was formed in 1918 by BP WadiaIndustrial strikes took place in Kanpur Calcutta Madras Jamshedpur and Ahmedabad AITUC was formed in Bombay in 1927 The growth of Trade union among the workers was slow because of the fear of the dismissal of the jobIn the mean time the Moderates as well as Communists left AITUC and formed separate organization

on National struggle with the help of workers and peasant class of the society7 QUESTION ndash What was the name of the party founded by Subhas Chandra Bose7 ANSWER- Forward Block8QUESTION ndash Who was Shasipada Banerjee8 ANSWER ndash Shasipada Banerjee was a radical Brahmo He founded a working menrsquos club to protest against exploitation of the British rulers towards the working class of India9 QUESTION ndash What was the weekly published by NM Lokhande9ANSWER- Dinabandhu10 QUESTION ndash Who founded Bombay Mill-Hands Association and in which year10 ANSWER- NM Lokhande in189011 QUESTION- Who was BP WadiaANSWER- BPWadia was the founder of Madras Labour Union in191812 QUESTION- What was the name of the first labour union of India12 ANSWER- Madras Labour Union13 QUESTION Who founded the Majur Mahajan 13 ANSWER GANDHIJI14 QUESTION What was the full form of AITUC When it was formed14 ANSWER All India Trade Union Congressin 192715QUESTION Who formed the Red Trade Union Congress and in which year15ANSWER The Communists formed the Red Trade Union Congress16 QUESTION What do you mean by Socialism16 ANSWER Socialism describes any political and economic theory that says the community rather than individuals should own and manage property and natural resources

Subject Eng Literature (The Tempest ndash William Shakespeare) Topic Act III Scene 3 Lines 1 to 52 (Line 52 ndash Brother my lord the Duke Stand to and do as we) Date 13th April 2020 (4th Period)

[Students should read the original play and also the paraphrase given in the school prescribed textbook]Summary Questions amp Answers

o Alonso Sebastian Antonio Gonzalo Adrian Francisco and others wandered about the island in search of Ferdinand and gets tired and hungry of the toil and at the same time gives up all hope of finding him

o Antonio and Sebastian are happy that Alonso is out of hope and decide to make another attempt on his life that night when being so tired they will be sleeping soundly

o Suddenly a solemn and strange music is heard in the air and several strange shapes enter bringing in a banquet These strange shapes then dance round it with gestures of salutation and then inviting the King to eat they depart

o Seeing this strange scene all are inclined to believe the tales told by travelers that there truly are ldquounicornsrdquo and ldquothe phoenixrsquo thronerdquo

1 ALONSO What harmony is this My good friends hark (L18-27)

GONZALO Marvellous sweet music

[Enter several strange shapes bringing in a banquet

they dance about it with gentle actions of salutation

and inviting the King and his companions to eat they depart]ALONSO Give us kind keepers heavens What were theseSEBASTIAN A living drollery Now I will believe

That there are unicorns that in Arabia

There is one tree the phoenixrsquo throne one phoenix

At this hour reigning thereANTONIO Ill believe both

And what does else want credit come to me

And Ill be sworn rsquotis true Travellers neer did lie

Though fools at home condemn rsquoem

(a) How did Prospero present an amazing spectacle before Alonso and his companions

Using his magic powers Prospero ordered strange shapes to lay a banquet before Alonso and his companions The shapes brought several dishes with tasty eatables in them They placed the dishes on a table before Alonso and his companions Then the strange shapes began to dance gracefully around the banquet While dancing they made gestures inviting them to eat the food Then suddenly the shapes disappeared(b) Who were the guests at the strange banquet Describe the lsquoliving drolleryrsquo

Alonso Sebastian Antonio Gonzalo Adrian and Francisco were the guests at the strange banquet

The term ldquoliving drolleryrdquo refers to live entertainment show In this context when Alonso the King of Naples Sebastian his brother Antonio the treacherous brother of Prospero Gonzalo the kind and loyal councillor to the King Adrian and Francisco came to the island they were hungry and weary in their spirits They heard a solemn and strange music They were shocked to see several strange shapes bringing in a banquet and these shapes danced about it with gentle action of salutation inviting the King and his companions to eat After this Sebastian described this show as lsquoliving drolleryrsquo(c) What is lsquophoenixrsquo What are lsquoUnicornsrdquo

The term lsquophoenixrsquo refers to a mythical Arabian bird which lived alone and perched on a solitary tree After one hundred years it expired in flames and rose again from its own ashes

lsquoUnicornsrsquo refers to the mythological four-footed beasts having horns in the centre of their foreheads When the horns are ground into powder the powder was believed to be

an aphrodisiac(d) How does Sebastian explain the puppet show OR Why does the speaker now believe in unicorns and phoenix

Sebastian finds several strange shapes bringing in the banquet They invite the king and his party for dinner and soon depart He tells that if such a strange sight can be a reality there is nothing incredible in the world and from the present moment he will believe anything He says that it is a strange dumb show enacted not by puppets but by living beings It is stranger than a travellerrsquos tale Seeing such a thing

before his own eyes he will no longer disbelieve the story about unicorns and phoenix(e) How do the other characters present respond to this living drollery

At the sight of the lsquoliving drolleryrsquo like Sebastian Gonzalo and Antonio too acted strangely Antonio told that he too now believes in unicorns and phoenix and anything else that seems to be incredible He too now believes in travellersrsquo tales Gonzalo told that if he would report those happenings in Naples nobody will believe him He considers that those gentle shapes were gentler in manner in comparison to the living beings Alonso was at first sight suspicious and told them that those strange shapes conveyed their meaning in expressive gestures when they seemed to lack speech by their movements and sounds Francisco was amazed at their mysterious disappearance

2 ALONSO Not I

(Line 43-52)GONZALO Faith sir you need not fear When we

were boysWho would believe that there were mountaineers

Dewlapped like bulls whose throats had hanging at rsquoem

Wallets of flesh Or that there were such men

Whose heads stood in their breasts Which now we find

Each putter-out of five for one will bring us

Good warrant ofALONSO I will stand to and feed

Although my lastmdashno matter since I feel

The best is past Brother my lord the Duke

Stand to and do as we

(a) How does Alonso respond at the spectacle of the shapes which were sent to them at the instruction of Prospero

After seeing the strange sight of appearing and disappearing of the shapes sent by Prospero to arrange a banquet for them Alonso says that his surprise at having seen those creatures is infinite and he is fully justified in feeling so much surprise He thinks that their shapes their gestures and the sounds they made were indeed amazing Although they do not possess the gift of speech yet they were able to convey their

thoughts by means of their gestures only

(b) What does Prospero say about the views expressed by Alonso regarding the shapes What does Francisco think about the shapesAfter hearing Alonsorsquos views about the shapes Prospero says that this manrsquos praise of the spirits is rather hasty He means to say that Alonso has shown great haste in reaching the conclusion about the shapes Francisco is amazed to see that those shapes disappeared in a mysterious way(c) What does Sebastian ask Alonso to doSebastian tells Alonso that the shapes having disappeared should not matter to them because they have left the eatables behind He asks Alonso to enjoy eating as they are extremely hungry but the king does not accept his offer of enjoying the dishes(d) How does Gonzalo try to dispel Alonsorsquos fear of those strange shapes What kind of references does he give to AlonsoGonzalo says that those who have travelled abroad have reported seeing even stranger sights than these shapes that Alonso and his companions have beheld Hence there is no reason to feel afraid of these shapes Gonzalo further adds that in his younger days he had heard strange stories from travelers and Alonso might have heard similar stories For instance it was said that there existed a certain race of

human beings who had huge lumps of flesh hanging at their throats and who therefore resembled bulls Then Gonzalo tells about a race of human beings whose heads were located at their breasts Gonzalo says that such stories were not believed by most people in those days but now-a-days these stories have become common(e) Explain the following lsquoEach putter-out of five for onersquoEnglish travellers often insured their trips with London brokers Those that went on foreign travels those days used to deposit a certain amount with some firm or company in London before their departure If the travelers failed to return the money was forfeited by the company with which it had been deposited But this money was repaid five-fold if the travelers returned safe and sound In this way a traveler stood a great chance of recovering the entire cost of his

travels(f) Give the explanatory meanings of the following expressions in the context of the above extract (i) Dewlapped (ii) Wallets of flesh

(iii) Putter-out(i) Dewlapped having big lumps of flesh at the necks(ii) Wallets of flesh large masses of flesh looking like bags(iii) Putter-out to invest money before commencing the travel

  • General methods of preparation of hydrogen
  • Chapter Dimensional Analysis (Summary)
    • Properties of Charge
Page 43:   · Web viewSubject. Topic. Summary. Execution. Hindi. व्याकरण. शरीरके अंगो के नाम लिखिए. 1) आँख 2) नाक 3

were the other ships

(ii) Why was the ship in that area Where was it coming from or going where

(iii) The ship broke apart What happened to those who were in the ship

passengers on board must prepare for death which they will possibly soon have to meetThey can retire to their cabins and offer prayers to the Almighty to save them from drowning

(e) What does the boatswain say when he is asked to be patient What does he order to the royal party

When the boatswain is asked to be patient and remain calm he says that he will be patient only when the storm will be over and the sea will be calm but as long as the storm blows and there is danger to the ship he cannot think of being patient He orders the royal party to go to the cabin and leave the mariners to their work

(2) GONZALO I have great comfort from this fellow (Line 27-36)

Methinks he hath no drowning mark upon him his complexion is perfect

gallows Stand fast good Fate to his hanging Make the rope of his destiny our cable for our own doth little advantage If he be not born to be hanged our case is miserable

(a) Why does Gonzalo regard the Boatswain in the midst of danger

In the midst of danger Gonzalo regards the boatswain because he feels that the Boatswain is a source of comfort and is bent upon to do his work sincerely which in this case is saving the ship and its passengers from the severest of raging storm

(b) What reasons does Gonzalo give when he says that none in the ship will die of drowning

Gonzalo is almost sure that none in the ship will die by drowning His says that there is no mark on the face of the boatswain that indicates that he will die by drowning On the other hand the lines on his face are strong indications that he will be hanged to death Therefore there shall be no danger of the shiprsquos sinking

(c) Explain the following ldquoStand fast good Fate to his hanging Make the rope of his destiny our cable for our own doth little advantage If he be not born to be hanged our case is miserablerdquo

The stated lines mean that if the will of destiny is to be carried out then the ship will not get wrecked and all the passengers will be saved The safety of the passengers therefore depends upon the will of fate being carried out in the case of the boatswain If however the boatswain is not to die by hanging then the passengers are also very unsafe because in that case the ship is likely to sink

(d) What order does the Boatswain give to the sailors

when he re-enters What does he say about the crying of the fellows inside the cabin

The boatswain orders the sailors to bring the topmast lower and bring the ship close to a stationary position with the help of the main sail He says that the fellows inside the cabin are moaning and crying in their distress louder than his voice and louder even than the roaring of the storm

Class XII (ScienceCommerceHumanities) Subject Topic Summary Execution

Computer Science

PropositionalLogic

Propositional logic is a procedure to provide reasoning through statementProposition A ststement that results in True or False is said to be proposition There are two types of propositionSimple proposition amp compound propositionSimple proposioton A simple proposition is one that is not a part of any other proposition Such sentential form of proposition is symbolized with english letters in short For example Ram is a claver student (TrueFalse)Where do you live (Not in True or False)Grapes are sweet (TrueFalse)It rains today (TrueFalse)Here we can see some statements anwer would be true or false but some staements answer can not give in terms of true or false Thus the sentences which can be answered in true or false are known as simple propositionAssigning propositon to a variableThe general syntax to assign propostion to a variable is as followsVariable = Simple propositonFor example A=Ram is a clever studentB= Grapes are sweetC= it rains todayCompound proposition

helliphellipto be continued in next classhelliphellipMath Relation Relation If A and B are two non-empty sets

then a relation R from A to B is a subset of AxB If R A x B and (a b) R then we say that a sube isinis related to b by the relation R written as aRbeg Let A be the set of students of class XII and B be the set of students of class XI Then some of the examples of relation from A to B arei) (a b) AXB a is brother of bisinii) (a b) AXB age of a is more than age of isinb Types of relation In this section we would like to study different types of relations We know that a relation in a set A is a subset of A times A Thus the empty set φ and A times A are two extreme relations For illustration consider a relation R in the set A = 1 2 3 4 given by R = (a b) a ndash b = 10 This is the empty set as no pair (a b) satisfies the condition a ndash b = 10 Similarly R = (a b) | a ndash b | ge 0 is the whole primeset A times A as all pairs (a b) in A times A satisfy | a ndash

Example 1 Let A be the set of all students of a boys school Show that the relation R in A given by R = (a b) a is sister of b is the empty relation and R = (a b) the primedifference between heights of a and b is less than 3 meters is the universal relationSolution Since the school is boys school no student of the school can be sister of any student of the school Hence R = φ showing that R is the empty relation It is also obvious that the difference between heights of any two students of the school has to be less than 3 meters This shows that R = A times A is primethe universal relation Example 2 Show that the relation R in the set 1 2 3 given by R = (1 1) (2 2) (3 3) (1 2) (2 3) is reflexive

b | ge 0 These two extreme examples lead us to the following definitionsDefinition 1 A relation R in a set A is called empty relation if no element of A isrelated to any element of A ie R = φ A times AsubDefinition 2 A relation R in a set A is called universal relation if each element of A is related to every element of A ie R = A times A Both the empty relation and the universal relation are some times called trivial relation Definition 3 A relation R in a set A is called(i) reflexive if (a a) R for every a Aisin isin(ii) symmetric if (a1 a2) R implies that (aisin 2a1)

R for all aisin 1 a2 Aisin(iii) transitive if (a1 a2) R and (aisin 2 a3) R isinimplies that (a1 a3) R for all aisin 1 a2 a3 AisinDefinition 4 A relation R in a set A is said to be an equivalence relation if R is reflexive symmetric and transitive

but neither symmetric nor transitiveSolution R is reflexive since (1 1) (2 2) and (3 3) lie in R Also R is not symmetric as (1 2) R but (2 1) isin notinR Similarly R is not transitive as (1 2) R and (2 3) R but (1 3) R isin isin notinExample 3 Show that the relation R in the set Z of integers given byR = (a b) 2 divides a ndash b is an equivalence relationSolution R is reflexive as 2 divides (a ndash a) for all a Z isinFurther if (a b) R then 2 divides a isinndash b Therefore 2 divides b ndash a Hence (b a) R which shows that R is isinsymmetric Similarly if (a b) R and (b c) R isin isinthen a ndash b and b ndash c are divisible by 2 Now a ndash c = (a ndash b) + (b ndash c) is even (Why) So (a ndash c) is divisible by 2 This shows that R is transitive Thus R is an equivalence relation in ZExample 4 Let L be the set of all lines in a plane and R be the relation in L defined as R = (L1 L2) L1 is perpendicular to L2 Show that R is symmetric but neither reflexive nor transitiveSolution R is not reflexive as a line L1 can not be perpendicular to itself ie (L1 L1) R notinR is symmetric as (L1 L2) Risin

L1 is perpendicular to L2rArr L2 is perpendicular to L1rArr (L2 L1) RrArr isin

R is not transitive Indeed if L1 is perpendicular to L2 and L2 is perpendicular to L3 then L1 can never be perpendicular to L3 In fact L1 is parallel to L3 ie (L1 L2) R isin(L2 L3) R but (L1 L3) Risin notin

Chemistry Solid state Characteristics if Solids(i)The particles are locked in fixed positions they are unable to change their relative positions and this brings a definite shape and volume of a solid(ii)In a solid the constituent particles are held by strong forces of attractionThe forces of attraction may be bonding or non bonding(iii)The constituent particles in a solid pack together as closely as possibleoccupying most of the available space within the solidThus the empty space in a solid is very smallThis makes a solid highly rigid and nearly incompressibleThis also explains why a solid has high density and exhibits slow diffusionClassification of Solids

Q1)Define Crystalline solids AnsA Solid that has a definite geometrical shape and a sharp melting pointand whose constituent particles (atomsmolecules or ions) are arranged in a long range order of definite pattern extending throughout the solid is called a crystalline solidExNaClQ2)Define Amorphous solids AnsA solid that does not have a definite shape and a sharp melting pointand whose constituent particles (atomsmolecules or ions) are not arranged in a definite pattern is called an amorphoussolid

Crystalline solidsAmorphous solids

ExGlassRubberQ3)Classify Crystalline Solids Crystalline Solids

Physics Coloumbrsquos Law (Summary)

Before Going Into Coloumbrsquos Law We Will First Learn What is Charge Properties of Charge and Always remember that charge is quantized ie a body always have static charge of magnitude equal to some integral multiple of fundamental electronic charge e= 16 x 10- 19 C

Charge is the property of matter that causes it to produce and experience electrical and magnetic effects The study of the electrical charges at rest is called electrostatics When both electrical and magnetic effects are present the interaction between charges is referred to as electromagnetic

There exist two types of charges in nature positive and negative Like charges repel and unlike charges attract each other

The type of charge on an electron is negative The charge of a proton is the same as that of an electron but with a positive sign In an atom the number of electrons and the number of protons are equal The atom is therefore electrically neutral If one or more electrons are added to it it becomes negatively charged and is designated as negative ion However if one or more electrons are removed from an atom it becomes positively charged and is called a positive ion

The excess or deficiency of electrons in a body gives the concept of charge If there is an excess of electrons in a body it is negatively charged And if there is deficiency of electrons the body becomes positively charged Whenever addition or removal of electrons takes places the body acquires a charge

The SI Unit of charge is coulomb (C) In SI units the current is a fundamental quantity having a unit of ampere (A) The unit of charge is defined in terms of the unit of current Thus one coulomb is the charge transferred in one second across the section of a wire carrying a

Ionic SolidsMetallicSolids

Molecular Solids

current of one ampere

As q = It we have1 C = (1 A) (1 s)

The dimensions of charge are [A T]

Properties of Charge

(1) Quantization of Charge Electric charge can have only discrete values rather than any value That is charge is quantized The smallest discrete value of charge that can exist in nature is the charge on an electron given as

e = plusmn 16 x 10- 19 C

This is the charge attained by an electron and a protonA charge q must be an integral multiple of this basic unit That is

Q = plusmn ne where n = 1 2 hellip

Charge on a body can never be (frac12)e (23)e or 57e etcWhen we rub a glass rod with silk some electrons are transferred from the rod to the silk The rod becomes positively charged The silk becomes negatively charged The coulomb is a very large amount of charge A typical charge acquired by a rubbed body is 10 - 8 C

Biology Reproduction in organisms

Welcome to this new session 2020-21Today in this first chapter we mainly discuss about reproduction types needs and life span of some organismsWe also discuss about difference between sexual and asexual reproduction

Q1 What is reproductionReproduction is defined as a biological processin which an organism gives rise to young onessimilar to itselfQ2 What are the needs of reproductionbulli) Reproduction maintain life on earthii) It enables the continuity of the species generation after generationiii) It creates genetic variation among populationsQ3 Define Life span and write some orgnisms life spanbull Life span is the period from birth to

the natural death of an organism- OrganismsLife span1 Butterfly 1 - 2 weeks2 Fruit fly 30 days3Dog 10-13 years4 Rose5-7 years5 Tortoise100-150 years6 Banyan Tree -200 - 250 yearsQ4 Reproduction is of two types in case ofanimals but in case of plants vegetative propagation is also present

Asexual Reproduction Sexual Reproductioni) Always uniparentalii) Gametes are not involvediii) Only mitotic division involvediv) Somatic cells of parents are involvedv) Offsprings are genetically similar to the parents

i) Usually biparentalii) Gametes are involvediii) Meiosis occurs during gametogenesis Mitosis occurs after fertilisationiv) Germ cells of the parents are involvedv) offsprings are genetically different from the parents

COMMERCE BUSINESS ENVIRONMENT

Welcome to the new sessiontoday we are going to start the first chapter of Class XII The name of the chapter is Business Environment

Already many of you have got some idea about the word business environment form the first chapter of business studies in class XI

In todayrsquos world every business enterprise is a part of the society It exists and operates in association with various groups in society such as customers suppliers competitors banks and financial institutions government agencies trade unions media and so on All these groups influence the functioning of business in one way or the other They constitute the environment of businessConcept of Business Environment

The term lsquobusiness environmentrsquo refers to the sum total of all individuals institutions and other forces that lie outside a business enterprise but that may influence its functioning and performance

The main features of business environment Totality of External forces General and Specific forces Interrelatedness Complexity Dynamic Uncertainty

Prepare the following questions from todayrsquos assignment

2 What do you mean by business environment

The term lsquobusiness environmentrsquo means the aggregate of all forces factors and institutions which are external to and beyond the control of an individual business enterprise but they may influence its functioning and performance Business environment is the macro framework within which a business firm a micro unit operates It consists of several interrelated and interacting elements

2 Explain the main features of business environment in brief

Totality of External forces-Business environment is the sum total of all things external to a business environment

General and Specific forces-It

Relativity

The Interrelation between business and its environment

The business enterprise is an open system It continuously interacts with its environment It takes inputs (such as raw materials capital labour energy and so on) from its environment transforms them into goods and services and sends them back to the environment

Fig 1 Business Environment Relationship

includes both the forces general forces are the economic social political legal and technological conditions which indirectly influence all business enterprise Specific forces are the investors customers competitors and suppliers which influence individual enterprise directly

Interrelatedness-Different elements of environment are interrelated for an example growing awareness for health care has increased the demand for health foods

Complexity- Business environment id complex in nature as the elements keep on changing example economic technological and other forces changes in demand for a product and service

Dynamic-Business environment is not static it keeps on changing

Uncertainty- Itrsquos very difficult to predict future events such as technology and fashion which occur fast and frequently

Business Studies

Human Resources Management

Human resource of an organisation are the aggregate of knowledge skills attitudes of people working in it

The management system which deals with human resources is called human resource management

Features of HRMbullComprehensive functionbullPeople-oriented

Question1) What do you mean by human

resource management Answer) Human resource management may be defined as that field of Management which has to do with planning organising and controlling the functions of procuring developing maintaining and utilising the labour force

bullAction oriented bullPervasive function bullContinuous function

2) Explain the features of HRM in brief

Answer)bullHuman Resource Management is concerned with managing people at work bull Human Resource Management is concerned with employees which bring people and organisations together so that the goals of each are met bullHuman resource management considered every employees as an individual and also promote their satisfaction and growth bull Human resource management is inherent in all organisations and at all levelsbullManagement of human resources are ongoing on never ending process which requires a constant alertness and Awareness of human relations

3) ldquoHR function is said to be pervasiverdquowhy

Answer) Human resource management is required in all organisations whether it is private or government organisations armed forces sports organisations etc It permeatsall the functional areas like production marketing finance research etc This from this feature of human resource management it can be said that it is pervasive in nature

Economics Demand Q1DEFINITION OF DEMANDIn economics demand is the quantity of a good that consumers are willing and able to purchase at various prices during a given period of timeQ2DEMAND CURVEIn economics a demand curve is a graph depicting the relationship between the price of a certain commodity and the quantity of that commodity that is demanded at that pricQ3LAW OF DEMANDIn microeconomics the law of demand states that conditional on all else being equal as the price of a good increases quantity demanded decreases conversely as the price of a good decreases quantity demanded increasesQ4ASSUMPTION of LAW OF DEMAND(i)No change in price of related commodities(ii) No change in income of the consumer(iii) No change in taste and preferences customs habit and fashion of the consumer( No expectation regarding future change in priceQ5MARKET DEMAND SCHEDULEIn economics a market demand schedule is a tabulation of the quantity of a good that all consumers in a market will purchase at a

given price At any given price the corresponding value on the demand schedule is the sum of all consumersrsquo quantities demanded at that priceQ6INDIVIDUAL DEMAND SCHEDULEIndividual demand schedule refers to a tabular statement showing various quantities of a commodity that a consumer is willing to buy at various levels of price during a given period of timeQ7 FACTORS AFFECTING INDIVIDUAL DEMAND FOR A COMMODITY

The factors that influence a consumerrsquos decision to purchase a commodity are also known as determinants of demand The following factors affect the individual demand for a commodity1 price of the commodity2 price of related goods3 income of buyer of the commodity4 tastes and preferences of the buyer1 Price of the CommodityYou must have observed that when price of a commodity falls you tend to buy more of it and when its price rises you tend to buy less of it when all other factors remain constant (lsquoother things remaining the samersquo) In other words other things remaining the same there is an inverse relationship between the price of a commodity and its quantity demanded by its buyers This statement is in accordance with law of demand which you will study in the later part of this lesson Price of a commodity and its quantity demanded by its buyers are inversely related only when lsquoother things remain the samersquo So lsquoother things remaining the samersquo is an assumption when we study the effect of changes in the price of a commodity on its quantity demanded2 Price of Related goodsA consumer may demand a particular good But while buying that good heshe also asks the price of its related goods Related goods can be of two types-(i) Substitute goods(ii) Complementary goods While purchasing a good prices of its substitutes and complements do affect its quantity purchased(i) Price of Substitute Goods Substitute goods are those goods which can easily be used in place of one another for satisfaction of a particular want like tea and coffee An increase in price of substitute good leads to an increase in demand for the given commodity and a decrease in price of substitute good leads to a decrease in demand for the given commodity It means demand for a given commodity is directly affected by change in price of substitute goods For example if price of coffee increases the demand for tea will rise as tea will become relatively cheaper in comparison to coffee(ii) Price of Complementary goods Complementary goods are those goods which are used together to satisfy a particular want like car and petrol An increase in the price of complementary goods leads to a decrease in demand for the given commodity and a decrease in the price of complementary goods leads to an increase in demand for the given commodity For example if price of petrol falls then the demand for cars will increase as it will be relatively cheaper to use both the goods together So demand for a given commodity is inversely affected by change in price of complementary goods3 Income of the Buyer of CommodityDemand for a commodity is also affected by income of its buyer However the effect of change in income on demand depends on the nature of the commodity under consideration In case of some goods like full cream milk fine quality of rice (Basmati rice) etc demand for these commodities increases when income of the buyer increases and

demand for these commodities decreases when income of the buyer decreases Such goods whose demand increases with the increase in income of the buyer are called normal goods But there are some goods like coarse rice toned milk etc whose demand decreases when income of buyer increases and their demand increases when income of the buyer decreases Such goods whose demand decreases with the increase in income of the buyer are called inferior goods Suppose a consumer buys 10 Kgs of rice whose price is ` 25 per Kg He cannot afford to buy better quality of rice because the price of such rice is ` 50 per Kg The consumer is spending ` 250 per month on the purchase of rice Now if income of the consumer increases and he can afford ` 350 on purchase of 10 Kg of rice Now he can afford to buy some quantity of rice say 6 Kgs whose price is ` 25 per Kg and may buy 4 Kgs of rice whose price is ` 50 per Kg Thus he will buy 10 Kgs of rice by spending ` 350 per month Therefore we may conclude that demand for normal goods is directly related to the income of the buyer but demand for inferior goods is inversely related to the income of the buyer4 Tastes and Preferences of the BuyerThe demand for a commodity is also affected by the tastes and preferences of the buyers They include change in fashion customs habits etc Those commodities are preferred by the consumers which are in fashion So demand for those commodities rises which are in fashion On the other hand if a commodity goes out of the fashion its demand falls because no consumer will like to buy it(5) Number of Buyers in the Market(Population)Increase in population raises the market demand whereas decrease in population reduces the market demand for a commodity Not only the size of population but its composition like age (ratio of males females children and old people in population) also affects the demand for a commodity It is because of needs of children young old male and female population differs(6) Distribution of Income and WealthIf the distribution of income and wealth is more in favour of the rich demand for the commodities preferred by the rich such as comforts and luxuries is likely to be higher On the other hand if the distribution of income and wealth is more in favour of poor demand for commodities preferred by the poor such as necessities will be more(7) Season and Weather ConditionsThis is generally observed that the demand for woolens increases during winter whereas demand for ice creams and cold drinks increases during summer Similarly market demand for umbrellas rain coats increases during rainy seasonQ8 REASONS FOR OPERATION OF LAW OF DEMAND WHY DEMAND CURVE SLOPES DOWNWARDNow we will try to explain why does a consumer purchase more quantity of a commodity at a lower price and less of it at a higher price or why does the law of demand operate ie why does the demand curve slope downwards from left to right The main reasons for operation of law of demand are1 Law of Diminishing Marginal UtilityAs you have studied earlier law of diminishing marginal utility states that as we consume more and more units of a commodity the utility derived from each successive unit goes on decreasing The consumer will be ready to pay more for those units which provide him more utility and less for those which provide him less utility It implies that he will purchase more only when the price of the commodity falls2 Income Effect

When price of a commodity falls purchasing power or real income of the consumer increases which enables him to purchase more quantity of the commodity with the same money income Let us take an example Suppose you buy 4 ice creams when price of each ice cream is ` 25 If price of ice creams falls to ` 20 then with same money income you can buy 5 ice creams now3 Substitution EffectWhen price of a commodity falls it becomes comparatively cheaper as compared to its substitutes (although price of substitutes has not been changed) This will lead to rise in demand for the given commodity For example if coke and Pepsi both are sold at ` 10 each and price of coke falls Now coke has become relatively cheaper and will be substituted for Pepsi It will lead to rise in demand for coke4 Change in Number of BuyersWhen price of a commodity falls some old buyers may demand more of the commodity at the reduced price and some new buyers may also start buying this commodity who were not in a position to buy it earlier due to higher price This will lead to increase in number of buyers when price of the commodity falls As a result demand for the commodity rises when its price falls5 Diverse Uses of a CommoditySome commodities have diverse uses like milk It can be used for drinking for sweet preparation for ice cream preparation etc If price of milk rises its use may be restricted to important purpose only This will lead to reduction in demand for other less important uses When price of milk falls it can be put to other uses also leading to rise n demand for itQ9 EXCEPTIONS TO THE LAW OF DEMANDYou have studied in law of demand that a buyer is willing to buy more quantity of a commodity at a lower price and less of it at a higher price But in certain circumstances a rise in price may lead to rise in demand These circumstances are called Exceptions to the Law of Demand Some important exceptions are1 Giffen GoodsGiffen goods are special type of inferior goods in which negative income effect is stronger than negative substitution effect Giffen goods do not follow law of demand as their demand rises when their price rises Examples of Giffen goods are jowar and bajra etc2 Status Symbol GoodsSome goods are used by rich people as status symbols eg diamonds gold jewellary etc The higher the price the higher will be the demand for these goods When price of such goods falls these goods are no longer looked at as status symbol goods and tehrefore therir demand falls3 NecessitiesCommodities such as medicines salt wheat etc do not follow law of demandbecause we have to purchase them in minimum required quantity whatever their price may be4 Goods Expected to be ScarceWhen the buyers expect a scarcity of a particular good in near future they start buying more and more of that good even if their prices are rising For example during war famines etc people tend to buy more of some goods even at higher prices due to fear of their scarcity in near future

Political Science

Constitution of India-The

Preamble

The preamble-

Preamble-

The preamble is the most precious part of the constitution We the people of India having solemnly resolved to constitute India into a Sovereign Socialist Secular Democratic Republic and to secure to all its citizensA preamble is an introductory and expressionary statement in a document that explains the documents purpose and underlying philosophy When applied to the opening paragraphs of a statute it may recite historical facts pertinent to the subject of the statuteNature and purpose of the constitution-Purpose of the Constitution dictates permanent framework of the government to form a more perfect union to establish justice and ensure peace of thenationconstitution provide principles how the government can run itself following the rules and laws written in the constitution of each state keeps them balanced

Answer the following questions-

1 What is preambleA preamble is an introductory and expressionary statement in a document that explains the documents purpose and underlying philosophy2 What is the nature and

purpose of the constitutionConstitution dictatespermanent framework of the government to form a more perfect union to establish justice and ensure peace of the nation

Homework-Learn

Accounts Compatibilty mode

1MEANING OF PARTNERSHIPPartnership is a form of business organisation where two or more persons join hands to run a business They share the profits and losses according to the agreement amongst them According to the Indian Partnership Act 1932 ldquoPartnership is relation between persons who have agreed to share profits of a business carried on by all or any one of them acting for allrdquo For example one of your friends has passed class XII from National Institute of Open Schooling (NIOS) and wants to start a business Heshe approaches you to join in this venture Heshe wants you to contribute some money and participate in the business activities Both of you if join hands constitute a partnership2CHARACTERISTICS1048698 Agreement A partnership is formed by an agreement The agreement may be either oral or in writing It defines the relationship between the persons who agree to carry on business It may contain the terms of sharing profit and the capital to be invested by each partner etc The written agreement is known as partnership deed1048698 Number of persons There must be at least two persons to form a partnership

The maximum number of partners in a partnership firm can be 50 according toCompanies Act 20131048698 Business The Partnership is formed to carry on business with a purpose of earning profits The business should be lawful Thus if two or more persons agree to carry on unlawful activities it will not be termed as partnership1048698 Sharing Profits The partners agree to share profits in the agreed ratio In caseof loss all the partners have to bear it in the same agreed profit sharing ratio10486981048698Mutual Agency Every partner is an agent of the other partners Every partner can bind the firm and all other partners by hisher acts Each partner will be responsible and liable for the acts of all other partners10486981048698Unlimited liability The liability of each partner except that of a minor is unlimited Their liability extends to their personal assets also If the assets of the firm are insufficient to pay off its debts the partnersrsquo personal property can be used to satisfy the claim of the creditors of the partnership firm10486981048698Management All the partners have a right to mange the business However they may authorize one or more partners to manage the affairs of the business on their behalf10486981048698Transferability of Share No partner can transfer hisher share to any one including hisher family member without the consent of all other partners3PARTNERSHIP DEEDAgreement forms the basis of partnership The written form of the agreement is which a document of partnership is It contains terms and conditions regarding the conduct of the business It also explains relationship between the partners This document is called partnership deed Every firm can frame its own partnership deed in which the rights duties and liabilities of the partners are stated in detail It helps in settling the disputes arising among the partners during the general conduct of business 4CONTENTS OF PARTNERSHIP DEEDThe partnership deed generally contains the following (i) Name and address of the partnership firm(ii) Nature and objectives of the business(iii) Name and address of each partner(iv) Ratio in which profits is to be shared(v) Capital contribution by each partner(vi) Rate of Interest on capital if allowed(vii) Salary or any other remuneration to partners if allowed(viii) Rate of interest on loans and advances by a partner to the firm(ix) Drawings of partners and interest thereon if any(x) Method of valuation of goodwill and revaluation of assets and liabilities on the reconstitution of the partnership ie on the admission retirement or death of a partner(xi) Settlement of disputes by arbitration(xii) Settlement of accounts at the time of retirement or death of a partner5IN ABSENCE OF PARTNERSHIP DEEDThe partnership deed lays down the terms and conditions of partnership in regard to rights duties and obligations of the partners In the absence of partnership deed there may arise a controversy on certain issues like profit sharing ratio interest on

capital interest on drawings interest on loan and salary of the partners In such cases the provisions of the Indian Partnership Act becomes applicableSome of the Issues are(i) Distribution of Profit Partners are entitled to share profits equally(ii) Interest on Capital Interest on capital is not allowed(iii) Interest on Drawings No interest on drawing of the partners is to be charged(iv) Interest on Partnerrsquos Loan A Partner is allowed interest 6 per annum on the amount of loan given to the firm by himher(v) Salary and Commission to Partner A partner is not entitled to anysalary or commission or any other remuneration for managing the business

History TOPIC-TOWARDS INDEPENDENCE AND PARTITION THE LAST PHASE (1935-1947)

SUB TOPIC-IMPORTANT POLITICAL DEVELOPMENTS ndash GROWTH OF SOCIAL IDEAS

Socialism is a political social and economic philosophyLike in other parts of the world the Russian revolution of 1917 served as a great inspiration for revolutionaries in India who at that time were engaged in the struggle for liberation from British ruleSocialist ideas led to the formation of communist party of IndiaJAWAHARLAL NEHRU Among the early Congress leaders Jawaharlal Nehru was very much impressed and influenced by the Socialist ideas He also learnt about the Economic activities of the Soviet Union after the Bolshevic Revolution 1917 He made full use of them in IndiaThe election of Jawaharlal Nehru and Subhas Chandra Bose showed the Left wing tendency within CongressJawaharlal Nehru demanded economic freedom along with political freedom of the people in order to end the exploitation of masses

Nehrus working committee included three socialists leaders The Lucknow session was a landmark in the evolution of socialist ideas of the congressSUBHAS CHANDRA BOSE ndash Subhas Chandra Bose had socialist leaning Both Jawaharlal Nehru and Subhas Chandra Bose were known as leftist Congress men Later on National Congress divided into Leftist and rightist campCONGRESS SOCIALIST Within the Congress some leaders formed the Congress Socialist partyPattavi Sitaramyya Sardar Patel Rajendra Prasad had hostile attitude towards the Congress Socialist partyJawaharlals attitude was hesitant

1 QUESTION ndash Mention name of two Congress leaders who had socialist leaning

1ANSWER ndash Subhas Chandra Bose and Jawaharlal Nehru2QUESTION- In which session of the congress Jawaharlal elaborated his Socialist ideas2 ANSWER ndash Lucknow and Faizpur Session in December 1935 and 19363QUESTION ndash Why Congress was sharply divided into leftist and rightist camp 3ANSWER ndash Subhas Chandra Bosersquos attempt to seek re election for congress presidentship in 1939sharply divided the National Congress into Leftist and Rightist camp4 QUESTION ndash Who was MN Roy 4 ANSWER ndash Manabendra Roy first formed the Communist Party of India outside the country at Tashkent in 19205QUESTION ndash Who formed the Congress Socialist Party within the Congress5 ANSWER ndash Jaya Prakash Narayan Achyut Patwardhan Acharya Narendra Dev Ram Mohan Lohia Aruna Asaf Ali6QUESTION ndash When was the Congress Socialist Party formed What was its object6 ANSWER ndash 1934The Congress Socialist Party sought to work out socialist programme through the Congress They joined hands with the Congress and wanted to carry

Subhas Chandra Bose being expelled from the congress after the Tripuri rift he formed Forward BlockThere were basic differences between the Congress Socialists and the communistsTRADE UNION ACTIVITIES Maximum working class people lived in Bombay and Calcutta The working and living conditions of those workers were very miserable In this situation Shasipada Banerjee NM Lokhande protested against the oppression of the working class peopleThe first Trade Union Madras Labour Union was formed in 1918 by BP WadiaIndustrial strikes took place in Kanpur Calcutta Madras Jamshedpur and Ahmedabad AITUC was formed in Bombay in 1927 The growth of Trade union among the workers was slow because of the fear of the dismissal of the jobIn the mean time the Moderates as well as Communists left AITUC and formed separate organization

on National struggle with the help of workers and peasant class of the society7 QUESTION ndash What was the name of the party founded by Subhas Chandra Bose7 ANSWER- Forward Block8QUESTION ndash Who was Shasipada Banerjee8 ANSWER ndash Shasipada Banerjee was a radical Brahmo He founded a working menrsquos club to protest against exploitation of the British rulers towards the working class of India9 QUESTION ndash What was the weekly published by NM Lokhande9ANSWER- Dinabandhu10 QUESTION ndash Who founded Bombay Mill-Hands Association and in which year10 ANSWER- NM Lokhande in189011 QUESTION- Who was BP WadiaANSWER- BPWadia was the founder of Madras Labour Union in191812 QUESTION- What was the name of the first labour union of India12 ANSWER- Madras Labour Union13 QUESTION Who founded the Majur Mahajan 13 ANSWER GANDHIJI14 QUESTION What was the full form of AITUC When it was formed14 ANSWER All India Trade Union Congressin 192715QUESTION Who formed the Red Trade Union Congress and in which year15ANSWER The Communists formed the Red Trade Union Congress16 QUESTION What do you mean by Socialism16 ANSWER Socialism describes any political and economic theory that says the community rather than individuals should own and manage property and natural resources

Subject Eng Literature (The Tempest ndash William Shakespeare) Topic Act III Scene 3 Lines 1 to 52 (Line 52 ndash Brother my lord the Duke Stand to and do as we) Date 13th April 2020 (4th Period)

[Students should read the original play and also the paraphrase given in the school prescribed textbook]Summary Questions amp Answers

o Alonso Sebastian Antonio Gonzalo Adrian Francisco and others wandered about the island in search of Ferdinand and gets tired and hungry of the toil and at the same time gives up all hope of finding him

o Antonio and Sebastian are happy that Alonso is out of hope and decide to make another attempt on his life that night when being so tired they will be sleeping soundly

o Suddenly a solemn and strange music is heard in the air and several strange shapes enter bringing in a banquet These strange shapes then dance round it with gestures of salutation and then inviting the King to eat they depart

o Seeing this strange scene all are inclined to believe the tales told by travelers that there truly are ldquounicornsrdquo and ldquothe phoenixrsquo thronerdquo

1 ALONSO What harmony is this My good friends hark (L18-27)

GONZALO Marvellous sweet music

[Enter several strange shapes bringing in a banquet

they dance about it with gentle actions of salutation

and inviting the King and his companions to eat they depart]ALONSO Give us kind keepers heavens What were theseSEBASTIAN A living drollery Now I will believe

That there are unicorns that in Arabia

There is one tree the phoenixrsquo throne one phoenix

At this hour reigning thereANTONIO Ill believe both

And what does else want credit come to me

And Ill be sworn rsquotis true Travellers neer did lie

Though fools at home condemn rsquoem

(a) How did Prospero present an amazing spectacle before Alonso and his companions

Using his magic powers Prospero ordered strange shapes to lay a banquet before Alonso and his companions The shapes brought several dishes with tasty eatables in them They placed the dishes on a table before Alonso and his companions Then the strange shapes began to dance gracefully around the banquet While dancing they made gestures inviting them to eat the food Then suddenly the shapes disappeared(b) Who were the guests at the strange banquet Describe the lsquoliving drolleryrsquo

Alonso Sebastian Antonio Gonzalo Adrian and Francisco were the guests at the strange banquet

The term ldquoliving drolleryrdquo refers to live entertainment show In this context when Alonso the King of Naples Sebastian his brother Antonio the treacherous brother of Prospero Gonzalo the kind and loyal councillor to the King Adrian and Francisco came to the island they were hungry and weary in their spirits They heard a solemn and strange music They were shocked to see several strange shapes bringing in a banquet and these shapes danced about it with gentle action of salutation inviting the King and his companions to eat After this Sebastian described this show as lsquoliving drolleryrsquo(c) What is lsquophoenixrsquo What are lsquoUnicornsrdquo

The term lsquophoenixrsquo refers to a mythical Arabian bird which lived alone and perched on a solitary tree After one hundred years it expired in flames and rose again from its own ashes

lsquoUnicornsrsquo refers to the mythological four-footed beasts having horns in the centre of their foreheads When the horns are ground into powder the powder was believed to be

an aphrodisiac(d) How does Sebastian explain the puppet show OR Why does the speaker now believe in unicorns and phoenix

Sebastian finds several strange shapes bringing in the banquet They invite the king and his party for dinner and soon depart He tells that if such a strange sight can be a reality there is nothing incredible in the world and from the present moment he will believe anything He says that it is a strange dumb show enacted not by puppets but by living beings It is stranger than a travellerrsquos tale Seeing such a thing

before his own eyes he will no longer disbelieve the story about unicorns and phoenix(e) How do the other characters present respond to this living drollery

At the sight of the lsquoliving drolleryrsquo like Sebastian Gonzalo and Antonio too acted strangely Antonio told that he too now believes in unicorns and phoenix and anything else that seems to be incredible He too now believes in travellersrsquo tales Gonzalo told that if he would report those happenings in Naples nobody will believe him He considers that those gentle shapes were gentler in manner in comparison to the living beings Alonso was at first sight suspicious and told them that those strange shapes conveyed their meaning in expressive gestures when they seemed to lack speech by their movements and sounds Francisco was amazed at their mysterious disappearance

2 ALONSO Not I

(Line 43-52)GONZALO Faith sir you need not fear When we

were boysWho would believe that there were mountaineers

Dewlapped like bulls whose throats had hanging at rsquoem

Wallets of flesh Or that there were such men

Whose heads stood in their breasts Which now we find

Each putter-out of five for one will bring us

Good warrant ofALONSO I will stand to and feed

Although my lastmdashno matter since I feel

The best is past Brother my lord the Duke

Stand to and do as we

(a) How does Alonso respond at the spectacle of the shapes which were sent to them at the instruction of Prospero

After seeing the strange sight of appearing and disappearing of the shapes sent by Prospero to arrange a banquet for them Alonso says that his surprise at having seen those creatures is infinite and he is fully justified in feeling so much surprise He thinks that their shapes their gestures and the sounds they made were indeed amazing Although they do not possess the gift of speech yet they were able to convey their

thoughts by means of their gestures only

(b) What does Prospero say about the views expressed by Alonso regarding the shapes What does Francisco think about the shapesAfter hearing Alonsorsquos views about the shapes Prospero says that this manrsquos praise of the spirits is rather hasty He means to say that Alonso has shown great haste in reaching the conclusion about the shapes Francisco is amazed to see that those shapes disappeared in a mysterious way(c) What does Sebastian ask Alonso to doSebastian tells Alonso that the shapes having disappeared should not matter to them because they have left the eatables behind He asks Alonso to enjoy eating as they are extremely hungry but the king does not accept his offer of enjoying the dishes(d) How does Gonzalo try to dispel Alonsorsquos fear of those strange shapes What kind of references does he give to AlonsoGonzalo says that those who have travelled abroad have reported seeing even stranger sights than these shapes that Alonso and his companions have beheld Hence there is no reason to feel afraid of these shapes Gonzalo further adds that in his younger days he had heard strange stories from travelers and Alonso might have heard similar stories For instance it was said that there existed a certain race of

human beings who had huge lumps of flesh hanging at their throats and who therefore resembled bulls Then Gonzalo tells about a race of human beings whose heads were located at their breasts Gonzalo says that such stories were not believed by most people in those days but now-a-days these stories have become common(e) Explain the following lsquoEach putter-out of five for onersquoEnglish travellers often insured their trips with London brokers Those that went on foreign travels those days used to deposit a certain amount with some firm or company in London before their departure If the travelers failed to return the money was forfeited by the company with which it had been deposited But this money was repaid five-fold if the travelers returned safe and sound In this way a traveler stood a great chance of recovering the entire cost of his

travels(f) Give the explanatory meanings of the following expressions in the context of the above extract (i) Dewlapped (ii) Wallets of flesh

(iii) Putter-out(i) Dewlapped having big lumps of flesh at the necks(ii) Wallets of flesh large masses of flesh looking like bags(iii) Putter-out to invest money before commencing the travel

  • General methods of preparation of hydrogen
  • Chapter Dimensional Analysis (Summary)
    • Properties of Charge
Page 44:   · Web viewSubject. Topic. Summary. Execution. Hindi. व्याकरण. शरीरके अंगो के नाम लिखिए. 1) आँख 2) नाक 3

when he re-enters What does he say about the crying of the fellows inside the cabin

The boatswain orders the sailors to bring the topmast lower and bring the ship close to a stationary position with the help of the main sail He says that the fellows inside the cabin are moaning and crying in their distress louder than his voice and louder even than the roaring of the storm

Class XII (ScienceCommerceHumanities) Subject Topic Summary Execution

Computer Science

PropositionalLogic

Propositional logic is a procedure to provide reasoning through statementProposition A ststement that results in True or False is said to be proposition There are two types of propositionSimple proposition amp compound propositionSimple proposioton A simple proposition is one that is not a part of any other proposition Such sentential form of proposition is symbolized with english letters in short For example Ram is a claver student (TrueFalse)Where do you live (Not in True or False)Grapes are sweet (TrueFalse)It rains today (TrueFalse)Here we can see some statements anwer would be true or false but some staements answer can not give in terms of true or false Thus the sentences which can be answered in true or false are known as simple propositionAssigning propositon to a variableThe general syntax to assign propostion to a variable is as followsVariable = Simple propositonFor example A=Ram is a clever studentB= Grapes are sweetC= it rains todayCompound proposition

helliphellipto be continued in next classhelliphellipMath Relation Relation If A and B are two non-empty sets

then a relation R from A to B is a subset of AxB If R A x B and (a b) R then we say that a sube isinis related to b by the relation R written as aRbeg Let A be the set of students of class XII and B be the set of students of class XI Then some of the examples of relation from A to B arei) (a b) AXB a is brother of bisinii) (a b) AXB age of a is more than age of isinb Types of relation In this section we would like to study different types of relations We know that a relation in a set A is a subset of A times A Thus the empty set φ and A times A are two extreme relations For illustration consider a relation R in the set A = 1 2 3 4 given by R = (a b) a ndash b = 10 This is the empty set as no pair (a b) satisfies the condition a ndash b = 10 Similarly R = (a b) | a ndash b | ge 0 is the whole primeset A times A as all pairs (a b) in A times A satisfy | a ndash

Example 1 Let A be the set of all students of a boys school Show that the relation R in A given by R = (a b) a is sister of b is the empty relation and R = (a b) the primedifference between heights of a and b is less than 3 meters is the universal relationSolution Since the school is boys school no student of the school can be sister of any student of the school Hence R = φ showing that R is the empty relation It is also obvious that the difference between heights of any two students of the school has to be less than 3 meters This shows that R = A times A is primethe universal relation Example 2 Show that the relation R in the set 1 2 3 given by R = (1 1) (2 2) (3 3) (1 2) (2 3) is reflexive

b | ge 0 These two extreme examples lead us to the following definitionsDefinition 1 A relation R in a set A is called empty relation if no element of A isrelated to any element of A ie R = φ A times AsubDefinition 2 A relation R in a set A is called universal relation if each element of A is related to every element of A ie R = A times A Both the empty relation and the universal relation are some times called trivial relation Definition 3 A relation R in a set A is called(i) reflexive if (a a) R for every a Aisin isin(ii) symmetric if (a1 a2) R implies that (aisin 2a1)

R for all aisin 1 a2 Aisin(iii) transitive if (a1 a2) R and (aisin 2 a3) R isinimplies that (a1 a3) R for all aisin 1 a2 a3 AisinDefinition 4 A relation R in a set A is said to be an equivalence relation if R is reflexive symmetric and transitive

but neither symmetric nor transitiveSolution R is reflexive since (1 1) (2 2) and (3 3) lie in R Also R is not symmetric as (1 2) R but (2 1) isin notinR Similarly R is not transitive as (1 2) R and (2 3) R but (1 3) R isin isin notinExample 3 Show that the relation R in the set Z of integers given byR = (a b) 2 divides a ndash b is an equivalence relationSolution R is reflexive as 2 divides (a ndash a) for all a Z isinFurther if (a b) R then 2 divides a isinndash b Therefore 2 divides b ndash a Hence (b a) R which shows that R is isinsymmetric Similarly if (a b) R and (b c) R isin isinthen a ndash b and b ndash c are divisible by 2 Now a ndash c = (a ndash b) + (b ndash c) is even (Why) So (a ndash c) is divisible by 2 This shows that R is transitive Thus R is an equivalence relation in ZExample 4 Let L be the set of all lines in a plane and R be the relation in L defined as R = (L1 L2) L1 is perpendicular to L2 Show that R is symmetric but neither reflexive nor transitiveSolution R is not reflexive as a line L1 can not be perpendicular to itself ie (L1 L1) R notinR is symmetric as (L1 L2) Risin

L1 is perpendicular to L2rArr L2 is perpendicular to L1rArr (L2 L1) RrArr isin

R is not transitive Indeed if L1 is perpendicular to L2 and L2 is perpendicular to L3 then L1 can never be perpendicular to L3 In fact L1 is parallel to L3 ie (L1 L2) R isin(L2 L3) R but (L1 L3) Risin notin

Chemistry Solid state Characteristics if Solids(i)The particles are locked in fixed positions they are unable to change their relative positions and this brings a definite shape and volume of a solid(ii)In a solid the constituent particles are held by strong forces of attractionThe forces of attraction may be bonding or non bonding(iii)The constituent particles in a solid pack together as closely as possibleoccupying most of the available space within the solidThus the empty space in a solid is very smallThis makes a solid highly rigid and nearly incompressibleThis also explains why a solid has high density and exhibits slow diffusionClassification of Solids

Q1)Define Crystalline solids AnsA Solid that has a definite geometrical shape and a sharp melting pointand whose constituent particles (atomsmolecules or ions) are arranged in a long range order of definite pattern extending throughout the solid is called a crystalline solidExNaClQ2)Define Amorphous solids AnsA solid that does not have a definite shape and a sharp melting pointand whose constituent particles (atomsmolecules or ions) are not arranged in a definite pattern is called an amorphoussolid

Crystalline solidsAmorphous solids

ExGlassRubberQ3)Classify Crystalline Solids Crystalline Solids

Physics Coloumbrsquos Law (Summary)

Before Going Into Coloumbrsquos Law We Will First Learn What is Charge Properties of Charge and Always remember that charge is quantized ie a body always have static charge of magnitude equal to some integral multiple of fundamental electronic charge e= 16 x 10- 19 C

Charge is the property of matter that causes it to produce and experience electrical and magnetic effects The study of the electrical charges at rest is called electrostatics When both electrical and magnetic effects are present the interaction between charges is referred to as electromagnetic

There exist two types of charges in nature positive and negative Like charges repel and unlike charges attract each other

The type of charge on an electron is negative The charge of a proton is the same as that of an electron but with a positive sign In an atom the number of electrons and the number of protons are equal The atom is therefore electrically neutral If one or more electrons are added to it it becomes negatively charged and is designated as negative ion However if one or more electrons are removed from an atom it becomes positively charged and is called a positive ion

The excess or deficiency of electrons in a body gives the concept of charge If there is an excess of electrons in a body it is negatively charged And if there is deficiency of electrons the body becomes positively charged Whenever addition or removal of electrons takes places the body acquires a charge

The SI Unit of charge is coulomb (C) In SI units the current is a fundamental quantity having a unit of ampere (A) The unit of charge is defined in terms of the unit of current Thus one coulomb is the charge transferred in one second across the section of a wire carrying a

Ionic SolidsMetallicSolids

Molecular Solids

current of one ampere

As q = It we have1 C = (1 A) (1 s)

The dimensions of charge are [A T]

Properties of Charge

(1) Quantization of Charge Electric charge can have only discrete values rather than any value That is charge is quantized The smallest discrete value of charge that can exist in nature is the charge on an electron given as

e = plusmn 16 x 10- 19 C

This is the charge attained by an electron and a protonA charge q must be an integral multiple of this basic unit That is

Q = plusmn ne where n = 1 2 hellip

Charge on a body can never be (frac12)e (23)e or 57e etcWhen we rub a glass rod with silk some electrons are transferred from the rod to the silk The rod becomes positively charged The silk becomes negatively charged The coulomb is a very large amount of charge A typical charge acquired by a rubbed body is 10 - 8 C

Biology Reproduction in organisms

Welcome to this new session 2020-21Today in this first chapter we mainly discuss about reproduction types needs and life span of some organismsWe also discuss about difference between sexual and asexual reproduction

Q1 What is reproductionReproduction is defined as a biological processin which an organism gives rise to young onessimilar to itselfQ2 What are the needs of reproductionbulli) Reproduction maintain life on earthii) It enables the continuity of the species generation after generationiii) It creates genetic variation among populationsQ3 Define Life span and write some orgnisms life spanbull Life span is the period from birth to

the natural death of an organism- OrganismsLife span1 Butterfly 1 - 2 weeks2 Fruit fly 30 days3Dog 10-13 years4 Rose5-7 years5 Tortoise100-150 years6 Banyan Tree -200 - 250 yearsQ4 Reproduction is of two types in case ofanimals but in case of plants vegetative propagation is also present

Asexual Reproduction Sexual Reproductioni) Always uniparentalii) Gametes are not involvediii) Only mitotic division involvediv) Somatic cells of parents are involvedv) Offsprings are genetically similar to the parents

i) Usually biparentalii) Gametes are involvediii) Meiosis occurs during gametogenesis Mitosis occurs after fertilisationiv) Germ cells of the parents are involvedv) offsprings are genetically different from the parents

COMMERCE BUSINESS ENVIRONMENT

Welcome to the new sessiontoday we are going to start the first chapter of Class XII The name of the chapter is Business Environment

Already many of you have got some idea about the word business environment form the first chapter of business studies in class XI

In todayrsquos world every business enterprise is a part of the society It exists and operates in association with various groups in society such as customers suppliers competitors banks and financial institutions government agencies trade unions media and so on All these groups influence the functioning of business in one way or the other They constitute the environment of businessConcept of Business Environment

The term lsquobusiness environmentrsquo refers to the sum total of all individuals institutions and other forces that lie outside a business enterprise but that may influence its functioning and performance

The main features of business environment Totality of External forces General and Specific forces Interrelatedness Complexity Dynamic Uncertainty

Prepare the following questions from todayrsquos assignment

2 What do you mean by business environment

The term lsquobusiness environmentrsquo means the aggregate of all forces factors and institutions which are external to and beyond the control of an individual business enterprise but they may influence its functioning and performance Business environment is the macro framework within which a business firm a micro unit operates It consists of several interrelated and interacting elements

2 Explain the main features of business environment in brief

Totality of External forces-Business environment is the sum total of all things external to a business environment

General and Specific forces-It

Relativity

The Interrelation between business and its environment

The business enterprise is an open system It continuously interacts with its environment It takes inputs (such as raw materials capital labour energy and so on) from its environment transforms them into goods and services and sends them back to the environment

Fig 1 Business Environment Relationship

includes both the forces general forces are the economic social political legal and technological conditions which indirectly influence all business enterprise Specific forces are the investors customers competitors and suppliers which influence individual enterprise directly

Interrelatedness-Different elements of environment are interrelated for an example growing awareness for health care has increased the demand for health foods

Complexity- Business environment id complex in nature as the elements keep on changing example economic technological and other forces changes in demand for a product and service

Dynamic-Business environment is not static it keeps on changing

Uncertainty- Itrsquos very difficult to predict future events such as technology and fashion which occur fast and frequently

Business Studies

Human Resources Management

Human resource of an organisation are the aggregate of knowledge skills attitudes of people working in it

The management system which deals with human resources is called human resource management

Features of HRMbullComprehensive functionbullPeople-oriented

Question1) What do you mean by human

resource management Answer) Human resource management may be defined as that field of Management which has to do with planning organising and controlling the functions of procuring developing maintaining and utilising the labour force

bullAction oriented bullPervasive function bullContinuous function

2) Explain the features of HRM in brief

Answer)bullHuman Resource Management is concerned with managing people at work bull Human Resource Management is concerned with employees which bring people and organisations together so that the goals of each are met bullHuman resource management considered every employees as an individual and also promote their satisfaction and growth bull Human resource management is inherent in all organisations and at all levelsbullManagement of human resources are ongoing on never ending process which requires a constant alertness and Awareness of human relations

3) ldquoHR function is said to be pervasiverdquowhy

Answer) Human resource management is required in all organisations whether it is private or government organisations armed forces sports organisations etc It permeatsall the functional areas like production marketing finance research etc This from this feature of human resource management it can be said that it is pervasive in nature

Economics Demand Q1DEFINITION OF DEMANDIn economics demand is the quantity of a good that consumers are willing and able to purchase at various prices during a given period of timeQ2DEMAND CURVEIn economics a demand curve is a graph depicting the relationship between the price of a certain commodity and the quantity of that commodity that is demanded at that pricQ3LAW OF DEMANDIn microeconomics the law of demand states that conditional on all else being equal as the price of a good increases quantity demanded decreases conversely as the price of a good decreases quantity demanded increasesQ4ASSUMPTION of LAW OF DEMAND(i)No change in price of related commodities(ii) No change in income of the consumer(iii) No change in taste and preferences customs habit and fashion of the consumer( No expectation regarding future change in priceQ5MARKET DEMAND SCHEDULEIn economics a market demand schedule is a tabulation of the quantity of a good that all consumers in a market will purchase at a

given price At any given price the corresponding value on the demand schedule is the sum of all consumersrsquo quantities demanded at that priceQ6INDIVIDUAL DEMAND SCHEDULEIndividual demand schedule refers to a tabular statement showing various quantities of a commodity that a consumer is willing to buy at various levels of price during a given period of timeQ7 FACTORS AFFECTING INDIVIDUAL DEMAND FOR A COMMODITY

The factors that influence a consumerrsquos decision to purchase a commodity are also known as determinants of demand The following factors affect the individual demand for a commodity1 price of the commodity2 price of related goods3 income of buyer of the commodity4 tastes and preferences of the buyer1 Price of the CommodityYou must have observed that when price of a commodity falls you tend to buy more of it and when its price rises you tend to buy less of it when all other factors remain constant (lsquoother things remaining the samersquo) In other words other things remaining the same there is an inverse relationship between the price of a commodity and its quantity demanded by its buyers This statement is in accordance with law of demand which you will study in the later part of this lesson Price of a commodity and its quantity demanded by its buyers are inversely related only when lsquoother things remain the samersquo So lsquoother things remaining the samersquo is an assumption when we study the effect of changes in the price of a commodity on its quantity demanded2 Price of Related goodsA consumer may demand a particular good But while buying that good heshe also asks the price of its related goods Related goods can be of two types-(i) Substitute goods(ii) Complementary goods While purchasing a good prices of its substitutes and complements do affect its quantity purchased(i) Price of Substitute Goods Substitute goods are those goods which can easily be used in place of one another for satisfaction of a particular want like tea and coffee An increase in price of substitute good leads to an increase in demand for the given commodity and a decrease in price of substitute good leads to a decrease in demand for the given commodity It means demand for a given commodity is directly affected by change in price of substitute goods For example if price of coffee increases the demand for tea will rise as tea will become relatively cheaper in comparison to coffee(ii) Price of Complementary goods Complementary goods are those goods which are used together to satisfy a particular want like car and petrol An increase in the price of complementary goods leads to a decrease in demand for the given commodity and a decrease in the price of complementary goods leads to an increase in demand for the given commodity For example if price of petrol falls then the demand for cars will increase as it will be relatively cheaper to use both the goods together So demand for a given commodity is inversely affected by change in price of complementary goods3 Income of the Buyer of CommodityDemand for a commodity is also affected by income of its buyer However the effect of change in income on demand depends on the nature of the commodity under consideration In case of some goods like full cream milk fine quality of rice (Basmati rice) etc demand for these commodities increases when income of the buyer increases and

demand for these commodities decreases when income of the buyer decreases Such goods whose demand increases with the increase in income of the buyer are called normal goods But there are some goods like coarse rice toned milk etc whose demand decreases when income of buyer increases and their demand increases when income of the buyer decreases Such goods whose demand decreases with the increase in income of the buyer are called inferior goods Suppose a consumer buys 10 Kgs of rice whose price is ` 25 per Kg He cannot afford to buy better quality of rice because the price of such rice is ` 50 per Kg The consumer is spending ` 250 per month on the purchase of rice Now if income of the consumer increases and he can afford ` 350 on purchase of 10 Kg of rice Now he can afford to buy some quantity of rice say 6 Kgs whose price is ` 25 per Kg and may buy 4 Kgs of rice whose price is ` 50 per Kg Thus he will buy 10 Kgs of rice by spending ` 350 per month Therefore we may conclude that demand for normal goods is directly related to the income of the buyer but demand for inferior goods is inversely related to the income of the buyer4 Tastes and Preferences of the BuyerThe demand for a commodity is also affected by the tastes and preferences of the buyers They include change in fashion customs habits etc Those commodities are preferred by the consumers which are in fashion So demand for those commodities rises which are in fashion On the other hand if a commodity goes out of the fashion its demand falls because no consumer will like to buy it(5) Number of Buyers in the Market(Population)Increase in population raises the market demand whereas decrease in population reduces the market demand for a commodity Not only the size of population but its composition like age (ratio of males females children and old people in population) also affects the demand for a commodity It is because of needs of children young old male and female population differs(6) Distribution of Income and WealthIf the distribution of income and wealth is more in favour of the rich demand for the commodities preferred by the rich such as comforts and luxuries is likely to be higher On the other hand if the distribution of income and wealth is more in favour of poor demand for commodities preferred by the poor such as necessities will be more(7) Season and Weather ConditionsThis is generally observed that the demand for woolens increases during winter whereas demand for ice creams and cold drinks increases during summer Similarly market demand for umbrellas rain coats increases during rainy seasonQ8 REASONS FOR OPERATION OF LAW OF DEMAND WHY DEMAND CURVE SLOPES DOWNWARDNow we will try to explain why does a consumer purchase more quantity of a commodity at a lower price and less of it at a higher price or why does the law of demand operate ie why does the demand curve slope downwards from left to right The main reasons for operation of law of demand are1 Law of Diminishing Marginal UtilityAs you have studied earlier law of diminishing marginal utility states that as we consume more and more units of a commodity the utility derived from each successive unit goes on decreasing The consumer will be ready to pay more for those units which provide him more utility and less for those which provide him less utility It implies that he will purchase more only when the price of the commodity falls2 Income Effect

When price of a commodity falls purchasing power or real income of the consumer increases which enables him to purchase more quantity of the commodity with the same money income Let us take an example Suppose you buy 4 ice creams when price of each ice cream is ` 25 If price of ice creams falls to ` 20 then with same money income you can buy 5 ice creams now3 Substitution EffectWhen price of a commodity falls it becomes comparatively cheaper as compared to its substitutes (although price of substitutes has not been changed) This will lead to rise in demand for the given commodity For example if coke and Pepsi both are sold at ` 10 each and price of coke falls Now coke has become relatively cheaper and will be substituted for Pepsi It will lead to rise in demand for coke4 Change in Number of BuyersWhen price of a commodity falls some old buyers may demand more of the commodity at the reduced price and some new buyers may also start buying this commodity who were not in a position to buy it earlier due to higher price This will lead to increase in number of buyers when price of the commodity falls As a result demand for the commodity rises when its price falls5 Diverse Uses of a CommoditySome commodities have diverse uses like milk It can be used for drinking for sweet preparation for ice cream preparation etc If price of milk rises its use may be restricted to important purpose only This will lead to reduction in demand for other less important uses When price of milk falls it can be put to other uses also leading to rise n demand for itQ9 EXCEPTIONS TO THE LAW OF DEMANDYou have studied in law of demand that a buyer is willing to buy more quantity of a commodity at a lower price and less of it at a higher price But in certain circumstances a rise in price may lead to rise in demand These circumstances are called Exceptions to the Law of Demand Some important exceptions are1 Giffen GoodsGiffen goods are special type of inferior goods in which negative income effect is stronger than negative substitution effect Giffen goods do not follow law of demand as their demand rises when their price rises Examples of Giffen goods are jowar and bajra etc2 Status Symbol GoodsSome goods are used by rich people as status symbols eg diamonds gold jewellary etc The higher the price the higher will be the demand for these goods When price of such goods falls these goods are no longer looked at as status symbol goods and tehrefore therir demand falls3 NecessitiesCommodities such as medicines salt wheat etc do not follow law of demandbecause we have to purchase them in minimum required quantity whatever their price may be4 Goods Expected to be ScarceWhen the buyers expect a scarcity of a particular good in near future they start buying more and more of that good even if their prices are rising For example during war famines etc people tend to buy more of some goods even at higher prices due to fear of their scarcity in near future

Political Science

Constitution of India-The

Preamble

The preamble-

Preamble-

The preamble is the most precious part of the constitution We the people of India having solemnly resolved to constitute India into a Sovereign Socialist Secular Democratic Republic and to secure to all its citizensA preamble is an introductory and expressionary statement in a document that explains the documents purpose and underlying philosophy When applied to the opening paragraphs of a statute it may recite historical facts pertinent to the subject of the statuteNature and purpose of the constitution-Purpose of the Constitution dictates permanent framework of the government to form a more perfect union to establish justice and ensure peace of thenationconstitution provide principles how the government can run itself following the rules and laws written in the constitution of each state keeps them balanced

Answer the following questions-

1 What is preambleA preamble is an introductory and expressionary statement in a document that explains the documents purpose and underlying philosophy2 What is the nature and

purpose of the constitutionConstitution dictatespermanent framework of the government to form a more perfect union to establish justice and ensure peace of the nation

Homework-Learn

Accounts Compatibilty mode

1MEANING OF PARTNERSHIPPartnership is a form of business organisation where two or more persons join hands to run a business They share the profits and losses according to the agreement amongst them According to the Indian Partnership Act 1932 ldquoPartnership is relation between persons who have agreed to share profits of a business carried on by all or any one of them acting for allrdquo For example one of your friends has passed class XII from National Institute of Open Schooling (NIOS) and wants to start a business Heshe approaches you to join in this venture Heshe wants you to contribute some money and participate in the business activities Both of you if join hands constitute a partnership2CHARACTERISTICS1048698 Agreement A partnership is formed by an agreement The agreement may be either oral or in writing It defines the relationship between the persons who agree to carry on business It may contain the terms of sharing profit and the capital to be invested by each partner etc The written agreement is known as partnership deed1048698 Number of persons There must be at least two persons to form a partnership

The maximum number of partners in a partnership firm can be 50 according toCompanies Act 20131048698 Business The Partnership is formed to carry on business with a purpose of earning profits The business should be lawful Thus if two or more persons agree to carry on unlawful activities it will not be termed as partnership1048698 Sharing Profits The partners agree to share profits in the agreed ratio In caseof loss all the partners have to bear it in the same agreed profit sharing ratio10486981048698Mutual Agency Every partner is an agent of the other partners Every partner can bind the firm and all other partners by hisher acts Each partner will be responsible and liable for the acts of all other partners10486981048698Unlimited liability The liability of each partner except that of a minor is unlimited Their liability extends to their personal assets also If the assets of the firm are insufficient to pay off its debts the partnersrsquo personal property can be used to satisfy the claim of the creditors of the partnership firm10486981048698Management All the partners have a right to mange the business However they may authorize one or more partners to manage the affairs of the business on their behalf10486981048698Transferability of Share No partner can transfer hisher share to any one including hisher family member without the consent of all other partners3PARTNERSHIP DEEDAgreement forms the basis of partnership The written form of the agreement is which a document of partnership is It contains terms and conditions regarding the conduct of the business It also explains relationship between the partners This document is called partnership deed Every firm can frame its own partnership deed in which the rights duties and liabilities of the partners are stated in detail It helps in settling the disputes arising among the partners during the general conduct of business 4CONTENTS OF PARTNERSHIP DEEDThe partnership deed generally contains the following (i) Name and address of the partnership firm(ii) Nature and objectives of the business(iii) Name and address of each partner(iv) Ratio in which profits is to be shared(v) Capital contribution by each partner(vi) Rate of Interest on capital if allowed(vii) Salary or any other remuneration to partners if allowed(viii) Rate of interest on loans and advances by a partner to the firm(ix) Drawings of partners and interest thereon if any(x) Method of valuation of goodwill and revaluation of assets and liabilities on the reconstitution of the partnership ie on the admission retirement or death of a partner(xi) Settlement of disputes by arbitration(xii) Settlement of accounts at the time of retirement or death of a partner5IN ABSENCE OF PARTNERSHIP DEEDThe partnership deed lays down the terms and conditions of partnership in regard to rights duties and obligations of the partners In the absence of partnership deed there may arise a controversy on certain issues like profit sharing ratio interest on

capital interest on drawings interest on loan and salary of the partners In such cases the provisions of the Indian Partnership Act becomes applicableSome of the Issues are(i) Distribution of Profit Partners are entitled to share profits equally(ii) Interest on Capital Interest on capital is not allowed(iii) Interest on Drawings No interest on drawing of the partners is to be charged(iv) Interest on Partnerrsquos Loan A Partner is allowed interest 6 per annum on the amount of loan given to the firm by himher(v) Salary and Commission to Partner A partner is not entitled to anysalary or commission or any other remuneration for managing the business

History TOPIC-TOWARDS INDEPENDENCE AND PARTITION THE LAST PHASE (1935-1947)

SUB TOPIC-IMPORTANT POLITICAL DEVELOPMENTS ndash GROWTH OF SOCIAL IDEAS

Socialism is a political social and economic philosophyLike in other parts of the world the Russian revolution of 1917 served as a great inspiration for revolutionaries in India who at that time were engaged in the struggle for liberation from British ruleSocialist ideas led to the formation of communist party of IndiaJAWAHARLAL NEHRU Among the early Congress leaders Jawaharlal Nehru was very much impressed and influenced by the Socialist ideas He also learnt about the Economic activities of the Soviet Union after the Bolshevic Revolution 1917 He made full use of them in IndiaThe election of Jawaharlal Nehru and Subhas Chandra Bose showed the Left wing tendency within CongressJawaharlal Nehru demanded economic freedom along with political freedom of the people in order to end the exploitation of masses

Nehrus working committee included three socialists leaders The Lucknow session was a landmark in the evolution of socialist ideas of the congressSUBHAS CHANDRA BOSE ndash Subhas Chandra Bose had socialist leaning Both Jawaharlal Nehru and Subhas Chandra Bose were known as leftist Congress men Later on National Congress divided into Leftist and rightist campCONGRESS SOCIALIST Within the Congress some leaders formed the Congress Socialist partyPattavi Sitaramyya Sardar Patel Rajendra Prasad had hostile attitude towards the Congress Socialist partyJawaharlals attitude was hesitant

1 QUESTION ndash Mention name of two Congress leaders who had socialist leaning

1ANSWER ndash Subhas Chandra Bose and Jawaharlal Nehru2QUESTION- In which session of the congress Jawaharlal elaborated his Socialist ideas2 ANSWER ndash Lucknow and Faizpur Session in December 1935 and 19363QUESTION ndash Why Congress was sharply divided into leftist and rightist camp 3ANSWER ndash Subhas Chandra Bosersquos attempt to seek re election for congress presidentship in 1939sharply divided the National Congress into Leftist and Rightist camp4 QUESTION ndash Who was MN Roy 4 ANSWER ndash Manabendra Roy first formed the Communist Party of India outside the country at Tashkent in 19205QUESTION ndash Who formed the Congress Socialist Party within the Congress5 ANSWER ndash Jaya Prakash Narayan Achyut Patwardhan Acharya Narendra Dev Ram Mohan Lohia Aruna Asaf Ali6QUESTION ndash When was the Congress Socialist Party formed What was its object6 ANSWER ndash 1934The Congress Socialist Party sought to work out socialist programme through the Congress They joined hands with the Congress and wanted to carry

Subhas Chandra Bose being expelled from the congress after the Tripuri rift he formed Forward BlockThere were basic differences between the Congress Socialists and the communistsTRADE UNION ACTIVITIES Maximum working class people lived in Bombay and Calcutta The working and living conditions of those workers were very miserable In this situation Shasipada Banerjee NM Lokhande protested against the oppression of the working class peopleThe first Trade Union Madras Labour Union was formed in 1918 by BP WadiaIndustrial strikes took place in Kanpur Calcutta Madras Jamshedpur and Ahmedabad AITUC was formed in Bombay in 1927 The growth of Trade union among the workers was slow because of the fear of the dismissal of the jobIn the mean time the Moderates as well as Communists left AITUC and formed separate organization

on National struggle with the help of workers and peasant class of the society7 QUESTION ndash What was the name of the party founded by Subhas Chandra Bose7 ANSWER- Forward Block8QUESTION ndash Who was Shasipada Banerjee8 ANSWER ndash Shasipada Banerjee was a radical Brahmo He founded a working menrsquos club to protest against exploitation of the British rulers towards the working class of India9 QUESTION ndash What was the weekly published by NM Lokhande9ANSWER- Dinabandhu10 QUESTION ndash Who founded Bombay Mill-Hands Association and in which year10 ANSWER- NM Lokhande in189011 QUESTION- Who was BP WadiaANSWER- BPWadia was the founder of Madras Labour Union in191812 QUESTION- What was the name of the first labour union of India12 ANSWER- Madras Labour Union13 QUESTION Who founded the Majur Mahajan 13 ANSWER GANDHIJI14 QUESTION What was the full form of AITUC When it was formed14 ANSWER All India Trade Union Congressin 192715QUESTION Who formed the Red Trade Union Congress and in which year15ANSWER The Communists formed the Red Trade Union Congress16 QUESTION What do you mean by Socialism16 ANSWER Socialism describes any political and economic theory that says the community rather than individuals should own and manage property and natural resources

Subject Eng Literature (The Tempest ndash William Shakespeare) Topic Act III Scene 3 Lines 1 to 52 (Line 52 ndash Brother my lord the Duke Stand to and do as we) Date 13th April 2020 (4th Period)

[Students should read the original play and also the paraphrase given in the school prescribed textbook]Summary Questions amp Answers

o Alonso Sebastian Antonio Gonzalo Adrian Francisco and others wandered about the island in search of Ferdinand and gets tired and hungry of the toil and at the same time gives up all hope of finding him

o Antonio and Sebastian are happy that Alonso is out of hope and decide to make another attempt on his life that night when being so tired they will be sleeping soundly

o Suddenly a solemn and strange music is heard in the air and several strange shapes enter bringing in a banquet These strange shapes then dance round it with gestures of salutation and then inviting the King to eat they depart

o Seeing this strange scene all are inclined to believe the tales told by travelers that there truly are ldquounicornsrdquo and ldquothe phoenixrsquo thronerdquo

1 ALONSO What harmony is this My good friends hark (L18-27)

GONZALO Marvellous sweet music

[Enter several strange shapes bringing in a banquet

they dance about it with gentle actions of salutation

and inviting the King and his companions to eat they depart]ALONSO Give us kind keepers heavens What were theseSEBASTIAN A living drollery Now I will believe

That there are unicorns that in Arabia

There is one tree the phoenixrsquo throne one phoenix

At this hour reigning thereANTONIO Ill believe both

And what does else want credit come to me

And Ill be sworn rsquotis true Travellers neer did lie

Though fools at home condemn rsquoem

(a) How did Prospero present an amazing spectacle before Alonso and his companions

Using his magic powers Prospero ordered strange shapes to lay a banquet before Alonso and his companions The shapes brought several dishes with tasty eatables in them They placed the dishes on a table before Alonso and his companions Then the strange shapes began to dance gracefully around the banquet While dancing they made gestures inviting them to eat the food Then suddenly the shapes disappeared(b) Who were the guests at the strange banquet Describe the lsquoliving drolleryrsquo

Alonso Sebastian Antonio Gonzalo Adrian and Francisco were the guests at the strange banquet

The term ldquoliving drolleryrdquo refers to live entertainment show In this context when Alonso the King of Naples Sebastian his brother Antonio the treacherous brother of Prospero Gonzalo the kind and loyal councillor to the King Adrian and Francisco came to the island they were hungry and weary in their spirits They heard a solemn and strange music They were shocked to see several strange shapes bringing in a banquet and these shapes danced about it with gentle action of salutation inviting the King and his companions to eat After this Sebastian described this show as lsquoliving drolleryrsquo(c) What is lsquophoenixrsquo What are lsquoUnicornsrdquo

The term lsquophoenixrsquo refers to a mythical Arabian bird which lived alone and perched on a solitary tree After one hundred years it expired in flames and rose again from its own ashes

lsquoUnicornsrsquo refers to the mythological four-footed beasts having horns in the centre of their foreheads When the horns are ground into powder the powder was believed to be

an aphrodisiac(d) How does Sebastian explain the puppet show OR Why does the speaker now believe in unicorns and phoenix

Sebastian finds several strange shapes bringing in the banquet They invite the king and his party for dinner and soon depart He tells that if such a strange sight can be a reality there is nothing incredible in the world and from the present moment he will believe anything He says that it is a strange dumb show enacted not by puppets but by living beings It is stranger than a travellerrsquos tale Seeing such a thing

before his own eyes he will no longer disbelieve the story about unicorns and phoenix(e) How do the other characters present respond to this living drollery

At the sight of the lsquoliving drolleryrsquo like Sebastian Gonzalo and Antonio too acted strangely Antonio told that he too now believes in unicorns and phoenix and anything else that seems to be incredible He too now believes in travellersrsquo tales Gonzalo told that if he would report those happenings in Naples nobody will believe him He considers that those gentle shapes were gentler in manner in comparison to the living beings Alonso was at first sight suspicious and told them that those strange shapes conveyed their meaning in expressive gestures when they seemed to lack speech by their movements and sounds Francisco was amazed at their mysterious disappearance

2 ALONSO Not I

(Line 43-52)GONZALO Faith sir you need not fear When we

were boysWho would believe that there were mountaineers

Dewlapped like bulls whose throats had hanging at rsquoem

Wallets of flesh Or that there were such men

Whose heads stood in their breasts Which now we find

Each putter-out of five for one will bring us

Good warrant ofALONSO I will stand to and feed

Although my lastmdashno matter since I feel

The best is past Brother my lord the Duke

Stand to and do as we

(a) How does Alonso respond at the spectacle of the shapes which were sent to them at the instruction of Prospero

After seeing the strange sight of appearing and disappearing of the shapes sent by Prospero to arrange a banquet for them Alonso says that his surprise at having seen those creatures is infinite and he is fully justified in feeling so much surprise He thinks that their shapes their gestures and the sounds they made were indeed amazing Although they do not possess the gift of speech yet they were able to convey their

thoughts by means of their gestures only

(b) What does Prospero say about the views expressed by Alonso regarding the shapes What does Francisco think about the shapesAfter hearing Alonsorsquos views about the shapes Prospero says that this manrsquos praise of the spirits is rather hasty He means to say that Alonso has shown great haste in reaching the conclusion about the shapes Francisco is amazed to see that those shapes disappeared in a mysterious way(c) What does Sebastian ask Alonso to doSebastian tells Alonso that the shapes having disappeared should not matter to them because they have left the eatables behind He asks Alonso to enjoy eating as they are extremely hungry but the king does not accept his offer of enjoying the dishes(d) How does Gonzalo try to dispel Alonsorsquos fear of those strange shapes What kind of references does he give to AlonsoGonzalo says that those who have travelled abroad have reported seeing even stranger sights than these shapes that Alonso and his companions have beheld Hence there is no reason to feel afraid of these shapes Gonzalo further adds that in his younger days he had heard strange stories from travelers and Alonso might have heard similar stories For instance it was said that there existed a certain race of

human beings who had huge lumps of flesh hanging at their throats and who therefore resembled bulls Then Gonzalo tells about a race of human beings whose heads were located at their breasts Gonzalo says that such stories were not believed by most people in those days but now-a-days these stories have become common(e) Explain the following lsquoEach putter-out of five for onersquoEnglish travellers often insured their trips with London brokers Those that went on foreign travels those days used to deposit a certain amount with some firm or company in London before their departure If the travelers failed to return the money was forfeited by the company with which it had been deposited But this money was repaid five-fold if the travelers returned safe and sound In this way a traveler stood a great chance of recovering the entire cost of his

travels(f) Give the explanatory meanings of the following expressions in the context of the above extract (i) Dewlapped (ii) Wallets of flesh

(iii) Putter-out(i) Dewlapped having big lumps of flesh at the necks(ii) Wallets of flesh large masses of flesh looking like bags(iii) Putter-out to invest money before commencing the travel

  • General methods of preparation of hydrogen
  • Chapter Dimensional Analysis (Summary)
    • Properties of Charge
Page 45:   · Web viewSubject. Topic. Summary. Execution. Hindi. व्याकरण. शरीरके अंगो के नाम लिखिए. 1) आँख 2) नाक 3

b | ge 0 These two extreme examples lead us to the following definitionsDefinition 1 A relation R in a set A is called empty relation if no element of A isrelated to any element of A ie R = φ A times AsubDefinition 2 A relation R in a set A is called universal relation if each element of A is related to every element of A ie R = A times A Both the empty relation and the universal relation are some times called trivial relation Definition 3 A relation R in a set A is called(i) reflexive if (a a) R for every a Aisin isin(ii) symmetric if (a1 a2) R implies that (aisin 2a1)

R for all aisin 1 a2 Aisin(iii) transitive if (a1 a2) R and (aisin 2 a3) R isinimplies that (a1 a3) R for all aisin 1 a2 a3 AisinDefinition 4 A relation R in a set A is said to be an equivalence relation if R is reflexive symmetric and transitive

but neither symmetric nor transitiveSolution R is reflexive since (1 1) (2 2) and (3 3) lie in R Also R is not symmetric as (1 2) R but (2 1) isin notinR Similarly R is not transitive as (1 2) R and (2 3) R but (1 3) R isin isin notinExample 3 Show that the relation R in the set Z of integers given byR = (a b) 2 divides a ndash b is an equivalence relationSolution R is reflexive as 2 divides (a ndash a) for all a Z isinFurther if (a b) R then 2 divides a isinndash b Therefore 2 divides b ndash a Hence (b a) R which shows that R is isinsymmetric Similarly if (a b) R and (b c) R isin isinthen a ndash b and b ndash c are divisible by 2 Now a ndash c = (a ndash b) + (b ndash c) is even (Why) So (a ndash c) is divisible by 2 This shows that R is transitive Thus R is an equivalence relation in ZExample 4 Let L be the set of all lines in a plane and R be the relation in L defined as R = (L1 L2) L1 is perpendicular to L2 Show that R is symmetric but neither reflexive nor transitiveSolution R is not reflexive as a line L1 can not be perpendicular to itself ie (L1 L1) R notinR is symmetric as (L1 L2) Risin

L1 is perpendicular to L2rArr L2 is perpendicular to L1rArr (L2 L1) RrArr isin

R is not transitive Indeed if L1 is perpendicular to L2 and L2 is perpendicular to L3 then L1 can never be perpendicular to L3 In fact L1 is parallel to L3 ie (L1 L2) R isin(L2 L3) R but (L1 L3) Risin notin

Chemistry Solid state Characteristics if Solids(i)The particles are locked in fixed positions they are unable to change their relative positions and this brings a definite shape and volume of a solid(ii)In a solid the constituent particles are held by strong forces of attractionThe forces of attraction may be bonding or non bonding(iii)The constituent particles in a solid pack together as closely as possibleoccupying most of the available space within the solidThus the empty space in a solid is very smallThis makes a solid highly rigid and nearly incompressibleThis also explains why a solid has high density and exhibits slow diffusionClassification of Solids

Q1)Define Crystalline solids AnsA Solid that has a definite geometrical shape and a sharp melting pointand whose constituent particles (atomsmolecules or ions) are arranged in a long range order of definite pattern extending throughout the solid is called a crystalline solidExNaClQ2)Define Amorphous solids AnsA solid that does not have a definite shape and a sharp melting pointand whose constituent particles (atomsmolecules or ions) are not arranged in a definite pattern is called an amorphoussolid

Crystalline solidsAmorphous solids

ExGlassRubberQ3)Classify Crystalline Solids Crystalline Solids

Physics Coloumbrsquos Law (Summary)

Before Going Into Coloumbrsquos Law We Will First Learn What is Charge Properties of Charge and Always remember that charge is quantized ie a body always have static charge of magnitude equal to some integral multiple of fundamental electronic charge e= 16 x 10- 19 C

Charge is the property of matter that causes it to produce and experience electrical and magnetic effects The study of the electrical charges at rest is called electrostatics When both electrical and magnetic effects are present the interaction between charges is referred to as electromagnetic

There exist two types of charges in nature positive and negative Like charges repel and unlike charges attract each other

The type of charge on an electron is negative The charge of a proton is the same as that of an electron but with a positive sign In an atom the number of electrons and the number of protons are equal The atom is therefore electrically neutral If one or more electrons are added to it it becomes negatively charged and is designated as negative ion However if one or more electrons are removed from an atom it becomes positively charged and is called a positive ion

The excess or deficiency of electrons in a body gives the concept of charge If there is an excess of electrons in a body it is negatively charged And if there is deficiency of electrons the body becomes positively charged Whenever addition or removal of electrons takes places the body acquires a charge

The SI Unit of charge is coulomb (C) In SI units the current is a fundamental quantity having a unit of ampere (A) The unit of charge is defined in terms of the unit of current Thus one coulomb is the charge transferred in one second across the section of a wire carrying a

Ionic SolidsMetallicSolids

Molecular Solids

current of one ampere

As q = It we have1 C = (1 A) (1 s)

The dimensions of charge are [A T]

Properties of Charge

(1) Quantization of Charge Electric charge can have only discrete values rather than any value That is charge is quantized The smallest discrete value of charge that can exist in nature is the charge on an electron given as

e = plusmn 16 x 10- 19 C

This is the charge attained by an electron and a protonA charge q must be an integral multiple of this basic unit That is

Q = plusmn ne where n = 1 2 hellip

Charge on a body can never be (frac12)e (23)e or 57e etcWhen we rub a glass rod with silk some electrons are transferred from the rod to the silk The rod becomes positively charged The silk becomes negatively charged The coulomb is a very large amount of charge A typical charge acquired by a rubbed body is 10 - 8 C

Biology Reproduction in organisms

Welcome to this new session 2020-21Today in this first chapter we mainly discuss about reproduction types needs and life span of some organismsWe also discuss about difference between sexual and asexual reproduction

Q1 What is reproductionReproduction is defined as a biological processin which an organism gives rise to young onessimilar to itselfQ2 What are the needs of reproductionbulli) Reproduction maintain life on earthii) It enables the continuity of the species generation after generationiii) It creates genetic variation among populationsQ3 Define Life span and write some orgnisms life spanbull Life span is the period from birth to

the natural death of an organism- OrganismsLife span1 Butterfly 1 - 2 weeks2 Fruit fly 30 days3Dog 10-13 years4 Rose5-7 years5 Tortoise100-150 years6 Banyan Tree -200 - 250 yearsQ4 Reproduction is of two types in case ofanimals but in case of plants vegetative propagation is also present

Asexual Reproduction Sexual Reproductioni) Always uniparentalii) Gametes are not involvediii) Only mitotic division involvediv) Somatic cells of parents are involvedv) Offsprings are genetically similar to the parents

i) Usually biparentalii) Gametes are involvediii) Meiosis occurs during gametogenesis Mitosis occurs after fertilisationiv) Germ cells of the parents are involvedv) offsprings are genetically different from the parents

COMMERCE BUSINESS ENVIRONMENT

Welcome to the new sessiontoday we are going to start the first chapter of Class XII The name of the chapter is Business Environment

Already many of you have got some idea about the word business environment form the first chapter of business studies in class XI

In todayrsquos world every business enterprise is a part of the society It exists and operates in association with various groups in society such as customers suppliers competitors banks and financial institutions government agencies trade unions media and so on All these groups influence the functioning of business in one way or the other They constitute the environment of businessConcept of Business Environment

The term lsquobusiness environmentrsquo refers to the sum total of all individuals institutions and other forces that lie outside a business enterprise but that may influence its functioning and performance

The main features of business environment Totality of External forces General and Specific forces Interrelatedness Complexity Dynamic Uncertainty

Prepare the following questions from todayrsquos assignment

2 What do you mean by business environment

The term lsquobusiness environmentrsquo means the aggregate of all forces factors and institutions which are external to and beyond the control of an individual business enterprise but they may influence its functioning and performance Business environment is the macro framework within which a business firm a micro unit operates It consists of several interrelated and interacting elements

2 Explain the main features of business environment in brief

Totality of External forces-Business environment is the sum total of all things external to a business environment

General and Specific forces-It

Relativity

The Interrelation between business and its environment

The business enterprise is an open system It continuously interacts with its environment It takes inputs (such as raw materials capital labour energy and so on) from its environment transforms them into goods and services and sends them back to the environment

Fig 1 Business Environment Relationship

includes both the forces general forces are the economic social political legal and technological conditions which indirectly influence all business enterprise Specific forces are the investors customers competitors and suppliers which influence individual enterprise directly

Interrelatedness-Different elements of environment are interrelated for an example growing awareness for health care has increased the demand for health foods

Complexity- Business environment id complex in nature as the elements keep on changing example economic technological and other forces changes in demand for a product and service

Dynamic-Business environment is not static it keeps on changing

Uncertainty- Itrsquos very difficult to predict future events such as technology and fashion which occur fast and frequently

Business Studies

Human Resources Management

Human resource of an organisation are the aggregate of knowledge skills attitudes of people working in it

The management system which deals with human resources is called human resource management

Features of HRMbullComprehensive functionbullPeople-oriented

Question1) What do you mean by human

resource management Answer) Human resource management may be defined as that field of Management which has to do with planning organising and controlling the functions of procuring developing maintaining and utilising the labour force

bullAction oriented bullPervasive function bullContinuous function

2) Explain the features of HRM in brief

Answer)bullHuman Resource Management is concerned with managing people at work bull Human Resource Management is concerned with employees which bring people and organisations together so that the goals of each are met bullHuman resource management considered every employees as an individual and also promote their satisfaction and growth bull Human resource management is inherent in all organisations and at all levelsbullManagement of human resources are ongoing on never ending process which requires a constant alertness and Awareness of human relations

3) ldquoHR function is said to be pervasiverdquowhy

Answer) Human resource management is required in all organisations whether it is private or government organisations armed forces sports organisations etc It permeatsall the functional areas like production marketing finance research etc This from this feature of human resource management it can be said that it is pervasive in nature

Economics Demand Q1DEFINITION OF DEMANDIn economics demand is the quantity of a good that consumers are willing and able to purchase at various prices during a given period of timeQ2DEMAND CURVEIn economics a demand curve is a graph depicting the relationship between the price of a certain commodity and the quantity of that commodity that is demanded at that pricQ3LAW OF DEMANDIn microeconomics the law of demand states that conditional on all else being equal as the price of a good increases quantity demanded decreases conversely as the price of a good decreases quantity demanded increasesQ4ASSUMPTION of LAW OF DEMAND(i)No change in price of related commodities(ii) No change in income of the consumer(iii) No change in taste and preferences customs habit and fashion of the consumer( No expectation regarding future change in priceQ5MARKET DEMAND SCHEDULEIn economics a market demand schedule is a tabulation of the quantity of a good that all consumers in a market will purchase at a

given price At any given price the corresponding value on the demand schedule is the sum of all consumersrsquo quantities demanded at that priceQ6INDIVIDUAL DEMAND SCHEDULEIndividual demand schedule refers to a tabular statement showing various quantities of a commodity that a consumer is willing to buy at various levels of price during a given period of timeQ7 FACTORS AFFECTING INDIVIDUAL DEMAND FOR A COMMODITY

The factors that influence a consumerrsquos decision to purchase a commodity are also known as determinants of demand The following factors affect the individual demand for a commodity1 price of the commodity2 price of related goods3 income of buyer of the commodity4 tastes and preferences of the buyer1 Price of the CommodityYou must have observed that when price of a commodity falls you tend to buy more of it and when its price rises you tend to buy less of it when all other factors remain constant (lsquoother things remaining the samersquo) In other words other things remaining the same there is an inverse relationship between the price of a commodity and its quantity demanded by its buyers This statement is in accordance with law of demand which you will study in the later part of this lesson Price of a commodity and its quantity demanded by its buyers are inversely related only when lsquoother things remain the samersquo So lsquoother things remaining the samersquo is an assumption when we study the effect of changes in the price of a commodity on its quantity demanded2 Price of Related goodsA consumer may demand a particular good But while buying that good heshe also asks the price of its related goods Related goods can be of two types-(i) Substitute goods(ii) Complementary goods While purchasing a good prices of its substitutes and complements do affect its quantity purchased(i) Price of Substitute Goods Substitute goods are those goods which can easily be used in place of one another for satisfaction of a particular want like tea and coffee An increase in price of substitute good leads to an increase in demand for the given commodity and a decrease in price of substitute good leads to a decrease in demand for the given commodity It means demand for a given commodity is directly affected by change in price of substitute goods For example if price of coffee increases the demand for tea will rise as tea will become relatively cheaper in comparison to coffee(ii) Price of Complementary goods Complementary goods are those goods which are used together to satisfy a particular want like car and petrol An increase in the price of complementary goods leads to a decrease in demand for the given commodity and a decrease in the price of complementary goods leads to an increase in demand for the given commodity For example if price of petrol falls then the demand for cars will increase as it will be relatively cheaper to use both the goods together So demand for a given commodity is inversely affected by change in price of complementary goods3 Income of the Buyer of CommodityDemand for a commodity is also affected by income of its buyer However the effect of change in income on demand depends on the nature of the commodity under consideration In case of some goods like full cream milk fine quality of rice (Basmati rice) etc demand for these commodities increases when income of the buyer increases and

demand for these commodities decreases when income of the buyer decreases Such goods whose demand increases with the increase in income of the buyer are called normal goods But there are some goods like coarse rice toned milk etc whose demand decreases when income of buyer increases and their demand increases when income of the buyer decreases Such goods whose demand decreases with the increase in income of the buyer are called inferior goods Suppose a consumer buys 10 Kgs of rice whose price is ` 25 per Kg He cannot afford to buy better quality of rice because the price of such rice is ` 50 per Kg The consumer is spending ` 250 per month on the purchase of rice Now if income of the consumer increases and he can afford ` 350 on purchase of 10 Kg of rice Now he can afford to buy some quantity of rice say 6 Kgs whose price is ` 25 per Kg and may buy 4 Kgs of rice whose price is ` 50 per Kg Thus he will buy 10 Kgs of rice by spending ` 350 per month Therefore we may conclude that demand for normal goods is directly related to the income of the buyer but demand for inferior goods is inversely related to the income of the buyer4 Tastes and Preferences of the BuyerThe demand for a commodity is also affected by the tastes and preferences of the buyers They include change in fashion customs habits etc Those commodities are preferred by the consumers which are in fashion So demand for those commodities rises which are in fashion On the other hand if a commodity goes out of the fashion its demand falls because no consumer will like to buy it(5) Number of Buyers in the Market(Population)Increase in population raises the market demand whereas decrease in population reduces the market demand for a commodity Not only the size of population but its composition like age (ratio of males females children and old people in population) also affects the demand for a commodity It is because of needs of children young old male and female population differs(6) Distribution of Income and WealthIf the distribution of income and wealth is more in favour of the rich demand for the commodities preferred by the rich such as comforts and luxuries is likely to be higher On the other hand if the distribution of income and wealth is more in favour of poor demand for commodities preferred by the poor such as necessities will be more(7) Season and Weather ConditionsThis is generally observed that the demand for woolens increases during winter whereas demand for ice creams and cold drinks increases during summer Similarly market demand for umbrellas rain coats increases during rainy seasonQ8 REASONS FOR OPERATION OF LAW OF DEMAND WHY DEMAND CURVE SLOPES DOWNWARDNow we will try to explain why does a consumer purchase more quantity of a commodity at a lower price and less of it at a higher price or why does the law of demand operate ie why does the demand curve slope downwards from left to right The main reasons for operation of law of demand are1 Law of Diminishing Marginal UtilityAs you have studied earlier law of diminishing marginal utility states that as we consume more and more units of a commodity the utility derived from each successive unit goes on decreasing The consumer will be ready to pay more for those units which provide him more utility and less for those which provide him less utility It implies that he will purchase more only when the price of the commodity falls2 Income Effect

When price of a commodity falls purchasing power or real income of the consumer increases which enables him to purchase more quantity of the commodity with the same money income Let us take an example Suppose you buy 4 ice creams when price of each ice cream is ` 25 If price of ice creams falls to ` 20 then with same money income you can buy 5 ice creams now3 Substitution EffectWhen price of a commodity falls it becomes comparatively cheaper as compared to its substitutes (although price of substitutes has not been changed) This will lead to rise in demand for the given commodity For example if coke and Pepsi both are sold at ` 10 each and price of coke falls Now coke has become relatively cheaper and will be substituted for Pepsi It will lead to rise in demand for coke4 Change in Number of BuyersWhen price of a commodity falls some old buyers may demand more of the commodity at the reduced price and some new buyers may also start buying this commodity who were not in a position to buy it earlier due to higher price This will lead to increase in number of buyers when price of the commodity falls As a result demand for the commodity rises when its price falls5 Diverse Uses of a CommoditySome commodities have diverse uses like milk It can be used for drinking for sweet preparation for ice cream preparation etc If price of milk rises its use may be restricted to important purpose only This will lead to reduction in demand for other less important uses When price of milk falls it can be put to other uses also leading to rise n demand for itQ9 EXCEPTIONS TO THE LAW OF DEMANDYou have studied in law of demand that a buyer is willing to buy more quantity of a commodity at a lower price and less of it at a higher price But in certain circumstances a rise in price may lead to rise in demand These circumstances are called Exceptions to the Law of Demand Some important exceptions are1 Giffen GoodsGiffen goods are special type of inferior goods in which negative income effect is stronger than negative substitution effect Giffen goods do not follow law of demand as their demand rises when their price rises Examples of Giffen goods are jowar and bajra etc2 Status Symbol GoodsSome goods are used by rich people as status symbols eg diamonds gold jewellary etc The higher the price the higher will be the demand for these goods When price of such goods falls these goods are no longer looked at as status symbol goods and tehrefore therir demand falls3 NecessitiesCommodities such as medicines salt wheat etc do not follow law of demandbecause we have to purchase them in minimum required quantity whatever their price may be4 Goods Expected to be ScarceWhen the buyers expect a scarcity of a particular good in near future they start buying more and more of that good even if their prices are rising For example during war famines etc people tend to buy more of some goods even at higher prices due to fear of their scarcity in near future

Political Science

Constitution of India-The

Preamble

The preamble-

Preamble-

The preamble is the most precious part of the constitution We the people of India having solemnly resolved to constitute India into a Sovereign Socialist Secular Democratic Republic and to secure to all its citizensA preamble is an introductory and expressionary statement in a document that explains the documents purpose and underlying philosophy When applied to the opening paragraphs of a statute it may recite historical facts pertinent to the subject of the statuteNature and purpose of the constitution-Purpose of the Constitution dictates permanent framework of the government to form a more perfect union to establish justice and ensure peace of thenationconstitution provide principles how the government can run itself following the rules and laws written in the constitution of each state keeps them balanced

Answer the following questions-

1 What is preambleA preamble is an introductory and expressionary statement in a document that explains the documents purpose and underlying philosophy2 What is the nature and

purpose of the constitutionConstitution dictatespermanent framework of the government to form a more perfect union to establish justice and ensure peace of the nation

Homework-Learn

Accounts Compatibilty mode

1MEANING OF PARTNERSHIPPartnership is a form of business organisation where two or more persons join hands to run a business They share the profits and losses according to the agreement amongst them According to the Indian Partnership Act 1932 ldquoPartnership is relation between persons who have agreed to share profits of a business carried on by all or any one of them acting for allrdquo For example one of your friends has passed class XII from National Institute of Open Schooling (NIOS) and wants to start a business Heshe approaches you to join in this venture Heshe wants you to contribute some money and participate in the business activities Both of you if join hands constitute a partnership2CHARACTERISTICS1048698 Agreement A partnership is formed by an agreement The agreement may be either oral or in writing It defines the relationship between the persons who agree to carry on business It may contain the terms of sharing profit and the capital to be invested by each partner etc The written agreement is known as partnership deed1048698 Number of persons There must be at least two persons to form a partnership

The maximum number of partners in a partnership firm can be 50 according toCompanies Act 20131048698 Business The Partnership is formed to carry on business with a purpose of earning profits The business should be lawful Thus if two or more persons agree to carry on unlawful activities it will not be termed as partnership1048698 Sharing Profits The partners agree to share profits in the agreed ratio In caseof loss all the partners have to bear it in the same agreed profit sharing ratio10486981048698Mutual Agency Every partner is an agent of the other partners Every partner can bind the firm and all other partners by hisher acts Each partner will be responsible and liable for the acts of all other partners10486981048698Unlimited liability The liability of each partner except that of a minor is unlimited Their liability extends to their personal assets also If the assets of the firm are insufficient to pay off its debts the partnersrsquo personal property can be used to satisfy the claim of the creditors of the partnership firm10486981048698Management All the partners have a right to mange the business However they may authorize one or more partners to manage the affairs of the business on their behalf10486981048698Transferability of Share No partner can transfer hisher share to any one including hisher family member without the consent of all other partners3PARTNERSHIP DEEDAgreement forms the basis of partnership The written form of the agreement is which a document of partnership is It contains terms and conditions regarding the conduct of the business It also explains relationship between the partners This document is called partnership deed Every firm can frame its own partnership deed in which the rights duties and liabilities of the partners are stated in detail It helps in settling the disputes arising among the partners during the general conduct of business 4CONTENTS OF PARTNERSHIP DEEDThe partnership deed generally contains the following (i) Name and address of the partnership firm(ii) Nature and objectives of the business(iii) Name and address of each partner(iv) Ratio in which profits is to be shared(v) Capital contribution by each partner(vi) Rate of Interest on capital if allowed(vii) Salary or any other remuneration to partners if allowed(viii) Rate of interest on loans and advances by a partner to the firm(ix) Drawings of partners and interest thereon if any(x) Method of valuation of goodwill and revaluation of assets and liabilities on the reconstitution of the partnership ie on the admission retirement or death of a partner(xi) Settlement of disputes by arbitration(xii) Settlement of accounts at the time of retirement or death of a partner5IN ABSENCE OF PARTNERSHIP DEEDThe partnership deed lays down the terms and conditions of partnership in regard to rights duties and obligations of the partners In the absence of partnership deed there may arise a controversy on certain issues like profit sharing ratio interest on

capital interest on drawings interest on loan and salary of the partners In such cases the provisions of the Indian Partnership Act becomes applicableSome of the Issues are(i) Distribution of Profit Partners are entitled to share profits equally(ii) Interest on Capital Interest on capital is not allowed(iii) Interest on Drawings No interest on drawing of the partners is to be charged(iv) Interest on Partnerrsquos Loan A Partner is allowed interest 6 per annum on the amount of loan given to the firm by himher(v) Salary and Commission to Partner A partner is not entitled to anysalary or commission or any other remuneration for managing the business

History TOPIC-TOWARDS INDEPENDENCE AND PARTITION THE LAST PHASE (1935-1947)

SUB TOPIC-IMPORTANT POLITICAL DEVELOPMENTS ndash GROWTH OF SOCIAL IDEAS

Socialism is a political social and economic philosophyLike in other parts of the world the Russian revolution of 1917 served as a great inspiration for revolutionaries in India who at that time were engaged in the struggle for liberation from British ruleSocialist ideas led to the formation of communist party of IndiaJAWAHARLAL NEHRU Among the early Congress leaders Jawaharlal Nehru was very much impressed and influenced by the Socialist ideas He also learnt about the Economic activities of the Soviet Union after the Bolshevic Revolution 1917 He made full use of them in IndiaThe election of Jawaharlal Nehru and Subhas Chandra Bose showed the Left wing tendency within CongressJawaharlal Nehru demanded economic freedom along with political freedom of the people in order to end the exploitation of masses

Nehrus working committee included three socialists leaders The Lucknow session was a landmark in the evolution of socialist ideas of the congressSUBHAS CHANDRA BOSE ndash Subhas Chandra Bose had socialist leaning Both Jawaharlal Nehru and Subhas Chandra Bose were known as leftist Congress men Later on National Congress divided into Leftist and rightist campCONGRESS SOCIALIST Within the Congress some leaders formed the Congress Socialist partyPattavi Sitaramyya Sardar Patel Rajendra Prasad had hostile attitude towards the Congress Socialist partyJawaharlals attitude was hesitant

1 QUESTION ndash Mention name of two Congress leaders who had socialist leaning

1ANSWER ndash Subhas Chandra Bose and Jawaharlal Nehru2QUESTION- In which session of the congress Jawaharlal elaborated his Socialist ideas2 ANSWER ndash Lucknow and Faizpur Session in December 1935 and 19363QUESTION ndash Why Congress was sharply divided into leftist and rightist camp 3ANSWER ndash Subhas Chandra Bosersquos attempt to seek re election for congress presidentship in 1939sharply divided the National Congress into Leftist and Rightist camp4 QUESTION ndash Who was MN Roy 4 ANSWER ndash Manabendra Roy first formed the Communist Party of India outside the country at Tashkent in 19205QUESTION ndash Who formed the Congress Socialist Party within the Congress5 ANSWER ndash Jaya Prakash Narayan Achyut Patwardhan Acharya Narendra Dev Ram Mohan Lohia Aruna Asaf Ali6QUESTION ndash When was the Congress Socialist Party formed What was its object6 ANSWER ndash 1934The Congress Socialist Party sought to work out socialist programme through the Congress They joined hands with the Congress and wanted to carry

Subhas Chandra Bose being expelled from the congress after the Tripuri rift he formed Forward BlockThere were basic differences between the Congress Socialists and the communistsTRADE UNION ACTIVITIES Maximum working class people lived in Bombay and Calcutta The working and living conditions of those workers were very miserable In this situation Shasipada Banerjee NM Lokhande protested against the oppression of the working class peopleThe first Trade Union Madras Labour Union was formed in 1918 by BP WadiaIndustrial strikes took place in Kanpur Calcutta Madras Jamshedpur and Ahmedabad AITUC was formed in Bombay in 1927 The growth of Trade union among the workers was slow because of the fear of the dismissal of the jobIn the mean time the Moderates as well as Communists left AITUC and formed separate organization

on National struggle with the help of workers and peasant class of the society7 QUESTION ndash What was the name of the party founded by Subhas Chandra Bose7 ANSWER- Forward Block8QUESTION ndash Who was Shasipada Banerjee8 ANSWER ndash Shasipada Banerjee was a radical Brahmo He founded a working menrsquos club to protest against exploitation of the British rulers towards the working class of India9 QUESTION ndash What was the weekly published by NM Lokhande9ANSWER- Dinabandhu10 QUESTION ndash Who founded Bombay Mill-Hands Association and in which year10 ANSWER- NM Lokhande in189011 QUESTION- Who was BP WadiaANSWER- BPWadia was the founder of Madras Labour Union in191812 QUESTION- What was the name of the first labour union of India12 ANSWER- Madras Labour Union13 QUESTION Who founded the Majur Mahajan 13 ANSWER GANDHIJI14 QUESTION What was the full form of AITUC When it was formed14 ANSWER All India Trade Union Congressin 192715QUESTION Who formed the Red Trade Union Congress and in which year15ANSWER The Communists formed the Red Trade Union Congress16 QUESTION What do you mean by Socialism16 ANSWER Socialism describes any political and economic theory that says the community rather than individuals should own and manage property and natural resources

Subject Eng Literature (The Tempest ndash William Shakespeare) Topic Act III Scene 3 Lines 1 to 52 (Line 52 ndash Brother my lord the Duke Stand to and do as we) Date 13th April 2020 (4th Period)

[Students should read the original play and also the paraphrase given in the school prescribed textbook]Summary Questions amp Answers

o Alonso Sebastian Antonio Gonzalo Adrian Francisco and others wandered about the island in search of Ferdinand and gets tired and hungry of the toil and at the same time gives up all hope of finding him

o Antonio and Sebastian are happy that Alonso is out of hope and decide to make another attempt on his life that night when being so tired they will be sleeping soundly

o Suddenly a solemn and strange music is heard in the air and several strange shapes enter bringing in a banquet These strange shapes then dance round it with gestures of salutation and then inviting the King to eat they depart

o Seeing this strange scene all are inclined to believe the tales told by travelers that there truly are ldquounicornsrdquo and ldquothe phoenixrsquo thronerdquo

1 ALONSO What harmony is this My good friends hark (L18-27)

GONZALO Marvellous sweet music

[Enter several strange shapes bringing in a banquet

they dance about it with gentle actions of salutation

and inviting the King and his companions to eat they depart]ALONSO Give us kind keepers heavens What were theseSEBASTIAN A living drollery Now I will believe

That there are unicorns that in Arabia

There is one tree the phoenixrsquo throne one phoenix

At this hour reigning thereANTONIO Ill believe both

And what does else want credit come to me

And Ill be sworn rsquotis true Travellers neer did lie

Though fools at home condemn rsquoem

(a) How did Prospero present an amazing spectacle before Alonso and his companions

Using his magic powers Prospero ordered strange shapes to lay a banquet before Alonso and his companions The shapes brought several dishes with tasty eatables in them They placed the dishes on a table before Alonso and his companions Then the strange shapes began to dance gracefully around the banquet While dancing they made gestures inviting them to eat the food Then suddenly the shapes disappeared(b) Who were the guests at the strange banquet Describe the lsquoliving drolleryrsquo

Alonso Sebastian Antonio Gonzalo Adrian and Francisco were the guests at the strange banquet

The term ldquoliving drolleryrdquo refers to live entertainment show In this context when Alonso the King of Naples Sebastian his brother Antonio the treacherous brother of Prospero Gonzalo the kind and loyal councillor to the King Adrian and Francisco came to the island they were hungry and weary in their spirits They heard a solemn and strange music They were shocked to see several strange shapes bringing in a banquet and these shapes danced about it with gentle action of salutation inviting the King and his companions to eat After this Sebastian described this show as lsquoliving drolleryrsquo(c) What is lsquophoenixrsquo What are lsquoUnicornsrdquo

The term lsquophoenixrsquo refers to a mythical Arabian bird which lived alone and perched on a solitary tree After one hundred years it expired in flames and rose again from its own ashes

lsquoUnicornsrsquo refers to the mythological four-footed beasts having horns in the centre of their foreheads When the horns are ground into powder the powder was believed to be

an aphrodisiac(d) How does Sebastian explain the puppet show OR Why does the speaker now believe in unicorns and phoenix

Sebastian finds several strange shapes bringing in the banquet They invite the king and his party for dinner and soon depart He tells that if such a strange sight can be a reality there is nothing incredible in the world and from the present moment he will believe anything He says that it is a strange dumb show enacted not by puppets but by living beings It is stranger than a travellerrsquos tale Seeing such a thing

before his own eyes he will no longer disbelieve the story about unicorns and phoenix(e) How do the other characters present respond to this living drollery

At the sight of the lsquoliving drolleryrsquo like Sebastian Gonzalo and Antonio too acted strangely Antonio told that he too now believes in unicorns and phoenix and anything else that seems to be incredible He too now believes in travellersrsquo tales Gonzalo told that if he would report those happenings in Naples nobody will believe him He considers that those gentle shapes were gentler in manner in comparison to the living beings Alonso was at first sight suspicious and told them that those strange shapes conveyed their meaning in expressive gestures when they seemed to lack speech by their movements and sounds Francisco was amazed at their mysterious disappearance

2 ALONSO Not I

(Line 43-52)GONZALO Faith sir you need not fear When we

were boysWho would believe that there were mountaineers

Dewlapped like bulls whose throats had hanging at rsquoem

Wallets of flesh Or that there were such men

Whose heads stood in their breasts Which now we find

Each putter-out of five for one will bring us

Good warrant ofALONSO I will stand to and feed

Although my lastmdashno matter since I feel

The best is past Brother my lord the Duke

Stand to and do as we

(a) How does Alonso respond at the spectacle of the shapes which were sent to them at the instruction of Prospero

After seeing the strange sight of appearing and disappearing of the shapes sent by Prospero to arrange a banquet for them Alonso says that his surprise at having seen those creatures is infinite and he is fully justified in feeling so much surprise He thinks that their shapes their gestures and the sounds they made were indeed amazing Although they do not possess the gift of speech yet they were able to convey their

thoughts by means of their gestures only

(b) What does Prospero say about the views expressed by Alonso regarding the shapes What does Francisco think about the shapesAfter hearing Alonsorsquos views about the shapes Prospero says that this manrsquos praise of the spirits is rather hasty He means to say that Alonso has shown great haste in reaching the conclusion about the shapes Francisco is amazed to see that those shapes disappeared in a mysterious way(c) What does Sebastian ask Alonso to doSebastian tells Alonso that the shapes having disappeared should not matter to them because they have left the eatables behind He asks Alonso to enjoy eating as they are extremely hungry but the king does not accept his offer of enjoying the dishes(d) How does Gonzalo try to dispel Alonsorsquos fear of those strange shapes What kind of references does he give to AlonsoGonzalo says that those who have travelled abroad have reported seeing even stranger sights than these shapes that Alonso and his companions have beheld Hence there is no reason to feel afraid of these shapes Gonzalo further adds that in his younger days he had heard strange stories from travelers and Alonso might have heard similar stories For instance it was said that there existed a certain race of

human beings who had huge lumps of flesh hanging at their throats and who therefore resembled bulls Then Gonzalo tells about a race of human beings whose heads were located at their breasts Gonzalo says that such stories were not believed by most people in those days but now-a-days these stories have become common(e) Explain the following lsquoEach putter-out of five for onersquoEnglish travellers often insured their trips with London brokers Those that went on foreign travels those days used to deposit a certain amount with some firm or company in London before their departure If the travelers failed to return the money was forfeited by the company with which it had been deposited But this money was repaid five-fold if the travelers returned safe and sound In this way a traveler stood a great chance of recovering the entire cost of his

travels(f) Give the explanatory meanings of the following expressions in the context of the above extract (i) Dewlapped (ii) Wallets of flesh

(iii) Putter-out(i) Dewlapped having big lumps of flesh at the necks(ii) Wallets of flesh large masses of flesh looking like bags(iii) Putter-out to invest money before commencing the travel

  • General methods of preparation of hydrogen
  • Chapter Dimensional Analysis (Summary)
    • Properties of Charge
Page 46:   · Web viewSubject. Topic. Summary. Execution. Hindi. व्याकरण. शरीरके अंगो के नाम लिखिए. 1) आँख 2) नाक 3

Crystalline solidsAmorphous solids

ExGlassRubberQ3)Classify Crystalline Solids Crystalline Solids

Physics Coloumbrsquos Law (Summary)

Before Going Into Coloumbrsquos Law We Will First Learn What is Charge Properties of Charge and Always remember that charge is quantized ie a body always have static charge of magnitude equal to some integral multiple of fundamental electronic charge e= 16 x 10- 19 C

Charge is the property of matter that causes it to produce and experience electrical and magnetic effects The study of the electrical charges at rest is called electrostatics When both electrical and magnetic effects are present the interaction between charges is referred to as electromagnetic

There exist two types of charges in nature positive and negative Like charges repel and unlike charges attract each other

The type of charge on an electron is negative The charge of a proton is the same as that of an electron but with a positive sign In an atom the number of electrons and the number of protons are equal The atom is therefore electrically neutral If one or more electrons are added to it it becomes negatively charged and is designated as negative ion However if one or more electrons are removed from an atom it becomes positively charged and is called a positive ion

The excess or deficiency of electrons in a body gives the concept of charge If there is an excess of electrons in a body it is negatively charged And if there is deficiency of electrons the body becomes positively charged Whenever addition or removal of electrons takes places the body acquires a charge

The SI Unit of charge is coulomb (C) In SI units the current is a fundamental quantity having a unit of ampere (A) The unit of charge is defined in terms of the unit of current Thus one coulomb is the charge transferred in one second across the section of a wire carrying a

Ionic SolidsMetallicSolids

Molecular Solids

current of one ampere

As q = It we have1 C = (1 A) (1 s)

The dimensions of charge are [A T]

Properties of Charge

(1) Quantization of Charge Electric charge can have only discrete values rather than any value That is charge is quantized The smallest discrete value of charge that can exist in nature is the charge on an electron given as

e = plusmn 16 x 10- 19 C

This is the charge attained by an electron and a protonA charge q must be an integral multiple of this basic unit That is

Q = plusmn ne where n = 1 2 hellip

Charge on a body can never be (frac12)e (23)e or 57e etcWhen we rub a glass rod with silk some electrons are transferred from the rod to the silk The rod becomes positively charged The silk becomes negatively charged The coulomb is a very large amount of charge A typical charge acquired by a rubbed body is 10 - 8 C

Biology Reproduction in organisms

Welcome to this new session 2020-21Today in this first chapter we mainly discuss about reproduction types needs and life span of some organismsWe also discuss about difference between sexual and asexual reproduction

Q1 What is reproductionReproduction is defined as a biological processin which an organism gives rise to young onessimilar to itselfQ2 What are the needs of reproductionbulli) Reproduction maintain life on earthii) It enables the continuity of the species generation after generationiii) It creates genetic variation among populationsQ3 Define Life span and write some orgnisms life spanbull Life span is the period from birth to

the natural death of an organism- OrganismsLife span1 Butterfly 1 - 2 weeks2 Fruit fly 30 days3Dog 10-13 years4 Rose5-7 years5 Tortoise100-150 years6 Banyan Tree -200 - 250 yearsQ4 Reproduction is of two types in case ofanimals but in case of plants vegetative propagation is also present

Asexual Reproduction Sexual Reproductioni) Always uniparentalii) Gametes are not involvediii) Only mitotic division involvediv) Somatic cells of parents are involvedv) Offsprings are genetically similar to the parents

i) Usually biparentalii) Gametes are involvediii) Meiosis occurs during gametogenesis Mitosis occurs after fertilisationiv) Germ cells of the parents are involvedv) offsprings are genetically different from the parents

COMMERCE BUSINESS ENVIRONMENT

Welcome to the new sessiontoday we are going to start the first chapter of Class XII The name of the chapter is Business Environment

Already many of you have got some idea about the word business environment form the first chapter of business studies in class XI

In todayrsquos world every business enterprise is a part of the society It exists and operates in association with various groups in society such as customers suppliers competitors banks and financial institutions government agencies trade unions media and so on All these groups influence the functioning of business in one way or the other They constitute the environment of businessConcept of Business Environment

The term lsquobusiness environmentrsquo refers to the sum total of all individuals institutions and other forces that lie outside a business enterprise but that may influence its functioning and performance

The main features of business environment Totality of External forces General and Specific forces Interrelatedness Complexity Dynamic Uncertainty

Prepare the following questions from todayrsquos assignment

2 What do you mean by business environment

The term lsquobusiness environmentrsquo means the aggregate of all forces factors and institutions which are external to and beyond the control of an individual business enterprise but they may influence its functioning and performance Business environment is the macro framework within which a business firm a micro unit operates It consists of several interrelated and interacting elements

2 Explain the main features of business environment in brief

Totality of External forces-Business environment is the sum total of all things external to a business environment

General and Specific forces-It

Relativity

The Interrelation between business and its environment

The business enterprise is an open system It continuously interacts with its environment It takes inputs (such as raw materials capital labour energy and so on) from its environment transforms them into goods and services and sends them back to the environment

Fig 1 Business Environment Relationship

includes both the forces general forces are the economic social political legal and technological conditions which indirectly influence all business enterprise Specific forces are the investors customers competitors and suppliers which influence individual enterprise directly

Interrelatedness-Different elements of environment are interrelated for an example growing awareness for health care has increased the demand for health foods

Complexity- Business environment id complex in nature as the elements keep on changing example economic technological and other forces changes in demand for a product and service

Dynamic-Business environment is not static it keeps on changing

Uncertainty- Itrsquos very difficult to predict future events such as technology and fashion which occur fast and frequently

Business Studies

Human Resources Management

Human resource of an organisation are the aggregate of knowledge skills attitudes of people working in it

The management system which deals with human resources is called human resource management

Features of HRMbullComprehensive functionbullPeople-oriented

Question1) What do you mean by human

resource management Answer) Human resource management may be defined as that field of Management which has to do with planning organising and controlling the functions of procuring developing maintaining and utilising the labour force

bullAction oriented bullPervasive function bullContinuous function

2) Explain the features of HRM in brief

Answer)bullHuman Resource Management is concerned with managing people at work bull Human Resource Management is concerned with employees which bring people and organisations together so that the goals of each are met bullHuman resource management considered every employees as an individual and also promote their satisfaction and growth bull Human resource management is inherent in all organisations and at all levelsbullManagement of human resources are ongoing on never ending process which requires a constant alertness and Awareness of human relations

3) ldquoHR function is said to be pervasiverdquowhy

Answer) Human resource management is required in all organisations whether it is private or government organisations armed forces sports organisations etc It permeatsall the functional areas like production marketing finance research etc This from this feature of human resource management it can be said that it is pervasive in nature

Economics Demand Q1DEFINITION OF DEMANDIn economics demand is the quantity of a good that consumers are willing and able to purchase at various prices during a given period of timeQ2DEMAND CURVEIn economics a demand curve is a graph depicting the relationship between the price of a certain commodity and the quantity of that commodity that is demanded at that pricQ3LAW OF DEMANDIn microeconomics the law of demand states that conditional on all else being equal as the price of a good increases quantity demanded decreases conversely as the price of a good decreases quantity demanded increasesQ4ASSUMPTION of LAW OF DEMAND(i)No change in price of related commodities(ii) No change in income of the consumer(iii) No change in taste and preferences customs habit and fashion of the consumer( No expectation regarding future change in priceQ5MARKET DEMAND SCHEDULEIn economics a market demand schedule is a tabulation of the quantity of a good that all consumers in a market will purchase at a

given price At any given price the corresponding value on the demand schedule is the sum of all consumersrsquo quantities demanded at that priceQ6INDIVIDUAL DEMAND SCHEDULEIndividual demand schedule refers to a tabular statement showing various quantities of a commodity that a consumer is willing to buy at various levels of price during a given period of timeQ7 FACTORS AFFECTING INDIVIDUAL DEMAND FOR A COMMODITY

The factors that influence a consumerrsquos decision to purchase a commodity are also known as determinants of demand The following factors affect the individual demand for a commodity1 price of the commodity2 price of related goods3 income of buyer of the commodity4 tastes and preferences of the buyer1 Price of the CommodityYou must have observed that when price of a commodity falls you tend to buy more of it and when its price rises you tend to buy less of it when all other factors remain constant (lsquoother things remaining the samersquo) In other words other things remaining the same there is an inverse relationship between the price of a commodity and its quantity demanded by its buyers This statement is in accordance with law of demand which you will study in the later part of this lesson Price of a commodity and its quantity demanded by its buyers are inversely related only when lsquoother things remain the samersquo So lsquoother things remaining the samersquo is an assumption when we study the effect of changes in the price of a commodity on its quantity demanded2 Price of Related goodsA consumer may demand a particular good But while buying that good heshe also asks the price of its related goods Related goods can be of two types-(i) Substitute goods(ii) Complementary goods While purchasing a good prices of its substitutes and complements do affect its quantity purchased(i) Price of Substitute Goods Substitute goods are those goods which can easily be used in place of one another for satisfaction of a particular want like tea and coffee An increase in price of substitute good leads to an increase in demand for the given commodity and a decrease in price of substitute good leads to a decrease in demand for the given commodity It means demand for a given commodity is directly affected by change in price of substitute goods For example if price of coffee increases the demand for tea will rise as tea will become relatively cheaper in comparison to coffee(ii) Price of Complementary goods Complementary goods are those goods which are used together to satisfy a particular want like car and petrol An increase in the price of complementary goods leads to a decrease in demand for the given commodity and a decrease in the price of complementary goods leads to an increase in demand for the given commodity For example if price of petrol falls then the demand for cars will increase as it will be relatively cheaper to use both the goods together So demand for a given commodity is inversely affected by change in price of complementary goods3 Income of the Buyer of CommodityDemand for a commodity is also affected by income of its buyer However the effect of change in income on demand depends on the nature of the commodity under consideration In case of some goods like full cream milk fine quality of rice (Basmati rice) etc demand for these commodities increases when income of the buyer increases and

demand for these commodities decreases when income of the buyer decreases Such goods whose demand increases with the increase in income of the buyer are called normal goods But there are some goods like coarse rice toned milk etc whose demand decreases when income of buyer increases and their demand increases when income of the buyer decreases Such goods whose demand decreases with the increase in income of the buyer are called inferior goods Suppose a consumer buys 10 Kgs of rice whose price is ` 25 per Kg He cannot afford to buy better quality of rice because the price of such rice is ` 50 per Kg The consumer is spending ` 250 per month on the purchase of rice Now if income of the consumer increases and he can afford ` 350 on purchase of 10 Kg of rice Now he can afford to buy some quantity of rice say 6 Kgs whose price is ` 25 per Kg and may buy 4 Kgs of rice whose price is ` 50 per Kg Thus he will buy 10 Kgs of rice by spending ` 350 per month Therefore we may conclude that demand for normal goods is directly related to the income of the buyer but demand for inferior goods is inversely related to the income of the buyer4 Tastes and Preferences of the BuyerThe demand for a commodity is also affected by the tastes and preferences of the buyers They include change in fashion customs habits etc Those commodities are preferred by the consumers which are in fashion So demand for those commodities rises which are in fashion On the other hand if a commodity goes out of the fashion its demand falls because no consumer will like to buy it(5) Number of Buyers in the Market(Population)Increase in population raises the market demand whereas decrease in population reduces the market demand for a commodity Not only the size of population but its composition like age (ratio of males females children and old people in population) also affects the demand for a commodity It is because of needs of children young old male and female population differs(6) Distribution of Income and WealthIf the distribution of income and wealth is more in favour of the rich demand for the commodities preferred by the rich such as comforts and luxuries is likely to be higher On the other hand if the distribution of income and wealth is more in favour of poor demand for commodities preferred by the poor such as necessities will be more(7) Season and Weather ConditionsThis is generally observed that the demand for woolens increases during winter whereas demand for ice creams and cold drinks increases during summer Similarly market demand for umbrellas rain coats increases during rainy seasonQ8 REASONS FOR OPERATION OF LAW OF DEMAND WHY DEMAND CURVE SLOPES DOWNWARDNow we will try to explain why does a consumer purchase more quantity of a commodity at a lower price and less of it at a higher price or why does the law of demand operate ie why does the demand curve slope downwards from left to right The main reasons for operation of law of demand are1 Law of Diminishing Marginal UtilityAs you have studied earlier law of diminishing marginal utility states that as we consume more and more units of a commodity the utility derived from each successive unit goes on decreasing The consumer will be ready to pay more for those units which provide him more utility and less for those which provide him less utility It implies that he will purchase more only when the price of the commodity falls2 Income Effect

When price of a commodity falls purchasing power or real income of the consumer increases which enables him to purchase more quantity of the commodity with the same money income Let us take an example Suppose you buy 4 ice creams when price of each ice cream is ` 25 If price of ice creams falls to ` 20 then with same money income you can buy 5 ice creams now3 Substitution EffectWhen price of a commodity falls it becomes comparatively cheaper as compared to its substitutes (although price of substitutes has not been changed) This will lead to rise in demand for the given commodity For example if coke and Pepsi both are sold at ` 10 each and price of coke falls Now coke has become relatively cheaper and will be substituted for Pepsi It will lead to rise in demand for coke4 Change in Number of BuyersWhen price of a commodity falls some old buyers may demand more of the commodity at the reduced price and some new buyers may also start buying this commodity who were not in a position to buy it earlier due to higher price This will lead to increase in number of buyers when price of the commodity falls As a result demand for the commodity rises when its price falls5 Diverse Uses of a CommoditySome commodities have diverse uses like milk It can be used for drinking for sweet preparation for ice cream preparation etc If price of milk rises its use may be restricted to important purpose only This will lead to reduction in demand for other less important uses When price of milk falls it can be put to other uses also leading to rise n demand for itQ9 EXCEPTIONS TO THE LAW OF DEMANDYou have studied in law of demand that a buyer is willing to buy more quantity of a commodity at a lower price and less of it at a higher price But in certain circumstances a rise in price may lead to rise in demand These circumstances are called Exceptions to the Law of Demand Some important exceptions are1 Giffen GoodsGiffen goods are special type of inferior goods in which negative income effect is stronger than negative substitution effect Giffen goods do not follow law of demand as their demand rises when their price rises Examples of Giffen goods are jowar and bajra etc2 Status Symbol GoodsSome goods are used by rich people as status symbols eg diamonds gold jewellary etc The higher the price the higher will be the demand for these goods When price of such goods falls these goods are no longer looked at as status symbol goods and tehrefore therir demand falls3 NecessitiesCommodities such as medicines salt wheat etc do not follow law of demandbecause we have to purchase them in minimum required quantity whatever their price may be4 Goods Expected to be ScarceWhen the buyers expect a scarcity of a particular good in near future they start buying more and more of that good even if their prices are rising For example during war famines etc people tend to buy more of some goods even at higher prices due to fear of their scarcity in near future

Political Science

Constitution of India-The

Preamble

The preamble-

Preamble-

The preamble is the most precious part of the constitution We the people of India having solemnly resolved to constitute India into a Sovereign Socialist Secular Democratic Republic and to secure to all its citizensA preamble is an introductory and expressionary statement in a document that explains the documents purpose and underlying philosophy When applied to the opening paragraphs of a statute it may recite historical facts pertinent to the subject of the statuteNature and purpose of the constitution-Purpose of the Constitution dictates permanent framework of the government to form a more perfect union to establish justice and ensure peace of thenationconstitution provide principles how the government can run itself following the rules and laws written in the constitution of each state keeps them balanced

Answer the following questions-

1 What is preambleA preamble is an introductory and expressionary statement in a document that explains the documents purpose and underlying philosophy2 What is the nature and

purpose of the constitutionConstitution dictatespermanent framework of the government to form a more perfect union to establish justice and ensure peace of the nation

Homework-Learn

Accounts Compatibilty mode

1MEANING OF PARTNERSHIPPartnership is a form of business organisation where two or more persons join hands to run a business They share the profits and losses according to the agreement amongst them According to the Indian Partnership Act 1932 ldquoPartnership is relation between persons who have agreed to share profits of a business carried on by all or any one of them acting for allrdquo For example one of your friends has passed class XII from National Institute of Open Schooling (NIOS) and wants to start a business Heshe approaches you to join in this venture Heshe wants you to contribute some money and participate in the business activities Both of you if join hands constitute a partnership2CHARACTERISTICS1048698 Agreement A partnership is formed by an agreement The agreement may be either oral or in writing It defines the relationship between the persons who agree to carry on business It may contain the terms of sharing profit and the capital to be invested by each partner etc The written agreement is known as partnership deed1048698 Number of persons There must be at least two persons to form a partnership

The maximum number of partners in a partnership firm can be 50 according toCompanies Act 20131048698 Business The Partnership is formed to carry on business with a purpose of earning profits The business should be lawful Thus if two or more persons agree to carry on unlawful activities it will not be termed as partnership1048698 Sharing Profits The partners agree to share profits in the agreed ratio In caseof loss all the partners have to bear it in the same agreed profit sharing ratio10486981048698Mutual Agency Every partner is an agent of the other partners Every partner can bind the firm and all other partners by hisher acts Each partner will be responsible and liable for the acts of all other partners10486981048698Unlimited liability The liability of each partner except that of a minor is unlimited Their liability extends to their personal assets also If the assets of the firm are insufficient to pay off its debts the partnersrsquo personal property can be used to satisfy the claim of the creditors of the partnership firm10486981048698Management All the partners have a right to mange the business However they may authorize one or more partners to manage the affairs of the business on their behalf10486981048698Transferability of Share No partner can transfer hisher share to any one including hisher family member without the consent of all other partners3PARTNERSHIP DEEDAgreement forms the basis of partnership The written form of the agreement is which a document of partnership is It contains terms and conditions regarding the conduct of the business It also explains relationship between the partners This document is called partnership deed Every firm can frame its own partnership deed in which the rights duties and liabilities of the partners are stated in detail It helps in settling the disputes arising among the partners during the general conduct of business 4CONTENTS OF PARTNERSHIP DEEDThe partnership deed generally contains the following (i) Name and address of the partnership firm(ii) Nature and objectives of the business(iii) Name and address of each partner(iv) Ratio in which profits is to be shared(v) Capital contribution by each partner(vi) Rate of Interest on capital if allowed(vii) Salary or any other remuneration to partners if allowed(viii) Rate of interest on loans and advances by a partner to the firm(ix) Drawings of partners and interest thereon if any(x) Method of valuation of goodwill and revaluation of assets and liabilities on the reconstitution of the partnership ie on the admission retirement or death of a partner(xi) Settlement of disputes by arbitration(xii) Settlement of accounts at the time of retirement or death of a partner5IN ABSENCE OF PARTNERSHIP DEEDThe partnership deed lays down the terms and conditions of partnership in regard to rights duties and obligations of the partners In the absence of partnership deed there may arise a controversy on certain issues like profit sharing ratio interest on

capital interest on drawings interest on loan and salary of the partners In such cases the provisions of the Indian Partnership Act becomes applicableSome of the Issues are(i) Distribution of Profit Partners are entitled to share profits equally(ii) Interest on Capital Interest on capital is not allowed(iii) Interest on Drawings No interest on drawing of the partners is to be charged(iv) Interest on Partnerrsquos Loan A Partner is allowed interest 6 per annum on the amount of loan given to the firm by himher(v) Salary and Commission to Partner A partner is not entitled to anysalary or commission or any other remuneration for managing the business

History TOPIC-TOWARDS INDEPENDENCE AND PARTITION THE LAST PHASE (1935-1947)

SUB TOPIC-IMPORTANT POLITICAL DEVELOPMENTS ndash GROWTH OF SOCIAL IDEAS

Socialism is a political social and economic philosophyLike in other parts of the world the Russian revolution of 1917 served as a great inspiration for revolutionaries in India who at that time were engaged in the struggle for liberation from British ruleSocialist ideas led to the formation of communist party of IndiaJAWAHARLAL NEHRU Among the early Congress leaders Jawaharlal Nehru was very much impressed and influenced by the Socialist ideas He also learnt about the Economic activities of the Soviet Union after the Bolshevic Revolution 1917 He made full use of them in IndiaThe election of Jawaharlal Nehru and Subhas Chandra Bose showed the Left wing tendency within CongressJawaharlal Nehru demanded economic freedom along with political freedom of the people in order to end the exploitation of masses

Nehrus working committee included three socialists leaders The Lucknow session was a landmark in the evolution of socialist ideas of the congressSUBHAS CHANDRA BOSE ndash Subhas Chandra Bose had socialist leaning Both Jawaharlal Nehru and Subhas Chandra Bose were known as leftist Congress men Later on National Congress divided into Leftist and rightist campCONGRESS SOCIALIST Within the Congress some leaders formed the Congress Socialist partyPattavi Sitaramyya Sardar Patel Rajendra Prasad had hostile attitude towards the Congress Socialist partyJawaharlals attitude was hesitant

1 QUESTION ndash Mention name of two Congress leaders who had socialist leaning

1ANSWER ndash Subhas Chandra Bose and Jawaharlal Nehru2QUESTION- In which session of the congress Jawaharlal elaborated his Socialist ideas2 ANSWER ndash Lucknow and Faizpur Session in December 1935 and 19363QUESTION ndash Why Congress was sharply divided into leftist and rightist camp 3ANSWER ndash Subhas Chandra Bosersquos attempt to seek re election for congress presidentship in 1939sharply divided the National Congress into Leftist and Rightist camp4 QUESTION ndash Who was MN Roy 4 ANSWER ndash Manabendra Roy first formed the Communist Party of India outside the country at Tashkent in 19205QUESTION ndash Who formed the Congress Socialist Party within the Congress5 ANSWER ndash Jaya Prakash Narayan Achyut Patwardhan Acharya Narendra Dev Ram Mohan Lohia Aruna Asaf Ali6QUESTION ndash When was the Congress Socialist Party formed What was its object6 ANSWER ndash 1934The Congress Socialist Party sought to work out socialist programme through the Congress They joined hands with the Congress and wanted to carry

Subhas Chandra Bose being expelled from the congress after the Tripuri rift he formed Forward BlockThere were basic differences between the Congress Socialists and the communistsTRADE UNION ACTIVITIES Maximum working class people lived in Bombay and Calcutta The working and living conditions of those workers were very miserable In this situation Shasipada Banerjee NM Lokhande protested against the oppression of the working class peopleThe first Trade Union Madras Labour Union was formed in 1918 by BP WadiaIndustrial strikes took place in Kanpur Calcutta Madras Jamshedpur and Ahmedabad AITUC was formed in Bombay in 1927 The growth of Trade union among the workers was slow because of the fear of the dismissal of the jobIn the mean time the Moderates as well as Communists left AITUC and formed separate organization

on National struggle with the help of workers and peasant class of the society7 QUESTION ndash What was the name of the party founded by Subhas Chandra Bose7 ANSWER- Forward Block8QUESTION ndash Who was Shasipada Banerjee8 ANSWER ndash Shasipada Banerjee was a radical Brahmo He founded a working menrsquos club to protest against exploitation of the British rulers towards the working class of India9 QUESTION ndash What was the weekly published by NM Lokhande9ANSWER- Dinabandhu10 QUESTION ndash Who founded Bombay Mill-Hands Association and in which year10 ANSWER- NM Lokhande in189011 QUESTION- Who was BP WadiaANSWER- BPWadia was the founder of Madras Labour Union in191812 QUESTION- What was the name of the first labour union of India12 ANSWER- Madras Labour Union13 QUESTION Who founded the Majur Mahajan 13 ANSWER GANDHIJI14 QUESTION What was the full form of AITUC When it was formed14 ANSWER All India Trade Union Congressin 192715QUESTION Who formed the Red Trade Union Congress and in which year15ANSWER The Communists formed the Red Trade Union Congress16 QUESTION What do you mean by Socialism16 ANSWER Socialism describes any political and economic theory that says the community rather than individuals should own and manage property and natural resources

Subject Eng Literature (The Tempest ndash William Shakespeare) Topic Act III Scene 3 Lines 1 to 52 (Line 52 ndash Brother my lord the Duke Stand to and do as we) Date 13th April 2020 (4th Period)

[Students should read the original play and also the paraphrase given in the school prescribed textbook]Summary Questions amp Answers

o Alonso Sebastian Antonio Gonzalo Adrian Francisco and others wandered about the island in search of Ferdinand and gets tired and hungry of the toil and at the same time gives up all hope of finding him

o Antonio and Sebastian are happy that Alonso is out of hope and decide to make another attempt on his life that night when being so tired they will be sleeping soundly

o Suddenly a solemn and strange music is heard in the air and several strange shapes enter bringing in a banquet These strange shapes then dance round it with gestures of salutation and then inviting the King to eat they depart

o Seeing this strange scene all are inclined to believe the tales told by travelers that there truly are ldquounicornsrdquo and ldquothe phoenixrsquo thronerdquo

1 ALONSO What harmony is this My good friends hark (L18-27)

GONZALO Marvellous sweet music

[Enter several strange shapes bringing in a banquet

they dance about it with gentle actions of salutation

and inviting the King and his companions to eat they depart]ALONSO Give us kind keepers heavens What were theseSEBASTIAN A living drollery Now I will believe

That there are unicorns that in Arabia

There is one tree the phoenixrsquo throne one phoenix

At this hour reigning thereANTONIO Ill believe both

And what does else want credit come to me

And Ill be sworn rsquotis true Travellers neer did lie

Though fools at home condemn rsquoem

(a) How did Prospero present an amazing spectacle before Alonso and his companions

Using his magic powers Prospero ordered strange shapes to lay a banquet before Alonso and his companions The shapes brought several dishes with tasty eatables in them They placed the dishes on a table before Alonso and his companions Then the strange shapes began to dance gracefully around the banquet While dancing they made gestures inviting them to eat the food Then suddenly the shapes disappeared(b) Who were the guests at the strange banquet Describe the lsquoliving drolleryrsquo

Alonso Sebastian Antonio Gonzalo Adrian and Francisco were the guests at the strange banquet

The term ldquoliving drolleryrdquo refers to live entertainment show In this context when Alonso the King of Naples Sebastian his brother Antonio the treacherous brother of Prospero Gonzalo the kind and loyal councillor to the King Adrian and Francisco came to the island they were hungry and weary in their spirits They heard a solemn and strange music They were shocked to see several strange shapes bringing in a banquet and these shapes danced about it with gentle action of salutation inviting the King and his companions to eat After this Sebastian described this show as lsquoliving drolleryrsquo(c) What is lsquophoenixrsquo What are lsquoUnicornsrdquo

The term lsquophoenixrsquo refers to a mythical Arabian bird which lived alone and perched on a solitary tree After one hundred years it expired in flames and rose again from its own ashes

lsquoUnicornsrsquo refers to the mythological four-footed beasts having horns in the centre of their foreheads When the horns are ground into powder the powder was believed to be

an aphrodisiac(d) How does Sebastian explain the puppet show OR Why does the speaker now believe in unicorns and phoenix

Sebastian finds several strange shapes bringing in the banquet They invite the king and his party for dinner and soon depart He tells that if such a strange sight can be a reality there is nothing incredible in the world and from the present moment he will believe anything He says that it is a strange dumb show enacted not by puppets but by living beings It is stranger than a travellerrsquos tale Seeing such a thing

before his own eyes he will no longer disbelieve the story about unicorns and phoenix(e) How do the other characters present respond to this living drollery

At the sight of the lsquoliving drolleryrsquo like Sebastian Gonzalo and Antonio too acted strangely Antonio told that he too now believes in unicorns and phoenix and anything else that seems to be incredible He too now believes in travellersrsquo tales Gonzalo told that if he would report those happenings in Naples nobody will believe him He considers that those gentle shapes were gentler in manner in comparison to the living beings Alonso was at first sight suspicious and told them that those strange shapes conveyed their meaning in expressive gestures when they seemed to lack speech by their movements and sounds Francisco was amazed at their mysterious disappearance

2 ALONSO Not I

(Line 43-52)GONZALO Faith sir you need not fear When we

were boysWho would believe that there were mountaineers

Dewlapped like bulls whose throats had hanging at rsquoem

Wallets of flesh Or that there were such men

Whose heads stood in their breasts Which now we find

Each putter-out of five for one will bring us

Good warrant ofALONSO I will stand to and feed

Although my lastmdashno matter since I feel

The best is past Brother my lord the Duke

Stand to and do as we

(a) How does Alonso respond at the spectacle of the shapes which were sent to them at the instruction of Prospero

After seeing the strange sight of appearing and disappearing of the shapes sent by Prospero to arrange a banquet for them Alonso says that his surprise at having seen those creatures is infinite and he is fully justified in feeling so much surprise He thinks that their shapes their gestures and the sounds they made were indeed amazing Although they do not possess the gift of speech yet they were able to convey their

thoughts by means of their gestures only

(b) What does Prospero say about the views expressed by Alonso regarding the shapes What does Francisco think about the shapesAfter hearing Alonsorsquos views about the shapes Prospero says that this manrsquos praise of the spirits is rather hasty He means to say that Alonso has shown great haste in reaching the conclusion about the shapes Francisco is amazed to see that those shapes disappeared in a mysterious way(c) What does Sebastian ask Alonso to doSebastian tells Alonso that the shapes having disappeared should not matter to them because they have left the eatables behind He asks Alonso to enjoy eating as they are extremely hungry but the king does not accept his offer of enjoying the dishes(d) How does Gonzalo try to dispel Alonsorsquos fear of those strange shapes What kind of references does he give to AlonsoGonzalo says that those who have travelled abroad have reported seeing even stranger sights than these shapes that Alonso and his companions have beheld Hence there is no reason to feel afraid of these shapes Gonzalo further adds that in his younger days he had heard strange stories from travelers and Alonso might have heard similar stories For instance it was said that there existed a certain race of

human beings who had huge lumps of flesh hanging at their throats and who therefore resembled bulls Then Gonzalo tells about a race of human beings whose heads were located at their breasts Gonzalo says that such stories were not believed by most people in those days but now-a-days these stories have become common(e) Explain the following lsquoEach putter-out of five for onersquoEnglish travellers often insured their trips with London brokers Those that went on foreign travels those days used to deposit a certain amount with some firm or company in London before their departure If the travelers failed to return the money was forfeited by the company with which it had been deposited But this money was repaid five-fold if the travelers returned safe and sound In this way a traveler stood a great chance of recovering the entire cost of his

travels(f) Give the explanatory meanings of the following expressions in the context of the above extract (i) Dewlapped (ii) Wallets of flesh

(iii) Putter-out(i) Dewlapped having big lumps of flesh at the necks(ii) Wallets of flesh large masses of flesh looking like bags(iii) Putter-out to invest money before commencing the travel

  • General methods of preparation of hydrogen
  • Chapter Dimensional Analysis (Summary)
    • Properties of Charge
Page 47:   · Web viewSubject. Topic. Summary. Execution. Hindi. व्याकरण. शरीरके अंगो के नाम लिखिए. 1) आँख 2) नाक 3

current of one ampere

As q = It we have1 C = (1 A) (1 s)

The dimensions of charge are [A T]

Properties of Charge

(1) Quantization of Charge Electric charge can have only discrete values rather than any value That is charge is quantized The smallest discrete value of charge that can exist in nature is the charge on an electron given as

e = plusmn 16 x 10- 19 C

This is the charge attained by an electron and a protonA charge q must be an integral multiple of this basic unit That is

Q = plusmn ne where n = 1 2 hellip

Charge on a body can never be (frac12)e (23)e or 57e etcWhen we rub a glass rod with silk some electrons are transferred from the rod to the silk The rod becomes positively charged The silk becomes negatively charged The coulomb is a very large amount of charge A typical charge acquired by a rubbed body is 10 - 8 C

Biology Reproduction in organisms

Welcome to this new session 2020-21Today in this first chapter we mainly discuss about reproduction types needs and life span of some organismsWe also discuss about difference between sexual and asexual reproduction

Q1 What is reproductionReproduction is defined as a biological processin which an organism gives rise to young onessimilar to itselfQ2 What are the needs of reproductionbulli) Reproduction maintain life on earthii) It enables the continuity of the species generation after generationiii) It creates genetic variation among populationsQ3 Define Life span and write some orgnisms life spanbull Life span is the period from birth to

the natural death of an organism- OrganismsLife span1 Butterfly 1 - 2 weeks2 Fruit fly 30 days3Dog 10-13 years4 Rose5-7 years5 Tortoise100-150 years6 Banyan Tree -200 - 250 yearsQ4 Reproduction is of two types in case ofanimals but in case of plants vegetative propagation is also present

Asexual Reproduction Sexual Reproductioni) Always uniparentalii) Gametes are not involvediii) Only mitotic division involvediv) Somatic cells of parents are involvedv) Offsprings are genetically similar to the parents

i) Usually biparentalii) Gametes are involvediii) Meiosis occurs during gametogenesis Mitosis occurs after fertilisationiv) Germ cells of the parents are involvedv) offsprings are genetically different from the parents

COMMERCE BUSINESS ENVIRONMENT

Welcome to the new sessiontoday we are going to start the first chapter of Class XII The name of the chapter is Business Environment

Already many of you have got some idea about the word business environment form the first chapter of business studies in class XI

In todayrsquos world every business enterprise is a part of the society It exists and operates in association with various groups in society such as customers suppliers competitors banks and financial institutions government agencies trade unions media and so on All these groups influence the functioning of business in one way or the other They constitute the environment of businessConcept of Business Environment

The term lsquobusiness environmentrsquo refers to the sum total of all individuals institutions and other forces that lie outside a business enterprise but that may influence its functioning and performance

The main features of business environment Totality of External forces General and Specific forces Interrelatedness Complexity Dynamic Uncertainty

Prepare the following questions from todayrsquos assignment

2 What do you mean by business environment

The term lsquobusiness environmentrsquo means the aggregate of all forces factors and institutions which are external to and beyond the control of an individual business enterprise but they may influence its functioning and performance Business environment is the macro framework within which a business firm a micro unit operates It consists of several interrelated and interacting elements

2 Explain the main features of business environment in brief

Totality of External forces-Business environment is the sum total of all things external to a business environment

General and Specific forces-It

Relativity

The Interrelation between business and its environment

The business enterprise is an open system It continuously interacts with its environment It takes inputs (such as raw materials capital labour energy and so on) from its environment transforms them into goods and services and sends them back to the environment

Fig 1 Business Environment Relationship

includes both the forces general forces are the economic social political legal and technological conditions which indirectly influence all business enterprise Specific forces are the investors customers competitors and suppliers which influence individual enterprise directly

Interrelatedness-Different elements of environment are interrelated for an example growing awareness for health care has increased the demand for health foods

Complexity- Business environment id complex in nature as the elements keep on changing example economic technological and other forces changes in demand for a product and service

Dynamic-Business environment is not static it keeps on changing

Uncertainty- Itrsquos very difficult to predict future events such as technology and fashion which occur fast and frequently

Business Studies

Human Resources Management

Human resource of an organisation are the aggregate of knowledge skills attitudes of people working in it

The management system which deals with human resources is called human resource management

Features of HRMbullComprehensive functionbullPeople-oriented

Question1) What do you mean by human

resource management Answer) Human resource management may be defined as that field of Management which has to do with planning organising and controlling the functions of procuring developing maintaining and utilising the labour force

bullAction oriented bullPervasive function bullContinuous function

2) Explain the features of HRM in brief

Answer)bullHuman Resource Management is concerned with managing people at work bull Human Resource Management is concerned with employees which bring people and organisations together so that the goals of each are met bullHuman resource management considered every employees as an individual and also promote their satisfaction and growth bull Human resource management is inherent in all organisations and at all levelsbullManagement of human resources are ongoing on never ending process which requires a constant alertness and Awareness of human relations

3) ldquoHR function is said to be pervasiverdquowhy

Answer) Human resource management is required in all organisations whether it is private or government organisations armed forces sports organisations etc It permeatsall the functional areas like production marketing finance research etc This from this feature of human resource management it can be said that it is pervasive in nature

Economics Demand Q1DEFINITION OF DEMANDIn economics demand is the quantity of a good that consumers are willing and able to purchase at various prices during a given period of timeQ2DEMAND CURVEIn economics a demand curve is a graph depicting the relationship between the price of a certain commodity and the quantity of that commodity that is demanded at that pricQ3LAW OF DEMANDIn microeconomics the law of demand states that conditional on all else being equal as the price of a good increases quantity demanded decreases conversely as the price of a good decreases quantity demanded increasesQ4ASSUMPTION of LAW OF DEMAND(i)No change in price of related commodities(ii) No change in income of the consumer(iii) No change in taste and preferences customs habit and fashion of the consumer( No expectation regarding future change in priceQ5MARKET DEMAND SCHEDULEIn economics a market demand schedule is a tabulation of the quantity of a good that all consumers in a market will purchase at a

given price At any given price the corresponding value on the demand schedule is the sum of all consumersrsquo quantities demanded at that priceQ6INDIVIDUAL DEMAND SCHEDULEIndividual demand schedule refers to a tabular statement showing various quantities of a commodity that a consumer is willing to buy at various levels of price during a given period of timeQ7 FACTORS AFFECTING INDIVIDUAL DEMAND FOR A COMMODITY

The factors that influence a consumerrsquos decision to purchase a commodity are also known as determinants of demand The following factors affect the individual demand for a commodity1 price of the commodity2 price of related goods3 income of buyer of the commodity4 tastes and preferences of the buyer1 Price of the CommodityYou must have observed that when price of a commodity falls you tend to buy more of it and when its price rises you tend to buy less of it when all other factors remain constant (lsquoother things remaining the samersquo) In other words other things remaining the same there is an inverse relationship between the price of a commodity and its quantity demanded by its buyers This statement is in accordance with law of demand which you will study in the later part of this lesson Price of a commodity and its quantity demanded by its buyers are inversely related only when lsquoother things remain the samersquo So lsquoother things remaining the samersquo is an assumption when we study the effect of changes in the price of a commodity on its quantity demanded2 Price of Related goodsA consumer may demand a particular good But while buying that good heshe also asks the price of its related goods Related goods can be of two types-(i) Substitute goods(ii) Complementary goods While purchasing a good prices of its substitutes and complements do affect its quantity purchased(i) Price of Substitute Goods Substitute goods are those goods which can easily be used in place of one another for satisfaction of a particular want like tea and coffee An increase in price of substitute good leads to an increase in demand for the given commodity and a decrease in price of substitute good leads to a decrease in demand for the given commodity It means demand for a given commodity is directly affected by change in price of substitute goods For example if price of coffee increases the demand for tea will rise as tea will become relatively cheaper in comparison to coffee(ii) Price of Complementary goods Complementary goods are those goods which are used together to satisfy a particular want like car and petrol An increase in the price of complementary goods leads to a decrease in demand for the given commodity and a decrease in the price of complementary goods leads to an increase in demand for the given commodity For example if price of petrol falls then the demand for cars will increase as it will be relatively cheaper to use both the goods together So demand for a given commodity is inversely affected by change in price of complementary goods3 Income of the Buyer of CommodityDemand for a commodity is also affected by income of its buyer However the effect of change in income on demand depends on the nature of the commodity under consideration In case of some goods like full cream milk fine quality of rice (Basmati rice) etc demand for these commodities increases when income of the buyer increases and

demand for these commodities decreases when income of the buyer decreases Such goods whose demand increases with the increase in income of the buyer are called normal goods But there are some goods like coarse rice toned milk etc whose demand decreases when income of buyer increases and their demand increases when income of the buyer decreases Such goods whose demand decreases with the increase in income of the buyer are called inferior goods Suppose a consumer buys 10 Kgs of rice whose price is ` 25 per Kg He cannot afford to buy better quality of rice because the price of such rice is ` 50 per Kg The consumer is spending ` 250 per month on the purchase of rice Now if income of the consumer increases and he can afford ` 350 on purchase of 10 Kg of rice Now he can afford to buy some quantity of rice say 6 Kgs whose price is ` 25 per Kg and may buy 4 Kgs of rice whose price is ` 50 per Kg Thus he will buy 10 Kgs of rice by spending ` 350 per month Therefore we may conclude that demand for normal goods is directly related to the income of the buyer but demand for inferior goods is inversely related to the income of the buyer4 Tastes and Preferences of the BuyerThe demand for a commodity is also affected by the tastes and preferences of the buyers They include change in fashion customs habits etc Those commodities are preferred by the consumers which are in fashion So demand for those commodities rises which are in fashion On the other hand if a commodity goes out of the fashion its demand falls because no consumer will like to buy it(5) Number of Buyers in the Market(Population)Increase in population raises the market demand whereas decrease in population reduces the market demand for a commodity Not only the size of population but its composition like age (ratio of males females children and old people in population) also affects the demand for a commodity It is because of needs of children young old male and female population differs(6) Distribution of Income and WealthIf the distribution of income and wealth is more in favour of the rich demand for the commodities preferred by the rich such as comforts and luxuries is likely to be higher On the other hand if the distribution of income and wealth is more in favour of poor demand for commodities preferred by the poor such as necessities will be more(7) Season and Weather ConditionsThis is generally observed that the demand for woolens increases during winter whereas demand for ice creams and cold drinks increases during summer Similarly market demand for umbrellas rain coats increases during rainy seasonQ8 REASONS FOR OPERATION OF LAW OF DEMAND WHY DEMAND CURVE SLOPES DOWNWARDNow we will try to explain why does a consumer purchase more quantity of a commodity at a lower price and less of it at a higher price or why does the law of demand operate ie why does the demand curve slope downwards from left to right The main reasons for operation of law of demand are1 Law of Diminishing Marginal UtilityAs you have studied earlier law of diminishing marginal utility states that as we consume more and more units of a commodity the utility derived from each successive unit goes on decreasing The consumer will be ready to pay more for those units which provide him more utility and less for those which provide him less utility It implies that he will purchase more only when the price of the commodity falls2 Income Effect

When price of a commodity falls purchasing power or real income of the consumer increases which enables him to purchase more quantity of the commodity with the same money income Let us take an example Suppose you buy 4 ice creams when price of each ice cream is ` 25 If price of ice creams falls to ` 20 then with same money income you can buy 5 ice creams now3 Substitution EffectWhen price of a commodity falls it becomes comparatively cheaper as compared to its substitutes (although price of substitutes has not been changed) This will lead to rise in demand for the given commodity For example if coke and Pepsi both are sold at ` 10 each and price of coke falls Now coke has become relatively cheaper and will be substituted for Pepsi It will lead to rise in demand for coke4 Change in Number of BuyersWhen price of a commodity falls some old buyers may demand more of the commodity at the reduced price and some new buyers may also start buying this commodity who were not in a position to buy it earlier due to higher price This will lead to increase in number of buyers when price of the commodity falls As a result demand for the commodity rises when its price falls5 Diverse Uses of a CommoditySome commodities have diverse uses like milk It can be used for drinking for sweet preparation for ice cream preparation etc If price of milk rises its use may be restricted to important purpose only This will lead to reduction in demand for other less important uses When price of milk falls it can be put to other uses also leading to rise n demand for itQ9 EXCEPTIONS TO THE LAW OF DEMANDYou have studied in law of demand that a buyer is willing to buy more quantity of a commodity at a lower price and less of it at a higher price But in certain circumstances a rise in price may lead to rise in demand These circumstances are called Exceptions to the Law of Demand Some important exceptions are1 Giffen GoodsGiffen goods are special type of inferior goods in which negative income effect is stronger than negative substitution effect Giffen goods do not follow law of demand as their demand rises when their price rises Examples of Giffen goods are jowar and bajra etc2 Status Symbol GoodsSome goods are used by rich people as status symbols eg diamonds gold jewellary etc The higher the price the higher will be the demand for these goods When price of such goods falls these goods are no longer looked at as status symbol goods and tehrefore therir demand falls3 NecessitiesCommodities such as medicines salt wheat etc do not follow law of demandbecause we have to purchase them in minimum required quantity whatever their price may be4 Goods Expected to be ScarceWhen the buyers expect a scarcity of a particular good in near future they start buying more and more of that good even if their prices are rising For example during war famines etc people tend to buy more of some goods even at higher prices due to fear of their scarcity in near future

Political Science

Constitution of India-The

Preamble

The preamble-

Preamble-

The preamble is the most precious part of the constitution We the people of India having solemnly resolved to constitute India into a Sovereign Socialist Secular Democratic Republic and to secure to all its citizensA preamble is an introductory and expressionary statement in a document that explains the documents purpose and underlying philosophy When applied to the opening paragraphs of a statute it may recite historical facts pertinent to the subject of the statuteNature and purpose of the constitution-Purpose of the Constitution dictates permanent framework of the government to form a more perfect union to establish justice and ensure peace of thenationconstitution provide principles how the government can run itself following the rules and laws written in the constitution of each state keeps them balanced

Answer the following questions-

1 What is preambleA preamble is an introductory and expressionary statement in a document that explains the documents purpose and underlying philosophy2 What is the nature and

purpose of the constitutionConstitution dictatespermanent framework of the government to form a more perfect union to establish justice and ensure peace of the nation

Homework-Learn

Accounts Compatibilty mode

1MEANING OF PARTNERSHIPPartnership is a form of business organisation where two or more persons join hands to run a business They share the profits and losses according to the agreement amongst them According to the Indian Partnership Act 1932 ldquoPartnership is relation between persons who have agreed to share profits of a business carried on by all or any one of them acting for allrdquo For example one of your friends has passed class XII from National Institute of Open Schooling (NIOS) and wants to start a business Heshe approaches you to join in this venture Heshe wants you to contribute some money and participate in the business activities Both of you if join hands constitute a partnership2CHARACTERISTICS1048698 Agreement A partnership is formed by an agreement The agreement may be either oral or in writing It defines the relationship between the persons who agree to carry on business It may contain the terms of sharing profit and the capital to be invested by each partner etc The written agreement is known as partnership deed1048698 Number of persons There must be at least two persons to form a partnership

The maximum number of partners in a partnership firm can be 50 according toCompanies Act 20131048698 Business The Partnership is formed to carry on business with a purpose of earning profits The business should be lawful Thus if two or more persons agree to carry on unlawful activities it will not be termed as partnership1048698 Sharing Profits The partners agree to share profits in the agreed ratio In caseof loss all the partners have to bear it in the same agreed profit sharing ratio10486981048698Mutual Agency Every partner is an agent of the other partners Every partner can bind the firm and all other partners by hisher acts Each partner will be responsible and liable for the acts of all other partners10486981048698Unlimited liability The liability of each partner except that of a minor is unlimited Their liability extends to their personal assets also If the assets of the firm are insufficient to pay off its debts the partnersrsquo personal property can be used to satisfy the claim of the creditors of the partnership firm10486981048698Management All the partners have a right to mange the business However they may authorize one or more partners to manage the affairs of the business on their behalf10486981048698Transferability of Share No partner can transfer hisher share to any one including hisher family member without the consent of all other partners3PARTNERSHIP DEEDAgreement forms the basis of partnership The written form of the agreement is which a document of partnership is It contains terms and conditions regarding the conduct of the business It also explains relationship between the partners This document is called partnership deed Every firm can frame its own partnership deed in which the rights duties and liabilities of the partners are stated in detail It helps in settling the disputes arising among the partners during the general conduct of business 4CONTENTS OF PARTNERSHIP DEEDThe partnership deed generally contains the following (i) Name and address of the partnership firm(ii) Nature and objectives of the business(iii) Name and address of each partner(iv) Ratio in which profits is to be shared(v) Capital contribution by each partner(vi) Rate of Interest on capital if allowed(vii) Salary or any other remuneration to partners if allowed(viii) Rate of interest on loans and advances by a partner to the firm(ix) Drawings of partners and interest thereon if any(x) Method of valuation of goodwill and revaluation of assets and liabilities on the reconstitution of the partnership ie on the admission retirement or death of a partner(xi) Settlement of disputes by arbitration(xii) Settlement of accounts at the time of retirement or death of a partner5IN ABSENCE OF PARTNERSHIP DEEDThe partnership deed lays down the terms and conditions of partnership in regard to rights duties and obligations of the partners In the absence of partnership deed there may arise a controversy on certain issues like profit sharing ratio interest on

capital interest on drawings interest on loan and salary of the partners In such cases the provisions of the Indian Partnership Act becomes applicableSome of the Issues are(i) Distribution of Profit Partners are entitled to share profits equally(ii) Interest on Capital Interest on capital is not allowed(iii) Interest on Drawings No interest on drawing of the partners is to be charged(iv) Interest on Partnerrsquos Loan A Partner is allowed interest 6 per annum on the amount of loan given to the firm by himher(v) Salary and Commission to Partner A partner is not entitled to anysalary or commission or any other remuneration for managing the business

History TOPIC-TOWARDS INDEPENDENCE AND PARTITION THE LAST PHASE (1935-1947)

SUB TOPIC-IMPORTANT POLITICAL DEVELOPMENTS ndash GROWTH OF SOCIAL IDEAS

Socialism is a political social and economic philosophyLike in other parts of the world the Russian revolution of 1917 served as a great inspiration for revolutionaries in India who at that time were engaged in the struggle for liberation from British ruleSocialist ideas led to the formation of communist party of IndiaJAWAHARLAL NEHRU Among the early Congress leaders Jawaharlal Nehru was very much impressed and influenced by the Socialist ideas He also learnt about the Economic activities of the Soviet Union after the Bolshevic Revolution 1917 He made full use of them in IndiaThe election of Jawaharlal Nehru and Subhas Chandra Bose showed the Left wing tendency within CongressJawaharlal Nehru demanded economic freedom along with political freedom of the people in order to end the exploitation of masses

Nehrus working committee included three socialists leaders The Lucknow session was a landmark in the evolution of socialist ideas of the congressSUBHAS CHANDRA BOSE ndash Subhas Chandra Bose had socialist leaning Both Jawaharlal Nehru and Subhas Chandra Bose were known as leftist Congress men Later on National Congress divided into Leftist and rightist campCONGRESS SOCIALIST Within the Congress some leaders formed the Congress Socialist partyPattavi Sitaramyya Sardar Patel Rajendra Prasad had hostile attitude towards the Congress Socialist partyJawaharlals attitude was hesitant

1 QUESTION ndash Mention name of two Congress leaders who had socialist leaning

1ANSWER ndash Subhas Chandra Bose and Jawaharlal Nehru2QUESTION- In which session of the congress Jawaharlal elaborated his Socialist ideas2 ANSWER ndash Lucknow and Faizpur Session in December 1935 and 19363QUESTION ndash Why Congress was sharply divided into leftist and rightist camp 3ANSWER ndash Subhas Chandra Bosersquos attempt to seek re election for congress presidentship in 1939sharply divided the National Congress into Leftist and Rightist camp4 QUESTION ndash Who was MN Roy 4 ANSWER ndash Manabendra Roy first formed the Communist Party of India outside the country at Tashkent in 19205QUESTION ndash Who formed the Congress Socialist Party within the Congress5 ANSWER ndash Jaya Prakash Narayan Achyut Patwardhan Acharya Narendra Dev Ram Mohan Lohia Aruna Asaf Ali6QUESTION ndash When was the Congress Socialist Party formed What was its object6 ANSWER ndash 1934The Congress Socialist Party sought to work out socialist programme through the Congress They joined hands with the Congress and wanted to carry

Subhas Chandra Bose being expelled from the congress after the Tripuri rift he formed Forward BlockThere were basic differences between the Congress Socialists and the communistsTRADE UNION ACTIVITIES Maximum working class people lived in Bombay and Calcutta The working and living conditions of those workers were very miserable In this situation Shasipada Banerjee NM Lokhande protested against the oppression of the working class peopleThe first Trade Union Madras Labour Union was formed in 1918 by BP WadiaIndustrial strikes took place in Kanpur Calcutta Madras Jamshedpur and Ahmedabad AITUC was formed in Bombay in 1927 The growth of Trade union among the workers was slow because of the fear of the dismissal of the jobIn the mean time the Moderates as well as Communists left AITUC and formed separate organization

on National struggle with the help of workers and peasant class of the society7 QUESTION ndash What was the name of the party founded by Subhas Chandra Bose7 ANSWER- Forward Block8QUESTION ndash Who was Shasipada Banerjee8 ANSWER ndash Shasipada Banerjee was a radical Brahmo He founded a working menrsquos club to protest against exploitation of the British rulers towards the working class of India9 QUESTION ndash What was the weekly published by NM Lokhande9ANSWER- Dinabandhu10 QUESTION ndash Who founded Bombay Mill-Hands Association and in which year10 ANSWER- NM Lokhande in189011 QUESTION- Who was BP WadiaANSWER- BPWadia was the founder of Madras Labour Union in191812 QUESTION- What was the name of the first labour union of India12 ANSWER- Madras Labour Union13 QUESTION Who founded the Majur Mahajan 13 ANSWER GANDHIJI14 QUESTION What was the full form of AITUC When it was formed14 ANSWER All India Trade Union Congressin 192715QUESTION Who formed the Red Trade Union Congress and in which year15ANSWER The Communists formed the Red Trade Union Congress16 QUESTION What do you mean by Socialism16 ANSWER Socialism describes any political and economic theory that says the community rather than individuals should own and manage property and natural resources

Subject Eng Literature (The Tempest ndash William Shakespeare) Topic Act III Scene 3 Lines 1 to 52 (Line 52 ndash Brother my lord the Duke Stand to and do as we) Date 13th April 2020 (4th Period)

[Students should read the original play and also the paraphrase given in the school prescribed textbook]Summary Questions amp Answers

o Alonso Sebastian Antonio Gonzalo Adrian Francisco and others wandered about the island in search of Ferdinand and gets tired and hungry of the toil and at the same time gives up all hope of finding him

o Antonio and Sebastian are happy that Alonso is out of hope and decide to make another attempt on his life that night when being so tired they will be sleeping soundly

o Suddenly a solemn and strange music is heard in the air and several strange shapes enter bringing in a banquet These strange shapes then dance round it with gestures of salutation and then inviting the King to eat they depart

o Seeing this strange scene all are inclined to believe the tales told by travelers that there truly are ldquounicornsrdquo and ldquothe phoenixrsquo thronerdquo

1 ALONSO What harmony is this My good friends hark (L18-27)

GONZALO Marvellous sweet music

[Enter several strange shapes bringing in a banquet

they dance about it with gentle actions of salutation

and inviting the King and his companions to eat they depart]ALONSO Give us kind keepers heavens What were theseSEBASTIAN A living drollery Now I will believe

That there are unicorns that in Arabia

There is one tree the phoenixrsquo throne one phoenix

At this hour reigning thereANTONIO Ill believe both

And what does else want credit come to me

And Ill be sworn rsquotis true Travellers neer did lie

Though fools at home condemn rsquoem

(a) How did Prospero present an amazing spectacle before Alonso and his companions

Using his magic powers Prospero ordered strange shapes to lay a banquet before Alonso and his companions The shapes brought several dishes with tasty eatables in them They placed the dishes on a table before Alonso and his companions Then the strange shapes began to dance gracefully around the banquet While dancing they made gestures inviting them to eat the food Then suddenly the shapes disappeared(b) Who were the guests at the strange banquet Describe the lsquoliving drolleryrsquo

Alonso Sebastian Antonio Gonzalo Adrian and Francisco were the guests at the strange banquet

The term ldquoliving drolleryrdquo refers to live entertainment show In this context when Alonso the King of Naples Sebastian his brother Antonio the treacherous brother of Prospero Gonzalo the kind and loyal councillor to the King Adrian and Francisco came to the island they were hungry and weary in their spirits They heard a solemn and strange music They were shocked to see several strange shapes bringing in a banquet and these shapes danced about it with gentle action of salutation inviting the King and his companions to eat After this Sebastian described this show as lsquoliving drolleryrsquo(c) What is lsquophoenixrsquo What are lsquoUnicornsrdquo

The term lsquophoenixrsquo refers to a mythical Arabian bird which lived alone and perched on a solitary tree After one hundred years it expired in flames and rose again from its own ashes

lsquoUnicornsrsquo refers to the mythological four-footed beasts having horns in the centre of their foreheads When the horns are ground into powder the powder was believed to be

an aphrodisiac(d) How does Sebastian explain the puppet show OR Why does the speaker now believe in unicorns and phoenix

Sebastian finds several strange shapes bringing in the banquet They invite the king and his party for dinner and soon depart He tells that if such a strange sight can be a reality there is nothing incredible in the world and from the present moment he will believe anything He says that it is a strange dumb show enacted not by puppets but by living beings It is stranger than a travellerrsquos tale Seeing such a thing

before his own eyes he will no longer disbelieve the story about unicorns and phoenix(e) How do the other characters present respond to this living drollery

At the sight of the lsquoliving drolleryrsquo like Sebastian Gonzalo and Antonio too acted strangely Antonio told that he too now believes in unicorns and phoenix and anything else that seems to be incredible He too now believes in travellersrsquo tales Gonzalo told that if he would report those happenings in Naples nobody will believe him He considers that those gentle shapes were gentler in manner in comparison to the living beings Alonso was at first sight suspicious and told them that those strange shapes conveyed their meaning in expressive gestures when they seemed to lack speech by their movements and sounds Francisco was amazed at their mysterious disappearance

2 ALONSO Not I

(Line 43-52)GONZALO Faith sir you need not fear When we

were boysWho would believe that there were mountaineers

Dewlapped like bulls whose throats had hanging at rsquoem

Wallets of flesh Or that there were such men

Whose heads stood in their breasts Which now we find

Each putter-out of five for one will bring us

Good warrant ofALONSO I will stand to and feed

Although my lastmdashno matter since I feel

The best is past Brother my lord the Duke

Stand to and do as we

(a) How does Alonso respond at the spectacle of the shapes which were sent to them at the instruction of Prospero

After seeing the strange sight of appearing and disappearing of the shapes sent by Prospero to arrange a banquet for them Alonso says that his surprise at having seen those creatures is infinite and he is fully justified in feeling so much surprise He thinks that their shapes their gestures and the sounds they made were indeed amazing Although they do not possess the gift of speech yet they were able to convey their

thoughts by means of their gestures only

(b) What does Prospero say about the views expressed by Alonso regarding the shapes What does Francisco think about the shapesAfter hearing Alonsorsquos views about the shapes Prospero says that this manrsquos praise of the spirits is rather hasty He means to say that Alonso has shown great haste in reaching the conclusion about the shapes Francisco is amazed to see that those shapes disappeared in a mysterious way(c) What does Sebastian ask Alonso to doSebastian tells Alonso that the shapes having disappeared should not matter to them because they have left the eatables behind He asks Alonso to enjoy eating as they are extremely hungry but the king does not accept his offer of enjoying the dishes(d) How does Gonzalo try to dispel Alonsorsquos fear of those strange shapes What kind of references does he give to AlonsoGonzalo says that those who have travelled abroad have reported seeing even stranger sights than these shapes that Alonso and his companions have beheld Hence there is no reason to feel afraid of these shapes Gonzalo further adds that in his younger days he had heard strange stories from travelers and Alonso might have heard similar stories For instance it was said that there existed a certain race of

human beings who had huge lumps of flesh hanging at their throats and who therefore resembled bulls Then Gonzalo tells about a race of human beings whose heads were located at their breasts Gonzalo says that such stories were not believed by most people in those days but now-a-days these stories have become common(e) Explain the following lsquoEach putter-out of five for onersquoEnglish travellers often insured their trips with London brokers Those that went on foreign travels those days used to deposit a certain amount with some firm or company in London before their departure If the travelers failed to return the money was forfeited by the company with which it had been deposited But this money was repaid five-fold if the travelers returned safe and sound In this way a traveler stood a great chance of recovering the entire cost of his

travels(f) Give the explanatory meanings of the following expressions in the context of the above extract (i) Dewlapped (ii) Wallets of flesh

(iii) Putter-out(i) Dewlapped having big lumps of flesh at the necks(ii) Wallets of flesh large masses of flesh looking like bags(iii) Putter-out to invest money before commencing the travel

  • General methods of preparation of hydrogen
  • Chapter Dimensional Analysis (Summary)
    • Properties of Charge
Page 48:   · Web viewSubject. Topic. Summary. Execution. Hindi. व्याकरण. शरीरके अंगो के नाम लिखिए. 1) आँख 2) नाक 3

the natural death of an organism- OrganismsLife span1 Butterfly 1 - 2 weeks2 Fruit fly 30 days3Dog 10-13 years4 Rose5-7 years5 Tortoise100-150 years6 Banyan Tree -200 - 250 yearsQ4 Reproduction is of two types in case ofanimals but in case of plants vegetative propagation is also present

Asexual Reproduction Sexual Reproductioni) Always uniparentalii) Gametes are not involvediii) Only mitotic division involvediv) Somatic cells of parents are involvedv) Offsprings are genetically similar to the parents

i) Usually biparentalii) Gametes are involvediii) Meiosis occurs during gametogenesis Mitosis occurs after fertilisationiv) Germ cells of the parents are involvedv) offsprings are genetically different from the parents

COMMERCE BUSINESS ENVIRONMENT

Welcome to the new sessiontoday we are going to start the first chapter of Class XII The name of the chapter is Business Environment

Already many of you have got some idea about the word business environment form the first chapter of business studies in class XI

In todayrsquos world every business enterprise is a part of the society It exists and operates in association with various groups in society such as customers suppliers competitors banks and financial institutions government agencies trade unions media and so on All these groups influence the functioning of business in one way or the other They constitute the environment of businessConcept of Business Environment

The term lsquobusiness environmentrsquo refers to the sum total of all individuals institutions and other forces that lie outside a business enterprise but that may influence its functioning and performance

The main features of business environment Totality of External forces General and Specific forces Interrelatedness Complexity Dynamic Uncertainty

Prepare the following questions from todayrsquos assignment

2 What do you mean by business environment

The term lsquobusiness environmentrsquo means the aggregate of all forces factors and institutions which are external to and beyond the control of an individual business enterprise but they may influence its functioning and performance Business environment is the macro framework within which a business firm a micro unit operates It consists of several interrelated and interacting elements

2 Explain the main features of business environment in brief

Totality of External forces-Business environment is the sum total of all things external to a business environment

General and Specific forces-It

Relativity

The Interrelation between business and its environment

The business enterprise is an open system It continuously interacts with its environment It takes inputs (such as raw materials capital labour energy and so on) from its environment transforms them into goods and services and sends them back to the environment

Fig 1 Business Environment Relationship

includes both the forces general forces are the economic social political legal and technological conditions which indirectly influence all business enterprise Specific forces are the investors customers competitors and suppliers which influence individual enterprise directly

Interrelatedness-Different elements of environment are interrelated for an example growing awareness for health care has increased the demand for health foods

Complexity- Business environment id complex in nature as the elements keep on changing example economic technological and other forces changes in demand for a product and service

Dynamic-Business environment is not static it keeps on changing

Uncertainty- Itrsquos very difficult to predict future events such as technology and fashion which occur fast and frequently

Business Studies

Human Resources Management

Human resource of an organisation are the aggregate of knowledge skills attitudes of people working in it

The management system which deals with human resources is called human resource management

Features of HRMbullComprehensive functionbullPeople-oriented

Question1) What do you mean by human

resource management Answer) Human resource management may be defined as that field of Management which has to do with planning organising and controlling the functions of procuring developing maintaining and utilising the labour force

bullAction oriented bullPervasive function bullContinuous function

2) Explain the features of HRM in brief

Answer)bullHuman Resource Management is concerned with managing people at work bull Human Resource Management is concerned with employees which bring people and organisations together so that the goals of each are met bullHuman resource management considered every employees as an individual and also promote their satisfaction and growth bull Human resource management is inherent in all organisations and at all levelsbullManagement of human resources are ongoing on never ending process which requires a constant alertness and Awareness of human relations

3) ldquoHR function is said to be pervasiverdquowhy

Answer) Human resource management is required in all organisations whether it is private or government organisations armed forces sports organisations etc It permeatsall the functional areas like production marketing finance research etc This from this feature of human resource management it can be said that it is pervasive in nature

Economics Demand Q1DEFINITION OF DEMANDIn economics demand is the quantity of a good that consumers are willing and able to purchase at various prices during a given period of timeQ2DEMAND CURVEIn economics a demand curve is a graph depicting the relationship between the price of a certain commodity and the quantity of that commodity that is demanded at that pricQ3LAW OF DEMANDIn microeconomics the law of demand states that conditional on all else being equal as the price of a good increases quantity demanded decreases conversely as the price of a good decreases quantity demanded increasesQ4ASSUMPTION of LAW OF DEMAND(i)No change in price of related commodities(ii) No change in income of the consumer(iii) No change in taste and preferences customs habit and fashion of the consumer( No expectation regarding future change in priceQ5MARKET DEMAND SCHEDULEIn economics a market demand schedule is a tabulation of the quantity of a good that all consumers in a market will purchase at a

given price At any given price the corresponding value on the demand schedule is the sum of all consumersrsquo quantities demanded at that priceQ6INDIVIDUAL DEMAND SCHEDULEIndividual demand schedule refers to a tabular statement showing various quantities of a commodity that a consumer is willing to buy at various levels of price during a given period of timeQ7 FACTORS AFFECTING INDIVIDUAL DEMAND FOR A COMMODITY

The factors that influence a consumerrsquos decision to purchase a commodity are also known as determinants of demand The following factors affect the individual demand for a commodity1 price of the commodity2 price of related goods3 income of buyer of the commodity4 tastes and preferences of the buyer1 Price of the CommodityYou must have observed that when price of a commodity falls you tend to buy more of it and when its price rises you tend to buy less of it when all other factors remain constant (lsquoother things remaining the samersquo) In other words other things remaining the same there is an inverse relationship between the price of a commodity and its quantity demanded by its buyers This statement is in accordance with law of demand which you will study in the later part of this lesson Price of a commodity and its quantity demanded by its buyers are inversely related only when lsquoother things remain the samersquo So lsquoother things remaining the samersquo is an assumption when we study the effect of changes in the price of a commodity on its quantity demanded2 Price of Related goodsA consumer may demand a particular good But while buying that good heshe also asks the price of its related goods Related goods can be of two types-(i) Substitute goods(ii) Complementary goods While purchasing a good prices of its substitutes and complements do affect its quantity purchased(i) Price of Substitute Goods Substitute goods are those goods which can easily be used in place of one another for satisfaction of a particular want like tea and coffee An increase in price of substitute good leads to an increase in demand for the given commodity and a decrease in price of substitute good leads to a decrease in demand for the given commodity It means demand for a given commodity is directly affected by change in price of substitute goods For example if price of coffee increases the demand for tea will rise as tea will become relatively cheaper in comparison to coffee(ii) Price of Complementary goods Complementary goods are those goods which are used together to satisfy a particular want like car and petrol An increase in the price of complementary goods leads to a decrease in demand for the given commodity and a decrease in the price of complementary goods leads to an increase in demand for the given commodity For example if price of petrol falls then the demand for cars will increase as it will be relatively cheaper to use both the goods together So demand for a given commodity is inversely affected by change in price of complementary goods3 Income of the Buyer of CommodityDemand for a commodity is also affected by income of its buyer However the effect of change in income on demand depends on the nature of the commodity under consideration In case of some goods like full cream milk fine quality of rice (Basmati rice) etc demand for these commodities increases when income of the buyer increases and

demand for these commodities decreases when income of the buyer decreases Such goods whose demand increases with the increase in income of the buyer are called normal goods But there are some goods like coarse rice toned milk etc whose demand decreases when income of buyer increases and their demand increases when income of the buyer decreases Such goods whose demand decreases with the increase in income of the buyer are called inferior goods Suppose a consumer buys 10 Kgs of rice whose price is ` 25 per Kg He cannot afford to buy better quality of rice because the price of such rice is ` 50 per Kg The consumer is spending ` 250 per month on the purchase of rice Now if income of the consumer increases and he can afford ` 350 on purchase of 10 Kg of rice Now he can afford to buy some quantity of rice say 6 Kgs whose price is ` 25 per Kg and may buy 4 Kgs of rice whose price is ` 50 per Kg Thus he will buy 10 Kgs of rice by spending ` 350 per month Therefore we may conclude that demand for normal goods is directly related to the income of the buyer but demand for inferior goods is inversely related to the income of the buyer4 Tastes and Preferences of the BuyerThe demand for a commodity is also affected by the tastes and preferences of the buyers They include change in fashion customs habits etc Those commodities are preferred by the consumers which are in fashion So demand for those commodities rises which are in fashion On the other hand if a commodity goes out of the fashion its demand falls because no consumer will like to buy it(5) Number of Buyers in the Market(Population)Increase in population raises the market demand whereas decrease in population reduces the market demand for a commodity Not only the size of population but its composition like age (ratio of males females children and old people in population) also affects the demand for a commodity It is because of needs of children young old male and female population differs(6) Distribution of Income and WealthIf the distribution of income and wealth is more in favour of the rich demand for the commodities preferred by the rich such as comforts and luxuries is likely to be higher On the other hand if the distribution of income and wealth is more in favour of poor demand for commodities preferred by the poor such as necessities will be more(7) Season and Weather ConditionsThis is generally observed that the demand for woolens increases during winter whereas demand for ice creams and cold drinks increases during summer Similarly market demand for umbrellas rain coats increases during rainy seasonQ8 REASONS FOR OPERATION OF LAW OF DEMAND WHY DEMAND CURVE SLOPES DOWNWARDNow we will try to explain why does a consumer purchase more quantity of a commodity at a lower price and less of it at a higher price or why does the law of demand operate ie why does the demand curve slope downwards from left to right The main reasons for operation of law of demand are1 Law of Diminishing Marginal UtilityAs you have studied earlier law of diminishing marginal utility states that as we consume more and more units of a commodity the utility derived from each successive unit goes on decreasing The consumer will be ready to pay more for those units which provide him more utility and less for those which provide him less utility It implies that he will purchase more only when the price of the commodity falls2 Income Effect

When price of a commodity falls purchasing power or real income of the consumer increases which enables him to purchase more quantity of the commodity with the same money income Let us take an example Suppose you buy 4 ice creams when price of each ice cream is ` 25 If price of ice creams falls to ` 20 then with same money income you can buy 5 ice creams now3 Substitution EffectWhen price of a commodity falls it becomes comparatively cheaper as compared to its substitutes (although price of substitutes has not been changed) This will lead to rise in demand for the given commodity For example if coke and Pepsi both are sold at ` 10 each and price of coke falls Now coke has become relatively cheaper and will be substituted for Pepsi It will lead to rise in demand for coke4 Change in Number of BuyersWhen price of a commodity falls some old buyers may demand more of the commodity at the reduced price and some new buyers may also start buying this commodity who were not in a position to buy it earlier due to higher price This will lead to increase in number of buyers when price of the commodity falls As a result demand for the commodity rises when its price falls5 Diverse Uses of a CommoditySome commodities have diverse uses like milk It can be used for drinking for sweet preparation for ice cream preparation etc If price of milk rises its use may be restricted to important purpose only This will lead to reduction in demand for other less important uses When price of milk falls it can be put to other uses also leading to rise n demand for itQ9 EXCEPTIONS TO THE LAW OF DEMANDYou have studied in law of demand that a buyer is willing to buy more quantity of a commodity at a lower price and less of it at a higher price But in certain circumstances a rise in price may lead to rise in demand These circumstances are called Exceptions to the Law of Demand Some important exceptions are1 Giffen GoodsGiffen goods are special type of inferior goods in which negative income effect is stronger than negative substitution effect Giffen goods do not follow law of demand as their demand rises when their price rises Examples of Giffen goods are jowar and bajra etc2 Status Symbol GoodsSome goods are used by rich people as status symbols eg diamonds gold jewellary etc The higher the price the higher will be the demand for these goods When price of such goods falls these goods are no longer looked at as status symbol goods and tehrefore therir demand falls3 NecessitiesCommodities such as medicines salt wheat etc do not follow law of demandbecause we have to purchase them in minimum required quantity whatever their price may be4 Goods Expected to be ScarceWhen the buyers expect a scarcity of a particular good in near future they start buying more and more of that good even if their prices are rising For example during war famines etc people tend to buy more of some goods even at higher prices due to fear of their scarcity in near future

Political Science

Constitution of India-The

Preamble

The preamble-

Preamble-

The preamble is the most precious part of the constitution We the people of India having solemnly resolved to constitute India into a Sovereign Socialist Secular Democratic Republic and to secure to all its citizensA preamble is an introductory and expressionary statement in a document that explains the documents purpose and underlying philosophy When applied to the opening paragraphs of a statute it may recite historical facts pertinent to the subject of the statuteNature and purpose of the constitution-Purpose of the Constitution dictates permanent framework of the government to form a more perfect union to establish justice and ensure peace of thenationconstitution provide principles how the government can run itself following the rules and laws written in the constitution of each state keeps them balanced

Answer the following questions-

1 What is preambleA preamble is an introductory and expressionary statement in a document that explains the documents purpose and underlying philosophy2 What is the nature and

purpose of the constitutionConstitution dictatespermanent framework of the government to form a more perfect union to establish justice and ensure peace of the nation

Homework-Learn

Accounts Compatibilty mode

1MEANING OF PARTNERSHIPPartnership is a form of business organisation where two or more persons join hands to run a business They share the profits and losses according to the agreement amongst them According to the Indian Partnership Act 1932 ldquoPartnership is relation between persons who have agreed to share profits of a business carried on by all or any one of them acting for allrdquo For example one of your friends has passed class XII from National Institute of Open Schooling (NIOS) and wants to start a business Heshe approaches you to join in this venture Heshe wants you to contribute some money and participate in the business activities Both of you if join hands constitute a partnership2CHARACTERISTICS1048698 Agreement A partnership is formed by an agreement The agreement may be either oral or in writing It defines the relationship between the persons who agree to carry on business It may contain the terms of sharing profit and the capital to be invested by each partner etc The written agreement is known as partnership deed1048698 Number of persons There must be at least two persons to form a partnership

The maximum number of partners in a partnership firm can be 50 according toCompanies Act 20131048698 Business The Partnership is formed to carry on business with a purpose of earning profits The business should be lawful Thus if two or more persons agree to carry on unlawful activities it will not be termed as partnership1048698 Sharing Profits The partners agree to share profits in the agreed ratio In caseof loss all the partners have to bear it in the same agreed profit sharing ratio10486981048698Mutual Agency Every partner is an agent of the other partners Every partner can bind the firm and all other partners by hisher acts Each partner will be responsible and liable for the acts of all other partners10486981048698Unlimited liability The liability of each partner except that of a minor is unlimited Their liability extends to their personal assets also If the assets of the firm are insufficient to pay off its debts the partnersrsquo personal property can be used to satisfy the claim of the creditors of the partnership firm10486981048698Management All the partners have a right to mange the business However they may authorize one or more partners to manage the affairs of the business on their behalf10486981048698Transferability of Share No partner can transfer hisher share to any one including hisher family member without the consent of all other partners3PARTNERSHIP DEEDAgreement forms the basis of partnership The written form of the agreement is which a document of partnership is It contains terms and conditions regarding the conduct of the business It also explains relationship between the partners This document is called partnership deed Every firm can frame its own partnership deed in which the rights duties and liabilities of the partners are stated in detail It helps in settling the disputes arising among the partners during the general conduct of business 4CONTENTS OF PARTNERSHIP DEEDThe partnership deed generally contains the following (i) Name and address of the partnership firm(ii) Nature and objectives of the business(iii) Name and address of each partner(iv) Ratio in which profits is to be shared(v) Capital contribution by each partner(vi) Rate of Interest on capital if allowed(vii) Salary or any other remuneration to partners if allowed(viii) Rate of interest on loans and advances by a partner to the firm(ix) Drawings of partners and interest thereon if any(x) Method of valuation of goodwill and revaluation of assets and liabilities on the reconstitution of the partnership ie on the admission retirement or death of a partner(xi) Settlement of disputes by arbitration(xii) Settlement of accounts at the time of retirement or death of a partner5IN ABSENCE OF PARTNERSHIP DEEDThe partnership deed lays down the terms and conditions of partnership in regard to rights duties and obligations of the partners In the absence of partnership deed there may arise a controversy on certain issues like profit sharing ratio interest on

capital interest on drawings interest on loan and salary of the partners In such cases the provisions of the Indian Partnership Act becomes applicableSome of the Issues are(i) Distribution of Profit Partners are entitled to share profits equally(ii) Interest on Capital Interest on capital is not allowed(iii) Interest on Drawings No interest on drawing of the partners is to be charged(iv) Interest on Partnerrsquos Loan A Partner is allowed interest 6 per annum on the amount of loan given to the firm by himher(v) Salary and Commission to Partner A partner is not entitled to anysalary or commission or any other remuneration for managing the business

History TOPIC-TOWARDS INDEPENDENCE AND PARTITION THE LAST PHASE (1935-1947)

SUB TOPIC-IMPORTANT POLITICAL DEVELOPMENTS ndash GROWTH OF SOCIAL IDEAS

Socialism is a political social and economic philosophyLike in other parts of the world the Russian revolution of 1917 served as a great inspiration for revolutionaries in India who at that time were engaged in the struggle for liberation from British ruleSocialist ideas led to the formation of communist party of IndiaJAWAHARLAL NEHRU Among the early Congress leaders Jawaharlal Nehru was very much impressed and influenced by the Socialist ideas He also learnt about the Economic activities of the Soviet Union after the Bolshevic Revolution 1917 He made full use of them in IndiaThe election of Jawaharlal Nehru and Subhas Chandra Bose showed the Left wing tendency within CongressJawaharlal Nehru demanded economic freedom along with political freedom of the people in order to end the exploitation of masses

Nehrus working committee included three socialists leaders The Lucknow session was a landmark in the evolution of socialist ideas of the congressSUBHAS CHANDRA BOSE ndash Subhas Chandra Bose had socialist leaning Both Jawaharlal Nehru and Subhas Chandra Bose were known as leftist Congress men Later on National Congress divided into Leftist and rightist campCONGRESS SOCIALIST Within the Congress some leaders formed the Congress Socialist partyPattavi Sitaramyya Sardar Patel Rajendra Prasad had hostile attitude towards the Congress Socialist partyJawaharlals attitude was hesitant

1 QUESTION ndash Mention name of two Congress leaders who had socialist leaning

1ANSWER ndash Subhas Chandra Bose and Jawaharlal Nehru2QUESTION- In which session of the congress Jawaharlal elaborated his Socialist ideas2 ANSWER ndash Lucknow and Faizpur Session in December 1935 and 19363QUESTION ndash Why Congress was sharply divided into leftist and rightist camp 3ANSWER ndash Subhas Chandra Bosersquos attempt to seek re election for congress presidentship in 1939sharply divided the National Congress into Leftist and Rightist camp4 QUESTION ndash Who was MN Roy 4 ANSWER ndash Manabendra Roy first formed the Communist Party of India outside the country at Tashkent in 19205QUESTION ndash Who formed the Congress Socialist Party within the Congress5 ANSWER ndash Jaya Prakash Narayan Achyut Patwardhan Acharya Narendra Dev Ram Mohan Lohia Aruna Asaf Ali6QUESTION ndash When was the Congress Socialist Party formed What was its object6 ANSWER ndash 1934The Congress Socialist Party sought to work out socialist programme through the Congress They joined hands with the Congress and wanted to carry

Subhas Chandra Bose being expelled from the congress after the Tripuri rift he formed Forward BlockThere were basic differences between the Congress Socialists and the communistsTRADE UNION ACTIVITIES Maximum working class people lived in Bombay and Calcutta The working and living conditions of those workers were very miserable In this situation Shasipada Banerjee NM Lokhande protested against the oppression of the working class peopleThe first Trade Union Madras Labour Union was formed in 1918 by BP WadiaIndustrial strikes took place in Kanpur Calcutta Madras Jamshedpur and Ahmedabad AITUC was formed in Bombay in 1927 The growth of Trade union among the workers was slow because of the fear of the dismissal of the jobIn the mean time the Moderates as well as Communists left AITUC and formed separate organization

on National struggle with the help of workers and peasant class of the society7 QUESTION ndash What was the name of the party founded by Subhas Chandra Bose7 ANSWER- Forward Block8QUESTION ndash Who was Shasipada Banerjee8 ANSWER ndash Shasipada Banerjee was a radical Brahmo He founded a working menrsquos club to protest against exploitation of the British rulers towards the working class of India9 QUESTION ndash What was the weekly published by NM Lokhande9ANSWER- Dinabandhu10 QUESTION ndash Who founded Bombay Mill-Hands Association and in which year10 ANSWER- NM Lokhande in189011 QUESTION- Who was BP WadiaANSWER- BPWadia was the founder of Madras Labour Union in191812 QUESTION- What was the name of the first labour union of India12 ANSWER- Madras Labour Union13 QUESTION Who founded the Majur Mahajan 13 ANSWER GANDHIJI14 QUESTION What was the full form of AITUC When it was formed14 ANSWER All India Trade Union Congressin 192715QUESTION Who formed the Red Trade Union Congress and in which year15ANSWER The Communists formed the Red Trade Union Congress16 QUESTION What do you mean by Socialism16 ANSWER Socialism describes any political and economic theory that says the community rather than individuals should own and manage property and natural resources

Subject Eng Literature (The Tempest ndash William Shakespeare) Topic Act III Scene 3 Lines 1 to 52 (Line 52 ndash Brother my lord the Duke Stand to and do as we) Date 13th April 2020 (4th Period)

[Students should read the original play and also the paraphrase given in the school prescribed textbook]Summary Questions amp Answers

o Alonso Sebastian Antonio Gonzalo Adrian Francisco and others wandered about the island in search of Ferdinand and gets tired and hungry of the toil and at the same time gives up all hope of finding him

o Antonio and Sebastian are happy that Alonso is out of hope and decide to make another attempt on his life that night when being so tired they will be sleeping soundly

o Suddenly a solemn and strange music is heard in the air and several strange shapes enter bringing in a banquet These strange shapes then dance round it with gestures of salutation and then inviting the King to eat they depart

o Seeing this strange scene all are inclined to believe the tales told by travelers that there truly are ldquounicornsrdquo and ldquothe phoenixrsquo thronerdquo

1 ALONSO What harmony is this My good friends hark (L18-27)

GONZALO Marvellous sweet music

[Enter several strange shapes bringing in a banquet

they dance about it with gentle actions of salutation

and inviting the King and his companions to eat they depart]ALONSO Give us kind keepers heavens What were theseSEBASTIAN A living drollery Now I will believe

That there are unicorns that in Arabia

There is one tree the phoenixrsquo throne one phoenix

At this hour reigning thereANTONIO Ill believe both

And what does else want credit come to me

And Ill be sworn rsquotis true Travellers neer did lie

Though fools at home condemn rsquoem

(a) How did Prospero present an amazing spectacle before Alonso and his companions

Using his magic powers Prospero ordered strange shapes to lay a banquet before Alonso and his companions The shapes brought several dishes with tasty eatables in them They placed the dishes on a table before Alonso and his companions Then the strange shapes began to dance gracefully around the banquet While dancing they made gestures inviting them to eat the food Then suddenly the shapes disappeared(b) Who were the guests at the strange banquet Describe the lsquoliving drolleryrsquo

Alonso Sebastian Antonio Gonzalo Adrian and Francisco were the guests at the strange banquet

The term ldquoliving drolleryrdquo refers to live entertainment show In this context when Alonso the King of Naples Sebastian his brother Antonio the treacherous brother of Prospero Gonzalo the kind and loyal councillor to the King Adrian and Francisco came to the island they were hungry and weary in their spirits They heard a solemn and strange music They were shocked to see several strange shapes bringing in a banquet and these shapes danced about it with gentle action of salutation inviting the King and his companions to eat After this Sebastian described this show as lsquoliving drolleryrsquo(c) What is lsquophoenixrsquo What are lsquoUnicornsrdquo

The term lsquophoenixrsquo refers to a mythical Arabian bird which lived alone and perched on a solitary tree After one hundred years it expired in flames and rose again from its own ashes

lsquoUnicornsrsquo refers to the mythological four-footed beasts having horns in the centre of their foreheads When the horns are ground into powder the powder was believed to be

an aphrodisiac(d) How does Sebastian explain the puppet show OR Why does the speaker now believe in unicorns and phoenix

Sebastian finds several strange shapes bringing in the banquet They invite the king and his party for dinner and soon depart He tells that if such a strange sight can be a reality there is nothing incredible in the world and from the present moment he will believe anything He says that it is a strange dumb show enacted not by puppets but by living beings It is stranger than a travellerrsquos tale Seeing such a thing

before his own eyes he will no longer disbelieve the story about unicorns and phoenix(e) How do the other characters present respond to this living drollery

At the sight of the lsquoliving drolleryrsquo like Sebastian Gonzalo and Antonio too acted strangely Antonio told that he too now believes in unicorns and phoenix and anything else that seems to be incredible He too now believes in travellersrsquo tales Gonzalo told that if he would report those happenings in Naples nobody will believe him He considers that those gentle shapes were gentler in manner in comparison to the living beings Alonso was at first sight suspicious and told them that those strange shapes conveyed their meaning in expressive gestures when they seemed to lack speech by their movements and sounds Francisco was amazed at their mysterious disappearance

2 ALONSO Not I

(Line 43-52)GONZALO Faith sir you need not fear When we

were boysWho would believe that there were mountaineers

Dewlapped like bulls whose throats had hanging at rsquoem

Wallets of flesh Or that there were such men

Whose heads stood in their breasts Which now we find

Each putter-out of five for one will bring us

Good warrant ofALONSO I will stand to and feed

Although my lastmdashno matter since I feel

The best is past Brother my lord the Duke

Stand to and do as we

(a) How does Alonso respond at the spectacle of the shapes which were sent to them at the instruction of Prospero

After seeing the strange sight of appearing and disappearing of the shapes sent by Prospero to arrange a banquet for them Alonso says that his surprise at having seen those creatures is infinite and he is fully justified in feeling so much surprise He thinks that their shapes their gestures and the sounds they made were indeed amazing Although they do not possess the gift of speech yet they were able to convey their

thoughts by means of their gestures only

(b) What does Prospero say about the views expressed by Alonso regarding the shapes What does Francisco think about the shapesAfter hearing Alonsorsquos views about the shapes Prospero says that this manrsquos praise of the spirits is rather hasty He means to say that Alonso has shown great haste in reaching the conclusion about the shapes Francisco is amazed to see that those shapes disappeared in a mysterious way(c) What does Sebastian ask Alonso to doSebastian tells Alonso that the shapes having disappeared should not matter to them because they have left the eatables behind He asks Alonso to enjoy eating as they are extremely hungry but the king does not accept his offer of enjoying the dishes(d) How does Gonzalo try to dispel Alonsorsquos fear of those strange shapes What kind of references does he give to AlonsoGonzalo says that those who have travelled abroad have reported seeing even stranger sights than these shapes that Alonso and his companions have beheld Hence there is no reason to feel afraid of these shapes Gonzalo further adds that in his younger days he had heard strange stories from travelers and Alonso might have heard similar stories For instance it was said that there existed a certain race of

human beings who had huge lumps of flesh hanging at their throats and who therefore resembled bulls Then Gonzalo tells about a race of human beings whose heads were located at their breasts Gonzalo says that such stories were not believed by most people in those days but now-a-days these stories have become common(e) Explain the following lsquoEach putter-out of five for onersquoEnglish travellers often insured their trips with London brokers Those that went on foreign travels those days used to deposit a certain amount with some firm or company in London before their departure If the travelers failed to return the money was forfeited by the company with which it had been deposited But this money was repaid five-fold if the travelers returned safe and sound In this way a traveler stood a great chance of recovering the entire cost of his

travels(f) Give the explanatory meanings of the following expressions in the context of the above extract (i) Dewlapped (ii) Wallets of flesh

(iii) Putter-out(i) Dewlapped having big lumps of flesh at the necks(ii) Wallets of flesh large masses of flesh looking like bags(iii) Putter-out to invest money before commencing the travel

  • General methods of preparation of hydrogen
  • Chapter Dimensional Analysis (Summary)
    • Properties of Charge
Page 49:   · Web viewSubject. Topic. Summary. Execution. Hindi. व्याकरण. शरीरके अंगो के नाम लिखिए. 1) आँख 2) नाक 3

Relativity

The Interrelation between business and its environment

The business enterprise is an open system It continuously interacts with its environment It takes inputs (such as raw materials capital labour energy and so on) from its environment transforms them into goods and services and sends them back to the environment

Fig 1 Business Environment Relationship

includes both the forces general forces are the economic social political legal and technological conditions which indirectly influence all business enterprise Specific forces are the investors customers competitors and suppliers which influence individual enterprise directly

Interrelatedness-Different elements of environment are interrelated for an example growing awareness for health care has increased the demand for health foods

Complexity- Business environment id complex in nature as the elements keep on changing example economic technological and other forces changes in demand for a product and service

Dynamic-Business environment is not static it keeps on changing

Uncertainty- Itrsquos very difficult to predict future events such as technology and fashion which occur fast and frequently

Business Studies

Human Resources Management

Human resource of an organisation are the aggregate of knowledge skills attitudes of people working in it

The management system which deals with human resources is called human resource management

Features of HRMbullComprehensive functionbullPeople-oriented

Question1) What do you mean by human

resource management Answer) Human resource management may be defined as that field of Management which has to do with planning organising and controlling the functions of procuring developing maintaining and utilising the labour force

bullAction oriented bullPervasive function bullContinuous function

2) Explain the features of HRM in brief

Answer)bullHuman Resource Management is concerned with managing people at work bull Human Resource Management is concerned with employees which bring people and organisations together so that the goals of each are met bullHuman resource management considered every employees as an individual and also promote their satisfaction and growth bull Human resource management is inherent in all organisations and at all levelsbullManagement of human resources are ongoing on never ending process which requires a constant alertness and Awareness of human relations

3) ldquoHR function is said to be pervasiverdquowhy

Answer) Human resource management is required in all organisations whether it is private or government organisations armed forces sports organisations etc It permeatsall the functional areas like production marketing finance research etc This from this feature of human resource management it can be said that it is pervasive in nature

Economics Demand Q1DEFINITION OF DEMANDIn economics demand is the quantity of a good that consumers are willing and able to purchase at various prices during a given period of timeQ2DEMAND CURVEIn economics a demand curve is a graph depicting the relationship between the price of a certain commodity and the quantity of that commodity that is demanded at that pricQ3LAW OF DEMANDIn microeconomics the law of demand states that conditional on all else being equal as the price of a good increases quantity demanded decreases conversely as the price of a good decreases quantity demanded increasesQ4ASSUMPTION of LAW OF DEMAND(i)No change in price of related commodities(ii) No change in income of the consumer(iii) No change in taste and preferences customs habit and fashion of the consumer( No expectation regarding future change in priceQ5MARKET DEMAND SCHEDULEIn economics a market demand schedule is a tabulation of the quantity of a good that all consumers in a market will purchase at a

given price At any given price the corresponding value on the demand schedule is the sum of all consumersrsquo quantities demanded at that priceQ6INDIVIDUAL DEMAND SCHEDULEIndividual demand schedule refers to a tabular statement showing various quantities of a commodity that a consumer is willing to buy at various levels of price during a given period of timeQ7 FACTORS AFFECTING INDIVIDUAL DEMAND FOR A COMMODITY

The factors that influence a consumerrsquos decision to purchase a commodity are also known as determinants of demand The following factors affect the individual demand for a commodity1 price of the commodity2 price of related goods3 income of buyer of the commodity4 tastes and preferences of the buyer1 Price of the CommodityYou must have observed that when price of a commodity falls you tend to buy more of it and when its price rises you tend to buy less of it when all other factors remain constant (lsquoother things remaining the samersquo) In other words other things remaining the same there is an inverse relationship between the price of a commodity and its quantity demanded by its buyers This statement is in accordance with law of demand which you will study in the later part of this lesson Price of a commodity and its quantity demanded by its buyers are inversely related only when lsquoother things remain the samersquo So lsquoother things remaining the samersquo is an assumption when we study the effect of changes in the price of a commodity on its quantity demanded2 Price of Related goodsA consumer may demand a particular good But while buying that good heshe also asks the price of its related goods Related goods can be of two types-(i) Substitute goods(ii) Complementary goods While purchasing a good prices of its substitutes and complements do affect its quantity purchased(i) Price of Substitute Goods Substitute goods are those goods which can easily be used in place of one another for satisfaction of a particular want like tea and coffee An increase in price of substitute good leads to an increase in demand for the given commodity and a decrease in price of substitute good leads to a decrease in demand for the given commodity It means demand for a given commodity is directly affected by change in price of substitute goods For example if price of coffee increases the demand for tea will rise as tea will become relatively cheaper in comparison to coffee(ii) Price of Complementary goods Complementary goods are those goods which are used together to satisfy a particular want like car and petrol An increase in the price of complementary goods leads to a decrease in demand for the given commodity and a decrease in the price of complementary goods leads to an increase in demand for the given commodity For example if price of petrol falls then the demand for cars will increase as it will be relatively cheaper to use both the goods together So demand for a given commodity is inversely affected by change in price of complementary goods3 Income of the Buyer of CommodityDemand for a commodity is also affected by income of its buyer However the effect of change in income on demand depends on the nature of the commodity under consideration In case of some goods like full cream milk fine quality of rice (Basmati rice) etc demand for these commodities increases when income of the buyer increases and

demand for these commodities decreases when income of the buyer decreases Such goods whose demand increases with the increase in income of the buyer are called normal goods But there are some goods like coarse rice toned milk etc whose demand decreases when income of buyer increases and their demand increases when income of the buyer decreases Such goods whose demand decreases with the increase in income of the buyer are called inferior goods Suppose a consumer buys 10 Kgs of rice whose price is ` 25 per Kg He cannot afford to buy better quality of rice because the price of such rice is ` 50 per Kg The consumer is spending ` 250 per month on the purchase of rice Now if income of the consumer increases and he can afford ` 350 on purchase of 10 Kg of rice Now he can afford to buy some quantity of rice say 6 Kgs whose price is ` 25 per Kg and may buy 4 Kgs of rice whose price is ` 50 per Kg Thus he will buy 10 Kgs of rice by spending ` 350 per month Therefore we may conclude that demand for normal goods is directly related to the income of the buyer but demand for inferior goods is inversely related to the income of the buyer4 Tastes and Preferences of the BuyerThe demand for a commodity is also affected by the tastes and preferences of the buyers They include change in fashion customs habits etc Those commodities are preferred by the consumers which are in fashion So demand for those commodities rises which are in fashion On the other hand if a commodity goes out of the fashion its demand falls because no consumer will like to buy it(5) Number of Buyers in the Market(Population)Increase in population raises the market demand whereas decrease in population reduces the market demand for a commodity Not only the size of population but its composition like age (ratio of males females children and old people in population) also affects the demand for a commodity It is because of needs of children young old male and female population differs(6) Distribution of Income and WealthIf the distribution of income and wealth is more in favour of the rich demand for the commodities preferred by the rich such as comforts and luxuries is likely to be higher On the other hand if the distribution of income and wealth is more in favour of poor demand for commodities preferred by the poor such as necessities will be more(7) Season and Weather ConditionsThis is generally observed that the demand for woolens increases during winter whereas demand for ice creams and cold drinks increases during summer Similarly market demand for umbrellas rain coats increases during rainy seasonQ8 REASONS FOR OPERATION OF LAW OF DEMAND WHY DEMAND CURVE SLOPES DOWNWARDNow we will try to explain why does a consumer purchase more quantity of a commodity at a lower price and less of it at a higher price or why does the law of demand operate ie why does the demand curve slope downwards from left to right The main reasons for operation of law of demand are1 Law of Diminishing Marginal UtilityAs you have studied earlier law of diminishing marginal utility states that as we consume more and more units of a commodity the utility derived from each successive unit goes on decreasing The consumer will be ready to pay more for those units which provide him more utility and less for those which provide him less utility It implies that he will purchase more only when the price of the commodity falls2 Income Effect

When price of a commodity falls purchasing power or real income of the consumer increases which enables him to purchase more quantity of the commodity with the same money income Let us take an example Suppose you buy 4 ice creams when price of each ice cream is ` 25 If price of ice creams falls to ` 20 then with same money income you can buy 5 ice creams now3 Substitution EffectWhen price of a commodity falls it becomes comparatively cheaper as compared to its substitutes (although price of substitutes has not been changed) This will lead to rise in demand for the given commodity For example if coke and Pepsi both are sold at ` 10 each and price of coke falls Now coke has become relatively cheaper and will be substituted for Pepsi It will lead to rise in demand for coke4 Change in Number of BuyersWhen price of a commodity falls some old buyers may demand more of the commodity at the reduced price and some new buyers may also start buying this commodity who were not in a position to buy it earlier due to higher price This will lead to increase in number of buyers when price of the commodity falls As a result demand for the commodity rises when its price falls5 Diverse Uses of a CommoditySome commodities have diverse uses like milk It can be used for drinking for sweet preparation for ice cream preparation etc If price of milk rises its use may be restricted to important purpose only This will lead to reduction in demand for other less important uses When price of milk falls it can be put to other uses also leading to rise n demand for itQ9 EXCEPTIONS TO THE LAW OF DEMANDYou have studied in law of demand that a buyer is willing to buy more quantity of a commodity at a lower price and less of it at a higher price But in certain circumstances a rise in price may lead to rise in demand These circumstances are called Exceptions to the Law of Demand Some important exceptions are1 Giffen GoodsGiffen goods are special type of inferior goods in which negative income effect is stronger than negative substitution effect Giffen goods do not follow law of demand as their demand rises when their price rises Examples of Giffen goods are jowar and bajra etc2 Status Symbol GoodsSome goods are used by rich people as status symbols eg diamonds gold jewellary etc The higher the price the higher will be the demand for these goods When price of such goods falls these goods are no longer looked at as status symbol goods and tehrefore therir demand falls3 NecessitiesCommodities such as medicines salt wheat etc do not follow law of demandbecause we have to purchase them in minimum required quantity whatever their price may be4 Goods Expected to be ScarceWhen the buyers expect a scarcity of a particular good in near future they start buying more and more of that good even if their prices are rising For example during war famines etc people tend to buy more of some goods even at higher prices due to fear of their scarcity in near future

Political Science

Constitution of India-The

Preamble

The preamble-

Preamble-

The preamble is the most precious part of the constitution We the people of India having solemnly resolved to constitute India into a Sovereign Socialist Secular Democratic Republic and to secure to all its citizensA preamble is an introductory and expressionary statement in a document that explains the documents purpose and underlying philosophy When applied to the opening paragraphs of a statute it may recite historical facts pertinent to the subject of the statuteNature and purpose of the constitution-Purpose of the Constitution dictates permanent framework of the government to form a more perfect union to establish justice and ensure peace of thenationconstitution provide principles how the government can run itself following the rules and laws written in the constitution of each state keeps them balanced

Answer the following questions-

1 What is preambleA preamble is an introductory and expressionary statement in a document that explains the documents purpose and underlying philosophy2 What is the nature and

purpose of the constitutionConstitution dictatespermanent framework of the government to form a more perfect union to establish justice and ensure peace of the nation

Homework-Learn

Accounts Compatibilty mode

1MEANING OF PARTNERSHIPPartnership is a form of business organisation where two or more persons join hands to run a business They share the profits and losses according to the agreement amongst them According to the Indian Partnership Act 1932 ldquoPartnership is relation between persons who have agreed to share profits of a business carried on by all or any one of them acting for allrdquo For example one of your friends has passed class XII from National Institute of Open Schooling (NIOS) and wants to start a business Heshe approaches you to join in this venture Heshe wants you to contribute some money and participate in the business activities Both of you if join hands constitute a partnership2CHARACTERISTICS1048698 Agreement A partnership is formed by an agreement The agreement may be either oral or in writing It defines the relationship between the persons who agree to carry on business It may contain the terms of sharing profit and the capital to be invested by each partner etc The written agreement is known as partnership deed1048698 Number of persons There must be at least two persons to form a partnership

The maximum number of partners in a partnership firm can be 50 according toCompanies Act 20131048698 Business The Partnership is formed to carry on business with a purpose of earning profits The business should be lawful Thus if two or more persons agree to carry on unlawful activities it will not be termed as partnership1048698 Sharing Profits The partners agree to share profits in the agreed ratio In caseof loss all the partners have to bear it in the same agreed profit sharing ratio10486981048698Mutual Agency Every partner is an agent of the other partners Every partner can bind the firm and all other partners by hisher acts Each partner will be responsible and liable for the acts of all other partners10486981048698Unlimited liability The liability of each partner except that of a minor is unlimited Their liability extends to their personal assets also If the assets of the firm are insufficient to pay off its debts the partnersrsquo personal property can be used to satisfy the claim of the creditors of the partnership firm10486981048698Management All the partners have a right to mange the business However they may authorize one or more partners to manage the affairs of the business on their behalf10486981048698Transferability of Share No partner can transfer hisher share to any one including hisher family member without the consent of all other partners3PARTNERSHIP DEEDAgreement forms the basis of partnership The written form of the agreement is which a document of partnership is It contains terms and conditions regarding the conduct of the business It also explains relationship between the partners This document is called partnership deed Every firm can frame its own partnership deed in which the rights duties and liabilities of the partners are stated in detail It helps in settling the disputes arising among the partners during the general conduct of business 4CONTENTS OF PARTNERSHIP DEEDThe partnership deed generally contains the following (i) Name and address of the partnership firm(ii) Nature and objectives of the business(iii) Name and address of each partner(iv) Ratio in which profits is to be shared(v) Capital contribution by each partner(vi) Rate of Interest on capital if allowed(vii) Salary or any other remuneration to partners if allowed(viii) Rate of interest on loans and advances by a partner to the firm(ix) Drawings of partners and interest thereon if any(x) Method of valuation of goodwill and revaluation of assets and liabilities on the reconstitution of the partnership ie on the admission retirement or death of a partner(xi) Settlement of disputes by arbitration(xii) Settlement of accounts at the time of retirement or death of a partner5IN ABSENCE OF PARTNERSHIP DEEDThe partnership deed lays down the terms and conditions of partnership in regard to rights duties and obligations of the partners In the absence of partnership deed there may arise a controversy on certain issues like profit sharing ratio interest on

capital interest on drawings interest on loan and salary of the partners In such cases the provisions of the Indian Partnership Act becomes applicableSome of the Issues are(i) Distribution of Profit Partners are entitled to share profits equally(ii) Interest on Capital Interest on capital is not allowed(iii) Interest on Drawings No interest on drawing of the partners is to be charged(iv) Interest on Partnerrsquos Loan A Partner is allowed interest 6 per annum on the amount of loan given to the firm by himher(v) Salary and Commission to Partner A partner is not entitled to anysalary or commission or any other remuneration for managing the business

History TOPIC-TOWARDS INDEPENDENCE AND PARTITION THE LAST PHASE (1935-1947)

SUB TOPIC-IMPORTANT POLITICAL DEVELOPMENTS ndash GROWTH OF SOCIAL IDEAS

Socialism is a political social and economic philosophyLike in other parts of the world the Russian revolution of 1917 served as a great inspiration for revolutionaries in India who at that time were engaged in the struggle for liberation from British ruleSocialist ideas led to the formation of communist party of IndiaJAWAHARLAL NEHRU Among the early Congress leaders Jawaharlal Nehru was very much impressed and influenced by the Socialist ideas He also learnt about the Economic activities of the Soviet Union after the Bolshevic Revolution 1917 He made full use of them in IndiaThe election of Jawaharlal Nehru and Subhas Chandra Bose showed the Left wing tendency within CongressJawaharlal Nehru demanded economic freedom along with political freedom of the people in order to end the exploitation of masses

Nehrus working committee included three socialists leaders The Lucknow session was a landmark in the evolution of socialist ideas of the congressSUBHAS CHANDRA BOSE ndash Subhas Chandra Bose had socialist leaning Both Jawaharlal Nehru and Subhas Chandra Bose were known as leftist Congress men Later on National Congress divided into Leftist and rightist campCONGRESS SOCIALIST Within the Congress some leaders formed the Congress Socialist partyPattavi Sitaramyya Sardar Patel Rajendra Prasad had hostile attitude towards the Congress Socialist partyJawaharlals attitude was hesitant

1 QUESTION ndash Mention name of two Congress leaders who had socialist leaning

1ANSWER ndash Subhas Chandra Bose and Jawaharlal Nehru2QUESTION- In which session of the congress Jawaharlal elaborated his Socialist ideas2 ANSWER ndash Lucknow and Faizpur Session in December 1935 and 19363QUESTION ndash Why Congress was sharply divided into leftist and rightist camp 3ANSWER ndash Subhas Chandra Bosersquos attempt to seek re election for congress presidentship in 1939sharply divided the National Congress into Leftist and Rightist camp4 QUESTION ndash Who was MN Roy 4 ANSWER ndash Manabendra Roy first formed the Communist Party of India outside the country at Tashkent in 19205QUESTION ndash Who formed the Congress Socialist Party within the Congress5 ANSWER ndash Jaya Prakash Narayan Achyut Patwardhan Acharya Narendra Dev Ram Mohan Lohia Aruna Asaf Ali6QUESTION ndash When was the Congress Socialist Party formed What was its object6 ANSWER ndash 1934The Congress Socialist Party sought to work out socialist programme through the Congress They joined hands with the Congress and wanted to carry

Subhas Chandra Bose being expelled from the congress after the Tripuri rift he formed Forward BlockThere were basic differences between the Congress Socialists and the communistsTRADE UNION ACTIVITIES Maximum working class people lived in Bombay and Calcutta The working and living conditions of those workers were very miserable In this situation Shasipada Banerjee NM Lokhande protested against the oppression of the working class peopleThe first Trade Union Madras Labour Union was formed in 1918 by BP WadiaIndustrial strikes took place in Kanpur Calcutta Madras Jamshedpur and Ahmedabad AITUC was formed in Bombay in 1927 The growth of Trade union among the workers was slow because of the fear of the dismissal of the jobIn the mean time the Moderates as well as Communists left AITUC and formed separate organization

on National struggle with the help of workers and peasant class of the society7 QUESTION ndash What was the name of the party founded by Subhas Chandra Bose7 ANSWER- Forward Block8QUESTION ndash Who was Shasipada Banerjee8 ANSWER ndash Shasipada Banerjee was a radical Brahmo He founded a working menrsquos club to protest against exploitation of the British rulers towards the working class of India9 QUESTION ndash What was the weekly published by NM Lokhande9ANSWER- Dinabandhu10 QUESTION ndash Who founded Bombay Mill-Hands Association and in which year10 ANSWER- NM Lokhande in189011 QUESTION- Who was BP WadiaANSWER- BPWadia was the founder of Madras Labour Union in191812 QUESTION- What was the name of the first labour union of India12 ANSWER- Madras Labour Union13 QUESTION Who founded the Majur Mahajan 13 ANSWER GANDHIJI14 QUESTION What was the full form of AITUC When it was formed14 ANSWER All India Trade Union Congressin 192715QUESTION Who formed the Red Trade Union Congress and in which year15ANSWER The Communists formed the Red Trade Union Congress16 QUESTION What do you mean by Socialism16 ANSWER Socialism describes any political and economic theory that says the community rather than individuals should own and manage property and natural resources

Subject Eng Literature (The Tempest ndash William Shakespeare) Topic Act III Scene 3 Lines 1 to 52 (Line 52 ndash Brother my lord the Duke Stand to and do as we) Date 13th April 2020 (4th Period)

[Students should read the original play and also the paraphrase given in the school prescribed textbook]Summary Questions amp Answers

o Alonso Sebastian Antonio Gonzalo Adrian Francisco and others wandered about the island in search of Ferdinand and gets tired and hungry of the toil and at the same time gives up all hope of finding him

o Antonio and Sebastian are happy that Alonso is out of hope and decide to make another attempt on his life that night when being so tired they will be sleeping soundly

o Suddenly a solemn and strange music is heard in the air and several strange shapes enter bringing in a banquet These strange shapes then dance round it with gestures of salutation and then inviting the King to eat they depart

o Seeing this strange scene all are inclined to believe the tales told by travelers that there truly are ldquounicornsrdquo and ldquothe phoenixrsquo thronerdquo

1 ALONSO What harmony is this My good friends hark (L18-27)

GONZALO Marvellous sweet music

[Enter several strange shapes bringing in a banquet

they dance about it with gentle actions of salutation

and inviting the King and his companions to eat they depart]ALONSO Give us kind keepers heavens What were theseSEBASTIAN A living drollery Now I will believe

That there are unicorns that in Arabia

There is one tree the phoenixrsquo throne one phoenix

At this hour reigning thereANTONIO Ill believe both

And what does else want credit come to me

And Ill be sworn rsquotis true Travellers neer did lie

Though fools at home condemn rsquoem

(a) How did Prospero present an amazing spectacle before Alonso and his companions

Using his magic powers Prospero ordered strange shapes to lay a banquet before Alonso and his companions The shapes brought several dishes with tasty eatables in them They placed the dishes on a table before Alonso and his companions Then the strange shapes began to dance gracefully around the banquet While dancing they made gestures inviting them to eat the food Then suddenly the shapes disappeared(b) Who were the guests at the strange banquet Describe the lsquoliving drolleryrsquo

Alonso Sebastian Antonio Gonzalo Adrian and Francisco were the guests at the strange banquet

The term ldquoliving drolleryrdquo refers to live entertainment show In this context when Alonso the King of Naples Sebastian his brother Antonio the treacherous brother of Prospero Gonzalo the kind and loyal councillor to the King Adrian and Francisco came to the island they were hungry and weary in their spirits They heard a solemn and strange music They were shocked to see several strange shapes bringing in a banquet and these shapes danced about it with gentle action of salutation inviting the King and his companions to eat After this Sebastian described this show as lsquoliving drolleryrsquo(c) What is lsquophoenixrsquo What are lsquoUnicornsrdquo

The term lsquophoenixrsquo refers to a mythical Arabian bird which lived alone and perched on a solitary tree After one hundred years it expired in flames and rose again from its own ashes

lsquoUnicornsrsquo refers to the mythological four-footed beasts having horns in the centre of their foreheads When the horns are ground into powder the powder was believed to be

an aphrodisiac(d) How does Sebastian explain the puppet show OR Why does the speaker now believe in unicorns and phoenix

Sebastian finds several strange shapes bringing in the banquet They invite the king and his party for dinner and soon depart He tells that if such a strange sight can be a reality there is nothing incredible in the world and from the present moment he will believe anything He says that it is a strange dumb show enacted not by puppets but by living beings It is stranger than a travellerrsquos tale Seeing such a thing

before his own eyes he will no longer disbelieve the story about unicorns and phoenix(e) How do the other characters present respond to this living drollery

At the sight of the lsquoliving drolleryrsquo like Sebastian Gonzalo and Antonio too acted strangely Antonio told that he too now believes in unicorns and phoenix and anything else that seems to be incredible He too now believes in travellersrsquo tales Gonzalo told that if he would report those happenings in Naples nobody will believe him He considers that those gentle shapes were gentler in manner in comparison to the living beings Alonso was at first sight suspicious and told them that those strange shapes conveyed their meaning in expressive gestures when they seemed to lack speech by their movements and sounds Francisco was amazed at their mysterious disappearance

2 ALONSO Not I

(Line 43-52)GONZALO Faith sir you need not fear When we

were boysWho would believe that there were mountaineers

Dewlapped like bulls whose throats had hanging at rsquoem

Wallets of flesh Or that there were such men

Whose heads stood in their breasts Which now we find

Each putter-out of five for one will bring us

Good warrant ofALONSO I will stand to and feed

Although my lastmdashno matter since I feel

The best is past Brother my lord the Duke

Stand to and do as we

(a) How does Alonso respond at the spectacle of the shapes which were sent to them at the instruction of Prospero

After seeing the strange sight of appearing and disappearing of the shapes sent by Prospero to arrange a banquet for them Alonso says that his surprise at having seen those creatures is infinite and he is fully justified in feeling so much surprise He thinks that their shapes their gestures and the sounds they made were indeed amazing Although they do not possess the gift of speech yet they were able to convey their

thoughts by means of their gestures only

(b) What does Prospero say about the views expressed by Alonso regarding the shapes What does Francisco think about the shapesAfter hearing Alonsorsquos views about the shapes Prospero says that this manrsquos praise of the spirits is rather hasty He means to say that Alonso has shown great haste in reaching the conclusion about the shapes Francisco is amazed to see that those shapes disappeared in a mysterious way(c) What does Sebastian ask Alonso to doSebastian tells Alonso that the shapes having disappeared should not matter to them because they have left the eatables behind He asks Alonso to enjoy eating as they are extremely hungry but the king does not accept his offer of enjoying the dishes(d) How does Gonzalo try to dispel Alonsorsquos fear of those strange shapes What kind of references does he give to AlonsoGonzalo says that those who have travelled abroad have reported seeing even stranger sights than these shapes that Alonso and his companions have beheld Hence there is no reason to feel afraid of these shapes Gonzalo further adds that in his younger days he had heard strange stories from travelers and Alonso might have heard similar stories For instance it was said that there existed a certain race of

human beings who had huge lumps of flesh hanging at their throats and who therefore resembled bulls Then Gonzalo tells about a race of human beings whose heads were located at their breasts Gonzalo says that such stories were not believed by most people in those days but now-a-days these stories have become common(e) Explain the following lsquoEach putter-out of five for onersquoEnglish travellers often insured their trips with London brokers Those that went on foreign travels those days used to deposit a certain amount with some firm or company in London before their departure If the travelers failed to return the money was forfeited by the company with which it had been deposited But this money was repaid five-fold if the travelers returned safe and sound In this way a traveler stood a great chance of recovering the entire cost of his

travels(f) Give the explanatory meanings of the following expressions in the context of the above extract (i) Dewlapped (ii) Wallets of flesh

(iii) Putter-out(i) Dewlapped having big lumps of flesh at the necks(ii) Wallets of flesh large masses of flesh looking like bags(iii) Putter-out to invest money before commencing the travel

  • General methods of preparation of hydrogen
  • Chapter Dimensional Analysis (Summary)
    • Properties of Charge
Page 50:   · Web viewSubject. Topic. Summary. Execution. Hindi. व्याकरण. शरीरके अंगो के नाम लिखिए. 1) आँख 2) नाक 3

bullAction oriented bullPervasive function bullContinuous function

2) Explain the features of HRM in brief

Answer)bullHuman Resource Management is concerned with managing people at work bull Human Resource Management is concerned with employees which bring people and organisations together so that the goals of each are met bullHuman resource management considered every employees as an individual and also promote their satisfaction and growth bull Human resource management is inherent in all organisations and at all levelsbullManagement of human resources are ongoing on never ending process which requires a constant alertness and Awareness of human relations

3) ldquoHR function is said to be pervasiverdquowhy

Answer) Human resource management is required in all organisations whether it is private or government organisations armed forces sports organisations etc It permeatsall the functional areas like production marketing finance research etc This from this feature of human resource management it can be said that it is pervasive in nature

Economics Demand Q1DEFINITION OF DEMANDIn economics demand is the quantity of a good that consumers are willing and able to purchase at various prices during a given period of timeQ2DEMAND CURVEIn economics a demand curve is a graph depicting the relationship between the price of a certain commodity and the quantity of that commodity that is demanded at that pricQ3LAW OF DEMANDIn microeconomics the law of demand states that conditional on all else being equal as the price of a good increases quantity demanded decreases conversely as the price of a good decreases quantity demanded increasesQ4ASSUMPTION of LAW OF DEMAND(i)No change in price of related commodities(ii) No change in income of the consumer(iii) No change in taste and preferences customs habit and fashion of the consumer( No expectation regarding future change in priceQ5MARKET DEMAND SCHEDULEIn economics a market demand schedule is a tabulation of the quantity of a good that all consumers in a market will purchase at a

given price At any given price the corresponding value on the demand schedule is the sum of all consumersrsquo quantities demanded at that priceQ6INDIVIDUAL DEMAND SCHEDULEIndividual demand schedule refers to a tabular statement showing various quantities of a commodity that a consumer is willing to buy at various levels of price during a given period of timeQ7 FACTORS AFFECTING INDIVIDUAL DEMAND FOR A COMMODITY

The factors that influence a consumerrsquos decision to purchase a commodity are also known as determinants of demand The following factors affect the individual demand for a commodity1 price of the commodity2 price of related goods3 income of buyer of the commodity4 tastes and preferences of the buyer1 Price of the CommodityYou must have observed that when price of a commodity falls you tend to buy more of it and when its price rises you tend to buy less of it when all other factors remain constant (lsquoother things remaining the samersquo) In other words other things remaining the same there is an inverse relationship between the price of a commodity and its quantity demanded by its buyers This statement is in accordance with law of demand which you will study in the later part of this lesson Price of a commodity and its quantity demanded by its buyers are inversely related only when lsquoother things remain the samersquo So lsquoother things remaining the samersquo is an assumption when we study the effect of changes in the price of a commodity on its quantity demanded2 Price of Related goodsA consumer may demand a particular good But while buying that good heshe also asks the price of its related goods Related goods can be of two types-(i) Substitute goods(ii) Complementary goods While purchasing a good prices of its substitutes and complements do affect its quantity purchased(i) Price of Substitute Goods Substitute goods are those goods which can easily be used in place of one another for satisfaction of a particular want like tea and coffee An increase in price of substitute good leads to an increase in demand for the given commodity and a decrease in price of substitute good leads to a decrease in demand for the given commodity It means demand for a given commodity is directly affected by change in price of substitute goods For example if price of coffee increases the demand for tea will rise as tea will become relatively cheaper in comparison to coffee(ii) Price of Complementary goods Complementary goods are those goods which are used together to satisfy a particular want like car and petrol An increase in the price of complementary goods leads to a decrease in demand for the given commodity and a decrease in the price of complementary goods leads to an increase in demand for the given commodity For example if price of petrol falls then the demand for cars will increase as it will be relatively cheaper to use both the goods together So demand for a given commodity is inversely affected by change in price of complementary goods3 Income of the Buyer of CommodityDemand for a commodity is also affected by income of its buyer However the effect of change in income on demand depends on the nature of the commodity under consideration In case of some goods like full cream milk fine quality of rice (Basmati rice) etc demand for these commodities increases when income of the buyer increases and

demand for these commodities decreases when income of the buyer decreases Such goods whose demand increases with the increase in income of the buyer are called normal goods But there are some goods like coarse rice toned milk etc whose demand decreases when income of buyer increases and their demand increases when income of the buyer decreases Such goods whose demand decreases with the increase in income of the buyer are called inferior goods Suppose a consumer buys 10 Kgs of rice whose price is ` 25 per Kg He cannot afford to buy better quality of rice because the price of such rice is ` 50 per Kg The consumer is spending ` 250 per month on the purchase of rice Now if income of the consumer increases and he can afford ` 350 on purchase of 10 Kg of rice Now he can afford to buy some quantity of rice say 6 Kgs whose price is ` 25 per Kg and may buy 4 Kgs of rice whose price is ` 50 per Kg Thus he will buy 10 Kgs of rice by spending ` 350 per month Therefore we may conclude that demand for normal goods is directly related to the income of the buyer but demand for inferior goods is inversely related to the income of the buyer4 Tastes and Preferences of the BuyerThe demand for a commodity is also affected by the tastes and preferences of the buyers They include change in fashion customs habits etc Those commodities are preferred by the consumers which are in fashion So demand for those commodities rises which are in fashion On the other hand if a commodity goes out of the fashion its demand falls because no consumer will like to buy it(5) Number of Buyers in the Market(Population)Increase in population raises the market demand whereas decrease in population reduces the market demand for a commodity Not only the size of population but its composition like age (ratio of males females children and old people in population) also affects the demand for a commodity It is because of needs of children young old male and female population differs(6) Distribution of Income and WealthIf the distribution of income and wealth is more in favour of the rich demand for the commodities preferred by the rich such as comforts and luxuries is likely to be higher On the other hand if the distribution of income and wealth is more in favour of poor demand for commodities preferred by the poor such as necessities will be more(7) Season and Weather ConditionsThis is generally observed that the demand for woolens increases during winter whereas demand for ice creams and cold drinks increases during summer Similarly market demand for umbrellas rain coats increases during rainy seasonQ8 REASONS FOR OPERATION OF LAW OF DEMAND WHY DEMAND CURVE SLOPES DOWNWARDNow we will try to explain why does a consumer purchase more quantity of a commodity at a lower price and less of it at a higher price or why does the law of demand operate ie why does the demand curve slope downwards from left to right The main reasons for operation of law of demand are1 Law of Diminishing Marginal UtilityAs you have studied earlier law of diminishing marginal utility states that as we consume more and more units of a commodity the utility derived from each successive unit goes on decreasing The consumer will be ready to pay more for those units which provide him more utility and less for those which provide him less utility It implies that he will purchase more only when the price of the commodity falls2 Income Effect

When price of a commodity falls purchasing power or real income of the consumer increases which enables him to purchase more quantity of the commodity with the same money income Let us take an example Suppose you buy 4 ice creams when price of each ice cream is ` 25 If price of ice creams falls to ` 20 then with same money income you can buy 5 ice creams now3 Substitution EffectWhen price of a commodity falls it becomes comparatively cheaper as compared to its substitutes (although price of substitutes has not been changed) This will lead to rise in demand for the given commodity For example if coke and Pepsi both are sold at ` 10 each and price of coke falls Now coke has become relatively cheaper and will be substituted for Pepsi It will lead to rise in demand for coke4 Change in Number of BuyersWhen price of a commodity falls some old buyers may demand more of the commodity at the reduced price and some new buyers may also start buying this commodity who were not in a position to buy it earlier due to higher price This will lead to increase in number of buyers when price of the commodity falls As a result demand for the commodity rises when its price falls5 Diverse Uses of a CommoditySome commodities have diverse uses like milk It can be used for drinking for sweet preparation for ice cream preparation etc If price of milk rises its use may be restricted to important purpose only This will lead to reduction in demand for other less important uses When price of milk falls it can be put to other uses also leading to rise n demand for itQ9 EXCEPTIONS TO THE LAW OF DEMANDYou have studied in law of demand that a buyer is willing to buy more quantity of a commodity at a lower price and less of it at a higher price But in certain circumstances a rise in price may lead to rise in demand These circumstances are called Exceptions to the Law of Demand Some important exceptions are1 Giffen GoodsGiffen goods are special type of inferior goods in which negative income effect is stronger than negative substitution effect Giffen goods do not follow law of demand as their demand rises when their price rises Examples of Giffen goods are jowar and bajra etc2 Status Symbol GoodsSome goods are used by rich people as status symbols eg diamonds gold jewellary etc The higher the price the higher will be the demand for these goods When price of such goods falls these goods are no longer looked at as status symbol goods and tehrefore therir demand falls3 NecessitiesCommodities such as medicines salt wheat etc do not follow law of demandbecause we have to purchase them in minimum required quantity whatever their price may be4 Goods Expected to be ScarceWhen the buyers expect a scarcity of a particular good in near future they start buying more and more of that good even if their prices are rising For example during war famines etc people tend to buy more of some goods even at higher prices due to fear of their scarcity in near future

Political Science

Constitution of India-The

Preamble

The preamble-

Preamble-

The preamble is the most precious part of the constitution We the people of India having solemnly resolved to constitute India into a Sovereign Socialist Secular Democratic Republic and to secure to all its citizensA preamble is an introductory and expressionary statement in a document that explains the documents purpose and underlying philosophy When applied to the opening paragraphs of a statute it may recite historical facts pertinent to the subject of the statuteNature and purpose of the constitution-Purpose of the Constitution dictates permanent framework of the government to form a more perfect union to establish justice and ensure peace of thenationconstitution provide principles how the government can run itself following the rules and laws written in the constitution of each state keeps them balanced

Answer the following questions-

1 What is preambleA preamble is an introductory and expressionary statement in a document that explains the documents purpose and underlying philosophy2 What is the nature and

purpose of the constitutionConstitution dictatespermanent framework of the government to form a more perfect union to establish justice and ensure peace of the nation

Homework-Learn

Accounts Compatibilty mode

1MEANING OF PARTNERSHIPPartnership is a form of business organisation where two or more persons join hands to run a business They share the profits and losses according to the agreement amongst them According to the Indian Partnership Act 1932 ldquoPartnership is relation between persons who have agreed to share profits of a business carried on by all or any one of them acting for allrdquo For example one of your friends has passed class XII from National Institute of Open Schooling (NIOS) and wants to start a business Heshe approaches you to join in this venture Heshe wants you to contribute some money and participate in the business activities Both of you if join hands constitute a partnership2CHARACTERISTICS1048698 Agreement A partnership is formed by an agreement The agreement may be either oral or in writing It defines the relationship between the persons who agree to carry on business It may contain the terms of sharing profit and the capital to be invested by each partner etc The written agreement is known as partnership deed1048698 Number of persons There must be at least two persons to form a partnership

The maximum number of partners in a partnership firm can be 50 according toCompanies Act 20131048698 Business The Partnership is formed to carry on business with a purpose of earning profits The business should be lawful Thus if two or more persons agree to carry on unlawful activities it will not be termed as partnership1048698 Sharing Profits The partners agree to share profits in the agreed ratio In caseof loss all the partners have to bear it in the same agreed profit sharing ratio10486981048698Mutual Agency Every partner is an agent of the other partners Every partner can bind the firm and all other partners by hisher acts Each partner will be responsible and liable for the acts of all other partners10486981048698Unlimited liability The liability of each partner except that of a minor is unlimited Their liability extends to their personal assets also If the assets of the firm are insufficient to pay off its debts the partnersrsquo personal property can be used to satisfy the claim of the creditors of the partnership firm10486981048698Management All the partners have a right to mange the business However they may authorize one or more partners to manage the affairs of the business on their behalf10486981048698Transferability of Share No partner can transfer hisher share to any one including hisher family member without the consent of all other partners3PARTNERSHIP DEEDAgreement forms the basis of partnership The written form of the agreement is which a document of partnership is It contains terms and conditions regarding the conduct of the business It also explains relationship between the partners This document is called partnership deed Every firm can frame its own partnership deed in which the rights duties and liabilities of the partners are stated in detail It helps in settling the disputes arising among the partners during the general conduct of business 4CONTENTS OF PARTNERSHIP DEEDThe partnership deed generally contains the following (i) Name and address of the partnership firm(ii) Nature and objectives of the business(iii) Name and address of each partner(iv) Ratio in which profits is to be shared(v) Capital contribution by each partner(vi) Rate of Interest on capital if allowed(vii) Salary or any other remuneration to partners if allowed(viii) Rate of interest on loans and advances by a partner to the firm(ix) Drawings of partners and interest thereon if any(x) Method of valuation of goodwill and revaluation of assets and liabilities on the reconstitution of the partnership ie on the admission retirement or death of a partner(xi) Settlement of disputes by arbitration(xii) Settlement of accounts at the time of retirement or death of a partner5IN ABSENCE OF PARTNERSHIP DEEDThe partnership deed lays down the terms and conditions of partnership in regard to rights duties and obligations of the partners In the absence of partnership deed there may arise a controversy on certain issues like profit sharing ratio interest on

capital interest on drawings interest on loan and salary of the partners In such cases the provisions of the Indian Partnership Act becomes applicableSome of the Issues are(i) Distribution of Profit Partners are entitled to share profits equally(ii) Interest on Capital Interest on capital is not allowed(iii) Interest on Drawings No interest on drawing of the partners is to be charged(iv) Interest on Partnerrsquos Loan A Partner is allowed interest 6 per annum on the amount of loan given to the firm by himher(v) Salary and Commission to Partner A partner is not entitled to anysalary or commission or any other remuneration for managing the business

History TOPIC-TOWARDS INDEPENDENCE AND PARTITION THE LAST PHASE (1935-1947)

SUB TOPIC-IMPORTANT POLITICAL DEVELOPMENTS ndash GROWTH OF SOCIAL IDEAS

Socialism is a political social and economic philosophyLike in other parts of the world the Russian revolution of 1917 served as a great inspiration for revolutionaries in India who at that time were engaged in the struggle for liberation from British ruleSocialist ideas led to the formation of communist party of IndiaJAWAHARLAL NEHRU Among the early Congress leaders Jawaharlal Nehru was very much impressed and influenced by the Socialist ideas He also learnt about the Economic activities of the Soviet Union after the Bolshevic Revolution 1917 He made full use of them in IndiaThe election of Jawaharlal Nehru and Subhas Chandra Bose showed the Left wing tendency within CongressJawaharlal Nehru demanded economic freedom along with political freedom of the people in order to end the exploitation of masses

Nehrus working committee included three socialists leaders The Lucknow session was a landmark in the evolution of socialist ideas of the congressSUBHAS CHANDRA BOSE ndash Subhas Chandra Bose had socialist leaning Both Jawaharlal Nehru and Subhas Chandra Bose were known as leftist Congress men Later on National Congress divided into Leftist and rightist campCONGRESS SOCIALIST Within the Congress some leaders formed the Congress Socialist partyPattavi Sitaramyya Sardar Patel Rajendra Prasad had hostile attitude towards the Congress Socialist partyJawaharlals attitude was hesitant

1 QUESTION ndash Mention name of two Congress leaders who had socialist leaning

1ANSWER ndash Subhas Chandra Bose and Jawaharlal Nehru2QUESTION- In which session of the congress Jawaharlal elaborated his Socialist ideas2 ANSWER ndash Lucknow and Faizpur Session in December 1935 and 19363QUESTION ndash Why Congress was sharply divided into leftist and rightist camp 3ANSWER ndash Subhas Chandra Bosersquos attempt to seek re election for congress presidentship in 1939sharply divided the National Congress into Leftist and Rightist camp4 QUESTION ndash Who was MN Roy 4 ANSWER ndash Manabendra Roy first formed the Communist Party of India outside the country at Tashkent in 19205QUESTION ndash Who formed the Congress Socialist Party within the Congress5 ANSWER ndash Jaya Prakash Narayan Achyut Patwardhan Acharya Narendra Dev Ram Mohan Lohia Aruna Asaf Ali6QUESTION ndash When was the Congress Socialist Party formed What was its object6 ANSWER ndash 1934The Congress Socialist Party sought to work out socialist programme through the Congress They joined hands with the Congress and wanted to carry

Subhas Chandra Bose being expelled from the congress after the Tripuri rift he formed Forward BlockThere were basic differences between the Congress Socialists and the communistsTRADE UNION ACTIVITIES Maximum working class people lived in Bombay and Calcutta The working and living conditions of those workers were very miserable In this situation Shasipada Banerjee NM Lokhande protested against the oppression of the working class peopleThe first Trade Union Madras Labour Union was formed in 1918 by BP WadiaIndustrial strikes took place in Kanpur Calcutta Madras Jamshedpur and Ahmedabad AITUC was formed in Bombay in 1927 The growth of Trade union among the workers was slow because of the fear of the dismissal of the jobIn the mean time the Moderates as well as Communists left AITUC and formed separate organization

on National struggle with the help of workers and peasant class of the society7 QUESTION ndash What was the name of the party founded by Subhas Chandra Bose7 ANSWER- Forward Block8QUESTION ndash Who was Shasipada Banerjee8 ANSWER ndash Shasipada Banerjee was a radical Brahmo He founded a working menrsquos club to protest against exploitation of the British rulers towards the working class of India9 QUESTION ndash What was the weekly published by NM Lokhande9ANSWER- Dinabandhu10 QUESTION ndash Who founded Bombay Mill-Hands Association and in which year10 ANSWER- NM Lokhande in189011 QUESTION- Who was BP WadiaANSWER- BPWadia was the founder of Madras Labour Union in191812 QUESTION- What was the name of the first labour union of India12 ANSWER- Madras Labour Union13 QUESTION Who founded the Majur Mahajan 13 ANSWER GANDHIJI14 QUESTION What was the full form of AITUC When it was formed14 ANSWER All India Trade Union Congressin 192715QUESTION Who formed the Red Trade Union Congress and in which year15ANSWER The Communists formed the Red Trade Union Congress16 QUESTION What do you mean by Socialism16 ANSWER Socialism describes any political and economic theory that says the community rather than individuals should own and manage property and natural resources

Subject Eng Literature (The Tempest ndash William Shakespeare) Topic Act III Scene 3 Lines 1 to 52 (Line 52 ndash Brother my lord the Duke Stand to and do as we) Date 13th April 2020 (4th Period)

[Students should read the original play and also the paraphrase given in the school prescribed textbook]Summary Questions amp Answers

o Alonso Sebastian Antonio Gonzalo Adrian Francisco and others wandered about the island in search of Ferdinand and gets tired and hungry of the toil and at the same time gives up all hope of finding him

o Antonio and Sebastian are happy that Alonso is out of hope and decide to make another attempt on his life that night when being so tired they will be sleeping soundly

o Suddenly a solemn and strange music is heard in the air and several strange shapes enter bringing in a banquet These strange shapes then dance round it with gestures of salutation and then inviting the King to eat they depart

o Seeing this strange scene all are inclined to believe the tales told by travelers that there truly are ldquounicornsrdquo and ldquothe phoenixrsquo thronerdquo

1 ALONSO What harmony is this My good friends hark (L18-27)

GONZALO Marvellous sweet music

[Enter several strange shapes bringing in a banquet

they dance about it with gentle actions of salutation

and inviting the King and his companions to eat they depart]ALONSO Give us kind keepers heavens What were theseSEBASTIAN A living drollery Now I will believe

That there are unicorns that in Arabia

There is one tree the phoenixrsquo throne one phoenix

At this hour reigning thereANTONIO Ill believe both

And what does else want credit come to me

And Ill be sworn rsquotis true Travellers neer did lie

Though fools at home condemn rsquoem

(a) How did Prospero present an amazing spectacle before Alonso and his companions

Using his magic powers Prospero ordered strange shapes to lay a banquet before Alonso and his companions The shapes brought several dishes with tasty eatables in them They placed the dishes on a table before Alonso and his companions Then the strange shapes began to dance gracefully around the banquet While dancing they made gestures inviting them to eat the food Then suddenly the shapes disappeared(b) Who were the guests at the strange banquet Describe the lsquoliving drolleryrsquo

Alonso Sebastian Antonio Gonzalo Adrian and Francisco were the guests at the strange banquet

The term ldquoliving drolleryrdquo refers to live entertainment show In this context when Alonso the King of Naples Sebastian his brother Antonio the treacherous brother of Prospero Gonzalo the kind and loyal councillor to the King Adrian and Francisco came to the island they were hungry and weary in their spirits They heard a solemn and strange music They were shocked to see several strange shapes bringing in a banquet and these shapes danced about it with gentle action of salutation inviting the King and his companions to eat After this Sebastian described this show as lsquoliving drolleryrsquo(c) What is lsquophoenixrsquo What are lsquoUnicornsrdquo

The term lsquophoenixrsquo refers to a mythical Arabian bird which lived alone and perched on a solitary tree After one hundred years it expired in flames and rose again from its own ashes

lsquoUnicornsrsquo refers to the mythological four-footed beasts having horns in the centre of their foreheads When the horns are ground into powder the powder was believed to be

an aphrodisiac(d) How does Sebastian explain the puppet show OR Why does the speaker now believe in unicorns and phoenix

Sebastian finds several strange shapes bringing in the banquet They invite the king and his party for dinner and soon depart He tells that if such a strange sight can be a reality there is nothing incredible in the world and from the present moment he will believe anything He says that it is a strange dumb show enacted not by puppets but by living beings It is stranger than a travellerrsquos tale Seeing such a thing

before his own eyes he will no longer disbelieve the story about unicorns and phoenix(e) How do the other characters present respond to this living drollery

At the sight of the lsquoliving drolleryrsquo like Sebastian Gonzalo and Antonio too acted strangely Antonio told that he too now believes in unicorns and phoenix and anything else that seems to be incredible He too now believes in travellersrsquo tales Gonzalo told that if he would report those happenings in Naples nobody will believe him He considers that those gentle shapes were gentler in manner in comparison to the living beings Alonso was at first sight suspicious and told them that those strange shapes conveyed their meaning in expressive gestures when they seemed to lack speech by their movements and sounds Francisco was amazed at their mysterious disappearance

2 ALONSO Not I

(Line 43-52)GONZALO Faith sir you need not fear When we

were boysWho would believe that there were mountaineers

Dewlapped like bulls whose throats had hanging at rsquoem

Wallets of flesh Or that there were such men

Whose heads stood in their breasts Which now we find

Each putter-out of five for one will bring us

Good warrant ofALONSO I will stand to and feed

Although my lastmdashno matter since I feel

The best is past Brother my lord the Duke

Stand to and do as we

(a) How does Alonso respond at the spectacle of the shapes which were sent to them at the instruction of Prospero

After seeing the strange sight of appearing and disappearing of the shapes sent by Prospero to arrange a banquet for them Alonso says that his surprise at having seen those creatures is infinite and he is fully justified in feeling so much surprise He thinks that their shapes their gestures and the sounds they made were indeed amazing Although they do not possess the gift of speech yet they were able to convey their

thoughts by means of their gestures only

(b) What does Prospero say about the views expressed by Alonso regarding the shapes What does Francisco think about the shapesAfter hearing Alonsorsquos views about the shapes Prospero says that this manrsquos praise of the spirits is rather hasty He means to say that Alonso has shown great haste in reaching the conclusion about the shapes Francisco is amazed to see that those shapes disappeared in a mysterious way(c) What does Sebastian ask Alonso to doSebastian tells Alonso that the shapes having disappeared should not matter to them because they have left the eatables behind He asks Alonso to enjoy eating as they are extremely hungry but the king does not accept his offer of enjoying the dishes(d) How does Gonzalo try to dispel Alonsorsquos fear of those strange shapes What kind of references does he give to AlonsoGonzalo says that those who have travelled abroad have reported seeing even stranger sights than these shapes that Alonso and his companions have beheld Hence there is no reason to feel afraid of these shapes Gonzalo further adds that in his younger days he had heard strange stories from travelers and Alonso might have heard similar stories For instance it was said that there existed a certain race of

human beings who had huge lumps of flesh hanging at their throats and who therefore resembled bulls Then Gonzalo tells about a race of human beings whose heads were located at their breasts Gonzalo says that such stories were not believed by most people in those days but now-a-days these stories have become common(e) Explain the following lsquoEach putter-out of five for onersquoEnglish travellers often insured their trips with London brokers Those that went on foreign travels those days used to deposit a certain amount with some firm or company in London before their departure If the travelers failed to return the money was forfeited by the company with which it had been deposited But this money was repaid five-fold if the travelers returned safe and sound In this way a traveler stood a great chance of recovering the entire cost of his

travels(f) Give the explanatory meanings of the following expressions in the context of the above extract (i) Dewlapped (ii) Wallets of flesh

(iii) Putter-out(i) Dewlapped having big lumps of flesh at the necks(ii) Wallets of flesh large masses of flesh looking like bags(iii) Putter-out to invest money before commencing the travel

  • General methods of preparation of hydrogen
  • Chapter Dimensional Analysis (Summary)
    • Properties of Charge
Page 51:   · Web viewSubject. Topic. Summary. Execution. Hindi. व्याकरण. शरीरके अंगो के नाम लिखिए. 1) आँख 2) नाक 3

given price At any given price the corresponding value on the demand schedule is the sum of all consumersrsquo quantities demanded at that priceQ6INDIVIDUAL DEMAND SCHEDULEIndividual demand schedule refers to a tabular statement showing various quantities of a commodity that a consumer is willing to buy at various levels of price during a given period of timeQ7 FACTORS AFFECTING INDIVIDUAL DEMAND FOR A COMMODITY

The factors that influence a consumerrsquos decision to purchase a commodity are also known as determinants of demand The following factors affect the individual demand for a commodity1 price of the commodity2 price of related goods3 income of buyer of the commodity4 tastes and preferences of the buyer1 Price of the CommodityYou must have observed that when price of a commodity falls you tend to buy more of it and when its price rises you tend to buy less of it when all other factors remain constant (lsquoother things remaining the samersquo) In other words other things remaining the same there is an inverse relationship between the price of a commodity and its quantity demanded by its buyers This statement is in accordance with law of demand which you will study in the later part of this lesson Price of a commodity and its quantity demanded by its buyers are inversely related only when lsquoother things remain the samersquo So lsquoother things remaining the samersquo is an assumption when we study the effect of changes in the price of a commodity on its quantity demanded2 Price of Related goodsA consumer may demand a particular good But while buying that good heshe also asks the price of its related goods Related goods can be of two types-(i) Substitute goods(ii) Complementary goods While purchasing a good prices of its substitutes and complements do affect its quantity purchased(i) Price of Substitute Goods Substitute goods are those goods which can easily be used in place of one another for satisfaction of a particular want like tea and coffee An increase in price of substitute good leads to an increase in demand for the given commodity and a decrease in price of substitute good leads to a decrease in demand for the given commodity It means demand for a given commodity is directly affected by change in price of substitute goods For example if price of coffee increases the demand for tea will rise as tea will become relatively cheaper in comparison to coffee(ii) Price of Complementary goods Complementary goods are those goods which are used together to satisfy a particular want like car and petrol An increase in the price of complementary goods leads to a decrease in demand for the given commodity and a decrease in the price of complementary goods leads to an increase in demand for the given commodity For example if price of petrol falls then the demand for cars will increase as it will be relatively cheaper to use both the goods together So demand for a given commodity is inversely affected by change in price of complementary goods3 Income of the Buyer of CommodityDemand for a commodity is also affected by income of its buyer However the effect of change in income on demand depends on the nature of the commodity under consideration In case of some goods like full cream milk fine quality of rice (Basmati rice) etc demand for these commodities increases when income of the buyer increases and

demand for these commodities decreases when income of the buyer decreases Such goods whose demand increases with the increase in income of the buyer are called normal goods But there are some goods like coarse rice toned milk etc whose demand decreases when income of buyer increases and their demand increases when income of the buyer decreases Such goods whose demand decreases with the increase in income of the buyer are called inferior goods Suppose a consumer buys 10 Kgs of rice whose price is ` 25 per Kg He cannot afford to buy better quality of rice because the price of such rice is ` 50 per Kg The consumer is spending ` 250 per month on the purchase of rice Now if income of the consumer increases and he can afford ` 350 on purchase of 10 Kg of rice Now he can afford to buy some quantity of rice say 6 Kgs whose price is ` 25 per Kg and may buy 4 Kgs of rice whose price is ` 50 per Kg Thus he will buy 10 Kgs of rice by spending ` 350 per month Therefore we may conclude that demand for normal goods is directly related to the income of the buyer but demand for inferior goods is inversely related to the income of the buyer4 Tastes and Preferences of the BuyerThe demand for a commodity is also affected by the tastes and preferences of the buyers They include change in fashion customs habits etc Those commodities are preferred by the consumers which are in fashion So demand for those commodities rises which are in fashion On the other hand if a commodity goes out of the fashion its demand falls because no consumer will like to buy it(5) Number of Buyers in the Market(Population)Increase in population raises the market demand whereas decrease in population reduces the market demand for a commodity Not only the size of population but its composition like age (ratio of males females children and old people in population) also affects the demand for a commodity It is because of needs of children young old male and female population differs(6) Distribution of Income and WealthIf the distribution of income and wealth is more in favour of the rich demand for the commodities preferred by the rich such as comforts and luxuries is likely to be higher On the other hand if the distribution of income and wealth is more in favour of poor demand for commodities preferred by the poor such as necessities will be more(7) Season and Weather ConditionsThis is generally observed that the demand for woolens increases during winter whereas demand for ice creams and cold drinks increases during summer Similarly market demand for umbrellas rain coats increases during rainy seasonQ8 REASONS FOR OPERATION OF LAW OF DEMAND WHY DEMAND CURVE SLOPES DOWNWARDNow we will try to explain why does a consumer purchase more quantity of a commodity at a lower price and less of it at a higher price or why does the law of demand operate ie why does the demand curve slope downwards from left to right The main reasons for operation of law of demand are1 Law of Diminishing Marginal UtilityAs you have studied earlier law of diminishing marginal utility states that as we consume more and more units of a commodity the utility derived from each successive unit goes on decreasing The consumer will be ready to pay more for those units which provide him more utility and less for those which provide him less utility It implies that he will purchase more only when the price of the commodity falls2 Income Effect

When price of a commodity falls purchasing power or real income of the consumer increases which enables him to purchase more quantity of the commodity with the same money income Let us take an example Suppose you buy 4 ice creams when price of each ice cream is ` 25 If price of ice creams falls to ` 20 then with same money income you can buy 5 ice creams now3 Substitution EffectWhen price of a commodity falls it becomes comparatively cheaper as compared to its substitutes (although price of substitutes has not been changed) This will lead to rise in demand for the given commodity For example if coke and Pepsi both are sold at ` 10 each and price of coke falls Now coke has become relatively cheaper and will be substituted for Pepsi It will lead to rise in demand for coke4 Change in Number of BuyersWhen price of a commodity falls some old buyers may demand more of the commodity at the reduced price and some new buyers may also start buying this commodity who were not in a position to buy it earlier due to higher price This will lead to increase in number of buyers when price of the commodity falls As a result demand for the commodity rises when its price falls5 Diverse Uses of a CommoditySome commodities have diverse uses like milk It can be used for drinking for sweet preparation for ice cream preparation etc If price of milk rises its use may be restricted to important purpose only This will lead to reduction in demand for other less important uses When price of milk falls it can be put to other uses also leading to rise n demand for itQ9 EXCEPTIONS TO THE LAW OF DEMANDYou have studied in law of demand that a buyer is willing to buy more quantity of a commodity at a lower price and less of it at a higher price But in certain circumstances a rise in price may lead to rise in demand These circumstances are called Exceptions to the Law of Demand Some important exceptions are1 Giffen GoodsGiffen goods are special type of inferior goods in which negative income effect is stronger than negative substitution effect Giffen goods do not follow law of demand as their demand rises when their price rises Examples of Giffen goods are jowar and bajra etc2 Status Symbol GoodsSome goods are used by rich people as status symbols eg diamonds gold jewellary etc The higher the price the higher will be the demand for these goods When price of such goods falls these goods are no longer looked at as status symbol goods and tehrefore therir demand falls3 NecessitiesCommodities such as medicines salt wheat etc do not follow law of demandbecause we have to purchase them in minimum required quantity whatever their price may be4 Goods Expected to be ScarceWhen the buyers expect a scarcity of a particular good in near future they start buying more and more of that good even if their prices are rising For example during war famines etc people tend to buy more of some goods even at higher prices due to fear of their scarcity in near future

Political Science

Constitution of India-The

Preamble

The preamble-

Preamble-

The preamble is the most precious part of the constitution We the people of India having solemnly resolved to constitute India into a Sovereign Socialist Secular Democratic Republic and to secure to all its citizensA preamble is an introductory and expressionary statement in a document that explains the documents purpose and underlying philosophy When applied to the opening paragraphs of a statute it may recite historical facts pertinent to the subject of the statuteNature and purpose of the constitution-Purpose of the Constitution dictates permanent framework of the government to form a more perfect union to establish justice and ensure peace of thenationconstitution provide principles how the government can run itself following the rules and laws written in the constitution of each state keeps them balanced

Answer the following questions-

1 What is preambleA preamble is an introductory and expressionary statement in a document that explains the documents purpose and underlying philosophy2 What is the nature and

purpose of the constitutionConstitution dictatespermanent framework of the government to form a more perfect union to establish justice and ensure peace of the nation

Homework-Learn

Accounts Compatibilty mode

1MEANING OF PARTNERSHIPPartnership is a form of business organisation where two or more persons join hands to run a business They share the profits and losses according to the agreement amongst them According to the Indian Partnership Act 1932 ldquoPartnership is relation between persons who have agreed to share profits of a business carried on by all or any one of them acting for allrdquo For example one of your friends has passed class XII from National Institute of Open Schooling (NIOS) and wants to start a business Heshe approaches you to join in this venture Heshe wants you to contribute some money and participate in the business activities Both of you if join hands constitute a partnership2CHARACTERISTICS1048698 Agreement A partnership is formed by an agreement The agreement may be either oral or in writing It defines the relationship between the persons who agree to carry on business It may contain the terms of sharing profit and the capital to be invested by each partner etc The written agreement is known as partnership deed1048698 Number of persons There must be at least two persons to form a partnership

The maximum number of partners in a partnership firm can be 50 according toCompanies Act 20131048698 Business The Partnership is formed to carry on business with a purpose of earning profits The business should be lawful Thus if two or more persons agree to carry on unlawful activities it will not be termed as partnership1048698 Sharing Profits The partners agree to share profits in the agreed ratio In caseof loss all the partners have to bear it in the same agreed profit sharing ratio10486981048698Mutual Agency Every partner is an agent of the other partners Every partner can bind the firm and all other partners by hisher acts Each partner will be responsible and liable for the acts of all other partners10486981048698Unlimited liability The liability of each partner except that of a minor is unlimited Their liability extends to their personal assets also If the assets of the firm are insufficient to pay off its debts the partnersrsquo personal property can be used to satisfy the claim of the creditors of the partnership firm10486981048698Management All the partners have a right to mange the business However they may authorize one or more partners to manage the affairs of the business on their behalf10486981048698Transferability of Share No partner can transfer hisher share to any one including hisher family member without the consent of all other partners3PARTNERSHIP DEEDAgreement forms the basis of partnership The written form of the agreement is which a document of partnership is It contains terms and conditions regarding the conduct of the business It also explains relationship between the partners This document is called partnership deed Every firm can frame its own partnership deed in which the rights duties and liabilities of the partners are stated in detail It helps in settling the disputes arising among the partners during the general conduct of business 4CONTENTS OF PARTNERSHIP DEEDThe partnership deed generally contains the following (i) Name and address of the partnership firm(ii) Nature and objectives of the business(iii) Name and address of each partner(iv) Ratio in which profits is to be shared(v) Capital contribution by each partner(vi) Rate of Interest on capital if allowed(vii) Salary or any other remuneration to partners if allowed(viii) Rate of interest on loans and advances by a partner to the firm(ix) Drawings of partners and interest thereon if any(x) Method of valuation of goodwill and revaluation of assets and liabilities on the reconstitution of the partnership ie on the admission retirement or death of a partner(xi) Settlement of disputes by arbitration(xii) Settlement of accounts at the time of retirement or death of a partner5IN ABSENCE OF PARTNERSHIP DEEDThe partnership deed lays down the terms and conditions of partnership in regard to rights duties and obligations of the partners In the absence of partnership deed there may arise a controversy on certain issues like profit sharing ratio interest on

capital interest on drawings interest on loan and salary of the partners In such cases the provisions of the Indian Partnership Act becomes applicableSome of the Issues are(i) Distribution of Profit Partners are entitled to share profits equally(ii) Interest on Capital Interest on capital is not allowed(iii) Interest on Drawings No interest on drawing of the partners is to be charged(iv) Interest on Partnerrsquos Loan A Partner is allowed interest 6 per annum on the amount of loan given to the firm by himher(v) Salary and Commission to Partner A partner is not entitled to anysalary or commission or any other remuneration for managing the business

History TOPIC-TOWARDS INDEPENDENCE AND PARTITION THE LAST PHASE (1935-1947)

SUB TOPIC-IMPORTANT POLITICAL DEVELOPMENTS ndash GROWTH OF SOCIAL IDEAS

Socialism is a political social and economic philosophyLike in other parts of the world the Russian revolution of 1917 served as a great inspiration for revolutionaries in India who at that time were engaged in the struggle for liberation from British ruleSocialist ideas led to the formation of communist party of IndiaJAWAHARLAL NEHRU Among the early Congress leaders Jawaharlal Nehru was very much impressed and influenced by the Socialist ideas He also learnt about the Economic activities of the Soviet Union after the Bolshevic Revolution 1917 He made full use of them in IndiaThe election of Jawaharlal Nehru and Subhas Chandra Bose showed the Left wing tendency within CongressJawaharlal Nehru demanded economic freedom along with political freedom of the people in order to end the exploitation of masses

Nehrus working committee included three socialists leaders The Lucknow session was a landmark in the evolution of socialist ideas of the congressSUBHAS CHANDRA BOSE ndash Subhas Chandra Bose had socialist leaning Both Jawaharlal Nehru and Subhas Chandra Bose were known as leftist Congress men Later on National Congress divided into Leftist and rightist campCONGRESS SOCIALIST Within the Congress some leaders formed the Congress Socialist partyPattavi Sitaramyya Sardar Patel Rajendra Prasad had hostile attitude towards the Congress Socialist partyJawaharlals attitude was hesitant

1 QUESTION ndash Mention name of two Congress leaders who had socialist leaning

1ANSWER ndash Subhas Chandra Bose and Jawaharlal Nehru2QUESTION- In which session of the congress Jawaharlal elaborated his Socialist ideas2 ANSWER ndash Lucknow and Faizpur Session in December 1935 and 19363QUESTION ndash Why Congress was sharply divided into leftist and rightist camp 3ANSWER ndash Subhas Chandra Bosersquos attempt to seek re election for congress presidentship in 1939sharply divided the National Congress into Leftist and Rightist camp4 QUESTION ndash Who was MN Roy 4 ANSWER ndash Manabendra Roy first formed the Communist Party of India outside the country at Tashkent in 19205QUESTION ndash Who formed the Congress Socialist Party within the Congress5 ANSWER ndash Jaya Prakash Narayan Achyut Patwardhan Acharya Narendra Dev Ram Mohan Lohia Aruna Asaf Ali6QUESTION ndash When was the Congress Socialist Party formed What was its object6 ANSWER ndash 1934The Congress Socialist Party sought to work out socialist programme through the Congress They joined hands with the Congress and wanted to carry

Subhas Chandra Bose being expelled from the congress after the Tripuri rift he formed Forward BlockThere were basic differences between the Congress Socialists and the communistsTRADE UNION ACTIVITIES Maximum working class people lived in Bombay and Calcutta The working and living conditions of those workers were very miserable In this situation Shasipada Banerjee NM Lokhande protested against the oppression of the working class peopleThe first Trade Union Madras Labour Union was formed in 1918 by BP WadiaIndustrial strikes took place in Kanpur Calcutta Madras Jamshedpur and Ahmedabad AITUC was formed in Bombay in 1927 The growth of Trade union among the workers was slow because of the fear of the dismissal of the jobIn the mean time the Moderates as well as Communists left AITUC and formed separate organization

on National struggle with the help of workers and peasant class of the society7 QUESTION ndash What was the name of the party founded by Subhas Chandra Bose7 ANSWER- Forward Block8QUESTION ndash Who was Shasipada Banerjee8 ANSWER ndash Shasipada Banerjee was a radical Brahmo He founded a working menrsquos club to protest against exploitation of the British rulers towards the working class of India9 QUESTION ndash What was the weekly published by NM Lokhande9ANSWER- Dinabandhu10 QUESTION ndash Who founded Bombay Mill-Hands Association and in which year10 ANSWER- NM Lokhande in189011 QUESTION- Who was BP WadiaANSWER- BPWadia was the founder of Madras Labour Union in191812 QUESTION- What was the name of the first labour union of India12 ANSWER- Madras Labour Union13 QUESTION Who founded the Majur Mahajan 13 ANSWER GANDHIJI14 QUESTION What was the full form of AITUC When it was formed14 ANSWER All India Trade Union Congressin 192715QUESTION Who formed the Red Trade Union Congress and in which year15ANSWER The Communists formed the Red Trade Union Congress16 QUESTION What do you mean by Socialism16 ANSWER Socialism describes any political and economic theory that says the community rather than individuals should own and manage property and natural resources

Subject Eng Literature (The Tempest ndash William Shakespeare) Topic Act III Scene 3 Lines 1 to 52 (Line 52 ndash Brother my lord the Duke Stand to and do as we) Date 13th April 2020 (4th Period)

[Students should read the original play and also the paraphrase given in the school prescribed textbook]Summary Questions amp Answers

o Alonso Sebastian Antonio Gonzalo Adrian Francisco and others wandered about the island in search of Ferdinand and gets tired and hungry of the toil and at the same time gives up all hope of finding him

o Antonio and Sebastian are happy that Alonso is out of hope and decide to make another attempt on his life that night when being so tired they will be sleeping soundly

o Suddenly a solemn and strange music is heard in the air and several strange shapes enter bringing in a banquet These strange shapes then dance round it with gestures of salutation and then inviting the King to eat they depart

o Seeing this strange scene all are inclined to believe the tales told by travelers that there truly are ldquounicornsrdquo and ldquothe phoenixrsquo thronerdquo

1 ALONSO What harmony is this My good friends hark (L18-27)

GONZALO Marvellous sweet music

[Enter several strange shapes bringing in a banquet

they dance about it with gentle actions of salutation

and inviting the King and his companions to eat they depart]ALONSO Give us kind keepers heavens What were theseSEBASTIAN A living drollery Now I will believe

That there are unicorns that in Arabia

There is one tree the phoenixrsquo throne one phoenix

At this hour reigning thereANTONIO Ill believe both

And what does else want credit come to me

And Ill be sworn rsquotis true Travellers neer did lie

Though fools at home condemn rsquoem

(a) How did Prospero present an amazing spectacle before Alonso and his companions

Using his magic powers Prospero ordered strange shapes to lay a banquet before Alonso and his companions The shapes brought several dishes with tasty eatables in them They placed the dishes on a table before Alonso and his companions Then the strange shapes began to dance gracefully around the banquet While dancing they made gestures inviting them to eat the food Then suddenly the shapes disappeared(b) Who were the guests at the strange banquet Describe the lsquoliving drolleryrsquo

Alonso Sebastian Antonio Gonzalo Adrian and Francisco were the guests at the strange banquet

The term ldquoliving drolleryrdquo refers to live entertainment show In this context when Alonso the King of Naples Sebastian his brother Antonio the treacherous brother of Prospero Gonzalo the kind and loyal councillor to the King Adrian and Francisco came to the island they were hungry and weary in their spirits They heard a solemn and strange music They were shocked to see several strange shapes bringing in a banquet and these shapes danced about it with gentle action of salutation inviting the King and his companions to eat After this Sebastian described this show as lsquoliving drolleryrsquo(c) What is lsquophoenixrsquo What are lsquoUnicornsrdquo

The term lsquophoenixrsquo refers to a mythical Arabian bird which lived alone and perched on a solitary tree After one hundred years it expired in flames and rose again from its own ashes

lsquoUnicornsrsquo refers to the mythological four-footed beasts having horns in the centre of their foreheads When the horns are ground into powder the powder was believed to be

an aphrodisiac(d) How does Sebastian explain the puppet show OR Why does the speaker now believe in unicorns and phoenix

Sebastian finds several strange shapes bringing in the banquet They invite the king and his party for dinner and soon depart He tells that if such a strange sight can be a reality there is nothing incredible in the world and from the present moment he will believe anything He says that it is a strange dumb show enacted not by puppets but by living beings It is stranger than a travellerrsquos tale Seeing such a thing

before his own eyes he will no longer disbelieve the story about unicorns and phoenix(e) How do the other characters present respond to this living drollery

At the sight of the lsquoliving drolleryrsquo like Sebastian Gonzalo and Antonio too acted strangely Antonio told that he too now believes in unicorns and phoenix and anything else that seems to be incredible He too now believes in travellersrsquo tales Gonzalo told that if he would report those happenings in Naples nobody will believe him He considers that those gentle shapes were gentler in manner in comparison to the living beings Alonso was at first sight suspicious and told them that those strange shapes conveyed their meaning in expressive gestures when they seemed to lack speech by their movements and sounds Francisco was amazed at their mysterious disappearance

2 ALONSO Not I

(Line 43-52)GONZALO Faith sir you need not fear When we

were boysWho would believe that there were mountaineers

Dewlapped like bulls whose throats had hanging at rsquoem

Wallets of flesh Or that there were such men

Whose heads stood in their breasts Which now we find

Each putter-out of five for one will bring us

Good warrant ofALONSO I will stand to and feed

Although my lastmdashno matter since I feel

The best is past Brother my lord the Duke

Stand to and do as we

(a) How does Alonso respond at the spectacle of the shapes which were sent to them at the instruction of Prospero

After seeing the strange sight of appearing and disappearing of the shapes sent by Prospero to arrange a banquet for them Alonso says that his surprise at having seen those creatures is infinite and he is fully justified in feeling so much surprise He thinks that their shapes their gestures and the sounds they made were indeed amazing Although they do not possess the gift of speech yet they were able to convey their

thoughts by means of their gestures only

(b) What does Prospero say about the views expressed by Alonso regarding the shapes What does Francisco think about the shapesAfter hearing Alonsorsquos views about the shapes Prospero says that this manrsquos praise of the spirits is rather hasty He means to say that Alonso has shown great haste in reaching the conclusion about the shapes Francisco is amazed to see that those shapes disappeared in a mysterious way(c) What does Sebastian ask Alonso to doSebastian tells Alonso that the shapes having disappeared should not matter to them because they have left the eatables behind He asks Alonso to enjoy eating as they are extremely hungry but the king does not accept his offer of enjoying the dishes(d) How does Gonzalo try to dispel Alonsorsquos fear of those strange shapes What kind of references does he give to AlonsoGonzalo says that those who have travelled abroad have reported seeing even stranger sights than these shapes that Alonso and his companions have beheld Hence there is no reason to feel afraid of these shapes Gonzalo further adds that in his younger days he had heard strange stories from travelers and Alonso might have heard similar stories For instance it was said that there existed a certain race of

human beings who had huge lumps of flesh hanging at their throats and who therefore resembled bulls Then Gonzalo tells about a race of human beings whose heads were located at their breasts Gonzalo says that such stories were not believed by most people in those days but now-a-days these stories have become common(e) Explain the following lsquoEach putter-out of five for onersquoEnglish travellers often insured their trips with London brokers Those that went on foreign travels those days used to deposit a certain amount with some firm or company in London before their departure If the travelers failed to return the money was forfeited by the company with which it had been deposited But this money was repaid five-fold if the travelers returned safe and sound In this way a traveler stood a great chance of recovering the entire cost of his

travels(f) Give the explanatory meanings of the following expressions in the context of the above extract (i) Dewlapped (ii) Wallets of flesh

(iii) Putter-out(i) Dewlapped having big lumps of flesh at the necks(ii) Wallets of flesh large masses of flesh looking like bags(iii) Putter-out to invest money before commencing the travel

  • General methods of preparation of hydrogen
  • Chapter Dimensional Analysis (Summary)
    • Properties of Charge
Page 52:   · Web viewSubject. Topic. Summary. Execution. Hindi. व्याकरण. शरीरके अंगो के नाम लिखिए. 1) आँख 2) नाक 3

demand for these commodities decreases when income of the buyer decreases Such goods whose demand increases with the increase in income of the buyer are called normal goods But there are some goods like coarse rice toned milk etc whose demand decreases when income of buyer increases and their demand increases when income of the buyer decreases Such goods whose demand decreases with the increase in income of the buyer are called inferior goods Suppose a consumer buys 10 Kgs of rice whose price is ` 25 per Kg He cannot afford to buy better quality of rice because the price of such rice is ` 50 per Kg The consumer is spending ` 250 per month on the purchase of rice Now if income of the consumer increases and he can afford ` 350 on purchase of 10 Kg of rice Now he can afford to buy some quantity of rice say 6 Kgs whose price is ` 25 per Kg and may buy 4 Kgs of rice whose price is ` 50 per Kg Thus he will buy 10 Kgs of rice by spending ` 350 per month Therefore we may conclude that demand for normal goods is directly related to the income of the buyer but demand for inferior goods is inversely related to the income of the buyer4 Tastes and Preferences of the BuyerThe demand for a commodity is also affected by the tastes and preferences of the buyers They include change in fashion customs habits etc Those commodities are preferred by the consumers which are in fashion So demand for those commodities rises which are in fashion On the other hand if a commodity goes out of the fashion its demand falls because no consumer will like to buy it(5) Number of Buyers in the Market(Population)Increase in population raises the market demand whereas decrease in population reduces the market demand for a commodity Not only the size of population but its composition like age (ratio of males females children and old people in population) also affects the demand for a commodity It is because of needs of children young old male and female population differs(6) Distribution of Income and WealthIf the distribution of income and wealth is more in favour of the rich demand for the commodities preferred by the rich such as comforts and luxuries is likely to be higher On the other hand if the distribution of income and wealth is more in favour of poor demand for commodities preferred by the poor such as necessities will be more(7) Season and Weather ConditionsThis is generally observed that the demand for woolens increases during winter whereas demand for ice creams and cold drinks increases during summer Similarly market demand for umbrellas rain coats increases during rainy seasonQ8 REASONS FOR OPERATION OF LAW OF DEMAND WHY DEMAND CURVE SLOPES DOWNWARDNow we will try to explain why does a consumer purchase more quantity of a commodity at a lower price and less of it at a higher price or why does the law of demand operate ie why does the demand curve slope downwards from left to right The main reasons for operation of law of demand are1 Law of Diminishing Marginal UtilityAs you have studied earlier law of diminishing marginal utility states that as we consume more and more units of a commodity the utility derived from each successive unit goes on decreasing The consumer will be ready to pay more for those units which provide him more utility and less for those which provide him less utility It implies that he will purchase more only when the price of the commodity falls2 Income Effect

When price of a commodity falls purchasing power or real income of the consumer increases which enables him to purchase more quantity of the commodity with the same money income Let us take an example Suppose you buy 4 ice creams when price of each ice cream is ` 25 If price of ice creams falls to ` 20 then with same money income you can buy 5 ice creams now3 Substitution EffectWhen price of a commodity falls it becomes comparatively cheaper as compared to its substitutes (although price of substitutes has not been changed) This will lead to rise in demand for the given commodity For example if coke and Pepsi both are sold at ` 10 each and price of coke falls Now coke has become relatively cheaper and will be substituted for Pepsi It will lead to rise in demand for coke4 Change in Number of BuyersWhen price of a commodity falls some old buyers may demand more of the commodity at the reduced price and some new buyers may also start buying this commodity who were not in a position to buy it earlier due to higher price This will lead to increase in number of buyers when price of the commodity falls As a result demand for the commodity rises when its price falls5 Diverse Uses of a CommoditySome commodities have diverse uses like milk It can be used for drinking for sweet preparation for ice cream preparation etc If price of milk rises its use may be restricted to important purpose only This will lead to reduction in demand for other less important uses When price of milk falls it can be put to other uses also leading to rise n demand for itQ9 EXCEPTIONS TO THE LAW OF DEMANDYou have studied in law of demand that a buyer is willing to buy more quantity of a commodity at a lower price and less of it at a higher price But in certain circumstances a rise in price may lead to rise in demand These circumstances are called Exceptions to the Law of Demand Some important exceptions are1 Giffen GoodsGiffen goods are special type of inferior goods in which negative income effect is stronger than negative substitution effect Giffen goods do not follow law of demand as their demand rises when their price rises Examples of Giffen goods are jowar and bajra etc2 Status Symbol GoodsSome goods are used by rich people as status symbols eg diamonds gold jewellary etc The higher the price the higher will be the demand for these goods When price of such goods falls these goods are no longer looked at as status symbol goods and tehrefore therir demand falls3 NecessitiesCommodities such as medicines salt wheat etc do not follow law of demandbecause we have to purchase them in minimum required quantity whatever their price may be4 Goods Expected to be ScarceWhen the buyers expect a scarcity of a particular good in near future they start buying more and more of that good even if their prices are rising For example during war famines etc people tend to buy more of some goods even at higher prices due to fear of their scarcity in near future

Political Science

Constitution of India-The

Preamble

The preamble-

Preamble-

The preamble is the most precious part of the constitution We the people of India having solemnly resolved to constitute India into a Sovereign Socialist Secular Democratic Republic and to secure to all its citizensA preamble is an introductory and expressionary statement in a document that explains the documents purpose and underlying philosophy When applied to the opening paragraphs of a statute it may recite historical facts pertinent to the subject of the statuteNature and purpose of the constitution-Purpose of the Constitution dictates permanent framework of the government to form a more perfect union to establish justice and ensure peace of thenationconstitution provide principles how the government can run itself following the rules and laws written in the constitution of each state keeps them balanced

Answer the following questions-

1 What is preambleA preamble is an introductory and expressionary statement in a document that explains the documents purpose and underlying philosophy2 What is the nature and

purpose of the constitutionConstitution dictatespermanent framework of the government to form a more perfect union to establish justice and ensure peace of the nation

Homework-Learn

Accounts Compatibilty mode

1MEANING OF PARTNERSHIPPartnership is a form of business organisation where two or more persons join hands to run a business They share the profits and losses according to the agreement amongst them According to the Indian Partnership Act 1932 ldquoPartnership is relation between persons who have agreed to share profits of a business carried on by all or any one of them acting for allrdquo For example one of your friends has passed class XII from National Institute of Open Schooling (NIOS) and wants to start a business Heshe approaches you to join in this venture Heshe wants you to contribute some money and participate in the business activities Both of you if join hands constitute a partnership2CHARACTERISTICS1048698 Agreement A partnership is formed by an agreement The agreement may be either oral or in writing It defines the relationship between the persons who agree to carry on business It may contain the terms of sharing profit and the capital to be invested by each partner etc The written agreement is known as partnership deed1048698 Number of persons There must be at least two persons to form a partnership

The maximum number of partners in a partnership firm can be 50 according toCompanies Act 20131048698 Business The Partnership is formed to carry on business with a purpose of earning profits The business should be lawful Thus if two or more persons agree to carry on unlawful activities it will not be termed as partnership1048698 Sharing Profits The partners agree to share profits in the agreed ratio In caseof loss all the partners have to bear it in the same agreed profit sharing ratio10486981048698Mutual Agency Every partner is an agent of the other partners Every partner can bind the firm and all other partners by hisher acts Each partner will be responsible and liable for the acts of all other partners10486981048698Unlimited liability The liability of each partner except that of a minor is unlimited Their liability extends to their personal assets also If the assets of the firm are insufficient to pay off its debts the partnersrsquo personal property can be used to satisfy the claim of the creditors of the partnership firm10486981048698Management All the partners have a right to mange the business However they may authorize one or more partners to manage the affairs of the business on their behalf10486981048698Transferability of Share No partner can transfer hisher share to any one including hisher family member without the consent of all other partners3PARTNERSHIP DEEDAgreement forms the basis of partnership The written form of the agreement is which a document of partnership is It contains terms and conditions regarding the conduct of the business It also explains relationship between the partners This document is called partnership deed Every firm can frame its own partnership deed in which the rights duties and liabilities of the partners are stated in detail It helps in settling the disputes arising among the partners during the general conduct of business 4CONTENTS OF PARTNERSHIP DEEDThe partnership deed generally contains the following (i) Name and address of the partnership firm(ii) Nature and objectives of the business(iii) Name and address of each partner(iv) Ratio in which profits is to be shared(v) Capital contribution by each partner(vi) Rate of Interest on capital if allowed(vii) Salary or any other remuneration to partners if allowed(viii) Rate of interest on loans and advances by a partner to the firm(ix) Drawings of partners and interest thereon if any(x) Method of valuation of goodwill and revaluation of assets and liabilities on the reconstitution of the partnership ie on the admission retirement or death of a partner(xi) Settlement of disputes by arbitration(xii) Settlement of accounts at the time of retirement or death of a partner5IN ABSENCE OF PARTNERSHIP DEEDThe partnership deed lays down the terms and conditions of partnership in regard to rights duties and obligations of the partners In the absence of partnership deed there may arise a controversy on certain issues like profit sharing ratio interest on

capital interest on drawings interest on loan and salary of the partners In such cases the provisions of the Indian Partnership Act becomes applicableSome of the Issues are(i) Distribution of Profit Partners are entitled to share profits equally(ii) Interest on Capital Interest on capital is not allowed(iii) Interest on Drawings No interest on drawing of the partners is to be charged(iv) Interest on Partnerrsquos Loan A Partner is allowed interest 6 per annum on the amount of loan given to the firm by himher(v) Salary and Commission to Partner A partner is not entitled to anysalary or commission or any other remuneration for managing the business

History TOPIC-TOWARDS INDEPENDENCE AND PARTITION THE LAST PHASE (1935-1947)

SUB TOPIC-IMPORTANT POLITICAL DEVELOPMENTS ndash GROWTH OF SOCIAL IDEAS

Socialism is a political social and economic philosophyLike in other parts of the world the Russian revolution of 1917 served as a great inspiration for revolutionaries in India who at that time were engaged in the struggle for liberation from British ruleSocialist ideas led to the formation of communist party of IndiaJAWAHARLAL NEHRU Among the early Congress leaders Jawaharlal Nehru was very much impressed and influenced by the Socialist ideas He also learnt about the Economic activities of the Soviet Union after the Bolshevic Revolution 1917 He made full use of them in IndiaThe election of Jawaharlal Nehru and Subhas Chandra Bose showed the Left wing tendency within CongressJawaharlal Nehru demanded economic freedom along with political freedom of the people in order to end the exploitation of masses

Nehrus working committee included three socialists leaders The Lucknow session was a landmark in the evolution of socialist ideas of the congressSUBHAS CHANDRA BOSE ndash Subhas Chandra Bose had socialist leaning Both Jawaharlal Nehru and Subhas Chandra Bose were known as leftist Congress men Later on National Congress divided into Leftist and rightist campCONGRESS SOCIALIST Within the Congress some leaders formed the Congress Socialist partyPattavi Sitaramyya Sardar Patel Rajendra Prasad had hostile attitude towards the Congress Socialist partyJawaharlals attitude was hesitant

1 QUESTION ndash Mention name of two Congress leaders who had socialist leaning

1ANSWER ndash Subhas Chandra Bose and Jawaharlal Nehru2QUESTION- In which session of the congress Jawaharlal elaborated his Socialist ideas2 ANSWER ndash Lucknow and Faizpur Session in December 1935 and 19363QUESTION ndash Why Congress was sharply divided into leftist and rightist camp 3ANSWER ndash Subhas Chandra Bosersquos attempt to seek re election for congress presidentship in 1939sharply divided the National Congress into Leftist and Rightist camp4 QUESTION ndash Who was MN Roy 4 ANSWER ndash Manabendra Roy first formed the Communist Party of India outside the country at Tashkent in 19205QUESTION ndash Who formed the Congress Socialist Party within the Congress5 ANSWER ndash Jaya Prakash Narayan Achyut Patwardhan Acharya Narendra Dev Ram Mohan Lohia Aruna Asaf Ali6QUESTION ndash When was the Congress Socialist Party formed What was its object6 ANSWER ndash 1934The Congress Socialist Party sought to work out socialist programme through the Congress They joined hands with the Congress and wanted to carry

Subhas Chandra Bose being expelled from the congress after the Tripuri rift he formed Forward BlockThere were basic differences between the Congress Socialists and the communistsTRADE UNION ACTIVITIES Maximum working class people lived in Bombay and Calcutta The working and living conditions of those workers were very miserable In this situation Shasipada Banerjee NM Lokhande protested against the oppression of the working class peopleThe first Trade Union Madras Labour Union was formed in 1918 by BP WadiaIndustrial strikes took place in Kanpur Calcutta Madras Jamshedpur and Ahmedabad AITUC was formed in Bombay in 1927 The growth of Trade union among the workers was slow because of the fear of the dismissal of the jobIn the mean time the Moderates as well as Communists left AITUC and formed separate organization

on National struggle with the help of workers and peasant class of the society7 QUESTION ndash What was the name of the party founded by Subhas Chandra Bose7 ANSWER- Forward Block8QUESTION ndash Who was Shasipada Banerjee8 ANSWER ndash Shasipada Banerjee was a radical Brahmo He founded a working menrsquos club to protest against exploitation of the British rulers towards the working class of India9 QUESTION ndash What was the weekly published by NM Lokhande9ANSWER- Dinabandhu10 QUESTION ndash Who founded Bombay Mill-Hands Association and in which year10 ANSWER- NM Lokhande in189011 QUESTION- Who was BP WadiaANSWER- BPWadia was the founder of Madras Labour Union in191812 QUESTION- What was the name of the first labour union of India12 ANSWER- Madras Labour Union13 QUESTION Who founded the Majur Mahajan 13 ANSWER GANDHIJI14 QUESTION What was the full form of AITUC When it was formed14 ANSWER All India Trade Union Congressin 192715QUESTION Who formed the Red Trade Union Congress and in which year15ANSWER The Communists formed the Red Trade Union Congress16 QUESTION What do you mean by Socialism16 ANSWER Socialism describes any political and economic theory that says the community rather than individuals should own and manage property and natural resources

Subject Eng Literature (The Tempest ndash William Shakespeare) Topic Act III Scene 3 Lines 1 to 52 (Line 52 ndash Brother my lord the Duke Stand to and do as we) Date 13th April 2020 (4th Period)

[Students should read the original play and also the paraphrase given in the school prescribed textbook]Summary Questions amp Answers

o Alonso Sebastian Antonio Gonzalo Adrian Francisco and others wandered about the island in search of Ferdinand and gets tired and hungry of the toil and at the same time gives up all hope of finding him

o Antonio and Sebastian are happy that Alonso is out of hope and decide to make another attempt on his life that night when being so tired they will be sleeping soundly

o Suddenly a solemn and strange music is heard in the air and several strange shapes enter bringing in a banquet These strange shapes then dance round it with gestures of salutation and then inviting the King to eat they depart

o Seeing this strange scene all are inclined to believe the tales told by travelers that there truly are ldquounicornsrdquo and ldquothe phoenixrsquo thronerdquo

1 ALONSO What harmony is this My good friends hark (L18-27)

GONZALO Marvellous sweet music

[Enter several strange shapes bringing in a banquet

they dance about it with gentle actions of salutation

and inviting the King and his companions to eat they depart]ALONSO Give us kind keepers heavens What were theseSEBASTIAN A living drollery Now I will believe

That there are unicorns that in Arabia

There is one tree the phoenixrsquo throne one phoenix

At this hour reigning thereANTONIO Ill believe both

And what does else want credit come to me

And Ill be sworn rsquotis true Travellers neer did lie

Though fools at home condemn rsquoem

(a) How did Prospero present an amazing spectacle before Alonso and his companions

Using his magic powers Prospero ordered strange shapes to lay a banquet before Alonso and his companions The shapes brought several dishes with tasty eatables in them They placed the dishes on a table before Alonso and his companions Then the strange shapes began to dance gracefully around the banquet While dancing they made gestures inviting them to eat the food Then suddenly the shapes disappeared(b) Who were the guests at the strange banquet Describe the lsquoliving drolleryrsquo

Alonso Sebastian Antonio Gonzalo Adrian and Francisco were the guests at the strange banquet

The term ldquoliving drolleryrdquo refers to live entertainment show In this context when Alonso the King of Naples Sebastian his brother Antonio the treacherous brother of Prospero Gonzalo the kind and loyal councillor to the King Adrian and Francisco came to the island they were hungry and weary in their spirits They heard a solemn and strange music They were shocked to see several strange shapes bringing in a banquet and these shapes danced about it with gentle action of salutation inviting the King and his companions to eat After this Sebastian described this show as lsquoliving drolleryrsquo(c) What is lsquophoenixrsquo What are lsquoUnicornsrdquo

The term lsquophoenixrsquo refers to a mythical Arabian bird which lived alone and perched on a solitary tree After one hundred years it expired in flames and rose again from its own ashes

lsquoUnicornsrsquo refers to the mythological four-footed beasts having horns in the centre of their foreheads When the horns are ground into powder the powder was believed to be

an aphrodisiac(d) How does Sebastian explain the puppet show OR Why does the speaker now believe in unicorns and phoenix

Sebastian finds several strange shapes bringing in the banquet They invite the king and his party for dinner and soon depart He tells that if such a strange sight can be a reality there is nothing incredible in the world and from the present moment he will believe anything He says that it is a strange dumb show enacted not by puppets but by living beings It is stranger than a travellerrsquos tale Seeing such a thing

before his own eyes he will no longer disbelieve the story about unicorns and phoenix(e) How do the other characters present respond to this living drollery

At the sight of the lsquoliving drolleryrsquo like Sebastian Gonzalo and Antonio too acted strangely Antonio told that he too now believes in unicorns and phoenix and anything else that seems to be incredible He too now believes in travellersrsquo tales Gonzalo told that if he would report those happenings in Naples nobody will believe him He considers that those gentle shapes were gentler in manner in comparison to the living beings Alonso was at first sight suspicious and told them that those strange shapes conveyed their meaning in expressive gestures when they seemed to lack speech by their movements and sounds Francisco was amazed at their mysterious disappearance

2 ALONSO Not I

(Line 43-52)GONZALO Faith sir you need not fear When we

were boysWho would believe that there were mountaineers

Dewlapped like bulls whose throats had hanging at rsquoem

Wallets of flesh Or that there were such men

Whose heads stood in their breasts Which now we find

Each putter-out of five for one will bring us

Good warrant ofALONSO I will stand to and feed

Although my lastmdashno matter since I feel

The best is past Brother my lord the Duke

Stand to and do as we

(a) How does Alonso respond at the spectacle of the shapes which were sent to them at the instruction of Prospero

After seeing the strange sight of appearing and disappearing of the shapes sent by Prospero to arrange a banquet for them Alonso says that his surprise at having seen those creatures is infinite and he is fully justified in feeling so much surprise He thinks that their shapes their gestures and the sounds they made were indeed amazing Although they do not possess the gift of speech yet they were able to convey their

thoughts by means of their gestures only

(b) What does Prospero say about the views expressed by Alonso regarding the shapes What does Francisco think about the shapesAfter hearing Alonsorsquos views about the shapes Prospero says that this manrsquos praise of the spirits is rather hasty He means to say that Alonso has shown great haste in reaching the conclusion about the shapes Francisco is amazed to see that those shapes disappeared in a mysterious way(c) What does Sebastian ask Alonso to doSebastian tells Alonso that the shapes having disappeared should not matter to them because they have left the eatables behind He asks Alonso to enjoy eating as they are extremely hungry but the king does not accept his offer of enjoying the dishes(d) How does Gonzalo try to dispel Alonsorsquos fear of those strange shapes What kind of references does he give to AlonsoGonzalo says that those who have travelled abroad have reported seeing even stranger sights than these shapes that Alonso and his companions have beheld Hence there is no reason to feel afraid of these shapes Gonzalo further adds that in his younger days he had heard strange stories from travelers and Alonso might have heard similar stories For instance it was said that there existed a certain race of

human beings who had huge lumps of flesh hanging at their throats and who therefore resembled bulls Then Gonzalo tells about a race of human beings whose heads were located at their breasts Gonzalo says that such stories were not believed by most people in those days but now-a-days these stories have become common(e) Explain the following lsquoEach putter-out of five for onersquoEnglish travellers often insured their trips with London brokers Those that went on foreign travels those days used to deposit a certain amount with some firm or company in London before their departure If the travelers failed to return the money was forfeited by the company with which it had been deposited But this money was repaid five-fold if the travelers returned safe and sound In this way a traveler stood a great chance of recovering the entire cost of his

travels(f) Give the explanatory meanings of the following expressions in the context of the above extract (i) Dewlapped (ii) Wallets of flesh

(iii) Putter-out(i) Dewlapped having big lumps of flesh at the necks(ii) Wallets of flesh large masses of flesh looking like bags(iii) Putter-out to invest money before commencing the travel

  • General methods of preparation of hydrogen
  • Chapter Dimensional Analysis (Summary)
    • Properties of Charge
Page 53:   · Web viewSubject. Topic. Summary. Execution. Hindi. व्याकरण. शरीरके अंगो के नाम लिखिए. 1) आँख 2) नाक 3

When price of a commodity falls purchasing power or real income of the consumer increases which enables him to purchase more quantity of the commodity with the same money income Let us take an example Suppose you buy 4 ice creams when price of each ice cream is ` 25 If price of ice creams falls to ` 20 then with same money income you can buy 5 ice creams now3 Substitution EffectWhen price of a commodity falls it becomes comparatively cheaper as compared to its substitutes (although price of substitutes has not been changed) This will lead to rise in demand for the given commodity For example if coke and Pepsi both are sold at ` 10 each and price of coke falls Now coke has become relatively cheaper and will be substituted for Pepsi It will lead to rise in demand for coke4 Change in Number of BuyersWhen price of a commodity falls some old buyers may demand more of the commodity at the reduced price and some new buyers may also start buying this commodity who were not in a position to buy it earlier due to higher price This will lead to increase in number of buyers when price of the commodity falls As a result demand for the commodity rises when its price falls5 Diverse Uses of a CommoditySome commodities have diverse uses like milk It can be used for drinking for sweet preparation for ice cream preparation etc If price of milk rises its use may be restricted to important purpose only This will lead to reduction in demand for other less important uses When price of milk falls it can be put to other uses also leading to rise n demand for itQ9 EXCEPTIONS TO THE LAW OF DEMANDYou have studied in law of demand that a buyer is willing to buy more quantity of a commodity at a lower price and less of it at a higher price But in certain circumstances a rise in price may lead to rise in demand These circumstances are called Exceptions to the Law of Demand Some important exceptions are1 Giffen GoodsGiffen goods are special type of inferior goods in which negative income effect is stronger than negative substitution effect Giffen goods do not follow law of demand as their demand rises when their price rises Examples of Giffen goods are jowar and bajra etc2 Status Symbol GoodsSome goods are used by rich people as status symbols eg diamonds gold jewellary etc The higher the price the higher will be the demand for these goods When price of such goods falls these goods are no longer looked at as status symbol goods and tehrefore therir demand falls3 NecessitiesCommodities such as medicines salt wheat etc do not follow law of demandbecause we have to purchase them in minimum required quantity whatever their price may be4 Goods Expected to be ScarceWhen the buyers expect a scarcity of a particular good in near future they start buying more and more of that good even if their prices are rising For example during war famines etc people tend to buy more of some goods even at higher prices due to fear of their scarcity in near future

Political Science

Constitution of India-The

Preamble

The preamble-

Preamble-

The preamble is the most precious part of the constitution We the people of India having solemnly resolved to constitute India into a Sovereign Socialist Secular Democratic Republic and to secure to all its citizensA preamble is an introductory and expressionary statement in a document that explains the documents purpose and underlying philosophy When applied to the opening paragraphs of a statute it may recite historical facts pertinent to the subject of the statuteNature and purpose of the constitution-Purpose of the Constitution dictates permanent framework of the government to form a more perfect union to establish justice and ensure peace of thenationconstitution provide principles how the government can run itself following the rules and laws written in the constitution of each state keeps them balanced

Answer the following questions-

1 What is preambleA preamble is an introductory and expressionary statement in a document that explains the documents purpose and underlying philosophy2 What is the nature and

purpose of the constitutionConstitution dictatespermanent framework of the government to form a more perfect union to establish justice and ensure peace of the nation

Homework-Learn

Accounts Compatibilty mode

1MEANING OF PARTNERSHIPPartnership is a form of business organisation where two or more persons join hands to run a business They share the profits and losses according to the agreement amongst them According to the Indian Partnership Act 1932 ldquoPartnership is relation between persons who have agreed to share profits of a business carried on by all or any one of them acting for allrdquo For example one of your friends has passed class XII from National Institute of Open Schooling (NIOS) and wants to start a business Heshe approaches you to join in this venture Heshe wants you to contribute some money and participate in the business activities Both of you if join hands constitute a partnership2CHARACTERISTICS1048698 Agreement A partnership is formed by an agreement The agreement may be either oral or in writing It defines the relationship between the persons who agree to carry on business It may contain the terms of sharing profit and the capital to be invested by each partner etc The written agreement is known as partnership deed1048698 Number of persons There must be at least two persons to form a partnership

The maximum number of partners in a partnership firm can be 50 according toCompanies Act 20131048698 Business The Partnership is formed to carry on business with a purpose of earning profits The business should be lawful Thus if two or more persons agree to carry on unlawful activities it will not be termed as partnership1048698 Sharing Profits The partners agree to share profits in the agreed ratio In caseof loss all the partners have to bear it in the same agreed profit sharing ratio10486981048698Mutual Agency Every partner is an agent of the other partners Every partner can bind the firm and all other partners by hisher acts Each partner will be responsible and liable for the acts of all other partners10486981048698Unlimited liability The liability of each partner except that of a minor is unlimited Their liability extends to their personal assets also If the assets of the firm are insufficient to pay off its debts the partnersrsquo personal property can be used to satisfy the claim of the creditors of the partnership firm10486981048698Management All the partners have a right to mange the business However they may authorize one or more partners to manage the affairs of the business on their behalf10486981048698Transferability of Share No partner can transfer hisher share to any one including hisher family member without the consent of all other partners3PARTNERSHIP DEEDAgreement forms the basis of partnership The written form of the agreement is which a document of partnership is It contains terms and conditions regarding the conduct of the business It also explains relationship between the partners This document is called partnership deed Every firm can frame its own partnership deed in which the rights duties and liabilities of the partners are stated in detail It helps in settling the disputes arising among the partners during the general conduct of business 4CONTENTS OF PARTNERSHIP DEEDThe partnership deed generally contains the following (i) Name and address of the partnership firm(ii) Nature and objectives of the business(iii) Name and address of each partner(iv) Ratio in which profits is to be shared(v) Capital contribution by each partner(vi) Rate of Interest on capital if allowed(vii) Salary or any other remuneration to partners if allowed(viii) Rate of interest on loans and advances by a partner to the firm(ix) Drawings of partners and interest thereon if any(x) Method of valuation of goodwill and revaluation of assets and liabilities on the reconstitution of the partnership ie on the admission retirement or death of a partner(xi) Settlement of disputes by arbitration(xii) Settlement of accounts at the time of retirement or death of a partner5IN ABSENCE OF PARTNERSHIP DEEDThe partnership deed lays down the terms and conditions of partnership in regard to rights duties and obligations of the partners In the absence of partnership deed there may arise a controversy on certain issues like profit sharing ratio interest on

capital interest on drawings interest on loan and salary of the partners In such cases the provisions of the Indian Partnership Act becomes applicableSome of the Issues are(i) Distribution of Profit Partners are entitled to share profits equally(ii) Interest on Capital Interest on capital is not allowed(iii) Interest on Drawings No interest on drawing of the partners is to be charged(iv) Interest on Partnerrsquos Loan A Partner is allowed interest 6 per annum on the amount of loan given to the firm by himher(v) Salary and Commission to Partner A partner is not entitled to anysalary or commission or any other remuneration for managing the business

History TOPIC-TOWARDS INDEPENDENCE AND PARTITION THE LAST PHASE (1935-1947)

SUB TOPIC-IMPORTANT POLITICAL DEVELOPMENTS ndash GROWTH OF SOCIAL IDEAS

Socialism is a political social and economic philosophyLike in other parts of the world the Russian revolution of 1917 served as a great inspiration for revolutionaries in India who at that time were engaged in the struggle for liberation from British ruleSocialist ideas led to the formation of communist party of IndiaJAWAHARLAL NEHRU Among the early Congress leaders Jawaharlal Nehru was very much impressed and influenced by the Socialist ideas He also learnt about the Economic activities of the Soviet Union after the Bolshevic Revolution 1917 He made full use of them in IndiaThe election of Jawaharlal Nehru and Subhas Chandra Bose showed the Left wing tendency within CongressJawaharlal Nehru demanded economic freedom along with political freedom of the people in order to end the exploitation of masses

Nehrus working committee included three socialists leaders The Lucknow session was a landmark in the evolution of socialist ideas of the congressSUBHAS CHANDRA BOSE ndash Subhas Chandra Bose had socialist leaning Both Jawaharlal Nehru and Subhas Chandra Bose were known as leftist Congress men Later on National Congress divided into Leftist and rightist campCONGRESS SOCIALIST Within the Congress some leaders formed the Congress Socialist partyPattavi Sitaramyya Sardar Patel Rajendra Prasad had hostile attitude towards the Congress Socialist partyJawaharlals attitude was hesitant

1 QUESTION ndash Mention name of two Congress leaders who had socialist leaning

1ANSWER ndash Subhas Chandra Bose and Jawaharlal Nehru2QUESTION- In which session of the congress Jawaharlal elaborated his Socialist ideas2 ANSWER ndash Lucknow and Faizpur Session in December 1935 and 19363QUESTION ndash Why Congress was sharply divided into leftist and rightist camp 3ANSWER ndash Subhas Chandra Bosersquos attempt to seek re election for congress presidentship in 1939sharply divided the National Congress into Leftist and Rightist camp4 QUESTION ndash Who was MN Roy 4 ANSWER ndash Manabendra Roy first formed the Communist Party of India outside the country at Tashkent in 19205QUESTION ndash Who formed the Congress Socialist Party within the Congress5 ANSWER ndash Jaya Prakash Narayan Achyut Patwardhan Acharya Narendra Dev Ram Mohan Lohia Aruna Asaf Ali6QUESTION ndash When was the Congress Socialist Party formed What was its object6 ANSWER ndash 1934The Congress Socialist Party sought to work out socialist programme through the Congress They joined hands with the Congress and wanted to carry

Subhas Chandra Bose being expelled from the congress after the Tripuri rift he formed Forward BlockThere were basic differences between the Congress Socialists and the communistsTRADE UNION ACTIVITIES Maximum working class people lived in Bombay and Calcutta The working and living conditions of those workers were very miserable In this situation Shasipada Banerjee NM Lokhande protested against the oppression of the working class peopleThe first Trade Union Madras Labour Union was formed in 1918 by BP WadiaIndustrial strikes took place in Kanpur Calcutta Madras Jamshedpur and Ahmedabad AITUC was formed in Bombay in 1927 The growth of Trade union among the workers was slow because of the fear of the dismissal of the jobIn the mean time the Moderates as well as Communists left AITUC and formed separate organization

on National struggle with the help of workers and peasant class of the society7 QUESTION ndash What was the name of the party founded by Subhas Chandra Bose7 ANSWER- Forward Block8QUESTION ndash Who was Shasipada Banerjee8 ANSWER ndash Shasipada Banerjee was a radical Brahmo He founded a working menrsquos club to protest against exploitation of the British rulers towards the working class of India9 QUESTION ndash What was the weekly published by NM Lokhande9ANSWER- Dinabandhu10 QUESTION ndash Who founded Bombay Mill-Hands Association and in which year10 ANSWER- NM Lokhande in189011 QUESTION- Who was BP WadiaANSWER- BPWadia was the founder of Madras Labour Union in191812 QUESTION- What was the name of the first labour union of India12 ANSWER- Madras Labour Union13 QUESTION Who founded the Majur Mahajan 13 ANSWER GANDHIJI14 QUESTION What was the full form of AITUC When it was formed14 ANSWER All India Trade Union Congressin 192715QUESTION Who formed the Red Trade Union Congress and in which year15ANSWER The Communists formed the Red Trade Union Congress16 QUESTION What do you mean by Socialism16 ANSWER Socialism describes any political and economic theory that says the community rather than individuals should own and manage property and natural resources

Subject Eng Literature (The Tempest ndash William Shakespeare) Topic Act III Scene 3 Lines 1 to 52 (Line 52 ndash Brother my lord the Duke Stand to and do as we) Date 13th April 2020 (4th Period)

[Students should read the original play and also the paraphrase given in the school prescribed textbook]Summary Questions amp Answers

o Alonso Sebastian Antonio Gonzalo Adrian Francisco and others wandered about the island in search of Ferdinand and gets tired and hungry of the toil and at the same time gives up all hope of finding him

o Antonio and Sebastian are happy that Alonso is out of hope and decide to make another attempt on his life that night when being so tired they will be sleeping soundly

o Suddenly a solemn and strange music is heard in the air and several strange shapes enter bringing in a banquet These strange shapes then dance round it with gestures of salutation and then inviting the King to eat they depart

o Seeing this strange scene all are inclined to believe the tales told by travelers that there truly are ldquounicornsrdquo and ldquothe phoenixrsquo thronerdquo

1 ALONSO What harmony is this My good friends hark (L18-27)

GONZALO Marvellous sweet music

[Enter several strange shapes bringing in a banquet

they dance about it with gentle actions of salutation

and inviting the King and his companions to eat they depart]ALONSO Give us kind keepers heavens What were theseSEBASTIAN A living drollery Now I will believe

That there are unicorns that in Arabia

There is one tree the phoenixrsquo throne one phoenix

At this hour reigning thereANTONIO Ill believe both

And what does else want credit come to me

And Ill be sworn rsquotis true Travellers neer did lie

Though fools at home condemn rsquoem

(a) How did Prospero present an amazing spectacle before Alonso and his companions

Using his magic powers Prospero ordered strange shapes to lay a banquet before Alonso and his companions The shapes brought several dishes with tasty eatables in them They placed the dishes on a table before Alonso and his companions Then the strange shapes began to dance gracefully around the banquet While dancing they made gestures inviting them to eat the food Then suddenly the shapes disappeared(b) Who were the guests at the strange banquet Describe the lsquoliving drolleryrsquo

Alonso Sebastian Antonio Gonzalo Adrian and Francisco were the guests at the strange banquet

The term ldquoliving drolleryrdquo refers to live entertainment show In this context when Alonso the King of Naples Sebastian his brother Antonio the treacherous brother of Prospero Gonzalo the kind and loyal councillor to the King Adrian and Francisco came to the island they were hungry and weary in their spirits They heard a solemn and strange music They were shocked to see several strange shapes bringing in a banquet and these shapes danced about it with gentle action of salutation inviting the King and his companions to eat After this Sebastian described this show as lsquoliving drolleryrsquo(c) What is lsquophoenixrsquo What are lsquoUnicornsrdquo

The term lsquophoenixrsquo refers to a mythical Arabian bird which lived alone and perched on a solitary tree After one hundred years it expired in flames and rose again from its own ashes

lsquoUnicornsrsquo refers to the mythological four-footed beasts having horns in the centre of their foreheads When the horns are ground into powder the powder was believed to be

an aphrodisiac(d) How does Sebastian explain the puppet show OR Why does the speaker now believe in unicorns and phoenix

Sebastian finds several strange shapes bringing in the banquet They invite the king and his party for dinner and soon depart He tells that if such a strange sight can be a reality there is nothing incredible in the world and from the present moment he will believe anything He says that it is a strange dumb show enacted not by puppets but by living beings It is stranger than a travellerrsquos tale Seeing such a thing

before his own eyes he will no longer disbelieve the story about unicorns and phoenix(e) How do the other characters present respond to this living drollery

At the sight of the lsquoliving drolleryrsquo like Sebastian Gonzalo and Antonio too acted strangely Antonio told that he too now believes in unicorns and phoenix and anything else that seems to be incredible He too now believes in travellersrsquo tales Gonzalo told that if he would report those happenings in Naples nobody will believe him He considers that those gentle shapes were gentler in manner in comparison to the living beings Alonso was at first sight suspicious and told them that those strange shapes conveyed their meaning in expressive gestures when they seemed to lack speech by their movements and sounds Francisco was amazed at their mysterious disappearance

2 ALONSO Not I

(Line 43-52)GONZALO Faith sir you need not fear When we

were boysWho would believe that there were mountaineers

Dewlapped like bulls whose throats had hanging at rsquoem

Wallets of flesh Or that there were such men

Whose heads stood in their breasts Which now we find

Each putter-out of five for one will bring us

Good warrant ofALONSO I will stand to and feed

Although my lastmdashno matter since I feel

The best is past Brother my lord the Duke

Stand to and do as we

(a) How does Alonso respond at the spectacle of the shapes which were sent to them at the instruction of Prospero

After seeing the strange sight of appearing and disappearing of the shapes sent by Prospero to arrange a banquet for them Alonso says that his surprise at having seen those creatures is infinite and he is fully justified in feeling so much surprise He thinks that their shapes their gestures and the sounds they made were indeed amazing Although they do not possess the gift of speech yet they were able to convey their

thoughts by means of their gestures only

(b) What does Prospero say about the views expressed by Alonso regarding the shapes What does Francisco think about the shapesAfter hearing Alonsorsquos views about the shapes Prospero says that this manrsquos praise of the spirits is rather hasty He means to say that Alonso has shown great haste in reaching the conclusion about the shapes Francisco is amazed to see that those shapes disappeared in a mysterious way(c) What does Sebastian ask Alonso to doSebastian tells Alonso that the shapes having disappeared should not matter to them because they have left the eatables behind He asks Alonso to enjoy eating as they are extremely hungry but the king does not accept his offer of enjoying the dishes(d) How does Gonzalo try to dispel Alonsorsquos fear of those strange shapes What kind of references does he give to AlonsoGonzalo says that those who have travelled abroad have reported seeing even stranger sights than these shapes that Alonso and his companions have beheld Hence there is no reason to feel afraid of these shapes Gonzalo further adds that in his younger days he had heard strange stories from travelers and Alonso might have heard similar stories For instance it was said that there existed a certain race of

human beings who had huge lumps of flesh hanging at their throats and who therefore resembled bulls Then Gonzalo tells about a race of human beings whose heads were located at their breasts Gonzalo says that such stories were not believed by most people in those days but now-a-days these stories have become common(e) Explain the following lsquoEach putter-out of five for onersquoEnglish travellers often insured their trips with London brokers Those that went on foreign travels those days used to deposit a certain amount with some firm or company in London before their departure If the travelers failed to return the money was forfeited by the company with which it had been deposited But this money was repaid five-fold if the travelers returned safe and sound In this way a traveler stood a great chance of recovering the entire cost of his

travels(f) Give the explanatory meanings of the following expressions in the context of the above extract (i) Dewlapped (ii) Wallets of flesh

(iii) Putter-out(i) Dewlapped having big lumps of flesh at the necks(ii) Wallets of flesh large masses of flesh looking like bags(iii) Putter-out to invest money before commencing the travel

  • General methods of preparation of hydrogen
  • Chapter Dimensional Analysis (Summary)
    • Properties of Charge
Page 54:   · Web viewSubject. Topic. Summary. Execution. Hindi. व्याकरण. शरीरके अंगो के नाम लिखिए. 1) आँख 2) नाक 3

Political Science

Constitution of India-The

Preamble

The preamble-

Preamble-

The preamble is the most precious part of the constitution We the people of India having solemnly resolved to constitute India into a Sovereign Socialist Secular Democratic Republic and to secure to all its citizensA preamble is an introductory and expressionary statement in a document that explains the documents purpose and underlying philosophy When applied to the opening paragraphs of a statute it may recite historical facts pertinent to the subject of the statuteNature and purpose of the constitution-Purpose of the Constitution dictates permanent framework of the government to form a more perfect union to establish justice and ensure peace of thenationconstitution provide principles how the government can run itself following the rules and laws written in the constitution of each state keeps them balanced

Answer the following questions-

1 What is preambleA preamble is an introductory and expressionary statement in a document that explains the documents purpose and underlying philosophy2 What is the nature and

purpose of the constitutionConstitution dictatespermanent framework of the government to form a more perfect union to establish justice and ensure peace of the nation

Homework-Learn

Accounts Compatibilty mode

1MEANING OF PARTNERSHIPPartnership is a form of business organisation where two or more persons join hands to run a business They share the profits and losses according to the agreement amongst them According to the Indian Partnership Act 1932 ldquoPartnership is relation between persons who have agreed to share profits of a business carried on by all or any one of them acting for allrdquo For example one of your friends has passed class XII from National Institute of Open Schooling (NIOS) and wants to start a business Heshe approaches you to join in this venture Heshe wants you to contribute some money and participate in the business activities Both of you if join hands constitute a partnership2CHARACTERISTICS1048698 Agreement A partnership is formed by an agreement The agreement may be either oral or in writing It defines the relationship between the persons who agree to carry on business It may contain the terms of sharing profit and the capital to be invested by each partner etc The written agreement is known as partnership deed1048698 Number of persons There must be at least two persons to form a partnership

The maximum number of partners in a partnership firm can be 50 according toCompanies Act 20131048698 Business The Partnership is formed to carry on business with a purpose of earning profits The business should be lawful Thus if two or more persons agree to carry on unlawful activities it will not be termed as partnership1048698 Sharing Profits The partners agree to share profits in the agreed ratio In caseof loss all the partners have to bear it in the same agreed profit sharing ratio10486981048698Mutual Agency Every partner is an agent of the other partners Every partner can bind the firm and all other partners by hisher acts Each partner will be responsible and liable for the acts of all other partners10486981048698Unlimited liability The liability of each partner except that of a minor is unlimited Their liability extends to their personal assets also If the assets of the firm are insufficient to pay off its debts the partnersrsquo personal property can be used to satisfy the claim of the creditors of the partnership firm10486981048698Management All the partners have a right to mange the business However they may authorize one or more partners to manage the affairs of the business on their behalf10486981048698Transferability of Share No partner can transfer hisher share to any one including hisher family member without the consent of all other partners3PARTNERSHIP DEEDAgreement forms the basis of partnership The written form of the agreement is which a document of partnership is It contains terms and conditions regarding the conduct of the business It also explains relationship between the partners This document is called partnership deed Every firm can frame its own partnership deed in which the rights duties and liabilities of the partners are stated in detail It helps in settling the disputes arising among the partners during the general conduct of business 4CONTENTS OF PARTNERSHIP DEEDThe partnership deed generally contains the following (i) Name and address of the partnership firm(ii) Nature and objectives of the business(iii) Name and address of each partner(iv) Ratio in which profits is to be shared(v) Capital contribution by each partner(vi) Rate of Interest on capital if allowed(vii) Salary or any other remuneration to partners if allowed(viii) Rate of interest on loans and advances by a partner to the firm(ix) Drawings of partners and interest thereon if any(x) Method of valuation of goodwill and revaluation of assets and liabilities on the reconstitution of the partnership ie on the admission retirement or death of a partner(xi) Settlement of disputes by arbitration(xii) Settlement of accounts at the time of retirement or death of a partner5IN ABSENCE OF PARTNERSHIP DEEDThe partnership deed lays down the terms and conditions of partnership in regard to rights duties and obligations of the partners In the absence of partnership deed there may arise a controversy on certain issues like profit sharing ratio interest on

capital interest on drawings interest on loan and salary of the partners In such cases the provisions of the Indian Partnership Act becomes applicableSome of the Issues are(i) Distribution of Profit Partners are entitled to share profits equally(ii) Interest on Capital Interest on capital is not allowed(iii) Interest on Drawings No interest on drawing of the partners is to be charged(iv) Interest on Partnerrsquos Loan A Partner is allowed interest 6 per annum on the amount of loan given to the firm by himher(v) Salary and Commission to Partner A partner is not entitled to anysalary or commission or any other remuneration for managing the business

History TOPIC-TOWARDS INDEPENDENCE AND PARTITION THE LAST PHASE (1935-1947)

SUB TOPIC-IMPORTANT POLITICAL DEVELOPMENTS ndash GROWTH OF SOCIAL IDEAS

Socialism is a political social and economic philosophyLike in other parts of the world the Russian revolution of 1917 served as a great inspiration for revolutionaries in India who at that time were engaged in the struggle for liberation from British ruleSocialist ideas led to the formation of communist party of IndiaJAWAHARLAL NEHRU Among the early Congress leaders Jawaharlal Nehru was very much impressed and influenced by the Socialist ideas He also learnt about the Economic activities of the Soviet Union after the Bolshevic Revolution 1917 He made full use of them in IndiaThe election of Jawaharlal Nehru and Subhas Chandra Bose showed the Left wing tendency within CongressJawaharlal Nehru demanded economic freedom along with political freedom of the people in order to end the exploitation of masses

Nehrus working committee included three socialists leaders The Lucknow session was a landmark in the evolution of socialist ideas of the congressSUBHAS CHANDRA BOSE ndash Subhas Chandra Bose had socialist leaning Both Jawaharlal Nehru and Subhas Chandra Bose were known as leftist Congress men Later on National Congress divided into Leftist and rightist campCONGRESS SOCIALIST Within the Congress some leaders formed the Congress Socialist partyPattavi Sitaramyya Sardar Patel Rajendra Prasad had hostile attitude towards the Congress Socialist partyJawaharlals attitude was hesitant

1 QUESTION ndash Mention name of two Congress leaders who had socialist leaning

1ANSWER ndash Subhas Chandra Bose and Jawaharlal Nehru2QUESTION- In which session of the congress Jawaharlal elaborated his Socialist ideas2 ANSWER ndash Lucknow and Faizpur Session in December 1935 and 19363QUESTION ndash Why Congress was sharply divided into leftist and rightist camp 3ANSWER ndash Subhas Chandra Bosersquos attempt to seek re election for congress presidentship in 1939sharply divided the National Congress into Leftist and Rightist camp4 QUESTION ndash Who was MN Roy 4 ANSWER ndash Manabendra Roy first formed the Communist Party of India outside the country at Tashkent in 19205QUESTION ndash Who formed the Congress Socialist Party within the Congress5 ANSWER ndash Jaya Prakash Narayan Achyut Patwardhan Acharya Narendra Dev Ram Mohan Lohia Aruna Asaf Ali6QUESTION ndash When was the Congress Socialist Party formed What was its object6 ANSWER ndash 1934The Congress Socialist Party sought to work out socialist programme through the Congress They joined hands with the Congress and wanted to carry

Subhas Chandra Bose being expelled from the congress after the Tripuri rift he formed Forward BlockThere were basic differences between the Congress Socialists and the communistsTRADE UNION ACTIVITIES Maximum working class people lived in Bombay and Calcutta The working and living conditions of those workers were very miserable In this situation Shasipada Banerjee NM Lokhande protested against the oppression of the working class peopleThe first Trade Union Madras Labour Union was formed in 1918 by BP WadiaIndustrial strikes took place in Kanpur Calcutta Madras Jamshedpur and Ahmedabad AITUC was formed in Bombay in 1927 The growth of Trade union among the workers was slow because of the fear of the dismissal of the jobIn the mean time the Moderates as well as Communists left AITUC and formed separate organization

on National struggle with the help of workers and peasant class of the society7 QUESTION ndash What was the name of the party founded by Subhas Chandra Bose7 ANSWER- Forward Block8QUESTION ndash Who was Shasipada Banerjee8 ANSWER ndash Shasipada Banerjee was a radical Brahmo He founded a working menrsquos club to protest against exploitation of the British rulers towards the working class of India9 QUESTION ndash What was the weekly published by NM Lokhande9ANSWER- Dinabandhu10 QUESTION ndash Who founded Bombay Mill-Hands Association and in which year10 ANSWER- NM Lokhande in189011 QUESTION- Who was BP WadiaANSWER- BPWadia was the founder of Madras Labour Union in191812 QUESTION- What was the name of the first labour union of India12 ANSWER- Madras Labour Union13 QUESTION Who founded the Majur Mahajan 13 ANSWER GANDHIJI14 QUESTION What was the full form of AITUC When it was formed14 ANSWER All India Trade Union Congressin 192715QUESTION Who formed the Red Trade Union Congress and in which year15ANSWER The Communists formed the Red Trade Union Congress16 QUESTION What do you mean by Socialism16 ANSWER Socialism describes any political and economic theory that says the community rather than individuals should own and manage property and natural resources

Subject Eng Literature (The Tempest ndash William Shakespeare) Topic Act III Scene 3 Lines 1 to 52 (Line 52 ndash Brother my lord the Duke Stand to and do as we) Date 13th April 2020 (4th Period)

[Students should read the original play and also the paraphrase given in the school prescribed textbook]Summary Questions amp Answers

o Alonso Sebastian Antonio Gonzalo Adrian Francisco and others wandered about the island in search of Ferdinand and gets tired and hungry of the toil and at the same time gives up all hope of finding him

o Antonio and Sebastian are happy that Alonso is out of hope and decide to make another attempt on his life that night when being so tired they will be sleeping soundly

o Suddenly a solemn and strange music is heard in the air and several strange shapes enter bringing in a banquet These strange shapes then dance round it with gestures of salutation and then inviting the King to eat they depart

o Seeing this strange scene all are inclined to believe the tales told by travelers that there truly are ldquounicornsrdquo and ldquothe phoenixrsquo thronerdquo

1 ALONSO What harmony is this My good friends hark (L18-27)

GONZALO Marvellous sweet music

[Enter several strange shapes bringing in a banquet

they dance about it with gentle actions of salutation

and inviting the King and his companions to eat they depart]ALONSO Give us kind keepers heavens What were theseSEBASTIAN A living drollery Now I will believe

That there are unicorns that in Arabia

There is one tree the phoenixrsquo throne one phoenix

At this hour reigning thereANTONIO Ill believe both

And what does else want credit come to me

And Ill be sworn rsquotis true Travellers neer did lie

Though fools at home condemn rsquoem

(a) How did Prospero present an amazing spectacle before Alonso and his companions

Using his magic powers Prospero ordered strange shapes to lay a banquet before Alonso and his companions The shapes brought several dishes with tasty eatables in them They placed the dishes on a table before Alonso and his companions Then the strange shapes began to dance gracefully around the banquet While dancing they made gestures inviting them to eat the food Then suddenly the shapes disappeared(b) Who were the guests at the strange banquet Describe the lsquoliving drolleryrsquo

Alonso Sebastian Antonio Gonzalo Adrian and Francisco were the guests at the strange banquet

The term ldquoliving drolleryrdquo refers to live entertainment show In this context when Alonso the King of Naples Sebastian his brother Antonio the treacherous brother of Prospero Gonzalo the kind and loyal councillor to the King Adrian and Francisco came to the island they were hungry and weary in their spirits They heard a solemn and strange music They were shocked to see several strange shapes bringing in a banquet and these shapes danced about it with gentle action of salutation inviting the King and his companions to eat After this Sebastian described this show as lsquoliving drolleryrsquo(c) What is lsquophoenixrsquo What are lsquoUnicornsrdquo

The term lsquophoenixrsquo refers to a mythical Arabian bird which lived alone and perched on a solitary tree After one hundred years it expired in flames and rose again from its own ashes

lsquoUnicornsrsquo refers to the mythological four-footed beasts having horns in the centre of their foreheads When the horns are ground into powder the powder was believed to be

an aphrodisiac(d) How does Sebastian explain the puppet show OR Why does the speaker now believe in unicorns and phoenix

Sebastian finds several strange shapes bringing in the banquet They invite the king and his party for dinner and soon depart He tells that if such a strange sight can be a reality there is nothing incredible in the world and from the present moment he will believe anything He says that it is a strange dumb show enacted not by puppets but by living beings It is stranger than a travellerrsquos tale Seeing such a thing

before his own eyes he will no longer disbelieve the story about unicorns and phoenix(e) How do the other characters present respond to this living drollery

At the sight of the lsquoliving drolleryrsquo like Sebastian Gonzalo and Antonio too acted strangely Antonio told that he too now believes in unicorns and phoenix and anything else that seems to be incredible He too now believes in travellersrsquo tales Gonzalo told that if he would report those happenings in Naples nobody will believe him He considers that those gentle shapes were gentler in manner in comparison to the living beings Alonso was at first sight suspicious and told them that those strange shapes conveyed their meaning in expressive gestures when they seemed to lack speech by their movements and sounds Francisco was amazed at their mysterious disappearance

2 ALONSO Not I

(Line 43-52)GONZALO Faith sir you need not fear When we

were boysWho would believe that there were mountaineers

Dewlapped like bulls whose throats had hanging at rsquoem

Wallets of flesh Or that there were such men

Whose heads stood in their breasts Which now we find

Each putter-out of five for one will bring us

Good warrant ofALONSO I will stand to and feed

Although my lastmdashno matter since I feel

The best is past Brother my lord the Duke

Stand to and do as we

(a) How does Alonso respond at the spectacle of the shapes which were sent to them at the instruction of Prospero

After seeing the strange sight of appearing and disappearing of the shapes sent by Prospero to arrange a banquet for them Alonso says that his surprise at having seen those creatures is infinite and he is fully justified in feeling so much surprise He thinks that their shapes their gestures and the sounds they made were indeed amazing Although they do not possess the gift of speech yet they were able to convey their

thoughts by means of their gestures only

(b) What does Prospero say about the views expressed by Alonso regarding the shapes What does Francisco think about the shapesAfter hearing Alonsorsquos views about the shapes Prospero says that this manrsquos praise of the spirits is rather hasty He means to say that Alonso has shown great haste in reaching the conclusion about the shapes Francisco is amazed to see that those shapes disappeared in a mysterious way(c) What does Sebastian ask Alonso to doSebastian tells Alonso that the shapes having disappeared should not matter to them because they have left the eatables behind He asks Alonso to enjoy eating as they are extremely hungry but the king does not accept his offer of enjoying the dishes(d) How does Gonzalo try to dispel Alonsorsquos fear of those strange shapes What kind of references does he give to AlonsoGonzalo says that those who have travelled abroad have reported seeing even stranger sights than these shapes that Alonso and his companions have beheld Hence there is no reason to feel afraid of these shapes Gonzalo further adds that in his younger days he had heard strange stories from travelers and Alonso might have heard similar stories For instance it was said that there existed a certain race of

human beings who had huge lumps of flesh hanging at their throats and who therefore resembled bulls Then Gonzalo tells about a race of human beings whose heads were located at their breasts Gonzalo says that such stories were not believed by most people in those days but now-a-days these stories have become common(e) Explain the following lsquoEach putter-out of five for onersquoEnglish travellers often insured their trips with London brokers Those that went on foreign travels those days used to deposit a certain amount with some firm or company in London before their departure If the travelers failed to return the money was forfeited by the company with which it had been deposited But this money was repaid five-fold if the travelers returned safe and sound In this way a traveler stood a great chance of recovering the entire cost of his

travels(f) Give the explanatory meanings of the following expressions in the context of the above extract (i) Dewlapped (ii) Wallets of flesh

(iii) Putter-out(i) Dewlapped having big lumps of flesh at the necks(ii) Wallets of flesh large masses of flesh looking like bags(iii) Putter-out to invest money before commencing the travel

  • General methods of preparation of hydrogen
  • Chapter Dimensional Analysis (Summary)
    • Properties of Charge
Page 55:   · Web viewSubject. Topic. Summary. Execution. Hindi. व्याकरण. शरीरके अंगो के नाम लिखिए. 1) आँख 2) नाक 3

The maximum number of partners in a partnership firm can be 50 according toCompanies Act 20131048698 Business The Partnership is formed to carry on business with a purpose of earning profits The business should be lawful Thus if two or more persons agree to carry on unlawful activities it will not be termed as partnership1048698 Sharing Profits The partners agree to share profits in the agreed ratio In caseof loss all the partners have to bear it in the same agreed profit sharing ratio10486981048698Mutual Agency Every partner is an agent of the other partners Every partner can bind the firm and all other partners by hisher acts Each partner will be responsible and liable for the acts of all other partners10486981048698Unlimited liability The liability of each partner except that of a minor is unlimited Their liability extends to their personal assets also If the assets of the firm are insufficient to pay off its debts the partnersrsquo personal property can be used to satisfy the claim of the creditors of the partnership firm10486981048698Management All the partners have a right to mange the business However they may authorize one or more partners to manage the affairs of the business on their behalf10486981048698Transferability of Share No partner can transfer hisher share to any one including hisher family member without the consent of all other partners3PARTNERSHIP DEEDAgreement forms the basis of partnership The written form of the agreement is which a document of partnership is It contains terms and conditions regarding the conduct of the business It also explains relationship between the partners This document is called partnership deed Every firm can frame its own partnership deed in which the rights duties and liabilities of the partners are stated in detail It helps in settling the disputes arising among the partners during the general conduct of business 4CONTENTS OF PARTNERSHIP DEEDThe partnership deed generally contains the following (i) Name and address of the partnership firm(ii) Nature and objectives of the business(iii) Name and address of each partner(iv) Ratio in which profits is to be shared(v) Capital contribution by each partner(vi) Rate of Interest on capital if allowed(vii) Salary or any other remuneration to partners if allowed(viii) Rate of interest on loans and advances by a partner to the firm(ix) Drawings of partners and interest thereon if any(x) Method of valuation of goodwill and revaluation of assets and liabilities on the reconstitution of the partnership ie on the admission retirement or death of a partner(xi) Settlement of disputes by arbitration(xii) Settlement of accounts at the time of retirement or death of a partner5IN ABSENCE OF PARTNERSHIP DEEDThe partnership deed lays down the terms and conditions of partnership in regard to rights duties and obligations of the partners In the absence of partnership deed there may arise a controversy on certain issues like profit sharing ratio interest on

capital interest on drawings interest on loan and salary of the partners In such cases the provisions of the Indian Partnership Act becomes applicableSome of the Issues are(i) Distribution of Profit Partners are entitled to share profits equally(ii) Interest on Capital Interest on capital is not allowed(iii) Interest on Drawings No interest on drawing of the partners is to be charged(iv) Interest on Partnerrsquos Loan A Partner is allowed interest 6 per annum on the amount of loan given to the firm by himher(v) Salary and Commission to Partner A partner is not entitled to anysalary or commission or any other remuneration for managing the business

History TOPIC-TOWARDS INDEPENDENCE AND PARTITION THE LAST PHASE (1935-1947)

SUB TOPIC-IMPORTANT POLITICAL DEVELOPMENTS ndash GROWTH OF SOCIAL IDEAS

Socialism is a political social and economic philosophyLike in other parts of the world the Russian revolution of 1917 served as a great inspiration for revolutionaries in India who at that time were engaged in the struggle for liberation from British ruleSocialist ideas led to the formation of communist party of IndiaJAWAHARLAL NEHRU Among the early Congress leaders Jawaharlal Nehru was very much impressed and influenced by the Socialist ideas He also learnt about the Economic activities of the Soviet Union after the Bolshevic Revolution 1917 He made full use of them in IndiaThe election of Jawaharlal Nehru and Subhas Chandra Bose showed the Left wing tendency within CongressJawaharlal Nehru demanded economic freedom along with political freedom of the people in order to end the exploitation of masses

Nehrus working committee included three socialists leaders The Lucknow session was a landmark in the evolution of socialist ideas of the congressSUBHAS CHANDRA BOSE ndash Subhas Chandra Bose had socialist leaning Both Jawaharlal Nehru and Subhas Chandra Bose were known as leftist Congress men Later on National Congress divided into Leftist and rightist campCONGRESS SOCIALIST Within the Congress some leaders formed the Congress Socialist partyPattavi Sitaramyya Sardar Patel Rajendra Prasad had hostile attitude towards the Congress Socialist partyJawaharlals attitude was hesitant

1 QUESTION ndash Mention name of two Congress leaders who had socialist leaning

1ANSWER ndash Subhas Chandra Bose and Jawaharlal Nehru2QUESTION- In which session of the congress Jawaharlal elaborated his Socialist ideas2 ANSWER ndash Lucknow and Faizpur Session in December 1935 and 19363QUESTION ndash Why Congress was sharply divided into leftist and rightist camp 3ANSWER ndash Subhas Chandra Bosersquos attempt to seek re election for congress presidentship in 1939sharply divided the National Congress into Leftist and Rightist camp4 QUESTION ndash Who was MN Roy 4 ANSWER ndash Manabendra Roy first formed the Communist Party of India outside the country at Tashkent in 19205QUESTION ndash Who formed the Congress Socialist Party within the Congress5 ANSWER ndash Jaya Prakash Narayan Achyut Patwardhan Acharya Narendra Dev Ram Mohan Lohia Aruna Asaf Ali6QUESTION ndash When was the Congress Socialist Party formed What was its object6 ANSWER ndash 1934The Congress Socialist Party sought to work out socialist programme through the Congress They joined hands with the Congress and wanted to carry

Subhas Chandra Bose being expelled from the congress after the Tripuri rift he formed Forward BlockThere were basic differences between the Congress Socialists and the communistsTRADE UNION ACTIVITIES Maximum working class people lived in Bombay and Calcutta The working and living conditions of those workers were very miserable In this situation Shasipada Banerjee NM Lokhande protested against the oppression of the working class peopleThe first Trade Union Madras Labour Union was formed in 1918 by BP WadiaIndustrial strikes took place in Kanpur Calcutta Madras Jamshedpur and Ahmedabad AITUC was formed in Bombay in 1927 The growth of Trade union among the workers was slow because of the fear of the dismissal of the jobIn the mean time the Moderates as well as Communists left AITUC and formed separate organization

on National struggle with the help of workers and peasant class of the society7 QUESTION ndash What was the name of the party founded by Subhas Chandra Bose7 ANSWER- Forward Block8QUESTION ndash Who was Shasipada Banerjee8 ANSWER ndash Shasipada Banerjee was a radical Brahmo He founded a working menrsquos club to protest against exploitation of the British rulers towards the working class of India9 QUESTION ndash What was the weekly published by NM Lokhande9ANSWER- Dinabandhu10 QUESTION ndash Who founded Bombay Mill-Hands Association and in which year10 ANSWER- NM Lokhande in189011 QUESTION- Who was BP WadiaANSWER- BPWadia was the founder of Madras Labour Union in191812 QUESTION- What was the name of the first labour union of India12 ANSWER- Madras Labour Union13 QUESTION Who founded the Majur Mahajan 13 ANSWER GANDHIJI14 QUESTION What was the full form of AITUC When it was formed14 ANSWER All India Trade Union Congressin 192715QUESTION Who formed the Red Trade Union Congress and in which year15ANSWER The Communists formed the Red Trade Union Congress16 QUESTION What do you mean by Socialism16 ANSWER Socialism describes any political and economic theory that says the community rather than individuals should own and manage property and natural resources

Subject Eng Literature (The Tempest ndash William Shakespeare) Topic Act III Scene 3 Lines 1 to 52 (Line 52 ndash Brother my lord the Duke Stand to and do as we) Date 13th April 2020 (4th Period)

[Students should read the original play and also the paraphrase given in the school prescribed textbook]Summary Questions amp Answers

o Alonso Sebastian Antonio Gonzalo Adrian Francisco and others wandered about the island in search of Ferdinand and gets tired and hungry of the toil and at the same time gives up all hope of finding him

o Antonio and Sebastian are happy that Alonso is out of hope and decide to make another attempt on his life that night when being so tired they will be sleeping soundly

o Suddenly a solemn and strange music is heard in the air and several strange shapes enter bringing in a banquet These strange shapes then dance round it with gestures of salutation and then inviting the King to eat they depart

o Seeing this strange scene all are inclined to believe the tales told by travelers that there truly are ldquounicornsrdquo and ldquothe phoenixrsquo thronerdquo

1 ALONSO What harmony is this My good friends hark (L18-27)

GONZALO Marvellous sweet music

[Enter several strange shapes bringing in a banquet

they dance about it with gentle actions of salutation

and inviting the King and his companions to eat they depart]ALONSO Give us kind keepers heavens What were theseSEBASTIAN A living drollery Now I will believe

That there are unicorns that in Arabia

There is one tree the phoenixrsquo throne one phoenix

At this hour reigning thereANTONIO Ill believe both

And what does else want credit come to me

And Ill be sworn rsquotis true Travellers neer did lie

Though fools at home condemn rsquoem

(a) How did Prospero present an amazing spectacle before Alonso and his companions

Using his magic powers Prospero ordered strange shapes to lay a banquet before Alonso and his companions The shapes brought several dishes with tasty eatables in them They placed the dishes on a table before Alonso and his companions Then the strange shapes began to dance gracefully around the banquet While dancing they made gestures inviting them to eat the food Then suddenly the shapes disappeared(b) Who were the guests at the strange banquet Describe the lsquoliving drolleryrsquo

Alonso Sebastian Antonio Gonzalo Adrian and Francisco were the guests at the strange banquet

The term ldquoliving drolleryrdquo refers to live entertainment show In this context when Alonso the King of Naples Sebastian his brother Antonio the treacherous brother of Prospero Gonzalo the kind and loyal councillor to the King Adrian and Francisco came to the island they were hungry and weary in their spirits They heard a solemn and strange music They were shocked to see several strange shapes bringing in a banquet and these shapes danced about it with gentle action of salutation inviting the King and his companions to eat After this Sebastian described this show as lsquoliving drolleryrsquo(c) What is lsquophoenixrsquo What are lsquoUnicornsrdquo

The term lsquophoenixrsquo refers to a mythical Arabian bird which lived alone and perched on a solitary tree After one hundred years it expired in flames and rose again from its own ashes

lsquoUnicornsrsquo refers to the mythological four-footed beasts having horns in the centre of their foreheads When the horns are ground into powder the powder was believed to be

an aphrodisiac(d) How does Sebastian explain the puppet show OR Why does the speaker now believe in unicorns and phoenix

Sebastian finds several strange shapes bringing in the banquet They invite the king and his party for dinner and soon depart He tells that if such a strange sight can be a reality there is nothing incredible in the world and from the present moment he will believe anything He says that it is a strange dumb show enacted not by puppets but by living beings It is stranger than a travellerrsquos tale Seeing such a thing

before his own eyes he will no longer disbelieve the story about unicorns and phoenix(e) How do the other characters present respond to this living drollery

At the sight of the lsquoliving drolleryrsquo like Sebastian Gonzalo and Antonio too acted strangely Antonio told that he too now believes in unicorns and phoenix and anything else that seems to be incredible He too now believes in travellersrsquo tales Gonzalo told that if he would report those happenings in Naples nobody will believe him He considers that those gentle shapes were gentler in manner in comparison to the living beings Alonso was at first sight suspicious and told them that those strange shapes conveyed their meaning in expressive gestures when they seemed to lack speech by their movements and sounds Francisco was amazed at their mysterious disappearance

2 ALONSO Not I

(Line 43-52)GONZALO Faith sir you need not fear When we

were boysWho would believe that there were mountaineers

Dewlapped like bulls whose throats had hanging at rsquoem

Wallets of flesh Or that there were such men

Whose heads stood in their breasts Which now we find

Each putter-out of five for one will bring us

Good warrant ofALONSO I will stand to and feed

Although my lastmdashno matter since I feel

The best is past Brother my lord the Duke

Stand to and do as we

(a) How does Alonso respond at the spectacle of the shapes which were sent to them at the instruction of Prospero

After seeing the strange sight of appearing and disappearing of the shapes sent by Prospero to arrange a banquet for them Alonso says that his surprise at having seen those creatures is infinite and he is fully justified in feeling so much surprise He thinks that their shapes their gestures and the sounds they made were indeed amazing Although they do not possess the gift of speech yet they were able to convey their

thoughts by means of their gestures only

(b) What does Prospero say about the views expressed by Alonso regarding the shapes What does Francisco think about the shapesAfter hearing Alonsorsquos views about the shapes Prospero says that this manrsquos praise of the spirits is rather hasty He means to say that Alonso has shown great haste in reaching the conclusion about the shapes Francisco is amazed to see that those shapes disappeared in a mysterious way(c) What does Sebastian ask Alonso to doSebastian tells Alonso that the shapes having disappeared should not matter to them because they have left the eatables behind He asks Alonso to enjoy eating as they are extremely hungry but the king does not accept his offer of enjoying the dishes(d) How does Gonzalo try to dispel Alonsorsquos fear of those strange shapes What kind of references does he give to AlonsoGonzalo says that those who have travelled abroad have reported seeing even stranger sights than these shapes that Alonso and his companions have beheld Hence there is no reason to feel afraid of these shapes Gonzalo further adds that in his younger days he had heard strange stories from travelers and Alonso might have heard similar stories For instance it was said that there existed a certain race of

human beings who had huge lumps of flesh hanging at their throats and who therefore resembled bulls Then Gonzalo tells about a race of human beings whose heads were located at their breasts Gonzalo says that such stories were not believed by most people in those days but now-a-days these stories have become common(e) Explain the following lsquoEach putter-out of five for onersquoEnglish travellers often insured their trips with London brokers Those that went on foreign travels those days used to deposit a certain amount with some firm or company in London before their departure If the travelers failed to return the money was forfeited by the company with which it had been deposited But this money was repaid five-fold if the travelers returned safe and sound In this way a traveler stood a great chance of recovering the entire cost of his

travels(f) Give the explanatory meanings of the following expressions in the context of the above extract (i) Dewlapped (ii) Wallets of flesh

(iii) Putter-out(i) Dewlapped having big lumps of flesh at the necks(ii) Wallets of flesh large masses of flesh looking like bags(iii) Putter-out to invest money before commencing the travel

  • General methods of preparation of hydrogen
  • Chapter Dimensional Analysis (Summary)
    • Properties of Charge
Page 56:   · Web viewSubject. Topic. Summary. Execution. Hindi. व्याकरण. शरीरके अंगो के नाम लिखिए. 1) आँख 2) नाक 3

capital interest on drawings interest on loan and salary of the partners In such cases the provisions of the Indian Partnership Act becomes applicableSome of the Issues are(i) Distribution of Profit Partners are entitled to share profits equally(ii) Interest on Capital Interest on capital is not allowed(iii) Interest on Drawings No interest on drawing of the partners is to be charged(iv) Interest on Partnerrsquos Loan A Partner is allowed interest 6 per annum on the amount of loan given to the firm by himher(v) Salary and Commission to Partner A partner is not entitled to anysalary or commission or any other remuneration for managing the business

History TOPIC-TOWARDS INDEPENDENCE AND PARTITION THE LAST PHASE (1935-1947)

SUB TOPIC-IMPORTANT POLITICAL DEVELOPMENTS ndash GROWTH OF SOCIAL IDEAS

Socialism is a political social and economic philosophyLike in other parts of the world the Russian revolution of 1917 served as a great inspiration for revolutionaries in India who at that time were engaged in the struggle for liberation from British ruleSocialist ideas led to the formation of communist party of IndiaJAWAHARLAL NEHRU Among the early Congress leaders Jawaharlal Nehru was very much impressed and influenced by the Socialist ideas He also learnt about the Economic activities of the Soviet Union after the Bolshevic Revolution 1917 He made full use of them in IndiaThe election of Jawaharlal Nehru and Subhas Chandra Bose showed the Left wing tendency within CongressJawaharlal Nehru demanded economic freedom along with political freedom of the people in order to end the exploitation of masses

Nehrus working committee included three socialists leaders The Lucknow session was a landmark in the evolution of socialist ideas of the congressSUBHAS CHANDRA BOSE ndash Subhas Chandra Bose had socialist leaning Both Jawaharlal Nehru and Subhas Chandra Bose were known as leftist Congress men Later on National Congress divided into Leftist and rightist campCONGRESS SOCIALIST Within the Congress some leaders formed the Congress Socialist partyPattavi Sitaramyya Sardar Patel Rajendra Prasad had hostile attitude towards the Congress Socialist partyJawaharlals attitude was hesitant

1 QUESTION ndash Mention name of two Congress leaders who had socialist leaning

1ANSWER ndash Subhas Chandra Bose and Jawaharlal Nehru2QUESTION- In which session of the congress Jawaharlal elaborated his Socialist ideas2 ANSWER ndash Lucknow and Faizpur Session in December 1935 and 19363QUESTION ndash Why Congress was sharply divided into leftist and rightist camp 3ANSWER ndash Subhas Chandra Bosersquos attempt to seek re election for congress presidentship in 1939sharply divided the National Congress into Leftist and Rightist camp4 QUESTION ndash Who was MN Roy 4 ANSWER ndash Manabendra Roy first formed the Communist Party of India outside the country at Tashkent in 19205QUESTION ndash Who formed the Congress Socialist Party within the Congress5 ANSWER ndash Jaya Prakash Narayan Achyut Patwardhan Acharya Narendra Dev Ram Mohan Lohia Aruna Asaf Ali6QUESTION ndash When was the Congress Socialist Party formed What was its object6 ANSWER ndash 1934The Congress Socialist Party sought to work out socialist programme through the Congress They joined hands with the Congress and wanted to carry

Subhas Chandra Bose being expelled from the congress after the Tripuri rift he formed Forward BlockThere were basic differences between the Congress Socialists and the communistsTRADE UNION ACTIVITIES Maximum working class people lived in Bombay and Calcutta The working and living conditions of those workers were very miserable In this situation Shasipada Banerjee NM Lokhande protested against the oppression of the working class peopleThe first Trade Union Madras Labour Union was formed in 1918 by BP WadiaIndustrial strikes took place in Kanpur Calcutta Madras Jamshedpur and Ahmedabad AITUC was formed in Bombay in 1927 The growth of Trade union among the workers was slow because of the fear of the dismissal of the jobIn the mean time the Moderates as well as Communists left AITUC and formed separate organization

on National struggle with the help of workers and peasant class of the society7 QUESTION ndash What was the name of the party founded by Subhas Chandra Bose7 ANSWER- Forward Block8QUESTION ndash Who was Shasipada Banerjee8 ANSWER ndash Shasipada Banerjee was a radical Brahmo He founded a working menrsquos club to protest against exploitation of the British rulers towards the working class of India9 QUESTION ndash What was the weekly published by NM Lokhande9ANSWER- Dinabandhu10 QUESTION ndash Who founded Bombay Mill-Hands Association and in which year10 ANSWER- NM Lokhande in189011 QUESTION- Who was BP WadiaANSWER- BPWadia was the founder of Madras Labour Union in191812 QUESTION- What was the name of the first labour union of India12 ANSWER- Madras Labour Union13 QUESTION Who founded the Majur Mahajan 13 ANSWER GANDHIJI14 QUESTION What was the full form of AITUC When it was formed14 ANSWER All India Trade Union Congressin 192715QUESTION Who formed the Red Trade Union Congress and in which year15ANSWER The Communists formed the Red Trade Union Congress16 QUESTION What do you mean by Socialism16 ANSWER Socialism describes any political and economic theory that says the community rather than individuals should own and manage property and natural resources

Subject Eng Literature (The Tempest ndash William Shakespeare) Topic Act III Scene 3 Lines 1 to 52 (Line 52 ndash Brother my lord the Duke Stand to and do as we) Date 13th April 2020 (4th Period)

[Students should read the original play and also the paraphrase given in the school prescribed textbook]Summary Questions amp Answers

o Alonso Sebastian Antonio Gonzalo Adrian Francisco and others wandered about the island in search of Ferdinand and gets tired and hungry of the toil and at the same time gives up all hope of finding him

o Antonio and Sebastian are happy that Alonso is out of hope and decide to make another attempt on his life that night when being so tired they will be sleeping soundly

o Suddenly a solemn and strange music is heard in the air and several strange shapes enter bringing in a banquet These strange shapes then dance round it with gestures of salutation and then inviting the King to eat they depart

o Seeing this strange scene all are inclined to believe the tales told by travelers that there truly are ldquounicornsrdquo and ldquothe phoenixrsquo thronerdquo

1 ALONSO What harmony is this My good friends hark (L18-27)

GONZALO Marvellous sweet music

[Enter several strange shapes bringing in a banquet

they dance about it with gentle actions of salutation

and inviting the King and his companions to eat they depart]ALONSO Give us kind keepers heavens What were theseSEBASTIAN A living drollery Now I will believe

That there are unicorns that in Arabia

There is one tree the phoenixrsquo throne one phoenix

At this hour reigning thereANTONIO Ill believe both

And what does else want credit come to me

And Ill be sworn rsquotis true Travellers neer did lie

Though fools at home condemn rsquoem

(a) How did Prospero present an amazing spectacle before Alonso and his companions

Using his magic powers Prospero ordered strange shapes to lay a banquet before Alonso and his companions The shapes brought several dishes with tasty eatables in them They placed the dishes on a table before Alonso and his companions Then the strange shapes began to dance gracefully around the banquet While dancing they made gestures inviting them to eat the food Then suddenly the shapes disappeared(b) Who were the guests at the strange banquet Describe the lsquoliving drolleryrsquo

Alonso Sebastian Antonio Gonzalo Adrian and Francisco were the guests at the strange banquet

The term ldquoliving drolleryrdquo refers to live entertainment show In this context when Alonso the King of Naples Sebastian his brother Antonio the treacherous brother of Prospero Gonzalo the kind and loyal councillor to the King Adrian and Francisco came to the island they were hungry and weary in their spirits They heard a solemn and strange music They were shocked to see several strange shapes bringing in a banquet and these shapes danced about it with gentle action of salutation inviting the King and his companions to eat After this Sebastian described this show as lsquoliving drolleryrsquo(c) What is lsquophoenixrsquo What are lsquoUnicornsrdquo

The term lsquophoenixrsquo refers to a mythical Arabian bird which lived alone and perched on a solitary tree After one hundred years it expired in flames and rose again from its own ashes

lsquoUnicornsrsquo refers to the mythological four-footed beasts having horns in the centre of their foreheads When the horns are ground into powder the powder was believed to be

an aphrodisiac(d) How does Sebastian explain the puppet show OR Why does the speaker now believe in unicorns and phoenix

Sebastian finds several strange shapes bringing in the banquet They invite the king and his party for dinner and soon depart He tells that if such a strange sight can be a reality there is nothing incredible in the world and from the present moment he will believe anything He says that it is a strange dumb show enacted not by puppets but by living beings It is stranger than a travellerrsquos tale Seeing such a thing

before his own eyes he will no longer disbelieve the story about unicorns and phoenix(e) How do the other characters present respond to this living drollery

At the sight of the lsquoliving drolleryrsquo like Sebastian Gonzalo and Antonio too acted strangely Antonio told that he too now believes in unicorns and phoenix and anything else that seems to be incredible He too now believes in travellersrsquo tales Gonzalo told that if he would report those happenings in Naples nobody will believe him He considers that those gentle shapes were gentler in manner in comparison to the living beings Alonso was at first sight suspicious and told them that those strange shapes conveyed their meaning in expressive gestures when they seemed to lack speech by their movements and sounds Francisco was amazed at their mysterious disappearance

2 ALONSO Not I

(Line 43-52)GONZALO Faith sir you need not fear When we

were boysWho would believe that there were mountaineers

Dewlapped like bulls whose throats had hanging at rsquoem

Wallets of flesh Or that there were such men

Whose heads stood in their breasts Which now we find

Each putter-out of five for one will bring us

Good warrant ofALONSO I will stand to and feed

Although my lastmdashno matter since I feel

The best is past Brother my lord the Duke

Stand to and do as we

(a) How does Alonso respond at the spectacle of the shapes which were sent to them at the instruction of Prospero

After seeing the strange sight of appearing and disappearing of the shapes sent by Prospero to arrange a banquet for them Alonso says that his surprise at having seen those creatures is infinite and he is fully justified in feeling so much surprise He thinks that their shapes their gestures and the sounds they made were indeed amazing Although they do not possess the gift of speech yet they were able to convey their

thoughts by means of their gestures only

(b) What does Prospero say about the views expressed by Alonso regarding the shapes What does Francisco think about the shapesAfter hearing Alonsorsquos views about the shapes Prospero says that this manrsquos praise of the spirits is rather hasty He means to say that Alonso has shown great haste in reaching the conclusion about the shapes Francisco is amazed to see that those shapes disappeared in a mysterious way(c) What does Sebastian ask Alonso to doSebastian tells Alonso that the shapes having disappeared should not matter to them because they have left the eatables behind He asks Alonso to enjoy eating as they are extremely hungry but the king does not accept his offer of enjoying the dishes(d) How does Gonzalo try to dispel Alonsorsquos fear of those strange shapes What kind of references does he give to AlonsoGonzalo says that those who have travelled abroad have reported seeing even stranger sights than these shapes that Alonso and his companions have beheld Hence there is no reason to feel afraid of these shapes Gonzalo further adds that in his younger days he had heard strange stories from travelers and Alonso might have heard similar stories For instance it was said that there existed a certain race of

human beings who had huge lumps of flesh hanging at their throats and who therefore resembled bulls Then Gonzalo tells about a race of human beings whose heads were located at their breasts Gonzalo says that such stories were not believed by most people in those days but now-a-days these stories have become common(e) Explain the following lsquoEach putter-out of five for onersquoEnglish travellers often insured their trips with London brokers Those that went on foreign travels those days used to deposit a certain amount with some firm or company in London before their departure If the travelers failed to return the money was forfeited by the company with which it had been deposited But this money was repaid five-fold if the travelers returned safe and sound In this way a traveler stood a great chance of recovering the entire cost of his

travels(f) Give the explanatory meanings of the following expressions in the context of the above extract (i) Dewlapped (ii) Wallets of flesh

(iii) Putter-out(i) Dewlapped having big lumps of flesh at the necks(ii) Wallets of flesh large masses of flesh looking like bags(iii) Putter-out to invest money before commencing the travel

  • General methods of preparation of hydrogen
  • Chapter Dimensional Analysis (Summary)
    • Properties of Charge
Page 57:   · Web viewSubject. Topic. Summary. Execution. Hindi. व्याकरण. शरीरके अंगो के नाम लिखिए. 1) आँख 2) नाक 3

Subhas Chandra Bose being expelled from the congress after the Tripuri rift he formed Forward BlockThere were basic differences between the Congress Socialists and the communistsTRADE UNION ACTIVITIES Maximum working class people lived in Bombay and Calcutta The working and living conditions of those workers were very miserable In this situation Shasipada Banerjee NM Lokhande protested against the oppression of the working class peopleThe first Trade Union Madras Labour Union was formed in 1918 by BP WadiaIndustrial strikes took place in Kanpur Calcutta Madras Jamshedpur and Ahmedabad AITUC was formed in Bombay in 1927 The growth of Trade union among the workers was slow because of the fear of the dismissal of the jobIn the mean time the Moderates as well as Communists left AITUC and formed separate organization

on National struggle with the help of workers and peasant class of the society7 QUESTION ndash What was the name of the party founded by Subhas Chandra Bose7 ANSWER- Forward Block8QUESTION ndash Who was Shasipada Banerjee8 ANSWER ndash Shasipada Banerjee was a radical Brahmo He founded a working menrsquos club to protest against exploitation of the British rulers towards the working class of India9 QUESTION ndash What was the weekly published by NM Lokhande9ANSWER- Dinabandhu10 QUESTION ndash Who founded Bombay Mill-Hands Association and in which year10 ANSWER- NM Lokhande in189011 QUESTION- Who was BP WadiaANSWER- BPWadia was the founder of Madras Labour Union in191812 QUESTION- What was the name of the first labour union of India12 ANSWER- Madras Labour Union13 QUESTION Who founded the Majur Mahajan 13 ANSWER GANDHIJI14 QUESTION What was the full form of AITUC When it was formed14 ANSWER All India Trade Union Congressin 192715QUESTION Who formed the Red Trade Union Congress and in which year15ANSWER The Communists formed the Red Trade Union Congress16 QUESTION What do you mean by Socialism16 ANSWER Socialism describes any political and economic theory that says the community rather than individuals should own and manage property and natural resources

Subject Eng Literature (The Tempest ndash William Shakespeare) Topic Act III Scene 3 Lines 1 to 52 (Line 52 ndash Brother my lord the Duke Stand to and do as we) Date 13th April 2020 (4th Period)

[Students should read the original play and also the paraphrase given in the school prescribed textbook]Summary Questions amp Answers

o Alonso Sebastian Antonio Gonzalo Adrian Francisco and others wandered about the island in search of Ferdinand and gets tired and hungry of the toil and at the same time gives up all hope of finding him

o Antonio and Sebastian are happy that Alonso is out of hope and decide to make another attempt on his life that night when being so tired they will be sleeping soundly

o Suddenly a solemn and strange music is heard in the air and several strange shapes enter bringing in a banquet These strange shapes then dance round it with gestures of salutation and then inviting the King to eat they depart

o Seeing this strange scene all are inclined to believe the tales told by travelers that there truly are ldquounicornsrdquo and ldquothe phoenixrsquo thronerdquo

1 ALONSO What harmony is this My good friends hark (L18-27)

GONZALO Marvellous sweet music

[Enter several strange shapes bringing in a banquet

they dance about it with gentle actions of salutation

and inviting the King and his companions to eat they depart]ALONSO Give us kind keepers heavens What were theseSEBASTIAN A living drollery Now I will believe

That there are unicorns that in Arabia

There is one tree the phoenixrsquo throne one phoenix

At this hour reigning thereANTONIO Ill believe both

And what does else want credit come to me

And Ill be sworn rsquotis true Travellers neer did lie

Though fools at home condemn rsquoem

(a) How did Prospero present an amazing spectacle before Alonso and his companions

Using his magic powers Prospero ordered strange shapes to lay a banquet before Alonso and his companions The shapes brought several dishes with tasty eatables in them They placed the dishes on a table before Alonso and his companions Then the strange shapes began to dance gracefully around the banquet While dancing they made gestures inviting them to eat the food Then suddenly the shapes disappeared(b) Who were the guests at the strange banquet Describe the lsquoliving drolleryrsquo

Alonso Sebastian Antonio Gonzalo Adrian and Francisco were the guests at the strange banquet

The term ldquoliving drolleryrdquo refers to live entertainment show In this context when Alonso the King of Naples Sebastian his brother Antonio the treacherous brother of Prospero Gonzalo the kind and loyal councillor to the King Adrian and Francisco came to the island they were hungry and weary in their spirits They heard a solemn and strange music They were shocked to see several strange shapes bringing in a banquet and these shapes danced about it with gentle action of salutation inviting the King and his companions to eat After this Sebastian described this show as lsquoliving drolleryrsquo(c) What is lsquophoenixrsquo What are lsquoUnicornsrdquo

The term lsquophoenixrsquo refers to a mythical Arabian bird which lived alone and perched on a solitary tree After one hundred years it expired in flames and rose again from its own ashes

lsquoUnicornsrsquo refers to the mythological four-footed beasts having horns in the centre of their foreheads When the horns are ground into powder the powder was believed to be

an aphrodisiac(d) How does Sebastian explain the puppet show OR Why does the speaker now believe in unicorns and phoenix

Sebastian finds several strange shapes bringing in the banquet They invite the king and his party for dinner and soon depart He tells that if such a strange sight can be a reality there is nothing incredible in the world and from the present moment he will believe anything He says that it is a strange dumb show enacted not by puppets but by living beings It is stranger than a travellerrsquos tale Seeing such a thing

before his own eyes he will no longer disbelieve the story about unicorns and phoenix(e) How do the other characters present respond to this living drollery

At the sight of the lsquoliving drolleryrsquo like Sebastian Gonzalo and Antonio too acted strangely Antonio told that he too now believes in unicorns and phoenix and anything else that seems to be incredible He too now believes in travellersrsquo tales Gonzalo told that if he would report those happenings in Naples nobody will believe him He considers that those gentle shapes were gentler in manner in comparison to the living beings Alonso was at first sight suspicious and told them that those strange shapes conveyed their meaning in expressive gestures when they seemed to lack speech by their movements and sounds Francisco was amazed at their mysterious disappearance

2 ALONSO Not I

(Line 43-52)GONZALO Faith sir you need not fear When we

were boysWho would believe that there were mountaineers

Dewlapped like bulls whose throats had hanging at rsquoem

Wallets of flesh Or that there were such men

Whose heads stood in their breasts Which now we find

Each putter-out of five for one will bring us

Good warrant ofALONSO I will stand to and feed

Although my lastmdashno matter since I feel

The best is past Brother my lord the Duke

Stand to and do as we

(a) How does Alonso respond at the spectacle of the shapes which were sent to them at the instruction of Prospero

After seeing the strange sight of appearing and disappearing of the shapes sent by Prospero to arrange a banquet for them Alonso says that his surprise at having seen those creatures is infinite and he is fully justified in feeling so much surprise He thinks that their shapes their gestures and the sounds they made were indeed amazing Although they do not possess the gift of speech yet they were able to convey their

thoughts by means of their gestures only

(b) What does Prospero say about the views expressed by Alonso regarding the shapes What does Francisco think about the shapesAfter hearing Alonsorsquos views about the shapes Prospero says that this manrsquos praise of the spirits is rather hasty He means to say that Alonso has shown great haste in reaching the conclusion about the shapes Francisco is amazed to see that those shapes disappeared in a mysterious way(c) What does Sebastian ask Alonso to doSebastian tells Alonso that the shapes having disappeared should not matter to them because they have left the eatables behind He asks Alonso to enjoy eating as they are extremely hungry but the king does not accept his offer of enjoying the dishes(d) How does Gonzalo try to dispel Alonsorsquos fear of those strange shapes What kind of references does he give to AlonsoGonzalo says that those who have travelled abroad have reported seeing even stranger sights than these shapes that Alonso and his companions have beheld Hence there is no reason to feel afraid of these shapes Gonzalo further adds that in his younger days he had heard strange stories from travelers and Alonso might have heard similar stories For instance it was said that there existed a certain race of

human beings who had huge lumps of flesh hanging at their throats and who therefore resembled bulls Then Gonzalo tells about a race of human beings whose heads were located at their breasts Gonzalo says that such stories were not believed by most people in those days but now-a-days these stories have become common(e) Explain the following lsquoEach putter-out of five for onersquoEnglish travellers often insured their trips with London brokers Those that went on foreign travels those days used to deposit a certain amount with some firm or company in London before their departure If the travelers failed to return the money was forfeited by the company with which it had been deposited But this money was repaid five-fold if the travelers returned safe and sound In this way a traveler stood a great chance of recovering the entire cost of his

travels(f) Give the explanatory meanings of the following expressions in the context of the above extract (i) Dewlapped (ii) Wallets of flesh

(iii) Putter-out(i) Dewlapped having big lumps of flesh at the necks(ii) Wallets of flesh large masses of flesh looking like bags(iii) Putter-out to invest money before commencing the travel

  • General methods of preparation of hydrogen
  • Chapter Dimensional Analysis (Summary)
    • Properties of Charge
Page 58:   · Web viewSubject. Topic. Summary. Execution. Hindi. व्याकरण. शरीरके अंगो के नाम लिखिए. 1) आँख 2) नाक 3

[Students should read the original play and also the paraphrase given in the school prescribed textbook]Summary Questions amp Answers

o Alonso Sebastian Antonio Gonzalo Adrian Francisco and others wandered about the island in search of Ferdinand and gets tired and hungry of the toil and at the same time gives up all hope of finding him

o Antonio and Sebastian are happy that Alonso is out of hope and decide to make another attempt on his life that night when being so tired they will be sleeping soundly

o Suddenly a solemn and strange music is heard in the air and several strange shapes enter bringing in a banquet These strange shapes then dance round it with gestures of salutation and then inviting the King to eat they depart

o Seeing this strange scene all are inclined to believe the tales told by travelers that there truly are ldquounicornsrdquo and ldquothe phoenixrsquo thronerdquo

1 ALONSO What harmony is this My good friends hark (L18-27)

GONZALO Marvellous sweet music

[Enter several strange shapes bringing in a banquet

they dance about it with gentle actions of salutation

and inviting the King and his companions to eat they depart]ALONSO Give us kind keepers heavens What were theseSEBASTIAN A living drollery Now I will believe

That there are unicorns that in Arabia

There is one tree the phoenixrsquo throne one phoenix

At this hour reigning thereANTONIO Ill believe both

And what does else want credit come to me

And Ill be sworn rsquotis true Travellers neer did lie

Though fools at home condemn rsquoem

(a) How did Prospero present an amazing spectacle before Alonso and his companions

Using his magic powers Prospero ordered strange shapes to lay a banquet before Alonso and his companions The shapes brought several dishes with tasty eatables in them They placed the dishes on a table before Alonso and his companions Then the strange shapes began to dance gracefully around the banquet While dancing they made gestures inviting them to eat the food Then suddenly the shapes disappeared(b) Who were the guests at the strange banquet Describe the lsquoliving drolleryrsquo

Alonso Sebastian Antonio Gonzalo Adrian and Francisco were the guests at the strange banquet

The term ldquoliving drolleryrdquo refers to live entertainment show In this context when Alonso the King of Naples Sebastian his brother Antonio the treacherous brother of Prospero Gonzalo the kind and loyal councillor to the King Adrian and Francisco came to the island they were hungry and weary in their spirits They heard a solemn and strange music They were shocked to see several strange shapes bringing in a banquet and these shapes danced about it with gentle action of salutation inviting the King and his companions to eat After this Sebastian described this show as lsquoliving drolleryrsquo(c) What is lsquophoenixrsquo What are lsquoUnicornsrdquo

The term lsquophoenixrsquo refers to a mythical Arabian bird which lived alone and perched on a solitary tree After one hundred years it expired in flames and rose again from its own ashes

lsquoUnicornsrsquo refers to the mythological four-footed beasts having horns in the centre of their foreheads When the horns are ground into powder the powder was believed to be

an aphrodisiac(d) How does Sebastian explain the puppet show OR Why does the speaker now believe in unicorns and phoenix

Sebastian finds several strange shapes bringing in the banquet They invite the king and his party for dinner and soon depart He tells that if such a strange sight can be a reality there is nothing incredible in the world and from the present moment he will believe anything He says that it is a strange dumb show enacted not by puppets but by living beings It is stranger than a travellerrsquos tale Seeing such a thing

before his own eyes he will no longer disbelieve the story about unicorns and phoenix(e) How do the other characters present respond to this living drollery

At the sight of the lsquoliving drolleryrsquo like Sebastian Gonzalo and Antonio too acted strangely Antonio told that he too now believes in unicorns and phoenix and anything else that seems to be incredible He too now believes in travellersrsquo tales Gonzalo told that if he would report those happenings in Naples nobody will believe him He considers that those gentle shapes were gentler in manner in comparison to the living beings Alonso was at first sight suspicious and told them that those strange shapes conveyed their meaning in expressive gestures when they seemed to lack speech by their movements and sounds Francisco was amazed at their mysterious disappearance

2 ALONSO Not I

(Line 43-52)GONZALO Faith sir you need not fear When we

were boysWho would believe that there were mountaineers

Dewlapped like bulls whose throats had hanging at rsquoem

Wallets of flesh Or that there were such men

Whose heads stood in their breasts Which now we find

Each putter-out of five for one will bring us

Good warrant ofALONSO I will stand to and feed

Although my lastmdashno matter since I feel

The best is past Brother my lord the Duke

Stand to and do as we

(a) How does Alonso respond at the spectacle of the shapes which were sent to them at the instruction of Prospero

After seeing the strange sight of appearing and disappearing of the shapes sent by Prospero to arrange a banquet for them Alonso says that his surprise at having seen those creatures is infinite and he is fully justified in feeling so much surprise He thinks that their shapes their gestures and the sounds they made were indeed amazing Although they do not possess the gift of speech yet they were able to convey their

thoughts by means of their gestures only

(b) What does Prospero say about the views expressed by Alonso regarding the shapes What does Francisco think about the shapesAfter hearing Alonsorsquos views about the shapes Prospero says that this manrsquos praise of the spirits is rather hasty He means to say that Alonso has shown great haste in reaching the conclusion about the shapes Francisco is amazed to see that those shapes disappeared in a mysterious way(c) What does Sebastian ask Alonso to doSebastian tells Alonso that the shapes having disappeared should not matter to them because they have left the eatables behind He asks Alonso to enjoy eating as they are extremely hungry but the king does not accept his offer of enjoying the dishes(d) How does Gonzalo try to dispel Alonsorsquos fear of those strange shapes What kind of references does he give to AlonsoGonzalo says that those who have travelled abroad have reported seeing even stranger sights than these shapes that Alonso and his companions have beheld Hence there is no reason to feel afraid of these shapes Gonzalo further adds that in his younger days he had heard strange stories from travelers and Alonso might have heard similar stories For instance it was said that there existed a certain race of

human beings who had huge lumps of flesh hanging at their throats and who therefore resembled bulls Then Gonzalo tells about a race of human beings whose heads were located at their breasts Gonzalo says that such stories were not believed by most people in those days but now-a-days these stories have become common(e) Explain the following lsquoEach putter-out of five for onersquoEnglish travellers often insured their trips with London brokers Those that went on foreign travels those days used to deposit a certain amount with some firm or company in London before their departure If the travelers failed to return the money was forfeited by the company with which it had been deposited But this money was repaid five-fold if the travelers returned safe and sound In this way a traveler stood a great chance of recovering the entire cost of his

travels(f) Give the explanatory meanings of the following expressions in the context of the above extract (i) Dewlapped (ii) Wallets of flesh

(iii) Putter-out(i) Dewlapped having big lumps of flesh at the necks(ii) Wallets of flesh large masses of flesh looking like bags(iii) Putter-out to invest money before commencing the travel

  • General methods of preparation of hydrogen
  • Chapter Dimensional Analysis (Summary)
    • Properties of Charge
Page 59:   · Web viewSubject. Topic. Summary. Execution. Hindi. व्याकरण. शरीरके अंगो के नाम लिखिए. 1) आँख 2) नाक 3

an aphrodisiac(d) How does Sebastian explain the puppet show OR Why does the speaker now believe in unicorns and phoenix

Sebastian finds several strange shapes bringing in the banquet They invite the king and his party for dinner and soon depart He tells that if such a strange sight can be a reality there is nothing incredible in the world and from the present moment he will believe anything He says that it is a strange dumb show enacted not by puppets but by living beings It is stranger than a travellerrsquos tale Seeing such a thing

before his own eyes he will no longer disbelieve the story about unicorns and phoenix(e) How do the other characters present respond to this living drollery

At the sight of the lsquoliving drolleryrsquo like Sebastian Gonzalo and Antonio too acted strangely Antonio told that he too now believes in unicorns and phoenix and anything else that seems to be incredible He too now believes in travellersrsquo tales Gonzalo told that if he would report those happenings in Naples nobody will believe him He considers that those gentle shapes were gentler in manner in comparison to the living beings Alonso was at first sight suspicious and told them that those strange shapes conveyed their meaning in expressive gestures when they seemed to lack speech by their movements and sounds Francisco was amazed at their mysterious disappearance

2 ALONSO Not I

(Line 43-52)GONZALO Faith sir you need not fear When we

were boysWho would believe that there were mountaineers

Dewlapped like bulls whose throats had hanging at rsquoem

Wallets of flesh Or that there were such men

Whose heads stood in their breasts Which now we find

Each putter-out of five for one will bring us

Good warrant ofALONSO I will stand to and feed

Although my lastmdashno matter since I feel

The best is past Brother my lord the Duke

Stand to and do as we

(a) How does Alonso respond at the spectacle of the shapes which were sent to them at the instruction of Prospero

After seeing the strange sight of appearing and disappearing of the shapes sent by Prospero to arrange a banquet for them Alonso says that his surprise at having seen those creatures is infinite and he is fully justified in feeling so much surprise He thinks that their shapes their gestures and the sounds they made were indeed amazing Although they do not possess the gift of speech yet they were able to convey their

thoughts by means of their gestures only

(b) What does Prospero say about the views expressed by Alonso regarding the shapes What does Francisco think about the shapesAfter hearing Alonsorsquos views about the shapes Prospero says that this manrsquos praise of the spirits is rather hasty He means to say that Alonso has shown great haste in reaching the conclusion about the shapes Francisco is amazed to see that those shapes disappeared in a mysterious way(c) What does Sebastian ask Alonso to doSebastian tells Alonso that the shapes having disappeared should not matter to them because they have left the eatables behind He asks Alonso to enjoy eating as they are extremely hungry but the king does not accept his offer of enjoying the dishes(d) How does Gonzalo try to dispel Alonsorsquos fear of those strange shapes What kind of references does he give to AlonsoGonzalo says that those who have travelled abroad have reported seeing even stranger sights than these shapes that Alonso and his companions have beheld Hence there is no reason to feel afraid of these shapes Gonzalo further adds that in his younger days he had heard strange stories from travelers and Alonso might have heard similar stories For instance it was said that there existed a certain race of

human beings who had huge lumps of flesh hanging at their throats and who therefore resembled bulls Then Gonzalo tells about a race of human beings whose heads were located at their breasts Gonzalo says that such stories were not believed by most people in those days but now-a-days these stories have become common(e) Explain the following lsquoEach putter-out of five for onersquoEnglish travellers often insured their trips with London brokers Those that went on foreign travels those days used to deposit a certain amount with some firm or company in London before their departure If the travelers failed to return the money was forfeited by the company with which it had been deposited But this money was repaid five-fold if the travelers returned safe and sound In this way a traveler stood a great chance of recovering the entire cost of his

travels(f) Give the explanatory meanings of the following expressions in the context of the above extract (i) Dewlapped (ii) Wallets of flesh

(iii) Putter-out(i) Dewlapped having big lumps of flesh at the necks(ii) Wallets of flesh large masses of flesh looking like bags(iii) Putter-out to invest money before commencing the travel

  • General methods of preparation of hydrogen
  • Chapter Dimensional Analysis (Summary)
    • Properties of Charge
Page 60:   · Web viewSubject. Topic. Summary. Execution. Hindi. व्याकरण. शरीरके अंगो के नाम लिखिए. 1) आँख 2) नाक 3

(b) What does Prospero say about the views expressed by Alonso regarding the shapes What does Francisco think about the shapesAfter hearing Alonsorsquos views about the shapes Prospero says that this manrsquos praise of the spirits is rather hasty He means to say that Alonso has shown great haste in reaching the conclusion about the shapes Francisco is amazed to see that those shapes disappeared in a mysterious way(c) What does Sebastian ask Alonso to doSebastian tells Alonso that the shapes having disappeared should not matter to them because they have left the eatables behind He asks Alonso to enjoy eating as they are extremely hungry but the king does not accept his offer of enjoying the dishes(d) How does Gonzalo try to dispel Alonsorsquos fear of those strange shapes What kind of references does he give to AlonsoGonzalo says that those who have travelled abroad have reported seeing even stranger sights than these shapes that Alonso and his companions have beheld Hence there is no reason to feel afraid of these shapes Gonzalo further adds that in his younger days he had heard strange stories from travelers and Alonso might have heard similar stories For instance it was said that there existed a certain race of

human beings who had huge lumps of flesh hanging at their throats and who therefore resembled bulls Then Gonzalo tells about a race of human beings whose heads were located at their breasts Gonzalo says that such stories were not believed by most people in those days but now-a-days these stories have become common(e) Explain the following lsquoEach putter-out of five for onersquoEnglish travellers often insured their trips with London brokers Those that went on foreign travels those days used to deposit a certain amount with some firm or company in London before their departure If the travelers failed to return the money was forfeited by the company with which it had been deposited But this money was repaid five-fold if the travelers returned safe and sound In this way a traveler stood a great chance of recovering the entire cost of his

travels(f) Give the explanatory meanings of the following expressions in the context of the above extract (i) Dewlapped (ii) Wallets of flesh

(iii) Putter-out(i) Dewlapped having big lumps of flesh at the necks(ii) Wallets of flesh large masses of flesh looking like bags(iii) Putter-out to invest money before commencing the travel

  • General methods of preparation of hydrogen
  • Chapter Dimensional Analysis (Summary)
    • Properties of Charge